7,173 Matching Annotations
  1. Oct 2025
    1. Author Response

      The following is the authors’ response to the original reviews.

      Reviewer #1 (Public Review):

      Summary

      The authors investigated the antigenic diversity of recent (2009- 2017) A/H3N2 influenza neuraminidases (NAs), the second major antigenic protein after haemagglutinin. They used 27 viruses and 43 ferret sera and performed NA inhibition. This work was supported by a subset of mouse sera. Clustering analysis determined 4 antigenic clusters, mostly in concordance with the genetic groupings. Association analysis was used to estimate important amino acid positions, which were shown to be more likely close to the catalytic site. Antigenic distances were calculated and a random forest model was used to determine potential important sites.

      This has the potential to be a very interesting piece of work. At present, there are inconsistencies in the methods, results and presentation that limit its impact. In particular, there are weaknesses in some of the computational work.

      Strengths

      (1) The data cover recent NA evolution and a substantial number (43) of ferret (and mouse) sera were generated and titrated against 27 viruses. This is laborious experimental work and is the largest publicly available neuraminidase inhibition dataset that I am aware of. As such, it will prove a useful resource for the influenza community.

      (2) A variety of computational methods were used to analyse the data, which give a rounded picture of the antigenic and genetic relationships and link between sequence, structure and phenotype.

      Weaknesses

      (1) Inconsistency in experimental methods

      Two ferret sera were boosted with H1N2, while recombinant NA protein for the others. This, and the underlying reason, are clearly explained in the manuscript. The authors note that boosting with live virus did not increase titres. Nevertheless, these results are included in the analysis when it would be better to exclude them (Figure 2 shows much lower titres to their own group than other sera).

      As an exercise, we have excluded the H1N2 boosted ferrets sera and no major impact was observed in the antigenic grouping (see Author response image 1a). Another way to control for differences in immunogenicity is to normalize the NAI values with the homologous ELISA titers for each antigen. Clustering based on these ELISA normalized NAI titers reveals the same 4 distinct antigenic groups but with one change: Kan17 is shifted from group 1 to group 2 (Author response image 1b). Note that a homologous ELISA titer is not available for A/West-Virginia/17/2012 and thus this serum sample is not included in Author response image 1b.

      Author response image 1.

      Antigenic and phylogenetic relatedness of N2 NAs. Phylogenetic tree based on the N2 NA head domain amino acid sequences and heat-map representing the average of normalized neuraminidase inhibition titer per H6N2 [log2 (max NAI/NAI)] determined in ferret sera after the boost (listed vertically). The red-to-blue scale indicates high-to-low NAI observed in ELLA against the H6N2 reassortants (listed at the bottom). UPGMA clustering of H6N2s inhibition profiles are shown on top of the heat map and colored according to the phylogenetic groups.(a) Based on the ferret sera with exclusion of the sera that were obtained following prime-boost by infection with H1N2 (A/Estonia/91625/2015 and A/Stockholm/15/2014). (b) Based on serum NAI titers that were normalized by the homologous ELISA titer.

      (2) Inconsistency in experimental results

      Clustering of the NA inhibition results identifies three viruses which do not cluster with their phylogenetic group. Again, this is clearly pointed out in the paper. Further investigation of this inconsistency is required to determine whether this has a genetic basis or is an experimental issue. It is difficult to trust the remaining data while this issue is unresolved.

      We understand the concern of the reviewer. It is important to keep in mind that discrete grouping of antigens allows to visualize major antigenic drifts. However, within closely related groups the cross reactivity of antisera is more likely distributed in a spectrum. When we constructed an antigenic map based on the antigenic cartography algorithm (as described by Smith D. et al, 2004), Kansas17, Wis15, and Ala15 are positioned more closely to antigenic group 1 than the majority of other antigens that were classified as group 2 (Author response image 2a). Similar results were obtained when individual ferret sera from the biological duplicates were used (Author response image 2b). This antigenic cartography map is now added as Figure 2. Figure supplement 3 to the revised manuscript.

      Author response image 2.

      The antigenic cartography was constructed using averaged data from pairs of ferrets (a). Similar analysis was performed on individual ferrets sera (b).

      (3) Inconsistency in group labelling

      A/Hatay/4990/2016 & A/New Caledonia/23/2016 are in phylogenetic group 1 in Figure 2 and phylogenetic group 1 in Figure 5 - figure supplement 1 panel a.

      Our apologies: there was indeed a mistake in labeling of Figure 5. A new antigenic cartography was constructed and included in the revised manuscript. As a result Figure 5 - figure supplement has now become redundant and was removed from the manuscript.

      A/Kansas/14/2017 is selected as a representative of antigenic group 2, when in Figure 2 it is labelled as AC1 (although Figure 2 - supplement 4 which the text is referring to shows data for A/Singapore/Infimh-16-0019/2016 as the representative of AC2). A/Kansas/14/2017 is coloured and labelled as AC2 in Figure 2 - supplement 5.

      Thank you for pointing out this inconsistency. Kan17 clustered antigenically in group 1 based on the NAI values that were normalized relative to the serum with the maximal NAI value against the H6N2 virus that was tested. When using NAI titers that are normalization with the homologous ELISA titer, Kan17 is positioned in group 2. Likewise, antigenic cartography mapping positions Kan17 in group 2. Therefore, we conclude that A/Kansas/14/2017 NA is a representative of group 2.

      The colouring is changed for Figure 3a at the bottom. A/Heilongjiang-Xiangyang/1134/2011 is coloured the same as AC4 viruses when it is AC1 in Figure 2. This lack of consistency makes the figures misleading.

      We apologize for this mistake. The coloring in Figure 3a has been corrected.

      (4) Data not presented, without explanation

      The paper states that 44 sera and 27 H6N2 viruses were used (line 158). However, the results for the Kansas/14/2017 sera do not appear to be presented in any of the figures (e.g. Figure 2 phylogenetic tree, Figure 5 - figure supplement 1). It is not obvious why these data were not presented. The exclusion of this serum could affect the results as often the homologous titre is the highest and several heatmaps show the fold down from the highest titre.

      Serum against A/Kansas/14/2017 was not prepared. For that reason, it is not included in the analysis. We agree that such homologous serum ideally should have been included and in the NAI assay would have resulted in a high if not the highest titer. However, we noticed that homologous sera did not always have the highest titers, especially in panels like ours were some antigens are closely related. The highest titer obtained against Kan17 H6N2 was from A/Bris/16 sera: 1/104, a titer that is in the range of other, homologous titers observed in the panel (Table S3). The Bris16 and Kan17 NAs have five amino acid differences. In summary, inclusion of Kan17 homologous sera would likely not impact the analysis and interpretation of the results because there are multiple highly cross-inhibiting heterologous serum samples against Kan17.

      (5) The cMDS plot does not have sufficient quality assurance A cMDS plot is shown in Figure 5 - figure supplement 1, generated using classical MDS. The following support for the appropriateness of this visualisation is not given. a. Goodness of fit of the cMDS projection, including per point and per titre. b. Testing of the appropriate number of dimensions (the two sera from phylogenetic group 3 are clustered with phylogenetic group 2; additional dimensions might separate these groups). c. A measure of uncertainty in positioning, e.g. bootstrapping. d. A sensitivity analysis of the assumption about titres below the level of detection (i.e. that <20 = 10). Without this information, it is difficult to judge if the projection is reliable.

      We agree with these comments. We have removed Figure 5 – figure supplement 1, and added new figure 2 – figure supplement 3 (antigenic cartography) instead.

      (6) Choice of antigenic distance measure

      The measure of antigenic distance used here is the average difference between titres for two sera. This is dependent on which viruses have been included in the analysis and will be biased by the unbalanced number of viruses in the different clusters (12, 8, 2, 5).

      To verify the impact of the number of antigens on our analysis, the matrix of differences was generated with only 4 H6N2s representing at least one phylogenetic group (Per09, Sin16, Hel823 and Ind11) (Author response image 3a). This matrix is very similar to the one calculated based on all 27 antigens (Author response image 3b). The obtained matrix (Author response image 3a) was used in random forest to model antigenic distances and the result of prediction was plotted against real differences calculated based on the full data. The correlation coefficient (R2) of predicted vs observed values dropped from 0.81 to 0.71, suggesting that the number of antigens tested does not drastically affect the antigenic differences calculated based on serum values (Author response image 3e). Importantly, amino acid substitutions potentially associated with increased antigenic distances are similarly identified (Author response image 3c, d and f).

      Author response image 3.

      Matrix of differences was calculated using only 4 H6N2 antigens (a) or the full panel (b). The matrixes from (c) 4 or (d) 27 antigens were used in random forest modeling to estimate the impact of amino acid changes, respectively. The rf modeling data generated from 4 H6N2 only was plotted and correlated with values calculated from the full panel of 27 H6N2s (e). The multi-way importance plot indicates in red that 7 out of the 10 most important substitutions were identified by the analysis using only 4 H6N2s (f).

      Interestingly, when matrix of differences is calculated using only 4 H6N2s data but not including at least one representative of antigenic group 1 and 2, the correlation coefficient between the predicted values and values obtained from the full panel is dramatically impacted (R2 values drops from 0.81 to 0.5 and 0.57. It is important to note that most of the sera also belong to phylogenetic antigens from groups 1 and 2. As a consequence, poorer prediction of those antigens would more drastically impact the correlation. No drastic drop was observed when representative H6N2s from group 3 or 4 were excluded from the data (from 0.81 to 0.75 and 0.73, Author response image 4 c and d).

      Author response image 4.

      Random forest analysis was repeated using only 4 antigens, but excluding representatives of one of the phylogenetic groups (a) no group 1, (b) no group 2, (c) no group 3, and (d) no group 4.

      We also used Euclidean distances as a measure of differences (Author response image 5). The predictive values obtained in rf have a slightly reduced R2 compared to the values obtained using average of differences.

      In conclusion the unbalanced number of antigens used per group and metric of distance does not seem to impact per se our analysis.

      Author response image 5.

      Antigenic distances were calculated using Euclidian distances of sera to sera. Those antigenic distances were used in rf for estimation of antigenic distance and importance of each amino acid substitution.

      (7) Association analysis does not account for correlations

      For each H6N2 virus and position, significance was calculated by comparing the titres between sera that did or did not have a change at that position. This does not take into account the correlations between positions. For haemagglutinin, it can be impossible to determine the true antigenic effects of such correlated substitutions with mutagenesis studies.

      Most of the potential correlated effects cannot be addressed with the panel of N2s, except for combinations of substitution that are included in the panel, such as 245/247 with or without 468. Only mutagenesis studies would shed light on the epistatic effects. However, it is important to keep in mind that those individual substitutions in such kind of study likely do not reflect natural evolution of N2 (cfr. the importance of the NA charge balance (Wang et al., 2021: 10.7554/eLife.72516).

      (8) Random forest method

      25 features are used to classify 43 sera, which seems high (p/3 is typical for classification). By only considering mismatches, rather than the specific amino acid changes, some signals may be lost (for example, at a given position, one amino acid change might be neutral while another has a large antigenic effect). Features may be highly, or perfectly correlated, which will give them a lower reported importance and skew the results.

      The number of features were optimized in the range from 5 to 80, with 25 being optimal (best R-value in predicted vs observed antigenic distances). Those features refer to the number of amino acid substitutions used in each tree. The number of trees was also optimized in the range of 100 to 2000.

      In random forest the matrix of differences is made considering only position based and not the type of substitution in pairs of NA. Indeed, substitutions with distinct effects may skew results by indicating lower reported importance.

      We have highlighted such potential bias in our discussion:

      “Also, our modelling does not consider that substitution by other amino acids can have a distinct impact on the antigenic distance. As a consequence, predictions based on the model could underestimate or overestimate the importance of a particular amino acid residue substitution in some cases.”

      Reviewer #2 (Public Review):

      Summary:

      The authors characterized the antigenicity of N2 protein of 44 selected A(H3N2) influenza A viruses isolated from 2009-2017 using ferret and mice immune sera. Four antigenic groups were identified, which correlated with their respective phylogenic/ genetic groups. Among 102 amino acids differed by the 44 selected N2 proteins, the authors identified residues that differentiate the antigenicity of the four groups and constructed a machine-learning model that provides antigenic distance estimation. Three recent A(H3N2) vaccine strains were tested in the model but there was no experimental data to confirm the model prediction results.

      Strengths:

      This study used N2 protein of 44 selected A(H3N2) influenza A viruses isolated from 2009-2017 and generated corresponding panels of ferret and mouse sera to react with the selected strains. The amount of experimental data for N2 antigenicity characterization is large enough for model building.

      Weaknesses:

      The main weakness is that the strategy of selecting 44 A(H3N2) viruses from 2009-2017 was not explained. It is not clear if they represent the overall genetic diversity of human A(H3N2) viruses circulating during this time. A comprehensive N2 phylogenetic tree of human A(H3N2) viruses from 2009-2017, with the selected 44 strains labeled in the tree, would be helpful to assess the representativeness of the strains included in the study.

      The selection of antigens was performed using the method described by Bien and Tibshirani 2011 (doi: 10.1198/jasa.2011.tm10183). This method calculates MinMax distances to identify a central representative among distinct clusters.

      To facilitate visualization of in a phylogenetic tree, only 180 representative N2 proteins from 2009-2017 were randomly selected (20 strains per year, unlabelled). Those 180 representatives and 44 readout panel strains (labelled) are shown in the phylogenetic tree below. Readout strains cover the major branches of the tree. The tree has been built using PhyML 3.0 using JTT substitution model and default parameters (Guindon S. et al, Systematic Biology 59(3):307-21, 2010) and visualized using ETE3 (Huerta-Cepas J. et al, Mol. Biol. Evol 33(6):1635-38, 2016).

      Author response image 6.

      The second weakness is the use of double-immune ferret sera (post-infection plus immunization with recombinant NA protein) or mouse sera (immunized twice with recombinant NA protein) to characterize the antigenicity of the selected A(H3N2) viruses. Conventionally, NA antigenicity is characterized using ferret sera after a single infection. Repeated influenza exposure in ferrets has been shown to enhance antibody binding affinity and may affect the cross-reactivity to heterologous strains (PMID: 29672713). The increased cross-reactivity is supported by the NAI titers shown in Table S3, as many of the double immune ferret sera showed the highest reactivity not against its own homologous virus but to heterologous strains. Although the authors used the post-infection ferret sera to characterize 5 viruses (Figure 2, Figure Supplement 4), the patterns did not correlate well. If the authors repeat the NA antigenic analysis using the post-infection ferret sera with lower cross-reactivity, will the authors be able to identify more antigenic groups instead of 4 groups?

      This is a very valuable remark. In their paper, Kosikova et al. (CID 2018) report that repeated infection of ferrets with antigenically slightly different H3N2 viruses results in a broader anti-HA response, compared to a prime infection of an influenza naïve ferret, which results in a narrower anti-HA response. In our ferret immunizations the boost was performed with recombinant, enzymatically active NA that was homologous to the NA of the H1N2 virus that was used for the priming by infection. We determined the NAI responses in sera from ferrets after H1N2 infection against 5 different H6N2 viruses (Figure 2 – figure supplement 5). Compared to NAI responses in sera from H1N2 infected and subsequently NA protein boosted ferrets, the NAI titers obtained after a single infection were considerably lower. Although the normalized NAI titers of day 14 and day 42 sera correlated well, we cannot exclude a degree of broadening of the NAI response in the NA protein boost sera (Author response image 7). On the other hand, repeated influenza antigen exposure is the reality for the majority of people.

      Author response image 7.

      Correlation obtained on NAI data from ferrets at day 14 after infection vs data from day 42 after boost.

      Another weakness is that the authors used the newly constructed model to predict the antigenic distance of three recent A(H3N2) viruses but there is no experimental data to validate their prediction (eg. if these viruses are indeed antigenically deviating from group 2 strains as concluded by the authors).

      Indeed, there is no experimental data from A/Hong_Kong/45/2018, A/Tasmania/503/2020, or A/Darwin/9/2021. The generation of data to determine experimental values for A/Hong_Kong/45/2018, A/Tasmania/503/2020, or A/Darwin/9/2021 would require the generation of new reassortant viruses (H1N2s), recombinant protein and immunization of new ferrets. The ferrets sera would have to be analyzed against all 27 H6N2s, including duplicated control sera for normalization. The major point of the modeling was to evaluate if it is possible to predict the antigenic behavior based on amino acid substitutions.

      As an exercise we have run the model again but this time excluding the Swe17 and HK17 antigens from the data set. Sequences of Sw17 or HK17 were then used to predict antigenic distances. The modeled versus experimental data are plotted in Author response image 8 and show a robust predictive outcome with R2 values of 0.94 and 0.91 for Sw17 and HK17, respectively.

      Author response image 8.

      Antigenic distances from Swe17 and HK17 calculated using the random forest algorithm that was constructed without experimental data from Swe17 and HK17. The predicted distances were plotted side by side to the experimental distances in (a) and correlations are shown in (b).

      Reviewer #3 (Public Review):

      Summary:

      This paper by Portela Catani et al examines the antigenic relationships (measured using monotypic ferret and mouse sera) across a panel of N2 genes from the past 14 years, along with the underlying sequence differences and phylogenetic relationships. This is a highly significant topic given the recent increased appreciation of the importance of NA as a vaccine target, and the relative lack of information about NA antigenic evolution compared with what is known about HA. Thus, these data will be of interest to those studying the antigenic evolution of influenza viruses. The methods used are generally quite sound, though there are a few addressable concerns that limit the confidence with which conclusions can be drawn from the data/analyses.

      Strengths:

      • The significance of the work, and the (general) soundness of the methods.

      • Explicit comparison of results obtained with mouse and ferret sera.

      Weaknesses:

      • Approach for assessing the influence of individual polymorphisms on antigenicity does not account for the potential effects of epistasis.

      Indeed, possible epistatic effects or individual polymorphisms were not assessed, which is limited by the nature of the panel of N2s selected in the study. We now emphasize this in the discussion as follows:

      “Also, our modelling does not consider that substitution by different amino acids can have distinct impact on antigenic distance. As a consequence, predictions based on the model could underestimate the importance of a particular amino acid residue substitution in some cases.”

      • Machine learning analyses were neither experimentally validated nor shown to be better than simple, phylogenetic-based inference.

      This is a valid remark and indeed we have found a clear correlation between NAI cross reactivity and phylogenetic relatedness. However, besides achieving good prediction of the experimental data (as shown in Figure 5 and in FigureR7), machine Learning analysis has the potential to rank or indicate major antigenic divergences based on available sequences before it has consolidated as new clade. ML can also support the selection and design of broader reactive antigens.

      Recommendations for the authors:

      Reviewer #1 (Recommendations For The Authors):

      Major corrections

      No major corrections, beyond the issues I touched on in the public review, for which I give a little more detail below:

      Point 2. If there's not a putative genetic basis for the unexpected clustering seen in the NAI, then reiterating a small subset of the data would show the reliability of the experimental methods and substantiate this unexpected finding.

      We thank the reviewer for this pertinent point and suggestion. We have modified our analysis by reiterating individual ferret data normalized with the homologous ELISA titers. This reiteration is shown in figure R1b. In this case both Kan17 and Wis15 are switched to antigenic group 2. The profile of sera inhibition against those 2 strains that shift from antigenic cluster 1 to 2, is clearly an intermediate between profiles observed in those 2 groups. Considering that antigenic evolution occurs gradually, it is not unexpected that those intermediate profiles would swing from one side to another when pushed to forced discrimination. Antigenic cartography mapping, as in Smith et al. (2004), also indicated that those H6N2s are located closer to G1 than overall antigens from G2. Raw data distribution (max and min EC50) also do not indicate potential bias in analysis.

      Point 5. If you want to use antigenic cartography (Smith et al 2004), there is the R CRAN package (https://CRAN.R-project.org/package=Racmacs) which can handle threshold titres (like <20) and has functions for the diagnostic tools I describe, in order to quality assure the resulting plot. It does use a different antigenic distance metric than the paper currently uses, so you might not want to take that route.

      Thank you for this suggestion. We have performed antigenic cartography using the methodology described by Smith et al made accessible by Sam Wilks. The outcome of this analysis has been added to the manuscript as Figure 2 – Figure supplement 3.

      Point 6. More robust measures of antigenic distance take into account the homologous titre, homologous and heterologous titres (Archetti & Horsfall, 1950) or use the highest observed titre for a serum (Smith et al 2004). A limitation of the first two is that the antigenic distance can only be calculated when you have the homologous titre, which will limit you as you only have this for 26/43 sera. They may give similar results to your average antigenic distance, in which case your analysis still stands. Calculating antigenic distance using the homologous or maximum titre only gives the antigenic distance between the antigen and the serum. If you want the distance between all the sera, then further analysis is required (making an antigenic map and outputting the serum-serum distances, see the point above).

      We thank the reviewer for these suggestions. A complete set of 43 H6N2 viruses that matches all 43 sera would have been ideal. This would require the generation of 17 additional H6N2 viruses and their testing in ELLA, a significant amount of work in terms of time and resources. Instead, we have generated an antigenic map of the 27 antigens and homologous sera (cfr. our response to point 5 above). Despite different methods the outcome showing 4 major antigenic groups is consistent.

      Minor corrections

      Table S1

      A/New_Castle/67/2016 should be A/Newcastle/67/2016

      A/Gambia/2012 is not the full virus name

      Corrected.

      Table S3 has multiple values of exactly 10.0. I think these should be <20 as they are below the threshold of detection for the assay.

      All the values lower than 20 in Table S3 were replaced by “< 20”.

      Line 376: A/Sidney/5/1997 should be A/Sydney/5/1997

      Corrected.

      Line 338: "25 randomly sampled data" is a bit vague, "25 randomly sampled features" would be better

      Corrected.

      Include RMSE of the random forest model.

      RMSE=19.6 RMSE/mean = 0.207 is now mentioned in the manuscript.

      Figure 5 - supplement 1: These plots are difficult to interpret as the aspect ratio is not 1:1, and panels a & b are difficult to compare as they have not been aligned (using a Procrustes analysis). It would be neater if they were labelled with short names.

      We have generated an antigenic cartography map instead. As a consequence, the MDS has become redundant and Figure 5 – supplement 1 was removed.

      Line 562: 98 variable residues, where it is 102 elsewhere in the text.

      There are 4 mutations near the end of the NA stalk domain, which are not resolved in the N2 structure. Therefore, amino acid distances to these residues cannot be calculated.

      No data availability statement. Some of the raw data is available in Table S3 and there is no link to the code.

      The data and code used for generation of rf modelling was uploaded to Github and made available. The following statement has been added to the manuscript: “The data and code used for the generation of the rf model is available at https://github.com/SaelensLAB/RF..”

      Reviewer #2 (Recommendations For The Authors):

      (1) More than 42,000 NA sequences are available for the mentioned period on GISAID, it is therefore important to understand the selection criteria for the 44 strains and if these strains represent the overall genetic diversity of N2 of human A(H3N2) viruses. To demonstrate the representativeness of the 44 selected strains, please construct a representative N2 phylogenetic tree for human A(H3N2) viruses circulated in 2009-2017 and label the 44 selected strains on the tree.

      The selection of antigens was performed using the method described by Bien and Tibshirani 2011 (doi: 10.1198/jasa.2011.tm10183). This method uses MinMax distances to identify a central representative among distinct clusters.

      To facilitate visualization tree only of 180 representative N2 proteins from 2009-2017 were randomly selected (20 strains per year, unlabelled). Those 180 representatives and 44 readout panel strains (labelled) are shown in the phylogenetic tree below. Readout strains cover the major branches of the tree. The tree has been built using PhyML 3.0 using JTT substitution model and default parameters (Guindon S. et al, Systematic Biology 59(3):307-21, 2010) and visualized using ETE3 (Huerta-Cepas J. et al, Mol. Biol. Evol 33(6):1635-38, 2016).

      Author response image 9.

      (2) Double immune ferret sera may increase antibody binding affinity and cross-reactivity against heterologous strains. Using single-infection ferret sera may yield different antigenic grouping results (eg. may identify more antigenic groups). Can the authors repeat the NA antigenic grouping using single-infection ferret sera? Although data from a subset of 5 strains was presented (Figure 2, Figure Supplement 4), the information was not sufficient to support if the use of single-infection or double immune ferret sera will yield similar antigenic grouping results.

      In our ferret immunizations the boost was performed with recombinant, enzymatically active NA that was homologous to the NA of the H1N2 virus that was used for the priming by infection. We determined the NAI responses in sera from ferrets after H1N2 infection against 5 different H6N2 viruses (Figure 2 – figure supplement 5). Compared to NAI responses in sera from H1N2 infected and subsequently NA protein boosted ferrets, the NAI titers obtained after a single infection were considerably lower. Although the normalized NAI titers of day 14 and day 42 sera correlated well, we cannot exclude a degree of broadening of the NAI response in the NA protein boost sera (Figure R6). On the other hand, repeated influenza antigen exposure is the reality for the majority of people.

      (3) NA antigenicity data is presented in heat maps and the authors would often describe the heat map patterns matches without further explanations. Line 234-235, the heat map of mouse sera (Figure 2. Figure supplement 5) was described to match the results of ferret sera (Figure 2), but this tends to be subjective. A correlation analysis of 7 selected antigens showed a positive correlation, what about the other 37 antigens?

      The interpretation of heatmaps is indeed very subjective, for this reason the correlation of the 7 selected antigens was also provided. The other 37 antigens were not tested. Considering the results using post boost sera, a simulation of using random forest modeling indicate that the data from one antigen of each antigenic group is sufficient to achieve a reliable predictive output (R2=0.71) (Figure R3 of this rebuttal).

      (4) Can the authors explain in more detail how data in Figure 4a was generated? According to the authors, residues close to the catalytic pocket are more likely to impact NAI. Can the authors explain how they define if a residue is close to the catalytic pocket?

      The correlation of distances of amino acid residues with significance values is explained as follows. Consider 7 distinct elements that are distributed horizontally as shown by the squares in the figure below (Author response image 10a). The elements highlighted in yellow have a numerical propriety (in case of N2 neuraminidase this was the significance values obtained in the association study). Taking P1 as reference we can calculate the distance (red arrows) between P1 and P2, P4 and P7, those distances can them be correlated to intrinsic values of P2, P4 and P7, which enables the calculation of the correlation coefficient Tau. This same process is repeated for each position (or each amino acid), as a consequence every position will have a correlation coefficient calculated (Author response image 8b). This correlation coefficient can be represented as a heat map at the surface of N2.

      Author response image 10.

      The 2D scheme represents the strategy used to calculate the correlation (i.e. the Tau values) between distances and p-values. Tau values can then be presented in a heat map.

      (5) Can the authors provide experimental data using the three recent A(H3N2) viruses as antigens and perform NAI assay to confirm if they are antigenic all deviating from group 2 viruses?

      The generation of data to determine experimental values for A/Hong_Kong/45/2018, A/Tasmania/503/2020, or A/Darwin/9/2021 would require the generation of new reassortant viruses (H1N2s), recombinant protein and immunization of new ferrets. The ferrets sera would have to be analyzed against all 27 H6N2s, including duplicated control sera for normalization. The major point of the modeling was to evaluate if it is possible to predict the antigenic behavior based on amino acid substitutions.

      As an exercise we have run the model again but this time excluding the Swe17 and HK17 antigens from the data set. Sequences of Sw17 or HK17 were then used to predict antigenic distances. The modeled versus experimental data are plotted in Author response image 7 and show a robust predictive outcome with R2 values of 0.94 and 0.91 for Sw17 and HK17, respectively.

      (6) According to Ge et al. 2022 (PMID: 35387078), N2 NA's before 2014 (2007-2013) showed a 329-N-glycosylation and E344, and they were subsequently replaced by H3N2 viruses with E344K and 329 non-glycosylation changing the NI reactivity in ferret antisera towards later strains. Were these residues also predicted to be important to N2 antigenicity from your machine-learning method?

      Three of the N2 NAs used in our panel, A/Victoria/361/2011, A/Hong_Kong/3089/2017, and A/Tennessee/18/2017, lack this N-glycosylation motif. The E344K substitution is present in another 3 NAs, derived from A/Nagano/2153/2017, A/Minnesota/11/2010, and A/Indiana/08/2011. The importance of those mutations is among the lowest ones predicted in our modeling. However, the differences in NAI reported by Ge et al. are low (not even twofold). The experimental variability in our study potentially limits the identification of substitutions with a subtle impact NAI. We have added the following to the discussion in our revised manuscript:

      “It has been reported that an N-glycosylation site at position 329 combined with E344 in NA from human H3N2 viruses from 2007 to 2013 was gradually lost in later H3N2 viruses (Ge et al., 2022). This loss of an N-glycosylation site at position 329 combined with an E344K substitution was associated with a change in NAI reactivity in ferret sera. Three N2 NAs in our panel, derived from A/Victoria/361/2011, A/Hong_Kong/3089/2017, and A/Tennessee/18/2017, lack this N-glycosylation motif. The E344K substitution is present in three other NAs, derived from A/Nagano/2153/2017, A/Minnesota/11/2010, and A/Indiana/08/2011. The importance of those mutations is among the lowest ones predicted by our modeling. However, the differences in NAI reported by Ge et al. are very modest (lower than twofold). The experimental variability in our study potentially limits the identification of substitutions with a subtle impact NAI.”

      Reviewer #3 (Recommendations For The Authors):

      Specific suggestions:

      Line 132: Did the authors confirm the absence of compensatory mutations due to a heterologous H6 background that could potentially confound downstream NAI results?

      All NAs genes of the rescued H6N2 viruses were fully sequenced and were found to be identical to the expected NA sequences, with the only exception being the A/Tasmania/1018/2015 were a mixed population of wt and M467I was found. This substitution is located at the surface and at the top of the NA head domain, and thus could potentially impact NA antigenicity. However, A/Tasmania/1018/2015 H6N2s had a similar inhibition profile as other H6N2s in phylogenetic and antigenic group 1. This indicates that, at least in this mixed population, antigenicity was not drastically affected by the M467I substitution.

      Line 96: how do these data rule out variation in the fraction of properly folded protein across NAs? They certainly show that properly folded NA protein is present, but not whether amounts vary between the different NAs.

      SEC-MALS (size exclusion chromatography-Multiangle light scattering) data and enzymatic activity were considered as a proxy for correctly folded NA. Although the specific activity of the recombinant N2 NAs is expressed per mass unit (microgram), we cannot exclude that the fraction of properly folded protein across the different recombinant NAs may vary.

      Lines 262-269: this analysis approach (based on my reading) seems to consider each polymorphism in isolation and thus does not seem well suited for accounting for epistatic interactions within the NA. For example, the effect of a substitution on NAI may be contingent upon other alleles within NA that are not cleanly segregated between the two serum comparator groups. Can the authors address the potential of epistasis within NA to confound the results shown in Figure 3?

      Unfortunately, epistatic interactions cannot be solved using the panel of N2 selected for the study. This limitation is mentioned in our discussion:

      “It is important to highlight that co-occurring substitutions in our panel (the ones present in the main branches of the phylogenetic tree) cannot be individually assessed by association analysis or the random forest model. The individual weight of those mutation on NA drift thus remains to be experimentally demonstrated.”

      Line 331: is there a way to visualize and/or quantify how these two plots (F5 supplement 1a/b) reflect each other or not? Without this, it is hard to ascertain how they relate to each other.

      We have generated an antigenic cartography map instead. As a consequence, the MDS has become redundant and Figure 5 – supplement 1 was removed.

      Figure 4B structural images are not well labelled.

      The active site in 1 of the protomers is now indicated with an arrow in the top and side views of the NA tetramer.

      Lines 339-359: the ML predictions are just predictions and kind of meaningless without experimental validation of the predicted antigenic differences between recent NAs. This section would also be strengthened by an assessment of whether the ML approach obtains more accurate results than simply using phylogeny to predict antigenic relationships.

      Indeed, there is no experimental data from A/Hong_Kong/45/2018, A/Tasmania/503/2020, or A/Darwin/9/2021. The generation of data to determine experimental values for A/Hong_Kong/45/2018, A/Tasmania/503/2020, or A/Darwin/9/2021 would require the generation of new reassortant viruses (H1N2s), recombinant protein and immunization of new ferrets. The ferrets sera would have to be analyzed against all 27 H6N2s, including duplicated control sera for normalization. The major point of the modeling was to evaluate if it is possible to predict the antigenic behavior based on amino acid substitutions.

      As an exercise we have run the model again but this time excluding the Swe17 and HK17 antigens from the data set. Sequences of Sw17 or HK17 were then used to predict antigenic distances. The modeled versus experimental data are plotted in figure R7 and show a robust predictive outcome with R2 values of 0.94 and 0.91 for Sw17 and HK17, respectively. A major advantage of antigenic modeling is the potential to rank or indicate major antigenic divergences based on available sequences before it has consolidated as new clade. The support in selecting or designing broader reactive antigens is another advantage of machine learning analysis.

      Lines 416-421: appreciate the direct comparison of results obtained from ferrets versus mice.

      We thank the reviewer for expressing this appreciation.

    1. Author response:

      The following is the authors’ response to the original reviews

      Reviewer #1 (Public review):

      Summary:

      This manuscript by Tesmer and colleagues uses fiber photometry recordings, sophisticated analysis of movement, and deep learning algorithms to provide compelling evidence that activity in hypothalamic hypocretin/orexin neurons (HONs) correlates with net body movement over multiple behaviors. By examining projection targets, the authors show that hypocretin/orexin release differs in projection targets to the locus coeruleus and substantia nigra, pars compacta. Ablation of HONs does not cause differences in the power spectra of movements. The movement-tracking ability of HONs is independent of HON activity that correlates with blood glucose levels. Finally, the authors show that body movement is not encoded to the same extent in other neural populations.

      Strengths:

      The major strengths of the study are the combination of fiber photometry recordings, analysis of movement in head-fixed mice, and sophisticated classification of movement using deep learning algorithms. The experiments seem to be well performed, and the data are well presented, visually. The data support the main conclusions of the manuscript.

      We thank the reviewer for their supportive feedback.

      Weaknesses:

      The weaknesses are minor, mostly consisting of writing and data visualization throughout the manuscript. To some degree, it is already known that hypocretin/orexin neurons correlate with movement and arousal, although this manuscript studies this correlation with unprecedented sophistication and scale. It is also unfortunate that most of the experiments throughout the study were only performed in male mice. Taken together, this study is likely to be impactful to the field and our understanding of HONs across behavioral states.

      We agree that disentangling movement from arousal is an important aspect, and in the revised manuscript, we now include new data and analyses towards this (pupillometry to directly assess arousal, and multivariate analysis to assess contributions of arousal vs movemement to HON activity). In addition, we now implement many of the reviewer’s recommendations regarding writing, data presentation, and visual clarity (see our replies in the “recommendations for authors” section).

      Reviewer #1 (Recommendations for the authors):

      Some recommendations for the authors:

      (1) The first sentence of the Introduction states: "Neural activity related to body movement recently received much attention." I would rephrase or clarify this statement, as neuroscientists have been studying neural activity related to body movement for decades.

      The reviewer is correct. Our intention was to highlight the resurgence of movementrelated neurosciences enabled by modern techniques such as deep learning applied to video data (e.g. DeepLabCut, etc). The passage has been updated for clarity.

      (2) The Introduction also states that HONs orchestrate "consciousness and arousal." I would delete the word "consciousness," as consciousness represents a lofty, global concept that is challenging to define and quantify in humans, let alone mice.

      We used the word consciousness to be consistent with current literature on the function of the mouse hypothalamus (e.g. Nat Neurosci 2016 Feb;19(2):290-8). But we agree it is not necessary here, and so we followed the advice to delete it.

      (3) The authors state that HON dynamics were recorded while mice were head-fixed while on a running wheel. For clarity, it would be helpful to visualize this head-fixation in Figures 1A and 5B. It would also be helpful to clarify how certain behaviors (e.g. grooming, chewing) were performed and recorded while the mouse was head-fixed.

      In the revised manuscript, updated graphics with a head-fixed mouse have now been added to relevant figures. Representative RGB frames (colors representing sequential frames) of each behaviour have been added to Figure 2A.

      (4) In the legend for Figure 1A, the reference to Gonzalez et al. 2016 seems out of place (at least the reader should be informed why the text is referring to this previous study). Additionally, because the references are ordered by number instead of alphabetically, it would be more helpful to refer to a numbered reference rather than a name.

      Gonzalez et al. 2016 references the source of the AAV construct used in this figure. This has been moved to the methods. Following eLife formatting guidelines, references will be alphabetized upon publication.

      (5) In Figure 3F, it would be helpful to show visual validation that the HON-DTR method indeed ablates all HONs. This is depicted conceptually, but representative figures would be much more convincing.

      A representative histological slice is now included for both wild type (WT) and HON-DTR mice in the new Figure 4B.

      Reviewer #2 (Public review):

      Summary:

      Despite several methodological strengths, the major and highly significant drawback is the confound of arousal with movement. This confound is not resolved, so the results could be explained by previously established relationships between orexin and arousal/wakefulness.

      This an excellent point, and we agree. To address this directly in the revised manuscript, we now include new data and analyses towards this (pupillometry to directly assess arousal, and multivariate analysis to assess contributions of arousal vs movemement to HON activity).

      Strengths:

      The authors show that orexin neuron activity is associated with body movement and that this information is conveyed irrespective of the fasted state. They also report differences in different orexin target brain regions for orexin release during movement. This paper contains an impressive array of cutting-edge techniques to examine a very important brain system, the orexin-hypocretin system. The authors offer an original perspective on the function of this system. The authors showed that orexin neuron activity scales to some degree with the magnitude of body movement change; this is unaffected by a fasted state and seems to be somewhat unique to orexin neurons.

      The investigation of other genetically defined subcortical neuron populations to determine the specificity of findings is also a strength, as is the ability to quantify movement and use deep learning to classify specific behaviors adds sophistication to analysis. The authors also show heterogeneity in orexin projections to specific target nuclei, which is interesting.

      The authors "speculate that narcolepsy-cataplexy, caused by HON loss-of-function, is perhaps explained by oscillations into unwanted sleep-states and motor programs due to impaired control loops for wakefulness and movement". This is quite an interesting aspect of their work and deserving of further study.

      We thank the reviewer for their supportive feedback.

      Weaknesses:

      Despite the strengths, there are several major and minor weaknesses that detract significantly from the study.

      My main concern with this work is the confound of arousal with movement so that correlations with one might reflect a relationship instead with the other. The orexin system is well known to play an important role in arousal, with elevated activity of orexin neurons reported for waking and high arousal. Orexin signaling has also been strongly associated with motivation, which also is associated with arousal and movement. The authors offer no compelling evidence that the relationships they describe between different movements and orexin signaling do not simply reflect the known relationship between arousal and motivation.

      The authors could address this concern by including classical arousal measurements, eg, cortical EEG recorded simultaneously with movements. Often, EEG arousal occurs independently of movement, so this could provide one approach to disentangling this confound. The idea that orexin signaling plays a role in arousal rather than movement is supported by their finding that orexin lesions using the orexin-DTR mouse model did not impact movements. In contrast, prior lesion and pharmacologic studies have found that decreased orexin signaling significantly decreases arousal and waking.

      Another way they could test their idea would be to paralyze and respirate animals so that orexin activity could be recorded without movement. Alternatively, animals could be trained to remain motionless to receive a reward. Thus, there are several ways to test the overall hypothesis of this work that have not been examined here.

      The authors propose that "a simple interpretation of their results is that, via HON movement tracking, the brain creates a "wake up" signal in proportion to movement". This seems to argue for the role of the orexin system in arousal and motivation rather than in movement per se.

      Thank you. We agree that disentangling between arousal and movement is indeed critical. A classic approach is a multivariate analysis, wherein multiple simultaneously recorded “predictors” of HON activity – such as arousal and movement - can be directly compared. While EEG arousal is an option, another well-accepted metric for arousal is pupil diameter. Using n = 7 mice, we now simultaneously record HON activity, movement, running speed, pupil size fluctuations, and ocular movements:

      We then fit a partial least squares multivariate regression (a regression type more robust to collinearity) using the movement metric, pupil size, and ocular movements as predictors of orexin neuron activity. Consistent with previous publications, we found that pupil size alone has a positive correlation with hORX.GCaMP6s (~0.45). However, using a drop-one feature analysis in multivariate regression, we found that movement had the highest % contribution to statistically explaining orexin neuron activity. Here are the new results (which we now added as Fig. 7A-B).

      Author response image 1.

      Furthermore, we also expanded this analysis to incorporate the different frequencies found in HON dynamics, using empirical mode decomposition. We found that pupil size had a maximum correlation at lower HON frequencies than the movement metric, while ocular movements were maximally correlated in higher frequencies (now added as Fig. 7D,E).

      Overall, this analysis suggests that – while HONs encode both movement and arousal – arousal and movement do not always co-fluctuate at the same timescales, and their impacts on HONs can be disentangled in a number of ways. We now mention this in revised text on page 5.

      There are several studies that have examined the effect of orexin antagonist treatment in rodents on locomotor and other motor activities. These studies have largely found no consistent effect of antagonizing orexin signaling, especially at the OxR1 receptor, on simple motor activity. These studies are not referenced here but should be taken into account in the authors' conclusions.

      We agree. Prior studies found that orexin antagonism – or optogenetic silencing of HONs – evokes either reduced locomotion, or no effect on locomotor movements. We now added text and references to paragraph 4 of Discussion, summarising this.

      Figure 3, panel F: I understand HON-DTR is a validated model but a picture of HONs ablation is necessary, including pictures of HONs outputs ablation within the SNc and LC.

      A representative histological slice is now included for both wild type (WT) and HON-DTR mice in the new Figure 4B. Because HONs are only found in the hypothalamus, somatic deletion of HONs in this region will result in axonal degradation in output regions.

      The discussion lacks a more extensive paragraph on the distinct signal and role of Ox>SNc and Ox-LC projections.

      We now added sentences discussing potential implications of this to Discussion (middle of paragraph 4).

      Reviewer #2 (Recommendations for the authors):

      Minor weaknesses

      A very important movement in rodents is head orientation, especially given the limitation in ocular movement. However, this paper used a fixed head model which obviated this movement and did not attempt to analyze ocular movements.

      Analysing ocular movements is something we had not considered but is very easy to check using pupillometry. In n = 7 mice, we recorded both orexin neurons, and ocular movements captured through an infrared camera under constant lighting. Ocular movements had a small positive correlation with orexin neuron photometry (r = ~0.26). See response to the public review above.

      Author response image 2.

      The "HON" abbreviation is not commonly used for orexin neurons, and I suggest replacing that with a more well-known abbreviation.

      To the best of our knowledge, there is no universally agreed or best-known abbreviation for hypocretin/orexin neurons (we agree it would be nice if there was one!). “HONs” is a simple first letter abbreviation of hypocretin/orexin neurons, which acknowledges the two names for this peptide given by the original discoverers (de Lecea et al, and Sakurai et al, in 1998). Although this may not be the perfect abbreviation, we have kept it for now, also to be consistent with the large number (>10) of other published studies that recently used this abbreviation.

      The graphs showing Pearson's r values do not demonstrate a very strong correlation between neural activity and movement change; they also lack validation of genetic expression/ablation in some cases. The results would more strongly support the conclusions if statistically significant correlations could be demonstrated between activity and movement.

      We agree that a correlation of ~0.68 is probably not worthy of a “very strong” classification. While there is no universal ruleset for categorizing the strength of a correlation, we have toned down our language throughout the manuscript.

      Comment regarding statistical testing of correlations: we are cautious to stand behind correlation significance testing for large sample sizes (~48’000 photometry & video samples in a 40-minute session). In our case, correlations were always extremely significant p<0.0001. The reason for this is that correlation p-values become “too big to fail” (see Lin et al. 2013) with inflated sample size. We therefore refrain from commenting on p-values and rather report between or within-subjects statistical tests, or tests against zero. See four example experiments below.

      Author response image 3.

      Citation: Lin, M., Lucas, H. C., Jr & Shmueli, G. Research Commentary—Too Big to Fail: Large Samples and the p-Value Problem. Information Systems Research 24, 906–917 (2013).

      The rationale for looking at running speed, general movement, and specific types of nonlocomotor movements could be clarified and explained more thoroughly in the introduction. Why is it important to distinguish between locomotion (represented here with running) and all other movements? Presumably, this is because orexin is known to regulate arousal/locomotion. What evidence is there for orexin's role in other types of movements, which are being grouped together in Figure 1? This could be laid out in more detail in the Introduction. Relatedly, it is not very clear in the text whether the correlation between movement and orexin neuron activity includes movement related to running.

      The main focus of our paper is on movement in general (i.e. video pixel difference, described in Results and Methods). This movement metric includes everything captured by the video, it is agnostic to the type of movement or behaviour.  To connect this to some of the specific innate movements/behaviours typically studied in mouse literature (running, grooming, sniffing, etc), we also performed plots in Figure 2. We attempted to explain this better in revised section 1 of Results.

      What exactly is being correlated in Figure 1C (and throughout the rest of the paper?) Is this the average signal correlated with the average movement change over the entire recording time? This could be more explicitly stated in methods/results. The correlations themselves/p-values could be shown in addition to/instead of Pearson's r values. Are the correlations themselves significant? This would strengthen the claim that orexin activity is strongly coupled to the magnitude of body movement change. As another example, in Figure 2D, there are no statistics reported on the correlation between movement metric and average neural signal. In Figure 6G, orexin neuron activity is more strongly correlated with movement than MVe glut neurons, but are either of these correlations significant? The correlation between MVe glut activity and movement overall seems similar to that of orexin neurons, and may be worth noting more explicitly.

      Throughout the paper, we have recorded both neural activity (photometry) and movement at 20 Hz. This would generate, for example, 48’000 samples of photometry and movement from a 40-minute session. All the samples were used to calculate a pearson’s r between variables. To clarify this, we now added the subtext “wholesession” to relevant figures, as well as a clarification in the methods.

      Individual experiment correlations for orexin neurons and MVe glut neurons were always significant p<0.0001, even after a Bonferroni multiple comparisons correction was applied to each population. See the “too big to fail” nature of correlation hypothesis testing above.

      It could be made clearer at the end of Figure 2 that orexin neuron activity is tracking the magnitude of movement change (shown in Figure 2D), not that it is encoding different types of movement.

      We intended for original Figure 2E to illustrate this concept, however this panel has caused a great deal of confusion to several readers and was perhaps ill conceived. We have replaced Figure 2E with a new panel more directly addressing the reviewer’s statement. We can construct three models where orexin neuron activity is predicted from the behavioral classification (sometimes called “one-hot” encoding) and/or the movement metric.

      Model 1 predicts orexin neuron activity using only a categorical predictor of behavioral state. Model 2 only uses the movement metric, and model 3 allows a different movement-metric correlation within each behavioral state. We can compare these models using AIC (Akaike Information Criterion) which is a point estimate. While the most complex model 3 was the best, model 2 was much closer to model 3 than model 1. Similarly, model 2 was much better than model 1. From this we conclude that the magnitude of movement change is a more powerful predictor than behavioral state (“type of movement”). This is now Figure 2E.

      It would be interesting to see the raw movement metric data as shown in Figures 1 and 2 in the DTR mice to show that ablating orexin neurons does not impair the movement profile seen in Figures 1 and 2.

      The requested visualization has been added to Figure 4B.

      Validation that orexin was selectively ablated in these mice would be ideal.

      Histology (see response to public review) was added to a new Figure 4B.

      Figure 4A - OxLight expression in SNc does not look very robust.

      Please note this is a membrane-targeted indicator, the staining this produces is thus much weaker than cyctosolic indicators such as calcium indicator GCaMP.

      Figure 4 - It would be beneficial to see the same correlations that were done in Figures 1 and 2 to show OxLight activity vs. movement metric. Are they correlated?

      Individual traces had significant correlations with OxLight and movement, and the population averages revealed similar trends:

      Author response image 4.

      Figure 6B - Targeting of MVe neurons does not look very specific. The sample size for orexintargeted mice should be re-stated in the figure legend for clarity.

      Legend has been updated to clarify n = 15 for orexin targeted mice.

      Some citations didn't seem to match what was being referenced in the text. Similarly, in the legend for Figure 1C, the statistics do not match what is reported in the text. In Figure 1, the sample size is not noted in the text. When referring to running in Figure 1, is this referring to running speed? Perhaps the language could be more consistent.

      These typos (due to a rounding error) in the legend and text have been corrected. Sample size has been added to the text, and we have changed Figure 1D to clarify we are referring to running speed. We moved some citations to improve clarity.

      Methods - where were Cre mice obtained from?

      Sources now better referenced in Methods (JAX or Parlato et al).

      Figure 1, panel C: The authors compared Pearson's r-coefficient results for each animal and for each variable. However, it would be interesting to show the correlation curves for each variable. However, it would be interesting to show the correlation curves for each variable as well here. Also, there is mention of a strong correlation but it is unclear whether these correlations are significant.

      See below for an example mouse.

      Author response image 5.

      Figure 3, panel F: I understand HON-DTR is a validated model but a picture orexin ablation is necessary, including pictures of orexin fibers ablation within the SNc and LC.

      See our reply to the public review above.

      Figure 5, Panel A: Same comment as Figure 1, panel C.

      We have similarly clarified the panel and legend.

      Page 4: The authors mention "Within the 1st and 4th quartile of blood glucose, movement-HON correlations were not significantly different. Please add the figures.

      The requested plot has been added to Figure 6, panel G.

      Reviewer #3 (Public review):

      Summary

      The study presents an investigation into how hypothalamic orexin neurons (HONs) track body movement with high precision. Using techniques including fiber photometry, video-based movement metrics, and empirical mode decomposition (EMD), the authors demonstrate that HONs encode net body movement consistently across a range of behaviors and metabolic states. They test the ability of HONs to track body movement to that of other subcortical neural populations, from which they distinguish HONs activity from other subcortical neural populations.

      Strengths:

      The study characterizes HONs activity as key indicators of movement and arousal, and this method may have potential implications for understanding sleep disorders, energy regulation, and brain-body coordination. Overall, I think this is a very interesting story, with novel findings and implications about sensorimotor systems in animals. The manuscript is clearly written and the evidence presented is rigorous. The conclusions are well supported by experimental data with clear statistical analyses.

      We thank the reviewer for their supportive feedback.

      Weaknesses/suggestions:

      There are a couple of issues I think the authors could address to make the paper better and more complete:

      (1) The study primarily focuses on steady-state behaviors. It would be interesting if the authors' current dataset allows analyses of HON dynamics during transitions between behavioral states (e.g., resting to running or grooming to sniffing). This could provide additional insights into how HONs adapt to rapid changes in body movement.

      This is a fantastic idea, and easy to check using our classification CNN. We identified the six most frequent behavioral transitions and plotted them in Figure 2H. HONs show rapid dynamics in activity aligned with behavioral changes.

      These changes are very similar to the movement magnitude along these transitions, which is now also plotted in Figure 2G.

      (2) Given the established role of HONs in arousal and wakefulness, the study could further investigate how movement-related HON dynamics interact with arousal states. For example, does HON encoding of movement differ during sleep versus wakefulness?

      To further investigate how movement encoding interacts with arousal, we now include quantification and analysis of pupil-linked arousal (see new Figure 7). We agree it would be interesting to look at what happens during sleep, especially REM sleep when some HONs are thought to be active where there is no/little body movement, but this is beyond the scope of the present study.

      (3) Although HON ablation experiments suggest that HONs do not shape movement frequency profiles. It would be more compelling if the authors could investigate whether HONs contribute to specific types of movements (e.g., fine motor vs. gross motor movements) or modulate movement initiation thresholds.

      We performed this analysis using the k-means classifier for small/large movements. Consistent with previous results, we found no significant effect (p = 0.2767) of genotype on the frequency of identified small (fine) or large (gross) movement clusters. This plot has been added to Figure 4E.

      (4) The heterogeneous movement-related orexin dynamics observed in the LC and SNc raise intriguing questions about the circuit-level mechanisms underlying these differences. Optogenetic or chemogenetic manipulation of these projections could validate the functional implications of these dynamics.

      We agree. We now discuss some implications of this in revised Discussion (paragraph 4). Please note that previous work already demonstrated that orexin action in the SNc can produce locomotion (referenced in the paragraph), though we agree that further work would be valuable.

      Reviewer #3 (Recommendations for the authors):

      Additional feedback:

      (1) Figure 1C: the individual data points are hard to track or see. Consider using a larger marker face to help data visualization. Similar issues can be found in Figures 2C, 2E, 5E, 6C, 6F, and 6G.

      Thickness of the lines and scatterplots have been increased.

      (2) First Section of Results: the authors claim to use a deep-learning network to automatically classify video recordings into five distinct behaviors. However, several issues need to be addressed here:

      a. In Results, the corresponding sentence lacks a reference to the Methods Section.

      Reference has been added to the text.

      b. In Methods, the description of the CNN model is quite limited, lacking many basic, necessary components including necessary references to published papers, the model training, characterization (only an overall accuracy is not enough), as well as dataset definition, preparation, augmentation (if any), etc.

      We have expanded the methods section regarding the CNN model.

      (3) First Section of Results: in the second paragraph, the authors claim that "Overall, these results reveal HON population activity precisely tracks a general degree of body movement across recorded behaviors." This is not accurate. To indicate that HONs activity tracks the general degree of body movement across behavior states, they need to further show that behavioral states with similar levels of movement metrics can be differentiated via HON activities. However, as they showed in Figure 2D, some behaviors with similar values of movement metric do not seem to be easily discerned by HON activity levels.

      We agree with you, and this is also what we originally intended to convey – now reworded for clarity.

      (4) Technical issue: Figures 3B, 3C, 3G, using local regression to plot the solid lines makes them touch negative values, which does not make sense for "power proportion" (this quantity is always non-negative).

      This is a good point. To fix this, we first log-transformed the power metric, then performed a local regression, and used the link function to transform the model predictions back to %-units for visualization. This has been noted in the methods.

      (5) Figure 3G: For a better comparison, consider combining the two plots into a single plot.

      The two plots have been merged as shown in Figure 4C.

      (6) Figure 5E: For a better data visualization, the current pair of plots can be consolidated into one single plot where the x-axis is Move and the y-axis is dGlu. In this way, it is easier to understand and the orthogonality as claimed in the manuscript can be more apparent.

      The requested plot has been added as Figure 6F.

    1. Author response:

      The following is the authors’ response to the original reviews

      Reviewer #1 (Public review):

      Summary:

      This study takes a detailed approach to understanding the effect of menopausal hormone therapy (MHT) in the brain aging of females. Neuroimaging data from the UK Biobank is used to explore brain aging and shows an unexpected effect of current MHT use and poorer brain health outcomes relative to never users. There is considerable debate about the benefits of MHT and estrogens in particular for brain health, and this analysis illustrates that the effects are certainly not straightforward and require greater consideration.

      Strengths:

      (1) The detailed approach to obtaining important information about MHT use from primary care records. Prior studies have suggested that factors such as estrogen/progestin type, route of administration, duration, and timing of use relative to menopause onset can contribute to whether MHT benefits brain health.

      (2) Consideration of type of menopause (spontaneous, or surgical) in the analysis, as well as sensitivity diagnoses to rule out the effect being driven by those with clinical conditions.

      (3) The incorporation of the brain age estimate along with hippocampal volume to address brain health.

      (4) The complex data are also well explained and interpretations are reasonable.

      (5) Limitations of the UK Biobank data are acknowledged

      We thank the reviewer for their time and the positive evaluation of our manuscript.

      Weaknesses:

      (1) Lifestyle factors are listed and the authors acknowledge group differences (at least between current users and never users of MHT). I was not able to find these analyses showing these differences.

      We highlighted and tested for group differences in lifestyle scores, and the results are shown in Table 1-3, column p-value. As highlighted in the method section (page 9): “The lifestyle score was calculated using a published formula (69), and included data on sleep, physical activity, nutrition, smoking, and alcohol consumption (see supplementary Note 3, Table S2)”. In line with reviewer 1 suggestion to the authors, we now included an additional table testing for group differences in the specific lifestyle factors constituting the lifestyle score in the supplementary materials (Table S2). Please find a more detailed response below (Recommendations for the authors, Response to Comment 1).

      (2) The distribution of women who were not menopausal was unequal across groups, and while the authors acknowledge this, one wonders to what extent this explains the observed findings.

      We agree with the reviewer that the unequal distribution of women across groups can influence the observed findings. We have made minor edits to highlight this important topic more explicitly in the discussion:

      Discussion (page 21): “Current MHT users were significantly younger than past- and never-users, and around 67 % were menopausal relative to over 80% in the past- and never-user groups. The unequal distribution of age and menopausal status across groups may have influenced the observed findings. For instance, a larger proportion of the current users might be in the perimenopausal phase, which is often associated with debilitating neurological and vasomotor symptoms (1). MHT is commonly prescribed to minimize such symptoms. Although MHT initiation during perimenopause has been associated with improved memory and hippocampal function, as well as lower AD risk later in life (15), the need for MHT might in itself be an indicator of neurological changes (71); here potentially reflected in higher BAG and lower hippocampal volumes. After the transition to menopause, symptoms might subside and some perimenopausal brain changes might revert or stabilize in the postmenopausal phase 5. Although the UK Biobank lacks detailed information on menopausal symptoms and perimenopausal staging, our results might be capturing subtle disturbances during perimenopause that later stabilize. This could explain why the largely postmenopausal groups of past MHT users and never-users present with lower GM and WM BAG than the current user group. Considering the critical window hypothesis emphasizing perimenopause as a key phase for MHT action (29,43), future longitudinal studies are crucial to clarify the interplay between neurological changes and MHT use across the menopause transition.”

      Discussion (page 25): “In addition, previous studies highlight that UK Biobank participants are considered healthier than the general population based on several lifestyle and health-related factors (89, 90). This healthy volunteer bias increases with age, likely resulting in a disproportionate number of healthier older adults. Together with the imbalance in age distributions across groups, this might explain the less apparent brain aging in the older MHT user groups. We have previously highlighted that age is negatively associated with the number of APOE ε4 carriers in the UK Biobank (21), which is indicative of survivor bias.”

      (3) While the interpretations are reasonable, and relevant theories (healthy cell & critical window) are mentioned, the discussion is missing a more zoomed-out perspective of the findings. While I appreciate wanting to limit speculation, the reader is left having to synthesize a lot of complex details on their own. A particularly difficult finding to reconcile is under what conditions these women benefit from MHT and when do they not (and why that may be).

      We thank the reviewer for this comment. As the presented data is cross-sectional and does not enable causal inference, we have refrained from a more zoomed-out interpretation of the results to avoid undue speculations. However, where applicable, we have discussed our findings in a broader context such as the effects of MHT use on the brain across the menopausal transition (discussion page 21) and the effects of MHT use on the brain in the presence and absence of bilateral oophorectomy and/or hysterectomy (discussion page 25).

      To best inform the reader about the scope of our paper, we would like to highlight the following sentences in our discussion (page 24):

      “The current work represents the most comprehensive study of detailed MHT data, APOE ε4 genotype, and several brain measures in a large population-based cohort to date. Overall, our findings do not unequivocally support general neuroprotective effects of MHT, nor do they indicate severe adverse effects of MHT use on the female brain. The results suggest subtle yet complex relationships between MHT’s and brain health, highlighting the necessity for a personalized approach to MHT use. Importantly, our analyses provide a broad view of population-based associations and are not designed to guide individual-level decisions regarding the benefits versus risks of MHT use.”

      And the conclusion (page 25): “In conclusion, our findings suggest that associations between MHT use and female brain health might vary depending on duration of use and past surgical history. Although the effect sizes were generally modest, future longitudinal studies and RCTs, particularly focused on the perimenopausal transition window, are warranted to fully understand how MHT use influences female brain health. Importantly, considering risks and benefits, decisions regarding MHT use should be made within the clinical context unique to each individual.”

      Reviewer #1 (Recommendations for the authors):

      Can the authors provide:

      (1) More information about which aspects of lifestyle factors were different between the groups, and how these factors may have contributed to the observed findings (if possible, without burying this information in the supplemental)?

      We thank the reviewer for this suggestion. We now added a table comparing lifestyle factors contained in the lifestyle score by MHT user status using t-tests (continuous variables) or χ2 tests (see Table S2). The results are referred to in the main manuscript result section under “Sample characteristics”, and the table (Table S2) is provided in the supplements not to overburden the main text, in line with input from reviewer 3.

      We updated the main text to refer to Table S2 and updated the supplementary Note 3 (page 2-3) to include the results of the comparison of the lifestyle factors contained in the lifestyle score by MHT user status.

      Methods, page 9:“The lifestyle score was calculated using a published formula (69), and included data on sleep, physical activity, nutrition, smoking, and alcohol consumption (see supplementary Note 3, Table S2).”

      Results, page 13: “Sample demographics including lifestyle score, stratified by MHT user group, surgical history among MHT users, and estrogen only MHT or combined MHT use, are summarized in Table 1, 2 and 3, respectively. MHT user group differences for each lifestyle factor contained in the lifestyle score are shown in Table S2.”

      “Note 3| Lifestyle Score

      The lifestyle score was calculated based on sleep duration, time spent watching television, current and past smoking status, alcohol consumption frequency, physical activity level (number of days per week of moderate/vigorous activity for at least 10 minutes), intake of fruits and vegetables, and intake of oily fish, beef, lamb/mutton, pork and processed meat (for details see (10)). Each unhealthy lifestyle factor was scored with 1 point (e.g., smoking), and participants points were summed to generate an unweighted score (from 0-9): the higher the lifestyle score, the unhealthier the participant’s lifestyle.

      A comparison of the lifestyle factors contained in the lifestyle score by MHT user status is presented in Table S2. In summary, we found that current MHT were more often smokers than never-users, had a higher alcohol intake than never- and past MHT users, reported the lowest fruit and vegetable intake relative to never-users and past MHT users, and stated lower moderate activity levels relative to past MHT users. Past MHT users reported higher alcohol intake than never-users, spend more time watching TV relative to never- and current-users, consumed more beef, pork, lamb/mutton, and processed meat than never-users, and reported lower vigorous activity levels relative to never-users. However, oily fish intake and fruit and vegetable intake was higher among past MHT users relative to never-and current-users. Self-reported sleep duration did not differ between MHT user groups.”

      (2) A greater description of the 2 main theories of MHT effects on the brain (healthy cell vs critical window). Can the authors also provide a more thorough explanation for how the findings fit with these theories.

      We thank the reviewer for this comment. We have described our findings in the context of the critical window hypothesis (discussion, page 21, paragraph 2), the healthy cell bias hypothesis (discussion, page 22, paragraph 3), and healthy user bias hypothesis (discussion, page 22, paragraph 4). We refrained from a more thorough explanation to avoid undue speculations.

      (3) Reflect more on what the findings may indicate as to who benefits from MHT, and why. There are some references that the authors may want to add, particularly related to recent findings from premenopausal bilateral oophortectomies that also speak to when (and for whom) MHT use might benefit.

      We thank the reviewer for this feedback. We have included additional references in the revised manuscript as follows:

      Discussion, page 23: “It is also possible that the timing between MHT use and surgery is more tightly controlled and therefore more beneficial for brain aging (43). For instance, studies suggest that MHT may mitigate the potential long-term adverse effects of bilateral oophorectomy before natural menopause on bone mineral density as well as cardiovascular, cognitive and mental health (79-81). In addition, a 2024 UK Biobank study found that ever used MHT was associated with decreased odds of Alzheimer’s disease in women with bilateral oophorectomy (82).”  

      (79) Blumel JE, Arteaga E, Vallejo MS, et al. Association of bilateral oophorectomy and menopause hormone therapy with mild cognitive impairment: the REDLINC X study. Climacteric 2022;25:195-202.

      (80) Kaunitz AM, Kapoor E, Faubion S. Treatment of Women After Bilateral Salpingo-oophorectomy Performed Prior to Natural Menopause. JAMA 2021;326:1429-1430.

      (81) Stuursma A, Lanjouw L, Idema DL, de Bock GH, Mourits MJE. Surgical Menopause and Bilateral Oophorectomy: Effect of Estrogen-Progesterone and Testosterone Replacement Therapy on Psychological Well-being and Sexual Functioning; A Systematic Literature Review. J Sex Med 2022;19:1778-1789.

      (82) Calvo N, McFall GP, Ramana S, et al. Associated risk and resilience factors of Alzheimer's disease in women with early bilateral oophorectomy: Data from the UK Biobank. J Alzheimers Dis 2024;102:119-128.

      Reviewer #2 (Public review):

      Summary:

      In this observational study, Barth et al. investigated the association between menopausal hormone therapy and brain health in middle- to older-aged women from the UK Biobank. The study evaluated detailed MHT data (never, current, or past user), duration of mHT use (age first/last used), history of hysterectomy with or without bilateral oophorectomy, APOEE4 genotype, and brain characteristics in a large, population-based sample. The researchers found that current mHT use (compared to never-users), but not past use, was associated with a modest increase in gray and white matter brain age gap (GM and WM BAG) and a decrease in hippocampal volumes. No significant association was found between the age of mHT initiation and brain measures among mHT users. Longer duration of use and older age at last MHT use post-menopause were associated with higher GM and WM BAG, larger WMH volumes, and smaller hippocampal volumes. In a sub-sample, after adjusting for multiple comparisons, no significant associations were found between detailed mHT variables (formulations, route of administration, dosage) and brain measures. The association between mHT variables and brain measures was not influenced by APOEE4 allele carrier status. Women with a history of hysterectomy with or without bilateral oophorectomy had lower GM BAG compared to those without such a history. Overall, these observational data suggest that the association between mHT use and brain health in women may vary depending on the duration of use and surgical history.

      Strengths:

      (1) The study has several strengths, including a large, population-based sample of women in the UK, and comprehensive details of demographic variables such as menopausal status, history of oophorectomy/hysterectomy, genetic risk factors for Alzheimer's disease (APOE ε4 status), age at mHT initiation, age at last use, duration of mHT, and brain imaging data (hippocampus and WMH volume).

      (2) In a sub-sample, the study accessed detailed mHT prescription data (formulations, route of administration, dosage, duration), allowing the researchers to study how these variables were associated with brain health outcomes. This level of detail is generally missing in observational studies investigating the association of mHT use with brain health.

      We thank the reviewer for their time and the positive evaluation of our manuscript.

      Weaknesses:

      (1) While the study has many strengths, it also has some weaknesses. As highlighted in an editorial by Kantarci & Manson (2023), women with symptoms such as subjective cognitive problems, sleep disturbances, and elevated vasomotor symptoms combined with sleep disturbances tend to seek mHT more frequently than those without these symptoms. The authors of this study have also indicated that the need of mHT use which might be associated with these symptoms may be indicators of preexisting neurological changes, potentially reflecting worse brain health scores, including higher BAG and lower hippocampal volume and/or higher WMH. However, among current users, how many of these women have these symptoms could not be reported in the study. Women with these vasomotor symptoms who are using mHT are more likely to stay longer in the healthcare system compared with those without these symptoms and no MHT use history. The authors noted that the UK Biobank lacks detailed information on menopausal symptoms and perimenopausal staging, limiting the study's ability to understand how these variables influence outcomes.

      We thank the reviewer for the succint synopsis of the limitations highlighted in discussion, page 21. We have now added the mentioned reference, 2023 editoral by Kantarci & Manson, to the discussion as well (see reference 71).

      Discussion (page 21): “Current MHT users were significantly younger than past- and never-users, and around 67 % were menopausal relative to over 80% in the past- and never-user groups. The unequal distribution of age and menopausal status across groups may have influenced the observed findings. For instance, a larger proportion of the current users might be in the perimenopausal phase, which is often associated with debilitating neurological and vasomotor symptoms (1). MHT is commonly prescribed to minimize such symptoms. Although MHT initiation during perimenopause has been associated with improved memory and hippocampal function, as well as lower AD risk later in life (15), the need for MHT might in itself be an indicator of neurological changes (71); here potentially reflected in higher BAG and lower hippocampal volumes. After the transition to menopause, symptoms might subside and some perimenopausal brain changes might revert or stabilize in the postmenopausal phase 5. Although the UK Biobank lacks detailed information on menopausal symptoms and perimenopausal staging, our results might be capturing subtle disturbances during perimenopause that later stabilize. This could explain why the largely postmenopausal groups of past MHT users and never-users present with lower GM and WM BAG than the current user group. Considering the critical window hypothesis emphasizing perimenopause as a key phase for MHT action (29,43), future longitudinal studies are crucial to clarify the interplay between neurological changes and MHT use across the menopause transition.”

      (2)  Earlier observational studies have reported conflicting results regarding the association between mHT use and the risk of dementia and brain health. Contrary to some observational studies, three randomized trials (WHI, KEEPS, ELITE) (Espeland et al 2013, Gleason et al 2015; Henderson et al 2016) demonstrated neither beneficial nor harmful effects of mHT (with varying doses and formulations) when initiated closer to menopause (<5 years). While strong efforts were made to run proper statistical analyses to investigate the association between mHT use and brain health, these results reflect mainly associations, but not causal relationships as also stated by the authors.

      We thank the reviewer for pointing that out.

      (3)  Furthermore, observational studies have intrinsic limitations, such as a lack of control over switching mHT doses and formulations, a lack of laboratory measures to confirm mHT use, and reliance on self-reported data, which may not always be reliable. The authors caution that these findings should not guide individual-level decisions regarding the benefits versus risks of mHT use. However, the study raises new questions that should be addressed by randomized clinical trials to investigate the varying effects of MHT on brain health and dementia risk.

      We thank the reviewer for making our efforts in providing proper disclaimers in the discussion visible.

      Reviewer #2 (Recommendations for the authors):

      (1) The study could benefit from extending these findings by adding plasma biomarkers of AD and PET imaging markers to further study the association of mHT variables with brain health.

      We agree with the reviewer that such markers would be beneficial for elucidating the association between MHT variables and brain health. Unfortunately, these markers are not readily available in the UK Biobank.

      (2) The study's reliance on a predominantly white cohort limits the generalizability of the findings to more diverse populations. This homogeneity may not capture the full spectrum of responses to MHT across different ethnic and genetic backgrounds.

      We fully agree with the reviewers statement and state this limitation in the discussion (page 25) as follows:

      “In addition to these inherent biases in aging cohorts, the ethnic background of the sample is homogeneous (> 96% white), further reducing the generalizability of the results.”

      (3) The study may benefit by editing the following information in the introduction: "In summary, WHIMS, HERS, and KEEPS mainly relied on orally administered CEE in older-aged or recently postmenopausal females." KEEPS used two routes and formulations (transdermal estradiol and oCEE, both with micronized progesterone).

      We thank the reviewer for catching this oversight. We removed the sentence to avoid ambiguities and revised the sentence specifically refering to the KEEPS study as follows:

      Introduction, page 3: “In contrast, administering oral CEE or transdermal estradiol plus micronized progesterone in recently postmenopausal females did not alter cognition in the Kronos Early Estrogen Prevention Study (KEEPS) (28).”

      (4) The study may benefit by editing the following statement in the introduction: "oral CEE use in combination with MPA seems to increase the risk for AD regardless of timing": I would suggest revising this statement, which is based on review article 29. The statement of the adverse effect of oCEE regardless of the time of start contradicts earlier randomized clinical findings. I think it is important to make a distinction between the outcomes of randomized control trials and observational studies. The WMIHS (Shumaker et al., 2003) (randomized control trial) reported that there was an increased risk of dementia for women (who were more than 10 years from the onset of menopause when the therapy was initiated) in oCEE + MPA compared to placebo. Two other long-duration randomized trials tested the effect of oral oestrogen and progesterone treatment on cognitive function in women who started treatment shortly after menopause (within 3 or 6 years) did not find evidence that treatment benefits or harms cognitive function compared with placebo (Gleason et al., 2015; Henderson et al., 2016). A short-term (4 months) randomized trial (Maki et al 2007 (Maki et al., 2007) (mentioned in ref 29) reported a potential negative effect of CEE/MPA on verbal memory in women who started HT shortly after menopause (within 3 years). The study did not investigate the risk of dementia, and the duration of use of HT was short-term.

      We thank the reviewer for this detailed input. After checking the provided references, we rephrased the sentence as follows:

      Introduction, page 4:“Although emerging evidence supports this hypothesis (30, 31), oral CEE use in combination with MPA has been found to increase the risk for memory decline regardless of timing (26, 29, 32).”

      We believe this formulation is more in line with the evidence provided by Shumaker et al. 2003, Maki et al. 2007 and the other references provided in the review paper by Maki and colleagues (mentioned in ref. 29). The reviewer further refers to Gleason et al. 2015 and Henderson et al. 2016, however both RCTs use micronized progesterone, not MPA, thereby not supporting the statement.

      (26) Shumaker SA, Legault C, Rapp SR, et al. Estrogen plus progestin and the incidence of dementia and mild cognitive impairment in postmenopausal women: the Women's Health Initiative Memory Study: a randomized controlled trial. JAMA 2003;289:2651-2662.

      (29) Maki PM. Critical window hypothesis of hormone therapy and cognition: a scientific update on clinical studies. Menopause 2013;20:695-709.

      (32) Maki PM, Gast MJ, Vieweg AJ, Burriss SW, Yaffe K. Hormone therapy in menopausal women with cognitive complaints: a randomized, double-blind trial. Neurology 2007;69:1322-1330.

      Reviewer #3 (Public review):

      In this study Barth et al. present results of detailed analyses of the relationships between menopausal hormone therapy (MHT), APOE ε4 genotype, and measures of anatomical brain age in women in the UK Biobank. While past studies have investigated the links between some of these variables (including works by the authors themselves), this new study adds more detailed MHT variables, surgical status, and additional brain aging measures. The UK biobank sample is large, but it is a population cohort and many of the MHT measures are self-reported (as the authors point out). However, the authors present a solid analysis of the available information which shows associations between MHT user status, length of MHT use, as well as surgical status with brain age. However, as the authors themselves state, the results do not unequivocally support the neuroprotective or adverse effect of MHT on the brain. I think this work strengthens the case for the need of better-designed longitudinal studies investigating the effect of MHT on the brain in the peri/post-menopausal stage.

      Strengths:

      (1) The authors addressed the statistical analyses rigorously. For example, multiple testing corrections, outlier removal, and sensitivity analysis were performed carefully. Ample background information is provided in the introduction allowing even individuals not familiar with the field to understand the motivation behind the work. The discussion section also does a great job of addressing open questions and limitations. Very detailed results of all statistical tests are provided either in the main text or in the supplementary information.

      We thank the reviewer for their time and the positive evaluation of our manuscript.

      Weaknesses:

      (1) For me, the biggest weakness was the presentation of the results. As many variables are involved and past studies have investigated several of these questions, it would have helped to better clarify the analysis and questions that are addressed by this study in particular and what sets this work apart from past studies. The information is present in the manuscript but better organization might have helped. For example, a figure depicting the key questions near the beginning of the manuscript would have been very helpful for me. The Tables also contain a lot of information but I wonder if there might be a way to capture the most relevant information more succinctly (either in Table format or in a figure) for the main text.

      We thank the reviewer for this comment. We do agree that with the large number of analyses it can be hard to keep an overview. We now added a Figure summarizing the main and sensitity analyses by sample.

      (2) Another concern I had was the linear models investigating the effects of these MHT variables on the brain age gap. The authors have included "age" as one of the parameters in this analysis. I wonder if adding a quadratic age factor age2 in the model might have improved the fit since many brain phenotypes tend to show quadratic brain age effects in the 40 to 80-year age range.

      We thank the reviewer for this suggestion. We have rerun the main analysis in the whole sample (model 1) with age squared as an additional covariate, and compared the gray matter brain age gap model fits using the corrected Akaike Information Criterion (AIC). All models with age squared had a better model fit than models without age squared (see Author response table 1). Hence, in the revised manuscript, we added a sensitivity analysis rerunning the model 1 with age squared to account for potential non-linear effect. The results were largely consistent. The manuscript was revised as follows to reflect the added analysis:

      Sensitivity analysis (Methods, Page 11): “To test whether the results were influenced by the inclusion of participants with ICD-10 diagnosis or by non-linear effects of age, the main analyses (models 1-2) were re-run excluding the sub-sample with diagnosed brain disorders (see supplementary Note 2) or adding age(2) as additional covariate, respectively.”

      Sensitivity analysis (Results, Page 20): “The results were consistent after removing participants with ICD-10 diagnoses known to impact the brain (see Table S9 for model 1 analyses and Table S10 for model 2 analyses), after additionally adjusting for age(2) (see Table S11), and after removing extreme values (see Table S12 for model 1 analyses).”

      Author response table 1.

      Gray matter brain age gap model selection based on corrected Akaike Information Criterion (AICc)

      Abbreviations and explanations of parameters: MHT = menopausal hormone therapy, K = number of estimated parameters for each model, AICc = the information criterion requested for each model, ΔAICc = the appropriate delta AIC component depending on the information criteria selectedModelLik = the relative likelihood of the model given the data, AICcWT = Akaike weights to indicate the level of support in favor of any given model being the most parsimonious among the candidate model sets, LL = log-likelihood of each model.

      Reviewer #3 (Recommendations for the authors):

      (1) Please note typo in Figures 2 and 3 legend "GM WM".

      We thank the reviewer for catching this typo and we changed it to BAG GM and BAG WM for all Figures for consistency.

    1. Author response:

      The following is the authors’ response to the original reviews

      Public Reviews:

      Reviewer #1 (Public review):

      The chromophore molecule of animal and microbial rhodopsins is retinal which forms a Schiff base linkage with a lysine in the 7-th transmembrane helix. In most cases, the chromophore is positively charged by protonation of the Schiff base, which is stabilized by a negatively charged counterion. In animal opsins, three sites have been experimentally identified, Glu94 in helix 2, Glu113 in helix 3, and Glu181 in extracellular loop 2, where a glutamate acts as the counterion by deprotonation. In this paper, Sakai et al. investigated molecular properties of anthozoan-specific opsin II (ASO-II opsins), as they lack these glutamates. They found an alternative candidate, Glu292 in helix 7, from the sequences. Interestingly, the experimental data suggested that Glu292 is not the direct counterion in ASO-II opsins. Instead, they found that ASO-II opsins employ a chloride ion as the counterion. In the case of microbial rhodopsin, a chloride ion serves as the counterion of light-driven chloride pumps. This paper reports the first observation of a chloride ion as the counterion in animal rhodopsin. Theoretical calculation using a QM/MM method supports their experimental data. The authors also revealed the role of Glu292, which serves as the counterion in the photoproduct, and is involved in G protein activation.

      The conclusions of this paper are well supported by data, while the following aspects should be considered for the improvement of the manuscript.

      We thank the reviewer for carefully reading the manuscript and providing important suggestions. Below, we address the specific comments.

      (1) Information on sequence alignment only appears in Figure S2, not in the main figures. Figure S2 is too complicated by so many opsins and residue positions. It will be difficult for general readers to follow the manuscript because of such an organization. I recommend the authors show key residues in Figure 1 by picking up from Figure S2.

      We thank the reviewer for pointing this out. As suggested, we have selected key residues (potential counterion sites) from Fig. S2 and show them now as Fig. 1B in the revised manuscript. Fig. S2 has also been simplified by showing only the most important residues.

      (2) Halide size dependence. The authors observed spectral red-shift for larger halides. Their observation is fully coincident with the chromophore molecule in solution (Blatz et al. Biochemistry 1972), though the isomeric states are different (11-cis vs all-trans). This suggests that a halide ion is the hydrogen-bonding acceptor of the Schiff base N-H group in solution and ASO-II opsins. A halide ion is not the hydrogen-bonding acceptor in the structure of halorhodopsin, whose halide size dependence is not clearly correlated with absorption maxima (Scharf and Engelhard, Biochemistry 1994). These results support their model structure (Figure 4), and help QM/MM calculations.

      We appreciate the comment, which provides a deeper insight into our results and reinforces our conclusions. We have revised the discussion of the effect of halide size on the λ<sub>max</sub> shift to cite the prior work mentioned by the reviewer.

      (3) QM/MM calculations. According to Materials and Methods, the authors added water molecules to the structure and performed their calculations. However, Figure 4 does not include such water molecules, and no information was given in the manuscript. In addition, no information was given for the chloride binding site (contact residues) in Figure 4. More detailed information should be shown with additional figures in Figure SX.

      We thank the reviewer for making us realize that Fig. 4 was oversimplified.

      We have added following text in the “Structural modelling and QM/MM calculations of the dark state of Antho2a” section:

      Lines 220 – 223

      “The chloride ion is also coordinated by two water molecules and the backbone of Cys187 which is part of a conserved disulfide bridge (Fig. S2). The retinylidene Schiff base region also includes polar (Ser186, Tyr91) and non-polar (Ala94, Leu113) residues (Fig. 4).”

      We have updated Fig. 4 and its legend to show a more detailed environment of the protonated Schiff base and the chloride ion, including water molecules and other nearby residues.

      (4) Figure 5 clearly shows much lower activity of E292A than that of WT, whose expression levels are unclear. How did the authors normalize (or not normalize) expression levels in this experiment?

      We thank the reviewer for this valuable comment. In the previous version of the manuscript, we did not normalize the activity based on expression levels. We have considered this in the amended version.

      First, we evaluated the expression levels of wild type and E292A Antho2a by comparing absorbances at λ<sub>max</sub> (± 5 nm) of these pigments that were expressed and purified under the same conditions. Assuming that their molar absorption coefficients at the absorption maximum wavelengths are approximately the same, this can allow us to roughly compare their expression levels. The relative expression of the E292A mutant compared to the wild type (set as 1) was 0.81 at pH 6.5 and 140 mM NaCl, in which 94.0% (for E292A) and 99.8% (for wild type) of the Schiff base is protonated (Fig. 3A and B). As we conducted the live cell Ca<sup>2+</sup> assay in media at pH 7.0, we estimated the proportion of the protonated states of wild type and E292A mutant at same pH. The relative amounts of the protonated states to the wild type at pH 6.5 (set as 1) were estimated to be 0.99 for wild type and 0.84 for E292A. Together, the protonated pigment of the E292A mutant was calculated to be about 73% of that of the wild type at pH 7.0. From Fig. 5, the amplitude of Ca<sup>2+</sup> response of the E292A mutant was 12.1% of the wild type, showing that even after normalizing the expression levels, the Ca<sup>2+</sup> response amplitude was lower in the E292A mutant than in the wild type. This leads to our conclusion that the E292A mutation can also influence the G protein activation efficiency.

      We have added Fig. S11 showing the comparison of expression levels between the wild type and E292A of Antho2a (Fig. S11A) and maximum Ca<sup>2+</sup> responses after normalizing the expression levels (Fig. S11B).

      We have also revised the discussion section as follows:

      Lines 324 – 335

      “The relative expression level of the E292A mutant of Antho2a was approximately 0.81 of the wild type (set as 1), as determined by comparing absorbances at λ<sub>max</sub> for both pigments expressed and purified under identical conditions (Fig. S11A). Additionally, the fraction of protonated pigment relative to the wild type (set as 1 at pH 6.5) was estimated to be 0.94 for the E292A mutant at pH 6.5, and 0.99 and 0.84 for the wild type and the E292A mutant at pH 7.0, respectively (Fig. 3A and B). Since pH 7.0 corresponds to the conditions used in the live cell Ca<sup>2+</sup> assays, the effective amount of protonated pigment for the E292A mutant was approximately 73% of the wild type. Nevertheless, even after normalization for these differences, the Ca<sup>2+</sup> response amplitude of the E292A mutant remained significantly lower (~ 17% of wild type, compared to the observed 12% prior to normalization; Fig. 5 and Fig. S11B). These observations suggest that Glu292 serves not only as a counterion in the photoproduct but also plays an allosteric role in influencing G protein activation.”

      (5) The authors propose the counterion switching from a chloride ion to E292 upon light activation. A schematic drawing on the chromophore, a chloride ion, and E292 (and possible surroundings) in Antho2a and the photoproduct will aid readers' understanding.

      We thank the reviewer for this excellent suggestion. We have prepared a new figure with a schematic drawing of the environment of the protonated Schiff base depicting the counterion switch in Fig. S10.

      Reviewer #2 (Public review):

      Summary:

      This work reports the discovery of a new rhodopsin from reef-building corals that is characterized experimentally, spectroscopically, and by simulation. This rhodopsin lacks a carboxylate-based counterion, which is typical for this family of proteins. Instead, the authors find that a chloride ion stabilizes the protonated Schiff base and thus serves as a counterion.

      Strengths:

      This work focuses on the rhodopsin Antho2a, which absorbs in the visible spectrum with a maximum at 503 nm. Spectroscopic studies under different pH conditions, including the mutant E292A and different chloride concentrations, indicate that chloride acts as a counterion in the dark. In the photoproduct, however, the counterion is identified as E292.

      These results lead to a computational model of Antho2a in which the chloride is modeled in addition to the Schiff base. This model is improved using the hybrid QM/MM simulations. As a validation, the absorption maximum is calculated using the QM/MM approach for the protonated and deprotonated E292 residue as well as the E292A mutant. The results are in good agreement with the experiment. However, there is a larger deviation for ADC(2) than for sTD-DFT. Nevertheless, the trend is robust since the wt and E292A mutant models have similar excitation energies. The calculations are performed at a high level of theory that includes a large QM region.

      Weaknesses:

      I have a couple of questions about this study:

      We thank the reviewer for providing critical comments, particularly on the QM/MM calculations. We have carefully considered all comments and have addressed them as detailed below. Corresponding revisions have been made to the manuscript.

      (1) I find it suspicious that the absorption maximum is so close to that of rhodopsin when the counterion is very different. Is it possible that the chloride creates an environment for the deprotonated E292, which is the actual counterion?

      We think it is unlikely that the chloride ion merely facilitates deprotonation of Glu292 in such a way that it acts as the counterion of the dark state Antho2a. This conclusion is based on two results from our study. (1) λ<sub>max</sub> of wild type Antho2a in the dark is positively correlated with the ionic radius of the halide in the solution; the λ<sub>max</sub> is red shifted in the order Cl- < Br- < I- (Fig. 2E and F in the revised manuscript). This tendency is observed when the halide anion acts as a counterion of the protonated Schiff base (Blatz et al. Biochemistry 11: 848–855, 1972). (2) The QM/MM models of the dark state of Antho2a show that the calculated λ<sub>max</sub> of Antho2a with a protonated (neutral) Glu292 is much closer to the experimentally observed λ<sub>max</sub> than with a deprotonated (negatively charged) Glu292 (Fig. 4), suggesting that the Glu292 is likely to be protonated even in the presence of chloride ion. Therefore, we conclude that a solute anion, and not Glu292, acts as the counterion of the protonated Schiff base in the dark state of Antho2a. We have discussed this in the revised manuscript as follows:

      Lines 274 – 291

      “We found that the type of halide anions in the solution has a small but noticeable effect on the λ<sub>max</sub> values of the dark state of Antho2a. This is consistent with the effect observed in a counterion-less mutant of bovine rhodopsin, in which halide ions serve as surrogate counterions (Nathans, 1990; Sakmar et al., 1991). Similarly, our results align with earlier observations that the λ<sub>max</sub> of a retinylidene Schiff base in solution increases with the ionic radius of halides acting as hydrogen bond acceptors (i.e., I− > Br− > Cl−) (Blatz et al., 1972). In contrast, the λ<sub>max</sub> of halorhodopsin from Natronobacterium pharaonic does not clearly correlate with halide ionic radius (Scharf and Engelhard, 1994), as the halide ion in this case is not a hydrogen-bonding acceptor of the protonated Schiff base (Kouyama et al., 2010; Mizuno et al., 2018). Altogether, these findings support our hypothesis that in Antho2a, a solute halide ion forms a hydrogen bond with the Schiff base, thereby serving as the counterion in the dark state. Moreover, QM/MM calculations for the dark state of Antho2a suggest that Glu292 is protonated and neutral, further supporting the hypothesis that Glu292 does not serve as the counterion in the dark state. However, unlike dark state, Cl− has little to no effect on the visible light absorption of the photoproduct (Fig. S5). Therefore, we conclude that Cl− and Glu292, respectively, act as counterions for the protonated Schiff base of the dark state and photoproduct of Antho2a. This represents a unique example of counterion switching from exogeneous anion to a specific amino acid residue upon light irradiation (Fig. S10).”

      (2) The computational protocol states that water molecules have been added to the predicted protein structure. Are there water molecules next to the Schiff base, E292, and Cl-? If so, where are they located in the QM region?

      We have updated Fig. 4 to show amino acids and water molecules near the Schiff base, E292, and the chloride ion. These include Ser186, Tyr91, Ala94, Leu113, Cys187, and two water molecules coordinating the chloride ion. We have added following text in the “Structural modelling and QM/MM calculations of the dark state of Antho2a” section of the revised manuscript.

      Lines 220 – 223

      “The chloride ion is also coordinated by two water molecules and the backbone of Cys187 which is part of a conserved disulfide bridge (Fig. S2). The retinylidene Schiff base region also includes polar (Ser186, Tyr91) and non-polar (Ala94, Leu113) residues (Fig. 4).”

      Water molecules, which have been modelled by homology to other GPCR structures, were not included in the QM region. In the revised version of the manuscript, we clarify this point in the “Computational modelling and QM/MM calculations” section as follows.

      Lines 515 – 517

      “The retinal-binding pocket also contains predicted water molecules (modelled based on homologous GPCR structures) close to the Schiff base and the chloride ion which were not included in the QM region.”

      (3) If the E292 residue is the counterion in the photoproduct state, I would expect the retinal Schiff base to rotate toward this side chain upon isomerization. Can this be modeled based on the recent XFEL results on rhodopsin?

      The recent XFEL studies of rhodopsin reveal that at very early stages (1 ps after photoactivation), structural changes in retinal are limited primarily to the isomerization around the C11=C12 bond of the polyene chain, without significant rotation of the Schiff base.

      Although modelling of a later active state with planar retinal and a rotated Schiff base is feasible—e.g., guided by high-resolution structures of bovine rhodopsin’s Meta II state such as PDB ID: 3PQR, see Author response image 1 below—active states of GPCRs typically exhibit substantial conformational flexibility and heterogeneity, making the generation of precise structural models suitable for accurate QM/MM calculations challenging. Despite these uncertainties, this preliminary modelling does indicate that upon isomerization to the all-trans configuration, the retinal Schiff base would rotate towards E292, supporting our hypothesis that E292 serves as the counterion in the Antho2a photoproduct. This is now shown better in the revised Fig. S10.

      Author response image 1.

      Reviewer #3 (Public review):

      Summary:

      The paper by Saito et al. studies the properties of anthozoan-specific opsins (ASO-II) from organisms found in reef-building coral. Their goal was to test if ASO-II opsins can absorb visible light, and if so, what the key factors involved are.

      The most exciting aspect of this work is their discovery that ASO-II opsins do not have a counterion residue (Asp or Glu) located at any of the previously known sites found in other animal opsins.

      This is very surprising. Opsins are only able to absorb visible (long wavelength light) if the retinal Schiff base is protonated, and the latter requires (as the name implies) a "counter ion". However, the authors clearly show that some ASO-II opsins do absorb visible light.

      To address this conundrum, they tested if the counterion could be provided by exogenous chloride ions (Cl-). Their results find compelling evidence supporting this idea, and their studies of ASO-II mutant E292A suggest E292 also plays a role in G protein activation and is a counterion for a protonated Schiff base in the light-activated form.

      Strengths:

      Overall, the methods are well-described and carefully executed, and the results are very compelling.

      Their analysis of seven ASO-II opsin sequences undoubtedly shows they all lack a Glu or Asp residue at "normal" (previously established) counter-ion sites in mammalian opsins (typically found at positions 94, 113, or 181). The experimental studies clearly demonstrate the necessity of Cl- for visible light absorbance, as do their studies of the effect of altering the pH.

      Importantly, the authors also carried out careful QM/MM computational analysis (and corresponding calculation of the expected absorbance effects), thus providing compelling support for the Cl- acting directly as a counterion to the protonated retinal Schiff base, and thus limiting the possibility that the Cl- is simply altering the absorbance of ASO-II opsins through some indirect effect on the protein.

      Altogether, the authors achieved their aims, and the results support their conclusions. The manuscript is carefully written, and refreshingly, the results and conclusions are not overstated.

      This study is impactful for several reasons. There is increasing interest in optogenetic tools, especially those that leverage G protein-coupled receptor systems. Thus, the authors' demonstration that ASO-II opsins could be useful for such studies is of interest.

      Moreover, the finding that visible light absorbance by an opsin does not absolutely require a negatively charged amino acid to be placed at one of the expected sites (94, 113, or 181) typically found in animal opsins is very intriguing and will help future protein engineering efforts. The argument that the Cl- counterion system they discover here might have been a preliminary step in the evolution of amino acid based counterions used in animal opsins is also interesting.

      Finally, given the ongoing degradation of coral reefs worldwide, the focus on these curious opsins is very timely, as is the authors' proposal that the lower Schiff base pKa they discovered here for ASO-II opsins may cause them to change their spectral sensitivity and G protein activation due to changes in their environmental pH.

      We thank the reviewer for the comprehensive summary of the manuscript and for finding it well-described and impactful.

      Recommendations for the Authors:

      Reviewer #1 (Recommendations for the authors):

      (1) p. 5, l. 102: The authors obtained three absorption spectra out of seven. Did the authors examine the reasons for no absorption spectra for the remaining four proteins?

      We have not identified the reasons for the absence of detectable absorption spectra for the remaining four opsins. We speculate that this could result from poor retinal binding under detergent-solubilized conditions, but we have not directly tested this possibility.

      (2) p. 7, l. 141: The pH value is 7.5 in the text and 7.4 in Figure S4B.

      We thank the reviewer for finding this mistake. The correct value is 7.4 and we have revised the text accordingly.

      Reviewer #2 (Recommendations for the authors):

      The structures and the simulations should be made available to the reader by providing them in a repository.

      We have deposited the Antho2a models in Zenodo (https://zenodo.org/; an open-access repository for research data). We have added the following description in the “Data and materials availability” section of the revised manuscript.

      Lines 559 – 560

      “The structural models of wild type Antho2a with a neutral or charged Glu292 and the Antho2a E292A mutant are available in Zenodo (10.5281/zenodo.15064942).”

      Reviewer #3 (Recommendations for the authors):

      (1) In the homology models for the ASO-II opsins, are there any other possible residues that could act as counter-ion residues outside of the "normal" positions at 94, 113, or 181?

      We have updated Fig. 4 to show all residues near the retinylidene Schiff base region, which include Cl−, Glu292, Ser186, Tyr91, Ala94, Leu113, Cys187, and two water molecules.

      Apart from Cl− and Glu292, the homology models of the ASO-II opsins do not reveal any other candidate as the counterion of Schiff base. This is also suggested by the sequence alignment between opsins of the ASO-II group and other animal opsins in Fig. S2, where we show amino acid residues near the Schiff base (in addition to key motifs important for G protein activation).

      (2) It is mentioned that the ASO-II opsins do not appear to be bistable opsins in detergents - do these opsins show any ability to photo-switch back and forth when in cellular membranes?

      We have not directly tested whether Antho2a exhibits photo-switching in cellular membranes due to technical limitations associated with high light scattering in spectroscopic measurements. Instead, we recorded absorption spectra from crude extracts of detergent-solubilized cell membranes expressing Antho2a wild type (without purification) in the dark and after sequential light irradiation (Fig. S3C). This approach, which retains cellular lipids, can better preserve the photochemical properties of opsins, such as thermal stability and photoreactivity of their photoproducts, similar to intact cellular membranes. The first irradiation with green light (500 nm) led to a decrease in absorbance around the 550 nm region and an increase around the 450 nm region, indicating the formation of a photoproduct, consistent with observations using purified Antho2a.

      However, subsequent irradiation with violet light (420 nm) did not reverse these spectral changes but resulted in only a slight decrease in absorbance around 400 nm. Re-exposure to green light produced no further spectral changes aside from baseline distortions. These findings suggest that the Antho2a photoproduct has limited ability to revert to its original dark state under these conditions. Nevertheless, because detergent solubilization may influence these observations, further studies in intact cellular membranes using live-cell assay will be required to conclusively assess bistability or photo-switching properties.

      (3) The idea that E292 acts as a counterion for the protonated active state is intriguing - do the authors think the retinal decay process after light activation occurs with hydrolysis of the non-protonated form with subsequent retinal release?

      We thank the reviewer for raising this important question. We first examined whether the increased UV absorbance observed after incubating the photoproduct for 20 hours in the dark (Fig. S3D, E, violet curves) originated from free retinal released from the opsin pigment. Acid denaturation (performed at pH 1.9) of this photoproduct resulted in a main product absorbing around 400 nm (Fig. S3G). Typically, when retinal binds opsin via the Schiff base (whether protonated or deprotonated), acid denaturation traps the retinal chromophore as a protonated Schiff base, yielding an absorption spectrum with a λ<sub>max</sub> at approximately 440 nm, as observed in the dark state of Antho2a (Fig. S3F). Our results thus indicate that the UV absorbance in the photoproduct did not result from a deprotonated Schiff base but rather from retinal released during incubation. We have not directly tested whether the protonated or deprotonated form is more prone to retinal release. However, the decay of visible absorbance (associated with the protonated photoproduct) occurred more rapidly under alkaline conditions (pH 8.0), which generally favors deprotonation of the Schiff base (Fig. S3H). Thus, it is possible that the deprotonated photoproduct releases retinal more rapidly than the protonated form, but further studies are necessary to confirm this hypothesis.

      To answer the comments (2) and (3) by the reviewer, we have added new panels (C and F–H) to Fig. S3.

      We have revised the Results section as follows:

      Lines 136 – 141

      “The photoproduct remained stable for at least 5 minutes (Fig. S3A, curves 2 and 3) but did not revert to the original dark state upon subsequent irradiation (Fig. S3A and C). Instead, it underwent gradual decay accompanied by retinal release over time (Fig. S3D–G). These findings indicate that purified Antho2a is neither strictly bleach resistant nor bistable (see also Fig. S3 legend). We also observed that the protonated photoproduct decayed more rapidly at pH 8.0 (Fig. S3H) than at pH 6.5 (Fig. 3A, D, E).”

      Text:

      (4) Page 3, line 38. Consider defining eumetazoan (for lay readers).

      As suggested, we have defined eumetazoans and revised the sentence as follows:

      Lines 38 – 40

      “Opsins are present in the genomes of all eumetazoans (i.e., all animal lineages except sponges), and based on their phylogenetic relationships, they can be classified into eight groups…”

      (5) Page 3, line 42. "But, furthermore, ..." should be changed to either word alone.

      Revised as suggested.

      (6) Page 18, line 447. The HPLC method is well-described and helpful. If possible, please add a Reference, or indicate if this is a new variation of the method.

      This is a well-established method for analyzing the composition of retinal isomers bound to different states of rhodopsin pigments. We have now cited a reference describing the methodology (Terakita et al. Vision Res. 6: 639–652, 1989).

      (7) Page 11, line 267. "..type of halide anions in the solution affected the λ<sub>max</sub> values of the dark state of".

      Since the changes are not large (but clearly occur), consider changing this sentence to "..type of halide anions in the solution has a small but visible effect on the λ<sub>max</sub> values of the dark state ..."

      We have revised this sentence as suggested.

      Figures:

      (9) Consider combining Figure FS6 with Figure 2 (effect of anions on visible absorbance).

      As suggested, the previous Fig. S6 has been included in the main text as Fig. 2E and F in the revised manuscript.

    1. Author response:

      The following is the authors’ response to the original reviews.

      Public Reviews:

      Reviewer #1 (Public Review):

      The manuscript by Li et al. investigates the metabolism-independent role of nuclear IDH1 in chromatin state reprogramming during erythropoiesis. The authors describe accumulation and redistribution of histone H3K79me3, and downregulation of SIRT1, as a cause for dyserythropoiesis observed due to IDH1 deficiency. The authors studied the consequences of IDH1 knockdown, and targeted knockout of nuclear IDH1, in normal human erythroid cells derived from hematopoietic stem and progenitor cells and HUDEP2 cells respectively. They further correlate some of the observations such as nuclear localization of IDH1 and aberrant localization of histone modifications in MDS and AML patient samples harboring IDH1 mutations. These observations are intriguing from a mechanistic perspective and they hold therapeutic significance, however there are major concerns that make the inferences presented in the manuscript less convincing.

      (1) The authors show the presence of nuclear IDH1 both by cell fractionation and IF, and employ an efficient strategy to knock out nuclear IDH1 (knockout IDH1/ Sg-IDH1 and rescue with the NES tagged IDH1/ Sg-NES-IDH1 that does not enter the nucleus) in HUDEP2 cells. However, some important controls are missing.

      A) In Figure 3C, for IDH1 staining, Sg-IDH1 knockout control is missing.

      Thanks for the reviewer’s suggestion. We have complemented the staining of Sg-IDH1 knockout cells, and made corresponding revision in Figure 3C in the revised manuscript.

      B) Wild-type IDH1 rescue control (ie., IDH1 without NES tag) is missing to gauge the maximum rescue that is possible with this system.

      Thanks for the reviewer’s suggestion. We have overexpressed wild-type IDH1 in the IDH1-deficient HUDEP2 cell line and detected the phenotype. The results are presented in Supplementary Figure 9 in the revised manuscript. As shown in Supplementary Figure 9A, IDH1 deficiency resulted in reduced cell number in HUDEP2 cells, a phenotype that was rescued by overexpression of wild-type IDH1 but not by NES-IDH1. Given IDH1's well-established role in redox homeostasis through catalyzing isocitrate to α-KG conversion, we hypothesized that both wild-type IDH1 and NES-IDH1 overexpression would significantly restore α-KG levels compared to the IDH1-deficient group. Supplementary Figure 9B demonstrates that IDH1 depletion resulted in a dramatic decrease in α-KG levels, whereas overexpression of either wild-type IDH1 or NES-IDH1 almost completely restored α-KG levels, as anticipated. These results suggest that wild-type IDH1 overexpression can restore metabolic regulatory functions as effectively as NES-IDH1 overexpression. To investigate whether apoptosis contributes to the impaired cell expansion caused by IDH1 deficiency, we performed Annexin V/PI staining to quantify apoptotic cells. As shown in Supplementary Figure 9C and D, flow cytometry analysis revealed no significant changes in apoptosis rates following either IDH1 depletion or ectopic expression of wild-type IDH1 or NES-IDH1 in IDH1 deficient HUDEP2 cells.

      Flow cytometric analysis demonstrated that IDH1 deficiency triggered S-phase accumulation at day 8, indicative of cell cycle arrest. Whereas ectopic expression of wild-type IDH1 significantly rescued this cell cycle defect, overexpression of NES-IDH1 failed to ameliorate the S-phase accumulation phenotype induced by IDH1 depletion, as presented in Supplementary Figure 9E and F. Although NES-IDH1 overexpression rescued metabolic regulatory function defect, it failed to alleviate the cell cycle arrest induced by IDH1 deficiency. In contrast, wild-type IDH1 overexpression fully restored normal cell cycle progression. This functional dichotomy demonstrates that nuclear-localized IDH1 executes critical roles distinct from its cytoplasmic counterpart, and overexpression of wild-type IDH1 could efficient restore the functional impairment induced by depletion of nuclear localized IDH1.

      (2) Considering the nuclear knockout of IDH1 (Sg-NES-IDH1 referenced in the previous point) is a key experimental system that the authors have employed to delineate non-metabolic functions of IDH1 in human erythropoiesis, some critical experiments are lacking to make convincing inferences.

      A) The authors rely on IF to show the nuclear deletion of Sg-NES-IDH1 HUDEP2 cells. As mentioned earlier since a knockout control is missing in IF experiments, a cellular fractionation experiment (similar to what is shown in Figure 2F) is required to convincingly show the nuclear deletion in these cells.

      We sincerely thank the reviewer for raising this critical point. As suggested, we have performed additional IF experiments and cellular fractionation experiments to comprehensively address the subcellular localization of IDH1.

      The results of IF staining were shown in Figure 3C of the revised manuscript. In Control HUDEP2 cells, endogenous IDH1 was detected in both the cytoplasm and nucleus. This dual localization may reflect its dynamic roles in cytoplasmic metabolic processes and potential nuclear functions under specific conditions. In Sg-IDH1 cells (IDH1 knockout), IDH1 signal was undetectable, confirming efficient depletion of the protein. In Sg-NES-IDH1 cells (overexpressing NES-IDH1 in IDH1 deficient cells), IDH1 predominantly accumulated in the cytoplasm, consistent with the disruption of its nuclear export signal. The dual localization of IDH1 that was determined by IF staining experiment were then further confirmed by cellular fractionation assays, as shown in Figure 3D.

      B) Since the authors attribute nuclear localization to a lack of metabolic/enzymatic functions, it is important to show the status of ROS and alpha-KG in the Sg-NES-IDH1 in comparison to control, wild type rescue, and knockout HUDEP2 cells. The authors observe an increase of ROS and a decrease of alpha-KG upon IDH1 knockdown. If nuclear IDH1 is not involved in metabolic functions, is there only a minimal or no impact of the nuclear knockout of IDH1 on ROS and alpha-KG, in comparison to complete knockout? These studies are lacking.

      We appreciate the insightful suggestions of the reviewers and agree that the detection of ROS and alpha-KG is useful for the demonstration of the non-canonical function of IDH1. We examined alpha-KG concentrations in control, IDH1 knockout and nuclear IDH1 knockout HUDEP2 cell lines. The results showed a significant decrease in alpha-KG content after complete knockout of IDH1, whereas there was no significant change in nuclear knockout IDH1 (Supplementary Figure 9B). As to the detection of ROS level, the commercial ROS assay kits that we can get are detected using PE (Excitation: 565nm; Emission: 575nm) and FITC (Excitation: 488nm; Emission: 518nm) channels in flow cytometry. We constructed HUDEP2 cell lines of Sg-IDH1 and Sg-NES-IDH1 to express green fluorescent protein (Excitation: 488nm; Emission: 507nm) and Kusabira Orange fluorescent protein (Excitation: 500nm; Emission: 561nm) by themselves. Unfortunately, due to the spectral overlap of the fluorescence channels, we were unable to detect the changes in ROS levels in these HUDEP2 cell lines using the available commercial kit.

      (3) The authors report abnormal nuclear phenotype in IDH1 deficient erythroid cells. It is not clear what parameters are used here to define and quantify abnormal nuclei. Based on the cytospins (eg., Figure 1A, 3D) many multinucleated cells are seen in both shIDH1 and Sg-NES-IDH1 erythroid cells, compared to control cells. Importantly, this phenotype and enucleation defects are not rescued by the administration of alpha-KG (Figures 1E, F). The authors study these nuclei with electron microscopy and report increased euchromatin in Figure 4B. However, there is no discussion or quantification of polyploidy/multinucleation in the IDH1 deficient cells, despite their increased presence in the cytospins.

      A) PI staining followed by cell cycle FACS will be helpful in gauging the extent of polyploidy in IDH1 deficient cells and could add to the discussions of the defects related to abnormal nuclei.

      We appreciate the reviewer’s insightful suggestion. Since PI dye is detected using the PE channel (Excitation: 565nm; Emission: 575nm) of the flow cytometer and the HUDEP2 cell line expresses Kusabira orange fluorescent protein (Excitation: 500nm; Emission: 561nm), we were unable to use PI staining to detect the cell cycle. Edu staining is another commonly used method to determine cell cycle progression, and we performed Edu staining followed by flow cytometry analysis on Control, Sg-IDH1 and Sg-NES-IDH1 HUDEP2 cells, respectively. The results showed that complete knockdown of IDH1 resulted in S-phase block and increased polyploidy in HUDEP2 cells on day 8 of erythroid differentiation, and overexpression of IDH1-NES did not reverse this phenotype (Supplemental Figure 9E-F). Moreover, we have added a discussion of abnormal nuclei being associated with impaired erythropoiesis.

      B) For electron microscopy quantification in Figures 4B and C, how the quantification was done and the labelling of the y-axis (% of euchromatin and heterochromatin) in Figure 4 C is not clear and is confusingly presented. The details on how the quantification was done and a clear label (y-axis in Figure 4C) for the quantification are needed.

      Thanks for the reviewer's suggestion. In this study, we calculated the area of nuclear, heterochromatin and euchromatin by using Image J software. We addressed the quantification strategy in the section of Supplementary methods of the revised Supplementary file. In addition, the y-axis label in Figure 4C was changed to “the area percentage of euchromatin and heterochromatin’’.

      C) As mentioned earlier, what parameters were used to define and quantify abnormal nuclei (e.g. Figure 1A) needs to be discussed clearly. The red arrows in Figure 1A all point to bi/multinucleated cells. If this is the case, this needs to be made clear.

      We thank the reviewer for their suggestion. In our present study, nuclear malformations were defined as cells exhibiting binucleation or multinucleation based on cytospin analysis. A minimum of 300 cells per group were evaluated, and the proportion of aberrant nuclei was calculated as (number of abnormal cells / total counted cells) × 100%.

      (4) The authors mention that their previous study (reference #22) showed that ROS scavengers did not rescue dyseythropoiesis in shIDH1 cells. However, in this referenced study they did report that vitamin C, a ROS scavenger, partially rescued enucleation in IDH1 deficient cells and completely suppressed abnormal nuclei in both control and IDH1 deficient cells, in addition to restoring redox homeostasis by scavenging reactive oxygen species in shIDH1 erythroid cells. In the current study, the authors used ROS scavengers GSH and NAC in shIDH1 erythroid cells and showed that they do not rescue abnormal nuclei phenotype and enucleation defects. The differences between the results in their previous study with vitamin C vs GSH and NAC in the context of IDH1 deficiency need to be discussed.

      We appreciate the reviewer’s insightful observation. The apparent discrepancy between the effects of vitamin C (VC) in our previous study and glutathione (GSH)/N-acetylcysteine (NAC) in the current work can be attributed to divergent molecular mechanisms beyond ROS scavenging. A growing body of evidence has identified vitamin C as a multifunctional regulator. In addition to acting as an antioxidant maintaining redox homeostasis, VC also acts as a critical epigenetic modulator. VC have been identified as a cofactor for α-ketoglutarate (α-KG)-dependent dioxygenases, including TET2, which catalyzes 5-methylcytosine (5mC) oxidation to 5-hydroxymethylcytosine (5hmC) [1,2]. Structural studies confirm its direct interaction with TET2’s catalytic domain to enhance enzymatic activity in vitro [3]. The biological significance of the epigenetic modulation induced by vitamin C is illustrated by its ability to improve the generation of induced pluripotent stem cells and to induce a blastocyst-like state in mouse embryonic stem cells by promoting demethylation of H3K9 and 5mC, respectively [4,5]. In contrast, GSH and NAC are canonical ROS scavengers lacking intrinsic epigenetic-modifying activity. While they effectively neutralize oxidative stress (as validated by reduced ROS levels in our current data, Supplemental Figure 7), their inability to rescue nuclear abnormalities or enucleation defects in IDH1 deficient cells suggests that IDH1 deficiency-driven dyserythropoiesis is not solely ROS-dependent.

      References:

      (1) Blaschke K, Ebata KT, Karimi MM, Zepeda-Martínez JA, Goyal P, et al. Vitamin C induces Tet-dependent DNA demethylation and a blastocyst-like state in ES cells. Nature. 20138;500(7461): 222-226.

      (2) Minor EA, Court BL, Young JI, Wang G. Ascorbate induces ten-eleven translocation (Tet) methylcytosine dioxygenase-mediated generation of 5-hydroxymethylcytosine. J Biol Chem. 2013;288(19): 13669-13674.

      (3) Yin R, Mao S, Zhao B, Chong Z, Yang Y, et al. Ascorbic acid enhances Tet-mediated 5-methylcytosine oxidation and promotes DNA demethylation in mammals. J Am Chem Soc. 2013;135(28):10396-10403.

      (4) Esteban MA, Wang T, Qin B, Yang J, Qin D, et al. Vitamin C enhances the generation of mouse and human induced pluripotent stem cells. Cell Stem Cell. 2010;6(1):71-79.

      (5) Chung T, Brena RM, Kolle G, Grimmond SM, Berman BP, et al. Vitamin C promotes widespread yet specific DNA demethylation of the epigenome in human embryonic stem cells. Stem Cells. 2010;28(10):1848-1855.

      (5) The authors describe an increase in euchromatin as the consequential abnormal nuclei phenotype in shIDH1 erythroid cells. However, in their RNA-seq, they observe an almost equal number of genes that are up and down-regulated in shIDH1 cells compared to control cells. If possible, an RNA-Seq in nuclear knockout Sg-NES-IDH1 erythroid cells in comparison with knockout and wild-type cells will be helpful to tease out whether a specific absence of IDH1 in the nucleus (ie., lack of metabolic functions of IDH) impacts gene expression differently.

      Thanks for the reviewer's suggestion. ATAC-seq showed an increase in chromatin accessibility after IDH1 deletion, but the number of up-regulated genes was slightly larger than that of down-regulated genes, which may be caused by the metabolic changes affected by IDH1 deletion. In order to explore the effect of chromatin accessibility changes on gene expression after IDH1 deletion, we analyzed the changes in differential gene expression at the differential ATAC peak region (as shown in Author response image 1), and the results showed that the gene expression at the ATAC peak region with increased chromatin accessibility was significantly up-regulated. This may explain the regulation of chromatin accessibility on gene expression.

      Author response image 1.

      Box plots of gene expression differences of differential ATAC peaks located in promoter for the signal increasing and decreasing groups.

      (6) In Figure 8, the authors show data related to SIRT1's role in mediating non-metabolic, chromatin-associated functions of IDH1.

      A) The authors show that SIRT1 inhibition leads to a rescue of enucleation and abnormal nuclei. However, whether this rescues the progression through the late stages of terminal differentiation and the euchromatin/heterochromatin ratio is not clear.

      Thanks for the reviewer's suggestion. As shown in Supplementary Figure 14 and 15 in the revised Supplementary Data, our data showed that both the treatment of SRT1720 on normal erythroid cells and treatment of IDH1-deficient erythroid cells with SIRT1 inhibitor both have no effect on the terminal differentiation.

      (7) In Figure 4 and Supplemental Figure 8, the authors show the accumulation and altered cellular localization of H3K79me3, H3K9me3, and H3K27me2, and the lack of accumulation of other three histone modifications they tested (H3K4me3, H3K35me4, and H3K36me2) in shIDH1 cells. They also show the accumulation and altered localization of the specific histone marks in Sg-NES-IDH1 HUDEP2 cells.

      A) To aid better comparison of these histone modifications, it will be helpful to show the cell fractionation data of the three histone modifications that did not accumulate (H3K4me3, H3K35me4, and H3K36me2), similar to what was shown in Figure 4E for H3K79me3, H3K9me3, and H3K27me2).

      We appreciate the reviewer’s insightful suggestion. We collected erythroblasts on day 15 of differentiation from cord blood-derived CD34<sup>+</sup> hematopoietic stem cells to erythroid lineage and performed ChIP assay. As shown in Author response image 2, the results showed that the concentration of bound DNA of H3K9me3, H3K27me2 and H3K79me3 was too low to meet the sequencing quality requirement, which was consistent with that of WB. In addition, we tried to perform ChIP-seq for H3K79me3, and the results showed that there was no marked peak signal.

      Author response image 2.

      ChIP-seq analysis show that there was no marked peak signal of H3K79me3 on D15. (A) Quality control of ChIP assay for H3K9me3, H3K27me2, and H3K79me3. (B) Representative peaks chart image showed normalized ChIP signal of H3K79me3 at gene body regions. (C) Heatmaps displayed normalized ChIP signal of H3K79me3 at gene body regions. The window represents ±1.5 kb regions from the gene body. TES, transcriptional end site; TSS, transcriptional start site.

      C) Among the three histone marks that are dysregulated in IDH1 deficient cells (H3K79me3, H3K9me3, and H3K27me2), the authors show via ChIP-seq (Fig5) that H3K79me3 is the critical factor. However, the ChIP-seq data shown here lacks many details and this makes it hard to interpret the data. For example, in Figure 5A, they do not mention which samples the data shown correspond to (are these differential peaks in shIDH1 compared to shLuc cells?). There is also no mention of how many replicates were used for the ChIP seq studies.

      We thank the reviewer for pointing this out. We apologize for not clearly describing the ChIP-seq data for H3K9me3, H3K27me2 and H3K79me3 and we have revised them in the corresponding paragraphs. Since H3 proteins gradually translocate from the nucleus to the cytoplasm starting at day 11 (late Baso-E/Ploy-E) of erythroid lineage differentiation, we performed ChIP-seq for H3K9me3, H3K27me2 and H3K79me3 only for the shIDH1 group, and set up three independent biological replicates for each of them.

      Reviewer #2 (Public Review):

      Li and colleagues investigate the enzymatic activity-independent function of IDH1 in regulating erythropoiesis. This manuscript reveals that IDH1 deficiency in the nucleus leads to the redistribution of histone marks (especially H3K79me3) and chromatin state reprogramming. Their findings suggest a non-typical localization and function of the metabolic enzyme, providing new insights for further studies into the non-metabolic roles of metabolic enzymes. However, there are still some issues that need addressing:

      (1) Could the authors show the RNA and protein expression levels (without fractionation) of IDH1 on different days throughout the human CD34+ erythroid differentiation?

      We sincerely appreciate the reviewer’s constructive feedback. To address this point, we have now systematically quantified IDH1 expression dynamics across erythropoiesis stages using qRT-PCR and Western blot analyses. As quantified in Supplementary fige 1, IDH1 expression exhibited a progressive upregulation during early erythropoiesis and subsequently stabilized throughout terminal differentiation.

      (2) Even though the human CD34+ erythroid differentiation protocol was published and cited in the manuscript, it would be helpful to specify which erythroid stages correspond to cells on days 7, 9, 11, 13, and 15.

      We sincerely thank the reviewer for raising this important methodological consideration. Our research group has previously established a robust platform for staged human erythropoiesis characterization using cord blood-derived CD34<sup>+</sup> hematopoietic stem cells (HSCs) [6-9]. This standardized protocol enables high-purity isolation and functional analysis of erythroblasts at defined differentiation stages.

      Thanks for the reviewer’s suggestion. Our previous work (Jingping Hu et.al, Blood 2013. Xu Han et.al, Blood 2017.Yaomei Wang et.al, Blood 2021.) have isolation and functional characterization of human erythroblasts at distinct stages by using Cord blood. These works illustrated that using cord blood-derived hematopoietic stem cells and purification each stage of human erythrocytes can facilitate a comprehensive cellular and molecular characterization.

      Following isolation from cord blood, CD34<sup>+</sup> cells were cultured in a serum-free medium and induced to undergo erythroid differentiation using our standardized protocol. The process of erythropoiesis was comprised of 2 phases. During the early phase (day 0 to day 6), hematopoietic stem progenitor cells expanded and differentiated into erythroid progenitors, including BFU-E and CFU-E cells.

      During terminal erythroid maturation (day 7 to day 15), CFU-E cells progressively transitioned through defined erythroblast stages, as validated by daily cytospin morphology and expression of band 3/α4 integrin. The stage-specific composition was quantitatively characterized as follows:

      Author response table 1.

      The composition of erythroblast during terminal stage erythropoiesis.

      This differentiation progression from proerythroblasts (Pro-E) through basophilic (Baso-E), polychromatic (Poly-E), to orthochromatic erythroblasts (Ortho-E) recapitulates physiological human erythropoiesis, confirming the validity of our differentiation system for mechanistic studies.

      Reference:

      (6) Li J, Hale J, Bhagia P, Xue F, Chen L, et al. Isolation and transcriptome analyses of human erythroid progenitors: BFU-E and CFU-E. Blood. 2014;124(24):3636-3645.

      (7) Hu J, Liu J, Xue F, Halverson G, Reid M, et al. Isolation and functional characterization of human erythroblasts at distinct stages: implications for understanding of normal and disordered erythropoiesis in vivo. Blood. 2013;121(16):3246-3253.

      (8) Wang Y, Li W, Schulz VP, Zhao H, Qu X, et al. Impairment of human terminal erythroid differentiation by histone deacetylase 5 deficiency. Blood. 2021;138(17):1615-1627.

      (9) Li M, Liu D, Xue F, Zhang H, Yang Q, et al. Stage-specific dual function: EZH2 regulates human erythropoiesis by eliciting histone and non-histone methylation. Haematologica. 2023;108(9):2487-2502.

      (3) It is important to mention on which day the lentiviral knockdown of IDH1 was performed. Will the phenotype differ if the knockdown is performed in early vs. late erythropoiesis? In Figures 1C and 1D, on which day do the authors begin the knockdown of IDH1 and administer NAC and GSH treatments?

      We sincerely appreciate the reviewer’s inquiry regarding the experimental timeline. The day of getting CD34<sup>+</sup> cells was recorded as day 0. Lentivirus of IDH1-shRNA and Luciferase -shRNA was transduced in human CD34<sup>+</sup> at day 2. Puromycin selection was initiated 24h post-transduction to eliminate non-transduced cells. IDH1-KD cells were then selected for 3 days. The knock down deficiency of IDH1 was determined on day 7. NAC or GSH was added to culture medium and replenished every 2 days.

      (4) While the cell phenotype of IDH1 deficiency is quite dramatic, yielding cells with larger nuclei and multi-nuclei, the authors only attribute this phenotype to defects in chromatin condensation. Is it possible that IDH1-knockdown cells also exhibit primary defects in mitosis/cytokinesis (not just secondary to the nuclear condensation defect)?), given the function of H3K79Me in cell cycle regulation?

      We appreciate the reviewer’s insightful suggestion. We performed Edu based cell cycle analysis on Control, Sg-IDH1 and Sg-NES-IDH1 HUDEP2 cells, respectively. The results showed that IDH1 deficiency resulted in S-phase block and increased polyploidy in HUDEP2 cells on day 8 of erythroid differentiation. NES-IDH1 overexpression failed to rescue these defects, indicating nuclear IDH1 depletion as the primary driving factor (Figure 3E,F). Recent studies have established a clear link between cell cycle arrest and nuclear malformation. Chromosome mis-segregation, nuclear lamina disruption, mechanical stress on the nuclear envelope, and nucleolar dysfunction all contribute to nuclear abnormalities that trigger cell cycle checkpoints [10,11]. Based on all these findings, it reasonable for us to speculate that the cell cycle defect in IDH1 deficient cells might caused by the nuclear malfunction.

      Reference:

      (10) Hong T, Hogger AC, Wang D, Pan Q, Gansel J, et al. Cell Death Discov. CDK4/6 inhibition initiates cell cycle arrest by nuclear translocation of RB and induces a multistep molecular response. 2024;10(1):453.

      (11) Hervé S, Scelfo A, Marchisio GB, Grison M, Vaidžiulytė K, et al. Chromosome mis-segregation triggers cell cycle arrest through a mechanosensitive nuclear envelope checkpoint. Nat Cell Biol. 2025;27(1):73-86. 

      (5) Why are there two bands of Histone H3 in Figure 4A?

      We sincerely appreciate the reviewer's insightful observation regarding the dual bands of Histone H3 in our original Figure 4A. Upon rigorous investigation, we identified that the observed doublet pattern likely originated from the inter-batch variability of the original commercial antibody. To conclusively resolve this technical artifact, we have procured a new lot of Histone H3 antibody and repeated the western blot experimental under optimized conditions, and the results demonstrates a single band of H3.

      (6) Displaying a heatmap and profile plots in Figure 5A between control and IDH1-deficient cells will help illustrate changes in H3K79me3 density in the nucleus after IDH1 knockdown.

      Thank you for your suggestion. As presented in Author response image 2, we performed ChIP assays on erythroblasts collected at day 15. However, the concentration of H3K79me3-bound DNA was insufficient to meet the quality threshold required for reliable sequencing. Consequently, we are unable to generate the requested heatmap and profile plots due to limitations in data integrity from this experimental condition.

      Reviewer #3 (Public Review):

      Li, Zhang, Wu, and colleagues describe a new role for nuclear IDH1 in erythroid differentiation independent from its enzymatic function. IDH1 depletion results in a terminal erythroid differentiation defect with polychromatic and orthochromatic erythroblasts showing abnormal nuclei, nuclear condensation defects, and an increased proportion of euchromatin, as well as enucleation defects. Using ChIP-seq for the histone modifications H3K79me3, H3K27me2, and H3K9me3, as well as ATAC-seq and RNA-seq in primary CD34-derived erythroblasts, the authors elucidate SIRT1 as a key dysregulated gene that is upregulated upon IDH1 knockdown. They furthermore show that chemical inhibition of SIRT1 partially rescues the abnormal nuclear morphology and enucleation defect during IDH1-deficient erythroid differentiation. The phenotype of delayed erythroid maturation and enucleation upon IDH1 shRNA-mediated knockdown was described in the group's previous co-authored study (PMID: 33535038). The authors' new hypothesis of an enzyme- and metabolism-independent role of IDH1 in this process is currently not supported by conclusive experimental evidence as discussed in more detail further below. On the other hand, while the dependency of IDH1 mutant cells on NAD+, as well as cell survival benefit upon SIRT1 inhibition, has already been shown (see, e.g, PMID: 26678339, PMID: 32710757), previous studies focused on cancer cell lines and did not look at a developmental differentiation process, which makes this study interesting.

      (1) The central hypothesis that IDH1 has a role independent of its enzymatic function is interesting but not supported by the experiments. One of the author's supporting arguments for their claim is that alpha-ketoglutarate (aKG) does not rescue the IDH1 phenotype of reduced enucleation. However, in the group's previous co-authored study (PMID: 33535038), they show that when IDH1 is knocked down, the addition of aKG even exacerbates the reduced enucleation phenotype, which could indicate that aKG catalysis by cytoplasmic IDH1 enzyme is important during terminal erythroid differentiation. A definitive experiment to test the requirement of IDH1's enzymatic function in erythropoiesis would be to knock down/out IDH1 and re-express an IDH1 catalytic site mutant. The authors perform an interesting genetic manipulation in HUDEP-2 cells to address a nucleus-specific role of IDH1 through CRISPR/Cas-mediated IDH1 knockout followed by overexpression of an IDH1 construct containing a nuclear export signal. However, this system is only used to show nuclear abnormalities and (not quantified) accumulation of H3K79me3 upon nuclear exclusion of IDH1. Otherwise, a global IDH1 shRNA knockdown approach is employed, which will affect both forms of IDH1, cytoplasmic and nuclear. In this system and even the NES-IDH1 system, an enzymatic role of IDH1 cannot be excluded because (1) shRNA selection takes several days, prohibiting the assessment of direct effects of IDH1 loss of function (only a degron approach could address this if IDH1's half-life is short), and (2) metabolic activity of this part of the TCA cycle in the nucleus has recently been demonstrated (PMID: 36044572), and thus even a nuclear role of IDH1 could be linked to its enzymatic function, which makes it a challenging task to separate two functions if they exist.

      We appreciate the reviewer’s emphasis on rigorously distinguishing between enzymatic and enzymatic independent roles of IDH1. In our revised manuscript, we have removed all assertions of a "metabolism-independent" mechanism. Instead, we focus on demonstrating that nuclear-localized IDH1 contributes to chromatin state regulation during terminal erythropoiesis (e.g., H3K79me3 accumulation). While we acknowledge that nuclear IDH1’s enzymatic activity may still play a role [12], our data emphasize its spatial association with chromatin remodeling. We now explicitly state that nuclear IDH1’s function may involve both enzymatic and structural roles, and further studies are required to dissect these mechanisms.

      Reference:

      (12) Kafkia E, Andres-Pons A, Ganter K, Seiler M, Smith TS, et al.Operation of a TCA cycle subnetwork in the mammalian nucleus. Sci Adv. 2022;8(35):eabq5206.

      (2) It is not clear how the enrichment of H3K9me3, a prominent marker of heterochromatin, upon IDH1 knockdown in the primary erythroid culture (Figure 4), goes along with a 2-3-fold increase in euchromatin. Furthermore, in the immunofluorescence (IF) experiments presented in Figure 4Db, it seems that H3K9me3 levels decrease in intensity (the signal seems more diffuse), which seems to contrast the ChIP-seq data. It would be interesting to test for localization of other heterochromatin marks such as HP1gamma. As a related point, it is not clear at what stage of erythroid differentiation the ATAC-seq was performed upon luciferase- and IDH1-shRNA-mediated knockdown shown in Figure 6. If it was done at a similar stage (Day 15) as the electron microscopy in Figure 4B, then the authors should explain the discrepancy between the vast increase in euchromatin and the rather small increase in ATAC-seq signal upon IDH1 knockdown.

      Thank you for raising this important point. We agree that while H3K9me3 and H3K27me2 modifications are detectable in the nucleus, their functional association with chromatin in this context remains unclear. Our ChIP-seq data did not reveal distinct enrichment peaks for H3K9me3 or H3K27me2 (unlike the well-defined H3K79me3 peaks), suggesting that these marks may not be stably bound to specific chromatin regions under the experimental conditions tested. However, we acknowledge that the absence of clear peaks in our dataset does not definitively rule out chromatin interactions, as technical limitations or transient binding dynamics could influence these results. To avoid over-interpretation, we have removed speculative statements about the chromatin-unbound status of H3K9me3 and H3K27me2 from the revised manuscript. This revision aligns with our broader effort to present conclusions strictly supported by the current data while highlighting open questions for future investigation.

      (3)The subcellular localization of IDH1, in particular its presence on chromatin, is not convincing in light of histone H3 being enriched in the cytoplasm on the same Western blot. H3 would be expected to be mostly localized to the chromatin fraction (see, e.g., PMID: 31408165 that the authors cite). The same issue is seen in Figure 4A.

      We sincerely appreciate the reviewer's insightful comment regarding the subcellular distribution of histone H3 in our study. We agree that histone H3 is classically associated with chromatin-bound fractions, and its cytoplasmic enrichment in our Western blot analyses appears counterintuitive at first glance. However, this observation is fully consistent with the unique biology of terminal erythroid differentiation, which involves drastic nuclear remodeling and histone release - a hallmark of terminal stage erythropoiesis. Terminal erythroid differentiation is characterized by progressive nuclear condensation, chromatin compaction, and eventual enucleation. During this phase, global chromatin reorganization leads to the active eviction of histones from the condensed nucleus into the cytoplasm. This process has been extensively documented in erythroid cells, with studies demonstrating cytoplasmic accumulation of histones H3 and H4 as a direct consequence of nuclear envelope breakdown and chromatin decondensation preceding enucleation [13-16]. Our experiments specifically analyzed terminal-stage polychromatic and orthochromatic erythroblasts. At this stage, histone releasing into the cytoplasm is a dominant biological event, explaining the pronounced cytoplasmic H3 signal in our subcellular fractionation assays.

      In summary, the cytoplasmic enrichment of histone H3 in our data aligns with established principles of erythroid biology and reinforces the physiological relevance of our findings. We thank the reviewer for raising this critical point, which allowed us to better articulate the unique aspects of our experimental system.

      Reference:

      (13) Hattangadi SM, Martinez-Morilla S, Patterson HC, Shi J, Burke K, et al. Histones to the cytosol: exportin 7 is essential for normal terminal erythroid nuclear maturation. Blood. 2014;124(12):1931-1940.

      (14) Zhao B, Mei Y, Schipma MJ, Roth EW, Bleher R, et al. Nuclear Condensation during Mouse Erythropoiesis Requires Caspase-3-Mediated Nuclear Opening. Dev Cell. 2016;36(5): 498-510.

      (15) Zhao B, Liu H, Mei Y, Liu Y, Han X, et al. Disruption of erythroid nuclear opening and histone release in myelodysplastic syndromes. Cancer Med. 2019;8(3):1169-1174. 

      (16) Zhen R, Moo C, Zhao Z, Chen M, Feng H, et al.  Wdr26 regulates nuclear condensation in developing erythroblasts. Blood. 2020;135(3):208-219.

      (4) This manuscript will highly benefit from more precise and complete explanations of the experiments performed, the material and methods used, and the results presented. At times, the wording is confusing. As an example, one of the "Key points" is described as "Dyserythropoiesis is caused by downregulation of SIRT1 induced by H3K79me3 accumulation." It should probably read "upregulation of SIRT1".

      We sincerely thank the reviewer for highlighting the need for improved clarity in our experimental descriptions and textual precision. We fully agree that rigorous wording is essential to accurately convey scientific findings. Specific modifications have been made and are highlighted in Track Changes mode in the resubmitted manuscript.

      The reviewer correctly identified an inconsistency in the original phrasing of one key finding. The sentence in question ("Dyserythropoiesis is caused by downregulation of SIRT1 induced by H3K79me3 accumulation") has been revised to:"Dyserythropoiesis is caused by the upregulation of SIRT1 mediated through H3K79me3 accumulation." This correction aligns with our experimental data showing that H3K79me3 elevation promotes SIRT1 transcriptional activation. We apologize for this oversight and have verified the consistency of all regulatory claims in the text.

      Recommendations for the authors:

      Reviewer #1 (Recommendations For The Authors):

      (1) It will be helpful to mention/introduce the cells used for the study at the beginning of the results section. For example, for Figure 1A neither the figure legend nor the results text includes information on the cells used.

      Thanks for the reviewer’s suggestion. The detail information of the cells that were used in our study have been provided in the revised manuscript.

      (2) Important details for many figures are lacking. For example, in Figure 5, there is no mention of the replicates for ChIP-Seq studies. Also, the criteria used for quantifications of abnormal nuclei, % euchromatin vs heterochromatin, the numbers of biological replicates, and how many fields/cells were used for these quantifications are missing.

      We thank the reviewer for emphasizing the importance of methodological transparency. It has been revised accordingly. The ChIP-Seq data in Figure 5 was generated from three independent biological replicates to ensure reproducibility. In this study, Image J software was used to calculate the area of nuclear, heterochromatin/euchromatin and to quantify the percentage of euchromatin and heterochromatin. A minimum of 300 cells per group were evaluated, and the proportion of aberrant nuclei was calculated as (number of abnormal cells / total counted cells) × 100%.

      (3) It will be helpful if supplemental data are ordered according to how they are discussed in the text. Currently, the order of the supplemental data is hard to keep track of eg., the results section starts describing supplemental Figure 1, then the text jumps to supplemental Figure 5 followed by Supplemental Figure 3 (and so on).

      Thanks for the reviewer’s suggestion. It has been revised accordingly.

      (4) Overall, there are many incomplete sentences and typos throughout the manuscript including some of the figures e.g. on page 10 the sentence "Since the generation of erythroid with abnormal nucleus and reduction of mature red blood cells caused by IDH1 absence are notable characteristics of MDS and AML." is incomplete. On page 11, it reads "Histone post-modifications". This needs to be either histone modifications or histone post-translational modifications. In Figure 4C, the y-axis title is hard to understand "% of euchromatin and heterochromatin". Overall, the document needs to be proofread and revised carefully.

      Thanks for the reviewer’s suggestion. We have made revision accordingly in the revised manuscript. The sentence "Since the generation of erythroid with abnormal nucleus and reduction of mature red blood cells caused by IDH1 absence are notable characteristics of MDS and AML." has been revised to “The production of erythrocytes with abnormal nuclei and the reduction of mature erythrocytes due to IDH1 deletion are prominent features of MDS and AML.”  “% of euchromatin and heterochromatin” has been modified to “Area ratio of euchromatin to heterochromatin”.

      Reviewer #3 (Recommendations For The Authors):

      The following critique points aim to help the authors to improve their manuscript:

      (1) The authors reason (p. 10) that because mutant IDH1 has been shown to result in altered chromatin organization, this could be the case in their system, too. However, mutant IDH1 has an ascribed metabolic consequence, the generation of 2-HG, which further weakens the author's argument for an enzymatically independent role of IDH1 in their system. The same is true for the author's observation in Supplementary Figure 9B that in IDH1-mutant AML/MDS samples, H3K79me3 colocalized with the IDH1 mutants in the nucleus. Again, this speaks in favor of IDH1's role being linked to metabolism. The authors could re-write this manuscript, not so much emphasizing the separation of function between different subcellular forms of IDH1 but rather focusing on the chromatin changes and how they could be linked to the actual phenotype, the nuclear condensation and enucleation defect - if so, addressing the surprising finding of enrichment of both active and repressive chromatin marks will be important.

      Thanks for the reviewer’s suggestion. We agree with the reviewers and editors all the data we present in the current are not robust enough to rigorously distinguish between enzymatic and enzymatic-independent roles of IDH1. In our revised manuscript, we have removed all assertions of a "metabolism-independent" mechanism. Instead, we focus on demonstrating that nuclear-localized IDH1 contributes to chromatin state regulation during terminal erythropoiesis (e.g., H3K79me3 accumulation).

      (2) How come so many genes were downregulated by RNA-seq (about an equal number as upregulated genes) but not more open by ATAC-seq? The authors should discuss this result.

      Thanks for the reviewer's suggestion. ATAC-seq showed an increase in chromatin accessibility after IDH1 deletion, but the number of up-regulated genes was slightly larger than that of down-regulated genes, which may be caused by the metabolic changes affected by IDH1 deletion. In order to explore the effect of chromatin accessibility changes on gene expression after IDH1 deletion, we analyzed the changes in differential gene expression at the differential ATAC peak region (as shown in the figure below), and the results showed that the gene expression at the ATAC peak region with increased chromatin accessibility was significantly up-regulated. This may explain the regulation of chromatin accessibility on gene expression.

      (3) For the ChIP-seq analyses of H3K79me3, H3K27me2, and H3K9me3, the authors should not just show genome-wide data but also several example gene tracks to demonstrate the differential abundance of peaks in control versus IDH1 knockdown. Furthermore, the heatmap shown in Figure 5A should include broader regions spanning the gene bodies, to visualize the intergenic H3K27me2 and H3K9me3 peaks. Expression could very well be regulated from these intergenic regions as they could bear enhancer regions. ChIP-seq for H3K27Ac in the same setting would be very useful to identify those enhancers.

      Thanks for the reviewer’s suggestion. It has been revised accordingly. We reanalyzed the ChIP-seq peak signal of H3K79me3, H3K27me2 and H3K9me3 in a wider region (±5Kb) at gene body, and the results showed that the H3K27me2 and H3K9me3 peak signals did not change significantly. Since H3K79me3 showed a higher peak signal and was mainly enriched in the promoter region, our subsequent analysis focusing on the impact of H3K79me3 accumulation on chromatin accessibility and gene expression might be more valuable.

      Author response image 3.

      ChIP-seq analysis show that the peak signal of H3K79me3,H3K27me2 and H3K9me3. (A) Heatmaps displayed normalized ChIP signal of H3K9me3, H3K27me2, and H3K79me3 at gene body regions. The window represents ±5 kb regions from the gene body. TES, transcriptional end site; TSS, transcriptional start site. (B) Representative peaks chart image showed normalized ChIP signal of H3K9me3, H3K27me2, and H3K79me3 at gene body regions.

      (4) The absent or very mild delay (also no significance visible in the quantification plots) in the generation of orthochromatic erythroblasts on Day 13 upon IDH1 shRNA knockdown as per a4-integrin/Band3 flow cytometry does not correspond to the already quite prominent number of multinucleated cells at that stage seen by cytospin/Giemsa staining. Why do the authors think this is the case? Cytospin/Giemsa staining might be the better method to quantify this phenotype and the authors should quantify the cells at different stages in at least 100 cells from non-overlapping cytospin images.

      Thanks for the reviewer’s suggestion. We have supplemented the cytpspin assay and the results were presented in Supplemental Figure 4.

      (5) The pull-down assay in Figure 7E does not show a specific binding of H3K79me3 to the SIRT1 promoter. Rather, there is just more H3K79me3 in the nucleus, thus leading to generally increased binding. The authors should show that H3K79me3 does not bind more just everywhere but to specific loci. The ChIP-seq data mention only categories but don't show any gene lists that could hint at the specificity of H3K79me3 binding at genes that would promote nuclear abnormalities and enucleation defects.

      We thank the reviewer for pointing this out. The GSEA results of H3K79me3 peak showed enrichment of chromatin related biological processes, and the list of associated genes is shown Figure 7B. In addition, we also displayed the changes in H3K79me3 peak signals, ATAC peak signals, and gene expression at gene loci of three chromatin-associated genes (SIRT1, KMT5A and NUCKS1).

      (6) P. 12: "Representatively, gene expression levels and ATAC peak signals at SIRT1 locus were elevated in IDH1-shRNA group and were accompanied by enrichment of H3K9me3 (Figure 7F)." Figure 7F does not show an enrichment of H3K9me3, but if the authors found such, they should explain how this modification correlates with the activation of gene expression.

      Thank you for bringing this issue to our attention. We sincerely apologize for the mistake in the description of Figure 7F on page 12. We have already corrected this error in the revised manuscript.

      (7) Related to the mild phenotype by flow cytometry on Day 13, are the "3 independent biological replicates" from culturing and differentiating CD34 cells from 3 different donors? If all are from the same donor, experiments from at least a second donor should be performed to generalize the results.

      In our current study, CD34<sup>+</sup> cells were derived from different donors. 

      (8) If the images in Supplementary Figure 4 are only the indicated cell type, then it is not clear how the data were quantified since only some cells in each image are pointed at and others do not seem to have as large nuclei. There is also no explanation in the legend what the colors mean (nuclei were presumably stained with DAPI, not clear what the cytoplasm stain is - GPA?).

      We thank the reviewer for pointing this out. We have revised the manuscript accordingly. Specifically, the nuclei was stained with DAPI and the color was blue. The cell membrane was stained with GPA and the color was red. This staining method allows for clear visualization of the cell structure and helps to better understand the localization of the proteins of interest.

      (9) It is not clear to this reviewer whether Figure 4F is a quantification of the Western Blot or of the IF data.

      Figure 4F is a quantification of the Western Blot experiment.

      (10) The authors sometimes do not describe experiments well, e.g., "treatment of IDH1-deficient erythroid cells with IDH1-EX527" (p. 13). EX-527 is a SIRT1 inhibitor, which the authors only explicitly mention later in that paragraph. It is unclear to this reviewer, why the authors call it IDH1-EX527.

      Thank you for pointing out the unclear description in our manuscript. We apologize for the confusion caused by the unclear statement. We have revised the manuscript accordingly. The compound EX-527 is a SIRT1 inhibitor, and we have corrected the description to simply "EX-527" in the revised manuscript.

      (11) The end of the introduction needs revising to be more concise; the last paragraph on p. 4 ("Recently, the decreased expression of IDH1...") partially should be integrated with the previous paragraph, and partially is repeated in the last paragraph (top paragraph on p. 5). The last sentence on p. 4, "These findings strongly suggest that aberrant expression of IDH1 is also an important factor in the pathogenesis of AML and MDS.", should rather read "increased expression of IDH1", to distinguish it from mutant IDH1 (mutant IDH1 is also aberrantly expressed IDH1).

      We appreciated the reviewer for the helpful suggestion. Considering that the inclusion of this paragraph did not provide a valuable contribution to the formulation of the scientific question, we have removed it after careful consideration, and the revised manuscript is generally more logically smooth.

      (12) Abstract and last sentence of the introduction: "innovative perspective" should be re-worded, as the authors present data, not a perspective. Maybe could use "evidence".

      Thanks for the reviewer’s suggestion. It has been revised accordingly.

      (13) "IDH1-mut AML/MDS" on p. 11. The authors should provide more information about these AML/MDS samples. The legend contains no information about them/their mutational status. How many samples did the authors look at? Do these cells contain mutations other than IDH1?

      Thanks for the reviewer’s suggestion. The detail information of these AML/MDS samples are provide in supplemental table 1. In our current study, we collected ten AML/MDS samples and the majority of the samples only contain IDH1 mutations at different sites.

      (14) The statement, "Taken together, these results indicated that IDH1 deficiency reshaped chromatin states and subsequently altered gene expression pattern, especially for genes regulated by H3K79me3, which was the mechanism underlying roles of IDH1 in modulation of terminal erythropoiesis." (p. 10), is not correct at that point in the manuscript as the authors have not yet introduced the RNA-seq data.

      Thanks for the reviewer’s suggestion. The statement has been revised to “Taken together, these results indicated that IDH1 deficiency reshaped chromatin states by altering the abundance and distribution of H3K79me3, which was the mechanism underlying roles of IDH1 in modulation of terminal erythropoiesis”.

      (15) For easier readability, the authors should present the data in order. For example, the supplemental data for IDH shRNA and siRNA should be presented together and not in Supplementary Figures 1 and 5. Supplementary Figure 3 is mentioned after Supplementary Figure 1, but before Supplementary Figure 2 - again, all data need to be presented in subsequent figures to be viewed together.

      Thank you for your suggestion regarding the order of data presentation. We have reorganized the figures in the manuscript to improve readability. We apologize for any confusion caused by the previous arrangement and hope that the revised version meets your expectations.

    1. Author response:

      The following is the authors’ response to the original reviews.

      Public Reviews:

      Reviewer #1 (Public review):

      Summary:

      The authors investigate the role of HSPA2 during mouse preimplantation development. Knocking down HSPA2 in zygotes, the authors describe lower chances of developing into blastocysts, which show a reduced number of inner cell mass cells. They find that HSPA2 mRNA and protein levels show some heterogeneity among blastomeres at the 4-cell stage and propose that HSPA2 could contribute to skewing their relative contribution to embryonic lineages. To test this, the authors try to reduce HSPA2 expression in one of the 2-cell stage blastomere and propose that it biases their contribution to towards extra-embryonic lineages. To explain this, the authors propose that HSPA2 would interact with CARM1, which controls chromatin accessibility around genes regulating differentiation into embryonic lineage.

      Strengths:

      (1) The study offers simple and straightforward experiments with large sample sizes.

      Thanks for your kind recognition.

      (2) Unlike most studies in the field, this research often relies on both mRNA and protein levels to analyses gene expression and differentiation.

      Thanks for your kind recognition.

      Weaknesses:

      (1) Image and statistical analyses are not well described.

      Thanks for your advisable comment. We redescribe the image and statistical analyses in our revised version (line 255-257).

      (2) The functionality of the overexpression construct is not validated.

      Thanks for your kind suggestion. We validate the functionality of the overexpression construct in our revised version (Figure S3).

      (3) Tracking of KD cells in embryos injected at the 2-cell stage with GFP is unclear.

      Thanks for your kind suggestion. We randomly co-injected green fluorescent protein (Gfp) mRNA as a linage tracer with either Hspa2-siRNA or NC-FAM into one of the 2 -cell, and then monitored embryo development to the blastocyst stage (line 342-344).

      (4) A key rationale of the study relies on measuring small differences in the levels of mRNA and proteins using semi-quantitative methods to compare blastomeres. As such, it is not possible to know whether those subtle differences are biologically meaningful. For example, the lowest HSPA2 level of the embryo with the highest level is much higher than the top cell from the embryo with the lowest level. What does this level mean then? Does this mean that some blastomeres grafted from strong embryos would systematically outcompete all other blastomeres from weaker embryos? That would be very surprising. I think the authors should be more careful and consider the lack of quantitative power of their approach before reaching firm conclusions. Although to be fair, the authors only follow a long trend of studies with the same intrinsic flaw of this approach.

      Thanks for your advisable comment. Indeed, despite the approach drew on previous research (Zhou Cell 2018), we were clearly aware that this approach can only reflect relative comparisons. This means that the relative difference among the blastomeres from the same embryo were detected and compared. We did not compare the absolute levels of mRNA between different embryos. We also offered simple and straightforward experiments with large sample sizes to confirm this conclusion.

      (5) Some of the analyses on immunostaining do not take into account that this technique only allows for semi-quantitative measurements and comparisons.

      a) Some of the microscopy images are shown with an incorrect look-up table.

      b) Some of the schematics are incorrect and misleading.

      Thanks for your advisable comment. We revised microscopy images and schematics in our revised version.

      Reviewer #2 (Public review):

      Summary:

      In this study, Gao et al. use RNA-seq to identify Hspa2 as one of the earliest transcripts heterogeneously distributed between blastomeres. Functional studies are performed using siRNA knockdown showing Hspa2 may bias cells toward the ICM lineage via interaction with the known methyltransferase CARM1.

      Strengths:

      This study tackles an important question regarding the origins of the first cell fate decision in the preimplantation embryo. It provides novelty in its identification of Hspa2 as a heterogeneous transcript in the early embryo and proposes a plausible mechanism showing interactions with Carm1. Multiple approaches are used to validate their functional studies (FISH, WB, development rates, proteomics). Given only 4 other transcripts/RNA have been identified at or before the 4-cell stage (LincGET, CARM1, PRDM14, HMGA1), this would be an important addition to our understanding of how TE vs ICM fate is established.

      Thanks for your kind recognition.

      The RNA-seq results leading the authors to focus on Hspa2 are not included in the manuscript. This dataset would serve as an important resource but is neither included nor discussed. Nor is it mentioned whether Hspa2 was identified in prior RNA-seq embryos studies (for example Deng Science 2014).

      Thanks for your advisable comment. To identify genes that show a significantly high variability across blastomeres in the same embryo, we regressed out the embryo effect by established a new method, which will be published and uploaded to the database in the future. Thus, the RNA-seq results leading the we focus on Hspa2 are not included in the manuscript.   

      In addition, the functional studies are centered on Hspa2 knockdown at the zygote (1-cell) stage, which would largely target maternal transcript. Given the proposed mechanism relies on Hspa2 heterogeneity post-ZGA (late 2-cell stage), the knockdown studies don't necessarily test this and thus don't provide direct support to the authors' conclusions. The relevance of the study would be improved if the authors could show that zygotic knockdown leads to symmetric Hspa2 levels at the late 2-cell and/or 4-cell stage. It may be possible that zygotic knockdown leads to lower global Hspa2 levels, but that asymmetry is still generated at the 4-cell stage.

      Thanks for your advisable comment. We showed that the Hspa2 levels at the late 2-cell and 4cell stage after zygotic knockdown in our revised version (Figure S1 G-H, line 450-452).

      Furthermore, the authors show that Hspa2 knockdown at the 1-cell stage lowers total Carm1 levels at the 4-cell stage. However, it is unclear how total abundance within the embryo alters lineage specification within blastomeres. The authors go on to propose a plausible mechanism involving Hspa2 and Carm1 interaction, but do not discuss how expression levels may be involved.

      Thanks for your advisable comment. Previous research suggests that heterogeneous activity of the methyltransferase CARM1 results in differential methylation of histone H3R26 to modulate establishment of lineage specification (Zernicka-Goetz Cell 2018). Thus, we didn't discuss the total abundance within the embryo alters lineage specification.

      Recommendations for the authors:  

      Reviewer #1 (Recommendations for the authors):

      (1) Major issue with analyses:

      Image analysis needs to be much better explained than simply saying that ImageJ was used. Where are cells measured (at their equatorial plane? What is the size of the ROI?)? Ideally, the ROI and/or raw measurements should be provided.

      Thanks for your advisable comment. We redescribe the Image analysis in our revised version (line 187-194). 

      What are the objective criteria determining whether a cell is counted as GFP positive, CDX2 positive, or OCT4 positive? This is very unclear and key to the interpretation of many experiments.

      Thanks for your advisable comment. We think that the cell containing fluorescence signals above background noise were counted positive.

      Statistical analyses mention ANOVA in the methods but the student's t-test in the figure legend. Which is which? Most data are heavily normalized, which would unlikely fit the description for Student's t-test analyses.

      Thanks for your advisable comment. We redescribe the statistical analyses in our materials and methods (line 253-260).

      Figure 5H describes a relative fluorescence intensity with control at 1. The legend describes a normalization to "DNA" (I guess the authors meant DAPI), which is unlikely to give 1. This suggests that additional normalization was done and is not described. Is that the case? Also, since the authors propose that HSPA2 would control Histone modification and chromatin packing, I do not think that using DAPI is an appropriate way of normalizing the fluorescence signal.

      Thanks for your advisable comment. We replaced DNA with DAPI in our revised version. Based on previous studies, we adopted DAPI as a normalized fluorescence signal (Zhou Cell 2018, Zernicka-Goetz Cell 2018).

      Figure 1E shows data normalized to the lowest level while Figure 1H is normalized to the highest level. A consistent representation would be welcome.

      Thanks for your advisable comment. We revised the Figure 1H in our revised version.

      Is Figure 1C showing a t-test between correlations?

      Yes, Figure 1C shows the t-test between correlation.

      (2) Major issue with the interpretation of semi-quantitative methods and measurements:

      qPCR, WB, immunostaining are all semi-quantitative methods that require some kind of normalization due to non-linear bias in the way the molecules are picked up. Such normalization makes it difficult to know whether a detectable difference is meaningful biologically speaking i.e. if a difference of 1 CT between blastomeres can be detected after qPCR, is it meaningful? If that were the case, then embryos with lower CT than others (Figure 1D) would not be able to develop into blastocyst, like siRNA injected embryos, or grafting a blastomere with a high CT onto an embryo with low CT would lead to the systematic differentiation of these strong blastomeres into ICM.

      Thanks for your advisable comment. The CT values represent the relative mRNA levels of Hspa2 between blastomeres, and the higher CT value represents the lower expression of Hspa2 at mRNA level. Figure 1D shows the Hspa2 mRNA levels between blastomeres. The blastomere with lowlevel expression of the Hspa2 mRNA is not bias an ICM fates.  

      The same goes for fluorescence analyses (Figure 1F). Can the authors also provide the measurements for DAPI as they did for HSPA2? I am sure that with enough measurements, DAPI is variable enough to give a statistical difference among blastomeres with questionable biological meaning.

      I think the reasoning used here (unfortunately following the reasoning that has been used in a series of studies by other groups) of ranking blastomeres after semi-quantitative measurement is fundamentally flawed.

      Thanks for your advisable comment. The DAPI was determined by the maximal area using a custom Python script. Based on previous studies, we adopted DAPI as a normalized fluorescence signal (Zhou Cell 2018). This approach is to normalize embryo-to-embryo variance from the technical reason.

      (3) Major issue with overexpression experiment:

      While the siRNA experiment is partially validated by qPCR and WB measurements of HSPA2 after KD, the overexpression experiment is not. Do the authors have any evidence that the construct they use is produced into protein and functional? Can the authors check by WB? Can the authors rescue the siRNA with their overexpression?

      Thanks for your advisable comment. We verified the overexpression experiment by WB in in our revised version (Figure S3, line 360-361). Considering that siRNA degrades mRNA and prevents the mRNA translation process, we did not co-inject the siRNA with their overexpression.

      The lack of effect of HSPA2 overexpression on blastocyst formation is difficult to reconcile with the interpretation from the authors that levels of HSPA2 bias lineages.

      Have the authors tried lower concentrations? Have the authors tried FISH on their half-injected 2cell embryos? Of course, if the antibody against HSPA2 would work with immunostaining, that would be ideal.

      Thanks for your advisable comment. We chose the concentrations for our study based on previous research (Zernicka-Goetz Cell 2016). To verified Hspa2 was successfully inject into one blastomere at the 2-cell stage, we observed green fluorescence after co-injected GFP mRNA with either siRNA or NC-FAM into one blastomere of the two-cell embryos. Thus, we didn't try FISH on half-injected 2-cell embryos. We tried to perform immunostaining experiments with various HSPA2 antibodies (Proteintech: 12797-1-AP, Abcam: ab108416) and no good results were achieved.

      Author response image 1.

      (4) Major issue with tracking of injected cells:

      It is unclear what counts as a GFP-positive cell. In Figure 3D, most cells appear to have the same level of GFP.

      Thanks for your advisable comment. The cell containing green fluorescence signals above background noise were counted GFP-positive in Figure 3D. Most cells seem to have the same level of GFP because they are daughter cells of the blastomeres injected with GFP.

      In the images of GFP-expressing cells used to track the control of KD cells shown in Figure 3A, it seems that the control embryos have mostly GFP cells in the ICM. Is that the case, or just a bad example?

      Thanks for your advisable comment. The green fluorescent signals in Figure 3A represented OCT4 protein, an ICM marker.

      Can the authors do FISH against HSPA2 and visualize their GFP cells to validate the heterogeneous expression in situ?

      Thanks for your advisable comment. We have verified the heterogeneous expression of HSPA2 in Figure1.

      (5) Issue with fluorescent images:

      Many images are shown with inappropriate look-up tables with saturated DAPI, OCT4, CDX2, and FISH. This raises the doubt that analyses were made on saturated images, which would be incorrect.

      The LUT of Figure 5H should be adjusted similarly between the control and siRNA.

      Thanks for your advisable comment. We revised some images which showed inappropriate lookup tables in our revised version. The LUT of Figure 5H had been adjusted between the control and siRNA. 

      (6) Issue with schematics:

      Schematics of blastomere isolation grown into blastocyst-like structures are misleading since the final blastocyst-like structure should not have a zona pellucida and should have fewer cells than regular blastocysts.

      Thanks for your advisable comment. We revised schematics of blastomere grown into morula in our revised version (Figure 1A and Figure S1A).

      The summary schematics in the final figure should not state HSPA2 -/- since experiments in the study did not use KO but KD.

      Thanks for your advisable comment. We revised the summary schematics in our revised version.

      The blastocysts are the same sizes as the cleavage stage or morula embryos which implies that cells lose volume to the lumen, which is not the case.

      Thanks for your advisable comment. We revised the schematics in our revised version.

      (7) Issue with data presentation:

      In the tables within the figures, the number of decimals given should be the same for the mean and SE (one decimal should be more than enough).

      Thanks for your advisable comment. We revised the figure 2H in our revised version.

      The comparison of cell number and distribution within embryos (e.g. Figure 2B) would be best represented by a correlation analysis of TE vs ICM cells.

      Thanks for your advisable comment. We add the figure of a correlation analysis of TE vs ICM cells in our revised version (Figure 3B).

      The docking simulations are described in the main text as "experiments".

      Thanks for your advisable comment. We redescribed the docking simulations in our revised version.

      (8) Issue with data interpretation:

      The reduced number of ICM cells is interpreted as a slowed-down cell cycle. This could also be explained by failed cytokinesis and the generation of binucleated or polyploid cells. Have the authors checked for that? For example, by looking at their DAPI staining. 

      Thanks for your advisable comment. Our RNA-seq results revealed that the differentially expressed genes (DEGs) at blastocyst stage with HSPA2 knocking down are closely related to negative regulation of cell cycle, G1/S transition of mitotic cell cycle, mitotic cell cycle phase transition and regulation of mitotic cell cycle phase transition. Additionally, the previous study demonstrated that knockdown of HSPA2 reduced cell proliferation and led to G1/S phase cell cycle arrest (Hu Ann Transl Med 2019). Additionally, the lower cell number in ICM may also associated with failed cytokinesis and the generation of binucleated or polyploid cells. Thus, we guessed that HSPA2 has a role in ICM lineage establishment, although half of the ICM cells were able to survive with HSPA2 deficiency (line 463-472).

      It is unclear to me why reduced ICM should lead to fewer blastocysts. Blastocysts should be able to form as long as their TE is fine. In Figure 2G, embryos seem to be cultured in close proximity, which is fine if they are healthy but not if some of the embryos start dying and releasing toxic compounds (e.g. ROS). Have the authors tried removing the dying KD embryos to see if the development of the remaining embryos would improve?

      Thanks for your advisable comment. We think HSPA2 may affect blastocyst development by affecting other signaling pathways. And, the GO enriched terms was closely related to blastocyst development (Figure 2E). There was no significant difference in morula formation rate between Hspa2-KD group and NC group, thus the assumption that the toxic compounds released by some of the embryos that lead to downregulation of blastocyst rate may not be correct. Indeed, the rate of blastocyst formation in Hspa2-KD embryos was reduced significantly lower when few embryos was cultured separately. In addition, we discussed the possibility that the lower cell number in ICM may also associated with failed cytokinesis and the generation of binucleated or polyploid cells.

      Author response image 2.

      Reviewer #2 (Recommendations for the authors):

      One of the significant findings in the paper is the discovery portion where Hspa2 is identified as a heterogeneous transcript. To improve the logic and impact of the manuscript, it may benefit from reorganizing some of the figures and text. For example:

      (1) The paragraph in the introduction (Lines 56-68) should be moved to the discussion as the Hspa2 reveal should be in section 3.1, not prior to the RNA-seq results presented in Figure 1.

      Thanks for your advisable comment. We think it is more logical that HSPA2 needs to be introduced in the introduction.

      (2) Add text at the beginning of Section 3.1 to describe the rationale and results for the RNAseq. It would help the readers if the authors clearly stated why they chose the 4-cell stage.

      Thanks for your advisable comment. We explain why we chose the 4-cell stage in our revised version (line 272-273).

      (3) As this is the first time Hspa2 is identified, consider moving Figure S1C to the main figure to show expression throughout development.

      Thanks for your advisable comment. We moved Figure S1C to the main figure in our revised version (line 286-291).

      (4) Figure 1C: the correlation between Hspa2 and ICM markers would be strengthened if additional transcripts were used (Oct4, Sox2, Sox21). The graph in 1C would also be more informative if represented as a scatter plot with correlation coefficients (Nanog log2TPM vs Hspa2 log2TPM), rather than bar graphs.

      Thanks for your advisable comment. We chose Nanog as the correlation between Hspa2 and Nanog, a ICM markers, was showing the strongest correlation in result. And, the figure 1C shows the stronger positive correlation between Nanog and Hspa2 in gene expression than random gene pairs (n=100, n means the number of random gene pairs). Thus, the figure 1C with bar graphs is easier to understand.

      (5) Figure 1D: how were individual blastomeres grouped into B1-4? Individually run and then pooled based on relative expression?

      Thanks for your advisable comment. Blastomeres are named B1 to B4 according to increasing Hspa2 concentration in figure 1E.

      (6) Figures 1F, 1I, 5H: the DAPI channel appears to be saturated, but is used to normalize fluorescence intensity and may incorrectly account for light scattering within the embryo. Please clarify by adding more details regarding image analysis. Were partial stacks through the nucleus used for analysis, or max projections? Graph axes should be "relative fluorescence intensity."

      Thanks for your advisable comment. We added the details of fluorescence images analysis. The graph axes had revised in our revised version.

      (7) Line 278: the results in Figure S1C would benefit from more text regarding expression patterns throughout development. The maternal transcript appears to have a sharp downregulation by the early 2-cell stage, and is then upregulated coinciding with ZGA.

      Thanks for your advisable comment. We added more describe of the Figure in main text (LINE 285-290).

      (8) For the analyses in Figure 2 I-J and 2K-L, were arrested embryos excluded from analysis? This is an important detail as including arrested embryos would significantly bias the RNA-seq results. 

      Thanks for your advisable comment. The arrested embryos were excluded in Figure 2 I-J and 2K-L.

      (9) Figures 2G-H would be aided by converting the table in 2H to a bar graph and adding development rates for all stages (2-, 4-, 8-, morula, and blast). This would also show when an arrest occurs.

      Thanks for your advisable comment. We converted the table in 2H to a bar graph.

      (10) Blast rates are represented with too many significant digits (Figures 2H, 4B). They should only be reported to the closest ones given the unit of measure (number of blasts divided by number of zygotes). For instance, a blast rate of 81.63 {plus minus} 2.000 reflects excessive precision that is not measured in the data, it should rather read 82 {plus minus} 2%. This is also true for % cells (Figures 3E, 4H).

      Thanks for your advisable comment. Values were rounded down to the one decimal place (rounded down).

      (11) The clarity and impact of Figure 3A and 3D would benefit from 2D slices through the ICM. 

      Thanks for your advisable comment. In order to get more comprehensive understanding of the 3D structure of blastocyst of Figure 3A and 3D, we did not choose 2D slices.

      (12) To improve clarity and logic, separate the 1-cell and 2-cell knockdown experiments in the text and figures:

      a) 1-cell knockdown with RNA-seq results (Fig 2A-F).

      b) 1-cell knockdown showing less ICM/pluripotency markers in (combine Figures 2G-M and Figures 3A-B; "new Fig 3").

      c) 2-cell knockdown tracing lineage (Figures 2D-E; "new Fig 4").

      The new Figures 3 and 4 should mirror one another (i.e. for each knockdown experiment, development rates and cell counts should be included). For the 2-cell knockdown (Figures 2 D-E), what were the developmental rates (8-cell, morula, blast)?

      Thanks for your advisable comment. However, in order to the overall logical of the article, we do not separate the 1-cell and 2-cell knockdown experiments in the text and figures. And, we added the developmental rates (8-cell, morula, blast) of 2-cell knockdown group in our revised version (Figure S2).

      For the overexpression experiment (Figure 4), why were injections performed at the zygote stage versus the 2-cell stage? Given the significant downregulation of maternal transcript demonstrated in Figure S1C, it seems plausible that the injected RNA was also downregulated.

      Thanks for your advisable comment. For the overexpression experiment, we first chose to inject Hspa2 mRNA at the zygote stage and found that the overexpression of Hspa2 does not induce blastomere cells to bias an ICM fate. The qRT-PCR results indicated that the expression level of Hspa2 in overexpression group was significantly increased compared with normal group at 4cell and blastocyst stage (Figure 4C, 4D).  In addition, there is no guarantee that an equal amount of Hspa2 mRNA be injected into each blastomere in 2-cell stage. Thus, we did not microinject Hspa2 mRNA into the 2-cell stage.

      The 3.5 subheading overstates the results as the Hspa2-Carm1 interaction is not linked to lineage segregation. For example, a more specific subtitle might be, "Hspa2 interacts with Carm1 and alters H3R26me2 levels."

      Thanks for your advisable comment. We revised the subtitle in our revised version (line 376).

      Figures 5B-C and 5D-E. The qRT-PCR and WB analysis of knockdown blasts shows a correlation between Hspa2 downregulation and Carm1 downregulation. However, if the proposed mechanism is Hspa2 binding to Carm1 to mediate downstream methylation, why would it be expected to alter transcript levels at the 4-cell or blast stage? Please add further details and discussion in the results and discussion sections.

      Thanks for your advisable comment. The reason we chose to work at the 4-cell stage is because previous studies on CARM1 have focused on the 4-cell stage (Zernicka-Goetz Cell 2018,2016). 

      In the discussion, the statement in Lines 430-431 is an overinterpretation: "the heterogeneity of HSPA2... acts as an upstream factor to drive [the] first cell-fate decision." The knockdown experiments don't alter heterogeneity per se, but total abundance. Furthermore, the results do not show that heterogeneity drives heterogeneity in H3R26me2 patterns, for example.

      Thanks for your advisable comment. We redescribe the relevant statement in the discussion.

      More needs to be said regarding the ICM cells that persisted in the 1-cell KD experiment (Fig 3B). Lines 449-450 point out this result, but do not propose any plausible explanations. For instance, ICM cells may still form due to the incomplete knockdown achieved or the possibility that redundant pathways exist.

      Thanks for your advisable comment. We redescribe the relevant statement in our revised version (line 468-473).

      The 5th paragraph of the discussion seems incomplete. The authors point out a possible link between Hspa2 and Hippo and Wnt signaling pathways, but need to expand their discussion on how this may act as an additional mechanism incorporating Hspa2 with lineage segregation.

      Thanks for your advisable comment. We redescribe the 5th paragraph of the discussion (line 483-494).

      Statistics: all comparisons with greater than 2 groups should be performed with a one-way ANOVA and multiple comparisons, rather than Student's t-test (Figures 1B, 1D, 1E, 1F).

      All figure legends lack statistical test details.

      Thanks for your advisable comment. All figure legends added statistical test details in statistical analysis.

      Minor comments:

      In all graphs, individual blastomere expression levels should be represented as boxwhisker/bar/scatter/violin plots since the comparison is groups rather than time points (i.e. symbols should not be connected with a line in Figures 1B, 1D, 1F-G, 1I, S1D, S1F).

      Thanks for your advisable comment. Each colored line represents a single cell, and the dots of the same color represent the blastomere of the same cell. Thus, we use a line representation individual blastomere.

      For all fluorescent images, having two representative images may be confusing for the reader. Figures may be improved by just including one representative image for each stage/treatment (Figures 1F, 1I, S1F, 3A, 3D, 4E, 4G).

      Thanks for your advisable comment. The figures just including one representative image for each stage in our revised version. In addition, two representative images from each group were shown for each treatment (Figures 3A, 3D, 4E, 4G).

      The manuscript would be improved with thorough grammar and typo editing.

      For example:

      (1) Lines 18, 73, the wording is confusing, consider: "knockdown of Hspa2 in one of the two-cell blastomeres biased its progeny towards the trophectoderm lineage.".

      (2) Line 23, overstatement. Consider: "we demonstrated that HSPA2 levels correlate with ICMassociated genes and that it interacts with the CARM1.".

      (3) Line 25 confusing wording, "via the execution of commitment and differentiation phases.".

      (4) Line 37, replace "that" with "of;" replace "cell-fate decisions" with "cell-fate decision".

      (5) Line 40: needs space before (CARM1).

      (6) Line 43: the wording is confusing, consider "can result in higher expression levels of".

      (7) Line 45: wording, consider "Recent [studies have] further suggested".

      (8) Line 70: plurality, consider "analyzed gene expression pattern".

      (9) Line 73 typo: "prevents its".

      (10) Line 76-77 wording, consider "Hspa2 expression patterns can bias cell fate in the mouse embryo".

      (11) Line 276: remove "in whole embryos," since MII eggs are not embryos.

      (12) Line 617 "There" should be "Three".

      (13) Axis label in Fig 3b "Totle" should be "Total".

      (14) Lines 417, 419 missing spaces.

      (15) Line 448 missing word, "interfering [with] the cell cycle".

      (16) Line 462 incorrect word, "[a]polar cells being specified as ICM".

      (17) Line 469 incorrect plural, "cell differentiation".

      Thanks for your advisable comment. We revised the whole manuscript carefully according to the reviewers' suggestions.

    1. Author response:

      The following is the authors’ response to the original reviews

      Reviewer #1:

      Summary:

      The manuscript by Zhang et al describes the use of a protein language model (pLM) to analyse disordered regions in proteins, with a focus on those that may be important in biological phase separation. While the paper is relatively easy to read overall, my main comment is that the authors could perhaps make it clearer which observations are new, and which support previous work using related approaches. Further, while the link to phase separation is interesting, it is not completely clear which data supports the statements made, and this could also be made clearer.

      We thank the reviewer for their thoughtful evaluation of our manuscript and for the supportive comments. As outlined in the responses below, we have made substantial revisions to clarify the novel observations presented in our study and to strengthen the connection between sequence conservation and phase separation.

      Comment 1: With respect to putting the work in a better context of what has previously been done before, this is not to say that there is not new information in it, but what the authors do is somewhat closely related to work by others. I think it would be useful to make those links more directly.

      We have addressed the specific comments as outlined below.

      Comment 1a: Alderson et al (reference 71) analysed in detail the conservation of IDRs (via pLDDT, which is itself related to conservation) to show, for example, that conserved residues fold upon binding. This analysis is very similar to the analysis used in the current study (using ESM2 as a different measure of conservation). Thus, the result that "Given that low ESM2 scores generally reflect mutational constraint in folded proteins, the presence of region a among disordered residues suggests that certain disordered amino acids are evolutionarily conserved and likely functionally significant" is in some ways very similar to the results of that (Alderson et al) paper .

      We thank the reviewer for the comment. However, we would like to clarify that our findings show subtle but important differences from those reported by Alderson et al. Specifically, Alderson et al. used AlphaFold2 predictions to identify IDRs that undergo disorder-to-order transitions, which the authors termed as conditionally folded IDRs. These regions could potentially be functionally important, assuming that function of IDRs necessitate folding.

      We argue, however, that, the validity of this structure-function relationship for IDRs remains to be tested. In our opinion, The most direct way to evaluate the functional significance is via evaluating the evolutionary conservation.

      As shown in Author response image 1, the correlation between pLDDT scores and the conservation score, while noticable, is significantly weaker than that between the ESM2 score and the conservation score.

      Author response image 1.

      Comparison of the correlation between AlphaFold2 pLDDT scores and conservation scores with the correlation between ESM2 scores and conservation scores. Calculations were performed using proteins in the MLO-hProt dataset. (A) Correlation between the mean AlphaFold2 pLDDT scores and conservation scores for various amino acids. Pearson correlation coefficients (r) are indicated in the figure legends. The four panels on the right present analogous correlation plots for amino acids grouped by structural order, as defined by their pLDDT scores. (B) Similar as in part A but for ESM2 scores.

      Therefore, we believe that ESM2 score is a better indicator than AlphaFold2 pLDDT score for functional relevance.

      Furthermore, for the human IDRs, we explicitly selected amino acids with pLDDT scores ≤ 70.

      These would be classified as structureless, disordered amino acids, according to the study by Alderson et al. Nevertheless, as shown in Figures 2 and 3 of the main text, our analyses still identifies conserved regions. Therefore, these regions may function via distinct mechanisms than the disorder to order transition.

      We now discuss the novelty of our work in the context of existing studies in the newly added Conclusions and Discussion: Related Work, as quoted below.

      “Numerous studies have sought to identify functionally relevant amino acid groups within IDRs [cite]. For instance, using multiple sequence alignment, several groups have identified evolutionarily conserved residues that contribute to phase separation [cite]. Alderson et al. employed AlphaFold2 to detect disordered regions with a propensity to adopt structured conformations, suggesting potential functional relevance [alderson et al].

      In contrast, our approach based on ESM2 is more direct: it identifies conserved residues without relying on alignment or presupposing that functional significance requires folding into stable 3D structures. Notably, many of the conserved residues identified in our analysis exhibit low pLDDT scores (Figure 2), implying potential functional roles independent of stable conformations.”

      Comment 1b: Dasmeh et al, Lu et al and Ho & Huang analysed conservation in IDRs, including aromatic residues and their role in phase separation.

      We thank the reviewer for bringing these works to our attention! We now explicitly discuss these studies in both the Discussion section as mentioned above and in the Introduction as quoted below.

      “Evolutionary analysis of IDRs is challenging due to difficulties in sequence alignment [cite], though several studies have attempted alignment of disordered proteins with promising results [Dasmeh et al, Lu et al and Ho & Huang].”

      Comment 1c: A number of groups have performed proteomewide saturation scans using pLMs, including variants of the ESM family, including Meier (reference 89, but cited about something else) and Cagiada et al (https://doi.org/10.1101/2024.05.21.595203) that analysed variant effects in IDRs using a pLM. Thus, I think statements such as "their applicability to studying the fitness and evolutionary pressures on IDRs has yet to be established" should possibly be qualified.

      We added a new paragraph in the Introduction to discuss the application of protein language models to IDRs and cited the suggested references.

      “While protein language models have been widely applied to structured proteins [cite], it is important to emphasize that these models themselves are not inherently biased toward folded domains. For example, the Evolutionary Scale Model (ESM2) [cite] is trained as a probabilistic language model on raw protein sequences, without incorporating any structural or functional annotations. Its unsupervised learning paradigm enables ESM2 to capture statistical patterns of residue usage and evolutionary constraints without relying on explicit structural information. Thus, the success of ESM2 in modeling the mutational landscapes of folded proteins [cite] reflects the model’s ability to learn sequence-level constraints imposed by natural selection — a property that is equally applicable to IDRs if those regions are also under functional selection. Indeed, protein language models are increasingly been used to analyze variant effects in IDRs [cite].”

      Comment 2: On page 4, the authors write, "The conserved residues are primarily located in regions associated with phase separation." These results are presented as a central part of the work, but it is not completely clear what the evidence is.

      We thank the reviewer this insightful comment. We realized that our wording is not as precise as we should have been. We meant to state that the regions associated with phase separation are significantly enriched in these conserved residues. This is a significant finding and indicates that phase separation could be a source of evolutionary pressure in dictating IDP sequence conservation. However, we do not intend to suggest that phase separation is the only evolutionary pressure.

      The sentence has been revised to

      “Notably, regions associated with phase separation are significantly enriched in these conserved residues.”

      We further replaced the section title "Conserved, Disordered Residues Localize in Regions Driving Phase Separation" with "Regions Driving Phase Separation Are Enriched with Conserved, Disordered Residues" to further clarify our findings and avoid overinterpretation.

      Finally, we revised the following sentence in the discussion

      “Notably, these conserved, disordered residues are predominantly located in regions actively involved in phase separation, contributing to the formation of membraneless organelles.”

      to

      “Notably, regions actively involved in phase separation are enriched with these conserved, disordered residues, supporting their potential role in the formation of membraneless organelles.”

      The submitted manuscript provides clear evidence supporting the enrichment of conserved residues in MLO-driving IDRs. Specifically, Figures 4A and 4C demonstrate that these IDRs exhibit a substantially higher fraction of conserved residues compared to other IDRs involved in phase separation.

      In this analysis, the nMLO-hIDR group serves as a baseline, representing the distribution of conservation in disordered regions lacking MLO-related functions. In contrast, IDRs from MLOassociated groups show a pronounced lower shift in their median and interquartile ranges, indicating stronger evolutionary constraints. Within the dMLO cohort, the degree of conservation follows a distinct gradient: driving residues exhibit the highest levels of conservation, followed by participant residues, with non-participant residues showing values closer to the nMLO baseline. This pattern reflects the relative functional importance of each group in phase separation, with conservation levels corresponding to their roles in MLO scaffolding.

      To further support this, we computed, for each IDR, the fraction of conserved amino acids. As shown in Figure S11B, for IDRs that actively contribute to phase separation, the fraction is indeed higher than those not involved in phase separation. This analysis is now included in SI.

      During the revision, we explicitly evaluated whether conserved residues are preferentially located in regions associated with phase separation. To this end, for each protein in the MLO-hProt dataset, we calculated the probability p of finding conserved residues within regions contributing to phase separation. These regions include both "driving" and "participating" segments as defined in Figure 4 of the main text.

      Figure S11A presents the distribution of p across all proteins. For comparison, we also include the distribution of 1− p, representing the probability of finding conserved residues in regions not associated with phase separation. On average, p exceeds 0.5, suggesting a tendency for conserved residues to be more frequently located in phase-separating regions. However, the difference between the two distributions is not statistically significant. This result may be due to the generally low density of conserved residues in IDRs, which makes the estimation of p challenging for individual proteins. Additionally, some conserved sites may be involved in functions unrelated to phase separation.

      We added the following text to the Discussion section of the main text.

      “We emphasize that the results presented in Figure 4 do not directly demonstrate that conserved residues are preferentially located in regions associated with phase separation. Although these regions are more enriched in conserved amino acids, their total sequence length can be smaller than that of non-phase-separating regions. As a result, the absolute number of conserved residues may still be higher outside phase-separating regions. To quantitatively assess this, we calculated, for each protein in the MLO-hProt dataset, the probability p of finding conserved residues within regions contributing to phase separation. These regions include both "driving" and "participating" segments, as defined in Figure 4 of the main text. Figure S11 shows the distribution of p across all proteins. For comparison, we also present the distribution of 1− p, which reflects the probability of finding conserved residues in non-phase-separating regions. While the average value of p exceeds 0.5, indicating a trend toward conserved residues being more frequently located in phase-separating regions, the difference between the two distributions is not statistically significant. Future studies with expanded datasets may be necessary to clarify this trend.”

      Comment 3: It would be useful with an assessment of what controls the authors used to assess whether there are folded domains within their set of IDRs.

      We acknowledge that our previous labeling may have caused some confusion. Protein sequences used in Figures 2 and 3 include both folded and disordered domains. Results presented in these figures were constructed using full-length protein sequences to highlight the similarities and differences in ESM2 scores between folded and disordered domains.

      In contrast, the analyses presented in Figures 4 and 5 focus exclusively on IDRs to examine their role in phase separation.

      To prevent further confusion, we have renamed the dataset used in Figures 2 and 3 as MLO-hProt, emphasizing that the analysis pertains to entire protein sequences. The term MLO-hIDR is now reserved for a new dataset that includes only disordered residues, as used in Figures 4 and 5, and corresponding SI Figures.

      For the dMLO-IDR dataset, all except one amino acid (P40967, residue G592) are annotated as disordered in the MobiDB database (https://mobidb.org/). This database characterizes disordered regions based on a combination of predictive algorithms and experimental data. As illustrated in Figure S5A, 25.5% of the proteins in the dataset have direct experimental evidence supporting their disorderedness. These experimental annotations are derived from a diverse range of techniques (Figure S5B). For the remaining proteins, disorder was predicted by one or more computational tools. Although not all tools were applied to every protein, each protein in the dataset was identified as disordered by at least one method.

      For human proteins, IDRs were identified based on AlphaFold2 pLDDT scores, using a threshold of 70. As established in prior studies [1, 2], the pLDDT score provides a quantitative measure of local structural confidence, with lower values indicating greater structural disorder. IDRs associated with conditional folding or disorder-to-order transitions generally exhibit high pLDDT values (e.g., >70).

      Author response image 2 shows a violin plot of AlphaFold2 pLDDT scores for the various MLO-hIDR groups. The consistently low scores support the conclusion that these regions are structurally disordered.

      We also cross-checked the MLO-hIDR regions against the MobiDB database. As shown in Figure S6, approximately 76% of the proteins in the dataset are predicted to contain disordered regions. Among the non-labeled segments with pLDDT scores ≤ 70, the majority are relatively short, with segments of 1–5 residues accounting for approximately 80%.

      Author response image 2.

      AlphaFold pLDDT scores of hIDRs in different MLO-related groups.

      In addition to renaming the dataset, we also revised the manuscript to highlight the validation of disorderedness in section of Results: Regions Driving Phase Separation Are Enriched with Conserved, Disordered Residues.

      “The presence of evolutionarily conserved disordered residues raises the question of their functional significance. To explore this, we identified disordered regions of MLO-hProt using a pLDDT score less than 70 and partitioned these regions into two categories: drivers (dMLO-hIDR), which actively drive phase separation, and clients (cMLO-hIDR), which are present in MLOs under certain conditions but do not promote phase separation themselves [cite]. Additionally, IDRs from human proteins not associated with MLOs, termed nMLO-hIDR, were included as a control. To enhance statistical robustness, we extended our dataset by incorporating driver proteins from additional species [cite], resulting in the expanded dMLO-IDR dataset. Beyond the pLDDT-based classification, the majority of residues in these datasets are also predicted to be disordered by various computational tools and supported by experimental evidence (Figures S5 and S6).”

      Recommendation 1: The authors use the terms "evolutionary fitness of IDRs" (abstract and p. 5, for example), "fitness of amino acids" (p. 4), and "quantify the fitness of particular residues at specific sites" (p. 5). It is not clear what is meant by fitness in this context.

      We thank the reviewer for pointing out the ambiguity in the term fitness. To enhance clarity, we have replaced “fitness" with “mutational tolerance" to more directly emphasize the evolutionary conservation of specific residues.

      Recommendation 2: The authors write (p. 6) "Previous studies have demonstrated a strong correlation between ESM2 scores and changes in free energy related to protein structure stability". While that may be true, it might be worth noting that ESM2 scores report on the effects of mutations and function more broadly than stability because these models have previously been shown to capture conservation effects beyond stability.

      We fully agree with the reviewer’s comment and have revised the main text accordingly. Specifically, the referenced sentence has been revised and relocated, as shown below.

      “Our analysis demonstrated that HP1_α_’s structured domains consistently yield low ESM2 scores, reflecting strong mutational constraints characteristic of folded regions. These constraints are further evident in the local LLR predictions, as shown in Figure 2B, where we illustrate the folded region G120-T130. Given the functional importance of preserving the 3D of structured domains, mutations with greater detrimental effects are likely to disrupt protein folding substantially. This interpretation is consistent with previous studies reporting a significant correlation between ESM2 LLRs and changes in free energy associated with protein structural stability [cite].”

      Recommendation 3: p. 10: The authors write "To exclude sequences that no longer qualify as homologs, we filtered for sequences with at least 20% identity to the reference". How did they decide on 20% and why? And over which residues are these 20% calculated.

      We apologize for the earlier lack of clarity. Sequence alignment was performed using the full-length protein sequences, encompassing both folded and disordered regions. For each sequence, we calculated the percent identity by counting the number of positions, denoted as n, at which the amino acid matches the reference. The percent identity was then computed as n/N, where N represents the total length of the aligned reference sequence. This total includes residues in folded and disordered regions, as well as gap positions introduced during alignment.

      We updated the Methods section of the main text to clarify.

      “We performed multi-sequence alignment (MSA) analysis using HHblits from the HH-suite3 software suite [citations], a widely used open-source toolkit known for its sensitivity in detecting sequence similarities and identifying protein folds. HHblits builds MSAs through iterative database searches, sequentially incorporating matched sequences into the query MSA with each iteration. Sequence alignment was performed using the full-length protein sequences, encompassing both folded and disordered regions.

      ...

      To refine alignment quality by focusing on closely related homologs, we filtered out sequences with ≤ 20% identity to the query, excluding weakly related sequences where only short segments show similarity to the reference. For each sequence, we calculated the percent identity by counting the number of positions, denoted as n, at which the amino acid matches the reference. The percent identity was then computed as n/N, where N represents the total length of the aligned reference sequence. This total includes residues in folded and disordered regions, as well as gap positions introduced during alignment.”

      We selected a 20% sequence identity threshold to balance inclusion of true homologs with exclusion of distant matches that may not share functional relevance. To determine this cutoff, we compared identity thresholds of 0%, 10%, 20%, and 40% and examined the resulting distributions of conservation and ESM2 scores across aligned residues for MLO-hProt dataset (Author response image 3). Thresholds of 10%, 20%, and 40% produced qualitatively similar results, with a consistent correspondence between low ESM2 scores and high conservation. Lower thresholds introduced highly divergent sequences that added noise to the alignment, resulting in reduced overall conservation scores. In contrast, higher thresholds excluded homologs with potentially meaningful conservation, particularly in disordered regions where conservation scores tend to be relatively low.

      Author response image 3.

      Histograms of the ESM2 score and the conservation score, presented in a format consistent with Figure 3B of the main text. The conservation scores were computed using aligned sequences with identity thresholds of ≥0, ≥10%, ≥20%, and ≥40% (left to right). Contour lines represent different levels of −log_P_(CS,ESM2), where P is the joint probability density of conservation score (CS) and ESM2 score. Contours are spaced at 0.5-unit intervals, highlighting regions of distinct density.

      Recommendation 4: In their description of "motif" searching algorithm (p. 20) I think that the search algorithm would give a different result whether the search is performed N->C or C->N (because the first residue (i) needs to have a score <0.5 but the last (j) could have a score >0.5 as long as the average is below 0.5. Is that correct? And if so, why did they choose an asymmetric algorithm? .

      We thank the reviewer for highlighting the asymmetry in our motif-search algorithm.

      To investigate this issue, we repeated the algorithm starting from the C-terminus and compared the resulting motifs with those obtained from the N-terminal scan. We found that the two sets of motifs overlap entirely: each motif identified from the C-terminal direction has a corresponding counterpart from the N-terminal scan. However, the motifs are not identical. The directionality of the search introduces additional amino acids—referred to here as peripheral residues—at the motif boundaries, which differ between the two sets.

      As shown in Author response image 4, the number of peripheral residues is small relative to the total motif length.

      To eliminate asymmetry and ambiguity, we have revised our method to perform bidirectional scans—from both the N- and C-termini—and define each motif as the overlapping region identified by both directions. This approach emphasizes the conserved core and avoids the inclusion of spurious terminal residues. The updated procedure is described in Methods: Motif Identification.

      “To identify motifs within a given IDR, we implemented the following iterative procedure. Starting from either the N– or C–terminus of the sequence, we first locate the initial residue i whose ESM2 score falls within 0.5. From i, residues are sequentially appended…”

      Author response image 4.

      Number of peripheral residues and their relative length to the full-motif length identified from both sides. (A). The unique motifs identified from N-to-C terminal direction. (B) The unique motifs identified from C-to-N terminal direction.

      “…in the direction toward the opposite terminus until the segment’s average ESM2 score exceeds 0.5; the first residue to breach this threshold is denoted j. The segment (i,i+1,..., j−1) is then recorded as a candidate motif. This process repeats starting from j until the end of the IDR is reached.

      We perform this full procedure independently from both termini and designate the final motif as the intersection of the two candidate-motif sets. This bidirectional overlap strategy excludes terminal residues that might transiently satisfy the average-score criterion only due to adjacent low-scoring regions, thereby isolating the conserved core of each motif. All other residues—those not included in either directional pass—are classified as non-motif regions, minimizing peripheral artifacts.”

      Accordingly, we have updated the Supplementary material: ESM2_motif_with_exp_ref.csv for the new identified motifs commonly exited from both N-terminal and C-terminal searches. Minor changes were observed in the set of motifs as being discussed, but these do not affect the main conclusions. Figures 5C, 5D, and S6 have been revised accordingly.

      Reviewer #2:

      Summary:

      Unfortunately, I do not believe that the results can be trusted. ESM2 has not been validated for IDRs through experiments. The authors themselves point out its little use in that context. In this study, they do not provide any further rationale for why this situation might have changed. Furthermore, they mention that experimental perturbations of the predicted motifs in in vivo studies may further elucidate their functional importance, but none of that is done here. That some of the motifs have been previously validated does not give any credibility to the use of ESM2 here, given that such systems were probably seen during the training of the model.

      We thank the reviewer for their detailed and thoughtful critique of our manuscript. We recognize the importance of careful model validation, especially in the context of IDRs, and appreciate the opportunity to clarify the scope and rationale of our study. Below, we respond point-by-point to the main concerns.

      (1) The use of ESM2 is not validated for IDRs, and the authors provide no rationale for its applicability in this context.

      We thank the reviewer for raising this important point.

      First, we emphasize that ESM2 is a probabilistic language model trained entirely on amino acid sequences, without any structural supervision. The model does not receive any input about protein structure — folded or disordered — during training. Instead, it learns to estimate the likelihood of each amino acid at a given position, conditioned on the surrounding sequence context. This makes ESM2 agnostic to whether a sequence is folded or disordered; the model’s capacity to identify patterns of residue usage arises solely from the statistics of natural sequences.

      As such, ESM2 is not inherently biased toward folded proteins, even though previous studies have demonstrated its usefulness in identifying conserved and functionally constrained residues in structured domains [3–9]. These findings support the broader utility of language models for uncovering evolutionary constraints — and by extension, suggest that similar signatures could exist in IDRs, particularly if they are under functional selection.

      Indeed, if certain residues or motifs in IDRs are conserved due to their importance in biological processes (e.g., phase separation), we would expect such selection to be reflected in sequence-based features, which ESM2 is designed to detect. The model’s applicability to IDRs, then, is a natural extension of its core probabilistic architecture.

      To further evaluate this, we carried out an independent in silico validation using multiple sequence alignments (MSAs). This analysis allowed us to compute the evolutionary conservation of individual amino acids without any reliance on ESM2. We then compared these conservation scores to ESM2 scores and found a strong correlation between the two. This provides direct, quantitative support for the idea that ESM2 is capturing biologically meaningful sequence constraints — even in disordered regions.

      While we agree that experimental testing would ultimately provide the most compelling validation, we believe that our MSA-based comparison constitutes a strong and arguably ideal computational validation of the model’s predictions. It offers an orthogonal measure of evolutionary pressure that confirms the biological plausibility of ESM2 scores.

      We added the following text in the introduction to highlight the applicability of ESM2 to IDRs.

      “While protein language models have been widely applied to structured proteins, it is important to emphasize that these models themselves are not inherently biased toward folded domains. For example, the Evolutionary Scale Model (ESM2) [cite] is trained as a probabilistic language model on raw protein sequences, without incorporating any structural or functional annotations. It operates by estimating the likelihood of observing a given amino acid at a particular position, conditioned on the entire surrounding sequence context. This unsupervised learning paradigm enables ESM2 to capture statistical patterns of residue usage and evolutionary constraints without relying on explicit structural information. Thus, the success of ESM2 in modeling fitness landscapes of folded proteins reflects the model’s ability to learn sequence-level constraints imposed by natural selection — a property that is equally applicable to IDRs if those regions are also under functional selection. Indeed, protein language models are increasingly been used to analyze variant effects in IDRs [cite].”

      (2) There is no experimental validation of the ESM2-based predictions in this study.

      We agree that experimental validation would provide definitive support for the utility of ESM2 in IDRs, and we explicitly state this as a limitation in the revised manuscript as quoted below.

      “Limitations: Despite the promising findings, our study has several limitations. Most notably, our analysis is purely computational, relying on ESM2-derived predictions and sequence-based conservation without accompanying experimental validation. While the strong correlation between ESM2 scores and evolutionary conservation provides compelling evidence that the identified motifs are functionally constrained, the precise biological roles of these motifs remain uncharacterized. ESM2 is well-suited for highlighting regions under selective pressure, but it does not provide mechanistic insights into how conserved motifs contribute to specific molecular functions such as phase separation, molecular recognition, or dynamic regulation. Determining these roles will require targeted experimental investigations, including mutagenesis and biophysical characterization.”

      In addition, we revised the manuscript title from “Protein Language Model Identifies Disordered, Conserved Motifs Driving Phase Separation" to “Protein Language Model Identifies Disordered, Conserved Motifs Implicated in Phase Separation". This revision softens the original claim to better reflect the absence of direct experimental evidence for the motifs’ role in phase separation.

      However, we also emphasize that the goal of our study is not to claim definitive predictive power, but rather to explore whether ESM2-derived mutational profiles align with known biological features of IDRs — and in doing so, to generate new, testable hypotheses.

      In addition, while no in vivo experiments were performed, our study does include an in silico validation step, as detailed in the response to the previous comment. The strong correlation between ESM2 scores and conservation scores provides direct support for the utility of ESM2 in identifying residues under evolutionary constraint in disordered regions.

      (3) The overlap between predicted motifs and known ones may be due totraining data leakage.

      We respectfully clarify that training data leakage is not possible in this case, as ESM2 is trained using unsupervised learning on raw protein sequences alone. The model has no access to experimental annotations, functional labels, or knowledge of which motifs are involved in phase separation. It only models statistical sequence patterns derived from evolutionarily observed proteins.

      Therefore, any agreement between ESM2-derived predictions and previously validated motifs arises not from memorization of experimental data, but from the model’s ability to learn meaningful sequence constraints from the natural distribution of proteins.

      (4) The authors should revamp the study with a testable predictive framework.

      We respectfully suggest that a full revamp is not necessary or appropriate in this context.

      As outlined in our previous responses, we believe that certain misunderstandings about the nature and capabilities of ESM2 may have influenced the reviewer’s assessment.

      Importantly, both Reviewer 1 and Reviewer 3 express strong support for the significance and novelty of this work, and recommend publication following minor revisions.

      In this context, we believe the manuscript provides a useful contribution as a first step toward understanding disordered regions using language models, and that it has value even in the absence of direct experimental testing. We have now better positioned the manuscript in this light, clarified limitations, and suggested concrete next steps for follow-up research.

      We hope these clarifications and revisions address the reviewer’s concerns, and we thank them again for helping us strengthen the framing, rigor, and clarity of our study.

      Reviewer #3:

      Summary:

      This is a very nice and interesting paper to read about motif conservation in protein sequences and mainly in IDRs regions using the ESM2 language model. The topic of the paper is timely, with strong biological significance. The paper can be of great interest to the scientific community in the field of protein phase transitions and future applications using the ESM models. The ability of ESM2 to identify conserved motifs is crucial for disease prediction, as these regions may serve as potential drug targets. Therefore, I find these findings highly significant, and the authors strongly support them throughout the paper. The work motivates the scientific community towards further motif exploration related to diseases.

      Strengths:

      (1) Revealing conserved regions in IDRs by the ESM-2 language model.

      (2) Identification of functionally significant residues within protein sequences, especially in IDRs.

      (3) Findings supported by useful analyses.

      We appreciate the reviewer’s thoughtful words and support for our work.

      Weaknesses:

      (1) Lack of examples demonstrating the potential biological functions of these conserved regions.

      As detailed in the Response to Recommendation 6, we conducted additional analyses to connect the identified conserved regions with their biological functions.

      (2) Very limited discussion of potential future work and of limitations.

      We have substantially revised the Conclusions and Discussion section to provide a detailed analysis of the study’s limitations and to propose several directions for future research, as elaborated in our Response to Recommendation 5 below.

      Recommendation 1: The authors describe the ESM2 score such that lower scores are associated with conserved residues, stating that "lower scores indicate higher mutational constraint and reduced flexibility, implying that these residues are more likely essential for protein function, as they exhibit fewer permissible mutational states." However, when examining intrinsically disordered regions (IDRs), which are known to drive phase separation, I observe that the ESM2 score is relatively high (Figure 3C, pLDDT < 50, and Supplementary Figure S2). Could the authors clarify how this relatively high score aligns with the conservation of motifs that drive phase separation?

      We thank the reviewer for this insightful comment. We would like to clarify that most amino acids in the IDRs are not conserved, even for IDRs that contribute to phase separation. Only a small set of amino acids in these IDRs, which we term as motifs, are evolutionarily conserved with low ESM2 scores. Therefore, the ESM2 scores exhibit bimodal distribution at high and low values, as shown in Figures 4A and 4C of the manuscript. When averaged over all the amino acids, the mean ESM2 scores, plotted in Figure 3C, are relatively high due to dominant population of non-conserved amino acids.

      Recommendation 2: The authors mention: "We first analyzed the relationship between ESM2 and pLDDT scores for human Heterochromatin Protein 1 (HP1, residues 1-191)". I appreciate this example as a demonstration of amino acid conservation in IDRs. However, it is questionable whether the authors could provide some more examples to support amino acid conservation particularly within the IDRs along with lower ESM2 score (e.g, Could the authors provide some additional examples of "conserved disordered" regions in various proteins which are associated with relatively low ESM2 score as appear in Figure 2A).

      We thank the reviewer for this valuable suggestion. We want to kindly noted that the conserved residues on IDRs are prevalent as indicated in Figures 2D and 3B. To further illustrate the prevalence of “conserved disordered” regions, we generated ESM2 versus pLDDT score plots for the full dMLO–hProt dataset (82 proteins) in Figure S2. In these plots, residues with pLDDT ≤ 70 are highlighted in blue to denote structural disorder (dMLO-hIDR), and these disordered residues with ESM2 score ≤ 1.5 are shown in purple to indicate conserved disordered segments.

      Recommendation 3: Could the authors plot a Violin conservation score plot for Figure 4A to emphasise the relationship between ESM2 scores and conservation scores of disordered residues?

      We thank the reviewer for this suggestion. We included a violin plot illustrating the distribution of conservation scores for disordered residues across all four IDR groups, shown in Author response image 5. Consistent with the findings in Figure 4A, the phase separation drivers (dMLO-hIDR and dMLOIDR) exhibit a higher proportion of conserved amino acids compared to the client group (cMLOhIDR).

      We also note that the nMLO-hIDR group may contain conserved residues due to functions unrelated to MLO formation, which could contribute to the higher observed levels of conservation in this group.

      Author response image 5.

      Violin plots illustrating the distribution of conservation scores for disordered residues across the nMLO–hIDR, cMLO–hIDR, dMLO–hIDR, and dMLO–IDR datasets. Pairwise statistical comparisons were conducted using two-sided Mann–Whitney U tests on the conservation score distributions (null hypothesis: the two groups have equal medians). P-values indicate the probability of observing the observed rank differences under the null hypothesis. Statistical significance is denoted as follows: ***: p < 0.001; **: p < 0.01; *:p < 0.05;

      Recommendation 4: It will be appreciated if the authors could add to Figure 4 Violin plots, a statistical comparison between the different groups.

      We thank the reviewer for this valuable suggestion. We included the p-values for Figures 4A and 4C to quantify the statistical significance of differences in the distributions.

      Most comparisons are highly significant (p < 0.001), while the largest p-value (p = 0.089) between the conservation score of driving and non-participating groups (Figure 4C) still suggests a marginally significant trend.

      Recommendation 5: Could the authors expand more on potential future research directions using ESM2, given its usefulness in identifying conserved motifs? Specifically, how do the authors envision conserved motifs will contribute to future discoveries/applications/models using ESM (e.g, discuss the importance of conserved motifs, especially in IDRs motifs, in protein phase transition prediction in relation to diseases).

      We thank the reviewer for this insightful comment. To further assess the functional relevance of the conserved motifs, we incorporated pathogenic variant data from ClinVar [10, 11] to evaluate mutational impacts. As shown in Figure S12A and B, a substantial number of pathogenic variants in MLO-hProt proteins are associated with low ESM2 LLR values. This pattern holds for both folded and disordered residues.

      Moreover, we observed that variants located within motifs are more frequently pathogenic compared to those outside motifs (Figure S12C). In the main text, motifs were defined only for driver proteins; however, the available variant data for this subset are limited (6 data points). To improve statistical power, we extended motif identification to include both client and driver human proteins, following the same methodology described in the main text. Consistent with previous findings, variants within motifs in this expanded set are also more likely to be pathogenic. These results further support the functional importance of both low ESM2-scoring residues and the conserved motifs in which they reside.

      The following text was added in the Discussion section of the manuscript to discuss these results and outline future research directions.

      “Several promising directions could extend this work, both to refine our mechanistic understanding and to explore clinical relevance. One avenue is testing the hypothesis that conserved motifs in scaffold proteins act as functional stickers, mediating strong intermolecular interactions. This could be evaluated computationally via free energy calculations or experimentally via interaction assays. Deletion of such motifs in client proteins may also reduce their partitioning into condensates, illuminating their roles in molecular recruitment.

      To explore potential clinical implications, we analyzed pathogenicity data from Clin-Var [10, 11]. As shown in Figure S12A, single-point mutations with low LLR values—indicative of constrained residues—are enriched among clinically reported pathogenic variants, while benign variants typically exhibit higher LLR values. Moreover, mutations within conserved motifs are significantly more likely to be pathogenic than those in non-motif regions (Figure S12B). These findings highlight the potential of ESM2 as a first-pass screening tool for identifying clinically relevant residues and suggest that the conserved motifs described here may serve as priorities for future studies, both mechanistic and therapeutic.”

      Moreover, the functional significance of conserved motifs, particularly their implications in disease and pathology, warrants further investigation. As an initial analysis, we incorporated ClinVar pathogenic variant data [citation] to assess mutational effects within our datasets. As illustrated in Figure R12A, single-point mutations with low LLR values are enriched among clinically reported pathogenic variants, whereas benign variants are more commonly associated with higher LLR values. Notably, mutations within conserved motifs are substantially more likely to be pathogenic compared to those in non-motif regions. These findings highlight the potential of ESM2 as a firstpass tool for identifying residues of clinical relevance. The conserved motifs identified here may be prioritized in future studies aimed at elucidating their biological roles and evaluating their viability as therapeutic targets.

      Recommendation 6: The authors mention: "Our findings provide strong evidence for evolutionary pressures acting on specific IDRs to preserve their roles in scaffolding phase separation mechanisms, emphasizing the functional importance of entire motifs rather than individual residues in MLO formation." They also present a word cloud of functional motifs in Figure 5D. Although it makes sense that evolutionarily conserved motifs, especially within the IDRs regions, act as functional units, I think there is no direct evidence for such functionality (e.g., examples of biological pathways associated with IDRs and phase separation). Hence, there is no justification to write in the figure caption: "ESM2 Identifies Functional Motifs in driving IDRs" unless the authors provide some examples of such functionality. This will even make the paper stronger by establishing a clear connection to biological pathways, and hence these motifs can serve as potential drug targets.

      We thank the reviewer for this insightful suggestion. We have replaced “functional motifs" with “conserved motifs" in the figure caption.

      Identifying the precise biological pathways associated with the conserved motifs is a complex task and a comprehensive investigation lies beyond the scope of this study. Nonetheless, as an initial effort, we explored the potential functions of these motifs using annotations available in DisProt (https://disprot.org/).

      DisProt is the leading manually curated database dedicated to IDPs, providing both structural and functional annotations. Expert curators compile experimentally validated data, including definitions of disordered regions, associated functional terms, and supporting literature references. Author response image 6 presents a representative DisProt entry for DNA topoisomerase 1 (UniProt ID: P11387), illustrating its structural and biological annotation.

      For each motif, we located the corresponding DisProt entry and assigned a functional annotation based on the annotated IDR from which the motif originates. We emphasize that this functional assignment should be regarded as an approximation. Because experimental annotations often pertain to the entire IDR, regions outside the motif may also contribute to the reported function.

      Nevertheless, the annotations provide valuable insights.

      Author response image 6.

      Screenshot of information provided by the DisProt database. Detailed annotations of biological functions and structural features, along with experimental references, are accessible via mouse click.

      Approximately 50% of ESM2-predicted IDR motifs lack functional annotations. Among those that are annotated, motifs from the dMLO-IDR dataset are predominantly associated with “molecular condensate scaffold activity,” followed by various biomolecular binding functions (Author response image 7A). These findings support the role of these motifs in MLO formation.

      For comparison, we applied the same identification procedure (described in Methods: Motif Identification) to motifs from the nMLO-hIDR dataset. In contrast to the dMLO-IDR motifs, these exhibit a broader range of annotated functions related to diverse cellular processes. Collectively, these results suggest that motifs identified by ESM2 are aligned with biologically relevant functions captured in current databases.

      Finally, as illustrated in Figure S12 and discussed in the Response to Recommendation 5, variants occurring within identified motifs are more likely to be pathogenic than those in non-motif regions, further underscoring their functional importance.

      Author response image 7.

      Biological functions of ESM2-predicted motifs. (A) Distribution of biological functions associated with all identified motifs from dMLO-IDR driving groups. (B) Distribution of biological functions associated with all identified motifs from nMLO-hIDR groups.

      Recommendation 7: In Figure 2C the authors present FE (I assume this is free energy), some discussion about the difference in the free energy referring to the "a" region is missing (i.e. both "Folded" and "Disordered" regions are associated with low ESM score but with low and high free energy (FE), respectively.

      We thank the reviewer for the comments. FE indeed abbreviates free energy. To improve clarify and avoid confusion, we have updated all figure captions by replacing “FE” with “−logP” to explicitly denote the logarithm of probability in the contour density plots.

      We used “a" in Figures 2C and 2D to refer to regions with low ESM2 scores, which appears a local minimum in both plots. Since most residues in folded regions are conserved, region a has lower free energy than region b in Figure 2C. On the other hand, as most residues in disordered regions are not conserved, as we elaborated in Response to Recommendation 1, region a has lower population and higher free energy than region b.

      To avoid confusion, we have replaced “a" and “b" in Figure 2D with “I" and “II".

      Recommendation 8: Figure S2: It would be useful to plot the same figure for structured and disordered regions as well.

      We are not certain we fully understood this comment, as we believe the requested analysis has already been addressed. In Figure S2, we used the AlphaFold2 pLDDT score to represent the structural continuum of different protein regions, where residues with pLDDT > 70 (red and lightred bars) are classified as structured, while those with pLDDT ≤ 70 (blue and light-blue bars) are classified as disordered.

      Minor suggestion 1: Could the authors clarify the meaning of the abbreviation "FE" in the colorbar of the contour line? I assume this is free energy.

      We have updated all contour density plot figure captions by replacing “FE” with “−logP” to explicitly denote the logarithm of probability.

      Minor suggestion 2: In Figure 2A - do the authors mean "Conserved folded" instead of just "Folded"? If so, could the authors indicate this?

      We thank the reviewer for this comment. The ESM2 scores indeed suggest that, within folded regions, there may be multiple distinct groups exhibiting varying degrees of evolutionary conservation. However, as our primary focus is on IDRs, we chose not to investigate these distinctions further.

      Figure 2A illustrates a randomly selected folded region based on AlphaFold2 pLDDT scores.

      References

      (1) Ruff, K. M.; Pappu, R. V. AlphaFold and Implications for Intrinsically Disordered Proteins. Journal of Molecular Biology 2021, 433, 167208.

      (2) Alderson, T. R.; Pritišanac, I.; Kolaric, Ð.; Moses, A. M.; Forman-Kay, J. D. Systematic´ Identification of Conditionally Folded Intrinsically Disordered Regions by AlphaFold2. Proceedings of the National Academy of Sciences of the United States of America, 120, e2304302120.

      (3) Brandes, N.; Goldman, G.; Wang, C. H.; Ye, C. J.; Ntranos, V. Genome-Wide Prediction of Disease Variant Effects with a Deep Protein Language Model. Nature Genetics 2023, 55, 1512–1522.

      (4) Lin, Z. et al. Evolutionary-Scale Prediction of Atomic-Level Protein Structure with a Language Model. 2023.

      (5) Zeng, W.; Dou, Y.; Pan, L.; Xu, L.; Peng, S. Improving Prediction Performance of General Protein Language Model by Domain-Adaptive Pretraining on DNA-binding Protein. Nature Communications 2024, 15, 7838.

      (6) Gong, J. et al. THPLM: A Sequence-Based Deep Learning Framework for Protein Stability Changes Prediction upon Point Variations Using Pretrained Protein Language Model. Bioinformatics 2023, 39, btad646.

      (7) Lin, W.; Wells, J.; Wang, Z.; Orengo, C.; Martin, A. C. R. Enhancing Missense Variant Pathogenicity Prediction with Protein Language Models Using VariPred. Scientific Reports 2024, 14, 8136.

      (8) Saadat, A.; Fellay, J. Fine-Tuning the ESM2 Protein Language Model to Understand the Functional Impact of Missense Variants. Computational and Structural Biotechnology Journal 2025, 27, 2199–2207.

      (9) Chu, S. K. S.; Narang, K.; Siegel, J. B. Protein Stability Prediction by Fine-Tuning a Protein Language Model on a Mega-Scale Dataset. PLOS Computational Biology 2024, 20, e1012248.

      (10) Landrum, M. J.; Lee, J. M.; Riley, G. R.; Jang, W.; Rubinstein, W. S.; Church, D. M.; Maglott, D. R. ClinVar: Public Archive of Relationships among Sequence Variation and Human Phenotype. Nucleic Acids Research 2014, 42, D980–D985.

      (11) Landrum, M. J. et al. ClinVar: Improving Access to Variant Interpretations and Supporting Evidence. Nucleic Acids Research 2018, 46, D1062–D1067.

    1. Author Response

      The following is the authors’ response to the original reviews.

      Thank you for the thoughtful consideration of our work, including both reviewers’ constructive comments. Our apologies for taking some extra time for this revision, but we wanted to adress comments thoroughly with new analyses, not to mention a PhD defense, parental leave and my teaching ultimately being the bottleneck for the team’s work!

      Reviewer #1 (Public Review):

      The authors use a combination of structural and MD simulation approaches to characterize phospholipid interactions with the pentameric ligand-gated ion channel, GLIC. By analyzing the MD simulation data using clusters of closed and open states derived previously, the authors also seek to compare lipid interactions between putative functional states. The ultimate goal of this work is to understand how lipids shape the structure and function of this channel.

      The strengths of this article include the following:

      1) The MD simulation data provide extensive sampling of lipid interactions in GLIC, and these interactions were characterized in putative closed and open states of the channel. The extensive sampling permits confident delineation of 5-6 phospholipid interaction sites per subunit. The agreement in phospholipid binding poses between structures and the all-atom MD simulations supports the utility of MD simulations to examine lipid interactions.

      2) The study presents phospholipid binding sites/poses that agree with functionally-important lipid binding sites in other pLGICs, supporting the notion that these sites are conserved. For example, the authors identify interactions of POPC at an outer leaflet intersubunit site that is specific for the open state. This result is quite interesting as phospholipids or drugs that positively modulate other pLGICs are known to occupy this site. Also, the effect of mutating W217 in the inner leaflet intersubunit site suggests that this residue, which is highly conserved in pLGICs, is an important determinant of the strength of phospholipid interactions at this site. This residue has been shown to interact with phospholipids in other pLGICs and forms the binding site of potentiating neurosteroids in the GABA(A) receptor.

      Weaknesses of this article include the following:

      1) The authors describe in detail state-dependent lipid interactions from the MD simulations; however, the functional significance of these findings is unclear. GLIC function appears to be insensitive to lipids, although this understanding is based on experiments where GLIC proteoliposomes were fused to oocyte membranes, which may not be optimal to control the lipid environment. Without functional studies of GLIC in model membranes, the lipid dependence of GLIC function is not definitively known. Therefore, it is difficult to interpret the meaning of these state-dependent lipid interactions in GLIC.

      2) It is unlikely that the bound phospholipids in the GLIC structures, which are co-purified from e. coli membranes, are POPC. Rather, these are most like PE or PG lipids. While it is difficult to accommodate mixed phospholipid membranes in all-atom MD simulations, the choice of POPC for this model, while practically convenient, seems suboptimal, especially since it is not known if PE or PG lipids modulate GLIC function. Nevertheless, it is striking that the overall binding poses of POPC from the simulations agree with those identified in the structures. It is possible that the identity of the phospholipid headgroup will have more of an impact on the strength of interactions with GLIC rather than the interaction poses (see next point).

      3) The all-atom MD simulations provide limited insight into the strength of the POPC interactions at each site, which is important to interpret the significance of these interactions. It is unlikely that the system has equilibrated within the 1.7 microseconds of simulation for each replicate preventing a meaningful assessment of the lipid interaction times. Although the authors report exchange of up to 4 POPC interacting at certain residues in M4, this may not represent binding/unbinding events (depending on how binding/interaction is defined), since the 4 Å cutoff distance for lipid interactions is relatively small. This may instead be a result of small movements of POPC in and out of this cutoff. The ability to assess interaction times may have been strengthened if the authors performed a single extended replicate up to, for example, 10-20 microseconds instead of extending multiple replicates to 1.7 microseconds.

      Reviewer #2 (Public Review):

      The authors convincingly show multiple inner and outer leaflet non-protein (lipid) densities in a cryo-EM closed state structure of GLIC, a prokaryotic homologue of canonical pentameric ligand-gated ion channels, and observe lipids in similar sites during extensive simulations at both resting and activating pH. The simulations not only corroborate structural observations, but also suggest the existence of a state-dependent lipid intersubunit site only occupied in the open state. These important findings will be of considerable interest to the ion channel community and provide new hypotheses about lipid interactions in conjunction with channel gating.

      Recommendations for the authors: please note that you control which, if any, revisions, to undertake

      In particular, a discussion of whether the timescale of the simulations permit measurements of residence or interaction times of the lipids should be addressed.

      Reviewer #1 (Recommendations for the authors):

      Comment 1.1: The authors may consider expanding the discussion about the significance of state-dependent lipid interactions. On the one hand, they emphasize state-dependent interactions of POPC with closed and open states in the outer leaflet in the results. On the other hand, they state that GLIC is insensitive to its lipid environment. What is the significance of the state-dependent interactions of POPC in GLIC, if any? It is possible that GLIC agonist responses are sensitive to phospholipids (such as PE or PG found in e. coli)? The state-dependent differences in lipid interaction identified in this study support this possibility and suggest the need to better understand the effects of phospholipids on GLIC function.

      Response 1.1: We agree with the reviewer that this is an interesting question and we have therefore extended the discussion with additional references on the functional effects on GLIC of various lipid membranes:

      p. 11 (Discussion)

      “Sampling was further simplified by performing simulations in a uniform POPC membrane. Prior experiments have been conducted to assess the sensitivity of GLIC in varying lipid environments (Labriola et al., 2013; Carswell et al., 2015; Menny et al., 2017), indicating that GLIC remains fully functional in pure POPC bilayers. In our cryo-EM experiments, the protein was recombinantly expressed from E. coli, which means that the experimental density would likely represent phosphatidylglycerol or phosphatidylethanolamine lipids. However, as the molecular identities of bound lipids could not be precisely determined, POPC lipids were built for straightforward comparison with simulation poses. While it appears that GLIC is capable of gating in a pure POPC bilayer, it remains plausible that its function could be influenced by different lipid species, especially due to the presence of multiple charged residues around the TMD/ECD interface which might interact differently with different lipid head groups. Further experiments would be needed to confirm whether the state dependence observed in simulations is also lipid-dependent. It is possible that certain types of lipids bind in one but not the other state, or that certain states are stabilized by a particular lipid type.”

      Comment 1.2: It would be helpful to state in the discussion that the co-purified lipids from GLIC structures are likely PE or PG from e. coli membranes. Nevertheless, it is interesting that the phospholipid poses from the structures generally agree with those identified from the MD simulations using PC.

      Response 1.2: Good point. We have clarified in the discussion that the native lipids in the cryo-EM structure are likely PG or PE lipids, as quoted in the preceding Response.

      Comment 1.3: The authors describe a more deeply penetrating interaction of POPC in the outer intrasubunit cleft in the open state, but this is difficult to appreciate from the images in Fig. 4B, 4E or S3B. The same is true of the deep POPC interaction at the outer intersubunit site. It may be helpful to show these densities from a different perspective to appreciate the depth of these binding poses.

      Response 1.3: We have added Figure 4 – figure supplement 1 to better show the depth of lipid binding poses, especially the ones in the outer leaflet intrasubunit cleft and at the inner intersubunit site, and cited the figure on p. 7 (Results).

      Comment 1.4: The representation of the lipid densities in Fig. 4B is not easy to interpret. First, the meaning of resting versus activating conditions and closed versus open states can be easily missed for readers who are not familiar with the author's previous study. It may be helpful to describe this (i.e. how open and closed state clusters were generated from structures determined in resting and activating conditions) in greater detail in either the figure legend, results or methods. Second, the authors state that there are differences in lipid poses between the closed and open states but not resting and activating conditions. With the exception of the intersubunit density, this is difficult to appreciate from Fig. 4B. As stated in point #3, the difference, for example, in the complementary intrasubunit site may be better appreciated with an image from a different perspective.

      Response 1.4: Acknowledged - the distinction between resting and activating conditions v.s. open and closed states can be confusing. We have tried to clarify these differences at the beginning of the results section, the methods section, and in the caption of Figure 4. Regarding differences in lipid poses between open and closed states, we agree it is difficult to appreciate from Figure 4, but here we refer the reader to Figure 4 – figure supplement 2 for an overlay between open and closed densities. Additionally, we now added Figure 1 – figure supplement 1 which provides lipid densities for all five subunits and overlays with the build cryo-EM lipids, possibly making differences easier to appreciate. Regarding images from different perspectives, we trust the new figure supplement described in Response 1.3 provides a better perspective.

      p. 3 (Results)

      “For computational quantification of lipid interactions and binding sites, we used molecular simulations of GLIC conducted under either resting or activating conditions (Bergh et al., 2021a). As described in Methods, resting conditions corresponded to neutral pH with most acidic residues deprotonated; activating conditions corresponded to acidic pH with several acidic residues protonated. Both open and closed conformations were present in both conditions, albeit with different probabilities.”

      p. 8 (Figure 4)

      “Overlaid densities for each state represent simulations conducted under resting (dark shades) or activating (light shades) conditions, which were largely superimposable within each state.”

      p. 24 (Methods)

      “We analyzed previously published MSMs of GLIC gating under both resting and activating conditions (Bergh et al., 2021a). Resting conditions corresponded to pH 7, at which GLIC is nonconductive in functional experiments, with all acidic residues modeled as deprotonated. Activating conditions corresponded to pH 4.6, at which GLIC is conductive and has been crystallized in an open state (Bocquet et al., 2009). These conditions were modeled by protonating a group of acidic residues (E26, E35, E67, E75, E82, D86, D88, E177, E243; H277 doubly protonated) as previously described (Nury et al., 2011).”

      Comment 1.5: The new closed GLIC structure was obtained by merging multiple datasets. What were the conditions of the datasets used? Was it taken from samples in resting or also activating conditions?

      Response 1.5: We have updated the Results, Discussion, and Methods to clarify this important point, in particular by merging datasets and rerunning the classification:

      p. 3 (Results)

      “In our cryo-EM work, a new GLIC reconstruction was generated by merging previously reported datasets collected at pH 7, 5, and 3 (Rovšnik et al., 2021). The predominant class from the merged data corresponded to an apparently closed channel at an overall resolution of 2.9 Å, the highest resolution yet reported for GLIC in this state (Figure 1 – figure supplement 2, Table 1).”

      p. 11 (Discussion)

      “Interestingly, the occupational densities varied remarkably little between resting and activating conditions (Figure 1 – figure supplement 1), indicating state- rather than pH- dependence in lipid interactions, also further justifying the approach of merging closed- state GLIC cryo-EM datasets collected at different pH conditions to resolve lipids.”

      p. 14 (Methods)

      “After overnight thrombin digestion, GLIC was isolated from its fusion partner by size exclusion in buffer B at pH 7, or in buffer B with citrate at pH 5 or 3 substituted for Tris. The purified protein was concentrated to 3–5 mg/mL by centrifugation. [...] Data from three different grids, at pH 7, 5, and 3, were merged and processed together.”

      Comment 1.6: In Fig. 3D, do the spheres represent the double bond? If so, please state in the legend

      Response 1.6: We have clarified in the legend of Figure 3D that the yellow spheres on the lipid tails represent a double bond.

      Comment 1.7: In Fig. 3E, what is the scale of the color representation?

      Response 1.7: We have clarified in the legend of Figure 3E that colors span 0 (white) to 137015 contacts (dark red).

      Reviewer #2 (Recommendations For The Authors):

      Comment 2.1: I'm not sure I fully understand how the final lipids were modeled (built). Fig. 1 caption suggests they may have been manually built? I understand that the idea was to place them in the overlap of simulation densities and structure densities, but can the authors please clarify if there were any quantifiable conditions that were employed during this process or if this was entirely manual placement in a pose that looked good? Regardless, it would be helpful to see an overlay of the built lipids with both the cryo and simulation densities (e.g., overly of Fig. 1F/H and G/H) to better visualize how the final built lipids compare.

      Response 2.1: We thank the reviewer for pointing out unclarities regarding our methods. We have extended the methods section to clarify how the lipids were manually built in the cryo-EM structure. We have also added Figure 1 – figure supplement 1 showing overlays of the computational densities and built cryo-EM lipids.

      p. 15 (Methods)

      “Lipids were manually built in COOT by importing a canonical SMILES format of POPC (Kim et al., 2021) and adjusting it individually into the cryo-EM density in each of the sites associated with a single subunit, based in part on visual inspection of lipid densities from simulations, as described above. After building, 5-fold symmetry was applied to generate lipids at the same sites in the remaining four subunits.”

      Comment 2.2: Regarding the state-dependent lipid entry to the outer leaflet intersubunit site associated with channel opening, if the authors could include a movie depicting this process that would be great. The current short explanation does not do this justice. Also, what were the dynamics of this process? Beyond the correlation between site occupancy and the pore being open, how did the timing of lipid entry/exit and pore opening/closing correlate?

      Response 2.2: The point regarding the timing of state-dependent lipid binding at the subunit interface and pore opening is indeed an interesting one. We have added Figure 4 – figure supplement 3D showing that the state-dependent P250 lipid interaction precedes pore opening, as quantified by pore hydration levels, indicating a potential role in gating. The interaction between lipid binding and conformational change of the protein is also depicted in the newly added Figure 4 - video supplement 1, which we hope will be able to better communicate the conclusions regarding state-dependent interactions. We have also expanded the results and discussion to better explain these results:

      p. 9 (Results)

      “The lipid head made particularly close contacts with residue P250 on the M2-M3 loop, which undergoes substantial conformational change away from the pore upon channel opening, along with outer-leaflet regions of M1–M3 (Figure 4E, Figure 4—figure Supplement 3A,B,C, Figure 4—video 1). These conformational changes were accompanied by a flip of M1 residue F195, which blocked the site in the closed state but rotated inward to allow closer lipid interactions in the open state (Figure 4—figure Supplement 3C, Figure 4—video 1). Indeed, P250 was predominantly located within 3 Å of the nearest lipid atom in open- but not closed-state frames (Figure 4F). Despite being restricted to the open state, interactions with P250 were among the longest duration in all simulations (Figure 2C) and as these binding events preceded pore opening, it is plausible to infer a role for this state-dependent lipid interaction in the gating process (Figure 4 – figure supplement 3D).”

      p. 12 (Discussion)

      “The state-dependent binding event at this site preceded pore opening in MSMs, where lipid binding coincided with crossing a smaller energy barrier between closed and intermediate states, followed by pore opening at the main energy barrier between intermediate and open states (Figure 4 – figure supplement 3D). Further, since the P250- lipid interaction was characterized by relatively long residence times (Figure 2), it is possible this lipid interaction has a role to play in GLIC gating.”

      Comment 2.3: Although the interaction times are helpful, I didn't get a great sense of how mobile the lipids are during the simulations. Can the authors discuss this a bit more. For example, are interaction times dominated by lipids that jiggle a bit away from a residue and then back again, vs how often are lipids exchanging with other lipids initially further away from the protein?

      Response 2.3: We have now added various measures of lipid diffusion, both for initially interacting lipids and for bulk lipids, which are summarized in the new Figure 2 – figure supplement 1. We have further addressed the question of simulation timescales in Results, Discussion, and Methods. These numbers highlight that it is possible for lipids several nanometers away from the protein surface to exchange with lipids of the first lipid shell.

      p. 3,6 (Results)

      “Lateral lipid diffusion coefficients were estimated to 1.47 nm2/µs for bulk lipids and 0.68 nm2/µs for lipids of the first lipid shell (Figure 2 – figure supplement 1A), which is relatively slow compared to the timescales of each trajectory (1.7 µs). However, multiple residues throughout the M1, M3, and M4 helices exchanged contacts with 2-4 different lipid molecules in individual simulations (Figure 2C). Furthermore, 1.7-µs root mean square displacement of lipids originally in the first lipid shell was 2.15 nm, and 3.16 nm in the bulk bilayer, indicating such exchanges are not limited to nearby lipids (Figure 2 – figure supplement 1B). Thus, exchange events and diffusion estimates indicate that the duration of lipid contacts observed in this work can be at least partly attributed to interaction stabilities and not solely to sampling limitations.”

      p. 11 (Discussion)

      “Indeed, the unrestrained atomistic MD simulations studied here were not expected to capture the maximal duration of stable contacts, as indicated by some interaction times approaching the full 1.7-µs trajectory (Figure 2}). Nevertheless, simulations were of sufficient length to sample exchange of up to four lipids, particularly around the M4 helix. Calculation of lipid lateral diffusion coefficients resulted in average displacements at the end of simulations of 2.15 nm for lipids initially interacting with the protein surface, roughly corresponding to lipids diffusing out to the 4th lipid shell. Diffusion of bulk lipids was faster, allowing lipids originally 3.16 nm away from the protein surface to ingress the first lipid shell. This observation underscores the potential for lipid exchange events even among lipids initially distant from the protein surface. Of course, duration of exceptionally stable interactions, such as those involving T274 (Figure 2C), inevitably remain bounded by the length of our simulations. Still, diffusion metrics, supported by robust statistical analysis encompassing diverse starting conditions (500 trajectories), enable confident estimation of relative interaction times.“

      p. 13 (Methods)

      “Time-based measures of protein-lipid interactions, such as mean duration times and exchange of interactions, were calculated for the 100 x 1.7 µs-long simulations using prolintpy (Sejdiu and Tieleman, 2021) with a 4 Å interaction cutoff. Analysis of lateral lipid diffusion in individual simulations was carried out for two disjoint sets of lipids: the first lipid shell defined as lipids with any part within 4 Å of the protein surface (~90 lipids), and bulk lipids consisting of all other lipids (~280 lipids). Mean square displacements of each lipid set were calculated using GROMACS 2021.5 (Abraham et al., 2015b) with contributions from the protein center of mass removed. Diffusion coefficients for each set, DA, were calculated using the Einstein relation (Equation 1) by estimating the slope of the linear curve fit to the data.

      where ri(t) is the coordinate of the center of mass of lipid i of set A at time t and DA is the self-diffusion coefficient.”

      Comment 2.4: How symmetric or asymmetric are the cryo and simulation densities across subunits and was there subunit asymmetry in the final build lipids? I could not tell from any of the figures beyond the casual observation that they maybe look somewhat similar in Fig. 1?

      Response 2.4: We thank the reviewer for this useful remark. We have clarified in the methods that the cryo-EM lipids were built in C5-symmetry, and thus the positions are symmetric. The computational densities were calculated independently for each subunit and are thus not necessarily symmetric. We have added Figure 1 – figure supplement 1 showing densities for all five subunits, also serving as an indication of convergence of the results.

      p. 3 (Results) “Although the stochastic nature of simulations resulted in nonidentical lipid densities associated with the five GLIC subunits, patterns of lipid association were notably symmetric (Figure 1 – figure supplement 1).”

      p. 14-15 (Methods)

      “A smaller subset of particles was used to generate an initial model. All subsequent processing steps were done using 5-fold symmetry. […] A monomer of that model was fit to the reconstructed density and 5-fold symmetry was applied with PHENIX 1.19.2-4158 through NCS restraints detected from the reconstructed cryo-EM map, to generate a complete channel. […] After building, 5-fold symmetry was applied to generate lipids at the same sites in the remaining four subunits.”

      Minor comments:

      Comment 2.5: Fig. 1 is probably not easy to follow for the general reader and the caption is very brief. I suggest adding an additional explanation to the caption and/or additional annotations to the figure to help a general reader step through this.

      Response 2.5: We have expanded the caption of Figure 1 and clarified the meanings of colors, labels, and annotations.

      Comment 2.6: Fig. 1B - Caption is confusing. I would not call the state separation lines outlines as they are not closed loops. Also, I see red/orange and two shades of blue whereas the caption mentions orange and blue only. The caption should also explicitly say what the black lines are (other cluster separations).

      Response 2.6: We have edited the caption to better describe colors, annotations, and the meaning of the data:

      p. 4 (Figure 1)

      “(B) Markov state models were used to cluster simulations conducted under resting (R) or activating (A) conditions into five states, including closed (left of the light or dark orange lines) and open (right of the light or dark blue lines). Black lines mark edges of other state clusters derived from MSM eigenvectors. Experimental structures are highlighted as white circles.”

      Comment 2.7: Fig. 3F caption appears to conflict with data where interaction with W217A appears longer than W217. I think the authors want to suggest here that W217A reduces contact time with T274 as stated in the main text.

      Response 2.7: We have clarified in this legend that “Mutation of residue W217, lining this pocket, reveals shortened interactions at the T274 binding site” (p. 6, Figure 3).

      Comment 2.8: Ref 25 and 26 are the same.

      Response 2.8: Apologies; this mistake has been corrected.

    1. Author Response

      The following is the authors’ response to the original reviews.

      Reviewer #1 (Public Review):

      The paper from Hsu and co-workers describes a new automated method for analyzing the cell wall peptidoglycan composition of bacteria using liquid chromatography and mass spectrometry (LC/MS) combined with newly developed analysis software. The work has great potential for determining the composition of bacterial cell walls from diverse bacteria in high-throughput, allowing new connections between cell wall structure and other important biological functions like cell morphology or host-microbe interactions to be discovered. In general, I find the paper to be well written and the methodology described to be useful for the field. However, there are areas where the details of the workflow could be clarified. I also think the claims connecting cell wall structure and stiffness of the cell surface are relatively weak. The text for this topic would benefit from a more thorough discussion of the weak points of the argument and a toning down of the conclusions drawn to make them more realistic.

      Thank you for your thorough and insightful review of our manuscript. We greatly appreciate your positive and constructive feedbacks on our methodology. We have carefully reviewed your comments and have responded to each point as follows:

      Specific points:

      1) It was unclear to me from reading the paper whether or not prior knowledge of the peptidoglycan structure of an organism is required to build the "DBuilder" database for muropeptides. Based on the text as written, I was left wondering whether bacterial samples of unknown cell wall composition could be analyzed with the methods described, or whether some preliminary characterization of the composition is needed before the high-throughput analysis can be performed. The paper would be significantly improved if this point were explicitly addressed in the main text. We apologize for not making it clearer. The prior knowledge of the peptidoglycan structure of an organism is indeed required to build the “DBuilder” database to accurately identify muropeptides; otherwise, the false discovery rate might increase. While peptidoglycan structures of certain organisms might not have been extensively studied, users still remain the flexibility to adapt the muropeptide compositions based on their study, referencing closely related species for database construction. We have addressed this aspect in the main text to ensure a clearer understanding.

      “(Section HAMA platform: a High-throughput Automated Muropeptide Analysis for Identification of PGN Fragments) …(i) DBuilder... Based on their known (or putative) PGN structures, all possible combinations of GlcNAc, MurNAc and peptide were input into DBuilder to generate a comprehensive database that contains monomeric, dimeric, and trimeric muropeptides (Figure 1b)."

      2) The potential connection between the structure of different cell walls from bifidobacteria and cell stiffness is pretty weak. The cells analyzed are from different strains such that there are many possible reasons for the change in physical measurements made by AFM. I think this point needs to be explicitly addressed in the main text. Given the many possible explanations for the observed measurement differences (lines 445-448, for example), the authors could remove this portion of the paper entirely. Conclusions relating cell wall composition to stiffness would be best drawn from a single strain of bacteria genetically modified to have an altered content of 3-3 crosslinks.

      We understand your concern regarding the weak connection between cell wall structure and cell stiffness. We will make a clear and explicit statement in the main text to acknowledge that the cells analyzed are derived from different strains, introducing the possibility of various factors influencing the observed changes in physical measurements as determined by AFM. Furthermore, we greatly appreciate your suggestion to consider genetically modified strains to investigate the role of cross-bridge length in determining cell envelope stiffness. In this regard, we are in the process of developing a CRISPR/Cas genome editing toolbox for Bifidobacterium longum, and we plan on this avenue of investigation for future work.

      Reviewer #2 (Public Review):

      The authors introduce "HAMA", a new automated pipeline for architectural analysis of the bacterial cell wall. Using MS/MS fragmentation and a computational pipeline, they validate the approach using well-characterized model organisms and then apply the platform to elucidate the PG architecture of several members of the human gut microbiota. They discover differences in the length of peptide crossbridges between two species of the genus Bifidobacterium and then show that these species also differ in cell envelope stiffness, resulting in the conclusion that crossbridge length determines stiffness.

      We appreciate your thoughtful review of our manuscript and your recognition of the potential significance of our work in elucidating the poorly characterized peptidoglycan (PGN) architecture of the human gut microbiota.

      The pipeline is solid and revealing the poorly characterized PG architecture of the human gut microbiota is worthwhile and significant. However, it is unclear if or how their pipeline is superior to other existing techniques - PG architecture analysis is routinely done by many other labs; the only difference here seems to be that the authors chose gut microbes to interrogate.

      We apologize if this could have been clearer. The HAMA platform stands apart from other pipelines by utilizing automatic analysis of LC-MS/MS data to identify muropeptides. In contrast, most of the routine PGN architecture analyses often use LC-UV/Vis or LC-MS platform, where only the automatic analyzing PGFinder software is supported. To our best knowledge, a comparable pipeline on automatically analyzing LC-MS/MS data was reported by Bern et al., which they used commercial Byonic software with an in-house FASTA database and specific glycan modifications. They achieved accurate and sensitive identification on monomer muropeptides, but struggled with cross-linked muropeptides due to the limitations of the Byonic software. We believe that our pipeline introducing the automatic and comprehensive analysis on muropeptide identification (particularly for Gram-positive bacterial peptidoglycans) would be a valuable addition to the field. To enhance clarity, we have adjusted the context as follows:

      (Introduction) … Although they both demonstrated great success in identifying muropeptide monomers, the accurate identification of muropeptide multimers and other various bacterial PGN structures still remains unresolved. This is because deciphering the compositions requires MS/MS fragmentation, but it is still challenging to automatically annotate MS/MS spectra from these complex muropeptide structures."

      I do not agree with their conclusions about the correlation between crossbridge length and cell envelope stiffness. These experiments are done on two different species of bacteria and their experimental setup therefore does not allow them to isolate crossbridge length as the only differential property that can influence stiffness. These two species likely also differ in other ways that could modulate stiffness, e.g. turgor pressure, overall PG architecture (not just crossbridge length), membrane properties, teichoic acid composition etc.

      Regarding the conclusions drawn about the correlation between cross-bridge length and cell envelope stiffness, we understand your point and appreciate your feedback. We revisit this section of our manuscript and tone down the conclusions drawn from this aspect of the study. We also recognize the importance of considering other potential factors that could influence stiffness, as you mentioned above. In light of this, we mentioned the need for further investigations, potentially involving genetically modified strains, in the main text to isolate and accurately determine the impact of bridge length on cell envelope stiffness.

      Reviewer #1 (Recommendations For The Authors):

      Minor points:

      1) One thing to consider would be testing the robustness of the analysis pipeline with one the well-characterized bacteria studied, but genetically modifying them to change the cell wall composition in predictable ways. Does the analysis pipeline detect the expected changes?

      We appreciate the reviewer's suggestion and would like to provide a clear response. Regarding to testing the pipeline with genetically modified strains, our lab previously worked on genetically modified S. maltophilia (KJΔmrdA).1 Inactivation of mrdA turned out the increasing level of N-acetylglucosaminyl-1,6-anhydro-N-acetylmuramyl-L-alanyl-D-glutamyl-meso-diamnopimelic acid-D-alanine (GlcNAc-anhMurNAc tetrapeptide) in muropeptide profiles, which is the critical activator ligands for mutant strain ΔmrdA-mediated β-lactamase expression. In this case, our platform could provide rapid PGN analysis for verifying the expected change of muropeptide profiles (see Author response image 1). Besides, if the predictable changes involve genetically modifications on interpeptide bridges within the PGN structure, for example, the femA/B genes of S. aureus, which are encoded for the synthesis of interpeptide bridges,2 our current HAMA pipeline is capable of detecting these anticipated changes. However, if the genetically modifications involve the introduce of novel components to PGN structures, then it would need to create a dedicated database specific to the genetically modified strain.

      Author response image 1.

      2) Line 368: products catalyzed > products formed

      The sentence has been revised.

      “(Section Inferring PGN Cross-linking Types Based on Identified PGN Fragments) …Based on the muropeptide compositional analysis mentioned above, we found high abundances of M3/M3b monomer and D34 dimer in the PGNs of E. faecalis, E. faecium, L. acidophilus, B. breve, B. longum, and A. muciniphila, which may be the PGN products formed by Ldts.”

      3) Lines 400-402: Is it possible the effect is related to porosity, not "hardness".

      Thank you for the suggestion. The possibility of the slower hydrolysis rate of purified PGN in B. breve being related to porosity is indeed noteworthy. While this could be a potential factor, we would like to acknowledge the limited existing literature that directly addresses the relation between PGN architecture and porosity. It is plausible that current methods available for assessing cell wall porosity may have certain limitations, contributing to the scarcity of relevant studies. In light of this, we would like to propose a speculative explanation for the observed effect. It is plausible that the tighter PGN architecture resulting from shorter interpeptide bridges in B. breve could contribute to its harder texture. This speculation is grounded in the concept that a more compact PGN structure might lead to increased stiffness, aligning with our observations of higher cell stiffness in B. breve.

      4) Lines 403-408: See point #2 above.

      Thank you for the suggestion. We have explicitly addressed this point in the main text:

      “(Section Exploring the Bridge Length-dependent Cell Envelope Stiffness in B. longum and B. breve) … Taken all together, we speculate that a tight peptidoglycan network woven by shorter interpeptide bridges or 3-3 cross-linkages could give bacteria stiffer cell walls. However, it is important to note that cell stiffness is a mechanical property that also depends on PGN thickness, overall architecture, and turgor pressure. These parameters may vary among different bacterial strains. Hence, carefully controlled, genetically engineered strains with similar characteristics will be needed to dissect the role of cross-bridge length in cell envelope stiffness.”

      5) Lines 428-429: It is not clear to me how mapping the cell wall architecture provides structural information about the synthetic system. It is also not clear how antibiotic resistance can be inferred. More detail is needed here to flesh out these points.

      Thank you for the suggestion. To provide further clarity on these important aspects, the context in the manuscript has been revised.

      “(Discussion) …Importantly, our HAMA platform provides a powerful tool for mapping peptidoglycan architecture, giving structural information on the PGN biosynthesis system. This involves the ability to infer possible PGN cross-linkages based on the type of PGN fragments obtained from hydrolysis. For instance, the identification of 3-3 cross-linkage formed by L,D-transpeptidases (Ldts) is of particular significance. Unlike 4-3 cross-linkages, the 3-3 cross-linkage is resistant to inhibition by β-Lactam antibiotics, a class of antibiotics that commonly targets bacterial cell wall synthesis through interference with 4-3 cross-linkages. Therefore, by elucidating the specific cross-linkage types within the peptidoglycan architecture, our approach offers insights into antibiotic resistance mechanisms.”

      6) Line 478: "maneuvers are proposed for" > "work is needed to generate". Also, delete "innovative". Also "in silico" > "in silico-based".

      The sentence has been revised.

      “(Discussion) …To achieve a more comprehensive identification of muropeptides, future work is needed to generate an expanded database, in silico-based fragmentation patterns, and improved MS/MS spectra acquisition.”

      7) Line 485: "Its" > "It has potential"

      The sentence has been revised.

      “(Discussion) …It has potential applications in identifying activation ligands for antimicrobial resistance studies, characterizing key motifs recognized by pattern recognition receptors for host-microbiota immuno-interaction research, and mapping peptidoglycan in cell wall architecture studies.”

      8) Figure 1 legend: Define Gb and Pb.

      Gb and Pb are the abbreviations of glycosidic bonds and peptide bonds. We have revised the Figure legend 1 as follow:

      “(Figure legend 1) …(b) DBuilder constructs a muropeptide database containing monomers, dimers, and trimers with two types of linkage: glycosidic bonds (Gb) and peptide bonds (Pb).”

      9) Figure 2: It is hard to see what is going on in panel a and c with all the labels. Consider removing them and showing a zoomed inset with labels in addition to ab unlabeled full chromatogram.

      We apologize for not making this clearer. The panel a and c in Figure 2 were directly generated by the Analyzer as a software screenshot of the peak annotations on chromatogram. Our intention was to present a comprehensive PGN mapping (approximately 70% of the peak area was assigned to muropeptide signals) using this platform. We understand the label density might affect clarity, so we have added the output tables of the whole muropeptide identifications as source data (Table 1–Source Data 1&2). Additionally, we have uploaded the Analyzer output files (see Additional Files), which can be better visualized in the Viewer program, and it also allows users zoom in for detailed labeling information.

      10) Figure 3: It is worth pointing out what features of the MS/MS fingerprints are helping to discriminate between species.

      Thank you for the suggestion. We have revised Figure 3 and the legend as follow:

      “(Figure legend 3) …The sequence of each isomer was determined using in silico MS/MS fragmentation matching, with the identified sequence having the highest matching score. The key MS/MS fragments that discriminate between two isomers are labeled in bold brown.”

      Author response image 2.

      11) Figure 4 and 5 legend: Can you condense the long descriptions of the abbreviations - or at least only refer to them once?

      Certainly, to enhance clarity and conciseness in the figure legends, we have revised Figure legend 5 as follow:

      “(Figure legend 5) …(b) Heatmap displaying …. Symbols: M, monomer; D, dimer; T, trimer (numbers indicate amino acids in stem peptides). Description of symbol abbreviations as in Figure legend 4, with the addition of "Glycan-T" representing trimers linked by glycosidic bonds.”

      Reviewer #2 (Recommendations For The Authors):

      1. Please read the manuscript carefully for spelling errors.

      We appreciate your careful review of our manuscript. We have thoroughly rechecked the entire manuscript for spelling errors and have made the necessary corrections to ensure the accuracy and quality of the text.

      1. Line 46 - "multilayered" is likely only true for Gram-positive bacteria.

      We thank reviewer #2 for bringing up this concern. Indeed, Gram-negative bacteria mostly possess single layer of peptidoglycan, but could be up to three layers in some part of the cell surface.3, 4 In order to reduce the confusion, we have rewritten the context as follow: “(Introduction) …PGN is a net-like polymeric structure composed of various muropeptide molecules, with their glycans linearly conjugated and short peptide chains cross-linked through transpeptidation.”

      1. Methods section: It seems like pellets from a 10 mL bacterial culture were ultimately suspended in 1.5 L (750 mL water + 750 mL tris) - why such a large volume? And how were PG fragments subsequently washed (centrifugation? There is no information on this in the Methods).

      We apologize for the mislabeling on the units. The accurate volume should be “1.5 mL (750 µL water + 750 µL tris)”. We have updated the correct volume in the Methods section (lines 99-100). For the washing process of purified PGN, we added 1 mL water, centrifuged at 10,000 rpm for 5 minutes, and removed supernatant. This information has added to the Methods section (lines 95-98).

      1. Line 183 - why were 6 modifications chose as the cutoff? Please make rationale more clear.

      We thank reviewer #2 for the comments. We set the maximum modification number of 6 in the assumption of one modification on each sugar of a trimeric muropeptide. A lower cutoff could effectively limit the identification of muropeptides with unlikely numbers of modifications, whereas a higher cutoff could allow for having multiple modifications on a muropeptide. In our hand, muropeptide modifications of E. coli are mostly N-deacetyl-MurNAc and anhydro-MurNAc, and modifications of gut microbes used here are mostly N-deacetyl-GlcNAc, anhydro-MurNAc, O-acetyl-MurNAc, loss of GlcNAc, and amidated iso-Glu. While we recommend starting data analysis with the cutoff of 6 modifications, users are free to adjust this based on their studies.

      1. Line 339 - define donor vs. acceptor here (can be added in parentheses after explaining the relevant chemical reactions further above in the text)

      Thank you for the suggestion. To provide greater clarity regarding the roles of the donor and acceptor substrates in the transpeptidation process, we have revised the content in the manuscript as follows:

      “(Section Inferring PGN Cross-linking Types Based on Identified PGN Fragments) …In general, there are two types of PGN cross-linkage…. Transpeptidation involves two stem peptides which function as acyl donor and acceptor substrates, respectively. As the enzyme names imply, the donor substrates that Ddts and Ldts bind to are terminated as D,D-stereocenters and L,D-stereocenters, which structurally means pentapeptides and tetrapeptides. During D,D-transpeptidation, Ddts recognize D-Ala4-D-Ala5 of the donor stem (pentapeptide) and remove the terminal D-Ala5 residue, forming an intermediate. The intermediate then cross-links the NH2 group in the third position of the neighboring acceptor stem, forming a 4-3 cross-link.”

      1. Line 366 following - can you calculate % crosslinks based on these numbers? What does "high abundance" of 3,3 crosslinks mean in this context? Is this the majority of PG?

      Thank you for your questions. Calculating the percentage of crosslinks based on the muropeptide compositional numbers is a valid consideration. However, it's important to note that the muropeptides we analyzed were hydrolyzed by mutanolysin, and as such, deriving an accurate % crosslink value from these data might not provide a true representation of the crosslinking percentage within the PGN network. For a more precise determination of % crosslinks, methods such as solid-phase NMR on purified peptidoglycan would be required. Our research provides insights into the characterization of PGN fragments and allows us to infer potential PGN cross-linkage types and the enzymes involved based on the dominant muropeptide fragments. Regarding the phrase "high abundance" in the context, it indicates that the M3b/M4b monomer and D34 dimer muropeptides represent a significant portion of the hydrolysis products. These muropeptides are major constituents within the PGN fragments obtained from the enzymatic hydrolysis.

      1. Line 375 - I am not sure PG is a meaningful diffusion barrier for drugs and signaling molecules, give that even larger proteins can apparently diffuse through the pores.

      Thank you for raising this point. Peptidoglycan indeed possesses relatively wide pores that allow for the diffusion of larger molecules, including proteins.5 Research has provided a rough estimate of the porosity of the PGN meshwork, suggesting that it allows for the diffusion of proteins with a maximum molecular mass of around 50 kDa.6 Considering this, we acknowledge that PGN may not serve as a significant diffusion barrier for drugs and signaling molecules. The porosity of the PGN scaffold, which is defined by the degree of cross-linking, plays a role in influencing the transport of molecules to the cell membrane. Thus, while PGN may not serve as a strict diffusion barrier, its structural characteristics still impact bacterial cell mechanics and interactions. We have revised the manuscript to reflect this understanding:

      “(Section Exploring the Bridge Length-dependent Cell Envelope Stiffness in B. longum and B. breve) …The porosity of the PGN scaffold, defined by the degree of cross-linking, influences the transport of larger molecules such as proteins. Therefore, modifications to PGN structure are anticipated to significantly affect bacterial cell mechanics and interactions.”

      1. Line 400 - what does "slower hydrolysis rate" refer to, is this chemical hydrolysis or enzymatic (autolysins?). also, I am not sure hydrolysis rate of either modality allows for solid conclusions about how hard (line 402) the PG is.

      Thank you for your comments. The hydrolysis rate here refers to the enzymatic hydrolysis, specifically the mutanolysin cleaving the β-N-acetylmuramyl-(1,4)-N-acetylglucosamine linkage. Indeed, there is no direct correlation between the hydrolysis rate and the hardness of PGN architecture, although the structure rigidity is a key determinant in protein digestion.7 Considering the enzymatic hydrolysis rate depending on the accessibility of the substrate to the enzyme, we proposed that the tighter PGN architecture could also lead to a slower hydrolysis rate. This speculation aligns with our observations of higher cell stiffness or more compact PGN structure of B. breve and its slower hydrolysis rate. We understand this is indirect proof, so the revised sentence now reads:

      “(Section Exploring the Bridge Length-dependent Cell Envelope Stiffness in B. longum and B. breve) …Furthermore, B. breve also showed a slower enzymatic hydrolysis rate in purified PGNs, implying that the cell wall structure of B. breve is characterized by a compact PGN architecture.”

      1. Line 424 - I am not convinced this pipeline can detect PG architectures that other pipelines cannot; likely, the difference between previous analyses and theirs is due to different growth conditions (3,3 crosslink formation is often modulated by environmental factors/growth stage). In the next sentence, it sounds like mutanolysin treatment is a novelty in PG analysis (which it is not).

      We apologize if this could have been clearer and we have revised the paragraph to describe our study more accurately. We agree that different growth conditions could influence PGN architecture and other pipelines could manually identify the PGN architectures or automatically identify them if they are not too complex. Our original intention was to highlight the ability of the HAMA program to automatically identify unreported PGN structure. Here are the revised sentences:

      “(Discussion) …We speculate that this finding may be influenced by the comprehensive mass spectrometric approaches we employed or by variations in growth conditions. Moreover, we utilized the well-established enzymatic method involving mutanolysin to cleave the β-N-acetylmuramyl-(1,4)-N-acetylglucosamine linkage, which preserves the original peptide linkage in intact PGN subunits.”

      1. Line 440- 442: As outlined in more detail above: I don't think you can conclude something about the relationship between bridge length and envelope stiffness based on these data. Thank you for your valuable feedback. We agree that our data may not definitively support the direct conclusion about the relationship between bridge length and envelope stiffness in Bifidobacterium species. Instead, we will rephrase this section to accurately present the observed correlations without overgeneralizing:

      “(Discussion) … Notably, our study suggested a potential correlation between the cell stiffness and the compactness of bacterial cell walls in Bifidobacterium species (Figure 5). B. longum, which predominantly harbors tetrapeptide bridges (Ser-Ala-Thr-Ala), exhibits a trend towards lower stiffness, whereas B. breve, characterized by PGN cross-linked with monopeptide bridges (Gly), demonstrates a trend towards higher stiffness. These findings suggested that it may be correlated between the increased rigidity and the more compact PGN architecture built by shorter cross-linked bridges.”

      References: 1. Huang, Y.-W.; Wang, Y.; Lin, Y.; Lin, C.; Lin, Y.-T.; Hsu, C.-C.; Yang, T.-C., Impacts of Penicillin Binding Protein 2 Inactivation on β-Lactamase Expression and Muropeptide Profile in Stenotrophomonas maltophilia. mSystems 2017, 2 (4), 00077-00017.

      1. Jarick, M.; Bertsche, U.; Stahl, M.; Schultz, D.; Methling, K.; Lalk, M.; Stigloher, C.; Steger, M.; Schlosser, A.; Ohlsen, K., The serine/threonine kinase Stk and the phosphatase Stp regulate cell wall synthesis in Staphylococcus aureus. Sci. Rep. 2018, 8 (1), 13693.

      2. Labischinski, H.; Goodell, E. W.; Goodell, A.; Hochberg, M. L., Direct proof of a "more-than-single-layered" peptidoglycan architecture of Escherichia coli W7: a neutron small-angle scattering study. J. Bacteriol. 1991, 173 (2), 751-756.

      3. Rohde, M., The Gram-Positive Bacterial Cell Wall. Microbiol. Spectr. 2019, 7 (3), gpp3-0044-2018.

      4. Vollmer, W.; Höltje, J. V., The architecture of the murein (peptidoglycan) in gram-negative bacteria: vertical scaffold or horizontal layer(s)? J. Bacteriol. 2004, 186 (18), 5978-5987.

      5. Vollmer, W.; Blanot, D.; De Pedro, M. A., Peptidoglycan structure and architecture. FEMS Microbiol. Rev. 2008, 32 (2), 149-167.

      6. Li, Q.; Zhao, D.; Liu, H.; Zhang, M.; Jiang, S.; Xu, X.; Zhou, G.; Li, C., "Rigid" structure is a key determinant for the low digestibility of myoglobin. Food Chem.: X 2020, 7, 100094.

    1. Author response:

      The following is the authors’ response to the original reviews

      Public Reviews:

      Reviewer #1 (Public review):

      Summary:

      Chen et al. identified the role of endocardial id2b expression in cardiac contraction and valve formation through pharmaceutical, genetic, electrophysiology, calcium imaging, and echocardiography analyses. CRISPR/Cas9 generated id2b mutants demonstrated defective AV valve formation, excitation-contraction coupling, reduced endocardial cell proliferation in AV valve, retrograde blood flow, and lethal effects.

      Strengths:

      Their methods, data and analyses broadly support their claims.

      Weaknesses:

      The molecular mechanism is somewhat preliminary.

      We thank the reviewer for the positive assessment of our work. A detailed point-by-point response has been incorporated in the response to “Recommendations for the authors” section.

      Reviewer #2 (Public review):

      Summary:

      Biomechanical forces, such as blood flow, are crucial for organ formation, including heart development. This study by Shuo Chen et al. aims to understand how cardiac cells respond to these forces. They used zebrafish as a model organism due to its unique strengths, such as the ability to survive without heartbeats, and conducted transcriptomic analysis on hearts with impaired contractility. They thereby identified id2b as a gene regulated by blood flow and is crucial for proper heart development, in particular, for the regulation of myocardial contractility and valve formation. Using both in situ hybridization and transgenic fish they showed that id2b is specifically expressed in the endocardium, and its expression is affected by both pharmacological and genetic perturbations of contraction. They further generated a null mutant of id2b to show that loss of id2b results in heart malformation and early lethality in zebrafish. Atrioventricular (AV) and excitation-contraction coupling were also impaired in id2b mutants. Mechanistically, they demonstrate that Id2b interacts with the transcription factor Tcf3b to restrict its activity. When id2b is deleted, the repressor activity of Tcf3b is enhanced, leading to suppression of the expression of nrg1 (neuregulin 1), a key factor for heart development. Importantly, injecting tcf3b morpholino into id2b-/- embryos partially restores the reduced heart rate. Moreover, treatment of zebrafish embryos with the Erbb2 inhibitor AG1478 results in decreased heart rate, in line with a model in which Id2b modulates heart development via the Nrg1/Erbb2 axis. The research identifies id2b as a biomechanical signaling-sensitive gene in endocardial cells that mediates communication between the endocardium and myocardium, which is essential for heart morphogenesis and function.

      Strengths:

      The study provides novel insights into the molecular mechanisms by which biomechanical forces influence heart development and highlights the importance of id2b in this process.

      Weaknesses:

      The claims are in general well supported by experimental evidence, but the following aspects may benefit from further investigation:

      (1) In Figure 1C, the heatmap demonstrates the up-regulated and down-regulated genes upon tricane-induced cardiac arrest. Aside from the down-regulation of id2b expression, it was also evident that id2a expression was up-regulated. As a predicted paralog of id2b, it would be interesting to see whether the up-regulation of id2a in response to tricane treatment was a compensatory response to the down-regulation of id2b expression.

      We thank the reviewer for the comment. As suggested, we performed qRT-PCR analysis to assess id2a expression in tricaine-treated heart. Our results demonstrate a significant upregulation of id2a following the inhibition of cardiac contraction, suggesting a potential compensatory response to the decreased id2b. These new results have been incorporated into the revised manuscript (Figure 1D).

      (2) The study mentioned that id2b is tightly regulated by the flow-sensitive primary cilia-klf2 signaling axis; however aside from showing the reduced expression of id2b in klf2a and klf2b mutants, there was no further evidence to solidify the functional link between id2b and klf2. It would therefore be ideal, in the present study, to demonstrate how Klf2, which is a transcriptional regulator, transduces biomechanical stimuli to Id2b.

      We have examined the expression levels of id2b in both klf2a and klf2b mutants. The whole mount in situ results clearly demonstrate a decrease in id2b signal in both mutants (Figure 3E). As noted by the reviewer, klf2 is a transcriptional regulator, suggesting that the regulation of id2b may occur at the transcriptional level. However, dissecting the molecular mechanisms underlying the crosstalk between klf2 and id2b is beyond the scope of the present study.

      (3) The authors showed the physical interaction between ectopically expressed FLAG-Id2b and HA-Tcf3b in HEK293T cells. Although the constructs being expressed are of zebrafish origin, it would be nice to show in vivo that the two proteins interact.

      We thank the reviewer for this insightful comment. As suggested, we synthesized Flag-id2b and HA-tcf3b mRNA and co-injected them into 1-cell stage zebrafish embryos. We collected 100-300 embryos at 12, 24, and 48 hpf and performed western blot analysis using the same anti-HA and anti-Flag antibodies validated in HEK293 cell experiments. Despite multiple independent attempts, we were unable to detect clear bands of the tagged proteins in zebrafish embryos. We speculate that this could be due to mRNA instability, translational efficiency, or the low abundance of Id2b and Tcf3b proteins. We have acknowledged these technical limitations in the revised manuscript and clarified that the HEK293 cell data support a potential interaction between Id2b and Tcf3b, while confirming their endogenous interaction will require further investigations (Lines 295-296).

      Reviewer #3 (Public review):

      Summary:

      How mechanical forces transmitted by blood flow contribute to normal cardiac development remains incompletely understood. Using the unique advantages of the zebrafish model system, Chen et al make the fundamental discovery that endocardial expression of id2b is induced by blood flow and required for normal atrioventricular canal (AVC) valve development and myocardial contractility by regulating calcium dynamics. Mechanistically, the authors suggest that Id2b binds to Tcf3b in endocardial cells, which relieves Tcf3b-mediated transcriptional repression of Neuregulin 1 (NRG1). Nrg1 then induces expression of the L-type calcium channel component LRRC1. This study significantly advances our understanding of flow-mediated valve formation and myocardial function.

      Strengths:

      Strengths of the study are the significance of the question being addressed, use of the zebrafish model, and data quality (mostly very nice imaging). The text is also well-written and easy to understand.

      Weaknesses:

      Weaknesses include a lack of rigor for key experimental approaches, which led to skepticism surrounding the main findings. Specific issues were the use of morpholinos instead of genetic mutants for the bmp ligands, cilia gene ift88, and tcf3b, lack of an explicit model surrounding BMP versus blood flow induced endocardial id2b expression, use of bar graphs without dots, the artificial nature of assessing the physical interaction of Tcf3b and Id2b in transfected HEK293 cells, and artificial nature of examining the function of the tcf3b binding sites upstream of nrg1.

      We thank the reviewer for the positive assessment and the constructive suggestions. We have performed additional experiments and data analysis to address these issues. A detailed point-by-point response has been incorporated in the response to “Recommendations for the authors” section.

      Recommendations for the authors:

      Reviewer #1 (Recommendations for the authors):

      Questions/Concerns:

      (1) In the introduction, it would be beneficial to include background information on the id2b gene, what is currently known about its function in heart development/regeneration and in other animal models than just the zebrafish.

      We thank the reviewer for the constructive suggestion. In the revised manuscript, we have added a paragraph in the Introduction to provide background on id2b and its role in heart development. Specifically, we discuss its function as a member of the ID (inhibitor of DNA binding) family of helix-loop-helix (HLH) transcriptional regulators and highlight its involvement in cardiogenesis in both zebrafish and mouse models. These additions help place our findings in a broader developmental and evolutionary context (Lines 91-100).

      (2) Of the 6 differentially expressed genes identified in Figure 1C, why did the authors choose to focus on id2b and not the other 5 downregulated genes?

      We thank the reviewer for the comments. As suggested, we have added a sentence in the revised manuscript to clarify the rationale for selecting id2b as the focus of the present study (Lines 117-121).

      (3) As the authors showed representative in situ images for id2b expression with blebbistatin treatment in Figure 1E, and tnn2a MO in Figure 1F, it would also be beneficial to show relative mRNA expression levels for id2b in conditions of blebbistatin treatment and tnn2a MO knockdown. In Fig. 1C: id2b is downregulated with tricaine, but id2a is upregulated with tricaine. Do these genes perform similar or different functions, results of gene duplication events?

      We thank the reviewer for the thoughtful suggestion. Our in situ hybridization results demonstrate reduced id2b expression following tricaine, blebbistatin, and tnn2 morpholino treatment. To further validate these observations and enhance cellular resolution, we generated an id2b:eGFP knockin line. Analysis of this reporter line confirmed a significant reduction in id2b expression in the endocardium upon inhibition of cardiac contraction and blood flow (Figure 3A-D), supporting our in situ results. The divergent expression patterns of id2a and id2b in response to tricaine treatment likely reflect functional specification following gene duplication in zebrafish. While our current study focuses on characterizing the role of id2b in zebrafish heart development, the specific function of id2a remains to be determined. 

      (4) In Fig. 2b, could the authors compare the id2b fluorescence with RNAscope ISH at 24, 48, and 72 hpf? RNAscope ISH allows for the visualization of single RNA molecules in individual cells. The authors should at least compare these in the heart to demonstrate that id2b accurately reflects the endogenous id2b expression. In Fig. 2E: Suggest showing the individual fluorescent images for id2b:eGFP and kdrl:mCherry in the same colors as top panel images instead of in black and white. In Fig. 2F: The GFP fluorescence from id2b:eGFP signals looks overexposed.

      We thank the reviewer for the valuable comment. In response, we attempted RNAscope in situ hybridization on embryos carrying the id2b:eGFP reporter to directly compare fluorescent reporter expression with endogenous id2b transcripts. However, we encountered a significant reduction in id2b:eGFP fluorescence following the RNAscope procedure, and even subsequent immunostaining with anti-GFP antibodies yielded only weak signals. Despite this technical limitation, the RNAscope results independently confirmed id2b expression in endocardial cells (Figure 2E), supporting the specificity and cell-type localization observed with the reporter line. As suggested by the reviewer, we have updated Figure 2G to display id2b:eGFP and kdrl:mCherry images in the same color scheme as the top panel to improve consistency and clarity. Additionally, we have replaced the images in Figure 2F to avoid overexposure and better represent the spatial distribution of id2b:eGFP in adult heart.

      (5) In Fig. 3A: are all the images in panel A taken with the same magnification? In Fig. 3e, could the authors show the localization of klf2 and id2b and confirm their expression in the same endocardial cells? In Fig. 3, the authors conclude that klf2-mediated biomechanical signaling is essential for activating id2b expression. This statement is somewhat overstated because they only demonstrated that knockout of klf2 reduced id2b expression.

      We thank the reviewer for these constructive comments. All images presented in Figure 3A were captured using the same magnification, as now clarified in the revised figure legend. We appreciate the reviewer’s question regarding the localization of klf2 and id2b. While we were unable to directly visualize both markers in the same embryos due to the current unavailability of klf2 reporter lines, prior studies using klf2a:H2B-eGFP transgenic zebrafish have demonstrated that klf2a is broadly expressed in endocardial cells, with enhanced expression in the atrioventricular canal region (Heckel et al., Curr Bio 2015, PMID: 25959969; Gálvez-Santisteban et al., Elife 2019, PMID: 31237233). Our id2b:eGFP reporter analysis revealed a similarly broad endocardial expression pattern. These independent observations support the likelihood that klf2a and id2b are co-expressed in the same endocardial cell population.   

      We also appreciate the reviewer’s comments regarding the connection between biomechanical signaling and id2b expression. Previous studies have already established that biomechanical cues directly regulate klf2 expression in zebrafish endocardial cells (Vermot et al., Plos Biol 2009, PMID: 19924233; Heckel et al., Curr Bio 2015, PMID: 25959969). In the present study, we observed a significant reduction in id2b expression in both klf2a and klf2b mutants, suggesting that id2b acts downstream of klf2. These observations together establish the role of biomechanical cues-klf2-id2b signaling axis in endocardial cells. Nevertheless, we agree with the reviewer that further investigation is required to elucidate the precise mechanism by which klf2 regulates id2b expression.

      (6) In Fig. 4: What's the mRNA expression for id2b in WT and id2b mutant fish hearts?

      We performed qRT-PCR analysis on purified zebrafish hearts and observed a significant reduction in id2b mRNA levels in id2b mutants compared to wild-type controls. These new results have been incorporated into the revised manuscript (Figure 4A).

      (7) In Fig. 5E, the heart rate shows no difference between id2b+/+ and id2b-/- fish according to echocardiography analysis. However, Fig. 5B indicates a difference in heart rate. Could the authors explain this discrepancy?

      We thank the reviewer for this insightful observation. In our study, we observed a reduction in heart rate in id2b mutants during embryonic stages (120 hpf), as shown in Figure 5B. However, this difference was not evident in adult fish based on echocardiography analysis (Figure 5E). While the exact reason for these changes during development remains unclear, it is possible that the reduction in cardiac output observed in id2b mutants during early development triggers compensatory mechanisms over time, ultimately restoring heart rate in adulthood. Given that heart rate is primarily regulated by pacemaker activity, further investigation will be required to determine whether such compensatory adaptations occur and to elucidate the underlying mechanisms.

      (8) In Fig. 6A: it's a little hard to read the gene names in the left most image in the panel. In Fig. 6B, the authors conducted qRT-PCR analysis of 72 hpf embryonic hearts and validated decreased nrg1 levels in id2b-/- compared to control. Since nrg1 is not specifically expressed in endocardial cells in the developing heart, the authors should isolate endocardial cells and compare nrg1 expression in id2b-/- to control. This would ensure that the loss of id2b affects nrg1 expression derived from endocardial cells rather than other cell types. In Supp Figure S6: Suggest adding an image of the UMAP projection to show tcf3b expression in endocardial cells from sequencing analysis.

      We thank the reviewer for these helpful suggestions. In response, we have increased the font size of gene names in the leftmost panel of Figure 6A to improve readability. Regarding nrg1 expression, we acknowledge the importance of assessing its cell-type specificity. Unfortunately, due to the lack of reliable transgenic or knock-in tools for nrg1, its precise expression pattern in embryonic hearts remains unclear. We attempted to isolate endocardial cells from embryonic hearts using FACS, but the limited number of cells obtained at this stage precluded reliable qRT-PCR analysis. Nonetheless, our data show that id2b is specifically expressed in endocardial cells, and publicly available single-cell RNA-seq datasets also support that nrg1 is predominantly expressed in endocardial, but not myocardial or epicardial cells during embryonic heart development (Figure 6-figure supplement 1). These findings suggest that id2b may regulate nrg1 expression in a cell-autonomous manner within the endocardium. As suggested, we have also added a UMAP image to Figure 7-figure supplement 1 to show tcf3b expression in endocardial cells, further supporting the cell identity in single-cell dataset.

      (9) In Fig. 6, Nrg1 knockout shows no gross morphological defects and normal trabeculation in larvae. Could the authors explain why they propose that endocardial id2b promotes nrg1 synthesis, thereby enhancing cardiomyocyte contractile function? Did Nrg1 knockdown with Mo lead to compromised calcium signaling and cardiac contractile function? Nrg2a has been reported to be expressed in endocardial cells in larvae, and its loss leads to heart function defects. Perhaps Nrg2a plays a more important role than Nrg1.

      We thank the reviewer for raising this important point. Although we did not directly test nrg1 knockout in our study, previous reports have shown that genetic deletion of nrg1 in zebrafish does not impair cardiac trabeculation during embryonic stages (Rasouli et al., Nat Commun 2017, PMID: 28485381; Brown et al., J Cell Mol Med 2018, PMID: 29265764). However, reduced trabecular area and signs of arrhythmia were observed in juvenile and adult fish (Brown et al., J Cell Mol Med 2018, PMID: 29265764), suggesting a potential role for nrg1 in maintaining cardiac structure and function later in development. Whether calcium signaling and cardiac contractility are affected at these stages remains to be determined. Given that morpholino-induced knockdown is limited to early embryonic stages, it is not suitable for assessing nrg1 function in juvenile or adult hearts.

      As noted by the reviewer, nrg2a is expressed in endocardial cells, and its deletion has been associated with cardiac defects (Rasouli et al., Nat Commun 2017, PMID: 28485381). To assess its potential involvement in our model, we performed qRT-PCR analysis and observed increased nrg2a expression in id2b mutant hearts (Author response image 1). This upregulation may reflect a compensatory response to the loss of id2b. Therefore, nrg2a is unlikely to play an essential role in mediating the depressed cardiac function in this context.

      Author response image 1.

      Expression levels of nrg2a. qRT-PCR analysis of nrg2a mRNA in id2b<sup>+/+</sup> and id2b<sup>-/-</sup> adult hearts. Data were normalized to the expression of actb1. N=5 biological replicates, with each sample containing two adult hearts.

      (10) In Fig. 7A of the IP experiment, it is recommended that the authors establish a negative control using control IgG corresponding to the primary antibody source. This control helps to differentiate non-specific background signal from specific antibody signal.

      As suggested, we have included an IgG control corresponding to the primary antibody species in the immunoprecipitation (IP) experiment to distinguish specific from non-specific binding. The updated data are presented in Figure 7A of the revised manuscript.

      (11) In Pg. 5, line 115: there is no reference included for previous literature on blebbistatin.

      We have added the corresponding reference (Line 126, Reference #5).

      In Pg. 5, lines 118-119; pg. 6 line 144: It would be beneficial to include a short sentence describing why choosing a tnnt2a morpholino knockdown to help provide mechanistic context to readers.

      We thank the reviewer for the constructive suggestion. In cardiomyocytes, tnnt2a encodes a sarcomeric protein essential for cardiac contraction, and its knockdown is a well-established method for abolishing heartbeat and blood flow in zebrafish embryos, thereby allowing investigation of flow-dependent gene regulation. In the revised manuscript, we have added a sentence and corresponding reference to clarify the rationale for using tnnt2a morpholino in our study (Lines 128-129, Reference #35).

      In Pg. 6, line 140: Results title of "Cardiac contraction promotes endocardial id2b expression through primary cilia but not BMP" is misleading and contradicts the results presented in this section and corresponding figure. For example, the bmp Mo knockdown experiments led to decreased id2b fluorescence and the last statement of this results section contradicts the title that BMP does not promote endocardial id2b in lines 179-180: "Collectively, these results suggest that BMP signaling and blood flow modulate id2b expression in a developmental-stage-dependent manner." It would be helpful to clarify whether BMP signaling is involved in id2b expression or not.

      We apologize for any confusion caused by the section title. Our results demonstrate that id2b expression is regulated by both BMP signaling and biomechanical forces in a developmental-stage-specific manner. Specifically, morpholino-mediated knockdown of bmp2b, bmp4, and bmp7a at the 1-cell stage significantly reduced id2b:eGFP fluorescence at 24 hpf (Figure 3-figure supplement 1A, B), suggesting that id2b is responsive to BMP signaling during early embryonic development. However, treatment with the BMP inhibitor Dorsomorphin during later stages (24-48 or 36-60 hpf) did not significantly alter id2b:eGFP fluorescence intensity in individual endocardial cells, although a modest reduction in total endocardial cell number was noted (Figure 3-figure supplement 1C, D). These results suggest that BMP signaling is required for id2b expression during early development but becomes dispensable at later stages, when biomechanical cues may play a more prominent role. To address this concern and better reflect the data, we have revised the Results section title to: "BMP signaling and cardiac contraction regulate id2b expression". This revised title more accurately reflects the dual regulation of id2b expression (Line 153).

      In line 205: Any speculation on why the hemodynamics was preserved between id2b mutant and WT siblings at 96 hpf?

      As suggested, we have included a sentence to address this observation. “Surprisingly, the pattern of hemodynamics was largely preserved in id2b<sup>-/-</sup> embryos compared to id2b<sup>+/+</sup> siblings at 96 hpf (Figure 4-figure supplement 1E, Video 1, 2), suggesting that the reduced number of endocardial cells in the AVC region was not sufficient to induce functional defects.” (Lines 223-225)

      In line 246: Fig. 6k and 6j are referenced, but should be figure 5k and 5j.

      We have corrected this in the revised manuscript.

      Reviewer #2 (Recommendations for the authors):

      he manuscript was overall well explained, aside from a few minor points that would help facilitate reader comprehension:

      (1) The last paragraph of the introduction could be a brief summary of the study.

      We thank the reviewer for this constructive suggestion. As recommended, we have included a paragraph in the Introduction section summarizing our key findings to provide clearer context for the study (Lines 96-100).

      (2) Lines 127-128: 'revealed a substantial recapitulation of the... of endogenous id2b expression' may need to be rephrased.

      We thank the reviewer for the valuable suggestion. In the revised manuscript, we have changed the sentence to: “Comparison of id2b:eGFP fluorescence with in situ hybridization at 24, 48, and 72 hpf revealed that the reporter signal closely recapitulates the endogenous id2b expression pattern.” (Lines 137-139)

      (3) Line 182: '... in a developmental-stage-dependent manner' sounds a bit ambiguous, may need to slightly elaborate/ clarify what this means.

      We thank the reviewer for the helpful comment. To improve clarity, we have revised the statement to: “Collectively, these results suggest that id2b expression is regulated by both BMP and biomechanical signaling, with the relative contribution of each pathway varying across developmental stages.” (Lines 195-197)

      Reviewer #3 (Recommendations for the authors):

      (1) The conclusion that BMP signaling prior to 24 hpf is necessary for id2b expression is not fully supported by the data. How do the authors envision pre-linear heart tube BMP signaling impacting endocardial id2b expression during later chamber stages? Id2b reporter fluorescence can be clearly visualized in the linear heart tube in panel B from Figure 1. Does id2b expression initiate prior to contraction? Can the model be refined by showing when id2b endocardial reporter fluorescence is first observed, and whether this early/pre-contractile expression is dependent on BMP signaling?

      We thank the reviewer for the important comment. As suggested, we performed morpholino-mediated knockdown of bmp2b, bmp4, and bmp7a at the 1-cell stage. Live imaging at 24 hpf showed significantly reduced id2b:eGFP fluorescence compared to controls (Figure 3-figure supplement 1A, B), suggesting that id2b is responsive to BMP signaling during early embryonic development. However, treatment with the BMP inhibitor Dorsomorphin during 24-48 or 36-60 hpf did not significantly impact id2b:eGFP fluorescence intensity in individual endocardial cells, although a reduction in endocardial cell number was observed (Figure 3-figure supplement 1C, D). These results suggest that BMP signaling is essential for id2b expression during early embryonic development, while it becomes dispensable at later stages, when biomechanical cues exert a more significant role.

      (2) Overexpressing tagged versions of TCF3b and Id2b in HEK293 cells is a very artificial way to make the major claim that these two proteins interact in endogenous endocardial cells. Can this be done in zebrafish embryonic or adult hearts?

      We thank the reviewer for this insightful comment. As suggested, we synthesized Flag-id2b and HA-tcf3b mRNA and co-injected them into 1-cell stage zebrafish embryos. We collected 100-300 embryos at 12, 24, and 48 hpf and performed western blot analysis using the same anti-HA and anti-Flag antibodies validated in HEK293 cell experiments. Despite multiple independent attempts, we were unable to detect clear bands of the tagged proteins in zebrafish embryos. We speculate that this could be due to mRNA instability, translational efficiency, or the low abundance of Id2b and Tcf3b proteins. We have acknowledged these technical limitations in the revised manuscript and clarified that the HEK293 cell data support a potential interaction between Id2b and Tcf3b, while confirming their endogenous interaction will require further investigations (Lines 295-296).

      (3) The data presented are consistent with the claim that the tcf3b binding sites are functional upstream of nrg1 to repress its transcription. To fully support this idea, those two sites should be disrupted with gRNAs if possible.

      We thank the reviewer for the valuable suggestion. In response, we attempted to disrupt the tcf3b binding sites using sgRNAs. However, we encountered technical difficulties in identifying sgRNAs that specifically and efficiently target these binding sites without affecting adjacent regions. Despite these challenges, our luciferase reporter assay, using tcf3b mRNA overexpression and morpholino knockdown, clearly demonstrated that tcf3b binds to and regulates nrg1 promoter region. Nevertheless, we acknowledge that future study using genome editing will be necessary to validate the direct binding of tcf3b to nrg1 promoter.

      Minor Points:

      (1) Must remove all of the "data not shown" statements and add the primary data to the Supplemental Figures.

      As suggested, we have removed all of the “data not shown” statements and added the original data to the revised manuscript (Figure 4E, middle panels, and Figure 4-figure supplement 1F)

      (2) Must present the order of the panels in the figure as they are presented in the text. One example is Figure 6 where 6E is discussed in the text before 6C and 6D.

      We thank the reviewer for bring up this important point. In the revised manuscript, we have carefully revised the manuscript to ensure that the order of figure panels matches the sequence in which they are discussed in the text. Specifically, we have reorganized the presentation of Figure 6 panels to align with the text flow, discussing panels 6C and 6D before panel 6E. The updated figure and corresponding text have been corrected accordingly in the revised manuscript.

      (3) Change the italicized gene names (e.g. tcf3b) to non-italicized names with the first letter capitalized (e.g. Tcf3b) when referencing the protein.

      As suggested, we have revised the manuscript to use non-italicized names with the first letter capitalized when referring to proteins.

      (4) All bar graphs should be replaced with dot bar graphs.

      We have replaced all bar graphs with dot bar graphs throughout the manuscript.

      (5) The new id2b mutant allele should be validated as a true null using quantitative RT-PCR to show that the message becomes destabilized through non-sense mediated decay or by immunostaining/western blot analysis if there is a zebrafish Id2b-specific antibody available.

      We thank the reviewer for this important suggestion. We have performed qRT-PCR analysis and detected a significant reduction in id2b mRNA levels in id2b<sup>-/-</sup> compared to id2b<sup>+/+</sup> controls. These new results are presented in Figure 4A of the revised manuscript.

      (6) Was tricaine used to anesthetize embryos for capturing heart rate and percent fractional area change? This analysis should be performed with no or very limited tricaine as it affects heart rate and systolic function. These parameters were captured at 120 hpf, but the authors should also look earlier at 72 hpf at a time when valves are not present by calcium transients are necessary to support heart function.

      We thank the reviewer for this important comment. When performing live imaging to assess cardiac contractile function, we used low-dose tricaine (0.16 mg/mL) to anesthetize the zebrafish embryos. We have included this important information in the Methods section (Line 503). As suggested, we have also included the heart function results at 72 hpf, which are now presented in Figure 5-figure supplement 2A-C of the revised manuscript.

      (7) The alpha-actinin staining in Figure 5-supplement 2D is very pixelated and unconvincing. This should be repeated and imaged at a higher resolution.

      As suggested, we have re-performed the α-actinin staining and acquired higher-resolution images. The updated results are now presented in Figure 5-figure supplement 2G of the revised manuscript.

      (8) The authors claim that reductions in id2b mutant heart contractility are due to perturbed calcium transients instead of sarcomere integrity. Why do the authors think that regulation of calcium dynamics was not observed in the DEG enriched GO-terms? Was significant downregulation of cacna1 identified in the bulk RNAseq?

      We thank the reviewer for raising this important point. In our bulk RNAseq dataset comparing id2b mutant and control hearts, GO term enrichment was primarily associated with pathways related to cardiac muscle contraction and heart contraction (Figure 5-figure supplement 1B). We speculate that the transcriptional changes related to calcium dynamics may be relatively subtle and thus were not captured as significantly enriched GO terms. In addition, our qRT-PCR analysis revealed a significant reduction in cacna1c expression in id2b mutant hearts compared to controls, suggesting that id2b deletion impairs calcium channel expression. However, this change was not detected by RNA-seq, likely due to limitations in sensitivity.

      (9) In line 277, the authors say, "To determine whether this interaction occurs in zebrafish, Flag-id2b and HA-tcf3b were co-expressed in HEK293 cells...". This should be re-phrased to, "To determine if zebrafish Id2b and Tcf3b interact in vitro, Flag-id2b and HA-tcf3b were co-expressed in HEK293 cells for co-immunoprecipitation analysis." The sentence in line 275 should be changed to, "....heterodimer with Tcf3b to limit its function as a potent transcriptional repressor."

      We thank the reviewer for these constructive comments and have revised the text accordingly (Lines 291-294).

      (10) Small text corrections or ideas:

      Line 63: emphasized

      We have corrected this in the revised manuscript.

      Line 71: studied signaling pathways

      We have corrected this in the revised manuscript.

      Line 106: the top 6 DEGS (I think that the authors mean top 6 GO-terms) and is Id2b in one of the enriched GO categories?

      id2b is one of the top DEGs. This point has been clarified in the revised manuscript (Lines 116-117).

      Line 125: a knockin id2b:eGFP reporter line

      We have corrected this in the revised manuscript (Line 136).

      Line 138: This paragraph could use a conclusion sentence.

      We have added a conclusion sentence in the revised manuscript (Lines 150-151).

      Line 190: id2b-/- zebrafish experienced early lethality

      We have revised the statement as suggested (Line 206).

      Line 193: The prominent enlargement of the atrium with a smaller ventricle has characterized as cardiomyopathy in zebrafish (Weeks et al. Cardiovasc Res, 2024, PMID: 38900908), which has also been associated with disruptions in calcium transients (Kamel et al J Cardiovasc Dev Dis, 2021, PMID: 33924051 and Kamel et al, Nat Commun 2021, PMID: 34887420). This information should be included in the text along with these references.

      We thank the reviewer for this helpful suggestion. We have incorporated these important references into the revised manuscript and included the relevant information to acknowledge the established link between atrial enlargement, cardiomyopathy, and disrupted calcium transients in zebrafish models (Reference #41, 42, and 45; Lines 210 and 260).

    1. Author Response

      The following is the authors’ response to the original reviews.

      Reviewer #1 (Public Review):

      [...] Weaknesses

      Showing that A-2 and especially A-3 are outliers in the PCA analysis is useful, but it may be hiding other interesting signals in the data. The other strains are remarkably colinear on these plots, hinting that if the outliers were removed, one main component would emerge along which they are situated. It also seems possible that this additional analysis step would allow the second dimension to better differentiate them in a way that is interesting with respect to their mutator status or mutations in key metabolic or regulatory genes.

      We thank the reviewer for their positive comments and their constructive feedback on the manuscript. Following reviewer’s recommendation, we performed the PCA analysis on metabolism data after removing A-2 and A-3 data. We have detailed those results below. Consistent with a similar analysis performed on RNA-seq datasets in our previous publication, we find that removing these outliers has only a modest effect on separating mutators from non-mutators. We find that, while the new PC2 separates most mutators from the non-mutators, the separation is rather weak. Moreover, we do not see a similar distinction when looking at metabolic data in the Stationary phase. In the interest of improving the readability of the manuscript, we recommend not including these analysis in the final manuscript. We have presented the data for the reviewer’s benefit in Author response image 1, 2 and 3.

      Author response image 1.

      Author response image 2.

      Author response image 3.

      There is a missed opportunity to connect some key results to what is known about LTEE mutations that reduce the activity of pykF (pyruvate kinase I). This gene is mutated in all 12 LTEE populations, and often these mutations are frameshifts or transposon insertions that should completely knock out its activity. At first glance, inactivating an enzyme for a step in glycolysis does not make sense when the nutrient source in the growth medium is glucose, even though PykF is only one of two isozymes E. coli encodes for this reaction. There has been speculation that inactivating pykF increases the concentration of phosphoenolpyruvate (PEP) in cells and that this can lead to increased rates of glucose import because PEP is used by the phosphotransferase system of E. coli to import glucose (see https://doi.org/10.1002/bies.20629). The current study has confirmed the higher PEP levels, which is consistent with this model.

      We thank the reviewer for pointing out this missed opportunity. We have expanded the discussion around the role of pykF mutations and the elevated concentrations of PEP observed in our data in section 3.4.

      In the introduction, the papers cited to show the importance of changes in metabolism for adaptation do not seem to fit the focus of this study very well. They stress production of toxins and secondary metabolites, which do not seem to be mechanisms that are at work in the LTEE. I can think of two areas of background that would be more relevant: (1) studies of how bacterial metabolism evolves in adaptive laboratory evolution (ALE) experiments to optimize metabolic fluxes toward biomass production (for example, https://doi.org/10.1038/nature01149), and (2) discussions of how cross-feeding, metabolic niche specialization, and metabolic interdependence evolve in microbial communities, including in other evolution experiments (for example, https://doi.org/10.1073/pnas.0708504105 and https://doi.org/10.1128/mBio.00036-12).

      We thank the reviewer for pointing out missed citations in our introduction. We agree that these papers are relevant to the topic and have added their citations. Additionally, following the suggestion of another reviewer, we have reorganized the introduction so that the concept of the role of metabolism in evolution is presented first and the LTEE second.

      Reviewer #2 (Public Review):

      [...] Overall, this is a significant and well-executed research study. It offers new insights into the complex relationship between genetic changes and observable traits in evolving populations and utilizes metabolomics in the LTEE, a novel approach in combination with RNA-seq and mutation datasets.

      However, the paper's overall clarity is lacking. It is spread too thin and covers many topics without a clear focus. I strongly recommend a substantial rewrite of the manuscript, emphasizing structure and readability. The science is well executed, but the current writing does not do it justice.

      We thank the reviewer for their positive comments and their constructive feedback on the lack of clarity in writing. Following the reviewer’s suggestions, we have rewritten parts of the manuscript and reorganizd a few sections to improve readability. We hope the revised manuscript is significantly improved.

      Recommendations for the authors

      Reviewer #1 (Recommendations For The Authors):

      1) Title and Abstract: Add the study organism to the abstract, and probably also the title. Currently, E. coli is not mentioned in either! I'm also not sure that the LTEE is a sufficiently well-known acronym to abbreviate this in the title.

      We have revised the title of the manuscript and now spell out LTEE and included E. coli in the title and the abstract.

      2) Abstract: I would switch the usage of metabolome to metabolism in a few more places. For example, "changes in its metabolism", "networked and convoluted nature of metabolism". The metabolome, the concentrations of all metabolites, is what is being measured, but I think of this as a phenotypic readout of how metabolism evolving.

      We have changed “metabolome” to “metabolism” in cases where we refer to what is evolving and use “metabolome” when we refer to what is being measured.

      3) Line 16: Technically, the 12 LTEE populations were not initially identical. The Ara- differed from the Ara+ ancestors by one intentional mutation and one unintentional mutation that was not discovered until whole genomes were sequenced. I would rephrase this to "where 12 replicate populations of E. coli are propagated" or something similar so that it can be correct without needing to describe this unnecessary detail.

      The line has been rephrased as suggested.

      4) General Note: The text refers to populations as Ara-3 but the figures use A-3. I'd suggest going with A-3 and similar throughout for consistency.

      Instances of Ara have been changed to A+/-, and a sentence specifying as such has been added to the intro to make mention of this.

      5) Lines 43-44, 97-98. My understanding is that both S and L ecotypes in A-2 can use both glucose and acetate, but that the differentiation is related to their specialization that leads to each one being better on one or the other nutrient. The descriptions make it sound like each grows at a different time. Also, by definition, cells are not growing during "stationary phase". The change from glucose utilization (and acetate secretion) to acetate utilization during one cycle of growth is better described as a diauxic shift.

      We have reworded this part to remove mention of “growth” during stationary phase and changed the wording such that it no longer sounds like they grow at different times.

      6) Line 54: The statement "provide the ability to test hypotheses from previous data" is vague. Either provide an example or delete.

      We have removed this sentence as suggested.

      7) Lines 71-72: The terms "interphase" and "intraphase" sound too much like parts of the cell cycle. I'd suggest describing the comparisons as between and within growth phases.

      The use of intra and interphase have been changed as suggested.

      8) Line 79: The citrate is presumably still a chelating agent, so change phrasing to "Citrate is present in the medium because it was originally added as a chelating agent" or something similar.

      This sentence has been rewritten as suggested.

      9) Line 83: Write out "mutation accumulations" so it is easier to understand as "the number of mutations that have accumulated".

      The phrase has been changed as suggested.

      10) Line 116: It's unclear whether the abundances of metabolites are "strategies of survival" in stationary phase. An equally valid explanation is that there is less selection on the metabolome to have a specific composition during stationary phase to have high fitness.

      We have added a line about the possibility for alternative hypotheses.

      11) Figure 1: There seems to be some information missing from the legend. What are R06 and R07 in Panels A and B? Is panel D exponential phase and panel E stationary phase?

      This information was inadvertently missing from the caption and has been added.

      12) Figures 2 and 3: Gene names should be in italics. To me, the gray for deleted genes is hard to tell apart from the blue/red. Perhaps you could put a little X in these boxes instead? I think that having a little triangle pointing from each gene or metabolite name its corresponding abundance panel would help the reader track which information goes with which features. In Fig. 3 the placement of L-aspartate is a bit awkward. I'd suggest moving it down so the dashed line does not have to go through the abundance panel.

      These figures have been edited to include small triangles that link a gene or metabolite and its heatmap. Additionally, an X has been added where genes have suffered inactivating mutations and the placement of some elements has been moved to improve overall clarity.

      13) Lines 183-185: It would be easier to see and judge the consistency of these argR related relationships if a correlation graph of some kind was shown, probably as a supplemental figure. This plot could, for example, have genes/metabolites across the x-axis and fold-change on the y-axis with lines connecting points corresponding to each of the twelve populations across these categories (like Fig S8 but with lines added). Alternatively, it could be a heat map with the populations across one axis and the genes/metabolites across the other axis (like Fig S3).

      We have added a supplementary figure consisting of heatmaps showing the consistency of these changes within an evolved line. It is now figure S9.

      14) Line 195: I think adding a sentence elaborating on what exactly mutation accumulation means in this context would be helpful to readers.

      We have attempted to clarify the meaning of this by specifically stating that it is due to the accumulation of deleterious mutations.

      15) Line 293: Is standard LTEE medium DM25? These omics experiments with the LTEE sometimes use similar media with different glucose concentrations, and this is a very important detail to precisely specify.

      We reference “standard” LTEE medium in the methods section and have additionally specified the amount of sugar to make it clear that we are not supplementing the media with additional sugar.

      16) Figure S8B. Is "cystine" used instead of "cysteine" on purpose here since the compound is oxidized in the metabolomics treatment?

      The use of cystine is intentional, we detect the oxidized compound.

      Reviewer #2 (Recommendations For The Authors):

      Title:

      The abbreviation "LTEE" should not be in the title. Most readers will not recognize what it means. Instead, either the full name of the experiment, "Long-Term Evolution Experiment with E. coli," should be used, or the title should be rephrased to "Linking genotypic and phenotypic changes during a long-term evolution experiment using metabolomics."

      We have spelled out LTEE and included E. coli in the title.

      Abstract:

      Sentence 1: Consider softening the statement: "Do changes in an organism's environment, genome, or gene expression patterns often lead to changes in its metabolome?"

      We have rephrased this sentence to “Changes in an organism's environment, genome, or gene expression patterns can lead to changes in its metabolism”.

      Sentence 4: Use a hyphen for "Long-Term."

      This addition has been made.

      Sentence 4: Replace "transduce" with a more appropriate term: "...how the effects of mutations can be distributed through a cellular network to eventually affect metabolism and fitness."

      We have rewritten this sentence as “to understand how mutations can eventually affect metabolism and perhaps fitness”.

      Sentence 5: Clarify the use of "both" to refer to the ancestor of the LTEE and its descendant populations as two classes.

      We have reworded this sentence so it’s clear that the ancestors and evolved lines are two separate classes “We used mass-spectrometry to broadly survey the metabolomes of the ancestral strains and all 12 evolved lines…”.

      Sentence 6: Reverse the order for better emphasis: "Our work provides a better understanding of how mutations might affect fitness through the metabolome in the LTEE, and thus provides a major step in developing a complete genotype-phenotype map for this experimental system."

      We have rearranged this sentence per the reviewers suggestion.

      Introduction:

      Revise the introduction for clarity, readability, and logical narrative progression. Start with the second paragraph to set up the basic scientific principles being studied and then transition to describing the LTEE as a model system to examine those principles.

      The introduction has been rearranged and reworded in parts to increase clarity.

      Sentence 1: Revise for clarity: "The Long-Term Evolution Experiment (LTEE) has studied 12 initially identical populations of Escherichia coli as they have evolved in a carbon-limited, minimal glucose medium under a daily serial transfer regime."

      Sentence 2: Suggestion: "Begun in 1988, the LTEE populations have evolved for more than 75,000 generations, making it the longest-running experiment of its kind."

      Paragraph 2, sentence 2: Italicize "Drosophila."

      Paragraph 3, sentence 2: Make an important distinction: "Ara-3 is unique in that it evolved the ability to grow aerobically on citrate."

      Paragraph 3, sentence 4: Introduce the IS-mediated loss of the rbs operon in the LTEE as if it has not been described elsewhere.

      These suggestions have been incorporated into the manuscript.

      Results:

      Section 3.1: The use of samples from hours 2 and 24 to represent exponential and stationary phase may present some issues. For instance, capturing Ara-3 during its exponential growth on glucose, but not citrate, at hour 2. Furthermore, except for Ara-3, the LTEE populations reach stationary phase after approximately 4 hours, and there could be significant differences between early, mid, and late stationary phase. This possibility should be acknowledged, and future follow-up work should consider exploring these differences.

      We have added sentences in the first paragraph of the results section to include these details. We have also added a short paragraph to the conclusions suggesting additional studies of stationary phase, citing work on evolution of E. coli during long term stationary phase.

      Paragraph 3: While Turner et al. 2017 is an essential reference regarding resource use differences between Ara-3 and other LTEE populations, it would be more suitable to reference Blount et al. 2012 for the mutations that enabled access to citrate. Also, it is important to note that the difference lies in the ability to grow aerobically on citrate, rather than the ability to metabolize it.

      This citation has been added.

      Paragraph 4: As mentioned elsewhere, most LTEE populations exhibit balanced polymorphisms. Therefore, it is more appropriate to state that Ara-2 is the best-understood example of long-term diversity. It is likely that there are important metabolic differences between co-existing lineages in other LTEE populations.

      We now refer to Ara-2 as being the best-understood example of long term diversity..

      Paragraph 5: The first sentence of this paragraph should likely end with "levels."

      The word “levels” was added to the end of this sentence.

      Figure 3: It is preferable to refer to the "Superpathway of arginine and polyamine biosynthesis," citing EcoCyc as a reference, rather than a descriptor.

      This has been changed to a reference.

      Section 3.3, Paragraph 3: While higher intracellular amino acid abundances may facilitate higher translation rates and faster growth, the higher abundances themselves do not evaluate the hypothesis. To evaluate the hypothesis, it is necessary to demonstrate that higher abundances are associated with higher translation or growth rates. Therefore, the final sentence of this paragraph is not meaningful.

      We have reworded this sentence to say that it’s not possible to tell what the additional amino acids are being used for given only this data and that additional experiments are needed to confirm this hypothesis.

      Section 3.4: The first paragraph of this section misstates how evolution works. The low level of glucose in the LTEE does not drive innovation; instead, innovation occurs at random through the introduction of variation by mutation. Although the existence of the citrate resource acts as a reward that selects for variation that provides access to it, it is essential to remember that evolution is blind to such a reward. Moreover, regarding the evolution of the Cit+ trait, it is incorrect to assert that low glucose contributed to its evolution. As shown by Quandt et al. (2015), it seems probable that Cit+ evolution was potentiated by adaptation to specialization on acetate, which is produced by overflow metabolism resulting from rapid growth on glucose. This rapid growth only occurs when glucose is relatively abundant. The level of glucose seems low to us because it is low relative to traditional levels in bacteriological media, but not to the bacteria.

      We agree that this is a semantical, but important distinction. We have reworded this part as to not suggest that evolution has any forward thinking properties and is indeed blind to any rewards that might occur as the result of adaptation.

      In general, all instances of "utilize" and its cognates should be replaced with "use" and its cognates.

      Instances of “utilize” have been changed to use and its cognates.

      There is some uncertainty about the expectation of ramping up the TCA cycle in the LTEE. Overflow metabolism and acetate production appear to be prevalent in the LTEE, suggesting that many lineages only partially oxidize carbon derived from glucose, thereby bypassing the TCA cycle. While it is possible that this interpretation is incorrect, it would be helpful to see it addressed in the manuscript.

      We agree that this is a plausible hypothesis, we have added a paragraph at the end of this section that discusses the implications of overflow metabolism as an alternative hypothesis.

    1. Author Response

      The following is the authors’ response to the original reviews.

      eLife assessment

      This study provides potentially important, new information about the combination of information from the two eyes in humans. The data included frequency tagging of each eye's inputs and measures reflecting both cortical (EEG) and sub-cortical processes (pupillometry). Binocular combination is of potentially general interest because it provides -in essence- a case study of how the brain combines information from different sources and through different circuits. The strength of supporting evidence appears to be solid, showing that temporal modulations are combined differently than spatial modulations, with additional differences between subcortical and cortical pathways. However, the manuscript's clarity could be improved, including by adding more convincing motivations for the approaches used.

      We thank the editor and reviewers for their detailed comments and suggestions regarding our paper. We have implemented most of the suggested changes. In doing so we noticed a minor error in our analysis code that affected the functions shown in Figure 2e (previously Figure 1e), and have fixed this and rerun the modelling. Our main results and conclusions are unaffected by this change. We have also added a replication data set to the Appendix, as this bears on one of the points raised by a reviewer, and included a co-author who helped run this experiment.

      Reviewer #1 (Public Review):

      In this paper, the interocular/binocular combination of temporal luminance modulations is studied. Binocular combination is of broad interest because it provides a remarkable case study of how the brain combines information from different sources. In addition, the mechanisms of binocular combination are of interest to vision scientists because they provide insight into when/where/how information from two eyes is combined.

      This study focuses on how luminance flicker is combined across two eyes, extending previous work that focused mainly on spatial modulations. The results appear to show that temporal modulations are combined in different ways, with additional differences between subcortical and cortical pathways.

      1. Main concern: subcortical and cortical pathways are assessed in quite different ways. On the one hand, this is a strength of the study (as it relies on unique ways of interrogating each pathway). However, this is also a problem when the results from two approaches are combined - leading to a sort of attribution problem: Are the differences due to actual differences between the cortical and subcortical binocular combinations, or are they perhaps differences due to different methods. For example, the results suggest that the subcortical binocular combination is nonlinear, but it is not clear where this nonlinearity occurs. If this occurs in the final phase that controls pupillary responses, it has quite different implications.

      At the very least, this work should clearly discuss the limitations of using different methods to assess subcortical and cortical pathways.

      The modelling asserts that the nonlinearity is primarily interocular suppression, and that this is stronger in the subcortical pathway. Moreover the suppression impacts before binocular combination. So this is quite a specific location. We now say more about this in the Discussion, and also suggest that fMRI might avoid the limits on the conclusions we can draw from different methods.

      1. Adding to the previous point, the paper needs to be a better job of justifying not only the specific methods but also other details of the study (e.g., why certain parameters were chosen). To illustrate, a semi-positive example: Only page 7 explains why 2Hz modulation was used, while the methods for 2Hz modulation are described in detail on page 3. No justifications are provided for most of the other experimental choices. The paper should be expanded to better explain this area of research to non-experts. A notable strength of this paper is that it should be of interest to those not working in this particular field, but this goal is not achieved if the paper is written for a specialist audience. In particular, the introduction should be expanded to better explain this area of research, the methods should include justifications for important empirical decisions, and the discussion should make the work more accessible again (in addition to addressing the issues raised in point 1 above). The results also need more context. For example, why EEG data have overtones but pupillometry does not?

      We now explain the choice of frequency in the final paragraph of the introduction as follows:

      ‘We chose a primary flicker frequency of 2Hz as a compromise between the low-pass pupil response (see Barrionuevo et al., 2014; Spitschan et al., 2014), and the relatively higher-pass EEG response (Regan, 1966).’

      We also mention why the pupil response is low-pass:

      ‘The pupil response can be modulated by periodic changes in luminance, and is temporally low-pass (Barrionuevo et al., 2014; Spitschan et al. 2014), most likely due to the mechanical limitations of the iris sphincter and dilator muscles’.

      Reviewer #2 (Public Review):

      Previous studies have extensively explored the rules by which patterned inputs from the two eyes are combined in the visual cortex. Here the authors explore these rules for un-patterned inputs (luminance flicker) at both the level of the cortex, using Steady-State Visual Evoked Potentials (SSVEPs) and at the sub-cortical level using pupillary responses. They find that the pattern of binocular combination differs between cortical and sub-cortical levels with the cortex showing less dichoptic masking and somewhat more binocular facilitation.

      Importantly, the present results with flicker differ markedly from those with gratings (Hou et al., 2020, J Neurosci, Baker and Wade 2017 cerebral cortex, Norcia et al, 2000 Nuroreport, Brown et al., 1999, IOVS). When SSVEP responses are measured under dichoptic conditions where each eye is driven with a unique temporal frequency, in the case of grating stimuli, the magnitude of the response in the fixed contrast eye decreases as a function of contrast in the variable contrast eye. Here the response increases by varying (small) magnitudes. The authors favor a view that cortex and perception pool binocular flicker inputs approximately linearly using cells that are largely monocular. The lack of a decrease below the monocular level when modulation strength increase is taken to indicate that previously observed normalization mechanism in pattern vision does not play a substantial role in the processing of flicker. The authors present a computational model of binocular combination that captures features of the data when fit separately to each data set. Because the model has no frequency dependence and is based on scalar quantities, it cannot make joint predictions for the multiple experimental conditions which is one of its limitations.

      A strength of the current work is the use of frequency-tagging of both pupil and EEG responses to measure responses for flicker stimuli at two anatomical levels of processing. Flicker responses are interesting but have been relatively neglected. The tagging approach allows one to access responses driven by each eye, even when the other eye is stimulated which is a great strength. The tagging approach can be applied at both levels of processing at the same time when stimulus frequencies are low, which is an advantage as they can be directly compared. The authors demonstrate the versatility of frequency tagging in a novel experimental design which may inspire other uses, both within the present context and others. A disadvantage of the tagging approach for studying sub-cortical dynamics via pupil responses is that it is restricted to low temporal frequencies given the temporal bandwidth of the pupil. The inclusion of a behavioral measure and a model is also a strength, but there are some limitations in the modeling (see below).

      The authors suggest in the discussion that luminance flicker may preferentially drive cortical mechanisms that are largely monocular and in the results that they are approximately linear in the dichoptic cross condition (no effect of the fixed contrast stimulus in the other eye). By contrast, prior research using dichoptic dual frequency flickering stimuli has found robust intermodulation (IM) components in the VEP response spectrum (Baitch and Levi, 1988, Vision Res; Stevens et al., 1994 J Ped Ophthal Strab; France and Ver Hoeve, 1994, J Ped Ophthal Strab; Suter et al., 1996 Vis Neurosci). The presence of IM is a direct signature of binocular interaction and suggests that at least under some measurement conditions, binocular luminance combination is "essentially" non-linear, where essential implies a point-like non-linearity such as squaring of excitatory inputs. The two views are in striking contrast. It would thus be useful for the authors could show spectra for the dichoptic, two-frequency conditions to see if non-linear binocular IM components are present.

      This is an excellent point, and one that we had not previously appreciated the importance of. We have generated a figure (Fig 8) showing the IM response in the cross frequency conditions. There is a clear response at 0.4Hz in the pupillometry data (2-1.6Hz), and at 3.6Hz in the EEG data (2+1.6Hz). We therefore agree that this shows the system is essentially nonlinear, despite the binocular combination appearing approximately linear. We now say in the Discussion:

      ‘In the steady-state literature, one hallmark of a nonlinear system is the presence of intermodulation responses at the sums and differences of fundamental flicker frequencies (Baitch & Levi, 1988; Tsai et al., 2012). In Figure 8 we plot the amplitude spectra of conditions from Experiment 1 in which the two eyes were stimulated at different frequencies (2Hz and 1.6Hz) but at the same contrast (48%; these correspond to the binocular cross and dichoptic cross conditions in Figures 2d,e and 3d,e). Consistent with the temporal properties of pupil responses and EEG, Figure 8a reveals a strong intermodulation difference response at 0.4Hz (red dashed line), and Figure 8b reveals an intermodulation sum response at 3.6Hz (red dashed line). The presence of these intermodulation terms is predicted by nonlinear gain control models of the type considered here (Baker and Wade, 2017; Tsai et al., 2012), and indicates that the processing of monocular flicker signals is not fully linear prior to the point at which they are combined across the eyes.’

      If the IM components are indeed absent, then there is a question of the generality of the conclusions, given that several previous studies have found them with dichoptic flicker. The previous studies differ from the authors' in terms of larger stimuli and in their use of higher temporal frequencies (e.g. 18/20 Hz, 17/21 Hz, 6/8 Hz). Either retinal area stimulated (periphery vs central field) or stimulus frequency (high vs low) could affect the results and thus the conclusions about the nature of dichoptic flicker processing in cortex. It would be interesting to sort this out as it may point the research in new directions.

      This is a great suggestion about retinal area. As chance would have it, we had already collected a replication data set where we stimulated the periphery, and we now include a summary of this data set as an Appendix. In general the results are similar, though we obtain a measurable (though still small) second harmonic response in the pupillometry data with this configuration, which is a further indication of nonlinear processing.

      Whether these components are present or absent is of interest in terms of the authors' computational model of binocular combination. It appears that the present model is based on scalar magnitudes, rather than vectors as in Baker and Wade (2017), so it would be silent on this point. The final summation of the separate eye inputs is linear in the model. In the first stage of the model, each eye's input is divided by a weighted input from the other eye. If we take this input as inhibitory, then IM would not emerge from this stage either.

      We have performed the modelling using scalar values here for simplicity and transparency, and to make the fitting process computationally feasible (it took several days even done this way). This type of model is quite capable of processing sine waves as inputs, and producing a complex output waveform which is Fourier transformed and then analysed in the same way as the experimental data (see e.g. Tsai, Wade & Norcia, 2012, J Neurosci; Baker & Wade, 2017, Cereb Cortex). However our primary aim here was to fit the model, and make inferences about the parameter values, rather than to use a specific set of parameter values to make predictions. We now say more about this family of models and how they can be applied in the methods section:

      “Models from this family can handle both scalar contrast values and continuous waveforms (Tsai et al., 2012) or images (Meese and Summers, 2007) as inputs. For time-varying inputs, the calculations are performed at each time point, and the output waveform can then be analysed using Fourier analysis in the same way as for empirical data.This means that the model can make predictions for the entire Fourier spectrum, including harmonic and intermodulation responses that arise as a consequence of nonlinearities in the model (Baker and Wade, 2017). However for computational tractability, we performed fitting here using scalar contrast values.”

      As a side point, there are quite a lot of ways to produce intermodulation terms, meaning they are not as diagnostic as one might suppose. We demonstrate this in Author response image 1, which shows the Fourier spectra produced by a toy model that multiplies its two inputs together (for an interactive python notebook that allows various nonlinearities to be explored, see here). Intermodulation terms also arise when two inputs of different frequencies are summed, followed by exponentiation. So it would be possible to have an entirely linear binocular summation process, followed by squaring, and have this generate IM terms (not that we think this is necessarily what is happening in our experiments).

      Author response image 1

      Related to the model: One of the more striking results is the substantial difference between the dichoptic and dichoptic-cross conditions. They differ in that the latter has two different frequencies in the two eyes while the former has the same frequency in each eye. As it stands, if fit jointly on the two conditions, the model would make the same prediction for the dichoptic and dichoptic-cross conditions. It would also make the same prediction whether the two eyes were in-phase temporally or in anti-phase temporally. There is no frequency/phase-dependence in the model to explain differences in these cases or to potentially explain different patterns at the different VEP response harmonics. The model also fits independently to each data set which weakens its generality. An interpretation outside of the model framework would thus be helpful for the specific case of differences between the dichoptic and dichoptic-cross conditions.

      As mentioned above, the limitations the reviewer highlights are features of the specific implementation, rather than the model architecture in general. Furthermore, although this particular implementation of the model does not have separate channels for different phases, these can be added (see e.g. Georgeson et al., 2016, Vis Res, for an example in the spatial domain). In future work we intend to explore the phase relationship of flicker, but do not have space to do this here.

      Prior work has defined several regimes of binocular summation in the VEP (Apkarian et al.,1981 EEG Journal). It would be useful for the authors to relate the use of their terms "facilitation" and "suppression" to these regimes and to justify/clarify differences in usage, when present. Experiment 1, Fig. 3 shows cases where the binocular response is more than twice the monocular response. Here the interpretation is clear: the responses are super-additive and would be classed as involving facilitation in the Apkarian et al framework. In the Apkarian et al framework, a ratio of 2 indicates independence/linearity. Ratios between 1 and 2 indicate sub-additivity and are diagnostic of the presence of binocular interaction but are noted by them to be difficult to interpret mechanistically. This should be discussed. A ratio of <1 indicates frank suppression which is not observed here with flicker.

      Operationally, we use facilitation to mean an increase in response relative to a monocular baseline, and suppression to mean a decrease in response. We now state this explicitly in the Introduction. Facilitation greater than a factor of 2 indicates some form of super-additive summation. In the context of the model, we also use the term suppression to indicate divisive suppression between channels, however this feature does not always result in empirical suppression (it depends on the condition, and the inhibitory weight). We think that interpretation of results such as these is greatly aided by the use of a computational modelling framework, which is why we take this approach here. The broad applicability of the model we use in the domain of spatial contrast lends it credibility for our stimuli here.

      Can the model explore the full range of binocular/monocular ratios in the Apkarian et al framework? I believe much of the data lies in the "partial summation" regime of Apkarian et al and that the model is mainly exploring this regime and is a way of quantifying varying degrees of partial summation.

      Yes, in principle the model can produce the full range of behaviours. When the weight of suppression is 1, binocular and monocular responses are equal. When the weight is zero, the model produces linear summation. When the weight is greater than 1, suppression occurs. It is also possible to produce super-additive summation effects, most straightforwardly by changing the model exponents. However this was not required for our data here, and so we kept these parameters fixed. We agree that the model is a good way to unify the results across disparate experimental paradigms, and that is our main intention with Figure 7i.

      Reviewer #3 (Public Review):

      This manuscript describes interesting experiments on how information from the two eyes is combined in cortical areas, sub-cortical areas, and perception. The experimental techniques are strong and the results are potentially quite interesting. But the manuscript is poorly written and tries to do too much in too little space. I had a lot of difficulty understanding the various experimental conditions, the complicated results, and the interpretations of those results. I think this is an interesting and useful project so I hope the authors will put in the time to revise the manuscript so that regular readers like myself can better understand what it all means.

      Now for my concerns and suggestions:

      The experimental conditions are novel and complicated, so readers will not readily grasp what the various conditions are and why they were chosen. For example, in one condition different flicker frequencies were presented to the two eyes (2Hz to one and 1.6Hz to the other) with the flicker amplitude fixed in the eye presented to the lower frequency and the flicker amplitude varied in the eye presented to the higher frequency. This is just one of several conditions that the reader has to understand in order to follow the experimental design. I have a few suggestions to make it easier to follow. First, create a figure showing graphically the various conditions. Second, come up with better names for the various conditions and use those names in clear labels in the data figures and in the appropriate captions. Third, combine the specific methods and results sections for each experiment so that one will have just gone through the relevant methods before moving forward into the results. The authors can keep a general methods section separate, but only for the methods that are general to the whole set of experiments.

      We have created a new figure (now Fig 1) that illustrates the conditions from Experiment 1, and is referenced throughout the paper. We have kept the names constant, as they are rooted in a substantial existing literature, and it will be confusing to readers familiar with that work if we diverge from these conventions. We did consider separating out the methods section, but feel it helps the flow of the results section to keep it as a single section.

      I wondered why the authors chose the temporal frequencies they did. Barrionuevo et al (2014) showed that the human pupil response is greatest at 1Hz and is nearly a log unit lower at 2Hz (i.e., the change in diameter is nearly a log unit lower; the change in area is nearly 2 log units lower). So why did the authors choose 2Hz for their primary frequency? And why did the authors choose 1.6Hz which is quite close to 2Hz for their off frequency? The rationale behind these important decisions should be made explicit.

      We now explain this in the Introduction as follows:

      ‘We chose a primary flicker frequency of 2Hz as a compromise between the low-pass pupil response (see Barrionuevo et al., 2014; Spitschan et al., 2014), and the relatively higher-pass EEG response (Regan, 1966).’

      It is a compromise frequency that is not optimal for either modality, but generates a measurable signal for both. The choice of 1.6 Hz was for similar reasons - for a 10-second trial it is four frequency bins away from the primary frequency, so can be unambiguously isolated in the spectrum.

      By the way, I wondered if we know what happens when you present the same flicker frequencies to the two eyes but in counter-phase. The average luminance seen binocularly would always be the same, so if the pupil system is linear, there should be no pupil response to this stimulus. An experiment like this has been done by Flitcroft et al (1992) on accommodation where the two eyes are presented stimuli moving oppositely in optical distance and indeed there was no accommodative response, which strongly suggests linearity.

      We have not tried this yet, but it’s on our to-do list for future work. The accommodation work is very interesting, and we now cite it in the manuscript as follows:

      ‘Work on the accommodative response indicates that binocular combination there is approximately linear (Flitcroft et al. 1992), and can even cancel when signals are in antiphase (we did not try this configuration here).’

      Figures 1 and 2 are important figures because they show the pupil and EEG results, respectively. But it's really hard to get your head around what's being shown in the lower row of each figure. The labeling for the conditions is one problem. You have to remember how "binocular" in panel c differs from "binocular cross" in panel d. And how "monocular" in panel d is different than "monocular 1.6Hz" in panel e. Additionally, the colors of the data symbols are not very distinct so it makes it hard to determine which one is which condition. These results are interesting. But they are difficult to digest.

      We hope that the new Figure 1 outlining the conditions has helped with interpretation here.

      The authors make a strong claim that they have found substantial differences in binocular interaction between cortical and sub-cortical circuits. But when I look at Figures 1 and 2, which are meant to convey this conclusion, I'm struck by how similar the results are. If the authors want to continue to make their claim, they need to spend more time making the case.

      Indeed, it is hard to make direct comparisons across figures - this is why Figure 4 plots the ratio of binocular to monocular conditions, and shows a clear divergence between the EEG and pupillometry results at high contrasts.

      Figure 5 is thankfully easy to understand and shows a very clear result. These perceptual results deviate dramatically from the essentially winner-take-all results for spatial sinewaves shown by Legge & Rubin (1981); whom they should cite by the way. Thus, very interestingly the binocular combination of temporal variation is quite different than the binocular combination of spatial variation. Can the pupil and EEG results also be plotted in the fashion of Figure 5? You'd pick a criterion pupil (or EEG) change and use it to make such plots.

      We now cite Legge & Rubin. We see what you mean about plotting the EEG and pupillometry results in the same coordinates as the matching data, but we don’t think this is especially informative as we would end up only with data points along the axes and diagonal of the plot, without the points at other angles. This is a consequence of how the experiments were conducted.

      My main suggestion is that the authors need to devote more space to explaining what they've done, what they've found, and how they interpret the data. I suggest therefore that they drop the computational model altogether so that they can concentrate on the experiments. The model could be presented in a future paper.

      We feel that the model is central to the understanding and interpretation of our results, and have retained it in the revised version of the paper.

      Reviewer #2 (Recommendations For The Authors):

      I found the terms for the stimulus conditions confusing. I think a simple schematic diagram of the conditions would help the reader.

      Now added (the new Fig 1).

      In reporting the binocular to monocular ratio, please clarify whether the monocular data was from one eye alone (and how that eye was chosen) or from both eyes and then averaged, or something else. It would be useful to plot the results from the dichoptic condition in this form, as well.

      These were averaged across both eyes. We now say in the Methods section:

      ‘We confirmed in additional analyses that the monocular consensual pupil response was complete, justifying our pooling of data across the eyes.’

      Also, clarify whether the term facilitation is used as above throughout (facilitation being > 2 times monocular response under binocular condition) or if a different criterion is being used. If we take facilitation to mean a ratio > 2, then facilitation depends on temporal frequency in Figure 4.

      We now explain our use of these terms in the final paragraph of the Introduction:

      ‘Relative to the response to a monocular signal, adding a signal in the other eye can either increase the response (facilitation) or reduce it (suppression).’

      The magnitude of explicit facilitation attained is interesting, but not without precedent. Ratios of binocular to mean monocular > 2, have been reported previously and values of summation depend strongly on the stimulus used (see for example Apkarian et al., EEG Journal, 1981, Nicol et al., Doc Ophthal, 2011).

      We now mention this in the Discussion as follows:

      ‘(however we note that facilitation as substantial as ours has been reported in previous EEG work by Apkarian et al. (1981))’

      In Experiment 3, the authors say that the psychophysical matching results are consistent with the approximately linear summation effects observed in the EEG data of Experiment 1. In describing Fig. 3, the claim is that the EEG is non-linear, e.g. super-additive - at least at high contrasts. Please reconcile these statements.

      We think that the ‘superadditive’ effects are close enough to linear that we don’t want to make too much of a big deal about them - this could be measurement error, for example. So we use terms such as near-linear, or approximately linear, when referring to them throughout.

      Reviewer #3 (Recommendations For The Authors):

      Let me make some more specific comments using a page/paragraph/line format to indicate where in the text they're relevant.

      1/2 (middle)/3 from end. "In addition" seems out of place here.

      Removed.

      1/3/4. By "intensities" do you mean "contrasts"?

      Fixed.

      1/3/last. "... eyes'...".

      Fixed.

      2/5/3. By "one binocular disc", you mean into "one perceptually fused disc".

      Rewritten as: ‘to help with their perceptual fusion, giving the appearance of a single binocular disc’

      3/1/1. "calibrated" seems like the wrong word here. I think you're just changing the vergence angle to enable fusion, right?

      Now rewritten as: ‘Before each experiment, participants adjusted the angle of the stereoscope mirrors to achieve binocular fusion’

      3/1/1. "adjusting the angles...". And didn't changing the mirror angles affect the shapes of the discs in the retinal images?

      Perhaps very slightly, but this is well within the tolerance of the visual system to compensate for in the fused image, especially for such high contrast edges.

      3/3/5. "fixed contrast" is confusing here because it's still a flickering stimulus if I follow the text here. Reword.

      Now ‘fixed temporal contrast’

      3/4/1. It would be clearer to say "pupil tracker" rather than "eye tracker" because you're not really doing eye tracking.

      True, but the device is a commercial eye tracker, so this is the appropriate term regardless of what we are using it for.

      3/5/6. I'm getting lost here. "varying contrast levels" applies to the dichoptic stimulus, right?

      Yes, now reworded as ‘In the other interval, a target disc was displayed, flickering at different contrast levels on each trial, but with a fixed interocular contrast ratio across the block.’

      3/5/7. Understanding the "ratio of flicker amplitudes" is key to understanding what's going on here. More explanation would be helpful.

      Addressed in the above point.

      4/3/near end. Provide some explanation about why the Fourier approach is more robust to noise.

      Added ‘(which can make the phase and amplitude of a fitted sine wave unstable)’

      Figure 1. In panel a, explain what the numbers on the ordinate mean. What's zero, for example? Which direction is dilation? Same question for panel b. It's interesting in panel c that the response in one eye to 2Hz increases when the other eye sees 1.6Hz. Would be good to point that out in the text.

      Good idea about panel (a) - we have changed the y-axis to ‘Relative amplitude’ for clarity, and now note in the figure caption that ‘Negative values indicate constriction relative to baseline, and positive values indicate dilation.’ Panel (b) is absolute amplitude, so is unsigned. Panel (c) only contains 2Hz conditions, but there is some dichoptic suppression across the two frequencies in panels (d,e) - we now cover this in the text and include statistics.

      6/2/1. Make clear in the text that Figure 1c shows contrast response functions for the pupil.

      Now noted in the caption.

      Figure 3. I'm lost here. I feel like I should be able to construct this figure from Figures 1 and 2, but don't know how. More explanation is needed at least in the caption.

      Done. The caption now reads:

      ‘Ratio of binocular to monocular response for three data types. These were calculated by dividing the binocular response by the monocular response at each contrast level, using the data underlying Figures 2c, 3c and 3f. Each value is the average ratio across N=30 participants, and error bars indicate bootstrapped standard errors.’

      9/1/1-2. I didn't find the evidence supporting this statement compelling.

      We now point the reader to Figure 4 as a reminder of the evidence for this difference.

      9/1/6-9. You said this. But this kind of problem can be fixed by moving the methods sections as I suggested above.

      As mentioned, we feel that the results section flows better with the current structure.

      Figure 4. Make clear that this is EEG data.

      Now added to caption.

      Figure 5 caption. Infinite exponent in what equation?

      Now clarified as: ‘models involving linear combination (dotted) or a winner-take-all rule (dashed)’

      Figure 6. I hope this gets dropped. No one will understand how the model predictions were derived. And those who look at the data and model predictions will surely note (as the authors do) that they are rather different from one another.

      As noted above, we feel that the model is central to the paper and have retained this figure. We have also worked out how to correct the noise parameter in the model for the number of participants included in the coherent averaging, which fixes the discrepancy at low contrasts. The correspondence between the data and model in is now very good, and we have plotted the data points and curves in the same panels, which makes the figure less busy.

      12/1. Make clear in this paragraph that "visual cortex" is referring to EEG and perception results and that "subcortical" is referring to pupil. Explain clearly what "linear" would be and what the evidence for "non-linear" is.

      Good suggestion, we have added qualifiers linking to both methods. Also tidied up the language to make it clearer that we are talking about binocular combination specifically in terms of linearity, and spelled out the evidence for each point.

      12/2/6-9. Explain the Quaia et al results enough for the reader to know what reflexive eye movements were studied and how.

      We now specify that these eye movements are also known as the ‘ocular following response’ and were measured using scleral search coils.

      12/2/9-10. Same for Spitchan and Cajochen: more explanation.

      Added:

      “(melatonin is a hormone released by the pineal gland that regulates sleep; its production is suppressed by light exposure and can be measured from saliva assays)”

      12/3/2-3. Intriguing statements about optimally combining noisy signals, but explain this more. It won't be obvious to most readers.

      We have added some more explanation to this section.

      13/1. This is an interesting paragraph where the authors have a chance to discuss what would be most advantageous to the organism. They make the standard argument for perception, but basically punt on having an argument for the pupil.

      Indeed, we agree that this point is necessarily speculative, however we think it is interesting for the reader to consider.

      13/2/1. "Pupil size affects the ..." is more accurate.

      Fixed.

      13/2/2 from end. Which "two pathways"? Be clear.

      Changed to ‘the pupil and perceptual pathways’

    1. Author response:

      The following is the authors’ response to the original reviews.

      Reviewer #1 (Public review):

      (1) The mechanism by which STAMBPL1 mediates GRHL3 transcription through its interaction with FOXO1 is not sufficiently discussed, especially in relation to how STAMBPL1 regulates FOXO1. Some reported effects are modest.

      We appreciate the reviewer’s comments. In response, we have added a discussion on the potential mechanisms by which STAMPBL1 regulates FOXO1 transcriptional activity in Discussion, highlighted in red on page 18, lines 342 to 352. The specific reply content is as follows: “The transcriptional activity of FOXO1 is primarily regulated by its nucleocytoplasmic shuttling process (Van Der Heide, Hoekman et al. 2004). The PI3K/AKT pathway promotes the phosphorylation of FOXO1, resulting in the formation of a complex with members of the 14-3-3 family (including 14-3-3σ, 14-3-3ε, and 14-3-3ζ), which facilitates its export from the nucleus and inhibits its transcriptional activity (Huang and Tindall 2007, Tzivion, Dobson et al. 2011). It’s reported that TDAG51 prevents the binding of 14-3-3ζ to FOXO1 in the nucleus by interacting with FOXO1, thereby enhancing its transcriptional activity through increased accumulation within the nucleus (Park, Jeon et al. 2023). Our results indicate that the overexpression of STAMBPL1 and STAMBPL1-E292A did not affect the protein levels of FOXO1 (Fig.7E and Fig.S5E), but STAMBPL1 co-localizes with FOXO1 in the nucleus (Fig.7M) and interacts with it (Fig.7N and Fig.S5I-J). This suggests that STAMBPL1 enhances the transcriptional activity of FOXO1 on GRHL3 by interacting with nuclear FOXO1.” The result was added to Supplementary Figure 5 as Fig.S5E.

      Reviewer #2 (Public review):

      (1) A potential limitation of the study is the reliance on specific cellular and animal models, which may constrain the extrapolation of these findings to the broader spectrum of human TNBC biology. Furthermore, while the study provides evidence for a novel regulatory axis involving STAMBPL1, FOXO1, and GRHL3, the multifaceted nature of angiogenesis may implicate additional regulatory factors not exhaustively addressed in this research.

      We appreciate the valuable suggestions provided by the reviewer. In Discussion, we have added an in-depth discussion of the limitations of the study, as well as an analysis of the regulatory factors related to tumor angiogenesis, which highlighted in red on pages 20 to 21, lines 396 to 412. The relevant content added is as follows: “In this study, we utilized two triple-negative breast cancer cell lines, HCC1806 and HCC1937, along with human primary umbilical vein endothelial cells (HUVECs) and a nude mouse breast orthotopic transplantation tumor model to investigate the regulatory mechanism by which STAMBPL1 activates the GRHL3/HIF1α/VEGFA signaling pathway through its interaction with FOXO1, thereby promoting angiogenesis in TNBC. The results of this study have certain limitations regarding their applicability to human TNBC biology. Furthermore, in addition to the HIF1α/VEGFA signaling pathway emphasized in this study, tumor cells can continuously release or upregulate various pro-angiogenic factors, such as Angiopoietin and FGF, which activate endothelial cells, pericytes (PCs), cancer-associated fibroblasts (CAFs), endothelial progenitor cells (EPCs), and immune cells (ICs). This leads to capillary dilation, basement membrane disruption, extracellular matrix remodeling, pericyte detachment, and endothelial cell differentiation, thereby sustaining a highly active state of angiogenesis (Liu, Chen et al. 2023). It is important to collect clinical TNBC tissue samples in the future to analyze the expression of the STAMBPL1/FOXO1/GRHL3/HIF1α/VEGFA signaling axis. Furthermore, patient-derived organoid and xenograft models are useful to elucidate the regulatory relationship of this axis in TNBC angiogenesis”

      Reviewer #3 (Public review):

      The main weaknesses of this work are that the relevance of this molecular axis to the pathogenesis of TNBC is not clear, and it is not clearly established whether this is a regulatory pathway that occurs in hypoxic conditions or independently of oxygen levels.

      (1) With respect to the first point, both FOXO1 and GRHL3 have been previously described as tumor suppressors, with reports of FOXO1 inhibiting tumor angiogenesis. Therefore, this works describes an apparently contradictory function of these proteins in TNBC. While it is not surprising that the same genes perform divergent functions in different tumor contexts, a stronger evidence in support of the oncogenic function of these two genes should be provided to make the data more convincing. As an example, the data in support of high STAMBPL1, FOXO and GRHL3 gene expression in TNBC TCGA specimens provided in Figure 8 is not very strong and it is not clear what the non-TNBC specimens are (whether other breast cancers or other tumors, perhaps those tumors whether these genes perform tumor suppressive functions). To strengthen the notion that STAMBPL1, FOXO and GRHL3 are overexpressed in TNCB, the authors could provide a comparison with normal tissue, as well as the analysis of other publicly available datasets (like the NCI Clinical Proteomic Tumor Analysis Consortium as an example). Finally, is it not clear what are the basal protein expression levels of STAMBPL1 in the cell lines used in this study, as based on the data presented in Figures 2D and F it appears that the protein is not expressed if not exogenously overexpressed. It would be helpful if the authors addressed this issue and provided further evidence of STAMBPL1 expression in TNBC cell lines.

      We appreciate the suggestions. In this study, we utilized the BCIP online tool to analyze the Metabric database, incorporating adjacent normal tissues as controls. Although the expression levels of STAMBPL1, FOXO1, and GRHL3 in breast cancer tissues are not uniformly higher than those in adjacent tissues, their expression levels in triple-negative breast cancer (TNBC) are significantly elevated compared to non-TNBC. The results of this re-analysis have been added in Supplementary Figure 6 as Fig.S6A-C.

      About the question of the basal protein expression levels of STAMBPL1 in the cell lines used in this study, our response is that Fig. 2A showed the endogenous level of STAMBPL1 in HCC1806 and HCC1937. For Fig. 2D and 2F, the overexpressed STAMBPL1 was fused with a 3xFlag tag, resulting in a higher molecular weight compared to the endogenous STAMBPL1. In the revised Figure 2, we have indicated the positions of the endogenous (Endo.) and exogenous (OE.) STAMBPL1 bands with arrows.

      (2) Linked to these considerations is the second major criticism, namely that it is not made clear if this new regulatory axis is proposed to act in normoxic or hypoxic conditions. The experiments presented in this paper are performed in both conditions but a clear explanation as to why cells are exposed to hypoxia is not given and would be necessary being that HIF-1a transcription and not protein stability is being analyzed. Also, different hypoxic conditions are sometimes used, resulting in different mRNA levels of HIF-1a and its downstream targets and quite significant fluctuations within the same cell line from one experimental setting to the next. The authors should provide an explanation as to why experimental conditions are changed and, more importantly, the experiments presented in Figure 2 should be performed also in normoxia.

      Thanks for the comments. Under normoxic conditions, HIF1α is recognized by pVHL due to hydroxylation and is rapidly degraded via the proteasomal pathway. In contrast, under hypoxic conditions, HIF1α protein is accumulated. To investigate the effect of STAMBPL1 knockdown on HIF1A gene transcription levels, we conducted experiments under hypoxic conditions to avoid interference from the rapid degradation of HIF1α at the protein level, as shown in Figures 2B-C. Furthermore, under normoxic conditions, the overexpression of STAMBPL1 had been demonstrated to significantly enhance the protein levels of HIF1α and upregulate the transcription of VEGFA through HIF1α. To avoid the potential impact of excessive accumulation of HIF1α protein under hypoxic conditions on its protein level detection and the transcription of downstream VEGFA, the related experiments shown in Figure 2D-G were performed under normoxic conditions. We have explained the corresponding experimental conditions in the “Result” and “Figure legends” according to the reviewer's comments, highlighted in red.

      (3) Another critical point is that necessary experimental controls are sometimes missing, and this is reducing the strength of some of the conclusions enunciated by the authors. As examples, experiments where overexpression of STAMBPL1 is coupled to silencing of FOXO1 to demonstrate dependency lack FOXO1 silencing the absence of STAMBPL1 overexpression. Because diminishing FOXO1 expression affects HIF-1a/VEGF transcription even in the absence of STAMBPL1 (shown in Figure 7C, D), it is not clear if the data presented in Figure 7G are significant. The difference between HIF-1a expression upon FOXO1 silencing should be compared in the presence or absence of STAMBPL1 overexpression to understand if FOXO1 impacts HIF-1a transcription dependently or independently of STAMBPL1.

      Thank you for this comment. For Fig.7G-H, our experimental objective was to determine whether the activation of HIF1A/VEGFA transcription by STAMBPL1 via FOXO1. Therefore, under STAMBPL1 overexpression, we knocked down FOXO1 to investigate whether FOXO1 silencing could reverse the upregulation of HIF1A/VEGFA transcription induced by STAMBPL1 overexpression.

      (4) In addition, some minor comments to improve the quality of this manuscript are provided.

      (4.1) As a general statement, the manuscript is extremely synthetic. While this is not necessarily a negative feature, sometimes results are discussed in the figure legends and not in the main text (as an example, western blots showing HIF-1a expression) and this makes it hard to read thought the data in an easy and enjoyable manner.

      Thank you for this suggestion. We have revised the figure legends to make them clearer and more concise, highlighted in red.

      (4.2) The effect of STAMBPL1 overexpression on HIF-1a transcription is minor (Figure 2) The authors should explain why they think this is the case and whether hypoxia may provide a molecular environment that is more permissive to this type of regulation.

      Thank you for the comment. Under normoxic conditions, we conducted WB to examine the protein expression of HIF1α after the overexpression of STAMBPL1 and the knockdown of HIF1α. To visually illustrate the impact of STAMBPL1 overexpression on HIF1A protein levels, as well as the effectiveness of HIF1α knockdown, we annotated the grayscale analysis results of the bands in Figures 2D and 2F. As the reviewer pointed out, under normoxic conditions, HIF1α is rapidly degraded, which may explain why the upregulation of HIF1α protein levels by STAMBPL1 overexpression is not very pronounced.

      (4.3) HIF-1a does not appear upregulated at the protein level protein by STAMBPL1 or GRLH3 overexpression, even though this is stated in the legends of Figures 2 and 6. The authors should show unsaturated western blots images and provide quantitative data of independent experiments to make this point.

      Thank you for this comment. We have added the unsaturated image of HIF1α into Fig.2D, and performed a grayscale analysis of the HIF1α bands in Fig.2D and Fig.6A to indicate the relative protein level of HIF1α.

      Reviewer #1 (Recommendations for the authors):

      (1) The authors previously reported that STAMBPL1 stabilizes MKP1 in TNBC. However, in this study, they focus on HIF1a. Given that STAMBPL1 affects HIF1a expression, it would be valuable to examine the levels of ROS in TNBC cells with or without STAMBPL1, as ROS is known to influence HIF1a stability.

      Thank you for your comments. It’s known that STAMBPL1 functions as a deubiquitinating enzyme. However, our study reveals that the upregulation of HIF1α by STAMBPL1 is independent of its deubiquitinating activity. This conclusion is supported by the observation that overexpression of the deubiquitinase active site mutant, STAMBPL1-E292A, also upregulated HIF1α expression (Figure 1F). Moreover, STAMBPL1 overexpression enhanced HIF1α transcription (Figures 4E and S3E), while STAMBPL1 knockdown was able to inhibit the transcription of HIF1α (Figures 2B-C). These results indicate that STAMBPL1 mediates the transcription of HIF1α but does not affect the stability of HIF1α. For these reasons, we think that it is unnecessary to examine the ROS levels.

      (2) Figure 1A: The regulation of HIF1a mRNA by STAMBPL1, but not its protein levels, could be better addressed by using MG132 to rule out the impact of protein degradation.

      Thanks for this comment. Under normoxic conditions, the oxygen-sensitive prolyl hydroxylases PHD1-3 act on HIF1α, specifically inducing hydroxylation at the proline 402 and 564 residues. These hydroxylated residues are recognized by the pVHL/E3 ubiquitin ligase complex, leading to ubiquitination and subsequent degradation via the proteasome pathway. Conversely, under hypoxic conditions, PHD1-3 are inactivated, and non-hydroxylated HIF1α is not recognized by the pVHL/E3 ubiquitin ligase complex, thereby avoiding ubiquitination and proteasomal degradation (DOI: 10.1073/pnas.95.14.7987, DOI: 10.1515/BC.2004.016, and DOI: 10.1042/BJ20040620). The mechanism of HIF1α accumulation under hypoxia is analogous to the action of the proteasome inhibitor MG132. When we treated cells with hypoxia, the ubiquitination and proteasomal degradation pathway of HIF1α was blocked. At this time, STAMBPL1 knockdown could downregulate the expression of HIF1α (Fig.1A). Meanwhile, since the knockdown of STAMBPL1 significantly downregulated the mRNA level of HIF1α under hypoxia (Fig.2B-C), we concluded that STAMBPL1 affects the expression of HIF1α by mediating its transcription. In addition, MG132 will block all proteasomal substrate degradation and may affect HIF1α mRNA levels indirectly.

      (3) Figure 2D and 2F: The effect of STAMBPL1 in promoting HIF1a expression is quite mild, and the effect of HIF1a knockdown is also modest. Given the high levels of STAMBPL1 in TNBC cell lines (Figure 2A), it would be better to repeat these experiments in a STAMBPL1-knockdown setting for clearer insights.

      We appreciate this insightful suggestion. Considering that the regulation of HIF1α expression by STAMBPL1 occurs at the transcriptional level, and to prevent excessive accumulation of HIF1a during hypoxia that could confound the effect of STAMBPL1 overexpression on HIF1α regulation, we opted to overexpress STAMBPL1 under normoxic conditions and subsequently knock down HIF1α, as shown in Fig.2D and Fig.2F. This approach allowed us to observe that STAMBPL1 overexpression can upregulate HIF1a expression to some extent. Additionally, in response to the reviewer's suggestion to knock down STAMBPL1, we have conducted the corresponding experiments, with results presented in Fig.1A-E and Fig.2B-C.

      (4) Figure 4A: Why does the RNA-seq pattern differ significantly between the two siRNAs? Additionally, the authors should clarify why they focus primarily on transcription factors, as other mechanisms, such as mRNA stability and RNA modification, could also influence gene transcription.

      Thank you for this comment. Two siRNAs for STAMBPL1 were designed and synthesized by a biotechnology company. Although both siRNAs target STAMBPL1, they target different sequences. While both siRNAs effectively knocked down STAMBPL1 (Fig. 1A and Fig. 2A), the possibility of off-target effects cannot be completely ruled out. Therefore, we needed to use two siRNAs simultaneously for RNA-seq, ensuring that the gene expression changes observed are due to the knockdown of STAMBPL1 by focusing on genes downregulated by both two siRNAs. Additionally, among the 27 genes downregulated by both two siRNAs, only 18 genes were annotated. Of these 18 genes, except for GRHL3, which is a transcription factor reported to be involved in gene transcription regulation, the remaining 17 genes have no documented association with RNA transcription, stability, or modification. Therefore, we focused on the GRHL3 gene.

      (5) Figure 5G: To investigate whether STAMBPL1 and GRHL3 function epistatically in the pathway, a double knockdown of STAMBPL1 and GRHL3 should be examined. Additionally, a double knockdown of STAMBPL1 and FOXO1 should be assessed.

      Thank you for your comment. In Figure 5G, we aimed to assess the knockdown efficiency of GRHL3 using siRNAs. To determine whether STAMBPL1 upregulates the HIF1a/VEGFA axis via GRHL3, we overexpressed STAMBPL1 and subsequently knocked down GRHL3. Our findings indicated that STAMBPL1 overexpression indeed enhanced the HIF1a/VEGFA axis, which was rescued by the knockdown of GRHL3, as shown in Figures 4E-F and S3E-F. Similarly, upon overexpressing STAMBPL1 and knocking down FOXO1, we observed that STAMBPL1 overexpression increased the GRHL3/HIF1a/VEGFA axis, which could also be rescued by knocking down FOXO1, as shown in Figures 7F-H. These results suggest that STAMBPL1 upregulates the GRHL3/HIF1a/VEGFA axis through FOXO1. We do not think it is a right way to double knock down STAMBPL1 and FOXO1 or GRHL3.

      (6) Figure 7: It remains unclear how STAMBPL1 regulates FOXO1. The authors show that STAMBPL1 increases the transcriptional activation of FOXO1 at the GRHL3 promoter, but it is not clear if STAMBPL1 is required for FOXO1 binding to the GRHL3 promoter. To address this, STAMBPL1-knockdown should be included to examine its effect on FOXO1 binding to the GRHL3 promoter. Furthermore, it would be important to determine whether the STAMBPL1-FOXO1 interaction is essential for GRHL3 transcription. Since the interaction sites of STAMBPL1-FOXO1 have been mapped, a mutant disrupting the interaction would provide better insight into how STAMBPL1 promotes GRHL3 transcription by interacting with FOXO1.

      Thank you for this comment. It has been reported that FOXO1 promotes the transcription of the GRHL3 gene by interacting with its promoter (DOI: 10.1093/nar/gkw1276). We also verified through ChIP assay that FOXO1 can bind to the promoter of GRHL3 gene (Fig.7I) and mediate its transcription. Specifically, knocking down FOXO1 significantly down-regulated the mRNA level of GRHL3 (Fig.7B), and the GRHL3 promoter lacking FOXO1 binding site almost completely lost transcriptional activity (Fig.7J), indicating that FOXO1 is crucial for the transcriptional activity of the GRHL3 promoter. Overexpression of STAMBPL1 enhances the activating effect of FOXO1 on the transcriptional activity of the GRHL3 promoter (Fig.7K). However, the up-regulation of GRHL3 transcription by overexpression of STAMBPL1 is completely blocked by FOXO1 knockdown (Fig.7F), and the knockdown of FOXO1 essentially blocks the binding of STAMBPL1 to the GRHL3 promoter (Fig.7L), suggesting that STAMBPL1 affects the transcriptional expression of GRHL3 based on FOXO1. As we added in Discussion, the transcription factor activity of FOXO1 is mainly regulated by its nucleoplasm shuttling process, and the accumulation of FOXO1 in nucleus can enhance its transcription factor activity (DOI: 10.1042/BJ20040167; DOI: 10.15252/embj.2022111867). In our research, neither STAMBPL1 nor its mutant of deubiquitinating enzyme site affected the expression of FOXO1 (Fig.S5E), but STAMBPL1 and FOXO1 co-located in the nucleus (Fig.7M), and they interacted with each other (Fig.7N, Fig.S5I-J). Therefore, we speculate that STAMBPL1 interacts with FOXO1 in the nucleus, obstructs the binding of FOXO1 with the members of 14-3-3 family, inhibits the export of FOXO1, thereby enhancing its transcriptional activity. This interaction between STAMBPL1 and FOXO1 does not necessarily affect the binding of FOXO1 with DNA, including the GRHL3 promoter.

      (7) Figure 8 A-C: What is the correlation among the expressions of STAMBPL1, FOXO1, and GRHL3 in TNBC tumors compared to non-TNBC tumors?

      Thank you for your comment. In Figure 8A-C, we analyzed the expression levels of STAMBPL1, FOXO1, and GRHL3 in both TNBC and non-TNBC samples using the BCIP. The results indicate that the expression levels of these three genes are significantly higher in TNBC compared to non-TNBC samples. To investigate the correlation among the expressions of STAMBPL1, FOXO1, and GRHL3 in TNBC versus non-TNBC, we further utilized the Metabric data. Besides the positive correlation trend between STAMBPL1 and GRHL3 expression in TNBC clinical samples (Pearson R = 0.27), no significant correlation was observed in the expression levels of STAMBPL1, FOXO1, and GRHL3 in TNBC and non-TNBC clinical samples (as shown in Author response image 1 below). Since STAMBPL1 and FOXO1 are involved as protein molecules in the transcriptional regulation of GRHL3 gene, and the data obtained from the Metabric database are the transcriptional levels of these three genes, this might be the reason why the correlation between their expressions was not observed.

      Author response image 1.

      Reviewer #2 (Recommendations for the authors):

      The authors have thoroughly elucidated the role of STAMBPL1 in TNBC. However, it would be beneficial to discuss the potential clinical implications of these findings, such as how targeting STAMBPL1 or FOXO1 might impact current treatment strategies for TNBC. However, several issues need to be addressed.

      Major:

      (1) While the study provides an exhaustive analysis of the molecular mechanisms, a comparison with other subtypes of breast cancer could enhance our understanding of the specificity of the STAMBPL1/FOXO1/GRHL3/HIF1α/VEGFA axis in TNBC.

      Thank you for your comment. According to report, STAMBPL1 is significantly associated with the mesenchymal characteristics of breast cancer (DOI: 10.1038/s41416-020-0972-x). We utilized cBioPortal (http://www.cbioportal.org/) to analyze the expression of STAMBPL1 across various clinical subtypes of breast cancer. The results indicated that STAMBPL1 is highly expressed in invasive breast cancer, which has been added to Supplementary Figure 6 as Fig.S6D. Given that TNBC is an aggressive type of invasive breast cancer, we further examined the expression of STAMBPL1 in TNBC compared to non-TNBC using BCIP (http://omicsnet.org/bcancer/database). Our findings revealed that the expression level of STAMBPL1 in TNBC was elevated relative to its levels in non-TNBC (Fig.8A). Additionally, since tumor angiogenesis is a critical factor influencing the metastasis of cancer cells, our study focused specifically on the pro-angiogenic effects of STAMBPL1 in TNBC.

      (2) The authors might consider discussing any potential off-target effects of the siRNA and shRNA used in the study to bolster the conclusions drawn from the knockdown experiments.

      We appreciate the reviewer's suggestion. It is well-known that siRNA or shRNA have off-target effects. To address this concern, we employed two siRNAs for each gene knockdown in our study. Specifically, we knocked down genes such as STAMBPL1, FOXO1, GRHL3, and HIF1A in two TNBC cell lines, HCC1806 and HCC1937, using two siRNAs. Except for siRNA#1 targeting HIF1A, which did not show a significant knockdown effect in HCC1806 cells (Fig.2D and Fig.6A), the knockdown effects of other siRNAs on their respective genes were effective, and the resulting phenotypes were consistent. As shown in Fig.2F and Fig.S4H, siRNA#1 targeting HIF1A had a significant knockdown effect in HCC1937 cells. The lower knockdown efficiency of this siRNA in HCC1806 cell line might be attributed to cell-specific factors.

      (3) It would be advantageous if the authors could provide further details on the patient demographics and tumor characteristics in the TCGA database analysis to better comprehend the clinical relevance of their findings.

      Thanks for the reviewer's suggestions. We have now indicated the number of clinical samples in each group in the legend of Fig.8A-C. Since we utilized the BCIP online database to analyze and compare the expression levels of the three genes STAMBPL1, FOXO1, and GRHL3 in TNBC and non-TNBC, we are unable to obtain more specific information regarding the tumor characteristics of each sample. However, our analysis clearly shows that the expression levels of these three genes are significantly higher in TNBC compared to non-TNBC.

      (4) The authors should consider discussing any limitations regarding the generalizability of their findings, such as potential variations among different TNBC subtypes or the specificity of their observations to certain stages of the disease.

      We appreciate the reviewer's comment. Accordingly, we have added a discussion on the limitation of this study in Discussion, highlighted in red font on pages 20 to 21, lines 396 to 412. In addition, we utilized the bc-GenExMiner online database to conduct a comparative analysis of STAMBPL1 expression in different subtypes of non-TNBC and TNBC. The result indicates that STAMBPL1 is highly expressed in mesenchymal-like and basal-like TNBC, which has been added into Supplementary Figure 6 as Fig.S6E. Since these two subtypes of TNBC are highly invasive and metastatic, it suggests that targeting the signaling pathway of STAMBPL1/FOXO1/GRHL3/HIF1α/VEGFA may offer clinical benefits for patients with invasive TNBC.

      Minor:

      The paper is generally well-written, but it's crucial to maintain vigilance for subject-verb agreement, proper use of tense, and consistent terminology.

      Thank you for this suggestion. We have thoroughly revised the article for issues such as grammar, including tense, subject-verb agreement, and terminology.

    1. Author response:

      The following is the authors’ response to the original reviews

      Reviewer #1 (Public review):

      Summary:

      This study aimed at replicating two previous findings that showed (1) a link between prediction tendencies and neural speech tracking, and (2) that eye movements track speech. The main findings were replicated which supports the robustness of these results. The authors also investigated interactions between prediction tendencies and ocular speech tracking, but the data did not reveal clear relationships. The authors propose a framework that integrates the findings of the study and proposes how eye movements and prediction tendencies shape perception.

      Strengths:

      This is a well-written paper that addresses interesting research questions, bringing together two subfields that are usually studied in separation: auditory speech and eye movements. The authors aimed at replicating findings from two of their previous studies, which was overall successful and speaks for the robustness of the findings. The overall approach is convincing, methods and analyses appear to be thorough, and results are compelling.

      Weaknesses:

      Linking the new to the previous studies could have been done in more detail, and the extent to which results were replicated could have been discussed more thoroughly.

      Eye movement behavior could have been presented in more detail and the authors could have attempted to understand whether there is a particular component in eye movement behavior (e.g., microsaccades) that drives the observed effects.

      We would like to thank you for your time and effort in reviewing our work and we appreciate the positive comments!

      We extended our manuscript, now providing intermediate results on individual prediction tendency, which can be compared to our results from Schubert et al., (2023).

      Furthermore, we expanded our discussion now detailing the extent to which our results (do not) replicate the previous findings (e.g. differences in horizontal vs. vertical ocular speech tracking, lack of distractor tracking, link between ocular speech tracking and behavioral outcomes).

      While we agree with the reviewer that it is an important and most interesting question, to what extent individual features of gaze behavior (such as microsaccades, blinks etc.) contribute to the ocular speech tracking effect, it is beyond the scope of the current manuscript. It will be methodologically and conceptually challenging to distinguish these features from one another and to relate them to diverse cognitive processes. We believe that a separate manuscript is needed to give these difficult questions sufficient space for new methodological approaches and control analyses. The primary goal of this manuscript was to replicate the findings of Gehmacher et al. (2024) using similar methods and to relate them to prediction tendencies, attention, and neural speech tracking. 

      Reviewer #2 (Public review):

      Summary

      Schubert et al. recorded MEG and eye-tracking activity while participants were listening to stories in single-speaker or multi-speaker speech. In a separate task, MEG was recorded while the same participants were listening to four types of pure tones in either structured (75% predictable) or random (25%) sequences. The MEG data from this task was used to quantify individual 'prediction tendency': the amount by which the neural signal is modulated by whether or not a repeated tone was (un)predictable, given the context. In a replication of earlier work, this prediction tendency was found to correlate with 'neural speech tracking' during the main task. Neural speech tracking is quantified as the multivariate relationship between MEG activity and speech amplitude envelope. Prediction tendency did not correlate with 'ocular speech tracking' during the main task. Neural speech tracking was further modulated by local semantic violations in the speech material, and by whether or not a distracting speaker was present. The authors suggest that part of the neural speech tracking is mediated by ocular speech tracking. Story comprehension was negatively related to ocular speech tracking.

      Strengths

      This is an ambitious study, and the authors' attempt to integrate the many reported findings related to prediction and attention in one framework is laudable. The data acquisition and analyses appear to be done with great attention to methodological detail (perhaps even with too much focus on detail-see below). Furthermore, the experimental paradigm used is more naturalistic than was previously done in similar setups (i.e. stories instead of sentences).

      Weaknesses

      For many of the key variables and analysis choices (e.g. neural/ocular speech tracking, prediction tendency, mediation) it is not directly clear how these relate to the theoretical entities under study, and why they were quantified in this particular way. Relatedly, while the analysis pipeline is outlined in much detail, an overarching rationale and important intermediate results are often missing, which makes it difficult to judge the strength of the evidence presented. Furthermore, some analysis choices appear rather ad-hoc and should be made uniform and/or better motivated.

      We would like to thank you very much for supporting our paper and your thoughtful feedback!

      To address your concerns, that our theoretical entities as well as some of our analytical choices lack transparency, we expanded our manuscript in several ways:

      (1) We now provide the intermediate results of our prediction tendency analysis (see new Figure 2 of our manuscript). These results are comparable to our findings from Schubert et al. (2023), demonstrating that on a group level there is a tendency to pre-activate auditory stimuli of high probability and illustrating the distribution of this tendency value in our subject population.

      (2) We expanded our methods section in order to explain our analytical choices (e.g. why this particular entropy modulation paradigm was used to measure individual prediction tendency).

      (3) We now provide an operationalisation of the terms “neural speech tracking” and “ocular speech tracking” at their first mention, to make these metrics more transparent to the reader.

      (4) We are summarizing important methodological information ahead of each results section, in order to provide the reader with a comprehensible background, without the necessity to read through the detailed methods section. 

      (5) We expanded our discussion section, with a special emphasis on relating the key variables of the current investigation to theoretical entities.

      Reviewer #3 (Public review):

      Summary:

      In this paper, the authors measured neural activity (using MEG) and eye gaze while individuals listened to speech from either one or two speakers, which sometimes contained semantic incongruencies.

      The stated aim is to replicate two previous findings by this group: (1) that there is "ocular speech tracking" (that eye-movements track the audio of the speech), (2) that individual differences in neural response to tones that are predictable vs. not-predictable in their pitch is linked to neural response to speech. In addition, here they try to link the above two effects to each other, and to link "attention, prediction, and active sensing".

      Strengths:

      This is an ambitious project, that tackles an important issue and combines different sources of data (neural data, eye-movements, individual differences in another task) in order to obtain a comprehensive "model" of the involvement of eye-movements in sensory processing.

      The authors use many adequate methods and sophisticated data-analysis tools (including MEG source analysis and multivariate statistical models) in order to achieve this.

      Weaknesses:

      Although I sympathize with the goal of the paper and agree that this is an interesting and important theoretical avenue to pursue, I am unfortunately not convinced by the results and find that many of the claims are very weakly substantiated in the actual data.

      Since most of the analyses presented here are derivations of statistical models and very little actual data is presented, I found it very difficult to assess the reliability and validity of the results, as they currently stand. I would be happy to see a thoroughly revised version, where much more of the data is presented, as well as control analyses and rigorous and well-documented statistical testing (including addressing multiple comparisons).

      We thank you for your thoughtful feedback. We appreciate your concerns and will address them below in greater detail.

      These are the main points of concern that I have regarding the paper, in its current format.

      (1) Prediction tendencies - assessed by listening to sequences of rhythmic tones, where the pitch was either "predictable" (i.e., followed a fixed pattern, with 25% repetition) or "unpredictable" (no particular order to the sounds). This is a very specific type of prediction, which is a general term that can operate along many different dimensions. Why was this specific design selected? Is there theoretical reason to believe that this type of prediction is also relevant to "semantic" predictions or other predictive aspects of speech processing?

      Theoretical assumptions and limitations of our quantification of individual prediction tendency are now shortly summarized in the first paragraph of our discussion section. With this paradigm we focus on anticipatory “top-down” predictions, whilst controlling for possibly confounding “bottom-up” processes. Since this study aimed to replicated our previous work we chose the same entropy-modulation paradigm as in other studies from our group (e.g. Demarchi et al. 2019, Schubert et al. 2023;2024, Reisinger et al. 2024), which has proven to give reproducible findings of feature-specific preactivations of sounds in a context of low entropy. One advantage of this design is that it gives us the opportunity to directly compare the processing of “predictable” and “unpredictable” sounds of the same frequency in a time-resolved manner (this argument is now also included in the Methods section).

      Regarding the question to what extent this type of prediction might also be relevant to “semantic” predictions we would like to refer to our previous study (Schubert et al., 2023), where we explicitly looked at the interaction between individual prediction tendency and encoding of semantic violations in the cortex. (In short, there we found a spatially dissociable interaction effect, indicating an increased encoding of semantic violations that scales with prediction tendency in the left hemisphere, as well as a disrupted encoding of semantic violations for individuals with stronger prediction tendency in the right hemisphere.) We did not aim to replicate all our findings in the current study, but instead we focused on merging the most important results from two independent phenomena in the domain of speech processing and bringing them into a common framework. However, as now stated in our discussion, we believe that “predictions are directly linked to the interpretation of sensory information. This interpretation is likely to occur at different levels along the cognitive (and anatomical) hierarchy…” and that “this type of prediction is relevant for acoustic processing such as speech and music, whose predictability unfolds over time.”

      (2) On the same point - I was disappointed that the results of "prediction tendencies" were not reported in full, but only used later on to assess correlations with other metrics. Even though this is a "replication" of previous work, one would like to fully understand the results from this independent study. On that note, I would also appreciate a more detailed explanation of the method used to derive the "prediction tendency" metric (e.g, what portion of the MEG signal is used? Why use a pre-stimulus and not a post-stimulus time window? How is the response affected by the 3Hz steady-state response that it is riding on? How are signals integrated across channels? Can we get a sense of what this "tendency" looks like in the actual neural signal, rather than just a single number derived per participant (an illustration is provided in Figure 1, but it would be nice to see the actual data)? How is this measure verified statistically? What is its distribution across the sample? Ideally, we would want enough information for others to be able to replicate this finding).

      We now included a new figure (similar to Schubert et al. 2023) showing the interim results of the “prediction tendency” effect as well as individual prediction tendency values of all subjects.

      Furthermore we expanded the description of the “prediction tendency” metric in the Methods section, where we explain our analytical choices in more detail. In particular we used a pre-stimulus time window in order to capture “anticipatory predictions”. The temporally predictably design gives us the opportunity to capture this type of predictions. The integration across channels is handled by the multivariate pattern analysis (MVPA), which inherently integrates multidimensional data (as mentioned in the methods section we used data from 102 magnetometers) and links it to (in this case) categorical information.

      (3) Semantic violations - half the nouns ending sentences were replaced to create incongruent endings. Can you provide more detail about this - e.g., how were the words selected? How were the recordings matched (e.g., could they be detected due to audio editing?)? What are the "lexically identical controls that are mentioned"? Also, is there any behavioral data to know how this affected listeners? Having so many incongruent sentences might be annoying/change the nature of listening. Were they told in advance about these?

      We expanded the Methods section and included the missing information: 

      “We randomly selected half of the nouns that ended a sentence (N = 79) and replaced them with the other half to induce unexpected semantic violations. The swap of nouns happened in the written script before the audio material was recorded in order to avoid any effects of audio clipping. Narrators were aware of the semantic violations and had been instructed to read out the words as normal. Consequently all target words occurred twice in the text, once in a natural context (serving as lexical controls) and once in a mismatched context (serving as semantic violations) within each trial, resulting in two sets of lexically identical words that differed greatly in their contextual probabilities (see Figure 1F for an example). Participants were unaware of these semantic violations.” Since we only replaced 79 words with semantic violations in a total of ~ 24 minutes of audio material we believe that natural listening was not impaired. In fact none of the participants mentioned to have noticed the semantic violations during debriefing (even though they had an effect on speech tracking in the brain). 

      (4) TRF in multi-speaker condition: was a univariate or multivariate model used? Since the single-speaker condition only contains one speech stimulus - can we know if univariate and multivariate models are directly comparable (in terms of variance explained)? Was any comparison to permutations done for this analysis to assess noise/chance levels?

      For mTRF models it depends on the direction (“encoding” vs. “decoding”) whether or not the model is comparable to a univariate model. In our case of an encoding model the TRFs are fitted to each MEG channel independently. This gives us the possibility to explore the effect over different areas (whereas a multivariate “decoding” model would result in only one speech reconstruction value).

      In both conditions (single and multi speaker) a single input feature (the envelope of the attended speech stream) was used. Of course it would be possible to fit the model to use a multivariate encoding model, predicting the brain’s response to the total input of sounds. This would, however, target a slightly different question than ours as we aimed to investigate how much of the attended speech is tracked.

      Regarding your suggestion of a comparison to permutations to assess noise levels we would like to point out that we chose the same methodological approach as in our previous studies, that we aimed to replicate here. Indeed in these original studies no permuted versions (with exception of the mediation analysis where comparing a model with an additional input predictor to a single predictor model would not result in a fair comparison) have been used. We conducted the mTRF approach considering the guidelines of Crosse et al. (2016) to the best of our knowledge and in accordance with similar studies in this field.

      Crosse, M. J., Di Liberto, G. M., Bednar, A., & Lalor, E. C. (2016). The multivariate temporal response function (mTRF) toolbox: a MATLAB toolbox for relating neural signals to continuous stimuli. Frontiers in human neuroscience, 10, 604.

      (5) TRF analysis at the word level: from my experience, 2-second segments are insufficient for deriving meaningful TRFs (see for example the recent work by Mesik & Wojtczak). Can you please give further details about how the analysis of the response to semantic violations was conducted? What was the model trained on (the full speech or just the 2-second long segments?) Is there a particular advantage to TRFs here, relative - say - to ERPs (one would expect a relatively nice N400 response, not)? In general, it would be nice to see the TRF results on their own (and not just the modulation effects).

      We fully agree with the reviewers statement that 2-second segments would have been too short to derive meaningful TRFs. To investigate the effect of semantic violations, we used the same TRFs trained on the whole dataset (with 4-fold cross validation). The resulting true as well as the predicted data was segmented into single word epochs of 2 seconds. We selected semantic violations as well as their lexically identical controls and correlated true with predicted responses for every word. Thus, we conducted the same analysis as for the overall encoding effect, focusing on only part of the data. We have reformulated the Methods section accordingly to clear up this misunderstanding. Since the TRFs are identical to the standard TRFs from the overall neural speech tracking, they are not informative to the semantic violation effect. However, since the mTRF approach is the key method throughout the manuscript (and our main focus is not on the investigations of brain responses to semantic violations) we have favoured this approach over the classical ERF analysis. 

      (6) Another related point that I did not quite understand - is the dependent measure used for the regression model "neural speech envelope tracking" the r-value derived just from the 2sec-long epochs? Or from the entire speech stimulus? The text mentions the "effect of neural speech tracking" - but it's not clear if this refers to the single-speaker vs. twospeaker conditions or to the prediction manipulation. Or is it different in the different analyses? Please spell out exactly what metric was used in each analysis.

      As suggested we now provide a clear definition of each dependent metric for each analysis.

      “Neural speech tracking” refers to the correlation coefficients between predicted and true brain responses from the aforementioned encoding model, trained and tested on the whole audio material within condition (single vs. multi-speaker).

      Recommendations for the authors:

      Reviewing Editor Comments:

      The reviewers have provided a number of recommendations to improve the manuscript, particularly requesting that more data be reported, with an emphasis on the measurements themselves (eye movements and TRFs) rather than just the numerical outputs of mathematical models.

      We appreciate all the reviewers' and editor’s comments and effort to improve our manuscript. In the revised version we provide interim findings and missing data, updated figures that include an intuitive illustration of the metrics (such as TRFs), and a thoroughly revised discussion section where we focus on the relationship between our observed quantities and theoretical entities. We now offer operationalized definitions of the relevant concepts (“prediction tendency”, “active ocular sensing” and “selective attention”) and suggest how these entities might be related in the context of speech processing, based on the current findings. We are confident that this revision has improved the quality of our paper a lot and we are grateful for all the feedback and suggestions. 

      Reviewer #1 (Recommendations for the authors):

      (1) Participants had to fixate throughout the tasks. How did the authors deal with large eye movements that violated the instructed fixation?

      As described in the Methods section: “Participants were instructed to look at a black fixation cross at the center of a grey screen.” This instruction was not intended to enforce strict fixation but rather to provide a general reference point, encouraging participants to keep their gaze on the grey screen and avoid freely scanning the room or closing their eyes. Unlike trial-based designs, where strict fixation is feasible due to shorter trial durations, this approach did not impose rigid fixation requirements. Consequently, the threshold for "instruction violation" was inherently more flexible, and no additional preprocessing was applied to the gaze vectors.

      Fixating for such an extended period of time (1.5 hours?) is hard. Did fixation behavior change over time? Could (fixation) fatigue affect the correlations between eye movements and speech tracking? For example, fatigued participants had to correct their fixation more often and this drives, in part, the negative correlation with comprehension?

      Yes, participants spent approximately 2 hours in the MEG, including preparation time (~30 minutes). However, participants were given opportunities to rest their eyes between different parts and blocks of the experiment (e.g., resting state, passive listening, and audiobook blocks), which should help mitigate fatigue to some extent.

      That said, we agree that it is an intriguing idea that fatigue could drive the ocular speech tracking effect, with participants potentially needing to correct their gaze more as the experiment progresses. However, our analysis suggests this is unlikely for several reasons:

      (1) Cross-validation in encoding models: Ocular speech tracking effects were calculated using a 4-fold cross-validation approach (this detail has now been added to the Methods section; please see our response to public review #3). This approach reduces the influence of potential increases in gaze corrections over time, as the models are trained and validated on independent data splits.  Moreover, if there were substantial differences in underlying response magnitudes between folds - for instance, between the first and fourth fold - this would likely compromise the TRF's ability to produce valid response functions for predicting the left-out data. Such a scenario would not result in significant tracking, further supporting the robustness of the observed effects.

      (2) TRF time-course stability: If fatigue were driving increased gaze corrections, we would expect this to be reflected in a general offset (capturing the mean difference between folds) in the TRF time-courses shown in Figure 4 (right panel). However, no such trend / offset is evident.

      (3) Comparison of eye movement data: To directly investigate this possibility, we compared the amount of total eye movements between the first and last blocks for both the single and multi-speaker conditions. Total movement was calculated by first calculating the differences in pixel values between consecutive eye positions on both the x- and y-axes. The Euclidean distance was then computed for each difference, providing a measure of movement between successive time points. Summing these distances yielded the total movement for each block. Statistical analysis was performed separately for the single speaker (ASS) and multi-speaker (AMS) conditions. For each condition, paired comparisons were made between the first and last blocks (we resorted to non-parametric tests, if assumptions of normality were violated):

      For the single speaker condition (ASS), the normality assumption was not satisfied (p≤0.05p, Kolmogorov-Smirnov test). Consequently, a Wilcoxon signedrank test was conducted, which revealed no significant difference in total movements between the first and last blocks (z=−1.330, p=0.184). For the multi-speaker condition (AMS), the data met the normality assumption (p>0.05), allowing the use of a paired t-test. The results showed no significant difference in total movements between the first and last blocks (t=−0.184, p=0.855).

      The results are visualized in a bar plot (see below), where individual data points are displayed alongside the mean and standard error for each block. Statistical annotations indicate that neither condition demonstrated significant differences between the blocks. These findings suggest that total eye movements remained stable across the experimental conditions, regardless of whether participants were exposed to a single or multiple speakers.

      Author response image 1.

      (4) Behavioral responses: Participants’ behavioral responses did not indicate any decrease in comprehensibility for later blocks compared to earlier ones. Specifically, a comparison of comprehension scores between the first and last blocks revealed no significant difference in either the single-speaker condition (ASS; Wilcoxon signed-rank test Z=−0.5911, p=0.5545) or the multi-speaker condition (AMS; Wilcoxon signed-rank test: Z=0.5018, p=0.6158). These findings suggest that participants maintained consistent levels of comprehension throughout the experiment, regardless of the condition or block order. The results are visualized in a bar plot (see below), where individual data points are displayed alongside the mean and standard error for each block. Statistical annotations indicate that neither condition demonstrated significant differences between the blocks.

      Author response image 2.

      Together, these factors suggest that fatigue is unlikely to be a significant driver of the ocular speech tracking effects observed in this study.

      (2) The authors should provide descriptive statistics of fixation behavior /fixational eye movements. What was the frequency and mean direction of microsaccades, do they follow the main sequence, etc., quantify drift and tremor?

      Thank you for their suggestion regarding descriptive statistics. To address this, we computed the rates of microsaccades (which were extracted using the microsaccade detection algorithm as proposed in Liu, B., Nobre, A. C. & van Ede, F. Functional but not obligatory link between microsaccades and neural modulation by covert spatial attention. Nat. Commun. 13, 3503 (2022)) and fixations as these metrics are directly relevant to our study and the requests above.

      Microsaccade Rates:

      - Single speaker Condition: Mean = 2.306 Hz, SD = 0.363 Hz. ○ Multi speaker: Mean = 2.268 Hz, SD = 0.355 Hz.

      Fixation Rates:

      - Single speaker Condition: Mean = 2.858 Hz, SD = 1.617 Hz. ○ Multi speaker Condition: Mean = 2.897 Hz, SD = 1.542 Hz.

      These values fall within the expected ranges reported in the literature (fixation rates: 2– 4 Hz, microsaccade rates: ~0.5–2.5 Hz) and serve as a sanity check, confirming the plausibility of our eye-tracking data. Regarding the reviewer’s request for additional metrics (e.g., microsaccade direction, main sequence analysis, drift, and tremor), extracting these features would require advanced algorithms and analyses not supported by our current preprocessing pipeline or dataset. We hope that the provided metrics, which were the main focus of this study, serve as a sufficient sanity check and highlight the robustness of our data.

      Related to this, I am wondering whether microsaccades are the feature that drives speech tracking.

      This is an important and pressing question that we aim to address in future publications. Currently, our understanding - and the reason microsaccades and blinks are not analysed in this manuscript - is limited by methodological constraints. Specifically, microsaccades are binary response vectors, which are not compatible with TRF analyses. Addressing this would require adapting future models to handle timecontinuous binary response data or exploring alternative approaches, such as regression-based ERFs (for example as in Heilbron et al. 2022). As the primary goal of this manuscript was to replicate the findings of Gehmacher et al. (2024) using similar methods and to integrate these findings into an initial unified framework, we did not investigate additional eye movement features here. However, we agree that microsaccades (and also blinks, see below) likely contribute, at least in part, to the observed ocular speech tracking effects, and we now suggest this in the Discussion:  

      “Relatedly, it remains an open question whether microsaccades are a key feature driving ocular speech tracking. However, our current study does not analyze microsaccades due to methodological constraints: microsaccades are binary response vectors, which are incompatible with TRF analyses used here. Addressing this would require adapting models to handle time-continuous binary response data or potentially exploring alternative approaches, such as regression-based ERFs (e.g., as in Heilbron et al., 2022). While these limitations preclude microsaccade analysis in the current study, we hypothesize that they could enhance temporal precision and selectively amplify relevant sensory input, supporting auditory perception. Future studies should explore this possibility to uncover the specific contributions of microsaccades to speech tracking.”

      (3) Can the authors make sure that interpolated blinks did not drive any of the effects? Can interpolated blink trials be excluded?

      Using continuous audiobooks as stimuli meant that we could not exclude blink periods from the analysis without introducing substantial continuation artifacts in the TRF analysis. Importantly, the concept of covert motor routines and active sensing suggests that participants engage more strongly in motor routines - including ocular behaviors such as microsaccades and blinks - during tasks like speech tracking. These motor routines are inherently tied to individual gaze patterns, making microsaccades and blinks correlated with other ocular behaviors. This complicates efforts to disentangle their individual contributions to the observed ocular speech tracking effects.

      Engagement in these motor routines, as posited by active sensing, would naturally load onto various viewing behaviors, further intertwining their roles.

      Even if we were to examine correlations, such as the amount of blinks with the ocular speech tracking effect, it is unlikely to provide a clearer understanding due to these inherent overlaps. The methodological and conceptual challenge lies in distinguishing these features from one another and understanding their respective roles in driving the observed effects.

      However, the aim of this manuscript was not to dissect the ocular speech tracking effect in greater detail, but rather to relate it - based on similar analytical choices as in Gehmacher et al - to prediction tendencies, attention, and neural speech tracking. While it will be crucial in future work to differentiate these patterns and their connections to diverse cognitive processes, it is beyond the scope of this study to address all these questions comprehensively.

      We acknowledge that eye movements, including microsaccades and blinks (however, see challenges for this in response 2), remain underexplored in many experimental paradigms. Their interplay with cognitive processes - such as attention, prediction, and sensory integration - will undoubtedly be an important focus for future studies. 

      (4) Could the authors provide more details on how time shuffling was done for the eyemovement predictor, and include a circularly shifted version (or a version that does not destroy temporal contiguity) in their model comparisons? Some types of shuffling can result in unrealistic time series, which would end up in an unfair comparison with the model that has the real eye movement traces as predictors.

      We thank the reviewer for their insightful question regarding the time-shuffling procedure for the eye-movement predictor and for suggesting the inclusion of a circularly shifted version in our model comparisons. Below, we provide further details about our approach and the rationale behind it:

      (1) Random Shuffling: In our analysis, the eye-movement predictor was randomly shuffled over time, meaning that individual samples were randomly replaced. This method completely disrupts the temporal structure of the signal, providing a null model that directly tests whether the temporal mediation observed is due to the specific temporal relationship between ocular movements and envelope tracking.

      (2) Circular Shifting: While circular shifting maintains temporal contiguity, it introduces certain challenges in the context of TRF analysis. Specifically:

      - Adaptation to Shifts: The TRF model could adapt to the introduced shift, potentially reducing the validity of the null comparison.

      - Similarity due to Repetition: The broadband envelope exhibits strong repetitive patterns over time, such as rhythms inherent to speech. Circular shifting can therefore produce predictors that are very similar to the original signal. As a result, this similarity may lead to null distributions that do not adequately disrupt the temporal mediation we aim to test, making it less robust as a control.

      (3) Rationale for Random Shuffling: The primary goal of our mediation analysis is to determine whether there is a temporal mediation of envelope tracking by ocular movements. By deliberately destroying the temporal structure through random shuffling, we ensure that the null model tests for the specific temporal relationship that is central to our hypothesis. Circularly shifted predictors, on the other hand, may partially preserve temporal dependencies, making them less suitable for this purpose.

      In summary, while circular shifting is a valuable approach in other contexts, it is less appropriate for the specific goals of this study. We hope this explanation clarifies our methodological choices and demonstrates their alignment with the aims of our analysis.

      (5) Replication: I want to point out that it is great that the previous findings were in principle replicated. However, I would like to suggest a more nuanced evaluation of the replication:

      a) Instead of a (direct) replication, the present study should be called a 'conceptual replication', since modifications in design and procedure were made.

      Thank you very much for this suggestion! We now use the term ‘conceptual replication’ throughout the manuscript.

      b) Not all the findings from the Gehmacher et al., 2024 study were replicated to a full extent:

      Did the authors find indications of a vertical vs. horizontal tracking difference in the Gehmacher 2024 data? Could they check this in the Gehmacher 2024 data?

      The findings for horizontal and vertical gaze tracking in Gehmacher et al. (2024) are detailed in the supplementary material of that publication. Both single-speaker and multi-speaker target conditions showed significant speech tracking effects in both horizontal and vertical directions. However, there was a slightly stronger tracking effect for the single-speaker condition in the vertical direction. Due to the highly predictable structure of words in Gehmacher et al. effects here were probably overall boosted as compared to continuous audiobook listening, likely leading to the differentiation of horizontal and vertical gaze. See figures in Gehmacher et al. supplementary file for reference.

      c) Another difference between their previous and this study is the non-existent tracking of the multi-speaker distractor in this study. The authors should point this out clearly in the discussion and potentially provide an explanation.

      Thank you for highlighting this point! We now address this in the discussion:

      “Importantly, in contrast to Gehmacher et al. (2024), we did not observe ocular tracking of the multi-speaker distractor in this study. This difference is likely attributable to the simplistic single-trial, 5-word task structure in Gehmacher et al., which resulted in high temporal overlap between the target and distractor speech streams and likely drove the significant distractor-tracking effects observed in that study. The absence of such an effect during continuous listening in our study suggests that ocular tracking is indeed more specific to selective attention.”

      Minor:

      (1) I was a little surprised to not see an indication of eyes/eye movements in Figure 6. The intention of the authors might have been to create a general schematic illustration, but I find this a bit misleading. This paper provides nice evidence for a specific ocular effect in speech tracking. There is, to my knowledge, no indication that speech would be influenced by different kinds of active sensing (if there are, please include them in the discussion). Given that the visuomotor system is quite dominant in humans, it might actually be the case that the speech tracking the authors describe is specifically ocular.

      Taking into account all the reviewers' remarks on the findings and interpretations, we have updated this figure (now Fig. 7) in the manuscript to make it more specific and aligned with the revised discussion section. Throughout the manuscript, we now explicitly refer to active ocular sensing in relation to speech processing and have avoided the broader term 'active sensing' in this context. We hope these revisions address the concerns raised.

      (2) I find the part in the discussion (page 2, last paragraph) on cognitive processes hard to follow. I don't agree that 'cognitive processes' are easily separable from any of the measured responses (eye and brain). Referring to the example they provide, there is evidence that eye movements are correlated with brain activity that is correlated with memory performance. How, and more importantly, why would one separate those?

      Thank you for raising this important point. We have carefully considered your comments, particularly regarding the interplay between cognitive processes and measured responses (eye and brain), as well as the challenge of conceptually separating them. Additionally, we have incorporated Reviewer #2's query (13) into a unified and complementary reasoning. In response, we have rewritten the relevant paragraph in the discussion to provide a clearer and more detailed explanation of how ocular and neural responses contribute to speech processing in an interdependent manner. We hope this revision addresses your concerns and offers a more precise and coherent discussion on this topic:

      “Despite the finding that eye movements mediate neural speech tracking, the behavioural relevance for semantic comprehension appears to differ between ocular and neural speech tracking. Specifically, we found a negative association between ocular speech tracking and comprehension, indicating that participants with lower comprehension performance exhibited increased ocular speech tracking. Interestingly, no significant relationship was observed between neural tracking and comprehension.

      In this context, the negative association between ocular tracking and comprehension might reflect individual differences in how participants allocate cognitive resources. Participants with lower comprehension may rely more heavily on attentional mechanisms to process acoustic features, as evidenced by increased ocular tracking. This reliance could represent a compensatory strategy when higher-order processes, such as semantic integration or memory retrieval, are less effective. Importantly, our comprehension questions (see Experimental Procedure) targeted a broad range of processes, including intelligibility and memory, suggesting that this relationship reflects a trade-off in resource allocation between low-level acoustic focus and integrative cognitive tasks.

      Rather than separating eye and brain responses conceptually, our analysis highlights their complementary contributions. Eye movements may enhance neural processing by increasing sensitivity to acoustic properties of speech, while neural activity builds on this foundation to integrate information and support comprehension. Together, these systems form an interdependent mechanism, with eye and brain responses working in tandem to facilitate different aspects of speech processing.

      This interpretation is consistent with the absence of a difference in ocular tracking for semantic violations (e.g., words with high surprisal versus lexically matched controls), reinforcing the view that ocular tracking primarily reflects attentional engagement with acoustic features rather than direct involvement in semantic processing. This aligns with previous findings that attention modulates auditory responses to acoustic features (e.g., Forte et al., 2017), further supporting the idea that ocular tracking reflects mechanisms of selective attention rather than representations of linguistic content.

      Future research should investigate how these systems interact and explore how ocular tracking mediates neural responses to linguistic features, such as lexical or semantic processing, to better understand their joint contributions to comprehension.”.  

      (3) Attention vs. predictive coding. I think the authors end up with an elegant description of the observed effects, "as an "active sensing" mechanism that implements the attentional optimization of sensory precision." However, I feel the paragraph starts with the ill-posed question "whether ocular speech tracking is modulated not by predictive, but other (for example attentional) processes". If ocular tracking is the implementation of a process (optimization of sensory precision, aka attention), how could it be at the same time modulated by that process? In my opinion, adding the notion that there is a modulation by a vague cognitive concept like attention on top of what the paper shows does not improve our understanding of how speech tracking in humans works.

      Thank you for raising this point. We agree that it is critical to clarify the relationship between ocular speech tracking, attention, and predictive processes, and we appreciate the opportunity to refine this discussion.  

      To avoid the potential confusion that active ocular sensing represents on the one hand an implementation of selective attention on the other it seems to be modulated by it, we now use  the formulation “ocular speech tracking reflects attentional mechanisms rather than predictive processes.”

      To address your concern that the conceptualization of attention seems rather vague, we have revised the whole paragraph in order to redefine the theoretical entities in question (including selective attention) and to provide a clearer and more precise picture (see also our revised version of Fig. 6, now Fig. 7). We now focus on highlighting the distinct yet interdependent roles of selective attention and individual prediction tendencies for speech tracking.:

      “With this speculative framework we attempt to describe and relate three important phenomena with respect to their relevance for speech processing: 1) “Anticipatory predictions” that are created in absence of attentional demands and contain probabilistic information about stimulus features (here, inferred from frequency-specific pre-activations during passive listening to sound sequences). 2) “Selective attention” that allocates resources towards relevant (whilst suppressing distracting) information (which was manipulated by the presence or absence of a distractor speaker). And finally 3) “active ocular sensing”, which refers to gaze behavior that is temporally aligned to attended (but not unattended) acoustic speech input (inferred from the discovered phenomenon of ocular speech tracking). We propose that auditory inflow is, at a basic level, temporally modulated via active ocular sensing, which “opens the gates” in the sensory periphery at relevant timepoints. How exactly this mechanism is guided (for example where the information about crucial timepoints comes from, if not from prediction, and whether it requires habituation to a speechstream etc.) is yet unclear. Unlike predictive tendencies, active ocular sensing appears to reflect selective attention, manifesting as a mechanism that optimizes sensory precision. Individual differences with respect to anticipatory predictions on the other hand, seem to be independent from the other two entities, but nevertheless relevant for speech processing. We therefore support the notion that representational content is interpreted based on prior probabilistic assumptions. If we consider the idea that “a percept” of an (auditory) object is actually temporally and spatially distributed (across representational spacetime - see Fig. 7), the content of information depends on where and when it is probed (see for example Dennett, 1991 for similar ideas on consciousness). Having to select from multiple interpretations across space and time requires a careful balance between the weighting of internal models and the allocation of resources based on current goals. We suggest that in the case of speech processing, this challenge results in an independent adaptation of feature-based precision-weighting by predictions on the one hand and temporal precision-weighting by selective attention on the other.”

      Reviewer #2 (Recommendations for the authors):

      My main recommendation is outlined in the Weaknesses above: the overarching rationale for many analysis choices should be made explicit, and intermediate results should be shown where appropriate, so the reader can follow what is being quantified and what the results truly mean. Specifically, I recommend to pay attention to the following (in no particular order):

      (1) Define 'neural speech tracking' early on. (e.g.: 'The amount of information in the MEG signal that can multivariately be explained by the speech amplitude envelope.' (is that correct?))

      Thank you for pointing out that this important definition is missing. It is now defined at the first mention in the Introduction as follows: “Here (and in the following) “neural speech tracking” refers to a correlation coefficient between actual brain responses and responses predicted from an encoding model based solely on the speech envelope”.

      (2) Same for 'ocular speech tracking'. Here even reading the Methods does not make it unambiguous how this term is used.

      It is now defined at the first mention in the Introduction as follows: “Ocular speech tracking” (similarly to “neural speech tracking” refers to the correlation coefficient between actual eye movements and movements predicted from an encoding model based on the speech envelope”.

      In addition also define both (neural and ocular speech tracking) metrics in the Methods Section.

      (3) Related to this: for ocular speech tracking, are simply the horizontal and vertical eye traces compared to the speech envelope? If so, this appears somewhat strange: why should the eyes move more rightward/upward with a larger envelope? And the direction here depends on the (arbitrary) sign of right = positive, etc. (It would make more sense to quantify 'amount of movement' in some way, but if this is done, I missed it in Methods.)

      Thank you for your insightful comments. You are correct that the horizontal and vertical traces were used for ocular speech tracking, and no additional details were included in the Methods. While we agree that the observed rightward/upward movement may seem unusual, this pattern is consistent with previous findings, including those reported in Gehmacher et al. (2024). In that study, we discussed how ocular speech tracking could reflect a broader engagement of the motor system during speech perception. For example, we observed a general right-lateralized gaze bias when participants attended to auditory speech, which we hypothesized might resemble eye movements during text reading, with a similar temporal alignment (~200 ms). We also speculated that this pattern might differ in cultures that read text from right to left.

      We appreciate your suggestion to explore alternative methods for quantifying gaze patterns, such as the "amount of movement" or microsaccades. While these approaches hold promise for future studies, our primary aim here was to replicate previous findings using the same signal and analysis methods to establish a basis for further exploration.  

      (4) In the Introduction, specifically blink-related ocular activity is mentioned as being related to speech tracking (for which a reference is, incidentally, missing), while here, any blink-related activity is excluded from the analysis. This should be motivated, as it appears in direct contradiction.

      Thank you for pointing this out. The mention of blink-related ocular activity in the Introduction refers to findings by Jin et al. (2018), where such activity was shown to align with higher-order syntactic structures in artificial speech. We have now included the appropriate reference for clarity.

      While Jin et al. focused on blink-related activity, in the present study, we focused on gaze patterns to investigate ocular speech tracking, replicating findings from

      Gehmacher et al. (2024). This approach was motivated by our goal to validate previous results using the same methodology. Importantly to this point, the exclusion of blinks in our analysis was due to methodological constraints of TRF analysis, which requires a continuous response signal; blinks, being discrete and artifact-prone, are incompatible with this approach.

      To address your concern, we revised the Introduction to clarify this distinction and provide explicit motivation for focusing on gaze patterns. It now reads:

      “Along these lines, It has been shown that covert, mostly blink related eye activity aligns with higher-order syntactic structures of temporally predictable, artificial speech (i.e. monosyllabic words; Jin et al, 2018). In support of ideas that the motor system is actively engaged in speech perception (Galantucci et al., 2006; Liberman & Mattingly, 1985), the authors suggest a global entrainment across sensory and (oculo)motor areas which implements temporal attention. 

      In another recent study from our lab (Gehmacher et al., 2024), we showed that eye movements continuously track intensity fluctuations of attended natural speech, a phenomenon we termed ocular speech tracking. In the present study, we focused on gaze patterns rather than blink-related activity, both to replicate findings from

      Gehmacher et al. (2024) and because blink activity is unsuitable for TRF analysis due to its discrete and artifact-prone nature. Hence, “Ocular speech tracking” (similarly to “neural speech tracking” refers to the correlation coefficient between actual eye movements and movements predicted from an encoding model based on the speech envelope.”

      Jin, P., Zou, J., Zhou, T., & Ding, N. (2018). Eye activity tracks task-relevant structures during speech and auditory sequence perception. Nature communications, 9(1), 5374.

      (5) The rationale for the mediation analysis is questionable. Let speech envelope = A, brain activity = B, eye movements = C. The authors wish to claim that A -> C -> B. But it is equally possible that A -> B -> C. They reflect on this somewhat in Discussion, but throughout the rest of the paper, the mediation analysis is presented as specifically testing whether A -> B is mediated by C, which is potentially misleading.

      Indeed we share your concern regarding the directionality of the relationships in the mediation analysis. Our choice of ocular movements as a mediator was motivated by the fact that the relationship between acoustic speech and neural activity is well established, as well as previous results indicating that oculomotor activity contributes to cognitive effects in auditory attention (Popov et al., 2022). 

      Indeed, here we treat both interpretations (“ocular movements contribute to neural speech tracking” versus “neural activity contributes to ocular speech tracking”) as equal.  We now emphasise this point in our discussion quite thoroughly:

      “It is important to note that our current findings do not allow for inference on directionality. Our choice of ocular movements as a mediator was motivated by the fact that the relationship between acoustic speech and neural activity is well established, as well as previous results indicating that oculomotor activity contributes to cognitive effects in auditory attention (Popov et al., 2022). However, an alternative model may suggest that neural activity mediates the effect of ocular speech tracking. Hence, it is possible that ocular mediation of speech tracking may reflect a) active (ocular) sensing for information driven by (top-down) selective attention or b) improved neural representations as a consequence of temporally aligned increase of sensory gain or c) (not unlikely) both. In fact, when rejecting the notion of a single bottom-up flow of information and replacing it with a model of distributed parallel and dynamic processing, it seems only reasonable to assume that the direction of communication (between our eyes and our brain) will depend on where (within the brain) as well as when we look at the effect. Thus, the regions and time-windows reported here should be taken as an illustration of oculo-neural communication during speech processing rather than an attempt to "explain" neural speech processing by ocular movements.”

      (6) The mediation analysis can be improved by a proper quantification of the effect (sizes or variance explained). E.g. how much % of B is explained by A total, and how much of that can in turn be explained by C being involved? For drawing directional conclusions perhaps Granger causality could be used.

      In Figure 4 (now Figure 5) of our manuscript we use standardized betas (which correspond to effect sizes) to illustrate the mediation effect. With the current mTRF approach it is however not possible (or insightful) to compare the variance explained. It is reasonable to assume that variance in neural activity will be explained better when including oculomotor behavior as a second predictor along with acoustic simulation. However this increase gives no indication to what extent this oculomotor behavior was task relevant or irrelevant (since all kinds of “arbitrary” movements will be captured with brain activity and therefore lead to an increase in variance explained). For this reason we chose to pursue the widely accepted framework of mediation (Baron & Kenny, 1986). This (correlational) approach is indeed limited in its interpretations (see prev. response), however the goal of the current study was to replicate and illustrate the triad relationship of acoustic speech input, neural activity and ocular movements with no particular hypotheses on directionality.

      (7) Both prediction tendency and neural speech tracking depend on MEG data, and thus on MEG signal-to-noise ratio (SNR). It is possible some participants may have higher SNR recordings in both tasks, which may result in both higher (estimated) prediction tendency and higher (estimated) speech tracking. This would result in a positive correlation, as the authors observe. This trivial explanation should be ruled out, by quantifying the relative SNR and testing for the absence of a mediation here.

      We agree that for both approaches (MVPA and mTRF models) individual MEG SNR plays an important role. This concern has been raised previously and addressed in our previous manuscript (Schubert et al., 2023). First, it should be noted that our prediction tendency value is the result of a condition contrast (rather than simple decoding accuracy) which compensates for the influence of subject specific signal-to-noise ratio (as no vacuous difference in SNR is to be expected between conditions). Second, in our previous study we also used frequency decoding accuracy as a control variable to correlate with speech tracking variables of interest and found no significant effect.

      (8) Much of the analysis pipeline features temporal response functions (TRFs). These should be shown in a time-resolved manner as a key intermediate step.

      We now included the Neural Speech tracking TRFs into the Figure (now Figure 3).

      (9) Figure 2 shows much-condensed results from different steps in the pipeline. If I understand correctly, 2A shows raw TRF weights (averaged over some time window?), while 2B-F shows standardized mean posterior regressor weights after Bayesian stats? It would be very helpful to make much more explicit what is being shown here, in addition to showing the related TRFs.

      Thank you for pointing this out! The figure description so far has been indeed not very insightful on this issue. We now adapted the caption and hope this clarifies the confusion: “ Neural speech tracking is related to prediction tendency and word surprisal, independent of selective attention. A) Envelope (x) - response (y) relationships are estimated using deconvolution (Boosting). The TRF (filter kernel, h) models how the brain processes the envelope over time. This filter is used to predict neural responses via convolution. Predicted responses are correlated with  actual neural activity to evaluate model fit and the TRF's ability to capture response dynamics. Correlation coefficients from these models are then used as dependent variables in Bayesian regression models. (Panel adapted from Gehmacher et al., 2024b). B) Temporal response functions (TRFs) depict the time-resolved neural tracking of the speech envelope for the single speaker and multi speaker target condition, shown here as absolute values averaged across channels. Solid lines represent the group average. Shaded areas represent 95% Confidence Intervals. C–H) The beta weights shown in the sensor plots are derived from Bayesian regression models in A). For Panel C, this statistical model is based on correlation coefficients computed from the TRF models (further details can be found in the Methods Section). C) In a single speaker condition, neural tracking of the speech envelope was significant for widespread areas, most pronounced over auditory processing regions. D) The condition effect indicates a decrease in neural speech tracking with increasing noise (1 distractor). E) Stronger prediction tendency was associated with increased neural speech tracking over left frontal areas. F) However, there was no interaction between prediction tendency and conditions of selective attention. G) Increased neural tracking of semantic violations was observed over left temporal areas. H) There was no interaction between word surprisal and speaker condition, suggesting a representation of surprising words independent of background noise. Marked sensors indicate ‘significant’ clusters, defined as at least two neighboring channels showing a significant result. N = 29.”

      Gehmacher, Q., Schubert, J., Kaltenmaier, A., Weisz, N., & Press, C. (2024b). The "Ocular Response Function" for encoding and decoding oculomotor related neural activity. bioRxiv, 2024-11.

      (10) Bayesian hypothesis testing is not done consistently. Some parts test for inclusion of 0 in 94% HDI, while some parts adopt a ROPE approach. The same approach should be taken throughout. Additionally, Bayes factors would be very helpful (I appreciate these depend on the choice of priors, but the default Bambi priors should be fine).

      Our primary aim in this study was to replicate two recent findings: (1) the relationship between individual prediction tendencies and neural speech tracking, and (2) the tracking of the speech envelope by eye movements. To maintain methodological consistency with the original studies, we did not apply a ROPE approach when analyzing these replication effects. Instead, we followed the same procedures as the original work, focusing on the inclusion of 0 in the HDI for the neural effects and using the same methods for the ocular effects. Additionally, we were not specifically interested in potential null effects in these replication analyses, as our primary goal was to test whether we could reproduce the previously reported findings.

      For the mediation analysis, however, we chose to extend the original approach by not only performing the analysis in a time-resolved manner but also applying a ROPE approach. This decision was motivated by our interest in gaining more comprehensive insights — beyond the replication goals — by also testing for potential null effects, which can provide valuable information about the presence or absence of mediation effects.

      We appreciate your thoughtful feedback and hope this clarifies our rationale for the differing approaches in our Bayesian hypothesis testing. 

      Regarding Bayes Factors, 

      We understand that some researchers find Bayes Factors appealing, as they offer a seemingly simple and straightforward way to evaluate the evidence in favor of/ or against H0 in relation to H1 (e.g. BF10 > 102 =  Decisive; according to the Jeffreys Scale). However, in practice Bayes Factors are often misunderstood e.g. by interpreting Bayes Factor as posterior odds or not acknowledging the notion of relative evidence in the Bayes Factor (see Wong et al. 2022). Instead of using Bayes Factors, we prefer to rely on estimating and reporting the posterior distribution of parameters given the data, prior and model assumptions (in form of the 94% HDI). This allows for a continuous evaluation of evidence for a given hypothesis that is in our eyes easier to interpret as a Bayes Factor.

      Jeffreys, Harold (1998) [1961]. The Theory of Probability (3rd ed.). Oxford, England. p. 432. ISBN 9780191589676.

      Wong, T. K., Kiers, H., & Tendeiro, J. (2022). On the Potential Mismatch Between the Function of the Bayes Factor and Researchers’ Expectations. Collabra: Psychology, 8(1), 36357. https://doi.org/10.1525/collabra.36357

      (11) It would be helpful if Results could be appreciated without a detailed read of Methods. I would recommend a recap of each key methodological step before introducing the relevant Result. (This may also help in making the rationale explicit.)

      In addition to the short recaps of methods that were already present, and information on quantifications of neural and ocular tracking and bayes statistics (see responses 1, 2, 9), we now added the following parts below to the results sections. Please refer to them in the context of the manuscript where they should now complement a key recap of methodological steps necessary to readily understand each analysis and rational that led to the results:

      Individual prediction tendency is related to neural speech tracking:

      “Thus, this measure is a single value per subject, which comprises a) differences between two contextual probabilities (i.e. ordered vs. random) in b) feature-specific tone representations c) in advance of their observation (summed over a time-window of -0.3 - 0 s). Importantly, this prediction tendency was assessed in an independent entropy modulation paradigm (see Fig. 1). On a group level we found an increased tendency to pre-activate a stimulus of high probability (i.e. forward transition) in an ordered context compared to a random context (see Fig, 2A). This effect replicates results from our previous work (Schubert et al., 2023, 2024). Using the summed difference between entropy levels (ordered - random) across pre-stimulus time, one value was extracted per subject (Fig. 2B). This value was used as a proxy for “individual prediction tendency” and correlated with encoding of clear speech across different MEG sensors. [...]

      Neural speech tracking, quantified as the correlation coefficients between predicted and observed MEG responses to the speech envelope, was used as the dependent variable in Bayesian regression models. These models included condition (single vs. multi-speaker) as a fixed effect, with either prediction tendency or word surprisal as an additional predictor, and random effects for participants.”

      Eye movements track acoustic speech in selective attention:

      “For this, we separately predicted horizontal and vertical eye movements from the acoustic speech envelope using temporal response functions (TRFs). The resulting model fit (i.e. correlation between true and predicted eye movements) is commonly referred to as “speech tracking”. Bayesian regression models were applied to evaluate tracking effects under different conditions of selective attention (single speaker, attended multi-speaker, unattended multi-speaker). Furthermore, we assessed whether individual prediction tendency or semantic word surprisal influenced ocular speech tracking.”

      Neural speech tracking is mediated by eye movements:

      “This model evaluates to what extent gaze behaviour functions as a mediator between acoustic speech input and brain activity.”

      Neural and ocular speech tracking are differently related to comprehension: “Bayesian regression models were used to investigate relationships between neural/ocular speech tracking and comprehension or difficulty. Ocular speech tracking was analyzed separately for horizontal and vertical eye movements.”

      (12) The research questions in the Introduction should be sharpened up, to make explicit when a question concerns a theoretical entity, and when it concerns something concretely measured/measurable.

      We sharpened them up:

      “Taking into account the aforementioned study by Schubert and colleagues (2023), the two recently uncovered predictors of neural tracking (individual prediction tendency and ocular tracking) raise several empirical questions regarding the relationship between predictive processes, selective attention, and active ocular sensing in speech processing:

      (1) Are predictive processes related to active ocular sensing in the same way they are to neural speech tracking? Specifically, do individuals with a stronger tendency to anticipate predictable auditory features, as quantified through prestimulus neural representations in an independent tone paradigm, show increased or even decreased ocular speech tracking, measured as the correlation between predicted and actual eye movements? Or is there no relationship at all?

      (2) To what extent does selective attention influence the relationship between prediction tendency, neural speech tracking, and ocular speech tracking? For example, does the effect of prediction tendency or ocular speech tracking on neural tracking differ between a single-speaker and multi-speaker listening condition?

      (3) Are individual prediction tendency and ocular speech tracking related to behavioral outcomes, such as comprehension and perceived task difficulty? Speech comprehension is assessed through accuracy on comprehension questions, and task difficulty is measured through subjective ratings.

      Although predictive processes, selective attention, and active sensing have been shown to contribute to successful listening, their potential interactions and specific roles in naturalistic speech perception remain unclear. Addressing these questions will help disentangle their contributions and establish an integrated framework for understanding how neural and ocular speech tracking support speech processing.”

      (13) The negative relationship between story comprehension and ocular speech tracking appears to go against the authors' preferred interpretation, but the reflection on this in the Discussion is very brief and somewhat vague.

      Thank you for pointing this out. We have taken your comments into careful consideration and also incorporated Reviewer #1's query (Minor point 2) into a unified and complementary reasoning. We have rewritten the relevant paragraph in the discussion to provide a clearer and more detailed explanation. We hope this revision offers a more precise and less vague discussion on this important point.

      “Despite the finding that eye movements mediate neural speech tracking, the behavioural relevance for semantic comprehension appears to differ between ocular and neural speech tracking. Specifically, we found a negative association between ocular speech tracking and comprehension, indicating that participants with lower comprehension performance exhibited increased ocular speech tracking. Interestingly, no significant relationship was observed between neural tracking and comprehension.

      In this context, the negative association between ocular tracking and comprehension might reflect individual differences in how participants allocate cognitive resources. Participants with lower comprehension may rely more heavily on attentional mechanisms to process acoustic features, as evidenced by increased ocular tracking. This reliance could represent a compensatory strategy when higher-order processes, such as semantic integration or memory retrieval, are less effective. Importantly, our comprehension questions (see Experimental Procedure) targeted a broad range of processes, including intelligibility and memory, suggesting that this relationship reflects a trade-off in resource allocation between low-level acoustic focus and integrative cognitive tasks.

      Rather than separating eye and brain responses conceptually, our analysis highlights their complementary contributions. Eye movements may enhance neural processing by increasing sensitivity to acoustic properties of speech, while neural activity builds on this foundation to integrate information and support comprehension. Together, these systems form an interdependent mechanism, with eye and brain responses working in tandem to facilitate different aspects of speech processing.

      This interpretation is consistent with the absence of a difference in ocular tracking for semantic violations (e.g., words with high surprisal versus lexically matched controls), reinforcing the view that ocular tracking primarily reflects attentional engagement with acoustic features rather than direct involvement in semantic processing. This aligns with previous findings that attention modulates auditory responses to acoustic features (e.g., Forte et al., 2017), further supporting the idea that ocular tracking reflects mechanisms of selective attention rather than representations of linguistic content.

      Future research should investigate how these systems interact and explore how ocular tracking mediates neural responses to linguistic features, such as lexical or semantic processing, to better understand their joint contributions to comprehension.”.  

      (14) Page numbers would be helpful.

      We added the page numbers.

      Reviewer #3 (Recommendations for the authors):

      Results

      (1) Figure 2 - statistical results are reported in this figure, but they are not fully explained in the text, nor are statistical values provided for any of the analyses (as far as I can tell).

      Also, how were multiple comparisons dealt with (the choice of two neighboring channels seems quite arbitrary)? Perhaps for this reason, the main result - namely the effect of "prediction tendency" and "semantic violations" - is quite sparse and might not survive more a rigorous statistical criterion. I would feel more comfortable with these results if the reporting of the statistical analysis had been more thorough (ideally, including comparison to control models).

      We would like to thank you again for your detailed queries, comments, and questions on our work. We first of all adapted this figure (now Figure 3 in the manuscript, please see responses 8 and 9 to Reviewer #2) to help readers understand the metrics and values within each statistical analysis. In addition, we indeed did not include the detailed statistics in the text! We now added the missing statistic reports calculated as averages over ‘clusters’:

      “Replicating previous findings (Schubert et al., 2023), we found widespread encoding of clear speech (average over cluster: β = 0.035, 94%HDI = [0.024, 0.046]), predominantly over auditory processing regions (Fig. 3C), that was decreased (β = -0.018, 94%HDI = [0.029, -0.006]) in a multi-speaker condition (Fig. 3D). Furthermore, a stronger prediction tendency was associated with increased neural speech tracking (β = 0.014, 94%HDI = [0.004, 0.025]) over left frontal sensors (see Fig. 3E). We found no interaction between prediction tendency and condition (see Fig. 3F).” [...] “In a direct comparison with lexically identical controls, we found an increased neural tracking of semantic violations (β = 0.039, 94%HDI = [0.007, 0.071]) over left temporal areas (see Fig. 3G). Furthermore, we found no interaction between word surprisal and speaker condition (see Fig. 3H).”

      Regarding the "prediction tendency" effect, it is important to note that this finding replicates a result from Schubert et al. (2023). The left frontal location of this effect is also consistent over studies, which convinces us of the robustness of the finding. Furthermore, testing this relationship properly requires a mixed-effects model in order to account for the variability across subjects that is not explained by fixed effects and the repeated measures design. For this reason a random Intercept had to be fitted for each subject (1|subject in the respective model formula). This statistical requirement motivated our decision to use bayesian statistics as (at least to our knowledge) there is no implementation of a cluster-based permutation mixed effects model (yet). In order to provide a more conservative criterion (as bayesian statistics don’t require a multiple comparison correction) we chose to impose in addition the requirement of a “clustered” effect.

      The choice of using two neighboring channels is consistent with the default parameter settings in FieldTrip’s cluster-based permutation testing (cfg.minnbchan = 2). This parameter specifies the minimum number of neighboring channels required for a sample to be included in the clustering algorithm, ensuring spatial consistency in the identified clusters. This alignment ensures that our methodology is comparable to numerous prior studies in the field, where such thresholds are standard. While it is true that all statistical analyses involve some degree of arbitrariness in parameter selection (e.g., alpha levels or clustering thresholds), our approach reflects established conventions and ensures comparability with previous findings.

      While the original study utilized source space analyses, we replicated this effect using only 102 magnetometers. This choice was made for computational simplicity, demonstrating that the effect is robust even without source-level modeling. Similarly, the "semantic violation" effect, while perceived as sparse, is based solely on magnetometer data and - in our opinion - should not be viewed as overly sparse given the methods employed. This effect aligns with the two-neighbor clustering approach, ensuring spatial consistency across magnetometers. The results reflect the robustness of the effects within the constraints of magnetometer-level analyses.

      Overall, the methodological choices, including the choice of a bayesian linear mixed effects model, the use of two neighboring channels and the reliance on magnetometers, are grounded in established practices and methodological considerations. While stricter thresholds or alternative approaches might yield different results, our methods align with best practices in the field and ensure the robustness, comparability, and replicability of our findings.

      (2) Figure 3 - the difference between horizontal and vertical eye-movements. This result is quite confusing and although the authors do suggest a possible interpretation for this in the discussion, I do wonder how robust this difference is or whether the ocular signal (in either direction) is simply too noisy or the effect too small to be detected consistently across conditions. Also, the ocular-TRFs themselves are not entirely convincing in suggesting reliable response/tracking of the audio - despite the small-but-significant increase in prediction accuracy.

      The horizontal versus vertical comparison was conducted to explore potential differences in how these dimensions contribute to ocular tracking of auditory stimuli (please also see our response to Reviewer #1, Response 5b, that includes the vertical vs. horizontal effects of Gehmacher at al. 2024). It would indeed be interesting to develop a measure that combines the two directions into a more natural representation of 'viewing,' such as a combined vector. However, this approach would require the use of complex numbers to represent both magnitude and direction simultaneously, hence the development of novel TRF algorithms capable of modeling this multidimensional signal. While beyond the scope of the current study, this presents an exciting avenue for future research and would allow us to move closer to understanding ocular speech tracking and the robustness of these effects, above and beyond the already successful replication.

      It is also important to emphasize that ocular-TRFs are derived from (viewing) behavioral data rather than neural signals, and are thus inherently subject to greater variability across participants and time. This higher variability does not necessarily indicate a small or unreliable effect but reflects the dynamic and task-dependent nature of eye movement behavior. The TRFs with shaded error margins represent this variability, highlighting how eye movements are influenced by both individual differences and moment-to-moment changes in task engagement.

      Despite this inherent variability, the significant prediction accuracy improvements confirm that ocular-TRFs reliably capture meaningful relationships between eye movements and auditory stimuli. The observed differences between horizontal and vertical TRFs further support the hypothesis that these dimensions are differentially involved in the task, possibly driven by the specific roles they play in sensorimotor coupling.

      (3) Figure 4 - this figure shows source distribution of 3 PCA components, derived from the results of the mediation effect of eye movements on the speech-tracking. Here too I am having difficulty in interpreting what the results actually are. For one, all three components are quite widespread and somewhat overlapping, so although they are statistically "independent" it is hard to learn much from them about the brain regions involved and whether they truly represent separable contributions. Similarly difficult to interpret are the time courses, which share some similarities with the known TRFs to speech (especially PC3). I would have expected to find a cleaner "auditory" response, and clearer separation between sensory regions and regions involved in the control of eye movements. I also wonder why the authors chose not to show the sourcelocalization of the neural and ocular speech-tracking responses alone - this could have helped us between understand what "mediation" of the neural response might look like.

      We appreciate the reviewer’s interest in better understanding the source distribution and time courses of the PCA components. While we acknowledge that the widespread and overlapping nature of the components may make a more fine grained interpretation challenging, it is important to emphasize that our analysis simply reflects the data, hence we can only present and interpret what the analysis revealed.

      Regarding your suggestion to show the source localization of ocular speech tracking and neural speech tracking alone, we would like to point out that ocular tracking is represented by only one channel for vertical and one channel for horizontal eye movements. Thus, in this case the estimated source of the effect are the eyes themselves. We believe that the source localization of neural speech tracking has been a thoroughly studied topic in research so far (locating it to perisylvian, auditory areas with a stronger preference for the left hemisphere) and can also be seen in Schubert et al., (2023). Nevertheless, we believe the observed PCA components still provide valuable, and most importantly novel insights into the interplay between eye movements and neural responses in speech tracking.  

      Discussion/interpretation

      (1) Although I appreciate the authors' attempt to propose a "unified" theoretical model linking predictions about low-level features to higher features, and the potential involvement of eye movements in 'active sensing' I honestly think that this model is overambitious, given the data presented in the current study. Moreover, there is very little discussion of past literature and existing models of active sensing and hierarchical processing of speech, that could have helped ground the discussion in a broader theoretical context. The entire discussion contains fewer than 20 citations (some of which are by these authors) and needs to be substantially enriched in order to provide context for the authors' claims.

      Thank you very much for your thoughtful feedback and for appreciating our approach. We hope that the revised manuscript addresses your concerns. Specifically, we now emphasize that our proposal is a conceptual framework, with the main goal to operationale “prediction tendency”, “active ocular sensing”, and “selective attention” and to “organise these entities according to their assumed function for speech processing and to describe their relationship with each other.” We did this by thoroughly revising our discussion section with a clear emphasis on the definition of terms, for example: 

      “With this speculative framework we attempt to describe and relate three important phenomena with respect to their relevance for speech processing: 1) “Anticipatory predictions” that are created in absence of attentional demands and contain probabilistic information about stimulus features (here, inferred from frequency-specific pre-activations during passive listening to sound sequences). 2) “Selective attention” that allocates resources towards relevant (whilst suppressing distracting) information (which was manipulated by the presence or absence of a distractor speaker). And finally 3) “active ocular sensing”, which refers to gaze behavior that is temporally aligned to attended (but not unattended) acoustic speech input (inferred from the discovered phenomenon of ocular speech tracking).”

      Our theoretical proposals are now followed by a recap of our results that support the respective idea, for example: 

      “...these predictions are formed in parallel and carry high feature-specificity but low temporal precision (as they are anticipatory in nature). This idea is supported by our finding that pure-tone anticipation is visible over a widespread prestimulus interval, instead of being locked to sound onset”

      “....we suggest that active (ocular) sensing does not necessarily convey feature- or content-specific information, it is merely used to boost (and conversely filter) sensory input at specific timescales (similar to neural oscillations). This assumption is supported by our finding that semantic violations are not differentially encoded in gaze behaviour than lexical controls.”

      And we put a strong focus on highlighting the boundaries of these ideas, in order to avoid theoretical confusion, misunderstandings or implicit theoretical assumption that are not grounded in data, in particular: 

      “In fact, when rejecting the notion of a single bottom-up flow of information and replacing it with a model of distributed parallel and dynamic processing, it seems only reasonable to assume that the direction of communication (between our eyes and our brain) will depend on where (within the brain) as well as when we look at the effect. Thus, the regions and time-windows reported here should be taken as an illustration of oculo-neural communication during speech processing rather than an attempt to "explain" neural speech processing by ocular movements.”

      “Even though the terminology [“hierarchy”] is suggestive of a fixed sequence (similar to a multi storey building) with levels that must be traversed one after each other (and even the more spurious idea of a rooftop, where the final perceptual experience is formed and stored into memory), we distance ourselves from these (possibly unwarranted) ideas. Our usage of “higher” or “lower” simply refers to the observation that the probability of a feature at a higher (as in more associative) level affects the interpretation (and thus the representation and prediction) of a feature at lower (as in more segregated) levels (Caucheteux et al., 2023).”

      Additionally, we have made substantial efforts to present complementary results (see response to Reviewer #2, point 8) to further substantiate our interpretation. Importantly, we have updated the illustration of the model (see response to Reviewer #, minor point 1) and refined both our interpretations and the conceptual language in the Discussion. Furthermore, we have included additional citations where appropriate to strengthen our argument.

      We would also like to briefly note that this section of the Discussion aimed to highlight existing literature that bridges the gap our model seeks to address. However, as this is a relatively underexplored area, the references available are necessarily limited.

      (2) Given my many reservations about the data, as presented in the current version of the manuscript, I find much of the discussion to be an over-interpretation of the results. This might change if the authors are able to present more robust results, as per some of my earlier comments.

      We sincerely hope that our comprehensive revisions have addressed your concerns and improved the manuscript to your satisfaction.

    1. Author response:

      The following is the authors’ response to the original reviews.

      Reviewer 1:

      The main weaknesses of the paper are a lack of significance in key findings, and relatedly, concluding effects from insignificant findings. Additional elements could be improved to help strengthen this overall well-rounded and intriguing set of results.

      In the original manuscript, we reported that chemogenetic silencing of POA-social neurons (previously called POA-iso neurons; more details on rationale for renaming below in our responses to reviewer recommendations) tended to reduce mounting in both single-housed female and single-housed male mice, although these effects were non-significant. We have added samples to both datasets and now report that chemogenetic silencing of POA-social neurons significantly reduces the proportion of trials with mounting in both sexes (Fig. 2C and Fig. 6G). 

      We have also included new analyses to test whether optogenetic activation of POAsocial neurons in group-housed females promotes social investigation (in addition to USV production, as reported in the original manuscript). We now report that optogenetic activation of POA-social neurons significantly increases the probability of social investigation (Fig. 4E-F) and significantly increases the duration of social investigation bouts (Fig. 4G). 

      Additional recommendations from the reviewer are addressed in detail below. Thank you for your critical and insightful feedback.

      Reviewer 2:

      All the activity-dependent labeling experiments with TRAP mice, including the subsequent neural activity manipulation experiments (Figures 2, 3, 4, 5E-F), were conducted by labeling neurons only in socially isolated animals, not group-housed animals. The authors labeled neurons after 30-minute social interactions, raising the possibility that the labeled neurons simply represent a "social interaction/behavior population" (mediating mounting and USVs in females and males) rather than a set of neurons specific to social isolation.

      I strongly recommend including experimental groups that involve labeling neurons after 30minute social interactions in group-housed female or male mice and inhibit TRAPed neurons after social isolation or activate TRAPed neurons after group housing. If manipulating the grouphoused TRAP neurons has similar effects to manipulating the isolated TRAP neurons, it would suggest the current labeling paradigm is not isolating neurons specific to the effect of social isolation per se. Rather, the neurons may mediate more general social interaction or motivationrelated activities. Given the known role of POA in male mating behavior, a group-housed TRAP experiment in males with a female visitor is especially important for understanding the selectivity of the labeled cells.

      Without proper controls, referring to the labeled neurons as "POAiso" neurons is potentially misleading. The data thus far suggests these neurons may predominantly reflect a "POA social behavior" population rather than a set of cells distinctly responsive to isolated housing.

      We agree with the reviewer that the POA neurons we are studying regulate the production of social behaviors in females and males, rather than representing a set of cells distinctly responsive to single housing. To more clearly reflect our thinking, we have changed the name of the neurons from “POA-iso neurons” to “POA-social neurons”. Thank you for this helpful criticism.

      Our Fos data are consistent with the idea that the POA may regulate social behaviors in group-housed females (not just single-housed females). Namely, we found that counts of Fospositive POA neurons are significantly related to rates of social investigation (p = 0.01) and tend to be related to USV rates (p = 0.05) in group-housed females that engaged in same-sex interactions (Fig. S1C). We now include two new sets of experiments aimed at further testing the idea this idea. 

      First, we include 2 control groups in which TRAPing sessions were performed in grouphoused females following same-sex interactions. We find that chemogenetic silencing of grouphoused-TRAPed POA neurons fails to reduce social behaviors in females that are subsequently single-housed and given a same-sex social interaction (Fig. 5A-D), and that optogenetic activation of group-housed-TRAPed POA neurons fails to promote female social behavior (Fig. 5E-H). At face value, these findings do not support the idea that the POA contains neurons that regulate social behaviors in group-housed females.

      However, one important caveat is that group-housed females engage in low rates of social behaviors (low investigation time, no mounting, and few USVs), and thus TRAP-based labeling may not work efficaciously in these mice. There may be POA neurons that regulate social behaviors in group-housed females but that do not upregulate Fos following production of relatively low rates of social behaviors. To test this idea, we also include females in which POA neurons are chemogenetically silenced using a viral strategy that does not depend on activitydependent labeling. In this new experiment, we report that silencing of POA neurons significantly reduces USV production in group-housed females (Fig. 5J-L) and significantly reduces social investigation, mounting, and USV production when these same females are retested following single-housing (Fig. 5M-O). Together, these experiments suggest that the POA may regulate the production of social behaviors during same-sex interactions in group-housed females, but that these effects may be difficult to detect in some cases given the low rates at which group-housed females engage in social behaviors during same-sex interactions relative to single-housed females.

      Finally, we want to highlight an additional new dataset that supports the idea that POAsocial neurons regulate social behaviors, rather than encoding the “state” of social isolation. We now include a control group for the chemogenetic silencing of female POA-social neurons, in which females were single-housed but were not given a social interaction prior to 4-OHT treatment (N = 5 non-social controls). Rates of social behaviors were subsequently unaffected following CNO delivery in these females (Fig. S2D-G). These new data support the conclusion that POA-social neurons regulate the production of social behaviors, rather than encoding the state of social isolation. 

      Reviewer 3:

      While the authors should be commended for performing and reporting multiple circuit perturbation experiments (e.g., chemogenetics, ablation), the conflicting effects on behavior are hard to interpret without additional experiments. For example, chemogenetic silencing of the POA neurons (using DREADDs) attenuated all three behavioral measures but the ablation of the same POA neurons (using CASPACE) decreased mounting duration without impacting social investigation or USV production. Similarly, optogenetic activation of POA neurons was sufficient to generate USV production as reported in earlier studies but mounting or social investigation remained unaffected. 

      Do these discrepancies arise due to the efficiency differences between DREADD-mediated silencing vs. Casp3 ablation? Or does the chemogenetic result reflect off-manifold effects on downstream circuitry whereas a more permanent ablation strategy allows other brain regions to compensate due to redundancy? It is important to resolve whether these arise due to technical reasons or whether these reflect the underlying (perhaps messy) logic of neural circuitry. Therefore, while it is clear that POA neurons likely contribute to multiple behavioral readouts of social isolation, understanding their exact roles in any greater detail will require further experiments.

      We have added new analyses to consider the possibility that optogenetic activation of female POA-social neurons promotes social investigation. In the original manuscript, we analyzed the duration of social investigation bouts in POA-social-ChR2 females according to whether they overlapped with laser stimulation or whether they did not overlap. We realized that we made an error in this first analysis and inadvertently included social investigation bouts that occurred during the first 5 minutes of the social sessions, prior to any laser stimulation. Because these earlier bouts tend to be longer duration than later bouts, this mistake washed out the effect of laser stimulation on social bout duration. After correcting that error, we now report that optogenetic activation of female POA-social neurons lengthens social investigation bout duration (Fig. 4G). Inspired by this interesting finding, we also included analyses of the probability of social investigation following laser stimulation (Fig. 4E-F; excluding laser stimulations that were preceded by social investigation in the pre-laser baseline period). These analyses support the conclusion that optogenetic activation of POA-social neurons promotes both USV production and social investigation in group-housed females.  

      The majority of the females that we used in our TRAP2-based ablation experiments were heterozygous for TRAP2 (N = 11 of 15 POA-social-caspase subjects were TRAP2;Ai14 females), whereas all females used in our chemogenetic silencing experiments were homozygous for TRAP2. To test whether a more effective ablation of POA-social neurons might drive decreases in social investigation and USV production, we set up additional TRAP2 homozygous POA-social-caspase females and directly compare the effects of ablation between the two genotypes (Fig. S3; N = 11 hets in total and N = 9 homozygotes in total). These experiments revealed that effects on mounting were more pronounced following POA-social ablation in TRAP2 homozygotes vs. heterozygotes, but that neither group exhibited decreased social investigation or USV production following 4-OHT treatment.

      To ask whether caspase-mediated ablation in TRAP2 homozygotes was effective in eliminating neural activity associated with social behaviors in females, we performed Fos immunostaining in a subset of the POA-social-caspase TRAP2 homozygotes following a samesex interaction. We found that POA Fos expression was robustly reduced in these females relative to control group-housed and control single-housed females that also engaged in samesex interactions, down to levels seen in group-housed and single-housed females that did not engage in a social interaction (comparison shown in Fig. S3D; control female data same as in Fig. 1). Moreover, the remaining POA Fos in these TRAP2 homozygotes was no longer positively correlated to social investigation or USV production (Fig. S3E-F). Together, these findings lead us to favor the interpretation suggested by the reviewer below, that permanent ablation of POA-social neurons leads to compensation from other brain regions due to redundancy. In addition, our finding that optogenetic activation of POA-social neurons promotes both USV production and social investigation supports the idea that POA-social neurons directly regulate these behaviors. We agree with the reviewer that additional work is needed to understand the complex sex- and context-dependent role played by the POA in the regulation of mouse social behaviors.

      Recommendations for the Authors:

      Reviewer 1 Recommendations:

      (1) The largest issue is that many of the stated "key" behavioral findings are not statistically significant.

      (1a) Figure 2C is not significant and Figure 5G is not significant

      We have added N = 5 POA-social-hM4Di females, N = 3 POA-social-hM4Di males, and N = 3 POA-social-GFP males to the dataset. The decrease in mounting following chemogenetic silencing of POA-social neurons is now statistically significant in both sexes (p < 0.05 for both; see current Figs. 2C and 6G). We also simplified our statistical analysis of mounting in these experiments to consider the proportion of trials with and without resident-initiated mounting on saline vs. CNO days, using McNemar’s test for paired proportions. 

      (1b) Mounting graphs are completely omitted in Figure 4. 

      Given that mounting was only observed infrequently in POA-social-ChR2 females, we simply report this information in the Results text (lines 382-388). In our prior summary of the mounting results, we reported that mounting was observed in a total of 3 trials from 2 females, but we inadvertently included information from a duplicate trial from one of the POA-socialChR2 females in this summary (all other analyses of the POA-social-ChR2 females included one trial per female). We have corrected that error and now report that we observed mounting following laser stimulation in 1 trial from 1 POA-social-ChR2 female. We have expanded our consideration of potential effects of optogenetic activation of POA-social neurons on social investigation and include these new analyses as part of Figure 4 (Fig. 4E-G), following the existing analyses of USV production.

      (1c) Figure 3C shows a reduction of mounting following the ablation of POA (although no stats on the graph to denote significance), but this ablation approach can't resolve whether POA is required to encode the state produced by the short period of isolation, and/or whether it needs to be online at test.

      We have now added an asterisk in Fig. 3C to denote a p value less than 0.05. Thank you for catching our oversight.

      We designed our activity-dependent labeling experiments to TRAP and express viruses in POA neurons that increase their activity in conjunction with the production of social behaviors in single-housed females. We believe our findings our most consistent with the conclusion that these neurons regulate the production of social behaviors, rather than encoding the state of social isolation, and we have renamed these neurons as “POA-social” neurons to better reflect our thinking.

      We also now include control experiments (albeit chemogenetic inhibition, not caspase ablation) in which the TRAP2 strategy is used to express hM4Di in the POA of single-housed females that do not experience a social interaction prior to 4-OHT delivery (non-social controls, Fig. S2D-G). We report that chemogenetic inhibition of these neurons does not decrease social behavior in single-housed females during a subsequent same-sex interaction (p > 0.05 for saline vs. CNO rates of social investigation, mounting, and USVs). These additional findings support the idea that the activity of POA-social neurons is related to the production of social behaviors rather than to the state of social isolation. 

      The reviewer is correct that our ablation approach cannot resolve the question of whether POA-social neuronal activity is required online during testing, but our reversible chemogenetic inhibition experiments provide evidence that the activity of POA-social neurons is required online at the time of testing to regulate social behavior.

      (1d) A similar issue is seen regarding investigation (a general lack of significance with most of the LOF and GOF manipulations).

      As reported in the original manuscript, we find that chemogenetic inhibition of POAsocial neurons reduces social investigation in females, while caspase-mediated ablation of female POA-social neurons does not. Our original caspase dataset used mostly but not all TRAP2 heterozygous females (N = 11 TRAP2 heterozygotes (TRAP2;Ai14), generated by crossing TRAP2 mice with Ai14 mice, for the purpose of visualizing the absence of tdTomato labeling to estimate spread of the caspase virus; and N = 4 TRAP2 homozygotes). By adding to the TRAP2 homozygous caspase dataset and comparing the effects on female social behavior of ablation of POA-social neurons in TRAP2 heterozygous vs. TRAP2 homozygous females, we

      now provide evidence that the attenuation of mounting is more efficacious in TRAP2 homozygous females than in heterozygotes (Fig. S3B). Nonetheless, we fail to see effects on social investigation and USV production, even when caspase ablation of POA-social neurons is performed in TRAP2 homozygous females (Fig. S3A,C). 

      In spite of the lack of effect on these behaviors, we show that caspase-mediated ablation of POA-social neurons in TRAP2 homozygous females leads to a dramatic reduction in social interaction-induced Fos expression in the POA. POA Fos expression in these caspase females is reduced to the levels seen in control group-housed and single-housed females that are not given social interactions and are significantly lower than Fos expression in group-housed and single-housed females that are given a same-sex interaction (Fig. S3D). Moreover, the remaining POA Fos expression in the caspase females is no longer related to rates of social investigation (Fig. S3E), as is normally the case in group-housed and single-housed control females (Fig. S1C, left). Together, these data support the idea that some type of neuronal compensation outside of the POA is occurring following ablation of POA-social neurons, and this compensation permits normal levels of USV production and social investigation.

      As in the original manuscript, we report that chemogenetic inhibition of POA-social neurons in male mice reduces mounting but does not reduce social investigation (or USV production). We now include quantification of social behaviors produced by male and female POA-social-hM4Di mice in the TRAPing sessions that preceded 4-OHT delivery (Fig. S5). These measurements show that males spent significantly more time than females engaged in mounting, and we speculate that this bias in TRAPing session behavior might have led to a bias in TRAP-mediated viral labeling of male POA neurons that regulate mounting, at the expense of male POA neurons that regulate social investigation (or USV production).

      We have added new analyses to consider the possibility that optogenetic activation of female POA-social neurons promotes social investigation. In the original manuscript, we analyzed the duration of social investigation bouts in POA-social-ChR2 females according to whether they overlapped with laser stimulation or whether they did not overlap. We realized that we made an error in this first analysis and inadvertently included social investigation bouts that occurred during the first 5 minutes of the social sessions, prior to any laser stimulation. Because these earlier bouts tend to be longer duration than later bouts, this mistake washed out the effect of laser stimulation on social bout duration. After correcting that error, we now report that optogenetic activation of female POA-social neurons lengthens social investigation bout duration (Fig. 4G). Inspired by this encouraging finding, we also included analyses of the probability of social investigation following laser stimulation (Fig. 4E-F; excluding laser stimulations that were preceded by social investigation in the pre-laser baseline period). These analyses support the conclusion that optogenetic activation of POA-social neurons promotes both USV production and social investigation in group-housed females.

      (2) In Figure 1 and elsewhere, the authors use a Mann-Whitney U test, which should be used for non-parametric data, but in other places, they use statistical tests for normally distributed data. Why? How was the normality of distributions tested?

      We tested the normality of data distributions using the Shapiro-Wilk test. Parametric tests were used for analyses that contained normally distributed data, and non-parametric tests were used for analyses that contained non-normally distributed data. This information is included in the Methods (lines 997-1000), and full details of statistical analyses can be found in Table S1.

      (3) The method for "trapping" neurons that are part of the short-term isolation ensemble has some caveats that have not been adequately addressed. First, 4-OHT was administered after social interaction, but before 24 hours of isolation, making it unclear exactly WHAT is being trapped.

      i) Is it neurons that encode the recent 3-day iso experience? (seems unlikely, as this would have been hours after the end of that iso window)

      We now include a group of control females to directly test this possibility (Fig. S2D-G). These TRAP2 females were single-housed for 3 days but were not given a social interaction prior to 4-OHT treatment (N = 5 non-social controls). Presumably, POA neurons TRAPed in these females might encode the experience of short-term isolation. However, we found that chemogenetic inactivation of these TRAPed neurons during a subsequent same-sex interaction failed to decrease social behaviors in single-housed females (Fig. S2E-G; p > 0.05 for CNO vs. saline rates of social investigation, mounting, and USV production). These control experiments support the idea that we are TRAPing neurons whose activity is related to the production of social behaviors, and we have renamed the neurons as “POA-social” neurons to reflect this thinking.

      ii) Is it neurons that encode the recent behavior impacted by the 3-day iso? (this seems to be the goal, but the authors do not provide evidence that the time course of their injection is efficient enough to recruit the recently activated neurons, nor do they provide evidence that opening the trapping window directly after the behavior is better than directly before)

      We opted to perform IP injections of 4-OHT immediately following the behavior session, rather than behavior, due to concern that handling the mice and delivering IP injections prior to behavior sessions would stress the mice, leading to lower rates of social behaviors. The nonsocial female hM4Di experiments described above support the idea that we are TRAPing neurons related to the production of social behaviors, as the reviewer suggests. 

      iii) Is it trapping neurons active during the subsequent 24 hours of isolation? (seems possible, but this would mean that the authors are looking at a different population of neurons than they claim).

      If chemogenetic silencing of POA neurons that were TRAPed following 3-days of social isolation but in the absence of a social interaction (N = 5 non-social controls, Fig. S2D-G) does not alter social behaviors, there is no compelling reason to hypothesize that TRAPing POA neurons activated following the 24 hours of social isolation that follow a social interaction would do so. Moreover, in the original study characterizing the TRAP2 mice (DeNardo et al., 2019), the authors performed experiments to characterize the time course of TRAPing relative to 4-OHT treatment and concluded that the majority of TRAPing occurs within a 6-hour window centered around the 4-OHT injection.

      (4) Relatedly, the authors seem to find a fair bit of variability in their TRAP-mediated experiments. This begs the question - are the effects of their GOF and LOF approaches

      i) dependent on the iso-behaviors that were "trapped" for each animal (in other words, how does behavior at test 1 correlate with behavior at test 2)? 

      To test the reviewer’s idea, we compared rates of TRAPing session behaviors for the POA-social-hM4Di females to the subsequent effects of neuronal silencing on these behaviors (calculated as (CNO behavior – saline behavior). These correlations are shown in Fig. S2A-C and are all non-significant. We also include below for the reviewer the same types of correlations for the other datasets in our study (loss-of-function experiments: female POAsocial-caspase, male POA-social-hM4Di; and gain-of-function experiments: female POA-socialChR2).

      Author response image 1.

      The only loss-of-function experiment comparison in the above figure that reveals a negative and significant correlation is the mounting comparison for the POA-social-hM4Di males (time spent mounting during TRAPing session vs. (CNO time spent mounting -saline time spent mounting). This significant correlation likely reflects that fact that (1) no males mounted in the CNO session and (2) that mounting rates for individual males are relatively consistent over time (in comparison to female mounting, which is more variable; see Author response image 2 below of TRAPing session vs. saline mounting in male vs. female POA-social-hM4Di experiments). The correlation between TRAPing session and testing session mounting is significant for the POA-social-ChR2 females, but despite the significant correlation, we would want to see more instances of optogenetically-elicited mounting to make any claim about its relationship to TRAPing session behavior.

      Author response image 2.

      Nonetheless, we agree with the reviewer’s intuition that one would expect the effects of POA activity manipulations on different behaviors to scale with rates at which these behaviors were performed during the TRAPing session. We speculate that variability in the TRAPing process might have obscured such a relationship. There is inevitable variability in the exact body cavity placement of IP injections, which can affect drug absorption, and another point is that we delivered a fixed volume of 4-OHT (10 mg/mL 4-OHT in 150 uL filtered corn oil) to all mice in the study, regardless of their weight, which likely added variability in TRAPing efficacy from animal to animal. This detail was reported inaccurately in the Methods, and that error has been corrected (line 920). With regard to our male POA-social-hM4Di dataset, we find that these males spend more time mounting during their TRAPing sessions than female POA-socialhM4Di (Fig. S5; males also spent less time investigating and tended to produce fewer USVs than females), a fact that we hypothesize may have led to a bias toward TRAPing mountingrelated POA neurons in male subjects. In addition, however, the fact that male mice typically weigh more than females and would have received a slightly lower effective dosage of 4-OHT may also have contributed to the weaker effects on behavior in the male POA-social-hM4Di experiments relative to the female POA-social-hM4Di experiments.

      We also want to highlight that interpreting correlations for females between time spent mounting during the TRAPing session and time spent mounting during the test sessions can be complicated. For example, we see 2 cases in the female POA-social-hM4Di dataset in which the female did not mount in the TRAPing session, and then mounted on the saline day (12s and 10s total mounting for those 2 females) but not on the CNO day. One interpretation of the data from these 2 females is that mounting on the TRAPing day is not required to attenuate mounting on the later test days. However, female mounting behavior itself is variable, both across different females and across different tests of a given female, as noted above. If we consider all singlehoused females included in our dataset for which we quantified control behavioral data (i.e., behavior trials from unmanipulated females and TRAPing sessions from females that were later manipulated), we find that mounting is not observed in ~30% of the females (24 of 83). In ongoing behavioral experiments not included in this manuscript, we are investigating factors that regulate female mounting following single-housing. In that dataset, we also see little evidence that female mounting in one social interaction predicts mounting in a subsequent interaction

      (i.e., there don’t appear to stable “high mounters” and “low mounters” following single housing). Thus, the small number of cases in which females did not mount in the TRAPing session and then displayed mounting on the CNO only day are difficult to interpret. 

      Two additional considerations are that TRAPing may not be equally efficacious for POA neurons that regulate different behaviors, and that different behaviors may be differentially sensitive to perturbations of the POA. Previous elegant calcium imaging work has shown that different subsets of Esr1+ POA neurons exhibit activity that is “tuned” to specific behaviors (sniffing vs. mounting in males interacting with females; Yang et al., 2023). However, it is possible that these subsets of neurons display differential levels of Fos expression following the production of their preferred behavior and that some behavior-related subsets may thus be more easily TRAPed than others. It may also be the case that some behaviors are more easily disrupted by POA activity manipulations than others (e.g., perturbation in a smaller percentage of behavior-related POA neurons may be required to disrupt some behaviors relative to others). 

      Despite these caveats, we have two lines of evidence that the effects of chemogenetic silencing of POA-social neurons depends on the behaviors produced during the TRAPing sessions.

      (1) Social behavior is required during the TRAPing session to see subsequent effects on social behavior following chemogenetic silencing of TRAPed POA neurons. In control females that were single-housed but were not given a social interaction prior to 4OHT treatment, social behaviors are not reduced by chemogenetic silencing of TRAPed POA neurons (Figs. S2D-G).

      (2) To directly test whether mounting in the TRAPing session is required to see attenuation of mounting during subsequent chemogenetic silencing of POA-social neurons, we performed control experiments in which single-housed females interacted with a female visitor that was placed under a cup during the TRAPing session prior to 4-OHT treatment. Mounting was not possible in this context, and we also found that females produced lower rates of USVs during the TRAPing session relative to single-housed females engaged in free social interaction. However, subject females spent more time engaged in social investigation of the visitor relative to single-housed females engaged in free social interactions (see Author response image 3 below).

      Author response image 3.

      Unfortunately, none of the experimental females in this cohort displayed mounting in the CNO or saline sessions. Given that we could use this dataset to address the intended question, we did not include it in the manuscript. However, it is quite interesting that female subjects displayed higher than normal social investigation and lower than normal USV production in their TRAPing sessions (relative to single-housed females engaged in free interactions), and subsequently, chemogenetic inhibition of TRAPed POA neurons decreased social investigation but did not decrease USV production (Author response image 4 below). 

      Author response image 4.

      Together, we think our data support the idea that the POA neurons that are TRAPed are related to the social behaviors performed by the animals, but these relationships may be complex and difficult to detect from comparisons across animals within a single experimental group.

      And/or are they

      ii) influenced by the spread or amount of virus for each animal? These correlations could help shed light on what exactly is being trapped - is it specific behaviors or is it the "state" of shortterm isolation?

      Our control experiments with females that were single-housed but did not receive a social interaction prior to 4-OHT treatment provide evidence that the production of social behaviors is required to see subsequent effects on behavior following chemogenetic inhibition of TRAPed POA neurons (Figs. S2D-G).

      The same volume of virus was injected across all activity manipulation experiments (200 nL). Because of the trajectory of our POA viral injections (performed at a slight rostral angle relative to vertical), we did sometimes see viral labeling that spread into the AH caudal to the POA. For this reason, we included the AH TRAPed control group (Fig. 2), to rule out the possibility that viral spread into the AH could account for the effects of chemogenetic silencing of POA-social neurons on female social behaviors. Also because of the injection angle used, we don’t see substantial viral spread rostral to our injection coordinates. In short, there isn’t systematic variability in the targeting or spread of our POA viral injections that can account for variability in the effects on USV production and social investigation of our LOF and GOF manipulations (female hM4Di and female ChR2 experiments).

      In older lesion studies in male rodents and birds, there is some support for the idea that rostral vs. caudal POA neurons differentially regulate appetitive vs. consummatory sexual behaviors (as reviewed in Balthazart and Ball, 2007). However, all of our viral injections were placed in what that review paper would have considered ‘caudal’ POA. We also note that more recent imaging studies have reported that subsets of POA neurons are differentially tuned to male sniffing vs. male mounting (Yang et al.,2023), and these subsets must be relatively co-localized given that they are imaged in the same field of view. Whether distinct subsets of POA neurons regulate the production of different female social behaviors, and if so, how these subsets are localized within the POA, remains an important question for future study.

      (5) The authors label their region of interest as the "POA" but images throughout (e.g. their fos image, Figure 1E), look more like the MPO. Why label it POA?

      The POA neurons in our study are found in a band that spans the medial POA, as well as a bit of the lateral POA. To avoid over-specifying, we call this region the POA more generally.

      (6) In all the experiments, mice are isolated and then re-group housed with siblings. Do all the siblings in the group belong to the same experimental group, or are siblings naïve? This may be critical to help determine whether some of the effects observed may be "group" effects.

      In general, multiple (although not always all) mice in a cage belonged to the same experimental group. In our inhibitory DREADDs experiments, it is unclear how that could drive our observed effects on behavior, given that home cage behavior would only be expected to differ for a given mouse in the time period following their CNO session. 

      For the female POA-social-caspase mice, we cannot rule out the possibility that their home cage behaviors differed in the time period following 4-OHT treatment and re-grouphousing and prior to post-4-OHT behavior measurements. However, given that the only social behavior affected by ablation of POA-social neurons was mounting, and that rates of mounting would be expected to be very low in group-housed females within home cages, it is unclear how our experimental result could be attributed to group effects.

      If by “group” effects the reviewer means “litter” effects, we include a plot below that shows the CNO vs. saline behaviors for the POA-social-hM4Di females, separated by cage ID. There is no evidence that the effects of chemogenetic silencing of POA-social-hM4Di females are being driven by only certain cages (only social investigation and USVs are shown, because mounting was uniformly low (1 of 17 females mounted) in the CNO session).

      Author response image 5.

      (7) For chemogenetic experiments, the authors state that CNO and Saline were given in a counterbalanced order (eg line 189). Did the authors see any order effects?

      We did not see order effects, and we can include plots of those data below for the female and male POA-social-hM4Di groups, with mice plotted according to which treatment they received first.

      Author response image 6.

      (8) In the control experiments in Figure 2 where VMH or AH are chemogenetically silenced, it isn't clear whether these groups include mice that were subjected to 3 days of isolation. Please clarify.

      Yes, these female groups were also subjected to 3 days of isolation (first prior to the TRAPing session, and for a second time prior to the onset of the CNO/saline testing sessions). That information has been clarified in the Results section (line 214) and in the Methods (lines 935-938).

      (9) Line 312. The title for this section, "POA neurons increase their activity....." is somewhat misleading. It sounds like the authors imaged trapped neurons. I think what they mean is that more POA neurons are activated following opposite-sex interactions with males.

      Thanks for this catch. We have modified the section title, as well as the title of the first results sub-section.

      (10) Figure 5A, right panels. The authors fail to find an increase in the investigation of male-male pairs following the short-term isolation of one. This contrasts with the main finding in Matthews et al., 2016 Cell, where short periods of isolation are said to promote pro-social behaviors. The authors could comment on this discrepancy in their discussion (eg difference in testing apparatus/test type? Difference in the number of days of isolation? etc.).

      In current Fig. 6A, there is no significant interaction between the two main effects, but each main effect is significant: single-housed males spend more time investigating partners than group-housed males, and males spend more time investigating female partners than male partners. The significant main effect of housing condition is consistent with the findings of Matthews et al., 2016 and is included within the Results (lines 486-492). 

      (11) Figure 5F, the authors seem to have a main effect of virus (more overall investigation in dreadds mice). Nothing about this is addressed.

      We sometimes see differences in social behavior between cohorts of males when they are tested at different times and, correspondingly, with different groups of female social partners. Our POA-social-hM4Di and POA-social-GFP males were set-up and tested at largely non-overlapping times. We have added a brief note to the Results section to include this information (lines 535-539).

      Reviewer 2 Recommendations:

      (1) (C)ritical control experiments are missing to support this claim (that a population of preoptic hypothalamic neurons contribute to the effects of short-term social isolation on the social behaviors of female mice).  

      (1a) All the activity-dependent labeling experiments with TRAP mice, including the subsequent neural activity manipulation experiments (Figures 2, 3, 4, 5E-F), were conducted by labeling neurons only in socially isolated animals, not group-housed animals. The authors labeled neurons after 30-minute social interactions, raising the possibility that the labeled neurons simply represent a "social interaction/behavior population" (mediating mounting and USVs in females and males) rather than a set of neurons specific to social isolation behaviors of mice)… The data thus far suggests these neurons may predominantly reflect a "POA social behavior" population rather than a set of cells distinctly responsive to isolated housing.

      We agree with the reviewer that the POA neurons we are studying regulate the production of social behaviors in females and males, rather than representing a set of cells distinctly responsive to single housing. To more clearly reflect our thinking, we have changed the name of the neurons from “POA-iso neurons” to “POA-social neurons”. Thank you for this helpful criticism.

      Our Fos data are consistent with the idea that the POA may regulate social behaviors in group-housed females (not just single-housed females). Namely, we found that counts of Fospositive POA neurons are significantly related to rates of social investigation (p = 0.01) and tend to be related to USV rates (p = 0.05) in group-housed females that engaged in same-sex interactions (Fig. S1C). We now include two new sets of experiments aimed at further testing the idea this idea. 

      First, we include 2 control groups in which TRAPing sessions were performed in grouphoused females following same-sex interactions. We find that chemogenetic silencing of these group-housed-TRAPed POA neurons fails to reduce social behaviors in females that are subsequently single-housed and given a same-sex social interaction (Fig. 5A-D; GH-TRAPed POA hM4Di females), and that optogenetic activation of group-housed-TRAPed POA neurons fails to promote female social behavior (Fig. 5E-H; GH-TRAPed POA ChR2 females). At face value, these findings do not support the idea that the POA contains neurons that regulate social behaviors in group-housed females.

      However, one important caveat is that group-housed females engage in low rates of social behaviors (low investigation time, no mounting, and few USVs), and thus TRAP-based labeling may not work efficaciously in these mice. There may be POA neurons that regulate social behaviors in group-housed females but that do not upregulate Fos following production of relatively low rates of social behaviors. To test this idea, we also include females in which POA neurons are chemogenetically silenced using a viral strategy that does not depend on activitydependent labeling. In this new experiment, we report that silencing of POA neurons significantly reduces USV production in group-housed females (Fig. 5J-L) and significantly reduces social investigation, mounting, and USV production when these same females are retested following single-housing (Fig. 5M-O).

      (2) Please add strain background information of subject animals in the methods.

      This information has been added to the Animals section within the Methods (lines 788802).

      Responses to Reviewer 3 Recommendations:

      (1a) (T)he conflicting effects on behavior are hard to interpret without additional experiments….Similarly, optogenetic activation of POA neurons was sufficient to generate USV production as reported in earlier studies but mounting or social investigation remained unaffected. 

      We have added new analyses to consider the possibility that optogenetic activation of female POA-social neurons promotes social investigation. In the original manuscript, we analyzed the duration of social investigation bouts in POA-social-ChR2 females according to whether they overlapped with laser stimulation or whether they did not overlap. We realized that we made an error in this first analysis and inadvertently included social investigation bouts that occurred during the first 5 minutes of the social sessions, prior to any laser stimulation. Because these earlier bouts tend to be longer duration than later bouts, this mistake washed out the effect of laser stimulation on social bout duration. After correcting that error, we now report that optogenetic activation of female POA-social neurons lengthens social investigation bout duration (Fig. 4G). Inspired by this interesting finding, we also included analyses of the probability of social investigation following laser stimulation (Fig. 4E-F; excluding laser stimulations that were preceded by social investigation in the pre-laser baseline period). These analyses support the conclusion that optogenetic activation of POA-social neurons promotes both USV production and social investigation in group-housed females.

      (1b) Do these discrepancies (between hM4Di and caspase) arise due to the efficiency differences between DREADD-mediated silencing vs. Casp3 ablation? Or does the chemogenetic result reflect off-manifold effects on downstream circuitry whereas a more permanent ablation strategy allows other brain regions to compensate due to redundancy? It is important to resolve whether these arise due to technical reasons or whether these reflect the underlying (perhaps messy) logic of neural circuitry.  

      The possibility that the difference in effects on behavior between chemogenetic silencing and caspase ablation at face value seems inconsistent with the findings of previous experiments, in which ablation of large numbers of POA neurons failed to reduce USV production in male mice (POA lesions in Bean et al., 1981; ablation of VGAT+ POA neurons by Gao et al., 2018). These findings stand in contrast to those using chemogenetic silencing of large numbers of POA neurons, which report reduced USV production in male mice (VGAT+/Esr1+ in Karigo et al., 2021; Esr1+ in Chen et al., 2021).

      However, it is the case that the majority of the females that we used in our TRAP2-based ablation experiments were heterozygous for TRAP2 (N = 11 of 15 POA-social-caspase subjects were TRAP2;Ai14 females), whereas all females used in our chemogenetic silencing experiments were homozygous for TRAP2. To test whether a more effective ablation of POAsocial neurons might drive decreases in social investigation and USV production, we set up additional TRAP2 homozygous POA-social-caspase females and directly compare the effects of ablation between the two genotypes (Fig. S3; N = 11 hets in total and N = 9 homozygotes in total). These experiments revealed that effects on mounting were more pronounced following POA-social ablation in TRAP2 homozygotes vs. heterozygotes, but that neither group exhibited decreased social investigation or USV production following 4-OHT treatment.

      To ask whether caspase-mediated ablation in TRAP2 homozygotes was effective in eliminating neural activity associated with social behaviors in females, we performed Fos immunostaining in a subset of the POA-social-caspase TRAP2 homozygotes following a samesex interaction. We found that POA Fos expression was robustly reduced in these females relative to control group-housed and control single-housed females that also engaged in samesex interactions, down to levels seen in group-housed and single-housed females that did not engage in a social interaction (comparison shown in Fig. S3D; control female data same as in Fig. 1). Moreover, the remaining POA Fos in these TRAP2 homozygotes was no longer positively correlated to social investigation or USV production (Fig. S3E-F). Together, these findings lead us to favor the interpretation suggested by the reviewer below, that permanent ablation of POA-social neurons leads to compensation from other brain regions due to redundancy.

      Given the negative results above, we favor this possibility and indicate so in our Discussion. In addition, our finding that optogenetic activation of POA-social neurons promotes both USV production and social investigation supports the idea that POA-social neurons directly regulate these behaviors. We agree with the reviewer that additional work is needed to understand the complex sex- and context-dependent role played by the POA in the regulation of mouse social behaviors.

      (2) L 49: Please define Mesolimbic circuitry the first time it is mentioned.

      We have added a definition (lines 52-53).

      (3) L 210: In Figure 2C, the mounting duration baseline (saline) distribution seems lower than the same experimental baseline in Figures 1C and 3C. Does this reflect natural variability in the behavioral assay and might this be mitigated by additional sampling of animals?

      Yes, there is substantial variability in the display of mounting behavior by single-housed females, including in the proportion of trials with mounting as well as in the total duration of mounting. In the revised manuscript, we have simplified our analysis of mounting in our TRAPbased experiments to quantify the proportion of trials with mounting, rather than considering the total time spent mounting. After adding N = 5 additional females to the POA-social-hM4Di dataset, we now report a statistically significant decrease in the proportion of trials with mounting following chemogenetic silencing of POA-social neurons (Fig. 2C; McNemar’s test for paired proportions). 

      (4) L 310: The authors claim that "These findings suggest that a subset of POAiso neurons overlap with GABAergic, PAG-projecting POA neurons that have been demonstrated in previous work to promote USVs via disinhibition of excitatory PAG neurons important to USV production (Chen et al., 2021; Michael et al., 2020)." I think the data reported suggests the opposite since only 18.3% of all POA->PAG neurons are cFos+. Perhaps better rephrased as "A subset (18.3%) of POA->PAG neurons are labelled by cFos and that is sufficient to drive the production of USVs". Is it surprising?

      We modified the phrasing (lines 468-469), but a bit differently than suggested above, because although we suspect that optogenetic activation of the PAG-projecting neurons within the larger population of POA-social neurons is responsible for eliciting USV production, we did not technically demonstrate this to be the case in the current dataset. 

      We do find it surprising that so few (only ~20%) of PAG-projecting POA neurons upregulate Fos following female-female interactions marked by high rates of USV production. Even though optogenetic activation of PAG-projecting POA neurons elicits USV production, our finding suggests that the majority of PAG-projecting POA neurons may not play a role in regulating vocalization. In future work, it may be useful to apply an intersectional approach to further understand how the POA regulates USV production (for example, measure or manipulate activity selectively in projection-defined subsets of POA-social neurons).

      (5) Given the considerable prior evidence of POA->PAG circuit in promoting USVs, it is hard to understand why chemogenetic inactivation of POA neurons in males affects mounting but not USV production (Figures 5F-H). Any potential explanation for this discrepancy?

      We have two ideas about this surprising result. First, we examined the TRAPing session social behaviors of female and male POA-social-hM4Di mice. We found that male POA-socialhM4Di mice spent more time than female subjects mounting during the TRAPing sessions, and conversely, males spent less time investigating visitors and tended to produce fewer USVs than female subjects (Fig. S5). Given that our labeling method is activity-dependent, one possibility is that this bias in behavior is reflected in a bias toward labeling of POA neurons related to mounting.  

      Second, each mouse in the TRAP2-based hM4Di datasets received an IP injection of the same amount of 4-OHT (150 nL of 10 mg/mL 4-OHT in filtered corn oil) not adjusted for weight of the mouse. This information was not reported accurately in the Methods, and we have adjusted that section accordingly (line 920). As a result, because male mice typically weigh more than females and would have received a lower effective dosage of 4-OHT, another possibility is that TRAPing in males was less efficient than in females and accounts for the less complete effects on social behaviors. We have added language to the Results to discuss these possibilities (lines 540-560).

      (6) L 472: Typo. "we found that short-term isolation exerts more robust on the effects of male behavior during subsequent interactions with females than during interactions with males."

      Thank you for catching this mistake.

    1. Author response:

      The following is the authors’ response to the original reviews.

      Public Reviews:

      Reviewer #1 (Public Review):

      Summary:

      In this study, the authors address whether the dorsal nucleus of the inferior colliculus (DCIC) in mice encodes sound source location within the front horizontal plane (i.e., azimuth). They do this using volumetric two-photon Ca2+ imaging and high-density silicon probes (Neuropixels) to collect single-unit data. Such recordings are beneficial because they allow large populations of simultaneous neural data to be collected. Their main results and the claims about those results are the following:

      (1) DCIC single-unit responses have high trial-to-trial variability (i.e., neural noise);

      (2) approximately 32% to 40% of DCIC single units have responses that are sensitive tosound source azimuth;

      (3) single-trial population responses (i.e., the joint response across all sampled single unitsin an animal) encode sound source azimuth "effectively" (as stated in title) in that localization decoding error matches average mouse discrimination thresholds;

      (4) DCIC can encode sound source azimuth in a similar format to that in the central nucleusof the inferior colliculus (as stated in Abstract);

      (5) evidence of noise correlation between pairs of neurons exists;

      and 6) noise correlations between responses of neurons help reduce population decoding error.

      While simultaneous recordings are not necessary to demonstrate results #1, #2, and #4, they are necessary to demonstrate results #3, #5, and #6.

      Strengths:

      - Important research question to all researchers interested in sensory coding in the nervous system.

      - State-of-the-art data collection: volumetric two-photon Ca2+ imaging and extracellularrecording using high-density probes. Large neuronal data sets.

      - Confirmation of imaging results (lower temporal resolution) with more traditionalmicroelectrode results (higher temporal resolution).

      - Clear and appropriate explanation of surgical and electrophysiological methods. I cannot comment on the appropriateness of the imaging methods.

      Strength of evidence for claims of the study:

      (1) DCIC single-unit responses have high trial-to-trial variability - The authors' data clearlyshows this.

      (2) Approximately 32% to 40% of DCIC single units have responses that are sensitive tosound source azimuth - The sensitivity of each neuron's response to sound source azimuth was tested with a Kruskal-Wallis test, which is appropriate since response distributions were not normal. Using this statistical test, only 8% of neurons (median for imaging data) were found to be sensitive to azimuth, and the authors noted this was not significantly different than the false positive rate. The Kruskal-Wallis test was not performed on electrophysiological data. The authors suggested that low numbers of azimuth-sensitive units resulting from the statistical analysis may be due to the combination of high neural noise and relatively low number of trials, which would reduce statistical power of the test. This may be true, but if single-unit responses were moderately or strongly sensitive to azimuth, one would expect them to pass the test even with relatively low statistical power. At best, if their statistical test missed some azimuthsensitive units, they were likely only weakly sensitive to azimuth. The authors went on to perform a second test of azimuth sensitivity-a chi-squared test-and found 32% (imaging) and 40% (e-phys) of single units to have statistically significant sensitivity. This feels a bit like fishing for a lower p-value. The Kruskal-Wallis test should have been left as the only analysis. Moreover, the use of a chi-squared test is questionable because it is meant to be used between two categorical variables, and neural response had to be binned before applying the test.

      The determination of what is a physiologically relevant “moderate or strong azimuth sensitivity” is not trivial, particularly when comparing tuning across different relays of the auditory pathway like the CNIC, auditory cortex, or in our case DCIC, where physiologically relevant azimuth sensitivities might be different. This is likely the reason why azimuth sensitivity has been defined in diverse ways across the bibliography (see Groh, Kelly & Underhill, 2003 for an early discussion of this issue). These diverse approaches include reaching a certain percentage of maximal response modulation, like used by Day et al. (2012, 2015, 2016) in CNIC, and ANOVA tests, like used by Panniello et al. (2018) and Groh, Kelly & Underhill (2003) in auditory cortex and IC respectively. Moreover, the influence of response variability and biases in response distribution estimation due to limited sampling has not been usually accounted for in the determination of azimuth sensitivity.

      As Reviewer #1 points out, in our study we used an appropriate ANOVA test (KruskalWallis) as a starting point to study response sensitivity to stimulus azimuth at DCIC. Please note that the alpha = 0.05 used for this test is not based on experimental evidence about physiologically relevant azimuth sensitivity but instead is an arbitrary p-value threshold. Using this test on the electrophysiological data, we found that ~ 21% of the simultaneously recorded single units reached significance (n = 4 mice). Nevertheless these percentages, in our small sample size (n = 4) were not significantly different from our false positive detection rate (p = 0.0625, Mann-Whitney, See Author response image 1 below).  In consequence, for both our imaging (Fig. 3C) and electrophysiological data, we could not ascertain if the percentage of neurons reaching significance in these ANOVA tests were indeed meaningfully sensitive to azimuth or this was due to chance. 

      Author response image 1.

      Percentage of the neuropixels recorded DCIC single units across mice that showed significant median response tuning, compared to false positive detection rate (α = 0.05, chance level).

      We reasoned that the observed markedly variable responses from DCIC units, which frequently failed to respond in many trials (Fig. 3D, 4A), in combination with the limited number of trial repetitions we could collect, results in under-sampled response distribution estimations. This under-sampling can bias the determination of stochastic dominance across azimuth response samples in Kruskal-Wallis tests. We would like to highlight that we decided not to implement resampling strategies to artificially increase the azimuth response sample sizes with “virtual trials”, in order to avoid “fishing for a smaller p-value”, when our collected samples might not accurately reflect the actual response population variability.

      As an alternative to hypothesis testing based on ranking and determining stochastic dominance of one or more azimuth response samples (Kruskal-Wallis test), we evaluated the overall statistical dependency to stimulus azimuth of the collected responses.  To do this we implement the Chi-square test by binning neuronal responses into categories. Binning responses into categories can reduce the influence of response variability to some extent, which constitutes an advantage of the Chi-square approach, but we note the important consideration that these response categories are arbitrary.

      Altogether, we acknowledge that our Chi-square approach to define azimuth sensitivity is not free of limitations and despite enabling the interrogation of azimuth sensitivity at DCIC, its interpretability might not extend to other brain regions like CNIC or auditory cortex. Nevertheless we hope the aforementioned arguments justify why the Kruskal-Wallis test simply could not “have been left as the only analysis”.

      (3) Single-trial population responses encode sound source azimuth "effectively" in that localization decoding error matches average mouse discrimination thresholds - If only one neuron in a population had responses that were sensitive to azimuth, we would expect that decoding azimuth from observation of that one neuron's response would perform better than chance. By observing the responses of more than one neuron (if more than one were sensitive to azimuth), we would expect performance to increase. The authors found that decoding from the whole population response was no better than chance. They argue (reasonably) that this is because of overfitting of the decoder modeltoo few trials used to fit too many parameters-and provide evidence from decoding combined with principal components analysis which suggests that overfitting is occurring. What is troubling is the performance of the decoder when using only a handful of "topranked" neurons (in terms of azimuth sensitivity) (Fig. 4F and G). Decoder performance seems to increase when going from one to two neurons, then decreases when going from two to three neurons, and doesn't get much better for more neurons than for one neuron alone. It seems likely there is more information about azimuth in the population response, but decoder performance is not able to capture it because spike count distributions in the decoder model are not being accurately estimated due to too few stimulus trials (14, on average). In other words, it seems likely that decoder performance is underestimating the ability of the DCIC population to encode sound source azimuth.

      To get a sense of how effective a neural population is at coding a particular stimulus parameter, it is useful to compare population decoder performance to psychophysical performance. Unfortunately, mouse behavioral localization data do not exist. Therefore, the authors compare decoder error to mouse left-right discrimination thresholds published previously by a different lab. However, this comparison is inappropriate because the decoder and the mice were performing different perceptual tasks. The decoder is classifying sound sources to 1 of 13 locations from left to right, whereas the mice were discriminating between left or right sources centered around zero degrees. The errors in these two tasks represent different things. The two data sets may potentially be more accurately compared by extracting information from the confusion matrices of population decoder performance. For example, when the stimulus was at -30 deg, how often did the decoder classify the stimulus to a lefthand azimuth? Likewise, when the stimulus was +30 deg, how often did the decoder classify the stimulus to a righthand azimuth?

      The azimuth discrimination error reported by Lauer et al. (2011) comes from engaged and highly trained mice, which is a very different context to our experimental setting with untrained mice passively listening to stimuli from 13 random azimuths. Therefore we did not perform analyses or interpretations of our results based on the behavioral task from Lauer et al. (2011) and only made the qualitative observation that the errors match for discussion.

      We believe it is further important to clarify that Lauer et al. (2011) tested the ability of mice to discriminate between a positively conditioned stimulus (reference speaker at 0º center azimuth associated to a liquid reward) and a negatively conditioned stimulus (coming from one of five comparison speakers positioned at 20º, 30º, 50º, 70 and 90º azimuth, associated to an electrified lickport) in a conditioned avoidance task. In this task, mice are not precisely “discriminating between left or right sources centered around zero degrees”, making further analyses to compare the experimental design of Lauer et al (2011) and ours even more challenging for valid interpretation.

      (4) DCIC can encode sound source azimuth in a similar format to that in the central nucleusof the inferior colliculus - It is unclear what exactly the authors mean by this statement in the Abstract. There are major differences in the encoding of azimuth between the two neighboring brain areas: a large majority of neurons in the CNIC are sensitive to azimuth (and strongly so), whereas the present study shows a minority of azimuth-sensitive neurons in the DCIC. Furthermore, CNIC neurons fire reliably to sound stimuli (low neural noise), whereas the present study shows that DCIC neurons fire more erratically (high neural noise).

      Since sound source azimuth is reported to be encoded by population activity patterns at CNIC (Day and Delgutte, 2013), we refer to a population activity pattern code as the “similar format” in which this information is encoded at DCIC. Please note that this is a qualitative comparison and we do not claim this is the “same format”, due to the differences the reviewer precisely describes in the encoding of azimuth at CNIC where a much larger majority of neurons show stronger azimuth sensitivity and response reliability with respect to our observations at DCIC. By this qualitative similarity of encoding format we specifically mean the similar occurrence of activity patterns from azimuth sensitive subpopulations of neurons in both CNIC and DCIC, which carry sufficient information about the stimulus azimuth for a sufficiently accurate prediction with regard to the behavioral discrimination ability.

      (5) Evidence of noise correlation between pairs of neurons exists - The authors' data andanalyses seem appropriate and sufficient to justify this claim.

      (6) Noise correlations between responses of neurons help reduce population decodingerror - The authors show convincing analysis that performance of their decoder increased when simultaneously measured responses were tested (which include noise correlation) than when scrambled-trial responses were tested (eliminating noise correlation). This makes it seem likely that noise correlation in the responses improved decoder performance. The authors mention that the naïve Bayesian classifier was used as their decoder for computational efficiency, presumably because it assumes no noise correlation and, therefore, assumes responses of individual neurons are independent of each other across trials to the same stimulus. The use of decoder that assumes independence seems key here in testing the hypothesis that noise correlation contains information about sound source azimuth. The logic of using this decoder could be more clearly spelled out to the reader. For example, if the null hypothesis is that noise correlations do not carry azimuth information, then a decoder that assumes independence should perform the same whether population responses are simultaneous or scrambled. The authors' analysis showing a difference in performance between these two cases provides evidence against this null hypothesis.

      We sincerely thank the reviewer for this careful and detailed consideration of our analysis approach. Following the reviewer’s constructive suggestion, we justified the decoder choice in the results section at the last paragraph of page 18:

      “To characterize how the observed positive noise correlations could affect the representation of stimulus azimuth by DCIC top ranked unit population responses, we compared the decoding performance obtained by classifying the single-trial response patterns from top ranked units in the modeled decorrelated datasets versus the acquired data (with noise correlations). With the intention to characterize this with a conservative approach that would be less likely to find a contribution of noise correlations as it assumes response independence, we relied on the naive Bayes classifier for decoding throughout the study. Using this classifier, we observed that the modeled decorrelated datasets produced stimulus azimuth prediction error distributions that were significantly shifted towards higher decoding errors (Fig. 5B, C) and, in our imaging datasets, were not significantly different from chance level (Fig. 5B). Altogether, these results suggest that the detected noise correlations in our simultaneously acquired datasets can help reduce the error of the IC population code for sound azimuth.”

      Minor weakness:

      - Most studies of neural encoding of sound source azimuth are done in a noise-free environment, but the experimental setup in the present study had substantial background noise. This complicates comparison of the azimuth tuning results in this study to those of other studies. One is left wondering if azimuth sensitivity would have been greater in the absence of background noise, particularly for the imaging data where the signal was only about 12 dB above the noise. The description of the noise level and signal + noise level in the Methods should be made clearer. Mice hear from about 2.5 - 80 kHz, so it is important to know the noise level within this band as well as specifically within the band overlapping with the signal.

      We agree with the reviewer that this information is useful. In our study, the background R.M.S. SPL during imaging across the mouse hearing range (2.5-80kHz) was 44.53 dB and for neuropixels recordings 34.68 dB. We have added this information to the methods section of the revised manuscript.

      Reviewer #2 (Public Review):

      In the present study, Boffi et al. investigate the manner in which the dorsal cortex of the of the inferior colliculus (DCIC), an auditory midbrain area, encodes sound location azimuth in awake, passively listening mice. By employing volumetric calcium imaging (scanned temporal focusing or s-TeFo), complemented with high-density electrode electrophysiological recordings (neuropixels probes), they show that sound-evoked responses are exquisitely noisy, with only a small portion of neurons (units) exhibiting spatial sensitivity. Nevertheless, a naïve Bayesian classifier was able to predict the presented azimuth based on the responses from small populations of these spatially sensitive units. A portion of the spatial information was provided by correlated trial-to-trial response variability between individual units (noise correlations). The study presents a novel characterization of spatial auditory coding in a non-canonical structure, representing a noteworthy contribution specifically to the auditory field and generally to systems neuroscience, due to its implementation of state-of-the-art techniques in an experimentally challenging brain region. However, nuances in the calcium imaging dataset and the naïve Bayesian classifier warrant caution when interpreting some of the results.

      Strengths:

      The primary strength of the study lies in its methodological achievements, which allowed the authors to collect a comprehensive and novel dataset. While the DCIC is a dorsal structure, it extends up to a millimetre in depth, making it optically challenging to access in its entirety. It is also more highly myelinated and vascularised compared to e.g., the cerebral cortex, compounding the problem. The authors successfully overcame these challenges and present an impressive volumetric calcium imaging dataset. Furthermore, they corroborated this dataset with electrophysiological recordings, which produced overlapping results. This methodological combination ameliorates the natural concerns that arise from inferring neuronal activity from calcium signals alone, which are in essence an indirect measurement thereof.

      Another strength of the study is its interdisciplinary relevance. For the auditory field, it represents a significant contribution to the question of how auditory space is represented in the mammalian brain. "Space" per se is not mapped onto the basilar membrane of the cochlea and must be computed entirely within the brain. For azimuth, this requires the comparison between miniscule differences between the timing and intensity of sounds arriving at each ear. It is now generally thought that azimuth is initially encoded in two, opposing hemispheric channels, but the extent to which this initial arrangement is maintained throughout the auditory system remains an open question. The authors observe only a slight contralateral bias in their data, suggesting that sound source azimuth in the DCIC is encoded in a more nuanced manner compared to earlier processing stages of the auditory hindbrain. This is interesting, because it is also known to be an auditory structure to receive more descending inputs from the cortex.

      Systems neuroscience continues to strive for the perfection of imaging novel, less accessible brain regions. Volumetric calcium imaging is a promising emerging technique, allowing the simultaneous measurement of large populations of neurons in three dimensions. But this necessitates corroboration with other methods, such as electrophysiological recordings, which the authors achieve. The dataset moreover highlights the distinctive characteristics of neuronal auditory representations in the brain. Its signals can be exceptionally sparse and noisy, which provide an additional layer of complexity in the processing and analysis of such datasets. This will be undoubtedly useful for future studies of other less accessible structures with sparse responsiveness.

      Weaknesses:

      Although the primary finding that small populations of neurons carry enough spatial information for a naïve Bayesian classifier to reasonably decode the presented stimulus is not called into question, certain idiosyncrasies, in particular the calcium imaging dataset and model, complicate specific interpretations of the model output, and the readership is urged to interpret these aspects of the study's conclusions with caution.

      I remain in favour of volumetric calcium imaging as a suitable technique for the study, but the presently constrained spatial resolution is insufficient to unequivocally identify regions of interest as cell bodies (and are instead referred to as "units" akin to those of electrophysiological recordings). It remains possible that the imaging set is inadvertently influenced by non-somatic structures (including neuropil), which could report neuronal activity differently than cell bodies. Due to the lack of a comprehensive ground-truth comparison in this regard (which to my knowledge is impossible to achieve with current technology), it is difficult to imagine how many informative such units might have been missed because their signals were influenced by spurious, non-somatic signals, which could have subsequently misled the models. The authors reference the original Nature Methods article (Prevedel et al., 2016) throughout the manuscript, presumably in order to avoid having to repeat previously published experimental metrics. But the DCIC is neither the cortex nor hippocampus (for which the method was originally developed) and may not have the same light scattering properties (not to mention neuronal noise levels). Although the corroborative electrophysiology data largely eleviates these concerns for this particular study, the readership should be cognisant of such caveats, in particular those who are interested in implementing the technique for their own research.

      A related technical limitation of the calcium imaging dataset is the relatively low number of trials (14) given the inherently high level of noise (both neuronal and imaging). Volumetric calcium imaging, while offering a uniquely expansive field of view, requires relatively high average excitation laser power (in this case nearly 200 mW), a level of exposure the authors may have wanted to minimise by maintaining a low the number of repetitions, but I yield to them to explain.

      We assumed that the levels of heating by excitation light measured at the neocortex in Prevedel et al. (2016), were representative for DCIC also. Nevertheless, we recognize this approximation might not be very accurate, due to the differences in tissue architecture and vascularization from these two brain areas, just to name a few factors. The limiting factor preventing us from collecting more trials in our imaging sessions was that we observed signs of discomfort or slight distress in some mice after ~30 min of imaging in our custom setup, which we established as a humane end point to prevent distress. In consequence imaging sessions were kept to 25 min in duration, limiting the number of trials collected. However we cannot rule out that with more extensive habituation prior to experiments the imaging sessions could be prolonged without these signs of discomfort or if indeed influence from our custom setup like potential heating of the brain by illumination light might be the causing factor of the observed distress. Nevertheless, we note that previous work has shown that ~200mW average power is a safe regime for imaging in the cortex by keeping brain heating minimal (Prevedel et al., 2016), without producing the lasting damages observed by immunohistochemisty against apoptosis markers above 250mW (Podgorski and Ranganathan 2016, https://doi.org/10.1152/jn.00275.2016).

      Calcium imaging is also inherently slow, requiring relatively long inter-stimulus intervals (in this case 5 s). This unfortunately renders any model designed to predict a stimulus (in this case sound azimuth) from particularly noisy population neuronal data like these as highly prone to overfitting, to which the authors correctly admit after a model trained on the entire raw dataset failed to perform significantly above chance level. This prompted them to feed the model only with data from neurons with the highest spatial sensitivity. This ultimately produced reasonable performance (and was implemented throughout the rest of the study), but it remains possible that if the model was fed with more repetitions of imaging data, its performance would have been more stable across the number of units used to train it. (All models trained with imaging data eventually failed to converge.) However, I also see these limitations as an opportunity to improve the technology further, which I reiterate will be generally important for volume imaging of other sparse or noisy calcium signals in the brain.

      Transitioning to the naïve Bayesian classifier itself, I first openly ask the authors to justify their choice of this specific model. There are countless types of classifiers for these data, each with their own pros and cons. Did they actually try other models (such as support vector machines), which ultimately failed? If so, these negative results (even if mentioned en passant) would be extremely valuable to the community, in my view. I ask this specifically because different methods assume correspondingly different statistical properties of the input data, and to my knowledge naïve Bayesian classifiers assume that predictors (neuronal responses) are assumed to be independent within a class (azimuth). As the authors show that noise correlations are informative in predicting azimuth, I wonder why they chose a model that doesn't take advantage of these statistical regularities. It could be because of technical considerations (they mention computing efficiency), but I am left generally uncertain about the specific logic that was used to guide the authors through their analytical journey.

      One of the main reasons we chose the naïve Bayesian classifier is indeed because it assumes that the responses of the simultaneously recorded neurons are independent and therefore it does not assume a contribution of noise correlations to the estimation of the posterior probability of each azimuth. This model would represent the null hypothesis that noise correlations do not contribute to the encoding of stimulus azimuth, which would be verified by an equal decoding outcome from correlated or decorrelated datasets. Since we observed that this is not the case, the model supports the alternative hypothesis that noise correlations do indeed influence stimulus azimuth encoding. We wanted to test these hypotheses with the most conservative approach possible that would be least likely to find a contribution of noise correlations. Other relevant reasons that justify our choice of the naive Bayesian classifier are its robustness against the limited numbers of trials we could collect in comparison to other more “data hungry” classifiers like SVM, KNN, or artificial neuronal nets. We did perform preliminary tests with alternative classifiers but the obtained decoding errors were similar when decoding the whole population activity (Author response image 2A). Dimensionality reduction following the approach described in the manuscript showed a tendency towards smaller decoding errors observed with an alternative classifier like KNN, but these errors were still larger than the ones observed with the naive Bayesian classifier (median error 45º). Nevertheless, we also observe a similar tendency for slightly larger decoding errors in the absence of noise correlations (decorrelated, Author response image 2B). Sentences detailing the logic of classifier choice are now included in the results section at page 10 and at the last paragraph of page 18 (see responses to Reviewer 1).

      Author response image 2.

      A) Cumulative distribution plots of the absolute cross-validated single-trial prediction errors obtained using different classifiers (blue; KNN: K-nearest neighbors; SVM: support vector machine ensemble) and chance level distribution (gray) on the complete populations of imaged units. Cumulative distribution plots of the absolute cross-validated singletrial prediction errors obtained using a Bayes classifier (naive approximation for computation efficiency) to decode the single-trial response patterns from the 31 top ranked units in the simultaneously imaged datasets across mice (cyan), modeled decorrelated datasets (orange) and the chance level distribution associated with our stimulation paradigm (gray). Vertical dashed lines show the medians of cumulative distributions. K.S. w/Sidak: Kolmogorov-Smirnov with Sidak.

      That aside, there remain other peculiarities in model performance that warrant further investigation. For example, what spurious features (or lack of informative features) in these additional units prevented the models of imaging data from converging?

      Considering the amount of variability observed throughout the neuronal responses both in imaging and neuropixels datasets, it is easy to suspect that the information about stimulus azimuth carried in different amounts by individual DCIC neurons can be mixed up with information about other factors (Stringer et al., 2019). In an attempt to study the origin of these features that could confound stimulus azimuth decoding we explored their relation to face movement (Supplemental Figure 2), finding a correlation to snout movements, in line with previous work by Stringer et al. (2019).

      In an orthogonal question, did the most spatially sensitive units share any detectable tuning features? A different model trained with electrophysiology data in contrast did not collapse in the range of top-ranked units plotted. Did this model collapse at some point after adding enough units, and how well did that correlate with the model for the imaging data?

      Our electrophysiology datasets were much smaller in size (number of simultaneously recorded neurons) compared to our volumetric calcium imaging datasets, resulting in a much smaller total number of top ranked units detected per dataset. This precluded the determination of a collapse of decoder performance due to overfitting beyond the range plotted in Fig 4G.

      How well did the form (and diversity) of the spatial tuning functions as recorded with electrophysiology resemble their calcium imaging counterparts? These fundamental questions could be addressed with more basic, but transparent analyses of the data (e.g., the diversity of spatial tuning functions of their recorded units across the population). Even if the model extracts features that are not obvious to the human eye in traditional visualisations, I would still find this interesting.

      The diversity of the azimuth tuning curves recorded with calcium imaging (Fig. 3B) was qualitatively larger than the ones recorded with electrophysiology (Fig. 4B), potentially due to the larger sampling obtained with volumetric imaging. We did not perform a detailed comparison of the form and a more quantitative comparison of the diversity of these functions because the signals compared are quite different, as calcium indicator signal is subject to non linearities due to Ca2+ binding cooperativity and low pass filtering due to binding kinetics. We feared this could lead to misleading interpretations about the similarities or differences between the azimuth tuning functions in imaged and electrophysiology datasets. Our model uses statistical response dependency to stimulus azimuth, which does not rely on features from a descriptive statistic like mean response tuning. In this context, visualizing the trial-to-trial responses as a function of azimuth shows “features that are not obvious to the human eye in traditional visualizations” (Fig. 3D, left inset).

      Finally, the readership is encouraged to interpret certain statements by the authors in the current version conservatively. How the brain ultimately extracts spatial neuronal data for perception is anyone's guess, but it is important to remember that this study only shows that a naïve Bayesian classifier could decode this information, and it remains entirely unclear whether the brain does this as well. For example, the model is able to achieve a prediction error that corresponds to the psychophysical threshold in mice performing a discrimination task (~30 {degree sign}). Although this is an interesting coincidental observation, it does not mean that the two metrics are necessarily related. The authors correctly do not explicitly claim this, but the manner in which the prose flows may lead a non-expert into drawing that conclusion.

      To avoid misleading the non-expert readers, we have clarified in the manuscript that the observed correspondence between decoding error and psychophysical threshold is explicitly coincidental.

      Page 13, end of middle paragraph:

      “If we consider the median of the prediction error distribution as an overall measure of decoding performance, the single-trial response patterns from subsamples of at least the 7 top ranked units produced median decoding errors that coincidentally matched the reported azimuth discrimination ability of mice (Fig 4G, minimum audible angle = 31º) (Lauer et al., 2011).”

      Page 14, bottom paragraph:

      “Decoding analysis (Fig. 4F) of the population response patterns from azimuth dependent top ranked units simultaneously recorded with neuropixels probes showed that the 4 top ranked units are the smallest subsample necessary to produce a significant decoding performance that coincidentally matches the discrimination ability of mice (31° (Lauer et al., 2011)) (Fig. 5F, G).”

      We also added to the Discussion sentences clarifying that a relationship between these two variables remains to be determined and it also remains to be determined if the DCIC indeed performs a bayesian decoding computation for sound localization.

      Page 20, bottom:

      “… Concretely, we show that sound location coding does indeed occur at DCIC on the single trial basis, and that this follows a comparable mechanism to the characterized population code at CNIC (Day and Delgutte, 2013). However, it remains to be determined if indeed the DCIC network is physiologically capable of Bayesian decoding computations. Interestingly, the small number of DCIC top ranked units necessary to effectively decode stimulus azimuth suggests that sound azimuth information is redundantly distributed across DCIC top ranked units, which points out that mechanisms beyond coding efficiency could be relevant for this population code.

      While the decoding error observed from our DCIC datasets obtained in passively listening, untrained mice coincidentally matches the discrimination ability of highly trained, motivated mice (Lauer et al., 2011), a relationship between decoding error and psychophysical performance remains to be determined. Interestingly, a primary sensory representations should theoretically be even more precise than the behavioral performance as reported in the visual system (Stringer et al., 2021).”

      Moreover, the concept of redundancy (of spatial information carried by units throughout the DCIC) is difficult for me to disentangle. One interpretation of this formulation could be that there are non-overlapping populations of neurons distributed across the DCIC that each could predict azimuth independently of each other, which is unlikely what the authors meant. If the authors meant generally that multiple neurons in the DCIC carry sufficient spatial information, then a single neuron would have been able to predict sound source azimuth, which was not the case. I have the feeling that they actually mean "complimentary", but I leave it to the authors to clarify my confusion, should they wish.

      We observed that the response patterns from relatively small fractions of the azimuth sensitive DCIC units (4-7 top ranked units) are sufficient to generate an effective code for sound azimuth, while 32-40% of all simultaneously recorded DCIC units are azimuth sensitive. In light of this observation, we interpreted that the azimuth information carried by the population should be redundantly distributed across the complete subpopulation of azimuth sensitive DCIC units.

      In summary, the present study represents a significant body of work that contributes substantially to the field of spatial auditory coding and systems neuroscience. However, limitations of the imaging dataset and model as applied in the study muddles concrete conclusions about how the DCIC precisely encodes sound source azimuth and even more so to sound localisation in a behaving animal. Nevertheless, it presents a novel and unique dataset, which, regardless of secondary interpretation, corroborates the general notion that auditory space is encoded in an extraordinarily complex manner in the mammalian brain.

      Reviewer #3 (Public Review):

      Summary:

      Boffi and colleagues sought to quantify the single-trial, azimuthal information in the dorsal cortex of the inferior colliculus (DCIC), a relatively understudied subnucleus of the auditory midbrain. They used two complementary recording methods while mice passively listened to sounds at different locations: a large volume but slow sampling calcium-imaging method, and a smaller volume but temporally precise electrophysiology method. They found that neurons in the DCIC were variable in their activity, unreliably responding to sound presentation and responding during inter-sound intervals. Boffi and colleagues used a naïve Bayesian decoder to determine if the DCIC population encoded sound location on a single trial. The decoder failed to classify sound location better than chance when using the raw single-trial population response but performed significantly better than chance when using intermediate principal components of the population response. In line with this, when the most azimuth dependent neurons were used to decode azimuthal position, the decoder performed equivalently to the azimuthal localization abilities of mice. The top azimuthal units were not clustered in the DCIC, possessed a contralateral bias in response, and were correlated in their variability (e.g., positive noise correlations). Interestingly, when these noise correlations were perturbed by inter-trial shuffling decoding performance decreased. Although Boffi and colleagues display that azimuthal information can be extracted from DCIC responses, it remains unclear to what degree this information is used and what role noise correlations play in azimuthal encoding.

      Strengths:

      The authors should be commended for collection of this dataset. When done in isolation (which is typical), calcium imaging and linear array recordings have intrinsic weaknesses. However, those weaknesses are alleviated when done in conjunction with one another - especially when the data largely recapitulates the findings of the other recording methodology. In addition to the video of the head during the calcium imaging, this data set is extremely rich and will be of use to those interested in the information available in the DCIC, an understudied but likely important subnucleus in the auditory midbrain.

      The DCIC neural responses are complex; the units unreliably respond to sound onset, and at the very least respond to some unknown input or internal state (e.g., large inter-sound interval responses). The authors do a decent job in wrangling these complex responses: using interpretable decoders to extract information available from population responses.

      Weaknesses:

      The authors observe that neurons with the most azimuthal sensitivity within the DCIC are positively correlated, but they use a Naïve Bayesian decoder which assume independence between units. Although this is a bit strange given their observation that some of the recorded units are correlated, it is unlikely to be a critical flaw. At one point the authors reduce the dimensionality of their data through PCA and use the loadings onto these components in their decoder. PCA incorporates the correlational structure when finding the principal components and constrains these components to be orthogonal and uncorrelated. This should alleviate some of the concern regarding the use of the naïve Bayesian decoder because the projections onto the different components are independent. Nevertheless, the decoding results are a bit strange, likely because there is not much linearly decodable azimuth information in the DCIC responses. Raw population responses failed to provide sufficient information concerning azimuth for the decoder to perform better than chance. Additionally, it only performed better than chance when certain principal components or top ranked units contributed to the decoder but not as more components or units were added. So, although there does appear to be some azimuthal information in the recoded DCIC populations - it is somewhat difficult to extract and likely not an 'effective' encoding of sound localization as their title suggests.

      As described in the responses to reviewers 1 and 2, we chose the naïve Bayes classifier as a decoder to determine the influence of noise correlations through the most conservative approach possible, as this classifier would be least likely to find a contribution of correlated noise. Also, we chose this decoder due to its robustness against limited numbers of trials collected, in comparison to “data hungry” non linear classifiers like KNN or artificial neuronal nets. Lastly, we observed that small populations of noisy, unreliable (do not respond in every trial) DCIC neurons can encode stimulus azimuth in passively listening mice matching the discrimination error of trained mice. Therefore, while this encoding is definitely not efficient, it can still be considered effective.

      Although this is quite a worthwhile dataset, the authors present relatively little about the characteristics of the units they've recorded. This may be due to the high variance in responses seen in their population. Nevertheless, the authors note that units do not respond on every trial but do not report what percent of trials that fail to evoke a response. Is it that neurons are noisy because they do not respond on every trial or is it also that when they do respond they have variable response distributions? It would be nice to gain some insight into the heterogeneity of the responses.

      The limited number of azimuth trial repetitions that we could collect precluded us from making any quantification of the unreliability (failures to respond) and variability in the response distributions from the units we recorded, as we feared they could be misleading. In qualitative terms, “due to the high variance in responses seen” in the recordings and the limited trial sampling, it is hard to make any generalization. In consequence we referred to the observed response variance altogether as neuronal noise. Considering these points, our datasets are publicly available for exploration of the response characteristics.

      Additionally, is there any clustering at all in response profiles or is each neuron they recorded in the DCIC unique?

      We attempted to qualitatively visualize response clustering using dimensionality reduction, observing different degrees of clustering or lack thereof across the azimuth classes in the datasets collected from different mice. It is likely that the limited number of azimuth trials we could collect and the high response variance contribute to an inconsistent response clustering across datasets.

      They also only report the noise correlations for their top ranked units, but it is possible that the noise correlations in the rest of the population are different.

      For this study, since our aim was to interrogate the influence of noise correlations on stimulus azimuth encoding by DCIC populations, we focused on the noise correlations from the top ranked unit subpopulation, which likely carry the bulk of the sound location information.  Noise correlations can be defined as correlation in the trial to trial response variation of neurons. In this respect, it is hard to ascertain if the rest of the population, that is not in the top rank unit percentage, are really responding and showing response variation to evaluate this correlation, or are simply not responding at all and show unrelated activity altogether. This makes observations about noise correlations from “the rest of the population” potentially hard to interpret.

      It would also be worth digging into the noise correlations more - are units positively correlated because they respond together (e.g., if unit x responds on trial 1 so does unit y) or are they also modulated around their mean rates on similar trials (e.g., unit x and y respond and both are responding more than their mean response rate). A large portion of trial with no response can occlude noise correlations. More transparency around the response properties of these populations would be welcome.

      Due to the limited number of azimuth trial repetitions collected, to evaluate noise correlations we used the non parametric Kendall tau correlation coefficient which is a measure of pairwise rank correlation or ordinal association in the responses to each azimuth. Positive rank correlation would represent neurons more likely responding together. Evaluating response modulation “around their mean rates on similar trials” would require assumptions about the response distributions, which we avoided due to the potential biases associated with limited sample sizes.

      It is largely unclear what the DCIC is encoding. Although the authors are interested in azimuth, sound location seems to be only a small part of DCIC responses. The authors report responses during inter-sound interval and unreliable sound-evoked responses. Although they have video of the head during recording, we only see a correlation to snout and ear movements (which are peculiar since in the example shown it seems the head movements predict the sound presentation). Additional correlates could be eye movements or pupil size. Eye movement are of particular interest due to their known interaction with IC responses - especially if the DCIC encodes sound location in relation to eye position instead of head position (though much of eye-position-IC work was done in primates and not rodent). Alternatively, much of the population may only encode sound location if an animal is engaged in a localization task. Ideally, the authors could perform more substantive analyses to determine if this population is truly noisy or if the DCIC is integrating un-analyzed signals.

      We unsuccessfully attempted eye tracking and pupillometry in our videos. We suspect that the reason behind this is a generally overly dilated pupil due to the low visible light illumination conditions we used which were necessary to protect the PMT of our custom scope.

      It is likely that DCIC population activity is integrating un-analyzed signals, like the signal associated with spontaneous behaviors including face movements (Stringer et al., 2019), which we observed at the level of spontaneous snout movements. However investigating if and how these signals are integrated to stimulus azimuth coding requires extensive behavioral testing and experimentation which is out of the scope of this study. For the purpose of our study, we referred to trial-to-trial response variation as neuronal noise. We note that this definition of neuronal noise can, and likely does, include an influence from un-analyzed signals like the ones from spontaneous behaviors.

      Although this critique is ubiquitous among decoding papers in the absence of behavioral or causal perturbations, it is unclear what - if any - role the decoded information may play in neuronal computations. The interpretation of the decoder means that there is some extractable information concerning sound azimuth - but not if it is functional. This information may just be epiphenomenal, leaking in from inputs, and not used in computation or relayed to downstream structures. This should be kept in mind when the authors suggest their findings implicate the DCIC functionally in sound localization.

      Our study builds upon previous reports by other independent groups relying on “causal and behavioral perturbations” and implicating DCIC in sound location learning induced experience dependent plasticity (Bajo et al., 2019, 2010; Bajo and King, 2012), which altogether argues in favor of DCIC functionality in sound localization.

      Nevertheless, we clarified in the discussion of the revised manuscript that a relationship between the observed decoding error and the psychophysical performance, or the ability of the DCIC network to perform Bayesian decoding computations, both remain to be determined (please see responses to Reviewer #2).

      It is unclear why positive noise correlations amongst similarly tuned neurons would improve decoding. A toy model exploring how positive noise correlations in conjunction with unreliable units that inconsistently respond may anchor these findings in an interpretable way. It seems plausible that inconsistent responses would benefit from strong noise correlations, simply by units responding together. This would predict that shuffling would impair performance because you would then be sampling from trials in which some units respond, and trials in which some units do not respond - and may predict a bimodal performance distribution in which some trials decode well (when the units respond) and poor performance (when the units do not respond).

      In samples with more that 2 dimensions, the relationship between signal and noise correlations is more complex than in two dimensional samples (Montijn et al., 2016) which makes constructing interpretable and simple toy models of this challenging. Montijn et al. (2016) provide a detailed characterization and model describing how the accuracy of a multidimensional population code can improve when including “positive noise correlations amongst similarly tuned neurons”. Unfortunately we could not successfully test their model based on Mahalanobis distances as we could not verify that the recorded DCIC population responses followed a multivariate gaussian distribution, due to the limited azimuth trial repetitions we could sample.

      Significance:

      Boffi and colleagues set out to parse the azimuthal information available in the DCIC on a single trial. They largely accomplish this goal and are able to extract this information when allowing the units that contain more information about sound location to contribute to their decoding (e.g., through PCA or decoding on top unit activity specifically). The dataset will be of value to those interested in the DCIC and also to anyone interested in the role of noise correlations in population coding. Although this work is first step into parsing the information available in the DCIC, it remains difficult to interpret if/how this azimuthal information is used in localization behaviors of engaged mice.

      Recommendations for the authors:

      Reviewer #2 (Recommendations For The Authors):

      General:

      The manuscript is generally well written, but could benefit from a quick proof by a native English speaker (e.g., "the" inferior colliculus is conventionally used with its article). The flow of arguments is also generally easy to follow, but I would kindly ask the authors to consider elaborating or clarifying the following points (including those already mentioned in my public review).

      (1) Choice of model:

      There are countless ways one can construct a decoder or classifier that can predict a presented sensory stimulus based on a population neuronal response. Given the assumptions of independence as mentioned in my public review, I would ask the authors to explicitly justify their choice of a naïve Bayesian classifier.

      A section detailing the logic of classifier choice is now included in the results section at page 10 and the last paragraph of page 18 from the revised version of the manuscript.

      (2) Number of imaging repetitions:

      For particularly noisy datasets, 14 repetitions is indeed quite few. I reckon this was not the choice of the authors, but rather limited by the inherent experimental conditions. Despite minimisation of required average laser power during the development of s-TeFo imaging, the authors still required almost 200 mW (which is still quite a lot of exposure). Although 14 repetitions for 13 azimuthal locations every 5 s is at face value a relatively short imaging session (~15 min.), at 191 mW, with the desire to image mice multiple times, I could imagine that this is a practical limitation the authors faced (to avoid excessive tissue heating or photodamage, which was assessed in the original Nature Methods article, but not here). Nevertheless, this logic (or whatever logic they had) should be explained for non-imaging experts in the readership.

      This is now addressed in the answers to the public reviews.

      (3) Redundancy:

      It is honestly unclear to me what the authors mean by this. I don't speculate that they mean there are "redundant" (small) populations of neurons that sufficiently encode azimuth, but I'm actually not certain. If that were the case, I believe this would need further clarification, since redundant representations would be both inconsistent with the general (perhaps surprising) finding that large populations are not required in the DCIC, which is thought to be the case at earlier processing stages.

      In the text we are referring to the azimuth information being redundantly distributed across DCIC top ranked units. We do not mention redundant “populations of neurons”.

      (4) Correspondence of decoding accuracy with psychometric functions in mice: While this is an interesting coincidental observation, it should not be interpreted that the neuronal detection threshold in the DCIC somehow is somehow responsible its psychometric counterpart (which is an interesting yet exceedingly complex question). Although I do not believe the authors intended to suggest this, I would personally be cautious in the way I describe this correspondence. I mention this because the authors point it out multiple times in the manuscript (whereas I would have just mentioned it once in passing).

      This is now clarified in the revised manuscript.

      (5) Noisy vs. sparse:

      I'm confident that the authors understand the differences between these terms, both in concept (stochastic vs. scattered) and in context (neuronal vs. experimental), but I personally would be cautious in the way I use them in the description of the study. Indeed, auditory neuronal signals are to my knowledge generally thought to be both sparse and noisy, which is in itself interesting, but the study also deals with substantial experimental (recording) noise, and I think it's important for the readership to understand when "noise" refers to the recordings (in particular the imaging data) and to neuronal activity. I mention this specifically because "noisy" appears in the title.

      We have clarified this issue at the bottom of page 5 by adding the following sentences to the revised manuscript:

      “In this section we used the word “noise” to refer to the sound stimuli used and recording setup background sound levels or recording noise in the acquired signals. To avoid confusion, from now on in the manuscript the word “noise” will be used in the context of neuronal noise, which is the trial-to-trial variation in neuronal responses unrelated to stimuli, unless otherwise noted.”

      (6)  More details in the Methods:

      The Methods section is perhaps the least-well structured part of the present manuscript in my view, and I encourage the authors to carefully go through it and add the following information (in case I somehow missed it).

      a. Please also indicate the number of animals used here.

      Added.

      b. How many sessions were performed on each mouse?

      This is already specified in the methods section in page 25:

      “mice were imaged a total of 2-11 times (sessions), one to three times a week.”

      We added for clarification:

      “Datasets here analyzed and reported come from the imaging session in which we observed maximal calcium sensor signal (peak AAV expression) and maximum number of detected units.”

      c. For the imaging experiments, was it possible to image the same units from session tosession?

      This is not possible for sTeFo 2P data due to low spatial resolution which makes precisely matching neuron ROIs across sessions challenging.

      d. Could the authors please add more detail to the analyses of the videos (to track facialmovements) or provide a reference?

      Added citation.

      e. The same goes for the selection of subcellular regions of interest that were used as"units."

      Added to page 25:

      “We used the CaImAn package (Giovannucci et al., 2019) for automatic ROI segmentation through constrained non negative matrix factorization and selected ROIs (Units) showing clear Ca transients consistent with neuronal activity, and IC neuron somatic shape and size (Schofield and Beebe, 2019).”

      Specific: In order to maximise the efficiency of my comments and suggestions (as there are no line numbers), my numerated points are organised in sequential order.

      (1) Abstract: I wouldn't personally motivate the study with the central nucleus of the IC (i.e. Idon't think this is necessary). I think the authors can motivate it simply with the knowledge gaps in spatial coding throughout the auditory system, in which such large data sets such as the ones presented here are of general value.

      (2) Page 4: 15-50 kHz "white" noise is incorrect. It should be "band-passed" noise.

      Changed.

      (3) Supplemental figure 1, panel A: Since the authors could not identify cell bodiesunequivocally from their averaged volume timeseries data, it would be clearer to the readership if larger images are shown, so that they can evaluate (speculate) for themselves what subcellular structures were identified as units. Even better would be to include a planar image through a cross-section. As mentioned above, not everything determined for the cortex or hippocampus can be assumed to be true for the DCIC.

      The raw images and segmentations are publicly available for detailed inspections.

      (4) Supplemental figure 2, panel A: This panel requires further explanation, in particular thepanel on the right. I assume that to be a simple subtraction of sequential frames, but I'm thrown off by the "d(Grey)" colour bar. Also, if "grey" refers to the neutral colour, it is conventionally spelled "gray" in US-American English.

      Changed.

      (5) Supplemental figure 2, panel B: I'm personally curious why the animals exhibitedmovement just prior to a stimulus. Did they learn to anticipate the presentation of a sound after some habituation? Is that somehow a pre-emptive startle response? We observe that in our own experiments (but as we stochastically vary the inter-trial-intervals, the movement typically occurs directly after the stimulus). I don't suggest the authors dwell on this, but I find it an interesting observation.

      It is indeed interesting, but we can’t conclude much about it without comparing it to random inter-trial-intervals.

      (6) Supplemental figure 3: I personally find these data (decoding of all electrophysiologicaldata) of central relevance to the study, since it mirrors the analyses presented for its imaging data counterpart and encourage the authors to move it to the main text.

      Changed.

      (7) Page 12: Do the authors have any further analyses of spatial tuning functions? We allknow they can parametrically obscure (i.e., bi-lobed, non-monotonic, etc.), but having these parameters (even if just in a supplemental figure) would be informative for the spatial auditory community.

      We dedicated significant effort to attempt to parametrize and classify the azimuth response dependency functions from the recorded DCIC cells in an unbiased way. Nevertheless, given the observed response noise and the “obscure” properties of spatial tuning functions mentioned by the reviewer, we could only reach the general qualitative observation of having a more frequent contralateral selectivity.

      (8) Page 14 (end): Here, psychometric correspondence is referenced. Please add theLauer et al., (2011) reference, or, as I would, remove the statement entirely and save it for the discussion (where it is also mentioned and referenced).

      Changed.

      (9) Figure 5, Panels B and C: Why don't the authors report the Kruskal-Wallis tests (forincreasing number of units training the model), akin to e.g., Panel G of Figure 4? I think that would be interesting to see (e.g., if the number of required units to achieve statistical significance is the same).

      Within class randomization produced a moderate effect on decoder performance, achieving statistical significance at similar numbers of units, as seen in figure 5 panels B and C. We did not include these plots for the sake of not cluttering the figure with dense distributions and fuzzing the visualization of the differences between the distributions shown.

      (10) Figure 5, Panels B and C (histograms): I see a bit of skewedness in the distributions(even after randomisation). Where does this come from? This is just a small talking point.

      We believe this is potentially due to more than one distribution of pairwise correlations combined into one histogram (like in a Gaussian mixture model).

      (11) Page 21: Could the authors please specify that the Day and Delgutte (2013) study wasperformed on rabbits? Since rabbits have an entirely different spectral hearing range compared to mice, spatial coding principles could very well be different in those animals (and I'm fairly certain such a study has not yet been published for mice).

      Specified.

      (12) Page 22: I'd encourage the authors to remove the reference to Rayleigh's duplextheory, since mice hardly (if at all) use interaural time differences for azimuthal sound localisation, given their generally high-frequency hearing range.

      That sentence is meant to discuss beyond the mouse model an exciting outlook of our findings in light of previous reports, which is a hypothetical functional relationship between the tonotopy in DCIC and the spatial distribution of azimuth sensitive DCIC neurons. We have clarified this now in the text.

      (13) Page 23: I believe the conventional verb for gene delivery with viruses is still"transduce" (or "infect", but not "induce"). What was the specific "syringe" used for stereotactic injections? Also, why were mice housed separately after surgery? This question pertains to animal welfare.

      Changed. The syringe was a 10ml syringe to generate positive or negative pressure, coupled to the glass needle through a silicon tubing via a luer 3-way T valve. Single housing was chosen to avoid mice compromising each other’s implantations. Therefore this can be seen as a refinement of our method to maximize the chances of successful imaging per implanted mouse.

      (14) Page 25: Could the authors please indicate the refractory period violation time windowhere? I had to find it buried in the figure caption of Supplementary figure 1.

      Added.

      (15) Page 27: What version of MATLAB was used? This could be important for reproductionof the analyses, since The Mathworks is infamously known to add (or even more deplorably, modify) functions in particular versions (and not update older ones accordingly).

      Added.

      Reviewer #3 (Recommendations For The Authors):

      Overall I thought this was a nice manuscript and a very interesting dataset. Here are some suggestions and minor corrections:

      You may find this work of interest - 'A monotonic code for sound azimuth in primate inferior colliculus' 2003, Groh, Kelly & Underhill.

      We thank the reviewer for pointing out this extremely relevant reference, which we regrettably failed to cite. It is now included in the revised version of the manuscript.

      In your introduction, you state "our findings point to a functional role of DCIC in sound location coding". Though your results show that there is azimuthal information contained in a subset of DCIC units there's no evidence in the manuscript that shows a functional link between this representation and sound localization.

      This is now addressed in the answers to the public reviews.

      I found the variability in your DCIC population quite striking - especially during the intersound intervals. The entrainment of the population in the imaging datatset suggests some type of input activating the populations - maybe these are avenues for further probing the variability here:

      (1) I'm curious if you can extract eye movements from your video. Work from Jennifer Grohshows that some cells in the primate inferior colliculus are sensitive to different eye positions (Groh et. al., 2001). With recent work showing eye movements in rodents, it may explain some of the variance in the DCIC responses.

      This is now addressed in the answers to the public reviews.

      (2) I was also curious if the motor that moves the speaker made noise It could be possiblesome of the 'on going' activity could be some sound-evoked response.

      We were careful to set the stepper motor speed so that it produced low frequency noise, within a band mostly outside of the hearing range of mice (<4kHz). Nevertheless, we cannot fully rule out that a very quiet but perhaps very salient component of the motor noise could influence the activity during the inter trial periods. The motor was stationary and quiet for a period of at least one stimulus duration before and during stimulus presentation.  

      (3) Was the sound you present frozen or randomly generated on each trial? Could therebe some type of structure in the noise you presented that sometimes led cells to respond to a particular azimuth location but not others?

      The sound presented was frozen noise. This is now clarified in the methods section.

      It may be useful to quantify the number of your units that had refractory period violations.

      Our manual curation of sorted units was very stringent to avoid mixing differently tuned neurons. The single units analyzed had very infrequent refractory period violations, in less than ~5% of the spikes, considering a 2 ms refractory period.

      Was the video recording contralateral or ipsilateral to the recording?

      The side of the face ipsilateral to the imaged IC was recorded. Added to methods.

      I was struck by the snout and ear movements - in the example shown in Supplementary Figure 2B it appears as they are almost predicting sound onset. Was there any difference in ear movements in the habituated and non-habituated animals? Also, does the placement of the cranial window disturb any of the muscles used in ear movement?

      Mouse snout movements appear to be quite active perhaps reflecting arousal (Stringer et al., 2019). We cannot rule out that the cranial window implantation disturbed ear movement but while moving the mouse headfixed we observed what could be considered normal ear movements.

      Did you correlate time-point by time-point in the average population activity and movement or did you try different temporal labs/leads in case the effect of the movements was delayed in some way?

      Point by point due to 250ms time resolution of imaging.

      Are the video recordings only available during the imaging? It would be nice to see the same type of correlations in the neuropixel-acquired data as well.

      Only imaging. For neuropixels recordings, we were skeptical about face videography as we suspected that face movements were likely influenced by the acute nature of the preparation procedure. Our cranial window preparation in the other hand involved a recovery period of at least 4 weeks. Therefore we were inclined to perform videographical interrogation of face movements on these mice instead.

      If you left out more than 1 trial do you think this would help your overfitting issue (e.g. leaving out 20% of the data).

      Due to the relatively small number of trial repetitions collected, fitting the model with an even smaller training dataset is unlikely to help overfitting and will likely decrease decoder performance.

      It would be nice to see a confusion matrix - even though azimuthal error and cumulative distribution of error are a fine way to present the data - a confusion matrix would tell us which actual sounds the decoder is confusing. Just looking at errors could result in some funky things where you reduce the error generally but never actually estimate the correct location.

      We considered confusion matrices early on in our study but they were not easily interpretable or insightful, likely due to the relatively low discrimination ability of the mouse model with +/- 30º error after extensive training. Therefore, we reasoned that in passively listening mice (and likely trained mice too) with limited trial repetitions, an undersampled and diffuse confusion matrix is expected which is not an ideal means of visualizing and comparing decoding errors. Hence we relied on cumulative error distributions.

      Do your top-ranked units have stronger projections onto your 10-40 principal components?

      It would be interesting to know if the components are mostly taking into account those 30ish percent of the population that is dependent upon azimuth.

      Inspection of PC loadings across units ranked based on response dependency to stimulus azimuth does not show a consistent stronger projection of top ranked units onto the first 10-40 principal components (Author response image 3).

      Author response image 3.

      PC loading matrices for each recorded mouse. The units recorded in each mouse are ranked in descending order of response dependency to stimulus azimuth based on  the p value of the chi square test. Units above the red dotted line display a chi square p value < 0.05, units below this line have p values >= 0.05.

      How much overlap is there in the tuning of the top-ranked units?

      This is quite varying from mouse to mouse and imaging vs electrophysiology, which makes it hard to make a generalization since this might depend on the unique DCIC population sampled in each mouse.

      I'm not really sure I follow what the nS/N adds - it doesn't really measure tuning but it seems to be introduced to discuss/extract some measure of tuning.

      nS/N is used to quantify how noisy neurons are, independent of how sensitive their responses are to the stimulus azimuth.

      Is the noise correlation - observed to become more positive - for more contralateral stimuli a product of higher firing rates due to a more preferred stimulus presentation or a real effect in the data? Was there any relationship between distance and strength of observed noise correlation in the DCIC?

      We observed a consistent and homogeneous trend of pairwise noise correlation distributions either shifted or tailed towards more positive values across stimulus azimuths, for imaging and electrophysiology datasets (Author response image 3). The lower firing frequency observed in neuropixels recordings in response to ipsilateral azimuths could have affected the statistical power of the comparison between the pairwise noise correlation coefficient distribution to its randomized chance level, but the overall histogram shapes qualitatively support this consistent trend across azimuths (Author response image 4).

      Author response image 4.

      Distribution histograms for the pairwise correlation coefficients (Kendall tau) from pairs of simultaneously recorded top ranked units across mice (blue) compared to the chance level distribution obtained through randomization of the temporal structure of each unit’s activity to break correlations (purple). Vertical lines show the medians of these distributions. Imaging data comes from n = 12 mice and neuropixels data comes from n = 4 mice.

      Typos:

      'a population code consisting on the simultaneous" > should on be of?

      'half of the trails' > trails should be trials?

      'referncing the demuxed channels' > should it be demixed?

      Corrected.

    1. Author Response

      The following is the authors’ response to the original reviews.

      eLife assessment

      This study presents a valuable finding on the immunophenotypes of cancer treatment-related pneumonitis. The evidence supporting the claims of the authors is solid, although the inclusion of controls, as suggested by one of the reviewers, strengthened the study. The work will be of interest to cancer immunologists.

      Response: We are thankful for the editor's recognition of the contribution our study makes to understanding the immunophenotypes associated with cancer treatment-related pneumonitis. We agree that the inclusion of control data is pivotal for benchmarking biomarkers. While our initial study design was constrained by the availability of BALF from healthy individuals within clinical settings, we addressed this limitation by incorporating scRNA-seq data from healthy control and COVID-19 BALF cells sourced from the GSE145926 dataset. This additional analysis has provided a baseline for comparison, revealing that CD16 is expressed in a minority of T cells in healthy BALF, specifically 1.0% of CD4+ T cells and 1.6% of CD8+ T cells. The inclusion of this data as Figures 6H and 6I in our manuscript offers a robust context for the significant increase in CD16-expressing T cells observed in patients with PCP, thus enhancing the robustness of our study's conclusions.

      Author response image 1.

      Reviewer #1 (Recommendations For The Authors):

      Many thanks for giving me the opportunity to review your paper. I really enjoyed the way you carried out this work - for example, your use of a wide panel of markers and the use of two analytical methods - you have clearly given great thought to bias avoidance. I also greatly appreciated your paragraph on the limitations, as there are several, but you do not 'over-sell' your conclusions so there is no issue here for me.

      To improve the piece, there are a few typos (eg 318 - specific to alpha-myosin) and I was briefly confused about the highlighted clusters in Figure 4. Perhaps mention why they are highlighted when they first appear in 4D instead of E?

      Response: We have corrected the typos, and we have rearranged the sequence of Figures 3E and 3F, as well as 4D and 4E, to ensure a logical flow. Citrus-generated violin plots are now presented prior to the heatmap of the clusters, which better illustrates the progression of our analysis and the derivation of the clusters.

      In terms of improvements to the data, obviously it would have been ideal if you had had some sort of healthy control as a point of reference for all cohorts, but working in the field I understand the difficulties in getting healthy BAL. It would be worth your while however trying to find more supportive data in the literature in general. There are studies which assess various immune markers in healthy BAL eg https://journal-inflammation.biomedcentral.com/articles/10.1186/1476-9255-11-9. and so I think it is worth looking wrt the main findings. For example, are CD16+ T cells seen in healthy BAL or any other conditions (at present the COVID study is being over-relied on)? Could these cells be gamma deltas? (gamma deltas frequently express CD8 and CD16, and can switch to APC like phenotypes).

      Response: We are grateful for the reviewer's consideration of the practical challenges associated with collecting BALF from healthy individuals. Alternatively, we have supplemented our analysis with single-cell RNA sequencing data from BALF cells of healthy controls, as found in existing literature (Nature Medicine 2020; 26: 842-844). We have accessed to GSE145926 and downloaded data of BALF cells from healthy control (n=3) and severe COVID19 (n=6). The filtered gene-barcode matrix was first normalized using ‘NormalizeData’ methods in Seurat v.4 with default parameters. The top 2,000 variable genes were then identified using the ‘vst’ method in Seurat FindVariableFeatures function. Then PCA and UMAP was performed. T cells were identified as CD2 >1 and CD3E >1, and FCGR3A expression was explored using an expression threshold of 0.5. Violin plots and bar plots were generated by ggplot function.

      Regarding the pivotal finding of increased CD16-expressing T cells in patients with PCP, the scRNA-seq data mining indicates that CD16 is expressed by a minority of T cells in healthy BALF—1.0% of CD4+ T cells and 1.6% of CD8+ T cells. These figures, now incorporated into our revised manuscript as Figures 6H and 6I, substantiate our findings. These cells could be gamma delta T cells, but we could not confirm it with the limited data. We will investigate in the future study. The main text has been updated to reflect these findings.

      Author response image 2.

      I would agree with your approach of not going down the transcript route, so just focus on protein expression.

      I think you need to mention more about the impact of ICI on PD1 expression - in the methods you lose one approach owing to low T cell expression (132) but in the discussion you mention ICI induced high expression (311) as previously reported. This apparent contradiction needs an explanation.

      Response: We acknowledge the need for clarification regarding the impact of ICIs on PD-1 expression. In the methods section, the low detection of PD-1 expression on T cells in patients treated with nivolumab was indeed noted; this was due to the competitive nature of the PD-1 detection antibody EH12.2 with nivolumab. As reported by Suzuki et al. (International Immunology 2020; 32: 547-557), T cells from patients with ICI-induced ILD, including those treated with nivolumab, exhibit upregulated PD-1 expression, where the PD-1 detection antibody (clone: MIH4). Conversely, as outlined by Yanagihara et al. (BBRC 2020; 527: 213-217), the PD-1 detection antibody clone EH12.2 conjugated with 155Gd (#3155009B) used in our study is unable to detect PD-1 when patients are under nivolumab treatment due to competitive inhibition. The absence of a metal-conjugated PD-1 antibody with the MIH4 clone presented a limitation in our study. Ideally, we would have conjugated the MIH4 antibody with 155Gd for our analysis, which is a refinement we aim to incorporate in future research. We have now included this discussion in our manuscript to clarify the contradiction between the methodological limitations and the high PD-1 expression induced by ICIs, as reported in the literature. This addition will guide readers through the nuances of antibody selection and its implications for detecting PD-1 expression in the context of ICI treatment.

      Finally, since you have the severity data, it would be good to assess all the significantly different clusters against this metric, as you have done for CD16+ T cells. Not only may this reveal more wrt the impact of other immune populations, but it'll also give a point of reference for the CD16+ T cell data.

      Response: Thank you for the suggestion to assess all significantly different clusters against the disease severity metric. We have expanded our analysis to include a thorough correlation study between the disease severity and intensity of various T-cell markers. Notably, we observed that intensity of CCR7 expression correlates with the disease severity. Although the precise biological significance of this correlation remains to be elucidated, it may suggest a role for CCR7+ T cells in the pathogenesis or progression of the disease. We have considered the potential implications of this finding and included it as Supplementary Figure 5. We have also discussed this observation in the discussion section.

      Author response image 3.

      Overall though I think this is a really nice study, with a potentially very significant finding in linking CD16+ T cells with severity. Congratulations.

      Response: We would like to thank the reviewer’s heartful comments on our manuscript.

      Reviewer #2 (Recommendations For The Authors):

      General:

      1) The fact that this is a retrospective study should be indicated earlier in the paper.

      Response: Now we have mentioned the retrospective nature of the study in the method section as follows: In this retrospective study, patients who were newly diagnosed with PCP, DI-ILD, and ICI-ILD and had undergone BALF collection at Kyushu University Hospital from January 2017 to April 2022 were included. The retrospective study was approved by the Ethics Committee of Kyushu University Hospital (reference number 22117-00).

      2) tSNE and UMAP are dimensionality reduction techniques that don't cluster the cells, the authors should specify what clustering algorithm was used subsequently (e.g FlowSOM)

      Response: The cluster was determined manually by their expression pattern.

      3) With regards to the role of CD16 in a potential exacerbated cytotoxicity in the fatal PCP case, the authors could measure the levels of C3a related proteins in patient serum to link to a common immunopathogenic pathway with COVID.

      Response: We did not collect serum from the patients in this study as our research protocol was approved by the Ethics committee for the use of BALF only. However, we agree with your assessment that the measurement of serum C3a levels would be informative. In future studies, we will incorporate the measurement of serum C3a levels to provide more comprehensive insights into the impact of C3a on immune function. Thank you for your valuable feedback and for helping us to improve the quality of our research.

      Line-specific:

      101 The authors should provide some information on how the cryopreservation of the BALF was carried out.

      Response: Upon collection, BALF samples were immediately centrifuged at 300 g for 5 minutes to pellet the cells. The resultant cell pellets were then resuspended in Cellbanker 1 cryopreservation solution (Takara, catalog #210409). This suspension was aliquoted into cryovials and gradually frozen to –80ºC using a controlled rate freezing method to ensure cell viability. The samples were stored at –80ºC until required for experimental analysis. We have added the information in the method section.

      Fig 3B: It would be very helpful if the authors could add a supplementary figure with marker expression on the UMAP projection.

      Response: We have added Supplementary Figure 4 with marker expression on the UMAP projection in Figure 3B.

      Fig 4A: Same as Fig 3B

      Response: We have added Supplementary Figure 5 with marker expression on the UMAP projection in Figure 4A.

      Fig 5B: Same as Fig 3B

      Response: We have added Supplementary Figure 6 with marker expression on the tSNE projection in Figure 5B.

      266 Authors should state if the data is not shown with regards to differences in myeloid cell fractions

      430 Marker intensity is not shown in panel D

      Re: Corrected as follows: “Citrus network tree visualizing the hierarchical relationship of each marker between identified T cell ~”

      446 The legend says patients have IPF, CTD-ILD, sarcoidosis but the figure shows PCP, DI-ILD, ICI-ILD.

      Re: Corrected.

      451 What do the authors mean in "Graphical plots represent individual samples"? Panel B is a dot plot of all samples.

      Response: Corrected as “Dot plots represent ~”.

      472 What do the authors mean in "Graphical plots represent individual samples"? Panel C is a dot plot of all samples.

      Response: Corrected as “Dot plots represent ~”.

      Reviewer #3 (Recommendations For The Authors):

      An important thing is to add comparisons against healthy donors, at least. A common baseline is needed to firmly establish any biomarkers.

      Response: We acknowledge the reviewer's concern regarding the comparison with healthy donors. Although our study did not initially include BALF collection from healthy controls due to the constraints of clinical practice, we recognize the importance of a control baseline to validate biomarkers. To address this, we have integrated scRNA-seq data from healthy control BALF cells available in public datasets (Nature Medicine 2020; 26: 842-844), accessed from GSE145926. This dataset includes BALF cells from healthy controls (n=3) alongside severe COVID-19 patients (n=6). Data mining confirmed that CD16 expression is in a minority of T cells in healthy BALF—1.0% of CD4+ T cells and 1.6% of CD8+ T cells. We have included this comparative data in our manuscript as Figures 6H and 6I to provide context for the observed increase in CD16-expressing T cells in PCP patients, which substantiates our findings.

      Author response image 4.

      Data analysis needs to go deeper. There are several other tools on Cytobank alone that would allow a more quantitative analysis of the data. Fold changes in marker expressions would be very important as measurements of phenotypic changes.

      Response: We thank the reviewer for their constructive feedback on the depth of our data analysis. We acknowledge the value of a more quantitative approach, including the use of fold change measurements to assess phenotypic alterations, and recognize the potential insights such tools on Cytobank could provide. Due to the scope and limited space of the current study, we have focused our analysis on the most pertinent findings relevant to our research questions. We believe the present analysis serves the immediate objectives of this study. However, we agree that further quantitative analysis would enhance the understanding of the data. We have expanded our analysis to include a thorough correlation study between the disease severity of PCP and intensity of various T-cell markers. Notably, we observed that intensity of CCR7 expression correlates with the disease severity of PCP. Although the precise biological significance of this correlation remains to be elucidated, it may suggest a role for CCR7+ T cells in the pathogenesis or progression of the disease. We have considered the potential implications of this finding and included it as Supplementary Figure 5. We have also discussed this observation in the discussion section. We aim to consider these approaches in future work to build upon the foundation laid by this study. Your suggestions are invaluable and will be kept at the forefront as we plan subsequent research phases.

      Author response image 5.

      Reviewer #1 (Public Review):

      Cytotoxic agents and immune checkpoint inhibitors are the most commonly used and efficacious treatments for lung cancers. However their use brings two significant pulmonary side-effects; namely Pneumocystis jirovecii infection and resultant pneumonia (PCP), and interstitial lung disease (ILD). To observe the potential immunological drivers of these adverse events, Yanagihara et al. analysed and compared cells present in the bronchoalveolar lavage of three patient groups (PCP, cytotoxic drug-induced ILD [DI-ILD], and ICI-associated ILD [ICI-ILD]) using mass cytometry (64 markers). In PCP, they observed an expansion of the CD16+ T cell population, with the highest CD16+ T proportion (97.5%) in a fatal case, whilst in ICI-ILD, they found an increase in CD57+ CD8+ T cells expressing immune checkpoints (TIGIT+ LAG3+ TIM-3+ PD-1+), FCRL5+ B cells, and CCR2+ CCR5+ CD14+ monocytes. Given the fatal case, the authors also assessed for, and found, a correlation between CD16+ T cells and disease severity in PCP, postulating that this may be owing to endothelial destruction. Although n numbers are relatively small (n=7-9 in each cohort; common numbers for CyTOF papers), the authors use a wide panel (n=65) and two clustering methodologies giving greater strength to the conclusions. The differential populations discovered using one or two of the analytical methods are robust: whole population shifts with clear and significant clustering. These data are an excellent resource for clinical disease specialists and pan-disease immunologists, with a broad and engaging contextual discussion about what they could mean.

      Strengths:

      • The differences in immune cells in BAL in these specific patient subgroups is relatively unexplored.

      • This is an observational study, with no starting hypothesis being tested.

      • Two analytical methods are used to cluster the data.

      • A relatively wide panel was used (64 markers), with particular strength in the alpha beta T cells and B cells.

      • Relevant biomarkers, beta-D-glucan and KL-6 were also analysed

      • Appropriate statistics were used throughout.

      • Numbers are low (7 cases of PCP, 9 of DI-ILD, and 9 of ICI-ILD) but these are difficult samples to collect and so in relative terms, and considering the use of CyTOF, these are good numbers.

      • Beta-D-glucan shows potential as a biomarker for PCP (as previously reported) whilst KL-6 shows potential as a biomarker for ICI-ILD (not reported before). Interestingly, KL-6 was not seen to be increased in DI-ILD patients.

      • Despite the relatively low n numbers and lack of matching there are some clear differentials. The CD4/CD8+CD16+HLA-DR+CXCR3+CD14- T cell result is striking - up in PCP (with EM CD4s significantly down) - whilst the CD8 EMRA population is clear in ICI-ILD and 'non-exhausted' CD4s, with lower numbers of EMRA CD8s in DI-ILD.

      • The authors identify 17/31 significantly differentiated clusters of myeloid cells, eg CD11bhi CD11chi CD64+ CD206+ alveolar macrophages with HLA-DRhi in PCP.

      • With respect to B cells, the authors found that FCRL5+ B cells were more abundant in patients with ICI-ILD compared to those with PCP and DI-ILD, suggesting these FCRL5+ B cells may have a role in irAE.

      • One patient's extreme CD16+ T cell (97.5% positive) and death, led the authors to consider CD16+ T cells as an indicator of disease severity in PCP. This was then tested and found to be correct.

      • Authors discuss results in context of literature leading them to suggest that CD16+ T cells may target endothelial cells and wonder if anti-complement therapy may be efficacious in PCP.

      • Great discussion on auto-reactive T cell clones where the authors suggest that in ICI-ILD CD8s may react against healthy lung, driving ILD.

      • An observation of CXCR3 in different CD8 populations in ICI-ILD and PCP lead the authors to hypothesise on the chemoattractants in the microenvironment.

      • Excellent point suggesting CD57 may not always be a marker of senescence on T cells - reflective of growing change within the community.

      • Well considered suggestion that FCRL5+ B cells may be involved in ICI-ILD driven autoimmunity.

      • The authors discuss the main weaknesses in the discussion and stress that the findings detailed in the paper "demonstrate a correlation rather than proof of causation".

      • Figures and legends are clear and pleasing to the eye.

      Weaknesses:

      • This is an observational study, with no starting hypothesis being tested.

      • Only patients who were able to have a lavage taken have been recruited.

      • One set of analysis wasn't carried out for one subgroup (ICI-ILD) as PD1 expression was negative owing to the use of nivolumab.

      • Some immune cell subsets wouldn't be picked up with the markers and gating strategies used; e.g. NK cells.

      • Some immune cells would be disproportionately damaged by the storage, thawing and preparation of the samples; e.g. granulocytes.

      • Numbers are low (7 cases of PCP, 9 of DI-ILD, and 9 of ICI-ILD), sex, age and adverse event matching wasn't performed, and treatment regimen are varied and 'suspected' (suggesting incomplete clinical data) - but these are difficult samples to collect. These numbers drop further for some analyses e.g. T cell clustering owing to factors such as low cell number.

      • The disease comparisons are with each other, there is no healthy control.

      • Samples are taken at one time point.

      • The discussion on probably the stand out result - the CD16+ T cells in PCP - relies on two papers - leading to a slightly skewed emphasis on one paper on CD16+ cells in COVID. There are other papers out there that have observed CD16+ T cells in other conditions. It is also worth being in mind that given the markers used, these CD16+ T cell may be gamma deltas.

      • The discussion on ICI patient consistently showing increased PD1, could have been greater, as given the ICI is targeting PD1, one would expect the opposite as commented on, and observed, in the methods section.

      Reviewer #2 (Public Review):

      Yanagihara and colleagues investigated the immune cell composition of bronchoalveolar lavage fluid (BALF) samples in a cohort of patients with malignancy undergoing chemotherapy and with with lung adverse reactions including Pneumocystis jirovecii pneumonia (PCP) and immune-checkpoint inhibitors (ICIs) or cytotoxic drug induced interstitial lung diseases (ILDs). Using mass cytometry, their aim was to characterize the cellular and molecular changes in BAL to improve our understanding of their pathogenesis and identify potential biomarkers and therapeutic targets. In this regard, the authors identify a correlation between CD16 expression in T cells and the severity of PCP and an increased infiltration of CD57+ CD8+ T cells expressing immune checkpoints and FCLR5+ B cells in ICI-ILD patients.

      The conclusions of this paper are mostly well supported by data, but some aspects of the data analysis need to be clarified and extended.

      1) The authors should elaborate on why different set of markers were selected for each analysis step. E.g., Different set of markers were used for UMAP, CITRUS and viSNE in the T cell and myeloid analysis.

      2) The authors should state if a normality test for the distribution of the data was performed. If not, non-parametric tests should be used.

      3) The authors should explore the correlation between CD16 intensity and the CTCAE grade in T cell subsets such as EMRA CD8 T cells, effector memory CD4, etc as identified in Figure 1B.

      4) The authors could use CITRUS to better assess the B cell compartment.

      Reviewer #3 (Public Review):

      The authors collected BALF samples from lung cancer patients newly diagnosed with PCP, DI-ILD or ICI-ILD. CyTOF was performed on these samples, using two different panels (T-cell and B-cell/myeloid cell panels). Results were collected, cleaned-up, manually gated and pre-processed prior to visualisation with manifold learning approaches t-SNE (in the form of viSNE) or UMAP, and analysed by CITRUS (hierarchical clustering followed by feature selection and regression) for population identification - all using Cytobank implementation - in an attempt to identify possible biomarkers for these disease states. By comparing cell abundances from CITRUS results and qualitative inspection of a small number of marker expressions, the authors claimed to have identified an expansion of CD16+ T-cell population in PCP cases and an increase in CD57+ CD8+ T-cells, FCRL5+ B-cells and CCR2+ CCR5+ CD14+ monocytes in ICI-ILD cases.

      By the authors' own admission, there is an absence of healthy donor samples and, perhaps as a result of retrospective experimental design, also an absence of pre-treatment samples. The entire analysis effectively compares three yet-established disease states with no common baseline - what really constitutes a "biomarker" in such cases? The introduction asserts that "y characterizing the cellular and molecular changes in BAL from patients with these complications, we aim to improve our understanding of their pathogenesis and identify potential therapeutic targets" (lines 82-84). Given these obvious omissions, no real "changes" have been studied in the paper. These are very limited comparisons among three, and only these three, states.

      Even assuming more thorough experimental design, the data analysis is unfortunately too shallow and has not managed to explore the wealth of information that could potentially be extracted from the results. CITRUS is accessible and convenient, but also make a couple of big assumptions which could affect data analysis - 1) Is it justified to concatenate all FCS files to analyse the data in one batch / small batches? Could there be batch effects or otherwise other biological events that could confuse the algorithm? 2) With a relatively small number of samples, and after internal feature selection of CITRUS, is the regression model suitable for population identification or would it be too crude and miss out rare populations? There are plenty of other established methods that could be used instead. Have those methods been considered?

      Colouring t-SNE or UMAP (e.g. Figure 6C) plots by marker expression is useful for quick identification of cell populations but it is not a quantitative analysis. In a CyTOF analysis like this, it is common to work out fold changes of marker expressions between conditions. It is inadequate to judge expression levels and infer differences simply by looking at colours.

      The relatively small number of samples also mean that most results presented in the paper are not statistical significant. Whilst it is understandable that it is not always possible to collect a large number of patient samples for studies like this, having several entire major figures showing "n.s." (e.g. Figures 3A, 4B and 5C), together with limitations in the comparisons themselves and inadequate analysis, make the observations difficult to be convincing, and even less so for the single fatal PCP case where N = 1.

      It would also be good scientific practice to show evidence of sample data quality control. Were individual FCS files examined? Did the staining work? Some indication of QC would also be great.

      This dataset generated and studied by the authors have the potential to address the question they set out to answer and thus potentially be useful for the field. However, in the current state of presentation, more evidence and more thorough data analysis are needed to draw any conclusions, or correlations, as the authors would like to frame them.

    1. Author response:

      The following is the authors’ response to the original reviews.

      eLife assessment

      This paper provides useful information about how the ionome of Arabidopsis thaliana adapts to very high CO2-levels, backed up by solid evidence and carefully designed studies. However, the broader claims of the paper about climate change and food security - heavily emphasized in the abstract, introduction, and discussion - are inappropriate, as there is no direct link to the presented work.

      We sincerely thank you for the work you have done in reviewing our manuscript. We very much appreciate your overall positive assessment of the experimental work as a whole, its value and robustness.

      In this revised version, we took on board the majority of your suggestions and your comments. In particular, we understood your critical point about overstating our objectives, which might in turn seem uncorrelated with our results. We fully agree with the comments that have been made on this point. Consequently, we have made substantial modifications and corrections in order to clarify our objectives and their implications: exploring in depth the natural variation of the shoot ionome response to elevated CO2, and generating a valuable resource allowing a better understanding of the genetic and molecular mechanisms involved in the regulation of plant mineral nutrition by the elevation of atmospheric CO2.

      We also made modifications in response to the other suggestions, including a clarification of the functional experiments carried out around the function of TIP2;2 in response to elevated CO2. Figure 7 now comprises the comparison between both ambient and elevated CO2 conditions, which is much more informative that what appeared in the previous version.

      Public Reviews:

      Reviewer #1 (Public Review):

      Summary:

      The study's abstract, introduction, and conclusions are not supported by the methods and results conducted. In fact, the results presented suggest that Arabidopsis could easily adapt to an extremely high CO2 environment.

      We understand the reviewer’s comment. Although our work is considered useful, robust and well designed, we agree with the reviewer's point. We have certainly overemphasized the significance of our work to address the issue of food security in response to rising atmospheric CO2, at the expense of the factual description of the results of our fundamental study of the mechanisms at the interface between CO2 and mineral nutrition. We have clarified this focus by modifying the text of the introduction, objectives and discussion. We hope that these modifications will enable readers to better appreciate the core of this work.

      Regarding the last part of the comment, our results do suggest that genetic variation could allow adaptation to rising atmospheric CO2, and our study does indeed aim to identify the extent and basis of this genetic variation.

      This study offers good evidence pointing to a genetic basis for Arabidopsis thaliana's response to elevated CO2 (eCO2) levels and its subsequent impact on the leaf ionome. The natural variation analyses in the study support the hypothesis that genetic factors, rather than local adaptation, guide the influence of eCO2 on the ionome of rosette leaves in Arabidopsis. However, the manuscript's claim regarding its role in "the development of biofortified crops adapted to a high-CO2 world" (line 23) is overstated, especially given the absence of any analysis on the influence of eCO2 on the seed ionome and Arabidopsis is a poor model for harvest index for any crop. The manuscript, in its current form, necessitates massive revisions, particularly in clarifying its broader implications and in providing more substantial evidence for some of its assertions.

      We thank the reviewer for this comment, and we would like to thank the reviewer for the positive appreciation for the identification of genetic basis for Arabidopsis thaliana's response to elevated CO2 and its subsequent impact on the leaf ionome. Nevertheless, it is true that the study of the leaf ionome is far from being able to lead to the development of biofortified plants. Some papers described that nutrient harvest index in Arabidopsis is a potential indicator of nutrient use efficiency (for instance, Masclaux-Daubresse and Chardon, Journal of Experimental Botany 2011 or Aranjuelo et al., Journal of Experimental Botany 2013). However, as we did not include any seed ionome data in the paper, we added clear mentions that our analyses were made on leaves (lines 56/57/250/319) and a comment in the discussion section to address this limitation (lines 325-328).

      Major Drawbacks and Questions:

      (1) Evidence for the Central Premise:

      The foundational premise of the study is the assertion that rising atmospheric CO2 levels result in a decline in plant mineral content. This phenomenon is primarily observed in C3 plants, with C4 plants seemingly less affected. The evidence provided on this topic is scant and, in some instances, contradicts the authors' own references. The potential reduction of certain minerals, especially in grains, can be debated. For instance, reduced nitrogen (N) and phosphorus (P) content in grains might not necessarily be detrimental for human and animal consumption. In fact, it could potentially mitigate issues like nitrogen emissions and phosphorus leaching. Labeling this as a "major threat to food security" (line 30) is exaggerated. While the case for microelements might be more compelling, the introduction fails to articulate this adequately. Furthermore, the introduction lacks any discussion on how eCO2 might influence nutrient allocation to grains, which would be crucial in substantiating the claim that eCO2 poses a threat to food security. A more comprehensive introduction that clearly delineates the adverse effects of eCO2 and its implications for food security would greatly enhance the manuscript.

      We partially agree with this comment. The decline in mineral status of C3 plants under conditions of elevated atmospheric CO2 has been widely described in the literature, and specifically documented for the cereal grains. While there are variations in this effect (depending on species, ecotype, cultivar), there is no debate about its acceptance. Here are just a few of the many works describing this effect, both on a global scale and at the level of the individual plant (Cotrufo MF (1998) Elevated CO2 reduces the nitrogen concentration of plant tissues. Global Change Biology 4: 43-54; Loladze I (2014) Hidden shift of the ionome of plants exposed to elevated CO(2)depletes minerals at the base of human nutrition. eLife 3: e02245; Myers SS (2014) Increasing CO2 threatens human nutrition. Nature 510: 139-142; Poorter H (1997) The effect of elevated CO2 on the chemical composition and construction costs of leaves of 27 C3 species. Plant, Cell & Environment 20: 472-482 ; Soares JC (2019) Preserving the nutritional quality of crop plants under a changing climate: importance and strategies. Plant and Soil 443: 1-26; Stitt] M (1999) The interaction between elevated carbon dioxide and nitrogen nutrition: the physiological and molecular background. Plant, Cell & Environment 22: 583-621; Uddling J (2018) Crop quality under rising atmospheric CO2. Curr Opin Plant Biol 45: 262-267).

      In addition to this, the threat to food security posed by this alteration in plant mineral status has also been well described in the literature by several modeling approaches (Beach RH (2019) Combining the effects of increased atmospheric carbon dioxide on protein, iron, and zinc availability and projected climate change on global diets: a modelling study. Lancet Planet Health 3: e307-e317; Ebi KL (2019) Elevated atmospheric CO(2) concentrations and climate change will affect our food's quality and quantity. Lancet Planet Health 3: e283-e284; Medek DE (2017) Estimated Effects of Future Atmospheric CO2 Concentrations on Protein Intake and the Risk of Protein Deficiency by Country and Region. Environ Health Perspect 125: 087002; Smith MR (2018) Impact of anthropogenic CO2 emissions on global human nutrition. Nature Climate Change 8: 834-839; Weyant C (2018) Anticipated burden and mitigation of carbon-dioxide-induced nutritional deficiencies and related diseases: A simulation modeling study. PLoS Med 15: e1002586; Zhu C (2018) Carbon dioxide (CO2) levels this century will alter the protein, micronutrients, and vitamin content of rice grains with potential health consequences for the poorest rice-dependent countries. Sci Adv 4: eaaq1012). To reinforce this point, we have added a sentence and references (lines 30-33). Nevertheless, we understand the reviewer's comment on the nuance to be given to the intensity of this potential threat. We have therefore modified the text, replacing "major threat" by "significant threat" (lines 3 and 29).

      We also would like to answer the reviewer’s comment on the potential environmental benefit associated with reduced N and P content in grains (mitigation of N emissions and P leaching). Indeed, if this reduced N and P content results from a lowered use efficiency of soil nutrients by plants, as suggested by several studies (Bloom 2010, Cassan 2023, Gojon 2023 and references therein), this may at the opposite favor N oxides emission and P leaching from the soil.

      (2) Exaggerated Concerns:

      The paper begins with the concern that carbon fertilization will lead to carbon dilution in our foods. While we indeed face numerous genuine threats in the coming decades, this particular issue is manageable. The increase in CO2 alone offers many opportunities for boosting yield. However, the heightened heat and increased evapotranspiration will pose massive challenges in many environments.

      While there are indeed multiple threats that we are facing in the coming decades, we don't fully agree with this comment. At present, there's no evidence to say that the negative effect of CO2 on plant mineral content will be manageable. Furthermore, there is compelling evidence that altered mineral nutrition and mineral status of plants will be an important factor limiting the high CO2-induced increase in yield, as will be heat or increased evapotranspiration (see for instance Coskun et al (2016) Nutrient constraints on terrestrial carbon fixation: The role of Nitrogen. J. Plant Physiol. 203: 95-109; Jiang M (2020) Low phosphorus supply constrains plant responses to elevated CO2 : A meta-analysis. Glob Chang Biol 26: 5856-5873 ; Reich PB (2006) Nitrogen limitation constrains sustainability of ecosystem response to CO2. Nature 440: 922-925). Thus, although we do not negate the crucial importance of heat and water stress, we believe it is relevant to study the basic mechanisms responsible for the negative effect of CO2 on plant mineral composition.

      Figure 4 in fact suggests that 43% of the REGMAP panel (cluster 3) is already pre-adapted to very high CO2 levels. This suggests annual species could adapt very rapidly.

      We agree with the reviewer. However, this suggests that genetic variation exists in some ecotypes to support adaptation to elevated CO2. The purpose of this work is indeed to identify this genetic variation, in order to characterize the mechanisms behind.

      (3) Assumptions on CO2 Levels:

      The assumption of 900ppm seems to be based on a very extreme climate change scenario. Most people believe we will overshoot the 1.5°C scenario, however, it seems plausible that 2.5 to 3°C scenarios are more likely. This would correspond to around 500ppm of CO2. https://www.nature.com/articles/s41597-022-01196-7/tables/4

      We agree with the reviewer that the CO2 concentration we used corresponds to a high value in the IPCC projections. That said, this value is currently considered very plausible: the following figure (from Smith and Myers (2018) Nature Climate Change) shows that current CO2 emissions align with the IPCC's most extreme model (RCP 8.5), which would result in a CO2 concentration of around 900 ppm in 2100. Furthermore, nothing allows to exclude the 4°C scenario in the 6th IPCC report.

      Author response image 1.

      (4) Focus on Real Challenges:

      We have numerous real challenges, such as extreme heat and inconsistent rainfall, to address in the context of climate change. However, testing under extreme CO2 conditions and then asserting that carbon dilution will negatively impact nutrition is exaggerated.

      While we fully agree that several threats linked to climate change exist, and all deserve to be studied, we find it questionable to consider that the potential effect of high CO2 on the mineral nutrition of plants is not a real challenge. The mineral nutrition of plants is already a current major environmental challenge. This perspective seems to reflect the reviewer's personal opinion rather than an analysis of our work.

      In contrast, the FACE experiments are fundamental and are conducted at more realistic eCO2 levels. Understanding the interaction between a 20% increase in CO2 and new precipitation patterns is key for global carbon flux prediction.

      Again, we do not fully understand this comment, as the aim of our study was not to perform a global carbon flux prediction, but to unravel genes and mechanisms underlying the negative effect of elevated CO2 on the nutrient content of Arabidopsis rosettes. However, we agree with the reviewer’s comment and with the fact that FACE are useful facilities to explore the CO2 response in more natural environments, and we highlight the fact that the decrease in mineral status of C3 plants has been widely documented in FACE studies. FACE experiments do not facilitate, however, to conduct fully controlled experiments (temperature, rainfall, wind and light intensities are not controllable in FACE), that allow to disentangle the mechanisms by which elevated CO2 regulates the signaling pathways associated with the plant mineral composition. In the longer term, studying the mechanisms we have identified in a more global context of climate change could be highly relevant.

      As I look at the literature on commercial greenhouse tomato production, 1000ppm of eCO2 is common, but it also looks like the breeders and growers have already solved for flavor and nutrition under these conditions.

      Indeed, tomato is often cultivated in CO2-enriched greenhouses at 1000 ppm. According to the literature, this results in a 20-25% reduction in vitamin C or lycopene, and requires a significantly higher nitrogen and water intake to reach expected sugar levels (Doddrell H (2023) Horticulture Research). In addition, the negative effect of elevated CO2 on tomato nutrient content seems to have significant repercussions on nutrition-health properties (Boufeldja (2023), Molecules).

      Conclusion:

      While the study provides valuable insights into the genetic underpinnings of Arabidopsis thaliana's response to elevated CO2 levels, it requires an entirely revised writeup, especially in its abstract, broader claims and implications. The manuscript would benefit from a more thorough introduction, a clearer definition of its scope, and a clear focus on the limits of this study.

      We thank the reviewer for the comments made on our manuscript. In addition to the responses that we provide to these comments, we have modified the main text of the introduction, objectives and discussion to take these comments into consideration. We believe that this will significantly improve the manuscript.

      Reviewer #2 (Public Review):

      Strengths:

      The authors have conducted a large, well-designed experiment to test the response to eCO2. Overall, the experimental design is sound and appropriate for the questions about how a change in CO2 affects the ionome of Arabidopsis. Most of the conclusions in this area are well supported by the data that the authors present.

      We thank the reviewer for this positive appreciation.

      Weakness:

      While the authors have done good experiments, it is a big stretch from Arabidopsis grown in an arbitrary concentration of CO2 to relevance to human and animal nutrition in future climates. Arabidopsis is a great model plant, but its leaves are not generally eaten by humans or animals.

      We agree with the reviewer’s comment. We recognized that implying a direct contribution of our work to human nutrition in the future climates is overstated, as mentioned by the reviewer 1 as well. This was not an intentional overstatement, as we have always been convinced that our work contributed to the understanding of the basic mechanisms involved in the negative regulation of plant mineral nutrition by high CO2. We have significantly modified the text to correct any misunderstanding of our work’s implication.

      The authors don't justify their choice of a CO2 concentration. Given the importance of the parameter for the experiment, the rationale for selecting 900 ppm as elevated CO2 compared to any other concentration should be addressed. And CO2 is just one of the variables that plants will have to contend with in future climates, other variables will also affect elemental concentrations.

      We agree with this comment. We added a justification of the high CO2 concentration used in this work in the Material and Methods section (lines 343-344). You can also read the explanation of this choice in the response to the reviewer 1’s point 3.

      Given these concerns, I think the emphasis on biofortification for future climates is unwarranted for this study.

      Anew, we agree with this comment and we have significantly modified the text to correct any misunderstanding of our work’s implication.

      Additionally, I have trouble with these conclusions:

      -Abstract "Finally, we demonstrate that manipulating the function of one of these genes can mitigate the negative effect of elevated CO2 on the plant mineral composition."

      -Discussion "Consistent with these results, we show that manipulating TIP2;2 expressions with a knock-out mutant can modulate the Zn loss observed under high CO2."

      The authors have not included the data to support this conclusion as stated. They have shown that this mutant increases the Zn content of the leaves when compared to WT but have not demonstrated that this response is different than in ambient CO2. This is an important distinction: one way to ameliorate the reduction of nutrients due to eCO2 is to try to identify genes that are involved in the mechanism of eCO2-induced reduction. Another way is to increase the concentration of nutrients so that the eCO2-induced reduction is not as important (i.e. a 10% reduction in Zn due to eCO2 is not as important if you have increased the baseline Zn concentration by 20%). The authors identified tip2 as a target from the GWAS on difference, but their validation experiment only looks at eCO2.

      We thank the reviewer for this comment, and we agree with it. It is much more interesting, especially in the context of this paper, to analyze the function of a candidate gene not only in elevated CO2, but in both ambient and elevated CO2. Therefore, we added in Figure 7 data for the expression of TIP2;2 in contrasted haplotypes under ambient CO2, in comparison to those already presented under elevated CO2 (now Fig. 7C and 7D). This showed that TIP2;2 expression is lower in haplotype 0 also under ambient CO2. We also added in Figure 7 (Fig. 7E) the Zn level in WT and tip2;2-1 mutant under ambient CO2, in comparison to those already presented under elevated CO2. This showed that that the tip2;2-1 mutant line did not present any decrease in Zn shoot content in response to elevated CO2, in opposition to what is observed for the WT.

      We have added comments associated to these new results in the Results and Discussion sections and in the discussion section (lines 233-242 in the results section, and lines 310-314 in the discussion section).

      Recommendations for the authors:

      Reviewer #1 (Recommendations For The Authors):

      Reviewer Comments on the Article's Approach to Ionome Analysis

      (1) Omission of Phosphorus from the Ionome:

      It's surprising that phosphorus (P) was not measured in the ionome. After nitrogen (N), P is often the most limiting mineral for plant development and yield, making it a significant component of the ionome. Why did the authors omit this crucial element?

      We agree with the reviewer that P is an important mineral for plant growth. The absence of data related to P content is due to feasibility constraints rather than oversight. The MP-AES instrument we used to analyze the ionome (except N and C, that we obtained from an Elementar Analyzer) would have required an extra-step and an extra-analysis to obtain data for macronutrient such as P or K. In the context of this large-scale experiment, we faced the necessity to compromise and proceed without these data.

      (2) Relationship Between Leaf Ionome and Seed:

      The manuscript lacks evidence demonstrating the relationship between the leaf ionome and the seed. This connection is vital to establish the study's aims as outlined in lines 20-24. If the central argument is that eCO2 threatens food security, it's essential for the authors to either:

      • Provide evidence that eCO2 induces changes in the ionome profiles of seeds.

      • Show that changes in the rosette leaf ionome lead to alterations in seed ionome profiles.

      We agree with the reviewer. Although we know that seed ionome composition of Arabidopsis model accession such as Columbia is indeed negatively affected by eCO2, we do not provide the data that support some of the terms used in lines 20-24. The correspondence between leaf and seed ionome in natural population under eCO2 is certainly a next question that we will address. Therefore, to align our stated objectives with our data, we have modified the sentence in lines 20-24. We also added a comment on this point lines on the discussion section (lines 324-328).

      (3) Analysis of Ionome in Rosette Leaves:

      Why did the authors choose to analyze the ionome specifically in rosette leaves? Is there a known correlation between the ionome profile in rosette leaves and seeds?

      See our answer to the above comment.

      (4) Experimental Design Comments:

      • The layout of the accession growouts, the methods of randomization, blocking, and controls/checks should be detailed.

      • Were BLUEs (Best Linear Unbiased Estimators) or BLUPs (Best Linear Unbiased Predictors) employed to account for experimental design conditions? If not, it's recommended that they be used.

      We thank the reviewer for this comment. A note on replicates has been added in the Method/Plant Material section. Concerning the BLUEs/BLUPs, although I am not familiar with their use, I do not think that these approaches are relevant in our experimental design. Indeed, we pooled 3 to 5 replicates for each accession to measure the ionome (as mentioned in the Method/Ionome analysis section – we realized this was perhaps not clear enough, and thus we reinforced this point in this section). Therefore, we do not have the variance data required to perform BLUEs/BLUPs.

      (5) Carbon Dilution Effect:

      The statement, "The first component of the PCA described a clear antagonistic trend between C content and the change of other mineral elements (Fig. 3B)..." suggests a well-understood carbon dilution effect. These results are anticipated and align with existing knowledge.

      We thank the reviewer for this comment. However, this sentence does not relate to the biomass dilution hypothesis referred to by the reviewer. Indeed, the composition of each mineral (C and others) is expressed as a percentage of biomass, not as an absolute value. Therefore, this reflects more a probable effect of the increase in carbon compounds (notably soluble sugars), which could influence mineral composition.

      (6) Heritability Estimates:

      The authors should report both the broad-sense heritability and an estimate of heritability based on a GRM or Kinship matrix.

      We thank the reviewer for this suggestion. We are skeptical of using a kinship matrix to estimate heritability in our study. Estimating narrow-sense heritability using a kinship matrix is conceptually based on the infinitesimal model of Fisher, thereby meaning that phenotypic variation is driven by hundreds to thousands of QTLs with small effects. If this is the case, GWAS conducted on several hundred (or even thousands) of genotypes will not be powerful enough to detect such QTLs. Accordingly, estimates of broad-sense heritability based on estimates of variance components can drastically differ from estimates of narrow-sense heritability based on the use of a kinship matrix, as illustrated in the study of Bergelson et al. (2019 Scientific Reports).

      (7) Application of the Breeder's Equation:

      It would be beneficial if the authors applied the breeder's equation to estimate the species' potential rate of response. Based on the allele frequency of the adapted cluster 3 (69 ecotypes or 43% frequency of Figure 3B), it seems plausible that the populations could adapt within 23 generations.

      We thank the reviewer for this suggestion. Indeed, it would be really interesting to test whether sub-populations could adapt in comparison with others, and over what period of time. It is nevertheless not possible to do so using the Breeder’s equation in our case, as this requires fitness data under conditions of ambient or elevated CO2 (i.e. production of seeds) to be applied, and we do not have these data at the level of the whole population.

      (8) Overall Quality:

      In general, the authors have executed a high-quality ionome mapping experiment. However, the abstract, introduction, and discussion should be entirely rewritten and reframed.

      We thank the reviewer for the positive evaluation of our experiment. As previously mentioned, we are for the most part in agreement with the comments made about the need to align our stated objectives with our experimental data and conclusions. To do so, we have rewritten part of the abstract, introduction and discussion. The details of these modifications are described in the responses made to each comment.

      Here's a line-by-line list of suggestions on writing:

      Line 30 would read better with a comma after thus (or by replacing thus with therefore and then a comma at the start of the sentence).

      Line 33 nevertheless would read better in between commas.

      Lines 45 - 48 sentence is too long, could probably divide it into two.

      Lines 90 - 94 are hard to interpret, recommend rephrasing for clarity.

      Line 130 - keep verbs in the past tense for consistency (ran instead of run).

      Line 194 - what do the authors mean by crossed? I'm inferring they looked at the intersection of DEGs with the list of genes identified by GWA mapping, probably should use a more concise word.

      There's a concurrent use of the adjective strong (Lines 80, 142, 144, 197, 245). I would advise using a more concise adjective or avoiding its use to let the reader form their own opinion on the data.

      Lines 174-176 the cited reference (No. 15) is incorrect. The study by Katz et al. (2022) does not provide information on the role of ZIF1 in zinc sequestration mechanisms under elevated CO2 conditions.

      We thank the reviewer for these detailed recommendations. We have corrected or rephrased the text according to these suggestions.

      Reviewer #2 (Recommendations For The Authors):

      Technical points:

      900 ppm as elevated CO2: Given the importance of the parameter for the experiment, the rationale for selection 900 ppm as elevated CO2 compared to any other concentration should be addressed.

      We acknowledge the reviewer's point and have previously addressed related aspects earlier in our response. In line with this, we have included a justification for this particular parameter in the Method section.

      The authors do not mention what genotype was used for their root/shoot RNAseq experiment.

      We thank the reviewer for this comment, and indeed, this information was not mentioned. This is now done, in the Method section.

      Line 125: Spelling error "REGMPA".

      This has been corrected.

      Line 338: Removal of outlier observations - "Prior to GWAS and multivariate analyses such as PCA or clustering, mineral composition measures were pre-processed to remove technical outliers". The authors should mention the exact number of outliers that were removed and what the explicit criteria were for removal.

      The number of outliers removed from each dataset is now indicated in Supplemental Table 7 (this is cited in the Method section). The explicit criteria used for this analysis is actually mentioned in the corresponding Method section: “the values positioned more than 5 median absolute deviations away from the median were removed from the dataset”.

      Line 379: "Lowly expressed genes with an average value across conditions under 25 reads were excluded from the analysis". Providing information about the number of the lowly expressed genes that were removed from the analysis can help with the interpretation of the likelihood of the candidates selected being correct.

      This is a standard procedure in RNAseq analysis. It avoids many false positives in the differential analysis of gene expression based on ratios (where a very small number in the denominator can lead to a very high variation in expression, of no real significance). For information, this step led to the removal of 11607 and 10121 genes for the shoot and root datasets.

      Line 384: It's not clear how many biological replicates were used.

      This has been corrected.

      Additional comment: We have also become aware of a confusion concerning one of the candidate genes located close to GWA peaks: line 180 of the first version, we mentioned CAX1 (AT1G16380) for its role on nutrient deficiency response. There are actually two genes annotated as CAX1 in TAIR (both are cation exchangers), but the one involved in nutrient deficiency response is AT2G38170. We therefore removed the sentence mentioning AT1G16380/CAX1 as a potential candidate gene.

    1. Author Response

      The following is the authors’ response to the original reviews.

      Reviewer #1 (Public Review):

      This paper performed a functional analysis of the poorly characterized pseudo-phosphatase Styxl2, one of the targets of the Jak/Stat pathway in muscle cells. The authors propose that Styxl2 is essential for de novo sarcomere assembly by regulating autophagic degradation of non-muscle myosin IIs (NM IIs). Although a previous study by Fero et al. (2014) has already reported that Styxl2 is essential for the integrity of sarcomeres, this study provides new mechanistic insights into the phenomenon. In vivo studies in this manuscript are compelling; however, I feel the contribution of autophagy in the degradation of NM IIs is still unclear.

      Major concerns:

      1) The contribution of autophagy in the degradation of Myh9 is still unclear to this reviewer.

      It has been reported that autophagy is dispensable for sarcomere assembly in mice (Cell Metab, 2009, PMID; 1994508). In Fig. 7A, the authors showed that overexpressed Styxl2 downregulated the amount of ectopically expressed Myh9 in an ATG5-dependent manner in C2C12 cells; however, the experiment is far from a physiological condition. Therefore, the authors should test ATG5 knockdown and the genetic interaction between Styxl2 and ATG5 in vivo. That is, 1) loss of ATG5 on sarcomere assembly in zebrafish, and 2) the genetic interaction between Styxl2 and ATG5; co-injection of Styxl2 mRNA and ATG5-MO into the zebrafish embryos.

      Our response: In fact, the reference cited by the reviewer (Cell Metab, 2009; PMID; 19945408) clearly indicated that autophagy is required for sarcomere assembly. Moreover, another paper using the fish extraocular muscle regeneration model (Autophagy, 2014, PMID: 27467399), also showed that the sarcomere structure was disrupted in the regenerated muscles when autophagy was inhibited by chloroquine. In addition, other references (Nature medicine, 2007, PMID: 17450150; Autophagy, 2010, PMID: 20431347) also showed that loss of Atg5 in mouse cardiac muscles led to disorganized sarcomere structure. We also performed the Atg5 knockdown experiments as suggested by the reviewer. However, the sarcomere structure defects were not so obvious as Styxl2 knockdown (see Author response image 1 below). In fact, it was reported that Atg5 knockdown may not be a desirable strategy to disrupt autophagy as it was found “--- only a small amount of Atg5 is needed for autophagy, knockdown of Atg5 to levels low enough to block autophagy might be difficult to achieve, --” (Nature medicine, 2007, PMID: 17450150). Due to the ineffectiveness of the Atg5 MO in our assays, we did not perform the second experiment suggested by the reviewer. Moreover, as Styxl2 is not a key component of the autophagy machinery, it is less likely that overexpression of Styxl2 alone can rescue the autophagy defects caused by Atg5.

      Author response image 1.

      The fish zygotes were injected with Atg5 or Ctrl MO. 48 hpf, the fish were stained with an anti-Actinin antibody. Some fast muscle fibers were disrupted when Atg5 was knocked down. The number in numerator at the bottom of each image represents fish embryos showing normal Actinin staining pattern, while that in denominator represents the total number of embryos examined. Scale bar, 10 µm.

      2) As referenced, Yamamoto et al. reported that Myh9 is degraded by autophagy. Mechanistically, Nek9 acts as an autophagic adaptor that bridges Atg8 and Myh9 through interactions with both. Inconsistent with the model, the authors mentioned on page 12, lines 365-367, "A recent report showed that Myh9 could also undergo Nek9-mediated selective autophagy (Yamamoto et al., 2021), suggesting that Myh9 is ubiquitinated". I think it is not yet explored whether autophagic degradation of Myh9 requires its ubiquitination. Moreover, I cannot judge whether Myh9 is ubiquitinated in a Styxl2-dependent manner from the data in Fig. 7C. The author should test whether Nek9 is required for Myh9 degradation in muscles. If Nek plays a role in the Myh9 degradation, it would be better to remove Fig. 7C.

      Our response: Indeed, as pointed out by the reviewer, it has not been explored whether Myh9 is ubiquitinated or not. However, it has been well-established that some proteins undergoing autophagic degradation are ubiquitinated, which are linked to Atg8/LC3 via p62 and NBR1 (Mol Cell, 2009, PMID: 19250911; J Biol Chem, 2007, PMID: 17580304). To improve the data quality, we repeated the Myh9 ubiquitination experiment in cells with or without Styxl2 by using a slightly different strategy: as shown in the revised Figure 7C, we first co-transfect HEK 293T cells with HA-Myh9, Myc-ubiquitin, and Flag-Styxl2. We then immunoprecipitated Myc-tagged Ubiquitin from the whole cell lysates, and then blot for HAMyh9. We detected an obvious increase in Ubiquitin-conjugated HA-Myh9 (revised Figure 7C). As suggested by the reviewer, we also tested whether knockdown of Nek9 affects the degradation of Myh9. We failed to detect an obvious effect (see Author response image 2 below) caused by Nek9 knockdown. One possible explanation for this negative result is that Nek9 itself is a negative regulator of selective autophagy (J Biol Chem, 2020, PMID: 31857374). By knocking it down, the functions of the autophagy machinery are expected to be enhanced instead of being impaired. This may explain why we failed to detect an effect on Myh9 degradation simply by knocking down Nek9. To further elucidate whether Nek9 is involved in Myh9 degradation in myoblasts, we may need to use a dominant-negative mutant of Nek9 missing the LCIII-binding motif as shown by Yamamoto (Nat Commun, 2021, PMID: 34078910). This will be addressed in our future study.

      Author response image 2.

      C2C12 cells were transfected with negative control siRNA (NC), siNek9#2 or siNek9#3. 18 h later, the cells were transfected with plasmids HA-Myh9 and Flag-Styxl2 or Flag-Stk24. After another 24 h, the cells were harvested for RT-qPCR (left panel) or western blot (right panel).

      3) In Fig. 5F, the protein level of Styxl2 and Myh10 should be checked because the efficiency of Myh10-MO was not shown anywhere in this manuscript.

      Our response: As suggested by the reviewer, a Western blot showing the protein levels of Myh10 was shown in Figure 5-figure supplement 1B.

      Reviewer #2 (Public Review):

      The authors investigated the role of the Jak1-Stat1 signaling pathway in myogenic differentiation by screening the transcriptional targets of Jak1-Stat1 and identified Styxl2, a pseudophosphatase, as one of them. Styxl2 expression was induced in differentiating muscles. The authors used a zebrafish knockdown model and conditional knockout mouse models to show that Styxl2 is required for de novo sarcomere assembly but is dispensable for the maintenance of existing sarcomeres. Styxl2 interacts with the non-muscle myosin IIs, Myh9 and Myh10, and promotes the replacement of these non-muscle myosin IIs by muscle myosin IIs through inducing autophagic degradation of Myh9 and Myh10. This function is independent of its phosphatase domain.

      A previous study using zebrafish found that Styxl2 (previously known as DUSP27) is expressed during embryonic muscle development and is crucial for sarcomere assembly, but its mechanism remains unknown. This paper provides important information on how Styxl2 mediates the replacement of non-muscle myosin with muscle myosin during differentiation. This study may also explain why autophagy deficiency in muscles and the heart causes sarcomere assembly defects in previous mouse models.

      Reviewer #3 (Public Review):

      Wu and colleagues are characterising the function of Styxl2 during muscle development, a pseudo-phosphatase that was already described to have some function in sarcomere morphogenesis or maintenance (Fero et al. 2014). The authors verify a role for Styxl2 in sarcomere assembly/maintenance using zebrafish embryonic muscles by morpholino knockdown and by a conditional Styxl2 allele in mice (knocked-out in satellite cells with Pax7 Cre).

      Experiments using a tamoxifen inducible Cre suggest that Styxl2 is dispensable for sarcomere maintenance and only needed for sarcomere assembly.

      BioID experiments with Styxl2 in C2C 12 myoblasts suggest binding of nonmuscle myosins (NMs) to Styxl2. Interestingly, both NMs are downregulated when muscles differentiate after birth or during regeneration in mice. This down-regulation is reduced in the Styxl2 mutant mice, suggesting that Styxl2 is required for the degradation of these NMs.

      Impressively, reducing one NM (zMyh10) by double morpholino injection in a Styxl2 morphant zebrafish, does improve zebrafish mobility and sarcomere structure. Degradation of Mhy9 is also stimulated in cell culture if Styxl2 is co-expressed. Surprisingly, the phosphatase domain is not needed for these degradation and sarcomere structure rescue effects. Inhibitor experiments suggest that Styxl2 does promote the degradation of NMs by promoting the selective autophagy pathway.

      Strengths:

      A major strength of the paper is the combination of various systems, mouse and fish muscles in vivo to test Styxl2 function, and cell culture including a C2C12 muscle cell line to assay protein binding or protein degradation as well as inhibitor studies that can suggest biochemical pathways.

      Weakness:

      The weakness of this manuscript is that the sarcomere phenotypes and also the western blots are not quantified. Hence, we rely on judging the results from a single image or blot. Also, Styxl2 role in sarcomere biology was not entirely novel.

      Few high resolution sarcomere images are shown, myosins have not been stained for.

      Reviewer #1 (Recommendations For The Authors):

      Minor concerns:

      4) The position of molecular weight markers should be shown in all Western blot data.

      Our response: As suggested by the reviewer, the molecular weight markers have been added in the Western blot data.

      5) Schematic models of Styxl2deltaN509 and N513 construct would be helpful for the readers.

      Our response: A schematic has been added in Figure 6B (upper panel) to show Styxl2deltaN509 and Styxl2N513.

      6) Several data were described but not shown (data not shown). I think the data need to be included in the main or supplemental figures.

      Our response: As suggested by the reviewer, the raw data were now included in the Figure 6-figure supplement 1A and Figure 7-figure supplement 1.

      Reviewer #2 (Recommendations For The Authors):

      1) In Fig. 5E, the authors suggest that the needle touch response was improved by additional knockdown of Myh10. This is a bit confusing because the germline knockout of Myh10 is lethal (line 445). The authors should provide more explanation on this point. Additionally, it would be better to include Myh10-MO in Fig. 5E.

      Our response:<br /> In line 445 of our original manuscript, we stated that germline knockout of mouse Myh10 gene is lethal based on a published report (Proc Natl Acad Sci USA, 1997, PMID: 9356462). Here, in zebrafish zygotes, we only knocked down zMyh10, thus, we do not expect to get a lethal phenotype. In addition, other groups who knocked down Myh10 in fish also did not get a lethal phenotype (Dev Biol, 2015, PMID: 25446029). As to the control involving Myh10MO in the experiment in Fig.5E, we did include it in our experiments. As we did not observe any obvious effects on either motility or sarcomere structures, we did not include the data set in the figure.

      2) It was suggested that Myh9 and Myh10 form a complex (Rao et al. PLoS One 9, e114087, 2014). Thus, the IP experiments do not rule out the possibility that Styxl2 directly interacts with either Myh9 or Myh10 and indirectly with the other.

      Our response: In known myosin-II complexes, different myosin molecules can associate with each other through their tail domains (Bioarchitecture, 2013, PMID: 24002531). Thus, if we use fulllength myosin molecules in our co-immunoprecipitation assays, it will be difficult to exclude the possibility raised by the reviewer. However, by using truncated myosin proteins, we showed that the head domain of either Myh9 or Myh10 could interact with Styxl2 in the absence of the tail domain (Figure 4E, F). This result strongly suggests that both Myh9 and Myh10 can independently interact with Styxl2.

      Reviewer #3 (Recommendations For The Authors):

      1) The western blot shown in Figure 3B supporting the induced deletion of Styxl2 should be quantified. Ideally, some other blots, e.g., in Figure 5, too. Please add the age of the mice in Figure 5B to the figure legend.

      Our response:<br /> As suggested by the reviewer, we quantified the data in Figures.3B, 3F, 5B, 5D, and 7A and the data were included in the revised figures. In Fig.5B, we already indicated the age of the mice (i.e., P1) in the legend.

      2) A quantification of the sarcomere phenotypes in the double knock-down of zMyh10 and Styxl2 compared to Styxl2 single would make the paper significantly stronger. Furthermore, a double morpholino control should be included to rule out any RNAi machinery 'dilution effect'.

      Our response: As suggested by the reviewer, we quantified the sarcomere structures using the line scan analysis in ImageJ and the scan images were placed as inserts in the upper corner of the immunofluorescent images (revised Figures 5F, and 6C). To avoid potential “dilution effects”, in all the experiments involving the use of two different MOs, the total amount of MO was kept the same in all control samples by including a control MO (e.g., in samples treated with one specific MO, an equal amount of a control MO was also included, while in samples without any specific MO, twice as much control MO was used).

      3) The sarcomere phenotypes in figure 6 should also be better quantified, for example using simple line scans of the alpha-actinin stains and assay periodicity or calculating the autocorrelation coefficients. How about myosin stains?

      Our response: We quantified Figure 6C as suggested by the reviewer. We also performed myosin staining. The results were similar to that shown by the a-actinin antibody (see revised Figure 6-Fig supplement 1B).

      4) Do the authors see periodic NMs patterns in developing mouse muscle fibers as indicated by the model in in in figure 7D? It is unclear if nonmuscle myosin is present in a PERIODIC pattern in early myofibrils. NM myosin periodic patterns that have been observed have a periodicity of only about 1 µm fitting the shorter length of the NM bipolar filaments (about 300 nm only, PMID 28114270).

      Our response: The reviewer raised a good point here. Ideally, we should examine developing mouse muscle fibers to prove that NM shows periodic patterns. However, due to the difficulty in catching myocytes undergoing sarcomere assembly, the majority of the studies involving NM in sarcomeres use cultured cardiomyocytes. Using TA muscles from P1 new-born mice, we failed to detect the presence of NM in sarcomeres (see Author response image 3 below). Actually, nearly all the myofibers showed mature sarcomere pattern without the NM signal. More work is needed in the future to examine developing mouse fibers at different embryonic stages to look for the presence of NM in developing sarcomeres.

      Author response image 3.

      The TA muscles were collected from male and female P1 mice. The muscles were sectioned and co-stained for a-actinin (Actn) and Myh9. The majority of myofibrils is mature without the NM II signal. Scale bar, 10 µm.

      5) Recent work suggested that mechanical tension is key to assemble the first long periodic myofibril containing immature sarcomeres. Tension is likely produced by a combination of NM and Mhc in the assembling sarcomeres themselves. This could be included in the introduction or discussion (PMIDs 24631244, 29316444, 29702642, 35920628).

      Our response: We thank the reviewer for pointing to us additional relevant references. We have added them in the Introduction.

      6) I suggest replacing "sarcomeric muscles" with "striated muscles".

      Our response: We revised the term in the manuscript as suggested by the reviewer.

    1. Author response:

      The following is the authors’ response to the original reviews.

      Reviewer #1 (Public Review):

      We appreciate the valuable and constructive comments of Reviewer #1 on our manuscript. We have addressed the comments from Reviewer #1 in the public review in the response to the recommendations for the authors, as the public review comments largely overlap with that of the recommendations for the authors.

      Reviewer #1 (Recommendations For The Authors):

      (1.1) Figure 1 did not use a mock-infected control for the development of R-loops but only a time before infection. I think it would have been a good control to have that after the same time of infection non-infected cells did not show increases in R-loops and this is not a product of the cell cycle.

      We prepared our DRIPc-seq library using cell extracts harvested at 0, 3, 6, and 12 h post-infection (hpi), all at the same post-seeding time point. Each sample was infected with HIV-1 virus in a time-dependent manner. Therefore, it is unlikely that the host cellular R-loop induction observed in our DRIPc-seq results was due to R-loop formation during the cell cycle. In Lines 93–95 of the Results section of the revised manuscript, we have provided a more detailed description of our DRIPc-seq library experimental scheme. Thank you. 

      (1.2) Figure 2 should have included a figure showing the proportion of DRIPc-seq peaks located in different genome features relative to one another instead of whether they were influenced by time post-infection. Figure 2C was performed in HeLa cells, but primary T cell data would have been more relevant as primary CD4+ T cells are more relevant to HIV infection.

      We have included a new figure presenting the relative proportion of DRIPc-seq peaks mapped to different genomic features at each hpi (Fig. 2C of the revised manuscript). We found that the proportion of DRIPc-seq peaks mapped to various genomic compartments remained consistent over the hours following the HIV-1 infection. This further supports our original claim that HIV-1 infection does not induce R-loop enrichment at specific genomic features but that the accumulation of R-loops after HIV-1 infection is widely distributed.

      We considered HeLa cells as the primary in vitro infection model, therefore, we conducted RNA-seq only on HeLa cells. However, we agree with the reviewer's opinion that data from primary CD4+ T cells may be more physiologically relevant. Nevertheless, as demonstrated in the new figure (Fig. 2C of the revised manuscript), HIV-1 infection did not significantly alter the proportion of R-loop peaks mapped to specific genomic compartments, such as gene body regions, in HeLa, primary CD4+ T, and Jurkat cells. Therefore, we anticipate no clear correlation between changes in gene expression levels and R-loop peak detection upon HIV-1 infection, even in primary T cells. Thank you.   

      (1.3) Figure 5G is very hard to see when printed, is there a change in brightness or contrast that could be used? The arrows are helpful but they don't seem to be pointing to much.

      We have highlighted the intensity of the PLA foci and magnified the images in Fig. 5G in the revised manuscript. While editing the images according to your suggestion, we found a misannotation regarding the multiplicity of infection in the number of PLA foci per nucleus quantification analysis graph in Fig. 5G of the original manuscript. We have corrected this issue and hope that it is now much clearer. 

      (1.4) The introduction provided a good background for those who may not have a comprehensive understanding of DNA-RNA hybrids and R-loops, but the rationale that integration in non-expressed sequence implies that R-loops may be involved is very weak and was not addressed experimentally. A better rationale would have been to point out that, although integration in genes is strongly associated with gene expression, the association is not perfect, particularly in that some highly expressed genes are, nonetheless, poor integration targets.

      In accordance with the reviewer's comment, we revised the Introduction. We have deleted the statement and reference in the introduction "... the most favored region of HIV-1 integration is an intergenic locus, ...”, which may overstate the relevance of the R-loop in HIV-1 integration events in non-expressed sequences. Instead, we introduced a more recent finding that high levels of gene expression do not always predict high levels of integration, together with the corresponding citation (Lines 46– 47 of the revised manuscript), according to the reviewer’s suggestion in the reviewer's public review 2)-(a).

      (1.5) The discussion was seriously lacking in connecting their conclusions regarding R-loop targeting of integration to how integration works at the structural level, where it is very clear that concerted integration on the two DNA strands ca 5 bp apart is essential to correct, 2-ended integration. It is very difficult to visualize how this would be possible with the triple-stranded R-loop as a target. The manuscript would be greatly strengthened by an experiment showing concerted integration into a triplestranded structure in vitro using PICs or pure integrase.

      We believe there has been a misunderstanding of our interpretation regarding the putative role of R-loop structures in the HIV-1 integration site mechanism because of some misleading statements in our original manuscript. Based primarily on our current data, we believe that R-loop structures are bound by HIV-1 integrase proteins and lead to HIV-1 viral genome integration into the vicinity regions of the host genomic R-loops. By carefully revising our manuscript, we found that the title, abstract, and discussion of our original manuscript includes phrases, such as “HIV-1 targets R-loops for integration,” which may overstate our finding on the role of R-loop in HIV-1 integration site selection. We replaced these phrases. For example, we used phrases, such as, “HIV-1 favors vicinity regions of R-loop for the viral genome integration,” in the revised manuscript. We apologize for the inconvenience caused by the unclear and nonspecific details of our findings.  

      Using multiple biochemical experiments, we successfully demonstrated the interaction between the cellular R-loop and HIV-1 integrase proteins in cells and in vitro (Fig. 5 of the revised manuscript). However, we could not validate whether the center of the triple-stranded R-loops is the extraction site of HIV-1 integration, where the strand transfer reaction by integrase occurs. This is because an R-loop can be multi-kilobase in size (1, 2); therefore, we displayed a large-scale genomic region (30-kb windows) to present the integration sites surrounding the R-loop centers. Nevertheless, we believe that we validated R-loop-mediated HIV-1 integration in R-loop-forming regions using our pgR-poor and pgR-rich cell line models. When infected with HIV-1, pgR-rich cells, but not pgR-poor cells, showed higher infectivity upon R-loop induction in designated regions following DOX treatment (Fig. 3C and 3D of the revised manuscript). In addition, we quantified site-specific integration events in R-loop regions, and found that a greater number of integration events occurred in designated regions of the pgR-rich cellular genome upon R-loop induction by DOX treatment, but not in pgR-poor cells (Fig. 3E–G of the revised manuscript). 

      We agree with the reviewer that an experiment showing the concerted integration of purified PICs into a triple-stranded structure in vitro would greatly strengthen our manuscript. We attempted the purification of viral DNA (vDNA)-bound PICs using either Sso7d-tagged HIV-1 integrase proteins or non-tagged HIV-1 integrase proteins (F185K/C280S) procured from the NIH HIV reagent program (HRP-20203), following the method described by Passos et al., Science, 2017; 355 (89-92) (3). Despite multiple attempts, we could not purify the nucleic acid-bound protein complexes for in vitro integration assays. However, we believe that pgR-poor and pgR-rich cell line models provide a strong advantage in specificity of our primer readouts. Compounded with our in cellulo observation, we believe that our work provides strong evidence for a causative relationship between R-loop formation/R-loop sites and HIV-1 integration.

      Additionally, in the Discussion section of the revised manuscript, we have expanded our discussion on the role of genomic R-loops contributing in molding the host genomic environment for HIV-1 integration site selection, and the potential explanation on how R-loops are driving integration over long-range genomic regions. Thank you. 

      (1.6) There are serious concerns with the quantitation of integration sites used here, which should be described in detail following line 503 but isn't. In Figure 3, E-G, they are apparently shown as reads per million, while in Figure 4B as "sites (%)" and in 4C as log10 integration frequency." Assuming the authors mean what they say, they are using the worst possible method for quantitation. Counting reads from restriction enzyme-digested, PCR-digested DNA can only mislead. At the numbers provided (MOI 0.6, 10 µg DNA assayed) there would be about 1 million proviruses in the samples assayed, so the probability of any specific site being used more than once is very low, and even less when one considers that a 10% assay efficiency is typical of integration site assays. Although the authors may obtain millions of reads per experiment, the number of reads per site is an irrelevant value, determined only by technical artefacts in the PCR reactions, most significantly the length of the amplicons, a function of the distance from the integration site to the nearest MstII site, further modified by differences in Tm. Better is to collapse identical reads to 1 per site, as may have been done in Figure 4B, however, the efficiency of integration site detection will still be inversely related to the length of the amplicon. Indeed, if the authors were to plot the read frequency against distance to the nearest MstII site, it is likely that they would get plots much like those in Figure 4B.

      Detailed methods for integration site sequencing data processing are described in the Materials and Methods section of the revised manuscript (Line 621–631 of the revised manuscript). We primarily followed HIV-1 integration site sequencing data processing methods previously described by Li et al., mBio, 2020; 11(5) (4).  

      While it may be correct that the HIV-1 integration event cannot occur more than once at a given site, our Fig. 3E, 4C, and 4D of the revised manuscript present the number of integration-site sequencing read counts expressed in reads-per-million (RPM) units or as log10-normalized values. Based on the number of mapped reads from the integration site sequencing results, we can infer that there was an integration event at this site, whether it was a single or multiple event.

      We believe that the original annotation of y-axis, “Integration frequency,” may be misleading as it can be interpreted as a probability of any specific site being used for HIV-1 integration. Therefore, we corrected it as “number of mapped read” for clarity (Fig. 3E–G, 4C and 4D, and the corresponding figure legends of the revised manuscript). We apologize for any confusion. Thank you.

      Other points:

      (1.7) Overall: There are numerous grammatical and usage errors, especially in agreement of subject and verb, and missing articles, sometimes multiple times in the same sentence. These must be corrected prior to resubmission.

      The revised manuscript was edited by a professional editing service. Thank you.

      (1.8) Line 126-134: A striking result, but it needs more controls, as discussed above, including a dose-response analysis.

      We determined the doses of NVP and RAL inhibitors in HeLa cells by optimizing the minimum dose of drug treatment that provided a sufficient inhibitory effect on HIV1 infection (Author response image 1). The primary objective of this experiment was to determine R-loop formation while reverse transcription or integration of the HIV-1 life cycle was blocked, therefore, we do not think that a dose-dependent analysis of inhibitors is required.

      Author response image 1.

      (A and B) Representative flow cytometry histograms of VSV-G-pseudotyped HIV-1-EGFP-infected HeLa cells at an MOI of 1, harvested at 48 hpi. The cells were treated with DMSO, the indicated doses of nevirapine (NVP) (A) or indicated doses of raltegravir (RAL) (B) for 24 h before infection. 

      (1.9) Line 183: Please tell us what ECFP is and why it was chosen. Is there a reference for its failure to form R-loops?

      Ibid: The human AIRN gene is a very poor target for HIV integration in PBMC.

      A high GC skew value (> 0) is a predisposing factor for R-loop formation at the transcription site. This is because a high GC skew causes a newly synthesized RNA strand to hybridize to the template DNA strand, and the non-template DNA strand remains looped out in a single-stranded conformation (5) (Ref 36 in the revised manuscript). The ECFP sequence possessed a low GC skew value, as previously used for an R-loop-forming negative sequence (6) (Ref 17 of the revised manuscript). We have added this description and the corresponding references to Lines 188–192 of the revised manuscript.  

      The human AIRN gene (RefSeq DNA sequence: NC_000006.12) sequence possesses a GC skew value of -0.04, in a window centered at base 2186, while the mouse AIRN (mAIRN) sequence is characterized by a GC skew value of 0.213. The ECFP sequence gave a GC skew value of -0.086 in our calculation. We anticipated that the human AIRN gene region does not form a stable R-loop, and in fact, it did not harbor R-loop enrichment upon HIV-1 infection in our DRIPc-seq data analysis of multiple cell types (Author response image 2)

      Author response image 2.

      Genome browser screenshot over the chromosomal regions in 20-kb windows centered on human AIRN showing results from DRIPc-seq in the indicated HIV-1-infected cells (blue, 0 hpi; yellow, 3 hpi; green, 6 hpi; red, 12 hpi)

      (1.10) Line 190: You haven't shown dependence. Associated is a better word.

      Thank you for the suggestion. We have changed “R-loop-dependent site-specific HIV-1 integration events...” to “R-loop-associated site-specific HIV-1 integration events...” (Line 198 of the revised manuscript) according to the reviewer’s suggestion in the revised manuscript. 

      (1.11) Line 239: What happened to P1? What is the relationship of the P and N regions to genes?

      We have added superimpositions of the P1 chromatin region on DRIPc-seq and the HIV-1 integration frequency to Figure 4C of the revised manuscript. We observed a relevant integration event within the P1 R-loop region, but to a lesser extent than in the P2 and P3 R-loop regions, perhaps because the P1 region has relatively less R-loop enrichment than the P2 and P3 regions, as examined by DRIP-qPCR in S3A Fig. of the revised manuscript.

      Genome browser screenshots with annotations of accommodating genes in the P and N regions are shown in S2A–E Fig. of the revised manuscript, and RNA-seq analysis of the relative gene expression levels of the P1-3 and N1,2 R-loop regions are shown in S4 Table of the revised manuscript. Thank you.

      (1.12) Line 261: But the binding affinity of integrase to the R-loop is somewhat weaker than to double-stranded DNA according to Figure 5A.

      Nucleic acid substrates were loaded at the same molarity, and the percentage of the unbound fraction was calculated by dividing the intensity of the unbound fraction in each lane by the intensity of the unbound fraction in the lane with 0 nM integrase in the binding reaction. The calculated percentages of the unbound fraction from three independent replicate experiments are shown in Fig. 5A, right of the revised manuscript. In our analysis and measurements, the integrase proteins showed higher binding affinities to the R-loop and R-loop comprising nucleic acid structures than to dsDNA in vitro. We hope that this explanation clarifies this point. 

      (1.13) Line 337: "accumulate". This is a not uncommon misinterpretation of the results of studies on the distribution of intact proviruses in elite controllers. The only possible correct interpretation of the finding is that proviruses form everywhere else but cells containing them are eliminated, most likely by the immune system.

      Thank you for the suggestion. We have changed the Line 337 of the original manuscript to “... HIV-1 proviruses in heterochromatic regions are not eliminated but selected by immune system,” in Lines 361-363 of the revised manuscript. 

      (1.14) Line 371 How many virus particles per cell does this inoculum amount to?

      We determined the amount of GFP reporter viruses required to transduce ∼50% of WT Jurkat T cells, corresponding to an approximate MOI of 0.6. We repeatedly obtained 30–50% of VSV-G-pseudotyped HIV-1-EGFP positively infected cells for HIV1 integration site sequencing library construction for Jurkat T cells. 

      (1.15) Line 503 and Figures 3 and 4: There must be a clear description of how integration events are quantitated.

      Detailed methods for integration site sequencing data processing are described in the Materials and Methods section of the revised manuscript (Line 621–631 of the revised manuscript). We primarily followed HIV-1 integration site sequencing data processing methods previously described in Li et al., mBio, 2020; 11(5) (4).

      Reviewer #2 (Public Review):

      Retroviral integration in general, and HIV integration in particular, takes place in dsDNA, not in R-loops. Although HIV integration can occur in vitro on naked dsDNA, there is good evidence that, in an infected cell, integration occurs on DNA that is associated with nucleosomes. This review will be presented in two parts. First, a summary will be provided giving some of the reasons to be confident that integration occurs on dsDNA on nucleosomes. The second part will point out some of the obvious problems with the experimental data that are presented in the manuscript.

      We appreciate your comments. We have carefully addressed the concerns expressed as follows (your comments are in italics):  

      (2.1) 2017 Dos Passos Science paper describes the structure of the HIV intasome. The structure makes it clear that the target for integration is dsDNA, not an R-loop, and there are very good reasons to think that structure is physiologically relevant. For example, there is data from the Cherepanov, Engelman, and Lyumkis labs to show that the HIV intasome is quite similar in its overall structure and organization to the structures of the intasomes of other retroviruses. Importantly, these structures explain the way integration creates a small duplication of the host sequences at the integration site. How do the authors propose that an R-loop can replace the dsDNA that was seen in these intasome structures?

      We do appreciate the current understanding of the HIV-1 integration site selection mechanism and the known structure of the dsDNA-bound intasome. Our study proposes an R-loop as another contributor to HIV-1 integration site selection. Recent studies providing new perspectives on HIV-1 integration site targeting motivated our current work. For instance, Ajoge et al., 2022 (7) indicated that a guanine-quadruplex (G4) structure formed in the non-template DNA strand of the R-loop influences HIV-1 integration site targeting. Additionally, I. K. Jozwik et al., 2022 (8) showed retroviral integrase protein structure bound to B-to-A transition in target DNA. R-loop structures are a prevalent class of alternative non-B DNA structures (9). We acknowledge the current understanding of HIV-1 integration site selection and explore how R-loop interactions may contribute to this knowledge in the Discussion section of our manuscript. 

      Primarily based on our current data, we believe that R-loop structures are bound by HIV-1 integrase proteins and lead to HIV-1 viral genome integration into the vicinity regions of the host genomic R-loops, but we do not claim that R-loops completely replace dsDNA as the target for HIV-1 integration. An R-loop can be multi-kilobase in size and the R-loop peak length widely varies depending on the immunoprecipitation and library construction methods (1, 2), therefore, we could not validate whether the center of triple-stranded R-loops is the extraction site of HIV-1 integration where the strand transfer reaction by integrase occurs. Therefore, we replaced phrases such as, “HIV-1 targets R-loops for integration,” which may overstate our finding on the role of R-loop in HIV-1 integration site selection, with phrases, such as, “HIV-1 favors vicinity regions of R-loop for the viral genome integration,” in the revised manuscript. We apologize for the inconvenience caused by the unclear and non-specific details of our findings. Nevertheless, we believe that we validated R-loop-mediated HIV-1 integration in R-loop-forming regions using our pgR-poor and pgR-rich cell line models. We quantified site-specific integration events in the R-loop regions, and found that a greater number of integration events occurred in designated regions of the pgR-rich cellular genome upon R-loop induction by DOX treatment, but not in pgR-poor cells (Fig. 3E–G of the revised manuscript). 

      dsDNA may have been the sole target of the intasome demonstrated in vitro possibly because dsDNA has only been considered as a substrate for in vitro intasome assembly. We hope that our work will initiate and advance future investigations on target-bound intasome structures by considering R-loops as potential new targets for integrated proteins and intasomes.  

      (2.2) As noted above, concerted (two-ended) integration can occur in vitro on a naked dsDNA substrate. However, there is compelling evidence that, in cells, integration preferentially occurs on nucleosomes. Nucleosomes are not found in R loops. In an infected cell, the viral RNA genome of HIV is converted into DNA within the capsid/core which transits the nuclear pore before reverse transcription has been completed. Integration requires the uncoating of the capsid/core, which is linked to the completion of viral DNA synthesis in the nucleus. Two host factors are known to strongly influence integration site selection, CPSF6 and LEDGF. CPSF6 is involved in helping the capsid/core transit the nuclear pore and associate with nuclear speckles. LEDGF is involved in helping the preintegration complex (PIC) find an integration site after it has been released from the capsid/core, most commonly in the bodies of highly expressed genes. In the absence of an interaction of CPSF6 with the core, integration occurs primarily in the lamin-associated domains (LADs). Genes in LADs are usually not expressed or are expressed at low levels. Depending on the cell type, integration in the absence of CPSF6 can be less efficient than normal integration, but that could well be due to a lack of LEDGF (which is associated with expressed genes) in the LADs. In the absence of an interaction of IN with LEDGF (and in cells with low levels of HRP2) integration is less efficient and the obvious preference for integration in highly expressed genes is reduced. Importantly, LEDGF is known to bind histone marks, and will therefore be preferentially associated with nucleosomes, not R-loops. LEDGF fusions, in which the chromatin binding portion of the protein is replaced, can be used to redirect where HIV integrates, and that technique has been used to map the locations of proteins on chromatin. Importantly, LEDGF fusions in which the chromatin binding component of LEDGF is replaced with a module that recognizes specific histone marks direct integration to those marks, confirming integration occurs efficiently on nucleosomes in cells. It is worth noting that it is possible to redirect integration to portions of the host genome that are poorly expressed, which, when taken with the data on integration into LADs (integration in the absence of a CPSF6 interaction) shows that there are circumstances in which there is reasonably efficient integration of HIV DNA in portions of the genome in which there are few if any R-loops.

      Although R-loops may not wrap around nucleosomes, long and stable R-loops likely cover stretches of DNA corresponding to multiple nucleosomes (10). For example, R-loops are associated with high levels of histone marks, such as H3K36me3, which LEDGF recognizes (2, 11). R-loops dynamically regulate the chromatin architecture. Possibly by altering nucleosome occupancy, positioning, or turnover, R-loop structures relieve superhelical stress and are often associated with open chromatin marks and active enhancers (2, 10). These features are also distributed over HIV-1 integration sites (12). In the Discussion section of the revised manuscript, we explored the R-loop molding mechanisms in the host genomic environment for HIV-1 integration site selection and its potential collaborative role with LEDGF/p75 and CPSF6 governing HIV-1 integration site selection. 

      By carefully revising our original manuscript, with respect to the reviewer's comment, we recognized the need to tone down our statements. We found that the title, abstract, and discussion of our original manuscript includes phrases, such as, “HIV-1 targets Rloops for integration,” which may overstate our finding on the role of R-loop in HIV-1 integration site selection. We replaced these phrases. For example, we used phrases, such as “HIV-1 favors vicinity regions of R-loop for the viral genome integration,” in the revised manuscript. We apologize for the inconvenience caused by the unclear and non-specific details of our findings.

      (2.3) Given that HIV DNA is known to preferentially integrate into expressed genes and that R-loops must necessarily involve expressed RNA, it is not surprising that there is a correlation between HIV integration and regions of the genome to which R loops have been mapped. However, it is important to remember that correlation does not necessarily imply causation.

      We understand the reviewer's concern regarding the possibility of a coincidental correlation between the R-loop regions and HIV-1 integration sites, particularly when the interpretation of this correlation is primarily based on a global analysis. 

      Therefore, we designed pgR-poor and pgR-rich cell lines, which we believe are suitable models for distinguishing between integration events driven by transcription and the presence of R-loops. Although the two cell lines showed comparable levels of transcription at the designated region upon DOX treatment via TRE promoter activation (Fig. 3B of the revised manuscript), only pgR-rich cells formed R-loops at the designated regions (Fig. 3C of the revised manuscript). When infected with HIV1, pgR-rich cells, but not pgR-poor cells, showed higher infectivity after DOX treatment (Fig. 3D of the revised manuscript). Moreover, we quantified site-specific integration events in the R-loop regions, and found that a greater number of integration events occurred in designated regions of the pgR-rich cellular genome upon R-loop induction by DOX treatment, but not in pgR-poor cells (Fig. 3E of the revised manuscript). Therefore, we concluded that transcriptional activation without an R-loop (in pgR-poor cells) may not be sufficient to drive HIV-1 integration. We believe that our work provides strong evidence for a causative relationship between R-loop formation/Rloop sites and HIV-1 integration. We hope that our explanation addresses your concerns. Thank you.

      If we consider some of the problems in the experiments that are described in the manuscript:

      (2.4) In an infected individual, cells are almost always infected by a single virion and the infecting virion is not accompanied by large numbers of damaged or defective virions. This is a key consideration: the claim that infection by HIV affects R-loop formation in cells was done with a VSVg vector in experiments in which there appears to have been about 6000 virions per cell. Although most of the virions prepared in vitro are defective in some way, that does not mean that a large fraction of the defective virions cannot fuse with cells. In normal in vivo infections, HIV has evolved in ways that avoid signaling infected the cell of its presence. To cite an example, carrying out reverse transcription in the capsid/core prevents the host cell from detecting (free) viral DNA in the cytoplasm. The fact that the large effect on R-loop formation which the authors report still occurs in infections done in the absence of reverse transcription strengthens the probability that the effects are due to the massive amounts of virions present, and perhaps to the presence of VSVg, which is quite toxic. To have physiological relevance, the infections would need to be carried out with virions that contain HIV even under circumstances in which there is at most one virion per cell.

      Our virus production and in vitro and ex vivo HIV-1 infection experimental conditions, designed for infecting cell types, such as HeLa cells and primary CD4+ T cells with VSV-G pseudotyped HIV, were based on a comprehensive review of numerous references. At the very beginning of this study, we tested HIV-1-specific host genomic R-loop induction using empty virion particles (virus-like particles, VLP) or other types of viruses (non-retrovirus, SeV; retroviruses, FMLV and FIV), all produced with a VSV G protein donor. We could not include a control omitting the VSV G protein or using natural HIV-1 envelope protein to prevent viral spread in culture. We observed that despite all types of virus stocks being prepared using VSV-G, only cells infected with HIV-1 viruses showed R-loop signal enrichment (Author response image 3). Therefore, we omitted the control for the VSV G protein in subsequent analyses, such as DRIPcseq. We have also revised our manuscript to provide a clearer description of the experimental conditions. In particular, we now clearly stated that we used VSV-G pseudotyped HIV-1 in this study, throughout the abstract, results, and discussion sections of the revised manuscript. Thank you.

      Author response image 3.

      (A) Dot blot analysis of the R-loop in gDNA extracts from HIV-1 infected U2OS cells with MOI of 0.6 harvested at 6 hpi. The gDNA extracts were incubated with or without RNase H in vitro before membrane loading (anti-S9.6 signal). (B) Dot blot analysis of the R-loop in gDNA extracts from HeLa cells infected with 0.3 MOI of indicated viruses. The infected cells were harvested at 6 hpi. The gDNA extracts were incubated with or without RNase H in vitro before membrane loading (anti-S9.6 signal).

      HIV-1 co-infection may also be expected in cell-free HIV-1 infections. However, it was previously suggested that the average number of infection events varies within 1.02 to 1.65 based on a mathematical model that estimates the frequency of multiple infections with the same virus (Figure 4c of Ito et al., Sci. Rep, 2017; 6559) (13). 

      (2.5) Using the Sso7d version of HIV IN in the in vitro binding assays raises some questions, but that is not the real question/problem. The real problem is that the important question is not what/how HIV IN protein binds to, but where/how an intasome binds. An intasome is formed from a combination of IN bound to the ends of viral DNA. In the absence of viral DNA ends, IN does not have the same structure/organization as it has in an intasome. Moreover, HIV IN (even Sso7d, which was modified to improve its behavior) is notoriously sticky and hard to work with. If viral DNA had been included in the experiment, intasomes would need to be prepared and purified for a proper binding experiment. To make matters worse, there are multiple forms of multimeric HIV IN and it is not clear how many HIV INs are present in the PICs that actually carry out integration in an infected cell.

      As the reviewer has noted, HIV IN, even with Sso7d tagging, is difficult. We attempted the purification of viral DNA (vDNA)-bound PICs using either Sso7d-tagged HIV-1 integrase proteins or non-tagged HIV-1 integrase proteins (F185K/C280S), procured from the NIH HIV reagent program (HRP-20203), following the method described by Passos et al., Science, 2017; 355 (89-92) (3). Despite multiple attempts, we were unable to purify the vDNA-bound IN protein complexes for in vitro assays. However, through multiple biochemical experiments, we believe that we have successfully demonstrated the interaction between cellular R-loops and HIV-1 integrase proteins both in cells and in vitro (Fig. 5A–F of the revised manuscript). We also observed a close association between integrase proteins and host cellular Rloops in HIV-1-infected cells, using a fluorescent recombinant virus (HIV-IN-EGFP) with intact IN-EGFP PICs (Fig. 5G of the revised manuscript). 

      (2.6) As an extension of comment 2, the proper association of an HIV intasome/PIC with the host genome requires LEDGF and the appropriate nucleic acid targets need to be chromatinized.

      The interaction between cellular R-loops and HIV-1 integrase proteins in HeLa cells endogenously expressing LEDGF/p75 was examined using reciprocal immunoprecipitation assays in Fig. 5C–F, S6B, and S6D Fig. of the revised manuscript. In addition, as discussed in more detail in our response to comment [28], we observed a close association between host cellular R-loops and HIV-1 integrase proteins by PLA assay, in HIV-1-infected HeLa cells. 

      (2.7) Expressing any form of IN, by itself, in cells to look for what IN associates with is not a valid experiment. A major factor that helps to determine both where integration takes place and the sites chosen for integration is the transport of the viral DNA and IN into the nucleus in the capsid core. However, even if we ignore that important part of the problem, the IN that the authors expressed in HeLa cells won't be bound to the viral DNA ends (see comment 2), even if the fusion protein would be able to form an intasome. As such, the IN that is expressed free in cells will not form a proper intasome/PIC and cannot be expected to bind where/how an intasome/PIC would bind.

      As discussed in more detail in our response to comment [2-8], we believe that our PLA experiment using the pVpr-IN-EGFP virus, which has previously been examined for virion integrity, as well as the IN-EGFP PICs (14), demonstrated a close association between host cellular R-loops and HIV-1 integrase proteins in HIV-1-infected cells. 

      (2.8) As in comment 1, for the PLA experiments presented in Figure 5 to work, the number of virions used per cell (which differs from the MOI measured by the number of cells that express a viral marker) must have a high, which is likely to have affected the cells and the results of the experiment. However, there is the additional question of whether the IN-GFP fusion is functional. The fact that the functional intasome is a complex multimer suggests that this could be a problem. There is an additional problem, even if IN-GFP is fully functional. During a normal infection, the capsid core will have delivered copies of IN (and, in the experiments reported here, the IN-GFP fusion) into the nucleus that is not part of the intasome. These "free" copies of IN (here IN-GFP) are not likely to go to the same sites as an intasome, making this experiment problematic (comment 4).

      The HIV-IN-EGFP virus stock was produced by polyethylenimine-mediated transfection of HEK293T cells with 6 µg of pVpr-IN-EGFP, 6 µg of HIV-1 NL4-3 noninfectious molecular clone (pD64E; NIH AIDS Reagent Program 10180), and 1 µg of pVSV-G as previously described in (14), and described in the Materials and Methods section of our manuscript. The pVpr-IN-EGFP vector used to produce HIV-1-IN-EGFP virus stock was provided by Anna Cereseto group (Albanese et al., PLOS ONE, 2008; 6(6); Ref 34 of the revised manuscript). It was previously reported that the HIV-1INEGFP virions produced by IN-EGFP trans-incorporation through Vpr are intact and infective viral particles (Figure 1 of Albanese et al., PLOS ONE, 2008; 6(6)). Therefore, we believe that the HIV-IN-EGFP used in our PLA experiments was functional. 

      Additionally, Albanese et al. showed that the EGFP signal of HIV-IN-EGFP virions colocalizes with the viral protein matrix (p17MA) and capsid (P24CA) as well as with the newly synthesized cDNA produced by reverse transcriptase by labeling and visualizing the synthesized cDNA (14). In addition, the fluorescent recombinant virus (HIV-INEGFP) was structurally intact at the nuclear level (Figure 6 of Albanese et al., PLOS ONE, 2008; 6(6)). Therefore, we believe that our PLA experimental result is not likely misled as the reviewer concerns due to the integrity of the HIV-IN-EGFP virion as well as IN-EGFP PICs.

      Furthermore, the in vitro HIV-1 infection setting of our PLA experiments was carefully determined based on multiple studies that performed image-based assays on HIV-1infected cells. For instance, Albanese et al. infected 4 × 104 cells with viral loads equivalent to 1.5 or 3 µg of HIV-1 p24 for their immunofluorescence analysis, in their previous report (14). We titrated the fluorescent HIV-1 virus stocks by examining both the multiplicity of infection (MOI) and quantifying the HIV-1 p24 antigen content (Author response image 4). In our calculation, we infected 5 × 104 HeLa cells with viral loads equivalent to 1.3 ug of HIV-1 p24, which is indicated as 2 MOI in Fig. 5G of our manuscript, for our PLA experiments. 

      Image-Based Assays often require increased and enhanced signal for statistical robustness. For example, Achuthan et al. infected cells with VSV-G-pseudotyped HIV1 at the approximate MOI of 350 for vDNA and PIC visualization (15). Therefore, we believe our experimental condition for PLA experiments, which we carefully designed based on previous study that are frequently referred, are reasonable. We really hope that our discussion sufficiently addressed the reviewer’s concern. 

      Author response image 4.

      Gating strategy used to determine HIV-1-infectivity in HeLa cells at 48 hpi. Cells were infected with a known p24 antigen content in the stock of the VSV-G-pseudotyped HIV-1-EGFP-virus. The percentages of GFP-positive cell population are indicated.

      (2.9) In the Introduction, the authors state that the site of integration affects the probability that the resulting provirus will be expressed. Although this idea is widely believed in the field, the actual data supporting it are, at best, weak. See, for example, the data from the Bushman lab showing that the distribution of integration sites is the same in cells in which the integrated proviruses are, and are not, expressed. However, given what the authors claim in the introduction, they should be more careful in interpreting enzyme expression levels (luciferase) as a measure of integration efficiency in experiments in which they claim proviruses are integrated in different places.

      We thank the reviewer for the constructive comment. We have changed the statement in Lines 41–42 in the Introduction section of our original manuscript to “The chromosomal landscape of HIV-1 integration influences proviral gene expression, persistence of integrated proviruses, and prognosis of antiretroviral therapy.” (Lines 39-41 of the revised manuscript). We believe that this change can tone-down the relevance between the site of integration and the provirus expression level.

      The piggyBac transposase randomly insert the “cargo (transposon)” into TTAA chromosomal sites of the target genome, generating efficient insertions at different genomic loci (16, 17). We believe that this random insertion of the pgR-poor/rich vector mediated by the piggyBac system allows us not to mislead the R-loop-mediated HIV1 integration site because of the genome locus bias of the vector insertion. Therefore, Figure 3 in our manuscript does not claim any relevance between the site of integration and the resulting provirus expression levels. Instead, as noted in Line 214 of the revised manuscript, using the luciferase reporter HIV-1 virus, we attempted to examine HIV-1 infection in cells with an "extra number of R-loops” in the host cellular genome. We observed that pgR-rich cells showed higher luciferase activity upon DOX treatment than pgR-poor cells (Fig. 3D of the revised manuscript). We believe that this is because a greater number of HIV-1 integration events may occur in pgR-rich cells, where DOX-inducible de novo R-loop regions are introduced. This has been further examined in Fig. 3E–G of the revised manuscript. We hope this explanation clarifies the Figure 3. Thank you. 

      (2.10) Using restriction enzymes to create an integration site library introduces biases that derive from the uneven distribution of the recognition sites for the restriction enzymes.

      As described in the Materials and Methods section, we adopted a sequencing library construction method using a previously established protocol (18, 19). Although we recognize the advantages of DNA fragmentation by sonication, in in vitro or ex vivo HIV-1 infection settings, where the multiplicity of infection is carefully determined based on multiple references, more copies of integrated viral sequences are expected compared to that in samples from infected patients (18). Therefore, in these settings, restriction enzyme-based DNA fragmentation and ligation-mediated PCR sequencing are well-established methods that provide significant data sources for HIV-1 integration site sequencing (15, 20-22). Furthermore, our data showing the proportion of integration sites over R-loop regions (Fig. 4B of the revised manuscript) are presented alongside the respective random controls (i.e., proportion of integration sites within the 30-kb windows centered on randomized DRIPc-seq peaks, gray dotted lines; control comparisons between randomized integration sites with DRIPc-seq peaks, black dotted lines; and randomized integration sites with randomized DRIPcseq peaks, gray solid lines), which do not show such a correlation between the HIV-1 integration sites and nearby areas of the R-loop regions. Therefore, we believe that our results from the integration site sequencing data analysis are unlikely to be biased. 

      Reviewer #3 (Public Review):

      In this manuscript, Park and colleagues describe a series of experiments that investigate the role of R-loops in HIV-1 genome integration. The authors show that during HIV-1 infection, R-loops levels on the host genome accumulate. Using a synthetic R-loop prone gene construct, they show that HIV-1 integration sites target sites with high R-loop levels. They further show that integration sites on the endogenous host genome are correlated with sites prone to R-loops. Using biochemical approaches, as well as in vivo co-IP and proximity ligation experiments, the authors show that HIV-1 integrase physically interacts with R-loop structures.

      My primary concern with the paper is with the interpretations the authors make about their genome-wide analyses. I think that including some additional analyses of the genome-wide data, as well as some textual changes can help make these interpretations more congruent with what the data demonstrate. Here are a few specific comments and questions:

      We are grateful for the time and effort we spent on our behalf and the reviewer’s appreciation for the novelty of our work, in particular, R-loop induction by HIV-1 infection and the correlation between host R-loops and the genomic site of HIV-1 integration. In the following sections, we provide our responses to your comments and suggestions. Your comments are in italics. We have carefully addressed the following issues.

      (3.1) I think Figure 1 makes a good case for the conclusion that R-loops are more easily detected HIV-1 infected cells by multiple approaches (all using the S9.6 antibody). The authors show that their signals are RNase H sensitive, which is a critical control. For the DRIPc-Seq, I think including an analysis of biological replicates would greatly strengthen the manuscript. The authors state in the methods that the DRIPc pulldown experiments were done in biological replicates for each condition. Are the increases in DRIPc peaks similar across biological replicates? Are genomic locations of HIV-1-dependent peaks similar across biological replicates? Measuring and reporting the biological variation between replicate experiments is crucial for making conclusions about increases in R-loop peak frequency. This is partially alleviated by the locus-specific data in Figure S3A. However, a better understanding of how the genome-wide data varies across biological replicates will greatly enhance the quality of Figure 1.

      DRIPc-seq experiments were conducted with two biological replicates. To define consensus DRIPc-seq peaks using these two replicates, we used two methods applicable to ChIP-seq analysis: the irreproducible discovery rate (IDR) method and sequencing data pooling. We found that the sequencing data pooling method yielded significantly more DRIPc-seq peaks than consensus peak identification through IDR, and we decided to utilize R-loop peaks from pooled sequencing data for our downstream analyses, as described in the figure legends and Materials and Methods of the revised manuscript. 

      As noted by the reviewer, it is important to verify whether the increasing trend in the number of R-loop peaks and genomic locations of HIV-1 dependent R-loops were consistently observed across the two biological replicates. Therefore, we independently performed R-loop calling on each replicate of the sequencing data of primary CD4+ T cells from two individual donors to verify that the increase in R-loop numbers was consistent (Author response image 5). Additionally, the overlap of the R-loop peaks between the two replicates was statistically significant across the genome (Author response table 1). Thank you.

      Author response image 5.

      Bar graph indicating DRIPc-seq peak counts for HIV-1-infected primary CD4+ T cells harvested at the indicated hours post infection (hpi). Pre-immunoprecipitated samples were untreated (−) or treated (+) with RNase H, as indicated. Each dot corresponds to an individual data set from two biologically independent experiments.

      Author response table 1.

      DRIPc-seq peak length and Chi-square p-value in CD4+ T cells from individual donor 1 and 2 

      (3.2) I think that the conclusion that R-loops "accumulate" in infected cells is acceptable, given the data presented. However, in line 134 the authors state that "HIV1 infection induced host genomic R-loop formation". I suggest being very specific about the observation. Accumulation can happen by (a) inducing a higher frequency of the occurrence of individual R-loops and/or (b) stabilizing existing R-loops. I'm not convinced the authors present enough evidence to claim one over the other. It is altogether possible that HIV-1 infection stabilizes R-loops such that they are more persistent (perhaps by interactions with integrase?), and therefore more easily detected. I think rephrasing the conclusions to include this possibility would alleviate my concerns.

      We thank the reviewer for the considerable discussion on our manuscript. We have now changed Line 134 to, “HIV-1 infection induces host genomic R-loop enrichment” (Lines 132-133 of the revised manuscript), and added a new conclusion sentence implicating the possible explanation for the R-loop signal enrichment upon HIV-1 infection (Lines 133–135 of the revised manuscript), according to the reviewer's suggestion.    

      (3.3) A technical problem with using the S9.6 antibody for the detection of R-loops via microscopy is that it cross-reacts with double-stranded RNA. This has been addressed by the work of Chedin and colleagues (as well as others). It is absolutely essential to treat these samples with an RNA:RNA hybrid-specific RNase, which the authors did not include, as far as their methods section states. Therefore, it is difficult to interpret all of the immunofluorescence experiments that depend on S9.6 binding.

      We understand the reviewer's concern regarding the cross-reactivity of the S9.6 antibody with more abundant dsRNA, particularly in imaging applications. We carefully designed the experimental and analytical methods for R-loop detection using microscopy. For example, we pre-extracted the cytoplasmic fraction before staining with the S9.6 antibody and quantified the R-loop signal by subtracting the nucleolar signal. Both of these steps were taken to eliminate the possibility of misdetecting Rloops via microscopy because of the prominent cytoplasmic and nucleolar S9.6 signals, which primarily originate from ribosomal RNA. In addition, we included R-loop negative control samples in our microscopy analysis that were subjected to intensive RNase H treatment (60U/mL RNase H for 36 h) and observed a significant reduction in the S9.6 signal (Figure 1E of the revised manuscript). RNase H-treated samples served as essential and widely accepted negative controls for R-loop detection. 

      We would like to point out that recent studies have reported strong intrinsic specificity of S9.6 anybody for DNA:RNA hybrid duplex over dsDNA and dsRNA, along with the structural elucidations of S9.6 antibody recognition of hybrids (23, 24). Therefore, our interpretation of host cellular R-loop enrichment after HIV-1 infection using S9.6 antibodies in multiple biochemical approaches is well supported. Nevertheless, we agree with the reviewer's opinion that additional negative controls for the detection of R-loops via microscopy, such as RNase T1-and RNase III-treated samples, could improve the robustness and accuracy of R-loop imaging data (25).  

      (3.4) Given that there is no clear correlation between expression levels and R-loop peak detection, combined with the data that show increased detection of R-loop frequency in non-genic regions, I think it will be important to show that the R-loop forming regions are indeed transcribed above background levels. This will help alleviate possible concerns that there are technical errors in R-loop peak detection.

      Figures S5D and S5E in the revised manuscript show the relative gene expression levels of the R-loop-forming positive regions (P1-3) and the referenced Rloop-positive loci (RPL13A and CALM3). The gene expression levels of these R-loopforming regions were significantly higher than those of the ECFP or mAIRN genes without DOX treatment, which can be considered background levels of transcription in cells. Thank you. 

      (3.5) In Figures 4C and D the hashed lines are not defined. It is also interesting that the integration sites do not line up with R-loop peaks. This does not necessarily directly refute the conclusions (especially given the scale of the genomic region displayed), but should be addressed in the manuscript. Additionally, it would greatly improve Figure 4 to have some idea about the biological variation across replicates of the data presented 4A.

      We thank the reviewer for the considerable comment on our study. First of all, we added an annotation for the dashed lines in the figure legends of Figures 4C and 4D in the revised manuscript.

      We agree with the reviewer's interpretation of the relationship between the integration sites and R-loop peaks. Primarily based on our current data, we believe R-loop structures are bound by HIV-1 integrase proteins and lead HIV-1 viral genome integration into the “vicinity” regions of the host genomic R-loops. We displayed a large-scale genomic region (30-kb windows) to present integration sites surrounding R-loop centers because an R-loop can be multi-kilobase in size (1, 2). Depending on the immunoprecipitation and library construction methods, the R-loop peaks varied in size, and the peak length showed a wide distribution (Figure 3B of Malig et al., 2020, Figure 1B of Sanz et al., 2016, and Figure 2A of the revised manuscript). Therefore, presenting integration site events within a wide window of R-loop peaks could be more informative and better reflect the current understanding of R-loop biology.

      R-loop formation recruits diverse chromatin-binding protein factors, such as H3K4me1, p300, CTCF, RAD21, and ZNF143 (Figure 6A and 6B of Sanz et al., 2016) (26), which allow R-loops to exhibit enhancer and insulator chromatin states, which can act as distal regulatory elements (26, 27). We have demonstrated physical interactions between host cellular R-loops and HIV-1 integrase proteins (Figure 5 of the revised manuscript), therefore, we believe that this ‘distal regulatory element-like feature’ of the R-loop can be a potential explanation for how R-loops drive integration over longrange genomic regions.

      According to your suggestion, we added this explanation to the relevant literature in the Discussion section of the revised manuscript.

      Author response image 6 which represents the biological variation across replicates of the data shown in Figure 4A. The integration site sequencing data for Jurkat cells were adopted from SRR12322252 (4), which consists of the integration site sequencing data of HIV-1-infected wild type Jurkat cells with one biological replicate. We hope that our explanations and discussion have successfully addressed your concerns. Thank you. 

      Author response image 6.

      Bar graphs showing the quantified number of HIV-1 integration sites per Mb pair in total regions of 30-kb windows centered on DRIPc-seq peaks from HIV-1 infected HeLa cells and primary CD4+ T cells (magenta) or non-R-loop region in the cellular genome (gray). Each dot corresponds to an individual data set from two biologically independent experiments.

      (3.6) The authors do not adequately describe the Integrase mutant that they use in their biochemical experiments in Figure 5A. Could this impact the activity of the protein in such a way that interferes with the interpretation of the experiment? The mutant is not used in subsequent experiments for Figure 5 and so even though the data are consistent with each other (and the conclusion that Integrase interacts with R-loops) a more thorough explanation of why that mutant was used and how it impacts the biochemical activity of the protein will help the interpretation of the data presented in Figure 5.

      We appreciate the reviewer’s suggestions. In our EMSA analysis, we purified and used Sso7d-tagged HIV-1 integrase proteins with an active-site amino acid substitution, E152Q. First, we used the Sso7d-tagged HIV-1 integrase protein, as it has been suggested in previous studies that the fusion of small domains, such as Sso7d (DNA binding domain) can significantly improve the solubility of HIV integrase proteins without affecting their ability to assemble with substrate nucleic acids and their enzymatic activity (Figure 1B of Li et al., PLOS ONE, 2014;9 (8) (28, 29). We used an integrase protein with an active site amino acid substitution, E152Q, in our mobility shift assay, because the primary goal of this experiment was to examine the ability of the protein to bind or form a complex with different nucleic acid substrates. We thought that abolishing the enzymatic activity of the integrase protein, such as 3'-processing that cleaves DNA substrates, would be more appropriate for our experimental objective. This Sso7d tagged- HIV-1 integrase with the E152Q mutation has also been used to elucidate the structural model of the integrase complex with a nucleic acid substrate by cryo-EM (3) and has been shown to not disturb substrate binding.   Based on the reviewer’s comments, we have added a description of the E152Q mutant integrase protein in Lines 268–270 of the revised manuscript. Thank you.

      Reviewer #3 (Recommendations For The Authors):

      The paper suffers from many grammatical errors, which sometimes interfere with the interpretations of the experiments. In the view of this reviewer, the manuscript must be carefully revised prior to publication. For example, lines 247-248 "Intasomes consist of HIV-1 viral cDNA and HIV-1 coding protein, integrases." It is unclear from this sentence whether there are multiple integrases or multiple proteins that interact with the viral genome to facilitate integration. This makes the subsequent experiments in Figure 5 difficult to interpret. There are many other examples, too numerous to point out individually.

      We thoughtfully revised the original manuscript, making the best efforts to provide clearer details of our findings. We believe that we have made substantial changes to the manuscript, including Lines 247–248 of the original manuscript that the reviewer noted. Furthermore, the revised manuscript was edited by a professional editing service. Thank you.     (1) M. Malig, S. R. Hartono, J. M. Giafaglione, L. A. Sanz, F. Chedin, Ultra-deep Coverage Singlemolecule R-loop Footprinting Reveals Principles of R-loop Formation. J Mol Biol 432, 22712288 (2020).

      (2) L. A. Sanz et al., Prevalent, Dynamic, and Conserved R-Loop Structures Associate with Specific Epigenomic Signatures in Mammals. Mol Cell 63, 167-178 (2016).

      (3) D. O. Passos et al., Cryo-EM structures and atomic model of the HIV-1 strand transfer complex intasome. Science 355, 89-92 (2017).

      (4) W. Li et al., CPSF6-Dependent Targeting of Speckle-Associated Domains Distinguishes Primate from Nonprimate Lentiviral Integration. mBio 11,  (2020).

      (5) P. A. Ginno, Y. W. Lim, P. L. Lott, I. Korf, F. Chedin, GC skew at the 5' and 3' ends of human genes links R-loop formation to epigenetic regulation and transcription termination. Genome Res 23, 1590-1600 (2013).

      (6) S. Hamperl, M. J. Bocek, J. C. Saldivar, T. Swigut, K. A. Cimprich, Transcription-Replication Conflict Orientation Modulates R-Loop Levels and Activates Distinct DNA Damage Responses. Cell 170, 774-786 e719 (2017).

      (7) H. O. Ajoge et al., G-Quadruplex DNA and Other Non-Canonical B-Form DNA Motifs Influence Productive and Latent HIV-1 Integration and Reactivation Potential. Viruses 14,  (2022).

      (8) I. K. Jozwik et al., B-to-A transition in target DNA during retroviral integration. Nucleic Acids Res 50, 8898-8918 (2022).

      (9) F. Chedin, C. J. Benham, Emerging roles for R-loop structures in the management of topological stress. J Biol Chem 295, 4684-4695 (2020).

      (10) F. Chedin, Nascent Connections: R-Loops and Chromatin Patterning. Trends Genet 32, 828838 (2016).

      (11) P. B. Chen, H. V. Chen, D. Acharya, O. J. Rando, T. G. Fazzio, R loops regulate promoterproximal chromatin architecture and cellular differentiation. Nat Struct Mol Biol 22, 9991007 (2015).

      (12) A. R. Schroder et al., HIV-1 integration in the human genome favors active genes and local hotspots. Cell 110, 521-529 (2002).

      (13) Y. Ito et al., Number of infection events per cell during HIV-1 cell-free infection. Sci Rep 7, 6559 (2017).

      (14) A. Albanese, D. Arosio, M. Terreni, A. Cereseto, HIV-1 pre-integration complexes selectively target decondensed chromatin in the nuclear periphery. PLoS One 3, e2413 (2008).

      (15) V. Achuthan et al., Capsid-CPSF6 Interaction Licenses Nuclear HIV-1 Trafficking to Sites of Viral DNA Integration. Cell Host Microbe 24, 392-404 e398 (2018).

      (16) X. Li et al., piggyBac transposase tools for genome engineering. Proc Natl Acad Sci U S A 110, E2279-2287 (2013).

      (17) Y. Cao et al., Identification of piggyBac-mediated insertions in Plasmodium berghei by next generation sequencing. Malar J 12, 287 (2013).

      (18) E. Serrao, P. Cherepanov, A. N. Engelman, Amplification, Next-generation Sequencing, and Genomic DNA Mapping of Retroviral Integration Sites. J Vis Exp,  (2016).

      (19) K. A. Matreyek et al., Host and viral determinants for MxB restriction of HIV-1 infection. Retrovirology 11, 90 (2014).

      (20) G. A. Sowd et al., A critical role for alternative polyadenylation factor CPSF6 in targeting HIV-1 integration to transcriptionally active chromatin. Proc Natl Acad Sci U S A 113, E10541063 (2016).

      (21) B. Lucic et al., Spatially clustered loci with multiple enhancers are frequent targets of HIV-1 integration. Nat Commun 10, 4059 (2019).

      (22) P. K. Singh, G. J. Bedwell, A. N. Engelman, Spatial and Genomic Correlates of HIV-1 Integration Site Targeting. Cells 11,  (2022).

      (23) C. Bou-Nader, A. Bothra, D. N. Garboczi, S. H. Leppla, J. Zhang, Structural basis of R-loop recognition by the S9.6 monoclonal antibody. Nat Commun 13, 1641 (2022).

      (24) Q. Li et al., Cryo-EM structure of R-loop monoclonal antibody S9.6 in recognizing RNA:DNA hybrids. J Genet Genomics 49, 677-680 (2022).

      (25) J. A. Smolka, L. A. Sanz, S. R. Hartono, F. Chedin, Recognition of RNA by the S9.6 antibody creates pervasive artifacts when imaging RNA:DNA hybrids. J Cell Biol 220,  (2021).

      (26) L. A. Sanz, F. Chedin, High-resolution, strand-specific R-loop mapping via S9.6-based DNARNA immunoprecipitation and high-throughput sequencing. Nat Protoc 14, 1734-1755 (2019).

      (27) M. Merkenschlager, D. T. Odom, CTCF and cohesin: linking gene regulatory elements with their targets. Cell 152, 1285-1297 (2013).

      (28) M. Li, K. A. Jurado, S. Lin, A. Engelman, R. Craigie, Engineered hyperactive integrase for concerted HIV-1 DNA integration. PLoS One 9, e105078 (2014).

      (29) M. Li et al., A Peptide Derived from Lens Epithelium-Derived Growth Factor Stimulates HIV1 DNA Integration and Facilitates Intasome Structural Studies. J Mol Biol 432, 2055-2066 (2020).

    1. Author Response

      The following is the authors’ response to the original reviews.

      General remarks for the Editor and the Reviewers

      We would like to thank the Editor and the Reviewers for their feedback. Below we address their comments and present our point-by-point responses as well as the related changes in the manuscript.

      In addition to these changes, in a few cases we have found it necessary to move some texts and provide some additional explanations within the manuscript. We emphasize that these amendments have been made for only technical reasons, and do not alter the results and conclusions of the paper, but may help to render the text more coherent and understandable to readers with little knowledge of the subject.

      These minor corrections are:

      • We extended the Introduction section by a sentence (lines 40-42) that is intended to fit the proposed template directed, non-enzymatic replication mechanism into a more general prebiotic evolutionary context, thus emphasizing its biological relevance. This sentence includes an additional reference (Rosenberger et al., 2021).

      • Two very methodologically oriented and repeated descriptions of random sequence generation have been moved to the Methods section (lines 178-185) from the Results section (lines 336-339 and lines 351-354).

      • We complemented the Data availability statement with licensing information (lines 684-685).

      • Further minor changes (also indicated by red texts) have been implemented to remedy logical and grammatical glitches.

      Public Reviews:

      Reviewer #1 (Public Review):

      Summary:

      Szathmary and colleagues explore the parabolic growth regime of replicator evolution. Parabolic growth occurs when nucleic acid strain separation is the rate-limiting step of the replication process which would have been the case for non-enzymatic replication of short oligonucleotide that could precede the emergence of ribozyme polymerases and helicases. The key result is that parabolic replication is conducive to the maintenance of genetic diversity, that is, the coexistence of numerous master sequences (the Gause principle does not apply). Another important finding is that there is no error threshold for parabolic replication except for the extreme case of zero fidelity.

      Strengths:

      I find both the analytic and the numerical results to be quite convincing and well-described. The results of this work are potentially important because they reveal aspects of a realistic evolutionary scenario for the origin of replicators.

      Weaknesses:

      There are no obvious technical weaknesses. It can be argued that the results represent an incremental advance because many aspects of parabolic replication have been explored previously (the relevant publications are properly cited). Obviously, the work is purely theoretical, experimental study of parabolic replication is due. In the opinion of this reviewer, though, these are understandable limitations that do not actually detract from the value of this work.

      We are grateful that this Reviewer appreciates our work. We completely agree that the ultimate validation must come from experiments. It is important to stress that in this field theory often preceded experimental work by decades, and the former often guided the latter. We hope that for the topic of the present paper experiments will follow considerably faster.

      Reviewer #2 (Public Review):

      Summary:

      A dominant hypothesis concerning the origin of life is that, before the appearance of the first enzymes, RNA replicated non-enzymatically by templating. However, this replication was probably not very efficient, due to the propensity of single strands to bind to each other, thus inhibiting template replication. This phenomenon, known as product inhibition, has been shown to lead to parabolic growth instead of exponential growth. Previous works have shown that this situation limits competition between alternative replicators and therefore promotes RNA population diversity. The present work examines this scenario in a model of RNA replication, taking into account finite population size, mutations, and differences in GC content. The main results are (1) confirmation that parabolic growth promotes diversity, but that when the population size is small enough, sequences least efficient at replicating may nevertheless go extinct; (2) the observation that fitness is not only controlled by the replicability of sequences, but also by their GC content; (3) the observation that parabolic growth attenuates the impact of mutations and, in particular, that the error threshold to which exponentially growing sequences are subject can be exceeded, enabling sequence identity to be maintained at higher mutation rates.

      Strengths:

      The analyses are sound and the observations are intriguing. Indeed, it has been noted previously that parabolic growth promotes coexistence, its role in mitigating the error threshold catastrophe - which is often presented as a major obstacle to our understanding of the origin of life - had not been examined before.

      Weaknesses:

      Although all the conclusions are interesting, most are not very surprising for people familiar with the literature. As the authors point out, parabolic growth is well known to promote diversity (SzathmaryGladkih 89) and it has also been noted previously that a form of Darwinian selection can be found at small population sizes (Davis 2000).

      Given that under parabolic growth, no sequence is ever excluded for infinite populations, it is also not surprising to find that mutations have a less dramatic exclusionary impact.

      In the two articles cited (Szathmary-Gladkih 1989 and Davis 2000) the subexponentiality of the system was implemented in a mechanistic way, by introducing the exponent 0 < 𝑝 < 1. Although the behaviour of these models is more or less consistent with experimental findings (von Kiedrowski, 1986; Zielinski and Orgel, 1987), the divergence of per capita growth rates (𝑥̇/𝑥) at very low concentrations–which guarantees the ability to maintain unlimited diversity in the case of infinite population sizes–makes this formal approach partly unrealistic.

      To avoid the possible artefacts of this mechanistic approach, and as there are no previous studies analysing the diversity maintaining ability of finite populations of parabolic replicators in an individual-based model context, we implemented a simplified template replication mechanism leading to parabolic growth and analysed the dynamics in an individual-based stochastic model context. The key point of our investigation is that considerable diversity can be maintained in the system even when the population size is quite small.

      Regarding the Reviewer’s comment on selection: Darwinian selection can only occur in a simple subexponential dynamics if the ratio of replicabilities diverges, cf. Eq. (8) and the preceding paragraph in Davis, 2000.

      Our results also show (Figs. 4B and 4C) that high mutation rates and the error threshold problem can still be considered as a major limiting factor for parabolically replicating systems in terms of their diversity-maintaining ability. In the light of the above, potential mechanisms to relax the error threshold in such systems, one of which is demonstrated in the present study, seem to be important steps to account for the sequence diversification and increase in molecular complexity during the early evolution of RNA replicators.

      A general weakness is the presentation of models and parameters, whose choices often appear arbitrary. Modeling choices that would deserve to be further discussed include the association of the monomers with the strands and the ensuing polymerization, which are combined into a single association/polymerization reaction (see also below), or the choice to restrict to oligomers of length L = 10. Other models, similar to the one employed here, have been proposed that do not make these assumptions, e.g. Rosenberger et al. Self-Assembly of Informational Polymers by Templated Ligation, PRX 2021. To understand how such assumptions affect the results, it would be helpful to present the model from the perspective of existing models.

      The assumption of one-step polymerization reactions that we used here is a common technique for modelling template replication of sequence-represented replicators [see, e.g., Fontana and Schuster, 1998 (10.1126/science.280.5368.1451), Könnyű et al., 2008 (10.1186/1471-2148-8267), Vig-Milkovics et al, 2019 (10.1016/j.jtbi.2018.11.020) or Szilágyi et al., 2020 (10.1371/journal.pgen.1009155)]. This is because assuming base-to-base polymerisation of the copy would lead to a very large number of different types of intermediates, which a Gillespietype stochastic simulation algorithm could not handle in reasonable computation times, even if the sequences were relatively short. For comparison, in our model, where polymerization is one-step, the characteristic time of a simulation for 𝐿 = 10, 𝑁 = 105 and 𝛿 = 0.01 was 552 hours.

      Note that in Rosenberg et al. (PRX 2021), in contrast to a pioneering work [Fernando et al, 2007 (10.1007/s00239-006-0218-4)], sequences of replicators are not represented, which makes this approach completely inapplicable to our case, in which sequence defines the fitness. In sum, we suggest that this valid criticism points to possible future work.

      The values of the (many) parameters, often very specific, also very often lack justifications. For example, why is the "predefined error factor" ε = 0.2 and not lower or higher? How would that affect the results?

      A general remark. For the more important parameters , several values were used to test the behaviour of the model (see Table 1), but due to the considerable number of parameters, it is impossible to examine all possible combinations. 𝑐+ = 1 fixes the timescale, 𝐿 is set to 10 to obtain reasonable running times (see above).

      𝜀 characterizes how replicability decreases as the number of mutations increases. In the manuscript we used the following default vector: 𝜀 = (0.05, 0.2, 1) in which the third element corresponds to the mutation-free sequence, so it must to be 1. The first element determines the baseline replicability (see Methods), which we preferred not to change because it would fundamentally alter the ratio of replication propensities to association and dissociation propensities (as the substantial amount of complementary sequences of the master sequences are of baseline replicability) and thus would alter the reaction kinetics to an extent that it is not comparable with the original results. Therefore, only the second element can be adjusted. Accordingly, we have analysed the behaviour of the model in the cases of a steeper and a more gradual loss of replicability using the following two vectors, respectively: 𝜀, = (0.05, 𝟎. 𝟎𝟓, 1) and 𝜀,, = (0.05, 𝟎. 𝟓, 1). The choice of 𝜀, is chemically more plausible, since for very short oligomers the loss of chemical activity and replicability as a function of the number of mutations can be very sharp. We performed a series of simulations with all possible combinations of 𝛿 = 0.001, 0.005, 0.1 and 𝑁 = 103, 104, 105 for 𝜀′ and 𝜀,,in the constant population and chemostat model context (36 different runs). For other parameters, we took the default values, see Table 1. These values also correspond to the parameters we used in Figures 2 and 6. The results show that the steeper loss of replicability (𝜀,) slightly increases the diversity maintaining ability of the system, whereas the more gradual loss of replicability (𝜀,,) moderately decreases the diversity-maintaining ability of the system, and that these shifts are more pronounced in the constant population size model (Author response image 1) than in the chemostat model (Author response image 2). Altogether, these results confirm that the qualitative outcome of the model is robust in a wide range of loss of replicability (𝜀 vector) values.

      Author response image 1.

      Replicator coexistence in the constant population model with different loss of replicability (𝜀 vector) values. Within a given combination of 𝛿 and 𝑁 parameter values, the upper panel corresponds to the steeper loss of replicability (𝜀!), the middle panel to the default 𝜀 vector (Figure 2A), and the bottom panel to the more gradual loss of replicability vector (𝜀!!). Within each 𝛿; 𝑁 parameter combination, the same master sequence set was used with the three different 𝜀 vectors for comparability.

      Author response image 2.

      Replicator coexistence in the chemostat model with different loss of replicability (𝜀 vector) values. Within a given combination of 𝛿 and 𝑁 parameter values, the upper panel corresponds to the steeper loss of replicability (𝜀!), the middle panel to the default 𝜀 vector (Figure 6A), and the bottom panel to the more gradual loss of replicability vector (𝜀!!). Within each 𝛿; 𝑁 parameter combination, the same master sequence set was used with the three different 𝜀 vectors for comparability.

      Similarly, in equation (11), where does the factor 0.8 come from?

      This factor scales the decay rate of duplex sequences (𝑐"!") as the function of the binding energy

      (𝐸b). The value of 0.8 is an arbitrary choice, the value should be in the interval (0,1) and is only relevant in the chemostat model. It is expected to have a similar effect on the dynamics as the duplex decay factor parameter 𝑓, which we have investigated in a wide range of different values (cf. Table 1, Fig. 6), although 𝑓 is independent of the binding energy (𝐸/): increasing/decreasing the 0.8 factor is expected to decrease/increase the average total population size. We have investigated the diversity maintaining ability of the system at smaller (0.6) and larger (0.9) parameter values at different population sizes (𝑁 ≈ 103, 104 and 105) and at different replicability distances (δ = 0.001, 0.005 and 0.01) as shown in Fig. 6. We have found that the number of coexisting master types changes very little in response to changes in this factor. Only two shifts could be detected (underlined): factor 0.9 combined with 𝑁 ≈ 104 and 𝛿 = 0.001 caused the number of surviving master types to decrease by one, while factor 0.9 combined with 𝑁 ≈ 103 and 𝛿 = 0.01 caused the number of surviving master types to increase by one (Author response table 1). Factor 0.6 produced the same number of surviving types as the default (Author response table 1). In summary, the model shows marked robustness to changes in the values of this parameter.

      Author response table 1.

      Number of coexisting master types in the chemostat model with different binding energy dependent duplex decay rates. Within each 𝛿; 𝑁 parameter combination, the same master sequence set was used with the three different factor values: 0.6, 0.8 (the original) and 0.9 for comparability.

      Why is the kinetic constant for duplex decay reaction 1.15e10−8?

      Note that this value is the minimum of the duplex decay rate, Table 1 correctly shows the interval of this kinetic constant as: [1.15 ⋅ 10-8, 6.4 ⋅ 10-5]. Both values are derived from the basic parameters of the system and can be computed according to Eq. (11). The minimum: as the parameter set corresponding to this value is: . The maximum: with .

      Are those values related to experiments, or are they chosen because specific behaviors can happen only then?

      See above.

      The choice of the model and parameters potentially impact the two main results, the attenuation of the error threshold and the role of GC content:

      Regarding the error threshold, it is also noted (lines 379-385) that it disappears when back mutations are taken into account. This suggests that overcoming the error threshold might not be as difficult as suggested, and can be achieved in several ways, which calls into question the importance of the particular role of parabolic growth. Besides, when the concentration of replicators is low, product inhibition may be negligible, such that a "parabolic replicator" is effectively growing exponentially and an error catastrophe may occur. Do the authors think that this consideration could affect their conclusion? Can simulations be performed?

      The assumption of back mutation only provides a theoretical solution to the error threshold problem: back mutation guarantees a positive (non-zero) concentration of a master type, but, since the probability of back mutation is generally very low, this equilibrium concentration may be extremely low, or negligible for typical system sizes. Consequently, back mutation alone does not solve the problem of the error catastrophe: in our system back mutation is present (the probability that a sequence with 𝑘 errors mutates back to a master sequence is 𝜇k(1−𝜇)L-k), and the diversity-maintaining ability is limited. The effect of back mutation decreases exponentially with increasing sequence length.

      Regarding the role of the GC content, GC-rich oligomers are found to perform the worst but no rationale is provided.

      For GC-rich oligonucleotides the dissociation probability of a template-copy complex is relatively low (cf. Eqs. (9, 10)), thus they have a relatively low number of offspring, cf. lines 557-561: “a relatively high dissociation probability and the consequential higher propensity of being in a simple stranded form provides an advantage for sequences with relatively low GC content in terms of their replication affinity, that is, the expected number of offspring in case of such variants will be relatively high.”. Note that the simulation results shown in Fig. 3A, demonstrate the realization of this effect with prepared sequences (along a GC content gradient).

      One may assume that it happens because GC-rich sequences are comparatively longer to release the product. However, it is also conceivable that higher GC content may help in the polymerization of the monomers as the monomers attach longer on the template (as described in Eq. (9)). This is an instance where the choice to pull into a single step the association and polymerization reactions are pulled into a single step independent of GC content may be critical.

      It would be important to show that the result arises from the actual physics and not from this modeling choice.

      Some more specific points that would deserve to be addressed:

      • Line 53: it is said that p "reflects how easily the template-reaction product complex dissociates". This statement is not correct. A reaction order p<1 reflects product inhibition, the propensity of templates to bind to each other, not slow product release. Product release can be limiting, yet a reaction order of 1 can be achieved if substrate concentrations are sufficiently high relative to oligomer concentrations (von Kiedrowski et al., 1991).

      We think the key reference is Von Kiedrowski (1993) in this case. Other things being equal, his Table 1 on p. 134 shows that a sufficient increase in 𝐾4, i.e., the stability of the duplex (template and copy) (association rate divided by dissociation rate) throws the system into the parabolic regime. This is what we had in mind. In order to clarify this, we modified the quoted sentence thus: “In this kinetics, the growth order is equal or close to 0.5 (i.e., the dynamics is sub-exponential) because increased stability of the template-copy complex (rate of association divided by dissociation) promotes parabolic growth (von Kiedrowski et al., 1991; von Kiedrowski & Szathmáry, 2001).”

      • Population size is a key parameter, and a comparison is made between small (10^3) and large (10^5) populations, but without explaining what determines the scale (small/large relative to what?).

      The “small” value (103) corresponds to the smallest meaningful population size, significantly smaller population sizes (e.g. 102) cannot maintain the 10 master types (or any subset of them) and are chemically unrealistic. The “large value” (105) is the largest population size for which simulation times are still acceptable, in the case of 106 the runtimes are in the order of months.

      • In the same vein, we might expect size not to be the only important parameter, but also concentration.

      With constant volume population size and concentration are strictly coupled.

      • Lines 543-546: if understanding correctly, the quantitative result is that the error threshold rises from 0.1 in the exponential case to 0.196 in the parabolic. Are the authors suggesting that a factor of 2 is a significant difference?

      In this paragraph we compared the empirical error threshold of our system (which is close to 𝑝"#$ = 0.15) with the error threshold of the well-known single peak fitness landscape (which can be approximated by ) as a reference case. To make the message even clearer we have extended the last sentence (lines 596-597) as follows: “but note that applying this approach to our system is a serious oversimplification”. The 0.196 is simply the probability of error-free replication of a sequence when , but we have removed this sentence (“corresponding to the replication accuracy of a master sequence”) from the manuscript as it seems to be confusing.

      • Figure 3C: this figure shows no statistically significant effect?

      Thank you for pointing out this. We statistically tested the hypothesis that the GC content between the survived and the extinct master subsets are different. This analysis revealed that the differences between these two groups are statistically significant, which we now included in the manuscript at lines 380-390: “A direct investigation of whether the sequence composition of the master types is associated with their survival outcome was conducted using the data from the constant population model simulation results (Figure 2). In these data, the average GC content was measured to be lower in the surviving master subpopulations than in the extinct subpopulations (Figure 3C). To determine whether this difference was statistically significant, nonparametric, two-sample Wilcoxon rank-sum tests (Hollander & Wolfe, 1999) were performed on the GC content of the extinct-surviving master subsets. The GC content was significantly different between these two groups in all nine investigated parameter combinations of population size (N) and replicability distance (δ) at p<0.05 level, indicating a selective advantage for a lower GC content in the constant population model context. The exact p values obtained from this analysis are shown in Figure 3C.”

      • line 542: "phase transition-like species extension (Figure 4B)": such a clear threshold is not apparent.

      Thank you for pointing out the incorrect phrasing. As there is no clear threshold in the number of coexisting types as a function of the mutation rate, we removed the “phase transition-like” expression: “However, when finite population sizes and stochastic effects are taken into account, at the largest investigated per-base mutation rate (𝑝mut = 0.15), the summed relative steady-state master frequencies approach zero (Figure 4C) with accelerating species extinction (Figure 4B), indicating that this value is close to the system׳s empirical error threshold.” (lines 589-594).

      Recommendations for the authors:

      Reviewer #1 (Recommendations For The Authors):

      On the whole, the work is well done and presented, there are no major recommendations. It seems a good idea to cite and briefly discuss this recent paper: https://pubmed.ncbi.nlm.nih.gov/36996101/ which develops a symbiotic scenario of the coevolution of primordial replicators and reproducers that appears to be fully compatible with the results of the current work.

      Thank you for bringing this article to our attention. We have inserted the following sentence at lines 621-624: “The demonstrated diversity-maintaining mechanism of finite parabolic populations can be used as a plug-in model to investigate the coevolution of naked and encapsulated molecular replicators (e.g., Babajanyan et al., 2023).”

      The manuscript is well written, but there are some minor glitches that merit attention. For example:

      l. 5 "carriers presents a problem, because product formation and mutual hybridization" - "mutual" is superfluous here, delete

      l. 13 "amplification. In addition, sequence effects (GC content) and the strength of resource" - hardly "effects" - should be 'features' or 'properties'

      l. 41 "If enzyme-free replication of oligomer modules with a high degree of sequence" - "modules" here is only confusing - simply, "oligomers"

      l. 44 "under ecological competition conditions with which distinct replicator types with different" - delete "with" etc, there are many such minor glitches that are best corrected.

      Thank you for pointing out, we have corrected! Other drafting errors, glitches, superfluous sentences have also been corrected.

      Reviewer #2 (Recommendations For The Authors):

      None

      Editor (Recommendations For The Authors):

      In the manuscript, it appears that coexistence is assessed at a given point in time, while figures seem to show that it remains time-dependent. It would be great if the authors could clarify this and/or discuss this.

      We appreciate you bringing this to our attention, as we have indeed missed to elaborate on this important point. The steady state characteristic of the coexistence is assessed in our model in the following way: the relative frequency of each master sequence is tested for the condition of ≥ 100- (cut-off relative frequency for survival) in every 2,000th replication step in the interval between 10,000 replication steps before termination and actual termination (10= replication steps). If the above condition is true more than once, we consider the master type in question as survived (we have included this explanation in the Methods section: lines 258-268). Although this relatively narrow time interval can still be regarded as a snapshot of the state of the system, according to our numerical experiences, the resulting measure is a reliable quantitative indicator of the apparent stability of species coexistence in the parabolic dynamics.

    1. Author response:

      The following is the authors’ response to the original reviews.

      Public Reviews:

      Reviewer 1:

      (1) General comment: The evidence for these highly novel, potentially interesting roles (of the exocyst) would need to be more compelling to support direct involvement.

      We wish to thank the reviewer for his/her comments, and for considering that the proposed functions are highly novel and potentially interesting. To strengthen the evidence supporting the new roles of the exocyst, we have performed a number of additional experiments that are depicted in novel figures or figure panels of the new version of the manuscript. Particularly, we aimed at providing further support of the direct involvement of the exocyst in different steps of the regulated secretory pathway. Please see the details below.

      (2) For instance, the localization of exocyst to Golgi or to granule-granule contact sites does not seem substantial.

      We have performed quantitative colocalization studies, as suggested by the reviewer to further substantiate our initial findings. We have carefully analysed GFP-Sec15 distribution in relation to the Golgi complex and secretory Glue granules at relevant time points of salivary gland development. Overall, we found that GFP-Sec15 distribution is dynamic during salivary gland development. Before Glue synthesis (72 h AEL), Sec15 was observed in close association (defined as a distance equal to, or less than 0.6 µm) with the Golgi complex (please see below Author response image 1). This association was lost once Glue granules have begun to form (96 h AEL). Importantly, we do not see relevant association between GFP-Sec15 and the ER (please see Author response image 2). These observations support our conclusion that the exocyst plays a role at the Golgi complex. New images supporting these conclusions, as well as quantitative data, have been included in Figure 5 of the new version of the manuscript. In addition, real time imaging, as well as 3D reconstruction analyses, confirming the close association between Sec15 and Golgi cisternae are now included in the manuscript. Please see Supplementary Videos 1-3. These new data are described in the text lines 200-210 of the Results section and text lines 359368 of the Discussion section.

      Interestingly, at the time when Sec15-Golgi association is lost (96 h AEL), Sec15 foci associate instead with newly formed secretory granules (< 1µm diameter). This association persists during secretory granule maturation (100-116 h AEL), when Sec15 foci localize specifically in between neighbouring, immature secretory granules. When maturation has ended and Glue granule exocytosis begins (116-120 h AEL), this localization between granules is lost. These observations are consistent with a role of the exocyst in homotypic fusion during SG maturation. We have included new images showing that association between Sec15 and secretory granules is dynamic and depends on the developmental stage. We have quantified this association both during maturation and at a stage when SGs are already mature. We have in addition performed a 3D reconstruction analysis of these images to confirm the close association between Sec15 and immature SGs. These new data are now depicted in Figure 7BC, Supplementary Videos 4-5, and described in text lines 216-221 of the Results section. In addition, a lower magnification image is provided below in this letter (Author response image 3), quantifying the proportion of Sec15 foci localized in between SGs (yellow arrows) relative to the total number of Sec15 foci (yellow arrows + green arrowheads).

      Author response image 1.

      Criteria utilized to define Sec15 focithat were“associated” or“not associated” withthe trans-Golgi network in the experiments of Figure 5C-E of the manuscript.When the distance between maximal intensities of GFP-Sec15 and Golgi-RFP signals was equal or less than 0.6 m, the signals were considered “associated” (upper panels). When the distance was more than 0.6 m, the signals were considered “not associated” (lower panels).

      Author response image 2.

      Criteria utilized to define Sec15 focithat were“associated” or“not associated” withthe ERin the experiments of Figure 5A-Bof the manuscript.When the distance between maximal intensities of GFP-Sec15 and KDEL-RFP signals was equal or less than 0.6 m, the signals were considered “associated”. When the distance was more than 0.6 m, the signals were considered “not associated”.

      Author response image 3.

      (A) GFP-Sec15 foci (cyan) and SGs (red) are shown in cells bearing Immature SGs or (B) with mature SGs. Yellow arrows indicate GFP-Sec15 foci localized in between SGs; green arrowheads indicate GFP-Sec15 foci that arenot in between SGs. (C) Quantification of the percentage (%) of Sec15 foci localized in between SGs respect to the total number of Sec15 foci in cells filled with immature SGs (ISG)vs cells with mature SGs (MSG).

      It is interesting to mention that previous evidence from mammalian cultured cells (Yeaman et al,  2001) show that the exocyst localizes both at the trans-Golgi network and at the plasma membrane, weighing in favour of our claim that the exocyst is required at various steps of the exocytic pathway. Thus, the exocyst may play multiple roles in the secretion pathway in other biological models as well. This concept has now been included at the Discussion section of the revised version of the manuscript (lines 359-368).

      To make the conclusions of our work clearer, in the revised version of the manuscript, we have now included a graphical abstract, summarizing the dynamic localization of the exocyst in relation to the processes of SG biogenesis, maturation and exocytosis reported in our work. 

      (3) Instead, it is possible that defects in Golgi traffic and granule homotypic fusion are not due to direct involvement of the exocyst in these processes, but secondary to a defect in canonical exocyst roles at the plasma membrane. A block in the last step of glue exocytosis could perhaps propagate backward in the secretory pathway to disrupt Golgi complexes or cause poor cellular health due to loss of cell polarity or autophagy.

      We thank the reviewer for these thoughtful comments. We have performed a number of additional experiments to assess “cellular health” or to identify possible defects in cell polarity after knock-down of exocyst subunits. These new data have been included in new supplementary figures 5 and 6 of the revised version of the manuscript (please see below). 

      In our view, the precise localization of GFP-Sec15 at the Golgi complex (Figure 5C-E), as well as in between immature secretory granules (Figure 7B-D), argues in favour of a direct involvement of the exocyst in SG biogenesis and homofusion respectively. 

      We truly appreciate the comment of the reviewer raising the possibility that the defects that we observe at early steps of the pathway (SG biogenesis and SG maturation) may actually stem from a backward effect of the role of the exocyst in SG-plasma membrane tethering. We wish to respectfully point out that the processes of biogenesis, maturation and plasma membrane tethering/fusion of SGs do not occur simultaneously in the Drosophila larval salivary gland in vivo, as they do in other secretory model systems (i.e. cell culture). In this regard, the experimental model is unique in terms of synchronization. In each cell of the salivary gland, the three processes (biogenesis, maturation and exocytosis) occur sequentially, and controlled by developmental cues. At the developmental stage when SGs fuse with the plasma membrane, SG biogenesis has already ceased many hours earlier: SG biogenesis occurs at 96-100 hours after egg lay (AEL), SG maturation takes place at 100-112 hours AEL, and SG-plasma membrane fusion happens only when all SGs have undergone maturation and are ready to fuse with the plasma membrane at 116-120 h AEL. Thus, in our view it is not conceivable that a defect in SG-plasma membrane tethering/fusion (116-120 h AEL) may affect backwards the processes of SG biogenesis or SG maturation, which have occurred earlier in development (96-112 h AEL).

      As suggested by the reviewer, we have analysed several markers of cellular health and cell polarity, comparing conditions of exocyst subunit silencing (exo70RNAi, sec3RNAi or exo84RNAi) with wild type controls (whiteRNAi). These new data are depicted in Supplementary Figures 5 and 6, and described in lines 172-179 of the Results section of the revised version of the manuscript. Noteworthy, for these experiments we have applied silencing conditions that block secretory granule maturation, bringing about mostly immature SGs. Our analyses included: 1) Subcellular distribution of PI(4,5)P2, 2) subcellular distribution of the tetraspanin CD63, 3) of Rab11, 4) of filamentous actin, and 5) of CD8. We have also compared 6) nuclear size and nuclear general morphology, 7) the number and distribution of mitochondria, 8) morphology and subcellular distribution of the cis- and 9) trans-Golgi networks. Finally, 10) we have compared basal autophagy in salivary cells with or without knocking down exocyst subunits. The markers that we have analysed behaved similarly to those of control salivary glands, suggesting that the observed defects in regulated exocytosis indeed reflect different roles of the exocyst in the secretory pathway, rather than poor cellular health or impaired cell polarity.  

      Our conclusions are in line with previous studies in which apico-basal polarity, Golgi complex morphology and distribution, as well as apical membrane trafficking were also evaluated in exocyst mutant backgrounds, finding no anomalies (Jafar-Nejad et al, 2005). 

      Conversely, in studies in which apical polarity was disturbed by interfering with Crumbs levels, SG biogenesis, maturation and exocytosis were not affected (Lattner et al, 2019), indicating that these processes not necessarily interfere with one another.  

      (4) Final recommendation: In the absence of stronger evidence for these other exocyst roles, I would suggest focusing the study on the canonical role (interesting, as it was previously reported that Drosophila exocyst had no function in the salivary gland and limited function elsewhere [DOI: 10.1034/j.1600-0854.2002.31206.x]), and leave the alternative roles for discussion and deeper study in the future.  

      We appreciate the reviewer´s recommendation. However, we believe that the major strength of our work is the discovery of non-canonical roles of the exocyst complex, unrelated to its function as a tethering complex for vesicle-plasma membrane fusion. We believe that in the new version of our manuscript, we provide stronger evidence supporting the two novel roles of the exocyst:

      a) Its participation in maintaining the normal structure of the Golgi complex, and b) Its function in secretory granule maturation.

      Reviewer 2:

      (5) General comment: A key strength is the breadth of the assays and study of all 8 exocyst subunits in a powerful model system (fly larvae). Many of the assays are quantitated and roles of the exocyst in early phases of granule biogenesis have not been ascribed. 

      We are grateful that the reviewer appreciates the novelty of our contribution.

      (6) However there are several weaknesses, both in terms of experimental controls, concrete statements about the granules (better resolution), and making a clear conceptual framework. Namely, why do KD of different exocysts have different effects on presumed granule formation

      The reviewer has raised a point that is central to the interpretation of all our data throughout the manuscript. The short answer is that the extent of RNAi-dependent silencing of exocyst subunits determines the phenotype: 

      1) Maximum silencing affects Golgi complex morphology and prevents SG biogenesis. 2) Intermediate silencing blocks SG maturation, without affecting Golgi complex morphology and SG biogenesis. 3) Weak silencing blocks SG tethering and fusion with the plasma membrane, without affecting Golgi complex morphology, SG biogenesis or SG maturation. 

      In other words, 1) Low levels of exocyst subunits are sufficient for normal Golgi complex morphology and SG biogenesis. 2) Intermediate levels of exocyst subunits are sufficient for SG maturation (and also sufficient for SG biogenesis). 3) High levels of exocyst subunits are required for SG tethering and subsequent fusion with the plasma membrane. 

      Based on the above notion, we have exploited the fact that temperature can fine-tune the level of Gal4/UAS-dependent transcription, thereby achieving different levels of silencing, as shown by Norbert Perrimon et al in their seminal paper “the level of RNAi knockdown can also be altered by using Gal4 lines of various strengths, rearing flies at different temperatures, or via coexpression of UAS-Dicer2” (Perkins et al, 2015). 

      We found in our system that indeed, by applying appropriate silencing conditions (RNAi line and temperature) to any of the eight subunits of the exocyst, we have been able to obtain one of the three alternative phenotypes: Impaired SG biogenesis, or impaired SG maturation, or impaired SG tethering/fusion with the plasma membrane.

      These concepts are summarized below in Author response image 4. Please see also at point 26, the general comment of Reviewer #3. 

      We have conducted qRT-PCR assays to provide experimental support to the notions summarized above in Author response image 4. We measured the remaining levels of mRNAs of some of the exocyst subunits, after inducing RNAi-mediated silencing at different temperatures, or with different RNAi transgenic lines. The remaining RNA levels after silencing correlate well with the observed phenotypes, following the predictions of Author response image 4 and summarized in Author response image 5. These new data are now shown in Supplementary Figure 2 of the revised version of the manuscript, and described in lines 153-159 at the Results section.

      (7) Why does just overexpression of a single subunit (Sec15) induce granule fusion?

      The reviewer raises a very important point. Based on available data from the literature, Sec15 behaves as a seed for assembly of the holocomplex and it also mediates the recruitment of the holocomplex to SGs through its interaction with Rab11 (Escrevente et al, 2021; Bhuin and Roy, 2019; Wu et al, 2005; Zhang et al, 2004; Guo et al, 1999). Thus, overexpression of Sec15 is expected to enhance exocyst assembly, thereby potentiating the activities carried out by the complex in the cell, including SG homofusion. In the revised version of the manuscript we have also performed the overexpression of Sec8, finding that, unlike Sec15, Sec8 fails to induce homotypic fusion. These results were expected, as they confirm that Sec8 does not behave as a seed for mounting the whole complex. These new data have been included in Figure 7E-H, and are described in text lines 221-229 of the Results section. 

      Author response image 4.

      Conceptual model of RNAi expression at different temperatures , remaining levels of mRNA/protein levels and phenotypes obtained at each temperature.

      Author response image 5.

      qRT-PCR assays presented in Supplementary Figure 2 are shown in combination with the phenotypes observed at each of the conditions analyzed. Note the correlation between phenotypes and the extent of mRNA downregulation.

      (8) While the paper is fascinating, the major comments need to be addressed to really be able to make better sense of this work, which at present is hard to disentangle direct vs. secondary effects, especially as much of the TGN seems to be altered in the KDs.  

      We hope that our response to point 6) has helped to clarify this important point raised by the Reviewer. After applying silencing conditions where normal structure of the trans-Golgi network is impaired, SG biogenesis does not occur. Thus, since SGs do not form, it is not conceivable to detect defects in SG maturation or SG fusion with the plasma membrane in the same cell.

      (9) The authors conveniently ascribe many of the results to the holocomplex, but their own data (Fig. 4 and Fig. 6) are at odds with this.

      This is another central point of our work, so we thank the reviewer for his/her comment. In Figures 4A, 7A and 9A of the revised version of the manuscript, we show that, by inducing appropriate levels of silencing of any of the 8 subunits of the exocyst, each of the three alternative phenotypic manifestations can occur. In our opinion, this argues in favour of a function for the whole exocyst complex in each of the three specific activities proposed in our study: 1) SG biogenesis, 2) SG maturation, and 3) SG tethering/fusion with the plasma membrane. In detailed characterizations of these three phenotypes performed throughout the study, we decided to induce silencing of just two or three of the subunits of the exocyst, assuming that the whole complex accounts the mechanisms involved.

      Major comments

      (10) Resolution not sufficient. Identification of "mature secretory granules" (MSG) in Fig. 3 is based on low-resolution images in which the MSG are not clearly seen (see control in Fig. 3A) and rather appear as a diffuse haze, and not as clear granules. There may be granules here, but as shown it is not clear. Thus it would be helpful to acquire images at higher resolution (at the diffraction limit, or higher) to see and count the MSG.

      We thank the reviewer for raising this point, as it may not be straightforward to the reader to identify the SGs throughout the figures of our study. To make it clearer, in Figure 3A (magnified insets on the right), we have delimitated individual SGs with a green dotted line, and included diagrams (far right), which we hope will help the identification of SGs. In Figure 3B, we show that after silencing Sec84, a mosaic phenotype was observed: In some cells SGs fail to undergo maturation, and remain smaller than normal. In other cells of this mosaic phenotype, biogenesis of SGs was impaired and the fluorescent cargo remained trapped in a mesh-like structure (that we later show that corresponds to the ER). The dotted line marks individual SGs, and the diagrams included on the right intend to help the interpretation of the phenotype. The mesh-like structures where Sgs3-GFP was retained are also marked with dotted line, and schematized on the right. These new schemes are described in the Figure 3 caption of the revised version of the manuscript.

      We wish to mention that all the confocal images depicted in this figure and throughout the manuscript  have been captured at high resolution, with a theoretical resolution limit of 168177nm (d = γ/2NA). Given that secretory granules range from 0.8-7µm in diameter, the resolution is more than sufficient to clearly resolve these structures. 

      (11) Note: the authors are not clear on which objective was used. Maybe the air objective as the resolution appears poor).  

      In this particular figure, we have utilized a Plan-Apochromat 63X/1.4NA oil objective of the inverted Carl Zeiss LSM 880 confocal microscope (mentioned in materials and methods).

      (12) They need to prove that the diffuse Sgs3-GFP haze is indeed due to MSG.  

      If we interpret correctly the concern of the reviewer, what he/she calls “diffuse haze” is actually the distribution of Sgs3-GFP within individual SGs, which, as previously reported by other authors, is not homogeneous at this stage (Syed et al. 2022). We hope that the diagrams that we have included in Figure 3 A, B (point 10) will help the readers interpreting the images.   

      (13) Related it is unclear what are the granule structures that correspond to Immature secretory granules (ISG) and cells with mesh-like structures (MLS)?

      We are confident that the diagrams now included in Figure 3A and B will help the interpretation, and particularly to identify immature granules and the mesh-like structure generated after silencing of exocyst subunits.

      (14) Similarly, Sgs3 images of KD of 8 exocyst subunits were interpreted to be identical, in Fig. 4, but the resolution is poor.

      We hope that the issue related to resolution of our images has been properly addressed in the response to point 10) of this letter. In Figure 4A, we show that after silencing of any of the 8 subunits (with the appropriate conditions), in all cases SG biogenesis was impaired, and Sgs3GFP was instead retained in a mesh-like structure. Images obtained after silencing different exocyst subunits are of course not identical, but in all cases, a mesh-like structure has replaced the formation of SGs (Figure 4A). Hopefully, the diagrams now included in Figure 3A and B help the correct interpretation of the phenotypes throughout the study.

      To demonstrate that the structure in which Sgs3-GFP was retained upon exocyst complex knockdown corresponds to the ER, we performed a colocalization analysis between Sgs3-GFP and the ER markers GFP-KDEL or Bip-sfGFP-HDEL, after which we calculated the Pearsons Coefficient, which indicated substantial colocalization (Figure 4B-G and Supplementary Figures 7 and 8). These new data are described in lines 196-199 of the revised version of the manuscript. To facilitate the visualization of the results, in the revised version of the manuscript we have included magnified cropped areas of the images shown in Figure 4A.

      (15) What is remarkable is a highly variable effect of different subunit KD on the percentage of cells with MLS (Fig. 4C). Controls = 100 %, Exo70=~75% (at 19 deg), Sec3 = ~30%, Sec10 = 0%, Exo84 = 100% ... This is interesting for the functional exocyst is an octameric holocomples, thus why the huge subunit variability in the phenotypes? The trivial explanation is either: i) variable exocyst subunit KD (not shown) or ii) variability between experiments (no error bars are shown). Both should be addressed by quantification of the KD of different proteins and secondly by replicating the experiments.

      We agree with the reviewer statement. We believe that both, variability of KD efficiency (i) and variability between experiments (ii) contribute to the variable effect observed after knocking down the different subunits. As detailed in the response to point 6), we have performed qRT-PCR determinations to confirm that the severity of the phenotype depends on the efficiency of RNAimediated silencing. We chose to analyse in detail the effect on the subunits exo70 and sec3, which were those with the highest phenotypic differences between the three silencing temperatures utilized. We found that as expected, the levels of silencing were temperaturedependent, being higher at 29°C and lower at 19°C. These data were included in Supplementary Figure 2, and described lines 153-159 of the Results section and also summarized in Author response images 4 and 5 of this rebuttal letter.

      We thank the reviewer for his/her comment on the replication of experiments and statistics. We failed to include detailed numerical information in the original submission, such as the number of replicas and standard deviations of the data depicted in Figure 3C and Supplementary Figure 1, so we apologize for this omission. In the revised version of the manuscript, we have included a table (Supplementary Table 3) in which all the raw data of Figure 3C and Supplementary Figure 1, including standard deviations, are now depicted.

      (16) If their data holds up then the underlying mechanism here needs to be considered.

      (Note: there is some precedent from the autophagy field of differential exocyst effects)

      Our proposed mechanism is essentially that the holocomplex is required for multiple processes along the secretory pathway. Each of these actions (Golgi structure maintenance, SG maturation and SG tethering/fusion with the plasma membrane) requires different amounts of holocomplex activity, being this the reason why each phenotype manifests at different levels of RNAi-mediated silencing (Author response image 4 of this letter). The model predicts that Golgi structure maintenance requires minimal levels of complex activity, and that is why strong knock-down of exocyst subunits is required to obtain this phenotype. In line with our results, it has been reported that other tethering complexes of the CATCHR family are also required for maintaining Golgi cisternae stuck together (D'Souza et al, 2020; Khakurel and Lupashin, 2023; Liu et al, 2019). One possibility is that the exocyst may play a redundant role in the maintenance of the normal structure of the Golgi complex, along with other CATCHR complexes. This potential redundancy could explain why severe exocyst knock-down is required to observe structural anomalies at this organelle. On the other end of the spectrum, we propose that tethering/fusion with the plasma membrane is very susceptible to even slight reduction of complex activity, so that mild RNAi-mediated silencing is sufficient to provoke defects in this process. This proposed model is depicted in Author response image 4 and discussed in lines 395-405 of the Discussion section. 

      (17) In the salivary glands the authors state that the exocyst is needed for Sgs3-GFP exit from the ER. First, Pearson's coefficient should be shown so as to quantitate the degree of ER localizations of all KDs.

      We thank the reviewer for this comment that helped us to strengthen the observation that when SG biogenesis is impaired, Sgs3-GFP remains trapped in the ER. In the revised version of the manuscript, we have calculated Pearson´s coefficient to assess colocalization between ER markers (GFP-KDEL or Bip-sfGFP-HDEL) and Sgs3-GFP in salivary gland cells that express sec15RNAi. The Pearson’s coefficient was around 0.6 for both ER markers, indicating that colocalization with Sgs3-GFP was substantial (Supplementary Figure 8, text lines 196-199 of the Results section).

      (18) Second, there should be some rescue performed (if possible) to support specificity. 

      As suggested by the reviewer, we have performed a rescue experiment of the phenotype provoked by the expression of sec15 RNAi, which consisted on the retention of Sgs3-GFP in the endoplasmic reticulum: Expression of Sec15-GFP reverted substantially the ER retention phenotype, rescuing SG biogenesis and also SG maturation in most cells (over 60% of the cells). These new data are now shown in Supplementary Figure 4, and described in lines 168-171 of the Results section.

      (19) Third, importantly other proteins that should traffic to the PM need to be shown to traffic normally so as to rule out a non-specific effect.

      We have addressed this issue (also mentioned by Reviewer #1), by analyzing the localization of a number of polarization markers, finding that the overall polarization of the cell was not affected by loss of function of exocyst subunits. Please, see our response to the point 3) raised by Reviewer #1. The new data showing cell polarization markers are shown in Supplementary Figure 6 of the revised version of the manuscript, and described on text lines 172-179 of the Results section.

      (20) It is unclear from their model (Fig. 5) why after exocyst KD of Sec15 the cis-Golgi is more preserved than the TGN, which appears as large vacuoles. This is not quantitated and not shown for the 8 subunits.

      We thank the reviewer for this relevant comment. We agree that the phenotype of either, sec15 or sec3 loss-of-function cells manifests differently with cis-Golgi and trans-Golgi markers. While the cis-Golgi marker looked fragmented and aggregated, the trans-Golgi marker adopted a swollen appearance. However, in our view, the different appearance of the two markers does not necessarily imply that one compartment is more preserved than the other. In the revised version of the manuscript, we have quantified the penetrance of the phenotypes provoked by sec15 or sec3 silencing, using both cis-Golgi and trans-Golgi markers. In both cases, the penetrance was high, although even higher with the trans-Golgi marker. These new data are now depicted in Supplementary Figure 9 of the revised version of the manuscript. 

      It is interesting to mention that in HeLa cells, as well as in the retinal epithelial cell line hTERT, Golgi phenotypes similar to those we have described here have been reported after loss-offunction of other tethering complexes, which were shown to maintain the Golgi cisternae stuck together, including the GOC and GARP complexes (D'Souza et al, 2020, Khakurel and Lupashin, 2023; Shijie Liu et al, 2019). As we did throughout our work, not every aspect of the analysis included the silencing of all eight subunits. In this case, we chose to silence Sec3 and Sec15. Please note that we have modified the model depicted in Figure 6E-F, to highlight the cis- and transGolgi phenotypes upon exocyst knock-down, as well as the localization of the exocyst in cisternae of the Golgi complex.

      (21) Acute/Chronic control: It would be nice to acutely block the exocyst so as to better distinguish if the effects observed are primary or secondary effects (e.g. on a recycling pathway).

      We thank the reviewer for raising this important issue. To address this point, and to be able to induce silencing of exocyst subunits at specific time intervals of larval development, we utilized a strategy based on a thermosensitive variant of the Gal4 inhibitor Gal80 (Gal80ts)(Lee and Luo, 1999). We blocked Gal4 activity (and therefore RNAi expression) by maintaining the larvae at 18 °C during the 1st and 2nd instars (until 120 hours after egg lay), and then induced the activity of Gal4 specifically at the 3rd larval instar by raising the temperature to 29 ºC, a condition in which Gal80ts becomes inactive. After silencing the expression of sec3 or sec15 at the 3rd larval instar only, the phenotype was very similar to that observed after chronic silencing of exocyst subunits (larvae maintained at 29 ºC all throughout development, where Gal4 was never inhibited). These observations suggest that the defects observed in the secretory pathway after knock down of exocyst subunits reflect genuine functions of the exocyst in this pathway, rather than a secondary effect derived from impaired development of the salivary glands at early larval stages. These new results are now shown in Supplementary Figure 3, and described in manuscript lines 160-171 of the Results section.   

      (22) Granule homotypic fusion. Strangely over-expression of just one subunit, Sec15-GFP, made giant secretory granules (SG) that were over 8 microns big! Why is that, especially if normally the exocyst is normally a holocomplex. Was this an effect that was specific to Sec15 or all exocyst subunits? Is the Sec15 level rate limiting in these cells? It may be that a subcomplex of Sec15/10 plays earlier roles, but in any case this needs to be addressed across all (or many) of the exocyst subcomplex members.

      Please, see our response to point 7) of this letter. Sec15 is believed to act as a seed for the formation of the whole complex.

      (23) In summary, there are clearly striking effects on secretory granule biogenesis by dysfunction of the exocyst, however right now it is hard to disentangle effects on ERGolgi traffic, loss of the TGN, and a problem in maturation or fusion of granules. 

      As discussed in detail in our response to the point 3 raised by Reviewer #1, the secretory pathway is highly synchronized in each of the cells of the Drosophila salivary gland. SG biogenesis, SG maturation and SG fusion with the plasma membrane never occur simultaneously in the same cell. Thus, in a cell in which ER-Golgi traffic is impaired (and SG biogenesis does not occur), SGs do not exist, and therefore, they cannot exhibit defects in the process of maturation or fusion with the plasma membrane. In summary, we believe that our work has shown that in Drosophila larval salivary glands the exocyst holocomplex is required for (at least) three functions along the secretory pathway: 1) To maintain the appropriate Golgi complex architecture, thus enabling ERGolgi transport; 2) For secretory granule maturation: both, homotypic fusion and acquisition of maturation factors; 3) For secretory granule exocytosis: secretory granule tethering to enable subsequent fusion with the plasma membrane. As mentioned above (point 6 of this letter), these three functions require different amounts of the holocomplex, and therefore can be revealed by inducing different levels of silencing.  

      (24) It is also confusing if the entire exocyst holocomplex or subcomplex plays a key role 

      The fact that, by silencing any of the subunits (with the appropriate conditions) it is possible obtain any of the 3 phenotypes (impaired SG biogenesis, impaired SG maturation or impaired SG fusion with the plasma membrane) argues in favour of a function of the complex as a whole in each of these three functions.

      Reviewer 3:

      (25) General comment: Freire and co-authors examine the role of the exocyst complex during the formation and secretion of mucins from secretory granules in the larval salivary gland of Drosophila melanogaster. Using transgenic lines with a tagged Sgs3 mucin the authors KD expression of exocyst subunit members and observe a defect in secretory granules with a heterogeneity of phenotypes. By carefully controlling RNAi expression using a Gal4-based system the authors can KD exocyst subunit expression to varying degrees. The authors find that the stronger the inhibition of expression of exocyst the earlier in the secretory pathway the defect. The manuscript is well written, the model system is physiological, and the techniques are innovative.

      We appreciate the reviewer´s assessment of our work. 

      (26) My major concern is that the evidence underlying the fundamental claim of the manuscript that "the exocyst complex participates" in multiple secretory processes lacks direct evidence.

      We thank the reviewer for raising this important issue. We believe that the analysis of Sec15 subcellular localization during salivary gland development (Figures 5, 7B-D and 9E-F), in combination with the detailed analysis of the phenotypes provoked by loss-of-function of each of the exocyst subunits, provide evidence supporting multiple functions of the exocyst in the secretory pathway. We have also included 3D reconstructions and videos of GFP-Sec15 colocalization with Golgi and SG markers to support exocyst localization associated to these structures (Supplementary Videos 1-7), text lines 200-210; 216-221 and 303-305.

      (27) It is clear from multiple lines of evidence, which are discussed by the authors, that exocyst is essential for an array of exocytic events. The fundamental concern is that loss of homeostasis on the plasma membrane proteome and lipidome might have severe pleiotropic effects on the cell.

      We agree with the reviewer that this is an important point that needed to be addressed. As discussed in detail above at the response to point 3 raised by Reviewer #1, we have analysed several plasma membrane markers (including a PI(4,5)P2 lipid reporter), and found that overall, plasma membrane integrity and polarity were not substantially affected (Supplementary Figure 6). In addition, we have analyzed several markers of general cellular “health” that indicate that salivary gland cells do not seem to be distressed by the reduction of exocyst complex activity (Supplementary Figure 5). These new data are described in lines 172-179 of the Results section.

      (28) Perhaps the authors have more evidence that exocyst is important for homeotypic fusion of the SGs, as supported by the localisation of Sec15 on the fusion sites.

      We believe that the fact that, by silencing any of the exocyst subunits (with the appropriate conditions), immature smaller-than-normal granules were observed, argus in favour that the exocyst as a whole participates in SG homofusion (Figure 7A). In addition, we have included more images, quantifications, 3D reconstructions and videos of GFP-Sec15 localized just at the contact sites between immature SGs. We have quantified and compared GFP-Sec15 localization at immature SG vs its localization at mature SGs, finding that localizes preferentially at immature SGs, supporting a role of the exocyst as a tethering complex during homotypic fusion (shown Figure 7B-C and Supplementary Videos 4-6, and described in lines 216-221 of the Results section). Please see also our response to the point 2 raised by reviewer 1 in this rebuttal letter, and to Author response image 3 above in this letter.

      (29) The second question that I think is important to address is, what exactly do the varying RNAi levels correspond to in terms of experiments, and have these been validated? Due to the fundamental claim being that the severity of the phenotype being correlated with the level of KD, I think validation of this model is absolutely essential.  

      We thank the Reviewer for raising this important point, and agree it was lacking in the original version of our manuscript. As discussed in our response to the point 6) raised by Reviewer #2, we have performed qRT-PCR determinations for exo70 and sec3 mRNA levels after inducing silencing of these subunits at different temperatures, or with different RNAi transgenic lines. The remnant mRNA levels correlate well with the observed phenotypes. Please see Supplementary Figure 2 of the revised manuscript, and Author response image 5 of this rebuttal letter; described in lines 155-159 of the Results section. 

      Recommendations for the authors:

      Reviewer #1 (Recommendations For The Authors):

      -  The authors assert in the discussion that exocyst involvement in constitutive secretion is well documented. This is based on a very recent study in mammalian culture cells. Therefore, I would not dismiss the issue as completely settled. Furthermore, a previous study of Drosophila sec10 reported no roles outside the ring gland (DOI: 10.1034/j.1600-0854.2002.31206.x).

      We have included these observations in the Discussion section. Lines 326-329.

      -  A salivary gland screening by Julie Brill's lab reported exocyst components as hits (DOI: 10.1083/jcb.201808017).

      We have referred to this paper in the Discussion section. Lines 326-329.

      -  It should be explained in more detail what is measured in graphs 7C, F, and others quantifying fluorescence around secretory granules. Looking at the images, the decrease in Rab1 and Rab11 seems less convincing.

      We have made a clearer description of how fluorescence intensity was measured in the Methods section lines 558-561. Also, we have uploaded a source data file in which the raw data of each experiment used for quantifications are disclosed. 

      Please note that the data indicates that Rab11 levels are higher in sec5 (Figure 8J-L) and sec3 (supplementary Figure 11M-R).

      Reviewer #2 (Recommendations For The Authors):

      No major issues.

      Writing - The authors should better frame their interpretations of other studies of the exocyst that include the role in autophagy, Palade body trafficking, and differential roles of the subunits.

      We have discussed these specific points in the Discussion section, lines 348-355 and 409-410.

      Minor - Fig. 6A: Why are variable temperatures (19-29 deg C used for the 8 KD experiments)?

      Please show it all at the same temperature (control too).

      The need for the usage of specific temperatures to obtain specific phenotypes with each of the RNAi lines used was explained in point 6 of this letter.

      Reviewer #3 (Recommendations For The Authors):

      In the abstract, the authors refer to the exocytic process and go on to describe secretory granule biogenesis and exocytosis. However, there are many exocytic processes aside from secretory granule biogenesis, and I think the authors should clarify this.

      Corrected in the Abstract. Lines 19-21

      Page 17 Thomas, 2021 reference, there is a glitch with the reference.

      Thanks for noticing. Fixed.

      References

      Bhuin T, Roy JK. Developmental expression, co-localization and genetic interaction of exocyst component Sec15 with Rab11 during Drosophila development. Exp Cell Res. 2019 Aug 1;381(1):94-104. doi: 10.1016/j.yexcr.2019.04.038. Epub 2019 May 7. PMID: 31071318.

      D'Souza Z, Taher FS, Lupashin VV. Golgi inCOGnito: From vesicle tethering to human disease. Biochim Biophys Acta Gen Subj. 2020 Nov;1864(11):129694. doi: 10.1016/j.bbagen.2020.129694. Epub 2020 Jul 27. PMID: 32730773; PMCID: PMC7384418.

      Escrevente C, Bento-Lopes L, Ramalho JS, Barral DC. Rab11 is required for lysosome exocytosis through the interaction with Rab3a, Sec15 and GRAB. J Cell Sci. 2021 Jun 1;134(11):jcs246694. doi: 10.1242/jcs.246694. Epub 2021 Jun 8. PMID: 34100549; PMCID: PMC8214760.

      Guo W, Roth D, Walch-Solimena C, Novick P. The exocyst is an effector for Sec4p, targeting secretory vesicles to sites of exocytosis. EMBO J. 1999 Feb 15;18(4):1071-80. doi: 10.1093/emboj/18.4.1071. PMID: 10022848; PMCID: PMC1171198.

      Jafar-Nejad H, Andrews HK, Acar M, Bayat V, Wirtz-Peitz F, Mehta SQ, Knoblich JA, Bellen HJ. Sec15, a component of the exocyst, promotes notch signaling during the asymmetric division of Drosophila sensory organ precursors. Dev Cell. 2005 Sep;9(3):351-63. doi: 10.1016/j.devcel.2005.06.010. PMID: 16137928.

      Khakurel A, Lupashin VV. Role of GARP Vesicle Tethering Complex in Golgi Physiology. Int J Mol Sci. 2023 Mar 23;24(7):6069. doi: 10.3390/ijms24076069. PMID: 37047041; PMCID: PMC10094427.

      Lattner J, Leng W, Knust E, Brankatschk M, Flores-Benitez D. Crumbs organizes the transport machinery by regulating apical levels of PI(4,5)P2 in Drosophila. Elife. 2019 Nov 7;8:e50900. doi: 10.7554/eLife.50900. PMID: 31697234; PMCID: PMC6881148.

      Lee T, Luo L. Mosaic analysis with a repressible cell marker for studies of gene function in neuronal morphogenesis. Neuron. 1999 Mar;22(3):451-61. doi: 10.1016/s08966273(00)80701-1. PMID: 10197526.

      Liu S, Majeed W, Grigaitis P, Betts MJ, Climer LK, Starkuviene V, Storrie B. Epistatic Analysis of the Contribution of Rabs and Kifs to CATCHR Family Dependent Golgi Organization. Front Cell Dev Biol. 2019 Aug 2;7:126. doi: 10.3389/fcell.2019.00126. PMID: 31428608; PMCID: PMC6687757.

      Perkins LA, Holderbaum L, Tao R, Hu Y, Sopko R, McCall K, Yang-Zhou D, Flockhart I, Binari R, Shim HS, Miller A, Housden A, Foos M, Randkelv S, Kelley C, Namgyal P, Villalta C, Liu LP, Jiang X, Huan-Huan Q, Wang X, Fujiyama A, Toyoda A, Ayers K, Blum A, Czech B, Neumuller R, Yan D, Cavallaro A, Hibbard K, Hall D, Cooley L, Hannon GJ, Lehmann R, Parks A, Mohr SE, Ueda R, Kondo S, Ni JQ, Perrimon N. The Transgenic RNAi Project at Harvard Medical School: Resources and Validation. Genetics. 2015 Nov;201(3):843-52. doi: 10.1534/genetics.115.180208. Epub 2015 Aug 28. PMID: 26320097; PMCID: PMC4649654.

      Wu S, Mehta SQ, Pichaud F, Bellen HJ, Quiocho FA. Sec15 interacts with Rab11 via a novel domain and affects Rab11 localization in vivo. Nat Struct Mol Biol. 2005 Oct;12(10):879-85. doi: 10.1038/nsmb987. Epub 2005 Sep 11. PMID: 16155582.

      Yeaman C, Grindstaff KK, Wright JR, Nelson WJ. Sec6/8 complexes on trans-Golgi network and plasma membrane regulate late stages of exocytosis in mammalian cells. J Cell Biol. 2001 Nov 12;155(4):593-604. doi: 10.1083/jcb.200107088. Epub 2001 Nov 5. PMID: 11696560; PMCID: PMC2198873.

      Zhang XM, Ellis S, Sriratana A, Mitchell CA, Rowe T. Sec15 is an effector for the Rab11 GTPase in mammalian cells. J Biol Chem. 2004 Oct 8;279(41):43027-34. doi: 10.1074/jbc.M402264200. Epub 2004 Jul 29. PMID: 15292201.

    1. Author response:

      The following is the authors’ response to the original reviews.

      Public Reviews:

      Reviewer #1 (Public Review):

      The paper proposes an interesting perspective on the spatio-temporal relationship between FC in fMRI and electrophysiology. The study found that while similar network configurations are found in both modalities, there is a tendency for the networks to spatially converge more commonly at synchronous than asynchronous time points. However, my confidence in the findings and their interpretation is undermined by an apparent lack of justification for the expected outcomes for each of the proposed scenarios, and in the analysis pipeline itself.

      Main Concerns

      (1) Figure 1 makes sense to me conceptually, including the schematics of the trajectories, i.e.

      Scenario 1: Temporally convergent, same trajectories through connectome state space

      Scenario 2: Temporally divergent, different trajectories through connectome state space

      However, based on my understanding I am concerned that these scenarios do not necessarily translate into the schematic CRP plots shown in Figure 2C, or the statements in the main text:

      For Scenario 1: "epochs of cross-modal spatial similarity should occur more frequently at on-diagonal (synchronous) than off-diagonal (asynchronous) entries, resulting in an on-/off-diagonal ratio larger than unity"

      For Scenario 2: "epochs of spatial similarity could occur equally likely at on-diagonal and off-diagonal entries (ratio≈1)"

      Where do the authors get these statements and the schematics in Figure 2C from? Are they based on previous literature, theory, or simulations?

      I am not convinced based on the evidence currently in the paper, that the ratio of off- to on-diagonal entries (and under what assumptions) is a definitive way to discriminate between scenarios 1 and 2.

      For example, what about the case where the same network configuration reoccurs in both modalities at multiple time points? It seems to me that one would get a CRP with entries occurring equally on the on-diagonal as on the off-diagonal, regardless of whether the dynamics are matched between the two modalities or not (i.e. regardless of scenario 1 or 2 being true).

      This thought experiment example might have a flaw in it, and the authors might ultimately be correct, but nonetheless, a systematic justification needs to be provided for using the ratio of off- to on-diagonal entries to discriminate between scenarios 1 and 2 (and under what assumptions it is valid).

      In the absence of theory, a couple of ways I can think of to gain insight into this key aspect are:

      (1) Use surrogate data for scenarios 1 and 2:

      a. For scenario 1: Run the CRP using a single modality. E.g. feed in the EEG into the analysis as both modality 1 AND modality 2. This should provide at least one example of CRP under scenario 1 (although it does not ensure that all CRPs under this scenario will look like this, it is at least a useful sanity check)

      b. For scenario 2: Run the CRP using a single modality plus a shuffled version. E.g. feed in the EEG into the analysis as both modality 1 AND a temporally shuffled version of the EEG as modality 2. The temporal shuffling of the EEG could be done by simply splitting the data into blocks of say ~10s and then shuffling them into a new order. This should provide a version of the CRP under scenario 2 (although it does not ensure that all CRPs under this scenario will look like this, it is at least a useful sanity check).

      (2) Do simulations, with clearly specified assumptions, for scenarios 1 and 2. One way of doing this is to use a simplified (state-space) setup and randomly simulate N spatially fixed networks that are independently switching on and off over time (i.e. "activation" is 0 or 1). Note that this would result in a N-dimensional connectome state space.

      The authors would only need to worry about simulating the network activation time courses, i.e. they would not need to bother with specifying the spatial configuration of each network, instead, they would make the implied assumption that each of these networks has the same spatial configuration in modality 1 and modality 2.

      With that assumption, the CRP calculation should simply correspond to calculating, at each time i in modality 1 and time j in modality 2, the number of networks that are activating in both modality 1 and modality 2, by using their activation time courses. Using this, one can simulate and compute the CRPs for the two scenarios:

      a. Scenario 1: where the simulated activation timecourses are set to be the same between both modalities

      b. Scenario 2: where the simulated activation timecourses are simulated separately for each of the modalities

      We thank the reviewer for raising this important matter as it directly relates to our study hypothesis. To address this point, we chose to focus on the first of the two alternative suggestions of the reviewer, as it provides evidence based on empirical data. In line with the reviewer’s suggestion 1, recurrence plots have indeed been previously applied to connectome dynamics data from the same modality [Hansen et al., NeuroImage 2015; Fig. 2B]. As shown in the referenced study, where the recurrence plot has been estimated within fMRI connectome dynamics, the on-diagonal entries have noticeably larger correlation values in comparison to off-diagonal entries. As the authors state, this contrast emphasizes the autocorrelation of connectome dynamics in their single modality recurrence plot. Extending these findings to our cross-modal recurrence plots, more synchronicity of connectome dynamics across fMRI and EEG will -by theory- translate into stronger correlation values along the diagonal axis as it represents neighboring timepoints in the data. On the other hand, less cross-modal synchronicity translates to a lack of such correlation prevalence along the diagonal axis.

      Complementing these statements with empirical data, Author response image 1 shows the fMRI-to-iEEG and fMRI-to-fMRI CRPs side by side as suggested by the reviewer. For simplicity, we thresholded each CRP at the top 5% of entries and calculated their corresponding on-/off-diagonal ratios. The on/off-diagonal ratio for fMRI-to-fMRI CRP was 4.32 ± 6.26 across -5 to +5 TR lags (with a maximum of 16.56 at a lag of one TR), while this value was 1.00 ± 0.31 for fMRI-to-iEEG CRP. Thus, it becomes apparent that synchronicity of connectome dynamics directly translates to the on-/off-diagonal ratio in CRP.

      Author response image 1.

      Sample CRP shown for a subject for comparing two cases: fMRI-to-iEEG (left) and fMRI-to-fMRI (right). The comparison shows that in the presence of genuine synchronous connectome dynamics, as expected for the within-molality case (right panel), the on-/off-diagonal ratio is expected to show noticeably higher values. This figure establishes a strong link between our proposed metric of on-/off-diagonal ratio and the extent of synchronicity of connectome dynamics.

      Author response image 2.

      On-/off-diagonal ratio in the fMRI-to-fMRI recurrence plot is considerably higher than the cross-modal fMRI-to-iEEG case. Horizontal axis shows the lag where the metric was calculated in the CRP. The bars reflect the group average metric while the whickers show standard deviation. Note that for the within-modality case, ratio is not defined at lag zero because of identical connectome frames.

      (2) Choices in the analysis pipeline leading up to the computation of FC in fMRI or EEG will affect the quality of information available in the FC. For example, but not only, the choice of parcellation (in the study, the number of parcels is very high given the number of EEG sensors). I think it is important that we see the impact of the chosen pipeline on the time-averaged connectomes, an output that the field has some idea about what is sensible. This would give confidence that the information being used in the main analyses in the paper is based on a sensible footing and relates to what the field is used to thinking about in terms of FC. This should be trivial to compute, as it is just a case of averaging the time-varying FCs being used for the CRP over all time points. Admittedly, this approach is less useful for the intracranial EEG.

      We agree with the reviewer on ensuring that the time-averaged FC aligns with expectations of the field and prior work. For this reason, our supplementary analysis already included an analysis that replicates the well-established (albeit modest) spatial similarity between fMRI static connectome and EEG/iEEG static connectomes:

      “In scalp EEG-fMRI data, cross-modal spatial (2D) Pearson correlation of group-level time-averaged connectomes between fMRI and EEG-FCAmp or fMRI and EEG-FCPhase were calculated across all frequency bands. The average spatial correlation value across frequency bands r = 0.28 and r = 0.28 for EEG-FCAmp and EEG-FCPhase, respectively. The spatial correlation values across all frequency bands and connectivity measures were significantly higher than the corresponding null distributions generated by phase-permuted group-level fMRI-FC spatial organization (p<0.005; 200 repetitions; FDR-corrected at q<0.05 for the number of frequency bands). …. Of note, the small effect sizes are strongly in line with prior literature (Hipp and Siegel, 2015; Wirsich et al., 2017; Betzel et al., 2019) and may point to possible divergence in the dynamic domain as investigated in the main manuscript.”

      This replication directly confirms the validity of our selected atlas for further investigations into the connectome dynamics. We acknowledge that with 64 EEG channels, one can only estimate a relatively coarse connectome. Among the well-known coarse atlases, we chose the Desikan-Killiany atlas as it is based on anatomical features, eliminating possible biases towards a particular functional data modality. Moreover, this atlas has been commonly used for multimodal functional connectivity studies, facilitating the confirmation of prior findings in the time-averaged domain [Deligianni et al. Front. Neurosci 2104, Wirsich et al. NeuroImage, 2020, Wirsich et al., NeuroImage 2021].

      (3) Leakage correction. The paper states: "To mitigate this issue, we provide results from source-localized data both with and without leakage correction (supplementary and main text, respectively)." Given that FC in EEG is dominated by spatial leakage (see Hipp paper), then I cannot see how it can be justified to look at non-spatial leakage correction results at all, let alone put them up front as the main results. All main results/figures for the scalp EEG should be done using spatial leakage-corrected EEG data.

      We agree that relying on leakage-uncorrected scalp EEG alone would be problematic. It is for this reason that the intracranial data constructs the core of our results, emphasizing that the observed multiplex architecture of connectomes is indeed present in the absence of source leakage. Only when this finding is established in the intracranial EEG, do we provide the scalp EEG data as a generalization to whole-cortex coverage connectomes of healthy subjects. Moreover, it is known that existing source-leakage correction algorithms may inadvertently remove some of the genuine zero-lag connectivity. For instance, Finger and colleagues have shown that the similarity of functional connectivity to structural connectivity diminishes after correction for source-leakage (Finger et. al, PLOS Comp. Biol. 2016). Therefore, we have deliberately chosen to include our generalization findings before source-leakage correction (main text) as well as after source-leakage correction reflecting a more stringent approach (supplementary analysis). Importantly, our conclusions hold true for both before and after source-leakage correction.

      Reviewer #2 (Public Review):

      Summary:

      The study investigates the brain's functional connectivity (FC) dynamics across different timescales using simultaneous recordings of intracranial EEG/source-localized EEG and fMRI. The primary research goal was to determine which of three convergence/divergence scenarios is the most likely to occur.

      The results indicate that despite similar FC patterns found in different data modalities, the time points were not aligned, indicating spatial convergence but temporal divergence.

      The researchers also found that FC patterns in different frequencies do not overlap significantly, emphasizing the multi-frequency nature of brain connectivity. Such asynchronous activity across frequency bands supports the idea of multiple connectivity states that operate independently and are organized into a multiplex system.

      Strengths:

      The data supporting the authors' claims are convincing and come from simultaneous recordings of fMRI and iEEG/EEG, which has been recently developed and adapted.

      The analysis methods are solid and involve a novel approach to analyzing the co-occurrence of FC patterns across modalities (cross-modal recurrence plot, CRP) and robust statistics, including replication of the main results using multiple operationalizations of the functional connectome (e.g., amplitude, orthogonalized, and phase-based coupling).

      In addition, the authors provided a detailed interpretation of the results, placing them in the context of recent advances and understanding of the relationships between functional connectivity and cognitive states.

      Weaknesses:

      Despite the impressive work, the paper still lacks some analyses to make it complete.

      Firstly, the effect of the window size is unclear, especially in the case of different frequencies where the number of cycles that fall in a window will vary drastically. A typical oscillation lasts just a few cycles (see Myrov et al., 2024), and brain states are usually short-lived because of meta-stability (see Roberts et al., 2019).

      We now replicate our results with an additional window size. Please see section “Recommendations for the authors”.

      Secondly, the authors didn't examine frequencies lower than 1Hz despite similarities between fMRI and infra-slow oscillations found in prior literature (see Palva et al., 2014; Zhang et al., 2023).

      We address this issue below. Please see section “Recommendations for the authors”.

      On a minor note, the phase-locking value (PLV) is positively biased for EEG data (see Palva et al., 2018) and a different metric for phase coupling could be a more appropriate choice (e.g., iPLV/wPLI, see Vinck et al., 2011).

      While iPLV and wPLI are not positively biased, they may reduce genuine zero-phase connectivity as they were initially designed to address spurious zero-phase connectivity from source leakage in scalp EEG. Indeed, PLV connectivity is shown to be more strongly correlated with structural connectivity than wPLI and other phase coupling methods [Finger et al., PLOS Comp. Biol. 2016], emphasizing that it contains genuine connectivity that may be lacking when zero-phase connectivity is removed. We chose PLV because it is a widely used functional connectivity metric, particularly in intracranial data where source leakage is not a critical concern. Thus, using PLV facilitates cross-study comparisons including to our prior work [e.g. Mostame et al. NeuroImage 2020, Mostame et al. J Neurosci 2021].

      The repository with the code is also unavailable.

      Thank you for bringing this to our attention. We have now made our repository publicly accessible at: https://github.com/connectlab/Mostame2024_Multiplex_iEEG_fMRI.

      Recommendations for the authors:

      Reviewer #1 (Recommendations For The Authors):

      The window widths used to compute FC as a function of time are an important aspect, so I feel that this should be briefly described up-front in the main Results text.

      Methods. "Finally, to compensate for the time lag between hemodynamic and neural responses of the brain (Logothetis et al., 2001), we shifted the fMRI-FC time course 6 seconds backwards in time." What about the effects of temporal blurring from the HRF? Do we need to care about that?

      We agree with the importance to investigate the effect if temporal blurring of the HRF. The main text already included a replication of findings from CRPs generated using fMRI data and EEG amplitude signals convolved with the canonical HRF. This method serves as an alternative to the 6-second shifting. Both approaches produced similar results.

      Methods. In fMRI connectome computation it is common to look at partial correlation rather than full correlation. Partial correlation focuses more on direct connections. It would be good if the paper acknowledged and justified why it is OK to use full correlation.

      We have now added a brief explanation in this regard in the main text (Methods section) as follows:

      “In fMRI connectome computation, some prior work has used partial correlation instead of full correlation. Partial correlation emphasizes direct connections by calculating correlation between any pair of bran regions after regressing out the timeseries of all other regions. However, we have opted to use full correlation because this permits interpretation of our outcomes in the context of the vast existing literature that uses full correlations in fMRI including the majority of bimodal (EEG-fMRI) connectome studies (e.g. Tagliazucchi et al., 2012; Deligianni et al., 2014; Wirsich et al., 2017b, 2020, 2021; Allen et al., 2018).”

      The paper should relate the results to findings showing clear links between simultaneously recorded EEG and fMRI beyond FC. E.g. Mantini (PNAS) 2007 and Van De Ville (PNAS) 2010 to name two.

      In line with this important point, we have extended the existing discussion section that compares our outcomes to EEG-fMRI beyond functional connectivity:

      “Prior multi-modal studies of neural dynamics have predominantly aimed at methodologically cross-validating hemodynamic and electrophysiological observations, thus focusing on their convergence. These important foundational studies include e.g., the cross-modal comparison of region-wise (Mukamel et al., 2005; Nir et al., 2007) or ICN-wise (Mantini et al., 2007) activity fluctuations, instantaneous activity maps (Hunyadi et al., 2019; Zhang et al., 2020) or EEG microstates (Van de Ville 2010), infraslow connectome states (Abreu et al., 2020), or connection-wise FC including studies in the iEEG-fMRI and scalp EEG-fMRI data used in the current study (Ridley et al., 2017; and Wirsich et al., 2020, respectively). In contrast to this prior work, the current study investigated the highly time-resolved cross-modal temporal relationship at the level of FC patterns distributed over all available pairwise connections, and found a connectome-level temporal divergence. The discrepancy between temporal divergence in our study and convergence in prior studies implies that infraslow fluctuations of activity in individual regions or of FC in individual region-pairs observable in both modalities (prior studies) are neurally distinct from connectome-wide FC dynamics observable separately in each modality (current study). Indeed, we confirmed the existence of infraslow electrophysiological FC dynamics driving cross-modal temporal associations at the level of individual connections (Fig. S3) …”

      Reviewer #2 (Recommendations For The Authors):

      (1) Check different window sizes and stability of the FC patterns as a function of it.

      We thank the reviewer for the helpful feedback. We agree that the window size could possibly affect the estimation of individual connectome frames, particularly given that neural processes unfold at hundreds of milliseconds rather than seconds. However, we expect that the asynchronous nature of cross-modal convergence observed in our data would remain intact regardless of the specific window length used for FC calculations. To confirm this, we replicated some of our main analyses in the iEEG-fMRI data with a window length of 500ms (as opposed to 3s, equivalent to one TR) as follows:

      First, we showed that changing the window length does not substantially impact the overall architecture of the connectomes (Author response image 3). Particularly, the time-averaged connectome patterns across different frequency bands were all strongly correlated between the two analyses (500ms and 3s window lengths).

      Author response image 3.

      Time-averaged connectome patterns are highly replicable when calculated using 3s or 500ms window lengths. Horizontal axis represents frequency bands, while each dot represents a subject. Vertical axis shows 2D Pearson correlation of the two connectomes. The group average within each frequency band is marked by a horizontal line.

      Second, we replicated our major findings of CRP and its on-/off-diagonal ratio in the iEEG-fMRI dataset using a window length of 500ms for FC calculations. Indeed, the data does not show a substantial difference in the on-/off-diagonal ratios of the CRP entries between the 3s and 500ms window lengths. Specifically, the ratio was equal to 1.02 ± 0.07 for 500ms window length, emphasizing absence of significant temporal convergence of the connectome dynamics (see Author response image 4). A paired t-test between group-averaged ratios across different lags confirms a lack of significant difference between the two analyses (p= 0.50). This finding further emphasizes the genuine asynchronous nature of connectome dynamics across the neural timescales measured in fMRI and electrophysiology. We have added this analysis to the supplementary data.

      Author response image 4.

      On-/off-diagonal ratio is shown across lags for both analyses: 3s window length (blue) and 500ms window length (red). Each bar shows the mean across subjects, while the whiskers show the corresponding standard deviations.

      (2) Try to decrease the lowest frequency of the analysis below 1Hz or just compute it for multiple log-spaced frequencies from infra-slow delta to high-gamma band.

      Thank you for pointing out this matter. We do not expect considerable signal in the frequency range below the current lower bound of delta (1Hz) because as in most other EEG recordings, EEG was not recorded in DC setting and has a hardware high-pass filter of 0.1Hz. Nonetheless, we investigated the power spectral density of our iEEG-fMRI data and found that there is indeed little signal power left in the available infraslow range [0.5 – 1 Hz] after the preprocessing steps (Author response image 5).

      Author response image 5.

      Power spectral density of all subjects in the fMRI-iEEG dataset shows lack of sufficient power in the infraslow range. Infraslow range signals are almost always filtered out during recording unless the recording setup includes a DC amplifier. The infraslow signal of EEG that is often considered correlated with the fMRI signals in the literature most commonly are extracted from the slow-changing envelope of the bandlimited signals, like envelope of gamma oscillations.

      Accordingly, when the iEEG signals are filtered within the range of [0.5, 1], there is little signal variation observed in the signal timeseries, contrasting the adjacent delta band signal (Author response image 6). Importantly, the power envelope of the delta band (and all other canonical bands not shown here) comprise major fluctuations in the infraslow range, as expected. We would like to emphasize that the existing studies addressing infraslow EEG signal dynamics typically consider the infraslow envelope fluctuations of band-limited signals in traditional frequency bands [e.g. Nir et. al, Nat Neurosci 2008] rather than direct recordings in the infraslow frequency range. Investigating HRF-convolved EEG signals similarly captures the infraslow characteristics of the timeseries [e.g. Mantini et al. PNAS 2007, Sadaghiani et al., J Neurosci 2010] (and note that HRF-convolved analyses are included as supplementary investigation in the current study). To the best of our knowledge, very few studies have investigated direct infraslow EEG signals using DC EEG, and we are aware of only two DC-EEG studies with concurrent fMRI [Hiltunen et al., J Neurosci 2014, Grooms et al., Brain Connectivity 2017]. The infraslow correlates of fMRI in electrophysiological signals reported in prior work therefore reflect the slow changes in faster activity or connectivity of traditional frequency bands, which is indeed already included in the current study.

      Author response image 6.

      Sample timeseries of the iEEG signal of the nine subjects (nine rows) for a 400 second interval. Blue signals show the bandlimited delta with its envelope shown as darker blue. The red signal represents the infraslow signal component left in the data, which is much lower in power.

    1. Author response:

      The following is the authors’ response to the original reviews.

      Public Reviews:

      Reviewer #1 (Public Review):

      Ritvo and colleagues present an impressive suite of simulations that can account for three findings of differentiation in the literature. This is important because differentiation-in which items that have some features in common, or share a common associate are less similar to one another than are unrelated items-is difficult to explain with classic supervised learning models, as these predict the opposite (i.e., an increase in similarity). A few of their key findings are that differentiation requires a high learning rate and low inhibitory oscillations, and is virtually always asymmetric in nature.

      This paper was very clear and thoughtful-an absolute joy to read. The model is simple and elegant, and powerful enough to re-create many aspects of existing differentiation findings. The interrogation of the model and presentation of the findings were both extremely thorough. The potential for this model to be used to drive future work is huge. I have only a few comments for the authors, all of which are relatively minor.

      (1) I was struck by the fact that the "zone" of repulsion is quite narrow, compared with the zone of attraction. This was most notable in the modeling of Chanales et al. (i.e., just one of the six similarity levels yielded differentiation). Do the authors think this is a generalizable property of the model or phenomenon, or something idiosyncratic to do with the current investigation? It seems curious that differentiation findings (e.g., in hippocampus) are so robustly observed in the literature despite the mechanism seemingly requiring a very particular set of circumstances. I wonder if the authors could speculate on this point a bit-for example, might the differentiation zone be wider when competitor "pop up" is low (i.e., low inhibitory oscillations), which could help explain why it's often observed in hippocampus? This seems related a bit to the question about what makes something "moderately" active, or how could one ensure "moderate" activation if they were, say, designing an experiment looking at differentiation.

      We thank the reviewer for this comment. In the previous version of the manuscript, in the section entitled “Differentiation Requires a High Learning Rate and Is Sensitive to Activation Dynamics”, we discussed some reasons why differentiation may be more likely to be found in the hippocampus – namely, the high learning rate of the hippocampus and the sparsity of hippocampal activation patterns (pp. 27-28):

      “These results have implications for where to look for differentiation in the brain. Our finding that differentiation requires a high learning rate suggests that differentiation will be more evident in the hippocampus than in neocortex, insofar as hippocampus is thought to have a higher learning rate than neocortex (McClelland et al., 1995). In keeping with this prediction, numerous studies have found differentiation effects in hippocampus but not in neocortical regions involved in sensory processing (e.g., Chanales et al., 2017; Favila et al., 2016; Zeithamova et al., 2018). At the same time, some studies have found differentiation effects in neocortex (e.g., Schlichting et al., 2015; Wammes et al., 2022). One possible explanation of these neocortical differentiation effects is that they are being ``propped up’’ by top-down feedback from differentiated representations in the hippocampus. This explanation implies that disruptions of hippocampal processing (e.g., lesions, stimulation) will eliminate these neocortical differentiation effects; we plan to test this prediction in future work.

      Additionally, the simulations where we adjusted the oscillation amount (using our model of Schlichting et al., 2015) imply that differentiation will be most evident in brain regions where it is relatively hard to activate competitors. Given the U shape of the NMPH learning rule, limiting competitor activity makes it less likely that plasticity will ``cross over'' from weakening (and differentiation) to strengthening (and integration). Thus, within the hippocampus, subregions with sparser activity (e.g., dentate gyrus, and to a lesser extent, CA3; Barnes et al., 1990, GoodSmith et al., 2017; West et al., 1991) will be more prone to differentiation. There is strong empirical support for this prediction. For example, Wammes et al. (2022) manipulated the similarity of stimuli in a statistical learning experiment and found that moderate levels of visual similarity were associated with significant differentiation in the dentate gyrus but not other subregions. Also, numerous studies have found greater differentiation in dentate gyrus / CA3 than in CA1 (e.g., Dimsdale-Zucker et al., 2018; Wanjia et al., 2021; Molitor et al., 2021; Kim et al., 2017; but see Zheng et al., 2021).”

      In the revised draft we have supplemented this discussion with a new section entitled “Reconciling the Prevalence of Differentiation in the Model and in the Data” (pp. 30-31):

      “A key lesson from our model is that, from a computational perspective, it is challenging to obtain differentiation effects: The region of parameter space that gives rise to differentiation is much smaller than the one that gives rise to integration (for further discussion of this issue, see the section in Methods on Practical Advice for Getting the Model to Show Differentiation). However, the fact that integration is more prevalent in our simulations across parameter configurations does not mean that integration will be more prevalent than differentiation in real-life circumstances. What really matters in predicting the prevalence of differentiation in real life is how the parameters of the brain map on to parameters of the model: If the parameters of the brain align with regions of model parameter space that give rise to differentiation (even if these regions are small), this would explain why differentiation has been so robustly observed in extant studies. Indeed, this is exactly the case that we sought to make above about the hippocampus – i.e., that its use of especially sparse coding and a high learning rate will give rise to the kinds of neural dynamics that cause differentiation (as opposed to integration). As another example, while it is true that half of the overlap conditions in our simulation of Chanales et al. (2021) give rise to integration, this does not imply that integration will occur half of the time in the Chanales et al. (2021) study; it may be that the levels of overlap that are actually observed in the brain in Chanales et al. (2021) are more in line with the levels of overlap that give rise to differentiation in our model.”

      (2) With real fMRI data we know that the actual correlation value doesn't matter all that much, and anti-correlations can be induced by things like preprocessing decisions. I am wondering if the important criterion in the model is that the correlations (e.g., as shown in Figure 6) go down from pre to post, versus that they are negative in sign during the post learning period. I would think that here, similar to in neural data, a decrease in correlation would be sufficient to conclude differentiation, but would love the authors' thoughts on that.

      We thank the reviewer for bringing this up. In the paper, we define differentiation as the moving apart of representations – so we agree with the reviewer that it would be appropriate to conclude that differentiation is taking place when correlations go down from pre to post.

      In addition to the definitional question (“what counts as differentiation”), one can also ask the mechanistic question of what is happening in the model at the (simulated) neuronal level in conditions where differentiation (i.e., an average decrease in similarity from pre to post) occurs. Here, the model’s answer is clear: When the similarity of two pairmates decreases, it is because the pairmates have acquired anticorrelated representations at the (simulated) neuronal level. When similarity decreases on average from pre to post, but the average “post” similarity value is not negative, this is because there is a mix of outcomes across runs of the model (due to variance in the initial, random model weights and also variance in the order in which items are presented across training epochs) – some runs lead to differentiation (manifested as anticorrelated pairmate representations) whereas others lead to no change or integration. The average pre-to-post change depends on the relative frequencies with which these different outcomes occur.

      We have made several edits to the paper to clarify this point.

      We added a new section under “Results” in our simulation of Chanales et al. (2021) entitled, “Pairs of Items that Differentiate Show Anticorrelated Representations” (p. 15):

      “Figure 6B also highlights that, for learning rates where robust differentiation effects occur in aggregate (i.e., there is a reduction in mean pattern similarity, averaging across model runs), these aggregate effects involve a bimodal distribution across model runs: For some model runs, learning processes give rise to anticorrelated representations, and for other model runs the model shows integration; this variance across model runs is attributable to random differences in the initial weight configuration of the model. The aggregate differentiation effect is therefore a function of the proportion of model runs showing differentiation (here, anticorrelation) and the proportion of model runs showing integration. The fact that differentiation shows up as anticorrelation in the model's hidden layer relates to the learning effects discussed earlier:

      Unique competitor units are sheared away from (formerly) shared units, so the competitor ends up not having any overlap with the target representation (i.e., the level of overlap is less than you would expect due to chance, which mathematically translates into anticorrelation). We return to this point and discuss how to test for anticorrelation in the Discussion section.”

      We added new text to the “Take-Home Lessons” section in the Chanales et al. (2021) simulation (p. 17):

      “In particular, the simulations expose some important boundary conditions for when representational change can occur according to the NMPH (e.g., that differentiation depends on a large learning rate, but integration does not), and the simulations provide a more nuanced account of exactly how representations change (e.g., that differentiation driven by the NMPH is always asymmetric, whereas integration is sometimes asymmetric and sometimes symmetric; and that, when differentiation occurs on a particular model run, it tends to give rise to anticorrelated representations in the model's hidden layer).”

      We added new text to the “Nature of Representational Change” section in the Favila et al. (2016) simulation (p. 21):

      “Figure 8 - Supplement 1 also indicates that, as in our simulation of Chanales et al. (2021), individual model runs where differentiation occurs show anticorrelation between the pairmate representations, and gradations in the aggregate level of differentiation that is observed across conditions reflect differences in the proportion of trials showing this anticorrelation effect.”

      We added new text to the “Take-Home Lessons” section in the Favila et al. (2016) simulation (p.21):

      “As in our simulation of \cite{chanales2021adaptive}, we found that the NMPH-mediated differentiation was asymmetric, manifested as anticorrelation between pairmate representations on individual model runs, and required a high learning rate, leading to abrupt representational change.”

      We added new text to the “Nature of Representational Change” section in the Schlichting et al. (2015) simulation (p. 26):

      “Also, as in our other simulations, when differentiation occurs on a particular model run it tends to give rise to anticorrelated representations (results not shown).”

      We added new text to the “Take-Home Lessons” section in the Schlichting et al. (2015) simulation (pp. 26-27):

      “As in the other versions of our model, differentiation requires a high learning rate, and – on model runs when it occurs – it is asymmetric and gives rise to anticorrelated representations.”

      We added new text at the start of the Discussion (p. 27):

      “In addition to qualitatively replicating the results from the studies we simulated, our model gives rise to several novel predictions – most notably, that differentiation driven by the NMPH requires a rapid learning rate and, when it occurs for a particular pair of items, it is asymmetric and gives rise to anticorrelated representations.”

      We also added a new section in the Discussion entitled “Testing the Model's Prediction about Anticorrelation”, which (among other things) highlights the reviewer’s point that fMRI pattern similarity values can be affected by preprocessing choices (p. 30):

      “Even though we operationally define differentiation as a reduction in similarity with learning, the way that it actually shows up on individual model runs is as anticorrelation between pairmates; in the model, the size of the aggregate differentiation effect is determined by the proportion of model runs that show this anticorrelation effect (vs. no change or integration). This implies that, if we could get a clean measurement of the similarity of pairmates in an experiment, we might see a multimodal distribution, with some pairmates showing anticorrelation, and others showing increased correlation (integration) or no change in similarity. This kind of clean readout of the similarity of individual pairs might be difficult to obtain with fMRI; it is more feasible that this could be obtained with electrophysiology. Another challenge with using fMRI to test this prediction is that anticorrelation at the individual-neuron level might not scale up to yield anticorrelation at the level of the BOLD response; also, fMRI pattern similarity values can be strongly affected by preprocessing choices – so a negative pattern similarity value does not necessarily reflect anticorrelation at the individual-neuron level. A final caveat is that, while we predict that differentiation will show up as anticorrelation in the brain region that gives rise to the differentiation effect, this might not translate into anticorrelation in areas that are downstream of this region (e.g., if the hippocampus is the source of the differentiation effect, we would expect anticorrelation there, but not necessarily in neocortical regions that receive input from the hippocampus; we revisit this point later in the discussion, when we address limitations and open questions).”

      We added new text in the Discussion, under “Limitations and Open Questions” (p. 31):

      “Importantly, while hippocampus can boost the representation of unique features in neocortex, we expect that neocortex will continue to represent shared perceptual features (e.g., in Favila et al., 2016, the fact that both pairmates are photos of barns). For this reason, in paradigms like the one used by Favila et al. (2016), the predicted effect of hippocampal differentiation on neocortical representations will be a reduction in pattern similarity (due to upregulation in the representation of unique pairmate features) but neocortex should not cross over into anticorrelation in these paradigms (due to its continued representation of shared perceptual features). Indeed, this is exactly the pattern that Wanjia et al. (2021) observed in their study, which used similar stimuli to those used in Favila et al. (2016).”

      Lastly, we updated the Abstract (p. 1)

      “What determines when neural representations of memories move together (integrate) or apart (differentiate)? Classic supervised learning models posit that, when two stimuli predict similar outcomes, their representations should integrate. However, these models have recently been challenged by studies showing that pairing two stimuli with a shared associate can sometimes cause differentiation, depending on the parameters of the study and the brain region being examined. Here, we provide a purely unsupervised neural network model that can explain these and other related findings. The model can exhibit integration or differentiation depending on the amount of activity allowed to spread to competitors – inactive memories are not modified, connections to moderately active competitors are weakened (leading to differentiation), and connections to highly active competitors are strengthened (leading to integration). The model also makes several novel predictions – most importantly, that when differentiation occurs as a result of this unsupervised learning mechanism, it will be rapid and asymmetric, and it will give rise to anticorrelated representations in the region of the brain that is the source of the differentiation. Overall, these modeling results provide a computational explanation for a diverse set of seemingly contradictory empirical findings in the memory literature, as well as new insights into the dynamics at play during learning.”

      (3) For the modeling of the Favila et al. study, the authors state that a high learning rate is required for differentiation of the same-face pairs. This made me wonder what happens in the low learning rate simulations. Does integration occur?

      For the same-face condition of the Favila simulation, lowering learning rate does not result in an overall integration effect:

      Author response image 1.

      In other cases, we do see integration emerge at lower learning rates – e.g., in the Schlichting interleaved condition we see a small integration effect emerge for a learning rate value of 0.3:

      Author response image 2.

      Our view is that, while integration can emerge at low learning rates, it is not a reliable property of the model – in some cases, there is a “window” of learning rates where there is enough learning to drive integration but not enough to drive differentiation, and in other cases there is not. Given this lack of reliability across simulations, we would prefer not to discuss this in the paper.

      This paradigm has a lot of overlap with acquired equivalence, and so I am thinking about whether these are the sorts of small differences (e.g., same-category scenes and perhaps a high learning rate) that bias the system to differentiate instead of integrate.

      We agree that it would be very interesting to use the model to explore acquired equivalence and related phenomena, but we think it is out of scope of the current paper. We have added some text to the Discussion under “Limitations and Open Questions” (p. 32):

      “Another important future direction is to apply the model to a wider range of learning phenomena involving representational change – for example, acquired equivalence, which (like some of the studies modeled here) involves linking distinct stimuli to a shared associate (see, e.g., Honey and Hall, 1989; Shohamy and Wagner, 2008; Myers et al., 2003; Meeter et al., 2009; de Araujo Sanchez and Zeithamova, 2023). It is possible that some of these phenomena might be better explained by supervised learning, or a mixture of unsupervised and supervised learning, than by unsupervised learning alone.”

      (4) For the simulations of the Schlichting et al. study, the A and B appear to have overlap in the hidden layer based on Figure 9, despite there being no similarity between the A and B items in the study (in contrast to Favila et al., in which they were similar kinds of scenes, and Chanales et al., in which they were similar colors). Why was this decision made? Do the effects depend on some overlap within the hidden layer? (This doesn't seem to be explained in the paper that I saw though, so maybe just it's a visualization error?)

      Overlap in the pretrained hidden representations of A and B is not strictly necessary for these effects – it would be possible to reconfigure other parameters to get high levels of competition even if there were no overlap (e.g., by upregulating the strengths of connections from shared input features). Having said that, it is definitely true that overlap between the pretrained hidden representations boosts competition, and we think it is justified to posit this in the Schlichting simulation. We have now added an explanation for this in the paper (p. 23):

      “New text in Schlichting, “Knowledge Built into the Network”

      Matching the previous two simulations, we pretrained the weights so the hidden representations of the stimuli initially had 2/6 units in common. Even though the A and B stimuli used in the actual experiment did not have obvious feature overlap (they were randomly selected novel objects), it is important to note that the hidden layer is not simply a representation of the sensory features of the A and B stimuli; the hidden layer also receives input from the output layer, which represents the shared associate of A and B (X). We think that the presence of this shared associate justifies our use of initially-overlapping hidden representations.”

      (5) It seems as though there were no conditions under which the simulations produced differentiation in both the blocked and intermixed conditions, which Schlichting et al. observed in many regions (as the present authors note). Is there any way to reconcile this difference?

      We thank the reviewer for bringing this up. If we set the connection strength between X (in the output layer) and A (in the hidden layer) in the blocked condition to .9 instead of .999 (keeping this connection strength at .8 for the interleaved condition) and we set Osc to .0615, we observe differentiation in both conditions.

      Rather than replacing the original results in the paper, which would entail re-making the associated videos, etc., we have added a supplementary figure (Figure 10 - Supplement 1), which is included on p. 46.

      We also added the following to the Results section of the Schlichting simulation in the main text (p. 26):

      “Figure 10 - Supplement 1 shows results from an alternative parameterization where, in the low-oscillation-amplitude condition, differentiation is observed in both the blocked and interleaved conditions (mirroring results from Schlichting et al., 2015, who found differentiation in both conditions in several regions of interest, including parts of the hippocampus and medial prefrontal cortex).”

      (6) A general question about differentiation/repulsion and how it affects the hidden layer representation in the model: Is it the case that the representation is actually "shifted" or repelled over so it is no longer overlapping? Or do the shared connections just get pruned, such that the item that has more "movement" in representational space is represented by fewer units on the hidden layer (i.e., is reduced in size)? I think, if I understand correctly, that whether it gets shifted vs. reduce would depend on the strength of connections along the hidden layer, which would in turn depend on whether it represents some meaningful continuous dimension (like color) or not. But, if the connections within the hidden layer are relatively weak and it is the case that representations become reduced in size, would there be any anticipated consequences of this (e.g., cognitively/behaviorally)?

      The representations are shifted – this is discussed in the Chanales results section:

      “Because the activity ``set point'' for the hidden layer (determined by the kWTA algorithm) involves having 6 units active, and the unique parts of the competitor only take up 4 of these 6 units, this leaves room for activity to spread to additional units. Given the topographic projections in the output layer, the model is biased to ``pick up'' units that are adjacent in color space to the currently active units; because activity cannot flow easily from the competitor back to the target (as a result of the aforementioned severing of connections), it flows instead {\em away} from the target, activating two additional units, which are then incorporated into the competitor representation. This sequence of events (first a severing of the shared units, then a shift away from the target) completes the process of neural differentiation, and is what leads to the behavioral repulsion effect in color recall (because the center-of-mass of the color representation has now shifted away from the target).”

      Reviewer #2 (Public Review):

      This paper addresses an important computational problem in learning and memory. Why do related memory representations sometimes become more similar to each other (integration) and sometimes more distinct (differentiation)? Classic supervised learning models predict that shared associations should cause memories to integrate, but these models have recently been challenged by empirical data showing that shared associations can sometimes cause differentiation. The authors have previously proposed that unsupervised learning may account for these unintuitive data. Here, they follow up on this idea by actually implementing an unsupervised neural network model that updates the connections between memories based on the amount of coactivity between them. The goal of the authors' paper is to assess whether such a model can account for recent empirical data at odds with supervised learning accounts. For each empirical finding they wish to explain, the authors built a neural network model with a very simple architecture (two inputs layers, one hidden layer, and one output layer) and with prewired stimulus representations and associations. On each trial, a stimulus is presented to the model, and inhibitory oscillations allow competing memories to pop up. Pre-specified u-shaped learning rules are used to update the weights in the model, such that low coactivity leaves model connections unchanged, moderate coactivity weakens connections, and high coactivity strengthens connections. In each of the three models, the authors manipulate stimulus similarity (following Chanales et al), shared vs distinct associations (following Favila et al), or learning strength (a stand in for blocked versus interleaved learning schedule; following Schlichting et al) and evaluate how the model representations evolve over trials.

      As a proof of principle, the authors succeed in demonstrating that unsupervised learning with a

      simple u-shaped rule can produce qualitative results in line with the empirical reports. For instance, they show that pairing two stimuli with a common associate (as in Favila et al) can lead to *differentiation* of the model representations. Demonstrating these effects isn't trivial and a formal modeling framework for doing so is a valuable contribution. Overall, the authors do a good job of both formally describing their model and giving readers a high level sense of how their critical model components work, though there are some places where the robustness of the model to different parameter choices is unclear. In some cases, the authors are very clear about this (e.g. the fast learning rate required to observe differentiation). However, in other instances, the paper would be strengthened by a clearer reporting of the critical parameter ranges.

      We thank the reviewer for raising this point. The interdependence of parameters in our model makes it infeasible to identify critical parameter ranges. We have added a paragraph to the “Approach to Parameterization and Data Fitting” section in the Methods to address this point (p. 33):

      “The overall goal of this modeling work is to account for key empirical regularities regarding differentiation and integration and to establish boundary conditions on these regularities. As such, the modeling work described below focuses more on qualitative fits to general properties of the data space than on quantitative fits to results from specific studies. Automatic parameter optimization is not feasible for this kind of model, given the large number of model parameters and the highly interactive, nonlinear nature of competitive dynamics in the model; consequently, model fitting was done by hand.

      These complex interactions between parameters also make it infeasible to list “critical parameter ranges” for generating particular model outcomes. Our experience in working with the model has been that activation dynamics are what matter most for learning, and that disparate parameter sets can give rise to the same activation dynamics and -- through this -- the same learning effects; likewise, similar parameter sets can give rise to different activation dynamics and different learning outcomes. Consequently, in this paper we have focused on characterizing the dynamics that give rise to different learning effects (and how they can be affected by local parameter perturbations, e.g., relating to learning rate and oscillation size), rather than the – impossible, we believe – task of enumerating the full set of parameter configurations that give rise to a particular result.”

      For instance, it's clear from the manipulation of oscillation strength in the model of Schlichting et al that this parameter can dramatically change the direction of the results. The authors do report the oscillation strength parameter values that they used in the other two models, but it is not clear how sensitive these models are to small changes in this value.

      In some cases, the effects of oscillation strength are relatively smooth. For example, in the Favila simulation, increasing the oscillation amplitude Osc effectively recapitulates the U-shaped curve (i.e., higher levels of Osc lead to more competitor activation, which initially leads to weakening / differentiation but then gives way to strengthening / integration), as is shown for the Favila Different Face condition in this plot:

      Author response image 3.

      In the Chanales 2/6 overlap condition, the effects of varying Osc are more nonlinear:

      Author response image 4.

      We think this is attributable to the increased “all-or-none” recurrent dynamics in this simulation (due to the recurrent projections within the output layer), which make it more difficult to evoke moderate (vs. high) levels of activation. This difficulty in reliably obtaining graded activation dynamics is likely a consequence of the small-scale (“toy”) nature of the model and the simple inhibitory mechanisms employed here, as opposed to being a generalizable property of the brain – presumably, the actual brain employs more nuanced and effective means of controlling activation. Furthermore, we don’t think that the high prevalence of integration in the model’s parameter space necessarily translates into a prediction that integration should be more prevalent overall – see the new “Reconciling the Prevalence of Differentiation in the Model and in the Data” section described in response to one of the reviewer’s other points below. Due to the paper already being quite long, we have opted not to include the above plots / discussion in the paper.

      Similarly, it's not clear whether the 2/6 hidden layer overlap (only explicitly manipulated in the model of Chanales et al) is required for the other two models to work.

      When we were parameterizing the model, we opted to keep the 2/6 level of overlap for all of the simulations and we adjusted other parameters to fit the data; in part, this was because overlap can only be adjusted in discrete jumps, whereas other influential parameters in the model can be adjusted in a more graded, real-valued way. Our use of 2/6 overlap (as opposed to, say, 1/6 or 3/6 overlap) for the Favila and Schlichting models was done out of convenience, and should not be interpreted as a strong statement that this particular level of overlap is necessary for obtaining differentiation; we could easily get the model to show differentiation given other overlap levels by adjusting other parameters.

      Finally, though the u-shaped learning rule is essential to this framework, the paper does little formal investigation of this learning rule. It seems obvious that allowing the u-shape to collapse too much toward a horizontal line would reduce the model's ability to account for empirical results, but there may be other more interesting features of the learning rule parameterization that are essential for the model to function properly.

      Given that the paper is already quite long, we have opted not to include further exploration of the parameters of the U-shaped learning rule in the paper. However, for the reviewer’s information, we report the effects of a few illustrative manipulations of these parameters below. As a general principle, the effects of these manipulations make sense in light of the theoretical framework described in the paper.

      For example, the parameter “DRevMag” controls the size of the negative “dip” in the U-shaped curve (more negative values = a larger dip). Given that this negative dip is essential for severing weights to competitors and causing differentiation, shifting DRevMag upwards towards zero should shift the balance of the model away from differentiation and towards integration. This is indeed what we observe, as shown in this parameter sweep from the Chanales simulation:

      Author response image 5.

      As another example: The “DRev” parameter controls where the U-shaped curve transitions from negative weight change to positive weight change. Lower values of DRev mean that the region of coactivity values leading to negative weight change will be smaller, and the region of coactivity values leading to positive weight change will be larger. As such, we would expect that lower values of DRev would bias the model toward integration. That is indeed the case, as shown in this parameter sweep from the Schlichting Blocked simulation:

      Author response image 6.

      There are a few other points that may limit the model's ability to clearly map onto or make predictions about empirical data. The model(s) seems very keen to integrate and do so more completely than the available empirical data suggest. For instance, there is a complete collapse of representations in half of the simulations in the Chanales et al model and the blocked simulation in the Schlichting et al model also seems to produce nearly complete integration Even if the Chanales et al paper had observed some modest behavioral attraction effects, this model would seem to over-predict integration. The author's somewhat implicitly acknowledge this when they discuss the difficulty of producing differentiation ("Practical Advice for Getting the Model to Show Differentiation") and not of producing integration, but don't address it head on.

      We thank the reviewer for this comment – R1 had a similar comment. We have added a new section to the Discussion to address this point (p. 30):

      “Reconciling the Prevalence of Differentiation in the Model and in the Data.

      A key lesson from our model is that, from a computational perspective, it is challenging to obtain differentiation effects: The region of parameter space that gives rise to differentiation is much smaller than the one that gives rise to integration (for further discussion of this issue, see the section in Methods on Practical Advice for Getting the Model to Show Differentiation). However, the fact that integration is more prevalent in our simulations across parameter configurations does not mean that integration will be more prevalent than differentiation in real-life circumstances. What really matters in predicting the prevalence of differentiation in real life is how the parameters of the brain map on to parameters of the model: If the parameters of the brain align with regions of model parameter space that give rise to differentiation (even if these regions are small), this would explain why differentiation has been so robustly observed in extant studies. Indeed, this is exactly the case that we sought to make above about the hippocampus – i.e., that its use of especially sparse coding and a high learning rate will give rise to the kinds of neural dynamics that cause differentiation (as opposed to integration). As another example, while it is true that half of the overlap conditions in our simulation of Chanales et al. (2021) give rise to integration, this does not imply that integration will occur half of the time in the Chanales et al. (2021) study; it may be that the levels of overlap that are actually observed in the brain in Chanales et al. (2021) are more in line with the levels of overlap that give rise to differentiation in our model.”

      Second, the authors choice of strongly prewiring associations in the Chanales and Favila models makes it difficult to think about how their model maps onto experimental contexts where competition is presumably occurring while associations are only weakly learned. In the Chanales et al paper, for example, the object-face associations are not well learned in initial rounds of the color memory test. While the authors do justify their modeling choice and their reasons have merit, the manipulation of AX association strength in the Schlichting et al model also makes it clear that the association strength has a substantial effect on the model output. Given the effect of this manipulation, more clarity around this assumption for the other two models is needed.

      We thank the reviewer for bringing this up. We have edited the section entitled “A Note on Prewiring Representations” in the Methods to further justify our choice to prewire associations in the Chanales and Favila models (p. 37):

      “In our model, our practice of ``prewiring'' memory representations for the A and B pairmates serves two functions. In some cases, it is meant to stand in for actual training (as in the blocked / interleaved manipulation; the connections supporting the AX association are prewired to be stronger in the blocked condition than in the interleaved condition). However, the other, more fundamental role of prewiring is to ensure that the A and B input patterns evoke sparse distributed representations in the hidden layer (i.e., where some units are strongly active but most other units are inactive). In the real brain, this happens automatically because the weight landscape has been extensively sculpted by both experience and evolution. For example, in the real hippocampus, when the second pairmate is presented for the first time, it will evoke a sparse distributed representation in the CA3 subfield (potentially overlapping with the first pairmate’s CA3 representation) even before any learning of the second pairmate has occurred, due to the strong, sparse mossy fiber projections that connect the dentate gyrus to CA3 (McNaughton & Morris, 1987). As discussed above, we hypothesize that this initial, partial overlap between the second pairmate’s representation and the first pairmate’s representation can lead to pop-up of the unique features of the first pairmate’s representation, triggering learning that leads to differentiation or integration. In our small-scale model, we are effectively starting with a ``blank brain''; in the absence of prewiring, the A and B inputs would activate overly diffuse representations that do not support these kinds of competitive dynamics. As such, prewiring in our model is necessary for proper functioning. The presence of prewired A and B representations should therefore not be interpreted as reflecting a particular training history (except in the blocked / interleaved case above); rather, these prewired representations constitute the minimum step we would take to ensure well-defined competitive dynamics in our small-scale model.

      The fact that connection strengths serve this dual function – sometimes reflecting effects of training (as in our simulation of Schlichting et al., 2015) and in other cases reflecting necessary prewiring – complicates the interpretation of these strength values in the model. Our view is that this is a necessary limitation of our simplified modeling approach – one that can eventually be surmounted through the use of more biologically-detailed architectures (see Limitations and Open Questions in the Discussion).”

      Overall, this is strong and clearly described work that is likely to have a positive impact on computational and empirical work in learning and memory. While the authors have written about some of the ideas discussed in this paper previously, a fully implemented and openly available model is a clear advance that will benefit the field. It is not easy to translate a high-level description of a learning rule into a model that actually runs and behaves as expected. The fact that the authors have made all their code available makes it likely that other researchers will extend the model in numerous interesting ways, many of which the authors have discussed and highlighted in their paper.

      Reviewer #3 (Public Review):

      This paper proposes a computational account for the phenomenon of pattern differentiation (i.e., items having distinct neural representations when they are similar). The computational model relies on a learning mechanism of the nonmonotonic plasticity hypothesis, fast learning rate and inhibitory oscillations. The relatively simple architecture of the model makes its dynamics accessible to the human mind. Furthermore, using similar model parameters, this model produces simulated data consistent with empirical data of pattern differentiation. The authors also provide insightful discussion on the factors contributing to differentiation as opposed to integration. The authors may consider the following to further strengthen this paper:

      The model compares different levels of overlap at the hidden layer and reveals that partial overlap seems necessary to lead to differentiation. While I understand this approach from the perspective of modeling, I have concerns about whether this is how the human brain achieves differentiation. Specifically, if we view the hidden layer activation as a conjunctive representation of a pair that is the outcome of encoding, differentiation should precede the formation of the hidden layer activation pattern of the second pairmate. Instead, the model assumes such pattern already exists before differentiation. Maybe the authors indeed argue that mechanistically differentiation follows initial encoding that does not consider similarity with other memory traces?

      Related to the point above, because the simulation setup is different from how differentiation actually occurs, I wonder how valid the prediction of asymmetric reconfiguration of hidden layer connectivity pattern is.

      We thank the reviewer for this comment. In the revised manuscript, we have edited the “Note on Prewiring Representations” in the Methods to clarify how our assumptions about prewiring relate to what we really think is happening in the brain (p. 37):

      “In our model, our practice of ``prewiring'' memory representations for the A and B pairmates serves two functions. In some cases, it is meant to stand in for actual training (as in the blocked / interleaved manipulation; the connections supporting the AX association are prewired to be stronger in the blocked condition than in the interleaved condition). However, the other, more fundamental role of prewiring is to ensure that the A and B input patterns evoke sparse distributed representations in the hidden layer (i.e., where some units are strongly active but most other units are inactive). In the real brain, this happens automatically because the weight landscape has been extensively sculpted by both experience and evolution. For example, in the real hippocampus, when the second pairmate is presented for the first time, it will evoke a sparse distributed representation in the CA3 subfield (potentially overlapping with the first pairmate’s CA3 representation) even before any learning of the second pairmate has occurred, due to the strong, sparse mossy fiber projections that connect the dentate gyrus to CA3 (McNaughton & Morris, 1987). As discussed above, we hypothesize that this initial, partial overlap between the second pairmate’s representation and the first pairmate’s representation can lead to pop-up of the unique features of the first pairmate’s representation, triggering learning that leads to differentiation or integration. In our small-scale model, we are effectively starting with a ``blank brain''; in the absence of prewiring, the A and B inputs would activate overly diffuse representations that do not support these kinds of competitive dynamics. As such, prewiring in our model is necessary for proper functioning. The presence of prewired A and B representations should therefore not be interpreted as reflecting a particular training history (except in the blocked / interleaved case above); rather, these prewired representations constitute the minimum step we would take to ensure well-defined competitive dynamics in our small-scale model.

      The fact that connection strengths serve this dual function – sometimes reflecting effects of training (as in our simulation of Schlichting et al., 2015) and in other cases reflecting necessary prewiring – complicates the interpretation of these strength values in the model. Our view is that this is a necessary limitation of our simplified modeling approach – one that can eventually be surmounted through the use of more biologically-detailed architectures (see Limitations and Open Questions in the Discussion).”

      Although as the authors mentioned, there haven't been formal empirical tests of the relationship between learning speed and differentiation/integration, I am also wondering to what degree the prediction of fast learning being necessary for differentiation is consistent with current data. According to Figure 6, the learning rates lead to differentiation in the 2/6 condition achieved differentiation after just one-shot most of the time. On the other hand, For example, Guo et al (2021) showed that humans may need a few blocks of training and test to start showing differentiation.

      We thank the reviewer for mentioning this. We have added a paragraph to the “Differentiation Requires a High Learning Rate and Is Sensitive to Activity Dynamics” section of the Discussion that addresses this point (pp. 28-29):

      “Although the results from Wanjia et al. (2021) provide strong support for the model's prediction that differentiation will be abrupt, they raise another question: What explains variance across items in when this abrupt change takes place? The answer to this question remains to be seen, but one possibility is encoding variability: If we assume that participants stochastically sample (i.e., attend to) the features of the scene pairmates, it is possible that participants might initially fail to sample the features that distinguish the scene pairmates, which can be quite subtle – and if the distinguishing features of the pairmates are not represented in high-level visual regions (i.e., the pairmates are represented in these regions as having the same features), this could delay the onset of differentiation until the point at which the distinguishing features happen (by chance) to be sampled.”

      Related to the point above, the high learning rate prediction also seems to be at odds with the finding that the cortex, which has slow learning (according to the theory of complementary learning systems), also shows differentiation in Wammes et al (2022).

      We now address this point in the section of the Discussion entitled “Differentiation Requires a High Learning Rate and Is Sensitive to Activity Dynamics” (p. 27):

      “Our finding that differentiation requires a high learning rate suggests that differentiation will be more evident in the hippocampus than in neocortex, insofar as hippocampus is thought to have a higher learning rate than neocortex (McClelland et al., 1995). In keeping with this prediction, numerous studies have found differentiation effects in hippocampus but not in neocortical regions involved in sensory processing (e.g., Chanales et al., 2017; Favila et al., 2016; Zeithamova et al., 2018). At the same time, some studies have found differentiation effects in neocortex (e.g., Schlichting et al., 2015; Wammes et al., 2022). One possible explanation of these neocortical differentiation effects is that they are being ``propped up’’ by top-down feedback from differentiated representations in the hippocampus.”

      More details about the learning dynamics would be helpful. For example, equation(s) showing how activation, learning rate and the NMPH function work together to change the weight of connections may be added. Without the information, it is unclear how each connection changes its value after each time point.

      We thank the reviewer for this comment. We have made two major changes to address this concern. First, we have edited the “Learning” section within “Basic Network Properties” in the main text (pp. 6-7):

      “Connection strengths in the model between pairs of connected units x and y were adjusted at the end of each trial (i.e., after each stimulus presentation) as a U-shaped function of the coactivity of x and y, defined as the product of their activations on that trial. The parameters of the U-shaped learning function relating coactivity to change in connection strength (i.e., weakening / strengthening) were specified differently for each projection where learning occurs (bidirectionally between the input and hidden layers, the hidden layer to itself, and the hidden to output layer). Once the U-shaped learning function for each projection in each version of the model was specified, we did not change it for any of the various conditions. Details of how we computed coactivity and how we specified the U-shaped function can be found in the Methods section.”

      Second, we have added the requested equations to the “Learning” part of the Methods (pp. 37-38):

      The right side of the function, strong activation leads to strengthening of the connectivity, which I assume will lead to stronger activation on the next time point. The model has an upper limit of connection strength to prevent connection from strengthening too much. The same idea can be applied to the left side of the function: instead of having two turning points, it can be a linear function such that low activation keeps weakening connection until the lower limit is reached. This way the NMPH function can take a simpler form (e.g., two line-segments if you think the weakening and strengthening take different rates) and may still simulate the data.

      We thank the reviewer for mentioning this. We have added a new paragraph in the “Learning” section of the Methods to justify the particular shape of the learning curve (pp. 38-39):

      “Evidence for the U-shaped plasticity function used here (where low activation leads to no change, moderate activation leads to weakening, and higher levels of activation lead to strengthening) was previously reviewed in Ritvo et al. (2019). In brief, there are three lines of work that support the U shape: First, multiple neurophysiological studies have found that moderate postsynaptic depolarization leads to synaptic weakening and higher levels of depolarization lead to synaptic strengthening (e.g., Artola et al., 1990; Hansel et al., 1996). Second, human neuroscience studies have used pattern classifiers, applied to fMRI and EEG data, to measure memory activation, and have related this measure to subsequent memory accessibility; several studies using this approach have found that low levels of activation lead to no change in memory strength, moderate levels of activation lead to impaired subsequent memory, and higher levels of activation lead to increased subsequent memory (e.g., Newman and Norman, 2010; Detre et al., 2013; Kim et al., 2014; for related findings, see Lewis-Peacock and Norman, 2014; Wang et al., 2019). Third, a recent human fMRI study by Wammes et al. (2022) manipulated memory activation by varying the visual similarity of pairmates and observed a U-shaped function relating visual similarity to representational change in the hippocampus, whereby low levels of pairmate similarity were associated with no change, moderate levels of similarity were associated with differentiation, and the differentiation effect went away at higher levels of similarity.

      We have also included a pointer to this new paragraph in the “Nonmonotonic Plasticity Hypothesis” section of Introduction (p. 2):

      (for further discussion of the empirical justification for the NMPH, see the Learning subsection in the Methods)”

      Recommendations for the authors:

      Reviewer #1 (Recommendations For The Authors):

      A few additional minor things about data presentation and the like:

      (1) Figure 1 legend - a more general description of how to interpret the figure might be helpful for more naive readers (e.g., explaining how one can visualize in the schematic that there is overlap in the hidden layer between A and B). Also, from the Figure 1 depiction, it's not clear what is different about the setup from the initial left hand side panels in A, B, C, to make it such that activity spreads strongly to A in panel A, weakly in panel B, and not at all in panel C since the weights are the same. Is there a way to incorporate this into the graphic, or describe it in words?

      To address this point, we have added the following text to the Figure 1 caption (p. 3):

      “Note that the figure illustrates the consequences of differences in competitor activation for learning, without explaining why these differences would arise. For discussion of circumstances that could lead to varying levels of competitor activation, see the simulations described in the text.”

      (2) I believe not all of the papers cited on lines 193-195 actually have similarity manipulations in them. I'd recommend double checking this list and removing those less relevant to the statement.

      Thank you for pointing this out; we have removed the Ballard reference and we have clarified what we mean by similarity reversal (p. 7):

      “The study was inspired by recent neuroimaging studies showing ``similarity reversals'', wherein stimuli that have more features in common (or share a common associate) show less hippocampal pattern similarity (Favila et al., 2016; Schlichting et al., 2015; Molitor et al., 2021; Chanales et al., 2017; Dimsdale-Zucker et al., 2018; Wanjia et al., 2021; Zeithamova et al., 2018; Jiang et al., 2020; Wammes et al., 2022).”

      (3) I wanted a bit more detail about how the parameters were set in the main paper, not just in the methods. Even something as brief as noting that model fitting was done by hand by tweaking parameters to re-create the empirical patterns (if I'm understanding correctly) would have been helpful for me.

      To address this point, we have added the following text under “Basic Network Properties” (p. 4):

      “Our goal was to qualitatively fit key patterns of results from each of the aforementioned studies. We fit the parameters of the model by hand as they are highly interdependent (see the Methods section for more details).”

      (4) In Figure 4E, it would be helpful to describe the x and y axes of the MDS plots in the legend.

      To address this point, we have added the following new text to the Figure 4 caption that clarifies how the MDS plots were generated (p. 11):

      “MDS plots were rotated, shifted, and scaled such that pairmate 1before is located at (0,0), pairmate 2before is located directly to the right of pairmate 1before, and the distance between pairmate 1before and pairmate 2before is proportional to the baseline distance between the pairmates.”

      (5) Figure 6 - at first I thought the thicker line was some sort of baseline, but I think it is just many traces on top of one another. If other readers may be similarly confused, perhaps this could be stated.

      Thanks for this comment. We have updated Figure 6 (p. 16).

      We have also updated the caption.

      I am having a lot of difficulty understanding the terms "competitor-to-competitor,"

      "competitor-to-target/shared," and "target/shared-to-target/shared," and therefore I don't fully get Figure 5. I think it might be helpful to expand the description of these terms where they are first introduced in the paper (p. 13?). I think I am missing something crucial here, and I am not quite sure what that is-which I know is not very helpful! But, to narrate my confusion a bit, I thought that these terms would somehow relate to connections between different connections of the network. For example is competitor-to-competitor within the hidden layer? Or is this somehow combining across relevant connections that might span different pairs of layers in the model? And, I really have no idea why it is "target/shared."

      Thank you for these comments. We have updated Figure 5 and we have also made several changes to the main text and the figure caption to address these points.

      Changes to the main text (p. 13):

      “Whether symmetric or asymmetric integration occurs depends on the relative strengths of connections between pairs of unique competitor units (competitor-competitor connections) compared to connections between unique competitor units and shared units (competitor-shared connections) after the first trial (Figure 5; note that the figure focuses on connections between hidden units, but the principle also applies to connections that span across layers). Generally, coactivity between unique competitor units (competitor-competitor coactivity) is less than coactivity between unique competitor units and shared units (competitor-shared coactivity), which is less than coactivity between unique target units and shared units (target-shared coactivity).”

      (7) Relatedly in Figure 13, I understand how some competitor-to-target/shared connections could be spared in the bottom instance given panel B. However, I'm struggling to understand how that relates to the values in the corresponding chart in panel A. What about panel A, bottom (vs. the top) means lower coactivities between some competitor-to-target/shared? Is it because if the noise level is higher, the "true" activation of competitor-to-target/shared connections is weaker? I think again, I'm missing something critical here! and wonder if other readers may be in the same situation. (I know the authors described this also on p. 36, but I'm still confused!)

      We have updated Figure 13 to clarify these points.

      (8)  In Figure 9, I believe there is no caption for panel D. Also, it looks as though the item unit active for A and B is the same. I wonder if this is an error?

      Thank you for catching these errors! They have both been fixed.

      Reviewer #2 (Recommendations For The Authors):

      -Perhaps I missed it, but I think defining coactivity (how it is computed) in the main text would be useful for readers, as this is critical for understanding the model. I did find it in the methods.

      We thank the reviewer for this suggestion. We have updated the “Learning” section within “Basic Network Properties” in the main text to address this point (pp. 6-7):

      “Connection strengths in the model between pairs of connected units x and y were adjusted at the end of each trial (i.e., after each stimulus presentation) as a U-shaped function of the coactivity of x and y, defined as the product of their activations on that trial. The parameters of the U-shaped learning function relating coactivity to change in connection strength (i.e., weakening / strengthening) were specified differently for each projection where learning occurs (bidirectionally between the input and hidden layers, the hidden layer to itself, and the hidden to output layer). Once the U-shaped learning function for each projection in each version of the model was specified, we did not change it for any of the various conditions. Details of how we computed coactivity and how we specified the U-shaped function can be found in the Methods section.”

      -The modeling results in the different face condition are at odds with the data for the Favila et al model (they observe some differentiation in the paper and the model predicts no change). This could be due to a number of unmodeled factors, but it is perhaps worth noting.

      Thank you for pointing this out. It is possible to better capture the pattern of results observed by Favila et al. in their paper (with some differentiation in the different-face condition and even more differentiation in the same-face condition) by slightly adjusting the model parameters (specifically, by setting the oscillation amplitude Osc for the hidden layer to .1 instead of .067).

      Rather than replacing the old (Osc \= .067) results in the paper, which would entail re-making the associated videos, etc., we have added a supplementary figure (Figure 8 - Supplement 1; see p.45):

      We also added new text to the Favila Results, under “Differentiation and Integration” (p. 20):

      “Note also that the exact levels of differentiation that are observed in the different-face and same-face conditions are parameter dependent; for an alternative set of results showing some differentiation in the different-face condition (but still less than is observed in the same-face condition), see Figure 8 - Supplement 1.”

      -Related to my comment in the public review about pre-wiring associations, in the caption for Figure 9 (Schlichting model), the authors report "In both conditions, the pre-wired connection linking the "item B" hidden units to the "item X" output unit is set to .7. In the interleaved condition, the connection linking the "item A" hidden units to the "item X" output unit is set to .8, to reflect some amount of initial AX learning. In the blocked condition, the connection linking the "item A" hidden units to the "item X" output unit is set a higher value (.999), to reflect extra AX learning." What are the equivalent values for the other models, especially the Favila model since the structure is the same as Schlichting? I understood all the "strong" connections to be .99 unless otherwise stated. If that's the case, I don't understand why the blocked Schlichting model and the Favila model produce opposite effects. More clarity would be useful here.

      We have added a new paragraph to the results section for the Schlicting model (under “Differentiation and Integration”) to clarify why the blocked Schlichting model and the Favila model show different results (p. 24):

      “Note that the key feature driving integration in the blocked condition of this simulation is not the high strength of the connection from X to A on its own – rather, it is the asymmetry in the pretrained connection strengths from X to A (.999) and from X to B (.7). This asymmetry, which is meant to reflect the extensive training on A-X that occurred before the initial presentation of B-X, results in the A-X hidden representation decisively winning the competition during B-X presentation, which then leads to the B input also being linked to this representation (i.e., integration). It is instructive to compare this to the same-face condition from our simulation of Favila et al. (2016): In that simulation, the two pairmates are also linked strongly (.99 initial connection strength) to a shared associate, but in that case the connections are equally strong, so there is more balanced competition -- in this case, the competitor representation only comes to mind moderately (instead of displacing the target representation), so the result is differentiation instead of integration.”

      -The meaning of the different colored dots in Figure 5 is bit hard to keep track of, even given the legend labels. The figure might benefit from a model sketch highlighting each of the different coactivity types. The left side of Fig 13 was useful but again somehow mapping on the colors would help further. Another note on these figures: what does having two dots of each color mean? Is it just an illustration of the variance? There would be more dots if there was one dot per coactivity value.

      We have updated Figure 5 and Figure 13 to clarify these points (including a clarification that the dots only represent a subset of the possible pairings between units).

      -While I appreciate the goal of the paper is to account for these three studies, readers who aren't familiar with or specifically interested in these studies may appreciate a small amount of intuition on why formalizing unsupervised learning models may be broadly important for computational investigations of learning/memory/cognition.

      We have added the following text under “Basic Network Properties” in the Introduction to address this point (p. 4):

      “Achieving a better understanding of unsupervised learning is an important goal for computational neuroscience, given that learning agents have vastly more opportunities to learn in an unsupervised fashion than from direct supervision (for additional discussion of this point, see, e.g., Zhuang et al., 2021).”

    1. Author response:

      The following is the authors’ response to the original reviews.

      Public Reviews:

      Reviewer #1 (Public Review):

      Summary:

      This paper presents a compelling and comprehensive study of decision-making under uncertainty. It addresses a fundamental distinction between belief-based (cognitive neuroscience) formulations of choice behaviour with reward-based (behavioural psychology) accounts. Specifically, it asks whether active inference provides a better account of planning and decision-making, relative to reinforcement learning. To do this, the authors use a simple but elegant paradigm that includes choices about whether to seek both information and rewards. They then assess the evidence for active inference and reinforcement learning models of choice behaviour, respectively. After demonstrating that active inference provides a better explanation of behavioural responses, the neuronal correlates of epistemic and instrumental value (under an optimised active inference model) are characterised using EEG. Significant neuronal correlates of both kinds of value were found in sensor and source space. The source space correlates are then discussed sensibly, in relation to the existing literature on the functional anatomy of perceptual and instrumental decision-making under uncertainty.

      Strengths:

      The strengths of this work rest upon the theoretical underpinnings and careful deconstruction of the various determinants of choice behaviour using active inference. A particular strength here is that the experimental paradigm is designed carefully to elicit both information-seeking and reward-seeking behaviour; where the information-seeking is itself separated into resolving uncertainty about the context (i.e., latent states) and the contingencies (i.e., latent parameters), under which choices are made. In other words, the paradigm - and its subsequent modelling - addresses both inference and learning as necessary belief and knowledge-updating processes that underwrite decisions.

      The authors were then able to model belief updating using active inference and then look for the neuronal correlates of the implicit planning or policy selection. This speaks to a further strength of this study; it provides some construct validity for the modelling of belief updating and decision-making; in terms of the functional anatomy as revealed by EEG. Empirically, the source space analysis of the neuronal correlates licences some discussion of functional specialisation and integration at various stages in the choices and decision-making.

      In short, the strengths of this work rest upon a (first) principles account of decision-making under uncertainty in terms of belief updating that allows them to model or fit choice behaviour in terms of Bayesian belief updating - and then use relatively state-of-the-art source reconstruction to examine the neuronal correlates of the implicit cognitive processing.

      Response: We are deeply grateful for your careful review of our work and for the thoughtful feedback you have provided. Your dedication to ensuring the quality and clarity of the work is truly admirable. Your comments have been invaluable in guiding us towards improving the paper, and We appreciate your time and effort in not just offering suggestions but also providing specific revisions that I can implement. Your insights have helped us identify areas where I can strengthen the arguments and clarify the methodology.

      Comment 1:

      The main weaknesses of this report lies in the communication of the ideas and procedures. Although the language is generally excellent, there are some grammatical lapses that make the text difficult to read. More importantly, the authors are not consistent in their use of some terms; for example, uncertainty and information gain are sometimes conflated in a way that might confuse readers. Furthermore, the descriptions of the modelling and data analysis are incomplete. These shortcomings could be addressed in the following way.

      First, it would be useful to unpack the various interpretations of information and goal-seeking offered in the (active inference) framework examined in this study. For example, it will be good to include the following paragraph:

      "In contrast to behaviourist approaches to planning and decision-making, active inference formulates the requisite cognitive processing in terms of belief updating in which choices are made based upon their expected free energy. Expected free energy can be regarded as a universal objective function, specifying the relative likelihood of alternative choices. In brief, expected free energy can be regarded as the surprise expected following some action, where the expected surprise comes in two flavours. First, the expected surprise is uncertainty, which means that policies with a low expected free energy resolve uncertainty and promote information seeking. However, one can also minimise expected surprise by avoiding surprising, aversive outcomes. This leads to goal-seeking behaviour, where the goals can be regarded as prior preferences or rewarding outcomes.

      Technically, expected free energy can be expressed in terms of risk plus ambiguity - or rearranged to be expressed in terms of expected information gain plus expected value, where value corresponds to (log) prior preferences. We will refer to both decompositions in what follows; noting that both decompositions accommodate information and goal-seeking imperatives. That is, resolving ambiguity and maximising information gain have epistemic value, while minimising risk or maximising expected value have pragmatic or instrumental value. These two kinds of values are sometimes referred to in terms of intrinsic and extrinsic value, respectively [1-4]."

      Response 1: We deeply thank you for your comments and corresponding suggestions about our interpretations of active inference. In response to your identified weaknesses and suggestions, we have added corresponding paragraphs in the Methods section (The free energy principle and active inference, line 95-106):

      “Active inference formulates the necessary cognitive processing as a process of belief updating, where choices depend on agents' expected free energy. Expected free energy serves as a universal objective function, guiding both perception and action. In brief, expected free energy can be seen as the expected surprise following some policies. The expected surprise can be reduced by resolving uncertainty, and one can select policies with lower expected free energy which can encourage information-seeking and resolve uncertainty. Additionally, one can minimize expected surprise by avoiding surprising or aversive outcomes (oudeyer et al., 2007; Schmidhuber et al., 2010). This leads to goal-seeking behavior, where goals can be viewed as prior preferences or rewarding outcomes.

      Technically, expected free energy can also be expressed as expected information gain plus expected value, where the value corresponds to (log) prior preferences. We will refer to both formulations in what follows. Resolving ambiguity, minimizing risk, and maximizing information gain has epistemic value while maximizing expected value have pragmatic or instrumental value. These two types of values can be referred to in terms of intrinsic and extrinsic value, respectively (Barto et al., 2013; Schwartenbeck et al., 2019).”

      Oudeyer, P. Y., & Kaplan, F. (2007). What is intrinsic motivation? A typology of computational approaches. Frontiers in neurorobotics, 1, 108.

      Schmidhuber, J. (2010). Formal theory of creativity, fun, and intrinsic motivation (1990–2010). IEEE transactions on autonomous mental development, 2(3), 230-247.

      Barto, A., Mirolli, M., & Baldassarre, G. (2013). Novelty or surprise?. Frontiers in psychology, 4, 61898.

      Schwartenbeck, P., Passecker, J., Hauser, T. U., FitzGerald, T. H., Kronbichler, M., & Friston, K. J. (2019). Computational mechanisms of curiosity and goal-directed exploration. elife, 8, e41703.

      Comment 2:

      The description of the modelling of choice behaviour needs to be unpacked and motivated more carefully. Perhaps along the following lines:

      "To assess the evidence for active inference over reinforcement learning, we fit active inference and reinforcement learning models to the choice behaviour of each subject. Effectively, this involved optimising the free parameters of active inference and reinforcement learning models to maximise the likelihood of empirical choices. The resulting (marginal) likelihood was then used as the evidence for each model. The free parameters for the active inference model scaled the contribution of the three terms that constitute the expected free energy (in Equation 6). These coefficients can be regarded as precisions that characterise each subjects' prior beliefs about contingencies and rewards. For example, increasing the precision or the epistemic value associated with model parameters means the subject would update her beliefs about reward contingencies more quickly than a subject who has precise prior beliefs about reward distributions. Similarly, subjects with a high precision over prior preferences or extrinsic value can be read as having more precise beliefs that she will be rewarded. The free parameters for the reinforcement learning model included..."

      Response 2: We deeply thank you for your comments and corresponding suggestions about our description of the behavioral modelling. In response to your identified weaknesses and suggestions, we have added corresponding content in the Results section (Behavioral results, line 279-293):

      “To assess the evidence for active inference over reinforcement learning, we fit active inference (Eq.9), model-free reinforcement learning, and model-based reinforcement learning models to the behavioral data of each participant. This involved optimizing the free parameters of active inference and reinforcement learning models. The resulting likelihood was used to calculate the Bayesian Information Criterion (BIC) (Vrieze 2012) as the evidence for each model. The free parameters for the active inference model (AL, AI, EX, prior, and α) scaled the contribution of the three terms that constitute the expected free energy in Eq.9. These coefficients can be regarded as precisions that characterize each participant's prior beliefs about contingencies and rewards. For example, increasing α means participants would update their beliefs about reward contingencies more quickly, increasing AL means participants would like to reduce ambiguity more, and increasing AI means participants would like to learn the hidden state of the environment and avoid risk more. The free parameters for the model-free reinforcement learning model are the learning rate α and the temperature parameter γ and the free parameters for the model-based are the learning rate α, the temperature parameter γ and prior (the details for the model-free reinforcement learning model can be seen in Eq.S1-11 and the details for the model-based reinforcement learning model can be seen Eq.S12-23 in the Supplementary Method). The parameter fitting for these three models was conducted using the `BayesianOptimization' package in Python (Frazire 2018), first randomly sampling 1000 times and then iterating for an additional 1000 times.”

      Vrieze, S. I. (2012). Model selection and psychological theory: a discussion of the differences between the Akaike information criterion (AIC) and the Bayesian information criterion (BIC). Psychological methods, 17(2), 228.

      Frazier, P. I. (2018). A tutorial on Bayesian optimization. arXiv preprint arXiv:1807.02811.

      Comment 3:

      In terms of the time-dependent correlations with expected free energy - and its constituent terms - I think the report would benefit from overviewing these analyses with something like the following:

      "In the final analysis of the neuronal correlates of belief updating - as quantified by the epistemic and intrinsic values of expected free energy - we present a series of analyses in source space. These analyses tested for correlations between constituent terms in expected free energy and neuronal responses in source space. These correlations were over trials (and subjects). Because we were dealing with two-second timeseries, we were able to identify the periods of time during decision-making when the correlates were expressed.

      In these analyses, we focused on the induced power of neuronal activity at each point in time, at each brain source. To illustrate the functional specialisation of these neuronal correlates, we present whole-brain maps of correlation coefficients and pick out the most significant correlation for reporting fluctuations in selected correlations over two-second periods. These analyses are presented in a descriptive fashion to highlight the nature and variety of the neuronal correlates, which we unpack in relation to the existing EEG literature in the discussion. Note that we did not attempt to correct for multiple comparisons; largely, because the correlations observed were sustained over considerable time periods, which would be almost impossible under the null hypothesis of no correlations."

      Response 3: We deeply thank you for your comments and corresponding suggestions about our description of the regression analysis in the source space. In response to your suggestions, we have added corresponding content in the Results section (EEG results at source level, line 331-347):

      “In the final analysis of the neural correlates of the decision-making process, as quantified by the epistemic and intrinsic values of expected free energy, we presented a series of linear regressions in source space. These analyses tested for correlations over trials between constituent terms in expected free energy (the value of avoiding risk, the value of reducing ambiguity, extrinsic value, and expected free energy itself) and neural responses in source space. Additionally, we also investigated the neural correlate of (the degree of) risk, (the degree of) ambiguity, and prediction error. Because we were dealing with a two-second time series, we were able to identify the periods of time during decision-making when the correlates were expressed. The linear regression was run by the "mne.stats.linear regression" function in the MNE package (Activity ~ Regressor + Intercept). Activity is the activity amplitude of the EEG signal in the source space and regressor is one of the regressors that we mentioned (e.g., expected free energy, the value of reducing ambiguity, etc.).

      In these analyses, we focused on the induced power of neural activity at each time point, in the brain source space. To illustrate the functional specialization of these neural correlates, we presented whole-brain maps of correlation coefficients and picked out the brain region with the most significant correlation for reporting fluctuations in selected correlations over two-second periods. These analyses were presented in a descriptive fashion to highlight the nature and variety of the neural correlates, which we unpacked in relation to the existing EEG literature in the discussion. Note that we did not attempt to correct for multiple comparisons; largely, because the correlations observed were sustained over considerable time periods, which would be almost impossible under the null hypothesis of no correlations.”

      Comment 4:

      There was a slight misdirection in the discussion of priors in the active inference framework. The notion that active inference requires a pre-specification of priors is a common misconception. Furthermore, it misses the point that the utility of Bayesian modelling is to identify the priors that each subject brings to the table. This could be easily addressed with something like the following in the discussion:

      "It is a common misconception that Bayesian approaches to choice behaviour (including active inference) are limited by a particular choice of priors. As illustrated in our fitting of choice behaviour above, priors are a strength of Bayesian approaches in the following sense: under the complete class theorem [5, 6], any pair of choice behaviours and reward functions can be described in terms of ideal Bayesian decision-making with particular priors. In other words, there always exists a description of choice behaviour in terms of some priors. This means that one can, in principle, characterise any given behaviour in terms of the priors that explain that behaviour. In our example, these were effectively priors over the precision of various preferences or beliefs about contingencies that underwrite expected free energy."

      Response 4: We deeply thank you for your comments and corresponding suggestions about the prior of Bayesian methods. In response to your suggestions, we have added corresponding content in the Discussion section (The strength of the active inference framework in decision-making, line 447-453):

      “However, it may be the opposite. As illustrated in our fitting results, priors can be a strength of Bayesian approaches. Under the complete class theorem (Wald 1947; Brown 1981), any pair of behavioral data and reward functions can be described in terms of ideal Bayesian decision-making with particular priors. In other words, there always exists a description of behavioral data in terms of some priors. This means that one can, in principle, characterize any given behavioral data in terms of the priors that explain that behavior. In our example, these were effectively priors over the precision of various preferences or beliefs about contingencies that underwrite expected free energy.”

      Wald, A. (1947). An essentially complete class of admissible decision functions. The Annals of Mathematical Statistics, 549-555.

      Brown, L. D. (1981). A complete class theorem for statistical problems with finite sample spaces. The Annals of Statistics, 1289-1300.

      Reviewer #2 (Public Review):

      Summary:

      Zhang and colleagues use a combination of behavioral, neural, and computational analyses to test an active inference model of exploration in a novel reinforcement learning task.

      Strengths:

      The paper addresses an important question (validation of active inference models of exploration). The combination of behavior, neuroimaging, and modeling is potentially powerful for answering this question.

      Response: We want to express our sincere gratitude for your thorough review of our work and for the valuable comments you have provided. Your attention to detail and dedication to improving the quality of the work are truly commendable. Your feedback has been invaluable in guiding us towards revisions that will strengthen the work. We have made targeted modifications based on most of the comments. However, due to factors such as time and energy constraints, we have not added corresponding analyses for several comments.

      Comment 1:

      The paper does not discuss relevant work on contextual bandits by Schulz, Collins, and others. It also does not mention the neuroimaging study of Tomov et al. (2020) using a risky/safe bandit task.

      Response 1:

      We deeply thank you for your suggestions about the relevant work. We now discussion and cite these representative papers in the Introduction section (line 42-55):

      “The decision-making process frequently involves grappling with varying forms of uncertainty, such as ambiguity - the kind of uncertainty that can be reduced through sampling, and risk - the inherent uncertainty (variance) presented by a stable environment. Studies have investigated these different forms of uncertainty in decision-making, focusing on their neural correlates (Daw et al., 2006; Badre et al., 2012; Cavanagh et al., 2012).

      These studies utilized different forms of multi-armed bandit tasks, e.g the restless multi-armed bandit tasks (Daw et al., 2006; Guha et al., 2010), risky/safe bandit tasks (Tomov et al., 2020; Fan et al., 2022; Payzan et al., 2013), contextual multi-armed bandit tasks (Schulz et al., 2015; Schulz et al., 2015; Molinaro et al., 2023). However, these tasks either separate risk from ambiguity in uncertainty, or separate action from state (perception). In our work, we develop a contextual multi-armed bandit task to enable participants to actively reduce ambiguity, avoid risk, and maximize rewards using various policies (see Section 2.2) and Figure 4(a)). Our task makes it possible to study whether the brain represents these different types of uncertainty distinctly (Levy et al., 2010) and whether the brain represents both the value of reducing uncertainty and the degree of uncertainty. The active inference framework presents a theoretical approach to investigate these questions. Within this framework, uncertainties can be reduced to ambiguity and risk. Ambiguity is represented by the uncertainty about model parameters associated with choosing a particular action, while risk is signified by the variance of the environment's hidden states. The value of reducing ambiguity, the value of avoiding risk, and extrinsic value together constitute expected free energy (see Section 2.1).”

      Daw, N. D., O'doherty, J. P., Dayan, P., Seymour, B., & Dolan, R. J. (2006). Cortical substrates for exploratory decisions in humans. Nature, 441(7095), 876-879.

      Badre, D., Doll, B. B., Long, N. M., & Frank, M. J. (2012). Rostrolateral prefrontal cortex and individual differences in uncertainty-driven exploration. Neuron, 73(3), 595-607.

      Cavanagh, J. F., Figueroa, C. M., Cohen, M. X., & Frank, M. J. (2012). Frontal theta reflects uncertainty and unexpectedness during exploration and exploitation. Cerebral cortex, 22(11), 2575-2586.

      Guha, S., Munagala, K., & Shi, P. (2010). Approximation algorithms for restless bandit problems. Journal of the ACM (JACM), 58(1), 1-50.

      Tomov, M. S., Truong, V. Q., Hundia, R. A., & Gershman, S. J. (2020). Dissociable neural correlates of uncertainty underlie different exploration strategies. Nature communications, 11(1), 2371.

      Fan, H., Gershman, S. J., & Phelps, E. A. (2023). Trait somatic anxiety is associated with reduced directed exploration and underestimation of uncertainty. Nature Human Behaviour, 7(1), 102-113.

      Payzan-LeNestour, E., Dunne, S., Bossaerts, P., & O’Doherty, J. P. (2013). The neural representation of unexpected uncertainty during value-based decision making. Neuron, 79(1), 191-201.

      Schulz, E., Konstantinidis, E., & Speekenbrink, M. (2015, April). Exploration-exploitation in a contextual multi-armed bandit task. In International conference on cognitive modeling (pp. 118-123).

      Schulz, E., Konstantinidis, E., & Speekenbrink, M. (2015, November). Learning and decisions in contextual multi-armed bandit tasks. In CogSci.

      Molinaro, G., & Collins, A. G. (2023). Intrinsic rewards explain context-sensitive valuation in reinforcement learning. PLoS Biology, 21(7), e3002201.

      Levy, I., Snell, J., Nelson, A. J., Rustichini, A., & Glimcher, P. W. (2010). Neural representation of subjective value under risk and ambiguity. Journal of neurophysiology, 103(2), 1036-1047.

      Comment 2:

      The statistical reporting is inadequate. In most cases, only p-values are reported, not the relevant statistics, degrees of freedom, etc. It was also not clear if any corrections for multiple comparisons were applied. Many of the EEG results are described as "strong" or "robust" with significance levels of p<0.05; I am skeptical in the absence of more details, particularly given the fact that the corresponding plots do not seem particularly strong to me.

      Response 2: We deeply thank you for your comments about our statistical reporting. We have optimized the fitting model and rerun all the statistical analyses. As can be seen (Figure 6, 7, 8, S3, S4, S5), the new regression results are significantly improved compared to the previous ones. Due to the limitation of space, we place the other relevant statistical results, including t-values, std err, etc., on our GitHub (https://github.com/andlab-um/FreeEnergyEEG). Currently, we have not conducted multiple comparison corrections based on Reviewer 1’s comments (Comments 3) “Note that we did not attempt to correct for multiple comparisons; largely, because the correlations observed were sustained over considerable time periods, which would be almost impossible under the null hypothesis of no correlations”.

      Author response image 1.

      Comment 3:

      The authors compare their active inference model to a "model-free RL" model. This model is not described anywhere, as far as I can tell. Thus, I have no idea how it was fit, how many parameters it has, etc. The active inference model fitting is also not described anywhere. Moreover, you cannot compare models based on log-likelihood, unless you are talking about held-out data. You need to penalize for model complexity. Finally, even if active inference outperforms a model-free RL model (doubtful given the error bars in Fig. 4c), I don't see how this is strong evidence for active inference per se. I would want to see a much more extensive model comparison, including model-based RL algorithms which are not based on active inference, as well as model recovery analyses confirming that the models can actually be distinguished on the basis of the experimental data.

      Response 3: We deeply thank you for your comments about the model comparison details. We previously omitted some information about the comparison model, as classical reinforcement learning is not the focus of our work, so we put the specific details in the supplementary materials. Now we have placed relevant information in the main text (see the part we have highlighted in yellow). We have now added the relevant information regarding the model comparison in the Results section (Behavioral results, line 279-293):

      “To assess the evidence for active inference over reinforcement learning, we fit active inference (Eq.9), model-free reinforcement learning, and model-based reinforcement learning models to the behavioral data of each participant. This involved optimizing the free parameters of active inference and reinforcement learning models. The resulting likelihood was used to calculate the Bayesian Information Criterion (BIC) as the evidence for each model. The free parameters for the active inference model (AL, AI, EX, prior, and α) scaled the contribution of the three terms that constitute the expected free energy in Eq.9. These coefficients can be regarded as precisions that characterize each participant's prior beliefs about contingencies and rewards. For example, increasing α means participants would update their beliefs about reward contingencies more quickly, increasing AL means participants would like to reduce ambiguity more, and increasing AI means participants would like to learn the hidden state of the environment and avoid risk more. The free parameters for the model-free reinforcement learning model are the learning rate α and the temperature parameter γ and the free parameters for the model-based are the learning rate α, the temperature parameter γ and prior (the details for the model-free reinforcement learning model can be found in Eq.S1-11 and the details for the model-based reinforcement learning model can be found in Eq.S12-23 in the Supplementary Method). The parameter fitting for these three models was conducted using the `BayesianOptimization' package in Python, first randomly sampling 1000 times and then iterating for an additional 1000 times.”

      We have now incorporated model-based reinforcement learning into our comparison models and placed the descriptions of both model-free and model-based reinforcement learning algorithms in the supplementary materials. We have also changed the criterion for model comparison to Bayesian Information Criterion. As indicated by the results, the performance of the active inference model significantly outperforms both comparison models.

      Sorry, we didn't do model recovery before, but now we have placed the relevant results in the supplementary materials. From the result figures, we can see that each model fits its own generated simulated data well:

      “To demonstrate how reliable our models are (the active inference model, model-free reinforcement learning model, and model-based reinforcement learning model), we run some simulation experiments for model recovery. We use these three models, with their own fitting parameters, to generate some simulated data. Then we will fit all three sets of data using these three models.

      The model recovery results are shown in Fig.S6. This is the confusion matrix of models: the percentage of all subjects simulated based on a certain model that is fitted best by a certain model. The goodness-of-fit was compared using the Bayesian Information Criterion. We can see that the result of model recovery is very good, and the simulated data generated by a model can be best explained by this model.”

      Author response image 2.

      Comment 4:

      Another aspect of the behavioral modeling that's missing is a direct descriptive comparison between model and human behavior, beyond just plotting log-likelihoods (which are a very impoverished measure of what's going on).

      Response 4: We deeply thank you for your comments about the comparison between the model and human behavior. Due to the slight differences between our simulation experiments and real behavioral experiments (the "you can ask" stage), we cannot directly compare the model and participants' behaviors. However, we can observe that in the main text's simulation experiment (Figure 3), the active inference agent's behavior is highly consistent with humans (Figure 4), exhibiting an effective exploration strategy and a desire to reduce uncertainty. Moreover, we have included two additional simulation experiments in the supplementary materials, which demonstrate that active inference may potentially fit a wide range of participants' behavioral strategies.

      Author response image 3.

      (An active inference agent with AL=AI=EX=0. It can accomplish tasks efficiently like a human being, reducing the uncertainty of the environment and maximizing the reward.)

      Author response image 4.

      (An active inference agent with AL=AI=0, EX=10. It will only pursue immediate rewards (not choosing the "Cue" option due to additional costs), but it can also gradually optimize its strategy due to random effects.)

      Author response image 5.

      (An active inference agent with EX=0, AI=AL=10. It will only pursue environmental information to reduce the uncertainty of the environment. Even in "Context 2" where immediate rewards are scarce, it will continue to explore.) (a) shows the decision-making of active inference agents in the Stay-Cue choice. Blue corresponds to agents choosing the "Cue" option and acquiring "Context 1"; orange corresponds to agents choosing the "Cue" option and acquiring "Context 2"; purple corresponds to agents choosing the "Stay" option and not knowing the information about the hidden state of the environment. The shaded areas below correspond to the probability of the agents making the respective choices. (b) shows the decision-making of active inference agents in the Stay-Cue choice. The shaded areas below correspond to the probability of the agents making the respective choices. (c) shows the rewards obtained by active inference agents. (d) shows the reward prediction errors of active inference agents. (e) shows the reward predictions of active inference agents for the "Risky" path in "Context 1" and "Context 2".

      Comment 5:

      The EEG results are intriguing, but it wasn't clear that these provide strong evidence specifically for the active inference model. No alternative models of the EEG data are evaluated.

      Overall, the central claim in the Discussion ("we demonstrated that the active inference model framework effectively describes real-world decision-making") remains unvalidated in my opinion.

      Response 5: We deeply thank you for your comments. We applied the active inference model to analyze EEG results because it best fit the participants' behavioral data among our models, including the new added results. Further, our EEG results serve only to verify that the active inference model can be used to analyze the neural mechanisms of decision-making in uncertain environments (if possible, we could certainly design a more excellent reinforcement learning model with a similar exploration strategy). We aim to emphasize the consistency between active inference and human decision-making in uncertain environments, as we have discussed in the article. Active inference emphasizes both perception and action, which is also what we wish to highlight: during the decision-making process, participants not only passively receive information, but also actively adopt different strategies to reduce uncertainty and maximize rewards.

      Reviewer #3 (Public Review):

      Summary:

      This paper aims to investigate how the human brain represents different forms of value and uncertainty that participate in active inference within a free-energy framework, in a two-stage decision task involving contextual information sampling, and choices between safe and risky rewards, which promotes a shift from exploration to exploitation. They examine neural correlates by recording EEG and comparing activity in the first vs second half of trials and between trials in which subjects did and did not sample contextual information, and perform a regression with free-energy-related regressors against data "mapped to source space." Their results show effects in various regions, which they take to indicate that the brain does perform this task through the theorised active inference scheme.

      Strengths:

      This is an interesting two-stage paradigm that incorporates several interesting processes of learning, exploration/exploitation, and information sampling. Although scalp/brain regions showing sensitivity to the active-inference-related quantities do not necessarily suggest what role they play, it can be illuminating and useful to search for such effects as candidates for further investigation. The aims are ambitious, and methodologically it is impressive to include extensive free-energy theory, behavioural modelling, and EEG source-level analysis in one paper.

      Response: We would like to express our heartfelt thanks to you for carefully reviewing our work and offering insightful feedback. Your attention to detail and commitment to enhancing the overall quality of our work are deeply admirable. Your input has been extremely helpful in guiding us through the necessary revisions to enhance the work. We have implemented focused changes based on a majority of your comments. Nevertheless, owing to limitations such as time and resources, we have not included corresponding analyses for a few comments.

      Comment 1:

      Though I could surmise the above general aims, I could not follow the important details of what quantities were being distinguished and sought in the EEG and why. Some of this is down to theoretical complexity - the dizzying array of constructs and terms with complex interrelationships, which may simply be part and parcel of free-energy-based theories of active inference - but much of it is down to missing or ambiguous details.

      Response 1: We deeply thank you for your comments about our work’s readability. We have significantly revised the descriptions of active inference, models, research questions, etc. Focusing on active inference and the free energy principle, we have added relevant basic descriptions and unified the terminology. We have added information related to model comparison in the main text and supplementary materials. We presented our regression results in clearer language. Our research focused on the brain's representation of decision-making in uncertain environments, including expected free energy, the value of reducing ambiguity, the value of avoiding risk, extrinsic value, ambiguity, and risk.

      Comment 2:

      In general, an insufficient effort has been made to make the paper accessible to readers not steeped in the free energy principle and active inference. There are critical inconsistencies in key terminology; for example, the introduction states that aim 1 is to distinguish the EEG correlates of three different types of uncertainty: ambiguity, risk, and unexpected uncertainty. But the abstract instead highlights distinctions in EEG correlates between "uncertainty... and... risk" and between "expected free energy .. and ... uncertainty." There are also inconsistencies in mathematical labelling (e.g. in one place 'p(s|o)' and 'q(s)' swap their meanings from one sentence to the very next).

      Response 2: We deeply thank you for your comments about the problem of inconsistent terminology. First, we have unified the symbols and letters (P, Q, s, o, etc.) that appeared in the article and described their respective meanings more clearly. We have also revised the relevant expressions of "uncertainty" throughout the text. In our work, uncertainty refers to ambiguity and risk. Ambiguity can be reduced through continuous sampling and is referred to as uncertainty about model parameters in our work. Risk, on the other hand, is the inherent variance of the environment and cannot be reduced through sampling, which is referred to as uncertainty about hidden states in our work. In the analysis of the results, we focused on how the brain encodes the value of reducing ambiguity (Figure 8), the value of avoiding risk (Figure 6), and (the degree of) ambiguity (Figure S5) during action selection. We also analyzed how the brain encodes reducing ambiguity and avoiding risk during belief update (Figure 7).

      Comment 3:

      Some basic but important task information is missing, and makes a huge difference to how decision quantities can be decoded from EEG. For example:

      - How do the subjects press the left/right buttons - with different hands or different fingers on the same hand?

      Response 3: We deeply thank you for your comments about the missing task information. We have added the relevant content in the Methods section (Contextual two-armed bandit task and Data collection, line 251-253):

      “Each stage was separated by a jitter ranging from 0.6 to 1.0 seconds. The entire experiment consists of a single block with a total of 120 trials. The participants are required to use any two fingers of one hand to press the buttons (left arrow and right arrow on the keyboard).”

      Comment 4:

      - Was the presentation of the Stay/cue and safe/risky options on the left/right sides counterbalanced? If not, decisions can be formed well in advance especially once a policy is in place.

      Response 4: The presentation of the Stay/cue and safe/risky options on the left/right sides was not counterbalanced. It is true that participants may have made decisions ahead of time. However, to better study the state of participants during decision-making, our choice stages consist of two parts. In the first two seconds, we ask participants to consider which option they would choose, and after these two seconds, participants are allowed to make their choice (by pressing the button).

      We also updated the figure of the experiment procedure as below (We circled the time that the participants spent on making decisions).

      Author response image 6.

      Comment 5:

      - What were the actual reward distributions ("magnitude X with probability p, magnitude y with probability 1-p") in the risky option?

      Response 5: We deeply thank you for your comments about the missing task information. We have placed the relevant content in the Methods section (Contextual two-armed bandit task and Data collection, line 188-191):

      “The actual reward distribution of the risky path in "Context 1" was [+12 (55%), +9 (25%), +6 (10%), +3 (5%), +0 (5%)] and the actual reward distribution of the risky path in "Context 2" was [+12 (5%), +9 (5%), +6 (10%), +3 (25%), +0 (55%)].”

      Comment 6:

      The EEG analysis is not sufficiently detailed and motivated.

      For example,

      - why the high lower-filter cutoff of 1 Hz, and shouldn't it be acknowledged that this removes from the EEG any sustained, iteratively updated representation that evolves with learning across trials?

      Response 6: We deeply thank you for your comments about our EEG analysis. The 1Hz high-pass filter may indeed filter out some useful information. We chose a 1Hz high-pass filter to filter out most of the noise and prevent the noise from affecting our results analysis. Additionally, there are also many decision-related works that have applied 1Hz high-pass filtering in EEG data preprocessing (Yau et al., 2021; Cortes et al., 2021; Wischnewski et al., 2022; Schutte et al., 2017; Mennella et al., 2020; Giustiniani et al., 2020).

      Yau, Y., Hinault, T., Taylor, M., Cisek, P., Fellows, L. K., & Dagher, A. (2021). Evidence and urgency related EEG signals during dynamic decision-making in humans. Journal of Neuroscience, 41(26), 5711-5722.

      Cortes, P. M., García-Hernández, J. P., Iribe-Burgos, F. A., Hernández-González, M., Sotelo-Tapia, C., & Guevara, M. A. (2021). Temporal division of the decision-making process: An EEG study. Brain Research, 1769, 147592.

      Wischnewski, M., & Compen, B. (2022). Effects of theta transcranial alternating current stimulation (tACS) on exploration and exploitation during uncertain decision-making. Behavioural Brain Research, 426, 113840.

      Schutte, I., Kenemans, J. L., & Schutter, D. J. (2017). Resting-state theta/beta EEG ratio is associated with reward-and punishment-related reversal learning. Cognitive, Affective, & Behavioral Neuroscience, 17, 754-763.

      Mennella, R., Vilarem, E., & Grèzes, J. (2020). Rapid approach-avoidance responses to emotional displays reflect value-based decisions: Neural evidence from an EEG study. NeuroImage, 222, 117253.

      Giustiniani, J., Nicolier, M., Teti Mayer, J., Chabin, T., Masse, C., Galmès, N., ... & Gabriel, D. (2020). Behavioral and neural arguments of motivational influence on decision making during uncertainty. Frontiers in Neuroscience, 14, 583.

      Comment 7:

      - Since the EEG analysis was done using an array of free-energy-related variables in a regression, was multicollinearity checked between these variables?

      Response 7: We deeply thank you for your comments about our regression. Indeed, we didn't specify our regression formula in the main text. We conducted regression on one variable each time, so there was no need for a multicollinearity check. We have now added the relevant content in the Results section (“EEG results at source level” section, line 337-340):

      “The linear regression was run by the "mne.stats.linear regression" function in the MNE package (Activity ~ Regressor + Intercept). Activity is the activity amplitude of the EEG signal in the source space and regressor is one of the regressors that we mentioned (e.g., expected free energy, the value of reducing ambiguity, etc.).”

      Comment 8:

      - In the initial comparison of the first/second half, why just 5 clusters of electrodes, and why these particular clusters?

      Response 8: We deeply thank you for your comments about our sensor-level analysis. These five clusters are relatively common scalp EEG regions to analyze (left frontal, right frontal, central, left parietal, and right parietal), and we referred previous work analyzed these five clusters of electrodes (Laufs et al., 2006; Ray et al., 1985; Cole et al., 1985). In addition, our work pays more attention to the analysis in source space, exploring the corresponding functions of specific brain regions based on active inference models.

      Laufs, H., Holt, J. L., Elfont, R., Krams, M., Paul, J. S., Krakow, K., & Kleinschmidt, A. (2006). Where the BOLD signal goes when alpha EEG leaves. Neuroimage, 31(4), 1408-1418.

      Ray, W. J., & Cole, H. W. (1985). EEG activity during cognitive processing: influence of attentional factors. International Journal of Psychophysiology, 3(1), 43-48.

      Cole, H. W., & Ray, W. J. (1985). EEG correlates of emotional tasks related to attentional demands. International Journal of Psychophysiology, 3(1), 33-41.

      Comment 9:

      How many different variables are systematically different in the first vs second half, and how do you rule out less interesting time-on-task effects such as engagement or alertness? In what time windows are these amplitudes being measured?

      Response 9 (and the Response for Weaknesses 11): There were no systematic differences between the first half and the second half of the trials, with the only difference being the participants' experience. In the second half, participants had a better understanding of the reward distribution of the task (less ambiguity). The simulation results can well describe these.

      Author response image 7.

      As shown in Figure (a), agents can only learn about the hidden state of the environment ("Context 1" (green) or "Context 2" (orange)) by choosing the "Cue" option. If agents choose the "Stay" option, they will not be able to know the hidden state of the environment (purple). The risk of agents is only related to wh

      ether they choose the "Cue" option, not the number of rounds. Figure (b) shows the Safe-Risky choices of agents, and Figure (e) is the reward prediction of agents for the "Risky" path in "Context 1" and "Context 2". We can see that agents update the expected reward and reduce ambiguity by sampling the "Risky" path. The ambiguity of agents is not related to the "Cue" option, but to the number of times they sample the "Risky" path (rounds).

      In our choosing stages, participants were required to think about their choices for the first two seconds (during which they could not press buttons). Then, they were asked to make their choices (press buttons) within the next two seconds. This setup effectively kept participants' attention focused on the task. And the two second during the “Second choice” stage when participants decide which option to choose (they cannot press buttons) are measured for the analysis of the sensor-level results.

      Comment 10:

      In the comparison of asked and not-asked trials, what trial stage and time window is being measured?

      Response 10: We have added relevant descriptions in the main text. The two second during the “Second choice” stage when participants decide which option to choose (they cannot press buttons) are measured for the analysis of the sensor-level results.

      Author response image 8.

      Comment 11:

      Again, how many different variables, of the many estimated per trial in the active inference model, are different in the asked and not-asked trials, and how can you know which of these differences is the one reflected in the EEG effects?

      Response 11: The difference between asked trials and not-asked trials lies only in whether participants know the specific context of the risky path (the level of risk for the participants). A simple comparison indeed cannot tell us which of these differences is reflected in the EEG effects. Therefore, we subsequently conducted model-based regression analysis in the source space.

      Comment 12:

      The authors choose to interpret that on not-asked trials the subjects are more uncertain because the cue doesn't give them the context, but you could equally argue that they don't ask because they are more certain of the possible hidden states.

      Response 12: Our task design involves randomly varying the context of the risky path. Only by choosing to inquire can participants learn about the context. Participants can only become increasingly certain about the reward distribution of different contexts of the risky path, but cannot determine which specific context it is. Here are the instructions for the task that we will tell the participants (line 226-231).

      "You are on a quest for apples in a forest, beginning with 5 apples. You encounter two paths: 1) The left path offers a fixed yield of 6 apples per excursion. 2) The right path offers a probabilistic reward of 0/3/6/9/12 apples, and it has two distinct contexts, labeled "Context 1" and "Context 2," each with a different reward distribution. Note that the context associated with the right path will randomly change in each trial. Before selecting a path, a ranger will provide information about the context of the right path ("Context 1" or "Context 2") in exchange for an apple. The more apples you collect, the greater your monetary reward will be."

      Comment 13:

      - The EEG regressors are not fully explained. For example, an "active learning" regressor is listed as one of the 4 at the beginning of section 3.3, but it is the first mention of this term in the paper and the term does not arise once in the methods.

      Response 13: We have accordingly revised the relevant content in the main text (as in Eq.8). Our regressors now include expected free energy, the value of reducing ambiguity, the value of avoiding risk, extrinsic value, prediction error, (the degree of) ambiguity, reducing ambiguity, and avoiding risk.

      Comment 14:

      - In general, it is not clear how one can know that the EEG results reflect that the brain is purposefully encoding these very parameters while implementing this very mechanism, and not other, possibly simpler, factors that correlate with them since there is no engagement with such potential confounds or alternative models. For example, a model-free reinforcement learning model is fit to behaviour for comparison. Why not the EEG?

      Response 14: We deeply thank you for your comments. Due to factors such as time and effort, and because the active inference model best fits the behavioral data of the participants, we did not use other models to analyze the EEG data. At both the sensor and source level, we observed the EEG signal and brain regions that can encode different levels of uncertainties (risk and ambiguity). The brain's uncertainty driven exploration mechanism cannot be explained solely by a simple model-free reinforcement learning approach.

      Recommendations for the authors:

      Response: We have made point-to-point revisions according to the reviewer's recommendations, and as these revisions are relatively minor, we have only responded to the longer recommendations here.

      Reviewer #1 (Recommendations For The Authors)

      I enjoyed reading this sophisticated study of decision-making. I thought your implementation of active inference and the subsequent fitting to choice behaviour - and study of the neuronal (EEG) correlates - was impressive. As noted in my comments on strengths and weaknesses, some parts of your manuscript with difficult to read because of slight collapses in grammar and an inconsistent use of terms when referring to the mathematical quantities. In addition to the paragraphs I have suggested, I would recommend the following minor revisions to your text. In addition, you will have to fill in some of the details that were missing from the current version of the manuscript. For example:

      Recommendation 1:

      Which RL model did you use to fit the behavioural data? What were its free parameters?

      Response 1: We have now added information related to the comparison models in the behavioral results and supplementary materials. We applied both simple model-free reinforcement learning and model-based reinforcement learning. The free parameters for the model-free reinforcement learning model are the learning rate α and the temperature parameter γ, while the free parameters for the model-based approach are the learning rate α, the temperature parameter γ, and the prior.

      Recommendation 2:

      When you talk about neuronal activity in the final analyses (of time-dependent correlations) what was used to measure the neuronal activity? Was this global power over frequencies? Was it at a particular frequency band? Was it the maximum amplitude within some small window et cetera? In other words, you need to provide the details of your analysis that would enable somebody to reproduce your study at a certain level of detail.

      Response 2: In the final analyses, we used the activity amplitude at each point in the source space for our analysis. Previously, we had planned to make our data and models available on GitHub to facilitate easier replication of our work.

      Reviewer #3 (Recommendations For The Authors)

      Recommendation 1:

      It might help to explain the complex concepts up front, to use the concrete example of the task itself - presumably, it was designed so that the crucial elements of the active inference framework come to the fore. One could use hypothetical choice patterns in this task to exemplify different factors such as expected free energy and unexpected uncertainty at work. It would also be illuminating to explain why behaviour on this task is fit better by the active inference model than a model-free reinforcement learning model.

      Response 1: Thank you for your suggestions. We have given clearer explanations to the three terms in the active inference formula: the value of reducing ambiguity, the value of avoiding risk, and the extrinsic value (Eq.8), which makes it easier for readers to understand active inference.

      In addition, we can simply view active inference as a computational model similar to model-based reinforcement learning, where the expected free energy represents a subjective value, without needing to understand its underlying computational principles or neurobiological background. In our discussion, we have argued why the active inference model fits the participants' behavior better than our reinforcement learning model, as the active inference model has an inherent exploration mechanism that is consistent with humans, who instinctively want to reduce environmental uncertainty (line 435-442).

      “Active inference offers a superior exploration mechanism compared with basic model-free reinforcement learning  (Figure 4 (c)). Since traditional reinforcement learning models determine their policies solely on the state, this setting leads to difficulty in extracting temporal information (Laskin et al., 2020) and increases the likelihood of entrapment within local minima. In contrast, the policies in active inference are determined by both time and state. This dependence on time (Wang et al., 2016) enables policies to adapt efficiently, such as emphasizing exploration in the initial stages and exploitation later on. Moreover, this mechanism prompts more exploratory behavior in instances of state ambiguity. A further advantage of active inference lies in its adaptability to different task environments (Friston et al., 2017). It can configure different generative models to address distinct tasks, and compute varied forms of free energy and expected free energy.”

      Laskin, M., Lee, K., Stooke, A., Pinto, L., Abbeel, P., & Srinivas, A. (2020). Reinforcement learning with augmented data. Advances in neural information processing systems, 33, 19884-19895.

      Wang, J. X., Kurth-Nelson, Z., Tirumala, D., Soyer, H., Leibo, J. Z., Munos, R., ... & Botvinick, M. (2016). Learning to reinforcement learn. arXiv preprint arXiv:1611.05763.

      Friston, K., FitzGerald, T., Rigoli, F., Schwartenbeck, P., & Pezzulo, G. (2017). Active inference: a process theory. Neural computation, 29(1), 1-49.

      Recommendation 2:

      Figure 1A provides a key example of the lack of effort to help the reader understand. It suggests the possibility of a concrete example but falls short of providing one. From the caption and text, applied to the figure, I gather that by choosing either to run or to raise one's arms, one can control whether it is daytime or nighttime. This is clearly wrong but it is what I am led to think by the paper.

      Response 2: Thank you for your suggestion, which we had not considered before. In this figure, we aim to illustrate that "the agent receives observations and optimizes his cognitive model by minimizing variational free energy → the agent makes the optimal action by minimizing expected free energy → the action changes the environment → the environment generates new observations for the agent." We have now modified the image to be simpler to prevent any possible confusion for readers. Correspondingly, we removed the figure of a person raising their hand and the shadowed house in Figure a.

      Author response image 9.

      Recommendation 3:

      I recommend an overhaul in the labelling and methodological explanations for consistency and full reporting. For example, line 73 says sensory input is 's' and the cognitive model is 'q(s),' and the cause of the sensory input is 'p(s|o)' but on the very next line, the cognitive model is 'p(s|o)' and the causes of sensory input are 'q(s).' How this sensory input s relates to 'observations' or 'o' is unclear, and meanwhile, capital S is the set of environmental states. P seems to refer to the generative distribution, but it also means probability.

      Response 3: Thank you for your advice. Now we have revised the corresponding labeling and methodological explanations in our work to make them consistent. However, we are not sure how to make a good modification to P here. In many works, P can refer to a certain probability distribution or some specific probabilities.

      Recommendation 4:

      Even the conception of a "policy" is unclear (Figure 2B). They list 4 possible policies, which are simply the 4 possible sequences of steps, stay-safe, cue-risky, etc, but with no contingencies in them. Surely a complete policy that lists 'cue' as the first step would entail a specification of how they would choose the safe or risky option BASED on the information in that cue

      Response 4: Thank you for your suggestion. In active inference, a policy actually corresponds to a sequence of actions. The policy of "first choosing 'Cue' and then making the next decision based on specific information" differs from the meaning of policy in active inference.

      Recommendation 5:

      I assume that the heavy high pass filtering of the EEG (1 Hz) is to avoid having to baseline-correct the epochs (of which there is no mention), but the authors should directly acknowledge that this eradicates any component of decision formation that may evolve in any way gradually within or across the stages of the trial. To take an extreme example, as Figure 3E shows, the expected rewards for the risky path evolve slowly over the course of 60 trials. The filter would eliminate this.

      Response 5: Thank you for your suggestion. The heavy high pass filtering of the EEG (1 Hz) is to minimize the noise in the EEG data as much as possible.

      Recommendation 6:

      There is no mention of the regression itself in the Methods section - the section is incomplete.

      Response 6: Thank you for your suggestion. We have now added the relevant content in the Results section (EEG results at source level, line 337-340):

      “The linear regression was run by the "mne.stats.linear regression" function in the MNE package (Activity ∼ Regressor + Intercept, Activity is the activity amplitude of the EEG signal in the source space and regressor is one of the regressors that we mentioned).”

      Recommendation 7:

      On Lines 260-270 the same results are given twice.

      Response 7: Thank you for your suggestion. We have now deleted redundant content.

      Recommendation 8:

      Frequency bands are displayed in Figure 5 but there is no mention of those in the Methods. In Figure 5b Theta in the 2nd half is compared to Delta in the 1st half- is this an error?

      Response 8: Thank you for your suggestion. It indeed was an error (they should all be Theta) and now we have corrected it.

      Author response image 10.

    1. Author Response

      The following is the authors’ response to the original reviews.

      Reviewer 1

      Major points:

      R1C1: I appreciate that the data are aligned, in some points, with related studies of this niche. However, it would help the reader to have this alignment explored more extensively in the Discussion as well.

      Answer: We acknowledge that the discussion would benefit from additional comparisons to the available datasets. We thus add the following comment after the first paragraph of the discussion: “Previous studies of the different sub-populations of SVZ progenitors were carried out using transcriptomic approaches based on the expression of various more or less specific markers. These approaches have made it possible to identify quiescent and activated neural stem cells as well as mature neuroblasts, but have been faced with the strong influence of the cell cycle on cell clustering. Indeed, neural progenitors in these studies cycling have been gathered in either “mitotic” clusters (Llorens et al. 2015, Zywitza et al. 2018, Cebrian et al. 2021) or “neural progenitor cells” clusters (Dulken et al. 2017) that had no clear biological significance and hindering identification of subtypes of SVZ cycling progenitors. Our study, combining, for the first time, characterization of Facs-isolated cells and an irradiation-based model of sequential regeneration, allowed to clearly distinguish the molecular profiles of TAP and iNB among cycling progenitors reflecting differences in their in vitro and in vivo respective potentials”.

      R1C2: The data on multilineage differentiation, both in culture and upon engraftment, would be greatly strengthened by quantification. What is the relative yield of TUJ1/DCX-positive cells versus the other marker combinations? Specifically regarding the multilineage differentiation in vitro - because different media conditions are used to generate each lineage, it may be difficult to determine relative yield. Could a differentiation system that allows production of all 3 lineages be used instead?

      If the fraction of non-DCX/TUJ1-labeled progeny is low, particularly in vivo, this might suggest that while multilineage differentiation is possible, it is a much less likely cellular state outcome than production of mature neuroblasts. Some suggested references with examples of the culture conditions, experimental conditions, and discussions highlighted in the public review: Culture conditions that allow simultaneous trilineage differentiation. PMID: 17615304 Influence of culture conditions on potency: similar to issues covered in PMID: 21549325.

      Answer: We agree with the reviewer that quantification of a multilineage differentiation in vitro would improve the characterization of the relative potencies of the different SVZ progenitor.

      According to PMID: 17615304 and PMID: 21549325, and in agreement with our own experience, the only culture condition that allows neurosphere-derived neural progenitors to differentiate in vitro into the three lineages is the removal of mitogens from the culture medium. However, this does not work on freshly isolated SVZ cells, which remain in an undifferentiated state in this condition.

      This is why we chose to use specific differentiation media for each of the 3 lineages as in Figure 1C. It is also for this reason that we performed as many experiments as possible in vivo rather than in vitro as in Figure S2. In the new version, we have added a quantitative analysis of stainings by antibodies against GFAP, CNPase or DCX of GFP-positive cells persisting at IS, where high number of grafted cells were found in Figure S2B. This was performed by using the NIS software measuring eGFP-, GFAP-, CNPase- and DCX-positive areas. The intersection between each marker and eGFP areas was then determined as a percentage of staining (Figure S2C). The results showed that approximately one third of GFP+ cells expressed GFAP or DCX. The quantitative analysis of CNPase expression was complicated by CNPase-positive host cells, but the stronger CNPase staining in eGFP-positive areas clearly revealed the expression of CNPase by a significant proportion of eGFP-positive cells.

      R1C3: Additionally, for claims similar to what is currently made in the text, it would be extremely valuable to confirm the purity of the sort for each population - for example by fixing and staining the sorted fraction with additional antibodies that confirm cell identity.

      Answer: We have previously shown in Daynac et al. 2013 that s-iNB expressed the neuroblast markers CD24 and DCX, but also markers of neural progenitors such as Mash1, a basic helix-loop-helix transcription factor. As suggested by the reviewer, we have further investigated the expression of other markers of neural progenitors by sorted cells. The results showed that the proportion of DLX2+ cells a marker of proliferating progenitors (Doetsch et al. 2002) was very high in aNSC/TAP (98%) and progressively decreased in iNB (82%) and mNB (25%). Similarly, the expression of the transcription factor SOX2 that plays an essential role in the maintenance of neural progenitors (PMID: 25126380) accounted for 78% of aNSC/TAP, 70% of iNB and 17% of mNB.

      Altogether, these new data confirmed the identity of the different cell populations and particularly that of iNB. They are commented at the beginning of the Results and shown in Figure S1.

      R1C4: Line 125: GFAP alone doesn't necessarily indicate a "conversion to NSCs" - this conclusion could be greatly strengthened by inclusion of more markers, particularly at the protein level, or cyto-architectural studies.

      Answer: We agree with the reviewer that GFAP expression alone is not sufficient to evidence the presence of NSC in the SVZ. We have thus modified the text accordingly: “Importantly, eGFP+ cells were present in the SVZ of all the animals transplanted with eGFP+s-iNB and eGFP+s-NSC/TAP (Fig. 1Db, Fig. 1Dc), some of them expressing GFAP indicating the generation of astrocytes, and therefore possibly NSC”.

      R1C5: Could these cellular states be reflective of preferential translation of DCX? It would be very helpful to see the flow cytometry sort data for iNBs / mNBs used in Figure 6, particularly if these cells were also fixed and stained directly for DCX protein.

      Answer: As suggested by the reviewer, freshly FAC-sorted iNB and mNB were fixed and labelled with an anti-DCX monoclonal antibody after permeabilization. As shown in the figure below, we found a higher level of DCX expression in mNB than in iNB. Therefore, this result tends to indicate that the proliferation capacity is somehow related to the level of DCX expression. However, because of the relatively low importance of this result, we decided not to include them in the manuscript.

      Author response image 1.

      Modal histogram representation of DCX expression level in unstained, iNB and mNB cells determined by flow cytometry (FlowJo).

      <R1C6: Figure S8 is all zeroes, showing the GFP+Dcxhigh NBs do not retain proliferative capacity. But we don't get a direct experimental comparison to EGFPnegative/lowDcxlow iNB engraftment, which would strengthen the conclusions of the paper.

      Answer: Unfortunately, there is no method available to analyse the eGFPnegative/lowDcxlow iNB engraftment: by definition, these cells do not express eGFP and the use of a tracker is not appropriate for long periods of time — and thus a high number of cell divisions — after engraftment. However, to us, this control is not needed to conclude that GFP+Dcxhigh iNB have no (or at least a lower) stem cell potential in vivo considering that we have shown in Figure 1 and Table 1 that the whole iNB population is able to generate the different types of neural cells.

      R1C7: Transplant data in Table 1 - a relatively small proportion of transplant derived cells are in OB, etc. Given that A cells are thought to cycle at least once in vivo, is this expected?

      Answer: The reviewer is right considering that a relatively small proportion of transplant derived cells were found in the OB. However, we should consider that we used immunocompetent mice as receivers, which could have significantly reduced the engraftment efficiency, and the migration of engrafted cells outside the injection site.

      R1C8: A caveat is that there is not much functional testing of the proposed model, especially for the interconversion of iNB states suggested by the diagram in Figure 7. The text is relatively restrained in proposing this model, so it is reasonable to keep - but perhaps should be noted that this part of the model will need additional testing.

      Answer: Data presented in Figure 6 clearly suggest that Dcxhigh iNB have similar in vitro potential than Dcxlow iNB, whereas they don’t have such potential in vivo (Figure S10). This suggests that, providing they are in appropriate conditions, Dcxhigh iNB could reacquire stem/progenitor properties. However, we agree that this hypothesis requires further investigation. Therefore, as suggested by the reviewer, we have added in the Figure 7 legend: “Possible interconversion of iNB states would require further experimental confirmation.”

      Additional minor points:

      R1C9: Introduction: the SVZ is described as "the lateral wall" - however, several works in the mouse have also examined the medial wall and callosal roof, as cited later in the intro. Suggest rephrasing the second sentence (line 48) and later sentence (line 66) to clarify that "the SVZ" encompasses all of these subregions, they are not necessarily separate niches. Answer: As indicated by the reviewer, the SVZ encompasses distinct subdomains, with NSCs having a regional identity based on their location in the lateral or septal wall of the ventricle and generating different types of neuronal and glial progeny (PMID:34259628.). To address the reviewer concern about possible confusion and clearly indicate that SVZ encompass several subdomains, we have modified the sentence line 66 as follows: “Since then, the single cell RNA-sequencing has revolutionized the field and has made it possible to precisely elucidate the transcriptome of SVZ cells present in the LW and in the septal wall which also harbors NSC niches”.

      However, we did not modify the line 48, since in this sentence we just indicate that the largest neurogenic niche in the adult brain reside in the LW of the SVZ.

      R1C10: Line 77: "exposure" not "exposition"

      Answer: The error has been corrected in the revised manuscript.

      R1C11: As noted in the Public Review - the use of the term "D1/D2" cells seems likely to confuse readers who are also versed in dentate gyrus neurogenesis. Recommend removing this term from the manuscript.

      Answer: We agree that the D1/D2 terminology could bring confusion, D cells referring to Tanycytes in the hypothalamus. We now refer to iNB1 for DcxLow iNB and iNB2 for DcxHigh iNB in the revised manuscript.

      Reviewer 2

      Major comments:

      Lack of rigor

      R2C1: There is a lack of appropriate normalization controls for the microarray data. As there is a decreased level of transcription in quiescent NSCs, there needs to be a cell number control (spike-ins based on cell numbers). Without this normalization, the readout can be greatly skewed.

      Answer: We agree that qNSC are marked by a decreased level of transcription due to quiescence. To overcome this problem in the Clariom assays, we thus chose to calibrate each population, with a fixed amount of cRNA and cDNA using Hela cells as internal control. We totally agree that this method is not optimal but it appears to be efficient in the end. Indeed, it should be noticed that it has been adopted, thus with the same rigor, in other microarray studies published in the field (PMID: 24811379) and also on skeletal muscle cells (PMID: 29273087). Moreover, interestingly the transcriptomic signature of qNSC matches perfectly with those from other studies and particularly to those of related clusters in single cell experiments (including ours, Figure S5). This is probably linked to the fact that more importantly that the number of cells, the main characteristic of these cells is the lack of expression of genes involved in cell proliferation and metabolism. Whatever so, these data confirming previously published are not the main information of our manuscript, which is mainly dedicated to the characterization of proliferating cells, which is not impaired by our choices of normalization.

      R2C2: The absolute segregation of clusters in the single-cell analysis is currently entirely in agreement with the cell cycle stage. This suggests that in the author's analysis, the clustering in 3F is entirely shaped by the cell cycle, making that the defining characteristic of the author's definitions for their cell types. Has an analysis been done that regresses out cell cycle-associated genes to see if there are clusters for different cell states/types that are identified in the absence of cell cycle stage being the defining factor? (Barron and Li, 2016). For example, just as you would see a difference in cluster if you are a quiescent or activated NSC as compared to a neuroblast for example, even without the contribution of cell cycle. These are different cell types.

      Answer: We agree that cell cycle regression would theoretically allow for further discrimination between cycling cells along successive neurogenic stages. We have already performed regression using several methods, including regressing using S- and G2/M-score regression as indicated in the Seurat workflow, removing cell cycle-related PCs from UMAP calculation as used in the Cebrian-Sylla study, and using alternative gene sets such as the ones provided by the tricycle method (PMID: 35101061). These regression methods have all been used on our datasets, the original Cebrian-Sylla datasets and a combination of our datasets with the Cebrian-Sylla original datasets to increase cell number and clustering resolution. However, none of these methods modified the clustering of cycling cells.

      In fact, the strong influence of the cell cycle over clustering highlights the relevance of our depletion/replenishment approaches to decipher the molecular changes masked by the cell cycle, as discussed below.

      R2C3: The use of the DCX-CreERT2 line is a lineage tracing line. Once DCX is expressed, Cre recombines the DNA to allow for fluorescence. It is binary, on or off associated with DCX expression. And once on, it is always on, whether the cell is currently expressing DCX or not. As the authors had previously described a DCXlow condition, the eGFP- cells would not reflect DCXlow, but no DCX at all. And the eGFP+ cells may not be currently expressing DCX anymore. The authors should have used a system where the DCX promoter itself drives fluorescence.

      Answer: We took advantage of the DCX-CreERT2 line to demonstrate that some neural cells that have recently acquired DCX expression (i.e. eGFP+ iNB) could keep (or recover) the potential of neural progenitors in vitro. Of course, some of these GFP+ cells could have stopped to express DCX. This is probably the case when they differentiate into astrocytes and oligodendrocytes in vitro as shown in Figure 6.

      Whatever so, the use of the Dcx promoter as a direct driver of eGFP fluorescence would have totally impeded our capacity to demonstrate such changes in cell fate in vivo because of the impossibility to track oligodendrocytes or astrocytes derived from iNB because of the loss of Dcx expression.

      R2C4: The lack of analysis of images (differentiation, for example) limits the conclusions of the in-vitro data, and the images with unclear staining, limit the conclusions of the in-vivo experiments.

      Answer: This comment is similar to that of R1C2. We have now added a quantification in Figure S2.

      R2C5: The cited difference in splicing differences in cell types was interesting (though did not show up in the transcriptome enrichment analyses Fig S2) and would be something to further pursue, however, this was a very limited analysis. There was no further study of these splicing mediators beyond single-cell data.

      Answer: We now show enrichments of GO terms corresponding to mRNA splicing isoforms in the different types of sorted SVZ cells (Figure S4). This analysis clearly revealed that spliced genes in SVZ cells are mainly involved in neuron development and neurogenesis. Interestingly this also showed that qNSC logically differed from the other cell types by splicing concerning genes involved in mitosis and cell cycle, consistently with their quiescent state. More importantly, GO annotations of differentially spliced isoforms further confirmed that s-TAP and s-iNB have distinct features. We agree with the reviewer that further analysis of splicing mediators would be very important for understanding molecular changes involved in neurogenesis. However, we think that it is largely beyond the scope of this study.

      R2C6: Fig 1C - Show values, not just pictures. You may need to shift your current differentiation paradigm to do so by removing growth factors instead of unique differentiation conditions.

      Answer: See the answer to R1C2.

      R2C7: Fig S1A - Stainings for GFAP and DCX are not clear. It is very hard to distinguish which cells are associated with these signals.

      Answer: This figure (now Figure S2A) shows an eGFP+iNB cell (white arrow) that has reached the rostral migratory stream and expressed DCX (inset a3), but not GFAP (inset a2). This is now indicated in the figure legend. We have also moved the arrow for more clarity.

      R2C8: Fig S1B2 - There is red staining everywhere, so it is very hard to see a specific CNPase signal.

      Answer: We have added a new figure (Fig S2B) distinguishing eGFP+CNPase+ cells (yellow arrows) from eGFP+CNPase- cells (white arrow).

      R2C9: Line 174 - It's the mRNA that you are detecting is being downregulated - be more specific as you are not showing protein downregulation.

      Answer: We specified, "encoding" a major splicing repressor in the Line 174 text to refer to the mRNA: “Interestingly, Ptbp1, encoding a major splicing repressor”.

      R2C10: Line 189 - text in this line have some clusters not shown in the figure - (clusters 6 and 15, DCX+ Ki67+ neuroblasts) - which would be an important thing to visualize. As is shown now, the authors are only showing that iNBs are similar to mitotic TAPs.

      Answer: Clusters 6 and 15 have been added to Figure S5.

      R2C11: Fig 3D-E - Why is cluster 17 called aNSCs (3E) when it has the highest GFAP (Fig 3D). Typically, the highest GFAP cells are qNSCs or astrocytes, not aNSCs.

      Answer: We previously reported that the level of gfap mRNA expression in neural stem cells (quiescent and activated) did not exactly reflect the amount of protein in these cells. This is the reason why we also used the Slc1a3 marker (Glast), which is highly expressed both at the RNA and protein levels in quiescent NSCs (Daynac et al. 2013).

      R2C12: Line 216 - You said in line 216 cluster 13 were astrocytes, then you said in line 227 that cluster 13 was s-qNSC. Which is it?

      Answer: This is due to the fact that we performed two distinct analyses.

      In the first one (line 216), cells were scored based on datasets provided by Cebrian et al. with one dataset containing genes enriched in astrocytes, and another one, genes enriched in quiescent B-cells. Therefore, cluster 13 was shown to contain 73% cells expressing astrocyte markers, whereas cluster 4 gathered cells expressing both qNSC (B-cells, 48%) and astrocyte (52%) genes.

      In the second one (line 227), cells were scored using our transcriptomic signatures of FAC-sorted SVZ cells, which do not include differentiated astrocytes. We demonstrated that the cluster 13 cells only expressed s-qNSC genes.

      R2C13: Line 214 - While other clusters were all named in lines 214-221 that were then further discussed in lines 227-230, clusters 15 and 19 were not. You associate both of those clusters with s-iNB - what was it associated with in the above section?

      Answer: Lines 219-221 have been reworded as follows: Clusters 10, 5, 15, 12, and 8 were defined as cycling progenitors based on the expression of proliferative markers such as Top2a, Mki67, Ascl1. Clusters 1, 3, 7 and 9 were identified as mNB due to the loss of Mki67, Top2 a and Ascl1 expressions and the expression of Robo2 and Dcx. Cluster 19 that have lost Ascl1 but still expressing Top2a and Mki67 together with Robo2 and Dcx appears at the transition between iNB and mNB.

      R2C14: Fig 3I-J - 5 days after irradiation, I would like to see from tissue slices how many cells are dividing compared to 1day post-irradiation and controls. In other paradigms, such as temozolomide experiments (Kalamakis et al), by 5 days we should see less cells in quiescence and more of those quiescent cells exiting quiescence into the cell cycle. Why would there be more cells in quiescence in the irradiated brain? Even if they are radiation resistant, the base number should be comparative between controls and irradiated, which is not what you show in Fig 3I-J. And R2C14)

      Line 234-235 - the text says normalized to numbers of qNSCs which is supposed to be the same (which I agree should be the same). However, your graph in 3I and J shows more qNSCs in irradiated conditions, which would influence greatly and is currently hard to interpret.

      Answer: As stated by the reviewer, there is no increase in the absolute number of quiescent cells in the irradiated SVZ. The reconstitution of SVZ cell populations after 4Gy irradiation has already been studied by our group (Daynac et al. 2013, see Fig. 3F), showing that s-iNB and s-mNB are still under-represented after 5 days, while qNSC are in similar numbers as in unirradiated SVZ. Therefore, this led to an over-representation of quiescent cells and early SVZ progenitors in Figure 3J as compared in Figure 3I.

      R2C15: Fig 6A - the authors show a significant difference in neurospheres between eGFP- (DCX-) and eGFP+ (DCX+) iNBs - as would be expected as DCX suggests a further commitment towards neurogenic fates, yet your population doubling is the same.

      Answer: To determine the population doublings, the medium was changed and cells numbered every 7 days. This condition masked the differences between two cell populations reaching the plateau phase at different time, explaining why eGFP-iNB and eGFP+iNB could not be clearly distinguished by this technique.

      R2C16: Fig 6C - Differentiation data (in-vitro) should be quantified in 6C, just as was mentioned for 1C. These values should be done for both of the populations (eGFP-iNB, and eGFP+iNB) and not just compared to the previous pictures which were on total iNB. Again, numbers are required, not just picture examples.

      Answer: Quantitative data have been given in Figure 6D showing that approximately 60-80% of cells eGFP+iNB are able to differentiate in either neurons, oligodendrocytes or astrocytes. We did not analyze the differentiation of eGFP-iNB since it would not add any supplementary information.

      R2C17: Fig S8 - The authors did not show if the lack of engraftment of eGFP+ cells is due to the transplant (previously you showed only 2/3 worked in a similar paradigm). It would be helpful if the authors would have some means to visualize the DCX low cells to confirm they worked as before in the transplantation (another color? Another type of mouse (Thy1 antigen differences)?) Answer: Unfortunately, the Thy1 antigen has not been documented in mouse subventricular zone progenitors, but only in neurons (PMID: 10813783). Thy1 antigen has also been described in bipotent glial progenitor cell (GCP) from the developing human brain giving rise to oligodendrocytes (PMID: 36931245).

      As shown, in Figure S10 we have performed 5 grafts with s-iNB eGFP+ cells, 2 alone and 3 mixed with eGFP- cells and never found any eGFP+ cells 5 weeks after grafting. Moreover, we did not find any eGFP+ cells in the brains of 3 other animals 2 weeks after grafting with s-iNB eGFP+ cells (These data have been added to Figure S10). As compared to the results described in Figure 1 this clearly shows that iNB DCXhigh are not able to generate persistent cells in the grafted brains similarly as mNB.

      R2C18: Fig S8 - Why were there no eGFP cells even at the injection site? DCX expression promotes migration, indeed DCX expression becomes very high in cells in the SVZ as they begin to exit to go to the migratory stream. If one didn't see migration, one would expect you would still have survival. Currently, the authors show no cells at 5 weeks, however, they would need to show earlier timepoints as well to determine what is happening with these cells. It is possible these GFP+ cells are not even expressing DCX anymore (see above).

      Answer: As stated above, we did not find any GFP+ cells in the brains of 3 other animals 2 weeks after grafting with s-iNB eGFP+ cells (see Figure S10).

      R2C19: Line 320 - the authors suggest a subpopulation of NEURONS continues to divide and cite 2 works from the 1990s showing proliferating SVZ cells can differentiate. Our knowledge of this system has come dramatically forward since the 1990s as well as technologically, and to date, neurons have not been shown to divide.

      Answer: We apologize for this lack of clarity, as we agree that neurons correspond to differentiated non-cycling cells, but we used the terminology used in these articles. The incorrect part of the sentence Line 320 has thus been deleted from the text.

      R2C20: Fig 7 - The whole figure is based on changing levels of RSR genes which were not confirmed in any way to be involved in any of these stages, only descriptively in single-cell analyses.

      Answer: As stated above, in our opinion, further characterization of the involvement of RSR genes in neurogenesis is largely beyond the scope of our manuscript. Nevertheless, we think that the role of RSR genes in neurogenesis is an important question that should be addressed in further studies.

      Overstatement of findings

      R2C21: Fig 1 - Authors did not compare all cell types in each condition but made overstatements about their relationships to each other between graphs. There should also be separate graphs showing all cell types at 4% and a separate one at 20%.

      Answer: In the revised version, Figure 1 shows the graph comparing all cell types at 4%O2 and a separate one at 20% as requested by the reviewer. The graphs clearly shows that 4%O2 promotes iNB proliferation compared to the 20% condition.

      R2C22: Fig 1D-b2 - Why does DCX look nuclear? One can't say they are only NSCs if they are GFAP as astrocytes also express GFAP. The authors would need another marker to separate those populations. In the text, the authors say expressing GFAP (line 124) which means NSC, but then in line 127 expressing GFAP means astrocytes - which further shows you need additional markers to validate those 2 different cell types. Answer: DCX nuclear translocation has been shown to improve cellular proliferation (PMID:32050972).

      As indicated in R1C4. The text has been modified as follows: “Importantly, eGFP+ cells were present in the SVZ of all the animals transplanted with s-iNB eGFP+ and s-NSC/TAP eGFP+ (Fig. 1Db, 1Dc), some of them expressing GFAP indicating the generation of astrocytes, and therefore possibly NSC”.

      R2C23: Fig S2 - The transcriptome signature for s-iNBs is very similar to s-TAP, basically suggesting the iNBs are further along in cell cycle.

      Answer: This is now the Figure S3. Functional enrichment analysis of individual transcriptome signatures revealed that both s-TAP and s-iNB are enriched in genes related to the cell cycle although with different GO terms enrichments. Indeed, s-TAP are enriched in genes related to G1, G1/S and S phase (but with low -log10 adjusted p-values) and s-iNB with genes related to cell cycle mitosis and M phase (with high -log10 adjusted p-values).

      We have previously shown that around 33 % s-iNB have DNA content>2N, versus around 26% of s-TAP and s- aNSC (Daynac et al. 2013), which is in accordance with GO terms enrichments. However, these data have also shown that most s-iNB and s-TAP are in G1, indicating that siNB are not just further along mitosis than TAP.

      Moreover, our transcriptomic data clearly show that s-iNB are distinct from s-TAP: 1) according to principal component analyses (Figure 2B et C), the whole transcriptome of s-TAP is closer to that of s-aNSCs than to that of s-iNB (10% variations in PCA2), 2) the heatmap in Figure 2D shows that they have different RSR genes expression profiles, 3) the new Figure S4 shows that GO annotations of differentially spliced isoforms further confirmed that s-TAP and s-iNB have distinct features, and 5) Figure S5 shows that s-iNB expressed genes associated to either TAP or NB that have been described in previous studies, whereas s-TAP did not express genes associated to NB, but look closer to aNSC. Finally, scRNAsq cell clusters related to s-iNB are distinct from the cluster related to s-TAP as shown 1) in Figure 3D and 2) in Figure 4.

      R2C24: Fig 3 - The lack of information about timepoint 0 after irradiation, and when proliferation and cell cycle entry begins again following irradiation, limits our interpretation of the single-cell irradiated data.

      Answer: We have previously reported the relative abundance of each SVZ neural progenitors in the young adult mouse brain in several papers. Particularly, we based our interpretation on our SVZ irradiation model reported in Daynac et al. 2013 demonstrating a radio resistance of qNSC re-entering into the cell cycle as early as 2 days after 4Gy irradiation successively regenerating aNSC, TAP then iNB and mNB.

      R2C25: Fig S3 - These results effectively show that the s-aNSCs and s-TAPs are actually less specific when compared to that same identity in other studies, and that the iNBs are most similar to mitotic TAPs. This supports what was mentioned above, which is that the transcriptional signatures are very similar between the s-TAPs and i-NBs, showing these are not a unique cell state, but just a bit further along mitosis within the TAP cell state.

      Answer: This is now the Figure S5. In this figure, we show that s-iNB expressed genes associated to either TAP or NB that have been described in previous studies, whereas s-TAP did not express genes associated to NB, but look like closer to aNSC. As indicated above in R2C23, s-iNB are not just a bit further along mitosis within the TAP cell state. Indeed, we give several data showing that s-iNB and s-TAP have different transcriptomic profiles.

      R2C26: Fig 4B - The focus on Ptbp1 as being associated with the iNB cluster border to mNB is expected as all previous studies of Ptbp1 have focused on its role in the progression of other cell types through the cell cycle, its control of cell cycle regulators, and a cell cycle mRNA regulon (Monzon-Casanova et al, 2018, 2019, 2020). This further supports these analyses are specifically defined by cell cycle stages.

      Answer: We totally agree that Ptbp1 expression distinguishes cycling cells from postmitotic neuroblasts in accordance with previously published paper, and that based on this unique gene we cannot find any differences between cycling cells ie. aNSC, TAP and iNB. However, as shown in the manuscript and stated above (R2C23 and 25), these cells can be distinguished by their respective expression of many other genes, including other RSR genes.

      R2C27: Line 281-282 is an overstatement - the authors suggest that this is a new type of cycling neural progenitor - when all studies point to it being the end of mitosis TAPs as they go on their way to mNBs. This clearly shows a trajectory and not a defined, binary cell type.

      Answer: We agree with this statement that the use of the word "type" was misleading, and changed it to "stage" to better reflect that s-iNB are a distinct stage along the differentiation process according to our pseudotime cell-trajectory analysis.

      Author response image 2.

      Pseudotime analysis using Monocle 3 (excluding the cluster 13 corresponding to astrocytes and starting from s-qNSC) revealed two branches starting from s-TAP, one towards cell cycle the other towards neuronal differentiation.

      minor comments:

      R2C28: Fig 3D - For ease, please define what you called the clusters in 3D - not just cluster numbers

      Answer: We chose not to call the clusters in 3D because their identification (Group names) is based on data presented after in Figures 3E, F and G.

      R2C29: Fig 3E-F - Show astrocytes by text in 3E and F

      Answer: As discussed above, astrocytes cannot be shown in these figures because they are based on our signatures which did not include astrocyte signature.

    1. Author Response

      The following is the authors’ response to the original reviews.

      eLife Assessment

      This study investigated the factors related to understudied genes in biomedical research. It showed that understudied genes are largely abandoned at the writing stage, and it identified a number of biological and experimental factors that influence which genes are selected for investigation. The study is a valuable contribution to this branch of meta-research, and while the evidence in support of the findings is solid, the interpretation and presentation of the results (especially the figures) needs to be improved.

      We thank the editor and reviewers for their detailed and thoughtful assessment of our work. Below, we present detailed responses to reviewers’ comments and suggestions. We are also submitting a version edited for clarity of presentation and precision of interpretation.

      Following the eLife assessment, we also tried to identify further statements where results could be presented in a more precise way.

      First, in the section Subsequent reception by other scientists does not penalize studies on understudied genes, we now state “This result again opposes the hypothesis that less-investigated genes will yield articles with lower impact.”

      Second, in section Identification of biological and experimental factors associated with selection of highlighted genes, we now state:

      “We cautiously hypothesize that this might reflect on many different research groups producing reagents surrounding the genes that they actively study. The most informative continuous factor is the number of research articles about a gene (Figure 1B).”, removing claims of causality.

      Finally, for improved readability, we have moved all supplemental tables into separate .xlsx files.

      Reviewer #1 (Public Review):

      Summary and strengths

      The authors tried to address why only a subset of genes are highlighted in many publications. Is it because these highlighted genes are more important than others? Or is it because there are non-genetic reasons? This is a critical question because in the effort to discover new genes for drug targets and clinical benefit, we need to expand a pool of genes for deep analyses. So I appreciate the authors' efforts in this study, as it is timely and important. They also provided a framework called FMUG (short for Find My Understudied Gene) to evaluate genes for a number of features for subsequent analyses.

      We thank the reviewer for their insightful comments and are pleased that the reviewer shares our appreciation for the gravity of these questions. As the reviewer emphasizes, it is critical to understand whether the choice of genes reflects their importance or non-genetic reasons. Previously we and others demonstrated that this choice does not reflect biological importance, when the latter is assessed through unbiased genome-wide data (e.g.: Haynes et al., 2018; Stoeger et al. 2018). Now we contribute to this critical question by systematically evaluating individual non-genetic reasons. We address the reviewer’s comments below.

      Weaknesses

      Many of the figures are hard to comprehend, and the figure legends do not sufficiently explain them.

      For example, what was plotted in Fig 1b? The number of articles increased from results -> write-ups -> follow-ups in all four categories with different degrees. But it does not seem to match what the authors meant to deliver.

      We apologize for the lack of clarity. We identified two interrelated elements that we have now fixed: i) the prior figure legend provided for each genomics approach n number of articles, such as “GWAS (n=450 articles)”; ii) the prior y-axis was labelled “Number of articles”.

      Addressing the first element, we now rephrased the legend for clarity:

      “b, We identified articles reporting on genome-wide CRISPR screens (CRISPR, 15 focus articles and 18 citing articles), transcriptomics (T-omics, 148 focus articles and 1,678 citing articles), affinity purification–mass spectrometry (AP-MS, 296 focus articles and 1,320 citing articles), and GWAS (450 focus articles and 3,524 citing articles). Focusing only on protein-coding genes (white box plot), we retrieved data uploaded to repositories describing which genes came up as “hits” in each experiment (first colored box plot). We then retrieved the hits mentioned in the titles and abstracts of those articles (second colored box plot) and hits mentioned in the titles and abstracts of articles citing those articles (third colored box plot). Unique hit genes are only counted once.”

      The number of genes in each box plot is now reported in the x-axis labels for each step. For example, the results for CRISPR were obtained from 15 focus studies (original research) and 18 subsequent studies (papers citing focus articles). Those 15 studies identified 9,268 genes where loss-of-function changed phenotypes but, in their titles and abstracts, mentioned only 18 of those 9,268 genes. While the 9,268 hit genes have received similar research attention to the entirety of protein-coding genes, the 18 hit genes mentioned in the title or abstract are significantly more well studied. The articles citing the focus articles also only mentioned in their titles and abstracts 19 highly studied hit genes.

      Addressing the second element, we updated the axis label to “Number of articles about gene”, to distinguish it from number of articles mentioned in the legend, convey that this is the number of articles about each gene that were published independently of the genomics assays we inspect. To further underscore this point we now label the “20% highest-studied genes” that we mention in the main text, and reworded the figure caption to better capture where the critical increase occurs: “A shift in focus towards well-studied genes occurs during the summarization and write-up of results and remains in subsequent studies.”.

      Fig 4 is also confusing. It appears that the genes were clustered by many features that the authors developed. But does it have any relationship with genes being under- or over-studied?

      We again apologize for the lack of clarity. As is described in the main text, while the results of Figs. 1-2 suggest that gene popularity may be predict the highlighting of a differentially expressed gene in the title or abstract, we want to conduct a systematically analysis of the factors that correlate with such a decision. We thus build a set of 45 factors that have been discussed as factors explaining why some genes receive increased research attention.

      The data in Fig. 4 shows that those 45 factors are not independent but that some are highly correlated. Because of those correlations, we are able to select a smaller number as representative of the full set. Those are the default factors shown to users of FMUG. While users can choose all factors that are significantly correlated with the highlighting in title or abstract, the default of presenting factors representing different clusters of factors enabled us to limit the number of factors that are initially displayed.

      Please note that following the suggestion of Reviewer 3, we have now moved this Figure to the supplemental material, as Figure S11.

      Reviewer #2 (Public Review)

      Summary and strengths

      In this manuscript the authors analyse the trajectory of understudied genes (UGs) from experiment to publication and study the reasons for why UGs remain underrepresented in the scientific literature. They show that UGs are not underrepresented in experimental datasets, but in the titles and abstracts of the manuscripts reporting experimental data as well as subsequent studies referring to those large-scale studies. They also develop an app that allows researchers to find UGs and their annotation state. Overall, this is a timely article that makes an important contribution to the field. It could help to boost the future investigation of understudied genes, a fundamental challenge in the life sciences. It is concise and overall well-written, and I very much enjoyed reading it. However, there are a few points that I think the authors should address.

      We thank the reviewer for their kind assessment.

      Weaknesses

      The authors conclude that many UGs "are lost" from genome-wide assay at the manuscript writing stage. If I understand correctly, this is based on gene names not being reported in the title or abstract of these manuscripts. However, for genome-wide experiments, it would be quite difficult for authors to mention large numbers of understudied genes in the abstract. In contrast, one might highlight the expected behaviour of a well-studied protein simply to highlight that the genome-wide study provides credible results.

      We agree that it is not reasonable to expect a title or abstract to highlight hundreds or even thousands of differentially expressed genes. We’ve now extended our Study Limitations section to address this:

      “we take a gene being mentioned in the title or abstract of an article as a proxy for a gene receiving attention by the article’s authors. The title and abstract are space-limited and thus cannot accommodate discussion of large numbers of genes.”

      We also agree that highlighting the expected behavior of a well-studied protein may provide credibility to a study and increase confidence on other results. The soundness of such a strategy was quantitatively studied in a study by Uzzi et al. (Science 2013), which we now include in the section on study limitations as:

      “authors beginning manuscripts with something familiar before introducing something new”.

      To convey the practical limitation of abstracts needing to be concise, we added the following sentence to our discussion section, when suggesting controlled trials that add genes to abstracts:

      “This intervention would need to be carefully designed since abstracts are limited in their size.”

      To avoid over-interpretation we have in the discussion also extended the sentence on “lost in a leaky pipeline” to “lost to titles and abstracts of research articles in a leaky pipeline”.

      Our focus on titles and abstracts has been equally motivated by their availability (full text still is often behind paywalls and/or not accessible for bulk-download and text-mining) and by abstracts being the most visible and most read parts of research articles (e.g.: bioRxiv estimates that for the preprint for the present manuscript, the abstract was read ~10 times more frequently than full-text HTML and 4 times more frequently than the pdf).

      Could this bias the authors' conclusions and, if so, how could this be addressed? For example, would it be worth to normalise studies based on the total number of genes they cover?

      We previously described that – in line with the reviewer’s expectations – unstudied genes are preferentially added to the title or abstract of articles that feature more genes in the title or abstract (Stoeger et al., Plos Biology, 2022; Fig. 2B). Normalizing by the total number of genes should thus preserve the pronounced division between well-studied genes and unstudied genes show in Figure 1B. In line with these predictions, we randomly select one gene per title/abstract and find that the effect remains (see new Figure S7).

      Author response image 1.

      Figure 1B is confusing in its present form. I think the plot and/or the legend need revising. For example, what "numbers to the right of each box plot" are the authors referring to? Also, I assume that the filled boxes are understudied genes and the empty/white box is "all genes", but that's not explained in the legend. In the main text, the figure is referred to with the sentence "we found that hit genes that are highlighted in the title or abstract are strongly over-represented among the 20% highest-studied genes in all biomedical literature ". I cannot follow how the figure shows this. My interpretation is that the y-axis is not showing the number of articles, but represents the percentage of articles mentioning a gene in the title/abstract, displayed on a log scale. If so, perhaps a better axis labels and legend text could be sufficient. But then one would also need to somehow connect this to the statement in the main text about the 20% highest-studied genes (a dashed line?). Alternatively, the authors could consider other ways of plotting these data, e.g. simply plotting the "% of publication in which a gene appears" from 0-100% or so.

      Reviewer 1 raised a similar point on overall figure clarity. We identified two interrelated elements that contribute to overall confusion and have now fixed them (see response to Reviewer 1 beginning on page 2 of this document).

      We attempted an alternative plotting of Fig 1B according to the reviewer’s suggestion. In the version below, the y-axis instead shows the percent of gene-related articles that are about each gene. We chose to keep the original y-axis (showing number of articles about each gene) as it additionally conveys the absolute scale of scholarship on individual genes.

      Author response image 2.

      Reviewer #3 (Public Review):

      Summary and strengths

      The manuscript investigated the factors related to understudied genes in biomedical research. It showed that understudied are largely abandoned at the writing stage and identified biological and experimental factors associated with selection of highlighted genes.

      It is very important for the research community to recognize the systematic bias in research of human genes and take precautions when designing experiments and interpreting results. The authors have tried to profile this issue comprehensively and promoted more awareness and investigation of understudied genes.

      We thank the reviewer for their kind assessment of our work.

      Weaknesses

      Regarding result section 1 "Understudied genes are abandoned at synthesis/writing stage", the figures are not clear and do not convey the messages written in the main text. For example, in Figure 1B, figure S5 and S6,

      • There is no "numbers to the right of each box plot".

      The “numbers to the right” statement in the caption was an erroneous inclusion from an earlier version of the figure. We apologize for our error and have now removed this statement.

      • Do these box plots only show understudied genes? How many genes are there in each box plot? The definition and numbers of understudied genes are not clear.

      The x-axis describes genes featured in each stage of the publication process (from all protein-coding genes to genes found as hits in genome-wide screen to genes found in the title/abstract to genes found in the title/abstract of citing articles) and the y-axis describes the number of articles annotated to those genes. We have also now added the number of genes in each box plot to the figure. This information is also in Materials and Methods under each technology’s heading (see also response to Reviewer 1 beginning on page 2 of this document).

      Author response image 3.

      • "We found that hit genes that are highlighted in the title or abstract are strongly over-represented among the 20% highest-studied genes in all biomedical literature (Figure 1B)". This is not clear from the figure.

      We have revised Figure 1B and its caption to better communicate the main point of the figure: that genes which make it to the title/abstract of the reporting article tend to be more popular than genes which are hits in genome-wide experiments from those articles. We have added a horizontal line that shows the cutoff for the top 20% most popular genes.

      Regarding result section 2 "Subsequent reception by other scientists does not penalize studies on understudied genes", the authors showed in figure 2 that there is a negative correlation between articles per gene before 2015 and median citations to articles published in 2015. Another explanation could be that for popular genes, there are more low-quality articles that didn't get citations, not necessarily that less popular genes attract more citations.

      We believe that both explanations for the observed phenomenon are not mutually exclusive. Previously, we focused on the median of citations to articles about a gene to capture the typical effect. In a new analysis, we also find support for the possibility outlined by the reviewer and believe that adding this to our manuscript complements and balances our analysis of citations. Specifically, in the new Figure S8B we find that most popular genes are slightly more likely to be among least cited papers (and in Figure S8A that the least studied genes have been much more likely to be among the most cited papers). In-text, we state:

      “Further, since 1990, articles about the least popular genes have at times been 3 to 4 times more likely to be among the most cited articles than articles on the most popular genes whereas articles on the most popular genes have been slightly less to be highly cited than lowly cited (Figure S8)”.

      We thank the reviewer for their suggestion, which strengthens our manuscript. The figure caption reads:

      “Figure S8: Likelihoods of being highly cited (top 5% of citations among all articles about genes, panel a) or lowly cited (bottom 5% of citations among all articles about genes, panel b) for articles about the most popular genes (top 5% accumulated articles) versus articles about the least popular genes (bottom 5% accumulated articles) by year of publication. Only articles with a single gene in the title/abstract are considered. Shaded regions show ±1 standard error of the proportion."

      Author response image 4.

      Regarding result section 3 "Identification of biological and experimental factors associated with selection of highlighted genes", in Figure 3 and table s2, the author stated that "hits with a compound known to affect gene activity are 5.114 times as likely to be mentioned in the title/abstract in an article using transcriptomics", The number 5.144 comes out of nowhere both in the figure and the table. In addition, figure 4 is not informative enough to be included as a main figure.

      This is the result of both a typo and imprecise terminology. The number should read 4.262 (the likelihood ratio of being mentioned in the title/abstract between genes with and without a compound), which corresponds to an odds ratio of 4.331. We have clarified this in the table caption, stating:

      “e.g. hits with a compound known to affect gene activity are 4.262 times as likely to be mentioned in the title/abstract in an article using transcriptomics, corresponding to an odds ratio of 4.331".

      We have removed Figure 4 as a main-text figure and added a version, with revised color scheme along comments of Reviewer 1, as Figure S11. We added to the figure caption “Bold indicates FMUG ‘s default factors, which we selected based on this clustering and based on their strength of association with gene selection (Figure 3, Table S2 and Table S3)."

      Recommendations for the authors:

      Reviewer #1 (Recommendations for the authors):

      • Fig 2a shows that papers highlighting understudied genes are actually cited more. I wonder why authors only looked at data before 2015. Fig 2b shows an increased correlation since 2015. Please consider redrawing Fig 2a to include data from 2015-2020?

      We highlight data from 2015 since, from our used version of iCite (v32, released July 2022, covering citations made through most of 2021), papers published in 2015 have had about 6 years to accumulate citations. With fewer years to accumulate citations, insufficient signal may cause correlation to converge toward zero. Below, we repeat the analysis in Figure 2 but only considering citations made within a year of an article’s publication, which substantially reduces correlation (although remaining significant).

      Author response image 5.

      We added a note to the figure caption:

      “We forgo depicting more recent years than 2015 to allow for citations to accumulate over multiple years, providing a more sensitive and robust readout of long-term impact.”

      For Figure 2B, we add:

      “For more recent years, where articles have had less time to accumulate citations, insufficient signal may cause correlation to converge toward zero.”

      • Can FMUG be posted on the web for easy access by researchers with non-computational backgrounds?"

      We presently regretfully do not have the resources to create or maintain a web-based version. We hope that the publication of this manuscript will enable us to attract resources to create and maintain a web-based version.

      Reviewer #2 (Recommendations for the authors):

      • Related to the first weakness in my public review: The observed disparity between CRISPR and GWAS study in terms of which genes they promote to the abstract is interesting. I wonder if this has to do with the application of these techniques. GWAS studies will often highlight that they retrieve known associations between a gene and a phenotype, to show that a screen is working. I guess often the point is to subsequently identify more genes associated with a particular phenotype, but often it is unclear how to validate/verify newly found associations. In contrast, CRISPR screens might be more focussed on functionally/mechanistically understanding unknown processes, e.g. observing a phenotype that appears/disappears in response to a gene deletion. In such studies, the follow-up of a previously unknown gene could be more straightforward and relevant to the outcome. Does that mean CRIPSR screens are better than GWAS studies for addressing the UG problem? Perhaps the authors could briefly discuss this issue.

      The number of studies we included featuring CRISPR screens is relatively small (n = 15 compared to n = 450 for GWAS). Thus, it is not possible to conclude in a statistically sound manner whether authors of CRISPR screens are truly more likely to highlight understudied genes.

      However, the reviewer raises compelling reasons for why this might be the case, and we now embed the broader discussion point that some techniques might be more powerful toward understudied genes.

      The discussion now includes:

      “Further, the observed discrepancy between the popularity of hits highlighted by GWAS versus other technologies suggests that some -omics technologies may be more powerful than others for characterizing understudied genes. This possibility merits further research and researchers participating in unknomics should consider the relative strengths of each technology towards providing tractable results for follow-up.”

      • Affinity capture mass spectrometry (Aff-MS): Perhaps I misunderstood this, but typically this is referred to as affinity purification MS (AP-MS)

      Thank you for the clarification. We have changed ‘Aff-MS’ to ‘AP-MS’ throughout the manuscript.

      • Page 3, line 96. The sentence "The first possibility is that seemingly understudied genes are, in fact, not understudied as they would rarely be identified through experiments.". Would they not still be understudied, just not intentionally?

      We have rephrased this sentence to:

      “The first possibility is that some genes are less studied because they are rarely identified as hits in experiments.”

      • Fig 4 is very interesting, but I also found it a bit confusing. First, the choice of colour scheme, where blue shows the absence and white shows the presence of something, seems counterintuitive, especially on a white background. Second, I find it confusing that only some of the experiments are labelled in the heatmap. Could the authors not simply use Fig S9 as Fig 4? Or alternatively, only include the 8 labelled factors in the simplified figure.

      In line with this feedback and that of Review #1 and #3, we have removed Figure 4 as a main-text figure and instead include this figure as Supplementary Figure S11. We have reversed the color scheme so that purple indicates one and white indicates zero. We also now label all factors. Previously we had only listed the default features of FMUG. We also now updated the figure legend to convey how it assisted the choice of default factors in FMUG. It reads:

      “Bold indicates FMUG ‘s default factors, which we selected based on this clustering and based on their strength of association with gene selection (Figure 3, Table S2 and Table S3)”.

      • The FMUG app is fantastic and sounds exactly like something that is required to boost the visibility of understudied genes and overcome the understudied gene bias. However, I did not understand the choice of reporting this in the Discussion section.

      We thank the reviewer for their enthusiasm, and have now moved FMUG into the results section.

      • To further increase usability of the FMUG app, is there a way it could be deployed online? I appreciate this could require a major amount of coding work, which would not be reasonable to demand. So please consider this a suggestion, potentially for a future implementation.

      We presently regretfully do not have the resources to create or maintain a web-based version. We hope that the publication of this manuscript will enable us to attract resources to create and maintain a web-based version.

      Reviewer #3 (Recommendations for the authors):

      Table s2 and s3: p values are indicated by star signs. However, with so many hypothesis tests, the p values should be corrected for multiple tests.

      We have now applied Benjamini-Hochberg multiple hypothesis correction to these tables, correcting p-values within each of the four technologies. We update our significance calling to read:

      “We identified 45 factors that relate to genes and found 33 (12 out of 23 binary factors and 21 out of 22 continuous factors) associated with selection in at least one assay type at Benjamini-Hochberg FDR < 0.001.”

      Figure S1 - S4

      These figures contain too many noninformative boxes. In all the figures, only the last three boxes are informative (reports assessed for eligibility, reports excluded, and studies included in review). The rest boxes convey little information and should be simplified.

      We have simplified these diagrams, removing boxes which contained no information.

      Figure S6: what does it mean by "prior to the publication of the first article represented in this sample"? What is "this sample"?

      “This sample” refers to the collection of 450 GWAS articles, 296 articles using AP-MS, 148 transcriptomics articles, and 15 genome-wide CRISPR screen articles. We have rephrased this sentence to make this clear. It now reads:

      “Variant of Figure 1B only considering articles published in 2002 or before, prior to the publication of any of the articles featuring -omics experiments which we considered for this analysis.”

    1. Author response:

      The following is the authors’ response to the current reviews.

      eLife Assessment

      This neuroimaging and electrophysiology study in a small cohort of congenital cataract patients with sight recovery aims to characterize the effects of early visual deprivation on excitatory and inhibitory balance in visual cortex. While contrasting sight-recovery with visually intact controls suggested the existence of persistent alterations in Glx/GABA ratio and aperiodic EEG signals, it provided only incomplete evidence supporting claims about the effects of early deprivation itself. The reported data were considered valuable, given the rare study population. However, the small sample sizes, lack of a specific control cohort and multiple methodological limitations will likely restrict usefulness to scientists working in this particular subfield.

      We thank the reviewing editors for their consideration and updated assessment of our manuscript after its first revision.

      In order to assess the effects of early deprivation, we included an age-matched, normally sighted control group recruited from the same community, measured in the same scanner and laboratory. This study design is analogous to numerous studies in permanently congenitally blind humans, which typically recruited sighted controls, but hardly ever individuals with a different, e.g. late blindness history. In order to improve the specificity of our conclusions, we used a frontal cortex voxel in addition to a visual cortex voxel (MRS). Analogously, we separately analyzed occipital and frontal electrodes (EEG).

      Moreover, we relate our findings in congenital cataract reversal individuals to findings in the literature on permanent congenital blindness. Note, there are, to the best of our knowledge, neither MRS nor resting-state EEG studies in individuals with permanent late blindness.

      Our participants necessarily have nystagmus and low visual acuity due to their congenital deprivation phase, and the existence of nystagmus is a recruitment criterion to diagnose congenital cataracts.

      It might be interesting for future studies to investigate individuals with transient late blindness. However, such a study would be ill-motivated had we not found differences between the most “extreme” of congenital visual deprivation conditions and normally sighted individuals (analogous to why earlier research on permanent blindness investigated permanent congenitally blind humans first, rather than permanently late blind humans, or both in the same study). Any result of these future work would need the reference to our study, and neither results in these additional groups would invalidate our findings.

      Since all our congenital cataract reversal individuals by definition had visual impairments, we included an eyes closed condition, both in the MRS and EEG assessment. Any group effect during the eyes closed condition cannot be due to visual acuity deficits changing the bottom-up driven visual activation.

      As we detail in response to review 3, our EEG analyses followed the standards in the field.

      Public Reviews:

      Reviewer (1 (Public review):

      Summary

      In this human neuroimaging and electrophysiology study, the authors aimed to characterise effects of a period of visual deprivation in the sensitive period on excitatory and inhibitory balance in the visual cortex. They attempted to do so by comparing neurochemistry conditions ('eyes open', 'eyes closed') and resting state, and visually evoked EEG activity between ten congenital cataract patients with recovered sight (CC), and ten age-matched control participants (SC) with normal sight.

      First, they used magnetic resonance spectroscopy to measure in vivo neurochemistry from two locations, the primary location of interest in the visual cortex, and a control location in the frontal cortex. Such voxels are used to provide a control for the spatial specificity of any effects, because the single-voxel MRS method provides a single sampling location. Using MR-visible proxies of excitatory and inhibitory neurotransmission, Glx and GABA+ respectively, the authors report no group effects in GABA+ or Glx, no difference in the functional conditions 'eyes closed' and 'eyes open'. They found an effect of group in the ratio of Glx/GABA+ and no similar effect in the control voxel location. They then perform multiple exploratory correlations between MRS measures and visual acuity, and report a weak positive correlation between the 'eyes open' condition and visual acuity in CC participants.

      The same participants then took part in an EEG experiment. The authors selected two electrodes placed in the visual cortex for analysis and report a group difference in an EEG index of neural activity, the aperiodic intercept, as well as the aperiodic slope, considered a proxy for cortical inhibition. Control electrodes in the frontal region did not present with the same pattern. They report an exploratory correlation between the aperiodic intercept and Glx in one out of three EEG conditions.

      The authors report the difference in E/I ratio, and interpret the lower E/I ratio as representing an adaptation to visual deprivation, which would have initially caused a higher E/I ratio. Although intriguing, the strength of evidence in support of this view is not strong. Amongst the limitations are the low sample size, a critical control cohort that could provide evidence for higher E/I ratio in CC patients without recovered sight for example, and lower data quality in the control voxel. Nevertheless, the study provides a rare and valuable insight into experience-dependent plasticity in the human brain.

      Strengths of study

      How sensitive period experience shapes the developing brain is an enduring and important question in neuroscience. This question has been particularly difficult to investigate in humans. The authors recruited a small number of sight-recovered participants with bilateral congenital cataracts to investigate the effect of sensitive period deprivation on the balance of excitation and inhibition in the visual brain using measures of brain chemistry and brain electrophysiology. The research is novel, and the paper was interesting and well written.

      Limitations

      Low sample size. Ten for CC and ten for SC, and further two SC participants were rejected due to lack of frontal control voxel data. The sample size limits the statistical power of the dataset and increases the likelihood of effect inflation.

      In the updated manuscript, the authors have provided justification for their sample size by pointing to prior studies and the inherent difficulties in recruiting individuals with bilateral congenital cataracts. Importantly, this highlights the value the study brings to the field while also acknowledging the need to replicate the effects in a larger cohort.

      Lack of specific control cohort. The control cohort has normal vision. The control cohort is not specific enough to distinguish between people with sight loss due to different causes and patients with congenital cataracts with co-morbidities. Further data from a more specific populations, such as patients whose cataracts have not been removed, with developmental cataracts, or congenitally blind participants, would greatly improve the interpretability of the main finding. The lack of a more specific control cohort is a major caveat that limits a conclusive interpretation of the results.

      In the updated version, the authors have indicated that future studies can pursue comparisons between congenital cataract participants and cohorts with later sight loss.

      MRS data quality differences. Data quality in the control voxel appears worse than in the visual cortex voxel. The frontal cortex MRS spectrum shows far broader linewidth than the visual cortex (Supplementary Figures). Compared to the visual voxel, the frontal cortex voxel has less defined Glx and GABA+ peaks; lower GABA+ and Glx concentrations, lower NAA SNR values; lower NAA concentrations. If the data quality is a lot worse in the FC, then small effects may not be detectable.

      In the updated version, the authors have added more information that informs the reader of the MRS quality differences between voxel locations. This increases the transparency of their reporting and enhances the assessment of the results.

      Because of the direction of the difference in E/I, the authors interpret their findings as representing signatures of sight improvement after surgery without further evidence, either within the study or from the literature. However, the literature suggests that plasticity and visual deprivation drives the E/I index up rather than down. Decreasing GABA+ is thought to facilitate experience dependent remodelling. What evidence is there that cortical inhibition increases in response to a visual cortex that is over-sensitised to due congenital cataracts? Without further experimental or literature support this interpretation remains very speculative.

      The updated manuscript contains key reference from non-human work to justify their interpretation.

      Heterogeneity in patient group. Congenital cataract (CC) patients experienced a variety of duration of visual impairment and were of different ages. They presented with co-morbidities (absorbed lens, strabismus, nystagmus). Strabismus has been associated with abnormalities in GABAergic inhibition in the visual cortex. The possible interactions with residual vision and confounds of co-morbidities are not experimentally controlled for in the correlations, and not discussed.

      The updated document has addressed this caveat.

      Multiple exploratory correlations were performed to relate MRS measures to visual acuity (shown in Supplementary Materials), and only specific ones shown in the main document. The authors describe the analysis as exploratory in the 'Methods' section. Furthermore, the correlation between visual acuity and E/I metric is weak, not corrected for multiple comparisons. The results should be presented as preliminary, as no strong conclusions can be made from them. They can provide a hypothesis to test in a future study.

      This has now been done throughout the document and increases the transparency of the reporting.

      P.16 Given the correlation of the aperiodic intercept with age ("Age negatively correlated with the aperiodic intercept across CC and SC individuals, that is, a flattening of the intercept was observed with age"), age needs to be controlled for in the correlation between neurochemistry and the aperiodic intercept. Glx has also been shown to negatively correlates with age.

      This caveat has been addressed in the revised manuscript.

      Multiple exploratory correlations were performed to relate MRS to EEG measures (shown in Supplementary Materials), and only specific ones shown in the main document. Given the multiple measures from the MRS, the correlations with the EEG measures were exploratory, as stated in the text, p.16, and in Fig.4. yet the introduction said that there was a prior hypothesis "We further hypothesized that neurotransmitter changes would relate to changes in the slope and intercept of the EEG aperiodic activity in the same subjects." It would be great if the text could be revised for consistency and the analysis described as exploratory.

      This has been done throughout the document and increases the transparency of the reporting.

      The analysis for the EEG needs to take more advantage of the available data. As far as I understand, only two electrodes were used, yet far more were available as seen in their previous study (Ossandon et al., 2023). The spatial specificity is not established. The authors could use the frontal cortex electrode (FP1, FP2) signals as a control for spatial specificity in the group effects, or even better, all available electrodes and correct for multiple comparisons. Furthermore, they could use the aperiodic intercept vs Glx in SC to evaluate the specificity of the correlation to CC.

      This caveat has been addressed. The authors have added frontal electrodes to their analysis, providing an essential regional control for the visual cortex location.

      Comments on the latest version:

      The authors have made reasonable adjustments to their manuscript that addressed most of my comments by adding further justification for their methodology, essential literature support, pointing out exploratory analyses, limitations and adding key control analyses. Their revised manuscript has overall improved, providing valuable information, though the evidence that supports their claims is still incomplete.

      We thank the reviewer for suggesting ways to improve our manuscript and carefully reassessing our revised manuscript.

      Reviewer 2 (Public review):

      Summary:

      The study examined 10 congenitally blind patients who recovered vision through the surgical removal of bilateral dense cataracts, measuring neural activity and neuro chemical profiles from the visual cortex. The declared aim is to test whether restoring visual function after years of complete blindness impacts excitation/inhibition balance in the visual cortex.

      Strengths:

      The findings are undoubtedly useful for the community, as they contribute towards characterising the many ways in which this special population differs from normally sighted individuals. The combination of MRS and EEG measures is a promising strategy to estimate a fundamental physiological parameter - the balance between excitation and inhibition in the visual cortex, which animal studies show to be heavily dependent upon early visual experience. Thus, the reported results pave the way for further studies, which may use a similar approach to evaluate more patients and control groups.

      Weaknesses:

      The main methodological limitation is the lack of an appropriate comparison group or condition to delineate the effect of sight recovery (as opposed to the effect of congenital blindness). Few previous studies suggested that Excitation/Inhibition ratio in the visual cortex is increased in congenitally blind patients; the present study reports that E/I ratio decreases instead. The authors claim that this implies a change of E/I ratio following sight recovery. However, supporting this claim would require showing a shift of E/I after vs. before the sight-recovery surgery, or at least it would require comparing patients who did and did not undergo the sight-recovery surgery (as common in the field).

      We thank the reviewer for suggesting ways to improve our manuscript and carefully reassessing our revised manuscript.

      Since we have not been able to acquire longitudinal data with the experimental design of the present study in congenital cataract reversal individuals, we compared the MRS and EEG results of congenital cataract reversal individuals  to published work in congenitally permanent blind individuals. We consider this as a resource saving approach. We think that the results of our cross-sectional study now justify the costs and enormous efforts (and time for the patients who often have to travel long distances) associated with longitudinal studies in this rare population.

      There are also more technical limitations related to the correlation analyses, which are partly acknowledged in the manuscript. A bland correlation between GLX/GABA and the visual impairment is reported, but this is specific to the patients group (N=10) and would not hold across groups (the correlation is positive, predicting the lowest GLX/GABA ratio values for the sighted controls - opposite of what is found). There is also a strong correlation between GLX concentrations and the EEG power at the lowest temporal frequencies. Although this relation is intriguing, it only holds for a very specific combination of parameters (of the many tested): only with eyes open, only in the patients group.

      Given the exploratory nature of the correlations, we do not base the majority of our conclusions on this analysis. There are no doubts that the reported correlations need replication; however, replication is only possible after a first report. Thus, we hope to motivate corresponding analyses in further studies.

      It has to be noted that in the present study significance testing for correlations were corrected for multiple comparisons, and that some findings replicate earlier reports (e.g. effects on EEG aperiodic slope, alpha power, and correlations with chronological age).

      Conclusions:

      The main claim of the study is that sight recovery impacts the excitation/inhibition balance in the visual cortex, estimated with MRS or through indirect EEG indices. However, due to the weaknesses outlined above, the study cannot distinguish the effects of sight recovery from those of visual deprivation. Moreover, many aspects of the results are interesting but their validation and interpretation require additional experimental work.

      We interpret the group differences between individuals tested years after congenital visual deprivation and normally sighted individuals as supportive of the E/I ratio being impacted by congenital visual deprivation. In the absence of a sensitive period for the development of an E/I ratio, individuals with a transient phase of congenital blindness might have developed a visual system indistinguishable  from normally sighted individuals. As we demonstrate, this is not so. Comparing the results of congenitally blind humans with those of congenitally permanently blind humans (from previous studies) allowed us to identify changes of E/I ratio, which add to those found for congenital blindness.  

      We thank the reviewer for the helpful comments and suggestions related to the first submission and first revision of our manuscript. We are keen to translate some of them into future studies.

      Reviewer 3 (Public review):

      This manuscript examines the impact of congenital visual deprivation on the excitatory/inhibitory (E/I) ratio in the visual cortex using Magnetic Resonance Spectroscopy (MRS) and electroencephalography (EEG) in individuals whose sight was restored. Ten individuals with reversed congenital cataracts were compared to age-matched, normally sighted controls, assessing the cortical E/I balance and its interrelationship and to visual acuity. The study reveals that the Glx/GABA ratio in the visual cortex and the intercept and aperiodic signal are significantly altered in those with a history of early visual deprivation, suggesting persistent neurophysiological changes despite visual restoration.

      First of all, I would like to disclose that I am not an expert in congenital visual deprivation, nor in MRS. My expertise is in EEG (particularly in the decomposition of periodic and aperiodic activity) and statistical methods.

      Although the authors addressed some of the concerns of the previous version, major concerns and flaws remain in terms of methodological and statistical approaches along with the (over)interpretation of the results. Specific concerns include:

      (1 3.1 Response to Variability in Visual Deprivation<br /> Rather than listing the advantages and disadvantages of visual deprivation, I recommend providing at least a descriptive analysis of how the duration of visual deprivation influenced the measures of interest. This would enhance the depth and relevance of the discussion.

      Although Review 2 and Review 3 (see below) pointed out problems in interpreting multiple correlational analyses in small samples, we addressed this request by reporting such correlations between visual deprivation history and measured EEG/MRS outcomes.

      Calculating the correlation between duration of visual deprivation and behavioral or brain measures is, in fact, a common suggestion. The existence of sensitive periods, which are typically assumed to not follow a linear gradual decline of neuroplasticity, does not necessary allow predicting a correlation with duration of blindness. Daphne Maurer has additionally worked on the concept of “sleeper effects” (Maurer et al., 2007), that is, effects on the brain and behavior by early deprivation which are observed only later in life when the function/neural circuits matures.

      In accordance with this reasoning, we did not observe a significant correlation between duration of visual deprivation and any of our dependent variables.

      (2 3.2) Small Sample Size

      The issue of small sample size remains problematic. The justification that previous studies employed similar sample sizes does not adequately address the limitation in the current study. I strongly suggest that the correlation analyses should not feature prominently in the main manuscript or the abstract, especially if the discussion does not substantially rely on these correlations. Please also revisit the recommendations made in the section on statistical concerns.

      In the revised manuscript, we explicitly mention that our sample size is not atypical for the special group investigated, but that a replication of our results in larger samples would foster their impact. We only explicitly mention correlations that survived stringent testing for multiple comparisons in the main manuscript.

      Given the exploratory nature of the correlations, we have not based the majority of our claims on this analysis.

      (3 3.3) Statistical Concerns

      While I appreciate the effort of conducting an independent statistical check, it merely validates whether the reported statistical parameters, degrees of freedom (df), and p-values are consistent. However, this does not address the appropriateness of the chosen statistical methods.

      We did not intend for the statcheck report to justify the methods used for statistics, which we have done in a separate section with normality and homogeneity testing (Supplementary Material S9), and references to it in the descriptions of the statistical analyses (Methods, Page 13, Lines 326-329 and Page 15, Lines 400-402).

      Several points require clarification or improvement:

      (4) Correlation Methods: The manuscript does not specify whether the reported correlation analyses are based on Pearson or Spearman correlation.

      The depicted correlations are Pearson correlations. We will add this information to the Methods.

      (5) Confidence Intervals: Include confidence intervals for correlations to represent the uncertainty associated with these estimates.

      We will add the confidence intervals to the second revision of our manuscript.

      (6) Permutation Statistics: Given the small sample size, I recommend using permutation statistics, as these are exact tests and more appropriate for small datasets.

      Our study focuses on a rare population, with a sample size limited by the availability of participants. Our findings provide exploratory insights rather than make strong inferential claims. To this end, we have ensured that our analysis adheres to key statistical assumptions (Shapiro-Wilk as well as Levene’s tests, Supplementary Material S9),and reported our findings with effect sizes, appropriate caution and context.

      (7) Adjusted P-Values: Ensure that reported Bonferroni corrected p-values (e.g., p > 0.999) are clearly labeled as adjusted p-values where applicable.

      In the revised manuscript, we will change Figure 4 to say ‘adjusted p,’  which we indeed reported.

      (8) Figure 2C

      Figure 2C still lacks crucial information that the correlation between Glx/GABA ratio and visual acuity was computed solely in the control group (as described in the rebuttal letter). Why was this analysis restricted to the control group? Please provide a rationale.

      Figure 2C depicts the correlation between Glx/GABA+ ratio and visual acuity in the congenital cataract reversal group, not the control group. This is mentioned in the Figure 2 legend, as well as in the main text where the figure is referred to (Page 18, Line 475).

      The correlation analyses between visual acuity and MRS/EEG measures were only performed in the congenital cataract reversal group since the sighed control group comprised of individuals with vision in the normal range; thus this analyses would not make sense. Table 1 with the individual visual acuities for all participants, including the normally sighted controls, shows the low variance in the latter group.  

      For variables in which no apiori group differences in variance were predicted, we performed the correlation analyses across groups (see Supplementary Material S12, S15).

      We will highlight these motivations more clearly in the Methods of the revised manuscript.

      (9 3.4) Interpretation of Aperiodic Signal

      Relying on previous studies to interpret the aperiodic slope as a proxy for excitation/inhibition (E/I) does not make the interpretation more robust.

      How to interpret aperiodic EEG activity has been subject of extensive investigation. We cite studies which provide evidence from multiple species (monkeys, humans) and measurements (EEG, MEG, ECoG), including studies which pharmacologically manipulated E/I balance.

      Whether our findings are robust, in fact, requires a replication study. Importantly, we analyzed the intercept of the aperiodic activity fit as well, and discuss results related to the intercept.

      Quote:

      “3.4 Interpretation of aperiodic signal:

      - Several recent papers demonstrated that the aperiodic signal measured in EEG or ECoG is related to various important aspects such as age, skull thickness, electrode impedance, as well as cognition. Thus, currently, very little is known about the underlying effects which influence the aperiodic intercept and slope. The entire interpretation of the aperiodic slope as a proxy for E/I is based on a computational model and simulation (as described in the Gao et al. paper).

      Response: Apart from the modeling work from Gao et al., multiple papers which have also been cited which used ECoG, EEG and MEG and showed concomitant changes in aperiodic activity with pharmacological manipulation of the E/I ratio (Colombo et al., 2019; Molina et al., 2020; Muthukumaraswamy & Liley, 2018). Further, several prior studies have interpreted changes in the aperiodic slope as reflective of changes in the E/I ratio, including studies of developmental groups (Favaro et al., 2023; Hill et al., 2022; McSweeney et al., 2023; Schaworonkow & Voytek, 2021) as well as patient groups (Molina et al., 2020; Ostlund et al., 2021).

      - The authors further wrote: We used the slope of the aperiodic (1/f) component of the EEG spectrum as an estimate of E/I ratio (Gao et al., 2017; Medel et al., 2020; Muthukumaraswamy & Liley, 2018). This is a highly speculative interpretation with very little empirical evidence. These papers were conducted with ECoG data (mostly in animals) and mostly under anesthesia. Thus, these studies only allow an indirect interpretation by what the 1/f slope in EEG measurements is actually influenced.

      Response: Note that Muthukumaraswamy et al. (2018) used different types of pharmacological manipulations and analyzed periodic and aperiodic MEG activity in humans, in addition to monkey ECoG (Muthukumaraswamy & Liley, 2018). Further, Medel et al. (now published as Medel et al., 2023) compared EEG activity in addition to ECoG data after propofol administration. The interpretation of our results are in line with a number of recent studies in developing (Hill et al., 2022; Schaworonkow & Voytek, 2021) and special populations using EEG. As mentioned above, several prior studies have used the slope of the 1/f component/aperiodic activity as an indirect measure of the E/I ratio (Favaro et al., 2023; Hill et al., 2022; McSweeney et al., 2023; Molina et al., 2020; Ostlund et al., 2021; Schaworonkow & Voytek, 2021), including studies using scalp-recorded EEG from humans.

      In the introduction of the revised manuscript, we have made more explicit that this metric is indirect (Page 3, Line 91), (additionally see Discussion, Page 24, Lines 644-645, Page 25, Lines 650-657).

      While a full understanding of aperiodic activity needs to be provided, some convergent ideas have emerged. We think that our results contribute to this enterprise, since our study is, to the best of our knowledge, the first which assessed MRS measured neurotransmitter levels and EEG aperiodic activity.“

      (10) Additionally, the authors state:

      "We cannot think of how any of the exploratory correlations between neurophysiological measures and MRS measures could be accounted for by a difference e.g. in skull thickness."

      (11) This could be addressed directly by including skull thickness as a covariate or visualizing it in scatterplots, for instance, by representing skull thickness as the size of the dots.

      We are not aware of any study that would justify such an analysis.

      Our analyses were based on previous findings in the literature.

      Since to the best of our knowledge, no evidence exists that congenital cataracts go together with changes in skull thickness, and that skull thickness might selectively modulate visual cortex Glx/GABA+ but not NAA measures, we decided against following this suggestion.

      Notably, the neurotransmitter concentration reported here is after tissue segmentation of the voxel region. The tissue fraction was shown to not differ between groups in the MRS voxels (Supplementary Material S4). The EEG electrode impedance was lowered to <10 kOhm in every participant (Methods, Page 13, Line 344), and preparation was identical across groups.

      (12 3.5) Problems with EEG Preprocessing and Analysis

      Downsampling: The decision to downsample the data to 60 Hz "to match the stimulation rate" is problematic. This choice conflates subsequent spectral analyses due to aliasing issues, as explained by the Nyquist theorem. While the authors cite prior studies (Schwenk et al., 2020; VanRullen & MacDonald, 2012) to justify this decision, these studies focused on alpha (8-12 Hz), where aliasing is less of a concern compared of analyzing aperiodic signal. Furthermore, in contrast, the current study analyzes the frequency range from 1-20 Hz, which is too narrow for interpreting the aperiodic signal as E/I. Typically, this analysis should include higher frequencies, spanning at least 1-30 Hz or even 1-45 Hz (not 20-40 Hz).

      As mentioned in the Methods (Page 15 Line 376) and the previous response, the pop_resample function used by EEGLAB applies an anti-aliasing filter, at half the resampling frequency (as per the Nyquist theorem https://eeglab.org/tutorials/05_Preprocess/resampling.html). The upper cut off of the low pass filter set by EEGlab prior to down sampling (30 Hz) is still far above the frequency of interest in the current study  (1-20 Hz), thus allowing us to derive valid results.

      Quote:

      “- The authors downsampled the data to 60Hz to "to match the stimulation rate". What is the intention of this? Because the subsequent spectral analyses are conflated by this choice (see Nyquist theorem).

      Response: This data were collected as part of a study designed to evoke alpha activity with visual white-noise, which ranged in luminance with equal power at all frequencies from 1-60 Hz, restricted by the refresh rate of the monitor on which stimuli were presented (Pant et al., 2023). This paradigm and method was developed by VanRullen and colleagues (Schwenk et al., 2020; Vanrullen & MacDonald, 2012), wherein the analysis requires the same sampling rate between the presented frequencies and the EEG data. The downsampling function used here automatically applies an anti-aliasing filter (EEGLAB 2019) .”

      Moreover, the resting-state data were not resampled to 60 Hz. We will make this clearer in the Methods of the revised manuscript.

      Our consistent results of group differences across all three  EEG conditions, thus, exclude any possibility that they were driven by aliasing artifacts.

      The expected effects of this anti-aliasing filter can be seen in the attached Figure R1, showing an example participant’s spectrum in the 1-30 Hz range (as opposed to the 1-20 Hz plotted in the manuscript), clearly showing a 30-40 dB drop at 30 Hz. Any aliasing due to, for example, remaining line noise, would additionally be visible in this figure (as well as Figure 3) as a peak.

      Author response image 1.

      Power spectral density of one congenital cataract-reversal (CC) participant in the visual stimulation condition across all channels. The reduced power at 30 Hz shows the effects of the anti-aliasing filter applied by EEGLAB’s pop_resample function.

      As we stated in the manuscript, and in previous reviews, so far there has been no consensus on the exact range of measuring aperiodic activity. We made a principled decision based on the literature (showing a knee in aperiodic fits of this dataset at 20 Hz) (Medel et al., 2023; Ossandón et al., 2023), data quality (possible contamination by line noise at higher frequencies) and the purpose of the visual stimulation experiment (to look at the lower frequency range by stimulating up to 60 Hz, thereby limiting us to quantifying below 30 Hz), that 1-20 Hz would be the fit range in this dataset.

      Quote:

      “(3) What's the underlying idea of analyzing two separate aperiodic slopes (20-40Hz and 1-19Hz). This is very unusual to compute the slope between 20-40 Hz, where the SNR is rather low.

      "Ossandón et al. (2023), however, observed that in addition to the flatter slope of the aperiodic power spectrum in the high frequency range (20-40 Hz), the slope of the low frequency range (1-19 Hz) was steeper in both, congenital cataract-reversal individuals, as well as in permanently congenitally blind humans."

      Response: The present manuscript computed the slope between 1-20 Hz. Ossandón et al. as well as Medel et al. (2023) found a “knee” of the 1/f distribution at 20 Hz and describe further the motivations for computing both slope ranges. For example, Ossandón et al. used a data driven approach and compared single vs. dual fits and found that the latter fitted the data better. Additionally, they found the best fit if a knee at 20 Hz was used. We would like to point out that no standard range exists for the fitting of the 1/f component across the literature and, in fact, very different ranges have been used (Gao et al., 2017; Medel et al., 2023; Muthukumaraswamy & Liley, 2018).“

      (13) Baseline Removal: Subtracting the mean activity across an epoch as a baseline removal step is inappropriate for resting-state EEG data. This preprocessing step undermines the validity of the analysis. The EEG dataset has fundamental flaws, many of which were pointed out in the previous review round but remain unaddressed. In its current form, the manuscript falls short of standards for robust EEG analysis. If I were reviewing for another journal, I would recommend rejection based on these flaws.

      The baseline removal step from each epoch serves to remove the DC component of the recording and detrend the data. This is a standard preprocessing step (included as an option in preprocessing pipelines recommended by the EEGLAB toolbox, FieldTrip toolbox and MNE toolbox), additionally necessary to improve the efficacy of ICA decomposition (Groppe et al., 2009).

      In the previous review round, a clarification of the baseline timing was requested, which we added. Beyond this request, there was no mention of the appropriateness of the baseline removal and/or a request to provide reasons for why it might not undermine the validity of the analysis.

      Quote:

      “- "Subsequently, baseline removal was conducted by subtracting the mean activity across the length of an epoch from every data point." The actual baseline time segment should be specified.

      Response: The time segment was the length of the epoch, that is, 1 second for the resting state conditions and 6.25 seconds for the visual stimulation conditions. This has been explicitly stated in the revised manuscript (Page 13, Line 354).”

      Prior work in the time (not frequency) domain on event-related potential (ERP) analysis has suggested that the baselining step might cause spurious effects (Delorme, 2023) (although see (Tanner et al., 2016)). We did not perform ERP analysis at any stage. One recent study suggests spurious group differences in the 1/f signal might be driven by an inappropriate dB division baselining method (Gyurkovics et al., 2021), which we did not perform.

      Any effect of our baselining procedure on the FFT spectrum would be below the 1 Hz range, which we did not analyze.  

      Each of the preprocessing steps in the manuscript match pipelines described and published in extensive prior work. We document how multiple aspects of our EEG results replicate prior findings (Supplementary Material S15, S18, S19), reports of other experimenters, groups and locations, validating that our results are robust.

      We therefore reject the claim of methodological flaws in our EEG analyses in the strongest possible terms.

      Quote:

      “3.5 Problems with EEG preprocessing and analysis:

      - It seems that the authors did not identify bad channels nor address the line noise issue (even a problem if a low pass filter of below-the-line noise was applied).

      Response: As pointed out in the methods and Figure 1, we only analyzed data from two occipital channels, O1 and O2 neither of which were rejected for any participant. Channel rejection was performed for the larger dataset, published elsewhere (Ossandón et al., 2023; Pant et al., 2023). As control sites we added the frontal channels FP1 and Fp2 (see Supplementary Material S14)

      Neither Ossandón et al. (2023) nor Pant et al. (2023) considered frequency ranges above 40 Hz to avoid any possible contamination with line noise. Here, we focused on activity between 0 and 20 Hz, definitely excluding line noise contaminations (Methods, Page 14, Lines 365-367). The low pass filter (FIR, 1-45 Hz) guaranteed that any spill-over effects of line noise would be restricted to frequencies just below the upper cutoff frequency.

      Additionally, a prior version of the analysis used spectrum interpolation to remove line noise; the group differences remained stable (Ossandón et al., 2023). We have reported this analysis in the revised manuscript (Page 14, Lines 364-357).

      Further, both groups were measured in the same lab, making line noise (~ 50 Hz) as an account for the observed group effects in the 1-20 Hz frequency range highly unlikely. Finally, any of the exploratory MRS-EEG correlations would be hard to explain if the EEG parameters would be contaminated with line noise.

      - What was the percentage of segments that needed to be rejected due to the 120μV criteria? This should be reported specifically for EO & EC and controls and patients.

      Response: The mean percentage of 1 second segments rejected for each resting state condition and the percentage of 6.25 long segments rejected in each group for the visual stimulation condition have been added to the revised manuscript (Supplementary Material S10), and referred to in the Methods on Page 14, Lines 372-373).

      - The authors downsampled the data to 60Hz to "to match the stimulation rate". What is the intention of this? Because the subsequent spectral analyses are conflated by this choice (see Nyquist theorem).

      Response: This data were collected as part of a study designed to evoke alpha activity with visual white-noise, which changed in luminance with equal power at all frequencies from 1-60 Hz, restricted by the refresh rate of the monitor on which stimuli were presented (Pant et al., 2023). This paradigm and method was developed by VanRullen and colleagues (Schwenk et al., 2020; VanRullen & MacDonald, 2012), wherein the analysis requires the same sampling rate between the presented frequencies and the EEG data. The downsampling function used here automatically applies an anti-aliasing filter (EEGLAB 2019) .

      - "Subsequently, baseline removal was conducted by subtracting the mean activity across the length of an epoch from every data point." The actual baseline time segment should be specified.

      The time segment was the length of the epoch, that is, 1 second for the resting state conditions and 6.25 seconds for the visual stimulation conditions. This has now been explicitly stated in the revised manuscript (Page 14, Lines 379-380).<br /> - "We excluded the alpha range (8-14 Hz) for this fit to avoid biasing the results due to documented differences in alpha activity between CC and SC individuals (Bottari et al., 2016; Ossandón et al., 2023; Pant et al., 2023)." This does not really make sense, as the FOOOF algorithm first fits the 1/f slope, for which the alpha activity is not relevant.

      Response: We did not use the FOOOF algorithm/toolbox in this manuscript. As stated in the Methods, we used a 1/f fit to the 1-20 Hz spectrum in the log-log space, and subtracted this fit from the original spectrum to obtain the corrected spectrum. Given the pronounced difference in alpha power between groups (Bottari et al., 2016; Ossandón et al., 2023; Pant et al., 2023), we were concerned it might drive differences in the exponent values. Our analysis pipeline had been adapted from previous publications of our group and other labs (Ossandón et al., 2023; Voytek et al., 2015; Waschke et al., 2017).

      We have conducted the analysis with and without the exclusion of the alpha range, as well as using the FOOOF toolbox both in the 1-20 Hz and 20-40 Hz ranges (Ossandón et al., 2023). The findings of a steeper slope in the 1-20 Hz range as well as lower alpha power in CC vs SC individuals remained stable. In Ossandón et al., the comparison between the piecewise fits and FOOOF fits led the authors to use the former, as it outperformed the FOOOF algorithm for their data.

      - The model fits of the 1/f fitting for EO, EC, and both participant groups should be reported.

      Response: In Figure 3 of the manuscript, we depicted the mean spectra and 1/f fits for each group.

      In the revised manuscript, we added the fit quality metrics (average R<sup>2</sup> values > 0.91 for each group and condition) (Methods Page 15, Lines 395-396; Supplementary Material S11) and additionally show individual subjects’ fits (Supplementary Material S11).“

      (14) The authors mention:

      "The EEG data sets reported here were part of data published earlier (Ossandón et al., 2023; Pant et al., 2023)." Thus, the statement "The group differences for the EEG assessments corresponded to those of a larger sample of CC individuals (n=38) " is a circular argument and should be avoided."

      The authors addressed this comment and adjusted the statement. However, I do not understand, why not the full sample published earlier (Ossandón et al., 2023) was used in the current study?

      The recording of EEG resting state data stated in 2013, while MRS testing could only be set up by the end of 2019. Moreover, not all subjects who qualify for EEG recording qualify for being scanned (e.g. due to MRI safety, claustrophobia)

      References

      Bottari, D., Troje, N. F., Ley, P., Hense, M., Kekunnaya, R., & Röder, B. (2016). Sight restoration after congenital blindness does not reinstate alpha oscillatory activity in humans. Scientific Reports. https://doi.org/10.1038/srep24683

      Colombo, M. A., Napolitani, M., Boly, M., Gosseries, O., Casarotto, S., Rosanova, M., Brichant, J. F., Boveroux, P., Rex, S., Laureys, S., Massimini, M., Chieregato, A., & Sarasso, S. (2019). The spectral exponent of the resting EEG indexes the presence of consciousness during unresponsiveness induced by propofol, xenon, and ketamine. NeuroImage, 189(September 2018), 631–644. https://doi.org/10.1016/j.neuroimage.2019.01.024

      Delorme, A. (2023). EEG is better left alone. Scientific Reports, 13(1), 2372. https://doi.org/10.1038/s41598-023-27528-0

      Favaro, J., Colombo, M. A., Mikulan, E., Sartori, S., Nosadini, M., Pelizza, M. F., Rosanova, M., Sarasso, S., Massimini, M., & Toldo, I. (2023). The maturation of aperiodic EEG activity across development reveals a progressive differentiation of wakefulness from sleep. NeuroImage, 277. https://doi.org/10.1016/J.NEUROIMAGE.2023.120264

      Gao, R., Peterson, E. J., & Voytek, B. (2017). Inferring synaptic excitation/inhibition balance from field potentials. NeuroImage, 158(March), 70–78. https://doi.org/10.1016/j.neuroimage.2017.06.078

      Groppe, D. M., Makeig, S., & Kutas, M. (2009). Identifying reliable independent components via split-half comparisons. NeuroImage, 45(4), 1199–1211. https://doi.org/10.1016/j.neuroimage.2008.12.038

      Gyurkovics, M., Clements, G. M., Low, K. A., Fabiani, M., & Gratton, G. (2021). The impact of 1/f activity and baseline correction on the results and interpretation of time-frequency analyses of EEG/MEG data: A cautionary tale. NeuroImage, 237. https://doi.org/10.1016/j.neuroimage.2021.118192

      Hill, A. T., Clark, G. M., Bigelow, F. J., Lum, J. A. G., & Enticott, P. G. (2022). Periodic and aperiodic neural activity displays age-dependent changes across early-to-middle childhood. Developmental Cognitive Neuroscience, 54, 101076. https://doi.org/10.1016/J.DCN.2022.101076

      Maurer, D., Mondloch, C. J., & Lewis, T. L. (2007). Sleeper effects. In Developmental Science. https://doi.org/10.1111/j.1467-7687.2007.00562.x

      McSweeney, M., Morales, S., Valadez, E. A., Buzzell, G. A., Yoder, L., Fifer, W. P., Pini, N., Shuffrey, L. C., Elliott, A. J., Isler, J. R., & Fox, N. A. (2023). Age-related trends in aperiodic EEG activity and alpha oscillations during early- to middle-childhood. NeuroImage, 269, 119925. https://doi.org/10.1016/j.neuroimage.2023.119925

      Medel, V., Irani, M., Crossley, N., Ossandón, T., & Boncompte, G. (2023). Complexity and 1/f slope jointly reflect brain states. Scientific Reports, 13(1), 21700. https://doi.org/10.1038/s41598-023-47316-0

      Molina, J. L., Voytek, B., Thomas, M. L., Joshi, Y. B., Bhakta, S. G., Talledo, J. A., Swerdlow, N. R., & Light, G. A. (2020). Memantine Effects on Electroencephalographic Measures of Putative Excitatory/Inhibitory Balance in Schizophrenia. Biological Psychiatry: Cognitive Neuroscience and Neuroimaging, 5(6), 562–568. https://doi.org/10.1016/j.bpsc.2020.02.004

      Muthukumaraswamy, S. D., & Liley, D. T. (2018). 1/F electrophysiological spectra in resting and drug-induced states can be explained by the dynamics of multiple oscillatory relaxation processes. NeuroImage, 179(November 2017), 582–595. https://doi.org/10.1016/j.neuroimage.2018.06.068

      Ossandón, J. P., Stange, L., Gudi-Mindermann, H., Rimmele, J. M., Sourav, S., Bottari, D., Kekunnaya, R., & Röder, B. (2023). The development of oscillatory and aperiodic resting state activity is linked to a sensitive period in humans. NeuroImage, 275, 120171. https://doi.org/10.1016/J.NEUROIMAGE.2023.120171

      Ostlund, B. D., Alperin, B. R., Drew, T., & Karalunas, S. L. (2021). Behavioral and cognitive correlates of the aperiodic (1/f-like) exponent of the EEG power spectrum in adolescents with and without ADHD. Developmental Cognitive Neuroscience, 48, 100931. https://doi.org/10.1016/j.dcn.2021.100931

      Pant, R., Ossandón, J., Stange, L., Shareef, I., Kekunnaya, R., & Röder, B. (2023). Stimulus-evoked and resting-state alpha oscillations show a linked dependence on patterned visual experience for development. NeuroImage: Clinical, 103375. https://doi.org/10.1016/J.NICL.2023.103375

      Schaworonkow, N., & Voytek, B. (2021). Longitudinal changes in aperiodic and periodic activity in electrophysiological recordings in the first seven months of life. Developmental Cognitive Neuroscience, 47. https://doi.org/10.1016/j.dcn.2020.100895

      Schwenk, J. C. B., VanRullen, R., & Bremmer, F. (2020). Dynamics of Visual Perceptual Echoes Following Short-Term Visual Deprivation. Cerebral Cortex Communications, 1(1). https://doi.org/10.1093/TEXCOM/TGAA012

      Tanner, D., Norton, J. J. S., Morgan-Short, K., & Luck, S. J. (2016). On high-pass filter artifacts (they’re real) and baseline correction (it’s a good idea) in ERP/ERMF analysis. Journal of Neuroscience Methods, 266, 166–170. https://doi.org/10.1016/j.jneumeth.2016.01.002

      Vanrullen, R., & MacDonald, J. S. P. (2012). Perceptual echoes at 10 Hz in the human brain. Current Biology. https://doi.org/10.1016/j.cub.2012.03.050

      Voytek, B., Kramer, M. A., Case, J., Lepage, K. Q., Tempesta, Z. R., Knight, R. T., & Gazzaley, A. (2015). Age-related changes in 1/f neural electrophysiological noise. Journal of Neuroscience, 35(38). https://doi.org/10.1523/JNEUROSCI.2332-14.2015

      Waschke, L., Wöstmann, M., & Obleser, J. (2017). States and traits of neural irregularity in the age-varying human brain. Scientific Reports 2017 7:1, 7(1), 1–12. https://doi.org/10.1038/s41598-017-17766-4


      The following is the authors’ response to the original reviews.

      eLife Assessment

      This potentially useful study involves neuro-imaging and electrophysiology in a small cohort of congenital cataract patients after sight recovery and age-matched control participants with normal sight. It aims to characterize the effects of early visual deprivation on excitatory and inhibitory balance in the visual cortex. While the findings are taken to suggest the existence of persistent alterations in Glx/GABA ratio and aperiodic EEG signals, the evidence supporting these claims is incomplete. Specifically, small sample sizes, lack of a specific control cohort, and other methodological limitations will likely restrict the usefulness of the work, with relevance limited to scientists working in this particular subfield.

      As pointed out in the public reviews, there are very few human models which allow for assessing the role of early experience on neural circuit development. While the prevalent research in permanent congenital blindness reveals the response and adaptation of the developing brain to an atypical situation (blindness), research in sight restoration addresses the question of whether and how atypical development can be remediated if typical experience (vision) is restored. The literature on the role of visual experience in the development of E/I balance in humans, assessed via Magnetic Resonance Spectroscopy (MRS), has been limited to a few studies on congenital permanent blindness. Thus, we assessed sight recovery individuals with a history of congenital blindness, as limited evidence from other researchers indicated that the visual cortex E/I ratio might differ compared to normally sighted controls.

      Individuals with total bilateral congenital cataracts who remained untreated until later in life are extremely rare, particularly if only carefully diagnosed patients are included in a study sample. A sample size of 10 patients is, at the very least, typical of past studies in this population, even for exclusively behavioral assessments. In the present study, in addition to behavioral assessment as an indirect measure of sensitive periods, we investigated participants with two neuroimaging methods (Magnetic Resonance Spectroscopy and electroencephalography) to directly assess the neural correlates of sensitive periods in humans. The electroencephalography data allowed us to link the results of our small sample to findings documented in large cohorts of both, sight recovery individuals and permanently congenitally blind individuals. As pointed out in a recent editorial recommending an “exploration-then-estimation procedure,” (“Consideration of Sample Size in Neuroscience Studies,” 2020), exploratory studies like ours provide crucial direction and specific hypotheses for future work.

      We included an age-matched sighted control group recruited from the same community, measured in the same scanner and laboratory, to assess whether early experience is necessary for a typical excitatory/inhibitory (E/I) ratio to emerge in adulthood. The present findings indicate that this is indeed the case. Based on these results, a possible question to answer in future work, with individuals who had developmental cataracts, is whether later visual deprivation causes similar effects. Note that even if visual deprivation at a later stage in life caused similar effects, the current results would not be invalidated; by contrast, they are essential to understand future work on late (permanent or transient) blindness.

      Thus, we think that the present manuscript has far reaching implications for our understanding of the conditions under which E/I balance, a crucial characteristic of brain functioning, emerges in humans.

      Finally, our manuscript is one of the first few studies that relate MRS neurotransmitter concentrations to parameters of EEG aperiodic activity. Since present research has been using aperiodic activity as a correlate of the E/I ratio, and partially of higher cognitive functions, we think that our manuscript additionally contributes to a better understanding of what might be measured with aperiodic neurophysiological activity.

      Public Reviews:<br /> Reviewer #1 (Public Review):

      Summary:

      In this human neuroimaging and electrophysiology study, the authors aimed to characterize the effects of a period of visual deprivation in the sensitive period on excitatory and inhibitory balance in the visual cortex. They attempted to do so by comparing neurochemistry conditions ('eyes open', 'eyes closed') and resting state, and visually evoked EEG activity between ten congenital cataract patients with recovered sight (CC), and ten age-matched control participants (SC) with normal sight.

      First, they used magnetic resonance spectroscopy to measure in vivo neurochemistry from two locations, the primary location of interest in the visual cortex, and a control location in the frontal cortex. Such voxels are used to provide a control for the spatial specificity of any effects because the single-voxel MRS method provides a single sampling location. Using MR-visible proxies of excitatory and inhibitory neurotransmission, Glx and GABA+ respectively, the authors report no group effects in GABA+ or Glx, no difference in the functional conditions 'eyes closed' and 'eyes open'. They found an effect of the group in the ratio of Glx/GABA+ and no similar effect in the control voxel location. They then performed multiple exploratory correlations between MRS measures and visual acuity, and reported a weak positive correlation between the 'eyes open' condition and visual acuity in CC participants.

      The same participants then took part in an EEG experiment. The authors selected only two electrodes placed in the visual cortex for analysis and reported a group difference in an EEG index of neural activity, the aperiodic intercept, as well as the aperiodic slope, considered a proxy for cortical inhibition. They report an exploratory correlation between the aperiodic intercept and Glx in one out of three EEG conditions.

      The authors report the difference in E/I ratio, and interpret the lower E/I ratio as representing an adaptation to visual deprivation, which would have initially caused a higher E/I ratio. Although intriguing, the strength of evidence in support of this view is not strong. Amongst the limitations are the low sample size, a critical control cohort that could provide evidence for a higher E/I ratio in CC patients without recovered sight for example, and lower data quality in the control voxel.

      Strengths of study:

      How sensitive period experience shapes the developing brain is an enduring and important question in neuroscience. This question has been particularly difficult to investigate in humans. The authors recruited a small number of sight-recovered participants with bilateral congenital cataracts to investigate the effect of sensitive period deprivation on the balance of excitation and inhibition in the visual brain using measures of brain chemistry and brain electrophysiology. The research is novel, and the paper was interesting and well-written.

      Limitations:

      (1.1) Low sample size. Ten for CC and ten for SC, and a further two SC participants were rejected due to a lack of frontal control voxel data. The sample size limits the statistical power of the dataset and increases the likelihood of effect inflation.

      Applying strict criteria, we only included individuals who were born with no patterned vision in the CC group. The population of individuals who have remained untreated past infancy is small in India, despite a higher prevalence of childhood cataract than Germany. Indeed, from the original 11 CC and 11 SC participants tested, one participant each from the CC and SC group had to be rejected, as their data had been corrupted, resulting in 10 participants in each group.

      It was a challenge to recruit participants from this rare group with no history of neurological diagnosis/intake of neuromodulatory medications, who were able and willing to undergo both MRS and EEG. For this study, data collection took more than 2.5 years.

      We took care of the validity of our results with two measures; first, we assessed not just MRS, but additionally, EEG measures of E/I ratio. The latter allowed us to link results to a larger population of CC individuals, that is, we replicated the results of a larger group of 28 additional individuals (Ossandón et al., 2023) in our sub-group.

      Second, we included a control voxel. As predicted, all group effects were restricted to the occipital voxel.

      (1.2) Lack of specific control cohort. The control cohort has normal vision. The control cohort is not specific enough to distinguish between people with sight loss due to different causes and patients with congenital cataracts with co-morbidities. Further data from more specific populations, such as patients whose cataracts have not been removed, with developmental cataracts, or congenitally blind participants, would greatly improve the interpretability of the main finding. The lack of a more specific control cohort is a major caveat that limits a conclusive interpretation of the results.

      The existing work on visual deprivation and neurochemical changes, as assessed with MRS, has been limited to permanent congenital blindness. In fact, most of the studies on permanent blindness included only congenitally blind or early blind humans (Coullon et al., 2015; Weaver et al., 2013), or, in separate studies, only late-blind individuals (Bernabeu et al., 2009). Thus, accordingly, we started with the most “extreme” visual deprivation model, sight recovery after congenital blindness. If we had not observed any group difference compared to normally sighted controls, investigating other groups might have been trivial. Based on our results, subsequent studies in late blind individuals, and then individuals with developmental cataracts, can be planned with clear hypotheses.

      (1.3) MRS data quality differences. Data quality in the control voxel appears worse than in the visual cortex voxel. The frontal cortex MRS spectrum shows far broader linewidth than the visual cortex (Supplementary Figures). Compared to the visual voxel, the frontal cortex voxel has less defined Glx and GABA+ peaks; lower GABA+ and Glx concentrations, lower NAA SNR values; lower NAA concentrations. If the data quality is a lot worse in the FC, then small effects may not be detectable.

      Worse data quality in the frontal than the visual cortex has been repeatedly observed in the MRS literature, attributable to magnetic field distortions (Juchem & Graaf, 2017) resulting from the proximity of the region to the sinuses (recent example: (Rideaux et al., 2022)). Nevertheless, we chose the frontal control region rather than a parietal voxel, given the potential neurochemical changes in multisensory regions of the parietal cortex due to blindness. Such reorganization would be less likely in frontal areas associated with higher cognitive functions. Further, prior MRS studies of the visual cortex have used the frontal cortex as a control region as well (Pitchaimuthu et al., 2017; Rideaux et al., 2022). In the revised manuscript, we more explicitly inform the reader about this data quality difference between regions in the Methods (Pages 11-12, MRS Data Quality/Table 2) and Discussion (Page 25, Lines 644- 647).

      Importantly, while in the present study data quality differed between the frontal and visual cortex voxel, it did not differ between groups (Supplementary Material S6).  

      Further, we checked that the frontal cortex datasets for Glx and GABA+ concentrations were of sufficient quality: the fit error was below 8.31% in both groups (Supplementary Material S3). For reference, Mikkelsen et al. reported a mean GABA+ fit error of 6.24 +/- 1.95% from a posterior cingulate cortex voxel across 8 GE scanners, using the Gannet pipeline. No absolute cutoffs have been proposed for fit errors. However, MRS studies in special populations (I/E ratio assessed in narcolepsy (Gao et al., 2024), GABA concentration assessed in Autism Spectrum Disorder (Maier et al., 2022) have used frontal cortex data with a fit error of <10% to identify differences between cohorts (Gao et al., 2024; Pitchaimuthu et al., 2017). Based on the literature, MRS data from the frontal voxel of the present study would have been of sufficient quality to uncover group differences.

      In the revised manuscript, we added the recently published MRS quality assessment form to the supplementary materials (Supplementary Excel File S1). Additionally, we would like to allude to our apriori prediction of group differences for the visual cortex, but not for the frontal cortex voxel. Finally, EEG data quality did not differ between frontal and occipital electrodes; therefore, lower sensitivity of frontal measures cannot easily explain the lack of group differences for frontal measures.

      (1.4) Because of the direction of the difference in E/I, the authors interpret their findings as representing signatures of sight improvement after surgery without further evidence, either within the study or from the literature. However, the literature suggests that plasticity and visual deprivation drive the E/I index up rather than down. Decreasing GABA+ is thought to facilitate experience-dependent remodelling. What evidence is there that cortical inhibition increases in response to a visual cortex that is over-sensitised due to congenital cataracts? Without further experimental or literature support this interpretation remains very speculative.

      Indeed, higher inhibition was not predicted, which we attempt to reconcile in our discussion section. We base our discussion mainly on the non-human animal literature, which has shown evidence of homeostatic changes after prolonged visual deprivation in the adult brain (Barnes et al., 2015). It is also interesting to note that after monocular deprivation in adult humans, resting GABA+ levels decreased in the visual cortex (Lunghi et al., 2015). Assuming that after delayed sight restoration, adult neuroplasticity mechanisms must be employed, these studies would predict a “balancing” of the increased excitatory drive following sight restoration by a commensurate increase in inhibition (Keck et al., 2017). Additionally, the EEG results of the present study allowed for speculation regarding the underlying neural mechanisms of an altered E/I ratio. The aperiodic EEG activity suggested higher spontaneous spiking (increased intercept) and increased inhibition (steeper aperiodic slope between 1-20 Hz) in CC vs SC individuals (Ossandón et al., 2023).

      In the revised manuscript, we have more clearly indicated that these speculations are based primarily on non-human animal work, due to the lack of human studies on the subject (Page 23, Lines 609-613).

      (1.5) Heterogeneity in the patient group. Congenital cataract (CC) patients experienced a variety of duration of visual impairment and were of different ages. They presented with co-morbidities (absorbed lens, strabismus, nystagmus). Strabismus has been associated with abnormalities in GABAergic inhibition in the visual cortex. The possible interactions with residual vision and confounds of co-morbidities are not experimentally controlled for in the correlations, and not discussed.

      The goal of the present study was to assess whether we would observe changes in E/I ratio after restoring vision at all. We would not have included patients without nystagmus in the CC group of the present study, since it would have been unlikely that they experienced congenital patterned visual deprivation. Amongst diagnosticians, nystagmus or strabismus might not be considered genuine “comorbidities” that emerge in people with congenital cataracts. Rather, these are consequences of congenital visual deprivation, which we employed as diagnostic criteria. Similarly, absorbed lenses are clear signs that cataracts were congenital. As in other models of experience dependent brain development (e.g. the extant literature on congenital permanent blindness, including anophthalmic individuals (Coullon et al., 2015; Weaver et al., 2013), some uncertainty remains regarding whether the (remaining, in our case) abnormalities of the eye, or the blindness they caused, are the factors driving neural changes. In case of people with reversed congenital cataracts, at least the retina is considered to be intact, as they would otherwise not receive cataract removal surgery.

      However, we consider it unlikely that strabismus caused the group differences, because the present study shows group differences in the Glx/GABA+ ratio at rest, regardless of eye opening or eye closure, for which strabismus would have caused distinct effects. By contrast, the link between GABA concentration and, for example, interocular suppression in strabismus, have so far been documented during visual stimulation (Mukerji et al., 2022; Sengpiel et al., 2006), and differed in direction depending on the amblyopic vs. non-amblyopic eye. Further, one MRS study did not find group differences in GABA concentration between the visual cortices of 16 amblyopic individuals and sighted controls (Mukerji et al., 2022), supporting that the differences in Glx/GABA+ concentration which we observed were driven by congenital deprivation, and not amblyopia-associated visual acuity or eye movement differences. 

      In the revised manuscript, we discussed the inclusion criteria in more detail, and the aforementioned reasons why our data remains interpretable (Page 5, Lines 143 – 145, Lines 147-149). 

      (1.6) Multiple exploratory correlations were performed to relate MRS measures to visual acuity (shown in Supplementary Materials), and only specific ones were shown in the main document. The authors describe the analysis as exploratory in the 'Methods' section. Furthermore, the correlation between visual acuity and E/I metric is weak, and not corrected for multiple comparisons. The results should be presented as preliminary, as no strong conclusions can be made from them. They can provide a hypothesis to test in a future study.

      In the revised manuscript, we have clearly indicated that the exploratory correlation analyses are reported to put forth hypotheses for future studies (Page 4, Lines 118-128; Page 5, Lines 132-134; Page 25, Lines 644- 647).

      (1.7) P.16 Given the correlation of the aperiodic intercept with age ("Age negatively correlated with the aperiodic intercept across CC and SC individuals, that is, a flattening of the intercept was observed with age"), age needs to be controlled for in the correlation between neurochemistry and the aperiodic intercept. Glx has also been shown to negatively correlate with age.

      The correlation between chronological age and aperiodic intercept was observed across groups, but the correlation between Glx and the intercept of the aperiodic EEG activity was seen only in the CC group, even though the SC group was matched for age. Thus, such a correlation was very unlikely to be predominantly driven by an effect of chronological age.

      In the revised manuscript, we added the linear regressions with age as a covariate (Supplementary Material S16, referred to in the main Results, Page 21, Lines 534-537), demonstrating the significant relationship between aperiodic intercept and Glx concentration in the CC group. 

      (1.8) Multiple exploratory correlations were performed to relate MRS to EEG measures (shown in Supplementary Materials), and only specific ones were shown in the main document. Given the multiple measures from the MRS, the correlations with the EEG measures were exploratory, as stated in the text, p.16, and in Figure 4. Yet the introduction said that there was a prior hypothesis "We further hypothesized that neurotransmitter changes would relate to changes in the slope and intercept of the EEG aperiodic activity in the same subjects." It would be great if the text could be revised for consistency and the analysis described as exploratory.

      In the revised manuscript, we improved the phrasing (Page 5, Lines 130-132) and consistently reported the correlations as exploratory in the Methods and Discussion. We consider the correlation analyses as exploratory due to our sample size and the absence of prior work. However, we did hypothesize that both MRS and EEG markers would concurrently be altered in CC vs SC individuals.

      (1.9) The analysis for the EEG needs to take more advantage of the available data. As far as I understand, only two electrodes were used, yet far more were available as seen in their previous study (Ossandon et al., 2023). The spatial specificity is not established. The authors could use the frontal cortex electrode (FP1, FP2) signals as a control for spatial specificity in the group effects, or even better, all available electrodes and correct for multiple comparisons. Furthermore, they could use the aperiodic intercept vs Glx in SC to evaluate the specificity of the correlation to CC.

      The aperiodic intercept and slope did not differ between CC and SC individuals for Fp1 and Fp2, suggesting the spatial specificity of the results. In the revised manuscript, we added this analysis to the Supplementary Material (Supplementary Material S14) and referred to it in our Results (Page 20, Lines 513-514).

      Further, Glx concentration in the visual cortex did not correlate with the aperiodic intercept in the SC group (Figure 4), suggesting that this relationship was indeed specific to the CC group.

      The data from all electrodes has been analyzed and published in other studies as well (Pant et al., 2023; Ossandón et al., 2023). 

      Reviewer #2 (Public Review):

      Summary:

      The manuscript reports non-invasive measures of activity and neurochemical profiles of the visual cortex in congenitally blind patients who recovered vision through the surgical removal of bilateral dense cataracts. The declared aim of the study is to find out how restoring visual function after several months or years of complete blindness impacts the balance between excitation and inhibition in the visual cortex.

      Strengths:

      The findings are undoubtedly useful for the community, as they contribute towards characterising the many ways this special population differs from normally sighted individuals. The combination of MRS and EEG measures is a promising strategy to estimate a fundamental physiological parameter - the balance between excitation and inhibition in the visual cortex, which animal studies show to be heavily dependent upon early visual experience. Thus, the reported results pave the way for further studies, which may use a similar approach to evaluate more patients and control groups.

      Weaknesses:

      (2.1) The main issue is the lack of an appropriate comparison group or condition to delineate the effect of sight recovery (as opposed to the effect of congenital blindness). Few previous studies suggested an increased excitation/Inhibition ratio in the visual cortex of congenitally blind patients; the present study reports a decreased E/I ratio instead. The authors claim that this implies a change of E/I ratio following sight recovery. However, supporting this claim would require showing a shift of E/I after vs. before the sight-recovery surgery, or at least it would require comparing patients who did and did not undergo the sight-recovery surgery (as common in the field).

      Longitudinal studies would indeed be the best way to test the hypothesis that the lower E/I ratio in the CC group observed by the present study is a consequence of sight restoration.

      We have now explicitly stated this in the Limitations section (Page 25, Lines 654-655).

      However, longitudinal studies involving neuroimaging are an effortful challenge, particularly in research conducted outside of major developed countries and dedicated neuroimaging research facilities. Crucially, however, had CC and SC individuals, as well as permanently congenitally blind vs SC individuals (Coullon et al., 2015; Weaver et al., 2013), not differed on any neurochemical markers, such a longitudinal study might have been trivial. Thus, in order to justify and better tailor longitudinal studies, cross-sectional studies are an initial step.

      (2.2) MR Spectroscopy shows a reduced GLX/GABA ratio in patients vs. sighted controls; however, this finding remains rather isolated, not corroborated by other observations. The difference between patients and controls only emerges for the GLX/GABA ratio, but there is no accompanying difference in either the GLX or the GABA concentrations. There is an attempt to relate the MRS data with acuity measurements and electrophysiological indices, but the explorative correlational analyses do not help to build a coherent picture. A bland correlation between GLX/GABA and visual impairment is reported, but this is specific to the patients' group (N=10) and would not hold across groups (the correlation is positive, predicting the lowest GLX/GABA ratio values for the sighted controls - the opposite of what is found). There is also a strong correlation between GLX concentrations and the EEG power at the lowest temporal frequencies. Although this relation is intriguing, it only holds for a very specific combination of parameters (of the many tested): only with eyes open, only in the patient group.

      We interpret these findings differently, that is, in the context of experiments from non-human animals and the larger MRS literature (Page 23, Lines 609-611).

      Homeostatic control of E/I balance assumes that the ratio of excitation (reflected here by Glx) and inhibition (reflected here by GABA+) is regulated. Like prior work (Gao et al., 2024, 2024; Narayan et al., 2022; Perica et al., 2022; Steel et al., 2020; Takado et al., 2022; Takei et al., 2016), we assumed that the ratio of Glx/GABA+ is indicative of E/I balance rather than solely the individual neurotransmitter levels. One of the motivations for assessing the ratio vs the absolute concentration is that as per the underlying E/I balance hypothesis, a change in excitation would cause a concomitant change in inhibition, and vice versa, which has been shown in non-human animal work (Fang et al., 2021; Haider et al., 2006; Tao & Poo, 2005) and modeling research (Vreeswijk & Sompolinsky, 1996; Wu et al., 2022). Importantly, our interpretation of the lower E/I ratio is not just from the Glx/GABA+ ratio, but additionally, based on the steeper EEG aperiodic slope (1-20 Hz). 

      As stated in the Discussion section and Response 1.4, we did not expect to see a lower Glx/GABA+ ratio in CC individuals. We discuss the possible reasons for the direction of the correlation with visual acuity and aperiodic offset during passive visual stimulation, and offer interpretations and (testable) hypotheses.

      We interpret the direction of the Glx/GABA+ correlation with visual acuity to imply that patients with highest (compensatory) balancing of the consequences of congenital blindness (hyperexcitation), in light of visual stimulation, are those who recover best. Note, the sighted control group was selected based on their “normal” vision. Thus, clinical visual acuity measures are not expected to sufficiently vary, nor have the resolution to show strong correlations with neurophysiological measures. By contrast, the CC group comprised patients highly varying in visual outcomes, and thus were ideal to investigate such correlations.

      This holds for the correlation between Glx and the aperiodic intercept, as well. Previous work has suggested that the intercept of the aperiodic activity is associated with broadband spiking activity in neural circuits (Manning et al., 2009). Thus, an atypical increase of spiking activity during visual stimulation, as indirectly suggested by “old” non-human primate work on visual deprivation (Hyvärinen et al., 1981) might drive a correlation not observed in healthy populations.

      In the revised manuscript, we have more clearly indicated in the Discussion that these are possible post-hoc interpretations (Page 23, Lines 584-586; Page 24, Lines 609-620; Page 24, Lines 644-647; Pages 25, Lines 650 - 657). We argue that given the lack of such studies in humans, it is all the more important that extant data be presented completely, even if the direction of the effects are not as expected.

      (2.3) For these reasons, the reported findings do not allow us to draw firm conclusions on the relation between EEG parameters and E/I ratio or on the impact of early (vs. late) visual experience on the excitation/inhibition ratio of the human visual cortex.

      Indeed, the correlations we have tested between the E/I ratio and EEG parameters were exploratory, and have been reported as such.

      We have now made this clear in all the relevant parts of the manuscript (Introduction, Page 5, Lines 132-135; Methods, Page 16, Line 415; Results, Page 21, Figure 4; Discussion, Page 22, Line 568, Page 25, Lines 644-645, Page 25, Lines 650-657).

      The goal of our study was not to compare the effects of early vs. late visual experience. The goal was to study whether early visual experience is necessary for a typical E/I ratio in visual neural circuits. We provided clear evidence in favor of this hypothesis. Thus, the present results suggest the necessity of investigating the effects of late visual deprivation. In fact, such research is missing in permanent blindness as well.

      Reviewer #3 (Public Review):

      This manuscript examines the impact of congenital visual deprivation on the excitatory/inhibitory (E/I) ratio in the visual cortex using Magnetic Resonance Spectroscopy (MRS) and electroencephalography (EEG) in individuals whose sight was restored. Ten individuals with reversed congenital cataracts were compared to age-matched, normally sighted controls, assessing the cortical E/I balance and its interrelationship to visual acuity. The study reveals that the Glx/GABA ratio in the visual cortex and the intercept and aperiodic signal are significantly altered in those with a history of early visual deprivation, suggesting persistent neurophysiological changes despite visual restoration.

      My expertise is in EEG (particularly in the decomposition of periodic and aperiodic activity) and statistical methods. I have several major concerns in terms of methodological and statistical approaches along with the (over)interpretation of the results. These major concerns are detailed below.

      (3.1) Variability in visual deprivation:

      - The document states a large variability in the duration of visual deprivation (probably also the age at restoration), with significant implications for the sensitivity period's impact on visual circuit development. The variability and its potential effects on the outcomes need thorough exploration and discussion.

      We work with a rare, unique patient population, which makes it difficult to systematically assess the effects of different visual histories while maintaining stringent inclusion criteria such as complete patterned visual deprivation at birth. Regardless, we considered the large variance in age at surgery and time since surgery as supportive of our interpretation: group differences were found despite the large variance in duration of visual deprivation. Moreover, the existing variance was used to explore possible associations between behavior and neural measures, as well as neurochemical and EEG measures.

      In the revised manuscript, we have detailed the advantages (Methods, Page 5, Lines 143 – 145, Lines 147-149; Discussion, Page 26, Lines 677-678) and disadvantages (Discussion, Page 25, Lines 650-657) of our CC sample, with respect to duration of congenital visual deprivation.

      (3.2) Sample size:

      - The small sample size is a major concern as it may not provide sufficient power to detect subtle effects and/or overestimate significant effects, which then tend not to generalize to new data. One of the biggest drivers of the replication crisis in neuroscience.

      We address the small sample size in our Discussion, and make clear that small sample sizes were due to the nature of investigations in special populations. In the revised manuscript, we added the sample sizes of previous studies using MRS in permanently blind individuals (Page 4, Lines 108 - 109). It is worth noting that our EEG results fully align with those of larger samples of congenital cataract reversal individuals (Page 25, Lines 666-676, Supplementary Material S18, S19) (Ossandón et al., 2023), providing us confidence about their validity and reproducibility. Moreover, our MRS results and correlations of those with EEG parameters were spatially specific to occipital cortex measures.

      The main problem with the correlation analyses between MRS and EEG measures is that the sample size is simply too small to conduct such an analysis. Moreover, it is unclear from the methods section that this analysis was only conducted in the patient group (which the reviewer assumed from the plots), and not explained why this was done only in the patient group. I would highly recommend removing these correlation analyses.

      In the revised manuscript, we have more clearly marked the correlation analyses as exploratory (Introduction, Page 4, Lines 118-128 and Page 5, Lines 132-134; Methods Page 16, Line 415; Discussion Page 22, Line 568, Page 24, Lines 644-645, Page 25, Lines 650-657); note that we do not base most of our discussion on the results of these analyses.

      As indicated by Reviewer 1, reporting them allows for deriving more precise hypothesis for future studies. It has to be noted that we investigate an extremely rare population, tested outside of major developed economies and dedicated neuroimaging research facilities. In addition to being a rare patient group, these individuals come from poor communities. Therefore, we consider it justified to report these correlations as exploratory, providing direction for future research.

      (3.3) Statistical concerns:

      - The statistical analyses, particularly the correlations drawn from a small sample, may not provide reliable estimates (see https://www.sciencedirect.com/science/article/pii/S0092656613000858, which clearly describes this problem).

      It would undoubtedly be better to have a larger sample size. We nonetheless think it is of value to the research community to publish this dataset, since 10 multimodal data sets from a carefully diagnosed, rare population, representing a human model for the effects of early experience on brain development, are quite a lot. Sample sizes in prior neuroimaging studies in transient blindness have most often ranged from n = 1 to n = 10. They nevertheless provided valuable direction for future research, and integration of results across multiple studies provides scientific insights. 

      Identifying possible group differences was the goal of our study, with the correlations being an exploratory analysis, which we have clearly indicated in the methods, results and discussion.

      - Statistical analyses for the MRS: The authors should consider some additional permutation statistics, which are more suitable for small sample sizes. The current statistical model (2x2) design ANOVA is not ideal for such small sample sizes. Moreover, it is unclear why the condition (EO & EC) was chosen as a predictor and not the brain region (visual & frontal) or neurochemicals. Finally, the authors did not provide any information on the alpha level nor any information on correction for multiple comparisons (in the methods section). Finally, even if the groups are matched w.r.t. age, the time between surgery and measurement, the duration of visual deprivation, (and sex?), these should be included as covariates as it has been shown that these are highly related to the measurements of interest (especially for the EEG measurements) and the age range of the current study is large.

      In our ANOVA models, the neurochemicals were the outcome variables, and the conditions were chosen as predictors based on prior work suggesting that Glx/GABA+ might vary with eye closure (Kurcyus et al., 2018). The study was designed based on a hypothesis of group differences localized to the occipital cortex, due to visual deprivation. The frontal cortex voxel was chosen to indicate whether these differences were spatially specific. Therefore, we conducted separate ANOVAs based on this study design.

      We have now clarified the motivation for these conditions in the Introduction (Page 4, Lines 122-125) and the Methods (Page 9, Lines 219-224).

      In the revised manuscript, we added the rationale for parametric analyses for our outcomes (Shapiro-Wilk as well as Levene’s tests, Supplementary Material S9). Note that in the Supplementary Materials (S12, S14), we have reported the correlations between visual history metrics and MRS/EEG outcomes, thereby investigating whether the variance in visual history might have driven these results. Specifically, we found a (negative) correlation between visual cortex Glx/GABA+ concentration during eye closure and the visual acuity in the CC group (Figure 2c). None of the other exploratory correlations between MRS/EEG outcomes vs time since surgery, duration of blindness or visual acuity were significant in the CC group (Supplementary Material S12, S15).  

      The alpha level used for the ANOVA models specified in the Methods section was 0.05. The alpha level for the exploratory analyses reported in the main manuscript was 0.008, after correcting for (6) multiple comparisons using the Bonferroni correction, also specified in the Methods. Note that the p-values following correction are expressed as multiplied by 6, due to most readers assuming an alpha level of 0.05 (see response regarding large p-values).

      We used a control group matched for age, recruited and tested in the same institutes, using the same setup. We feel that we followed the gold standards for recruiting a healthy control group for a patient group.

      - EEG statistical analyses: The same critique as for the MRS statistical analyses applies to the EEG analysis. In addition: was the 2x3 ANOVA conducted for EO and EC independently? This seems to be inconsistent with the approach in the MRS analyses, in which the authors chose EO & EC as predictors in their 2x2 ANOVA.

      The 2x3 ANOVA was not conducted independently for the eyes open/eyes closed condition. The ANOVA conducted on the EEG metrics was 2x3 because it had two groups (CC, SC) and three conditions (eyes open (EO), eyes closed (EC) and visual stimulation (LU)) as predictors.

      - Figure 4: The authors report a p-value of >0.999 with a correlation coefficient of -0.42 with a sample size of 10 subjects. This can't be correct (it should be around: p = 0.22). All statistical analyses should be checked.

      As specified in the Methods and Figure legend, the reported p values in Figure 4 have been corrected using the Bonferroni correction, and therefore multiplied by the number of comparisons, leading to the seemingly large values.

      Additionally, to check all statistical analyses, we put the manuscript through an independent Statistics Check (Nuijten & Polanin, 2020) (https://michelenuijten.shinyapps.io/statcheck-web/) and have uploaded the consistency report with the revised Supplementary Material (Supplementary Report 1).

      - Figure 2c. Eyes closed condition: The highest score of the *Glx/GABA ratio seems to be ~3.6. In subplot 2a, there seem to be 3 subjects that show a Glx/GABA ratio score > 3.6. How can this be explained? There is also a discrepancy for the eyes-closed condition.

      The three subjects that show the Glx/GABA+ ratio > 3.6 in subplot 2a are in the SC group, whereas the correlations plotted in figure 2c are only for the CC group, where the highest score is indeed ~3.6.

      (3.4) Interpretation of aperiodic signal:

      - Several recent papers demonstrated that the aperiodic signal measured in EEG or ECoG is related to various important aspects such as age, skull thickness, electrode impedance, as well as cognition. Thus, currently, very little is known about the underlying effects which influence the aperiodic intercept and slope. The entire interpretation of the aperiodic slope as a proxy for E/I is based on a computational model and simulation (as described in the Gao et al. paper).

      Apart from the modeling work from Gao et al., multiple papers which have also been cited which used ECoG, EEG and MEG and showed concomitant changes in aperiodic activity with pharmacological manipulation of the E/I ratio (Colombo et al., 2019; Molina et al., 2020; Muthukumaraswamy & Liley, 2018). Further, several prior studies have interpreted changes in the aperiodic slope as reflective of changes in the E/I ratio, including studies of developmental groups (Favaro et al., 2023; Hill et al., 2022; McSweeney et al., 2023; Schaworonkow & Voytek, 2021) as well as patient groups (Molina et al., 2020; Ostlund et al., 2021).

      In the revised manuscript, we have cited those studies not already included in the Introduction (Page 3, Lines 92-94).

      - Especially the aperiodic intercept is a very sensitive measure to many influences (e.g. skull thickness, electrode impedance...). As crucial results (correlation aperiodic intercept and MRS measures) are facing this problem, this needs to be reevaluated. It is safer to make statements on the aperiodic slope than intercept. In theory, some of the potentially confounding measures are available to the authors (e.g. skull thickness can be computed from T1w images; electrode impedances are usually acquired alongside the EEG data) and could be therefore controlled.

      All electrophysiological measures indeed depend on parameters such as skull thickness and electrode impedance. As in the extant literature using neurophysiological measures to compare brain function between patient and control groups, we used a control group matched in age/sex, recruited in the same region, tested with the same devices, and analyzed with the same analysis pipeline. For example, impedance was kept below 10 kOhm for all subjects.

      This is now mentioned in the Methods, Page 13, Line 344.

      There is no evidence available suggesting that congenital cataracts are associated with changes in skull thickness that would cause the observed pattern of group results. Moreover, we cannot think of how any of the exploratory correlations between neurophysiological measures and MRS measures could be accounted for by a difference e.g. in skull thickness.

      - The authors wrote: "Higher frequencies (such as 20-40 Hz) have been predominantly associated with local circuit activity and feedforward signaling (Bastos et al., 2018; Van Kerkoerle et al., 2014); the increased 20-40 Hz slope may therefore signal increased spontaneous spiking activity in local networks. We speculate that the steeper slope of the aperiodic activity for the lower frequency range (1-20 Hz) in CC individuals reflects the concomitant increase in inhibition." The authors confuse the interpretation of periodic and aperiodic signals. This section refers to the interpretation of the periodic signal (higher frequencies). This interpretation cannot simply be translated to the aperiodic signal (slope).

      Prior work has not always separated the aperiodic and periodic components, making it unclear what might have driven these effects in our data. The interpretation of the higher frequency range was intended to contrast with the interpretations of lower frequency range, in order to speculate as to why the two aperiodic fits might go in differing directions. Note that Ossandón et al. reported highly similar results (group differences for CC individuals and for permanently congenitally blind humans) for the aperiodic activity between 20-40 Hz and oscillatory activity in the gamma range.

      In the revised Discussion, we removed this section. We primarily interpret the increased offset and prior findings from fMRI-BOLD data (Raczy et al., 2023) as an increase in broadband neuronal firing.

      - The authors further wrote: We used the slope of the aperiodic (1/f) component of the EEG spectrum as an estimate of E/I ratio (Gao et al., 2017; Medel et al., 2020; Muthukumaraswamy & Liley, 2018). This is a highly speculative interpretation with very little empirical evidence. These papers were conducted with ECoG data (mostly in animals) and mostly under anesthesia. Thus, these studies only allow an indirect interpretation by what the 1/f slope in EEG measurements is actually influenced.

      Note that Muthukumaraswamy et al. (2018) used different types of pharmacological manipulations and analyzed periodic and aperiodic MEG activity in humans, in addition to monkey ECoG (Muthukumaraswamy & Liley, 2018). Further, Medel et al. (now published as Medel et al., 2023) compared EEG activity in addition to ECoG data after propofol administration. The interpretation of our results are in line with a number of recent studies in developing (Hill et al., 2022; Schaworonkow & Voytek, 2021) and special populations using EEG. As mentioned above, several prior studies have used the slope of the 1/f component/aperiodic activity as an indirect measure of the E/I ratio (Favaro et al., 2023; Hill et al., 2022; McSweeney et al., 2023; Molina et al., 2020; Ostlund et al., 2021; Schaworonkow & Voytek, 2021), including studies using scalp-recorded EEG from humans.

      In the introduction of the revised manuscript, we have made more explicit that this metric is indirect (Page 3, Line 91), (additionally see Discussion, Page 24, Lines 644-645, Page 25, Lines 650-657).

      While a full understanding of aperiodic activity needs to be provided, some convergent ideas have emerged. We think that our results contribute to this enterprise, since our study is, to the best of our knowledge, the first which assessed MRS measured neurotransmitter levels and EEG aperiodic activity.

      (3.5) Problems with EEG preprocessing and analysis:

      - It seems that the authors did not identify bad channels nor address the line noise issue (even a problem if a low pass filter of below-the-line noise was applied).

      As pointed out in the methods and Figure 1, we only analyzed data from two occipital channels, O1 and O2 neither of which were rejected for any participant. Channel rejection was performed for the larger dataset, published elsewhere (Ossandón et al., 2023; Pant et al., 2023). As control sites we added the frontal channels FP1 and Fp2 (see Supplementary Material S14)

      Neither Ossandón et al. (2023) nor Pant et al. (2023) considered frequency ranges above 40 Hz to avoid any possible contamination with line noise. Here, we focused on activity between 0 and 20 Hz, definitely excluding line noise contaminations (Methods, Page 14, Lines 365-367). The low pass filter (FIR, 1-45 Hz) guaranteed that any spill-over effects of line noise would be restricted to frequencies just below the upper cutoff frequency.

      Additionally, a prior version of the analysis used spectrum interpolation to remove line noise; the group differences remained stable (Ossandón et al., 2023). We have reported this analysis in the revised manuscript (Page 14, Lines 364-357).

      Further, both groups were measured in the same lab, making line noise (~ 50 Hz) as an account for the observed group effects in the 1-20 Hz frequency range highly unlikely. Finally, any of the exploratory MRS-EEG correlations would be hard to explain if the EEG parameters would be contaminated with line noise.

      - What was the percentage of segments that needed to be rejected due to the 120μV criteria? This should be reported specifically for EO & EC and controls and patients.

      The mean percentage of 1 second segments rejected for each resting state condition and the percentage of 6.25 long segments rejected in each group for the visual stimulation condition have been added to the revised manuscript (Supplementary Material S10), and referred to in the Methods on Page 14, Lines 372-373).

      - The authors downsampled the data to 60Hz to "to match the stimulation rate". What is the intention of this? Because the subsequent spectral analyses are conflated by this choice (see Nyquist theorem).

      This data were collected as part of a study designed to evoke alpha activity with visual white-noise, which changed in luminance with equal power at all frequencies from 1-60 Hz, restricted by the refresh rate of the monitor on which stimuli were presented (Pant et al., 2023). This paradigm and method was developed by VanRullen and colleagues (Schwenk et al., 2020; VanRullen & MacDonald, 2012), wherein the analysis requires the same sampling rate between the presented frequencies and the EEG data. The downsampling function used here automatically applies an anti-aliasing filter (EEGLAB 2019) .

      - "Subsequently, baseline removal was conducted by subtracting the mean activity across the length of an epoch from every data point." The actual baseline time segment should be specified.

      The time segment was the length of the epoch, that is, 1 second for the resting state conditions and 6.25 seconds for the visual stimulation conditions. This has now been explicitly stated in the revised manuscript (Page 14, Lines 379-380).

      - "We excluded the alpha range (8-14 Hz) for this fit to avoid biasing the results due to documented differences in alpha activity between CC and SC individuals (Bottari et al., 2016; Ossandón et al., 2023; Pant et al., 2023)." This does not really make sense, as the FOOOF algorithm first fits the 1/f slope, for which the alpha activity is not relevant.

      We did not use the FOOOF algorithm/toolbox in this manuscript. As stated in the Methods, we used a 1/f fit to the 1-20 Hz spectrum in the log-log space, and subtracted this fit from the original spectrum to obtain the corrected spectrum. Given the pronounced difference in alpha power between groups (Bottari et al., 2016; Ossandón et al., 2023; Pant et al., 2023), we were concerned it might drive differences in the exponent values. Our analysis pipeline had been adapted from previous publications of our group and other labs (Ossandón et al., 2023; Voytek et al., 2015; Waschke et al., 2017).

      We have conducted the analysis with and without the exclusion of the alpha range, as well as using the FOOOF toolbox both in the 1-20 Hz and 20-40 Hz ranges (Ossandón et al., 2023). The findings of a steeper slope in the 1-20 Hz range as well as lower alpha power in CC vs SC individuals remained stable. In Ossandón et al., the comparison between the piecewise fits and FOOOF fits led the authors to use the former, as it outperformed the FOOOF algorithm for their data.

      - The model fits of the 1/f fitting for EO, EC, and both participant groups should be reported.

      In Figure 3 of the manuscript, we depicted the mean spectra and 1/f fits for each group.

      In the revised manuscript, we added the fit quality metrics (average R<sup>2</sup> values > 0.91 for each group and condition) (Methods Page 15, Lines 395-396; Supplementary Material S11) and additionally show individual subjects’ fits (Supplementary Material S11).

      (3.6) Validity of GABA measurements and results:

      - According the a newer study by the authors of the Gannet toolbox (https://analyticalsciencejournals.onlinelibrary.wiley.com/doi/abs/10.1002/nbm.5076), the reliability and reproducibility of the gamma-aminobutyric acid (GABA) measurement can vary significantly depending on acquisition and modeling parameter. Thus, did the author address these challenges?

      We took care of data quality while acquiring MRS data by ensuring appropriate voxel placement and linewidth prior to scanning (Page 9, Lines 229-237). We now address this explicitly in the Methods in the “MRS Data Quality” section. Acquisition as well as modeling parameters were constant for both groups, so they cannot have driven group differences.

      The linked article compares the reproducibility of GABA measurement using Osprey (Oeltzschner et al., 2020), which was released in 2020 and uses linear combination modeling to fit the peak, as opposed to Gannet’s simple peak fitting (Hupfeld et al., 2024). The study finds better test-retest reliability for Osprey compared to Gannet’s method.

      As the present work was conceptualized in 2018, we used Gannet 3.0, which was the state-of-the-art edited-spectrum analysis toolbox at the time, and still is widely used.

      In the revised manuscript, we re-analyzed the data using linear combination modeling with Osprey (Oeltzschner et al., 2020), and reported that the main findings remained the same, i.e. the Glx/GABA+ concentration ratio was lower in the visual cortex of congenital cataract reversal individuals compared to normally sighted controls, regardless of whether participants were scanned with eyes open or with eyes closed. Further, NAA concentration did not differ between groups (Supplementary Material S3). Thus, we demonstrate that our findings were robust to analysis pipelines, and state this in the Methods (Page 9, Lines 242-246) and Results (Page 19, Lines 464-467).

      - Furthermore, the authors wrote: "We confirmed the within-subject stability of metabolite quantification by testing a subset of the sighted controls (n=6) 2-4 weeks apart. Looking at the supplementary Figure 5 (which would be rather plotted as ICC or Blant-Altman plots), the within-subject stability compared to between-subject variability seems not to be great. Furthermore, I don't think such a small sample size qualifies for a rigorous assessment of stability.

      Indeed, we did not intend to provide a rigorous assessment of within-subject stability. Rather, we aimed to confirm that data quality/concentration ratios did not systematically differ between the same subjects tested longitudinally; driven, for example, by scanner heating or time of day. As with the phantom testing, we attempted to give readers an idea of the quality of the data, as they were collected from a primarily clinical rather than a research site.

      In the revised manuscript, we have removed the statement regarding stability and the associated section.

      - "Why might an enhanced inhibitory drive, as indicated by the lower Glx/GABA ratio" Is this interpretation really warranted, as the results of the group differences in the Glx/GABA ratio seem to be rather driven by a decreased Glx concentration in CC rather than an increased GABA (see Figure 2).

      We used the Glx/GABA+ ratio as a measure, rather than individual Glx or GABA+ concentration, which did not significantly differ between groups. As detailed in Response 2.2, we think this metric aligns better with an underlying E/I balance hypothesis and has been used in many previous studies (Gao et al., 2024; Liu et al., 2015; Narayan et al., 2022; Perica et al., 2022).

      Our interpretation of an enhanced inhibitory drive additionally comes from the combination of aperiodic EEG (1-20 Hz) and MRS measures, which, when considered together, are consistent with a decreased E/I ratio.

      In the revised manuscript, we have rewritten the Discussion and removed this section.   

      - Glx concentration predicted the aperiodic intercept in CC individuals' visual cortices during ambient and flickering visual stimulation. Why specifically investigate the Glx concentration, when the paper is about E/I ratio?

      As stated in the methods, we exploratorily assessed the relationship between all MRS parameters (Glx, GABA+ and Glx/GABA+ ratio) with the aperiodic parameters (slope, offset), and corrected for multiple comparisons accordingly. We think this is a worthwhile analysis considering the rarity of the dataset/population (see 1.2, 1.6, 2.1 and Reviewer 1’s comments about future hypotheses). We only report the Glx – aperiodic intercept correlation in the main manuscript as it survived correction for multiple comparisons.

      (3.7) Interpretation of the correlation between MRS measurements and EEG aperiodic signal:

      - The authors wrote: "The intercept of the aperiodic activity was highly correlated with the Glx concentration during rest with eyes open and during flickering stimulation (also see Supplementary Material S11). Based on the assumption that the aperiodic intercept reflects broadband firing (Manning et al., 2009; Winawer et al., 2013), this suggests that the Glx concentration might be related to broadband firing in CC individuals during active and passive visual stimulation." These results should not be interpreted (or with very caution) for several reasons (see also problem with influences on aperiodic intercept and small sample size). This is a result of the exploratory analyses of correlating every EEG parameter with every MRS parameter. This requires well-powered replication before any interpretation can be provided. Furthermore and importantly: why should this be specifically only in CC patients, but not in the SC control group?

      We have indicated clearly in all parts of the manuscript that these correlations are presented as exploratory. Further, we interpret the Glx-aperiodic offset correlation, and none of the others, as it survived the Bonferroni correction for multiple comparisons. We offer a hypothesis in the Discussion as to why such a correlation might exist in the CC but not the SC group (see response 2.2), and do not speculate further.

      (3.8) Language and presentation:

      - The manuscript requires language improvements and correction of numerous typos. Over-simplifications and unclear statements are present, which could mislead or confuse readers (see also interpretation of aperiodic signal).

      In the revised manuscript, we have checked that speculations are clearly marked, and typos are removed.

      - The authors state that "Together, the present results provide strong evidence for experience-dependent development of the E/I ratio in the human visual cortex, with consequences for behavior." The results of the study do not provide any strong evidence, because of the small sample size and exploratory analyses approach and not accounting for possible confounding factors.

      We disagree with this statement and allude to convergent evidence of both MRS and neurophysiological measures. The latter link to corresponding results observed in a larger sample of CC individuals (Ossandón et al., 2023). In the revised manuscript, we have rephrased the statement as “to provide initial evidence” (Page 22, Line 676).

      - "Our results imply a change in neurotransmitter concentrations as a consequence of *restoring* vision following congenital blindness." This is a speculative statement to infer a causal relationship on cross-sectional data.

      As mentioned under 2.1, we conducted a cross-sectional study which might justify future longitudinal work. In order to advance science, new testable hypotheses were put forward at the end of a manuscript.

      In the revised manuscript, we rephrased the sentence and added “might imply” to better indicate the hypothetical character of this idea (Page 22, Lines 586-587).

      - In the limitation section, the authors wrote: "The sample size of the present study is relatively high for the rare population , but undoubtedly, overall, rather small." This sentence should be rewritten, as the study is plein underpowered. The further justification "We nevertheless think that our results are valid. Our findings neurochemically (Glx and GABA+ concentration), and anatomically (visual cortex) specific. The MRS parameters varied with parameters of the aperiodic EEG activity and visual acuity. The group differences for the EEG assessments corresponded to those of a larger sample of CC individuals (n=38) (Ossandón et al., 2023), and effects of chronological age were as expected from the literature." These statements do not provide any validation or justification of small samples. Furthermore, the current data set is a subset of an earlier published paper by the same authors "The EEG data sets reported here were part of data published earlier (Ossandón et al., 2023; Pant et al., 2023)." Thus, the statement "The group differences for the EEG assessments corresponded to those of a larger sample of CC individuals (n=38) " is a circular argument and should be avoided.

      Our intention was not to justify having a small sample, but to justify why we think the results might be valid as they align with/replicate existing literature.

      In the revised manuscript, we added a figure showing that the EEG results of the 10 subjects considered here correspond to those of the 28 other subjects of Ossandón et al (Supplementary Material S18). We adapted the text accordingly, clearly stating that the pattern of EEG results of the ten subjects reported here replicate those of the 28 additional subjects of Ossandón et al. (2023) (Page 25, Lines 671-672).

      References (Public Review)

      Barnes, S. J., Sammons, R. P., Jacobsen, R. I., Mackie, J., Keller, G. B., & Keck, T. (2015). Subnetwork-specific homeostatic plasticity in mouse visual cortex in vivo. Neuron, 86(5), 1290–1303. https://doi.org/10.1016/J.NEURON.2015.05.010

      Bernabeu, A., Alfaro, A., García, M., & Fernández, E. (2009). Proton magnetic resonance spectroscopy (1H-MRS) reveals the presence of elevated myo-inositol in the occipital cortex of blind subjects. NeuroImage, 47(4), 1172–1176. https://doi.org/10.1016/j.neuroimage.2009.04.080

      Bottari, D., Troje, N. F., Ley, P., Hense, M., Kekunnaya, R., & Röder, B. (2016). Sight restoration after congenital blindness does not reinstate alpha oscillatory activity in humans. Scientific Reports. https://doi.org/10.1038/srep24683

      Colombo, M. A., Napolitani, M., Boly, M., Gosseries, O., Casarotto, S., Rosanova, M., Brichant, J. F., Boveroux, P., Rex, S., Laureys, S., Massimini, M., Chieregato, A., & Sarasso, S. (2019). The spectral exponent of the resting EEG indexes the presence of consciousness during unresponsiveness induced by propofol, xenon, and ketamine. NeuroImage, 189(September 2018), 631–644. https://doi.org/10.1016/j.neuroimage.2019.01.024

      Consideration of Sample Size in Neuroscience Studies. (2020). Journal of Neuroscience, 40(21), 4076–4077. https://doi.org/10.1523/JNEUROSCI.0866-20.2020

      Coullon, G. S. L., Emir, U. E., Fine, I., Watkins, K. E., & Bridge, H. (2015). Neurochemical changes in the pericalcarine cortex in congenital blindness attributable to bilateral anophthalmia. Journal of Neurophysiology. https://doi.org/10.1152/jn.00567.2015

      Fang, Q., Li, Y. T., Peng, B., Li, Z., Zhang, L. I., & Tao, H. W. (2021). Balanced enhancements of synaptic excitation and inhibition underlie developmental maturation of receptive fields in the mouse visual cortex. Journal of Neuroscience, 41(49), 10065–10079. https://doi.org/10.1523/JNEUROSCI.0442-21.2021

      Favaro, J., Colombo, M. A., Mikulan, E., Sartori, S., Nosadini, M., Pelizza, M. F., Rosanova, M., Sarasso, S., Massimini, M., & Toldo, I. (2023). The maturation of aperiodic EEG activity across development reveals a progressive differentiation of wakefulness from sleep. NeuroImage, 277. https://doi.org/10.1016/J.NEUROIMAGE.2023.120264

      Gao, Y., Liu, Y., Zhao, S., Liu, Y., Zhang, C., Hui, S., Mikkelsen, M., Edden, R. A. E., Meng, X., Yu, B., & Xiao, L. (2024). MRS study on the correlation between frontal GABA+/Glx ratio and abnormal cognitive function in medication-naive patients with narcolepsy. Sleep Medicine, 119, 1–8. https://doi.org/10.1016/j.sleep.2024.04.004

      Haider, B., Duque, A., Hasenstaub, A. R., & McCormick, D. A. (2006). Neocortical network activity in vivo is generated through a dynamic balance of excitation and inhibition. Journal of Neuroscience. https://doi.org/10.1523/JNEUROSCI.5297-05.2006

      Hill, A. T., Clark, G. M., Bigelow, F. J., Lum, J. A. G., & Enticott, P. G. (2022). Periodic and aperiodic neural activity displays age-dependent changes across early-to-middle childhood. Developmental Cognitive Neuroscience, 54, 101076. https://doi.org/10.1016/J.DCN.2022.101076

      Hupfeld, K. E., Zöllner, H. J., Hui, S. C. N., Song, Y., Murali-Manohar, S., Yedavalli, V., Oeltzschner, G., Prisciandaro, J. J., & Edden, R. A. E. (2024). Impact of acquisition and modeling parameters on the test–retest reproducibility of edited GABA+. NMR in Biomedicine, 37(4), e5076. https://doi.org/10.1002/nbm.5076

      Hyvärinen, J., Carlson, S., & Hyvärinen, L. (1981). Early visual deprivation alters modality of neuronal responses in area 19 of monkey cortex. Neuroscience Letters, 26(3), 239–243. https://doi.org/10.1016/0304-3940(81)90139-7

      Juchem, C., & Graaf, R. A. de. (2017). B0 magnetic field homogeneity and shimming for in vivo magnetic resonance spectroscopy. Analytical Biochemistry, 529, 17–29. https://doi.org/10.1016/j.ab.2016.06.003

      Keck, T., Hübener, M., & Bonhoeffer, T. (2017). Interactions between synaptic homeostatic mechanisms: An attempt to reconcile BCM theory, synaptic scaling, and changing excitation/inhibition balance. Current Opinion in Neurobiology, 43, 87–93. https://doi.org/10.1016/J.CONB.2017.02.003

      Kurcyus, K., Annac, E., Hanning, N. M., Harris, A. D., Oeltzschner, G., Edden, R., & Riedl, V. (2018). Opposite Dynamics of GABA and Glutamate Levels in the Occipital Cortex during Visual Processing. Journal of Neuroscience, 38(46), 9967–9976. https://doi.org/10.1523/JNEUROSCI.1214-18.2018

      Liu, B., Wang, G., Gao, D., Gao, F., Zhao, B., Qiao, M., Yang, H., Yu, Y., Ren, F., Yang, P., Chen, W., & Rae, C. D. (2015). Alterations of GABA and glutamate-glutamine levels in premenstrual dysphoric disorder: A 3T proton magnetic resonance spectroscopy study. Psychiatry Research - Neuroimaging, 231(1), 64–70. https://doi.org/10.1016/J.PSCYCHRESNS.2014.10.020

      Lunghi, C., Berchicci, M., Morrone, M. C., & Russo, F. D. (2015). Short‐term monocular deprivation alters early components of visual evoked potentials. The Journal of Physiology, 593(19), 4361. https://doi.org/10.1113/JP270950

      Maier, S., Düppers, A. L., Runge, K., Dacko, M., Lange, T., Fangmeier, T., Riedel, A., Ebert, D., Endres, D., Domschke, K., Perlov, E., Nickel, K., & Tebartz van Elst, L. (2022). Increased prefrontal GABA concentrations in adults with autism spectrum disorders. Autism Research, 15(7), 1222–1236. https://doi.org/10.1002/aur.2740

      Manning, J. R., Jacobs, J., Fried, I., & Kahana, M. J. (2009). Broadband shifts in local field potential power spectra are correlated with single-neuron spiking in humans. The Journal of Neuroscience : The Official Journal of the Society for Neuroscience, 29(43), 13613–13620. https://doi.org/10.1523/JNEUROSCI.2041-09.2009

      McSweeney, M., Morales, S., Valadez, E. A., Buzzell, G. A., Yoder, L., Fifer, W. P., Pini, N., Shuffrey, L. C., Elliott, A. J., Isler, J. R., & Fox, N. A. (2023). Age-related trends in aperiodic EEG activity and alpha oscillations during early- to middle-childhood. NeuroImage, 269, 119925. https://doi.org/10.1016/j.neuroimage.2023.119925

      Medel, V., Irani, M., Crossley, N., Ossandón, T., & Boncompte, G. (2023). Complexity and 1/f slope jointly reflect brain states. Scientific Reports, 13(1), 21700. https://doi.org/10.1038/s41598-023-47316-0

      Molina, J. L., Voytek, B., Thomas, M. L., Joshi, Y. B., Bhakta, S. G., Talledo, J. A., Swerdlow, N. R., & Light, G. A. (2020). Memantine Effects on Electroencephalographic Measures of Putative Excitatory/Inhibitory Balance in Schizophrenia. Biological Psychiatry: Cognitive Neuroscience and Neuroimaging, 5(6), 562–568. https://doi.org/10.1016/j.bpsc.2020.02.004

      Mukerji, A., Byrne, K. N., Yang, E., Levi, D. M., & Silver, M. A. (2022). Visual cortical γ−aminobutyric acid and perceptual suppression in amblyopia. Frontiers in Human Neuroscience, 16. https://doi.org/10.3389/fnhum.2022.949395

      Muthukumaraswamy, S. D., & Liley, D. T. (2018). 1/F electrophysiological spectra in resting and drug-induced states can be explained by the dynamics of multiple oscillatory relaxation processes. NeuroImage, 179(November 2017), 582–595. https://doi.org/10.1016/j.neuroimage.2018.06.068

      Narayan, G. A., Hill, K. R., Wengler, K., He, X., Wang, J., Yang, J., Parsey, R. V., & DeLorenzo, C. (2022). Does the change in glutamate to GABA ratio correlate with change in depression severity? A randomized, double-blind clinical trial. Molecular Psychiatry, 27(9), 3833—3841. https://doi.org/10.1038/s41380-022-01730-4

      Nuijten, M. B., & Polanin, J. R. (2020). “statcheck”: Automatically detect statistical reporting inconsistencies to increase reproducibility of meta-analyses. Research Synthesis Methods, 11(5), 574–579. https://doi.org/10.1002/jrsm.1408

      Oeltzschner, G., Zöllner, H. J., Hui, S. C. N., Mikkelsen, M., Saleh, M. G., Tapper, S., & Edden, R. A. E. (2020). Osprey: Open-source processing, reconstruction & estimation of magnetic resonance spectroscopy data. Journal of Neuroscience Methods, 343, 108827. https://doi.org/10.1016/j.jneumeth.2020.108827

      Ossandón, J. P., Stange, L., Gudi-Mindermann, H., Rimmele, J. M., Sourav, S., Bottari, D., Kekunnaya, R., & Röder, B. (2023). The development of oscillatory and aperiodic resting state activity is linked to a sensitive period in humans. NeuroImage, 275, 120171. https://doi.org/10.1016/J.NEUROIMAGE.2023.120171

      Ostlund, B. D., Alperin, B. R., Drew, T., & Karalunas, S. L. (2021). Behavioral and cognitive correlates of the aperiodic (1/f-like) exponent of the EEG power spectrum in adolescents with and without ADHD. Developmental Cognitive Neuroscience, 48, 100931. https://doi.org/10.1016/j.dcn.2021.100931

      Pant, R., Ossandón, J., Stange, L., Shareef, I., Kekunnaya, R., & Röder, B. (2023). Stimulus-evoked and resting-state alpha oscillations show a linked dependence on patterned visual experience for development. NeuroImage: Clinical, 103375. https://doi.org/10.1016/J.NICL.2023.103375

      Perica, M. I., Calabro, F. J., Larsen, B., Foran, W., Yushmanov, V. E., Hetherington, H., Tervo-Clemmens, B., Moon, C.-H., & Luna, B. (2022). Development of frontal GABA and glutamate supports excitation/inhibition balance from adolescence into adulthood. Progress in Neurobiology, 219, 102370. https://doi.org/10.1016/j.pneurobio.2022.102370

      Pitchaimuthu, K., Wu, Q. Z., Carter, O., Nguyen, B. N., Ahn, S., Egan, G. F., & McKendrick, A. M. (2017). Occipital GABA levels in older adults and their relationship to visual perceptual suppression. Scientific Reports, 7(1). https://doi.org/10.1038/S41598-017-14577-5

      Rideaux, R., Ehrhardt, S. E., Wards, Y., Filmer, H. L., Jin, J., Deelchand, D. K., Marjańska, M., Mattingley, J. B., & Dux, P. E. (2022). On the relationship between GABA+ and glutamate across the brain. NeuroImage, 257, 119273. https://doi.org/10.1016/J.NEUROIMAGE.2022.119273

      Schaworonkow, N., & Voytek, B. (2021). Longitudinal changes in aperiodic and periodic activity in electrophysiological recordings in the first seven months of life. Developmental Cognitive Neuroscience, 47. https://doi.org/10.1016/j.dcn.2020.100895

      Schwenk, J. C. B., VanRullen, R., & Bremmer, F. (2020). Dynamics of Visual Perceptual Echoes Following Short-Term Visual Deprivation. Cerebral Cortex Communications, 1(1). https://doi.org/10.1093/TEXCOM/TGAA012

      Sengpiel, F., Jirmann, K.-U., Vorobyov, V., & Eysel, U. T. (2006). Strabismic Suppression Is Mediated by Inhibitory Interactions in the Primary Visual Cortex. Cerebral Cortex, 16(12), 1750–1758. https://doi.org/10.1093/cercor/bhj110

      Steel, A., Mikkelsen, M., Edden, R. A. E., & Robertson, C. E. (2020). Regional balance between glutamate+glutamine and GABA+ in the resting human brain. NeuroImage, 220. https://doi.org/10.1016/J.NEUROIMAGE.2020.117112

      Takado, Y., Takuwa, H., Sampei, K., Urushihata, T., Takahashi, M., Shimojo, M., Uchida, S., Nitta, N., Shibata, S., Nagashima, K., Ochi, Y., Ono, M., Maeda, J., Tomita, Y., Sahara, N., Near, J., Aoki, I., Shibata, K., & Higuchi, M. (2022). MRS-measured glutamate versus GABA reflects excitatory versus inhibitory neural activities in awake mice. Journal of Cerebral Blood Flow & Metabolism, 42(1), 197. https://doi.org/10.1177/0271678X211045449

      Takei, Y., Fujihara, K., Tagawa, M., Hironaga, N., Near, J., Kasagi, M., Takahashi, Y., Motegi, T., Suzuki, Y., Aoyama, Y., Sakurai, N., Yamaguchi, M., Tobimatsu, S., Ujita, K., Tsushima, Y., Narita, K., & Fukuda, M. (2016). The inhibition/excitation ratio related to task-induced oscillatory modulations during a working memory task: A multtimodal-imaging study using MEG and MRS. NeuroImage, 128, 302–315. https://doi.org/10.1016/J.NEUROIMAGE.2015.12.057

      Tao, H. W., & Poo, M. M. (2005). Activity-dependent matching of excitatory and inhibitory inputs during refinement of visual receptive fields. Neuron, 45(6), 829–836. https://doi.org/10.1016/J.NEURON.2005.01.046

      Vanrullen, R., & MacDonald, J. S. P. (2012). Perceptual echoes at 10 Hz in the human brain. Current Biology. https://doi.org/10.1016/j.cub.2012.03.050

      Voytek, B., Kramer, M. A., Case, J., Lepage, K. Q., Tempesta, Z. R., Knight, R. T., & Gazzaley, A. (2015). Age-related changes in 1/f neural electrophysiological noise. Journal of Neuroscience, 35(38). https://doi.org/10.1523/JNEUROSCI.2332-14.2015

      Vreeswijk, C. V., & Sompolinsky, H. (1996). Chaos in neuronal networks with balanced excitatory and inhibitory activity. Science, 274(5293), 1724–1726. https://doi.org/10.1126/SCIENCE.274.5293.1724

      Waschke, L., Wöstmann, M., & Obleser, J. (2017). States and traits of neural irregularity in the age-varying human brain. Scientific Reports 2017 7:1, 7(1), 1–12. https://doi.org/10.1038/s41598-017-17766-4

      Weaver, K. E., Richards, T. L., Saenz, M., Petropoulos, H., & Fine, I. (2013). Neurochemical changes within human early blind occipital cortex. Neuroscience. https://doi.org/10.1016/j.neuroscience.2013.08.004

      Wu, Y. K., Miehl, C., & Gjorgjieva, J. (2022). Regulation of circuit organization and function through inhibitory synaptic plasticity. Trends in Neurosciences, 45(12), 884–898. https://doi.org/10.1016/J.TINS.2022.10.006

      Recommendations for the Authors:

      Reviewer #1 (Recommendations for The Authors):

      Thank you for the interesting submission. I have inserted my comments to the authors here. Some of them will be more granular comments related to the concerns raised in the public review.

      (1) Introduction:

      Could you please justify the rationale for using eyes open and eyes closed in the MRS condition, and the use of the three different conditions in the EEG experiment? If these resulted in negative findings, then the implications should be discussed.

      Previous work with MRS in sighted individuals has suggested that eye opening in darkness results in a decrease of visual cortex GABA+ concentration, while visual stimulation results in an increase of Glx concentration, compared to a baseline concentration at eye closure (Kurcyus et al., 2018). Moreover visual stimulation/eye opening is known to result in an alpha desynchronization (Adrian & Matthews, 1934).

      While previous work of our group has shown significantly reduced alpha oscillatory activity in congenital cataract reversal individual, desynchronization following eye opening was indistinguishable when compared to normally sighted controls (Ossandón et al., 2023; Pant et al., 2023).

      Thus, we decided to include both conditions to test whether a similar pattern of results would emerge for GABA+/Glx concentration.

      We added our motivation to the Introduction of the revised manuscript (Page 4, Lines 122-125) along with the Methods (Page 9, Lines 219-223).

      It does not become clear from the introduction why a higher intercept is predicted in the EEG measure. The rationale for this hypothesis needs to be explained better.

      Given the prior findings suggesting an increased E/I ratio in CC individuals and the proposed link between neuronal firing (Manning et al., 2009) and the aperiodic intercept, we expected a higher intercept for the CC compared to the SC group.

      We have now added this explanation to the Introduction (Page 4, Lines 126-128).

      (2) Participants

      Were participants screened for common MRS exclusion criteria such as history of psychiatric conditions or antidepressant medication, which could alter neurochemistry? If not, then this needs to be pointed out.

      All participants were clinically screened at the LV Prasad Eye Institute, and additionally self-reported no neurological or psychiatric conditions or medications. Moreover, all subjects were screened based exclusion criteria for being scanned using the standard questionnaire of the radiology center.

      We have now made this clear in the Methods (Page 7, Lines 168-171).

      Table 1 needs to show the age of the participant, which can only be derived by adding the columns 'duration of deprivation' and 'time since surgery'. Table 1 also needs to include the controls.

      We have accordingly modified Table 1 in the revised manuscript and added age for the patients as well as the controls (Table 1, Pages 6-7).

      The control cohort is not specific enough to exclude reduced visual acuity, or co-morbidities, as the primary driver of the differences between groups. Ideally, a cohort with developmental cataracts is recruited. Normally sighted participants as a control cohort cannot distinguish between different types of sight loss, or stages of plasticity.

      The goal of this study was not to distinguish between different types of sight loss or stages of plasticity. We aimed to assess whether the most extreme forms of visual deprivation (i.e. congenital and total patterned vision loss) affected the E/I ratio. Low visual acuity and nystagmus are genuine diagnostic criteria (Methods, Page 5, Lines 142-145). Visual acuity cannot solely explain the current findings, since the MRS data were acquired both with eyes closed or diffuse visual stimulation in a dimly lit room, without any visual task.

      With the awareness of the present results, we consider it worthwhile for the future to investigate additional groups such as developmental cataract-reversal individuals, to narrow down the contribution of the age of onset and degree of visual deprivation to the observed group differences.

      (3) Data collection and analysis

      - More detail is needed: how long were the sessions, how long was each part?

      We have added this information on Page 7, Lines 178-181 of the Methods. MRS scanning took between 45 and 60 minutes, EEG testing took 20 minutes excluding the time for capping, and visual acuity testing took 3-5 minutes.

      - It should be mentioned here that the EEG data is a reanalysis of a subset of legacy data, published previously in Ossandón et al., 2023; Pant et al., 2023.

      In the revised manuscript, we explicitly state at the beginning of the “Electrophysiology recordings” section of the Methods (Page 13, Lines 331-334) that the EEG datasets were a subset of previously published data.

      (4) MRS Spectroscopy

      - Please fill out the minimum reporting standards form (Lin et al., 2021), or report all the requested measures in the main document https://pubmed.ncbi.nlm.nih.gov/33559967/

      We have now filled out this form and added it as Supplementary Material (Supplementary Excel File 1). Additionally, all the requested information has been moved to the Methods section of the main document (MRS Data Quality, Pages 10-12).

      - Information on how the voxels were placed is missing. The visual cortex voxel is not angled parallel to the calcarine, as is a common way to capture processing in the early visual cortex. Describe in the paper what the criteria for successful placement were, and how was it ensured that non-brain tissue was avoided in a voxel of this size.

      Voxel placement was optimized in each subject to avoid the meninges, ventricles, skull and subcortical structures, ensured by examining the voxel region across slices in the acquired T1 volume for each subject. Saturation bands were placed to nullify the skull signal during MRS acquisition, at the anterior (frontal) and posterior (visual) edge of the voxel for every subject. Due to limitations in the clinical scanner rotated/skewed voxels were not possible, and thus voxels were not always located precisely parallel to the calcarine.

      We have added this information to Page 9 (Lines 229-237) of the revised manuscript.

      - Figure 1. shows voxels that are very close to the edge of the brain (frontal cortex) or to the tentorium (visual cortex). Could the authors please calculate the percentage overlap between the visual cortex MRS voxel and the visual cortex, and compare them across groups to ensure that there is no between-group bias from voxel placement?

      We have now added the requested analysis to Supplementary Material S2 and referred to it in the main manuscript on Page 9, Lines 236-237.

      Briefly, the percentage overlap with areas V1-V6 in every individual subject’s visual cortex voxel was 60% or more; the mean overlap in the CC group was 67% and the SC group 70%. The percentage overlap did not differ between groups ( t-test (t(18) = -1.14, p = 0.269)).

      - Figure 1. I would recommend displaying data on a skull-stripped image to avoid identifying information from the participant's T1 profile.

      We have now replaced the images in Figure 1 with skull-stripped images. Note that images from SPM12 were used instead of GannetCoregister, as GannetCoregister only displays images with the skull.

      - Please show more rigor with the MRS quality measures. Several examples of inconsistency and omissions are below.

      • SNR was quantified and shows a difference in SNR between voxel positions, with lower SNR in the frontal cortex. No explanation or discussion of the difference was provided.

      • Looking at S1, the linewidth of NAA seems to be a lot broader in the frontal cortex than in the visual cortex. The figures suggest that acquisition quality was very different between voxel locations, making the comparison difficult.

      • Linewidth of NAA is a generally agreed measure of shim quality in megapress acquisitions (Craven et al., 2022).

      The data quality difference between the frontal and visual cortices has been observed in the literature (Juchem & Graaf, 2017; Rideaux et al., 2022). We nevertheless chose a frontal cortex voxel as control site instead of the often-chosen sensorimotor cortex. The main motivation was to avoid any cortical region linked to sensory processing since crossmodal compensation as a consequence of visual deprivation is a well-documented phenomenon.

      We now make this clearer in the Methods (Page 11, Lines 284 – 299), in the Discussion/Limitations (Page 25, Lines 662 - 665).  

      - To get a handle on the data quality, I would recommend that the authors display their MRS quality measures in a separate section 'MRS quality measure', including NAA linewidth, NAA SNR, GABA+ CRLB, Glx CRLB, and test for the main effects and interaction of voxel location (VC, FC) and group (SC, CC) and discuss any discrepancies.

      We have moved all the quality metric values for GABA+, Glx and NAA from the supplement to the Methods section (see Table 2), and added the requested section titled “MRS Data quality.”

      We have conducted the requested analyses and reported them in Supplementary Material S6: there was a strong effect of region confirming that data quality was better in the visual than frontal region. We have referred to this in the main manuscript on Page 11, Line 299.

      In the revised manuscript, we discuss the data quality in the frontal cortex, and how we ensured it was comparable to prior work. Moreover, there were no significant group effects, or group-by-region interactions, suggesting that group differences observed for the visual cortex voxel cannot be accounted for by differences in data quality. We now included a section on data quality, both in the Methods (Page 11, Lines 284 – 299), and the limitations section of the Discussion (Page 25, Lines 662 - 665).

      Please clarify the MRS acquisition, "Each MEGA- PRESS scan lasted for 8 minutes and was acquired with the following specifications: TR = 2000 ms, TE = 68 ms, Voxel size = 40 mm x 30 mm x 25mm, 192 averages (each consists of two TRs). "192 averages x 2 TRs x 2s TR = 12.8 min, not 8 min, apologies if I have misunderstood these details.

      We have corrected this error in the revised manuscript and stated the parameters more clearly – there were a total of 256 averages, resulting in an (256 repetitions with 1 TR * 2 s/60) 8.5-minute scan (Page 8, Lines 212-213).

      - What was presented to participants in the eyes open MRS? Was it just normal room illumination or was it completely dark? Please add details to your methods.

      The scans were conducted in regular room illumination, with no visual stimulation.

      We have now clarified this on Page 9 (Lines 223-224) of the Methods.

      (5) MRS analysis

      How was the tissue fraction correction performed? Please add or refer to the exact equation from Harris et al., 2015.

      We have clarified that the reported GABA+/Glx values are water-normalized alpha corrected values (Page 10, Line 249), and cited Harris et al., 2015 on Page 10 (Line 251) of the Methods.

      (6) Statistical approach

      How was the sample size determined? Please add your justification for the sample size

      We collected as many qualifying patients as we were able to recruit for this study within 2.5 years of data collection (commencing August 2019, ending February 2022), given the constraints of the patient population and the pandemic. We have now made this clear in the Discussion (Page 25, Lines 650-652).

      Please report the tests for normality.

      We have now reported the Shapiro-Wilk test results for normality as well as Levene’s test for homogeneity of variance between groups for every dependent variable in our dataset in Supplementary Material S9, and added references to it in the descriptions of the statistical analyses (Methods, Page13, Lines 326-329 and Page 15, Lines 400-402).

      Calculate the Bayes Factor where possible.

      As our analyses are all frequentist, instead of re-analyzing the data within a Bayesian framework, we added partial eta squared values for all the reported ANOVAs (η<sub>p</sub><sup>²</sup>) for readers to get an idea of the effect size (Results).

      I recommend partial correlations to control for the influence of age, duration, and time of surgery, rather than separate correlations.

      Given the combination of small sample size and the expected multicollinearity in our variables (duration of blindness, for example, would be expected to correlate with age, as well as visual acuity post-surgery), partial correlations could not be calculated on this data.

      We are aware of the limits of correlational analyses. Given the unique data set of a rare population we had exploratorily planned to relate behavioral, EEG and MRS parameters by calculating correlations. Since no similar data existed when we started (and to the best of our knowledge our data set is still unique), these correlation analyses were explorative, but the most transparent to run.

      We have now clearly outlined these limitations in our Introduction (Page 5, Lines 133-135), Methods (Page 15, Lines 408-410) and Discussion section (Page 24, Line 634, Page 25, Lines 652-65) to ensure that the results are interpreted with appropriate caution.

      (7) Visual acuity

      Is the VA monocular average, from the dominant eye, or bilateral?

      We have now clarified that the VA reported here is bilateral (Methods, Page 7 Line 165 and Page 15, Line 405). Bilateral visual acuity in congenital cataract-reversal individuals typically corresponds to the visual acuity of the best eye.

      It is mentioned here that correlations with VA are exploratory, please be consistent as the introduction mentions that there was a hypothesis that you sought to test.

      We have now accordingly modified the Introduction (Page 5, Lines 133-135) and added the appropriate caveats in the discussion with regards to interpretations (Page 25, Lines 652-665).

      (8) Correlation analyses between MRS and EEG

      It is mentioned here that correlations between EEG and MRS are exploratory, please consistently point out the exploratory nature, as these results are preliminary and should not be overinterpreted ("We did not have prior hypotheses as to the best of our knowledge no extant literature has tested the correlation between aperiodic EEG activity and MRS measures of GABA+,Glx and Glx/GABA+." ).

      In the revised manuscript, we explicitly state the reported associations between EEG (aperiodic component) and MRS parameters allow for putting forward directed / more specific hypotheses for future studies (Introduction, Page 5, Lines 133-135; Methods, Page 15, Line 415. Discussion, Page 25, Lines 644-645 and Lines 652-665).

      (9) Results

      Figure 2 uses the same y-axis for the visual cortex and frontal cortex to facilitate a comparison between the two locations. Comparing Figure 2 a with b demonstrates poorer spectral peaks and reduced amplitudes. Lower spectral quality in the frontal cortex voxel could contribute to the absence of a group effect in the control voxel location. The major caveat that spectral quality differs between voxels needs to be pointed out and the limitations thereof discussed.

      We have now explicitly pointed out this issue in the Methods (MRS Data Quality, Supplementary Material S6) and Discussion in the Limitations section (Page 25, Lines 662-665). While data quality was lower for the frontal compared to the visual cortex voxels, as has been observed previously (Juchem & Graaf, 2017; Rideaux et al., 2022), this was not an issue for the EEG recordings. Thus, lower sensitivity of frontal measures cannot easily explain the lack of group differences for frontal measures. Crucially, data quality did not differ between groups.

      The results in 2c are the result of multiple correlations with metabolite values ("As in previous studies, we ran a number of exploratory correlation analyses between GABA+, Glx, and Glx/GABA+ concentrations, and visual acuity at the date of testing, duration of visual deprivation, and time since surgery respectively in the CC group"), it seems at least six for the visual acuity measure (VA vs Glx, VA vs GABA+, VA vs Glx/GABA+ x 2 conditions). While the trends are interesting, they should be interpreted with caution because of the exploratory nature, small sample size, the lack of multiple comparison correction, and the influence of two extreme data points. The authors should not overinterpret these results and should point out the need for replication.

      See response to (6) last section, which we copy here for convenience:

      We are aware of the limits of correlational analyses. Given the unique data set of a rare population we exploratorily related behavioral, EEG and MRS parameters by calculating correlations. Since no similar data existed when we started (and to the best of our knowledge our data set is still unique), these correlation analyses were explorative, but the most transparent to run.

      We have now clearly outlined these limitations in our Discussion section to ensure that the results are interpreted with appropriate caution (Discussion, Page 25, Lines 644-645 and Lines 652-665).

      (10) Discussion:

      Please explain the decrease in E/I balance from MRS in view of recent findings on an increase in E/I balance in CC using RSN-fMRI (Raczy et al., 2022) and EEG (Ossandon et al. 2023).

      We have edited our Abstract (Page 1-2, Lines 31-35) and Discussion (Page 23, Lines 584-590; Page 24, Lines 613-620). In brief, we think our results reflect a homeostatic regulation of E/I balance, that is, an increase in inhibition due to an increase in stimulus driven excitation following sight restoration.

      Names limitations but does nothing to mitigate concerns about spatial specificity. The limitations need to be rewritten to include differences in SNR between the visual cortex and frontal lobe. Needs to include caveats of small samples, including effect inflation.

      We have now discussed the data quality differences between the visual and frontal cortex voxel in MRS data quality, which we find irrespective of group (MRS Data Quality, Supplementary Material S6). We also reiterate why this might not explain our results; data quality was comparable to prior studies which have found group differences in frontal cortex (Methods Page 11, Lines 284 – 299), and data quality did not differ between groups. Further, EEG data quality did not differ across frontal and occipital regions, but group differences in EEG datasets were localized to the occipital cortex.

      Reviewer #2 (Recommendations for The Authors):

      To address the main weakness, the authors could consider including data from a third group, of congenitally blind individuals. Including this would go a very long way towards making the findings interpretable and relating them to the rest of the literature.

      Unfortunately, recruitment of these groups was not possible due to the pandemic. Indeed, we would consider a pre- vs post- surgery approach the most suitable design in the future, which, however, will require several years to be completed. Such time and resource intensive longitudinal studies are justified by the present cross-sectional results.

      We have explicitly stated our contribution and need for future studies in the Limitations section of the Discussion (Page 25, Lines 650-657).

      Analysing the amplitude of alpha rhythms, as well as the other "aperiodic" components, would be useful to relate the profile of the tested patients with previous studies. Visual inspection of Figure 3 suggests that alpha power with eyes closed is not reduced in the patients' group compared to the controls. This would be inconsistent with previous studies (including research from the same group) and it could suggest that the small selected sample is not really representative of the sight-recovery population - certainly one of the most heterogeneous study populations. This further highlights the difficulty of drawing conclusions on the effects of visual experience merely based on this N=10 set of patients.

      Alpha power was indeed reduced in the present subsample of 10 CC individuals (Supplementary Material S19). A possible source of the confusion (that the graphs of the CC and SC group look so similar for the EC condition in Figure 3) likely is that the spectra are shown with aperiodic components not yet removed, and scales to accommodate very different alpha power values. As documented in Supplementary Material S18 and S19, alpha power and the aperiodic intercept/slope results of the resting state data in the present 10 CC individuals correspond to the results from a larger sample of CC individuals (n = 28) in Ossandón et al., 2023. We explicitly highlight this “replication” in the main manuscript (Page 25 -26, Lines 671-676). Thus, the present sub-sample of CC individuals are representative for their population.

      To further characterise the MRS results, the authors may consider an alternative normalisation scheme. It is not clear whether the lack of significant GABA and GLX differences in the face of a significant group difference in the GLX/GABA ratio is due to the former measures being noisier since taking the ratio between two metabolites often helps reduce inter-individual variability and thereby helps revealing group differences. It remains an open question whether the GABA or GLX concentrations would show significant group differences after appropriate normalisation (e.g. NAA?).

      We repeated the analysis with Creatine-normalized values of GABA+ and Glx, and the main results i.e. reduced Glx/GABA+ concentration in the visual cortex of CC vs SC individuals, and no such difference in the frontal cortex, remained the same (Supplementary Material S5).

      Further, we re-analyzed the data using Osprey, an open-source toolbox that uses linear combination modeling, and found once more that our results did not change (Supplementary Material S3). We refer to these findings in the Methods (Page 10, Lines 272-275) and Results (Page 10, Lines 467-471) of the main manuscript.

      In fact, the Glx concentration in the visual cortex of CC vs SC individuals was significantly decreased when Cr-normalized values were used (which was not significant in the original analysis). However, we do not interpret this result as it was not replicated with the water-normalized values from Gannet or Osprey.

      I suggest revising the discussion to present a more balanced picture of the existent evidence of the relation between E/I and EEG indices. Although there is evidence that the 1/f slope changes across development, in a way that could be consistent with a higher slope reflecting more immature and excitable tissue, the link with cortical E/I is far from established, especially when referring to specific EEG indices (intercept vs. slope, measured in lower vs. higher frequency ranges).

      We have revised the Introduction (Page 4, Line 91, Lines 101-102) and Discussion (Page 22, Lines 568-569, Page 24, Lines 645-647 and Lines 654-657) in the manuscript accordingly; we allude to the fact that the links between cortical E/I and aperiodic EEG indices have not yet been unequivocally established in the literature.

      Minor:

      - The authors estimated NAA concentration with different software than the one used to estimate GLX and GABA; this examined the OFF spectra only; I suggest that the authors consider running their analysis with LCModel, which would allow a straightforward approach to estimate concentrations of all three metabolites from the same edited spectrum and automatically return normalised concentrations as well as water-related ones.

      We re-analyzed all of the MRS datasets using Osprey, which uses linear combination modelling and has shown quantification results similar to LCModel for NAA (Oeltzschner et al., 2020). The results of a lower Glx/GABA+ concentration in the visual cortex of CC vs SC individuals, and no difference in NAA concentration, were replicated using this pipeline.

      We have now added these analyses to the Supplementary Material S3 and referred to them in the Methods (Page 9, Lines 242-246) and Results (Page 18, Lines 464-467).

      - Of course the normalisation used to estimate GABA and GLX values is completely irrelevant when the two values are expressed as ratio GLX/GABA - this may be reflected in the text ("water normalised GLX/GABA concentration" should read "GLX/GABA concentration" instead).

      We have adapted the text on Page 16 (Line 431) and have ensured that throughout the manuscript the use of “water-normalized” is in reference to Glx or GABA+ concentration, and not the ratio.

      - Please specify which equation was used for tissue correction - is it alpha-correction?

      We have clarified that the reported GABA+/Glx values are water-normalized alpha corrected values (Page 10, Line 249), and cited Harris et al., 2015 on Page 10 (Line 251) of the Methods.

      - Since ANOVA was used, the assumption is that values are normally distributed. Please report evidence supporting this assumption.

      We have now reported the Shapiro-Wilk test results for normality as well as Levene’s test for homogeneity of variance between groups for every dependent variable in our dataset in Supplementary Material S9, and added references to it in the Methods (Page 13, Lines 326-329 and Page 15, Lines 400-402).

      Reviewer #3 (Recommendations for The Authors):

      In addition to addressing major comments listed in my Public Review, I have the following, more granular comments, which should also be addressed:

      (1) The paper's structure could be improved by presenting visual acuity data before diving into MRS and EEG results to better contextualize the findings.

      We now explicitly state in the Methods (Page 5, Line 155) that lower visual acuity is expected in a cohort of CC individuals with long lasting congenital visual deprivation.

      We have additionally included a plot of visual acuities of the two groups (Supplementary Material S1).

      (2) The paper should better explain the differences between CC for which sight is restored and congenitally blind patients. The authors write in the introduction that there are sensitive periods/epochs during the lifespan for the development of local inhibitory neural circuits. and "Human neuroimaging studies have similarly demonstrated that visual experience during the first weeks and months of life is crucial for the development of visual circuits. If human infants born with dense bilateral cataracts are treated later than a few weeks from birth, they suffer from a permanent reduction of not only visual acuity (Birch et al., 1998; Khanna et al., 2013) and stereovision (Birch et al., 1993; Tytla et al., 1993) but additionally from impairments in higher-level visual functions, such as face perception (Le Grand et al., 2001; Putzar et al., 2010; Röder et al., 2013)...".

      Thus it seems that the current participants (sight restored after a sensitive period) seem to be similarly affected by the development of the local inhibitory circuits as congenitally blind. To assess the effect of plasticity and sight restoration longitudinal data would be necessary.

      In the Introduction (Page 2, Lines 59-64; Page 3, Lines 111-114) we added that in order to identify sensitive periods e.g. for the elaboration of visual neural circuits, sight recovery individuals need to be investigated. The study of permanently blind individuals allows for investigating the role of experience (whether sight is necessary to introduce the maturation of visual neural circuits), but not whether visual input needs to be available at early epochs in life (i.e. whether sight restoration following congenital blindness could nevertheless lead to the development of visual circuits).

      This is indeed the conclusion we make in the Discussion section. We have now highlighted the need for longitudinal assessments in the Discussion (Page 25, Lines 654-656).

      (3) What's the underlying idea of analyzing two separate aperiodic slopes (20-40Hz and 1-19Hz). This is very unusual to compute the slope between 20-40 Hz, where the SNR is rather low.

      "Ossandón et al. (2023), however, observed that in addition to the flatter slope of the aperiodic power spectrum in the high frequency range (20-40 Hz), the slope of the low frequency range (1-19 Hz) was steeper in both, congenital cataract-reversal individuals, as well as in permanently congenitally blind humans."

      The present manuscript computed the slope between 1-20 Hz. Ossandón et al. as well as Medel et al. (2023) found a “knee” of the 1/f distribution at 20 Hz and describe further the motivations for computing both slope ranges. For example, Ossandón et al. used a data driven approach and compared single vs. dual fits and found that the latter fitted the data better. Additionally, they found the best fit if a knee at 20 Hz was used. We would like to point out that no standard range exists for the fitting of the 1/f component across the literature and, in fact, very different ranges have been used (Gao et al., 2017; Medel et al., 2023; Muthukumaraswamy & Liley, 2018).

      (4) "For this scan, participants were instructed to keep their eyes closed and stay as still as possible." Why should it be important to have the eyes closed during a T1w data acquisition? This statement at this location does not make sense.

      To avoid misunderstandings, we removed this statement in this context.

      (5) "Two SC subjects did not complete the frontal cortex scan for the EO condition and were excluded from the statistical comparisons of frontal cortex neurotransmitter concentrations."<br /> Why did the authors not conduct whole-brain MRS, which seems to be on the market for quite some time (e.g. https://www.ncbi.nlm.nih.gov/pmc/articles/PMC3590062/) ?

      Similar to previous work (Coullon et al., 2015; Weaver et al., 2013) our hypothesis was related to the visual cortex, and we chose the frontal cortex voxel as a control. This has now been clarified in the Introduction (Page 4, Lines 103-114), Methods (Page 9, Lines 225-227) and Discussion (Page 25, Lines 662-665).

      (6) In "....during visual stimulation with stimuli that changed in luminance (LU) (Pant et al., 2023)." the authors should provide a link on the visual stimulation, which is provided further below

      In the revised manuscript, we have moved up the description of the visual stimulation (Page 13, Line 336).

      (7) "During the EO condition, participants were asked to fixate on a blank screen." This is not really possible. Typically, resting state EO conditions include a fixation cross, as the participants would not be able to fixate on a blank screen and move their eyes, which would impact the recordings.

      We have now rephrased this as “look towards” with the goal of avoiding eye movements (Page 14, Line 347).

      (8) "Components corresponding to horizontal or vertical eye movements were identified via visual inspection and removed (Plöchl et al., 2012)." It is unclear what the Plöchl reference should serve for. Is the intention of the authors to state that manual (and subjective) visual inspection of the ICA components is adequate? I would recommend removing this reference.

      The intention was to provide the basis for classification during the visual inspection, as opposed to an automated method such as ICLabel.

      We stated this clearly in the revised manuscript (Page 14 Lines 368-370).

      (9) "The datasets were divided into 6.25 s long epochs corresponding to each trial." This is a bit inaccurate, as the trial also included some motor response task. Thus, I assume the 6.25 s are related to the visual stimulation.

      We have modified the sentence accordingly (Page 15, Line 378).

      (10) Figure 2. a & b. Just an esthetic suggestion: I would recommend removing the lines between the EC and EO conditions, as they suggest some longitudinal changes. Unless it is important to highlight the changes between EC and EO within each subject.

      In fact, EC vs. EO was a within-subject factor with expected changes for the EEG and possible changes in the MRS parameters. To allow the reader to track changes due to EC vs. EO for individual subjects (rather than just comparing the change in the mean scores), we use lines.  

      (11) Figure 3A: I would plot the same y-axis range for both groups to make it more comparable.

      We have changed Figure 3A accordingly.

      (12) " flattening of the intercept" replaces flattening, as it is too related to slope.

      We have replaced “flattening” with “reduction” (Page 20, Line 517).

      (13) The plotting of only the significant correlation between MRS measures and EEG measures seems to be rather selective reporting. For this type of exploratory analysis, I would recommend plotting all of the scatter plots and moving the entire exploratory analysis to the supplementary (as this provides the smallest evidence of the results).

      We have made clear in the Methods (Page 16, Lines 415-426), Results and Discussion (page 24, Lines 644-645), as well as in the Supplementary material, that the reason for only reporting the significant correlation was that this correlation survived correction for multiple comparisons, while all other correlations did not. We additionally explicitly allude to the Supplementary Material where the plots for all correlations are shown (Results, Page 21, Lines 546-552).

      (14) "Here, we speculate that due to limited structural plasticity after a phase of congenital blindness, the neural circuits of CC individuals, which had adapted to blindness after birth, employ available, likely predominantly physiological plasticity mechanisms (Knudsen, 1998; Mower et al., 1985; Röder et al., 2021), in order to re-adapt to the newly available visual excitation following sight restoration."

      I don't understand the logic here. The CC individuals are congenitally blind, thus why should there be any physiological plasticity mechanism to adapt to blindness, if they were blind at birth?

      With “adapt to blindness” we mean adaptation of a brain to an atypical or unexpected condition when taking an evolutionary perspective (i.e. the lack of vision). We have made this clear in the revised manuscript (Introduction, Page 4, Lines 111-114; Discussion, Page 23, Lines 584-591).

      (15) "An overall reduction in Glx/GABA ratio would counteract the aforementioned adaptations to congenital blindness, e.g. a lower threshold for excitation, which might come with the risk of runaway excitation in the presence of restored visually-elicited excitation."

      This could be tested by actually investigating the visual excitation by visual stimulation studies.

      The visual stimulation condition in the EEG experiment of the present study found a higher aperiodic intercept in CC compared to SC individuals. Given the proposed link between the intercept and spontaneous neural firing (Manning et al., 2009), we interpreted the higher intercept in CC individuals as increased broadband neural firing during visual stimulation (Results Figure 3; Discussion Page 24, Lines 635-640). This idea is compatible with enhanced BOLD responses during an EO condition in CC individuals (Raczy et al., 2022). Future work should systematically manipulate visual stimulation to test this idea.

      (16) As the authors also collected T1w images, the hypothesis of increased visual cortex thickness in CC. Was this investigated?

      This hypothesis was investigated in a separate publication which included this subset of participants (Hölig et al., 2023), and found increased visual cortical thickness in the CC group. We refer to this publication, and related work (Feng et al., 2021) in the present manuscript.

      (17) The entire discussion of age should be omitted, as the current data set is too small to assess age effects.

      We have removed this section and just allude to the fact that we replicated typical age trends to underline the validity of the present data (Page 26, Lines 675-676).

      (18) Table1: should include the age and the age at the time point of surgery.

      We added age to the revised Table 1. We clarified that in CC individuals, duration of blindness is the same as age at the time point of surgery (Page 6, Line 163).

      (19) Why no group comparisons of visual acuity are reported?

      Lower visual acuity in CC than SC individuals is a well-documented fact.

      We have now added the visual acuity plots for readers (Supplementary Material S1, referred to in the Methods, Page 5, Line 155) which highlight this common finding.

      References (Recommendations to the Authors)

      Adrian, E. D., & Matthews, B. H. C. (1934). The berger rhythm: Potential changes from the occipital lobes in man. Brain. https://doi.org/10.1093/brain/57.4.355

      Coullon, G. S. L., Emir, U. E., Fine, I., Watkins, K. E., & Bridge, H. (2015). Neurochemical changes in the pericalcarine cortex in congenital blindness attributable to bilateral anophthalmia. Journal of Neurophysiology. https://doi.org/10.1152/jn.00567.2015

      Feng, Y., Collignon, O., Maurer, D., Yao, K., & Gao, X. (2021). Brief postnatal visual deprivation triggers long-lasting interactive structural and functional reorganization of the human cortex. Frontiers in Medicine, 8, 752021. https://doi.org/10.3389/FMED.2021.752021/BIBTEX

      Gao, R., Peterson, E. J., & Voytek, B. (2017). Inferring synaptic excitation/inhibition balance from field potentials. NeuroImage, 158(March), 70–78. https://doi.org/10.1016/j.neuroimage.2017.06.078

      Hölig, C., Guerreiro, M. J. S., Lingareddy, S., Kekunnaya, R., & Röder, B. (2023). Sight restoration in congenitally blind humans does not restore visual brain structure. Cerebral Cortex, 33(5), 2152–2161. https://doi.org/10.1093/CERCOR/BHAC197

      Juchem, C., & Graaf, R. A. de. (2017). B0 magnetic field homogeneity and shimming for in vivo magnetic resonance spectroscopy. Analytical Biochemistry, 529, 17–29. https://doi.org/10.1016/j.ab.2016.06.003

      Kurcyus, K., Annac, E., Hanning, N. M., Harris, A. D., Oeltzschner, G., Edden, R., & Riedl, V. (2018). Opposite Dynamics of GABA and Glutamate Levels in the Occipital Cortex during Visual Processing. Journal of Neuroscience, 38(46), 9967–9976. https://doi.org/10.1523/JNEUROSCI.1214-18.2018

      Manning, J. R., Jacobs, J., Fried, I., & Kahana, M. J. (2009). Broadband shifts in local field potential power spectra are correlated with single-neuron spiking in humans. The Journal of Neuroscience : The Official Journal of the Society for Neuroscience, 29(43), 13613–13620. https://doi.org/10.1523/JNEUROSCI.2041-09.2009

      Medel, V., Irani, M., Crossley, N., Ossandón, T., & Boncompte, G. (2023). Complexity and 1/f slope jointly reflect brain states. Scientific Reports, 13(1), 21700. https://doi.org/10.1038/s41598-023-47316-0

      Muthukumaraswamy, S. D., & Liley, D. T. (2018). 1/F electrophysiological spectra in resting and drug-induced states can be explained by the dynamics of multiple oscillatory relaxation processes. NeuroImage, 179(November 2017), 582–595. https://doi.org/10.1016/j.neuroimage.2018.06.068

      Oeltzschner, G., Zöllner, H. J., Hui, S. C. N., Mikkelsen, M., Saleh, M. G., Tapper, S., & Edden, R. A. E. (2020). Osprey: Open-source processing, reconstruction & estimation of magnetic resonance spectroscopy data. Journal of Neuroscience Methods, 343, 108827. https://doi.org/10.1016/j.jneumeth.2020.108827

      Ossandón, J. P., Stange, L., Gudi-Mindermann, H., Rimmele, J. M., Sourav, S., Bottari, D., Kekunnaya, R., & Röder, B. (2023). The development of oscillatory and aperiodic resting state activity is linked to a sensitive period in humans. NeuroImage, 275, 120171. https://doi.org/10.1016/J.NEUROIMAGE.2023.120171

      Pant, R., Ossandón, J., Stange, L., Shareef, I., Kekunnaya, R., & Röder, B. (2023). Stimulus-evoked and resting-state alpha oscillations show a linked dependence on patterned visual experience for development. NeuroImage: Clinical, 103375. https://doi.org/10.1016/J.NICL.2023.103375

      Raczy, K., Holig, C., Guerreiro, M. J. S., Lingareddy, S., Kekunnaya, R., & Roder, B. (2022). Typical resting-state activity of the brain requires visual input during an early sensitive period. Brain Communications, 4(4). https://doi.org/10.1093/BRAINCOMMS/FCAC146

      Rideaux, R., Ehrhardt, S. E., Wards, Y., Filmer, H. L., Jin, J., Deelchand, D. K., Marjańska, M., Mattingley, J. B., & Dux, P. E. (2022). On the relationship between GABA+ and glutamate across the brain. NeuroImage, 257, 119273. https://doi.org/10.1016/J.NEUROIMAGE.2022.119273

      Weaver, K. E., Richards, T. L., Saenz, M., Petropoulos, H., & Fine, I. (2013). Neurochemical changes within human early blind occipital cortex. Neuroscience. https://doi.org/10.1016/j.neuroscience.2013.08.004

    1. Author Response

      The following is the authors’ response to the original reviews.

      Reviewer #1 (Public Review):

      This manuscript by Neininger-Castro and colleagues presents a novel automatic image analysis method for assessing sarcomeres, the basic units of myofibrils and validates this tool in a couple of experimental approaches that interfere with sarcomere assembly in iPSCcardiomyocytes (iPSC-CM).

      Automatic quantification of sarcomeres is definitely something that is useful to the field. I am surprised that there is no reference in the manuscript to SarcTrack, published by Toepfer and colleagues in 2019 (PMID 30700234), which has exactly the same purpose. The advantage of the image analysis software presented in the current manuscript appears to me to be that it can cover both mature sarcomeres and nascent sarcomeres in premyofibrils effectively.

      We whole-heartedly disagree that SarcTrack has the exact same purpose as sarcApp. sarcApp measures more than the frequency of actinin2 images, and can measure real-space quantifications of actinin, myomesin, and titin, which has not been done before in this way. However, SarcTrack is an interesting method that we hope many researchers find helpful in their research. SarcTrack is a particle tracker that outputs the dimensions of the objects found, but does not distinguish between Z-Lines and other actinin2-positive structures (Z-Bodies, adhesions). It also does not group these structures into higher order structures such as myofibrils and muscle stress fibers.

      When going through the manuscript there were a few issues that should be addressed in a revised version of the manuscript:

      1) I am a bit puzzled that they took 1.4 um length as a cutoff length for a mature A-band in their quantifications, since the consensus in the field for thick filament length seems to be 1.6 um?

      We use 1.4 µm as a cutoff length for the length of a Z-Line rather than the A-Band. We believe the reviewer is referring to the width of the A-Band perpendicular to the Z-lines, which is indeed 1.6 µm. However, we are referring to the length of the Z-Lines, which can span anywhere from 1.4 µm to up to 10 or more µm. Thank you for allowing us to make the clarification.

      2) When doing the knockdown for alpha and beta-myosin heavy chain, respectively, why did they not also do a Western blot for the "other" isoform as well (Figure 7)? We know that iPSCCM express a mixture, so the relatively mild phenotype that they observe in single knockdown experiments may well be due to concomitant upregulation of the expression of the other isoform. In my point of view this should be checked.

      It is likely that in the single knockdown experiments the other isoform is upregulated, which is why we were careful in stating that neither muscle myosin alone is required for sarcomere formation. We do agree this would be an interesting experiment to check beyond the scope of this manuscript.

      3) There seems to be a disconnect between the images for myomesin knockdown shown in Figure 8H and the quantification shown in Figure 8I, which makes me wonder whether the image shown in H middle (MYOM1 (1) KD), where the beta-myosin doublets do not seem to be much affected is really representative?

      The image shown in the middle of H is representative of the mean length of beta-myosin doublets in MYOM1 (1) KD hiCMs. While the beta-myosin doublets are still present and organized, they are significantly shorter. In the zoomed out image, you can appreciate much shorter arrays of beta-myosin doublets that, while extending across the entire cell, are thinner than control cells.

      Reviewer #2 (Public Review):

      Neininger-Castro et al report on their original study entitled "Independent regulation of Z-lines and M-lines during sarcomere assembly in cardiac myocytes revealed by the automatic image analysis software sarcApp", In this study, the research team developed two software, yoU-Net and sarcApp, that provide new binarization and sarcomere quantification methods. The authors further utilized human induced pluripotent stem cell-derived cardiomyocytes (hiCMs) as their model to verify their software by staining multiple sarcomeric components with and without the treatment of Blebbistatin, a known myosin II activity inhibitor. With the treatment of different Blebbistatin concentrations, the morphology of sarcomeric proteins was disturbed. These disrupted sarcomeric structures were further quantified using sarcApp and the quantification data supported the phenotype. The authors further investigated the roles of muscle myosins in sarcomere assembly by knocking down MYH6, MYH7, or MYOM in hiCMs. The knockdown of these genes did not affect Z-line assembly yet the knockdown of MYOM affected M-line assembly. The authors demonstrated that different muscle myosins participate in sarcomere assembly in different manners.

      Reviewer #3 (Public Review):

      Neininger-Castro and colleagues developed software tools for the quantification of sarcomeres and sarcomere-precursor features in immunostained human induced pluripotent stem cellderived cardiac myocytes (hiCMs). In the first part they used a deep-learning- based model called a U-Net to construct and train a network for binarization of immunostained cardiomyocyte images. They also wrote graphical user interface (GUI) software that will assist other labs in using this approach and made it publicly available. They did not compare their approach to existing ones, but an example from one image suggests their binarization tool outperforms Otsu thresholding binarization.

      In the second part they developed a software tool called sarcApp that classifies sarcomere structures in the binarized image as a Z-Line or Z-Body and assigns each to either a myofibril or to stress fibers. The tools can then automatically count and measure multiple features (33 per cell and 24 per myofibril) and report them on a per-cell, per-myofibril, and per- stress fiber basis.

      To test the tools they used Blebbistatin to inhibit sarcomere assembly and showed that the sarcApp tool could capture changes in multiple features such as fewer myofibrils, fewer Z-Lines, decreased myofibril persistence, decreased Z-Line length and altered myofibril orientation in the Blebbistatin treated cells. With some changes the tool was also shown to quantify sarcomeres in titin and myomesin stained cardiomyocytes.

      Finally they used sarcApp to quantify the changes in sarcomere assembly after siRNA mediated knockout of MYH7, MYH7, or MYOM. The analysis indicates that neither MYH6 nor MYH7 knockdown perturbed the assembly of Z- or M-lines, and that knockdown of MYOM perturbed the A-band/M-Line but not the Z-Line assembly according to features captured by the sarcApp tool.

      Overall the authors developed and made publicly available an excellent software tool that will be very useful for labs that are interested in studying sarcomere assembly. Multiple features that are difficult to measure or count manually can be automatically measured by the software quickly and accurately.

      There are however some remaining questions about these tools:

      1) The binarization tool which is tailored to sarcomere image binarization appears promising but was not systematically compared with existing approaches.

      We compared it with the existing approach we used previously in the lab, which was Otsu’s method for binarization. We are not aware of several other binarization approaches to compare to, other than using other machine learning techniques that are less advanced than a U-Net, the current standard in image-to-image translation.

      2) How robust is the tool? The tool was tested on images from one type of cardiomyocytes (hiCMs) taken from one lab using Nikon Spinning Disk confocal microscope equipped with Apo TIRF Oil 100X 1.49 NA objective or instant Structured Illumination Microscopy (iSIM), using deconvolution (Microvolution software) and in a specific magnification. It remains to be seen whether the tool would be equally effective with images taken with other microscopy systems, with other cardiomyocytes (chick or neonatal rat), with different magnifications, live imaging, etc.

      We tested the software with several magnifications, with live imaging, and with other tissues. We did not include the information in the manuscript because the data we tested the software with is for future manuscripts studying different aspects of sarcomere formation and maintenance. sarcApp reliably identifies Z-Lines and sarcomeres with deconvolved widefield fluorescence images of hiCMs and frozen human tissue, and are currently using it to measure zebrafish data for another study. Further, it works for live imaging with an actinin2-GFP (or similar) label. For the titin quantification, we would recommend using only 60-100X magnification, as the titin structures (doublets and rings) are not resolvable at lower magnifications.

      3) The tool was developed for evaluation of sarcomere assembly. The authors show that for this application it can detect the perturbation by Blebbistatin, or knockdown of sarcomeric genes. It remains to be seen if this tool is also useful for assessment of sarcomere structure for other questions beside sarcomere assembly and in other sarcomere pathologies.

      While this is beyond the scope of this specific methods paper, we welcome other researchers to use our software for other questions in other pathologies. We are currently doing the same for other manuscripts from our lab.

      Reviewer #1 (Recommendations For The Authors):

      1)"alpha-actinin..., which border the sarcomeric contractile machinery (thin and thick filaments); Z-lines do NOT border thick filaments in a relaxed sarcomere

      We have removed “(thin and thick filaments)” from the text.

      2) myomesin targeting siRNAs (gene name MYOM): there are actually three genes encoding for myomesin family members, specify, which one was targeted (I am assuming MYOM1).

      Thank you for the clarification: we do target MYOM1

      3) I am not surprised that they found not many mature Z-lines in the absence of both sarcomeric myosins; a similar codependence of assembly of mature Z-discs and the presence of functional thick filaments was previously shown by Geach and colleagues in 2015 (PMID 25845369)

      Thank you for sharing this manuscript: we have added a reference to it in our study.

      Reviewer #2 (Recommendations For The Authors):

      This work offers the possibility to gain more insights into the process of sarcomere assembly through the advancement in sarcomeric or myofibril structure analyses. However, some clarifications are needed from the authors, please see below for the comments.

      1) It is recommended that the authors include the time points for replating and harvesting hiCMs. After replating, the cardiomyocytes require at least three to four days for sarcomeric structures to reform. If the hiCMs were fixed before sarcomere assembly had completed, the staining of sarcomeric proteins including ACTN2 and titin could be compromised and it is difficult to tell if the phenotypes observed were consequences of drug treatments or knockdown of sarcomeric genes or simply because the replating hiCMs were fixed before their sarcomeric structures had fully regrown. It is also recommended that the authors replate hiCMs at a fixed time point to avoid discrepancies in the data.

      Cardiomyocytes do not require three to four days for sarcomeric structures to re-form, and indeed only require 24 hours, with the first sarcomeres typically appearing at ~6 hours. We and others have published several studies demonstrating this (Fenix et al., eLIfe 2018, Taneja, Neininger and Burnette MBoC 2020, Chen et al. Nature Methods, 2022). While sarcomeres continue to develop and turn over after this time, our lab is interested in the beginning steps of sarcomerogenesis rather than the turnover of mature structures.

      2) The sarcApp automatically identifies Z-lines and Z-bodies; however, is there an option for the users to set their own thresholds? Some users may select different criterions when quantifying sarcomeres. Moreover, the Z-lines and Z-bodies identified by the software are not always accurate. Can the users modify the list manually in an unbiased way. If this function is not available, the authors may consider adding this function to their software. sarcApp measures Zline and Z-bodies length but does not measure Z-line and Z-bodies width, but sometimes it is also necessary to measure the width.

      Absolutely, users can modify the thresholds to identify Z-Lines and Z-Bodies. There is not a way for users to modify the list in an unbiased way per se, as editing the list of Z-Lines and Z-Bodies based on non-mathematical measurements is inherently biased, but the user is free to add in other Z-Lines and Z-Bodies as they wish. In this context, “manually” and “unbiased” is mutually exclusive.

      3) It is recommended that the authors include the original images beside the sarcomeric structures identified by sarcApp (Figure 2A, 2C, 4C-F and more). It would be easier to compare the original Z-lines and Z-bodies with those identified by the software.

      We have added these in Author response image 1.

      Author response image 1.

      Uncropped images and merges from Figures 2, 4 and 6, respectively.

      4) The M-line length quantification data in Figure 3G, 5F, and 6H showed different colored-dots labeling n1 to n3, but the authors did not discuss the significance of these symbols.

      We are not sure what the reviewer means by this statement: there is no significance of the different colored dots other than to mark the biological replicate shown. These graphs were created using SuperPlots, which was not stated in the original methods. It has now been added to the Statistical Analysis section.

      5) Can the authors elaborate more on the reasons why they treated Blebbistatin at concentrations of 50µM and 100µM. Previous studies showed that 25µM of Blebbistatin was sufficient to delay the transformation of cardiomyocytes (PMID 27072942). Can the authors also comment on why they selected 6 hours, 12 hours, and 24 hours post replating for drug treatment. Moreover, the drug treatment at different time points was only done on ACTN2 but not titin or myomesin.

      We selected 6, 12, and 24 hours for actinin2 to show the time course of sarcomere formation and to show that sarcomeres are developed by 24 hours, as also mentioned above. We are interested in future studies of the time course of titin and myomesin over time, and are working on it in the lab.

      We chose 50 and 100 µM Blebbistatin as these completely blocked sarcomere assembly whereas treatment with 25 µM did not. This manuscript is a methods paper that aims to validate sarcApp and show how it could be used. We did not intend for it to be a comprehensive study of how different concentrations of blebbistatin affects sarcomere assembly.

      We are also unsure what the reviewer means by “transformation of cardiomyocytes”. The manuscript with the PMID of 27072942 does not address this issue. The paper is a “review and analyze readmission data for patients who received a continuous flow left ventricular assist device (LVAD)”. We assume the reviewer is referring to differentiation. The model system we developed and published in eLife in 2018 does not use differentiating iPSC cardiac myocytes. The hiCMs we use are terminally differentiated but still immature, as they are more transcriptionally similar to primary fetal myocytes. As such, they do not maintain their sarcomeres when they removed from the 96 well and plated onto a glass coverslip for highresolution microscopy. These assemble sarcomeres within 24 hours with the sarcomeres forming close to the dorsal membrane and then rearrange overtime (e.g., moving from the top of the cell to the bottom) (Fenix et al., eLife 2018). With that said, we do agree with the reviewer that a study of sarcomere assembly in the context of cardiac myocyte differentiation would be a fascinating direction for future studies, and we think sarcApp could facilitate such studies.

      6) The authors mentioned that the myofibrils of Z-line, titin, and M-line were randomly oriented after Blebbistatin treatments. The myofibrils were randomly oriented for titin and M-line. However, the orientation of Z-line after 50µM Blebbistatin treatment was not necessarily random, only the orientation after 100µM Blebbistatin treatment was randomized. The authors might consider changing bar graph to other types of charts if the orientation was really randomized after quantification.

      We find that the bar chart is the most informative to us, but users can consider other types of charts in their analyses.

      7) It is recommended that the authors include images staining ACTN2 at lower magnifications (Figure 1A, 1C). With current images, it is true that yoU-Net can separate Z-lines from Z-bodies yet it is difficult to tell if yoU-Net can still distinguish Z-lines from Z-bodies with larger images or it only applies to a small portion of the image.

      The yoU-Net can distinguish Z-Lines from Z-Bodies with images of any size, as image size (height vs. width in pixels) does not affect how binarization occurs. During binarization, the only pixel requirement is that the width and height are divisible by 8 (for downsampling purposes). Usually this is not the case with raw images, so the image borders are slightly cropped to make them usable. In terms of resolution, we recommend using 60X-100X objectives on confocal or superresolution data for the clearest results. We have, however, successfully binarized deconvolved widefield images at 100X as well.

      8) The authors mentioned that the knockdown of MYH7 did not affect Z-lines and M-lines; however, the structures of ACTN2, myomesin, and titin appeared more organized as compared to those in control.

      We agree that the sarcomeres and myofibrils look slightly more organized, and did mean to state that the knockdown did not negatively affect Z-Lines and M-Lines and have updated the manuscript to be more accurate.

      9) Please provide the merge images for Fig. 4D, 4E, 6B

      The merge images for Fig. 4D, 4E, and 6B are included with the original images requested above (point 3)

      10) In the text, they described" "antibodies to the titin I-band localize to both MSFs and sarcomeres in hiCMs (Figure 4A). Titin forms ring-like structures around the Z-Bodies of MSFs that are closer to the apparent sarcomere transition point (Figure 4A)" However, based on the antibody information they provided, it is not explicitly recognized for N-or C-terminus TITIN. Please provide TTN N-terminus or TTN-C terminus co-stainings with ACTN2 antibody to understand which part of TTN together with ACTN2 forms a Z-Body.

      The TTN antibody is an N-terminal antibody localizing to the I-Band region of sarcomeres. We agree with the reviewer that a more thorough study of titin will be of interest and we are currently undertaking such a study. However, this is a methods paper presenting a tool. While some of the data we present does point to mechanistic hypotheses, it is beyond the scope of this study to fully characterize titin during sarcomere assembly.

      11) TITIN doublet was used to indicate a sarcomere in Fig. 4C-D. Moreover, they also used another combination (myomesin and F-ACTIN) to label a sarcomere in Fig. 6D. Can they compare the difference between these two methods or by using these two methods (TITIN doublet) and (myomesin and F-ACTIN), how is the average length of sarcomere? Will the sarcomere length be the same?

      We noted in the manuscript that due to the organization of titin doublets (wrapping around the ends of Z-Lines) that the average titin doublet will be approximately 0.3 um longer than the ZLine. We did not expect to see a difference in lengths of myomesin M-Lines and mature actinin2 Z-Lines and indeed do not see major differences in the average lengths (between 2.0 and 2.5 um in 24 hour control cells)

      12) They used siRNA method to knockdown MYH6, MYH7 and MYOM and concluded that the knockdown of these genes did not affect the Z-line assembly. Even though they showed very nice knockdown efficiency of these proteins, they should (1) co-stain MYH6/TITIN/actinin2 and MYH6/ myomesin /actinin2 for Fig. 7C. (2) MYH7/TITIN/actinin2 and MYH7/ myomesin /actinin2 for Fig. 7I. (3) MYOM1/TITIN/actinin2 and MYOM2/TITIN/actinin2 for Fig. 8A. (4) MYH7/MYOM1 and MYH7/MYOM2 for Fig. 8H to make sure the cells they measured were truly knockdownpositive cells,

      The antibodies for alpha and beta myosin are not very efficient for immunofluorescence, and work best for western blots. We decided also to choose a random subset of the cells on the dish to be sure to eliminate any risk of cherry-picking. While imaging cells on the dish, we looked only at the DAPI nuclear channel and selected 50 cells minimum per dish with only this channel, then imaged the other channels.

      Minor comments:

      1) Well-organized sarcomere structure on DMSO treated cells in Fig.5A and Fig. 6A, but it was disarray in Fig. S3M. Why?

      Figure S3 shows hiCMs that have only been allowed to spread for 6 hours, which have not formed mature sarcomeres yet, hence the disarray.

      2) Fig 1A, Fig2B: please label the name of the antibody, not the actin filament

      We used phalloidin labelling here, which marks actin filaments. We have updated the figure legends to be more clear. Thank you!

      3) Fig. 7I: actinin2 instead of actinin

      Thank you for catching this! We have fixed it.

      Reviewer #3 (Recommendations For The Authors):

      Testing the app using images shot by other microscopy systems, magnifications, and cardiomyocytes from other species, as noted in the public review above, should make the app even more wildly useful.

      A more formal head-to-head comparison with other approaches will be more convincing in showing the new tool is superior

      I also think that a more detailed protocol for using the app will help other investigators.

      The app counts and measures many features, but it is not always clear how and using what algorithm these are measured. Including these details in a protocol or even as comments in the code will be very helpful for others.

      The protocol found on the public GitHub for the app will help other investigators to download, use, and understand the application. We have received contact from researchers who have been able to use the application without assistance from us, which is a good sign that the application is user-friendly and that the online protocol is sufficient.

    1. Author response:

      The following is the authors’ response to the original reviews.

      The reviewers praised multiple aspects of our study. Reviewer 1 noted that “the work aligns well with current research trends and will greatly interest researchers in the field.” Reviewer 2 highlighted the unique capability of our imaging approach, which “allows for investigation of the heterogeneity of response across individual dopamine axons, unlike other common approaches such as fiber photometry.” Reviewer 3 commented that “the experiments are beautifully executed” and “are revealing novel information about how aversive and rewarding stimuli is encoded at the level of individual axons, in a way that has not been done before.”

      In addition to the positive feedback, the reviewers also provided useful criticisms and suggestions, some of which may not be fully addressed in a single study. For instance, questions regarding whether dopamine axons encode the valence or specific identity of the stimuli, or the most salient aspects of the environment, remain open. At the same time, as all the reviewers agreed, our report on the diversity of dopamine axonal responses using a novel imaging design introduces significant new insights to the neuroscience community. Following the reviewers’ recommendations, we have refrained from making interpretations that could be perceived as overinterpretation, such as concluding that “dopamine axons are involved in aversive processing.” This has necessitated extensive revisions, including modifying the title of our manuscript to make clear that the novelty of our work is revealing ‘functional diversity’ using our new imaging approach.

      Below, we respond to the reviewers’ comments point by point.

      eLife assessment

      This valuable study shows that distinct midbrain dopaminergic axons in the medial prefrontal cortex respond to aversive and rewarding stimuli and suggest that they are biased toward aversive processing. The use of innovative microprism based two-photon calcium imaging to study single axon heterogeneity is solid, although the experimental design could be optimized to distinguish aversive valence from stimulus salience and identity in this dopamine projection. This work will be of interest to neuroscientists working on neuromodulatory systems, cortical function and decision making.

      Reviewer #1

      Summary:

      In this manuscript, Abe and colleagues employ in vivo 2-photon calcium imaging of dopaminergic axons in the mPFC. The study reveals that these axons primarily respond to unconditioned aversive stimuli (US) and enhance their responses to initially-neutral stimuli after classical association learning. The manuscript is well-structured and presents results clearly. The utilization of a refined prism-based imaging technique, though not entirely novel, is well-implemented. The study's significance lies in its contribution to the existing literature by offering single-axon resolution functional insights, supplementing prior bulk measurements of calcium or dopamine release. Given the current focus on neuromodulator neuron heterogeneity, the work aligns well with current research trends and will greatly interest researchers in the field.

      However, I would like to highlight that the authors could further enhance their manuscript by addressing study limitations more comprehensively and by providing essential details to ensure the reproducibility of their research. In light of this, I have a number of comments and suggestions that, if incorporated, would significantly contribute to the manuscript's value to the field.

      Strengths:

      • Descriptive.

      • Utilization of a well-optimized prism-based imaging method.

      • Provides valuable single-axon resolution functional observations, filling a gap in existing literature.

      • Timely contribution to the study of neuromodulator neuron heterogeneity.

      We thank the reviewer for this positive assessment.

      Weaknesses:

      (1) It's important to fully discuss the fact that the measurements were carried out only on superficial layers (30-100um), while major dopamine projections target deep layers of the mPFC as discussed in the cited literature (Vander Weele et al., 2018) and as illustrated in FigS1B,C. This limitation should be explicitly acknowledged and discussed in the manuscript, especially given the potential functional heterogeneity among dopamine neurons in different layers. This potential across-layer heterogeneity could also be the cause of discrepancy among past recording studies with different measurement modalities. Also, mentioning technical limitations would be informative. For example: how deep the authors can perform 2p-imaging through the prism? was the "30-100um" maximum depth the authors could get?

      Thank you for pointing out this important issue about layer differences.

      It is possible that the mesocortial pathway has layer-specific channels, with some neurons targeting supra granular layers and others targeting infragranular ones. Alternatively, it is also plausible that the axons of the same neurons branch into both superficial and deep layers. This is a critical issue that has not been investigated in anatomical studies and will require single-cell labeling of dopamine neurons (Matsuda et al 2009 and Aransay et al 2015). We now discuss this issue in the Discussion.

      As for the imaging depth of 30–100 m, we were unable to visualize deeper axons in a live view mode. Our imaging system has already been optimized to detect weak signals (e.g., we have employed an excitation wavelength of 980 nm, dispersion compensation, and a hybrid photodetector). It is possible that future studies using improved imaging approaches may be able to visualize deeper layers. Importantly, sparse axons in the supragranular layers are advantageous in detecting weak signals; dense labeling of axons would increase the background fluorescence relative to signals. We now reference this layer issue in the Results and Discussion sections.

      (2) In the introduction, it seems that the authors intended to refer to Poulin et al. 2018 regarding molecular/anatomical heterogeneity of dopamine neurons, but they inadvertently cited Poulin et al. 2016 (a general review on scRNAseq). Additionally, the statement that "dopamine neurons that project to the PFC show unique genetic profiles (line 85)" requires clarification, as Poulin et al. 2018 did not specifically establish this point. Instead, they found at least the Vglut2/Cck+ population projects into mPFC, and they did not reject the possibility of other subclasses projecting to mPFC. Rather, they observed denser innervation with DAT-cre, suggesting that non-Vglut2/Cck populations would also project to mPFC. Discuss the potential molecular heterogeneity among mPFC dopamine axons in light of the sampling limitation mentioned earlier.

      We thank the reviewer for pointing this out. Genetic profiles of PFC-projecting DA neurons are still being investigated, so describing them as “unique” was misleading. We have edited the Introduction accordingly, and now discuss this issue in detail in the Discussion.

      (3) I find the data presented in Figure 2 to be odd. Firstly, the latency of shock responses in the representative axons (right panels of G, H) is consistently very long - nearly 500ms. It raises a query whether this is a biological phenomenon or if it stems from a potential technical artifact, possibly arising from an issue in synchronization between the 2-photon imaging and stimulus presentation. My reservations are compounded by the notable absence of comprehensive information concerning the synchronization of the experimental system in the method section.

      The synchronization of the stimulus and data acquisition is accomplished at a sub-millisecond resolution. We use a custom-made MATLAB program that sends TTL commands to standard imaging software (ThorImage or ScanImage) and a stimulator for electrical shocks. All events are recorded as analogue inputs to a different DAQ to ensure synchronization. We have provided additional details regarding the configuration in the Methods section.

      We consider that the long latency of shock response is biological. For instance, a similar long latency was found after electrical shock in a photometry imaging study (Kim, …, Deisseroth, 2016).

      Secondly, there appear to be irregularities in Panel J. While the authors indicate that "Significant axons were classified as either reward-preferring (cyan) or aversive-preferring (magenta), based on whether the axons are above or below the unity line of the reward/aversive scatter plot (Line 566)," a cyan dot slightly but clearly deviates above the unity line (around coordinates (x, y) = (20, 21)). This needs clarification. Lastly, when categorizing axons for analysis of conditioning data in Fig3 (not Fig2), the authors stated "The color-coded classification (cyan/magenta) was based on k-means clustering, using the responses before classical conditioning (Figure 2J)". I do not understand why the authors used different classification methods for two almost identical datasets.

      We thank the reviewer for pointing out these insufficient descriptions. We classified the axons using k-means clustering, and the separation of the two clusters happened to roughly coincide with the unity line of the reward/aversive scatter plot in Fig 2J. In other words, we did not use the unity line to classify the data points (which is why the color separation of the histogram is not at 45 degrees). We have clarified this point in the Methods section.

      (4) In connection with Point 3, conducting separate statistical analyses for aversive and rewarding stimuli would offer a fairer approach. This could potentially reveal a subset of axons that display responses to both aversive and appetitive stimuli, aligning more accurately with the true underlying dynamics. Moreover, the characterization of Figure 2J as a bimodal distribution while disregarding the presence of axons responsive to both aversive and appetitive cues seems somewhat arbitrary and circular logic. A more inclusive consideration of this dual-responsive population could contribute to a more comprehensive interpretation.

      We also attempted k-means clustering with additional dimensions (e.g., temporal domains as shown in Fig. 3I, J), but no additional clusters were evident. We note that the lack of other clusters does not exclude the possibility of their existence, which may only become apparent with a substantial increase in the number of samples. In the current report, we present the clusters that were the easiest/simplest for us to identify.

      Additionally, we have revised our manuscript to reflect that many axons respond to both reward and aversive stimuli, and that aversive-preferring axons do not exclusively respond to the aversive stimulus.

      (5) The contrast in initialization to novel cues between aversive and appetitive axons mirrors findings in other areas, such as the tail-of-striatum (TS) and ventral striatum (VS) projecting dopamine neurons (Menegas et al., 2017, not 2018). You might consider citing this very relevant study and discussing potential collateral projections between mPFC and TS or VS.

      Thank you for pointing this out. We have now included Menegas et al., 2017, and also discuss the possibility of collaterals to these areas. In addition, we also referred to Azcorra et al., 2023 - this was published after our initial submission.

      (6) The use of correlation values (here >0.65) to group ROIs into axons is common but should be justified based on axon density in the FOV and imaging quality. It's important to present the distribution of correlation values and demonstrate the consistency of results with varying cut-off values. Also, provide insights into the reliability of aversive/appetitive classifications for individual ROIs with high correlations. Importantly, if you do the statistical testing and aversive/appetitive classifications for individual ROIs with above-threshold high correlation (to be grouped into the same axon), do they always fall into the same category? How many false positives/false negatives are observed?


      "Our results remained similar for different correlation threshold values (Line 556)" (data not shown) is obsolete.

      We have conducted additional analysis using correlation values 0.5 and 0.3 that resulted in a smaller number of axon terminals. In essence, the relationship between reward responses and aversive responses remained very similar to Fig. 2J, K.

      Author response image 1.

      Reviewer #2 (Public Review):

      Summary:

      This study aims to address existing differences in the literature regarding the extent of reward versus aversive dopamine signaling in the prefrontal cortex. To do so, the authors chose to present mice with both a reward and an aversive stimulus during different trials each day. The authors used high spatial resolution two-photon calcium imaging of individual dopaminergic axons in the medial PFC to characterize the response of these axons to determine the selectivity of responses in unique axons. They also paired the reward (water) and an aversive stimulus (tail shock) with auditory tones and recorded across 12 days of associative learning.

      The authors find that some axons respond to both reward and aversive unconditioned stimuli, but overall, there is a strong preference to respond to aversive stimuli consistent with expectations from prior studies that used other recording methods. The authors find that both of their two auditory stimuli initially drive responses in axons, but that with training axons develop more selective responses for the shock associated tone indicating that associative learning led to changes in these axon's responses. Finally, the authors use anticipatory behaviors during the conditioned stimuli and facial expressions to determine stimulus discrimination and relate dopamine axons signals with this behavioral evidence of discrimination. This study takes advantage of cutting-edge imaging approaches to resolve the extent to which dopamine axons in PFC respond appetitive or aversive stimuli. They conclude that there is a strong bias to respond to the aversive tail shock in most axons and weaker more sparse representation of water reward.

      Strengths:

      The strength of this study is the imaging approach that allows for investigation of the heterogeneity of response across individual dopamine axons, unlike other common approaches such as fiber photometry which provide a measure of the average population activity. The use of appetitive and aversive stimuli to probe responses across individual axons is another strength.

      We thank the reviewer for this positive assessment.

      Weaknesses:

      A weakness of this study is the design of the associative conditioning paradigm. The use of only a single reward and single aversive stimulus makes it difficult to know whether these results are specific to the valence of the stimuli versus the specific identity of the stimuli. Further, the reward presentations are more numerous than the aversive trials making it unclear how much novelty and habituation account for results. Moreover, the training seems somewhat limited by the low number of trials and did not result in strong associative conditioning. The lack of omission responses reported may reflect weak associative conditioning. Finally, the study provides a small advance in our understanding of dopamine signaling in the PFC and lacks evidence for if and what might be the consequence of these axonal responses on PFC dopamine concentrations and PFC neuron activity.

      We thank the reviewer for the suggestions.

      We agree that interpreting the response change during classical conditioning is not straightforward. Although the reward and aversive stimuli we employed are commonly used in the field, future studies with more sophisticated paradigms will be necessary to address whether dopamine axons encode the valence of the stimuli, the specific identity of the stimuli, or novelty and habituation. In our current manuscript, we refrain from making a conclusion that distinct groups of neurons encode different valances. In fact, many axons respond to both stimuli, at different ratios. We have removed descriptions that may suggest exclusive coding of reward or aversive processing. Additionally, we have extensively discussed possible interpretations.

      In terms of the strength of the conditioning association, behavioral results indicated that the learning plateaued – anticipatory behaviors did not increase during the last two phases when the conditioned span was divided into six phases (Figure 3–figure supplement 1).

      Our goal in the current manuscript is to provide new insight into the functional diversity of dopamine axons in the mPFC. Investigating the impact of dopamine axons on local dopamine concentration and neural activity in the mPFC is important but falls beyond the scope of our current study. In particular, given the functional diversity of dopamine axons, interpreting bulk optogenetic or chemogenetic axonal manipulation experiments would not be straightforward. As suggested, measuring the dopamine concentration through two-photon imaging of dopamine sensors and monitoring the activity of dopamine recipient neurons (e.g., D1R- or D2R-expressing neurons) is a promising approach that we plan to undertake in the near future.

      Reviewer #3 (Public Review):

      Summary:

      The authors image dopamine axons in medial prefrontal cortex (mPFC) using microprism-mediated two-photon calcium imaging. They image these axons as mice learn that two auditory cues predict two distinct outcomes, tailshock or water delivery. They find that some axons show a preference for encoding of the shock and some show a preference for encoding of water. The authors report a greater number of dopamine axons in mPFC that respond to shock. Across time, the shock-preferring axons begin to respond preferentially to the cue predicting shock, while there is a less pronounced increase in the water-responsive axons that acquire a response to the water-predictive cue (these axons also increase non-significantly to the shock-predictive cue). These data lead the authors to argue that dopamine axons in mPFC preferentially encode aversive stimuli.

      Strengths:

      The experiments are beautifully executed and the authors have mastered an impressively complex technique. Specifically, they are able to image and track individual dopamine axons in mPFC across days of learning. This technique is used the way it should be: the authors isolate distinct dopamine axons in mPFC and characterize their encoding preferences and how this evolves across learning of cue-shock and cue-water contingencies. Thus, these experiments are revealing novel information about how aversive and rewarding stimuli is encoded at the level of individual axons, in a way that has not been done before. This is timely and important.

      We thank the reviewer for this positive assessment.

      Weaknesses:

      The overarching conclusion of the paper is that dopamine axons preferentially encode aversive stimuli. This is prevalent in the title, abstract, and throughout the manuscript. This is fundamentally confounded. As the authors point out themselves, the axonal response to stimuli is sensitive to outcome magnitude (Supp Fig 3). That is, if you increase the magnitude of water or shock that is delivered, you increase the change in fluorescence that is seen in the axons. Unsurprisingly, the change in fluorescence that is seen to shock is considerably higher than water reward.

      We agree that the interpretation of our results is not straightforward. Our current manuscript now focuses on our strength, which is reporting the functional diversity of dopamine axons. Therefore, we avoid using the word ‘encode’ when describing the response.

      We believe that our results could reconcile the apparent discrepancy as to why some previous studies reported only aversive responses while others reported reward responses. In particular, if the reward volume were very small, the reward response could go undetected.

      Further, when the mice are first given unexpected water delivery and have not yet experienced the aversive stimuli, over 40% of the axons respond [yet just a few lines below the authors write: "Previous studies have demonstrated that the overall dopamine release at the mPFC or the summed activity of mPFC dopamine axons exhibits a strong response to aversive stimuli (e.g., tail shock), but little to rewards", which seems inconsistent with their own data].

      We always recorded the reward and aversive response together, which might have confused the reviewer. Therefore, there is no inconsistency in our data. We have clarified our methods and reasoning accordingly.

      Given these aspects of the data, it could be the case that the dopamine axons in mPFC encodes different types of information and delegates preferential processing to the most salient outcome across time.

      This is certainly an exciting interpretation, so we have included it in our discussion. Meanwhile, ‘the most salient outcome’ alone cannot fully capture the diverse response patterns of the dopaminergic axons, particularly reward-preferring axons. We discuss our findings in more detail in the revised manuscript.

      The use of two similar sounding tones (9Khz and 12KHz) for the reward and aversive predicting cues are likely to enhance this as it requires a fine-grained distinction between the two cues in order to learn effectively. There is considerable literature on mPFC function across species that would support such a view. Specifically, theories of mPFC function (in particular prelimbic cortex, which is where the axon images are mostly taken) generally center around resolution of conflict in what to respond, learn about, and attend to. That is, mPFC is important for devoting the most resources (learning, behavior) to the most relevant outcomes in the environment. This data then, provides a mechanism for this to occur in mPFC. That is, dopamine axons signal to the mPFC the most salient aspects of the environment, which should be preferentially learned about and responded towards. This is also consistent with the absence of a negative prediction error during omission: the dopamine axons show increases in responses during receipt of unexpected outcomes, but do not encode negative errors. This supports a role for this projection in helping to allocate resources to the most salient outcomes and their predictors, and not learning per se. Below are a just few references from the rich literature on mPFC function (some consider rodent mPFC analogous to DLPFC, some mPFC), which advocate for a role in this region in allocating attention and cognitive resources to most relevant stimuli, and do not indicate preferential processing of aversive stimuli.

      Distinguishing between 9 kHz and 12 kHz sound tones may not be that difficult, considering anticipatory licking and running are differentially manifested. In addition, previous studies have shown that mice can distinguish between two sound tones when they are separated by 7% (de Hoz and Nelken 2014). Nonetheless, we agree with the attractive interpretation that “the mPFC devotes the most resources (learning, behavior) to the most relevant outcomes in the environment” and that dopamine is a mechanism for this. Therefore, we discuss this interpretation in the revised text.

      References:

      (1) Miller, E. K., & Cohen, J. D. (2001). An integrative theory of prefrontal cortex function. Annual review of neuroscience, 24(1), 167-202.

      (2) Bissonette, G. B., Powell, E. M., & Roesch, M. R. (2013). Neural structures underlying set-shifting: roles of medial prefrontal cortex and anterior cingulate cortex. Behavioural brain research, 250, 91101.

      (3) Desimone, R., & Duncan, J. (1995). Neural mechanisms of selective visual attention. Annual review of neuroscience, 18(1), 193-222.

      (4) Sharpe, M. J., Stalnaker, T., Schuck, N. W., Killcross, S., Schoenbaum, G., & Niv, Y. (2019). An integrated model of action selection: distinct modes of cortical control of striatal decision making. Annual review of psychology, 70, 53-76.

      (5) Ridderinkhof, K. R., Ullsperger, M., Crone, E. A., & Nieuwenhuis, S. (2004). The role of the medial frontal cortex in cognitive control. science, 306(5695), 443-447.

      (6) Nee, D. E., Kastner, S., & Brown, J. W. (2011). Functional heterogeneity of conflict, error, taskswitching, and unexpectedness effects within medial prefrontal cortex. Neuroimage, 54(1), 528-540.

      (7) Isoda, M., & Hikosaka, O. (2007). Switching from automatic to controlled action by monkey medial frontal cortex. Nature neuroscience, 10(2), 240-248.

      Reviewer #1 (Recommendations For The Authors):

      Specific Suggestions and Questions on the Methods Section:

      In general, the methods part is not well documented and sometimes confusing. Thus, as it stands, it hinders reproducible research. Specific suggestions/questions are listed in the following section.

      (1) Broussard et al. 2018 introduced axon-GCaMP6 instead of axon-jGCaMP8m. The authors should provide details about the source of this material. If it was custom-made, a description of the subcloning process would be appreciated. Additionally, consider depositing sequence information or preferably the plasmid itself. Furthermore, the introduction of the jGCaMP8 series by Zhang, Rozsa, et al. 2023 should be acknowledged and referenced in your manuscript.

      We thank the reviewer for pointing this out. We have now included details on how we prepared the axon-jGCaMP8m, which was based on plasmids available at Addgene. Additionally, we have deposited our construct to Addgene ( https://www.addgene.org/216533/ ). We have also cited Janelia’s report on jGCaMP8, Zhang et al.

      (2) The authors elaborate on the approach taken for experimental synchronization. Specifically, how was the alignment achieved between 2-photon imaging, treadmill recordings, aversive/appetitive stimuli, and videography? It would be important to document the details of the software and hardware components employed for generating TTLs that trigger the pump, stimulator, cameras, etc.

      We have now included a more detailed explanation about the timing control. We utilize a custommade MATLAB program that sends TTL square waves and analogue waves via a single National Instruments board (USB-6229) to control two-photon image acquisition, behavior camera image acquisition, water syringe movement, current flow from a stimulator, and sound presentation. We also continuously recorded at 30 kHz via a separate National Instrument board (PCIe-6363) the frame timing of two-photon imaging, the frame timing of a behavior camera, copies of command waves (sent to the syringe pump, the stimulator, and the speaker), and signals from the treadmill corresponding to running speed.

      (3) The information regarding the cameras utilized in the study presents some confusion. In one instance, you mention, "To monitor licking behavior, the face of each mouse was filmed with a camera at 60 Hz (CM3-U3-13Y3M-CS, FLIR)" (Line 488). However, there's also a reference to filming facial expressions using an infrared web camera (Line 613). Could you clarify whether the FLIR camera (which is an industrial CMOS not a webcam) is referred to as a webcam? Alternatively, if it's a different camera being discussed, please provide product details, including pixel numbers and frame rate for clarity.

      We thank the reviewer for pointing this out. This was a mistake on our end. The camera used in the current project was a CM3-U3-13Y3M-CS, not a web camera. We have now corrected this.

      (4) Please provide more information about the methodology employed for lick detection. Specifically, did the authors solely rely on videography for this purpose? If so, why was an electrical (or capacitive) detector not used? It would provide greater accuracy in detecting licking.

      Lick detection was performed offline based on videography, using DeepLabCut. As licking occurs at a frequency of ~6.5 Hz (Xu, …, O’Connor Nature Neurosci, 2022), the movement can be detected at a frame rate of 60 Hz. Initially, we used both a lick sensor and videography. However, we favored videography because it could potentially provide non-binary information.

      Other Minor Points:

      (5) Ensure consistency in the citation format; both Vander Weele et al. 2018 and Weele et al. 2019, share the same first author.

      Thank you for pointing this out. Endnote processes the first author’s name differently depending on the journal. We fixed the error manually. The first paper (2018) is an original research paper, and the second one (2019) is a review about how dopamine modulates aversive processing in the mPFC. We cited the second one in three instances where we mentioned review papers.

      (6) The distinction between "dashed vs dotted lines" in Figure 3K and 3M appears to be very confusing. Please consider providing a clearer visualization/labeling to mitigate this confusion.

      We have now changed the line styles.

      (7) Additionally plotting mean polar angles of aversive/appetitive axons as vectors in the Cartesian scatter plots (2J, 3I,J) would make interpretation easier.

      We have now made this change to Figures 2, 3, 4.

      (8) Data and codes should be shared in a public database. This is important for reproducible research and we believe that "available from the corresponding author upon reasonable request" is outdated language.

      We have uploaded the data to GitHub, https://github.com/pharmedku/2024-elife-da-axon.

      Reviewer #2 (Recommendations For The Authors):

      (1) Authors don't show which mouse each axon data comes from making it hard to know if differences arise from inter-mouse differences vs differences in axons. The best way to address this point is to show similar plots as Figure 2J & K but broken down by mouse to shows whether each mouse had evidence of these two clusters.

      We have now made this change to Figure 2-figure supplement 3.

      (2) Line 166: Should this sentence point to panels 2F, G, H rather than 2I which doesn't show a shock response?

      We thank the reviewer for pointing this out. We have fixed the incorrect labels.

      Line 195: The population level bias to aversive stimuli was shown previously using photometry so it is not justified to say "for the first time" regarding this statement.

      We have adjusted this sentences so the claim of ”for the first time” is not associated with the population-level bias.

      (4) The paper lacks a discussion of the potential role that novelty plays in the amplitude of the responses given that tail shocks occur less often that rewards. Is the amplitude of the first reward of the day larger than subsequent rewards? Would tail shock responses decay if they occurred in sequential trials?

      Following the reviewer's suggestion, we conducted a comparison of individual axonal responses to both conditioned and unconditioned stimuli across the first trial and subsequent trials. Our findings reveal a notable trend: aversive-preferring axons exhibited attenuation in response to CSreward, yet enhancement in response to CSaversive. Conversely, the response of these axons to USreward was attenuated, with no significant change observed for USaversive. In contrast, reward-preferring axons displayed an invariable activity pattern from the initial trial, highlighting the functional diversity present within dopamine axons. This analysis has been integrated into Figure 3-figure supplement 4 and is elaborated upon in the Discussion section.

      (5) Fix typo in Figure 1 - supplement 1. Shift

      We have now corrected this. Thank you.

      (6) The methods section needs information about trial numbers. Please indicate how many trials were presented to each mouse per day.

      We have now added the information about trial numbers to the Methods section.

      Reviewer #3 (Recommendations For The Authors):

      In line with the public review, my recommendation is for the authors to remain as objective about their data as possible. There are many points in the manuscript where the authors seem to directly contradict their own data. For example, they first detail that dopamine axons respond to unexpected water rewards. Indeed, they find that there are 40% of dopamine axons that respond in this way. Then, a few paragraphs later they state: "Previous studies have demonstrated that the overall dopamine release at the mPFC or the summed activity of mPFC dopamine axons exhibits a strong response to aversive stimuli (e.g., tail shock), but little to rewards". As detailed above, I do not think these data support an idea that dopamine axons in mPFC preferentially encode aversive outcomes. If the authors wanted to examine a role for mPFC in preferential encoding of aversive stimuli, you would first have to equate the outcomes by magnitude and then compare how the axons acquire preferences across time. Alternatively, a prediction of a more general process that I detail above would predict that you could give mice two rewards that differ in magnitude (e.g., lots of food vs. small water) and you would see the same results that the authors have seen here (i.e., a preference for the food, which is the larger and more salient outcome). Without other tests of how dopamine axons in mPFC respond to situations like this, I don't think any conclusion around mPFC in favoring aversive stimuli can be made.

      As suggested, we have made the current manuscript as objective as possible, removing interpretation aspects regarding what dopamine axons encode and emphasizing their functional diversity. In particular, we remove the word ‘encode’ when describing the response of dopamine axons.

      Although it may have appeared unclear, there was no contradiction within our data regarding the response to reward and aversive stimuli. We have now improved the readability of the Results and Methods sections. Concerning the interpretation of what exactly the mPFC dopamine axons encode, we have rewritten the discussion to be as objective about our data as possible, as suggested. We also have edited our title and abstract accordingly. Meanwhile, we wish to emphasize that our reward and aversive stimuli are standard paradigms commonly used in the field. We believe, and all the reviewers agreed, that reporting the diversity of dopamine axonal responses with a novel imaging design constitutes new insight for the neuroscience community. Therefore, we have decided to leave the introduction of new behavioral tasks for future studies and instead expanded our discussion.

      As mentioned, I think the experiments are executed really well and the technological aspects of the authors' methods are impressive. However, there are also some aspects of the data presentation that would be improved. Some of the graphs took a considerable amount of effort to unpack. For example, Figure 4 is hard going. Is there a way to better illustrate the main points that this figure wants to convey? Some of this might be helped by a more complete description in the figure captions about what the data are showing. It would also be great to see how the response of dopamine axons changes across trial within a session to the shock and water-predictive cues. Supp Figure 1 should be in the main text with standard error and analyses across time. Clarifying these aspects of the data would make the paper more relevant and accessible to the field.

      We thank the reviewer for pointing out that the legend of Figure 4 was incomplete. We have fixed it, along with improving the presentation of the figure. We have also prepared a new figure (Figure 3– figure supplement 4) to compare CSaversive and CSreward signals for the first and rest of the trials within daily sessions, revealing further functional diversity in dopamine axons. We have decided to keep Figure 1–figure supplement 2 as a figure supplement with an additional analysis, as another reviewer pointed out that the design is not completely new. Furthermore, as eLife readers can easily access figure supplements, we believe it is appropriate to maintain it in this way.

      Minor points:

      (1) What is the control period for the omission test? Was omission conducted for the shock?

      The control period for reward omission is a 2-second period just before the CS onset. We did not include shock omission, because a sufficient number of trials (> 6 trials) for the rare omission condition could not be achieved within a single day.

      (2) The authors should mention how similar the tones were that predicted water and shock.

      According to de Hoz and Nelken (2014), a frequency difference of 4–7% is enough for mice to discriminate between tones. In addition, anticipatory licking and running confirmed that the mice could discriminate between the frequencies. We have now included this information in the Discussion.

      (3) I realize the viral approach used in the current studies may not allow for an idea of where in VTA dopamine neurons are that project to mPFC- is there data in the literature that speak to this? Particularly important as we now know that there is considerable heterogeneity in dopamine neuronal responses, which is often captured by differences in medial/lateral position within VTA.

      Some studies have suggested that mesocortical dopamine neurons are located in the medial posterior VTA (e.g., Lammel et al., 2008). However, in mouse anterograde tracing, it is not possible to spatially confine the injection of conventional viruses/tracers. We now refer to Lammel et al., 2008 in the Introduction.

    1. Author Response

      The following is the authors’ response to the original reviews.

      REVIEWER #1:

      The authors present a carefully controlled set of experiments that demonstrate an additional complexity for GPCR signaling in that endosomal signaling make be different when b-arrestin is or isn't associated with a G protein-bound V2R vasopressin receptor. It uses state of the art biosensorbased approaches and b-arrestin KO lines to assess this. It adds to a growing body of evidence that G proteins and b-arrestin can associate with GPCR complexes simultaneously. They also demonstrate the possibility that Gaq might also be activated by the V2R receptor. My sense is one thing they may need to be considered is the possibility of such "megacomplexes" might actually involve receptor dimers or oligomers.

      1.1 Can the authors please review the data that describes the concept of "GPCR megacomplexes"? I feel this is missing from the introduction. The notion means different things to different people. As you will see from my other comments, you should especially focus on evidence at the level of the single receptor.

      We appreciate the reviewer’s comments and have now included a more wholesome description of the GPCR megacomplex, or ‘megaplex’, concept in the introduction (page 2, 1st paragraph).

      1.2 The authors use mini-G proteins to conclude that V2R receptors interact with Gaq (in addition to Gas). I would prefer if there were a more direct measure of this. Can the authors show that the receptor interacts with full length Gaq (and not the other G proteins in Figure)? Is there a signaling phenotype associated with Gaq coupling? Is it sensitive to Gaq inhibition?

      Excellent point and we are happy to expand further on this. The ability of the V2R to activate Gq/11 has already been demonstrated before (Zhu, X. et al. Mol Pharmacol 46(3):460-9 (1994); Lykke, K. et al. Physiol Rep. 3(8):e12519 (2015); Avet, C. et al. eLife 11: e74101 (2022); Heydenreich, F.M. et al. Mol Pharmacol 102(3):139-49 (2022). Therefore, we did not attempt to document this activation using more traditional assays. On the other hand, to demonstrate an interaction between V2R and Ga subunit in cells is challenging for several reasons. First, the full-length Ga subunit is already located at the plasma membrane at basal state, and thus, generates high background signals in proximity assays. Second, upon receptor activation, the Ga subunit interaction with V2R is so transient that it is difficult, if not impossible, to catch this transient moment in a proximity assay. Although the miniG proteins are highly engineered, coupling specificity of the different subtypes (Gas, Gai/o, Gaq/11, and Ga12/13) to GPCRs is maintained. In addition, as they are homogenously expressed in the cytosol under basal states rather than at the membrane, they generate low background noise. Upon agonist stimulation, miniG proteins are recruited from the cytosol to the V2R at the plasma membrane, resulting in a robust signal in proximity assays. Thus, miniG proteins are unique in that they can actually detect GPCR–G protein interactions in cellular proximity assays, which is very challenging using full-length Ga subunits.

      That being said, we fully understand the reviewer’s concern and greatly value the effort in enhancing robustness of our study. Therefore, we have now monitored downstream signaling events of Gaq/11 in the absence or presence of the selective Gaq/11 inhibitor YM-254890 as a secondary method of documenting Gaq/11 activity. Specifically, we used a newly developed biosensor to measure diacylglycerol (DAG) production, a downstream second messenger of Gaq/11 activation, at both the plasma membrane and endosomes. Using a second biosensor, we detect general protein kinase C (PKC) activation, which is another downstream signaling event of Gaq/11 activation. Together, we demonstrated that AVP-stimulation leads to DAG production at both the plasma membrane and endosomes (Fig. 1C-D) as well as PKC activation (Fig. 1E), which all are sensitive to YM-254890 inhibition (Fig. 1C-D and E). Together these results rigorously suggest that the V2R interacts with and activates Gaq/11.

      1.3 I raise a similar concern with Gaq coupling in endosomes.

      For similar reasons that miniG proteins are excellent tools for demonstrating V2R interaction with G proteins at the plasma membrane, miniG proteins can also be used to detect V2R interaction with G proteins at endosomes by measuring proximity between miniG and an endosomal marker in response to agonist challenge. However, to ensure that the endosomal recruitment of miniGsq to the V2R demonstrated in our study corresponds to endosomal Gaq/11 activation, we monitored the production of DAG at the early endosomes in a similar way to which we detected DAG production at the plasma membrane. As shown in Fig. 1D, stimulation of V2R with AVP induces recruitment of the DAG-binding biosensor to the early endosomal marker Rab5. Pre-treatment of the cells with the selective Gaq/11 inhibitor YM-254890 abrogated this response, confirming that V2R activation leads to production of DAG at the early endosomes in a Gaq/11-dependent manner (Fig. 1D).

      1.4 Can the confocal data be shown for Gai and Ga12?

      Yes, we can certainly show this data as negative control. We have now included the confocal data using Halo-mGsi as a negative control for confocal microscopy (Fig. 2). As seen on this figure, mGsi does not colocalize with Lck (plasma membrane), nor with EEA1 (early endosomes) upon stimulation of cells with AVP in line with a receptor that does not couple to Gai/o.

      We did not include data using Halo-mG12, as this G protein subtype, similar to Gi/o, does not couple functionally to V2R. Therefore, it is highly unlikely we would obtain different results from the experiments using Halo-mGsi.

      1.5 The authors want us to believe that there is simultaneous binding of G proteins and b-arrestin. This is never demonstrated and is at odds with the structural basis of G protein and b-arrestin binding. Have the authors considered that "simultaneous" occupancy might simply reflect binding at distinct GPCR monomers in the context of dimeric or oligomeric receptors? They could I suppose provide data at the level of a single receptor rather than using the bulk BRET approaches used.

      We appreciate the comment and opportunity to highlight some of our previous work, which address the megacomplexes at the level of a single receptor. First, we have characterized the megacomplex biochemically and structurally at a low resolution (Thomsen ARB et al. 2016, Cell 166(4):907-19). The results unequivocally demonstrate that a single GPCR interacts simultaneously with heterotrimeric G protein, at the receptor core, and with b-arrestin via the phosphorylated receptor carboxy-terminal. We also documented functionality of the megacomplex as the receptor can interact with and activate the G protein, which were shown by 3 different biochemical approaches (Thomsen ARB et al. 2016, Cell 166(4):907-19). In addition, we solved a high-resolution cryo-EM structure of a megacomplex further highlighting the architecture of this complex (Nguyen AH et al. 2019, Nat Struct Mol Biol 26:1123-31). As both biochemical and structural analyses were done in vitro in which the receptor was embedded in a detergent micelle, we also confirmed that the megacomplex structural architecture fits naturally within the context of a membrane in molecular dynamics simulation experiments (Nguyen AH et al. 2019, Nat Struct Mol Biol 26:1123-31).

      In cells, we and others have also showed that GPCRs such as the V2R can bind b-arrestins exclusively via the phosphorylated carboxy-terminal tail as it does in the megacomplex (Kumari P et al. 2016, Nat Commun 7:13416; Cahill III TJ et al. 2017, PNAS 114(10):2562-67; Kumari P et al. 2017, Mol Biol Cell 28(8):1003-10; Chen K et al. 2023, Nature (online doi: https://doi.org/10.1038/s41586-023-06420-x). In addition, we and others have used BRET and confocal microscopy to show that the V2R and other GPCRs recruit G protein and b-arrestin simultaneously and that the three components colocalize in endosomes upon prolonged agonist exposure (Thomsen ARB et al. 2016, Cell 166(4):907-19; Chen K et al. 2023, Nature (online doi: https://doi.org/10.1038/s41586-023-06420-x). As the reviewer correctly points out, in these cellular experiments (as well as in single molecule microscopy), the working resolution is not high enough to rule out that the receptors that co-recruit G protein and b-arrestin in endosomes could be dimeric instead of monomeric. Thus, we conducted a series of experiments with GPCR–b-arrestin fusions where the two proteins are covalently attached at the receptor carboxy-terminal tail. We showed that despite the GPCR–b-arrestin coupling being fully functional (in respect to b-arrestin promoting a highaffinity state of the receptor for agonist binding and constitutively internalizing the receptor) the receptor could still activate G proteins (Thomsen ARB et al. 2016, Cell 166(4):907-19; Nguyen AH et al. 2019, Nat Struct Mol Biol 26:1123-31), which demonstrates that the single receptor megaplex can physically form in cells.

      We have now included an extra paragraph in the discussion to go over these megaplex-related considerations (5th paragraph in the discussion), and we thank the reviewer for raising this point.

      1.6 Please introduce abbreviations when you first use this- this was not done consistently.

      Thank you for noticing these errors, which we now have corrected.  

      REVIEWER #2:

      This manuscript by Daly et al., probes the emerging paradigm of GPCR signaling from endosomes using the V2R as a model system with an emphasis on Gaq/11 and b-arrestins. The study employs cellular imaging, enzyme complementation assays and energy transfer-based sensors to probe the potential formation of GPCR-G-protein-b-arrestin megaplexes. While the study is certainly very interesting, it appears to be very preliminary at many levels, and clearly requires further development in order to make robust conclusions. The authors should consider expanding on this work further to make the points more convincingly to make the work solid and impactful. The two corresponding authors are among the leaders in the field having demonstrated the existence of megaplexes, and building on the work in a systematic fashion should certainly move the paradigm forward. As the work presented in the current manuscript is already pre-printed, the authors should take this opportunity to present a completer and more comprehensive story to the field.

      We are grateful for the time and efforts the reviewer has put into reviewing our work. We are certainly excited to learn that the reviewer finds our work “very interesting”. Regarding the robustness, we have added extra control experiments to increase the completeness of the study. These experiments include:

      • Measurements of AVP-stimulated diacylglycerol production, a signaling event downstream of Gaq/11 activation. These measurements were conducted both at plasma membrane (Fig. 1C) and early endosomes (Fig. 1D) using a newly developed DAG-binding biosensor, and demonstrate that the V2R activates Gaq/11 at both of these subcellular locations.

      • Monitoring AVP-promoted protein kinase C activation, another downstream signaling effect of Gaq/11 activation (Fig. 1E). The result of this approach shows in another way that V2R activates of Gaq/11.

      • Inhibition of signaling events downstream of Gaq/11 activation using the selective of Gaq/11 inhibitor YM254890. YM-254890 inhibits both AVP-stimulated DAG production at plasma membrane and endosomes as well as PKC activation (Fig. 1C-E), which strongly confirms that these signaling outputs are results of Gaq/11 activation.

      • We have also included the confocal data using Halo-mGsi as a negative control for confocal microscopy (Fig. 2). As seen in this figure, mGsi does not translocate to the plasma membrane or early endosomes upon stimulation with AVP, which validates that V2R activation does not couple to and activate Gai/o.

      Finally, we would like to kindly remind the reviewer that the production of the pre-print manuscript is part of the peer-review process in eLife.

      2.1 The use of miniG proteins in these experiments is a major concern as these are highly engineered and may not represent the true features of G proteins. While these have been used as a readout in other publications, their use in demonstrating megaplex formation is sub-optimal, and native, full-length G proteins should be used.

      We are a bit unsure as to what the reviewer means by using native full-length G proteins. If the reviewer is suggesting to co-immunoprecipitate V2R with native unlabeled G protein and b-arrestin, it should be considered that the G protein interaction with the receptor is extremely transient and unlikely to survive the pull-down procedure unless stabilized by a nanobody or crosslinking. Although the b-arrestin interaction with the receptor is more stable of nature, co-immunoprecipitation with the receptor requires crosslinking or stabilization with a Fab/nanobody. Therefore, we do not think this approach can be used as a more accurate way of detecting native megaplexes.

      If the reviewer is suggesting the use of full-length G proteins in our cell-based proximity assays instead of miniG proteins, we would like to highlight that this approach is somewhat prone to false-positive responses. The major reason behind this is that G proteins are located at regions in membranes close to the receptor whereas b-arrestins are distributed throughout the cytosol. Upon activation of the V2R, barrestins translocate to the receptor at the plasma membrane, which results in enhanced BRET between V2R-coupled G protein subtypes and b-arrestins (see Author response image 1 below of preliminary data). This translocation also results in non-specific BRET signals between b-arrestins and G protein subtypes at the plasma membrane that do not couple to V2R but are located in close proximity to the receptor. As these nonspecific BRET signals do not report on the formation of functional V2R megaplexes (see Author response image 1), we have purposely not used this approach.

      Author response image 1.

      To overcome this technical hurdle in detection of functional megaplexes, we have replaced full-length G proteins by miniG proteins as the latter are located in the cytosol at resting states and only translocate to the membrane area if a receptor adopts an active conformation. This replacement is advantageous since activation of megaplex-forming receptors such as the V2R results in simultaneous translocation of miniG proteins and b-arrestins from the cytosol to the receptor at the plasma membrane, which produces a highly specific proximity signal (see Author response image 2 below of preliminary data). When stimulating the V2R, we only observe increases in proximity between b-arrestin1 and miniG proteins that are activated by the V2R (miniGs and miniGsq) but not the miniG proteins that are not activated by this receptor (miniGsi and miniG12) (see Author response image 2). Therefore, usage of miniG proteins offers a more accurate experimental approach to detect functional megaplexes as compared to the usage of full-length G proteins.

      Author response image 2.

      2.2 The interpretation of complementation (NanoLuc) or proximity (BRET) as evidence of signaling is not appropriate, especially when overexpression system and engineered constructs are being used.

      We thank the reviewer for raising this concern. We have previously demonstrated global Gas activation and Gas signaling in form of cAMP stimulated by internalized V2R (Thomsen ARB et al. 2016, Cell 166(4):907-19). As mentioned previously, in the current updated manuscript we have now included experiments to document downstream signaling events in response to Gaq/11 activation. These experiments include measurement of production of DAG at the plasma membrane (Fig. 1C) and early endosomes (Fig. 1D), as well as phosphorylation/activation of PKC (Fig. 1E). Pre-incubation with the selective Gaq/11 inhibitor YM-254890, abrogated all these downstream signals and confirms that the V2R stimulates Gaq/11 protein signaling at both the plasma membrane and endosomes (Fig. 1C-E).

      2.3 After the original work from the same corresponding authors on megaplex formation, the major challenge in the field is to demonstrate the existence and relevance of megaplex formation at endogenous levels of components, and the current study focuses solely on showing the proximity of Gaq and b-arrestins.

      We completely agree with the reviewer that it will be important to demonstrate functionality endogenous megaplexes and we are currently working on this in other studies using different receptor systems. However, doing this is not trivial and we will have to overcome major technical barriers that we feel is somewhat out of the scope of the current study. The goal of our V2R study is to demonstrate that V2R megaplexes form with Gaq/11 resulting to Gaq/11 activation at endosomes, and that endosomal G protein activation by the V2R can occur independently of b-arrestin, which we in our humble opinion accomplish.

      2.4 The study lacks a coherent approach, and the assays are often shifted back and forth between the two b-arrestin isoforms (1 and 2), for example, confocal vs. complementation etc.

      We understand the reviewer’s concern. However, as opposed to the β2-adrenergic receptor that binds βarrestin2 with higher affinity than β-arrestin1, V2R has a strong affinity for both β-arrestin1 and β-arrestin2 (Oakley et al. 2000, JBC 275(22):17201-10). The V2R’s almost identical affinity for β-arrestin1 and βarrestin2 is well illustrated in Fig. 3B. Thus, although different β-arrestin isoforms were used in some experiments, it is very unlikely that the overall results and conclusions from this study will change by adding extra experiments to ensure that both β-arrestin isoforms are used in every experiment.

      2.5 In every assay, only the G proteins and b-arrestins are monitored without a direct assessment of the presence of receptor, and absent that data, it is difficult to justify calling these entities megaplexes.

      Mini G proteins and b-arrestin come into close proximity upon agonist stimulation of the V2R. Using confocal microscopy, we observed this co-recruitment of miniGs/miniGsq and b-arrestin in response to prolonged V2R stimulation at endosomes specifically (Fig. 3D-F). In absence of GPCR stimulation, both miniG and b-arrestin would be homogenously distributed throughout the cytosol, and thus, the only reason to why both proteins have been recruited to endosomes in response to AVP challenge is that they are recruited to internalized and active V2R. This point was obviously not adequately described in the original manuscript, and thus, we have now clarified this further in the updated manuscript at the 8th sentence of the last paragraph of the "The V2R recruits Gas/Gaq and barrs simultaneously" section.

      REVIEWER #3:

      The manuscript by Daly et al. examines endosomal signaling of the vasopressin type 2 receptors using engineered mini G protein (mG proteins) and a number of novel techniques to address if sustained G protein signaling in the endosomal compartment is enhanced by b-arrestin. Employing these interesting techniques they have how V2R could activates Gas and Gaq in the endosomal compartments and how this modulation could occur in arrestin-dependent and -independent manner. Although the phenomenon of endosomal signaling is complex to address the authors have tried their best to examine these using a number of well controlled set of experiments. Though this is an interesting and well carried out study of endosomal signaling of G proteins, my concerns are:

      3.1 The study is done in overexpressed HEK 293 cells with these engineered constructs making me wonder if the kinetics would be the same in primary cells?

      The reviewer raises an interesting and valid point. It is possible that in the context of primary cells the kinetic would differ slightly and it would definitely be interesting to address this in a subsequent study. However, despite being an interesting aspect of our study, the kinetic itself is not our major take home message, but rather the subcellular localization of the G protein activation and the role of β-arrestin in these events. We have now highlighted this aspect in our updated manuscript (1st paragraph of the discussion) and we thank the reviewer for addressing this.

      3.2 The use of the phrase "G protein activation independent of b-arrestins to a minor degree" would make me question its physiological relevance. The authors should discuss the relevance of their findings in physiological or pathological context.

      We are glad that the reviewer focuses on this point, and we would like to highlight that other GPCRs including the glucagon-like peptide-1 receptor (GLP1R) internalizes in a β-arrestin-independent manner (Claing A et al. 2000 PNAS 97(3):1119-24), while signaling through Gas from endosomes. In the case of the GLP1R, this endosomal Gas signaling promotes glucose-stimulated insulin secretion in pancreatic βcells (Kuna RS et al. 2013 Am J Physiol Endocrinol Metab 305:E161-70). Consequently, β-arrestinindependent endosomal G protein signaling appears to have some physiological relevance. Similarly, in a very recent pre-print from the von Zastrow group (Blythe EE and von Zastrow M 2023 BioRxiv https://doi.org/10.1101/2022.09.07.506997), it was reported that endogenously-expressed vasoactive intestinal peptide receptor 1 (VIPR1), which regulates gastro-intestinal functions, promotes robust G protein signaling from endosomes in a completely β-arrestin-independent fashion. This again suggest that endogenously expressed GPCRs can internalize and activate G proteins from endosomes independently from β-arrestin to produce physiological responses. We have now discussed about these studies in the 6th paragraph of the discussion.

      3.3 The confocal colocalization studies shown in Figure 2 and their conclusion "suggesting a certain level of endosomal Gas/Gaq signaling despite the absence of barr2" seems rather inconclusive.

      As opposed to V2R a receptor that retains β-arrestin in endosomes upon internalization, β-arrestin quickly dissociates from V2b2AR after internalization due to the low affinity of the carboxy-terminal of β2AR for βarrestin. In the previous Fig. 2 (now Fig. 3), after 45 minutes of AVP stimulation, no β-arrestin is visible at endosomes in cells expressing V2b2AR as β-arrestin has already dissociated from the receptor and translocated back to the cytosol. However, clear green clusters of mGs and mGsq are still visible at endosomes indicating the presence of active receptor interacting with Gas or Gaq despite the fact that βarrestin is back to the cytosol. We quantified the percentage of the green mGs or mGsq clusters that do not colocalize with β-arrestin and have added this information to the updated version of the manuscript (Fig. 3G). In V2R-expressing cells, almost all active receptors that interact with Gas or Gaq/11 also associate with β-arrestin (Fig. 3G). In contrast, in V2b2AR-expressing cells, approximately 75% of the active receptors do not interact with β-arrestin (Fig. 3G). This suggests that β-arrestin binding to V2R is not an absolute requirement for endosomal Gas and Gaq activation by V2R. This point was obviously not addressed adequately in the original manuscript, and thus, we have now elaborated further on this in the updated version in the last paragraph of the "The V2R recruits Gas/Gaq and βarrs simultaneously" section.

      3.4 Though a novel observation it is not clear to me how V2R would internalize after activation without arrestin. Is it some sort of generalized microcytosis occurring in these overexpressed cells? Should discuss.

      This is certainly a very interesting observation and something other research laboratories also have seen recently – in particular, in context to endosomal G protein signaling (Blythe EE and von Zastrow M 2023 BioRxiv https://doi.org/10.1101/2022.09.07.506997). The main and best characterized pathway for GPCR internalization is clathrin-dependent where receptors most commonly are associated with β-arrestins. However, for some GPCRs, the β-arrestin association is not required for clathrin-mediated internalization. One example is the apelin receptor that can internalize via clathrin-coated pits, but in β-arrestinindependent manner (Pope GR et al. 2016 Moll Cell Endocrinol. 437:108-19). Alternatively, GPCRs can also internalize independently of any clathrin and β-arrestin associations via caveolae or fast endophilinmediated endocytosis (FEME). We have now expanded our discussion of possible mechanisms for βarrestin-independent receptor internalization in the updated manuscript in the 6th paragraph of the discussion, and we thank the reviewer for the suggestion.

      3.5 Is use of mini G protein a good representation? The authors should justify.

      Excellent point and something we have comprehensively discussed in our response to reviewer 1 and 2 (points 1.2 and 2.1).

    1. Author response:

      The following is the authors’ response to the original reviews.

      Reviewer #1 (Public Review):

      Summary:

      Bendzunas, Byrne et al. explore two highly topical areas of protein kinase regulation in this manuscript. Firstly, the idea that Cys modification could regulate kinase activity. The senior authors have published some standout papers exploring this idea of late, and the current work adds to the picture of how active site Cys might have been favoured in evolution to serve critical regulatory functions. Second, BRSK1/2 are understudied kinases listed as part of the "dark kinome" so any knowledge of their underlying regulation is of critical importance to advancing the field.

      Strengths:

      In this study, the author pinpoints highly-conserved, but BRSK-specific, Cys residues as key players in kinase regulation. There is a delicate balance between equating what happens in vitro with recombinant proteins relative to what the functional consequence of Cys mutation might be in cells or organisms, but the authors are very clear with the caveats relating to these connections in their descriptions and discussion. Accordingly, by extension, they present a very sound biochemical case for how Cys modification might influence kinase activity in cellular environs.

      Weaknesses:

      I have very few critiques for this study, and my major points are barely major.

      Major points

      (1) My sense is that the influence of Cys mutation on dimerization is going to be one of the first queries readers consider as they read the work. It would be, in my opinion, useful to bring forward the dimer section in the manuscript.

      We agree that the influence of Cys on BRSK dimerization is a topic of significant interest. Our primary focus was to explore oxidative regulation of the understudied BRSK kinases as they contain a conserved T-loop Cys, and we have previously demonstrated that equivalent residues at this position in related kinases were critical drivers of oxidative modulation of catalytic activity. We have demonstrated here that BRSK1 & 2 are similarly regulated by redox and this is due to oxidative modification of the T+2 Cys, in addition to Cys residues that are conserved amongst related ARKs as well as BRSK-specific Cys. Although we also provide evidence for limited redox-sensitive higher order BRSK species (dimers) in our in vitro analysis, these represent a small population of the total BRSK protein pool (this was validated by SEC-MALs analysis). As such, we do not have strong evidence to suggest that these limited dimers significantly contribute to the pronounced inhibition of BRSK1 & 2 in the presence of oxidizing agents, and instead believe that other biochemical mechanisms likely drive this response. This may result from oxidized Cys altering the conformation of the activation loop. Indeed, the formation of an intramolecular disulfide within the T-loop of BRSK1 & 2, which we detected by MS, is one such regulatory modification. It is noteworthy, that intramolecular disulfide bonds within the T-loop of AKT and MELK have already been shown to induce an inactive state in the kinase, and we posit a similar mechanism for BRSKs.

      While we recognize the potential importance of dimerization in this context, our current data from in vitro and cell-based assays do not provide substantial evidence to assert dimerization as a primary regulatory mechanism. Hence, we maintained a more conservative stance in our manuscript, discussing dimerization in later sections where it naturally followed from the initial findings. That being said, we acknowledge the potential significance of dimerization in the regulation of the BRSK T-loop cysteine. We believe this aspect merits further investigation and could indeed be the focus of a follow-up study.

      (2) Relatedly, the effect of Cys mutation on the dimerization properties of preparations of recombinant protein is not very clear as it stands. Some SEC traces would be helpful; these could be included in the supplement.

      In order to determine whether our recombinant BRSK proteins (and T-loop mutants) existed as monomers or dimers, we performed SDS-PAGE under reducing and non-reducing conditions (Fig 7). This unambiguously revealed that a monomer was the prominent species, with little evidence of dimers under these experimental conditions (even in the presence of oxidizing agents). Although we cannot discount a regulatory role for BRSK dimers in other physiological contexts, we could not produce sufficient evidence to suggest that multimerization played a substantial role in modifying BRSK kinase activity in our assays. We note that our in vitro analysis was performed using truncated forms of the protein, and as such it is entirely possible that regions of the protein that flank the kinase domain may serve additional regulatory functions that may include higher order BRSK conformations. In this regard, although we have not included SEC traces of our recombinant proteins, we have included analytical SEC-MALS of the truncated proteins (Supplementary Figure 6) which we believe to be more informative. We have also now included additional SEC-MALS data for BRSK2 C176A and C183A (Supplementary Figure 6d and e), which supports our findings in Fig 7, demonstrating the presence of limited dimer species under non-reducing conditions.

      (3) Is there any knowledge of Cys mutants in disease for BRSK1/2?

      We have conducted an extensive search across several databases: COSMIC (Catalogue of Somatic Mutations in Cancer), ProKinO (Protein Kinase Ontology), and TCGA (The Cancer Genome Atlas). These databases are well-regarded for their comprehensive and detailed records of mutations related to cancer and protein kinases. Our analysis using the COSMIC and TCGA databases focused on identifying any reported instances of Cys mutations in BRSK1/2 that are implicated in cancer. Additionally, we utilized the ProKinO database to explore the broader landscape of protein kinase mutations, including any potential disease associations of Cys mutations in BRSK1/2. However, we found no evidence to indicate the presence of Cys mutations in BRSK1/2 that are associated with cancer or disease. This lack of association in the current literature and database records suggests that, as of our latest search, Cys mutations in BRSK1/2 have not been reported as significant contributors to pathogenesis.

      (4) In bar charts, I'd recommend plotting data points. Plus, it is crucial to report in the legend what error measure is shown, the number of replicates, and the statistical method used in any tests.

      We have added the data points to the bar charts and included statistical methods in figure legends.

      (5) In Figure 5b, the GAPDH loading control doesn't look quite right.

      The blot has been repeated and updated.

      (6) In Figure 7 there is no indication of what mode of detection was used for these gels.

      We have updated the figure legend to confirm that the detection method was western blot.

      (7) Recombinant proteins - more detail should be included on how they were prepared. Was there a reducing agent present during purification? Where did they elute off SEC... consistent with a monomer of higher order species?

      We have added ‘produced in the absence of reducing agents unless stated otherwise’ in the methods section to improve clarity. Although we have not added additional sentences to describe the elution profile of the BRSK proteins by SEC during purification, we believe that the inclusion of analytical SEC-MALS data is sufficient evidence that the proteins are largely monomeric under non-reducing conditions.

      Reviewer #2 (Public Review):

      Summary:

      In this study by Bendzunas et al, the authors show that the formation of intra-molecular disulfide bonds involving a pair of Cys residues near the catalytic HRD motif and a highly conserved T-Loop Cys with a BRSK-specific Cys at an unusual CPE motif at the end of the activation segment function as repressive regulatory mechanisms in BSK1 and 2. They observed that mutation of the CPE-Cys only, contrary to the double mutation of the pair, increases catalytic activity in vitro and drives phosphorylation of the BRSK substrate Tau in cells. Molecular modeling and molecular dynamics simulations indicate that oxidation of the CPE-Cys destabilizes a conserved salt bridge network critical for allosteric activation. The occurrence of spatially proximal Cys amino acids in diverse Ser/Thr protein kinase families suggests that disulfide-mediated control of catalytic activity may be a prevalent mechanism for regulation within the broader AMPK family. Understanding the molecular mechanisms underlying kinase regulation by redox-active Cys residues is fundamental as it appears to be widespread in signaling proteins and provides new opportunities to develop specific covalent compounds for the targeted modulation of protein kinases.

      The authors demonstrate that intramolecular cysteine disulfide bonding between conserved cysteines can function as a repressing mechanism as indicated by the effect of DTT and the consequent increase in activity by BSK-1 and -2 (WT). The cause-effect relationship of why mutation of the CPE-Cys only increases catalytic activity in vitro and drives phosphorylation of the BRSK substrate Tau in cells is not clear to me. The explanation given by the authors based on molecular modeling and molecular dynamics simulations is that oxidation of the CPE-Cys (that will favor disulfide bonding) destabilizes a conserved salt bridge network critical for allosteric activation. However, no functional evidence of the impact of the salt-bridge network is provided. If you mutated the two main Cys-pairs (aE-CHRD and A-loop T+2-CPE) you lose the effect of DTT, as the disulfide pairs cannot be formed, hence no repression mechanisms take place, however when looking at individual residues I do not understand why mutating the CPE only results in the opposite effect unless it is independent of its connection with the T+2residue on the A-loop.

      Strengths:

      This is an important and interesting study providing new knowledge in the protein kinase field with important therapeutic implications for the rationale design and development of next-generation inhibitors.

      Weaknesses:

      There are several issues with the figures that this reviewer considers should be addressed.

      Reviewer #1 (Recommendations for The Authors):

      Major points

      Page 26 - the discussion could be more concise. There's an element of recapping the results, which should be avoided.

      Regarding the conciseness of the discussion section, we have thoroughly revised it to ensure a more succinct presentation, deliberately avoiding the recapitulation of results. The revised discussion now focuses on interpreting the findings and their implications, steering clear of redundancy with the results section.

      Figure 1b seems to be mislabeled/annotated. I recommend checking whether the figure legends match more broadly. Figure 1 appears to be incorrectly cited throughout the results.

      Thank you for pointing out the discrepancies in the labeling and citation of Figure 1b. We have carefully reviewed and corrected these issues to ensure that all figure labels, legends, and citations accurately reflect the corresponding data and illustrations. We appreciate your attention to detail and the opportunity to improve the clarity and accuracy of our presentation.

      Figure 6 - please include a color-coding key in the figure. Further support for these simulations could be provided by supplementary movies or plots of the interaction. Figure 4 colour palette should be adjusted for the spheres in the Richardson diagrams to have greater distinction.

      As suggested, we have amended the colour palette in Figure 4 to improve conformity throughout the figure.

      Minor points

      Figure 2 - it'd be helpful to know what the percentage coverage of peptides is.

      We have updated the figure legend to include peptide coverage for both proteins

      Some typos - Supp 2 legend "Domians".

      Fixed

      Figure 6 legend - analyzed by needs a space;

      Fixed

      Fig 8 legend schematic misspelled.

      Fixed

      Broadly, if you Google T-loop you get a pot pourri of enzyme answers. Why not just use Activation loop?

      The choice of "T-loop" over "Activation loop" in our manuscript was made to maintain consistency with other literature in the field, and in particular our previous paper “Aurora A regulation by reversible cysteine oxidation reveals evolutionarily conserved redox control of Ser/Thr protein kinase activity” where we refer to the activation loop cysteine as T-loop + 2. We acknowledge the varied enzyme contexts in which "T-loop" is used and agree on the importance of clarity. To address this, we made an explicit note in the manuscript that the "T-loop" is also referred to as the "Activation loop", ensuring readers are aware of the interchangeable use of these terms. Additionally, this nomenclature facilitates a more straightforward designation of cysteine residues within the loop (T+2 Cysteine). We believe this approach balances adherence to established conventions with the need for clarity and precision in our descriptions.

      Methods - what is LR cloning. Requires some definition. Some manufacturer detail is missing in methods, and referring to prior work is not sufficient to empower readers to replicate.

      We agree, and have added the following to the methods section:

      “BRSK1 and 2 were sub-cloned into pDest vectors (to encode the expression of N-terminal Flag or HA tagged proteins) using the Gateway LR Clonase II system (Invitrogen) according to the manufacturer’s instructions. pENtR BRSK1/2 clones were obtained in the form of Gateway-compatible donor vectors from Dr Ben Major (Washington University in St. Louis). The Gateway LR Clonase II enzyme mix mediates recombination between the attL sites on the Entry clone and the attR sites on the destination vector. All cloned BRSK1/2 genes were fully sequenced prior to use.”

      Page 7 - optimal settings should be reported. How were pTau signals quantified and normalised?

      We have added the following to the methods section:

      “Two-color Western blot detection method employing infrared fluorescence was used to measure the ratio of Tau phospho serine 262 to total Tau. Total GFP Tau was detected using a mouse anti GFP antibody and visualized at 680 nm using goat anti mouse IRdye 680 while phospho-tau was detected using a Tau phospho serine 262 specific antibody and visualized at 800 nm using goat anti rabbit IRdye 800. Imaging was performed using a Licor Odessey Clx with scan control settings set to 169 μm, medium quality, and 0.0 mm distance. Quantification was performed using Licor image studio on the raw image files. Total Tau to phospho Tau ratio was determined by measuring the ratio of the fluorescence intensities measured at 800 nm (pTau) to those at 680 nm (total tau).”

      In the Figure 6g-j legend, the salt bridge is incorrectly annotated as E185-R248 rather than 258.

      Fixed

      Lines 393-395 provides a repeat statement on BRSKs phosphorylating Tau (from 388-389).

      We have removed the repetition and reworded the opening lines of the results section to improve the overall flow of the manuscript.

      Supp. Figure 1 is difficult to view - would it be possible to increase the size of the phylogenetic analysis?

      We thank the reviewer for this observation. We have rotated (90°) and expanded the figure so that it can be more clearly viewed

      Supp. Figure 2 - BRSK1/2 incorrectly spelled.

      Fixed

      Please check the alignment of labels in Supp. Figure 3e.

      Fixed

      Reviewer #2 (Recommendations For The Authors):

      (1) In Figure 1, current panel b is not mentioned/described in the figure legend and as a consequence, the rest of the panels in the legends do not fit the content of the figure.

      Reviewer 1 also noted this error, and we have amended the manuscript accordingly.

      What is the rationale for using the HEK293T cells as the main experimental/cellular system? Are there cell lines that express both proteins endogenously so that the authors can recapitulate the results obtained from ectopic overexpression?

      The selection of HEK-293T cells was driven by their well-established utility in overexpression studies, which make them ideal for the investigation of protein interactions and redox regulation. This cell line's robust transfection efficiency and well-characterized biology provide a reliable platform for dissecting the molecular mechanisms underlying the redox regulation of proteins. Furthermore, the use of HEK-293T cells aligns with the broader scientific practice, serving as a common ground for comparability with existing literature in the field of BRSK1/2 signaling, protein regulation and interaction studies.

      The application of HEK-293T cells as a model system in our study serves as a foundational step towards eventually elucidating the functions of BRSK1/2 in neuronal cells, where these kinases are predominantly expressed and play critical roles. Given the fact that BRSKs are classed as ‘understudied’ kinases, the choice of a HEK-293T co-overexpression system allowed us to analyze the direct effects of BRSK kinase activity (using phosphorylation of Tau as a readout) in a cellular context and in more controlled manner. This approach not only aids in the establishment of a baseline understanding of the redox regulation of BRSK1/2, but also sets the stage for subsequent investigations in more physiologically relevant neuronal models

      In current panel d, could the authors recapitulate the same experimental conditions as in current panel c?

      Figure 1 panel c shows that both BRSK1 and 2 are reversibly inhibited by oxidizing agents such as H2O2, whilst panels d and e show the concentration dependent activation and inhibition of the BRSKs with increasing concentrations of DTT and H2O2 respectively. The experimental conditions were identical, other than changing amounts of reducing and oxidizing agents, and used the same peptide coupled assays. Data for all experiments were originally collected in ‘real time’ as depicted in Fig 1c (increase in substrate phosphorylation over time). However, to aid interpretation of the data, we elected to present the latter two panels as dose response curves by calculating the change in the rate of enzyme activity (shown as pmol phosphate incorporated into the peptide substrate per min) for each condition. To aid the reader, we now include an additional supplementary figure (new supplementary figure 2) depicting BRSK1 and 2 dependent phosphorylation of the peptide substrate in the presence of different concentrations of DTT and H2O2 in a real time (kinetic) assay. The new data shown is a subset of the unprocessed data that was used to calculate the rates of BRSK activity in Fig 1d & e.

      Why did the authors use full-length constructs in these experiments and did not in e.g. Figure 2 where they used KD constructs instead?

      In the initial experiments, illustrated in Figure 1, we employed full-length protein constructs to establish a proof of concept, demonstrating the overall behavior and interactions of the proteins in their full-length form. This confirmed that BRSK1 & 2, which both contain a conserved T + 2 Cys residue that is frequently prognostic for redox sensitivity in related kinases, displayed a near-obligate requirement for reducing agents to promote kinase activity.  

      Subsequently, in Figure 2, our focus shifted towards delineating the specific regions within the proteins that are critical for redox regulation. By using constructs that encompass only the kinase domain, we aimed to demonstrate that the redox-sensitive regulation of these proteins is predominantly mediated by specific cysteine residues located within the kinase domain itself. This strategic use of the kinase domain of the protein allowed for a more targeted investigation. Furthermore, in our hands these truncated forms of the protein were more stable at higher concentrations, enabling more detailed characterization of the proteins by DSF and SEC-MALS. We predict that the flanking disordered regions of the full-length protein (as predicted by AlphaFold) contribute to this effect.

      (2) In Figure 2, Did the authors try to do LC/MS-MS in the same experimental conditions as in Figure 1 (e.g. buffer minus/plus DTT, H2O2, H2O2 + DTT)?

      We would like to clarify that the mass spectrometry experiments were conducted exclusively on proteins purified under native (non-reducing) conditions. We did not extend the LC/MS-MS analyses to include proteins treated with various buffer conditions such as minus/plus DTT, H2O2, or H2O2 + DTT as used in the experiments depicted in Figure 1. Given that we could readily detect disulfides in the absence of oxidizing agents, we did not see the benefit of additional treatment conditions as peroxide treatment of protein samples can frequently complicate interpretation of MS data. However, it should be noted that prior to MS analysis, tryptic peptides were subjected to a 50:50 split, with one half alkylated in the presence of DTT (as described in the methods section) to eliminate disulfides and other transiently oxidized Cys forms. Comparative analysis between reduced and non-reduced tryptic peptides improved our confidence when assigning disulfide bonds (which were eliminated in identical peptides in the presence of DTT).

      On panel b, why did the authors show alphafold predictions and not empiric structural information (e.g. X-ray, EM,..)?

      The AlphaFold models were primarily utilized to map the general locations of redox-sensitive cysteine pairs within the proteins of interest. Although we have access to the crystal structure of mouse BRSK2, they do not fully capture the active conformation seen in the Alphafold model of the human version. The use of AlphaFold models for human proteins in this study aids in consistently tracking residue numbering across the manuscript, offering a useful framework for understanding the spatial arrangement of these critical cysteine pairs in their potentially active-like states. This approach facilitates our analysis and discussion by providing a reference for the structural context of these residues in the human proteins.

      What was the rationale for using the KD construct and not the FL as in Figure 1?

      The rationale to use the kinase domain was primarily based on the significantly lower confidence in the structural predictions for regions outside the kinase domain (KD). Our experimental focus was to investigate the role of conserved cysteine residues within the kinase domain, which are critical for the protein's function and regulation. This targeted approach allowed us to concentrate our analyses on the most functionally relevant and structurally defined portion of the protein, thereby enhancing the precision and relevance of our findings. As is frequently the case, truncated forms of the protein, consisting only of the kinase domain, are much more stable than their full length counterparts and are therefore more amenable to in vitro biochemical analysis. In our hands this was true for both BRSK1 and 2, and as such much of the data collected here was generated using kinase-domain (KD) constructs. Simulations using the KD structures are therefore much more representative of our original experimental setup.

      The BSK1 KD construct appears to be rather inactive and not responsive to DTT treatment. Could the authors comment on the differences observed with the FL construct of Figure 1

      It is important to note that BRSK1, in general, exhibits lower intrinsic activity compared to BRSK2. This reduced activity could be attributed to a range of factors, including the need for activation by upstream kinases such as LKB1, as well as potential post-translational modifications (PTMs) that may be absent in the bacterially expressed KD construct. The full-length forms of the protein were purified from Sf21 cells, and as such may have additional modifications that are lacking in the bacterially derived KD counterparts. We also cannot discount additional regulatory roles of the regions that flank the KD, and these may contribute in part to the modest discrepancy observed between constructs.  Despite these differences, it is crucial to emphasize that both the KD and FL constructs of BRSK1 are regulated by DTT, indicating a conserved redox-dependent activation for both of the related BRSK proteins.  

      (3) In Figure 4, on panel A wouldn´t the authors expect that mutating on the pairs e.g. C198A in BSK1 would have the same effect as mutating the C191 from the T+2 site? Did they try mutating individual sites of the aE/CHRD pair? The same will apply to BSK2

      We appreciate the insightful comment. It's important to clarify that the redox regulation of these proteins is influenced not solely by the formation of disulfide bonds but also by the oxidation state of individual cysteine residues, particularly the T+2 Cys. This nuanced mechanism of regulation allows for a diverse range of functional outcomes based on the specific cysteine involved and its state of oxidation. This aspect forms a key finding of our paper, highlighting the complexity of redox regulation beyond mere disulfide bond formation. For example, AURA kinase activity is regulated by oxidation of a single T+2 Cys (Cys290, equivalent to Cys191 and Cys176 of BRSK1 and 2 respectively), but this regulation can be supplemented through artificial incorporation of a secondary Cys at the DFG+2 position (Byrne et al., 2020). This targeted genetic modification or AURA mirrors equivalent regulatory disulfide-forming Cys pairs that naturally occur in kinases such as AKT and MELK, and which provide an extra layer of regulatory fine tuning (and a possible protective role to prevent deleterious over oxidation) to the T+2 Cys. We surmise that the CPE Cys is also an accessory regulatory element to the T+2 Cys in BRSK1 +2, which is the dominant driver of BRSK redox sensitivity (as judged by the fact that CPE Cys mutants are still potently regulated by redox [Fig 4]), by locking it in an inactive disulfide configuration.

      In our preliminary analysis of BRSK1, we observed that mutations of individual sites within the aE/CHRD pair was similarly detrimental to kinase activity as a tandem mutation (see reviewer figure 1). As discussed in the manuscript, we think that these Cys may serve important structural regulatory functions and opted to focus on co-mutations of the aE/CHRD pair for the remainder of our investigation.

      Author response image 1.

      In vitro kinase assays showing rates of in vitro peptide phosphorylation by WT and Cys-to-Ala (aE/CHRD residues) variants of BRSK1 after activation by LKB1.

      In panels C and D, the same experimental conditions should have been measured as in A and B.

      Panels A and B were designed to demonstrate the enzymatic activity and the response to DTT treatment to establish the baseline redox regulation of the kinase and a panel of Cys-to-Ala mutant variants. In contrast, panels C and D were specifically focused on rescue experiments with mutants that showed a significant effect under the conditions tested in A and B. These panels were intended to further explore the role of redox regulation in modulating the activity of these mutants, particularly those that retained some level of activity or exhibited a notable response to redox changes.

      The rationale for this experimental design was to prioritize the investigation of mutants, such as those at the T+2 and CPE cysteine sites, which provided the most insight into the redox-dependent modulation of kinase activity. Other mutants, which resulted in inactivation, were deprioritized in this context as they offered limited additional information regarding the redox regulation mechanism. This focused approach allowed us to delve deeper into understanding how specific cysteine residues contribute to the redox-sensitive control of kinase function, aligning with the overall objective of elucidating the nuanced roles of redox regulation in kinase activity.

      (4) In figure 5: Why did the authors use reduced Glutathione instead of DTT? The authors should have recapitulated the same experimental conditions as in Figure 4 and not focused only on the T+2 or the CPE single mutants but using the double and the aE/CHRD mutants as well, as internal controls and validation of the enzymatic assays using the modified peptide

      Regarding the use of reduced glutathione (GSH) instead of DTT in Figure 5, we chose GSH for its well characterized biological relevance as an antioxidant in cellular responses to oxidative stress. Furthermore, while DTT has been widely used in experimental setups, it is also potentially cytotoxic at high concentrations.

      Addressing the point on experimental consistency with Figure 4, we appreciate the suggestion and indeed had already conducted such experiments (Previously Supp Fig 3, now changed to current Supp Fig 4). These experiments include analyses of BRSK mutant activity in a HEK-293T model. However, we chose not to focus on inactivating mutants (such as the aE/CHRD mutants which had depleted expression levels possibly as a consequence of compromised structural integrity) or pursue the generation of double mutant CMV plasmids, as these were deemed unlikely to add significant insights into the core narrative of our study. Our focus remained on the mutants that yielded the most informative results regarding the redox regulation mechanisms in the in vitro setting, ensuring a clear and impactful presentation of our findings.

      A time course evaluation of the reducing or oxidizing reagents should have been performed. Would we expect that in WT samples, and in the presence of GSH, and also in the case of the CPE mutant, an increment in the levels of Tau phosphorylation as a readout of BSK1-2 activity?

      We acknowledge the importance of such analyses in understanding the dynamic nature of redox regulation on kinase activity and have included a time course (Supp Fig 2 e-g). These results confirm a depletion of Tau phosphorylation over time in response to peroxide generated by the enzyme glucose oxidase.

      (5) In Figure 6, did the authors look at the functional impact of the residues with which interact the T+2 and the CPE motifs e.g. T174 and the E185-R258 tether?

      Our primary focus was on the salt bridges, as this is a key regulatory structural feature that is conserved across many kinases. Regarding the additional interactions mentioned, we have thoroughly evaluated their roles and dynamics through molecular dynamics (MD) simulations but did not find any results of significant relevance to warrant inclusion.

      (6) In Figure 7: Did the author look at the oligomerization state of the BSK1-2 multimers under non-reducing conditions? Were they also observed in the case of the FL constructs? What was the stoichiometry?

      Our current work indicates that the kinase domain of BRSK1-2 primarily exists in a monomeric state, with some evidence of dimerization or multimer formation under specific conditions. Our SEC-MALS (Supp Fig 6) and SDS-PAGE analysis (Figure 7) clearly demonstrates that monomers are overwhelmingly the dominant species under non-reducing conditions (>90 %). We also conclude that these limited oligomeric species can be removed by inclusion of reducing agents such as DTT (Figure 7), which may suggest a role for a Cys residue(s). Notably, removal of the T+2 Cys was insufficient to prevent multimerization.

      We were unable to obtain reliable SEC-MALS data for the full-length forms of the protein, likely due to the presence of disordered regions that flank the kinase domain which results in a highly heterodispersed and unstable preparation (at the concentrations required for SEC-MALS). Although we are therefore unable to comment on the stoichiometry of FL BRSK dimers, we can detect BRSK1 and 2 hetero- and homo-complexes in HEK-293T cells by IP, which supports the existence of limited BRSK1 & 2 dimers (Supp Fig 6a). However, we were unable to detect intermolecular disulfide bonds by MS, although this does not necessarily preclude their existence. The physiological role of BRSK multimerization (if any) and establishing specifically which Cys residues drive this phenomenon is of significant interest to our future investigations.

    1. Author response:

      The following is the authors’ response to the original reviews.

      Reviewer #1:

      I will summarize my comments and suggestions below.

      (1) Abstract:

      "Non-catalytic (pseudo)kinase signaling mechanisms have been described in metazoans, but information is scarce for plants." To the best of my understanding EFR is an active protein kinase in vitro and in vivo and cannot be considered a pseudokinase. Consider rephrasing.

      We rephrased to: “Non-catalytic signaling mechanisms of protein kinase domains have been described in metazoans, but information is scarce for plants.”

      (2) Page 4: It should be noted, that while membrane associated Rap-RiD systems have been used in planta to activate receptor kinase intracellular domains by promoting interaction with a co-receptor kinase domain, this system does not resemble the actual activation mechanism in the plasma membrane. This would be worth discussing when introducing the system. For example, the first substrates of the RK signaling complex may also be membrane associated and not freely diffuse in solution, which may be important for enzyme-substrate interaction.

      We inserted on page 4: “The RiD system was previously applied in planta, maintaining membrane-association by N-terminal myristoylation (Kim et al., 2021). For the in vitro experiments, the myristoylation sites were excluded to facilitate the production of recombinant protein.”

      (3) Page 4 and Fig 1: The catalytic Asp in BRI1 is D1027 and not D1009 (https://pubmed.ncbi.nlm.nih.gov/21289069/). Please check and prepare the correct mutant protein if needed.

      We clarified this in the text by stating that we mutated the HRD-aspartate to asparagine in all our catalytic-dead mutants: “Kinase-dead variants with the catalytic residue (HRD-aspartate) replaced by asparagine (EFRD849N and BRI1D1009N), had distinct effects […]”. D1027 in BRI1 is the DFG-Asp, which was not mutated in our study.

      (4) Page 4 and Fig 1: Is BIK1 a known component of the BR signaling pathway and a direct BRI1 substrate? Or in other words how specific is the trans-phosphorylation assay? In my opinion, a more suitable substrate for BRI1/BAK1 would be BSK1 or BSK3 (for example https://pubmed.ncbi.nlm.nih.gov/30615605/).

      Kinase-dead BIK1 is a reported substrate of BRI1. We clarified this in the results section by inserting: “BIK1 was chosen as it is reported substrate of both, EFR/BAK1 and BRI1/BAK1 complexes (Lin et al., 2013).”

      (5) Fig. 1B Why is BIK1 D202N partially phosphorylated in the absence of Rap? I would suggest to add control lanes showing BRI1, EFR, FLS2, BAK1 and BIK1 in isolation. Given that a nice in vitro activation system with purified components is available, why not compare the different enzyme kinetics rather than band intensities at only 1 enzyme : substrate ratio?

      BIK1 D202N is partially phosphorylated due to the presence of active BAK1 that is capable of transphosphorylating BIK1 D202N as it has been reported in a previous study: (DOI: 10.1038/s41586-018-0471-x).

      (6) Page 4 and Fig 1: Is the kinase dead variant of EFR indeed kinase dead? I could still see a decent autorad signal for this mutant when expressed in E. coli (Fig 1 A in Bender et al., 2021; https://pubmed.ncbi.nlm.nih.gov/34531323/)? If this mutant is not completely inactive, could this change the interpretation of the experiments performed with the mutant protein in vitro and in planta in the current manuscript? In my opinion, it could be possible that a partially active EFR mutant can be further activated by BAK1, and in turn can phosphorylate BIK1 D202N. The differences in autorad signal for BRI1D1009?N and EFRD849N is very small, and the entire mechanism hinges on this difference.

      We would like to emphasize that the mechanism hinges on the difference between non-dimerized and dimerized kinase domains in the in vitro kinase assay. BRI1 D1009N fails to enhance BIK1 D202N trans-phosphorylation compared to the non-dimerized sample, while EFR D849N is still capable of enhancing BIK1 transphosphorylation upon dimerization as indicated by quantification of autorads (Figure 1B/C). We have also addressed this point in a section on the limitations of our study.

      (7) Fig 1B. "Our findings therefore support the hypothesis that EFR increases BIK1 phosphorylation by allosterically activating the BAK1 kinase domain." To the best of my understanding presence of wild-type EFR in the EFR-BAK1 signaling complex leads to much better phosphorylation of BIK1D202N when compared to the EFRD849N mutant. How does that support the allosteric mechanism? By assuming that the D849N mutant is in an inactive conformation and fully catalytically inactive (see above)? Again, I think the data could also be interpreted in such a way that the small difference in autorad signal for BIK1 between BRI1 inactive (but see above) and ERF inactive are due to EFR not being completely kinase dead (see above), rather than EFR being an allosteric regulator. To clarify this point I would suggest to a) perform quantitative auto- and trans-(generic substrate) phosphorylation assays with wt and D849N EFR to derive enzyme kinetic parameters, to (2) include the EFRD849 mutant in the HDX analysis and (3) to generate transgenic lines for EFRD489N/F761H/Y836F // EFRD489N/F761H/SSAA and compare them to the existing lines in Fig. 3.

      Mutations of proteins, especially those that require conformational plasticity for their function can have pleiotropic effects as the mutation may affect the conformational plasticity and consequently catalytic and non-catalytic functions that depend on the conformational plasticity. In such cases, it is difficult to fully untangle catalytic and non-catalytic functions. Coming back to EFR D849N, the D849N mutation may also impact the non-catalytic function by altering the conformational plasticity, explaining the difference observed in EFR vs EFR D849N. As you rightly suggested, HDX would be a way to address this but would still not clarify whether catalytic activity contributes to activation. We instead attempted to produce analog sensitive EFR variants for in vivo characterization of EFR-targeted catalytic inhibition. Unfortunately, we failed in producing an analog-sensitive variant for which we could show ATP-analog binding. To address your concern, we inserted a section on limitations of the study.

      (8) Fig. 2B,C, supplement 3 C,D. Has it been assessed if the different EFR versions were expressed to similar protein levels and still localized to the PM?

      Localization of the mutant receptors has not been explicitly evaluated by confocal microscopy. However, the selected mutation EFRF761H is shown to accumulate in stable Arabidopsis lines (Figure 3 – Supplement 1C) and BAK1 could be coIPed by all EFR variants upon elf18-treatment (Figure 3 B), indicating plasma membrane localization.

      (9) How the active-like conformation of EFR is in turn activating BAK1 is poorly characterized, but appears to be the main step in the activation of the receptor complex. Extending the HDX analyses to resting and Rap-activated receptor complexes could be a first step to address this question. I tried to come up with an experimental plan to test if indeed the kinase activity of BAK1 and not of EFR is essential for signal propagation, but this is a complex issue. You would need to be able to mimic an activated form of EFR (which you can), to make sure its inactive (possibly, see above) and likewise to engineer a catalytically inactive form of BAK1 in an active-like state (difficult). As such a decisive experiment is difficult to implement, I would suggest to discuss different possible interpretations of the existing data and alternative scenarios in the discussion section of the manuscript.

      We addressed your concern whether BAK1 kinase activity is essential for signaling propagation by pairing EFRF761H and BAK1D416N (Figure 4 Supplement 2 C) which fails to induce signaling. In this case, EFRF761H is in its activated conformation but cannot activate downstream signaling. We also attempted to address your concern by an in vitro kinase assay by pairing EFR and BAK1D416N and using a range of concentrations of the substrate BIK1D202N. We observed that catalytic activity of BAK1 but not EFR was essential for BIK1 phosphorylation. However, this experiment does not address whether activated EFR can efficiently propagate signaling in the absence of BAK1 catalytic activity. In the limitations of the study section, we now discuss the catalytic importance of EFR for signaling activation.

      Author response image 1.

      BIK1 trans-phosphorylation depends on BAK1 catalytic activity. Increasing concentrations of BIK1 D202N were used as substrate for Rap-induced dimers of EFR-BAK1, EFR D849N-BAK1, and EFR-BAK1 D416N respectively. BIK1 trans-phosphorylation depended on the catalytic activity of BAK1. Proteins were purified from E. coli λPP cells. Three experiments yielded similar results of which a representative is shown here.

      Reviewer #2:

      All of my suggestions are minor.

      Figure 1B, I think it would be more useful to readers to explain the amino acid in the D-N change, rather than just call it D-to-N? Also, please label the bands on the stained gel; the shift on FKBP-BRI1 and FKBP-EFR are noticeable on the Coomassie stain.

      We implemented your suggestions.

      Figure 1-Supplement 1. There is still a signal in pS612 BAK1 (it states 'also failed to induce BAK1 S612 phosphorylation' in the text, which is not quite correct). Also, could mention the gel shift seen in BAK1, which appears absent in Y836F.

      We corrected the text which now states: “To test whether the requirement for Y836 phosphorylation is similar, we immunoprecipitated EFR-GFP and EFRY836F-GFP from mock- or elf18-treated seedlings and probed co-immunoprecipitated BAK1 for S612 phosphorylation. EFRY836F also obstructed the induction of BAK1 S612 phosphorylation (Figure 1 – Supplement 1), indicating that EFRY836F and EFRSSAA impair receptor complex activation.” The gel shift of BAK1 you pointed out was not observed in replications and thus we prefer not to comment on it.

      Figure 2 and 3 are full of a, b, c,d's, which I don't understand. Sorry

      We used uppercase letters to indicate subpanels and lowercase letters to indicate the results of the statistical testing. In the figure caption, we have clarified that the lowercase letters refer to statistical comparisons.

      Figure 2 A. If each point on the x-axis is one amino acid, I think it would again be useful to name the amino acids that the gold or purple or blue colored lines extend through.

      Each point stands for a peptide which are sorted by position of their starting amino acid from N-terminus to C-terminus. We now added plots of HDX for individual peptides that correspond to the highlighted region in subpanel A.

      Figure Supplement 1 is very small for what it is trying to show, even on the printed page. If this residue were to be phosphorylated, what would happen to the H-bond?

      We suppose that VIa-Tyr phosphorylation would break the H-bond and causes displacement of the aC-b4 loop. Recent studies, published after our submission, highlight the importance of this loop for substrate coordination and ATP binding. Thus, phosphorylation of VIa-Tyr and displacing this loop may render the kinase rather unproductive. We have expanded the discussion to include this point.

      Figure 2B: Tyr 836 is not present in any of the alignments in Figure 2A. This should be rectified, because the text talks about the similarity to Tyr 156 in PKA.

      We have adjusted the alignments such that they now contain the VIa-Tyr residues of EFR and PKA.

      Figure 4D. Is there any particular reason that these Blots are so hard to compare or FKBP and BAK1?

      We assume it is referred to Figure 4 – Supplement 2 D. FKBP-EFR and FRB-BAK1 both are approximately the size of RubisCo, the most abundant protein in plant protein samples and which overlay the FKBP- and FRB-tagged kinase. Thus, it is difficult to detect these proteins.

      Reviewer #3:

      (1) The paper reporting the allosteric activation mechanism of EGFR should be cited.

      Will be included.

      (2)The authors showed that "Rap addition increased BIK1 D202N phosphorylation when the BRI1 or EFR kinase domains were dimerized with BAK1, but no such effect was observed with FLS2". Please explain why FLS2 failed to enhance BIK1 transphosphorylation by Rap treatment?

      Even though BIK1 is a reported downstream signaling component of FLS2/BAK1, it might be not the most relevant downstream signaling component and rather related RLCKs, like PBL1, might be better substrates for dimerized FLS2/BAK1. We haven’t tested this, however. Alternatively, the purified FLS2 kinase domain might be labile and quickly unfolds even though it was kept on ice until the start of the assay, or the N-terminal FKBP-tag may disrupt function. As the reason for our observation is not clear, we have removed FLS2 in vitro dimerization experiments from the manuscript.

      (3) Based solely on the data presented in Figure 1, it can be concluded that EFR's kinase activity is not required to facilitate BIK1 transphosphorylation. Therefore, the title of Figure 1, "EFR Allosterically Activates BAK1," may be inappropriate.

      We have changed the figure title to: “EFR facilitates BIK1 trans-phosphorylation by BAK1 non-catalytically.”

      (4) In Figure 1- Supplement 1, I could not find any bands in anti-GFP and anti-BAK1 pS612 of input. Please redo it.

      Indeed, we could not detect protein in the input samples of this experiment. BAK1 S612 phosphorylation is an activation mark and not necessarily expected to be abundant enough for detection in input samples. EFR-GFP, however, is usually detected in input samples and is reported in Macho et al. 2014 from which manuscript these lines come. Why EFR-GFP is not detected in this set of experiments is unclear but, in our opinion, does not detract from the conclusions drawn since similar amounts of EFR-GFP are pulled-down across all samples.

      (5) For Figure 2A, please mark the structure represented by each color directly in the figure.

      We have made the suggested change.

      (6) Please modify "EFRF761/Y836F and EFRF761H/SSAA restore BIK1 trans-phosphorylation" to "EFRF761H/Y836F and EFRF761H/SSAA restore BIK1 trans-phosphorylation".

      Thank you for spotting this. We changed it.

      (7) The HDX-MS analysis demonstrated that the EFR (Y836F) mutation inhibits the formation of the active-like conformation. Conversely, the EFR (F761H) mutation serves as a potent intragenic suppressor, significantly stabilizing the active-like conformation. Confirming through HDX-MS conformational testing that the EFR (Y836F F761H) double mutation does not hinder the formation of the active-like EFR kinase conformation would greatly strengthen the conclusions of the article.

      Response: We agree that this is beneficial, and we attempted to do it but failed to produce enough protein for HDX-MS analysis. We stated this now in an extra section of the paper (“Limitations of the study”).

    1. Author response:

      The following is the authors’ response to the original reviews.

      Public Reviews: 

      Reviewer #1 (Public review): 

      Summary: 

      This is a comprehensive study that clearly and deeply investigates the function of GATA6 in human early cardiac development. 

      Strengths: 

      This study combines hESC engineering, differentiation, detailed gene expression, genome occupancy, and pathway modulation to elucidate the role of GATA6 in early cardiac differentiation. The work is carefully executed and the results support the conclusions. The use of publicly available data is well integrated throughout the manuscript. The RIME experiments are excellent. 

      Weaknesses: 

      Much has been known about GATA6 in mesendoderm development, and this is acknowledged by the authors. 

      We appreciate the comments and have tried to highlight both the early role of GATA6 in cardiac progenitor biology as well as the haploinsufficiency for relevance to human congenital heart disease, which we believe adds value to other recent published work, among others Sharma et al. eLife 2020.

      Reviewer #2 (Public review): 

      Summary: 

      This manuscript by Bisson et al describes the role of GATA6 to regulate cardiac progenitor cell (CPC) specification and cardiomyocyte (CM) generation using human embryonic stem cells (hESCs). The authors found that GATA6 loss-of-function hESC exhibits early defects in mesendoderm and lateral mesoderm patterning stages. Using RNA-seq and CUT&RUN assays the genes of the Wnt and BMP programs were found to be affected by the loss of GATA6 expression. Modulating Wnt and BMP during early cardiac differentiation can partially rescue CPC and CM defects in GATA6 hetero- and homozygous mutant hESCs. 

      Strengths: 

      The studies performed were rigorous and the rationale for the experimental design was logical. The results obtained were clear and supported the conclusions that the authors made regarding the role of GATA6 on Wnt and BMP pathway gene expression. 

      Weaknesses: 

      Given the wealth of studies that have been performed in this research area previously, the amount of new information provided in this study is relatively modest. Nevertheless, the results and quite clear and should make a strong contribution to the field. 

      Likewise for reviewer 2, we appreciate the comments and have tried to highlight both the early role of GATA6 in cardiac progenitor biology as well as the haploinsufficiency for relevance to human congenital heart disease.

      Reviewer #3 (Public review): 

      In this study, Bison et al. analyzed the role of the GATA6 transcription factor in patterning the early mesoderm and generating cardiomyocytes, using human embryonic stem cell differentiation assays and patient-derived hiPSCs with heart defects associated with mutations in the GATA6 gene. They identified a novel role for GATA6 in regulating genes involved in the WNT and BMP pathways -findings not previously noted in earlier analyses of GATA6 mutant hiPSCs during early cardiac mesoderm specification (Sharma et al., 2020). Modulation of the WNT and BMP pathways may partially rescue early cardiac mesoderm defects in GATA6 mutant hESCs. These results provide significant insights into how GATA6 loss-of-function and heterozygous mutations contribute to heart defects. 

      I have the following comments: 

      (1) Throughout the manuscript, Bison et al. alternate between different protocols to generate cardiomyocytes, which creates some confusion (e.g., Figure 1 vs. Supplemental Figure 2A). The authors should provide a clear justification for using alternative protocols. 

      We agree and clarified this issue in the revision (p. 6). The reviewer is correct that there are two widely used protocols for directed differentiation of PSCs to cardiac fate. One is a cytokine-based protocol (Fig. 1A) and the other uses small molecules to manipulate the WNT pathway (CHIR protocol, Supplemental Fig. 2B). In our study, we used the CHIR protocol only for experiments in Supplemental Figure 2B-E. Since our data implicated BMP and WNT as mediators of the GATA6-dependent program, we did this mainly to confirm that the phenotype we observed with the cytokine-based protocol was not biased by the differentiation protocol. However, we found the CHIR protocol to be overall relatively inefficient for cardiac differentiation using the parental H1 hESCs and the various isogenic lines. The in vitro cardiac differentiation protocols for hPSCs are known to be variable depending on lines and sometimes require extensive optimization for various media components and concentrations, cell seeding densities, and batch variations for crucial reagents. The cytokine-based protocol we optimized worked most efficiently with our hPSC lines to generate cardiomyocytes, therefore we committed to using it for the bulk of experiments in this study.  

      (2) The authors should characterise the mesodermal identity and cardiomyocyte subtypes generated with the activin/BMP-induction protocol thoroughly and clarify whether defects in the expression of BMP and WNTrelated gene affect the formation of specific cardiomyocyte subtypes in a chamber-specific manner. This analysis is important, as Sharma et al. suggested a role for GATA6 in orchestrating outflow tract formation, and Bison et al. similarly identified decreased expression of NRP1, a gene involved in outflow tract septation, in their GATA6 mutant cells. 

      We agree it is important that the mesodermal identities are quite thoroughly characterized.

      For example, Fig. 2 (K+P+, Brachyury, EOMES), Fig. 3G&H (lateral mesoderm, cardiac mesoderm RNAseq & GSEA comparing datasets from Koh et al.). The capacity of the cytokine-based protocol to generate both FHF and SHF derived sub-types has been rigorously evaluated by Keller and colleagues, which we now cite (Yang et al. 2022). Since the null cells do not generate CMs, chamber specific subtypes cannot be evaluated; whether the GATA6 heterozygous mutants are biased is an interesting question. Indeed, the top GO term identified by CUT&RUN analysis for GATA6 at day 2 of

      differentiation is outflow tract morphogenesis, which is consistent with the interpretation by Sharma et al., but implicates this program at a much earlier developmental stage, long before cardiomyocyte differentiation. We think this is one of the most important findings of our study and appreciate the chance to highlight this in the revision (p. 9, 17). When we evaluated chamber-specificity for differentiated cardiomyocytes, we did not find significant differences, as indicated for the reviewer in the panel below (day 20 of differentiation). Since our study focuses on early stages of progenitor specification rather than cardiomyocyte differentiation, we agree that a more rigorous analysis would be of value, and indicated this as a limitation of our current study (p. 18).

      Author response image 1.

      (3) The authors developed an iPSC line derived from a congenital heart disease (CHD) patient with an atrial septal defect and observed that these cells generate cTnnT+ cells less efficiently. However, it remains unclear whether atrial cardiomyocytes (or those localised specifically at the septum) are being generated using the activin/BMP-induction protocol and the patient-derived iPSC line.

      As indicated above, our study is focused on cardiac progenitor specification, and we found similar differences with the patient-derived iPSC-CMs compared to using hESC heterozygous targeted mutants. While we did not note any major differences in expression of cardiomyocyte markers, whether the mutants show any biases toward sub-types of cardiomyocytes is an interesting question to be pursued in subsequent work.

      (4) The authors should also justify the necessity of using the patient-derived line to further analyse GATA6 function. 

      This is a good point, and as suggested we provided the justification (p. 5-6). This is the first patient-derived iPSC line published with a heterozygous GATA6 mutation along with an isogenic mutation-corrected control generated for cardiac directed differentiation. Patients with congenital heart disease (CHD) associated with GATA6 mutations are typically heterozygous (also true for many other CHD variants; presumably homozygous null embryos would not survive). It is important to query if phenotypes found using targeted mutations in hESCs (or iPSCs) model the human disease, since the patient cells (or the hESCs) likely have additional genetic variants that might interact with the GATA6 mutation. The fact that both types of heterozygous cells (patient-derived iPSCs and targeted hESCs) generate similar defects in CM differentiation provides evidence supporting the use of these human cellular models to study the genetic and cellular basis for congenital heart disease. This is particularly important, since other models, such as heterozygous mice, do not show such phenotypes.

      (5) Figure 3 suggests an enrichment of paraxial mesoderm genes in the context of GATA6 loss-of-function, which is intriguing given the well-established role of GATA6 in specifying cardiac versus pharyngeal mesoderm lineages in model organisms. Could the authors expand their analysis beyond GO term enrichment to explore which alternative fates GATA6 mutant cells may acquire? Additionally, how does the potential enrichment of paraxial mesoderm, rather than pharyngeal mesoderm, relate to the initial mesodermal induction from their differentiation protocol? Could the authors also rule out the possibility of increased neuronal cell fates? 

      We need to interpret our in vitro differentiation data cautiously in relation to what has been shown in vivo, since we are unlikely to be reproducing all the complex signaling taking place in the embryo. Yet we do see modest increases in gene expression levels including signatures of paraxial mesoderm and ECM/mesenchymal at days 2 or 3 of differentiation in the GATA6 mutant cells. Therefore, we now include a heatmap showing enriched paraxial mesoderm gene expression in the mutant cells, new Fig. 3I (see page 10).

      A caveat of this result is that the cells are being differentiated toward cardiac fate, so a bias for alternative fates might be suppressed. We modified the protocol to favor paraxial fate by adding CHIR at day 2 (rather than XAV) and performing qPCR assays at day 3. We found this successfully induced paraxial mesoderm gene expression, but equally comparing wildtype, heterozygous, or null cells, so do not feel it warrants highlighting further. 

      Recommendations for the authors:  

      Reviewing Editor (Recommendations for the authors): 

      Incorporation of marker analysis for various stages of iPSC to CM differentiation (mesoderm, cardiac progenitor, CM subtypes) would increase the significance and support for the findings presented. Further data on the link (direct or indirect) between GATA6 and Wnt/BMP signalling would also add to the significance of this study. A number of textual changes/clarifications are also suggested to improve the manuscript. 

      We appreciate the feedback and provide responses for issues raised for markers, direct or indirect interactions, and textual changes/clarifications in the following sections. As indicated above, we did not find obvious alterations in cardiac subtypes, but since our study is focused on early progenitor specification, this is an interesting question that we think should be more rigorously evaluated in subsequent work.  

      Reviewer #1 (Recommendations for the authors)

      Minor details: 

      (1) On p6 "Principal component analysis (PCA) showed that the cells derived from each genotype were well separated from each other (Supplemental Figure 2C)". All genotypes should be in one PCA plot to better evaluate the three genotypes. 

      We prepared the new plot as suggested, presented as new Supplemental Fig. 2C. 

      (2) p10: "Chia et al.22 and found a significantly decreased enrichment in GATA6-/- cells relative to WT at day 2" decreased enrichment of what? Direct target genes? 

      Thank you for catching this. Yes, the text was changed to indicate a “decreased enrichment in GATA6-/- cells relative to WT at day 2 for putative direct GATA6 target genes.” 

      Reviewer #2 (Recommendations for the authors): 

      Overall, this is an interesting study that addresses the early developmental roles of GATA6 on cardiac differentiation. While the identification of Wnt and BMP pathway genes to be involved in GATA6 regulation is not entirely unexpected, the authors do bring forth some useful knowledge that helps to further elucidate the mechanism of pre-cardiac mesoderm regulation. Some suggestions for improvement are included below - 

      Major points: 

      (1) Since the loss of Gata6 in this study is global (either as heterozygous or homozygous, it is likely that the very early requirement of Gata6 (e.g. mesodermal stage of differentiation) is responsible for the cardiac transcriptional phenotype observed and not due to specific role of Gata6 in the cardiac lineage which would need to be addressed using conditional knock out of Gata6 in hPSC model. The authors should be more explicit when discussing the results as disruption of mesodermal differentiation leading to loss of downstream cardiac lineage cells. For example, I would change the title "GATA6 loss-of-function impairs CM differentiation" to "GATA6 loss-of-function impairs mesodermal (or mesodermal lineage) differentiation" and show the changes in cardiac progenitor cells genes (Isl1, Tbx1, Hand1, and BAF50c/Smarcd3) in addition to cardiomyocyte genes but no change in mesodermal (e.g. Brachyury, T, Eomes, Mesp1/2, etc) genes. 

      We agree with the reviewer’s interpretation. The title for the section was changed as suggested. In Fig. 1, we show changes in cardiac progenitor cell genes (Isl1, Hand1, and BAF50c/Smarcd3) while not seeing changes in mesodermal genes in Fig. 2 (e.g. Brachyury, Eomes, Mesp1/2). We note that the defect may be specific to cardiac (or anterior lateral) mesoderm, as the ability to express paraxial mesoderm markers was not impaired.  

      (2) The use of NKX2.5, TBX5, TBX20, and GATA4 as markers for CPC is not ideal. These markers are also expressed in differentiated cardiomycytes. ISL1 or TBX1 for second heart field progenitors and HAND1 or BAF60c/Smarcd3 for first heart field progenitors would be ideal.  

      As suggested, we included additional day 6 qPCR panel (new Fig. 1E) to evaluate the heart field progenitor markers. 

      (3) Much of the findings described in this study have been known in the field including the requirement of Wnt and BMP to induce mesodermal and subsequently cardiomyocyte differentiation. The key new information here is that Gata6 knockout disrupts Wnt and BMP signaling. It would help to further validate experimentally some of the Wnt and BMP genes as either direct or indirect targets of Gata6 using reporter assays. 

      While reporter assays are feasible and do provide relevant outputs, we feel that the use of any one or even several response elements in a reporter assay adds relatively little value compared to comprehensive analysis of bona fide network components. To address the reviewers concern we have included profiling heat maps for WNT and BMP pathway components to more rigorously and specifically evaluate the disruption in the signaling networks caused by loss of GATA6. Proving direct targets of endogenous genes is challenging, but we mapped many binding peaks for GATA6 to putative enhancers of WNT/BMP pathway genes (based on histone marks). We provide a list of these genes (new Fig. 4F) and distinguish these from WNT/BMP pathway genes that were not bound by GATA6 yet are down-regulated in the GATA6 mutant cells and are likely to be indirect targets (p. 12). 

      Minor points: 

      (1) Figures 1 and 2 - in the figure legend the labels w2, w4, m2, m5, m11, and m14 should be explained as the name of the clones of targeted hESC.  

      The legends were edited to provide this information.  

      (2) Supplemental Figure 3A - the resolution of the FACS plot is suboptimal. 

      We apologize and have corrected the plot resolution in the revised manuscript.  

      (3) Supplemental Table 1 - it's intriguing that amongst all the SWI/SNF factors, the one that is known to be cardiac-specific (SMARCD3) did not come up in the GATA6-RIME-enriched proteins. Is this a reflection of the early stage in which GATA6 plays a role in development (e.g. mesendoderm development but not precardiac mesoderm development when SMARCD3 is expressed)? 

      We agree and have noted this feature in the revised manuscript (p. 17). We note that SMARCD3 is expressed in the RNA-seq data as early as day 2. Although speculative, it may be that GATA6 primarily interacts with SWI/SNF complexes prior to the role for SMARCD3 in cardiac specification.

      Reviewer #3 (Recommendations for the authors): 

      (1) Figures 3G and 3H, as well as others, have resolution issues. The gene names are unreadable, and higherresolution images should be provided. 

      We apologize for the resolution issues and these have been fixed in the revised version. 

      (2) In their early manipulation of the WNT and BMP pathways (Figure 6A), it is unclear whether the activin/BMP protocol shown in Figure 1A was used. If this is the case, the authors should compare their results to a wild-type + DOX EV condition for consistency. 

      We clarified in the revision (Fig. 6A) that all the experiments in Fig. 6 use the cytokine protocol. In the revised figure, we included the wild-type + DOX EV condition as suggested. 

      (3) In Figures 6C and 6D, the authors should include an analysis of a wild-type isogenic line under their new CHIR/LB condition for comparison. 

      As suggested, we included the WT isogenic line in the comparison. For Fig. 6C these are shown on a separate graph because the Y-axis values are very different. Note that the CHIR/LB treatments that improve mutant cell differentiation impact the WT cells in the opposite manner.

    1. Author response:

      The following is the authors’ response to the original reviews.

      Reviewer #1:

      Summary:

      The manuscript by Bohra et al. describes the indirect effects of ligand-dependent gene activation on neighboring non-target genes. The authors utilized single-molecule RNA-FISH (targeting both mature and intronic regions), 4C-seq, and enhancer deletions to demonstrate that the non-enhancer-targeted gene TFF3, located in the same TAD as the target gene TFF1, alters its expression when TFF1 expression declines at the end of the estrogen signaling peak. Since the enhancer does not loop with TFF3, the authors conclude that mechanisms other than estrogen receptor or enhancer-driven induction are responsible for TFF3 expression. Moreover, ERα intensity correlations show that both high and low levels of ERα are unfavorable for TFF1 expression. The ERa level correlations are further supported by overexpression of GFP-ERa. The authors conclude that transcriptional machinery used by TFF1 for its acute activation can negatively impact the TFF3 at peak of signaling but once, the condensate dissolves, TFF3 benefits from it for its low expression.

      Strengths:

      The findings are indeed intriguing. The authors have maintained appropriate experimental controls, and their conclusions are well-supported by the data.

      Weaknesses:

      There are some major and minor concerns that related to approach, data presentation and discussion. But I think they can be fixed with more efforts.

      We thank the reviewer for their positive comments on the paper. We have addressed all their specific recommendations below.  

      The deletion of enhancer reveals the absolute reliance of TFF1 on its enhancers for its expression. Authors should elaborate more on this as this is an important finding.

      We thank the reviewer for the comment. We have now added a more detailed discussion on the requirement of enhancer for TFF1 expression in the revised manuscript (line 368-385).  

      In Fig. 1, TFF3 expression is shown to be induced upon E2 signaling through qRT-PCR, while smFISH does not display a similar pattern. The authors attribute this discrepancy to the overall low expression of TFF3. In my opinion, this argument could be further supported by relevant literature, if available. Additionally, does GRO-seq data reveal any changes in TFF3 expression following estrogen stimulation? The GRO-seq track shown in Fig.1 should be adjusted to TFF3 expression to appreciate its expression changes.

      We have now included a browser shot image of TFF3 region showing GRO-Seq signal at E2 time course (Fig. S1C). We observed an increased transcription towards the 3’ end of TFF3 gene body at 3h.  The increased transcription at 3h, corroborates with smFISH data. The relative changes of TFF3 expression measured by qRT-PCR and smFISH for intronic transcripts are somewhat different, we speculate that such biased measurements that are dependent on PCR amplifications could be more for genes that express at low levels and smFISH using intronic probes may be a more sensitive assay to detect such changes.    

      Since the mutually exclusive relationship between TFF1 and TFF3 is based on snap shots in fixed cells, can authors comment on whether the same cell that expresses TFF1 at 1h, expresses TFF3 at 3h? Perhaps, the calculations taking total number of cells that express these genes at 1 and 3h would be useful.

      Like pointed out by the reviewer, since these are fixed cells, we cannot comment on the fate of the same cell at two time points. To further address this limitation, future work could employ cells with endogenous tags for TFF1 and TFF3 and utilize live cell imaging techniques. In a fixed cell assay, as the reviewer suggests, it can be investigated whether a similar fraction shows high TFF3 expression at 3h, as the fraction that shows high TFF1 expression at 1 h. To quantify the fractions as suggested by the reviewer, we plotted the fraction of cells showing high TFF1 and TFF3 expression at 1h and 3h. We identify truly high expressing cells by taking mean and one standard deviation (for single cell level data) at E2-1hr as the threshold for TFF1 (80 and above transcript counts) and mean and one standard deviation (for single cell level data) at E2-3hr as the threshold for TFF3 (36 and above transcript counts). The fraction with high TFF1 expression at 1h  (12.06 ± 2.1) is indeed comparable to that with high TFF3 expression at 3h (12.50 ± 2.0) (Fig. 2C and Author response image 1). We should note that if the transcript counts were normally distributed, a predetermined fraction would be expected to be above these thresholds and comparable fractions can arise just from underlying statistics. But in our experiments, this is unlikely to be the case given the many outliers that affect both the mean and the standard deviation, and the lack of normality and high dispersion in single cell distributions. Of course, despite the fractions being comparable, we cannot be certain if it is the same set of cells that go from high expression of TFF1 to high expression of TFF3, but definitely that is a possibility. We thank the reviewer for pointing out this comparison.

      Author response image 1.

      The graph represents the percent of cells that show high expression for TFF1 and TFF3 at 1h and 3h post E2 signaling. The threshold was collected by pooling in absolute RNA counts from 650 analyzed cells (as in Fig. 2C). The mean and standard deviation over single cell data were calculated. Mean plus one standard deviation was used to set the threshold for identifying high expressing cells. For TFF1, as it maximally expresses at 1h the threshold used was 80. For TFF3, as it maximally expresses at 3h the threshold used was 36. Fraction of cells expressing above 80 and 36 for TFF1 and TFF3 respectively were calculated from three different repeats. Mean of means and standard deviations from the three experiments are plotted here.

      Authors conclude that TFF3 is not directly regulated by enhancer or estrogen receptor. Does ERa bind on TFF3 promoter? 

      The ERa ChIP-seq performed at 1h and 3h of signaling suggests that TFF3 promoter is not bound by ERa as shown in supplementary Fig. 1B and S1B. However, one peak upstream to TFF1 promoter is visible and that is lost at 3h. 

      Minor comments:

      Reviewer’s comment -The figures would benefit from resizing of panels. There is very little space between the panels.

      We have now resized the figures in the revised manuscript.

      The discussion section could include an extrapolation on the relationship between ERα concentration and transcriptional regulation. Given that ERα levels have been shown to play a critical role in breast cancer, exploring how varying concentrations of ERα affect gene expression, including the differential regulation of target and non-target genes, would provide valuable insights into the broader implications of this study.

      This is a very important point that was missing from the manuscript. We have included this in the discussion in the revised manuscript (line 426-430).

      Reviewer #2:

      Summary:

      In this manuscript by Bohra et al., the authors use the well-established estrogen response in MCF7 cells to interrogate the role of genome architecture, enhancers, and estrogen receptor concentration in transcriptional regulation. They propose there is competition between the genes TFF1 and TFF3 which is mediated by transcriptional condensates. This reviewer does not find these claims persuasive as presented. Moreover, the results are not placed in the context of current knowledge.

      Strengths:

      High level of ERalpha expression seems to diminish the transcriptional response. Thus, the results in Fig. 4 have potential insight into ER-mediated transcription. Yet, this observation is not pursued in great depth however, for example with mutagenesis of ERalpha. However, this phenomenon - which falls under the general description of non monotonic dose response - is treated at great depth in the literature (i.e. PMID: 22419778). For example, the result the authors describe in Fig. 4 has been reported and in fact mathematically modeled in PMID 23134774. One possible avenue for improving this paper would be to dig into this result at the single-cell level using deletion mutants of ERalpha or by perturbing co-activators.

      We thank the reviewer for pointing us to the relevant literature on our observation which will enhance the manuscript. We have discussed these findings in relations to ours in the discussion section (Line 400-413). We thank the reviewer for insight on non-monotonic behavior.

      Weaknesses:

      There are concerns with the sm-RNA FISH experiments. It is highly unusual to see so much intronic signal away from the site of transcription (Fig. 2) (PMID: 27932455, 30554876), which suggests to me the authors are carrying out incorrect thresholding or have a substantial amount of labelling background. The Cote paper cited in the manuscript is likewise inconsistent with their findings and is cited in a misleading manner: they see splicing within a very small region away from the site of transcription. 

      We thank the reviewer for this comment, and apologize if they feel we misrepresented the argument from Cote et al. This has now been rectified in the manuscript. However, we do not agree that the intronic signals away from the site of transcription are an artefact. First, the images presented here are just representative 2D projections of 3D Z-stacks; whereas the full 3D stack is used for spot counting using a widely-used algorithm that reports spot counts that are constant over wide range of thresholds (Raj et al., 2008). The veracity of automated counts was first verified initially by comparison to manual counts. Even for the 2D representations the extragenic intronic signals show up at similar thresholds to the transcription sites. 

      The signal is not non-specific arising from background labeling, explained by following reasons:

      • To further support the time-course smFISH data and its interpretation without depending on the dispersed intronic signal, we have analyzed the number of alleles firing/site of transcription at a given time in a cell under the three conditions. We counted the sites of transcription in a given cell and calculated the percentage of cells showing 1,2,3,4 or >4 sites. We see that the percent of cells showing a single site of transcription for TFF1 is very high in uninduced cells and this decreases at 1h. At 1h, the cells showing 2, 3 and 4 sites of transcription increase which again goes down at 3h (Author response image 2A). This agrees with the interpretation made from mean intronic counts away from the site of transcription. Similarly, for TFF3, the number of cells showing 2,3 and 4 sites of transcription increase slightly at 3hr compared to uninduced and 1hr (Author response image 2B).  We can also see that several cells have no alleles firing at a given time as has been quantified in the graphs on right showing total fraction of cells with zero versus non-zero alleles firing (Author response image 2A-B). A non-specific signal would be present in all cells.

      • There is literature on post-transcriptional splicing of RNA beyond our work, which suggests that intronic signal can be found at relatively large distances away from the site of transcription. Waks et al. showed that some fraction of unspliced RNA could be observed up to 6-10 microns away from the site of transcription suggesting that there can be a delay between transcription and (alternative) splicing (Waks et al., 2011). Pannuclear disperse intronic signals can arise as there can be more than one allele firing at a time in different nuclear locations. The spread of intronic transcripts in our images is also limited in cells in which only 1 allele is firing at E2-1 hour (Author response image 2C) or uninduced cells (Author response image 2D). Furthermore, Cote et al. discuss that “Of note, we see that increased transcription level correlates with intron dispersal, suggesting that the percentage of splicing occurring away from the transcription site is regulated by transcription level for at least some introns. This may explain why we observe posttranscriptional splicing of all genes we measured, as all were highly expressed.” This is in line with our interpretation that intron signal dispersal can occur in case of posttranscriptional splicing (Coté et al., 2023). Additionally, other studies have suggested that transcripts in cells do not necessarily undergo co-transcriptional splicing which leads us to conclude that intronic signal can be found farther away from the site of transcription. Coulon et al. showed that splicing can occur after transcript release from the site and suggested that no strict checkpoint exists to ensure intron removal before release which results in splicing and release being kinetically uncoupled from each other (Coulon et al., 2014). Similarly, using live-cell imaging, it was shown that splicing is not always coupled with transcription, and this could depend on the nature and structural features of transcript (such as blockage of polypyrimidine tract which results in delayed recognition) (Vargas et al., 2011). Drexler  et al. showed that as opposed to drosophila transcripts that are shorter, in mammalian cells, splicing of the terminal intron can occur post-transcriptionally (Drexler et al., 2020). Using RNA polymerase II ChIP-Seq time course data from ERα activation in the MCF-7 cells, Honkela et al. showed that large number of genes can show significant delays between the completion of transcription and mRNA production (Honkela et al., 2015). This was attributed to faster transcription of shorter genes which results in splicing  delays suggesting rapid completion of transcription on shorter genes can lead to splicing-associated delays (Honkela et al., 2015). More recently, comparisons of nascent and mature RNA levels suggested a time lapse between transcription and splicing for the genes that are early responders during signaling (Zambrano et al., 2020). The presence of significant numbers of TFF1 nascent RNA in the nucleus in our data corroborates with above observations. 

      • Uniform intensities across many transcripts suggests these are true signal arising from RNA molecules which would not be the case for non-specific, background signal (Author response image 2E).

      • Splicing occurs in the nucleus and intron containing pre-transcripts should be nuclear localized. Thus, intronic signals should remain localized to the nucleus unlike the mature mRNA which translocate to the cytoplasm after processing and thus exonic signals can be found both in the nucleus and the cytoplasm. In keeping with this, we observe no signal in the cytoplasm for the intronic probes and it remains localized within the nucleus as expected and can be seen in Author response image 2F, while exonic signals are observed in both compartments. This suggests to us that the signal is coming from true pre-transcripts. There is no reason for non-specific background labelling to remain restricted to the nucleus.

      • We observe that the mean intronic label counts for both the genes TFF1 and TFF3 increases upon E2-induction compared to uninduced condition (Fig. 2B). Similarly, the mean intronic count for both genes reduce drastically in the TFF1-enhancer deleted cells (Fig. 3C, D). This change in the number of intronic signal specifically on induction and enhancer deletion suggests that the signal is not an artefact and arises from true nascent transcripts that are sensitive to stimulus or enhancer deletion.

      • We expect colocalization of intronic signal with exonic signals in the nucleus, while there can be exonic signals that do not colocalize with intronic, representing more mature mRNA. Indeed, we observe a clear colocalization between the intronic and exonic signals in the nucleus, while exonic signals can occur independent of intronic both in the nucleus and the cytoplasm. This clearly demonstrates that the intronic signals in our experiments are specific and not simply background labelling (Author response image 2G).

      These studies and the arguments above lead us to conclude that the presence of intronic transcripts in the nucleus, away from the site of transcription is not an artefact. We hope the reviewer will agree with us. These analyses have now been included in the manuscript as Supplementary Figure 6 and have been added in the manuscript at line numbers 106-111, 201204,  215-217 and line 231-235. We thank the reviewer for raising this important point.

      Author response image 2.

      Dynamic induction and RNA localization of TFF1 and TFF3 transcription across cell populations using smRNA FISH A. Bar graph depicting the percentage of cells with 1,2,3,4, or greater than 4 sites of transcription for TFF1 (left) is shown. The graph shows the mean of means from different repeats of the experiment, and error bars denote SEM (n>200, N=3). Only the cells with at least one allele firing were counted and cells with no alleles were not included in this. The graph on right shows the number of cells with zero or non-zero number of alleles firing. B. Bar graph depicting the percentage of cells with 1,2,3,4 or greater than 4 sites of transcription for TFF3 (left) is shown. The graph shows the mean of means from different repeats of the experiment, and error bars denote SEM (n>200, N=3). Only the cells with at least one allele firing were counted and cells with no alleles were not included in this. The graph in the middle shows the number of cells with 2,3,4 or greater than 4 sites of transcription for TFF3.The graph on the right shows the number of cells with zero or non-zero number of alleles firing. C. Images from single molecule RNA FISH experiment showing transcripts for InTFF1 in cells induced for 1 hour with E2. The image shows that when a single allele of TFF1 is firing, the transcripts show a more spatially restricted localisation. The scale bar is 5 microns. D. Images from single molecule RNA FISH experiment showing transcripts for InTFF1 in uninduced cells. The image shows that when a single allele of TFF1 is firing and transcription is low, the transcripts show a more spatially restricted localisation. The scale bar is 5 microns. E. Line profile through several transcripts in the nucleus show uniform and similar intensities indicating that these are true signals. F. 60X Representative images from a single molecule RNA FISH experiment showing transcripts for InTFF1 and ExTFF1 (top) and InTFF3 and ExTFF3 (bottom). The image shows that there is no intronic signal in the cytoplasm, while exonic signals can be found both in the nucleus and the cytoplasm. The scale bar is 5 microns. G. 60X Representative images from single molecule RNA FISH experiment showing transcripts for InTFF1 and ExTFF1. The image shows that all intronic signals are colocalized with exonic signals, but all exonic signals are expectedly not colocalized with intronic signals, representing more mature mRNA. The scale bar is 5 microns.

      One substantial way to improve the manuscript is to take a careful look at previous single cell analysis of the estrogen response, which in some cases has been done on the exact same genes (PMID: 29476006, 35081348, 30554876, 31930333). In some of these cases, the authors reach different conclusions than those presented in the present manuscript. Likewise, there have been more than a few studies that have characterized these enhancers (the first one I know of is: PMID 18728018). Also, Oh et al. 2021 (cited in the manuscript) did show an interaction between TFF1e and TFF3, which seems to contradict the conclusion from Fig. 3. In summary, the results of this paper are not in dialogue with the field, which is a major shortcoming. 

      We thank the reviewer for pointing out these important studies. The studies from Prof. Larson group are particularly very insightful (Rodriguez et al., 2019). We have now included this in the discussion (line 106-111 and line 420-424) where we suggest the differences and similarities between our, Larson’s group and also Mancini’s group (Patange et al., 2022; Stossi et al., 2020). 

      The 4C-Seq data from the manuscript Oh et al. 2021 is exactly consistent with our observation from Fig 3 as they also observed little to no interaction between TFF1e and TFF3p in WT cells, only upon TFF1p deletion, did the TFF1e become engaged with the TFF3p. In agreement with this, we also observe little to no interaction between TFF1e and TFF3p in WT cells (Fig.3A). This is also consistent with our competition model for resources between these two genes. Oh et al. shows interaction between TFF1e and TFF3 when the TFF1 promoter is deleted showing that when the primary promoter is not available the enhancer is retargeted to the next available gene (Oh et al., 2021). It does not show that in WT or at any time point of E2 signalling does TFF1e and TFF3 interact.

      In the opinion of this reviewer, there are few - if any - experiments to interrogate the existence of LLPS for diffraction-limited spots such as those associated with transcription. This difficulty is a general problem with the field and not specific to the present manuscript. For example, transient binding will also appear as a dynamic 'spot' in the nucleus, independently of any higher-order interactions. As for Fig. 5, I don't think treating cells with 1,6 hexanediol is any longer considered a credible experiment. For example, there are profound effects on chromatin independent of changes in LLPS (PMID: 33536240).  

      We are cognizant of and appreciate the limitations pointed out by the reviewer. We and others have previously shown that ERa forms condensates on TFF1 chromatin region using ImmunoFISH assay (Saravanan et al., 2020).  The data below shows the relative mean ERα intensity on TFF1 FISH spots and random regions clearly showing an appearance of the condensate at the TFF1 site. Further, the deletion of TFF1e causes the reduction in size of this condensate. Thus, we expect that these ERα condensates are characterized by higher-order interactions and become disrupted on treatment with 1,6-hexanediol. These condensates are the size of below micron as mentioned by the reviewer, but most TF condensates are of the similar sizes. We agree with the reviewer that 1,6- hexanediol treatment is a brute-force experiment with several irreversible changes to the chromatin. Although we have tried to use it at a low concentration for a short period of time and it has been used in several papers (Chen et al., 2023; Gamliel et al., 2022). The opposite pattern of TFF1 vs. TFF3 expression upon 1,6- hexanediol treatment suggests that there is specificity. Further, to perturb condensates, mutants of ERa can be used (N-terminus IDR truncations) however, the transcriptional response of these mutants is also altered due to perturbed recruitment of coactivators that recognize Nterminus of ER, restricting the distinction between ERa functions and condensate formation.

      References:

      Chen, L., Zhang, Z., Han, Q., Maity, B. K., Rodrigues, L., Zboril, E., Adhikari, R., Ko, S.-H., Li, X., Yoshida, S. R., Xue, P., Smith, E., Xu, K., Wang, Q., Huang, T. H.-M., Chong, S., & Liu, Z. (2023). Hormone-induced enhancer assembly requires an optimal level of hormone receptor multivalent interactions. Molecular Cell, 83(19), 3438-3456.e12. https://doi.org/10.1016/j.molcel.2023.08.027

      Coté, A., O’Farrell, A., Dardani, I., Dunagin, M., Coté, C., Wan, Y., Bayatpour, S., Drexler, H. L., Alexander, K. A., Chen, F., Wassie, A. T., Patel, R., Pham, K., Boyden, E. S., Berger, S., Phillips-Cremins, J., Churchman, L. S., & Raj, A. (2023). Post-transcriptional splicing can occur in a slow-moving zone around the gene. eLife, 12. https://doi.org/10.7554/eLife.91357.2

      Coulon, A., Ferguson, M. L., de Turris, V., Palangat, M., Chow, C. C., & Larson, D. R. (2014). Kinetic competition during the transcription cycle results in stochastic RNA processing. eLife, 3, e03939. https://doi.org/10.7554/eLife.03939

      Drexler, H. L., Choquet, K., & Churchman, L. S. (2020). Splicing Kinetics and Coordination Revealed by Direct Nascent RNA Sequencing through Nanopores. Molecular Cell, 77(5), 985-998.e8. https://doi.org/10.1016/j.molcel.2019.11.017

      Gamliel, A., Meluzzi, D., Oh, S., Jiang, N., Destici, E., Rosenfeld, M. G., & Nair, S. J. (2022). Long-distance association of topological boundaries through nuclear condensates. Proceedings of the National Academy of Sciences of the United States of America, 119(32), e2206216119. https://doi.org/10.1073/pnas.2206216119

      Honkela, A., Peltonen, J., Topa, H., Charapitsa, I., Matarese, F., Grote, K., Stunnenberg, H. G., Reid, G., Lawrence, N. D., & Rattray, M. (2015). Genome-wide modeling of transcription kinetics reveals patterns of RNA production delays. Proceedings of the National Academy of Sciences of the United States of America, 112(42), 13115. https://doi.org/10.1073/pnas.1420404112

      Oh, S., Shao, J., Mitra, J., Xiong, F., D’Antonio, M., Wang, R., Garcia-Bassets, I., Ma, Q., Zhu, X., Lee, J.-H., Nair, S. J., Yang, F., Ohgi, K., Frazer, K. A., Zhang, Z. D., Li, W., & Rosenfeld, M. G. (2021). Enhancer release and retargeting activates disease-susceptibility genes. Nature, 595(7869), Article 7869. https://doi.org/10.1038/s41586-021-03577-1

      Patange, S., Ball, D. A., Wan, Y., Karpova, T. S., Girvan, M., Levens, D., & Larson, D. R. (2022). MYC amplifies gene expression through global changes in transcription factor dynamics. Cell Reports, 38(4). https://doi.org/10.1016/j.celrep.2021.110292

      Raj, A., van den Bogaard, P., Rifkin, S. A., van Oudenaarden, A., & Tyagi, S. (2008). Imaging individual mRNA molecules using multiple singly labeled probes. Nature Methods, 5(10), Article 10. https://doi.org/10.1038/nmeth.1253

      Rodriguez, J., Ren, G., Day, C. R., Zhao, K., Chow, C. C., & Larson, D. R. (2019). Intrinsic Dynamics of a Human Gene Reveal the Basis of Expression Heterogeneity. Cell, 176(1–2), 213-226.e18. https://doi.org/10.1016/j.cell.2018.11.026

      Saravanan, B., Soota, D., Islam, Z., Majumdar, S., Mann, R., Meel, S., Farooq, U., Walavalkar, K., Gayen, S., Singh, A. K., Hannenhalli, S., & Notani, D. (2020). Ligand dependent gene regulation by transient ERα clustered enhancers. PLOS Genetics, 16(1), e1008516. https://doi.org/10.1371/journal.pgen.1008516

      Stossi, F., Dandekar, R. D., Mancini, M. G., Gu, G., Fuqua, S. A. W., Nardone, A., De Angelis, C., Fu, X., Schiff, R., Bedford, M. T., Xu, W., Johansson, H. E., Stephan, C. C., & Mancini, M. A. (2020). Estrogeninduced transcription at individual alleles is independent of receptor level and active conformation but can be modulated by coactivators activity. Nucleic Acids Research, 48(4), 1800. https://doi.org/10.1093/nar/gkz1172

      Vargas, D. Y., Shah, K., Batish, M., Levandoski, M., Sinha, S., Marras, S. A. E., Schedl, P., & Tyagi, S. (2011). Single-Molecule Imaging of Transcriptionally Coupled and Uncoupled Splicing. Cell, 147(5), 1054–1065. https://doi.org/10.1016/j.cell.2011.10.024

      Waks, Z., Klein, A. M., & Silver, P. A. (2011). Cell-to-cell variability of alternative RNA splicing. Molecular Systems Biology, 7(1), 506. https://doi.org/10.1038/msb.2011.32

      Zambrano, S., Loffreda, A., Carelli, E., Stefanelli, G., Colombo, F., Bertrand, E., Tacchetti, C., Agresti, A., Bianchi, M. E., Molina, N., & Mazza, D. (2020). First Responders Shape a Prompt and Sharp NF-κB-Mediated Transcriptional Response to TNF-α. iScience, 23(9), 101529. https://doi.org/10.1016/j.isci.2020.101529

    1. Author response:

      The following is the authors’ response to the original reviews.

      Reviewer #1 (Public Review):

      The authors in this paper investigate the nature of the activity in the rodent EPN during a simple freely moving cue-reward association task. Given that primate literature suggests movement coding whereas other primate and rodent studies suggest mainly reward outcome coding in the EPNs, it is important to try to tease apart the two views. Through careful analysis of behavior kinematics, position, and neural activity in the EPNs, the authors reveal an interesting and complex relationship between the EPN and mouse behavior.

      Strengths:

      (1) The authors use a novel freely moving task to study EPN activity, which displays rich movement trajectories and kinematics. Given that previous studies have mostly looked at reward coding during head-fixed behavior, this study adds a valuable dataset to the literature. (2) The neural analysis is rich and thorough. Both single neuron level and population level (i.e. PCA) analysis are employed to reveal what EPN encodes.

      Thank you very much for this appreciation.

      Weaknesses:

      (1) One major weakness in this paper is the way the authors define the EPN neurons. Without a clear method of delineating EPN vs other surrounding regions, it is not convincing enough to call these neurons EPNs solely from looking at the electrode cannula track from Figure 2B. Indeed, EPN is a very small nucleus and previous studies like Stephenson-Jones et al (2016) have used opto-tagging of Vglut2 neurons to precisely label EPN single neurons. Wallace et al (2017) have also shown the existence of SOM and PV-positive neurons in the EPN. By not using transgenic lines and cell-type specific approaches to label these EPN neurons, the authors miss the opportunity to claim that the neurons recorded in this study do indeed come from EPN. The authors should at least consider showing an analysis of neurons slightly above or below EPN and show that these neurons display different waveforms or firing patterns.

      We thank the reviewer for their comment, and we thank the opportunity to expand on the inclusion criteria of studied units after providing an explanation. 

      As part of another study, we performed experiments recording in EPN with optrodes and photoidentification in PV-Cre animals. We found optoidentified units in both: animals with correct placement (within the EPN) and on those with off-target placement (within the thalamus or medial to the EPN). Thus, despite the use of Cre animals, we relied on histology to ensure correct EPN recording. We believe that the optotagging based purely on neural makers such as PV, SOM, VGLUT, VGAT would not provide a better anatomical delineation of the EPN since adjacent structures are rich in those same markers. The thalamic reticular nucleus is just dorsal to the EPN and it has been shown to express both SOM and PV (Martinez-Garcia et al., 2020). 

      On the other hand, the lateral hypothalamus (just medial to the EPN) also expresses vGlut2 and SOM. Stephenson-Jones (2016), Extended Data Figure 1, panel g, shows vGluT2 and somatostatin labeling of neurons, with important expression of neurons dorsal, ventral and medial to the EPN. Thus, we believe that viral strategies relying on single neuronal markers still depend on careful histological analysis of recording sites.

      A combination of neural markers or more complex viral strategies might be more suitable to delineate the EPN. As an example, for anatomical tracing Stephenson-Jones et al. 2016 performed a rabies-virus based approach involving retrogradely transported virus making use of projection sites through two injections. Two step viral approaches were also performed in Wallace, M. et al. 2017. We attempted to perform a two-step viral approach, using an anterogradely transported Cre-expressing virus (AAV1.hSyn.Cre.WPRE.hGH) injected into the striatum and a second Cre dependent ChR2 into the EPN. However, our preliminary experiments showed that this double viral approach had a stark effect decreasing the performance of animals during the task (we attempted re-training 2-3 weeks after viral infections and animals failed to turn to the contralateral side of the injections). We believe that this approach might have had a toxic effect (Zingg et al., 2017). 

      To this point, a recent paper (Lazaridis et al., 2019) repeated an optogenetic experiment performed in the Stephenson-Jones et al. study, using a set of different viral approaches and concluded that increasing the activity of GPi-LHb is not aversive, as it had been previously reported. Thus, future studies attempting to increase anatomical specificity are a must, but they will require using viral approaches amenable to the behavioral paradigm.

      We attempted to find properties regarding waveforms, firing rate, and firing patterns from units above or below, however, we did not find a marker that could generate a clear demarcation. We show here a figure that includes the included units in this study as well as excluded ones to show that there is a clear overlap.

      Author response image 1.

      Finally, we completely agree with the reviewer in that there is still room for improvement. We have further expanded the Methods section to explain better our efforts to include units recorded within the EPN. Further, we have added a paragraph within the Discussion section to point out this limitation (lines 871-876).

      Methods (lines 116-131):

      “Recordings. Movable microwire bundles (16 microwires, 32 micrometers in diameter, held inside a cannula, Innovative Neurophysiology, Durham, NC)] were stereotaxtically implanted just above the entopeduncular nucleus (-0.8 AP, 1.7 ML, 3.9 DV). Post surgical care included antibiotic, analgesic and antiinflammatory pharmacological treatment. After 5 days of recovery, animals were retrained for 1-2 weeks. Unitary activity was recorded for 2-6 days at each dorsoventral electrode position and the session with the best electrophysiological (signal to noise ratio (>2), stability across time) and behavioral [performance, number of trials (>220)] quality was selected. Microwire electrodes were advanced in 50 micrometer dorsoventral steps for 500 micrometers in total. After experiment completion, animals were perfused with a 4% paraformaldehyde solution. Brains were extracted, dehydrated with a 30% sucrose solution and sectioned in a cryostat into 30micron thick slices. Slices were mounted and photographed using a light microscope. Microwire tracks of the 16-microwire bundle were analyzed (Fig. 2A-B) and only animals with tracks traversing the EPN were selected (6 out of 10). Finally, we located the final position of microwire tips and inferred the dorsoventral recording position of each of the recording sessions. Only units recorded within the EPN were included.” 

      Discussion (lines 871-876):

      “A weakness of the current study is the lack of characterization of neuronal subtypes. An area of opportunity for future research could be to perform photo-identification of neuronal subtypes within the EPN which could contribute to the overall description of the information representation. Further, detailed anatomical viral vector strategies could aid to improve anatomical localization of recordings, reduce reliance on histological examination, and solve some current controversies (Lazaridis et al., 2019).” 

      (2) The authors fail to replicate the main finding about EPN neurons which is that they encode outcome in a negative manner. Both Stephenson-Jones et al (2016) and Hong and Hikosaka (2008) show a reward response during the outcome period where firing goes down during reward and up during neutral or aversive outcome. However, Figure 2 G top panel shows that the mean population is higher during correct trials and lower during incorrect trials. This could be interesting given that the authors might try recording from another part of EPN that has not been studied before. However, without convincing evidence that the neurons recorded are from EPN in the first place (point 1), it is hard to interpret these results and reconcile them with previous studies.

      We really thank the reviewer for pointing out that we need to better explain how EPN units encode outcome. We now provide an additional panel in Figure 4, its corresponding text in the results section (lines 544-562) and a new paragraph in the discussion related to this comment.

      We believe that we do indeed recapitulate findings of both of Stephenson-Jones et al (2016) and Hong and Hikosaka (2008). Both studies focus on a specific subpopulation of GPi/EPN neurons that project to the lateral habenula (LHb). Stephenson-Jones et al (2016) posit that GPi-LHb neurons (which they opto-tag as vGluT2) exhibit a decreased firing rate during rewarding outcomes. Hong and Hikosaka (2008) antidromically identified LHb projecting neurons through within the GPi and found reward positive and reward negative neurons, which were respectively modulated either by increasing or decreasing their firing rate with a rewarding outcome (red and green dots on the x-axis of Figure 5A in their paper).

      As the reviewer pointed out the zScore may be misleading. Therefore, in our study we also decomposed population activity on reward axis through dPCA. When marginalizing for reward in Figure 3F, we find that the weights of individual units on this axis are centered around zero, with positive and negative values (Figure 3F, right panel). Thus, units can code a rewarding outcome as either an increase or a decrease of activity. We show example units of such modulation in Figure 3-1g and h.

      We had segregated our analysis of spatio-temporal and kinematic coding upon the reward coding of units in Figure 4L-M. Yet, following this comment and in an effort of further clarifying this segregation, we introduced panels with the mean zScore of units during outcome evaluation in Figure 4L.

      We amended the main text to better explain these findings (lines 544-562).

      “Previous reports suggest that EPN units that project to the lateral habenula encode reward as a decrease in firing rate. Thus, we wished to ask whether reward encoding units can code kinematic and spatio-temporal variables as well.

      To this end, we first segregated units upon their reward coding properties: reward positive (which increased activity with reward) and reward negative units (which decreased activity with reward). We performed auROC on the 250ms after head entry comparing rewarded trials and incorrect trails (p<0.001, permutation test). Mean activity of reward insensitive, positive and negative units is shown in Fig. 4L. Next, we performed a dimensionality reduction on the coefficients of the model that best explained both contexts (kinematic + spatio-temporal model on pooled data) using UMAP (McInnes et al., 2018). We observe a continuum rather than discrete clusters (Fig. 4L). Note that individual units are color coded according to their responsivity to reward. We did not find a clear clustering either.”  

      Paragraph added in the discussion (lines 749-755):

      “In this study, we found that rewarding outcomes can be represented by EPN units through either an increase or a decrease in firing rate (Fig. 3F, 3-1g-h, 4L). While Stephenson-Jones et al., 2016 found that lateral habenula (LHb)-projecting neurons within the EPN of mice primarily encoded rewarding outcomes by a decrease in firing rate, Hong and Hikosaka, 2008 observed that in primates, LHb-projecting units could encode reward through either a decrease or an increase in firing rate. Thus, our results align more closely with the latter study, which also employed an operant conditioning task.”

      (3) The authors say that: 'reward and kinematic doing are not mutually exclusive, challenging the notion of distinct pathways and movement processing'. However, it is not clear whether the data presented in this work supports this statement. First, the authors have not attempted to record from the entire EPN. Thus it is possible that the coding might be more segregated in other parts of EPN. Second, EPNs have previously been shown to display positive firing for negative outcomes and vice versa, something which the authors do not find here. It is possible that those neurons might not encode kinematic and movement variables. Thus, the authors should point out in the main text the possibility that the EPN activity recorded might be missing some parts of the whole EPN.

      We thank the reviewer for the opportunity to expand on this topic. We believe it is certainly possible that other not-recorded regions of the EPN might exhibit greater segregation of reward and kinematics. However, we considered it worthwhile pointing out that from the dataset collected in this study reward-sensitive units encode kinematics in a similar fashion to reward-insensitive ones (Fig. 4L,M). Moreover, we asked specifically whether reward-negative units (that decrease firing rate with rewarding outcomes, as previously reported) could encode kinematics and spatio-temporal variables with different strength than reward-insensitive ones and could not find significant differences (Fig. 4M).

      We did indeed find units that displayed decreased firing rate upon rewarding outcomes, as has been previously reported. We have addressed this fact more thoroughly in point (2). 

      Finally, we agree with the reviewer that the dataset collected in this study is by no means exhaustive of the entire EPN and have thus included a sentence pointing this out in the Discussion section (lines 805-806):

      “Given that we did not record from the entire EPN, it is still possible that another region of the nucleus might exhibit more segregation.”

      (4) The authors use an IR beam system to record licks and make a strong claim about the nature of lick encoding in the EPN. However, the authors should note that IR beam system is not the most accurate way of detecting licks given that any object blocking the path (paw or jaw-dropping) will be detected as lick events. Capacitance based, closed-loop detection, or video capturing is better suited to detect individual licks. Given that the authors are interested in kinematics of licking, this is important. The authors should either point this out in the main text or verify in the system if the IR beam is correctly detecting licks using a combination of those methods.

      We thank the reviewer for the opportunity of clarifying the lick event acquisition. We have experience using electrical alternatives to lickometers; however, we believe they were not best suited to this application. Closed-loop lickometers generally use a metallic grid upon which animals stand so that the loop can be closed; however, we wanted to have a transparent floor. We have found capacitance based lickometers to be useful in head-fixed conditions but have noticed that they are very dependent on animal position and proximity of other bodyparts such as limbs. Given the freely moving aspect of the task this was difficult to control. Finally, both electric alternatives for lickometers are more prone to noise and may introduce electrical artifacts that might contaminate the spiking signal. This is why we opted to use a slit in combination with an IR beam that would only fit the tongue and that forced enough protrusion such that individual licks could be monitored. Further, the slit could not fit other body-parts like the paw or jaw. We have now included a video (Supp. Video 2) showing a closeup of this behavior that better conveys how the jaw and paw do not fit inside the slit. The following text has been added in the corresponding methods section (lines 97-98):

      “The lickometer slit was just wide enough to fit the tongue and deep enough to evoke a clear tongue protrusion.”

      Reviewer #1 (Recommendations For The Authors):

      (1)The authors should verify using opto-tagging of either Vglut2, SOM, or PV neurons whether they can see the same firing pattern. If not, the authors should address this weakness in the paper.

      We thank the reviewer for this important point, we have provided a more detailed reply above.

      (2)The way dPCA or PCA is applied to the data is not stated at all in the main text. Are all units from different mice combined? Or applied separately for each mouse? How does that affect the interpretation of the data? At least a brief text should be included in the main text to guide the readers.

      We thank the reviewer for pointing out this important omission. We have included an explanation in the Methods section and in the Main text.

      Methods (lines 182-184):

      “For all population level analyses individual units recorded from all sessions and all animals were pooled to construct pseudo-simultaneous population response of combined data mostly recorded separately.”

      Main text (lines 397-399):

      “For population level analyses throughout the study, we pooled recorded units from all animals to construct a pseudo-simultaneous population.”

      Discussion (lines 729-730):

      “…(from pooled units from all animals to construct a pseudo-simultaneous population, which assumes homogeneity across subjects)”

      (3) The authors argue that they do not find 'value coding' in this study. However, the authors never manipulate reward size or probability, but only the uncertainty or difficulty of the task. This might be better termed 'difficulty', and it is difficult to say whether this correlates with value in this task. For instance, mice might be very confident about the choice, even for an intermediate frequency sweep, if the mouse had waited long enough to hear the full sweep. In that case, the difficulty would not correlate with value, given that the mouse will think the value of the port it is going to is high. Thus, authors should avoid using the term value.

      We agree with the reviewer. We have modified the text to specify that difficulty was the variable being studied and added the following sentence in the Discussion (lines 747-748):

      “It is still possible that by modifying reward contingencies such as droplet size value coding could be evidenced.”

      (4) How have the authors obtained Figure 7D bottom panel? It is unclear at all what this correlation represents. Are the authors looking at a correlation between instantaneous firing rate and lick rate during a lick bout?

      We thank the reviewer for pointing out that omission. It is indeed correlation coefficient between the instantaneous firing rate and the instantaneous lick rate for a lick bout. We have included labeling in Figure 7D and pointed this out in the main text [lines 680-681]:

      “Fig.7D, lower panel shows the correlation coefficient between the instantaneous firing rate and the instantaneous lick rate within a lick bout for all units.”

      Reviewer #2 (Public Review):

      This paper examined how the activity of neurons in the entopeduncular nucleus (EPN) of mice relates to kinematics, value, and reward. The authors recorded neural activity during an auditory-cued two-alternative choice task, allowing them to examine how neuronal firing relates to specific movements like licking or paw movements, as well as how contextual factors like task stage or proximity to a goal influence the coding of kinematic and spatiotemporal features. The data shows that the firing of individual neurons is linked to kinematic features such as lick or step cycles. However, the majority of neurons exhibited activity related to both movement types, suggesting that EPN neuronal activity does not merely reflect muscle-level representations. This contradicts what would be expected from traditional action selection or action specification models of the basal ganglia.

      The authors also show that spatiotemporal variables account for more variability compared to kinematic features alone. Using demixed Principal Component Analysis, they reveal that at the population level, the three principal components explaining the most variance were related to specific temporal or spatial features of the task, such as ramping activity as mice approached reward ports, rather than trial outcome or specific actions. Notably, this activity was present in neurons whose firing was also modulated by kinematic features, demonstrating that individual EPN neurons integrate multiple features. A weakness is that what the spatiotemporal activity reflects is not well specified. The authors suggest some may relate to action value due to greater modulation when approaching a reward port, but acknowledge action value is not well parametrized or separated from variables like reward expectation.

      We thank the reviewer for the comment. We indeed believe that further exploring these spatiotemporal signals is important and will be the subject of future studies.

      A key goal was to determine whether activity related to expected value and reward delivery arose from a distinct population of EPN neurons or was also present in neurons modulated by kinematic and spatiotemporal features. In contrast to previous studies (Hong & Hikosaka 2008 and Stephenson-Jones et al., 2016), the current data reveals that individual neurons can exhibit modulation by both reward and kinematic parameters. Two potential differences may explain this discrepancy: First, the previous studies used head-fixed recordings, where it may have been easier to isolate movement versus reward-related responses. Second, those studies observed prominent phasic responses to the delivery or omission of expected rewards - responses largely absent in the current paper. This absence suggests a possibility that neurons exhibiting such phasic "reward" responses were not sampled, which is plausible since in both primates and rodents, these neurons tend to be located in restricted topographic regions. Alternatively, in the head-fixed recordings, kinematic/spatial coding may have gone undetected due to the forced immobility.

      Thank you for raising this point. Nevertheless, there is some phasic activity associated with reward responses, which can be seen in the new panel in Figure 4L.

      Overall, this paper offers needed insight into how the basal ganglia output encodes behavior. The EPN recordings from freely moving mice clearly demonstrate that individual neurons integrate reward, kinematic, and spatiotemporal features, challenging traditional models. However, the specific relationship between spatiotemporal activity and factors like action value remains unclear.

      We really appreciate this reviewer for their valuable comments.

      Reviewer #2 (Recommendations For The Authors):

      One small suggestion is to make sure that all the panels in the figures are well annotated. I struggled in places to know what certain alignments or groupings meant because they were not labelled. An example would be what do the lines correspond to in the lower panels of Figure 2D and E. I could figure it out from other panels but it would have helped if each panel had better labelling.

      Thanks for pointing this out, we have improved labelling across the figures and corrected the specific example you have pointed out.

      The paper is very nice though. Congratulations!

      Thank you very much.

      Editor's note:

      Should you choose to revise your manuscript, please include full statistical reporting including exact p-values wherever possible alongside the summary statistics (test statistic and df) and 95% confidence intervals. These should be reported for all key questions and not only when the p-value is less than 0.05 in the main manuscript.

      We thank the editor for the comment. A statistics table has been added.

      References:

      Lazaridis, I., Tzortzi, O., Weglage, M., Märtin, A., Xuan, Y., Parent, M., Johansson, Y., Fuzik, J., Fürth, D., Fenno, L. E., Ramakrishnan, C., Silberberg, G., Deisseroth, K., Carlén, M., & Meletis, K. (2019). A hypothalamus-habenula circuit controls aversion. Molecular Psychiatry, 24(9), 1351–1368. https://doi.org/10.1038/s41380-019-0369-5

      Martinez-Garcia, R. I., Voelcker, B., Zaltsman, J. B., Patrick, S. L., Stevens, T. R., Connors, B. W., & Cruikshank, S. J. (2020). Two dynamically distinct circuits drive inhibition in the sensory thalamus. Nature, 583(7818), 813–818. https://doi.org/10.1038/s41586-0202512-5

      McInnes, L., Healy, J., Saul, N., & Großberger, L. (2018). UMAP: Uniform Manifold Approximation and Projection. Journal of Open Source Software, 3(29), 861. https://doi.org/10.21105/joss.00861

      Zingg, B., Chou, X. lin, Zhang, Z. gang, Mesik, L., Liang, F., Tao, H. W., & Zhang, L. I. (2017). AAV-Mediated Anterograde Transsynaptic Tagging: Mapping Corticocollicular Input-Defined Neural Pathways for Defense Behaviors. Neuron, 93(1), 33–47. https://doi.org/10.1016/j.neuron.2016.11.045

    1. Author Response

      The following is the authors’ response to the original reviews.

      eLife assessment

      This paper reports valuable results regarding the potential role and time course of the prefrontal cortex in conscious perception. Although the sample size is small, the results are clear and convincing, and strengths include the use of several complementary analysis methods. The behavioral test includes subject report so the results do not allow for distinguishing between theories of consciousness; nevertheless, results do advance our understanding of the contribution of prefrontal cortex to conscious perception. We appreciate very much for editor and reviewers encouraged review opinion. Particularly, we thank three reviewers very much for their professional and constructive comments that help us to improve the manuscript substantially.

      Public Reviews:

      Reviewer #1 (Public Review):

      This is a clear and rigorous study of intracranial EEG signals in the prefrontal cortex during a visual awareness task. The results are convincing and worthwhile, and strengths include the use of several complementary analysis methods and clear results. The only methodological weakness is the relatively small sample size of only 6 participants compared to other studies in the field. Interpretation weaknesses that can easily be addressed are claims that their task removes the confound of report (it does not), and claims of primacy in showing early prefrontal cortical involvement in visual perception using intracranial EEG (several studies already have shown this). Also the shorter reaction times for perceived vs not perceived stimuli (confident vs not confident responses) has been described many times previously and is not a new result.

      We appreciate very much for the reviewer’s encouraged opinion. We are going to address reviewer’s specific questions and comments point-by-point in following.

      ‘The only methodological weakness is the relatively small sample size of only 6 participants compared to other studies in the field.’

      We agree that the sample size is relatively small in the present study. To compensate such shortcoming, we rigorously verified each result at both individual and population levels, resembling the data analysis method in non-human primate study.

      Interpretation weaknesses that can easily be addressed are claims that their task removes the confound of report (it does not),

      Thank you very much for your comment. We agree that our task does not remove the confound of report entirely. However, we believe that our task minimizes the motor confounds by dissociating the emergence of awareness from motor in time and balanced direction of motor between aware and unaware conditions. We have modified the text according to reviewer’s comment in the revised manuscript as following: “This task removes the confound of motor-related activity”.

      ..and claims of primacy in showing early prefrontal cortical involvement in visual perception using intracranial EEG (several studies already have shown this).

      We agree that several iEEG studies, including ERP and HFA, have shown the early involvement of prefrontal cortical in visual perception. However, in these studies, the differential activity between conscious and unconscious conditions was not investigated, thus, the activity in prefrontal cortex might be correlated with unconscious processing, rather than conscious processing. In present study, we compared the neural activity in PFC between conscious and unconscious trials, and found the correlation between PFC activity and conscious perception. Although one iEEG study(Gaillard et al., 2009) reported awareness-specific PFC activation, the awareness-related activity started 300 ms after the onset of visual stimuli, which was ~100 ms later than the early awareness related activity in our study. Also, due to the limited number of electrodes in the previous study (2 patients with 19 recording sites mostly in mesiofrontal and peri-insular regions), it was restricted while exploring the awareness-related activity in PFC. In the present study, the number of recording sites (245) were much more than previous study and covered multiple areas in PFC. Our results further show earlier awareness-related activity (~ 200 ms after visual stimuli onset), including ERP, HFA and PLV, which sheds new light on understanding of the role of PFC in conscious perception.

      We have added this discussion in the MS (lines 522-536);

      Also the shorter reaction times for perceived vs not perceived stimuli (confident vs not confident responses) has been described many times previously and is not a new result. Thank you very much for your comment. We agree that the reaction time is strongly modulated by the confident level, which has been described previously (Broggin, Savazzi, & Marzi, 2012; Marzi, Mancini, Metitieri, & Savazzi, 2006). However, in previous studies, the confident levels were usually induced by presenting stimulus with different physical property, such as spatial frequency, eccentricity and contrast. It is well known that the more salient stimuli will induce the faster process of visual information and speed up the process of visuomotor transformation, eventually shorten the reaction time (Corbetta & Shulman, 2002; Posner & Petersen, 1990). Therefore, the dependence of visual processing on the salience of visual stimulus confounds with the effect of visual awareness on the reaction time, which is hard to attribute the shorter reaction time in more salient condition purely to visual awareness. In contrast, we create a condition (near perceptual threshold) in the present study, in which the saliency (contrast) of visual stimulus is very similar in both aware and unaware conditions in order to eliminate the influence of stimulus saliency in reaction time. We think that the difference in reaction time in our study is mainly due to the modulation of awareness state, which was not reported previously.

      We have added the discussion in the MS (lines 497-507).

      Reviewer #1 (Recommendations For The Authors):

      Specific comments follow:

      Abstract: "we designed a visual awareness task that can minimize report-related confounding" and in the Introduction lines 112-115: "Such a paradigm can effectively dissociate awareness-related activity from report-related activity in terms of time... and report behavior"; Discussion lines 481-483 "even after eliminating the influence of the confounding variables related to subjective reports such as motion preparation" and other similar statements in the manuscript should be removed. The task involves report using eye movements with every single stimulus. The fact that there is report for both perceived and not perceived stimuli, that the direction of report is not determined until the time of report, and that there is delay between stimulus and report, does not remove the report-related post-perceptual processing that will inevitably occur in a task where overt report is required for every single trial. For example, brain activity related to planning to report perception will only occur after perceived trials, regardless of the direction of eye movement later decided upon. This preparation to respond is different for perceived and not perceived stimuli, but is not part of the perception itself. In this way the current task is not at all unique and does not substantially differ from many other report-based tasks used previously.

      The objective of present study is to assess whether PFC is involved in the emergence of visual awareness. To do so, it is crucial to determine the subjective awareness state as correct as possible. Considering the disadvantage of non-report paradigms in determining the subjective awareness state (Tsuchiya et al. TiCS, 2015; Mashour et al, Neuron, 2020), we employed a balanced report paradigm. It has been argued (Merten & Nieder, PNAS, 2011) that, in the balanced report paradigms, subjects could not prepare any motor response during the delay period because only the appearance of a rule cue (change color of fixation point at the end of delay period) informed subjects about the appropriate motor action. In this case, the post-perceptual processing during delay period might reflect the non-motor cognitive activity. Alternatively, as being mentioned by reviewer, the post-perceptual processing might relate to planning to report perception, which is different for perceived and not perceived stimuli. Therefore, up to date, the understanding of the post-perceptual processing remains controversial. According to reviewer’s comment, we have modified the description of our task as following: “we designed a visual awareness task that can minimize report-related motor confounding”. Also, have changed “report-related” to “motorrelated” in the text of manuscript.

      Figures 3, 4 changes in posterior middle frontal gyri suggest early frontal eye field involvement in perception. This should be interpreted in the context of many previous studies showing FEF involvement in signal detection. The authors claim that "earlier visual awareness related activities in the prefrontal cortex were not found in previous iEEG studies, especially in the HG band" on lines 501-502 of the Discussion. This statement is not true and should be removed. The following statement in the Discussion on lines 563-564 should be removed for the same reasons: "our study detected 'ignition' in the human PFC for the first time." Authors should review and cite the following studies as precedent among others:

      Blanke O, Morand S, Thut G, Michel CM, Spinelli L, Landis T, Seeck M (1999) Visual activity in the human frontal eye field. Neuroreport 10 (5):925-930. doi:10.1097/00001756-19990406000006

      Foxe JJ, Simpson GV (2002) Flow of activation from V1 to frontal cortex in humans. A framework for defining "early" visual processing. Exp Brain Res 142 (1):139-150. doi:10.1007/s00221-001-0906-7

      Gaillard R, Dehaene S, Adam C, Clemenceau S, Hasboun D, Baulac M, Cohen L, Naccache L (2009) Converging intracranial markers of conscious access. Plos Biology 7 (3):e61

      Gregoriou GG, Gotts SJ, Zhou H, Desimone R (2009) High-frequency, long-range coupling between prefrontal and visual cortex during attention. Science 324:1207-1210

      Herman WX, Smith RE, Kronemer SI, Watsky RE, Chen WC, Gober LM, Touloumes GJ, Khosla M, Raja A, Horien CL, Morse EC, Botta KL, Hirsch LJ, Alkawadri R, Gerrard JL, Spencer DD, Blumenfeld H (2019) A Switch and Wave of Neuronal Activity in the Cerebral Cortex During the First Second of Conscious Perception. Cereb Cortex 29 (2):461-474.

      Khalaf A, Kronemer SI, Christison-Lagay K, Kwon H, Li J, Wu K, Blumenfeld H (2022) Early neural activity changes associated with stimulus detection during visual conscious perception. Cereb Cortex. doi:10.1093/cercor/bhac140

      Kwon H, Kronemer SI, Christison-Lagay KL, Khalaf A, Li J, Ding JZ, Freedman NC, Blumenfeld H (2021) Early cortical signals in visual stimulus detection. Neuroimage 244:118608.

      We agree that several iEEG studies, including ERP and HFA, have shown the early involvement of prefrontal cortical in visual perception. However, in these studies, the differential activity between conscious and unconscious conditions was not investigated, thus, the activity in prefrontal cortex might be correlated with unconscious processing, rather than conscious processing. In present study, we compared the neural activity in PFC between conscious and unconscious trials, and found the correlation between PFC activity and conscious perception. Although one iEEG study reported awareness-specific PFC activation, the awareness-related activity started 300 ms after the onset of visual stimuli, which was ~100 ms later than the early awareness related activity in our study. Also, due to the limited number of electrodes in the previous study (2 patients with 19 recording sites mostly in mesiofrontal and peri-insular regions), it was restricted while exploring the awareness-related activity in PFC. In the present study, the number of recording sites (245) were much more than previous study and covered multiple areas in PFC. Our results further show earlier awareness-related activity (~ 200 ms after visual stimuli onset), including ERP, HFA and PLV, which sheds new light on understanding of the role of PFC in conscious perception.

      We have added this discussion in the MS (lines 522-533);

      Minor weakness that should be mentioned in the Discussion: The intervals for the FP (fixation period) and Delay period were both fixed at 600 ms instead of randomly jittered, so that subjects likely had anticipatory activity predictably occurring with each grating and cue stimulus.

      Thank you very much for your comment. We agree that subjects might have anticipatory activity during experiment. Actually, the goal for us to design the task in this way is to try to balance the effect of attention and anticipation between aware and unaware conditions. We have added this discussion in the MS (lines 467-469);

      The faster reaction times for perceived/confident responses vs not perceived/unconfident responses has been reported many times previously in the literature and should be acknowledged rather than being claimed as a novel finding. Authors should modify p. 163 lines 160-162, first sentence of the Discussion lines 445-446 "reaction time.. shorter" claiming this was a novel finding; same for lines 464-467. Please see the following among others:

      Broggin E, Savazzi S, Marzi CA (2012) Similar effects of visual perception and imagery on simple reaction time. Q J Exp Psychol (Hove) 65 (1):151-164. doi:10.1080/17470218.2011.594896

      Chelazzi L, Marzi CA, Panozzo G, Pasqualini N, Tassinari G, Tomazzoli L (1988) Hemiretinal differences in speed of light detection in esotropic amblyopes. Vision Res 28 (1):95-104 Marzi CA, Mancini F, Metitieri T, Savazzi S (2006) Retinal eccentricity effects on reaction time to imagined stimuli. Neuropsychologia 44 (8):1489-1495. doi:10.1016/j.neuropsychologia.2005.11.012

      Posner MI (1994) Attention: the mechanisms of consciousness. Proceedings of the National Academy of Sciences of the United States of America 91 (16):7398-7403

      Sternberg S (1969) Memory-scanning: mental processes revealed by reaction-time experiments. Am Sci 57 (4):421-457

      Thanks. We have cited some of these papers in the revised manuscript due to the restricted number of citations.

      Methods lines 658-659: "results under LU and HA conditions were classified as the control group and were only used to verify and check the results during calculation." However the authors show these results in the figures and they are interesting. HA stimuli show earlier responses than NA stimuli. This is a valuable result which should be discussed and interpreted in light of the other findings.

      We thank very much for reviewer’s comment. We have made discussion accordingly in the revised MS (lines 535-536).

      General comment on figures: Many of the figure elements are tiny and the text labels and details can't be seen at all, especially single trial color plots, and the brain insets showing recording sites.

      We have modified the figures accordingly.

      Other minor comments: Typo: Figure 2 legend, line 169 "The contrast level resulted in an awareness percentage greater than 25%..." is missing a word and should say instead something like "The contrast level that resulted in an awareness percentage greater than 25%..."

      Thanks. We have corrected the typo accordingly.

      Figure 2 Table description in text line 190 says "proportions of recording sites" but the Table only shows number of recording sites and number of subjects, not "proportions." This should be corrected in the text.

      Thanks. We have corrected the error.

      Figure 3, and other figures, should always label the left and right hemispheres to avoid ambiguity.

      Thanks. We have made correction accordingly. In caption of Figure 2D (line 189), we modified the sentence as ‘In all brain images, right side of the image represents the right side of the brain’.

      Methods line 666. The saccadic latency calculations paragraph should have a separate heading before it, to separate it from the Behavioral data analysis section.

      Thanks. It has been corrected in line 725.

      Reviewer #2 (Public Review):

      The authors attempt to address a long-standing controversy in the study of the neural correlates of visual awareness, namely whether neurons in prefrontal cortex are necessarily involved in conscious perception. Several leading theories of consciousness propose a necessary role for (at least some sub-regions of) PFC in basic perceptual awareness (e.g., global neuronal workspace theory, higher order theories), while several other leading theories posit that much of the previously reported PFC contributions to perceptual awareness may have been confounded by task-based cognition that co-varied between the aware and unaware reports (e.g., recurrent processing theory, integrated information theory). By employing intracranial EEG in human patients and a threshold detection task on low-contrast visual stimuli, the authors assessed the timing and location of neural populations in PFC that are differentially activated by stimuli that are consciously perceived vs. not perceived. Overall, the reported results support the view that certain regions of PFC do contribute to visual awareness, but at time-points earlier than traditionally predicted by GNWT and HOTs.

      Reply: We appreciate very much for the reviewer’s encouraged opinion.

      Major strengths of this paper include the straightforward visual threshold detection task including the careful calibration of the stimuli and the separate set of healthy control subjects used for validation of the behavioral and eye tracking results, the high quality of the neural data in six epilepsy patients, the clear patterns of differential high gamma activity and temporal generalization of decoding for seen versus unseen stimuli, and the authors' interpretation of these results within the larger research literature on this topic. This study appears to have been carefully conducted, the data were analyzed appropriately, and the overall conclusions seem warranted given the main patterns of results.

      Reply: We appreciate very much for the reviewer’s encouraged opinion.

      Weaknesses include the saccadic reaction time results and the potential flaws in the design of the reporting task. This is not a "no report" paradigm, rather, it's a paradigm aimed at balancing the post-perceptual cognitive and motor requirements between the seen and unseen trials. On each trial, subjects/patients either perceived the stimulus or not, and had to briefly maintain this "yes/no" judgment until a fixation cross changed color, and the color change indicated how to respond (saccade to the left or right). Differences in saccadic RTs (measured from the time of the fixation color change to moving the eyes to the left or right response square) were evident between the seen and unseen trials (faster for seen). If the authors' design achieved what they claim on page 3, "the report behaviors were matched between the two awareness states ", then shouldn't we expect no differences in saccadic RTs between the aware and unaware conditions? The fact that there were such differences may indicate differences in post-perceptual cognition during the time between the stimulus and the response cue. Alternatively, the RT difference could reflect task-strategies used by subjects/patients to remember the response mapping rules between the perception and the color cue (e.g., if the YES+GREEN=RIGHT and YES+RED=LEFT rules were held in memory, while the NO mappings were inferred secondarily rather than being actively held in memory). This saccadic RT result should be better explained in the context of the goals of this particular reporting-task.

      The objective of present study is to assess whether PFC is involved in the emergence of visual awareness. To do so, it is crucial to determine the subjective awareness state as correct as possible. Considering the disadvantage of non-report paradigms in determining the subjective awareness state (Tsuchiya et al, TiCS, 2015; Mashour et al, Neuron, 2020), we employed a balanced report paradigm. It has been argued (Merten & Nieder, PNAS, 2011) that, in the balanced report paradigms, subjects could not prepare any motor response during the delay period because only after the appearance of a rule cue (change color of fixation point at the end of delay period) subjects were informed about the appropriate motor action. In this case, the post-perceptual processing during delay period might reflect the non-motor cognitive activity, such as working memory (Mashour et al. Neuron, 2020). Alternatively, as being mentioned by reviewer, the postperceptual processing might relate to planning to report perception, which is different for perceived and not perceived stimuli (Aru et al. Neurosci Biobehav Rev, 2012 ). Therefore, up to date, the understanding of the post-perceptual processing remains controversial. Considering reviewer’s comment together with other opinions, we have modified the description of our task as following: “we designed a visual awareness task that can minimize report-related motor confounding”. Also, we have changed “report-related” to “motor-related” in the rest of manuscript.

      Regarding the question whether the saccadic RT in our balanced response paradigm should be expected to be similar between aware and unaware condition, we think that the RT should be similar in case if the delay period is long enough for the decision of “no” to be completed. In fact, in a previous study (Merten & Nieder, PNAS, 2011), the neuronal encoding of “no” decision didn’t appear until 2s after the stimulus cue onset. However, in our task, the delay period lasted only 600 ms that was long enough to form the “yes” decision, but was not enough to form the “no” decision. It might be the reason that our data show shorter RT in aware condition than in unaware condition.

      We totally agree reviewer’s comment about the alternative interpretation for RT difference between aware and unaware condition in our study, i.e., reflecting task-strategies used by subjects/patients to remember the response mapping rules between the perception and the color cue (e.g., if the YES+GREEN=RIGHT and YES+RED=LEFT rules were held in memory, while the NO mappings were inferred secondarily rather than being actively held in memory). We have made additional discussion about these questions in the revised manuscript (lines 492496).

      Nevertheless, the current results do help advance our understanding of the contribution of PFC to visual awareness. These results, when situated within the larger context of the rapidly developing literature on this topic (using "no report" paradigms), e.g., the recent studies by Vishne et al. (2023) Cell Reports and the Cogitate consortium (2023) bioRxiv, provide converging evidence that some sub-regions of PFC contribute to visual awareness, but at latencies earlier than originally predicted by proponents of, especially, global neuronal workspace theory.

      We appreciate very much for the reviewer’s encouraged opinion.

      Reviewer #2 (Recommendations For The Authors):

      Abstract: "the spatiotemporal overlap between the awareness-related activity and the interregional connectivity in PFC suggested that conscious access and phenomenal awareness may be closely coupled." I strongly suggest revising this sentence. The current results cannot be used to make such a broad claim about p-consciousness vs. a-consciousness. This study used a balanced trial-by-trial report paradigm, which can only measure conscious access.

      We thank reviewer for this comment. We have withdrawn this sentence from the revised manuscript.

      Task design: A very similar task was used previously by Schröder et al. (2021) J Neurosci. See specifically, their Figure 1, and Figure 4B-C. Using almost the exact same "matching task", the authors of this previous study show that they get a P3b for both the perceived and not-perceived conditions, confirming that post-perceptual cognition/report confounds were not eliminated, but instead were present in (and balanced between) both the perceived/not-perceived trials due to the delayed matching aspect of the design. This previous paper should be cited and the P3b result should be considered when assessing whether cognition/report confounds were addressed in the current study.

      Thank you very much for your reminding about the study of Schröder et al. We are sorry for not citing this closely related study in our previous manuscript. Schröder et al. found while P3b showed significant difference between perceived and not-perceived trials in direct report task, the P3b was presented in both perceived/not-perceived trials and not significantly different in the matched task. Based on these findings, Schröder et al. argued that P3b represented the task specific post-perceptual cognition/report rather than the emergence of awareness per se. Considering the similarity of tasks between Schröder et al. and ours, we agree that our task is not able to totally eliminate the confound of post-perceptual cognition/report related activity with awareness related activity. Nevertheless, our task is able to minimize the confound of motorrelated activity with the emergence of awareness by separating them in time and balancing the direction of responsive movements. Therefore, we modified the term of “report-related” to “motor-related” in the text of revised manuscript.

      On page 2, lines 71-75, the authors' review of the Frassle et al. (2014) experiment should be revised for accuracy. In this study, all PFC activity did not disappear as the authors claim. Also, the main contrast in the Frassle et al. study was rivalry vs. replay. However, in both of these conditions, visual awareness was changing with the main difference being whether there was sensory conflict between the two eyes or not. Such a contrast would presumably subtract out the common activity patterns related to visual awareness changes, while isolating rivalry (and the resulting neural competition) vs. non-rivalry (and the lack of such competition) which is not broadly relevant for the goal of measuring neural correlates of visual awareness which are present in both sides of the contrast (rivalry and replay).

      Thank you very much for your suggestion. We agree that and revised in the MS (lines 71-76).

      ‘For instance, a functional magnetic resonance imaging (fMRI) study employing human binocular rivalry paradigms found that when subjects need to manually report the changing of their awareness between conflict visual stimuli, the frontal, parietal, and occipital lobes all exhibited awareness-related activity. However, when report was not required, awareness-related activation was largely diminished in the frontal lobe but remained in the occipital and parietal lobes’

      On page 2, lines 76-78, the authors write, "no-report paradigm may overestimate unconscious processing because it cannot directly measure the awareness state". This should be reworded for clarity, as report paradigms also do not "directly measure the awareness state". All measures of awareness are indirect, either via subjects verbal or manual reports, or via behaviors or other physiological measures like OKN, pupillometry, etc. It's also not clear as written why no-report paradigms might overestimate unconscious processing.

      Thank you very much for your suggestion. We agreed and modified the description. In lines 76-80:

      ‘Nevertheless, the no-report paradigm may overestimate the neural correlates of awareness by including unconscious processing, because it infers the awareness state through other relevant physiological indicators, such as optokinetic nystagmus and pupil size(Tsuchiya, Wilke, Frassle, & Lamme, 2015). In the absence of subjective reports, it remains controversial regarding whether the presented stimuli are truly seen or not.’

      However, the no-report paradigm may overestimate the neural correlates of awareness, because it infers the awareness state through other relevant physiological indicators, such as optokinetic nystagmus and pupil size(Tsuchiya et al., 2015) , in the absence of subjective reports and it remains controversial that whether the stimuli presented in such paradigm are truly seen as opposed to being merely potentially visible but unattended.

      On page 5, line 155, there is a typo. This should be Figure 2C, not 2B.

      Thanks. We have modified the description.

      On page 5, lines 160-162, the authors state, "The results showed that the saccadic reaction time in the aware trials was systematically shorter than that in the unaware trials. Such results demonstrate that visual awareness significantly affects the speed of information processing in the brain." I don't understand this. If subjects can never make a saccade until the fixation cross changes color, both for Y and N decisions, why would a difference in saccadic reaction times indicate anything about visual awareness affecting the speed of information processing in the brain? Doesn't this just show that the Red/Green x Left/Right response contingencies were easier to remember and execute for the Yes-I-did-see-it decisions compared to the No-I-didn't-see-it decisions?

      We agree and have made additional discussion about these questions in the revised manuscript (lines 492-496).

      ‘An alternative interpretation for RT difference between aware and unaware condition in our study is that the difference in task-strategies used by subjects/patients to remember the response mapping rules between the perception and the color cue (e.g., if the YES+GREEN=RIGHT and YES+RED=LEFT rules were held in memory, while the NO mappings were inferred secondarily rather than being actively held in memory).’

      In Figure 3B (and several other figures) due to the chosen view and particular brain visualization used, many readers will not know whether the front of brain is up and back of brain down or vise versa (there are no obvious landmarks like the cerebellum, temporal sulcus, etc.). I suggest specifying this in the caption or better yet on the figure itself.

      Thanks. We have added these descriptions in the caption of Figure 2D.

      Line 189 ‘In all brain images, right and up sides of each image represent the right and up sides of the brain’.

      In Figure 3B, the color scale may confuse some readers. When I first inspected this figure, I immediately thought the red meant positive voltage or activation, while the blue meant negative voltage or deactivation. Only later, I realized that any color here is meaningful. Not sure if an adjustment of the color scale might help, or perhaps not normalizing (and not taking absolute values of the voltage diffs, but maintaining the +/- diffs)?

      Thanks for reviewer’s comment. We are sorry for not clearly describing the reason why we normalized the activity in absolute value and chose the color scale from 0 to 20. The major reason is that it is not clearly understood so far regarding the biological characteristics of LFP polarity (Einevoll et al, Nat Rev Neurosci, 2013). To simplify such complex issue, we consider the change in magnitude of LFP during delay period in our task represents awareness related activity, regardless its actual value being positive or negative. Therefore, we first calculated the absolute value of activity difference between aware and unaware trials in individual recording site, then used Shepard's method (see Method for detailed information) to calculate the activity in each vertex and projected on the surface of brain template as shown in Fig. 3B.

      We have added the description in the MS (lines 794-800).

      We have tried to adjust the color scale from -20 to 20 according to reviewer’s suggestion. However, the topographic heatmap showed less distinguishable between brain regions with different strength of awareness related activity. Thus, we would like to keep the way as we used to analyze and present these results.

      Figure 3B: Why choose seemingly arbitrary time points in this figure? What's the significance of 247 and 314 and 381ms (why not show 200, 250, 300, etc.)? Also, are these single time-points or averages within a broader time window around this time-point, e.g., 225-275ms for the 250ms plot?

      Thank reviewer for this helpful comment. We are sorry for not clearly describing why we chose the 8 time points to demonstrate the spatiotemporal characteristics of awareness related activity in Fig. 3B. To identify the awareness related activity, we analyzed the activity difference between aware and unaware trials during delay period (180-650 ms after visual stimulus onset). The whole dynamic process has been presented in SI with a video (video S1). Here, we just sampled the activity at 8 time points (180 ms, 247 ms, 314 ms, etc.) that equally divided the 430 ms delay period.

      We have added the description in the MS (lines 213-215).

      Figure 3D: It's not clear how this figure panel is related to the data shown in Fig3A. In Fig3A, the positive amplitude diffs all end at around 400ms, but in Fig3D, these diffs extend out to 600+ms. I suggest adding clarity about the conversion being used here.

      Thanks for reviewer’s comment. We are sorry for not clearly describing the way to analyze the population activity (Fig. 3D) in the previous version of manuscript. Since it is not clearly understood so far regarding the biological characteristics of LFP polarity, to simplify such complex issue, we consider the change in magnitude of LFP during delay period in our task is awareness related activity, regardless its actual value being positive or negative. Therefore, while analyzing the awareness related population activity, we first calculate the absolute value of activity difference between aware and unaware trials in individual recording site, then pool the data of 43 recording sites together and calculate the mean and standard error of mean (SEM)(Fig. 3D). As you can see in Fig. 3A, the activity difference between aware (red) and unaware (blue) trials lasts until/after the end of delay period. Thus, the awareness related population activity in Fig 3D extends out to 600 ms.

      We have added the description in the MS (lines 769-777).

      Figure 6D could be improved by making the time labels much bigger, perhaps putting them on the time axis on the bottom rather than in tiny text above each brain.

      Thanks for reviewer’s comment. We have modified it accordingly.

      Page 18, line 480: "our results show that the prefrontal cortex still displays visual awareness-related activities even after eliminating the influence of the confounding variables related to subjective reports such as motion preparation" This is too strong of a statement. It's not at all clear whether confounding variables related to subjective reports (especially the cognition needed to hold in mind the Y/N decision about seeing the stimulus prior to the response cue) were eliminated with the design used here. In other places of the manuscript, the authors use "minimized" which is more accurate.

      Thanks for reviewer’s comment. We have modified it accordingly.

      Page 19, section starting on line 508: The authors should consider citing the study by Vishne et al. (2023), which was just accepted for publication recently, but has been posted on bioRxiv for almost a year now: https://www.biorxiv.org/content/10.1101/2022.08.02.502469v1 . And on page 20, line 563, the authors claim that to the best of their knowledge, they were the first to detect "ignition" in PFC in human subjects. Consider revising this statement, now that you know about the Vishne et al. paper.

      We agree.

      Thanks for your reminding about these papers. We have cited this study and made discussion in the revised manuscript (line 522-533). We agree that several iEEG studies have shown the early involvement of PFC in visual perception (Vishne et al. 2023; Khalaf et al. 2023; Kwon et al. 2021). However, in these studies, authors did not compare the neural activity between conscious and unconscious conditions, leaving the possibility that the ERP and HFA were correlated with the unconscious information processing rather than awareness-specific processing. In the present study, we compared the neural activity in PFC between conscious and unconscious trials, and found that the activity of PFC specifically correlated with conscious perception. As we mentioned in the previous version of manuscript, there is one iEEG study (Gaillard et al. 2009) that reported awareness-specific activity in PFC. However, the awareness related activity started more than 300 ms after the onset of visual stimuli, which was about 100 ms longer than the early awareness related activity in our study. Nevertheless, according to reviewer’s comment, we modified our argument as following in lines 621-623:

      ‘However, as discussed above, in contrast with previous studies, our study detected earlier awareness-specific ‘ignition’ in the human PFC, while minimizing the motor-related confounding.’

      Experimental task section of Methods: Were any strategies for learning the response cue matching task suggested to patients/subjects, and/or did any patients/subjects report which strategy they ended up using? For example, if I were a subject in this experiment, I would remember and mentally rehearse the rules: "YES+GREEN = RIGHT" and "YES+RED = LEFT". For trials in which I didn't see anything, I wouldn't need to hold 2 more rules in mind, as they can be inferred from the inverse of the YES rules (and it's much harder to hold 4 things in mind than 2). This extra inference needed to get to the NO+GREEN = LEFT and NO+RED = RIGHT rules would likely cause me to respond slightly slower to the NO trials compared to the YES trials, leading to saccadic RT effects in the same direction the authors found. More information about the task training and strategies used by patients/subjects would be helpful.

      We agree and discussed this in lines 492-496.

      Reviewer #3 (Public Review):

      The authors report a study in which they use intracranial recordings to dissociate subjectively aware and subjectively unaware stimuli, focusing mainly on prefrontal cortex. Although this paper reports some interesting findings (the videos are very nice and informative!) the interpretation of the data is unfortunately problematic for several reasons. I will detail my main comments below. If the authors address these comments well, I believe the paper may provide an interesting contribution to further specifying the neural mechanisms important for conscious access (in line with Gaillard et al., Plos Biology 2009).

      Reply: We appreciate very much for the reviewer’s encouraged opinion.

      The main problem with the interpretation of the data is that the authors have NOT used a so called "no-report paradigm". The idea of no report paradigms is that subjects passively view a certain stimulus without the instruction to "do something with it", e.g., detect the stimulus, immediately or later in time. Because of the confusion of this term, specifically being related to the "act of reporting", some have argued we should use the term no-cognition paradigm instead (Block, TiCS, 2019, see also Pitts et al., Phil Trans B 2018). The crucial aspect is that, in these types of paradigms, the critical stimulus should be task-irrelevant and thus not be associated with any task (immediately or later). Because in this experiment subjects were instructed to detect the gratings when cued 600 ms later in time, the stimuli are task relevant, they have to be reported about later and therefore trigger all kinds of (known and potentially unknown) cognitive processes at the moment the stimuli are detected in real-time (so stimulus-locked). You could argue that the setup of this delayed response task excludes some very specific report related processes (e.g., the preparation of an eye-movement), which is good, however this is usually not considered the main issue. For example when comparing masked versus unmasked stimuli (Gaillard et al., 2009 Plos Biology), these conditions usually also both contain responses but these response related processes are "averaged out" in the specific contrasts (unmasked > masked). In this paper, RT differences between conditions (that are present in this dataset) are taken care of by using this delayed response in this paper, which is a nice feature for that and is not the case for the above example set-up.

      Given the task instructions, and this being merely a delayed-response task, it is to be expected that prefrontal cortex shows stronger activity for subjectively aware versus subjectively unaware stimuli. Unfortunately, given the nature of this task, the novelty of the findings is severely reduced. The authors cannot claim that prefrontal cortex is associated with "visual awareness", or what people have called phenomenal consciousness (this is the goal of using no-cognition paradigms). The only conclusion that can be drawn is that prefrontal cortex activity is associated with accessing sensory input: and hence conscious access. This less novel observation has been shown many times before and there is also little disagreement about this issue between different theories of consciousness (e.g., global workspace theory and local recurrency theories both agree on this).

      We totally agree that the no-report/no-cognition paradigms contain less cognition within the post-perceptual processing than the report paradigms. We designed the balanced response task in order to minimize the motor related component from post-perceptual processing, even though this task does not eliminate the entire cognition from post-perceptual processing. Regarding reviewer’s comment that our task is not able to assess the involvement of PFC in the emergence of awareness, we have different opinion. As we mentioned in the manuscript, the findings of early awareness related activity (~200 ms) in PFC, which resemble the VAN activity in EEG studies, indicate the association of PFC with the emergence of visual awareness (phenomenal consciousness).

      The best solution at this point seems to rewrite the paper entirely in light of this. My advice would be to state in the introduction that the authors investigate conscious access using iEEG and then not refer too much to no-cognition paradigm or maybe highlight some different strategies about using task-irrelevant stimuli (see Canales-Johnson et al., Plos Biology 2023; Hesse et al., eLife 2020; Hatamimajoumerd et al Curr Bio 2022; Alilovic et al., Plos Biology 2023; Pitts et al., Frontiers 2014; Dwarakanth et al., Neuron 2023 and more). Obviously, the authors should then also not claim that their results solve debates about theories regarding visual awareness (in the "no-cognition" sense, or phenomenal consciousness), for example in relation to the debate about the "front or the back of the brain", because the data do not inform that discussion. Basically, the authors can just discuss their results in detail (related to timing, frequency, synchronization etc) and relate the different signatures that they have observed to conscious access.

      The objective of present study is to assess whether PFC is involved in the emergence of visual awareness (i.e., phenomenal consciousness). Interestingly, we found the early awareness related activity (~200 ms after visual stimulus onset), including ERP, high gamma activity and phase synchronization, in PFC, which indicate the association of PFC with the emergence of visual awareness. Therefore, we would like to keep the basic context of manuscript and make revision according to reviewers’ comments.

      On the other hand, we totally agree reviewer’s argument that the report paradigm is more suitable to study the access consciousness. Indeed, we have found that the awareness related activity in PFC could be separated into two subgroups, i.e., early activity with shorter latency (~200 ms after stimulus onset) and late activity with longer latency (> 350 ms after stimulus onset). In addition, the early activity was declined to the baseline level within ~200 ms during delay period, whereas the late activity lasted throughout the delay period and reached to the next stage of task (change color of the fixation point). Moreover, the early activity occurs primarily within the contralateral PFC of the visual stimulus, whereas the late activity occurs within both contralateral and ipsilateral PFC. While the early awareness related activity resembles the VAN activity in EEG studies (associating with p-consciousness), the late awareness related activity resembles the P3b activity (associating with a-consciousness). We are going to report these results in a separated paper soon.

      I think the authors have to discuss the Gaillard et al PLOS Biology 2009 paper in much more detail. Gaillard et al also report a study related to conscious access contrasting unmasked and masked stimuli using iEEG. In this paper they also report ERP, time frequency and phase synchronization results (and even Granger causality). Because of the similarities in approach, I think it would be important to directly compare the results presented in that paper with results presented here and highlight the commonalities and discrepancies in the Discussion.

      Thanks for reviewer’s comment. We have made additional analysis and detailed discussion accordingly. In addition, we also extended discussion with other relevant studies in the revised manuscript.

      In lines 528-549,

      ‘Although one iEEG study reported awareness-specific PFC activation, the awareness-related activity started 300 ms after the onset of visual stimuli, which was ~100 ms later than the early activity in our study. Also, due to the limited number of electrodes in PFC (2 patients with 19 recording sites mostly in mesiofrontal and peri-insular regions), their experiments were restricted while exploring the awareness-related activity in PFC. In the present study, the number of recording sites (245) were much more than previous study and covered more areas in PFC. Our results further show earlier awareness-related activity (~ 200 ms after visual stimuli onset), including ERP, HFA and PLV. These awareness-related activity in PFC occurred even earlier (~150 ms after stimulus onset) for the salient stimulus trials (Fig. 3A\D and Fig. 4A\D, HA condition).

      However, the proportions are much smaller than that reported by Gaillard et al, which peaked at ~60%. We think that one possibility for the difference may be due to the more sampled PFC subregions in present study and the uneven distribution of awareness-related activity in PFC. Meanwhile, we noticed that the peri-insula regions and middle frontal gyrus (MFG), which were similar with the regions reported by Gaillard et al, seemed to show more fraction of awarenessrelated sites than other subregions during the delay period (0-650 ms after stimulus onset). To test such possibility and make comparison with the study of Gaillard et al. we calculated the proportion of awareness-related site in peri-insula and MFG regions. We found although the proportion of awareness-related site was larger in peri-insula and MFG than in other subregions, it was much lower than the report of Gaillard et al. One alternative possibility for the difference between these two studies might be due to the more complex task in Gaillard et al. Nevertheless, we think these new results would contribute to our understanding of the neural mechanism underlying conscious perception, especially for the role of PFC.’ In lines 601-603:

      ‘The only human iEEG study reported that the phase synchronization of the beta band in the aware condition also occurred relatively late (> 300 ms) and mainly confined to posterior zones but not PFC.’

      As for the Granger Causality analysis between PFC and occipital lobe, while the aim of this study focused mainly on PFC and there were few recoding sites in occipital lobe, we would like to do this analysis in later studies after we collect more data.

      In the Gaillard paper they report a figure plotting the percentage of significant frontal electrodes across time (figure 4A) in which it can be seen that significant electrodes emerge after approximately 250 ms in PFC as well. It would be great if the authors could make a similar figure to compare results. In the current paper there are much more frontal electrode contacts than in the Gaillard paper, so that is interesting in itself.

      Thanks reviewer for this constructive comment. We made similar analysis as Gaillard et al. and plotted the results in the figure bellow. As you can see, the awareness related sites started to emerge about 200 ms after visual stimulus onset according to both ERP and HG activity. The proportion of awareness related sites reached peak at ~14% (8% for HG) in 300-400ms. However, the proportions are much smaller than that reported by Gaillard et al, which peaked at ~60%. We think that one possibility for the difference may be due to the more sampled PFC subregions in present study and the uneven distribution of awareness-related activity in PFC. Meanwhile, we noticed that the peri-insula regions and middle frontal gyrus (MFG), which were similar with the regions reported by Gaillard et al, seemed to show more fraction of awareness-related sites than other subregions during the delay period (0-650 ms after stimulus onset). To test such possibility and make comparison with the study of Gaillard et al. we calculated the proportion of awareness-related site in peri-insula and MFG regions. We found although the proportion of awareness-related site was larger in peri-insula and MFG than in other subregions, it was much lower than the report of Gaillard et al. One alternative possibility for the difference between these two studies might be due to the more complex task in Gaillard et al.

      We have added this figure and discussion to the revised manuscript as a new result (Figure 4E & S2 and lines 537-549).

      Author response image 1.

      Percentage of awareness-related sites in ERP and HG analysis. n, number of recording sites in PFC.

      Author response image 2.

      Percentage of awareness-related sites in ERP and HG analysis at parsopercularis and middle frontal gyrus (MFG). n, number of recording sites.

      In my opinion, some of the most interesting results are not highlighted: the findings that subjectively unaware stimuli show increased activations in the prefrontal cortex as compared to stimulus absent trials (e.g., Figure 4D). Previous work has shown PFC activations to masked stimuli (e.g., van Gaal et al., J Neuroscience 2008, 2010; Lau and Passigngham J Neurosci 2007) as well as PFC activations to subjectively unaware stimuli (e.g., King, Pescetelli, and Dehaene, Neuron 2016) and this is a very nice illustration of that with methods having more detailed spatial precision. Although potentially interesting, I wonder about the objective detection performance of the stimuli in this task. So please report objective detection performance for the patients and the healthy subjects, using signal detection theoretic d'. This gives the reader an idea of how good subjects were in detecting the presence/absence of the gratings. Likely, this reveals far above chance detection performance and in that case I would interpret these findings as "PFC activation to stimuli indicated as subjectively unaware" and not unconscious stimuli. See Stein et al., Plos Biology 2021 for a direct comparison of subjectively and objectively unaware stimuli.

      We gratefully appreciate for reviewer’s helpful and valuable comments. We do notice that the activity of PFC in subjectively unawareness condition (stimulus contrast near perceptual threshold) is significantly higher than stimulus absent condition. Such results, by using sEEG recordings with much higher spatial resolution than brain imaging and scalp EEG, support findings of previous studies (citations). Considering the question of neural correlation of unawareness processing is a hot and interesting topic, after carefully considering, we would like to report these results in a separate paper, rather than add these results in the current manuscript in order to avoid the distraction.

      According to reviewer’s comment about the objective detection performance of the stimuli in our task, we analyzed the signal detection theoretic d’. The values of d’ in patients and healthy subjects are similar (1.81±0.27 in patients and 2.12±0.37 in healthy subjects). Such results indicate that the objective detection performance of subjects in our task is well above the chance level. Since our task merely measures the subjective awareness, we agree reviewer’s comment about the interpretation of our results as “PFC activation to stimuli indicated the subjective unawareness rather than objective unawareness”. We will emphasize this point in our next paper.

      We have added the d prime in the MS (lines149-150).

      In Figure 7 of the paper the authors want to make the case that the contrast does not differ between subjectively aware stimuli and subjectively unaware stimuli. However so far they've done the majority of their analyses across subjects, and for this analysis the authors only performed within-subject tests, which is not a fair comparison imo. Because several P values are very close to significance I anticipate that a test across subjects will clearly show that the contrast level of the subjectively aware stimuli is higher than of the subjectively unaware stimuli, at the group level. A solution to this would be to sub-select trials from one condition (NA) to match the contrast of the other condition (NU), and thereby create two conditions that are matched in contrast levels of the stimuli included. Then do all the analyses on the matched conditions.

      Thank reviewer for the helpful comment. Regarding reviewer’s comment “However so far they've done the majority of their analyses across subjects, and for this analysis the authors only performed within-subject tests, which is not a fair comparison imo”, if we understand correctly, reviewer considered that it was fair if the analysis of neural activity in PFC was done across subjects but the stimulus contrast analysis between NA and NU was done individually. Actually, it is not the case. In neural activity analysis, the significant awareness-related sites were identified firstly in each individual subject (Fig. 3A and Fig 4A, and Methods), same as the analysis of stimulus contrast (see Methods). Only in the neural population activity analysis, the activity of awareness-related sites was pooled together and made further analysis.

      To further evidence the awareness related activity in PFC is not highly correlated with stimulus contrast, we compared the activity difference between two different stimulus contrast conditions, i.e., stimulus contrast difference between high-contrast aware (HA) and NA conditions (large difference, ~14%), and between NA and NU conditions (slight difference, ~0.2%). The working hypothesis is that, if PFC activity is closely correlated with the contrast of stimulus contrast, we expect to see the activity difference between HA and NA conditions is much larger than that between NA and NU conditions. To test this hypothesis, we analyzed data of two patients in which the previous analysis showed significant or near significant difference of stimulus contrast between NA and NU conditions (Author response image 1, below, patient #2 and 1). The results (Author response image 1) show that the averaged activity difference (0-650 ms after visual stimulus onset) between HA and NA was similar as the averaged activity difference between NA and NU trials, even though the stimulus contrast difference was much larger between HA and NA conditions than between NA and NU conditions. Such results indicate that the awareness-related activity in PFC cannot be solely explained by the contrast difference between NA and NU conditions. Based on these results, we think that it is not necessary to perform the analysis as reviewer’s comment “A solution to this would be to sub-select trials from one condition (NA) to match the contrast of the other condition (NU), and thereby create two conditions that are matched in contrast levels of the stimuli included. Then do all the analyses on the matched conditions”. Another reason that impedes us to do this analysis is due to the limited trial numbers in our dataset.

      Author response image 3.

      Relationship between stimulus contract and PFC activity. X axis represents the stimulus contrast difference between two paired conditions, i.e., aware versus unaware in near perceptual threshold conditions (NA – NU, red dots); aware in high contrast condition versus aware in near perceptual threshold condition (HA – NA, blue dots). Y axis represents the activity difference between paired stimulus conditions. The results show that activity difference is similar between two paired conditions regardless the remarkable contrast difference between two paired conditions. Such results indicate that the greater activity in NA trials than in NU trials (Fig. xx-xx) could not be interpreted by the slight difference in stimulus contrast between NA and NU trials.

      Related, Figure 7B is confusing and the results are puzzling. Why is there such a strong below chance decoding on the diagonal? (also even before stimulus onset) Please clarify the goal and approach of this analysis and also discuss/explain better what they mean.

      We have withdrawn Figure7B for the confusing decoding results on the diagonal.

      I was somewhat surprised by several statements in the paper and it felt that the authors may not be aware of several intricacies in the field of consciousness. For example, a statement like the following "Consciousness, as a high-level cognitive function of the brain, should have some similar effects as other cognitive functions on behavior (for example, saccadic reaction time). With this question in mind, we carefully searched the literature about the relationship between consciousness and behavior; surprisingly, we failed to find any relevant literature." This is rather problematic for at least two reasons. First, not everyone would agree that consciousness is a highlevel cognitive function and second there are many papers arguing for a certain relationship between consciousness and behavior (Dehaene and Naccache, 2001 Cognition; van Gaal et al., 2012, Frontiers in Neuroscience; Block 1995, BBS; Lamme, Frontiers in Psychology, 2020; Seth, 2008 and many more). Further, the explanation for the reaction time differences in this specific case is likely related to the fact that subjects' confidence in that decision is much higher in the aware trials than in the unaware trials, hence the speeded response for the first. This is a phenomenon that is often observed if one explores the "confidence literature". Although the authors have not measured confidence I would not make too much out of this RT difference.

      We agree that and modified accordingly in lines 492-507.

      ‘An alternative interpretation for RT difference between aware and unaware condition in our study, i.e., reflecting task-strategies used by subjects/patients to remember the response mapping rules between the perception and the color cue (e.g., if the YES+GREEN=RIGHT and YES+RED=LEFT rules were held in memory, while the NO mappings were inferred secondarily rather than being actively held in memory).

      Another possibility is that the reaction time is strongly modulated by the confident level, which has been described in previous studies(Broggin et al., 2012; Marzi et al., 2006). However, in previous studies, the confident levels were usually induced by presenting stimulus with different physical property, such as spatial frequency, eccentricity and contrast. However, the dependence of visual process on the salience of visual stimulus confounds with the effect of visual awareness on the reaction time of responsive movements, which is hard to attribute the shorter reaction time in more salient condition purely to visual awareness. In contrast, we create a condition (near aware threshold) in the present study, in which the saliency (contrast) of visual stimulus is very similar in both aware and unaware conditions in order to eliminate the influence of stimulus saliency in reaction time. We think that the difference in reaction time in our study is mainly due to the modulation of awareness state, which was not reported previously.’

      I would be interested in a lateralized analysis, in which the authors compare the PFC responses and connectivity profiles using PLV as a factor of stimulus location (thus comparing electrodes contralateral to the presented stimulus and electrodes ipsilateral to the presented stimulus). If possible this may give interesting insights in the mechanism of global ignition (global broadcasting), supposing that for contralateral electrodes information does not have to cross from one hemisphere to another, whereas for ipsilateral electrodes that is the case (which may take time). Gaillard et al refer to this issue as well in their paper, and this issue is sometimes discussed regarding to Global workspace theory. This would add novelty to the findings of the paper in my opinion.

      We gratefully appreciate reviewer’s helpful and available suggestions. We have made the analysis accordingly. We find that the awareness-related ERP activation in PFC occurs earlier only in the contralateral PFC with latency about 200 ms and then occurs in both contralateral and ipsilateral PFC about 100 ms later. In addition, the magnitude of awareness-related activity is stronger in the contralateral PFC than in ipsilateral PFC during the early phase (200-400 ms), then the activity becomes similar between contralateral and ipsilateral PFC. Moreover, the awareness related HG activity only appears in the contralateral PFC. Such results show the spatiotemporal characteristics of visual awareness related activity between two hemispheres. We are going to report these results in a separate paper soon.

      Reviewer #3 (Recommendations For The Authors):

      Some of the font sizes in the figures are too small.

      We have modified accordingly.

      To me, the abbreviations are confusing, (NA/NU etc). I would try to come up with easier ones or just not use abbreviations.

      We have modified accordingly and try to avoid to use the abbreviations.

      The data/scripts availability statement states "available upon reasonable request". I would suggest that the authors make the data openly available when possible, and I believe eLife requires that as well.

      Thanks for reviewer’s suggestions. Due to several ongoing studies based on this dataset, we would like to open our data after complete these studies if there is no restriction from national policy.

    1. Author Response

      The following is the authors’ response to the original reviews.

      Public Comments

      Reviewer 1

      (1) Despite the well-established role of Netrin-1 and UNC5C axon guidance during embryonic commissural axons, it remains unclear which cell type(s) express Netrin-1 or UNC5C in the dopaminergic axons and their targets. For instance, the data in Figure 1F-G and Figure 2 are quite confusing. Does Netrin-1 or UNC5C express in all cell types or only dopamine-positive neurons in these two mouse models? It will also be important to provide quantitative assessments of UNC5C expression in dopaminergic axons at different ages.

      Netrin-1 is a secreted protein and in this manuscript we did not examine what cell types express Netrin-1. This question is not the focus of the study and we consider it irrelevant to the main issue we are addressing, which is where in the forebrain regions we examined Netrin-1+ cells are present. As per the reviewer’s request we include below images showing Netrin-1 protein and Netrin-1 mRNA expression in the forebrain. In Figure 1 below, we show a high magnification immunofluorescent image of a coronal forebrain section showing Netrin-1 protein expression.

      Author response image 1.

      This confocal microscope image shows immunofluorescent staining for Netrin-1 (green) localized around cell nuclei (stained by DAPI in blue). This image was taken from a coronal section of the lateral septum of an adult male mouse. Scale bar = 20µm

      In Figures 2 and 3 below we show low and high magnification images from an RNAscope experiment confirming that cells in the forebrain regions examined express Netrin-1 mRNA.

      Author response image 2.

      This confocal microscope image of a coronal brain section of the medial prefrontal cortex of an adult male mouse shows Netrin-1 mRNA expression (green) and cell nuclei (DAPI, blue). Brain regions are as follows: Cg1: Anterior cingulate cortex 1, DP: dorsopeduncular cortex, fmi: forceps minor of the corpus callosum, IL: Infralimbic Cortex, PrL: Prelimbic Cortex

      Author response image 3.

      A higher resolution image from the same sample as in Figure 2 shows Netrin-1 mRNA (green) and cell nuclei (DAPI; blue). DP = dorsopeduncular cortex

      Regarding UNC5c, this receptor homologue is expressed by dopamine neurons in the rodent ventral tegmental area (Daubaras et al., 2014; Manitt et al., 2010; Phillips et al., 2022). This does not preclude UNC5c expression in other cell types. UNC5c receptors are ubiquitously expressed in the brain throughout development, performing many different developmental functions (Kim and Ackerman, 2011; Murcia-Belmonte et al., 2019; Srivatsa et al., 2014). In this study we are interested in UNC5c expression by dopamine neurons, and particularly by their axons projecting to the nucleus accumbens. We therefore used immunofluorescent staining in the nucleus accumbens, showing UNC5 expression in TH+ axons. This work adds to the study by Manitt et al., 2010, which examined UNC5 expression in the VTA. Manitt et al. used Western blotting to demonstrate that UNC5 expression in VTA dopamine neurons increases during adolescence, as can be seen in the following figure:

      References:

      Daubaras M, Bo GD, Flores C. 2014. Target-dependent expression of the netrin-1 receptor, UNC5C, in projection neurons of the ventral tegmental area. Neuroscience 260:36–46. doi:10.1016/j.neuroscience.2013.12.007

      Kim D, Ackerman SL. 2011. The UNC5C Netrin Receptor Regulates Dorsal Guidance of Mouse Hindbrain Axons. J Neurosci 31:2167–2179. doi:10.1523/jneurosci.5254-10.20110.2011

      Manitt C, Labelle-Dumais C, Eng C, Grant A, Mimee A, Stroh T, Flores C. 2010. Peri-Pubertal Emergence of UNC-5 Homologue Expression by Dopamine Neurons in Rodents. PLoS ONE 5:e11463-14. doi:10.1371/journal.pone.0011463

      Murcia-Belmonte V, Coca Y, Vegar C, Negueruela S, Romero C de J, Valiño AJ, Sala S, DaSilva R, Kania A, Borrell V, Martinez LM, Erskine L, Herrera E. 2019. A Retino-retinal Projection Guided by Unc5c Emerged in Species with Retinal Waves. Current Biology 29:1149-1160.e4. doi:10.1016/j.cub.2019.02.052

      Phillips RA, Tuscher JJ, Black SL, Andraka E, Fitzgerald ND, Ianov L, Day JJ. 2022. An atlas of transcriptionally defined cell populations in the rat ventral tegmental area. Cell Reports 39:110616. doi:10.1016/j.celrep.2022.110616

      Srivatsa S, Parthasarathy S, Britanova O, Bormuth I, Donahoo A-L, Ackerman SL, Richards LJ, Tarabykin V. 2014. Unc5C and DCC act downstream of Ctip2 and Satb2 and contribute to corpus callosum formation. Nat Commun 5:3708. doi:10.1038/ncomms4708

      (2) Figure 1 used shRNA to knockdown Netrin-1 in the Septum and these mice were subjected to behavioral testing. These results, again, are not supported by any valid data that the knockdown approach actually worked in dopaminergic axons. It is also unclear whether knocking down Netrin-1 in the septum will re-route dopaminergic axons or lead to cell death in the dopaminergic neurons in the substantia nigra pars compacta?

      First we want to clarify and emphasize, that our knockdown approach was not designed to knock down Netrin-1 in dopamine neurons or their axons. Our goal was to knock down Netrin-1 expression in cells expressing this guidance cue gene in the dorsal peduncular cortex.

      We have previously established the efficacy of the shRNA Netrin-1 knockdown virus used in this experiment for reducing the expression of Netrin-1 (Cuesta et al., 2020). The shRNA reduces Netrin-1 levels in vitro and in vivo.

      We agree that our experiments do not address the fate of the dopamine axons that are misrouted away from the medial prefrontal cortex. This research is ongoing, and we have now added a note regarding this to our manuscript.

      Our current hypothesis, based on experiments being conducted as part of another line of research in the lab, is that these axons are rerouted to a different brain region which they then ectopically innervate. In these experiments we are finding that male mice exposed to tetrahydrocannabinol in adolescence show reduced dopamine innervation in the medial prefrontal cortex in adulthood but increased dopamine input in the orbitofrontal cortex. In addition, these mice show increased action impulsivity in the Go/No-Go task in adulthood (Capolicchio et al., Society for Neuroscience 2023 Abstracts)

      References:

      Capolicchio T., Hernandez, G., Dube, E., Estrada, K., Giroux, M., Flores, C. (2023) Divergent outcomes of delta 9 - tetrahydrocannabinol in adolescence on dopamine and cognitive development in male and female mice. Society for Neuroscience, Washington, DC, United States [abstract].

      Cuesta S, Nouel D, Reynolds LM, Morgunova A, Torres-Berrío A, White A, Hernandez G, Cooper HM, Flores C. 2020. Dopamine Axon Targeting in the Nucleus Accumbens in Adolescence Requires Netrin-1. Frontiers Cell Dev Biology 8:487. doi:10.3389/fcell.2020.00487

      (3) Another issue with Figure1J. It is unclear whether the viruses were injected into a WT mouse model or into a Cre-mouse model driven by a promoter specifically expresses in dorsal peduncular cortex? The authors should provide evidence that Netrin-1 mRNA and proteins are indeed significantly reduced. The authors should address the anatomic results of the area of virus diffusion to confirm the virus specifically infected the cells in dorsal peduncular cortex.

      All the virus knockdown experiments were conducted in wild type mice, we added this information to Figure 1k.

      The efficacy of the shRNA in knocking down Netrin-1 was demonstrated by Cuesta et al. (2020) both in vitro and in vivo, as we show in our response to the reviewer’s previous comment above.

      We also now provide anatomical images demonstrating the localization of the injection and area of virus diffusion in the mouse forebrain. In Author response image 4 below the area of virus diffusion is visible as green fluorescent signal.

      Author response image 4.

      Fluorescent microscopy image of a mouse forebrain demonstrating the localization of the injection of a virus to knock down Netrin-1. The location of the virus is in green, while cell nuclei are in blue (DAPI). Abbreviations: DP: dorsopeduncular cortex IL: infralimbic cortex

      References:

      Cuesta S, Nouel D, Reynolds LM, Morgunova A, Torres-Berrío A, White A, Hernandez G, Cooper HM, Flores C. 2020. Dopamine Axon Targeting in the Nucleus Accumbens in Adolescence Requires Netrin-1. Frontiers Cell Dev Biology 8:487. doi:10.3389/fcell.2020.00487

      (4) The authors need to provide information regarding the efficiency and duration of knocking down. For instance, in Figure 1K, the mice were tested after 53 days post injection, can the virus activity in the brain last for such a long time?

      In our study we are interested in the role of Netrin-1 expression in the guidance of dopamine axons from the nucleus accumbens to the medial prefrontal cortex. The critical window for these axons leaving the nucleus accumbens and growing to the cortex is early adolescence (Reynolds et al., 2018b). This is why we injected the virus at the onset of adolescence, at postnatal day 21. As dopamine axons grow from the nucleus accumbens to the prefrontal cortex, they pass through the dorsal peduncular cortex. We disrupted Netrin-1 expression at this point along their route to determine whether it is the Netrin-1 present along their route that guides these axons to the prefrontal cortex. We hypothesized that the shRNA Netrin-1 virus would disrupt the growth of the dopamine axons, reducing the number of axons that reach the prefrontal cortex and therefore the number of axons that innervate this region in adulthood.

      We conducted our behavioural tests during adulthood, after the critical window during which dopamine axon growth occurs, so as to observe the enduring behavioral consequences of this misrouting. This experimental approach is designed for the shRNa Netrin-1 virus to be expressed in cells in the dorsopeduncular cortex when the dopamine axons are growing, during adolescence.

      References:

      Capolicchio T., Hernandez, G., Dube, E., Estrada, K., Giroux, M., Flores, C. (2023) Divergent outcomes of delta 9 - tetrahydrocannabinol in adolescence on dopamine and cognitive development in male and female mice. Society for Neuroscience, Washington, DC, United States [abstract].

      Reynolds LM, Yetnikoff L, Pokinko M, Wodzinski M, Epelbaum JG, Lambert LC, Cossette M-P, Arvanitogiannis A, Flores C. 2018b. Early Adolescence is a Critical Period for the Maturation of Inhibitory Behavior. Cerebral cortex 29:3676–3686. doi:10.1093/cercor/bhy247

      (5) In Figure 1N-Q, silencing Netrin-1 results in less DA axons targeting to infralimbic cortex, but why the Netrin-1 knocking down mice revealed the improved behavior?

      This is indeed an intriguing finding, and we have now added a mention of it to our manuscript. We have demonstrated that misrouting dopamine axons away from the medial prefrontal cortex during adolescence alters behaviour, but why this improves their action impulsivity ability is something currently unknown to us. One potential answer is that the dopamine axons are misrouted to a different brain region that is also involved in controlling impulsive behaviour, perhaps the dorsal striatum (Kim and Im, 2019) or the orbital prefrontal cortex (Jonker et al., 2015).

      We would also like to note that we are finding that other manipulations that appear to reroute dopamine axons to unintended targets can lead to reduced action impulsivity as measured using the Go No Go task. As we mentioned above, current experiments in the lab, which are part of a different line of research, are showing that male mice exposed to tetrahydrocannabinol in adolescence show reduced dopamine innervation in the medial prefrontal cortex in adulthood, but increased dopamine input in the orbitofrontal cortex. In addition, these mice show increased action impulsivity in the Go/No-Go task in adulthood (Capolicchio et al., Society for Neuroscience 2023 Abstracts)

      References

      Capolicchio T., Hernandez, G., Dube, E., Estrada, K., Giroux, M., Flores, C. (2023) Divergent outcomes of delta 9 - tetrahydrocannabinol in adolescence on dopamine and cognitive development in male and female mice. Society for Neuroscience, Washington, DC, United States [abstract].

      Jonker FA, Jonker C, Scheltens P, Scherder EJA. 2015. The role of the orbitofrontal cortex in cognition and behavior. Rev Neurosci 26:1–11. doi:10.1515/revneuro2014-0043 Kim B, Im H. 2019. The role of the dorsal striatum in choice impulsivity. Ann N York Acad Sci 1451:92–111. doi:10.1111/nyas.13961

      (6) What is the effect of knocking down UNC5C on dopamine axons guidance to the cortex?

      We have found that mice that are heterozygous for a nonsense Unc5c mutation, and as a result have reduced levels of UNC5c protein, show reduced amphetamine-induced locomotion and stereotypy (Auger et al., 2013). In the same manuscript we show that this effect only emerges during adolescence, in concert with the growth of dopamine axons to the prefrontal cortex. This is indirect but strong evidence that UNC5c receptors are necessary for correct adolescent dopamine axon development.

      References

      Auger ML, Schmidt ERE, Manitt C, Dal-Bo G, Pasterkamp RJ, Flores C. 2013. unc5c haploinsufficient phenotype: striking similarities with the dcc haploinsufficiency model. European Journal of Neuroscience 38:2853–2863. doi:10.1111/ejn.12270

      (7) In Figures 2-4, the authors only showed the amount of DA axons and UNC5C in NAcc. However, it remains unclear whether these experiments also impact the projections of dopaminergic axons to other brain regions, critical for the behavioral phenotypes. What about other brain regions such as prefrontal cortex? Do the projection of DA axons and UNC5c level in cortex have similar pattern to those in NAcc?

      UNC5c receptors are expressed throughout development and are involved in many developmental processes (Kim and Ackerman, 2011; Murcia-Belmonte et al., 2019; Srivatsa et al., 2014). We cannot say whether the pattern we observe here is unique to the nucleus accumbens, but it is certainly not universal throughout the brain.

      The brain region we focus on in our manuscript, in addition to the nucleus accumbens, is the medial prefrontal cortex. Close and thorough examination of the prefrontal cortices of adult mice revealed practically no UNC5c expression by dopamine axons. However, we did observe very rare cases of dopamine axons expressing UNC5c. It is not clear whether these rare cases are present before or during adolescence.

      Below is a representative set of images of this observation, which is now also included as Supplementary Figure 4:

      Author response image 5.

      Expression of UNC5c protein in the medial prefrontal cortex of an adult male mouse. Low (A) and high (B) magnification images demonstrate that there is little UNC5c expression in dopamine axons in the medial prefrontal cortex. Here we identify dopamine axons by immunofluorescent staining for tyrosine hydroxylase (TH, see our response to comment #9 regarding the specificity of the TH antibody for dopamine axons in the prefrontal cortex). This figure is also included as Supplementary Figure 4 in the manuscript. Abbreviations: fmi: forceps minor of the corpus callosum, mPFC: medial prefrontal cortex.

      References:

      Kim D, Ackerman SL. 2011. The UNC5C Netrin Receptor Regulates Dorsal Guidance of Mouse Hindbrain Axons. J Neurosci 31:2167–2179. doi:10.1523/jneurosci.5254- 10.20110.2011

      Murcia-Belmonte V, Coca Y, Vegar C, Negueruela S, Romero C de J, Valiño AJ, Sala S, DaSilva R, Kania A, Borrell V, Martinez LM, Erskine L, Herrera E. 2019. A Retino-retinal Projection Guided by Unc5c Emerged in Species with Retinal Waves. Current Biology 29:1149-1160.e4. doi:10.1016/j.cub.2019.02.052

      Srivatsa S, Parthasarathy S, Britanova O, Bormuth I, Donahoo A-L, Ackerman SL, Richards LJ, Tarabykin V. 2014. Unc5C and DCC act downstream of Ctip2 and Satb2 and contribute to corpus callosum formation. Nat Commun 5:3708. doi:10.1038/ncomms4708

      (8) Can overexpression of UNC5c or Netrin-1 in male winter hamsters mimic the observations in summer hamsters? Or overexpression of UNC5c in female summer hamsters to mimic the winter hamster? This would be helpful to confirm the causal role of UNC5C in guiding DA axons during adolescence.

      This is an excellent question. We are very interested in both increasing and decreasing UNC5c expression in hamster dopamine axons to see if we can directly manipulate summer hamsters into winter hamsters and vice versa. We are currently exploring virus-based approaches to design these experiments and are excited for results in this area.

      (9) The entire study relied on using tyrosine hydroxylase (TH) as a marker for dopaminergic axons. However, the expression of TH (either by IHC or IF) can be influenced by other environmental factors, that could alter the expression of TH at the cellular level.

      This is an excellent point that we now carefully address in our methods by adding the following:

      In this study we pay great attention to the morphology and localization of the fibres from which we quantify varicosities to avoid counting any fibres stained with TH antibodies that are not dopamine fibres. The fibres that we examine and that are labelled by the TH antibody show features indistinguishable from the classic features of cortical dopamine axons in rodents (Berger et al., 1974; 1983; Van Eden et al., 1987; Manitt et al., 2011), namely they are thin fibres with irregularly-spaced varicosities, are densely packed in the nucleus accumbens, sparsely present only in the deep layers of the prefrontal cortex, and are not regularly oriented in relation to the pial surface. This is in contrast to rodent norepinephrine fibres, which are smooth or beaded in appearance, relatively thick with regularly spaced varicosities, increase in density towards the shallow cortical layers, and are in large part oriented either parallel or perpendicular to the pial surface (Berger et al., 1974; Levitt and Moore, 1979; Berger et al., 1983; Miner et al., 2003). Furthermore, previous studies in rodents have noted that only norepinephrine cell bodies are detectable using immunofluorescence for TH, not norepinephrine processes (Pickel et al., 1975; Verney et al., 1982; Miner et al., 2003), and we did not observe any norepinephrine-like fibres.

      Furthermore, we are not aware of any other processes in the forebrain that are known to be immunopositive for TH under any environmental conditions.

      To reduce confusion, we have replaced the abbreviation for dopamine – DA – with TH in the relevant panels in Figures 1, 2, 3, and 4 to clarify exactly what is represented in these images. As can be seen in these images, fluorescent green labelling is present only in axons, which is to be expected of dopamine labelling in these forebrain regions.

      References:

      Berger B, Tassin JP, Blanc G, Moyne MA, Thierry AM (1974) Histochemical confirmation for dopaminergic innervation of the rat cerebral cortex after destruction of the noradrenergic ascending pathways. Brain Res 81:332–337.

      Berger B, Verney C, Gay M, Vigny A (1983) Immunocytochemical Characterization of the Dopaminergic and Noradrenergic Innervation of the Rat Neocortex During Early Ontogeny. In: Proceedings of the 9th Meeting of the International Neurobiology Society, pp 263–267 Progress in Brain Research. Elsevier.

      Levitt P, Moore RY (1979) Development of the noradrenergic innervation of neocortex. Brain Res 162:243–259.

      Manitt C, Mimee A, Eng C, Pokinko M, Stroh T, Cooper HM, Kolb B, Flores C (2011) The Netrin Receptor DCC Is Required in the Pubertal Organization of Mesocortical Dopamine Circuitry. J Neurosci 31:8381–8394.

      Miner LH, Schroeter S, Blakely RD, Sesack SR (2003) Ultrastructural localization of the norepinephrine transporter in superficial and deep layers of the rat prelimbic prefrontal cortex and its spatial relationship to probable dopamine terminals. J Comp Neurol 466:478–494.

      Pickel VM, Joh TH, Field PM, Becker CG, Reis DJ (1975) Cellular localization of tyrosine hydroxylase by immunohistochemistry. J Histochem Cytochem 23:1–12.

      Van Eden CG, Hoorneman EM, Buijs RM, Matthijssen MA, Geffard M, Uylings HBM (1987) Immunocytochemical localization of dopamine in the prefrontal cortex of the rat at the light and electron microscopical level. Neurosci 22:849–862.

      Verney C, Berger B, Adrien J, Vigny A, Gay M (1982) Development of the dopaminergic innervation of the rat cerebral cortex. A light microscopic immunocytochemical study using anti-tyrosine hydroxylase antibodies. Dev Brain Res 5:41–52.

      (10) Are Netrin-1/UNC5C the only signal guiding dopamine axon during adolescence? Are there other neuronal circuits involved in this process?

      Our intention for this study was to examine the role of Netrin-1 and its receptor UNC5C specifically, but we do not suggest that they are the only molecules to play a role. The process of guiding growing dopamine axons during adolescence is likely complex and we expect other guidance mechanisms to also be involved. From our previous work we know that the Netrin-1 receptor DCC is critical in this process (Hoops and Flores, 2017; Reynolds et al., 2023). Several other molecules have been identified in Netrin-1/DCC signaling processes that control corpus callosum development and there is every possibility that the same or similar molecules may be important in guiding dopamine axons (Schlienger et al., 2023).

      References:

      Hoops D, Flores C. 2017. Making Dopamine Connections in Adolescence. Trends in Neurosciences 1–11. doi:10.1016/j.tins.2017.09.004

      Reynolds LM, Hernandez G, MacGowan D, Popescu C, Nouel D, Cuesta S, Burke S, Savell KE, Zhao J, Restrepo-Lozano JM, Giroux M, Israel S, Orsini T, He S, Wodzinski M, Avramescu RG, Pokinko M, Epelbaum JG, Niu Z, Pantoja-Urbán AH, Trudeau L-É, Kolb B, Day JJ, Flores C. 2023. Amphetamine disrupts dopamine axon growth in adolescence by a sex-specific mechanism in mice. Nat Commun 14:4035. doi:10.1038/s41467-023-39665-1

      Schlienger S, Yam PT, Balekoglu N, Ducuing H, Michaud J-F, Makihara S, Kramer DK, Chen B, Fasano A, Berardelli A, Hamdan FF, Rouleau GA, Srour M, Charron F. 2023. Genetics of mirror movements identifies a multifunctional complex required for Netrin-1 guidance and lateralization of motor control. Sci Adv 9:eadd5501. doi:10.1126/sciadv.add5501

      (11) Finally, despite the authors' claim that the dopaminergic axon project is sensitive to the duration of daylight in the hamster, they never provided definitive evidence to support this hypothesis.

      By “definitive evidence” we think that the reviewer is requesting a single statistical model including measures from both the summer and winter groups. Such a model would provide a probability estimate of whether dopamine axon growth is sensitive to daylight duration. Therefore, we ran these models, one for male hamsters and one for female hamsters.

      In both sexes we find a significant effect of daylength on dopamine innervation, interacting with age. Male age by daylength interaction: F = 6.383, p = 0.00242. Female age by daylength interaction: F = 21.872, p = 1.97 x 10-9. The full statistical analysis is available as a supplement to this letter (Response_Letter_Stats_Details.docx).

      Reviewer 3

      (1) Fig 1 A and B don't appear to be the same section level.

      The reviewer is correct that Fig 1B is anterior to Fig 1A. We have changed Figure 1A to match the section level of Figure 1B.

      (2) Fig 1C. It is not clear that these axons are crossing from the shell of the NAC.

      We have added a dashed line to Figure 1C to highlight the boundary of the nucleus accumbens, which hopefully emphasizes that there are fibres crossing the boundary. We also include here an enlarged image of this panel:

      Author response image 6.

      An enlarged image of Figure1c in the manuscript. The nucleus accumbens (left of the dotted line) is densely packed with TH+ axons (in green). Some of these TH+ axons can be observed extending from the nucleus accumbens medially towards a region containing dorsally oriented TH+ fibres (white arrows).

      (3) Fig 1. Measuring width of the bundle is an odd way to measure DA axon numbers. First the width could be changing during adult for various reasons including change in brain size. Second, I wouldn't consider these axons in a traditional bundle. Third, could DA axon counts be provided, rather than these proxy measures.

      With regards to potential changes in brain size, we agree that this could have potentially explained the increased width of the dopamine axon pathway. That is why it was important for us to use stereology to measure the density of dopamine axons within the pathway. If the width increased but no new axons grew along the pathway, we would have seen a decrease in axon density from adolescence to adulthood. Instead, our results show that the density of axons remained constant.

      We agree with the reviewer that the dopamine axons do not form a traditional “bundle”. Therefore, throughout the manuscript we now avoid using the term bundle.

      Although we cannot count every single axon, an accurate estimate of this number can be obtained using stereology, an unbiassed method for efficiently quantifying large, irregularly distributed objects. We used stereology to count TH+ axons in an unbiased subset of the total area occupied by these axons. Unbiased stereology is the gold-standard technique for estimating populations of anatomical objects, such as axons, that are so numerous that it would be impractical or impossible to measure every single one. Here and elsewhere we generally provide results as densities and areas of occupancy (Reynolds et al., 2022). To avoid confusion, we now clarify that we are counting the width of the area that dopamine axons occupy (rather than the dopamine axon “bundle”).

      References:

      Reynolds LM, Pantoja-Urbán AH, MacGowan D, Manitt C, Nouel D, Flores C. 2022. Dopaminergic System Function and Dysfunction: Experimental Approaches. Neuromethods 31–63. doi:10.1007/978-1-0716-2799-0_2

      (4) TH in the cortex could also be of noradrenergic origin. This needs to be ruled out to score DA axons

      This is the same comment as Reviewer 1 #9. Please see our response below, which we have also added to our methods:

      In this study we pay great attention to the morphology and localization of the fibres from which we quantify varicosities to avoid counting any fibres stained with TH antibodies that are not dopamine fibres. The fibres that we examine and that are labelled by the TH antibody show features indistinguishable from the classic features of cortical dopamine axons in rodents (Berger et al., 1974; 1983; Van Eden et al., 1987; Manitt et al., 2011), namely they are thin fibres with irregularly-spaced varicosities, are densely packed in the nucleus accumbens, sparsely present only in the deep layers of the prefrontal cortex, and are not regularly oriented in relation to the pial surface. This is in contrast to rodent norepinephrine fibres, which are smooth or beaded in appearance, relatively thick with regularly spaced varicosities, increase in density towards the shallow cortical layers, and are in large part oriented either parallel or perpendicular to the pial surface (Berger et al., 1974; Levitt and Moore, 1979; Berger et al., 1983; Miner et al., 2003). Furthermore, previous studies in rodents have noted that only norepinephrine cell bodies are detectable using immunofluorescence for TH, not norepinephrine processes (Pickel et al., 1975; Verney et al., 1982; Miner et al., 2003), and we did not observe any norepinephrine-like fibres.

      References:

      Berger B, Tassin JP, Blanc G, Moyne MA, Thierry AM (1974) Histochemical confirmation for dopaminergic innervation of the rat cerebral cortex after destruction of the noradrenergic ascending pathways. Brain Res 81:332–337.

      Berger B, Verney C, Gay M, Vigny A (1983) Immunocytochemical Characterization of the Dopaminergic and Noradrenergic Innervation of the Rat Neocortex During Early Ontogeny. In: Proceedings of the 9th Meeting of the International Neurobiology Society, pp 263–267 Progress in Brain Research. Elsevier.

      Levitt P, Moore RY (1979) Development of the noradrenergic innervation of neocortex. Brain Res 162:243–259.

      Manitt C, Mimee A, Eng C, Pokinko M, Stroh T, Cooper HM, Kolb B, Flores C (2011) The Netrin Receptor DCC Is Required in the Pubertal Organization of Mesocortical Dopamine Circuitry. J Neurosci 31:8381–8394.

      Miner LH, Schroeter S, Blakely RD, Sesack SR (2003) Ultrastructural localization of the norepinephrine transporter in superficial and deep layers of the rat prelimbic prefrontal cortex and its spatial relationship to probable dopamine terminals. J Comp Neurol 466:478–494.

      Pickel VM, Joh TH, Field PM, Becker CG, Reis DJ (1975) Cellular localization of tyrosine hydroxylase by immunohistochemistry. J Histochem Cytochem 23:1–12.

      Van Eden CG, Hoorneman EM, Buijs RM, Matthijssen MA, Geffard M, Uylings HBM (1987) Immunocytochemical localization of dopamine in the prefrontal cortex of the rat at the light and electron microscopical level. Neurosci 22:849–862.

      Verney C, Berger B, Adrien J, Vigny A, Gay M (1982) Development of the dopaminergic innervation of the rat cerebral cortex. A light microscopic immunocytochemical study using anti-tyrosine hydroxylase antibodies. Dev Brain Res 5:41–52.

      (5) Netrin staining should be provided with NeuN + DAPI; its not clear these are all cell bodies. An in situ of Netrin would help as well.

      A similar comment was raised by Reviewer 1 in point #1. Please see below the immunofluorescent and RNA scope images showing expression of Netrin-1 protein and mRNA in the forebrain.

      Author response image 7.

      This confocal microscope image shows immunofluorescent staining for Netrin-1 (green) localized around cell nuclei (stained by DAPI in blue). This image was taken from a coronal section of the lateral septum of an adult male mouse. Scale bar = 20µm

      Author response image 8.

      This confocal microscope image of a coronal brain section of the medial prefrontal cortex of an adult male mouse shows Netrin-1 mRNA expression (green) and cell nuclei (DAPI, blue). RNAscope was used to generate this image. Brain regions are as follows: Cg1: Anterior cingulate cortex 1, DP: dorsopeduncular cortex, IL: Infralimbic Cortex, PrL: Prelimbic Cortex, fmi: forceps minor of the corpus callosum

      Author response image 9.

      A higher resolution image from the same sample as in Figure 2 shows Netrin-1 mRNA (green) and cell nuclei (DAPI; blue). DP = dorsopeduncular cortex

      (6) The Netrin knockdown needs validation. How strong was the knockdown etc?

      This comment was also raised by Reviewer 1 #1.

      We have previously established the efficacy of the shRNA Netrin-1 knockdown virus used in this experiment for reducing the expression of Netrin-1 (Cuesta et al., 2020). The shRNA reduces Netrin-1 levels in vitro and in vivo.

      References:

      Cuesta S, Nouel D, Reynolds LM, Morgunova A, Torres-Berrío A, White A, Hernandez G, Cooper HM, Flores C. 2020. Dopamine Axon Targeting in the Nucleus Accumbens in Adolescence Requires Netrin-1. Frontiers Cell Dev Biology 8:487. doi:10.3389/fcell.2020.00487

      (7) If the conclusion that knocking down Netrin in cortex decreases DA innervation of the IL, how can that be reconciled with Netrin-Unc repulsion.

      This is an intriguing question and one that we are in the planning stages of addressing with new experiments.

      Although we do not have a mechanistic answered for how a repulsive receptor helps guide these axons, we would like to note that previous indirect evidence from a study by our group also suggests that reducing UNC5c signaling in dopamine axons in adolescence increases dopamine innervation to the prefrontal cortex (Auger et al, 2013).

      References

      Auger ML, Schmidt ERE, Manitt C, Dal-Bo G, Pasterkamp RJ, Flores C. 2013. unc5c haploinsufficient phenotype: striking similarities with the dcc haploinsufficiency model. European Journal of Neuroscience 38:2853–2863. doi:10.1111/ejn.12270

      (8) The behavioral phenotype in Fig 1 is interesting, but its not clear if its related to DA axons/signaling. IN general, no evidence in this paper is provided for the role of DA in the adolescent behaviors described.

      We agree with the reviewer that the behaviours we describe in adult mice are complex and are likely to involve several neurotransmitter systems. However, there is ample evidence for the role of dopamine signaling in cognitive control behaviours (Bari and Robbins, 2013; Eagle et al., 2008; Ott et al., 2023) and our published work has shown that alterations in the growth of dopamine axons to the prefrontal cortex leads to changes in impulse control as measured via the Go/No-Go task in adulthood (Reynolds et al., 2023, 2018a; Vassilev et al., 2021).

      The other adolescent behaviour we examined was risk-like taking behaviour in male and female hamsters (Figures 4 and 5), as a means of characterizing maturation in this behavior over time. We decided not to use the Go/No-Go task because as far as we know, this has never been employed in Siberian Hamsters and it will be difficult to implement. Instead, we chose the light/dark box paradigm, which requires no training and is ideal for charting behavioural changes over short time periods. Indeed, risk-like taking behavior in rodents and in humans changes from adolescence to adulthood paralleling changes in prefrontal cortex development, including the gradual input of dopamine axons to this region.

      References:

      Bari A, Robbins TW. 2013. Inhibition and impulsivity: Behavioral and neural basis of response control. Progress in neurobiology 108:44–79. doi:10.1016/j.pneurobio.2013.06.005

      Eagle DM, Bari A, Robbins TW. 2008. The neuropsychopharmacology of action inhibition: cross-species translation of the stop-signal and go/no-go tasks. Psychopharmacology 199:439–456. doi:10.1007/s00213-008-1127-6

      Ott T, Stein AM, Nieder A. 2023. Dopamine receptor activation regulates reward expectancy signals during cognitive control in primate prefrontal neurons. Nat Commun 14:7537. doi:10.1038/s41467-023-43271-6

      Reynolds LM, Hernandez G, MacGowan D, Popescu C, Nouel D, Cuesta S, Burke S, Savell KE, Zhao J, Restrepo-Lozano JM, Giroux M, Israel S, Orsini T, He S, Wodzinski M, Avramescu RG, Pokinko M, Epelbaum JG, Niu Z, Pantoja-Urbán AH, Trudeau L-É, Kolb B, Day JJ, Flores C. 2023. Amphetamine disrupts dopamine axon growth in adolescence by a sex-specific mechanism in mice. Nat Commun 14:4035. doi:10.1038/s41467-023-39665-1

      Reynolds LM, Pokinko M, Torres-Berrío A, Cuesta S, Lambert LC, Pellitero EDC, Wodzinski M, Manitt C, Krimpenfort P, Kolb B, Flores C. 2018a. DCC Receptors Drive Prefrontal Cortex Maturation by Determining Dopamine Axon Targeting in Adolescence. Biological psychiatry 83:181–192. doi:10.1016/j.biopsych.2017.06.009

      Vassilev P, Pantoja-Urban AH, Giroux M, Nouel D, Hernandez G, Orsini T, Flores C. 2021. Unique effects of social defeat stress in adolescent male mice on the Netrin-1/DCC pathway, prefrontal cortex dopamine and cognition (Social stress in adolescent vs. adult male mice). Eneuro ENEURO.0045-21.2021. doi:10.1523/eneuro.0045-21.2021

      (9) Fig2 - boxes should be drawn on the NAc diagram to indicate sampled regions. Some quantification of Unc5c would be useful. Also, some validation of the Unc5c antibody would be nice.

      The images presented were taken medial to the anterior commissure and we have edited Figure 2 to show this. However, we did not notice any intra-accumbens variation, including between the core and the shell. Therefore, the images are representative of what was observed throughout the entire nucleus accumbens.

      To quantify UNC5c in the accumbens we conducted a Western blot experiment in male mice at different ages. A one-way ANOVA analyzing band intensity (relative to the 15-day-old average band intensity) as the response variable and age as the predictor variable showed a significant effect of age (F=5.615, p=0.01). Posthoc analysis revealed that 15-day-old mice have less UNC5c in the nucleus accumbens compared to 21- and 35-day-old mice.

      Author response image 10.

      The graph depicts the results of a Western blot experiment of UNC5c protein levels in the nucleus accumbens of male mice at postnatal days 15, 21 or 35 and reveals a significant increase in protein levels at the onset adolescence.

      Our methods for this Western blot were as follows: Samples were prepared as previously (Torres-Berrío et al., 2017). Briefly, mice were sacrificed by live decapitation and brains were flash frozen in heptane on dry ice for 10 seconds. Frozen brains were mounted in a cryomicrotome and two 500um sections were collected for the nucleus accumbens, corresponding to plates 14 and 18 of the Paxinos mouse brain atlas. Two tissue core samples were collected per section, one for each side of the brain, using a 15-gauge tissue corer (Fine surgical tools Cat no. NC9128328) and ejected in a microtube on dry ice. The tissue samples were homogenized in 100ul of standard radioimmunoprecipitation assay buffer using a handheld electric tissue homogenizer. The samples were clarified by centrifugation at 4C at a speed of 15000g for 30 minutes. Protein concentration was quantified using a bicinchoninic acid assay kit (Pierce BCA protein assay kit, Cat no.PI23225) and denatured with standard Laemmli buffer for 5 minutes at 70C. 10ug of protein per sample was loaded and run by SDS-PAGE gel electrophoresis in a Mini-PROTEAN system (Bio-Rad) on an 8% acrylamide gel by stacking for 30 minutes at 60V and resolving for 1.5 hours at 130V. The proteins were transferred to a nitrocellulose membrane for 1 hour at 100V in standard transfer buffer on ice. The membranes were blocked using 5% bovine serum albumin dissolved in tris-buffered saline with Tween 20 and probed with primary (UNC5c, Abcam Cat. no ab302924) and HRP-conjugated secondary antibodies for 1 hour. a-tubulin was probed and used as loading control. The probed membranes were resolved using SuperSignal West Pico PLUS chemiluminescent substrate (ThermoFisher Cat no.34579) in a ChemiDoc MP Imaging system (Bio-Rad). Band intensity was quantified using the ChemiDoc software and all ages were normalized to the P15 age group average.

      Validation of the UNC5c antibody was performed in the lab of Dr. Liu, from whom it was kindly provided. Briefly, in the validation study the authors showed that the anti-UNC5C antibody can detect endogenous UNC5C expression and the level of UNC5C is dramatically reduced after UNC5C knockdown. The antibody can also detect the tagged-UNC5C protein in several cell lines, which was confirmed by a tag antibody (Purohit et al., 2012; Shao et al., 2017).

      References:

      Purohit AA, Li W, Qu C, Dwyer T, Shao Q, Guan K-L, Liu G. 2012. Down Syndrome Cell Adhesion Molecule (DSCAM) Associates with Uncoordinated-5C (UNC5C) in Netrin-1mediated Growth Cone Collapse. The Journal of biological chemistry 287:27126–27138. doi:10.1074/jbc.m112.340174

      Shao Q, Yang T, Huang H, Alarmanazi F, Liu G. 2017. Uncoupling of UNC5C with Polymerized TUBB3 in Microtubules Mediates Netrin-1 Repulsion. J Neurosci 37:5620–5633. doi:10.1523/jneurosci.2617-16.2017

      (10) "In adolescence, dopamine neurons begin to express the repulsive Netrin-1 receptor UNC5C, and reduction in UNC5C expression appears to cause growth of mesolimbic dopamine axons to the prefrontal cortex".....This is confusing. Figure 2 shows a developmental increase in UNc5c not a decrease. So when is the "reduction in Unc5c expression" occurring?

      We apologize for the mistake in this sentence. We have corrected the relevant passage in our manuscript as follows:

      In adolescence, dopamine neurons begin to express the repulsive Netrin-1 receptor UNC5C, particularly when mesolimbic and mesocortical dopamine projections segregate in the nucleus accumbens (Manitt et al., 2010; Reynolds et al., 2018a). In contrast, dopamine axons in the prefrontal cortex do not express UNC5c except in very rare cases (Supplementary Figure 4). In adult male mice with Unc5c haploinsufficiency, there appears to be ectopic growth of mesolimbic dopamine axons to the prefrontal cortex (Auger et al., 2013). This miswiring is associated with alterations in prefrontal cortex-dependent behaviours (Auger et al., 2013).

      References:

      Auger ML, Schmidt ERE, Manitt C, Dal-Bo G, Pasterkamp RJ, Flores C. 2013. unc5c haploinsufficient phenotype: striking similarities with the dcc haploinsufficiency model. European Journal of Neuroscience 38:2853–2863. doi:10.1111/ejn.12270

      Manitt C, Labelle-Dumais C, Eng C, Grant A, Mimee A, Stroh T, Flores C. 2010. Peri-Pubertal Emergence of UNC-5 Homologue Expression by Dopamine Neurons in Rodents. PLoS ONE 5:e11463-14. doi:10.1371/journal.pone.0011463

      Reynolds LM, Pokinko M, Torres-Berrío A, Cuesta S, Lambert LC, Pellitero EDC, Wodzinski M, Manitt C, Krimpenfort P, Kolb B, Flores C. 2018a. DCC Receptors Drive Prefrontal Cortex Maturation by Determining Dopamine Axon Targeting in Adolescence. Biological psychiatry 83:181–192. doi:10.1016/j.biopsych.2017.06.009

      (11) In Fig 3, a statistical comparison should be made between summer male and winter male, to justify the conclusions that the winter males have delayed DA innervation.

      This analysis was also suggested by Reviewer 1, #11. Here is our response:

      We analyzed the summer and winter data together in ANOVAs separately for males and females. In both sexes we find a significant effect of daylength on dopamine innervation, interacting with age. Male age by daylength interaction: F = 6.383, p = 0.00242. Female age by daylength interaction: F = 21.872, p = 1.97 x 10-9. The full statistical analysis is available as a supplement to this letter (Response_Letter_Stats_Details.docx).

      (12) Should axon length also be measured here (Fig 3)? It is not clear why the authors have switched to varicosity density. Also, a box should be drawn in the NAC cartoon to indicate the region that was sampled.

      It is untenable to quantify axon length in the prefrontal cortex as we cannot distinguish independent axons. Rather, they are “tangled”; they twist and turn in a multitude of directions as they make contact with various dendrites. Furthermore, they branch extensively. It would therefore be impossible to accurately quantify the number of axons. Using unbiased stereology to quantify varicosities is a valid, well-characterized and straightforward alternative (Reynolds et al., 2022).

      References:

      Reynolds LM, Pantoja-Urbán AH, MacGowan D, Manitt C, Nouel D, Flores C. 2022. Dopaminergic System Function and Dysfunction: Experimental Approaches. Neuromethods 31–63. doi:10.1007/978-1-0716-2799-0_2

      (13) In Fig 3, Unc5c should be quantified to bolster the interesting finding that Unc5c expression dynamics are different between summer and winter hamsters. Unc5c mRNA experiments would also be important to see if similar changes are observed at the transcript level.

      We agree that it would be very interesting to see how UNC5c mRNA and protein levels change over time in summer and winter hamsters, both in males, as the reviewer suggests here, and in females. We are working on conducting these experiments in hamsters as part of a broader expansion of our research in this area. These experiments will require a lengthy amount of time and at this point we feel that they are beyond the scope of this manuscript.

      (14) Fig 4. The peak in exploratory behavior in winter females is counterintuitive and needs to be better discussed. IN general, the light dark behavior seems quite variable.

      This is indeed a very interesting finding, which we have expanded upon in our manuscript as follows:

      When raised under a winter-mimicking daylength, hamsters of either sex show a protracted peak in risk taking. In males, it is delayed beyond 80 days old, but the delay is substantially less in females. This is a counterintuitive finding considering that dopamine development in winter females appears to be accelerated. Our interpretation of this finding is that the timing of the risk-taking peak in females may reflect a balance between different adolescent developmental processes. The fact that dopamine axon growth is accelerated does not imply that all adolescent maturational processes are accelerated. Some may be delayed, for example those that induce axon pruning in the cortex. The timing of the risk-taking peak in winter female hamsters may therefore reflect the amalgamation of developmental processes that are advanced with those that are delayed – producing a behavioural effect that is timed somewhere in the middle. Disentangling the effects of different developmental processes on behaviour will require further experiments in hamsters, including the direct manipulation of dopamine activity in the nucleus accumbens and prefrontal cortex.

      Full Reference List

      Auger ML, Schmidt ERE, Manitt C, Dal-Bo G, Pasterkamp RJ, Flores C. 2013. unc5c haploinsufficient phenotype: striking similarities with the dcc haploinsufficiency model. European Journal of Neuroscience 38:2853–2863. doi:10.1111/ejn.12270

      Bari A, Robbins TW. 2013. Inhibition and impulsivity: Behavioral and neural basis of response control. Progress in neurobiology 108:44–79. doi:10.1016/j.pneurobio.2013.06.005

      Cuesta S, Nouel D, Reynolds LM, Morgunova A, Torres-Berrío A, White A, Hernandez G, Cooper HM, Flores C. 2020. Dopamine Axon Targeting in the Nucleus Accumbens in Adolescence Requires Netrin-1. Frontiers Cell Dev Biology 8:487. doi:10.3389/fcell.2020.00487

      Daubaras M, Bo GD, Flores C. 2014. Target-dependent expression of the netrin-1 receptor, UNC5C, in projection neurons of the ventral tegmental area. Neuroscience 260:36–46. doi:10.1016/j.neuroscience.2013.12.007

      Eagle DM, Bari A, Robbins TW. 2008. The neuropsychopharmacology of action inhibition: crossspecies translation of the stop-signal and go/no-go tasks. Psychopharmacology 199:439– 456. doi:10.1007/s00213-008-1127-6

      Hoops D, Flores C. 2017. Making Dopamine Connections in Adolescence. Trends in Neurosciences 1–11. doi:10.1016/j.tins.2017.09.004

      Jonker FA, Jonker C, Scheltens P, Scherder EJA. 2015. The role of the orbitofrontal cortex in cognition and behavior. Rev Neurosci 26:1–11. doi:10.1515/revneuro-2014-0043

      Kim B, Im H. 2019. The role of the dorsal striatum in choice impulsivity. Ann N York Acad Sci 1451:92–111. doi:10.1111/nyas.13961

      Kim D, Ackerman SL. 2011. The UNC5C Netrin Receptor Regulates Dorsal Guidance of Mouse Hindbrain Axons. J Neurosci 31:2167–2179. doi:10.1523/jneurosci.5254-10.2011

      Manitt C, Labelle-Dumais C, Eng C, Grant A, Mimee A, Stroh T, Flores C. 2010. Peri-Pubertal Emergence of UNC-5 Homologue Expression by Dopamine Neurons in Rodents. PLoS ONE 5:e11463-14. doi:10.1371/journal.pone.0011463

      Murcia-Belmonte V, Coca Y, Vegar C, Negueruela S, Romero C de J, Valiño AJ, Sala S, DaSilva R, Kania A, Borrell V, Martinez LM, Erskine L, Herrera E. 2019. A Retino-retinal Projection Guided by Unc5c Emerged in Species with Retinal Waves. Current Biology 29:1149-1160.e4. doi:10.1016/j.cub.2019.02.052

      Ott T, Stein AM, Nieder A. 2023. Dopamine receptor activation regulates reward expectancy signals during cognitive control in primate prefrontal neurons. Nat Commun 14:7537. doi:10.1038/s41467-023-43271-6

      Phillips RA, Tuscher JJ, Black SL, Andraka E, Fitzgerald ND, Ianov L, Day JJ. 2022. An atlas of transcriptionally defined cell populations in the rat ventral tegmental area. Cell Reports 39:110616. doi:10.1016/j.celrep.2022.110616

      Purohit AA, Li W, Qu C, Dwyer T, Shao Q, Guan K-L, Liu G. 2012. Down Syndrome Cell Adhesion Molecule (DSCAM) Associates with Uncoordinated-5C (UNC5C) in Netrin-1-mediated Growth Cone Collapse. The Journal of biological chemistry 287:27126–27138. doi:10.1074/jbc.m112.340174

      Reynolds LM, Hernandez G, MacGowan D, Popescu C, Nouel D, Cuesta S, Burke S, Savell KE, Zhao J, Restrepo-Lozano JM, Giroux M, Israel S, Orsini T, He S, Wodzinski M, Avramescu RG, Pokinko M, Epelbaum JG, Niu Z, Pantoja-Urbán AH, Trudeau L-É, Kolb B, Day JJ, Flores C. 2023. Amphetamine disrupts dopamine axon growth in adolescence by a sex-specific mechanism in mice. Nat Commun 14:4035. doi:10.1038/s41467-023-39665-1

      Reynolds LM, Pantoja-Urbán AH, MacGowan D, Manitt C, Nouel D, Flores C. 2022. Dopaminergic System Function and Dysfunction: Experimental Approaches. Neuromethods 31–63. doi:10.1007/978-1-0716-2799-0_2

      Reynolds LM, Pokinko M, Torres-Berrío A, Cuesta S, Lambert LC, Pellitero EDC, Wodzinski M, Manitt C, Krimpenfort P, Kolb B, Flores C. 2018a. DCC Receptors Drive Prefrontal Cortex Maturation by Determining Dopamine Axon Targeting in Adolescence. Biological psychiatry 83:181–192. doi:10.1016/j.biopsych.2017.06.009

      Reynolds LM, Yetnikoff L, Pokinko M, Wodzinski M, Epelbaum JG, Lambert LC, Cossette M-P, Arvanitogiannis A, Flores C. 2018b. Early Adolescence is a Critical Period for the Maturation of Inhibitory Behavior. Cerebral cortex 29:3676–3686. doi:10.1093/cercor/bhy247

      Schlienger S, Yam PT, Balekoglu N, Ducuing H, Michaud J-F, Makihara S, Kramer DK, Chen B, Fasano A, Berardelli A, Hamdan FF, Rouleau GA, Srour M, Charron F. 2023. Genetics of mirror movements identifies a multifunctional complex required for Netrin-1 guidance and lateralization of motor control. Sci Adv 9:eadd5501. doi:10.1126/sciadv.add5501

      Shao Q, Yang T, Huang H, Alarmanazi F, Liu G. 2017. Uncoupling of UNC5C with Polymerized TUBB3 in Microtubules Mediates Netrin-1 Repulsion. J Neurosci 37:5620–5633. doi:10.1523/jneurosci.2617-16.2017

      Srivatsa S, Parthasarathy S, Britanova O, Bormuth I, Donahoo A-L, Ackerman SL, Richards LJ, Tarabykin V. 2014. Unc5C and DCC act downstream of Ctip2 and Satb2 and contribute to corpus callosum formation. Nat Commun 5:3708. doi:10.1038/ncomms4708

      Torres-Berrío A, Lopez JP, Bagot RC, Nouel D, Dal-Bo G, Cuesta S, Zhu L, Manitt C, Eng C, Cooper HM, Storch K-F, Turecki G, Nestler EJ, Flores C. 2017. DCC Confers Susceptibility to Depression-like Behaviors in Humans and Mice and Is Regulated by miR-218. Biological psychiatry 81:306–315. doi:10.1016/j.biopsych.2016.08.017

      Vassilev P, Pantoja-Urban AH, Giroux M, Nouel D, Hernandez G, Orsini T, Flores C. 2021. Unique effects of social defeat stress in adolescent male mice on the Netrin-1/DCC pathway, prefrontal cortex dopamine and cognition (Social stress in adolescent vs. adult male mice). Eneuro ENEURO.0045-21.2021. doi:10.1523/eneuro.0045-21.2021

      Private Comments

      Reviewer #1

      (12) The language should be improved. Some expression is confusing (line178-179). Also some spelling errors (eg. Figure 1M).

      We have removed the word “Already” to make the sentence in lines 178-179 clearer, however we cannot find a spelling error in Figure 1M or its caption. We have further edited the manuscript for clarity and flow.

      Reviewer #2

      (1) The authors claim to have revealed how the 'timing of adolescence is programmed in the brain'. While their findings certainly shed light on molecular, circuit and behavioral processes that are unique to adolescence, their claim may be an overstatement. I suggest they refine this statement to discuss more specifically the processes they observed in the brain and animal behavior, rather than adolescence itself.

      We agree with the reviewer and have revised the manuscript to specify that we are referring to the timing of specific developmental processes that occur in the adolescent brain, not adolescence overall.

      (2) Along the same lines, the authors should also include a more substantiative discussion of how they selected their ages for investigation (for both mice and hamsters), For mice, their definition of adolescence (P21) is earlier than some (e.g. Spear L.P., Neurosci. and Beh. Reviews, 2000).

      There are certainly differences of opinion between researchers as to the precise definition of adolescence and the period it encompasses. Spear, 2000, provides one excellent discussion of the challenges related to identifying adolescence across species. This work gives specific ages only for rats, not mice (as we use here), and characterizes post-natal days 28-42 as being the conservative age range of “peak” adolescence (page 419, paragraph 1). Immediately thereafter the review states that the full adolescent period is longer than this, and it could encompass post-natal days 20-55 (page 419, paragraph 2).

      We have added the following statement to our methods:

      There is no universally accepted way to define the precise onset of adolescence. Therefore, there is no clear-cut boundary to define adolescent onset in rodents (Spear, 2000). Puberty can be more sharply defined, and puberty and adolescence overlap in time, but the terms are not interchangeable. Puberty is the onset of sexual maturation, while adolescence is a more diffuse period marked by the gradual transition from a juvenile state to independence. We, and others, suggest that adolescence in rodents spans from weaning (postnatal day 21) until adulthood, which we take to start on postnatal day 60 (Reynolds and Flores, 2021). We refer to “early adolescence” as the first two weeks postweaning (postnatal days 21-34). These ranges encompass discrete DA developmental periods (Kalsbeek et al., 1988; Manitt et al., 2011; Reynolds et al., 2018a), vulnerability to drug effects on DA circuitry (Hammerslag and Gulley, 2014; Reynolds et al., 2018a), and distinct behavioral characteristics (Adriani and Laviola, 2004; Makinodan et al., 2012; Schneider, 2013; Wheeler et al., 2013).

      References:

      Adriani W, Laviola G. 2004. Windows of vulnerability to psychopathology and therapeutic strategy in the adolescent rodent model. Behav Pharmacol 15:341–352. doi:10.1097/00008877-200409000-00005

      Hammerslag LR, Gulley JM. 2014. Age and sex differences in reward behavior in adolescent and adult rats. Dev Psychobiol 56:611–621. doi:10.1002/dev.21127

      Hoops D, Flores C. 2017. Making Dopamine Connections in Adolescence. Trends in Neurosciences 1–11. doi:10.1016/j.tins.2017.09.004

      Kalsbeek A, Voorn P, Buijs RM, Pool CW, Uylings HBM. 1988. Development of the Dopaminergic Innervation in the Prefrontal Cortex of the Rat. The Journal of Comparative Neurology 269:58–72. doi:10.1002/cne.902690105

      Makinodan M, Rosen KM, Ito S, Corfas G. 2012. A critical period for social experiencedependent oligodendrocyte maturation and myelination. Science 337:1357–1360. doi:10.1126/science.1220845

      Manitt C, Mimee A, Eng C, Pokinko M, Stroh T, Cooper HM, Kolb B, Flores C. 2011. The Netrin Receptor DCC Is Required in the Pubertal Organization of Mesocortical Dopamine Circuitry. J Neurosci 31:8381–8394. doi:10.1523/jneurosci.0606-11.2011

      Reynolds LM, Flores C. 2021. Mesocorticolimbic Dopamine Pathways Across Adolescence: Diversity in Development. Front Neural Circuit 15:735625. doi:10.3389/fncir.2021.735625

      Reynolds LM, Yetnikoff L, Pokinko M, Wodzinski M, Epelbaum JG, Lambert LC, Cossette MP, Arvanitogiannis A, Flores C. 2018. Early Adolescence is a Critical Period for the Maturation of Inhibitory Behavior. Cerebral cortex 29:3676–3686. doi:10.1093/cercor/bhy247

      Schneider M. 2013. Adolescence as a vulnerable period to alter rodent behavior. Cell and tissue research 354:99–106. Doi:10.1007/s00441-013-1581-2

      Spear LP. 2000. Neurobehavioral Changes in Adolescence. Current directions in psychological science 9:111–114. doi:10.1111/1467-8721.00072

      Wheeler AL, Lerch JP, Chakravarty MM, Friedel M, Sled JG, Fletcher PJ, Josselyn SA, Frankland PW. 2013. Adolescent Cocaine Exposure Causes Enduring Macroscale Changes in Mouse Brain Structure. J Neurosci 33:1797–1803. doi:10.1523/jneurosci.3830-12.2013

      (3) Figure 1 - the conclusions hinge on the Netrin-1 staining, as shown in panel G, but the cells are difficult to see. It would be helpful to provide clearer, more zoomed images so readers can better assess the staining. Since Netrin-1 expression reduces dramatically after P4 and they had to use antigen retrieval to see signal, it would be helpful to show some images from additional brain regions and ages to see if expression levels follow predicted patterns. For instance, based on the allen brain atlas, it seems that around P21, there should be high levels of Netrin-1 in the cerebellum, but low levels in the cortex. These would be nice controls to demonstrate the specificity and sensitivity of the antibody in older tissue.

      We do not study the cerebellum and have never stained this region; doing so now would require generating additional tissue and we’re not sure it would add enough to the information provided to be worthwhile. Note that we have stained the forebrain for Netrin-1 previously, providing broad staining of many brain regions (Manitt et al., 2011)

      References:

      Manitt C, Mimee A, Eng C, Pokinko M, Stroh T, Cooper HM, Kolb B, Flores C. 2011. The Netrin Receptor DCC Is Required in the Pubertal Organization of Mesocortical Dopamine Circuitry. J Neurosci 31:8381–8394. doi:10.1523/jneurosci.0606-11.2011

      (4) Figure 3 - Because mice tend to avoid brightly-lit spaces, the light/dark box is more commonly used as a measure of anxiety-like behavior than purely exploratory behavior (including in the paper they cited). It is important to address this possibility in their discussion of their findings. To bolster their conclusions about the coincidence of circuit and behavioral changes in adolescent hamsters, it would be useful to add an additional measure of exploratory behaviors (e.g. hole board).

      Regarding the light/dark box test, this is an excellent point. We prefer the term “risk taking” to “anxiety-like” and now use the former term in our manuscript. Furthermore, our interest in the behaviour is purely to chart the development of adolescent behaviour across our treatment groups, not to study a particular emotional state. Regardless of the specific emotion or emotions governing the light/dark box behaviour, it is an ideal test for charting adolescent shifts in behaviour as it is well-characterized in this respect, as we discuss in our manuscript.

      (5) Supplementary Figure 4,5 The authors defined puberty onset using uterine and testes weights in hamsters. While the weights appear to be different for summer and winter hamsters, there were no statistical comparison. Please add statistical analyses to bolster claims about puberty start times. Also, as many studies use vaginal opening to define puberty onset, it would be helpful to discuss how these measurements typically align and cite relevant literature that described use of uterine weights. Also, Supplementary Figures 4 and 5 were mis-cited as Supp. Fig. 2 in the text (e.g. line 317 and others).

      These are great suggestions. We have added statistical analyses to Supplementary Figures 5 and 6 and provided Vaginal Opening data as Supplementary Figure 7. The statistical analyses confirm that all three characters are delayed in winter hamsters compared to summer hamsters.

      We have also added the following references to the manuscript:

      Darrow JM, Davis FC, Elliott JA, Stetson MH, Turek FW, Menaker M. 1980. Influence of Photoperiod on Reproductive Development in the Golden Hamster. Biol Reprod 22:443–450. doi:10.1095/biolreprod22.3.443

      Ebling FJP. 1994. Photoperiodic Differences during Development in the Dwarf Hamsters Phodopus sungorus and Phodopus campbelli. Gen Comp Endocrinol 95:475–482. doi:10.1006/gcen.1994.1147

      Timonin ME, Place NJ, Wanderi E, Wynne-Edwards KE. 2006. Phodopus campbelli detect reduced photoperiod during development but, unlike Phodopus sungorus, retain functional reproductive physiology. Reproduction 132:661–670. doi:10.1530/rep.1.00019

      (6) The font in many figure panels is small and hard to read (e.g. 1A,D,E,H,I,L...). Please increase the size for legibility.

      We have increased the font size of our figure text throughout the manuscript.

      Reviewer #3

      (15) Fig 1 C,D. Clarify the units of the y axis

      We have now fixed this.

      Full Reference List

      Adriani W, Laviola G. 2004. Windows of vulnerability to psychopathology and therapeutic strategy in the adolescent rodent model. Behav Pharmacol 15:341–352. doi:10.1097/00008877-200409000-00005

      Hammerslag LR, Gulley JM. 2014. Age and sex differences in reward behavior in adolescent and adult rats. Dev Psychobiol 56:611–621. doi:10.1002/dev.21127

      Hoops D, Flores C. 2017. Making Dopamine Connections in Adolescence. Trends in Neurosciences 1–11. doi:10.1016/j.tins.2017.09.004

      Kalsbeek A, Voorn P, Buijs RM, Pool CW, Uylings HBM. 1988. Development of the Dopaminergic Innervation in the Prefrontal Cortex of the Rat. The Journal of Comparative Neurology 269:58–72. doi:10.1002/cne.902690105

      Makinodan M, Rosen KM, Ito S, Corfas G. 2012. A critical period for social experiencedependent oligodendrocyte maturation and myelination. Science 337:1357–1360. doi:10.1126/science.1220845

      Manitt C, Mimee A, Eng C, Pokinko M, Stroh T, Cooper HM, Kolb B, Flores C. 2011. The Netrin Receptor DCC Is Required in the Pubertal Organization of Mesocortical Dopamine Circuitry. J Neurosci 31:8381–8394. doi:10.1523/jneurosci.0606-11.2011

      Reynolds LM, Flores C. 2021. Mesocorticolimbic Dopamine Pathways Across Adolescence: Diversity in Development. Front Neural Circuit 15:735625. doi:10.3389/fncir.2021.735625 Reynolds LM, Yetnikoff L, Pokinko M, Wodzinski M, Epelbaum JG, Lambert LC, Cossette M-P, Arvanitogiannis A, Flores C. 2018. Early Adolescence is a Critical Period for the Maturation of Inhibitory Behavior. Cerebral cortex 29:3676–3686. doi:10.1093/cercor/bhy247

      Schneider M. 2013. Adolescence as a vulnerable period to alter rodent behavior. Cell and tissue research 354:99–106. doi:10.1007/s00441-013-1581-2

      Spear LP. 2000. Neurobehavioral Changes in Adolescence. Current directions in psychological science 9:111–114. doi:10.1111/1467-8721.00072

      Wheeler AL, Lerch JP, Chakravarty MM, Friedel M, Sled JG, Fletcher PJ, Josselyn SA, Frankland PW. 2013. Adolescent Cocaine Exposure Causes Enduring Macroscale Changes in Mouse Brain Structure. J Neurosci 33:1797–1803. doi:10.1523/jneurosci.3830-12.2013

    1. Author response:

      The following is the authors’ response to the original reviews.

      Public Reviews:

      Reviewer #1 (Public review):

      Summary:

      The manuscript co-authored by Pál Barzó et al is very clear and very well written, demonstrating the electrophysiological and morphological properties of human cortical layer 2/3 pyramidal cells across a wide age range, from age 1 month to 85 years using whole-cell patch clamp. To my knowledge, this is the first study that looks at the cross-age differences in biophysical and morphological properties of human cortical pyramidal cells. The community will also appreciate the significant effort involved in recording data from 485 cells, given the challenges associated with collecting data from human tissue. Understanding the electrophysiological properties of individual cells, which are essential for brain function, is crucial for comprehending human cortical circuits. I think this research enhances our knowledge of how biophysical properties change over time in the human cortex. I also think that by building models of human single cells at different ages using these data, we can develop more accurate representations of brain function. This, in turn, provides valuable insights into human cortical circuits and function and helps in predicting changes in biophysical properties in both health and disease.

      Strengths:

      The strength of this work lies in demonstrating how the electrophysiological and morphological features of human cortical layer 2/3 pyramidal cells change with age, offering crucial insights into brain function throughout life.

      Weaknesses:

      One potential weakness of the paper is that the methodology could be clearer, especially in how different cells were used for various electrophysiological measurements and the conditions under which the recordings were made. Clarifying these points would improve the study's rigor and make the results easier to interpret.

      Reviewer #2 (Public review):

      Summary:

      In this study, Barzo and colleagues aim to establish an appraisal for the development of basal electrophysiology of human layer 2/3 pyramidal cells across life and compare their morphological features at the same ages.

      Strengths:

      The authors have generated recordings from an impressive array of patient samples, allowing them to directly compare the same electrophysiological features as a function of age and other biological features. These data are extremely robust and well organised.

      Weaknesses:

      The use of spine density and shape characteristics is performed from an extremely limited sample (2 individuals). How reflective these data are of the population is not possible to interpret. Furthermore, these data assume that spines fall into discrete types - which is an increasingly controversial assumption.

      Many data are shown according to somewhat arbitrary age ranges. It would have been more informative to plot by absolute age, and then perform more rigourous statistics to test age-dependent effects.

      Overall, the authors achieve their aims by assessing the physiological and morphological properties of human L2/3 pyramidal neurons across life. Their findings have extremely important ramifications for our understanding of human life and implications for how different neuronal properties may influence neurological conditions.

      Reviewer #3 (Public review):

      Summary:

      To understand the specificity of age-dependent changes in the human neocortex, this paper investigated the electrophysiological and morphological characteristics of pyramidal cells in a wide age range from infants to the elderly.

      The results show that some electrophysiological characteristics change with age, particularly in early childhood. In contrast, the larger morphological structures, such as the spatial extent and branching frequency of dendrites, remained largely stable from infancy to old age. On the other hand, the shape of dendritic spines is considered immature in infancy, i.e., the proportion of mushroom-shaped spines increases with age.

      Strengths:

      Whole-cell recordings and intracellular staining of pyramidal cells in defined areas of the human neocortex allowed the authors to compare quantitative parameters of electrophysiological and morphological properties between finely divided age groups.

      They succeeded in finding symmetrical changes specific to both infants and the elderly, and asymmetrical changes specific to either infants or the elderly. The similarity of pyramidal cell characteristics between areas is unexpected.

      Weaknesses:

      Human L2/3 pyramidal cells are thought to be heterogeneous, as L2/3 has expanded to a high degree during the evolution from rodents to humans. However, the diversity (subtyping) is not revealed in this paper.

      Recommendations for the authors: 

      Reviewer #1 (Recommendations for the authors):

      The manuscript co-authored by Pál Barzó et al is very clear and very well written, demonstrating the electrophysiological and morphological properties of the human cortical layer 2/3 pyramidal cells across a wide age range, from age 1 month to 85 years using whole-cell patch clamp. To my knowledge, this is the first study that looks at the cross-age differences in morphological and electrophysiological properties of human cortical pyramidal cells. The community will also appreciate the significant effort involved in recording data from 485 cells, given the challenges associated with collecting data from human tissue. understanding the electrophysiological properties of individual cells, which are essential for brain function, is crucial for comprehending human cortical circuits. I think this research enhances our knowledge of how biophysical properties change over time in the human cortex. I also think that by building models of human single cells at different ages using these data, we can develop more accurate representations of brain function. This, in turn, provides valuable insights into human cortical circuits and function and helps in predicting changes in biophysical properties in both health and disease.

      We are grateful for the positive evaluation of our work. We also thank the reviewers for their comments and believe that our manuscript has improved significantly with their help. In addition to the reviewer’s suggestions for improvement, further cell reconstructions were performed to make the anatomical data more robust (n = 1,2,3,3,4,3,2 additional reconstruction in age groups infant, early childhood, late childhood, adolescence, young adulthood, middle adulthood and late adulthood, respectively; Σn = 18). Four additional cells were added to the spine analysis and the statistics associated with each additional dataset were updated.

      I have some comments, particularly regarding the methodology and data presentation, to improve the clarity of the paper

      (1) I assume the tissue is from the resected area adjacent to the tumor. Could you please clarify this in the Methods section?

      Thank you for this comment, it has been clarified in the Methods section with the following sentence: “We used human cortical tissue adjacent to the pathological lesion  that had to be surgically removed from patients (n = 63 female  n = 45 male) as part of the treatment for tumors, hydrocephalus, apoplexy, cysts, and arteriovenous malformation.”

      (2) Regarding the presentation of data in the Methods section, could you please clarify whether the authors used different cells for measuring the various electrophysiological properties? The number of recorded cells for calculating subthreshold properties (e.g., late adulthood: n = 113) differs from the number the cells used for calculating suprathreshold properties (e.g., late adulthood: n = 83). If this is the case, it may make it difficult to compare the electrophysiological properties. Could you please clarify this?

      The different element numbers are indeed due to the fact that different quality criteria were defined for the analysis of fast and slow signals. For the analysis of fast signals (e.g. AP half-width, AP upstroke velocity, AP amplitude), higher quality requirements were established therefore cells with high series resistance (> 30 MΩ) were excluded. We have updated and clarified the recording conditions in the text, figures, and methodology section accordingly.

      (3) Additionally, they mentioned that their recordings were done at zero holding current and at more than -50 pA. Could you clarify whether the data from these two sets of experiments were combined? If so, please provide an explanation in the methods section.

      Basically, we wanted to determine the parameters of the potential changes of the membrane at rest. However, for technical reasons related to the biological amplifier, in some of the experiments a certain continuous holding current may be present during the measurement (3.5% of all experiments). The holding currents were in the range of -50 pA to +60 pA. Within this range, previously checked on mouse neurons we have not found linear correlation between the electrophysiological properties and the holding current. This is reported in the Methods section.

      (4) This section needs revision. It is unclear why different series resistances (Rs) or different cells were used to compute various electrophysiological properties." To calculate passive membrane properties (resting membrane potential, input resistance, time constant, and sag) either cells with series resistance (Rs): 22.85 {plus minus} 9.04 MΩ (ranging between -4.55 MΩ and 56.76 MΩ) and 0 pA holding current (n = 154), or cells with holding current > -50 pA (-7.46 {plus minus} 28.56 pA, min: -49.89 pA, max: 59.68pA) and Rs < 30 MΩ (18.96 {plus minus} 6.48 MΩ) (n = 23) were used. For the analysis of high frequency action potential features (AP half-width, AP up-stroke velocity, AP amplitude and rheobase) cells with Rs < 30 MΩ (n = 331 cells with Rs 19.2 {plus minus} 6.6 MΩ) and holding current > -50pA (n = 308 with 0 pA holding current and Rs: 19.22 {plus minus} 6.59 MΩ, n = 23 withholding current: -7.46 {plus minus} 28.56 pA and Rs: 18.96 {plus minus} 6.48 MΩ) were used."

      To make the chapter clearer, we simplified the cell groups used to analyse the different electrophysical properties and revised the Method section as follows: “For the analysis of the electrophysiological recordings n = 457 recordings with a series resistance (Rs) of 24.93 ± 11.18 MΩ (max: 63.77 MΩ) were used. For the analysis of fast parameters related to the action potential (AP half-width, AP upstroke velocity, AP amplitude and rheobase), higher quality requirements were set and cells with Rs > 30 MΩ were excluded. This reduced the data set to n = 331 cells with Rs 19.42 ± 6.2 MΩ.”

      (5) The authors recorded the sag ratio using a -100 pA injected current. Is there a technical reason why they did not inject more than -100 PA?

      There is no particular technical reason, we use similar to others this current amplitude for voltage response recordings over the years to record electrophysiological traces.

      (6) In the abstract, the authors mentioned that data were recorded from ages 1 month to 85 years. However, in the results, they stated that data were recorded from ages 0 to 85 years. Could you please clarify this discrepancy?

      We corrected this discrepancy.

      (7) Additionally, the results mention that data were collected from 485 human cortical layer 2/3 (L2/3) pyramidal cells, but subthreshold membrane features such as resting membrane potential, input resistance, time constant (tau), and sag ratio were calculated in 475 cortical pyramidal cells from 99 patients. Could you please clarify these discrepancies? In the discussion "We recorded from n = 457 human cortical excitatory pyramidal cells from the supragranular layer from birth to 85 years"

      Thank you for pointing this out, we have corrected the error. Although our full data set contained 485 pyramidal cells, 28 recordings were excluded from the electrophysiological analysis and were used for morphological evaluation only, therefore 457 recordings were used for passive parameter measurements.

      (8) Regarding the distance from the pia to the border layer L1/L2, did the authors notice any differences across ages?

      To investigate whether the thickness of cortical layer 1 changes throughout life, we measured the L1 thickness and found no significant differences between age groups (P = 0.09, Kruskal-Wallis test) (Author response image 1).

      Author response image 1.

      Thickness of cortical layer 1 at different life stages. (A) Boxplot shows the thickness of layer 1. (B) Scatter plot shows the distribution of L1 thickness measured on the reconstructed cells. Age is shown in years on a logarithmic scale, dots are color-coded according to the corresponding age groups.

      (9) I am not sure why they referred to the data as layer 2/3 when most of the data, based on Figure 1E, were recorded from a distance of 0-200 µm from the L1/L2 border. Could it be that there is no significant depth-dependent variation in electrophysiological properties, as reported by Berg (2021), Kalmbach (2018), and Chameh (2021)?

      Although the vast majority of our data comes from a distance of less than 200 μm from the L1/L2 border, we cannot neglect the fact that our dataset also contains a small number of cells deeper than this, which are layer 3 cells. Apart from some differences shown in Supplementary Figures 7-9, we found no general difference between cells located at a distance of less than 200 μm and more than 200 μm from the L1 border.

      (10) In Figure 1, there is variability in resting membrane potential (RMP), tau, and input resistance (IR) within the infant age group. However, this trend is not observed in the sag ratio. Could you please discuss this finding?

      The large variance in the data is due to dramatic changes in these three parameters during the first year of life. Supplementary Figure 3 shows the comparisons of parameter distributions of patients between 0-6 months and 6-12 months. The sag amplitude in these cells is generally low therefore no such large changes could have occurred in them.

      (11) Did the authors use a K-Nearest Neighbors (KNN) test to assess the accuracy of the infant cluster in Figure 3F?

      Based on eight electrophysiological features of the cells (resting Vm, input resistance, tau, sag ratio, rheobase, AP half-width, AP up-stroke, and AP amplitude), the infant pyramidal cells on a UMAP form a distinct group (Author response image 2A) represented by cluster 4 on Author response image 2B. When calculating the sum of the Euclidean distances of cells within the cluster from the centroid, the isolated infant group (cluster 4) shows the smallest distance value from the centroid (cluster 1: 40.2, cluster 2: 36.21, cluster 3: 39.96, cluster 4: 5.72, cluster 5: 39.2, cluster 6: 55.74, cluster 7: 54.27), demonstrating that infant cells create a discrete cluster distinct from other age groups (Author response image 2B).

      Author response image 2.

      (A) Uniform Manifold Approximation and Projection (UMAP) of 8 selected electrophysiological properties (resting Vm, input resistance, tau, sag ratio, rheobase, AP half-width, AP up-stroke, and AP amplitude) with data points for 331 cortical L2/3 pyramidal cells, colored with the corresponding age groups. (B) UMAP colored by k-means clustering with 7 clusters, red crosses represent the centroids of the clusters.

      (12) Missing citation: 'Previous research has shown that the biophysical properties of human pyramidal cells show depth-related correlations throughout L2/3 (Berg et al., 2021).' Please include citations for Kalmbach (2018) and Chameh (2021).

      We thank for the additional references, these studies are now cited.

      (13) Have they noticed any morphological properties differences among the different cortical lobes (Parietal, Temporal, Frontal, and Occipital). It would be beneficial to present this data, especially since they have a sufficient sample size from each cortical lobe.

      The majority of our data set on the morphological properties of pyramidal cells comes from the parietal (n = 17 cells) and temporal lobe (n = 15). We found no significant differences in the morphological properties of cells from these two brain regions and no differences between age groups in the same cortical lobes.

      (14) Have the authors found differences in spine characteristics among different cortical areas, as reported previously by 10.1023/a:1024134312173).

      We found morphological differences in dendritic spines in the different brain regions, yet, our data are limited to draw definitive conclusions.

      Reviewer #2 (Recommendations for the authors):

      Major

      (1) I believe that these data presented in all main text figures would be more intuitive to be plotted on a log(age) scale, such as shown in supplementary Figure 13. The bounds of the ages used for different groups, as summarised in Figure 1 feel somewhat arbitrary.

      Recent neuroscientific studies on postnatal ageing mainly use the age-group comparison format (Kang 2011, Bethlehem 2022), which has been defined based on milestones in the cognitive, motor, social-emotional, and language/communications domains of observable behaviour (Zubler et al. 2022, for detailed definitions see Kang 2011). Since many parameters do not vary linearly but take a U-shape (or inverted U-shape), statistical quantification of these is not straightforward, so we would retain the age-group format for the main graphs. However, at the reviewer's suggestion, electrophysiological and morphological parameters are presented on a log(age) scale as supplementary figures (Supplementary Figures 2,4 and 6), also further statistical analysis was also carried out without grouping the data (see response 5).

      (2) The authors present a lot of data values in the text, which is also shown in the figures. This makes reading of the manuscript somewhat difficult in places. For brevity, it may be best to present this data as supplementary tables.

      Thank you for this suggestion. We have inserted these data as tables.

      (3) I am unclear why the authors excluded cells that fired doublets or triplets in Figure 4? Were these included in the passive and AP-specific analysis - but excluded from F-I plots? Please clarify the rationale and the relative abundance of these physiological types based on age - one might predict that more initial-burst firing types are associated with older neurons?

      Thank you for drawing attention to this anomaly. We have updated the figures and text by adding the cells with initial burst firing. These cells are also included in the analysis of passive and action potential properties. In our overall dataset, 6.78% of cells show burst firing; infant: 0%, early childhood: 3.57% (1 cell), late childhood: 0%, adolescence: 11.11% (6 cells), young adulthood: 10.11% (9), middle adulthood: 10.71% (6 cells), late adulthood: 7.96 (9 cells) of all cells including the age groups.

      (4) The statistical analyses performed in Figure 6 are not justified. From the authors' description of these data, they derive spine density measurements from 1 infant and 1 aged adult, then perform pseudoreplicated analysis in these individuals. These data would require greater replication from infant and aged groups - with the possible inclusion of a younger adult group also. It would be ideal to have n=3/age group to allow robust statistical analysis.

      Thank you for this point. Accordingly, we have expanded our data set to include n = 3 infant pyramidal cells (83 days old, from one patient) and n = 3 pyramidal cells from three late adulthood patients (64.3 ± 2.08 years old).

      (5) Given the high number of individuals and replicates throughout this manuscript, a more circumspect approach to statistics would be appreciated, e.g. a generalised linear mixed effects model - with age as a fixed effect and sex, patient, etc as random effects. This may reveal the greatest statistical power of these important and rich data.

      Of the generative models we used the Generalized Additive Mixed Model (GAMM) to describe the relationship between age and the various passive and active electrophysiological features. We defined age with cubic spline smoothing term as the fixed effect and gender, brain area, surgical procedure, and hemisphere as random effects. With GAMM we found that the age-dependent correlation of the examined parameters (resting membrane potential, input resistance, tau, sag ratio, rheobase current, AP half-width, AP up-stroke velocity, AP amplitude, first AP latency, adaptation) was significant, except for F-I slope, described by the model incorporating the four random effects.  We also observed correlation with gender, brain area, hemisphere, and surgical procedure in various intrinsic properties. The Author response table 1 below shows the statistical values of GAMM and the statistical tests used in the manuscript to compare.

      Author response table 1.

      Statistical significance of patient attributes *In the pairwise comparison, the age of cells in the two groups was significantly different: female (subthreshold: 37.36 ± 26.25 years old, suprathreshold: 38.3 ± 25.6 y.o.) - male (subthreshold: 24.86 ± 23.7 y.o., suprathreshold: 25.7 ± 23.93 y.o.), subthreshold: P = 1.96*10-6, suprathreshold: P = 3.25*10-5 Mann-Whitney test. **In the pairwise comparison, the age of cells in the two groups was significantly different: surgical procedure: tumor removal (subthreshold: 33.72 ± 24.33 y.o., suprathreshold: 36.43 ± 27.07 y.o.) - VP shunt (subthreshold: 27.38 ± 29.69 y.o., suprathreshold: 27.07 ± 29.37 y.o.) subthreshold: P = 3.68*10-3, suprathreshold: P = 1.64-10-3, Mann-Whitney test)

      (6) Regarding the morphological diversity of dendritic spines. There is some debate in the field as to whether the distinction of specific dendritic spine types - as conveyed in this manuscript - are true subtypes or reflect a continuum of diverse morphology (see Tønneson et al., 2014 Nature Neuroscience). It is appreciated that the approach taken by the authors is the dogma within the field - however, dogma should continue to be challenged. Given that the authors have used DAB labelling combined with light microscopy, the possibility of accurately measuring spine morphology required for determining this continuum is extremely limited (e.g. Li et al., (2023) ACS Chemical Neuroscience). I would suggest that alongside the inclusion of further replicates for their spine analysis, the authors tone down their discussion of spine subtypes given the absence of any synaptic data presented in this current study to support the maturation (or otherwise) of dendritic spine synapses.

      Many thanks to the reviewer for this comment. We agree with the drawbacks of our method for testing spine categorization. To increase the reliability of our results, we increased the number of pyramidal cells in the infant and late adult groups. We also revised the figure and as suggested by Reviewer#3 added photos of spines to each category in addition to schematic drawings to give an impression of the phenotype. In the discussion, we only address the differences between two readily separable mushroom and filopodial forms and highlight results that only confirm findings already known in the literature. Although the concerns are valid, we apply the sentence from the above Li et al. (2023) reference “...the most sophisticated equipment may not always be necessary for answering some research questions”. We believe that it is worth sharing our data and the somewhat subjective grouping, which we hope to report in more detail in the future.

      Minor

      (1) The order of the supplemental materials is out of order with their introduction in the text. These should be revised to reflect the order mentioned in the text.

      Thank you for your comment, we have corrected the order of the supplementary figures.

      (2) In Supplementary Figure 13, it would be informative to include some form of linear regression to confirm whether an age-dependent effect on neuronal morphology exists.

      We have added linear regression to the figure.

      (3) Figure 3D = should this be AP - not Ap?

      Thank you for drawing attention to this, we have corrected the incorrect typing on the figure.

      (4) For UMAP analysis in Figure 3, please provide a table of the features that were used for the 32 & 8-parameter UMAPs respectively.

      We have added a table to the Materials and methods section of all the electrophysiological features included in the UMAP.

      (5) For morphology, please include pia and L1/2 border for reconstructions shown for clarity.

      We indicated both the pia mater and the L1/2 border on the figure showing all the reconstructions (Supplementary Figure 10).

      Reviewer #3 (Recommendations for the authors):

      Major:

      (1) Data were obtained from different cortical areas of human patients of different ages. The electrophysiological characteristics were largely independent of other attributes such as disease, gender, and cortical areas (Supplementary Figure 2). To support the conclusion that age is one of the key attributes responsible for change, a similar morphological analysis would be necessary for gender.

      We updated the text and the supplementary section with Supplementary Figures 18-21. to determine if age-related differences in biophysical characteristics are affected by the patient's gender.

      (2) 'mushroom-shaped, thin, filopodial, branched, and stubby spines'

      Show photographs of individual typical spine types to make the classification easier to understand.

      To make the classification more understandable, we have updated the corresponding figure (Figure 6) with representative photos of the dendritic spine types.

      (3) Some electrophysiological parameters of the infant group showed higher deviations compared to other age groups. A UMAP (Supplementary Figure 2) shows that some infant neurons form a small cluster, while other infant neurons are scattered with neurons of other ages. Are there any differences between infant neurons in the small cluster and other infant neurons with respect to attributes other than age?

      For most of the electrophysiological parameters, the infant age group showed age-dependent variability, as illustrated in Supplementary Figures 3, 2,4 and 6 . The small group of infant cells is not clustered by gender, brain region, or medical condition, as shown in Supplementary Figure 5.

      (4) A recent paper (Benavides-Piccione et al. 2024, doi:10.1093/cercor/bhae180) reported that some morphological parameters of human layer 3 neurons differ between occipital and temporal regions. Area-dependent morphological differences have been also reported in non-human primates. Discussion of potential contradictions may therefore be requested.

      Most of the cells we reconstructed originated from the parietal and temporal regions (parietal: n = 20, temporal: n = 23, frontal: n = 15, occipital: n = 5). We found no differences in morphological features between these two regions, and we also found no significant differences when we compared the cells from the same brain regions by age group.

      (5) L2/3 cells of rodents are morphologically differentiated according to cortical depth. If individual L2/3 cells of humans are less differentiated than those of rodents, this point should be discussed.

      Depth-related morphological heterogeneity has already been reported previously (Berg 2021), however, our dataset on the morphological characteristics of pyramidal cells is from the upper L2/3 region, with their soma located at a distance of 117.85 ± 65.3 μm (between: 11.05 and 243.3 μm) from the L1/L2 border. Therefore, we cannot conclude from our data whether humans are less differentiated than rodents.

      Minor:

      (1) Cell body morphology may affect electrophysiological properties. However, morphological quantification of cell bodies has not been reported. It may be added.

      In our DAB-labeled samples, we could not perfectly measure the total volume of the cell body in the reconstructions, therefore our measurements regarding the soma morphology are not shown in the manuscript. When comparing the cell body area of the middle sections of the soma of the reconstructed cells between the age groups, we found no significant differences (P = 0.082, Kruskal–Wallis test).

      (2) 'The adaptation of the AP frequency response'

      Describe how this parameter was obtained.

      The adaptation of the AP frequency response or adaptation was calculated as the average adaptation of the interspike interval between consecutive APs.

      (3) 'we excluded cells showing initial duplet or triplet action potential bursts'

      Why were the burst cells excluded from the analysis?

      We have modified the figures and text to include cells with initial burst firing.

      (4) Electrophysiological characteristics to be analyzed:

      Spike thresholds and afterhyperpolarizations

      We found age-related differences in the amplitude of the afterhyperpolarization (P = 2.56*10<sup>-30</sup>, Kruskal-Wallis test) and in the threshold of the action potential (P = 5.24*10<sup>-12</sup>, Kruskal-Wallis test) (Author response image 3).

      Author response image 3.

      Age-dependence of afterhyperpolarization and AP threshold. (A-B) Boxplots show the differences in afterhyperpolarization (AHP) amplitude (A) and AP threshold (B) between age groups. Asterisks indicate statistical significance (* P < 0.05, ** P < 0.01, *** P < 0.001, Kruskal-Wallis test with post-hoc Dunn test). (C-D) Scatter plots show AHP amplitude (C) and AP threshold (D) across the lifespan. Age is shown on a logarithmic scale, dots are colored according to the corresponding age group.

      (5) 'We identified and labeled each spine on n = 2 fully 3D-reconstructed cells'

      To which cortical area do these cells belong?

      At what depths are they distributed?

      Is it possible to report the number of spines, in addition to the density per unit length?

      We increased the number of cells in which we analyzed dendritic spine density. The data shown in Figure 6. are from pyramidal cells from an infant patient (n = 3 from a single patient) and late adulthood patients (n = 3 from 3 patients) (Supplementary Figure 13). The infant cells are from the same patient, the sample is from the right parietal lobe, and the patient is 83 days old. The older cells are from three different patients (#1: 65 years old, right temporal lobe; #2: 66 years old, right parietal lobe; #3: 62 years old, right frontal lobe). Infant cells are located 144.43 ± 45.26 µm (#1: 109.3, #2: 128.49, #3: 195.5 µm), late adult cells 161.22 ± 66.22 µm (#1: 183.5, #2: 213.42, #3: 86.73 µm) from the L1/2 border. We provide the number of spines in an additional supplementary table (Supplementary table 2.).

    1. Author response:

      The following is the authors’ response to the original reviews.

      Thank you for your time and consideration on our submission. We also thank the reviewers for their consideration and helpful comments.  We have revised the introduction, results, and discussion sections of the revised manuscript in accordance with the reviewers’ suggestions, which have enhanced the clarity of our work. Specifically, we have clarified that the aim of the study is to report newly discovered sperm behaviours inside the uterus via high resolution deep tissue live imaging, and to stimulate further studies and discussion in the field of postcopulatory sexual selection in mice based on our observations. To the best of our knowledge, many of the specific sperm behaviours described in our manuscript are being reported for the first time, proven through direct observation inside the living reproductive tract.

      We have also restructured our manuscript and moved our hypothetical interpretations based on our experimental observations to the discussion section. We hope that these revisions have clarified our claims and that our revised manuscript effectively communicates the importance of our findings and its values in prompting new questions and insight that encourage further studies. We believe that our work clearly demonstrates the importance of sperm/reproductive tract interaction, which cannot be adequately studied in artificial environments, and may become an important guideline for designing future experiments and studies.

      Public Reviews: 

      Reviewer #1 (Public Review): 

      Summary: 

      The authors want to determine the role of the sperm hook of the house mouse sperm in movement through the uterus. The authors are trying to distinguish between two hypotheses put forward by others on the role of the sperm hook: (1) the sperm cooperation hypothesis (the sperm hook helps to form sperm trains) vs (2) the migration hypothesis (that the sperm hook is needed for sperm movement through the uterus). They use transgenic lines with fluorescent labels to sperm proteins, and they cross these males to C57BL/6 females in pathogen-free conditions. They use 2-photon microscopy on ex vivo uteri within 3 hours of mating and the appearance of a copulation plug. There are a total of 10 post-mating uteri that were imaged with 3 different males. They provide 10 supplementary movies that form the basis for some of the quantitative analysis in the main body figures. Their data suggest that the role of the sperm hook is to facilitate movement along the uterine wall. 

      We thank the reviewer for summarizing our work and the critical review of our paper. As summarized, the sperm hook has been primarily associated with the sperm cooperation (sperm hook) hypothesis and the migration hypothesis. However, we would like to emphasize that the aim of our work is not to cross check between the two hypotheses. Our aim was not to disprove either hypothesis, but rather to develop an experimental platform that enables detailed observation of sperm migration dynamics within the live reproductive tract. 

      Through live imaging, we observed both the formation of sperm trains as well as interaction between the sperm and female reproductive tract epithelium. However, in our observations, we could not find advantage in terms of faster movement for the rarely observed sperm trains. While these events were infrequent in our experiments, we are not asserting that the sperm train hypothesis is invalid but rather reporting our observations as is. 

      The main findings of our work lie in the newly observed dynamic behaviours of mouse sperm interacting with the female reproductive tract epithelium. Specifically, tapping and associated guided movement along the uterus wall, anchoring and related resistance to internal fluid flow and migration through the utero-tubal junction, and self-organized behaviour while clinging onto the colliculus tubarius. We have extensively revised the manuscript structure to clarify our findings.

      Strengths: 

      Ex vivo live imaging of fluorescently labeled sperm with 2-photon microscopy is a powerful tool for studying the behavior of sperm. 

      Weaknesses: 

      The paper is descriptive and the data are correlations. 

      The data are not properly described in the figure legends. 

      When statistical analyses are performed, the authors do not comment on the trend that sperm from the three males behave differently from each other. This weakens confidence in the results. For example, in Figure 1 the sperm from male 3613 (blue squares) look different from male 838 (red circles), but all of these data are considered together. The authors should comment on why sperm across males are considered together when the individual data points appear to be different across males. 

      Thank you for your comments and suggestions. We have revisited all figure legends and made the necessary amendments (shown in the red-lined manuscript). Please note that, for a better flow of the paper, the previous Figure 1 has been changed to Figure 2 in the revised manuscript.

      Regarding the analysis using different males, we would like to explain the statistics used. We used generalized linear mixed models to test the effect of the Angle and Distance to the wall on the migration kinetic parameters. The advantage of the generalized linear mixed models is that they consider individual variations in the data as an error term, thereby controlling such individual variations. 

      There are two main factors contributing to individual variations. One is, as you pointed out, the difference in sperm from different males. However, we used genetically similar mice, so genetical variations must be minimal. Nonetheless, there must be individual differences that caused variations including age, stress level as well as body conditions. As these factors cannot be controlled, we used the mixed model approach where individual variations are grouped within the individual. This approach enabled us to test the effect of each explanatory variable (Angle and Distance) within an individual. 

      The second factor that could cause variations is the female oestrous status. To avoid artifacts that could influence sperm behaviour, we did not use any invasive methods, such as hormone injections, to control or induce female oestrus. We controlled for this possible effect by including the mating date as a random effect. Since each female was used only once, the mating date reflects the variation caused by each female.

      To provide further verification that the variation between individual males do not affect our results, we conducted analysis per individual male and mating dates (per each female). As clearly shown, sperm data points from individual males or female also show consistent clear correlations with the distance from the uterus wall. As pointed out, while the mean sperm speed could be different between individuals, they are not the topic we are interested in here. Our interest here is the effect of the distance between sperm and the uterine wall. Additionally, the variation between males is not always larger than those effect of the day (female), which in total suggest that integrating male variation is not essential. We have added this information to Supplementary Figure (Fig. S3) of the revised supplementary materials.

      Moving forward, we can also consider the same analysis for the effects of the distance from wall on sperm SWR and LIN (linearity of forward progression) where no statistical significance was found. As see in the following figures, no statistically significant effect of the distance to wall on SWR and LIN are seen in that the regression lines drawn for each male and mating dates.

      In summary, the statistical approach we used here has successfully reflected variations in sperm kinetics from different males as well as the variance from different females. We hope that our explanations and additional analysis answer your concerns. 

      Movies S8-S10 are single data points and no statistical analyses are performed. Therefore, it is unclear how penetrant the sperm movements are. 

      With respect to Movie S8, Figure 4A and B (Figure 5A and B in the current revised manuscript) depict the trajectories of accumulated spermatozoa (sperm trains) in the female uterus, as shown in Movie S8. We have added this information to the revised figure legend (L 293) for clarity. We could not observe sperm trains that moved faster than single sperms during over 100 hours of observation and collection of over 10TB of images. The three sperm trains presented in Fig. 5B were the sperm trains that moved in the head-forward direction. Most other identifiable trains, or clusters, did not move or could not move forward as their heads were entangled randomly. Although we of course agree that a statistical test for Movie S8 (also Fig. 5B) would be great, due to the small number of sperm trains we found, we could not perform meaningful statistical tests. Instead, we provided all data in the box plots in Fig. 5C so that readers can evaluate and understand our points. We believe that this is a more neutral way of presenting our data rather than providing statistical significance.

      Regarding Movies S9 and S10, we are not entirely sure whether we understood your comments clearly. It would be very helpful if you could point out more specifically to the manuscript with line numbers as we would like to address your concerns and suggestions, and we believe that your input will improve our manuscript. We did not describe the penetration of sperm in these movies. Movies S9 and S10 are newly found sperm behaviours inside the UTJ and Isthmus. We observed that sperm beating is influenced by the width of luminal space as well as internal flow as see in Movies S9 and S10. As our animal model only expresses red fluorescence in the midpiece, accurate beating frequency measurement cannot be performed. However, we can clearly observe that beating is not continuous and almost results in a halt with respect to reproductive tract variations. We revised our description about the findings about beating speed changes in the revised manuscript (LL 305-335).  

      Movies S1B - did the authors also track the movement of sperm located in the middle of the uterus (not close to the wall)? Without this measurement, they can't be certain that sperm close to the uterus wall travels faster. 

      We revised the new Movie S1B to include videos that were used for the sperm migration kinetics analysis in Figure 2 (previously Figure 1). As you can see in the movies, the graph, and statistical analysis, there is a clear trend showing spermatozoa migration is slower as a function of distance from the uterus wall. Regarding your comment with respect to the middle of the uterus (not close to the wall), we have added another movie (Movie S1C) that was acquired at different depths from the wall (going towards the centre of the uterus). As clearly seen in Movie S1c, when imaging deeper into the uterus, there are an increasing number of inactive or slow-moving spermatozoa. Since the diameter of the uterus is easily over 2mm, we currently do not have optical access to exactly the centre of the uterus, but for all depths that are observable, spermatozoa near the wall were clearly faster.

      Movie S5A - is of lower magnitude (200 um scale bar) while the others have 50 and 20 uM scale bars. Individual sperm movement can be observed in the 20 uM (Movie 5SC). If the authors went to prove that there is no upsucking movement of sperm by the uterine contractions, they need to provide a high magnification image. 

      The main focus of video S5A, is the intramural UTJ where spermatozoa are located in rows within narrow luminal space (see Author response image 1). When there is up-suck like sperm passive carriage, there must be sperm movement from the uterus to intramural UTJ as in Author response image 1 left. However, there is no such sperm movement could be seen in our observations, as shown in Movie 5A. Importantly, as you can see in Movie 5A, indicated by an arrow from 5 sec to 6 sec, some spermatozoa are moving downward (see also Author response image 1 right). This is the opposite direction of movement with respect to possible up-suck like sperm carriage. 

      Genetical evidence also support up-suck like passive sperm carriage is not the case for sperm migration from the uterus to UTJ. If environmental up-suck like passive transfer plays an important role, it is unlikely that genetically modified spermatozoa cannot pass the entrance of the intramural UTJ (Nakanishi et al., 2004, Biol. Reprod.; Li et al., 2013, J. Mol. Cell Biol.; Larasati et al., 2020, Biol. Reprod.; Qu et al., 2021, Protein Cell). 

      Author response image 1.

      The left image represents what is expected when up-suck like passive sperm carriage occurs. The right image represents what is actually experimentally observed in the intramural UTJ (see Movie S5A). The direction of the arrowheads indicates the direction of sperm movement.

      Movie S8 - if the authors want to make the case that clustered sperm do not move faster than unclustered sperm, then they need to show Movie S8 at higher magnification. They also need to quantify these data. 

      We understand your concern. As shown in Figure 5B, we included all sperm kinetics data of each sperm train and unlinked spermatozoon around the trains as individual dots. The only analysis we did not conduct was a statistical test with the data as it could be erroneous due to the large sample size difference (3 trains vs 181 unlinked spermatozoa). As the medians of the four sperm kinetic parameters are similar except SWR, we concluded that they are not necessarily faster than unlinked single spermatozoa. Since there is no known advantage to spermatozoa (including sperm trains) with intermediate moving speeds for sperm competition – for example in IVF, success fertilization rate is high when faster and active spermatozoa with normal shape are selected (Vaughan & Sakkas, 2019, Biol. Reprod.) – it is questionable whether there can be an advantage to the formation of sperm trains whose speed is not faster than unlinked spermatozoa in our data.

      However, we do not agree with your comment regarding the need for higher magnification. Measurement of the sperm migration speeds (kinetic parameters) does not require measurement of exact tail movements in this study. Only sperm heads were tracked to measure their trajectory and such tracking was better done at low mag. For example, measuring the speed of a car does not need higher magnifications to visualize the rotation of the wheels. Additionally, including the effect of observation magnification on the sperm kinetic parameters for all 4 GLMM models for Figure 2 (Table S3) does not change the result, which shows that magnification is not a factor that influences our analysis. 

      Movie S9C - what is the evidence that these sperm are dead or damaged? 

      Thank you for your valid comment. We tracked sperm movements for at least 10 minutes and such entangled spermatozoa in the UTJ never became re-active. As you can see in the new Movie S9b, entangled spermatozoa were also acrosome re-acted (green acrosome head is gone) while active spermatozoa are responding to peristaltic movement by exhibiting movements within the same video. However, as you pointed out, we did not measure their viability with appropriate dyes. Although we also considered about extracting these spermatozoa and performing viability tests, we could not come up with a way to specifically extract the exact spermatozoa that were imaged. Considering your comments, we changed the term damaged or dead to inactive in the revised manuscript (LL 313-316, Legend Figure 6D. LL 380-384).

      Movie S10 - both slow- and fast-moving sperm are seen throughout the course of the movie, which does not support the authors' conclusion that sperm tails beat faster over time. 

      There must have been a misunderstanding. We did not indicate that sperm beating got faster over time anywhere in the main manuscript, including the figure legend and related movie captions. As correctly pointed out, the sperm beating speed changes over time (not getting faster over time) and shows a correlation with internal fluid flow and width of luminal space (LL 320-332). Please let us know if you meant something else. 

      Reviewer #2 (Public Review): 

      Summary: 

      The specific objective of this study was to determine the role of the large apical hook on the head of mouse sperm (Mus musculus) in sperm migration through the female reproductive tract. The authors used a custom-built two-photon microscope system to obtain digital videos of sperm moving within the female reproductive tract. They used sperm from genetically modified male mice that produce fluorescence in the sperm head and flagellar midpiece to enable visualization of sperm moving within the tract. Based on various observations, the authors concluded that the hook serves to facilitate sperm migration by hooking sperm onto the lining of the female reproductive tract, rather than by hooking sperm together to form a sperm train that would move them more quickly through the tract. The images and videos are excellent and inspirational to researchers in the field of mammalian sperm migration, but interpretations of the behaviors are highly speculative and not supported by controlled experimentation. 

      Thank you for your critical review and valuable comments on our manuscript. As pointed out, some of our findings and suggestions were largely observation based. However, to the best of our knowledge, many of our observations are novel, particularly in the context of live imaging inside the female uterus and reproductive tract. We believe these observations open doors to many questions and follow up studies that can be envisioned based on our findings, which is what drives science forward. 

      That being said, we entirely agree that many follow up experiments need to be designed and performed, especially to validate the exact molecular mechanisms of the observed dynamics. We acknowledge that it is unfortunate we currently lack the proper molecular experimental toolsets to perform further tests. We have removed much of the hypothetical discussions from the results section and moved them to the discussion section. We hope that our revision more clearly defines the observed experimental data and our interpretations.

      Strengths: 

      The microscope system developed by the authors could be of interest to others investigating sperm migration. 

      The new behaviors shown in the images and videos could be of interest to others in the field, in terms of stimulating the development of new hypotheses to investigate. 

      Weaknesses: 

      The authors stated several hypotheses about the functions of the sperm behaviors they saw, but the hypotheses were not clearly stated or tested experimentally. 

      The hypothesis statements were weakened by the use of hedge words, such as "may". 

      We appreciate your helpful comments and have revised our hypotheses and suggestions accordingly. We have removed instances of “may” or revised it to be more direct. We have also moved most of our interpretations and hypotheses from the results to the discussion section. 

      It is important to note that experimental approaches to test what we suggested from our findings in the current ex-vivo observation platform are not trivial and require extensive investigation of several unknown factors of the female reproductive tract. For instance, obtaining detailed information on the chemical characteristics and fluid dynamics in the female reproductive tract is essential to build a microfluidic channel that accurately resembles the uterus and oviduct, replicating what we found in an extracted living entire organ. This poses a significant challenge and requires collaborative expertise from many labs, which we hope to build in the near future. 

      Furthermore, our biggest concern is that, even if we were to construct the appropriate microfluidic channel to test sperm migration, it is very likely that the sperm behaviours that we observed under natural conditions may not be replicated in artificial environments. This raises questions about whether in-silico or in-vitro findings can truly resemble what we reported here using the ex-vivo observation inside a living organ.

      To share our experience related to this difficulty, at the initial stage of our study, we attempted sperm injection combined with fluorescent beads to visualize the fluid flow, as well as dyeing the female reproductive tract and spermatozoa after mating. However, none of these resulted in meaningful results. Another potential approach to perform similar research regarding our claims is using genetical engineering to indirectly confirm the influence of the sperm hook morphology on sperm behaviour. However, such an approach lacks a mechanical demonstration about how the sperm hook interacts with the female reproductive tract. 

      It is unfortunate that the sperm behaviours that we found and reported here are considered as highly speculative. The main findings of our work lie in the newly observed dynamic behaviours of mouse sperm interacting with the female reproductive tract epithelium. Specifically, these behaviours include tapping and associated guided movement along the uterus wall, anchoring and related resistance to internal fluid flow and migration through the utero-tubal junction, and self-organized behaviour while clinging onto the colliculus tubarius. 

      We have extensively revised the manuscript structure to clarify our findings and integrated our points in the introduction. Although we understand our following hypotheses may be considered speculative and the causative relationship between the sperm hook and its role in sperm migration requires further experimental approaches, we believe that the image-based observation of dynamic behaviours of spermatozoa are solid. We believe our findings will facilitate further studies and discussion in the field of studies on postcopulatory sexual selection in rodents.

      Recommendations for the authors:

      Reviewer #1 (Recommendations For The Authors): 

      The manuscript is written for an expert in a fairly small field. I recommend that the authors rewrite the manuscript to make it more accessible to people outside of the field. These suggestions include 

      (1) Provide a diagram of the female reproductive tract in Figure 1. 

      a. Indicate where sperm enter the tract and the location of the oocyte they are trying to reach. 

      b. Label all areas of the uterus that are mentioned in this study and be consistent about the label. 

      (2) All movies should have a diagram of the location of the uterus that is being imaged. 

      Thank you for the great suggestion. We have added a diagram of the female reproductive tract in the revised Figure 1A. In response to your comments 1a and b, we have indicated such information by including eggs in the ampulla and arrows that indicate sperm migration direction. We have also labelled the name of the specific areas that were studied in the manuscript.

      We are unsure how to integrate the diagram in all movies without reframing the videos, which could cause serious corruption of the files. More importantly, we think that adding the same diagram to all movies may complicate the visuals and disrupt indications and subject in the movie. Instead, we have referred to the common diagram (Figure 1A) in each movie caption, specifying where the video was taken. Thank you for the suggestion. With this information, we hope readers can now more easily understand where we made the observations. 

      (3) The major questions in the field need to be better described in the introduction. 

      Thank you for your valuable suggestions and specific comments which have greatly helped improve our manuscript. We have revised our introduction and discussion sections by adding more literature reviews and integrating studies across a wider range of the postcopulatory sexual selection, as per your suggestion (LL 34-57, LL 385-398).

      (4) The major question that the authors are trying to address should be described in the introduction. 

      Thank you for the helpful suggestion. We have clarified in the introduction that our aim was to contribute to the field of postcopulatory sexual selection in rodents by advancing methodological progress and to stimulate discussion and future research on the function of the sperm hook in murine rodents (LL 76-94) based on our observations.

      (5) A discussion of the sperm hook should be provided. How many species have this structure (or similar structure)? 

      We have integrated your point into the revised discussion section. Essentially, most murine rodent species have sperm hooks (while their exact shapes differ). However, as there are over 500 species and not all of them have been tested, we do not know exactly how many of them have this structure. Therefore, we included paper references that examined species variations in sperm hook characteristics and their possible correlation with sperm competition (LL 385417) in the discussion. Additionally, we also included papers by Breed (2004) and by Roldan et al (1992) that investigated murine rodents with a sperm hook in the introduction section as well (LL 58-61).  

      (6) The figure legends must describe everything in the figure or movie. 

      Thank you for the helpful suggestion. We previously thought that our figure legends may be too long. We have included further information in the figure legends and movie captions. We have also revised the movies by adding some clips following our revision (Movie S1).

      Reviewer #2 (Recommendations For The Authors): 

      Here are some specific concerns I had about the clarity of approach to experiments and interpretations of results. 

      In the Introduction, the authors stated that the study was intended to determine the function of the hooks on the mouse sperm heads. However, in the Results section, the authors did not explain the rationale for the first set of experiments with respect to the overall objective of the study. In this experiment, the authors measured the velocities of sperm swimming in the uterus and found that the sperm moved faster when closer to the uterine wall (VCL, VSL). They concluded that migration along the uterine wall "may" be an efficient strategy for reaching the entrance to the uterotubal junction (UTJ) and did not explain how this related to the function of the hooks. 

      Thank you for your critical comment and guidance. We have changed the order of Figure 1 and Figure 2 and revised the result section to integrate your points. At the initial stage of the study, we expected to find evidence of the function of sperm trains in aiding sperm migration in the female uterus (which has not been observed in the live uterus; previous works were done invitro with extracted sperm from epididymis or uterus after mating). However, what we found was something unexpected: dynamic sperm hook related movements facilitating sperm migration inside the female uterus by playing a mechanical role in sperm interaction with the uterine wall. These results that were presented in the previous Figure 2 has been reorganized as the new Figure 1.

      Based on this observation, our research later moved to clarify whether such sperm-epithelium interaction indeed helps sperm migration. This led us to measure sperm kinetics in relation to their distance and angle to the uterine wall. We have revised our introduction and result parts by integrating these points. We hope that our revision will answer your questions. We have also reduced the use of ‘may’ or ‘can’ in the results section. In the revised manuscript, we have moved such hypotheses to the discussion section and focused on what we observed in the results section.

      The authors proposed that the sperm hook "may" play a crucial role in determining the direction of migration. When sperm encountered a uterine wall, significantly more changed migration direction toward the pro-hook direction than toward the anti-hook direction. In Figure 2B, sperm behavior is not visually understandable nor clearly explained. 

      Thank you for the helpful comments. We have removed “may” and “might” to make our claim clearer and more concise. We have also revised the previous Figure 2B by combining it with the previous Figure 2C (they have been combined into Figure 1C now). We have also revised Figure 1B by increasing the line thickness of the sperm trajectory of the pro-wall-hook direction and added the anti-wall-hook trajectory. We hope that these revisions make the figure easier to understand.

      In Figure 2E, are the authors showing that the tip of the hook is caught between two epithelial cells? Please clarify the meaning of this figure. 

      Please clarify the difference between "tapping" and "anchoring". 

      Thank you for the detailed comments. As you pointed out, we currently have no evidence whether sperm can be caught in epithelia inter-cellular gaps. We have revised this source of confusion by removing the gap in the revised figure (Figure 1E). We have also included the definition of anchoring (LL 142-143) and tapping (LL 128-130). Anchoring facilitates the attachment of sperm to the uterine epithelia. Such anchoring also involves the catching of the sperm head in the inter-mucosal fold or gap, particularly at the entrance of the intramural UTJ at the end of the uterus. Tapping is the interaction between the head hook and epithelia in which the sperm hook is tapping (or patting) on the surface. Sperm tapping can be a byproduct that results from flagella beating when spermatozoa migrate toward the pro-wall-hook direction along the uterine wall (epithelia) or can play some role in sperm migration. As we currently cannot draw a conclusion, we did not integrate the possible function of the tapping in the manuscript.

      The authors proposed that opposite sliding of neighboring mucosal folds lining the UTJ would cause small openings to form, through which only perhaps one sperm at a time could enter and pass through the UTJ into the uterus. This hypothesis was not actually tested. 

      Imaging inside deep tissue is challenging due to light scattering as it penetrates through biological tissue. While this is also true for the uterus, the intramural UTJ is especially difficult to image because the UTJ consists of several thick muscle and cell layers (see Movie S5A). Another challenge is that the peristaltic movement of the UTJ results in constant movement, making continuous tracking of single sperms while passing through the entirety of the UTJ impossible in our current experiments. We have moved this hypothesis to the discussion section and restated that this is a pure hypothetical model (LL 399-406). We hope that our model encourages the community in designing or establishing an improved ex-vivo observation system that may be able to test this hypothetical model in the near future.

      Next, the authors hypothesized that sperm that encounter the small openings in the UTJ may then be guided onward and the hooks could prevent backward slipping. This was also not tested. 

      As you’ve noted, the function of the sperm hook that aids in sliding and preventing backward slipping could not be tested directly in our ex-vivo observation platform that relies on natural movement of the living organ. However, we believe that these limitations also highlight the importance of continued research and the development of more advanced methodologies in this field.

      We would also like to note that we provide direct observations of spermatozoa resisting internal flow due to reproductive tract contractions in Movie S3A, B as well as Movie S5B. We referred to these movies and pointed out the role of anchoring (sperm attachment) in preventing sperm from being squeezing out (LL 140-149, LL 224-241). Unfortunately, we cannot conceive of how this behaviour can be tested additionally in any uterus-resembling microfluidic device or ex-vivo systems. In line with your suggestion, we have rewritten the related result section and moved our related discussions in the result part to the discussion section (LL 224-241, LL 399-417). 

      The authors observed that large numbers of uterine sperm are attached to the entrance of the UTJ. Some sperm clustered and synchronized their flagellar beating. The authors speculated that this behavior served to push sperm in clusters onward through the UTJ. 

      We would like to note that we did not speculate that sperm clustering and their synchronization could serve to push spermatozoa in a cluster to move onward through the UTJ. We only pointed out our observation in recorded videos, that generative flow from the clustered spermatozoa pushed away other spermatozoa as seen in Movie S7 (LL 261-264). Although such sperm cooperation is possible (blocking passage of later sperm), we cannot draw that conclusion from our observation. The possibility you pointed out (pushing sperm onward through the UTJ) was suggested by Qu et al in 2021 [Cooperation-based sperm clusters mediate sperm oviduct entry and fertilization, Protein & Cell] based on their observations on cleared dead reproductive tracts.

      The authors found only a few sperm trains in the uterus, UTJ, and oviduct, so they could not measure sufficient numbers of samples to test whether sperm trains swim faster than single sperm. Without sufficient data, they concluded that the "sperm trains did not move faster than unlinked single spermatozoa." 

      We would like to take this opportunity to clarify our claims. We do not claim that our current experiments can give the final verdict on whether the sperm train hypothesis for faster swimming is correct or not. The phrase “sperm trains did not move faster” was not intended to mean that the sperm train hypothesis is invalid.  We did not draw a conclusion but dryly described the experimental data that we observed (LL 279-286).  We would once again like to emphasize that the main claim of our manuscript is not to rule out the sperm train hypothesis, but to present the various dynamic interactions of the sperm head with the female reproductive tract. To make the statement more balanced, we revised the sentence as “observed sperm trains did not move faster or slower than unlinked single spermatozoa” (LL 281-282).

      The authors hypothesized that the dense sperm clusters at the entrance into the UTJ could prevent the rival's sperm from entering the UTJ (due to plugging entrance and/or creating an outward flow to sweep back the rival's sperm), but they did not test it. 

      We agree that we were not able to test such possible function of the sperm cluster at UTJ entrance. Following your concerns, we revised the result part (LL 256-264) by removing most of our discussions related to the observed phenomena. We also integrated some interpretation rather to the discussion section (LL 421-437) and suggested that future works using appropriate microfluidic channel designs or sequential double mating experiments may be performed for additional tests (LL 443-447). However, we would like to point out that Movie S7C clearly shows surrounding sperms that are swept away from the sperm clusters. Since the sperm density is high, this is almost equivalent to a particle image velocimetry experiment, and we can clearly see the effect of the outward flow generated by the sperm clusters.

    1. Author response:

      The following is the authors’ response to the original reviews

      Public Reviews:

      Reviewer #1 (Public review):

      Weakness#1: The authors claim to have identified drivers that label single DANs in Figure 1, but their confocal images in Figure S1 suggest that many of those drivers label additional neurons in the larval brain. It is also not clear why only some of the 57 drivers are displayed in Figure S1.

      As described in the Results section, we screened 57 GAL4 driver lines based on previous reports. These included drivers that had been shown to label a single dopaminergic neuron (DAN) or a small subset of DANs in the larval or adult brain hemisphere, suggesting potential for specific DAN labeling in larvae.

      In Figure 1, TH-GAL4 was used to cover all neurons in the DL1 cluster, while R58E02 and R30G08 were well known drivers for pPAM. Fly strains in Figure 1h, k, l, and m were reported as single DAN strains in larvae[1], while strains in Figure 1e, f, g were reported identifying only several DANs in adult brains[2,3]. We examined these strains and only some of them labeled single DANs in 3rd instar larval brain hemisphere (Figure 1f, g, h, l and m). Among them, only strains in Figure 1f and h labeled single DAN in the brain hemisphere, without labeling other non-DANs. Other strains labeled non-DANs in addition to single DANs (Figure 1g, l and m). Taking ventral nerve cord (VNC) into consideration, strain in Figure 1h also labeled neurons in VNC (Figure S1e), while strain in Figure 1f did not (Figure S1c).

      In summary, the driver shown in Figure 1f (R76F02AD;R55C10DBD, labeling DAN-c1) is the only line we identified that labels a single DAN in the 3rd instar larval brain hemisphere without additional labeling. The other lines shown in Figure 1 (g, h, l, m) label a single DAN but also include some non-DANs. Figure 1 focuses on strains that label a single or a pair of DANs.

      Labeling patterns for all 57 driver lines are summarized in Table 1. Figure S1 includes representative examples; full confocal images for all screened strains are available upon request, as stated in the figure legend.

      Weakness #2: Critically, R76F02-AD; R55C10-DBD labels more than one neuron per hemisphere in Figure S1c, and the authors cite Xie et al. (2018) to note that this driver labels two DANs in adult brains. Therefore, the authors cannot argue that the experiments throughout their paper using this driver exclusively target DAN-c1.

      Figure S1c shows a single dopaminergic (DA) neuron in each brain hemisphere. While additional GFP-positive signals were occasionally observed, they did not originate from the cell bodies of DA neurons, as these were not labeled by the tyrosine hydroxylase (TH) antibody. These additional GFP signals primarily appeared to be neurites, including axonal terminals, although we cannot rule out the possibility that some represent false-positive signals or weakly stained non-neuronal cell bodies. This interpretation is based on the analysis of 22 third-instar larval brains.

      To clarify this point in the manuscript, we added the following sentence to the Results section: “Based on the analysis of 22 brain samples, we observed this driver strain labels one neuron per hemisphere in the third-instar larval brain (Figure 2a–d, Figure S1c, Table S3).” Additionally, Table S3 was included to summarize the DAN-c1 labeling pattern across all 22 samples. An enlarged inset highlighting GFP-positive signals was also added to Figure S1c.

      Weakness #3: Missing from the screen of 57 drivers is the driver MB320C, which typically labels only PPL1-γ1pedc in the adult and should label DAN-c1 in the larva. If MB320C labels DAN-c1 exclusively in the larva, then the authors should repeat their key experiments with MB320C to provide more evidence for DAN-c1 involvement specifically.

      We thank the reviewer for this insightful suggestion. The MB320C driver primarily labels the PPL1-γ1pedc neuron in the adult brain, along with one or two additional weakly labeled cells. It would indeed be interesting to examine the expression pattern of this driver in third-instar larval brains. If it is found to label only DAN-c1 at this stage, we could consider using it to knock down D2R and assess whether this recapitulates our current findings.

      While we agree that this is a promising direction for future studies, we believe it is not essential for the current manuscript, given the specificity of the DAN-c1 driver (please see our response to Reviewer #3 for details). Nonetheless, we appreciate the reviewer’s suggestion, and we recognize that MB320C could be a valuable tool for future experiments.

      Weakness #4: The authors claim that the SS02160 driver used by Eschbach et al. (2020) labels other neurons in addition to DAN-c1. Could the authors use confocal imaging to show how many other neurons SS02160 labels? Given that both Eschbach et al. and Weber et al. (2023) found no evidence that DAN-c1 plays a role in larval aversive learning, it would be informative to see how SS02160 expression compares with the driver the authors use to label DAN-c1.

      We did not have our own images showing DANs in brains of SS02160 driver cross line. However, Extended Data Figure 1 in the paper of Eschbach et al. shows strongly labeled four neurons on each brain hemisphere[4], indicating that this driver is not a strain only labeling one neuron, DAN-c1.

      Weakness #5: The claim that DAN-c1 is both necessary and sufficient in larval aversive learning should be reworded. Such a claim would logically exclude any other neuron or even the training stimuli from being involved in aversive learning (see Yoshihara and Yoshihara (2018) for a detailed discussion of the logic), which is presumably not what the authors intended because they describe the possible roles of other DANs during aversive learning in the discussion.

      We agree with the reviewer that the terms “necessary” and “sufficient” may be too exclusive and could unintentionally exclude contributions from other neurons. As noted in the Discussion section, we acknowledge that additional dopaminergic neurons may also play roles in larval aversive learning. To reflect this, we have revised our wording to use “important” and “mediates” instead of the more definitive terms “necessary” and “sufficient,” making our conclusions more accurate and appropriately measured.

      Weakness #6: Moreover, if DAN-c1 artificial activation conveyed an aversive teaching signal irrespective of the gustatory stimulus, then it should not impair aversive learning after quinine training (Figure 2k). While the authors interpret Figure 2k (and Figure 5) to indicate that artificial activation causes excessive DAN-c1 dopamine release, an alternative explanation is that artificial activation compromises aversive learning by overriding DAN-c1 activity that could be evoked by quinine.

      This is an excellent point, and we agree that we cannot rule out the possibility that artificial activation interferes with aversive learning by overriding the natural activity of DAN-c1 that would normally be evoked by quinine. The observed results with TRPA1 could potentially be attributed to dopamine depletion, inactivation due to prolonged depolarization, or neural adaptation. However, we believe that our hypothesis - that over-excitation of DAN-c1 impairs learning - is more consistent with our experimental findings and with previously published data. Our rationale is as follows: (1) Associative learning in larvae occurs only when the conditioned stimulus (CS, e.g., an odor such as pentyl acetate) and unconditioned stimulus (US, e.g., quinine) are paired. In wild-type larvae, the CS depolarizes a subset of Kenyon cells in the mushroom body (MB), while the US induces dopamine (DA) release from DAN-c1 into the lower peduncle (LP) compartment (Figure 7a). When both stimuli coincide, calcium influx from CS activation and Gαs signaling via D1-type dopamine receptors activate the MB-specific adenylyl cyclase, rutabaga, which functions as a coincidence detector (Figure 7d). (2) Rutabaga converts ATP to cAMP, activating the PKA signaling pathway and modifying synaptic strength between Kenyon cells and mushroom body output neurons (MBONs) (Figure 7d). These changes in synaptic strength underlie learned behavioral responses to future presentations of the same odor. (3) Our results show that D2R is expressed in DAN-c1, and that D2R knockdown impairs aversive learning. Since D2Rs typically inhibit neuronal excitability and reduce cAMP levels[5], we hypothesize that D2R acts as an autoreceptor in DAN-c1 to restrict DA release. When D2R is knocked down, this inhibition is lifted, leading to increased DA release in response to the US (quinine). The resulting excess DA, in combination with CS-induced calcium influx, would elevate cAMP levels in Kenyon cells excessively - disrupting normal learning processes (Figure 7b). This is supported by studies showing that dunce mutants, which have elevated cAMP levels, also exhibit aversive learning deficits[6]. (4) The TRPA1 activation results are consistent with our over-excitation model. When DAN-c1 was artificially activated at 34°C in the distilled water group, this mimicked the natural activation by quinine, producing an aversive learning response toward the odor (Figure 2k or new Figure 2i, DW group). Similarly, in the sucrose group, artificial activation mimicked quinine, producing a learning response that reflected both appetitive and aversive conditioning (Figure 2k, SUC group). (5) Over-excitation impairs learning in the quinine group. When DAN-c1 was activated during quinine exposure, both artificial and natural activation combined to produce excessive DA release. This over-excitation likely disrupted the cAMP balance in Kenyon cells, impairing learning and resulting in failure of aversive memory formation (Figure 2k, QUI group). This phenotype closely mirrors the effect of D2R knockdown in DAN-c1. (6) Optogenetic activation of DAN-c1 during aversive training similarly produced elevated DA levels due to both natural and artificial stimulation. This again would result in MBN over-excitation and a corresponding learning deficit. When optogenetic activation occurred during non-training phases (resting or testing), no additional DA was released during training, and aversive learning remained intact (Figure 5b). (7) Notably, when optogenetic activation was applied during training, we observed no aversive learning in the distilled water group and no reduction in the sucrose group (Figure 5c, 5d). We interpret this as evidence that the optogenetic stimulation was strong enough to cause elevated DA release in both groups, impairing learning in a manner similar to D2R knockdown or TRPA1 overactivation. (8) We extended this over-excitation framework to directly activate Kenyon cells (MBNs). Since MBNs are involved in both appetitive and aversive learning, their over-excitation disrupted both types of learning (Figure 6), further supporting our hypothesis. In summary, we propose that DAN-c1 activity is tightly regulated by D2R autoreceptors to ensure appropriate levels of dopamine release during aversive learning. Disruption of this regulation - either through D2R knockdown or artificial overactivation of DAN-c1 - results in excessive DA release, over-excitation of Kenyon cells, and impaired learning. This over-excitation model is consistent with both our experimental results and prior literature.

      Weakness #7: The authors should not necessarily expect that D2R enhancer driver strains would reflect D2R endogenous expression, since it is known that TH-GAL4 does not label p(PAM) dopaminergic neurons.

      Just like the example of TH-GAL4, it is possible that the D2R driver strains may partially reflect the expression pattern of endogenous D2R in larval brains. When we crossed the D2R driver strains with the GFP-tagged D2R strain, however, we observed co-localization in DM1 and DL2b dopaminergic neurons, as well as in mushroom body neurons (Figure S3c to h). In addition, D2R knockdown with D2R-miR directly supported that the GFP-tagged D2R strain reflected the expression pattern of endogenous D2R (Figure 4b to d, signals were reduced in DM1). In summary, we think the D2R driver strains supported the expression pattern we observed from the GFP-tagged D2R strain, especially in DM1 DANs.

      Weakness #8: Their observations of GFP-tagged D2R expression could be strengthened with an anti-D2R antibody such as that used by Lam et al., (1999) or Love et al., (2023).

      Love et al. (2023) used the antibody originally described by Draper et al.[6]. We attempted to use the same antibody in our experiments; however, we were unable to detect clear signals following staining. This may be due to a lack of specificity for neurons in the Drosophila larval brain or incompatibility with our staining protocol. Unfortunately, we were unable to locate a copy of the Lam (1999) paper for further reference.

      Weakness #9: Finally, the authors could consider the possibility other DANs may also mediate aversive learning via D2R. Knockdown of D2R in DAN-g1 appears to cause a defect in aversive quinine learning compared with its genetic control (Figure S4e). It is unclear why the same genetic control has unexpectedly poor aversive quinine learning after training with propionic acid (Figure S5a). The authors could comment on why RNAi knockdown of D2R in DAN-g1 does not similarly impair aversive quinine learning (Figure S5b).

      We re-analyzed the data related to DAN-g1. Interestingly, knockdown of D2R in DAN-g1 larvae trained with quinine (QUI) showed a significant difference in response index (R.I.) compared to the distilled water (DW) control group. However, it also differed significantly from the DAN-g1 genetic control group trained with QUI (two-way ANOVA with Tukey’s multiple comparisons, p = 0.0002), while it was not significantly different from the UAS-D2R-miR genetic control group (p = 0.2724). Furthermore, knockdown of D2R in DAN-g1 did not lead to aversive learning deficits when larvae were trained with a different odorant, propionic acid (ProA; Figure S5a). Similarly, using an RNAi line to knock down D2R in DAN-g1 did not result in learning impairment when larvae were trained with pentyl acetate (PA; Figure S5b). These inconsistencies may stem from differences in stimulus intensity across odorants, as well as the variable efficiency of the knockdown strategies (microRNA vs. RNAi). Based on these results, we propose that D2Rs in DAN-g1 may modulate larval aversive learning in a quantitative manner but do not play as critical a role as those in DAN-c1, where knockdown produces a clear qualitative effect. We have added this paragraph to the Discussion section of the manuscript.

      Reviewer #2 (Public review):

      Weakness#1: Is not completely clear how the system DAN-c1, MB neurons and Behavioral performance work. We can be quite sure that DAN-c1;Shits1 were reducing dopamine release and impairing aversive memory (Figure 2h). Similarly, DAN-c1;ChR2 were increasing dopamine release and also impaired aversive memory (Figure 5b). However, is not clear what is happening with DAN-c1;TrpA1 (Figure 2K). In this case the thermos-induction appears to impair the behavioral performance of all three conditions (QUI, DW and SUC) and the behavior is quite distinct from the increase and decrease of dopamine tone (Figure 2h and 5b).

      The study successfully examined the role of D2R in DAN-c1 and MB neurons in olfactory conditioning. The conclusions are well supported by the data, with the exception of the claim that dopamine release from DAN-c1 is sufficient for aversive learning in the absence of unconditional stimulus (Figure 2K). Alternatively, the authors need to provide a better explanation of this point.

      Please refer to our response to Weakness #6 of Reviewer #1 above.

      Reviewer #3 (Public review):

      Weakness #1: It is a strength of the paper that it analyses the function of dopamine neurons (DANs) at the level of single, identified neurons, and uses tools to address specific dopamine receptors (DopRs), exploiting the unique experimental possibilities available in larval Drosophila as a model system. Indeed, the result of their screening for transgenic drivers covering single or small groups of DANs and their histological characterization provides the community with a very valuable resource. In particular the transgenic driver to cover the DANc1 neuron might turn out useful. However, I wonder in which fraction of the preparations an expression pattern as in Figure 1f/ S1c is observed, and how many preparations the authors have analyzed. Also, given the function of DANs throughout the body, in addition to the expression pattern in the mushroom body region (Figure 1f) and in the central nervous system (Figure S1c) maybe attempts can be made to assess expression from this driver throughout the larval body (same for Dop2R distribution).

      We thank the reviewer for the positive comments and thoughtful suggestions.

      Regarding the R76F02AD; R55C10DBD strain, we examined 22 third instar larval brains expressing GFP, Syt-GFP, or Den-mCherry. All brains clearly labeled DAN-c1. In approximately half of the samples, only DAN-c1 was labeled. In the remaining samples, 1 to 5 additional weakly labeled soma were observed, typically without associated neurites. Only 1 or 2 strongly labeled non-DAN-c1 cells were occasionally detected. These additional labeled neurons were rarely dopaminergic. In the ventral nerve cord (VNC), 8 out of 12 samples showed no labeled cells. The remaining 4 samples had 2–4 strongly labeled cells. These results support our conclusion that the R76F02AD; R55C10DBD combination predominantly and specifically labels DAN-c1 in the third instar larval brain. As for the reviewer’s question about the expression pattern of R76F02AD; R55C10DBD and D2R in the larval body, we agree that this is a very interesting avenue for further investigation. However, our current study is focused on the central nervous system and larval learning behaviors. We hope to explore this question more fully in future work.

      We added the following sentence to the Results section: “Based on analysis of 22 brain samples, we believe this driver strain consistently labels one neuron per hemisphere in the third-instar larval brain (Figure 2a - d, Figure S1c, Table S3).” In addition, we included Table S3 to summarize the DAN-c1 labeling patterns observed across these samples.

      Weakness #2: A first major weakness is that the main conclusion of the paper, which pertains to associative memory (last sentence of the abstract, and throughout the manuscript), is not justified by their evidence. Why so? Consider the paradigm in Figure 2g, and the data in Figure 2h (22 degrees, the control condition), where the assay and the experimental rationale used throughout the manuscript are introduced. Different groups of larvae are exposed, for 30min, to an odour paired with either i) quinine solution (red bar), ii) distilled water (yellow bar), or iii) sucrose solution (blue bar); in all cases this is followed by a choice test for the odour on one side and a distilled-water blank on the other side of a testing Petri dish. The authors observe that odour preference is low after odour-quinine pairing, intermediate after odour-water pairing and high after odour-sucrose pairing. The differences in odour preference relative to the odour-water case are interpreted as reflecting odour-quinine aversive associations and odour-sucrose appetitive associations, respectively. However, these differences could just as well reflect non-associative effects of the 30-min quinine or sucrose exposure per se (for a classical discussion of such types of issues see Rescorla 1988, Annu Rev Neurosci, or regarding Drosophila Tully 1988, Behav Genetics, or with some reference to the original paper by Honjo & Furukubo-Tokunaga 2005, J Neurosci that the authors reference, also Gerber & Stocker 2007, Chem Sens).

      As it stands, therefore, the current 3-group type of comparison does not allow conclusions about associative learning.

      We adopted the single-odor larval learning paradigm from Honjo et al., who first developed and validated this method for studying larval olfactory associative learning7,8. To address the reviewer’s concern regarding potential non-associative effects from 30-minute exposure to quinine or sucrose, we refer to multiple lines of evidence provided in Honjo’s studies: (1) Honjo et al. demonstrated that only larvae receiving paired presentations of odor and unconditioned stimulus (quinine or sucrose) exhibited learned responses. Exposure to either stimulus alone, or temporally dissociated presentations, failed to induce any learning response. (2) When tested with a second, non-trained odorant, larvae only responded to the odorant previously paired with the unconditioned stimulus. This rules out generalized olfactory suppression and confirms odor-specific associative learning. (3) Well-characterized learning mutants (e.g., rutabaga, dunce) that show deficits in adult reciprocal odor learning also failed to exhibit learned responses in this single-odor paradigm, further supporting its validity. (4) In our study, we used two distinct odorants (pentyl acetate and propionic acid) and two independent D2R knockdown approaches (UAS-miR and UAS-RNAi). We consistently observed that D2R knockdown in DAN-c1 impaired aversive learning. Importantly, naïve olfactory, gustatory, and locomotor assays ruled out general sensory or motor defects. Comparisons with control groups (odor paired with distilled water) also ruled out non-associative effects such as habituation. Taken together, these results strongly support that the single-odor paradigm is a robust and reliable assay for assessing larval olfactory associative learning in Drosophila. We have added a section in the Discussion to clarify and defend the use of this paradigm in our study.

      Weakness #3: A second major weakness is apparent when considering the sketch in Figure 2g and the equation defining the response index (R.I.) (line 480). The point is that the larvae that are located in the middle zone are not included in the denominator. This can inflate scores and is not appropriate. That is, suppose from a group of 30 animals (line 471) only 1 chooses the odor side and 29, bedazzled after 30-min quinine or sucrose exposure or otherwise confused by a given opto- or thermogenetic treatment, stay in the middle zone... a P.I. of 1.0 would result.

      We gave 5 min during the testing stage to allow the larvae to wander on the testing plate. Under most conditions, more than half of larvae (>50%) will explore around, and the rest may stay in the middle zone (will not be calculated). We used 25-50 larvae in each learning assay, so finally around 10-30 larvae will locate in two semicircular areas. Indeed, based on our raw data, a R.I. of 1 seldom appears. Most of the R.I.s fall into a region from -0.2 to 0.8. We should admit that the calculation equation of R. I. is not linear, so it would be sharper (change steeply) when it approaches -1 and 1. However, as most of the values fall into the region from -0.2 to 0.8, we think ‘border effects’ can be neglected if we have enough numbers of larvae in the calculation (10-30).

      Weakness #4: Unless experimentally demonstrated, claims that the thermogenetic effector shibire/ts reduces dopamine release from DANs are questionable. This is because firstly, there might be shibire/ts-insensitive ways of dopamine release, and secondly because shibire/ts may affect co-transmitter release from DANs.

      Shibire<sup>ts1</sup> gene encodes a thermosensitive mutant of dynamin, expressing this mutant version in target neurons will block neurotransmitter release at the ambient temperature higher than 30C, as it represses vesicle recycling[7]. It is a widely used tool to examine whether the target neuron is involved in a specific physiological function. We cannot rule out that there might be Shibire<sup>ts1</sup> insensitive ways of dopamine release exist. However, blocking dopamine release from DAN-c1 with Shibire<sup>ts1</sup> has already led to learning responses changing (Figure 2h). This result indicated that the dopamine release from DAN-c1 during training is important for larval aversive learning, which has already supported our hypothesis.

      For the second question about the potential co-transmitter release, we think it is a great question. Recently Yamazaki et al. reported co-neurotransmitters in dopaminergic system modulate adult olfactory memories in Drosophila[9], and we cannot rule out the roles of co-released neurotransmitters/neuropeptides in larval learning. Ideally, if we could observe the real time changes of dopamine release from DAN-c1 in wild type and TH knockdown larvae would answer this question. However, live imaging of dopamine release from one dopaminergic neuron is not practical for us at this time. On the other hand, the roles of dopamine receptors in olfactory associative learning support that dopamine is important for Drosophila learning. D1 receptor, dDA1, has been proven to be involved in both adult and larval appetitive and aversive learning[10,11]. In our work, D2R in the mushroom body showed important roles in both larval appetitive and aversive learning (Figure 6a). All this evidence reveals the importance of dopamine in Drosophila olfactory associative learning. In addition, there is too much unknow information about the co-release neurotransmitter/neuropeptides, as well as their potential complex ‘interaction/crosstalk’ relations. We believe that investigation of co-released neurotransmitter/neuropeptides is beyond the scope of this study at this time.

      Weakness #5: It is not clear whether the genetic controls when using the Gal4/ UAS system are the homozygous, parental strains (XY-Gal4/ XY-Gal4 and UAS-effector/ UAS-effector), or as is standard in the field the heterozygous driver (XY-Gal4/ wildtype) and effector controls (UAS-effector/ wildtype) (in some cases effector controls appear to be missing, e.g. Figure 4d, Figure S4e, Figure S5c).

      Almost all controls we used were homozygous parental strains. They did not show abnormal behaviors in either learnings or naïve sensory or locomotion assays. The only exception is the control for DAN-c1, the larvae from homozygous R76F02AD; R55C10DBD strain showed much reduced locomotion speed (Figure S6). To prevent this reduced locomotion speed affecting the learning ability, we used heterozygous R76F02AD; R55C10DBD/wildtype as control, which showed normal learning, naïve sensory and locomotion abilities (Figure 4e to i).

      For Figure 4d, it is a column graph to quantify the efficiency of D2R knockdown with miR. Because we need to induce and quantify the knockdown effect in specific DANs (DM1), only TH-GAL4 can be used as the control group, rather than UAS-D2R-miR. For the missing control groups in Figure S4e and S5c, we have shown them in other Figures (Figure 4e).

      We described this in the Materials and Methods part, “All control strains used in learning assays were homozygous (except DAN-c1×WT), while all experimental groups (D2R knockdown and thermogenetics) used were heterozygous by crossing the corresponding control strains”.

      We also re-organized the Figure S4e and S5c along with the control groups to make it easier to understand.

      Weakness #6: As recently suggested by Yamada et al 2024, bioRxiv, high cAMP can lead to synaptic depression (sic). That would call into question the interpretation of low-Dop2R leading to high-cAMP, leading to high-dopamine release, and thus the authors interpretation of the matching effects of low-Dop2R and driving DANs.

      We appreciate the reviewer’s suggestion. We read through this literature, which also addresses the question we mentioned in the Discussion section, about the discrepancy between the cAMP elevation in the mushroom body neurons and the reduced MBN-MBON synaptic plasticity after olfactory associative learning in Drosophila. The author gave an explanation to the existing D1R-cAMP elevation-MBN-MBON LTD axis, which is really helpful to our understanding about the learning mechanism. However, unfortunately, we do not think this offers a possible explanation for our D2R-related mechanisms. We added this literature into our citation.

      Recommendations for the authors:

      Reviewer #1 (Recommendations for the authors):

      (1) Throughout the behavioral experiments, a defect in aversive learning is defined as a relative increase in the response index (RI) after olfactory training with quinine (red) and a defect in appetitive learning as a relative decrease in RI after training with sucrose (blue). Training with distilled water (yellow) is intended to be a control for comparisons within genotypes/treatment groups but causes interpretation issues if it is also affected by experimental manipulations.

      The authors typically make comparisons between quinine, water, and sucrose within each group, but this often forces readers to infer the key comparisons of interest. For example, the key comparison in Figure 2h is the statistically significant difference between the red groups, which differ only in the temperature used during training. Many other figure panels in the paper would also benefit from more direct statistical comparisons, particularly Figure 2k.

      While I recognize the value of the water control, I strongly recommend that the authors make statistical comparisons directly between genotypes/treatment groups where possible and to interpret results with more caution when the water RI score differs substantially between groups. Also, since the authors are conducting two-way ANOVAs before Dunnett's multiple comparisons tests, they ideally should report the p-value for the main effect of each factor, plus the interaction p-value between the two factors before making multiple comparisons.

      We appreciate the reviewer’s suggestion. In response, we re-analyzed all learning assay data in Figures 2 and 4 using two-way ANOVA followed by Tukey’s multiple comparisons test. Unlike our previous analysis, which only compared each experimental group to its corresponding DW control, we now compared all groups against one another. First, we found that most R.I. values from different temperature conditions (Figure 2) or genotypes (Figure 4) trained with DW were not significantly different, with the exception of the data in Figure 2i (formerly Figure 2k; discussed further below). The R.I. from DAN-c1 × D2R-miR larvae trained with QUI was significantly different from both genotype control groups (DAN-c1 × WT and UAS-D2R-miR), while no significant difference was observed between the two controls trained with QUI. Thus, this more comprehensive statistical approach supports the conclusions we previously reported. Second, as the reviewer noted, the new analysis allows for a more direct interpretation of our findings. For example, in the thermogenetic experiments using the Shibire<sup>ts1</sup> strain, the R.I. of DAN-c1 × UAS-Shibire<sup>ts1</sup> larvae trained with QUI at 34°C was not significantly different from the DW group at 34°C, but was significantly different from the QUI group at 22°C. Both findings support our conclusion that blocking dopamine release from DAN-c1 impairs larval aversive learning (Figure 2f).

      In the dTRPA1 activation experiments, the R.I. of DAN-c1 × UAS-dTRPA1 larvae trained with DW at 34°C was significantly lower than that of the DW group at 22°C and the QUI group at 34°C, but not significantly different from the QUI group at 22°C (Figure 2i). These results indicate that activating DAN-c1 during training is sufficient to drive aversive learning even in the absence of QUI. Interestingly, when DAN-c1 × UAS-dTRPA1 larvae were trained with QUI at 34°C, their R.I. was significantly higher than that of the DW group at 34°C and significantly different from the QUI group at 22°C, but not significantly different from the DW group at 22°C (Figure 2i). We interpret this as evidence that simultaneous activation of DAN-c1 by both QUI and dTRPA1 leads to over-excitation, which in turn impairs aversive learning.

      We have revised the figures (Figures 2, 4, 5, and 6) and updated the corresponding Results sections to reflect this new statistical analysis. Additionally, we now report the p-values for interaction, row factor, and column factor - either in Table S4 (for Figure 2) or in the figure captions for Figures 4, 5, 6, S4, S5, and S7.

      (2) The authors' motivation to find tools that label DANs other than DAN-c1 was unclear until much later in the paper when I saw the screening experiments in Figures S4 and S5. The authors could provide a clearer justification for why they focus on DAN-c1 in Figure 2 rather than another DAN for which they found a specific driver in Figure 1. The motivation for looking at individual pPAM neurons was also unclear.

      We sincerely appreciate the reviewer’s thoughtful suggestion. Our study was initially motivated by the goal of characterizing the expression pattern of D2R in the larval brain. From there, we aimed to identify DAN drivers that label specific pairs of dopaminergic neurons, enabling us to assess the functional role of D2R in distinct DAN subtypes through targeted knockdown experiments. This approach ultimately led us to focus on DAN-c1, as it was the only neuronal population for which D2R knockdown resulted in a learning deficit. We then returned to examine the functional significance of DAN-c1 in aversive learning. While we recognize that a more comprehensive narrative might be desirable, the current structure of our manuscript reflects the most logical progression of our work based on our research priorities and experimental outcomes. We did explore alternative manuscript structures - such as beginning with the D2R expression pattern - but found that the current format best conveys our findings and rtionale.

      Regarding our motivation to study individual PAM neurons: we aimed to identify whether D2R plays a role in a specific pair of pPAM neurons involved in larval appetitive learning. However, we were unable to find a driver that exclusively labels DAN-j1, which we believe to be the key neuron in this context (see Figure 1). As a result, our investigation into appetitive learning did not progress beyond the observation of D2R expression in pPAM neurons (Figure 3d), and we did not proceed with learning assays in this context. While we acknowledge the limitations of our study, we believe that our focus on DAN-c1 is well-justified based on both our findings and the tools currently available. We respectfully note that a major restructuring of the manuscript would not necessarily clarify the rationale for focusing on DAN-c1, and therefore we have maintained the current organization.

      (3) The authors should also double-check and update the expression patterns of the drivers in Table 1 using references such as the FlyLight online resource. For example, MB438B labels PPL1-α'2α2, PPL1-α3, PPL1-γ1pedc according to FlyLight, not just PPL1-γ1pedc as initially reported by Aso and Hattori et al. (2014).

      We appreciate the reviewer’s suggestion. We have double-checked and updated the driver expression patterns in Table 1, using FlyLight data as a reference.

      (4) Interpreting overlaid green-and-red fluorescence confocal images would be difficult for any colorblind readers; I suggest that the authors consider using a more friendly color set.

      We thank the reviewer for the suggestion. In our study, we need three distinct colors to represent different channels. We also tested an alternative color scheme using and cyan , magenta, and yellow (CMY) instead of the standard red, green, and blue (RGB). As a comparison (see below), we used a R76F02AD;R55C10DBD (DAN-c1) GFP-labeled brain as an example. In our evaluation, the RGB combination provided clearer visualization and appeared more natural, while the CMY scheme looked somewhat artificial. Therefore, we decided to retain the original RGB color scheme and did not modify the colors in the figures.

      Author response image 1.

      (5) For Figure 4d, counting each DAN as an individual N would violate the assumption of independence made by the unpaired t test, since multiple DANs are found in each brain and therefore are not independent. Instead, it would be better to count each individual N as the average intensity of the four DANs measured in each brain.

      We revised the analysis of microRNA efficiency by averaging the fluorescence intensity of DANs within each brain, treating each brain as a single sample. Based on this approach, we re-plotted Figure 4d.

      (6) Finally, the authors ought to make it clearer throughout the paper that they have implicated a pair of DAN-c1 neurons in aversive learning, not just a single DAN as currently stated in the title.

      We thank the reviewer for the suggestion about the phrase we are using under this scenario. We have changed all “single neuron” to “a pair of neurons”.

      Reviewer #2 (Recommendations for the authors):

      (1) The results section presents: "Activation of DAN-c1 with dTRPA1 at 34°C during training induced repulsion to PA in the distilled water group (Figure 2k). These data suggested that DAN-c1 excitation and presumably increased dopamine release is sufficient for larval aversive learning in the absence of gustatory pairing."<br /> An alternative interpretation is that 30 min of TrpA activation depletes synaptic vesicle pool, or inactivates neurons because of prolonged depolarization, or DAN shows firing rate adaptation (e.g. see Pulver et al. 2009; doi:10.1152/jn.00071.2009). In such a case DA release would be reduced and not increased. Therefore, the interpretation that DAN-c1 activation is both necessary and sufficient in larval aversive learning is difficult to be sustained.

      In this regard it is important to know how the sensory motor abilities are during a thermos-induction at 34°C during 30 min.

      We thank the reviewer for the thoughtful suggestion. Regarding the concern about potential dopamine depletion or neuronal inactivation, we believe a comparison with the Shibire<sup>ts1</sup> experiments helps clarify the interpretation. Activation of Shibire<sup>ts1</sup> during training with distilled water did not result in aversive learning (Figure 2f), which is a distinct phenotype from that observed with dTRPA1 activation (Figure 2i). This suggests that the phenotypes seen with dTRPA1 activation are not due to reduced dopamine release. Additionally, as the reviewer suggested, we have revised our conclusion to state that “DAN-c1 is important for larval aversive learning,” rather than claiming it is both necessary and sufficient.

      (2) The GRASP system can label the contact of a cell in close proximity like synaptic contacts, but also other situations like no synaptic contact. It would be useful to use a more specific synaptic labelling tool, like the trans-synaptic tracing system (Talay et al., 2017 https://doi.org/10.1016/j.neuron.2017.10.011), which provides a better label of synaptic contact.

      We really appreciate the reviewer’s suggestion. First, we acknowledge that there are four general methods to reveal synaptic connections between neurons: immunohistochemistry (IHC), neuron labeling, viral tracing, GRASP, and electron microscopy (EM). Among these, IHC is not sufficiently convincing, viral tracing is challenging and rarely used in Drosophila, and EM, while the most accurate, is prohibitively expensive for our current goals. For these reasons, we chose the GRASP system to demonstrate the synaptic connections from dopaminergic neurons to the mushroom body. Second, we utilized an activity-dependent version of the GRASP system, linking split-GFP1-10 with synaptic proteins (e.g., synaptobrevin)[12] rather than with cell surface proteins like CD4 or CD8. This version significantly reduces false positive signals compared to the previous version, which was tagged with cell surface proteins. While we admit that this method does not provide as solid evidence of synaptic connections as EM, it is the most efficient method available to us for showing the synaptic connections from dopaminergic neurons to the mushroom body. Finally, we thank the reviewer for suggesting the literature on trans-synaptic tracing methods. Unfortunately, this method is not suitable for our goal, as it labels the entire postsynaptic neuron. In our study, we use GRASP to identify the specific dopaminergic neurons based on the synaptic locations and compartments within the mushroom body lobe. We require a labeling system at the subcellular level because, as noted, DAN-c1 forms synapses specifically in the lower peduncle (LP) of the mushroom body lobe, which is part of the axonal bundles from mushroom body neurons. Using the trans-synaptic tracing method would label the entire mushroom body, making it impossible to distinguish DAN-c1 from other DL1 dopaminergic neurons.

      (3) Previously, Honjo et al (2009) used a petri dish of 8.5 cm and a filter paper for reinforcement of 5.5 cm. In this study the petri dish was 10 cm and the size of the filter paper was not informed. That is important information because it will determine the probability of conditioning.

      A piece of filter paper (0.25cm<sup>2</sup> square) was used to hold odorants in this study. We have added this information to the Materials and Methods.

      (4) Statistic analysis of Behavioral performance of Fig 2H-I was made by ANOVA followed by Dunnett multiple comparisons test. Which was the control group? In each graph 2 independent Dunnett tests were performed against the DW control group?

      We have re-analyzed the data using a two-way ANOVA followed by Tukey’s multiple comparison test, as suggested by Reviewer #1. In Figure 2f-j (previously Figure 2h-l), the DW groups serve as the control groups. In our new analysis, we compared data across all groups using Tukey’s multiple comparison test, with particular focus on comparisons to the corresponding DW control groups.

      (5) The sample size in staining experiments of figures 1-4 were not informed.

      We have added Table S2 in the supplementary materials to provide the N numbers for brain samples used in the figures.

      (6) Color code in Fig 5 is missing, I assumed that is the same as in figure 4e

      We added color code in the figure legend of Figure 5.

      (7) Line 506 "0.1% QH solutions" should be 0.1% QUI solutions

      Changed.

      (8) There is no information on the availability of data

      We added Data Availability Statement: Data will be made available on request.

      Reviewer #3 (Recommendations for the authors):

      (1) Axes of behavioural experiments should better show the full span of possible values (-1;1) to allow a fair assessment.

      We have adjusted the axes in all learning assay graphs to a range from -1 to 1 for consistency and clarity.

      (2) Ns should better be given within the figures.

      We have added Table S2 in the supplementary materials to provide the N numbers for brain samples used in the figures. Additionally, Tables S4 to S6 include the N numbers for the learning assays. While we initially considered including the N numbers within the figure captions, we found it challenging to present this information clearly and efficiently. Therefore, we decided to summarize the N numbers in the tables instead.

      (3) Dot- or box-plots would be better for visualizing the data than means and SEMs.

      We agree with the reviewer’s suggestion. In the behavioral assay graphs, both dot plots and mean ± SEM have been included for better visualization of the data.

      (4) The paper reads as if Dop2R would reduce neuronal activity, rather than "just" cAMP levels. Such a misunderstanding should be avoided.

      We appreciate the reviewer’s comment. Under most conditions, dopamine binding to D2Rs activates the Gαi/o pathway, which inhibits adenylyl cyclase (AC) and reduces cAMP levels. This reduction in cAMP ultimately leads to decreased neuronal activity. In other words, D2R activation typically has an inhibitory effect on neurons. Additionally, D2R can exert inhibitory effects through other signaling pathways, such as the inhibition of voltage-gated associative learning, we continue to emphasize the importance of the D2R-mediated AC-cAMP-PKA signaling pathway. However, we do not rule out the potential involvement of additional signaling pathways, such as inhibition of voltage-gated calcium channels via Gβγ subunits[5]. As noted in the Introduction, dopamine receptors are also involved in other signaling cascades, including PKC, MAPK, and CaMKII pathways. In the context of our study, based on current understanding of molecular signaling in Drosophila olfactory, we still think D2R mediated AC-cAMP-PKA signaling pathway would be the most important one. However, we cannot rule out the involvement of other signaling pathways.

      (5) It would be better if citations were more clearly separated into ones that refer to adult flies versus work on larvae.

      We separated the citations related to adult flies from those working on larvae.

      (6) Line 81-83. DopECR is not found in mammals, is it?

      You are correct. DopECR is not found in mammals. This non-canonical receptor shares structural homology with vertebrate β-adrenergic-like receptors. It can be activated rapidly by dopamine as well as insect ecdysteroids[13,14].

      (7) Line 99: Better "a" learning center (some forms of learning work without mushroom bodies).

      We have revised the text from "the learning center" to "a learning center," as suggested by the reviewer.

      (8) Supplemental figures should be numbered according to the sequence in which they are mentioned in the text.

      We have rearranged the sequence of supplemental figures to match the order in which they are referenced in the text.

      (9) It is striking that dTRPA1-driving DANc1 is punishing in the water condition but that this effect does not summate with quinine punishment (but rather seems to impair it). Maybe you can back this up by ChR- or Chrimson-driving DANc1? Or by silencing DANc1 by GtACR1?

      We appreciate the reviewer’s suggestion. Indeed, we observed similar but not identical results when we used ChR2 to activate DAN-c1 during the training stage (Figure 5b and c). We found that activating DAN-c1 with quinine (QUI) impaired aversive learning (Figure 5b), consistent with our findings using dTRPA1 activation of DAN-c1 when trained in QUI at 34°C (Figure 2i). We propose that the over-excitation of DAN-c1, whether induced by QUI or artificial manipulation (optogenetics and thermogenetics), impairs aversive learning, which aligns with our findings for D2R knockdown (Figure 4e). However, there are some differences between dTRPA1 and ChR2 activation. While dTRPA1 activation induced aversive learning when trained with distilled water (DW) at 34°C (Figure 2i), ChR2 did not induce aversive learning under the same conditions (Figure 5c). We believe this difference is due to the varying activation levels between the two manipulations. Our optogenetic stimulus may have been stronger than the thermogenetic one, potentially leading to over-excitation in the DW group, preventing aversive learning. In the QUI group, the more severe over-excitation impaired aversive learning, producing a phenotype similar to that observed with other over-excitation methods (e.g., thermogenetics or D2R knockdown), where the phenotype reached a maximum level. We have also addressed these points in the Discussion section.

      (10) Unless I got the experimental procedure wrong, isn't it surprising that Figure S7b does not uncover a punishing effect of driving TH-Gals neurons?

      This optogenetic experiment with ChR2 expression in TH-GAL4 neurons was a pioneering attempt to activate DAN-c1 using ChR2. As explained in response to question (9), the failure to observe a punishing effect in the DW group when TH-GAL4 neurons were activated during training may be due to our optogenetic stimulus being too strong. This likely resulted in over-excitation of DAN-c1 (among the neurons labeled by TH-GAL4), impairing aversive learning and preventing the appearance of typical aversive behaviors.

      (11) It seems that Figure1f´ is repeated, in a mirrored manner, in Figure 2e.

      We have removed Figure 2e, as it was deemed redundant and not necessary for this section.

      Reference

      (1) Saumweber, T. et al. Functional architecture of reward learning in mushroom body extrinsic neurons of larval Drosophila. Nat Commun 9, 1104 (2018). https://doi.org/10.1038/s41467-018-03130-1

      (2) Aso, Y. & Rubin, G. M. Dopaminergic neurons write and update memories with cell-type-specific rules. Elife 5 (2016). https://doi.org/10.7554/eLife.16135

      (3) Xie, T. et al. A Genetic Toolkit for Dissecting Dopamine Circuit Function in Drosophila. Cell Rep 23, 652-665 (2018). https://doi.org/10.1016/j.celrep.2018.03.068

      (4) Eschbach, C. et al. Recurrent architecture for adaptive regulation of learning in the insect brain. Nat Neurosci 23, 544-555 (2020). https://doi.org/10.1038/s41593-020-0607-9

      (5) Neve, K. A., Seamans, J. K. & Trantham-Davidson, H. Dopamine receptor signaling. J Recept Signal Transduct Res 24, 165-205 (2004). https://doi.org/10.1081/rrs-200029981

      (6) Draper, I., Kurshan, P. T., McBride, E., Jackson, F. R. & Kopin, A. S. Locomotor activity is regulated by D2-like receptors in Drosophila: an anatomic and functional analysis. Dev Neurobiol 67, 378-393 (2007). https://doi.org/10.1002/dneu.20355

      (7) Honjo, K. & Furukubo-Tokunaga, K. Induction of cAMP response element-binding protein-dependent medium-term memory by appetitive gustatory reinforcement in Drosophila larvae. J Neurosci 25, 7905-7913 (2005). https://doi.org/10.1523/JNEUROSCI.2135-05.2005

      (8) Honjo, K. & Furukubo-Tokunaga, K. Distinctive neuronal networks and biochemical pathways for appetitive and aversive memory in Drosophila larvae. J Neurosci 29, 852-862 (2009). https://doi.org/10.1523/JNEUROSCI.1315-08.2009

      (9) Yamazaki, D., Maeyama, Y. & Tabata, T. Combinatory Actions of Co-transmitters in Dopaminergic Systems Modulate Drosophila Olfactory Memories. J Neurosci 43, 8294-8305 (2023). https://doi.org/10.1523/jneurosci.2152-22.2023

      (10) Selcho, M., Pauls, D., Han, K. A., Stocker, R. F. & Thum, A. S. The role of dopamine in Drosophila larval classical olfactory conditioning. PLoS One 4, e5897 (2009). https://doi.org/10.1371/journal.pone.0005897

      (11) Kim, Y. C., Lee, H. G. & Han, K. A. D1 dopamine receptor dDA1 is required in the mushroom body neurons for aversive and appetitive learning in Drosophila. J Neurosci 27, 7640-7647 (2007). https://doi.org/10.1523/JNEUROSCI.1167-07.2007

      (12) Macpherson, L. J. et al. Dynamic labelling of neural connections in multiple colours by trans-synaptic fluorescence complementation. Nat Commun 6, 10024 (2015). https://doi.org/10.1038/ncomms10024

      (13) Abrieux, A., Duportets, L., Debernard, S., Gadenne, C. & Anton, S. The GPCR membrane receptor, DopEcR, mediates the actions of both dopamine and ecdysone to control sex pheromone perception in an insect. Front Behav Neurosci 8, 312 (2014). https://doi.org/10.3389/fnbeh.2014.00312

      (14) Lark, A., Kitamoto, T. & Martin, J. R. Modulation of neuronal activity in the Drosophila mushroom body by DopEcR, a unique dual receptor for ecdysone and dopamine. Biochim Biophys Acta Mol Cell Res 1864, 1578-1588 (2017). https://doi.org/10.1016/j.bbamcr.2017.05.015

    1. Author Response

      The following is the authors’ response to the original reviews.

      We would like to first thank the Editor as well as the two reviewers for their enthusiasm and careful evaluation of our manuscript. We also appreciate their thoughtful and constructive comments and suggestions. They did, however, have concerns regarding experimental design, data analysis, and over-interpretation of our findings. We endeavored to address these concerns through refinement of our framing, inclusion of additional new analyses, and rewriting some parts of our discussion section. We hope our response can better explain the rationale of our experimental design and data interpretation. In addition, we also acknowledge the limitations of our present study, so that it will benefit future investigations into this topic. Our detail responses are provided below.

      Reviewer #1 (Public Review)

      This study examines whether the human brain uses a hexagonal grid-like representation to navigate in a non-spatial space constructed by competence and trustworthiness. To test this, the authors asked human participants to learn the levels of competence and trustworthiness for six faces by associating them with specific lengths of bar graphs that indicate their levels in each trait. After learning, participants were asked to extrapolate the location from the partially observed morphing bar graphs. Using fMRI, the authors identified brain areas where activity is modulated by the angles of morphing trajectories in six-fold symmetry. The strength of this paper lies in the question it attempts to address. Specifically, the question of whether and how the human brain uses grid-like representations not only for spatial navigation but also for navigating abstract concepts, such as social space, and guiding everyday decision-making. This question is of emerging importance.

      Thanks very much again for the evaluation and comments. Please find our revision plans to each comment below.

      The weak points of this paper are that its findings are not sufficiently supporting their arguments, and there are several reasons for this:

      (1) Does the grid-like activity reflect 'navigation over the social space' or 'navigation in sensory feature space'? The grid-like representation in this study could simply reflect the transition between stimuli (the length of bar graphs). Participants in this study associated each face with a specific length of two bars, and the 'navigation' was only guided by the morphing of a bar graph image. Moreover, any social cognition was not required to perform the task where they estimate the gridlike activity. To make social decision-making that was conducted separately, we do not know if participants needed to navigate between faces in a social space. Instead, they can recall bar graphs associated with faces and compute the decision values by comparing the length of bars. Notably, in the trust game in this study, competence and trustworthiness are not equally important to make a decision (Equation 1). The expected value is more sensitive to one over the other. This also suggests that the space might not reflect social values but perceptual differences.

      The Reviewer raises an interesting point. We apologize for not being clear enough to address this possibility in our original manuscript and we will improve the clarity in our revision. To address this issue, we would like to break it into two sub-questions and answer them separately: 1) Are participants merely memorizing the values associated with each avatar or do they place the avatars on a two-dimensional map in their internal representation. 2) If so, are the two dimensions of this internal representation social dimensions relating to competence and trust or sensory dimensions relating to bar height (i.e., social space or sensory space).

      For the first question, we hope our analysis of the distance effect on the reaction time in the comparison task can address this issue. Specifically, it came from the idea that distance is a measure of similarity between two avatars in the 2D social space. The closer two avatars are, the more similar they are, hence distinguishing them will be harder and result in longer reaction time. If participants are merely memorizing the avatars as six isolated instances without integrating them into a low-dimensional map, then avatars should be equidistant (as if they were lying on the vertices of a 5-simplex), and would not show a distance effect. Therefore, we interpreted the stronger distance effect as a behavioural index of having a better internal map-like representation. This approach is adopted from the work by Park et al. (2020), where they used the distance effect to demonstrate human brains map abstract relationships among entities from piecemeal learning.

      For the second question of ‘social space’ vs. ‘sensory space’, our study adopted the paradigm developed by, in which they used a similar way to construct a conceptual space and found that such space can be represented with grid-like code in the entorhinal and prefrontal cortex. We stayed close to the original design by Constantinescu et al. (2016) and hoped that our work could provide, to some extent, a close replication of their result but using non-spatial social concepts instead. Indeed, this led to the limitation of our study that participants are passively traversing the artificial space rather than actively navigating in the space to make decisions/inferences. And we did not find sufficient evidence as reported in previous grid-like coding fMRI studies. This may have to do with low signal quality in the medial temporal region, we are not entirely sure. Nevertheless, we don’t think our findings contradict or disprove previous findings in any way. Here we would also like to point to the work by Park et al. (2021). Their task involves making novel inferences in a 2D social hierarchy space and found that grid-like code in the entorhinal cortex and medial prefrontal cortex support such novel inferences. Hence, we argue that results from these studies and partial evidence from our study collectively support the idea that the entorhinal is important for representing abstract knowledge (spatial and non-spatial).

      (2) Does the brain have a common representation of faces in a social space? In this study, participants don't need to have a map-like representation of six faces according to their levels of social traits. Instead, they can remember the values of each trait. The evidence of neural representations of the faces in a 2-dimensional social space is lacking. The authors argued that the relationship between the reaction times and the distances between faces provides evidence of the formation of internal representations. However, this can be found without the internal representation of the relationships between faces. If the authors seek internal representations of the faces in the brain, it would be important to show that this representation is not simply driven by perceptual differences between bar graphs that participants may recall in association with each face.

      Considering these caveats, it is hard for me to agree if the authors provide evidence to support their claims.

      With regard to the common representation of faces, this is a potential limitation of our paradigm because our current task design didn’t include a stage of face presentation to properly test this question. With regard to the asymmetry between the two dimensions in determining expected value. We think that the prerequisite for identifying six-fold grid-like coding is to have an abstract space formed by orthogonal dimensions, i.e., competence and trustworthiness in our task are not correlated. In addition, the scanner task does not require computation of expected value. However, we do think that it is worth investigating whether the extent to which each dimension contributes to decision-making and inference will distort the grid-like representation of the map. Our prediction is that the entorhinal cortex will maintain a representation of the map invariant to this aspect so that it can support inferences in different contexts where different weights may be assigned to different dimensions. But this will be an interesting hypothesis for future studies to test. We hope that our revision plans with above considerations could address the Reviewer’s comments.

      Reviewer #2 (Public Review)

      Summary:

      In this work, Liang et al. investigate whether an abstract social space is neurally represented by a grid-like code. They trained participants to 'navigate' around a two-dimensional space of social agents characterized by the traits of warmth and competence, then measured neural activity as participants imagined navigating through this space. The primary neural analysis consisted of three procedures: 1) identifying brain regions exhibiting the hexagonal modulation characteristic of a grid-like code, 2) estimating the orientation of each region's grid, and 3) testing whether the strength of the univariate neural signal increases when a participant is navigating in a direction aligned with the grid, compared to a direction that is misaligned with the grid.

      From these analyses, the authors find the clearest evidence of a grid-like code in the prefrontal cortex and weaker evidence in the entorhinal cortex.

      Strengths:

      The work demonstrates the existence of a grid-like neural code for a socially-relevant task, providing evidence that such coding schemes may be relevant for a variety of two-dimensional task spaces.

      Thank you very much again for your careful evaluation and thoughtful comments. Please find our response to the comments below.

      Weaknesses:

      In various parts of this manuscript, the authors appear to use a variety of terms to refer to the (ostensibly) same neural regions: prefrontal cortex, frontal pole, ventromedial prefrontal cortex (vmPFC), and orbitofrontal cortex (OFC). It would be useful for the authors to use more consistent terminology to avoid confusing readers.

      Thanks for pointing out the use of terms, we will try to improve that in the revision of our manuscript.

      Claims about a grid code in the entorhinal cortex are not well-supported by the analyses presented. The whole-brain analysis does not suggest that the entorhinal cortex exhibits hexagonal modulation; the strength of the entorhinal BOLD signal does not track the putative alignment of the grid code there; multivariate analyses do not reveal any evidence of a grid-like representational geometry.

      On a conceptual level, it is not entirely clear how this work advances our understanding of gridlike encoding of two-dimensional abstract spaces, or of social cognition. The study design borrows heavily from Constantinescu et al. 2016, which is itself not an inherent weakness, but the Constantinescu et al. study already suggests that grid codes are likely to underlie two-dimensional spaces, no matter how abstract or arbitrary. If there were a hypothesis that there is something unique about how grid codes operate in the social domain, that would help motivate the search for social grid codes specifically, but no such theory is provided. The authors do note that warmth and competence likely have ecological importance as social traits, but other past studies have used slightly different social dimensions without any apparent loss of generality (e.g., Park et al. 2021). There are some (seemingly) exploratory analyses examining how individual difference measures like social anxiety and avoidance might affect the brain and behavior in this study, but a strong theoretical basis for examining these particular measures is lacking.

      We acknowledge that we used very similar dimensions to the work by Park et al. (2021). While Park and colleagues (2021) took a more innovative and rigorous approach, we tried to stay close to the original design by Constantinescu et al. (2016) with the hope that our work could provide, to some extent, a close replication of their result. Our data was collected before the 2021 paper came out and as the comment points out, we did not find as complete and convincing evidence as in these previous grid-like coding fMRI papers. This may be due to low signal quality in the medial temporal region, we are not entirely sure. But we don’t think our current findings can contradict or disprove previous findings in any way.

      I found it difficult to understand the analyses examining whether behavior (i.e., reaction times) and individual difference measures (i.e., social anxiety and avoidance) can be predicted by the hexagonal modulation strength in some region X, conditional on region X having a similar estimated grid alignment with some other region Y. It is possible that I have misunderstood the authors' logic and/or methodology, but I do not feel comfortable commenting on the correctness or implications of this approach given the information provided in the current version of this manuscript.

      We apologize for not being clear enough in the manuscript and we will improve the clarity in our revision. This exploratory analysis aims to examine if there is any correlation between the strength of grid-like representation of social value map and behavioral indicators of map-like representation; and test if there are any correlation between the strength of grid-like representation of this social value map and participants’ social trait. For the behavioral indicator, we used the distance effect in the reaction time of the comparison task outside the scanner. The closer a pair of avatars are, the more similar they are, hence distinguishing them will be harder and results in longer reaction time when making comparison judgement. If participants are merely memorizing the avatars as six isolated instances without integrating them into a map, all avatars should be equidistant and there wouldn’t be a distance effect. We interpreted stronger grid-like activity as a neural index of better representation of the 2D social space, and we interpreted stronger distance effect as a behavioral index of having better internal map-like representation.

      It was puzzling to see passing references to multivariate analyses using representational similarity analysis (RSA) in the main text, given that RSA is only used in analyses presented in the supplementary material.

      We speculate if RSA in entorhinal ROI would be more sensitive than the wholebrain univariate analysis to identify grid-like code because a previous paper on grid-like code in olfactory space (Bao et al., 2019) didn’t identify grid-like representation with univariate analysis but identified it with RSA analysis. However, we failed to find evidence of grid-like code in the entorhinal ROI aligned to its own putative grid orientation with the RSA approach. We reported this result in the main text to show that we carried out a relatively thorough investigation to test the hypothesis using various approaches and decided to add references to the RSA approach in the main text as well.

      Reviewer #3 (Public Review)

      Liang and colleagues set out to test whether the human brain uses distance and grid-like codes in social knowledge using a design where participants had to navigate in a two-dimensional social space based on competence and warmth during an fMRI scan. They showed that participants were able to navigate the social space and found distance-based codes as well as grid-like codes in various brain regions, and the grid-like code correlated with behavior (reaction times).

      On the whole, the experiment is designed appropriately for testing for distant-based and grid-like codes and is relatively well-powered for this type of study, with a large amount of behavioral training per participant. They revealed that a number of brain regions correlated positively or negatively with distance in the social space, and found grid-like codes in the frontal polar cortex and posterior medial entorhinal cortex, the latter in line with prior findings on grid-like activity in the entorhinal cortex. The current paper seems quite similar conceptually and in design to previous work, most notably by Park et al., 2021, Nature Neuroscience.

      Thanks very much again for your careful evaluation and comments. Please find our response to the comments below.

      Below, I raise a few issues and questions on the evidence presented here for a grid-like code as the basis of navigating abstract social space or social knowledge.

      (1) The authors claim that this study provides evidence that humans use a spatial / grid code for abstract knowledge like social knowledge.

      This data does specifically not add anything new to this argument. As with almost all studies that test for a grid code in a similar "conceptual" space (not only the current study), the problem is that when the space is not a uniform, square/circular space, and 2-dimensional then there is no reason the code will be perfectly grid-like, i.e., show six-fold symmetry. In real-world scenarios of social space (as well as navigation, semantic concepts), it must be higher dimensional - or at least more than two-dimensional. It is unclear if this generalizes to larger spaces where not all part of the space is relevant. Modelling work from Tim Behrens' lab (e.g., Whittington et al., 2020) and Bradley Love's lab (e.g., Mok & Love, 2019) have shown/argued this to be the case. In experimental work, like in mazes from the Mosers' labs (e.g., Derdikman et al., 2009), or trapezoid environments from the O'Keefe lab (Krupic et al., 2015), there are distortions in mEC cells, and would not pass as grid cells in terms of the six-fold symmetry criterion.

      The authors briefly discuss the limitations of this at the very end but do not really say how this speaks to the goal of their study and the claim that social space or knowledge is organized as a grid code and if it is in fact used in the brain in their study and beyond. This issue deserves to be discussed in more depth, possibly referring to prior work that addressed this, and raising the issue for future work to address the problem - or if the authors think it is a problem at all.

      Thanks very much for the references to the papers that we haven’t considered enough in our discussion. We will endeavour to discuss the topic in more depth in our revision. In summary, we raise this discussion point because various research groups have found gridlike representations in 2D artificial conceptual space. We think that the next step for a stronger claim would be to find the representation of more spontaneous non-spatial maps.

      Data and analysis

      (2) Concerning the negative correlation of distance with activation in the fusiform gyrus and visual cortex: this is a slightly puzzling but potentially interesting finding. However, could this be related to reaction times? The larger the distance, the longer the reaction times, so the original finding might reflect larger activations with smaller distances.

      Thanks very much for the suggestion. However, we didn’t find a correlation between response time in the choice stage in the scanner task and the negative distance activation in the fusiform gyrus (Figures below). Meanwhile, the morph period in each trial remains the same, the negative correlation of distance with activation in the fusiform gyrus could also be interpreted as a positive correlation of morphing speed with activation in the fusiform gyrus. Indeed, stronger negative activation indicates larger activation for smaller distances, but we are uncertain what it indicates concerning the functional role of Fusiform in our current task.

      Author response image 1.

      (3) Concerning the correlation of grid-like activity with behavior: is the correlation with reaction time just about how long people took (rather than a task-related neural signal)? The authors have only reported correlations with reaction time. The issue here is that the duration of reaction times also relates to the starting positions of each trial and where participants will navigate to. Considering the speed-accuracy tradeoff, could performance accuracy be negatively correlated with these grid consistency metrics? Or it could be positively correlated, which would suggest the grid signal reflects a good representation of the task.

      We apologize for not being clear enough in the manuscript and we will improve the clarity in our revision. The reaction time used to calculate the distance effect is from a task outside the scanner. The closer a pair of avatars are, the more similar they are, hence distinguishing them will be harder and results in longer reaction time when making comparison judgement. If participants are merely memorizing the avatars as six isolated instances without integrating them into a map, all avatars should be equidistant and there wouldn’t be a distance effect. We interpreted stronger grid-like activity as a neural index of better representation of the 2D social space, and we interpreted stronger distance effect as a behavioural index of having better internal map-like representation. This was the motivation behind this analysis.

      References

      Bao, X., Gjorgieva, E., Shanahan, L. K., Howard, J. D., Kahnt, T., & Gottfried, J. A. (2019). Grid-like Neural Representations Support Olfactory Navigation of a Two-Dimensional Odor Space. Neuron, 102(5), 1066-1075 e1065. https://doi.org/10.1016/j.neuron.2019.03.034

      Constantinescu, A. O., O'Reilly, J. X., & Behrens, T. E. J. (2016). Organizing conceptual knowledge in humans with a gridlike code. Science,352(6292), 1464-1468. https://doi.org/10.1126/science.aaf0941

      Park, S. A., Miller, D. S., & Boorman, E. D. (2021). Inferences on a multidimensional social hierarchy use a grid-like code. Nat Neurosci, 24(9), 1292-1301. https://doi.org/10.1038/s41593-02100916-3

      Park, S. A., Miller, D. S., Nili, H., Ranganath, C., & Boorman, E. D. (2020). Map Making: Constructing, Combining, and Inferring on Abstract Cognitive Maps. Neuron, 107(6), 1226-1238 e1228. https://doi.org/10.1016/j.neuron.2020.06.030

    1. Author response:

      The following is the authors’ response to the original reviews.

      Public Reviews:

      Reviewer #1 (Public review):

      The Bagnat and Rawls groups' previous published work (Park et al., 2019) described the kinetics and genetic basis of protein absorption in a specialized cell population of young vertebrates termed lysosome-rich enterocytes (LREs). In this study they seek to understand how the presence and composition of the microbiota impacts the protein absorption function of these cells and reciprocally, how diet and intestinal protein absorption function impact the microbiome.

      Strengths of the study include the functional assays for protein absorption performed in live larval zebrafish, which provides detailed kinetics on protein uptake and degradation with anatomic precision, and the gnotobiotic manipulations. The authors clearly show that the presence of the microbiota or of certain individual bacterial members slows the uptake and degradation of multiple different tester fluorescent proteins.

      To understand the mechanistic basis for these differences, the authors also provide detailed single-cell transcriptomic analyses of cells isolated based on both an intestinal epithelial cell identity (based on a transgenic marker) and their protein uptake activity. The data generated from these analyses, presented in Figures 3-5, are valuable for expanding knowledge about zebrafish intestinal epithelial cell identities, but of more limited interest to a broader readership. Some of the descriptive analysis in this section is circular because the authors define subsets of LREs (termed anterior and posterior) based on their fabp2 expression levels, but then go on to note transcriptional differences between these cells (for example in fabp2) that are a consequence of this initial subsetting.

      Inspired by their single-cell profiling and by previous characterization of the genes required for protein uptake and degradation in the LREs, the authors use quantitative hybridization chain reaction RNA-fluorescent in situ hybridization to examine transcript levels of several of these genes along the length of the LRE intestinal region of germ-free versus mono-associated larvae. They provide good evidence for reduced transcript levels of these genes that correlate with the reduced protein uptake in the mono-associated larval groups.

      The final part of the study (shown in Figure 7) characterized the microbiomes of 30-day-old zebrafish reared from 6-30 days on defined diets of low and high protein and with or without homozygous loss of the cubn gene required for protein uptake. The analysis of these microbiomes notes some significant differences between fish genotypes by diet treatments, but the discussion of these data does not provide strong support for the hypothesis that "LRE activity has reciprocal effects on the gut microbiome". The most striking feature of the MDS plot of Bray Curtis distance between zebrafish samples shown in Figure 7B is the separation by diet independent of host genotype, which is not discussed in the associated text. Additionally, the high protein diet microbiomes have a greater spread than those of the low protein treatment groups, with the high protein diet cubn mutant samples being the most dispersed. This pattern is consistent with the intestinal microbiota under a high protein diet regimen and in the absence of protein absorption machinery being most perturbed in stochastic ways than in hosts competent for protein uptake, consistent with greater beta dispersal associated with more dysbiotic microbiomes (described as the Anna Karenina principle here: https://pubmed.ncbi.nlm.nih.gov/28836573/). It would be useful for the authors to provide statistics on the beta dispersal of each treatment group.

      Overall, this study provides strong evidence that specific members of the microbiota differentially impact gene expression and cellular activities of enterocyte protein uptake and degradation, findings that have a significant impact on the field of gastrointestinal physiology. The work refines our understanding of intestinal cell types that contribute to protein uptake and their respective transcriptomes. The work also provides some evidence that microbiomes are modulated by enterocyte protein uptake capacity in a diet-dependent manner. These latter findings provide valuable datasets for future related studies.

      We thank the Reviewer for their thorough and kind assessment. We appreciate the suggestion for edits and for pointing out areas that needed further clarification.

      One point in need of further explanation is the use fabp6 (referred to as fabp2 by the reviewer) to define anterior LREs and their gene expression pattern, which includes high levels of fabp6, something that was deemed a “circular argument” by the reviewer.  The rationale for using fabp6 as a reference is that we were able to define its spatial pattern in relation to other LRE markers and the neighboring ileocyte population using transgenic markers (Lickwar et al., 2017; Wen et al., 2021). Thus, far from being a circular argument, using fabp6 allowed us to identify other markers that are differentially expressed between anterior and posterior LREs, which share a core program that we highlight in our study. In the revised manuscript, we clarified this point (lines 166 – 169).

      We followed the Reviewer’s suggestion to test if LRE activity and dietary protein affected beta dispersal. Our analyses revealed that beta dispersion was not significantly different between our experimental conditions. We added details about this analysis (lines 384 – 386) and a new supplemental figure panel (Figure S7C).

      Reviewer #2 (Public review):

      Summary:

      The authors set out to determine how the microbiome and host genotype impact host protein-based nutrition.

      Strengths:

      The quantification of protein uptake dynamics is a major strength of this work and the sensitivity of this assay shows that the microbiome and even mono-associated bacterial strains dampen protein uptake in the host by causing down-regulation of genes involved in this process rather than a change in cell type.

      The use of fluorescent proteins in combination with transcript clustering in the single cell seq analysis deepens our understanding of the cells that participate in protein uptake along the intestine. In addition to the lysozome-rich enterocytes (LRE), subsets of enteroendocrine cells, acinar, and goblet cells also take up protein. Intriguingly, these non-LRE cells did not show lysosomal-based protein degradation; but importantly analysis of the transcripts upregulated in these cells include dab2 and cubn, genes shown previously as being essential to protein uptake.

      The derivation of zebrafish mono-associated with single strains of microbes paired with HCR to localize and quantify the expression of host protein absorption genes shows that different bacterial strains suppress these genes to variable extents.

      The analysis of microbiome composition, when host protein absorption is compromised in cubn-/- larvae or by reducing protein in the food, demonstrates that changes to host uptake can alter the abundance of specific microbial taxa like Aeramonas.

      Weaknesses:

      The finding that neurons are positive for protein uptake in the single-cell data set is not adequately discussed. It is curious because the cldn:GFP line used for sorting does not mark neurons and if the neurons are taking up mCherry via trans-synaptic uptake from EECs, those neurons should be mCherry+/GFP-; yet methods indicate GFP+ and GFP+/mCherry+ cells were the ones collected and analyzed.

      We thank the Reviewer for the kind and positive assessment of our work, for suggestions to improve the accessibility and clarity of the manuscript, and for pointing out an issue related to a neuronal population that needed further clarification.

      It turns out that there is a population of neurons that express cldn15la. They are not easily visualized by microscopy because IECs express this gene much more highly. However, the endogenous cldn15la transcripts can be found in neurons as shown in a recently published dataset (PMID: 35108531) as well as in this study We added a discussion point to clarify this issue (lines 463 – 465).

      Reviewer #3 (Public review):

      Summary:

      Childers et al. address a fundamental question about the complex relationship within the gut: the link between nutrient absorption, microbial presence, and intestinal physiology. They focus on the role of lysosome-rich enterocytes (LREs) and the microbiota in protein absorption within the intestinal epithelium. By using germ-free and conventional zebrafishes, they demonstrate that microbial association leads to a reduction in protein uptake by LREs. Through impressive in vivo imaging of gavaged fluorescent proteins, they detail the degradation rate within the LRE region, positioning these cells as key players in the process. Additionally, the authors map protein absorption in the gut using single-cell sequencing analysis, extensively describing LRE subpopulations in terms of clustering and transcriptomic patterns. They further explore the monoassociation of ex-germ-free animals with specific bacterial strains, revealing that the reduction in protein absorption in the LRE region is strain-specific.

      Strengths:

      The authors employ state-of-the-art imaging to provide clear evidence of the protein absorption rate phenotype, focusing on a specific intestinal region. This innovative method of fluorescent protein tracing expands the field of in vivo gut physiology.

      Using both conventional and germ-free animals for single-cell sequencing analysis, they offer valuable epithelial datasets for researchers studying host-microbe interactions. By capitalizing on fluorescently labelled proteins in vivo, they create a new and specific atlas of cells involved in protein absorption, along with a detailed LRE single-cell transcriptomic dataset.

      Weaknesses:

      While the authors present tangible hypotheses, the data are primarily correlative, and the statistical methods are inadequate. They examine protein absorption in a specific, normalized intestinal region but do not address confounding factors between germ-free and conventional animals, such as size differences, transit time, and oral gavage, which may impact their in vivo observations. This oversight can lead to bold conclusions, where the data appear valuable but require more nuance.

      The sections of the study describing the microbiota or attempting functional analysis are elusive, with related data being overinterpreted. The microbiome field has long used 16S sequencing to characterize the microbiota, but its variability due to experimental parameters limits the ability to draw causative conclusions about the link between LRE activity, dietary protein, and microbial composition. Additionally, the complex networks involved in dopamine synthesis and signalling cannot be fully represented by RNA levels alone. The authors' conclusions on this biological phenomenon based on single-cell data need support from functional and in vivo experiments.

      We thank the Reviewer for their assessment and for pointing out some areas that needed to be explained better and/or discussed.

      The Reviewer mentions some potential confounding factors (ie., size differences, transit time, oral gavage) in the gnotobiology experiments. We would like to convey that these aspects have been addressed in our experimental design and are now clarified in the revised manuscript: 1- larval sizes were recorded and found to be similar between GF and monoassociated larvae (Figure S6A); 2- while intestinal transit time may be affected by microbes and is a topic of interest, in our assay luminal mCherry cargo is present at high levels throughout the gut and is not limiting at any point during the experiment; 3- gavage, which is necessary for quantitative assays, is indeed an experimental manipulation that may somehow alter the subjects (the same is true for microscopy and virtually any research method). However, it cannot explain differences between GF and CV or alter our conclusions via microbial or dietary effects. We now elaborate the former point in the revised discussion (line 426). A new panel has been added for Fig.S6 to show that standard length was similar in GF and monoassociated larvae (Figure S6A).

      We are aware that microbial community composition is often highly variable between experiments and this necessitates adequately high biological replication and inclusion of internal controls to allow conclusions to be drawn. Nevertheless, studies evaluating the utility of 16S rRNA gene sequencing have found that this analysis reveals important impacts of environmental factors on the gut microbiome (PMIDs: 21346791, 31409661, 31324413). Our results provide further evidence that 16S rRNA gene sequencing remains a useful method to detect perturbations to the zebrafish gut microbiome. Reproducing previous findings, we detected many of the core zebrafish microbiota strains in our samples that have been identified by other studies (PMIDs: 26339860, 21472014, 17055441). To ensure the robustness of our results, we included several biological replicates for each condition, co-housed genotypes and included large sample sizes to minimize environmental variability between groups. In response to this reviewer concern, we have added a supplemental beta diversity plot and statistical analyses showing that the microbiomes in our larvae were significantly different from the diets or tank water (Figure S7A). This analysis shows that the host environment influenced microbial community composition (lines 376 – 378). We also added an additional supplemental panel and performed analysis showing that the experimental replicates (i.e., different tanks) were not a significant source of variation in this study (lines 378 – 380) (Figure S7B). This result underscores that the microbiota in these larvae were influenced by both the host and diet.

      Regarding dopamine pathways, we acknowledge that it involves complex biology that will require dedicated studies. In this work, we simply point out gene expression patterns we find interesting as they may inform future studies.

      Finally, the Reviewer mentions the use of inadequate statistical methods for some analyses without specifying or indicating alternative analyses, only the need to justify the use of two-way ANOVA is made explicit. In this point, we respectfully disagree and would like to emphasize that we use statistical methods that are standard in the field (PMID: 37707499). We nevertheless added a justification for the use of two-way ANOVA where appropriate (lines 635-637, 653-654, 773-776). The two-way ANOVA test was to compare fluorescence profiles of gavages cargoes or HCR probes along the length of the LRE region. This test accounts for differences in fluorescence between experimental conditions in segments (30 μm) along the LRE region (~300 μm). This allows us to capture differences in fluorescence between experimental conditions while accounting for heterogeneity in the LRE region. Please see our comment below for more information about our use of the 2-way ANOVA.

      Recommendations for the authors:

      Reviewer #1 (Recommendations for the authors):

      Please provide in the materials and methods the strain identifiers and sources of the bacteria used in the study.

      Thank you for the suggestions. Strain identifiers and source information were added to the methods (lines 576-579).

      Reviewer #2 (Recommendations for the authors):

      (1) This is a very satisfying and thorough analysis of the reciprocal influence of diet, microbiome, and host genotype on protein absorption by the host. Below I make suggestions that mainly relate to making the paper more accessible to a broader audience.

      (2) Line 233 Starts a section that reports the findings of the scRNA dataset. The writing is inconsistent with respect to how the genes are listed: whether abbreviation only or spelled out followed by abbreviation. I prefer the latter. For example, slc10a2 is a bile acid Na cotransporter but for those not in the know, they would have to look this up. Perhaps adding a supplementary table that provides a gene list of those discussed in the text with abbreviation/spelled-out, and KEGG terms.

      Thank you for pointing out inconsistent gene labeling. We have revised the text with spelled out gene names followed by abbreviations.

      (3) Line 461 Where did the neurons come from when you were sorting cldn+ cells?

      Neuronal expression of cldn15la was detected in our data and other published datasets (PMID: 37995681, 35108531). We added a note to the text clarifying that neuronal cells can express cldn15la (lines 463-465).

      (4) Line 561 1x tricaine should be converted to percentage in solution or concentration throughout.

      The tricaine concentration was 0.2 mg/mL. We added this detail to the methods (line 596).

      (5) Line 612 Please clarify how normalizations are carried out: is it to the peak value in the germ-free condition? CV never reaches 1.

      AUC values were normalized to the peak value in the GF condition at 60 minutes PG. We clarified this step in the methods (lines 618-619).

      (6) Line 654-663 I think mCherry here should be mTourquoise?

      Thank you for catching this typo. We corrected it in the text.

      (7) In Figure 1 Please consider adding a color so that magenta does not represent BOTH germ-free AND mCherry.

      Due to the many colors of fluorescent proteins and HCR probes in this paper, we were not able to find an alternative plot line color to represent GF.

      (8) In Figure 2 I suggest consistency with respect to the order you present GF/CV

      Figure 1 GF->CV

      Figure 2 CV->GF

      My preference is GF->CV

      Images in Figure 2 were re-ordered following reviewer’s recommendation.

      Here, 20 minute time point also appears qualitatively different between GF and CV.

      There can be slight differences in LREs between individuals. These images were selected because they represented the average differences in the amount of mTurquoise degradation activity that occurred between 20 – 60 minutes post-flushing in the GF and CV conditions.

      In Figure 3E Figure legend refers to being able to see BSA in vacuoles. The image should be modified to show this- currently too small.

      In response, we enlarged the confocal microscopy images showing DQ red BSA in the LRE region (Figure 3E). We added a panel with confocal microscopy images of the LREs in 6 dpf larva gavaged with DQ red BSA (Figure S3F). These images show that DQ red BSA fluorescence was localized to the LRE lysosomal vacuole.

      In Figure 5D, Posterior LRE should be pink not green in the key to the right of the heatmap.

      Thank you for catching this error. We have corrected the colors (Figure 5D).

      Reviewer #3 (Recommendations for the authors):

      (1) Introduction and context:

      Expand the introduction to include more background on microbial-mediated protein absorption, with references to relevant findings in Drosophila. This will provide a stronger foundation for the study's contributions to the field.

      Thank you for this suggestion. We added information about microbe-mediated amino acid harvest in Drosophila to the introduction (lines 49-53).

      (12) Methodological suggestions:

      Measure and report differences between germ-free (GF) and conventional (CV) animals, such as transit time, to account for potential confounding factors in protein absorption dynamics.

      We respectfully assert that a transit assay is not required for this study and could actually create confusion as an effect in transit time could be interpreted as a contributing factor when it is in fact not the case due to the experimental design. This is because the concentration of luminal protein was equivalent in GF and CV larvae (Figure S1E), so the LREs had equal saturating access to those proteins in both conditions. Furthermore, we showed the microbiota did not degrade fluorescent protein (Figure S1F). Therefore, we feel confident that there was lower protein uptake in the LREs of CV larvae because the microbiome exerted regulatory effects on LRE activity.

      Provide detailed information on the gating strategy used for single-cell sorting to enhance the dataset's utility and support claims about cell changes.

      The methods we used for sorting cells were previously described (PMID: 31474562). In this manuscript, we describe them under the heading “Fluorescence activated cell sorting for single cell RNA-sequencing.”

      Explain the "GeneRatio" metric in figure legends for clarity.

      The GeneRatio is the ratio of genes associated with each individual GO term to the number of genes associated with the domain. An explanation was added to the caption (Figure S3C).

      (13) Visual and statistical improvements:

      Include images of labeled peptidases within lysosome-rich enterocytes (LREs) to reinforce findings.

      Thank you for the suggestion. We added images of labeled peptidases in the LRE region (Figure S6E-D).

      For Panels 4-F and 5-D, consider using violin plots of selected genes to improve clarity and emphasize major ideas.

      In Figure 4F, the heatmap shows multiple genes were upregulated in mCherry-positive cells. We tried the plotting suggested by the reviewer and felt that violin plots could not convey this message as clearly. Likewise, the heatmap in Figure 5D effectively shows the gradient of expression between ileocytes, anterior and posterior LREs.

      Strengthen statistical analysis by employing more rigorous methods and justifying their selection, such as using two-way ANOVA where appropriate.

      The two-way ANOVA was used to quantify protein uptake or HCR probe fluorescence along the length of the LRE region. This statistical test allowed us to compare differences in fluorescence between experimental conditions in multiple LRE segments (see Authoer response image 1 below for example). As our assays show, the LRE region is heterogenous with segments showing different levels of activity and gene expression. The two-way ANOVA is appropriate because it allows us to account for this heterogeneity by comparing fluorescence across multiple segments.

      Author response image 1.

      Our figures display these fluorescent levels in line plots (above, left) rather than bar plots (above, right). The results are easier to visualize interpret in line plots, and they display the fluorescence profiles in greater detail.

      (14) Technical corrections:

      Correct figure references: Figure 5 about tryptophan metabolism should be 5A, S5G-S5H.

      We corrected the figure references.

      Line 518: Spell out "heterozygotes" instead of using "gets".

      We changed the term from “hets” to “heterozygotes.”

      (15) Revise Figure S2 citation to match the actual figure labeling.

      We corrected the text to indicate “Figure S2” rather than “Figure S2A.”

      Additional manuscript modification

      · Figure panels 3B-C, S3A-B, 4A-C: Two cluster were relabeled with improved descriptors based on our updated annotations. The clusters “Pharynx-esophagus-cloaca 1” (PEC1) and PEC2 were relabeled as “Pharynx-cloaca 1” and “Pharynx-cloaca 2.”

    1. Author response:

      The following is the authors’ response to the original reviews.

      Public Reviews:

      Reviewer #1:

      This study of mixed glutamate/GABA transmission from axons of the supramammillary nucleus to dentate gyrus seeks to sort out whether the two transmitters are released from the same or different synaptic vesicles. This conundrum has been examined in other dual-transmission cases and even in this particular pathway, there are different views. The authors use a variety of electrophysiological and immunohistochemical methods to reach the surprising (to me) conclusion that glutamate and GABA- filled vesicles are distinct yet released from the same nerve terminals. The strength of the conclusion rests on the abundance of data (approaches) rather than the decisiveness of any one approach, and I came away believing that the boutons may indeed produce and release distinct types of vesicles, but have reservations. 

      We thank the reviewer for his/her evaluation of our work. At present, several studies reported that a variety of combinations of two transmitters are co-released from different synaptic vesicles in the central nervous system. In this regard, we think the cotransmission of glutamate/GABA from different synaptic vesicles is not surprising. To better explain to the reader how much we know about co-release of dual transmitters in the brain, we have now added new sentences describing segregated co-release of two neurotransmitters in other synapses in the Introduction (line 63-80).

      Accepting the conclusion, one is now left with another conundrum, not addressed even in the discussion: how can a single bouton sort out VGLUTs and VIAATs to different vesicles, position them in distinct locations with nm precision, and recycle them without mixing? And why do it this way instead of with single vesicles having mixed chemical content? For example, could a quantitative argument be made that separate vesicles allow for higher transmitter concentrations? I feel the paper needs to address these problems with some coherent discussion, at minimum. 

      Although these questions are very important and interesting to address, little is known about molecular mechanisms how VGluT2 and VIAAT are sorted to different vesicles and each synaptic vesicle is segregated. That is why we had not mentioned the sorting mechanisms in the original manuscript. Nevertheless, in response to the reviewer’s suggestion, we have now added new sentences describing possible mechanisms for the sorting and segregation of VGluT2 and VIAAT in the Discussion (line 439-462).

      As for the question regarding why glutamate and GABA are released from different synaptic vesicles, we mentioned the functional roles of separate release of two transmitters over release from single vesicles several times in the Introduction (line 94100), Results (line 300-302), and Discussion (line 406-408, 521-522). Although it seems to be an interesting point to think about transmitter concentrations in the vesicles, we think this issue is beyond the scope of the present study. Given that manipulation of vesicular transmitter contents is technically possible (Hori and Takamori, 2021), this issue awaits further investigation.

      Major concerns: 

      (1) Throughout the paper, the authors use repetitive optogenetic stimulation to activate SuM fibers and co-release glutamate and GABA. There are several issues here: first, can the authors definitively assure the reader that all the short-term plasticity is presynaptic and not due to ChR2 desensitization? This has not been addressed. Second, can the authors also say that all the activated fibers release both transmitters? If for example 20% of the fibers retained a onetransmitter identity and had distinct physiological properties, could that account for some of the physiological findings? 

      Thank you for raising this important point. To examine whether repetitive light illumination induces ChR2 desensitization, the fiber volley was extracellularly recorded. We found that paired-pulse or 10 stimuli at 5, 10, and 20 Hz reliably evoked similar amplitudes of fiber volley during light stimulation. These results clearly indicate that repetitive light stimulation can reliably activate ChR2 and elicit action potentials in the SuM axons. These new findings are now included in Figure 1-figure supplement 2 and Figure 5-figure supplement 2. We also previously demonstrated that by direct patch-clamp recordings from ChR2-expressing hippocampal mossy fiber terminals, 125 times light stimulation at 25 Hz reliably elicited action potentials (Fig. S1: Fukaya et al., 2023). Therefore, we believe that if expression level of ChR2 is high, activation of ChR2 induces action potentials in response to repetitive light stimulation and mediates synaptic transmission with high efficiency.

      We found that most of the SuM terminals (95%) have both VGluT2 and VIAAT (Figure 1E). This anatomical evidence strongly indicates that most of the SuM terminals have the ability to release both glutamate and GABA, and the SuM fibers having one transmitter identity should be minor populations.

      (2) PPR differences in Figures 1F-I are statistically significant but still quite small. You could say they are more similar than different in fact, and residual differences are accounted for by secondary factors like differential receptor saturation. 

      In this experiment, the light intensity was adjusted to yield less than 80% of the maximum response as described in the method section of original and revised manuscript, minimizing the possibility of receptor saturation. We also excluded the possibility that PPR differences could be attributed to differential receptor saturation and desensitization by using a low-affinity AMPA receptor antagonist and a low-affinity GABAA receptor antagonist (Figure 5-figure supplement 3). These results indicate that PPR differences are mediated by the presynaptic origin.

      (3) The logic of the GPCR experiments needs a better setup. I could imagine different fibers released different transmitters and had different numbers of mGluRs, so that one would get different modulations. On the assumption that all the release is from a single population of boutons, then either the mGluRs are differentially segregated within the bouton, or the vesicles have differential responsiveness to the same modulatory signal (presumably a reduced Ca current). This is not developed in the paper. 

      Based on our minimal stimulation results and anatomical analysis, we believe that many SuM terminals contain both glutamate and GABA. Therefore, both transmissions are able to be modulated by mGluRs and GABAB receptors within the same terminals. As the reviewer pointed out, differential responsiveness of glutamate-containing and GABA-containing vesicles to the GPCR signal could be one of the molecular mechanisms for differential effects of GPCRs on EPSCs and IPSCs. In addition, the spatial coupling between GPCRs and active zones for glutamate and GABA in the same SuM terminals may be different, which may give rise to differential modulation of glutamate and GABA release. These possible mechanisms are now described in the Discussion (line 469-476).

      (4) The biphasic events of Figures 3 and S3: I find these (unaveraged) events a bit ambiguous. Another way to look at them is that they are not biphasic per se but rather are not categorizable. Moreover, these events are really tiny, perhaps generated by only a few receptors whose open probability is variable, thus introducing noise into the small currents. 

      We agree with the reviewer that some events are tiny and some small currents could be masked by background noise. We understand that detecting the biphasic events by minimal stimulation has technical limitations. Because we automatically detected biphasic events, which were defined as an EPSC-IPSC sequence, only if an outward peak current following an inward current appeared within 20 ms of light illumination as described in the method section, we cannot exclude the possibility that the biphasic events we detected might include false biphasic responses. To compensate these technical issues, we also performed strontium-induced asynchronous release as another approach and found similar results as minimal stimulation experiments (Figures 3E and 3F). Furthermore, we confirmed that the amplitudes and kinetics of minimal light stimulation-evoked EPSCs or IPSCs were not altered by blockade of their counterpart currents (Figure 3-figure supplement 2). Even if false biphasic responses were accidentally included in the analysis, eventually biphasic events are a minor population and we successfully detected discernible independent EPSCs and IPSCs, which were the major population of uniquantal release-mediated synaptic responses. Thus, multiple pieces of evidence support distinct release of glutamate and GABA from SuM terminals.

      (5) Figure 4 indicates that the immunohistochemical analysis is done on SuM terminals, but I do not see how the authors know that these terminals come from SuM vs other inputs that converge in DG. 

      We thank the reviewer for raising an important point. As shown in Figure 4A, B, almost all VGluT2-positive terminals in the GC layer co-expressed with VIAAT. We are aware that VTA neurons reportedly project to the GC layer of the DG and co-release glutamate and GABA (Ntamati and Luscher, 2016). Contrary to this report, our retrograde tracing analysis did not reveal direct projections from the VTA to the DG. This new data is now included in Figure 4-figure supplement 1. We also added pre-embedding immunogold EM analysis, in which SuM terminals were virally labeled with eYFP, confirming that they form both asymmetric and symmetric synapses (revised Figure 4F). Together with these new data, our results clearly demonstrate that SuM terminals in the GC layer form both asymmetric and symmetric synapses. While our results strongly suggest that VGluT2positive terminals and SuM terminals in the GC layer are nearly identical, we cannot fully exclude the possibility that other inputs originating from unidentified brain regions may co-express VGluT2 and VIAAT in the GC layer. Therefore, in Figure 4 of the revised manuscript, we described “VGluT2-positive terminals” instead of “SuM terminals”.

      (6) Figure 4E also shows many GluN1 terminals not associated with anything, not even Vglut, and the apparent numbers do not mesh with the statistics. Why? 

      In triple immunofluorescence for VGluT2, VIAAT, and GluN1, free GluN1 puncta were predominantly observed in the molecular layer. Given that VGluT2-positive terminals are sparse in the molecular layer, these GluN1 puncta are primarily associated with VGluT1, the dominant subtype. In this study, we focused the analysis of GluN1 puncta specifically on the GC layer, excluding the molecular layer. To avoid miscommunication, we changed the original Figure 4E to the new Figure 4G, which focuses on the GC layer and aligns with the quantitative analysis. Additionally, we used ultrathin sections (100-nm-thick) to enhance spatial resolution, which limits the detection of co-localization events within this confined spatial range, as noted in the Discussion (line 485-488).

      (7) Do the conclusions based on the fluorescence immuno mesh with the apparent dimensions of the EM active zones and the apparent intermixing of labeled vesicles in immuno EM? 

      To further support our immunofluorescence results, we performed EM study and found that a single SuM terminal formed both asymmetric and symmetric synapses on a GC soma (revised Figures 4E and 4F). These new data and our immunofluorescence results clearly indicate that a single SuM terminal forms both glutamatergic and GABAergic synapses on a GC and co-release glutamate and GABA. 

      As the reviewer pointed out, our immuno EM shows that VGluT2 and VIAAT labeled vesicles appear to intermix in asymmetric and symmetric synapses. Accordingly, in the revised manuscript, Figure 7 has been modified to show the intermixing of glutamate and GABA-containing vesicles in the SuM terminal. It should be noted that because of low labeling efficiency, our immuno-EM images don’t represent the whole picture of synaptic vesicles for glutamate and GABA. There could be biased distribution of vesicles close to their release site (more VGluT2-containing vesicles close to asymmetric synapses and more VIAAT-containing vesicles close to symmetric synapses) as reported previously (Root et al., 2018). Additionally, our results could be explained by other mechanisms: co-release of glutamate and GABA from the same vesicles, with one transmitter undetected due to the absence of its postsynaptic receptor. This possibility is now mentioned in the Discussion (line 512-520). More detailed vesicle configuration in a single SuM terminal will have to be investigated in future studies.

      (8) Figure 6 is not so interesting to me and could be removed. It seems to test the obvious: EPSPs promote firing and IPSPs oppose it. 

      We believe these results are necessary for the following two reasons. First, we showed that glutamate/GABA co-transmission balance is dynamically changed in a frequency-dependent manner (Figure 5). In terms of physiological significance, it is important to demonstrate how these frequency-dependent dynamic changes affect GC firing. Therefore, we believe that figure 6, which shows how SuM inputs modulate GC firing by repetitive SuM stimulation, is necessary for this paper. Second, we previously reported the excitatory effects of the SuM inputs on GC firing, suggesting the important roles of glutamatergic transmission of the SuM inputs in synaptic plasticity (Hashimotodani et al., 2018; Hirai et al., 2022; Tabuchi et al., 2022). In contrast, how GABAergic cotransmission contributes to SuM-GC synaptic plasticity and DG information processing was not well understood. Our results in figure 6, which demonstrate the inhibitory effects of GABAergic co-transmission on GC firing by high frequency repetitive SuM input activity, clearly show the contribution of GABAergic co-transmission to short-term plasticity at SuM-GC synapses. For these reasons, we would like to keep Figure 6. We hope that our explanations convince the reviewer. 

      Reviewer #2:

      Summary:

      In this study, the authors investigated the release properties of glutamate/GABA co-transmission at the supramammillary nucleus (SuM)-granule cell (GC) synapses using in vitro electrophysiology and anatomical approaches at the light and electron microscopy level. They found that SuM to dentate granule cell synapses, which co-release glutamate and GABA, exhibit distinct differences in paired-pulse ratio, Ca2+ sensitivity, presynaptic receptor modulation, and Ca2+ channel-vesicle coupling configuration for each neurotransmitter. The study shows that glutamate/GABA co-release produces independent glutamatergic and GABAergic synaptic responses, with postsynaptic targets segregated. They show that most SuM boutons form distinct glutamatergic and GABAergic synapses in close proximity, characterized by GluN1 and GABAAα1 receptor labeling, respectively. Furthermore, they demonstrate that glutamate/GABA co-transmission exhibits distinct short-term plasticity, with glutamate showing frequencydependent depression and GABA showing frequency-independent stable depression. 

      Their findings suggest that these distinct modes of glutamate/GABA co-release by SuM terminals serve as frequency-dependent filters of SuM inputs. 

      Strengths:

      The conclusions of this paper are mostly well supported by the data. 

      We thank the reviewer for their positive and constructive comments on our manuscript.

      Weaknesses: 

      Some aspects of Supplementary Figure 1A and the table need clarification. Specifically, the claim that the authors have stimulated an axon fiber rather than axon terminals is not convincingly supported by the diagram of the experimental setup. Additionally, the antibody listed in the primary antibodies section recognizes the gamma2 subunit of the GABAA receptor, not the alpha1 subunit mentioned in the results and Figure 4. 

      We have now answered these questions in recommendations section below.

      Reviewer #3:

      Summary: 

      In this manuscript, Hirai et al investigated the release properties of glutamate/GABA cotransmission at SuM-GC synapses and reported that glutamate/GABA co-transmission exhibits distinct short-term plasticity with segregated postsynaptic targets. Using optogenetics, whole-cell patch-clamp recordings, and immunohistochemistry, the authors reveal distinct transmission modes of glutamate/GABA co-release as frequency-dependent filters of incoming SuM inputs. 

      Strengths: 

      Overall, this study is well-designed and executed; conclusions are supported by the results. This study addressed a long-standing question of whether GABA and glutamate are packaged in the same vesicles and co-released in response to the same stimuli in the SuM-GC synapses (Pedersen et al., 2017; Hashimotodani et al., 2018; Billwiller et al., 2020; Chen et al., 2020; Li et al., 2020; Ajibola et al., 2021). Knowledge gained from this study advances our understanding of neurotransmitter co-release mechanisms and their functional roles in the hippocampal circuits. 

      Weaknesses:

      No major issues are noted. Some minor issues related to data presentation and experimental details are listed below. 

      We appreciate the reviewer’s positive view of our study. We responded in more detail in recommendations section below.

      Recommendations for the authors:

      Reviewer #1:

      (1) The blue color for VIAAT in panel 1C is extremely hard to see. 

      Thank you for pointing out. We have changed to the cyan color for VIAAT in Figure 1C and D in the revised manuscript.

      (2) Line 329 "perforant" not "perfomant".  

      We appreciate the reviewer’s careful attention. In the revised manuscript, we corrected this misword.

      Reviewer #2:

      To convincingly demonstrate that the authors stimulated SuM axon fiber instead of SuM terminals (Supplementary Figures 1A), they should provide an image showing the distribution of SuMlabeled fibers and axon terminals reaching the dentate gyrus (DG) and the trace of the optic fiber, rather than providing a diagram of the experimental setup. 

      We appreciate the reviewer’s suggestion. We have now provided a new experimental setup image (Figure 1-figure supplement 1A) showing a single GC, the distribution of SuM fibers in the GC layer, and the illumination area at each location. As SuM inputs make synapses onto the GC soma and dendrite close to the GC cell body, SuM-GC synapses in the recording GCs exist in a very limited area. This characteristic synaptic localization allowed us to control the illumination area without applying light to the SuM terminals in the recording GCs. Delayed onsets of EPSCs/IPSCs by over-axon stimulation (Figure 1-figure supplement 1C, D) also support that SuM terminals in the recording GCs were out of illumination area.

      Additionally, the authors should clarify the discrepancy between the antibody mentioned in the list of primary antibodies, which recognizes the gamma2 subunit of the GABAA receptor, and the alpha1 subunit of the GABAA receptor mentioned in the results and Figure 4. 

      We apologize for this mistake. As described in the main text and figure, we used the antibody for a1 subunit of the GABAA receptor. Table S1 has been corrected in the revised version of the paper.

      Reviewer #3:

      (1) In Figure 1, the authors used two [Ca2+]o concentrations to study the EPSC and IPSC amplitudes. How does the Ca2+ concentration affect the PPR in the EPSC and IPSC, respectively? 

      Given that lowering the extracellular Ca2+ concentration reduces the release probability, it is expected that 1 mM extracellular Ca2+ concentration increases PPR compared to 2.5 mM. Actually, we observed that lowering the extracellular Ca2+ concentration increased the synaptic responses from 2nd to 10th (both EPSC and IPSC) by train stimulation (Figure 5).

      (2) In Figure 2D, does baclofen also have a dose-dependent effect on the inhibition of the EPSC and IPSC similar to the DCG-IV in Figure 2C? 

      Thank you for your question. Because we aimed to demonstrate the differential inhibitory effects of baclofen at a certain concentration on glutamatergic and GABAergic co-transmission, we did not go into detail regarding a dose-dependent effect. In response to the reviewer’s comment, we performed the effects of higher concentration of baclofen on EPSCs and IPSCs. As shown in the figure below, 50 µM baclofen inhibited EPSCs and IPSCs to the similar extent. Therefore, by comparing inhibitory effect of two different concentrations of baclofen (5 and 50 µM), we believe that baclofen also has a dose-dependent inhibitory effect on both EPSCs and IPSCs similar to the DCGIV.

      Author response image 1.

      (3) In Figure 2E, statistical labels, such as "*" or "n.s." (not significant), should be provided on the plots to facilitate the reading of figures. 

      In response to the reviewer’s comment, we have provided statistical labels in the Figure 2E.

      (4) In Figure 3A, the latency of the evoked EPSC for the lower light stimulation groups seems to be much slower than the one shown on the left or other figures in the paper, such as Figure 1F.

      Please double-check if the blue light stimulation label is placed in the right location. 

      Corrected, thanks.

      (5) The use of minimal light stimulation in optogenetic experiments is not appropriately justified or described. More detailed information should be provided, such as whether the optogenetic stimulation is performed on the axon or the terminals of the SuM. 

      We appreciate the reviewer’s suggestion. To effectively detect stochastic synaptic responses, the light stimulation was applied on the terminals of the SuM. We have now stated this information (line 212). We also further described the justification of use of minimal light stimulation in the revised manuscript (line 207-209). 

      References

      Fukaya R, Hirai H, Sakamoto H, Hashimotodani Y, Hirose K, Sakaba T (2023) Increased vesicle fusion competence underlies long-term potentiation at hippocampal mossy fiber synapses. Sci Adv 9:eadd3616.

      Hashimotodani Y, Karube F, Yanagawa Y, Fujiyama F, Kano M (2018) Supramammillary Nucleus Afferents to the Dentate Gyrus Co-release Glutamate and GABA and Potentiate Granule Cell Output. Cell Rep 25:2704-2715 e2704.

      Hirai H, Sakaba T, Hashimotodani Y (2022) Subcortical glutamatergic inputs exhibit a Hebbian form of long-term potentiation in the dentate gyrus. Cell Rep 41:111871.

      Hori T, Takamori S (2021) Physiological Perspectives on Molecular Mechanisms and Regulation of Vesicular Glutamate Transport: Lessons From Calyx of Held Synapses. Front Cell Neurosci 15:811892.

      Ntamati NR, Luscher C (2016) VTA Projection Neurons Releasing GABA and Glutamate in the Dentate Gyrus. eNeuro 3.

      Root DH, Zhang S, Barker DJ, Miranda-Barrientos J, Liu B, Wang HL, Morales M (2018) Selective Brain Distribution and Distinctive Synaptic Architecture of Dual Glutamatergic-GABAergic Neurons. Cell Rep 23:3465-3479.

      Tabuchi E, Sakaba T, Hashimotodani Y (2022) Excitatory selective LTP of supra-mammillary glutamatergic/GABAergic co-transmission potentiates dentate granule cell firing. Proc Natl Acad Sci U S A 119:e2119636119.

    1. Author Response

      The following is the authors’ response to the original reviews.

      Public Reviews:

      Reviewer #1 (Public Review):

      The manuscript by Goetz et al. takes a new perspective on sensory information processing in cells. In contrast to previous studies, which have used population data to build a response distribution and which estimate sensory information at about 1 bit, this work defines sensory information at the single cell level. To do so, the authors take two approaches. First, they estimate single cells' response distributions to various input levels from time-series data directly. Second, they infer these single-cell response distributions from the population data by assuming a biochemical model and extracting the cells' parameters with a maximum-entropy approach. In either case, they find, for two experimental examples, that single-cell sensory information is much higher than 1 bit, and that the reduction to 1 bit at the population level is due to the fact that cells' response functions are so different from each other. Finally, the authors identify examples of measurable cell properties that do or do not correlate with single-cell sensory information.

      The work brings an important and distinct new insight to a research direction that generated strong interest about a decade ago: measuring sensory information in cells and understanding why it is so low. The manuscript is clear, the results are compelling, and the conclusions are well supported by the findings. Several contributions should be of interest to the quantitative biology community (e.g., the demonstration that single cells' sensory information is considerably larger than previously implied, and the approach of inferring single-cell data from population data with the help of a model and a maximum-entropy assumption).

      We thank the reviewer for the excellent summary of our research.

      Reviewer #2 (Public Review):

      In this paper the authors present an existing information theoretic framework to assess the ability of single cells to encode external signals sensed through membrane receptors.

      The main point is to distinguish actual noise in the signaling pathway from cell-cell variability, which could be due to differences in their phenotypic state, and to formalize this difference using information theory.

      After correcting for this cellular variability, the authors find that cells may encode more information than one would estimate from ignoring it, which is expected. The authors show this using simple models of different complexities, and also by analyzing an imaging dataset of the IGF/FoxO pathway.

      The implications of the work are limited because the analysed data is not rich enough to draw clear conclusions. Specifically,

      • the authors do not distinguish what could be methodological noise inherent to microscopy techniques (segmentation etc), and actual intrinsic cell state. It's not clear that cell-cell variability in the analyzed dataset is not just a constant offset or normalization factor. Other authors (e.g. Gregor et al Cell 130, 153-164) have re-centered and re-normalized their data before further analysis, which is more or less equivalent to the idea of the conditional information in the sense that it aims to correct for this experimental noise.

      We thank the reviewer for the comment. However, we do not believe our analysis is a consequence of normalization artifacts. Prior to modeling the single cell data, we removed well-dependent background fluorescence. This should take care of technical variation related to overall offsets in the data. We agree with the reviewer that background subtraction may not fully account for technical variability. For example, some of the cell-to-cell variability may potentially be ascribed to issues such as incorrect segmentation. Unfortunately, however, attempting to remove this technical variability through cell-specific normalization as suggested by the reviewer1 will diminish to a very large extent the true biological effects related to extensivity (cell size, total protein abundance). We note that these effects are a direct function of cell state-variables (see for example Cohen-Saidon et al.2 who use cell-state specific normalization to improve signaling fidelity). Therefore, an increase in mutual information after normalization does not only reflect removal of technical noise but also accounts for effect of cell state variables.

      Nonetheless, as the reviewer suggested, we performed a cell-specific normalization wherein the mean nuclear FoxO levels in each cell (in the absence of IGF) were normalized to one. Then, for each ligand concentration, we collated FoxO response across all cells and computed the channel capacity corresponding to cell-state agnostic mutual information ICSA. As expected, ICSA increases from ∼0.9 bits to ∼1.3 bits when cell-specific normalization was performed (Author response image 1). However, this value is significantly lower than the average ∼1.95 of cell-state specific mutual information ⟨ICee⟩. Finally, we note that the cell specific normalization does not change the calculations of channel capacity at the single cell level as these calculations do not depend on linear transformations of the data (centering and normalization). Therefore, we do not think that our analysis of experimental data suffers from artifacts related to microscopy.

      Author response image 1.

      Author response image 1. Left: nuclear FoxO response averaged over all cells in the population across different ligand concentration. Right: nuclear FoxO response was first normalized at the single cell level and then averaged over all cells in the population across different ligand concentrations.

      • in the experiment, each condition is shown only once and sequentially. This means that the reproducibility of the response upon repeated exposures in a single cell was not tested, casting doubt on the estimate of the response fidelity (estimated as the variance over time in a single response).

      The reviewer raises an excellent question about persistence of cell states. To verify that cell states are indeed conserved at the time scale of the experiment, we reanalyzed data generated by Gross et al.3 wherein cells were perturbed with IGF (37.5 pM), followed by a washout which allowed the cells to reach pre-stimulation nuclear FoxO levels, followed by a re-perturbation with the same amount of IGF. Nuclear FoxO response was measured at the single cell level after 90 minutes with IGF exposure both these times. Since the response x to the same input u was measured twice in the same cell (x1 and x2), we could evaluate the intrinsic variability in response at the single cell level. We then compared this intrinsic variability to the extrinsic cell-state dependent variability in the population.

      To do so, we computed for each cell δ=x1-x2 the difference between the two responses. reviewer Figure 2 show the histogram p(δ) as computed from the data (pink) and the same computed from the model that was trained on the single cell data (blue). We also computed p(δ0) which represented the difference between responses of two different cells both from the data and from the model.

      As we see in Author response image 2, the distribution p(δ) is significantly narrower than p(δ0) suggesting that intracellular variability is significantly smaller than across-population variability and that cells’ response to the same stimuli are quite conserved, especially when compared to responses in randomly picked pairs of cells. This shows that cell states and the corresponding response to extracellular perturbations are conserved, at least at the time scale of the experiment. Therefore, our estimates of cell-to-cell variability signaling fidelity are stable and reliable. We have now incorporated this discussion in the manuscript (lines 275-281).

      Author response image 2.

      Author response image 2. Left: Cells were treated with 37.5 pM of IGF for 90 minutes, washed out for 120 minutes and again treated with 37.5 pM of IGF. Nuclear FoxO was measured during the treatment and the washout. The distributions on the left show the difference in FoxO levels in single cells after the two 90 minutes IGF stimulations (pink: data, blue: model). Right: Distribution of difference in FoxO levels in two randomly picked cells after 90 minutes of exposure to 37.5 pM IGF.

      • another dataset on the EGF/EGFR pathway is analyzed, but no conclusion can be drawn from it because single-cell information cannot be directly estimated from it. The authors instead use a maximum-entropy Ansatz, which cannot be validated for lack of data.

      We thank the reviewer for this comment. We agree with the reviewer that we have not verified our predictions for the EGF/EGFR pathway. That study was meant to show the potential generality of our analysis. We look forward to validating our predictions for the EGF/EGFR pathway in future studies.

      Reviewer #3 (Public Review):

      Goetz, Akl and Dixit investigated the heterogeneity in the fidelity of sensing the environment by individual cells in a population using computational modeling and analysis of experimental data for two important and well-studied mammalian signaling pathways: (insulin-like growth factor) IGF/FoxO and (epidermal growth factor) EFG/EFGR mammalian pathways. They quantified this heterogeneity using the conditional mutual information between the input (eg. level of IGF) and output (eg. level of FoxO in the nucleus), conditioned on the "state" variables which characterize the signaling pathway (such as abundances of key proteins, reaction rates, etc.) First, using a toy stochastic model of a receptor-ligand system - which constitutes the first step of both signaling pathways - they constructed the population average of the mutual information conditioned on the number of receptors and maximized over the input distribution and showed that it is always greater than or equal to the usual or "cell state agnostic" channel capacity. They constructed the probability distribution of cell state dependent mutual information for the two pathways, demonstrating agreement with experimental data in the case of the IGF/FoxO pathway using previously published data. Finally, for the IGF/FoxO pathway, they found the joint distribution of the cell state dependent mutual information and two experimentally accessible state variables: the response range of FoxO and total nuclear FoxO level prior to IGF stimulation. In both cases, the data approximately follow the contour lines of the joint distribution. Interestingly, high nuclear FoxO levels, and therefore lower associated noise in the number of output readout molecules, is not correlated with higher cell state dependent mutual information, as one might expect. This paper contributes to the vibrant body of work on information theoretic characterization of biochemical signaling pathways, using the distribution of cell state dependent mutual information as a metric to highlight the importance of heterogeneity in cell populations. The authors suggest that this metric can be used to infer "bottlenecks" in information transfer in signaling networks, where certain cell state variables have a lower joint distribution with the cell state dependent mutual information.

      The utility of a metric based on the conditional mutual information to quantify fidelity of sensing and its heterogeneity (distribution) in a cell population is supported in the comparison with data. Some aspects of the analysis and claims in the main body of the paper and SI need to be clarified and extended.

      1. The authors use their previously published (Ref. 32) maximum-entropy based method to extract the probability distribution of cell state variables, which is needed to construct their main result, namely p_CeeMI (I). The salient features of their method, and how it compares with other similar methods of parameter inference should be summarized in the section with this title. In SI 3.3, the Lagrangian, L, and Rm should be defined.

      We thank the reviewer for the comment and apologize for the omission. We have now rewritten the manuscript to include references to previous reviews of works that infer probability distributions4 of cell state variables (lines 156-168). Notably, as we argued in our previous work5, no current method can efficiently estimate the joint distribution over parameters that is consistent with measured single cell data and models of signaling networks. Therefore, we could not use multiple approaches to infer parameter distributions. We have now expanded our discussion of the method in the supplementary information sections.

      1. Throughout the text, the authors refer to "low" and "high" values of the channel capacity. For example, a value of 1-1.5 bits is claimed to be "low". The authors need to clarify the context in which this value is low: In some physically realistic cases, the signaling network may need to simply distinguish between the present or absence of a ligand, in which case this value would not be low.

      We agree with the reviewer that small values of channel capacities might be sufficient for cells to carry out some tasks, in which case a low channel capacity does not necessarily indicate a network not performing its task. Indeed, how much information is needed for a specific task is a related but distinct question from how much information is provided though a signaling network. Both questions are essential to understand a cell's signaling behavior, with the former being far less easy to answer in a way which is generalizable. In contrast, the latter can be quantitatively answered using the analysis presented in our manuscript.

      1. Related to (2), the authors should comment on why in Fig. 3A, I_Cee=3. Importantly, where does the fact that the network is able to distinguish between 23 ligand levels come from? Is this related to the choice (and binning) of the input ligand distribution (described in the SI)?

      We thank the reviewer for the comment. The network can distinguish between all inputs used in the in silico experiment precisely because the noise at the cellular level is small enough that there is negligible overlap between single cell response distributions. Indeed, the mutual information will not increase with the number of equally spaced inputs in a sub-linear manner, especially when the input number is very high.

      1. The authors should justify the choice of the gamma distribution in a number of cases (eg. distribution of ligand, distribution cell state parameters, such as number of receptors, receptor degradation rate, etc.).

      We thank the reviewer for the comment. We note that previous works in protein abundances and gene expression levels (e.g. see6) have reported distributions with positive skews that can be fit well with gamma distributions or log-normal distributions. Moreover, many stochastic models of protein abundance levels and signaling networks are also known to result in abundances that are distributed according to a negative binomial distribution, the discrete counterpart of gamma distribution. Therefore, we chose Gamma distributions in our study. We have now clarified this point in the Supplementary Information. At the same time, gamma distribution only serves as a regularization for the finite data and in principle, our analysis and conclusion do not depend on choice of gamma distribution for abundances of proteins, ligands, and cell parameters.

      1. Referring to SI Section 2, it is stated that the probability of the response (receptor binding occupancy) conditioned on the input ligand concentration and number of receptors is a Poisson distribution. Indeed this is nicely demonstrated in Fig. S2. Therefore it is the coefficient of variation (std/mean) that decreases with increasing R0, not the noise (which is strictly the standard deviation) as stated in the paper.

      We thank the reviewer of the comment. We have now corrected our text.

      1. In addition to explicitly stating what the input (IGF level) and the output (nuclear GFP-tagged FoxO level) are, it would be helpful if it is also stated what is the vector of state variables, theta, corresponding to the schematic diagram in Fig. 2C.

      We thank the reviewer of the comment. We have now corrected our text in the supplementary material as well as the main text (Figure 2 caption).

      1. Related to Fig. 2C, the statement in the caption: "Phosphorylated Akt leads to phosphorylation of FoxO which effectively shuttles it out of the nucleus." needs clarification: From the figure, it appears that pFoxO does not cross the nuclear membrane, in which case it would be less confusing to say that phosphorylation prevents reentry of FoxO into the nucleus.

      We thank the reviewer of the comment. We have now corrected our text (Figure 2 caption).

      1. The explanations for Fig. 2D, E and insets are sparse and therefore not clear. The authors should expand on what is meant by model and experimental I(theta). What is CC input dose? Also in Fig. 2E, the overlap between the blue and pink histograms means that the value of the blue histogram for the final bin - and therefore agreement or lack thereof with the experimental result - is not visible. Also, the significance of the values 3.25 bits and 3 bits in these plots should be discussed in connection with the input distributions.

      We thank the reviewer of the comment. We have now corrected our text (Figure 2 caption and lines 249-251).

      1. While the joint distribution of the cell state dependent mutual information and various biochemical parameters is given in Fig. S7, there is no explanation of what these results mean, either in the SI or main text. Related to this, while a central claim of the work is that establishing this joint distribution will allow determination of cell state variables that differentiate between high and low fidelity sensing, this claim would be stronger with more discussion of Figs. 3 and S7. The related central claim that cell state dependent mutual information leads to higher fidelity sensing at the population level would be made stronger if it can be demonstrated that in the limit of rapidly varying cell state variables, the I_CSA is retrieved.

      We thank the reviewer for this excellent comment. We have now added more discussion about interpreting the correlation between cell state variables and cell-state specific mutual information (lines 294-306). We also appreciate the suggestion about a toy model calculation to show that dynamics of cell state variables affects cell state specific mutual information. We have now performed a simple calculation to show how dynamics of cell state variables affects cells’ sensing ability (lines 325-363). Specifically, we constructed a model of a receptor binding to the ligand wherein the receptor levels themselves changed over time through a slow process of gene expression (Author response image 3, main text Figure 4). In this model, the timescales of fluctuations of ligand-free receptors on the cell surface can be tuned by speeding up/slowing down the degradation rate of the corresponding mRNA while keeping the total amount of steady state mRNA constant. As shown in Author response image 3, the dependence of cell-specific mutual information on cell state variable diminishes when the time scale of change of cell state variables is fast.

      Author response image 3.

      Author response image 3. Cell state dynamics governs cell state conditioned mutual information. A. In a simple stochastic model, receptor mRNA is produced at a constant rate from the DNA and the translated into ligand-free receptors. The number of ligand-bound receptors after a short exposure to ligands is considered the output. B. A schematic showing dynamics of receptor numbers when mRNA dynamics are slower compared to signaling time scales. C. Conditioning on receptor numbers leads to differing abilities in sensing the environment when the time scale of mRNA dynamics τ is slow. In contrast, when the mRNA dynamics are fast (large τ-1), conditioning on cell state variables does not lead to difference in sensing abilities.

      Reviewer #1 (Recommendations For The Authors):

      My major concerns are mainly conceptual, as described below. With proper attention to these concerns, I feel that this manuscript could be a good candidate for the eLife community.

      Major concerns:

      1. The manuscript convincingly demonstrates that cells good sensors after all, and that heterogeneity makes their input-output functions different from each other. This raises the question of what happens downstream of sensing. For single-celled organisms, where it may be natural to define behavioral consequences at the single-cell level, it may very well be relevant that single-cell information is high, even if cells respond differently to the environment. But for cells in multicellular organisms, like those studied here, I imagine that most behavioral consequences of sensing occur at the multicellular level. Thus, many cells' responses are combined into a larger response. Because their responses are different, their high-information individual responses may combine into a low-information collective response. In fact, one could argue that a decent indicator of the fidelity of this collective response is indeed the population-level information measure estimated in previous works. Thus, a fundamental question that the authors must address is: what is the ultimate utility of reliable, but heterogeneous, responses for a multicellular system? This question has an important bearing for the relevance of their findings.

      We thank the reviewer for this thought-provoking comment. We agree that the fidelity with which cells sense their environment, especially those in multicellular organisms, may not always need to be very high. We speculate that when the biological function of a collection of cells can be expressed as an average over the response of individual cells; high-information but heterogeneous cells can be considered equivalent to low-information homogeneous cells. An example of such a function is population differentiation to maintain relative proportions of different cell types in a tissue or producing a certain amount of extracellular enzyme.

      In contrast, we believe that when the biological function involves collective action, spatial patterning, or temporal memory, the difference between reliable but heterogeneous population and unreliable homogeneous population will become significant. We plan to explore this topic in future studies.

      1. The authors demonstrate that the agreement is good between their inference approach and the direct estimation of response distributions from single-cell time series data. In fact, the agreement is so good that it raises the question of why one would need the inference approach at all. Is it because single-cell time series data is not always available? Is that why the authors used it for one example and not the other? The validation is an asset, but I imagine that the inference approach is complicated and may make assumptions that are not always true. Thus, its utility and appropriate use must be clarified.

      We thank the reviewer for the comment. As the reviewer correctly pointed out, live cell imaging data is not always available and has limited scope. Specifically, optical resolution limits measurements of multiple targets. Moreover, typical live cell measurements measure total abundance or localization and not post-translational modification (phosphorylation, methylation, etc.) which are crucial to signaling dynamics. The most readily available single cell data such those measured using single cell RNA sequencing, immunofluorescence, or flow cytometry are necessarily snapshots. Therefore, computational models that can connect underlying signaling networks to snapshot data become essential when imputing single cell trajectories. In addition, the modeling also allows us to identify network parameters that correlate most strongly with cellular heterogeneity. We have now clarified this point in the manuscript (lines 366-380).

      Minor comments:

      1. I would point out that the maximum values in the single-cell mutual information distributions (Fig 2D and E) correspond to log2 of the number of inputs levels, corresponding to perfect distinguishability of each of the equally-weighted input states. It is clear that many of the mutual information values cluster toward this maximum, and it would help readers to point out why.

      We thank the reviewer for the comment. We have now included a discussion about the skew in the distribution in the text (lines 251-260).

      1. Line 216 references Fig 2C for the EGF/EGFR pathway, but Fig 2C shows the FoxO pathway. In fact, I did not see a schematic of the EGF/EGFR pathway. It may be helpful to include one, and for completeness perhaps also one for the toy model, and organize the figures accordingly.

      We thank the reviewer for the comment. We did not include three separate schematics because the schematics of the EGF/EGFR model and the toy model are subsets of the schematic of the IGF/FoxO model. We have now clarified this point in the manuscript (Figure 2 caption).

      Reviewer #2 (Recommendations For The Authors):

      • the simple model of Fig. 2A would gain from a small cartoon explaining the model and its parameters.

      We thank the reviewer for the comment. We did not include a schematic for the toy model as it is a subset of the schematic of the IGF/FoxO model. The schematic of the toy model is included in the supplementary information.

      • L should be called u, and B should be called x, to be consistent with the rest of the notations in the paper.

      We have decided to keep the notation originally presented in the manuscript.

      • legend of 2E and D should be clarified. "CC input dose" is cryptic. The x axis is the input dose, the y axis is its distribution at the argmax of I. CC is the max of I, not its argmax. Likewise "I" in the legend for the colors should not be used to describe the insets, which are input distributions.

      We have now changed this in the manuscript.

      • the data analysis of the IGF/FoxO pathway should be explained in the main text, not the SI. Otherwise it's impossible to understand how one arrives at, or how to intepret, figure 2E, which is central to the paper. For instance the fact that p(x|u,theta) is assumed to be Gaussian, and how the variance and mean are estimated from the actual data is very important to understand the significance of the results.

      While we have added more details in the manuscript in various places, for the sake of brevity and clarity, we have decided to keep the details of the calculations in the supplementary materials.

      • there's no Method's section. Most of the paper's theoretical work is hidden in the SI, while it should be described in the methods.

      We thank the review of the comment. However, we believe that adding a methods section will break the narrative of the paper. The methods are described in detail in the supplementary materials with sufficient detail to reproduce our results. Additionally, we also provide a link to the github page that has all scripts related to the manuscript.

      PS: please submit a PDF of the SI for review, so that people can read it on any platform (as opposed to a word document, especially with equations)

      We have now done this.

      Reviewer #3 (Recommendations For The Authors):

      1. Subplots in Fig. 1, inset in Fig. 3 are not legible due to small font.

      We have now increased the font.

      1. Mean absolute error in Fig. S5 and relative error in related text should be clarified.

      We have now clarified this in the manuscript.

      1. Acronyms (MACO, MERIDIAN) should be defined.

      We have now made these changes.

      References

      1. Gregor T, Tank DW, Wieschaus EF, Bialek W. Probing the limits to positional information. Cell. 2007;130(1):153-64. doi: 10.1016/j.cell.2007.05.025. PubMed PMID: WOS:000248587000018.

      2. Cohen-Saidon C, Cohen AA, Sigal A, Liron Y, Alon U. Dynamics and Variability of ERK2 Response to EGF in Individual Living Cells. Mol Cell. 2009;36(5):885-93. doi: 10.1016/j.molcel.2009.11.025. PubMed PMID: WOS:000272965400020.

      3. Gross SM, Dane MA, Bucher E, Heiser LM. Individual Cells Can Resolve Variations in Stimulus Intensity along the IGF-PI3K-AKT Signaling Axis. Cell Syst. 2019;9(6):580-8 e4.

      4. Loos C H, J. Mathematical modeling of variability in intracellular signaling. Current Opinion in Systems Biology. 2019;16:17-24.

      5. Dixit PD, Lyashenko E, Niepel M, Vitkup D. Maximum Entropy Framework for Predictive Inference of Cell Population Heterogeneity and Responses in Signaling Networks. Cell Syst. 2020;10(2):204-12 e8.

      6. Taniguchi Y, Choi PJ, Li GW, Chen H, Babu M, Hearn J, Emili A, Xie XS. Quantifying E. coli proteome and transcriptome with single-molecule sensitivity in single cells. Science. 2010;329(5991):533-8. doi: 10.1126/science.1188308. PubMed PMID: 20671182; PMCID: PMC2922915.

    1. Author response:

      The following is the authors’ response to the previous reviews

      Public Reviews:

      Reviewer #1 (Public review):

      Summary:

      The authors attempted to dissect the function of a long non-coding RNA, lnc-FANCI-2, in cervical cancer. They profiled lnc-FANCI-2 in different cell lines and tissues, generated knockout cell lines, and characterized the gene using multiple assays.

      Strengths:

      A large body of experimental data has been presented and can serve as a useful resource for the scientific community, including transcriptomics and proteomics datasets. The reported results also span different parts of the regulatory network and open up multiple avenues for future research.

      Thanks for your positive comments on the strengths.

      Weaknesses:

      The write-up is somewhat unfocused and lacks deep mechanistic insights in some places.

      As the lnc-FANCI-2 as a novel lncRNA had never been explored for any functional study, our report found that it regulates RAS signaling. Thus, this report focuses on lnc-FANCI-2 and RAS signaling pathway but also includes some important screening data, which are important for our readers to understand how we could reach the RAS signaling.

      Reviewer #2 (Public review):

      The study by Liu et al provides a functional analysis of lnc-FANCI-2 in cervical carcinogenesis, building on their previous discovery of FANCI-2 being upregulated in cervical cancer by HPV E7.

      The authors conducted a comprehensive investigation by knocking out (KO) FANCI-2 in CaSki cells and assessing viral gene expression, cellular morphology, altered protein expression and secretion, altered RNA expression through RNA sequencing (verification of which by RT-PCR is well appreciated), protein binding, etc. Verification experiments by RT-PCR, Western blot, etc are notable strengths of the study.

      The KO and KD were related to increased Ras signaling and EMT and reduced IFN-y/a responses.

      Thanks for your positive comments. It did take us a few years to reach this scientific point for understanding of lnc-FANCI-2 function.

      Although the large amount of data is well acknowledged, it is a limitation that most data come from CaSki cells, in which FANCI-2 localization is different from SiHa cells and cancer tissues (Figure 1). The cytoplasmic versus nuclear localization is somewhat puzzling.

      Regarding lnc-FANCI-2 localization, it could be both cytoplasmic and nuclear in cervical cancer tissues, HPV16 or HPV18 infected keratinocytes, and HPV16+ cervical cancer cell line CaSki cells which contain multiple integrated HPV16 DNA copies. But surprisingly, it is most detectable in the nucleus in HPV16+ SiHa cells which contain only one copy of integrated HPV16 DNA (Yu, L., et al. mBio 15: e00729-24, 2024). No matter what, knockdown of lnc-FANCI-2 expression from SiHa cells induces RAS signaling leading to an increase in the expression of p-AKT and p-Erk1/2 (suppl. Fig. S6B).

      Reviewer #3 (Public review):

      Summary:

      A long noncoding RNA, lnc-FANCI-2, was reported to be regulated by HPV E7 oncoprotein and a cell transcription factor, YY1 by this group. The current study focuses on the function of lnc-FANCI-2 in HPV-16 positive cervical cancer is to intrinsically regulate RAS signaling, thereby facilitating our further understanding of additional cellular alterations during HPV oncogenesis. The authors used advanced technical approaches such as KO, transcriptome and (IRPCRP) and LC- MS/MS analyses in the current study and concluded that KO Inc-FANCI-2 significantly increases RAS signaling, especially phosphorylation of Akt and Erk1/2.

      Strengths:

      (1) HPV E6E7 are required for full immortalization and maintenance of the malignant phenotype of cervical cancer, but they are NOT sufficient for full transformation and tumorigenesis. This study helps further understanding of other cellular alterations in HPV oncogenesis.

      (2) lnc-FANCI-2 is upregulated in cervical lesion progression from CIN1, CIN2-3 to cervical cancer, cancer cell lines, and HPV transduced cell lines.

      (3) Viral E7 of high-risk HPVs and host transcription factor YY1 are two major factors promoting lnc-FANCI-2 expression.

      (4) Proteomic profiling of cytosolic and secreted proteins showed inhibition of MCAM, PODXL2, and ECM1 and increased levels of ADAM8 and TIMP2 in KO cells.

      (5) RNA-seq analyses revealed that KO cells exhibited significantly increased RAS signaling but decreased IFN pathways.

      (6) Increased phosphorylated Akt and Erk1/2, IGFBP3, MCAM, VIM, and CCND2 (cyclin D2) and decreased RAC3 were observed in KO cells.

      Thanks for your positive comments. It has taken us almost nine years to reach this point to gradually understand lnc-FANCI-2 functions, which are more complex than our initial thoughts.  

      Weaknesses:

      (1) The authors observed the increased Inc-FANCI-2 in HPV 16 and 18 transduced cells, and other cervical cancer tissues as well, HPV-18 positive HeLa cells exhibited different expressions of Inc-FANCI-2.

      Both HPV16 and HPV18 infections induce lnc-FANCI-2 expression in keratinocytes (Liu H., et al. PNAS, 2021). However, HPV18+ cervical cancer cell lines HeLa and C4II cells (Figure S1A and S1B) do not express lnc-FANCI-2 as we see in HPV-negative cell lines such as HCT116, HEK293, HaCaT, and BCBL1 cells. Although we don’t know why, our preliminary data show that the lnc-FANCI-2 promoter functions well and is sensitive to YY1 binding in lnc-FANCI-2 expressing CaSki and C33A cells in our dual luciferase assays but is much less sensitive to YY1 binding in HeLa and HCT116 cells, indicating some unknown cellular factors negatively regulating lnc-FANCI-2 promoter activity.

      Author response image 1.

      A firefly luciferase (FLuc) reporter containing either the wild-type (−600 wt) or YY1-binding-site-mutated lnc-FANCI-2 promoter was evaluated in CaSki, HeLa, C33A, and HCT116 cells for its promoter activity, with Renilla luciferase (RLuc) activity driven by a TK promoter serving as an internal control. The two YY1-binding motifs (A and B) with a X for mutation are illustrated in the right diagram.

      (2) Previous studies and data in the current showed a steadily increased Inc-FANCI-2 during cancer progression, however, the authors did not observe significant changes in cell behaviors (both morphology and proliferation) in KO Inc-FANCI-2.

      Thanks. We do see decreases in cell proliferation, colony formation, and cell migration, accompanied by increased cell senescence, from the lnc-FANCI-2 KO cells to the parent WT cells.  These data are now added to the revised Fig. 1 and the revised supplemental Fig. S3.

      (3) The authors observed the significant changes of RAS signaling (downstream) in KO cells, but they provided limited interpretations of how these results contributed to full transformation or tumorigenesis in HPV-positive cancer.

      As we stated in the title of this function of lnc-FANCI-2, the lnc-FANCI-2 intrinsically restricts RAS signaling and phosphorylation of Akt and Erk in HPV16-infected cervical cancer. Presumably, high RAS-AKT-ERK signaling inhibits tumor cell survival due to senescence induction as we show in our new Figure 1 and supplemental Fig. S3. A similar report was found in a lung cancer study (Patricia Nieto, et al. Nature 548: 239-243, 2017).

      Recommendations for the authors:

      Reviewer #1 (Recommendations for the authors):

      Major comments:

      (1) A major issue is that parts of the manuscript read like a collection of experimental results. However, some of the results do not contribute directly to the central story. Besides confusing the reader, the large amount of apparently disparate results can raise more questions. For example:

      a) Why is lnc-FANCI-2 highly expressed in HPV16-infected cervical cancer cell lines (but not in HPV18-infected cells)?

      b) How do p53 and RB repress the expression of lnc-FANCI-2?

      c) What regulates the sub-cellular localization of lnc-FANCI-2?

      d) How does lnc-FANCI-2 negatively regulate RAS signalling?

      e) How does MAP4K4 bind to lnc-FANCI-2?

      f) Do lnc-FANCI-2 and MAP4K4 require each other to regulate RAS signalling?

      g) How does RAS signalling regulate the transcription of MCAM and IGFBP3?

      h) How does MCAM feedback on RAS? Do the different MCAM isoforms impact on RAS signalling differently?

      i) How does IGFBP3 feedback on ERK but not AKT?

      j) How do the other mentioned proteins like ADAM8 fit into the regulatory network?

      k) Each question will require a lot more work to address. I think it would be good if the authors could think through carefully what the key message(s) in the current manuscript should be and then present a more focused write-up.

      Thanks for the critical comments. Because this study is the first time to explore lnc-FANCI-2 functions, we would like to be collective. We believe these data are important to guide any future studies. We really appreciate our reviewer listing many questions related to HPV infection, cell biology, RAS signaling, cancer biology from questions a to k. To address each question in a satisfactory way will be a separate study, but fortunately, our report has pointed out such a direction with some preliminary data for future studies. Here below are our responses to each question from a to k:

      a) Both HPV16 and HPV18 infection induce lnc-FANCI-2 expression in keratinocytes (Liu H., et al. PNAS, 2021). However, HPV18+ cervical cancer cell lines HeLa and C4II cells (Figure S1A and S1B) do not express lnc-FANCI-2 as we see in HPV-negative cell lines such as HCT116, HEK293, HaCaT, and BCBL1 cells. Although we don’t know why, our preliminary data show that lnc-FANCI-2 promoter functions well and is sensitive to YY1 binding in lnc-FANCI-2 expressing CaSki and C33A cells but is much less sensitive to YY1 in HeLa and HCT116 cells, indicating some unknown cellular factors negatively regulating lnc-FANCI-2 promoter activity.

      b) We don’t know whether p53 and pRB could repress the expression of lnc-FANCI-2 although C33A cells bearing a mutant p53 and mutant pRB express high amount of lnc-FANCI-2. However, KD of E2F1 had no effect on lnc-FANCI-2 promoter activity in CaSki cells (Liu, H., et al. PNAS, 2021).

      c) RNA cellular localization can be affected by many factors, including splicing, export, and polyadenylation. As lnc-FANCI-2 is a long non-coding RNA, its regulation of cellular location could be more complicated than mRNAs and thus could be a future research direction.  

      d) The conclusion that lnc-FANCI-2 negatively regulates RAS signaling is based on both lnc-FANCI-2 KO and KD studies.  Please see the proposed hypothetic model in Figure 8E.

      e) The MAP4K4 binding to lnc-FANCI-2 was demonstrated by our IRPCRP-Mass spectrometry (Fig. 8A and 8C), although the exact binding site on lnc-FANCI-2 was not explored. As you probably know, many enzymes today turn out an RNA-binding enzyme (Castello A., et al. Trends Endocrinol. Metab. 26: 746-757, 2015; Hentze MW., et al. Nat. Rev. Mol. Cell Biol. 19: 327-341, 2018)    

      f) Yes, they are slightly relied on each other in regulating RAS signaling. We found that KD of MAP4K4 in parent CaSki cells (Figure 8D) led to more effect on RAS signaling (MCAM, IGFBP3, p-Akt) than that in lnc-FANCI-2 KO ΔPr-A9 cells. In contrast, the latter displayed more p-Erk1/2 than that induced by KD of lnc-FANCI-2 in the parental CaSki cells (Figure S7C).

      g) We believe RAS signaling regulates most likely the transcription of MCAM and IGFBP3 through phosphorylated transcription factors (Figure 8E diagram).

      h) As a signal molecule with at least 13 ligands/coreceptors (Joshkon A., et al. Biomedicines 8: 633, 2020), the increased MCAM appears to sustain RAS signaling (Fig. 7J and Fig. 8E). We are assuming the full-length cytoplasmic MCAM plays a predominant role in RAS signaling due to its abundance than the cleaved nuclear MCAM missing both transmembrane and cytoplasmic regions. Plus, RAS signaling mainly occurs in the cytosol.  

      i) Exact mechanism remains unknown. Lnc-FANCI-2 KO cells exhibit high expression levels of IGFBP3 RNA and protein and p-Erk1/2, but not so much for p-Akt, possibly due to IGFBP3 regulation of MAPK for Erk phosphorylation, but not much so on PI3K for Akt phosphorylation.

      j) The dysregulation of RAS signaling and ADAM protein activity is implicated in various cancers. ADAM proteins can modulate RAS signaling by cleaving and releasing ligands that activate or inactivate RAS-related pathways (Schafer B., et al. JBC 279: 47929-38, 2004; Ohtsu H., et al. Am J Physiol Cell Physiol 291: C1-C10, 2006; Dang M, et al. JBC 286: 17704-17713, 2011; Kleino I, et al. PLoS One 10: e0121301, 2015). Some ADAM proteins are Involved in the migration and invasion of cancer cells, and its loss can promote the degradation of KRAS (Huang Y-K., et al. Nat Cancer 5: 400-419, 2024). In this revision, we have a brief discussion on ADAMs and RAS signaling.

      k) We agree with our reviewer that each question will require a lot more work to address. As this study is to explore the lnc-FANCI-2 function for the first time, however, we prefer to include all of these data that have been selectively included in this write-up. We hope reviewer 1 will be satisfied with our response to each question from a to j. 

      (2) Figures S1A & S1C - Replicates are needed.

      Yes, we have repeated all of the experiments. The quantification shown in Figure S1A and S1C was performed in triplicate, and error bars have been added to the updated figure.

      3) Figure S1D - There seems to be some lnc-FANCI-2 RNA in the nucleus of CaSki cells as well. Please quantify the relative amount of lnc-FANCI-2 in the nucleus vs cytoplasm.

      Yes, a small fraction of lnc-FANCI-2 is in the nucleus of CaSki cells as we reported (Liu H., PNAS, 2021, Movies S1 and S2). We did quantify by fractionation and RT-qPCR the relative amount of lnc-FANCI-2 in the nucleus vs cytoplasm in Figure S1C. 

      (4) Figure S2B - (a) For ΔPr-A9 cells, it looks like there is an increase in E6 and a decrease in E7, instead of "little change" as the authors claimed. (b) I suggest checking the protein levels for all the control and KO clones.

      Thanks for the questions. We had some variation in E6 and E7 detection and the submitted one was one representative.  We grew again the lnc-FANCI-2 KO clones A9 and B3 and reexamined the expression of HPV16 E6/E7 proteins and their downstream targets, p53 and E2F1. As shown in new Figure S3A expt II, we saw again some variations in the detections (~20-30%) and these variations do not reflect a noticeable change for their downstream targets. Thus, we do not consider these changes significantly enough to draw a conclusion in our study, but rather most likely from sampling in the assays.

      (5) In the Proteome Profiler Human sReceptor Array analysis, multiple proteins were highlighted as having at least 30% change. But it is unclear how they relate to RAS signaling.

      Thanks for this comment.  Cellular soluble receptors are essential for RAS signaling, EMT pathway and IFN responses. For example, the dysregulation of RAS signaling and ADAM protein activity is implicated in various cancers. ADAM proteins can modulate RAS signaling by cleaving and releasing ligands that activate or inactivate RAS-related pathways (Schafer B., et al. JBC 279: 47929-38, 2004; Ohtsu H., et al. Am J Physiol Cell Physiol 291: C1-C10, 2006; Dang M, et al. JBC 286: 17704-17713, 2011; Kleino I, et al. PLoS One 10: e0121301, 2015). Some ADAM proteins are Involved in the migration and invasion of cancer cells, and its loss can promote the degradation of KRAS (Huang Y-K., et al. Nat Cancer 5: 400-419, 2024). In this revision, we have a brief discussion on ADAMs and RAS signaling.

      (6) Does knockdown of MAP4K4 lead to an increase in MCAM and IGFBP3?

      Yes, the MAP4K4 KD from parental WT CaSki cells does lead an increase in MCAM (~70%) and IGFBP3 (~30%) which is like the knockdown of lnc-FANCI-2 shown in the revised Figure 8D.

      Minor comments:

      (7) In the opinion of this reviewer the title is somewhat unwieldy.

      Thanks. We have shortened the title as “The lnc-FANCI-2 intrinsically restricts RAS signaling in HPV16-infected cervical cancer”

      (8) The abstract can be more focused and doesn't have to mention so many gene names. In fact, the significance paragraph works better as an abstract. For the significance, the authors can provide another write-up on the implications of their research instead.

      Thanks. We have revised the abstract and added the implications of this research.

      (9) The last sentence of the introduction feels a little abrupt. It would be good to elaborate a little more on the key findings.

      Thanks for this critical comment. We have revised as in the following: In this report, we demonstrate that lnc-FANCI-2 in HPV16-infected cells controls RAS signaling by interaction with MAP4K4 and other RNA-binding proteins. Ablation of lnc-FANCI-2 in the cells promotes RAS signaling and phosphorylation of Akt and Erk. High levels of lnc-FANCI-2 and low level of MCAM expression in cervical cancer patients correlate with improved survival, indicating that lnc-FANCI-2 plays a critical role in regulating RAS signaling to affect cervical cancer progression and patient outcomes.

      (10) Typo on line 191: Should be ADAM8 and not ADMA8.

      Corrected.

      Reviewer #2 (Recommendations for the authors):

      The paper contains a vast amount of data and would greatly benefit from an expanded version of the schematic of Figure 8E summarizing the main results. Including additional details on FANCI-2 regulation by HPV (primarily from previous studies) and its implications for HPV16-driven carcinogenesis would provide a more comprehensive overview.

      Thanks for the suggestion. We have modified our Figure 8E to include HR-HPV E7 and YY1 in regulation of lnc-FANCI-2 transcription.

      Further specific comments:

      (1) The introduction may be shortened to increase readability (e.g. lines 77-90; 94-105).

      We have shortened the introduction by deletion of the lines 94-105 from our initial submission.

      (2) Lines 55-57 the number of cervical cancer diagnoses and mortality need to be updated to the latest literature. The reference is from 2012.

      Thanks. We have revised and updated accordingly with a new citation (Bray F., et al: Global cancer statistics 2022: GLOBOCAN estimates of incidence and mortality worldwide for 36 cancers in 185 countries. CA Cancer J Clin 74, 229-263 (2024))

      (3) Line 61: Progression rate of CIN3 is incorrect (31% in 30 years according to reference 5).

      Thanks. Corrected.

      (4) Lines 108-112 are difficult to understand and should be rewritten.

      Thanks. Revised accordingly.

      (5) Line 116 Is this correct or should 'but' be 'and'?

      Thanks. Corrected accordingly.

      (6) Figure 1A top: The difference between cervical cancer and normal areas is hard to see in the top figure. The region labeled as "normal" does not resemble typical differentiating epithelium or normal glandular epithelium, though this is difficult to assess accurately from the image provided. I suggest adding HE staining and also the histotypes.

      We have added an H&E staining panel in the corresponding region to Figure 1A, which clearly shows the normal and cancer regions. Both cervical cancer tissues were cervical squamous cell carcinoma.

      (7) HFK-HPV16 & 18 cells (Figure 1B) are not described in the Materials & Methods.

      Thanks. We revised our Materials and Methods by citing our two previous publications.

      (8) Figure 2E (RNA scope on FANCI-2 KO) only shows 2 to 3 cells, which makes it somewhat difficult to assess downregulated expression in the KO. I suggest replacing these with pictures showing more cells (i.e. >10) to strengthen the results.

      We have replaced the image in Figure 2E to include more cells.

      (9) The spindle-like morphology in deltaPr-A9 cells shown in FigS2A is not very distinct. Including images at higher magnification could help clarify this feature.

      Good comment. We have enlarged the images for better view and revised the context.

      (10) Both protein and RNA expression analysis have been performed on WT CaSki cells and FANCI-2 KO cells. If I am correct there is little overlap between the significantly changed gene products. What does this mean? Have you looked into the comparison?

      The DEGs identified from RNA-seq indicated a genome wide transcriptome change, while the protein array we used only covered 105 soluble protein receptors. However, we did find 9/15 (60%) membrane proteins in cell lysates (PODXL2, ECM1, NECTIN2, MCAM, ADAM9, CDH5, ADAM10, ITGA5, NOTCH1, SCARF2, ADAM8, TIMP2, LGALS3BP, CDH13, and ITGB6) exhibited consistent changes in expression (underlined) by both RNA-seq and protein array assays. We have revised the text with this information (page 11). Other six proteins (40%) had inconsistent expression correlation in two assays could be due to post-translational mechanisms, such as protein stability, modifications and secretion, etc.  

      (11) Figure S7, which represents TCGA data and survival is quite complex. It would be more effective to display a similar figure for FANCI-2, as was done for MCAM in Figure 7I, to simplify the comparison and enhance clarity.

      Thanks. However, the suggested figure for lnc-FANCI-2 was published in PNAS paper already (Liu H., et al. PNAS, 2021).  The Figure S8 in this revision is the result from our in-house GradientScanSurv pipeline, a new way to correlate the expression and survival more accurately.

      What do the Figures look like if you analyse only HPV16+ patients versus HPV18+ patients, considering that FANCI-2 upregulation in cell lines is related to HPV16 and not 18? Is there an effect of histotype? Or tumor stage?

      HPV18 infected keratinocytes express high level of lnc-FANCI-2. Two HPV18<sup>+</sup> HeLa and C4II cell lines and HPV-negative cell lines, such as HCT116 cells, which do not express lnc-FANCI-2 could be due to the presence of some unknow repressive factors. We found that lnc-FANCI-2 promoter functions well in responding to YY1 binding in CaSki and C33A cells expressing lnc-FANCI-2 but does not so in HeLa and HCT116 cells in our dual luciferase assays. 

      (12) It remains puzzling that FANCI-2 upregulation was previously shown to already occur in CIN lesions and increase further in cervical cancer, while the current data indicate that FANCI-2 suppresses AKT activation. If I am correct Akt activation has been linked to cervical carcinogenesis. Similarly, line 434 states that increased MCAM might promote cervical tumorigenesis, implying that low FANCI-2 would stimulate tumorigenesis. If I understand correctly, the increase in FANCI-2 observed in CIN lesions would reflect a "brake" on the carcinogenic pathway and its sustained increase in cancer might indicate that growth is still (partly) controlled. As mentioned earlier, a Figure illustrating the relation between FANCI-2, HPV, and the carcinogenic process would be beneficial for clarity.

      Yes. Increased MCAM, but low level of lnc-FANCI-2, correlates with poor cervical cancer survival. We have revised Figure 8E to illustrate this relation better.  

      (13) May part of the potentially conflicting findings be explained by CaSki cells being of metastatic origin? Related to this, does the expression of FANCI-2 or MALM depend on the tumor stage?

      Thanks for this important suggestion. Unfortunately, we found that the expression of lnc-FANCI-2 and MCAM is not associated with cervical cancer stage based on the TCGA data (http://gepia.cancer-pku.cn/index.html). See the data below:

      Author response image 2.

      Despite some lingering uncertainty, the extensive experiments conducted using KO and KD cells do provide compelling evidence that lnc-FANCI-2 function is linked to RAS signaling and EMT.

      Thanks for your positive review and instructive comments.

      Reviewer #3 (Recommendations for the authors):

      (1) The authors observed the increased Inc-FANCI-2 in HPV 16 and 18 transduced cells, and other cervical cancer tissues as well, HPV-18 positive HeLa cells exhibited different expressions of Inc-FANCI-2. I suggest authors provide more discussions on this difference, for example, HPV genotypes. HPV genome status in host cells? Cell types?

      Thanks. We found the keratinocyte infections with HPV16, HPV18, and other HR-HPVs could induce lnc-FANCI-2 expression (Liu H., et al. PNAS, 2021). In this report, we found HPV18<sup>+</sup> HeLa and C4II cells and other HPV-negative cell lines do not. Our preliminary data on lnc-FANCI-2 promoter activity assays showed the presence of a negative regulatory factor (s) in non-lnc-FANCI-2 expressing cells. See the data in Author response image 1.

      We have revised our discussion by inclusion these sets of the luciferase data as data not shown.

      (2) I suggest the authors discuss more details on how the changes of RAS signaling in KO cells help our further understanding of the molecular mechanisms for HPV-associated full-cell transformation and malignancy in addition to the well-known functions of HPV E6 and E7.

      Thanks. We have modified the Figure 8E as suggested by reviewer 2 and revised the discussion further.

    1. Author Response

      The following is the authors’ response to the original reviews.

      Reviewer #1:

      Summary:

      This paper performs fine-mapping of the silkworm mutants bd and its fertile allelic version, bdf, narrowing down the causal intervals to a small interval of a handful of genes. In this region, the gene orthologous to mamo is impaired by a large indel, and its function is later confirmed using expression profiling, RNAi, and CRISPR KO. All these experiments are convincingly showing that mamo is necessary for the suppression of melanic pigmentation in the silkworm larval integument. The authors also use in silico and in vitro assays to probe the potential effector genes that mamo may regulate. Strengths: The genotype-to-phenotype workflow, combining forward (mapping) and reverse genetics (RNAi and CRISPR loss-of-function assays) linking mamo to pigmentation are extremely convincing.

      Response: Thank you very much for your affirmation of our work. The reviewer discussed the parts of our manuscript that involve evolution sentence by sentence. We have further refined the description in this regard and improved the logical flow. Thank you again for your help.

      Weaknesses:

      1) The last section of the results, entitled "Downstream target gene analysis" is primarily based on in silico genome-wide binding motif predictions.

      While the authors identify a potential binding site using EMSA, it is unclear how much this general approach over-predicted potential targets. While I think this work is interesting, its potential caveats are not mentioned. In fact the Discussion section seems to trust the high number of target genes as a reliable result. Specifically, the authors correctly say: "even if there are some transcription factor-binding sites in a gene, the gene is not necessarily regulated by these factors in a specific tissue and period", but then propose a biological explanation that not all binding sites are relevant to expression control. This makes a radical short-cut that predicted binding sites are actual in vivo binding sites. This may not be true, as I'd expect that only a subset of binding motifs predicted by Positional Weight Matrices (PWM) are real in vivo binding sites with a ChIP-seq or Cut-and-Run signal. This is particularly problematic for PWM that feature only 5-nt signature motifs, as inferred here for mamo-S and mamo-L, simply because we can expect many predicted sites by chance.

      Response: Thank you very much for your careful work. The analysis and identification of transcription factor-binding sites is an important issue in gene regulation research. Techniques such as ChIP-seq can be used to experimentally identify the binding sites of transcription factors (TFs). However, reports using these techniques often only detect specific cell types and developmental stages, resulting in a limited number of downstream target genes for some TFs. Interestingly, TFs may regulate different downstream target genes in different cell types and developmental stages.

      Previous research has suggested that the ZF-DNA binding interface can be understood as a “canonical binding model”, in which each finger contacts DNA in an antiparallel manner. The binding sequence of the C2H2-ZF motif is determined by the amino acid residue sequence of its α-helical component. Considering the first amino acid residue in the α-helical region of the C2H2-ZF domain as position 1, positions -1, 2, 3, and 6 are key amino acids for recognizing and binding DNA. The residues at positions -1, 3, and 6 specifically interact with base 3, base 2, and base 1 of the DNA sense sequence, respectively, while the residue at position 2 interacts with the complementary DNA strand (Wolfe SA et al., 2000; Pabo CO et al., 2001). Based on this principle, the binding sites of C2H2-ZF have good reference value. For the 5-nt PWM sequence, we referred to the study of D. melanogaster, which was identified by EMSA (Shoichi Nakamura et al., 2019). In the new version, we have rewritten this section.

      Pabo CO, Peisach E, Grant RA. Design and selection of novel Cys2His2 zinc finger proteins. Annu Rev Biochem. 2001;70:313-340.

      Wolfe SA, Nekludova L, Pabo CO. DNA recognition by Cys2His2 zinc finger proteins. Annu Rev Biophys Biomol Struct. 2000;29:183-212.

      Nakamura S, Hira S, Fujiwara M, et al. A truncated form of a transcription factor Mamo activates vasa in Drosophila embryos. Commun Biol. 2019;2:422. Published 2019 Nov 20.

      2) The last part of the current discussion ("Notably, the industrial melanism event, in a short period of several decades ... a more advanced self-regulation program") is flawed with important logical shortcuts that assign "agency" to the evolutionary process. For instance, this section conveys the idea that phenotypically relevant mutations may not be random. I believe some of this is due to translation issues in English, as I understand that the authors want to express the idea that some parts of the genome are paths of least resistance for evolutionary change (e.g. the regulatory regions of developmental regulators are likely to articulate morphological change). But the language and tone is made worst by the mention that in another system, a mechanism involving photoreception drives adaptive plasticity, making it sound like the authors want to make a Lamarckian argument here (inheritance of acquired characteristics), or a point about orthogenesis (e.g. the idea that the environment may guide non-random mutations).

      Because this last part of the current discussion suffers from confused statements on modes and tempo of regulatory evolution and is rather out of topic, I would suggest removing it.

      In any case, it is important to highlight here that while this manuscript is an excellent genotype-to-phenotype study, it has very few comparative insights on the evolutionary process. The finding that mamo is a pattern or pigment regulatory factor is interesting and will deserve many more studies to decipher the full evolutionary study behind this Gene Regulatory Network.

      Response: Thank you very much for your careful work. In this part of the manuscript, we introduced some assumptions that make the statement slightly unconventional. The color pattern of insects is an adaptive trait. The bd and bdf mutants used in the study are formed spontaneously. As a frequent variation and readily observable phenotype, color patterns have been used as models for evolutionary research (Wittkopp PJ et al., 2011). Darwin's theory of natural selection has epoch-making significance. I deeply believe in the theory that species strive to evolve through natural selection. However, with the development of molecular genetics, Darwinism’s theory of undirected random mutations and slow accumulation of micromutations resulting in phenotype evolution has been increasingly challenged.

      The prerequisite for undirected random mutations and micromutations is excessive reproduction to generate a sufficiently large population. A sufficiently large population can contain sufficient genotypes to face various survival challenges. However, it is difficult to explain how some small groups and species with relatively low fertility rates have survived thus far. More importantly, the theory cannot explain the currently observed genomic mutation bias. In scientific research, every theory is constantly being modified to adapt to current discoveries. The most famous example is the debate over whether light is a particle or a wave, which has lasted for hundreds of years. However, in the 20th century, both sides seemed to compromise with each other, believing that light has a wave‒particle duality.

      In summary, we have rewritten this section to reduce unnecessary assumptions.

      Wittkopp PJ, Kalay G. Cis-regulatory elements: molecular mechanisms and evolutionary processes underlying divergence. Nat Rev Genet. 2011;13(1):59-69.

      Minor Comment:

      The gene models presented in Figure 1 are obsolete, as there are more recent annotations of the Bm-mamo gene that feature more complete intron-exon structures, including for the neighboring genes in the bd/bdf intervals. It remains true that the mamo locus encodes two protein isoforms.

      An example of the Bm-mamo locus annotation, can be found at: https://www.ncbi.nlm.nih.gov/gene/101738295 RNAseq expression tracks (including from larval epidermis) can be displayed in the embedded genome browser from the link above using the "Configure Tracks" tool.

      Based on these more recent annotations, I would say that most of the work on the two isoforms remains valid, but FigS2, and particularly Fig.S2C, need to be revised.

      Response: Thank you very much for your careful work. In this study, we referred to the predicted genes of SilkDB, NCBI and Silkbase. In different databases, there are varying degrees of differences in the number of predicted genes and the length of gene mRNA. Because the SilkDB database is based on the first silkworm genome, it has been used for the longest time and has a relatively large number of users. In the revised manuscript, we have added the predicted genes of NCBI and Silkbase in Figure S1.

      Author response image 1.

      The predicted genes and qPCR analysis of candidate genes in the responsible genomic region for bd mutant. (A) The predicted genes in SilkDB;(B) the predicted genes in Genbak;(C) the predicted genes in Silkbase;(D) analysis of nucleotide differences in the responsible region of bd;(E) investigation of the expression level of candidate genes.

      Reviewer #2 (Public Review):

      Summary:

      The authors tried to identify new genes involved in melanin metabolism and its spatial distribution in the silkworm Bombyx mori. They identified the gene Bm-mamo as playing a role in caterpillar pigmentation. By functional genetic and in silico approaches, they identified putative target genes of the Bm-mamo protein. They showed that numerous cuticular proteins are regulated by Bm-mamo during larval development.

      Strengths:

      • preliminary data about the role of cuticular proteins to pattern the localization of pigments

      • timely question

      • challenging question because it requires the development of future genetic and cell biology tools at the nanoscale

      Response: Thank you very much for your affirmation of our work. The reviewer's familiarity with the color patterns of Lepidoptera is helpful, and the recommendation raised has provided us with very important assistance. This has allowed us to make significant progress with our manuscript.

      Weaknesses:

      • statistical sampling limited

      • the discussion would gain in being shorter and refocused on a few points, especially the link between cuticular proteins and pigmentation. The article would be better if the last evolutionary-themed section of the discussion is removed.

      A recent paper has been published on the same gene in Bombyx mori (https://www.sciencedirect.com/science/article/abs/pii/S0965174823000760) in August 2023. The authors must discuss and refer to this published paper through the present manuscript.

      Response: Thank you very much for your careful work. First, we believe that competitive research is sometimes coincidental and sometimes intentional. Our research began in 2009, when we began to configure the recombinant population. In 2016, we published an article on comparative transcriptomics (Wu et al. 2016). The article mentioned above has a strong interest in our research and is based on our transcriptome analysis for further research, with the aim of making a preemptive publication. To discourage such behavior, we cannot cite it and do not want to discuss it in our paper.

      Songyuan Wu et al. Comparative analysis of the integument transcriptomes of the black dilute mutant and the wild-type silkworm Bombyx mori. Sci Rep. 2016 May 19:6:26114. doi: 10.1038/srep26114.

      Reviewer #1 (Recommendations For The Authors):

      1) please consider using a more recent annotation model of the B. mori genome to revise your Result Section 1, Fig.1, and Fig. S2. https://www.ncbi.nlm.nih.gov/gene/101738295

      Specifically, you used BGIM_ gene models, while the current annotation such as the one above featured in the NCBI database provides more accurate intron-exon structures without splitting mamo into tow genes. I believe this can be done with minor revisions of the figures, and you could keep the BGIM_ gene names for the text.

      Response: Thank you very much for your careful work. The GenBank of NCBI (National Center for Biotechnology Information) is a very good database that we often use and refer to in this research process. Our research started in 2009, so we mainly referred to the SilkDB database (Jun Duan et al., 2010), although other databases also have references, such as NCBI and Silkbase (https://silkbase.ab.a.u-tokyo.ac.jp/cgi-bin/index.cgi). Because the SilkDB database was constructed based on the first published silkworm genome data, it has been used for the longest time and has a relatively large number of users. Recently, researchers are still using these data (Kejie Li et al., 2023).

      The problem with predicting the mamo gene as two genes (BGIBMGA012517 and BGIBMGA012518) in SilkDB is mainly due to the presence of alternative splicing of the mamo gene. BGIBMGA012517 corresponds to the shorter transcript (mamo-s) of the mamo gene. Due to the differences in sequencing individuals, sequencing methods, and methods of gene prediction, there are differences in the number and sequence of predicted genes in different databases. We added the pattern diagram of predicted genes from NCBI and Silkbase, and the expression levels of new predicted genes are shown in Supplemental Figure S1.

      Jun Duan et al., SilkDB v2.0: a platform for silkworm (Bombyx mori) genome biology. Nucleic Acids Res. 2010 Jan;38(Database issue): D453-6. doi: 10.1093/nar/gkp801. Kejie Li et al., Transcriptome analysis reveals that knocking out BmNPV iap2 induces apoptosis by inhibiting the oxidative phosphorylation pathway. Int J Biol Macromol. 2023 Apr 1;233:123482. doi: 10.1016/j.ijbiomac.2023.123482. Epub 2023 Jan 31.

      Author response image 2.

      The predicted genes and qPCR analysis of candidate genes in the responsible genomic region for bd mutant. (A) The predicted genes in SilkDB;(B) the predicted genes in Genbak;(C) the predicted genes in Silkbase;(D) analysis of nucleotide differences in the responsible region of bd;(E) investigation of the expression level of candidate genes.

      2) As I mentioned in my public review, I strongly believe the interpretation of the PWM binding analyses require much more conservative statements taking into account the idea that short 5-nt motifs are expected by chance. The work in this section is interesting, but the manuscript would benefit from a quite significant rewrite of the corresponding Discussion section, making it that the in silico approach is prone to the identification of many sites in the genomes, and that very few of those sites are probably relevant for probabilistic reasons. I would recommend statements such as "Future experiments assessing the in vivo binding profile of Bm-mamo (eg. ChIP-seq or Cut&Run), will be required to further understand the GRNs controlled by mamo in various tissues".

      Response: Thank you very much for your careful work. Previous research has suggested that the ZF-DNA binding interface can be understood as a “canonical binding model”, in which each finger contacts DNA in an antiparallel manner. The binding sequence of the C2H2-ZF motif is determined by the amino acid residue sequence of its α-helical component. Considering the first amino acid residue in the α-helical region of the C2H2-ZF domain as position 1, positions -1, 2, 3, and 6 are key amino acids for recognizing and binding DNA. The residues at positions -1, 3, and 6 specifically interact with base 3, base 2, and base 1 of the DNA sense sequence, respectively, while the residue at position 2 interacts with the complementary DNA strand (Wolfe SA et al., 2000; Pabo CO et al., 2001). Based on this principle, the prediction of DNA recognition motifs of C2H2-type zinc finger proteins currently has good accuracy.

      The predicted DNA binding sequence (GTGCGTGGC) of the mamo protein in Drosophila melanogaster was highly consistent with that of silkworms. In addition, in D. melanogaster, the predicted DNA binding sequence of mamo, the bases at positions 1 to 7 (GTGCGTG), was highly similar to the DNA binding sequence obtained from EMSA experiments (Seiji Hira et al., 2013). Furthermore, in another study on the mamo protein of Drosophila melanogaster, five bases (TGCGT) were used as the DNA recognition core sequence of the mamo protein (Shoichi Nakamura et al., 2019). In the JASPAR database (https://jaspar.genereg.net), there are also some shorter (4-6 nt) DNA recognition sequences; for example, the DNA binding sequence of Ubx is TAAT (ID MA0094.1) in Drosophila melanogaster. However, we used longer DNA binding motifs (9 nt and 15 nt) of mamo to study the 2 kb genomic regions near the predicted gene. Over 70% of predicted genes were found to have these feature sequences near them. This analysis method is carried out with common software and processes. Due to sufficient target proteins, the accessibility of DNA, the absence of suppressors, the suitability of ion environments, etc., zinc finger protein transcription factors are more likely to bind to specific DNA sequences in vitro than in vivo. Using ChIP-seq or Cut&Run techniques to analyze various tissues and developmental stages in silkworms can yield one comprehensive DNA-binding map of mamo, and some false positives generated by predictions can be excluded. Thank you for your suggestion. We will conduct this work in the next research step. In addition, for brevity, we deleted the predicted data (Supplemental Tables S7 and S8) that used shorter motifs.

      Pabo CO, Peisach E, Grant RA. Design and selection of novel Cys2His2 zinc finger proteins. Annu Rev Biochem. 2001;70:313-340.

      Wolfe SA, Nekludova L, Pabo CO. DNA recognition by Cys2His2 zinc finger proteins. Annu Rev Biophys Biomol Struct. 2000;29:183-212.

      Anton V Persikov et al., De novo prediction of DNA-binding specificities for Cys2His2 zinc finger proteins. Nucleic Acids Res. 2014 Jan;42(1):97-108. doi: 10.1093/nar/gkt890. Epub 2013 Oct 3.

      Seiji Hira et al., Binding of Drosophila maternal Mamo protein to chromatin and specific DNA sequences. Biochem Biophys Res Commun. 2013 Aug 16;438(1):156-60. doi: 10.1016/j.bbrc.2013.07.045. Epub 2013 Jul 20.

      Shoichi Nakamura et al., A truncated form of a transcription factor Mamo activates vasa in Drosophila embryos. Commun Biol. 2019 Nov 20;2: 422. doi: 10.1038/s42003-019-0663-4. eCollection 2019.

      3) In my opinion, the last section of the Discussion needs to be completely removed ("Notably, the industrial melanism event, in a short period of several decades ... a more advanced self-regulation program"), as it is over-extending the data into evolutionary interpretations without any support. I would suggest instead writing a short paragraph asking whether the pigmentary role of mamo is a Lepidoptera novelty, or if it could have been lost in the fly lineage.

      Below, I tried to comment point-by-point on the main issues I had.

      Wu et al: Notably, the industrial melanism event, in a short period of several decades, resulted in significant changes in the body color of multiple Lepidoptera species(46). Industrial melanism events, such as changes in the body color of pepper moths, are heritable and caused by genomic mutations(47).

      Yes, but the selective episode was brief, and the relevant "carbonaria" mutations may have existed for a long time at low-frequency in the population.

      Response: Thank you very much for your careful work. Moth species often have melanic variants at low frequencies outside industrial regions. Recent molecular work on genetics has revealed that the melanic (carbonaria) allele of the peppered moth had a single origin in Britain. Further research indicated that the mutation event causing industrial melanism of peppered moth (Biston betularia) in the UK is the insertion of a transposon element into the first intron of the cortex gene. Interestingly, statistical inference based on the distribution of recombined carbonaria haplotypes indicates that this transposition event occurred in approximately 1819, a date highly consistent with a detectable frequency being achieved in the mid-1840s (Arjen E Van't Hof, et al., 2016). From molecular research, it is suggested that this single origin melanized mutant (carbonaria) was generated near the industrial development period, rather than the ancient genotype, in the UK. We have rewritten this part of the manuscript.

      Arjen E Van't Hof, et al., The industrial melanism mutation in British peppered moths is a transposable element. Nature. 2016 Jun 2;534(7605):102-5. doi: 10.1038/nature17951.

      Wu et al: If relying solely on random mutations in the genome, which have a time unit of millions of years, to explain the evolution of the phenotype is not enough.

      What you imply here is problematic for several reasons.

      First, as you point out later, some large-effect mutations (e.g. transpositions) can happen quickly.

      Second, it's unclear what "the time units of million of years" means here... mutations occur, segregate in populations, and are selected. The speed of this process depends on the context and genetic architectures.

      Third, I think I understand what you mean with "to explain the evolution of the phenotype is not enough", but this would probably need a reformulation and I don't think it's relevant to bring it here. After all, you used loss-of-function mutants to explain the evolution of artificially selected mutants. The evolutionary insights from these mutants are limited. Random mutations at the mamo locus are perfectly sufficient here to explain the bd and bdf phenotypes and larval traits.

      Response: Thank you very much for your careful work. Charles Darwin himself, who argued that “natural selection can act only by taking advantage of slight successive variations; she can never take a leap, but must advance by the shortest and slowest steps” (Darwin, C. R. 1859). This ‘micromutational’ view of adaptation proved extraordinarily influential. However, the accumulation of micromutations is a lengthy process, which requires a very long time to evolve a significant phenotype. This may be only a proportion of the cases. Interestingly, recent molecular biology studies have shown that the evolution of some morphological traits involves a modest number of genetic changes (H Allen Orr. 2005).

      One example is the genetic basis analysis of armor-plate reduction and pelvic reduction of the three-spined stickleback (Gasterosteus aculeatus) in postglacial lakes. Although the marine form of this species has thick armor, the lake population (which was recently derived from the marine form) does not. The repeated independent evolution of lake morphology has resulted in reduced armor plate and pelvic structures, and there is no doubt that these morphological changes are adaptive. Research has shown that pelvic loss in different natural populations of three-spined stickleback fish occurs by regulatory mutations deleting a tissue-specific enhancer (Pel) of the pituitary homeobox transcription factor 1 (Pitx1) gene. The researchers genotyped 13 pelvic-reduced populations of three-spined stickleback from disparate geographic locations. Nine of the 13 pelvic-reduced stickleback populations had sequence deletions of varying lengths, all of which were located at the Pel enhancer. Relying solely on random mutations in the genome cannot lead to such similar mutation forms among different populations. The author suggested that the Pitx1 locus of the stickleback genome may be prone to double-stranded DNA breaks that are subsequently repaired by NHEJ (Yingguang Frank Chan et al., 2010).

      The bd and bdf mutants used in the study are formed spontaneously. Natural mutation is one of the driving forces of evolution. Nevertheless, we have rewritten the content of this section.

      Darwin, C. R. The Origin of Species (J. Murray, London, 1859).

      H Allen Orr. The genetic theory of adaptation: a brief history. Nat Rev Genet. 2005 Feb;6(2):119-27. doi: 10.1038/nrg1523.

      Yingguang Frank Chan et al., Adaptive evolution of pelvic reduction in sticklebacks by recurrent deletion of a Pitx1 enhancer. Science. 2010 Jan 15;327(5963):302-5. doi: 10.1126/science.1182213. Epub 2009 Dec 10.

      Wu et al: Interestingly, the larva of peppered moths has multiple visual factors encoded by visual genes, which are conserved in multiple Lepidoptera, in the skin. Even when its compound eyes are covered, it can rely on the skin to feel the color of the environment to change its body color and adapt to the environment(48). Therefore, caterpillars/insects can distinguish the light wave frequency of the background. We suppose that perceptual signals can stimulate the GRN, the GRN guides the expression of some transcription factors and epigenetic factors, and the interaction of epigenetic factors and transcription factors can open or close the chromatin of corresponding downstream genes, which can guide downstream target gene expression.

      This is extremely confusing because you are bringing in a plastic trait here. It's possible there is a connection between the sensory stimulus and the regulation of mamo in peppered moths, but this is a mere hypothesis. Here, by mentioning a plastic trait, this paragraph sounds as if it was making a statement about directed evolution, especially after implying in the previous sentence that (paraphrasing) "random mutations are not enough". To be perfectly honest, the current writing could be misinterpreted and co-opted by defenders of the Intelligent Design doctrine. I believe and trust this is not your intention.

      Response: Thank you very much for your careful work. The plasticity of the body color of peppered moth larvae is very interesting, but we mainly wanted to emphasize that their skin shows the products of visual genes that can sense the color of the environment by perceiving light. Moreover, these genes are conserved in many insects. Human skin can also perceive light by opsins, suggesting that they might initiate light–induced signaling pathways (Haltaufderhyde K et al., 2015). This indicates that the perception of environmental light by the skin of animals and the induction of feedback through signaling pathways is a common phenomenon. For clarity, we have rewritten this section of the manuscript.

      Haltaufderhyde K, Ozdeslik RN, Wicks NL, Najera JA, Oancea E. Opsin expression in human epidermal skin. Photochem Photobiol. 2015;91(1):117-123.

      Wu et al: In addition, during the opening of chromatin, the probability of mutation of exposed genomic DNA sequences will increase (49).

      Here again, this is veering towards a strongly Lamarckian view with the environment guiding specific mutation. I simply cannot see how this would apply to mamo, nothing in the current article indicates this could be the case here. Among many issues with this, it's unclear how chromatin opening in the larval integument may result in heritable mutations in the germline.

      Response: Thank you very much for your careful work. Previous studies have shown that there is a mutation bias in the genome; compared with the intergenic region, the mutation frequency is reduced by half inside gene bodies and by two-thirds in essential genes. In addition, they compared the mutation rates of genes with different functions. The mutation rate in the coding region of essential genes (such as translation) is the lowest, and the mutation rates in the coding region of specialized functional genes (such as environmental response) are the highest. These patterns are mainly affected by the traits of the epigenome (J Grey Monroe et al., 2022).

      In eukaryotes, chromatin is organized as repeating units of nucleosomes, each consisting of a histone octamer and the surrounding DNA. This structure can protect DNA. When one gene is activated, the chromatin region of this gene is locally opened, becoming an accessible region. Research has found that DNA accessibility can lead to a higher mutation rate in the region (Radhakrishnan Sabarinathan et al., 2016; Schuster-Böckler B et al., 2012; Lawrence MS et al., 2013; Polak P et al., 2015). In addition, the BTB-ZF protein mamo belongs to this family and can recruit histone modification factors such as DNA methyltransferase 1 (DMNT1), cullin3 (CUL3), histone deacetylase 1 (HDAC1), and histone acetyltransferase 1 (HAT1) to perform chromatin remodeling at specific genomic sites. Although mutations can be predicted by the characteristics of apparent chromatin, the forms of mutations are diverse and random. Therefore, this does not violate randomness. For clarity, we have rewritten this section of the manuscript.

      J Grey Monroe, Mutation bias reflects natural selection in Arabidopsis thaliana. Nature. 2022 Feb;602(7895):101-105.

      Sabarinathan R, Mularoni L, Deu-Pons J, Gonzalez-Perez A, López-Bigas N. Nucleotide excision repair is impaired by binding of transcription factors to DNA. Nature. 2016;532(7598):264-267.

      Schuster-Böckler B, Lehner B. Chromatin organization is a major influence on regional mutation rates in human cancer cells. Nature. 2012;488(7412):504-507.

      Lawrence MS, Stojanov P, Polak P, et al. Mutational heterogeneity in cancer and the search for new cancer-associated genes. Nature. 2013;499(7457):214-218.

      Polak P, Karlić R, Koren A, et al. Cell-of-origin chromatin organization shapes the mutational landscape of cancer. Nature. 2015;518(7539):360-364.

      Mathew R, Seiler MP, Scanlon ST, et al. BTB-ZF factors recruit the E3 ligase cullin 3 to regulate lymphoid effector programs. Nature. 2012;491(7425):618-621.

      Wu et al: Transposon insertion occurs in a timely manner upstream of the cortex gene in melanic pepper moths (47), which may be caused by the similar binding of transcription factors and opening of chromatin.

      No, we do not think that the peppered moth mutation is Lamarckian at all, as seems to be inferred here (notice that by mentioning the peppered moth twice, you are juxtaposing a larval plastic trait and then a purely genetic wing trait, making it even more confusing). Also, the "in a timely manner" is superfluous, because all the data are consistent with a chance mutation being eventually picked up by strong directional mutation. The mutation and selection did NOT occur at the same time.

      Response: Thank you very much for your careful work. The insertion of one transposon into the first intron of the cortex gene of industrial melanism in peppered moth occurred in approximately 1819, which is similar to the time of industrial development in the UK (Arjen E Van't Hof, et al., 2016). In multiple species of Heliconius, the cortex gene is the shared genetic basis for the regulation of wing coloring patterns. Interestingly, the SNP of the cortex, associated with the wing color pattern, does not overlap among different Heliconius species, such as H. erato dephoon and H. erato favorinus, which suggests that the mutations of this cortex gene have different origins (Nadeau NJ et al., 2016). In addition, in Junonia coenia (van der Burg KRL et al., 2020) and Bombyx mori (Ito K et al., 2016), the cortex gene is a candidate for regulating changes in wing coloring patterns. Overall, the cortex gene is an evolutionary hotspot for the variation of multiple butterfly and moth wing coloring patterns. In addition, it was observed that the variations in the cortex are diverse in these species, including SNPs, indels, transposon insertions, inversions, etc. This indicates that although there are evolutionary hotspots in the insect genome, this variation is random. Therefore, this is not completely detached from randomness.

      Arjen E Van't Hof, et al., The industrial melanism mutation in British peppered moths is a transposable element. Nature. 2016 Jun 2;534(7605):102-5. doi: 10.1038/nature17951.

      Nadeau NJ, Pardo-Diaz C, Whibley A, et al. The gene cortex controls mimicry and crypsis in butterflies and moths. Nature. 2016;534(7605):106-110.

      van der Burg KRL, Lewis JJ, Brack BJ, Fandino RA, Mazo-Vargas A, Reed RD. Genomic architecture of a genetically assimilated seasonal color pattern. Science. 2020;370(6517):721-725.

      Ito K, Katsuma S, Kuwazaki S, et al. Mapping and recombination analysis of two moth colour mutations, Black moth and Wild wing spot, in the silkworm Bombyx mori. Heredity (Edinb). 2016;116(1):52-59.

      Wu et al: Therefore, we proposed that the genetic basis of color pattern evolution may mainly be system-guided programmed events that induce mutations in specific genomic regions of key genes rather than just random mutations of the genome.

      While the mutational target of pigment evolution may involve a handful of developmental regulator genes, you do not have the data to infer such a strong conclusion at the moment.

      The current formulation is also quite strong and teleological: "system-guided programmed events" imply intentionality or agency, an idea generally assigned to the anti-scientific Intelligent Design movement. There are a few examples of guided mutations, such as the adaptation phase of gRNA motifs in bacterial CRISPR assays, where I could see the term ""system-guided programmed events" to be applicable. But it is irrelevant here.

      Response: Thank you very much for your careful work. The CRISPR-CAS9 system is indeed very well known. In addition, recent studies have found the existence of a Cas9-like gene editing system in eukaryotes, such as Fanzor. Fanzor (Fz) was reported in 2013 as a eukaryotic TnpB-IS200/IS605 protein encoded by the transposon origin, and it was initially thought that the Fz protein (and prokaryotic TnpBs) might regulate transposon activity through methyltransferase activity (Saito M et al., 2023). Fz has recently been found to be a eukaryotic CRISPR‒Cas system. Although this system is found in fungi and mollusks, it raises hopes for scholars to find similar systems in other higher animals. However, before these gene-editing systems became popular, zinc finger nucleases (ZFNs) were already being studied as a gene-editing system in many species. The mechanism by which ZFN recognizes DNA depends on its zinc finger motif (Urnov FD et al., 2005). This is consistent with the mechanism by which transcription factors recognize DNA-binding sites.

      Furthermore, a very important evolutionary event in sexual reproduction is chromosome recombination during meiosis, which helps to produce more abundant alleles. Current research has found that this recombination event is not random. In mice and humans, the PRDM9 transcription factors are able to plan the sites of double-stranded breaks (DSBs) in meiosis recombination. PRDM9 is a histone methyltransferase consisting of three main regions: an amino-terminal region resembling the family of synovial sarcoma X (SSX) breakpoint proteins, which contains a Krüppel-associated box (KRAB) domain and an SSX repression domain (SSXRD); a PR/SET domain (a subclass of SET domains), surrounded by a pre-SET zinc knuckle and a post-SET zinc finger; and a long carboxy-terminal C2H2 zinc finger array. In most mammalian species, during early meiotic prophase, PRDM9 can determine recombination hotspots by H3K4 and H3K36 trimethylation (H3K4me3 and H3K36me3) of nucleosomes near its DNA-binding site. Subsequently, meiotic DNA DSBs are formed at hotspots through the combined action of SPO11 and TOPOVIBL. In addition, some proteins (such as RAD51) are involved in repairing the break point. In summary, programmed events of induced and repaired DSBs are widely present in organisms (Bhattacharyya T et al., 2019).

      These studies indicate that on the basis of randomness, the genome also exhibits programmability.

      Saito M, Xu P, Faure G, et al. Fanzor is a eukaryotic programmable RNA-guided endonuclease. Nature. 2023;620(7974):660-668.

      Urnov FD, Miller JC, Lee YL, et al. Highly efficient endogenous human gene correction using designed zinc-finger nucleases. Nature. 2005;435(7042):646-651.

      Bhattacharyya T, Walker M, Powers NR, et al. Prdm9 and Meiotic Cohesin Proteins Cooperatively Promote DNA Double-Strand Break Formation in Mammalian Spermatocytes [published correction appears in Curr Biol. 2021 Mar 22;31(6):1351]. Curr Biol. 2019;29(6):1002-1018.e7.

      Wu et al: Based on this assumption, animals can undergo phenotypic changes more quickly and more accurately to cope with environmental changes. Thus, seemingly complex phenotypes such as cryptic coloring and mimicry that are highly similar to the background may have formed in a short period. However, the binding sites of some transcription factors widely distributed in the genome may be reserved regulatory interfaces to cope with potential environmental changes. In summary, the regulation of genes is smarter than imagined, and they resemble a more advanced self-regulation program.

      Here again, I can agree with the idea that certain genetic architectures can evolve quickly, but I cannot support the concept that the genetic changes are guided or accelerated by the environment. And again, none of this is relevant to the current findings about Bm-mamo.

      Response: Thank you very much for your careful work. Darwin's theory of natural selection has epoch-making significance. I deeply believe in the theory that species strive to evolve through natural selection. However, with the development of molecular genetics, Darwinism’s theory of undirected random mutations and slow accumulation of micromutations resulting in phenotype evolution has been increasingly challenged.

      The prerequisite for undirected random mutations and micromutations is excessive reproduction to generate a sufficiently large population. A sufficiently large population can contain sufficient genotypes to face various survival challenges. However, it is difficult to explain how some small groups and species with relatively low fertility rates have survived thus far. More importantly, the theory cannot explain the currently observed genomic mutation bias. In scientific research, every theory is constantly being modified to adapt to current discoveries. The most famous example is the debate over whether light is a particle or a wave, which has lasted for hundreds of years. However, in the 20th century, both sides seemed to compromise with each other, believing that light has a wave‒particle duality.

      Epigenetics has developed rapidly since 1987. Epigenetics has been widely accepted, defined as stable inheritance caused by chromosomal conformational changes without altering the DNA sequence, which differs from genetic research on variations in gene sequences. However, an increasing number of studies have found that histone modifications can affect gene sequence variation. In addition, both histones and epigenetic factors are essentially encoded by genes in the genome. Therefore, genetics and epigenetics should be interactive rather than parallel. However, some transcription factors play an important role in epigenetic modifications. Meiotic recombination is a key process that ensures the correct separation of homologous chromosomes through DNA double-stranded break repair mechanisms. The transcription factor PRDM9 can determine recombination hotspots by H3K4 and H3K36 trimethylation (H3K4me3 and H3K36me3) of nucleosomes near its DNA-binding site (Bhattacharyya T et al., 2019). Interestingly, mamo has been identified as an important candidate factor for meiosis hotspot setting in Drosophila (Winbush A et al., 2021).

      Bhattacharyya T, Walker M, Powers NR, et al. Prdm9 and Meiotic Cohesin Proteins Cooperatively Promote DNA Double-Strand Break Formation in Mammalian Spermatocytes [published correction appears in Curr Biol. 2021 Mar 22;31(6):1351]. Curr Biol. 2019;29(6):1002-1018.e7.

      Winbush A, Singh ND. Genomics of Recombination Rate Variation in Temperature-Evolved Drosophila melanogaster Populations. Genome Biol Evol. 2021;13(1): evaa252.

      Reviewer #2 (Recommendations For The Authors):

      Major comments

      Response: Thank you very much for your careful work. First, we believe that competitive research is sometimes coincidental and sometimes intentional. Our research began in 2009, when we began to configure the recombinant population. In 2016, we published an article on comparative transcriptomics (Wu et al. 2016). The article mentioned above has a strong interest in our research and is based on our transcriptome analysis for further research, with the aim of making a preemptive publication.

      To discourage such behavior, we cannot cite it and do not want to discuss it in our paper.

      Songyuan Wu et al. Comparative analysis of the integument transcriptomes of the black dilute mutant and the wild-type silkworm Bombyx mori. Sci Rep. 2016 May 19:6:26114. doi: 10.1038/srep26114.

      • line 52-54. The numerous biological functions of insect coloration have been thoroughly investigated. It is reasonable to expect more references for each function.

      Response: Thank you very much for your careful work. We have made the appropriate modifications.

      Sword GA, Simpson SJ, El Hadi OT, Wilps H. Density-dependent aposematism in the desert locust. Proc Biol Sci. 2000;267(1438):63-68. … Behavior.

      Barnes AI, Siva-Jothy MT. Density-dependent prophylaxis in the mealworm beetle Tenebrio molitor L. (Coleoptera: Tenebrionidae): cuticular melanization is an indicator of investment in immunity. Proc Biol Sci. 2000;267(1439):177-182. … Immunity.

      N. F. Hadley, A. Savill, T. D. Schultz, Coloration and Its Thermal Consequences in the New-Zealand Tiger Beetle Neocicindela-Perhispida. J Therm Biol. 1992;17, 55-61…. Thermoregulation.

      Y. G. Hu, Y. H. Shen, Z. Zhang, G. Q. Shi, Melanin and urate act to prevent ultraviolet damage in the integument of the silkworm, Bombyx mori. Arch Insect Biochem. 2013; 83, 41-55…. UV protection.

      M. Stevens, G. D. Ruxton, Linking the evolution and form of warning coloration in nature. P Roy Soc B-Biol Sci. 2012; 279, 417-426…. Aposematism.

      K. K. Dasmahapatra et al., Butterfly genome reveals promiscuous exchange of mimicry adaptations among species. Nature.2012; 487, 94-98…. Mimicry.

      Gaitonde N, Joshi J, Kunte K. Evolution of ontogenic change in color defenses of swallowtail butterflies. Ecol Evol. 2018;8(19):9751-9763. Published 2018 Sep 3. …Crypsis.

      B. S. Tullberg, S. Merilaita, C. Wiklund, Aposematism and crypsis combined as a result of distance dependence: functional versatility of the colour pattern in the swallowtail butterfly larva. P Roy Soc B-Biol Sci.2005; 272, 1315-1321…. Aposematism and crypsis combined.

      • line 59-60. This general statement needs to be rephrased. I suggest remaining simple by indicating that insect coloration can be pigmentary, structural, or bioluminescent. About the structural coloration and associated nanostructures, the authors could cite recent reviews, such as: Seago et al., Interface 2009 + Lloyd and Nadeau, Current Opinion in Genetics & Development 2021 + "Light as matter: natural structural colour in art" by Finet C. 2023. I suggest doing the same for recent reviews that cover pigmentary and bioluminescent coloration in insects. The very recent paper by Nishida et al. in Cell Reports 2023 on butterfly wing color made of pigmented liquid is also unique and worth to consider.

      Response: Thank you very much for your careful work. We have made the appropriate modifications.

      Insect coloration can be pigmentary, structural, or bioluminescent. Pigments are mainly synthesized by the insects themselves and form solid particles that are deposited in the cuticle of the body surface and the scales of the wings (10, 11). Interestingly, recent studies have found that bile pigments and carotenoid pigments synthesized through biological synthesis are incorporated into body fluids and passed through the wing membranes of two butterflies (Siproeta stelenes and Philaethria diatonica) via hemolymph circulation, providing color in the form of liquid pigments (12). The pigments form colors by selective absorption and/or scattering of light depending on their physical properties (13). However, structural color refers to colors, such as metallic colors and iridescence, generated by optical interference and grating diffraction of the microstructure/nanostructure of the body surface or appendages (such as scales) (14, 15). Pigment color and structural color are widely distributed in insects and can only be observed by the naked eye in illuminated environments. However, some insects, such as fireflies, exhibit colors (green to orange) in the dark due to bioluminescence (16). Bioluminescence occurs when luciferase catalyzes the oxidation of small molecules of luciferin (17). In conclusion, the color patterns of insects have evolved to be highly sophisticated and are closely related to their living environments. For example, cryptic color can deceive animals via high similarity to the surrounding environment. However, the molecular mechanism by which insects form precise color patterns to match their living environment is still unknown.

      • RNAi approach. I have no doubt that obtaining phenocopies by electroporation might be difficult. However, I find the final sampling a bit limited to draw conclusions from the RT-PCR (n=5 and n=3 for phenocopies and controls). Three control individuals is a very low number. Moreover, it would nice to see the variability on the plot, using for example violin plots.

      Response: Thank you very much for your careful work. In the RNAi experiment, we injected more than 20 individuals in the experimental group and control group. We have added the RNAi data in Figure 4.

      Author response table 1.

      • Figure 6. Higher magnification images of Dazao and Bm-mamo knockout are needed, as shown in Figure 5 on RNAi.

      Response: Thank you very much for your careful work. We have added enlarged images.

      Author response image 3.

      • Phylogenetic analysis/Figure S6. I am not sure to what extent the sampling is biased or not, but if not, it is noteworthy that mamo does not show duplicated copies (negative selection?). It might be interesting to discuss this point in the manuscript.

      Response: Thank you very much for your careful work. mamo belongs to the BTB/POZ zinc finger family. The members of this family exhibit significant expansion in vertebrates. For example, there are 3 members in C. elegans, 13 in D. melanogaster, 16 in Bombyx mori, 58 in M. musculus and 63 in H. sapiens (Wu et al, 2019). These members contain conserved BTB/POZ domains but vary in number and amino acid residue compositions of the zinc finger motifs. Due to the zinc finger motifs that bind to different DNA recognition sequences, there may be differences in their downstream target genes. Therefore, when searching for orthologous genes from different species, we required high conservation of their zinc finger motif sequences. Due to these strict conditions, only one orthologous gene was found in these species.

      • Differentially-expressed genes and CP candidate genes (line 189-191). The manuscript would gain in clarity if the authors explain more in details their procedure. For instance, they moved from a list of 191 genes to CP genes only. Can they say a little bit more about the non-CP genes that are differentially expressed? Maybe quantify the number of CPs among the total number of differentially-expressed genes to show that CPs are the main class?

      Response: Thank you very much for your careful work. The nr (Nonredundant Protein Sequence Database) annotations for 191 differentially expressed genes in Supplemental Table S3 were added. Among them, there were 19 cuticular proteins, 17 antibacterial peptide genes, 6 transporter genes, 5 transcription factor genes, 5 cytochrome genes, 53 enzyme-encoding genes and others. Because CP genes were significantly enriched in differentially expressed genes (DEGs), previous studies have found that BmorCPH24 can affect pigmentation. Therefore, we first conducted an investigation into CP genes.

      • Interaction between Bm-mamo. It is not clear why the authors chose to investigate the physical interaction of Bm-mamo protein with the putative binding site of yellow, and not with the sites upstream of tan and DDC. Do the authors test one interaction and assume the conclusion stands for the y, tan and DDC?

      Response: Thank you very much for your careful work. In D. melanogaster, the yellow gene is the most studied pigment gene. The upstream and intron sequences of the yellow gene have been identified as containing multiple cis-regulatory elements. Due to the important pigmentation role of the yellow gene and its variable cis-regulatory sequence among different species, it has been considered a research model for cis-regulatory elements (Laurent Arnoult et al. 2013, Gizem Kalay et al. 2019, Yaqun Xin et al. 2020, Yann Le Poul et al. 2020). We use yellow as an example to illustrate the regulation of the mamo gene. We added this description to the discussion.

      Laurent Arnoult et al. Emergence and diversification of fly pigmentation through evolution of a gene regulatory module. Science. 2013 Mar 22;339(6126):1423-6. doi: 10.1126/science.1233749.

      Gizem Kalay et al. Redundant and Cryptic Enhancer Activities of the Drosophila yellow Gene. Genetics. 2019 May;212(1):343-360. doi: 10.1534/genetics.119.301985. Epub 2019 Mar 6.

      Yaqun Xin et al. Enhancer evolutionary co-option through shared chromatin accessibility input. Proc Natl Acad Sci U S A. 2020 Aug 25;117(34):20636-20644. doi: 10.1073/pnas.2004003117. Epub 2020 Aug 10.

      Yann Le Poul et al. Regulatory encoding of quantitative variation in spatial activity of a Drosophila enhancer. Sci Adv. 2020 Dec 2;6(49):eabe2955. doi: 10.1126/sciadv.abe2955. Print 2020 Dec.

      • Please note that some controls are missing for the EMSA experiments. For instance, the putative binding-sites should be mutated and it should be shown that the interaction is lost.

      Response: Thank you very much for your careful work. In this study, we found that the DNA recognition sequence of mamo is highly conserved across multiple species. In D. melanogaster, studies have found that mamo can directly bind to the intron of the vasa gene to activate its expression. The DNA recognition sequence they use is TGCGT (Shoichi Nakamura et al. 2019). We chose a longer sequence, GTGCGTGGC, to detect the binding of mamo. This binding mechanism is consistent across species.

      • Figure 7 and supplementary data. How did the name of CPs attributed? According to automatic genome annotation of Bm genes and proteins? Based on Drosophila genome and associated gene names? Did the authors perform phylogenetic analyses to name the different CP genes?

      Response: Thank you very much for your careful work. The naming of CPs is based on their conserved motif and their arrangement order on the chromosome. In previous reports, sequence identification and phylogenetic analysis of CPs have been carried out in silkworms (Zhengwen Yan et al. 2022, Ryo Futahashi et al. 2008). The members of the same family have sequence similarity between different species, and their functions may be similar. We have completed the names of these genes in the text, for example, changing CPR2 to BmorCPR2.

      Zhengwen Yan et al. A Blueprint of Microstructures and Stage-Specific Transcriptome Dynamics of Cuticle Formation in Bombyx mori. Int J Mol Sci. 2022 May 5;23(9):5155.

      Ningjia He et al. Proteomic analysis of cast cuticles from Anopheles gambiae by tandem mass spectrometry. Insect Biochem Mol Biol. 2007 Feb;37(2):135-46.

      Maria V Karouzou et al. Drosophila cuticular proteins with the R&R Consensus: annotation and classification with a new tool for discriminating RR-1 and RR-2 sequences. Insect Biochem Mol Biol. 2007 Aug;37(8):754-60.

      Ryo Futahashi et al. Genome-wide identification of cuticular protein genes in the silkworm, Bombyx mori. Insect Biochem Mol Biol. 2008 Dec;38(12):1138-46.

      • Discussion. I think the discussion would gain in being shorter and refocused on the understudied role of CPs. Another non-canonical aspect of the discussion is the reference to additional experiments (e.g., parthogenesis line 290-302, figure S14). This is not the place to introduce more results, and it breaks the flow of the discussion. I encourage the authors to reshuffle the discussion: 1) summary of their findings on mamo and CPs, 2) link between pigmentation mutant phenotypes, pigmentation pattern and CPs, 3) general discussion about the (evo-)devo importance of CPs and link between pigment deposition and coloration. Three important papers should be mentioned here:

      1) Matsuoka Y and A Monteiro (2018) Melanin pathway genes regulate color and morphology of butterfly wing scales. Cell Reports 24: 56-65... Yellow has a pleiotropic role in cuticle deposition and pigmentation.

      2) https://arxiv.org/abs/2305.16628... Link between nanoscale cuticle density and pigmentation

      3) https://www.cell.com/cell-reports/pdf/S2211-1247(23)00831-8.pdf... Variation in pigmentation and implication of endosomal maturation (gene red).

      Response: Thank you very much for your careful work. We have rewritten the discussion section.

      1) We have summarized our findings.

      Bm-mamo may affect the synthesis of melanin in epidermis cells by regulating yellow, DDC, and tan; regulate the maturation of melanin granules in epidermis cells through BmMFS; and affect the deposition of melanin granules in the cuticle by regulating CP genes, thereby comprehensively regulating the color pattern in caterpillars.

      2) We describe the relationship among the pigmentation mutation phenotype, pigmentation pattern, and CP.

      Previous studies have shown that the lack of expression of BmorCPH24, which encodes important components of the endocuticle, can lead to dramatic changes in body shape and a significant reduction in the pigmentation of caterpillars (53). We crossed Bo (BmorCPH24 null mutation) and bd to obtain F1(Bo/+Bo, bd/+), then self-crossed F1 and observed the phenotype of F2. The lunar spots and star spots decreased, and light-colored stripes appeared on the body segments, but the other areas still had significant melanin pigmentation in double mutation (Bo, bd) individuals (Fig. S13). However, in previous studies, introduction of Bo into L (ectopic expression of wnt1 results in lunar stripes generated on each body segment) (24) and U (overexpression of SoxD results in excessive melanin pigmentation of the epidermis) (58) strains by genetic crosses can remarkably reduce the pigmentation of L and U (53). Interestingly, there was a more significant decrease in pigmentation in the double mutants (Bo, L) and (Bo, U) than in (Bo, bd). This suggests that Bm-mamo has a stronger ability than wnt1 and SoxD to regulate pigmentation. On the one hand, mamo may be a stronger regulator of the melanin metabolic pathway, and on the other hand, mamo may regulate other CP genes to reduce the impact of BmorCPH24 deficiency.

      3) We discussed the importance of (evo-) devo in CPs and the relationship between pigment deposition and coloring.

      CP genes usually account for over 1% of the total genes in an insect genome and can be categorized into several families, including CPR, CPG, CPH, CPAP1, CPAP3, CPT, CPF and CPFL (68). The CPR family is the largest group of CPs, containing a chitin-binding domain called the Rebers and Riddiford motif (R&R) (69). The variation in the R&R consensus sequence allows subdivision into three subfamilies (RR-1, RR-2, and RR-3) (70). Among the 28 CPs, 11 RR-1 genes, 6 RR-2 genes, 4 hypothetical cuticular protein (CPH) genes, 3 glycine-rich cuticular protein (CPG) genes, 3 cuticular protein Tweedle motif (CPT) genes, and 1 CPFL (like the CPFs in a conserved C-terminal region) gene were identified. The RR-1 consensus among species is usually more variable than RR-2, which suggests that RR-1 may have a species-specific function. RR-2 often clustered into several branches, which may be due to gene duplication events in co-orthologous groups and may result in conserved functions between species (71). The classification of CPH is due to their lack of known motifs. In the epidermis of Lepidoptera, the CPH genes often have high expression levels. For example, BmorCPH24 had a highest expression level, in silkworm larvae epidermis (72). The CPG protein is rich in glycine. The CPH and CPG genes are less commonly found in insects outside the order Lepidoptera (73). This suggests that they may provide species specific functions for the Lepidoptera. CPT contains a Tweedle motif, and the TweedleD1 mutation has a dramatic effect on body shape in D. melanogaster (74). The CPFL members are relatively conserved in species and may be involved in the synthesis of larval cuticles (75). CPT and CPFL may have relatively conserved functions among insects. The CP genes are a group of rapidly evolving genes, and their copy numbers may undergo significant changes in different species. In addition, RNAi experiments on 135 CP genes in brown planthopper (Nilaparvata lugens) showed that deficiency of 32 CP genes leads to significant defective phenotypes, such as lethal, developmental retardation, etc. It is suggested that the 32 CP genes are indispensable, and other CP genes may have redundant and complementary functions (76). In previous studies, it was found that the construction of the larval cuticle of silkworms requires the precise expression of over two hundred CP genes (22). The production, interaction, and deposition of CPs and pigments are complex and precise processes, and our research shows that Bm-mamo plays an important regulatory role in this process in silkworm caterpillars. For further understanding of the role of CPs, future work should aim to identify the function of important cuticular protein genes and the deposition mechanism in the cuticle.

      Minor comments - Title. At this stage, there is no evidence that Bm-mamo regulates caterpillar pigmentation outside of Bombyx mori. I suggest to precise 'silkworm caterpillars' in the title.

      Response: Thank you very much for your careful work. We have modified the title.

      • Abstract, line 29. Because the knowledge on pigmentation pathway(s) is advanced, I would suggest writing 'color pattern is not fully understood' instead of 'color pattern is not clear'.

      Response: Thank you very much for your careful work. We have modified this sentence.

      • line 29. I suggest 'the transcription factor' rather than 'a transcription factor'.

      Response: Thank you very much for your careful work. We have modified this sentence.

      • line 30. If you want to mention the protein, the name 'Bm-mamo' should not be italicized.

      Response: Thank you very much for your careful work. We have modified this sentence.

      • line 30. 'in the silkworm'.

      Response: Thank you very much for your careful work. We have modified this sentence.

      • line 31. 'mamo' should not be italicized.

      Response: Thank you very much for your careful work. We have modified this sentence.

      • line 31. 'in Drosophila' rather 'of Drosophila'.

      Response: Thank you very much for your careful work. We have modified this sentence.

      • line 32. Bring detail if the gamete function is conserved in insects? In all animals?

      Response: Thank you very much for your careful work. The sentence was changed to “This gene has a conserved function in gamete production in Drosophila and silkworms and evolved a pleiotropic function in the regulation of color patterns in caterpillars.”

      • Introduction, line 51. I am not sure what the authors mean by 'under natural light'. Please rephrase.

      Response: Thank you very much for your careful work. We have deleted “under natural light”.

      • line 43. I find that the sentence 'In some studies, it has been proven that epidermal proteins can affect the body shape and appendage development of insects' is not necessary here. Furthermore, this sentence breaks the flow of the teaser.

      Response: Thank you very much for your careful work. We have deleted this sentence.

      • line 51-52. 'Greatly benefit them' should be rephrased in a more neutral way. For example, 'colours pattern have been shown to be involved in...'.

      Response: Thank you very much for your careful work. We have modified to “and the color patterns have been shown to be involved in…”

      • line 62. CPs are secreted by the epidermis, but I would say that CPs play their structural role in the cuticle, not directly in the epidermis. I suggest rephrasing this sentence and adding references.

      Response: Thank you very much for your careful work. We have modified “epidermis” to “cuticle”.

      • line 67. Please indicate that pathways have been identified/reported in Lepidoptera (11). Otherwise, the reader does not understand if you refer to previous biochemical in Drosophila for example.

      Response: Thank you very much for your careful work. We have modified this sentence. “Moreover, the biochemical metabolic pathways of pigments used for color patterning in Lepidoptera…have been reported.”

      • line 69. Missing examples of pleiotropic factors and associated references. For example, I suggest adding: engrailed (Dufour, Koshikawa and Finet, PNAS 2020) + antennapedia (Prakash et al., Cell Reports 2022) + optix (Reed et al., Science 2011), etc. Need to add references for clawless, abdominal-A.

      Response: Thank you very much for your careful work. We have made modifications.

      • line 76. The simpler term moth might be enough (instead of Lepidoptera).

      Response: Thank you very much for your careful work. We have modified this to “insect”.

      • line 96. I would simplify the text by writing "Then, quantitative RT-PCR was performed..."

      Response: Thank you very much for your careful work. We have modified this sentence.

      • line 112. 'Predict' instead of 'estimate'?

      Response: Thank you very much for your careful work. We have modified this sentence.

      • line 113. I would rather indicate the full name first, then indicate mamo between brackets.

      Response: Thank you very much for your careful work. We have modified this sentence.

      • line 144. The Perl script needs to be made accessible on public repository.

      Response: Thank you very much for your careful work.

      • line 147-150. Too many technical details here. The details are already indicated in the material and methods section. Furthermore, the details break the flow of the paragraph.

      Response: Thank you very much for your careful work. We have modified this section.

      • line 152. Needs to make the link with the observed phenotypes in Figure 1. Just needs to state that RNAi phenocopies mimic the mutant alleles.

      Response: Thank you very much for your careful work. We have modified this sentence.

      • line 153-157. Too many technical details here. The details are already indicated in the material and methods section. Furthermore, the details break the flow of the paragraph.

      Response: Thank you very much for your careful work. We have simplified this paragraph.

      • line 170. Please rephrase 'conserved in 30 species' because it might be understood as conserved in 30 species only, and not in other species.

      Response: Thank you very much for your careful work. We have modified this sentence.

      • line 182. Maybe explain the rationale behind restricting the analysis to +/- 2kb. Can you cite a paper that shows that most of binding sites are within 2kb from the start codon?

      Response: Thank you very much for your careful work. We have modified this sentence.

      • line 182. '14,623 predicted genes'.

      Response: Thank you very much for your careful work. We have modified this sentence.

      • line 183. '10,622 genes'

      Response: Thank you very much for your careful work. We have modified this sentence.

      • line 183. Redundancy. Please remove 'silkworm' or 'B. mori'.

      Response: Thank you very much for your careful work. We have modified this sentence.

      • line 187. '10,072 genes'

      Response: Thank you very much for your careful work. We have modified this sentence.

      • line 188. '9,853 genes'

      Response: Thank you very much for your careful work. We have modified this sentence.

      • line 200. "Therefore, the differential...in caterpillars" is a strong statement.

      Response: Thank you very much for your careful work. We have modified this sentence.

      • line 204. Remove "The" in front of eight key genes. Also, needs a reference... maybe a recent review on the biochemical pathway of melanin in insects.

      Response: Thank you very much for your careful work. We have modified this sentence.

      • line 220. This sentence is too general and vague. Please explicit what you mean by "in terms of evolution". Number of insect species? Diversity of niche occupancy? Morphological, physiological diversity?

      Response: Thank you very much for your careful work. We have modified this sentence.

      • line 285. The verb "believe" should be replaced by a more neutral one.

      Response: Thank you very much for your careful work. We have modified this sentence.

      • line 354-355. This sentence needs to be rephrased in a more objective way.

      Response: Thank you very much for your careful work. We have rewritten this sentence.

      • line 378. Missing reference for MUSCLE.

      Response: Thank you very much for your careful work. We have modified this sentence.

      • line 379. Pearson model?

      Response: Thank you very much for your careful work. We have modified this sentence.

      • line 408. "The CRISPRdirect online software was used...".

      Response: Thank you very much for your careful work. We have modified this sentence.

      • Figure 1. In the title, I suggest indicating Dazao, bd, bdf as it appears in the figure. Needs to precise 'silkworm larval development'.

      Response: Thank you very much for your careful work. We have modified this figure title.

      • Figure 3. In the title, is the word 'pattern' really necessary? In the legend, please indicate the meaning of the acronyms AMSG and PSG.

      Response: Thank you very much for your careful work. We have modified this figure legend.

      • Figure S7A. Typo 'Znic finger 1', 'Znic finger 2', 'Znic finger 3',

      Response: Thank you very much for your careful work. We have fixed these typos. .

    1. Author Response:

      Reviewer #1 (Public Review):

      Summary:

      The authors identified that genetically and pharmacological inhibition of CERS1, an enzyme implicated in ceramides biosynthesis worsen muscle fibrosis and inflammation during aging.<br /> Strengths:

      The study points out an interesting issue on excluding CERS1 inhibition as a therapeutic strategy for sarcopenia. Overall, the article it's well written and clear.<br /> Weaknesses:

      Many of the experiments confirmed previous published data, which also show a decline of CERS1 in ageing and the generation and characterization of a muscle specific knockout mouse line. The mechanistic insights of how the increased amount of long ceramides (cer c24) and the decreased of shorter ones (cer c18) might influence muscle mass, force production, fibrosis and inflammation in aged mice have not been addressed.

      We thank the reviewer for the assessment and would like to point out that Cers1 had not previously been studied in the context of aging. Moreover, our unbiased pathway analyses in human skeletal muscle implicate CERS1 for the first time with myogenic differentiation, which we validate in cell culture systems. To improve mechanistic insights, as suggested by Reviewer #1, we performed more experiments to gain insights how Cers1 derived c18, and Cers2 derived c24 ceramide species affect myogenesis. We recently showed that knocking out Cers2 reduces c24:0/c24:1 and promotes muscle cell maturation (PMID: 37118545, Fig. 6m-r and Supplementary Fig. 5e). This suggests that the very long chain ceramides c24 might indeed be driving the effect we see upon Cers1 inhibition because we observe an accumulation of c24 ceramides upon Cers1 (c18) inhibition (Fig 2B, Fig 3B, Fig 4A, Fig S3E), which is associated with impaired muscle maturation (Fig 4B-C, Fig S3G-I, Fig S4G-I). To study whether impaired muscle cell differentiation upon Cers1 inhibition is dependent on Cers2, we knocked-down Cers1 alone, or in combination with the knockdown of Cers2. Results show that reduced muscle cell maturation mediated by Cers1KD is rescued by the simultaneous knockdown of Cers2 as shown by gene expression analyses and immunohistochemical validation and quantification. Hence, we believe that reducing Cers1 function during aging might lead to an increase in sphingosine levels as has been shown previously (PMID: 31692231). Increased sphingosine triggers cell apoptosis due to its toxicity (PMID: 12531554). Therefore, channeling accumulating sphingosine towards C24 ceramides may avoid toxicity but, as we show in this manuscript, will reduce the myogenic potential in muscle. However, if also C24 production is blocked by Cers2 inhibition, sphingosine is forced towards the production of other, potentially less toxic or myogenesis-impairing ceramides. We added these new data to the revised manuscript as new Fig 5D-E and new Fig S5G-I.

      Reviewer #2 (Public Review):

      Summary:

      The manuscript by Wohlwend et al. investigates the implications of inhibiting ceramide synthase Cers1 on skeletal muscle function during aging. The authors propose a role for Cers1 in muscle myogenesis and aging sarcopenia. Both pharmacological and AAV-driven genetic inhibition of Cers1 in 18month-old mice lead to reduced C18 ceramides in skeletal muscle, exacerbating age-dependent features such as muscle atrophy, fibrosis, and center-nucleated fibers. Similarly, inhibition of the Cers1 orthologue in C. elegans reduces motility and causes alterations in muscle morphology.<br /> Strengths:

      The study is well-designed, carefully executed, and provides highly informative and novel findings that are relevant to the field.

      Weaknesses:

      The following points should be addressed to support the conclusions of the manuscript.

      (1) It would be essential to investigate whether P053 treatment of young mice induces age-dependent features besides muscle loss, such as muscle fibrosis or regeneration. This would help determine whether the exacerbation of age-dependent features solely depends on Cers1 inhibition or is associated with other factors related to age- dependent decline in cell function. Additionally, considering the reported role of Cers1 in whole-body adiposity, it is necessary to present data on mice body weight and fat mass in P053treated aged-mice.

      We thank the reviewer to suggest that we study Cers1 inhibition in young mice. In fact, a previous study shows that muscle-specific Cers1 knockout in young mice impairs muscle function (PMID: 31692231). Similar to our observation, these authors report reduced muscle fiber size and muscle force. Therefore, we do not believe that our observed effects of Cers1 inhibition in aged mice are specific to aging, although the phenotypic consequences are accentuated in aged mice. As requested by the reviewer, we attached the mice body weights and fat mass (Author response image 1A-B). The reduced fat mass upon P053 treatment is in line with previously reported reductions in fat mass in chow diet or high fat diet fed young mice upon Cers1 inhibition (PMID: 30605666, PMID: 30131496), again suggesting that the effect of Cers1 inhibition might not be specific to aging.

      Author response image 1.

      (A-B) Body mass (A) and Fat mass as % of body mass (B) were measured in 22mo C57BL/6J mice intraperitoneally injected with DMSO or P053 using EchoMRI (n=7-12 per group). (C-D) Grip strengh measurements in all limbs (C) or only the forelimbs (D) in 24mo C57BL/6J mice intramuscularly injected with AAV9 particles containing scramble, or shRNA targeting Cers1 (n=8 per group). (E-F) Pax7 gene expression in P053 or AAV9 treated mice (n=6-7 per group) (E), or in mouse C2C12 muscle progenitor cells treated with 25nM scramble or Cers1 targeting shRNA (n=8 per group) (F). (G) Proliferation as measured by luciferase intensity in mouse C2C12 muscle muscle cells treated with 25nM scramble or Cers1 targeting shRNA (n=24 per group). Each column represents one biological replicate. (H) Overlayed FACS traces of Annexin-V (BB515, left) and Propidium Iodide (Cy5, right) of mouse C2C12 muscle myotubes treated with 25nM scramble or Cers1 targeting shRNA (n=3 per group). Quantification right: early apoptosis (Annexin+-PI-), late apoptosis (Annexin+-PI+), necrosis (Annexin--PI+), viability (Annexin--PI-). (I) Normalized Cers2 gene expression in mouse C2C12 muscle muscle cells treated with 25nM scramble or Cers1 targeting shRNA (n=6-7 per group). (J-K) Representative mitochondrial respiration traces of digitonin-permeablized mouse C2C12 muscle muscle cells treated DMSO or P053 (J) with quantification of basal, ATP-linked, proton leak respiration as well as spare capacity and maximal capacity linked respiration (n=4 per group). (L) Reactive oxygen production in mitochondria of mouse C2C12 muscle muscle cells treated DMSO or P053. (M) Enriched gene sets related to autophagy and mitophagy in 24mo C57BL/6J mouse muscles intramuscularly injected with AAV9 particles containing scramble, or shRNA targeting Cers1 (left), or intraperitoneally injected with DMSO or P053 (right). Color gradient indicates normalized effect size. Dot size indicates statistical significance (n=6-8 per group). (N) Representative confocal Proteostat® stainings with quantifications of DMSO and P053 treated mouse muscle cells expressing APPSWE (top) and human primary myoblasts isolated from patients with inclusion body myositis (bottom). (O) Stillness duration during a 90 seconds interval in adult day 5 C. elegans treated with DMSO or 100uM P053. (P) Lifespan of C. elegans treated with DMSO or P053. (n=144-147 per group, for method details see main manuscript page 10).

      (2) As grip and exercise performance tests evaluate muscle function across several muscles, it is not evident how intramuscular AAV-mediated Cers1 inhibition solely in the gastrocnemius muscle can have a systemic effect or impact different muscles. This point requires clarification.

      The grip strength measurements presented in the manuscript come from hindlimb grip strength, as pointed out in the Methods section. We measured grip strength in all four limbs, as well as only fore- (Author response image 1C-D). While forelimb strength did not change, only hindlimb grip strength was significantly different in AAV-Cers1KD compared to the scramble control AAV (Fig 3I), which is in line with the fact that we only injected the AAV in the hindlimbs. This is similar to the effect we observed with our previous data where we saw altered muscle function upon IM AAV delivery in the gastrocnemius (PMID: PMID: 34878822, PMID: 37118545). The gastrocnemius likely has the largest contribution to hindlimb grip strength given its size, and possibly even overall grip strength as suggested by a trend of reduced grip strength in all four limbs (Author response image 1C). We also suspect that the hindlimb muscles have the largest contribution to uphill running as we could also see an effect on running performance. While we carefully injected a minimal amount of AAV into gastrocnemius to avoid leakage, we cannot completely rule out that some AAV might have spread to other muscles. We added this information to the discussion of the manuscript as a potential limitation of the study.

      (3) To further substantiate the role of Cers1 in myogenesis, it would be crucial to investigate the consequences of Cers1 inhibition under conditions of muscle damage, such as cardiotoxin treatment or eccentric exercise.<br /> While it would be interesting to study Cers1 in the context of muscle regeneration, and possibly mouse models of muscular dystrophy, we think such work would go beyond the scope of the current manuscript.

      (4) It would be informative to determine whether the muscle defects are primarily dependent on the reduction of C18-ceramides or the compensatory increase of C24-ceramides or C24-dihydroceramides.

      To improve mechanistic insights, as suggested by Reviewer #2, we performed more experiments to gain insights how Cers1 derived c18, and Cers2 derived c24 ceramide species affect myogenesis. We recently showed that knocking out Cers2 reduces c24:0/c24:1 and promotes muscle cell maturation (PMID: 37118545, Fig. 6m-r and Supplementary Fig. 5e). This suggests that the very long chain ceramides c24 might indeed be driving the effect we see upon Cers1 inhibition because we observe an accumulation of c24 ceramides upon Cers1 (c18) inhibition (Fig 2B, Fig 3B, Fig 4A, Fig S3E), which is associated with impaired muscle maturation (Fig 4B-C, Fig S3G-I, Fig S4G-I). To study whether impaired muscle cell differentiation upon Cers1 inhibition is dependent on Cers2, we knocked-down Cers1 alone, or in combination with the knockdown of Cers2. Results show that reduced muscle cell maturation mediated by Cers1KD is rescued by the simultaneous knockdown of Cers2 as shown by gene expression analyses and immunohistochemical validation and quantification. We added these data to the manuscript as new Fig 5D-E, new Fig S5G-I. These data, together with our previous results showing that Degs1 knockout reduces myogenesis (PMID: 37118545, Fig. 6s-x and Fig. 7) suggest that C24/dhC24 might contribute to the age-related impairments in myogenesis. We added the new results to the revised manuscript.

      (5) Previous studies from the research group (PMID 37118545) have shown that inhibiting the de novo sphingolipid pathway by blocking SPLC1-3 with myriocin counteracts muscle loss and that C18-ceramides increase during aging. In light of the current findings, certain issues need clarification and discussion. For instance, how would myriocin treatment, which reduces Cers1 activity because of the upstream inhibition of the pathway, have a positive effect on muscle? Additionally, it is essential to explain the association between the reduction of Cers1 gene expression with aging (Fig. 1B) and the age-dependent increase in C18-ceramides (PMID 37118545).

      Blocking the upstream enzyme of the ceramide pathway (SPT1) shuts down the entire pathway that is overactive in aging, and therefore seems beneficial for muscle aging. While most enzymes in the ceramide pathway that we studied so far (SPTLC1, CERS2) revealed muscle benefits in terms of myogenesis, inflammation (PMID: 35089797; PMID: 37118545) and muscle protein aggregation (PMID: 37196064), the CERS1 enzyme shows opposite effects. This is also visible in the direction of CERS1 expression compared to the other enzymes in one of our previous published studies (PMID: 37118545, Fig. 1e and Fig. 1f). In the current study, we show that Cers1 inhibition indeed exacerbates age-related myogenesis and inflammation as opposed to the inhibition of Sptlc1 or Cers2. As the reviewer points out, both C18- and C24-ceramides seem to accumulate upon muscle aging. We think this is due to an overall overactive ceramide biosynthesis pathway. Blocking C18-ceramides via Cers1 inhibition results in the accumulates C24-ceramides and worsens muscle phenotypes (see reply to question #4). On the other hand, blocking C24-ceramides via Cers2 inhibition improves muscle differentiation. These observations together with the finding that Cers1 mediated inhibition of muscle differentiation is dependent on proper Cers2 function (new Fig 5D-E, new Fig S5G-I) points towards C24-ceramides as the main culprit of reduced muscle differentiation. Hence, at least a significant part of the benefits of blocking SPTLC1 might have been related to reducing very long-chain ceramides. We believe that reduced Cers1 expression in skeletal muscle upon aging, observed by us and others (PMID: 31692231), might reflect a compensatory mechanism to make up for an overall overactive ceramide flux in aged muscles. Reducing Cers1 function during aging might lead to an increase in sphingosine levels as has been shown previously (PMID: 31692231). Increased sphingosine triggers cell apoptosis due to its toxicity (PMID: 12531554). Therefore, channeling accumulating sphingosine towards C24 ceramides may avoid toxicity but, as we show in this manuscript, will reduce the myogenic potential in muscle. However, if also C24 production is blocked by Cers2 inhibition (new Fig 5E-D, new Fig S5G-I), sphingosine is forced towards the production of other, potentially less toxic, or myogenesis-impairing ceramides. These data are now added to the revised manuscript (see page 7). Details were added to the discussion of the manuscript (see page 8).

      Addressing these points will strengthen the manuscript's conclusions and provide a more comprehensive understanding of the role of Cers1 in skeletal muscle function during aging.

      Reviewer #1 (Recommendations For The Authors):

      The authors identified that genetical and pharmacological inhibition of CERS1, an enzyme implicated in ceramides biosynthesis worsen muscle fibrosis and inflammation during aging.

      Even though many of the experiments only confirmed previous published data (ref 21, 11,37,38), which also show a decline of CERS1 in ageing and the generation and characterization of a muscle specific knockout mouse line, the study points out an interesting issue on excluding CERS1 inhibition as a therapeutic strategy for sarcopenia and opens new questions on understanding how inhibition of SPTLC1 (upstream CERS1) have beneficial effects in healthy aging (ref 15 published by the same authors).

      Overall, the article it's well written and clear. However, there is a major weakness. The mechanistic insights of how the increased amount of long ceramides (c24) and the decreased of shorter ones (cer c18) might influence muscle mass, force production, fibrosis and inflammation in aged mice have not been addressed. At the present stage the manuscript is descriptive and confirmatory of CERS1 mediated function in preserving muscle mass. The authors should consider the following points:

      Comments:

      (1) Muscle data

      (a) The effect of CERS1 inhibition on myotube formation must be better characterized. Which step of myogenesis is affected? Is stem cell renewal or MyoD replication/differentiation, or myoblast fusion or an increased cell death the major culprit of the small myotubes? Minor point: Figure S1C: show C14:00 level at 200 h; text of Fig S2A and 1F: MRF4 and Myogenin are not an early gene in myogenesis please correct, Fig S2B and 2C: changes in transcript does not mean changes in protein or myotube differentiation and therefore, authors must test myotube formation and myosin expression.

      Cers1 inhibition seems to affect differentiation and myoblast fusion. To test other suggested effects we performed more experiments as delineated. Inhibiting Cers1 systemically with the pharmacological inhibitor of Cers1 (P053) or with intramuscular delivery of AAV expressing a short hairpin RNA (shRNA) against Cers1 in mice did not affect Pax7 transcript levels (Author response image 1E). Moreover, we did also not observe an effect of shRNA targeting Cers1 on Pax7 levels in mouse C2C12 muscle progenitor cells (Author response image 1F). To characterize the effect of Cers1 inhibition on muscle progenitor proliferation/renewal, we used scramble shRNA, or shRNA targeting Cers1 in C2C12 muscle progenitors and measured proliferation using CellTiter-Glo (Promega). Results showed that Cers1KD had no significant effect on cell proliferation (Author response image 1G). Next, we assayed cell death in differentiating C2C12 myotubes deficient in Cers1 using FACS Analysis of Annexin V (left) and propidium iodide (right). We found no difference in early apoptosis, late apoptosis, necrosis, or muscle cell viability, suggesting that cell death can be ruled out to explain smaller myotubes (Author response image 1H). These findings support the notion that the inhibitory effect of Cers1 knockdown on muscle maturation are primarily based on effects on myogenesis rather than on apoptosis. Our data in the manuscript also suggests that Cers1 inhibition affects myoblast fusion, as shown by reduced myonucleation upon Cers1KD (Fig S3H right, Fig S5I).

      (b) The phenotype of CESR1 knockdown is milder than 0P53 treated mice (Fig S5D and Figure 3F, 3H are not significant) despite similar changes of Cer18:0, Cer24:0, Cer 24:1 concentration in muscles . Why?

      Increases in very long chain ceramides were in fact larger upon P053 administration compared to AAVmediated knockdown. For example, Cer24:0 levels increased by >50% upon P053 administration, compared to 20% by AAV injections. Moreover, dhC24:1 increased by 6.5-fold vs 2.5-fold upon P053 vs AAV treatment, respectively. These differences might not only explain the slightly attenuated phenotypes in the AA- treated mice but also underlines the notion that very long chain ceramides might cause muscle deterioration. We believe inhibiting the enzymatic activity of Cers1 (P053) as compared to degrading Cers1 transcripts is a more efficient strategy to reduce ceramide levels. However, we cannot completely rule out multi-organ, systemic effects of P053 treatment beyond its direct effect on muscle. We added these details in the discussion of the revised manuscript (see page 8 of the revised manuscript).

      (c) The authors talk about a possible compensation of CERS2 isoform but they never showed mRNA expression levels or CERS2 protein levels aner treatment. Is CERS2 higher expressed when CERS1 is downregulated in skeletal muscle?

      We appreciate the suggestion of the reviewer. We found no change in Cers2 mRNA levels upon Cers1 inhibition in mouse C2C12 myoblasts (Author response image 1I). We would like to point out that mRNA abundance might not be the optimal measurement for enzymes due to enzymatic activities. Therefore, we think metabolite levels are a better proxy of enzymatic activity. It should also be pointed out that “compensation” might not be an accurate description as sphingoid base substrate might simply be more available upon Cers1KD and hence, more substrate might be present for Cers2 to synthesize very long chain ceramides. This “re-routing” has been previously described in the literature and hypothesized to be related to avoid toxic (dh)sphingosine accumulation (PMID: 30131496). Therefore, we changed the wording in the revised manuscript to be more precise.

      (d) Force measurement of AAV CERS1 downregulated muscles could be a plus for the study (assay function of contractility)

      In the current study we measured grip strength in mice, which had previously been shown to be a good proxy of muscle strength and general health (PMID: 31631989). Indeed, our results of reduced muscle grip strength are in line with previous work that shows reduced contractility in muscles of Cers1 deficient mice (PMID: 31692231).

      (e) How are degradation pathways affected by the downregulation of CERS1. Is autophagy/mitophagy affected? How is mTOR and protein synthesis affected? There is a recent paper that showed that CerS1 silencing leads to a reduction in C18:0-Cer content, with a subsequent increase in the activity of the insulin pathway, and an improvement in skeletal muscle glucose uptake. Could be possible that CERS1 downregulation increases mTOR signalling and decreases autophagy pathway? Autophagic flux using colchicine in vivo would be useful to answer this hypothesis

      Cers1 in skeletal muscle has indeed been linked to metabolic homeostasis (see PMID: 30605666). In line with their finding in young mice we also find reduced fat mass upon P053 treatment in aged mice (Author response image 1A-B). We also looked into mitochondrial bioenergetics upon blocking Cers1 with P053 treatment using an O2k oxygraphy (Author response image 1J-L). Results show that Cers1 inhibition in mouse muscle cells increases mitochondrial respiration, similar to what has been shown before (PMID: 30131496). However, we also found that reactive oxygen species production in mouse muscle cells is increased upon P053 treatment, suggesting the presence of dysfunctional mitochondria upon inhibiting Cers1 with P053.We next looked into the mitophagy/autophagy degradation pathways suggested by the reviewer and do not find convincing evidence supporting that Cers1 has a major impact on autophagy or mitophagy derived gene sets in mice treated with shRNA against Cers1, or the Cers1 pharmacological inhibitor P053 (Author response image 1M).

      We then assessed the effect of Cers1 inhibition on transcripts levels related to the mTORC1/protein synthesis, as suggested by the reviewer. Cers1 knockdown in differentiating mouse muscle cells showed only a weak trend to reduce mTORC1 and its downstream targets (new Fig S4A). In line with this, there was no notable difference in protein synthesis in differentiating, Cers1 deficient mouse C2C12 myoblasts as assessed by L-homopropargylglycine (HPG) amino acid labeling using confocal microscopy (new Fig S4B) or FACS analyses (new Fig S4C). However, Cers1KD increased transcripts related to the myostatin-Foxo1 axis as well as the ubiquitin proteasome system (e.g. atrogin-1, MuRF1) (new Fig S4D), suggesting Cers1 inhibition increases protein degradation. We added these details to the revised manuscript on page 7. We recently implicated the ceramide pathway in regulating muscle protein homeostasis (PMID: 37196064). Therefore, we assessed the effect of Cers1 inhibition with the P053 pharmacological inhibitor on protein folding in muscle cells using the Proteostat dye that intercalates into the cross-beta spine of quaternary protein structures typically found in misfolded and aggregated proteins. Interestingly, inhibiting Cers1 further increased misfolded proteins in C2C12 mouse myoblasts expressing the Swedish mutation in APP and human myoblasts isolated from patients with inclusion body myositis (Author response imageure 1N). These findings suggest that deficient Cers1 might upregulate protein degradation to compensate for the accumulation of misfolded and aggregating proteins, which might contribute to impaired muscle function observed upon Cers1 knockdown. Further studies are needed to disentangle the underlying mechanstics.

      (f) The balances of ceramides have been found to play roles in mitophagy and fission with an impact on cell fate and metabolism. Did the authors check how are mitochondria morphology, mitophagy or how dynamics of mitochondria are altered in CERS1 knockdown muscles? (fission and fusion). There is growing evidence relating mitochondrial dysfunction to the contribution of the development of fibrosis and inflammation.

      Previously, CERS1 has been studied in the context of metabolism and mitochondria (for reference, please see PMID: 26739815, PMID: 29415895, PMID: 30605666, PMID: 30131496). In summary, these studies demonstrate that C18 ceramide levels are inversely related to insulin sensitivity in muscle and mitochondria, and that Cers1 inhibition improves insulin-stimulated suppression of hepatic glucose production and reduced high-fat diet induced adiposity. Moreover, improved mitochondrial respiration, citrate synthase activity and increased energy expenditure were reported upon Cers1 inhibition. Lack of Cers1 specifically in skeletal muscle was also reported to improve systemic glucose homeostasis. While these studies agree on the effect of Cers1 inhibition on fat loss, results on glucose homeostasis and insulin sensitivity differ depending on whether a pharmacologic or a genetic approach was used to inhibit Cers1. The current manuscript describes the effect of CERS1 on muscle function and myogenesis because these were the most strongly correlated pathways with CERS1 in human skeletal muscle (Fig 1C) and impact of Cers1 on these pathways is poorly studied, particularly in the context of aging. Therefore, we would like to refer to the mentioned studies investigating the effect of CERS1 on mitochondria and metabolism.

      (2) C.elegans data:

      (a) The authors checked maternal RNAi protocol to knockdown lagr-1 and showed alteration of muscle morphology at day 5. They also give pharmacological exposure of P053 drug at L4 stage. Furthermore, the authors also used a transgenic ortholog lagr-1 to perform the experiments. All of them were consistent showing a reduced movement. It would be important to show rescue of the muscle phenotype by overexpressing CERS1 ortholog in knockdown transgenic animals.

      We used RNAi to knockdown the Cers1 orthologue, lagr-1, in C.elegans. Therefore, we do not have transgenic animals. Overexpressing lagr-1 in the RNAi treated animals would also not be possible as the RNA from the overexpression would just get degraded.

      (b) The authors showed data about distance of C.elegans. It would be interesting to specify if body bends, reversals and stillness are affected in RNAi and transgenic Knockdown worms.

      As suggested, we measured trashing and stillness as suggested by the reviewer and found reduced trashing (new Fig S5B) and a trend towards an increase in stillness (Author response image 1O) in P053 treated worms on day 5 of adulthood, which is the day we observed significant differences in muscle morphology and movement (Fig 4D-E, Fig S5A). These data are now included in the revised manuscript.

      (c) Is there an effect on lifespan extension by knocking down CERS1?

      We performed two independent lifespan experiments in C.elegans treated with the Cers1 inhibitor P053 and found reduced lifespan in both replicate experiments (for second replicate, see Author response image 1P). We added these data to the revised manuscript as new Fig 4H.

      How do the authors explain the beneficial effect of sptlc1 inhibition on healthy aging muscle? Discuss more during the article if there is no possible explanation at the moment.

      We believe that blocking the upstream enzyme of the ceramide pathway (SPT1) shuts down the entire pathway that is overactive in aging, and therefore is more beneficial for muscle aging. Our current work suggests that at least a significant part of Sptlc1-KD benefits might stem from blocking very long chain ceramides. While SPTLC1 and CERS2 revealed muscle benefits in terms of myogenesis, inflammation (PMID: 35089797; PMID: 37118545) and muscle protein aggregation (PMID: 37196064), the CERS1 enzyme shows opposite effects, which is also visible in Fig 1e and Fig 1f of PMID: 37118545. In the current study, we show that Cers1 inhibition indeed exacerbates aging defects in myogenesis and inflammation as opposed to the inhibition of Sptlc1 or Cers2. The fact that the effect of Cers1 on inhibiting muscle differentiation is dependent on the clearance of Cers2-derived C24-ceramides suggests that reducing very long chain ceramides might be crucial for healthy muscle aging. We added details to the discussion.

    1. Author response:

      The following is the authors’ response to the original reviews

      Public Reviews:

      Reviewer #1 (Public Review):

      Summary:

      This study aims to understand the malaria antigen-specific cTfh profile of children and adults living in a malaria holoendemic area. PBMC samples from children and adults were unstimulated or stimulated with PfSEA-1A or PfGARP in vitro for 6h and analysed by a cTfh-focused panel. Unsupervised clustering and analysis on cTfh were performed.

      The main conclusions are:

      (1) the cohort of children has more diverse (cTfh1/2/17) recall responses compared to the cohort of adults (mainly cTfh17) and

      (2) Pf-GARP stimulates better cTfh17 responses in adults, thus a promising vaccine candidate.

      Strengths:

      This study is in general well-designed and with excellent data analysis. The use of unsupervised clustering is a nice attempt to understand the heterogeneity of cTfh cells. Figure 9 is a beautiful summary of the findings.

      Weaknesses:

      (1) Most of my concerns are related to using PfSEA-1A and PfGARP to analyse cTfh in vitro stimulation response. In vitro, stimulation on cTfh cells has been frequently used (e.g. Dan et al, PMID: 27342848), usually by antigen stimulation for 9h and analysed CD69/CD40L expression, or 18h and CD25/OX40. However, the authors use a different strategy that has not been validated to analyse in vitro stimulated cTfh. Also, they excluded CD25+ cells which might be activated cTfh. I am concerned about whether the conclusions based on these results are reliable.

      It has been shown that cTfh cells can hardly produce cytokines by Dan et al. However, in this paper, the authors report the significant secretion of IL-4 and IFNg on some cTfh clusters after 6h stimulation. If the stimulation is antigen-specific through TCR, why cTfh1 cells upregulate IL-4 but not IFNg in Figure 6? I believe including the representative FACS plots of IL-4, IFNg, IL21 staining, and using %positive rather than MFI can make the conclusion more convincing. Similarly, the author should validate whether TCR stimulation under their system for 6h can induce robust BCL6/cMAF expression in cTfh cells. Moreover, there is no CD40L expression. Does this mean TCR stimulation mediated BCl6/cMAF upregulation and cytokine secretion precede CD40L expression?

      In summary, I am particularly concerned about the method used to analyse PfSEA-1A and PfGARP-specific cTfh responses because it lacks proper validation. I am unsure if the conclusions related to PfSEA-1A/PfGARP-specific responses are reliable.

      An unfortunate reality of these types of complex immunologic studies is that it takes time to optimize a multiparameter flow cytometry panel, run this number of samples, and then conduct the analysis (not to mention the time it takes for a manuscript to be accepted for peer-review). An unexpected delay, frankly, was the COVID-19 pandemic when non-essential research lab activities were put on hold. We designed our panel in 2019 and referred to the “T Follicular Helper Cells” Methods and Protocols book from Springer 2015. Obviously the field of human immunology took a huge leap forward during the pandemic as we sought to characterize components of protective immunity, and as a result there are several new markers we will choose for future studies of Tfh subsets. We agree with the reviewer that cytokine expression kinetics differ depending on the in vitro stimulation conditions. Due to small blood volumes obtained from healthy children, we were limited in the number of timepoints we could test. However, since we were most interested in IL21 expression, we found 6 hrs to be the best in combination with the other markers of interest during our optimization experiments. We did find IFNg expression from non-Tfh cells, therefore we believe our stimulation conditions worked.

      Dan et al used stimulated tonsils cells to assess the CXCR5<sup>pos</sup>PD1<sup>pos</sup>CD45RA<sup>neg</sup> Tfh and CXCR5<sup>neg</sup> CD45RA<sup>neg</sup> non-Tfh whereas in our study, we evaluated CXCR5<sup>pos</sup>PD1<sup>pos</sup>CD45RA<sup>neg</sup> Tfh from PBMCs. Dan et al PBMCs’ work used EBV/CMV or other pathogen product stimuli and only gated on CD25<sup>pos</sup>OX40<sup>pos</sup> cells which are not the cells we are assessing in our study. This might explain in part the differences in cytokine kinetics, as we evaluated CD25<sup>neg</sup> PBMCs only. However, we agree that more recent studies focused on CXCR5<sup>pos</sup>PD1<sup>pos</sup> cells included more Activation-induced marker (AIM) markers, which are missing in our study, inducing a lack of depth in our analysis.

      Percentage of positive cells and MFI are complementary data. Indeed, the percentage of positive cells only indicates which cells express the marker of interest without giving a quantitative value of this expression. MFI indicates how much the marker of interest is expressed by cells which is important as it can indicate degree of activation or exhaustion per cell. Meta-cluster analysis is not ideal to assess the percentage of positivity whereas it does provide essential information regarding the intensity of expression. We added supplemental figures 14 (Bcl6 and cMAF), 15 (INFg and IL21) and 16 (IL4 and IL21) where percentage of positive cells were manually gated directly from the total CXCR5<sup>pos</sup>CD4<sup>pos</sup>CD45RA<sup>neg</sup>CD25<sup>neg</sup> TfH based on the FMO or negative control, and we overlaid the positive cells on the UMAP of all the CXCR5<sup>pos</sup>CD4<sup>pos</sup>CD45RA<sup>neg</sup>CD25<sup>neg</sup> meta-clusters. Results from the manual gating are consistent with the results we show using clustering. However, it helps to better visualize that antigen-specific IL21 expression was statistically significant in children whereas the high background observed for adults did not reveal higher expression after stimulation, perhaps suggesting an upper threshold of cytokine expression (supplemental figure 15). The following sentence has been added in the methods at the end of the “OMIQ analysis” section: “ However, the percentage of positive IFN𝛾, IL-4, IL-21, Bcl6, or cMAF using manual gating can be found in Supplemental Figures 14, 15, and 16 along with the overlay of the gated positive cells on the CD4<sup>pos</sup>CXCR5<sup>pos</sup>CD25<sup>neg</sup> UMAP and the cytoplots of the gated positive cells for each meta-cluster (Supplemental Figures 14, 15, and 16).”

      Indeed cMAF can be induced by TCR signaling, ICOS and IL6 (Imbratta et. al, 2020). However, in our study populations, ICOS was expressed (see Author response image 1, panel A) in absence of any stimulation suggesting that CXCR5<sup>pos</sup>CD4<sup>pos</sup>CD25<sup>neg</sup>CD45RA<sup>neg</sup> cells were already capable of expressing cMAF. Indeed, after gating Bcl6 and cMAF positive cells based on their FMOs (Author response image 1, panel B and C, respectively), we overlaid positive cells on the CXCR5<sup>pos</sup>CD4<sup>pos</sup>CD25<sup>neg</sup>CD45RA<sup>neg</sup> cells UMAP and we can see that most of our cells already express cMAF alone (Author response image 1, panel D), co-express cMAF and Bcl6 (Author response image 1, panel E), confirming that they are TfH cells, whereas very few cells only expressed Bcl6 alone (Author response image 1, panel F). Because we knew that cT<sub>FH</sub> already expresses Bcl6 and cMAF, we focused our analysis on the intensity of their expression to assess if our vaccine candidates were inducing more expression of these transcription factors.

      Author response image 1.

      (2) The section between lines 246-269 is confusing. Line 249, comparing the abundance after antigen stimulation is improper because 6h stimulation (under Golgi stop) should not induce cell division. I think the major conclusions are contained in Figure 5e, that (A) antigen stimulation will not alter cell number in each cluster and (B) children have more MC03, 06 and fewer MC02, etc.). The authors should consider removing statements between lines 255-259 because the trends are the same regardless of stimulations.

      We agree, there is no cell division after 6h and that different meta clusters did not proliferate after this short of in vitro stimulation. The use of the word ‘abundance’ in the context of cluster analysis is in reference to comparing the contribution of events by each group to the concatenated data. After the meta clusters are defined and then deconvoluted by study group, certain meta clusters could be more abundant in one group compared to another - meaning they contributed more events to a particular metacluster.

      Dimensionality reduction is more nuanced than manual gating and reveals a continuum of marker expression between the cell subsets, as there is no hard “straight line” threshold, as observed when using in 2D gating. Because of this, differences are revealed in marker expression levels after stimulation making them shift from one cluster to another - thereby changing their abundance.

      To clarify how this type of analysis is interpreted, we have modified lines 255-259 as follows:

      “In contrast, the quiescent PfSEA-1A- and PfGARP-specific cT<sub>FH</sub>2-like cluster (MC02) was significantly more abundant in adults compared to children (Figure 5c and 5d, pf<0.05). Interestingly, following PfGARP stimulation, the activated cT<sub>FH</sub>1/17-like subset (MC09) became more abundant in children compared to adults (Figure 5d, pf<0.05 with a False Discovery Rate=0.08), but no additional subsets shifted phenotype after PfSEA-1A stimulation (Figure 5c).”

      Reviewer #2 (Public Review):

      Summary:

      Forconi et al explore the heterogeneity of circulating Tfh cell responses in children and adults from malaria-endemic Kenya, and further compare such differences following stimulation with two malaria antigens. In particular, the authors also raised an important consideration for the study of Tfh cells in general, which is the hidden diversity that may exist within the current 'standard' gating strategies for these cells. The utility of multiparametric flow cytometry as well as unbiased clustering analysis provides a potentially potent methodology for exploring this hidden depth. However, the current state of analysis presented does not aid the understanding of this heterogeneity. This main goal of the study could hopefully be achieved by putting all the parameters used in one context, before dissecting such differences into their specific clinical contexts.

      Strengths:

      Understanding the full heterogeneity of Tfh cells in the context of infection is an important topic of interest to the community. The study included clinical groupings such as age group differences and differences in response to different malaria antigens to further highlight context-dependent heterogeneity, which offers new knowledge to the field. However, improvements in data analyses and presentation strategies should be made in order to fully utilize the potential of this study.

      Weaknesses:

      In general, most studies using multiparameter analysis coupled with an unbiased grouping/clustering approach aim to describe differences between all the parameters used for defining groupings, prior to exploring differences between these groupings in specific contexts. However, the authors have opted to separate these into sections using "subset chemokine markers", "surface activation markers" and then "cytokine responses", yet nuances within all three of these major groups were taken into account when defining the various Tfh identities. Thus, it would make sense to show how all of these parameters are associated with one another within one specific context to first logically establish to the readers how can we better define Tfh heterogeneity. When presented this way, some of the identities such as those that are less clear such as "MC03/MC04/ MC05/ MC08" may even be better revealed. once established, all of these clusters can then be subsequently explored in further detail to understand cluster-specific differences in children vs adults, and in the various stimulation conditions. Since the authors also showed that many of the activation markers were not significantly altered post-stimulation thus there is no real obstacle for merging the entire dataset for the first part of this study which is to define Tfh heterogeneity in an unbiased manner regardless of age groups or stimulation conditions. Other studies using similar approaches such as Mathew et al 2020 (doi: 10.1126/science.abc8) or Orecchioni et al 2017 (doi: 10.1038/s41467-017-01015-3) can be referred to for more effective data presentation strategies.

      Accordingly, the expression of cytokines and transcription factors can only be reliably detected following stimulation. However, the underlying background responses need to be taken into account for understanding "true" positive signals. The only raw data for this was shown in the form of a heatmap where no proper ordering was given to ensure that readers can easily interpret the expression of these markers following stimulation relative to no stimulation. Thus, it is difficult to reliably interpret any real differences reported without this. Finally, the authors report differences in either cluster abundance or cluster-specific cytokine/ transcription factor expression in Tfh cell subsets when comparing children vs adults, and between the two malaria antigens. The comparisons of cytokine/transcription factor between groups will be more clearly highlighted by appropriately combining groupings rather than keeping them separate as in Figures 6 and 7.

      Thank you for sharing these references. Similar to SPADE clustering and ViSNE dimensionality algorithms used in Orecchioni et al, we used all the extracellular markers from our panel in our FlowSOM algorithm with consensus meta-clustering which includes both the chemokine receptors and activation markers even though they are presented separately in our manuscript across the figure 3 and 4. This was explained in the methods section (lines 573 - 587). We then chose the UMAP algorithm as visual dimensionality reduction of the meta-clusters generated by FlowSOM-consensus meta-clustering as explained under the “OMIQ analysis” subpart of our methods (lines 588- 604). Therefore, we believe we have conducted the analysis as this reviewer suggests even if we chose to show the figures that were informative to our story. The heatmap of the results brings the possibility to see which combination of markers respond or not to the different conditions and between groups, all the raw data are present from the supplemental figures 10 to 13 showing, using bar plots, the differences expressed in the heatmaps. We believe it strengthens our interpretation of the results.

      Regarding the transcription factor and cytokine background, we added supplemental figures 14, 15 and 16 where we used manual gating to select Bcl6, cMAF, IFNg, IL21 or IL4 positive cells directly from total CXCR5<sup>pos</sup>CD4<sup>pos</sup>CD45RA<sup>neg</sup>CD25<sup>neg</sup> TfH cells based on the FMO or negative control, and we overlaid the positive cells on the UMAP of all the CXCR5<sup>pos</sup>CD4<sup>pos</sup>CD45RA<sup>neg</sup>CD25<sup>neg</sup> meta-clusters. Moreover, all the dot plots (with their statistics) used for the heatmap figure 6 and 7 can be found in the supplemental figures 10, 11, 12 and 13. These supplemental figures address the concerns above by showing the difference of signals between unstimulated and stimulated conditions.

      Reviewer #3 (Public Review):

      Summary:

      The goal of this study was to carry out an in-depth granular and unbiased phenotyping of peripheral blood circulating Tfh specific to two malaria vaccine candidates, PfSEA-1A and PfGARP, and correlate these with age (children vs adults) and protection from malaria (antibody titers against Plasmodium antigens.). The authors further attempted to identify any specific differences in the Tfh responses to these two distinct malaria antigens.

      Strengths:

      The authors had access to peripheral blood samples from children and adults living in a malaria-endemic region of Kenya. The authors studied these samples using in vitro restimulation in the presence of specific malaria antigens. The authors generated a very rich data set from these valuable samples using cutting-edge spectral flow cytometry and a 21-plex panel that included a variety of surface markers, cytokines, and transcription factors.

      Weaknesses:

      - Quantifying antigen-specific T cells by flow cytometry requires the use of either 1- tetramers or 2- in vitro restimulation with specific antigens followed by identification of TCR-activated cells based on de-novo expression of activation markers (e.g. intracellular cytokine staining and/or surface marker staining). Although authors use an in vitro restimulation strategy, they do not focus their study on cells de-novo expressing activation markers as a result of restimulation; therefore, their study is not really on antigen-specific cTfh. Moreover, the authors report no changes in the expression of activation markers commonly used to identify antigen-specific T cells upon in vitro restimulation (including IFNg and CD40L); therefore, it is not clear if their in vitro restimulation with malaria antigens actually worked.

      We understand the reviewer’s point of view and apologies for any confusion. IFNg was expressed but not statistically different between groups. Indeed, looking at the CD8 T cells and using manual gating, we were able to show that IFNg was increased but not statistically significant upon stimulation from CD4<sup>pos</sup>CXCR5<sup>pos</sup> cells (supplemental figure 15, panel C), confirming our primary observation using clustering analysis. These results showed that our malaria antigen induced IFNg response in some participants, but not all of them, revealing heterogeneity in this response among individuals within the same group.

      Regarding CD40L, in the supplemental figure 7, we can see that some of our meta-clusters expressed more CD40L upon stimulation, but again without leading to statistical differences between groups. Combined with the increased expression of other cytokines and transcription factors, we showed that our stimulation did indeed work. However, because of the high variation within groups, there were no statistical differences across our groups. Because CD40L is not the only marker showing specific T cell activation, and not all T cells respond using this marker alone, a more comprehensive multimarker AIM panel might have highlighted differences between groups. We recognized the limitations of our study and believe that future study will benefit from more activation markers commonly used to identify antigone-specific T cells such as CD69, OX40, 4-1BB (AIM panel), among other markers.

      - CXCR5+CD4+ memory T cells have been shown to present multi-potency and plasticity, capable of differentiating to non-Tfh subsets upon re-challenge. Although authors included in their flow panel a good number of markers commonly used in combination to identify Tfh (CXCR5, PD-1, ICOS, Bcl-6, IL-21), they only used one single marker (CXCR5) as their basis to define Tfh, thus providing a weak definition for Tfh cells and follow up downstream analysis.

      Sorry for the confusion, even though the subsampled on the CD4<sup>pos</sup>CXCR5<sup>pos</sup> CD25<sup>neg</sup> cells to run our FlowSOM, we showed the different levels of expression across meta-clusters (figure 4 panels A and B) of PD1 (Tfh being PD1 positive cells) and ICOS (indicating the activation stage of the Tfh, “T Follicular Helper Cells” Methods and Protocols book from Springer 2015). We also included an overlay of the manually gated double positive Bcl6-cMAF cells on the CXCR5<sup>pos</sup>CD45RA<sup>neg</sup>CD25<sup>neg</sup> CD4 T cell UMAP plot to show that most of them express Bcl6 (supplemental figure 14). Interestingly, the manually gated IL21 positive cells were less abundant, particularly for children (supplemental figure 15). Because we were not able to include all the markers that are now used to define Tfh cells, we referred to our cell subsets as “TFH-like”. This is an acknowledged limitation of our study. Due to the limited blood volume obtained from children and cost of running multiplex flow cytometry assays, our results showing antigen-specific heterogeneity of Tfh subset will have to be validated in future studies that include these additional defining markers.

      - Previous works have used FACS-sorting and in vitro assays for cytokine production and B cell help to study the functional capacity of different cTfh subsets in blood from Plasmodium-infected individuals. In this study, authors do not carry out any such assays to isolate and evaluate the functional capacity of the different Tfh subsets identified. Thus, all the suggestions for the role that these different cTfh subsets may have in vivo in the context of malaria remain highly hypothetical.

      Unfortunately, low blood volumes obtained from children prevented us from running in vitro functional assays and the study design did not allow us to correlate them with protection. However, since the function of identified Tfh subsets from malaria-exposed individuals has been evaluated using Pf lysates in other studies, we referenced them when interpreting the differences we reported in Tfh subset recognition between malaria antigens. If either of these antigens move forward into vaccine trials, then evaluating their function would be important.

      - The authors have not included malaria unexposed control groups in their study, and experimental groups are relatively small (n=13).

      This study design did not include the recruitment of malaria naive negative controls as its goal was to assess malaria antigen-specific responses comparing the quality and abundance between malaria-exposed children to adults to these potential new vaccine targets PfSEA-1A and PfGARP. We did however test 3 malaria-naive adults and found no non-specific activation after stimulation with these two malaria antigens. Since this was done as part of our assay optimization, we did not feel the need to show these negative findings.

      And even with our small sample size, we demonstrated significant age-associated differences in malaria antigen-specific responses from cT<sub>FH</sub>-like subsets.

      Recommendations for the authors:

      Reviewer #1 (Recommendations For The Authors):

      Minor points are:

      (1) Line 88, cTfh cells are not only from GC-Tfh, they have GC-independent origin (He et al, PMID: 24138884).

      The following sentence was added line 88 “Interestingly, cT<sub>FH</sub> cells can also come from peripheral cT<sub>FH</sub> precursor CCR7<sup>low</sup>PD1<sup>high</sup>CXCR5<sup>pos</sup> cells; thus, they also have a GC-independent origin (He, Cell, 2013 PMID: 24138884).

      (2) I believe all participants were free of blood-stage infection upon enrolment. But can authors clearly state this information between lines 151-159?

      We mentioned in the methods, line 495-496 “Participants were eligible if they were healthy and not experiencing any symptoms of malaria at the time venous blood was collected”. However, using qPCR we found 5 children with malaria blood stage. As shown in Author response image 2, comparing malaria free to blood-stage children, no differences were observed without any stimulation. However, MC03 is more abundant upon malaria antigen stimulation in the blood-stage group whereas MC04 is more abundant in the malaria free group upon PfGARP stimulation only confirming that our stimulation worked.

      Author response image 2.

      Reviewer #3 (Recommendations For The Authors):

      (1) The strategy for gating on antigen-specific cTfh cells needs to be revised. The correct approach would be to gate on those cells that respond by de-novo expression of activation markers upon antigen restimulation (also termed activation-induced markers. e.g. CD69, CD40L, CXCL13 and IL-21, Niessl 2020; CD69, CD40L, CD137 and OX40, Lemieux 2023; CD137 and OX40, Grifoni 2020). As it stands, the study is not really on antigen-specific T cells, but rather on the overall CD4 T cell compartment plus or minus antigenic stimulation.

      We recognized the limitation in our flow panel design which prevents us from performing this gating. We originally based our panel design on the “T follicular helper cells methods and protocols” book (Springer 2015) which used CD45RA, CD25, CXCR5, CCR6, CXCR3, CCR7, ICOS and PD1 to define cT<sub>FH</sub>. We had already optimized our 21-color panel, purchased reagents and started to run our experiments by the time these publications modified how to define TFH cells Niessl, Lemieux and Grifoni’s publication. Indeed we optimized and performed our assay from November 2019 to March 2020, finishing to run the samples during the first quarantine. Because of the urgent needs of research on SARS-CoV-2 that we were involved with from this time and moving forward, the analysis of our TFH work got highly postponed. Moreover, 2020 is also the year where many TFH papers came out with better ways to define cT<sub>FH</sub> and responses to antigen stimulations. In our future studies, our panel will include AIM.

      (2) It is not clear if the antigenic stimulation actually worked. Does the proportion of IFNg+ or IL-4+ or IL-21+ or CD40L+ or CD25+ CD4 or CD8 T cells increase following in vitro antigen restimulation?

      Yes, using manual gating, we are able to show an increase of IL4 (supplemental figure 16 panel B and C), and IL21 (supplemental figure 15 panel J and K) production in both children and adults. However, we did not observe significant production of IFNg (supplemental figure 15, panel C) and changes in CD40L expression (supplemental figure 7) after malaria antigen stimulation, however, our positive control SEB worked. So, yes our stimulation assay worked but these 2 malaria antigens did not significantly induce these cytokines. This could be that they are too low to detect in every participant since they are single antigens and not whole parasite lysates, as other studies have used. It could also be that these antigens don’t stimulate CD40L or IFNg in all our participants. We brought up this limitation as follow in the discussion, line 473: “Although the heterogeneity in the response of CD40L and IFNγ suggests that our tested malaria antigens did not induce significant differences in the expression of these markers in all our participants, our panel did not include other activated induced markers, such as OX40, 4-1BB, and CD69”.

      (3) It is not clear what is the proportion of cTfh over the total CD4 T cell compartment among the different groups. Does this vary among different groups? It would be valuable to display this as an old-fashioned combination of contour plots with outliers for illustrating flow cytometry and bar graphs for the cumulative data.

      The proportion of CD3<sup>pos</sup>CD4<sup>pos</sup>CD25<sup>neg</sup>CXCR5<sup>pos</sup> cTfh cells did not differ within the total number of CD4 T cells between groups (figure 2).

      (4) The gating strategy could be refined and become more robust if adding additional markers in combination with CXCR5 for identifying cTfh (e.g. CXCR5+Bcl6+).

      Thank you for this suggestion. An overlay of Bcl6 expression can be found in supplemental figure 14 where we confirm that our CXCR5+ cT<sub>FH</sub>-like subsets express cMAF and Bcl6.

      (5) The protocols for intracellular and intranuclear staining seem to be incomplete in Materials and Methods. In particular, cell permeabilization strategies seem to be missing.

      Our apologies for this oversight, we added the following sentences in the methods line 545: “Cells were fixed and permeabilized for 45 mins using the transcription factor buffer set (BD Pharmingen) followed by a wash with the perm-wash buffer. Intracellular staining was performed at 4 °C for 45 more mins followed by two washes using the kit’s perm-wash buffer”.

      (6) In Materials and Methods, the authors mention they have used fluorescence minus one control to set their gating strategy. It would be valuable to show these, either on the main body or as part of supplementary figures.

      We added the cytoplots of the FMOs and/or negative controls as appropriate in the supplemental figures 14 (cMAF and Bcl6), 15 (IFNg and IL21) and 16 (IL4 and IL21).

      (7) Line 194 and Figure 3, it is not clear the criteria that the authors used for down-sampling events before FlowSOM analysis. Was this random? Was this done with unstimulated or stimulated samples?

      We chose to down-sample on CD3posCD4<sup>pos</sup>CD25<sup>neg</sup>CD45RA<sup>neg</sup> and CXCR5<sup>pos</sup> cells prior to our FlowSOM to allow more cluster analysis to focus only on the differences among those cells. The down-sampling used 1,000 CD3posCD4<sup>pos</sup>CD25<sup>neg</sup> CD45RA<sup>neg</sup>CXCR5<sup>pos</sup> cells from each fcs file (unstimulated and stimulated samples). If the fcs file had more than 1,000 CXCR5<sup>pos</sup> cells, the down-sampling was done randomly by the OMIQ platform algorithm to select only 1,000 CXCR5<sup>pos</sup> cells within this specific fcs file. The latest sentence was added to the methods line 593.

      (8) Lanes 201, 202, As it stands, the take of the authors on the role of different cTfh subsets during infection remains highly speculative. Are these differences in cTfh phenotypes actually reflected in their in vitro capacity to provide B cell help (e.g. as in the Obeng-Adjei 2015 paper) or to produce IL-21, express co-stimulatory molecules, or any other characteristic that would allow them to better infer their functional roles during infection? Any additional in vitro analysis of the functional capacity of isolated cTfh subsets identified in this research would greatly increase its value.

      We agree with the reviewer that this sentence is speculative, and we rephrase it as follow: “First, we found different CXCR5 expression levels between meta-clusters (Figure 3b); CXCR5 is essential for cT<sub>FH</sub> cells to migrate to the lymph nodes and interact with B-cells”. We would have liked to perform in vitro functional assays. However, as explained above, we did not have sufficient cells collected from children to do so.

      (9) It is not clear why authors omitted IL-17 and did not use IFNg and IL-4 to refine their definition of Th1, Th2 and Th17 cTfh.

      We would have liked to include IL-17, however we were constrained by only having access to a 4 lasers cytometer at the time we ran our assay. In light of needing to prioritize markers, when we were designing our flow panel, cTfh1 were shown to be preferentially activated during episodes of acute febrile malaria children (Obeng-Adjei). Therefore, we chose to focus on IFNg and IL4 to differentiate Tfh1 from Tfh2, in addition to other markers as surrogate of functional potential. We did not use IFNg and IL4 to refine our definition of Tfh1, Tfh2 and Tfh17 as recent publications have shown that IL4 is not only expressed in Tfh2 but also in the other Tfh subsets, at lower intensity (Gowthaman among others). Therefore IFNg and IL4 by themselves were not sufficient to properly define the different Tfh subsets. In future studies, we plan to include transcription factor profiles (T-bet, BATF, GATA3) to further refine definitions of Tfh subsets.

      (10) Lines, 226, 228, based on the combination of markers that the MC03 subset expresses, it is tempting to think that this is the only "truly" committed Tfh subset from the entire analysis. Please, discuss.

      If the reviewer is referring to changes in marker expression levels that indicate they have not reached a level of differentiation that would make them reliable (ie “true) Tfh cells, we agree that this is an important question now that we have technology that can measure and analyse so many phenotypic markers at once. This brings forward the need for the scientific method - to replicate study findings to determine whether they are consistent given the same study design and experimental conditions.

      (11) Lines 243 244, Again, is this reflected in functional capacity?

      The study described in this manuscript did not include functional assays. However, this did not change the key finding that different malaria antigens behaved differently, demonstrating heterogeneity in Tfh recognition of malaria antigens. Regarding CD40L expression, we did not observe differences between groups, however some individuals had an increase of their CD40L (supplemental figure 7). It is possible that some individuals had responded through other activated induced markers (CD69, ICOS, OX40, 4-1BB among others) and that our stimulation condition was not long enough to assess CD40L expression upon malaria antigen stimulation. This limitation has been addressed by editing the line 243-244 as follows: “we were unable to find statistical differences in the CD40L expression between groups as only few individuals responded through it (supplemental figure 7).”

      (12) Lines 243, 244, Are these cTfh subsets exclusively detected in malaria-exposed individuals? This is confounded by the lack of a malaria unexposed control group in this study, which would have been highly valuable.

      We agree with the reviewer that having non-naive children would have been valuable as a negative control group. However, this study was conducted in Kenya where all children are suspected to have had at least one malaria infection. We also did not have ethical approval or the means to enroll children in the USA who would not have been exposed to malaria as a negative control group. Since we were also evaluating differences by age group, comparing US adults would not have helped to address this point. Therefore, this remains an open question that might be addressed by another study recruiting children in non-malaria endemic areas.

      (13) Line 267, as the authors have not gated on T cells de-novo expressing activation markers in response to antigen restimulation, how do they know these are indeed antigen-specific cTfh?

      Omiq analysis accounts for marker expression levels in the resting cells (unstimulated well) for each individual compared to each experimental/stimulated well. The algorithm computationally determines whether that expression level changed without an arbitrary positive threshold, keeping the expression levels as a continuous variable, not dichotomous - which is the power of unbiased cluster analyses. Therefore, we know that these cells are antigen-specific based on the statistical difference in intensity expression between the resting cells and the stimulated ones. Nevertheless, manual gating to show “de-novo” responding cells, produced the same results as assessing the MFI of each meta-cluster (supplemental figures 14, 15 and 16).

      (14) Lines, 292-295, it is very surprising that Tfh cells would not produce IL-21 upon restimulation. Have the authors observed upregulation of IL-21 following SEB restimulation?

      Yes, we observed IL21 positive cells upon SEB stimulation (supplemental figure 15, panel J and K). However we found unexpectedly high background levels of IL21, specifically within the adult group (supplemental figure 15, panel K and M) making it challenging to find antigen-specific increases above background. Interestingly, an increase in IL21 using manual gating was observed upon PfSEA-1A or PfGARP stimulation in children (supplemental figure 15, panel J and L).

      (15) In Figures 3 and 4, it is not clear if there are any significant differences in expression of different markers between different cTfh subsets and/or different conditions. Moreover, the lack of differences in response to antigen stimulation seems to suggest that it did not work adequately.

      We intentionally chose 6-hours stimulation to better assess changes in cytokines which we did. However, because it is a short stimulation, we did not expect dramatic changes in the extracellular markers presented in the figure 3 and 4. A longer stimulation, such as 24h, will highlight properly these changes.

      (16) Figure 5b would benefit from bar graphs.

      Please find below the bar-graphs for the highlighted meta-clusters in figure 5b. We did not include these bar-graphs to our figure 5 as they do not bring new information. They repeat the information already presented through the EdgeR plot.

      Author response image 3.

      (17) Figures 6 and 7 would greatly benefit from showing individual examples of old-fashioned contour with outliers flow plots to illustrate the different cTfh subsets identified in the study.

      The different cT<sub>FH</sub> subsets can be found with a contour plot with outliers in the supplemental figure 4.

      (18) Figures 3,4, 6, and 7, the authors exclusively focused on the study of MFI to measure the expression of cytokine and transcription factors among different groups/stimulations. Have the authors observed any differences in the percentage or absolute counts of cytokine+ and/or TF+ between different subsets of cTfh and/or different conditions?

      Yes. We added the supplemental figures 14 (transcription factors) and 15/16 (cytokines) where cytokines and transcription factors were assessed using manual gating. We found that total CD4<sup>pos</sup>CXCR5<sup>pos</sup> IL4 was significantly increased upon stimulation in both adults and children while IFNg was not. However, we found significantly higher IFNg on total CD8<sup>pos</sup> cells showing that the stimulation worked, but the total CD4<sup>pos</sup>CXCR5<sup>pos</sup> did not express IFNg. Finally, we observed a trend of higher IL21<sup>pos</sup>CD4<sup>pos</sup>CXCR5<sup>pos</sup> in adults, not significant due to high background whereas IL21 was significantly increased upon stimulation in children. Regarding cMAF and Bcl6, both transcription factors were significantly increased upon stimulation within children only.

      (19) Figure 8, the definition for high and low PfGARP antibody titers seems rather arbitrary. Are these associations still significant when attempting a regular correlation analysis between Ab values (i.e. Net MFI) and different cTfh subsets?

      Yes, the definition for high and low PfGARP antibody levels is arbitrary but when looking at the antibody data (figure 1b), it was naturally bimodal. Therefore as a sub-analysis, we assess the association between PfGARP antibodies levels and cT<sub>FH</sub> subsets, see Author response image 4. We checked the correlation between the abundance of the meta-clusters and the level of IgG anti-PfGARP and anti-PfSEA after PfGARP and PfSEA stimulation. We also checked the correlation between the MFI expression of Bcl6 and cMAF after stimulation (PfGARP or PfSEA-1A minus the unstimulated) by the meta-clusters and the level of IgG anti-PfGARP and anti-PfSEA. However, we believe that because of our small sample size, our results are not robust enough and that we risk over-interpreting the data. Therefore, we choose not to include this analysis in the manuscript.

      Author response image 4.

      (20) The comprehensive 21-plex panel that authors used in this study could generate insights on additional immune cells beyond cTfh (e.g. additional CD4 T cell subsets, CD8 T cells, CD19 B cells). It is not clear why the authors limited their analysis to cTfh only.

      The primary goal of the study was to assess the cT<sub>FH</sub> response to malaria vaccine candidates. However, we were able to assess the IFNg expression for CD8 T cells upon stimulation using the manual gating as indicated in the supplemental figure 15. Without additional markers to more clearly define other CD4 T cell or B cell subsets, we do not believe this dataset would go deep enough into characterizing antigen-specific responses to malaria antigens that would yield new insight.

      (21) Minor point, the punctuation should be revised throughout the manuscript.

      Punctuation was revised throughout the manuscript by our departmental scientific writer Dr. Trombly, as per reviewer request.

    1. Reviewer #2 (Public Review):

      Assessment

      This study develops a potentially useful metric for quantifying codon usage adaptation – the Codon Adaptation Index of Species (CAIS) – that is intended to allow for more direct comparisons of the strength of selection at the molecular level across species by controlling for interspecies variation in amino acid usage and GC content. As evidence to support there claim CAIS better controls for GC content and amino acid usage across species, they note that CAIS has only a weak positive correlation with GC% (that does not stand up to multiple hypothesis testing correction) while CAI has a clear negative correlation with GC%. Using CAIS, they find better adapted species have more disordered protein domains; however, excitement about these findings is dampened due to (1) this result is also observed using the effective number of codons (ENC) and

      (2) concerns over the interpretation of CAIS as a proxy for the effectiveness of selection.

      Public Review

      Summary

      The goal of the authors in this study is to develop a more reliable approach for quantifying codon usage such that it is more comparable across species. Specifically, the authors wish to estimate the degree of adaptive codon usage, which is potentially a general proxy for the strength of selection at the molecular level. To this end, the authors created the Codon Adaptation Index for Species (CAIS) that attempts to control for differences in amino acid usage and GC% across species. Using their new metric, the authors observe a positive relationship between CAIS and the overall “disorderedness” of a species protein domains. I think CAIS has the potential to be a valuable tool for those interested in comparing codon adaptation across species in certain situations. However, I have certain theoretical concerns about CAIS as a direct proxy for the efficiency of selection sNe when mutation bias changes across species.

      Strengths

      (1) I appreciate that the authors recognize the potential issues of comparing CAI when amino acid usage varies and correct for this in CAIS. I think this is sometimes an under-appreciated point in the codon usage literature, as CAI is a relative measure of codon usage bias (i.e. only considers synonyms). However, the strength of natural selection on codon usage can potentially vary across amino acids, such that comparing mean CAI between protein regions with different amino acid biases may result in spurious signals of statistical significance.

      (2) The CAIS metric presented here is generally applicable to any species that has an annotated genome with protein-coding sequences. A significant improvement over the previous version is the implementation of software tool for applying this method.

      (3) The authors do a better job of putting their results in the context of the underlying theory of CAIS compared to the previous version.

      (4) The paper is generally well-written.

      Weaknesses

      (1) The previously observed correlation between CAIS and body size was due to a bug when calculating phylogenetic independent contrasts. I commend the authors for acknowledging this mistake and updating the manuscript accordingly. I feel that the unobserved correlation between CAIS and body size should remain in the final version of the manuscript. Although it is disappointing that it is not statistically significant, the corrected results are consistent with previous findings (Kessler and Dean 2014).

      (2) I appreciate the authors for providing a more detailed explanation of the theoretical basis model. However, I remain skeptical that shifts in CAIS across species indicates shifts in the strength of selection. I am leaving the math from my previous review here for completeness.

      As in my previous review, let’s take a closer look at the ratio of observed codon frequencies vs. expected codon frequencies under mutation alone, which was previously notated as RSCUS in the original formulation. In this review, I will keep using the RSCUS notation, even though it has been dropped from the updated version. The key point is this is the ratio of observed and expected codon frequencies. If this ratio is 1 for all codons, then CAIS would be 0 based on equation 7 in the manuscript – consistent with the complete absence of selection on codon usage. From here on out, subscripts will only be used to denote the codon and it will be assumed that we are only considering the case of r = genome for some species s.

      I think what the authors are attempting to do is “divide out” the effects of mutation bias (as given by Ei), such that only the effects of natural selection remain, i.e. deviations from the expected frequency based on mutation bias alone represents adaptive codon usage. Consider Gilchrist et al. GBE 2015, which says that the expected frequency of codon i at selection-mutation-drift equilibrium in gene g for an amino acid with Na synonymous codons is

      where ∆M is the mutation bias, ∆η is the strength of selection scaled by the strength of drift, and φg is the gene expression level of gene g. In this case, ∆M and ∆η reflect the strength and direction of mutation bias and natural selection relative to a reference codon, for which ∆M,∆η = 0. Assuming the selection-mutation-drift equilibrium model is generally adequate to model of the true codon usage patterns in a genome (as I do and I think the authors do, too), the Ei,g could be considered the expected observed frequency codon i in gene g

      E[Oi,g].

      Let’s re-write the  in the form of Gilchrist et al., such that it is a function of mutation bias ∆M. For simplicity we will consider just the two codon case and assume the amino acid sequence is fixed. Assuming GC% is at equilibrium, the term gr and 1 − gr can be written as

      where µx→y is the mutation rate from nucleotides x to y. As described in Gilchrist et al. MBE 2015 and Shah and Gilchrist PNAS 2011, the mutation bias . This can be expressed in terms of the equilibrium GC content by recognizing that

      As we are assuming the amino acid sequence is fixed, the probability of observing a synonymous codon i at an amino acid becomes just a Bernoulli process.

      If we do this, then

      Recall that in the Gilchrist et al. framework, the reference codon has ∆MNNG,NNG \= 0 =⇒ e−∆MNNG,NNG \=

      (1) Thus, we have recovered the Gilchrist et al. model from the formulation of Ei under the assumption that natural selection has no impact on codon usage and codon NNG is the pre-defined reference codon. To see this, plug in 0 for ∆η in equation (1).

      We can then calculate the expected RSCUS using equation (1) (using notation E[Oi]) and equation (6) for the two codon case. For simplicity assume, we are only considering a gene of average expression (defined as ). Assume in this case that NNG is the reference codon (∆MNNG,∆ηNNG \= 0).

      This shows that the expected value of RSCUS for a two codon amino acid is expected to increase as the strength of selection ∆η increases, which is desired. Note that ∆η in Gilchrist et al. is formulated in terms of selection against a codon relative to the reference, such that a negative value represents that a codon is favored relative to the reference. If ∆η = 0 (i.e. selection does not favor either codon), then E[RSCUS] = 1. Also note that the expected RSCUS does not remain independent of the mutation bias. This means that even if sNe (i.e. the strength of natural selection) does not change between species, changes to the strength and direction of mutation bias across species could impact RSCUS. Assuming my math is right, I think one needs to be cautious when interpreting CAIS as representative of the differences in the efficiency of selection across species except under very particular circumstances.

      Consider our 2-codon amino acid scenario. You can see how changing GC content without changing selection can alter the CAIS values calculated from these two codons. Particularly problematic appears to be cases of extreme mutation biases, where CAIS tends toward 0 even for higher absolute values of the selection parameter. Codon usage for the majority of the genome will be primarily determined by mutation biases,

      with selection being generally strongest in a relatively few highly-expressed genes. Strong enough mutation biases ultimately can overwhelm selection, even in highly-expressed genes, reducing the fraction of sites subject to codon adaptation.

      Peer review image 1.

      Peer review image 2.

      CAIS (Low Expression)

      Peer review image 3.

      CAIS (Average Expression)

      Peer review image 4.

      CAIS (High Expression)

      If we treat the expected codon frequencies as genome-wide frequencies, then we are basically assuming this genome made up entirely of a single 2-codon amino acid with selection on codon usage being uniform across all genes. This is obviously not true, but I think it shows some of the potential limitations of the CAIS approach. Based on these simulations, CAIS seems best employed under specific scenarios. One such case could be when it is known that mutation bias varies little across the species of interest. Looking at the species used in this manuscript, most of them have a GC content around 0.41, so I suspect their results are okay (assuming things like GC-biased gene conversion are not an issue). Outliers in GC content probably are best excluded from the analysis.

      Although I have not done so, I am sure this could be extended to the 4 and 6 codon amino acids. One potential challenge to CAIS is the non-monotonic changes in codon frequencies observed in some species (again, see Shah and Gilchrist 2011 and Gilchrist et al. 2015).

    1. Author response:

      The following is the authors’ response to the original reviews.

      Public Reviews:

      Reviewer #1 (Public review):

      Summary:

      In this manuscript, Liu et al. present CROWN-seq, a technique that simultaneously identifies transcription-start nucleotides and quantifies N6,2'-O-dimethyladenosine (m6Am) stoichiometry. This method is derived from ReCappable-seq and GLORI, a chemical deamination approach that differentiates A and N6-methylated A. Using ReCappable-seq and CROWN-seq, the authors found that genes frequently utilize multiple transcription start sites, and isoforms beginning with an Am are almost always N6-methylated. These findings are consistently observed across nine cell lines. Unlike prior reports that associated m6Am with mRNA stability and expression, the authors suggest here that m6Am may increase transcription when combined with specific promoter sequences and initiation mechanisms. Additionally, they report intriguing insights on m6Am in snRNA and snoRNA and its regulation by FTO. Overall, the manuscript presents a strong body of work that will significantly advance m6Am research.

      Strengths:

      The technology development part of the work is exceptionally strong, with thoughtful controls and well-supported conclusions.

      We appreciate the reviewer for the very positive assessment of the study. We have addressed the concerns below.

      Weaknesses:

      Given the high stoichiometry of m6Am, further association with upstream and downstream sequences (or promoter sequences) does not appear to yield strong signals. As such, transcription initiation regulation by m6Am, suggested by the current work, warrants further investigation.

      We thank the reviewer for the insightful comments. We have softened the language related to m<sup>6</sup>Am and transcription regulation. We totally agree with the reviewer that future investigation is required to determine the molecular mechanism behind m<sup>6</sup>Am and transcription regulation.

      Reviewer #2 (Public review):

      Summary:

      In the manuscript "Decoding m6Am by simultaneous transcription-start mapping and methylation quantification" Liu and co-workers describe the development and application of CROWN-Seq, a new specialized library preparation and sequencing technique designed to detect the presence of cap-adjacent N6,2'-O-dimethyladenosine (m6Am) with single nucleotide resolution. Such a technique was a key need in the field since prior attempts to get accurate positional or quantitative measurements of m6Am positioning yielded starkly different results and failed to generate a consistent set of targets. As noted in the strengths section below the authors have developed a robust assay that moves the field forward.

      Furthermore, their results show that most mRNAs whose transcription start nucleotide (TSN) is an 'A' are in fact m6Am (85%+ for most cell lines). They also show that snRNAs and snoRNAs have a substantially lower prevalence of m6Am TSNs.

      Strengths:

      Critically, the authors spent substantial time and effort to validate and benchmark the new technique with spike-in standards during development, cross-comparison with prior techniques, and validation of the technique's performance using a genetic PCIF1 knockout. Finally, they assayed nine different cell lines to cross-validate their results. The outcome of their work (a reliable and accurate method to catalog cap-adjacent m6Am) is a particularly notable achievement and is a needed advance for the field.

      Weaknesses:

      No major concerns were identified by this reviewer.

      We thank the reviewer for the positive assessment of the method and dataset. We have addressed the concerns below.

      Mid-level Concerns:

      (1) In Lines 625 and 626, the authors state that “our data suggest that mRNAs initate (mis-spelled by authors) with either Gm, Cm, Um, or m6Am.” This reviewer took those words to mean that for A-initiated mRNAs, m6Am was the ‘default’ TSN. This contradicts their later premise that promoter sequences play a role in whether m6Am is deposited.

      We thank the reviewer for the comment. We have changed this sentence into “Instead, our data suggest that mRNAs initiate with either Gm, Cm, Um, or Am, where Am are mostly m<sup>6</sup>Am modified.” The revised sentence separates the processes of transcription initiation and m<sup>6</sup>Am deposition, which will not confuse the reader.

      (2) Further, the following paragraph (lines 633-641) uses fairly definitive language that is unsupported by their data. For example in lines 637 and 638 they state “We found that these differences are often due to the specific TSS motif.” Simply, using ‘due to’ implies a causative relationship between the promoter sequences and m6Am has been demonstrated. The authors do not show causation, rather they demonstrate a correlation between the promoter sequences and an m6Am TSN. Finally, despite claiming a causal relationship, the authors do not put forth any conceptual framework or possible mechanism to explain the link between the promoter sequences and transcripts initiating with an m6Am.

      (3) The authors need to soften the language concerning these data and their interpretation to reflect the correlative nature of the data presented to link m6Am and transcription initiation.

      For (2) and (3). We have softened the language in the revised manuscript. Specifically, for lines 633-641 in the original manuscript, we have changed “are often due to” into “are often related to” in the revised manuscript, which claims a correlation rather than a causation.

      Reviewer #3 (Public review):

      Summary:

      m6Am is an abundant mRNA modification present on the TSN. Unlike the structurally similar and abundant internal mRNA modification m6A, m6Am’s function has been controversial. One way to resolve controversies surrounding mRNA modification functions has been to develop new ways to better profile said mRNA modification. Here, Liu et al. developed a new method (based on GLORI-seq for m6A-sequencing), for antibody-independent sequencing of m6Am (CROWN-seq). Using appropriate spike-in controls and knockout cell lines, Liu et al. clearly demonstrated CROWN-seq’s precision and quantitative accuracy for profiling transcriptome-wide m6Am. Subsequently, the authors used CROWN-seq to greatly expand the number of known m6Am sites in various cell lines and also determine m6Am stoichiometry to generally be high for most genes. CROWN-seq identified gene promoter motifs that correlate best with high stoichiometry m6Am sites, thereby identifying new determinants of m6Am stoichiometry. CROWN-seq also helped reveal that m6Am does not regulate mRNA stability or translation (as opposed to past reported functions). Rather, m6Am stoichiometry correlates well with transcription levels. Finally, Liu et al. reaffirmed that FTO mainly demethylates m6Am, not of mRNA but of snRNAs and snoRNAs.

      Strengths:

      This is a well-written manuscript that describes and validates a new m6Am-sequencing method: CROWN-seq as the first m6Am-sequencing method that can both quantify m6Am stoichiometry and profile m6Am at single-base resolution. These advantages facilitated Liu et al. to uncover new potential findings related to m6Am regulation and function. I am confident that CROWN-seq will likely be the gold standard for m6Am-sequencing henceforth.

      Weaknesses:

      Though the authors have uncovered a potentially new function for m6Am, they need to be clear that without identifying a mechanism, their data might only be demonstrating a correlation between the presence of m6Am and transcriptional regulation rather than causality.

      We thank the reviewer for the very positive assessment of the CROWN-seq method. We have softened the language which is related to the correlation between m<sup>6</sup>Am and transcription regulation.

      Reviewer recommendations:

      We thank the reviewers for their constructive suggestions. In the revised manuscript, we have corrected the errors and updated the requested discussions and figures.

      Reviewer #1 (Recommendations for the authors):

      (1) The prior work from the research group, "Reversible methylation of m6Am in the 5′ cap controls mRNA stability" (PMID: 28002401), should be cited, even if the current findings differ from earlier conclusions-particularly in line 58 and the section titled "m6Am does not substantially influence mRNA stability or translation".

      We thank the reviewer for this comment. We have added the citation.

      (2) I wonder why the authors chose to convert A to I before capping and recapping, as RNA fragmentation caused by chemical treatment may introduce noise into these processes.

      We thank the reviewer for this comment. This is a very good point. We have indeed considered this alternative protocol. There are two concerns in performing decapping-and-recapping before A-to-I conversion: (1) it is unclear whether the 3’-desthiobiotin, which is essential for the 5’ end enrichment, is stable or not during the harsh A-to-I conversion; (2) performing decapping-and-recapping first requires more enzyme and 3’-desthiobiotin-GTP, which are the major cost of the library preparation. This is because the input of CROWN-seq (~1 μg mRNA) is much higher than that in ReCappable-seq (~5 μg total RNA or ~250 ng mRNA). In the current protocol, many 5’ ends are highly fragmented and therefore are lost during the A-to-I conversion. As a result, less enzyme and 3’-desthiobiotin-GTP are needed.

      (3) During CROWN-seq benchmarking, the authors found that 93% of reads mapped to transcription start sites, implying a 7% noise level with a spike-in probe. This noise could lead to false positives in TSN assignments in real samples. It appears that additional filters (e.g., a known TSS within 100 nt) were applied to mitigate false positives. If so, I recommend that the authors clarify these filters in the main text.

      We thank the reviewer for this comment. We think that the spike-in probes might lead to an underestimation of the accuracy of TSN mapping. The spike-in probes are made by in vitro transcription with m<sup>7</sup>Gpppm<sup>6</sup>AmG or m<sup>7</sup>GpppAmG analogs. We found that the in vitro transcription exhibits a small amount of non-specific initiation, which leads to spike-in probes with 5’ ends that are not precisely aligned with the desired TSS. To better illustrate the mapping accuracy of CROWN-seq, we provided Figure 2H, which compares the non-conversion rates of newly found A-TSNs between wild-type and PCIF1 knock cells. If the newly found A-TSNs are real, they should show high non-conversion rates in wild-type cells (i.e., high m<sup>6</sup>Am) and almost zero non-conversion rates (i.e., Am) in PCIF1 knockout cells. As expected, most of the newly found A-TSNs are true A-TSNs since they are m6Am in wild-type and Am in PCIF1 knockout. Thus, we think that CROWN-seq is very precise in TSS mapping. We have clarified this in the Discussion.

      (4) I wonder if PCIF1 knockout affects TSN choice and abundance. If not, this data should be presented. If so, how are these changes accounted for in Figure 2H and Figure S5?

      We thank the reviewer for this comment.  PCIF1 KO does not really affect TSN choice. Here we calculate the correlation of relative TSN expression within genes between wild-type and PCIF1 KO cells (shown using Pearson’s r). It shows that most of the genes have similar TSN choices (with higher Pearson’s r) in both wild-type and PCIF1 KO cells. Thus, PCIF1 KO does not alter global TSN expressions.

      Author response image 1.

      (5) The manuscript refers to Am as a rare modification in mRNA (e.g., introduction lines 101-102; discussion lines 574, 608; and possibly other locations) without specifying this only applies to transcription start sites. As this study does not cover entire mRNA sequences, these statements may not be misleading.

      We thank the reviewer for this comment.  We have clarified it.

      Reviewer #2 (Recommendations for the authors):

      (1) On line 122, the authors state that: "On average, a gene uses 9.5{plus minus}9 (mean and s.d., hereafter) TSNs (Figure 1A)." However, they do not discuss the dispersion apparent in the TSNs they observed. Figure panels 1A, B, and S1A, B show a range of 120 bases or less. What is the predominant range of distances between annotated TSNs and the newly identified ones?

      1a) For example, what percentage of new TSNs fall within 20? 50? 75? bases of the annotated sites? Additional text describing the distribution of these TSNs would help readers better understand the diversity inherent in these novel 5' RNA ends. Notably, this additional text likely is best placed in the CROWN-Seq section related to Figure 2 or S2.

      We thank the reviewer for this comment. We have updated Figure S2 to describe the newly found TSSs. Depending on the coverage in CROWN-seq, the TSSs with higher coverage tend to overlap with or locate proximally to known TSSs. In contrast, the TSSs with low coverage tend to be located further away from annotated TSSs.

      1b) The alternate TSNs can have effects on splicing patterns and isoform identity. Providing a few sentences to explain how regularly this occurs would be helpful.

      We thank the reviewer for this comment. It is a very interesting point. Different TSNs can indeed have different splicing patterns. Although the discovery of splicing patterns regulated by TSNs is out of the scope of this study, we have discussed this possibility in the revised Discussion section.

      (2) On Lines 241 and 242, the authors mentioned that 1284 sites were excluded from the analysis based on low (under 20-explained in the figure legend) read count, distance from TSS, or false negatives (which are not explained). Although I agree that the authors are justified in setting these reads aside, the information could be useful to readers willing to perform follow-up work if their mRNAs of interest were included in these 1284 sites.

      2a) An annotation of all of these sites (broken down by category, i.e. the 811, the 343, and the 130) as a supplementary table should be provided.

      We thank the reviewer for this comment. We have added the categories to the revised Table S1.

      (3) Although I have marked several typos/grammar mistakes in several parts of this review, others exist elsewhere in the text and should be corrected.

      We thank the reviewer for this comment. We have corrected them.

      (4) In lines 122 and 123 the authors say "Only ~9% of genes contain a single TSN (Figure 1A)." However, their figure shows 81% with a single TSN. Why is there a 10% discrepancy?

      We thank the reviewer for this comment. We have corrected the plot in Figure 1A, to match the description.

      (5) The first Tab of Table S2 is labeled 'Legend', but is blank. Is this intentional?

      We thank the reviewer for this comment. We have updated the table legends.

      (6) On lines 70 and 76 of the supplementary figure file pertaining to Figure S2, the legend labels for Figure S2E and S2F are not accurate, they need to be changed to G and H.

      (7) In Figure 4A 'percentile' is misspelled.

      (8) The color-coding legend for the 4 bases is missing from (and should be added to) Figure S4A.

      (9) On Lines 984, 1163, and 1194 the '2s' should be properly sub-scripted where appropriate.

      For (6) to (9). We thank the reviewer for finding these issues. We have now corrected them.

      Reviewer #3 (Recommendations for the authors):

      (1) The authors should discuss if their results can definitively distinguish between the SSCA+1GC motif promoting m6Am that, in turn, promotes transcription, versus the SCA+1GC motif promoting m6Am but also separately promoting transcription in a m6Am-independent manner. The authors should also discuss this in light of recent findings by An et al. (2024 Mol. Cell), which support the former conclusion.

      We thank the reviewer for the suggestion. We now have updated the Discussion to address that our paper and An et al. can support each other.

      (2) Given that the authors showed m6Am promotes gene expression (Figure 5) but does not affect mRNA stability (Fig. S5), logic dictates that m6Am must regulate mRNA transcription. However, the authors should explain why this regulation focuses on the initiation aspect of transcription rather than other aspects of transcriptional e.g. premature termination, pause release, and elongation.

      We thank the reviewer for this comment. In this study, we did not profile the 3’ ends of nascent RNAs and thus we can only make conclusions about the overall transcription process but not a specific aspect. We have updated the revised Discussion section to mention that An et al. discovered that m<sup>6</sup>Am can sequester PCF11 and thus promote transcription, and therefore some of the effects we see could be related to differential premature termination.

      (3) Authors should add alternative versions of Figure 1D but with 3 colours corresponding to Am vs. m6Am vs. Cm/Gm/Um for all the cells, they performed CROWN-seq on.

      We thank the reviewer for this comment. We have updated Figure S5 as the corresponding figure showing the fraction of Am vs. m6Am vs. Cm/Gm/Um.

      (4) Figure 2H (left): Please comment on the few outliers that still show high non-conversion even in PCIF1-KO cells.

      We thank the reviewer for this comment. We have discussed the outliers in the main text. These outliers can be found in the revised Table S3.

      (5) Line 254: "Second, if these sites were RNA fragments they would not contain m6Am." is missing a comma.

      (6) S2G and S2H labelling in Figure S2 legends is wrong.

      For (5) and (6). We thank the reviewer for these comments. We have corrected them.

      (7) Figure 3D: Many gene names are printed multiple times (e.g. ACTB is printed 5 times). Is this correct; is each dot representing 1 cell line?

      We thank the reviewer for this comment. These gene names represent different transcription-start nucleotides. We now clarify that each instance refers to a different start site.

      (8) S5A-C: Even if there's no substantial difference, authors should still display the Student's T-test P-values as they did for S5D-G.

      We thank the reviewer for this comment. We have updated the P-values.

      (9) Figure 5C and S5E: Why are the authors not showing the respective analysis for C-TSN and U-TSN genes?

      We thank the reviewer for this comment. Most mRNAs start with A or G. We therefore selected G-TSN as the control. Unlike G-TSNs which occur in diverse sequence and promoter contexts, C-TSNs and U-TSNs are unusual. Genes that mainly use C-TSNs and U-TSNs are the so-called “5’ TOP (Terminal OligoPyrimidine)” genes. The 5’ TOP genes are mostly genes related to translation and metabolism, and thus their expressions reflect the homeostasis of cell metabolism. Thus, we were concerned that any differential expression of the C-TSN and U-TSN genes between wild-type and PCIF1 knockout cells might reflect specific effects on TOP transcriptional regulation rather than the general effects of PCIF1 on transcription.

      (10) Line 82, 470, 506, 676: The authors should also cite Koh et al (2019 Nat. Comm.) in these lines that describe how snRNAs can also be m6Am-methylated and how FTO targets these same snRNAs for demethylation.

      We thank the reviewer for this comment. We have updated the citation.

    1. Author response:

      The following is the authors’ response to the original reviews

      Reviewer #1 (Public Review): 

      Summary: 

      This manuscript presents a method to infer causality between two genes (and potentially proteins or other molecules) based on the non-genetic fluctuations among cells using a version of the dual-reporter assay as a causal control, where one half of the dual-reporter pair is causally decoupled, as it is inactive. The authors propose a statistical invariant identity to formalize this idea. 

      We thank the referee for this summary of our work. 

      Strengths: 

      The paper outlines a theoretical formalism, which, if experimentally used, can be useful in causal network inference, which is a great need in the study of biological systems. 

      We thank the referee for highlighting the potential value of our proposed method.

      Weaknesses: 

      The practical utility of this method may not be straightforward and potentially be quite difficult to execute. Additionally, further investigations are needed to provide evidence of the broad applicability of the method to naturally occurring systems and its scalability beyond the simple circuit in which it is experimentally demonstrated. 

      We agree with these two points and have rewritten the manuscript, in particular highlighting the considerable future work that remains to be done to establish the broad applicability and scalability of our method.

      In the rewritten manuscript we explicitly spell out potential practical issues and we explicitly state that our presented proof–of–principle feasibility study does not guarantee that our method will successfully work in systems beyond the narrowly sampled test circuits. This helps readers to clearly distinguish between what we claim to have done from what remains to be done. The re-written parts and additional clarifications are:

      Abstract (p. 1), Introduction (p. 1-2), Sec. “Proposed additional tests” (p. 8), and “Limitations of this study” (p. 10).

      Reviewer #2 (Public Review): 

      Summary: 

      This paper describes a new approach to detecting directed causal interactions between two genes without directly perturbing either gene. To check whether gene X influences gene Z, a reporter gene (Y) is engineered into the cell in such a way that (1) Y is under the same transcriptional control as X, and (2) Y does not influence Z. Then, under the null hypothesis that X does not affect Z, the authors derive an equation that describes the relationship between the covariance of X and Z and the covariance of Y and Z. Violation of this relationship can then be used to detect causality. 

      The authors benchmark their approach experimentally in several synthetic circuits. In four positive control circuits, X is a TetR-YFP fusion protein that represses Z, which is an RFP reporter. The proposed approach detected the repression interaction in two or three of the positive control circuits. The authors constructed sixteen negative control circuit designs in which X was again TetR-YFP, but where Z was either a constitutively expressed reporter or simply the cellular growth rate. The proposed method detected a causal effect in one of the eight negative controls, which the authors argue is not a false positive, but due to an unexpected causal effect. Overall, the data support the practical usefulness of the proposed approach. 

      We thank the referee for their summary of our work.

      Strengths: 

      The idea of a "no-causality control" in the context of detected directed gene interactions is a valuable conceptual advance that could potentially see play in a variety of settings where perturbation-based causality detection experiments are made difficult by practical considerations. 

      By proving their mathematical result in the context of a continuous-time Markov chain, the authors use a more realistic model of the cell than, for instance, a set of deterministic ordinary differential equations. 

      We thank the referee for summarizing the value of our work. 

      Caveats: 

      The term "causally" is used in the main-text statement of the central theorem (Eq 2) without a definition of this term. This makes it difficult to fully understand the statement of the paper's central theorem without diving into the supplement.  

      We thank the referee for this suggestion. In the revised manuscript we now define causal effects right before the statement of the main theorem of the main text (p. 2). We have also added a definition of the causal network arrows in the caption of Fig. 1 to help readers better understand our central claim.

      The basic argument of theorem 1 appears to rely on establishing that x(t) and y(t) are independent of their initial conditions. Yet, there appear to be some scenarios where this property breaks down: 

      (1) Theorem 1 does not seem to hold in the edge case where R=beta=W=0, meaning that the components of interest do not vary with time, or perhaps vary in time only due to measurement noise. In this case x(t), y(t), and z(t) depend on x(0), y(0), and z(0). Since the distributions of x(0), y(0), and z(0) are unspecified, a counterexample to the theorem may be readily constructed by manipulating the covariance matrix of x(0), y(0), and z(0). 

      (2) A similar problem may occur when transition probabilities decay with time. For example, suppose that again R=0 and X are degraded by a protease (B), but this protease is subject to its own first-order degradation. The deterministic version of this situation can be written, for example, dx/dt=-bx and db/dt=-b. In this system, x(t) approaches x(0)exp(-b(0)) for large t. Thus, as above, x(t) depends on x(0). If similar dynamics apply to the Y and Z genes, we can make all genes depend on their initial conditions, thus producing a pathology analogous to the above example. 

      The reviewer does not know when such examples may occur in (bio)physical systems. Nevertheless, since one of the advantages of mathematics is the ability to correctly identify the domain of validity for a claim, the present work would be strengthened by "building a fence" around these edge cases, either by identifying the comprehensive set of such edge cases and explicitly prohibiting them in a stated assumption set, or by pointing out how the existing assumptions already exclude them.  

      We thank the referee for bringing to our attention these edge cases that indeed violate our theorem as stated. In the revised manuscript we have “built a fence” around these edge cases by adding two requirements to the premise of our theorem: First, we have added the requirement that the degradation rate does not decay to zero for any possible realization. That is, if beta(t) is the degradation rate of X and Y for a particular cell over time, then taking the time average of beta(t) over all time must be non-zero. Second, we have added the requirement that the system has evolved for enough time such that the dual reporter averages <x> and <y>, along with the covariances Cov(x, z_{k}) and Cov(y, z_{k}) have reached a time-independent stationary state.  

      With these requirements, no assumptions need to be made about the initial conditions of the system, because any differences in the initial conditions will decay away as the system reaches stationarity. For instance, the referee’s example (1) is not possible with these requirements because beta(t) can no longer remain zero. Additionally, example (2) is no longer possible because the time average of the degradation rate would be zero, which is no longer allowed (i.e., we would have that integral from 0 to T of b(0)exp(-t)/T dt =  0 when T goes to infinity). 

      Note that adding the condition that degradation cannot decay to exactly zero does not reduce the biological applicability of the theorem. But as the referee correctly points out any mathematical theorem needs to be accurately stated and stand on its own regardless of whether biological systems could realize particular edge cases. Also note, that the requirement that the cellular ensemble has reached a time-independent distribution of cell-to-cell variability can be (approximately) experimentally verified by taking snapshots of ensemble variability at two sufficiently separate different moments in time. 

      In response to the referee’s comment, we have added the above requirements when stating the theorem in the main text. We have also added the requirement of non-decay of the degradation rate to the definition of the system in SI Sec. 4, along with the stationarity requirement in theorem 1 in SI Sec 5. We have also added mathematical details to the proof of the invariant in SI Sec 5.

      Recommendations for the authors:

      Reviewer #1 (Recommendations For The Authors): 

      This manuscript presents a method to infer causality between two genes (and potentially proteins or other molecules) based on the non-genetic fluctuations among cells using a version of the dual-reporter assay as a causal control, where one half of the dual-reporter pair is causally decoupled, as it is inactive. The authors propose a statistical invariant identity to formalize this idea. They propose and experimentally demonstrate the utility of this idea with a synthetic reporter system in bacteria. 

      The paper is well written and clearly outlines the principle, the mathematical invariant relationship both to give the reader an intuitive understanding of why the relationship must be true and in their mathematical derivation of the proof of Theorem 1. 

      The paper outlines a theoretical formalism, which, if experimentally used, can be useful in causal network inference, which is a great need in the study of biological systems. However, the practical utility of this method may not be straightforward and potentially be quite difficult to execute. We think this work could offer a platform to advance the field of network inference, but would encourage the authors to address the following comments. 

      We thank the reviewer for the positive comments on readability, summarizing the value of our work, as well as the critical comments below that helped us improve the manuscript.

      Major comments: 

      (1) Although the invariant identity seems theoretically sound, the data from synthetic engineered circuits in this manuscript do not support that the invariant holds for natural causal relations between genes in wild-type cells. In all the positive control synthetic circuits (numbers 1 to 4) the target gene Z i.e. RFP was always on the plasmid, and in circuit #4 there was an additional endogenous copy. The authors recapitulate the X-to-Z causality in circuits 1, 2, and 3 but not 4. Ultimately, the utility of this method lies in the ability to capture causality from endogenous correlations, this observation suggests that the method might not be useful for that task. 

      We thank the referee for their careful reading of our synthetic circuits and sincerely apologize for an error in our description of circuit #4 in the schematic of Table S2 of the supplement. We incorrectly stated that this circuit contained a chromosomally expressed RFP. In fact, in circuit #4 RFP was only on the plasmid just like in the circuits #1-3. We have corrected the schematic in the revised manuscript and have verified that the other circuits are correctly depicted.

      In the revised manuscript, we now explicitly spell out that all our “positive control” test cases had the genes of interest expressed on plasmids, and that we have not shown that our method successfully detected causal interactions in a chromosomally encoded gene regulatory circuit, see additional statements in Sec. “Causally connected genes that break the invariant” on p. 6. 

      In the absence of any explicit experimental evidence, it is then important to consider whether chromosomally encoded circuits are expected to cause problems for our method which is based on a fluctuation test. Due to plasmid copy number fluctuations, X and Z will fluctuate significantly more when expressed on plasmids than when expressed chromosomally. However, because this additional variability is shared between X and Z it does not help our analysis which relies on stochastic differences in X and Z expression due to “intrinsic noise” effects downstream of copy number fluctuations. The additional “extrinsic noise” fluctuations due to plasmid copy number variability would wash out violations of Eq. (2) rather than amplify them. If anything, we thus expect our test cases to have been harder to analyze than endogenous fluctuations. This theoretical expectation is indeed borne out by numerical test cases presented in the revised supplement where plasmid copy fluctuations severely reduced the violations of Eq. 2, see new additional SI Sec. 15. 

      Additionally, the case of the outlier circuit (number 12) suggests that exogenous expression of certain genes may lead to an imbalance of natural stoichiometry and lead to indirect effects on target genes which can be misinterpreted as causal relations. Knocking out the endogenous copy may potentially ameliorate this issue but that remains to be tested. 

      We agree with the referee that the expression of exogenous genetic reporters can potentially affect cellular physiology and lead to undesired effects. In the revised manuscript we now explicitly spell out that the metabolic burden or the phototoxicity of introducing fluorescent proteins could in principle cause artificial interactions that do not correspond to the natural gene regulatory network, see Sec. “Proposed additional tests” on p. 8.

      However, it is also important to consider that the test circuit #12 represents a synthetic circuit with genes that were expressed at extremely high levels (discussed in 3rd paragraph of Sec. “Evidence that RpoS mediated stress response affected cellular growth in the outlier circuit”, p. 8), which led to the presumed cellular burden. Arguably, natural systems would not typically exhibit such high expression levels, but importantly even if they did, our method does not necessarily rely on fluorescently tagged proteins but can, in principle, also be applied to other methods such as transcript counting through sequencing or in-situ hybridization of fluorescent probes.  

      Ultimately, the value of this manuscript will be greatly elevated if the authors successfully demonstrate the recapitulation of some known naturally existing causal and non-causal relations. For this, the authors can choose any endogenous gene Z that is causally controlled by gene X. The gene X can be on the exogenous plasmid along with the reporter and the shared promoter. Same for another gene Z' which is not causally controlled by gene X. Potentially a knockout of endogenous X may be required but it might depend  on what genes are chosen. 

      If the authors think the above experiments are outside the scope of this manuscript, they should at least address these issues and comment on how this method could be effectively used by other labs to deduce causal relations between their favorite genes. 

      Because a full analysis of naturally occurring gene interactions was beyond the scope of our work, we agree with the referee’s suggestion to add a section to discuss the limitations of our experimental results. In the revised manuscript we reiterate that additional investigations are needed to show that the method works to detect causal interactions between endogenous genes, see Abstract (p. 1), Introduction (p. 1-2), Sec. “Proposed additional tests” (p. 8), and “Limitations of this study”  (p. 9). In the original manuscript we explicitly spelled out how other researchers can potentially carry out this further work in the subsections titled “Transcriptional dual reporters” (p. 3) and ”Translational dual reporters” (p. 3).  In the revised manuscript, we have added a section “Proposed additional tests” (p. 8) in which we propose an experiment analogous to the one proposed by the referee above, involving an endogenous gene circuit found in E. coli, as an example to test our invariant. 

      (2) For a theoretical exposition that is convincing, we suggest the authors simulate a larger network (for instance, a network with >10 nodes), like the one shown schematically in Figure 1, and demonstrate that the invariant relationship holds for the causally disconnected entities, but is violated for the causally related entities. It would also be interesting to see if any quantification for the casual distance between "X" and the different causally related entities could be inferred.  

      We thank the referee for this suggestion. We have added SI Sec. 14 where we present simulation results of a larger network with 10 nodes. We find that all of the components not affected by X satisfy Eq. (2) as they must. However, it is important to consider that we have analytically proven the invariant of Eq. (2) for all possible systems. It provably applies equally to networks with 5, 100, or 10,000 components. The main purpose of the simulations presented in Fig. (2) is to illustrate our results and to show that correlation coefficients do not satisfy such an invariant. However, they are not used as a proof of our mathematical statements.

      We thank the referee for the interesting suggestion of quantifying a “causal distance”. Unfortunately, the degree to which Eq. (2) is violated cannot directly equate to an absolute measure for the “causal distance” of an interaction. This is because both the strength of the interaction and the size of the stochastic fluctuations in X affect the degree to which Eq. (2) is violated. The distance from the line should thus be interpreted as a lower bound on the causal effect from X to Z because we do not know the magnitude of stochastic effects inherent to the expression of the dual reporters X and Y. While the dual reporters X and Y are identically regulated, they will differ due to stochastic fluctuations. Propagation of these fluctuations from X to Z are what creates an asymmetry between the normalized covariances. In the most extreme example, if X and Y do not exhibit any stochastic fluctuations we have x(t)=y(t) for all times and Eq. (2) will not be violated even in the presence of a strong causal link from X to Z.

      However, it might be possible to infer a relative causal distance to compare causal interactions within cells.

      That is, in a given network, the normalized covariances between X, Y and two other components of interest Z1, Z2 that are affected by X can be compared. If the asymmetry between (η𝑥𝑧1 , η𝑦𝑧1) is larger than the asymmetry between (η𝑥𝑧2 , η𝑦𝑧2) , then we might be able to conclude that X affects Z1 with a stronger interaction than the interaction from X to Z2, because here the intrinsic fluctuations in X are the same in both cases. 

      In response to the referee’s comment and to test the idea of a relative causal distance, we have simulated a larger network made of 10 components. In this network, X affects a cascade of components called Z8, Z9, and Z10, see the additional SI Sec. 14. Here the idea of a causal distance can be defined as the distance down the cascade: Z8 is closest to X and so has the largest causal strength, whereas Z10 has the weakest. Indeed, simulating this system we find that the asymmetry between η𝑥𝑧8 and η𝑦𝑧8 is the largest whereas that between  η𝑥𝑧10 and η𝑦𝑧10 the smallest. We also find that all of the components not affected by X have normalized covariances that satisfy Eq. (2). This result suggests that the relative causal distance or strength in a network could potentially be estimated from the degree of the violations of Eq. (2). 

      However, we note that these are preliminary results. In the case of the specific regulatory cascade now considered in SI Sec. 14, the idea of a causal distance can be well defined. Once feedback is introduced into the system, this definition may no longer make sense. For instance, consider the same network that we simulate in SI Sec. 14, but where the most downstream component in the cascade, Z10, feeds back and affects X and Y. In such a circuit it is unclear whether Z8 or Z10 is “causally closer” to X. A more thorough theoretical analysis, equipped with a more universal quantitative definition for causal distance or strength, would be needed to deduce what information can be inferred from the relative distances in the violations of Eq. (2). While this defines an interesting research question, answering it goes beyond the scope of the current manuscript. 

      Minor comments: 

      - The method relies on the gene X and the reporter Y having the same control which would result in similar dynamics. The authors do not quantitatively compare the YFP and CFP expression if this indeed holds for the synthetic circuits. It would be useful to know how much deviation between the two can be tolerated while not affecting the outcome. 

      We thank the referee for their comment. The invariant of Eq. (2) is indeed only guaranteed to hold only when the transcription rate of Y is proportional to that of X. How much levels of X and Y covary depends on the stochastic effects intrinsic to the expression of the dual reporters as well as how similar the transcriptional control of X and Y is. The stochastic difference between X and Y is exactly what we exploit. 

      However, in the limit of high YFP and CFP levels, intrinsic fluctuations that cause stochastic expression differences between X and Y become negligible and we can directly infer whether they are indeed tightly co-regulated from time-traces: Below, we show two single cell traces taken with our experimental setup in which the YFP and CFP fluorescence trajectories are almost exactly proportional. Both of these traces are from circuit #10 as defined in Table. S4. 

      Author response image 1.

      We chose the above traces because they showed the highest correlation between YFP and CFP levels. Other traces for lower expression levels have lower correlations due to effects of intrinsic noise (see Tables S2-S4). However, the existence of one trace in which YFP is almost perfectly proportional to CFP throughout can only occur if the YFP and CFP genes are under the same control. And, since the control of YFP and CFP genes in all of our synthetic circuits are identical (with the same promoters and plasmid positions), these data strongly suggest that our dual reporters are tightly co-regulated in all the synthetic circuits. Moreover, the negative control experiments presented in Fig. 3E provide a natural consistency check that the YFP and CFP are under the same control and satisfy Eq. (1).

      We agree that it would be useful to know how much the X and Y production rates can differ for Eq. (2) to hold. Importantly, our proven theorem already allows for the rates to differ by an unspecified proportionality constant. In response to the referee’s comment we have derived a more general condition under which our approach holds. In the newly added SI Sec. 7 we prove that Eq. (2) holds also when rates differ as long as the difference is stochastic in nature with an average of zero. We also prove that Eq. (2) holds in the face of multiplicative noise that is independent of the X and Y production rates.

      However, the production rates of X and Y cannot differ in all ways. Some types of differences between the X and Y production rates can lead to deviations of Eq. (2) even when there is no causal interaction. To highlight this, we added the results of simulations of a toy model in which the X and Y production rates differ by an additive noise term that does not average to zero, see Fig. S19B of the newly added SI Sec. 7.

      - The invariant should potentially hold true for any biological species that are causally related e.g. protein-protein interactions. Also, this method could potentially find many applications in eukaryotic cells. Although it's outside the scope of current work to experimentally demonstrate such applications, the authors should comment on experimental strategies to apply this method to overcome potential pitfalls (e.g. presence of enhancers in eukaryotic cells). 

      We thank the referee for this suggestion. We agree that there are potential pitfalls that could come into effect when our proposed approach is applied on more complex systems such as eukaryotic gene expression. In response to the referee’s comment, we have added an explicit discussion of these potential pitfalls in the discussion section “Limitations of this study” (see p. 10). 

      In particular, in eukaryotes there are many genes in which promoter sequences may not be the sole factor determining transcription rates. Other factors that can be involved in gene regulation include the presence of enhancers, epigenetic modifications, and bursts in gene expression, to name a few. We thus propose a few strategies, which include positioning the passive reporter at a similar gene loci as the gene of interest, measuring the gene regulation activities of the gene of interest and its passive reporter using a separate method, and exploiting the invariant with a third gene, where it is known there is no causal interaction, as a consistency check. In addition, we include in the SI a new section SI Sec. 8 which shows that the invariant holds in the face of many types of bursty gene expression dynamics.

      However, the above is not a comprehensive list. Some of the issues the referee mentions are serious and may not be straightforward to overcome. We now spell this out explicitly in the revised manuscript (p. 10). 

      - In the legend of Fig. 1, the sentence "Data points here are for..." is missing a few words, or needs to be rephrased. 

      We thank the referee for this comment. We have rewritten the figure caption, which now reads “Data points are numerical simulations of specific example networks (see SI for details) to illustrate the analytically proven theorem of Eq. 2.”

      - Fig. 2 talks about the uncertainties associated with each point on the scatter plots. However, it is difficult to understand the quantification in such a plot. It would be great to have a plot quantifying the uncertainties in the invariant relation for the different topologies studied, specifically in order to understand if one topology is consistently deviating more from the x=y line than the other topologies studied here.  

      We thank the referee for this suggestion. In the supplement of the revised manuscript we have added supplemental Figs. S3, S4, and  S5 to separately quantify the uncertainty of the difference processes plotted in Fig. 2 and have added a new section (SI Sec. 11) to discuss the processes simulated in Fig. 2 in more detail. In short, each simulated process generated less than ~5% of outliers when considering 95% confidence intervals (with the max percentage deviation being 5.01% for process 5, see Fig. S5). These outliers were then simulated over a larger number of simulations to reduce the sampling error, which resulted in 0% of outliers (see Sec. “Confidence intervals for finite sampling error” on Materials and Methods on p. 11). Some simulated processes generated larger percentage errors in the normalized covariances than others, but this is expected as different processes have different dynamics which will result in different degrees of sampling of the underlying distributions.

      Note, that the invariant of Eq. 2 is analytically proven for all tested topologies as none of the topologies include a causal effect from X to Z. Any deviation of the numerical data from the straight line prediction of Eq. 2 (right column in Fig. 2C) is due to the finite sampling of a stochastic process to estimate the true covariance from the sampling covariance. Any given parameter set was simulated several times which allowed us to estimate the sampling error from differences in between repeated samples. In the additional SI figures we now quantify this error for the different topologies. 

      In addition to the above changes we want to highlight that the purpose of the simulations presented in Fig. (2) is not to prove our statements or explore the behavior of different topologies. The purpose of the data presented in the right column of Fig. 2C is to illustrate the theoretical invariant and act as a numerical sanity check of our analytically proven result. In contrast, the data in the left column of Fig 2C illustrates that the correlations do not satisfy an invariant like Eq. 2 which applies to covariances but not correlations.  

      - The legend for Fig. 3 seems to end abruptly. There likely needs to be more.  

      We thank the referee for catching this mistake. We have corrected the accidentally truncated figure caption of Fig. 3.

      - There is a typo in equation (5.3) on page 23 of supplementary material, there should be x instead of y in the degradation equation of x. 

      We thank the referee for catching this mistake which has been corrected in the revised manuscript.

      - In the supplemental material, to understand the unexpected novel discovery of causality, Figure S5 is presented. However, this doesn't give the context for other negative controls designed, and the effect of rfp dynamics (which can be seen in the plots both in the main paper and the supplement) in the growth rate of cells in those constructs. As a baseline, it would be nice to have those figures.  

      We thank the referee for this suggestion. We have now included representative RFP traces with the growth rates for other negative control circuits, see Fig. S10. In addition, we have now included the cross correlation functions between RFP and growth rate in these negative control circuits, see Fig. S10A. While in all cases, RFP and growth rate are negatively correlated, the outlier circuit exhibits the largest negative correlation.

      The suggested comparison of the referee thus highlights that – in isolation – a negative correlation between RFP and growth rate is only weak evidence for our hypothesized causal interaction because negative correlations can result from the effect of growth rate affecting volume dilution and thus RFP concentration. Crucially, we thus additionally considered the overall variability of growth rate and found the outlier circuit has the largest growth rate variability which is indicative of something that is affecting the growth rate of those cells, see Fig. S10B. To compare the magnitude of RFP variability against other strains requires constraining the comparison group to other synthetic circuits that have RFP located on the chromosome rather than a plasmid. This is why we compare the CV of the outlier with the CV of circuit #5, which corresponds to the “regular” repressilator (i.e., the outlier circuit without the endogenous lacI gene). As an additional comparison, we computed the CV for a strain of E. coli that does not contain a synthetic plasmid at all, but still contains the RFP gene on the chromosome. We find that the CVs in the outlier circuit to be larger than in these two additional circuits, suggesting that the outlier circuit causes additional fluctuations in the RFP and growth rate. We now spell this out explicitly in the revised manuscript (see Sec. “Evidence that RpoS mediated stress response affected cellular growth in the outlier circuit“, p. 8).

      The referee is correct that the above arguments are only circumstantial evidence, but they do show that the data is consistent with a plausible explanation of the hypothesized causal interaction. Our main evidence for an RpoS mediated stress response that explains the deviations from Eq. 2 in the outlier circuit is the perturbation experiment in which the deviation disappears for the RpoS knockout strain. We now spell out this argument explicitly in the revised manuscript (see Sec. “Evidence that RpoS mediated stress response affected cellular growth in the outlier circuit“, p. 8).

      Reviewer #2 (Recommendations For The Authors): 

      The proof of theorem 1 relies on an earlier result, lemma 1. Lemma 1 only guarantees the existence of a "dummy" system that satisfies the separation requirement and preserves the dynamics of X and Y. However, in principle, it may be possible to maintain the dynamics of X and Y while still changing the relationship between Cov(X,Zk) and Cov(Y,Zk). This could occur if the dynamics of Zk differ in a particular way between the original system and the dummy system. So lemma 1 needs to be a little stronger- it needs  to mention that the dynamics of Zk are preserved, or something along these lines. The proof of lemma 1 appears to contain the necessary ingredients for what is actually needed, but this should be clarified. 

      We agree with the referee that this is an important distinction. Lemma 1 does in fact guarantee that any component Zk that is not affected by X and Y will have the same dynamics in the “dummy” system. However, as the referee points out, this is not stated in the lemma statement nor in the proof of the lemma. In response to the referee’s comment, we have made it clear in the lemma statement that the Zk dynamics are preserved in the “dummy” system, and we have also added details to the proof to show that this is the case, see Lemma 1 on p. 27 of the SI. 

      Readers who are familiar with chemical reaction diagrams, but not birth-death process diagrams may waste some time trying to interpret Equation 1 as a chemical reaction diagram with some sort of rate constant as a label on each arrow (I did this). It may be helpful to either provide a self-contained definition of the notation used, or mention a source where the necessary definitions can be found. 

      We agree with the referee. In the revised manuscript we have added a description of the notation used below Equation 1 of the main text, see p. 2. The notational overloading of the “arrow notation” is a perennial problem in the field and we thank the referee for reminding us of the need to clarify what the arrows mean in our diagrams.

      It would be helpful if the authors could propose a rule for deciding whether dependence is detected or not. As it stands presently, the output of the approach seems to be a chart like that in Figure 3D where you show eta_xz and eta_yz with confidence interval bars and the reader must visually assess whether the points more-or-less fall on the line of unity. It would be better to have some systematic procedure for making a "yes or no" call as to whether a causal link was detected or not. Having a systematic detection rule would allow you to make a call as to whether dependence in circuit 3 was detected or not. It would also allow you or a future effort to evaluate the true positive rate of the approach in simulated settings. 

      We thank the referee for this suggestion. In the revised manuscript we have added an explicit rule for detecting causality using the invariant of Eq. (2). Specifically, Eq. (2) can be re-written as r = 1 where r is the covariability ratio r = etaXZ/etaYZ. In that case, given 95% confidence intervals for the experimentally determined covariability ratio r, we say that there is a causal interaction if the confidence intervals overlap with the value of r = 1. 

      This corresponds to a null hypothesis test at the 2.5% significance level. The reason that it is at 2.5% significance and not 5% significance is as follows. Let’s say we measure a covariability ratio of r_m, and the 95% confidence interval is [r_m - e_m, r_m + e_m] for some error e_m. Without loss of generality, let’s say that r_m > 1 (the same applies if r_m < 1). This means that Prob(r < r_m - e_m) = 2.5% and Prob(r > r_m + e_m) = 2.5% , where r is the actual value of the covariability ratio. Under the null hypothesis that there is no causal interaction, we set r = 1. However, we now have Prob(1 < r_m + e_m) = 0, because we know that r_m > 1 and so we must have r_m + e_m > 1. The probability that the value of 1 falls outside the error bars is therefore 2.5% under the null hypothesis. 

      This proposed rule is the same rule that we used to detect statistical outliers in our simulations, where we found a “false positive” rate of 2.3% over 6522 simulated systems due to statistical sampling error (as discussed in the Materials and Methods section). In response to the referee’s suggestion, we have added the section “A rule for detecting causality in the face of measurement uncertainty” (p. 4). We also apply the rule to the experimental data and find that the rule detects 2/4 causal interactions in Fig. 3D. We have clarified this in the Fig. 3D caption, in the main text, and we have added a figure in the SI (Fig. S2) where we apply the null hypothesis test on the measured covariability ratios. 

      Note, whether the third interaction is “detected” or not depends on the cut-off value used. We picked the most common 95% rule to be consistent with the traditional statistical approaches. With this rule one of the data points lies right at the cusp of detection, but ultimately falls into the “undetected” category if a strictly binary answer is sought under the above rule. 

      It would be helpful to mention what happens when the abundance of a species hits zero. Specifically, there are two ways to interpret the arrow from X to X+d with a W on top: 

      Interpretation (1): 

      P(X+d | X) = W if X+d {greater than or equal to} 0  P(X+d | X) = 0 if X_i+d_i < 0 for at least one i 

      Interpretation (2): 

      P(X+d | X) = W regardless of whether X+d < 0  W = 0 whenever X_i < d_i for at least one i 

      Interpretation (1) corresponds to a graph where the states are indexed on the non-negative integers. Interpretation (2) corresponds to a graph where the states are indexed on the integers (positive or negative), and W is responsible for enforcing the non-negativity of mass. I believe you need the second interpretation because the first interpretation leads to problems with your definition of causality. For example, consider the reaction: 

      (Na, K) -- 0.1 --> (Na-1, K+1) 

      This could occur if Na and K are the intracellular concentrations of sodium and potassium ions in a cell that has an ATP-driven sodium-potassium exchanger whose rate is limited by the frequency with which extracellular potassium ions happen to flow by. Per the definition of causality found in the appendix, Na has no causal effect on K since Na does not show up in the reaction rate term. However, under interpretation (1), Na clearly has a causal effect on K according to a reasonable definition of causality because if Na=0, then the reaction cannot proceed, whereas if Na>0 then it can. However, under interpretation (2), the reaction above cannot exist and so this scenario is excluded. 

      We thank the referee for this comment that helped us clarify the meaning of arrows with propensities. In short, interpretation (2) corresponds to the definition of our stochastic systems. This is consistent with the standard notation used for the chemical master equation. As the referee points out, because molecular abundances cannot be negative, any biochemical system must then have the property that the propensity of a reaction must be equal to zero when the system is in a state in which an occurrence of that reaction would take one of the abundances to negative numbers. Stochastic networks that do not have this property cannot correspond to biochemical reaction networks.

      In the revised manuscript, we now spell this out explicitly to avoid any confusion, see SI page 25.

      Furthermore, we additionally discuss the referee’s example in which the rate of exchanging Na for K through an ion exchanger is approximately independent of the intracellular Na concentration. Because biochemical systems cannot become negative, it cannot be that the rate is truly constant, but at some point for low concentrations must go down until it becomes exactly zero for zero molecules. 

      Importantly, agreement with Eq. (2) does not imply that there is no causal effect from X to Zk. It is the deviation from Eq. (2) that implies the existence of a causal effect from X to Zk. Therefore, although the above referee’s example would constitute a causal interaction in our framework, it would not lead to a deviation of Eq. (2) because the fluctuations in Na (which we exploit) do not propagate to K. From a practical point of view, our method thus detects whether changing X over the observed range affects the production and degradation rates of Zk. 

      In the course of setting up the negative control benchmark circuits, a perturbation-based causal validation would be nice. For instance, first, verify that X does not affect Z by intervening on X (e.g. changing its copy number or putting it under the control of an inducible promoter), and ensuring that Z's activity is not affected by such interventions upon X. This approach would help to adjudicate questions of whether the negative control circuits actually have an unknown causal link. The existing benchmark is already reasonably solid in my view, and I do not know how feasible this would be with the authors' setup, but I think that a perturbation-based validation could in principle be the gold standard benchmark.  

      We agree that additional perturbation-based validation tests on all of the negative control circuits would indeed improve the evidence that our method worked as advertised. While such experiments are indeed beyond the scope of our current work we now explicitly point out the benefits of such additional controls in the revised Discussion.

      Below is a series of comments about typography, mostly about section 4 of the supplement. 

      We thank the referee for their careful reading and highlighting those mistakes.

      At the bottom of page 21, Z_aff is defined as the set of components that are affected by X. However, later Z_aff seems to refer to components affected by X or Y. For instance, in the proof of lemma 1, it is written "However, because a is part of z_aff, the {ak} variables must be affected by X and/or Y." 

      We thank the referee for catching this mistake. We have changed the definition of Z_aff throughout the supplement to refer to components affected by X or Y. If it can be experimentally ensured that Y is a passive reporter (i.e., it does not affect other components in the cell), then the theorem can only be violated if X affects Z. 

      In the equation following Eq 5.2, W_k and d_k should be W_i and d_i ?  

      Yes, the referee is correct. In the revised manuscript we have corrected W_k and d_k to W_i and d_i. 

      In Eq 5.3 in the lower-left transition diagram, I think a "y" should be an "x". 

      Yes, the referee is correct. In the revised manuscript  we have fixed this typo.

      In the master equation above Eq 5.5, the "R" terms for the y reactions are missing the alpha term, and I think two of the beta terms need to be multiplied by x and y respectively.  

      The referee is correct. In the revised manuscript  we have fixed this typo.

      The notation of Eq 5.8, where z_k(t) is the conditional expectation of z_kt, is strange and difficult to follow. Why does z_k(t) not get a bar over it like its counterparts for x, y, R, and beta? The bars, although not a perfect solution, do help.  

      We agree with the referee’s comment and have added further explanations to define the averages in question, see SI p. 28. In short, when we condition on the history of the components not affected by X or Y, we in effect condition on the time trajectories of z_{k} (when it is part of the components not affected by X and/or Y) and beta (since it only depends on the components not affected by X or Y). We thus previously did not include the bars when taking the averages of these components in the conditional space because the conditioning in effect sets their time-trajectories (so they become deterministic functions of time). In the revised manuscript we now also denote these conditional expectations with bars and we have added comments to the proof to clarify their definition.

      I think it would be helpful to show how the relationship <x>=<y>/alpha is obtained from Eq 5.5.  

      We agree with this suggestion and have added the derivations, see Eqs. (5.9) - (5.13) in the revised SI. 

      In the main text, the legend of Fig 3 cuts off mid-sentence.  

      We thank the referee for catching this mistake which has been fixed in the revised manuscript.

    1. Author response:

      The following is the authors’ response to the original reviews

      Public Reviews:

      Reviewer #1 (Public review):

      Summary:

      Oor et al. report the potentially independent effects of the spatial and feature-based selection history on visuomotor choices. They outline compelling evidence, tracking the dynamic history effects based on their clever experimental design (urgent version of the search task). Their finding broadens the framework to identify variables contributing to choice behavior and their neural correlates in future studies.

      Strengths:

      In their urgent search task, the variable processing time of the visual cue leads to a dichotomy in choice performance - uninformed guesses vs. informed choices. Oor et al. did rigorous analyses to find a stronger influence of the location-based selection history on the uninformed guesses and a stronger influence of the feature-based selection history on the informed choices. It is a fundamental finding that contributes to understanding the drivers of behavioral variance. The results are clear.

      Weaknesses:

      (1) In this urgent search task, as the authors stated in line 724, the variability in performance was mainly driven by the amount of time available for processing the visual cue. The authors used processing time (PT) as the proxy for this "time available for processing the visual cue." But PT itself is already a measure of behavioral variance since it is also determined by the subject's reaction time (i.e., PT = Reaction time (RT) - Gap). In that sense, it seems circular to explain the variability in performance using the variability in PT. I understand the Gap time and PT are correlated (hinted by the RT vs. Gap in Figure 1C), but Gap time seems to be more adequate to use as a proxy for the (imposed) time available for processing the visual cue, which drives the behavioral variance. Can the Gap time better explain some of the results? It would be important to describe how the results are different (or the same) if Gap time was used instead of PT and also discuss why the authors would prefer PT over Gap time (if that's the case).

      Thanks to Rev 1 for requesting clarification of this important point. As Rev 1 notes, PT is a derived variable, computed for each trial by subtracting the Gap interval from RT (PT=RT‒Gap). While it is true that Gap and PT are correlated (inversely), it is precisely because of the variance in RT that Gap alone is not an adequate (or certainly not the best) predictor of choice outcome. First, note that, if the Gap were fixed, there would still be variance in RT and in outcome, and any dependence of outcome on time would be explained necessarily by the PT. This is true at any Gap. So, clearly, the PT predicts outcome in a way that the Gap cannot. It is easy to see why: the Gap is the part of the RT interval during which no cue information is present, whereas the PT is the part of the same interval during which it is. Therefore, if one accepts the logical premise that the likelihood of a correct choice depends on the amount of time available to view the Cue before making that choice (i.e., the definition of PT), it follows that the relationship between PT and performance should be tighter than that between performance and Gap. And, indeed, this is the case. Mean accuracy declines systematically as a function of Gap, as expected, but its correlation with performance is much weaker than for PT.

      Rev 1’s request for a comparison of how accuracy varies as function of PT versus how it varies with Gap has appeared in earlier publications (Stanford et al., 2010; Shankar et al., 2011; Salinas et al., 2014) and we now include it here for the current dataset by adding plots of accuracy versus Gap as a new panel in Fig. 1 (Fig. 1c). That PT (not Gap) better predicts the likelihood of success on a given trial is evident in comparing the tachometric (Fig. 1b) and psychometric curves (Fig. 1c). The tachometric curves vary from chance to asymptotic performance and do so over a short range of PT (~75 ms) with well-defined inflection points identifying key transitions in performance (e.g., from guesses to increasingly informed choices). In contrast, the psychometric function plotting average accuracy versus Gap (Fig. 1c) varies much more gradually, a reduction in temporal definition attributable to the failure to account for the RT’s contribution to determining PT for each trial at a given Gap.

      (2) The authors provide a compelling account of how the urgent search task affords

      (i) more pronounced selection history effects on choice and

      (ii) dissociating the spatial and feature-based history effects by comparing their different effects on the tachometric curves. However, the authors didn't discuss the limits of their task design enough. It is a contrived task (one of the "laboratory tasks"), but the behavioral variability in this simple task is certainly remarkable. Yet, is there any conclusion we should avoid from this study? For instance, can we generalize the finding in more natural settings and say, the spatial selection history influences the choice under time pressure? I wonder whether the task is simple yet general enough to make such a conclusion.

      As Rev. 1 notes, the CO task is a laboratory task that produces large history effects. But importantly, we don't think urgency is causal or essential to the existence of such effects (this is now more explicitly stated in the first section of the Results); it is simply a powerful tool for revealing and characterizing them. As noted in the Discussion, our results are consistent with studies that, based on simpler, non-urgent tasks, demonstrated either reward-driven spatial biases or color priming effects. The CO task uses urgency to generate a psychometric function that time resolves perceptually informed from perceptually uninformed choices, and thereby provides the logical key to disambiguating the simultaneous contributions of perceptual and non-perceptual biases to performance. Such was essential to our demonstration that distinct biases act independently on the same saccade choices.

      In a natural setting, we would certainly expect the respective magnitudes of such non-volitional history-based biases to be highly context dependent, but it would be difficult, if not impossible, to discern their relative impact on natural behavior. That said, we think that the biases revealed by the CO task are exemplary of those that would manifest in natural behaviors depending on the real-world context to which such behaviors correspond. Here, it is important to emphasize that the spatial- and feature-based biases we observed were not strategic, on average neither helping nor hindering overall performance. Thus, in the real-world we might expect the expression of similar biases to be an important source of behavioral variance. These observations are now summarized in the penultimate paragraph of the Discussion.

      (3) Although the authors aimed to look at both inter- and intra-trial temporal dynamics, I'm not sure if the results reflect the true within-trial dynamics. I expected to learn more about how the spatial selection history bias develops as the Gap period progresses (as the authors mentioned in line 386, the spatial history bias must develop during the Gap interval). Does Figure 3 provide some hints in this within-trial temporal dynamics?

      Because it is based on the location of the saccadic choice(s) on previous trial(s), we might expect a signal of spatial bias to be present before and during the Gap period and perhaps even before a trial begins (i.e., intertrial interval). However, because behavioral bias is a probabilistic measure of saccade tendency, we have no way of knowing if such a signal is present during periods devoid of saccadic choices. Note that, for both monkey subjects, average RT exceeded the duration of the longest Gap employed (Fig. 1), and this means that relatively few saccades occurred prior to Cue onset. That said, it's clear in both Figs. 2, 3, and 6 that location bias is evident for saccades initiated at the transition between Gap and Cue intervals (PT=0). Anecdotally, we can report that that spatial bias is evident when we extend our analysis back further into the range of negative PTs (i.e., Gap interval), but the statistics are weak given the paucity of trials at that point. Nevertheless, this is consistent with a bias that exists from the beginning of the trial, as would be expected based on neurophysiological studies from Hikosaka's lab in a simpler but comparable spatial bias task.

      Although our data do not unequivocally identify the temporal origin of the spatial bias, they clearly show that the bias is present early (at short PTs) and diminishes rapidly as the perceptual information accrues (at long PTs). Thus, the PT-dependent temporal dynamics that are revealed clearly suggest that spatial and perceptual biases operate over different intra-trial time frames, one decreasing and the other increasing. As mentioned by Rev. 1, Fig. 3 emphasizes this dichotomy.

      (4) The monkeys show significant lapse rates (enough error trials for further analyses). Do the choices in the error trials reflect the history bias? For example, if errors are divided in terms of PTs, do the errors with short PT reflect more pronounced spatial history bias (choosing the previously selected location) compared to the errors with long PT?

      The short answer is “yes”. Errors generally show a PT-dependent influence of history bias. However, correct and error trials are the result of the same biased dynamics, and analyzing them separately post-hoc does not provide much additional insight about the history effects beyond that provided by the tachometric curves themselves.

      To see this, first consider the figure below (Author response image 1). Two tachometric curves conditioned on color history are shown (left). These are the two extreme curves plotted in Fig. 2a, which correspond to the 4S (i.e., 4 repeats of the current target color) and 4D (4 color repeats and then a switch) conditions. Each of these curves already shows the probability of making an error at each PT but, indeed, we can compare the proportions of correct and error trials at short PTs (guesses) and long PTs (informed choices). These are indicated by the bar graphs on the right. Now, the effect of a bias would be to create a difference in success rate between repetitions (4S, blue) and switches (4D, red) relative to the overall, unbiased expectation (indicated by dotted lines). For color-based history, there is no bias at short PT: the proportions of correct choices are almost exactly at the expected chance level (filled bars coincide with dotted line). In contrast, at long PTs, there is a differential effect, but it is due both to a proportion of correct trials that is higher than expected in the 4S case (filled blue bar above dotted line) and to a proportion of correct trials that is lower than expected in the 4D case (filled orange bar below dotted line). This is exactly as one would expect if the current choice was biased by target color history.

      Author response image 1.

      A similar analysis can be done for location history (Author response image 2, which shows the two extreme curves from Fig. 2e). In this case the bias is much stronger at short PTs, and the difference between repeats (4S, blue) and switches (4D, red) is largely explained by a proportion of correct choices that is much higher than expected by chance in the 4S condition (filled blue bar well above dotted line). This makes sense, because a rewarded location is likely to become the next guess, so if the target happens to appear again at that same location, the subsequent guess is more likely than chance to be correct. At longer PTs, the differential effect is smaller, as would be expected for more informed choices, but it is again driven by the 4S condition. Importantly, in the case of location the total number of S trials is much smaller than the total number of D trials (because a target-location repetition has a probability of 0.25 only), so it only makes sense to compare the proportions of correct (or error) trials, not the absolute numbers, between those conditions.

      Author response image 2.

      In summary, although it is possible to examine the separate dependencies of correct and error trials on history and PT, the distinction is not very useful. Only the frequency of errors relative to that of correct choices makes complete sense, not so much, say, the frequency of short PT errors relative to that of long PT errors.  

      Reviewer #2 (Public review):

      Summary:

      This is a clear and systematic study of trial history influences on the performance of monkeys in a target selection paradigm. The primary contribution of the paper is to add a twist in which the target information is revealed after, rather than before, the cue to make a foveating eye movement. This twist results in a kind of countermanding of an earlier "uninformed" saccade plan by a new one occurring right after the visual information is provided. As with countermanding tasks in general, time now plays a key factor in the success of this task, and it is time that allows the authors to quantitatively assess the parametric influences of things like previous target location, previous target identity, and previous correctness rate on choice performance. The results are logical and consistent with the prior literature, but the authors also highlight novelties in the interpretation of prior-trial effects that they argue are enabled by the use of their paradigm.

      Strengths:

      Careful analysis of a multitude of variables influencing behavior

      Weaknesses:

      Results appear largely confirmatory.

      Recommendations for the authors:

      Reviewer #1 (Recommendations for the authors):

      (1) The authors provide comprehensive accounts of the urgent search task in multiple places in the manuscript. But the description can be simpler and more consistent throughout. I found it confusing when the authors compared their task with previous search tasks used by Bichot and Schall, McPeek et al. I believe the authors wanted to explain that it is not just the urgency but the fact that the target color being randomly interleaved also contributes to the pronounced history bias in their task. I appreciate their thorough comparison with previous studies but it can be distracting or lose focus. It might read better if this statement can be expanded in the Discussion, not in the Results (lines 366-376).

      We thank the reviewer for pointing this out. We agree that the paragraph in question was ambiguous and appeared to elaborate a Discussion point, which was not our intent. Indeed, as the reviewer noted, the main point was that the randomization of the target colors (and not urgency) is the critical aspect of the task that makes it surprisingly difficult for the monkeys. We have revised the paragraph to emphasize this conclusion and the two empirical results from our own data that support it. The agreement with prior studies, which is somewhat tangential, is now briefly mentioned at the end of the paragraph. It should now be clear that the text mainly describes current data that are relevant to the interpretation of the main results.

      (2) It's important to state that feature-based selection history bias is not merely due to the monkey's intrinsic bias to one color over the other (red vs green). The authors did a nice job controlling that, as mentioned in Methods (lines 194-196) and supplementary figure (Figure 1 - Figure Supplement 2). It would be helpful for readers to read in Results as well.

      Thank you for the suggestion. We now mention this in the second section of the Results.

      (3) D trial examples for the location history in Results can be confusing to readers (lines 407-409; left-left-right, up-up-left). The examples in Methods (lines 224-229; left-up-right, up-down-left) are better to convey the preceding (different) trials can be of any kind.

      Indeed. Both types of example are now mentioned in the Results.

      Reviewer #2 (Recommendations for the authors):

      I have only minor comments:

      (1) In the abstract, I'm not sure what "when combined" means in the last sentence. What is combined? Selection history and stimulus salience? If so, this is not very clear. Also, it might be nice to end the abstract on how the study addresses the three components of attention that the abstract started with in the first place (salience, task, and history). Otherwise, I spent multiple abstract reads (before even reading the rest of the paper) trying to see whether indeed the paper addresses the three components of attention that were so prominently described at the beginning of the abstract or not. And, I still could not convince myself of whether all three were addressed by the study or not (I then resorted to proceeding with a reading of the rest of the paper).

      Thanks for pointing this out. We have reworded the abstract to clarify that we are focusing on selection history, not salience or top-down attention.

      (2) Line 72: isn't stimulus location still a feature????

      Our nomenclature here is intended to be consistent with the commonly applied distinction between “spatial” and “feature” -based attention that underscores the distinct mechanistic underpinnings of “where” and “what”.

      (3) Lines 76-79: I'm very confused here. The part about "guesses can be strongly biased toward an arbitrary location early on". However, I expected the later part of the sentence to still stick to location and mention what the temporal dynamic is. Instead, it discusses perceptual bias, which I presume is the color thing. So, the net result is that I'm a bit confused about how *both* location and color behave in *both* early and late times.

      We have rewritten the end of this paragraph to clarify when and how location and feature biases manifest in behavior. It may be useful to note the following. The tachometric curve describes different types of choices distinguished by their timing, guesses at short PTs vs informed decisions at long PTs. However, this also corresponds to the degree to which perceptual information becomes available over time within a single trial. Namely, perceptual information is initially absent but arrives later on. The revised text now reflects this distinction, making the logic for the expected results clearer.

      (4) Last paragraph of the introduction (lines 80-82): it would be helpful to justify here why the psychophysics were done in monkeys in this study, instead of humans.

      We now allude to the reason these studies were done in monkeys but feel that more elaboration of this point is better left to Discussion. The Discussion now more explicitly states that the current data are closely related to neurophysiological studies of spatial attention and color priming in monkeys (beginning of 4th paragraph).

      - Line 389: this kind of formulation is much clearer to me than lines 76-79 mentioned above.

      As noted, the above-mentioned section has been revised.

      - I'm a bit confused by Figure 4 in the sense that some of the effect sizes are not too different from Figure 2, even when there are some intermediate inconsistent trials. I guess the problem is aggravated by the different axis ranges in Figures 2, and 4.

      All the 1S and 1D data points are the same in both figures, as they should, but the problem is that, otherwise, the two figures are just not comparable. Apples and oranges. To see this, note that the trends for the difference between S and D conditions should go in opposite directions as trials go further into the past, and indeed they do. In Figures 2c, f, the differences between 1S and 1D results are small, and those between 4S and 4D results are the largest because both S and D effects grow away from the average with more repetitions. In contrast, in Figure 4b-d, the differences between S and D shrink as the effect of a single trial becomes more distant (differences are largest between 1S and 1D results, smallest between 1S9x and 1D9x results). The only slightly ambiguous trend is that of Figure 2g, because the S data are more noisy. We have expanded the text surrounding Figure 4 to highlight the different expected trends for this analysis in contrast to that presented in Figure 2. This should clarify the qualitative difference between the two.

      - On a related note, it is odd that the summary figures (e.g. Figures. 2, 4, etc) are vertically aligned such that the dependent measure is on the x-axis rather than the y-axis. For example, looking at Figure 2, it would make much more sense if panels b-d and f-h were rotated by 90 deg, such that the vertical axis is indeed the low asymptote or high asymptote or RT. This would directly correlate with the same data in panels a and e in the same figure and would be much easier to follow. Then, later in the paper, Fig. 8 suddenly does the dependent measure on the y-axis, as I said. I think it can help to use similarly consistent plotting approaches across all (or most) analyses.

      We tried other formats but settled on the current one because we felt it made it (slightly) easier to compare the patterns across history conditions between any two of the 6 bar graphs in each figure (in Figs 2, 5, 6), in part because it prevents any confusion with the PT axes. As this does not make a substantial difference either way, we prefer to maintain the present arrangement. Additional labels are now included, which should make the figures a bit more friendly.

      - At the beginning of the paper, I was under the impression that this will really be a free viewing search task (e.g. Wolfe search arrays or old Nakayama search arrays), but then it became clear later that it was still an instructed task, with the only difference being that the target onset is now 4 targets. I think this distinction should be clarified very early on, in order to avoid confusion by the readers. The reason I say this is that with enforced fixation, there are other factors in this task that come into play, like the monkey's individual microsaccade rates etc, which can modulate performance since they also have a form of countermanding that is like the one imposed by the compelled saccade task. So, better alert the readers to the context of the task early on.

      Thanks. We have provided additional detail when introducing the task for the first time in the Introduction, along with a citation to an earlier publication in which the specific task is described. There should be no ambiguity now.

      Reviewing Editor Comments:

      Short Assessment:

      This important study makes compelling use of the monkey animal model to capture the long-time course over which trial history affects decision-making under time pressure, showing decisions are affected by the stimulus sequence extending back as many as four trials previously.

      Summary:

      Decision-making is variable, but how much of this variability can be accounted for by the immediate previous history is not well known. Using an "urgent" saccade, Oor et al manipulated how much time monkeys had to process evidence, and evaluated what they did when there was too little time to make an evidence-based decision. They report that the history affected performance as far back as 4 previous trials and that different aspects of the stimulus history (color and location) affected performance differently.

      Strengths:

      The key strengths of this paper are that the monkey paradigm permitted a study under highly controlled conditions with stable performance across sessions and enough trials to conduct the history analysis farther back in time than is possible with smaller data sets. While the fact that prior history affects decisions was previously known, this study provides a careful quantification of the effect -- which proves to be quite large - as well as an assessment of both location and feature histories in combination with each other. The manuscript is well-written and easy to follow.

      Weaknesses and recommendations for the authors:

      (1) The figures are lovely but could use some more text/design elements to clarify, and there is space to do so. e.g., in Figure 2, there could be titles to indicate that the top row involves the color history and the bottom row involves location history. The information is there, in the y labels of panels B and F, but it takes a while to see that.

      Done. Titles have been added to Figure 2 and several others.

      (2) Furthermore, the abbreviations 1D, 4S, etc are explained in the legend but it seems there is room to spell them out or include a graphic to indicate what they mean.

      The labels 1D, 4S, etc are difficult to spell out because each one represents multiple conditions; for instance, 2S may correspond to green-green or red-red target colors, and so on. Figure legends have been edited to more clearly indicate that S and D labels correspond to repeat and switch trials, respectively, and that the associated number indicates how far back the history goes.

      (3) The terms "low asymptote" and "high asymptote" could be indicated in a graphic of a tachymetric function, smoothing the transition to the rightmost panels. (Consider also alternative terms - perhaps "floor" and "ceiling" might be more readily understandable than asymptote to the student reader??).

      Thanks for the suggested terms, “floor” and “ceiling”, which we’ve adopted. They are indeed more natural. Figure 2a now indicates that floor and ceiling accuracies correspond to opposite ends of the PT axis.

      (4) The units for the asymptotes are not indicated - I assume these are "% correct" but that would be helpful to clarify.

      Yes. Units for floor and ceiling (and RT) are now indicated in all figures.

      (5) Figure 3 - "PT", and "1S-1D" could be spelled out, and the meaning of the two colored traces could be in the figure itself rather than only in the legend. Similar suggestions apply about labeling, abbreviations apply in subsequent figures.

      PT is now spelled out in all figures other than Figure 1, and labels for the two traces were added to Figure 3. Thanks for all the detailed suggestions.

    1. Author response:

      The following is the authors’ response to the original reviews

      Public Reviews:

      Reviewer #1 (Public review):

      This study provides a thorough analysis of Nup107's role in Drosophila metamorphosis, demonstrating that its depletion leads to developmental arrest at the third larval instar stage due to disruptions in ecdysone biosynthesis and EcR signaling. Importantly, the authors establish a novel connection between Nup107 and Torso receptor expression, linking it to the hormonal cascade regulating pupariation.

      However, some contradictory results weaken the conclusions of the study. The authors claim that Nup107 is involved in the translocation of EcR from the cytoplasm to the nucleus. However, the evidence provided in the paper suggests it more likely regulates EcR expression positively, as EcR is undetectable in Nup107-depleted animals, even below background levels.

      We appreciate the concern raised in this public review. However, we must clarify that we do not claim that Nup107 directly regulates the translocation of EcR from the cytoplasm to nucleus, rather Nup107 regulates Ecdysone hormone (20E) synthesis which in turn affects EcR translocation. In the manuscript, we posited this hypothesis if Nup107 will regulate EcR nuclear translocation (9th line of 2nd paragraph on page 6). We have spelled this out more clearly as the 3rd subsection title of the Results section, and in the discussion (8th line of 2nd paragraph on page 11).

      20E acts through the EcR to induce the transcription of EcR responsive genes including the EcR. This creates a positive autoregulatory loop that enhances the EcR level through ecdysone signaling (1). Since Nup107 depletion leads to a reduction in ecdysone levels, it disrupts the transcription autoregulatory EcR expression loop. This can contribute to the reduced EcR levels seen in Nup107-depleted animals. 

      Additionally, the link between Nup107 and Torso is not fully substantiated. While overexpression of Torso appears to rescue the lack of 20E production in the prothoracic gland, the distinct phenotypes of Torso and Nup107 depletion-developmental delay in the former versus complete larval arrest in the latter complicate understanding of Nup107's precise role.

      We understand that there are differences in the developmental delay when Tosro and Nup107 depletion is analyzed. However, the two molecules being compared here are very different, and variability in their depletion could contribute observed phenotypic differences (2). Even if there is no variability of depletion of Torso and Nup107­­­, we believe that Nup107, being more widely expressed, and involved in the regulation of various cellular processes, induces stronger defects.

      Further, we think that RNAi-mediated depletion of Nup107 in prothoracic glands (PG) causes significant reduction in the PG size, which may exert a pronounced defect in 20E biosynthesis through the Halloween genes, inducing a stronger developmental arrest.

      To clarify these discrepancies, further investigation into whether Nup107 interacts with other critical signaling pathways related to the regulation of ecdysone biosynthesis, such as EGFR or TGF-β, would be beneficial and could strengthen the findings.

      In summary, although the study presents some intriguing observations, several conclusions are not well-supported by the experimental data.

      We agree with the reviewer’s suggestion. As noted in the literature, five RTKs-torso, InR, EGFR, Alk, and Pvr-stimulate the PI3K/Akt pathway, which plays a crucial role in the PG functioning and controlling pupariation and body size (3). We have checked the torso and EGFR signaling. We rescued Nup107 defects with the torso overexpression, however, constitutively active EGFR (BL-59843) did not rescue the phenotype (data was not shown). Nonetheless, we plan to examine the EGFR pathway activation by measuring the pERK levels in Nup107-depleted PGs.

      Reviewer #2 (Public review):

      Summary:

      The manuscript by Kawadkar et al investigates the role of Nup107 in developmental progression via the regulation of ecdysone signaling. The authors identify an interesting phenotype of Nup107 whole-body RNAi depletion in Drosophila development - developmental arrest at the late larval stage. Nup107-depleted larvae exhibit mis-localization of the Ecdysone receptor (EcR) from the nucleus to the cytoplasm and reduced expression of EcR target genes in salivary glands, indicative of compromised ecdysone signaling. This mis-localization of EcR in salivary glands was phenocopied when Nup107 was depleted only in the prothoracic gland (PG), suggesting that it is not nuclear transport of EcR but the presence of ecdysone (normally secreted from PG) that is affected. Consistently, whole-body levels of ecdysone were shown to be reduced in Nup107 KD, particularly at the late third instar stage when a spike in ecdysone normally occurs. Importantly, the authors could rescue the developmental arrest and EcR mislocalization phenotypes of Nup107 KD by adding exogenous ecdysone, supporting the notion that Nup107 depletion disrupts biosynthesis of ecdysone, which arrests normal development. Additionally, they found that rescue of the Nup107 KD phenotype can also be achieved by over-expression of the receptor tyrosine kinase torso, which is thought to be the upstream regulator of ecdysone synthesis in the PG. Transcript levels of the torso are also shown to be downregulated in the Nup107KD, as are transcript levels of multiple ecdysone biosynthesis genes. Together, these experiments reveal a new role of Nup107 or nuclear pore levels in hormone-driven developmental progression, likely via regulation of levels of torso and torso-stimulated ecdysone biosynthesis.

      Strengths:

      The developmental phenotypes of an NPC component presented in the manuscript are striking and novel, and the data appears to be of high quality. The rescue experiments are particularly significant, providing strong evidence that Nup107 functions upstream of torso and ecdysone levels in the regulation of developmental timing and progression.

      Weaknesses:

      The underlying mechanism is however not clear, and any insight into how Nup107 may regulate these pathways would greatly strengthen the manuscript. Some suggestions to address this are detailed below.

      Major questions:

      (1) Determining how specific this phenotype is to Nup107 vs. to reduced NPC levels overall would give some mechanistic insight. Does knocking down other components of the Nup107 subcomplex (the Y-complex) lead to similar phenotypes? Given the published gene regulatory function of Nup107, do other gene regulatory Nups such as Nup98 or Nup153 produce these phenotypes?

      We thank this public review for raising this concern. Working with a Nup-complex like the Nup107 complex, this concern is anticipated but difficult to address as many Nups function beyond their complex identity. Our observations with all other members of the Nup107-complex, including dELYS, suggest that except Nup107, none of the other tested Nup107-complex members could induce larval developmental arrest.

      In this study, we primarily focused on the Nup107 complex (outer ring complex) of the NPC. However, previous studies have reported that Nup98 and Nup153 interact with chromatin, with these investigations conducted in Drosophila S2 cells (4, 5, 6). We have now examined other nucleoporins outside of this complex, such as Nup153.

      We ubiquitously depleted Nup153 using the Actin5C-Gal4 driver and assessed the pupariation profile of the knockdown larvae in comparison to control larvae. In contrast to the Nup107 knockdown, when Nup153 is depleted to less than 50% levels, no impact on pupariation was observed (Auhtor response image 1)

      Author response image 1.

      Nup153 depletion does not affect the Drosophila metamorphosis. Actin5C-Gal4 is used as a ubiquitous driver. (A) Comparison of pupariation profiles of control and Nup153 knockdown organisms. (B) Quantification of Nup153 knockdown efficiency. Data are represented from at least three independent experiments. Statistical significance was derived from the Student’s t-test. Error bars represents SEM. ***p = <0.001.

      (2) In a related issue, does this level of Nup107 KD produce lower NPC levels? It is expected to, but actual quantification of nuclear pores in Nup107-depleted tissues should be added. These and the above experiments would help address a key mechanistic question - is this phenotype the result of lower numbers of nuclear pores or specifically of Nup107?

      We agree with the concern raised here, and to address the concern raised here, we stained the control and Nup107 depleted salivary glands with mAb414 antibody (exclusively FG-repeat Nup recognizing antibody). While Nup107 intensities are significantly reduced at the nuclear envelope in Nup107 depleted salivary glands, the mAb414 staining seems unperturbed (Author response image 2).

      Author response image 2.

      Nup107 depletion does not perturb overall NPC composition. Comparison of salivary gland nucleus upon control and Nup107 knockdown. The Nup107 is shown in green and mAb414, staining for other FG-repeat containing nucleoporins is shown in red. Scale bars, 5µm.

      (3) Additional experiments on how Nup107 regulates the torso would provide further insight. Does Nup107 regulate transcription of the torso or perhaps its mRNA export? Looking at nascent levels of the torso transcript and the localization of its mRNA can help answer this question. Or alternatively, does Nup107 physically bind the torso?

      While the concern regarding torso transcript level is genuine, we have already reported in the manuscript that Nup107 directly regulates torso expression. When Nup107 is depleted, torso levels go down, which in turn controls ecdysone production and subsequent EcR signaling (Figure 6B of the manuscript).

      However, the exact nature of Nup107 regulation on torso expression is still unclear. Since the Nup107 is known to interact with chromatin (7), it may affect torso transcription. The possibility of a stable and physiologically relevant interaction between Nup107 and the torso in a cellular context is unlikely largely due to their distinct subcellular localizations. If we investigate this further, it will require a significant amount of time for having reagents and experimentation, and currently stands beyond the scope of this manuscript.

      (4) The depletion level of Nup107 RNAi specifically in the salivary gland vs. the prothoracic gland should be compared by RT-qPCR or western blotting.

      Although we know that the Nup107 protein signal is reduced in SG upon knockdown (Figure 3B), we have not compared the Nup107 transcript level in these two tissues (SG and PG) upon RNAi. As suggested here, we evaluated the knockdown efficiency of Nup107 using the salivary gland-specific driver AB1-Gal4 and the prothoracic gland-specific driver Phm-Gal4. Our results indicate a significant reduction in Nup107 transcript levels upon Nup107 RNAi in both SG and PG compared to their respective controls (Author response image 3).

      Author response image 3.

      Nup107 levels are significantly reduced upon Nup107<sup>KK</sup> RNAi. Quantification of Nup107 transcript levels from control and Nup107 depleted larvae [tissue specific depletion using AB1-Gal4 (A) and Phm-Gal4 (B)]. Data are represented from at least three independent experiments. Statistical significance was derived from the Student’s t-test. Error bars represent SEM. **p = <0.004

      (5) The UAS-torso rescue experiment should also include the control of an additional UAS construct - so Nup107; UAS-control vs Nup107; UAS-torso should be compared in the context of rescue to make sure the Gal4 driver is functioning at similar levels in the rescue experiment.

      This is a very valid point, and we took this into account while planning the experiment. In such cases, often the GAL4 dilution can be critical. We have demonstrated in Figure S7, that GAL4 dilution is not blurring our observations. We used the Nup107<sup>KK</sup>; UAS-GFP as control alongside the Nup107<sup>KK</sup>; UAS-torso. We conclude that the presence of GFP signals in prothoracic glands and their reduced size indicates genes downstream to both UAS sequences are transcribed, and GAL4 dilution does not play a role here.

      Minor:

      (6) Figures and figure legends can stand to be more explicit and detailed, respectively.

      We have revisited all figures and their corresponding legends to ensure appropriate and explicit details are provided.

      Reviewer #3 (Public review):

      Summary:

      In this study by Kawadkar et al, the authors investigate the developmental role of Nup107, a nucleoporin, in regulating the larval-to-pupal transition in Drosophila through RNAi knockdown and CRISPR-Cas9-mediated gene editing. They demonstrate that Nup107, an essential component of the nuclear pore complex (NPC), is crucial for regulating ecdysone signaling during developmental transitions. The authors show that the depletion of Nup107 disrupts these processes, offering valuable insights into its role in development.

      Specifically, they find that:

      (1) Nup107 depletion impairs pupariation during the larval-to-pupal transition.

      (2) RNAi knockdown of Nup107 results in defects in EcR nuclear translocation, a key regulator of ecdysone signaling.

      (3) Exogenous 20-hydroxyecdysone (20E) rescues pupariation blocks, but rescued pupae fail to close.

      (4) Nup107 RNAi-induced defects can be rescued by activation of the MAP kinase pathway.

      Strengths:

      The manuscript provides strong evidence that Nup107, a component of the nuclear pore complex (NPC), plays a crucial role in regulating the larval-to-pupal transition in Drosophila, particularly in ecdysone signaling.

      The authors employ a combination of RNAi knockdown, CRISPR-Cas9 gene editing, and rescue experiments, offering a comprehensive approach to studying Nup107's developmental function.

      The study effectively connects Nup107 to ecdysone signaling, a key regulator of developmental transitions, offering novel insights into the molecular mechanisms controlling metamorphosis.

      The use of exogenous 20-hydroxyecdysone (20E) and activation of the MAP kinase pathway provides a strong mechanistic perspective, suggesting that Nup107 may influence EcR signaling and ecdysone biosynthesis.

      Weaknesses:

      The authors do not sufficiently address the potential off-target effects of RNAi, which could impact the validity of their findings. Alternative approaches, such as heterozygous or clonal studies, could help confirm the specificity of the observed phenotypes.

      This is a very valid point raised, and we are aware of the consequences of the off-target effects of RNAi. To assert the effects of authentic RNAi and reduce the off-target effects, we have used two RNAi lines (Nup107<sup>GD</sup> and Nup107<sup>KK</sup>) against Nup107. Both RNAi induced comparable levels of Nup107 reduction, and using these lines, ubiquitous and PG specific knockdown produced similar phenotypes. Although the Nup107<sup>GD</sup> line exhibited a relatively stronger knockdown compared to the Nup107<sup>KK</sup> line, we preferentially used the Nup107<sup>KK</sup> line because the Nup107<sup>GD</sup> line is based on the P-element insertion, and the exact landing site is unknown. Furthermore, there is an off-target predicted for the Nup107<sup>GD</sup> line, where a 19bp sequence aligns with the bifocal (bif) sequence. The bif-encoded protein is involved in axon guidance and regulation of axon extension. However, the Nup107<sup>KK</sup> line does not have a predicted off-target molecule, and we know its precise landing site on the second chromosome. Thus, the Nup107<sup>KK</sup> line was ultimately used in experimentation for its clearer and more reliable genetic background.

      We are also investigating Nup107 knockdown in the prothoracic gland, which exhibits polyteny. Additionally, the number of cells in the prothoracic gland is quite limited, approximately 50-60 cells (8). Given this, there is a possibility that a clonal study may not yield the phenotype.

      NPC Complex Specificity: While the authors focus on Nup107, it remains unclear whether the observed defects are specific to this nucleoporin or if other NPC components also contribute to similar defects. Demonstrating similar results with other NPC components would strengthen their claims.

      We thank this public review for raising this concern. Working with a Nup-complex like the Nup107 complex, this concern is anticipated but difficult to address as many Nups function beyond their complex identity. Our observations with all other members of the Nup107-complex, including dELYS, suggest that except Nup107, none of the other Nup107-complex members could induce larval developmental arrest. Since the study is primarily focused on the Nup107 complex (outer ring complex) of the NPC, we have not examined many more nucleoporins outside of this complex. But our observations with Nup153 knockdown, a nuclear basket nucleoporin, is comparable to control, with no delay in development (Author response image 1)

      Although the authors show that Nup107 depletion disrupts EcR signaling, the precise molecular mechanism by which Nup107 influences this process is not fully explored. Further investigation into how Nup107 regulates EcR nuclear translocation or ecdysone biosynthesis would improve the clarity of the findings.

      We appreciate the concern raised. Through our observation, we have proposed the upstream effect of Nup107 on the PTTH-torso-20E-EcR axis regulating developmental transitions. We know that Nup107 regulates torso levels, but we do not know if Nup107 directly interacts with torso. We would like to address whether Nup107 exerts control on PTTH levels also.

      However, we must emphasize that Nup107 does not directly regulate the translocation of EcR. On the contrary, we have demonstrated that when Nup107 is depleted only in the salivary gland, EcR translocates into the nucleus. Thus we conclude that the EcR translocation is 20E dependent and Nup107 independent. Further, we have argued that Nup107 regulates the expression of Halloween genes required for ecdysone biosynthesis. We are interested in identifying if Nup107 associates directly or through some protein to chromatin to bring about the changes in gene expression required for normal development.

      There are some typographical errors and overly strong phrases, such as "unequivocally demonstrate," which could be softened. Additionally, the presentation of redundant data in different tissues could be streamlined to enhance clarity and flow.

      Response: We thank the reviewer for this observation. We have put our best efforts to remove all typographical errors and have now made more reasonable statements based on our conclusions.

      Recommendations for the Authors:

      Reviewer #1 (Recommendations for the authors):

      The manuscript presents compelling evidence that Nup107 plays a role in regulating ecdysone production. However, significant concerns remain regarding the effects on EcR localization and expression, as well as the claimed link between PTTH/Torso signaling and Nup107's function, as the evidence provided is not conclusive.

      The hypothesis that Nup107 mediates EcR translocation from the cytoplasm to the nucleus appears misinterpreted by the authors. Based on the presented images, particularly for the prothoracic gland (PG) Figure 3C, Nup107 depletion seems to impact EcR protein levels rather than its localization. This conclusion is supported by data showing that EcR transcripts are autonomously downregulated in the absence of Nup107. Furthermore, the restoration of nuclear EcR levels upon exogenous 20E supplementation suggests that (1) Nup107 is dispensable for EcR activation and function, and (2) its primary role lies in regulating ecdysone production.

      We appreciate the concern raised by reviewer. However, we must clarify that we do not claim that Nup107 directly regulates the translocation of EcR from the cytoplasm, rather Nup107 regulates Ecdysone hormone (20E) synthesis which in turn affects EcR translocation. In the manuscript, we posited this hypothesis if Nup107 will regulate EcR nuclear translocation (9th line of 2nd paragraph on page 6). We have spelled this out more clearly as the 3rd subsection title of the Results section, and in the discussion (8th line of 2nd paragraph on page 11).

      20E acts through the EcR to induce the transcription of EcR responsive genes including the EcR. This creates a positive autoregulatory loop that enhances the EcR level through ecdysone signaling (1). Since Nup107 depletion leads to a reduction in ecdysone levels, it disrupts the transcription autoregulatory EcR expression loop. This can contribute to the reduced EcR levels seen in Nup107-depleted animals.

      Given that nucleoporins are known to influence mRNA transport-for instance, Nup107 has been shown to control Scn5a mRNA transport (Guan et al., 2019)-the observed effects on Halloween gene and EcR expression may stem from disruptions in mRNA transport to the cytoplasm. The downregulation of Shade further supports this hypothesis, as restricted ecdysone biosynthesis typically induces Shade upregulation in peripheral tissues. Quantifying potential mRNA accumulation in the nuclei of PG cells in Nup107-depleted animals would clarify this.

      The reviewer raised a valid point, and we fully agree with the concern that Nup107 has been shown to control Scn5a mRNA transport (Guan et al., 2019). The observed effects on Halloween gene and EcR expression could indeed stem from disruptions in efficient mRNA export to the cytoplasm. However, if Nup107 were regulating the mRNA export of Halloween genes and EcR, we should not expect a rescue of the Nup107 developmental delay phenotype with torso overexpression. But, by overexpressing the torso in the Nup107 depletion background, we are activating the torso pathway dependent Halloween gene expression, and rescuing the developmental delay phenotype of Nup107 depletion.

      With the current data, it is difficult to conclusively claim a role for Nup107 in EcR translocation or expression. Additional experiments, such as EcR overexpression in Nup107-depleted animals or Nup107 overexpression, would help determine its precise role.

      We appreciate the concern raised by reviewer. We did attempt to rescue the Nup107 depletion phenotype by overexpressing EcR (BL-6868) in the Nup107-RNAi background. However, we were unable to rescue the Nup107 depletion dependent developmental delay phenotype with this approach. This further suggests that the phenotype is not merely due to low level of EcR, but it is due to low availability of ecdysone hormone and EcR signaling.

      The second major issue is the proposed link between Nup107 and PTTH/Torso signaling. The authors suggest that Nup107 regulates ecdysone production through Torso expression based on rescue experiments. However, this is inconsistent with the distinct phenotypes observed when Nup107 or Torso signaling is disrupted. While PTTH/Torso signaling causes only a modest developmental delay (12 hours to 2 days, depending on the mutant), Nup107 depletion results in a complete developmental arrest at the larval stage. This discrepancy raises doubts about the assertion that Torso overexpression alone rescues such a severe phenotype. One possibility is that PTTH levels are upregulated in Nup107-depleted animals, leading to overactivation of the pathway when Torso is overexpressed. Quantifying PTTH levels in Nup107-depleted animals could address this.

      The reviewer raised a valid point, and we fully acknowledge this concern. While we do not completely agree with the idea of PTTH upregulation in Nup107 depleted larvae, as suggested here, we believe that quantifying PTTH levels upon Nup107 depletion can provide a useful insight. To address it, we quantified PTTH levels in Nup107-depleted larvae and found no significant change in PTTH expression compared to controls (Author response image 4).

      Author response image 4.

      Nup107 knockdown does not affect the PTTH level. Quantitation of PTTH transcript levels from control and Nup107 depleted larvae (Prothoracic specific depletion Phm-Gal4). Data are represented from at least three independent experiments. Statistical significance was derived from the Student's t-test. ns is non-significant.

      Another possibility is that the stock used for Torso overexpression, which includes a trk mutant, may introduce genetic interactions that overactivate the pathway. Using a clean UAS-Torso stock would resolve this issue.

      We appreciate the reviewer’s observation regarding the use of the Torso overexpression line (BL-92604), which carries the trk null allele on the second chromosome. The cleaved form of the trk serves as ligand for the troso receptor. Since it may serve as ligand for the torso, I am not sure how trk null allele bearing line when used along for torso overexpression studies will overactivate the pathway. 

      We realized this concern and the fly line used in this study and reported in the manuscript was generated through the following genetic strategy using the BL-92604 line.  First, a double balancer stock (Sco/CyO; MKRS/TM6.Tb) was used to generate the Sco/CyO; UAS-torso/ UAS-torso genotype. This recombinant line was subsequently combined with the Nup107<sup>KK</sup> line. Through the use of the double balancer strategy, we effectively replaced Nup107 RNAi genotype on the second chromosome, thereby ensuring that our final experimental setup is free from trk mutant contamination, if at all.

      Moreover, the rescue of Nup107 depletion phenotypes by RasV12 overexpression suggests that multiple RTKs, not just Torso, are affected. EGFR signaling, the primary regulator of ecdysone biosynthesis in the PG during the last larval stage, is notably absent from the authors' analysis. EGFR inactivation is known to arrest development, and previous studies indicate that Nup107 can reduce EGFR pathway activity (Kim et al, 2010). The authors should analyze EGFR pathway activity in the absence of Nup107. Overexpressing EGF ligands like Vein or Spitz in the PG (rather than the receptor) in a Nup107-depleted background would provide more relevant insights.

      The RasGTPase is one of the common effector molecules downstream of an activated receptor kinase. Rescue with a constitutively activated form of RasGTPase (RasV12) suggests one of the routes which is activated downstream of the torso receptor. It does not directly suggest all different RTKs are affected and are involved. Our idea of performing a rescue experiment was to see if the pathway activated downstream of the torso involves RasGTPase. 

      As noted in the literature, five RTKs—torso, InR, EGFR, Alk, and Pvr—stimulate the PI3K/Akt pathway, which plays a crucial role in the PG for controlling pupariation and body size (3). Although EGFR signaling is important, PTTH/Torso signaling is considered the primary mediator of metamorphic timing. In response to the suggestion to analyze EGFR pathway activity in the absence of Nup107, we attempted to rescue the phenotype by overexpressing constitutively active EGFR (BL-59843) in the Nup107-depleted background (data was not shown). We used constitutively active EGFR to bypass the availability of its ligands (vein and spitz). Unfortunately, we were unable to rescue the phenotype with this approach, which further suggests that EGFR is not the targeted RTK pathway in this context. By rescuing with torso, we found that Nup107 regulates torso-mediated Ras/Erk signaling to control metamorphosis.

      Additional issues require clarification:

      (1) RNAi Efficiency: In Figure 1C, the Nup107GD line shows a stronger knockdown effect than Nup107KK, yet most experiments were conducted with the weaker line. This might explain the residual Nup107 protein observed in Figure 2. Could the authors justify this choice?

      This is a very valid point raised, and we are aware of the consequences of the off-target effects of RNAi. To assert the effects of authentic RNAi and reduce the off-target effects, we have used two RNAi lines (Nup107<sup>GD</sup> and Nup107<sup>KK</sup>) against Nup107. Both RNAi induced comparable levels of Nup107 reduction, and using these lines, ubiquitous and PG specific knockdown produced similar phenotypes. Although the Nup107<sup>GD</sup> line exhibited a relatively stronger knockdown compared to the Nup107<sup>KK</sup> line, we preferentially used the Nup107<sup>KK</sup> line because the Nup107<sup>GD</sup> line is based on the P-element insertion, and the exact landing site is unknown. Furthermore, there is an off-target predicted for the Nup107<sup>GD</sup> line, where a 19bp sequence aligns with the bifocal (bif) sequence. The bif-encoded protein is involved in axon guidance and regulation of axon extension. However, the Nup107<sup>KK</sup> line does not have a predicted off-target molecule, and we know its precise landing site on the second chromosome. Thus, the Nup107<sup>KK</sup> line was ultimately used in experimentation for its clearer and more reliable genetic background.

      (2) Control Comparisons: In Figure 3, the effects of Nup107 depletion on EcR expression in salivary glands (SG) and PG are shown, but only SG controls are provided. Including PG controls would enable proper comparisons. These controls should also be added to Figures 5, 6, and S5.

      As suggested by the reviewer, we have checked the EcR localization in prothoracic gland (Author response image 5), also. As shown in figure R5, when PGs isolated from control, Nup107-RNAi and torso overexpression in Nup107 background were stained for EcR, the observations made were indistinguishable from those made in SGs of the indicated genetic combinations. This indicated that Nup107 regulates EcR signaling by regulating the 20E biosynthesis.

      Author response image 5.

      Prothoracic gland’s specific torso expression rescues EcR nuclear translocation defects. Immunofluorescence-based detection of nucleocytoplasmic distribution of EcR (EcR antibody, red) in control, prothoracic gland specific Nup107 knockdown (Phm-Gal4>Nup107<sup>KK</sup>) and torso overexpressing PG-specific Nup107 knockdown (Phm-Gal4>Nup107<sup>KK</sup>; UAS-torso) third instar larval Prothoracic gland nuclei. DNA is stained with DAPI. Scale bars, 20 μm.

      (3) Clarify the function of Torso in the text: The authors must revise their description of Torso signaling as the primary regulator of ecdysone production in both the results and discussion sections. Specifically, in the results section, the claim that Torso depletion induces developmental arrest is inaccurate. Instead, available evidence, including Rewitz et al. 2009, demonstrates that Torso depletion causes a delay of approximately five days rather than a complete developmental arrest. This discrepancy should be corrected to avoid overstating the role of Torso signaling in ecdysone regulation and to align the manuscript with established findings.

      We agree with the reviewer. We have incorporated the suggestion at the relevant place in the main manuscript.

      Reviewer #3 (Recommendations for the authors):

      These findings suggest that Nup107 is involved in regulating ecdysone signaling during developmental transitions, with depletion of Nup107 disrupting hormone-regulated processes. Moreover, the rescue experiments hint that Nup107 might directly influence EcR signaling and ecdysone biosynthesis, though the precise molecular mechanism remains unclear.

      Overall, the manuscript presents compelling data supporting Nup107's role in regulating developmental transitions. However, I have a few comments for consideration:

      Major Comments:

      RNAi Specificity: While RNAi is a powerful tool, the authors do not sufficiently address potential off-target effects, which could undermine the conclusions. Although a mutant Nup107 is described, it is lethal-are heterozygous or clonal studies possible to validate the findings more robustly?

      This is a very valid point raised, and we are aware of the consequences of the off-target effects of RNAi. To assert the effects of authentic RNAi and reduce the off-target effects, we have used two RNAi lines (Nup107<sup>GD</sup> and Nup107<sup>KK</sup>) against Nup107. Both RNAi induced comparable levels of Nup107 reduction, and using these lines, ubiquitous and PG specific knockdown produced similar phenotypes. Although the Nup107<sup>GD</sup> line exhibited a relatively stronger knockdown compared to the Nup107<sup>KK</sup> line, we preferentially used the Nup107<sup>KK</sup> line because the Nup107<sup>GD</sup> line is based on the P-element insertion, and the exact landing site is unknown. Furthermore, there is an off-target predicted for the Nup107<sup>GD</sup> line, where a 19bp sequence aligns with the bifocal (bif) sequence. The bif-encoded protein is involved in axon guidance and regulation of axon extension. However, the Nup107<sup>KK</sup> line does not have a predicted off-target molecule, and we know its precise landing site on the second chromosome. Thus, the Nup107<sup>KK</sup> line was ultimately used in experimentation for its clearer and more reliable genetic background.

      Following the suggestion from the reviewer, we considered conducting heterozygous and clonal analyses using the Nup107 mutant. We have carried out Nup107 knockdown studies in the prothoracic gland, which has a limited number of cells (50-60 cells) and is known to exhibit polyteny (8). Keeping these aspects of the Prothoracic gland in mind, the possibility that a clonal study will yield the phenotype is scarce. However, we will consider moving forward with this approach also.

      (2) NPC Complex Specificity: It remains unclear whether the observed defects are specific to Nup107 or if other NPC components also cause similar defects. If the authors are unable to use Nup107 mutants, they could demonstrate similar defects with other critical NPC members to bolster their claim.

      We thank this public review for raising this concern. Working with a Nup-complex like the Nup107 complex, this concern is anticipated but difficult to address as many Nups function beyond their complex identity. Our analysis of Nup153 depleted organisms indicates no developmental delay/defect. We have also assessed effects of knockdown of all other members of the Nup107-complex, including dELYS, but except Nup107 no other member of the Nup107-complex could induce developmental arrest in the third instar stage causing lack of pupariation. However, the null mutant of Nup133, the direct interactor of Nup107 in the Nup107-complex, induces a delay in pupariation (unpublished data).

      (3) Molecular Mechanism of EcR Signaling: The manuscript shows that Nup107 depletion affects EcR signaling and ecdysone biosynthesis, but the molecular basis of this regulation is not fully explored. Does phosphorylated ERK (p-ERK) fail to enter the nucleus? Clarifying this mechanism would strengthen the study's impact.

      We appreciate the reviewer’s insightful comment and fully agree with the concern. To address this, we examined the subcellular localization of phosphorylated ERK (p-ERK) in the prothoracic gland of control larvae, Nup107-depleted larvae, and Nup107-depleted larvae with torso overexpression. In control larvae, p-ERK was predominantly localized in the nucleus. However, in Nup107-depleted larvae, p-ERK was largely retained in the cytoplasm, indicating impaired pathway activation and nuclear translocation. Notably, overexpression of the torso in the Nup107-depleted background restored nuclear localization of p-ERK in the prothoracic gland (Author response image 6). These findings suggest that Nup107 regulates Drosophila metamorphosis, in part, through modulation of torso-mediated MAPK signaling.

      Author response image 6.

      Nup107 regulates torso activation dependent p-ERK localization. Detection of nucleocytoplasmic distribution of p-ERK (anti- p-ERK antibody, green) in the third instar larval prothoracic glands of control, PG-specific Nup107 knockdown (Phm-Gal4>Nup107<sup>KK</sup>) and PG-specific torso overexpression in Nup107 knockdown background (Phm-Gal4>Nup107<sup>KK</sup>; UAS-torso). DNA is stained with DAPI. Scale bars, 20 µm.

      Minor Comments:

      (1) The manuscript contains typographical errors that may hinder readability. Additionally, some phrases (e.g., "unequivocally demonstrate") may be overly strong. Consider adjusting language to reflect the nature of the data more accurately.

      We agree with the reviewer. We have edited the manuscript accordingly to crease out such typographical errors at relevant places in the main manuscript.

      (2) The data presentation could be improved by eliminating redundancy. Some sections repeat similar findings in different tissues, which could be consolidated to improve clarity and flow.

      While we agree with the comment, we could not help ourselves in tissue redundancy for presenting our data for EcR translocation studies. I wish we could use another tissue. However, we have put EcR localization and p-ERK translocation data in the responses to present another non-redundant tissue perspective (Figures R5 and R6).

      References:

      (1) Varghese, Jishy, and Stephen M Cohen. “microRNA miR-14 acts to modulate a positive autoregulatory loop controlling steroid hormone signaling in Drosophila.” Genes & development vol. 21,18 (2007): 2277-82. doi:10.1101/gad.439807

      (2) Rewitz, Kim F et al. “The insect neuropeptide PTTH activates receptor tyrosine kinase torso to initiate metamorphosis.” Science (New York, N.Y.) vol. 326,5958 (2009): 1403-5. doi:10.1126/science.1176450

      (3) Pan, Xueyang, and Michael B O'Connor. “Coordination among multiple receptor tyrosine kinase signals controls Drosophila developmental timing and body size.” Cell reports vol. 36,9 (2021): 109644. doi:10.1016/j.celrep.2021.109644

      (4) Pascual-Garcia, Pau et al. “Metazoan Nuclear Pores Provide a Scaffold for Poised Genes and Mediate Induced Enhancer-Promoter Contacts.” Molecular cell vol. 66,1 (2017): 63-76.e6. doi:10.1016/j.molcel.2017.02.020

      (5) Pascual-Garcia, Pau et al. “Nup98-dependent transcriptional memory is established independently of transcription.” eLife vol. 11 e63404. 15 Mar. 2022, doi:10.7554/eLife.63404

      (6) Kadota, Shinichi et al. “Nucleoporin 153 links nuclear pore complex to chromatin architecture by mediating CTCF and cohesin binding.” Nature communications vol. 11,1 2606. 25 May. 2020, doi:10.1038/s41467-020-16394-3

      (7) Gozalo, Alejandro et al. “Core Components of the Nuclear Pore Bind Distinct States of Chromatin and Contribute to Polycomb Repression.” Molecular cell vol. 77,1 (2020): 67-81.e7. doi:10.1016/j.molcel.2019.10.017

      (8) Shimell, MaryJane, and Michael B O'Connor. “Endoreplication in the Drosophila melanogaster prothoracic gland is dispensable for the critical weight checkpoint.” microPublication biology vol. 2023 10.17912/micropub.biology.000741. 21 Feb. 2023, doi:10.17912/micropub.biology.000741

    1. Author response:

      The following is the authors’ response to the original reviews.

      We have responded to these criticisms below and have revised the main text and figures. Here, we outline the major points of our responses:

      (1) The reviewers asked for more clarification regarding cell type annotation in the lung mesenchyme as shown in Figure 3C. We have included a new supplementary figure (Supplementary Figure 2) which shows differentially expressed genes amongst these mesenchymal cell subsets using a variety of visualization tools including a heatmap, UMAP plots, and the dotplot which was originally shown in Supplementary Figure 1D. The other supplemental figures have been re-numbered.

      (2) We acknowledge the lack of consensus in the field regarding the nomenclature of fibroblast subsets in the developing mouse lung. We are not attempting to define new subsets, but rather we adopted annotations based on previously published work. Specifically, we used Seurat to define mesenchymal cell clusters and then compared the gene expression patterns of these clusters to published work by Hurskainen et al. (Bernard Thebaud’s group) and Narvaez Del Pilar et al. (Jichou Chen’s group). We acknowledge these annotations might conflict with other published data, but any approach to choosing a cell label would be subject to scrutiny. For example, Col13a1 fibroblasts share markers with cells which have been defined by others as lipofibroblasts or alveolar fibroblasts. Similarly, Col14a1 fibroblasts appear to share markers with matrix fibroblasts. Further work is clearly needed to address these discrepancies, and we hope that making our data publicly available will help that effort. 

      (3) The reviewers asked us to interrogate changes in canonical markers of fibroblast subsets (i.e. lipofibroblasts, matrix fibroblasts) to address whether the apparent loss of myofibroblasts could be explained by a change in myofibroblast specification/differentiation. We have included these data in the responses, but because we are unable to draw any clear conclusions from these results, we do not feel these data warrant inclusion in the manuscript/figures.

      (4) As highlighted in the eLife assessment, our study does not include tissue validation (i.e. immunohistochemistry) of myofibroblast markers to distinguish whether the loss of myofibroblasts is attributable to lack of proliferation and/or changes in differentiation/specification. We spent considerable time over the past few months attempting to address these questions, however we were unable to produce convincing PDGFRa staining on tissues that we had collected during our original studies. Without PDGFRa staining, we regretfully could not co-stain for other useful markers to assess proliferation (EdU), apoptosis (TUNEL or caspase), or fibroblast function/specification (ACTA2, SM22a/TAGLN, ADRP, etc). We suspect that these experiments would require optimization of tissue fixation/processing at the time of harvest or the inclusion of a Pdgfra lineage tool for better identification of these cells by immunohistochemistry. Given that the majority of Pdgfra lineage tools require a knock-in/knock-out approach, data generated using these tools should be interpreted with caution given our results here show that Pdgfra-haploinsufficiency alone worsens disease outcomes after hyperoxia exposure.

      In summary, we have addressed several concerns raised by the reviewers and have attempted to perform some of the additional experiments suggested.

      Public Reviews:

      Reviewer #1 (Public Review):

      Summary:

      In this study, the authors used both the commonly used neonatal hyperoxia model as well as cell-type-specific genetic inactivation of Tgfbr2 models to study the basis of BPD. The bulk of the analyses focus on the mesenchymal cells. Results indicate impaired myofibroblast proliferation, resulting in decreased cell number. Inactivation of Etc2 in Pdgfra-lineaged cells, preventing cytokinesis of myofibroblasts, led to alveolar simplification. Together, the findings demonstrate that disrupted myofibroblast proliferation is a key contributor to BPD pathogenesis.

      Strengths:

      Overall, this comprehensive study of BPD models advances our understanding of the disease. The data are of high quality.

      Weaknesses:

      The critiques are mostly minor and can be addressed without extensive experimentation.

      Reviewer #2 (Public Review):

      Summary:

      In this study, the authors systematically explore the mechanism(s) of impaired postnatal lung development with relevance to BPD (bronchopulmonary dysplasia) in two murine models of 'alveolar simplification', namely hyperoxia and epithelial loss of TGFb signaling. The work presented here is of great importance, given the limited treatment options for a clinical entity frequently encountered in newborns with high morbidity and mortality that is still poorly understood, and the unclear role of TGFb signaling, its signaling levels, and its cellular effects during secondary alveolar septum formation, a lung structure generating event heavily impacted by BPD. The authors show that hyperoxia and epithelial TGFb signaling loss have similar detrimental effects on lung structure and mechanical properties (emphysema-like phenotype) and are associated with significantly decreased numbers of PDGFRa-expressing cells, the major cell pool responsible for generation of postnatal myofibroblasts. They then use a single-cell transcriptomic approach combined with pathway enrichment analysis for both models to elucidate common factors that affect alveologenesis. Using cell communication analysis (NicheNet) between epithelial and myofibroblasts they confirm increased projected TGFb-TGFbR interactions and decreased projected interactions for PDGFA-PDGFRA, and other key pathways, such as SHH and WNT. Based on these results they go on to uncover in a sequela of experiments that surprisingly, increased TGFb appears reactive to postnatal lung injury and rather protective/homeostatic in nature, and the authors establish the requirement for alpha V integrins, but not the subtype alphaVbeta6, a known activator of TGFb signaling and implied in adult lung fibrosis. The authors then go beyond the TGFb axis evaluation to show that mere inhibition of proliferation by conditional KO of Ect2 in Pdgfra lineage results in alveolar simplification, pointing out the pivotal role of PDGFRa-expressing myofibroblasts for normal postnatal lung development.

      Strengths:

      (1) The approach including both pharmacologic and mechanistically-relevant transgenic interventions both of which produced consistent results provides robustness of the results presented here.

      (2) Further adding to this robustness is the use of moderate levels of hyperoxia at 75% FiO2, which is less extreme than 100% FiO2 frequently used by others in the field, and therefore favors the null hypothesis.

      (3) The prudent use of advanced single-cell analysis tools, such as NicheNet to establish cell interactions through the pathways they tested and the validation of their scRNA-seq results by analysis of two external datasets. Delineation of the complexity of signals between different cell types during normal and perturbed lung development, such as attempted successfully in this study, will yield further insights into the underlying mechanism(s).

      (4) The combined readout of lung morphometric (MLI) and lung physiologic parameters generates a clinically meaningful readout of lung structure and function.

      (5) The systematic evaluation of TGFb signaling better determines the role in normal and postnatally-injured lungs.

      Weaknesses:

      (1) While the study convincingly establishes the effect of lung injury on the proliferation of PDGFRa-expressing cells, differentiation is equally important. Characterization of PDGFRa expressing cells and tracking the changes in the injury models in the scRNA analysis, a key feature of this study, would benefit from expansion in this regard. PDGFRa lineage gives rise to several key fibroblast populations, including myofibroblasts, lipofibroblasts, and matrix-type fibroblasts (Collagen13a1, Collagen14a1). Lipofibroblasts constitute a significant fraction of PDGFRa+ cells, and expand in response to hyperoxic injury, as shown by others. Collagen13a1-expressing fibroblasts expand significantly under both conditions (Figure 3), and appear to contain a significant number of PDGFRa-expressing cells (Suppl Fig.1). Effects of the applied injuries on known differentiation markers for these populations should be documented. Another important aspect would be to evaluate whether the protective/homeostatic effect of TGFb signaling is supporting the differentiation of myofibroblasts. Postnatal Gli1 lineage gains expression of PDGFRa and differentiation markers, such as Acta2 (SMA) and Eln (Tropoelastin). Loss of PDGFRa expression was shown to alter Elastin and TGFb pathway-related genes. TGFb signaling is tightly linked to the ECM via LTBPs, Fibrillins, and Fibulins. An additional analysis in the aforementioned regard has great potential to more specifically identify the cell type(s) affected by the loss of TGFb signaling and allow analysis of their specific transcriptomic changes in response and underlying mechanism(s) to postnatal injury.

      We attempted to conduct additional analyses on our sequencing data to evaluate the impact of lung injury on the differentiation of Pdgfra-expressing cells towards other fibroblast lineages. To specifically address the impact of hyperoxia on fibroblast differentiation, we subsetted wildtype cells collected at the P7 timepoint (while pups were still undergoing hyperoxia treatment) from the larger data set. Shown below are several Violin Plots comparing gene expression between RA and O2 conditions across the mesenchymal populations.

      Although there are some interesting observations in this analysis, we could not identify a consistent theme from these data which could clearly answer the reviewers’ questions. We see a clear reduction of Pdgfra and Eln in both myofibroblast subsets with hyperoxia, which support our findings of reductions in the myofibroblast subsets. Acta2 and Tagln appear slightly lower in alveolar myofibroblasts, but both are higher in ductal myofibroblasts. Interestingly, both Acta2 and Tagln are higher in Col14a1 fibroblasts with hyperoxia. The functional relevance of these data are unclear because there appears to be higher per-cell expression of Acta2 in ductal myofibroblasts while the relative contribution of these cells is reduced (Figure 3D-E). Col14a1 fibroblasts show increased Acta2 and Tagln expression and are slightly increased in proportion at P7 with hyperoxia treatment (Figure 3D), albeit to a much lesser degree compared to Col13a1 fibroblasts.

      Author response image 1.

      Markers of ductal myofibroblasts including Hhip, Cdh4, and Aspn all appear lower with hyperoxia. Interestingly Plin2 expression is only slightly increased in Col13a1 fibroblasts with hyperoxia treatment, and there is also increased expression in alveolar myofibroblasts. Tcf21 is another marker commonly used to identify lipofibroblasts and its expression is similarly increased in myofibroblasts during hyperoxia, although its expression is conversely lower in Col13a1 and Col14a1 fibroblasts in our data. Overall, these data would appear consistent with recently published data by Ricetti et al. in which the authors observed an increase in lipofibroblast gene signatures and reduced myofibroblast gene signatures with hyperoxia treatment.

      Author response image 2.

      Author response image 3.

      The ability of our data to clearly identify changes in cell fate differentiation is limited by our use of Seurat to define cell clusters because these methods are likely to mask subtle gene expression changes in a small number of cells nested within a parent cluster. In the example above with Plin2, the change in Plin2 expression within myofibroblasts is not significant enough for Seurat to pull these cells out from their parent clusters to define a different lineage, nor are these cells similar enough in their current moment in time to be considered Col13a1 fibroblasts or lipofibroblasts. Increasing the dimensions used to define Seurat clusters might be sufficient to identify this subset of cells as a distinct cluster, however this approach would come at the expense of creating several more cell subsets with increasingly small populations which would be difficult to further analyze.

      One alternative approach to address these questions regarding differentiation might include using pseudo-time analysis of our sequencing data to predict cell lineage. Unfortunately, these analyses are beyond the scope of our current study, but we hope that our public data set can be used by investigators hoping to utilize this approach. Another method to address these questions could utilize a pulse-chase lineage experiment where one could label Pdgfra-expressing cells at the onset of injury and compare the differentiation of these labeled cells following injury. Li et al. conducted a similar experiment with hyperoxia in which Pdgfra-expressing cells were labeled during embryonic development and then postnatally following hyperoxia exposure. The authors noted a decrease in both lineaged myofibroblasts and lineaged lipofibroblasts and concluded that Pdgfra-lineaged cells were lost with hyperoxia treatment rather than undergoing aberrant differentiation. While these experiments likely have their own caveats related to the timing and efficiency of labeling, they represent a more conclusive approach to addressing differences in cell specification as compared to our sequencing- and flow cytometry-based approaches.

      Author response image 4.

      Author response image 5.

      (2) Of the three major lung abnormalities encountered in BPD, the authors focus on alveolarization impairment in great detail, to a very limited extent on inflammation, and not on vascularization impairment. However, this would be important not only to better capture the established pathohistologic abnormalities of BPD, but also it is needed since the authors alter TGFb signaling, and inflammatory and vascular phenotypes with developmental loss of TGFb signaling and its activators have been described. Since the authors make the point about the absence of inflammation in their BPD model, it will be important to show the evidence.

      We acknowledge that vascular changes significantly contribute to BPD pathogenesis, however our study was not designed to adequately characterize changes in vascular/endothelial cells. We were motivated to focus on the lung mesenchyme after observing a dramatic loss of PDGFRa+ cells with our initial characterization of the hyperoxia injury model (Figure 2). At the onset of our study, the existing publicly available data did not contain enough mesenchymal cells for in-depth analysis. To generate new observations and hypotheses within the lung mesenchyme we enriched our single cell prep for mesenchymal cells at the time of FACS-sorting to ensure we would have sufficient cell numbers for downstream analysis.

      (3) Conceptually it would be important that in the discussion the authors reconcile their findings in the experimental BPD models in light of human BPD and the potential implications it might have on new ways to target key pathways and cell types for treatment. This allows the scientific community to formulate the next set of questions in a disease-relevant manner.

      We have edited text in the discussion to address this point.

      Reviewer #3 (Public Review):

      Summary:

      This paper seeks to understand the role of alveolar myofibroblasts in abnormal lung development after saccular stage injury.

      Strengths:

      Multiple models of neonatal injury are used, including hyperoxia and transgenic models that target alveolar myofibroblasts.

      Weaknesses:

      There are several weaknesses that leave the conclusions significantly undersupported by the data as presented:

      (1) There is no validation of the decreased number of myofibroblasts suggested by flow cytometry/scRNAseq at the level of the tissue. Given that multiple groups have reported increased myofibroblasts (aSMA+ fibroblasts) in humans with BPD and in mouse models, demonstrating a departure from prior findings with tissue validation in the mouse models is essential. There are many reasons for decreased numbers of a subpopulation by flow cytometry, most notably that injured cells may be less likely to survive the cell sorting process.

      Unfortunately, we were unable to produce convincing PDGFRa staining on tissues that we had collected during our original studies. Without PDGFRa staining, we regretfully could not co-stain for other useful markers to assess proliferation (EdU), apoptosis (TUNEL or caspase), or fibroblast function/specification (aSMA/ACTA2, SM22a/TAGLN, ADRP, etc). We suspect that these experiments would require optimization of tissue fixation/processing at the time of harvest or the inclusion of a Pdgfra lineage tool for better identification of these cells by immunohistochemistry. Given that the majority of Pdgfra lineage tools require a knock-in/knock-out approach, data generated using these tools should be interpreted with caution given our results here show that Pdgfra-haploinsufficiency alone worsens disease outcomes after hyperoxia exposure.

      Our single cell data show that there is increased expression of Acta2 and Tagln shown in the plots which might be consistent with the increased aSMA staining which others have observed in these settings. Interestingly, the transcripts of both genes are reduced in alveolar fibroblasts while increased in ductal myofibroblasts, Col13a1 fibroblasts, Col14a1 fibroblasts, and vascular smooth muscle. We did not include aSMA antibody staining in our flow cytometry experiments, but this would certainly add value to future attempts to characterize the phenotypic changes occurring during these injury models. 

      (2) The hallmark genes used to define the subpopulations are not given in single-cell data. As the definition of fibroblast subtypes remains an area of unsettled discussion in the field, it is possible that the decreased number by classification and not a true difference. Tissue validation and more transparency in the methods used for single-cell sequencing would be critical here.

      See response above and new Supplemental Figure 2.

      (3) There is an oversimplification of neonatal hyperoxia as a "BPD model" used here without a reference to detailed prior work demonstrating that the degree and duration of hyperoxia dramatically change the phenotype. For example, Morty et al have shown that hyperoxia of 85% or more x 14 days is required to demonstrate the septal thickening observed in severe human BPD. Other than one metric of lung morphometry (MLI), which is missing units on the y-axis and flexivent data, the authors have not fully characterized this model. Prior work comparing 75% O2 exposure for 5, 8, or 14 days shows that in the 8-day exposed group (similar to the model used here), much of the injury was reversible. What evidence do the authors have that hyperoxia alone is an accurate model of the permanent structural injury seen in human BPD?

      At the onset of our studies, we noted that several groups were using widely variable protocols ranging from 60-100% O2 exposure. Morty et al. have indeed conducted thorough experiments to characterize various different hyperoxia exposure protocols. In their 2017 study (https://www.ncbi.nlm.nih.gov/pmc/articles/PMC5312005/) they showed that 85% O2 from P1-P7 was sufficient to produce increased septal thickness compared to control mice, and this change was comparable to P1-P14 exposure with 85% O2. Interestingly, they also noted that some therapeutic interventions could rescue disease caused by 60% O2 but not 85% O2 exposure. Our criteria in choosing a treatment protocol were: (1) nursing dams and pups survived hyperoxia exposure, (2) injury was reproducible across cohorts, and (3) injury was not reversible simply by recovering in room air. We found that recent work utilizing 75% O2 exposure was sufficient to cause the alveolar simplification phenotype which we sought to investigate. In our hands, we did not observe mortality of nursing dams or pups except for litters lost to cannibalism/failure of cross-fostering.

      We are confident that the injury caused by our hyperoxia protocol is not reversible simply by recovering mice in room air. Several groups have phenotyped mice at P4, P10, or P14 immediately following the conclusion of hyperoxia treatment. To ensure that we were studying a lasting, irreversible phenotype, we conducted our endpoint studies (morphometry and lung physiology) at P40. Because mice continue to undergo alveolarization until ~P36-P39, we reasoned that this additional recovery time following cessation of hyperoxia would allow for spontaneous recovery if this injury was transient. Additionally, shown below are unpublished flexiVent data in which mice were treated for 10 days with 75% O2 and recovered until analysis at 10 weeks of age. These results are entirely consistent with the flexiVent data we have included in the manuscript, and the persistence of lung physiologic changes in adult mice suggest the presence of permanent underlying structural changes. We did not conduct morphometry/MLI studies at later timepoints, but we have no reason to suspect a different outcome given the clear results from lung physiology.

      Author response image 6.

      (4) Thibeault et al published a single-cell analysis of neonatal hyperoxia in 2021, with seemingly contrasting findings. How does this dataset compare in context?

      Our data is complimentary to the single-cell analysis published by Thebaud et al. We included a re-analysis of their mesenchymal data in Supplementary Figure 2 which shows they also observed a relative decrease in myofibroblast clusters at the P7 and P14 timepoints following hyperoxia treatment. Figure 4 of their paper highlights the top differentially expressed genes between RA and O2 in Col13a1 FB and myofibroblasts, and we observe nearly identical findings in our data set within each of these clusters. Below we have created dotplots of P7 wildtype samples for the same selected genes shown in Figure 4G of the Thebaud et al. paper. It is important to note that their clustering pooled all myofibroblasts into one cluster, while our data is divided into alveolar myofibroblasts and ductal myofibroblasts. The other difference is their data set includes all timepoints P3, P7 and P14 pooled for display, while the plot we selected for simplicity here is only P7 cells. From these data we can see that the general trends are identical to those observed by Thebaud et al., and the differences in genes such as Acta2 can be accounted for by different changes observed in the different myofibroblast clusters – which is identical to what is shown in the violin plots above – namely that Acta2 is reduced in hyperoxia in alveolar myofibroblasts while increased in the ductal myofibroblasts.

      Author response image 7.

      Alveolar myoFB

      Author response image 8.

      Ductal myoFB

      One difference between our two datasets is the relative contribution of myofibroblast and Col13a1 fibroblasts to the entire mesenchymal population of cells. Over 50% of all mesenchymal cells in our preps consist of myofibroblasts, while most of their mesenchymal cells are Col13a1 fibroblasts. These differences are likely accounted for by differences in tissue digestion and cell preparation protocols. However, despite these differences, their data show the same trends of decreased myofibroblasts and a relative expansion in Col13a1 fibroblasts.

      Recommendations for the authors:

      Reviewer #1 (Recommendations For The Authors):

      (1) Figure 1, for the hyperoxia model, it is informative to have the analysis done at P40, while most of the previous studies using this model focus on outcomes shortly after the end of the hyperoxia regimen. The authors state "we did not see evidence of fibrosis, scarring, or inflammation." It will be helpful to include data supporting this conclusion, especially ACTA2, CTHRC1, and CD45 staining.

      We did not conduct trichrome staining or hydroxyproline assays to quantify the absence of fibrotic changes because there were no gross histologic changes consistent with scarring or fibrosis by H&E staining. We have amended the text to say “we did not see evidence of fibrosis or scarring” since we did not publish any changes to characterize the immune cell compartment.

      (2) Figure 3, single cell analysis, naming of the clusters is confusing. Is "alveolar myofibroblasts" the same as "secondary crest myofibroblasts"? Is "Col13a1 FB" the same as "alveolar fibroblasts" and "Col14a1 FB" the same as "adventitial fibroblasts"? The loss of myofibroblasts is intriguing because, by staining, there is an increase of ACTA2+ cells. Are ACTA2+ cells not myofibroblasts in scRNAseq data?

      As mentioned in responses above, we used Jichou Chen’s nomenclature of “alveolar myofibroblasts” and “ductal myofibroblasts”, but we agree that the former cluster is most consistent with “secondary crest myofibroblasts”. To distinguish the two remaining clusters of fibroblasts we used the same nomenclature as found in Thebaud et al’s single cell data set- “Col13a1 FB and “Col14a1 FB”. The Col13a1 FB cluster is most consistent with “alveolar fibroblasts” and contains high expression of several genes used to define “lipofibroblasts”, though it is unclear whether the latter may represent a subcluster within the Col13a1 FB cluster.

      As shown above, Acta2 is expressed broadly within the lung mesenchyme with highest levels found in myofibroblasts and smooth muscle cells.

      (3) Phosphorylated SMAD2/3 staining (e.g. Cell Signaling antibody) in the two models will be informative to show where TGF signaling activity is altered.

      We have not been successful in using SMAD2/3 staining to infer changes in TGFb signaling at the resolution needed to address this question. Other groups have shown qPCR and western blot data for SMAD2/3 signaling from whole lung extracts, but these approaches lack cell type and specificity and do not address spatial changes. We attempted to incorporate pSMAD2/3 staining into our flow cytometry experiments, but the staining protocol did not work in our hands.

      (4) Is cell death increased in the multiple models that showed simplification?

      While our EdU experiments address proliferation, we were unable to perform PDGFRa and TUNEL/caspase co-staining by histology to address apoptosis/cell death in our different models. Shown here is data from P7 wildtype mice in which Cdkn1a (promoting arrest of cell cycle), and pro-apoptotic genes Bax, Bak1, and Fas are all upregulated in hyperoxia in several mesenchymal cell populations including myofibroblasts.

      Author response image 9.

      (5) Wording: "These data suggest that avb6 does not play a role in TGFb activation during normal development or neonatal hyperoxia, while av-integrins in the lung mesenchyme are required for normal development and play a protective role in response to hyperoxia." The first half of the sentence is missing a reference to the epithelium.

      Text now reads "These data suggest that epithelial avb6 does not play a role…”

      Reviewer #2 (Recommendations For The Authors):

      The reviewer greatly appreciates the work presented here, especially the hard task of addressing combined signaling pathway input into key mesenchymal cell types during an essential expansion of alveolar surface area in postnatal lung and its effect upon disturbance.

      The issues of concern are mentioned in the public review and are expanded upon below:

      (1) Expanded characterization of PDGFRa+ expressing cells in the scRNA dataset is needed (see public review). Also included should be some of the key myofibroblast genes (elastin, Acta2, etc.) and their changes in the relevant cell populations. It would be important to show (at least at the transcriptional level) that myofibroblast differentiation is impaired if the author claims that the alveolarization defect is due to functional myofibroblast impairment. Furthermore, Ect2 expression and changes with treatments should be shown for the different cell populations (relevant to Figure 9).

      See responses above

      (2) The authors stated that they did not find evidence of fibrosis, scarring, and inflammation, but did not provide data to support this statement. Given the importance of at least the inflammation component in BPD, the absence of inflammation needs to be shown, especially in the model using the TGFBR2-cKO mouse, where at least their data show a trend to increased CD45 cell numbers (Figure 2), and upregulated inflammatory upstream regulators (IL10, IFNa, IKBKB, CEBPB upregulated) in the IPA (Figure 3). BAL and/or tissue by flow or IHC have been used to assess different immune cell populations. In terms of evaluation of vascular impairment, the single-cell data set contains endothelial cells, vascular smooth muscle, and pericytes, which allows interrogation following the two different types of injury (hyperoxia cKO TGFbR2) used for the scRNA-seq experiments).

      A full characterization of the immune cell or vascular/endothelial cell compartment within our models is beyond the scope of this current study as we were focusing on the shared changes observed within the lung mesenchyme. None of these compartments exist in isolation, so of course there are likely to be correlative and/or causative changes observed in each of the different models which we studied. We did consider further phenotypic analysis of the immune cells by flow cytometry within our different models, but deferred these experiments for future studies. As mentioned earlier we have omitted the reference to “no inflammation”.

      (3) The authors should report several litters per experiment and experimental group, mortality in the groups, and if present, visualize using e.g. Caplan-Meyer curves. The switch of the mothers during treatment, the early postnatal injections and treatments, and variability in outcome measures between different litters have to be anticipated. Therefore at least 2 litters, but preferably 3 litters per experiment should be examined, to show reproducibility.

      All experiments were conducted with at least 2-3 contemporaneous litters in each treatment group as this was necessary to have enough animals per treatment condition/group to achieve statistical significance. This was essential as all experiments were conducted on the C57BL/6 background where litter sizes are typically 6-8 pups in our colony. We did not encounter any maternal mortality related to hyperoxia exposure while rotating between hyperoxia and normoxia every 48 hrs. Loss of pups in our experiments was mostly due to cannibalism either immediately after birth or from neglect due to failure of cross-fostering.

      (4) The reviewer is concerned about using PBS as a control for experiments involving antibody treatment, in this case, 1D 11. The use of an isotype IgG would be the most appropriate and convincing control. In this case, an isotype-matched murine IgG1 control (13C4) has already been generated and is commercially available. While the reviewer does not suggest repeating all experiments, at least one small experiment showing that control IgG does not alter the lung phenotype with hyperoxia when compared with 1D11 would be important.

      We appreciate the reviewer’s suggestion and will consider an isotype antibody comparison in future studies. While not directly comparing 1D11 to isotype, we can share data in which we compared PBS to a different antibody. In this experiment, we attempted to use antibody blockade during the first 10 days of life while mice were undergoing hyperoxia treatment to target a specific component of the TGFb pathway. We observed no difference in outcomes either in RA or O2 when comparing PBS to xxx antibody. We cannot share the antibody identity due to intellectual property reasons, however additional studies confirmed that this antibody likely had no impact due to poor in vivo blocking activity.

      Author response image 10.

      (5) While inhibited proliferation is one possible explanation for the decrease of PDGFRa expression in the injured mice, there should be consideration of increased and/or premature apoptosis (before the physiologically observed wave P14-P20) as another reason. Also, do the authors propose that only proliferation results in alveolarization impairment, but differentiation plays no significant role here? If that is the case that would mean that there are some fully-differentiated myofibroblasts in the alveolar septa, but not enough to create the multitude of alveolar septal walls. Have the authors evaluated the decrease in secondary alveolar septa formed per alveolar airspace? This measure would give some sense of whether septum initiation was prevented or whether septa were formed, but are structurally abnormal, e.g. due to altered ECM (suspected decrease in Elastin and SMA expression, if myofibroblast differentiation was impaired or cell content (suspected decrease in myofibroblasts and increase of other cell types, such as lipofibroblasts).

      Apoptosis/cell death are likely to play a role in addition to inhibited proliferation. See violin plots shown above with cell cycle arrest and pro-apoptotic genes upregulated within the mesenchyme. Because we were unable to optimize tissue sections/staining with the samples collected during the early time points of our experiments (ie P4, P7, P10, P14), we are unable to co-stain for markers of apoptosis and answer this question in a direct manner. Future experiments will focus on additional characterization of these early changes with particular attention to altered fibroblast phenotypes within the alveolar septae.

      (6) An illustration depicting key cells and the pathways involved in cartoon format would be a useful addition and visualize the important conclusions of this paper for the reader.

      We appreciate this suggestion but think the results are sufficiently straightforward that a summary cartoon would not add much.

      Figure 4A: the legend appears to be switched. The gray square seems to align with the epithelial ligands, while the blue square aligns with receptors.

      Thank you for identifying this mistake – fixed.

      Names of transgenic lines used through manuscript:

      Please use the correct name, as per JAX would be either Gli1tm3(cre/ERT2)Alj/J or Gli1-CreERT2.

      Please use the correct name, as per JAX would be either Pdgfratm1.1(cre/ERT2)Blh/J or Pdgfrα-CreERT2.

      PDGFRa-CRE would be JAX# 013148.

      The transgenic lines have been noted in the methods, and we have edited the text of the manuscript to reflect the correct names of these lines. For the supplementary figure 4 which compares Gli1-CreERT2 to Pdgfrα-CreERT2, we left our prior nomenclature intact because it better reflects that each of these lines are haploinsufficient at their targeted loci, and that the controls are cre-negative littermates.

      We did not use the PDGFRa-CRE line (JAX# 013148).

      Reviewer #3 (Recommendations For The Authors):

      - More transparency about the single-cell analysis is required: 1) how are cell types and clusters defined? 2) what strategy was used for ambient RNA? 3) how do the controls compare with recently published mouse developmental datasets? 4) how does this model compare with the single-cell dataset published by Thibeault et al in 2021 (neonatal hyperoxia x 14 days with multiple time points used)?

      See responses above.

      - Tissue level validation of these findings is essential by RNA ISH or IF. While validation that the same process is at play in human tissue would be ideal, if this is not available, the conclusions must be tempered in the discussion.

      See responses above.

      - Is this more mild neonatal injury reversible in mice? As noted above, more characterization of this model (and placing it in the context of other more widely published models would be helpful).

      See responses above.

    1. Author response:

      The following is the authors’ response to the original reviews.

      Reviewer #1 (Public review): 

      Summary: 

      The innate immune system serves as the first line of defense against invading pathogens. Four major immune-specific modules - the Toll pathway, the Imd pathway, melanization, and phagocytosis- play critical roles in orchestrating the immune response. Traditionally, most studies have focused on the function of individual modules in isolation. However, in recent years, it has become increasingly evident that effective immune defense requires intricate interactions among these pathways. 

      Despite this growing recognition, the precise roles, timing, and interconnections of these immune modules remain poorly understood. Moreover, addressing these questions represents a major scientific undertaking. 

      Strengths: 

      In this manuscript, Ryckebusch et al. systematically evaluate both the individual and combined contributions of these four immune modules to host defense against a range of pathogens. Their findings significantly enhance our understanding of the layered architecture of innate immunity. 

      We thank the reviewer for their kind assessment.

      Weaknesses: 

      While I have no critical concerns regarding the study, I do have several suggestions to offer that may help further strengthen the manuscript. These include: 

      (1) Have the authors validated the efficiency of the mutants used in this study? It would be helpful to include supporting data or references confirming that the mutations effectively disrupted the intended immune pathways. 

      We have done so in Figure 1.

      (2) Given the extensive use of double, triple, and quadruple mutants, a more detailed description of the mutant construction process is warranted. 

      We now provide a supplement (File S1) that details the successive genetic crosses and recombinations that were required to generate these compound fly stocks carrying multiple mutations. We also provide some information regarding rapid screening of stocks for phenotypes. Of note some of these fly stocks have been deposited at VDRC as they will be useful to fly community to assess immune modules in a controlled background, and complete stock information will be tied to these stocks there.

      Reviewer #2 (Public review): 

      Summary: 

      In this work, the authors take a holistic view of Drosophila immunity by selecting four major components of fly immunity often studied separately (Toll signaling, Imd signaling, phagocytosis, and melanization), and studying their combinatory effects on the efficiency of the immune response. They achieve this by using fly lines mutant for one of these components, or modules, as well as for a combination of them, and testing the survival of these flies upon infection with a plethora of pathogens (bacterial, viral, and fungal). 

      Strengths: 

      It is clear that this manuscript has required a large amount of hands-on work, considering the number of pathogens, mutations, and timepoints tested. In my opinion, this work is a very welcome addition to the literature on fly immune responses, which obviously do not occur in one type of response at a time, but in parallel, subsequently, and/or are interconnected. I find that the major strength of this work is the overall concept, which is made possible by the mutations designed to target the specific immune function of each module (at least seemingly) without major effects on other functions. I believe that the combinatory mutants will be of use for the fly community and enable further studies of the interplay of these components of immune response in various settings. 

      To control for the effects arising from the genetic variation other than the intended mutations, the mutants have been backcrossed into a widely used, isogenized Drosophila strain called w1118. Therefore, the differences accounted for by the genotype are controlled. 

      I also appreciate that the authors have investigated the two possible ways of dealing with an infection: tolerance and resistance, and how the modules play into those. 

      We thank the reviewer for their kind assessment. 

      Weaknesses: 

      While controlling for the background effects is vital, the w1118 background is problematic (an issue not limited to this manuscript) because of the wide effects of the white mutation on several phenotypes (also other than eye color/eyesight). It is a possibility that the mutation influences the functionality of the immune response components, for example, via effects of the faulty tryptophan handling on the metabolism of the animal. 

      I acknowledge that it is not reasonable to ask for data in different backgrounds better representing a "wild type" fly (however, that is defined is another question), but I think this matter should be brought up and discussed. 

      We agree with the reviewer and have included caveats on the different genetic effects brought about the combinatory mutant approach including differences in white gene status, insertion of GFP or DsRed markers, and nature of genetic mutations (Line 142-on).

      “Of note, the strains used in this study differ in their presence/absence of the white<sup>+</sup> gene, present in the PPO1<sup>∆</sup>, NimC1<sup>1</sup> and eater<sup>1</sup> mutations.  In addition to its well established function in eye pigmentation, the white gene can also impact host neurology and intestinal stem cell proliferation (Ferreiro et al., 2017; Sasaki et al., 2021). We did not observe any obvious correlations between white<sup>+</sup> gene status and susceptibilities in this study. Moreover,  in a previous study looking at the cumulative effects of AMP mutations on lifespan, white gene status and fluorescent markers did not readily explain differences in longevity (Hanson and Lemaitre, 2023). We therefore believe that the extreme immune susceptibility we have created through deficiencies for pathways regulating hundreds of genes, or major immune modules, overwhelms the potential effects of white<sup>+</sup> and other transgenic markers. For additional information on which stocks bear which markers, see discussion in Supplementary file 1.”

      Of interest, we were highly conscious of this concern in working with combinatory AMP mutants which differed in white, GFP, and DsRed copies. However, even over the many weeks of snowballing effects on microbiota community composition and structure, we found no trends tied strictly to white+ or to other genetic insertions on lifespan (Hanson and Lemaitre, 2023; DMM).

      The whole study has been conducted on male flies. Immune responses show quite extensive sex-specific variation across a variety of species studied, also in the fly. But the reasons for this variation are not fully understood. Therefore, I suggest that the authors conduct a subset of experiments on female flies to see if the findings apply to both sexes, especially the infection-specificity of the module combinations.  

      We thank the reviewer for this suggestion. We have performed the requested experiments, and include female survival trends in Figure 4supp1. We have added the following text to the main manuscript (Line 554):

      “All survival experiments to this point were done with males. We therefore assessed key survival trends for these infections in females to learn whether the dynamics we observed were consistent across sexes (Figure 4supp1). For all three pathogens (Pr rettgeri, Sa aureus, C. albicans) the rank order of susceptibility was broadly similar between males and females, with higher rates of mortality in females overall. Thus, we found no marked sex-bygenotype interaction. Interestingly, the greater susceptibility of females in our hands is true even for ∆ITPM flies, although there are only a few surviving flies on which we can base these conclusions. However, these data may suggest the sexual dimorphism in defense against infection that we see against these pathogens is due to factors independent of the immune modules we disrupted.”

      It is worth noting that male-female sex dichotomies in infection are inconsistent across the literature, with strong lab-specific effects (Belmonte et al., 2020 and personal observation). In our lab setting, we consistently see female mortality higher than males when compared, independent of pathogen and mutant background. We have not seen notable interaction terms of sex and genotype for most immune deficient mutants. It is quite interesting to have done these experiments with ITPM, however, which reveals that there is at least a trend suggesting this dichotomy is independent of the four immune modules we deleted. Still, our infection conditions kill most males, and so it would be good to replicate this sex-specific ∆ITPM result in a dedicated study with doses chosen to improve the resolution of male-female differences. For now, we prefer to use conservative language and avoid overinterpreting this trend, but do feel it merits mentioning.  

      Recommendations for the authors:

      Comment on statistical requests

      Both reviewers requested further clarity on the statistical analyses supplemental to Figure 3. We haved address these comments as follows.

      First, we now provide an additional supplementary .zip file containing summary statistics for all survival data in Figure 3 (Supplementary File 3). We have additionally added this text to line 226 to make this data treatment more clear:

      …” we chose to focus on major differences apparent in summary statistics,Highlighting”…

      And we highlight that all survival data are also provided as Kaplan-Meier survival curves in the main or supplementary figures in Line 233:

      “Kaplan-Meier survival curves for all experiments are provided in the main text or supplementary information”.

      Second, as outlined in the main text, we were unable to sample across all pathogenby-genotype interactions systematically, and this unfortunately obfuscates robust statistical modelling. We addressed the challenge of finding meaningful statistical differences by focusing on trends only if they were i) consistent across experimental replicates, ii) of a consistent logic across comparable genotypes, ensuring random inter-experimental noise was not unduly shaping interpretations, and iii) of a mean lifespan difference ≥1.0 days compared to wild-type, and compared to relevant unchallenged or clean-injury controls. This last choice was especially important because not all experimental replicates included all genotypes due to challenges of animal husbandry and coordination among multiple researchers over five years of data collection. As a result, our initial analyses using a cox mixed-effects model found it to be rather useless, being insensitive to important experiment batch effects visible to the eye because statistically-affected genotypes were not present in all experiments.

      We therefore ensured that behaviour relative to controls within* experiments was consistent, rather than the comparison of genotypes to controls across the sum of experiments with a post-hoc treatment attempting to apportion variance to experiment batch (but unable to do so for some genotypes and some batches). Due to differeces in baseline health and the dynamics explained by studies like Duneau et al. (2017; eLife, there is an expected unequal variance of genotype*pathogen interactions across experiment batches. Unfortunately, this unequal variance, coupled with incomplete sampling across experiment batches, means “highly significant” differences can emerge that don’t hold up to scrutiny of comparisons to controls taken only from within an experiment batch. Thus, we chose to forego a cox mixed effect model approach entirely. Instead, our highly conservative approach, focusing on only very large effects with a mean lifespan difference ≥1.0 days, mitigates these issues. We have taken great care to ensure that any results we highlight stand up to inter-experiment batch effects. We would further draw the reviewers’ attention to our response to Reviewer 2 relating to Figure 3, which emphasizes the level of conservativism that we are applying.

      At the end of the Discussion, we have added the following sentence to emphasize these limitations:

      “…a combinatorial mutation approach to deciphering immune function can be extended even to the broad level of whole immune modules. Of note, we were unable to systematically sample all genotype-bypathogen interactions equally. We have therefore been highly conservative in our reporting of major effects. There are likely many important interactions” not discussed in our study. Future investigations may highlight important biology that is apparent in our data, but which we may not have mentioned here. To this end, we have deposited our isogenic immunity fly stocks in the Vienna Drosophila Resource Centre to facilitate their use. Beyond immunity, our tools can also be of use to study various questions at the cutting edge of aging, memory, neurodegeneration, cancer, and more, where immune genes are repeatedly implicated. We hope that this set of lines will be useful to the community to better characterize the Drosophila host defense.”

      We recognise this response may not fully satisfy the reviewers’ requests. While use of summary statistics is simple, our rules for highlighting interactions of importance are defined, readily understood and interpreted, and draw attention to key trends in that are backed by a solid understanding of the data and its limitations. We have taken this approach out of a responsibility to avoid making spurious assertions that stem from underpowered statistical models rather than from the biology itself.

      Reviewer #1 (Recommendations for the authors): 

      (1) Lines 1092-1093 - Please double-check the labeling of the panels in Figure 2. It appears that panels A and C correspond to single-module mutants, whereas panels B and D refer to compound-module mutants. 

      We have modified Figure 2 and Figure 2supp1 labelling. We also realise there was an error in the column titling that contributed to the confusion. We hope the new layout is clear, and thank the reviewers for noting this issue.

      (2) Lines 347-377 - Figure 2D is not cited in the text. 

      We now cite Fig2D in Line 356.

      (3) P values should be indicated in Figure 2 and Figure 3 for all relevant comparisons. Additionally, "ns" (not significant) should be added in Figure 5A-B. 

      We make the effort to show key uninfected survival trends in Figure 2, and list the total flies (n_flies) in Fig3 to provide the reader with the underlying confidence in the trends observed. We focus on differences of mean lifespan of at least 1 day, and which are consistent in direction across combinatory mutations.  We have avoided the multiple comparisons of cox proportional hazard survival analyses throughout this study because they are overly sensitive for our purposes, as we have previously when systematically comparing many genotypes to each other (see Hanson and Lemaitre, 2023; DMM).

      (4) Minor points: Hml-Gal4, UAS-GFP should be italic; Line 192-- "uL" and "uM"; Line 596: P>.05.

      We have made these changes. We’re unsure what the comment regarding P>.05 referred to, but have removed spaces and made it non-italics. 

      Reviewer #2 (Recommendations for the authors): 

      Statistical analyses and their outcomes are clearly indicated only for the data in Figure 1 and Figure 5 and in the supplement for Figure 1, while they are not reported/not easily accessible for other data. For the main figures, statistics should be indicated in the figure for an easier assessment of the data. In case of multiple comparisons potentially crowding the plots too much, statistics may be in a supplementary file/table. 

      See response above.

      In case of the hemocytes, besides phagocytosis, I would think that ROS generation via the DUOX/NOX system is also an integral part of the immune response against pathogens, and that has not been included here. That might be an interesting addition for future experiments. As the NimC1, eater double mutant flies are said to have fewer hemocytes, it is possible that this function of the hemocytes is affected as well. This could be commented on in the text. 

      The reviewer raises a good point. The role of DUOX and NOX in ROS responses is not assessed in our study. To our knowledge, DUOX and NOX participate primarily in the wound repair response, or in epithelial renewal at damage sites or in the gut. In our study on systemic immunity, we did not assess the role of clotting, the precise function of ROS, and we have missed other host defense or stress response mechanisms as well (e.g. constitutively-expressed AMP-like genes, TEPs, JAK-STAT) that likely play a role in the systemic immune defense. Considering the lethality caused by Nox and Duox mutation, there would be inherent genetic difficulties to recombine these as multiple mutations. Unfortunately, this makes it  difficult to include these processes in our analysis in a systematic manner.  We are already happy to have generated fly lines lacking four immune modules simultaneously, even if they are not fully immune deficient. We have mentioned this point in the discussion (Line 613-on).

      Of note, the NimC1, eater double mutants actually have decreased hemocyte counts at the adult stage (Melcarne et al,. 2019). Thus NimC1, eater double mutants are not impaired only in phagocytosis, but the overall cellular response. We make a point to outline this in Line 225-257, and 607.

      I think it could be mentioned that the melanization response at larval stage (against parasitoids) functions differently from the melanization described here (requiring hemocyte differentiation and PPO3).

      A good point. We have added this mention in Line 97:

      “In addition, a third PPO gene (PPO3) is specifically expressed by lamellocytes, specialized hemocytes that differentiate in larvae responding to and enveloping invading parasites (Dudzic et al., 2015)”.

      Overall, the clarity of the figures and figure legends could be worked on to make them a bit easier to follow. Below are some of my suggestions: 

      (1) In Figure 2, adding headings to parts C & D (similarly to A & B) would make it easier to follow what is happening in the figure at a glance. Also, it is rather difficult to visually follow which strain is which in the plots. I'd suggest adding the key/legend for single mutants below 2A & B, and the key for the double mutants below C & D. If a mutant is present in A & B and in C & D, it could be included in both keys. I also think that it would be intuitive to present the single mutants by dashed lines and double mutants by continuous lines (or vice versa), so that one would easily distinguish between them. Of note, the figure legend says that A & B are single mutants, but for example in B there are also some double mutants (?). 

      We have modified Figure 2 and Figure 2supp1 labelling. We also realise there was an error in the column titling that contributed to the confusion. We hope the new layout is clear, and thank the reviewers for noting this issue.

      (2) In Figure 3, it looks like ΔMel is almost identical to controls in the clean injury survival, but in Figure 2C, it is clearly doing worse. I might be missing something here, but would like the authors to clarify the matter. Also, the meaning of the numbers in the heat map could be explained in the figure legend and/or added to the figure (color key). 

      The reviewer is correct. We thank the reviewer for this astute observation. Inadvertently, we used an old version of the Figure 2 preparation where only a subset of experiments was entered in the Prism data file rather than the total data used to inform Figure 3. This issue affected all genotypes.

      We have reviewed the data in Figure 2, Figure 2supp1, and Figure 3, and updated these figures accordingly to ensure they represent the full survival data. We have also incorporated new experiments into the sum data related to male-female differences and to fill gaps in the data from the 1<sup>st</sup> submission. We will also note due to the nature of 1<sup>st</sup> decimal rounding that the difference between WT and ΔMel appears slightly underrepresented: the true difference (over the 7-day lifespan) is 0.37. We’ve provided a version of this figure rounded to 2 decimal places below, but prefer the simpler 1 decimal place in the main text for readability. The updated Figure 2 shows the full data in Figure 3 accurately.

      We will also take this opportunity to highlight how conservative our ≥1.0 days difference approach is. Breaking down survival curve patterns in Figure 2 relative to mean differences in Figure 3, for clean injury, approximately ~75% of ΔMel flies survive to day 7 with mortality mostly taking place between days 3-7. The result is a mean lifespan of 6.37 days. On a survival curve, this difference appears quite strong, but in our mean lifespan table the difference is rather muted (WT vs. ΔMel difference = 0.37 days). Thus, differences of ≥1.0 days reflect very strong trends in survival data that are near-guaranteed to be independent of experimental noise. While we note issues that prevented us from a fully systematic sampling for all experiments, we are confident that the ≥1.0 day differences we highlight, using the rules explained in the main text, are robust. While this approach could be seen as overly conservative, it is our preference in this initial study, containing combinations of 25 treatments and 14 genotypes, to be highly conservative. Future studies may investigate other strong differences we have not highlighted, and the data we provide here can help generate expectations and guide those studies.

      Author response image 1.

      Figure 3 with 2 decimals places of rounding for mean lifespans. The 7-day clean injury mean lifespan of WT is 6.74 days, and of ΔMel is 6.37 days. Due to rounding, in the 1 decimal Figure 3 this difference appears as if it is only 0.3 days, but it closer to 0.4 days. Regardless, this level of difference, which appears rather clearly in a survival curve, is well below the level of difference we have chosen to highlight in our study.

      (1) Figure 4: I find it very tedious to compare CFUs among different mutants from the plots. As the idea is to compare bacterial loads among the mutants at different timepoints, it would be easier to compare them if the data were shown within a timepoint (CFUs of each mutant at 2h, at 6h, and so on). This is also how the results are written in the text (within a time point). Would it also be clearer if the CFU plots were named, for example: " A', B', and C'"? 

      We appreciate this note. We feel both representations have merits and pitfalls, but prefer our original design showing the progression of bacterial growth within genotype first. However, we have added dotted lines representing the wild-type bacterial loads at 2hpi, 12hpi, and 24hpi to assist the reader in making acrossgenotype comparisons at key time points. Like this, the reader can see if the error bars (StDev) overlap the mean of the wild-type, and so make more intuitive judgements about whether these differences are meaningful.

      (2) Figure 2D is not referred to in the text. 

      We now cite Fig2D in Line 356.

    1. Author Response

      The following is the authors’ response to the original reviews.

      Reviewer #1 (Public Review):

      The modeling approaches are very sophisticated, and clearly demonstrate the selective nature of acute ketamine to reduce the impact of trial losses on subsequent performance, relative to neutral or gain outcomes. The authors then, not unreasonably, suggest that this effect is important in the context of the negative bias in interpreting events that is prominent in depression, in that if ketamine reduces the ability of negative outcomes to alter behavior, this may be a mechanism for its rapid acting antidepressant effects.

      However, there is a very strong assumption in this regard, as shown by the first sentence of the discussion which implies this is a systematic study of ketamine's acute antidepressant effects. In actuality, this is a study of the acute effects of ketamine on reinforcement learning (RL) modeled parameters. A primary concern here is that an effect presented as a "robust antidepressant-like behavioral effect" should be more enduring than just an alteration during the acute administration. As it is, the link to an "anti-depressant effect" is based solely on the selective effects on losses. This is not to say this is not an interesting observation, worthy of exploration. It is noted that a similar lack of enduring effects on outcome evaluation is observed in humans, as shown in supplemental fig. S4, but there is not accompanying citation for the human work.

      We agree with the reviewer that the way we linked the study results to ketamine’s antidepressant action can be misleading and based on a rather strong assumption which was not systematically tested in the study. We made the following changes to the manuscript:

      (1) These results constitute a rare report of a robust antidepressant-like behavioral effect produced by therapeutic doses of ketamine during acute phase (<1 hour) after injection (Introduction, 3rd paragraph, line 8-9 in the original manuscript).

      Changed to: These results constitute a rare report of an acute effect of therapeutic dose of ketamine on the processing of affectively negative events during dynamic decision-making.

      (2) We clarified in the Discussion that our study is to gain insights into, but not a systematic investigation of ketamine’s antidepressant action as follows:

      (2.1) A sentence was added (1st paragraph of Discussion): Using a token-based decision task and extensive computational modeling, we examined the behavioral modulation induced by therapeutic doses of ketamine to gain insights into possible early signs of ketamine’s antidepressant activity.

      (2.2) Consistent with the findings from humans, ketamine’s effect on outcome evaluation was acute and did not last over subsequent days (Supplemental Figure S4) (Discussion, 2nd paragraph, line 6-7 in the original manuscript).

      Changed to: While ketamine’s antidepressant effect is reported to be sustained over a week of period (5), ketamine’s effect on outcome evaluation was acute and did not last over subsequent days (Supplemental Figure S4). This discrepancy might be attributable to the possible differences in the state of brain network between healthy subjects and those with depression as well as the type of measures taken to assess ketamine’s effect.

      (2.3) A sentence was added (Discussion, last sentence of the 2nd paragraph) : Nevertheless, systematic studies are required to understand whether the reduced aversiveness to loss in our task might share the same mechanisms that underlie ketamine’s antidepressant action.

      One question that comes to mind in terms of the selectivity observed is whether similar work has been done to examine the acute effects of any other drugs. If ketamine is unique in this regard, that would be quite interesting.

      We think this is an interesting idea. However, comparing ketamine’s effect to that of other drugs is not the scope of the current study. We hope that we will be able to answer this question with future studies.

      Reviewer #2 (Public Review):

      Oemisch and Seo set out to examine the effects of low-dose ketamine on reinforcement learning, with the idea that alterations in reinforcement learning and/or motivation might inform our understanding of what alterations co-occur with potential antidepressant effects. Macaques performed a reinforced/punished matching pennies task while under effects of saline or ketamine administration and the data were fit to a series of reinforcement learning models to determine which model described behavior under saline most closely and then what parameters of this best-fitting model were altered by ketamine. They found a mixed effect, with two out of three macaques primarily exhibiting an effect of ketamine on processing of losses and one out of three macaques exhibiting an effect of ketamine on processing of losses and perseveration. They found that these effects of ketamine appeared to be dissociable from the nystagmus effects of the ketamine.

      The findings are novel and the data suggesting that ketamine is primarily having its effects on processing of losses (under the procedures used) are solid. However, it is unclear whether the connection between processing of losses and the antidepressant effects of ketamine is justified and the current findings may be more useful for those studying reinforcement learning than those studying depression and antidepressant effects. In addition, the co-occurrence of different behavioral procedures with different patterns of ketamine effects, with one macaque tested with different parameters than the other two exhibiting effects of ketamine that were best fit with a different model than the other two macaques, suggests that there may be difficulty in generalizing these findings to reinforcement learning more generally.

      (1) First, the authors should be more explicit and careful in the connection they are trying to make about the link between loss processing and depression. The authors call their effect a "robust antidepressant-like behavioral effect" but there are no references to support this or discussion of how the altered loss processing would relate directly to the antidepressant effects.

      We agree with the reviewer’s point on the way we made the connection between the study results and ketamine’s antidepressant action. This concern overlaps with the reviewer #1’s concern. Please refer to our response 2, 2-1, 2-2 and 2-3.

      (2) It appears that the monkey P was given smaller rewards and punishers than the other two monkeys and this monkey had an effect of ketamine on perseveration that was not observed in the other two monkeys. Is this believed to be due to the different task, or was this animal given a different task because of some behavioral differences that preceded the experiment? The authors should also discuss what these differences may mean for the generality of their findings. For example, might there be some set of parameters where ketamine would only alter perseveration and not processing of losses?

      Although the best-fitting ketamine model for monkey P includes an additional element – perseveration, we believe that monkey P’s baseline behavior and ketamine’s effect are not significantly different from the other two monkeys for the following reasons.

      First, monkey P was the first animal that we tested ketamine’s effect, and therefore we aimed to match the other two monkeys’ baseline behavior similar to monkey P’s behavior in order to reduce variability in ketamine’s effect potentially attributable to the difference in baseline behavior before pharmacological manipulation. We had to adjust the payoff matrix for the subsequent animals (Y and B) because these monkeys were more sensitive to loss, and seldom chose “risky” target (yielding loss). In order to make the other two monkeys’ behavior similar to that of monkey P, we adjusted the asymmetry between the risky and the safe target in the way that loss (neutral) outcome occurred from the safe (risky) target as well. Eventually, this adjustment made the baseline behavior similar across all three monkeys. The goal of the study was to reliably measure the ketamine’s effect, and not to study individual differences that can naturally occur with the same task parameters. Therefore, we believe that the adjustment of payoff matrix helped to reliably detect ketamine’s effect starting from the common baseline behavior.

      Second, the best-fitting model for monkey P (K-model 7) and that for the other two monkeys (K-model 4) make very similar predictions both qualitatively and quantitatively as are seen in the revised Figure 4. The parameters for outcome values estimated from these two models in monkey P are very similar as is seen in the revised Table 3. In addition, the difference in BIC between the model which includes only perseveration modulation (K-model 6) and the model incorporating outcome value modulation as well (K-model 7) is 441, whereas the difference in BIC between K-model 7 and the model that includes only outcome value modulation (K-model 4) is as small as 4. These BIC results indicate that the variability explained by ketamine’s modulation of outcome evaluation is remarkably larger that that explained by its modulation of perseveration in monkey P.

      Therefore, we conclude that ketamine’s effect was not significantly different between monkey P and the other two monkeys. We clarified this in the revised manuscript by adding the following paragraph in the Result section:

      “Unlike monkey Y and B, the best-fitting model for monkey P indicated that ketamine increased overall tendency to switch choice in addition to outcome-dependent modulation of outcome evaluation. However, BIC differed only slightly (dBIC = 3.99) between the best-fitting (K-model 7) and the second-best model (K-model 4) and the model predictions for choice behavior were very similar both qualitatively and quantitatively (Table 3, Figure 4). We conclude that the behavioral effects of ketamine were consistent across all three monkeys.”

      (3) The authors should discuss whether the plasma ketamine levels they observed are similar to those seen with rapid antidepressant ketamine or are higher or lower.

      We added a sentence in the first paragraph of the Result section as follows with a reference.

      “Plasma concentration and its time course over 60 minutes were also comparable to those measured after 0.5mg/kg in human subjects (35).”

      (35) Zarate CA, Brutsche N, Laje G, Luckenbaugh DA, Venkata SLV, Ramamoorthy A, et al (2012): Relationship of ketamine’s plasma metabolites with response, diagnosis, and side effects in major depression. Biol Psychiatry, 72: 331-338.

      (4) For Figure 4 or S3, the authors should show the data fitted to model 7, which was the best for one of the animals.

      We added the parameters and model predictions from both K-model 7 and K-model 4 for monkey P to help comparison between two models in Table 3, and Figure 4. Revised Table 3 and Figure 4 are as follows:

      Author response table 1.

      Maximum likelihood parameter estimates of the best models for saline and ketamine sessions.

      In all three animals, the model incorporating valence-dependent change in outcome evaluation best fit the choice data from ketamine sessions with (K-model 7 in the parenthesis, P) or without (K-model 4, P and Y/B) additional change in the tendency of choice perseveration (Figure 3, Table 3).

      Author response image 1.

      ketamine-induced behavioral modulation simulated with differential forgetting model (for saline session) and best-fitting K-model (for ketamine session).

    1. Author response:

      The following is the authors’ response to the original reviews.

      Response to Public Comments

      (1) BioRxiv version history.

      Reviewer 1 correctly noted that we have posted different versions of the paper on bioRxiv and that there were significant changes between the initial version and the one posted as part of the eLife preprint process. Here we provide a summary of that history.

      We initially posted a bioRxiv preprint in November, 2021 (Version I) that included the results of two experiments. In Experiment 1, we compared conditions in which the stimulation frequency was at 2 kHz, 3.5 kHz, or 5.0 kHz. In Experiment 2, we replicated the 3.5 kHz condition of Experiment 1 and included two amplitude-modulated (AM) conditions, with a 3.5 kHz carrier signal modulated at 20 Hz or 140 Hz. Relative to the sham stimulation, non-modulated kTMP at 2 kHz and 3.5 kHz resulted in an increase in cortical excitability in Experiment 1. This effect was replicated in Experiment 2.

      In the original posting, we reported that there was an additional boost in excitability in the 20 Hz AM condition above that of the non-modulated condition. However, in re-examining the results, we recognized that the 20 Hz AM condition included an outlier that was pulling the group mean higher. We should have caught this outlier in the initial submission given that the resultant percent change for this individual is 3 standard deviations above the mean. Given the skew in the distribution, we also performed a log transform on the MEPs (which improves the normality and homoscedasticity of MEP distributions) and repeated the analysis. However, even here the participant’s results remained well outside the distribution. As such, we removed this participant and repeated all analyses. In this new analysis, there was no longer a significant difference between the 20 Hz AM and non-modulated conditions in Experiment 2. Indeed, all three true stimulation conditions (non-modulated, AM 20 Hz, AM 140 Hz) produced a similar boost in cortical excitability compared to sham. Thus, the results of Experiment 2 are consistent with those of Experiment 1, showing, in three new conditions, the efficacy of kHz stimulation on cortical excitability. But the results fail to provide evidence of an additional boost from amplitude modulation. 

      We posted a second bioRxiv preprint in May, 2023 (Version 2) with the corrected results for Experiment 2, along with changes throughout the manuscript given the new analyses.

      Given the null results for the AM conditions, we decided to run a third experiment prior to submitting the work for publication. Here we used an alternative form of amplitude modulation (see Kasten et. al., NeuroImage 2018). In brief, we again observed a boost in cortical excitability in from non-modulated kTMP at 3.5 kHz, but no additional effect of amplitude modulation.  This work is included in the third bioRrxiv preprint (Version 3), the paper that was submitted and reviewed at eLife.

      (2) Statistical analysis.

      Reviewer 1 raised a concern with the statistical analyses performed on aggregate data across experiments.  We recognize that this is atypical and was certainly not part of an a priori plan. Here we describe our goal with the analyses and the thought process that led us to combine the data across the experiments.

      Our overarching aim is to examine the effect of corticospinal excitability of different kTMP waveforms (carrier frequency and amplitude modulated frequency) matched at the same estimated cortical E-field (2 V/m). Our core comparison was of the active conditions relative to a sham condition (E-field = 0.01 V/m). We included the non-modulated 3.5 kHz condition in Experiments 2 and 3 to provide a baseline from which we could assess whether amplitude modulation produced a measurable difference from that observed with non-modulated stimulation. Thus, this non-modulated condition as well as the sham condition was repeated in all three experiments. This provided an opportunity to examine the effect of kTMP with a relatively large sample, as well as assess how well the effects replicate, and resulted in the strategy we have taken in reporting the results. 

      As a first step, we present the data from the 3.5 kHz non-modulated and sham conditions (including the individual participant data) for all three experiments in   4. We used a linear mixed effect model to examine if there was an effect of Experiment (Exps 1, 2, 3) and observed no significant difference within each condition. Given this, we opted to pool the data for the sham and 3.5 kHz non-modulated conditions across the three experiments. Once data were pooled, we examined the effect of the carrier frequency and amplitude modulated frequency of the kTMP waveform. 

      (3) Carry-over effects

      As suggested by Reviewer 1, we will examine in the revision if there is a carry-over effect across sessions (for the most part, 2-day intervals between sessions). For this, we will compare MEP amplitude in baseline blocks (pre-kTMP) across the four experimental sessions.

      Reviewer 1 also commented that mixing the single- and paired-pulse protocols might have impacted the results. While our a priori focus was on the single-pulse results, we wanted to include multiple probes given the novelty of our stimulation method. Mixing single- and different paired-pulse protocols has been relatively common in the non-invasive brain stimulation literature (e.g., Nitsche 2005, Huang et al, 2005, López-Alonso 2014, Batsikadze et al 2013) and we are unaware of any reports suggested that mixed designs (single and paired) distort the picture compared to pure designs (single only).

      (4) Sensation and Blinding

      Reviewer 2 bought up concerns about the sham condition and blinding of kTMP stimulation. We do think that kTMP is nearly ideal for blinding. The amplifier does emit an audible tone (at least for individuals with normal hearing) when set to an intensity to produce a 2 V/m E-field. For this reason, the participants and the experimenter wore ear plugs. Moreover, we played a 3.5 kHz tone in all conditions, including the sham condition, which effectively masked the amplifier sound. We measured the participant’s subjective rating of annoyance, pain, and muscle twitches after each kTMP session (active and sham). Using a linear mixed effect model, we found no difference between active and sham for each of these ratings suggesting that sensation was similar for active and sham (Fig 8). This matches our experience that kHz stimulation in the range used here has no perceptible sensation induced by the coil. To blind the experimenters (and participants) we used a coding system in which the experimenter typed in a number that had been randomly paired to a stimulation condition that varied across participants in a manner unknown to the experimenter.

      Reviewer 1 asked why we did not explicitly ask participants if they thought they were in an active or sham condition. This would certainly be a useful question. However, we did not want to alert them of the presence of a sham condition, preferring to simply describe the study as one testing a new method of non-invasive brain stimulation. Thus, we opted to focus on their subjective ratings of annoyance, pain, and finger twitches after kTMP stimulation for each experimental session.

      Response to Recommendations for the Authors

      Reviewer #1: 

      Reviewer # 1 in the public review noted the possibility of carry-over effects and suggested that we compare the amplitude of the MEPS in the pre blocks across the four sessions.

      Although we did not anticipate carry-over effects lasting 2 or more days, we have now conducted an analysis in which we use a linear mixed effect model with a fixed factor of Session and a random factor of Participant. The results show that there is not an effect of session [χ2(3) = 4.51, p \= 0.211].

      Author response table 1.

      Detailed comments and some suggestions to maybe improve the writing and figures: 

      Abstract: 

      BioRxiv Version 1: "We replicated this effect in Experiment 2 and found that amplitude-modulation at 20 Hz produced an additional boost in cortical excitability. " 

      BioRxiv Version 2, 3 and current manuscript: "Although amplitude-modulated kTMP increased MEP amplitude compared to sham, no enhancement was found compared to non-modulated kTMP." 

      I am a little concerned about this history because the conclusions seem to have changed. It looks like the new data has a larger number of subjects, which could explain the divergence. Although it is generally not good practice to analyze the data at interim time points, without accounting for alpha spending. It appears that data analysis methods may have also changed, as some of the extreme points in version 1 seem to be no longer in the new manuscript (Figure 4 Sham Experiment 1). 

      In the public review above we explain in detail the different versions of the bioRxiv preprint and how the results changed from the first version to the current manuscript.

      Introduction: <br /> "Second, the E-fields for the two methods exist in orthogonal subspaces" Can you explain what this means? 

      Thank you for this suggestion, we have updated the paper (pg. 4, line 78-81) by adding two sentences to explain what we mean by orthogonal subspaces and describe the consequences of this with respect to the E-fields resulting from tES and TMS. Specifically, we now comment that even if the E-fields of tES and TMS are similar in focality, they may target different populations of neurons.  

      "In addition, the kTMP waveform can be amplitude modulated to potentially mimic E-fields at frequencies matching endogenous neural rhythms [15]." That may be so, but reference [15] makes the exact opposite point, namely, that kHz stimulation has little effect on neuronal firing until you get to very strong fields. The paper that makes that claim is by Nir Grossman, but in my view, it is flawed as responses are most likely due to peripheral nerve (axon) stimulation there given the excessive currents used in that study. The reference to Wang and Peterchev [17] is in agreement with that by showing that you need 2 orders of magnitude stronger fields to activate neurons. 

      The reviewers are correct that that Ref 15 (Esmaeilpour et al, 2021), as well as Wang et al, 2023 use much higher E-fields than we target in our present study. However, our point here is that, while we cannot use our approach to apply E-fields at endogenous frequencies, we can do amplitude modulation of the kHz carrier frequency at these lower frequencies. We cited Esmaeilpour et al., (2021) because they show that high frequency stimulation with amplitude-modulated waveforms resulted in dynamic modulation at the “beating” frequency. Given we are well in subthreshold space in this paper, and well below the E-field levels in Esmaeilpour et al (2021), the open question is whether amplitude modulation at this level will be able to perturb neural activity (e.g., increase power of endogenous oscillations at the targeted frequency). 

      To address this concern, we modified the sentence (pg.6, lines 120-121) to now read "In addition, the kTMP waveform can be amplitude modulated at frequencies matching endogenous neural rhythms." In this way, we are describing a general property of kTMP (as well as other methods that can use high frequency signals).

      I am not aware of any in-vitro study showing the effects of kHz stimulation at 2V/m. The review paper by Neudorfer et al is very good. But if I got it correctly in a quick read it is not clear that there is experimental evidence for subthreshold effects. They do talk about facilitation, but the two experimental papers cited there on the auditory nerve don't quantify field magnitudes. I would really love it if you could point me to a relevant empirical study showing the effects of kHz stimulation at 2 V/m. 

      Perhaps all this is a moot point as you are interested in lasting (plastic) effects on MEP. For this, you cite one study with 11 subjects showing the effects of kHz tACS on MEPs [20]. I guess that is a start. The reference [21] is only a safety study, so it is probably not a good reference for that. Reference [22] also seems out of place as it is a modeling study. The effects on depression of low-intensity magnetic stimulation in references [23-26] are intriguing. 

      We agree with the reviewer that Ref 20 (now Ref 18: Chaieb, Antal & Paulus; 2011) is the most relevant one to cite here since it provides empirical evidence for changes in neural excitability from kHz stimulation, and in fact, serves as the model for the current study. We have retained Refs 23-26 (now Ref 19-22: Rohan et al., 2014; Carlezon et al., 2005; Rohan et al., 2004 & Dublin et al., 2019) since they also do show kHz effects on mood and removed Refs 21 (Chaieb et al., 2014) and 22 (Wang et al., 2018) for the reasons cited by the Reviewer.

      Figure 1: "The gray dashed function depicts the dependence of scalp stimulation threshold upon frequency [14]." It's hard to tell from that reference what the exact shape is, but the frequency dependence is likely steeper than what is shown here, i.e. 2 mA at 10 Hz can be really quite unpleasant. 

      We have removed the gray dashed line given that this might be taken to suggest a discrete transition. We now just have a graded transition to reflect that the tolerance of tES is subjective. We start the shading at 2 mA for the lowest frequencies given that there is general agreement that 2 mA is well-tolerated and decrease the shading intensity as frequency increases. The general aim of the figure is not to make strong claims about the threshold of scalp discomfort for tES, but to show that kTMP can target much higher cortical E-fields within the tolerable range.

      Methods: <br /> Procedures: <br /> It does not seem like double-blinding has been directly assessed. 

      We did not assess double blinding by directly assessing whether the participant was in a sham or active condition. We did not want to alert the participants of the presence of a sham condition after the first session of the 4-session study, preferring to simply describe the study as a test of a new method of non-invasive brain stimulation. For this reason, we opted to focus on their subjective ratings of annoyance, pain, and finger twitches after kTMP stimulation for each experimental session. These ratings did not differ between active and sham kTMP, which suggests kTMP has good potential for double blinding.

      MEP data analysis: Taking the mean of log power is unusual, but I suppose the reference provided gives a good justification. Does this explain the deviation from the biorxiv v1 results? 

      We opted to perform a logarithmic transformation of MEP amplitudes to improve the normality and homoscedasticity of the MEP distribution. We cite three papers (Refs 50-52: Peterchev et al., 2013, Nielsen 1996a, & Nielsen 1996b) that have applied a similar approach in handling MEP data. We had not done the transformation in the first bioRxiv but opted to do so in the eLife submission based on further review of the literature. We note that the two analyses produce similar statistical outcomes once we removed the outlier discussed in the Public Review.

      "Interactions were tested by comparing a model in which the fixed effects were restricted to be additive against a second model that could have multiplicative and additive effects." Not sure what this means. Why not run a full model with interactions included and read off the stats from that single model for the various factors? Should one not avoid running multiple models as one would have to correct p-values for multiple comparisons for every new test? 

      We used the lme4 package in R to fit our linear mixed effect models (Ref 54: Bates, Mächler, Bolker & Walker, 2015). In this package they intentionally leave out p-values for individual models or factors because they note there is a lack of convergence in the field about how to calculate parameter estimates in complex situations for linear mixed effect models (e.g., unbalanced designs). They suggest model comparison using the likelihood-ratio test to obtain and report p-values, which is what we report in the current manuscript.

      We revised the text in the section Linear Mixed Effects Models to state that likelihood ratio tests were used to obtain p-values to remove any confusion.

      Procedures: <br /> kTPM: Nice that fields were measured. Would be nice to see the data that established the empirical constant k. 

      We have expanded our discussion of how we established k in the Methods section. We first derived k using the equation E0 \= kfcI based on previously published reports of the current (I) and frequency (fc) of the MagVenture Cool-B65 coil (now Refs 29-30: Deng, Lisanby & Peterchev, 2013; Drakaki, Mathiesen, Siebner, Madsen & Thielscher, 2022). We then verified this value using the triangular E-field probe to within 5% error.

      Figure 3, spectrum. The placement of the fm label on the left panel is confusing. It suggests that fm was at the edge of the spectrum shown, which would not be the best way to show that there is nothing there - obviously, there isn't, but the figure could be more didactic. 

      Thanks for pointing this out. We modified the figure, moving the ‘fm’ label to the center of the first panel. This change makes it clear that there is no peak at the amplitude modulated frequency.

      "a trio of TMS assays of cortical excitability" Can you clarify what this means? 

      Sorry for the confusion. The trio of TMS assays refers to the single pulse and two paired-pulse protocols (SICI - ICF). We edited the Procedure section to clarify this (pg 9, line 195-197).

      Figure 2A: it would be nice to indicate which TMS blocks were single pulse and which were the two paired-pulse protocols. It is hard to keep track of it all for the three different experiments. 

      We have now clarified in the text (see above) that all three probes were used in each block for Experiments 1 and 2, and only the single-pulse probe in Experiment 3. We have modified the legend for Figure 2 to also provide this information.

      Results: <br /> "Based on these results, we combined the data across the three experiments for these two conditions in subsequent analyses." This strikes me as inappropriate. Should not a single model have been used with a fixed effect of experiment and fixed effect of stimulation condition? 

      We recognize that pooling data across experiments may be atypical. Indeed, our initial plan was to simply analyze each experiment on its own (completely within-subject analysis). However, after completing the three experiments, we realized that since the sham and non-modulated 3.5 kHz conditions were included in each experiment, we had an opportunity to examine the effect of kTMP in a relatively large N study (for NIBS research). Before pooling the data, we wanted to make sure that the factor of experiment did not impact the results and our analysis showed there was no effect of experiment. Note that we did not include the factor of stimulation condition in this model because we did not want to do multiple comparisons of the same contrast (3.5 kHz compared to sham). By pooling the data before analysis of the stimulation conditions we could then focus on our two key independent variables: 1) kTMP carrier frequency and 2) kTMP amplitude modulated frequency, doing fewer significance tests to minimize multiple comparisons. The linear mixed effect (LME) model allows us to include a random effect of participant. In this way, we account for the fact that some comparisons are within subjects and some comparisons are between subjects.

      The reviewer is correct that after pooling the data, we could have continued to include the factor of experiment in the LME models. This factor could still account for variance even though it was not significant in the initial test. Given this, we have now reanalyzed the data including the fixed factor of experiment in all the comparisons that contain data from multiple experiments. This has led us to modify the text in the Methods section under Linear Mixed Effects Models and in the Results section under Repeated kTMP Conditions (3.5 kHz and Sham) across Experiments. In addition, the results of the LME models have been updated throughout the Results section. We note that the pattern of results was unchanged with this modification of our analyses.

      "Pairwise comparisons of each active condition to sham showed that an increase was observed following both 2 kHz ..." I suppose this is all for Experiment 1? It is a little confusing to go back and forth between combining experiments and then separate analyses per experiment without some guiding text, aside from being a bit messy from the statistical point of view. 

      We did not go back to performing separate analyses of the experiments after pooling the data. Once we ran the test to justify pooling the data, subsequent tests were done with the pooled data to evaluate the effects of carrier frequency and amplitude modulation.

      Figure 5 is confusing because the horizontal lines with ** on top seem to refer to the same set of sham subjects, but the subjects of Experiments 2 and 3 are different from Experiment 1, so in these pairwise comparisons there is a mix of between-subject and within subject-comparison going on here. Did I get that right? 

      Yes – that is correct. As noted above we pooled the data after showing that there was no effect of experiment. Thus, the data for the sham and 3.5 kHz non-modulated conditions are from three different experiments. There was some overlap of subjects in Experiments 1 and Experiment 2 (Experiment 3 was all new participants).  We used a linear mixed effect model so that we could account for this mixed design. Participant was always included as a random factor, which allows us to account for the fact that some comparisons are within, and some are between. Based on a previous comment, we now include Experiment as a fixed factor (see above) which provides a way to evaluate variance across the different experiments.

      "We next compared sham vs. active non-modulated kTMP and found that active kTMP produced a significant increase in corticospinal excitability [χ2(1) = 23.46 p < 0.001" Is this for the 3.5Hz condition? 

      No, that is for an omnibus comparison of non-modulated kTMP (including 2 kHz, 3.5 kHz and 5 kHz conditions) vs. sham. We have edited the paper to include the three conditions that are included as the active non-modulated kTMP conditions for clarity (pg. 22, line 463). Having observed a significant omnibus result, we continued with paired comparisons: “Pairwise comparisons of each active condition to sham showed that an increase was observed following both 2 kHz [χ2(1) = 6.90, p = 0.009; d = 0.49] and 3.5 kHz kTMP [χ2(1) = 37.75, p < 0.001; d = 0.70; Fig 5: Non-Modulated conditions]. The 5 kHz condition failed to reach significance [χ2(1) = 1.43, p = 0.232; d = 0.21].”

      Paired-Pulse Assays: There are a number of results here without pointing to a figure, and at one point there is a reference to Figure 6, which may be in error. It would help to point the reader to some visual corresponding the the stats. 

      Thank you. This was an error on line 542. It should have read Figure 7. We have added two other pointers to Figure 7 where we discuss the absence of an effect of kTMP on SICI.

      Reviewer #2 (Recommendations For The Authors):

      I would recommend a couple of changes to the background.

      "Orthogonal subspaces" line 78. This is a fairly formal term that has little relevance here, although the difference between scalar and vector potential-based fields is interesting to think about. If it stays, it should be mathematically supported, but it's easily rewritten to deliver the gist of it. 

      We have updated the paper by adding text that we hope will clarify what we mean by orthogonal subspaces (pg. 4, line 78-81). We note that we developed the math behind this statement in a previous paper (Ref # 10: Sheltraw et al., 2021). We have changed the location of the citation so that it directly follows these sentences and will provide a pointer to readers interested in the physics and math concerning orthogonal subspaces. 

      The statement that the scalp e-field for TES is greater than the e-field for TMS for similar cortical fields needs a little more clarification, since historically they have operated orders of magnitude apart, and it is easy to misread and trip over this statement (although it is factually true). Presenting a couple of numbers at cortical and scalp positions would help illustrate the point. That you are not considering applying TES at traditional TMS levels but rather TMS at TES values is what is initially easy to miss. 

      We appreciate the feedback and have updated this section to provide the reader with a better intuition of this point. We now specify that the scalp to cortical E-field ratio is approximately 18 times larger for tES compared to TMS and cite our previous paper which has much more detail about how this was calculated.

      A note that the figures show scalp sensation around 1.0 V/m while the text states 0.5; cortical depths are an important thing for the reader to keep in mind. 

      This comment, when considered in tandem with one of the comments of Reviewer 1 led us to revise Figure 1. We removed the dashed gray line which might be taken to suggest a strict cutoff in terms of tolerability (which we did not intend). We now use shading that fades away to make the point of continuity. We have extended this down to a cortical E-field of 0.5 V/m to correspond with the text.  

      This is a nicely done and carefully reported experiment and I look forward to seeing more. 

      Thank you for your kind note!

    1. Author Response

      The following is the authors’ response to the original reviews.

      Reviewer 1

      Summary:

      In the present study, authors found the ternary complex formed by NCAN, TNC, and HA as an important factor facilitating the multipolar to bipolar transition in the intermediate zone (IZ) of the developing cortex. NCAM binds HA via the N-terminal Link modules, meanwhile, TNC cross-links NCAN through the CDL domain at the C-terminal. The expression and right localization of these three factors facilitate the multipolar-bipolar transition necessary for immature neurons to migrate radially. TNC and NCAM are also involved in neuronal morphology. The authors used a wide range of techniques to study the interaction between these three molecules in the developing cortex. In addition, single and double KO mice for NCAN and TNC were analyzed to decipher the role of these molecules in neuronal migration and morphology.

      Strengths:

      The study of the formation of the cerebral cortex is crucial to understanding the pathophysiology of many neurodevelopmental disorders associated with malformation of the cerebral cortex. In this study, the authors showed, for the first time, that the ternary complex formed by NCAN, TNC, and HA promotes neuronal migration. The results regarding the interaction between the three factors forming the ternary complex are convincing.

      We appreciate the reviewers' positive assessment of our research.

      Weaknesses:

      However, regarding the in vivo experiments, the authors should consider some points for the interpretation of the results:

      • The authors did not use the proper controls in their experiments. For embryonic analysis, such as cortical migration, neuronal morphology, and protein distribution (Fig. 6, 7, and 9), mutant mice should be compared with control littermates, since differences in the results could be due to differences in embryonic stages. For example, in Fig. 6 the dKO is more developed than the WT embryo.

      It was challenging to compare double knockout mice with control littermates. When crossing Ncan and Tcn double heterozygous mice, the probability of obtaining double knockout mice is 1/16. Given an average litter size of around 8, acquiring a substantial number of double knockout mice would necessitate an impractical number of breeding pairs. Consequently, we were constrained to use non-littermate control mice. To address potential differences in developmental stages, we analyzed 19-20 embryos obtained from five individuals in each group, demonstrating that the observed differences between the two groups are more substantial than the inherent variability within each group.

      • The authors claim that NCAM and TNC are involved in neuronal migration from experiments using single KO embryos. This is a strong statement considering the mild results, with no significant difference in the case of TNC KO embryos, and once again, using embryos from different litters.

      We agree with the reviewer's comment that a single deletion of TNC has a minimal impact on neuronal migration. We have revised the Results section to reflect the mild nature of the TNC KO phenotype more accurately.

      Page 8, line 225: "In NCAN KO mice, a significantly lower percentage of labeled cells resided in the upper layer (Bin2), and more cells remained in the lower layer (Bin5) than in WT mice (Figure 7a). In contrast, the impact of a single deletion of TNC on neuronal cell migration was minimal. Although TNC KO mice exhibited a tendency to have a higher proportion of labeled cells in the lower layer (Bin4) than in WT mice, this did not reach statistical significance (Figure 7a). The delay in neuronal migration observed in the single KO mice was milder when compared to that observed in DKO mice (Figure 6a-c), suggesting that simultaneous deletion of both NCAN and TNC is necessary for a more pronounced impairment in neuronal cell migration."

      • The measurement of immunofluorescence intensity is not the right method to compare the relative amount of protein between control and mutant embryos unless there is a right normalization.

      We agree that measuring immunofluorescence intensity alone is insufficient for comparing the relative amount of protein. In Figure 8, we have employed Western blotting to compare the protein levels, revealing an approximately 50% reduction in NCAN and TNC following hyaluronidase digestion. In Figures 7b and 7c, we demonstrated alterations in the localization patterns of TNC and NCAN in Ncan KO and Tnc KO mice; however, we did not mention their quantity.

      • Page 7, line 206. "No significant abnormalities were observed in the laminar structure in 4-week-old DKO mice". The authors should be more careful with this statement since they did not check the lamination of the adult cortex. I would recommend staining, control and mutant mice, with markers of different cortical populations, such as Cux1, Ctip2, Tbr1, to asses this point.

      In response to the suggestion, we have conducted additional experiments to provide a more detailed examination of the laminar structure in the cerebral cortex. The results have been incorporated into the revised manuscript as follows:

      Page 7, line 209: "To investigate the laminar organization of the postnatal cerebral cortex, we analyzed the distribution of NeuN-positive postmitotic neurons in DKO mice at 2 weeks of age. No notable abnormalities were observed in the laminar structure of DKO mice (Figure 6-figure supplement 3a, b). Additionally, the laminar distribution of Ctip2-positive deep layer neurons showed no significant differences between WT and DKO mice (Figure 6-figure supplement 3a, c)."

      • The authors do not explain how they measured the intensity of TNC around the transfected Turbo-RFP-positive neurons.

      We added the following description to the Materials and Methods:

      Page 18, line 608: "Images were captured in the IZ region containing Turbo-RFP-positive neurons using a 100X magnification objective lens with 3.0X optical zoom on an AX R confocal microscope (Nikon). A total of 10 optical sections were acquired with a step size of 190 nm. Z-projection views were generated, and the staining intensity of TNC around Turbo-RFP-positive neurons was measured in a 59 × 59 µm area using ImageJ FIJI."

      • The loading control of the western blots should be always included.

      In Figure 6-figure supplement 1, we have incorporated western blot data using a GAPDH antibody as a loading control. We have added an explanation in the figure legend of Figure 3c, stating that we analyzed the same samples as those used in Figure 1e.

      • For Fig. 3e, I think values are represented relative to E18 instead to P2.

      Thank you for pointing that out. As suggested, we have corrected the representation in Fig. 3e to be relative to E18 instead of P2.

      • I would recommend authors use the standard nomenclature for the embryonic stages. The detection of the vaginal plug is considered as E0.5 and therefore, half a day should be added to embryonic stages (E14.5...).

      We have revised our manuscript to designate the detection of the vaginal plug as E0.5, and subsequently, we have adjusted all embryonic stages by adding half a day, such as E14.5.

      • Fig 10K: I do not see the differences in the number of neurites in the graph.

      We have modified the presentation from a box-and-whisker plot to a bar graph to enhance the visibility of differences in the average number of neurites.

      • Line 37: Not all of the cerebral cortex is structured in 6 layers but the neocortex.

      We have changed 'cerebral cortex' to 'cerebral neocortex.'

      Reviewer 2

      Summary:

      ECM components are prominent constituents of the pericellular environment of CNS cells and form complex and dynamic interactomes in the pericellular spaces. Based on bioinformatic analysis, more than 300 genes have been attributed to the so-called matrisome, many of which are detectable in the CNS. Yet, not much is known about their functions while increasing evidence suggests important contributions to developmental processes, neural plasticity, and inhibition of regeneration in the CNS. In this respect, the present work offers new insights and adds interesting aspects to the facets of ECM contributions to neural development. This is even more relevant in view of the fact that neurocan has recently been identified as a potential risk gene for neuropsychiatric diseases. Because ECM components occur in the interstitial space and are linked in interactomes their study is very difficult. A strength of the manuscript is that the authors used several approaches to shed light on ECM function, including proteome studies, the generation of knockout mouse lines, and the analysis of in vivo labeled neural progenitors. This multi-perspective approach permitted to reveal hitherto unknown properties of the ECM and highlighted its importance for the overall organization of the CNS.

      Strengths:

      Systematic analysis of the ternary complex between neurons, TNC, and hyaluronic acid; establishment of KO mouse lines to study the function of the complex, use of in utero electroporation to investigate the impact on neuronal migration;

      We appreciate the reviewers' insightful comments.

      Weaknesses:

      The analysis is focused on neuronal progenitors, however, the potential impact of the molecules of interest, in particular, their removal on differentiation and /or survival of neural stem/progenitor cells is not addressed. The potential receptors involved are not considered. It also seems that rather the passage to the outer areas of the forming cortex is compromised, which is not the same as the migration process. The movement of the cells is not included in the analysis.

      In this study, we demonstrated that the ternary complex of NCAN, TNC, and HA is predominantly localized in the subplate/intermediate zone. This region lacks neural stem/progenitor cells but serves as the initiation site for the radial migration of postmitotic neurons. Consequently, our study focused on the role of the ternary complex in neuronal migration and polarity formation. We acknowledge that we did not investigate in-depth the potential effects of ECM perturbation on the differentiation and survival of neural stem/progenitor cells. However, as highlighted by the reviewer, it is important to explore the effects on neural stem/progenitor cells. To address this concern, we analyzed Pax6-positive radial glial cells and Tbr2-positive intermediate progenitor cells in the ventricular zone of wild-type and Ncan/Tnc double knockout (DKO) mice. Immunohistochemical analysis revealed no significant differences between WT and DKO mice (Figure 6-figure supplement 4a). Furthermore, the morphology of nestin-positive radial fibers exhibited no distinguishable variations between WT and DKO mice (Figure 6-figure supplement 4b, c).

      (1) In the description of the culture of cortical neurons the authors mentioned the use of 5% horse serum as a medium constituent. HS is a potent stimulus for astrocyte differentiation and astrocytes in vitro release neurocan. Therefore, the detection of neurocan in the supernatant of the cultures as shown in Figure 1h might as well reflect release by cultivated astrocytes.

      As pointed out by the reviewer, Figure 1h did not conclusively demonstrate that neurons are the sole source of NCAN production. Indeed, in situ hybridization analysis revealed the widespread distribution of Ncan mRNA throughout the cerebral cortex (Figure 2a). This result suggests that the production of NCAN involves not only neurons but also other cell populations, including radial glial cells and astrocytes. While we acknowledge the potential contribution of other cell types to NCAN production, Ncan expression by neurons during radial migration is a crucial aspect of our findings (Figure 1i, j). We have revised the manuscript as follows:

      Page 5, line 111: "This result suggested the secretion of NCAN by developing neurons; however, we cannot rule out the involvement of coexisting glial cells in the culture system. To investigate the expression of Ncan mRNA during radial migration in vivo, we labeled radial glial cells in the VZ with GFP through in utero electroporation at E14.5 (Figure 1i, Figure 1-figure supplement 1)."

      (2) It is known that neurocan in vivo is expressed by neurons, but may be upregulated in astrocytes after lesion, or in vitro, where the cells become reactive.

      We have incorporated the following description into the discussion:

      Page 11, line 359: "Previous studies have reported an upregulation of NCAN and TNC in reactive astrocytes, indicating the potential formation of the ternary complex of NCAN, TNC, and HA in the adult brain in response to injury (Deller et al., 1997; Haas et al., 1999)."

      (3) Do NCAN KO neurons show an increase in neurite growth on the TNC substrates? The response on POL was changed (Fig. 10h-k), but the ECM substrates were not tested with the KO neurons.

      The impact of ECM substrates on NCAN KO neurons has not been investigated, and this remains an avenue for further exploration in our ongoing research. Future studies aim to elucidate the NCAN-TNC connection by identifying TNC cell surface receptors and unraveling the subsequent intracellular signaling pathways.

      (4) Do the authors have an explanation for why the ternary complex is concentrated in the SP/IZ zone?

      In the mature brain, hyaluronan acts as a scaffold that facilitates the accumulation of ECM components, including proteoglycans and tenascins around neurons. Therefore, it is conceivable that the ECM components bind to hyaluronan in the embryonic brain, resulting in its accumulation in the subplate/intermediate zone. In support of this hypothesis, enzymatic digestion of hyaluronan in the subplate/intermediate zone led to the disappearance of TNC and NCAN accumulation (Figure 8a-c). This result may account for the disparity observed, where Tnc mRNA is expressed in the ventricular zone while the TNC protein localizes to the subplate/intermediate zone.

      (5) Are hyaluronic acid synthesizing complexes (HAS) concentrated in the SP/IZ?

      According to the reviewer's comment, we have investigated the localization of Has2 and Has3 mRNA using in situ hybridization. However, due to the relatively low expression levels of these enzymes, we encountered challenges in obtaining clear signals (Author response image 1). Further research is needed to understand the mechanisms behind the localization of hyaluronan in the intermediate zone.

      Author response image 1.

      In situ hybridization analysis of Has2 and 3 mRNA on the E16.5 cerebral cortex. Upper images show results of in situ hybridization using antisense against Has2 and 3. Lower images are in situ hybridization using sense probes as negative controls.

      (6) CSPGs as well as TNC are part of the neural stem/progenitors cell niche environment. Does the removal of either of the ECM compounds affect the proliferation, differentiation, and/or survival of NSPCs, or their progeny?

      )7) This question relates to the fact that the migration process itself is not visualized in the present study, rather its outcome - the quantitative distribution of labeled neurons in the different bins of the analysis. This could also derive from modified cell numbers.

      As pointed out by the reviewer, previous studies have shown the role of CSPGs and TNC as components of the neural stem/progenitor cell niche (see reviews by (Faissner et al., 2017; Faissner and Reinhard, 2015). However, as mentioned in Response #2, based on our analyses, we did not observe a reduction in neural stem/progenitor cells in NCAN/TNC double-knockout mice. While we cannot precisely explain this discrepancy, it is worth noting that many past studies evaluated the activities of the ECM molecules in in vitro systems such as neurospheres. The observed differences may stem from variations in experimental systems.

      (8) What is the role of the ECM in the SP/IZ area? Do the cells need the ECM to advance, the reduction would then leave the neuronal progenitors in the VZ area? This somehow contrasts with interpretations that the ECM acts as an obstacle for neurite growth or cell migration, or as a kind of barrier.

      The role of the ECM is multifaceted, with certain ECM molecules known to inhibit neurite outgrowth while others facilitate it. Additionally, the effects of ECM can vary depending on the cell type. It is established that after migrating neurons adhere to radial fibers, they utilize these fibers as a scaffold to migrate toward the cortical surface. However, in the subplate/intermediate zone, migrating neurons have not yet adhered to radial fibers. This study provides evidence that multipolar neurons undergo morphological changes into bipolar cells with the assistance of the NCAN, TNC, and HA complex. Subsequently, this facilitates their movement along radial fibers.

      (9) A direct visualization of the movement of neural progenitors in the tissue as has been for example performed by the Kriegstein laboratory might help resolve some of these issues.

      As suggested by the reviewer, utilizing live imaging techniques to directly observe the movement of neural progenitors within the tissue is indeed a powerful tool. We recognize the significance of addressing these points in future research.

    1. Author Response

      The following is the authors’ response to the original reviews.

      Public Reviews:

      Reviewer #1 (Public Review):

      Summary:

      Zhang et al., investigated the relationship between monocular and binocular responses of V1 superficial-layer neurons using two-photon calcium imaging. They found a strong relationship in their data: neurons that exhibited a greater preference for one eye or the other (high ocular dominance) were more likely to be suppressed under binocular stimulation, whereas neurons that are more equivalently driven by each other (low ocular dominance) were more likely to be enhanced by binocular stimulation. This result chiefly demonstrates the relationship between ocular dominance and binocular responses in V1, corroborating what has been shown previously using electrophysiological techniques but now with greater spatial resolution (albeit less temporal resolution). The binocular responses were well-fitted by a model that institutes divisive normalization between the eyes that accounts for both the suppression and enhancement phenomena observed in the subpopulation of binocular neurons. In so doing, the authors reify the importance of incorporating ocular dominance in computational models of binocular combination.

      The conclusions of this paper are mostly well supported by the data, but there are some limitations of the methodology that need to be clarified, and an expansion of how the results relate to previous work would better contextualize these important findings in the literature.

      Strengths:

      The two-photon imaging technique used to resolve the activity of individual neurons within intact brain tissue grants a host of advantages. Foremost, two-photon imaging confers considerably high spatial resolution. As a result, the authors were able to sample and analyze the activity from thousands of verified superficial-layer V1 neurons. The animal model used, awake macaques, is also highly relevant for the study of binocular combination. Macaques, like humans, are binocular animals, meaning they have forward-facing eyes that confer overlapping visual fields. Importantly, macaque V1 is organized into cortical columns that process specific visual features from the separate eyes just like in humans. In combination with a powerful imaging technique, this allowed the authors to evaluate the monocular and binocular response profiles of V1 neurons that are situated within neighboring ocular dominance columns, a novel feat. To this aim, the approach was well-executed and should instill further confidence in the notion that V1 neurons combine monocular information in a manner that is dependent on the strength of their ocular dominance.

      Weaknesses:

      While two-photon imaging provides excellent spatial resolution, its temporal resolution is often lower compared to some other techniques, such as electrophysiology. This limits the ability to study the fast dynamics of neuronal activity, a well-understood trade-off of the method. The issue is more so that the authors draw comparisons to electrophysiological studies without explicit appreciation of the temporal difference between these techniques. In a similar vein, two-photon imaging is limited spatially in terms of cortical depth, preferentially sampling from neurons in layers 2/3. This limitation does not invalidate any of the interpretations but should be considered by readers, especially when making comparisons to previous electrophysiological reports using microelectrode linear arrays that sample from all cortical layers. Indeed, it is likely that a complete picture of early cortical binocular processing will require high spatial resolution (i.e., sampling from neurons in neighboring ocular dominance columns, from pia mater to white matter) at the biophysically relevant timescales (1ms resolution, capturing response dynamics over the full duration of the stimulus presentation, including the transient onset and steady-state periods).

      To address the same concern from all three reviewers, we discussed the technical limitations of two photon calcium imaging at the end of Discussion, including limited imaging depth, low temporal resolution, and nonlinearity. The relevant texts are copied here:

      (Ln 304) “Limitations of the current study

      Although capable of sampling a large number of neurons at cellular resolution and with low sampling bias, two-photon calcium imaging has its known limitations that may better make it a complementary research tool to electrophysiological recordings.

      For example, two-photon imaging can only sample neurons from superficial-layers, while binocular neurons also exist in deeper layers, and even neurons in the input layer are affected by feedback from downstream binocular neurons to exhibit binocular response properties (Dougherty, Cox, Westerberg, & Maier, 2019). Furthermore, calcium signals are relatively slow and cannot reveal the fast dynamics of neuronal responses. Due to these spatial and temporal limitations, a more complete picture of the neuronal mechanisms underlying binocular combination of monocular responses may come from studies using both technologies.

      In addition, calcium signals may exaggerate the nonlinear properties of neurons. Although calcium signals indicated by GCaMP5, our favored choice of calcium indicator, displays a linear relationship to neuronal spike rates within a range of 10-150 Hz (Li, Liu, Jiang, Lee, & Tang, 2017), weak and strong signals out of this range are more nonlinear, and may appear poorer and stronger, respectively, than electrode-recorded effects. Consequently, the differences in population responses between monocular and binocular stimulations revealed by this study might be less pronounced.”

      (Recommendations For The Authors):

      Overall, my main suggestion for the authors to improve the paper is to revise some of the interpretations of their results in relation to previous research. The purpose of the present study was to illustrate a more complete picture of the binocular combination of monocular responses by taking into consideration the ocular dominance of V1 cells (lines 34-36). A study published earlier this year had an identical purpose (Mitchell et al., Current Biology, 2023) and arrived at a highly similar conclusion (and also applied divisive normalization to fit their data). I would ask that this paper be mentioned in the introduction and discussed.

      The Mitchell et al 2023 paper is added to the Introduction and Discussion:

      (Ln 50) “In addition (to the Dougherty et al 2019 paper from the same group), Mitchell, Carlson, Westerberg, Cox, and Maier (2023) reported that binocular combination of monocular stimuli with different contrasts is also affected by neurons’ eye preference.”

      (Ln 286) “The critical roles of ocular dominance have been largely overlooked by extant binocular vision models to our knowledge, except that Anderson and Movshon (1989) demonstrated that a model consisting of multiple ocular dominance channels can better explain their psychophysical adaptation data, and that Mitchell et al. (2023) revealed that binocular combination of different contrasts presented to different eyes are affected by neurons’ ocularity preference.”

      Nevertheless, the results of the present study are very valuable. They add substantial spatial resolution and sophisticated relational analysis of monocular and binocular responses that Mitchell et al., 2023 did not include. Therefore, my suggestion is to emphasize the advantages of two-photon imaging in the introduction, focusing on the ability to image neurons in neighboring ocular dominance columns. The rigorous modeling of the relationship between nearby neurons with a range of eye preferences, in tandem with the incredible yield of two-photon imaging, is what sets this paper apart from previous electrophysiological work.

      The finding that binocular responses were dependent on ocular dominance is largely consistent with previous electrophysiological results. However, there should be a paragraph in the discussion section that speaks to the limitations of comparing two-photon imaging data to electrophysiological data. Namely, there are two limitations:

      (1) These two techniques confer different temporal resolutions. It is conceivable that some of the electrophysiology relationships (for example, described by Dougherty et al., 2019) may be dependent on the temporal window over which the data was averaged, typically over 50-100ms around stimulus onset, or 100-250ms comprising the neurons' sustained response to the stimulus. This possible explanation of the difference in obtained results would be especially useful for the discussion paragraph starting at line 232. It would also be helpful to readers for there to be some mention of the advantage of having high temporal resolution (i.e., the benefits of electrophysiology) since (a) recent work has distinguished between sequential stages of binocular combination (Cox et al., 2019) and (b) modern models of V1 neurons emphasize recurrent feedback to explain V1 temporal dynamics (see Heeger et al., 2019; Rubin et al., 2015), which could prove to be relevant for combination of stimuli in the two eyes (Fleet et al., 1997).

      Our discussion regarding the technical limitations of 2-p calcium imaging has been listed earlier. Specific to the Dougherty et 2019 paper, we added the following discussion to address the issue of temporal resolution difference between two technologies.

      (Ln 266) “In addition, it is unclear whether the discrepancies are caused by different temporal resolutions of electrode recording and calcium imaging. The results of Dougherty et al. (2019) represent changes of neuronal spike activities over a period of approximately 50-200 ms after the stimulus onset, which may reflect the sustained neuronal responses to the stimulus and possible feedback signals. Calcium signals are much slower and indicative of the aggregated neuronal responses over a longer period (up to 1000 ms in the current study). They should have smeared, rather than exaggerated, the differences between monocular and binocular responses, although we cannot exclude the possibility that some neuronal response changes beyond 200 ms are responsible for the discrepancies.”

      (2) The sample of V1 neurons in this study is limited to cells in the most superficial layers of the cortex (layers 2/3). This limitation is, of course, well understood, but it should be mentioned at least in the context of studying the formative mechanisms of binocular combination in V1 (since we know that binocular neurons also exist in layers 5/6, and there is now substantial evidence that even layer 4 neurons are not as "monocular" as we previously thought (Dougherty et al., 2019)).

      See our discussion regarding the technical limitations of 2-p calcium imaging listed earlier.

      In short, I believe the paper would be improved by (1) adding the above citations in the appropriate places, (2) acknowledging in the introduction that this question has been investigated electrophysiologically but emphasizing the advantages of two-photon imaging, and (3) adding a paragraph to the discussion section that discusses the temporal and spatial limitations when using two-photon imaging to study binocular combination, particularly when comparing the results to electrophysiology.

      Reviewer #2 (Public Review):

      Summary:

      This study examines the pattern of responses produced by the combination of left-eye and right-eye signals in V1. For this, they used calcium imaging of neurons in V1 of awake, fixating monkeys. They take advantage of calcium imaging, which yields large populations of neurons in each field of view. With their data set, they observe how response magnitude relates to ocular dominance across the entire population. They analyze carefully how the relationship changed as the visual stimulus switched from contra-eye only, ipsi-eye only, and binocular. As expected, the contra-eye-dominated neurons responded strongly with a contra-eye-only stimulus. The ipsi-eye-dominated neurons responded strongly with an ipsi-eye-only stimulus. The surprise was responses to a binocular stimulus. The responses were similarly weak across the entire population, regardless of each neuron's ocular dominance. They conclude that this pattern of responses could be explained by interocular divisive normalization, followed by binocular summation.

      Strengths:

      A major strength of this work is that the model-fitting was done on a large population of simultaneously recorded neurons. This approach is an advancement over previous work, which did model-fitting on individual neurons. The fitted model in the manuscript represents the pattern observed across the large population in V1, and washes out any particular property of individual neurons. Given the large neuronal population from which the conclusion was drawn, the authors provide solid evidence supporting their conclusion. They also observed consistency across 5 fields of view.

      The experiments were designed and executed appropriately to test their hypothesis. Their data support their conclusion.

      Weaknesses:

      One weakness of their study is that calcium signals can exaggerate the nonlinear properties of neurons. Calcium imaging renders poor responses poorer and strong responses stronger, compared to single-unit recording. In particular, the dramatic change in the population response between monocular stimulation and binocular stimulation could actually be less pronounced when measured with single-unit recording methods. This means their choice of recording method could have accidentally exaggerated the evidence of their finding.

      We discussed the nonlinearity of calcium signals as part of the technical limitations of 2-p imaging calcium. The calcium indicator we use, GCaMP5, has a reasonable range of linear relationship with spike rates. But out of this range, the nonlinearity is indeed a concern.

      (Ln 314) “In addition, calcium signals may exaggerate the nonlinear properties of neurons. Although signals indicated by GCaMP5, our favored choice of calcium indicator, displays a linear relationship to neuronal spike rate within a range of 10-150 Hz (Li et al., 2017), weak and strong signals out of this range are more nonlinear, and may appear poorer and stronger, respectively, than electrode-recorded effects. Consequently, the changes in population responses between monocular and binocular stimulations revealed by this study might be less pronounced.”

      The implication of their finding is that strong ocular dominance is the result of release from interocular suppression by a monocular stimulus, rather than the lack of binocular combination as many traditional studies have assumed. This could significantly advance our understanding of the binocular combination circuitry of V1. The entire population of neurons could be part of a binocular combination circuitry present in V1.

      This is a very good insight. We added the following sentences to the end of the first paragraph of Discussion:

      (Ln 242) “These findings implicate that at least for neurons in superficial layers of V1, significant ocular dominance may result from a release of interocular suppression during monocular stimulation, an unusual viewing condition as our vision is typically binocular, rather than a lack of binocular combination of inputs from upstream monocular neurons.”

      (Recommendations For The Authors):

      Line 150: "To model interocular response suppression, responses from each eye in Eq. 2 were further normalized by an interocular suppression factor wib or wcb," I recommend the authors improve their explanation of how they arrived at Eq. 3 from Eq. 2. As it stands, my impression is that they have one model for the responses to monocular stimulation, and another model for the responses to binocular stimulation. What I think is missing is that both equations are derived from the same model. Monocular stimulation is a situation in which the stimulus in one eye's contrast is zero. Could the authors clarify whether this situation produces an interocular suppression of zero, and how that leads to Eq. 2?

      We rewrote the modeling part to show that Equations 1-3 are sequential steps of development for the same model. We also added a brief paragraph to discuss how Eq. 3 could lead to Eq. 2 under monocular viewing:

      (Ln 166) “Although not shown in Eq. 3, we also assumed that the nonlinear exponent b also depends on the contrast of the stimulus presented to the other eye (i.e., Sc or Si). Consequently, when Sc or Si = 0 under monocular stimulation, Rc or Ri = 0 (Eq. 1), and interocular suppression wib or wcb = 1, so Eq. 3 changes back to Eq. 2. It is only when Sc and Si are equal and close to 1, as in the current study, that interocular suppression and binocular combination would be in the current Eq. 3 format.”

      Line 225: "However, individually, compared to monocular responses, responses of monocular neurons more preferring the stimulated eye are actually suppressed, and only responses of binocular neurons are increased by binocular stimulation." This sentence is difficult to follow. I recommend the authors improve clarity by breaking up the sentence into several sentences. If I understand correctly, they summarize the pattern in the data that is indicative of interocular divisive normalization, i.e., their final conclusion.

      This sentence no longer exists in the Discussion.

      Line 426: "Third, for those showing significant orientation difference, the trial-based orientation responses of each neuron were fitted with a Gaussian model with a MATLAB nonlinear least squares function:" The choice of using a Gaussian function to fit orientation tuning was probably suboptimal. A Gaussian function provides an adequate fit only for neurons whose tuning is very sharp. The responses outside of the peak fall down to the baseline and the two ends meet. Otherwise, the two ends do not meet. An adequate fit would be achieved with a function of a circular variable, which wraps around 180 deg. I recommend using a Von Mises function for fitting orientation tuning.

      We agree with the reviewer that the Von Mises function is more accurate than Gaussian for fitting orientation tuning functions. Indeed we are using it to fit orientation tuning of V4 neurons, many of which have two peaks. For the current V1 data, the differences between Von Mises and Gaussian fittings are very small, as shown in the orientation functional maps from three macaques below. Because we also use the same Gaussian fitting of orientation tuning in several published and current under-review papers, we prefer to keep the Gaussian fitting results in the manuscript.

      Author response image 1.

      Reviewer #3 (Public Review):

      The authors have made simultaneous recordings of the responses of large numbers of neurons from the primary visual cortex using optical two-photon imaging of calcium signals from the superficial layers of the cortex. Recordings were made to compare the responses of the cortical neurons under normal binocular viewing of a flat screen with both eyes open and monocular viewing of the same screen with one eye's view blocked by a translucent filter. The screen displayed visual stimuli comprising small contrast patches of Gabor function distributions of luminance, a stimulus that is known to excite cortical neurons.

      This is an important data set, given the large numbers of neurons recorded. The authors present a simple model to explain the binocular combination of neuronal signals from the right and left eyes.

      The limitations of the paper as written are as follows. These points can be addressed with some additional analysis and rewriting of sections of the paper. No new experimental data need to be collected.

      (1) The authors should acknowledge the fact that these recordings arise from neurons in the superficial layers of the cortex. This limitation arises from the usual constraints on optical imaging in the macaque cortex. This means that the sample of neurons forming this data set is not fully representative of the population of binocular neurons within the visual cortex. This limitation is important in comparing the outcome of these experiments with the results from other studies of binocular combination, which have used single-electrode recording. Electrode recording will result in a sample of neurons that is drawn from many layers of the cortex, rather than just the superficial layers.

      See our discussion regarding the technical limitations of 2-p calcium imaging listed earlier.

      (2) Single-neuron recording of binocular neurons in the primary visual cortex has shown that these neurons often have some spontaneous activity. Assessment of this spontaneous level of firing is important for accurate model fitting [1]. The paper here should discuss the level of spontaneous neuronal firing and its potential significance.

      We have noticed previously that at non-optimal spatial frequencies, calcium responses to a moving Gabor grating are close to zero (Guan et al., Prog Neurobiology, 2021, Fig. 1B), but we cannot tell whether this is due to calcium response nonlinearity, or a close-to-zero level of spontaneous neuronal activity. Prince et al (2002) reported low spontaneous responses of V1 neurons with moving grating stimuli (e.g., about 3 spikes/sec in one exemplar neuron, their Fig. 1B), so this appears not a big effect. In our data fitting, we do have an orientation-unspecific component in the Gaussian model, which represents the neuronal response at a non-preferred orientation, but not necessarily the spontaneous activity.

      (3) The arrangements for visual stimulation and comparison of binocular and monocular responses mean that the stereoscopic disparity of the binocular stimuli is always at zero or close to zero. The animal's fixation point is in the centre of a single display that is viewed binocularly. The fixation point is, by definition, at zero disparity. The other points on the flat display are also at zero disparity or very close to zero because they lie in the same depth plane. There will be some small deviations from exactly zero because the geometry of the viewing arrangements results in the extremities of the display being at a slightly different distance than the centre. Therefore, the visual stimulation used to test the binocular condition is always at zero disparity, with a slight deviation from zero at the edges of the display, and never changes. [There is a detail that can be ignored. The experimenters tested neurons with visual stimulation at different real distances from the eyes, but this is not relevant here. Provided the animals accurately converged their eyes on the provided binocular fixation point, then the disparity of the visual stimuli will always be at or close to zero, regardless of viewing distance in these circumstances.] However, we already know from earlier work that neurons in the visual cortex exhibit a range of selectivity for binocular disparity. Some neurons have their peak response at non-zero disparities, representing binocular depths nearer than the fixation depth or beyond it. The response of other neurons is maximally suppressed by disparities at the depth of the fixation point (so-called Tuned Inhibitory [TI] neurons). The simple model and analysis presented in the paper for the summation of monocular responses to predict binocular responses will perform adequately for neurons that are tuned to zero disparity, so-called tuned excitatory neurons [TE], but is necessarily compromised when applied to neurons that have other, different tuning profiles. Specifically, when neurons are stimulated binocularly with a non-preferred disparity, the binocular response may be lower than the monocular response[2, 3]. This more realistic view of binocular responses needs to be considered by the authors and integrated into their modelling.

      We agree and include the following texts when discussing the future work:

      (Ln 298) “In addition, in our experiments, binocular stimuli were presented with zero disparity, which best triggered the responses of neurons with zero-disparity tuning. A more realistic model of binocular combination also requires the consideration of neurons with other disparity-tuning profiles.”

      (4) The data in the paper show some features that have been reported before but are not captured by the model. Notably for neurons with extreme values of ocular dominance, the binocular response is typically less than the larger of the two monocular responses. This is apparent in the row of plots in Figure 2D from individual animals and in the pooled data in Figure 2E. Responses of this type are characteristic of tuned inhibitory [TI] neurons[2]. It is not immediately clear why this feature of the data does not appear in the summary and analysis in Figure 3.

      This difference is indeed captured by the model, which can be more easily appreciated in Fig. 4A where monocular and binocular model simulations are plotted in the same panel. In the text, we also wrote: (Ln 195) “It is apparent that binocular responses cannot be explained by the sum of monocular responses, as binocular responses are substantially lower than the summed monocular responses for both monocular and binocular neurons. Nor can binocular responses be explained by the responses to the preferred eye, as binocular responses are also lower than those to the preferred eye (the larger of the two monocular responses) for monocular neurons.”

      The paper text states that the responses were "first normalized by the median of the binocular responses". This will certainly get rid of this characteristic of the data, but this step needs better justification, or an amendment to the main analysis is needed.

      The relevant sentence has been rewritten as “Monocular and binocular data of each FOV/depth, as well as the pooled data, were first normalized by the respective median of the binocular responses of all neurons in the same FOV/depth.” This normalization would render the overall binocular responses to be around unity, for the purpose of facilitating comparisons among all FOV/depth, but it would not affect the overall characteristic of the data.

      In the present form, the model and analysis do not appear to fit the data in Figure 2 as accurately as needed.

      Thanks for pointing out the problem, as data fitting for FOV C_270 and the pooled data were especially inaccurate. The issue has been mostly fixed when each datum was weighted by its standard deviation (please see the updated Fig. 3).

    1. Author response:

      The following is the authors’ response to the original reviews.

      Reviewer #1 (Recommendations For The Authors):

      In its current form, I would exclude the cryo-EM data from the manuscript. It does not add much and it is distracting from the excellent work that you did on the functional characterization of the variant. Alternatively, you could try to improve the resolution and see if you can get some more meaningful analysis out of the structures? I noticed that you only collected very small datasets. If you decide to pursue a higher resolution reconstruction, collecting more movies will give you a better chance to obtain a higher resolution.

      We express our gratitude to the reviewer for their invaluable feedback. While acknowledging that our structure currently maintains a low resolution, it still provides valuable insights into the splice's proximity to the N412 glycan density. This proximity and low-resolution map hindered the complete modeling of all the splice residues. Notably, this structure represents the first depiction of this particular splice variant. Consequently, it lays a foundation for subsequent studies in the field, and hence, we would want to keep it in the manuscript. As per reviewers’ suggestions, we have now included comparisons of our structure with the GluK1-2a receptor structure reported recently (Mayerson et al. 2022). We do plan to carry out higher-resolution structures in the future.

      I would probably also exclude the RNAseq analysis. I think that Figure 1 is fine, but the supplement 1 is not very successful in convincing me that the exon 9 is expressed mainly in early stages of brain development. In addition, the plot in Figure 1 indicates strong expression in the cerebellar cortex in 20s and 30s. If you decide to keep the data, I strongly encourage you to include more details on the analysis in the methods section.

      Thanks for this insightful comment. We have now modified this section extensively for better clarity. Indeed, the expression of this variant seems to be dynamic in different brain regions. This has now been specified in the revised manuscript. Figure 1 shows the expression of GRIK1 exon 9 gene in different regions of the human brain and donor age. The supplementary figure 1 is a zoom-in on one such region, the Cerebral cortex, where we observe the maximum expression of GRIK1. In this region, we also observed higher expression of exon 9 in the early stages of development. The scales of Figure 1 (0-4 RPKM) and supplemental Figure 1(06RPKM) are different due to more expression of other exons in supplemental Figure 1 (example, we observe 4RPKM expression in the shade of red, for figure 1, whereas similar values of 4RPKM are orange-yellow in the supplemental figure1). Using Supplemental Figure 1, we wanted to show the expression of exon 9 with respect to other exons during developmental stages that prove that GluK1-1 is highly expressed in the initial stages of life. more details on the analysis in the methods section has been added now.

      Additionally, there are a few minor issues in the data presentation:

      (1) in Fig. 2C there seems to be a mismatch between the green dose response plot and the GluK12a trace shown. The plot reports an EC50 of 187.7 uM, whereas in the sample trace 0.25 mM agonist activates only to ~20%.

      We have verified the data and statistics, confirming their consistency with the values reported in the manuscript. For Figure 2C, we present representative traces from a single cell. However, the EC50 value was calculated using Hill's equation based on averaged data from 5 cells.

      (2) The axis label is misprinted in Figure 3C

      Thanks. Corrected.

      (3) In Fig 5 supplement 1, panel B - the 3 last labels above the western blot lanes are off so it is difficult to see which sample corresponds to which lane.

      Thanks. We have corrected the figure.

      Reviewer #2 (Recommendations For The Authors):

      Overall I congratulate the authors of this study nicely done. It represents a large body of work.

      We thank the reviewer for his/her time and positive comments.

      I have several minor corrections that authors could consider for the revision of the manuscript P7. The desensitization rate of GluK1-2a was "delayed"... replace by "increased".

      Corrected.

      P9. Last line 0.37; P.. Add the P value.

      P value has been added as suggested.

      P11 authors indicate that K368/375//379/382H376-E mutant exhibit significant difference in desensitization properties in presence of NEto1, but on the 1st line of p11, they provide a P value above 0.05

      We thank the reviewer for pointing out this discrepancy and have fixed the same. We have discussed two mutants that show slower desensitization when compared to GluK1-1a co-expressed with Neto1. The K to E mutant has significance, while the des value for the K368/375//379/382H376-E mutant shows the same pattern, though not significantly. We have now modified the text to explain this more clearly.

      P19 the calculation of mean weighted tau TDes is not clear and should be better explained.

      Thanks. We have added more details in the Methods sections. We analyzed the current decays in response to 1–2 ms or 1 s applications by employing an exponential function or the sum of two exponential functions. This analysis allowed us to derive a weighted mean τdes using the formula [(τ1 × amplitude1) + (τ2 × amplitude2)]/[amplitude1 + amplitude2]. The tau values represent the time constants obtained from the exponential fits, while the amplitudes correspond to the estimated contributions of each component to the total peak current amplitude.

      [(A1 * t1) + (A2 * t2)] / (A1 + A2)

      It represents the calculation of a weighted mean, where A1 and A2 are the amplitudes, and t1 and t2 are the corresponding time constants. The formula calculates the overall mean time constant by taking into account the contribution of each component to the total amplitude.

      P19 the rate of recovery was obtained by fitting the one-phase association "with" exponential function. With is missing.

      We have corrected this error.  Thanks.

      P21 which method has been used for site directed mutagenesis

      Overlapping PCR was carried out for mutagenesis using the primers listed in Figure 4-table supplement 1. A ligation-free cloning approach (Zhang et al., 2017) was used. It has now been elaborated in the methodology section under Site directed mutagenesis.

      P21 and 22. Provide complete reference of reagent including species of antibodies.

      Thanks. We have added all the details in the methods section now. 

      Anti-His: Rabbit mAb #12698 (Cell Signaling Technology)

      Anti-Neto1: Rabbit #SAB3500679 (Sigma Aldrich)

      Anti-GFP: Mouse mAb G1546 (Sigma Aldrich)

      Anti-actin: Mouse mAb A3853 (Sigma Aldrich)

      P22 How much anti His antibody was used with 40microliter of protein A?

      We have used 2µg/ 40uL of Protein A slurry. This has now been added to the methodology.

      P23 Authors seem to have used a virus to express protein but the protocol is not given. For example what is P2 virus?

      We have now modified the manuscript to include details of baculovirus generation as per the protocol described in Goehring et al. 2014. We followed the same protocol wherein the 2nd generation of virus (P2) generated in insect (SF9) cells was used for infecting suspensionadapted HEK293-T cells for large-scale GluK1-1aEM protein expression.

      Reviewer #3 (Recommendations For The Authors):

      Major concerns:

      (1) The effect of the splice insert on Gluk1 regulation by Neto proteins is not fully clear. For example, experiments in Fig. 3G indicate that the desensitization time for Gluk1-1a + Neto2 is ~32ms. This value is half compared with data obtained from whole-cell experiments shown in Fig. 3A (~70ms). What is the reason for this discrepancy? If variability is observed between experiments, I wonder how valid are the comparisons made in panel A between GluK11a+Neto2 vs GluK1-2a+Neto2 groups. In the case of recovery analysis, authors found significant differences comparing both groups in the presence of Neto (Fig. 3B) but recovery times are not identic for Gluk1-1a vs Gluk1-2a (without Neto). Thus, I wonder if the fold change related to the control group (without Neto) is different. 

      We appreciate your detailed feedback, which has allowed us to clarify and reinforce the validity of our experimental findings. Different recording configurations (e.g., outside-out patch (Fig. 3G) versus whole-cell recordings (Fig. 3A) have been used. Whole-cell recordings average responses over a larger membrane area and also have slower solution exchange times compared to outside-out patch recordings. This may have contributed to the variability in desensitization times. However, similar trends in our whole cell vs. outside-out patch recordings were observed. Further, all the data except those presented in Figs 3G and 3H are from whole-cell recordings. We have performed multiple independent experiments and utilized rigorous statistical analyses to validate our comparisons. We report mean values with standard deviations or confidence intervals to provide a more accurate representation of the data.

      Neto1 significantly speeds up the recovery from desensitization for both variants, with a more pronounced effect on GluK1-1a (GluK1-1a +Neto1: 0.68 s) compared to GluK1-2a (GluK1-2a +Neto1: 1.15 s). The recovery times are not identical for the two variants, likely due to the presence of splice insert in GluK1-1a. Neto2, on the other hand, slows recovery for both variants without significant differential effects. However, the recovery rate from the desensitized state is faster for GluK1-1 compared to GluK1-2a alone, although insignificant (without Neto). 

      In the case of the glutamate concentration-response curve (Fig. 3C), EC50 values for Neto1 and Neto2 are relatively the same, but this approach on its own does not provide insights about the role of the splice insert. Previous experiments with the Gluk1 reveal differences between EC50 in the presence of Neto1 or 2 (Fisher, 2015), suggesting that the insert could regulate glutamate binding affinity, but still, this point is not directly demonstrated in this work.

      Thanks for this insightful comment. Indeed, we cannot conclude that splice residues directly affect glutamate sensitivity and have modified the text accordingly. The Fisher paper demonstrated that both Neto1 and Neto2 can influence glutamate sensitivity in GluK1-2a, with EC50 values of 124.6 ± 16.2 µM. Specifically, in the presence of Neto1 and Neto2, the EC50 values are 4.4 ± 0.4 µM and 13.7 ± 4.2 µM, respectively, indicating a noticeable effect though not substantially different for GluK1-2a coexpressed with either Neto1 and Neto2. Our observation for the GluK1-1a has been similar, with both Neto1 and Neto2 showing a leftward shift.

      (2) Similar to the previous point, a proper interpretation of mutant data is missing in the manuscript. From current data, it is difficult to visualize the role of the insert on Netodependent regulation, mainly, because of the fact that some mutations alone affect Gluk1-1 channel properties. The authors conclude their data by stating that "while the modulation of the receptor by Neto 1 is affected by mutations in splice insert, the modulation by Neto 2 remains largely unaffected" (Page 13). However, this statement is confusing since the co-expression of Gluk1-1a with Neto2 (Fig. 5) prevents the effect caused by mutation K368 alone (Fig. 4), indicating that modulations by Neto 2 are indeed potentially affected by the mutations. Please, clarify. Also, the effect of the K368/375/379/382H376-E mutant on Neto modulation (pink bar in Fig. 5) is impossible to interpret properly since the effect of the mutation alone is not shown in the manuscript.

      Thanks for seeking this important clarification. It is indeed true that splice residue mutations themselves affect the receptor functional properties in comparison to the wild-type receptors. For the sake of clarity, we have presented the effect of splice mutants on receptor properties separately from the effect of mutations on modulation by Neto proteins. Figure 4 demonstrates a comparison between wild-type and mutant receptors without the Neto proteins, showcasing different kinetic properties, while Figure 5 provides detailed information on the role of the insert in Neto-dependent regulation. 

      It’s true we could not record the effect of the K368/375/379/382H376-E mutant alone or when coexpressed with Neto 2 due to low peak amplitudes (mentioned in Table 1) that prevented reliable comparisons. However, robust currents were observed when the same mutant was coexpressed with Neto1, and hence comparisons were shown for this mutant with GluK1-1a wild-type + Neto1. 

      We have now modified the statement "while the modulation of the receptor by Neto 1 is affected by mutations in splice insert, the modulation by Neto 2 remains largely unaffected" and the last paragraph as follows:

      “Neto1 appears to have more pronounced effects on the mutant receptors compared to Neto2. Specifically, Neto1 significantly slowed desensitization for the K368-E mutant, accelerated recovery from desensitization for K368-E and K368/375/379/382H376-E mutants, increased agonist efficacy for K368-E and K375/379/382H376-E mutants, and altered rectification properties for K368E and K368/375/379/382H376-E mutants. In contrast, Neto2 had fewer significant effects on the mutant receptors, with the main impact being an increase in agonist efficacy for the K368-E mutant. Notably, Neto2 did not significantly affect desensitization, recovery from desensitization, or rectification properties of the mutant receptors when compared with wildtype GluK1-1a coexpressed with Neto2. These findings suggest that the splice residues in GluK1-1a differentially influence receptor modulation by Neto1 and Neto2, with Neto1 showing more extensive modulation of the mutant receptors' functional properties.”

      (3) An open question after reading this interesting work is if the proposed change in Neto regulation because of the splice insert is due to changes in Gluk1-Neto interactions or because the rearrangement after interaction with Neto proteins is different. Pull-down experiments (Fig 5 Sup.1) suggest that the splice insert and all the mutants tested do not prevent interaction with Neto proteins. I wonder if the authors could complement their data with a quantitative approach/analysis to demonstrate if the splice insert and the mutants affect Neto1/2 interactions (as expected for the rationale when creating the mutants).

      Thank you for this insightful suggestion. You raise an important point about distinguishing between changes in GluK1-Neto interactions and potential differences in receptor rearrangement after Neto binding. While our pull-down experiments suggest that the splice insert and mutants don't prevent Neto interactions (probably due to a larger interaction interface all along the receptor), a quantitative approach would indeed provide more nuanced information. In future studies, we do plan to perform a quantitative approach like Surface plasmon resonance to assess the changes in interactions upon mutations in the splice and/or Neto proteins in different states of the receptor. In addition, obtaining cryo-EM structures of GluK1 splice variants in complex with Neto1 and Neto2 would provide crucial insights into their interaction interfaces and any conformational changes induced by binding. 

      (4) Related to the Gluk1-1a structure, the authors state that the overall structure is similar to the one without the insert (page 14); however, this is not properly shown in the manuscript. Even if the overall architecture of the channel is the same, authors should make a proper/adequate comparison between both structures/domains to support their claims. Also, one should expect that the insertion of 15 amino acids would affect in some way the closing neighboring domains. The differential effect of the splice insert on glutamate and kainate EC50 values (Fig. 2 and Fig. 2 sup.1), suggests that the insert could introduce a sort of rearrangement in the binding domain. Thus, I wonder if a more elaborated analysis of the current structural data could reveal some structural insights that would explain the specific functional differences due to the splice insert. If the low resolution and the missing residues avoid making some comparisons and establish differences between sidechain orientations, still, a proper comparison between the domain backbones would be helpful to validate the author's statement at least. Also, I wonder if the changes could be resolved better in a closed state or APO structure, instead of the desensitized structure. Finally, are the structures obtained in DDM and nanodiscs similar?

      As per the reviewer’s suggestion, we have now added a new figure in the supplementary information, “Figure 6-figure supplement 9,” where we show a superimposition of GluK11aEM (detergent-solubilized or reconstituted in nanodiscs) and GluK1-2a (PDB:7LVT; silver) showing overall conservation of the structures in the desensitized state.

      As evident from the figure and rmsd values mentioned above, we do not observe significant movements at both ATD and LBD layers of GluK1-1a with respect to GluK1-2a. Also as can be observed the DDM solubilized and nanodisc reconstituted GluK1-1a (Panel A) are very similar with a rmsd of ~2.19Å across all the 2664 Calpha atom pairs. Due to low resolution of our structures, we have refrained from carrying out detailed structural comparisions.

      Our efforts to capture the closed state or apo state structures have failed due to either severe orientation bias (only top views) or increased heterogeneity. 

      (5) Methods section lacks relevant information for proper data interpretation as well as for replicating some experiments in the future. For example:

      A) The experimental design to determine the rectification index with a Ramp protocol is not clear: 1) Why the authors applied a ramp protocol if receptors desensitize along the time? Please clarify the protocol.

      Ramp protocols were used only for the wild-type receptors to compare their voltage-dependent behavior, as this was the first study to compare the two splice variants. All kainate receptors (GluK1-GluK5) desensitize over time. However, their rectification properties have been studied previously (both the absence and presence of Neto proteins) using Ramp protocols as they are faster than step protocols.  

      B) Are polyamines included in the solutions to perform the rectification assays?

      No, polyamines were not added to the intracellular solution, and the effect of the endogenous polyamine block was measured. This has now been specified in the results as well as the methods section.

      C) It is not clear if the experiments to calculate IK/IG ratios were performed in the same preparation (This is, the same cell was stimulated with glutamate and then kainate or vice versa).

      Indeed, the current responses for glutamate vs kainate are performed in the same cell (the same cell was stimulated by glutamate then kainate) so that the responses can be compared. It’s now been specified in the methods section.

      D) The experimental design for calculating recovery is not clear.

      We employed a double pulse protocol to measure receptor recovery. The protocol involved applying two consecutive pulses of agonist stimulation to the receptor. Initially, we applied a brief agonist pulse to activate the receptor, followed by a specific recovery period. After the recovery period, we administered a second agonist pulse to assess the receptor's recovery response. The receptor's recovery was determined by comparing the response amplitude of the second pulse to that of the first pulse, providing valuable insights into the receptor's recovery kinetics. Recovery rates were calculated with single exponential association fits in Prism. We have now modified the text for better clarity.

      E) Please indicate the species used for both functional and Cryo-EM (rat Gluk1 isoform?).

      Thanks for pointing this out. We have now specified in relevant methodology sections that Rattus norvegicus GluK1 and Neto proteins were used in this study.

      F) Please describe the nanodisc reconstitution protocol and how the nanodisc protein was purified, if appropriate.

      The MSP1E3D1 was purified by following the protocol given by the Sligar group in 2014 (doi: 10.1016/S0076-6879(09)64011-8). The nanodisc reconstitution protocol has now been elaborated in the revised manuscript.

      G) Site-directed mutagenesis methodology is incomplete. Please check.

      We have now elaborated this section to include more details.

      Minor concerns:

      (1) Authors state that splice residues are ~30A away from the TM domain. Currently, there is no friendly representation showing the localization of the splice in the structure, besides Fig.6E. The manuscript could benefit itself if authors include a better 3D representation or a scheme to highlight the position of the splice relative to critical domains.

      Thanks for pointing this out. The distance between TRP 381 CA (ATD) and LEU 636 CA (TM3) is 92.10 Å. We have changed the value in the text to ~92 Å.

      Author response image 1.

      (2) Authors mention that mutations in the insert to alanine show normal traffic to the plasma membrane but low current amplitude. Then, I wonder if single-channel conductance, mean open time or open probability is affected by the splice insert. Showing the effects of the insert on single-channel properties would strengthen the manuscript's quality.

      It is a good suggestion. However, as can be observed from our whole cell or outside out patch data, we obtained low peak amplitudes (<50 pA) for many of our receptor-only constructs and also suffered from high SEM for some recordings due to heterogeneity between cells of the same population. The suggestion to study the single channel properties of these receptors is considered for future experiments

      (3) It is unclear how the insert or the mutations specifically affect glutamate- or kainate-induced responses because authors analyze IK/IG ratios only. Maybe authors could consider including an analysis of the role of the insert on specific glutamate- or kainate-induced response to gain insights about ligand selectivity.

      All the values have been included in the excel for raw data. We have included the desensitization kinetics of mutant receptors in the presence of glutamate and compared it to the wild type GluK1-1a. Kainate induced responses were very heterogenous (high SEM for % desensitization) and hence have not been included in the main data.

      (4) Please be consistent with nomenclature along the manuscript to avoid confusion. For example, Are Gluk-1-1 and Gluk-1-1a referring to the same variant?

      GluK1-1 has been used in the abstract and the introduction where we introduce the N-terminal splice variant which either has the 15 residues (termed as GluK1-1) or lacks it (GluK1-2). The C- terminal splice variants for GluK1 are named as “a-d”, with “a” being the smallest Cterminal domain variant. Later in the manuscript, we have used only GluK1-1a terminology to represent the ATD splice variant with shortest C-terminal domain.

      The introduction and spatiotemporal results talk about the GluK1-1 receptors wherein the 

      (5) Legend figure 2: Repeated phrase should be removed. Please check.

      (6) Page 8: "This is similar to the effect observed in GluK1-2 receptors whereby the glutamate EC50 was shown to increase by Neto proteins [Neto1: 34-fold and Neto2: 7.5-fold (Palacios-Filardo et al., 2016) and Neto1/2: 10-30X (Fisher, 2015)]". It seems that values from Fisher's paper are backward. Please correct. 

      (7) Page 9. Second paragraph. Spelling mistake when referring to Fig. 3G.

      Thanks for pointing out the inadvertent errors; we have now corrected all of them.

      (8) Figure 3: The title in Y axis overlaps with the figure. Please check.

      We have corrected the error.

      (9) Page 10: "In addition, K375/379/382H376-E mutant also exhibited a slowdown in the recovery (K375/379/382H376-E: 4.83 {plus minus} 0.31 s P=0.2774) (Figure 4C; Table 1)." Statistical analysis indicates this is not correct. Please tone down this statement. For example: "...mutant also exhibited a trend to a slowdown in the recovery although differences do not reach statistical significance".

      Thanks. We have modified the statement as suggested.

      (10) Page 11: "and a reduction was observed for K375/379/382H376-E receptors (1.17 {plus minus} 0.28 P=0.3733) compared to wild-type (Figure 4D; Table 1)." Same issue as the previous minor comment.

      Thanks. We have modified the statement as suggested.

      (11) Page 11: "We observed that mutants K368-E and K368/375/379/382H376-E, desensitize significantly slower in the presence of Neto1" This statement is not true for K368/375/379/382H376-E mutant. Please correct.

      Thanks. We have modified the statement as suggested and specified the difference.

      (12) Legend Figure 4. Colored asterisks are not clear in the figure. Please check.

      Thanks. The reference to colored asterisks has been removed from the legend as they are not used.

      (13) Representative data shown in Fig 5 sup.2A do not match very well with the final quantification shown in Fig 5A. Please check. Also, the authors state in the result section (page 10) that data shown in Fig. 5A indicate that "GluK1-1a modulation by Neto 1 is influenced by the splice residues". This could be true only for residue K368; however, this is not so obvious since the two mutants containing K368E are inconsistent. Please check and clarify.

      Only representative traces are shown in Fig 5 sup 2 A. However, the quantification shown in Fig 5 A is from multiple cells. We have rechecked all the data and found it to be consistent. We have rewritten this section and modified it for better clarity.

      (14) Figure 6-supplement 2: Please incorporate missing values of MW standards in panel B.

      Thanks. We have modified the figure to include values for MW standards.

      (15) It is not clear the rationale for showing construct C552Y C557V C575S in Fig. 6 sup.3, panel A. This mutant is not mentioned in the manuscript.

      It has been mentioned in the methodology section under “Construct design for expression and purification of rat GluK1-1aEM”. It (C552Y C557V C576S) is one of the constructs used in optimizations that were checked for good protein yields. Based on FSEC protein profiles, we used C552Y, C557V (2X Cys mutant) as GluK1-1aEM, which is mentioned in the same section.

      (16) Fig. 6 sup.4 Not clear what does mean w.r.c. Please specify in the legend.

      With respect to (w. r. t.) has been specified in the manuscript.

      (17) Suggestion to improve data presentation in Fig. 4D and Fig. 3 sup.1B: For easier comparison of IK/IG ratios, representative traces for kainate and glutamate in the same group could be shown using the same Y-scale.

      It has been purposely shown with two different Y-scales due to the differences in peak amplitudes in the presence of glutamate or kainate. 

      (18) Fig. 3 sup.1A: Based on the figure legend, horizontal bars representing the application of glutamate are not consistent with time scale bars. Please, check. In the same figure, panel B, the representative traces shown for GluK-1a-Neto1 are not consistent with IK/IG ratio shown in Fig. 3D.

      Thanks, we have corrected the horizontal bars representing glutamate application. The representative traces shown for GluK-1a-Neto1 were rechecked and are consistent with the IK/IG ratio shown in Fig. 3D.

      (19) I wonder if the authors could discuss the lack of Neto1 effect on the wild type Gluk1-2a channel, as proposed previously.

      Sheng et al., 2015 showed that Neto1 enhances the desensitization onset of GluK1. However, it is unclear which GluK1 splice variants were used in that study. GluK1 has several splice variants, but in the present study, we specifically compared GluK1-1a and 2a. In our case, we did not observe the effect of Neto1 on wild-type GluK1-2a in either of the two techniques (whole cell and outside-out patch) we utilized for our study. However, as can be observed from our data, the GluK1-2a receptor alone shows a faster desensitization kinetics than the previous study (Copits et al., 2011). The differences could stem from different experimental conditions such as constructs, recording conditions used etc.

      Copits BA, Robbins JS, Frausto S, Swanson GT. Synaptic targeting and functional modulation of GluK1 kainate receptors by the auxiliary neuropilin and tolloid-like (NETO) proteins. Journal of Neuroscience. 2011 May 18;31(20):7334-40.

      Sheng N, Shi YS, Lomash RM, Roche KW, Nicoll RA. Neto auxiliary proteins control both the trafficking and biophysical properties of the kainate receptor GluK1. Elife. 2015 Dec 31;4:e11682. doi: 10.7554/eLife.11682. PMID: 26720915; PMCID: PMC4749551.

    1. Author response:

      The following is the authors’ response to the original reviews

      Public Reviews:

      Reviewer #1 (Public review):

      Strengths:

      The authors embarked on an ambitious journey to seek the answer regarding 3D genome changes predisposing to metastatic organotropism. The authors succeeded in the assembly of a comprehensive panel of breast cancer cell lines and the aggregation of the 3D genome structure data to conduct a hypothesis-driven computation analysis. The authors also achieved in including proper controls representing normal non-cancerous epithelium and the end organ of interest. The authors did well in the citation of relevant references in 3D genome organization and EMT.

      Weaknesses:

      (1) The authors should clearly indicate how they determine the patterns of spread of the breast cancer cell lines being utilized in this manuscript. How did the authors arrive at the conclusion that certain cell lines would be determined as "localized spread" and "metastatic tropism to the lung"? This definition is crucial, and I will explain why.

      It is indeed a critical point to clearly define and explain what qualifies as metastatic potential to particular organs in our system. Here, we intentionally limited our scope to metastasis that had occurred within the human system. Our cell lines are chosen based on their sites of origin and etiological history in the patients from which they were derived. For example, the cancer cell line BT474 was classified as “localized” because these cells were derived from a solid tumor in the breast itself. Meanwhile, MCF7 and T47D cell lines are considered lung metastatic because these cells were collected from the pleural effusion from the lung. We therefore model human organotropism from the breast to the lung by using cells that originated from infiltrative ductal carcinoma (human breast) but were collected from pleural effusions (human lung). We then use as a comparison a human lung cancer-derived cell line that was itself purified from a pleural effusion. In this way, we can compare the genome structure of a lung cancer cell in the lung environment to a breast cancer cell that has metastasized to the lung environment.

      In our revised version, we further clarify this definition in the text as well as in additional annotations in our supplemental table of all cell line information.

      Todd Golub's team from the Broad Institute of MIT and Harvard published "A metastasis map of human cancer cell lines" to exhaustively create a first-generation metastasis map (MetMap) that reveals organspecific patterns of metastasis. (By the way, this work was not cited in the reference in this manuscript.) The MetMap Explorer (https://depmap.org/metmap/vis-app/index.html) is a public resource that could be openly accessed to visualize the metastatic potential of each cell line as determined by the in vivo barcoding approach as described in the MetMap paper in the format of petal plots. 5 organs were tested in the MetMap paper, including brain, lung, liver, kidney, and bone. The authors would discover that some of the organ-specific metastasis patterns defined in the MetMap Explorer would be different from the authors' classification. For example, the authors defined MCF7 as a line as lung metastatic, and rightly so the MetMap charted a signal towards lung with low penetrance and low metastatic potential. The authors defined ZR751 as a line with localized spread, however, the MetMap charted a signal towards the kidney with low penetrance and low metastatic potential, the signal strength similar to the lung metastasis in MCF7. A similar argument could be made for T47D. The TNBC line MDA-MB-231 is indeed highly metastatic, however, in MetMap data, its metastasis is not only specific to the lung but towards all 5 organs with high penetrance and metastatic potential. The 2 lung cancer cell lines mentioned in this study, A549 and H460, the authors defined them as localized spread to the lung. However, the MetMap data clearly indicated that A549 and H460 are highly metastatic to all 5 organs with high penetrance and high metastatic potential.

      We acknowledge the valuable contributions of animal models in metastatic cancer studies, but we also want to avoid the potentially confounding variable of the animal microenvironment. The MetMap Explorer contains valuable information (and as part of our clarification on this point, we now cite the MetMap in the text), but the “metastatic potential of each cell line” for this tool is measured in a mouse environment. Knowing that a particular cell line, which originated from a human lung metastasis, can further metastasize to other organs in a mouse does not necessarily mean that those cells could do so in humans. The microenvironment responses to metastatic colonization recapitulate the events in wound repair, and these can differ among species (https://pubmed.ncbi.nlm.nih.gov/28916657/ https://pubmed.ncbi.nlm.nih.gov/39729995/ ). Further, the changes a cell needs to make to adapt to a new organ system in a mouse could be confounded by the changes needed to adapt to mouse conditions in general. Finally, migration from a site of ectopic injection may not mimic migration from an initial tumor site. These factors lead to well known cases where MetMap does not reflect the metastatic potential of cancers in humans. As a classic example, prostate cancer frequently metastasizes to bone in humans, and the PC3 cell line was derived from a bone metastatic prostate cancer. However, MetMap shows no evidence of PC3 being able to metastasize to bone in a mouse.

      We agree that the very best data would come from matched primary and metastatic tumors in the same human patient, but those data do not currently exist and generating them would require future work beyond the scope of this study.

      Since results will vary among different experimental models testing metastatic organotropism, (intracardiac injection was the metastasis model being adopted in the MetMap), the authors should state more clearly which experimental model system served as the basis for their definition of organ-specific metastasis. In my opinion, this is the most crucial first step for this entire study to be sound and solid.

      Taking all the above into account, in our revision, we have now included further clarification in the main text to more clearly explain how and why we chose the cell lines we did and what the advantages and limitations of this choice are.

      (2) Figure 1b: The authors found that "MDA-MB-231 cells were grouped with the lung carcinoma cells. This implies that the genome organization of this cell line is closer to that of lung cells than to other breast epithelial cell lines.". In fact, another TNBC line BT549 was also clustered under the same clade. So this clade consisted of normal-like and highly metastatic lines. Therefore, the authors should be mindful of the fact that the compartment features might not directly link to metastasis (or even metastatic organotropism).

      In figure 1b, the grouping that includes MDA-MB-231 (lung metastatic breast cancer) connected to A549, and H460 (lung cancer) occurs at a distance of about 0.2. If the clustering tree were cut at a distance of 0.26, 6 separate clusters would result: two clusters of Luminal subtypes (all labeled red), one that includes all healthy epithelial cells (both lung and breast, all labeled green), one that links two localized breast cancers, one that links MDA-MB-231 to lung carcinoma cell lines, and then BT549 by itself. So, while BT549 appears next to MDA-MB-231 along the horizontal axis, this is just coincidence of the representation: the dendrogram shows it is quite distant from all the other cell lines in this cluster according to compartment profile.

      So, it is only MDA-MB-231 that is very closely linked with the lung cancer cell types.

      It is true that the healthy lung cells (HTBE) are clustered separately and are more similar to normal/non tumorigenic breast epithelial cells (HMEC and MCF10A) than to any cancer cell type. This could suggest that there are aspects of the compartment pattern that represent any healthy epithelium as compared to cancer. What we find in the compartment profile, in both the clustering and the PCA analysis, is that compartment signatures contain information about cell properties on several overlapping levels: there is an aspect of the compartment profile that distinguishes healthy from cancerous cells, an aspect that distinguishes luminal cancers from other subtypes, a part that associates with organotropism, and an aspect that captures EMT status. The final compartment status is a composite of these numerous factors.

      We have clarified the text to indicate that we mean MDA-MB-231 clusters near lung cancer, not necessarily healthy lung cell models.

      (3) Figure 3: In the text, the authors stated, "To further investigate this result, we examined the transcription status of genes that changed compartment across the EMT spectrum and, conversely, the compartment status of genes that changed transcription (Fig. 3b, c, and d)". However, it was not apparent in the figure that the cell lines were arranged according to an EMT spectrum.

      To display these comparisons more clearly, we have now revised figure 3b, c, and d in two ways: First, we have defined the gene and cell line clustering by one set of data (for example, compartment identity in 3b) and then displayed the other data (gene expression) with all genes and cell lines in the same order. Therefore, for each column, genes and cell lines can be compared visually between top and bottom rows. Second, we have colored cell line names from purple to yellow according to their EMT scores as shown in Supplementary Figure 1a. This allows a visual indication of how the clustering separates cell lines by EMT status.

      Also, the clustering heatmaps did not provide sufficient information regarding the genes with concordant/divergent compartments vs transcription changes. It would be more informative if the authors could spend more effort in annotating these genes/pathways.

      We want to clarify that the genes plotted in the heatmaps in Figure 3 are also the genes whose functional enrichment we present in figures 1 and 2. So, the genes that segregate strongly based on A/B compartment (but not gene expression) in figure 3b are the same genes whose GO terms are annotated in Figure 1d. Likewise, the genes that segregate strongly based on gene expression, but not A/B compartment, in figure 3c and d are the same genes whose GO terms are annotated in Figure 2b. We have now made this connection clearer in the text.

      But, we also agree with the reviewer that it is important to explore a bit further the relationship between these divergent sets of genes. Our explorations have led to several observations:

      (1) In some cases, the compartment-segregated genes and the transcription-segregated genes are different members of the same pathways. In Author response image 1 below, for example, we show interactions (according to STRING) for genes from figure 3c that are highly expressed in the epithelial-like cell lines and are annotated as involved in epithelial development (green). We then added to the network genes from figure 3b that are specifically in the A compartment in the epithelial-like cell lines but not mesenchymal cell lines that are also annotated as involved in epithelial development (red). Most of these epithelial development genes that change expression are in the A compartment in all cell lines and therefore do not rely on spatial compartment changes for their regulation. But some additional epithelial development genes, which are interconnected in this same network, are changing compartments across the EMT spectrum. One example, FOXA1, is a key hub in the network and is known to be a pioneer transcription factor involved in development and differentiation. Controlling this gene at the level of spatial genome organization rather than local transcriptional control could be important in the stable cell fate changes that can happen with EMT.

      Author response image 1.

      (2) Overall, the set of genes that change compartments does not have as strong functional enrichment as the transcription change set of genes. This could indicate that some of the compartment changes that occur with EMT are not directly gene regulatory but rather enable an overall conformational change of the chromatin that is needed for the alterations in physical cell state or to accomplish long distance gene regulation changes.

      (3) Related to long distance gene regulation changes, we also see cases in which the gene that changes transcription but not compartment across EMT is adjacent to regions that switch compartments.

      A good example is TFF3 (yellow, Supplementary figure 1C). TFF3 is one of the genes that strongly segregates across EMT by transcription, being more highly expressed in epithelial-like (bottom 4 tracks) but not mesenchymal-like (top 4 tracks) cancers. Despite this differential expression, it is almost always in the A compartment across all cell lines. However, it is adjacent to regions that show strong compartment change EMT signatures. So, even though this specific gene region is not changing compartment, its regulation may be influenced by the entire region being Aassociated in epithelial-like but neighboring regions becoming B-associated in mesenchymal like cancers.

      TFF3 is expressed in normal breast epithelium and has been implicated as a biomarker for endocrine therapy response in breast cancer.

      Meanwhile, many genes that are in these compartment switching regions (BACE2, DSCAM, PDE9A) are not among the strongest expression signature genes.

      (4) Interestingly, some of the regions (such as the region shown in Supplementary figure 1C) that change compartment across the breast cancer spectrum overlap with regions that we found change compartment in the progression of prostate cancer, as shown in the string.db enrichment analysis below.

      Author response image 2.

      In our revised manuscript, we now include more of these explanations in the text and include the example offset compartment and transcription change region shown about as panel c of Supplementary Figure 1.

      (4) Figure 4: The title of the subheading of this section was 'Lung metastatic breast cancer cell lines acquire lung-like genome architecture". Echoing my comments in point 1, I am a bit hesitant to term it as "lung metastatic" but rather "metastatic' in general since cell lines such as MDA-MD-231 do metastasize to other organs as well. However, I do get the point that the definition of "lung metastasis" is derived from the common metastasis features among the cell lines here (MCF7, T47D, SKBR3, MDAMB-231). There might be another argument about whether the "lung" carcinoma cell lines can be considered "localized" since they are also capable of metastasizing to other organs.

      Rather than classifying cells on metastatic “potential” (as measured in a mouse), our cell lines are chosen based on their sites of origin and etiological history in the patients from which they were derived. Cancer cell lines called “lung metastasis” were collected from the pleural effusion from the human lung. Likewise, we call a cancer “localized” because it was taken from the tissue where the cancer originated, even if it might, if placed into a different context, be able to metastasize. We would argue that the genome structure features of the “localized” cancers reflect cancers that have not yet metastasized (even if they could in the future) while the “metastatic” cancers have already gone to a certain location (even if they could in theory have gone to a different location).

      In a way, what the authors probably were trying to leverage here is the "tissue" identity of that organ.

      Having said this, in addition to showing the "lung permissive changes", the authors should show the "breast identity conservation" as well. Because this section started to deal with the concept of "tissue/lineage identify", the authors should also clarify whether these breast cancer cell lines capable of making lung metastasis are also preserving their original tissue identity from the compartment features (which would most likely be the case).

      This is a great question. We have now more explicitly checked the proportions of genomic regions that change compartments to match lung vs. maintaining breast-specific compartment identity. The graphs in Supplementary Figure 2 begin with all genomic bins that have distinctive compartment identity between non-cancerous breast and lung epithelial cells. Then, the plots show what fraction of these tissue-specific bins change compartment to match lung vs. maintaining breast identity in each breast cancer cell line category. As we have shown in other graphs, particularly for switches to the A compartment, more bins change to match lung in the metastatic vs. primary site cell lines. In most cases, more than 50% of the tissue-specific bins shift to look more like lung.

      (5) Rest of the sections: The authors started to claim that the organ-specific metastasis permissive compartmental features mimic the destinated end organ. The authors utilized additional non-breast cancer cell lines (prostate cancer cell lines LNCaP as localized and DU145 as brain metastatic) in brain metastasis to strengthen this claim. (DU145 in MetMap again is highly metastatic to lung, brain, and kidney). However, this makes one wonder that for cell lines that are capable of metastasizing to multiple organ sites (eg. MDA-MB-231, DU145, A459, H460), does it mean that they all acquire the permissive features for all these organs? This scenario is clinically relevant in Stage 4 patients who often present with not only one metastatic lesion in one single organ but multiple metastatic lesions in more than one organ (eg. concomitant liver and lung metastasis). Do the authors think that there might be different clones having different tropism-permissive 3D genome features or there might be evolutionary trajectory in this?

      In my opinion, to further prove this point, the authors might need to consider doing in vivo experiments to collect paired primary and organ-specific metastatic samples to look at the 3D genome changes.

      We agree that an ideal experimental follow up to this study would be to collect paired metastatic and primary tumors, either in mouse xenograft or, even better, from patients. This is beyond the scope of what we can do for our current paper, but we have added a statement to the discussion of further experiments that would be required to clarify this point.

      (6) Technically, the study utilized public Hi-C data without generating new Hi-C data. The resolution of the Hi-C data for compartments was set at 250KB as the binning size indicating that the Hi-C data was at lower resolution so it might not be ideal to address other 3D genome architecture changes such as TADs or long-range loops. It is therefore unknown whether there might be permissive TAD/loop changes associated with organotropism and this is the limitation of this study.

      Our decision to focus on A/B compartmentalization rather than TAD or loop structure in this analysis was intentional and biologically motivated, rather than solely being a reflection of data resolution. Both compartments and topologically associated domains (TADs) are key parts of genome organization and disruption of these structures has the potential to alter downstream gene regulation, as shown by numerous studies. However, compartments have been found, more so than TADs, to be strongly associated with cell type and cell fate. Therefore, in this manuscript, we decided to focus only on the compartment organization changes between different healthy and cancerous cells as they are more likely to represent the stable alterations of the genome organization malignant transformations.

      (7) In the final sentence of the discussion the authors stated "Overall, our results suggest that genome spatial compartment changes can help encode a cell state that favors metastasis (EMT)". The "metastasis (EMT)" was in fact not clearly linked inside the manuscript. The authors did not provide a strong link between metastasis and EMT in their result description. It is also unclear whether the EMTassociated compartment identity would also correlate with the organotropic compartment identity.

      We agree that this statement involves too strong of an assumption. The literature on this topic is vast and complex, and while there is abundant evidence that pathways of EMT can play important roles in facilitating metastasis, there are other pathways at play in the metastatic process as well (https://journals.plos.org/Plosbiology/article?id=10.1371/journal.pbio.3002487). We have made a clearer statement about this in the text now.

      To address the question of whether the organotropic changes related to the EMT changes, we calculated the overlap between the genomic bins that strongly segregated cell lines in the compartment principal component analysis (PC1) with those that showed “organotropic” changes. As you can see in supplementary table 3, this overlap is actually very small, where only 3% of bins are important both for the EMT segregation of cell lines and organotropism.

      We have now included this overlap information as supplementary table 3 and have addressed this in the text.

      Reviewer #2 (Public review):

      Summary:

      This work addresses an important question of chromosome architecture changes associated with organotopic metastatic traits, showing important trends in genome reorganization. The most important observation is that 3D genome changes consistent with adaptations for new microenvironments, including lung metastatic breast cells exhibiting signatures of the genome architecture typical to a lung cell-like conformation and brain metastatic prostate cancer cells showing compartment shifts toward a brain-like state.

      Strengths:

      This work presents interesting original results, which will be important for future studies and biomedical implications of epigenetic regulation in norm and pathology.

      Weaknesses:

      The authors used publicly available data for 15 cell types. They should show how many different sources the data were obtained from and demonstrate that obtained results are consistent if the data from different sources were used.

      In our revised version, we have provided a clarified table of information about all the publicly available data used from all the cell lines, indicating the sources of the data. The 17 datasets used come from 8 different studies. So, indeed, the reviewer is correct that many different sources of data were used. To address the question of whether our results would be consistent if data from different sources were used, we created a comparison map of the A/B compartment profiles for data from multiple sources when it was available. You can see below that the Hi-C data from different sources for the same cell lines cluster quite closely and show high correlation and are well separated from different cell lines. So, we do not think that source batch effects play a major role in our results.

      Author response image 3.

      Recommendations for the authors:  

      Reviewer #1 (Recommendations for the authors):

      (1) Figure 1a: This figure could be re-formatted without the arrows. Arrows usually indicate upstreamto-downstream relationships along certain processes. Using arrows here would mislead people to think that the cell lines were derived from one another. The same could apply to the supplementary figures.

      We have now edited figure 1a to include lines linking cell lines, indicating conceptual relationships, rather than arrows, which would imply direct derivation.

      (2) Figure 1c: The PCA (PC2 axis) indeed seemed to separate the HER2 status quite well. One concern is MCF7, it is labeled as ERpos/HER2neg in MetMap but seems to be clustered as HER2pos in this study. Are they the same? (This again highlights the importance of cell line definition and annotation).

      It is a good point that MCF7, while generally considered HER2 negative (we indicate this negative status in Supplementary Table 1), falls near HER2 positive cells in PCA space. This indicates that PCA captures tendencies but is not a perfect classifier. In a high dimensional, complex system, it is expected that an unsupervised analysis such as this will not capture just one biological feature in a given principal component, and therefore something like HER2 status may not segregate perfectly. However, this analysis does suggest that MCF7 3D genome structure has features that are more similar to other HER2+ cell lines. This raises the interesting possibility that it may actually behave like HER2+ cells in some ways even while being HER2- itself. We have more clearly stated the MCF7 discrepancy in the text.

      Reviewer #2 (Recommendations for the authors):

      (1) The description of results can be shortened, to make it easier to read and understand.

      In our revision, we have tried to clarify where possible, but it was difficult to shorten without losing important caveats and context (especially to make important points emphasized by reviewer 1).

      (2) "100 most positive and negative eigenvalues for PC1" - please provide the correct description.

      We have altered this to make it clearer and more correct: “using the genes from the regions with the top 100 most positive and 100 most negative eigenvector loadings for this PC1”

    1. Author Response

      The following is the authors’ response to the original reviews.

      eLife assessment

      This manuscript provides novel and important findings regarding the impact of noradrenergic signaling from the locus coeruleus on hippocampal gene expression. The locus coeruleus is the sole source of noradrenaline to the hippocampus and many rapid molecular changes induced by stress are regulated by noradrenaline. This manuscript provides a rigorous investigation into hippocampal genes uniquely regulated by noradrenaline in the presence or absence of stress. Data were collected and analyses were performed using solid methodology, and the results mostly convincingly support the conclusion made with few weaknesses. The study would benefit from a more comprehensive analyses of sex differences.

      Response: We thank the reviewers and the editors for the positive evaluation of our work and for the constructive feedback. To address some of the key criticisms, we have performed several new experiments and analyses. Importantly, we now provide a much more rigorous comparison of males and females, which strongly suggests that there are no major sex differences in the transcriptomic response to stress and noradrenaline in the hippocampus. We think that these - and other additions discussed below - significantly strengthen the manuscript. We provide detailed responses to all the reviewers comments. We have added numbers to the reviewers’ comments for easier referencing.

      Reviewer #1 (Public Review):

      Comment 1: Privitera et al., provide a comprehensive and rigorous assessment of how noradrenaline (NA) inputs from the locus coeruleus (LC) to the hippocampus regulate stress-induced acute changes in gene expression. They utilize RNA-sequencing with selective activation/inhibition of LC-NA activity using pharmacological, chemogenetic and optogenetic manipulations to identify a great number of reproducible sets of genes impacted by LC activation. It is noteworthy that this study compares transcriptomic changes in the hippocampus induced by stress alone, as compared with selective circuit activation/inhibition. This reveals a small set of genes that were found to be highly reproducible. Further, the publicly available data will be highly useful to the scientific community.

      Response: We are very grateful for this positive evaluation.

      Comment 2: A major strength of the study is the inclusion of both males and females. However, with this aspect of the study also lies the biggest weakness. While the experiments tested males and females, they were not powered for identifying sex differences. There are vast amounts of literature documenting the inherent sex differences, both under resting and stress-evoked conditions, in the LC-NA system and this is a major missed opportunity to better understand if there is an impact of these sex-specific differences at the genetic level in a major LC projection region. There are many instances whereby sex effects are apparent, but do not pass multiple testing correction due to low n's. The authors highlight one of them (Ctla2b) in supplemental figure 6. This gene is only upregulated by stress in females. It is appreciated that the manuscript provides an incredible amount of novel data, making the investigation of sex differences ambitious. Data are publicly available for others to conduct follow up work, and therefore it may be useful if a list of those genes that were different based on targeted interrogation of the dataset be provided with a clear statement that multiple testing corrections failed. This will aid further investigations that are powered to evaluate sex effects.

      Response: The assessment of the reviewers and the editorial feedback encouraged us to look more thoroughly into potential sex differences, because we believe it would indeed be a major additional strength if our manuscript could make a firm statement on this important issue. To this end, we have expanded the manuscript in two major ways:

      (1) To expand the analysis of sex effects also to the dorsal hippocampus, and to increase robustness of the data, we have performed RNA-seq in 32 additional samples of male and female mice exposed to stress (or control) and propranolol (or saline) injection. Figure 1fh and Supplementary Figure 1d-f have been updated to reflect this new addition, and the results are presented in a new section on Pages 3-4 (pasted below for ease of reviewing). In summary, the strongly support our initial observation that the effects of stress on gene expression, as well as the effects of propranolol on blocking stress-induced effects, are highly similar in both sexes.

      (2) To further increase the power for detection of sex-effects, we have performed a small meta-analysis. For this, we combined several RNAseq datasets from the current manuscript and published datasets from our previous work (Floriou-Servou et al., 2018; von Ziegler et al., 2022), which also investigated transcriptomic sex-differences in the hippocampus 45 min after cold swim stress exposure in the same setup as used for the current manuscript. This approach increased our sample size to 51 males and 20 females. In summary, this well-powered approach shows no evidence for sex differences in the transcriptional response to stress, even when more lenient analyses were applied. These results are described in a new section on page 4, and summarized in Supplementary Figures 1f+g. This section is pasted below for ease of reviewing.

      "While blocking β-adrenergic receptors was able to block stress-induced gene expression, we did not test whether propranolol might decrease gene expression already at baseline, independent of stress. Additionally, all tests had thus far been conducted in male mice, raising the question about potential sex differences in NA-mediated transcriptomic responses. To address these two issues, we repeated the experiment in both sexes and included a group that received a propranolol injection but was not exposed to stress (Fig. 1f). Combining the data from both experiments, we repeated the analysis for each region, to identify genes whose response to stress was inhibited by propranolol (Figure 1g). As in the previous experiment, we found that many of the stress-induced gene expression changes were blocked by propranolol injection in both dHC (Figure 1g, left panel) and vHC (Figure 1g, right panel). Importantly, propranolol did not change the expression level of these genes in the absence of stress. We then directly compared the genes sensitive to stress and propranolol treatment in both dHC and vHC. To this end, we plotted the union of genes showing a significant stress:propranolol interaction in either region in one heatmap across both dHC and vHC (Supplementary Figure 1d). This showed again that the stress-induced changes were very similar in dHC and vHC, and that propranolol similarly blocked many of them. Finally, we asked whether the response differs between males and females. Despite clear sex differences in gene expression at baseline (data not shown), we found no significant sex differences in response to stress or propranolol between male and female mice (FDR<0.05; Fig. 1g). To more directly visualize this, we compared females and males by plotting the log2-fold changes of the stress:propranolol interaction across all stress-induced genes that were blocked by propranolol. We find very similar regulation patterns in both sexes (Figure 1h). Although none of these sex differences are significant, some genes seem to show quantitative differences, so we plotted the expression patterns of the 5 genes showing the largest difference in interaction term as box-plots, which suggest that these spurious differences are likely due to noisy coefficient estimates (Supplementary Fig. 1e). To address concerns that our analysis of sex differences might not have been sufficiently powered, we performed a meta-analysis of the experiments shown here along with previously published datasets from our lab (Floriou-Servou et al. 2018; von Ziegler et al. 2022). In all these experiments, the vHC of male and female mice was profiled 45 min after exposure to an acute swim stress challenge. This resulted in a sample size of 51 males and 20 females. Despite this high number of independent samples, we could not identify any statistically significant interaction between sex and the stress response. To identify candidates that might not reach significance while discounting differences due to noise in fold-change estimates, we reproduced the same analysis using DESeq2 with Approximate Posterior Estimation for generalized linear model (apeglm) logFC shrinkage (A. Zhu, Ibrahim, and Love 2018). This analysis also did not reveal any sex differences in the stress response (Supplementary Fig. 1f). We then tailored the meta-analysis specifically to the set of stress-responsive genes that were blocked by propranolol, and also for these genes the response to stress was strikingly similar in both sexes (Supplementary Fig. 1g). Altogether, we conclude that there are no major sex differences in the rapid transcriptomic stress response in the hippocampus, and that blocking beta-receptors prevents a large set of stress-induced genes in both females and males."

      To put these findings in context with existing literature, we agree with the reviewer that there are many studies that have reported sex differences in the LC-circuitry as summarized by Bangasser and colleagues (Bangasser et al., 2016, 2019). However, these studies primarily focus on the LC itself, suggesting that female rats have more LC neurons, denser LC-dendrites in the peri-LC region, and that LC neurons are more readily activated by stress in females because of heightened sensitivity to CRF-signaling. A recent study in mice reports, in contrast, that females have fewer TH-positive neurons in the LC, but they also find enhanced excitability of LC neurons in females (Mariscal et al., 2023). Similarly, one study has suggested molecular differences in the makeup of the LC (Mulvey et al., 2018). Our experiments, however, focus on the impact of NA release in a projection region (hippocampus). Further, we use a strong stress induction protocol (swim stress) and various potent modes of direct LC activation, so differences in "LC-excitability" are likely less relevant in this context. We added evidence showing that we trigger powerful NA release in both sexes (Supplementary Figure 2c-h; see response to Reviewer #2, Comment #3 for more details). In addition, we show that the intensity or pattern of LC stimulation does not appear to alter the molecular response (Figure 3a-b), and that various stressors (mild or intense) all trigger the same NA-dependent molecular changes (Figure 4a-b). Therefore, our results suggest that once NA is released (in the hippocampus), the molecular downstream effects on gene expression are very similar - independent of stimulation intensity, sex, or hippocampal subregion (dorsal/ventral). This does not mean that there are no sex differences for activation of LC, but rather that the transcriptional response to NA release in the hippocampus is robust across sexes, and that propranolol seems to block NA-dependent effects similarly in both sexes. This does not rule out quantitative differences between sexes that only emerge with targeted analyses of individual genes, or once fluctuations in ovarian hormones are taken into account. We have updated the section in the discussion to summarize these considerations in light of the new results (see pages 20-21, section: "A uniform molecular response to stress and noradrenaline release in both sexes").

      Comment 3: A major finding of the present study is the involvement of noradrenergic transcriptomic changes occurring in astrocytic genes in the hippocampus. Given the stated importance of this finding within the discussion, it seems that some additional dialogue integrating this with current literature about the role of astrocytes in the hippocampus during stress or fear memory would be important.

      Response: We thank the reviewer for giving us an opportunity to add a more detailed discussion about the role of astrocytes and thyroid hormones in the hippocampus during learning and memory formation. We have added these statements to the discussion:

      “Within the hippocampus, astrocytic pathways are emerging as important players for learning and memory processes (Gibbs, Hutchinson, and Hertz 2008; Bohmbach et al. 2022). In fact, it is well-known that NA enhances memory consolidation (Schwabe et al. 2022; McGaugh and Roozendaal 2002), and recent work suggests that these effects are mediated by astrocytic β-adrenergic receptors (Gao et al. 2016; Iqbal et al. 2023). Our transcriptomic screens revealed Dio2 as the most prominent target influenced by LC activity. Dio2 is selectively expressed in astrocytes and encodes for the intracellular type II iodothyronine deiodinase, which converts thyroxine (T4) to the bioactive thyroid hormone 3,3',5-triiodothyronine (T3) and therefore regulates the local availability of T3 in the brain (Bianco et al. 2019). Enzymatic activity of DIO2 has further been shown to be increased by prolonged noradrenergic transmission through desipramine treatment in LC projection areas (Campos-Barros et al. 1994). This suggests that the LC-NA system and its widespread projections could act as a major regulator of brain-derived T3. Notably, T3-signaling plays a role in hippocampal memory formation (Rivas and Naranjo 2007; Sui et al. 2006), raising the possibility that NA-induced Dio2 activity in astrocytes might mediate some of these effects.”

      Comment 4: The comparison of the candidate genes activated by the LC in the present study (swim) with datasets published by Floriou-Servou et al., 2018 (Novelty, swim, restraint, and footshock) is an interesting and important comparison. Were there other stressors identified in this paper or other publications that do not regulate these candidate genes? Further, can references be added to clarify to the reader, that prior studies have identified that novelty, restraint and footshock all activate LC-NA neurons.

      ponse: Thank you for the positive feedback. We have only tested the stressors reported in Figure 4a-b (novelty, swim, restraint, and footshock). It is known that all these stressors trigger noradrenaline release, in fact we are not aware of stressors that do not trigger NA release. This reproducible finding supports the notion that the identified set of genes is indeed highly NAresponsive. As suggested, we have now included references that show increased NA release in response to all these stressors:

      “Therefore, we assessed their expression in a dataset comparing the effect of various stressors on the hippocampal transcriptome (Floriou-Servou et al., 2018). The stressors included restraint, novelty and footshock stress, which have all previously been shown to increase hippocampal NA release (HajósKorcsok et al., 2003; Lima et al., 2019; Masatoshi Tanaka et al., 1982).”

      Comment 5: Comparisons are made between chemogenetic studies and yohimbine, stating that fewer genes were activated by chemogenetic activation of LC neurons. There is clear justification for why this may occur, but a caveat may need to be mentioned, that evidence of neuronal activation in the LC by each of these methods were conducted at 90 (yohimbine) versus 45 (hM3Dq) minutes, and therefore it cannot be ruled out that differences in LC-NA activity levels might also contribute.

      Response: The reviewer raises an important point about some inconsistencies between the time points chosen in our study, an aspect that was also pointed out by Reviewer #2. We have chosen the 45 and 90 min time points for two different reasons. On the one hand, cFos changes on the protein level are known to peak 90 min after neuronal activation, and we wanted to capture the strongest possible cFos signal in the LC. On the other hand, we wanted to measure gene expression changes triggered by NA release, which already occur 45 min after noradrenergic activation (Roszkowski et al., 2016). Thus, when the experimental design allowed separate experiments (e.g. systemic yohimbine injection), we chose to measure gene expression after 45 min, but to validate cFos activation in the LC separately after 90min. In response to DREADD activation, however, we wanted to confirm within the same animal that LC activation was successful, and thus we collected LC and hippocampus simultaneously (Figure 2c,d). While the cFos increase is already very pronounced at the 45min time point (Figure 2g), the quality of IHC is slightly lower because the tissue cannot be perfused in this experimental design. Therefore, we do not think that the time point for cFos sampling matters in this context. However, we agree with the reviewer that it remains unclear whether yohimbine and DREADDs activate the LC with similar potency. To directly compare NA release would require a set of photometry-based experiments to measure NA release using genetically-encoded NA-sensors. While we have added such experiments for LC activation with DREADDs and optogenetics to show rapid NA release indeed occurs in the hippocampus (see Reviewer #2, Comment 3; Supplementary Figure 2c-h), yohimbine interferes with the NA-sensors as explained in detail in response to Reviewer 2, Comment 3. Thus, it was too challenging for us to directly compare the release dynamics in response to DREADDs and yohimbine, which was also not the main focus of our work. To explicitly address this caveat, we have extended the corresponding section in the discussion:

      "Finally, our observation that systemic administration of the α2-adrenergic receptor antagonist yohimbine very closely recapitulates the transcriptional response to stress stands in contrast to the much more selective transcriptional changes observed after chemogenetic or optogenetic LC-NA activation. This difference could be due to various factors. First, it remains unclear how strong the LC gets activated by yohimbine versus hM3Dq-DREADDs. However, given the potent LC activation observed after DREADD activation, it seems unlikely that yohimbine would lead to a more pronounced LC activation, thus explaining the stronger transcriptional effects. Second, contrary to LC-specific DREADD-activation, systemic yohimbine injection will also antagonize postsynaptic α2-adrenergic receptors throughout the brain (and periphery). More research is needed to determine whether this could have a more widespread impact on the hippocampus (and other brain regions) than isolated LC-NA activation, further enhancing excitability by preventing α2-mediated inhibition of cAMP production. Finally, systemic yohimbine administration and noradrenergic activity have been shown to induce corticosterone release into the blood (Johnston, Baldwin, and File 1988; Leibowitz et al. 1988; Fink 2016). Thus, yohimbine injection could have broader transcriptional consequences, including corticosteroid-mediated effects on gene expression."

      Comment 6: Please add information about how virus or cannula placement was confirmed in these studies. Were missed placements also analyzed separately?

      Response: Pupillometry recordings were performed with all animals involving optogenetic or chemogenetic manipulations of the LC, before subjecting them to stress experiments. These assessments account for both correct optic fiber placement and virus expression (Privitera et al., 2020). If an animal did not show a clear pupil response, it was not included any further in the study. To demonstrate correct cannula placement for drug infusion of isoprotenerol in the dorsal hippocampus, we added a representative image of cannula placement in Supplementary Figure 1h.

      Comment 7: Time of day for tissue collection used in genetic analysis should be reported for all studies conducted or reanalyzed.

      Response: Thank you for pointing out this omission. Tissue collection for RNA-seq analysis was always performed between 11am and 5pm during the dark phase of the reversed light-dark cycle. We have added this information to the corresponding method section (“Tissue collection”).

      Reviewer #1 (Recommendations For The Authors):

      Comment 8: This is a well written, comprehensive and rigorous manuscript that will be of great interest to those in the scientific community.

      Response: Thank you for the positive evaluation of our work and for the constructive feedback.

      Reviewer #2 (Public Review):

      Comment 1: The present manuscript investigates the implication of locus coeruleus-noradrenaline system in the stress-induced transcriptional changes of dorsal and ventral hippocampus, combining pharmacological, chemogenetic, and optogenetic techniques. Authors have revealed that stress-induced release of noradrenaline from locus coeruleus plays a modulatory role in the expression of a large scale of genes in both ventral and dorsal hippocampus through activation of β-adrenoreceptors. Similar transcriptional responses were observed after optogenetic and chemogenetic stimulation of locus coeruleus. Among all the genes analysed, authors identified the most affected ones in response to locus coeruleus-noradrenaline stimulation as being Dio2, Ppp1r3c, Ppp1r3g, Sik1, and Nr4a1. By comparing their transcriptomic data with publicly available datasets, authors revealed that these genes were upregulated upon exposure to different stressors. Additionally, authors found that upregulation of Ppp1r3c, Ppp1r3g, and Dio2 genes following swim stress was sustained from 90 min up to 2-4 hours after stress and that it was predominantly restricted to hippocampal astrocytes, while Sik1 and Nr4a1 genes showed a broader cellular expression and a sharp rise and fall in expression, within 90 min of stress onset.

      Overall, the paper is well written and provides a useful inventory of dorsal and ventral hippocampal gene expression upregulated by activation of LC-NA system, which can be used as starting point for more functional studies related to the effects of stress-induced physiological and pathological changes.

      Response: We thank the reviewer for the careful assessment of our work.

      Comment 2: However, I believe that the study would have benefited of a more comprehensive analyses of sex differences. Experiments in females were conducted only in one experiment and analyses restricted to the ventral hippocampus.

      Response: In response to the comments by the reviewer, as well as Reviewer #1 and the editors, we have sequenced an additional 32 brain samples to expand the comparison of sex effects in females and males across dorsal and ventral hippocampus, and we included a new meta-analysis of 3 experimental datasets (51 male and 20 female) samples, to thoroughly assess sex differences in the transcriptomic response to stress. We refer the reviewer to our detailed response provided above to Reviewer #1, comment #2, and the updated results section on pages 3-4.

      Comment 3: Although, the experiments were overall sound and the results broadly support the conclusion made, I think some methodological choices should be better explained and rationalized. For instance, the study focuses on identifying transcriptional changes in the hippocampus induced by stress-mediated activation of the LC-NA system, however NA release following stress exposure and pharmacological or optogenetic manipulation was mostly measured in the cortex.

      Response: Because the hippocampus was used for RNA-sequencing, we could not assess NA release in the hippocampus (as this would require fiber implants that would interfere with molecular measures, or different tissue processing for HPLC). Nonetheless, we wanted to assess the transcriptional changes in the hippocampus, while simultaneously measuring successful stimulation of the LC-NA system in the same animals. To achieve this, we pursued 3 routes: 1) we used pupillometry to confirm functional LC activation; 2) we measured cFOS in the LC to directly demonstrate LC activation; 3) we assessed NA release using uHPLC (which requires larger tissue samples) and we chose the cortex because both cortex and hippocampus receive NA predominantly from the LC (Samuels & Szabadi, 2008). Importantly, we had previously shown that chemogenetic LC activation leads to a similar NA turnover in both the cortex and hippocampus, as measured by uHPLC (Zerbi et al., 2019). The relevant figure from that paper is inserted below to quickly show the striking similarity between hippocampus and cortex.

      Author response image 1.

      Levels of noradrenaline (NE) turnover (MHPG/NE ratio) in the cortex (CTX) and hippocampus (HC), measured in whole tissue with uHPLC 90min after hM3Dq-DREADD activation of the LC (copied and cropped from Zerbi et al, 2019, Neuron).

      In response to the reviewers comment, we performed additional experiments to directly demonstrate that LC-activation with DREADDs as well as optogenetics causes an increase in hippocampal NA-release. We recorded NA release in the hippocampus (using fiber photometry combined with genetically encoded NA sensors). For DREADD activation, we observed a strong increase in hippocampal noradrenaline that started a few minutes after clozapine administration, and this increase was sustained throughout the duration of the 21 minute recording (see Supplementary Figure2c-e). For optogenetic LC activation, we find a rapid and immediate sharp increase in NA levels in the hippocampus (Supplementary Figure 2f-h). These experiments were performed in females and males and triggered similar responses. An adapted and cropped version of Supplementary Figure 2 is pasted below for ease of reading.

      Please note that we could not perform a similar experiment using yohimbine, because the GRABNE sensors are based on the alpha-2 adrenergic receptor, thus yohimbine administration interferes with the photometry recording. However, we believe that it is clear from this response that strong activation of the LC leads to uniform release of NA in the hippocampus and cortex.

      Author response image 2.

      c, Schematic of fiber photometry recording of hippocampal NA during chemogenetic activation of the LC. After 5 min baseline recording in the homecage animals were injected with clozapine (0.03mg/kg, i.p.) and placed in the OFT for 21min. d, Average ΔF/F traces of GRABNE2m photometry recordings in response to chemogenetic activation of the LC (mean±SEM for hM3DGq+ and hM3DGq- split into females and males, n=3/group/sex). e, Peak ΔF/F response of fiber photometry trace. f, Schematic of fiber photometry recording of hippocampal NA during optogenetic activation of the LC. Animals were lightly anesthetized (1.5% isoflurane) and recorded in a stereotaxic frame. After 1 min baseline recording, animals were stimulated three times with 5Hz for 10s (10ms pulse width, ~8mW laser power) and recorded for 2 min post-stimulation. g, Average ΔF/F traces of the NA sensors GRABNE1m and nLightG in response to optogenetic activation of the LC (mean±SEM for females and males, n(females)= 10, n(males)=5. h, Peak ΔF/F response of fiber photometry trace.

      Comment 4: Furthermore, behavioral changes following systemic pharmacologic or chemogenetic manipulation were observed in the open field task immediately after peripheral injections of yohimbine or CNO, respectively. Is this timing sufficient for both drugs to cross the blood brain barrier and to exert behavioral effects?

      Response: We have previously shown that chemogenetic activation of the LC through clozapine elicits pupil responses within 1-2 minutes after injection (Privitera et al., 2020; Zerbi et al., 2019). This indicates that clozapine rapidly crosses the blood brain barrier and affects LC activity within a few minutes after injection. Our additional experiments using genetically encoded sensors in the hippocampus show this even more directly (Supplementary Figure 2d), see also the response to Comment 3 above.

      Similarly, yohimbine also rapidly crosses the blood brain barrier within the same time frame (Hubbard et al., 1988). These observations are consistent with the rapid behavioral effects that can be detected within a few minutes after injection of clozapine for LC-DREADD activation (Zerbi et al., 2019), and for yohimbine as well (von Ziegler et al., 2023). In response to another comment of this reviewer, we have also re-analyzed the behavior presented in the current manuscript in time-bins of 3 minutes, which also shows the rapid onset of effects in response to yohimbine (within the first 3 min) and DREADDs (within 6 min), see Supplementary Fig. 3.

      Comment 5: Finally, the study shows that activation of noradrenergic hippocampus-projecting LC neurons is sufficient to regulate the expression of several hippocampal genes, although the necessity of these projection to induce the observed transcriptional effects has been tested to some extent through systemic blockade of beta-adrenoceptor, I believe the study would have benefited of more selective (optogenetic or chemogenetic) necessity experiments.

      Response: We understand the reviewer's point that blocking the LC during stress exposure would be an interesting experiment. However, it is very hard to completely silence the LC during intense stressors. In fact, despite intense efforts, we have not been able to silence the LC during swim stress exposure using DREADDs or other chemogenetic approaches (PSAM/PSEM). We were in fact able to silence the LC with the optogenetic inhibitor JAWS (and others have reported successful LC silencing with GtACR2), but there is a major issue involving the "rebound effect", where more NA is released once the inhibition is stopped. We would thus have had to optogenetically silence the LC for 45-90 min, which would create heat artifacts, and require challenging control experiments to draw firm conclusions. Given all these issues, we reasoned that blocking adrenergic receptors is a simple and elegant solution, which provides clear evidence for the necessity of beta-adrenergic signaling.

      Reviewer #2 (Recommendations For The Authors):

      Major concerns:

      Comment 6: The study focuses on the identification of transcriptional changes in the hippocampus induced by stress-mediated activation of the LC-NA system, however, noradrenaline release following stress exposure or yohimbine injection was measured in the cortex. Authors should consider measuring NA concentrations in the hippocampus after exposure to swim stress or administration of yohimbine, or at least explain their choice to analyse to cortex in the manuscript.

      Response: We have addressed this issue in detail in Response to "Reviewer 2, Comment #3", where we provided an overview of the additional data that support our approach. As mentioned before, measuring NA release after yohimbine is not compatible with our GRABNE-photometry approach, as the GRAB-sensor is based on alpha2-adrenoceptor. Here, we would like to add that measuring NA release using photometry during swim stress is also challenging. The challenge is the vigorous movement (swimming, typically in one direction), which creates pressure on the cables/implants. We felt that overcoming these experimental challenges (setup, troubleshooting and controls) would be beyond the scope of the paper, given that it is already known that this stressor leads to strong NA release in the hippocampus. We have now included references that demonstrate that all the stressors used in our work trigger NA increase in the hippocampus (see response to Reviewer 1, Comment 3): “Therefore, we assessed their expression in a dataset comparing the effect of various stressors on the hippocampal transcriptome (Floriou-Servou et al., 2018). The stressors included restraint, novelty and footshock stress, which have all previously been shown to increase hippocampal NA release (Hajós-Korcsok et al., 2003; Lima et al., 2019; Masatoshi Tanaka et al., 1982).”

      Comment 7: Concerning the experiment aimed at investigating sex differences in gene expression, it is not clear the reason why authors decided to restrict their analyses in females to the ventral hippocampal only. The explanation that in males they did not detect major differences between the dorsal and ventral hippocampus is not sufficient, because there could have been different effects in females. Therefore, the conclusion made by the authors that their "results suggest that the transcriptomic response is independent of sex" is not entirely correct, since sex differences were only evaluated in the ventral hippocampus.

      Response: We appreciate the reviewer's critique. As described above, we have now also sequenced the dorsal hippocampal tissue from the propranolol experiment (males and females, 32 samples) and additionally added an extensive meta-analysis of three large datasets (n=71) to compare transcriptional sex differences in response to stress. A detailed description of these experiments and how they have extended/supported our conclusions have been provided in response to Reviewer #1, Comment #2.

      Comment 8: Besides the effects on females, the same experiment examined whether propranolol by itself (in the absence of stress) would have been able to alter gene expression: such effects were not examined in the dorsal hippocampus. In contrast, in a different experiment, the effects of isoproterenol on genes expression were restricted to the dorsal hippocampus only. Furthermore, related to this latter experiment, intra-dorsal hippocampal injection of isoproterenol should presumably mimic the rise in NA observed after stress exposure, why was gene expression measured 90 min after isoproterenol central injections while in the other experiments gene expression was determined 45 min after stress, that is when authors observe the peak NA concentration?

      Response: We have addressed the reviewer's critique of dorsal vs ventral hippocampus by reanalyzing 32 additional samples from dorsal hippocampus of male and female mice after propranolol (or saline) injection. Please see response to Reviewer #1, comment #2.

      Regarding the time points: We have chosen the 45 and 90 min time points mainly for two reasons. First, cFos protein changes are known to be strongest 90 min after neuronal activation. Second, because we wanted to capture gene expression changes triggered by NA release, we reasoned that these effects must be fast and should thus be measured at an early transcriptional time-point (45min). However, after performing the time-course experiment after swim stress exposure (Figure 4d,c), we observed that the LC-NA-sensitive genes (e.g. Dio2 and several PP1-subunits) show the strongest changes 90 min after stress exposure. Therefore, in some of our experiments we opted to analyze gene expression changes at 90min, converging with the time-point we typically use for cFos staining. Contrary to the reviewer's statement, peak NA concentrations are not observed 45 min after the various interventions, but rather the peak in the main metabolite (MHPG) is observed then, due to the temporal dynamics of NA release and breakdown. NA release occurs immediately upon stress exposure (or direct LC activation), which we also show in the new photometry data described above. Thus, rapid NA release triggers intracellular cascades that lead to downstream transcriptional changes, which peak presumably between 4590 min later.

      Comment 9: Behavioral changes following systemic pharmacologic or chemogenetic manipulation were observed in the open field task immediately after peripheral injections of yohimbine or CNO, respectively. Is this timing sufficient for both drugs to cross the blood brain barrier and to exert behavioral effects? It is also not immediately clear the reason why the open field tasks have different durations depending on the experiments, which can also impact the results. Authors might also consider to split the open field data analyses in 2 or 3 min time-bins, to allow for a better comparison across the different results.

      Response: We thank the reviewer for the suggestion to plot the behavior data as time-bins. We have implemented this change for the yohimbine and DREADD experiments, and updated the corresponding figure accordingly (Supplementary Figure 3, pasted below for ease of reading). The new visualization clearly shows that yohimbine injection triggers rapid behavioral effects already in the first three minutes, whereas the LC-DREADD activation triggers behavioral changes within 3-6 minutes after injection. Thus, clear drug effects are visible in the first 10 minutes, which is comparable to the standard OFT test (10min testing) shown in response to swim stress exposure (Suppl. Figure 3a). The choice to expose mice to the OFT for 21 minutes in total was due to the fact that we based our experimental approach on the optogenetic LC-stimulation protocol first published by McCall and colleagues (McCall et al, Neuron, 2015), in which the LC is stimulated for 3 min followed by 3 min pauses (see Suppl. Figure 3d). Because of this on-off design, we decided to keep the optogenetic analysis simple and show the overall effect (Supplementary Figure 3d), particularly as we know that NA dynamics do not recover rapidly enough after 3 min continuous stimulation to justify a bin-analysis (unpublished data).

      Author response image 3.

      Effects of acute stress and noradrenergic stimulation on anxiety-like behaviour in the open field test. a, Stress-induced changes in the open field test 45 min after stress onset. Stressed animals show overall reductions in distance traveled (unpaired t-test; t=3.55, df=22, p=0.0018), time in center (welch unpaired t-test; t=3.50, df=13.61, p=0.0036), supported rears (unpaired t-test; t=3.39, df=22, p=0.0026) and unsupported rears (unpaired t-test; t=5.53, df=22, p = 1.47e-05) compared to controls (Control n = 12; Stress n = 12). This data have been previously published (von Ziegler et al., 2022). b, Yohimbine (3 mg/kg, i.p.) injected animals show reduced distance traveled (unpaired t-test; t=2.39, df=10, p=0.03772), reduced supported rears (unpaired t-test; t=6.56, df=10, p=0.00006) and reduced unsupported rears (welch unpaired t-test; t=3.69, df=4.4, p = 0.01785) compared to vehicle injected animals (Vehicle n = 6; Yohimbine n = 7). c, Chemogenetic LC activation induced changes in the open field test immediately after clozapine (0.03 mg/kg, i.p.) injection. hM3Dq+ animals show reduced distance traveled (unpaired t-test; t=6.28, df=13, p=0.00003), reduced supported rears (unpaired t-test; t=4.28, df=13, p=0.0009), as well as reduced unsupported rears (welch unpaired t-test; t=4.28, df=13, p = 0.00437) compared to hM3D- animals (hM3Dq- n = 7; hM3Dq+ n = 8). d, Optogenetic 5 Hz LC activation induced changes during the open field test. ChR2+ animals show reduced supported rears (unpaired t-test; t=2.42, df=64, p=0.0185) and reduced unsupported rears (unpaired ttest; t=2.91, df=64, p = 0.00499) compared to ChR2- animals (ChR2- n = 32; ChR2+ n = 36). Data expressed as mean ± SEM. p < 0.05, p < 0.01, p < 0.001, **p < 0.0001.

      Comment 9: The study shows that activation of noradrenergic hippocampus-projecting LC neurons is sufficient to regulate the expression of several hippocampal genes. I believe the study would have benefited of more selective necessity experiments. Authors might consider adding optogenetic (or chemogenetic) experiments aimed at inhibiting LC-NA hippocampal projections during stress exposure (or, alternatively, perform intrahippocampal pharmacological blockade of β-adrenoreceptors during stress exposure), and determine the effects on gene expression.

      Response: We kindly refer the reviewer to our previous response to Comment #2 above.

      Minor concerns:

      There is a typo in the abstract. Please correct "LN-NA" with "LC-NA"

      Response: Thank you, we have corrected it.

      References

      Bangasser, D. A., Eck, S. R., & Ordoñes Sanchez, E. (1/2019). Sex differences in stress reactivity in arousal and attention systems. Neuropsychopharmacology: Official Publication of the American College of Neuropsychopharmacology, 44(1), 129–139.

      Bangasser, D. A., Wiersielis, K. R., & Khantsis, S. (06/2016). Sex differences in the locus coeruleusnorepinephrine system and its regulation by stress. Brain Research, 1641, 177–188.

    1. Author Response

      The following is the authors’ response to the original reviews.

      Reviewer #1 (Public Review):

      Summary:

      Mice can learn to associate sensory cues (sound and light) with a reward or activation of dopamine neurons in the ventral tegmental area (VTA), and then anticipate the reward from the sensory cue only. Using this paradigm, Harada et al. showed that after learning, the cue is able to induce dopamine release in the projection targets of the VTA, namely the nucleus accumbens and lateral hypothalamus (LH). Within the LH, dopamine release from VTA neurons (either by presentation of the cue or direct optical stimulation of VTA neurons) activates orexin neurons, measured as an increase in intracellular calcium levels.

      Strengths:

      This study utilized genetically encoded optical tools to selectively stimulate dopamine neurons and to monitor dopamine release in target brain areas and the calcium response of orexin neurons. This allowed a direct assessment of the relationship between the behavioral response of the animals, the release of a key neurotransmitter in select brain areas, and its effect on target cells, with a precision previously not possible. The results shed light on the mechanism underlying reward-related learning and expectation.

      Weaknesses: - The Ca increase in orexin neurons in response to optical stimulation of VTA DA neurons is convincing. However, there is an accumulated body of literature indicating that dopamine inhibits orexin neurons through D2 receptors, particularly at high concentrations both directly and indirectly (PMID 15634779, 16611835, 26036709, 30462527; but note that synaptic effects at low conc are excitatory - PMID 30462527, 26036709). There should be a clear acknowledgment of these previous studies and a discussion directly addressing the discrepancy. Furthermore, there are in-vivo studies that investigated the role of dopamine in the LH involving orexin neurons in different behavioral contexts (e.g. PMID 24236888). The statement found in the introduction "whether and how dopamine release modulates orexin neuronal activity has not been investigated vigorously" (3rd para of Introduction) is an understatement of these previous reports.

      We thank the Reviewer for pointing out that we missed several important citations. We added the references mentioned and the discrepancy of concern is addressed in the discussion section

      • Along these lines, previous reports of concentration-dependent bidirectional dopaminergic modulation of orexin neurons suggest that high and low levels of DA would affect orexin neurons differently. Is there any way to estimate the local concentration of DA released by the laser stimulation protocol used in this study? Could there be a dose dependency in the Intensity of laser stimulation and orexin neuron response?

      We agree that this is an interesting point. However, one limitation of our study, and of intensity-based genetically-encoded sensors in general, is that the estimation of the concentration is technically difficult. The sensor effectively reports changes in extra-synaptic levels of neurotransmitters, but to get the absolute value other modalities would be needed such as fast scan voltammetry. This limitation is now included in the discussion section.

      • The transient dip in DA signal during omission sessions in Fig2C (approx 1% decrease from baseline) is similar in amplitude compared to the decrease seen in non-laser trails shown in Fig 1C right panel (although the time course of the latter is unknown as the data is truncated). The authors should clarify whether those dips are a direct effect of the cue itself or indeed reward prediction error.

      Thanks for raising this important point. Indeed, there is a dip of the signal during non-stimulation trials. At day 1, the delivery of the cue triggered a dip and at day 10, there was a slight increase of the signal and followed by the dip. The data is difficult to interpret but our hypothesis is that two components trigger this dip of the signal. One is the aversiveness of the cue. Because a relatively loud sound (90dB) was used for the cue, it would not be surprising if the auditory cue was slightly aversive to the experimental animals. It has been shown that aversive stimuli induce a dip of dopamine in the NAc, although it is specific to NAc subregions. The second component is reward prediction error. Although the non-laser paired cue never triggered the laser stimulation, it is similar to the laser paired one. In a way both are composed of loud tone and same color of the visual cue (spatially different). We think it is possible that reward-related neuronal circuit was slightly activated by the non-laser paired cue. In line with this interpretation, a small increase of the signal was observed at day 10 but not day 1. If our hypothesis is true, since this signal was induced by two components, further analysis is unfortunately difficult.

      • There seem to be orexin-negative-GCaMP6 positive cells (Fig. 4B), suggesting that not all cells were phenotypically orexin+ at the time of imaging.<br /> The proportion of GCaMP6 cells that were ORX+ or negative and whether they responded differently to the stimuli should be indicated.

      While we acknowledge the observation of orexin-negative-GCaMP6 positive cells in Figure 4B, it's important to note that this phenomenon is consistent with the characteristics of the hOX-GCaMP virus used in prior experiments. The virus has undergone thorough characterization, and it has been reported to exhibit over 90% specificity, as demonstrated in prior work conducted in the laboratory of one of our contributing authors (PMID: 27546579). To address the concern raised by the reviewer, we have included Supplemental Figure 4 confirming that all mice consistently exhibited qualitatively similar hOX-GCaMP transients upon dopaminergic terminal stimulation. This additional evidence supports the reliability and specificity of our experimental approach.

      • Laser stimulation of DA neurons at the level of cell bodies (in VTA) induces an increase in DA release within the LH (Fig. 3C, D), however, there is no corresponding Ca signal in orexin neurons (Fig.4C).

      We realized that the figures were not clear and we understood that the reviewer did not see any corresponding Ca signal, but this description is not true. We now added Supplemental Figure 3 to show that there is Ca signal at day 1 already.

      In contrast, stimulating DA terminals within the LH induces a robust, long-lasting Ca signal (> 30s) in orexin neurons (Fig. 5). The initial peak is blocked by raclopride but the majority of Ca signal is insensitive to DA antagonists (please add a positive control or cite references indicating that the dose of antagonists used was sufficient; also the timing of antagonist administration should be indicated).

      This is now included in the discussion section. Also, the timing and dose of the antagonist is now described in the method section.

      Taken together, these results seem to suggest that DA does not directly increase Ca signal in orexin neurons. What could be mediating the remaining component?

      This point has been included in the discussion section.

      • Similarly, there is an elevation of Ca signal in orexin neurons that remains significantly higher after the cue/laser stimulation (Fig. 4F). It appears that it is this sustained component that is missing in omission trials. This can be analyzed further.

      It is true that there is a sustained component in stimulation trials, that is missing in omission trials. Most likely that is evoked by the stimulation of dopamine neurons. We argue that this component is isolated in Fig 5 and analyzed as much as we can.

      • Mice of both sexes were used in this study; it would be interesting to know whether sex differences were observed or not.

      We agree that this is an important point. However, our sample number is not high enough to make a meaningful comparison between male and female.

      Reviewer #2 (Public Review):

      Summary:

      This is an interesting and well-written study assessing the role of dopaminergic inputs from the VTA on orexin cell responses in an opto-pavlovian conditioning task. These data are consistent with a possible role of this system in reward expectation and are surprisingly one of the first demonstrations of a role for dopamine in this phenomenon.

      Strengths:

      The study has used an interesting opto-Pavlovian approach combined with fibre photometry.

      Weaknesses:

      It is unclear what n size was used or analysed, particularly for AUC measures e.g. Figures 1 D/E and 3 G. The number of trials reflected and the animal numbers need clarification.

      The sample size is indicated in the legend section.

      The study focused on opto-stim omissions - this work would be significantly strengthened by a comparison to a real-world examination where animals are trained for a radiation reward (food pellet).

      We agree that this would be an important experiment. This experiment is partially done in one of the contributing authors laboratories (doi.org/10.1101/2022.04.13.488195) and would be one of our follow up study.

      Have the authors considered the role of orexin in the opposing situation i.e. a surprise addition of reward?

      That would be an interesting experiment. To do that, natural reward, not optical stimulation, should be used as a reinforcer. This could be part of our follow up study.

      Similarly, there remains some conjecture regarding the role of these systems in reward and aversion - have the authors considered aversive learning paradigms - fear, or fear extinction - to further explore the roles of this system? There are some (important) discussions about the possible role of orexin in negative reinforcement. Further studies to address this could be warranted.

      It is true that dopamine also plays a significant role in aversive learning. Therefore, this would be an interesting experiment. The discussion section now includes this point.

      I think some further discussion of the work by Lineman concerning the interesting bidirectional actions of d1/d2 r signalling on glutamatergic transmission onto orexin neurons is worthwhile. While this work is currently cited, the nuance and perhaps relevance to d1 and d2 signalling could be contextualised a little more (https://doi.org/10.1152/ajpregu.00150.2018).

      Thanks for the suggestion. The discussion has been expanded.

      Reviewer #3 (Public Review):

      Summary:

      Harada and colleagues describe an interesting set of experiments characterizing the relationship between dopamine cell activity in the ventral tegmental area (VTA) and orexin neuron activity in the lateral hypothalamus (LH). All experiments are conducted in the context of an opto-Pavlovian learning task, in which a cue predicts optogenetic stimulation of VTA dopamine neurons. With training, cues that predict DA stimulation come to elicit dopamine release in LH (a similar effect is seen in accumbens). After training, omission trials (cue followed by no laser) result in a dip (inhibition) of dopamine release in LH, characteristic of reward prediction error observed in the striatum. Across cue training, the activity pattern of orexin neurons in LH mirrors that of LH DA levels. However, unlike the DA signal, orexin neurons do not exhibit a decrease in activity in omission trials. Systemic blockade of D2 but not D1 receptors blocked DA release in LH following VTA DA cell stimulation.

      Strengths: Although much work has been dedicated to examining projections from orexin cells to VTA, less has been done to characterize reciprocal projections and their function. In this way, this paper is a very important addition to the literature. The experiments are technically sound (with some limitations, below) and utilize sophisticated approaches, the manuscript is nicely written, and the conclusions are mostly reasonable based on the data collected.

      Weaknesses:

      I believe the impact of the paper could be enhanced by considering and/or addressing the following:

      Major:

      • I encourage the authors to discuss in the Introduction previous work on DA regulation of orexin neurons. In particular, the authors cite, but do not describe in any detail, the very relevant Linehan paper (2019; Am J Physiol Regul) which shows that DA differentially alters excitatory/inhibitory input onto orexin neurons and that these actions are reversed by D1 vs D2 receptor antagonists. Another paper (Bubser, 2005, EJN) showed that dopamine agonists increase the activity of orexin neurons and that these effects are blocked by D1/D2 antagonists. The current findings should be discussed in the context of these (and any other relevant) papers in the Discussion, too.

      Thanks for the valuable suggestion. This point has been integrated and the introduction and discussion sections have been revised carefully.

      • In the Discussion, the authors provide two (plausible) explanations for why they did not observe a dip in the calcium signal of orexin neurons during omission trials. Is it not possible that these cells do not encode for this type of RPE?

      We completely agree that it is possible. Now our current hypothesis is that dopamine in the LH encodes RPE and that information is transmitted to orexin neurons. Orexin neurons integrate other information and encode something else, we call it ‘multiplexed cognitive information’. It is still open question what this means exactly. This point is now mentioned in the discussion section.

      • Related to the above - I am curious about the authors' thoughts on why there is such redundancy in the system. i.e. why is dopamine doing the same thing in NAC and LH in the context of cue-reward learning?

      Thank you for the question. This is an important point, indeed. Our current hypothesis is described in the discussion section.

      ’Our data indicate that dopamine in both the NAc and LH encodes reward prediction error (RPE). One open question is the existence of such a redundant mechanism. We hypothesize that dopamine in the LH boosts dopamine release via a positive feedback loop between the orexin and dopamine systems. It has already been established that some orexin neurons project to dopaminergic neurons in the VTA, positively modulating firing. On the other hand, our data indicate that dopamine in the LH stimulates orexinergic neurons. These collective findings suggest that when either the orexin or dopamine system is activated, the other system is also activated consequently. Although the current findings align with this idea, the hypothesis should be carefully challenged and scrutinized.’

      • The data, as they stand, are largely correlative and do not indicate that DA recruitment of orexin neurons is necessary for learning to occur. It would be compelling if blocking the orexin cell recruitment affected some behavioral outcomes of learning. Similarly - does raclopride treatment across training prevent learning?

      We appreciate the insightful comment. It is indeed a limitation of our study that we lack behavioral data. However, given the extensive previous research on the crucial role of orexin in motivated behavior, we argue that establishing dopaminergic regulation of the orexin system itself is a valuable contribution. This perspective is thoroughly discussed in the dedicated section of our paper. It's important to note that the injection of D2 antagonists, including raclopride, is known to induce significant sedation. Due to this sedative effect, combining behavioral experiments with these drugs poses considerable challenges.

      • Only single doses of SCH23390 and raclopride were used. How were these selected? It would be nice to use more of a dose range to show that 1) and effect of D1R blockade was not missed, and 2) that the reduction in orexin signal with raclopride was dose-dependent.

      The rationale of the dose has been added to the discussion session. It is reported that these doses block dopamine receptors. We agree that it would be nice to have a dose-response curve, we are reluctant to increase the doses to avoid adverse effect to the experimental animals. The doses we used effectively induced hypo-locomotion, although data is not shown.

      • Fig 1C, could the effect the authors observed be due to movement?

      We argue this is unlikely. We recorded two channels one for the control and the other one for the signal. The motion-related artifact is corrected based on the control channel. One example trace around the laser stimulation is shown below. Please note that a typical motion-related artifact is a fast dip of the signal, normally observed in both 405 and 465 nm channels.

      Relatedly, what was the behavior like when the cue was on? Did mice orient/approach the cue?

      Although it has been reported that rats approach the cue (PMID: 30038277) in a similar task, it was not obvious in our case. It could be because we used both visual and auditory cues. Mice showed a general increase of locomotion during the cue and the stimulation but the direction was not clear to the experimenter.

      Also, when does the learning about the cue occur? Does it take all 10 days of learning or does this learning/cue-induced increase in dopamine signaling occur in less than 10 days?

      It is hard to say when the learning occurs. When we look at the learning curve of Figures 1,3 and 4, it seems the response to the cue plateaus at day 5 but since we don’t have behavioral data, the assessment is relayed only on the neuronal signal.

      • Also related to the above, could the observed dopamine signal be a result of just the laser turning on? It would seem important to include mice with a control sensor.

      We recorded two channels, 405 nm and 465 nm wavelength. 405 nm signal did not show increase of the signal while 465 nm signal did. The example trace is shown. Besides, the sensor has been characterized by the corresponding author already so we argue that this is unlikely.

      Author response image 1.

      Fig 1E, the effect seems to be driven by one mouse which looks like it could be a statistical outlier. The inclusion of additional animals would make these data more compelling.

      We agree that adding more mice would make data more compelling. However, considering the fact that dopamine in the accumbens has been investigated vigorously and our data is in line with the prior studies, we argue that we have enough data to claim our conclusion.

      • For Fig 1C, 3D, 3F, and 4D, could the authors please show the traces for the entire length of laser onset? It would be helpful to see both the rise and the fall of dopamine signals.

      For Fig 1C, one panel has been added. For fig 3, 4, supplemental figure was created to show the signal around laser stimulation.

      • Fig 2C, could the authors comment on how they compared the AUC to baseline? Was this comparison against zero? Because of natural hills and troughs during signals prior to cue (which may not equate to a zero), comparing the omission-induced dip to a zero may not be appropriate. A better baseline might be using the signals prior to the cue.

      The signal immediately before the cue onset was considered as a baseline, and baseline was subtracted. This means zero and baseline would be the same in our way of analysis.

      • Could the authors comment on how they came up with the 4-5.3s window to observe the AUC in Fig 3H?

      Since the kinetic of dopamine in the NAc and LH is different, different time windows have been used to observed a dip of dopamine. The analysis of the kinetics has been added.

      Recommendations for the authors:

      Reviewer #1 (Recommendations For The Authors):

      Specific feedback to the authors

      • Sample size for each experiment/group could not be found.

      The sample size is now included in the legends.

      • In most figures, the timing of onset for the cue and laser stimulation is unclear. This makes the data interpretation difficult. They should be labeled as in Fig. 3C, for example.

      Panels have been updated to address this point.

      • Please provide the rationale for selecting the time range for the measurement of AUC for different experiments (e.g. Fig. 2C, 3H, 4A, 5F).

      The kinetics of dopamine in NAc and LH are different. This is now shown in the new Supplemental Figure 2. Based on this difference, the different window was chosen.

      • Fig. 1E, 3G right, 4E right: statistical analysis should use two-way repeated measures ANOVA rather than one-way ANOVA. Fig 1D, 3G left and 4E left panels can also be analyzed by two-way repeated measures ANOVA.

      We realized that those panels were redundant. Some panels have been removed and the analysis has been conducted according to this point.

      Minor comments:

      Fig. 2C can also show non-omission trials as a comparison.

      The panel has been updated.

      • The term "laser cue" is confusing, as the cue itself does not involve a laser.

      ’Laser-paired cue’ is used instead.

      • Color contrast can be improved for some figures, including Fig. 2C right, Fig. 3H right, and green and blue fluorescent fonts.

      The panels have been updated.

      • Figure legends: Tukey's test, rather than Tekey's test.

      This has been fixed.

      • There are some long-winded sentences that are hard to follow.

      Edited.

      • p.2, line 11 from bottom: should read ...the VTA evokes the release of dopamine.

      Edited

      • p.3, line 9: remove e from release.

      This has been addressed.

      Reviewer #3 (Recommendations For The Authors):

      Minor:

      • When discussing the understudied role of dopamine in brain regions other than the striatum in the Introduction, it might be helpful to cite this article: https://elifesciences.org/articles/81980 where the authors characterize dopamine in the bed nucleus of stria terminalis in associative behaviors and reward prediction error.

      The discussion session has been updated accordingly.

      • In the Discussion, it might be better to refrain from describing the results as 'measuring dopamine release' in the LH. Since there was no direct detection of dopamine release, rather a dopamine binding to the dLight receptors, referring to the detection as dopamine signaling/binding/transients is a better alternative.

      This point has been addressed.

      • In the Discussion, without measuring tonic dopamine release, it is difficult to say that there was a tonic dopamine release in the LH prior to negative RPE. In addition, I wouldn't describe the negative RPE as silencing of dopamine neurons projecting to the LH since this was not directly measured and it is hard to say for sure if the dip in dopamine is caused by silencing of the neurons. There certainly seems to be a reduction in extra-synaptic dopamine signaling in LH, however, what occurs upstream is unknown.

      We respectfully disagree with this point. In our opinion, the dopamine transient is more important than the firing of dopamine neurons because what matters for downstream neurons is dopamine concentration. For example, administration of cocaine increases the dopamine concentration extra-synaptically via blockade of DAT, while the firing of dopamine neurons go down via activation of D2 receptors expressed in dopamine neurons. Administration of cocaine is not known to induce negative RPE.

      • Typo at multiple places: 'Tekey's multiple comparison test'.

      This has been fixed.

    1. Author response:

      The following is the authors’ response to the original reviews.

      eLife Assessment

      Building on their own prior work, the authors present valuable findings that add to our understanding of cortical astrocytes, which respond to synaptic activity with calcium release in subcellular domains that can proceed to larger calcium waves. The proposed concept of a spatial "threshold" is based on solid evidence from in vivo and ex vivo imaging data and the use of mutant mice. However, details of the specific threshold should be taken with caution and appear incomplete unless supported by additional experiments with higher resolution in space and time.

      We thank the reviewers and editors for the positive assessment of our work as containing valuable findings that add to our understanding of cortical astrocytes. We also appreciate their positive appraisal of the proposed concept of a spatial threshold supported by solid evidence. 

      Regarding their specific comments, we truly appreciate them because they have helped to clarify issues and to improve the study. Point-by-point responses to these comments are provided below. Regarding the general comment on the spatial and temporal resolution of our study, we would like to clarify that the spatial and temporal resolution used in the current study (i.e., 2 - 5 Hz framerate using a 25x objective with 1.7x digital zoom with pixels on the order of 1 µm2) is within the norm in the field, does not compromise the results, nor diminish the main conceptual advancement of the study, namely the existence of a spatial threshold for astrocyte calcium surge. 

      We respect the thoughtfulness of the reviewers and editors towards improving the paper.

      Public Reviews:

      Reviewer #1 (Public Review):

      Lines et al., provide evidence for a sequence of events in vivo in adult anesthetized mice that begin with a footshock driving activation of neural projections into layer 2/3 somatosensory cortex, which in turn triggers a rise in calcium in astrocytes within "domains" of their "arbor". The authors segment the astrocyte morphology based on SR101 signal and show that the timing of "arbor" Ca2+ activation precedes somatic activation and that somatic activation only occurs if at least {greater than or equal to}22.6% of the total segmented astrocyte "arbor" area is active. Thus, the authors frame this {greater than or equal to}22.6% activation as a spatial property (spatial threshold) with certain temporal characteristics - i.e., must occur before soma and global activation. The authors then elaborate on this spatial threshold by providing evidence for its intrinsic nature - is not set by the level of neuronal stimulus and is dependent on whether IP3R2, which drives Ca2+ release from the endoplasmic reticulum (ER) in astrocytes, is expressed. Lastly, the authors suggest a potential physiologic role for this spatial threshold by showing ex vivo how exogenous activation of layer 2/3 astrocytes by ATP application can gate glutamate gliotransmission to layer 2/3 cortical neurons - with a strong correlation between the number of active astrocyte Ca2+ domains and the slow inward current (SIC) frequency recorded from nearby neurons as a readout of glutamatergic gliotransmission. This is interesting and would potentially be of great interest to readers within and outside the glia research community, especially in how the authors have tried to systematically deconstruct some of the steps underlying signal integration and propagation in astrocytes. Many of the conclusions posited by the authors are potentially important but we think their approach needs experimental/analytical refinement and elaboration.

      We thank the reviewer for her/his positive appraisal and comments that has helped us to improve the study. In response to their insights, we aim to address the key points raised below:

      (1) Sequence of Events: We acknowledge the reviewer's interest in our findings regarding the sequence of events. We have provided a more detailed description of the methods and results to clarify the spatiotemporal relationships between domain activation and spatiotemporal clustering, to centripetal and centrifugal calcium propagation in relation to soma activation.

      (2) Spatial Threshold: The reviewer accurately identifies our characterization of a spatial threshold (≥22.6% activation) with temporal characteristics as a crucial aspect of our study. We have expanded upon this concept by offering a clearer illustration of how this threshold relates to somatic and global activation.

      (3) Intrinsic Nature of Spatial Threshold: The reviewer's insightful observation regarding the inherent quality of the spatial threshold, regardless of its dependence on neuronal stimuli is noteworthy. We have provided additional details to substantiate this claim, shedding more light on the fundamental nature of this phenomenon.

      (4) Physiological Implications: The reviewer rightly highlights the potential physiological significance of our findings, particularly in relation to gliotransmission in cortical neurons. We have enhanced our discussion by elaborating on the implications of these observations.

      The primary issue for us, and which we would encourage the authors to address, relates to the low spatialtemporal resolution of their approach. This issue does not necessarily compromise the concept of a spatial threshold, but more refined observations and analyses are likely to provide more reliable quantitative parameters and a more comprehensive view of the mode of Ca2+ signal integration in astrocytes. 

      We agree with the reviewer that our spatial-temporal resolution (2 – 5 Hz framerate using a 25x objective and 1.7x digital zoom with pixels on the order of 1 µm) does not compromise the proposed concept of the existence of a spatial threshold for the intracellular calcium expansion.

      For this reason, and because their observations might be perceived as both a conceptual and numerical standard in the field, we believe that the authors should proceed with both experimental and analytical refinement. Notably, we have difficulty with the reported mean delays of astrocyte Ca2+ elevations upon sensory stimulation. The 11s delay for response onset in "arbor" and 13s in the soma are extremely long, and we do not think they represent a true physiologic latency for astrocyte responses to the sensory activity. Indeed, such delays appear to be slower even than those reported in the initial studies of sensory stimulation in anesthetized mice with limited spatial-temporal resolution (Wang et al. Nat Neurosci., 2006) - not to say of more recent and refined ones in awake mice (Stobart et al. Neuron, 2018) that identified even sub-second astrocyte Ca2+ responses, largely preserved in IP3R2KO mice. Thus, we are inclined to believe that the slowness of responses reported here is an indicator of experimental/analytical issues. There can be several explanations of such slowness that the authors may want to consider for improving their approach: (a) The authors apparently use low zoom imaging for acquiring signals from several astrocytes present in the FOV: do all of these astrocytes respond homogeneously in terms of delay from sensory stimulus? Perhaps some are faster responders than others and only this population is directly activated by the stimulus. Others could be slower in activation because they respond secondarily to stimuli. In this case, the authors could focus their analysis specifically on the "fast-responding population". (b) By focusing on individual astrocytes and using higher zoom, the authors could unmask more subtle Ca2+ elevations that precede those reported in the current manuscript. These signals have been reported to occur mainly in regions of the astrocyte that are GCaMP6-positive but SR101-negative and constitute a large percentage of its volume (Bindocci et al., 2017). By restricting analysis to the SR101-positive part of the astrocyte, the authors might miss the fastest components of the astrocyte Ca2+ response likely representing the primary signals triggered by synaptic activity. It would be important if they could identify such signals in their records, and establish if none/few/many of them propagate to the SR-101-positive part of the astrocyte. In other words, if there is only a single spatial threshold, the one the authors reported, or two or more of them along the path of signal propagation towards the cell soma that leads eventually to the transformation of the signal into a global astrocyte Ca2+ surge. 

      We thank the reviewer for these excellent and important comments. The qualm with the mean delays of astrocyte activation is indeed a result of averaging together astrocyte responses to a 20 second stimulus. Indeed, astrocyte responses are heterogeneous and many astrocytes respond much quicker, as can be seen in example traces in Figs. 1D, 1G, and 3C. Indeed, with any biological system variability exists, however here we take the averaged responses in order to identify a general property of astrocyte calcium dynamics: the existence of the concept of a spatial threshold for astrocyte calcium surge. We have now included a paragraph in the Discussion section on this subject on P15, L16-22:

      “We were able to discover this general phenomenon of astrocyte physiology through the use of a novel computational tool that allowed us to combine almost 1000 astrocyte responses. Variation is rife in biological systems, and there are sure to be eccentricities within astrocyte calcium responses. Here, we focused on grouped data to better understand what appears to be an intrinsic property of astrocyte physiology. We used different statistical examinations and tested our hypothesis in vivo and in situ, and all these methods together provide a more complete picture of the existence of a spatial threshold for astrocyte calcium surge.“

      The specialized work of Stobart et al. 2018, was focused more on the fast activation of microdomain subpopulations than the induction of later somatic activation. Indeed, Stobart et al. 2018 and Wang et al. 2006 also found that somatic responses of astrocytes were delayed in the range of seconds. Importantly, Wang et al., 2006 describe that the activation of astrocytes is frequency dependent, that is, the higher the frequency, the faster and higher the activation. In the present, work we stimulated at just 2 Hz to better investigate the spatial threshold. Excitingly, the results showed by Stobart et al., 2018 agree with ours, Rupprecht et al. 2024 and Fedotova et al. 2023, that there is a sequence of activation from the domains to the somas, which could be due to the time that is required for the summation of the initial microdomain signal to reach a threshold capable to activate the soma. These above referenced studies have many similarities with our own but are different in the underlying scientific question that led to diverging methodology, however we want to stress that we agree with the reviewers that our methods provide sufficient evidence for the cell-scale scientific phenomenon that we are studying, which is the spatial threshold for astrocyte calcium surge. Finally, we have included an additional figure (new Figure 5) that only looks at the calcium dynamics of early responding cells and found no significant difference in the spatial threshold in this population compared to our original quantification.

      In this context, there is another concept that we encourage the authors to better clarify: whether the spatial threshold that they describe is constituted by the enlargement of a continuous wavefront of Ca2+ elevation, e.g. in a single process, that eventually reaches 22.6% of the segmented astrocyte, or can it also be

      constituted by several distinct Ca2+ elevations occurring in separate domains of the arbor, but overall totaling 22.6% of the segmented surface? Mechanistically, the latter would suggest the presence of a general excitability threshold of the astrocyte, whereas the former would identify a driving force threshold for the centripetal wavefront. In light of the above points, we think the authors should use caution in presenting and interpreting the experiments in which they use SIC as a readout. Their results might lead some readers to bluntly interpret the 22.6% spatial threshold as the threshold required for the astrocyte to evoke gliotransmitter release. Indeed, SIC are robust signals recorded somatically from a single neuron and likely integrate activation of many synapses all belonging to that neuron. On the other hand, an astrocyte impinges in a myriad of synapses belonging to several distinct neurons. In our opinion, it is quite possible that more local gliotransmission occurs at lower Ca2+ signal thresholds (see above) that may not be efficiently detected by using SIC as a readout; a more sensitive approach, such as the use of a gliotransmitter sensor expressed all along the astrocyte plasma-membrane could be tested to this aim.  

      The reviewer raised an excellent point. Whether the spatial threshold of 22.6% occur in the segmented astrocyte or may be reached occurring in separate domains of the arbor, is an important question and we address this by the inclusion of a novel analysis shown in the new figure (new Figure 5) in the revised version of the manuscript. In this new analysis, we demonstrate that the average distance between domain activation is not significantly different between subthreshold activity and the activity that precedes or follows the suprathreshold cellular activation. In contrast, we do find a significant difference in the average time between domain activation between subthreshold activity and activity that precedes and follows suprathreshold activation. We go further with a generalized linear model to show that percent area of active domains and temporal clustering is related to soma activation and not spatial clustering. This suggests that domain activation doesn’t need to be spatially clustered together to induce soma activation and subsequent calcium surge, but more importantly, domain activation must be over the spatial threshold and occur within a timeframe. This has been added to the Results on P10, L2-40:

      “Our results demonstrate the relationship between the percentage of active domains and soma activation and subsequent calcium surge. Next, we were interested in the spatiotemporal properties of domain activity leading up to and during calcium surge. Because we imaged groups of astrocytes, we were able to constrain our analyses to fast responders (onset < median population onset) in order to evaluate astrocytes that were more likely to respond to neuronal-evoked sensory stimulation and not nearby astrocyte activation (Figure 5A). In this population the spatial threshold was 23.8% within the 95% confidence intervals of [21.2%, 24.0%]. First, we created temporal maps, where each domain is labeled as its onset relative to soma activation, of individual astrocyte calcium responses to study the spatiotemporal profile of astrocyte calcium surge (Bindocci et al., 2017; Rupprecht et al., 2024) (Figure 5B). Using temporal maps, we quantified the spatial clustering of responding domains by measuring the average distance between active domains. We found that the average distance between active domains in subthreshold astrocyte responses were not significantly different from pre-soma suprathreshold activity (16.3 ± 0.4 µm in No-soma cells versus 16.2 ± 0.3 µm in Pre-soma cells, p = 0.75; n = 286 No-soma vs n = 326 Pre-soma, 30 populations and 3 animals; Figure 5C). Following soma activation, astrocyte calcium surge was marked with no significant change in the average distance between active domains (16.0 ± 0.3 µm in Post-soma cells versus 16.3 ± 0.4 µm in No-soma cells, p = 0.57 and 16.2 ± 0.3 µm in Presoma cells, p = 0.31; n = 326 soma active and n = 286 no soma active, 30 populations and 3 animals; Figure 5C). Taken together this suggests that on average domain activation happens in a nonlocal fashion that may illustrate the underlying nonlocal activation of nearby synaptic activity. Next, we interrogated the temporal patterning of domain activation by quantifying the average time between domain responses, and found that the average time between domain responses was significantly decreased in pre-soma suprathreshold activity compared to subthreshold activities without subsequent soma activation (9.4 ± 0.3 s in No-soma cells versus 4.4 ± 0.2 s in Pre-soma cells, p < 0.001; n = 326 soma active vs n = 286 not soma active, 30 populations and 3 animals; Figure 5D). The average time between domain activation was even less after the soma became active during calcium surge (2.1 ± 0.1 s in Post-soma versus 9.4 ± 0.3 s in No-Soma cells, p < 0.001 and 4.4 ± 0.1 s in Pre-soma cells, p < 0.001; n = 326 soma active and n = 286 not soma active, 30 populations and 3 animals; Figure 5D). This corroborates our findings in Figure S2 and highlights the difference in temporal profiles between subthreshold activity and astrocyte calcium surge. 

      We then tested the contribution of each of our three variables describing domain activation (percent area, average distance and time) to elicit soma activation by creating a general linear model. We found that overall, there was a significant relationship between these variables and the soma response (p = 5.5e-114), with the percent area having the largest effect (p = 3.5e-70) followed by the average time (p = 3.6e-7), and average distance having no significant effect (p = 0.12). Taken together this suggests that the overall spatial clustering of active domains has no effect on soma activation, and the percent area of active domains within a constrained time window having the largest effect.”

      Regarding comments on SIC, we fully agree with the reviewer. In the revised version of the manuscript, we have included text in the discussion to ensure the correct interpretation of the results, i.e., the observed 22.6% spatial threshold for the SIC does not necessarily indicate an intrinsic property of gliotransmitter release; rather, since SICs have been shown to be calcium-dependent, it is not surprising that their presence, monitored at the whole-cell soma, matches the threshold for the intracellular calcium extension. We have added to the Discussion P16, L15-30:

      “Astrocyte calcium activity induces multiple downstream signaling cascades, such as the release of gliotransmitters (Araque et al., 2014; de Ceglia et al., 2023). Using patch-clamp recordings of a single nearby neuron we showed that a nearby population of astrocyte calcium surge is also correlated to the increase in slow inward currents (SICs), previously demonstrated to be dependent on astrocytic vesicular release of glutamate (Araque et al., 2000; Durkee et al., 2019; Fellin et al., 2004). The increase of SICs we observed from patching a single neuron is likely the integration of gliotransmitter release onto synapses from a group of nearby astrocytes. Indeed, subthreshold astrocyte calcium increases alone can trigger activity in contacted dendrites (Di Castro et al., 2011). An exciting avenue of future research would be to observe the impact of a single astrocyte calcium surge on nearby neurons (Refaeli and Goshen, 2022). How many neurons would be affected, and would this singular event be observable through patch clamp from a single neuron? The output of astrocyte calcium surge is equally important to network communication as the labeling of astrocyte calcium surge, as it identifies a biologically relevant effect onto nearby neurons. Many downstream signaling mechanisms may be activated following astrocyte calcium surge, and the effect of locally concentrated domain activity vs astrocyte calcium surge should be studied further on different astrocyte outputs.”

      Additional considerations are that the authors propose an event sequence as follows: stimulus - synaptic drive to L2/3 - arbor activation - spatial threshold - soma activation - post soma activation - gliotransmission. This seems reminiscent of the sequence underlying neuronal spike propagation - from dendrite to soma to axon, and the resulting vesicular release. However, there is no consensus within the glial field about an analogous framework for astrocytes. Thus, "arbor activation", "soma activation", and "post soma activation" are not established `terms-of-art´. Similarly, the way the authors use the term "domain" contrasts with how others have (Agarwal et al., 2017; Shigetomi et al., 2013; Di Castro et al., 2011; Grosche et al., 1999) and may produce some confusion. The authors could adopt a more flexible nomenclature or clarify that their terms do not have a defined structural-functional basis, being just constructs that they justifiably adapted to deal with the spatial complexity of astrocytes in line with their past studies (Lines et al., 2020; Lines et al., 2021).

      We agree there is no consensus within the glial field about this event sequence. One major difference between this sequence of events and neuronal spike propagation is directionality from dendrite to soma to axon. It is unknown whether directionality of the calcium signal exists in astrocytes. However, our finding in Figure 5E suggests a directionality of centripetal propagation from the arborization to the soma to elicit calcium surge that leads to centrifugal propagation. In the Results on P10-11, L41-8:

      “Recent work studying astrocyte integration has suggested a centripetal model of astrocyte calcium, where more distal regions of the astrocyte arborization become active initially and activation flows towards the soma (Fedotova et al., 2023; Rupprecht et al., 2024). Here, we confirm this finding, where activated domains located distal from the soma respond sooner than domains more proximal to the soma (linear correlation: p < 0.05, R2 = 0.67; n = 30 populations, 3 animals; Figure 4E). Next, we build upon this result to also demonstrate that following soma activation, astrocyte calcium surge propagates outward in a centrifugal pattern, where domains proximal to the soma become active prior to distal domains (linear correlation: p < 0.01, R2 = 0.89; n = 30 populations, 3 animals; Figure 4E). Together these results detail that intracellular astrocyte calcium follows a centripetal model until the soma is activated leading to a calcium surge that flows centrifugally. This suggests that astrocytes have the capabilities to integrate the nearby local synaptic population, and if this activity exceeds the spatial threshold then it leads to a whole-cell response that spreads outward.” 

      And in the Discussion P15, L3-15:

      “Close examinations of the calcium surge uncovered distinct propagations whether before or after soma activation. Firstly, our analysis found that temporal clustering changed before and after calcium surge, with both being above subthreshold activity, and that this characteristic was absent when assessing spatial clustering. When comparing the percent area, spatial and temporal clustering of active domains using a GLM, we found that the percent area was the most significant parameter describing a threshold to soma activation. We then compared the delay of domain activation and its distance from the soma, and recreated previous results that suggest a centripetal model of astrocytic calcium responses from the distal arborizations to the soma (Fedotova et al., 2023; Rupprecht et al., 2023). Here, we went a step further and discovered that soma activation switches this directionality for astrocytic calcium surge to propagate outward in a centrifugal manner away from the soma. Taken together, these results demonstrate the integrative potential of astrocyte calcium responses and characterize further the astrocyte calcium surge to relay this other parts of the astrocyte.”

      The term “microdomain” is used in the references above to define distal subcellular domains in contact with synapses, and in order to dissociate from this term we adopt the nomenclature “domain” to define all subcellular domains in the astrocyte arborization. These items have been discussed and clarified in the revised version of the manuscript on P5, L17-19:

      “The concept of domain to define all subcellular domains in the astrocyte arborization should not be confused with the concept of microdomain, that usually refers to the distal subcellular domains in contact with synapses.”

      Our previous points suggest that the paper would be significantly strengthened by new experimental observations focusing on single astrocytes and using acquisitions at higher spatial and temporal resolution. If the authors will not pursue this option, we encourage them to at least improve their analysis, and at the same time recognize in the text some limitations of their experimental approach as discussed above. We indicate here several levels of possible analytical refinement.

      We believe our spatial (25x objective and 1.7x digital zoom with pixels on the order of 1µm) and temporal (2 – 5 Hz framerate) resolution is within the range used in the glial field. In any case the existence of a spatial threshold for astrocyte calcium surge is not compromised with the use of this imaging resolution.

      The first relates to the selection of astrocytes being analyzed, and the need to focus on a much narrower subpopulation than (for example) 987 astrocytes used for the core data. This selection would take into greater consideration the aspects of structure and latency. With the structural and latency-based criteria for selection, the number of astrocytes to analyze might be reduced by 10-fold or more, making our second analytical recommendation much more feasible.

      We agree that individual differences exist, however, establishing a general concept requires the sampling of many astrocytes. Nevertheless, we have included a new figure (new Figure 5) that analyzes early responders.

      For structure-based selection - Genetically-encoded Ca2+ indicators such as GCaMP6 are in principle expressed throughout an astrocyte, even in regions that are not labelled by SR101. Moreover, astrocytes form independent 3D territories, so one can safely assume that the GCaMP6 signal within an astrocyte volume belongs to that specific astrocyte (this is particularly evident if the neighboring astrocytes are GCaMP6negative). Therefore, authors could extend their analysis of Ca2+ signals in individual astrocytes to the regions that are SR101-negative and try to better integrate fast signals in their spatial threshold concept. Even if they decided to be conservative on their methods, and stick to the astrocyte segmentation based on the SR-101 signal, they should acknowledge that SR101 dye staining quality can vary considerably between individual astrocytes within a FOV - some astrocytes will have much greater structural visibility in the distal processes than others. This means that some astrocytes may have segmented domains extending more distally than others and we think that authors should privilege such astrocytes for analysis. However, cases like the representative astrocytes shown in Figure 4A or Figure S1B, have segmented domains localized only to proximal processes near the soma. Accordingly, given the reported timing differences between "arbor" and "soma" activation, one might expect there to be comparable timing differences between domains that are distal vs proximal to the soma as well. Fast signals in peripheral regions of astrocytes in contact with synapses are largely IP3R2-independent (Stobart et al., 2018). However, the quality of SR101 staining has implications for interpreting the IP3R2 KO data. There is evidence IP3R2 KO may preferentially impact activity near the soma (Srinivasan et al., 2015). Thus, astrocytes with insufficient staining - visible only in the soma and proximal domains - might show a biased effect for IP3R2 KO. While not necessarily disrupting the core conclusions made by the authors based on their analysis of SR101-segmented astrocytes, we think results would be strengthened if astrocytes with sufficient SR101 staining - i.e. more consistent with previous reports of L2/3 astrocyte area (Lanjakornsiripan et al., 2018) - were only included. This could be achieved by using max or cumulative projections of individual astrocytes in combination with SR101 staining to construct more holistic structural maps (Bindocci et al., 2017).

      We agree with the ideas concerning SR101, and indeed there could be variability in the origins of the astrocyte calcium signal. Astrocyte territory boundaries can be difficult to discern when both astrocytes express GCaMP6. Also, SR101-negative domains could encapsulate an area that is only partially that of astrocyte territory, including also extracellular space. Here we take a conservative approach to constrain ROIs to SR101positive astrocyte territory outlines without invading neighboring cells or extracellular space in order to reduce error in the estimate of a spatial threshold. The effect of IP3R2 KO preferentially impacting activity near the soma is interesting, and in line with our conclusions. We agree that the findings from SR101-negative pixels would not necessarily disrupt the core conclusions of the study, and the additional analysis suggested would further strengthen results. We have since included on the limitations of the study in the Discussion P15, L3137:

      “In this study, we chose to limit our examinations of calcium activity that was within the bounds determined by SR101 staining. Much work has shown that astrocyte territories are more akin to sponge-like morphology with small microdomains making up the end feet of their distal arborizations (Baldwin et al., 2024). Here, we took a conservative approach to not incorporate these fine morphological processes and only take SR101-postive pixels for analysis in order to reduce the possible error of including a neighboring astrocyte or extracellular space in our analyses. Much work can be done to extend these results.”

      For latency-based selection - The authors record calcium activity within a FOV containing at least 20+ astrocytes over a period of 60s, during which a 2Hz hindpaw stimulation at 2mA is applied for 20s. As discussed above, presumably some astrocytes in a FOV are the first to respond to the stimulus series, while others likely respond with longer latency to the stimulus. For the shorter-latency responders <3s, it is easier to attribute their calcium increases as "following the sensory information" projecting to L2/3. In other cases, when "arbor" responses occur at 10s or later, only after 20 stimulus events (at 2Hz), it is likely they are being activated by a more complex and recurrent circuit containing several rounds of neuron-glia crosstalk etc., which would be mechanistically distinct from astrocytes responding earlier. We suggest that authors focus more on the shorter latency response astrocytes, as they are more likely to have activity corresponding to the stimulus itself.

      We agree that different times of astrocyte calcium increases may be due to different mechanisms outside of the astrocyte. We believe the spatial threshold will be intrinsic to these external variables; yet we believe that longer latency responses are physiological and may carry important information to determining the astrocyte calcium responses. Indeed, we have performed the spatial threshold analysis on early responders (first half of responding cells), and found the spatial threshold in that population (23.8%) is within the 95% confidence interval [21.2%, 24.0%]. Additionally, the slow responders were also within the confidence interval (22.6%).

      The second level of analysis refinement we suggest relates specifically to the issue of propagation and timing for the activity within "arbor", "soma" and "post-soma". Currently, the authors use an ROI-based approach that segments the "arbor" into domains. We suggest that this approach could be supplemented by a more robust temporal analysis. This could for example involve starting with temporal maps that take pixels above a certain amplitude and plot their timing relative to the stimulus-onset, or (better) the first active pixel of the astrocyte. This type of approach has become increasingly used (Bindocci et al., 2017; Wang et al., 2019; Ruprecht et al., 2022) and we think its use can greatly help clarify both the proposed sequence and better characterize the spatial threshold. We think this analysis should specifically address several important points:

      We agree that the creation of temporal maps from our own data would be interesting, and we provide the results of the suggested analysis within the new figure (new Figure 5) in the revised version of the manuscript. In this analysis we show that subthreshold, pre-soma and post-soma dynamics are significantly different in time. These added results of including temporal maps strengthen our claim of a spatial threshold, by quantifying the distinct temporal and spatial dynamics of domain activation before and after the spatial threshold is met (i.e. soma activation), and highlights differences in subthreshold and suprathreshold activity.

      (1) Where/when does the astrocyte activation begin? Understanding the beginning is very important, particularly because another potential spatial threshold - preceding the one the authors describe in the paper - could gate the initial activation of more distal processes, as discussed above. This sequentially earlier spatial threshold could (for example) rely on microdomain interaction with synaptic elements and (in contrast) be IP3R2 independent (Srinivasan et al., 2015, Stobart et al., 2018). We would be interested to know whether, in a subset of astrocytes that meet the structure and latency criteria proposed above and can produce global activation, there is an initial local GCaMP6f response of a minimal size that must occur before propagation towards the soma begins. The data associated with varying stimulus parameters could potentially be useful here and reveal stimulus intensity/duration-dependent differences.

      This is a very important point. It is difficult to pinpoint the beginning of the signal, which is why we rely on the average of responses. The additional analysis we provide based on temporal maps (new Figure 5) shows a very interesting result in that there is no significant difference between the spatial clustering of, or average distance between, activated domains in subthreshold and pre-soma suprathreshold activity. This result, along with the General Linear Model, suggests that there is not another subcellular potential spatial threshold, as the activity is the same. Instead, the main difference between activity in the domains that leads to soma activation or not is the overall percentage of domains active and not necessarily how that spatial activity is organized. We have also added this point in the Discussion section to highlight the importance of this result. P15, L3-8:

      “Close examinations of the calcium surge uncovered distinct propagations whether before or after soma activation. Firstly, our analysis found that temporal clustering changed before and after calcium surge, with both being above subthreshold activity, and that this characteristic was absent when assessing spatial clustering. When comparing the percent area, spatial and temporal clustering of active domains using a GLM, we found that the percent area was the most significant parameter describing a threshold to soma activation.”

      (2) Whether the propagation in the authors' experimental model is centripetal? This is implied throughout the manuscript but never shown. We think establishing whether (or not) the calcium dynamics are centripetal is important because it would clarify whether spatially adjacent domains within the "arbor" need to be sequentially active before reaching the threshold and then reaching the soma. More broadly, visualizing propagation will help to better visualize summation, which is presumably how the threshold is first reached (and overcome).

      The alternative hypothesis of a general excitability threshold, as discussed above, would be challenged here and possibly rejected, thereby clarifying the nature of the Ca2+ process that needs to reach a threshold for further expansion to the soma and other parts of the astrocyte.

      We agree that our view is centripetal when considering activity leading up to soma activation. Indeed, we have found arborization activity precedes soma activity (Figure 3), soma activity appears to rely on the percent area of domain activity (Figure 4), and pre-soma domain activity comes online earlier in domains distal from the soma (new Figure 5). However, whether this is intrinsic or due to the fact that synapses are more likely to occur in the periphery requires further studies. Our new results in the new Figure 5 demonstrating that subthreshold activity has a spatial organization that is not significantly different than pre-soma activity in suprathreshold cases argues in favor of a general excitability threshold hypothesis. However, we do not see these hypotheses as mutually exclusive. Excitingly, we have also found that following soma activation, calcium surge appears to follow a centrifugal propagation. We have since added the topic of a centripetal-centrifugal experimental model to the Discussion P15, L8-15:

      “We then compared the delay of domain activation and its distance from the soma, and recreated previous results that suggest a centripetal model of astrocytic calcium responses from the distal arborizations to the soma (Fedotova et al., 2023; Rupprecht et al., 2024). Here, we went a step further and discovered that soma activation switches this directionality for astrocytic calcium surge to propagate outward in a centrifugal manner away from the soma. Taken together, these results demonstrate the integrative potential of astrocyte calcium responses and characterize further the astrocyte calcium surge to relay this other parts of the astrocyte.”

      (3) In complement to the previous point: we understand that the spatial threshold does not per se have a location, but is there some spatial logic underlying the organization of active domains before the soma response occurs? One can easily imagine multiple scenarios of sparse heterogeneous GCaMP6f signal distributions that correspond to {greater than or equal to}22.6% of the arborization, but that would not be expected to trigger soma activation. For example, the diagram in Figure 4C showing the astrocyte response to 2Hz stim (which lacks a soma response) underscores this point. It looks like it has {greater than or equal to}22.6% activation that is sparsely localized throughout the arborization. If an alternative spatial distribution for this activity occurred, such that it localized primarily to a specific process within the arbor, would it be more likely to trigger a soma response?

      This is an interesting point and our new spatiotemporal analysis found in the new figure (new Figure 5) aims to shed some light on this and is answered above. To our knowledge, there is no mechanism in astrocytes to impose directionality on calcium propagation, like rectifying voltage-gated sodium channels in neuronal voltage propagation. We found that the delay of domain activation compared to soma onset is significantly correlated to the distance from the soma (new Figure 5E). In addition, spatial clustering is not significantly different compared in pre-soma vs. non responders or post-soma. Together this suggests that centripetal propagation may be occurring throughout the entire cell and not in a local clustered way. Our findings also suggest that following soma activation astrocyte calcium surge follows a mostly centrifugal pattern (new Figure 5E).

      (4) Does "pre-soma" activation predict the location and onset time of "post-soma" activation? For example, are arbor domains that were part of the "pre-soma" response the first to exhibit GCaMP6f signal in the "post-soma" response?

      Please see above comments.

      Reviewer #2 (Public Review):

      Lines et al investigated the integration of calcium signals in astrocytes of the primary somatosensory cortex. Their goal was to better characterize the mechanisms that govern the spatial characteristics of calcium signals in astrocytes. In line with previous reports in the field, they found that most events originated and stayed localized within microdomains in distal astrocyte processes, occasionally coinciding with larger events in the soma, referred to as calcium surges. As a single astrocyte communicates with hundreds of thousands of synapses simultaneously, understanding the spatial integration of calcium signals in astrocytes and the mechanisms governing the latter is of tremendous importance to deepen our understanding of signal processing in the central nervous system. The authors thus aimed to unveil the properties governing the emergence of calcium surges. The main claim of this manuscript is that there would be a spatial threshold of ~23% of microdomain activation above which a calcium surge, i.e. a calcium signal that spreads to the soma, is observed. Although the study provides data that is highly valuable for the community, the conclusions of the current version of the manuscript seem a little too assertive and general compared with what can be deduced from the data and methods used.

      The major strength of this study is the experimental approach that allowed the authors to obtain numerous and informative calcium recordings in vivo in the somatosensory cortex in mice in response to sensory stimuli as well as in situ. Notably, they developed an interesting approach to modulating the number of active domains in peripheral astrocyte processes by varying the intensity of peripheral stimulation (its amplitude, frequency, or duration).

      We thank the reviewer for their kind and thoughtful review of our study.

      The major weakness of the manuscript is the method used to analyze and quantify calcium activity, which mostly relies on the analysis of averaged data and overlooks the variability of the signals measured. As a result, the main claims from the manuscript seem to be incompletely supported by the data. The choice of the use of a custom-made semi-automatic ROI-based calcium event detection algorithm rather than established state-of-the-art software, such as the event-based calcium event detection software AQuA (DOI: 10.1038/s41593-019-0492-2), is insufficiently discussed and may bias the analysis. Some references on this matter include: Semyanov et al, Nature Rev Neuro, 2020 (DOI: 10.1038/s41583-020-0361-8); Covelo et al 2022, J Mol Neurosci (DOI: 10.1007/s12031-022-02006-w) & Wang et al, 2019, Nat Neuroscience (DOI: 10.1038/s41593-019-0492-2). Moreover, the ROIs used to quantify calcium activity are based on structural imaging of astrocytes, which may not be functionally relevant.

      Unfortunately, there is no general consensus for calcium analysis in the astrocyte or neuronal field, and many groups use custom made software made in lab or custom software such as GECIquant, STARDUST, AQuA or AQuA2. While AQuA is an event-based calcium event detection software, it may be that not including inactive domains that are SR101 positive could underestimate the spatial threshold for calcium surge. Our data is not based on the functional events but is based on calcium with structural constraints within a single astrocyte. This is crucial to properly determine the ratio of active vs inactive pixels within a single astrocyte.

      For the reasons listed above, the manuscript would probably benefit from some rephrasing of the conclusions and a discussion highlighting the advantages and limitations of the methodological approach. The question investigated by this study is of great importance in the field of neuroscience as the mechanisms dictating the spatio-temporal properties of calcium signals in astrocytes are poorly characterized, yet are essential to understand their involvement in the modulation of signal integration within neural circuits.

      We thank the reviewer for their suggestions to benefit the conclusions and discussion. We have now included a paragraph outlining the limitations of the study in the Discussion P15, L23-37:

      “The investigation of the spatial threshold could be improved in the future in a number of ways. One being the use of state-of-the-art imaging in 3D(Bindocci et al., 2017). While the original publication using 3D imaging to study astrocyte physiology does not necessarily imply that there would be different calcium dynamics in one axis over another, the three-dimensional examination of the spatial threshold could refine the findings we present here. To better control the system, mice imaged here were under anesthesia, and this is a method that has been used to characterize many foundational physiological results in the field (Hubel and Wiesel, 1962; Mountcastle et al., 1957). However, assessing the spatial threshold in awake freely moving animals would be the next logical step. In this study, we chose to limit our examinations of calcium activity that was within the bounds determined by SR101 staining. Much work has shown that astrocyte territories are more akin to sponge-like morphology with small microdomains making up the end feet of their distal arborizations (Baldwin et al., 2024). Here, we took a conservative approach to not incorporate these fine morphological processes and only take SR101-postive pixels for analysis in order to reduce the possible error of including a neighboring astrocyte or extracellular space in our analyses. Much work can be done to extend these results.”

      Reviewer #3 (Public Review):

      Summary:

      The study aims to elucidate the spatial dynamics of subcellular astrocytic calcium signaling. Specifically, they elucidate how subdomain activity above a certain spatial threshold (~23% of domains being active) heralds a calcium surge that also affects the astrocytic soma. Moreover, they demonstrate that processes on average are included earlier than the soma and that IP3R2 is necessary for calcium surges to occur. Finally, they associate calcium surges with slow inward currents. Strengths:

      The study addresses an interesting topic that is only partially understood. The study uses multiple methods including in vivo two-photon microscopy, acute brain slices, electrophysiology, pharmacology, and knockout models. The conclusions are strengthened by the same findings in both in vivo anesthetized mice and in brain slices.

      We thank the reviewer for the positive assessment of the study and his/her comments.

      Weaknesses:

      The method that has been used to quantify astrocytic calcium signals only analyzes what seems to be a small proportion of the total astrocytic domain on the example micrographs, where a structure is visible in the SR101 channel (see for instance Reeves et al. J. Neurosci. 2011, demonstrating to what extent SR101 outlines an astrocyte). This would potentially heavily bias the results: from the example illustrations presented it is clear that the calcium increases in what is putatively the same astrocyte goes well beyond what is outlined with automatically placed small ROIs. The smallest astrocytic processes are an order of magnitude smaller than the resolution of optical imaging and would not be outlined by either SR101 or with the segmentation method judged by the ROIs presented in the figures. Completely ignoring these very large parts of the spatial domain of an astrocyte, in particular when making claims about a spatial threshold, seems inappropriate. Several recent methods published use pixel-by-pixel event-based approaches to define calcium signals. The data should have been analyzed using such a method within a complete astrocyte spatial domain in addition to the analyses presented. Also, the authors do not discuss how two-dimensional sampling of calcium signals from an astrocyte that has processes in three dimensions (see Bindocci et al, Science 2017) may affect the results: if subdomain activation is not homogeneously distributed in the three-dimensional space within the astrocyte territory, the assumptions and findings between a correlation between subdomain activation and somatic activation may be affected.

      In order to reduce noise from individual pixels, we chose to segment astrocyte arborizations into domains of several pixels. As pointed out previously, including pixels outside of the SR101-positive territory runs the risk of including a pixel that may be from a neighboring cell or mostly comprised of extracellular space, and we chose the conservative approach to avoid this source of error. We agree that the results have limitations from being acquired in 2D instead of 3D, but it is likely to assume the 3D astrocyte is homogeneously distributed and that the 2D plane is representative of the whole astrocyte. Indeed, no dimensional effects were reported in Bindocci et al, Science 2017. We have included a paragraph in the discussion to address this limitation in our study on P15, L23-27:

      “The investigation of the spatial threshold could be improved in the future in a number of ways. One being the use of state-of-the-art imaging in 3D(Bindocci et al., 2017). While the original publication using 3D imaging to study astrocyte physiology does not necessarily imply that there would be different calcium dynamics in one axis over another, the three-dimensional examination of the spatial threshold could refine the findings we present here.”

      The experiments are performed either in anesthetized mice, or in slices. The study would have come across as much more solid and interesting if at least a small set of experiments were performed also in awake mice (for instance during spontaneous behavior), given the profound effect of anesthesia on astrocytic calcium signaling and the highly invasive nature of preparing acute brain slices. The authors mention the caveat of studying anesthetized mice but claim that the intracellular machinery should remain the same. This explanation appears a bit dismissive as the response of an astrocyte not only depends on the internal machinery of the astrocyte, but also on how the astrocyte is stimulated: for instance synaptic stimulation or sensory input likely would be dependent on brain state and concurrent neuromodulatory signaling which is absent in both experimental paradigms. The discussion would have been more balanced if these aspects were dealt with more thoroughly.

      Yes, we agree that this is a limitation, and we acknowledge this is in the Discussion P15, L27-31:

      “To better control the system, mice imaged here were under anesthesia, and this is a method that has been used to characterize many foundational physiological results in the field (Hubel and Wiesel, 1962; Mountcastle et al., 1957). However, assessing the spatial threshold in awake freely moving animals would be the next logical step.”

      The study uses a heaviside step function to define a spatial 'threshold' for somata either being included or not in a calcium signal. However, Fig 4E and 5D showing how the method separates the signal provide little understanding for the reader. The most informative figure that could support the main finding of the study, namely a ~23% spatial threshold for astrocyte calcium surges reaching the soma, is Fig. 4G, showing the relationship between the percentage of arborizations active and the soma calcium signal. A similar plot should have been presented in Fig 5 as well. Looking at this distribution, though, it is not clear why ~23% would be a clear threshold to separate soma involvement, one can only speculate how the threshold for a soma event would influence this number. Even if the analyses in Fig. 4H and the fact that the same threshold appears in two experimental paradigms strengthen the case, the results would have been more convincing if several types of statistical modeling describing the continuous distribution of values presented in Fig. 4E (in addition to the heaviside step function) were presented.

      We agree with the reviewer and have added to the paper a discussion for our justification on the use of the Heaviside step function, and have included this in the methods section. We chose the Heaviside step function to represent the on/off situation that we observed in the data that suggested a threshold in the biology. We agree with the reviewer that Fig. 4G is informative and demonstrates that under 23% most of the soma fluorescence values are clustered at baseline. We agree that a different statistical model describing the data would be more convincing and confirmed the spatial threshold with the use of a confidence interval in the text and supported the use of percent domains active for this threshold over other properties such as spatial or temporal clustering using a general linear model. P18-19, L34-2:

      “Heaviside step function

      The Heaviside step function below in equation 4 is used to mathematically model the transition from one state to the next and has been used in simple integrate and fire models (Bueno-Orovio et al., 2008; Gerstner, 2000).

      The Heaviside step function 𝐻(𝑎) is zero everywhere before the threshold area (𝑎 ) and one everywhere afterwards. From the data shown in Figure 4E where each point (𝑆(𝑎)) is an individual astrocyte response with its percent area (𝑎) domains active and if the soma was active or not denoted by a 1 or 0 respectively. To determine 𝑎 in our data we iteratively subtracted 𝐻(𝑎) from  𝑆(𝑎) for all possible values of 𝑎 to create an error term over 𝑎. The area of the minimum of that error term was denoted the threshold area.”

      The description of methods should have been considerably more thorough throughout. For instance which temperature the acute slice experiments were performed at, and whether slices were prepared in ice-cold solution, are crucial to know as these parameters heavily influence both astrocyte morphology and signaling. Moreover, no monitoring of physiological parameters (oxygen level, CO2, arterial blood gas analyses, temperature etc) of the in vivo anesthetized mice is mentioned. These aspects are critical to control for when working with acute in vivo two-photon microscopy of mice; the physiological parameters rapidly decay within a few hours with anesthesia and following surgery.

      We have increased the thoroughness of our methods section. Especially including that body temperature and respiration were indeed monitored throughout anesthesia.

      Recommendations for the authors:  

      Reviewer #1 (Recommendations For The Authors):

      (1) We think it would improve the paper if the authors provided a frame-by-frame example over (for example) 10-15 frames showing the spatiotemporal evolution of responses, where each frame represents 1s or 2s. This could be included with the temporal maps we proposed above.

      We agree that this is a useful example and have included it in our new figure (new Figure 5, specifically see Figure 5A) that uses temporal maps to analyze the spatiotemporal properties of calcium dynamics (Figure 5B).

      (2) Concerning the evidence in the present manuscript, we are not clear on what "populations" means. Can the authors clarify in methods? It is our understanding that 987 astrocytes from 30 populations from 3 mice were the source for the core data in the paper. What are the 30 populations, and how were the 987 astrocytes distributed across the populations? Are they roughly 10 FOVs per mouse? If so, please clarify roughly how far apart FOVs from the same mouse were, and how much delay between stim protocol application there was when a FOV was changed to a new FOV. Also, if for example, the 10th FOV from mouse 1 "saw" 9 rounds of stimulation before recording the response to the 10th stim round. To this point, was there any indication of response differences in populations that were recorded earlier vs later in the experimental sequence for each mouse?

      Descriptions of data will be included with the uploaded datasets following acceptance.

      (3) The description of the results on page 6 is a bit confusing for us. In lines 1-4, are the authors saying that 57.7% of astrocytes in a FOV exhibited responses within their soma and arborization, while 15.1% had responses only in arborization? If so, this is not clear to us from Figure 2C, where we count ~25 astrocytes in the FOV, maybe 8 or 9 astrocytes with activity in the arborization + soma (after stimulation), and 8 or 9 astrocytes with responses only in arborization. Is there something we do not understand, or is the second panel simply not representative of the group data?

      Figure 2D is representative of the group data and does indeed show 57.7% of the population responds within the soma and arborization, and a 15.1% of astrocytes with responses in only their arborizations. It is unable to observe in this image whether arborizations are active or just increases in one or a few domains, as may not be enough activity to be detected when sampling over the entire arborization.

      (4) In the second part of page 6 - when the authors apply linear regression - are they saying that there is a linear relationship between the amount (area) of activity measured in the arborization versus the soma, where populations of astrocytes with 50% activation of the arborization also tend to have 50% activation in their somas? If so, then this is not apparent by the map provided in Figure 2C, where it looks like soma activation (within the subpopulation) is 100% irrespective of the apparent activity in the arborization. This needs to be clarified. If not, and what they mean is that the probability of finding an active soma is related to the amount of activation within the arborization, this needs to be stated more clearly.

      When testing the linear relationship between somas active vs arborizations active, we find a significant linear correlation (p < 0.001, R2 = 0.90).

      (5) In the experiments where stimulation duration, frequency, and intensity were varied to determine the percentage of domains that were on, it would be helpful to better understand the protocol in terms of sequence. In the methods it seems that hindpaw stimulation intensity was first pseudo-randomly varied at 2Hz for 10s, followed by pseudorandomly varied stimulation frequency and then pseudo-randomly varied duration - both at 2mA for 10s. Is this correct?

      We have since updated the methods section to better describe the experimental protocol.

      (6) In Figure 3E the alignment of the "arbor" to the somatic response is a bit misleading. The signals being averaged for the "arbor" are composed of temporally heterogeneous sources (from distal and proximal domains) and when averaged will produce an artificially slow rise time. In contrast, the averaged somatic signals are composed of much more homogenous sources (arising from a more singular event) and therefore have a sharp rise time. It would make more sense to align their kinetics relative to the stimulus onset. It would also make more sense to compare the somatic response of astrocytes to the "arbor" of astrocytes which respond rapidly vs slowly to the foot-shock.

      Aligning the responses to the stimulus onset would exacerbate the artificially slow rise time for the soma and arborization as not all cells come online at the same time from stimulus onset.

      Reviewer #2 (Recommendations For The Authors):

      Data availability

      It seems that the data is not shared on a public repository, while it appears to be necessary according to eLife's general principles (see https://elife-rp.msubmit.net/html/eliferp_author_instructions.html#dataavailability).

      We will upload raw data to a repository upon acceptance of the manuscript.

      Data analysis

      - Why did the authors choose the heaviside step function to characterize conditions for somatic event initiation? It seems that this approach is averaging very heterogeneous data (some cells do not display somatic events even with ~50% domains active while some display somatic events with < 5 it seems).

      Please see discussion to variability in the responses to the public reviews. We have since included more discussion on the use of the Heaviside step function in the Methods section.  

      - Averaging of the data. It seems that the approach chosen to quantify calcium activity overlooks the variability of the signals measured ("Astrocyte calcium quantifications were averaged over all astrocytes of a single video and these values were used in statistical testing.", l.22-23, page 15). What is the variability of the measured features between different astrocytes? Between different animals? To what extent does this averaging strategy overlook the variability of the signals/how much information do we expect to lose? The manuscript would probably benefit from a more advanced statistical approach to analyze the data.

      Is it possible to extract information from the data that would indicate mechanisms allowing somatic activity when the percentage of domain activation was lower than the threshold? How about the opposite (i.e when no global event was triggered even when the percentage of domain activation was high)?

      We are indeed combining the responses from many different diverse astrocyte responses, and we see this as a strength of the paper. Variation is a hallmark of biology, and we have added this to the discussion. In the rare cases where astrocyte somas do not come online when the percent of arborizations is over threshold, or the opposite when somas activate with little domain activation, we would say this is most likely due to imaging 2D instead of the entire 3D cell. We have also added this into our discussion.

      - Here are a few suggestions for additional analysis that might be of interest to the community:

      - Measuring calcium activity in domains depending on their distance from the soma. This would allow us to better understand the spatial integration of the signals and notably answer the following question: Does the emergence of somatic events depend on the spatial distribution of active domains? (and does a smaller domain-soma distance facilitate the emergence of a calcium surge with a lower percentage of active domains?) These measurements could be visualized with plots of xy position of the domains (domain-soma distance) = f(time) with a colormap reflecting dF/F0, for example, at different times pre- and post-somatic events. Instead of DF/F0, these plots could also display the correlation between domain activities.

      We have performed this analysis, and it is now in the new figure (new Figure 5).

      - Adding temporality to the data analysis. It seems that calcium activity is "concatenated" during the whole duration prior to the somatic event (pre-soma) and after (post-soma). However, it is unclear how long the domains remained active and how many domains were still active at the onset of the somatic event. Adding a finer temporal analysis might help answer questions such as the potential need for some degree of synchronization of domain activity to trigger calcium surges.

      It could notably be interesting to measure the level of synchrony of events as a function of their distance from the soma and to analyze how it correlates with the properties of the somatic event.

      We have now included temporal analysis of astrocyte calcium surge in our new figure (new Figure 5). While we did see examples of spatially clustered domain activation in our data, those examples usually included other non-clustered domain activities and when including all of the active domains within an astrocytes arborization, we found no difference between the distance between activated domains before and after soma activation, even when comparing to subthreshold domain activity.

      Experiments

      - Would it be possible to apply different levels of stimulation to a given cell in order to discriminate whether the "no-soma" cells can display somatic events when neuronal activity is enhanced?

      Increased sensory stimulation does increase soma activity (Please see Lines et al., Nature Communications, 2020). An example of increased stimulation leading to somatic activation where it was not present in lower stimuli can be seen in Figure 4A-C.

      - Why choose a stimulation of 2 mA, 2 Hz for 20 sec in the experiments on IP3R2-/- mice?

      Has the same set of various stimulation protocols featured in Figure 4 been applied to IP3R2-/- mice? If so, were more domains activated as stimulation intensity (amplitude; duration, or frequency) increased? Could it trigger somatic events? This information seems necessary to be able to assert that calcium surges rely on the IP3R2 pathway.

      These experiments were not performed.

      -  Adding intermediary values of ATP pulse duration to Figure 6 (e.g. 50 ms and 75 ms) might strengthen the claim that the linear increase of SIC frequency with ATP application duration is only observed above the ~23% threshold.

      Agreed, however these experiments were not performed.

      Minor corrections to the text and figures.

      Methods

      The reader might benefit from a little more detail regarding the analysis of calcium signals. Notably, what was the duration of the calcium recordings? Was it constant across the different conditions tested in the study? Was it different in slice experiments versus in vivo experiments? What were the durations of the pre- and post- soma recordings and their variability? Was the calcium activity normalized for each astrocyte or animal? If not, why not consider normalizing the post-stimulation activity with pre-stimulation baseline activity?

      Similarly, some information on the stimulation protocol seems to be lacking: what was the frequency and intensity of the stimulus in the experiments where stimulus duration varied? Concurrently, what were the duration and intensity when frequency varied? What were the duration and frequency when the intensity varied?

      It might be beneficial to add further information on the algorithm of the Calsee software. What is it performing? How was it tested? Why is it referred to as "semi"-automatic, i.e. what might the user be needing to do manually? The segmentation seems to be omitting some branches connecting distal ROIs to the soma (see e.g. Fig S1.E). How would this influence the analysis and results?

      Results

      - Some assessments in the manuscript seem a bit too assertive/general compared to what can be deduced from the evidence presented in the figures. It could be beneficial to the reader to rephrase the latter. Some examples are listed below:

      - "These results indicate that astrocyte responses occurred initially in the arborizations, which is consistent with the idea that synapses are likely to be accessed at the astrocyte arborization ", l.11-12 page 7. The fact that the time to peak is lower in the arborization does not necessarily mean that signals initiate there. It could be because the kinetics/pathways in those compartments are different or there could be a dilution effect in the soma. Indeed, an influx of the same amount of calcium ions in the soma vs in a small domain will not correspond to the same DF/F0 in those compartments and might thus remain undetected in the soma.

      - "Using transgenic IP3R2-/- mice, we found that the activation of type-2 IP3 receptors is necessary for the generation of astrocyte calcium surge" (page 4, line 1-2), "present data further demonstrate that IP3R2 are necessary for the propagation of astrocyte calcium surge." (l. 18-19 page 13) -> As discussed above, the evidence does not seem to be strong enough to assert that IP3R2 is necessary to trigger somatic events. The results indicate that the IP3R2 pathway seems to facilitate the emergence of somatic events. As astrocytes differ strongly in terms of morphology and expression profiles depending on physiological conditions, the conclusions of this study might only apply to the specific experimental conditions used: region studied, age of the animal, type of sensory stimuli performed, and so on.

      - "These results indicate that spatial threshold of the astrocyte calcium surge has a functional impact on gliotransmission, which have important consequences on the spatial extension of the astrocyte-neuron communication and synaptic regulation", l.41-48 page 11. Figure 6 seems to indicate a correlation between the proportion of astrocyte domains activated and the frequency of SICs. The data seems insufficient to conclude that there is a causal relationship between calcium surge in the astrocyte and gliotransmission or SIC frequency.

      -" These results indicate that, on average, subcellular calcium events located in astrocyte arborizations are related to soma activation.", page 6 l 15-16. It may be more informative to specify the correlation measured: i.e the larger the arborization activity, the larger the percentage of active somas.

      Figures

      Figure 2: Adding more details in the figure legend explaining how the different parameters are calculated might be useful to the reader. Notably, what does soma active (%) refer to?

      Figure 3: Could it be possible to add individual traces of calcium activity in the soma and arborization of individual cells to provide a glimpse of the variability of the signals measured?

      Fig4. B-C: Could it be possible to add in the legend information on the timeline between stimulation and calcium signal recording? (and the duration of the latter).

      Fig4 D-E: Why is the maximum number of active domains in panel D ~50-60% but goes up to ~100% in panel E? Could it be that plotting SEM rather than STD might misrepresent the variability in the percentage of active domains for each stimulus property?

      Fig4F: It seems that the threshold changes with the frequency of the stimulus: e.g. at 10 Hz, the threshold seems larger than 22.6%. What would that mean?

      Fig4G: - Why do some data points display a soma amplitude < 0 DF/F0 ?

      - Why choose a sigmoid fit? What are the statistics associated to the fit? Is it in accordance with the threshold of 23%? Would a linear fit provide a good fit?

      Fig5F: - It seems that a few IP3R2-/- astrocytes displayed somatic events? If so, it might be interesting to mention this in the discussion section and to speculate on why that might be. - It seems that panel 5F displays the average percentage of somas that got activated rather than the probability of somatic events.

      - Is it possible that the effect seen in domains vs arborization is due to statistical effects (as n=2450 vs 112)?

      Fig S1: Panel D legend: double labeling of the radius used for each plot might be useful, notably for colorblind readers as the colors might be hard to see.

      Discussion

      - The discussion section might benefit from a discussion on the similitude between the data presented here and previous reports that reported similar results, i.e that most calcium signals in astrocytes were located in the distal processes, forming microdomains that rarely propagated to the soma. These include Bindocci et al 2017 Science (DOI:10.1126/science.aai8185) and Georgiou et al, Science Advances, 2022 (DOI: 10.1126/sciadv.abe5371).

      Thank you for the suggestions. We have now changed portions of the Methods, Results  and Discussion sections.

      Reviewer #3 (Recommendations For The Authors):

      The text could potentially be improved somewhat.

      Thank you.

    1. Author response:

      The following is the authors’ response to the original reviews.

      Public Reviews:

      Reviewer #1 (Public Review):

      Summary:

      In this study, Maestri et al. use an integrative framework to study the evolutionary history of coronaviruses. They find that coronaviruses arose recently rather than having undergone ancient codivergences with their mammalian hosts. Furthermore, recent host switching has occurred extensively, but typically between closely related species. Humans have acted as an intermediate host, especially between bats and other mammal species.

      Strengths:

      The study draws on a range of data sources to reconstruct the history of virus-host codivergence and host switching. The analyses include various tests of robustness and evaluations through simulation.

      Weaknesses:

      The analyses are limited to a single genetic marker (RdRp) from coronaviruses, but using other sections of the genome might lead to different conclusions. The genetic marker also lacks resolution for recent divergences, which precludes the detailed examination of recent host switches. Careful and detailed reconstruction of the timescale would be helpful for clarifying the evolutionary history of coronaviruses alongside their hosts.

      The use of a single short genetic marker (the RdRp palmprint region) from coronaviruses is indeed a limitation. However, this marker is the one that is currently used for routinely delimiting operational taxonomic units in RNA viruses and reconstructing their evolutionary history (Edgar et al. 2022, see also the Serratus project; https://serratus.io/); therefore, we took the conscious decision early on to rely on this expertise. Unfortunately, this marker cannot provide robust timescale reconstructions for coronavirus evolution (previous estimates of coronavirus origin range from around 10 thousand years ago to 293 million years ago depending on modeling assumptions). Only future genomic work across Coronaviridae that will characterize multiple genetic regions with different evolutionary rates will allow us to precisely elucidate the timescale of the evolutionary history of coronaviruses alongside their hosts. In the meantime, we show here that, while the RdRp palmprint region cannot by itself resolve the precise timescale of coronavirus evolution, it strongly suggests, when used along with cophylogenetic approaches, a recent evolutionary origin in bats.

      We now further discuss these issues and the perspectives offered by future genomic work on lines 462-485.  

      Reviewer #2 (Public Review):

      Summary:

      In their study titled "Recent evolutionary origin and localized diversity hotspots of mammalian coronaviruses," authors Benoît Perez-Lamarque, Renan Maestri, Anna Zhukova, and Hélène Morlon investigate the complex evolutionary history of coronaviruses, particularly those affecting mammals, including humans. The study focuses on unraveling the evolutionary trajectory of these viruses, which have shown a high propensity for causing pandemics, as evidenced by the SARS-CoV2 outbreak.

      The research addresses a significant gap in our understanding of the evolutionary dynamics of coronaviruses, particularly their history, patterns of host-to-host transmission, and geographical spread. These aspects are important for predicting and managing future pandemic scenarios.

      Historically, studies have employed cophylogenetic tests to explore virus-host relationships within the Coronaviridae family, often suggesting a long history of virus-host codiversification spanning millions of years. However, the team led by Perez-Lamarque proposes a novel phylogenetic framework that contrasts this traditional view. Their approach, which involves adapting gene tree-species tree reconciliation, is designed to robustly test the validity of two competing scenarios: an ancient origination and codiversification versus a more recent emergence and diversification through host switching.

      Upon applying this innovative framework to the study of coronaviruses and their mammalian hosts, the authors' findings challenge the prevailing notion of a deep evolutionary history. Instead, their results strongly support a scenario where coronaviruses have a more recent origin, likely in bat populations, followed by diversification predominantly through hostswitching events. This diversification, interestingly, seems to occur preferentially within mammalian orders.

      A critical aspect of their findings is the identification of hotspots of coronavirus diversity, particularly in East Asia and Europe. These regions align with the proposed scenario of a relatively recent origin and subsequent localized host-switching events. The study also highlights the rarity of spillovers from bats to other species, yet underscores the relatively higher likelihood of such spillovers occurring towards humans, suggesting a significant role for humans as an intermediate host in the evolutionary journey of these viruses.

      The research also points out the high rates of host-switching within mammalian orders, including between humans, domesticated animals, and non-flying wild mammals.

      In conclusion, the study by Perez-Lamarque and colleagues presents an important quantitative advance in our understanding of the evolutionary history of mammalian coronaviruses. It suggests that the long-held belief in extensive virus-host codiversification may have been substantially overestimated, paving the way for a reevaluation of how we understand, predict, and potentially control the spread of these viruses.

      Strengths:

      The study is conceptually robust, and its conclusions are convincing.

      Weaknesses:

      Despite the availability of a dated host tree the authors were only able to use the "undated" model in ALE, with the dated method (which only allows time-consistent transfers) failing on their dataset (possibly due to dataset size?). Further exploration of the question would be potentially valuable.

      Our intuition is that ALE in its “dated” version does not necessarily fail on our dataset due to its size: ALE runs, but it provides unrealistic parameter estimates and is not able to output possible reconciliations, as mentioned in our Material and Methods section. We think this issue is mostly due to the fact that there is no pattern of codiversification: the coronavirus and mammal trees are so distinct that finding a reconciliation scenario between these trees with time-consistent switches is very difficult and ALE fails at estimating an amalgamated likelihood for such an unlikely scenario. We now ran the dated version of ALE independently on the smaller alpha and betacoronaviruses datasets. It still fails on the betacoronaviruses dataset.  On the alphacoronaviruses dataset, it does output significant reconciliations, however these reconciliations have a majority of events of transfers and losses, confirming that codiversification is unlikely in this clade.

      Reviewer #3 (Public Review):

      Summary:

      This work uses tools and concepts from co-phylogenetic analyses to reconstruct the evolutionary and diversification history of coronaviruses in mammals. It concludes that crossspecies transmissions from bats to humans are a relatively common event (compared to bats to other species). Across all mammals, the diversification history of coronaviruses suggests that there is potential for further evolutionary diversification.

      Strengths:

      The article uses an interesting approach based on jointly looking at the extant network of coronaviruses-mammals interactions, and the phylogenetic history of both these organisms. The authors do an impressive job of explaining the challenges of reconstructing evolutionary dynamics for RNA viruses, and this helps readers appraise the relevance of their approach.

      Weaknesses:

      I remain unconvinced by the argument that sampling does not introduce substantial biases in the analyses. As the authors highlight, incomplete knowledge of the extant interactions would lead to a biased reconstruction of the diversification history. In a recent paper (Poisot et al. 2023, Patterns), we look at sampling biases in the virome of mammals and suggest that is a fairly prominent issue, that is furthermore structured by taxonomy, space, and phylogenetic position. Case in point, even for betacoronaviruses, there have been many newly confirmed hosts in recent years. For organisms that have received less intense scrutiny, I think a thorough discussion of potential gaps in data would be required (see for example Cohen et al. 2022, Nat. Comms).

      I was also surprised to see little discussion of the differences between alpha and beta coronaviruses - there is evidence that they may differ in their cross-species transmission (see Caraballo et al. 2022 Micr. Spectr.), which could call into question the relevance of treating all coronaviruses as a single, homogeneous group.

      Some of the discussions in this paper also echo previous work by e.g. Geoghegan et al. (see 2017, PLOS Pathogens), which I was surprised to not see discussed, as it is a much earlier investigation of the relative frequencies of co-divergence and host switches for different viral families, with a deep discussion of how this may structure future evolutionary dynamics.

      We totally agree that sampling biases in the virome of mammals is a prominent issue, which is why we conducted a series of sensitivity analyses to test their effect on our main conclusions. We thoroughly tested the effect of (i) the unequal sampling effort across mammalian species that have been screened and (ii) the unequal screening of mammalian species across the mammalian tree of life by subsampling the data to correct for the unequal sampling effort (see Supporting Information Text). In both cases, we still reported low support for a scenario of codiversification, the origin in bats in East Asia, the preferential host switches within mammalian orders, and the rare spillovers from bats to humans. The robustness of our findings to sampling biases may be explained by the fact that the cophylogenetic approach we used (ALE) explicitly accounts for undersampling by assuming that all host switches involve unsampled intermediate hosts. To address the reviewer's comment, we now better underline the importance of sampling biases in our main text (see Discussion, lines 487-494) with supporting references (note that we did not find the Cohen et al. Nature Comm reference). We also better highlight our sensitivity analyses by moving them from the Supporting Information Text to the main text. 

      We agree that distinguishing between alpha and beta coronaviruses provides useful additional insights. We have run separate cophylogenetic analyses for these two sub-clades and now report the results of these additional analyses in the revised manuscript, and put them in context with the existing literature about the two sub-clades.

      We were not aware of the work of Geoghegan et al. (see 2017, PLOS Pathogens), thank you for providing this reference that is now cited. 

      Reviewer #1 (Recommendations For The Authors):

      (1) Overall I found this paper to be quite difficult to follow. The text needs clearer structure, which can be helped by writing in shorter paragraphs and adding section headings. For example, there are some very long paragraphs starting on L83, L176, L215, L511, and L598.

      We have now added section headings and divided these paragraphs into smaller ones.

      (2) It would be helpful to define some of the key terminology relating to the evolutionary interactions between the viruses and their hosts. Some of the terms that are typically used in the context include "coevolution", "cospeciation", "codivergence", and "codiversification". These have different meanings and need to be used carefully. The paper mostly deals with "codivergence" between coronaviruses and their host species.

      We now provide a list of definitions in Box S1. These definitions are as in our recent article clarifying the differences between these patterns/processes (Perez-Lamarque & Morlon 2024).

      Specific comments

      L83-L105: This paragraph can be written more concisely.

      We prefer to keep this paragraph like this as it contains key explanations that are necessary for understanding our approach and results.  

      Figure 1: The timescales of the trees are rather confusing. The different scales are indicated by the gray shading but this is easy to overlook. Maybe stretching or compressing the trees horizontally would help to emphasise the different timescales.

      Done.

      Figure 2: Note that the maximum clade credibility tree is a specific tree sampled from the posterior distribution - it is not a consensus tree. In the figure caption, the meaning of "location" is unclear.

      We have removed the word “consensus”, thank you for noting this. We have replaced “location” by “branching order”. 

      L461: How was the model chosen, and why were different models used in the BEAST and PhyloBayes analyses?

      We did our PhyloBayes analyses first and used the LG model following methodology outlined in previous studies using ALE (e.g. Groussin et al. 2017; Dorrell et al. 2021). Unfortunately, the LG model is not available in the default version of BEAST2 so we had to use a different model (the WAG model). We have now run BEAST2 with the LG model (thanks to the BEAST_CLASSIC package) and we obtained very similar results (see Figure below showing the BEAST consensus trees obtained with the WAG or LG models – they only slightly differ by the branching of the u7351 OTU). We have now added this information in the Methods section. 

      Author response image 1.

      L477: It is not clear to me how the PhyloBayes and BEAST analyses differ. Please expand the explanation of why PhyloBayes was used here.

      We have now clarified this (lines 594-597). 

      L568: Why not test explicitly for recombination?

      We did test for the occurrence of recombination using several approaches, including

      OpenRDP (https://github.com/PoonLab/OpenRDP), our own custom code, and Gubbins (Croucher et al. 2015). These tests were however inconclusive, indicating either the absence or presence of recombination, thus suggesting that the palmprint region is too short to infer anything about recombination. We thus do not exclude the possibility that recombination occurred, and test the robustness of our results to recombination by running our analyses on different sub-parts of the palmprint region. We have clarified this in our Material & Methods.

      L618: "DNA sequences" -> "RNA sequences"

      Done.

      The paper contains numerous minor grammatical errors and would benefit from careful proofreading and editing. Please check the use of plurals and apostrophes. Some of the errors are listed below:

      L49: "As several" -> "As with several"

      Done.

      L178: "reconciliates" -> "reconciles"?

      Done.

      L199: "extent" -> "extant"

      Done.

      L289: This sentence needs rephrasing to avoid a triple negative ("cannot ... reject ... not present")

      Done.

      L469: "temporary" -> "temporal"

      Done.

      L470: "neglectable" -> "negligible"

      Done.

      L577: "not only relying" -> "not relying only"

      Done.

      Reviewer #2 (Recommendations For The Authors):

      The study is generally well-constructed and its results are convincing. However, considering the availability of a dated host tree, conducting a dated reconciliation analysis could be beneficial. Creating a smaller sub-dataset and performing a dated reconciliation analysis would likely be a valuable addition to the research.

      We have now run the dated version of ALE on both the alpha and betacoronaviruses subclades. ALE dated still does not output reconciliations on the betacoronaviruses dataset, but it does on the smaller alphacoronaviruses dataset. We found significant reconciliations, indicating that mammal-alphacoronavirus associations are not random with respect to phylogeny, but the reconciliations involved more host switch and loss events (38 switches + 29 losses) than cospeciation events (65), indicating cophylogenetic signal in the absence of phylogenetic congruence (Perez-Lamarque & Morlon 2024). We now present the results on lines 264-282.  

      Reviewer #3 (Recommendations For The Authors):

      I think the results are written in a very speculative way, with many sentence fragments that should really be part of the discussion.

      We have carefully checked our Results section and rephrased or removed formulation that may have been perceived as speculative.  

      There are a lot of considerations in this manuscript about spread and future pandemics, but I think this is very far from the topic of this paper. When we quantified the coevolutionary risk of bats-betacovs in a recent paper (Forero et al. 2024, Virus Evol.), we only briefly touched upon this discussion because we compared our outputs with a measure of human population density. I don't think the manuscript needs to talk about epidemiology at all, and it would probably be more useful as a purely evo-bio piece.

      We think that it is useful to discuss the potential implications of our results for future pandemics, even though we agree that this discussion is rather speculative. We have removed the mention of predictions in the Abstract and have softened our wording in the Discussion.  

      References:

      Croucher, N.J., Page, A.J., Connor, T.R., Delaney, A.J., Keane, J.A., Bentley, S.D., et al. (2015). Rapid phylogenetic analysis of large samples of recombinant bacterial whole genome sequences using Gubbins. Nucleic Acids Res., 43, e15.

      Dorrell, R.G., Villain, A., Perez-Lamarque, B., Audren de Kerdrel, G., McCallum, G., Watson, A.K., et al. (2021). Phylogenomic fingerprinting of tempo and functions of horizontal gene transfer within ochrophytes. Proc. Natl. Acad. Sci., 118, e2009974118.

      Edgar, R.C. et al. (2022). Petabase-scale sequence alignment catalyses viral discovery. Nature 602, 142–147.

      Groussin, M., Mazel, F., Sanders, J.G., Smillie, C.S., Lavergne, S., Thuiller, W., et al. (2017).

      Unraveling the processes shaping mammalian gut microbiomes over evolutionary time. Nat. Commun., 8, 14319.

      Perez-Lamarque, B. & Morlon, H. (2024). Distinguishing cophylogenetic signal from phylogenetic congruence clarifies the interplay between evolutionary history and species interactions. Syst. Biol.

    1. Author response:

      The following is the authors’ response to the original reviews

      Reviewer 1 (Public Review):

      Thank you for the helpful comments. Below, we have quoted the relevant sections from the revised manuscript as we respond to the reviewer’s comments item-by-item.

      Weaknesses:

      While the task design in this study is intentionally stimulus-rich and places a minimal constraint on the animal to preserve naturalistic behavior, this is, unfortunately, a double-edged sword, as it also introduces additional variables that confound some of the neural analysis. Because of this, a general weakness of the study is a lack of clear interpretability of the task variable neural correlates. This is a limitation of the task, which includes many naturally correlated variables - however, I think with some additional analyses, the authors could strengthen some of their core arguments and significantly improve clarity.

      We acknowledge the weakness and have included additional analyses to compensate for it. The details are as follows in our reply to the subsequent comments.  

      For example, the authors argue, based on an ANN decoding analysis (Figure 2b), that PFC neurons encode spatial information - but the spatial coordinate that they decode (the distance to the active foraging zone) is itself confounded by the fact that animals exhibit different behavior in different sections of the arena. From the way the data are presented, it is difficult to tell whether the decoder performance reflects a true neural correlate of distance, or whether it is driven by behavior-associated activity that is evoked by different behaviors in different parts of the arena. The author's claim that PFC neurons encode spatial information could be substantiated with a more careful analysis of single-neuron responses to supplement the decoder analysis. For example, 1) They could show examples of single neurons that are active at some constant distance away from the foraging site, regardless of animal behavior, and 2) They could quantify how many neurons are significantly spatially modulated, controlling for correlates of behavior events. One possible approach to disambiguate this confound could be to use regression-based models of neuron spiking to quantify variance in neuron activity that is explained by spatial features, behavioral features, or both.

      First of all, we would like to point out that while the recording was made during naturalistic foraging with minimal constraints behaviorally, a well-trained rat displayed an almost fixed sequence of actions within each zone. The behavioral repertoire performed in each zone was very different from each other: exploratory behaviors in the N-zone, navigating back and forth in the F-zone, and licking sucrose while avoiding attacks in the E-zone. Therefore, the entire arena is not only divided by the geographical features but also by the distinct set of behaviors performed in each zone. This is evident in the data showing a higher decoding accuracy of spatial distance in the F-zone than in the N- or E-zone. In this sense, the heterogeneous encoding reflects heterogenous distribution of dominant behaviors (navigation in the F-zone and attack avoidance while foraging in the E-zone) and hence corroborate the reviewer’s comment at a macroscopic scale encompassing the entire arena.

      Having said that, the more critical question is whether the neural activity is more correlated with microscopic behaviors at every moment rather than the location decoded in the F-zone. As the reviewer suggested, the first-step is to analyze single-neuron activity to identify whether direct neural correlates of location exist. To this end, traditional place maps were constructed for individual neurons. Most neurons did not show cohesive place fields across different regions, indicating little-to-no direct place coding by individual neurons. Only a few neurons displayed recognizable place fields in a consistent manner. However, even these place fields were irregular and patchy, and therefore, nothing comparable to the place cells or grid cells found in the hippocampus or entorhinal cortex. Some examples firing maps have been added to Figure 2 and characterized in the text as below.

      “To determine whether location-specific neural activity exists at the single-cell level in our mPFC data, a traditional place map was constructed for individual neurons. Although most neurons did not show cohesive place fields across different regions in the arena, a few neurons modulated their firing rates based on the rat’s current location. However, even these neurons were not comparable to place cells in the hippocampus (O’Keefe & Dostrovsky, 1971) or grid cells in the entorhinal cortex (Hafting et al., 2005) as the place fields were patchy and irregular in some cases (Figure 2B; Units 66 and 125) or too large, spanning the entire zone rather than a discrete location within it (Units 26 and 56). The latter type of neuron has been identified in other studies (e.g., Kaefer et al., 2020).”

      Next, to verify whether the location decoding reflects neuronal activity due to external features or particular type of action, predicted location was compared between the opposite directions within the F-zone, inbound and outbound in reference to the goal area (Lobsterbot). If the encoding were specifically tied to a particular action or environmental stimuli, there should be a discrepancy when the ANN decoder trained with outbound trajectory is tested for predictions on the inbound path, and vice versa. However, the results showed no significant difference between the two trajectories, suggesting that the decoded distance was not simply reflecting neural responses to location-specific activities or environmental cues during navigation.

      “To determine whether the accuracy of the regressor varied depending on the direction of movement, we compared the decoding accuracy of the regressor for outbound (from the N- to E-zone) vs. inbound (from the E- to N- zone) navigation within the F-zone. There was no significant difference in decoding accuracy between outbound vs. inbound trips (paired t-test; t(39) = 1.52, p =.136), indicating that the stability of spatial encoding was maintained regardless of the moving direction or perceived context (Figure 2E).”

      Additionally, we applied the same regression analysis on a subset of data that were recorded while the door to the robot compartment was closed during the Lobsterbot sessions. This way, it is possible to test the decoding accuracy when the most salient spatial feature, the Lobsterbot, is blocked out of sight. The subset represents an average of 38.92% of the entire session. Interestingly, the decoding accuracy with the subset of data was higher accuracy than that with the entire dataset, indicating that the neural activities were not driven by a single salient landmark. This finding supports our conclusion that the location information can be decoded from a population of neurons rather than from individual neurons that are associated with environmental or proprioceptive cues. We have added the following description of results in the manuscript.

      “Previous analyses indicated that the distance regressor performed robustly regardless of movement direction, but there is a possibility that the decoder detects visual cues or behaviors specific to the E-zone. For example, neural activity related to Lobsterbot confrontation or licking behavior might be used by the regressor to decode distance. To rule out this possibility, we analyzed a subset of data collected when the compartment door was closed, preventing visual access to the Lobsterbot and sucrose port and limiting active foraging behavior. The regressor trained on this subset still decoded distance with a MAE of 12.14 (± 3.046) cm (paired t-test; t(39) = 12.17, p <.001). Notably, the regressor's performance was significantly higher with this subset than with the full dataset (paired t-test; t(39) = 9.895, p <.001).”

      As for the comment on “using regression-based models of neuron spiking to quantify variance in neuron activity that is explained by spatial features, behavioral features, or both”, it is difficult to separate a particular behavioral event let alone timestamping it since the rat’s location was being monitored in the constantly-moving, naturalistic stream of behaviors. However, as mentioned above, a new section entitled “Overlapping populations of mPFC neurons adaptively encode spatial information and defensive decision” argues against single-neuron based account by performing the feature importance analysis. The results showed that even when the top 20% of the most informative neurons were excluded, the remaining neural population could still decode both distance and events.  This analysis supports the idea of a population-wide mode shift rather than distinct subgroups of neurons specialized in processing different sensory or motor events. This idea is also expressed in the schematic diagrams featured in Figure 8 of the revision.

      To substantiate the claim that PFC neurons really switch between different coding "modes," the authors could include a version of this analysis where they have regressed out, or otherwise controlled for, these confounds. Otherwise, the claim that the authors have identified "distinctively different states of ensemble activity," as opposed to simple coding of salient task features, seems premature.

      A key argument in our study is that the mPFC neurons encode different abstract internal representations (distance and avoidance decision) at the level of population. This has been emphasized in the revision with additional analyses and discussions. Most of all, we performed single neuron-based analysis for both spatial encoding (place fields for individual neurons) and avoidance decision (PETHs for head entry and head withdrawal) and contrasted the results with the population analysis. Although some individual neurons displayed a fractured “place cell-like” activity, and some others showed modulated firing at the head-entry and the head-withdrawal events, the ensemble decoding extracted distance information for the current location of the animal at a much higher accuracy. Furthermore, the PCA analysis identified abstract feature dimensions especially regarding the activity in the E-zone that cannot be attributable to a small number of sensory- or motor-related neurons. 

      To mitigate the possibility that the PCA is driven primarily by a small subset of units responsive to salient behavioral events, we also applied PCA to the dataset excluding the activity in the 2-second time window surrounding the head entry and withdrawal. While this approach does not eliminate all cue- or behavior-related activity within the E-zone, it does remove the neural activity associated with emotionally significant events, such as entry into the E-zone, the first drop of sucrose, head withdrawal, and the attack. Even without these events, the PC identified in the E-zone was still separated from those in the F-zone and N-zone. This result again argues in support of distinct states of ensemble activity formed in accordance with different categories of behaviors performed in different zones. Finally, the Naïve Bayesian classifier trained with ensemble activity in the E-zone was able to predict the success and failure of avoidance that occur a few seconds later, indicating that the same population of neurons are encoding the avoidance decision rather than the location of the animal.

      Reviewer 1 (Recommendations):

      The authors include an analysis (Figure 4) of population responses using PCA on session-wide data, which they use to support the claim that PFC neurons encode distinctive neural states, particularly between the encounter zone and nesting/foraging zones. However, because the encounter zone contains unique stimulus and task events (sucrose, threat, etc.), and the samples for PCA are drawn from the entire dataset (including during these events), it seems likely that the Euclidean distance measures analyzed in Figure 4b are driven mostly by the neural correlates of these events rather than some more general change in "state" of PFC dynamics. This does not invalidate this analysis but renders it potentially redundant with the single neuron results shown in Figure 5 - and I think the interpretation of this as supporting a state transition in the coding scheme is somewhat misleading. The authors may consider performing a PCA/population vector analysis on the subset of timepoints that do not contain unique behavior events, rather than on session-wide data, or otherwise equalizing samples that correspond to behavioral events in different zones. Observing a difference in PC-projected population vectors drawn from samples that are not contaminated by unique encounter-related events would substantiate the idea that there is a general shift in neural activity that is more related to the change in context or goal state, and less directly to the distinguishing events themselves.

      Thank you for the comments. Indeed, this is a recurring theme where the reviewers expressed concerns and doubts about heterogenous encoding of different functional modes. Besides the systematic presentation of the results in the manuscript, from PETH to ANN and to Bayesian classifier, we argue, however, that the activity of the mPFC neurons is better represented by the population rather than loose collection of stimulus- or event-related neurons.

      The PCA results that we included as the evidence of distinct functional separation, might reflect activities driven by a small number of event-coding neurons in different zones. As mentioned in the public review, we conducted the same analysis on a subset of data that excluded neural activity potentially influenced by significant events in the E-zone. The critical times are defined as ± 1 second from these events and excluded from the neural data. Despite these exclusions, the results continued to show populational differences between zones, reinforcing the notion that neurons encode abstract behavioral states (decision to avoid or stay) without the sensory- or motor-related activity. Although this analysis does not completely eliminate all possible confounding factors emerging in different external and internal contexts, it provides extra support for the population-level switch occurring in different zones.

      In Figure 7, the authors include a schematic that suggests that the number of neurons representing spatial information increases in the foraging zone, and that they overlap substantially with neurons representing behaviors in the encounter zone, such as withdrawal. They show in Figure 3 that location decoding is better in the foraging zone, but I could not find any explicit analysis of single-neuron correlates of spatial information as suggested in the schematic. Is there a formal analysis that lends support to this idea? It would be simple, and informative, to include a quantification of the fraction of spatial- and behavior-modulated neurons in each zone to see if changes in location coding are really driven by "larger" population representations. Also, the authors could quantify the overlap between spatial- and behavior-modulated neurons in the encounter zone to explicitly test whether neurons "switch" their coding scheme.

      The Figure 7 (now Figure 8) is now completely revised. The schematic diagram is modified to show spatial and avoidance decision encoding by the overlapping population of mPFC neurons (Figure 8a). Most notably, there are very few neurons that encode location but not the avoidance decision or vice versa. This is indicated by the differently colored units in F-zone vs. E-zone. The model also included units that are “not” engaged in any type of encoding or engaged in only one-type of encoding although they are not the majority.

      We have also added a schematic for hypothetical switching mechanisms (Figure 8b) to describe the conceptual scheme for the initiation of encoding-mode switching (sensory-driven vs. arbitrator-driven process)

      “Two main hypotheses could explain this switch. A bottom-up hypothesis suggests sensory inputs or upstream signals dictate encoding priorities, while a top-down hypothesis proposes that an internal or external “arbitrator” selects the encoding mode and coordinates the relevant information (Figure 8B). Although the current study is only a first step toward finding the regulatory mechanism behind this switch, our control experiment, where rats reverted to a simple shuttling task, provide evidence that might favor the top-down hypothesis. The absence of the Lobsterbot degraded spatial encoding rather than enhancing it, indicating that simply reducing the task demand is not sufficient to activate one particular type of encoding mode over another.  The arbitrator hypothesis asserts that the mPFC neurons are called on to encode heterogenous information when the task demand is high and requires behavioral coordination beyond automatic, stimulus-driven execution. Future studies incorporating multiple simultaneous tasks and carefully controlling contextual variables could help determine whether these functional shifts are governed by top-down processes involving specific neural arbitrators or by bottom-up signals.”

      Related to this difference in location coding throughout the environment, the authors suggest in Figure 3a-b that location coding is better in the foraging zone compared to the nest or encounter zones, evidenced by better decoder performance (smaller error) in the foraging zone (Figure 3b). The authors use the same proportion of data from the three zones for setting up training/test sets for cross-validation, but it seems likely that overall, there are substantially more samples from the foraging zone compared to the other two zones, as the animal traverses this section frequently, and whenever it moves from the next into the encounter zone (based on the video). What does the actual heatmap of animal location look like? And, if the data are down-sampled such that each section contributes the same proportion of samples to decoder training, does the error landscape still show better performance in the foraging zone? It is important to disambiguate the effects of uneven sampling from true biological differences in neural activity.

      Thank you for the comment. We agree with the concern regarding uneven data size from different sections of the arena. Indeed, as the heatmap below indicates, the rats spent most of their time in two critical locations, one being a transition area between N-and F-zone and the other near the sucrose port. This imbalance needs to be corrected. In fact we have included methodology to correct this biased sampling. In the result section “Non-navigational behavior reduces the accuracy of decoded location” we have the following results.

      Author response image 1.

      Heatmap of the animal’s position during one example session. (Left) Unprocessed occupancy plot. Each dot represents 0.2 seconds. Right) Smoothed occupancy plot using a Gaussian filter (sigma: 10 pixels, filter size: 1001 pixels). The white line indicates a 10 cm length.

      “To correct for the unequal distribution of location visits (more visits to the F- than to other zones), the regressor was trained using a subset of the original data, which was equalized for the data size per distance range (see Materials and Methods). Despite the correction, there was a significant main effect of the zone (F(1.16, 45.43) = 119.2, p <.001) and the post hoc results showed that the MAEs in the N-zone (19.52 ± 4.46 cm; t(39) = 10.45; p <.001) and the E-zone (26.13 ± 7.57 cm; t(39) = 11.40; p <.001) had a significantly higher errors when compared to the F-zone (14.10 ± 1.64 cm).”

      Also in the method section, we have stated that:

      “In the dataset adjusted for uneven location visits, we divided distance values into five equally sized bins. Then, a sub-dataset was created that contains an equal number of data points for each of these bins.”

      Why do the authors choose to use a multi-layer neural network (Figure 2b-c) to decode the animal's distance to the encounter zone?(…) The authors may consider also showing an analysis using simple regression, or maybe something like an SVM, in addition to the ANN approach.

      We began with a simple linear regression model and progressed to more advanced methods, including SVM and multi-layer neural networks. As shown below, simpler methods could decode distance to some extent, but neural networks and random forest regressors outperformed others (Neural Network: 16.61 cm ± 3.673; Linear Regression: 19.85 cm ± 2.528; Quadratic Regression: 18.68 cm ± 4.674; SVM: 18.88 cm ± 2.676; Random Forest: 13.59 cm ± 3.174).

      We chose the neural network model for two main reasons: (1) previous studies demonstrated its superior performance compared to Bayesian regressors commonly used for decoding neural ensembles, and (2) its generalizability and robustness against noisy data. Although the random forest regressor achieved the lowest decoding error, we avoided using it due to its tendency to overfit and its limited generalization to unseen data.

      Overall, we expect similar results with other regressors but with different statistical power for decoding accuracy. Instead, we speculate that neural network’s use of multiple nodes contributes to robustness against noise from single-unit recordings and enables the network to capture distributed processing within neural ensembles.

      In Figure 6c, the authors show a prediction of withdrawal behavior based on neural activity seconds before the behavior occurs. This is potentially very interesting, as it suggests that something about the state of neural dynamics in PFC is potentially related to the propensity to withdraw, or to the preparation of this behavior. However, another possibility is that the behaves differently, in more subtle ways, while it is anticipating threat and preparing withdrawal behavior - since PFC neurons are correlated with behavior, this could explain decoder performance before the withdrawal behavior occurs. To rule out this possibility, it would be useful to analyze how well, and how early, withdrawal success can be decoded only on the basis of behavioral features from the video, and then to compare this with the time course of the neural decoder. Another approach might be to decode the behavior on the basis of video data as well as neural data, and using a model comparison, measure whether inclusion of neural features significantly increases decoder performance.

      We appreciate this important point, as mPFC activity might indeed reflect motor preparation preceding withdrawal behavior. Another reviewer raised a similar concern regarding potential micro-behavioral influences on mPFC activity prior to withdrawal responses. However, our behavioral analysis suggests that highly trained rats engage in sucrose licking which has little variability regardless of the subsequent behavioral decision. To support, 95% of inter-lick intervals were less than 0.25 seconds, which is not enough time to perform any additional behavior during encounters.

      Author response image 2.

      To further clarify this, we included additional video showing both avoidance and escape withdrawals at close range. This video was recorded during the development of the behavioral paradigm, though we did not routinely collect this view, as animals consistently exhibited stable licking behavior in the E-zone. As demonstrated in the video, the rat remains highly focused on the lick port with minimal body movement during encounters. Therefore, we believe that the neural ensemble dynamics observed in the mPFC are unlikely to be driven by micro-behavioral changes.

      Reviewer 2 (Public Review):

      Thank you for the positive comment on our behavior paradigm and constructive suggestions on additional analysis. We came to think that the role of mPFC could be better portrayed as representing and switching between different encoding targets under different contexts, which in part, was more clearly manifested by the naturalistic behavioral paradigm. In the revision we tried to convey this message more explicitly and provide a new perspective for this important aspect of mPFC function.

      It is not clear what proportion of each of the ensembles recorded is necessary for decoding distance from the threat, and whether it is these same neurons that directly 'switch' to responding to head entry or withdrawal in the encounter phase within the total population. The PCA gets closest to answering this question by demonstrating that activity during the encounter is different from activity in the nesting or foraging zones, but in principle this could be achieved by neurons or ensembles that did not encode spatial parameters. The population analyses are focused on neurons sensitive to behaviours relating to the threat encounter, but even before dividing into subtypes etc., this is at most half of the recorded population.

      In our study, the key idea we aim to convey is that mPFC neurons adapt their encoding schemes based on the context or functional needs of the ongoing task. Other reviewers also suggested strengthening the evidence that the same neurons directly switch between encoding two different tasks. The counteracting hypothesis to "switching functions within the same neurons" posits that there are dedicated subsets of neurons that modulate behavior—either by driving decisions/behaviors themselves or being driven by computations from other brain regions.

      To test this idea, we included an additional analysis chapter in the results section titled Overlapping populations of mPFC neurons adaptively encode spatial information and defensive decision. In this section, we directly tested this hypothesis by examining each neuron's contribution to the distance regressor and the event classifier. The results showed that the histogram of feature importance—the contribution to each task—is highly skewed towards zero for both decoders, and removing neurons with high feature importance does not impair the decoder’s performance. These findings suggest that 1) there is no direct division among neurons involved in the two tasks, and 2) information about spatial/defensive behavior is distributed across neurons.

      Furthermore, we tested whether there is a negative correlation between the feature importance of spatial encoding and avoidance encoding. Even if there were no “key neurons” that transmit a significant amount of information about either spatial or defensive behavior, it is still possible that neurons with higher information in the navigation context might carry less information in the active-foraging context, or vice versa. However, we did not observe such a trend, suggesting that mPFC neurons do not exhibit a preference for encoding one type of information over the other.

      Lastly, another reviewer raised the concern that the PCA results, which we used as evidence of functional separation of different ensemble functions, might be driven by a small number of event-coding neurons. To address this, we conducted the same analysis on a subset of data that excluded neural activity potentially influenced by significant events in the E-zone. In the Peri-Event Time Histogram (PETH) analysis, we observed that some neurons exhibit highly-modulated activity upon arrival at the E-zone (head entry; HE) and immediately following voluntary departure or attack (head withdrawal; HW). We defined 'critical event times' as ± one second from these events and excluded neural data from these periods to determine if PCA could still differentiate neural activities across zones. Despite these exclusions, the results continued to show populational differences between zones, reinforcing the notion that neurons adapt their activity according to the context. We acknowledge that this analysis still cannot eliminate all of the confounding factors due to the context change, but we confirmed that excluding two significant events (delivery onset of sucrose and withdrawal movement) does not alter our result.

      To summarize, these additional results further support the conclusion that spatial and avoidance information is distributed across the neural population rather than being handled by distinct subsets. The analyses revealed no negative correlation between spatial and avoidance encoding, and excluding event-driven neural activity did not alter the observed functional separation, confirming that mPFC neurons dynamically adjust their activity to meet contextual demands.

      A second concern is also illustrated by Fig. 7: in the data presented, separate reward and threat encoding neurons were not shown - in the current study design, it is not possible to dissociate reward and threat responses as the data without the threat present were only used to study spatial encoding integrity.

      Thank you for this valuable feedback. Other reviewers have also noted that Figure 7 (now Figure 8) is misleading and contains assertions not supported by our experiments. In response, we have revised the model to more accurately reflect our findings. We have eliminated the distinction between reward coding and threat coding neurons, simplifying it to focus on spatial encoding and avoidance encoding neurons. The updated figure will more appropriately align with our findings and claims. A. Distinct functional states (spatial vs. avoidance decision) encoded by the same population neurons are separable by the region (F- vs. E zone). B. Hypothetical control models by which mPFC neurons assume different functional states.

      Thirdly, the findings of this work are not mechanistic or functional but are purely correlational. For example, it is claimed that analyzing activity around the withdrawal period allows for ascertaining their functional contributions to decisions. But without a direct manipulation of this activity, it is difficult to make such a claim. The authors later discuss whether the elevated response of Type 2 neurons might simply represent fear or anxiety motivation or threat level, or whether they directly contribute to the decision-making process. As is implicit in the discussion, the current study cannot differentiate between these possibilities. However, the language used throughout does not reflect this. 

      We acknowledge that our experiments only involve correlational study and this serves as weakness. Although we carefully managed to select word to not to be deterministic, we agree that some of the language might mislead readers as if we found direct functional contribution. Thus, we changed expressions as below.

      “We then further analyzed the (functional contribution ->)correlation between neural activity and success and failure of avoidance behavior. If the mPFC neurons (encode ->)participate in the avoidance decisions, avoidance withdrawal (AW; withdrawal before the attack) and escape withdrawal (EW; withdrawal after the attack) may be distinguishable from decoded population activity even prior to motor execution.”

      Also, we added part below in discussion section to clarify the limitations of the study.

      “Despite this interesting conjecture, any analysis based on recording data is only correlational, mandating further studies with direct manipulation of the subpopulation to confirm its functional specificity.”

      Fourthly, the authors mention the representation of different functions in 'distinct spatiotemporal regions' but the bulk of the analyses, particularly in terms of response to the threat, do not compare recordings from PL and IL although - as the authors mention in the introduction - there is prior evidence of functional separation between these regions.

      Thank you for bringing this part to our attention. As we mentioned in the introduction, we acknowledge the functional differences between the PL and IL regions. Although differences in spatial encoding between these two areas were not deeply explored, we anticipated finding differences in event encoding, given the distinct roles of the PL and IL in fear and threat processing. However, our initial analysis revealed no significant differences in event encoding between the regions, and as a result, we did not emphasize these differences in the manuscript. To address this point, we have reanalyzed the data separately and included the following findings in the manuscript.

      “However, we did not observe a difference in decoding accuracy between the PL and IL ensembles, and there were no significant interactions between regressor type (shuffled vs. original) and regions (mixed-effects model; regions: p=.996; interaction: p=.782). These results indicate that the population activity in both the PL and IL contains spatial information (Figure 2D, Video 3).

      […]

      Furthermore, we analyzed whether there is a difference in prediction accuracy between sessions with different recorded regions, the PL and the IL. A repeated two-way ANOVA revealed no significant difference between recorded regions, nor any interaction (regions: F(1, 38) = 0.1828, p = 0.671; interaction: F(1, 38) = 0.1614, p = 0.690).

      […]

      We also examined whether there is a significant difference between the PL and IL in the proportion of Type 1 and Type 2 neurons. In the PL, among 379 recorded units, 143 units (37.73%) were labeled as Type 1, and 75 units (19.79%) were labeled as Type 2. In contrast, in the IL, 156 units (61.66%) and 19 units (7.51%) of 253 recorded units were labeled as Type 1 and Type 2, respectively. A Chi-square analysis revealed that the PL contains a significantly higher proportion of Type 2 neurons (χ²(1, 632) = 34.85, p < .001), while the IL contains a significantly higher proportion of Type 1 neurons compared to the other region (χ²(1, 632) = 18.07, p < .001).”

      To summarize our additional results, we did not observe performance differences in distance decoding or event decoding. The only difference we observed was the proportional variation of Type 1 and Type 2 neurons when we separated the analysis by brain region. These results are somewhat counterintuitive, considering the distinct roles of the two regions—particularly the PL in fear expression and the IL in extinction learning. However, since the studies mentioned in the introduction primarily used lesion and infusion methods, this discrepancy may be due to the different approach taken in this study. Considering this, we have added the following section to the discussion.

      “Interestingly, we found no difference between the PL and IL in the decoding accuracy of distance or avoidance decision. This somewhat surprising considering distinct roles of these regions in the long line of fear conditioning and extinction studies, where the PL has been linked to fear expression and the IL to fear extinction learning (Burgos-Robles et al., 2009; Dejean et al., 2016; Kim et al., 2013; Quirk et al., 2006; Sierra-Mercado et al., 2011; Vidal-Gonzalez et al., 2006). On the other hand, more Type 2 neurons were found in the PL and more Type 1 neurons were found in the IL. To recap, typical Type 1 neurons increased the activity briefly after the head entry and then remained inhibited, while Type 2 neurons showed a burst of activity during head entry and sustained increased activity. One study employing context-dependent fear discrimination task (Kim et al., 2013) also identified two distinct types of PL units: short-latency CS-responsive units, which increased firing during the initial 150 ms of tone presentation, and persistently firing units, which maintained firing for up to 30 seconds. Given the temporal dynamics of Type 2 neurons, it is possible that our unsupervised clustering method may have merged the two types of neurons found in Kim et al.’s study.

      While we did not observe decreased IL activity during dynamic foraging, prior studies have shown that IL excitability decreases after fear conditioning (Santini et al., 2008), and increased IL activity is necessary for fear extinction learning. In our paradigm, extinction learning was unlikely, as the threat persisted throughout the experiment. Future studies with direct manipulation of these subpopulations, particularly examining head withdrawal timing after such interventions, could provide insight into how these subpopulations guide behavior.”

      Additionally, we made some changes in the introduction, mainly replacing the PL/IL with mPFC to be consistent with the main body of results and conclusion and also specifying the correlational nature of the recording study.

      “Machine learning-based populational decoding methods, alongside single-cell analyses, were employed to investigate the correlations between neuronal activity and a range of behavioral indices across different sections within the foraging arena.”

      Reviewer 2 (Recommendations):

      The authors consistently use parametric statistical tests throughout the manuscript. Can they please provide evidence that they have checked whether the data are normally distributed? Otherwise, non-parametric alternatives are more appropriate.

      Thank you for mentioning this important issue in the analysis. We re-ran the test of normality for all our data using the Shapiro-Wilk test with a p-value of .05 and found that the following data sets require non-parametric tests, as summarized in Author response table 1 below. For those analyses which did not pass the normality test, we used a non-parametric alternative test instead. We also updated the methods section. For instance, repeated measures ANOVA for supplementary figure S1 and PCA results were changed to the Friedman test with Dunn’s multiple comparison test.

      Author response table 1.

      Line 107: it is not clear here or in the methods whether a single drop of sucrose solution is delivered per lick or at some rate during the encounter, both during the habituation or in the final task. This is important information in order to understand how animals might make decisions about whether to stay or leave and how to interpret neural responses during this time period. Or is it a large drop, such that it takes multiple licks to consume? Please clarify.

      The apparatus we used incorporated an IR-beam sensor-controlled solenoid valve. As the beam sensor was located right in front of the pipe, the rat’s tongue activated the sensor. As a result, each lick opened the valve for a brief period, releasing a small amount of liquid, and the rat had to continuously lick to gain access to the sucrose. We carefully regulated the flow of the liquid and installed a small sink connected to a vacuum pump, so any remaining sucrose not consumed by the rat was instantly removed from the port. We clarified how sucrose was delivered in the methods section and also in the results section.

      Method:

      “The sucrose port has an IR sensor which was activated by a single lick. The rat usually stays in front of the lick port and continuously lick up to a rate of 6.3 times per second to obtain sucrose. Any sucrose droplets dropped in the bottom sink were immediately removed by negative pressure so that the rat’s behavior was focused on the licking.”

      Result:

      “The lick port was activated by an IR-beam sensor, triggering the solenoid valve when the beam was interrupted. The rat gradually learned to obtain rewards by continuously licking the port.”

      However, I'm not sure I understand the authors' logic in the interpretation: does the S-phase not also consist of goal-directed behaviour? To me, the core difference is that one is mediated by threat and the other by reward. In addition, it would be helpful to visualize the behaviour in the S-phase, particularly the number of approaches. This difference in the amount of 'experience' so to speak might drive some of the decrease in spatial decoding accuracy, even if travel distance is similar (it is also not clear how travel distance is calculated - is this total distance?) Ideally, this would also be included as a predictor in the GLM.

      We agree that the behaviors observed during the shuttling phase can also be considered goal-directed, as the rat moves purposefully toward explicit goals (the sucrose port and the N-zone during the return trip). However, we argue that there is a significant difference in the level of complexity of these goals.

      During the L-phase, the rat not only has to successfully navigate to the E-zone for sucrose but also pay attention to the robots, either to avoid an attack from the robot's forehead or escape the fast-striking motion of the claw. When the rat runs toward the E-zone, it typically takes a side-approaching path, similar to Kim and Choi (2018), and exhibits defensive behaviors such as a stretched posture, which were not observed in the S-phase. This behavioral characteristic differs from the S-phase, where the rat adopted a highly stereotyped navigation pattern fairly quickly (within 3 sessions), evidenced by more than 50 shuttling trajectories per session. In this phase, the rat exhibited more stimulus-response behavior, simply repeating the same actions over time without deliberate optimization.

      In our additional experiment with two different levels of goal complexity (reward-only vs. reward/threat conflict), we used a between-subject design in which both groups experienced both the S-phase and L-phase before surgery and underwent only one type of session afterward. This approach ruled out the possibility of differences in contextual experience. Additionally, since we initially designed the S-phase as extended training, behaviors in the apparatus tended to stabilize after rats completed both the S-phase and L-phase before surgery. As a result, we compared the post-surgery Lobsterbot phase to the post-surgery shuttling phase to investigate how different levels of goal complexity shape spatial encoding strength.

      To clarify our claim, we edited the paragraph below.

      “This absence of spatial correlates may result from a lack of complex goal-oriented navigation behavior, which requires deliberate planning to acquire more rewards and avoid potential threats.

      […]

      After the surgery, unlike the Lob-Exp group, the Ctrl-Exp group returned to the shuttling phase, during which the Lobsterbot was removed. With this protocol, both groups experienced sessions with the Lobsterbot, but the Ctrl-Exp group's task became less complex, as it was reduced to mere reward collection.

      . Given these observations, along with the mPFC’s lack of consistency in spatial encoding, it is plausible that the mPFC operates in multiple functional modes, and the spatial encoding mode is preempted when the complexity of the task requires deliberate spatial navigation.”

      Additionally, we added behavior data during initial S-phase into Supplementary Figure 1.

      It is good point that the amount of experience might drive decrease in spatial decoding accuracy. To test this hypothesis, we added a new variable, the number of Lobsterbot sessions after surgery, to the previous GLM analysis. The updated model predicted the outcome variable with significant accuracy (F(4,44) = 10.31, p < .001), and with the R-squared value at 0.4838. The regression coefficients were as follows: presence of the Lobsterbot (2.76, standard error [SE] = 1.11, t = 2.42, p = .020), number of recorded cells (-0.43, SE = .08, t = -5.22, p < .001), recording location (0.90, SE = 1.11, p = .424), and number of L sessions (0.002, SE = 0.11, p = .981). These results indicate that the number of exposures to the Lobsterbot sessions, as a measure of experience, did not affect spatial decoding accuracy.

      For minor edit, we edited the term as “total travel distance”.

      Relating to the previous point, it should be emphasized in both sections on removing the Lobsterbot and on non-navigational behaviours that the spatial decoding is all in reference to distance from the threat (or reward location). The language in these sections differs from the previous section where 'distance from the goal' is mentioned. If the authors wish to discuss spatial decoding per se, it would be helpful to perform the same analysis but relative to the animals' own location which might have equal accuracy across locations in the arena. Otherwise, it is worth altering the language in e.g. line 258 onwards to state the fact that distance to the goal is only decodable when animals are actively engaged in the task.

      Thank you for this comment, we changed the term as “distance from the conflict zone” or “distance of the rat to the center of the E-zone” to clarify our experiment setup.

      In Fig. 5, why is the number of neurons shown in the PETHs less than the numbers shown in the pie charts?

      The difference in the number of neurons between the PETHs and the pie charts in Figure 5 is because PETHs are drawn only for 'event-responsive' units. For visualizing the neurons, we selectively included those that met certain criteria described in Method section (Behavior-responsive unit analysis). We have updated the caption for Figure 5 as follows to minimize confusion.

      “Multiple subpopulations in the mPFC react differently to head entry and head withdrawal.

      (A) Top: The PETH of head entry-responsive units is color-coded based on the Z-score of activity.

      (C) The PETH of head withdrawal-responsive units is color-coded based on the Z-score of activity.”

      I appreciate the amount of relatively unprocessed data plotted in Figure 5, but it would be great to visualize something similar for AW vs. EW responses within the HW2 population. In other words, what is there that's discernably different within these responses that results in the findings of Fig. 6?

      To visualize the difference in neural activity between AW and EW, we included an additional supplementary figure (Supplementary Figure 5). We divided the neurons into Type 1 and Type 2 and plotted PETH during Avoidance Withdrawal (AW) and Escape Withdrawal (EW). Consistent with the results shown in Figure 6d, we could visually observe increased activity in Type 2 neurons before the execution of AW compared to EW. However, we couldn’t find a similar pattern in Type 1 neurons.

      On a related note, it would add explanatory power if the authors were able to more tightly link the prediction accuracy of the ensemble (particularly the Type 2 neurons) to the timing of the behaviour. Earlier in the manuscript it would be helpful to show latency to withdraw in AW trials; are animals leaving many seconds before the attack happens, or are they just about anticipating the timing of the attack? And therefore when using ensemble activity to predict the success of the AW, is the degree to which this can be done in advance (as the authors say, up to 6 seconds before withdrawal) also related to how long the animal has been engaged with the threat?

      We agree that the timing of head withdrawal, particularly in AW trials, is a critical factor in describing the rat's strategy toward the task. To test whether the rat uses a precise timing strategy—for instance, leaving several seconds before the attack or exploiting the discrete 3- and 6-second attack durations—we plotted all head withdrawal timepoints during the 6-second trials. The distribution was more even, without distinguishable peaks (e.g., at the very initial period or at the 3- or 6-second mark). This indicates a lack of precise temporal strategy by the rat. We included additional data in the supplementary figure (Supplementary Figure 6) and added the following to the results section.

      “We monitored all head withdrawal timepoints to assess whether rats developed a temporal strategy to differentiate between the 3-second and 6-second attacks. We found no evidence of such a strategy, as the timings of premature head withdrawals during the 6-second attack trials were evenly distributed (see Supplementary Figure S1).”

      As depicted in the new supplementary figure, head withdrawal times during avoidance behavior vary from sub-seconds to the 3- or 6-second attack timepoints. After receiving the reviewer’s comment, we became curious whether there is a decoding accuracy difference depending on how long the animal engaged with the threat. We selected all 6-second attack and avoidance withdrawal trials and checked if correctly classified trials (AW trials classified as AW) had different head withdrawal times—perhaps shorter durations—compared to misclassified trials (AW trials classified as EW). As shown in Author response image 3 below, there was no significant difference between these two types, indicating that the latency of head withdrawal does not affect prediction accuracy.

      Author response image 3.

      Finally, there remain some open questions. One is how much encoding strength - of either space or the decision to leave during the encounter - relates to individual differences in animal performance or behaviour, particularly because this seems so variable at baseline. A second is how stable this encoding is. The authors mention that the distance encoding must be stable to an extent for their regressor to work; I am curious whether this stability is also found during the encounter coding, and also whether it is stable across experience. For example, in a session when an individual has a high proportion of anticipatory withdrawals, is the proportion of Type 2 neurons higher?

      Thank you for these questions. To recap the number of animals that we used, we used five rats during Lobsterbot experiments, and three rats for control experiment that we removed Lobsterbot after training. Indeed, there were individual differences in performance (i.e. avoidance success rate), number of recorded units (related to the recording quality), and baseline behaviors. To clarify these differences, see author response image 4 below.

      Author response image 4.

      We used a GLM to measure how much of the decoder’s accuracy was explained by individual differences. The result showed that 38.96% of distance regressor’s performance, and 12.14% of the event classifier’s performance was explained by the individual difference. Since recording quality was highly dependent on the animals, the high subject variability detected in the distance regression might be attributed to the number of recorded cells. Rat00 which had the lowest average mean absolute error had the highest number of recorded cells at average of 18. Compared to the distance regression, there was less subject variability in event classification. Indeed, the GLM results showed that the variability explained by the number of cells was only 0.62% in event classification.

      The reason we mentioned that "distance encoding must be stable for our regressor to work" is entirely based on the population-level analysis. Because we used neural data and behaviors from entire trials within a session, the regressor or classifier would have low accuracy if encoding dynamics changed within the session. In other words, if the way neurons encode avoidance/escape predictive patterns changed within a training set, the classifier would fail to generate an optimized separation function that works well across all datasets.

      To further investigate whether changes in experience affect event classification results over time, we plotted an additional graph below. Although there are individual and daily fluctuations in decoding accuracy, there was no observable trend throughout the experiments.

      Author response image 5.

      Regarding the correlation between the ratio of avoidance withdrawal and the proportion of Type 2 neurons, we were also curious and analyzed the data. Across 40 sessions, the correlation was -0.0716. For Type 1 neurons, it was slightly higher at 0.1459. We believe this indicates no significant relationship between the two variables.

      Minor points:

      I struggled with the overuse of acronyms in the paper. Some might be helpful but F-zone/N-zone, for example, or HE/HW, AW/EW are a bit of a struggle. After reading the paper a few times I learned them but a naive reader might need to often refer back to when they were first defined (as I frequently had to).

      To increase readability, we removed acronyms that are not often used and changed HE/HW to head-entry/head-withdrawal.

      I have a few questions about Figure 1F: in the text (line 150) it says that 'surgery was performed after three L sessions when the rats displayed a range of 30% to 60% AW'. This doesn't seem consistent with what is plotted, which shows greater variability in the proportion of AW behaviours both before and after surgery. It also appears that several rats only experienced two days of the L1 phase; please make clear if so. And finally, what is the line at 50% indicating? Neither the text nor the legend discuss any sort of thresholding at 50%. Instead, it would be best to make the distinction between pre- and post-surgery behaviour visually clearer.

      Thank you for pointing out this issue. We acknowledge there was an error in the text description. As noted in the Methods section, we proceeded with surgery after three Lobsterbot sessions. We have removed the incorrect part from the Results section and revised the Methods section for clarity.

      “After three days of Lobsterbot sessions, the rats underwent microdrive implant surgery, and recording data were collected from subsequent sessions, either Lobsterbot or shuttling sessions, depending on the experiment. For all post-surgery sessions, those with fewer than 20 approaches in 30 minutes were excluded from further analysis.”

      Among the five rats, Rat2 and Rat3 did not approach the robot during the entire Lob2 session, which is why these two rats do not have Lob2 data points. We updated the caption for regarding issue.

      Initially, we added a 50% reference line, but we agree it is unnecessary as we do not discuss this reference. We have updated the figure to include the surgery point, as shown in Supplementary Figure 1.

      Fig. 2C: each dot is an ensemble of simultaneously recorded neurons, i.e. a subset of the total 800-odd units if I understand correctly. How many ensembles does each rat contribute? Similarly, is this evenly distributed across PL and IL?

      Yes, each dot represents a single session, with a total of 40 sessions. Five rats contributed 11, 9, 8, 7, and 5 sessions, respectively. Although each rat initially had more than 10 sessions, we discarded some sessions with a low unit count (fewer than 10 sessions; as detailed in Materials and Methods - Data Collection). We collected 25 sessions from the PL and 15 sessions from the IL. Our goal was to collect more than 200 units per each region.

      Please show individual data points for Fig. 2D.

      We update the figure with individual data points.

      Is there a reason why the section on removing the Lobsterbot (lines 200 - 215) does not have associated MAE plots? Particularly the critical comparison between Lob-Exp and Ctl-Exp.

      We intentionally removed some graphs to create a more compact figure, but we appreciate your suggestion and have included the graph in Figure 2.

      Some references to supplementary materials are not working, e.g. line 333.

      Our submitted version of manuscript had reference error. For the current version, we used plane text, and the references are fixed.

      The legend for Supp. Fig. 2B is incorrect.

      We greatly appreciate this point. We changed the caption to match the figure.

      Reviewer 3 (Public Review):

      Thank you for recognizing our efforts in designing an ethologically relevant foraging task to uncover the multiple roles of the mPFC. While we acknowledge certain limitations in our methodology—particularly that we only observed correlations between neural activity and behavior without direct manipulation—we have conducted additional analyses to further strengthen our findings.

      Weakness:

      The primary concern with this study is the absence of direct evidence regarding the role of the mPFC in the foraging behavior of the rats. The ability to predict heterogeneous variables from the population activity of a specific brain area does not necessarily imply that this brain area is computing or using this information. In light of recent reports revealing the distributed nature of neural coding, conducting direct causal experiments would be essential to draw conclusions about the role of the mPFC in spatial encoding and/or threat evaluation. Alternatively, a comparison with the activity from a different brain region could provide valuable insights (or at the very least, a comparison between PL and IL within the mPFC).

      Thank you for the comment. Indeed, the fundamental limitation of the recording study is that it is only correlational, and any causal relationship between neural activity and behavioral indices is only speculative. We made it clearer in the revision and refrained from expressing any speculative ideas suggesting causality throughout the revision. While we did not provide direct evidence that the mPFC is computing or utilizing spatial/foraging information, we based our assertion on previous studies that have directly demonstrated the mPFC's role in complex decision-making tasks (Martin-Fernandez et al., 2023; Orsini et al., 2018; Zeeb et al., 2015) and in certain types of spatial tasks (De Bruin et al., 1994; Sapiurka et al., 2016) . We would like to emphasize that, to the best of our knowledge, there was no previous study which investigated the mPFC function while animal is solving multiple heterogenous problems in semi-naturalistic environment. Therefore, although our recording study only provides speculative causal inference, it certainly provides a foundation for investigating the mPFC function. Future study employing more sophisticated, cell-type specific manipulations would confirm the hypotheses from the current study.

      One of the key questions of this manuscript is how multiple pieces of information are represented in the recorded population of neurons. Most of the studies mentioned above use highly structured experimental designs, which allow researchers to study only one function of the mPFC. In the current study, the semi-naturalistic environment allows rats to freely switch between multiple behavioral sets, and our decoding analysis quantitatively assesses the extent to which spatial/foraging information is embedded during these sets. Our goal is to demonstrate that two different task hyperspaces are co-expressed in the same region and that the degree of this expression varies according to the rat’s current behavior (See Figure 8(b) in the revised manuscript).

      Alternatively, we added multiple analyses. First, we included a single unit-level analysis looking at the place cell-like property to contrast with the ensemble decoding. Most neurons did not show well-defined place fields although there were some indications for place cell-like property. For example, some neurons displayed fragmented place fields or unusually large place fields only at particular spots in the arena (mostly around the gates). The accuracy from this place information at the single-neuron level is much lower than that acquired from population decoding. Likewise, although there were neurons with modulated firing around the time of particular behavior (head entry and withdrawal), overall prediction accuracy of avoidance decision was much higher when the ensemble-based classifier was applied.

      Moreover, given that high-dimensional movement has been shown to be reflected in the neural activity across the entire dorsal cortex, more thorough comparisons between the neural encoding of task variables and movement would help rule out the possibility that the heterogeneous encoding observed in the mPFC is merely a reflection of the rats' movements in different behavioral modes.

      Thanks for the comment. We acknowledge that the neural activity may reflect various movement components across different zones in the arena. We performed several analyses to test this idea. First, we want to recap our run-and-stop event analysis may provide an insight regarding whether the mPFC neurons are encoding locations despite the significant motor events. The rats typically move across the F-zone fairly routinely and swiftly (as if they are “running”) to reach the E-zone at which they reduce the moving speed to almost a halt (“stopping”). The PETHs around these critical motor events, however, did not show any significant modulation of neural activity indicating that most neurons we recorded from mPFC did not respond to movement.

      We added this analysis to demonstrate that these sudden stops did not evoke the characteristic activation of Type 1 and Type 2 neurons observed during head entry into the E-zone. When we isolated these sudden stops outside the E-zone, we did not observe this neural signature (Supplementary Figure 2).

      Second, our PCA results showed that population activity in the E-zone during dynamic foraging behavior was distinct from the activity observed in the N- and F-zones during navigation. However, there is a possibility that the two behaviorally significant events—entry into the E-zone and voluntary or sudden exit—might be driving the differences observed in the PCA results. To account for this, we designated ±1 second from head entry and head withdrawal as "critical event times," excluded the corresponding neural data, and reanalyzed the data. This method removed neural activity associated with sudden movements in specific zones. Despite this exclusion, the PCA still revealed distinct population activity in the E-zone, different from the other zones (Supplementary Figure 4). This result reduces the likelihood that the observed heterogeneous neural activity is merely a reflection of zone-specific movements.

      Lastly, the main claim of the paper is that the mPFC population switches between different functional modes depending on the context. However, no dynamic analysis or switching model has been employed to directly support this hypothesis.

      Thank you for this comment. Since we did not conduct a manipulation experiment, there is a clear limitation in uncovering how switching occurs between the two task contexts. To make the most of our population recording data, we added an additional results section that examines how individual neurons contribute to both the distance regressor and the event classifier. Our findings support the idea that distance and dynamic foraging information are distributed across neurons, with no distinct subpopulations dedicated to each context. This suggests that mPFC neurons adjust their coding schemes based on the current task context, aligning with Duncan’s (2001) adaptive coding model, which posits that mPFC neurons adapt their coding to meet the task's current demands.

      Reviewer 3 (Recommendations):

      The evidence for spatial encoding is relatively weak. In the F-zone (50 x 48 cm), the average error was approximately 17 cm, constituting about a third of the box's width and likely not significantly smaller than the size of a rat's body. The errors in the shuffled data are also not substantially greater than those in the original data. An essential test indicates that spatial decoding accuracy decreases when the Losterbot is removed. However, assessing the validity of the results is difficult in the current state. There is no figure illustrating the results, and no statistics are provided regarding the test for matching the number of neurons.

      We acknowledge that the average error (~ 17 cm ) measured in our study is relatively large, even though the error is significantly smaller than that by the shuffled control model (22.6 cm). Previous studies reported smaller prediction errors but in different experimental conditions: 16 cm in Kaefer et al. (2020) and less than 10 cm in Ma et al. (2023) and Mashhoori et al. (2018). Most notably, the average number of units used in our study (15.8 units per session) is significantly smaller compared to the previous works, which used 63, 49, and 40 units, respectively. As our GLM results demonstrated, the number of recorded cells significantly influenced decoding accuracy (β = -0.43 cm/neuron). With a similar number of recorded cells, we would have achieved comparable decoding accuracy. In addition, unlike other studies that have employed a dedicated maze such as the virtual track or the 8-shaped maze, we exposed rats to a semi-naturalistic environment where they exhibited a variety of behaviors beyond simple navigation. As argued throughout the manuscript, we believe that the spatial information represented in the mPFC is susceptible to disruption when the animal engages in other activities. A similar phenomenon was reported by Mashhoori et al. (2018), where the decoder, which typically showed a median error of less than 10 cm, exhibited a much higher error—nearly 100 cm—near the feeder location.

      As for the reviewer’s request for comparing spatial decoding without the Lobsterbot, we added a new figure to illustrate the spatial decoding results, including statistical details. We also applied a Generalized Linear Model to regress out the effect of the number of recorded neurons and statistically assess the impact of Lobsterbot removal. This adjustment directly addresses the reviewer's request for a clearer presentation of the results and helps contextualize the decoding performance in relation to the number of recorded neurons.

      As indicated in the public review, drawing conclusions about the role of the mPFC in navigation and avoidance behavior during the foraging task is challenging due to the exclusively correlational nature of the results. The accuracy in AW/EW discrimination increases a few seconds before the response, implying that changes in mPFC activity precede the avoidance/escape response. However, one must question whether this truly reflects the case. Could this phenomenon be attributed to rats modifying their "micro-behavior" (as evidenced by changes in movement observed in the video) before executing the escape response, and subsequently influencing mPFC activity?

      We appreciate the reviewer's thoughtful observation regarding the correlational nature of our results and the potential influence of pre-escape micro-behaviors on mPFC activity. We acknowledge that the increased accuracy in AW/EW discrimination preceding the response could also be correlated with micro-behaviors. However, there is very little room for extraneous behavior other than licking the sucrose delivery port within the E-zone, as the rats are highly trained to perform this stereotypical behavior. To support this, we measured the time delays between licking events (inter-lick intervals). The results show a sharp distribution, with 95% of the intervals falling within a quarter second, indicating that the rats were stable in the E-zone, consistently licking without altering their posture.

      To complement the data presented in Author response image 2, a video clip showing a rat engaged in licking behavior was included. We carefully designed the robot compartment and adjusted the distance between the Lobsterbot and the sucrose port to ensure that rats could exhibit only limited behaviors inside the E-zone. The video confirms that no significant micro-behaviors were observed during the rat’s activity in the E-zone.

      If mPFC activity indeed switches mode, the results do not clearly indicate whether individual cells are specifically dedicated to spatial representation and avoidance or if they adapt their function based on the current goal. Figure 7, presented as a schematic illustration, suggests the latter option. However, the proportion of cells in the HE and HW categories that also encode spatial location has not been demonstrated. It has also not been shown how the switch is manifested at the level of the population.

      Thank you for this comment. As the reviewer pointed out, we suggest that mPFC neurons do not diverge based on their functions, but rather adapt their roles according to the current goal. To support this assertion, we added an additional results section that calculates the feature importance of decoders. This analysis allows us to quantitatively measure each neuron’s contribution to both the distance regressor and the event decoder. Our results indicate that distance and defensive behavior are not encoded by a small subset of neurons; instead, the information is distributed across the population. Shuffling the neural data of a single neuron resulted in a median increase in decoding error of 0.73 cm for the distance regressor and 0.01% for the event decoder, demonstrating that the decoders do not rely on a specific subset of neurons that exclusively encode spatial and/or defensive behavior

      Although we found supporting evidence that mPFC neurons encode two different types of information depending on the current context, we acknowledge that we could not go further in answering how this switch is manifested. One simple explanation is that the function is driven by current contextual information and goals—in other words, a bottom-up mechanism. However, in our control experiment, simplifying the navigation task worsened the encoding of spatial information in the mPFC. Therefore, we speculate that an external or internal arbitrator circuit determines what information to encode. A precise temporal analysis of the timepoint when the switch occurs in more controlled experiments might answer these questions. We have added this discussion to the discussion section.

      PL and IL are two distinct regions; however, there is no comparison between the two areas regarding their functional properties or the representations of the cells. Are the proportions of cell categories (HE vs HW or HE1 vs HE2, spatial encoding vs no spatial encoding) different in IL and PL? Are areas differentially active during the different behaviors?

      Thank you for bringing up this issue. As mentioned in our response to the public review, we included a comparison between the PL and IL regions. While we did not observe any differences in spatial encoding (feature importance scores), the only distinction was in the proportion of Type 1 and Type 2 neurons, as the reviewer suggested. We have incorporated our interpretation of these results into the discussion section.

      The results and interpretations of the cluster analysis appear to be highly dependent on the parameters used to define a cluster. For example, the HE2 category includes cells with activity that precedes events and gradually decreases afterward, as well as cells with activity that only follows the events.

      We strongly agree that dependency on hyperparameters is a crucial point when using unsupervised clustering methods. To eliminate any subjective criteria in defining clusters, we carefully selected our clustering approach, which requires only two hyperparameters: the number of initial clusters (set to 8) and the minimum number of cells required to be considered a valid cluster (cutoff limit, 50). The rationale behind these choices was: 1) a higher number of initial clusters would fail to generalize neural activity, 2) clusters with fewer than 50 neurons would be difficult to analyze, and 3) to prevent the separation of clusters that show noisy responses to the event.

      Author response table 2 shows the differences in the number of cell clusters when we varied these two parameters. As demonstrated, changing these two variables does result in different numbers of clusters. However, when we plotted each cluster type’s activity around head entry (HE) and head withdrawal (HW), an increased number of clusters resulted in the addition of small subsets with low variation in activity around the event, without affecting the general activity patterns of the major clusters.

      The example mentioned by the reviewer—possible separation of HE2—appears when using a hyperparameter set those results in 4 clusters, not 3. In this result, 83 units, which were labeled as HE2 in the 3-cluster hyperparameter set, form a new group, HE3 (Group 3). This group of units shows increased activity after head entry and exhibited characteristics similar to HE2, with most of the units classified as HW2, maintaining high activity until head withdrawal. Among the 83 HE3 units, 36 were further classified as HW2, 44 as non-significant, and 3 as HW1. Therefore, we believe this does not affect our analysis, as we observed the separation of two major groups, Type 1 (HE1-HW1) and Type 2 (HE2-HW2), and focused our analysis on these groups afterward.

      Despite this validation, there remains a strong possibility that our method might not fully capture small yet significant subpopulations of mPFC units. As a result, we have included a sentence in the methods section addressing the rationale and stability of our approach.

      “(Materials and Methods) To compensate for the limited number of neurons recorded per session, the hyperparameter set was chosen to generalize their activity and categorize them into major types, allowing us to focus on neurons that appeared across multiple recording sessions. Although changes in the hyperparameter sets resulted in different numbers of clusters, the major activity types remained consistent (Supplementary Figure S8). However, there is a chance that this method may not differentiate smaller subsets of neurons, particularly those with fewer than 50 recorded neurons.”

      Author response table 2.

      Minor points:

      Line 333: Error! Reference source not found. This was probably the place for citing Figure S2?

      Lines 339, 343: Error! Reference source not found.

      Thank you for mentioning these comments. In the new version, all reference functions from Word have been replaced with plain text.

    1. Author Response

      The following is the authors’ response to the original reviews.

      Public Reviews:

      Reviewer #1 (Public Review):

      Summary:

      This is a very well written and performed study describing a TOPBP1 separation of function mutation, resulting in defective MSCI maintenance but normal sex body formation. The phenotype differs from that of a previous TOPBP1 null allele, in which both MSCI and sex body formation were defective. Additional defects in CHK phosphorylation and SETX localization are also described.

      Strengths:

      The study is very rigorous, with a remarkably large number of MSCI marks assayed, phosphoproteomics (leading to the interesting SETX discovery) and 10X RNAseq, allowing the MSCI phenotype to be further deconvolved. The approaches in most cases are robust.

      Weaknesses:

      There aren't many; please find list below:

      1) The authors are committed to the idea that maintenance of MSCI is the major defect here. However, based on the data, an alternative would be that some cells achieve sex body formation and MSCI normally, while others do not. It would only take a small percentage of cells exhibiting MSCI failure to kill all the cells in the same germinal epithelium, so this could still explain the complete pachytene block. This isn't a major point...this phenotype is clearly different to the TOPBP1 KO, but a broader discussion of possibilities in the discussion would help. I raise this in the context of both the cytology and 10X analysis:

      a) The assessment that sex body formation is normal is based on cytology in Supp 8 and 9, but a more rigorous approach would be to assess condensation of the XY pair in stage-matched spread cells (maybe they have that data already) by measuring distances between the X and Y centromere, or looking at stage IV of the seminiferous cycle, where all cells should have oval sex bodies but sex body mutants have persistent elongated XY pairs (see work of Namekawa and Turner). The authors do actually mention that gH2AX spreading is defective in many cells....and if this is true, condensation to form a sex body would almost certainly not have taken place in those cells.

      We appreciate the reviewer’s comment and have performed the experiment suggested, counting the number of elongated sex bodies in all sex body-positive cells in seminiferous tubules stained with γH2AX and DAPI (as done by Turner in Hirota et al., 2018). The experiment did not show significant differences between Topbp1+/+ and Topbp1B5/B5 as shown in Author response image 1.

      Author response image 1.

      Topbp1B5/B5 displays normal condensation of the XY-pair. A) Immunostaining of XY condensation in Topbp1+/+ and Topbp1B5/B5 testes sections (γH2AX: green and DAPI: gray). B) Quantification of all sex body-positive cells per tubule (Topbp1+/+ number of cells counted = 781, number of tubules counted = 28, number of mice = 3; Topbp1B5/B5 number of cells counted = 967, number of tubules counted = 28, number of mice = 3). C) Quantification of elongated-sex body cells per tubule (Topbp1+/+ number of cells counted = 19 and 762 normal round/oval-sex bodies cells, number of tubules counted = 28, number of mice = 3; Topbp1B5/B5 number of cells counted = 45 and 922 normal round/oval-sex bodies cells, number of tubules counted = 28, number of mice = 3).

      b) Regarding the 10X data, the finding that expression of some XY genes is elevated and others are not is also consistent with a "partial" phenotype (some cells have normal XY bodies and MSCI, others fail in both). In Fig 6E, X expression looks to be elevated in B5 vs wt at all stages...if this were a maintenance issue, shouldn't it be equal to that in wt and then elevate later?

      We understand the point raised by the reviewer, however we do not favor the “partial” phenotype model because of the absence of any post-pachytene spermatocytes in the B5 mutant. If some cells had escaped the MSCI defect, we would expect to detect cells progressing further in meiosis. Because we cannot rule out completely the possibility of a subtle disruption in XY silencing initiation, we decided to better emphasize this point in the discussion (lines 391-394).

      In Figure 6E, the X-linked genes were normalized against chromosome 9-linked genes. The normalization against pre-leptotene was done for the results displayed on Figure 7, in which we demonstrate the maintenance issue. Furthermore, for the 10X analysis, while the same number of cells were loaded for wild-type and mutant, the composition of cells varied between these two samples. Despite the fact that very few “spermatocyte 3” cells were detected in the mutant, those cells displayed much higher X-linked gene expression than the wild-type spermatocyte 3 cells.

      2) How is the quantitation showing impaired localization of select markers (e.g. SETX) normalized? How do we know that the antibody staining simply didn't work as well on the mutant slides?

      The quantification showing impaired localization of the selected markers such as SETX was done as described by Sims, et al. 2022 and Adams, et al. 2018. In brief, the green signal was measured along (XY cores) or across (XY DNA loops) the X and Y chromosomes and normalized against the analogous signal on the autosomal chromosomes. The possibility that the antibody simply did not work as well on the mutant is unlikely since multiple biological replicates were performed and we reproducibly followed standard practices in the field for meiotic spreads staining, imaging, and quantification. We also note that our findings published in Sims et al, 2022 show that ATR inhibition strongly impairs SETX localization to the sex body, further substantiating our claim that signaling via ATR-TOPBP1 controls SETX.

      3) Is testis TOPBP1 protein expression reduced in the B5 mutant?

      TOPBP1 protein abundance in the B5 mutant is reduced in lysates from whole testis, measured via western blot. We did not detect a significant reduction in TOPBP1 signal intensity measured by immunofluorescence in pachytene spreads of the B5 mutant.

      4) 10X analysis: how were the genes on the y-axis in Supp 24 arranged? Is this by location on the X chromosome?

      These genes were sorted by location across the chromosome X.

      5) The final analyses in Fig 7: X-genes are subdivided based on their behavior (up, down, unchanged). What isn't clear to me is whether the authors have considered the fact that there are global changes in gene expression during meiosis (very low in lep , zyg and early pach, then ramps up hugely from mid pach). In other words, is this normalized to autosomal gene expression?

      For the final analysis in Fig7, the normalization was done by their expression at the pre-leptotene stage. Moreover, the analysis was made comparing X-linked gene behavior in Wild-type vs B5 mutant.

      6) Again regarding the 10X analysis, my prediction would be that not ALL X and Y gene would increase in pach if MSCI were ablated...we should remember that XY genes have been subject to MSCI for some 160 million years of evolution, and this will mean that many enhancers that originally drove their expression prior to the evolution of MSCI will now be lost. This has been our experience: many XY genes aren't elevated at pach even in mutants in which MSCI is totally defective. I'd urge the authors to consider this possibility when they use XY gene expression patterns to diagnose the severity or timing of the MSCI phenotype. This could be a discussion point.

      We greatly appreciate the reviewer’s suggestion and have added discussion about this point to lines 392400).

      Reviewer #2 (Public Review):

      Summary:

      This paper described the role of BRCT repeat 5 in TOPBP1, a DNA damage response protein, in the maintenance of meiotic sex chromosome inactivation (MSCI). By analyzing a Topbp1 mutant mouse with amino acid substitutions in BRCT repeat 5, the authors found reduced phosphorylation of a DNA/RNA helicase, Sentaxin, and decreased localization of the protein to the X-Y sex body in pachynema. Moreover, the authors also found decreased repression of several genes on the sex chromosomes in the male mice.

      Strengths:

      The works including phospho-proteomics and single-cell RNA sequencing with lots of data have been done with great care and most of the results are convincing.

      Weaknesses:

      One concern is that, although the Topbp1 mutant spermatocytes show very severe defects after the stage of late pachynema, the defect in the gene silencing in the sex body is relatively weak. It is a bit difficult to explain how such a weak mis regulation of the gene silencing in mice causes the complete loss of cells in the late stage of spermatogenesis.

      We appreciate the reviewer’s comment. We note that even subtle mis-regulation of XY gene silencing has been reported to lead to significant loss of cells in late stage of prophase I (Ichijima et al., 2011; Modzelewski et al., 2012). Moreover, it is possible that some cells with drastic changes in X-gene expression were excluded from the downstream analysis due to high levels of mitochondrial gene expression (cells that were likely dying due to apoptosis). The exclusion of cells with high levels of mitochondrial gene expression is a common practice in downstream analysis of sc-RNA sequencing data.

      Reviewer #3 (Public Review):

      The work presented by Ascencao and coworkers aims to deepen into the process of sex chromosome inactivation during meiosis (MSCI) as a critical factor in the regulation of meiosis progression in male mammals. For this purpose, they have generated a transgenic mouse model in which a specific domain of TOPBP1 protein has been mutated, hampering the binding of a number of protein partners and interfering with the regulatory cascade initiated by ATR. Through the use of immunolocalization of an impressive number of markers of MSCI, phosphoproteomics and single cell RNA sequencing (scRNAseq), the authors are able to show that despite a proper morphological formation of the sex body and the incorporation of most canonical MSCI makers, sex chromosome-liked genes are reactivated at some point during pachytene and this triggers meiosis progression breakdown, likely due to a defective phosphorylation of the helicase SETX.

      The manuscript presents a clear advance in the understanding of MSCI and meiosis progression with two main strengths. First, the generation of a mouse model with a very uncommon phenotype. Second, the use of a vast methodological approach. The results are well presented and illustrated. Nevertheless, the discussion could be still a bit tuned by the inclusion of some ideas, and perhaps speculations, that have not been considered.

      We appreciate the reviewer’s comment and have improved the discussion section addressing the points raised in the “recommendation For the Authors”.

      Reviewer #1 (Recommendations For The Authors):

      I don't have any additional points here

      Reviewer #2 (Recommendations For The Authors):

      The paper by Ascencao et al. describes a separation-in-function allele of TOPBP1 critical for DNA damage response (DDR) that confers a specific defect in XY sex chromosome inactivation during male mouse meiosis. The authors constructed a Topbp1 separation-of-function mouse by introducing amino acid substitutions in BRCT repeat 5 and found the mice with normal DDR response in mitosis and meiosis show male infertility. Topbp1(B5/B5) mice do not contain spermatocytes after diplonema, as a result, little spermatids/sperms. In the mice, most of the meiotic events in prophase I including chromosome synapsis and meiotic recombination as well as the formation of the sex body are normal. The detailed proteomic analysis revealed the reduced ATR-dependent phosphorylation of a DNA/RNA helicase, Sentaxin. And also single-cell RNA sequencing found that the expression of some of genes from sex chromosomes are not silenced well compared to the control. The works with lots of data have been done with great care and most of the results are convincing. One clear concern is that, although the authors nicely showed a defect in gene silencing in sex chromosomes in the Topbp1(B5/B5) mice, how a small defect in the gene silencing leads to the complete loss of diplotene spermatocytes remains unaddressed.

      Major points:

      Although the authors showed a change in the transcriptome in spermatocytes of Topbp1(B5/B5) male mice, the authors cannot explain the complete lack of spermatids in this mouse. Even the transcriptome seems not to provide a clue.

      1) Given that the TOPBP1-B5 protein cannot bind to both 53BP1 and BLM, it is interesting to check the localization of both proteins on meiotic chromosome spreads (in the case of 53BP1, the localization in MEFs with DNA damage).

      We appreciate the reviewer’s comment. We have tried to stain BLM in meiotic spreads using several different antibodies, however we were not successful getting specific signals for BLM. In the case of 53BP1, we monitored its localization, and it was not significantly different from Topbp1-/- meiotic spreads, please refer to Supplemental Figure 11. While we appreciate the reviewer’s suggestion of looking at the localization of 53BP1 in MEFs with DNA damage, we opted not to perform the experiment because we have shown that 53BP1 can still bind the BRCT 1 and 2 domains of TOPBP1 as previously described (Bigot et al., 2019; Cescutti et al., 2010; Liu et al., 2017). Additionally, both male and female 53BP1 KO mice are fertile (Ward et al., 2003), thus the partial disruption in binding to 53BP1 that we observed in TOPBP1 B5 mutant is likely not causing the infertility phenotype.

      2) A recent preprint by Fujiwara et al. (doi: https://doi.org/10.1101/2023.04.12.536672) showed the accumulation of R-loops in spermatocyte spreads in Senataxin knockout mice. The authors may check the R-loop on the sex body in Topbp1-B5 mice.

      We thank the reviewer for the suggestion. We have tried several protocols to stain R-loops (including the protocol used in the paper mentioned above) but were not successful.

      3) The authors need to check the protein level (and band shift) of Senataxin in the testis by western blotting analysis.

      We have tried several SETX antibodies, and none worked for western blot analysis.

      4) If possible, the authors can see any protein interaction between TOPBP1 and Senataxin.

      We appreciate the suggestion, and we will investigate this interaction in future work.

      5) The authors need to check the statistics in the paper.

      (1) It is better to show actual P-values in the case of "ns".

      P-values were added to the respective figure legends.

      (2) In focus counting such as Figures 3D, G, H, 4B, D, F, H, 5E, and F (and in Supplemental Figures), please indicate how many spreads were counted in each mouse. Moreover, the distribution of focus numbers and intensity of fluorescence are not parametric (not normal distribution). It is better to use a non-parametric method such as Mann-Whitney's U test.

      We appreciate the reviewer's comment and upon consulting with a Statistician at Cornell Statistical Consulting Unit (CSCU), we were advised to use a linear mixed effect model to take into account the variability in cells within each mouse when comparing mice between groups (Topbp1+/+ vs Topbp1B5/B5). We then reanalyzed all quantified meiotic spreads using this mixed effect model, and the p-value, number of mice, and number of cells counted for each group are displayed in the respective figure legends. Upon going through all the quantified meiotic spreads, we realized a minor error in one of the previous data points related to SETX staining in Topbp1+/+ and have fixed it. Using the previous quantification data and the new stats analysis the p-value for cores was 0.5598 and p-value for loops was 0.0273. Now using the correct values and the new stats analysis the p-value for cores is 0.5987 and p-value for loops is 0.0452. The correction did not change the conclusion of this data and is now displayed in the new Figure 5. We also realized a mistake in the ATR quantification when the spreadsheet was moved from excel to Graphpad. Using the previous quantification and the new stats analysis the p-value for cores was 0.2451 and p-value for loops was 0.8933. Now using the correct values and the new stats analysis the p-value for cores is 0.4068 and p-value for loops is 0.9396. The correction did not change the conclusion of this data and is now displayed in the new Figure 4. Moreover, we realized that we used n = 8 (n = number of mice) for MDC1 quantification and n = 2 for pCHK1_S345, instead of n =3 as shown in the preprint version of the manuscript. Corrected values were added to their respective figures and figure legends.

      (3) From Figures 6E, 7B, and 7C, the authors conclude the difference in the expression profile between wild type and Topbp1(B5) spermatocytes. It is better to show P-values for the comparison. Particularly, in Figure 7C, Xiap expression kinetics look similar between wild type and the mutant.

      We have added p-values to figures 6E and 7B and their respective figures or figure legends.<br /> In figure 7C, we now recognize that the Δ could have been misleading as we meant to compare Wild-type SP2 to Wild-type SP3 and Mutant SP2 to SP3; and not comparing Wild-type SP3 to Mutant SP3. Therefore, the Δ was excluded from Figure 7C. For the comparisons between expression levels of SP2 and SP3, it is challenging to calculate p-values for a single gene since these cells have started X-gene silencing and expression values are very low. Meaningful p-values for the comparisons between Wildtype SP3 to Mutant SP3 can be visualized in Figure 7B, where the comparison is based on number of genes instead of expression levels of each gene.

      Minor comments:

      1) Line 34: SPO11 is NOT a nuclease. Just delete it.

      It has been deleted (see line 34).

      2) Line 71, a protein: Is this protein ATR? Is so, please write it. If not, please give the name of the protein.

      In line 71 (now lines 79-80), we refer to TOPBP1-interacting proteins in general since many of these interactions happen through a phosphorylation in the TOPBP1’s interactor. This is the case for BLM, 53BP1, FANCJ, and RAD9. ATR interacts with TOPBP1 through TOPBP1’s AAD domain and this is not a phospho-mediated interaction. We restructured the sentence for clarity.

      3) In the Introduction, the authors often refer to a review by Cimprich and Cortez (2008) in various places. It is better to cite an original paper or the other an appropriate review.

      We have accepted the reviewer’s suggestion and added original papers when appropriate.

      4) Line 143-145: The authors generated eight charge reversal point mutations in the BRCT domain 5 of TOPBP1. If possible, it is helpful to mention the logic to generate these substitutions and also why BRCT domain 5, is not other domains.

      We generated eight charge reversal point mutations to abrogate all possible phospho-dependent interactions and avoid potential residual interactions. We have mutated other BRCT domains as well, which will be published separately.

      5) Line 174 (and Figure 2E): RPA should be either RPA2 or RPA32.

      Corrected (it is RPA2).

      6) Figure 5C-F: Please explain in more detail how the authors quantified the SETX signals. Why the two results are different?

      The quantification was done as described by Sims, et al. 2022, yielding separate data for XY cores and DNA loops. In brief, the green signal was measured along (XY cores) or across (XY DNA loops) the X and Y chromosomes. Signals were normalized by the signal in the autosomal chromosomes.

      Reviewer #3 (Recommendations For The Authors):

      I have no major criticisms, but I include a list of comments and suggestions (some of them conceptual, and disputable) that could help the authors to improve some parts of the manuscript.

      1) Line 52: I realize that the term protein "sequestration" (used in many instances along the manuscript) has been widespread in the literature related to MSCI in the last years. While this might be a cool way to describe the dynamics of proteins accumulating in the sex body, this reviewer considers this term is totally inappropriate. It is confusing and introduces at least to mistakes to the fact of protein accumulation in the sex body. First, it seems to indicate that once trapped in the sex body, proteins are incapable of leaving it, which might be completely wrong (histone replacement refutes this idea). Second, it is suggested that DDR proteins are attracted by the sex body and cannot remain associated to autosomes even if DNA repair has not been completed. This has also been demonstrated to be incorrect (see for example PDMI 19714216). Moreover, DDR proteins can associate de novo to chromosomes if needed, for instance upon DNA damage caused by chemicals or irradiation. Thus, I suggest that the use of "sequestration" should be evaluated more critically, evaluating the misleading ideas that are subjacent to this term. The use of protein "accumulation" is much more objective and descriptive of the real facts.

      We thank the reviewer’s suggestion and have addressed it in lines 52, 97 and 324.

      2) Line 88: Just as a deference to the original ideas, it would be nice to acknowledge that the inactivation of sex chromosomes and the formation of a sex body in mouse meiosis was described more than 50 years ago (PDMI 5833946; 4854664). Likewise, the ideas about the sequential achievement and reinforcement of MSCI during pachytene have been developed during the last 20 years, far before the recent reports cited in the manuscript. Citations to these "old fashion" works would be great.

      We appreciate the reviewer’s suggestion and have addressed it in line 86.

      3) Line 90. Please, take into consideration that such a strong effect on meiosis progression occurs mainly in some knockout mice models and that in many other models (including hybrid mice models from natural populations) autosomal regions can remain unsynapsed and accumulate DDR proteins without impairing meiosis. In other mammalian species, meiosis is even more permissive to these MSUC phenomena.

      We appreciate the reviewer’s suggestion and have addressed it at line 88.

      4) Line 211: The differences in the abundance of MLH1 and MLH3 are remarkable. If these two proteins are supposed to form a heterodimer leading to crossover formation, then the increase of only MLH1 might be related to a different process, not leading to crossover (even not class II ones).

      We agree with the reviewer’s comment and have included this point in the discussion (lines 491- 497).

      5) Line 217: I have some doubts about the results presented in Supplementary Figure 9. First, it is not clear to me how the represented cells counts were performed. Each spot is supposed to represent cell counts in a single individual, but how many cells were counted per individual? The proportion of cells could be a better indicator. Second, some B5/B5 individuals' counts were close to the ones displayed in the wild type. Did mutant animals show a high divergence compared to each other? It could be great to have each individual data displayed in a pie chart, and not only the aggregated data.

      We have now addressed this in the new Supplemental figure 9 legend. Each dot in the graph represents the sum of cells counted for each individual. We counted cells from 8 mice for each, Topbp1+/+ and Topbp1B5/B5.

      Here we summarize the total cells counted per individual:

      Author response table 1.

      6) Line 222: The data on 53BP1 deserve further attention. On the one side, from the analysis presented in Supplementary Figure 11, it seems that 53BP1 tends to show a lower intensity in Topbp1B5/B5 mice. Since only 2 mice were analyzed, while for most of the other proteins 3-8 animals were studied, I suggest increasing the number of animals analyzed for 53BP1 localization, to test if this slight difference turns significant. This is relevant since: 1) the association of 53BP1 protein in somatic cells was clearly affected, and 2) 53BP1 is one of the last MSCI markers incorporated to the sex body at mid-late pachytene. These results should be moved to the main text and not appear as supplementary data. On the other hand, if no differences were to be found in meiosis, compared to somatic cells, how do authors explain these differences? Would 53BP1 have another partner at the sex body apart from TOPBP1? Could TOPBP1 have other BRCT domains (apart from domain 5) able to bind 53BP1?

      We appreciate the reviewer’s suggestion; however, we had an issue with 53BP1 antibody. We analyzed 2 mice and needed to re-order the antibody. This antibody was backordered for almost one year, and when we finally received the order, the company had changed the clone for this antibody, and it no longer worked for meiotic spreads. In somatic cells, we see in HEK-293T a partial disruption in the binding to TOPBP1 B5 through IP-MS and IP-Western blot. The disruption is only partial due to the binding of 53BP1 to other domains in TOPBP1 such as BRCT 1 and 2 (Bigot et al., 2019; Cescutti et al., 2010; Liu et al., 2017). However, in assays in which we would expect a phenotypic response caused by impaired 53BP1, we did not see any effect, such as survival after IR (using the mice) and survival after phleomycin challenge (using Mefs). Moreover, 53BP1 KO mice, males and females, are fertile (Ward et al., 2003) so, the partial disruption in binding to 53BP1 that we observed in TOPBP1 B5 mutant is likely not causing the infertility phenotype.

      7) Line 250: I do not understand what is represented in Figure 5A. Why did the author mix two different experiments (differences in phosphoprotein abundance in B5/B5 compared to wild type and the interference of ATR with AZ20)?

      To account for the differences in cell population observed in the whole testis between Topbp1+/+ and Topbp1B5/B5, and to know exactly which phosphorylation changes were due to disruption in the ATR signaling and not pleiotropic effects, we combined two different phosphoproteomes: One phosphoproteome from the comparison between Topbp1+/+ and Topbp1B5/B5 and another one from the comparison between Vehicle or ATR inhibitor-treated mice. By utilizing this approach, we only consider hits that were disrupted in both analyses. A similar method was used by Sims et.al, 2022 (Sims et al., 2022).

      8) It is not clearly explained what is represented in Figure 6B. There is no explanation in the text or the figure legend. Do this represent the difference between scRNAseq in control and Topbp1B5/B5? If so, please, clarify.

      We thank the reviewer’s comment and have addressed it in the legend of Figure 6B.

      9) Line 342 and following. The authors describe a decrease of gene silencing. The use of two negative concepts is always confusing and results in the conversion to a positive one. I suggest considering the possibility of just talking about increase of gene expression, in order to make the message clearer.

      We appreciate the reviewer’s point here, but it is important to note that the phenomenon disrupted in our mutants is MSCI, which is by definition a gene silencing mechanism. This phenotype is not as simple as “increased gene expression”, it is the removal of a mechanism that is a key feature of prophase I. Thus, because we are focusing on the mechanism of MSCI, it is crucial to maintain this (albeit unusual) terminology.

      10) As for the classification of spermatocytes into 9 categories, I am curious about which spermatocytes are included in each of these categories. For instance, from cytology it seems that in Topbp1B5/B5 mice, spermatocytes are able to reach mid-late pachytene. However, in the spermatocyte categories established by scRNAseq they only reach class 3. Therefore, which are the populations included in the remaining 6 classes of spermatocytes? Do authors have any morphological correlation to these scRNAseq categories? Is it possible that in this mutant morphological advance of meiosis and gene expression profiles are uncoupled?

      The clustering of cells to a specific group is based on RNA expression, which does not always match cytological features. Moreover, during the analysis, cells with high expression of mitochondrial genes are excluded (these are dying cells that do not pass the quality control). Thus, while Topbp1B5/B5 reaches a mid-late-pachytene stage according to cytological analyses, in the single-cell RNA seq analysis we could only detect one pachytene stage. The other 6 remaining categories of spermatocytes can be classified according to their best-fit profile of gene expression. For that, we use the classification described by Chen et al., 2018 and Lau et al.,2020. Spermatocytes 3-5 = Pachytene, Spermatocytes 6-7 = Diplotene, Spermatocytes 8-9 = secondary spermatocytes (metaphase I/II). The gene markers used for this classification are displayed in Author response image 2.

      Author response image 2.

      Genes used as markers of spermatocytes captured in the scRNAseq analysis. Violin plots display the distribution of cells expressing Gm960 (Leptotene marker), Meiob (Leptotene/Zygotene marker), Psma8 (Pachytene marker), Pwill1 (Pachytene marker), Pou5f2 (Diplotene marker), and Ccna1 (Secondary Spermatocytes marker).

      11) Figure 6E shows that overexpression of X-linked genes is not a feature of spermatocytes but it is initiated in spermatogonia. This fact has not been properly stated in the text and perhaps not sufficiently highlighted.

      We noticed subtle changes during the spermatogonia stage and have addressed the reviewer’s comment in lines 317-322, however the downstream analyses related to a defect in X-gene silencing maintenance displayed in Figure 7 were done based on normalization of gene expression to its respective pre-leptotene stage.

      12) Supplementary Figure 24 shows that some X-linked genes are more expressed in Topbp1B5/B5 compared to control mice. In the figure it can be observed that many genes accumulate at the bottom of the graph. Does this have any correlation to the location of these genes along the X chromosome, for instance near or within the PAR? This could correlate with the defects in γH2AX accumulation at this region.

      These are the locations along the chromosome. Only the bottom 5 rows are within the PAR region, so this accumulation is not within the PAR region specifically. The bottom tenth of the genes in the heatmap correspond to roughly a 17 Mb region.

      13) The authors only analyzed the overexpression of genes located on the X chromosome. It would be interesting to show the behavior of Y-linked genes as well.

      The coverage of Y-linked genes was not very high and that is why we have not shown the results in the paper. However, the results for Y-linked genes were similar to the X-linked genes and can be visualized in Author response image 3.

      Author response image 3.

      Single cell RNAseq reveals that Topbp1B5/B5 spermatocytes initiate MSCI but fail to promote full silencing of Y chromosome-linked genes. Violin plot displaying the ratio of the average expression of Y chromosome genes by the average expression of chromosome 9 genes at different stages of spermatogenesis for Topbp1+/+ and Topbp1B5/B5 cells.

      14) Line 425: Authors indicate that it is not known if association of TOPBP1 and BLM, 53BP1 or other proteins is disrupted in Topbp1B5/B5 spermatocytes. Could these experiments be performed in the testis, as they were in somatic cells?

      The cellular composition in Topbp1+/+ and Topbp1B5/B5 testes is very different so it would not be a fair comparison. While we have tried to isolate pachytene cells to perform these experiments, we were successful only when using Topbp1+/+ but not Topbp1B5/B5, likely due to the extremely small size of the mutant testis.

      15) Line 455 and following. I find that the discussion about the role of SETX is not completely clear. It seems that a failure of SETX function could result in defective or no transcription, as a consequence of the impossibility to resolve RNA-DNA hybrid molecules. Therefore, should impairment of SETX lead to reduced or enhanced transcription? Please clarify. On the other hand, this defect in SETX function should affect the whole genome, and not only sex chromosomes. Do authors have any clues about this broad effect?

      We thank the reviewer’s comment and have expanded on discussion in lines 470-474. While we agree with the reviewer’s point that an impairment on SETX should affect the whole genome, however, during pachytene stage, SETX is mostly localized to the sex body. The Topbp1B5/B5 shows a specific defect in X and Y silencing maintenance during pachytene stage, thus we hypothesized that an impairment in SETX localization during pachytene should especially impair the X and Y chromosomes.

      16) As a general comment to the discussion section, I think authors could extend into some specific ideas or speculations. It is shocking that sex chromosome-linked genes are able to escape silencing without dismantling the complex (almost complete) MSCI response in the Topbp1 mutant (although perhaps this is not so surprising considering the high number of escapees reported in the inactivated X chromosome in female somatic cells).

      How to explain this paradox? One possibility (which would make a real breakthrough) is that the expression of sex chromosome-linked genes represents a regulated response to meiotic defects, and not just an unfortunate consequence of a defective MSCI. Thus, MSCI might be somehow irrelevant to prevent the execution of this sex chromosome-based program to stop meiosis progression when needed. The fact that this regulated activation was never proposed is perhaps due to the fact that most of the meiosis mutants characterized so far are unable to reach the stage at which MSCI is properly established, which is the most remarkable difference with the Topbp1 mutant studied here.

      Although naïve, the critical point for the activation of this sex chromosome-based program seems to depend simply on the transcription of Zfy1 and Zfy2 (encoding for transcription factors). The signaling cascades up and downstream these genes are the real mystery, awaiting further studies.

      We thank the very interesting point raised by the reviewer. Our interpretation of the data is that X and Y silencing being a dynamic process requires an initiation step and a maintenance step driven/controlled by the DDR machinery, and that Topbp1B5/B5 shows a grossly normal initiation of X and Y silencing but fails on maintain MSCI. Moreover, the expression of Zfy1 and Zfy2 have been previously demonstrated as enough to trigger cell death (Royo et al., 2010; Vernet et al., 2016), and Topbp1B5/B5 cells show increased expression of these genes. However, we do not exclude the very interesting possibility, raised by the reviewer, that the expression of XY-linked genes represents a regulated response to meiotic defects to stop meiosis progression, leading to the cell death observed in Topbp1B5/B5, which makes the Topbp1B5/B5 an unique model for these studies as most of the previous meiosis mutants are unable to reach the stage at which MSCI is properly established. We add discussion about this exciting point in lines 513-522.

      17) Scale bars are impossible to read in Figures 1I and J, and are missing in all the other image figures. Please, correct.

      We have addressed this in the new Figure 1. For figures displaying meiotic spreads, adding a scale bar is not a common practice in the field as these cells are swollen while being prepared.

      18) Line 828. Since Paula Cohen is an author of the manuscript, it seems weird to acknowledge herself in this section.

      Corrected.

      References

      Adams SR, Maezawa S, Alavattam KG, Abe H, Sakashita A, Shroder M, Broering TJ, Sroga Rios J, Thomas MA, Lin X, Price CM, Barski A, Andreassen PR, Namekawa SH. 2018. RNF8 and SCML2 cooperate to regulate ubiquitination and H3K27 acetylation for escape gene activation on the sex chromosomes. PLoS Genet 14. doi:10.1371/journal.pgen.1007233

      Bigot N, Day M, Baldock RA, Watts FZ, Oliver AW, Pearl LH. 2019. Phosphorylation-mediated interactions with topbp1 couple 53bp1 and 9-1-1 to control the g1 DNA damage checkpoint. Elife 8:1–28.

      Cescutti R, Negrini S, Kohzaki M, Halazonetis TD. 2010. TopBP1 functions with 53BP1 in the G1 DNA damage checkpoint. EMBO J 29:3723–3732.

      Chen Y, Zheng Y, Gao Y, Lin Z, Yang S, Wang T, Wang Q, Xie N, Hua R, Liu M, Sha J, Griswold MD, Li J, Tang F, Tong M-H. 2018. Single-cell RNA-seq uncovers dynamic processes and critical regulators in mouse spermatogenesis. Cell Res 28:879–896.

      Hirota T, Blakeley P, Sangrithi MN, Mahadevaiah SK, Encheva V, Snijders AP, ElInati E, Ojarikre OA, de Rooij DG, Niakan KK, Turner JMA. 2018. SETDB1 Links the Meiotic DNA Damage Response to Sex Chromosome Silencing in Mice. Dev Cell 47:645-659.e6.

      Ichijima Y, Ichijima M, Lou Z, Nussenzweig A, Daniel Camerini-Otero R, Chen J, Andreassen PR, Namekawa SH. 2011. MDC1 directs chromosome-wide silencing of the sex chromosomes in male germ cells. Genes and Development 25:959–971.

      Lau X, Munusamy P, Ng MJ, Sangrithi M. 2020. Single-Cell RNA Sequencing of the Cynomolgus Macaque Testis Reveals Conserved Transcriptional Profiles during Mammalian Spermatogenesis. Dev Cell 54:548-566.e7.

      Liu Y, Cussiol JR, Dibitetto D, Sims JR, Twayana S, Weiss RS, Freire R, Marini F, Pellicioli A, Smolka MB. 2017. TOPBP1Dpb11 plays a conserved role in homologous recombination DNA repair through the coordinated recruitment of 53BP1Rad9. J Cell Biol 216:623–639.

      Modzelewski AJ, Holmes RJ, Hilz S, Grimson A, Cohen PE. 2012. AGO4 regulates entry into meiosis and influences silencing of sex chromosomes in the male mouse germline. Dev Cell 23:251–264. Royo H, Polikiewicz G, Mahadevaiah SK, Prosser H, Mitchell M, Bradley A, De Rooij DG, Burgoyne PS, Turner JMA. 2010. Evidence that meiotic sex chromosome inactivation is essential for male fertility. Curr Biol 20:2117–2123.

      Sims JR, Faça VM, Pereira C, Ascenção C, Comstock W, Badar J, Arroyo-Martinez GA, Freire R, Cohen PE, Weiss RS, Smolka MB. 2022. Phosphoproteomics of ATR signaling in mouse testes. Elife 11. doi:10.7554/eLife.68648

      Vernet N, Mahadevaiah SK, de Rooij DG, Burgoyne PS, Ellis PJI. 2016. Zfy genes are required for efficient meiotic sex chromosome inactivation (MSCI) in spermatocytes. Hum Mol Genet 25:5300–5310.

      Ward IM, Minn K, van Deursen J, Chen J. 2003. p53 Binding protein 53BP1 is required for DNA damage responses and tumor suppression in mice. Mol Cell Biol 23:2556–2563.

      Yeo AJ, Becherel OJ, Luff JE, Graham ME, Richard D, Lavin MF. 2015. Senataxin controls meiotic silencing through ATR activation and chromatin remodeling. Cell Discovery 1. doi:10.1038/celldisc.2015.25

    1. Author response:

      The following is the authors’ response to the current reviews.

      Public Reviews:

      Reviewer #1 (Public review):

      Summary:

      This paper examines patterns of diversity and divergence in two closely related sub-species of Zea mays. While the data are interesting and the authors have tried to exclude multiple confounding factors, many patterns cannot clearly be ascribed to one cause or another.

      Strengths:

      The paper presents interesting data from sets of sympatric populations of the two sub-species, maize and teosinte. This sampling offers unique insights into the diversity and divergence between the two, as well as the geographic structure of each. Many analyses and simulations to check analyses have been carried out.

      Weaknesses:

      The strength of conclusions that can be drawn from the analyses was low, partly because there are many strange patterns. The authors have done a good job of adding caveats, but clearly, these species do not meet many assumptions of our methods.

      Thank you for the comments. We appreciate the multiple rounds of revision the manuscript has undergone and the work has improved as a consequence. Overall we disagree that the patterns are strange, and have made considerable efforts to explain in the text and in our responses why the patterns make sense based on what we know about the history of Zeamays from previous research. We agree that currently available methods are not capable of answering all questions we propose adequately. This reflects both limitations with the available data for these populations (i.e. phenotypes and spatially explicit sampling), and limitations in available methods tailored to the questions at hand (spatially explicit inference of the range over which an allele is adaptive). We have made considerable effort to point out the places where our inferences are likely to have low accuracy or limited resolution. These limitations are in many ways inherent to all inferential based science and should not be considered a weak point specific to this work, nor do they take away from the fundamental conclusions, which have changed quantitatively but not qualitatively over the course of peer review.

      Recommendations for the authors:

      Reviewer #1 (Recommendations for the authors):

      -The manuscript should say something about the fact that range-wide PSMC does not show a decline.

      We did not use PSMC methods but instead mushi as described in the methods. On line 356 we described how the lower sample size and strong regularization are the most likely explanations for the lack of a population size decline in the rangewide samples.

      - The manuscript should explain how rdmc was run and what "overlapping" means.

      We described how sweep intervals were inferred starting on line 823 (Methods subsection “Identifying Selective Sweeps”). Sweep regions were defined as the outermost coordinates from all populations that shared any overlap in their respectively defined sweep intervals. The details of how we ran rdmc, including all of the parameters, is described starting on line 895 (methods subsection “Inferring modes of convergent adaptation”).

      - Figure 4: "Negative log10" is messed up

      Thank you. This has been fixed for the Version Of Record.

      - Line 318: "accruacy"

      Thank you. We have edited this typo for the Version Of Record.

      - New Table S3: why don't the proportions add to 1?

      These values represent what proportion of fixed differences at 0 fold sites are unique to each population. The denominator is the total number of fixed differences for each population separately, so each proportion is distinct for each population and thus should not sum to one across them. The table caption has been reworded in efforts to clarify for the Version Of Record.


      The following is the authors’ response to the original reviews.

      Public Reviews:

      Reviewer #1 (Public Review):

      Summary:

      This paper examines patterns of diversity and divergence in two closely related sub-species of Zea mays. While the patterns are interesting, the strength of evidence in support of the conclusions drawn from these patterns is weak overall. Most of the main conclusions are not supported by convincing analyses.

      Strengths:

      The paper presents interesting data from sets of sympatric populations of the two sub-species, maize and teosinte. This sampling offers unique insights into the diversity and divergence between the two, as well as the geographic structure of each.

      Weaknesses:

      There were issues with many parts of the paper, especially with the strength of conclusions that can be drawn from the analyses. I list the major issues in the order in which they appear in the paper.

      (1) Gene flow and demography.

      The f4 tests of introgression (Figure 1E) are not independent of one another. So how should we interpret these: as gene flow everywhere, or just one event in an ancestral population? More importantly, almost all the significant points involve one population (Crucero Lagunitas), which suggests that the results do not simply represent gene flow between the sub-species. There was also no signal of increased migration between sympatric pairs of populations. Overall, the evidence for gene flow presented here is not convincing. Can some kind of supporting evidence be presented?

      We agree that the standard approach to f4 tests that we employed here is not without limitations, namely, that the tests are conducted independently, while the true evolutionary history is not. While a joint demographic inference across all populations would be useful, it did not seem tractable to perform over all of our populations with currently available methods, given the number of populations being analyzed, nor does it directly address the question of interest. Our purpose for including the f4 was testing if there was more gene flow between sympatric pairs than in other comparisons (we have made that point more clear in the text near line 174. As described in the text, the distribution of Z scores is generated by pairing focal populations with all other non-focal populations across both subspecies, which means the gene flow signal of interest is marginalized over the effects of gene flow in the other non-focal populations. This is not nearly as rich as inferring the full history, but it gives us some sense of the average amount of gene flow experienced between populations and allows us to address one of our primary questions of interest when conceiving this paper - do sympatric pairs show more geneflow than other pairs? We agree with the reviewer that that answer is largely no, and the writing reflects this.

      Overall, we think both points mentioned by the reviewer here; finding that most but not all tests involved Crucero Lagunitas maize, and that sympatric pairs don’t show higher gene flow; nicely contributes to the overall theme in the paper - the history of both subspecies is idiosyncratic and impacted by humans in ways that do not reflect geographic proximity that we did not anticipate (see expectations near line 110). We have emphasized the connection between f4 tests and the revised rdmc results near line 653.

      The paper also estimates demographic histories (changes in effective population sizes) for each population, and each sub-species together. The text (lines 191-194) says that "all histories estimated a bottleneck that started approximately 10 thousand generations ago" but I do not see this. Figure 2C (not 2E, as cited in the text) shows that teosinte had declines in all populations 10,000 generations ago, but some of these declines were very minimal. Maize has a similar pattern that started more recently, but the overall species history shows no change in effective size at all. There's not a lot of signal in these figures overall.

      I am also curious: how does the demographic model inferred by mushi address inbreeding and homozygosity by descent (lines 197-202)? In other words, why does a change in Ne necessarily affect inbreeding, especially when all effective population sizes are above 10,000?

      All maize populations show a decline beginning 10,000 generations ago. The smallest decline for maize is from 100,000 to 30,000. All teosinte populations show a reduction in population size. The smallest of these drops more than 70% from around 300,000 to 100,000. Three of the teosinte populations showed a reduction in population size from ~10^5 to ~10^3, which is well below 10,000. Thus all populations show declines.

      These large reductions should lead to inbreeding and increased homozygosity by descent. Mushi does not specifically model these features of the data, yet as we show, simulations under the model estimated by Mushi matched the true HBD levels fairly well (Figure 2D).

      The rangewide sample does not show declines, likely because there is enough isolation between populations that the reduction in variation at any given locus is not shared, and is maintained in the populations that did not experience the population decline.

      (2) Proportion of adaptive mutations.

      The paper estimates alpha, the proportion of nonsynonymous substitutions fixed by positive selection, using two different sampling schemes for polymorphism. One uses range-wide polymorphism data and one uses each of the single populations. Because the estimates using these two approaches are similar, the authors conclude that there is little local adaptation. However, this conclusion is not justified.

      There is little information as to how the McDonald-Kreitman test is carried out, but it appears that polymorphism within either teosinte or maize (using either sampling scheme) is compared to fixed differences with an outgroup. These species might be Z. luxurians or Z. diploperennis, as both are mentioned as outgroups. Regardless of which is used, this sampling means that almost all the fixed differences in the MK test will be along the ancestral branch leading to the ancestor of maize or teosinte, and on the branch leading to the outgroup. Therefore, it should not be surprising that alpha does not change based on the sampling scheme, as this should barely change the number of fixed differences (no numbers are reported).

      The lack of differences in results has little to do with range-wide vs restricted adaptation, and much more to do with how MK tests are constructed. Should we expect an excess of fixed amino acid differences on very short internal branches of each sub-species tree? It makes sense that there is more variation in alpha in teosinte than maize, as these branches are longer, but they all seem quite short (it is hard to know precisely, as no Fst values or similar are reported).

      The section “Genetic Diversity” in the methods provides details about how luxurians and diploperennis were used as outgroups. The section “Estimating the Rate of Positive Selection, α”, in the methods includes the definition of α and full joint non-linear regression equation and the software used to estimate it (brms), and the relevant citations crediting the authors of the original method. However, some of the relevant information about the SFS construction is provided in the previous section entitled, “Genetic Diversity”. We added reference to this in results near line 800.

      While we appreciate the concern that “almost all the fixed differences in the MK test will be along the ancestral branch leading to the ancestor of maize or teosinte”, this is only a problem if there aren’t enough fixed differences that are unshared between populations. This is more of a concern for maize than teosinte, which we make clear as a caveat in the manuscript in several places already. The fact that there is variation in alpha among teosinte populations is evidence that these counts do differ among pops. As we can see in the population trees in Figure 1, there is a considerable amount of terminal branch length for all the populations. Indeed if we look at the number of fixed differences at 0 fold sites across populations:

      The variation in the number of fixed differences, particularly across teosinte means that a large number cannot be shared between populations. We can estimate the fixed differences unique to each subpopulation (and total count) demonstrating that, in general, there are a large number of substitutions unique to each population. This is good evidence the rangewide estimates do not reflect a lack of variation within populations, at least not for teosinte. This is now included in the supplement (Table S3).

      Finally, we note that the branches leading to outgroups are likely not substantially longer than those among populations. Given our estimates of Ne, the coalescent within maize and teosinte should be relatively deep (with Ne of 30K it should be ~120K years). The divergence time between Zea mays and these outgroup taxa has been estimated at ~150K years (Chen et al. 2022). This is now mentioned in the text on line 407.

      We have added a caveat about the reviewers concern for the non-independence of fixed difference for maize near line 386.

      (3) Shared and private sweeps.

      In order to make biological inferences from the number of shared and private sweeps, there are a number of issues that must be addressed.

      One issue is false negatives and false positives. If sweeps occur but are missed, then they will appear to be less shared than they really are. Table S3 reports very high false negative rates across much of the parameter space considered, but is not mentioned in the main text. How can we make strong conclusions about the scale of local adaptation given this? Conversely, while there is information about the false positive rate provided, this information doesn't tell us whether it's higher for population-specific events. It certainly seems likely that it would be. In either case, we should be cautious saying that some sweeps are "locally restricted" if they can be missed more than 85% of the time in a second population or falsely identified more than 25% of the time in a single population.

      The reviewer brings up a worthwhile point. The simulation results indeed call into question how many of the sweeps we claim are exclusive to one population actually are. This caveat is already made, but we now make clearer the reviewer’s concern regarding the high false negative rate (near line 299). However, if anything this suggests sweeps are shared even more often than what is reported. One of the major takeaways from the paper is that convergent adaptation is more common than we expected. The most interesting part about the unique sweeps is the comparison between maize and teosinte. While the true proportions may vary, the relatively higher proportion of sweeps exclusive to one population in teosinte compared to maize is unlikely to be affected by false negatives, since the accuracy to identify sweeps pretty similar across subspecies (though perhaps with some exceptions for the populations with stronger bottlenecks). Further, these criticisms are specific to the raisd results. All sweeps shared across multiple populations were analyzed using rdmc. After adjustments made to the number of proposed sites for selection (see response below), there is good agreement between the raisd and rdmc results - the regions we proposed as selective sweeps with raisd all show evidence convergence using rdmc. Recall too that rdmc uses a quite different approach to inference - all populations are used jointly, labelling those that did and did not experience the sweep. If sweeps were present in populations that were labeled as neutral (or vice versa), this would weaken the power to infer selection at the locus. Much of the parameter space we explored is for quite weak selection, and the simulated analysis shows we are likely to miss those instances, often entirely. For strong sweeps, however, our simulations show we have appreciable accuracy.

      Together, there is reason to be optimistic about our detection of strong shared sweeps and that the main conclusions we make are sound.

      Finally, we note that we are unaware of any other empirical study that has performed similar estimates of the accuracy of the sweep calling in their data (as opposed to using simulations). We thus see these analyses as a significant contribution towards transparency that is completely lacking from most papers.

      A second, opposite, issue is shared ancestral events. Maize populations are much more closely related than teosinte (Figure 2B). Because of this, a single, completed sweep in the ancestor of all populations could much more readily show a signal in multiple descendant populations. This is consistent with the data showing more shared events (and possibly more events overall). There also appear to be some very closely (phylogenetically) related teosinte populations. What if there's selection in their shared ancestor? For instance, Los Guajes and Palmar Chico are the two most closely related populations of teosinte and have the fewest unique sweeps (Figure 4B). How do these kinds of ancestrally shared selective events fit into the framework here?

      The reviewer brings up another interesting point and one that likely impacts some of our results.

      As the reviewer describes, this is an issue that is of more concern to the more closely related populations and is less likely to explain results across the subspecies. We have added this as a caveat (near line 456). As is clear in the writing, sharing across subspecies is our primary interest for the rdmc results.

      These analyses of shared sweeps are followed by an analysis of sweeps shared by sympatric pairs of teosinte and maize. Because there are not more events shared by these pairs than expected, the paper concludes that geography and local environment are not important. But wouldn't it be better to test for shared sweeps according to the geographic proximity of populations of the same sub-species? A comparison of the two sub-species does not directly address the scale of adaptation of one organism to its environment, and therefore it is hard to know what to conclude from this analysis.

      We did not intend to conclude that local adaptation is not important. Especially for teosinte, we report and interpret evidence that many sweeps are happening exclusively to one population, which is consistent with the action of location adaptation and consistent with some of our expectations.

      More directly, this is another instance of us having clear hypotheses going into the paper and constructing specific analyses to test them. As we explain in the paper, we expected the scale of local adaptation to be very small, such that subspecies growing next to each other have more opportunities to exchange alleles that are locally adapted to their shared environment. The analysis we conducted makes sense in light of this expectation. We considered conducting tests regarding geographic proximity, but there is limited power with the number of populations we have within subspecies, and the meaning of the tests is unclear if all populations of both subspecies are naively included together. This analysis shows that, at least for sweeps and fixations, adaptation is larger than a single location. While it may not be a complete description on its own, the work here does provide information about the scale of adaptation and is useful to our overall claims and objectives of the paper. As mentioned in the paper, the story might be very different if we were to study through a lens of polygenic adaptation. We also now include in the discussion in several places mention of where broader sampling could improve inference.

      (4) Convergent adaptation

      My biggest concern involves the apparent main conclusion of the paper about the sources of "convergent adaptations". I believe the authors are misapplying the method of Lee and Coop (2017), and have not seriously considered the confounding factors of this method as applied. I am unconvinced by the conclusions that are made from these analyses.

      The method of Lee and Coop (referred to as rdmc) is intended to be applied to a single locus (or very tightly linked loci) that shows adaptation to the same environmental factor in different populations. From their paper: "Geographically separated populations can convergently adapt to the same selection pressure. Convergent evolution at the level of a gene may arise via three distinct modes." However, in the current paper, we are not considering such a restricted case. Instead, genome-wide scans for sweep regions have been made, without regard to similar selection pressures or to whether events are occurring in the same gene. Instead, the method is applied to large genomic regions not associated with known phenotypes or selective pressures.

      I think the larger worry here is whether we are truly considering the "same gene" in these analyses. The methods applied here attempt to find shared sweep regions, not shared genes (or mutations). Even then, there are no details that I could find as to what constitutes a shared sweep. The only relevant text (lines 802-803) describes how a single region is called: "We merged outlier regions within 50,000 Kb of one another and treated as a single sweep region." (It probably doesn't mean "50,000 kb", which would be 50 million bases.) However, no information is given about how to identify overlap between populations or sub-species, nor how likely it is that the shared target of selection would be included in anything identified as a shared sweep. Is there a way to gauge whether we are truly identifying the same target of selection in two populations?

      The question then is, what does rdmc conclude if we are simply looking at a region that happened to be a sweep in two populations, but was not due to shared selection or similar genes? There is little testing of this application here, especially its accuracy. Testing in Lee and Coop (2017) is all carried out assuming the location of the selected site is known, and even then there is quite a lot of difficulty distinguishing among several of the non-neutral models. This was especially true when standing variation was only polymorphic for a short time, as is estimated here for many cases, and would be confused for migration (see Lee and Coop 2017). Furthermore, the model of Lee and Coop (2017) does not seem to consider a completed ancestral sweep that has signals that persist into current populations (see point 3 above). How would rdmc interpret such a scenario?

      Overall, there simply doesn't seem to be enough testing of this method, nor are many caveats raised in relation to the strange distributions of standing variation times (bimodal) or migration rates (opposite between maize and teosinte). It is not clear what inferences can be made with confidence, and certainly the Discussion (and Abstract) makes conclusions about the spread of beneficial alleles via introgression that seem to outstrip the results.

      We have fixed the “50,000 Kb” typo.

      There are several important points the reviewer makes here worth considering. First and most importantly, the method of Lee and Coop (2017) actually does include sites as part of the composite likelihood calculation. For computational feasibility, the number of positions we initially considered was 20 (20 different positions along the input sequence were proposed as the site of the shared beneficial mutation). In efforts to further address the reviewer’s concern about adaptive mutations at distinct loci, we have increased the number of proposed selected sites to 200. This fact should greatly diminish the reviewer’s concern that we are picking up independent sweeps that happened at different nucleotide positions in the same region - evidence for a beneficial mutation must be shared by the selected populations at a proposed site. As the revisions show, this has modified the results of our paper in a number of ways, including changing all of the previous neutral regions to shared via standing variation or migration. Despite these changes, our previous conclusions are intact, including the pattern that migration rates are high when maize populations share the sweep. Relatedly, we disagree with the reviewer’s characterization of the migration results. The pattern is quite clear and makes sense - when a maize population is involved in the sweep, migration rate is inferred to be high. Sweeps exclusive to teosinte are rarer and are inferred to have a low migration rate. This relates directly to the idea that humans have moved maize relatively rapidly across the landscape.

      We have now included a plot showing how the difference between the maximum composite likelihood (CLE) site compares to the next highest CLE site varies across our inferences (Figure S8), which strongly suggests that patterns are not muddled across multiple loci, but are centered at a focal region where the beneficial allele is inferred to be located. While there are too many to show in the manuscript across all sweeps, here is a nice example of what inference looks like for one of the proposed sweep regions.

      Author response image 1.

      Furthermore, the situation the reviewer is describing would be selection acting on independent mutations (mutations at different loci), which would not create an increase in the amount of allele frequency covariance above and beyond what would be expected by drift under the migration and standing variation models.

      We also note that we are not alone in applying this approach to shared outlier signals in the absence of known genes; indeed the authors of the DMC method have applied it to regions of shared outlier signal themselves (e.g. https://journals.plos.org/plosgenetics/article?id=10.1371/journal.pgen.1008593).

      Reviewer #2 (Public Review):

      Summary:

      The authors sampled multiple populations of maize and teosinte across Mexico, aiming to characterise the geographic scale of local adaptation, patterns of selective sweeps, and modes of convergent evolution between populations and subspecies.

      Strengths & Weaknesses:

      The population genomic methods are standard and appropriate, including Fst, Tajima's D, α, and selective sweep scans. The whole genome sequencing data seems high quality. However, limitations exist regarding limited sampling, potential high false-positive sweep detection rates, and weak evidence for some conclusions, like the role of migration in teosinte adaptation.

      Aims & Conclusions:

      The results are interesting in supporting local adaptation at intermediate geographic scales, widespread convergence between populations, and standing variation/gene flow facilitating adaptation. However, more rigorous assessments of method performance would strengthen confidence. Connecting genetic patterns to phenotypic differences would also help validate associations with local adaptation.

      Impact & Utility:

      This work provides some of the first genomic insights into local adaptation and convergence in maize and teosinte. However, the limited sampling and need for better method validation currently temper the utility and impact. Broader sampling and connecting results to phenotypes would make this a more impactful study and valuable resource. The population genomic data itself provides a helpful resource for the community.

      Additional Context:

      Previous work has found population structure and phenotypic differences consistent with local adaptation in maize and teosinte. However, genomic insights have been lacking. This paper takes initial steps to characterise genomic patterns but is limited by sampling and validation. Additional work building on this foundation could contribute to understanding local adaptation in these agriculturally vital species.

      We appreciate the reviewer’s thoughtful reading of the paper and scrutiny. We hope that the added caveats made in response to reviewer 1 (as well as the previous rounds of peer review) will provide readers with the proper amount of skepticism in the accuracy of some of our initial sweep results, while also demonstrating that many of our conclusions are robust to the concerns raised over the various stages of review.

      We agree with the reviewer that better sampling and the incorporation inference about phenotypic data would be excellent additions, but the information is not available for the studied populations, and is outside scope of this paper.

      Recommendations for the authors:

      Reviewer #1 (Recommendations For The Authors):

      - Sometimes alpha is described as a rate, and sometimes as a proportion. The latter is correct.

      We have updated this. Thanks.

      - Line 79: are they really "discrete" populations?

      The teosinte populations sampled are all clearly separated from each other and are physically discrete. The maize population samples came from individual farmer fields. Traditional maize is grown as open-pollinated (outcrossing) populations, and farmers save seed for subsequent generations. An individual farmer’s field thus behaves as a discrete population for our purposes, impacted of course by gene flow, selection, and other evolutionary processes.

      - Lines 418-420: "Large genomes may lead to more soft sweeps, where no single mutation driving adaptive evolution would fix (Mei et al. 2018)." I'm not sure I understand this statement. Why is this a property of genome size?

      Mei et al. 2018 lay out the logic, but essentially they present data arguing that the total number of functionally relevant base pairs increases with genome size (less than linearly). If true, genomes with a large number of potentially functional bp are more likely to undergo soft sweeps (see theory by Hermisson and Pennings cited in Mei et al. 2018).

      - Lines 500-1: selection does not cause one to underestimate effective population sizes. Selection directly affects Ne. I'm not sure what biases the sentences on lines 502-508 are trying to explain.

      We have simplified this section. Not accounting for linked selection (especially positive selection) results in a biased inference of demographic history. See Marsh and Johri (2024) for another example. https://doi.org/10.1093/molbev/msae118

      - Line 511-3: does Uricchio et al. (2019) show any difference in the estimate of alpha from Messer and Petrov (2013) when taking background selection into account?

      What we initially wrote was incorrect. The aMK method of Messer and Petrov (2013) accounts for weakly deleterious polymorphisms, but it does not account for positively selected ones. We have updated this text and suggested our method may underestimate alpha if positively selected segregating alleles are common (near line 539).

      - Lines 598-599: "which would limit the rate of new and beneficial mutations." I don't understand this - shouldn't a bottleneck only affect standing variation? Why would a bottleneck affect new mutations?

      This is simply to say that during the low Ne period of a bottleneck, fewer total mutations (and therefore beneficial mutations) will be generated since there are fewer individuals for mutations to occur in. We have changed “rate” to amount to clarify we do not mean the mutation rate itself.

      Reviewer #2 (Recommendations For The Authors):

      Experiments/Analyses:

      (1) Consider simulating polygenic adaptation in addition to hard and soft sweeps to see if this improves the power to detect adaptive signatures shared between populations. This could involve simulating the coordinated change in allele frequencies across many loci to match a specified shift in trait value due to selection. The ability to detect shared polygenic adaptation between population replicates could be assessed using methods tailored to polygenic signals, such as the Polygenic Selection Score approach. Comparing the power to detect shared polygenic adaptation versus shared hard and soft sweeps would provide further insight into what adaptive modes current methods can uncover. If the power to detect shared polygenic adaptation is very low, the extent of shared adaptation between populations may be even more common than currently inferred. Adding simulations of polygenic adaptation would strengthen the study.

      While this would be a worthwhile undertaking in general, it would be a considerable amount of work outside of the scope and aims of this paper.

      (2) Explore using machine learning approaches like S/HIC to improve power over summary statistic methods potentially.

      We in fact put considerable effort into applying diplo S/HIC before switching to raisd for this project. While predictions on simulations had good power to detect sweeps, we found that applying to our actual data had a dubious number of windows classified as sweeps (e.g. >90% of the genome), which we believed to be false positives. We speculated that this may have to do with sensitivity to demographic or other types of misspecification in the simulations, such as our choice of window sizes compared to local recombination rates. It would likely be fruitful to our further efforts into using machine learning methods for maize and teosinte, but a deeper exploration of the right hyper parameters and simulation choices is likely needed to apply them effectively.

      (3) Increase geographic sampling density, if possible, especially near population pairs showing high differentiation, to better understand the scale of local adaptation.

      We agree this would be valuable research. Hopefully this work inspires further efforts into the question of the spatial and temporal scales of local adaptation with more ambitious spatial sampling designed at the onset

      Writing/Presentation:

      (1) Provide more intuition about the biological interpretation of the migration rates inferred under the migration model of convergence. What do the rates imply about the amount or timing of gene flow?

      We have expanded the discussion sections (starting near line 653) to elaborate on the migration results and connect the rdmc and f4 tests more explicitly. The timing of gene flow is more challenging to address directly with the approaches we used, but we agree it would be interesting to explore more in future papers.

      (2a) Expand the discussion of power limitations and the need for simulation tests. Consider adding ROC curves for sweep detection on simulated data. The relatively low proportion of shared selective sweeps between population replicates highlights limitations in the power to detect sweeps, especially incomplete or soft sweeps. I think it would be a good idea to expand the discussion of the power tradeoffs shown in the simulation analyses. In particular, the ROC curves in Figure S4 clearly show how power declines for weaker selection coefficients across the different sweep types. I suggest making these ROC curves part of the main figures to feature the issue of power limitations more prominently.

      (2b) The discussion would benefit from commenting on how power changes across the sweep simulation scenarios. Adding a summary figure to visualise the effects of sweep type, selection strength, and frequency on detectability could further clarify the power constraints. Stating the proportion of sweeps likely missed strengthens the argument that sharing adaptive alleles is likely even more common than inferred. Discussing power will also motivate the need for developing methods with improved abilities to uncover incomplete and soft sweeps.

      While these are useful suggestions (2a and 2b), the aim of this paper at its core is empirical, and was not intended to give an exhaustive analysis of the power to detect sweeps. We report what parts of the analysis may be impacted by low power and what aspects of our inferences have higher uncertainty due to power. We agree that there is more work to be done to improve methods to detect selection given our findings (see below concerning our efforts to use machine learning as well). While we do not highlight this in the paper, we also note that ours is one of extremely few empirical studies that actually perform power analyses on real data (as opposed to simulations). We think this extra transparency by itself is of substantial utility to the community in demonstrating that the results from simulation studies performed in publications describing a method do not necessarily translate well to empirical data.

      (3) Improve clarity in describing f4 test results. Consider visualising results on a map to show spatial patterns.

      We have expanded the discussion concerning f4 tests (see several comments to reviewer 1). We are not clear on how to effectively visualize f4 spatially, but hope the updates have made the results more clear.

      Minor:

      -  Increase the font size of figure axis labels for improved readability.

      We have looked over and figures and increased font sizes where possible.

      -  Add units to selection coefficient axis labels in Figure 5.

      Selection coefficients are derived in Lee and Coop (2017) from classical population genetics theory. They do not have units, but denote the relative fitness advantage of the heterozygous genotype carrying the beneficial mutation of interest.

      -  Fix the typo 'cophenetic' in Figure S3 caption.

      Fixed. Thank you.

    1. Author response:

      The following is the authors’ response to the original reviews.

      Public Reviews:

      Reviewer #1 (Public review):

      Summary:

      This study provides convincing evidence on the infraslow oscillation of DG cells during NREM sleep, and how serotonergic innervation modulates hippocampal activity pattern during sleep and memory.

      Strengths and Weaknesses:

      The authors used state-of-the-art techniques to carry out these experiments. Given that the functional role of infraslow rhythm still remains to be studied, this study provides convincing evidence of the role of DG cells in regulating infraslow rhythm, sleep microarchitecture, and memory.

      I have a few minor comments.

      (1) Decreased infraslow rhythm during NREMs in the 5ht1a KO mice is striking. It would be helpful to know whether sleep-wake states, MAs, and transitions to REMs are changed.

      We agree with the reviewer that serotonin receptors may be involved in sleep regulation therefore it is important to analyze the effect of their manipulation. We would also like to bring to the attention of the reviewer that in this case we restricted the 5ht1a manipulation to the hippocampus which does not have a known impact on sleep-wake regulation. The analysis of our recorded dataset from these mice confirmed this notion, because we did not see any changes in sleep metrics (see: supplementary figure 6A).

      (2) It would be interesting to discuss whether the magnitude in changes of infraslow rhythm strength is correlated with memory performance (Figure 6).

      We agree with the reviewer that this could be an interesting point. In our experiments we wanted to minimize the impact of the surgical procedures on the behavior, thus we used separate cohorts to record the photometry and to carry out the behavior experiments, therefore we are unable to correlate behavior and infraslow oscillatory amplitudes in our dataset.

      However, a similar experiment was carried out in a recent paper where the authors discovered that the norepinephrine system also displays infraslow oscillatory cycles during NREM sleep (Kjaerby et al 2022). The authors of that paper gradually decreased the magnitude of the NE pulses during NREM by optogenetic manipulation of the locus coeruleus which led to a fragmented sleep phenotype characterized by increased micro arousal occurrence, decreased REM and reduced spindle activity. They also tested the memory performance of the mice in a novel object recognition task and found diminished performance level in the opto group. Serotonin has multiple roles in the brain, many of them show overlap with proposed functions of the noradrenergic system including regulation of plasticity, signaling reward or fearful stimuli. Therefore, we speculate that the modification of serotonin dynamics during sleep will most likely interfere with memory performance.

      We inserted this paragraph in the discussion part of our paper.

      (3) The authors should cite the Oikonomou Neuron paper that describes slow oscillatory activity of DRN SERT neurons during NREM sleep.

      Thank you for the suggestion, we inserted this paper in the manuscript.

      (4) The authors should clarify how they define the phasic pattern of the photometry signal.

      We have added the details in the Methods.

      Reviewer #2 (Public review):

      Summary:

      The authors investigated DG neuronal activity at the population and single-cell level across sleep/wake periods. They found an infraslow oscillation (0.01-0.03 Hz) in both granule cells (GC) and mossy cells (MC) during NREM sleep.

      The important findings are:

      (1) The antiparallel temporal dynamics of DG neuron activities and serotonin neuron activities/extracellular serotonin levels during NREM sleep, and

      (2) The GC Htr1a-mediated GC infraslow oscillation.

      Strengths:

      (1) The combination of polysomnography, Ca-fiber photometry, two-photon microscopy, and gene depletion is technically sound. The coincidence of microarousals and dips in DG population activity is convincing. The dip in activity in upregulated cells is responsible for the dip at the population level.

      (2) DG GCs express excitatory Htr4 and Htr7 in addition to inhibitory Htr1a, but deletion of Htr1a is sufficient to disrupt DG GC infraslow oscillation, supporting the importance of Htr1a in DG activity during NREM sleep.

      Weaknesses:

      (1) The current data set and analysis are insufficient to interpret the observation correctly.

      a. In Figure 1A, during NREM, the peaks and troughs of GC population activities seem to gradually decrease over time. Please address this point.

      Thank you for the suggestion. We have analyzed and compared the magnitude of the oscillatory signals in the first and last minute of the NREM sleep epochs in Dock10-Cre mice and found no significant difference. However, we did observe that the ISO amplitude is smaller in the early stage of the first NREM epochs, defined as those with the prior wakefulness longer than 5 minutes (new supplementary figure 1).

      b. In Figure 1F, about 30% of Ca dips coincided with MA (EMG increase) and 60% of Ca dips did not coincide with EMG increase. If this is true, the readers can find 8 Ca dips which are not associated with MAs from Figure 1E. If MAs were clustered, please describe this properly.

      We did not find evidence that MAs were clustered in our dataset (see a representative example in supplementary figure 1A). We replaced the example trace with a new one which shows calcium dips with and without MAs. We believe this new trace better represents the data.

      c. In Figure 1F, the legend stated the percentage during NREM. If the authors want to include the percentage of wake and REM, please show the traces with Ca dips during wake and REM. This concern applies to all pie charts provided by the authors.

      Figure 1F (and all other pie charts) shows the outcome of brain states following a calcium-dip episode. That is, we found that the Ca-dips during NREM were followed by MAs in 30% of the cases, 59% of the Ca-dips led to the maintenance of NREM (no MAs) while in 2% and 9% of the cases we detected either REM state or wakening of the animal. These numbers correspond very well with similar analysis done in a recent paper which looked at the infraslow oscillatory behavior of the norepinephrine system (Kjaerby et al 2022) during NREM sleep. We apologize if the wording in the manuscript was misleading, we modified the figure legends to clarify what the pie charts represent. 

      d. In Figure 1C, please provide line plots connecting the same session. This request applies to all related figures.

      We have replaced the dot plots in all related figures with the line plots. 

      e. In Figure 2C, the significant increase during REM and the same level during NREM are not convincing. In Figure 2A, the several EMG increasing bouts do not appear to be MA, but rather wakefulness, because the duration of the EMG increase is greater than 15 seconds. Therefore, it is possible that the wake bouts were mixed with NREM bouts, leading to the decrease of Ca activity during NREM. In fact, In Figure 2E, the 4th MA bout seems to be the wake bout because the EMG increase lasts more than 15 seconds.

      We have replaced the Figure 2C with line plots as suggested above. It is clear that MC activity during REM sleep is higher, compared to that in NREM sleep, whereas the overall difference between wake and NREM is not significant (some increased, some decreased). Regarding the MAs, we have added a trace of averaged EMG signals in Figure 2G, showing that the averaged EMG bursts during MA are shorter than 5 seconds.

      f. Figure 5D REM data are interesting because the DRN activity is stably silenced during REM. The varied correlation means the varied DG activity during REM. The authors need to address it.

      We thank the reviewer for this suggestion. We have added this point to the discussion. We speculate that inputs from the supramammillary nucleus or entorhinal cortex to the DG during REM sleep may both contribute to this variability.

      g. In Figure 6, the authors should show the impact of DG Htr1a knockdown on sleep/wake structure including the frequency of MAs. I agree with the impact of Htr1a on DG ISO, but possible changes in sleep bout may induce the DG ISO disturbance.

      As suggested, we have performed sleep analysis in the Htr1a knockdown experiments including MA quantification. We have found no significant difference between Hrt1-knockdown and control mice in any of the sleep metrics (see: supplemental figure 6). Our interpretation is that the lack of changes in sleep/wake cycles is likely due to the hippocampus not being directly involved in regulating these brain states.

      (2) It is acceptable that DG Htr1a KO induces the reduced freezing in the CFC test (Figure 6E, F), but it is too much of a stretch that the disruption of DG ISO causes impaired fear memory. There should be a correlation.

      We have modified the discussion accordingly.

      (3) It is necessary to describe the extent of AAV-Cre infection. The authors injected AAV into the dorsal DG (AP -1.9 mm), but the histology shows the ventral DG (Supplementary Figure 4), which reduces the reliability of this study.

      The histology image shown in the manuscript was taken from the -2.5 mm anteroposterior level, which we still consider to be part of the dorsal DG. For additional clarity, we have replaced the figure with new histology images slightly more anterior position (AP~2.0mm). 

      Reviewer #3 (Public review):

      Summary:

      The authors employ a series of well-conceived and well-executed experiments involving photometric imaging of the dentate gyrus and raphe nucleus, as well as cell-type specific genetic manipulations of serotonergic receptors that together serve to directly implicate serotonergic regulation of dentate gyrus (DG) granule (GC) and mossy cell (MC) activity in association with an infra slow oscillation (ISO) of neural activity has been previously linked to general cortical regulation during NREM sleep and microarousals.

      Strengths:

      There are a number of novel and important results, including the modulation of dentage granule cell activity by the infraslow oscillation during NREM sleep, the selective association of different subpopulations of granule cells to microarousals (MA), the anticorrelation of raphe activity with infraslow dentate activity.

      The discussion includes a general survey of ISOs and recent work relating to their expression in other brain areas and other potential neuromodulatory system involvement, as well as possible connections with infraslow oscillations, micro-arousals, and sensory sensitivity.

      Weaknesses:

      (1) The behavioral results showing contextual memory impairment resulting from 5-HT1a knockdown are fine but are over-interpreted. The term memory consolidation is used several times, as well as references to sleep-dependence. This is not what was tested. The receptor was knocked down, and then 2 weeks later animals were found to have fear conditioning deficits. They can certainly describe this result as indicating a connection between 5-HT1a receptor function and memory performance, but the connection to sleep and consolidation would just be speculation. The fact that 5-HT1a knockdown also impacted DG ISOs does not establish dependency. Some examples of this are:

      a. The final conclusion asserts "Together, our study highlights the role of neuromodulation in organizing neuronal activity during sleep and sleep-dependent brain functions, such as memory.". However, the reported memory effects (impairment of fear conditioning) were not shown to be explicitly sleep-dependent.

      We thank the reviewer for this comment. We have revised the sentence.

      b. Earlier in the discussion it mentions "Finally, we showed that local genetic ablation of 5-HT1a receptors in GCs impaired the ISO and memory consolidation". The effect shown was on general memory performance - consolidation was not specifically implicated.

      We have revised the sentence.

      (2) The assertion on page 9 that the results demonstrate "that the 5-HT is directly acting in the DG to gate the oscillations" is a bit strong given the magnitude of effect shown in Figure 6D, and the absence of demonstration of negative effect on cortical areas that also show ISO activity and could impact DG activity (see requested cortical sigma power analysis).

      We have revised the sentence.

      (3) Recent work has shown that abnormal DG GC activity can result from the use of the specific Ca indicator being used (GCaMP6s). (Teng, S., Wang, W., Wen, J.J.J. et al. Expression of GCaMP6s in the dentate gyrus induces tonic-clonic seizures. Sci Rep 14, 8104 (2024). https://doi.org/10.1038/s41598-024-58819-9). The authors of that study found that the effect seemed to be specific to GCaMP6s and that GCaMP6f did not lead to abnormal excitability. Note this is of particular concern given similar infraslow variation of cortical excitability in epilepsy (cf Vanhatalo et al. PNAS 2004). While I don't think that the experiments need to be repeated with a different indicator to address this concern, you should be able to use the 2p GCaMP7 experiments that have already been done to provide additional validation by repeating the analyses done for the GCaMP6s photometry experiments. This should be done anyway to allow appropriate comparison of the 2p and photometry results.

      We would like to thank the reviewer for this comment. We also analyzed the two-photon data in the same manner as the photometry data. However, the only supportive evidence that might be related to ISO in the two-photon data, recorded at the somatic level, was decreased fluorescence during MAs in the NREM-upregulated cell group (see Figure 3 D, E). We are unsure why this discrepancy exists, but we have discussed it in the manuscript and offered some alternative explanations. One hypothesis we are currently exploring relates to the different subcellular compartments sampled by the two imaging techniques. The photometry probe was implanted above the dentate gyrus, and since light collection efficiency declines sharply with distance from the probe tip (Pisano et al., 2019), we hypothesize that ISO is stronger at the dendritic level which directly receive the inputs from entorhinal cortex, and which is closest to the probe's tip. We are now conducting multiplane two-photon imaging experiments in our labs to test this hypothesis.

      (4) While the discussion mentions previous work that has linked ISOs during sleep with regulation of cortical oscillations in the sigma band, oddly no such analysis is performed in the current work even though it is presumably available and would be highly relevant to the interpretation of a number of primary results including the relationship between the ISOs and MAs observed in the DG and similar results reported in other areas, as well as the selective impact of DG 5-HT1a knockdown on DG ISOs. For example, in the initial results describing the cross-correlation of calcium activity and EMG/EEG with MA episodes (paragraph 1, page 4), similar results relating brief arousals to the infraslow fluctuation in sleep spindles (sigma band) have been reported also at .02 Hz associated with variation in sensory arousability (cf. Cardis et al., "Cortico-autonomic local arousals and heightened somatosensory arousability during NREMS of mice in neuropathic pain", eLife 2021). It would be important to know whether the current results show similar cortical sigma band correlations. Also, in the results on ISO attenuation following 5-HT1 knockdown on page 7 (Figure 6), how is cortical EEG affected? Is ISO still seen in EEG but attenuated in DG?

      Thank you for this valuable comment. We performed the analysis and found a positive correlation between cortical sigma band activity and DG activity during NREM sleep (see supplementary figure 1C-1E). Additionally, we conducted further analyses using the local 5-HT1a KO mouse model but did not observe significant changes in sleep architecture or MA frequency (see supplementary figure 6A). It is also important to note that ISO was only analyzed using calcium signals, not EEG signals. The standard filtering settings in our EEG data collection (0.5-500 Hz) do not allow us to analyze signals in such a low-frequency range.

      (5) The illustrations of the effect of 5-HT1a knockdown shown in Figure 6 are somewhat misleading. The examples in panels B and C show an effect that is much more dramatic than the overall effect shown in panel D. Panels B and C do not appear to be representative examples. Which of the sample points in panel D are illustrated in panels B and C? It is not appropriate to arbitrarily select two points from different animals for comparison, or worse, to take points from the extremes of the distributions. If the intent is to illustrate what the effect shown in D looks like in the raw data, then you need to select examples that reflect the means shown in panel D. It is also important to show the effect on cortical EEG, particularly in sigma band to see if the effects are restricted to the DG ISOs. It would also be helpful to show that MAs and their correlations as shown in Figure 1 or G as well as broader sleep architecture are not affected.

      We agree with the reviewer that the chosen example may appear somewhat exaggerated. However, we must point out that visually assessing missing or downregulated frequency components can be challenging. To provide a more objective presentation, we included Supplementary Figure 6B-C, in which we performed analysis similar to that in Fig1G in 5HT1a mice. These figures show a significant decrease in ISO amplitude, though the blockade is not complete, due to the incomplete nature of genetic manipulation with viral injection (see Suppl Fig 5). Furthermore, recent studies (Dong et al., 2023; Zhang et al., 2024; Kjaerby et al., 2022) have identified several other neuromodulatory and peptidergic systems that might affect DG activity during MAs.

      To explore this further, we conducted pharmacological experiments. We administered 8-hydroxy-DPAT, a 5-HT1a agonist (i.p. 1 mg/kg) in Dock10-Cre mice injected with AAV-FLEX-GcaMP6s in the DG. Since 5-HT1a receptors act as autoreceptors on raphe 5-HT neurons, this treatment effectively silences the serotonergic system, thereby “removing” 5-HT signaling from the brain. The results, shown in Author response image 1, indicate that pharmacological suppression of 5-HT dampens the ISO in the DG during subsequent sleep intervals, with ISO recovering after the drug is washed out. These findings are consistent with the results obtained with the more specific local genetic manipulation. We have not included this result in the manuscript because we believe that the local downregulation is a cleaner experiment whose interpretation is more straightforward.

      Author response image 1.

      Finally, we also performed sleep analysis in 5-HT1a KO mice, showing that the local downregulation of 5-HT1a receptors had no significant effect on sleep metrics (Suppl Fig 6A). The hippocampus is not typically involved in regulating sleep-wake cycles, so we believe this result is consistent with that understanding.

      (6) On page 9 of the results it states that GCs and MCs are upregulated during NREM and their activity is abruptly terminated by MAs through a 5-HT mediated mechanism. I didn't see anything showing the 5-HT dependence of the MA activity correlation. The results indicate a reduction in ISO modulation of GC activity but not the MA-correlated activity. I would like to see the equivalent of Figure 1,2 G panels with the 5-HT1a manipulation.

      We agree with the reviewer on this point. We did not conduct any pharmacological or genetic manipulation in 2-photon calcium imaging experiments. We have removed that statement. As for the suggested analysis, please see our explanation above (Suppl Fig 6B-C).

      Recommendations for the authors:

      Reviewer #2 (Recommendations for the authors):

      (1) Since the authors did not monitor DG neuronal activity with an electrophysiological tool, please rephrase the following sentence: "In this study, we investigated the neuronal activity of the dentate gyrus (DG) with electrophysiological and optical imaging tools during sleep-wake cycles." in the Abstract.

      We have rephrased the sentence as suggested.

      (2) Since the authors did not manipulate the serotonin release during sleep to investigate whether serotonin release modulates DG ISO, please edit the following sentence: "Further experiments revealed that the infraslow oscillation in the DG is modulated by rhythmic serotonin release during sleep" in the Abstract.

      We have rephrased the sentence as suggested.

      (3) Single-cell recording in DG with two-photon microscopy may address the issue raised in the 4th paragraph of the Discussion. In addition, in Fig 6C, the photometry has only captured the diminished oscillation in Htr1a KO, but cannot distinguish whether the activity levels of GC remain at high or low, which is a clear disadvantage of photometry.

      We agree with the reviewer, and have added text to the discussion.

      Reviewer #3 (Recommendations for the authors):

      (1) Some of the figures are missing labels in the spectrogram panels (e.g. no freq units in Figures 4 and 6).

      We have added information in those figures.

      (2) Missing specific locations for EEG electrodes/screws. The text states "we predrilled 2 holes on the right side of the skull (1.5 mm posterior of the Bregma) for implanting recording electrodes". 2 holes on the right side of the skull are pretty vague.

      We have added this information in the Methods.

      (3) Some additional work that could be cited particularly when discussing the serotonergic impact on hippocampal function as it might relate to sleep and memory would include work linking mesopontine activity (both serotonergic and non-serotonergic) to memory-associated hippocampal sharp-wave ripple activity (e.g. Jelitai et al. Front. Neural Circ. 2021, Wang et al Nat. Neuro. 2015).

      We have cited these papers.

      (4) The work cited at the beginning of the Results describing higher population calcium activity during sleep states (15,18,30) is generally appropriate but not explicitly related to GCamP imaging. Pilz et al. "Functional Imaging of Dentate Granule Cells in the Adult Mouse Hippocampus", J.Neurosci. 2016 might be a more relevant citation.

      We have added the citation.

    1. Author response:

      The following is the authors’ response to the original reviews.

      We thank the three reviewers for their positive comments and useful suggestions. We have implemented most of the reviewers’ recommendations and hope the manuscript is clearer now.

      The main modifications are:

      - A revision of the introduction to better explain what Transitional Probabilities are and clarify the rationale of the experimental design

      - A revision of the discussion

      - To tune down and better explain the interpretation of the different responses between duplets after a stream with phonetic or voice regularities (possibly an N400).

      - To better clarify the framing of statistical learning as a universal learning mechanism that might share computational principles across features (or domains).

      Below, we provide detailed answers to each reviewer's point.

      Response to Reviewer 1:

      There are no significant weaknesses to signal in the manuscript. However, in order to fully conclude that there is no obvious advantage for the linguistic dimension in neonates, it would have been most useful to test a third condition in which the two dimensions were pitted against each other, that is, in which they provide conflicting information as to the boundaries of the words comprised in the artificial language.

      This last condition would have allowed us to determine whether statistical learning weighs linguistic and non-linguistic features equally, or whether phonetic content is preferentially processed.

      We appreciate the reviewers' suggestion that a stream with conflicting information would provide valuable insights. In the present study, we started with a simpler case involving two orthogonal features (i.e., phonemes and voices), with one feature being informative and the other uninformative, and we found similar learning capacities for both. Future work should explore whether infants—and humans more broadly—can simultaneously track regularities in multiple speech features. However, creating a stream with two conflicting statistical structures is challenging. To use neural entrainment, the two features must lead to segmentation at different chunk sizes so that their effects lead to changes in power/PLV at different frequencies—for instance, using duplets for the voice dimension and triplets for the linguistic dimension (or vice versa). Consequently, the two dimensions would not be directly comparable within the same participant in terms of the number of distinguishable syllables/voices, memory demand, or SNR given the 1/F decrease in amplitude of background EEG activity. This would involve comparisons between two distinct groups counter-balancing chunk size and linguistic non-linguistic dimension. Considering the test phase, words for one dimension would have been part-words for the other dimension. As we are measuring differences and not preferences, interpreting the results would also have been difficult. Additionally, it may be difficult to find a sufficient number of clearly discriminable voices for such a design (triplets imply 12 voices). Therefore, an entirely different experimental paradigm would need to be developed.

      If such a design were tested, one possibility is that the regularities for the two dimensions are calculated in parallel, in line with the idea that the calculation of statistical regularities is a ubiquitous implicit mechanism (see Benjamin et al., 2024, for a proposed neural mechanism). Yet, similar to our present study, possibly only phonetic features would be used as word candidates. Another possibility is that only one informative feature would be explicitly processed at a time due to the serial nature of perceptual awareness, which may prioritise one feature over the other.

      We added one sentence in the discussion stating that more research is needed to understand whether infants can track both regularities simultaneously (p.13, l.270 “Future work could explore whether they can simultaneously track multiple regularities.”).

      Note: The reviewer’s summary contains a typo: syllabic rate (4 Hz) –not 2 Hz, and word rate (2 Hz) –not 4 Hz.

      Response to Reviewer 2:

      N400: I am skeptical regarding the interpretation of the phoneme-specific ERP effect as a precursor of the N400 and would suggest toning it down. While the authors are correct in that infant ERP components are typically slower and more posterior compared to adult components, and the observed pattern is hence consistent with an adult N400, at the same time, it could also be a lot of other things. On a functional level, I can't follow the author's argument as to why a violation in phoneme regularity should elicit an N400, since there is no evidence for any semantic processing involved. In sum, I think there is just not enough evidence from the present paradigm to confidently call it an N400.

      The reviewer is correct that we cannot definitively determine the type of processing reflected by the ERP component that appears when neonates hear a duplet after exposure to a stream with phonetic regularities. We interpreted this component as a precursor to the N400, based on prior findings in speech segmentation tasks without semantic content, where a ~400 ms component emerged when adult participants recognised pseudowords (Sander et al., 2002) or during structured streams of syllables (Cunillera et al., 2006, 2009). Additionally, the component we observed had a similar topography and timing to those labelled as N400 in infant studies, where semantic processing was involved (Parise et al., 2010; Friedrich & Friederici, 2011).

      Given our experimental design, the difference we observed must be related to the type of regularity during familiarisation (either phonemes or voices). Thus, we interpreted this component as reflecting lexical search— a process which could be triggered by a linguistic structure but which would not be relevant to a non-linguistic regularity such as voices. However, we are open to alternative interpretations. In any case, this difference between the two streams reveals that computing regularities based on phonemes versus voices does not lead to the same processes.

      We revised the abstract (p.2, l.33) and the discussion of this result (p.15, l.299), toning them down. We hope the rationale of the interpretation is clearer now, as is the fact that it is just one possible interpretation of the results.

      Female and male voices: Why did the authors choose to include male and female voices? While using both female and male stimuli of course leads to a higher generalizability, it also introduces a second dimension for one feature that is not present for this other (i.e., phoneme for Experiment 1 and voice identity plus gender for Experiment 2). Hence, couldn't it also be that the infants extracted the regularity with which one gender voice followed the other? For instance, in List B, in the words, one gender is always followed by the other (M-F or F-M), while in 2/3 of the part-words, the gender is repeated (F-F and M-M). Wouldn't you expect the same pattern of results if infants learned regularities based on gender rather than identity?

      We used three female and three male voices to maximise acoustic variability. The streams were synthesised using MBROLA, which provides a limited set of artificial voices. Indeed, there were not enough French voices of acceptable quality, so we also used two Italian voices (the phonemes used existed in both Italian and French).

      Voices differ in timbre, and female voices tend to be higher pitched. However, it is sometimes difficult to categorise low-pitched female voices and high-pitched male voices. Given that gender may be an important factor in infants' speech perception (newborns, for instance, prefer female voices at birth), we conducted tests to assess whether this dimension could have influenced our results.

      We report these analyses in SI and referred to them in the methods section (p.25, l.468 “We performed post-hoc tests to ensure that the results were not driven by a perception of two voices: female and male (see SI).”).

      We first quantified the transitional probabilities matrices during the structured stream of Experiment 2, considering that there are only two types of voices: Female and Male.

      For List A, all transition probabilities are equal to 0.5 (P(M|F), P(F|M), P(M|M), P(F|F)), resulting in flat TPs throughout the stream (see Author response image 1, top). Therefore, we would not expect neural entrainment at the word rate (2 Hz), nor would we anticipate ERP differences between the presented duplets in the test phase.

      For List B, P(M|F)=P(F|M)=0.66 while P(M|M)=P(F|F)=0.33. However, this does not produce a regular pattern of TP drops throughout the stream (see Author response image 1, bottom). As a result, strong neural entrainment at 2 Hz was unlikely, although some degree of entrainment might have occasionally occurred due to some drops occurring at a 2 Hz frequency. Regarding the test phase, all three Words and only one Part-word presented alternating patterns (TP=0.6). Therefore, the difference in the ERPs between Words and Part- words in List B might be attributed to gender alternation.

      However, it seems unlikely that gender alternation alone explains the entire pattern of results, as the effect is inconsistent and appears in only one of the lists. To rule out this possibility, we analysed the effects in each list separately.

      Author response image 1.

      Transition probabilities (TPs) across the structured stream in Experiment 2, considering voices processed by gender (Female or Male). Top: List A. Bottom: List B.

      We computed the mean activation within the time windows and electrodes of interest and compared the effects of word type and list using a two-way ANOVA. For the difference between Words and Part-words over the positive cluster, we observed a main effect of word type (F(1,31) = 5.902, p = 0.021), with no effects of list or interactions (p > 0.1). Over the negative cluster, we again observed a main effect of word type (F(1,31) = 10.916, p = 0.0016), with no effects of list or interactions (p > 0.1). See Author response image 2.

      Author response image 2:

      Difference in ERP voltage (Words – Part-words) for the two lists (A and B); W=Words; P=Part-Words,

      We conducted a similar analysis for neural entrainment during the structured stream on voices. A comparison of entrainment at 2 Hz between participants who completed List A and List B showed no significant differences (t(30) = -0.27, p = 0.79). A test against zero for each list indicated significant entrainment in both cases (List A: t(17) = 4.44, p = 0.00036; List B: t(13) = 3.16, p = 0.0075). See Author response image 3.

      Author response image 3.

      Neural entrainment at 2Hz during the structured stream of Experiment 2 for Lists A and B.

      Words entrainment over occipital electrodes: Do you have any idea why the duplet entrainment effect occurs over the electrodes it does, in particular over the occipital electrodes (which seems a bit unintuitive given that this is a purely auditory experiment with sleeping neonates).

      Neural entrainment might be considered as a succession of evoked response induced by the stream. After applying an average reference in high-density EEG recordings, the auditory ERP in neonates typically consists of a central positivity and a posterior negativity with a source located at the electrical zero in a single-dipole model (i.e. approximately in the superior temporal region (Dehaene-Lambertz & Dehaene, 1994). In adults, because of the average reference (i.e. the sum of voltages is equal to zero at each time point) and because the electrodes cannot capture the negative pole of the auditory response, the negativity is distributed around the head. In infants, however, the brain is higher within the skull, allowing for a more accurate recording of the negative pole of the auditory ERP (see Figure 4 for the location of electrodes in an infant head model).

      Besides the posterior electrodes, we can see some entrainment on more anterior electrodes that probably corresponds to the positive pole of the auditory ERP.

      We added a phrase in the discussion to explain why we can expect phase-locked activity in posterior electrodes (p.14, l.277: “Auditory ERPs, after reference-averaged, typically consist of a central positivity and posterior negativity”).

      Author response image 4:

      International 10–20 sensors' location on the skull of an infant template, with the underlying 3-D reconstruction of the grey-white matter interface and projection of each electrode to the cortex. Computed across 16 infants (from Kabdebon et al, Neuroimage, 2014). The O1, O2, T5, and T6 electrodes project lower than in adults.

      Response to Reviewer 3:

      (1) While it's true that voice is not essential for language (i.e., sign languages are implemented over gestures; the use of voices to produce non-linguistic sounds, like laughter), it is a feature of spoken languages. Thus I'm not sure if we can really consider this study as a comparison between linguistic and non-linguistic dimensions. In turn, I'm not sure that these results show that statistical learning at birth operates on non-linguistic features, being voices a linguistic dimension at least in spoken languages. I'd like to hear the authors' opinions on this.

      On one hand, it has been shown that statistical learning (SL) operates across multiple modalities and domains in human adults and animals. On the other hand, SL is considered essential for infants to begin parsing speech. Therefore, we aimed to investigate whether SL capacities at birth are more effective on linguistic dimensions of speech, potentially as a way to promote language learning.

      We agree with the reviewer that voices play an important role in communication (e.g., for identifying who is speaking); however, they do not contribute to language structure or meaning, and listeners are expected to normalize across voices to accurately perceive phonemes and words. Thus, voices are speech features but not linguistic features. Additionally, in natural speech, there are no abrupt voice changes within a word as in our experiment; instead, voice changes typically occur on a longer timescale and involve only a limited number of voices, such as in a dialogue. Therefore, computing regularities based on voice changes would not be useful in real-life language learning. We considered that contrasting syllables and voices was an elegant way to test SL beyond its linguistic dimension, as the experimental paradigm is identical in both experiments.

      We have rephrased the introduction to make this point clearer. See p.5, l.88-92: “To test this, we have taken advantage of the fact that syllables convey two important pieces of information for humans: what is being said and who is speaking, i.e. linguistic content and speaker’s identity. While statistical learning…”.

      Along the same line, in the Discussion section, the present results are interpreted within a theoretical framework showing statistical learning in auditory non-linguistic (string of tones, music) and visual domains as well as visual and other animal species. I'm not sure if that theoretical framework is the right fit for the present results.

      (2) I'm not sure whether the fact that we see parallel and independent tracking of statistics in the two dimensions of speech at birth indicates that newborns would be able to do so in all the other dimensions of the speech. If so, what other dimensions are the authors referring to?

      The reviewer is correct that demonstrating the universality of SL requires testing additional modalities and acoustic dimensions. However, we postulate that SL is grounded in a basic mechanism of long-term associative learning, as proposed in Benjamin et al. (2024), which relies on a slow decay in the representation of a given event. This simple mechanism, capable of operating on any representational output, accounts for many types of sequence learning reported in the literature (Benjamin et al., in preparation).

      We have revised the discussion to clarify this theoretical framework.

      In p.13, l.264: “This mechanism might be rooted in associative learning processes relying on the co- existence of event representations driven by slow activation decays (Benjamin et al., 2024). ”

      In p., l. 364: “Altogether, our results show that statistical learning works similarly on different speech features in human neonates with no clear advantage for computing linguistically relevant regularities in speech. This supports the idea that statistical learning is a general learning mechanism, probably operating on common computational principles across neural networks (Benjamin et al., 2024)…”.

      (3) Lines 341-345: Statistical learning is an evolutionary ancient learning mechanism but I do not think that the present results are showing it. This is a study on human neonates and adults, there are no other animal species involved therefore I do not see a connection with the evolutionary history of statistical learning. It would be much more interesting to make claims on the ontogeny (rather than philogeny) of statistical learning, and what regularities newborns are able to detect right after birth. I believe that this is one of the strengths of this work.

      We did not intend to make claims about the phylogeny of SL. Since SL appears to be a learning mechanism shared across species, we use it as a framework to suggest that SL may arise from general operational principles applicable to diverse neural networks. Thus, while it is highly useful for language acquisition, it is not specific to it.

      We have removed the sentence “Statistical learning is an evolutionary ancient learning mechanism.”, and replaced it by (p.18, l.364) “Altogether, our results show that statistical learning works similarly on different speech features in human neonates with no clear advantage for computing linguistically relevant regularities in speech.” We now emphasise in the discussion that infants compute regularities on both features and propose that SL might be a universal learning mechanism sharing computational principles (Benjamin et al., 2024) (see point 2).

      (4) The description of the stimuli in Lines 110-113 is a bit confusing. In Experiment 1, e.g., "pe" and "tu" are both uttered by the same voice, correct? ("random voice each time" is confusing). Whereas in Experiment 2, e.g., "pe" and "tu" are uttered by different voices, for example, "pe" by yellow voice and "tu" by red voice. If this is correct, then I recommend the authors to rephrase this section to make it more clear.

      To clarify, in Experiment 1, the voices were randomly assigned to each syllable, with the constraint that no voice was repeated consecutively. This means that syllables within the same word were spoken by different voices, and each syllable was heard with various voices throughout the stream. As a result, neonates had to retrieve the words based solely on syllabic patterns, without relying on consistent voice associations or specific voice relationships.

      In Experiment 2, the design was orthogonal: while the syllables were presented in a random order, the voices followed a structured pattern. Similar to Experiment 1, each syllable (e.g., “pe” and “tu”) was spoken by different voices. The key difference is that in Experiment 2, the structured regularities were applied to the voices rather than the syllables. In other words, the “green” voice was always followed by the “red” voice for example but uttered different syllables.

      We have revised the description of the stimuli and the legend of Figure 1 to clarify these important points.

      See p.6, l. 113: “The structure consisted of the random concatenation of three duplets (i.e., two-syllable units) defined only by one of the two dimensions. For example, in Experiment 1, one duplet could be petu with each syllable uttered by a random voice each time they appear in the stream (e.g pe is produced by voice1 and tu by voice6 in one instance and in another instance pe is produced by voice3 and tu by

      voice2). In contrast, in Experiment 2, one duplet could be the combination [voice1- voice6], each uttering randomly any of the syllables.”

      p.20, l. 390 (Figure 1 legend): “For example, the two syllables of the word “petu” were produced by different voices, which randomly changed at each presentation of the word (e.g. “yellow” voice and “green” voice for the first instance, “blue” and “purple” voice for the second instance, etc..). In Experiment 2, the statistical structure was based on voices (TPs alternated between 1 and 0.5), while the syllables changed randomly (uniform TPs of 0.2). For example, the “green” voice was always followed by the “red” voice, but they were randomly saying different syllables “boda” in the first instance, “tupe” in the second instance, etc... “

      (5) Line 114: the sentence "they should compute a 36 x 36 TPs matrix relating each acoustic signal, with TPs alternating between 1/6 within words and 1/12 between words" is confusing as it seems like there are different acoustic signals. Can the authors clarify this point?

      Thank you for highlighting this point. To clarify, our suggestion is that neonates might not track regularities between phonemes and voices as separate features. Instead, they may treat each syllable-voice combination as a distinct item—for example, "pe" spoken by the "yellow" voice is one item, while "pe" spoken by the "red" voice is another. Under this scenario, there would be a total of 36 unique items (6 syllables × 6 voices), and infants would need to track regularities between these 36 combinations.

      We have modified this sentence in the manuscript to make it clearer.

      See p.7, l. 120: “If infants at birth compute regularities based on a neural representation of the syllable as a whole, i.e. comprising both phonetic and voice content, this would require computing a 36 × 36 TPs matrix relating each token.”

      Reviewer #1 (Recommendations for the authors):

      (1) The acronym TP should be spelled out, and a brief description of the fact that dips in TPs signal boundaries while high TPs signal a cohesive unit could be useful for non-specialist readers.

      We have added it at the beginning of the introduction (lines 52-60)

      (2) p.5, l.76: "Here, we aimed to further characterise the characteristics of this mechanism...". I suggest this is rephrased as "to further characterise this mechanism".

      We have changed it as suggested by the reviewer (now p.5, l.81)

      (3) p.9, l.172: "[...] this contribution is unlikely since the electrodes differ from the electrodes, showing enhanced word-rate activity at 2 Hz."

      It is unclear which electrodes differ from which electrodes. I figure that the authors mean that the electrodes showing stronger activity at 2 Hz differ from those showing it at 4 Hz, but the sentence could use rephrasing.

      This part has been rephrased (p.9, l.177-181)

      (4) p.10, l.182: "[...] the entrainment during the first minute of the structure stream [… ]".

      Structured stream.

      It has been corrected (p.10, l.190)

      (5) p.12, l.234: "we compared STATISTICAL LEARNING"

      Why the use of capitals?

      This was an error and it was corrected (p.12, l.242).

      (6) p.15, l.298: "[...] suggesting that such negativity might be related to semantic."

      The sentence feels incomplete. To semantics? To the processing of semantic information?

      The phrase has been corrected (p.15, l.314). Additionally, the discussion of the posterior negativity observed for duplets after familiarisation with a stream with regularities over phonemes has been rephrased (p.15, l.)

      (7) Same page, l.301: "3-mo-olds" 3-month-olds.

      It has been corrected (now in p.16, l.333)

      (8) Same page, l.307: "(see also (Bergelson and Aslin, 2017)" (see also Bergelson and Aslin, 2017).

      It has been corrected (now in p.17, l.340)

      (9) Same page, l.310: "[...] would be considered as possible candidate" As possible candidates.

      This has been rephrased and corrected (now in p.17, l.343)

      Reviewer #2 (Recommendations for the authors):

      (1) Figure 2: The authors mention a "thick orange line", which I think should be a "thick black line".

      We are sorry for this. It has been corrected.

      (2) Ln 166: Should be Figure 2C rather than 3C.

      It has been corrected (now in p.9, l.173)

      (3) Figure 4 is not referenced in the manuscript.

      We referred to it now on p. 12, l.236

    1. Author response:

      The following is the authors’ response to the original reviews.

      Public Reviews:

      Reviewer #1 (Public Review):

      The authors set out to define the molecular basis for LP as the origin of BRCA1deficient breast cancers. They showed that LPs have the highest level of replicative stress, and hypothesise that this may account for their tendency to transform. They went on to identify ELF3 as a candidate driver of LP transformation and showed that ELF3 expression is up-regulated in response to replicative stress as well as BRCA1 deficiency. They went on to show that ELF3 inactivation led to a higher level of DNA damage, which may result from compromised replicative stress responses.

      While the manuscript supports the interesting idea wherein ELF3 may fuel LP cell transformation, it remains obscure how ELF3 promotes cell tolerance to DNA damage. Interestingly the authors proposed that ELF3 suppresses excessive genomic instability, but in my opinion, I do not see any evidence that supports this claim. In fact, one might think that genomic instability is key to cell transformation.

      We greatly appreciate your thorough review and insightful comments on our manuscript. We have taken your feedback seriously and have made several key revisions to address your concerns.

      To your primary point about how ELF3 helps cells tolerate DNA damage, we have expanded our discussion to clarify the role of ELF3 in the context of BRCA1 deficiency and high replicative stress. We clarified that while ELF3 may not directly suppress excessive genomic instability, it plays a role in maintaining a balance that prevents catastrophic damage in BRCA1-deficient cells. Both BRCA1 deficiency and increased replication stress induce up-regulation of ELF3, which acts as a transcription factor, and it’s up-regulation leads to up-regulation of the expression of a variety of DNA replication-associated proteins that help to maintain homeostasis in the DNA replication process (Figure 5 E and F). Defects in ELF3 also do lead to disruption of the DNA replication process (Figure 5 G-I). While ELF3 cannot completely eliminate genomic instability, ELF3 essentially maintains genomic instability within a dangerous yet non-lethal range: higher than in normal cells, but not so high as to cause cell death.

      This precarious balance can facilitate the transformation of LPs into a malignant state, as you pointed out.

      In the revised manuscript, we emphasized that in cells with inherently low replicative stress, such as other non-LP mammary cells, the ELF3-associated mechanism might help cells endure the high replicative stress caused by BRCA1 deficiency without leading to cancerous changes. However, in LP cells, which naturally experience higher replicative stress, this ELF3-related mechanism may make them more susceptible to transformation into cancer cells. This supports our hypothesis that the combination of high replicative stress and BRCA1 deficiency specifically predisposes LP cells to tumorigenesis.

      We have modified the working model to make it clearer.

      Reviewer #2 (Public Review):

      Summary:

      The manuscript focuses on a persistent question of why germline mutations in BRCA1 which impair homology-directed repair of DNA double-strand breaks predispose to primarily breast and ovarian cancers but not other tissues. The authors propose that replication stress is elevated in the luminal progenitor (LP) cells and apply the gene signature from Dreyer et al as a measure of replication stress in populations of cells selected by FACS previously (published by Lim et al.) and suggest an enrichment of replication stress among the LP cells. This is followed by single-cell RNA seq data from a small number of breast tissues from a small number of BRCA1 mutation carriers but the pathogenic variants are not listed. The authors perform an elegant analysis of the effects of BRCA1 knockdown in MCF10A cells, but these cells are not considered a model of LP cells.

      Overall, the manuscript suffers from significant gaps and leaps in logic among the datasets used. The connection to luminal progenitor cells is not adequately established because the models used are not representative of this population of cells. Therefore, the central hypothesis is not sufficiently justified.

      Strengths:

      The inducible knockdown of BRCA1 provided compelling data pointing to an upregulation of ELF3 in this setting as well as a small number of other genes. It would be useful to discuss the other genes for completeness and explain the logic for focusing on ELF3. Nonetheless, the connection with ELF 3 is reasonable. The authors provide significant data showing a role for ELF3 in breast epithelial cells and its role in cell survival.

      Weaknesses:

      The initial observations in primary breast cells have small sample sizes. The mutations in BRCA1 seem to be presumed to be all the same, but we know that pathogenic variants differ among individuals and range from missense mutations affecting interactions with one critical partner to large-scale truncations of the protein.

      The figure legends are missing critical details that make it difficult for the reader to evaluate the data. The data support the notion that ELF3 may participate in relieving replication stress, but does not appear to be limited to LP cells as proposed in the hypothesis.

      We would like to sincerely thank you for your thorough review and constructive feedback on our manuscript. Your insightful comments and suggestions have been invaluable in guiding our revisions.

      (1) Acknowledgment of Data Set Limitations and Additional Analyses:    We fully acknowledge the importance of the concerns raised regarding the datasets used in our study. We have supplemented our manuscript with the missing information you pointed out and conducted additional analyses as suggested. These efforts have

      (2) Challenges in LP Cell Experiments:

      One of the most critical issues you raised was the lack of validation in LP cells, particularly concerning the role of ELF3 in these cells. We are acutely aware of the significance of this point. Following your review, we made extensive efforts to isolate and culture LP cells from both BRCA1-proficient and BRCA1-deficient patient samples. We tried various methods and invested substantial resources, including time, manpower, and materials, to establish a reliable protocol for isolating and cultivating LP cells in vitro. Unfortunately, despite our best efforts, we were unable to obtain a sufficient number of high-quality cells to generate solid and reproducible results.

      The challenges we faced included the limited availability of patient tissues and the technical difficulties in consistently obtaining viable LP cells. Given the already extended timeline for the revision of this manuscript, we regretfully decided to forgo further attempts to perform these critical experiments with LP cells. In the revised manuscript, we have explicitly addressed the limitations of our cell models and provided a detailed discussion of the challenges faced in isolating LP cells. Despite these limitations, we believe that the consistency between our results and LP cell sequencing data provides valuable insights and a solid foundation for future studies.

      (3) Data Presentation Improvements:

      In response to your feedback, we have also made significant improvements to the data presentation in our manuscript. We updated and optimized figure legends and narrative sections to ensure that the data are clearly and accurately conveyed. These changes aim to enhance the readability and comprehensibility of our findings.

      We greatly appreciate your valuable feedback, which has significantly contributed to the improvement of our manuscript. Your suggestions have helped us refine our arguments and present a more robust and nuanced interpretation of our data. 

      Thank you once again for your critical and constructive review. We look forward to your feedback on our revised manuscript.

      Recommendations for the authors:  

      Reviewer #1 (Recommendations For The Authors):  

      As such, in addition to consolidating the role of ELF3 in promoting cell tolerance to replicative stress (or in suppressing genomic instability), I have a few comments the authors should consider to improve their manuscript.  

      (1) I am not sure how cells have gained a growth advantage if they were arrested (Line 105-106). Perhaps the authors can elaborate.

      Thanks for pointing this out and we are sorry for the misleading statement. We have revised the manuscript and would like to clarify that “survival advantage” may be more accurate than “growth advantage”, and since long-term DOX treatment led to decreased cell survival indicated by decreased number of colonies in Supplemental Fig. S1D, thus many cells died during DOX treatment. Therefore, the cells able to survive throughout DOX treatment and being collected for sequencing may have gained survival advantage compared to their counterparts who fail to survive.  

      (2) Figure 3D - From Western blotting of ELF3, forced expression of E2F6 does not appear to "block" HU-induced ELF3 up-regulation, but merely down-regulate basal level of ELF3, with the effect of HU still notable.

      Thanks for the comment and we agree that E2F6 down-regulate ELF3 baseline expression levels and did not fully block ELF3 up-regulation. After calculating the foldchange after E2F6 overexpression, we did confirm that E2F6 overexpression still partially block HU-induced ELF3 up-regulation, with foldchange from 3.32 to 2.40, supporting our conclusion that HU-induced ELF3 upregulation is regulated by ATRChk1-E2F axis. It does, however, cannot be excluded that E2F6 also regulates ELF3 expression in other replication stress-independent ways, and we have revised the manuscript accordingly. 

      (3) Figure 3J & K - In my opinion, if BRCA1 knockdown were more efficient it remains formally possible that co-depletion of BRCA1 and GATA3 may exhibit additive effects in up-regulating ELF3 mRNA level.

      Thank you for the comment. Actually, the BRCA1 knockdown efficiency in Figure 3J was shown in Supplemental Fig. S3B, and notably both BRCA1 and GATA3 knockdown were numerically more efficient in the double-knockdown group than in the single-knockdown group, individually. Thus, the higher ELF3 up-regulation in double-knockdown group in Figure 3J could be cause by the superior knockdown efficiency of both BRCA1 and GATA3. Nonetheless, we agree that it might be possible that BRCA1 and GATA3 still have separate functions in this experimental setting and marginal additive effect may exist, and the manuscript was revised accordingly.

      (4) Figure 4 - Perhaps the authors can change its title to better summarise the findings. Cell sensitivity assays and xenograph experimentations may not necessarily relate to genomic instability.

      Thank you for the great suggestion. To summarize the results more accurately, we have revised the title as “ELF3 can help cells tolerate replication stress and sustain cell survival”.

      (5) Figure 5B&C - It would be important to document the time-dependent resolution of HU-induced DNA lesions by including additional time-points before, during, and after HU treatment.

      We appreciate the suggestion to include additional time points to document the timedependent resolution of HU-induced DNA lesions. In our experiments, we observed that ELF3 knockdown leads to genomic instability both in the presence and absence of HU treatment. Specifically, Figure 5A and Figure S5 demonstrate that ELF3 knockdown increases genomic instability without HU treatment, indicating its role in maintaining genomic stability under normal conditions. On the other hand, Figure 5B, 5C, and 5D show that ELF3 knockdown under HU-induced replication stress further exacerbates genomic instability. This observation aligns with our finding that ELF3 expression increases in response to replication stress, suggesting its critical role in maintaining replication homeostasis under such conditions. 

      6) Figure 5F&I - Which ELF3 siRNA was used in these experimentations? Since the authors did not exclude off-target effects perhaps it may be worthwhile to include both ELF3 siRNAs for Panel F.

      Thanks for your advice. The qPCR (Figure 5F) and DNA fiber assay (Figure 5I) were using siELF3-4 siRNA. And we repeat the qPCR experiments for Panel F using siELF3-5 siRNA (Supplement Fig. S5B).

      We sincerely thank you for your thoughtful feedback and constructive suggestions. Addressing these points has strengthened our manuscript, and we are grateful for the opportunity to refine and clarify our work. We appreciate your critical evaluation and look forward to further constructive dialogue.

      Reviewer #2 (Recommendations For The Authors):  

      (1) The data driving the hypothesis uses gene expression signatures as an indirect measure of replication stress. This is a critical concern.

      a. At this time, numerous gene expression signatures have been reported to be biomarkers of replication stress. Therefore, it would be valuable to apply additional gene expression signatures to examine the performance and the overlap in the results.

      The recent work by Takahashi et al., 2022 (https://pubmed.ncbi.nlm.nih.gov/36381660/) provides a signature that was derived independently and offers one that can be used to assess the performance of the signatures and stability of the conclusions.

      Thank you for the valuable suggestion. We have done the replication stress evaluation of mammary cell subgroups using the Repstress score developed in the work you mentioned. The result showed that LP cells have trends of higher replication stress compared with other subgroups, though no statistical significance. This result, consistent with our previous analysis, indicated that LP cells have higher trends of replication stress levels. And we have added this data as the last line of Figure 1A in revised vision.

      Author response image 1.

      Replication stress pathway scores of different human normal mammary cell  populations. The gene expression data were from Lim et al. (3).

      b. A direct measure of replication stress in LP cells would be important to confirm the gene expression signature. Therefore, performing immunostaining for markers of replication stress (eg gamma-H2AX foci, DNA fiber assays) would provide more direct data to support the assertions.

      Thank you for this suggestion and we totally agree that experiments revealing replication stress levels by investigating common markers, e.g., gamma-H2AX foci, DNA fiber assays, will provide vital evidence for our hypothesis. However, since our last response, we have been diligently trying to obtain LP cells for these experiments but encountered technical challenges while attempting to isolate and culture LP cells in vitro. 

      In the discussion part, we have revised the manuscript to emphasize that the data obtained from MCF10A should be interpreted with caution and there are certain gaps between the cell models and LP cells.

      (2) The depth of single-cell sequencing can often be limiting. Therefore, a supplementary table listing the genes used for the replication stress signature and the frequency that they are observed in the single-cell sequencing data. This is needed to ensure that the replication stress score does not reflect a small subset of the replication stress signature genes.

      Thanks very much for this evaluable suggestion. We have provided an expression matrix of genes for the replication stress signature in the revised version (Supplementary Table S1), And we also calculated the average expression level of each gene in the cells. As shown in Author response image 2, these genes expressed relatively low at the single-cell level (with counts≤10), The expression differences among genes are relatively small. Thus, we excluded the possibility that several high-expressed genes significantly affect the replicative stress score.

      Author response image 2.

      Average counts of Top 50 genes for the replication stress signature

      (3) As only 4 BRCA mutation carriers are analyzed, it is critical that the mutations be reported for these individuals because pathogenic variants differ in their effects and interactions with the DNA repair machinery in cells.

      Thanks for the suggestion and the information of 4 BRCA1 mutant carriers were added in Supplemental Table S2.

      (4) The figures throughout lack critical details making it difficult to evaluate. Figure 1A states that these are "replication stress pathway scores..." but there is no evaluation of levels of statistical differences. The heat map has what appears to be a log unit score between +2 and -2 but it is unclear whether it is log2 or log10 or some other unit. In 1B, the replication stress scores are visualized as relative values between 0 and 0.1, but there is no indication of what this means or whether there is a statistically significant difference in the levels among the populations. As tumors are composed of multiple cell types, it should be stated how the "tumor cells" are uniquely identified in the figure legend. The lack of critical information is common across many of the figures making review frustratingly difficult.

      Thanks for the suggestion. We have added the statistical analysis and scale in Figure 1A legend. For Figure 1B, replication stress was calculated by sum of replication stress gene expression and presented as ln value. We have provided a quantitative figure and statistical tests (by Mann-Whitney) of replication stress scores for various cell types (Supplementary Figure 1A). 

      In addition, we added details of identification of tumor cells in the method section in the revised manuscript. Briefly, the adjacent normal breast sample served as a control to filter various types of normal cells from tumor samples. the normal cells from the tumor sample were merged with the same types of normal cells from adjacent normal breast samples, leaving one cell cluster only generalized by tumor sample. These tumor specific clusters were considered as malignant cell populations. We further found that the malignant cell population showed higher UMI counts than the normal cell populations, consistent with active metabolism in the malignant cells. More importantly, ER, PR, and HER2 expression of the malignant cells in each case were exactly matched with the clinical records. Finally, we utilized InferCNV to validate malignant cells subset as higher copy number alterations (CNAs) detected in the malignant cells compared with normal cells.

      (5) The hypothesis states that the LP cells are uniquely sensitive to deficiency in BRCA1 compared to other cells. However, the authors use knockdown of BRCA1 in MCF10A cells which are generally considered to be basal cells and not LP cells.

      Thanks for the comment. We totally agree that MCF10A cannot reflect the LP features and was mainly used as a normal mammary cell line model. We have tried to obtain human LP to perform some experiments but have all failed due to the cell vulnerability and difficult to be passed on in vitro. The gap between MCF10A and LP cells was stressed in the discussion part.

      (6) Figure 2, the number of samples being compared is not listed for most of the panels. It appears that ELF3 is enriched in subsets of breast cancers, but much of the data is not focused on BRCA1-deficient tumors. Therefore, the data appears to show that ELF3 expression is more of a generalized feature of TNBCs (which has been reported previously) and dilutes the support for the hypothesis. Therefore, panels C-G raise concerns regarding the overall hypothesis that LP cells are the cell type that is affected.

      Thanks for the suggestion. We have added the number of samples in Figure 2 legends.

      Our analysis focus on basal subtype because of the well-known relationship between BRCA1 deficiency and this subtype. Our results demonstrate the association between ELF3 expression and basal, TNBC, as well as HER2+ subtype, consistent with previous reports. Since TNBC also has high replication stress levels (NPJ Breast Cancer. 2020 Sep 7;6:40.), ELF3 upregulation in this subtype may not be solely due to BRCA1 deficiency, and we totally agree that this analysis may dilute the relationship between ELF3 and BRCA1. We have revised the discussion part to be more precise on this. 

      (7) Figure 3 provides experimental support for the hypothesis. While panel A is of interest, the legend lacks any description beyond "normal mammary tissue" and that there are non-carriers and carriers of BRCA1 mutations. Is this from bulk RNAseq data or single-cell RNAseq data? How many carriers and how many noncarriers? Panel E is ENCODE data from MCF7 cells that are ER+ luminal subtype so it is unclear if this is relevant to the LP cells that are the focus of the hypothesis.

      Thanks for the comments. Figure 3 panel A was from single-cell RNAseq data, including 3 BRCA1 WT patients and 4 BRCA1 mutant patients. All cells (normal cells and tumor cells) are involving, and ELF3 expression was normalized by reads in each cell. We have added this information in the figure legend. 

      It has been difficult to obtain ENCODE data in LP cells. The effect of E2F1 on regulation of ELF3 was validated in MCF10A cells by experiment and consistent with MCF7 ENCODE data, thus we suggest this effect can be conserve in mammary cells, but further confirmation in LP cells is needed. We have revised the manuscript to note that.

      (8) In Figure 4, the authors use BRCA1-deficient breast cancer cells to show the reliance on ELF3 and suggest that this is specific to this genetic lesion and not other subtypes. However, there is no data to show that this is not observed using ER+ cells or TNBC that are not BRCA1-deficient cell lines or models.

      Thanks for pointing this out. As ELF3 knockdown in MCF10A resulted in increased genomic instability (Supplement Fig S5) and less capability to resolve replication stress (Figure 5B), we believe that ELF3 can help deal with replication stress not specifically in BRCA1-mutant cells, but also normal mammary cells, and also multiple cell lines with distinct backgrounds as suggested in Figure 4G, 4H and Supplement Fig S4G. The special link between ELF3 and BRCA1 is reflected by ELF3 significant upregulation upon BRCA1 deficiency, but not ELF3 downstream functions. 

      (9) Figure 5 provides the first direct evaluation of biomarkers of replication stress (gamma H2AX, 53BP1). DNA fiber assays provide the most direct evaluation of replication fork kinetics, and therefore, replication stress. The knockdown of BRCA1 and ELF3 appear to phenocopy one another in the HCC1937, but there is no other cell type to show whether this is specific for BRCA1-deficient cells. For example, the MCF7 cells show E2F1 binding to ELF3 (Figure 3E) and may show replication stress upon knockdown of ELF3. Without testing this, the authors cannot suggest that the effect is linked to BRCA1 status. The authors do not identify the BRCA1 mutation in these cells and whether there is homozygous loss. Similarly, the mutational status in the SUM149PT cells should also be stated. These need to be added to aid interpretation of the results.

      Thank you for the constructive advice. We have added information regarding BRCA1 status of HCC1937 and SUM149PT. As discussed before, the results from Figure 4G and 4H suggest that ELF3 expression is associated with sensitivity to replicationstress-inducing-drugs across many cell lines. Thus ELF3 can maintain the stability of DNA replication is not specific to BRCA1-deficient cells. The reliance of ELF3 in BRCA1-deficiency we proposed is mainly focus on the fact that ELF3 is upregulated in BRCA1 deficient conditions, plus ELF3 may help cells tolerate replication stress during the transformation, therefore the resulted tumor cells-that is BRCA1-deficient breast cancer cells-may be more sensitive when losing ELF3 expression.

      (10) While the data in Figure 6 are valuable extensions of the gene signature derived from the MCF10A cells with BRCA1 knockdown, only 2 BRCA1 carriers are reported. As carriers bear heterozygous mutations in BRCA1, haplo-insufficiency would be necessary to generate the signature. The authors do note the publication by Panthania et al, but there are relatively few examples of haploinsufficiency. It should be noted that Sedic et al., 2015 also suggested haploinsufficiency in breast epithelial cell cultures from BRCA1 heterozygotes which appears to cause premature senescence, possibly via replication stress. However, this was observed in the basal epithelial cells. Therefore, this appears to be a feature of the breast epithelium more generally and is not enriched or limited to the LP cells.

      Thanks very much for your valuable suggestion. We have revised the discussion part to involve this important work and we fully agree that BRCA1 deficiency can cause replication stress not limited to LP cells. While in fact, the point we would like to address in Figure 6 is that BRCA1 deficiency modules the transcription profile towards LP-like cells, but not other-subtype-like cells, in normal mammary cells. We observed surprisingly similar profile between BRCA1-deficient cells and LP cells, suggesting there might be an inherent function of BRCA1 to mediate LP genes transcription. Furthermore, the data indicate that ELF3 has a tighter association with LP genes than other recognized LP-specific transcription factors like ELF5 and EHF, which are of the same family of ELF3. This result is intriguing since ELF3 can be upregulated by BRCA1 deficiency and replication stress. We assume that ELF3 could be a transcription node downstream of BRCA1 deficiency and modulate LP genes expression, and this process might be limited to LP cells since ELF3 has the highest expression levels in LP. Nonetheless, this hypothesis is also needed to be validated in LP cells by experiments. 

      We would like to express our deepest gratitude to the reviewers for their thorough and constructive feedback. Their insightful comments have been invaluable in guiding the revisions of our manuscript, helping us to clarify our hypotheses and strengthen the presentation of our findings. While we encountered some challenges, particularly with the isolation and culturing of LP cells, we made significant efforts to address the reviewers' concerns to the best of our ability. We have updated our manuscript accordingly, ensuring that all issues raised have been addressed comprehensively. We believe that these revisions have substantially improved the quality and clarity of our work, and we are excited to share our findings with the scientific community. Thank you once again for the opportunity to revise our manuscript, and we look forward to your feedback on the updated version.

    1. Author response:

      The following is the authors’ response to the original reviews

      Public Reviews:

      Reviewer #1 (Public review):

      This paper by Poverlein et al reports the substantial membrane deformation around the oxidative phosphorylation super complex, proposing that this deformation is a key part of super complex formation. I found the paper interesting and well-written but identified a number of technical issues that I suggest should be addressed:

      We thank Reviewer 1 for finding our work interesting. We have addressed the technical issues below.

      (1) Neither the acyl chain chemical makeup nor the protonation state of CDL are specified. The acyl chain is likely 18:2/18:2/18:2/18:2, but the choice of the protonation state is not straightforward.

      We thank the Reviewer for highlighting this missing information. We have now added this information in the Materials and Methods section:

      "…were performed in a POPC:POPE:cardiolipin (2:2:1) membrane containing 5 mol% QH<sub>2</sub> / Q (1:1 ratio). Cardiolipin was modeled as tetraoleoyl cardiolipin (18:1/18:1/18:1/18:1) with a headgroup modeled in a singly protonated state (with Q<sub>tot</sub>=-1)."

      (2) The analysis of the bilayer deformation lacks membrane mechanical expertise. Here I am not ridiculing the authors - the presentation is very conservative: they find a deformed bilayer, do not say what the energy is, but rather try a range of energies in their Monte Carlo model - a good strategy for a group that focuses on protein simulations. The bending modulus and area compressibility modulus are part of the standard model for quantifying the energy of a deformed membrane. I suppose in theory these might be computed by looking at the per-lipid distribution in thickness fluctuations, but this route is extremely perilous on a per-molecule basis. Instead, the fluctuation in the projected area of a lipid patch is used to imply the modulus [see Venable et al "Mechanical properties of lipid bilayers from molecular dynamics simulation" 2015 and citations within]. Variations in the local thickness of the membrane imply local variations of the leaflet normal vector (the vector perpendicular to the leaflet surface), which is curvature. With curvature and thickness, the deformation energy is analyzed.

      See:

      Two papers: "Gramicidin A Channel Formation Induces Local Lipid Redistribution" by Olaf Andersen and colleagues. Here the formation of a short peptide dimer is experimentally linked to hydrophobic mismatch. The presence of a short lipid reduces the influence of the mismatch. See below regarding their model cardiolipin, which they claim is shorter than the surrounding lipid matrix.

      Also, see:

      Faraldo-Gomez lab "Membrane transporter dimerization driven by differential lipid solvation energetics of dissociated and associated states", 2021. Mondal et al "Membrane Driven Spatial Organization of GPCRs" 2013 and many citations within these papers.

      While I strongly recommend putting the membrane deformation into standard model terms, I believe the authors should retain the basic conservative approach that the membrane is strongly deformed around the proteins and that making the SC reduces the deformation, then exploring the consequences with their discrete model.

      We thank the Reviewer for the suggestions and for pointing out the additional references, which are now cited in the revised manuscript. The analysis is indeed significantly more complex for large multi-million atom supercomplexes in comparison to small peptides (gramicidin A) or model systems of lipid membranes. However, in the revised manuscript, we have conducted further analysis on the membrane curvature effects based on the suggestions. We were able to estimate the energetic contribution of the changes in local membrane thickness and curvature, which are now summarized in Table 1, and described in the main text and SI. We find that both the curvature and local thickness contribute to the increased stability of SC.

      We have now extensively modified the result to differentiate between different components of membrane strain properly:

      "We observe a local decrease in the membrane thickness at the protein-lipid interface (Fig. 2G, Fig S2A,D,E), likely arising from the thinner hydrophobic belt region of the OXPHOS proteins (ca. 30 Å, Fig. S1A) relative to the lipid membrane (40.5 Å, Fig. S1). We further observe ∼30% accumulation of cardiolipin at the thinner hydrophobic belt regions (Fig. 2H, Fig. S2B,F,G), with an inhomogeneous distribution around the OXPHOS complexes. While specific interactions between CDL and protein residues may contribute to this enrichment (Fig. 2N), CDL prefers thermodynamically thinner membranes (∼38 Å, Fig. S1B, Fig. S5F). These changes are further reflected in the reduced end-toend distance of lipid chains in the local membrane belt (see Methods, Fig. S6, cf. also Refs. (41-44). In addition to the perturbations in the local membrane thickness, the OXPHOS proteins also induce a subtle inward curvature towards the protein-lipid interface (Fig. S5G), which could modulate the accessibility of the Q/QH2 substrate into the active sites of CI and CIII<sub>2</sub> (see below, section Discussion). This curvature is accompanied by a distortion of the local membrane plane itself (Fig. 2A-F, Fig. S4AC, Fig. S7), with perpendicular leaflet displacements reaching up to ~2 nm relative to the average leaflet plane.

      To quantify the membrane strain effects, we analyzed the cgMD trajectories by projecting the membrane surface onto a 2-dimensional grid and calculating the local membrane height and thickness at each grid point. From these values, we quantified the local membrane curvature (Fig. S5H), which measures the energetic cost of deforming the membrane from a flat geometry (ΔG<sub>curv</sub>). We also computed the energetics associated with changes in the membrane thickness, assessed from the deviations from an ideal local membrane in the absence of embedded proteins (ΔG<sub>thick</sub>, see Supporting Information, for technical details). Our analysis suggests that both contributions are substantially reduced upon formation of the SC, with the curvature decreasing by 19.8 ± 1.3 kcal mol-1 and the thickness penalty by 2.8 ± 2.0 kcal mol-1 (Table 1). These results indicate a significant thermodynamic advantage for SC formation, as it minimizes lipid deformation and stabilizes the membrane environment surrounding Complex I and III.”

      […]

      “Taken together, the analysis suggests that the OXPHOS complexes affect the mechanical properties of the membranes by inducing a small inwards curvature towards the protein-lipid interface (Fig. S5), resulting in a membrane deformation effect, while the SC formation releases some deformation energy relative to the isolated OXPHOS complexes. The localization of specific lipids around the membrane proteins, as well as local membrane perturbation effects, is also supported by simulations of other membrane proteins (45, 46), suggesting that the effects could arise from general protein-membrane interactions.”

      Our Supporting Information section now provides additional information about the membrane curvature.

      (41) R. M. Venable, F. L. H. Brown, R. W. Pastor, Mechanical properties of lipid bilayers from molecular dynamics simulation. Chemistry and Physics of Lipids 192, 60-74 (2015).

      (42) R. Chadda et al., Membrane transporter dimerization driven by differential lipid solvation energetics of dissociated and associated states. eLife 10, e63288 (2021).

      (43) S. Mondal et al., Membrane Driven Spatial Organization of GPCRs. Scientific Reports 3, 2909 (2013).

      (44) J. A. Lundbæk, S. A. Collingwood, H. I. Ingólfsson, R. Kapoor, O. S. Andersen, Lipid bilayer regulation of membrane protein function: gramicidin channels as molecular force probes. Journal of The Royal Society Interface 7, 373-395 (2009).

      We also expanded our SI Method section to account for the new calculations:

      “Analysis of lipid chain end-to-end length

      To probe the protein-induced deformation effect of the membrane, the membrane curvature (H), and the end-to-end distance between the lipid chains, were computed based on aMD and cgMD simulations. The lipid chain length was computed from simulations A1-A6 and C1 based on the first and last carbon atoms of each lipid chain. For example, the end-to-end length of a cardiolipin chain was determined as the distance between atom “CA1” and atom “CA18”.

      “Membrane Curvature and Deformation Energy

      The local mean curvature of the membrane midplane was computed by approximating the membrane surface as a height function Z(x,y), defined as the average location of the N-side and P-side leaflets at each grid point. Based on this, the mean curvature H(x,y) was calculated as,

      where the derivatives are defined as .

      The thickness deformation energy was computed from the local thickness d(x,y) relative to a reference thickness distribution F(d), derived from membrane-only simulations, and converted to a free energy profile via Boltzmann inversion. At each grid point, the F(d) was summed over the grid,

      The bending deformation energy was computed from the mean curvature field H(x,y), assuming a constant bilayer bending modulus κ (taken as 20 kJ mol-1 = 4.78 kcal mol-1):

      where Δ_A_ is the area of the grid cell.

      The thickness and curvature fields were obtained by projecting the coarse-grained MD trajectories (one frame per ns) onto a 2D-grid with a resolution of 0.5 nm. Grid points with low occupancy were downweighted to mitigate noise. More specifically, points with counts below 50% of the median grid count were scaled linearly by their relative count value. To focus the analysis on the region around the protein– membrane interface, only grid points within a radius of 20 nm from the center of the complex were included in the energy calculations. Energies were normalized to an effective membrane area of 1000 nm2 to facilitate the comparison between systems. Bootstrapping with resampling over frames was performed to estimate the standard deviations of G<sub>thick</sub> and G<sub>curv</sub>.

      We find that G<sub>curve</sub> converges slowly due to its sensitivity to local derivatives and the small grid size required to resolve the curvature contribution near the protein. Consequently, tens of microseconds of simulations were necessary to obtain well-converged estimates of the curvature energy.”

      (1) If CDL matches the hydrophobic thickness of the protein it would disrupt SC formation, not favor it. The authors' hypothesis is that the SC stabilizes the deformed membrane around the separated elements. Lipids that are compatible with the monomer deformed region stabilize the monomer, similarly to a surfactant. That is, if CDL prefers the interface because the interface is thin and their CDL is thin, CDL should prevent SC formation. A simpler hypothesis is that CDL's unique electrostatics are part of the glue.

      We rephrased the corresponding paragraph in the Discussion section to reflect the role of electrostatics for the behavior of cardiolipin.

      "…supporting the involvement of CDL as a "SC glue". In this regard, electrostatic effects arising from the negatively charged cardiolipin headgroup could play an important role in the interaction of the OXPHOS complexes."

      Generally our simulations suggest that CDL prefers thinner membranes, which could rationalize these findings.

      "We find that CDL prefers thinner membranes relative to the neutral phospholipids (PE/PC, Fig. S5F),[…]”

      (2) Error bars for lipid and Q* enrichments should be computed averaging over multi-lipid regions of the protein interface, e.g., dividing the protein-lipid interface into six to ten domains, in particular functionally relevant regions. Anionic lipids may have long, >500 ns residence times, which makes lipid enrichment large and characterization of error bars challenging in short simulations. Smaller regions will be noisy. The plots depicted in, for example, Figure S2 are noisy.

      It is indeed challenging to capture lipid movements on the timescales accessible for atomistic MD, and hence the data in Figure S2 contains some noise. In this regard, for the cgMD data presented in the revised Fig. S2H,I, the concentration data was averaged for six domains of the protein-lipid interface.

      (3) The membrane deformation is repeatedly referred to as "entropic" without justification. The bilayer has significant entropic and enthalpic terms just like any biomolecule, why are the authors singling out entropy? The standard "Helfrich" energetic Hamiltonian is a free energy model in that it implicitly integrates over many lipid degrees of freedom.

      We apologize for the unclear message – our intention was not to claim that the effects are purely entropic, but could arise from a combination of both entropic and enthalpic effects. We hope that this has now been better clarified in the revised manuscript. We also agree that it is difficult to separate between entropic and enthalpic effects. However, we wish to point out that, e.g., the temperature-dependence of the SC formation suggests that the entropic contribution is also affecting the process.

      Regarding the Helfrich Hamiltonian, we note that the standard model assumes a homogeneous fluid-like sheet. We have thus difficulties in relating this model to capture the local effects.

      Revisions / clarifications in the main manuscript:

      "SC formation is affected by both enthalpic and entropic effects."

      "We have shown here that the respiratory chain complexes perturb the IMM by affecting the local membrane dynamics. The perturbed thickness and alteration in the lipid dynamics lead to an energetic penalty, which can be related to molecular strain effects, as suggested by the changes of both the internal energy of lipid and their interaction with the surroundings (Fig. S2, S5, S6), which are likely to be of enthalpic origin. However, lipid binding to the OXPHOS complex also results in a reduction in the translational and rotational motion of the lipids and quinone (Fig. S8-S9), which could result in entropic changes. The strain effects are therefore likely to arise from a combination of enthalpic and entropic effects."

      (4) Figure S7 shows the surface area per lipid and leaflet height. This appears to show a result that is central to the interpretation of SC formation but which makes very little sense. One simply does not increase both the height and area of a lipid. This is a change in the lipid volume! The bulk compressibility of most anything is much higher than its Young's modulus [similar to area compressibility]. Instead, something else has happened. My guess is that there is *bilayer* curvature around these proteins and that it has been misinterpreted as area/thickness changes with opposite signs of the two leaflets. If a leaflet gets thin, its area expands. If the manuscript had more details regarding how they computed thickness I could help more. Perhaps they measured the height of a specific atom of the lipid above the average mid-plane normal? The mid-plane of a highly curved membrane would deflect from zero locally and could be misinterpreted as a thickness change.

      We thank the Reviewer for this insightful comment. We chose to define the membrane thickness based on the height of the lipid P-atoms above the average midplane normal. The Reviewer is correct that this measurement gives a changing thickness for a highly curved membrane. However, in this scenario, the thickness would always be overestimated [d<sub>true</sub> = d<sub>measured</sub> / cos (angle between global mid-plane normal and local mid-plane normal)]. Therefore, since we observe a smaller thickness at the protein-lipid interface, the effect is not likely to result from an artifact. For further clarification, see Fig. S4I showing the averaged local position of the Patoms in the cgMD simulations, which further supports that there is a local deformation of the lipid.

      The changes in the local membrane thickness are also supported by our analysis of the membrane thickness (Fig.S2A) and by the lipid chain length distributions (Fig.S6).

      (5) The authors write expertly about how conformational changes are interpreted in terms of function but the language is repeatedly suggestive. Can they put their findings into a more quantitative form with statistical analysis? "The EDA thus suggests that the dynamics of CI and CIII2 are allosterically coupled."

      We extended our analysis on the allosteric effects, which is now described in the revised main text, the SI and the Methods section:

      "In this regard, our graph theoretical analysis (Fig. S11C,D) further indicates that ligand binding to Complex I induces a dynamic crosstalk between NDUFA5 and NDUFA10, consistent with previous work (50, 51), and affecting also the motion of UQCRC2 with respect to its surroundings. Taken together, these effects suggest that the dynamics of CI and CIII<sub>2</sub> show some correlation that could result in allosteric effects, as also indicated based on cryo-EM analysis (40)."

      “Extended Methods

      Allosteric Network Analysis. Interactions between amino acid residues were modeled as an interaction graph, where each residue was represented by a vertex. Two nodes were connected by an edge, if the Ca atoms of the corresponding amino acid residues were closer than 7.5 Å for more than 50% of the frames of simulations S1-S6 (time step of frames: 1 ns). (7) This analysis was carried out for the aMD simulations of the supercomplex, analyzing differences between the Q bound and apo states (simulations A1+A2+A3 vs. A4+A5+A6).”

      (6) The authors write "We find that an increase in the lipid tail length decreases the relative stability of the SC (Figure S5C)" This is a critical point but I could not interpret Figure S5C consistently with this sentence. Can the authors explain this?

      We apologize for this oversight. This sentence should refer to Fig. S5F, which has now been corrected. We have additionally updated the figure to provide an improved estimation of the thickness contribution based on the lipid tail length.

      "We find that an increase in the lipid tail length decreases the relative stability of the SC (Fig. S5F)"

      (7) The authors use a 6x6 and 15x15 lattice to analyze SC formation. The SC assembly has 6 units of E_strain favoring its assembly, which they take up to 4 kT. At 3 kT, the SC should be favored by 18 kT, or a Boltzmann factor of 10^8. With only 225 sites, specific and non-specific complex formation should be robust. Can the authors please check their numbers or provide a qualitative guide to the data that would make clear what I'm missing?

      In the revised manuscript, we have now clarified the definition of the lattice model and the respective energies:

      In summary, the qualitative data presented are interesting (especially the combination of molecular modeling with simpler Monte Carlo modeling aiding broader interpretation of the results) ... but confusing in terms of the non-standard presentation of membrane mechanics and the difficulty of this reviewer to interpret some of the underlying figures: especially, the thickness of the leaflets around the protein and the relative thickness of cardiolipin. Resolving the quantitative interpretation of the bilayer deformation would greatly enhance the significance of their Monte Carlo model of SC formation.

      We thank the Reviewer for the helpful suggestion. We hope that the revisions help to clarify the non-standard presentation and connect to concepts used in the lipid membrane community.

      Reviewer #2 (Public review):

      Summary:

      The authors have used large-scale atomistic and coarse-grained molecular dynamics simulations on the respiratory chain complex and investigated the effect of the complex on the inner mitochondrial membrane. They have also used a simple phenomenological model to establish that the super complex (SC) assembly and stabilisation are driven by the interplay between the "entropic" forces due to strain energy and the enthalpies forces (specific and non-specific) between lipid and protein domains. The authors also show that the SC in the membrane leads to thinning and there is preferential localisation of certain lipids (Cardiolipin) in the annular region of the complex. The data reports that the SC assembly has an effect on the conformational dynamics of individual proteins making up the assembled complex and they undergo "allosteric crosstalk" to maintain the stable functional complex. From their conformational analyses of the proteins (individual and while in the complex) and membrane "structural" properties (such as thinning/lateral organization etc) as well from the out of their phenomenological lattice model, the authors have provided possible implications and molecular origin about the function of the complex in terms of aspects such as charge currents in internal mitochondrion membrane, proton transport activity and ATP synthesis.

      Strengths:

      The work is bold in terms of undertaking modelling and simulation of such a large complex that requires simulations of about a million atoms for long time scales. This requires technical acumen and resources. Also, the effort to make connections to experimental readouts has to be appreciated (though it is difficult to connect functional pathways with limited (additive forcefield) simulations.

      We thank the Reviewer for recognizing the challenge in simulating multimillion atom membrane proteins. We also thank the Reviewer for recognizing the connections we have made to different experiments. Our work indeed relies on atomistic and coarse-grained molecular simulations, which are widely recognized to provide accurate models of membrane proteins.

      Weakness:

      There are several weaknesses in the paper (please see the list below). Claims such as "entropic effect", "membrane strain energy" and "allosteric cross talks" are not properly supported by evidence and seem far-fetched at times. There are other weaknesses as well. Please see the list below.

      We thank the Reviewer for pointing out that key concepts needed further clarification. Please see answers to specific questions below:

      (i) Membrane "strain energy" has been loosely used and no effort is made to explain what the authors mean by the term and how they would quantify it. If the membrane is simulated in stress-free conditions, where are strains building up from?

      We thank the Reviewer for this important question. In the revised manuscript, we have toned down the assignment of the effects into pure entropic or enthalpic effects. We have also provided further clarification of the effects observed in the membrane.

      Example of revisions / clarifications in the main text:

      "SC formation is affected by both enthalpic and entropic effects."

      "We have shown here that the respiratory chain complexes perturb the IMM by affecting the local membrane dynamics. The perturbed thickness and alteration in the lipid dynamics lead to an energetic penalty, which can be related to molecular strain effects, as suggested by the changes of both the internal energy of lipid and their interaction with the surroundings (Fig. S2, S5, S6), which are likely to be of enthalpic origin. However, lipid binding to the OXPHOS complex, also results in a reduction in the translational and rotational motion of the lipids and quinone (Fig. S8-S9), which could result in entropic changes. The strain effects are therefore likely to arise from a combination of enthalpic and entropic effects."

      We have also revised the result section, where we now have explicitly defined and clarified the different contributions to membrane strain, observed in our simulations:

      In the following, we define membrane strain as the local perturbations of the lipid bilayer induced by protein-membrane interactions. These include changes in (i) membrane thickness, (ii) the local membrane composition, (iii) lipid chain configurations, and (iv) local curvature of the membrane plane relative to an undisturbed, protein-free bilayer. Together, these phenomena reflect the thermodynamic effects associated with accommodating large protein complexes within the membrane.

      We now also provide a more quantitative estimation of the membrane strain based on the contribution of changes in local thickness and curvature, summarize in Table 1.

      (ii) In result #1 (Protein membrane interaction modulates the lipid dynamics ....), I strongly feel that the readouts from simulations are overinterpreted. Membrane lateral organization in terms of lipids having preferential localisation is not new (see doi: 10.1021/acscentsci.8b00143) nor membrane thinning and implications to function (https://doi.org/10.1091/mbc.E20-12-0794). The distortions that are visible could be due to a mismatch in the number of lipids that need to be there between the upper and lower leaflets after the protein complex is incorporated. Also, the physiological membrane will have several chemically different lipids that will minimise such distortions as well as would be asymmetric across the leaflets - none of which has been considered. Connecting chain length to strain energy is also not well supported - are the authors trying to correlate membrane order (Lo vs Ld) with strain energy?

      We thank the Reviewer for the suggestions. The role of the membrane in driving supercomplex formation has not, to our knowledge, been suggested before. There are certainly many important studies, which have been better highlighted in the revised manuscript. In this context, we also now cite the papers Srivastava & coworkers and Tielemann & coworkers.

      “The localization of specific lipids around the membrane proteins, as well as local membrane perturbation effects, are also supported by simulations of other membrane proteins (45, 46), suggesting that the effects could arise from general protein-membrane interactions.”

      (45) V. Corradi et al., Lipid–Protein Interactions Are Unique Fingerprints for Membrane Proteins. ACS Central Science 4 (June 13, 2018).

      (46) K. Baratam, K. Jha, A. Srivastava, Flexible pivoting of dynamin pleckstrin homology domain catalyzes fission: insights into molecular degrees of freedom. Molecular Biology of the Cell 32 (2021 Jul 1).

      Physiological membrane will have several chemically different lipids that will minimise such distortions as well as would be asymmetric across the leaflets

      We agree with this point. As shown in Figs. 2H,N, S6, S13, we suggest that cardiolipin functions as a buffer molecule. However, very little is experimentally known about the asymmetric distribution of lipids in the IMM. Therefore, modelling the effect of asymmetry across the left is outside the scope of this study. Moreover, as now better clarified in the revised manuscript, we agree that it is difficult to unambiguously divide the effect into enthalpic and entropic contributions.

      To address the main concern of the Reviewer, we have updated the main text and Supporting Information to clearly state the different aspects of how the proteinmembrane interactions induce perturbations of the lipid bilayer. We define these effects as membrane strain. We now use the changes in local thickness and local curvature to quantify the effect of membrane strain on the stability of the respiratory SC.

      (iii) Entropic effect: What is the evidence towards the entropic effect? If strain energy is entropic, the authors first need to establish that. They discuss enthalpy-entropy compensation but there is no clear data or evidence to support that argument. The lipids will rearrange themselves or have a preference to be close to certain regions of the protein and that generally arises because of enthalpies reasons (see the body of work done by Carol Robinson with Mass Spec where certain lipids prefer proteins in the GAS phase, certainly there is no entropy at play there). I find the claims of entropic effects very unconvincing.

      We agree that it is difficult to distinguish the entropic vs. enthalpic contributions. In the revised manuscript, we better clarify that both effects are likely to be involved.

      The native MS work by Robinson and coworkers and others support that many lipids are strongly bound to membrane proteins, as also supported by the local binding of certain lipid molecules, such as CDL to the SC (Figs. S2, S6, S13).

      We suggest that the accumulation of cardiolipin at the protein-lipid interface involves a combination of entropic and enthalpic effects, arising from the reduction of the lipid mobility (entropy) as indicated by lowered diffusion (Fig. S9), and formation of noncovalent bonds between the lipid and the OXPHOS protein (Fig. S14).

      We added further clarification to the Discussion section.

      “Taken together, our combined findings suggest that the SC formation is affected by thermodynamic effects that reduce the molecular strain in the lipid membrane, whilst the perturbed micro-environment also affects the lipid and Q dynamics, as well as the dynamics of the OXPHOS proteins (see below).”

      (iv) The changes in conformations dynamics are subtle as reported by the authors and the allosteric arguments are made based on normal mode analyses. In the complex, there are large overlapping regions between the CI, CIII2, and SCI/III2. I am not sure how the allosteric crosstalk claim is established in this work - some more analyses and data would be useful. Normal mode analyses (EDA) suggest that the motions are coupled and correlated - I am not convinced that it suggests that there is allosteric cross-talk.

      Our analysis suggests that the SC changes the dynamics of the system. Although it is difficult to assign how these effects result in activity modulation of the system, we note these changes relate to sites that are central for the charge transfer reactions. We thank the Reviewer for suggesting to extend the analysis, which further suggests that regions of the proteins could be allosterically coupled.

      (v) The lattice model should be described better and the rationale for choosing the equation needs to be established. Specific interactions look unfavourable in the equation as compared to non-specific interactions.

      We have now provided further clarification of the lattice model in the Methods section. Addition to the main text:

      “Lattice model of SC formation. A lattice model of the CI and CIII<sub>2</sub> was constructed (Fig. 4A,B) by modeling the OXPHOS proteins in unique grid positions on a 2D N×N lattice. Depending on the relative orientation, the protein-protein interaction was described by specific interactions (giving rise to the energetic contribution E<sub>specific</sub> < 0) and non-specific interactions (E<sub>non-specific</sub> > 0). The membrane-protein interaction determined the strain energy of the membrane (E<sub>strain</sub>), based on the number of neighboring "lipid" occupied grids that are in contact with proteins (Fig. 4A). The interaction between the lipids was indirectly accounted for by the background energy of the model. The proteins could occupy four unique orientations on a grid ([North, East, South, West]). The states and their respective energies that the system can visit are summarized in Table S6.”

      “The conformational landscape was sampled by Monte Carlo (MC) using 10<sup>7</sup> MC iterations with 100 replicas. Temperature effects were modeled by varying β, and the effect of different protein-to-lipid ratios by increasing the grid area. The simulation details can be found in Table S7.”

      Reviewer #3 (Public review):

      Summary:

      In this contribution, the authors report atomistic, coarse-grained, and lattice simulations to analyze the mechanism of supercomplex (SC) formation in mitochondria. The results highlight the importance of membrane deformation as one of the major driving forces for SC formation, which is not entirely surprising given prior work on membrane protein assembly, but certainly of major mechanistic significance for the specific systems of interest.

      Strengths:

      The combination of complementary approaches, including an interesting (re)analysis of cryo-EM data, is particularly powerful and might be applicable to the analysis of related systems. The calculations also revealed that SC formation has interesting impacts on the structural and dynamical (motional correlation) properties of the individual protein components, suggesting further functional relevance of SC formation. Overall, the study is rather thorough and highly creative, and the impact on the field is expected to be significant.

      Weaknesses:

      In general, I don't think the work contains any obvious weaknesses, although I was left with some questions.

      We thank the Reviewer for acknowledging that our work is thorough and creative, and that it is likely to have a significant impact on the field.

      Recommendations for the authors:

      Reviewer #1 (Recommendations for the authors):

      Diffusion is quantified in speed units (Figure S8). The authors should explain why they have used an apparently incorrect model for quantifying diffusion. The variance of the distribution of a diffusing molecule is linear with time, not its standard deviation (as I suppose I would use for computing effective molecular speed). Perhaps they are quantifying residence times, in which molecules near a wall (protein) will appear to have half the movements of a bulk molecule. This is confusing.

      We thank the Reviewer for the comment. The data shown in previous version of Figure S8 corresponded to the effective molecular velocity, which is now clarified in the revised figure (now Fig. S9). This measure was used to reflect the average residence time of the groups in the vicinity of the sites.

      However, as suggested by the Reviewer, we now also analyzed the positiondependent diffusion of the quinone in the new Figure S9:

      (2) With a highly charged bilayer a large water layer is necessary to verify that the concentration of salt is plateauing at 150 mM at the box edge. 45 A appears to be the default in CHARMM-GUI, but this default guidance is not based on the charge of the bilayer. I suggest the authors plot the average concentration of both anions and cations in mM units along the z coordinate of the simulation cell.

      We thank the Reviewer for the suggestion. We have now provided an analysis of the average ion concentrations along the z coordinate, supporting that the salt concentration plateaus at 150 mM at the box edge.

      Typos:

      SI: "POPC/POPE or CLD" should be CDL

      We apologize for the mistake. We have corrected the typos:

      "of the membrane thickness in a POPC/POPE/CDL/QH2 membrane and a CDL membrane."

      "a pure CDL membrane"

      Reviewer #2 (Recommendations for the authors):

      (1) Suggestion regarding membrane strain energy claims:

      Changes in area per lipid and membrane thinning are surely not akin to membrane strain energy changes. At best, the authors should calculate the area compressibility (both in bilayers with and without proteins) and then make comments. In general, if they are talking about the in-plane properties (bilayer being liquid in 2D), I do not see how they can discuss membrane strain energy with NPT=1 atms barostat reservoir that they are simulating against. At least they can try to plot the membrane lateral pressures in various conditions and then start making such comments. If it was a closed vesicle, I would expect some tension in the membrane due to the closed surface but in the conditions in which the simulations are run, I do not see how strain is so important. If the authors want to be more rigorous, they can calculate "atomic viral" values by doing a tessellation and showing the data to make their point. Strain energy would mean that there is a modulus in-plane. Bending modulus would surely change with membrane thinning and area compressibility changes (simple plate theory) but linear strain is surely something to be defined well before making claims out of it.

      Our work shows that the OXPHOS proteins alter the local membrane thickness and curvature, and we now quantify the deformation penalty associated with that (Table 1). As stated above, we now provide a better definition and description 'membrane strain’ and the observed effect, which is likely to contain both enthalpic and entropic contributions.

      As suggested by the Reviewer, we have computed the lateral pressure profiles around the OXPHOS proteins, further supporting that there are energetic effects related to the "solvation" of the membrane proteins in the IMM. To this end, Figs. S2D,E; Figure S4I and Fig. S5G,H shows the membrane distortion effect; while in Fig. S5A supports that there the 'internal energy' of the lipids changes as result of the SC formation, further justifying that these effects can be assigned as 'strain effects'. The analysis has also been extended by computing the end-to-end distances, shown in Fig. S6.

      Unfortunately, it is technically unfeasible to accurately estimate the area compressibility, bending modulus, or the atomic virial for the present multi-million membrane protein simulations.

      Summary of Revisions/Additions:

      Fig. S2 [...] (D, E) Difference in the membrane thickness around the SC relative to CI (left) or relative to CIII<sub>2</sub> (right) from (D) aMD and (E) cgMD.

      Fig. S4. [...] (I) Visualization of the membrane distortion effect.

      Fig. S5. Analysis of membrane-induced distortion effects. (A) Relative strain effect relative to a lipid membrane from atomistic MD simulations of the SCI/III2, CI, and CIII<sub>2</sub>, suggesting reduction of the membrane strain (blue patches) in the SC surroundings. The figure shows the non-bonded energies relative to the average non-bonded energies from membrane simulations (simulation M4, Table S1). (B) The lipid strain contribution for different lipids calculated from non-bonded interaction energies of the lipids relative to the average lipid interaction in a IMM membrane model (simulation M4). The figure shows the relative strain contribution for nearby lipids (r < 2 Å, in color from panel (C), and lipids >5 Å from the OXPHOS proteins. (C) Selection of lipids (< 2 Å) interacting with the OXPHOS proteins. (D) Potential of mean force (PMF) of membrane thickness derived from thickness distributions from cgMD simulations of a membrane, the SCI/III2, CI, and CIII<sub>2</sub>. (E) Membrane thickness as a function of CDL concentration from cgMD simulations. (F) ΔGthick of the SC as a function of membrane thickness based on cgMD simulations. (G) Membrane curvature around the SCI/III2 (left), CI (middle), and CIII<sub>2</sub> (right) from atomistic simulations. (H) Squared membrane curvature obtained from cgMD simulations, within a 20 nm radius around the center of the system. These maps correspond to the curvature field used in the calculation of the bending deformation energy term (G<sub>curv</sub>).

      Fig. S6. Analysis of lipid end-to-end distance from aMD simulations of (A) SC, (B) CI, (C) CIII<sub>2</sub>.

      (2) Membrane distortions:

      Membrane distortions can arise due to a mismatch in the area between the upper leaflet and the lower left especially when a protein is embedded. Authors can carefully choose the numbers to keep the membrane stable.

      We have further clarified in the revised manuscript that the membranes are stable in all simulation setups. During building the simulation setups, it was carefully considered that no leaflet introduced higher lipid densities that could result in artificial displacements. Our results of the local changes in the lipid dynamics and structure around the OXPHOS complexes are independently supported by both our atomistic and coarse-grained simulations, which contain significantly larger membranes. Moreover, as discussed in our work, the local membrane distortion is also experimentally supported by cryoEM analysis as well as recent in situ cryoTEM data, showing that the OXPHOS proteins indeed affect the local membrane properties.

      Clarifications/Additions to the main text:

      “We find that the individual OXPHOS complexes, CI and CIII<sub>2</sub>, induce pronounced membrane strain effects, supported both by our aMD (Fig. S2A) and cgMD simulations with a large surrounding membrane (Fig. 2G).“

      ” The localization of specific lipids around the membrane proteins, as well as local membrane perturbation effects, are also supported by simulations of other membrane proteins (45, 46), suggesting that the effects could arise from general protein-membrane interactions.”

      "During construction of the simulation setups, it was carefully considered that no leaflet introduced higher lipid densities that could result in artificial displacement effects."

      (3) Strain energy as an entropic effect:

      Please establish that the strain energy (if at all present) can be called an entropic effect.

      We have now better clarified that the SC formation results from combined enthalpic and entropic effects. We apologize that the previous version of the text was unclear in this respect.

      To further probe the involvement of entropic effects, we derived entropic and enthalpic contributions from our lattice model. The model supports that increased strain contributions also alters the entropic contributions, further supporting the coupling between the effects.

      We have also clarified our definition of the effects:

      " The perturbed thickness and alteration in the lipid dynamics leads to an energetic penalty, which can be related to molecular strain effects, as suggested by the changes of both the internal energy of lipid and their interaction with the surroundings (Fig. S2, S5, S6), which are likely to be of enthalpic origin. However, lipid binding to the OXPHOS complex, also results in a reduction in the translational and rotational motion of the lipids and quinone (Fig. S8-S9), which could result in entropic changes. The strain effects are therefore likely to arise from a combination of enthalpic and entropic effects."

      (4) Allosteric cross-talk:

      A thorough network analysis (looking at aspects like graph laplacian, edge weights, eigenvector centrality, changes in characteristic path length, etc can be undertaken to establish allostery (see https://doi.org/10.1093/glycob/cwad094, Ruth Nussinov/Ivet Bahar papers).

      We have expanded the network analysis as suggested by the Reviewer. In this regard, we have expanded the analysis by computing the covariance matrix, further supporting that the SC could involve correlated protein dynamics. We observe a prominent change especially with respect to the ligand state of Complex I, indicative of some degree of allostery, while we find that the apo state of Complex I leads to a slight uncoupling of the motion between CI and CIII<sub>2</sub>.

      Additions in the main text:

      In this regard, our graph theoretical analysis (Fig. S11) further indicates that ligand binding to Complex I induces a dynamic crosstalk between NDUFA5 and NDUFA10, consistent with previous work (48, 49), and affecting also the motion of UQCRC2 with respect to its surroundings_._ Taken together, these effects suggest that the dynamics of CI and CIII<sub>2</sub> show some correlation that could result in allosteric effects, as also indicated based on the cryoEM analysis.

      (5) Lattice model:

      The equation needs to be rationalised. For example, specific interaction (g_i g_j favours separation (lower energy when i and j are not next to each other), and nonspecific interaction favours proximity. Why is that? Also, the notation for degeneracy in partition function and the notation for lattice point. It is mentioned that the "interaction between the lipids was indirectly accounted for by the "background energy" of the model". If the packing/thinning etc are so important to the molecular simulations, will not the background energy change with changing lipid organising during complex formation?

      We have further expanded the technical discussion of the energy terms in our lattice model.

      For example, specific interaction (g_i g_j favours separation (lower energy when i and j are not next to each other), and non-specific interaction favours proximity. Why is that

      "The g<sub>i</sub>g<sub>j</sub> -term assigns a specific energy contribution when the OXPHOS complexes are in adjacent lattice points only in a correct orientation (modeling a specific non-covalent interaction between the complexes such as the Arg29<sup>FB4</sup>-Asp260<sup>C1</sup>/Glu259<sup>C1</sup> interaction between CI and CIII<sub>2</sub>). The d<sub>i</sub>d<sub>j</sub> -term assigns a non-specific interaction for the OXPHOS complexes when they are in adjacent lattice points, but in a "wrong" orientation relative to each other to form a specific interaction. The term introduces a strain into all lattice points surrounding an OXPHOS complex, mimicking the local membrane perturbation effects observed in our molecular simulations.

      This leads to the partition function,

      where wi is the degeneracy of the state, modeling that the SC and OXPHOS proteins can reside at any lattice position of the membrane, and where β=1/k<sub>B</sub>T (k<sub>B</sub>, Boltzmann's constant; T, temperature). The probability of a given state i was calculated as,

      with the free energy (G) defined as,

      This discussion has been included in the methods sections to ensure that our work remains readable for the biological community studying supercomplexes from a biochemical, metabolic, and physiological perspectives.

      (6) This is a minor issue but the paper is poorly organised and can be fixed readily. The figures are not referenced in order. For example, Figure 2G is discussed before discussing Figures 2A-2F (never discussed). Figure S2 is referenced before Figure S1.

      Answer: We thank the Reviewer for pointing this out. The order of the figures was revised.

      Reviewer #3 (Recommendations for the authors):

      A few minor questions/suggestions, not necessarily in the order of importance:

      (1) The discussion of the timescale of simulations is a bit misleading. For example, the discussion cites a timescale of 0.3 ms of CG simulations. The value is actually the sum of multiple CG simulations on the order of 50-75 microseconds. These are already very impressive lengths of CG simulations, there is no need to use the aggregated time to claim even longer time scales.

      We thank the Reviewer for the suggestion on this important clarification. We have now modified the text and tables accordingly:

      "(0.3 ms in cumulative simulation time, 50-75 μs/cgMD simulation)"

      (2) The observation of cardiolipin (CDL) accumulation is interesting. How close are the head groups, relative to the electrostatic screening length at the interface? Should one worry about the potential change of protonation state coupled with the CDL redistribution?

      Answer: We thank the Reviewer for this excellent comment, which has also been on our mind. The CDL indeed form contacts with various functional groups at the protein interface (as shown in Fig. S13), as well as bulk ions (sodium) that could tune the p_K_a of the CDLs, and result in a protonation change. We have clarified these effects in the revised manuscript:

      "While CDL was modeled here in the singly anionic charged state (but cf. Fig. S5E), we note that the local electrostatic environment could tune their p_K_a that result in protonation changes of the lipid, consistent with its function as a proton collecting antenna (62)."

      (3) The authors refer to the membrane strain effect as entropic. Since membrane bending implicates a free energy change that includes both enthalpic and entropic components, I wonder how the authors reached the conclusion that the effect is largely entropic in nature.

      We agree with the Reviewer that the effects are likely to comprise both enthalpic and entropic contributions, which are difficult to separate in practice. To reflect this, we have now better clarified why we consider that both contributions are involved. We apologize that our previous version of the manuscript was unclear in this respect. Clarifications in the main text:

      “The perturbed thickness and alteration in the lipid dynamics lead to an energetic penalty, which can be related to molecular strain effects, as suggested by the changes of both the internal energy of lipid and their interaction with the surroundings (Fig. S2, S5, S6), which are likely to be of enthalpic origin. However, lipid binding to the OXPHOS complex also results in a reduction in the translational and rotational motion of the lipids and quinone (Fig. S8-S9), which could result in entropic changes. The strain effects are therefore likely to arise from a combination of enthalpic and entropic effects."

      (4) The authors refer to the computed dielectric constant as epsilon_perpendicular. Did the authors really distinguish the parallel and perpendicular component of the dielectric tensor, as was done by, for example, R. Netz and co-workers for planar surfaces?

      We have extracted the perpendicular dielectric constant from the total dielectric profiles. We clarify that this differs from the formal definition of by Netz and coworkers.

      “The calculations were performed by averaging the total M over fixed z values from the membrane plane. Note that this treatment differs from extraction of radial and axial contributions of the dielectric tensor, as developed by Netz and co-workers (cf. Ref. (3) and refs therein) that requires a more elaborate treatment, which is outside the scope of the present work.”

      (3) P. Loche, C. Ayaz, A. Schlaich, Y. Uematsu, R.R. Netz. Giant Axial Dielectric Response in Water-Filled Nanotubes and Effective Electrostatic Ion-Ion Interactions from a Tensorial Dielectric Model. J Phys Chem B 123, 10850-10857 (2019).

      (5) Regarding the effect of SC formation on protein structure and dynamics, especially allosteric effects, most of the discussions are rather qualitative in nature. More quantitative analysis would be valuable. For example, the authors did compute covariance matrix although it appears that they chose not to discuss the results in depth. Is the convergence of concern and therefore no thorough discussion is given?

      We have now expanded the analysis by computing the covariance matrix, further supporting that the SC could involve correlated protein dynamics. We observe a prominent change, especially with respect to the ligand state of Complex I, indicative of some degree of allostery, while we find that the apo state of Complex I leads to a slight uncoupling of the motion between CI and CIII<sub>2</sub>.

      Additions in the main text:

      “In this regard, our graph theoretical analysis (Fig. S11) further indicates that ligand binding to Complex I induces a dynamic crosstalk between NDUFA5 and NDUFA10, consistent with previous work (48, 49), and affecting also the motion of UQCRC2 with respect to its surroundings. Taken together, these effects suggest that the dynamics of CI and CIII<sub>2</sub> show some correlation that could result in allosteric effects, as also indicated based on the cryoEM analysis (40).”

      (6) The discussion of quinone diffusion is interesting, although I'm a bit intrigued by the unit of the diffusion constant cited in the discussion. Perhaps a simple typo?

      The plot showed the molecular velocity, which roughly corresponding to the residence times. However, as suggested by the Reviewer, we now also analyzed the position-dependent diffusion of the quinone in the new Figure S9:

    1. Author Response

      Thank you for your letter and for the reviewers’ comments concerning our manuscript entitled “The cation channel mechanisms of subthreshold inward depolarizing currents in the VTA dopaminergic neurons and their roles in the depression-like behavior”. These comments are constructive and very helpful for improving our manuscript. We have studied comments carefully and have made provisional revision which we hope meet with approval. We also respond to the reviewer’s comments point by point as following.

      Reviewer #1 (Public Review):

      Comment 1:

      The pharmacological tools used in this study are highly non-selective. Gd3+, used here to block NALCN is actually more commonly used to block TRP channels. 2-APB inhibits not only TRPC channels, but also TRPM and IP3 receptors while stimulating TRPV channels (Bon and Beech, 2013), while FFA actually stimulates TRPC6 channels while inhibiting other TRPCs (Foster et al., 2009).

      We agree with the reviewer that the substances mentioned are not specific. Although we performed shRNA experiments against NALCN and TRPC6, we also used more specific pharmacological modulators for these two channels, L703,606 (the antagonist of NALCN)[1] and larixyl acetate (a potent TRPC6 inhibitor)[2]. The results are shown in figure 3E, F and figure 4C, E.

      Comment 2:

      -The multimodal approach including shRNA knockdown experiments alleviates much of the concern about the non-specific pharmacological agents. Therefore, the author's claim that NALCN is involved in VTA dopaminergic neuron pacemaking is well-supported.

      -However, the claim that TRPC6 is the key TRPC channel in VTA spontaneous firing is somewhat, but not completely supported. As with NALCN above, the pharmacology alone is much too non-specific to support the claim that TRPC6 is the TRP channel responsible for pacemaking. However, unlike the NALCN condition, there is an issue with interpreting the shRNA knockdown experiments. The issue is that TRPC channels often form heteromers with TRPC channels of other types (Goel, Sinkins and Schilling, 2002; Strübing et al., 2003). Therefore, it is possible that knocking down TRPC6 is interfering with the normal function of another TRPC channel, such as TRPC7 or TRPC4.

      From our single-cell RNA-seq results, TRPC7 and TRPC4 are found not to be present broadly like TRPC6 in the VTA DA neurons. And in experiments using single cell PCR (sFig. 9A), only a very small proportion of TRPC6-positive DA cells (DAT+) expressed TRPC4 (sFig. 9Bi) or TRPC7 (sFig. 9Bii), in consistent with the results of single-cell RNA-seq (Fig.2). Therefore, it is possible that knocking down TRPC6 maybe not interfering with the normal function of another TRPC channel, such as TRPC7 or TRPC4.

      Comment 3:

      The claim that TRPC6 channels in the VTA are involved in the depressive-like symptoms of CMUS is supported.

      • However, the connection between the mPFC-projecting VTA neurons, TRPC6 channels, and the chronic unpredictable stress model (CMUS) of depression is not well supported. In Figure 2, it appears that the mPFC-projecting VTA neurons have very low TRPC6 expression compared to VTA neurons projecting to other targets. However, in figure 6, the authors focus on the mPFC-projecting neurons in their CMUS model and show that it is these neurons that are no longer sensitive to pharmacological agents non-specifically blocking TRPC channels (2-APB, see above comment). Finally, in figure 7, the authors show that shRNA knockdown of TRPC6 channels (in all VTA dopaminergic neurons) results in depressive-like symptoms in CMUS mice. Due to the low expression of TRPC6 in mPFC-projecting VTA neurons, the author's claims of "broad and strong expression of TRPC6 channels across VTA DA neurons" is not fully supported. Because of the messy pharmacological tools used, it cannot be clamed that TRPC6 in the mPFC-projecting VTA neurons is altered after CMUS. And because the knockdown experiments are not specific to mPFC-projecting VTA neurons, it cannot be claimed that reducing TRPC6 in these specific neurons is causing depressive symptoms.

      The reason we focused on the mPFC-projecting VTA DA neurons is that this pathway is indicated in depressive-like behaviors of the CMUS model[3-5]. Although mPFC-projecting VTA DA neurons seem have lower level of TRPC6, we reason they are still functional there. However, we do agree with the reviewer that the statement “broad and strong expression of TRPC6 channels across VTA DA neurons" is not fully supported. We have changed the statements based on the reviewer suggestion. Furthermore, we did selectively knockdown TRPC6 in the mPFC-projecting VTA DA neurons, and then studied the behavior (Fig.8).

      Comment 4:

      It is important to note that the experiments presented in Figure 1 have all been previously performed in VTA dopaminergic neurons (Khaliq and Bean, 2010) including showing that low calcium increases VTA neuron spontaneous firing frequency and that replacement of sodium with NMDG hyperpolarizes the membrane potential.

      We agree with reviewer that similar experiments have been performed previously [6] for the flow of our manuscript and for general readers.

      Comment 5:

      -The authors explanation for the increase in firing frequency in 0 calcium conditions is that calcium-activated potassium channels would no longer be activated. However, there is a highly relevant finding that low calcium enhances the NALCN conductance through the calcium sensing receptor from Dejian Ren's lab (Lu et al., 2010) which is not cited in this paper. This increase in NALCN conductance with low calcium has been shown in SNc dopaminergic neurons (Philippart and Khaliq, 2018), and is likely a factor contributing to the low-calcium-mediated increase in spontaneous VTA neuron firing.

      We agree with the reviewer and thanks for the suggestions. A discussion for this has been added.

      Comment 6:

      -One of the only demonstrations of the expression and physiological significance of TRPCs in VTA DA neurons was published by (Rasmus et al., 2011; Klipec et al., 2016) which are not cited in this paper. In their study, TRPC4 expression was detected in a uniformly distributed subset of VTA DA neurons, and TRPC4 KO rats showed decreased VTA DA neuron tonic firing and deficits in cocaine reward and social behaviors.

      We thank the reviewer for the suggestion. The references and a discussion for this has been added.

      Comment 7:

      • Out of all seven TRPCs, TRPC5 is the only one reported to have basal/constitutive activity in heterologous expression systems (Schaefer et al., 2000; Jeon et al., 2012). Others TRPCs such as TRPC6 are typically activated by Gq-coupled GPCRs. Why would TRPC6 be spontaneously/constitutively active in VTA DA neurons?

      In a complex neuronal environment where VTA DA neurons are located, multiple modulatory factors including the GPCRs could be dynamically active, this could lead to the activation of TRP channels including TRPC6.

      Comment 8:

      A new paper from the group of Myoung Kyu Park (Hahn et al., 2023) shows in great detail the interactions between NALCN and TRPC3 channels in pacemaking of SNc DA neurons.

      The reference mentioned has been added. We thank the reviewer.

      Reviewer #2 (Public Review):

      Comment 1:

      These results do not show that TRPC6 mediates stress effects on depression-like behavior. As stated by the authors in the first sentence of the final paragraph, "downregulation of TRPC6 proteins was correlated with reduced firing activity of the VTA DA neurons, the depression-like behaviors, and that knocking down of TRPC6 in the VTA DA neurons confer the mice with depression behaviors." Therefore, the results show associations between TRPC6 downregulation and stress effects on behavior, occlusion of the effects of one by the other on some outcome measures, and cell manipulation effects that resemble stress effects. There is no experiment that shows reversal of stress effects with cell/circuit-specific TRPC6 manipulations. Please adjust the title, abstract and interpretation accordingly.

      We agree with the reviewer’s suggestion. The title was changed to ‘’The cation channel mechanisms of subthreshold inward depolarizing currents in the VTA dopaminergic neurons and their roles in the chronic stress-induced depression-like behavior” and the abstract and interpretation were also adjusted accordingly.

      Comment 2:

      Statistical tests and results are unclear throughout. For all analyses, please report specific tests used, factors/groups, test statistic and p-value for all data analyses reported. In some cases, the chosen test is not appropriate. For example, in Figure 6E, it is not clear how an experiment with 2 factors (stress and drug) can be analyzed with a 1-way RM ANOVA. The potential impact of inappropriate statistical tests on results makes it difficult to assess the accuracy of data interpretation.

      We have redone the statistical analysis as suggested by the reviewer and added specific tests used, factors/groups, test statistic and p-value for all data analyses into the figure legends of the revised manuscript.

      Comment 3:

      Why were only male mice used? Please justify and discuss in the manuscript. Also, change the title to reflect this.

      Although most similar previous studies used male mice or rats[7, 8], we do agree with the reviewer that the female animals should also be tested, in consideration possible role of sex hormones, as such we repeated some key experiments on female mice (sFig.1.6.8. and 13).

      Comment 4:

      Number of recorded cells is very low in Figure 1. Where in VTA did recordings occur? Given the heterogeneity in this brain region, this n may be insufficient. Additional information (e.g., location within VTA, criteria used to identify neurons) should be included. Report the number of mice (i.e., n = 6 cells from X mice) in all figures.

      Yes indeed, the number here is not high. More experiments were performed to increase the N/n number. And the location of recorded cells in VTA and the number of used mice is now shown in all figures; criteria to identify neurons is stated in the Methods-Identification of DA neurons and electrophysiological recordings. At the end of electrophysiological recordings, the recorded VTA neurons were collected for single-cell PCR. VTA DA neurons were identified by single-cell PCR for the presence of TH and DAT.

      Comment 5:

      Authors refer to VTA DA neurons as those that are DAT+ in line 276, although TH expression is considered the standard of DAergic identity, and studies (e.g., Lammel et al, 2008) have shown that a subset of VTA DA neurons have low levels of DAT expression. Authors should reword/clarify that these are DAT-expressing VTA DA neurons.

      The study published by Lammel[9] in 2015 has shown the low dopamine specificity of transgene expression in ventral midbrain of TH-Cre mice; on the other hand, DAT-Cre mice exhibit dopamine-specific Cre expression patterns, although DAT-Cre mice are likely to suffer from their own limitations (for example, low DAT expression in mesocortical DA neurons may make it difficult to target this subpopulation, see Lammel et al., 2008[10]).Hence, in our study, the DAT was used as criteria to identify DAT neurons. Of course, TH and DAT were all tested in single-cell PCR to identify whether the recorded cells were DA neurons.

      Comment 6:

      Neuronal subtype proportions should be quantified and reported (Fig. 1Aii).

      Neuronal subtype proportions are now quantified and reported in Fig. 1Aii.

      Comment 7:

      In addition to reporting projection specificity of neurons expressing specific channels, it would be ideal to report these data according to spatial location in VTA.

      The spatial location of recorded cells in VTA are now shown in all figures.

      Comment 8:

      The authors state that there are a small number of Glut neurons in VTA, then they state that a "significant proportion" of VTA neurons are glutamatergic.

      Thanks, “a significant proportion of neurons” has been changed to “less than half of sequenced DA neurons”.

      Comment 9:

      It is an overstatement that VTA DA neurons are the key determinant of abnormal behaviors in affective disorders.

      Thanks, we have amended the statement to that “Dopaminergic (DA) neurons in the ventral tegmental area (VTA) play an important role in mood, reward and emotion-related behaviors”.

      Reviewer #3 (Public Review):

      Comment 1:

      The authors of this study have examined which cation channels specifically confer to ventral tegmental area dopaminergic neurons their autonomic (spontaneous) firing properties. Having brought evidence for the key role played by NALCN and TRPC6 channels therein, the authors aimed at measuring whether these channels play some role in so-called depression-like (but see below) behaviors triggered by chronic exposure to different stressors. Following evidence for a down-regulation of TRPC6 protein expression in ventral tegmental area dopaminergic cells of stressed animals, the authors provide evidence through viral expression protocols for a causal link between such a down-regulation and so-called depression-like behaviors. The main strength of this study lies on a comprehensive bottom-up approach ranging from patch-clamp recordings to behavioral tasks. However, the interpretation of the results gathered from these behavioral tasks might also be considered one main weakness of the abovementioned approach. Thus, the authors make a confusion (widely observed in numerous publications) with regard to the use of paradigms (forced swim test, tail suspension test) initially aimed (and hence validated) at detecting the antidepressant effects of drugs and which by no means provide clues on "depression" in their subjects. Indeed, in their hands, the authors report that stress elicits changes in these tests which are opposed to those theoretically seen after antidepressant medication. However, these results do not imply that these changes reflect "depression" but rather that the individuals under scrutiny simply show different responses from those seen in nonstressed animals. These limits are even more valid in nonstressed animals injected with TRPC6 shRNAs (how can 5-min tests be compared to a complex and chronic pathological state such as depression?). With regard to anxiety, as investigated with the elevated plus-maze and the open field, the data, as reported, do not allow to check the author's interpretation as anxiety indices are either not correctly provided (e.g. absolute open arm data instead of percents of open arm visits without mention of closed arm behaviors) or subjected to possible biases (lack of distinction between central and peripheral components of the apparatus).

      We agree with the reviewer that behavior tests we used here is debatable whether they represent a real depression state, and this is an open question that could be discussed from different respective. Since these testes (forced swimming and tail suspension), as the reviewer noted, were “widely observed in numerous publications”, we used these seemly only options to reflect a “depression-like” state. One could argue that since these testes were initially used for testing antidepressants (“validated”), with decreased immobility time as indications of anti-depressive effects, why not an increased immobility time reflect a “depression-like” state. As for anxiety tests, the data concerning the elevated plus-maze are also changed based on the reviewer’s suggestion.

      Recommendations for the authors: please note that you control which, if any, revisions, to undertake

      Reviewer #1 (Recommendations For The Authors):

      Recommendation 1 for improving the paper:

      -The paper needs extensive editing for both overall structural clarity and for the high number of typos and grammatical errors.

      We thank the reviewer’s suggestion. The revised manuscript has been edited extensively.

      Recommendation 2 for improving the paper:

      -Retrobeads are often toxic to cells and build up with increasing time. It is surprising that the authors wait 14-21 days for retrobead expression in their target cells. It is also a problem that the mPFC projecting cells have a longer time with the retrobeads than the other projection-targeting cells because the toxicity could be more extensive with the longer wait time thus confounding the results. The authors should repeat some mPFC experiments at the 14 day time point to confirm that the longer time with the beads is not influencing the differential effects in these cells.

      According to the methods published by Stephan Lammel and Jochen Roeper, “For sufficient labeling, survival periods for retrograde tracer transport depended on respective injection areas: DS and NAc lateral shell, 7 days; NAc core, NAc medial shell, and BLA, 14 days; and mPFC, 21 days[10]”, we did the experiments related to mPFC projecting cells at the 21 day time point. Consistent with the mentioned above, the labeled mPFC projecting cells at 14 day time point, is not sufficient, compared with this at 21 day time point, which is shown as followings.

      Author response image 1.

      Confocal images showing the anatomical distribution of mPFC-projecting DA neurons labelled with retrobeads (red) in the VTA after DAT-immunofluorescence (green) staining at different day time point (A, 14d; B, 21d) after retrobeads injection; Scale bars=10 μm.

      Recommendation 3 for improving the paper:

      -The experiment with FFA in Figure 4E seems weird. Why is there no baseline before the FFA application? And why is the baseline trending downward immediately? The authors should explain why this example experiment is presented differently from all the others.

      We apologize for this part that this example time-course is not typical. Since the FFA is not specific antagonist for TRPC6 and actually stimulates TRPC6 channels, we repeated the experiments with a more specific pharmacological modulator for TRPC6, larixyl acetate (LA), and the results are shown in Figure 4C and 4F.

      Recommendation 4 for improving the paper:

      -It would be much more useful to see exact p values in the text, as it aids in interpreting the 'insignificance' of specific comparisons. Specifically, in Figure 5F, the 2-APB looks like it is having a small effect, and the already low firing rate (due to the TRPC6 knockdown) makes a big effect less likely. It would be useful to know what the actual p value is here (and everywhere).

      OK. We now report all P values in the figure legends of the revised version.

      Recommendation 5 for improving the paper:

      -In the results, it should be explained that the "RMP" of VTA DA neurons was obtained by treating the cells with TTX.

      A sentence indicating the presence of TTX when measuring “RMP” is added in the Results part of the revised version.

      Recommendation 6 for improving the paper:

      -The spacing of the panels in the figures is somewhat odd. The figures could be more compact.

      Thanks, we have re-arranged all figures.

      Recommendation 7 for improving the paper:

      The paper is difficult to read because of significant grammatical errors. Here are some examples by line number, but this list is not at all exhaustive.

      We thank the reviewer for pointing out grammatical errors and we corrected them.

      Reviewer #2 (Recommendations For The Authors):

      Recommendation 1 for improving the paper:

      Fix typos: e.g., change HCH to HCN, change EMP to EPM, "these finding", "compact par" should read "pars compacta", "substantial" in line 475 should read "substantia", Incomplete sentences on line 73 and line 107, etc. Also, what is meant by "autonomic" firing activity? What is meant by "expression files"? Change "depression behaviors" to depression-like behaviors. "The HCN" as written in line 69 is a bit misleading, as HCN channels in the heart and brain are different members of a family of channels, although as written in the text, it seems that they are identical. In Figure 2, rearrange order of brain regions (e.g., from "BLA-VTA" to "VTA-BLA"), because as written, it seems that the focus is on projections into the VTA from each brain region, rather than VTA neurons that project to each respective region.

      We thank the reviewer for pointing out these errors and we corrected them. Autonomic firing activity has been changed to spontaneous firing activity. Expression files has been changed to expression levels. All the “depression behavior” have been changed to depression-like behaviors. In the Figure 2, all “xx-VTA” have been changed to “VTA-xx”.

      Reviewer #3 (Recommendations For The Authors):

      Recommendation 1 for improving the paper:

      Methodology: as opposed to sFig. 8 where the order through which mice were repeatedly tested is precise, such a key information is lacking in Fig. 6 as well as in the Methods section (for example, when such traumatic stress as forced swimming is performed with regard to the other tests?). Relevant to this point is the possible bias triggered by such chronological testing as exposure to the forced swim test likely affects the behaviors recorded in the other tests. Furthermore, the way this test is conducted is appealing as it is mentioned that the water depth was set to 10 cms which is quite low given that immobility scores might be affected by the ability of mice to stand on their tails.

      With regard to the elevated plus-maze, data are erroneously provided. Absolute values regarding open arm behaviors should be provided as percentages of the number of visits (or time spent therein) over the total (open + closed) number of arm visits. Indeed, closed arm visits should also be provided. This variable, also considered an index of locomotor activity, would allow the reader to exclude any effect of locomotion on the exploration in the open field.

      As they stand, data in the open field seem to indicate parallel changes at the center(center time) and the periphery (total distance), hence suggesting locomotor effects rather than anxiogenic effects. Data related to the center and the periphery should be clearly distinguished. Lastly, the number of weeks allowed for the mice to recover from surgeries aimed at delivering viruses are not mentioned. This is important as it could have affected the amplitude of the sensitivity to the stressors.

      We thank the reviewer for the suggestion. The lack information in Figure 6 and the Methods is now supplied. We apologize for the wrong number of “10 cm” in the forced swimming test, this has been corrected. The data concerning the elevated plus-maze are also changed based on the reviewer’s suggestion. For a possible role of locomotor effect, we tested the mice on the rota-rod test. From the result, there is no difference in locomotor activity between control and depressed-like mice (sFig.10G, sFig.12I and sFig.13G). We modified the experimental procedure timeline in Figure 6 and in the method- AAV for gene knockdown or overexpression and viral construct and injection, we added “Mice were singly housed with enough food and water to recover for 4-5 weeks after injection of virus, before behavior tests and electrophysiological recordings.” to report the number of weeks allowed for the mice to recover from surgeries aimed at delivering virus.

      Recommendation 2 for improving the paper:

      Results/conclusions: as yet mentioned, the authors make a confusion in the interpretation of their tail suspension tests and forced swimming tests. I acknowledge that such a confusion is frequent but it is important to note that the tests used by the authors were INITIALLY aimed at detecting the antidepressant effects of drugs under investigation. However, it is not because a test reveals such antidepressant properties that they also provide indices of depression. The authors will surely agree that it is unlikely that a 5-min test provides a model of a chronic pathology accounted for by a complex intrication between genetics and environmental factors. I would propose the authors to read for example Molendijk and De Kloet (Eur J Neurosci 2022). I think that the authors should just neutrally mention their results without any interpretation related to depression. On the other hand, what could have been interesting is to test whether the so-called "depressive-like" responses recorded in the study were sensitive to chronic antidepressant treatments. This would have allowed the authors to further suggest some relevance (if any) with depression-like pathologies.

      As we discussed above, we again agree with the reviewer’s concern. However, if as stated by the reviewer that “However, it is not because a test reveals such antidepressant properties that they also provide indices of depression”, then the experiments suggested by the reviewer “….. to test whether the so-called "depressive-like" responses recorded in the study were sensitive to chronic antidepressant treatments”

      Recommendation 3 for improving the paper:

      A close examination of the responses to CMUS or chronic restraint suggests that indeed two populations of animals were detected, possibly sensitive and resilient to these stressors. Did the authors try to examine this possibility?

      Based on the results of behavior test in CMUS and CRS, animals might be divided into two populations of animals highly-sensitive and moderately-sensitive ones.

      Recommendation 4 for improving the paper:

      There are some text changes that need to be performed:

      Page 2 line 46: ref 4 uses a social stress model which brings no clearcut evidence for it being a "depression" model. Indeed, this model can also be suggested to be a model of chronic anxiety (Kalueff et al., Science 2006; Chaouloff, Cell tissue Res 2013), hence indicating that VTA dopaminergic neurons might also be involved in anxiety.

      page 11, line 329: the references supporting the hypothesis that VTA DA neurons are linked to depression cannot be found in the reference list (10-15 do not correspond to the appropriate references).

      page 11, line 3341: reference 47 does not fit with the authors' assertion as it did not include any behavior.

      Fig. S8: body weight data are likely provided as changes rather than absolute values (e.g. 8 g)

      We agreed with the reviewer’s comments. The line 46“……such as depression states” has been changed to “such as depression- or anxiety-related states”. And we corrected the references in line 329 and 341. Finally, the body weight has been changed to the change in body weight.

      References:

      1. Um, K.B., et al., TRPC3 and NALCN channels drive pacemaking in substantia nigra dopaminergic neurons. Elife, 2021. 10.

      2. Urban, N., et al., Identification and Validation of Larixyl Acetate as a Potent TRPC6 Inhibitor. Mol Pharmacol, 2016. 89(1): p. 197-213.

      3. Zhong, P., et al., HCN2 channels in the ventral tegmental area regulate behavioral responses to chronic stress. Elife, 2018. 7.

      4. Liu, D., et al., Brain-derived neurotrophic factor-mediated projection-specific regulation of depressive-like and nociceptive behaviors in the mesolimbic reward circuitry. Pain, 2018. 159(1): p. 175.

      5. Walsh, J.J. and M.H. Han, The Heterogeneity of Ventral Tegmental Area Neurons: Projection Functions in a Mood-Related Context. Neuroscience, 2014. 282: p. 101-108.

      6. Khaliq, Z.M. and B.P. Bean, Pacemaking in dopaminergic ventral tegmental area neurons: depolarizing drive from background and voltage-dependent sodium conductances. J Neurosci, 2010. 30(21): p. 7401-13.

      7. Li, L., et al., Selective targeting of M-type potassium K(v) 7.4 channels demonstrates their key role in the regulation of dopaminergic neuronal excitability and depression-like behaviour. Br J Pharmacol, 2017. 174(23): p. 4277-4294.

      8. Friedman, A.K., et al., Enhancing depression mechanisms in midbrain dopamine neurons achieves homeostatic resilience. Science, 2014. 344(6181): p. 313-9.

      9. Lammel, S., et al., Diversity of transgenic mouse models for selective targeting of midbrain dopamine neurons. Neuron, 2015. 85(2): p. 429-38.

      10. Lammel, S., et al., Unique properties of mesoprefrontal neurons within a dual mesocorticolimbic dopamine system. Neuron, 2008. 57(5): p. 760-73.

    1. Author response:

      The following is the authors’ response to the original reviews.

      Public Reviews:  

      Reviewer #1 (Public Review): 

      Summary: 

      In their manuscript entitled 'The domesticated transposon protein L1TD1 associates with its ancestor L1 ORF1p to promote LINE-1 retrotransposition', Kavaklıoğlu and colleagues delve into the role of L1TD1, an RNA binding protein (RBP) derived from a LINE1 transposon. L1TD1 proves crucial for maintaining pluripotency in embryonic stem cells and is linked to cancer progression in germ cell tumors, yet its precise molecular function remains elusive. Here, the authors uncover an intriguing interaction between L1TD1 and its ancestral LINE-1 retrotransposon. 

      The authors delete the DNA methyltransferase DNMT1 in a haploid human cell line (HAP1), inducing widespread DNA hypo-methylation. This hypomethylation prompts abnormal expression of L1TD1. To scrutinize L1TD1's function in a DNMT1 knock-out setting, the authors create DNMT1/L1TD1 double knock-out cell lines (DKO). Curiously, while the loss of global DNA methylation doesn't impede proliferation, additional depletion of L1TD1 leads to DNA damage and apoptosis.  

      To unravel the molecular mechanism underpinning L1TD1's protective role in the absence of DNA methylation, the authors dissect L1TD1 complexes in terms of protein and RNA composition. They unveil an association with the LINE-1 transposon protein L1-ORF1 and LINE-1 transcripts, among others.  

      Surprisingly, the authors note fewer LINE-1 retro-transposition events in DKO cells than in DNMT1 KO alone.  

      Strengths: 

      The authors present compelling data suggesting the interplay of a transposon-derived human RNA binding protein with its ancestral transposable element. Their findings spur interesting questions for cancer types, where LINE1 and L1TD1 are aberrantly expressed.  

      Weaknesses: 

      Suggestions for refinement:  

      The initial experiment, inducing global hypo-methylation by eliminating DNMT1 in HAP1 cells, is intriguing and warrants a more detailed description. How many genes experience misregulation or aberrant expression? What phenotypic changes occur in these cells? 

      This is an excellent suggestion. We have gene expression data on WT versus DNMT1 KO HAP1 cells and have included them now as Suppl. Figure S1. The  transcriptome analysis of DNMT1 KO cells showed hundreds of deregulated genes upon DNMT1 ablation. As expected, the majority were up-regulated and gene ontology analysis revealed that among the strongest up-regulated genes were gene clusters with functions in “regulation of transcription from RNA polymerase II promoter” and “cell differentiation” and genes encoding proteins with KRAB domains. In addition, the de novo methyltransferases DNMT3A and DNMT3B were up-regulated in DNMT1 KO cells suggesting the set-up of compensatory mechanisms in these cells. 

      Why did the authors focus on L1TD1? Providing some of this data would be helpful to understand the rationale behind the thorough analysis of L1TD1. 

      We have previously discovered that conditional deletion of the maintenance DNA methyltransferase DNMT1 in the murine epidermis results not only in the up-regulation of mobile elements, such as IAPs but also the induced expression of L1TD1 ([1], Suppl. Table 1 and Author response image 1). Similary, L1TD1 expression was induced by treatment of primary human keratinocytes or squamous cell carcinoma cells with the DNMT inhibitor azadeoxycytidine (Author response images 2 and 3). These findings are in accordance with the observation  that inhibition of DNA methyltransferase activity by aza-deoxycytidine in human non-small cell lung cancer cells (NSCLCs) results in up-regulation of L1TD1 [2]. Our interest in L1TD1 was further fueled by reports on a potential function of L1TD1 as prognostic tumor marker. We have included this information in the last paragraph of the Introduction in the revised manuscript.

      Author response image 1. RT-qPCR of L1TD1 expression in cultured murine control and Dnmt1 Δ/Δker keratinocytes. mRNA levels of L1td1 were analyzed in keratinocytes isolated at P5 from conditional Dnmt1 knockout mice [1]. Hprt expression was used for normalization of mRNA levels and wildtype control was set to 1. Data represent means ±s.d. with n=4. **P < 0.01 (paired t-test). 

      Author response image 2. RT-qPCR analysis of L1TD1 expression in primary human keratinocytes. Cells were treated with 5-aza-2-deoxycidine for 24 hours or 48 hours, with PBS for 48 hours or were left untreated. 18S rRNA expression was used for normalization of mRNA levels and PBS control was set to 1. Data represent means ±s.d. with n=3. **P < 0.01 (paired t-test).

      Author response image 3. Induced L1TD1 expression upon DNMT inhibition in squamous cell carcinoma cell lines SCC9 and SCCO12. Cells were treated with 5-aza-2-deoxycidine for 24 hours, 48 hours or 6 days. (A) Western blot analysis of L1TD1 protein levels using beta-actin as loading control. (B) Indirect immunofluorescence microscopy analysis of L1TD1 expression in SCC9 cells. Nuclear DNA was stained with DAPI. Scale bar: 10 µm. (C)  RT-qPCR analysis of L1TD1 expression in primary human keratinocytes. Cells were treated with 5-aza-2deoxycidine for 24 hours or 48 hours, with PBS for 48 hours or were left untreated. 18S rRNA expression was used for normalization of mRNA levels and PBS control was set to 1. Data represent means ±s.d. with n=3. *P < 0.05, **P < 0.01 (paired t-test).

      The finding that L1TD1/DNMT1 DKO cells exhibit increased apoptosis and DNA damage but decreased L1 retro-transposition is unexpected. Considering the DNA damage associated with retro-transposition and the DNA damage and apoptosis observed in L1TD1/DNMT1 DKO cells, one would anticipate the opposite outcome. Could it be that the observation of fewer transposition-positive colonies stems from the demise of the most transposition-positive colonies? Further exploration of this phenomenon would be intriguing. 

      This is an important point and we were aware of this potential problem. Therefore, we calibrated the retrotransposition assay by transfection with a blasticidin resistance gene vector to take into account potential differences in cell viability and blasticidin sensitivity. Thus, the observed reduction in L1 retrotransposition efficiency is not an indirect effect of reduced cell viability. We have added a corresponding clarification in the Results section on page 8, last paragraph. 

      Based on previous studies with hESCs and germ cell tumors [3], it is likely that, in addition to its role in retrotransposition, L1TD1 has further functions in the regulation of cell proliferation and differentiation. L1TD1 might therefore attenuate the effect of DNMT1 loss in KO cells generating an intermediate phenotype (as pointed out by Reviewer 2) and simultaneous loss of both L1TD1 and DNMT1 results in more pronounced effects on cell viability. This is in agreement with the observation that a subset of L1TD1 associated transcripts encode proteins involved in the control of cell division and cell cycle. It is possible that subtle changes in the expression of these protein that were not detected in our mass spectrometry approach contribute to the antiproliferative effect of L1TD1 depletion as discussed in the Discussion section of the revised manuscript. 

      Reviewer #2 (Public Review):           

      In this study, Kavaklıoğlu et al. investigated and presented evidence for the role of domesticated transposon protein L1TD1 in enabling its ancestral relative, L1 ORF1p, to retrotranspose in HAP1 human tumor cells. The authors provided insight into the molecular function of L1TD1 and shed some clarifying light on previous studies that showed somewhat contradictory outcomes surrounding L1TD1 expression. Here, L1TD1 expression was correlated with L1 activation in a hypomethylation-dependent manner, due to DNMT1 deletion in the HAP1 cell line. The authors then identified L1TD1-associated RNAs using RIP-Seq, which displays a disconnect between transcript and protein abundance (via Tandem Mass Tag multiplex mass spectrometry analysis). The one exception was for L1TD1 itself, which is consistent with a model in which the RNA transcripts associated with L1TD1 are not directly regulated at the translation level. Instead, the authors found the L1TD1 protein associated with L1-RNPs, and this interaction is associated with increased L1 retrotransposition, at least in the contexts of HAP1 cells. Overall, these results support a model in which L1TD1 is restrained by DNA methylation, but in the absence of this repressive mark, L1TD1 is expressed and collaborates with L1 ORF1p (either directly or through interaction with L1 RNA, which remains unclear based on current results), leads to enhances L1 retrotransposition. These results establish the feasibility of this relationship existing in vivo in either development, disease, or both.   

      Recommendations for the authors:

      Reviewer #2 (Recommendations For The Authors):        

      Major 

      (1) The study only used one knockout (KO) cell line generated by CRISPR/Cas9. Considering the possibility of an off-target effect, I suggest the authors attempt one or both of these suggestions. 

      A) Generate or acquire a similar DMNT1 deletion that uses distinct sgRNAs, so that the likelihood of off-targets is negligible. A few simple experiments such as qRT-PCR would be sufficient to suggest the same phenotype.  

      B) Confirm the DNMT1 depletion also by siRNA/ASO KD to phenocopy the KO effect.  (2) In addition to the strategies to demonstrate reproducibility, a rescue experiment restoring DNMT1 to the KO or KD cells would be more convincing. (Partial rescue would suffice in this case, as exact endogenous expression levels may be hard to replicate). 

      We have undertook several approaches to study the effect of DNMT1 loss or inactivation: As described above, we have generated a conditional KO mouse with ablation of DNMT1 in the epidermis. DNMT1-deficient keratinocytes isolated from these mice show a significant increase in L1TD1 expression.  In addition, treatment of primary human keratinocytes and two squamous cell carcinoma cell lines with the DNMT inhibitor aza-deoxycytidine led to upregulation of L1TD1 expression. Thus, the derepression of L1TD1 upon loss of DNMT1 expression or activity is not a clonal effect. Also, the spectrum of RNAs identified in RIP experiments as L1TD1-associated transcripts in HAP1 DNMT1 KO cells showed a strong overlap with the RNAs isolated by a related yet different method in human embryonic stem cells. When it comes to the effect of L1TD1 on L1-1 retrotranspostion, a recent study has reported a similar effect of L1TD1 upon overexpression in HeLa cells [4].  

      All of these points together help to convince us that our findings with HAP1 DNMT KO are in agreement with results obtained in various other cell systems and are therefore not due to off-target effects. With that in mind, we would pursue the suggestion of Reviewer 1 to analyze the effects of DNA hypomethylation upon DNMT1 ablation.

      (3) As stated in the introduction, L1TD1 and ORF1p share "sequence resemblance" (Martin 2006). Is the L1TD1 antibody specific or do we see L1 ORF1p if Fig 1C were uncropped?  (6) Is it possible the L1TD1 antibody binds L1 ORF1p? This could make Figure 2D somewhat difficult to interpret. Some validation of the specificity of the L1TD1 antibody would remove this concern (see minor concern below).  

      This is a relevant question. We are convinced that the L1TD1 antibody does not crossreact with L1 ORF1p for the following reasons: Firstly, the antibody does not recognize L1 ORF1p (40 kDa) in the  uncropped Western blot for Figure 1C (Author response image 4A). Secondly, the L1TD1 antibody gives only background signals in DKO cells in the  indirect immunofluorescence experiment shown in Figure 1E of the manuscript. 

      Thirdly, the immunogene sequence of L1TD1 that determines the specificity of the antibody was checked in the antibody data sheet from Sigma Aldrich. The corresponding epitope is not present in the L1 ORF1p sequence. Finally, we have shown that the ORF1p antibody does not cross-react with L1TD1 (Author response image 4B).

      Author response image 4. (A) Uncropped L1TD1 Western blot shown in Figure 1C. An unspecific band is indicated by an asterisk. (B) Westernblot analysis of WT, KO and DKO cells with L1 ORF1p antibody.

      (4) In abstract (P2), the authors mentioned that L1TD1 works as an RNA chaperone, but in the result section (P13), they showed that L1TD1 associates with L1 ORF1p in an RNAindependent manner. Those conclusions appear contradictory. Clarification or revision is required. 

      Our findings that both proteins bind L1 RNA, and that L1TD1 interacts with ORF1p are compatible with a scenario where L1TD1/ORF1p heteromultimers bind to L1 RNA. The additional presence of L1TD1 might thereby enhance the RNA chaperone function of ORF1p. This model is visualized now in Suppl. Figure S7C. 

      (5) Figure 2C fold enrichment for L1TD1 and ARMC1 is a bit difficult to fully appreciate. A 100 to 200-fold enrichment does not seem physiological. This appears to be a "divide by zero" type of result, as the CT for these genes was likely near 40 or undetectable. Another qRT-PCRbased approach (absolute quantification) would be a more revealing experiment. 

      This is the validation of the RIP experiments and the presentation mode is specifically developed for quantification of RIP assays (Sigma Aldrich RIP-qRT-PCR: Data Analysis Calculation Shell). The unspecific binding of the transcript in the absence of L1TD1 in DNMT1/L1TD1 DKO cells is set to 1 and the value in KO cells represents the specific binding relative the unspecific binding. The calculation also corrects for potential differences in the abundance of the respective transcript in the two cell lines. This is not a physiological value but the quantification of specific binding of transcripts to L1TD1. GAPDH as negative control shows no enrichment, whereas specifically associated transcripts show strong enrichement. We have explained the details of RIPqRT-PCR evaluation in Materials and Methods (page 14) and the legend of Figure 2C in the revised manuscript.       

      (6) Is it possible the L1TD1 antibody binds L1 ORF1p? This could make Figure 2D somewhat difficult to interpret. Some validation of the specificity of the L1TD1 antibody would remove this concern (see minor concern below).            

      See response to (3).  

      (7) Figure S4A and S4B: There appear to be a few unusual aspects of these figures that should be pointed out and addressed. First, there doesn't seem to be any ORF1p in the Input (if there is, the exposure is too low). Second, there might be some L1TD1 in the DKO (lane 2) and lane 3. This could be non-specific, but the size is concerning. Overexposure would help see this.

      The ORF1p IP gives rise to strong ORF1p signals in the immunoprecipitated complexes even after short exposure. Under these contions ORF1p is hardly detectable in the input. Regarding the faint band in DKO HAP1 cells, this might be due to a technical problem during Western blot loading. Therefore, the input samples were loaded again on a Western blot and analyzed for the presence of ORF1p, L1TD1 and beta-actin (as loading control) and shown as separate panel in Suppl. Figure S4A. 

      (8) Figure S4C: This is related to our previous concerns involving antibody cross-reactivity. Figure 3E partially addresses this, where it looks like the L1TD1 "speckles" outnumber the ORF1p puncta, but overlap with all of them. This might be consistent with the antibody crossreacting. The western blot (Figure 3C) suggests an upregulation of ORF1p by at least 2-3x in the DKO, but the IF image in 3E is hard to tell if this is the case (slightly more signal, but fewer foci). Can you return to the images and confirm the contrast are comparable? Can you massively overexpose the red channel in 3E to see if there is residual overlap? 

      In Figure 3E the L1TD1 antibody gives no signal in DNMT1/L1TD1 DKO cells confirming that it does not recognize ORF1p. In agreement with the Western blot in Figure 3C the L1 ORF1p signal in Figure 3E is stronger in DKO cells. In DNMT1 KO cells the L1 ORF1p antibody does not recognize all L1TD1 speckles. This result is in agreement with the Western blot shown above in Figure R4B and indicates that the L1 ORF1p antibody does not recognize the L1TD1 protein. The contrast is comparable and after overexposure there are still L1TD1 specific speckles. This might be due to differences in abundance of the two proteins.

      (9) The choice of ARMC1 and YY2 is unclear. What are the criteria for the selection?

      ARMC1 was one of the top hits in a pilot RIP-seq experiment (IP versus input and IP versus  IgG IP). In the actual RIP-seq experiment with DKO HAP1 cells instead of IgG IP as a negative control, we found ARMC1 as an enriched hit, although it was not among the top 5 hits. The results from the 2nd RIP-seq further confirmed the validity of ARMC1 as an L1TD1-interacting transcript. YY2 was of potential biological relevance as an L1TD1 target due to the fact that it is a processed pseudogene originating from YY1 mRNA as a result of retrotransposition. This is mentioned on page 6 of the revised manuscript.

      (10) (P16) L1 is the only protein-coding transposon that is active in humans. This is perhaps too generalized of a statement as written. Other examples are readily found in the literature. Please clarify.  

      We will tone down this statement in the revised manuscript. 

      (11) In both the abstract and last sentence in the discussion section (P17), embryogenesis is mentioned, but this is not addressed at all in the manuscript. Please refrain from implying normal biological functions based on the results of this study unless appropriate samples are used to support them.

      Much of the published data on L1TD1 function are related to embryonic stem cells [3-7]. Therefore, it is important to discuss our findings in the context of previous reports.

      (12) Figure 3E: The format of Figures 1A and 3E are internally inconsistent. Please present similar data/images in a cohesive way throughout the manuscript.  

      We show now consistent IF Figures in the revised manuscript.

      Minor: 

      (1) Intro:           

      - Is L1Td1 in mice and Humans? How "conserved" is it and does this suggest function?  

      Murine and human L1TD1 proteins share 44% identity on the amino acid level and it was suggested that the corresponding genes were under positive selection during evolution with functions in transposon control and maintenance of pluripotency [8].  

      - Why HAP1? (Haploid?) The importance of this cell line is not clear.          

      HAP1 is a nearly haploid human cancer cell line derived from the KBM-7 chronic myelogenous leukemia (CML) cell line [9, 10]. Due to its haploidy is perfectly suited and widely used for loss-of-function screens and gene editing. After gene editing  cells can be used in the nearly haploid or in the diploid state. We usually perform all experiments with diploid HAP1 cell lines.  Importantly, in contrast to other human tumor cell lines, this cell line tolerates ablation of DNMT1. We have included a corresponding explanation in the revised manuscript on page 5, first paragraph.

      - Global methylation status in DNMT1 KO? (Methylations near L1 insertions, for example?) 

      The HAP1 DNMT1 KO cell line with a 20 bp deletion in exon 4 used in our study was validated in the study by Smits et al. [11]. The authors report a significant reduction in overall DNA methylation. However, we are not aware of a DNA methylome study on this cell line. We show now data on the methylation of L1 elements in HAP1 cells and upon DNMT1 deletion in the revised manuscript in Suppl. Figure S1B.

      (2) Figure 1:  

      - Figure 1C. Why is LMNB used instead of Actin (Fig1D)?  

      We show now beta-actin as loading control in the revised manuscript.  

      - Figure 1G shows increased Caspase 3 in KO, while the matching sentence in the result section skips over this. It might be more accurate to mention this and suggest that the single KO has perhaps an intermediate phenotype (Figure 1F shows a slight but not significant trend). 

      We fully agree with the reviewer and have changed the sentence on page 6, 2nd paragraph accordingly.  

      - Would 96 hrs trend closer to significance? An interpretation is that L1TD1 loss could speed up this negative consequence. 

      We thank the reviewer for the suggestion. We have performed a time course experiment with 6 biological replicas for each time point up to 96 hours and found significant changes in the viability upon loss of DNMT1 and again significant reduction in viability upon additional loss of L1TD1 (shown in Figure 1F). These data suggest that as expexted loss of DNMT1 leads to significant reduction viability and that additional ablation of L1TD1 further enhances this effect.

      - What are the "stringent conditions" used to remove non-specific binders and artifacts (negative control subtraction?) 

      Yes, we considered only hits from both analyses, L1TD1 IP in KO versus input and L1TD1 IP in KO versus L1TD1 IP in DKO. This is now explained in more detail in the revised manuscript on page 6, 3rd paragraph.  

      (3) Figure 2:  

      - Figure 2A is a bit too small to read when printed. 

      We have changed this in the revised manuscript.

      - Since WT and DKO lack detectable L1TD1, would you expect any difference in RIP-Seq results between these two?

      Due to the lack of DNMT1 and the resulting DNA hypomethylation, DKO cells are more similar to KO cells than WT cells with respect to the expressed transcripts.

      - Legend says selected dots are in green (it appears blue to me). 

      We have changed this in the revised manuscript.           

      - Would you recover L1 ORF1p and its binding partners in the KO? (Is the antibody specific in the absence of L1TD1 or can it recognize L1?) I noticed an increase in ORF1p in the KO in Figure 3C.  

      Thank you for the suggestion. Yes, L1 ORF1p shows slightly increased expression in the proteome analysis and we have marked the corresponding dot in the Volcano plot (Figure 3A).

      - Should the figure panel reference near the (Rosspopoff & Trono) reference instead be Sup S1C as well? Otherwise, I don't think S1C is mentioned at all. 

      - What are the red vs. green dots in 2D? Can you highlight ERV and ALU with different colors? 

      We added the reference to Suppl. Figure S1C (now S3C) in the revised manuscript. In Figure 2D L1 elements are highlighted in green, ERV elements in yellow, and other associated transposon transcripts in red.     

      - Which L1 subfamily from Figure 2D is represented in the qRT-PCR in 2E "LINE-1"? Do the primers match a specific L1 subfamily? If so, which? 

      We used primers specific for the human L1.2 subfamily. 

      - Pulling down SINE element transcripts makes some sense, as many insertions "borrow" L1 sequences for non-autonomous retro transposition, but can you speculate as to why ERVs are recovered? There should be essentially no overlap in sequence. 

      In the L1TD1 evolution paper [8], a potential link between L1TD1 and ERV elements was discussed: 

      "Alternatively, L1TD1 in sigmodonts could play a role in genome defense against another element active in these genomes. Indeed, the sigmodontine rodents have a highly active family of ERVs, the mysTR elements [46]. Expansion of this family preceded the death of L1s, but these elements are very active, with 3500 to 7000 species-specific insertions in the L1-extinct species examined [47]. This recent ERV amplification in Sigmodontinae contrasts with the megabats (where L1TD1 has been lost in many species); there are apparently no highly active DNA or RNA elements in megabats [48]. If L1TD1 can suppress retroelements other than L1s, this could explain why the gene is retained in sigmodontine rodents but not in megabats." 

      Furthermore, Jin et al. report the binding of L1TD1 to repetitive sequences in transcripts [12]. It is possible that some of these sequences are also present in ERV RNAs.

      - Is S2B a screenshot? (the red underline). 

      No, it is a Powerpoint figure, and we have removed the red underline.

      (4) Figure 3: 

      - Text refers to Figure 3B as a western blot. Figure 3B shows a volcano plot. This is likely 3C but would still be out of order (3A>3C>3B referencing). I think this error is repeated in the last result section. 

      - Figure and legends fail to mention what gene was used for ddCT method (actin, gapdh, etc.). 

      - In general, the supplemental legends feel underwritten and could benefit from additional explanations. (Main figures are appropriate but please double-check that all statistical tests have been mentioned correctly).

      Thank you for pointing this out. We have corrected these errors in the revised manuscript.

      (5) Discussion: 

      -Aluy connection is interesting. Is there an "Alu retrotransposition reporter assay" to test whether L1TD1 enhances this as well? 

      Thank you for the suggestion. There is indeed an Alu retrotransposition reporter assay reported be Dewannieux et al. [13]. The assay is based on a Neo selection marker. We have previously tested a Neo selection-based L1 retrotransposition reporter assay, but this system failed to properly work in HAP1 cells, therefore we switched to a blasticidinbased L1 retrotransposition reporter assay. A corresponding blasticidin-based Alu retrotransposition reporter assay might be interesting for future studies (mentioned in the Discussion, page 11 paragraph 4 of the revised manuscript.

      (6) Material and Methods       : 

      - The number of typos in the materials and methods is too numerous to list. Instead, please refer to the next section that broadly describes the issues seen throughout the manuscript. 

      Writing style  

      (1) Keep a consistent style throughout the manuscript: for example, L1 or LINE-1 (also L1 ORF1p or LINE-1 ORF1p); per or "/"; knockout or knock-out; min or minute; 3 times or three times; media or medium. Additionally, as TE naming conventions are not uniform, it is important to maintain internal consistency so as to not accidentally establish an imprecise version. 

      (2) There's a period between "et al" and the comma, and "et al." should be italic. 

      (3) The authors should explain what the key jargon is when it is first used in the manuscript, such as "retrotransposon" and "retrotransposition".    

      (4) The authors should show the full spelling of some acronyms when they use it for the first time, such as RNA Immunoprecipitation (RIP).  

      (5) Use a space between numbers and alphabets, such as 5 µg.  

      (6) 2.0 × 105 cells, that's not an "x".  

      (7) Numbers in the reference section are lacking (hard to parse).  

      (8) In general, there are a significant number of typos in this draft which at times becomes distracting. For example, (P3) Introduction: Yet, co-option of TEs thorough (not thorough, it should be through) evolution has created so-called domesticated genes beneficial to the gene network in a wide range of organisms. Please carefully revise the entire manuscript for these minor issues that collectively erode the quality of this submission.  

      Thank you for pointing out these mistakes. We have corrected them in the revised manuscript. A native speaker from our research group has carefully checked the paper. In summary, we have added Supplementary Figure S7C and have changed Figures 1C, 1E, 1F, 2A, 2D, 3A, 4B, S3A-D, S4B and S6A based on these comments. 

      REFERENCES

      (1) Beck, M.A., et al., DNA hypomethylation leads to cGAS-induced autoinflammation in the epidermis. EMBO J, 2021. 40(22): p. e108234.

      (2) Altenberger, C., et al., SPAG6 and L1TD1 are transcriptionally regulated by DNA methylation in non-small cell lung cancers. Mol Cancer, 2017. 16(1): p. 1.

      (3) Narva, E., et al., RNA-binding protein L1TD1 interacts with LIN28 via RNA and is required for human embryonic stem cell self-renewal and cancer cell proliferation. Stem Cells, 2012. 30(3): p. 452-60.

      (4) Jin, S.W., et al., Dissolution of ribonucleoprotein condensates by the embryonic stem cell protein L1TD1. Nucleic Acids Res, 2024. 52(6): p. 3310-3326.

      (5) Emani, M.R., et al., The L1TD1 protein interactome reveals the importance of posttranscriptional regulation in human pluripotency. Stem Cell Reports, 2015. 4(3): p. 519-28.

      (6) Santos, M.C., et al., Embryonic Stem Cell-Related Protein L1TD1 Is Required for Cell Viability, Neurosphere Formation, and Chemoresistance in Medulloblastoma. Stem Cells Dev, 2015. 24(22): p. 2700-8.

      (7) Wong, R.C., et al., L1TD1 is a marker for undifferentiated human embryonic stem cells. PLoS One, 2011. 6(4): p. e19355.

      (8) McLaughlin, R.N., Jr., et al., Positive selection and multiple losses of the LINE-1-derived L1TD1 gene in mammals suggest a dual role in genome defense and pluripotency. PLoS Genet, 2014. 10(9): p. e1004531.

      (9) Andersson, B.S., et al., Ph-positive chronic myeloid leukemia with near-haploid conversion in vivo and establishment of a continuously growing cell line with similar cytogenetic pattern. Cancer Genet Cytogenet, 1987. 24(2): p. 335-43.

      (10) Carette, J.E., et al., Ebola virus entry requires the cholesterol transporter Niemann-Pick C1. Nature, 2011. 477(7364): p. 340-3.

      (11) Smits, A.H., et al., Biological plasticity rescues target activity in CRISPR knock outs. Nat Methods, 2019. 16(11): p. 1087-1093.

      (12) Jin, S.W., et al., Dissolution of ribonucleoprotein condensates by the embryonic stem cell protein L1TD1. Nucleic Acids Res, 2024.

      (13) Dewannieux, M., C. Esnault, and T. Heidmann, LINE-mediated retrotransposition of marked Alu sequences. Nat Genet, 2003. 35(1): p. 41-8.

    1. Author response:

      The following is the authors’ response to the original reviews

      Public Reviews:

      Reviewer #1 (Public review):

      Summary:

      The study by Deng et al reports single-cell expression analysis of developing mouse hearts and examines the requirements for cardiac fibroblasts in heart maturation. Much of this work is overlapping with previous studies, but the single-cell gene expression data may be useful to investigators in the field. The significance and scope of new findings are limited and major conclusions are largely based on correlative data.

      Strengths:

      The strengths of the manuscript are the new single-cell datasets and comprehensive approach to ablating cardiac fibroblasts in pre and postnatal development in mice.

      Weaknesses:

      There are several major weaknesses in the analysis and interpretation of the results.

      (1) The major conclusions regarding collagen signaling and heart maturation are based on gene expression patterns and are not functionally validated. The potential downstream signaling pathways were not examined and known structural contributions of fibrillar collagen to heart maturation are not discussed.

      We thank the reviewer for the comment. In this study, we mainly focused on the functional analysis of fibroblasts in heart development at embryonic and neonatal stages by using cell ablation system and single cell mRNA sequencing analysis. The further functional analysis of collagen pathway is interesting but out of the scope of this study. We will continue this line of research and share the results in the future. Moreover, through the analysis of single cell mRNA-sequencing data, we have predicted the downstream genes that are regulated by the collagen pathway in Fig 5C. We have also added sentences to highlight the structural role of collagen in affecting the related heart developmental processes.

      (2) The heterogeneity of fibroblast populations and contributions to multiple structures in the developing heart are not well-considered in the analysis. The developmental targeting of fibroblasts will likely affect multiple structures in the embryonic heart and other organs. Lethality is described in some of these studies, but additional analysis is needed to determine the effects on heart morphogenesis or other organs beyond the focus on cardiomyocyte maturation being reported. In particular, the endocardial cushions and developing valves are likely to be affected in the prenatal ablations, but these structures are not included in the analyses.

      We thank the reviewer for the comment. We have included a new figure presenting the fibroblast heterogeneity in developing hearts (Fig S3). We have also compared the valve structural differences at E18.5 (Fig S11).

      (3) ECM complexity and extensive previous work on specific ECM proteins in heart development and maturation are not incorporated into the current study. Different types of collagen (basement membrane Col4, filamentous Col6, and fibrillar Col1) are known to be expressed in fibroblast populations in the developing heart and have been studied extensively. Much also has been reported for other ECM components mentioned in the current work.

      We thank the reviewer for the comment. We agree that the ECM is complex, and the functions of many of its components have been previously reported, as mentioned in the introduction. In this study, our focus is to analyze the spatial and temporal expression patterns of various ECM genes in fibroblasts throughout developmental progression (Fig. S5–7). To further acknowledge previous work, we have added additional sentences and cited relevant literature on the role of collagen genes in developing hearts (page 4).

      Reviewer #2 (Public review):

      This study aims to elucidate the role of fibroblasts in regulating myocardium and vascular development through signaling to cardiomyocytes and endothelial cells. This focus is significant, given that fibroblasts, cardiomyocytes, and vascular endothelial cells are the three primary cell types in the heart. The authors employed a Pdgfra-CreER-controlled diphtheria toxin A (DTA) system to ablate fibroblasts at various embryonic and postnatal stages, characterizing the resulting cardiac defects, particularly in myocardium and vasculature development. scRNA-seq analysis of the ablated hearts identified collagen as a crucial signaling molecule from fibroblasts that influences the development of cardiomyocytes and vascular endothelial cells. This is an interesting manuscript; however, there are several major issues, including an over-reliance on the scRNA-seq data, which shows inconsistencies between replicates. Some of the major issues are described below.

      The comments are the same as the comments for “Recommendations for the authors”. Please see the responses below.

      Reviewer #3 (Public review):

      The authors investigated fibroblasts' communication with key cell types in developing and neonatal hearts, with a focus on the critical roles of fibroblast-cardiomyocyte and fibroblast-endothelial cell networks in cardiac morphogenesis. They tried to map the spatial distribution of these cell types and reported the major pathways and signaling molecules driving the communication. They also used Cre-DTA system to ablate Pdgfra labeled cells and observed myocardial and endothelial cell defects at development. They screened the pathways and genes using sequencing data of ablated hearts. Lastly, they reported compensatory collagen expression in long-term ablated neonate hearts. Overall, this study provides us with important insight into fibroblasts' roles in cardiac development and will be a powerful resource for collagens and ECM-focused research.

      Strengths:

      The authors utilized good analyzing tools to investigate multiple databases of single-cell sequencing and Multiseq. They identified significant pathways and cellular and molecular interactions of fibroblasts. Additionally, they compared some of their analytic findings with a human database, and identified several groups of ECM genes with varying roles in mice.

      Weaknesses:

      This study is majorly based on sequencing data analysis. At the bench, they used a very strident technique to study fibroblast functions by ablating one of the major cell populations of the heart. Considering the importance of the fibroblast population, intriguing in vivo findings were expected. Also, they analyzed the downstream genes in ablated hearts, but did not execute any experimental validation for any of the targets.

      Recommendations for the authors:

      Reviewing Editor Comments:

      All three reviewers found the large amount of scRNA-Seq data compelling and valuable, and they noted that the study's conclusions based on the scRNA Seq and fibroblast ablating align closely with previously published studies. Therefore, a more thorough discussion and integration of the current findings with prior studies are recommended. Each reviewer provided specific feedback to improve the manuscript, correct errors, and strengthen the overall presentation, and please edit the manuscript accordingly. Additionally, further validation of the scRNA-Seq data through more data analysis, reference comparisons, or additional experiments is encouraged.

      Reviewer #1 (Recommendations for the authors):

      (1) The heterogeneity of fibroblasts and ECM components in the developing heart needs to be considered in the analysis and description of results. There are extensive reports in both of these areas that would inform the gene expression and ablation studies being reported.

      We thank the reviewer for the comment. We have added a supplemental figure (Fig. S3) analyzing the heterogeneity of fibroblasts during development and described the results on page 3 and 4. Through the analysis of single-cell mRNA sequencing data, we identified four distinct populations of fibroblasts and further performed RNA scope to examine their spatial locations. Additionally, we agree with the reviewer that there are many types of ECM components, which we have addressed in the introduction (page 2). Furthermore, we have conducted a detailed analysis of the spatial and temporal expression patterns of ECM genes throughout developmental progression (Figs. S5–7).

      (2) One of the novel aspects of the work is the prenatal ablation of cardiac fibroblasts. Embryonic lethality was observed in some cases, but the specific cardiac structural anomalies or potential vascular effects were not described. The contributing role of cardiac fibroblasts to valvuloseptal development, which was likely affected in these studies, was not described.

      We thank the reviewer for the comment. Since the heart sections were not initially prepared to compare valve differences between control and ablation conditions, most sections do not include valve structures. However, in the small subset of sections that do contain valves, we have compared valve structures in control and ablated hearts at E18.5 following three doses of tamoxifen treatment from E15.5 to E17.5. In mutants, the valves appear shorter compared to controls. Specifically, we observed that in control hearts, the mitral valve was already connected to the papillary muscle, whereas in ablated hearts, the valve leaflet at similar position was not. We have included these images as a new supplemental figure (Fig. S11). Regarding vascular defects, we have described them in Fig. 3C and 3F.

      (3) The major conclusions regarding collagen signaling and heart development are based on correlations in gene expression and are not validated by functional data. What are the downstream signaling pathways affected and are they affected during development or with ablation? The main conclusions of the study do not take into account well-known structural functions of collagen in the developing heart.

      We thank the reviewer for the comment. Through regulatory prediction analysis, we identified the collagen ligands Col1a1, Col5a1, and Col4a1 from the collagen family (Fig. 5C), which regulate multiple genes in cardiomyocytes, including Masp1. Masp1 is a member of the lectin complement pathway and potentially regulates cardiomyocyte migration during development. These collagen ligands also regulate multiple mitochondria-related genes, such as Etfa, Ndufb10, Ndufs6, and Slc25a4, which are potentially important for cardiomyocyte development and maturation. Moreover, we agree with the reviewer that collagen is an important structural ECM protein, and its deletion or reduction could cause heart developmental defects due to its structural role. We have added a discussion on this possibility (page 8).

      (4) The postnatal ablation studies are very similar to studies with the same mouse lines reported by Kurabara et al 2022 in JMCC (PMID 35569524) which came to similar conclusions and was not cited in the current work.

      We thank the reviewer for the comment and apologize for overlooking this study. We have now included the citation on page 8.

      (5) The discussion of a regenerative response with DTA ablation of fibroblasts is confusing. Proliferation was examined in cardiomyocytes which lose their regenerative capacity after birth in mice. However, cardiac fibroblasts can proliferate in response to injury throughout life which is not really a regenerative process.

      We appreciate the reviewer’s comment. To avoid confusion, we have replaced the term "regeneration" with "response to cell loss" and "compensation."

      (6) Some of the descriptions of single-cell expression data are overstated (Page 7). Regulatory interactions, signaling pathway activation, or function cannot be determined from gene expression data alone.

      We thank the reviewer for the comment. We agree that these conclusions rely on results from multiple assays. We have weakened the description of the analysis by emphasizing that the findings are predictive results from scRNA-seq analysis.

      (7) In the last paragraph of the discussion "data not shown" should be shown or this information should be deleted. As written, the discussion does not present a clear description of what major new findings are being reported or why they are significant. The new insights into heart development are not specified.

      We thank the reviewer for the comment. We have added the data as a supplemental figure (Fig. S19). Since this paragraph is part of the discussion, we believe the results are not conclusive at this stage and require further research to explore the potential protective role of fibroblast ablation in neonatal hearts.

      Minor comments.

      (1) Figure legends are missing information needed to understand what is being shown. For example, in Figure 2, collagen is visualized using CHP staining.

      Thanks. We have gone through all figure legends to ensure that all necessary information has been provided.

      (2) The hearts in Figure S15 are upside down.

      Thanks. We have updated the figure.

      (3) In Figure S16A, "brian" should be "brain".

      Thanks. We have updated it.

      Reviewer #2 (Recommendations for the authors):

      This is an interesting manuscript; however, there are several major issues, including an overreliance on the scRNA-seq data, which shows inconsistencies between replicates. Some of the major issues are described below.

      (1) The CD31 immunostaining data (Figures 3B-G) indicate a reduction in endothelial cell numbers following fibroblast deletion using PdgfraCreER+/-; RosaDTA+/- mice. However, the scRNA-seq data show no percentage change in the endothelial cell population (Figure 4D). Furthermore, while the percentage of Vas_ECs decreased in ablated samples at E16.5, the results at E18.5 were inconsistent, showing an increase in one replicate and a decrease in another, raising concerns about the reliability of the RNA-seq findings.

      We thank the reviewer for the comment. We believe that measuring cell proportions in scRNA-seq results is sensitive and relies on a high number of total and target cells, similar to other cell counting assays such as FACS. As the reviewer pointed out, the proportions of Vas_EC in E18.5 replicates are inconsistent. Specifically, Col_4 at E18.5 showed a relatively low proportion of Vas_EC. Upon examining the cell numbers in each sample, we found that Col_4 had the lowest number of recovered cells, with approximately 760 in total, whereas the other samples had more than 920 cells each. Additionally, since immunofluorescence staining for CD31 marks both Vas_EC and Endo_EC, we combined these two cell types to increase the number of targeted cells. This analysis consistently showed that the ablated samples had lower proportions. However, given that the quantifications have also produced inconsistent results for other cell types, such as Ven_CM, as mentioned in the reviewer’s next question, we have decided to delete this plot to avoid confusion.

      Author response image 1.

      (2) Similarly, while the percentage of Ven_CMs increased at E18.5, it exhibited differing trends at E16.5 (Figure 4E), further highlighting the inconsistency of the scRNA-seq analysis with the other data.

      We thank the reviewer for the comment. Please see the response above.

      (3) Furthermore, the authors noted that the ablated samples had slightly higher percentages of cardiomyocytes in the G1 phase compared to controls (Figures 4H, S11D), which aligns with the enrichment of pathways related to heart development, sarcomere organization, heart tube morphogenesis, and cell proliferation. However, it is unclear how this correlates with heart development, given that the hearts of ablated mice are significantly smaller than those of controls (Figure 3E). Additionally, the heart sections from ablated samples used for CD31/DAPI staining in Figure 3F appear much larger than those of the controls, raising further inconsistencies in the manuscript.

      We thank the reviewer for the comment. We observed changes in G1-phase cardiomyocytes at both E16.5 and E18.5, with pathway enrichment primarily identified in E16.5 cardiomyocytes. At E16.5, the ablated hearts exhibited myocardial defects, including an increased trabecular-to-compact myocardium ratio and reduced vascular density. By E18.5, the ablated embryos had smaller hearts with reduced vascular density, although the trabecular-to-compact myocardium ratio showed no obvious changes. Regarding the larger section size in the ablated hearts compared to the control hearts, there are two reasons contributing to this discrepancy. First, the control and ablated heart sections have different scale bars. The ablated hearts were enlarged compared to control section. Secondly, the heart sections vary in size depending on their position. Sections taken from the middle of the heart are larger than those from the edges. In our initial comparison, we used an edge-positioned section from the control hearts and a middle-positioned section from the ablated hearts. To avoid confusion, we have now updated the control section to match the position of the ablated embryos more closely and used the same size of scale bars in the two images (Fig 3F).

      (4) The manuscript relies heavily on the scRNA-seq dataset, which shows inconsistencies between the two replicates. Furthermore, the morphological and histological analyses do not align with the scRNA-seq findings.

      We respectfully disagree with this comment from the reviewer. As shown in Figure 4B, the scRNAseq data from the two replicates are highly consistent. For inconsistencies in cell proportions and tissue section sizes, please refer to our responses above.

      (5) There is a lack of mechanistic insight into how collagen, as a key signaling molecule from fibroblasts, affects the development of cardiomyocytes and vascular endothelial cells.

      We thank the reviewer for the comment. In this study, we primarily focused on analyzing fibroblast function in heart development using cell ablation and single-cell mRNA sequencing. While further mechanistic analysis of the collagen pathway is intriguing, it falls outside the scope of this study. Additionally, our scRNAseq analysis identified multiple collagen ligands derived from fibroblasts that may regulate gene expression in Ven_CM and influence their development, as shown in Figure 5C. Although validating these predictions would be valuable, it is beyond the scope of this study. We will continue this line of research and share our findings in the future.

      (6) In Figure 1B, Col1a1 expression is observed in the epicardial cells (Figure 1A, E11.5), but this is not represented in the accompanying cartoon.

      We thank the reviewer for the comment. As stated in the main text (page 3), based on scRNA-seq and IF staining results, we observed that Col1a1 is also expressed in epicardial cells. In the cartoon, we depicted the pattern of fibroblasts rather than Col1a1-positive cells, which is why we did not include epicardial cells.

      (7) What is the genotype of the control animals used in the study?

      We thank the reviewer for the comment. We have added the genotype information for the control embryos in the legends of the relevant figures.

      (8) Do the PdgfraCreER+/-; RosaDTA+/- mice survive after birth when induced at E15.5, and do they exhibit any cardiac defects?

      We thank the reviewer for the comment. This is an interesting question; however, we did not perform the experiment because administering tamoxifen to pregnant mice from E15.5 to E18.5 causes delivery complications, as reported in the literature (PMID: 23139287). Unfortunately, this prevents us from exploring this question further.

      Reviewer #3 (Recommendations for the authors):

      Overall, this is a comprehensive study substantiated by the evidence the authors provided in their findings. However, I have a few concerns to be addressed.

      (1) The claim by the authors that "at E17.5 and P3, each FB was in contact with approximately one Vas_EC and four CMs at both stages" is not fully convincing. RNA scope images for Actn2 are not clear enough to lead the quantification (RNA scope images for Cdh5 look better). I suggest performing imaging at higher magnification and the Z stack technique to provide a better understanding of their localization. Also, no changes in FBs adjacent cell numbers (CM&EC) with ages (P3) compared to E17.5? Any thoughts on the explanation?

      We thank the reviewer for the comment. We imaged the staining results using a confocal microscope at 20X resolution. We also considered imaging them at 40X; however, due to the large areas that need to be imaged in these sections, it was challenging to do so. Additionally, we identified each CM based on Actn2 and DAPI staining information and are confident in the accuracy of our quantification results. Moreover, since each FB interacts with multiple CMs and Vas_ECs in 3D projections, but our calculations are based only on 2D imaging sections, there may be discrepancies compared to a true 3D environment. We have added a sentence to address this limitation (page 9). Regarding the similar number of interactions observed at E17.5 and P3, we think there are two possibilities. First, the three cell types may proliferate in a synchronized manner, maintaining a consistent number of interactions. Second, these cell types may exhibit minimal proliferation during late embryonic and early neonatal stages. Instead, heart growth primarily occurs through CM hypertrophy, which does not significantly alter the number of interactions.

      (2) Fix the Capitalized font of RNA markers in Figure S2.

      Thanks. We have updated them.

      (3) I appreciate the visualization of ligand-receptor interactions in collagen network comparison between FB to CM and FB to EC, and predictive analysis on the FB ligands that regulate differentially expressed genes in ablated heart CM and ECs.

      We appreciate the reviewer for the comment.

      (4) The authors depleted Pdgfra-Cre cells at E10.5, and reported 100% DTA+ lethality after 3 days. Induction at E13.5 to ablate Pdgfra-Cre cells resulted in survival at least up to E16.5 age. What could be the possible reasons authors think that lead to embryo lethality when induced at E10.5? Did the authors analyze the expression of Pdgfra at E10.5 to E13.5 using Pdgfra antibody or Pdgfra-Cre labeling, or using the ScRNA seq data?

      We thank the reviewer for the comment. The expression pattern of Pdgfra at E10.5 has been previously reported (PMID: 18297729) and shown to be highly expressed in the atrioventricular region, consistent with the Col1a1 expression pattern we profiled in this study. Therefore, we believe the embryonic lethality observed in the ablated embryos at E10.5 was likely due to the disruption of the atrioventricular structure. However, since Pdgfra is also expressed in other tissues at this stage, we cannot rule out the possibility that the ablation of non-cardiac tissues also contributed to the lethality.

      (5) In terms of the findings on the trabeculation and compaction defects, please provide the images of the ventricles with markers to indicate the compact and trabecular zones and their defects.

      Thanks! We have included images that illustrate the quantification of compact and trabecular myocardium thickness in control and ablated hearts (FigS10C).

      (6) Did the author check the expression of any other marker for the vascular system in addition to CD31 to see the effects of ablated FB on coronary vasculature development?

      We thank the reviewer for the comment. We analyzed only Cd31 to assess the effects of fibroblast ablation on the overall endothelial cell population. We did not separately examine the subpopulations, but this would be an interesting direction for future studies.

      (7) Can the authors interpret how findings from PHH3 proliferation explain thinner compact and thicker trabeculae in ablated hearts?

      We thank the reviewer for the comment and apologize for the misinterpretation of the results. We observed that the ablated hearts have a thinner compact myocardium, while the thickness of the trabecular myocardium remains unchanged, leading to an increased trabecular-to-compact myocardium ratio (Fig 3D). We have corrected the description in the manuscript accordingly. Moreover, since the compact myocardium has a higher proliferation rate than the trabecular myocardium, a reduction in overall cell proliferation is expected to have a more pronounced impact on the compact myocardium. Inhibition of compact myocardium proliferation has been reported to lead thinner compact myocardium and non-compaction defects (PMID: 31342111).

      (8) The authors did not execute experiments to find the downstream target that causes compaction defects and endothelial cell density defects upon ablation of FBs. Can you project from your sequencing analysis what could be the potential downstream if you could execute bench-side experiments on this?

      We appreciate the reviewer for the comment. We believe that the regulatory predictive results in Figures 5C and D from the scRNA-seq data analysis have provided a set of downstream candidates for validation. We could select some of the ligands, such as the collagen ligands Col1a1, Col4a1, and Col5a1, to treat the ablated embryos in vivo to assess whether they could partially rescue the myocardium defects. Additionally, we could conduct ex vivo experiments by co-culturing CM and FB, comparing them with CM alone and CM treated with the identified ligands. This would allow us to evaluate CM proliferation and the expression of downstream genes identified in the prediction results. However, as the reviewer suggested, these experiments are planned for future studies.

      (9) Please provide the echocardiographic M mode images with a comparable number of cardiac cycles in control and ablated (Fig. 6H). Also, the heart rate of the ablated heart is too low to compare other parameters with the control. If you could stabilize the heart rate at comparable values to control the heart, it is possible that EF and FS values will be largely changed.

      We thank the reviewer for the comment. As the echocardiographic analysis was performed on conscious mice, the lower heart rates in the ablated mice are a phenotype associated with the ablation. Unfortunately, we are unable to adjust them to the same as the control mice.

      (10) Can you provide a numerical dataset for any one of the cell chat figures? Like in figure 2A, supporting the claim "However, in terms of interaction strength, FB exhibited the highest values compared to those of other cell types (Fig. 2A)".

      Yes, we have added a supplemental table (Table S2) containing the numerical interaction weights. As shown in the table, the interactions between FB and other cell types have the highest values.

    1. Author response:

      The following is the authors’ response to the original reviews.

      Reviewer #1 (Public review):

      Summary:

      Chen and Phillips describe the dynamic appearance of cytoplasmic granules during embryogenesis analogous to SIMR germ granules, and distinct from CSR-1-containing granules, in the C. elegans germline. They show that the nuclear Argonaute NRDE-3, when mutated to abrogate small RNA binding, or in specific genetic mutants, partially colocalizes to these granules along with other RNAi factors, such as SIMR-1, ENRI-2, RDE-3, and RRF-1. Furthermore, NRDE-3 RIP-seq analysis in early vs. late embryos is used to conclude that NRDE-3 binds CSR-1-dependent 22G RNAs in early embryos and ERGO-1dependent 22G RNAs in late embryos. These data lead to their model that NRDE-3 undergoes small RNA substrate "switching" that occurs in these embryonic SIMR granules and functions to silence two distinct sets of target transcripts - maternal, CSR-1 targeted mRNAs in early embryos and duplicated genes and repeat elements in late embryos.

      Strengths:

      The identification and function of small RNA-related granules during embryogenesis is a poorly understood area and this study will provide the impetus for future studies on the identification and potential functional compartmentalization of small RNA pathways and machinery during embryogenesis.

      Weaknesses:

      (1) While the authors acknowledge the following issue, their finding that loss of SIMR granules has no apparent impact on NRDE-3 small RNA loading puts the functional relevance of these structures into question. As they note in their Discussion, it is entirely possible that these embryonic granules may be "incidental condensates." It would be very welcomed if the authors could include some evidence that these SIMR granules have some function; for example, does the loss of these SIMR granules have an effect on CSR-1 targets in early embryos and ERGO-1-dependent targets in late embryos?

      We appreciate reviewer 1’s concern that we do not provide enough evidence for the function of the SIMR granules. As suggested, we examined the NRDE-3 bound small RNAs more deeply, and we do observe a slight but significant increased CSR-class 22G-RNAs binding to NRDE-3 in late embryos of simr-1 and enri-2 mutants (see below, right). We hypothesize that this result could be due to a slower switch from CSR to ERGO 22G-RNAs in the absence of SIMR granules. We added these data to Figure 6G.

      (2) The analysis of small RNA class "switching" requires some clarification. The authors re-define ERGO1-dependent targets in this study to arrive at a very limited set of genes and their justification for doing this is not convincing. What happens if the published set of ERGO-1 targets is used? 

      As we mentioned in the manuscript, we initially attempted to use the previously defined ERGO targets. However, the major concern is fewer than half the genes classified as ERGO targets by Manage et al. and Fischer et al. overlap with one another (Figure 6—figure supplement 1D and below). We reason this might because the gene sets were defined as genes that lose small RNAs in various ERGO pathway mutants and because different criteria were used to define the lists as discussed in the manuscript (lines 471-476). As a result, some of the previously defined ERGO target genes may actually be indirect targets of the pathway. Here we focus on genes targeted by small RNAs enriched in an ERGO pathway Argonaute IP, which should be more specific.

      In this manuscript, we are interested specifically in the ERGO targets bound by NRDE-3, thus we utilized the IP-small RNA sequencing data from young adult animals (Seroussi et al, 2023), to define a new ERGO list. We are confident about this list because 1) Most of our new ERGO genes overlap with the overlap between ERGO-Manage and ERGO-Fischer list (see Figure 6—figure supplement 1D in our manuscript and below). 2) We observed the most significant decrease of small RNA levels and increase of mRNA levels in the nrde-3 mutants using our newly defined list (see Figure 6—figure supplement 1E-F in our manuscript).

      To further address reviewer 1’s concern about whether the data would look significantly different when using the ERGO-Manage and ERGO-Fischer lists, we made new scatter plots shown in Author response image 1 panels A-C below (ERGO-Manage – purple, ERGO-Fischer- yellow, and the overlap - yellow with purple ring). We found that the small switching pattern of NRDE-3 is consistent with our newly defined list, particularly if we look at the overlap of ERGO-Manage and ERGO-Fischer list (Author response image 1 panels D-F below, red).

      Author response image 1.

      Further, the NRDE-3 RIP-seq data is used to conclude that NRDE-3 predominantly binds CSR-1 class 22G RNAs in early embryos, while ERGO-1-dependent 22G RNAs are enriched in late embryos. a) The relative ratios of each class of small RNAs are given in terms of unique targets. What is the total abundance of sequenced reads of each class in the NRDE-3 IPs? 

      To address the reviewer’s question about the total abundance of sequenced reads of each class in the NRDE-3 IPs: Author response image 2 panel A-B below show the total RPM of CSR and ERGO class sRNAs in inputs and IPs at different stages. Focusing on late embryos, the total abundance of ERGO-dependent sRNAs is similar to CSR-class sRNAs in input, while much higher in IP, indicating an enrichment of ERGO-dependent 22G-RNAs in NRDE-3 consistent with our log2FC (IP vs input) in Figure 6B. This data supports our conclusion that NRDE-3 preferentially binds to ERGO targets in late embryos.

      Author response image 2.

      b) The "switching" model is problematic given that even in late embryos, the majority of 22G RNAs bound by NRDE-3 is the CSR-1 class (Figure 5D). 

      It is important to keep in mind the difference in the total number of CSR target genes (3834) and ERGO target genes (119).  The pie charts shown in Figure 6D are looking at the total proportion of the genes enriched in the NRDE-3 IP that are CSR or ERGO targets. For the NRDE-3 IP in late embryos, that would be 70/119 (58.8%) of ERGO targets are enriched, while 172/3834 (4.5%) of CSR targets are enriched. These data are also supported by the RPM graphs shown in Author response image 2 panels A-B above, which show that the majority of the small RNA bound by NRDE-3 in late embryos are ERGO targets. Nonetheless, NRDE-3 still binds to some CSR targets shown as Figure 6D and panel B, which may be because the amount of CSR-class 22G-RNAs is reduced gradually across embryonic development as the maternally-deposited NRDE-3 loaded with CSR-class 22G-RNAs is diluted by newly transcribed NRDE-3 loaded with ERGOdependent 22G-RNAs (lines 857-862). 

      c) A major difference between NRDE-3 small RNA binding in eri-1 and simr-1 mutants appears to be that NRDE-3 robustly binds CSR-1 22G RNAs in eri-1 but not in simr-1 in late embryos. This result should be better discussed.

      In the eri-1 mutant, we hypothesize that NRDE-3 robustly binds CSR-class 22G-RNAs because ERGOclass 22G-RNAs are not synthesized during mid-embryogenesis, so either NRDE-3 is unloaded (in granule at 100-cell stage in Figure 2A) or mis-loaded with CSR-class 22G-RNAs (in the nucleus at 100cell stage in Figure 2A). We don’t have a robust method to address the proportion of loaded vs. unloaded NRDE-3 so it is difficult to address the degree to which NRDE-3 is misloaded in the eri-1 mutant. In the simr-1 mutant, both classes of small RNAs are present and NRDE-3 is still preferentially loaded with ERGO-dependent 22G-RNAs, though we do see a subtle increase in association with CSR-class 22GRNAs. These data could suggest a less efficient loading of NRDE-3 with ERGO-dependent 22G-RNAs, but we would need more precise methods to address the loading dynamics in the simr-1 mutant.

      (3) Ultimately, if the switching is functionally important, then its impact should be observed in the expression of their targets. RNA-seq or RT-qPCR of select CSR-1 and ERGO-1 targets should be assessed in nrde-3 mutants during early vs late embryogenesis.

      The function of NRDE-3 at ERGO targets has been well studied (Guang et al, 2008) and is also assessed in our H3K9me3 ChIP-seq analysis in Figure 7E where, in mixed staged embryos, H3K9me3 level on ERGO targets (labeled as ‘NRDE-3 targets in young adults’) is reduced significantly in the nrde-3 mutant.

      To understand the function of NRDE-3 binding on CSR targets in early embryos, we attempted to do RTqPCR, smFISH, and anti-H3K9me3 CUT&Tag-seq on early embryos, and we either failed to obtain enough signal or failed to detect any significant difference (data not shown). We additionally tested the possibility that NRDE-3 functions with CSR-class 22G-RNAs in oocytes. We present new data showing that NRDE-3 represses RNA Pol II in oocytes to promote global transcriptional repression at the oocyteto-embryo transition, we now included these data in Figure 8. 

      Reviewer #2 (Public review):

      Summary:

      NRDE-3 is a nuclear WAGO-clade Argonaute that, in somatic cells, binds small RNAs amplified in response to the ERGO-class 26G RNAs that target repetitive sequences. This manuscript reports that, in the germline and early embryos, NRDE-3 interacts with a different set of small RNAs that target mRNAs. This class of small RNAs was previously shown to bind to a different WAGO-clade Argonaute called CSR1, which is cytoplasmic, unlike nuclear NRDE-3. The switch in NRDE-3 specificity parallels recent findings in Ascaris where the Ascaris NRDE homolog was shown to switch from sRNAs that target repetitive sequences to CSR-class sRNAs that target mRNAs.

      The manuscript also correlates the change in NRDE-3 specificity with the appearance in embryos of cytoplasmic condensates that accumulate SIMR-1, a scaffolding protein that the authors previously implicated in sRNA loading for a different nuclear Argonaute HRDE-1. By analogy, and through a set of corelative evidence, the authors argue that SIMR foci arise in embryogenesis to facilitate the change in NRDE-3 small RNA repertoire. The paper presents lots of data that beautifully documents the appearance and composition of the embryonic SIMR-1 foci, including evidence that a mutated NRDE-3 that cannot bind sRNAs accumulates in SIMR-1 foci in a SIMR-1-dependent fashion.

      Weaknesses:

      The genetic evidence, however, does not support a requirement for SIMR-1 foci: the authors detected no defect in NRDE-3 sRNA loading in simr-1 mutants. Although the authors acknowledge this negative result in the discussion, they still argue for a model (Figure 7) that is not supported by genetic data. My main suggestion is that the authors give equal consideration to other models - see below for specifics.

      We appreciate reviewer 2’s comments on the genetic evidence for the function of SIMR foci.  A similar concern was also brought up by reviewer 1. By re-examining our sequencing data, we found that there is a modest but significant increase in NRDE-3 association with CSR-class sRNAs in simr-1 and enri-2 mutants in late embryos. We believe that this data supports our model that SIMR-1 and ENRI-2 are required for an efficient switch of NRDE-3 bound small RNAs. Please refer our response to the reviewer 1 - point (1), and Figure 6G in the updated manuscript. 

      Reviewer #3 (Public review):

      Summary:

      Chen and Phillips present intriguing work that extends our view on the C. elegans small RNA network significantly. While the precise findings are rather C. elegans specific there are also messages for the broader field, most notably the switching of small RNA populations bound to an argonaute, and RNA granules behavior depending on developmental stage. The work also starts to shed more light on the still poorly understood role of the CSR-1 argonaute protein and supports its role in the decay of maternal transcripts. Overall, the work is of excellent quality, and the messages have a significant impact.

      Strengths:

      Compelling evidence for major shift in activities of an argonaute protein during development, and implications for how small RNAs affect early development. Very balanced and thoughtful discussion.

      Weaknesses:

      Claims on col-localization of specific 'granules' are not well supported by quantitative data

      We have now included zoomed images of individual granules to better show the colocalization in Figure 4 and Figure 4—figure supplement 1, and performed Pearson’s colocalization analysis between different sets of proteins in Figure 4B. 

      Reviewer #2 (Recommendations for the authors):

      - The manuscript is very dense and the gene names are not helpful. For example, the authors mention ERGO-1 without clarifying the type of protein, etc. I suggest the authors include a figure to go with the introduction that describes the different classes of primary and secondary sRNAs, associated Argonautes, and other accessory proteins. Also include a table listing relevant gene names, protein classes, main localizations, and proposed functions for easy reference by the readers.

      We agree that the genes names in different small RNA pathways are easily confused. We added a diagram and table in Figure 1—figure supplement 1 depicting the ERGO/NRDE and CSR pathways and added clarification about the ERGO/NRDE-3 pathway in the text from line 126-128.  

      - Line 424 - the wording here and elsewhere seems to imply that SIMR-1 and ENRI-2, although not essential, contribute to NRDE-3 sRNA loading. The sequencing data, however, do not support this - the authors should be clearer on this. If the authors believe there are subtle but significant differences, they should show them perhaps by adding a panel in Figure 5 that directly compares the NRDE-3 IPs in wildtype versus simr-1 mutants. Figure 5H however does not support such a requirement.

      As brought up by reviewer 1, we do not see difference in binding of ERGO-dependent sRNA in simr-1 mutant in late embryos. We do, however, see a modest, but significant, increase of CSR-sRNAs bound by NRDE-3 in simr-1 and enri-2 mutants, which we hypothesize could be due to a less efficient loading of ERGO-dependent 22G-RNAs by NRDE-3. The updated data are now in Figure 6G. We have also edited the text and model figure to soften these conclusions.

      - Condensates of PGL proteins appear at a similar time and place (somatic cells of early embryos) as the embryonic SIMR-1 foci. The PGL foci correspond to autophagy bodies that degrade PGL proteins. Is it possible that SIMR-1 foci also correspond to degradative structures? The possibility that SIMR-1 foci are targeted for autophagy and not functional would fit with the finding that simr-1 mutants do not affect NRDE-3 loading in embryos.

      We appreciate reviewer 2’s comments on possibility of SIMR granules acting as sites for degradation of SIMR-1 and NRDE-3. We think this is not the case for the following reasons: 1) if SIMR granules are sites of autophagic degradation, then we would expect that embryonic SIMR granules in somatic cells, like PGL granules, should only be observed in autophagy mutants; however we see them in wild-type embryos 2) we would not expect a functional Tudor domain to be required for granule localization; however in Figure 1—figure supplement 2B, we show that a point mutation in the Tudor domain of SIMR-1 abrogates SIMR granule formation, and 3) if NRDE-3(HK-AA) is recruited to SIMR granules for degradation while wild-type NRDE-3 is cytoplasmic, then NRDE-3(HK-AA) should shows a significantly reduced protein level comparing to wild-type NRDE-3. In the western blot in Figure 2—figure supplement 1B, NRDE-3 and NRDE-3(HK-AA) protein levels are similar, indicating that NRDE-3(HK-AA) is not degraded despite being unloaded. This is in contrast to what we have observed previously for HRDE-1, which is degraded in its unloaded state. If SIMR-1 played a role directly in promoting degradation of NRDE-3(HK-AA), we would similarly expect to see a change in NRDE-3 or NRDE-3(HK-AA) expression in a simr-1 mutant. We performed western blot and did not observe a significant change in protein expression for NRDE-3 (Figure 3—figure supplement 1A). 

      Although under wild-type conditions, SIMR granules do not appear to be sites of autophagic degradation, upon treatment with lgg-1 (an autophagy protein) RNAi, we found that SIMR-1, as well as many other germ granule and embryonic granule-localized proteins, increase in abundance in late embryos.  This data demonstrates that ZNFX-1, CSR-1, SIMR-1, MUT-2/RDE-3, RRF-1, and unloaded NRDE-3 are removed by autophagic degradation similar to what have been shown previously for PGL-1 proteins (Zhang et al, 2009, Cell). We added these data to Figure 5. It is important to emphasize, however, that the timing of degradation differs for each granule assayed (Lines 447-450), indicating that there must be multiple waves of autophagy to selectively degrade subsets of proteins when they are no longer needed by the embryo.

      - The observation that an NRDE-3 mutant that cannot load sRNAs localizes to SIMR-1 foci does not necessarily imply that wild-type unloaded NRDE-3 would also localize there. Unless the authors have additional data to support this idea, the authors should acknowledge that this hypothesis is speculative. In fact, why does cytoplasmic NRDE-3 not localize to granules in the rde-3;ego-1degron strain shown in Figure 6B?? Is it possible that the NRDE-3 mutant accumulates in SIMR-1 foci because it is unfolded and needs to be degraded?

      We believe that wild-type NRDE-3 also localize to SIMR foci when unloaded. This is supported by the localization of wild-type NRDE-3 in eri-1 and rde-3 mutants, where a subset of small RNAs are depleted. Wild-type NRDE-3 localizes to both somatic SIMR-1 granules and the nucleus, depending on embryo stage (Figure 2A, Figure 2—figure supplement 1C). The granule numbers in eri-1 and rde-3 mutants are less than the nrde-3(HK-AA) mutant, consistent with the imaging data that NRDE-3 only partially localize to somatic granule (Figure 2A – 100-cell stage).

      In the rde-3; ego-1 double mutant, the embryos have severe developmental defect: they cannot divide properly after 4-8 cell stage and exhibit morphology defects after that stage. In wild-type, SIMR foci does not appear until around 8-28-cell stage (shown in Figure 1C), so we believe that cytoplasmic NRDE-3 does not localize to foci in the double mutant is because of the timing.

      - The authors propose that NRDE-3 functions in nuclei to target mRNAs also targeted in the cytoplasm by CSR-1. If so, how do they propose that NRDE-3 might do this since little transcription occurs in oocytes/early embryos?? Are the authors suggesting that NRDE-3 targets germline genes for silencing specifically at the times that zygotic transcription comes back on, or already in maturing oocytes? Is the transcription of most CSR-1 targets silenced in early embryos??

      We appreciate the suggestions to check the function of NRDE-3 in oocytes. We tested this possibility and found it to be correct. NRDE-3 functions in oocytes for transcriptional repression by inhibiting RNA Pol II elongation. We added these data to Figure 8. We also attempted to do RT-qPCR, smFISH, and antiH3K9me3 Cut&Tag-seq on early embryos to further test the hypothesis that NRDE-3 acts with CSR-class 22G-RNAs in early embryos, but we either failed to obtain enough signal or failed to detect any significant difference (data not shown). Therefore, we think that the primary role for NRDE-3 bound to CSR-class 22G-RNAs may be for global transcriptional repression of oocytes prior to fertilization.

      - Line 684-686: "In summary, this work investigating the role of SIMR granules in embryos, together with our previous study of SIMR foci in the germline (Chen and Phillips 2024), has identified a new mechanism for small RNA loading of nuclear Argonaute proteins in C. elegans". This statement appears overstated/incorrect since there is no evidence that SIMR-1 foci are required for sRNA loading of NRDE3. The authors should emphasize other models, as suggested above.

      We have revised the text on line 869-871 to emphasize that SIMR granule regulate the localization of nuclear Argonaute proteins, rather than suggesting a direct role on controlling small RNA loading. We also edit the title, text, and legend for our model in Figure 9. 

      Reviewer #3 (Recommendations for the authors):

      Issues to be addressed:

      - The authors show a switch in 22G RNA binding by NRDE-3 during embryogenesis. While the data is convincing, it would be great if it could be tested if the preferred NRDE-3 replacement model is indeed correct. This could be done relatively easily by giving NRDE-3 a Dendra tag, allowing one to colour-switch the maternal WAGO-3 pool before the zygotic pool comes up. Such data would significantly enhance the manuscript, as this would allow the authors to follow the fate of maternal NRDE-3 more precisely, perhaps identifying a period of sharp decline of maternal NRDE-3.

      We think the NRDE-3 Dendra tag experiment suggested by the reviewer is a clever approach and we will consider generating this strain in the future. However, we feel that optimization of the color-switching tag between the maternal germline and the developing embryos is beyond the scope of this manuscript. To partially address the question about NRDE-3 fate during embryogenesis, we examined the single-cell sequencing data of C. elegans embryos from 1-cell to 16-cell stage (Tintori et al, 2016, Dev Cell; Visualization tool from John I Murray lab), as shown in Author response image 3 Panel A below, NRDE-3 transcript level increases as embryo develops, indicating that zygotic NRDE-3 is being actively expressed starting very early in development. We hypothesize that maternal NRDE-3 will either be diluted as the embryo develops or actively degraded during early embryogenesis. 

      Author response image 3.

      - Figure 3A: * should mark PGCs, but this seems incorrect. At the 8-cell stage there still is only one PGC (P4), not two, and at 100 cells there are only two, not three germ cells. Also, the identification of PGCs with a maker (PGL for instance) would be much more convincing.

      We apologize for the confusion in Figure 3A. We changed the figure legend to clarify that the * indicate nuclear NRDE-3 localization in somatic cells for 8- and 100-cell stage embryos rather than the germ cells.  

      - Overall, the authors should address colocalization more robustly. In the current manuscript, just one image is provided, and often rather zoomed-out. How robust are the claims on colocalization, or lack thereof? With the current data, this cannot be assessed. Pearson correlation, combined with line-scans through a multitude of granules in different embryos will be required to make strong claims on colocalization. This applies to all figures (main and supplement) where claims on different granules are derived from.

      We thank reviewer 3 for this important suggestion. To better address the colocalization, we included insets of individual granules in Figure 2D and Figure 4. We also performed colocalization analysis by calculating the Pearson’s R value between different groups of proteins in Figure 4B, to highlight that SIMR-1 colocalizes with ENRI-2, NRDE-3(HK-AA), RDE-3, and RRF-1, while CSR-1 colocalizes with EGO-1.

      For the proteins that lack colocalization in Figure 4—figure supplement 1, we also added insets of individual granules. Additionally, we included a new set of panels showing SIMR-1 localization compared to tubulin::GFP (Figure 4—figure supplement 1I) in response to a recent preprint (Jin et al, 2024, BioRxiv), which finds NRDE-3 (expressed under a mex-5 promoter) associating with pericentrosomal foci and the spindle in early embryos. We do not see SIMR-1 (or NRDE-3, data not shown) at centrosomes or spindles in wild-type conditions but made a similar observation for SIMR-1 in a mut-16 mutant (Figure 4E). All of the localization patterns were examined on at least 5 individual 100-cell staged embryos with same localization pattern.

      - Figure 7: Its title is: Function of cytoplasmic granules. This is a much stronger statement than provided in the nicely balanced discussion. The role of the granules remains unclear, and they may well be just a reflection of activity, not a driver. While this is nicely discussed in the text, figure 7 misses this nuance. For instance, the title suggests function, and also the legend uses phrases like 'recruited to granule X'. If granules are the results of activity, 'recruitment' is really not the right way to express the findings. The nuance that is so nicely worded in the discussion should come out fully in this figure and its legend as well.

      We have changed the title of Figure 7 (now Figure 9) to “Model for temporally- and developmentallyregulated NRDE-3 function” to deemphasize the role of the granules and to highlight the different functions of NRDE-3. Similarly, we have rephrased the text in the figure and legend and add a some details about our new results.

      Minor:

      Typo: line 663 Acaris

      We corrected the typo.

    1. Author response:

      The following is the authors’ response to the original reviews

      Public Reviews:

      Reviewer #1 (Public review):

      Summary:

      In this detailed study, Cohen and Ben-Shaul characterized the AOB cell responses to various conspecific urine samples in female mice across the estrous cycle. The authors found that AOB cell responses vary with the strains and sexes of the samples. Between estrous and non-estrous females, no clear or consistent difference in responses was found. The cell response patterns, as measured by the distance between pairs of stimuli, are largely stable. When some changes do occur, they are not consistent across strains or male status. The authors concluded that AOB detects the signals without interpreting them. Overall, this study will provide useful information for scientists in the field of olfaction.

      Strengths:

      The study uses electrophysiological recording to characterize the responses of AOB cells to various urines in female mice. AOB recording is not trivial as it requires activation of VNO pump. The team uses a unique preparation to activate the VNO pump with electric stimulation, allowing them to record AOB cell responses to urines in anesthetized animals. The study comprehensively described the AOB cell responses to social stimuli and how the responses vary (or not) with features of the urine source and the reproductive state of the recording females. The dataset could be a valuable resource for scientists in the field of olfaction.

      Weaknesses:

      (1) The figures could be better labeled.

      We revised all figures (except the model figure, Fig. 8), and among other improvements (many of which were suggested by the reviewers in other comments), added more labelling and annotation within the figures.

      (2) For Figure 2E, please plot the error bar. Are there any statistics performed to compare the mean responses?

      We added error bars (standard errors of the mean). We had not originally performed statistical comparisons between the stimuli, but now we have. The analysis of responses strength now appears in a new table (Table 1)

      (3) For Figure 2D, it will be more informative to plot the percentage of responsive units.

      Done.

      (4) Could the similarity in response be explained by the similarity in urine composition? The study will be significantly strengthened by understanding the "distance" of chemical composition in different urine.

      We agree. As we wrote in the Discussion: “Ultimately, lacking knowledge of the chemical space associated with each of the stimuli, this and all the other ideas developed here remain speculative.” We note however, that chemical distance (which in itself is hard to define) will provide only part of the picture. The other part is the “projection” of chemical space on the receptor array. This is an idea that we develop in the Discussion and in Figure 8. Specifically, that it is the combination of stimulus composition, and receptor tuning properties that will determine stimulus distances in neuronal space.

      That said, a better understanding of the chemical distance is an important aspect that we are working to include in our future studies. For this dataset unfortunately, we have no such data.

      (5) If it is not possible for the authors to obtain these data first-hand, published data on MUPs and chemicals found in these urines may provide some clues.

      This comment is directly related to the previous one. Measurements about some classes of molecules may be found for some of the stimuli that we used here, but not for all. We are not aware of any single dataset that contains this information for any type of molecule across the entire stimulus set that we have used and pooling results from different studies has limited validity because of the biological and technical variability across studies. In order to reliably interpret our current recordings, it would be necessary to measure the urinary content of the very same samples that were used for stimulation. Unfortunately, we are not able to conduct this analysis at this stage.

      (6) It is not very clear to me whether the female overrepresentation is because there are truly more AOB cells that respond to females than males or because there are only two female samples but 9 male samples.

      The definitive answer to this comment is given in our response to the next one.

      Nevertheless, we agree that this is an important point. It is true that the number of neurons fulfilling each of the patterns depends on the number of individual stimuli that define it (and on the frequency of neurons that respond to those stimuli). However, our measure of “over representation” was designed to overcome this bias, by using bootstrapping to reveal if the observed number of patterns is larger than expected by chance.  The higher frequency of responses to female, as compared to male stimuli, is observed in other studies by others and by us, also when the number of male and female stimuli is matched (e.g., Bansal et al BMC Biol 2021, Ben-Shaul et al, PNAS 2010, Hendrickson et al, JNS, 2008). However, here, by overrepresentation, we do not refer to the higher frequency of female responding neurons, but rather that given the number of responding neurons, the female pattern is more common than expected by chance.

      (7) If the authors only select two male samples, let's say ICR Naïve and ICR DOM, combine them with responses to two female samples, and do the same analysis as in Figure 3, will the female response still be overrepresented?

      Following this suggestion, we have performed this analysis, and we were glad to see that the result is the one we had anticipated. Below, we provide an image of the results, following the same approach that we applied before, and showed in Figure 3C. Here, we defined a female pattern (using the two female samples) and compared it to a male pattern (using the ICR naïve and ICR DOM as suggested). It is as if we had only four stimuli in our set. As in the article, we calculated the expected distribution with 100,000 shuffles. We denoted this pattern as F/M ICR. The results are shown below.

      Under the present conditions, the distribution of the number of female selective patterns is larger (i.e., shifted to the right, compare to the female category in Figure 3C. This is expected, since now the criterion is more permissive. Specifically, now to qualify as a “female pattern”, the two responses to female urine must be stronger only than the responses to the two male stimuli included in this analysis (and to all other responses). Notably, although the null distribution shifted to the right, the actual number of neurons fulfilling this pattern is also larger, so that the actual number remains significantly larger than expected by chance. This is also true for the reverse category (as is the case in the ~female category Figure 3C).  Thus, we conclude that overrepresentation of the female pattern is not a trivial consequence of the number of male and female stimuli.

      Author response image 1.

      (8) In Figure 4B and 4C, the pairwise distance during non-estrus is generally higher than that during estrus, although they are highly correlated. Does it mean that the cells respond to different urines more distinctively during diestrus than in estrus?

      This is an important observation (!) and we had originally overlooked it.  It is true that higher distance (as they are in estrus) imply more distinct population level responses and hence better discrimination among stimuli. However, this is inconsistent with all our other analyses that do not point to enhanced selectivity or discrimination in either state. If anything, we find somewhat higher sparseness in estrus.  Yet, there may be technical explanations for the differences.

      For Euclidean distances, the explanation may be trivial. The distance depends on the number of dimensions (i.e., units), and since our sample contains more neurons recorded during non-estrus, the larger distance is expected.

      In fact, there is a similar dependence on sample size for the correlation distance. Smaller samples are associated with higher (spurious) correlations, and hence larger samples are be associated with larger distances. To demonstrate this, we conducted a simple simulation, where we calculated the absolute correlation coefficients of random samples from standard normal distributions (using the MATLAB function randn), changing the size of the population. For each sample size, we conducted 1000 tests. We considered sample sizes from 10 to 100000, including 200 and 300 (which are similar to our sample sizes). The results are shown in the figure below. Note that the absolute value of the correlation coefficient decreases with sample size, while the p-value for the observed correlation is stable at ~0.5.

      While this is not a rigorous analysis of this issue, and while it does not exactly reflect the scenario in our data, where correlations are generally positive, it shows that the observed correlation (and hence correlation distance) is also affected by sample size.

      For these reasons, we focus on comparison of these distances, rather than the absolute values of the correlation distances.

      Author response image 2.

      Following this comment, we now write in the manuscript:

      “We first note that distances are generally larger during non-estrus, suggesting enhanced discrimination during this stage. However, further analyses of sparseness and selectivity do not support this idea (see below). Furthermore, we note that both Euclidean and correlation distances generally depend on sample size. In both cases, distances are expected to increase as a function of sample size, which in our dataset, is larger for the non-estrus (n = 305) as compared to the estrus (n = 241) neurons. Because of this factor, we focus here on the similarity of the relative within-state distances across the states (and not on their absolute magnitudes). Specifically, we find a positive and significant correlation among pairwise population distances under the two states. Thus, at the population level, representational space remains broadly stable across the estrus cycle. Nevertheless, several points in Fig. 4D, E clearly diverge from a linear relationship, implying that representational space differs under the two states. We next examine such state-dependent changes in more detail.”

      (9) The correlation analysis is not entirely intuitive when just looking at the figures. Some sample heatmaps showing the response differences between estrous states will be helpful.

      If we understand correctly, the idea is to show the correlation matrices from which the values in 4B and 4C are taken. The relevant images are now included in figure 4B, C and are references within the main text.

      Reviewer #2 (Public review):

      Summary:

      Many aspects of the study are carefully done, and in the grand scheme this is a solid contribution. I have no "big-picture" concerns about the approach or methodology. However, in numerous places the manuscript is unnecessarily vague, ambiguous, or confusing. Tightening up the presentation will magnify their impact.

      We have reviewed the text and made substantial editing changes. Along with other specific comments by made both reviewers, we hope that these changes improve the presentation.

      Strengths:

      (1) The study includes urine donors from males of three strains each with three social states, as well as females in two states. This diversity significantly enhances their ability to interpret their results.

      (2) Several distinct analyses are used to explore the question of whether AOB MCs are biased towards specific states or different between estrus and non-estrus females. The results of these different analyses are self-reinforcing about the main conclusions of the study.

      (3) The presentation maintains a neutral perspective throughout while touching on topics of widespread interest.

      Weaknesses:

      (1) Introduction:

      The discussion of the role of the VNS and preferences for different male stimuli should perhaps include Wysocki and Lepri 1991

      We assume that the reviewer is referring to “Consequences of removing the vomeronasal organ” by Wysocki CJ, Lepri JJ, a review article in J Steroid Biochem from 1991. We were not familiar with this specific article and have now read it. The article discusses various male behaviors, and some effects on female behavior and physiology (e.g., puberty acceleration, maternal behaviors, ovulation) but we could not find any mention of the preference of female mice in this article. We also expanded our search to all pubmed articles authored by Wysocki and Lepri and then all articles by Wysocki (with the keyword Vomeronasal). Despite our best intentions to give due credit, we found nothing that seems directly related to this statement. Please correct us if we had missed anything.

      (2) Results:

      a) Given the 20s gap between them, the distinction between sample application and sympathetic nerve trunk stimulation needs to be made crystal clear; in many places, "stimulus application" is used in places where this reviewer suspects they actually mean sympathetic nerve trunk stimulation.

      We realize that this is confusing, and we also agree that at least in one place, we have not been sufficiently clear about the distinction. To clarify, we distinguish between stimulus application (physical application of stimulus to the nostril), and stimulation (which refers to SNT stimulation, which typically induces VNO suction). The general term stimulus presentation refers to the entire process. As explained in the text, in our analysis, we consider the entire window starting at application and ending 40s after stimulation. This is because we sometimes observe immediate responses following application. One such responses is seen in Figure 2D, and this is directly related to a detailed comment made below (on Figure 1D, part c). Indeed, for this figure time 0 indicates stimulus application. This was indicated previously, but we have now rearranged order of the panels to make the distinction between this response and other clearer. We have also revised the figure caption and the text to clarify this issue.

      b) There appears to be a mismatch between the discussion of Figure 3 and its contents. Specifically, there is an example of an "adjusted" pattern in 3A, not 3B.

      True. we have revised the text to correctly refer to the figure. Thanks.

      c) The discussion of patterns neglects to mention whether it's possible for a neuron to belong to more than one pattern. For example, it would seem possible for a neuron to simultaneously fit the "ICR pattern" and the "dominant adjusted pattern" if, e.g., all ICR responses are stronger than all others, but if simultaneously within each strain the dominant male causes the largest response.

      This is true. In the legend to Figure 3B, we actually wrote: “A neuron may fulfill more than one pattern and thus may appear in more than one row.”, but we now also write in the main text:

      “We note that criteria for adjusted patterns are less stringent than for the standard patterns defined above. Furthermore, some patterns are not mutually exclusive, and thus, a neuron may fulfil more than a single pattern.”

      (3) Discussion:

      a) The discussion of chemical specificity in urine focuses on volatiles and MUPs (citation #47), but many important molecules for the VNS are small, nonvolatile ligands. For such molecules, the corresponding study is Fu et al 2015.

      Agreed. We now cite this work and several others that were not included before in the context of chemical and electrophysiological analyses.

      b) "Following our line of reasoning, this scarcity may represent an optimal allocation of resources to separate dominant from naïve males": 1 unit out of 215 is roughly consistent with a single receptor. Surely little would be lost if there could be more computational capacity devoted to this important axis than that? It seems more likely that dominance is computed from multiple neuronal types with mixed encoding.

      We fully agree, and we are not claiming that dominance, nor any other feature, is derived using dedicated feature selective neurons. Our discussion of resource allocation is inevitably speculative. Our main point in this context is that a lack of overrepresentation does not imply that a feature is not important. As a note, we do not think that there is good reason to suppose that AOB neurons reflect the activity of single receptors.

      To present this potential confusion, we now added the following sentences in the Discussion subsection titled “Response patterns of AOB-MCs”:

      “We stress that we do not suggest that features such as physiological state are encoded by the activity of single neurons. In fact, we believe that most ethologically relevant features are encoded by the activity of multiple neurons. Nevertheless, such population level representations ultimately depend on the response properties of individual neurons, and we thus ask: what can we learn from our analysis of response pattern frequency?”

      (4) Methods:

      a) Male status, "were unambiguous in most cases": is it possible to put numerical estimates on this? 55% and 99% are both "most," yet they differ substantially in interpretive uncertainty.

      Upon reexamination, we realized that this sentence is incorrect. Ambiguous cases were not considered as dominant for urine collection. We only classified mice as dominant if they “won” the tube test and exhibited dominant behavior in the subsequent observation period in the cage. The phrasing has now been corrected in the manuscript (Methods section).

      b) Surgical procedures and electrode positioning: important details of probes are missing (electrode recording area, spacing, etc).

      This information has been added to the Methods subsection “Surgical procedures and electrode positioning”

      c) Stimulus presentation procedure: Are stimuli manually pipetted or delivered by apparatus with precise timing?

      They are delivered manually. This has now been clarified in the text.

      d) Data analysis, "we applied more permissive criteria involving response magnitude": it's not clear whether this is what's spelled out in the next paragraph, or whether that's left unspecified. In either case, the next paragraph appears to be about establishing a noise floor on pattern membership, not a "permissive criterion."

      True, the next paragraph is not the explanation for the more permissive criteria. The more permissive criteria involving response magnitude are actually those described in Figure 3A and 3B. The sentence that was quoted above merely states that before applying those criteria, we had also searched for patterns defined by binary designation of neurons as responsive, or not responsive, to each of the stimuli (this is directly related to the next comment below). Using those binary definitions, we obtained a very small number of neurons for each pattern and thus decided to apply the approach actually used and described in the manuscript.

      To clarify this confusion, we thoroughly derived the description of this paragraph, and the beginning of the next one in the Methods section.

      e) Data analysis, method for assessing significance: there's a lot to like about the use of pooling to estimate the baseline and the use of an ANOVA-like test to assess unit responsiveness.

      But:

      i) for a specific stimulus, at 4 trials (the minimum specified in "Stimulus presentation procedure") kruskalwallis is questionable. They state that most trials use 5, however, and that should be okay.

      The exact values are now given in the text. The mean number of repeated presentations per stimulus: 5.1± 0.9, mean ± sd. In 72% of the cases, stimuli were given 5 or more times. Otherwise, they were presented 4 times. In the context of the statistical test, we note that we are not comparing 5 (or 4) values with another set of 5 (or 4 values), but with a much larger sample (~44-55 baseline trials – given 11 trials and 4-5 repeats of each). Under this scenario, we think that the statistical approach is sound. However, the more important consideration, in our opinion, is given below.

      ii) the methods statement suggests they are running kruskalwallis individually for each neuron/stimulus, rather than once per neuron across all stimuli. With 11 stimuli, there is a substantial chance of a false-positive if they used p < 0.05 to assess significance. (The actual threshold was unstated.) Were there any multiple comparison corrections performed? Or did they run kruskalwallis on the neuron, and then if significant assess individual stimuli? (Which is a form of multiple-comparisons correction.)

      First, we indeed failed to mention that our criterion was 0.05. This has been corrected, by adding the information to the results and the Methods sections. No, we did not apply any multiple comparison measures. We consider each neuron-stimulus pair as an independent entity, and we are aware that this leads to a higher false positive rate. On the other hand, applying multiple comparisons would be problematic, as the same number of stimuli used in different studies varies. Application of multiple comparison corrections would thus lead to different response criteria across different studies, which would be very problematic. This raises the almost philosophical question regarding the use of multiple comparisons (as well as one and two tailed tests), but practically, most, if not all of our conclusions involve comparisons across conditions. For this purpose, we think that our procedure is valid. More generally, while selection of responses according to significance has some obvious advantages, the decision to use any particular criterion is entirely arbitrary. Therefore, we do not attach any special meaning to the significance threshold used here. Rather, we think of it as a simple criterion that allows us to exclude weakly responding or non-responsive neurons, and to compare frequencies of neurons that fulfill this criterion, under different conditions and contexts.

      Recommendations for the authors:

      Reviewer #2 (Recommendations for the authors):

      Results:

      "are represented more than represented by chance" seems to have a misplaced word

      True. Thanks. Corrected.

      Figure 1D:

      a) Indicate the meaning of the number that appears in the top left for each unit (10, 5, 40, 5, 5) (I'm guessing it's the vertical scale for the PSTH, but best to spell it out explicitly.)

      This information has been added.

      b) "The red vertical line indicates stimulus application": is it the application of the chemical stimulus or SNT shock?

      Please see our answer to c

      c) "For unit 2, time 0 indicate stimulus application, as in this case, responses began after stimulus application, prior to stimulation." First, the meaning of time 0 for the other units is not clearly specified (we infer that unit 2 is an exception, but we don't know what most of them mean). Second, it seems as if the response (?) to ICR naive begins even before stimulus application.

      This issue was also mentioned above as the 2nd weakness raised by this reviewer. To explain the meaning of the red lines, and resolve this confusion, we revised the figure caption text to indicate that for all units (except the former unit 2) time 0 indicates SNT stimulation. We also changed the order of the unit examples, placing the former unit 2 in the rightmost position. It is true that for this unit, there is a firing rate change prior to stimulus application, which actually appears as rate attenuation following stimulus application. In this specific case, we consider this activity as “noise”, and note that this neuron-stimulus combination would not be classified as a response (since there is no consistent change across stimulus presentation).

      As a note, while reviewing this figure, we noted an error. We have previously written that the ITI was 10 s, whereas it was actually 18 s long. This has been corrected in the Figure and in the text.

      Figure 2B:

      "The mean error due to the reduced 2-D representation is 0.29 (arbitrary units)." This is unclear. MDS is often described in terms of % of variance explained, is that what this means? If so, the units are not arbitrary; otherwise, it's unclear whether specifying a value with arbitrary units adds any value.

      This is a very good point, and we thank the reviewer for identifying this mistake. The units are not arbitrary! They are units of correlation distance. We now added a scale bar (a square) to panel 2B to indicate what a distance of 0.1. Following this comment, we also calculated the mean error in the original data, and noted the ratio between the mean absolute error (due to considering only two dimensions) and the mean original distances. We also now report the value of the first two eigenvalues. Specifically, we now write:

      “Note that like all dimensionally reduced representations, the representation in Fig. 2B is an approximation. Here, the first two eigenvalues of account for 44.6% of the variance of the original distances (30.4% and 14.2%, respectively for the first and second dimension). Another way to evaluate the representation is via the mean error due to the reduced 2-D representation. Here, it is 0.29, whereas the mean of the original distances is 0.73.”

      Figure 3A:

      a) There is a truncated label (or something) above the panel letter.

      Thanks. Corrected. This was part of the “Figure” label

      b) The graphic for the "adjusted pattern" also fits the criterion of the "pattern": for example, in the top row the activity for ICR is still higher than for any other stimulus, thus fulfilling the criterion of a "pattern" and not just an "adjusted pattern."

      That was not our intention. An adjusted pattern does not necessarily fulfill the (non-adjusted) “pattern” (while the opposite is true). We have now revised the rightmost panel in figure 3A, adding both “&s” to indicate that all three conditions must be fulfilled, and in attempt for a more intuitive representation, applied a different background denoting stimuli with irrelevant responses. We also changed the terms in the legend within the panel, making them more accurate: (Thus, “strong activity” was changed to “stronger responses”). In addition, we revised the text and figure legends in attempt to better clarify these definitions.

      Figure 3B:

      I'm assuming that the columns of the heatmap correspond to different urine stimuli, and that the color is normalized firing rate. But readers should not have to guess.

      True, and agreed. We added legends to clarify this.

      Figure 4B:

      The caption should mention that the pairwise measures are between the stimulus columns of panel A.

      We revised the caption to indicate this. Note that we also added two additional panels to this figure.

      Figure 5A&B:

      Instead of a multiple-comparisons correction, it seems likely to be better to use a 2-way ANOVA. At a minimum, the nature of the multiple-comparisons correction needs to be specified (many are conservative, but they differ in the extent of how conservative they are).

      We now write in the text that we used a Bonferroni correction (this information previously appeared only in the caption). We also found an error in the caption. We previously wrote that we used a binomial exact test for both panels A and B. However, only the data in panel A was calculated with a binomial exact test. The data in panel B was calculated with a one-way ANOVA.

      We now also applied a 2-way ANOVA to response magnitudes (i.e., panel B). We find a main effect of stimulus, but not of state, and no effect of interaction between the two. This is consistent with our previous analyses. This analysis is now included in the text. We thank the reviewer for this suggestion.

      Editor's note:

      Should you choose to revise your manuscript, if you have not already done so, please include full statistical reporting including exact p-values wherever possible alongside the summary statistics (test statistic and df) and, where appropriate, 95% confidence intervals. These should be reported for all key questions and not only when the p-value is less than 0.05 in the main manuscript.

    1. Author response:

      The following is the authors’ response to the original reviews

      We thank the Reviewers for their thorough reading and thoughtful feedback. Below, we address each of the concerns raised in the public reviews, and outline our revisions that aim to further clarify and strengthen the manuscript.

      In our response, we clarify our conceptualization of elasticity as a dimension of controllability, formalizing it within an information-theoretic framework, and demonstrating that controllability and its elasticity are partially dissociable. Furthermore, we provide clarifications and additional modeling results showing that our experimental design and modeling approach are well-suited to dissociating elasticity inference from more general learning processes, and are not inherently biased to find overestimates of elasticity. Finally, we clarify the advantages and disadvantages of our canonical correlation analysis (CCA) approach for identifying latent relationships between multidimensional data sets, and provide additional analyses that strengthen the link between elasticity estimation biases and a specific psychopathology profile. 

      Public Reviews:

      Reviewer 1 (Public review): 

      This research takes a novel theoretical and methodological approach to understanding how people estimate the level of control they have over their environment, and how they adjust their actions accordingly. The task is innovative and both it and the findings are well-described (with excellent visuals). They also offer thorough validation for the particular model they develop. The research has the potential to theoretically inform the understanding of control across domains, which is a topic of great importance.

      We thank the Reviewer for their favorable appraisal and valuable suggestions, which have helped clarify and strengthen the study’s conclusion. 

      An overarching concern is that this paper is framed as addressing resource investments across domains that include time, money, and effort, and the introductory examples focus heavily on effort-based resources (e.g., exercising, studying, practicing). The experiments, though, focus entirely on the equivalent of monetary resources - participants make discrete actions based on the number of points they want to use on a given turn. While the same ideas might generalize to decisions about other kinds of resources (e.g., if participants were having to invest the effort to reach a goal), this seems like the kind of speculation that would be better reserved for the Discussion section rather than using effort investment as a means of introducing a new concept (elasticity of control) that the paper will go on to test.

      We thank the Reviewer for pointing out a lack of clarity regarding the kinds of resources tested in the present experiment. Investing additional resources in the form of extra tickets did not only require participants to pay more money. It also required them to invest additional time – since each additional ticket meant making another attempt to board the vehicle, extending the duration of the trial, and attentional effort – since every attempt required precisely timing a spacebar press as the vehicle crossed the screen. Given this involvement of money, time, and effort resources, we believe it would be imprecise to present the study as concerning monetary resources in particular. That said, we agree with the Reviewer that results might differ depending on the resource type that the experiment or the participant considers most. Thus, we now clarify the kinds of resources the experiment involved (lines 87-97): 

      “To investigate how people learn the elasticity of control, we allowed participants to invest different amounts of resources in attempting to board their preferred vehicle. Participants could purchase one (40 coins), two (60 coins), or three tickets (80 coins) or otherwise walk for free to the nearest location. Participants were informed that a single ticket allowed them to board only if the vehicle stopped at the station, while additional tickets provided extra chances to board even after the vehicle had left the platform. For each additional ticket, the chosen vehicle appeared moving from left to right across the screen, and participants could attempt to board it by pressing the spacebar when it reached the center of the screen. Thus, each additional ticket could increase the chance of boarding but also required a greater investment of resources—decreasing earnings, extending the trial duration, and demanding attentional effort to precisely time a button press when attempting to board.”

      In addition, in the revised discussion, we now highlight the open question of whether inferences concerning the elasticity of control generalize across different resource domains (lines 341-348):

      “Another interesting possibility is that individual elasticity biases vary across different resource types (e.g., money, time, effort). For instance, a given individual may assume that controllability tends to be highly elastic to money but inelastic to effort. Although the task incorporated multiple resource types (money, time, and attentional effort), the results may differ depending on the type of resources on which the participant focuses. Future studies could explore this possibility by developing tasks that separately manipulate elasticity with respect to different resource types. This would clarify whether elasticity biases are domain-specific or domaingeneral, and thus elucidate their impact on everyday decision-making.”

      Setting aside the framing of the core concepts, my understanding of the task is that it effectively captures people's estimates of the likelihood of achieving their goal (Pr(success)) conditional on a given investment of resources. The ground truth across the different environments varies such that this function is sometimes flat (low controllability), sometimes increases linearly (elastic controllability), and sometimes increases as a step function (inelastic controllability). If this is accurate, then it raises two questions.

      First, on the modeling front, I wonder if a suitable alternative to the current model would be to assume that the participants are simply considering different continuous functions like these and, within a Bayesian framework, evaluating the probabilistic evidence for each function based on each trial's outcome. This would give participants an estimate of the marginal increase in Pr(success) for each ticket, and they could then weigh the expected value of that ticket choice (Pr(success)*150 points) against the marginal increase in point cost for each ticket. This should yield similar predictions for optimal performance (e.g., opt-out for lower controllability environments, i.e., flatter functions), and the continuous nature of this form of function approximation also has the benefit of enabling tests of generalization to predict changes in behavior if there was, for instance, changes in available tickets for purchase (e.g., up to 4 or 5) or changes in ticket prices. Such a model would of course also maintain a critical role for priors based on one's experience within the task as well as over longer timescales, and could be meaningfully interpreted as such (e.g., priors related to the likelihood of success/failure and whether one's actions influence these). It could also potentially reduce the complexity of the model by replacing controllability-specific parameters with multiple candidate functions (presumably learned through past experience, and/or tuned by experience in this task environment), each of which is being updated simultaneously.

      We thank the Reviewer for suggesting this interesting alternative modeling approach. We agree that a Bayesian framework evaluating different continuous functions could offer advantages, particularly in its ability to generalize to other ticket quantities and prices. To test the Reviewer's suggestion, we implemented a Bayesian model where participants continuously estimate both controllability and its elasticity as a mixture of three archetypal functions mapping ticket quantities to success probabilities. The flat function provides no control regardless of how many tickets are purchased (corresponding to low controllability). The step function provides the same level of control as long as at least one ticket is purchased (inelastic controllability). The linear function increases control proportionally with each additional ticket (elastic controllability). The model computes the likelihood that each of the functions produced each new observation, and accordingly updates its beliefs. Using these beliefs, the model estimates the probability of success for purchasing each number of tickets, allowing participants to weigh expected control against increasing ticket costs. Despite its theoretical advantages for generalization to different ticket quantities, this continuous function approximation model performed significantly worse than our elastic controllability model (log Bayes Factor > 4100 on combined datasets). We surmise that the main advantage offered by the elastic controllability model is that it does not assume a linear increase in control as a function of resource investment – even though this linear relationship was actually true in our experiment and is required for generalizing to other ticket quantities, it likely does not match what participants were doing. We present these findings in a new section ‘Testing alternative methods’ (lines 686-701):

      “We next examined whether participant behavior would be better characterized as a continuous function approximation rather than the discrete inferences in our model. To test this, we implemented a Bayesian model where participants continuously estimate both controllability and its elasticity as a mixture of three archetypal functions mapping ticket quantities to success probabilities. The flat function provides no control regardless of how many tickets are purchased (corresponding to low controllability). The step function provides full control as long as at least one ticket is purchased (inelastic controllability). The linear function linearly increases control with the number of extra tickets (i.e., 0%, 50%, and 100% control for 1, 2, and 3 tickets, respectively; elastic controllability). The model computes the likelihood that each of the functions produced each new observation, and accordingly updates its beliefs. Using these beliefs, the model estimates the probability of success for purchasing each number of tickets, allowing participants to weigh expected control against increasing ticket costs. Despite its theoretical advantages for generalization to different ticket quantities, this continuous function approximation model performed significantly worse than the elastic controllability model (log Bayes Factor > 4100 on combined datasets), suggesting that participants did not assume that control increases linearly with resource investment.”

      We also refer to this analysis in our updated discussion (326-339): 

      “Second, future models could enable generalization to levels of resource investment not previously experienced. For example, controllability and its elasticity could be jointly estimated via function approximation that considers control as a function of invested resources. Although our implementation of this model did not fit participants’ choices well (see Methods), other modeling assumptions or experimental designs may offer a better test of this idea.”

      Second, if the reframing above is apt (regardless of the best model for implementing it), it seems like the taxonomy being offered by the authors risks a form of "jangle fallacy," in particular by positing distinct constructs (controllability and elasticity) for processes that ultimately comprise aspects of the same process (estimation of the relationship between investment and outcome likelihood). Which of these two frames is used doesn't bear on the rigor of the approach or the strength of the findings, but it does bear on how readers will digest and draw inferences from this work. It is ultimately up to the authors which of these they choose to favor, but I think the paper would benefit from some discussion of a common-process alternative, at least to prevent too strong of inferences about separate processes/modes that may not exist. I personally think the approach and findings in this paper would also be easier to digest under a common-construct approach rather than forcing new terminology but, again, I defer to the authors on this.

      We acknowledge the Reviewer's important point about avoiding a potential "jangle fallacy." We entirely agree with the Reviewer that elasticity and controllability inferences are not distinct processes. Specifically, we view resource elasticity as a dimension of controllability, hence the name of our ‘elastic controllability’ model. In response to this and other Reviewers’ comments, in the revised manuscript, we now offer a formal definition of elasticity as the reduction in uncertainty about controllability due to knowing the amount of resources available to the agent (lines 16-20; see further details in response to Reviewer 3 below).  

      With respect to how this conceptualization is expressed in the modeling, we note that the representation in our model of maximum controllability and its elasticity via different variables is analogous to how a distribution may be represented by separate mean and variance parameters. Even the model suggested by the Reviewer required a dedicated variable representing elastic controllability, namely the probability of the linear controllability function. More generally, a single-process account allows that different aspects of the said process would be differently biased (e.g., one can have an accurate estimate of the mean of a distribution but overestimate its variance). Therefore, our characterization of distinct elasticity and controllability biases (or to put it more accurately, 'elasticity of controllability bias' and 'maximum controllability bias') is consistent with a common construct account.

      To avoid misunderstandings, we have now modified the text to clarify that we view elasticity as a dimension of controllability that can only be estimated in conjunction with controllability. Here are a few examples:

      Lines 21-28: “While only controllable environments can be elastic, the inverse is not necessarily true – controllability can be high, yet inelastic to invested resources – for example, choosing between bus routes affords equal control over commute time to anyone who can afford the basic fare (Figure 1; Supplementary Note 1). That said, since all actions require some resource investment, no controllable environment is completely inelastic when considering the full spectrum of possible agents, including those with insufficient resources to act (e.g., those unable to purchase a bus fare or pay for a fixed-price meal).”

      Lines 45-47: “Experimental paradigms to date have conflated overall controllability and its elasticity, such that controllability was either low or elastic[16-20]. The elasticity of control, however, must be dissociated from overall controllability to accurately diagnose mismanagement of resources.”

      Lines 70-72: “These findings establish elasticity as a crucial dimension of controllability that guides adaptive behavior, and a computational marker of control-related psychopathology.”

      Lines 87-88: “To investigate how people learn the elasticity of control, we allowed participants to invest different amounts of resources in attempting to board their preferred vehicle.”

      Reviewer 2 (Public review):

      This research investigates how people might value different factors that contribute to controllability in a creative and thorough way. The authors use computational modeling to try to dissociate "elasticity" from "overall controllability," and find some differential associations with psychopathology. This was a convincing justification for using modeling above and beyond behavioral output and yielded interesting results. Interestingly, the authors conclude that these findings suggest that biased elasticity could distort agency beliefs via maladaptive resource allocation. Overall, this paper reveals some important findings about how people consider components of controllability.

      We appreciate the Reviewer's positive assessment of our findings and computational approach to dissociating elasticity and overall controllability.

      The primary weakness of this research is that it is not entirely clear what is meant by "elastic" and "inelastic" and how these constructs differ from existing considerations of various factors/calculations that contribute to perceptions of and decisions about controllability. I think this weakness is primarily an issue of framing, where it's not clear whether elasticity is, in fact, theoretically dissociable from controllability. Instead, it seems that the elements that make up "elasticity" are simply some of the many calculations that contribute to controllability. In other words, an "elastic" environment is inherently more controllable than an "inelastic" one, since both environments might have the same level of predictability, but in an "elastic" environment, one can also partake in additional actions to have additional control overachieving the goal (i.e., expend effort, money, time).

      We thank the Reviewer for highlighting the lack of clarity about the concept of elasticity. We first clarify that elasticity cannot be entirely dissociated from controllability because it is a dimension of controllability. If no controllability is afforded, then there cannot be elasticity or inelasticity. This is why in describing the experimental environments, we only label high-controllability, but not low-controllability, environments as ‘elastic’ or ‘inelastic’. For further details on this conceptualization of elasticity, and associated revisions of the text, see our response above to Reviewer 1. 

      Second, we now clarify that controllability can also be computed without knowing the amount of resources the agent is able and willing to invest, for instance by assuming infinite resources available or a particular distribution of resource availabilities. However, knowing the agent’s available resources often reduces uncertainty concerning controllability. This reduction in uncertainty is what we define as elasticity. Since any action requires some resources, this means that no controllable environment is entirely inelastic if we also consider agents that do not have enough resources to commit any action. However, even in this case, environments can differ in the degree to which they are elastic. For further details on this formal definition, and associated revisions of the text, see our response to Reviewer 3.

      Importantly, whether an environment is more or less elastic does not fully determine whether it is more or less controllable. In particular, environments can be more controllable yet less elastic. This is true even if we allow that investing different levels of resources (i.e., purchasing 0, 1, 2, or 3 tickets) constitute different actions, in conjunction with participants’ vehicle choices. Below, we show this using two existing definitions of controllability. 

      Definition 1, reward-based controllability[1]: If control is defined as the fraction of available reward that is controllably achievable, and we assume all participants are in principle willing and able to invest 3 tickets, controllability can be computed in the present task as:

      where P( S'= goal ∣ 𝑆, 𝐴, 𝐶 ) is the probability of reaching the treasure from present state 𝑆 when taking action A and investing C resources in executing the action. In any of the task environments, the probability of reaching the goal is maximized by purchasing 3 tickets (𝐶 = 3) and choosing the vehicle that leads to the goal (𝐴 = correct vehicle). Conversely, the probability of reaching the goal is minimized by purchasing 3 tickets (𝐶 = 3) and choosing the vehicle that does not lead to the goal (𝐴 = wrong vehicle). This calculation is thus entirely independent of elasticity, since it only considers what would be achieved by maximal resource investment, whereas elasticity consists of the reduction in controllability that would arise if the maximal available 𝐶 is reduced. Consequently, any environment where the maximum available control is higher yet varies less with resource investment would be more controllable and less elastic. 

      Note that if we also account for ticket costs in calculating reward, this will only reduce the fraction of achievable reward and thus the calculated control in elastic environments.   

      Definition 2, information-theoretic controllability[2]: Here controllability is defined as the reduction in outcome entropy due to knowing which action is taken:

      where H(S'|S) is the conditional entropy of the distribution of outcomes S' given the present state S, and H(S'|S, A, C) is the conditional entropy of the outcome given the present state, action, and resource investment. 

      To compare controllability, we consider two environments with the same maximum control:

      • Inelastic environment: If the correct vehicle is chosen, there is a 100% chance of reaching the goal state with 1, 2, or 3 tickets. Thus, out of 7 possible action-resource investment combinations, three deterministically lead to the goal state (≥1 tickets and correct vehicle choice), three never lead to it (≥1 tickets and wrong vehicle choice), and one (0 tickets) leads to it 20% of the time (since walking leads to the treasure on 20% of trials).

      • Elastic Environment: If the correct vehicle is chosen, the probability of boarding it is 0% with 1 ticket, 50% with 2 tickets, and 100% with 3 tickets. Thus, out of 7 possible actionresource investment combinations, one deterministically leads to the goal state (3 tickets and correct vehicle choice), one never leads to it (3 tickets and wrong vehicle choice), one leads to it 60% of the time (2 tickets and correct vehicle choice: 50% boarding + 50% × 20% when failing to board), one leads to it 10% of time (2 ticket and wrong vehicle choice), and three lead to it 20% of time (0-1 tickets).

      Here we assume a uniform prior over actions, which renders the information-theoretic definition of controllability equal to another definition termed ‘instrumental divergence’[3,4]. We note that changing the uniform prior assumption would change the results for the two environments, but that would not change the general conclusion that there can be environments that are more controllable yet less elastic. 

      Step 1: Calculating H(S'|S)

      For the inelastic environment:

      P(goal) = (3 × 100% + 3 × 0% + 1 × 20%)/7 = .46, P(non-goal) = .54  H(S'|S) = – [.46 × log<sub>2</sub>(.46) + .54 × log<sub>2</sub>(.54)] = 1 bit

      For the elastic environment:

      P(goal) = (1 × 100% + 1 × 0% + 1 × 60% + 1 × 10% + 3 × 20%)/7 = .33, P(non-goal) = .67 H(S'|S) = – [.33 × log<sub>2</sub>(.33) + .67 × log<sub>2</sub>(.67)] = .91 bits

      Step 2: Calculating H(S'|S, A, C)

      Inelastic environment: Six action-resource investment combinations have deterministic outcomes entailing zero entropy, whereas investing 0 tickets has a probabilistic outcome (20%). The entropy for 0 tickets is: H(S'|C = 0) = -[.2 × log<sub>2</sub> (.2) + 0.8 × log<sub>2</sub> (.8)] = .72 bits. Since this actionresource investment combination is chosen with probability 1/7, the total conditional entropy is approximately .10 bits

      Elastic environment: 2 actions have deterministic outcomes (3 tickets with correct/wrong vehicle), whereas the other 5 actions have probabilistic outcomes:

      2 tickets and correct vehicle (60% success): 

      H(S'|A = correct, C = 2) = – [.6 × log<sub>2</sub> (.6) + .4 × log<sub>2</sub> (.4)] = .97 bits 2 tickets and wrong vehicle (10% success): 

      H(S'|A = wrong, C = 2) = – [.1 × log<sub>2</sub> (.1) + .9 × log<sub>2</sub> (.9)] = .47 bits 0-1 tickets (20% success):

      H(S'|C = 0-1) = – [.2 × log<sub>2</sub> (.2) + .8 × log<sub>2</sub> (.8)] = .72 bits

      Thus the total conditional entropy of the elastic environment is: H(S'|S, A, C) = (1/7) × .97 + (1/7) × .47 + (3/7) × .72 = .52 bits

      Step 3: Calculating I(S'|A, S)  

      Inelastic environment: I(S'; A, C | S) = H(S'|S) – H(S'|S, A, C) = 1 – 0.1 = .9 bits 

      Elastic environment: I(S'; A, C | S) = H(S'|S) – H(S'|S, A, C) = .91 – .52 = .39 bits

      Thus, the inelastic environment offers higher information-theoretic controllability (.9 bits) compared to the elastic environment (.39 bits). 

      Of note, even if each combination of cost and success/failure to reach the goal is defined as a distinct outcome, then information-theoretic controllability is higher for the inelastic (2.81 bits) than for the elastic (2.30 bits) environment. These calculations are now included in the Supplementary materials (Supplementary Note 1). 

      In sum, for both definitions of controllability, we see that environments can be more elastic yet less controllable. We have also revised the manuscript to clarify this distinction (lines 21-28):

      “While only controllable environments can be elastic, the inverse is not necessarily true – controllability can be high, yet inelastic to invested resources – for example, choosing between bus routes affords equal control over commute time to anyone who can afford the basic fare (Figure 1; Supplementary Note 1). That said, since all actions require some resource investment, no controllable environment is completely inelastic when considering the full spectrum of possible agents, including those with insufficient resources to act (e.g., those unable to purchase a bus fare or pay for a fixed-price meal).”

      Reviewer 3 (Public review):

      A bias in how people infer the amount of control they have over their environment is widely believed to be a key component of several mental illnesses including depression, anxiety, and addiction. Accordingly, this bias has been a major focus in computational models of those disorders. However, all of these models treat control as a unidimensional property, roughly, how strongly outcomes depend on action. This paper proposes---correctly, I think---that the intuitive notion of "control" captures multiple dimensions in the relationship between action and outcome is multi-dimensional. In particular, the authors propose that the degree to which outcome depends on how much *effort* we exert, calling this dimension the "elasticity of control". They additionally propose that this dimension (rather than the more holistic notion of controllability) may be specifically impaired in certain types of psychopathology. This idea thus has the potential to change how we think about mental disorders in a substantial way, and could even help us better understand how healthy people navigate challenging decision-making problems.

      Unfortunately, my view is that neither the theoretical nor empirical aspects of the paper really deliver on that promise. In particular, most (perhaps all) of the interesting claims in the paper have weak empirical support.

      We appreciate the Reviewer's thoughtful engagement with our research and recognition of the potential significance of distinguishing between different dimensions of control in understanding psychopathology. We believe that all the Reviewer’s comments can be addressed with clarifications or additional analyses, as detailed below.  

      Starting with theory, the elasticity idea does not truly "extend" the standard control model in the way the authors suggest. The reason is that effort is simply one dimension of action. Thus, the proposed model ultimately grounds out in how strongly our outcomes depend on our actions (as in the standard model). Contrary to the authors' claims, the elasticity of control is still a fixed property of the environment. Consistent with this, the computational model proposed here is a learning model of this fixed environmental property. The idea is still valuable, however, because it identifies a key dimension of action (namely, effort) that is particularly relevant to the notion of perceived control. Expressing the elasticity idea in this way might support a more general theoretical formulation of the idea that could be applied in other contexts. See Huys & Dayan (2009), Zorowitz, Momennejad, & Daw (2018), and Gagne & Dayan (2022) for examples of generalizable formulations of perceived control.

      We thank the Reviewer for the suggestion that we formalize our concept of elasticity to resource investment, which we agree is a dimension of action. We first note that we have not argued against the claim that elasticity is a fixed property of the environment. We surmise the Reviewer might have misread our statement that “controllability is not a fixed property of the environment”. The latter statement is motivated by the observation that controllability is often higher for agents that can invest more resources (e.g., a richer person can buy more things). We clarify this in our revision of the manuscript in lines 8-15 (changes in bold): 

      “The degree of control we possess over our environment, however, may itself depend on the resources we are willing and able to invest. For example, the control a biker has over their commute time depends on the power they are willing and able to invest in pedaling. In this respect, a highly trained biker would typically have more control than a novice. Likewise, the control a diner in a restaurant has over their meal may depend on how much money they have to spend. In such situations, controllability is not fixed but rather elastic to available resources (i.e., in the same sense that supply and demand may be elastic to changing prices[14]).”

      To formalize elasticity, we build on Huys & Dayan’s definition of controllability1 as the fraction of reward that is controllably achievable, 𝜒 (though using information-theoretic definitions[2,3] would work as well). To the extent that this fraction depends on the amount of resources the agent is able and willing to invest (max 𝐶), this formulation can be probabilistically computed without information about the particular agent involved, specifically, by assuming a certain distribution of agents with different amounts of available resources. This would result in a probability distribution over 𝜒. Elasticity can thus be defined as the amount of information obtained about controllability due to knowing the amount of resources available to the agent: I(𝜒; max 𝐶). We have added this formal definition to the manuscript (lines 15-20): 

      “To formalize how elasticity relates to control, we build on an established definition of controllability as the fraction of reward that is controllably achievable[15], 𝜒. Uncertainty about this fraction could result from uncertainty about the amount of resources that the agent is able and willing to invest, 𝑚𝑎𝑥 𝐶. Elasticity can thus be defined as the amount of information obtained about controllability by knowing the amount of available resources: 𝐼(𝜒; 𝑚𝑎𝑥 𝐶).”

      Turning to experiment, the authors make two key claims: (1) people infer the elasticity of control, and (2) individual differences in how people make this inference are importantly related to psychopathology. Starting with claim 1, there are three sub-claims here; implicitly, the authors make all three. (1A) People's behavior is sensitive to differences in elasticity, (1B) people actually represent/track something like elasticity, and (1C) people do so naturally as they go about their daily lives. The results clearly support 1A. However, 1B and 1C are not supported. Starting with 1B, the experiment cannot support the claim that people represent or track elasticity because the effort is the only dimension over which participants can engage in any meaningful decision-making (the other dimension, selecting which destination to visit, simply amounts to selecting the location where you were just told the treasure lies). Thus, any adaptive behavior will necessarily come out in a sensitivity to how outcomes depend on effort. More concretely, any model that captures the fact that you are more likely to succeed in two attempts than one will produce the observed behavior. The null models do not make this basic assumption and thus do not provide a useful comparison.

      We appreciate the Reviewer's critical analysis of our claims regarding elasticity inference, which as detailed below, has led to an important new analysis that strengthens the study’s conclusions. However, we respectfully disagree with two of the Reviewer’s arguments. First, resource investment was not the only meaningful decision dimension in our task, since participant also needed to choose the correct vehicle to get to the right destination. That this was not trivial is evidenced by our exclusion of over 8% of participants who made incorrect vehicle choices more than 10% of the time. Included participants also occasionally erred in this choice (mean error rate = 3%, range [0-10%] now specified in lines 363-366). 

      Second, the experimental task cannot be solved well by a model that simply tracks how outcomes depend on effort because 20% of the time participants reached the treasure despite failing to board their vehicle of choice. In such cases, reward outcomes and control were decoupled. Participants could identify when this was the case by observing the starting location (since depending on the starting location, the treasure location could have been automatically reached by walking), which was revealed together with the outcome. To determine whether participants distinguished between control-related and non-control-related reward, we have now fitted a variant of our model to the data that allows learning from each of these kinds of outcomes by means of a different free parameter. The results show that participants learned considerably more from control-related outcomes. They were thus not merely tracking outcomes, but specifically inferred when outcomes can be attributed to control. We now include this new analysis in the revised manuscript (Methods lines 648-661):

      “To ascertain that participants were truly learning latent estimates of controllability rather than simpler associations, we conducted two complementary analyses.

      First, we implemented a simple Q-learning model that directly maps ticket quantities to expected values based on reward prediction errors, without representing latent controllability. This associative model performed substantially worse than even our simple controllability model (log Bayes Factor ≥ 1854 on the combined datasets). Second, we fitted a variant of the elastic controllability model that compared learning from control-related versus chance outcomes via separate parameters (instead of assuming no learning from chance outcomes). Chance outcomes were observed by participants in the 20% of trials where reward and control were decoupled, in the sense that participants reached the treasure regardless of whether they boarded their vehicle of choice. Results showed that participants learned considerably more from control-related, as compared to chance, outcomes (mean learning ratio=1.90, CI= [1.83, 1.97]). Together, these analyses show that participants were forming latent controllability estimates rather than direct action-outcome associations.”

      Controllability inference by itself, however, still does not suffice to explain the observed behavior. This is shown by our ‘controllability’ model, which learns to invest more resources to improve control, yet still fails to capture key features of participants’ behavior, as detailed in the manuscript. This means that explaining participants’ behavior requires a model that not only infers controllability—beyond merely outcome probability—but also assumes a priori that increased effort could enhance control. Building these a priori assumption into the model amounts to embedding within it an understanding of elasticity – the idea that control over the environment may be increased by greater resource investment. 

      That being said, we acknowledge the value in considering alternative computational formulations of adaptation to elasticity, as now expressed in the revised discussion (lines 326-333; reproduced below in response to the Reviewer’s comment on updating controllability beliefs when losing with less than 3 tickets).

      For 1C, the claim that people infer elasticity outside of the experimental task cannot be supported because the authors explicitly tell people about the two notions of control as part of the training phase: "To reinforce participants' understanding of how elasticity and controllability were manifested in each planet, [participants] were informed of the planet type they had visited after every 15 trips." (line 384).

      We thank the Reviewer for highlighting this point. We agree that our experimental design does not test whether people infer elasticity spontaneously. However, our research question was whether people can distinguish between elastic and inelastic controllability. The results strongly support that they can, and this does have potential implications for behavior outside of the experimental task. Specifically, to the extent that people are aware that in some contexts additional resource investment improves control, whereas in other contexts it does not, then our results indicate that they would be able to distinguish between these two kinds of contexts through trial-and-error learning. That said, we agree that investigating whether and how people spontaneously infer elasticity is an interesting direction for future work. We have now added this to the discussion of future directions (lines 287-295):

      “Additionally, real life typically doesn’t offer the streamlined recurrence of homogenized experiences that makes learning easier in experimental tasks, nor are people systematically instructed and trained about elastic and inelastic control in each environment. These complexities introduce substantial additional uncertainty into inferences of elasticity in naturalistic settings, thus allowing more room for prior biases to exert their influences. The elasticity biases observed in the present studies are therefore likely to be amplified in real-life behavior. Future research should examine how these complexities affect judgments about the elasticity of control to better understand how people allocate resources in real-life.”

      Finally, I turn to claim 2, that individual differences in how people infer elasticity are importantly related to psychopathology. There is much to say about the decision to treat psychopathology as a unidimensional construct. However, I will keep it concrete and simply note that CCA (by design) obscures the relationship between any two variables. Thus, as suggestive as Figure 6B is, we cannot conclude that there is a strong relationship between Sense of Agency and the elasticity bias---this result is consistent with any possible relationship (even a negative one). The fact that the direct relationship between these two variables is not shown or reported leads me to infer that they do not have a significant or strong relationship in the data.

      We agree that CCA is not designed to reveal the relationship between any two variables. However, the advantage of this analysis is that it pulls together information from multiple variables. Doing so does not treat psychopathology as unidimensional. Rather, it seeks a particular dimension that most strongly correlates with different aspects of task performance.

      This is especially useful for multidimensional psychopathology data because such data are often dominated by strong correlations between dimensions, whereas the research seeks to explain the distinctions between the dimensions. Similar considerations apply to the multidimensional task parameters, which although less correlated, may still jointly predict the relevant psychopathological profile better than each parameter does in isolation. Thus, the CCA enabled us to identify a general relationship between task performance and psychopathology that accounts for different symptom measures and aspects of controllability inference. 

      Using CCA can thus reveal relationships that do not readily show up in two-variable analyses. Indeed, the direct correlation between Sense of Agency (SOA) and elasticity bias was not significant – a result that, for completeness, we now report in Supplementary Figure 3 along with all other direct correlations. We note, however, that the CCA analysis was preregistered and its results were replicated. Additionally, participants scoring higher on the psychopathology profile also overinvested resources in inelastic environments but did not futilely invest in uncontrollable environments (Figure 6A), providing external validation to the conclusion that the CCA captured meaningful variance specific to elasticity inference. Most importantly, an auxiliary analysis specifically confirmed the contributions of both elasticity bias (Figure 6D, middle plot) and, although not reported in the original paper, of the Sense of Agency score (SOA; p=.03 permutation test; see updated Figure 6D, bottom plot) to the observed canonical correlation. The results thus enable us to safely conclude that differences in elasticity inferences are significantly associated with a profile of control-related psychopathology to which SOA contributed significantly. We now report this when presenting the CCA results (lines 255-257): 

      “Loadings on the side of psychopathology were dominated by an impaired sense of agency (SOA; contribution to canonical correlation: p=.03, Figure 6D, bottom plot), along with obsessive compulsive symptoms (OCD), and social anxiety (LSAS) – all symptoms that have been linked to an impaired sense of control[22-25].”

      Finally, whereas interpretation of individual CCA loadings that were not specifically tested remains speculative, we note that the pattern of loadings largely replicated across the initial and replication studies (see Figure 6B), and aligns with prior findings. For instance, the positive loadings of SOA and OCD match prior suggestions that a lower sense of control leads to greater compensatory effort7, whereas the negative loading for depression scores matches prior work showing reduced resource investment in depression[5-6].

      We have now revised the manuscript to clarify the justification for our analytical approach (lines 236-248):

      “To examine whether the individual biases in controllability and elasticity inference have psychopathological ramifications, we assayed participants on a range of self-report measures of psychopathologies previously linked to a distorted sense of control (see Methods, pg. 24). Examining the direct correlations between model parameters and psychopathology measures (reported in Supplementary Figure 3) does not account for the substantial variance that is typically shared among different forms of psychopathology. For this reason, we instead used a canonical correlation analysis (CCA) to identify particular dimensions within the parameter and psychopathology spaces that most strongly correlate with one another.”

      We also now include a cautionary note in the discussion (lines 309-315):

      “Whereas our pre-registered CCA effectively identified associations between task parameters and a psychopathological profile, this analysis method does not directly reveal relationships between individual variables. Auxiliary analyses confirmed significant contributions of both elasticity bias and sense of agency to the observed canonical correlation, but the contribution of other measures remains to be determined by future work. Such work could employ other established measures of agency, including both behavioral indices and subjective self-reports, to better understand how these constructs relate across different contexts and populations.”

      There is also a feature of the task that limits our ability to draw strong conclusions about individual differences in elasticity inference. As the authors clearly acknowledge, the task was designed "to be especially sensitive to overestimation of elasticity" (line 287). A straightforward consequence of this is that the resulting *empirical* estimate of estimation bias (i.e., the gamma_elasticity parameter) is itself biased. This immediately undermines any claim that references the directionality of the elasticity bias (e.g. in the abstract). Concretely, an undirected deficit such as slower learning of elasticity would appear as a directed overestimation bias. When we further consider that elasticity inference is the only meaningful learning/decisionmaking problem in the task (argued above), the situation becomes much worse. Many general deficits in learning or decision-making would be captured by the elasticity bias parameter. Thus, a conservative interpretation of the results is simply that psychopathology is associated with impaired learning and decision-making.

      We apologize for our imprecise statement that the task was ‘especially sensitive to overestimation of elasticity’, which justifiably led to Reviewer’s concern that slower elasticity learning can be mistaken for elasticity bias. To make sure this was not the case, we made use of the fact that our computational model explicitly separates bias direction (𝜆) from the rate of learning through two distinct parameters, which initialize the prior concentration and mean of the model’s initial beliefs concerning elasticity (see Methods pg. 23). The higher the concentration of the initial beliefs (𝜖), the slower the learning. Parameter recovery tests confirmed that our task enables acceptable recovery of both the bias λ<sub>elasticity</sub> (r=.81) and the concentration 𝜖<sub>elasticity</sub> (r=.59) parameters. And importantly, the level of confusion between the parameters was low (confusion of 0.15 for 𝜖<sub>elasticity</sub> → λ<sub>elasticity</sub> and 0.04 for λ<sub>elasticity</sub>→ 𝜖<sub>elasticity</sub> This result confirms that our task enables dissociating elasticity biases from the rate of elasticity learning. 

      Moreover, to validate that the minimal level of confusion existing between bias and the rate of learning did not drive our psychopathology results, we re-ran the CCA while separating concentration from bias parameters. The results (figure below) demonstrate that differences in learning rate (𝜖) had virtually no contribution to our CCA results, whereas the contribution of the pure bias (𝜆) was preserved. 

      We now report on this additional analysis in the text (lines 617-627):

      “To capture prior biases that planets are controllable and elastic, we introduced parameters λ<sub>controllability</sub> and λ<sub>elasticity</sub>, each computed by multiplying the direction (λ – 0.5) and strength (ϵ) of individuals’ prior belief. 𝜖<sub>controllability</sub> and 𝜖<sub>elasticity</sub> range between 0 and 1, with values above 0.5 indicating a bias towards high controllability or elasticity, and values below 0.5 indicating a bias towards low controllability or elasticity. 𝜖<sub>controllability</sub> and 𝜖<sub>elasticity</sub> are positively valued parameters capturing confidence in the bias. Parameter recovery analyses confirmed both good recoverability (see S2 Table) and low confusion between bias direction and strength (𝜖<sub>controllability</sub> → λ<sub>controllability</sub> = −. 07, λ<sub>controllability</sub> → 𝜖<sub>controllability</sub> =. 16, 𝜖<sub>elasticity</sub> → λ<sub>elasticity</sub> =. 15, λ<sub>elasticity</sub> → 𝜖<sub>elasticity</sub> =. 04), ensuring that observed biases and their relation to psychopathology do not merely reflect slower learning (Supplementary Figure 4), which can result from changes in bias strength but not direction.”

      We also more precisely articulate the impact of providing participants with three free tickets at their initial visits to each planet.

      Showing that a model parameter correlates with the data it was fit to does not provide any new information, and cannot support claims like "a prior assumption that control is likely available was reflected in a futile investment of resources in uncontrollable environments." To make that claim, one must collect independent measures of the assumption and the investment.

      We apologize if this and related statements seemed to be describing independent findings. They were meant to describe the relationship between model parameters and model-independent measures of task performance. It is inaccurate, though, to say that they provide no new information, since results could have been otherwise. For instance, whether a higher controllability bias maps onto resource misallocation in uncontrollable environments (as we observed) depends on the range of this parameter in our population sample. Had the range been more negative, a higher controllability bias could have instead manifested as optimal allocation in controllable environments. Additionally, these analyses serve two other purposes: as a validity check, confirming that our computational model effectively captured observed individual differences, and as a help for readers to understand what each parameter in our model represents in terms of observable behavior. We now better clarify the descriptive purposes of these regressions (lines 214-220, 231-235): 

      “To clarify how fitted model parameters related to observable behavior, we regressed participants’ opt-in rates and extra ticket purchases on the parameters (Figure 6A) ...”

      “... In sum, the model parameters captured meaningful individual differences in how participants allocated their resources across environments, with the controllability parameter primarily explaining variance in resource allocation in uncontrollable environments, and the elasticity parameter primarily explaining variance in resource allocation in environments where control was inelastic.”

      Did participants always make two attempts when purchasing tickets? This seems to violate the intuitive model, in which you would sometimes succeed on the first jump. If so, why was this choice made? Relatedly, it is not clear to me after a close reading how the outcome of each trial was actually determined.

      We thank the Reviewer for highlighting the need to clarify these aspects of the task in the revised manuscript. 

      When participants purchased two extra tickets, they attempted both jumps, and were never informed about whether either of them succeeded. Instead, after choosing a vehicle and attempting both jumps, participants were notified where they arrived at. This outcome was determined based on the cumulative probability of either of the two jumps succeeding. Success meant that participants arrived at where their chosen vehicle goes, whereas failure meant they walked to the nearest location (as determined by where they started from). 

      Though it is unintuitive to attempt a second jump before seeing whether the first succeed, this design choice ensured two key objectives. First, that participants would consistently need to invest not only more money but also more effort and time in planets with high elastic controllability. Second, that the task could potentially generalize to the many real-world situations where the amount of invested effort has to be determined prior to seeing any outcome, for instance, preparing for an exam or a job interview. We now explicitly state these details when describing the experimental task (lines 393-395):

      “When participants purchased multiple tickets, they made all boarding attempts in sequence without intermediate feedback, only learning whether they successfully boarded upon reaching their final destination. This served two purposes. First, to ensure that participants would consistently need to invest not only more money but also more effort and time in planets with high elastic controllability. Second, to ensure that results could potentially generalize to the many real-world situations where the amount of invested effort has to be determined prior to seeing any outcome (e.g., preparing for an exam or a job interview).”

      It should be noted that the model is heuristically defined and does not reflect Bayesian updating. In particular, it overestimates control by not using losses with less than 3 tickets (intuitively, the inference here depends on your beliefs about elasticity). I wonder if the forced three-ticket trials in the task might be historically related to this modeling choice.

      We apologize for not making this clear, but in fact losing with less than 3 tickets does reduce the model’s estimate of available control. It does so by increasing the elasticity estimates (a<sub>elastic≥1</sub>,a<sub>elastic2</sub> parameters), signifying that more tickets are needed to obtain the maximum available level of control, thereby reducing the average controllability estimate across ticket investment options. We note this now in the presentation of the computational model (caption Figure 4):

      “A failure to board does not change estimated maximum controllability, but rather suggests that 1 ticket might not suffice to obtain control (a<sub>elastic≥1</sub> + 1; 𝑙𝑖𝑔ℎ𝑡 𝑔𝑟𝑒𝑒𝑛 𝑑𝑖𝑚𝑖𝑛𝑖𝑠ℎ𝑒𝑑). As a result, the model’s estimate of average controllability across ticket options is reduced.”

      It would be interesting to further develop the model such that losing with less than 3 tickets would also impact inferences concerning the maximum available control, depending on present beliefs concerning elasticity, but the forced three-ticket purchases already expose participants to the maximum available control, and thus, the present data may not be best suited to test such a model. These trials were implemented to minimize individual differences concerning inferences of maximum available control, thereby focusing differences on elasticity inferences. We now explicitly address these considerations in the revised discussion (lines 326-333) with the following: 

      “Future research could explore alternative models for implementing elasticity inference that extend beyond our current paradigm. First, further investigation is warranted concerning how uncertainty about controllability and its elasticity interact. In the present study, we minimized individual differences in the estimation of maximum available control by providing participants with three free tickets at their initial visits to each planet. We made this design choice to isolate differences in the estimation of elasticity, as opposed to maximum controllability. To study how these two types of estimations interact, future work could benefit from modifying this aspect of our experimental design.”

      Furthermore, we have now tested a Bayesian model suggested by Reviewer 1, but we found that this model fitted participants’ choices worse (see details in the response to Reviewer 1’s comments). 

      Recommendations for the authors:

      Reviewer 1 (Recommendations for the authors):

      In the introduction, the definition of controllability and elasticity, and the scope of "resources" investigated in the current study were unclear. If I understand correctly, controllability is defined as "the degree to which actions influence the probability of obtaining a reward", and elasticity is defined as the change in controllability based on invested resources. This would define the controllability of the environment and the elasticity of controllability of the environment. However, phrases such as "elastic environment" seem to imply that elasticity can directly attach to an environment, instead of attaching to the controllability of the environment.

      We thank the Reviewer for highlighting the need to clarify our conceptualization of elasticity and controllability. We now provide formal definitions of both, with controllability defined as the fraction of controllably achievable reward[1], and elasticity as the reduction in uncertainty about controllability due to knowing the amount of resources the agent is willing and able to invest (see further details in the response to Reviewer 3’s public comments). In the revised manuscript, we now use more precise language to clarify that elasticity is a property of controllability, not of environments themselves. In addition, we now clarify that the current study manipulated monetary, attentional effort, and time costs together (see further details in the response to Reviewer 1’s public comments).   

      (2) Some of the real-world examples were confusing. For example, the authors mention that investing additional effort due to the belief that this leads to better outcomes in OCD patients is overestimated elasticity, but exercising due to the belief that this can make one taller is overestimated controllability. What's the distinction between the examples? The example of the chess expert practicing to win against a novice, because the amount of effort they invest would not change their level of control over the outcome is also unclear. If the control over the outcome depends on their skill set, wouldn't practicing influence the control over the outcome? In the case of the meeting time example, wouldn't the bus routes differ in their time investments even though they are the same price? In addition to focusing the introductory examples around monetary resources, I would also generally recommend tightening the link between those examples and the experimental task.

      We thank the Reviewer for highlighting the need to clarify the examples used to illustrate elasticity and controllability. We have now revised these examples to more clearly distinguish between the concepts and to strengthen their connection to the experimental task.

      Regarding the OCD example, the possibility that OCD patients overestimate elasticity comes from research suggesting they experience low perceived control but nevertheless engage in excessive resource investment2, reflecting a belief that only through repeated and intense effort can they achieve sufficient control over outcomes. As an example, consider an OCD patient investing unnecessary effort in repeatedly locking their door. This behavior cannot result from an overestimation of controllability because controllability truly is close to maximal. It also cannot result from an underestimation of the maximum attainable control, since in that case investing more effort is futile. Such behavior, however, can result from an overestimation of the degree to which controllability requires effort (i.e., overestimation of elasticity). 

      Similarly, with regards to the chess expert, we intended to illustrate a situation where given their current level, the chess expert is already virtually guaranteed to win, such that additional practice time does not improve their chances. Conversely, the height example illustrates overestimated controllability because the outcome (becoming taller through exercise) is in fact not amenable to control through any amount of resource investment.

      Finally, the meeting time example was meant to illustrate that if the desired outcome is reaching a meeting in time, then different bus routes that cost the same provide equal control over this outcome to anyone who can afford the basic fare. This demonstrates inelastic controllability with respect to money, as spending more on transportation doesn't increase the probability of reaching the meeting on time. The Reviewer correctly notes that time investment may differ between routes. However, investing more time does not improve the expected outcome. This illustrates that inelastic controllability does not preclude agents from investing more resources, but such investment does not increase the fraction of controllably achievable reward (i.e., the probability of reaching the meeting in time).

      In the revised manuscript, we’ve refined each of the above examples to better clarify the specific resources being considered, the outcomes they influence, and their precise relationship to both elasticity and controllability: 

      OCD (lines 40-43): Conversely, the repetitive and unusual amount of effort invested by people with obsessive-compulsive disorder in attempts to exert control[23,24] could indicate an overestimation of elasticity, that is, a belief that adequate control can only be achieved through excessive and repeated resource investment[25].  

      Chess expert (54-57): Alternatively, they may do so because they overestimate the elasticity of control – for example, a chess expert practicing unnecessarily hard to win against a novice, when their existing skill level already ensures control over the match's outcome.

      Height (lines 53-54): A given individual, for instance, may tend to overinvest resources because they overestimate controllability – for example, exercising due to a misguided belief that that this can make one taller, when in fact height cannot be controlled. 

      Meeting time (lines 26-28): Choosing between bus routes affords equal control over commute time to anyone who can afford the basic fare (Figure 1).

      Methods

      (1) In the elastic controllability model definition, controllability is defined as "the belief that boarding is possible" (with any number of tickets). The definition again is different from in the task description where controllability is defined as "the probability of the chosen vehicle stopping at the platform if purchasing a single ticket."

      We clarify that "the probability of the chosen vehicle stopping at the platform if purchasing a single ticket" is our definition for inelastic controllability, as opposed to overall/maximum controllability, as stated here (lines 101-103):

      "We defined inelastic controllability as the probability that even one ticket would lead to successfully boarding the vehicle, and elastic controllability as the degree to which two extra tickets would increase that probability."

      Overall controllability is the summation of the two. This summation is referred to in the elastic controllability model definition as the "the belief that boarding is possible". We now clarify this in the caption to figure 4:

      Elastic Controllability model: Represents beliefs about maximum controllability (black outline) and the degree to which one or two extra tickets are necessary to obtain it. These beliefs are used to calculate the expected control when purchasing 1 ticket (inelastic controllability) and the additional control afforded by 2 and 3 tickets (elastic controllability).    

      We also clarify this in the methods when describing the parameterization of the model (lines 529-531): 

      The expected value of one beta distribution (defined by a,sub>control</sub>, b,sub>control</sub>) represents the belief that boarding is possible (controllability) with any number of tickets. 

      (2) The free parameter K is confusing. What is the psychological meaning of this parameter? Is it there just to account for the fact that failure with 3 tickets made participants favor 3 tickets or is there meaning attached to including this parameter?

      This parameter captures how participants update their beliefs about resource requirements after failing to board with maximum resource investment. Our psychological interpretation is that participants who experience failure despite maximum investment (3 tickets) prioritize resolving uncertainty about whether control is fundamentally possible (before exploring whether control is elastic), which can only be determined by continuing to invest maximum resources. 

      We now clarify this in the methods (lines 555-559):

      To account for our finding that failure with 3 tickets made participants favor 3, over 1 and 2, tickets, we introduced a modified elastic controllability* model, wherein purchasing extra tickets is also favored upon receiving evidence of low controllability (loss with 3 tickets). This effect was modulated by a free parameter 𝜅 which reflects a tendency to prioritize resolving uncertainty about whether control is at all possible by investing maximum resources.

      This interpretation is supported by our analysis of 3-ticket choice trajectories (Supplementary Figure 2 presented in response to Reviewer 2). As shown in the figure, participants who win less than 50% of their 3-ticket attempts persistently purchase 3 tickets over the first 10 trials, despite frequent failures. This persistence gradually declines as participants accumulate evidence about their limited control, corresponding with an increase in opt-out rates.

      (3) Some additional details about the task design would be helpful. It seems that participants first completed 90 practice trials and were informed of the planet type every 15 trials (6 times during practice). What message is given to the participants about the planets? Did the authors analyze the last 15 trials of each condition in the regression analysis, and all 30 trials in the modeling analysis? How does the computational model (especially the prior beliefs parameters) reset when the planet changes? How do points accumulate over the session and/or are participants motivated to budget the points? Is it possible for participants to accumulate many points and then switch to a heuristic of purchasing 3 tickets on each trial?

      We apologize for not previously clarifying these details of the experimental design.

      During practice blocks, participants received explicit feedback about each planet's controllability characteristics, to help them understand when additional resources would or would not improve their boarding success. For high inelastic controllability planets, the message read: "Your ride actually would stop for you with 1 ticket! So purchasing extra tickets, since they do cost money, is a WASTE." For low controllability planets: "Doesn't seem like the vehicle stops for you nor does purchasing extra tickets help." Lastly, for high elastic controllability planets: "Hopefully by now it's clear that only by purchasing 3 tickets (LOADING AREA) are you consistently successful in catching your ride." We now include these messages in the methods section describing the task (lines 453-458).

      We indeed analyzed the last 15 trials of each condition in the regression analysis, and all 30 trials in the modeling analysis. Whereas the modeling attempted to explain participants’ learning process, the regression focused on explaining the resultant behavior, which in our pilot data (N=19), manifested fairly stably in the last 15 trials (ticket choices SD = 0.33 compared to .63 in the first 15 trials). The former is already stated in the text (lines 409-415), and we now also clarify the latter when discussing the model fitting procedure (line 695): 

      Reinforcement-learning models were fitted to all choices made by participants via an expectation maximization approach used in previous work.

      The computational model was initialized with the same prior parameters for all planets. When a participant moved to a new planet, the model's beliefs were reset to these prior values, capturing how participants would approach each new environment with their characteristic expectations about controllability and elasticity. We now clarify this in the methods (line 628): 

      For each new planet participants encountered, these parameters were used to initialize the beta distributions representing participants’ beliefs

      Points accumulated across all planets throughout the session, with participants explicitly motivated to maximize their total points as this directly determined their monetary bonus payment. To address the Reviewer's question about changes in ticket purchasing behavior, we conducted a mixed probit regression examining whether accumulated points influenced participants’ decisions to purchase extra tickets. We did not find such an effect (𝛽<sub>coins accumulated</sub> \= .01 𝑝 = .87), indicating that participants did not switch to simple heuristic strategies after accumulating enough coins. We now report this analysis in the methods (lines 421-427):

      Points accumulated across all planets throughout the session, with participants explicitly motivated to maximize their total points as this directly determined their monetary bonus payment. To ensure that accumulated gains did not lead participants to adopt a simple heuristic strategy of always purchasing 3 tickets, we conducted a mixed probit regression examining whether the number of accumulated coins influenced participants' decisions to purchase extra tickets. We did not find such an effect (𝛽<sub>coins accumulated</sub> = .01 𝑝 = .87), ruling out the potential strategy shift.

      Following the modeling section, it may be helpful to have a table of the fitted models, the parameters of each model, and the meaning/interpretation of each parameter.

      We thank the Reviewer for this suggestion. We have now added a table (Supplementary Table 3) that summarizes all fitted models, their parameters, and the meaning/interpretation of each parameter.

      (1) The conclusions from regressing the task choices (opt-in rates and ticket purchases) on the fitted parameters seem confusing given that the model parameters were fitted on the task behavior, and the relationship between these variables seems circular. For example, the authors found that preferences for purchasing 2 or 3 tickets (a2 and a3; computational parameters) were associated with purchasing more tickets (task behavior). But wouldn't this type of task behavior be what the parameters are explaining? It's not clear whether these correlation analyses are about how individuals allocate their resources or about the validity check of the parameters. Perhaps analyses on individual deviation from the optimal strategy and parameter associations with such deviation are better suited for the questions about whether individual biases lead to resource misallocation.

      We thank the Reviewer for highlighting this seeming confusion. These regressions were meant to describe the relationship between model parameters and model-independent measures of task performance. This serves three purposes. First, a validity check, confirming that our computational model effectively captured observed individual differences. Second, to help readers understand what each parameter in our model represents in terms of observable behavior. Third, to examine in greater detail how parameter values specifically mapped onto observable behavior. For instance, whether a higher controllability bias maps onto resource misallocation in uncontrollable environments (as we observed) depends on the range of this parameter in our population sample. Had the range been more negative, a higher controllability bias could have instead manifested as optimal allocation in controllable environments. We now better clarify the descriptive purposes of these regressions (lines 214-220, 231-235): 

      To clarify how fitted model parameters related to observable behavior, we regressed participants’ opt-in rates and extra ticket purchases on the parameters (Figure 6A) ... 

      ... In sum, the model parameters captured meaningful individual differences in how participants allocated their resources across environments, with the controllability parameter primarily explaining variance in resource allocation in uncontrollable environments, and the elasticity parameter primarily explaining variance in resource allocation in environments where control was inelastic.  

      Regarding the suggestion to analyze deviation from optimal strategy, this corresponds with our present approach in that opting in is always optimal in high controllability environments and always non-optimal in low controllability environments, and similarly, purchasing extra tickets is always optimal in elastic controllability environments and always non-optimal elsewhere. Thus, positive or negative coefficients can be directly translated into closer or farther from optimal, depending on the planet type, as indicated in the figure by color. We now clarify this mapping in the figure legend:

      (2) Minor: The legend of Figure 6A is difficult to read. It might be helpful to label the colors as their planet types (low controllability, high elastic controllability, high inelastic controllability).

      We thank the Reviewer for this helpful suggestion. We have revised the figure accordingly.

      Reviewer 2 (Recommendations for the authors):

      As noted above, I'm not sure I agree with (or perhaps don't fully understand) the claims the authors make about the distinctions between their "elastic" and "inelastic" experimental conditions. Let's take the travel example from Figure 1 - is this not just an example of “hierarchical” controllability calculations? In other words, in the elastic example, my choice is between going one speed or another (i.e., exerting more or less effort), and in the inelastic example, my choice is first, which route to take (also a consideration of speed, but with lower effort costs than the elastic scenario), and second, an estimate of the time cost (not within my direct control, but could be estimated). In the elastic scenarios, additional value considerations vary between options, and in others (inelastic), they don't, with control over the first choice point (which bus route to choose, or which lunch option to take), but not over the price. I wonder if the paper would be better framed (or emphasized) as exploring the influences of effort and related "costs" of control. There isn't really such a thing as controllability that does not have any costs associated with it (whether that be action costs, effort, money, or simply scenario complexity).

      We thank the Reviewer for highlighting the need to clarify our distinction between elastic and inelastic controllability as it manifests in our examples. We first clarify that elasticity concerns how controllability varies with resources, not costs. Though resource investment and costs are often tightly linked, that is not always the case, especially not when comparing between agents. For example, it may be equally difficult (i.e., costly) for a professional biker to pedal at a high speed as it is for a novice to pedal at a medium speed, simply because the biker’s muscles are better trained. This resource advantage increases the biker’s control over his commute time without incurring additional costs as compared to the novice. We now clarify this distinction in the text by revising our example to (lines 9-11): 

      “For example, the control a biker has over their commute time depends on the power they are willing and able to invest in pedaling. In this respect, a highly trained biker would typically have more control than a novice.”

      Second, whereas in our examples additional value considerations indeed vary in elastic environments, that does not have to be the case, and indeed, that is not the case in our experiment. In our experimental task, participants are given the option to purchase as many tickets as they wish regardless of whether they are in an elastic or an inelastic environment.  

      We agree that elastic environments often raise considerations regarding the cost of control (for instance, whether it is worth it to pedal harder to get to the destination in time). To consider this cost against potential payoffs, however, the agent must first determine what are the potential payoffs – that is, it must determine the degree to which controllability is elastic to invested resources. It is this antecedent inference that our experiment studies. We uniquely study this inference using environments where control may not only be low or high, but also, where high control may or may not require additional resource investments. We now clarify this point in Figure 1’s caption:

      “In all situations, agents must infer the degree to which controllability is elastic to be able to determine whether the potential gains in control outweigh the costs of investing additional resources (e.g., physical exertion, money spent, time invested).”

      For a formal definition of the elasticity of control, see our response to Reviewer 3’s public comments. 

      Relatedly, another issue I have with the distinctions between inelastic/elastic is that a high/elastic condition has inherently ‘more’ controllability than a high/inelastic condition, no matter what. For example, in the lunch option scenario, I always have more control in the elastic situation because I have two opportunities to exert choice (food option ‘and’ cost). Is there really a significant difference, then, between calling these distinctions "elastic/inelastic" vs. "higher/lower controllability?" Not that it's uninteresting to test behavioral differences between these two types of scenarios, just that it seems unnecessary to refer to these as conceptually distinct.

      As noted in the response above, control over costs may be higher in elastic environments, but it does not have to be so, as exemplified by the elastic environments in our experimental task. For a fuller explanation of why higher elasticity does not imply higher controllability, see our response to Reviewer 2’s public comments. 

      I also wonder whether it's actually the case that people purchased more tickets in the high control elastic condition simply because this is the optimal solution to achieve the desired outcome, not due to a preference for elastic control. To test this, you would need to include a condition in which people opted to spend more money/effort to have high elastic control in an instance where it was not beneficial to do so.

      We appreciate the Reviewer's question about potential preferences for elastic control. We first clarify that participants did not choose which environment type they encountered, so if control was low or inelastic, investing extra resources did not give them more control. Furthermore, our results show that the average participant did not prefer a priori to purchase more tickets. This is evidenced by participants’ successful adaptation to inelastic environments wherein they purchased significantly fewer tickets (see Figure 2B and 2C), and by participants’ parameter fits, which reveal an a priori bias to assume that controllability is inelastic (𝜆<sub>elasticity</sub> \= .16 ± .19), as well as a fixed preference against purchasing the full number of tickets (𝛼<sub>3</sub> \= −.74 ± .37). 

      We now clarify these findings by including a table of all parameter fits in the revised manuscript (see response to Reviewer 1). 

      It was interesting that the authors found that failure with 3 tickets made people more likely to continue to try 3 tickets, however, there is another possible interpretation. Could it be that this is simply evidence of a general controllability bias, where people just think that it is expected that you should be able to exert more money/effort/time to gain control, and if this initially fails, it is an unusual outcome, and they should try again? Did you look at this trajectory over time? i.e., whether repeated tries with 3 tickets immediately followed a failure with 3 tickets? Relatedly, does the perseveration parameter from the model also correlate with psychopathology?

      We thank the Reviewer for this suggestion. Our model accounts for a general controllability bias through the 𝜆<sub>controllability</sub> parameter, which represents a prior belief that planets are controllable. It also accounts, through the 𝜆<sub>elasticity</sub> parameter, for the prior belief that you should be able to exert more money/effort/time to gain control. Now, our addition of 𝜅 to the model captures the observation that failures with 3 tickets made participants more likely to purchase 3 tickets when they opted in. If this observation was due to participants not accepting that the planet is not controllable, then we would expect the increase in 3-ticket purchases when opting in to be coupled with a diminished reduction in opting in. To determine whether this was the case, we tested a variant of our model where 𝜅 not only increases the elasticity estimate but also reduces the controllability update (using 𝛽<sub>control</sub>+(1- 𝜅) instead of 𝛽<sub>control</sub>+1) after failures with 3 tickets. However, implementing this coupling diminished the model's fit to the data, as compared to allowing both effects to occur independently, indicating that the increase in 3 ticket purchases upon failing with 3 tickets did not result from participants not accepting that controllability is in fact low. Thus, we maintain our original interpretation that failure with 3 tickets increases uncertainty about whether control is possible at all, leading participants who continue to opt in to invest maximum resources to resolve this uncertainty. We now report these results in the revised text (lines 662-674). 

      The trajectory over time is consistent this interpretation (new Supplementary Figure 2 shown below). Specifically, we see that under low controllability (0-50%, orange line), over the first 10 trials participants show higher persistence with 3 tickets after failing, despite experiencing frequent failures, but also a higher opt-out probability. As these participants accumulate evidence about their limited control, we observe a gradual decrease in 3-ticket selections that corresponds directly with a further increase in opting out (right panel, orange line). This pattern qualitatively corresponds with the behavior of our computational model (empty circles). We present the results of the new analysis in lines 180-190: 

      “In fact, failure with 3 tickets even made participants favor 3, over 1 and 2, tickets. This favoring  of 3 tickets continued until participants accumulated sufficient evidence about their limited control to opt out (Supplementary Figure 2). Presumably, the initial failures with 3 tickets resulted in an increased uncertainty about whether it is at all possible to control one’s destination. Consequently, participants who nevertheless opted in invested maximum resources to resolve this uncertainty before exploring whether control is elastic.”

      Regarding correlations between the perseveration parameter and psychopathology, we have now conducted a comprehensive exploratory analysis of all two-way relationships between parameters and psychopathology scores (new Supplementary Figure 3). Whereas we observed modest negative correlations with social anxiety (LSAS, r=-0.13), cyclothymic temperament (r=0.13), and alcohol use (AUDIT, r=-0.13), none reached statistical significance after FDR correction for multiple comparisons. 

      Regarding the modeling, I also wondered whether a better alternative model than the controllability model would be a simple associative learning model, where a number of tickets are mapped to outcomes, regardless of elasticity.

      We thank the Reviewer for suggesting this alternative model. Following this suggestion, we implemented a simple associative learning model that directly maps each option to its expected value, without a latent representation of elasticity or controllability. Unlike our controllability model which learns the probability of reaching the goal state for each ticket quantity, this associative learning model simply updates option values based on reward prediction errors.

      We found that this simple Q-learning model performed worse than even the controllability model at explaining participant data (log Bayes Factor  ≥1854 on the combined datasets), further supporting our hypothesis that participants are learning latent estimates of control rather than simply associating options with outcomes. We present the results of this analysis in lines 662664:

      We implemented a simple Q-learning model that directly maps ticket quantities to expected values based on reward prediction errors, without representing latent controllability. This associative model performed substantially worse than even our simple controllability model (log Bayes Factor ≥ 1854 on the combined datasets).

      Reviewer 3 (Recommendations for the authors):

      Please make all materials available, including code (analysis and experiment) and data. Please also provide a link to the task or a video of a few trials of the main task.

      We thank the reviewer for this important suggestion. All requested materials are now available at https://github.com/lsolomyak/human_inference_of_elastic_control. This includes all experiment code, analysis code, processed data, and a video showing multiple sample trials of the main task.

      References

      (1)  Huys, Q. J. M., & Dayan, P. (2009). A Bayesian formulation of behavioral control. Cognition, 113(3), 314– 328.

      (2)  Ligneul, R. (2021). Prediction or causation? Towards a redefinition of task controllability. Trends in Cognitive Sciences, 25(6), 431–433.

      (3)  Mistry, P., & Liljeholm, M. (2016). Instrumental divergence and the value of control. Scientific Reports, 6, 36295.

      (4)  Lin, J. (1991). Divergence measures based on the Shannon entropy. IEEE Transactions on Information Theory, 37(1), 145–151

      (5)  Cohen RM, Weingartner H, Smallberg SA, Pickar D, Murphy DL. Effort and cognition in depression. Arch Gen Psychiatry. 1982 May;39(5):593-7. doi: 10.1001/archpsyc.1982.04290050061012. PMID: 7092490.

      (6)  Bi R, Dong W, Zheng Z, Li S, Zhang D. Altered motivation of effortful decision-making for self and others in subthreshold depression. Depress Anxiety. 2022 Aug;39(8-9):633-645. doi: 10.1002/da.23267. Epub 2022 Jun 3. PMID: 35657301; PMCID: PMC9543190.

      (7)  Tapal, A., Oren, E., Dar, R., & Eitam, B. (2017). The Sense of Agency Scale: A measure of consciously perceived control over one's mind, body, and the immediate environment. Frontiers in Psychology, 8, 1552

    1. Author response:

      The following is the authors’ response to the original reviews

      Reviewer #1 (Public review):

      Summary:

      There has been intense controversy over the generality of Hamilton's inclusive fitness rule for how evolution works on social behaviors. All generally agree that relatedness can be a game changer, for example allowing for otherwise unselectable altruistic behaviors when 𝑐 < 𝑟𝑏, where 𝑐 is the fitness cost to the altruism, 𝑏 is the fitness benefit to another, and 𝑟 their relatedness. Many complications have been successfully incorporated into the theory, including different reproductive values and viscous population structures.

      I agree, especially if by incorporating viscous population structures, the reviewer means the discovery of the cancellation effect (Wilson, Pollock, and Dugatkin, 1992, Taylor, 1992).

      The controversy has centered on another dimension; Hamilton's original model was for additive fitness, but how does his result hold when fitnesses are non-additive? One approach has been not to worry about a general result but just find results for particular cases. A consistent finding is that the results depend on the frequency of the social allele - nonadditivity causes frequency dependence that was absent in Hamilton's approach.

      Just to be extra precise: Hamilton’s (1964) original model did not use the Price equation nor the regression approach to define costs and benefits, and it did indeed simply presuppose fixed, additive fitness effects.

      Also for extra precision on terminology: many researchers will describe all fitnesses in social evolution as frequency dependent. The reason they do, is that with or without additivity, both the fitness of cooperators (with the social allele) and the fitness of defectors (without the social alle) typically increase in the frequency of cooperators in the population; the more cooperators there are, the more individuals run into them, which increases average fitness. The result depending on the frequency I take to mean that which of those two fitnesses is larger flips at a certain frequency, which automatically implies that the difference between them is depending on the frequency of the social allele. This is indeed the result of non-additivity. We will return to this in more detail in the response to Reviewer #3. Also at the end of Appendix B I have added a bit to be extra precise regarding frequency dependence.

      Two other approaches derive from Queller via the Price equation. Queller 1 is to find forms like Hamilton's rule, but with additional terms that deal with non-additive interaction, each with an r-like population structure variable multiplied by a b-like fitness effect (Queller, 1985). Queller 2 redefines the fitness effects c and b as partial regressions of the actor's and recipient's genes on fitness. This leaves Hamilton's rule intact, just with new definitions of c and b that depend on frequency (Queller, 1992a).

      Queller 2 is the version that has been most adopted by the inclusive fitness community along with assertions that Hamilton's rule in completely general. In this paper, van Veelen argues that Queller 1 is the correct approach. He derives a general form that Queller only hinted at. He does so within a more rigorous framework that puts both Price's equation and Hamilton's rule on firmer statistical ground. Within that framework, the Queller 2 approach is seen to be a statistical misspecification - it employs a model without interaction in cases that actually do have interaction. If we accept that this is a fatal flaw, the original version of Hamilton's rule is limited to linear fitness models, which might not be common.

      I totally agree.

      Strengths:

      While the approach is not entirely new, this paper provides a more rigorous approach and a more general result. It shows that both Queller 1 and Queller 2 are identities and give accurate results, because both are derived from the Price equation, which is an identity. So why prefer Queller 1? It identifies the misspecification issue with the Queller 2 approach and points out its consequences. For example, it will not give the minimum squared differences between the model and data. It does not separate the behavioral effects of the individuals from the population state (𝑏 and 𝑐 become dependent on 𝑟 and the population frequency).

      Just to be precise on a detail: in the data domain, as long as the number of parameters in a statistical model is lower than the number of data points, adding parameters typically (generically) lowers the sum of squared errors. That is to say, for an underspecified statistical model, the sum of squared errors goes down if a parameter is added, but for an already overspecified statistical model, the same is still true (although, typically, by how much the sum of squared errors is reduced will differ). The model specification task for a statistician includes knowing when to keep adding parameters, because the data suggest that the model is still underspecified, and when to stop adding parameters, because the model is well-specified, even if adding parameters still reduces the sum of squared errors.

      In a modeling context, on the other hand, one can say that sum of squared differences will stop decreasing at the point where the statistical model is well-specified, that is: when it matches the model we are considering.

      The paper also shows how the same problems can apply to non-social traits. Epistasis is the non-additivity of effects of two genes within the individual. (So one wonders why have we not had a similarly fierce controversy over how we should treat epistasis?)

      The paper is clearly written. Though somewhat repetitive, particularly in the long supplement, most of that repetition has the purpose of underscoring how the same points apply equally to a variety of different models.

      Finally, this may be a big step towards reconciliation in the inclusive fitness wars. Van Veelen has been one of the harshest critics of inclusive fitness, and now he is proposing a version of it.

      I am very happy to hear this, because I am indeed hopeful for reconciliation. I would like to add a comment, though. The debate on Hamilton’s rule/inclusive fitness is regularly thought of as a battle between two partizan camps, where both sides care at least as much about winning as they do about getting things right. This is totally understandable, because to some degree that is true. Also, I agree that it is fair to position me in the camp that is critical of the inclusive fitness literature. However, I would like to think that I have not been taking random shots at Hamilton’s rule. I have pointed to problems with the typical use of the Price equation and Hamilton’s rule, and I think I did for very good reasons. I am obviously very happy that finding the Generalized Price equation, and the general version of Hamilton’s rule, allowed me to go beyond this, and (finally) offer a correct alternative, and I totally appreciate that this opens the door for reconciliation, as this reviewer points out. But I would not describe this as a road-toDamascus moment. In order to illustrate the continuity in my work, I would like to point to three papers.

      In van Veelen (2007), I pointed to the missing link between the central result in Hamilton’s (1964) famous paper (which states that selection dynamics take the population to a state where mean inclusive fitness is maximized), and Hamilton’s actual rule (which states that selection will lead to individuals maximizing their individual inclusive fitness). My repair stated the additional assumptions that were necessary to make the latter follow from the former. I would say that this can hardly be characterized as an attack on Hamilton’s rule. Reading Hamilton (1964) with enough care to notice something is missing, and then repairing it, I think is a sign of respect, and not an attack.

      Van Veelen (2011) is about the replicator dynamics for n-player games, with the possibility of assortment. This puts the paper in a domain that does not assume weak selection, and that is typically not much oriented towards inclusive fitness. I included a theorem that implies that, under the condition of linearity, inclusive fitness not only gets the direction of selection right, but 𝑟𝑏 − 𝑐 becomes a parameter that also determines the speed of selection. This I think is representative, in the sense that in many of my papers, I carefully stake out when the classic version of Hamilton’s rule does work.

      In Akdeniz and van Veelen (2020), we moreover take a totally standard inclusive fitness approach in a model of the cancellation effect at the group level.

      I would say that this does not line up with the image of a harsh critic that takes random shots at Hamilton’s rule or inclusive fitness.

      Weaknesses:

      van Veelen argues that the field essentially abandoned the Queller 1 approach after its publication. I think this is putting it too strongly - there have been a number of theoretical studies that incorporate extra terms with higher-order relatednesses. It is probably accurate to say that there has been relative neglect. But perhaps this is partly due to a perception that this approach is difficult to apply.

      I can imagine that the perceived difficulty in application may have played a role in the neglect of the Queller 1 approach. What for sure has played a role, and I would think a much bigger one, is that the literature has been pretty outspoken that the Queller 1 approach is the wrong way to go. The main text cites a number of papers that hold this position very emphatically (The first one of those was a News and Views by Alan Grafen (1985) that accompanied the paper in which Queller presented his Queller 1 approach. I am very happy that Appendix B shows on how many levels this News and Views was wrong.). There is only a handful of papers that follow the Queller 1 example.

      The model in this paper is quite elegant and helps clarify conceptual issues, but I wonder how practical it will turn out to be. In terms of modeling complicated cases, I suspect most practitioners will continue doing what they have been doing, for example using population genetics or adaptive dynamics, without worrying about neatly separating out a series of terms multiplying fitness coefficients and population structure coefficients.

      I am not sure if I see what the reviewer envisions practitioners that use population genetics will keep on doing. I would think that the Generalized Price equation in regression form is a description of population genetic dynamics, and therefore, if practitioners will not make an effort to “neatly separate out a series of terms multiplying fitness coefficients and population structure coefficients”, then all I can say is that they should. I cannot do more than explain why, if they do not, they are at risk of mischaracterizing what gets selected and why.

      Regarding those that use adaptive dynamics, I would say that this is a whole different approach. Within this approach, one can also apply inclusive fitness; see Section 6 and Appendix D of van Veelen et al. (2017). Appendix D is full of deep technical results and was done by Benjamin Allen.

      For empirical studies, it is going to be hard to even try to estimate all those additional parameters. In reality, even the standard Hamilton's rule is rarely tested by trying to estimate all its parameters. Instead, it is commonly tested more indirectly, for example by comparative tests of the importance of relatedness. That of course would not distinguish between additive and non-additive models that both depend on relatedness, but it does test the core idea of kin selection. It will be interesting to see if van Veelen's approach stimulates new ways of exploring the real world.

      Regarding the impact on empirical studies, there are a few things that I would like to say. The first is that I would just like to repeat, maybe a bit more elaborately, what I wrote at the end of the main text. Given that the generalized version of Hamilton’s rule produces a host of Hamilton-like rules, and given the fact that all of them by construction indicate the direction of selection accurately, the question whether or not Hamilton’s rule holds turns out to be illposed. That means that we can stop doing empirical tests of Hamilton’s rule, which are predicated on the idea that Hamilton’s rule, with benefits and costs being determined by the regression method, could be violated – which it cannot (Side note: it is possible to violate Hamilton’s rule, if costs and benefits are defined according to the counterfactual method; see van Veelen et al. (2017) and van Veelen (2018). This way of defining costs and benefits is less common, although there are authors that find this definition natural enough to assume that this is the way in which everybody defines costs and benefits (Karlin and Matessi, 1983, Matessi and Karlin, 1984).). Instead, we should do empirical studies to find out which version of Hamilton’s rule applies to which behaviour in which species.

      would like to not understate what a step forward this is. The size of the step forwards is of course also due to the dismal point of departure. As theorists, we have failed our empiricists, because all 12 studies included in the review by Bourke (2014) of papers that explicitly test Hamilton’s rule are based on the misguided idea that the traditional Hamilton’s rule, with costs and benefits defined according to the regression method, can be violated. While the field does sometimes have disdain for mathematical nit-picking, this is a point where a little more attention to detail would have really helped. If the hypothesis is that Hamilton’s rule holds, and the null is that it does not, then trying to specify how the empirical quantity that reflects inclusive fitness would be distributed under the null hypothesis (in order to do the right statistical tests) would have forced researchers to do something with the information that this quantity is not distributed at all, because Hamilton’s rule is general (in the sense that it holds for any way in which the world works). If one would prefer to reverse the null and the alternative hypothesis, one would run into similar problems. Understanding that the question is ill-posed therefore is a big step forwards from the terrible state of statistics and the waste of research time, attention and money on the empirical side of this field (see also Section 8 of van Veelen et al., 2017).

      I would agree that doing comparative statics may not be much affected by this. Section 5 of van Veelen et al. (2017) indicates that there can be a large set of circumstances under which the general idea “relatedness up → cooperation up” still applies. But that may be a bit unambitious, and Section 8 of van Veelen et al. (2017), and the final section of van Veelen (2018) contain some reflections on empirical testing that may allow us to go beyond that. As long as there is change happening in the Generalized Price equation, the population is not in equilibrium. For empirical tests, one can either aim to capture selection as it happens, or assume that what we observe reflects properties of an equilibrium. This leads to interesting reflections on how to do empirics, which may differ between traits that are continuous and traits that are discrete (again: see van Veelen et al. (2017), and van Veelen (2018).

      Reviewer #2 (Public review):

      Summary:

      This manuscript reconsiders the "general form" of Hamilton's rule, in which "benefit" and "cost" are defined as regression coefficients. It points out that there is no reason to insist on Hamilton's rule of the form −𝑐 + 𝑏𝑟 > 0, and that, in fact, arbitrarily many terms (i.e. higherorder regression coefficients) can be added to Hamilton's rule to reflect nonlinear interactions. Furthermore, it argues that insisting on a rule of the form −𝑐 + 𝑏𝑟 > 0 can result in conditions that are true but meaningless and that statistical considerations should be employed to determine which form of Hamilton's rule is meaningful for a given dataset or model.

      Totally right. I cannot help to want to be extra precise, though, by distinguishing between the data domain and the modelling domain. In the data domain, statistical considerations apply in order to avoid misspecification. In this domain, avoiding misspecification can be complicated, because we do not know the underlying data generating process, and we depend on noisy data to make a best guess. In the modeling domain, however, there is no excuse for misspecification, as the model is postulated by the modeler. I therefore would think that in this domain, it does not really require “statistical considerations” to minimize the probability of misspecification; we can get the probability of misspecification all the way down to 0 by just choosing not to do it.

      Strengths:

      The point is an important one. While it is not entirely novel-the idea of adding extra terms to Hamilton's rule has arisen sporadically (Queller, 1985, 2011; Fletcher et al., 2006; van Veelen et al., 2017)--it is very useful to have a systematic treatment of this point. I think the manuscript can make an important contribution by helping to clarify a number of debates in the literature. I particularly appreciate the heterozygote advantage example in the SI.

      Me too, and I really hope the readers make it this far! I have thought of putting it in the main text, but did not know where that would fit.

      Weaknesses:

      Although the mathematical analysis is rigorously done and I largely agree with the conclusions, I feel there are some issues regarding terminology, some regarding the state of the field, and the practice of statistics that need to be clarified if the manuscript is truly to resolve the outstanding issues of the field. Otherwise, I worry that it will in some ways add to the confusion.

      (1) The "generalized" Price equation: I agree that the equations labeled (PE.C) and (GPE.C) are different in a subtle yet meaningful way. But I do not see any way in which (GPE.C) is more general than (PE.C). That is, I cannot envision any circumstance in which (GPE.C) applies but (PE.C) does not. A term other than "generalized" should be used.

      This is a great point! Just to make sure that those that read the reports online understand this point, let me add some detail. The equation labeled (PE.C) – which is short for Price equation in covariance form – is

      The derivation in Appendix A then assumes that we have a statistical model that includes a constant and a linear term for the p-score. It then defines the model-estimated fitness of individual 𝑖 as , where 𝑤<sub> 𝑖</sub> is the realized number of offspring of individual 𝑖, and 𝜀<sub> 𝑖</sub> is the error term – and it is the sum over all individuals of this error term-squared that is minimized. The vector of model-estimated fitnesses will typically be different for different choices of the statistical model. Appendix A then goes on to show that, whatever the statistical model is that is used, for all of them , as long as the statistical model includes a constant and a linear term for the p-score. That means that we can rewrite (PE.C) as

      The point that the reviewer is making, is that this is not really a generalization. For a given dataset (or, more generally, for a given population transition, whether empirical or in a model), is just a number, and it happens to be the case that 𝐶𝑜𝑣(𝑤:, 𝑝) returns the same number, whatever statistical model we use for determining what the model-estimated fitnesses 𝑤<sub> 𝑖</sub> are (as long as the statistical model includes a constant and a linear term for the p-score). In other words, (PE.C) is not really nested in (GPE.C), so (GPE.C) is not a proper generalization of (PE.C).

      This is a totally correct point, and I had actually struggled a bit with the question what terminology to use here. Equation (GPE.C) is definitely general, in the sense that we can change the statistical model, and thereby change the vector of model-estimated fitnesses , but as long as we keep the constant and the linear term in the statistical model, the equation still applies. But it is not a generalization of (PE.C).

      I do however have a hard time coming up with a better label. The General Price equation may be a bit better, but it still suggests generalization. The Statistical Model-based Price equation does not suggest or imply generalization, but it does not convey how general it is, and it suggests that it could be an alternative to the normal Price equation that one may or may not choose to use – while this version really is the one we should use. It may moreover create the impression that this is only for doing statistics, and one might use the traditional Price equation for anything that is not statistics. I cannot really think of other good alternatives, but I am of course open to suggestions.

      So, by lack of a better label, I called this the Generalized Price equation in covariance form. Though clearly imperfect, there are still a few good things about this label. The first is that, as mentioned above, this equation is general, in the sense that it holds, regardless of the statistical model. The second reason is that this is Step 1 in a sequence of three steps., the other two of which do produce proper generalizations. Step 2 goes from this equation in covariance form to the Generalized Price Equation in regression form, which is a proper generalization of the traditional Price equation in regression form. Step 3 goes from the Generalized Price Equation in regression form to the general version of Hamilton’s rule, which is also a proper generalization of the classical Hamilton’s rule. Since I would suggest that Step 1 on its own is kind of useless, and therefore Step 1 and Step 2 will typically come as a package, I would be tempted to think that this justifies the abuse of terminology for the Price Equation in covariance form. I did however add the observation made by the reviewer at the point where the Generalized Price equation (in both forms) is derived, so I hope this at least partly addresses this concern.

      (2) Regression vs covariance forms of the Price equation: I think the author uses "generalized" in reference to what Price called the "regression form" of his equation. But to almost everyone in the field, the "Price Equation" refers to the covariance form. For this reason, it is very confusing when the manuscript refers to the regression form as simply "the Price Equation".

      As an example, in the box on p. 15, the manuscript states "The Price equation can be generalized, in the sense that one can write a variety of Price-like equations for a variety of possible true models, that may have generated the data." But it is not the Price equation (covariance form) that is being generalized here. It is only the regression that Price used that is being generalized.

      To be consistent with the field, I suggest the term "Price Equation" be used only to refer to the covariance form unless it is otherwise specified as in "regression form of the Price equation".

      I am not sure about the level of confusion induced here, but I totally see that it can be helpful to avoid all ambiguity. I therefore went over everything, and whenever I wrote “Price equation”, I tried to make sure it comes either with “in covariance form” or with “in regression form”. At some places, it is a bit over the top to keep repeating “in regression form”, when it is abundantly clear which form is being discussed. Also, I added no qualifiers if a statement is true for both forms of the Price equation, or if the claim refers to the whole package of going through Step 1 and Step 2 mentioned above.

      (3) Sample covariance: The author refers to the covariance in the Price equation as “sample covariance”. This is not correct, since sample covariance has a denominator of N-1 rather than N (Bessel’s correction). The correct term, when summing over an entire population, is “population covariance”. Price (1972) was clear about this: “In this paper we will be concerned with population functions and make no use of sample functions”. This point is elaborated on by Frank (2012), in the subsection “Interpretation of Covariance”.

      I totally agree. On page 418 of van Veelen (2005), I wrote:

      “Another possibility is that we think of 𝑧<sub>i</sub> and 𝑞<sub>i</sub>, 𝑖 = 1,…,𝑁 as realizations of a jointly distributed random variable. […] In that case the expression between square brackets is a good approximation for what statisticians […] call a sample covariance. A sample covariance is defined as but in large samples it is OK to replace 𝑁 − 1 by 𝑁, and then this formula reduces to Price’s 𝐶𝑜𝑣(𝑧, 𝑞).”

      In van Veelen et al. (2012), I slid a little, because in Box 1 on page 66, I wrote that is the sample covariance, and only in footnote 1 on the same page did I include Bessel’s correction, when I wrote:

      “To be perfectly precise, the sample covariance is defined as

      In this manuscript, I slid a little further, and left Bessel’s correction out altogether. I am happy that the reviewer pointed this out, so I can make this maximally precise again.

      The reviewer also quotes Price (1972), page 485:

      “In this paper we will be concerned with population functions and make no use of sample functions”.

      Below, the reviewer will return to the issue of distinguishing between the sample covariance with Bessel’s correction, and the sample covariance without Bessel’s correction, where the latter is regularly also referred to as the population covariance. A natural interpretation of the quote from Price (1972), if we read a bit around this quote in the paper, is that the difference between his “population functions” and his “sample functions” is indeed Bessel’s correction.

      The reviewer also states that Frank (2012) elaborates on this in the subsection “Interpretation of Covariance”. What is interesting, though, is that, when Frank (2012) writes, on page 1017 “It is important to distinguish between population measures and sample measures”, the difference between those is not that one does, and the other does not include Bessel’s correction. The difference between “population measures” and “sample measures” in Frank (2012), page 1017

      “It is important to distinguish between population measures and sample measures”,

      the difference between those is not that one does, and the other does not include Bessel’s correction. The difference between “population measures” and “sample measures” in Frank (2012), page 1017, is that

      “In many statistical applications, one only has data on a subset of the full population, that subset forming a sample.”

      The distinction between a population covariance and a sample covariance in Frank (2012) therefore is that they are “covariances” of different things (where the word covariances is in quotation marks, because, again, they are not really covariances). Besides just making sure that Price (1972) and Frank (2012) are not using these terms in the same way, this also perfectly illustrates the mix-up between statistical populations (or data generating processes) and biological populations that I discuss on pages 8 and 9 of Appendix A. I will return to this below, when I explain why I want to avoid using the word “population covariance” for the sample covariance without Bessel’s correction.

      Of course, the difference is negligible when the population is large. However, the author applies the covariance formula to populations as small as 𝑁 = 2, for which the correction factor is significant.

      Absolutely right.

      The author objects to using the term "population covariance" (SI, pp. 8-9) on the grounds that it might be misleading if the covariance, regression coefficients, etc. are used for inference because in this case, what is being inferred is not a population statistic but an underlying relationship. However, I am not convinced that statistical inference is or should be the primary use of the Price equation (see next point). At any rate, avoiding potential confusion is not a sufficient reason to use incorrect terminology.

      There are a few related, but separate issues. One is what to call the 𝐶𝑜𝑣(𝑤, 𝑝)-term. Another, somewhat broader, is to avoid mixing up statistical populations and biological populations. A third is what the primary use of the Price equation is. The third issue I will respond to below, where it reappears. Here I will focus on the first two, which can be discussed without addressing the third.

      In a data context, I now call the 𝐶𝑜𝑣(𝑤, 𝑝)-term “’" times the sample covariance, or, in other words, the sample covariance without Bessel’s correction”. This should be unambiguous. In a modeling context I refer to 𝐶𝑜𝑣(𝑤, 𝑝)-term as “the 𝐶𝑜𝑣(𝑤, 𝑝)-term” and describe it as a summary statistic or a notational convention. There are two reasons for this choice.

      The first is that neither of these use the word “population”. I like this, because there is a persistent scope for confusion between statistical populations and biological populations (as exemplified by Frank, 2012). This leads to an incorrect, but widespread intuition that if we “know the entire (biological) population” in a data context, there is nothing that can be estimated. This is what pages 8 and 9 of Appendix A are all about.

      The second reason is that by using two labels, I also differentiate between the data context and the modeling context. This is important for reasons I will return to later.

      Relatedly, I suggest avoiding using 𝐸 for the second term in the Price equation, since (as the ms points out), it is not the expectation of any random variable. It is a population mean. There is no reason not to use something like Avg or bar notation to indicate population mean. Price (1972) uses "ave" for average.

      I totally agree that the second term in the Price equation is not an expectation. I made this point in van Veelen (2005), and I repeated this in the manuscript. This remark by the reviewer prompted me to spell this out a bit more emphatically in Appendix A. That still leaves me with the choice what notation to use.

      I therefore looked up all contributions to the Theme issue “Fifty years of the Price equation” in the Philosophical Transactions of the Royal Society B, and found that almost all contributions use 𝐸, sometimes saying that this refers to an expectation or an average. Of course, this is wrong. However (and this is another argument), it is equally wrong as using 𝐶𝑜𝑣 or 𝑉𝑎𝑟. The terms abbreviated as 𝐶𝑜𝑣 and 𝑉𝑎𝑟 are equally much not a covariance and a variance as the term abbreviated as 𝐸 is not an expectation. So I would think that there are a few reasons for sticking with 𝐸 here; 1) consistency with the literature; 2) consistency with the treatment of other terms; and 3) the fact that this term is not really of any importance in this manuscript. I do however totally understand the reviewer’s reasons, which I suppose include that for using 𝐸, there are relatively unproblematic alternatives (ave or upper bar) that are not available for the other terms. I hope therefore that being a bit more emphatic in the manuscript about 𝐸 not being an expectation at least partly addresses this concern.

      I should add, however, that the distinction between population statistics vs sample statistics goes away for regression coefficients (e.g. b, c, and r in Hamilton's rule) since in this case, Bessel's correction cancels out.

      Totally correct.

      (4) Descriptive vs. inferential statistics: When discussing the statistical quantities in the Price Equation, the author appears to treat them all as inferential statistics. That is, he takes the position that the population data are all generated by some probabilistic model and that the goal of computing the statistical quantities in the Price Equation is to correctly infer this model.

      Before I respond to this, I would like to point out that this literature has started going off the rails right from the very beginning. One of the initial construction errors was to use the ungeneralized Price equation in regression form. The other one is that the paper in which Price (1970) presented his equation is inconsistent, and suggests that the equation can be used for constructing hypotheses and for testing them at the same time (see van Veelen (2005), page 416). That, of course, is not possible; the first happens in the theory/modeling domain, and the second in the empirical testing/statistics domain, and they are separate exercises.

      These construction errors have warped the literature based on it, and have resulted in a lot of mental gymnastics and esoteric statements, which are needed if we are not willing to consider the possibility that there could be anything amiss with the original paper by Price (1970).

      In this paper, I undo both of these construction errors. Undoing the second one means exploring both domains separately. In Sections 2-4 of Appendix A I explore the possibility that the Price equation is applied to data. In Section 5 of Appendix A I explore the possibility that it is used in a modelling context. The primary effort here is just to do it right, and I have not read anything to suggest that I did not succeed in doing this. Secondarily, of course, I also want to contrast this to what happens in the existing literature. That is what this point by the reviewer is about. It is therefore important to be aware that seeing the contrast accurately is complicated by the apologetic warp in the existing literature.

      As a first effort to unwarp, I would like to point to the fact that I am not taking any position on what the Price equation should be used for. All I do here is explore (and find) possibilities, both in the statistical inference domain and in the modeling domain. I also find that there is scope for misspecification in both, and that, in both domains, we should want to avoid misspecification. The thing that I criticize in the existing literature therefore is not the choice of domain. The thing that I criticize is the insistence on, and celebrating of what is most accurately described as misspecification. This typically happens in the modeling domain.

      It is worth pointing out that those who argue in favor of the Price Equation do not see it this way: "it is a mistake to assume that it must be the evolutionary theorist, writing out covariances, who is performing the equivalent of a statistical analysis." (Gardner, West, and Wild, 2011); "Neither data nor inferences are considered here" (Rousset, 2015). From what I can tell, to the supporters of the Price equation and the regression form of Hamilton's rule, the statistical quantities involved are either population-level *descriptive* statistics (in an empirical context), or else are statistics of random variables (in a stochastic modeling context).

      Again, this description of the friction between my paper and the existing literature is predicated on the suggestion that I have only one domain in mind where the Price equation can be applied. That is not the case; I consider both.

      In the previous paragraph, the reviewer states that I “treat statistical quantities as inferential statistics”, and in this paragraph the reviewer contrasts that with the supporters of the (ungeneralized) Price equation that supposedly treat the same quantities as “descriptive statistics”. This is also beside the point, but it will take some effort to sort out the spaghetti of entangled arguments (where the spaghetti is the result of the history in this field, as indicated earlier).

      First of all, it is not unimportant to point out that the way most people use the terms “inferential statistics” and “descriptive statistics” is that the first refers to an activity, and the second to a function of a bunch of numbers, typically data. Inferential statistics is a combination of parameter estimation and model specification (those are activities). Descriptive statistics are for instance the average values of variables of interest (which makes them a function of a set of numbers). When doing inferential statistics (or statistical inference), looking at the descriptive statistics of the dataset is just a routine before the real work begins. It is important to remember that.

      Now I suppose that this reviewer uses these words a little differently. When he or she writes that I “treat statistical quantities as inferential statistics”, I assume that the reviewer means that I want to use a term like for doing statistical inference, or that, when I want to interpret such a term, I include considerations typical of statistical inference. Within the data domain, that is totally correct. In the paper I argue that there are very good reasons for this. We would like to know what the data can tell us about the actual fitness function, and if we do our statistical inference right, and choose our Price-like equation accordingly, then that means that we would be able to give a meaningful interpretation to a term like . It also means that we then have an equation that describes the genetic population dynamics accurately.

      When the reviewer states that other papers treat them as “population level descriptive statistics” in an empirical context, I have a hard time coming up with papers for which that is the case. Most papers apply the Price equation in the modeling domain (That is to say: this is true in evolution. In ecology the Price equation is often applied to data; see Pillai and Gouhier (2019) and Bourrat et al. (2023)). But even if there are researchers that apply the Price equation to data, then considering these statistical quantities as “descriptive statistics” would not make sense. Looking at the descriptive statistics alone is not an empirical exercise; it is just a routine that happens before the actual statistical inference starts. In a data context, saying that considerations that are standard in statistical inference do not apply, because one is just not doing statistical inference, is the equivalent of an admission of guilt. If you do not consider statistical significance, and never mention that sample size could matter, because you are using these terms as “descriptive statistics, not inferential statistics”, then you’re basically admitting to not doing a serious empirical study.

      Besides treating statistical quantities as descriptive statistics in a data context, the reviewer also states that, in a stochastic modeling context, other researchers treat the same statistical quantities as “statistics of random variables”. This is first of all very generous to the existing literature. I imagine that the reviewer is imagining a modeling exercise where for instance the covariance between two variables is postulated. A theory exercise would then take that as a starting point for the derivation of some theoretical result. This, however, is not what happens in most of the literature.

      There are two things that I would like to point out. First of all, postulating covariances and deriving results from assumptions regarding those covariances is not an activity that requires using the Price equation. There are many stochastic models that function perfectly fine without the Price equation. This is maybe a detail, but it is important to realize that what the reviewer probably thinks of as a legitimate theoretical exercise may be something that can very well be done without the Price equation.

      Secondly, I would like to repeat something that I have pointed out before, which is that the Price equation can be written for any transition, whether this transition is likely or unlikely, given a model, and even for transitions that are impossible. For all of those transitions, one can write the (ungeneralized) Price equation, and for all of those, the Price equation will be an identity, and it will contain the things that the reviewer refers to as “statistical quantities”. It is important to realize that these “statistical quantities”, therefore, are properties of a transition, and that every transition comes with its own ”statistical quantity”. That implies that they are not properties of random variables; they reflect something regarding one transition. What one could imagine, though, is the following. To fix ideas, let’s take the Price equation in regression form, and focus on . A meaningful modeling exercise starts with assumptions about the likelihood of all different transitions, and therefore the likelihood of different values of 𝛽 materializing – or it starts with assumptions that imply those probabilities. In a theoretical exercise, one could then derive statements about the expectation and variance of those “statistical quantities”. For instance, one can calculate the expected value 𝐸[𝛽] =𝐸, and the variance 𝑉𝑎𝑟[𝛽] = 𝑉𝑎𝑟 , where this expectation is a proper expectation (taken over the probabilities with which these transitions materialize) and this variance is a proper variance, for the same reason.

      This is what I do on page 416 of van Veelen (2005) and in Section 5 of Appendix A. I think something like this is what the reviewer may have in mind, but it is worth pointing out that this still does not mean that the from the Price equation for any given transition is now a property of a random variable. Much of the literature, however, is not at the level of sophistication that I imagine the reviewer has in mind – although there are papers that are; see the discussion below of Rousset and Billiard (2000) and Van Cleve (2015).

      In the appendix to this reply, I will address the quotes from Gardner, West, and Wild (2011) and Rousset (2015). This takes up some space, so that is why it is at the end of this reply.

      In short, the manuscript seems to argue that Price equation users are performing statistical inference incorrectly, whereas the users insist that they are not doing statistical inference at all.

      That is not what the manuscript argues, but I am happy to clarify. The manuscript explores both the use of the Price equation when applied to data (and therefore for statistical inference) and when applied to transitions in a model. The criticism on the existing literature is not that it performs statistical inference incorrectly. The criticism is that the literature insists on misspecification, which typically happens in a modelling context.

      The problem (and here I think the author would agree with me) arises when users of the Price equation go on to make predictive or causal claims that would require the kind of statistical analysis they claim not to be doing. Claims of the form "Hamilton's rule predicts.." or use of terms like "benefit" and "cost" suggest that one has inferred a predictive or causal relationship in the given data, while somehow bypassing the entire theory of statistical inference.

      I do not really know how to interpret this paragraph. The use of the word “data” suggests that this pertains to a data context, but I do not know what would qualify as a “predictive claim” in that domain, or how any study would go from data to a claim of the form “Hamilton’s rule predicts …”. Again, I do not really know papers that apply the Price equation to data. None of the empirical papers reviewed in Bourke (2014) for instance do. I would however agree that it is close to obvious that an approach that does indeed bypass the entire theory of statistical inference cannot identify causal relations in datasets. I think the examples in Section 2 of Appendix A also clearly illustrate that a literature in which the word “sample size” is absent, cannot be doing statistical inference.

      There is also a third way to use the Price equation which is entirely unobjectionable: as a way to express the relationship between individual-level fitness and population-level gene frequency change in a form that is convenient for further algebraic manipulation. I suspect that this is actually the most common use of the Price equation in practice.

      I am not sure if I understand what it means for the Price equation to “express the relationship between individual-level fitness and population-level gene frequency change”. That is a bit reminiscent of how John Maynard Smith saw the Price equation (Okasha, 2005), but he also emphasized that he was unable to follow George Price and his equation. For sure, it cannot be that one side of the Price equation reflects something at the individual level and the other something at the population level, because both sides of the Price equation are equally aggregated over the population. Just to be safe, and to avoid unwarranted associative thinking, I would therefore choose to be minimalistic, and say that the Price equation is an identity for a transition between a parent population and an offspring population.

      Regardless of the words we choose, however, the question how harmless or objectionable the use of the Price equation is in the literature is absolutely relevant. In earlier papers I have tried to cover a spectrum of examples of different ways to use (or misuse) the Price equation. In van Veelen (2005) I cover Grafen (1985a), Taylor (1989), Price (1972), and Sober and Wilson (2007). The main paper that is discussed in van Veelen et al. (2012) is Queller (1992b), but Section 7 of that paper also discusses the way the Price equation is used in Rousset and Billiard (2000), Taylor (1989), Queller (1985), and Page and Nowak (2002). These discussions also come with a description of how much it takes to repair them, and this varies all the way from nothing, or a bit of minor rewording, to being beyond repair.

      What is good to observe, is that the papers in which the use of the Price equation is the least problematic, are also the papers in which, if the reference to the Price equation would be taken out, nothing really changes. These are papers that start with a model, or a collection of models, and that, at some point in the derivation of their results, point to a step that can, but does not have to be described as using the Price equation. An example of this is Rousset and Billiard (2000); see the detailed description in Section 7 of van Veelen et al. (2012).

      I am happy to point to a few more papers on the no harm, no foul end of the spectrum here.

      Allen and Tarnita (2012) discuss properties of the dynamics in a well-defined set of models.

      Towards the end of the paper, a version of the Price equation more or less naturally appears. This is more of an interesting aside, though, and does not really play a role in derivation of the core results of the paper. Van Cleve (2015) is similar to Rousset and Billiard (2000), in that the “application of the Price equation” there is a minor ingredient of the derivation of the results. (A detail that this reviewer may find worth mentioning, given earlier comments, is that Van Cleve (2015) writes the left-hand side of the Price equation as 𝐸(𝑤Δ𝑝|𝐩), instead of . First two very unimportant things. Van Cleve (2015) uses 𝑤 for mean fitness, for which is a more common symbol. Another detail of lesser importance is that it includes the vector of parent p-scores in the notation, which in their notation is 𝐩. More importantly, however, is that Van Cleve (2015) writes 𝐸(Δ𝑝) for , which extends the (mis)use of the symbol 𝐸 for what really is just an average. This is consistent within the Price equation, in the sense that it now denotes the average with 𝐸, both on the right-hand side and on the left-hand side of the Price equation. It can however be a little bit confusing, because when Rousset and Billiard (2000) write , then this is a proper expectation. In their case, this summarizes all possible transitions out of a given state, and weighs them by their probabilities of happening, given a state summarized by 𝑝.). I am also happy to extend the spectrum a bit here. Some papers on inclusive fitness do not use the Price equation at all, even though one could imagine places where it could be inserted. A nice example of such a paper is Taylor et al. (2007).

      In this paper, I hope I can be excused from taking a complete inventory of this literature, and I hope that I do not have to count how many papers fall into the different categories. This would help assess the veracity of the suspicion the reviewer has, which is that the most common use of the Price equation is entirely unobjectionable, but I just do not have the time. I would however not want to underestimate the aggregate damage done in this field. The spectrum spanned in my earlier papers does include a fair amount of nonsense results. This typically happens in papers that do not study a specific model or set of models, but that take the Price equation as their point of departure for their theorizing. Also there seems to be a positive correlation between how exalted and venerating the language is that is used when describing the wonders and depths of the Price equation, and how little sense the claims make that are “derived” with it.

      We also should not set the bar too low. This is a literature that, at the starting point, has a few construction errors in it, as described in the paper. That is reason for concern. Moreover, one of the main end products of this literature is what we send our empiricists to the field with. As Section 8 of van Veelen et al. (2017) indicates, what we have supplied to our empiricists to work with is nothing short of terrible. I would therefore want to maintain that the damage done is enormous, and if there are also a few papers around that may use the ungeneralized Price equation in an innocuous way, then that is not enough redemption for my taste. We are still facing a literature in which, at every instance where the Price equation is used, we still need to check in which category it falls.

      For a paper that aims to clarify these thorny concepts in the literature, I think it is worth pointing out these different interpretations of statistical quantities in the Price equation (descriptive statistics vs inferential statistics vs algebraic manipulation). One can then critique the conclusions that are inappropriately drawn from the Price equation, which would require rigorous statistical inference to draw. Without these clarifications, supporters of the Price equation will again argue that this manuscript has misunderstood the purpose of the equation and that they never claimed to do inference in the first place.

      I would like to return to the point that I made at the beginning of my response to point (4), which is that the “thorniness” of these concepts is the result of the warp in the literature, resulting from the construction errors in Price (1970). If people want to understand how to apply the Price equation right, I think that reading Appendix A and B would work just fine. Again, I have not read anything that suggests that there is anything incorrect in there, so if the literature contains “thorny” concepts, it might just be that this is the result of the mental gymnastics necessitated by the unwillingness to accept that there might be something not completely right with Price (1970). Moreover, given my experiences in the field, I am not sure that there is anything that I could say that would convince the supporters of the ungeneralized Price equation.

      (5) "True" models: Even if one accepts that the statistical quantities in the Price equation are inferential in nature, the author appears to go a step further by asserting that, even in empirical populations, there is a specific "true" model which it is our goal to infer. This assumption manifests at many points in the SI when the author refers to the "true model" or "true, underlying population structure" in the context of an empirical population.

      Again, in Appendix A I explore both a data context and a modeling context. In the modeling context none of this applies, because in such a context, there is only the model that we postulate. In the part in which I explore what the Price equation can do in a data context, I do indeed use words like “true model” or "true underlying population structure".  

      I do not think it is necessary or appropriate, in empirical contexts, to posit the existence of a Platonic "true" model that is generating the data. Real populations are not governed by mathematical models. Moreover, the goal of statistical inference is not to determine the "true model" for given data but to say whether a given statistical model is justified based on this data. Fitting a linear model, for example, does not rule out the possibility there may be higher-order interactions - it just means we do not have a statistical basis to infer these higher-order interactions from the data (say, because their p-scores are insignificant), and so we leave them out.

      This remark suggests that the statistical approach in Sections 2-4 of Appendix A is more naïve than it should be, and that I would overlook the possibility of, for instance, interaction effects that are really nonzero, but that are statistically not significant. Now first of all, at a superficial level, I would like to say that this strikes me as somewhat inconsistent. In the remarks further back, the reviewer seems to excuse those that use the Price equation on data without any statistical considerations whatsoever. The reason why the reviewer is giving them a pass, is that they are “just not doing statistical inference”. Instead, they are doing this whole other thing with, you know, descriptive statistics. As I indicated above, that is just a fancy way of saying that they are not doing serious statistics – or serious empirics, for that matter.

      In this comment, on the other hand, the reviewer also suggests that the statistics that I use to replace the total absence of any statistical considerations with, is not quite up to snuff. Below, I will indicate why that is not the case at all, but I think it is also worth registering a touch of irony there.

      In order to address this issue, it is worth first observing that the whole of classical statistics is based on probability theory in the following sense. We are always asking ourselves the question: if the data generating process works like this, what would the likelihood be of certain outcomes (datasets); and if the data generating process works some other way (sometimes: the complement of whatever “this” is), what would the likelihood then be of the same outcomes. By comparing those, we draw inferences about the underlying data generating process (which is a word suggestive of a “Platonic” world view that the reviewer seems to reject). Therefore, if one would impose a ban on using Platonic words like “true data generating process”; “actual fitness function”; or “the population structure that is out there”, it would be impossible to teach any course in statistics, basic or advanced. Also it would be impossible to practice, and talk about, applied statistics.

      Now the reviewer claims that “Real populations are not governed by mathematical models”. I do not really know if I agree or disagree with that statement, but the example that the reviewer gives does not fit that claim. The reviewer suggests that if we find a higher order term not to be statistically significant (and therefore we reject the hypothesis that it is nonzero), then that would not necessarily mean that it is not there. That is totally true, and statisticians tend to be fully aware of that. But that does not imply that there is no true data-generating process; the whole premise of this example is that there is, but that the sample size is not large enough to determine it in a detailed enough way so as to include this interaction effect, that apparently is small relative to the sample size.

      The third thing to reflect on here, is that the reviewer seems to suggest that the Generalized Price equation in regression form, as presented in my paper, comes with a specific statistical approach, that he or she classifies as philosophically naïve or unsophisticated. That, however, is not the case, and I am very grateful that this remark by this reviewer allows me to make a point that I think shines a light on how the Generalized Price equation puts the train that started going off the rails in 1970 back on track, and reconnects it with the statistics it borrows its terminology from. To see that, it is good to be aware that statistics never gives certainty. The whole discipline is built around the awareness that it is possible to draw the wrong inference, and the aim is to determine, minimize, and balance, the likelihoods of making different wrong inferences. So, statistics produces statements about the confidence with which one can say that something works one way or the other. In some instances, the data are not enough to say anything with any confidence. In other cases, the data are rich enough so that it is really unlikely that we incorrectly infer that for instance a certain gene matters for fitness.

      The nice thing about the setup with the Generalized Price equation, is that those statistical considerations translate one-to-one to considerations regarding which Price-like equation to choose. If the data do not allow us to pick any model with confidence, then we should be equally agnostic about which Price-like equation describes the population genetic dynamics accurately. If the statistics gives us high confidence that a certain model matches the data, then we should pick the matching Price-like equation with the same confidence. This also carries over to higher level statistical considerations.

      If we think about terms that, if we would gather a gargantuan amount of data, might be statistically significant, but very small, then economists call those statistically significant, but economically insignificant. When rejecting the statistical significance on the basis of a not gargantuan dataset, statisticians are aware that terms that really have a zero effect, as well as terms, the effect of which is really small, are rejected with the same statistical test – and that we should be fine with that. All such considerations carry over to what we think of regarding the choice of a Price-like equation to describe the population genetic dynamics. Even if people disagree about whether or not to include a term that is statistically significant, but relatively small, such a disagreement can still happen within this setup, and just translates to a disagreement on which Price-like equation to choose.

      Similarly, people could also disagree about whether it is justified to use polynomials to characterize a fitness function. If we decide that we can, because of Taylor expansions, then the core result of the paper implies that the population genetic dynamics can be summarized by a generalized Hamilton’s rule (as long as the fitness function includes a constant and a linear term regarding the p-score). On the other hand, if we do not believe this is justified, and prefer to use an altogether different family of fitness functions, then we can no longer do this. All of this leaves space for all kinds of statistical considerations and disagreements, that just carry over to the choice for one or the other Price-like equation as an accurate description of the population genetic dynamics. Or, if one does not believe polynomials should be used, then this leads to not picking any Price-like equation at all.

      So, this is a long way of saying that the Generalized Price equation creates space for all statistical considerations to regain their place, and does not hinge on one approach to statistics or another.

      What we can say is that if we apply the statistical model to data generated by a probabilistic model, and if these models match, then as the number of observations grows to infinity, the estimators in the statistical model converge to the parameters of the data-generating one.

      But this is a mathematical statement, not a statement about real-world populations.

      Again, I do not know if I agree or disagree with the last sentence. However, that does not really matter, because either option only has implications for how we are to think of the relation between a Price-like equation describing a population genetic dynamics and real-world populations. It is not relevant for the question which Price-like equation to pick, or whether to pick one at all.

      A resolution I suggest to points 3, 4, and 5 above is:

      *A priori, the statistical quantities in the Price Equation are descriptive statistics, pertaining only to the specific population data given.

      *If one wishes to impute any predictive power, generalizability, or causal meaning to these statistics, all the standard considerations of inferential statistics apply. In particular, one must choose a statistical model that is justified based on the given data. In this case, one is not guaranteed to obtain the standard (linear) Hamilton's rule and may obtain any of an infinite family of rules.

      *If one uses a model that is not justified based on the given data, the results will still be correct for the given population data but will lack any meaning or generalizability beyond that.

      *In particular, if one considers data generated by a probabilistic model, and applies a statistical model that does not match the data-generating one, the results will be misleading, and will not generalize beyond the randomly generated realization one uses.

      Of course, the author may propose a different resolution to points 3-5, but they should be resolved somehow. Otherwise, the terminology in the manuscript will be incorrect and the ms will not resolve confusion in the field.

      I have outlined my solutions extensively above. I really appreciate that Reviewers #1 and #2 have spent time and attention on the manuscript and on the long appendices.  

      Appendix to the response to reviewer #2: Some remarks on Gardner, West & Wild (2011), Frank (2012), and Rousset (2015)

      An accurate response to the quote from Gardner, West, and Wild (2011) in the review report takes up space. I therefore wanted to put that in an appendix to the response to reviewer #2. I also include a few paragraphs regarding Frank (2012) and Rousset (2015), both of which are also mentioned by reviewer #2. All of this might also be of interest to people that are curious about how what I find in my paper relates to the existing literature.

      Gardner, West & Wild (2011) The quote I am responding to is “it is a mistake to assume that it must be the evolutionary theorist, writing out covariances, who is performing the equivalent of a statistical analysis” I want to put that into context, so I will go over the whole paragraph that surrounds the quote. The paragraph is called Statistics and Evolutionary Theory and can be found on page 1038 of the paper. I think that it is worth pointing out that it is not easy to respond to their somewhat impressionistic collages of words and formulas. I will therefore cut the paragraph up in a few smaller bits and try to make sense of it bit by bit. The paragraph begins with:

      “Our account of the general theory of kin selection has been framed in statistical terms.” Based on what they write two sentences down, the best match between those words and what they do in the paper would be: “our account uses words like “covariance”, “variance” and “expectation” for things that are not what “covariance”, “variance” and “expectation” mean in probability theory and statistics.” I would be totally open to an argument why that is nonetheless OK to do, but the way Gardner, West, and Wild (2011) phrase it obscures the fact that this needs any justification or reflection at all. “Framing something in statistical terms” is unspecific enough to sound completely harmless.

      “The use of statistical methods in the mathematical development of Darwinian theory has itself been subjected to recent criticism (van Veelen, 2005; Nowak et al., 2010b), so we address this criticism here.

      Also here, specifics would be helpful. The “use of statistical methods” sounds like it is more than just using terms from statistics, so this might refer to the minimizing of the sum of squared differences, which is also mentioned a sentence down in Gardner, West, and Wild (2011). If it does, then it is worth observing that in statistics, the minimizing of the sum of squared differences (or residuals, or errors) comes with theorems that point very clearly to what is being achieved by doing this. The Gauss–Markov theorem states that the ordinary least squares (OLS) estimator has the lowest variance within the class of linear unbiased estimators. This implies that minimizing the sum of squared errors helps answering a well-defined question in statistics; under certain conditions, an OLS estimator is our best shot at uncovering an unknown relation between variables. To also minimize a sum of squared differences, but now in the modeling domain, qualifies as “use of statistical methods” only in a very shallow way. It means that a similar minimization is performed. Without an equivalent of the Gauss-Markov theorem that would shine a light on what it is that is being achieved by doing so, that does not carry the same weight as it does in the statistics domain – in that it does not carry any weight at all.

      “The concern is that statistical terms – such as covariances and least-squares regressions – should properly be reserved for conventional statistical analyses, where hypotheses are tested against explicit data, and that they are out of place in the foundations of evolutionary theory (van Veelen, 2005; Nowak et al., 2010b).”

      Again, a few things are a bit vague. What are “explicit data”? Are there data that are not explicit? Why the generic “foundations of evolutionary theory”, instead of a more specific description of what these statistical terms are used for? But either way, this is a misrepresentation of what I wrote in van Veelen (2005). I did not suggest to “reserve statistical terms for conventional statistical analysis” just because. As I do here in the current paper, what I did there was explore the possibilities for the Price equation to help with what I then called Type I and Type II questions. Type I questions find themselves in the modeling domain and Type II questions find themselves in the statistical domain. I was not arguing for a ban on applying statistical concepts outside of the domain of statistical inference. All that I said is that in its current practice, it does not really help answering questions of either type.  

      “However, this concern is misplaced. First, natural selection is a statistical process, and it is therefore natural that this should be defined in terms of aggregate statistics, even if only strictly by analogy (Frank, 1997a, 1998).”

      This is a vague non-argument. Almost nothing is well-defined here. What does it mean for natural selection to be a statistical process? Is that just an unusual term for a random process? If so, then I suppose I agree, but that has nothing to do with what I state or claim. And what does it mean to be defined in terms of aggregate statistics? What is the alternative? I have no idea how any of this relates to anything that I claim or state in my papers.

      “Second, Fisher (1930, p198) coined the term ‘covariance’ in the context of his exposition of the genetical theory of natural selection, so the evolutionary usage of this term has precedent over the way the term is used in other fields.”

      This is what I would call a “historic fallacy”. The fact that Fisher coined the term “covariance” in a book on genetics and natural selection does not mean that any “evolutionary usage” of the term “covariance”, however nonsensical, now has precedent over the way the term is used in other fields. Irrespective of the path that the history of science, genetics, or statistics took, right now we are in a place where about every student at every university anywhere in the world that takes a course in probability theory and/or statistics, learns that covariance is a property of a random variable (see also Wikipedia). And they do for a very good reason; it is essential in recognizing the relation between probability theory on the one hand and statistics on the other. Being curious how this “evolutionary usage” of the term covariance works, if covariance turns out not to be a property of a random variable, is therefore perfectly justified, and “Fisher coined the term” is not a safe word that exempts it from scrutiny. 

      Third, it is a mistake to assume that it must be the evolutionary theorist, writing out covariances, who is performing the equivalent of a statistical analysis.

      Again, that is just not what anyone is saying. Nobody is suggesting that an evolutionary theorist should perform the equivalent of statistical analysis. All I did was point to how little is being achieved by transferring formulas from statistics to a modeling context.

      A better analogy is to regard Mother Nature in the role of statistician, analysing fitness effects of genes by the method of least-squares, and driving genetic change according to the results of her analyses (cf. Crow, 2008).

      I have no idea what any of this means. Mother Nature is a personification of something that is not a person, and that does not have cognition. Without sentience, “Mother Nature” cannot assume the role of statistician, and cannot analyse fitness effects.

      More generally, analogy is the basis of all understanding, so when isomorphisms arise unexpectedly between different branches of mathematics (in this case, theoretical population genetics and statistical least-squares analysis) this represents an opportunity for advancing scientific progress and not an anomaly that is to be avoided.

      This is a strawman argument, puffed up with platitudes. Nobody is arguing against analogies. But what is the analogy supposed to be here? Just taking least squares from statistical inference and performing it in a modeling context does not make it an analogy. The GaussMarkov theorem, which is the basis for why least squares helps answering questions in statistics, just does not mean anything in a modeling context. OLS in modeling is just willful misspecification, and nothing that it does in statistics translates to anything meaningful in modeling. Again, declaring it an analogy, or an isomorphism, does not make it one.

      Frank (2012) Because the reviewer also mentions Frank (2012), I would like to include a small remark on this paper too. “Natural Selection. IV. The Price equation” by Frank (2012) is partly a response to my earlier criticism of the use of the Price equation. Much like Gardner, West, and Wild (2011), I would describe this paper as what is called a ”flight forwards” in Dutch. While the questions I ask are relatively prosaic (such as: how does the Price equation help derive a prediction from model assumptions?), Frank (2012) pivots to suggesting that there is a profound philosophy-of-science disagreement that I am on the wrong side of. It is close to impossible to respond to Frank (2012), because it is a labyrinth of arguments that sound deep and impressive, but that are just not specific enough to know how they relate to points that I made – or even just what they mean in general. Just to pick a random paragraph:

      “Is there some reorientation for the expression of natural selection that may provide subtle perspective, from which we can understand our subject more deeply and analyse our problems with greater ease and greater insight? My answer is, as I have mentioned, that the Price equation provides that sort of reorientation. To argue the point, I will have to keep at the distinction between the concrete and the abstract, and the relative roles of those two endpoints in mature theoretical understanding.”

      For many of those terms, I have no real idea what they mean, and also reading the rest of the paper does not help understanding what this has to do with the more prosaic questions that are waiting for an answer. What is “reorientation”? What does “concrete” versus “abstract” have to do with the question what is being achieved by doing least squares regressions in modeling? What would be an example of a mature and an immature theoretical understanding?

      Rousset (2015) is also mentioned by the reviewer. This paper is not esoteric. It states, as reviewer #2 points out, that "neither data nor inferences are considered". This paper therefore finds itself in the modeling domain, and not in the data domain. It does however still dodge the question what the benefits are of misspecification in the modeling domain. As a matter of fact, it denies that there is misspecification at all.

      “In the presence of synergies, the residuals have zero mean and are uncorrelated to the predictors. No further assumption is made about the distribution of the residuals. Thus, there is no sense in which the regression is misspecified.”

      This is a remarkable quote, and testament to the lasting impact of the construction errors in Price (1970). Misspecification is literally defined as getting the model wrong. In statistics, avoiding misspecification can be complicated, because of the noise in the data. The real datagenerating process is unknown, and because of the noise, there is always the possibility that data that are generated by one model look like they could also have been generated by another. The challenge is to reduce the odds of getting the model wrong to acceptable proportions, which is what statistical tests are for. But in modeling, we know what the model is; it is postulated by the modeler. Therefore, misspecification can be avoided by just not replacing it with a different model.

      What is being discussed in this part of Rousset (2015) is replacing what in this manuscript is called Model 3 (𝑤<sub>𝑖</sub> = 𝛼 + 𝛽<sub>1,0</sub>𝑝<sub>𝑖</sub> + 𝛽<sub>1,1</sub>𝑝<sub>𝑖</sub> + 𝛽<sub>1,1</sub>𝑝<sub>𝑖</sub>𝑞<sub>𝑖</sub> + 𝜀<sub>𝑖</sub>) with Model 2 (𝑤<sub>𝑖</sub> = 𝛼 + 𝛽<sub>1,0</sub>𝑝<sub>𝑖</sub>+ 𝛽<sub>1,0</sub>𝑝<sub>𝑖</sub>𝑞<sub>𝑖</sub> + 𝜀<sub>𝑖</sub>), and choosing the parameters in Model 2 so that it is as close as it can be to Model

      (3) This is just the definition of misspecification. That is to say: the misspecification part is the choosing of Model 2 as a reference model. The minimizing of the sum of squared residuals one could consider as minimizing the damage.

      While Rousset (2015) finds itself in the modeling domain, it does nonetheless point to the field of statistics here, by stating that “the residuals have zero mean and are uncorrelated to the predictors”. From this, the paper concludes that “there is no sense in which the regression is misspecified”. That is just plain wrong. Minimizing the sum of the squared residuals guarantees that the residuals are uncorrelated with the variables that are included in the reference model, with respect to which the squared sum of residuals is minimized. The criterion that Rousset (2015) uses is that the model is well-specified if there is no correlation between the residuals (here: ) and the variables included in the reference model (here: 𝑝<sub>𝑖</sub> and 𝑞<sub>𝑖</sub>). But according to this criterion, all models would always be well-specified, and no model could ever be misspecified. The correct criterion, however, also requires that the residuals are not correlated with variables not included in the reference model. And here, the residuals are in fact correlated with 𝑝<sub>𝑖</sub>𝑞<sub>𝑖</sub>, which is the variable that is included in Model 3, but not in Model 2. Therefore, according to the correct version of this criterion, this model is in fact misspecified – as it should be, because getting the model wrong is the definition of misspecification.

      In order to make sure that there can be no misunderstanding, I have added subsections at the end of Section 2 and Section 4 of Appendix A, and at the end of Section 2 of Appendix B. These subsections show that the algebra of minimizing the sum of squared errors implies that there is no correlation between the errors, or the residuals, and the variables that are included in the model. This is by no means something new; it is the reason why we do OLS to begin with. For additional details about misspecification, I would refer to Section 1b (viii) in van Veelen (2020).

      Finally, there is a detail worth noticing. In the main text, as well as in Appendix B, I use an analogy (and, unlike what Gardner, West, and Wild, 2011, refer to as an analogy, this actually is one). This is an analogy between two choices. On the one hand, there is the choice between Price-like equation 1 (based on Model 1 as a reference model) and Price-like equation 2 (based on Model 2 as a reference model) both applied to Model 2. On the other hand, there is the choice between Price-like equation 2 (based on Model 2 as a reference model) and Price-like equation 3 (based on Model 3 as a reference model) both applied to Model 3. Model 1 is the non-social model, Model 2 is the social model without interaction term, and Model 3 is the social model with interaction term. That makes the first choice a choice between treating a social model as a social model, or as a non-social model. The second choice is between treating a social model with interaction term as a social model with interaction term, or as a social model without interaction term. The power of this analogy is that every argument against treating the social model as if it is a non-social model is also an argument against treating the social model with interaction term as if it is a social model without interaction term.

      This ties in with the incorrect criterion for when a model is well-specified from Rousset (2015) as follows. His criterion (that there should be no correlation between the residuals and the variables in the model) declares the social model without interaction term well-specified as a reference model, when we are considering a social model with interaction term. According to the same criterion, however, the non-social model would also have to be declared to be wellspecified as a reference model, when the model we are considering is a social model. The reason is that also here, there is no correlation between the residuals and the variables that are included in this model. This is clearly not what anyone is advocating for, and for good reasons. The residuals here would, after all, be correlated with the p-score of the partner, which is a variable that is not included in the non-social model. This is a good indication that we should not use the non-social model for a social trait.

      Reviewer #3 (Public review):

      Before responding to this review, I would like to express that I appreciate the fact that the reviews and the responses are public at eLife. Besides just being useful in general, this also allows readers to get a behind the scenes glimpse into the state of the field, and the level of the reviewing. While the reports by Reviewers #1 and #2 show openness and an interest in getting things right, the report by Reviewer #3 is representative of the many review reports that I have received from the inclusive fitness community in the past. These reports tend to be rhetorically strong, and to those who do not have the time to dig deeper in the details, these reports are probably also convincing. I will therefore go through this review line by line to show how little there is behind the confident off-hand dismissal.

      There is an interesting mathematical connection - an "isomorphism"-between Price's equation and least-squares linear regression.

      This is esoteric and needlessly vague. Why is the word “isomorphism” used? In mathematics, an isomorphism is a structure-preserving mapping. The Price equation is an equation, or an identity, which makes it a bit difficult to imagine what the set of objects is on one end of the mapping. Least-squares linear regression can perhaps be seen as a function of a dataset, which would make it a single object (one function). This complicates things at the other end of the mapping too, if that set is a singleton set. The only isomorphism that I can think of is a trivial isomorphism where one equation is mapped onto one function and vice versa. It seems unlikely that this is what the reviewer means. The word isomorphism moreover is in quotes, so maybe this is supposed to be figurative. But what would it be that is being suggested here by this figure of speech? Just saying that there is, as the reviewer puts it, an “interesting mathematical connection”, does not make it so. It would already be a start to just specify what the mathematical connection is, because I have a hard time seeing what that would be. Is it just that, if you divide the Cov(𝑤, 𝑝)-term by the Var(𝑝)-term, then you get a regression coefficient? If that is what the reviewer has in mind, that would be a rather shallow observation.

      Some people have misinterpreted this connection as meaning that there is a generalitylimiting assumption of linearity within Price's equation, and hence that Hamilton's rule-which is derived from Price's equation-provides only an approximation of the action of natural selection.

      Here, the reviewer pulls a switcheroo. The use of the word “general”, or “generality”, here refers to the fact that the classical Price equation is an identity for all possible transitions between a parent and an offspring population. This is the sense in which the inclusive fitness literature uses the word general, and so do I in the relevant places in the manuscript. When I do, I make sure to add phrases like “in the sense that whatever the true model is, it always gets the direction of selection right”. As a consequence, the classical Hamilton’s rule is also totally general, in the same sense.

      One of the core points of the paper is that this is not unique to the classical Price equation. As a matter of fact, there is a large set of Price-like equations and Hamilton-like rules that are equally much identities, and equally much general (in the sense that they get the direction of selection right for all possible transitions). The being an identity and being completely general (in this sense) therefore cannot be a decisive criterion in favour of the classical Price equation and the classical Hamilton’s rule.

      On the other hand, the way in which my Generalized Price equation and my generalized version of Hamilton’s rule are general, is that they do not restrict the statistical model with respect to which errors are squared, summed and minimized to one linear statistical model. This generalization generates the variety of Price-like equations and Hamilton-like rules mentioned above (all of which are general in the sense of always getting the direction of selection right) and it gives us the flexibility to pick one that separates terms that reflect the fitness function from terms that reflect the population state.

      In response to my generalizing the Price equation and Hamilton’s rule in this second sense, the criticism of the reviewer comes down to saying that the Price equation and Hamilton’s rule do not need generalizing, because they already are general – the switcheroo being that this refers to generality in the first sense. That makes it sound like this could be an honest mistake, confusing one way in which these can be described as general with another. However, I really hammered this point home in the manuscript. Even a cursory reading of the manuscript reveals that I am fully aware that the classical Price equation and the classical Hamilton’s rule are general in the first sense.

      It is also not helpful that, as a description of what I supposedly claim, this is impressionistic, and lacks specificity. The Price equation is an equation, or an identity. What does it mean for there to be an “assumption of linearity” within it? For the classical Price equation in covariance form (which Reviewer #2 argues is what most people think of as “the Price equation”) there is no way in which one can transform this into a meaningful statement. There is just nothing in there to which the adjective “linear” can be applied. Linearity only becomes a thing when we ask ourselves how we can interpret the regression coefficient in the classical Price equation in regression form. That would be the linearity of the statistical model the differences with which are squared, summed and minimized in the regression.

      This is in contrast to the majority view that Hamilton's rule is a fully general and exact result.

      Again, in this manuscript, I write, time and again, that the classical Hamilton’s rule is fully general (in the sense that it is applies to any transition), and exact (if that means that it always gets the direction of selection right). So, this is clearly not where the contrast with the majority view lies. The contrast with the majority view is that the majority insist on misspecification, and I suggest not to do that.

      To briefly give some mathematical details: Price's equation defines the action of natural selection in relation to a trait of interest as the covariance between fitness 𝑤 and the genetic breeding value 𝑔 for the trait, i.e. Cov(𝑤, 𝑔);

      The Price equation is an identity, not a definition. When deciding on a definition, there is some freedom. We can choose to define ⊂ so that 𝐴 ⊂ 𝐵 means that 𝐴 is a strict subset of 𝐵; or we can choose to define ⊂ so that 𝐴 ⊂ 𝐵 means that 𝐴 is a (not necessarily strict) subset of 𝐵. The Price equation does not “define the action of natural selection”, because it is an identity. There is no freedom to “define” any other way.

      The more serious reason why this is conceptually also a little dangerous, is the following. Imagine a locus with two alleles. Both of them are non-coding bits of DNA. Selection therefore does not act on either of them. Now imagine a parent population with an average p-score of 0.5, or, in other words, the frequency of these alleles in the parent population is 50-50. That makes the expected value of the p-score in the offspring population 0.5 too. In finite populations, however, randomness can make the p-score grow a bit larger or a bit smaller than 0.5. If the parent population is small, the variance (the expected squared deviation from 0.5) can actually be sizeable. If the p-score in the offspring population lands above 0.5, then the Price equation has a > 0 and a 𝐶𝑜𝑣(𝑤, 𝑝) > 0. Describing the Price equation as “defining the action of natural selection” now suggests that higher p-scores have been selected for (or, in other words, that “the action of natural selection in relation to a trait of interest” is positive). With equal probability, however, < 0 and therefore also 𝐶𝑜𝑣(𝑤, 𝑝) < 0, and this would then make us draw the opposite conclusion, that natural selection has acted to lower the p-scores in the population. Both of those would be wrong, because in this situation, it would have been randomness that changed the average p-score. 

      this is a fully general result that applies exactly to any arbitrary set of (𝑔, 𝑤) data; without any loss of generality this covariance can be expressed as the product of genetic variance Var(𝑝) and a coefficient 𝑏(𝑔, 𝑤), the coefficient simply being defined as 𝑏(𝑔, 𝑤) = for all Var(𝑝) > 0; it happens that if one fits a straight line to the same (𝑔, 𝑤) data by means of least-squares regression then the slope of that line is equal to 𝑏(𝑔, 𝑤).

      Why this needs to be explained is a bit of a mystery. These “mathematical details” are in almost all Price equation papers, and they are the point of departure of my Appendix A (it is on page 7 of a more than 90 page long set of appendices). Seeing the need to explain this suggests that the reviewer thinks that there is a chance that I or anyone reading this paper would have missed this. I have not, and, more importantly, none of this invalidates the point I make in the paper.   

      All of this has already been discussed, repeatedly, in the literature.

      All of this has already been discussed, repeatedly, in the literature indeed. It is just that it does not engage with anything I write in the manuscript, or that I wrote in my other papers.

      Now turn to the present paper: the first sentence of the Abstract says "The generality of Hamilton's rule is much debated", and then the next sentence says "In this paper, I show that this debate can be resolved by constructing a general version of Hamilton's rule".

      This is correct.

      But immediately it's clear that this isn't really resolving the debate, what this paper is actually doing is asserting the correctness of the minority view (i.e. that Hamilton's rule as it currently stands is not a general result)

      It seems to me that the reason why this is “immediately clear” to this reviewer is that the reviewer has not processed the contents of the paper. I am not sure if I have to repeat this, but I am not saying that “Hamilton’s rule as it currently stands” is not general (in the sense that it always gets the direction of selection right). It is, and I say that it is a bunch of times. But so are other rules.

      and then attempting to build a more general form of Hamilton's rule upon that shaky foundation.

      I am not just “attempting to build a more general form of Hamilton's rule”. I did in fact build a more general form of Hamilton’s rule (where the generality refers to the richer set of reference statistical models).

      Predictably, the paper erroneously interprets the standard formulation of Hamilton's rule as a linear approximation and develops non-linear extensions to improve the goodness of fit for a result that is already exactly correct.

      Nowhere in the paper or the appendices do I describe the standard formulation of Hamilton’s rule (or, for that matter, any formulation of Hamilton’s rule) as an “approximation”. It is just not a word that has anything to do with this. If we are doing statistical inference, and the sum of squared errors that is minimized decreases by adding a variable in the statistical model with regard to which the sum of squared errors is minimized, then that will typically improve the goodness of fit. In statistics this is not described that as an improvement in how well the statistical model “approximates” the data, or whatever it is that the reviewer would suggest is being approximated here.

      This is not a convincing contribution. It will not change minds or improve understanding of the topic.

      There is indeed plenty of scope for this not to change minds or improve understanding of the topic. It will not change the minds or improve the understanding of those that are not really interested in getting this right. Obviously, it will also not convince those that do not read it.

      Nor is it particularly novel. Smith et al (2010, "A generalisation of Hamilton's rule for the evolution of microbial cooperation" Science 328, 1700-1703) similarly interpreted Hamilton's rule as a linear model and provided a corresponding polynomial expansion - usefully fitting the model to microbial data so as to learn something about the costs and benefits of cooperation in an empirical setting. it's odd that this paper isn't cited here.

      Let me begin by pointing to what I agree with. Given that smith et al. (2010) and my manuscript are both in the business of generalizing Hamilton’s rule, it would be helpful to the reader if my paper includes more information about how the two efforts relate. I will discuss the relation below, and I will also include that in Appendix B, and point to it in the main text. Before I do, however, I would like to point to two details in the review report that fit a pattern.

      The first is that the reviewer describes what smith et al. (2010) do as “useful”, and seems to think of fitting polynomial expansions as a legitimate way to “learn something about the costs and benefits of cooperation in an empirical setting”. That sounds quite positive. My paper, in which I supposedly repeat this, however, is characterized as misguided. This fits a pattern; all of the reviews I received from the inclusive fitness community include a “done before”, and regularly the done before is described approvingly, while my paper is described as fundamentally flawed.

      Also customary is the lack of detail. What would be really useful here, is something like “equation A.14 in this manuscript is the same as equation 6 in smith et al. (2010) if we choose . This kind of statement would pin down the way in which what I do has been done before. That, however, would require going into detail, at the risk of finding out that what is done in my manuscript is actually quite different from what happens in smith et al. (2010). That is also a recurrent thing. When I look up the done before, I typically find something that is not quite the same.  

      Now on to the paper. What smith et al. (2010) try to do is something that I wholeheartedly support. It is an empirical study that tries to capture non-linearity. A first point of order is that it is worth asking ourselves: linear or non-linear in what? For that, I would like to go back to the setup of my manuscript. Model 2 from the Main Text is

      In this fitness function, 𝑝! is the p-score of individual 𝑖 and 𝑞! is the p-score of the partner that individual 𝑖 is matched with. This is a standard model of social behaviour if 𝛽<sub>1,0</sub> < 0 and 𝛽<sub>0,1</sub> > 0. Such choices for 𝛽<sub>1,0</sub> and 𝛽<sub>0,1</sub> indicate that having a higher p-score decreases the fitness of individual 𝑖 and increases the fitness of its partner. Here we assume that 𝛼 = 1, 𝛽<sub>1,0</sub> \= −1, and 𝛽<sub>0,1</sub> \= 2. We assume that p-scores can only be 0 or 1, or, in other words, we assume that there are only cooperators and defectors in the population (or, in terms of smith et al., 2010: cooperators and cheaters).

      For a well-mixed population, where the likelihood of being matched with a cooperator is the same for cooperators and defectors (it is equal to the frequency of cooperators for both), we can now plot the fitnesses of cooperators (red) and defectors (blue) as a function of the frequency of cooperators (Appendix 1-figure 6 left).

      We can do the same for a population with relatedness where the probability of being matched with a cooperator is + 𝑓<sub>c</sub> for cooperators, and 𝑓<sub>c</sub> for defectors, where 𝑓<sub>c</sub> is the frequency of cooperators (Appendix 1-figure 6 right). For relatedness 𝑟 = 0 and 𝑟 = "7, cooperation is selected against at every frequency.

      Increasing relatedness further, we would find that for 𝑟 = the lines coincide, which implies that at every frequency, cooperation is neither selected for nor against. For 𝑟 > ": cooperation will be selected for at every frequency. This pattern implies that, as we have seen in the manuscript, the classical Hamilton’s rule works perfectly fine for Model 2; with 𝑐 = −𝛽<sub>1,0</sub> = 1 and 𝑏 = 𝛽<sub>0,1</sub> \= 2, cooperation is selected for if and only if 𝑟𝑏 > 𝑐. The fitnesses of cooperators and defectors as functions of the frequency of cooperators, moreover, are always parallel lines, regardless of relatedness.

      Model 3 in the main text extends Model 2 by adding an interaction term:

      Now we choose 𝛼 = 1, 𝛽<sub>1,0</sub> = −1, 𝛽<sub>1,0</sub> = 1, and 𝛽<sub>1,1</sub>  \= 1. We again draw the fitnesses of cooperators and defectors, both at relatedness 𝑟 = 0 (Appendix 1-figure 7 left) and at relatedness 𝑟 = (Appendix 1-figure 7 right). In the manuscript, I argue that the appropriate version of Hamilton’s rule here is Queller’s rule: 𝑟<sub>0,1</sub>𝑏<sub>0,1</sub> + 𝑟<sub>1,1</sub>𝑏<sub>1,1</sub> > 𝑐 with 𝑐 = −𝛽<sub>1,0</sub> = 1, 𝑏<sub>0,1</sub> = 𝛽<sub>0,1</sub> = 1, and 𝑏<sub>1,1</sub> = 𝛽<sub>1,1</sub> = 1. The fitnesses of cooperators and defectors as functions of the frequency of cooperators are still straight lines, but they are no longer parallel.

      The first thing to observe, therefore, is that a model with synergy, in which the classic version of Hamilton’s rule would be misspecified, and Queller’s rule would be well-specified, does not require the fitnesses as functions of the frequencies of cooperators to be non-linear. All that changes with the addition of the interaction term, is that they stop being parallel.

      The paper by smith et al. (2010) is an effort to capture non-linearities in the way fitnesses depend on the frequency of cooperators. That, therefore, goes beyond the step from Model 2 to Model 3. Whether it uses the right method to capture those non-linearities, we will come back to in a second, but it is important to realize that also without these non-linearities, the classic version of Hamilton’s rule can be too limiting to accurately describe selection. (Here, I should add that this implies that we were wrong in Wu et al. (2013), when we suggested that “for this experiment, it seems unnecessary to use the generalized Hamilton’s rule, if instead the Malthusian fitness is adopted. In other words, the Wrightian fitness approach calls for a generalization of Hamilton’s rule, whereas the Malthusian fitness approach does not (or at least not in a drastic way, as Malthusian fitnesses are almost linear in the frequency of cooperators).” Using Malthusian fitnesses, the functions were close to linear, but not close to parallel, and therefore also here, Hamilton’s rule needs generalizing - albeit in a different way than smith et al. (2010) did).

      The cooperation that is observed in the Myxococcus xanthus studied by smith et al. (2010) is not a good match with a model where individuals are matched in pairs for an interaction that determines their fitnesses. These microbes cooperate in large groups, and a better match would therefore be the n-player public goods games studied in van Veelen (2018). There, we see that simple, straightforward ways to describe synergies (or anti-synergies) can easily lead to fitnesses not being linear in the frequency of cooperators.

      The way smith et al. (2010) try to capture those non-linearities, however, is not free of complications. We addressed those in Wu et al. (2013), and I summarized them, shortly, in van Veelen (2018). One of the issues is that most of the non-linearity smith et al. (2010) pick up is the result of considering Wrightian fitness rather than Malthusian fitness. In a continuous time model with a constant growth rate, the population size at time 𝑡 is 𝑁(𝑡) = 𝑒<sup>mt</sup>𝑁(0), where 𝑚 is the Malthusian fitness. In a discrete time model with a constant average number of offspring per individual, the population at time 𝑡 is 𝑁(𝑡) = 𝑤<sup>t</sup>𝑁(0), where 𝑤 is the Wrightian fitness. If we take 𝑚 = ln 𝑤, these are the same, and if 𝑤 is close to 1, then 𝑚 can be approximated by 𝑤 − 1. That also implies that if 𝑤 is close to 1 (or, equivalently, if 𝑚 is close to 0) one is locally linear if the other is too. However, in the experiment by smith et al. (2010) the aggregate fitness effects are not small, and what is highly nonlinear in terms of Wrightian fitness is close to linear in Malthusian fitness.

      Another complication is that the Taylor coefficients that smith et al. (2010) find are the result of a combination of the data and the choice of a functional form they choose to first apply to their data. That means that a different choice of a functional form would have given different Taylor coefficients, while the in-between transformation can also be skipped. Also, the number of Taylor coefficients is larger than the dimensionality of the data, which are based on averages for 6 frequencies. For more details on these complications, I would like to refer to Wu et al. (2013) and van Veelen (2018). A nice detail is that if we consider the way the fitnesses of cooperators and defectors compare when using Malthusian fitnesses, then a comparison of the slopes actually suggests anti-synergies, which leads to a stable mix of cooperators and cheaters, already in the absence of population structure. This matches what is suggested by Archetti and Scheuring, (2011, 2012) and Archetti (2018).

      Besides these technical complications, smith et al. (2010) is also different, in the sense that it is an empirical paper. It does not contain the Generalized Price equation, it contains no insights regarding how to derive population genetic dynamics from the Generalized Price equation, or how to derive the appropriate rules from those, and it has a very different approach to separating fitness effects and population structure.

      To end on a positive note, I would like to quote a bit out of Wu et al. (2013):

      “While we criticise these mathematical issues, we are convinced that smith et al. (2010) aim into the right direction: to incorporate the nonlinearities characteristic of biology into social evolution, we may have to extend and generalize the approach of inclusive fitness. It would be beautiful if such a generalization would ultimately include Hamilton’s original rule as a special case […].”

      I like to think that this is exactly what I have done in this paper.

      References

      Akdeniz, A., & van Veelen, M. (2020). The cancellation effect at the group level. Evolution, 74(7), 1246–1254. doi: 10.1111/evo.13995

      Allen, B., & Tarnita, C. E. (2012). Measures of success in a class of evolutionary models with fixed population size and structure. Journal of Mathematical Biology, 68, 109–143. doi: 10.1007/s00285-012-0622-x

      Archetti, M. (2018). How to Analyze Models of Nonlinear Public Goods. Games 2018, Vol. 9, Page 17, 9(2), 17. doi: 10.3390/g9020017

      Archetti, M., & Scheuring, I. (2011). Coexistence of cooperation and defection in public goods games. Evolution, 65(4), 1140–1148. doi: 10.1111/j.1558-5646.2010.01185.x

      Archetti, M., & Scheuring, I. (2012). Review: Game theory of public goods in one-shot social dilemmas without assortment. Journal of Theoretical Biology, 299, 9–20. doi: 10.1016/j.jtbi.2011.06.018

      Bourke, A. F. G. (2014). Hamilton’s rule and the causes of social evolution. Philosophical Transactions of the Royal Society B: Biological Sciences, 369(1642), 20130362. doi: 10.1098/rstb.2013.0362

      Bourrat, P., Godsoe, W., Pillai, P., Gouhier, T. C., Ulrich, W., Gotelli, N. J., & van Veelen, M. (2023). What is the price of using the Price equation in ecology? Oikos, 2023(8). doi: 10.1111/oik.10024

      Crow, J. F. (2008). Commentary: Haldane and beanbag genetics. International Journal of Epidemiology, 37(3), 442–445. doi: 10.1093/ije/dyn048

      Fisher, R. (1930). The genetical theory of natural selection. Retrieved from https://www.cabidigitallibrary.org/doi/full/10.5555/19601600934

      Fletcher, J. A., & Zwick, M. (2006). Unifying the theories of inclusive fitness and reciprocal altruism. American Naturalist, 168(2), 252–262. doi: 10.1086/506529

      Frank, S. A. (1997). The Price equation, Fisher’s fundamental theorem, kin selection, and causal analysis. Evolution, 51(6), 1712–1729. doi: 10.1111/j.1558-5646.1997.tb05096.x

      Frank, S. A. (1998). Foundations of social evolution. Princeton: Princeton University Press.

      Frank, S. A. (2012). Natural selection. IV. The Price equation*. Journal of Evolutionary Biology, 25(6), 1002–1019. doi: 10.1111/j.1420-9101.2012.02498.x

      Gardner, A., West, S. A., & Wild, G. (2011). The genetical theory of kin selection. Journal of Evolutionary Biology, 24(5), 1020–1043. doi: 10.1111/j.1420-9101.2011.02236.x

      Grafen, A. (1985a). A geometric view of relatedness. Oxford Surveys in Evolutionary Biology, 2(2), 28-89.

      Grafen, A. (1985b). News and Views. Evolutionary theory: Hamilton’s rule OK. Nature, 318(6044), 310–311. doi: 10.1038/318310a0

      Hamilton, W. D. (1964). The genetical evolution of social behaviour. I. Journal of Theoretical Biology, 7(1), 1–16. doi: 10.1016/0022-5193(64)90038-4

      Karlin, S., & Matessi, C. (1983). The eleventh R. A. Fisher Memorial Lecture - Kin selection and altruism. Proceedings of the Royal Society of London. Series B. Biological Sciences, 219(1216), 327–353. doi: 10.1098/rspb.1983.0077

      Matessi, C., & Karlin, S. (1984). On the evolution of altruism by kin selection. Proceedings of the National Academy of Sciences, 81(6), 1754–1758. doi: 10.1073/pnas.81.6.1754

      Nowak, M. A., Tarnita, C. E., & Wilson, E. O. (2010). The evolution of eusociality. Nature, 466(7310), 1057–1062. doi: 10.1038/nature09205

      Okasha, S. (2005). Maynard Smith on the levels of selection question. Biology and Philosophy, 20(5), 989–1010. doi: 10.1007/S10539-005-9019-1/METRICS

      Page, K. M., & Nowak, M. A. (2002). Unifying evolutionary dynamics. Journal of Theoretical Biology, 219(1). doi: 10.1016/S0022-5193(02)93112-7

      Pillai, P., & Gouhier, T. C. (2019). Not even wrong: the spurious measurement of biodiversity’s effects on ecosystem functioning. Ecology, 100(7), e02645. doi: 10.1002/ecy.2645

      Price, G. R. (1970). Selection and Covariance. Nature, 227(5257), 520–521. doi: 10.1038/227520a0

      Price, G. R. (1972). Extension of covariance selection mathematics. Annals of Human Genetics, 35(4), 485-490.

      Queller, D. C. (1985). Kinship, reciprocity and synergism in the evolution of social behaviour. Nature, 318(6044), 366–367. doi: 10.1038/318366a0

      Queller, D. C. (1992a). A general model for kin selection. Evolution, 46(2), 376–380. doi: 10.1111/j.1558-5646.1992.tb02045.x

      Queller, D. C. (1992b). Quantitative Genetics, Inclusive Fitness, and Group Selection. The American Naturalist, 139(3), 540–558. doi: 10.1086/285343

      Queller, D. C. (2011). Expanded social fitness and Hamilton’s rule for kin, kith, and kind. Proceedings of the National Academy of Sciences, 108(supplement_2), 10792–10799. doi: 10.1073/pnas.1100298108

      Rousset, & Billiard. (2000). A theoretical basis for measures of kin selection in subdivided populations: Finite populations and localized dispersal. Journal of Evolutionary Biology, 13(5). doi: 10.1046/j.1420-9101.2000.00219.x

      Rousset, F. (2015). Regression, least squares, and the general version of inclusive fitness. Evolution, 69(11), 2963–2970. doi: 10.1111/evo.12791

      Smith, J., Van Dyken, J. D., & Zee, P. C. (2010). A generalization of hamilton’s rule for the evolution of microbial cooperation. Science, 328(5986), 1700–1703. doi: 10.1126/science.1189675

      Sober, Elliott., & Wilson, D. Sloan. (2007). Unto others : the evolution and psychology of unselfish behavior. 394. Retrieved from https://www.hup.harvard.edu/books/9780674930476

      Taylor, P. D. (1992). Altruism in viscous populations - an inclusive fitness model. Evolutionary Ecology, 6(4), 352–356. doi: 10.1007/bf02270971

      Taylor, Peter D. (1989). Evolutionary stability in one-parameter models under weak selection. Theoretical Population Biology, 36(2), 125–143. doi: 10.1016/00405809(89)90025-7

      Taylor, Peter D., Day, T., & Wild, G. (2007). Evolution of cooperation in a finite homogeneous graph. Nature, 447(7143), 469–472. doi: 10.1038/nature05784

      Van Cleve, J. (2015). Social evolution and genetic interactions in the short and long term. Theoretical Population Biology, 103. doi: 10.1016/j.tpb.2015.05.002

      van Veelen, M. (2005). On the use of the Price equation. Journal of Theoretical Biology, 237(4). doi: 10.1016/j.jtbi.2005.04.026

      van Veelen, M. (2007). Hamilton’s missing link. Journal of Theoretical Biology, 246(3). doi: 10.1016/j.jtbi.2007.01.001

      van Veelen, M. (2011). The replicator dynamics with n players and population structure. Journal of Theoretical Biology, 276(1). doi: 10.1016/j.jtbi.2011.01.044

      van Veelen, M. (2018). Can Hamilton’s rule be violated? ELife, 7. doi: 10.7554/eLife.41901

      van Veelen, M. (2020). The problem with the Price equation. Philosophical Transactions of the Royal Society B: Biological Sciences, 375(1797), 20190355. doi: 10.1098/rstb.2019.0355

      van Veelen, M., Allen, B., Hoffman, M., Simon, B., & Veller, C. (2017). Hamilton’s rule. Journal of Theoretical Biology, 414. doi: 10.1016/j.jtbi.2016.08.019

      van Veelen, M., García, J., Sabelis, M. W., & Egas, M. (2012). Group selection and inclusive fitness are not equivalent; the Price equation vs. models and statistics. Journal of Theoretical Biology, 299. doi: 10.1016/j.jtbi.2011.07.025

      Wilson, D. S., Pollock, G. B., & Dugatkin, L. A. (1992). Can altruism evolve in purely viscous populations? Evolutionary Ecology, 6(4), 331–341. doi: 10.1007/bf02270969

      Wu, B., Gokhale, C. S., van Veelen, M., Wang, L., & Traulsen, A. (2013). Interpretations arising from Wrightian and Malthusian fitness under strong frequency dependent selection. Ecology and Evolution, 3(5). doi: 10.1002/ece3.500

    1. Author response:

      The following is the authors’ response to the original reviews.

      We thank the reviewers for their constructive comments on our manuscript and their appreciation of the results. We provide point-by-point responses bellow. For your convenience we highlight here the main changes to the manuscript.

      ·        More descriptive terminology for the contextual cues (Ctx.A / Ctx.noA is now referred to as LIGHT / DARK).

      ·        Schematic of experiment timeline highlighting the exclusion of non-discriminators following the initial acquisition period. This explains the absence of baseline sex differences post acquisition and clears up some misconceptions about lack of replicability.

      ·        New data (time in port preCS) showing that a prior reward does not cause continued presence in port.

      ·        Several text edits to address all the points raised by the reviewers.

      We hope that the editors and reviewers will be satisfied with this revised version and find the strength of the evidence more convincing.

      Reviewer #1 (Recommendations For The Authors):

      In relation to weaknesses points 1-4 in the public review:

      (1) With regards to the claim (page 4 of pdf), I think I can see what the authors are getting at when they claim "Only Ctx-dep.01 engages context-gated reward predictions", because the same reward is available in each context, and the animal must use contextual information to determine which cue will be rewarded. In other words, it has a discriminative purpose. In Ctx-dep.O1/O2, however, although the context doesn't serve a discriminative purpose in the sense that one cue will always earn a unique outcome, regardless of context, the fact that these cues are differentially rewarded in the different context means that animals may well form context-gated cue-outcome associations (e.g. CtxA-(CS1-O1), CtxnoA-(CS2-O2)). Moreover, the context is informative in this group in telling the animal which cue will be rewarded, even prior to outcome delivery, such that I don't think contextual information will fade to the background of the association and attention be lost to it in the way, say Mackintosh (1975) might predict. Therefore, I don't think this statement is correct.

      I suggest that the authors refine the statement to be more accurate.

      We agree with the reviewer —the context is absolutely relevant for rats trained in the Ctx-dep. O1/O2 task. We have edited the text in several places to make this clear. The question is how (by what mechanism) does the context participate in the control of behavior in this group. The reviewer correctly points out that, just like rats trained in the Ctx-dep. O1 task, rats trained in the Ctx-dep. O1/O2 might have formed context-gated cue-outcome associations. We now clearly acknowledge that in the text.

      However, because in this group the two outcomes are always encountered in different contexts, we argue that these rats could also have formed a direct association between the two contexts and the two outcomes. In other words, each context might directly evoke the expectation of a distinct reward outcome (prepare to drink, or prepare to eat). On a given trial, if the cue and context both tend to activate the same outcome representation, the converging cue+context excitation can add up. This would produce a context-sensitive response, but not via hierarchical modulation process (unlike Ctx-dep O1). Arguably, this last associative mechanism is much simpler and might explain why almost all rats in Ctx-dep. O1/O2 group learned the discrimination and at a much faster rate.

      Therefore, while rats trained in Ctx-dep O1/O2 might engage a combination of associative processes to achieve context-sensitive behavior (including hierarchical associations), only rats in the Ctx-dep O1 critically and unambiguously rely on hierarchical associations to achieve context-sensitive behavior.

      (2) I think the results shown in Figure 1 are very interesting, and well supported by the statistics. It's so nice to see a significant interaction, as so many papers try to report these types of effects without it. However, I do wonder how specific the results are to contextual modulation. That is, should a discriminative discrete cue be used instead of each context (e.g. CS1 indicates CS2 earns O1, CS3 indicates CS4 earns O1), would female rats still be as slow to learn the discrimination?

      I am just curious as to whether the authors have thoughts on this.

      We have not tested this and are not aware of a paper that examined this question specifically.

      However, we would like to point out that in the suggested design (CS1→[CS2→O1]; CS3→[CS4→O1]) the discriminative cues (CS1 and CS3) would almost certainly also acquire substantial reward-predictive value, either because of their direct association with the reward, or via second-order conditioning. This would complicate the interpretation of the results in terms of hierarchical associations. Incorporating non-rewarded presentation of CS1 and CS3 alone (i.e. extinguishing those cues, as is sometimes done in occasion setting experiments) would be one way to reduce the reward expectation evoked by those cues, but this approach has some limitations. Indeed, as mentioned by Rescorla (2006) “During extinction, the net associative strength of a stimulus declines to the level of [a response] threshold, but further decrement stops at that point”. So while extinguished CS1 and CS3 might no longer evoke overt behavioral responses, these cues could retain nonnegligible subthreshold excitatory connection with the US.  Individually, these cues might fail to evoke responding but could nonetheless increase responding during the CS1→CS2 trials (or CS3→CS4 trials), via simple summation. (Rescorla, 2006: “the compound of two [extinguished] stimuli has a strength that exceeds the threshold and so evokes responding”).

      This type of consideration is precisely why we opted for the behavioral task used in the study. In Ctx-dep. O1, the discriminative stimuli exert opposite effects on the two target cues, which rules out summation effects as a mechanism for context-sensitive behavior.

      (3) Pages 8-9 of pdf, where the biological basis or the delayed acquisition of contextual control in females is considered, I find this to be written from a place of assuming that what is observed in the males is the default behaviour. That is, although the estrous cycle and its effects on synaptic plasticity/physiology may well account for the results, is there not a similar argument to be made for androgens in males? Perhaps the androgens also somehow alter synaptic plasticity/physiology, leading to their faster speed, reduced performance stability, and increased susceptibility to stress.

      I would like the argument that female behaviour might be the default, and male behaviour the deviation to be considered in the discussion in addition to those already stated.

      We regret if we gave the impression that male behavior was the default. The paper is intended to report sex differences but we don’t view either sex as the default. To correct this impression, we have added a few sentences in the discussion to highlight male-hormonal factors as well as non-gonadal genetic factors that might have contributed to the observed sex differences.

      (4) In addition, the OFC - which is the brain region found to have differential expression of c-fos in males and females in Figure 5 - is not explicitly discussed with regard to the biological mechanisms of differences, which seems odd.

      I suggest OFC be discussed with regard to biological mechanisms of differences.

      We added a few sentences in the discussion to i) highlight the parallel between our study and human fMRI studies showing superior OFC activation in females during the regulation of emotional responses, ii) Suggest a potential relationship between the reported sex differences (speed of acquisition, robustness of performance, and OFC activation in context-gated reward prediction), iii) acknowledge our ignorance of the root causes of these sex differences.

      We wish we could offer a better answer. We have attempted to offer possible proximal explanations for the observed sex differences, but ultimately our work did not address the root causes of these behavioral and neural sex differences. Therefore we feel that further attempts to explain these differences would be too speculative.

      (5) I did wonder if the authors were aware that in the Rescorla-Wagner model, contextual stimuli are thought to summate with discrete cues to enter into the association with the outcome (i.e., the error term is between lambda and sigmaV, with sigmaV the 'summation' of all stimuli present on a trial, including contextual stimuli). Typically, this is not considered much, because the cue itself is so salient and more consistently paired with reward (whereas the ever-present context is often paired with no reward), but nevertheless, it is a part of the association. I'm not sure it's wrong to say that the background circumstances under which events occur are thought to play little role (as in the second sentence of the introduction), but I was wondering if the authors were aware of this fact when they wrote that.

      This sentence in the introduction was meant to introduce the distinction between eliciting stimuli and modulating contexts. Admittedly, this paints a naive picture, which we now acknowledge (we hope that the rest of the paper provides more nuance). As pointed out by this reviewer, the context is also a stimulus, and, just like any other stimulus, it is eligible for direct association with an outcome. The possibility for direct context→outcome association is precisely the rational for the Ctx-dep O1/O2 group.

      (6) Context-noA - Seems a little confusing for a name, why not just call it context B? NoA appears to imply that nothing happens in A or no outcome is available, whereas this is not always the case.

      We debated which terminology to use. We felt that “Context A vs. Context B” should perhaps be reserved to situations where the global context changes (e.g. two different conditioning boxes with different odors, floor texture etc., with proper counterbalancing procedures). We felt that “Context A vs noA” might be more appropriate here, as we are manipulating the local context by introducing (or removing) one single stimulus (the houselight). In this revised version we followed this reviewer’s advice and adopted a more descriptive, and hopefully less confusing, terminology: "Light vs Dark”.

      (7) Why is it that in the text the Ctx-dep O1/O2 is explained before simple and no discrimination, but in the Figure Ctx-dep O1/O2 is shown last? These should be consistent.

      Thanks for pointing that out. We have switched the order of task description to be consistent with the figures.

      (8) Page 6 (of pdf) - could the authors elaborate a little on why or how (or both) the delivery of reward can interfere with the expression of context-dependent discrimination? Do they just mean the performance of discrimination (e.g., animals will sit at the food port longer if there is food there because they are sitting there and eating it, which does not necessarily reflect the expectation of food based on cue presentations?), in which case it is not the discrimination itself that is being interfered with, just the measure of it. Perhaps the authors could elaborate by just inserting a sentence.

      We have added a few sentences to discuss this effect.

      The first clarification that we can make is that the reduced discrimination performance following reward is not simply due to animals’ continued presence in the reward port. We have added the time pre-cue to Fig. 3 B-F. This measure is not affected by previous reward history, showing that rats are leaving the port between trials.

      So what is driving this effect? At this stage, we are agnostic about the mechanism(s) for this effect. Kuchibhotla et al. (2019) —who first reported a similar effect— proposed a model in which recent rewards modify the threshold for behavioral responses (i.e. performance). In this model, a cue might evoke a weak reward prediction but evoke a strong behavioral response if presented after a reward. Additionally, we believe that learning factors might also contribute to the effect reported here. Indeed, the behavioral response on a given trial likely reflects the balance of hierarchical (context-dependent) associations vs. direct associations (Bradfield and Balleine, 2013). Naturally, this balance is dynamic and influenced by trial history. For instance, a Light:X+ trial might increase the value of cue X and promote responding during the following Dark:X- trial. The same logic could be applied to the influence of the context (e.g., Light:X+ trial might promote responding to a subsequent Light:Y- trial). We are currently working on a computational model that captures the dynamic interplay between hierarchical associations and direct associations. We hope that this model will provide some insight into the learning/performance mechanism for the effects reported here. However this computational work is still in the early stages and beyond the scope of the present study.

      (9) The lack of effect in the Ctx-dep O1/O2 groups in Figure 4 could be due to a lack of power - the group sizes are a lot smaller for this group than for Ctx-dep O1 where an interaction was detected. I think this should be at least addressed in the discussion (i.e., that this lack of effect is possibly due to less power here, as the effects are in the same direction).

      Good point. We now acknowledge this limitation in the text.

      Reviewer #2 (Recommendations For The Authors):

      (1) Please comment on the failure to replicate the sex differences across experiments. Perhaps this is due to some change in the training procedure that is briefly mentioned in the methods (a reduction in the number of rewarded trials) but it is unclear.

      The reviewer correctly observed that Fig. 3-5 do not show sex differences in baseline condition. This is not because of a replication failure, but because non-discriminating subjects were excluded from the experiment at the end of the acquisition period (after 72 training sessions). We now clarify this in the Method and Results section. We also added a schematic of the experiment timeline that highlights the exclusion of non-discriminators at the end of the acquisition period (Fig 1).

      On the topic of replicability, the data for Ctx-dep O1 was collected over 3 cohorts (over the course of 2 years) and the sex difference pattern was consistent.  For instance, the proportion of discriminators vs. non-discriminators for males and females trained in Ctx-dep O1, showed similar patterns across cohorts (see below).

      Author response table 1.

      (2) The design of this experiment makes it possible to analyse whether there is a differential outcome effect (DOE). The DOE would indeed predict better discrimination in group cxt-dep O1/O2 versus cxt-dep O1, which seems to be exactly what the authors observe although between-group statistics are not reported. Inspection of Figure 1 suggests that there may be a DOE in females but not in males. I wonder if the authors might consider reanalysing the data to check this.

      Indeed, there is clearly a differential outcome effect. We now point out this DOE in relation to the latency to achieve discrimination criterion (Fig. 2 C-D). Rats in the Ctx-dep. O1/O2 group acquired discrimination (reached criterion) much faster than rats in in the Ctx-dep. O1 group.

      Following the reviewer’s suggestion, we provide here the results of targeted ANOVAs (focusing exclusively on Ctx-dep. O1 and Ctx-dep. O1/O2) to investigate a potential sex-dependent effect of DOE (i.e. Sex x Task interactions), see figure below. A three-way ANOVA (Sex x Task x Session) conducted on the discrimination index reveal a main effect of Task (F1, 86 \= 173.560, P < 0.001), Session (F2.678, 230.329 \= 140.479, P<0.001) and a marginal effect of Sex (F1,86 = 3.929, P = 0.051), but critically no Task x Sex or Task x Sex x Session interaction (P ≥ 0.504). A two-way ANOVA (Sex x Task) conducted on the sessions to criterion revealed a main effect of both factors (Sex F1, 63 = 9.52, P = 0.003; Task F1, 62 = 184.143, P < 0.001) but critically, no Sex x Task interaction (P = 0.233).  These results indicate that the use of two different outcomes clearly facilitated the acquisition of context-dependent discrimination (DOE effect), but this effect benefited both sexes equally. We thank the reviewer for recommending this analysis.

      Author response image 1.

      Differential outcome effect (DOE) affects males and females equally. A. Discrimination ratio over the acquisition period. B. trials to criterion. Compared to animals trained with a single outcome (Ctx-dep. O1), the introducing dissociable outcomes for the two type of rewarded trials (Ctx-dep. O1/O2) profoundly facilitated the acquisition of discriminated behavior. This effect benefited both sexes equally.

      (3) Some minor points for clarification that the authors may also wish to address:

      - Figure 3: is data presented from sessions 71-80 only or for all sessions? I didn't fully follow the explanation offered in the results section.

      That’s right. The data presented in Fig. 3 considers only sessions 71-80, in discriminator rats —when performance is globally stable. We have edited the text to make this clearer. These 10 sessions represent a total of 800 trials (=10 session * 80 trials). The first trial of a session what not included in the analysis since it was not preceded by any trial. For the remaining 790 trials (10 session x 79 trials), we examined how the outcome of the past trial (reward or nonrewarded) influenced responding on the next trial.  This large sample size (790 trials / rat) was required to ensure that enough data was collected for each possible trial history scenario.

      - The authors argue that females are protected from the disrupting effect of stress. It might be useful if the authors offer further explanation as to what they mean by "protected".

      By “protected”, we simply mean “less sensitive”. We have reworded this sentence in that way. We do not claim to have an understanding of the precise mechanism for this sex dependent effect (although our data point to a possible role of the OFC).

      - The authors state that "delivery of reward, while critical for learning, can also interfere with the expression of context-dependent discrimination". This statement should be explained in further detail. For instance, why should reward delivery specifically impair context-dependent discrimination but not other forms of discrimination?

      We have reworded this sentence to be more inclusive. Indeed, delivery of reward also interferes with other forms of discrimination, particularly when discrimination performance is not yet optimal. We have also added a paragraph to discuss the possible mechanisms by which reward might interfere with discrimination performance in our task.   

      Reviewer #3 (Recommendations For The Authors):

      I do not suggest additional experiments, but I do hope you continue the behavioral work to characterize what is being learned in the task. I think the approach is promising. I would suggest reporting the % time in port and port entries for the entire CS. There is no justification for only analyzing the response in the last 5s.

      We thank the reviewer for the encouragement.

      We opted to focus on the time in port for two main reasons:

      (1) This measure is relatively consistent across the two different reward outcomes (unlike the rate of port entries). Indeed, consistent with prior studies (Delamater et al., 2017), we observed that the type of reward (solid or liquid) influences the topography of the anticipatory magazine-directed behavior. Specifically, cues paired with pellets elicited significantly more port entries than cues paired with chocolate milk. The opposite pattern was observed for time in port --cues paired with chocolate milk elicited more sustained time in port compared to cues paired with pellets (see figure below). While these measures (port entries and time in port) show opposite bias for the two possible outcomes, the size of this bias is much smaller for the time in port (Cohen’s d effect size: port entries: 1.41; time in port: 0.62). As a result, the discrimination ratio calculated from Time in port is consistent across the two outcomes (P = 0.078; effect size: 0.07), which is not the case for the discrimination ratio calculated from port entries (P = 0.007; effect size 0.32 see figure below).

      (2) Unlike the rate of port entries, the time in port shows monotonic increase during training in these tasks. Indeed, we observed here and in past work (Keiflin et al., 2019), that the rate of port entries initially increases with training, but then slightly decreases; particularly for cues paired with liquid reward. In contrast, the time in port continues to increase, or remains high, with extended training. This is easy to understand if we consider the extreme case of a hypothetical rat that might enter the port once upon cue presentation and maintain continued presence in port for the whole cue duration. This rat would have a relatively low rate of port entry (a single port entry per trial) but a high time in port.

      This is not to say that the rate of port entries is not a valid measure overall (we have used, and continue to use, this metric in other preparations). However, for the reasons explained above, we believe that the time in port is a better metric for reward anticipation in this specific study.

      Moreover, we chose to focus our analysis on the last 5s of the cue because that’s when anticipatory food cup behavior is more reliably observed (in our preparation >2/3 of the total time in port in occurs during the last 5s of the cue) and less contaminated by orienting behaviors (Holland, 1977, 1980, 2000). For these reasons, analysis of the last portion of the cue is relatively common in Pavlovian anticipatory approach preparations (El-Amamy and Holland, 2007; Olshavsky et al., 2013; Esber et al., 2015; Holland, 2016a, 2016b; Schiffino and Holland, 2016; Gardner et al., 2017; Sharpe et al., 2021; Maes et al., 2020; Sharpe et al., 2020; Siemian et al., 2021; Kang et al., 2021). Reporting time in port during the same cue epoch facilitates comparisons between these studies.

      We have edited the text in the Method section to provide a brief justification for focusing our analyses on this cue epoch.

      Author response image 2.

      Outcome identity influences the topography of the conditioned response. A-C: Conditioned responding expressed as the number of port entries per trial (A) or time in port per trials (C) for rats trained in the simple discrimination task with a chocolate milk reward (n= 19) or a sucrose pellet (n = 16). Data show the average of the last three 3 sessions. Compared to chocolate milk, pellets tend to produce more port entries. Conversely, chocolate milk tend to produce more time in port. However the magnitude of this bias is smaller for the Time in port. C-D: discrimination ratio calculate from the number of port entries (C) or the time in port (D); the latter is not affected by the outcome identity. *P<0.05; **P<0.01; ***P<0.001 T tests.

      The inconsistent use of terms is distracting throughout the paper. Is it discriminated or context-gated? Please provide a definition of your terms and then use them consistently. Is it a discriminative stimulus, a context, or an occasion setter? These all imply slightly different things and it would help the reader if you just used one term throughout the paper.

      Thanks for pointing that out. We have added a definition for “context-gated” and edited the text to keep the terminology consistent when appropriate. The words “discrimination”/”discriminated” still appear in the manuscript but without implying a mechanism (all tasks are variations of Pavlovian discrimination; the rats discriminating between rewarded and non-rewarded trials).

      As mentioned by this reviewer, the terms “context” and “occasion setter” are not synonymous. Therefore these terms still appear in the manuscript to refer to different concepts (e.g. in our task the visual stimulus is a context for all rats; this context acts as an occasion setter only for some rats).

      Minor:

      Intro, 2nd PP: "autism". This is abbreviated in the abstract but spelled out here. I suggest not abbreviating in the abstract and introducing abbreviations here, as you do with PTSD.

      Fixed as suggested

      Have deficits in contextual modulation been distinguished from potential deficits in binary associative learning in autism, PTSD, and substance use disorders? This is implied, but there are no citations provided.

      We provide a list of references showing deficits in contextual modulation in these disorders.

      This does not mean that these disorders are reducible to deficits in contextual modulation and it does not exclude other forms of deficits in those disorders --including alterations in certain aspects of binary associative learning.

      "In positive occasion-setting, animals learn that a target cue (X) results in a reward outcome (+) only when that cue is accompanied by a contextual feature (A); the same cue presented in absence of this contextual feature remains without consequence (A:X+ / X-)." - there are words missing in this sentence.

      We apologize but we fail identify the missing word(s). Perhaps the reviewer could be more specific and we will be happy to edit the sentence as needed.

      What is a contextual feature, is this redundant or can you provide a specific definition?

      We use the terminology “feature” and “target” as these are the standard terms in the description of occasion setting preparations (one stimulus, “the feature”, sets the occasion for responding –or not responding- to the “target” cue). By contextual feature, we meant that in this specific example the context was the feature. We have clarified this in the text. We believe that these terms are not redundant. Indeed, the context is not always a feature, and a feature is not necessarily a context (phasic cues can serve as “features”).

      Can you provide some background on studies of sex differences in simple associative learning? You imply these have been much more thoroughly studied than conditional discriminations.

      We added a few references as suggested.

      What is the rationale for studying stress?

      Stressful life events exacerbate several mental illnesses, potentially by impacting cognitive functions.

      Although the (sex-dependent) effects of stress on some cognitive function are well established (e.g. working memory, selective attention, spatial navigation), the effect of stress on contextual modulation (a core dysfunction in certain mental illnesses) --and the possible sex-differences in this effect-- had not been formally tested. We added a few sentences in the results section (at the beginning of the stress section) to remind the reminder of why we tested the effect of stress in this task.

      Method/Results:

      Cues are not counterbalanced; the feature is visual and targets are auditory - this should be noted as a limitation in the discussion section.

      We now acknowledge this limitation in the discussion. Moreover we believe that the new terminology for the context —Light vs Dark— (instead of A vs. noA in the original version) makes it abundantly clear that the “context” is this study was always visual.

      Summation is invoked to describe the discrimination with different outcomes, how is summation happening? This is not described. Perhaps incorporate the literature on conditional discriminations with differential outcomes (the "differential outcomes effect").

      We have edited the Result + Discussion section to clarify how summation might contribute to discrimination with different outcomes. We have also added references for the DOE in this task.

      The stress effect is confounded with test order; comparing stress vs. baseline.

      Sorry we don’t understand this point. The “baseline” refers to the animal’s performance on the last training session before the acute stress manipulation (we have edited the text to make this clear). Animals are first trained in the task and then we examine how stress alters their performance in this learned task. We don’t see how this could induce a test order confound.

      Throughout the results section, it would be helpful to have the number of animals reported for each analysis.

      The number of animals for each part of the experiment is now reported in the text, as well as in the figures.

      Discussion:

      "For Ctx-dep. O1, context is an occasion-setter, i.e. a stimulus that hierarchically modulates the associative strength between a target cue and its outcome." This is inaccurate. Occasion setters do not change or modulate the associative strength of a target cue. They modulate whether excitation or inhibition is expressed.

      We reworded the sentence as suggested: “For Ctx-dep. O1, context is an occasion-setter, i.e. a stimulus that modulates the response to a target cue”.

      "Together, these results indicate that the sex differences observed here are not attributable to simple associative, motivational, working-memory, or attentional processes, but are specific to the neurocomputational operations required for the hierarchical, contextual control of behavior." It should be noted here that the difference is one of degree, a quantitative difference, but not a difference in the qualitative features of the process.

      "Regardless of the precise mechanism, our results indicate that, compared to male rats, females ultimately achieved more stable contextual control over cued reward-seeking; their behavior remained context-regulated under stress or after recent rewards." Again this is a matter of degree.

      We absolutely agree. All the sex-difference reported here are a matter of degree. In the framework of McCarthy et al. (2012) the reported effects are type 2 or type 3 sex differences, not type 1 sexual dimorphism. We made a few edits in the Discussion to clarify this point.

      Procedure:

      Please clarify the percentage of trials that were reinforced in the No Discrimination group.

      From session 1-32 (acquisition period), 50% of the trials were reinforced. Following this acquisition period, only 25% of the trials were reinforced to match all the other groups. We have edited the method section to clarify this point.

      Please provide the dimensions of the restraint tubes and the model number if available.

      This information is now included.

      References

      Bradfield LA, Balleine BW (2013) Hierarchical and binary associations compete for behavioral control during instrumental biconditional discrimination. J Exp Psychol Anim Behav Process 39:2–13.

      Delamater AR, Garr E, Lawrence S, Whitlow JW (2017) Elemental, configural, and occasion setting mechanisms in biconditional and patterning discriminations. Behav Processes 137:40–52.

      El-Amamy H, Holland PC (2007) Dissociable effects of disconnecting amygdala central nucleus from the ventral tegmental area or substantia nigra on learned orienting and incentive motivation. Eur J Neurosci 25:1557–1567.

      Esber GR, Torres-Tristani K, Holland PC (2015) Amygdalo-striatal interaction in the enhancement of stimulus salience in associative learning. Behav Neurosci 129:87–95.

      Gardner MPH, Conroy JS, Shaham MH, Styer CV, Schoenbaum G (2017) Lateral Orbitofrontal Inactivation Dissociates Devaluation-Sensitive Behavior and Economic Choice. Neuron 96:1192–1203.e4.

      Holland PC (1977) Conditioned stimulus as a determinant of the form of the Pavlovian conditioned response. J Exp Psychol Anim Behav Process 3:77–104.

      Holland PC (1980) CS-US interval as a determinant of the form of Pavlovian appetitive conditioned responses. J Exp Psychol Anim Behav Process 6:155–174.

      Holland PC (2000) Trial and intertrial durations in appetitive conditioning in rats. Anim Learn Behav 28:121–135.

      Holland PC (2016a) Enhancing second-order conditioning with lesions of the basolateral amygdala. Behav Neurosci 130:176–181.

      Holland PC (2016b) Effects of amygdala lesions on overexpectation phenomena in food cup approach and autoshaping procedures. Behav Neurosci 130:357–375.

      Kang M, Reverte I, Volz S, Kaufman K, Fevola S, Matarazzo A, Alhazmi FH, Marquez I, Iordanova MD, Esber GR (2021) Agency rescues competition for credit assignment among predictive cues from adverse learning conditions. Sci Rep 11:16187.

      Keiflin R, Pribut HJ, Shah NB, Janak PH (2019) Ventral tegmental dopamine neurons participate in reward identity predictions. Curr Biol 29:93–103.e3.

      Kuchibhotla KV, Hindmarsh Sten T, Papadoyannis ES, Elnozahy S, Fogelson KA, Kumar R, Boubenec Y, Holland PC, Ostojic S, Froemke RC (2019) Dissociating task acquisition from expression during learning reveals latent knowledge. Nat Commun 10:2151.

      Maes EJP, Sharpe MJ, Usypchuk AA, Lozzi M, Chang CY, Gardner MPH, Schoenbaum G, Iordanova MD (2020) Causal evidence supporting the proposal that dopamine transients function as temporal difference prediction errors. Nat Neurosci 23:176–178.

      McCarthy MM, Arnold AP, Ball GF, Blaustein JD, De Vries GJ (2012) Sex differences in the brain: the not so inconvenient truth. J Neurosci 32:2241–2247.

      Olshavsky ME, Song BJ, Powell DJ, Jones CE, Monfils M-H, Lee HJ (2013) Updating appetitive memory during reconsolidation window: critical role of cue-directed behavior and amygdala central nucleus. Front Behav Neurosci 7:186.

      Rescorla RA (2006) Deepened extinction from compound stimulus presentation. J Exp Psychol Anim Behav Process 32:135–144.

      Schiffino FL, Holland PC (2016) Secondary visual cortex is critical to the expression of surprise-induced enhancements in cue associability in rats. Eur J Neurosci 44:1870–1877.

      Sharpe MJ, Batchelor HM, Mueller LE, Gardner MPH, Schoenbaum G (2021) Past experience shapes the neural circuits recruited for future learning. Nat Neurosci 24:391–400.

      Sharpe MJ, Batchelor HM, Mueller LE, Yun Chang C, Maes EJP, Niv Y, Schoenbaum G (2020) Dopamine transients do not act as model-free prediction errors during associative learning. Nat Commun 11:106.

      Siemian JN, Arenivar MA, Sarsfield S, Borja CB, Russell CN, Aponte Y (2021) Lateral hypothalamic LEPR neurons drive appetitive but not consummatory behaviors. Cell Rep 36:109615.

    1. Author response:

      The following is the authors’ response to the original reviews.

      Public Reviews:

      Reviewer #1 (Public review):

      Summary:

      Participants in this study completed three visits. In the first, participants received experimental thermal stimulations which were calibrated to elicit three specific pain responses (30, 50, 70) on a 0-100 visual analogue scale (VAS). Experimental pressure stimulations were also calibrated at an intensity to the same three pain intensity responses. In the subsequent two visits, participants completed another pre-calibration check (Visit 2 of 3 only). Then, prior to the exercise NALOXONE or a SALINE placebo-control was administered intravenously. Participants then completed 1 of 4 blocks of HIGH (100%) or LOW (55%) intensity cycling which was tailored according to a functional threshold power (FTP) test completed in Visit 1. After each block of cycling lasting 10 minutes, participants entered an MRI scanner and were stimulated with the same thermal and pressure stimulations that corresponded to 30, 50, and 70 pain intensity ratings from the calibration stage. Therefore, this study ultimately sought to investigate whether aerobic exercise does indeed incur a hypoalgesia effect. More specifically, researchers tested the validity of the proposed endogenous pain modulation mechanism. Further investigation into whether the intensity of exercise had an effect on pain and the neurological activation of pain-related brain centres were also explored.

      Results show that in the experimental visits (Visit 2 and 3), when participants exercised at two distinct intensities as intended. Power output, heart rate, and perceived effort ratings were higher during the HIGH versus LOW-intensity cycling. In particular. HIGH intensity exercise was perceived as "hard" / ~15 on the Borg (1974, 1998) scale, whereas LOW intensity exercise was perceived as "very light" / ~9 on the same scale.

      The fMRI data from Figure 1 indicates that the anterior insula, dorsal posterior insula, and middle cingulate cortex show pronounced activation as stimulation intensity and subsequent pain responses increased, thus linking these brain regions with pain intensity and corroborating what many studies have shown before.

      Results also showed that participants rated a higher pain intensity in the NALOXONE condition at all three stimulation intensities compared to the SALINE condition. Therefore, the expected effect of NALOXONE in this study seemed to occur whereby opioid receptors were "blocked" and thus resulted in higher pain ratings compared to a SALINE condition where opioid receptors were "not blocked". When accounting for participant sex, NALOXONE had negligible effects at lower experimental nociceptive stimulations for females compared to males who showed a hyperalgesia effect to NALOXONE at all stimulation intensities (peak effect at 50 VAS). Females did show a hyperalgesia effect at stimulation intensities corresponding to 50 and 70 VAS pain ratings. The fMRI data showed that the periaqueductal gray (PAG) showed increased activation in the NALOXONE versus SALINE condition at higher thermal stimulation intensities. The PAG is well-linked to endogenous pain modulation.

      When assessing the effects of NALOXONE and SALINE after exercise, results showed no significant differences in subsequent pain intensity ratings.

      When assessing the effect of aerobic exercise intensity on subsequent pain intensity ratings, authors suggested that aerobic exercise in the form of a continuous cycling exercise tailored to an individual's FTP is not effective at eliciting an exercise-induced hypoalgesia response irrespective of exercise intensity. This is because results showed that pain responses did not differ significantly between HIGH and LOW intensity exercise with (NALOXONE) and without (SALINE) an opioid antagonist. Therefore, authors have also questioned the mechanisms (endogenous opioids) behind this effect.

      Strengths:

      Altogether, the paper is a great piece of work that has provided some truly useful insight into the neurological and perceptual mechanisms associated with pain and exercise-induced hypoalgesia. The authors have gone to great lengths to delve into their research question(s) and their methodological approach is relatively sound. The study has incorporated effective pseudo-randomisation and conducted a rigorous set of statistical analyses to account for as many confounds as possible. I will particularly credit the authors on their analysis which explores the impact of sex and female participants' stage of menses on the study outcomes. It would be particularly interesting for future work to pursue some of these lines of research which investigate the differences in the endogenous opioid mechanism between sexes and the added interaction of stage of menses or training status.

      There are certainly many other areas that this article contributes to the literature due to the depth of methods the research team has used. For example, the authors provide much insight into: the impact of exercise intensity on the exercise-induced hypoalgesia effect; the impact of sex on the endogenous opioid modulation mechanism; and the impact of exercise intensity on the neurological indices associated with endogenous pain modulation and pain processing. All of which, the researchers should be credited for due to the time and effort they have spent completing this study. Indeed, their in-depth analysis of many of these areas provides ample support for the claims they make in relation to these specific questions. As such, I consider their evidence concerning the fMRI data to be very convincing (and interesting).

      Weaknesses:

      Although the authors have their own view of their results, I do however, have a slightly different take on what the post-exercise pain ratings seem to show and its implications for judging whether an exercise-induced hypoalgesia effect is present or not. From what I have read, I cannot seem to find whether the authors have compared the post-exercise pain ratings against any data that was collected pre-exercise/at rest or as part of the calibration. Instead, I believe the authors have only compared post-exercise pain ratings against one another (i.e., HIGH versus LOW, NALOXONE versus SALINE). In doing so, I think the authors cannot fully assume that there is no exercise-induced hypoalgesia effect as there is no true control comparison (a no-exercise condition).

      In more detail, Figure 6A appears to show an average of all pain ratings combined per participant (is this correct?). As participants were exposed to stimulations expected to elicit a 30, 50, or 70 VAS rating based on pre-calibration values, therefore the average rating would be expected to be around 50. What Figure 6A shows is that in the SALINE condition, average pain ratings are in fact ~10-15 units lower (~35) and then in the NALOXONE condition, average pain ratings are ~5 units lower (~45) for both exercise intensities. From this, I would surmise the following:

      It appears there is an exercise-induced hypoalgesia effect as average pain ratings are ~30% lower than pre-calibrated/resting pain ratings within the SALINE condition at the same temperature of stimulation (it would also be interesting to see if this effect occurred for the pressure pain).

      It appears there is evidence for the endogenous opioid mechanism as the NALOXONE condition demonstrates a minimal hypoalgesia effect after exercise. I.e., NALOXONE indeed blocked the opioid receptors, and such inhibition prevented the endogenous opioid system from taking effect.

      It appears there is no effect of exercise intensity on the exercise-induced hypoalgesia effect.

      That is, participants can cycle at a moderate intensity (55% FTP) and incur the same hypoalgesia benefits as cycling at an intensity that demarcates the boundary between heavy and severe intensity exercise (100%FTP). This is a great finding in my mind as anyone wishing to reduce pain can do so without having to engage in exercise that is too effortful/intense and therefore aversive - great news! This likely has many applications within the field of public health.

      I will very slightly caveat my summaries with the fact that a more ideal comparison here would be a control condition whereby participants did the same experimental visit but without any exercise prior to entering the MRI scanner. I consider the overall strength of the evidence to be solid, with the answer to the primary research question still a little ambiguous.

      Reviewer #2 (Public review):

      Summary:

      This interesting study compared two different intensities of aerobic exercise (low-intensity, high-intensity) and their efficacy in inducing a hypoalgesic reaction (i.e. exercise-induced hypoalgesia; EIH). fMRI was used to identify signal changes in the brain, with the infusion of naloxone used to identify hypoalgesia mechanisms. No differences were found in postexercise pain perception between the high-intensity and low-intensity conditions, with naloxone infusion causing increased pain perception across both conditions which was mirrored by activation in the medial frontal cortex (identified by fMRI). However, the primary conclusion made in this manuscript (i.e. that aerobic exercise has no overall effect on pain in a mixed population sample) cannot be supported by this study design, because the methodology did not include a baseline (i.e. pain perception following no exercise) to compare high/low-intensity exercise against. Therefore, some of the statements/implications of the findings made in this manuscript need to be very carefully assessed.

      Strengths:

      (1) The use of fMRI and naloxone provides a strong approach by which to identify possible mechanisms of EIH.

      (2) The infusion of naloxone to maintain a stable concentration helps to ensure a consistent effect and that the time course of the protocol won't affect the consistency of changes in pain perception.

      (3) The manipulation checks (differences in intensity of exercise, appropriate pain induction) are approached in a systematic way.

      (4) Whilst the exploratory analyses relating to the interactions for fitness level and sex were not reported in the study pre-registation, they do provide some interesting findings which should be explored further.

      Weaknesses:

      (1) Given that there is no baseline/control condition, it cannot be concluded that aerobic exercise has no effect on pain modulation because that comparison has not been made (i.e. pain perception at 'baseline' has not been compared with pain perception after high/lowintensity exercise). Some of the primary findings/conclusions throughout the manuscript state that there is 'No overall effect of aerobic exercise on pain modulation', but this cannot be concluded.

      (2) Across the manuscript, a number of terms are used interchangeably (and applied, it seems, incorrectly) which makes the interpretation of the manuscript difficult (e.g. how the author's use the term 'exercise-induced pain').

      (3) There is a lack of clarity on the interventions used in the methods, for example, it is not exactly clear the time and order in which the exercise tasks were implemented.

      (4) The exercise test (functional threshold power) used to set the intensity of the low/high exercise bouts is not an accurate means of demarcating steady state and non-steady state exercise. As a result, at the intensity selected for the high-intensity exercise in this study, it is likely that the challenge presented for the high-intensity exercise would have been very different between participants (e.g. some would have been in the 'heavy' domain, whereas others would be in the 'severe' domain).

      (5) It is likely that participants did not properly understand how to use the 6-20 Borg scale to rate their perceived effort, and so caution must be taken in how this RPE data is used/interpreted.

      (6) Although interesting, the secondary analyses (relating to the interaction effects of fitness level and sex) were not included in the study pre-registration, and so the study was not designed to undertake this analysis. These findings should be taken with caution.

      Recommendations for the authors:

      Reviewer #1 (Recommendations for the authors):

      Participants in this study completed three visits. In the first one, participants received experimental thermal stimulations which were calibrated to elicit three specific pain responses (30, 50, 70) on a visual analogue scale (VAS). Experimental pressure stimulations were also calibrated at an intensity to the same three pain intensity responses. In the subsequent two visits, participants completed another pre-calibration check (Visit 2 of 3 only). Then, prior to the exercise NALOXONE or a SALINE placebo-control was administered intravenously. Participants then completed 1 of 4 blocks of HIGH (100%) or LOW (55%) intensity cycling which was tailored according to a functional threshold power (FTP) test completed in Visit 1. After each block of cycling lasting 10 minutes, participants entered an MRI scanner and were stimulated with the same thermal and pressure stimulations that corresponded to 30, 50, and 70 pain intensity ratings from the calibration stage. Therefore, this study ultimately sought to investigate whether aerobic exercise does indeed incur a hypoalgesia effect. More specifically, researchers tested the validity of the proposed endogenous pain modulation mechanism.

      Further investigation into whether the intensity of exercise had an effect on pain and the neurological activation of pain-related brain centres was also explored.

      Results show that in the experimental visits (Visit 2 and 3) when participants exercised at two distinct intensities as intended. Power output, heart rate, and perceived effort ratings were higher during the HIGH versus LOW-intensity cycling. In particular, HIGH intensity exercise was perceived as "hard" / ~15 on the Borg (1974) scale, whereas LOW intensity exercise was perceived as "very light" / ~9 on the Borg (1974) scale.

      The fMRI data from Figure 1 indicates that the anterior insula, dorsal posterior insula, and middle cingulate cortex show pronounced activation as stimulation intensity and subsequent pain responses increase, thus linking these brain regions with the percept of pain intensity and corroborating what many studies have shown before.

      Results also showed that participants rated a higher pain intensity in the NALOXONE condition at all three stimulation intensities compared to the SALINE condition. Therefore, the expected effect of NALOXONE in this study seemed to occur whereby opioid receptors were "blocked" and thus resulted in higher pain ratings compared to a SALINE condition where opioid receptors were "not blocked". When accounting for participant sex, NALOXONE had negligible effects at lower experimental nociceptive stimulations for females compared to males who showed a hyperalgesia effect to NALOXONE at all stimulation intensities (peak effect at 50 VAS). Females did show a hyperalgesia effect at stimulation intensities corresponding to 50 and 70 VAS pain ratings. The fMRI data showed that the periaqueductal gray (PAG) showed increased activation in the NALOXONE versus SALINE condition at higher thermal stimulation intensities. The PAG is well-linked to endogenous pain modulation.

      When assessing the effects of NALOXONE and SALINE after exercise, results showed no significant differences in subsequent pain intensity ratings.

      When assessing the effect of aerobic exercise intensity on subsequent pain intensity ratings, authors suggested that aerobic exercise in the form of a continuous cycling exercise tailored to an individual's FTP is not effective at eliciting an exercise-induced hypoalgesia response irrespective of exercise intensity. This is because results showed that pain responses did not differ significantly between HIGH and LOW-intensity exercise with (NALOXONE) and without (SALINE) an opioid antagonist. Therefore, authors have also questioned the mechanisms (endogenous opioids) behind this effect.

      Altogether, the paper is a great piece of work that has provided some truly useful insight into the neurological and perceptual mechanisms associated with pain and exercise-induced hypoalgesia. The authors have gone to great lengths to delve into their research question(s) and their methodological approach is relatively sound. Although the authors have their own view of their results, I do however, have a slightly different take on what the post-exercise pain rating seems to show and its implications for judging whether an exercise-induced hypoalgesia effect is present or not. From what I have read, I cannot seem to find whether the authors have compared the post-exercise pain ratings against any data that was collected preexercise/at rest or as part of the calibration. Instead, I believe the authors have only compared post-exercise pain ratings against one another (i.e., HIGH versus LOW, NALOXONE versus SALINE). In doing so, I think the authors cannot fully question whether there is an exerciseinduced hypoalgesia effect as there is no true control comparison (a no-exercise condition). Nevertheless, there are certainly many other areas that this article contributes to the literature due to the depth of methods the research team has used. For example, the authors provide much insight into: the impact of exercise intensity on the exercise-induced hypoalgesia effect; the impact of sex on the endogenous opioid modulation mechanism; and the impact of exercise intensity on the neurological indices associated with endogenous pain modulation and pain processing. All of which, the researchers should be credited for due to the time and effort they have spent completing this study.

      I have provided some specific comments for the authors to consider. They are organised to correspond to each section as it is presented, and I have denoted the line I am referring to each time.

      To conclude, thank you to the authors for their work, and thank you to the editor for the opportunity to contribute to the review of this paper. I hope my comments are seen as useful and I look forward to seeing the authors' responses.

      We sincerely appreciate the reviewer's insightful comments, which highlight the strengths of our study. In response to the concerns raised, we have made several key revisions to the original manuscript to address the reviewers’ comments. As for the lack of a resting control condition, we acknowledge that our study was not designed to test the overall effect of exercise versus no exercise. However, our primary objective was to compare different exercise intensities, hypothesising that low-intensity (LI) exercise would induce less pain modulation as compared to high-intensity (HI) exercise. By exploring this, we aimed to enhance understanding of the dose-response relationship between exercise and pain modulation. To better reflect this focus, we have revised the misleading phrasing regarding the ‘overall’ effect of exercise to clearly emphasize our primary aim: comparing HI and LI exercise.

      This reviewer suggests an interesting interpretation of the data suggesting that exercise induced hypoalgesia might have occurred for both exercise intensities since the pain ratings provided were lower than the anticipated intensities as determined by the calibration. Given that this difference is lower in the naloxone (NLX) condition could provide evidence of opioidergic mechanisms underlying this effect. Unfortunately, the current study is not designed to comprehensively answer this question since there was no resting control condition. In particular, the lower pain ratings under SAL (Figure 6) could be due to exercise triggering the descending pain modulatory system (DPMS), but equally due to the default activation of the DPMS. Only an additional “no exercise” condition could disentangle this. Furthermore, habituation to noxious stimuli can influence pain ratings, resulting in lower pain ratings during the experiment as compared to the calibration. We have now provided a more detailed overview of the pain ratings at different stimulus intensities after HI and LI exercise in both drug treatment conditions for heat and pressure pain ratings. We elaborated on the specific comments raised in more detail in the following sections.

      Specific Comments

      (1) Abstract

      Line 25 - "we were unable to"... personal preference but this wording is a little 'weighted' in my view. I personally do not think researchers search to prove hypotheses correct, rather we search to prove hypotheses wrong, and therefore only through repeated attempts of falsification can we surmise that something holds true.

      We agree with the reviewer that the chosen wording can be perceived as weighted and have rephrased the sentence.

      Line 33 to 35 - the "...but individual factors... might play a role" is a crucial caveat to this sentence for me. Whilst I can understand that the results of the authors' study indicate that prior assumptions about exercise-induced hypoalgesia and its opioidergic mechanisms may be questioned, I think a little more evidence is needed to finally decide whether aerobic exercise has no overall effect on experimental pain responses. (see more in the Results comments below).

      We thank the reviewer for their comment. We agree that no claims can be made regarding the effect of aerobic exercise per se on pain modulation compared to no exercise based on the current data. Furthermore, we agree that more research is needed to further advance our understanding of (non-)opioidergic mechanisms in exercise-induced pain modulation. However, based on the data presented in this study we propose that the involvement of endogenous opioids in exercise-induced hypoalgesia could be influenced by sex and fitness levels since we could show differences in opioidergic involvement between males and females of different fitness levels. Future studies should account for the fitness levels and sex of the sample investigated.

      (2) Introduction

      Line 48 - please predefine anterior cingulate cortex here.

      We thank the reviewer for detecting this and have introduced the abbreviation for the anterior cingulate cortex in the referenced line.

      Line 49 - please predefine periaqueductal gray here instead of line 52.

      We have introduced the abbreviation for periaqueductal grey in the referenced line.

      Line 47 to 54 - when discussing the descending pain modulatory systems, authors seem to be relating specifically to the intensity/magnitude of pain experiences. However, the different brain regions that are mentioned may have varying "roles" according to which dimension of pain is of focus.

      Hofbauer et al. (2001) - https://doi.org/10.1152/jn.2001.86.1.402

      Rainville et al. (1997) - https://doi.org/10.1126/science.277.5328.968

      The two above studies provide some nice earlier findings on the brain regions - some of which are mentioned by the authors in this section - associated with the processing of pain quality in addition to the intensity of pain... simply attach here if they are of interest to the authors.

      The studies by Hofbauer et al. (2001) and Rainville et al. (1997) provide interesting findings on the effect of hypnotic suggestions on pain affect and the perceived intensity of a painful stimulus. However, these studies did not investigate exercise-induced changes in brain regions of the DPMS. The studies referenced in the relevant section of the manuscript are (one of the few) imaging studies that have indeed investigated brain structures of the DPMS in the context of exercise and pain modulation and, thus, were included in this paragraph to focus on the findings of these studies as well as emphasise the scarcity of imaging studies investigating exercise-induced pain modulation. Given these divergent research topics of the proposed studies, we suggest not including them in this paragraph to maintain a clearer line of argument and focus on exercise-induced pain modulation in brain regions of the DPMS.

      L59 to 61 - a minor comment about the phrasing within this sentence and a recommended change is provided below for the flow of the sentence/paragraph.

      "...there are instances where administration of µ-opioid antagonists has decreased exerciseinduced pain modulation (Droste et al. 1988; etc.) whereas in others there has been little effect (Droste et al. 1988; etc.).

      We have altered the sentence based on the reviewers' suggestions to improve the flow and coherence of the sentence.

      L56 to 72 - Whilst the current version of this paragraph scans well enough, I find that the narrative flits between the mechanisms being discussed and the rationale/shortcomings of current research. I think that the original content of this paragraph can be structured into:

      A- The endogenous opioid system is a likely candidate to explain how exercise elicits a hypoalgesia response.

      B- Citation(s) of the imaging studies (Boecker et al., 2008, etc.) and earlier literature which support A (e.g., Janal et al. 1984).

      C- Further support of this theory as µ-opioid antagonists like naloxone seem to counteract the endogenous opioid effect (Haier et al., 1981).

      D- Introduction of the caveats of previous research such as the studies that observed that µ-opioids did not impact the endogenous pain modulation system during exercise (e.g., Droste et al., 1991, etc.) and the range of different interventions and exercise modalities which make it difficult to draw clear conclusions of the pain modulation effect.

      To me, this structure would set out the details you have already put together in a more orderly and systematic way and also will lead nicely into your ensuing paragraph (Line 74 onwards).

      We appreciate the reviewers' constructive comments on structuring this paragraph. We agree that the proposed version eases the readability and comprehension of the paragraph and have, thus, adapted the restructured paragraph according to the reviewer’s suggestion.

      L75 - Why are single-arm pre-post measures and designs an issue? If you can elaborate a little more this would be very insightful for a reader.

      Single-arm pre-post measurement studies involve participants being assigned to a single experimental condition, with pain assessments conducted only once before and once following an intervention. This study design presents some limitations, particularly in the context of examining exercise-induced modulation of pain (Vaegter and Jones, 2020). Such designs are potentially confounded by the effects of habituation to noxious stimuli, as highlighted by Vaegter and Jones (2020). Incorporating randomised controlled trials with multiple measurement blocks not only mitigates these limitations but also provides a clearer understanding of how individual bouts of exercise influence pain perception. We have now added this to the paper.

      L80 - The reference for the functional threshold power assessment is provided as a number. Please could the authors change to reflect which study/studies they are referring to here (I presume it is the Borszcz and/or the McGrath studies?).

      We apologise for this oversight and have now updated the reference to be displayed correctly. The reviewer is correct in assuming that Borszcz et al. (2018) is the referenced study here.

      L88 - Did participants also receive pressure pain stimulations in addition to the thermal stimuli, as the figure suggests?

      Note Since read on to L102-104 and understood why pressure pain was included but not mentioned due to results. However, I would still recommend including pressure pain stimulations in this line, if possible, to be consistent with what Figure 1 shows and later text in the Methods section also shows.

      We thank the reviewer for their suggestion to mention pressure pain at the referenced line to increase the clarity and consistency of the experimental paradigm. Pressure and heat pain were applied in alternating fashion during scanning. Whilst the results of pressure pain are not included in this study we agree with the reviewer that it should be mentioned again as part of the methods and have added this.

      L94 - I really like Figure 1. Great job.

      Could the authors please define the inter-trial interval (ITI) in the legend? And please could the authors clarify what unit the 30, 50, and 70 figures in the "18 trials per block" section refer to.

      We thank the reviewer for their positive feedback. We have now included a definition of inter-trial-interval (ITI) in the figure legend. Furthermore, we adapted Figure 1 so that the units of the stimulus intensities (30, 50, 70) on the Visual Analog Scale (VAS) are included in the figure allowing for a clearer identification.

      (3) Results

      General comment for figures ... is there a specific reason the authors chose for error bars to be represented by an SE value as opposed to an SD value?

      The reason I ask is that participant responses seem to vary (See Figure 2A and 2E-G as an example). Error bars showing SD values would perhaps do justice to the variability in participant response(s), whereas the SE may be a better representation of the variability in responses due to the assessor's methods of collection. Whilst the SE error bars are narrow (great job on that!), the individual responses are clearly varied which I speculate could be because of the interventions that have been implemented (i.e., exercise intensity).

      The use of Standard Error (SE) is more common in the cognitive neuroscience literature.

      However, as this reviewer noted, we have also included individual data points alongside the SE, thereby providing a comprehensive view that allows for a thorough interpretation of the data distribution.

      L102 to 104 - In fact, it is interesting that exercise did not impact the pressure pain ratings whereas the same cannot be said for thermal pain. In line with some of my comments below about the impact of exercise on pain intensity responses, I would be intrigued to see the results of the pressure pain ratings in more detail.

      Another note on this... Whilst the results for the pressure pain may be beyond the scope of this paper and will be reported separately, knowing of this data is tantalising for a reader. I would suggest to: A) either mention the pressure pain and include the analysis of the data; or B) not mention the pressure pain altogether and save it for the subsequent paper. Either way, I look forward to seeing further discussion on this in future work.

      We have now summarised the behavioural results of exercise on pressure pain ratings below in Supplemental Figure S1.

      There was no hypoalgesic effect evident in the behavioural pain ratings comparing HI to LI exercise in the saline (SAL) condition (β = 0.57, CI [-1.73, 2.86], SE = 1.17, t(1354) = 0.48, P = 0.63; Supplemental Figure S1A, blue bars) as well as no interaction of drug treatment and exercise intensity on pressure pain ratings (β = -1.43, CI [-4.87, 2.01], SE = 1.75, t(2756.02) = -0.82, P = 0.42; Supplemental Figure S1). Post-hoc paired t-tests (Bonferroni-corrected) confirmed there to be no significant differences between the drug treatment conditions at LI (P = 0.18) or HI (P = 0.85) and no significant difference between the exercise intensities in the SAL (P = 0.65) and NLX (P = 0.48) conditions, confirming no significant differences in drug treatment between the exercise intensities.

      Furthermore, there was no significant effect of fitness level on differences in pain ratings (LI – HI exercise) in the SAL condition (β = 3.16, CI [-1.64, 7.97], SE = 2.37, t(38) = 1.34, P = 0.19; Supplemental Figure S1B) and no significant correlation between fitness level and difference pain ratings (r = 0.25, P = 0.13). Finally, there was no significant interaction of drug treatment, exercise intensity, and sex on difference pain ratings (β =-7.97, CI [-18.67, 2.73], SE = 5.51, t(190) = -1.45, P = 0.15; Supplemental Figure S1C-D).

      Exercise did not appear to affect pressure pain ratings and we have now added this to the discussion and in the methods section. However, we think that the figure should be part of the supplements.

      L112 to 113 - Fantastic work for including this analysis in your study. Great job.

      We appreciate the reviewers’ positive feedback on conducting these crucial analyses when investigating sex and gender differences in pain.

      L186 to 189 - It is fascinating that there appears to be no effect of NALOXONE on pain ratings within female participants at a VAS rating of 30 for thermal pain as well as a much diminished hyperalgesia effect at a VAS rating of 50 compared to males. Meanwhile, at higher intensity stimulations corresponding to a VAS rating of 70, females in fact demonstrate a more pronounced hyperalgesia effect compared to males. In addition, the hyperalgesia effect of NALOXONE for males seems to "peak" at a VAS rating of 50. The mechanisms behind these findings alone would be incredibly exciting to explore... but maybe in another study.

      We agree with the reviewer that the differences in males and females are fascinating results and concur that this may hint at varying degrees of opioidergic involvement at different stimulus intensities. This finding is intriguing and potentially clinically relevant, warranting further investigation in future research, although it lies beyond the scope of the current paper.

      L189 - To double check... Figures 4A and 4B refer to the entire cohort (male and female responses combined) whereas C-E are separated by sex?

      In addition, as there are no annotations to the top of Figures 4C-E were no significant differences observed between saline and naloxone conditions per each stimulus intensity? i.e., similar tests to what are shown in Table S6 but separated for each sex.

      Without getting too carried away, there may be something here that indicates a difference between sexes concerning the opioid-driven pain modulation response on a neurological level (i.e., brain region activation).

      The reviewer is correct in assuming that Figures 4A and 4B refer to the entire cohort whilst Fig. 4C – 4E are split for males and females. The full output of the analyses for Fig. 4A and 4B are reported in Supplemental Tables S5 – S7. Furthermore, the full output of the LMER analyses for Fig. 4E is reported in Supplemental Table S10. We agree with the reviewer that additional annotations in Fig. 4C – Fig. 4E ease interpretation and have, thus, added them to the respective figures, denoting the significance of the interaction term stimulus intensity and drug treatment for females (Fig. 4C) and males (Fig. 4D), respectively. For completeness, we now report the post-hoc paired samples t-tests for females and males in the Supplemental Tables S8 and S9, respectively.

      L254 to 258 - "we could not establish an overall hypoalgesia effect of exercise...". Do the results of the exercise intensity x drug treatment provide an answer for this exact hypothesis? After checking the methods section, I cannot seem to find whether the statistical analysis has involved a comparison of the pain ratings after the high (alone), low (alone), or high and low (combined) exercise compared to ratings during control or pre-calibration as part of precalibration (i.e., pain ratings in a rested state without any exercise yet completed).

      We concur with the reviewer's assessment that the study design and statistical analyses cannot address the ‘overall’ effect of exercise compared to no exercise. Please refer back to our general response before comment 1, where we have addressed this point.

      As it seems that the analysis assesses the differences between high and low-intensity exercise, to me, the results of the exercise intensity x drug treatment analysis do not assess whether there is an exercise-induced hypoalgesia effect or not. Instead, it seems to assess whether the intensity of exercise is a differentiating factor in the expected exercise-induced hypoalgesia effect to subsequent pain intensity ratings to experimental pain stimulation. For the authors to judge whether aerobic exercise does or does not have a hypoalgesia effect, then the exercise conditions (either combined or standalone) would have to be compared to a control condition or a data set that involved pain ratings from a pre-exercise timepoint.

      We thank the reviewer for their comment. We would like to point out the we concluded there to be no hypoalgesic effect between the LI and HI exercise based on the LMER model comparing the behavioural pain ratings between the exercise conditions in the SAL condition (β = 1.19, CI [-1.85, 4.22], SE = 1.55, t(1354) = 0.77, P = 0.44; Figure 6A, blue bars and Table S9). The statistical model investigating the interaction of exercise intensity and drug treatment served to show that NLX did not modulate pain differently between the LI and HI exercise conditions.

      Given that our experiment involved different exercise levels in a randomized order, a simple pre vs post analysis is not straightforward. Nevertheless, we have set up a model where we take into account the rating time point (pain ratings provided before each exercise block (prepain ratings) and following each exercise block (post-pain ratings)) at each stimulus intensity (VAS 30, 50, 70) and exercise intensity (LI and HI). The model also takes into account the exercise intensity performed in the previous block, the overall block number as well as the varying subject intercepts. The analysis was completed for heat (Author response image 1A) and pressure (Author response image 1B) pain ratings in the SAL condition to establish whether there was a significant effect of exercise intensity on the changes from pre to post-pain ratings. The model for heat pain yielded a significant main effect for stimulus intensity (β = 1.43, CI [1.34, 1.52], SE = 0.05, t(2054.95) = 31.61, P < 0.001) but no significant interaction of exercise intensity, rating time point, and stimulus intensity (P = 0.14). The model for pressure pain in the SAL condition yielded a significant main effect of stimulus intensity (β = 1.00, CI [0.92, 1.08], SE = 0.04, t(2054.99) = 24.68, P < 0.001) and block number (β = 1.14, CI [0.35, 1.94], SE = 0.41, t(2055.98) = 2.80, P = 0.005) but not interaction of exercise intensity, rating time point, and stimulus intensity (P = 0.38).

      Author response image 1.

      Heat (A) and Pressure (B) pain ratings in the saline (SAL) condition for pre (purple) and post (turquoise) exercise pain ratings at LI and HI exercise and all stimulus intensities (VAS 30, 50, 70). The bars depict the mean pain rating pre and post-exercise and the dots depict the subject-specific mean ratings. The error bars depict the SEM.

      Another point of consideration is that Figure 6A appears to show an average of all pain ratings combined per participant (is this correct?). As participants were exposed to stimulations expected to elicit a 30, 50, or 70 VAS rating based on pre-calibration values, therefore the average rating would be expected to be around 50. What Figure 6A shows is that in the SALINE condition, average pain ratings are in fact ~10-15 units lower (~35) and then in the NALOXONE condition, average pain ratings are ~5 units lower (~45) for both exercise intensities. From this, I would surmise the following:

      • It appears there is an exercise-induced hypoalgesia effect as average pain ratings are ~30% lower than pre-calibrated/resting pain ratings within the SALINE condition at the same temperature of stimulation (it would also be interesting to see if this effect occurred for the pressure pain).

      • It appears there is evidence for the endogenous opioid mechanism as the NALOXONE condition demonstrates a minimal hypoalgesia effect after exercise. I.e., NALOXONE indeed blocked the opioid receptors, and such inhibition prevented the endogenous opioid system from taking effect.

      • It appears there is no effect of exercise intensity on the exercise-induced hypoalgesia effect. That is, participants can cycle at a moderate intensity (55% FTP) and incur the same hypoalgesia benefits as cycling at an intensity that demarcates the boundary between heavy and severe intensity exercise (100%FTP). This is a winner in my mind. Anyone wishing to reduce pain can do so without having to engage in exercise that is too effortful and therefore aversive - great news!

      I will very slightly caveat my summaries with the fact that a more ideal comparison here would be a control condition whereby participants did the same experimental visit but without any exercise prior to entering the MRI scanner.

      As a result of this interpretation of your findings, I do not think that aerobic exercise as a means to cause subsequent hypoalgesia to experimental thermal nociception can be fully discounted. On the contrary, I think your results showed in Figure 6A are evidence for it.

      The reviewer is correct in assuming that Figure 6A shows the averaged pain ratings across all stimulus intensities (VAS 30, 50, and 70) for each subject. To provide more details, we have split Figure 6A by stimulus intensity, now depicting the pain ratings for LI and HI exercise and treatment condition (SAL and NLX) at VAS 30, 50, and 70 (Supplemental Fig. S8). The LMER was extended to include the stimulus intensity and yielded a significant main effect of stimulus intensity (β = 1.39, CI [1.31, 1.47], SE = 0.04, t(2753.12) = -34.082, P < 0.001) and a significant interaction of stimulus intensity and drug treatment (β = 0.12, CI [0.01, 0.24], SE = 0.06, t(2751) = 2.13, P = 0.03) but no significant interaction of exercise intensity, drug treatment, and stimulus intensity (β = -0.05, CI [-0.20, 0.11], SE = 0.08, t(2751) = -0.56, P = 0.58).

      The reviewer further suggests that the average pain ratings in the SAL condition are lower than the anticipated stimulus intensity, thus, indicating exercise-induced hypoalgesia. While this interpretation is one possibility, there is an alternative explanation: the lower pain ratings may stem from habituation to heat pain (Greffrath et al., 2007; Jepma et al., 2014; May et al., 2012). To support this perspective, we have visualised data from other studies in our lab that have been conducted with the same thermode head and device (TSA-2), using the same calibration procedure and aiming for the same stimulus intensities (VAS 30, 50, and 70). In both studies (Author response image 2A: Study 1: Behavioural sample; Author response image 2B: Study 2: fMRI sample; Author response image 2C: Original Exercise Study), participants did not engage in an exercise task and the pain ratings at VAS 30 and VAS 50 were lower than the anticipated intensities (VAS 30: 11.1/13.4; VAS 50: 35.0/35.9). Furthermore, in a previous study by (Wittkamp et al., 2024), the authors showed that, despite calibrating the heat stimuli at VAS 60, participants rated the pain stimuli with M = 48.58 (SD = 13.79).

      This discrepancy observed between calibrated intensities and ratings provided could be attributable to habituation effects, especially at low-intensity stimuli. Moreover, we would like to point the reviewer to the highest stimulus intensity at VAS 70 (Author response image 2C), where no habituation in all three data sets (including the current study) has taken place. This consistency suggests that exercise-induced hypoalgesia may not be present in our findings or potentially confounded by habituation effects.

      Author response image 2.

      Heat pain ratings at different intensities (30, 50, and 70 VAS) in different study samples. Bars depict the mean ratings in the saline (SAL) condition. Individual data points depict subject-specific mean pain ratings. Error bars depict the SEM.

      The reviewer further suggests that there is evidence for endogenous opioidergic modulation since the pain ratings in the NLX condition are lower than the anticipated intensities. We fully agree but, again, would argue that the DPMS can exert its effects on painful stimuli in a default manner, i.e. irrespective of any exercise effect.

      We concur with the reviewer’s interpretation that there is no effect of exercise intensity on exercise-induced hypoalgesia since the ratings between both exercise intensities are not significantly different.

      Finally, we agree that our data does not allow for the interpretation of an ‘overall’ effect of exercise-induced hypoalgesia and would like to point out that we did not aim to claim this. Rather, the data suggests there to be no effect of LI vs. HI aerobic exercise on pain modulation. We acknowledge, however, that the phrasing involving ‘overall’ can be misleading and have revised this to focus on the comparison between LI and HI exercise, thereby enhancing precision and clarity.

      Note This is also where it would be really interesting to see the pain pressure data if it were to be included. Mainly to see whether it coheres with what the thermal stimulation stuff shows.

      We have provided the ratings for the pressure pain ratings in the SAL condition below (Author response image 3).

      Author response image 3.

      Pressure pain ratings in the SAL condition at stimulus intensity (VAS 30, 50, and 70). Bars depict the mean ratings in the saline (SAL) condition. Individual data points depict subject-specific mean pain ratings. Error bars depict the SEM.

      L259 - As mentioned in the comment above. Could the authors distinguish what is being shown in Figure 6A? Are the data presented as the pooled mean for all stimulation intensities? If not, what data is displayed per bar/column?

      We thank the reviewer for their comment. The reviewer is correct in assuming that the bars in Figure 6A depict the pooled means across all stimulus intensities (VAS 30, 50, 70) for each drug treatment condition and exercise intensity. To allow for a more detailed comprehension of the data, we have split Figure 6A by stimulus intensity, now depicting the pain ratings for LI and HI exercise and treatment condition (SAL and NLX) at VAS 30, 50, and 70 (Supplemental Figure S8). The LMER was extended to include the stimulus intensity and yielded a significant main effect of stimulus intensity (β = 1.39, CI [1.31, 1.47], SE = 0.04, t(2753.12) = -34.082, P < 0.001) and a significant interaction of stimulus intensity and drug treatment (β = 0.12, CI [0.01, 0.24], SE = 0.06, t(2751) = 2.13, P = 0.03) but no significant interaction of exercise intensity, drug treatment, and stimulus intensity (β = -0.05, CI [-0.20, 0.11], SE = 0.08, t(2751) = -0.56, P = 0.58).

      L278 - Can the authors please provide a reference that explains how W.kg-1 at FTP is a measure of fitness level?

      We thank the reviewer for their comment. The obtained FTP value was corrected for the weight of each participant (Watt/kg), yielding a weight-corrected fitness measure that allows for better comparison between subjects. We denoted this in the figures as W*kg-1 which serves to be the equivalent term.

      L296 - Take the line away from Figure 7A... Does the individual data show a positive relation between pain rating changes and W.kg-1? Besides the three data points (1 on the far right of the figure and the two on the far left), I find it hard to see any real trend.

      We acknowledge the reviewers’ concern regarding the regression line and the visual clarity of the individual data points. However, it is important to note that the significant main effect of fitness level on differences in pain ratings in the SAL condition (β = 6.45, CI [1.25, 11.65], SE = 2.56, t(38) = 2.52, P = 0.02) supports the assertion that higher fitness levels are associated with greater hypoalgesia following HI exercise compared to LI exercise. While the trend may not be visible for all data points, the statistical analysis provides a robust basis for the observed relationship (r = 0.33, P = 0.038).

      We have conducted an additional LMER model where we have excluded the subjects with the highest and lowest FTP values (sub-28 with 3.19 W/kg and sub-06 with 0.76 W/kg, respectively.) The LMER still yields a significant main effect of fitness level (β = 6.82, CI [1.25, 11.65], SE = 3.18, t(34) = 2.14, P = 0.039; Author response image 4) and a positive correlation between the difference ratings and fitness level approaching significance (r = 0.32, P = 0.057).

      Author response image 4.

      Fitness level on difference pain ratings (LI-HI exercise) without subjects with highest and lowest FTP (N = 37). (A) Subject-specific differences in heat pain ratings (dots) between LI and HI exercise conditions (LI – HI exercise pain ratings) and corresponding regression line pooled across all stimulus intensities in the SAL condition. Fitness level (FTP) showed a significant positive relation to heat pain ratings with a significant main effect of FTP (P = 0.039) on difference ratings.

      (4) Discussion

      L356 to 358 - Exactly. What you write here, I agree with. Your testing allowed you to judge whether there is an effect of aerobic exercise intensity on pain modulation. However, I think this has been a little conflated with the idea that there is "no overall effect of aerobic exercise on pain modulation" in other areas of the article (L358-361, Results, and Abstract). As per my previous comment, I am not sure this (no overall effect) is true.

      We agree with the reviewer and have adapted the manuscript so that the misleading phrase including ‘overall’ is removed.

      L358 to 365 - One addition to this debate about whether this is a hypoalgesia effect of aerobic exercise. In 358 - 361 (particularly the end of 361) there is a strong conclusion that there is no direct involvement of the endogenous opioid system. Then glance onto L364 to 365 and there is then an almost conflicting summary that a hypoalgesia effect driven by opioidergic regions of the brain (and ergo endogenous opioids) is in effect. If there were no direct endogenous opioid involvement, then differences between NALOXONE (blockade of the opioid mechanism) and SALINE conditions would not exist.

      We thank the reviewer for their comment. The structure of this paragraph aimed to guide the reader towards a more nuanced understanding of the possible mechanisms and caveats in exercise-induced pain modulation. Whilst our data suggest an effect of NLX on pain ratings where we showed significantly higher pain ratings in the NLX condition compared to the SAL condition we could not identify an interaction between treatment and exercise intensity. This suggests that there is no significant difference in opioidergic involvement between HI and LI exercise. Our exploratory analyses, however, show an effect of endogenous opioids involved as an underlying mechanism dependant on sex and fitness level.

      My perspective is that an exercise-induced hypoalgesia effect has occurred (based on the data in Figure 6A) but that this effect is certainly caveated by the sex and fitness levels that this study has observed (and kudos for it).

      As mentioned above, based on the current data we cannot untangle whether the reduced pain ratings in the SAL condition are due to habituation to noxious stimuli or an actual hypoalgesic effect of exercise (or potentially a mix of both). However, we fully agree with the reviewer that exercise-induced pain modulation is influenced by fitness level and sex.

      L390 - "endogenous pain modulation through μ-opioid receptors increases with increasing pain intensity". Aside from the general discussion about whether aerobic exercise causes a post-exercise hypoalgesia effect. This finding is also interesting for the pain incurred during exercise in the form of naturally occurring muscle pain and may also be clinically relevant as it could be that the endogenous pain modulation "system" could be primed through repeated exercise as your results show that the fitness level (i.e., a close correlate of how much someone has engaged in exercise and therefore 'activated' the endogenous pain modulation system) is associated with a more pronounced post-exercise hypoalgesia effect.

      This is an interesting aspect. With regards to the pain induced by exercise itself (i.e. muscle pain) we did not gather any data on this type of pain and interpreting this would be mere speculation. However, it is an interesting hypothesis to investigate in future studies whether the pain induced by exercise is potentially influenced by the endogenous opioid system. We agree with the reviewers’ interpretation that repeated exercise might prime the endogenous opioid system, especially in fitter individuals who engage more frequently in exercise and, thus, ‘train’ the endogenous opioid system. We have included this line of interpretation in the original manuscript, where we suggest that the mFC, a brain region with high µ-opioid receptor density, might be ‘trained’ by repeated exercise and, therefore, shows increase activation in fitter individuals after short bouts of exercise.

      L404 to 405 - "a resting baseline does not control for unspecific factors such as attentional load or distraction (Brooks et al., 2017; Sprenger et al., 2012) through exercise." I am not sure I agree. A control condition allows one to truly deduce whether exercise causes a hypoalgesia effect or not. The attentional load may be a factor, but I would argue this is distinct from endogenous pain modulation - unless there is a study that shows cognitive load alone can elicit endogenous opioids like exercise. About distraction, this would be the case if the pain measures were taken during the exercise. However, as the pain measures taken in the MRI were post-exercise and there was no added distraction related to the exercise present anymore, then I do not think any added effect of distraction due to the exercise and its effect on postexercise pain measure is relevant any longer.

      We agree with the reviewer that a resting baseline condition in the context of exercise induced pain modulation would allow for the investigation of a potential hypoalgesic effect of exercise compared to no exercise. It is important to note that both studies (Brooks et al., 2017; Sprenger et al., 2012) have indeed shown that the effect of cognitive pain modulation is mediated by endogenous opioids.

      L406 - I do not think a low-intensity exercise is a true "control" condition. It certainly does allow the study to compare the dose-response relationship but as the individual is exercising (even at a moderate physiological intensity) then comparison of HIGH vs LOW does not tell us whether exercise does or does not cause hypoalgesia. In contrast, the results from Figure 6A seem to show that even LOW intensity exercise has a hypoalgesia effect and this is a good thing for those who cannot exercise at high intensities (e.g., chronic populations).

      Please refer back to our general response before comment 1, where we have addressed this point.

      L410 - A small digression in relation to the exercise intensities:

      The intensity domains (moderate - heavy - severe) are not truly controlled within this study (mainly for the LOW condition), and therefore some participants could have exercised within different exercise intensity domains than others. To explain, the exercise intensity domains are distinguishable by the physiological responses associated with the boundaries of each of these domains. The FTP is believed to be a demarcation point between heavy and severe intensity domains (though kinesiologists debate the validity of this). Other concepts similar to FTP are Critical Power or the Respiratory Compensation Point. Ultimately, the boundary between heavy and severe intensity domains is characterised by the highest possible intensity by which a steady-state in oxygen kinetics (V̇ O2) occurs (Burnley & Jones, 2018). If this is expressed as a power output (Watts) and then a percentage of this power output is used to prescribe exercise intensity, then the physiological response is not always as expected. The reason is that for some people the gaseous exchange threshold (the demarcation point between the moderate and heavy intensity domains) is not always the same percentage between resting and FTP/Critical Power/Respiratory Compensation Point for each person. As a result, some individuals who are prescribed an intensity of 55% FTP/Critical Power/Respiratory Compensation Point may subsequently exercise within the moderate intensity domain (most people did based on the heart rate and RPE responses) whilst some others might actually exercise more within the heavy intensity domain. A quick check of Figures 3B-C could indicate that this might have been the case for two or three participants, but that is inference and speculation as we cannot truly know unless gas parameters were taken (which is perfectly understandable that they have not been taken because this study has done so much else). However, the importance of this for this study is that if some participants did indeed exercise at a slightly higher physiological intensity, this undermines the LOW condition as a "control" as the physiological stimulus between conditions (Brownstein et al., 2023). It means that the proposed differences in endogenous opioids (Vaegter et al., 2015; 2019) between exercise intensities may not have been present and therefore summarising a lack of an exercise induced hypoalgesia effect is slightly confounded. This is one factor contributing to my scepticism about the conclusion that there is a lack of an exercise-induced hypoalgesia response.

      We thank the reviewer for their comment as it touches upon the challenges of estimating exercise intensities precisely. It is, indeed, crucial to consider the boundaries between moderate, heavy, and severe intensity domains, as delineated by physiological markers such as the Functional Threshold Power (FTP), Critical Power, and the Respiratory Compensation Point (VO2max) (Burnley & Jones, 2018). Previous research has shown that the FTP and FTP20 tests are reliable and convenient methods to estimate approximate measures of VO2max (Denham et al., 2020) and that the FTP test is a useful test for performance prediction in moderately trained cyclists (Sørensen et al., 2019).

      We acknowledge that without direct measurements of VO2max, it is challenging to determine the precise intensity domain in which each participant was operating. While the RPE and HR might suggest that some participants performed in the moderate intensity domain in the LI exercise condition, we could still ascertain there to be a significant difference in the relative power (%FTP), heart rate (HR), and rating of perceived exertion (RPE) between the LI and HI exercise conditions. In the overall sample, the consistency in relative power, heart rate, and RPE responses among participants suggests that the exercise doses were effectively communicated and adhered to; therefore, the validity of the LI exercise condition remains robust.

      While we did not include metabolic assessments in our protocol, our study focused on providing a comprehensive analysis of the exercise-induced hypoalgesia phenomenon across two distinct exercise intensities. Additionally, the rationale for selecting specific exercise intensities was grounded in the existing literature, which indicates significant differences in the hypoalgesic response between exercise intensity levels (Jones et al., 2019; Vaegter et al., 2014).

      According to the reviewer, the potential lack of difference between the exercise conditions might contribute to the fact that there was no difference in endogenous opioid release and, thus, no difference in pain ratings between the exercise conditions. However, our data still suggests that there is an influence of endogenous opioids in the HI exercise condition in males with higher fitness levels. Together with recent findings on the association of µ-opioid receptor activation and fitness levels in men (Saanijoki et al., 2022), as well as the difference in µ-opioid receptor availability between high and moderate aerobic exercise (Saanijoki et al., 2018), we would hypothesise that the release of endogenous opioids after short HI bouts of exercise depend on fitness levels (and potentially sex).

      Finally, we propose that discussing exercise intensity domains within the context of our study enriches the understanding of exercise-induced hypoalgesia without undermining the integrity of our findings. We have, therefore, included this in the discussion of the manuscript.

      L417 - For some reason I am doubting this value (r = 0.61). Could this be checked? I think it is higher in their study. r = 0.88?

      Also, as someone with a kinesiology background, I would argue this is a given anyway. The maximum power one can cycle for 20 minutes is related to the maximum power one can cycle for 60 minutes, this is expected. (That is no slight on the authors of this study, more a remark that readers could look and figure that for themselves if they needed to know).

      We thank the reviewer for their comment. We have carefully re-checked the correlation coefficient between the FTP20 and FTP60 tests in the study by Borsczc et al. (2018) and have corrected the correlation coefficient to r = 0.88. We thank the reviewer for detecting this. Whilst we agree that it seems somehow intuitive that the FTP20 and FTP60 should correlate highly, we wanted to provide the reader with a better understanding of where the FTP20 tests originated from and how it is suitable to assess aerobic fitness levels without having to maintain a steady power output for 60 minutes.

      L428 - Kudos to the authors for taking a standardised approach to this. Hopefully, my comment earlier might provide some extra food for thought about exercise intensity. I think there are several other ways future research could prescribe exercise without the need for expensive and cumbersome bits of equipment to know how hard people are exercising.

      We strongly agree with the reviewer and hope that our study can inspire future research to implement more convenient and inexpensive ways to establish aerobic (and anaerobic) fitness levels.

      L456 to 458 - Would it be possible to revisit this and check whether the pooled mean of all stimulation intensities for pain intensity ratings after pressure pain is lower than 50? If so, I think it can also be assumed that there is a slight hypoalgesia effect occurring for pressure pain too.

      We have revisited the pressure pain ratings pooled across all stimulus intensities (VAS 30,50, and 70). Indeed, the ratings are below 50 VAS (Supplemental Figure S1A) in the SAL and NLX conditions. As mentioned before lower pain ratings after LI exercise cannot be taken as evidence for exercise-induced analgesia.

      L495 to L499 - I find this fascinating. Great finding.

      We thank the reviewer for their positive feedback.

      (5) Methods

      L650 - "Watts"

      We have changed the sentence accordingly.

      L651 - beats per minute can also be represented as b.min-1 and cadence as revolutions.min-1.

      To allow for easier interpretation of the results in a broader readership we would like to propose to maintain the original abbreviations.

      L678 - Just to check what the authors mean by "on the second experimental day", they are actually referring to Visit 2 of 3 (first experimental visit of 2) as it is shown in Figure 1?

      We apologise for the lack of clarity. Indeed, the second experimental day refers to the third visit in the study. We have added this to the sentence to increase clarity.

      L708 - would change the end of the sentence to "and remained blinded throughout the study"

      We have changed the sentence accordingly.

      L742 - comma after "in one participant".

      We have added the missing comma.

      L746 - slight mistype... RPE in brackets instead of PRE

      We have changed the abbreviation to RPE.

      L747 - In case the authors are interested in affective measures in future studies... Hardy and Rejeski (1989) have a 9-point Likert scale rating affective valence which might be useful to check out.

      Thank you. The scale by Hary and Rejeski (1989) is a very relevant measure of affective valence during exercise, and we will consider this in future studies.

      L755 - Four squares for the thermode to be applied were drawn on the arm but through the methods I can only seem to see that the thermode was applied to the second square during calibration. During the MRI scan, did someone move the thermode to different squares for different stimulations?

      We appreciate the reviewers' question. Indeed, the heat calibration and recalibration on the first and second day, respectively, have always been completed on the same skin patch (patch 2) to allow for comparability of calibration across days. During the experimental sessions, the thermode head was repositioned in a randomised order across participants (i.e., skin patch 14-3-2) before each block. This was done manually before the MRI block commenced. The order of thermode head position was kept constant within participants across experimental days (day 2 and day 3).

      L764 - ITI predefined?

      We thank the reviewer for their comment and would like to point to line 130 in the revised manuscript where the abbreviation for inter-trial-interval (ITI) was first introduced.

      (6) Other Sections + Supplementary Materials

      L891 - I apologise in advance for this comment as it is the most trivial comment you will ever receive, but there is an extra "." On this line after J.N. initials for methodology.

      We have changed the punctuation accordingly.

      Table S1 - Strictly speaking, some of the intensity denominations in this table are not exactly an "intensity".

      Iannetta et al. (2020) - https://doi.org/10.1249/mss.0000000000002147 provides a commentary on intensity domains as well as Burnley and Jones (2018) - https://doi.org/10.1080/17461391.2016.1249524

      Likewise in this table - the term "without fatigue" in the description column is not strictly true as participants will naturally fatigue but authors are referring more to a "steady state".

      We have changed the name of the column to ‘Description’ to describe the test phase as proposed by Allen and Coggen (2012) and previously implemented by McGrath et al. (2019) and not the ‘intensity domains’ (as specified by Iannetta et al. (2020)). Further, we have refined the wording in Table S1 and replaced the term ‘without fatigue’ with ‘steady state’.

      Once again, thank you to the authors for their great work on this project and to the editor for the chance to review this paper.

      We would like to thank this reviewer for their very insightful and important comments and for pointing out the strengths of the manuscript. We believe the suggestions will help to improve the quality of the manuscript.

      Reviewer #2 (Recommendations for the authors):

      Summary:

      This interesting study compared two different intensities of aerobic exercise (low-intensity, high-intensity) and their efficacy in inducing a hypoalgesic reaction (i.e. exercise-induced hypoalgesia; EIH). fMRI was used to identify signal changes in the brain, with the infusion of naloxone used to identify hypoalgesia mechanisms. No differences were found in postexercise pain perception between the high-intensity and low-intensity conditions, with naloxone infusion causing increased pain perception across both conditions which was mirrored by activation in the medial frontal cortex (identified by fMRI). However, the primary conclusion made in this manuscript (i.e. that aerobic exercise has no overall effect on pain in a mixed population sample) cannot be supported by this study design, because the methodology did not include a baseline (i.e. pain perception following no exercise) to compare high/low-intensity exercise against. Therefore, some of the statements/implications of the findings made in this manuscript need to be very carefully assessed.

      Strengths:

      (1) The use of fMRI and naloxone provides a strong approach by which to identify possible mechanisms of EIH.

      (2) The infusion of naloxone to maintain a stable concentration helps to ensure a consistent effect and that the time course of the protocol won't affect the consistency of changes in pain perception.

      (3) The manipulation checks (differences in intensity of exercise, appropriate pain induction) are approached in a systematic way.

      (4) Whilst the exploratory analyses relating to the interactions for fitness level and sex were not reported in the study pre-registation, they do provide some interesting findings which should be explored further.

      Weaknesses:

      (1) Given that there is no baseline/control condition, it cannot be concluded that aerobic exercise has no effect on pain modulation because that comparison has not been made (i.e. pain perception at 'baseline' has not been compared with pain perception after high/low intensity exercise). Some of the primary findings/conclusions throughout the manuscript state that there is 'No overall effect of aerobic exercise on pain modulation', but this cannot be concluded.

      (2) Across the manuscript, a number of terms are used interchangeably (and applied, it seems, incorrectly) which makes the interpretation of the manuscript difficult (e.g. how the author's use the term 'exercise-induced pain').

      (3) There is a lack of clarity on the interventions used in the methods, for example, it is not exactly clear the time and order in which the exercise tasks were implemented.

      (4) The exercise test (functional threshold power) used to set the intensity of the low/high exercise bouts is not an accurate means of demarcating steady state and non-steady state exercise. As a result, at the intensity selected for the high-intensity exercise in this study, it is likely that the challenge presented for the high-intensity exercise would have been very different between participants (e.g. some would have been in the 'heavy' domain, whereas others would be in the 'severe' domain).

      (5) It is likely that participants did not properly understand how to use the 6-20 Borg scale to rate their perceived effort, and so caution must be taken in how this RPE data is used/interpreted.

      (6) Although interesting, the secondary analyses (relating to the interaction effects of fitness level and sex) were not included in the study pre-registration, and so the study was not designed to undertake this analysis. These findings should be taken with caution.

      We thank the reviewer for their insightful comments that contribute to improving the quality of the manuscript. In response to the identified weaknesses, we have made key revisions to enhance clarity and rigor. Regarding the lack of a resting control condition, we acknowledge that our study does not assess the overall effect of exercise versus no exercise. Our primary objective was to compare high- (HI) and low-intensity (LI) exercise on pain modulation, hypothesizing that lower intensities would have minimal effects. We revised the manuscript to eliminate misleading phrases about an "overall" effect, clearly emphasizing our aim to investigate the comparative effects of different exercise intensities. To address terminology inconsistencies, we have adopted "exercise-induced pain modulation," reflecting existing literature that recognizes both hypoalgesia and hyperalgesia associated with exercise (Vaegter and Jones, 2020). We clarified this terminology in the introduction and specified the pain modalities used in our study. We also improved methodological transparency by better describing the timing and order of exercise and drug treatment interventions. Concerning exercise intensity estimation, we acknowledge the complexities in classifying moderate, heavy, and severe domains. We added the study by Wong et al. (2023) to discuss the potential limitations of the FTP estimation protocol. Although direct measures of VO2max or blood lactate are absent in our study, our findings, including perceived exertion (RPE) scores and relative power data, support that participants were primarily in the heavy-intensity domain during HI exercise. To clarify RPE ratings, we adjusted the presentation to align with the Borg scale's intended anchor points, ensuring greater accuracy in reported exertion levels. Statistical analyses confirm significant differences in RPE between exercise intensities. These revisions aim to clarify our intent and methodologies, ultimately strengthening the contribution of our research to understanding exercise-induced pain modulation.

      (1) Lines 27-33 - please present some data and accompanying statistical output in the results section of the abstract.

      We thank the reviewer for their comment. In the results section of the abstract, we report whether the findings are (not) significant using the general threshold of P < 0.05. However, we prefer not to include more detailed data and statistical outputs here, as these are thoroughly presented in the results section and do not contribute to the abstract’s primary purpose of providing a concise summary.

      (2) Line 29 - please indicate how fitness level was quantified.

      The functional threshold power (FTP) adjusted for weight served as an indication of cardiovascular fitness level. We have now included this in the abstract.

      (3) Line 35 - please include a sentence detailing the implications of your findings.

      We have now included a sentence on the implications of our findings in the abstract.

      (4) Introduction general - I appreciate that it was an exploratory analysis, however, the introduction does not particularly lay the groundwork for this (e.g., the influence of fitness level, sex, etc) - please include some background within the introduction to establish the role level of fitness/exercise/training/physical activity on pain modulation.

      A paragraph detailing the role of fitness level and sex in the context of exercise-induced pain modulation and endogenous opioid release was part of the introduction of our manuscript but has been removed as per the reviewing editor’s request (as the inclusion of sex and fitness level was not part of the preregistration). We have now re-included a shortened version of this paragraph to provide some background on these potentially crucial factors in exercise-induced pain modulation.

      (5) Lines 40-41 - reference needed.

      We thank the reviewer for detecting this and have now included references concerning the release of endogenous opioids and the term exercise-induced hypoalgesia.

      (6) Lines 48-49 - please provide the full terms for ACC and PAG (PAG has been provided on line 52, but should be presented earlier).

      We thank the reviewer for detecting this. We now introduce the abbreviations for the periaqueductal grey (PAG) and anterior cingulate cortex (ACC) in the correct lines.

      (7) Line 49 - the term exercise-induced pain is often used interchangeably (incorrectly) with many different types of pain experienced during/after exercise (e.g. muscle burn/ache, DOMS, injury etc.). Please see O'Malley et al 2024 (doi: 10.1113/EP091687).

      We thank the reviewer for their comment. Despite the distinction between different types of pain induced by exercise being important, this is less relevant for the current study. We would like to point out that the full term used is exercise-induced pain modulation, referring to the modulation of (experimental) pain through exercise. We have deliberately chosen this term as it summarises exercise-induced hypoalgesia as well as hyperalgesia. Therefore, we did not refer to pain induced by exercise and would disagree that this term has been used interchangeably with different types of pain in the current manuscript.

      (8) Line 57 - neither of these studies looked at exercise-induced pain, rather they examined experimentally induced pain (e.g. cold pressor test) or chronic pain and how exercise might exacerbate it. This leads back to the previous comment - it is important to define what is meant by exercise-induced pain (EIP) from the offset, and then remain consistent in the reference to this.

      We agree with the reviewer and have cited the studies accordingly. We would like to point out that the current study does not investigate exercise-induced pain but the modulation of experimental pain through exercise and have used the term exercise-induced pain modulation consistently in the manuscript to describe this.

      (9) Line 61 - Droste et al and Olausson et al are missing from the reference list.

      We apologise for this oversight and have now updated the reference list to include the studies by Droste et al. (1991) and Olaussen et al. (1986).

      (10) Line 61 - Do you mean exercise-induced hypoalgesia, or modulation of exercise-induced pain - it is not clear? EIH is introduced in Line 40 and in consistent with what the Koltyn study explored. Conversely, Koltyn induced pain using heat and pressure, rather than exercise.

      In this manuscript, we have opted for the term ‘exercise-induced pain modulation’ since previous research has shown that exercise can elicit hypoalgesia as well as hyperalgesia (for review see Vaegter and Jones (2020)). Thus, the term refers to the modulation of pain through exercise. We have now included a sentence detailing the use of the term ‘exercise-induced pain modulation’ in the first passage of the introduction. Corresponding to Koltyn et al. (2014), we have used heat and pressure stimuli to induce pain and investigate the modulating effect of different exercise intensities on these pain modalities.

      (11) Line 62 and 64 - Both the Janal study and Haier study are missing from the reference list.

      We apologise for this oversight and have now updated the reference list to include the studies by Janal et al. (1984) and Haier et al. (1981).

      (12) Line 62 and 64 - define long/short distance/duration.

      We have revised the terminology from "short-duration" to "short-distance" to facilitate a more precise comparison of the exercise protocols employed in the studies by Janal et al. (1984) and Haier et al. (1981). Specifically, the long-distance run conducted by Janal et al. (1984) spanned 6.3 miles (10.3 km), while the short-distance run executed by Haier et al. (1981) covered 1 mile (1.6 km).

      (13) Line 62 - what type of pain?

      Janal et al. (1984) implemented thermal, ischemic, and cold pressor pain in their study and observed a hypoalgesic effect in response to thermal and ischemic pain that was reversed under NLX administration. We have now specified this in the text.

      (14) Line 67 - please place "i.e., the insula, ACC and prefrontal regions" in parentheses.

      Done.

      (15) Lines 67-69 - please provide clarity on the nature of the interventions being employed. For example, are you referring to interventions to reduce/overcome pain? Or are you referring to approaches to experimentally induce or increase pain during exercise? In either case, please be specific on the interventions employed, and why this variation in approach may make it challenging to draw a conclusion

      The interventions employed by several studies aimed to investigate the pharmacological underpinnings of the pain modulatory effect of exercise and were, thus, pharmacological interventions. The primary objective of these interventions is usually not to reduce/induce/decrease/increase pain but to block a specific receptor type to infer the involvement/role of these receptor types in pain modulation through exercise. In the context of exercise and pain specifically, the most frequently used pharmacological intervention consists of administering a µ-opioid receptor antagonist (naltrexone/naloxone (NLX)). Depending on which type of µ-opioid receptor antagonist is used, different administration protocols are employed (i.e., oral or intravenous administration, different doses, only bolus without constant injection). This variability in the administration protocols of these pharmacological interventions can account for different findings of the extent of opioidergic involvement in exercise-induced pain modulation. We have now refined the according section to increase the precision and clarity of the interventions used.

      (16) Line 69 - administration of what?

      This passage refers to the variability of administration of µ-opioid receptor antagonists such as naloxone (NLX) or naltrexone. We have now specified this in the according line.

      (17) Line 74 - EIH?

      As described above, we have chosen the term 'exercise-induced pain modulation' as an umbrella term for both exercise-induced hypoalgesia and hyperalgesia. However, the reviewer is correct that specifically studies investigating exercise-induced hypoalgesia have been criticised. Still, the proposed criticism also applies to studies detecting hyperalgesia and we would, thus, argue to retain the term ‘exercise-induced pain modulation’ here for the sake of consistency.

      (18) Line 75 - please define "single-arm pre-post measurements"

      We appreciate the reviewers' comment. Single-arm pre-post measurement studies involve participants being assigned to a single experimental condition, with pain assessments conducted only once prior to and once following the intervention. This study design presents several limitations, particularly in the context of examining exercise-induced modulation of pain (Vaegter and Jones, 2020). Such designs do not consider the effects of habituation to noxious stimuli, as highlighted by Vaegter and Jones (2020). Consequently, when measuring pain levels with only one pre- and one post-intervention assessment, there is a risk of misinterpreting the outcomes where a reduction in post-intervention pain ratings might erroneously be credited to the exercise intervention itself, rather than being a result of habituation to the noxious stimuli experienced. Incorporating randomised controlled trials with multiple measurement blocks not only mitigates these limitations but also provides a clearer understanding of how individual bouts of exercise influence pain perception.

      (19) Line 84 - is (40) a reference?

      We apologise for this oversight and have now updated the reference by Borszcz et al. (2018) to be displayed correctly.

      (20) Line 86 - is that 10 min per block (i.e. 40 min exercise time), or 10 min in total? If the former please include "per block" at the end of the sentence (Line 87).

      The reviewer is correct in assuming that we employed 10 min of cycling per block, resulting in a total of 40 minutes of cycling. We have updated the sentence now including ‘per block’ as suggested by the reviewer.

      (21) Line 89 - when you refer to "painfulness" are you referring to the intensity of pain experienced? If so, I think "pain intensity" would be more appropriate.

      In the current study, participants were asked about the ‘painfulness’ of each stimulus based on previous studies (Horing et al., 2019; Horing & Büchel, 2022; Tinnermann et al., 2022). The term ‘painfulness’ is a composite measure of ‘pain intensity’ (sensory dimension) and ‘pain unpleasantness’ (affective dimension) (Talbot et al., 2019). Since unpleasantness is also a definitional criterion of pain (‘Terminology | International Association for the Study of Pain’, n.d.) and previous research shows a high correlation between ‘pain unpleasantness’ and ‘pain intensity’ (Granot et al., 2008; Talbot et al., 2019) we have opted for the term ‘painfulness’ as a more comprehensive measure. Inherently, these two measures are highly correlated.

      (22) Line 91-93 - the way this is written could be suggestive of this being separate to the cycling blocks. Please rephrase to confirm that this was administered prior to the commencement of the cycling blocks.

      We have refined the sentence to make it clearer that the drug treatment was administered before the cycling block commenced on each of the experimental days. We would like to further specify, that whilst the bolus dose of the treatment was administered prior to the experiment, a constant intravenous supply of SAL/NLX was maintained throughout the experiment using an infusion pump.

      (23) Methods general - why only 10 min of exercise? It is likely that there is a 'dose effect' of exercise on EIH, whereby the intensity of exercise and the duration of the exercise are important. Short-duration but high-intensity exercise can induce EIH, as can moderate duration low-intensity exercise. But, for this protocol, was the intensity high enough or long enough to meet the 'dose' needed?

      We thank the reviewer for their question. Our decision to employ 10-minute exercise blocks was rooted in both scientific evidence on exercise-induced hypoalgesia and the (clinical) applicability of the findings. Research has shown that exercise durations ranging from 8 minutes to 2 hours of aerobic exercise can induce hypoalgesia (for review see Koltyn (2002)). Specifically, several studies induce hypoalgesia at 10-15 minutes of aerobic exercise (Gomolka et al., 2019; Gurevich et al., 1994; Haier et al., 1981; Jones et al., 2019; Sternberg et al., 2001; Vaegter et al., 2015). Furthermore, many prior studies have employed exercise durations that are tailored to professional or amateur athletes which may not be practical for healthy individuals with lower fitness levels who may find it challenging to engage in longer sessions, such as an hour of running. When considering applying these findings to the clinical chronic pain population it is crucial to assess the manageability of proposed exercise protocols. We believe that 10 minutes of exercise, whilst being a relatively brief exercise duration, may still be sufficient to elicit exercise-induced hypoalgesia.

      (24) Methods general - what was the time gap between each round (i.e. after the fMRI, how long before the participant started the next cycling block?).

      After each fMRI run the participants were taken out of the MR scanner. The HR and SPO2 were measured and participants were given the chance to go to the restroom before positioning them on the bike and starting the next block. All in all, the time following the fMRI scan and before the new block commenced ranged between 5-10 minutes. We have now included this specification in the methods section.

      (25) Methods general - there is some evidence to show that the EIH effect is less consistently shown when heat is used to induce pain - was there a reason heat was used as the pain induction method here?

      We thank the reviewer for their comment. Indeed, previous meta-analyses by Naugle et al. (2012) report larger effect sizes for pressure pain (Cohen’s d = 0.69) closely followed by heat pain (d = 0.59). In light of this evidence, we included both pain modalities in the current study. Notably, we found no significant differences in pressure pain responses between LI and HI exercise. It is important to emphasise that the term "pressure pain" predominantly encompasses studies employing handheld pressure algometry, whereas our investigation utilised a pressure cuff. This methodological variation raises the possibility that our findings—and corresponding effect sizes—may not be directly comparable to prior pressure pain studies.

      (26) Methods general - please be consistent in the use of terminology. In some areas, you use the phrase "cycling block" whereas in other areas it is referred to as a "cycling run".

      We have revised the methods section to be more precise with the terms ‘run’ and ‘block’.

      (27) Line 571-573 - Please detail how participants were excluded based on scores from STAI and BDI-II.

      We apologise for the misspelling, as it should be that one participant was excluded based on a BMI (body mass index) below 18. No participant had to be excluded based on the STAI or BDI-II score in the current study. We have corrected this in the manuscript.

      (28) Line 636-651 - the FTP20 test has been shown not to be a valid marker of the separation between the heavy and severe exercise intensity domains (see Wong et al 2023 - https://doi.org/10.1080/02640414.2023.2176045). Given that participants completed the high intensity cycle in 'zone 4' (91-106% of FTP), it is probable that participants could have completed this 10 min in either the heavy or the severe exercise intensity domains, with significant implications for the relative challenge this 10 min of exercise. Why was zone 4 used? What are the implications of this? Please discuss and include this as a limitation.

      We thank the reviewer for their comment as it touches upon the challenges of accurately estimating exercise intensities. It is indeed crucial to consider the boundaries between moderate, heavy, and severe intensity domains, as delineated by physiological markers.

      The study by Wong et al. (2023) is interesting; it assesses blood lactate and VO2 levels at FTP and FTP+15 Watts. Despite being highly relevant for the field some of the findings should be interpreted with caution due to the low sample size of 13 participants, consisting of 11 male and only 2 female cyclists, which may limit generalisability. Additionally, the testing protocol implemented in the study to determine participants' FTP consisted of a 5-minute self paced pedalling at 100 Watts followed by a 20-minute maximal, self-paced time trial. This differs from the FTP20 test as implemented in the current study (see Supplemental Table S1) or by other studies (McGrath et al., 2019). The finding in Wong et al. (2023) that participants were only able to sustain cycling at FTP for an average of 33 minutes suggests that the deviating protocol overestimates FTP. Mackey and Horner (2021) propose that the validity of the FTP20 test might rely on the warm-up used before FTP20 testing and the training status of athletes.

      However, we acknowledge that without direct measurements of VO2max or blood lactate levels, it is challenging to determine the precise intensity domain in which each participant was operating in the current study. Still, the RPE (low: M = 8.59, SD = 1.32; high: M = 14.92, SD = 1.98) suggests that participants operated in the heavy-intensity domain in the HI exercise condition. This is further supported by the relative power (%FTP) maintained in the HI (M = 105; SD = 0.05; Author response image 5, purple) and LI (M = 58; SD = 0.06; Author response image 5, green) exercise conditions (difference: t(37) = 44.58, P < 2.2e-16, d = 6.46) confirming the accuracy of the implemented FTP test as well as the maintained power throughout the cycling blocks. Thus, we would argue that participants in the current study predominantly exercised the heavy domain during the HI exercise condition. We have included the relative Power in Figure 3A, replacing the absolute Power.

      Finally, we propose that discussing exercise intensity domains within the context of our study enriches the understanding of exercise-induced hypoalgesia without undermining the integrity of our findings. We have now included a discussion of the validity of the FTP20 test as a demarcation point concerning the intensity domains.

      Author response image 5.

      Raincloud plot of relative power (%FTP) during low (green) and high (purple) intensity exercise. Individual data points depict subject-specific averages across blocks.

      (29) Line 676 - please provide further information on each cycling run/block. Did each participant complete a total of 4 runs (i.e., a total of 40 minutes of exercise), with 2 runs completed at a high intensity and 2 runs completed at a low intensity in a randomised order (e.g., for one participant this could be 10 minutes at low, followed by 10 minutes at high, followed by 10 minutes a low, followed by 10 minutes at high)? Figure 1 details this nicely, however, it would be helpful to read in-text.

      The reviewer is correct in assuming that there were a total of 4 blocks on each experimental day. Participants completed cycling in 2 blocks at HI and in 2 blocks at LI in a pseudorandomised order. This order was kept constant across experimental days (i.e. completing the same block order on Day 2 and Day 3). We have detailed this further in the Methods section.

      (30) Discussion general - it is possible that EIH could be induced via different mechanisms and that these mechanisms are at least in part due to exercise intensity. For example, EIH from higher-intensity exercise might have some contribution from CPM.

      We thank the reviewer for their comment. Previous research aimed to disentangle the two seemingly similar mechanisms of exercise-induced hypoalgesia (EIH) and conditioned pain modulation (CPM) (Ellingson et al., 2014; Rice et al., 2019; Samuelly-Leichtag et al., 2018; Vaegter et al., 2014). CPM is typically induced by applying a tonic noxious stimulus that decreases pain sensitivity to another noxious stimulus applied simultaneously or shortly after at a distant body part (Graven-Nielsen & Arendt-Nielsen, 2010). Despite EIH and CPM showing distinct mechanisms, it cannot be completely ruled out that there are at least partially overlapping mechanisms driving the two phenomena (Rice et al., 2019). Due to our study design, where the time difference between cycling blocks and the applied pain was on average five minutes, it is unlikely that CPM is the driving pain modulatory mechanism in our study setup.

      (31) Line 101 - as this was preregistered, should the study design be followed and then reported?

      We have conducted the study adhering to the preregistered study design and now report the results for pressure pain (Supplemental Figure S1). Some of the preregistered analyses (i.e. directly comparing heat and pressure pain) were beyond the scope of the current study and will be reported separately.

      (32) Line 110 - please provide some data on the fitness levels and how this is classified as high/low.

      The FTP (relative to body weight) was used as an estimate of cardiovascular and endurance fitness (Valenzuela et al., 2018). We refrained from classifying the fitness levels dichotomously as low or high since this is a subjective measure in a sample of healthy individuals of diverse fitness levels. Instead, we utilised the FTP as a more nuanced metric for comparison.

      (33) Lines 159-160 - in the context of the difference in intensity between the sessions. But, it is likely that the high-intensity exercise would have posed quite different relative challenge between participants.

      We thank the reviewer for their comment. As described above, we did not obtain direct measurements of VO2max or blood lactate levels making it challenging to determine the precise intensity domain in which each participant was operating in the current study. However, all participants received the same instructions to the BORG rating scale ensuring the comparability of RPE across participants to a certain extent.

      (34) Figure 3C - what instructions and familiarisation were given to participants regarding the 6-20 Borg scale? In Figure 3C it looks as though several participants rated the low exercise intensity at 6. This would/should be equivalent to sitting quietly, so it looks as though at least several participants did not understand how to use the RPE - please discuss.

      Indeed, three participants rated the LI exercise condition at 6 due to an error in the translation of the scale instruction. Participants were instructed that the lower anchor point of the scale (6) referred to ‘extremely light’ instead of ‘no exertion’. Thus, we have rescaled the RPE ratings where a rating of 6 now corresponds to a 7 (‘extremely light’) on the BORG scale and again calculated the paired t-test. There is still a significant difference in the RPE between exercise intensities (t(38) = 19.65, P < 2.2e-16, d = 3.69; Author response image 6). We have corrected this in the manuscript accordingly and updated Figure 3C.

      Author response image 6.

      Raincloud plot of rating of perceived exertion (RPE) on the BORG scale during low (green) and high (purple) intensity exercise. Individual data points depict subject-specific averages across blocks. A rating of 6 reflects ‘no exertion’ and 20 reflects ‘maximal exertion’.

      (35) Line 171 - is (37, 38) a reference?

      We apologise for this oversight and have now updated the references to be displayed correctly.

      (36) Line 176-18 - is this interaction sufficiently powered? Differences between sexes are not mentioned in the pre-registered study

      We have conducted an additional post-hoc power analysis for the interaction of drug, fitness level, and sex on differential heat pain ratings. We employed the power analysis for mixed models implemented in R (powerCurve) with 1000 simulations. This revealed that with a power of α = 0.8, a sample size of n = 27 would have been sufficient to detect this effect (Author response image 7). Despite not having preregistered the factor ‘sex’, we believe that the observed results provide valuable insights that contribute to a deeper understanding of the data. We have established these analyses to be exploratory, emphasising the need for caution in their interpretation. However, we feel it is essential to report these findings to inform future studies, ensuring that such factors are adequately considered.

      Author response image 7.

      Post-hoc power analysis for behavioural effects from the linear mixed effects (LMER) model with interaction drug, fitness level, and sex using the R package powerCurve with α = 0.8 and 1000 simulations.

      (37) Line 227 - this is not what this analysis shows. The comparison is low vs high-intensity exercise on pain modulation, not exercise vs. no exercise. You cannot conclude that aerobic exercise has no effect on pain modulation because you did not do that comparison (i.e. no baseline (without exercise) for pain).

      We agree with the reviewer and have rephrased the sub-headline accordingly to reflect that there is no difference in exercise-induced hypoalgesia between HI and LI aerobic exercise.

      (38) Methods General - why was a control condition not used, or at least a baseline pain response, so that low/high-intensity exercise could be compared to a baseline? Given this, I'm not sure I agree with the study conclusions (abstract: 'These results indicate that aerobic exercise has no overall effect on pain in a mixed population sample') because you have compared high vs low-intensity exercise, not exercise vs. no exercise.

      As for the lack of a resting control condition, we acknowledge that our study was not designed to test the overall effect of exercise versus no exercise. However, our primary objective was to compare different exercise intensities, hypothesising that low-intensity (LI) exercise would induce less pain modulation as compared to high-intensity (HI) exercise. By exploring this, we aimed to enhance understanding of the dose-response relationship between exercise and pain modulation. To better reflect this focus, we have revised the misleading phrasing regarding the ‘overall’ effect of exercise to clearly emphasize our primary aim: comparing HI and LI exercise. This reviewer suggests an interesting interpretation of the data suggesting that exercise-induced hypoalgesia might have occurred for both exercise intensities since the pain ratings provided were lower than the anticipated intensities as determined by the calibration. Given that this difference is lower in the naloxone (NLX) condition could provide evidence of opioidergic mechanisms underlying this effect.

      Unfortunately, the current study is not designed to comprehensively answer this question since there was no resting control condition. In particular, the lower pain ratings under SAL (Figure 6) could be due to exercise triggering the descending pain modulatory system (DPMS), but equally due to the default activation of the DPMS. Only an additional “no exercise” condition could disentangle this. Furthermore, habituation to noxious stimuli can influence pain ratings, resulting in lower pain ratings during the experiment as compared to the calibration.

      (39) Line 285 - or that better-trained individuals have a greater EIH response to higher intensity exercise, but both those of low and high fitness have established EIH after low intensity exercise. Given there isn't a 'no exercise' baseline, it is hard to make conclusions about EIH effect generally, only comparisons between high/low exercise intensity.

      We thank the reviewer for their comment. We agree that we cannot establish whether all participants showed a hypoalgesic response to the LI exercise with the current study design. However, our results show that participants with higher fitness levels showed increased hypoalgesia after HI exercise compared to those with lower fitness levels. We have refined the sentence accordingly.

      (40) Figure 7A - the regression line here is not that convincing.

      We acknowledge the reviewers’ concern regarding the regression line. However, it is important to note that the significant main effect of fitness level on differences in pain ratings in the SAL condition (β = 6.45, CI [1.25, 11.65], SE = 2.56, t(38) = 2.52, P = 0.02) supports the assertion that higher fitness levels are associated with greater hypoalgesia following HI exercise compared to LI exercise. While the trend may not be visible for all data points, the statistical analysis provides a robust basis for the observed relationship (r = 0.33, P = 0.038).

      (41) Line 354 - the NLX infusion was double-blind, but what are the implications of participants knowing that they completed high/low-intensity exercise - this cannot be blinded.

      The reviewer is correct that the exercise intensities cannot be blinded. To account for potential expectation effects of exercise on several psychological and physiological domains (including pain), participants completed a questionnaire on the calibration day where they had to indicate their expectations of to what extent acute exercise affects several domains (Lindheimer et al., 2019). They could rate each domain on a Likert scale ranging from ‘large decrease’ (-3) to ‘large increase’ (3) with 0 denoting ‘no effect’. This format was chosen to allow measuring the direction and magnitude of expectation effects and to avoid being directive or suggestive (Lindheimer et al., 2019). Despite including other psychological and physiological domains in the questionnaire (i.e., stress, anxiety, energy, memory) we focused on the specific pain domains (muscle pain, joint pain, and whole body pain) to establish participant’s expectations regarding the effect of acute exercise on pain. We tested whether the expectation ratings for each pain type were significantly different from 0 (no effect) using a one-sample t-test.

      There was no significant effect for muscle pain (t(38) = 1.78, P = 0.08, M = 0.39, SE = 0.12), joint pain (t(38) = -0.12, P = 0.90, M = -0.03, SE = 0.11), or ‘whole-body pain (t(38) = -1.05, P = 0.30, M = -0.21, SE = 0.12) suggesting there to be no expectation effect on these pain domains in the overall sample (Supplemental Figure S10A). Since there is variation in the data we calculated the correlation of the expectation ratings in the different pain domains with the difference score between the pain ratings in the SAL condition (LI – HI rating; Supplemental Figure S10B). This analysis yielded no significant correlation in either of the pain domains (joint pain: r = 0.11, P = 0.49; muscle pain: r = -0.07, P = 0.68; whole-body pain: r = 0.07, P = 0.68).

      Moreover, given that we have not been able to show a difference between the exercise intensities on pain modulation, expectation effects are likely not to contribute to this null effect.

      (42) Line 356-358 - and this comparison (and primary hypothesis) is not blinded.

      While we agree with the reviewer that this comparison is not – and potentially cannot be – blinded, we would like to reiterate our results from the previous paragraph that indicate that such expectation effects of exercise on pain were not present in the sample and, thus, did not seem to have influenced the results. It is noteworthy that the double-blind design of our study design specifically pertains to the pharmacological intervention employed.

      (43) Line 358-360 - this could be explained by both types of exercise inducing EIH via the same mechanism (which is disrupted by NLX).

      We thank the reviewer for their comment and would like to refer back to the reviewer's comment number 38 for a response to this.

      (44) Line 360-361 - this conclusion cannot be drawn, because you have only compared high vs low intensity exercise. So, the conclusion should be 'These results suggest that there is no difference between high and low aerobic exercise intensity on heat-induced pain'.

      We agree with the reviewer and have rephrased the sentence to reflect the claim accurately.

      (45) Line 396 - as previously discussed, this conclusion cannot be drawn through this study design.

      We agree with the reviewer and have rephrased the sub-headline accordingly to reflect that there is no difference in exercise-induced hypoalgesia between HI and LI aerobic exercise.

      (46) Line 399 - please expand on this point - it is critical to the hypothesis and should also be included in the introduction. What intensities/duration/dose of aerobic exercise is generally established to cause EIH?

      We thank the reviewer and agree that this is a crucial aspect that requires further specification. Below we have expanded on the duration/intensities shown to elicit exercise-induced hypoalgesia and included a concise version of this detailed paragraph in the manuscript introduction.

      For aerobic exercise, different methods have been employed to determine exercise intensity levels i.e., through the VO2max, age-predicted HRmax, or incremental intensities (Koltyn, 2002). Most studies using VO2max as a measure of exercise intensity (Koltyn et al., 1996; Micalos & Arendt-Nielsen, 2016; Vaegter et al., 2014) were able to induce hypoalgesia with HI levels ranging between 65%-75% VO2max. When using the HRmax as a measure of determining exercise intensities, HI exercise at 70%-75% of the HRmax has been shown to produce greater hypoalgesia compared to moderate intensity at 50% HRmax (Naugle et al., 2014; Vaegter et al., 2014). Furthermore, previous research has suggested that HI exercise produces greater hypoalgesia compared to LI exercise (60-70% HRmax vs. light activity: M. D. Jones et al., 2019; 70% vs. 50% HRmax: Naugle et al., 2014; 75% vs. 50% VO2max: Vaegter et al., 2014).

      Furthermore, different durations can be regarded as suitable with durations between 8 minutes to 2 hours of aerobic exercise having been shown to induce hypoalgesia (for review see Koltyn (2002)). Hoffman et al. (2004) showed a hypoalgesic response after 30 minutes but not after 10 minutes at 75% VO2max of cycling. In contrast, other studies were able to induce hypoalgesia at 10-15 minutes of HI aerobic exercise (75% VO2may: Gomolka et al., 2019; 63% VO2max: Gurevich et al., 1994; self-paced: Haier et al., 1981; 60-70% HRmax: Jones et al., 2019; 85% HRmax: Sternberg et al., 2001; 75% VO2max: Vaegter et al., 2015).

      (47) Line 400-401 - please define high intensity.

      We thank the reviewer for their comment. The referenced studies by Vaegter et al. (2014) and Jones et al. (2019) based the estimation of HI and LI exercise on an age-related target heart rate corresponding to VO2max and HRmax, respectively. In Vaegter et al. (2014), the HI condition corresponded to 75% VO2max, while the LI to 50% VO2max. In Jones et al. (2019), the HI exercise condition corresponded to 60% and 70% of HRmax, while the LI condition was defined as pedalling slowly against a light resistance of 0.5 kg of force to maintain a rating of perceived exertion (RPE) not above resting. We have included this clarification in the relevant section to elucidate the intensities of the chosen exercise conditions.

      (48) Line 403-405 - I'm not sure I follow (perhaps I have misunderstood) - pain induction was completed after exercise in the MRI scanner, so there was no distraction effect of exercise in either condition. A baseline could have been established in the same way and there would be exactly the same conditions, just without prior exercise.

      We agree with the reviewer that a resting baseline condition in the context of exercise induced pain modulation allows for the investigation of a potential hypoalgesic effect of exercise compared to no exercise. Nevertheless, it is important to note that previous studies (Brooks et al., 2017; Sprenger et al., 2012) have shown that cognitive pain modulation is mediated by endogenous opioids. Therefore, tasks with different attentional loads potentially influence post-task pain ratings. Although, we agree with the reviewer that the effect of distraction or attentional load would be minimal in the MR scanner, there still could be an effect of different cognitive loads from exercise vs. no exercise. Nevertheless, we focus the discussion on investigating the dose-response relationship between different exercise intensities where an ‘active’ control condition might contribute to a more nuanced understanding of exercise-induced pain modulation.

      (49) Line 403-411 - this is fine (although I do not agree that this was the best methodological decision), however, it does limit the conclusions that can be drawn (as previously mentioned). That is, you cannot conclude that no EIH occurred, only that there was no difference between low and high-intensity exercise in post-exercise pain response.

      We agree with the reviewer that the comparison of HI vs. LI exercise does not allow for an interpretation of the overall effect of exercise as opposed to no exercise on pain modulation. The comparison of HI and LI exercise allows the investigation of a dose-response relationship of these distinct exercise intensities. While LI exercise might not be a 'pure' control condition in the traditional sense, it is valuable for exploring the complexities of exercise and pain interaction.

      (50) Line 419-422 - sorry I do not follow - you say that moderate intensity exercise most reliably induces EIH but then select exercise intensities that are likely to be in the heavy or severe intensity domain? Please also include in this discussion the limitations of FTP20 as a threshold marker (see Wong et al) and the implications on the results/conclusions.

      We thank the reviewer for their comment. In the referenced sentence, we have defined the HI exercise as described in the reviews. Specifically, Wewege and Jones (2020) reported hypoalgesia to be greater after higher-intensity exercise, although the intensity was not further specified. Naugle et al. (2012) noted that HI exercise (i.e., 75% of VO2max) produced greater hypoalgesia, while Koltyn (2002) indicated that hypoalgesia occurs at intensities ranging from 60% to 75% of VO2max but more reliably at 75% VO2max or higher. Consequently, we have removed the term ‘moderate’, as it does not accurately reflect what has been reported in the reviews and could be misleading. Moreover, we have clarified the specific criteria for what is considered high (or higher) intensity exercise in the referenced reviews.

      We kindly ask the reviewers to refer back to the previous comment (reviewer comment number 28) regarding the discussion of the intensity domains and the FTP20 test as demarcation point for these intensity domains.

      (51) Line 422-425 - indeed, pacing is an important element of this test, which inexperienced cyclists have difficulty with when they are not provided with proper familiarisation.

      We agree with the reviewer that the FTP20 test has mainly been validated and employed in experienced cyclists and requires further validation in non-athletes of both sexes. However, since we have used an extensive warm-up period and several paced steps (intervals, 5-minute time-trial) as well as recovery periods (Supplemental Table S1) based on McGrath et al. (2019) we propose that participants were thoroughly familiarised with the elements of pacing before the estimation of the FTP in the 20-minutes took place. On average, participants showed a variation of M = 21.80 Watts (SE = 1.44 Watts) during the 20-minute paced FTP20 test (Supplemental Figure S11A). Interestingly, our data suggests that participants with a higher FTP showed higher variation of power output (Watts) during the 20-minute FTP test compared to individuals with lower fitness levels (Supplemental Figure S11B).

      (52) Line 425-427 - please remove this, the RPE difference between exercise bouts is not evidence that participants cycled at FTP.

      We thank the reviewer for their comment. However, we would propose to include the rating of perceived exertion (RPE) since it shows that the exercise intensities have been perceived as significantly different by the participants. This behavioural measure of exertion is potentially important for a broader audience to understand the exercise implementation beyond physiological markers.

      (53) Line 432 - high vs. low-intensity aerobic exercise

      We have changed the sentence accordingly to support the claim of the study that there was no difference in exercise-induced pain modulation between HI and LI aerobic exercise.

      (54) Line 447-449 - this seems contradictory to the first line of this paragraph (430-432) - i.e. that the heterogenous sample may have caused the null finding. Why deliberately select a participant sample that is likely to lead to a null effect?

      In the current study, we aimed to include participants of diverse fitness levels and both sexes to verify the findings on exercise-induced pain modulation in a broader population. We consider this important concerning translational aspects of EIH. Indeed, our heterogeneous sample may have ‘caused’ the observed null effect, but at the same time, it suggests that more homogenous (sometimes composed solely of male athletes) samples employed in many earlier studies might have skewed the understanding of exercise-induced pain modulation and thus unintentionally suggested a (non-existing) generalisation of this effect to the general population.

      (55) Line 532-456 - although Koltyn found electrical pain to have the greatest effect?

      The review by Naugle et al. (2012) reported effect sizes for heat (Cohens d = 0.59) and pressure pain intensity (d = 0.69) following aerobic exercise but did not provide effect sizes for electrical pain intensity. They noted that the effect size for electrical pain intensity after isometric exercise was d = 0.40, which is lower than that for heat and pressure pain. While Koltyn (2002) stated that electrical and pressure stimuli induce exercise-induced hypoalgesia more consistently than thermal pain, the study did not clarify whether this applies to pain threshold, intensity, or tolerance, nor did they provide effect sizes. Given that electrical, pressure, and heat pain are the most commonly used methods to induce quantifiable pain in the context of exercise studies (Vaegter and Jones, 2020), we based our decision to use heat and pressure pain primarily on Naugle et al.'s findings.

      (56) Line 468-469 - why leave out content that was pre-registered (i.e. difference between pressure and heat pain) but includes analysis that wasn't (i.e. sex differences)? If a study is going to be pre-registered, then isn't it important to follow that design?

      We thank the reviewer for this comment. We have conducted the study adhering to the preregistered study design and now report the results for pressure pain (Supplemental Figure S1). Some of the preregistered analyses (i.e. directly comparing heat and pressure pain) were beyond the scope of the current study and will be reported separately.

      (57) Line 532-525 - and how could this have been accounted for?

      We apologise for any confusion, as we are unsure about the specific reference the reviewer is making based on the provided line numbers. We believe the question relates to how the potential effects of endocannabinoids were considered in the current study design, and we've addressed that in our response. In human studies, it is not possible to centrally block endocannabinoids, which makes it difficult to directly estimate their role in exercise-induced pain modulation in humans. Measuring endocannabinoids in the blood might not adequately capture changes in endocannabinoid levels in the brain throughout the different exercise intensity conditions. Despite these limitations, exploring the role of endocannabinoids in exercise-induced pain modulation presents a promising avenue for future research that could enhance our understanding of pain mechanisms and improve pain management strategies.

      58) Limitations General - please include the other limitations discussed in this review.

      Done.

      (59)Line 530 - please amend this conclusion, in line with previous comments.

      Done.

      We would like to thank the reviewer for critically evaluating the manuscript and providing insightful comments. We appreciate the reviewer recognising the strengths of our work and believe that their suggestions will contribute to improving the quality of the manuscript.

    1. Author response:

      The following is the authors’ response to the current reviews.

      Reviewer #1 (Public review): 

      Devakinandan et al. present a revised version of their manuscript. Their scRNA-seq data is a valuable resource to the community, and they further validate their findings via in situ hybridizations and electron microscopy. Overall, they have addressed my major concerns. I only have two minor comments. 

      (1) The authors note in Figure 4I, and K that because the number of C2 V2Rs or H2-Mv receptors increased while the normalized expression of Gnao1 remained constant (and likewise for V1Rs and Gnai2 in Figure 4-S4C) that their results are unlikely to be capturing doublets. I'm not sure that this is the case. If the authors added together two V2R cells the total count of every gene might double, but the normalized expression of Gnao1 would remain the same. To address this concern, the authors should also show the raw counts for Gnao1 as well as the total number of UMIs for these cells. 

      In Figure 4I, 4K and Figure 4-Figure supplement 4C, on Y-axis, we plotted the sum of normalized counts of all V1R/V2R/H2-Mv genes expressed in each cell along with the normalized expression value of Gnao1/Gnai2. Both VR/H2-Mv and Gnao1/Gnai2 are normalized values, with normalization based on LogNormalize (mentioned in methods). We show here plots of total expression calculated from raw counts corresponding to the same Figure. Raw counts of VRs/H2-Mv, Gnao1/Gnai2 are plotted separately due to difference in scale. The overall trend matches normalized counts, with minor fluctuations in Gnao1/Gnai2.     

      Author response image 1.

      As mentioned in our response to version-1 reviews and in our manuscript, doublets generally are a random combination of two cells and the probability that a combinatorial pattern is due to doublet is proportional to the abundance of cells expressing those genes. It is possible that some of the family-C V2R combinations represented by 2 cells are doublets because of their widespread expression. The frequency of combinatorial expression patterns, greater than a set threshold of 2 cells, that we observed for family ABD V2Rs or V1Rs (supplementary tables 7, 8) is an indication of co-expression and unlikely from random doublets. For instance, 134 cells express two V1Rs, of which 44 cells express Vmn1r85+Vmn1r86, 21 cells express Vmn1r184+Vmn1r185, 13 express Vmn1r56+Vmn1r57, 6 express Vmn1r168+Vmn1r177. Some of the co-expression combinations we reported were also identified and verified experimentally in Lee et al., 2019 and Hills et. al., 2024.

      The co-expression of multiple family-C2 V2Rs (Vmn2r2-Vmn2r7) along with ABD V2Rs per cell as shown in our data, has been shown experimentally in earlier studies.      

      (2) As requested, the authors have now added a colorbar to the pseudocolored images in Figures 7. However, this colorbar still doesn't have any units. Can the authors add some units, or clarify in the methods how the raw data relates to the colors (e.g. is it mapped linearly, at a logscale, with gamma or other adjustments, etc.)? Moreover, it's also unclear what the dots in the backgrounds of plots like Figure 7E mean. Are they pixels? Showing the individual lines, the average for each animal, or omitting them entirely, might make more sense. 

      We used the Fire LUT with linear scale within Fiji / Image-J software to assign scale to the pseudo-colored images in Figure 7. We will include this description in our methods and thank the reviewer for pointing it out. The dots in the background are mentioned in Figure 7 legend as fluorescence intensity values normalized to a 0-1 scale and color coded for each antibody. The trendline was fitted on these values.  

      Reviewer #2 (Public review): 

      Summary: 

      The study focuses on the vomeronasal organ, the peripheral chemosensory organ of the accessory olfactory system, by employing single-cell transcriptomics. The author analyzed the mouse vomeronasal organ, identifying diverse cell types through their unique gene expression patterns. Developmental gene expression analysis revealed that two classes of sensory neurons diverge in their maturation from common progenitors, marked by specific transient and persistent transcription factors. A comparative study between major neuronal subtypes, which differ in their G-protein sensory receptor families and G-protein subunits (Gnai2 and Gnao1, respectively), highlighted a higher expression of endoplasmic reticulum (ER) associated genes in Gnao1 neurons. Moreover, distinct differences in ER content and ultrastructure suggest some intriguing roles of ER in Gnao1-positive vomeronasal neurons. This work is likely to provide useful data for the community and is conceptually novel with the unique role of ER in a subset of vomeronasal neurons. This reviewer has some minor concerns and some suggestions to improve the manuscript. 

      Strengths: 

      (1) The study identified diverse cell types based on unique gene expression patterns, using single-cell transcriptomic. 

      (2) The analysis suggest that two classes of sensory neurons diverge during maturation from common progenitors, characterized by specific transient and persistent transcription factors. 

      (3) A comparative study highlighted differences in Gnai2- and Gnao1-positive sensory neurons. 

      (4) Higher expression of endoplasmic reticulum (ER) associated genes in Gnao1 neurons. 

      (5) Distinct differences in ER content and ultrastructure suggest unique roles of ER in Gnao1-positive vomeronasal neurons. 

      (6) The research provides conceptually novel on the unique role of ER in a subset of vomeronasal neurons, offering valuable insights to the community. 

      Reviewer #3 (Public review): 

      Summary: 

      In this manuscript, Devakinandan and colleagues have undertaken a thorough characterization of the cell types of the mouse vomeronasal organ, focusing on the vomeronasal sensory neurons (VSNs). VSNs are known to arise from a common pool of progenitors that differentiate into two distinct populations characterized by the expression of either the G protein subunit Gnao1 or Gnai2. Using single-cell RNA sequencing followed by unsupervised clustering of the transcriptome data, the authors identified three Gnai2+ VSN subtypes and a single Gnao1+ VSN type. To study VSN developmental trajectories, Devakinandan and colleagues took advantage of the constant renewal of the neuronal VSN pool, which allowed them to harvest all maturation states. All neurons were re-clustered and a pseudotime analysis was performed. The analysis revealed the emergence of two pools of Gap43+ clusters from a common lineage, which differentiate into many subclusters of mature Gnao1+ and Gnai2+ VSNs. By comparing the transcriptomes of these two pools of immature VSNs, the authors identified a number of differentially expressed transcription factors in addition to known markers. Next, by comparing the transcriptomes of mature Gnao1+ and Gnai2+ VSNs, the authors report an enrichment of ER-related genes in Gnao1+ VSNs. Using electron microscopy, they found that this enrichment was associated with specific ER morphology in Gnao1+ neurons. Finally, the authors characterized chemosensory receptor expression and co-expression (as well as H2-Mv proteins) in mature VSNs, which recapitulated known patterns. 

      Strengths: 

      The data presented here provide new and interesting perspectives on the distinguishing features between Gnao1+ and Gnai2+ VSNs. These features include newly identified markers, such as transcription factors, as well as an unsuspected ER-related peculiarity in Gnao1+ neurons, consisting in a hypertrophic ER and an enrichment in ER-related genes. In addition, the authors provide a comprehensive picture of specific co-expression patterns of V2R chemoreceptors and H2-Mv genes. 

      Importantly, the authors provide a browser (scVNOexplorer) for anyone to explore the data, including gene expression and co-expression, number and proportion of cells, with a variety of graphical tools (violin plots, feature plots, dot plots, ...). 


      The following is the authors’ response to the original reviews.

      Public Reviews: 

      Reviewer #1 (Public Review): 

      Devakinandan and colleagues present a manuscript analyzing single-cell RNAsequencing data from the mouse vomeronasal organ. The main advances in this manuscript are to identify and verify the differential expression of genes that distinguish apical and basal vomeronasal neurons. The authors also identify the enriched expression of ER-related genes in Gnao1 neurons, which they verify with in situ hybridizations and immunostaining, and also explore via electron microscopy. Finally, the results of this manuscript are presented in an online R shiny app. Overall, these data are a useful resource to the community. I have a few concerns about the manuscript, which I've listed below. 

      General Concerns: 

      (1) The authors mention that they were unable to identify the cells in cluster 13. This cluster looks similar to the "secretory VSN" subtype described in a recent preprint from C. Ron Yu's lab (10.1101/2024.02.22.581574). The authors could try comparing or integrating their data with this dataset (or that in Katreddi et al. 2022) to see if this is a common cell type across datasets (or arises from a specific type of cell doublets). In situ hybridizations for some of the marker genes for this cluster could also highlight where in the VNO these cells reside. 

      Cluster13 (Obp2a+) cells identified in our study have similar gene expression markers to “putative secretory” cells mentioned in Hills et al.. At the time this manuscript was available publicly, our publication was already communicated. We have now performed RNA-ISH to Obp2a, the topmost marker identified with this cluster, and found it to be expressed in cells from glandular tissue on the non-sensory side. Some of the other markers associated with this cluster such as Obp2b, Lcn3, belong to the lipocalin family of proteins. Hence in our estimate these markers collectively represent non-sensory glandular tissue. We have added Obp2a RNA-ISH to Figure 2-figure supplement-1A and results section in our revised manuscript. Cluster-13 also has cells expressing Vmn1r37, which typically is expressed in neuronal cells. However, we do not see Obp2a mRNA in the sensory epithelium. It is possible that cluster-13 comprises a heterogenous mixture of cells, some of which are clearly non-sensory cells from glandular tissue, co-clustered with other cell types as well as a  possibility that Obp2a is expressed below the detection level of our assay in neurons, which will require further experiments. We do not have any possible reason to confidently assign this cluster as a neuronal cell type, hence, we excluded it in downstream analysis of neurons. 

      We used the data from Hills et al., to compare co-expression characteristic of V2Rs, which is added as Figure 3-figure supplement 3. 

      (2) I found the UMAPs for the neurons somewhat difficult to interpret. Unlike Katreddi et al. 2022 or Hills et al. 2024, it's tricky to follow the developmental trajectories of the cells in the UMAP space. Perhaps the authors could try re-embedding the data using gene sets that don't include the receptors? It would also be interesting to see if the neuron clusters still cluster by receptor-type even when the receptors are excluded from the gene sets used for clustering. Plots relating the original clusters to the neuronal clusters, or dot plots showing marker gene expression for the neuronal clusters might both be useful. For example, right now it's difficult to interpret clusters like n8-13. 

      a) We have revised the UMAP in Figure 3A, and labeled mature, immature, progenitor neurons so that it is easier to follow the developmental trajectory. 

      b) In our revised text we have explicitly drawn equivalence between neuronal clusters from Figure 1 to re-clustered neurons in subsequent figures (Figure 3 and 4 in revised submission). For developmental analysis, we merged mature Gnao1, Gnai2 neuronal subclusters to two major clusters that are equivalent to original neuronal clusters in Figure 1. As UMAP is an arbitrary representation of cells, we also show expression of markers for major neuronal cell types in Figure 1C and Figure 3-figure supplement 1B, helpful in making the connection.  

      c) The purpose of re-clustering with higher resolution was to identify sub-populations within Gnao1 and Gnai1 neurons. It was useful to make sense of mature Gnao1 neurons, where family-C Vmn2r and H2-Mv expression maps onto distinct subclusters. Along with neuronal subclusters in revised Figure 3-figure supplement-1 we include a dot plot of gene expression markers. 

      d) In Figure 3-figure supplement-2, we show a comparison of neuronal clusters with and without VRs. Exclusion of VRs did not substantially alter mature neuron dichotomy into Gnao1/Gnai2. Only Gnao1 subclusters n1/n3 whose organization is dependent on family-C Vmn2r expression were affected, as well as redistribution of subcluster n8 from Gnai2 neurons. VR expression does not seem to be the primary determinant of VSN cluster identity.

      Reviewer #2 (Public Review): 

      Summary: 

      The study focuses on the vomeronasal organ, the peripheral chemosensory organ of the accessory olfactory system, by employing single-cell transcriptomics. The author analyzed the mouse vomeronasal organ, identifying diverse cell types through their unique gene expression patterns. Developmental gene expression analysis revealed that two classes of sensory neurons diverge in their maturation from common progenitors, marked by specific transient and persistent transcription factors. A comparative study between major neuronal subtypes, which differ in their G-protein sensory receptor families and G-protein subunits (Gnai2 and Gnao1, respectively), highlighted a higher expression of endoplasmic reticulum (ER) associated genes in Gnao1 neurons. Moreover, distinct differences in ER content and ultrastructure suggest some intriguing roles of ER in Gnao1-positive vomeronasal neurons. This work is likely to provide useful data for the community and is conceptually novel with the unique role of ER in a subset of vomeronasal neurons. This reviewer has some minor concerns and some suggestions to improve the manuscript. 

      Strengths: 

      (1) The study identified diverse cell types based on unique gene expression patterns, using single-cell transcriptomic. 

      (2) The analysis suggests that two classes of sensory neurons diverge during maturation from common progenitors, characterized by specific transient and persistent transcription factors. 

      (3) A comparative study highlighted differences in Gnai2- and Gnao1-positive sensory neurons. 

      (4) Higher expression of endoplasmic reticulum (ER) associated genes in Gnao1 neurons. 

      (5) Distinct differences in ER content and ultrastructure suggest unique roles of ER in Gnao1-positive vomeronasal neurons. 

      (6) The research provides conceptually novel on the unique role of ER in a subset of vomeronasal neurons, offering valuable insights to the community. 

      Weaknesses: 

      (1) The connection between observations from sc RNA-seq and EM is unclear.

      (2) The lack of quantification for the ER phenotype is a concern. 

      We have extensively quantified the ER phenotype as shown in Figure 7, Figure 7-figure supplement-1 in our revised version. We would like to point out that the connection between scRNA-seq and EM was made due to our observations in the same figures, that levels of a number of ER luminal and ER membrane proteins were higher in Gnao1 compared to Gnai2 neurons. This led us to hypothesize a differential ER content or ultrastructure, which was verified by EM.

      Reviewer #3 (Public Review): 

      Summary: 

      In this manuscript, Devakinandan and colleagues have undertaken a thorough characterization of the cell types of the mouse vomeronasal organ, focusing on the vomeronasal sensory neurons (VSNs). VSNs are known to arise from a common pool of progenitors that differentiate into two distinct populations characterized by the expression of either the G protein subunit Gnao1 or Gnai2. Using single-cell RNA sequencing followed by unsupervised clustering of the transcriptome data, the authors identified three Gnai2+ VSN subtypes and a single Gnao1+ VSN type. To study VSN developmental trajectories, Devakinandan and colleagues took advantage of the constant renewal of the neuronal VSN pool, which allowed them to harvest all maturation states. All neurons were re-clustered and a pseudotime analysis was performed. The analysis revealed the emergence of two pools of Gap43+ clusters from a common lineage, which differentiate into many subclusters of mature Gnao1+ and Gnai2+ VSNs. By comparing the transcriptomes of these two pools of immature VSNs, the authors identified a number of differentially expressed transcription factors in addition to known markers. Next, by comparing the transcriptomes of mature Gnao1+ and Gnai2+ VSNs, the authors report the enrichment of ER-related genes in Gnao1+ VSNs. Using electron microscopy, they found that this enrichment was associated with specific ER morphology in Gnao1+ neurons. Finally, the authors characterized chemosensory receptor expression and coexpression (as well as H2-Mv proteins) in mature VSNs, which recapitulated known patterns. 

      Strengths: 

      The data presented here provide new and interesting perspectives on the distinguishing features between Gnao1+ and Gnai2+ VSNs. These features include newly identified markers, such as transcription factors, as well as an unsuspected ER-related peculiarity in Gnao1+ neurons, consisting of a hypertrophic ER and an enrichment in ER-related genes. In addition, the authors provide a comprehensive picture of specific co-expression patterns of V2R chemoreceptors and H2-Mv genes. 

      Importantly, the authors provide a browser (scVNOexplorer) for anyone to explore the data, including gene expression and co-expression, number and proportion of cells, with a variety of graphical tools (violin plots, feature plots, dot plots, ...). 

      Weaknesses: 

      The study still requires refined analyses of the data and rigorous quantification to support the main claims. 

      The method description for filtering and clustering single-cell RNA-sequencing data is incomplete. The Seurat package has many available pipelines for single-cell RNA-seq analysis, with a significant impact on the output data. How did the authors pre-process and normalize the data? Was the pipeline used with default settings? What batch correction method was applied to the data to mitigate possible sampling or technical effects? Moreover, the authors do not describe how cell and gene filtering was performed. The data in Figure 7-Supplement 3 show that one-sixth of the V1Rs do not express any chemoreceptor, while over a hundred cells express more than one chemoreceptor. Do these cells have unusually high or low numbers of genes or counts? To exclude the possibility of a technical artifact in these observations, the authors should describe how they dealt with putative doublet cells or debris. Surprisingly, some clusters are characterized by the expression of specific chemoreceptors (VRs). Have these been used for clustering? If so, clustering should be repeated after excluding these receptors. 

      The identification of the VSN types should be consistent across the different analyses and validated. The data presented in Figure 1 lists four mature VSN types, whereas the re-clustering of neurons presented in Figure 3 leads to a different subdivision. At present, it remains unclear whether these clusters reflect the biology of the system or are due to over-clustering of the data, and therefore correspond to either noise or arbitrary splitting of continua. Clusters should be merged if they do not correspond to discrete categories of cells, and correspondence should be established between the different clustering analyses. To validate the detected clusters as cell types, markers characteristic of each of these populations can be evaluated by ISH or IHC. 

      There is a lack of quantification of imaging data, which provides little support for the ERrelated main claim. Quantification of co-expression and statistics on labeling intensity or coverage would greatly strengthen the conclusions and the title of the paper. 

      a) scRNA-seq data analysis methods: Our revised submission has expanded on the methods section with details of parameters, filtering criterion and software used.

      b) Inclusion/exclusion of VRs: Figure 3-Figure supplement-2 of our revised submission shows a comparison of neuronal sub-clusters with and without VRs. Overall sub-cluster identities were not affected by VR exclusion, except for Gnao1 sub-clusters n1/n3 -governed by family C Vmn2r1/Vmn2r2 and redistribution of Gnai2 cluster n8. The minimal effect of VRs on Gnai2 sub-clustering can also be confirmed by lack of V1R in the dot plot showing markers of neuronal clusters. 

      c) Neuronal clusters and potential over-clustering: we pooled neuronal cells from Figure-1 and re-clustered to identify sub-populations within Gnao1 and Gnai1 neurons. Several neuronal sub-clusters identified by us including progenitors, immature neurons and mature neurons are validated by previous studies with wellknown markers. Amongst the mature neurons, the biological basis of four Gnao1 neuron sub-clusters (n1-n4) is discussed in our co-expression section (Figure 4AE) and these are also validated by previous experimental studies. These Gnao1 clusters are organized according to the expression of family-C V2Rs (Vmn2r1 or Vmn2r2) as well as H2M_v_ genes. Within Gnai2 sub-clusters, n12 and n13 exclusively express markers that distinguish them from n8-n11 which we have described in our revised version. However, n8-n11 do not have definitive markers and whether these sub-clusters are part of a continuum or over-clustered, will require further extensive experiments and analysis. We prefer to show all subclusters, including Gnai2 sub-clusters, in Figure 3-Figure supplement-1, along with a dot plot of sub-cluster gene expression, so that this data is available for future experiments and analysis.  We share the concern that some Gnai2 sub-clusters may not have an obvious biological basis at this time. Hence in our revised submission, we have merged mature Gnao1 and mature Gnai2 sub-clusters for the developmental analysis shown in Figure 3A. 

      d) Quantification of the ER phenotype: In our revised submission, we provide extensive quantification of the ER phenotype in Figure 7, Figure7-figure supplement-1.   

      e) We think that the cells expressing zero as well as two V1Rs are real and cannot be attributed to debris or doublets for the following reasons:

      i) Cells expressing no V1Rs are not necessarily debris because they express other neuronal markers at the same level as cells that express one or two V1Rs. For instance, Gnai2 expression level across cells expressing 0, 1, 2 V1Rs is the same, which we have included in Figure 4-figure supplement 4-C of our revised submission. Higher expression threshold value used in our analysis may have somewhat increased the proportion of cells with zero V1Rs. Similarly, Gnao1 levels across cells expressing multiple V2Rs and H2-M_v_ per cell stay the same, indicating that these are unlikely to be doublets (Figure 4 I-K). The frequency of each co-expression combination (Supplementary Table 7 and 8) itself is an indication of whether it is represented by a single cell or an artifact.

      ii) Cells co-expressing V1R genes: We listed the frequency of cells co-expressing V1R gene combinations in Supplementary table - 8. Among 134 cells that express two V1Rs, 44 cells express Vmn1r85+Vmn1r86, 21 express Vmn1r184+Vmn1r185, 13 express Vmn1r56+Vmn1r57, 6 express Vmn1r168+Vmn1r177, and so on. Doublets generally are a random combination of two cells. Here, each specific co-expression combination represents multiple cells and is highly unlikely by random chance. Some of the co-expression combinations we reported were also identified and verified experimentally in Lee et al., 2019 and Hills et. al., 2024.  

      Recommendations for the authors:

      Reviewing Editor (Recommendations for the Authors): 

      The editor had a query about the analysis of FPRs, which are a third family of sensory receptors in the rodent VNO. 

      FPRs were found in our study as expressed in subsets of Gnai2 and Gnao1 neurons as well as non-neuronal cells. These can be easily searched in www.scvnoexplorer.com. For instance, Fpr1 and Fpr2 are expressed in immune cell clusters - 2,6,8,10; whereas Fpr-3 is expressed in Gnao1 subcluster n1. Consistent with earlier reports (10.1073/pnas.0904464106, 10.1038/nature08029) expression of Fpr-rs3, Fpr-rs4, Fprrs6, Fpr-rs7 is restricted to Gnai2 neurons, of which Fpr-rs3 and Fpr-rs4 are limited to Tmbim1+ Gnai2 neurons.  

      Reviewer #1 (Recommendations For The Authors):

      (1) The reference to "genders" on page 3 should be changed to "sexes". 

      We have modified the text.   

      (2) Did the authors identify any Ascl1+ GBCs in their data? 

      Ascl1+ GBCs were identified and are now marked in our revised version Figure3-figure supplement 1B.    

      (3) The plots in Figures 1B and 2B say they're depicting gene "Expression", but it looks like the gene expression was z-scored. If so, the authors should describe how the expression was scaled. 

      We have modified the legend title to ‘scaled expression’ and described the basis of scaling in the methods section of our revised version. 

      (4) The main text mentions Figure 2C, but maybe this refers to the right part of Figure 2B?

      Panel 2C was mistakenly not marked in the figure. We have now marked it in revised Figure 2.    

      (5) The authors should attempt to describe the other branch points in the trajectory shown in Figure 3A. If they don't seem biologically plausible, then the authors might want to reconsider using Slingshot for their analyses.

      We do not seek to claim additional branch points within mature Gnao1 or Gnai2 neurons from our analysis. Whether there exist additional branch points leading to subcategories within mature neurons, requires extensive experimental investigation. Hence, in our revised submission, we have merged mature Gnai2 / Gnao1 subclusters for pseudotime developmental analysis and to keep our analysis focused on the single branch point at immature neurons.    

      (6) The most significantly enriched gene in Figure 3B in immature Gnao1+ neurons is Cnpy1, which is also an ER protein. It could also be interesting to look at its expression or speculate on its function in immature neurons. 

      Multiple ER genes were found to be enriched in Gnao1 neurons. We would not be comfortable speculating on the function of individual genes, without a proper study, which is beyond the scope of this manuscript.      

      (7) For figures with pseudo-colored expressions, it would be useful to have color bars. I'm also not sure the pseudocolors are necessary; presenting the data in grayscale or a single color like green might also be sufficient. 

      We used pseudocolor in the IHC images of ER proteins, because there is a wide variation in the fluorescence signal intensity across apical to basal axis for various proteins. In some cases, gray scale images could lead to the false impression that there is no signal in apical Gnai2 neurons, whereas pseudocolor shows low fluorescence level in these neurons. We have added intensity scale bar to the figures in our revision version.  

      (8) For in situ images with two colors it would be more colorblind-friendly to use green and magenta rather than green and red.

      Since no single color palette can help readers with different types of colorblindness, we decided to rely on user’s operating systems that offer rendering of the images to a color palette based on their type of colorblindness. We believe this  would be a better option as described here: https://markusmeister.com/2021/07/26/figure-design-for-colorblindreaders-is-outdated/

      (9) The heatmap in Figure 7E would likely look more accurate without interpolation/aliasing/smoothing. 

      We have not performed smoothening on any of the heatmaps. We have noticed that sometimes heatmaps take time to load in software (such as Adobe Acrobat) leading to the impression of smoothing. Changing the zoom level or reopening the file may fix this.     

      (10) Rather than just citing the literature on the unfolded protein response in the MOE, it could be useful to cite work on the ATF5 expression and the UPR in the VNO (e.g.

      10.1101/239830v1 or 10.12688/f1000research.13659.1).

      We have cited and commented on the ATF5 VNO expression in our discussion. 

      (11) I might try to condense the discussion. Additionally, in the discussion, the section on receptor co-expression comes before that on the VNO ER, so I might consider reorganizing the figures and results to present all of the scRNA-seq analyses (including the receptor co-expression figure) first before the figures on the ER. 

      We welcome this suggestion and have reorganized figures and results such that the scRNA-seq analysis flow is maintained before ER results.   

      Reviewer #2 (Recommendations For The Authors): 

      (1) Upregulation of ER-related mRNAs and expanded ER lumen in Gnao1-positive neurons is interesting, but the connection between these observations is unclear. The authors can strengthen the link by adding immunohistochemistry of representative ER proteins to test if the upregulation of mRNAs related to ER results in increased levels of these proteins in the ER of these neurons.

      Connection between scRNA-seq and EM was made due to our observations that levels of a number of ER luminal and membrane proteins were higher in Gnao1 compared to Gnai2 neurons (Figure 7, Figure 7-figure supplement-1 in our revised submission). This led us to hypothesize a differential ER content or ultrastructure, which was verified by EM. We have also addressed the question of whether upregulation of mRNAs related to ER proteins results in their increased levels (Figure 7-figure supplement-2). In some cases, for example Hspa5 (Bip), mRNA as well as protein levels are upregulated in Gnao1 neurons (see Figure 3A volcano plot, Figure 5-figure supplement-1 RNA-ISH, Figure 7-figure supplement-1 comparison of mRNA levels, Figure 7F immunofluorescence). However, there are other genes in the same figures, for which mRNA levels are not upregulated, yet protein levels are higher in Gnao1 neurons. As mentioned in our text and discussion, upregulated mRNA levels as well as post-transcriptional mechanisms are both likely to play a role in upregulating ER protein levels in Gnao1 neurons.       

      (2) In Figure 3, the authors seemed to exclude cluster 13 from Figure1 in the pseudotime analysis without justification. 

      Cluster13 has markers such as Obp2a, Obp2b, Lcn3. We confirmed via RNA-ISH (Figure 2-figure supplement-1A in our revised submission) that Obp2a maps to cells from glandular tissue on the non-sensory side. Cluster-13 also has cells expressing Vmn1r37, which typically is expressed in neuronal cells. However, we do not see Obp2a mRNA in the sensory epithelium. It is possible that cluster-13 comprises a heterogenous mixture of cells, some of which are non-sensory glandular cells, co-clustered with other cell types as well as the possibility that Obp2a is expressed in neurons, below the detection level of our assay. Further experiments will be required to distinguish between these possibilities. We do not have any possible reason to confidently assign this cluster as a neuronal cell type, hence, it was excluded in the downstream analysis of neurons.

      (3) In Figure 3, the line appears to suggest that Gnao1-positive cells can be progenitors of Gnai2-positive cells. Please clarify. 

      We thank the reviewer for pointing this out. We did not seek to give the impression that Gnao1 cells can be progenitors of Gnai2 cells. This may be due to the placement of dots in the trajectory leading to misinterpretation and the UMAP itself. We have modified the pseudotime trajectory in our revised version to make it more intuitive. 

      (4) Figure 3: Please label pseudotime lineage cluster identities. 

      Cluster identities are now labeled in Figure 3A pseudotime lineage as well as in Figure 3-figure supplement-1 dot plot.     

      (5) Figure 4: Please label the genes used for in situ hybridization in the volcano plot. 

      Genes used for RNA-ISH are labeled (bold font) in the volcano plot in Figure 5A.  

      (6) Figure 4: Please clarify which genes shown in the in situ hybridization figures correspond to which GO terms. 

      We have added supplementary table-10 containing gene ontology terms associated with genes for which RNA-ISH was performed. 

      (7) The EM shown in Figure 5 makes this work unique and intriguing. However, the lack of quantification for the ER phenotype is a concern. For example, does the ER area of a given cell correlate with the relative position of the cells along the apical-basal axis of the vomeronasal organ? What about the ER morphology in the progenitor cells? 

      We show here a quantification of the ER area from the low magnification EM image shown in Figure 8A. The ER area shows an increase going towards the basal side of the cross-section. However, this quantification is complicated by the following factors: a) Processing for EM, results in some shrinkage of the tissue, b) Gnao1 neurons follow an invaginating pattern in cross-sections. Due to these reasons, some Gnao1 neurons could come very close to, and at times lie adjacent to Gnai2 neurons in EM cross-section. Due to a lack of contrast, it is harder to identify the ER within the cell at low mag, especially in the apical zone. The plot shown here does indicate that roughly, the ER area of a cell correlates with its position along the apical-basal axis. In our revised submission, we have quantified the fluorescence intensities of various ER proteins along the apical basal axis from confocal images (Figure 7, Figure 7-figure supplement-1).    

      Author response image 2.

      ROIs (yellow) are manually drawn in the sensory epithelium, wherever possible to identify ER without ambiguity. Area and centroid of ROI are calculated and x coordinates of centroid of each ROI are used to position ER area along the apical-basal axis as shown in the plot below.

      Establishing ER ultrastructure in progenitor or immature cells, as well as unambiguous quantification of ER area in mature neurons, requires identification of these cells in crosssections using fluorescent molecular markers, followed by performing correlative light and electron microscopy (CLEM). This procedure being technically challenging is beyond the scope of our manuscript.      

      Reviewer #3 (Recommendations For The Authors): 

      (1) The main claim is about ER differences between Gnao1+ and Gnai2+ VSN. The ISH, IHC, and EM microscopy images are not quantified and, therefore, poorly support this main claim.

      In our revised submission, we provide extensive quantification of the ER phenotype in Figure 7, Figure7-Figure supplement-1.  Quantification of ER area from EM images is challenging and described above it in our response to reviewer #2 recommendation 7.

      (2) The annotation of VSN subclusters should be more rigorous, consistent throughout the paper (VSN clusters are inconsistent between Figure 1 and Figure 3, and the multiplication of subclusters in Figure 3 is not discussed), and verified (using ISH or IHC) that they reflect discrete, actual cell types. The authors should provide a list of differentiating marker genes for the clusters in Figure 3. At present, it remains unclear whether these clusters are the result of over-clustering of cells (and therefore represent either noise or arbitrary splits of continua) or whether they reflect the biology of the system. Subsequent characterization of these curated VSN subtypes (as done in Figure 4) would add value to the study.

      We pooled neuronal cells from Figure-1 and re-clustered at higher resolution to identify subtypes. Several neuronal sub-clusters identified by us including progenitors, immature neurons and mature neurons are validated by previous studies with well-known markers. Amongst the mature neurons, the biological basis of four Gnao1 neuron sub-clusters (n1n4) is discussed in our analysis and these are also validated by previous experimental studies. These Gnao1 clusters are organized according to the expression of family-C V2Rs (Vmn2r1 or Vmn2r2) as well as H2Mv genes. Within Gnai2 sub-clusters, n12 and n13 exclusively express markers that distinguish them from n8-n11 which we have described in our revised version. However, Gnai2 n8-n11 do not have definitive markers and whether these sub-clusters are part of a continuum or over-clustered, will require further extensive experiments and analysis. We prefer to show all sub-clusters, including Gnai2 sub-clusters, in Figure 3-Figure supplement-1, along with a dot plot of sub-cluster gene expression, so that this data is available for future experiments and analysis. We share the concern that some Gnai2 sub-clusters may not have an obvious biological basis at this time. Hence in our revised submission, we have merged mature Gnao1 and mature Gnai2 sub-clusters for the developmental analysis shown in Figure 3A.

      (3) Some clusters are characterized by the expression of specific chemoreceptors (VRs). Have these been used for clustering? If so, clustering should be repeated after excluding these receptors.

      Figure 3-Figure supplement-2 of our revised submission shows a comparison of neuron clusters with and without VRs. We also describe in the results, specific clusters that are affected by exclusion of VRs.  

      (4) Given the title and the data, the paper should be structured around its main claim (i.e. differential ER environment between VSN types). For example, Figure 7, which deals with the characterization of receptor expression and co-expression in VSNs, is sandwiched between the validation of ER substructure (Figure 6) and the timing of coexpression of ER chaperone genes (Figure 8). The data presented in Figure 7 would fit better if used as a validation of the dataset prior to the investigation presented in the current Figure 4. In addition, we suggest that expression and co-expression diagnostics should be used to filter cells for subsequent analyses.

      We appreciate this suggestion and have reorganized the figures in our revised version.  Our subsequent analysis showing enrichment of ER related genes at RNA, protein level covers all Gnao1 neurons and is not restricted to a specific subset. This is reflected in the ISH and IHC of ER genes. 

      (5) Figure 7-Supplement 3 suggests the presence of co-expressed V1Rs in VSNs. It is unclear from the data presented whether these co-expressing cells are artifactual cell doublets and should be removed from the analysis or whether the expression of the coexpressed receptors reflects a reality. To better address this observation, one may want to see the expression levels of the individual co-expressed V1rs in Figure 7-Supplemet 3 rather than the sum of V1r expression. I am also concerned about the unusually high frequency of "empty" neurons (i.e. without expressed VRs). Could these be debris? 

      We think that the cells expressing zero as well as two V1Rs are real and cannot be attributed to debris or doublets for the following reasons:

      i) Cells expressing no V1Rs are not necessarily debris because they express other neuronal markers at the same level as cells that express one or two V1Rs. For instance Gnai2 expression level across cells expressing 0, 1, 2 V1Rs is the same, which we have included in Figure 4-figure supplement 4-C of our revised submission. Higher expression threshold values used in our analysis may have somewhat increased the proportion of cells with zero V1Rs. Similarly, Gnao1 levels across cells expressing multiple V2Rs and H2-M_v_ per cell stay the same, indicating that these are unlikely to be doublets (Figure 4 I-K). As doublets are formed randomly, the frequency of each co-expression combination (Supplementary Table 7 and 8) itself is an indication of whether it is represented by a single cell or an artifact.

      ii) Cells co-expressing V1R genes: All cells used for co-expression analysis were filtered via an expression threshold (Figure 4-figure supplement 1D), which eliminates cells with low counts of V1R expression. We listed the frequency of cells co-expressing V1R gene combinations in Supplementary table - 8. Among 134 cells that express two V1Rs, 44 cells express Vmn1r85+Vmn1r86, 21 express Vmn1r184+Vmn1r185, 13 express Vmn1r56+Vmn1r57, 6 express Vmn1r168+Vmn1r177, and so on. Doublets generally are a random combination of two cells. Here, each specific co-expression combination represents multiple cells and is highly unlikely by random chance.  iii) Some of the co-expression combinations we reported were identified earlier and verified experimentally in Lee et al., 2019 using FACS based single collection in 96-well plates following the cellseq-2 protocol with very low chance of doublets, and Hills et. al., 2024.  

      (6) The authors use either dot plots or scatter plots to show gene expression in cell clusters. It looks nice, but it is very difficult to deduce population levels of expression from these plots. Could we see the distribution of gene expression across clusters using more quantitative visualizations such as violin or box plots?

      Dot plots are majorly used in our manuscript to show markers of cell clusters in Figure 1, Figure 2 and Figure 3-figure supplement 1. We would like to show at least 5 gene markers for each cluster that are important to identify the cell type. Using violin plot or bar plot for this will make the panel extremely big and overwhelming, especially with 16 clusters in Figure 1 and 13 clusters in Figure 3-figure supplement 1 or make the bars/violin too small to interpret.  Hence, for the sake of simplicity, we used dot plots to give our reader a birds-eye of gene expression differences across clusters. Scatter plots were used when we want to compare the expression levels of genes between male and female samples and show the expression of two genes (VRs) simultaneously in a single cell. This cannot be achieved by Violin/box plot. However, we have made our dataset available at scvnoexplorer.com to explore the expression patterns across cell clusters with different visualization options, including violin or box plots.  

      (7) To investigate whether sex might bias clustering, the authors calculated the Pearson coefficient of gene expression between sexes for each cluster. Given the high coefficient observed across all clusters (although no threshold is used), the authors conclude that there was no bias. While the overall effect may show a strong similarity in gene expression in each cluster between the sexes, this overlooks all the genes that are significantly differentially expressed. It would be worth investigating and discussing these differences. Relatedly, what batch correction method was applied to the data (to mitigate any possible sampling or technical effect)?

      We chose the Pearson coefficient as a representative parameter to show that there is no bias. In addition, we have performed differential expression analysis for each cluster and the results are in supplementary table-1. Except known sexually dimorphic genes, other genes are not differentially expressed significantly with adjusted p-values greater than 0.05. This was also shown by earlier studies using bulk RNAseq (doi.org/10.1371/journal.pgen.1004593, doi.org/10.1186/s12864-017-4364-4). We used depth normalization to integrate samples and described this in the methods section of our revised version.

      (8) We found the method description to be incomplete for the single-cell RNA sequencing analyses. The method section should include a detailed explanation of the code used by the authors to analyze the data. The Seurat package has many available pipelines for single-cell RNA-seq analysis, which have a major impact on the output data. It is therefore imperative to describe which of these pipelines were used and whether the pipeline was run with default settings. 

      Our revised submission has expanded on the methods section with details of parameters, filtering criterion and software used.

    1. Author response:

      The following is the authors’ response to the original reviews.

      Public Reviews:

      Reviewer #1 (Public Review):

      This study uses a variety of approaches to explore the role of the cerebellum, and in particular Purkinje cells (PCs), in the development of postural control in larval zebrafish. A chemogenetic approach is used to either ablate PCs or disrupt their normal activity and a powerful, high-throughput behavioural tracking system then enables quantitative assessment of swim kinematics. Using this strategy, convincing evidence is presented that PCs are required for normal postural control in the pitch axis. Calcium imaging further shows that PCs encode tilt direction. Evidence is also presented that suggests the role of the cerebellum changes over the course of early development, although this claim is rather less robust in the current version of the paper. Finally, the authors build on their prior work showing that both axial muscles and pectoral fins contribute to "climbs" and show evidence that suggests PCs are required for correct engagement of the fins during this behaviour. Overall, establishing a role for the cerebellum in postural control is not very surprising. However, a clear motivation of this study was to establish a robust experimental platform to investigate the changing role of cerebellar circuits in the development of postural control in the highly experimentally accessible zebrafish larvae, and in this regard, the authors have certainly succeeded.

      Overall, I consider this an excellent paper, with some room for improvement in aspects of presentation, discussion, and some aspects of the data analysis..

      We thank the reviewer for their kind comments and support. In the revision we have addressed their concerns regarding data presentation and analysis. Additionally, we have expanded our introduction and discussion to address questions of presentation.  

      Reviewer #2 (Public Review):

      Summary:

      Franziska Auer et al. investigate the role of cerebellar Purkinje cells in controlling posture in larval zebrafish using the chemogenetic tool TRPV1/capsaicin to bidirectionally manipulate (i.e., activate or ablate) these cells. This tool has been developed for zebrafish previously but has not been applied to Purkinje cells.

      High-throughput behavioral experiments are presented to monitor how body posture is affected by these perturbations. The analysis of postural control focuses on a specific subaspect of posture: the body tilt-angle relative to horizontal just before a swim bout is executed, quantified separately for pre-ascent and pre-dive bouts. They report a broad bimodal distribution of pre-ascent bout posture ranging from -20 to +40 degrees, while the pre-dive bout posture was more Gaussian, ranging between -40 and 0 degrees. The treatment effect is quantified as the change in the median of these distributions.

      Purkinje cell activation and ablation in 7 days post-fertilization (dpf) fish shifted the median of the ascending bout posture distributions to positive values. The authors hypothesize that the stochastic nature of the activation process might desynchronize Purkinje cell activity, thus abolishing Purkinje cells' role in postural control, similar to ablation. However, this does not explain why dive bout posture decreased upon activation but was unaffected by ablation. 

      To test whether the role of Purkinje cells in postural control matures over development, the authors repeated the ablation experiments at 14 dpf. They state that "at 14 dpf, the effects of Purkinje cell lesions on posture were more widespread than at 7 dpf." However, this effect size is comparable to that observed at 7 dpf, suggesting no further maturation of the role of Purkinje cells in pre-ascending bout postural control. The median pre-dive bout posture decreased at 14 dpf, contrasting with no effect at 7 dpf, yet this change was comparable in effect size to the activation effect on Purkinje cells at 7 dpf. The current data breadth may not be sufficient to conclude that signatures of emerging cerebellar control of posture across early development were uncovered.

      The study's exploration of activating Purkinje cells in freely swimming fish using TRPV1/ capsaicin is of special interest, but the practicability of this method is unclear from the current presentation. It would be beneficial to present the distribution of the percentage of activatable Purkinje cells across animals and time points to provide insight into the method's efficiency. Discussing this limitation and potential improvements would aid in evaluating the method, especially since the authors report that the activation experiments were labor-intensive, limiting repeat experiments. This may explain why the activation experiment at 7 dpf is the only data presented with cell activation, with other analyses performed using the cell ablation capabilities of the TRPV1/capsaicin method.

      Another data point at 14dpf would significantly strengthen the conclusions.

      The authors analyze Purkinje cell-controlled fin-trunk coordination by examining ascending bout posture across different swim bout speeds. They make the important finding that pectoral fin movements contribute significant lift for median and fast swim bouts but not for slow ones, and that Purkinje cell ablation disrupts lift generation at all speeds.

      Finally, the authors examined whether Purkinje cell activity encodes postural tilt-angle by performing calcium imaging on 31 cells from 8 fish using their Tilt In Place Microscope (TIPM). They report that they could decode the tilt-angle from individual neurons with a highly tuned response, and also from neurons that were not obviously tuned when pooling them and analyzing the population response. However, due to the non-simultaneous recordings across animals, definitive conclusions about populationlevel encoding should be made cautiously, it might be better to suggest potential population encoding that needs confirmation with more targeted experiments involving simultaneous recordings.

      Strengths:

      - The study introduces a novel application of the chemogenetic tool TRPV1/capsaicin to study cerebellar function in zebrafish.

      - High-throughput behavioral experiments provide detailed analysis of postural control.

      - The further investigation of Purkinje cell-controlled fin-trunk coordination offers new insights into motor control mechanisms.

      - The use of calcium imaging to decode postural tilt-angle from Purkinje cell activity presents interesting preliminary results on neuronal population encoding.

      Weaknesses:

      - The term "disruption" for postural control effects may lead to misleading expectations.

      - The supporting data show only subtle median shifts in postural angle, raising questions about the significance of observed effects. Statistical methods that account for the hierarchical structure of the data might be required to support the conclusions.

      - The study's data breadth may not be sufficient to conclude emerging cerebellar postural control across early development.

      - The current presentation does not adequately detail the practicability and efficiency of the TRPV1/capsaicin method for activating Purkinje cells, and the labor-intensive nature of these experiments constrains the ability to replicate and validate the findings.

      - Non-simultaneous recordings in calcium imaging necessitate cautious interpretation of population-level encoding results.

      We appreciate the reviewer's thoughtful and detailed feedback. In response, we have made several changes to highlight key points in our manuscript. We have adjusted our wording to more accurately reflect the scope of our findings. Finally, we have clarified and expanded the methods used.

      Reviewer #3 (Public Review):

      Summary:

      This paper uses a new chemogenetic tool to investigate the role of cerebellar Purkinje cells in postural control. Using a high-throughput behavioral assay, they show that activation or ablation of Purkinje cells affects various aspects of postural control in zebrafish larvae during spontaneous swimming and that the effects are more pronounced at later developmental time points, where the Purkinje cell number is much greater. Using a sophisticated imaging assay, they record Purkinje cell activity in response to the tilt of the fish and show that some Purkinje cells are tuned to tilt direction and that the direction can even be decoded from untuned neurons.

      Strengths:

      Overall the study is nice, using a range of tools to address a fundamental question about the role of the cerebellum in postural control in fish.

      Weaknesses:

      (1) The data in Figure 1 that establishes the method seems to be based on a very small number of experiments and lacks some statistical analysis.

      (2) The choice and presentation of the statistical and analysis methods used in Figures 2-5 could be improved.

      We thank the reviewer for their comments.  We have added additional statistical analyses for the activation experiments, and improved data presentation .

      Recommendations for the authors:

      Reviewer #1 (Recommendations For The Authors):

      Overall I think this is a great paper.

      * Introduction and Discussion.

      The Introduction (and Discussion) do little to explain what is understood about cerebellar control of posture and what major outstanding questions remain. The first paragraph of the Introduction seems to argue that the role of the cerebellum in control of posture is well established and line 24 attempts to motivate the present study by virtue of the fact that terrestrial locomotion is "complex". This might be true but is not necessarily a major obstacle given the suite of powerful approaches available in rodent neuroscience. What are the major challenges that are hard to tackle in rodents and what specific questions can the larval zebrafish help to answer? What about development (which gets no mention at all)? I'm not suggesting a comprehensive review of every aspect of cerebellar physiology, but I think the Introduction should attempt to outline the current hypotheses in a little more detail and highlight what we still need to understand.

      We take the Reviewer’s point that there is more to say in the Introduction. We feel that multi-dimensional limb biomechanics and proprioception are two aspects of terrestrial locomotion that support our use of the word “complexity.” However, we don’t dwell on this point because, as the reviewer correctly states, the suite of tools for rodent neuroscience & behavior is expansive and, in our opinion, not a limiting factor. Instead, we said what we felt we could regarding the potential contribution of the larval zebrafish in the last paragraph of the Discussion. In the revision, we have added details about the development of cerebellum to the introduction (though this, of course, is an expansive topic and well-beyond the scope of the Introduction), highlighted some of the historical limitations in rodent posture analysis, and set up the .

      * Figure 2: 'Arrows denote the shift towards more nose-up postures'. I think the distribution is quite easy to interpret without these arrows; I suggest removing them.

      We have removed the arrows.  

      * IQR is sometimes stated as a single number and sometimes as a range. It should be consistent and unless eLife has guidance to the contrary, I suggest that it be the latter.

      Thank you for pointing that out. We now report it as the value at the 25&75th %ile for all IQRs.  

      * Figure S2: For 14 dpf fish the axes are labelled PC2/3 - is this an error?

      We have changed it to a 3-dimensional plot for both 7 and 14 dpf data to show comparable plots for both ages (now Figure S5 F and G). For the analysis in the 14dpf fish the clearest separation was in the space defined by the 2nd and 3rd principal component.  

      * In the methods, there is insufficient detail given about fluorescent imaging.

      We added additional information to how the fluorescent imaging was performed to the ‘Confocal imaging’ section as well as to the ‘Functional imaging section’

      * Abstract

      In my opinion, the statement "Here, we used a powerful chemogenetic tool (TRPV1/ capsaicin) to *define the role of Purkinje cells*..." is too strong. Whilst the evidence that PCs are required for postural control is certainly strong, what exactly these cells do in the service of postural control is far from clear (as the authors indeed acknowledge in the Discussion). As such, I wouldn't say their role has been "defined".

      We change the word to “describe” to better reflect our findings

      * aldoca transgenic.

      This appears to be a beautiful transgenic line but the data showing the extent of its expression and evidence that in the cerebellum it exclusively labels PCs isn't clear enough.

      (i) Ideally Figure 1A would show an image of a whole animal to provide an overview of transgene expression but instead it seems to be (the legend is unclear) a cartoon with a confocal projection of part of the brain overlaid.

      We have updated the figure legend to be clearer that we show a cartoon of a larval zebrafish with the confocal image overlaid. The aldoca promotor has been previously described and exclusively labels Purkinje cells (10.1523/JNEUROSCI.3352-10.2010)

      (ii) Figure 1B shows expression in the cerebellum, but how are we to understand that all the labelled cells are PCs? Are all PCs labelled, or only a subset? Perhaps a double labelling with a PC in situ marker could be done to demonstrate colocalisation?

      As above, the aldoca promotor has been previously described; to the best of our knowledge in the Hibi lab’s hands (and ours) it labels Purkinje cells exclusively, and it labels all of them (10.1523/JNEUROSCI.3352-10.2010)  

      * Chemogenetic validation.

      Overall, the chemogenetic approach to abrogate PC function looks to be very powerful. The authors state in several places that a contribution of this paper is in its "establishing the validity of TRPV1/capsaicin-mediated perturbations". However, the data in Figure 1, along with various comments in other parts of the paper raise some questions:

      (i) For experiments depolarising PCs with 1µM CSn, the same size is tiny: Two transgenic animals and one control. Moreover, it is stated 'in one fish ... we observed a small number of neurons at the 9h timepoint with bright, speckled fluorescence suggestive of cell death". Was this one out of two transgenics?! In the discussion, I didn't understand the statement "ensure adequate brightness levels *to achieve sufficient depolarization without excitotoxicity*". Does this "excitotoxicity" relate to the specked fluorescence observation?

      Overall, the very small sample size and comments about excitotoxicity and cell death raise concerns about the approach that I think warrant clearer treatment in the results (including information about the assessment of transgene expression, % embryos judged to have suitable expression), especially as this paper is seeking to establish the validity of the method.

      We note first that the method has been previously validated (https://doi.org/10.1038/ nmeth.3691) and that we build on this work. For the experiment described, the point was to identify an acceptable duration for exposure. To that end, we analyzed 6 animals for up to 6h (including the washout experiments in Figure S1B) where we never observed any speckled fluorescence; we limited our behavioral experiments to 6h accordingly. We thought it would be worth including the observation of speckled fluorescence at 9h timepoint for future reference. To directly address the comment we have increased the number of analyzed cells and fish for the 1uM capsaicin experiments and added statistical analysis (lines 65-67).

      When screening for transgene expression we selected for fish that had clearly visible expression, but that did not look overly bright, and used the same criteria when screening fish for the GCaMP imaging and for behavior. Around a quarter of the fish that had aldoca:TRPV1-tagRFP expression had a usable expression level for the activation experiment. We have added this information to the Results (line 62) and Methods (line 369-372)

      (ii) The authors note "capsaicin could sporadically activate subsets of Purkinje cells" and further speculate about PC activity and synchrony in the discussion. Figure 1 seems to rely on single images at widely spaced time points but given that they are set up to do 2-photon calcium imaging, why didn't they collect continuous time series data and analyse the temporal patterns of activity across the transgenic PC population?

      We have added time series data for calcium imaging after 1uM of Capsaicin in TRPV1-  and TRPV1+ cells to Supplementary Figure S1A. Here too we see sporadic increases in calcium levels at similar rates: 0% for TRPV1- and 15-19% for TRPV1+ (see also Figure S1 legend)

      (iii) The axonopathy and cell death resulting from 10 µM Csn is quite dramatic.

      However, here the authors do not appear to have included a TRPV1 negative control (although oddly they did for 1 µM treatment) so it is currently unclear whether or not a high conc of Csn alone might be cytotoxic.

      Chen et al (https://doi.org/10.1038/nmeth.3691) have established the TRPV1/capsaicin method in zebrafish with broad neuronal label and did not see any effect with high doses of capsaicin in TRPV1 negative fish.  

      * Behavioural assessment - stats

      Overall, the disruption of postural stability after PC manipulations is convincing.

      However, I have a few queries about the statistics:

      (i) In this section, the statistical unit was not clear. The tables, which are otherwise very useful, give no indication of N. The legend text does report "8 repeats/149 control fish" and "across experimental repeats" suggesting the statistical unit might be the repeats rather than animals, but this should be clarified. In Figure 2G, individual data points should be plotted if N=8, or a representation of the distribution (eg violin or box and whisker plots) if N = 149.

      We apologize for the confusion. Given the variable numbers of bouts, a single experimental repeat does not allow for an accurate estimate of expected value. Below we simulated how accurately the median can be estimated based on increasing sample sizes (Author response image 1). Given that large numbers of bouts are necessary to accurately estimate the median we pool the data for all experiments and use resampling statistics to estimate bias in our estimate.

      Author response image 1.

      Median estimation based on increasing sample size

      (ii) Related to the above, I hope it might be easier to interpret the unexpected change in climb posture in ablation controls once the data for individual repeats is shown.

      When we analyze the data as single repeats we see considerable variability between different repeats due to undersampling. We tested the medians for the single repeats for outliers to ensure that the shift is not due to a single repeat skewing the distribution. We did not detect any outliers in the pre-lesion control or in the post-lesion control group. (Outliers were determined as deviating more than 3 times the scaled median absolute deviation (MAD) from the median. A scaling factor of 1.4826 was used to ensure that MAD-based outlier detection is consistent with other methods like Z-scores.) We added this information to line 133-134 and the method section under Statistics. 

      (iii) In some parts of this section, including the Tables, the authors report the 95% CI of the median, rather than IQR. In this case, they should report the z-value used for 95% CI estimation.

      As we are using resampling to estimate the 95% confidence interval of the median there is no z-value as in a traditional normal distribution based confidence interval; Instead, we explicitly define the 2.5th and 97.5th percentiles from the bootstrapped sample distribution, which captures the middle 95% of the data, representing the 95% confidence interval.

      * It is stated that "fish adopted more nose-up postures before *and throughout* climb bouts". Figure 2F seems to show posture before the climb, but where is the "throughout" data? It would be useful if Figure 2E, J could be extended to make a bit clearer these two phases of postural assessment.

      We removed the phrase ‘throughout climb bouts’ as we are not showing the posture throughout the bout and to avoid over complicating the interpretation.  

      * Why were PCs not activated at 14 dpf (eg using 1 µM Csn)?

      Due to shifts in priorities the first author will not be continuing this series of experiments, and so this additional experiment will have to wait for someone to pick up this line of inquiry

      * The authors appear to claim that the difference in phenotype in 7 versus 14 dpf animals following high conc Csn treatment is indicative of a changing role for cerebellar PCs over this developmental period. For instance, in reference to the 14 dpf ablation phenotype, the authors write "reveals the functional emergence of Purkinje cell control of dives" and in the abstract they talk about "emerging control of posture across early development". However, can they rule out that the phenotypic differences might instead reflect differential sensitivity of the relevant PC (sub)populations to CSn at the two ages? If this caveat cannot be discounted then I suggest it is acknowledged e.g. in the discussion.

      As previously established, all Purkinje cells are labeled in the aldoca line (10.1523/ JNEUROSCI.3352-10.2010). Fluorescence is brighter at 14dpf compared to 7dpf, suggesting higher levels of TRPV1. We therefore assume that at 14 dpf, the high concentration of Csn is sufficient to ablate Purkinje cells. At 14 dpf, cerebellar damage is visible under a standard dissecting microscope.The preponderance of evidence therefore speaks against a previously undiscovered subpopulation of TRPV1expressing Purkinje cells that are, by mechanisms yet unknown, resistant to high doses of capsaicin. 

      * Fin-body "coordination"

      The ideas and data around fin-body coordination are very intriguing.

      (i) The statement "fin engagement is speed-dependent" would benefit from a stats test to show this is indeed significant. The data in Figure 4B suggest a rather high degree of variance.

      This is an important point; we appreciate the Reviewer’s attention. We have added statistics to show this is speed dependent to line 167-169 and show the corresponding plot in the supplement in Figure S4.  "Here, we observed that fin engagement is speeddependent, with faster bouts producing greater lift for a given axial rotation (Spearman correlation coefficient: control 0.2193; 10uM capsaicin: 0.0397; Z-test after ztransformation: p < 0.001)  

      (ii) The statement "After capsaicin exposure, the slopes of the medium fast speed bins were significantly lower (Figure 4C), reflecting *a loss of speed-dependent modulation*" is not convincing. The slope is likely a function of both speed and Csn treatment, and the comparisons in Figure 4C appear to be testing the latter, not the former.

      We understand the reviewer’s point. However, the slope for the slow bouts remains unchanged. We therefore conclude that the reduction in fin-body slope is speed dependent and not a speed independent reduction of slope overall. 

      We have made this more clear by adding Supplementary Figure S4 and changing the text in line 177-179. 

      (iii) I'd like to understand more about the phenotype of the fin-amputated animals. Were any "bout" parameters changed? Did the animals still attempt climbs and was the distribution of the upward rotation parameter similar to controls? The text states "the slope of the relationship between upward rotation and lift was indistinguishable from zero" but the stats reported in the text are comparisons between groups while Table 5 shows 95% CIs that don't span zero. Some clarification would be useful here.

      We appreciate the Reviewer’s interest. We’ve studied climbing in fin-amputated animals at length here: https://doi.org/10.7554/eLife.45839 and here: https://doi.org/10.1016/ j.celrep.2023.112573 and have added these references in line 183.

      (iv) The authors repeatedly refer to fin-body *coordination* but it is not clear whether the loss of lift after PC ablation is a result of an explicit coordination defect (i.e. changes in the relative timing and/or kinematics between fins and axial motion components), versus a simple reduction in pectoral fin engagement. Either result could be interesting, but this should be clarified.

      Thank you for pointing that out. In the fastest speed bin, we observed an increase in upward rotation and a decrease in average fin lift. In contrast, the medium speed bin showed no significant changes in average fin lift or upward rotation (see Author response image 2 and Tables 4 and 5), yet already displayed coordination deficits. Based on these observations, we argue that Purkinje cell lesions primarily affect coordination, rather than simply reducing one specific parameter such as lift or rotation (line 293-298).

      We have added fin lift and rotation values from Author response image 2 for all speed bins to tables 4 and 5.  

      Author response image 2.

      Fin lift and rotation for slow, medium and fast bouts

      * PC activity and decoding of pitch direction.

      The clever TIPM method is used to collect calcium data that convincingly shows that individual PCs can encode pitch-tilt direction. However, a population of "not tuned" cells are also identified, and here I found the analysis of their responses and the argument that they encode pitch direction at a population level difficult to follow.

      (i) First, although the naming of the cells implies that individual neurons do not encode pitch direction, I did not find this convincing. Figures 5F/G suggest that several "not tuned" cells in fact show quite consistent differences in activity across trial types and indeed in terms of their average responses sit as far from the unity line as do several "tuned" cells.

      The Reviewer’s comment helped us clarify some key points. First, tuned and untuned cells were categorized based on a Directionality Index threshold of 0.35; some cells might look similar in 5F/G but the highly variable responses of Purkinje cells have highly variable response so overall there was no consistent tuning. We have clarified this in the text in line 203-207 Below we have plotted the Up versus Down responses for the 10 least tuned cells (sorted by directionality index). While some cells have higher responses on average to one direction we think that the variability makes it difficult to support a claim for “tuning.” We have also tested the support vector machine on the least tuned cells to confirm that the chosen cutoff for tuned/untuned is not affecting our claim that untuned cells can encode position.(see also Author response image 4)

      Author response image 3.

      Trial-by-trial variability

      (ii) It is therefore not very surprising that PCA (and the SVM decoder) distinguishes trial type. I would guess that PCA assigns the largest weights to these most tuned of the "not tuned" cells, and the 3-5 cell decoders do well when these cells happen to be sampled.

      Author response image 4.

      Decoding accuracy of the 3/5/7 least tuned cells

      This was an interesting idea. To rule out that it is only the most tuned cells that contain the information, we tested the decoder on the 3/5/7 least tuned cells; here too, 5 and more cells are better able to accurately decode the direction. We have add the decoding accuracy to the text in line 221-224

      (iii) As I understand the analysis, Figure 5G shows responses for "not tuned" cells over 21 trials (of each type) but these are not the same trials for the different cells? How then is population coding being assessed?

      We have updated the text and refer to this data as a “pseudo-population” in lines 216 and 218 for all experiments where we combined cells from different fish. For technical reasons, when we perform TIPM at eccentric angles we must use sparsely labelled fish to ensure that we can find the same cells over a 60 degree range. We have repeated our analyses for TIPM centered at the horizon, where we can record from entire populations from a single fish.  

      (iv) Furthermore, Figure S2 shows a somewhat different analysis with decoding accuracy measured on a fish-by-fish basis. In this case, are these decoders for simultaneously imaged neurons? Is this a cross-validated measure of decoding accuracy?

      Yes, as above, Figure S4 (former S2) looks at fish-by-fish basis of simultaneous recorded neurons. Yes, it was 5-fold cross validated. We have updated the text in line 490-494.

      Reviewer #2 (Recommendations For The Authors):

      - Postural control involves various aspects such as balance, coordination, relative body part orientations, and stability. Discussing these and presenting in this context the specific subaspect characterized in this study would help clarify which aspect of postural control the work focuses on.

      The Reviewer makes an interesting point, but we think their description of what constitutes postural control is overly broad. Specifically, control of “relative body part orientations in space” by definition requires coordination, and subserves balance and stability. We acknowledge, of course, that different aspects can be and often are treated independently. While interesting, a full treatment of what comprises “postural control” is beyond the scope of the paper, as it would require reconciling the terms across taxa, effectors, environments and well over a century of experiments.

      We contend that posture — particularly underwater — is best defined as the relative orientation of body parts in space. For fish, those parts consist of predominantly axial muscles and secondarily fins. We present these definitions in the Introduction and thank the Reviewer for encouraging us to more clearly shape our findings.

      - Disruption of posture or postural control: The use of the word "disruption" could lead to misleading expectations. While it may not be incorrect, it suggests a significant loss of equilibrium, an obvious increase in postural variability, or at least a noticeable effect when observing an individual animal's behavior. However, the supporting data show only a subtle median shift in postural angle within a very broad distribution averaged over many individuals. This effect was only significant when comparing fish with a control group, not when comparing fish posture before and after the treatment.

      Replacing "disruption" with "modification" would be more cautious.

      We take the Reviewer’s point and have adjusted our wording to "modifies postural control.” In lines 137, 266, and 283

      - Statistical significance: Consider aligning the asterisk notation with conventional standards (e.g., * for p < 0.05, ** for p < 0.01, *** for p < 0.001) to enhance clarity for readers. On the other hand, the individual measurements might not be independent (e.g., measurements from the same fish, or the same tank are likely to be correlated), so using the Wilcoxon rank-sum test (Mann-Whitney U test) on pooled data might lead to incorrect conclusions. Methods that account for the hierarchical structure of the data might be required to support the conclusions.

      We take the Reviewer’s point about the importance of conventions, however we have never found “more stars = more significant” to be all that helpful in evaluating claims. Instead, we’ve opted to have both a significance and effect size criteria; a “star” here reflects our considered confidence in the difference we observe. 

      We agree that the hierarchical nature of pooled data is worth considering/presenting.

      We performed a two-way analysis of variance (ANOVA) on the interquartile ranges (IQRs) of the single experimental repeats for the 7 days post-fertilization (dpf) activation, 7dpf lesion, and 14dpf lesion experiments. The ANOVA revealed no significant main effects, supporting the strategy of pooling experimental repeats to estimate distributions.

      The results of the ANOVA, along with the IQRs for all experimental repeats, are presented in Tables 6-11. We have also clarified this in the methods section in lines 505-509.

      - Data representation: All data of postural angles should be represented in the form of violin plots to show the underlying distributions of the postural angles, especially given that the effect size is small relative to the dispersion of the distribution of the postural angle and that this distribution is also not Gaussian but bimodal, and different before and after the treatments.

      We take the Reviewer’s point that seeing the full distribution can be useful. We have added plots of the raw distributions for the data in Figure 3 as supplemental Figure S3.

      - Showing the distributions will provide the necessary information for the reader to evaluate the importance of the effect. For all data shown in Table 1, the distributions should be presented in the supplementary information.

      As requested, we have added the distributions of the data in Table 1 to the supplement (Figure S2)

      - Roll posture: A statement about whether roll posture is perturbed by Purkinje cell manipulation would be a piece of important additional information helping to understand how strong the 'disruption' of posture is.

      We haven’t assessed roll posture, as this is not practical in the current version of the SAMPL apparatus. We have added this limitation to the results (line 116) but also note that as our manipulations are bilateral, we don’t anticipate any systematic changes to roll.   

      - Comparison with other methods: Add a discussion on how the TRPV1/capsaicin method compares with other methods, such as using nitroreductase (Ntr) for targeted pharmaco-genetic ablation of cells by treatment with metronidazole or the the possibility to to ablate Purkinje cells by KillerRed as the author lab has done previously. Both methods have been applied to ablate Purkinje cells in larval zebrafish. What are the advantages of the TRPV1 method compared to these when neglecting the activation possibility?

      Thank you for that suggestion, we have added a section to the discussion where we compare the TRPV1/capsaicin lesion to other lesion methods (lines 334-336)

      - Describe the decoding algorithm: The decoding algorithm used could be described more in detail in the methods section.

      We have described the decoding algorithm in more detail in the methods under ‘Functional GCaMP imaging in Purkinje cells.’ Line 488+ 

      We used a support vector machine (SVM) with a linear kernel. The SVM model was trained using k-fold cross-validation, which splits the data into k subsets (folds). At each iteration, the model was trained on k-1 folds and tested on the remaining fold, ensuring that the model performance was evaluated on unseen data in each fold. Permutations were performed on randomized trial identity as a null hypothesis (5-fold cross-validation; 100 shuffles for randomization). Accuracy was calculated as 1 minus the classification loss.  

      - Availability of code: The link to the data and code repository is not working.

      Thank you for pointing that out, we have fixed it now. In the lower right of the page you can see the history of all changes to the repository, including the entry on 2023-09-08 where the corresponding author set it to “public.” When we checked thanks to your comment, it had been set to “private,” without any record of when/why. We have reset it 2024-10-17. We will continue to check it periodically in the future and apologize in advance if it is unavailable; this is the first time we’ve seen that happen.

      - Electrophysiological Control: Including an electrophysiological characterization of the activation of Purkinje cells by the TRPV1/capsaicin would significantly strengthen the validity of the method.

      We take the Reviewer’s point that electrophysiological characterization is a way to strengthen the validity of the method. However, Chen et al (h"ps://doi.org/10.1038/ nmeth.3691) have performed electrophysiology during neuronal activation and concluded that TRPV1 activation with capsaicin indeed increases neuronal activity and firing rates increased. Our calcium imaging and lesion experiments amply demonstrate that Purkinje cells are sensitive to TRPV1-mediated currents. We therefore do not believe that the additional information gained by arduous electrophysiological evaluation is merited here.

      - Describe more in detail how climb and dive bouts are defined. The height difference between consecutive bouts measured 250ms before the bout of executions.

      Climb and Dive bouts are split by the angle of their trajectory. If the fish moves up (i.e. trajectory larger 0) it is considered a climb bout and vice versa for dive bouts. 250ms prior to the maximum speed is roughly the time the fish initiate a bout, so the pre-bout posture is measured when at this point. The time-courses of bouts are dissected extensively in Zhu et. al. 2023. We have added a definition for climb and dive bouts to the method section under ‘Behavior analysis’ line 453 and 454.  

      - Figure 1H: Why can't you ablate all Purkinje cells but only about 80%?

      This is an excellent question. We opted for an extremely conservative count, and included everything that was still resembling a cell, even if it might not be functional/ already dying. Our counts are therefore likely an underestimate of the percentage of cells that were lost. We have added this point to the text in lines 393 395

      - Figure 2C: The method is not fully clear. At 8dpf 0.1uM capsaicin is added to the chamber. At what time after the application of capsaicin did the behavioral recording start?

      We recorded after about 10-15min after adding the 1uM Csn to the chambers. The fish were fed after the 6h in capsaicin. We have added this information to the method section line 404 - 408.

      - Figure 2F: What indicates the shown confidence interval? Also median with a 95% confidence interval calculated over the experiments in parallel?

      The distributions shown in Figure 2F take data from all experiments pooled. We use resampling methods to determine the variability in our estimates. The distribution plots are showing the median and the 25th and 75th percentile of the resampled distribution. We have added this information to the figure legends.

      - Figure 3: Subtitles on panel D and E indicating <climb bout posture> and would facilitate reading.

      We have added the subtitles to those panels.

      - Figure 4: Describe in the methods how recordings from individual fish were mapped onto each other to superimpose the Purkinje cell locations recorded from the 8 fish.

      We have added the respective section to the methods: Line 481 - 483

      “To map the anatomical locations of the recorded cells, we imaged overview stacks for each fish. These stacks were manually aligned in Illustrator, and the cells included in the analysis were reidentified and color-coded according to their tuning properties.”

      Reviewer #3 (Recommendations For The Authors):

      Major points:

      (1) Lines 74-81. The data presented here and in later experiments to argue for an effect of capsaicin on neural activity lacks statistical rigor because of the apparently very small numbers of animals/cells assessed. For example, the control appears to involve 4 cells assessed from 1 animal, and the experimental group is just 2 animals. Given that the interpretation of the paper depends upon this result, it is worthwhile to show the result more clearly, and with some statistical analysis. They argue in the discussion that "Our imaging assay established that 1 µM of capsaicin would stochastically activate subsets of Purkinje cells" which seems a stretch from the data as presented.

      We appreciate this point, which was shared by Reviewer 1. We have added more data and performed statistical analysis (line 63 - 67 as well as Figure S1A)

      (2) I found the practice of sorting effects by a mixture of effect size and p-value to be a little arbitrary, although in this case, it seems likely that it identified the most relevant effects. I would have preferred to see some attempt to correct for multiple comparisons (e.g. by resampling with the identities of fish shuffled to estimate the distribution of each measurement for this population size), followed by filtering for effect size after establishing a corrected threshold for significance.

      We take the Reviewer’s point, though we note that critical values for effect size and pvalue are inevitably “a little arbitrary.” We can’t do the exact analysis the Reviewer suggests as we do not measure data from individual fish for these experiments. However, we did calculate new critical p-values (added to the Tables) that account for multiple comparisons using Šidák’s method.

      (3) Figure 4. The data here is a little strange in that the slope in the control condition for medium speed is given as much larger than for slow, but the data in the two cases appears largely overlapping for most of the range of behavior, only diverging for the most extreme rotations. It seems perhaps that the measurement of slope is strongly dependent on these most extreme values. The authors might want to consider the use of robust regression methods which might mitigate these effects.

      This is an interesting observation and we appreciate the Reviewer’s thoughtful suggestion. We now use a robust regression method (bisquare weighting of residuals).

      We have adjusted all values in lines 175 - 177  and added the regression method to the Methods section line 520.

      (4) Figure 5. The 'principal component analysis' description is extremely unclear. The text says that PCA 'showed near-complete segregation of trial types' but it is not explained how this was achieved with PCA or how this was quantified. Figure panels show the data plotted using different pairs of PCs showing visual evidence of segregation. In the methods, it is stated that "We performed principal component analysis" and that "cells were used for principal component analysis and subsequent support vector machine decoding analysis". What is meant exactly by 'performed PCA'? Was PCA used in a dimensionality reduction step? And if so, how many and which PCs were chosen and why? For visualization of the separation, the authors show arbitrary pairs of PCs. Could it be better to use a method more suited to that purpose such as linear discriminant analysis?

      PCA was used to define a subspace to qualitatively evaluate if different trials could be separated. Once it became clear that it could, we next trained a binary decoder on the complete dataset (i.e. no dimensionality reduction). We did not perform linear discriminant analysis as the unsupervised PCA already showed separation of trial types.  We have made this clearer in lines 212 - 214.

      (5) Why does the decoding analysis use only untuned cells? Isn't it equally, or more, interesting to know how well tilt can be encoded using all cells? It is unclear to me what we learn by selecting only untuned cells for this analysis (although I agree it is interesting that this does work).

      We focused exclusively on untuned cells because including even a single highly tuned cell for the population coding will lead to excellent results. By using untuned cells we test if there is some directionality information that is not visible just by looking at the up/ down responses of single cells. We have made this clear in lines 217 - 218

      Minor points and corrections:

      (1) Maybe consider losing the words 'powerful' (I think it is overused and not well defined) and 'reagent'. Reagent is normally used for something that participates in a reaction. It is a bit odd to use it to refer to a transgenic animal. Later it is called a 'tool' which seems better.

      We have changed the wording and refer to it as tool for the whole paper.  

      (2) Figure 1D. Please use a color bar to indicate the scale.

      We have added a color scale to the panel

      (3) Saying that 'posture' increases is confusing, although the meaning can be inferred from the overall context and the definitions in the Methods - could Posture be capitalized to indicate a specific definition is being used rather than the general meaning?

      This suggestion agrees with those made by Reviewer 2. We have changed the wording to “postural angle.” 

      (4) The arrowheads in Figure 2FHK are unnecessary and confusing (why are some horizontal and some vertical?).

      Thank you for that suggestion, we have removed the arrowheads.

      (5) Figure 3 The legend should indicate that the image is shown with an inverted lookup table.

      We have updated the legend

      (6) Figure 3 D and E Titles would be helpful, so it is not necessary to refer to the legend to understand the difference.

      We have added titles to the figure panels

      (7) The dwell time for the 2-photon experiments is given in the manuscript, but I think the authors meant microseconds?

      Thank you for pointing that out. We have corrected it to microseconds.

    1. Author response:

      The following is the authors’ response to the original reviews.

      We performed multiple new experiments and analyses in response to the reviewers concerns, and incorporated the results of these analyses in the main text, and in multiple substantially revised or new figures. Before embarking on a point-by-point reply to the reviewers’ concerns, we here briefly summarize our most important revisions.

      First, we addressed a concern shared by Reviewers #1-3 about a lack of information about our DNA sequences. To this end, we redesigned multiple figures (Figures 3, 4, 5, S8, S9, S10, S11, and S12) to include the DNA sequences of each tested promoter, the specific mutations that occurred in it, the resulting changes in position-weight-matrix (PWM) scores, and the spacing between promoter motifs. Second, Reviewers #1 and #2 raised concerns about a lack of validation of our computational predictions and the resulting incompleteness of the manuscript. To address this issue, we engineered 27 reporter constructs harboring specific mutations, and experimentally validated our computational predictions with them. Third, we expanded our analysis to study how a more complete repertoire of other sigma 70 promoter motifs such as the UP-element and the extended -10 / TGn motif affects gene expression driven by the promoters we study. Fourth, we addressed concerns by Reviewer #3 about the role of the Histone-like nucleoid-structuring protein (H-NS) in promoter emergence and evolution. We did this by performing both experiments and computational analyses, which are now shown in the newly added Figure 5. Fifth, to satisfy Reviewer #3’s concerns about missing details in the Discussion, we have rewritten this section, adding additional details and references. 

      We next describe these and many other changes in a point-by-point reply to each reviewer’s comments. In addition, we append a detailed list of changes to each section and figure to the end of this document.

      Reviewer #1 (Public Review):

      Summary:

      This study by Fuqua et al. studies the emergence of sigma70 promoters in bacterial genomes. While there have been several studies to explore how mutations lead to promoter activity, this is the first to explore this phenomenon in a wide variety of backgrounds, which notably contain a diverse assortment of local sigma70 motifs in variable configurations. By exploring how mutations affect promoter activity in such diverse backgrounds, they are able to identify a variety of anecdotal examples of gain/loss of promoter activity and propose several mechanisms for how these mutations interact within the local motif landscape. Ultimately, they show how different sequences have different probabilities of gaining/losing promoter activity and may do so through a variety of mechanisms.

      We thank Reviewer #1 for taking the time to read and provide critical feedback on our manuscript. Their summary is fundamentally correct.

      Major strengths and weaknesses of the methods and results:

      This study uses Sort-Seq to characterize promoter activity, which has been adopted by multiple groups and shown to be robust. Furthermore, they use a slightly altered protocol that allows measurements of bi-directional promoter activity. This combined with their pooling strategy allows them to characterize expressions of many different backgrounds in both directions in extremely high throughput which is impressive! A second key approach this study relies on is the identification of promoter motifs using position weight matrices (PWMs). While these methods are prone to false positives, the authors implement a systematic approach which is standard in the field. However, drawing these types of binary definitions (is this a motif? yes/no) should always come with the caveat that gene expression is a quantitative trait that we oversimplify when drawing boundaries.

      The point is well-taken. To clarify this and other issues, we have added a section on the limitations of our work to the Discussion. Within this section we include the following sentences (lines 675-680):

      “Additionally, future studies will be necessary to address the limitations of our own work. First, we use binary thresholding to determine i) the presence or absence of a motif, ii) whether a sequence has promoter activity or not, and iii) whether a part of a sequence is a hotspot or not. While chosen systematically, the thresholds we use for these decisions may cause us to miss subtle but important aspects of promoter evolution and emergence.”

      Their approach to randomly mutagenizing promoters allowed them to find many anecdotal examples of different types of evolutions that may occur to increase or decrease promoter activity. However, the lack of validation of these phenomena in more controlled backgrounds may require us to further scrutinize their results. That is, their explanations for why certain mutations lead or obviate promoter activity may be due to interactions with other elements in the 'messy' backgrounds, rather than what is proposed.

      Thank you for raising this important point. To address it, we have conducted extensive new validation experiments for the newest version of this manuscript. For the “anecdotal” examples you described, we created 27 reporter constructs harboring the precise mutation that leads to the loss or gain of gene expression, and validated its ability to drive gene expression. The results from these experiments are in Figures 3, 4, 5, and Supplemental Figures S8-S11, and are labeled with a ′ (prime) symbol.

      These experiments not only confirm the increases and decreases in fluorescence that our analysis had predicted. They also demonstrate, with the exception of two (out of 27) falsepositive discoveries, that background mutations do not confound our analysis. We mention these two exceptions (lines 364-367):

      “In two of these hotspots, our validation experiments revealed no substantial difference in gene expression as a result of the hotspot mutation (Fig S8F′ and Fig S8J′). In both of these false positives, new -10 boxes emerge in locations without an upstream -35 box.”

      An appraisal of whether the authors achieved their aims, and whether the results support their conclusions:

      The authors express a key finding that the specific landscape of promoter motifs in a sequence affects the likelihood that local mutations create or destroy regulatory elements. The authors have described many examples, including several that are non-obvious, and show convincingly that different sequence backgrounds have different probabilities for gaining or losing promoter activity. While this overarching conclusion is supported by the manuscript, the proposed mechanisms for explaining changes in promoter activity are not sufficiently validated to be taken for absolute truth. There is not sufficient description of the strength of emergent promoter motifs or their specific spacings from existing motifs within the sequence. Furthermore, they do not define a systematic process by which mutations are assigned to different categories (e.g. box shifting, tandem motifs, etc.) which may imply that the specific examples are assigned based on which is most convenient for the narrative.

      To summarize, Reviewer #1 criticizes the following three aspects of our work in this comment. 1) The mechanisms we proposed are not sufficiently validated. 2) The description of motifs, spacing, and PWM scores are not shown. 3) How mutations are classified into different categories (i.e. box-shifting, tandem motifs, etc.) is not systematically defined. 

      These are all valid criticisms. In response, we performed an extensive set of follow-up experiments and analyses, and redesigned the majority of the figures. Here is a more detailed response to each criticism:

      (1) Proposed mechanisms for explaining changes in promoter activity are not sufficiently validated. We engineered 27 reporter constructs harboring the specific mutations in the parents that we had predicted to change promoter activity. For each, we compared their fluorescence levels with their wild-type counterpart. The results from these experiments are in Figures 3 and 4, 5, and Supplemental Figures S8, S9, S10, S11, and S12, and are labeled with a ′ (prime) symbol.

      (2) No sufficient description of the strength of emergent promoter motifs or their specific spacings. We redesigned the figures to include the DNA sequences of the parent sequences, as well as the degenerate consensus sequences for each mutation. We additionally now highlight the specific motif sequences, their respective PWM scores, and by how much the score changes upon mutation. Finally, we annotated the spacing of motifs. These changes are in Figures 3, 4, 5, and Supplemental Figures S8, S9, S10, S11, and S12.

      We note that in many cases, high-scoring PWM hits for the same motif can overlap (i.e. two -10 motifs or two -35 motifs overlap). Additionally, the proximity of a -35 and -10 box does not guarantee that the two boxes are interacting. Together, these two facts can result in an ambiguity of the spacer size between two boxes. To avoid any reporting bias, we thus often report spacer sizes as a range (see Figure panels 4F, S8D, S8F-L, S9A, S9H, S10A, and S10E). The smallest spacer we annotate is in Figure 4F with 10 bp, and the largest is in Figure S8D with 26 bp. Any more “extreme” distances are not annotated and for the reader to decide if an interaction is present or not.

      (3) No systematic process by which mutations are assigned to different categories such as box shifting, tandem motifs, etc. We opted to reformulate these categories completely, because the phenotypic effects of a previously mentioned “tandem motif” was actually a byproduct of H-NS repression (see the newly added Figure S12). 

      We also agree that the categories were ambiguous. We now introduce two terms: homo-gain and hetero-gain of -10 and -35 boxes. The manuscript now clearly defines these terms, and the relevant passage now reads as follows (lines 430-435): 

      “We found that these mutations frequently create new boxes overlapping those we had identified as part of a promoter

      (Fig S9). This occurs when mutations create a -10 box overlapping a -10 box, a -35 box overlapping a -35 box, a -10 box overlapping a -35 box, or a -35 box overlapping a -10 box. We call the resulting event a “homo-gain” when the new box is of the same type as the one it overlaps, and otherwise a “hetero-gain”. In either case, the creation of the new box does not always destroy the original box.”

      Impact of the work on the field, and the utility of the methods and data to the community: From this study, we are more aware of different types of ways promoters can evolve and devolve, but do not have a better ability to predict when mutations will lead to these effects. Recent work in the field of bacterial gene regulation has raised interest in bidirectional promoter regions. While the authors do not discuss how mutations that raise expression in one direction may affect another, they have created an expansive dataset that may enable other groups to study this interesting phenomenon. Also, their variation of the Sort-Seq protocol will be a valuable example for other groups who may be interested in studying bidirectional expression. Lastly, this study may be of interest to groups studying eukaryotic regulation as it can inform how the evolution of transcription factor binding sites influences short-range interactions with local regulator elements. Any additional context to understand the significance of the work:

      The task of computationally predicting whether a sequence drives promoter activity is difficult. By learning what types of mutations create or destroy promoters from this study, we are better equipped for this task.

      We thank Reviewer #1 again for their time and their thoughtful comments.

      Reviewer #2 (Public Review):

      Summary:

      Fuqua et al investigated the relationship between prokaryotic box motifs and the activation of promoter activity using a mutagenesis sequencing approach. From generating thousands of mutant daughter sequences from both active and non-active promoter sequences they were able to produce a fantastic dataset to investigate potential mechanisms for promoter activation. From these large numbers of mutated sequences, they were able to generate mutual information with gene expression to identify key mutations relating to the activation of promoter island sequences.

      We thank Reviewer #2 for reading and providing a thorough review of our manuscript. 

      Strengths:

      The data generated from this paper is an important resource to address this question of promoter activation. Being able to link the activation of gene expression to mutational changes in previously nonactive promoter regions is exciting and allows the potential to investigate evolutionary processes relating to gene regulation in a statistically robust manner. Alongside this, the method of identifying key mutations using mutual information in this paper is well done and should be standard in future studies for identifying regions of interest.

      Thank you for your kind words.

      Weaknesses:

      While the generation of the data is superb the focus only on these mutational hotspots removes a lot of the information available to the authors to generate robust conclusions. For instance.

      (1) The linear regression in S5 used to demonstrate that the number of mutational hotspots correlates with the likelihood of a mutation causing promoter activation is driven by three extreme points.

      A fair criticism. In response, we have chosen to remove the analysis of this trend from the manuscript entirely. (Additionally, Pnew and mutual information calculations both relied on the fluorescence scores of daughter sequences, so the finding was circular in its logic.)

      (2) Many of the arguments also rely on the number of mutational hotspots being located near box motifs. The context-dependent likelihood of this occurring is not taken into account given that these sequences are inherently box motif rich. So, something like an enrichment test to identify how likely these hot spots are to form in or next to motifs.

      Another good point. To address it, we carried out a computational analysis where we randomly scrambled the nucleotides of each parent sequence while maintaining the coordinates for each mutual information “hotspot.” This scrambling results in significantly less overlap with hotspots and boxes. This analysis is now depicted in Figure 2C and described in lines 272-296.

      (3) The link between changes in expression and mutations in surrounding motifs is assessed with two-sided Mann Whitney U tests. This method assumes that the sequence motifs are independent of one another, but the hotspots of interest occur either in 0, 3, 4, or 5s in sequences. There is therefore no sequence where these hotspots can be independent and the correlation causation argument for motif change on expression is weakened.

      This is a fair criticism and a limitation of the MWU test. To better support our reasoning, we engineered 27 reporter constructs harboring the specific mutations in the parents that we had predicted to change promoter activity. For each, we compared their fluorescence levels with their wild-type counterpart. The results from these experiments are in Figures 3, 4, 5, and Supplemental Figures S8, S9, S10, S11, and S12 and are labeled with a ′ (prime) symbol.

      These experiments not only confirm the increases and decreases in fluorescence that our analysis had predicted. They also demonstrate, with the exception of two (out of 27) falsepositive discoveries, that background mutations do not confound our analysis. We mention these two exceptions (lines 364-367):

      “In two of these hotspots, our validation experiments revealed no substantial difference in gene expression as a result of the hotspot mutation (Fig S8F′ and Fig S8J′). In both of these false positives, new -10 boxes emerge in locations without an upstream -35 box.”

      (4) The distance between -10 and -35 was mentioned briefly but not taken into account in the analysis.

      We have now included these spacer distances where appropriate. These changes are in Figures 3, 4, 5, and Supplemental Figures S8, S9, S10, S11, and S12.

      We note that in many cases, high-scoring PWM hits for the same motif can overlap (i.e. two -10 motifs or two -35 motifs overlap). Additionally, the proximity of a -35 and -10 box does not guarantee that the two boxes are interacting. Together, these two facts can result in an ambiguity of the spacer size between two boxes. To avoid any reporting bias, we thus often report spacer sizes as a range (see Figure panels 4F, S8D, S8F-L, S9A, S9H, S10A, and S10E). The smallest spacer we annotate is in Figure 4F with 10 bp, and the largest is in Figure S8D with 26 bp. More “extreme” distances are not annotated, and for the reader to decide if an interaction is present or not.

      The authors propose mechanisms of promoter activation based on a few observations that are treated independently but occur concurrently. To address this using complementary approaches such as analysis focusing on identifying important motifs, using something like a glm lasso regression to identify significant motifs, and then combining with mutational hotspot information would be more robust.

      This is a great idea, and we pursued it as part of the revision. For each parent sequence, we mapped the locations of all -10 and -35 box motifs in the daughters, then reduced each sequence to a binary representation, either encoding or not encoding these motifs, also referred to as a “hot-encoded matrix.” We subsequently performed a Lasso regression between the hot-encoded matrices and the fluorescence scores of each daughter sequence. The regression then outputs “weights” to each of the motifs in the daughters. The larger a motif’s weight is, the more the motif influences promoter activity. The Author response image 1 describes our workflow.

      Author response image 1.

      We really wanted this analysis to work, but unfortunately, the computational model does not act robustly, even when testing multiple values for the hyperparameter lambda (λ), which accounts for differences in model biases vs variance.

      The regression assigns strong weights almost exclusively to -10 boxes, and assigns weak to even negative weights to -35 boxes. While initially exciting, these weights do not consistently align with the results from the 27 constructs with individual mutations that we tested experimentally. This ultimately suggests that the regression is overfitting the data.

      We do think a LASSO-regression approach can be applied to explore how individual motifs contribute to promoter activity. However, effectively implementing such a method would require a substantially more complex analysis. We respectfully believe that such an approach would distract from the current narrative, and would be more appropriate for a computational journal in a future study. 

      Because this analysis was inconclusive, we have not made it part of the revised manuscript. However, we hope that our 27 experimentally validated new constructs with individual mutations are sufficient to address the reviewer’s concerns regarding independent verification of our computational predictions.

      Other elements known to be involved in promoter activation including TGn or UP elements were not investigated or discussed.

      Thank you for highlighting this potentially important oversight. In response, we have performed two independent analyses to explore the role of TGn in promoter emergence in evolution. First, we computationally searched for -10 boxes with the bases TGn immediately upstream of them in the parent sequences, and found 18 of these “extended -10 boxes” in the parents (lines 143145):

      “On average, each parent sequence contains ~5.32 -10 boxes and ~7.04 -35 boxes (Fig S1). 18 of these -10 boxes also include the TGn motif upstream of the hexamer.”

      However, only 20% of these boxes were found in parents with promoter activity (lines 182-185):

      “We also note that 30% (15/50) of parents have the TGn motif upstream of a -10 box, but only 20% (3/15) of these parents have promoter activity (underlined with promoter activity: P4-RFP, P6-RFP, P8-RFP, P9-RFP, P10-RFP, P11GFP, P12-GFP, P17-GFP, P18-GFP, P18-RFP, P19-RFP, P22-RFP, P24-GFP, P25-GFP, P25-RFP). “

      Second, we computationally searched through all of the daughter sequences to identify new -10 boxes with TGn immediately upstream. We found 114 -10 boxes with the bases TGn upstream. However, only 5 new -10 boxes (2 with TGn) were associated with increasing fluorescence (lines 338-345):

      “On average, 39.5 and 39.4 new -10 and -35 boxes emerged at unique positions within the daughter sequences of each mutagenized parent (Fig 3A,B), with 1’562 and 1’576 new locations for -10 boxes and -35 boxes, respectively. ~22% (684/3’138) of these new boxes are spaced 15-20 bp away from their cognate box, and ~7.3% (114/1’562) of the new -10 boxes have the TGn motif upstream of them. However, only a mere five of the new -10 boxes and four of the new 35 boxes are significantly associated with increasing fluorescence by more than +0.5 a.u. (Fig 3C,D).”

      In addition, we now study the role of UP elements. This analysis showed that the UP element plays a negligible role in promoter emergence within our dataset.  It is discussed in a new subsection of the results (lines 591-608).

      Collectively, these additional analyses suggest that the presence of TGn plus a -10 box is insufficient to create promoter activity, and that the UP element does not play a significant role in promoter emergence or evolution.

      Reviewer #3 (Public Review):

      Summary:

      Like many papers in the last 5-10 years, this work brings a computational approach to the study of promoters and transcription, but unfortunately disregards or misrepresents much of the existing literature and makes unwarranted claims of novelty. My main concerns with the current paper are outlined below although the problems are deeply embedded.

      We thank Reviewer #3 for taking the time to review this manuscript. We have made extensive changes to address their concerns about our work.

      Strengths:

      The data could be useful if interpreted properly, taking into account i) the role of translation ii) other promoter elements, and iii) the relevant literature.

      Weaknesses:

      (1) Incorrect assumptions and oversimplification of promoters.

      - There is a critical error on line 68 and Figure 1A. It is well established that the -35 element consensus is TTGACA but the authors state TTGAAA, which is also the sequence represented by the sequence logo shown and so presumably the PWM used. It is essential that the authors use the correct -35 motif/PWM/consensus. Likely, the authors have made this mistake because they have looked at DNA sequence logos generated from promoter alignments anchored by either the position of the -10 element or transcription start site (TSS), most likely the latter. The distance between the TSS and -10 varies. Fewer than half of E. coli promoters have the optimal 7 bp separation with distances of 8, 6, and 5 bp not being uncommon (PMID: 35241653). Furthermore, the distance between the -10 and -35 elements is also variable (16,17, and 18 bp spacings are all frequently found, PMID: 6310517). This means that alignments, used to generate sequence logos, have misaligned -35 hexamers. Consequently, the true consensus is not represented. If the alignment discrepancies are corrected, the true consensus emerges. This problem seems to permeate the whole study since this obviously incorrect consensus/motif has been used throughout to identify sequences that resemble -35 hexamers.

      We respectfully but strongly disagree that our analysis has misrepresented the true nature of -35 boxes. First, accounting for more A’s at position 5 in the PWM is not going to lead to a “critical error.” This is because positions 4-6 of the motif barely have any information content (bits) compared to positions 1-3 (see Fig 1A). This assertion is not just based on our own PWM, but based on ample precedent in the literature. In PMID 14529615, TTG is present in 38% of all -35 boxes, but ACA only in 8%. In PMID 29388765, with the -10 instance TATAAT, the -35 instance TTGCAA yields stronger promoters compared to the -35 instance TTGACA (See their Figure 3B).

      In PMID 29745856 (Figure 2), the most information content lies in positions 1-3, with the A and C at position 5 both nearly equally represented, as in our PWM. In PMID 33958766 (Figure 1) an experimentally-derived -35 box is even reduced to a “partial” -35 box which only includes positions 1 and 2, with consensus: TTnnnn.

      In addition, we did not derive the PWMs as the reviewer describes. The PWMs we use are based on computational predictions that are in excellent agreement with experimental results. Specifically, the PWMs we use are from PMID 29728462, which acquired 145 -10 and -35 box sequences from the top 3.3% of computationally predicted boxes from Regulon DB. See PMID 14529615 for the computational pipeline that was used to derive the PWMs, which independently aligns the -10 and -35 boxes to create the consensus sequences. The -35 PWMs significantly and strongly correlates with an experimentally derived -35 box (see Supporting Information from Figure S4 of Belliveau et al., PNAS 2017. Pearson correlation coefficient = 0.89). Within the 145 -35 boxes, the exact consensus sequence (TTGACA) that Reviewer #3 is concerned about is present 6 times in our matrix, and has a PWM score above the significance threshold. In other words, TTGACA, is classified to be a -35 box in our dataset.

      We now provide DNA sequences for each of the figures to improve accessibility and reproducibility. A reader can now use any PWM or method they wish to interpret the data.

      - An uninformed person reading this paper would be led to believe that prokaryotic promoters have only two sequence elements: the -10 and -35 hexamers. This is because the authors completely ignore the role of the TG motif, UP element, and spacer region sequence. All of these can compensate for the lack of a strong -35 hexamer and it's known that appending such elements to a lone -10 sequence can create an active promoter (e.g. PMIDs 15118087, 21398630, 12907708, 16626282, 32297955). Very likely, some of the mutations, classified as not corresponding to a -10 or -35 element in Figure 2, target some of these other promoter motifs.

      Thank you for bringing this oversight to our attention. We have performed two independent analyses to explore the role of TGn in promoter emergence in evolution. First, we computationally searched for -10 boxes with the bases TGn immediately upstream of them in the parent sequences, and found 18 of these “extended -10 boxes” in the parents (lines 143145):

      “On average, each parent sequence contains ~5.32 -10 boxes and ~7.04 -35 boxes (Fig S1). 18 of these -10 boxes also include the TGn motif upstream of the hexamer.”

      However, only 20% of these boxes were found in parents with promoter activity (lines 182-185):

      “We also note that 30% (15/50) of parents have the TGn motif upstream of a -10 box, but only 20% (3/15) of these parents have promoter activity (underlined with promoter activity: P4-RFP, P6-RFP, P8-RFP, P9-RFP, P10-RFP, P11GFP, P12-GFP, P17-GFP, P18-GFP, P18-RFP, P19-RFP, P22-RFP, P24-GFP, P25-GFP, P25-RFP).”

      Second, we computationally searched through all of the daughter sequences to identify new -10 boxes with TGn immediately upstream. We found 114 -10 boxes with the bases TGn upstream. However, only 5 new -10 boxes (2 with TGn) were associated with increasing fluorescence (lines 338-345):

      “On average, 39.5 and 39.4 new -10 and -35 boxes emerged at unique positions within the daughter sequences of each mutagenized parent (Fig 3A,B), with 1’562 and 1’576 new locations for -10 boxes and -35 boxes, respectively. ~22% (684/3’138) of these new boxes are spaced 15-20 bp away from their cognate box, and ~7.3% (114/1’562) of the new -10 boxes have the TGn motif upstream of them. However, only a mere five of the new -10 boxes and four of the new 35 boxes are significantly associated with increasing fluorescence by more than +0.5 a.u. (Fig 3C,D).”

      In addition, we now study the role of UP elements. This analysis showed that the UP element plays a negligible role in promoter emergence within our dataset.  It is discussed in a new subsection of the results (lines 591-608) and in the newly added Figure S13.

      Collectively, these additional analyses suggest that the presence of TGn plus a -10 box is insufficient to create promoter activity, and that the UP element does not play a significant role in promoter emergence or evolution.

      - The model in Figure 4C is highly unlikely. There is no evidence in the literature that RNAP can hang on with one "arm" in this way. In particular, structural work has shown that sequencespecific interactions with the -10 element can only occur after the DNA has been unwound (PMID: 22136875). Further, -10 elements alone, even if a perfect match to the consensus, are non-functional for transcription. This is because RNAP needs to be directed to the -10 by other promoter elements, or transcription factors. Only once correctly positioned, can RNAP stabilise DNA opening and make sequence-specific contacts with the -10 hexamer. This makes the notion that RNAP may interact with the -10 alone, using only domain 2 of sigma, extremely unlikely.

      This is a valid criticism, and we thank the reviewer for catching this problem. In response, we have removed the model and pertinent figures throughout the entire manuscript.

      (2) Reinventing the language used to describe promoters and binding sites for regulators.

      - The authors needlessly complicate the narrative by using non-standard language. For example, On page 1 they define a motif as "a DNA sequence computationally predicted to be compatible with TF binding". They distinguish this from a binding site "because binding sites refer to a location where a TF binds the genome, rather than a DNA sequence". First, these definitions are needlessly complicated, why not just say "putative binding sites" and "known binding sites" respectively? Second, there is an obvious problem with the definitions; many "motifs" with also be "bindings sites". In fact, by the time the authors state their definitions, they have already fallen foul of this conflation; in the prior paragraph they stated: "controlled by DNA sequences that encode motifs for TFs to bind". The same issue reappears throughout the paper.

      We agree that this was needlessly complicated. We now just refer to every sequence we study as a motif. A -10 box is a motif, a -35 box is a motif, a putative H-NS binding site is an H-NS motif, etc. The word “binding site” no longer occurs in the manuscript.

      - The authors also use the terms "regulatory" and non-regulatory" DNA. These terms are not defined by the authors and make little sense. For instance, I assume the authors would describe promoter islands lacking transcriptional activity (itself an incorrect assumption, see below)as non-regulatory. However, as horizontally acquired sections of AT-rich DNA these will all be bound by H-NS and subject to gene silencing, both promoters for mRNA synthesis and spurious promoters inside genes that create untranslated RNAs. Hence, regulation is occurring.

      Another fair point. We have thus changed the terminology throughout to “promoter” and “nonpromoter.”

      - Line 63: "In prokaryotes, the primary regulatory sequences are called promoters". Promoters are not generally considered regulatory. Rather, it is adjacent or overlapping sites for TFs that are regulatory. There is a good discussion of the topic here (PMID: 32665585). 

      We have rewritten this. The sentence now reads (lines 67-69):

      “A canonical prokaryotic promoter recruits the RNA polymerase subunit σ70 to transcribe downstream sequences (Burgess et al., 1969; Huerta and Collado-Vides, 2003; Paget and Helmann, 2003; van Hijum et al., 2009).”

      (3) The authors ignore the role of translation.

      - The authors' assay does not measure promoter activity alone, this can only be tested by measuring the amount of RNA produced. Rather, the assay used measures the combined outputs of transcription and translation. If the DNA fragments they have cloned contain promoters with no appropriately positioned Shine-Dalgarno sequence then the authors will not detect GFP or RFP production, even though the promoter could be making an RNA (likely to be prematurely terminated by Rho, due to a lack of translation). This is known for promoters in promoter islands (e.g. Figure 1 in PMID: 33958766).

      We agree that this is definitely a limitation of our study, which we had not discussed sufficiently. In response, we now discuss this limitation in a new section of the discussion (lines 680-686):

      “Second, we measure protein expression through fluorescence as a readout for promoter activity. This readout combines transcription and translation. This means that we cannot differentiate between transcriptional and post-transcriptional regulation, including phenomena such as premature RNA termination (Song et al., 2022; Uptain and Chamberlin, 1997), post-transcriptional modifications (Mohanty and Kushner, 2006), and RNA-folding from riboswitch-like sequences (Mandal and Breaker, 2004).”

      - In Figure S6 it appears that the is a strong bias for mutations resulting in RFP expression to be close to the 3' end of the fragment. Very likely, this occurs because this places the promoter closer to RFP and there are fewer opportunities for premature termination by Rho.

      The reviewer raises a very interesting possibility. To validate it, we have performed the following analysis. We took the RFP expression values from the 9’934 daughters with single mutations in all 25 parent sequences (P1-RFP, P2-RFP, … P25-RFP), and plotted the location of the single mutation (horizontal axis) against RFP expression (vertical axis) in Author response image 2. 

      Author response image 2.

      The distribution is uniform across the sequences, showing that distance from the RBS is not likely the reason for this observation. Since this analysis was uninformative with respect to distance from the RBS, we chose not to include it in the manuscript.

      (4) Ignoring or misrepresenting the literature.

      - As eluded to above, promoter islands are large sections of horizontally acquired, high ATcontent, DNA. It is well known that such sequences are i) packed with promoters driving the expression on RNAs that aren't translated ii) silenced, albeit incompletely, by H-NS and iii) targeted by Rho which terminates untranslated RNA synthesis (PMIDs: 24449106, 28067866, 18487194). None of this is taken into account anywhere in the paper and it is highly likely that most, if not all, of the DNA sequences the authors have used contain promoters generating untranslated RNAs.

      Thank you for pointing out that our original submission was incomplete in this regard. We address these concerns by new analyses, including some new experiments. First, Rhodependent termination is associated with the RUT motif, which is very rich in Cytosines (PMID: 30845912). Given that our sequences confer between 65%-78% of AT-content, canonical rhodependent termination is unlikely. However, we computationally searched for rho-dependent terminators using the available code from PMID: 30845912, but the algorithm did not identify any putative RUTs. Because this analysis was not informative, we did not include it in the paper.

      We analyzed the role of H-NS on promoter emergence and evolution within our dataset using both experimental and computational approaches. These additional analyses are now shown in the newly-added Figure 5 and the newly-added Figure S12. We found that H-NS represses P22-GFP and P12-RFP and affects the bidirectionality of P20. More specifically, to analyze the effects of H-NS, we first compared the fluorescence levels of parent sequences in a Δhns background vs the wild-type (dh5α) background in Figure 5A. We found 6 candidate H-NS targets, with P22-GFP and P12-RFP exhibiting the largest changes in fluorescence (lines 496506):

      “We plot the fluorescence changes in Fig 5A as distributions for the 50 parents, where positive and negative values correspond to an increase or decrease in fluorescence in the Δhns background, respectively. Based on the null hypothesis that the parents are not regulated by H-NS, we classified outliers in these distributions (1.5 × the interquartile range) as H-NS-target candidates. We refer to these outliers as “candidates” because the fluorescence changes could also result from indirect trans-effects from the knockout (Mattioli et al., 2020; Metzger et al., 2016). This approach identified 6 candidates for H-NS targets (P2-GFP, P19-GFP, P20-GFP, P22-GFP, P12-RFP, and P20-RFP). For GFP, the largest change occurs in P22-GFP, increasing fluorescence ~1.6-fold in the mutant background (two-tailed t-test, p=1.16×10-8) (Fig 5B). For RFP, the largest change occurs in P12-RFP, increasing fluorescence ~0.5-fold in the mutant background (two-tailed t-test, p=4.33×10-10) (Fig 5B).” 

      We also observed that the Δhns background affected the bidirectionality of P20 (lines 507-511):

      “We note that for template P20, which is a bidirectional promoter, GFP expression increases ~2.6-fold in the Δhns background (two-tailed t-test, p=1.59×10-6). Simultaneously, RFP expression decreases ~0.42-fold in the Δhns background (two-tailed t-test, p=4.77×10-4) (Fig S12A). These findings suggest that H-NS also modulates the directionality of P20’s bidirectional promoter through either cis- or trans-effects.”

      We then searched for regions where losing H-NS motifs in hotspots significantly changed fluorescence. We identified 3 motifs in P12-RFP and P22-GFP (lines 522-528):

      “For P22-GFP, a H-NS motif lies 77 bp upstream of the mapped promoter. Mutations which destroy this motif significantly increase fluorescence by +0.52 a.u. (two-tailed MWU test, q=1.07×10-3) (Fig 5E). For P12-RFP, one H-NS motif lies upstream of the mapped promoter’s -35 box, and the other upstream of the mapped promoter’s -10 box. Mutations that destroy these H-NS motifs significantly increase fluorescence by +0.53 and +0.51 a.u., respectively (two-tailed MWU test, q=3.28×10-40 and q=4.42 ×10-50) (Fig 5F,G). Based on these findings, we conclude that these motifs are bound by H-NS.”

      We are grateful for the suggestion to look at the role of H-NS in our dataset. Our analysis revealed a more plausible explanation to what we formerly referred to as a “Tandem Motif” in the original submission. Previously, we had shown that in P12-RFP, when a -35 box is created next to the promoter’s -35 box, or a -10 box next to the promoter’s -10 box, that expression decreases. These new -10 and -35 boxes, however, also overlap with the two H-NS motifs in P12-RFP. We tested these exact point mutations in reporter plasmids and in the Δhns background, and found that the Δhns background rescues this loss in expression (see Figure S12). This analysis is in the newly added subsection: “The binding of H-NS changes when new 10 and -35 boxes are gained” and can be found at lines 529-563. We summarize the findings in a final paragraph of the section (lines 556-563):

      “To summarize, we present evidence that H-NS represses both P22-GFP and P12-RFP in cis. H-NS also modulates the bidirectionality of P20-GFP/RFP in cis or trans. In P22-GFP, the strongest H-NS motif lies upstream of the promoter. In P12-RFP, the strongest H-NS motifs lie  upstream of the -10 and -35 boxes of the promoter. We note that there are 16 additional H-NS motifs surrounding the promoter in P12-RFP that may also regulate P12-RFP (Fig S12G). Mutations in two of these two H-NS motifs can create additional -10 and -35 boxes that appear to lower expression. However, the effects of these mutations are insignificant in the absence of H-NS, suggesting that these mutations actually modulate H-NS binding.”

      We also agree that the majority of these sequences are likely driving the expression of many untranslated RNAs (see Purtov et al., 2014). We thus now define a promoter more carefully as follows (lines 113-119):

      “In this study, we define a promoter as a DNA sequence that drives the expression of a (fluorescent) protein whose expression level, measured by its fluorescence, is greater than a defined threshold. We use a threshold of 1.5 arbitrary units (a.u.) of fluorescence. This definition does not distinguish between transcription and translation. We chose it because protein expression is usually more important than RNA expression whenever natural selection acts on gene expression, because it is the primary phenotype visible to natural selection (Jiang et al., 2023).” 

      We also state this as a limitation of our study in the Discussion (lines 680-686):

      “Second, we measure protein expression through fluorescence as a readout for promoter activity. This readout combines transcription and translation. This means that we cannot differentiate between transcriptional and post-transcriptional regulation, including phenomena such as premature RNA termination (Song et al., 2022; Uptain and Chamberlin, 1997), post-transcriptional modifications (Mohanty and Kushner, 2006), and RNA-folding from riboswitch-like sequences (Mandal and Breaker, 2004).”

      - The authors state that GC content does not correlate with the emergence of new promoters. It is known that GC content does correlate to the emergence of new promoters because promoters are themselves AT-rich DNA sequences (e.g. see Figure 1 of PMID: 32297955). There are two reasons the authors see no correlation in this work. First, the DNA sequences they have used are already very AT-rich (between 65 % and 78 % AT-content). Second, they have only examined a small range of different AT-content DNA (i.e. between 65 % and 78 %). The effect of AT-content on promoter emerge is most clearly seen between AT-content of between around 40 % and 60 %. Above that level, the strong positive correlation plateaus.

      We respectfully disagree that the reviewer’s point is pertinent because what the reviewer is referring to is the likelihood that the sequence is a promoter, which indeed increases with AT content, but we are focused on the likelihood that a sequence becomes a promoter through DNA mutation. We note that if a DNA sequence is more AT-rich, then it is more likely to have -10 and -35 boxes, because their consensus sequences are also AT-rich. However, H-NS and other transcriptional repressors also bind to AT-rich sequences. This could also explain the saturation observed above 60% AT-content in PMID 32297955. Perhaps we can address this trend in future works.

      - Once these authors better include and connect their results to the previous literature, they can also add some discussion of how previous papers in recent years may have also missed some of this important context.

      We apologize for this oversight. We have rewritten the Discussion section to include the following points below. Many of the newly added references come from the group of David Grainger, who works on H-NS repression, bidirectional promoters, promoter emergence, promoter motifs, and spurious transcription in E. coli. More specifically:

      (1) The role of pervasive transcription and the likelihood of promoter emergence (lines 614-621):

      “Instead, we present evidence that promoter emergence is best predicted by the level of background transcription each non-promoter parent produces, a phenomenon also referred to as “pervasive transcription” (Kapranov et al., 2007).

      From an evolutionary perspective, this would suggest that sequences that produce such pervasive transcripts – including the promoter islands (Panyukov and Ozoline, 2013) and the antisense strand of existing promoters (Dornenburg et al., 2010; Warman et al., 2021), may have a proclivity for evolving de-novo promoters compared to other sequences (Kapranov et al., 2007; Wade and Grainger, 2014).”

      (2) How our results contradict the findings from Bykov et al., 2020 (lines 622-640):

      “A previous study randomly mutagenized the appY promoter island upstream of a GFP reporter, and isolated variants with increased and decreased GFP expression. The authors found that variants with higher GFP expression acquired mutations that 1) improve a -10 box to better match its consensus, and simultaneously 2) destroy other -10 and -35 boxes (Bykov et al., 2020). The authors concluded that additional -10 and -35 boxes repress expression driven by promoter islands. Our data challenge this conclusion in several ways. 

      First, we find that only ~13% of -10 and -35 boxes in promoter islands actually contribute to promoter activity. Extrapolating this percentage to the appY promoter island, ~87% (100% - 13%) of the motifs would not be contributing to its activity. Assuming the appY promoter island is not an outlier, this would insinuate that during random mutagenesis, these inert motifs might have accumulated mutations that do not change fluorescence. Indeed, Bykov et al. (Bykov et al., 2020) also found that a similar frequency of -10 and -35 boxes were destroyed in variants selected for lower GFP expression, which supports this argument. Second, we find no evidence that creating a -10 or -35 box lowers promoter activity in any of our 50 parent sequences. Third, we also find no evidence that destruction of a -10 or -35 box increases promoter activity without plausible alternative explanations, i.e. overlap of the destroyed box with a H-NS site, destruction of the promoter, or simultaneous creation of another motif as a result of the destruction. In sum, -10 and 35 boxes are not likely to repress promoter activity.”

      (3) How other sequence features besides the -10 and -35 boxes may influence promoter emergence and activity (lines 661-671):

      “These findings suggest that we are still underestimating the complexity of promoters. For instance, the -10 and -35 boxes, extended -10, and the UP-element may be one of many components underlying promoter architecture. Other components may include flanking sequences (Mitchell et al., 2003), which have been observed to play an important role in eukaryotic transcriptional regulation (Afek et al., 2014; Chiu et al., 2022; Farley et al., 2015; Gordân et al., 2013). Recent studies on E. coli promoters even characterize an AT-rich motif within the spacer sequence (Warman et al., 2020), and other studies use longer -10 and -35 box consensus sequences (Lagator et al., 2022). Another possibility is that there is much more transcriptional repression in the genome than anticipated (Singh et al., 2014). This would also coincide with the observed repression of H-NS in P22-GFP and P12-RFP, and accounts of H-NSrepression in the full promoter island sequences (Purtov et al., 2014).”

      (4) The limits of our experimental methodology (lines 675-686):

      “Additionally, future studies will be necessary to address the limitations of our own work. First, we use binary thresholding to determine i) the presence or absence of a motif, ii) whether a sequence has promoter activity or not, and iii) whether a part of a sequence is a hotspot or not. While chosen systematically, the thresholds we use for these decisions may cause us to miss subtle but important aspects of promoter evolution and emergence. Second, we measure protein expression through fluorescence as a readout for promoter activity. This readout combines transcription and translation. This means that we cannot differentiate between transcriptional and post-transcriptional regulation, including phenomena such as premature RNA termination (Song et al., 2022; Uptain and Chamberlin, 1997), posttranscriptional modifications (Mohanty and Kushner, 2006), and RNA-folding from riboswitch-like sequences (Mandal and Breaker, 2004) “

      (5) An updated take-home message (lines 687-694):

      “Overall, our study demonstrates that -10 and -35 boxes neither prevent existing promoters from driving expression, nor do they prevent new promoters from emerging by mutation. It shows how mutations can create new -10 and -35 boxes near or on top of preexisting ones to modulate expression. However, randomly creating a new -10 or -35 box will rarely create a new promoter, even if the new box is appropriately spaced upstream or downstream of a cognate box. Ultimately our study demonstrates that promoter models need to be further scrutinized, and that using mutagenesis to create de-novo promoters can provide new insights into promoter regulatory logic.”

      (5) Lack of information about sequences used and mutations.

      - To properly assess the work any reader will need access to the sequences cloned at the start of the work, where known TSSs are within these sequences (ideally +/- H-NS, which will silence transcription in the chromosomal context but may not when the sequences are removed from their natural context and placed in a plasmid). Without this information, it is impossible to assess the validity of the authors' work.

      Thank you for raising this point. Please see Data S1 for the 25 template sequences (P1-P25) used in this study, and Data S2 for all of the daughter sequences.

      For brevity, we have addressed the reviewer’s request to look at the role of H-NS in their comment (4) “Ignoring or misrepresenting the literature.”

      We do not have information about the predicted transcription start sites (TSS) for the parent sequences because the program which identified them (Platprom) is no longer available. Regardless, having TSS coordinates would not validate or invalidate our findings, since we already know that the promoter islands produce short transcripts throughout their sequences, and we are primarily interested in promoters which can produce complete transcripts.

      - The authors do not account for the possibility that DNA sequences in the plasmid, on either side of the cloned DNA fragment, could resemble promoter elements. If this is the case, then mutations in the cloned DNA will create promoters by "pairing up" with the plasmid sequences. There is insufficient information about the DNA sequences cloned, the mutations identified, or the plasmid, to determine if this is the case. It is possible that this also accounts for mutational hotspots described in the paper.

      We agree that these are important points. To address the criticism that we provided insufficient information, we now redesigned all our figures to provide this information. Specifically, the figures now include the DNA sequences, their PWM predictions, and the exact mutations that lead to promoter activity. The figures with these changes are Figures 3, 4, 5, and Supplemental Figures S8, S9, S10, S11, and S12. We now also provide more details about pMR1 in a new section of the methods (lines 740-748):

      “Plasmid MR1 (pMR1)

      The plasmid MR1 (pMR1) is a variant of the plasmid RV2 (pRV2) in which the kan resistance gene has been swapped with the cm resistance gene (Guazzaroni and Silva-Rocha, 2014). Plasmid pMR1 encodes the BBa_J34801 ribosomal binding site (RBS, AAAGAGGAGAAA) 6 bp upstream of the start codon for GFP(LVA). The plasmid also encodes a putative RBS (AAGGGAGG) (Cazemier et al., 1999) 5 bp upstream of the start codon for mCherry on the opposite strand.

      The plasmid additionally contains the low-to-medium copy number origin of replication p15A (Westmann et al., 2018).

      A map of the plasmid is available on the Github repository: https://github.com/tfuqua95/promoter_islands

      The reviewer also makes a valid point about promoter elements of the plasmid itself. We addressed it with the following new analyses. First we re-examined each of the examples where new -10 and -35 boxes are gained or lost, to see if any of these hotspots occur on the flanking ends of the parent sequences. We looked specifically at the ends because they could potentially interact with -10 and -35 box-like sequences on the plasmid to form a promoter. 

      Only one of these hotspots (out of 27) occurred at the end of the cloned sequences, and is thus a candidate for the phenomenon the reviewer hypothesized. This hotspot occurs in P9-GFP, where gaining a -10 box at the left flank increases expression (see Figure S8E-F’). There is indeed a -35 box 22-23 bp upstream of this -10 box on the plasmid, which could potentially affect promoter activity. 

      We tested the GFP expression of a construct harboring the point mutation which creates this -10 box on the left flank of P9-GFP. However, there was no significant difference in fluorescence between this construct and the wile-type P9-GFP (see Figure S8E-F’). Thus, this -35 box on pMR1 is not likely creating a new promoter.

      (6) Overselling the conclusions.

      Line 420: The paper claims to have generated important new insights into promoters. At the same time, the main conclusion is that "Our study demonstrates that mutations to -10 and -35 boxes motifs are the primary paths to create new promoters and to modulate the activity of existing promoters". This isn't new or unexpected. People have been doing experiments showing this for decades. Of course, mutations that make or destroy promoter elements create and destroy promoters. How could it be any other way?

      In hindsight, we agree that the original conclusion was not very novel. Our new conclusion is that -10 and -35 boxes do not repress transcription, and that our current promoter models, even with the additional motifs like the UP-element and the extended -10, are insufficient to understand promoters (lines 687-694):

      “Overall, our study demonstrates that -10 and -35 boxes neither prevent existing promoters from driving expression, nor do they prevent new promoters from emerging by mutation. It shows how mutations can create new -10 and -35 boxes near or on top of preexisting ones to modulate expression. However, randomly creating a new -10 or -35 box will rarely create a new promoter, even if the new box is appropriately spaced upstream or downstream of a cognate box. Ultimately our study demonstrates that promoter models need to be further scrutinized, and that using mutagenesis to create de-novo promoters can provide new insights into promoter regulatory logic.”

      Recommendations for the authors:

      Reviewer #1 (Recommendations For The Authors):

      I would like to start by thanking the authors for presenting an interesting and well-written article for review. This paper is a welcome addition to the field, addressing modern questions in the longstanding area of bacterial gene regulation. It is both enlightening and inspiring. While I do have suggestions, I hope these are not perceived as a lack of optimism for the work.

      Thank you for your kind words and suggestions, and for providing an astute and constructive review. We feel that manuscript has greatly improved with your suggested changes.

      ABSTRACT:

      Line 11: The sentence, "It is possible that these motifs influence..." Could be rewritten to be clearer as it is the most important point of the manuscript. It is not obvious that you're talking about how the local landscape of motifs affects the probability of promoters evolving/devolving in this location.

      We have changed the sentence to read, “Here, we ask whether the presence of such motifs in different genetic sequences influences promoter evolution and emergence.”

      INTRODUCTION:

      Line 68: Is the -35 consensus motif not TTGACA? Here it is listed as TTGAAA.

      Corrected from TTGAAA to TTGACA

      RESULTS:

      Line 92-94. In finding that the. The main takeaway from this work is that different sequences have different likelihoods of mutations creating promoters and so I believe this claim could be explored deeper with more quantitative information. Could the authors supplement this claim by including? Could you look at whether there is a correlation between the baseline expression of a parent sequence and Pnew? I expect even the inactive sequences to have some variability in measured expression.

      Thank you for this great idea. We followed up on it by plotting the baseline parent sequence fluorescence scores against Pnew. You are indeed correct, i.e., Pnew increases with baseline expression following a sigmoid function, and is now shown in Figure 1D. To report our new observations, we have added the following section to the Results (lines 219-232):

      “Although mutating each of the 40 non-promoter parent sequences could create promoter activity, the likelihood Pnew that a mutant has promoter activity, varies dramatically among parents. For each non-promoter parent, Fig 1D shows the percentage of active daughter sequences. The median Pnew is 0.046 (std. ± 0.078), meaning that ~4.6% of all mutants have promoter activity. The lowest Pnew is 0.002 (P25-GFP) and the highest 0.41 (P8-RFP), a 205-fold difference.

      We hypothesized that these large differences in Pnew could be explained by minute differences in the fluorescence scores of each parent, particularly if its score was below 1.5 a.u. Plotting the fluorescence scores of each parent (N=50) and their respective Pnew values as a scatterplot (Fig 1E), we can fit these values to a sigmoid curve (see methods). This finding helps to explain why P8-RFP has a high Pnew (0.41) and P25-GFP a low Pnew (0.002), as their fluorescence scores are 1.380 and 1.009 a.u., respectively. The fact that the inflection point of the fitted curve is at 1.51 a.u. further justifies our use of 1.5 a.u. as a cutoff for promoter and non-promoter activity.”

      Another potentially interesting analysis would be to see if k-mer content is correlated with Pnew. That is, determine the abundance of all hexamers in the sequence and see if Pnew is correlated with the number of hexamers present that is one nucleotide distance away from the consensus motifs (such as TcGACA or TAcAAT).

      We performed the suggested analysis by searching for k-mers that correlate with Pnew and found that no k-mer significantly correlates with Pnew (lines 240-248):

      “We then asked whether any k-mers ranging from 1-6 bp correlated with the non-promoter Pnew values (5,460 possible k-mers). 718 of these 1-6 bp k-mers are present 3 or more times in at least one non-promoter parent. We calculated a linear regression between the frequency of these 718 k-mers and each Pnew value, and adjusted the p-values to respective q-values (Benjamini-Hochberg correction, FDR=0.05). This analysis revealed six k-mers: CTTC, GTTG,

      ACTTC, GTTGA, AACTTC, TAACTT which correlate with Pnew. However, these correlations are heavily influenced by an outlying Pnew value of 0.41 (P8-RFP) (Fig S5C-H), and upon removing P8-RFP from the analysis, no k-mer significantly correlates with Pnew (data not shown)”

      Line 152-157: How did you define the thresholds for 'active' or 'inactive'? It is not clear in the methods how this distinction was made.

      We have more clearly defined these thresholds in the text. A sequence with promoter activity has a fluorescence score greater than 1.5 a.u. (lines 168-172):

      “We declared a daughter sequence to have promoter activity or to be a promoter if its score was greater than or equal to 1.5 a.u., as this score lies at the boundary between no fluorescence and weak fluorescence based on the sort-seq bins (methods). Otherwise, we refer to a daughter sequence as having no promoter activity or being a non-promoter.”

      Lines: 152-157: In trying to find the parent expression levels, no figure was available showing the distribution of parent expression levels. Furthermore, In looking at Data S2 & filtering out for sequences with distance 0 from the parent, I found the most active sequences did not match up with the sequences described as active in this section (e.g. p19 and p20 have a higher topstrand mean over P22, yet are not listed as active top strand sequences).

      We really appreciate you taking the time to examine the supplemental data. We previously listed the parents that had only GFP activity but no RFP activity (P22), and only RFP activity but no GFP activity (P6, P12, P13, P18, P21). We then said that P19 and P20 were bidirectional promoters, because they showed both GFP and RFP activity. In hindsight, we realize that our wording was confusing. We thus rewrote the affected paragraph, such that the bidirectional promoters are now in both lists of GFP/RFP active parents. We also now make the distinction between “templates” which comprise our 25 promoter island fragments, and “parents”, where we treat both strands separately (50 parents total). The paragraph in question now reads (lines 173-187):

      “Because some sequences in our library are unmutated parent sequences, we determined that 10/50 of the parent sequences already encode promoter activity before mutagenesis. Specifically, three parents drove expression on the top strand (P19-GFP, P20-GFP, P22-GFP), and five did on the bottom strand (P6-RFP, P12-RFP, P13-RFP, P18-RFP, P19-RFP, P20-RFP, P21-RFP). Two parents harbor bidirectional promoters (P19 and P20). The remaining 40 parent sequences are non-promoters, with an average fluorescence score of 1.39 a.u. We note that some of these parents have a fluorescence score higher than 1.39 a.u., but less than 1.50 a.u. such as P8-RFP (1.38 a.u.), P16-RFP (1.39 a.u.), P9-GFP (1.49 a.u.), and P1-GFP (1.47 a.u.). Whether these are truly “promoters” or not, is based solely on our threshold value of 1.5 a.u. We also note that 30% (15/50) of parents have the TGn motif upstream of a -10 box, but only 20% (3/15) of these parents have promoter activity (underlined with promoter activity: P4-RFP, P6-RFP, P8-RFP, P9RFP, P10-RFP, P11-GFP, P12-GFP, P17-GFP, P18-GFP, P18-RFP, P19-RFP, P22-RFP, P24-GFP, P25-GFP, P25RFP). See Fig S4 for fluorescence score distributions for each parent and its daughters, and Data S2 for all daughter sequence fluorescence scores.”

      Please include a supplementary figure showing the different parent expression levels (GFP mean +/- sd). Also, please explain the discrepancy in the 'active sequences' compared to Data S2 or correct my misunderstanding.

      We have added this plot to Figure S4B. The discrepancy arose because we listed the parents that had only GFP activity but no RFP activity (P22), and only RFP activity but no GFP activity (P6, P12, P13, P18, P21). We then said that P19 and P20 were bidirectional promoters, because they showed both GFP and RFP activity. previous response regarding the ambiguity.

      Line 182: I do not see 'Fuqua and Wagner 2023' in the references (though I am familiar with the preprint).

      We have added Fuqua and Wagner, BiorXiv 2023 to the references.

      Lines 197 - 200: The distribution of hotspot locations should be compared to the distribution of mutations in the library. e.g. It is not notable that 17% of mutations are in -10 motifs if 17% of all mutations are in -10 motifs.

      Thank you for raising this point. To address it, we carried out a computational analysis where we randomly scrambled the nucleotides of each parent sequence while maintaining the coordinates for each mutual information “hotspot.” This scrambling results in significantly less overlap with hotspots and boxes. This analysis is now depicted in Figure 2C and written in lines 272-296.

      Lines 253-264: Examples 3B, 3D, and 3F should indicate the spacing between the new and existing motifs. Are these close to the 15-19 bp spacer lengths preferred by sigma70?

      Point well taken. We now annotate the spacing of motifs in Figures 3, 4, 5, and Supplemental Figures S8, S9, S10, and S11. We note that in many cases, high-scoring PWM hits for the same motif can overlap (i.e. two -10 motifs or two -35 motifs overlap). Additionally, the proximity of a 35 and -10 box does not guarantee that the two boxes are interacting. Together, these two facts can result in an ambiguity of the spacer size between two boxes. To avoid any reporting bias, we thus often report spacer sizes as a range (see Figure panels 4F, S8D, S8F-L, S9A, S9H, S10A, and S10E). The smallest spacer we annotate is in Figure 4F with 10 bp, and the largest is in Figure S8D with 26 bp. Any more “extreme” distances are not annotated, and for the reader to decide if an interaction is present or not.

      Line 255: While fun, I am concerned about the 'Shiko' analogy. My understanding is the prevailing theory is that -35 recognition occurs before -10 recognition (https://doi.org/10.1073/pnas.94.17.9022, 10.1101/sqb.1998.63.141). Given this, the 'Shiko -35' concept in 3H is a bit awkward as it suggests that sigma70 stops at -10 motifs before planting down on the -35. Considering the cited paper is still in the preprint stages (and did not observe these Shiko -35 emergences), I am concerned about how this particular example will be received by the community. Perhaps more care could be done to verify that this example is consistent with generally accepted mechanisms of promoter recognition or a short clarification could be added to clarify the extent of the analogy.

      Thank you for raising this point. We decided to remove the Shiko analogy, because several readers assumed that it relates to the physical binding of RNA polymerase, rather than being an evolutionary mechanism of mutations forming complementary motifs in a stepwise manner.

      Lines 323-326: It would be helpful to describe a more systematic approach to defining emergence events into different categories. A clear definition of each category in the methods or main text would help others consistently refer to these concepts in the future. This could be helped by showing the actual parent vs daughter sequences as a supplementary figure to figures 4B, 4D, & 4G.

      We agree this could have been more clearly communicated. We have addressed this by 1) simplifying the nomenclatures of these categories and  2) clearly defining these categories, and 3) showing the actual parent vs daughter sequences in Figure 4, and Supplemental Figures S9, S10, S11, and S12. More specifically:

      (1) Simplifying the nomenclature. We highlight events where gaining new -10 and -35 boxes can modify the promoter activity of parent sequences with promoter activity. This occurs when a new -10 or -35 box appears that partially overlaps with the -10 or -35 box of the actual promoter. Thus, we rename two terms: hetero-gain and homo-gain, shown in Figure 4B:

      (2) We clearly define these categories (lines 430-435):

      “We found that these mutations frequently create new boxes overlapping those we had identified as part of a promoter (Fig S9). This occurs when mutations create a -10 box overlapping a -10 box, a -35 box overlapping a 35 box, a -10 box overlapping a -35 box, or a -35 box overlapping a -10 box. We call the resulting event a “homogain” when the new box is of the same type as the one it overlaps, and otherwise a “hetero-gain”. In either case, the creation of the new box does not always destroy the original box.”

      In the original manuscript, there was an additional third category, where gaining a -35 box upstream of the promoter’s -35 box, and gaining a -10 box upstream of the promoter’s -10 box decreased expression. We referred to this as a “tandem motif” and it can be found in Figure S12C,D. However, in response to comment “(4) Ignoring or misrepresenting the literature” from Reviewer #3, we carried out an analysis of the binding of H-NS (see Figure 5 and Figure S12). This analysis revealed that this “tandem motif” phenomenon was actually the result of changing the affinity of H-NS to these regions. Thus, the “tandem motif” is probably spurious.

      DISCUSSION:

      Line 378-379: Since hotspots are essentially areas where promoters appear, wouldn't it be obvious that having more hotspots (i.e. areas where more promoters appear) would equate to a higher probability of new promoters? It would be helpful to clarify why this isn't obvious. This could be resolved by adding more complexity to the statement, such as showing that the level of mutual information found in a hotspot or across all hotspots in a sequence is correlated with Pnew.

      A fair criticism. In response, we have chosen to remove the analysis of this trend from the manuscript entirely. (Additionally, Pnew and mutual information calculations both relied on the fluorescence scores of daughter sequences, so the finding was circular in its logic.)

      Line 394-396: This comparison of findings to Bykov et al should include a bit more justification for the proposed mechanism and how it specifically was observed in this paper. What did they observe and how do these findings relate?

      We gladly followed this suggestion, and added the following two paragraphs to the discussion (lines 622-640).

      “A previous study randomly mutagenized the appY promoter island upstream of a GFP reporter, and isolated variants with increased and decreased GFP expression. The authors found that variants with higher GFP expression acquired mutations that 1) improve a -10 box to better match its consensus, and simultaneously 2) destroy other -10 and -35 boxes (Bykov et al., 2020). The authors concluded that additional -10 and -35 boxes repress expression driven by promoter islands. Our data challenge this conclusion in several ways. 

      First, we find that only ~13% of -10 and -35 boxes in promoter islands actually contribute to promoter activity. Extrapolating this percentage to the appY promoter island, ~87% (100% - 13%) of the motifs would not be contributing to its activity. Assuming the appY promoter island is not an outlier, this would insinuate that during random mutagenesis, these inert motifs might have accumulated mutations that do not change fluorescence. Indeed, Bykov et al. (Bykov et al., 2020) also found that a similar frequency of -10 and -35 boxes were destroyed in variants selected for lower GFP expression, which supports this argument. Second, we find no evidence that creating a -10 or -35 box lowers promoter activity in any of our 50 parent sequences. Third, we also find no evidence that destruction of a -10 or -35 box increases promoter activity without plausible alternative explanations, i.e. overlap of the destroyed box with a H-NS site, destruction of the promoter, or simultaneous creation of another motif as a result of the destruction. In sum, -10 and 35 boxes are not likely to repress promoter activity. “

      METHODS:

      Line 500: Could you provide more details on PMR1 (e.g. size, copy number, RBS strength) or a reference? I could not find this easily.

      Thank you for pointing out this oversight. In response, we have added the following subsection to the methods (lines 740-748):

      “Plasmid MR1 (pMR1)

      The plasmid MR1 (pMR1) is a variant of the plasmid RV2 (pRV2) in which the kan resistance gene has been swapped with the cm resistance gene (Guazzaroni and Silva-Rocha, 2014). Plasmid pMR1 encodes the BBa_J34801 ribosomal binding site (RBS, AAAGAGGAGAAA) 6 bp upstream of the start codon for GFP(LVA). The plasmid also encodes a putative RBS (AAGGGAGG) (Cazemier et al., 1999) 5 bp upstream of the start codon for mCherry on the opposite strand.

      The plasmid additionally contains the low-to-medium copy number origin of replication p15A (Westmann et al., 2018).

      A map of the plasmid is available on the Github repository: https://github.com/tfuqua95/promoter_islands.”

      Line 581: What was the sequencing instrument &/or depth?

      We now report this information as follows (Methods, lines 918-922):

      “Illumina sequencing

      The amplicon pool was sequenced by Eurofins Genomics (Eurofins GmbH, Germany) using a NovaSeq 6000 (Illumina, USA) sequencer, with an S4 flow cell, and a PE150 (Paired-end 150 bp) run. In total, 282’843’000 reads and 84’852’900’000 bases were sequenced. Raw sequencing reads can be found here: https://www.ncbi.nlm.nih.gov/bioproject/1071572.”

      SUPPLEMENT:

      Supplementary Figure 2: Why does the GFP control produce a bimodal distribution?

      The GFP+ culture was inoculated directly from a glycerol stock. The bimodal distribution probably results from a subset of the bacteria having lost the GFP-coding insert, because the left-most peak coincides with the negative control.

      Reviewer #2 (Recommendations For The Authors):

      This paper would benefit from a clear definition of what constitutes an active promoter as this is only mentioned as justification for the use of arbitrary values for fluorescence.

      Good point. To clarify, we now include this new paragraph in the introduction (lines 112-119):

      “In this study, we define a promoter as a DNA sequence that drives the expression of a (fluorescent) protein whose expression level, measured by its fluorescence, is greater than a defined threshold. We use a threshold of 1.5 arbitrary units (a.u.) of fluorescence. This definition does not distinguish between transcription and translation. We chose it because protein expression is usually more important than RNA expression whenever natural selection acts on gene expression, because it is the primary phenotype visible to natural selection (Jiang et al., 2023).”

      There needs to be a clear distinction in the use of the word sequences as often interchange sequences when meaning the 25 parent sequences and then the 50 possible sequences directions the promoter can act. It is confusing going from one to the other.

      We agree that this distinction is important. To make it clearer, we now introduce an additional term (lines 119-130). Our experiments start from 25 promoter island fragments (P1-P25), which we now call template sequences. Each template sequence comprises both DNA strands. The parent sequences are the top and bottom strands of each template sequence. Therefore, there are now 50 parent sequences (P1-GFP, P1-RFP, P2-GFP…, P25-RFP). By treating each strand as its own sequence, we no longer have to refer to the strand, avoiding the earlier confusion.

      The description of the hotspots is often unclear and trying to determine if 3 out of 9 hotspots come from one parent sequence or multiple is not possible. A table denoting this information would be most helpful.

      We agree, and now provide this information in Data S3.

      Finally, the description of the proposed mechanism of promoter activation via mutation of motifs should not be in the results but in the discussion, as it has insufficient evidence and would require further experimental validation.

      We remedied this problem by providing experimental validation of the proposed mechanisms. Specifically, we created the precise mutations that caused a loss or gain of a -10 or a -35 box, and measured the level of gene expression they drive with a plate reader. Because we chose to provide this experimental validation, we opted to leave the mechanisms of promoter activation in the results section.

      The (Fuqua and Wagner 20023) paper is not in the references.

      We have added Fuqua and Wagner, BiorXiv 2023 to the references.

      I enjoyed the paper and wish the authors the best for their future work.

      Thank you for taking the time to review our manuscript!

      Reviewer #3 (Recommendations For The Authors):

      The paper has major flaws. For example:

      The data need to be analysed with correct promoter sequence element sequences (TTGACA for the -35 element).

      The discrepancy lies in the frequency of A’s vs C’s at position #5 of the PWM. Our PWM was built with more A’s than C’s at this position, but also includes C’s in this position. However, we respectfully disagree that using a different -35 box PWM is going to change the outcomes of our study. First, positions 4-6 of the PWM barely have any information content (bits) compared to positions 1-3 (see Fig 1A). This assertion is not just based on our own PWM, but based on ample precedent in the literature. In PMID 14529615, TTG is present in 38% of all -35 boxes, but ACA only 8%. In PMID 29388765, with the -10 instance TATAAT, the -35 instance TTGCAA yields stronger promoters compared to the -35 instance TTGACA (See their Figure 3B). In PMID 29745856 (Figure 2), the most information content lies in positions 1-3, with the A and C at position 5 both nearly equally represented, as in our PWM. In PMID 33958766 (Figure 1) an experimentally-derived -35 box is even reduced to a “partial” -35 box which only includes positions 1 and 2, with consensus: TTnnnn. Additionally, the -35 box PWM that we used significantly and strongly correlates with an experimentally derived -35 box (see Supporting Information from Figure S4 of Belliveau et al., PNAS 2017. Pearson correlation coefficient = 0.89). We now provide DNA sequences for each of the figures to improve accessibility and reproducibility. A reader can now use any PWM or method they wish to interpret the data.

      The data need to be analysed taking into account the role of other promoter elements and sequences for translation.

      Point well taken. 

      Thank you for bringing this oversight to our attention. We have performed two independent analyses to explore the role of TGn in promoter emergence in evolution. First, we computationally searched for -10 boxes with the bases TGn immediately upstream of them in the parent sequences, and found 18 of these “extended -10 boxes” in the parents (lines 143145):

      “On average, each parent sequence contains ~5.32 -10 boxes and ~7.04 -35 boxes (Fig S1). 18 of these -10 boxes also include the TGn motif upstream of the hexamer.”

      However, only 20% of these boxes were found in parents with promoter activity (lines 182-185):

      “We also note that 30% (15/50) of parents have the TGn motif upstream of a -10 box, but only 20% (3/15) of these parents have promoter activity (underlined with promoter activity: P4-RFP, P6-RFP, P8-RFP, P9-RFP, P10-RFP, P11GFP, P12-GFP, P17-GFP, P18-GFP, P18-RFP, P19-RFP, P22-RFP, P24-GFP, P25-GFP, P25-RFP).” 

      Second, we computationally searched through all of the daughter sequences to identify new -10 boxes with TGn immediately upstream. We found 114 -10 boxes with the bases TGn upstream. However, only 5 new -10 boxes (2 with TGn) were associated with increasing fluorescence (lines 338-345):

      “Mutations indeed created many new -10 and -35 boxes in our daughter sequences. On average, 39.5 and 39.4 new 10 and -35 boxes emerged at unique positions within the daughter sequences of each mutagenized parent (Fig 3A,B), with 1’562 and 1’576 new locations for -10 boxes and -35 boxes, respectively. ~22% (684/3’138) of these new boxes are spaced 15-20 bp away from their cognate box, and ~7.3% (114/1’562) of the new -10 boxes have the TGn motif upstream of them. However, only a mere five of the new -10 boxes and four of the new -35 boxes are significantly associated with increasing fluorescence by more than +0.5 a.u. (Fig 3C,D).”

      In addition, we now study the role of UP elements. This analysis showed that the UP element plays a negligible role in promoter emergence within our dataset.  It is discussed in a new subsection of the results (lines 591-608).

      “The UP-element does not strongly influence promoter activity in our dataset.

      The UP element is an additional AT-rich promoter motif that can lie stream of a -35 box in a promoter sequence (Estrem et al., 1998; Ross et al., 1993). We asked whether the creation of UP-elements also creates or modulates promoter activity in our dataset. To this end, we first identified a previously characterized position-weight matrix for the UP element (NNAAAWWTWTTTTNNWAAASYM, PWM threshold score = 19.2 bits) (Estrem et al., 1998) (Fig S13A). We then computationally searched for UP-element-specific hotspots within the parent sequences, i.e., locations in which mutations that gain or lose UP-elements lead to significant fluorescence increases (Mann-Whitney U-test, Fig S7 and methods. See Data S8 for the coordinates, fluorescence changes, and significance). The analysis did not identify any UP elements whose mutation significantly changes fluorescence. 

      We then repeated the analysis with a less stringent PWM threshold of 4.8 bits (1/4th of the PWM threshold score). This time, we identified 74 “UP-like” elements that are created or destroyed at unique positions within the parents. 23 of these motifs significantly change fluorescence when created or destroyed. However, even with this liberal threshold, none of these UP-like elements increase fluorescence by more than 0.5 a.u. when gained, or decrease fluorescence by more than 0.5 a.u. when lost (Fig S13B). This finding ultimately suggests that the UP element plays a negligible role in promoter emergence within our dataset.”

      Collectively, these additional analyses suggest that the presence of TGn plus a -10 box is insufficient to create promoter activity, and that the UP element does not play a significant role in promoter emergence or evolution.

      The full sequences used need to be provided and mutations resulting in new promoters need to be shown.

      To Figures 3, 4, 5, and Supplemental Figures S8, S9, S10, S11, and S12, we have added the sequences which created or the destroyed the promoters, and their PWM scores.

      The paper needs to be rewritten to take into account the relevant literature on i) promoter islands (i.e. sections of horizontally acquired AT-rich DNA) ii) generation and loss of promoters by mutation.

      We have rewritten the introduction. The majority of these points are now addressed in the following two new paragraphs (lines 92-112):

      “Recent work shows that mutations can help new promoters to emerge from promoter motifs or from sequences adjacent to such motifs (Bykov et al., 2020; Fuqua and Wagner, 2023; Yona et al., 2018). However, encoding -10 and -35 boxes is insufficient to drive complete transcription of a gene coding sequence. For instance, the E. coli genome contains clusters of -10 and -35 boxes that are bound by RNA polymerase and produce short oligonucleotide fragments, but rarely create complete transcripts. Such clusters are called promoter islands, and are strongly associated with horizontally-transferred DNA (Bykov et al., 2020; Panyukov and Ozoline, 2013; Purtov et al., 2014; Shavkunov et al., 2009). 

      There are two proposed explanations for why promoter islands do not create full transcripts. First, the TF H-NS may repress promoter activity in promoter islands. This is because in a Δhns background, transcript levels from the promoter islands increases (Purtov et al., 2014). However, mutagenizing a specific promoter island (appY) until it transcribes a GFP reporter, reveals that in-vitro H-NS binding does not significantly change when GFP levels increase (Bykov et al., 2020). Thus, it is not clear whether H-NS actually represses the complete transcription of these sequences. The second proposed explanation is that excessive promoter motifs silence transcription. The aforementioned study found that promoter activity increases when mutations improve a -10 box to better match its consensus (TAAAAAT→TATACT), while simultaneously destroying surrounding -10 and -35 boxes (Bykov et al., 2020). However, we note that if these surrounding motifs never contributed to GFP fluorescence to begin with, then mutations could also simply have accumulated in them during random mutagenesis without affecting promoter activity.”

      In closing, we would like to thank all three reviewers again for your time to engage with this manuscript.

      Summary of specific changes that we have made to each section of the manuscript 

      • Abstract

      - We updated the abstract to include the finding that more than 1’500 new -10s and 35s are created in our dataset, but only ~0.3% of them actually create de-novo promoter activity.

      - We no longer highlight the conclusion that the majority of promoters emerge and evolve from -10 and -35 boxes.

      • Introduction

      - We have added more background information about the UP-element and the TGn motif.

      - We better describe the promoter islands and the results identified by Bykov et al., 2020.

      • Results: Promoter island sequences are enriched with motifs for -10 and -35 boxes.

      - We clarify how the -10 and -35 PWMs we use were derived.

      - We refer to the 25 promoter island fragments as “Template sequences” (P1-P25). The “parent sequences” now correspond to the top and bottom strands of each template (N=50, P1-GFP, P1-RFP, P2-GFP, …, P25-RFP).

      - We elaborate that ~7% of the -10 boxes in the template sequences have the TGn motif.

      - In the previous version of the manuscript, if there were overlapping -10 boxes or overlapping -35 box, we counted these to be a single -10 box or a single -35 box, respectively. In the new version of the manuscript, we now treat each motif as an independent box. Because of this, the number of -10 and -35 boxes per parent have slightly increased.  

      •Results: Non-promoters vary widely in their potential to become promoters.

      - We make a clear distinction between promoters and non-promoters, and define the parent sequences.

      - We note that only 20% of parents with an “extended -10 box” have promoter activity.

      • Results: Promoter emergence correlates with minute differences in background promoter levels.

      - We added an analysis where we compare Pnew to the parent fluorescence levels, even if they are below 1.5 a.u. We find that the distribution of Pnew matches a sigmoid function.

      • Results: Promoter emergence does not correlate with simple sequence features

      - We added an analysis comparing k-mer counts to Pnew.

      - We updated the way we count -10 and -35 boxes, and recalculated the correlation with Pnew. The P and R2 values have changed, but Pnew still does not significantly correlate with -10 or -35 box counts.

      • Results: Promoters emerge and evolve only from specific subsets of -10 and -35 boxes

      - We have added an analysis where we computationally scramble the wild-type parent sequences while maintaining the coordinates of the mutual information hotspots. This reveals that the overlap with -10 and -35 motifs is not a coincidence of dense promoter motif encoding.

      We found a computational error in our analysis and updated the percent overlap between -10 boxes and -35 boxes with mutual information hotspots. The results are similar. o 14% of -10 boxes overlap with hotspots with our new way of defining -10 and -35 boxes.

      • Results: New -10 and -35 boxes readily emerge, but rarely lead to de-novo promoter activity

      - We quantify how often a new -10 and -35 box is created at a unique position within our collection of promoter fragments, and how often this results in a -10 and -35 box being appropriately spaced, and how often this actually leads to de-novo promoter activity. o We quantify how often a TGn sequence lies upstream of a new -10 box.

      • Results: Promoters can emerge when mutations create motifs but not by destroying them.

      - For each example, we added the DNA sequences of the wild-type region of interest and the mutant region of interest that results in the gain of promoter activity, and their respective PWM scores. 

      - We created constructs to validate each example by testing their fluorescence on a plate reader.

      - We removed the P1-GFP example from the main figure, as it was a false-positive in the dataset. It is now in Fig S8.

      - We removed the Shiko Emergence metaphor because it could be confused with a binding mechanism for RNA polymerase.

      • Results – Gaining new motifs over existing motifs increases and decreases promoter activity.

      - We removed the “Tandem motif” because it is more likely caused by H-NS binding.

      - We renamed the mechanisms to be “hetero-gain” and “homo-gain” for simplicity, and clearly define how we classified each sequence into each category.

      - We now include the DNA sequences, the PWM scores, the spacer lengths, and the fluorescence values from constructs harboring the predicted point mutations.

      • Results – Histone-like nucleoid-structuring protein (H-NS) represses P12-RFP and P22-GFP.

      - This is a new analysis, which explores the role of the TF H-NS in repressing the parent sequences. 

      - We identified putative H-NS motifs in P12-RFP and P22-GFP.

      - We show experimentally that in a H-NS null background, a bidirectional promoter (P20) becomes unidirectional, even though P20 does not contain an obvious H-NS motif.

      - In the original version of the manuscript, we describe a phenomenon where gaining a -35 box upstream of a promoter’s -35 box, or a -10 box upstream of a promoter’s -10 box significantly decreases expression. We called this phenomenon a “tandem motif.” However, in the newest version of the manuscript, we find that these fluorescence decreases are rescued in a H-NS null background, suggesting the finding was actually due to H-NS binding modulation and not -10 and -35 boxes.

      • Results – The UP-element does not strongly influence promoter activity in our dataset.

      We used a PWM for the UP element to see if gaining or losing UP motifs was significantly correlated with increasing or decreasing expression. Even with a liberal PWM threshold, the analysis did not find any UP elements.

      • Discussion

      - We rewrote the discussion to account for the new analyses and the results on H-NS, the UP-element, and the extended -10.

      - We better explain how our results clash with the results from the Bykov paper.

      - We fit our results into the context of David Grainger’s papers.

      • Methods

      - Added an explanation about pMR1.

      - Added methods describing how we created the point mutation constructs.

      - Added the methods for the plate reader.

      - Added the methods for Illumina sequencing.

      - Added the methods for the sigmoid curve-fitting.

      • Figure 1

      - Panel E compares how Pnew (the probability of a daughter sequence having a fluorescence score greater than 1.5 a.u.) associates with the fluorescence scores of each parent sequence.

      - Panel F was originally in Figure S5. In the originally submitted version of the manuscript, if there were overlapping -10s or overlapping -35s, we counted these to be a single -10 or a single -35, respectively. In the new version of the manuscript, we now treat each motif as an independent box. Because of this, the r2 and p values have changed, but the conclusions have not (Pnew still does not significantly correlate with -10 or -35 box counts).

      • Figure 2

      - Panel C now includes a stacked barplot showing the percentage of -10 and -35 boxes that overlap with mutual information hotspots when the parent sequences are randomly scrambled computationally.

      • Figure 3

      - Panels A-C were added to explain how we define a new -10/-35 box, how many such new boxes each parent has. These panels also illustrate how we associate the presence or absence of a motif with significant changes in fluorescence scores of the daughter sequences.

      - We moved the example of P1-GFP to Figure S8 because when we tested the specific mutation which leads to gaining the -10 box, fluorescence did not change.

      - We now include the DNA sequences, the PWM scores, the spacer lengths, and the fluorescence values from reporter constructs harboring the point mutations predicted by our computational analyses.

      - Cartoons of RNA polymerase have been removed.

      • Figure 4

      - The tandem-motif has been removed from the figure.

      - Cartoons of RNA polymerase have been removed.

      - We now include the DNA sequences, the PWM scores, the spacer lengths, and the fluorescence values from constructs harboring the point mutations predicted by our computational analyses.

      • Figure 5

      - This is a new figure analyzing the role of H-NS in promoter evolution and emergence.

      • Figure S4

      - Panel B now shows the wild-type parent scores and their standard deviations from the sort-seq experiment.

      • Figure S5

      - Panels with -10 and -35 box counts moved to Figure 1.

      - The panel comparing Pnew to hotspot counts was removed.

      - Correlations between different k-mers and Pnew are added to panels C-H.

      • Figure S8

      - We now include the DNA sequences, the PWM scores, the spacer lengths, and the fluorescence values from constructs harboring the point mutations predicted by our computational analyses.

      • Figure S9

      - We now include the DNA sequences, the PWM scores, the spacer lengths, and the fluorescence values from constructs harboring the point mutations predicted by our computational analyses.

      • Figure S10

      - We now include the DNA sequences, the PWM scores, the spacer lengths, and the fluorescence values from constructs harboring the point mutations predicted by our computational analyses.

      • Figure S11

      - Added DNA sequences and PWM scores.

      • Figure S12

      - A new figure with further insights about H-NS.

      • Figure S13

      - A new figure regarding the UP-element analysis.

      • Figure S14

      - Added Panel D to show how we created mutant reporter constructs for validation.

    1. Author response:

      The issue of a control without blue light illumination was raised. Clearly without the light we will not obtain any signal in the fluorescence microscopy experiments, which would not be very informative. Instead, we changed the level of blue light illumination in the fluorescence microscopy experiments (figure 4A) and the response of the bacteria scales with dosage. It is very hard to find an alternative explanation, beyond that the blue light is stressing the bacteria and modulating their membrane potentials.

      One of the referees refuses to see wavefronts in our microscopy data. We struggle to understand whether it is an issue with definitions (Waigh has published a tutorial on the subject in Chapter 5 of his book ‘The physics of bacteria: from cells to biofilms’, T.A.Waigh, CUP, 2024 – figure 5.1 shows a sketch) or something subtler on diffusion in excitable systems. We stand by our claim that we observe wavefronts, similar to those observed by Prindle et al<sup>1</sup> and Blee et al<sup>2</sup> for B. subtilis biofilms.

      The referee is questioning our use of ThT to probe the membrane potential. We believe the Pilizota and Strahl groups are treating the E. coli as unexcitable cells, leading to their problems. Instead, we believe E. coli cells are excitable (containing the voltage-gated ion channel Kch) and we now clearly state this in the manuscript. Furthermore, we include a section here discussing some of the issues with ThT.


      Use of ThT as a voltage sensor in cells

      ThT is now used reasonably widely in the microbiology community as a voltage sensor in both bacterial [Prindle et al]1 and fungal cells [Pena et al]12. ThT is a small cationic fluorophore that loads into the cells in proportion to their membrane potential, thus allowing the membrane potential to be measured from fluorescence microscopy measurements.

      Previously ThT was widely used to quantify the growth of amyloids in molecular biology experiments (standardized protocols exist and dedicated software has been created)13 and there is a long history of its use14. ThT fluorescence is bright, stable and slow to photobleach.

      Author response image 1 shows a schematic diagram of the ThT loading in E. coli in our experiments in response to illumination with blue light. Similar results were previously presented by Mancini et al15, but regimes 2 and 3 were mistakenly labelled as artefacts.

      Author response image 1.

      Schematic diagram of ThT loading during an experiment with E. coli cells under blue light illumination i.e. ThT fluorescence as a function of time. Three empirical regimes for the fluorescence are shown (1, 2 and 3).

      The classic study of Prindle et al on bacterial biofilm electrophysiology established the use of ThT in B. subtilis biofilms by showing similar results occurred with DiSc3 which is widely used as a Nernstian voltage sensor in cellular biology1 e.g. with mitochondrial membrane potentials in eukaryotic organisms where there is a large literature. We repeated such a comparative calibration of ThT with DiSc3 in a previous publication with both B. subtilis and P. aeruginosa cells2. ThT thus functioned well in our previous publications with Gram positive and Gram negative cells.

      However, to our knowledge, there are now two groups questioning the use of ThT and DiSc3 as voltage sensors with E. coli cells15-16. The first by the Pilizota group claims ThT only works as a voltage sensor in regime 1 of Author response image 1 using a method based on the rate of rotation of flagellar motors. Another slightly contradictory study by the Strahl group claims DiSc316 only acts as a voltage sensor with the addition of an ionophore for potassium which allows free movement of potassium through the E. coli membranes.

      Our resolution to this contradiction is that ThT does indeed work reasonably well with E. coli. The Pilizota group’s model for rotating flagellar motors assumes the membrane voltage is not varying due to excitability of the membrane voltage (otherwise a non-linear Hodgkin Huxley type model would be needed to quantify their results) i.e. E. coli cells are unexcitable. We show clearly in our study that ThT loading in E. coli is a function of irradiation with blue light and is a stress response of the excitable cells. This is in contradiction to the Pilizota group’s model. The Pilizota group’s model also requires the awkward fiction of why cells decide to unload and then reload ThT in regimes 2 and 3 of Author response image 1 due to variable membrane partitioning of the ThT. Our simple explanation is that it is just due to the membrane voltage changing and no membrane permeability switch needs to be invoked. The Strahl group’s16 results with DiSc3 are also explained by a neglect of the excitable nature of E. coli cells that are reacting to blue light irradiation. Adding ionophores to the E. coli membranes makes the cells unexcitable, reduces their response to blue light and thus leads to simple loading of DiSc3 (the physiological control of K+ in the cells by voltage-gated ion channels has been short circuited by the addition of the ionophore).

      Further evidence of our model that ThT functions as a voltage sensor with E. coli include:

      1) The 3 regimes in Author response image 1 from ThT correlate well with measurements of extracellular potassium ion concentration using TMRM i.e. all 3 regimes in Author response image 1 are visible with this separate dye (figure 1d).

      2) We are able to switch regime 3 in Author response image 1, off and then on again by using knock downs of the potassium ion channel Kch in the membranes of the E. coli and then reinserting the gene back into the knock downs. This cannot be explained by the Pilizota model.

      We conclude that ThT works reasonably well as a sensor of membrane voltage in E. coli and the previous contradictory studies15-16 are because they neglect the excitable nature of the membrane voltage of E. coli cells in response to the light used to make the ThT fluoresce.

      Three further criticisms of the Mancini et al method15 for calibrating membrane voltages include:

      1) E. coli cells have clutches that are not included in their models. Otherwise the rotation of the flagella would be entirely enslaved to the membrane voltage allowing the bacteria no freedom to modulate their speed of motility.

      2) Ripping off the flagella may perturb the integrity of the cell membrane and lead to different loading of the ThT in the E. coli cells.

      3) Most seriously, the method ignores the activity of many other ion channels (beyond H+) on the membrane voltage that are known to exist with E. coli cells e.g. Kch for K+ ions. The Pilizota groups uses a simple Nernstian battery model developed for mitochondria in the 1960s. It is not adequate to explain our results.

      An additional criticism of the Winkel et al study17 from the Strahl group is that it indiscriminately switches between discussion of mitochondria and bacteria e.g. on page 8 ‘As a consequence the membrane potential is dominated by H+’. Mitochondria are slightly alkaline intracellular organelles with external ion concentrations in the cytoplasm that are carefully controlled by the eukaryotic cells. E. coli are not i.e. they have neutral internal pHs, with widely varying extracellular ionic concentrations and have reinforced outer membranes to resist osmotic shocks (in contrast mitochondria can easily swell in response to moderate changes in osmotic pressure).

      A quick calculation of the equilibrium membrane voltage of E. coli can be easily done using the Nernst equation dependent on the extracellular ion concentrations defined by the growth media (the intracellular ion concentrations in E. coli are 0.2 M K+ and 10-7 M H+ i.e. there is a factor of a million fewer H+ ions). Thus in contradiction to the claims of the groups of Pilizota15 and Strahl17, H+ is a minority determinant to the membrane voltage of E. coli. The main determinant is K+. For a textbook version of this point the authors can refer to Chapter 4 of D. White, et al’s ‘The physiology and biochemistry of prokaryotes’, OUP, 2012, 4th edition.

      Even in mitochondria the assumption that H+ dominates the membrane potential and the cells are unexcitable can be questioned e.g. people have observed pulsatile depolarization phenomena with mitochondria18-19. A large number of K+ channels are now known to occur in mitochondrial membranes (not to mention Ca2+ channels; mitochondria have extensive stores of Ca2+) and they are implicated in mitochondrial membrane potentials. In this respect the seminal Nobel prize winning research of Peter Mitchell (1961) on mitochondria needs to be amended20. Furthermore, the mitochondrial work is clearly inapplicable to bacteria (the proton motive force, PMF, will instead subtly depend on non-linear Hodgkin-Huxley equations for the excitable membrane potential, similar to those presented in the current article). A much more sophisticated framework has been developed to describe electrophysiology by the mathematical biology community to describe the activity of electrically excitable cells (e.g. with neurons, sensory cells and cardiac cells), beyond Mitchell’s use of the simple stationary equilibrium thermodynamics to define the Proton Motive Force via the electrochemical potential of a proton (the use of the word ‘force’ is unfortunate, since it is a potential). The tools developed in the field of mathematical electrophysiology8 should be more extensively applied to bacteria, fungi, mitochondria and chloroplasts if real progress is to be made.


      Related to the previous point, we now cite articles from the Pilizota and Strahl groups in the main text (one from each group). Unfortunately, the space constraints of eLife mean we cannot make a more detailed discussion in the main article.

      In terms of modelling the ion channels, the Hodgkin-Huxley type model proposes that the Kch ion channel can be modelled as a typical voltage-gated potassium ion channel i.e. with a 𝑛<sup>4</sup> term in its conductivity. The literature agrees that Kch is a voltage-gated potassium ion channel based on its primary sequence<sup>3</sup>. The protein has the typical 6 transmembrane helix motif for a voltage-gated ion channel. The agent-based model assumes little about the structure of ion channels in E. coli, other than they release potassium in response to a threshold potassium concentration in their environment. The agent based model is thus robust to the exact molecular details chosen and predicts the anomalous transport of the potassium wavefronts reasonably well (the modelling was extended in a recent Physical Review E article(<sup>4</sup>). Such a description of reaction-anomalous diffusion phenomena has not to our knowledge been previously achieved in the literature<sup>5</sup> and in general could be used to describe other signaling molecules.

      1. Prindle, A.; Liu, J.; Asally, M.; Ly, S.; Garcia-Ojalvo, J.; Sudel, G. M., Ion channels enable electrical communication in bacterial communities. Nature 2015, 527, 59.

      2. Blee, J. A.; Roberts, I. S.; Waigh, T. A., Membrane potentials, oxidative stress and the dispersal response of bacterial biofilms to 405 nm light. Physical Biology 2020, 17, 036001.

      3. Milkman, R., An E. col_i homologue of eukaryotic potassium channel proteins. _PNAS 1994, 91, 3510-3514.

      4. Martorelli, V.; Akabuogu, E. U.; Krasovec, R.; Roberts, I. S.; Waigh, T. A., Electrical signaling in three-dimensional bacterial biofilms using an agent-based fire-diffuse-fire model. Physical Review E 2024, 109, 054402.

      5. Waigh, T. A.; Korabel, N., Heterogeneous anomalous transport in cellular and molecular biology. Reports on Progress in Physics 2023, 86, 126601.

      6. Hodgkin, A. L.; Huxley, A. F., A quantitative description of membrane current and its application to conduction and excitation in nerve. Journal of Physiology 1952, 117, 500.

      7. Dawson, S. P.; Keizer, J.; Pearson, J. E., Fire-diffuse-fire model of dynamics of intracellular calcium waves. PNAS 1999, 96, 606.

      8. Keener, J.; Sneyd, J., Mathematical Physiology. Springer: 2009.

      9. Coombes, S., The effect of ion pumps on the speed of travelling waves in the fire-diffuse-fire model of Ca2+ release. Bulletin of Mathematical Biology 2001, 63, 1.

      10. Blee, J. A.; Roberts, I. S.; Waigh, T. A., Spatial propagation of electrical signals in circular biofilms. Physical Review E 2019, 100, 052401.

      11. Gorochowski, T. E.; Matyjaszkiewicz, A.; Todd, T.; Oak, N.; Kowalska, K., BSim: an agent-based tool for modelling bacterial populations in systems and synthetic biology. PloS One 2012, 7, 1.

      12. Pena, A.; Sanchez, N. S.; Padilla-Garfias, F.; Ramiro-Cortes, Y.; Araiza-Villaneuva, M.; Calahorra, M., The use of thioflavin T for the estimation and measurement of the plasma membrane electric potential difference in different yeast strains. Journal of Fungi 2023, 9 (9), 948.

      13. Xue, C.; Lin, T. Y.; Chang, D.; Guo, Z., Thioflavin T as an amyloid dye: fibril quantification, optimal concentration and effect on aggregation. Royal Society Open Science 2017, 4, 160696.

      14. Meisl, G.; Kirkegaard, J. B.; Arosio, P.; Michaels, T. C. T.; Vendruscolo, M.; Dobson, C. M.; Linse, S.; Knowles, T. P. J., Molecular mechanisms of protein aggregation from global fitting of kinetic models. Nature Protocols 2016, 11 (2), 252-272.

      15. Mancini, L.; Tian, T.; Guillaume, T.; Pu, Y.; Li, Y.; Lo, C. J.; Bai, F.; Pilizota, T., A general workflow for characterization of Nernstian dyes and their effects on bacterial physiology. Biophysical Journal 2020, 118 (1), 4-14.

      16. Buttress, J. A.; Halte, M.; Winkel, J. D. t.; Erhardt, M.; Popp, P. F.; Strahl, H., A guide for membrane potential measurements in Gram-negative bacteria using voltage-sensitive dyes. Microbiology 2022, 168, 001227.

      17. Derk te Winkel, J.; Gray, D. A.; Seistrup, K. H.; Hamoen, L. W.; Strahl, H., Analysis of antimicrobial-triggered membrane depolarization using voltage sensitive dyes. Frontiers in Cell and Developmental Biology 2016, 4, 29.

      18. Schawarzlander, M.; Logan, D. C.; Johnston, I. G.; Jones, N. S.; Meyer, A. J.; Fricker, M. D.; Sweetlove, L. J., Pulsing of membrane potential in individual mitochondria. The Plant Cell 2012, 24, 1188-1201.

      19. Huser, J.; Blatter, L. A., Fluctuations in mitochondrial membrane potential caused by repetitive gating of the permeability transition pore. Biochemistry Journal 1999, 343, 311-317.

      20. Mitchell, P., Coupling of phosphorylation to electron and hydrogen transfer by a chemi-osmotic type of mechanism. Nature 1961, 191 (4784), 144-148.

      21. Baba, T.; Ara, M.; Hasegawa, Y.; Takai, Y.; Okumura, Y.; Baba, M.; Datsenko, K. A.; Tomita, M.; Wanner, B. L.; Mori, H., Construction of Escherichia Coli K-12 in-frame, single-gene knockout mutants: the Keio collection. Molecular Systems Biology 2006, 2, 1.

      22. Schinedlin, J.; al, e., Fiji: an open-source platform for biological-image analysis. Nature Methods 2012, 9, 676.

      23. Hartmann, R.; al, e., Quantitative image analysis of microbial communities with BiofilmQ. Nature Microbiology 2021, 6 (2), 151.


      The following is the authors’ response to the original reviews.

      Critical synopsis of the articles cited by referee 2:

      (1) ‘Generalized workflow for characterization of Nernstian dyes and their effects on bacterial physiology’, L.Mancini et al, Biophysical Journal, 2020, 118, 1, 4-14.

      This is the central article used by referee 2 to argue that there are issues with the calibration of ThT for the measurement of membrane potentials. The authors use a simple Nernstian battery (SNB) model and unfortunately it is wrong when voltage-gated ion channels occur. Huge oscillations occur in the membrane potentials of E. coli that cannot be described by the SNB model. Instead a Hodgkin Huxley model is needed, as shown in our eLife manuscript and multiple other studies (see above). Arrhenius kinetics are assumed in the SNB model for pumping with no real evidence and the generalized workflow involves ripping the flagella off the bacteria! The authors construct an elaborate ‘work flow’ to insure their ThT results can be interpreted using their erroneous SNB model over a limited range of parameters.

      (2) ‘Non-equivalence of membrane voltage and ion-gradient as driving forces for the bacterial flagellar motor at low load’, C.J.Lo, et al, Biophysical Journal, 2007, 93, 1, 294.

      An odd de novo chimeric species is developed using an E. coli  chassis which uses Na+ instead of H+ for the motility of its flagellar motor. It is not clear the relevance to wild type E. coli, due to the massive physiological perturbations involved. A SNB model is using to fit the data over a very limited parameter range with all the concomitant errors.

      (3) Single-cell bacterial electrophysiology reveals mechanisms of stress-induced damage’, E.Krasnopeeva, et al, Biophysical Journal, 2019, 116, 2390.

      The abstract says ‘PMF defines the physiological state of the cell’. This statement is hyperbolic. An extremely wide range of molecules contribute to the physiological state of a cell. PMF does not even define the electrophysiology of the cell e.g. via the membrane potential. There are 0.2 M of K+ compared with 0.0000001 M of H+ in E. coli, so K+ is arguably a million times more important for the membrane potential than H+ and thus the electrophysiology!

      Equation (1) in the manuscript assumes no other ions are exchanged during the experiments other than H+. This is a very bad approximation when voltage-gated potassium ion channels move the majority ion (K+) around!

      In our model Figure 4A is better explained by depolarisation due to K+ channels closing than direct irreversible photodamage. Why does the THT fluorescence increase again for the second hyperpolarization event if the THT is supposed to be damaged? It does not make sense.

      (4) ‘The proton motive force determines E. coli robustness to extracellular pH’, G.Terradot et al, 2024, preprint.

      This article expounds the SNB model once more. It still ignores the voltage-gated ion channels. Furthermore, it ignores the effect of the dominant ion in E. coli, K+. The manuscript is incorrect as a result and I would not recommend publication.

      In general, an important problem is being researched i.e. how the membrane potential of E. coli is related to motility, but there are serious flaws in the SNB approach and the experimental methodology appears tenuous.

      Answers to specific questions raised by the referees

      Reviewer #1 (Public Review):

      Summary:

      Cell-to-cell communication is essential for higher functions in bacterial biofilms. Electrical signals have proven effective in transmitting signals across biofilms. These signals are then used to coordinate cellular metabolisms or to increase antibiotic tolerance. Here, the authors have reported for the first time coordinated oscillation of membrane potential in E. coli biofilms that may have a functional role in photoprotection.

      Strengths:

      - The authors report original data.

      - For the first time, they showed that coordinated oscillations in membrane potential occur in E. Coli biofilms.

      - The authors revealed a complex two-phase dynamic involving distinct molecular response mechanisms.

      - The authors developed two rigorous models inspired by 1) Hodgkin-Huxley model for the temporal dynamics of membrane potential and 2) Fire-Diffuse-Fire model for the propagation of the electric signal.

      - Since its discovery by comparative genomics, the Kch ion channel has not been associated with any specific phenotype in E. coli. Here, the authors proposed a functional role for the putative K+ Kch channel : enhancing survival under photo-toxic conditions.

      We thank the referee for their positive evaluations and agree with these statements.

      Weaknesses:

      - Since the flow of fresh medium is stopped at the beginning of the acquisition, environmental parameters such as pH and RedOx potential are likely to vary significantly during the experiment. It is therefore important to exclude the contributions of these variations to ensure that the electrical response is only induced by light stimulation. Unfortunately, no control experiments were carried out to address this issue.

      The electrical responses occur almost instantaneously when the stimulation with blue light begins i.e. it is too fast to be a build of pH. We are not sure what the referee means by Redox potential since it is an attribute of all chemicals that are able to donate/receive electrons. The electrical response to stress appears to be caused by ROS, since when ROS scavengers are added the electrical response is removed i.e. pH plays a very small minority role if any.

      - Furthermore, the control parameter of the experiment (light stimulation) is the same as that used to measure the electrical response, i.e. through fluorescence excitation. The use of the PROPS system could solve this problem.

      >>We were enthusiastic at the start of the project to use the PROPs system in E. coli as presented by J.M.Krajl et al, ‘Electrical spiking in E. coli probed with a fluorescent voltage-indicating protein’, Science, 2011, 333, 6040, 345. However, the people we contacted in the microbiology community said that it had some technical issues and there have been no subsequent studies using PROPs in bacteria after the initial promising study. The fluorescent protein system recently presented in PNAS seems more promising, ‘Sensitive bacterial Vm sensors revealed the excitability of bacterial Vm and its role in antibiotic tolerance’, X.Jin et al, PNAS, 120, 3, e2208348120.

      - Electrical signal propagation is an important aspect of the manuscript. However, a detailed quantitative analysis of the spatial dynamics within the biofilm is lacking. In addition, it is unclear if the electrical signal propagates within the biofilm during the second peak regime, which is mediated by the Kch channel. This is an important question, given that the fire-diffuse-fire model is presented with emphasis on the role of K+ ions.

      We have presented a more detailed account of the electrical wavefront modelling work and it is currently under review in a physical journal, ‘Electrical signalling in three dimensional bacterial biofilms using an agent based fire-diffuse-fire model’, V.Martorelli, et al, 2024 https://www.biorxiv.org/content/10.1101/2023.11.17.567515v1

      - Since deletion of the kch gene inhibits the long-term electrical response to light stimulation (regime II), the authors concluded that K+ ions play a role in the habituation response. However, Kch is a putative K+ ion channel. The use of specific drugs could help to clarify the role of K+ ions.

      Our recent electrical impedance spectroscopy publication provides further evidence that Kch is associated with large changes in conductivity as expected for a voltage-gated ion channel (https://pubs.acs.org/doi/10.1021/acs.nanolett.3c04446, 'Electrical impedance spectroscopy with bacterial biofilms: neuronal-like behavior', E.Akabuogu et al, ACS Nanoletters, 2024, in print.

      - The manuscript as such does not allow us to properly conclude on the photo-protective role of the Kch ion channel.

      That Kch has a photoprotective role is our current working hypothesis. The hypothesis fits with the data, but we are not saying we have proven it beyond all possible doubt.

      - The link between membrane potential dynamics and mechanosensitivity is not captured in the equation for the Q-channel opening dynamics in the Hodgkin-Huxley model (Supp Eq 2).

      Our model is agnostic with respect to the mechanosensitivity of the ion channels, although we deduce that mechanosensitive ion channels contribute to ion channel Q.

      - Given the large number of parameters used in the models, it is hard to distinguish between prediction and fitting.

      This is always an issue with electrophysiological modelling (compared with most heart and brain modelling studies we are very conservative in the choice of parameters for the bacteria). In terms of predicting the different phenomena observed, we believe the model is very successful.

      Reviewer #2 (Public Review):

      Summary of what the authors were trying to achieve:

      The authors thought they studied membrane potential dynamics in E.coli biofilms. They thought so because they were unaware that the dye they used to report that membrane potential in E.coli, has been previously shown not to report it. Because of this, the interpretation of the authors' results is not accurate.

      We believe the Pilizota work is scientifically flawed.

      Major strengths and weaknesses of the methods and results:

      The strength of this work is that all the data is presented clearly, and accurately, as far as I can tell.

      The major critical weakness of this paper is the use of ThT dye as a membrane potential dye in E.coli. The work is unaware of a publication from 2020 https://www.sciencedirect.com/science/article/pii/S0006349519308793 [sciencedirect.com] that demonstrates that ThT is not a membrane potential dye in E. coli. Therefore I think the results of this paper are misinterpreted. The same publication I reference above presents a protocol on how to carefully calibrate any candidate membrane potential dye in any given condition.

      We are aware of this study, but believe it to be scientifically flawed. We do not cite the article because we do not think it is a particularly useful contribution to the literature.

      I now go over each results section in the manuscript.

      Result section 1: Blue light triggers electrical spiking in single E. coli cells

      I do not think the title of the result section is correct for the following reasons. The above-referenced work demonstrates the loading profile one should expect from a Nernstian dye (Figure 1). It also demonstrates that ThT does not show that profile and explains why is this so. ThT only permeates the membrane under light exposure (Figure 5). This finding is consistent with blue light peroxidising the membrane (see also following work Figure 4 https://www.sciencedirect.com/science/article/pii/S0006349519303923 [sciencedirect.com] on light-induced damage to the electrochemical gradient of protons-I am sure there are more references for this).

      The Pilizota group invokes some elaborate artefacts to explain the lack of agreement with a simple Nernstian battery model. The model is incorrect not the fluorophore.

      Please note that the loading profile (only observed under light) in the current manuscript in Figure 1B as well as in the video S1 is identical to that in Figure 3 from the above-referenced paper (i.e. https://www.sciencedirect.com/science/article/pii/S0006349519308793 [sciencedirect.com]), and corresponding videos S3 and S4. This kind of profile is exactly what one would expect theoretically if the light is simultaneously lowering the membrane potential as the ThT is equilibrating, see Figure S12 of that previous work. There, it is also demonstrated by the means of monitoring the speed of bacterial flagellar motor that the electrochemical gradient of protons is being lowered by the light. The authors state that applying the blue light for different time periods and over different time scales did not change the peak profile. This is expected if the light is lowering the electrochemical gradient of protons. But, in Figure S1, it is clear that it affected the timing of the peak, which is again expected, because the light affects the timing of the decay, and thus of the decay profile of the electrochemical gradient of protons (Figure 4 https://www.sciencedirect.com/science/article/pii/S0006349519303923 [sciencedirect.com]).

      We think the proton effect is a million times weaker than that due to potasium i.e. 0.2 M K+ versus 10-7 M H+. We can comfortably neglect the influx of H+ in our experiments.

      If find Figure S1D interesting. There authors load TMRM, which is a membrane voltage dye that has been used extensively (as far as I am aware this is the first reference for that and it has not been cited https://www.ncbi.nlm.nih.gov/pmc/articles/PMC1914430 [ncbi.nlm.nih.gov]/). As visible from the last TMRM reference I give, TMRM will only load the cells in Potassium Phosphate buffer with NaCl (and often we used EDTA to permeabilise the membrane). It is not fully clear (to me) whether here TMRM was prepared in rich media (it explicitly says so for ThT in Methods but not for TMRM), but it seems so. If this is the case, it likely also loads because of the damage to the membrane done with light, and therefore I am not surprised that the profiles are similar.

      The vast majority of cells continue to be viable. We do not think membrane damage is dominating.

      The authors then use CCCP. First, a small correction, as the authors state that it quenches membrane potential. CCCP is a protonophore (https://pubmed.ncbi.nlm.nih.gov/4962086 [pubmed.ncbi.nlm.nih.gov]/), so it collapses electrochemical gradient of protons. This means that it is possible, and this will depend on the type of pumps present in the cell, that CCCP collapses electrochemical gradient of protons, but the membrane potential is equal and opposite in sign to the DeltapH. So using CCCP does not automatically mean membrane potential will collapse (e.g. in some mammalian cells it does not need to be the case, but in E.coli it is https://www.biorxiv.org/content/10.1101/2021.11.19.469321v2 [biorxiv.org]). CCCP has also been recently found to be a substrate for TolC (https://journals.asm.org/doi/10.1128/mbio.00676-21 [journals.asm.org]), but at the concentrations the authors are using CCCP (100uM) that should not affect the results. However, the authors then state because they observed, in Figure S1E, a fast efflux of ions in all cells and no spiking dynamics this confirms that observed dynamics are membrane potential related. I do not agree that it does. First, Figure S1E, does not appear to show transients, instead, it is visible that after 50min treatment with 100uM CCCP, ThT dye shows no dynamics. The action of a Nernstian dye is defined. It is not sufficient that a charged molecule is affected in some way by electrical potential, this needs to be in a very specific way to be a Nernstian dye. Part of the profile of ThT loading observed in https://www.sciencedirect.com/science/article/pii/S0006349519308793 [sciencedirect.com] is membrane potential related, but not in a way that is characteristic of Nernstian dye.

      Our understanding of the literature is CCCP poisons the whole metabolism of the bacterial cells. The ATP driven K+ channels will stop functioning and this is the dominant contributor to membrane potential.

      Result section 2: Membrane potential dynamics depend on the intercellular distance

      In this chapter, the authors report that the time to reach the first intensity peak during ThT loading is different when cells are in microclusters. They interpret this as electrical signalling in clusters because the peak is reached faster in microclusters (as opposed to slower because intuitively in these clusters cells could be shielded from light). However, shielding is one possibility. The other is that the membrane has changed in composition and/or the effective light power the cells can tolerate (with mechanisms to handle light-induced damage, some of which authors mention later in the paper) is lower. Given that these cells were left in a microfluidic chamber for 2h hours to attach in growth media according to Methods, there is sufficient time for that to happen. In Figure S12 C and D of that same paper from my group (https://ars.els-cdn.com/content/image/1-s2.0-S0006349519308793-mmc6.pdf [ars.els-cdn.com]) one can see the effects of peak intensity and timing of the peak on the permeability of the membrane. Therefore I do not think the distance is the explanation for what authors observe.

      Shielding would provide the reverse effect, since hyperpolarization begins in the dense centres of the biofilms. For the initial 2 hours the cells receive negligible blue light. Neither of the referee’s comments thus seem tenable.

      Result section 3: Emergence of synchronized global wavefronts in E. coli biofilms

      In this section, the authors exposed a mature biofilm to blue light. They observe that the intensity peak is reached faster in the cells in the middle. They interpret this as the ion-channel-mediated wavefronts moved from the center of the biofilm. As above, cells in the middle can have different membrane permeability to those at the periphery, and probably even more importantly, there is no light profile shown anywhere in SI/Methods. I could be wrong, but the SI3 A profile is consistent with a potential Gaussian beam profile visible in the field of view. In Methods, I find the light source for the blue light and the type of microscope but no comments on how 'flat' the illumination is across their field of view. This is critical to assess what they are observing in this result section. I do find it interesting that the ThT intensity collapsed from the edges of the biofilms. In the publication I mentioned https://www.sciencedirect.com/science/article/pii/S0006349519308793#app2 [sciencedirect.com], the collapse of fluorescence was not understood (other than it is not membrane potential related). It was observed in Figure 5A, C, and F, that at the point of peak, electrochemical gradient of protons is already collapsed, and that at the point of peak cell expands and cytoplasmic content leaks out. This means that this part of the ThT curve is not membrane potential related. The authors see that after the first peak collapsed there is a period of time where ThT does not stain the cells and then it starts again. If after the first peak the cellular content leaks, as we have observed, then staining that occurs much later could be simply staining of cytoplasmic positively charged content, and the timing of that depends on the dynamics of cytoplasmic content leakage (we observed this to be happening over 2h in individual cells). ThT is also a non-specific amyloid dye, and in starving E. coli cells formation of protein clusters has been observed (https://pubmed.ncbi.nlm.nih.gov/30472191 [pubmed.ncbi.nlm.nih.gov]/), so such cytoplasmic staining seems possible.

      >>It is very easy to see if the illumination is flat (Köhler illumination) by comparing the intensity of background pixels on the detector. It was flat in our case. Protons have little to do with our work for reasons highlighted before. Differential membrane permittivity is a speculative phenomenon not well supported by any evidence and with no clear molecular mechanism.

      Finally, I note that authors observe biofilms of different shapes and sizes and state that they observe similar intensity profiles, which could mean that my comment on 'flatness' of the field of view above is not a concern. However, the scale bar in Figure 2A is not legible, so I can't compare it to the variation of sizes of the biofilms in Figure 2C (67 to 280um). Based on this, I think that the illumination profile is still a concern.

      The referee now contradicts themselves and wants a scale bar to be more visible. We have changed the scale bar.

      Result section 4: Voltage-gated Kch potassium channels mediate ion-channel electrical oscillations in E. coli

      First I note at this point, given that I disagree that the data presented thus 'suggest that E. coli biofilms use electrical signaling to coordinate long-range responses to light stress' as the authors state, it gets harder to comment on the rest of the results.

      In this result section the authors look at the effect of Kch, a putative voltage-gated potassium channel, on ThT profile in E. coli cells. And they see a difference. It is worth noting that in the publication https://www.sciencedirect.com/science/article/pii/S0006349519308793 [sciencedirect.com] it is found that ThT is also likely a substrate for TolC (Figure 4), but that scenario could not be distinguished from the one where TolC mutant has a different membrane permeability (and there is a publication that suggests the latter is happening https://onlinelibrary.wiley.com/doi/10.1111/j.1365-2958.2010.07245.x [onlinelibrary.wiley.com]). Given this, it is also possible that Kch deletion affects the membrane permeability. I do note that in video S4 I seem to see more of, what appear to be, plasmolysed cells. The authors do not see the ThT intensity with this mutant that appears long after the initial peak has disappeared, as they see in WT. It is not clear how long they waited for this, as from Figure S3C it could simply be that the dynamics of this is a lot slower, e.g. Kch deletion changes membrane permeability.

      The work that TolC provides a possible passive pathway for ThT to leave cells seems slightly niche. It just demonstrates another mechanism for the cells to equilibriate the concentrations of ThT in a Nernstian manner i.e. driven by the membrane voltage.

      The authors themselves state that the evidence for Kch being a voltage-gated channel is indirect (line 54). I do not think there is a need to claim function from a ThT profile of E. coli mutants (nor do I believe it's good practice), given how accurate single-channel recordings are currently. To know the exact dependency on the membrane potential, ion channel recordings on this protein are needed first.

      We have good evidence form electrical impedance spectroscopy experiments that Kch increases the conductivity of biofilms  (https://pubs.acs.org/doi/10.1021/acs.nanolett.3c04446, 'Electrical impedance spectroscopy with bacterial biofilms: neuronal-like behavior', E.Akabuogu et al, ACS Nanoletters, 2024, in print.

      Result section 5: Blue light influences ion-channel mediated membrane potential events in E. coli

      In this chapter the authors vary the light intensity and stain the cells with PI (this dye gets into the cells when the membrane becomes very permeable), and the extracellular environment with K+ dye (I have not yet worked carefully with this dye). They find that different amounts of light influence ThT dynamics. This is in line with previous literature (both papers I have been mentioning: Figure 4 https://www.sciencedirect.com/science/article/pii/S0006349519303923 [sciencedirect.com] and https://ars.els-cdn.com/content/image/1-s2.0-S0006349519308793-mmc6.pdf [ars.els-cdn.com] especially SI12), but does not add anything new. I think the results presented here can be explained with previously published theory and do not indicate that the ion-channel mediated membrane potential dynamics is a light stress relief process.

      The simple Nernstian battery model proposed by Pilizota et al is erroneous in our opinion for reasons outlined above. We believe it will prove to be a dead end for bacterial electrophysiology studies.

      Result section 6: Development of a Hodgkin-Huxley model for the observed membrane potential dynamics

      This results section starts with the authors stating: 'our data provide evidence that E. coli manages light stress through well-controlled modulation of its membrane potential dynamics'. As stated above, I think they are instead observing the process of ThT loading while the light is damaging the membrane and thus simultaneously collapsing the electrochemical gradient of protons. As stated above, this has been modelled before. And then, they observe a ThT staining that is independent from membrane potential.

      This is an erroneous niche opinion. Protons have little say in the membrane potential since there are so few of them. The membrane potential is mostly determined by K+.

      I will briefly comment on the Hodgkin Huxley (HH) based model. First, I think there is no evidence for two channels with different activation profiles as authors propose. But also, the HH model has been developed for neurons. There, the leakage and the pumping fluxes are both described by a constant representing conductivity, times the difference between the membrane potential and Nernst potential for the given ion. The conductivity in the model is given as gK*n^4 for potassium, gNa*m^3*h sodium, and gL for leakage, where gK, gNa and gL were measured experimentally for neurons. And, n, m, and h are variables that describe the experimentally observed voltage-gated mechanism of neuronal sodium and potassium channels. (Please see Hodgkin AL, Huxley AF. 1952. Currents carried by sodium and potassium ions through the membrane of the giant axon of Loligo. J. Physiol. 116:449-72 and Hodgkin AL, Huxley AF. 1952. A quantitative description of membrane current and its application to conduction and excitation in nerve. J. Physiol. 117:500-44).

      In the 70 years since Hodgkin and Huxley first presented their model, a huge number of similar models have been proposed to describe cellular electrophysiology. We are not being hyperbolic when we state that the HH models for excitable cells are like the Schrödinger equation for molecules. We carefully adapted our HH model to reflect the currently understood electrophysiology of E. coli.

      Thus, in applying the model to describe bacterial electrophysiology one should ensure near equilibrium requirement holds (so that (V-VQ) etc terms in authors' equation Figure 5 B hold), and potassium and other channels in a given bacterium have similar gating properties to those found in neurons. I am not aware of such measurements in any bacteria, and therefore think the pump leak model of the electrophysiology of bacteria needs to start with fluxes that are more general (for example Keener JP, Sneyd J. 2009. Mathematical physiology: I: Cellular physiology. New York: Springer or https://journals.plos.org/plosone/article?id=10.1371/journal.pone.0000144 [journals.plos.org])

      The reference is to a slightly more modern version of a simple Nernstian battery model. The model will not oscillate and thus will not help modelling membrane potentials in bacteria. We are unsure where the equilibrium requirement comes from (inadequate modelling of the dynamics?)

      Result section 7: Mechanosensitive ion channels (MS) are vital for the first hyperpolarization event in E. coli.

      The results that Mcs channels affect the profile of ThT dye are interesting. It is again possible that the membrane permeability of these mutants has changed and therefore the dynamics have changed, so this needs to be checked first. I also note that our results show that the peak of ThT coincides with cell expansion. For this to be understood a model is needed that also takes into account the link between maintenance of electrochemical gradients of ions in the cell and osmotic pressure.

      The evidence for permeability changes in the membranes seems to be tenuous.

      A side note is that the authors state that the Msc responds to stress-related voltage changes. I think this is an overstatement. Mscs respond to predominantly membrane tension and are mostly nonspecific (see how their action recovers cellular volume in this publication https://www.pnas.org/doi/full/10.1073/pnas.1522185113 [pnas.org]). Authors cite references 35-39 to support this statement. These publications still state that these channels are predominantly membrane tension-gated. Some of the references state that the presence of external ions is important for tension-related gating but sometimes they gate spontaneously in the presence of certain ions. Other publications cited don't really look at gating with respect to ions (39 is on clustering). This is why I think the statement is somewhat misleading.

      We have reworded the discussion of Mscs since the literature appears to be ambiguous. We will try to run some electrical impedance spectroscopy experiments on the Msc mutants in the future to attempt to remove the ambiguity.

      Result section 8: Anomalous ion-channel-mediated wavefronts propagate light stress signals in 3D E. coli biofilms.

      I am not commenting on this result section, as it would only be applicable if ThT was membrane potential dye in E. coli.

      Ok, but we disagree on the use of ThT.

      Aims achieved/results support their conclusions:

      The authors clearly present their data. I am convinced that they have accurately presented everything they observed. However, I think their interpretation of the data and conclusions is inaccurate in line with the discussion I provided above.

      Likely impact of the work on the field, and the utility of the methods and data to the community:

      I do not think this publication should be published in its current format. It should be revised in light of the previous literature as discussed in detail above. I believe presenting it in it's current form on eLife pages would create unnecessary confusion.

      We believe many of the Pilizota group articles are scientifically flawed and are causing the confusion in the literature.

      Any other comments:

      I note, that while this work studies E. coli, it references papers in other bacteria using ThT. For example, in lines 35-36 authors state that bacteria (Bacillus subtilis in this case) in biofilms have been recently found to modulate membrane potential citing the relevant literature from 2015. It is worth noting that the most recent paper https://journals.asm.org/doi/10.1128/mbio.02220-23 [journals.asm.org] found that ThT binds to one or more proteins in the spore coat, suggesting that it does not act as a membrane potential in Bacillus spores. It is possible that it still reports membrane potential in Bacillus cells and the recent results are strictly spore-specific, but these should be kept in mind when using ThT with Bacillus.

      >>ThT was used successfully in previous studies of normal B. subtilis cells (by our own group and A.Prindle, ‘Spatial propagation of electrical signal in circular biofilms’, J.A.Blee et al, Physical Review E, 2019, 100, 052401, J.A.Blee et al, ‘Membrane potentials, oxidative stress and the dispersal response of bacterial biofilms to 405 nm light’, Physical Biology, 2020, 17, 2, 036001, A.Prindle et al, ‘Ion channels enable electrical communication in bacterial communities’, Nature, 2015, 527, 59-63). The connection to low metabolism pore research seems speculative.

      Reviewer #3 (Public Review):

      It has recently been demonstrated that bacteria in biofilms show changes in membrane potential in response to changes in their environment, and that these can propagate signals through the biofilm to coordinate bacterial behavior. Akabuogu et al. contribute to this exciting research area with a study of blue light-induced membrane potential dynamics in E. coli biofilms. They demonstrate that Thioflavin-T (ThT) intensity (a proxy for membrane potential) displays multiphasic dynamics in response to blue light treatment. They additionally use genetic manipulations to implicate the potassium channel Kch in the latter part of these dynamics. Mechanosensitive ion channels may also be involved, although these channels seem to have blue light-independent effects on membrane potential as well. In addition, there are challenges to the quantitative interpretation of ThT microscopy data which require consideration. The authors then explore whether these dynamics are involved in signaling at the community level. The authors suggest that cell firing is both more coordinated when cells are clustered and happens in waves in larger, 3D biofilms; however, in both cases evidence for these claims is incomplete. The authors present two simulations to describe the ThT data. The first of these simulations, a Hodgkin-Huxley model, indicates that the data are consistent with the activity of two ion channels with different kinetics; the Kch channel mutant, which ablates a specific portion of the response curve, is consistent with this. The second model is a fire-diffuse-fire model to describe wavefront propagation of membrane potential changes in a 3D biofilm; because the wavefront data are not presented clearly, the results of this model are difficult to interpret. Finally, the authors discuss whether these membrane potential changes could be involved in generating a protective response to blue light exposure; increased death in a Kch ion channel mutant upon blue light exposure suggests that this may be the case, but a no-light control is needed to clarify this.

      In a few instances, the paper is missing key control experiments that are important to the interpretation of the data. This makes it difficult to judge the meaning of some of the presented experiments.

      (1) An additional control for the effects of autofluorescence is very important. The authors conduct an experiment where they treat cells with CCCP and see that Thioflavin-T (ThT) dynamics do not change over the course of the experiment. They suggest that this demonstrates that autofluorescence does not impact their measurements. However, cellular autofluorescence depends on the physiological state of the cell, which is impacted by CCCP treatment. A much simpler and more direct experiment would be to repeat the measurement in the absence of ThT or any other stain. This experiment should be performed both in the wild-type strain and in the ∆kch mutant.

      ThT is a very bright fluorophore (much brighter than a GFP). It is clear from the images of non-stained samples that autofluorescence provides a negligible contribution to the fluorescence intensity in an image.

      (2) The effects of photobleaching should be considered. Of course, the intensity varies a lot over the course of the experiment in a way that photobleaching alone cannot explain. However, photobleaching can still contribute to the kinetics observed. Photobleaching can be assessed by changing the intensity, duration, or frequency of exposure to excitation light during the experiment. Considerations about photobleaching become particularly important when considering the effect of catalase on ThT intensity. The authors find that the decrease in ThT signal after the initial "spike" is attenuated by the addition of catalase; this is what would be predicted by catalase protecting ThT from photobleaching (indeed, catalase can be used to reduce photobleaching in time lapse imaging).

      Photobleaching was negligible over the course of the experiments. We employed techniques such as reducing sample exposure time and using the appropriate light intensity to minimize photobleaching.

      (3) It would be helpful to have a baseline of membrane potential fluctuations in the absence of the proposed stimulus (in this case, blue light). Including traces of membrane potential recorded without light present would help support the claim that these changes in membrane potential represent a blue light-specific stress response, as the authors suggest. Of course, ThT is blue, so if the excitation light for ThT is problematic for this experiment the alternative dye tetramethylrhodamine methyl ester perchlorate (TMRM) can be used instead.

      Unfortunately the fluorescent baseline is too weak to measure cleanly in this experiment. It appears the collective response of all the bacteria hyperpolarization at the same time appears to dominate the signal (measurements in the eLife article and new potentiometry measurements).

      (4) The effects of ThT in combination with blue light should be more carefully considered. In mitochondria, a combination of high concentrations of blue light and ThT leads to disruption of the PMF (Skates et al. 2021 BioRXiv), and similarly, ThT treatment enhances the photodynamic effects of blue light in E. coli (Bondia et al. 2021 Chemical Communications). If present in this experiment, this effect could confound the interpretation of the PMF dynamics reported in the paper.

      We think the PMF plays a minority role in determining the membrane potential in E. coli. For reasons outlined before (H+ is a minority ion in E. coli compared with K+).

      (5) Figures 4D - E indicate that a ∆kch mutant has increased propidium iodide (PI) staining in the presence of blue light; this is interpreted to mean that Kch-mediated membrane potential dynamics help protect cells from blue light. However, Live/Dead staining results in these strains in the absence of blue light are not reported. This means that the possibility that the ∆kch mutant has a general decrease in survival (independent of any effects of blue light) cannot be ruled out.

      >>Both strains of bacterial has similar growth curve and also engaged in membrane potential dynamics for the duration of the experiment. We were interested in bacterial cells that observed membrane potential dynamics in the presence of the stress. Bacterial cells need to be alive to engage in membrane potential  dynamics (hyperpolarize) under stress conditions. Cells that engaged in membrane potential dynamics and later stained red were only counted after the entire duration. We believe that the wildtype handles the light stress better than the ∆kch mutant as measured with the PI.

      (6) Additionally in Figures 4D - E, the interpretation of this experiment can be confounded by the fact that PI uptake can sometimes be seen in bacterial cells with high membrane potential (Kirchhoff & Cypionka 2017 J Microbial Methods); the interpretation is that high membrane potential can lead to increased PI permeability. Because the membrane potential is largely higher throughout blue light treatment in the ∆kch mutant (Fig. 3AB), this complicates the interpretation of this experiment.

      Kirchhoff & Cypionka 2017 J Microbial Methods, using fluorescence microscopy, suggested that changes in membrane potential dynamics can introduce experimental bias when propidium iodide is used to confirm the viability of tge bacterial strains, B subtilis (DSM-10) and Dinoroseobacter shibae, that are starved of oxygen (via N2 gassing) for 2 hours. They attempted to support their findings by using CCCP in stopping the membrane potential dynamics (but never showed any pictoral or plotted data for this confirmatory experiment). In our experiment methodology, cell death was not forced on the cells by introducing an extra burden or via anoxia. We believe that the accumulation of PI in ∆kch mutant is not due to high membrane potential dynamics but is attributed to the PI, unbiasedly showing damaged/dead cells. We think that propidium iodide is good for this experiment. Propidium iodide is a dye that is extensively used in life sciences. PI has also been used in the study of bacterial electrophysiology (https://pubmed.ncbi.nlm.nih.gov/32343961/, ) and no membrane potential related bias was reported.

      Throughout the paper, many ThT intensity traces are compared, and described as "similar" or "dissimilar", without detailed discussion or a clear standard for comparison. For example, the two membrane potential curves in Fig. S1C are described as "similar" although they have very different shapes, whereas the curves in Fig. 1B and 1D are discussed in terms of their differences although they are evidently much more similar to one another. Without metrics or statistics to compare these curves, it is hard to interpret these claims. These comparative interpretations are additionally challenging because many of the figures in which average trace data are presented do not indicate standard deviation.

      Comparison of small changes in the absolute intensities is problematic in such fluorescence experiments. We mean the shape of the traces is similar and they can be modelled using a HH model with similar parameters.

      The differences between the TMRM and ThT curves that the authors show in Fig. S1C warrant further consideration. Some of the key features of the response in the ThT curve (on which much of the modeling work in the paper relies) are not very apparent in the TMRM data. It is not obvious to me which of these traces will be more representative of the actual underlying membrane potential dynamics.

      In our experiment, TMRM was used to confirm the dynamics observed using ThT. However, ThT appear to be more photostable than TMRM (especially towars the 2nd peak). The most interesting observation is that with both dyes, all phases of the membrane potential dynamics were conspicuous (the first peak, the quiescent period and the second peak). The time periods for these three episodes were also similar.

      A key claim in this paper (that dynamics of firing differ depending on whether cells are alone or in a colony) is underpinned by "time-to-first peak" analysis, but there are some challenges in interpreting these results. The authors report an average time-to-first peak of 7.34 min for the data in Figure 1B, but the average curve in Figure 1B peaks earlier than this. In Figure 1E, it appears that there are a handful of outliers in the "sparse cell" condition that likely explain this discrepancy. Either an outlier analysis should be done and the mean recomputed accordingly, or a more outlier-robust method like the median should be used instead. Then, a statistical comparison of these results will indicate whether there is a significant difference between them.

      The key point is the comparison of standard errors on the standard deviation.

      In two different 3D biofilm experiments, the authors report the propagation of wavefronts of membrane potential; I am unable to discern these wavefronts in the imaging data, and they are not clearly demonstrated by analysis.

      The first data set is presented in Figures 2A, 2B, and Video S3. The images and video are very difficult to interpret because of how the images have been scaled: the center of the biofilm is highly saturated, and the zero value has also been set too high to consistently observe the single cells surrounding the biofilm. With the images scaled this way, it is very difficult to assess dynamics. The time stamps in Video S3 and on the panels in Figure 2A also do not correspond to one another although the same biofilm is shown (and the time course in 2B is also different from what is indicated in 2B). In either case, it appears that the center of the biofilm is consistently brighter than the edges, and the intensity of all cells in the biofilm increases in tandem; by eye, propagating wavefronts (either directed toward the edge or the center) are not evident to me. Increased brightness at the center of the biofilm could be explained by increased cell thickness there (as is typical in this type of biofilm). From the image legend, it is not clear whether the image presented is a single confocal slice or a projection. Even if this is a single confocal slice, in both Video S3 and Figure 2A there are regions of "haze" from out-of-focus light evident, suggesting that light from other focal planes is nonetheless present. This seems to me to be a simpler explanation for the fluorescence dynamics observed in this experiment: cells are all following the same trajectory that corresponds to that seen for single cells, and the center is brighter because of increased biofilm thickness.

      We appreciate the reviewer for this important observation. We have made changes to the figures to address this confusion. The cell cover has no influence on the observed membrane potential dynamics. The entire biofilm was exposed to the same blue light at each time. Therefore all parts of the biofilm received equal amounts of the blue light intensity. The membrane potential dynamics was not influenced by cell density (see Fig 2C). 

      The second data set is presented in Video S6B; I am similarly unable to see any wave propagation in this video. I observe only a consistent decrease in fluorescence intensity throughout the experiment that is spatially uniform (except for the bright, dynamic cells near the top; these presumably represent cells that are floating in the microfluidic and have newly arrived to the imaging region).

      A visual inspection of Video S6B shows a fast rise, a decrease in fluorescence and a second rise (supplementary figure 4B). The data for the fluorescence was carefully obtained using the imaris software. We created a curved geometry on each slice of the confocal stack. We analyzed the surfaces of this curved plane along the z-axis. This was carried out in imaris.

      3D imaging data can be difficult to interpret by eye, so it would perhaps be more helpful to demonstrate these propagating wavefronts by analysis; however, such analysis is not presented in a clear way. The legend in Figure 2B mentions a "wavefront trace", but there is no position information included - this trace instead seems to represent the average intensity trace of all cells. To demonstrate the propagation of a wavefront, this analysis should be shown for different subpopulations of cells at different positions from the center of the biofilm. Data is shown in Figure 8 that reflects the velocity of the wavefront as a function of biofilm position; however, because the wavefronts themselves are not evident in the data, it is difficult to interpret this analysis. The methods section additionally does not contain sufficient information about what these velocities represent and how they are calculated. Because of this, it is difficult for me to evaluate the section of the paper pertaining to wave propagation and the predicted biofilm critical size.

      The analysis is considered in more detail in a more expansive modelling article, currently under peer review in a physics journal, ‘Electrical signalling in three dimensional bacterial biofilms using an agent based fire-diffuse-fire model’, V.Martorelli, et al, 2024 https://www.biorxiv.org/content/10.1101/2023.11.17.567515v1

      There are some instances in the paper where claims are made that do not have data shown or are not evident in the cited data:

      (1) In the first results section, "When CCCP was added, we observed a fast efflux of ions in all cells"- the data figure pertaining to this experiment is in Fig. S1E, which does not show any ion efflux. The methods section does not mention how ion efflux was measured during CCCP treatment.

      We have worded this differently to properly convey our results.

      (2) In the discussion of voltage-gated calcium channels, the authors refer to "spiking events", but these are not obvious in Figure S3E. Although the fluorescence intensity changes over time, it's hard to distinguish these fluctuations from measurement noise; a no-light control could help clarify this.

      The calcium transients observed were not due to noise or artefacts.

      (3) The authors state that the membrane potential dynamics simulated in Figure 7B are similar to those observed in 3D biofilms in Fig. S4B; however, the second peak is not clearly evident in Fig. S4B and it looks very different for the mature biofilm data reported in Fig. 2. I have some additional confusion about this data specifically: in the intensity trace shown in Fig. S4B, the intensity in the second frame is much higher than the first; this is not evident in Video S6B, in which the highest intensity is in the first frame at time 0. Similarly, the graph indicates that the intensity at 60 minutes is higher than the intensity at 4 minutes, but this is not the case in Fig. S4A or Video S6B.

      The confusion stated here has now been addressed. Also it should be noted that while Fig 2.1 was obtained with LED light source, Fig S4A was obtained using a laser light source. While obtaining the confocal images (for Fig S4A ), the light intensity was controlled to further minimize photobleaching. Most importantly, there is an evidence of slow rise to the 2nd peak in Fig S4B. The first peak, quiescence and slow rise to second peak are evident.

      Recommendations for the authors:

      Reviewer #1 (Recommendations For The Authors):

      Scientific recommendations:

      - Although Fig 4A clearly shows that light stimulation has an influence on the dynamics of cell membrane potential in the biofilm, it is important to rule out the contribution of variations in environmental parameters. I understand that for technical reasons, the flow of fresh medium must be stopped during image acquisition. Therefore, I suggest performing control experiments, where the flow is stopped before image acquisition (15min, 30min, 45min, and 1h before). If there is no significant contribution from environmental variations (pH, RedOx), the dynamics of the electrical response should be superimposed whatever the delay between stopping the flow stop and switching on the light.

      In this current research study, we were focused on studying how E. coli cells and biofilms react to blue light stress via their membrane potential dynamics. This involved growing the cells and biofilms, stopping the media flow and obtaining data immediately. We believe that stopping the flow not only helped us to manage data acquisition, it also helped us reduce the effect of environmental factors. In our future study we will expand the work to include how the membrane potential dynamics evolve in the presence of changing environmental factors for example such induced by stopping the flow at varied times.

      - Since TMRM signal exhibits a linear increase after the first response peak (Supplementary Figure 1D), I recommend mitigating the statement at line 78.

      - To improve the spatial analysis of the electrical response, I suggest plotting kymographs of the intensity profiles across the biofilm. I have plotted this kymograph for Video S3 and it appears that there is no electrical propagation for the second peak. In addition, the authors should provide technical details of how R^2(t) is measured in the first regime (Figure 7E).

      See the dedicated simulation article for more details. https://www.biorxiv.org/content/10.1101/2023.11.17.567515v1

      - Line 152: To assess the variability of the latency, the authors should consider measuring the variance divided by the mean instead of SD, which may depend on the average value.

      We are happy with our current use of standard error on the standard deviation. It shows what we claim to be true.

      - Line 154-155: To truly determine whether the amplitude of the "action potential" is independent of biofilm size, the authors should not normalise the signals.

      Good point. We qualitatively compared both normalized and unnormalized data. Recent electrical impedance spectroscopy measurements (unpublished) indicate that the electrical activity is an extensive quantity i.e. it scales with the size of the biofilms.

      - To precise the role of K+ in the habituation response, I suggest using valinomycin at sub-inhibitory concentrations (10µM). Besides, the high concentration of CCCP used in this study completely inhibits cell activity. Not surprisingly, no electrical response to light stimulation was observed in the presence of CCCP. Finally, the Kch complementation experiment exhibits a "drop after the first peak" on a single point. It would be more convincing to increase the temporal resolution (1min->10s) to show that there is indeed a first and a second peak.

      An interesting experiment for the future.

      - Line 237-238: There are only two points suggesting that the dynamics of hyperpolarization are faster at higher irradiance(Fig 4A). The authors should consider adding a third intermediate point at 17µW/mm^2 to confirm the statement made in this sentence.

      Multiple repeats were performed. We are confident of the robustness of our data.

      - Line 249 + Fig 4E: It seems that the data reported on Fig 4E are extracted from Fig 4D. If this is indeed the case, the data should be normalised by the total population size to compare survival probabilities under the two conditions. It would also be great to measure these probabilities (for WT and ∆kch) in the presence of ROS scavengers.

      - To distinguish between model fitting and model predictions, the authors should clearly state which parameters are taken from the literature and which parameters are adjusted to fit the experimental data.

      - Supplementary Figure 4A: why can't we see any wavefront in this series of images?

      For the experimental data, the wavefront was analyzed by employing the imaris software. We systematically created a ROI with a curved geometry within the confocal stack (the biofilm). The fluorescence of ThT was traced along the surface of the curved geometry was analyzed along the z-axis.

      - Fig 7B: Could the authors explain why the plateau is higher in the simulations than in the biofilm experiments? Could they add noise on the firing activities?

      See the dedicated Martorelli modelling article. In general we would need to approach stochastic Hodgkin-Huxley modelling and the fluorescence data (and electrical impedance spectroscopy data) presented does not have extensive noise (due to collective averaging over many bacteria cells).

      - Supplementary Figure 4B: Why can't we see the second peak in confocal images?

      The second peak is present although not as robust as in Fig 2B. The confocal images were obtained with a laser source. Therefore we tried to create a balance between applying sufficient light stress on the bacterial cells and mitigating photobleaching.

      Editing recommendations:

      The editing recommendations below has been applied where appropriate

      - Many important technical details are missing (e.g. R^2, curvature, and 445nm irradiance measurements). Error bars are missing from most graphs. The captions should clearly indicate if these are single-cell or biofilm experiments, strain name, illumination conditions, number of experiments, SD, or SE. Please indicate on all panels of all figures in the main text and in the supplements, which are the conditions: single cell vs. biofilm, strains, medium, centrifugal vs centripetal etc..., where relevant. Please also draw error bars everywhere.

      We have now made appropriate changes. We specifically use cells when we were dealing with single cells and biofilms when we worked on biofilms. We decided to describe the strain name either on the panel or the image description.

      - Line 47-51: The way the paragraph is written suggests that no coordinated electrical oscillations have been observed in Gram-negative biofilms. However, Hennes et al (referenced as 57 in this manuscript) have shown that a wave of hyperpolarized cells propagates in Neisseria gonorrhoea colony, which is a Gram-negative bacterium.

      We are now aware of this work. It was not published when we first submitted our work and the authors claim the waves of activity are due to ROS diffusion NOT propagating waves of ions (coordinated electrical wavefronts).

      - Line 59: "stressor" -> "stress" or "perturbation".

      The correction has been made.

      - Line 153: Please indicate in the Material&Methods how the size of the biofilm is measured.

      The biofilm size was obtained using BiofilmQ and the step by step guide for using BiofilmQ were stated..

      - Figure 2A: Please provide associated brightfield images to locate bacteria.

      - Line 186: Please remove "wavefront" from the caption. Fig2B only shows the average signal as a function of time.

      This correction has been implemented.

      - Fig 3B,C: Please indicate single cell and biofilm on the panels and also WT and ∆kch.

      - Line 289: I suggest adding "in single cell experiments" to the title of this section.

      - Fig 5A: blue light is always present at regular time intervals during regime I and II. The presence of blue light only in regime I could be misleading.

      - Fig 5C: The curve in Fig 5D seems to correspond to the biofilm case. The curve given by the model, should be compared with the average curve presented in Fig 1D.

      - Fig 6A, B, and C: These figures could be moved to supplements.

      - Line 392: Replace "turgidity" with "turgor pressure".

      - Fig 7C,E: Please use a log-log scale to represent these data and indicate the line of slope 1.

      - Fig 7E: The x-axis has been cropped.

      - Please provide a supplementary movie for the data presented in Fig 7E.

      - Line 455: E. Coli biofilms do not express ThT.

      - Line 466: "\gamma is the anomalous exponent". Please remove anomalous (\gamma can equal 1 at this stage).

      - Line 475: Please replace "section" with "projection".

      - Line 476: Please replace "spatiotemporal" with "temporal". There is no spatial dependency in either figure.

      - Line 500: Please define Eikonal approximation.

      - Fig 8 could be moved to supplements.

      - Line 553: "predicted" -> "predict".

      - Line 593: Could the authors explain why their model offers much better quantitative agreement?

      - Line 669: What does "universal" mean in that context?

      - Line 671: A volume can be pipetted but not a concentration.

      - Line 676: Are triplicates technical or biological replicates?

      - Sup Fig1: Please use minutes instead of seconds in panel A.

      - Model for membrane dynamics: "The fraction of time the Q+ channel is open" -> "The dynamics of Q+ channel activity can be written". Ditto for K+ channel...

      - Model for membrane dynamics: "the term ... is a threshold-linear". This function is not linear at all. Why is it called linear? Also, please describe what \sigma is.

      - ABFDF model: "releasing a given concentration" -> "releasing a local concentration" or "a given number" but it's not \sigma anymore. Besides, this \sigma is unlikely related to the previous \sigma used in the model of membrane potential dynamics in single cells. Please consider renaming one or the other. Also, ions are referred to as C+ in the text and C in equation 8. Am I missing something?

      Reviewer #2 (Recommendations For The Authors):

      I have included all my comments as one review. I have done so, despite the fact that some minor comments could have gone into this section, because I decided to review each Result section. I thus felt that not writing it as one review might be harder to follow. I have however highlighted which comments are minor suggestions or where I felt corrections.

      However, while I am happy with all my comments being public, given their nature I think they should be shown to authors first. Perhaps the authors want to go over them and think about it before deciding if they are happy for their manuscript to be published along with these comments, or not. I will highlight this in an email to the editor. I question whether in this case, given that I am raising major issues, publishing both the manuscript and the comments is the way to go as I think it might just generate confusion among the audience.

      Reviewer #3 (Recommendations For The Authors):

      I was unable to find any legends for any of the supplemental videos in my review materials, and I could not open supplemental video 5.

      I made some comments in the public review about the analysis and interpretation of the time-to-fire data. One of the other challenges in this data set is that the time resolution is limited- it seems that a large proportion of cells have already fired after a single acquisition frame. It would be ideal to increase the time resolution on this measurement to improve precision. This could be done by imaging more quickly, but that would perhaps necessitate more blue light exposure; an alternative is to do this experiment under lower blue light irradiance where the first spike time is increased (Figure 4A).

      In the public review, I mentioned the possible impact of high membrane potential on PI permeability. To address this, the experiment could be repeated with other stains, or the viability of blue light-treated cells could be addressed more directly by outgrowth or colony-forming unit assays.

      In the public review, I mentioned the possible combined toxicity of ThT and blue light. Live/dead experiments after blue light exposure with and without ThT could be used to test for such effects, and/or the growth curve experiment in Figure 1F could be repeated with blue light exposure at a comparable irradiance used in the experiment.

      Throughout the paper and figure legends, it would help to have more methodological details in the main text, especially those that are critical for the interpretation of the experiment. The experimental details in the methods section are nicely described, but the data analysis section should be expanded significantly.

      At the end of the results section, the authors suggest a critical biofilm size of only 4 µm for wavefront propagation (not much larger than a single cell!). The authors show responses for various biofilm sizes in Fig. 2C, but these are all substantially larger. Are there data for cell clusters above and below this size that could support this claim more directly?

      The authors mention image registration as part of their analysis pipeline, but the 3D data sets in Video S6B and Fig. S4A do not appear to be registered- were these registered prior to the velocity analysis reported in Fig. 8?

      One of the most challenging claims to demonstrate in this paper is that these membrane potential wavefronts are involved in coordinating a large, biofilm-scale response to blue light. One possible way to test this might be to repeat the Live/Dead experiment in planktonic culture or the single-cell condition. If the protection from blue light specifically emerges due to coordinated activity of the biofilm, the Kch mutant would not be expected to show a change in Live/Dead staining in non-biofilm conditions.

      Line 140: How is "mature biofilm" defined? Also on this same line, what does "spontaneous" mean here?

      Line 151: "much smaller": Given that the reported time for 3D biofilms is 2.73 {plus minus} 0.85 min and in microclusters is 3.27 {plus minus} 1.77 min, this seems overly strong.

      Line 155: How is "biofilm density" characterized? Additionally, the data in Figure 2C are presented in distance units (µm), but the text refers to "areal coverage"- please define the meaning of these distance units in the legend and/or here in the text (is this the average radius?).

      Lines 161-162: These claims seem strong given the data presented before, and the logic is not very explicit. For example, in the second sentence, the idea that this signaling is used to "coordinate long-range responses to light stress" does not seem strongly evidenced at this point in the paper. What is meant by a long-range response to light stress- are there processes to respond to light that occur at long-length scales (rather than on the single-cell scale)? If so, is there evidence that these membrane potential changes could induce these responses? Please clarify the logic behind these conclusions.

      Lines 235-236: In the lower irradiance conditions, the responses are slower overall, and it looks like the ThT intensity is beginning to rise at the end of the measurement. Could a more prominent second peak be observed in these cases if the measurement time was extended?

      Line 242-243: The overall trajectories of extracellular potassium are indeed similar, but the kinetics of the second peak of potassium are different than those observed by ThT (it rises some minutes earlier)- is this consistent with the idea that Kch is responsible for that peak? Additionally, the potassium dynamics also reflect the first peak- is this surprising given that the Kch channel has no effect on this peak?

      Line 255-256: Again, this seems like a very strong claim. There are several possible interpretations of the catalase experiment (which should be discussed); this experiment perhaps suggests that ROS impacts membrane potential, but does not obviously indicate that these membrane potential fluctuations mitigate ROS levels or help the cells respond to ROS stress. The loss of viability in the ∆kch mutant might indicate a link between these membrane potential experiments and viability, but it is hard to interpret without the no-light control I mention in the public review.

      Lines 313-315: "The model predicts... the external light stress". Please clarify this section. Where this prediction arises from in the modeling work? Second, I am not sure what is meant by "modulates the light stress" or "keeps the cell dynamics robust to the intensity of external light stress" (especially since the dynamics clearly vary with irradiance, as seen in Figure 4A).

      Line 322: I am not sure what "handles the ROS by adjusting the profile of the membrane potential dynamics" means. What is meant by "handling" ROS? Is the hypothesis that membrane potential dynamics themselves are protective against ROS, or that they induce a ROS-protective response downstream, or something else? Later in lines 327-8 the authors write that changes in the response to ROS in the model agree with the hypothesis, but just showing that ROS impacts the membrane potential does not seem to demonstrate that this has a protective effect against ROS.

      Line 365-366: This section title seems confusing- mechanosensitive ion channels totally ablate membrane potential dynamics, they don't have a specific effect on the first hyperpolarization event. The claim that mechanonsensitive ion channels are specifically involved in the first event also appears in the abstract.

      Also, the apparent membrane potential is much lower even at the start of the experiment in these mutants- is this expected? This seems to imply that these ion channels also have a blue light independent effect.

      Lines 368, 371: Should be VGCCs rather than VGGCs.

      Line 477: I believe the figure reference here should be to Figure 7B, not 6B.

      Line 567-568: "The initial spike is key to registering the presence of the light stress." What is the evidence for this claim?

      Line 592-594: "We have presented much better quantitative agreement..." This is a strong claim; it is not immediately evident to me that the agreement between model and prediction is "much better" in this work than in the cited work. The model in Figure 4 of reference 57 seems to capture the key features of their data. Clarification is needed about this claim.

      Line 613: "...strains did not have any additional mutations." This seems to imply that whole genome sequencing was performed- is this the case?

      Line 627: I believe this should refer to Figure S2A-B rather than S1.

      Line 719: What percentage of cells did not hyperpolarize in these experiments?

      Lines 751-754: As I mentioned above, significant detail is missing here about how these measurements were made. How is "radius" defined in 3D biofilms like the one shown in Video S6B, which looks very flat? What is meant by the distance from the substrate to the core, since usually in this biofilm geometry, the core is directly on the substrate? Most importantly, this only describes the process of sectioning the data- how were these sections used to compute the velocity of ThT signal propagation?

      I also have some comments specifically on the figure presentation:

      Normalization from 0 to 1 has been done in some of the ThT traces in the paper, but not all. The claims in the paper would be easiest to evaluate if the non-normalized data were shown- this is important for the interpretation of some of the claims.

      Some indication of standard deviation (error bars or shading) should be added to all figures where mean traces are plotted.

      Throughout the paper, I am a bit confused by the time axis; the data consistently starts at 1 minute. This is not intuitive to me, because it seems that the blue light being applied to the cells is also the excitation laser for ThT- in that case, shouldn't the first imaging frame be at time 0 (when the blue light is first applied)? Or is there an additional exposure of blue light 1 minute before imaging starts? This is consequential because it impacts the measured time to the first spike. (Additionally, all of the video time stamps start at 0).

      Please increase the size of the scale bars and bar labels throughout, especially in Figure 2A and S4A.

      In Figure 1B and D, it would help to decrease the opacity on the individual traces so that more of them can be discerned. It would also improve clarity to have data from the different experiments shown with different colored lines, so that variability between experiments can be clearly visualized.

      Results in Figure 1E would be easier to interpret if the frequency were normalized to total N. It is hard to tell from this graph whether the edges and bin widths are the same between the data sets, but if not, they should be. Also, it would help to reduce the opacity of the sparse cell data set so that the full microcluster data set can be seen as well.

      Biofilm images are shown in Figures 2A, S3A, and Video S3- these are all of the same biofilm. Why not take the opportunity to show different experimental replicates in these different figures? The same goes for Figure S4A and Video S6B, which again are of the same biofilm.

      Figure 2C would be much easier to read if the curves were colored in order of their size; the same is true for Figure 4A and irradiance.

      The complementation data in Figure S3D should be moved to the main text figure 3 alongside the data about the corresponding knockout to make it easier to compare the curves.

      Fig.ure S3E: Is the Y-axis in this graph mislabeled? It is labeled as ThT fluorescence, but it seems that it is reporting fluorescence from the calcium indicator?

      Video S6B is very confusing - why does the video play first forwards and then backwards? Unless I am looking very carefully at the time stamps it is easy to misinterpret this as a rise in the intensity at the end of the experiment. Without a video legend, it's hard to understand this, but I think it would be much more straightforward to interpret if it only played forward. (Also, why is this video labeled 6B when there is no video 6A?)

    1. Author Response

      The following is the authors’ response to the original reviews.

      eLife assessment

      This valuable study reports on the potential of neural networks to emulate simulations of human ventricular cardiomyocyte action potentials for various ion channel parameters with the advantage of saving simulation time in certain conditions. The evidence supporting the claims of the authors is solid, although the inclusion of open analysis of drop-off accuracy and validation of the neural network emulators against experimental data would have strengthened the study. The work will be of interest to scientists working in cardiac simulation and quantitative pharmacology.

      Thank you for the kind assessment. It is important for us to point out that, while limited, experimental validation was performed in this study and is thoroughly described in the work.

      Reviewer 1 - Comments

      This manuscript describes a method to solve the inverse problem of finding the initial cardiac activations to produce a desired ECG. This is an important question. The techniques presented are novel and clearly demonstrate that they work in the given situation. The paper is well-organized and logical.

      Strengths:

      This is a well-designed study, which explores an area that many in the cardiac simulation community will be interested in. The article is well written and I particularly commend the authors on transparency of methods description, code sharing, etc. - it feels rather exemplary in this regard and I only wish more authors of cardiac simulation studies took such an approach. The training speed of the network is encouraging and the technique is accessible to anyone with a reasonably strong GPU, not needing specialized equipment.

      Weaknesses:

      Below are several points that I consider to be weaknesses and/or uncertainties of the work:

      C I-(a) I am not convinced by the authors’ premise that there is a great need for further acceleration of cellular cardiac simulations - it is easy to simulate tens of thousands of cells per day on a workstation computer, using simulation conditions similar to those of the authors. I do not really see an unsolved task in the field that would require further speedup of single-cell simulations. At the same time, simulations offer multiple advantages, such as the possibility to dissect mechanisms of the model behaviour, and the capability to test its behaviour in a wide array of protocols - whereas a NN is trained for a single purpose/protocol, and does not enable a deep investigation of mechanisms. Therefore, I am not sure the cost/benefit ratio is that strong for single-cell emulation currently.

      An area that is definitely in need of acceleration is simulations of whole ventricles or hearts, but it is not clear how much potential for speedup the presented technology would bring there. I can imagine interesting applications of rapid emulation in such a setting, some of which could be hybrid in nature (e.g. using simulation for the region around the wavefront of propagating electrical waves, while emulating the rest of the tissue, which is behaving more regularly/predictable, and is likely to be emulated well), but this is definitely beyond of the scope of this article.

      Thank you for this point of view. Simulating a population of few thousand cells is completely feasible on single desktop machines and for fixed, known parameters, emulation may not fill ones need. Yet we still foresee a great untapped potential for rapid evaluations of ionic models, such as for the gradient-based inverse problem, presented in the paper. Such inverse optimization requires several thousand evaluations per cell and thus finding maximum conductances for the presented experimental data set (13 cell pairs control/drug → 26 APs) purely through simulations would require roughly a day of simulation time even in a very conservative estimation (3.5 seconds per simulation, 1000 simulations per optimization). Additionally, the emulator provides local sensitivity information between the AP and maximum conductances in the form of the gradient, which enables a whole new array of efficient optimization algorithms [Beck, 2017]. To further emphasize these points, we added the number of emulations and runtime of each conducted experiment in the specific section and a paragraph in the discussion that addresses this point:

      "Cardiomyocyte EP models are already very quick to evaluate in the scale of seconds (see Section 2.3.1), but the achieved runtime of emulations allows to solve time consuming simulation protocols markedly more efficient. One such scenario is the presented inverse maximum conductance estimation problem (see Section 3.1.2 and Section 3.1.3), where for estimating maximum conductances of a single AP, we need to emulate the steady state AP at least several hundred times as part of an optimization procedure. Further applications include the probabilistic use of cardiomyocyte EP models with uncertainty quantification [Chang et al., 2017, Johnstone et al., 2016] where thousands of samples of parameters are potentially necessary to compute a distribution of the steady-state properties of subsequent APs, and the creation of cell populations [Muszkiewicz et al., 2016, Gemmell et al., 2016, Britton et al., 2013]." (Section 4.2)

      We believe that rapid emulations are valuable for several use-cases, where thousands of evaluations are necessary. These include the shown inverse problem, but similarly arise in uncertainty quantification, or cardiomyocyte population creation. Similarly, new use-cases may arise as such efficient tools become available. Additionally, we provided the number of evaluations along with the runtimes for each of the conducted experiments, showing how essential these speedups are to realize these experiments in reasonable timeframes. Utilizing these emulations in organ-level electrophysiological models is a possibility, but the potential problems in such scenarios are much more varied and depend on a number of factors, making it hard to pin-point the achievable speed-up using ionic emulations.

      C I-(b) The authors run a cell simulation for 1000 beats, training the NN emulator to mimic the last beat. It is reported that the simulation of a single cell takes 293 seconds, while emulation takes only milliseconds, implying a massive speedup. However, I consider the claimed speedup achieved by emulation to be highly context-dependent, and somewhat too flattering to the presented method of emulation. Two specific points below:

      First, it appears that a not overly efficient (fixed-step) numerical solver scheme is used for the simulation. On my (comparable, also a Threadripper) CPU, using the same model (”ToR-ORd-dyncl”), but a variable step solver ode15s in Matlab, a simulation of a cell for 1000 beats takes ca. 50 seconds, rather than 293 of the authors. This can be further sped up by parallelization when more cells than available cores are simulated: on 32 cores, this translates into ca. 2 seconds amortized time per cell simulation (I suspect that the NN-based approach cannot be parallelized in a similar way?). By amortization, I mean that if 32 models can be simulated at once, a simulation of X cells will not take X50 seconds, but (X/32)50. (with only minor overhead, as this task scales well across cores).

      Second, and this is perhaps more important - the reported speed-up critically depends on the number of beats in the simulation - if I am reading the article correctly, the runtime compares a simulation of 1000 beats versus the emulation of a single beat. If I run a simulation of a single beat across multiple simulated cells (on a 32-core machine), the amortized runtime is around 20 ms per cell, which is only marginally slower than the NN emulation. On the other hand, if the model was simulated for aeons, comparing this to a fixed runtime of the NN, one can get an arbitrarily high speedup.

      Therefore, I’d probably emphasize the concrete speedup less in an abstract and I’d provide some background on the speedup calculation such as above, so that the readers understand the context-dependence. That said, I do think that a simulation for anywhere between 250 and 1000 beats is among the most reasonable points of comparison (long enough for reasonable stability, but not too long to beat an already stable horse; pun with stables was actually completely unintended, but here it is...). I.e., the speedup observed is still valuable and valid, albeit in (I believe) a somewhat limited sense.

      We agree that the speedup comparison only focused on a very specific case and needs to be more thoroughly discussed and benchmarked. One of the main strengths of the emulator is to cut the time of prepacing to steady state, which is known to be a potential bottleneck for the speed of the single-cell simulations. The time it takes to reach the steady state in the simulator is heavily dependant on the actual maximum conductance configuration and the speed-up is thus heavily reliant on a per-case basis. The differences in architecture of the simulator and emulator further makes direct comparisons very difficult. In the revised version we now go into more detail regarding the runtime calculations and also compare it to an adaptive time stepping simulation (Myokit [Clerx et al., 2016]) in a new subsection:

      "The simulation of a single AP (see Section 2.1) sampled at a resolution of 20kHz took 293s on one core of a AMD Ryzen Threadripper 2990WX (clock rate: 3.0GHz) in CARPentry. Adaptive timestep solver of variable order, such as implemented in Myokit [Clerx et al., 2016], can significantly lower the simulation time (30s for our setup) by using small step sizes close to the depolarization (phase 0) and increasing the time step in all other phases. The emulation of a steady state AP sampled at a resolution of 20kHz for t ∈ [−10, 1000]ms took 18.7ms on a AMD Ryzen 7 3800X (clock rate: 3.9GHz) and 1.2ms on a Nvidia A100 (Nvidia Corporation, USA), including synchronization and data copy overhead between CPU and GPU.

      "The amount of required beats to reach the steady state of the cell in the simulator has a major impact on the runtime and is not known a-priori. On the other hand, both simulator and emulator runtime linearly depends on the time resolution, but since the output of the emulator is learned, the time resolution can be chosen at arbitrarily without affecting the AP at the sampled times. This makes direct performance comparisons between the two methodologies difficult. To still be able to quantify the speed-up, we ran Myokit using 100 beats to reach steady state, taking 3.2s of simulation time. In this scenario, we witnessed a speed-up of 171 and 2 · 103 of our emulator on CPU and GPU respectively (again including synchronization and data copy overhead between CPU and GPU in the latter case). Note that both methods are similarly expected to have a linear parallelization speedup across multiple cells.

      For the inverse problem, we parallelized the problem for multiple cells and keep the problem on the GPU to minimize the overhead, achieving emulations (including backpropagation) that run in 120µs per AP at an average temporal resolution of 2kHz. We consider this the peak performance which will be necessary for the inverse problem in Section 3.1.2." (Section 2.3.1)

      Note that the mentioned parallelization across multiple machines/hardware applies equally to the emulator and simulator (linear speed-up), though the utilization for single cells is most likely different (single vs. multi-cell parallelization).

      C I-(c) It appears that the accuracy of emulation drops off relatively sharply with increasing real-world applicability/relevance of the tasks it is applied to. That said, the authors are to be commended on declaring this transparently, rather than withholding such analyses. I particularly enjoyed the discussion of the not-always amazing results of the inverse problem on the experimental data. The point on low parameter identifiability is an important one and serves as a warning against overconfidence in our ability to infer cellular parameters from action potentials alone. On the other hand, I’m not that sure the difference between small tissue preps and single cells which authors propose as another source of the discrepancy will be that vast beyond the AP peak potential (probably much of the tissue prep is affected by the pacing electrode?), but that is a subjective view only. The influence of coupling could be checked if the simulated data were generated from 2D tissue samples/fibres, e.g. using the Myokit software.

      Given the points above (particularly the uncertain need for further speedup compared to running single-cell simulations), I am not sure that the technology generated will be that broadly adopted in the near future.

      However, this does not make the study uninteresting in the slightest - on the contrary, it explores something that many of us are thinking about, and it is likely to stimulate further development in the direction of computationally efficient emulation of relatively complex simulations.

      We agree that the parameter identifiability is an important point of discussion. While the provided experimental data gave us great insights already, we still believe that given the differences in the setup, we can not draw conclusions about the source of inaccuracies with absolute certainty. The suggested experiment to test the influence of coupling is of interest for future works and has been integrated into the discussion. Further details are given in the response to the recommendation R III- (t)

      Reviewer 2 - Comments

      Summary:

      This study provided a neural network emulator of the human ventricular cardiomyocyte action potential. The inputs are the corresponding maximum conductances and the output is the action potential (AP). It used the forward and inverse problems to evaluate the model. The forward problem was solved for synthetic data, while the inverse problem was solved for both synthetic and experimental data. The NN emulator tool enables the acceleration of simulations, maintains high accuracy in modeling APs, effectively handles experimental data, and enhances the overall efficiency of pharmacological studies. This, in turn, has the potential to advance drug development and safety assessment in the field of cardiac electrophysiology.

      Strengths:

      1) Low computational cost: The NN emulator demonstrated a massive speed-up of more than 10,000 times compared to the simulator. This substantial increase in computational speed has the potential to expedite research and drug development processes

      2) High accuracy in the forward problem: The NN emulator exhibited high accuracy in solving the forward problem when tested with synthetic data. It accurately predicted normal APs and, to a large extent, abnormal APs with early afterdepolarizations (EADs). High accuracy is a notable advantage over existing emulation methods, as it ensures reliable modeling and prediction of AP behavior

      C II-(a) Input space constraints: The emulator relies on maximum conductances as inputs, which explain a significant portion of the AP variability between cardiomyocytes. Expanding the input space to include channel kinetics parameters might be challenging when solving the inverse problem with only AP data available.

      Thank you for this comment. We consider this limitation a major drawback, as discussed in Section 4.3. Identifiability is already an issue when only considering the most important maximum conductances. Further extending the problem to include kinetics will most likely only increase the difficulty of the inverse problem. For the forward problem though, it might be of interest to people studying ionic models to further analyze the effects of channel kinetics.

      C II-(b) Simplified drug-target interaction: In reality, drug interactions can be time-, voltage-, and channel statedependent, requiring more complex models with multiple parameters compared to the oversimplified model that represents the drug-target interactions by scaling the maximum conductance at control. The complex model could also pose challenges when solving the inverse problem using only AP data.

      Thank you pointing out this limitation. We slightly adapted Section 4.3 to further highlight some of these limitations. Note however that the experimental drugs used have been shown to be influenced by this drug interaction in varying degrees [Li et al., 2017] (e.g. dofetilide vs. cisapride). However, the discrepancy in identifiability was mostly channel-based (0%-100%), whereas the variation in identifiability between drugs was much lower (39%-66%).

      C II-(c) Limited data variety: The inverse problem was solved using AP data obtained from a single stimulation protocol, potentially limiting the accuracy of parameter estimates. Including AP data from various stimulation protocols and incorporating pacing cycle length as an additional input could improve parameter identifiability and the accuracy of predictions.

      The proposed emulator architecture currently only considers the discussed maximum conductances as input and thus can only compensate when using different stimulation protocols. However, the architecture itself does not prohibit including any of these as parameters for future variants of the emulator. We potentially foresee future works extending on the architecture with modified datasets to include other parameters of importance, such as channel kinetics, stimulation protocols and pacing cycle lengths. These will however vary between the actual use-cases one is interested in.

      C II-(d) Larger inaccuracies in the inverse problem using experimental data: The reasons for this result are not quite clear. Hypotheses suggest that it may be attributed to the low parameter identifiability or the training data set were collected in small tissue preparation.

      The low parameter identifiability on some channels (e.g. GK1) poses a problem, for which we state multiple potential reasons. As of yet, no final conclusion can be drawn, warranting further research in this area.

      Reviewer 3 - Comments

      Summary:

      Grandits and colleagues were trying to develop a new tool to accelerate pharmacological studies by using neural networks to emulate the human ventricular cardiomyocyte action potential (AP). The AP is a complex electrical signal that governs the heartbeat, and it is important to accurately model the effects of drugs on the AP to assess their safety and efficacy. Traditional biophysical simulations of the AP are computationally expensive and time-consuming. The authors hypothesized that neural network emulators could be trained to predict the AP with high accuracy and that these emulators could also be used to quickly and accurately predict the effects of drugs on the AP.

      Strengths:

      One of the study’s major strengths is that the authors use a large and high-quality dataset to train their neural network emulator. The dataset includes a wide range of APs, including normal and abnormal APs exhibiting EADs. This ensures that the emulator is robust and can be used to predict the AP for a variety of different conditions.

      Another major strength of the study is that the authors demonstrate that their neural network emulator can be used to accelerate pharmacological studies. For example, they use the emulator to predict the effects of a set of known arrhythmogenic drugs on the AP. The emulator is able to predict the effects of these drugs, even though it had not been trained on these drugs specifically.

      C III-(a) One weakness of the study is that it is important to validate neural network emulators against experimental data to ensure that they are accurate and reliable. The authors do this to some extent, but further validation would be beneficial. In particular for the inverse problem, where the estimation of pharmacological parameters was very challenging and led to particularly large inaccuracies.

      Thank you for this recommendation. Further experimental validation of the emulator in the context of the inverse problem would be definitely beneficial. Still, an important observation is that the identifiability varies greatly between channels. While the inverse problem is an essential reason for utilizing the emulator, it is also empirically validated for the pure forward problem and synthetic inverse problem, together with the (limited) experimental validation. The sources of problems arising in estimating the maximum conductances of the experimental tissue preparations are important to discuss in future works, as we now further emphasize in the discussion. See also the response to the recommendations R III-(t).

      Reviewer 1 - Recommendations

      R I-(a) Could further detail on the software used for the emulation be provided? E.g. based on section 2.2.2, it sounds like a CPU, as well as GPU-based emulation, is possible, which is neat.

      Indeed as suspected, the emulator can run on both CPUs and GPUs and features automatic parallelization (per-cell, but also multi-cell), which is enabled by the engineering feats of PyTorch [Paszke et al., 2019]. This is now outlined in a bit more detail in Sec. 2 and 5.

      "The trained emulator is provided as a Python package, heavily utilizing PyTorch [Paszke et al., 2019] for the neural network execution, allowing it to be executed on both CPUs and NVidia GPUs." (Section 5)

      R I-(b) I believe that a potential use of NN emulation could be also in helping save time on prepacing models to stability - using the NN for ”rough” prepacing (e.g. 1000 beats), and then running a simulation from that point for a smaller amount of time (e.g. 50 beats). One could monitor the stability of states, so if the prepacing was inaccurate, one could quickly tell that these models develop their state vector substantially, and they should be simulated for longer for full accuracy - but if the model was stable within the 50 simulated beats, it could be kept as it is. In this way, the speedup of the NN and accuracy and insightfulness of the simulation could be combined. However, as I mentioned in the public review, I’m not sure there is a great need for further speedup of single-cell simulations. Such a hybrid scheme as described above might be perhaps used to accelerate genetic algorithms used to develop new models, where it’s true that hundreds of thousands to millions of cells are eventually simulated, and a speedup there could be practical. However one would have to have a separate NN trained for each protocol in the fitness function that is to be accelerated, and this would have to be retrained for each explored model architecture. I’m not sure if the extra effort would be worth it - but maybe yes to some people.

      Thank you for this valuable suggestion. As pointed out in C I-(a), one goal of this study was to reduce the timeconsuming task of prepacing. Still, in its current form the emulator could not be utilized for prepacing simulators, as only the AP is computed by the emulator. For initializing a simulation at the N-th beat, one would additionally need all computed channel state variables. However, a simple adaptation of the emulator architecture would allow to also output the mentioned state variables.

      R I-(c) Re: ”Several emulator architectures were tried on the training and validation data sets and the final choice was hand-picked as a good trade-off between high accuracy and low computational cost” - is it that the emulator architecture was chosen early in the development, and the analyses presented in the paper were all done with one previously selected architecture? Or is it that the analyses were attempted with all considered architectures, and the well-performing one was chosen? In the latter case, this could flatter the performance artificially and a test set evaluation would be worth carrying out.

      We apologize for the unclear description of the architectural validation. The validation was in fact carried out with 20% of the training data (data set #1), which is however completely disjoint with the test set (#2, #3, #4, formerly data set #1 and #2) on which the evaluation was presented. To further clarify the four different data sets used in the study, we now dedicated an additional section to describing each set and where it was used (see also our response below R I-(d)), and summarize them in Table 1, which we also added at R II-(a). The cited statement was slightly reworked.

      "Several emulator architectures were tried on the training and validation data sets and the final choice was hand-picked as a good trade-off between high accuracy on the validation set (#1) and low computational runtime cost." (Section 2.2.2)

      R I-(d) When using synthetic data for the forward and inverse problem, with the various simulated drugs, is it that split of the data into training/validation test set was done by the drug simulated (i.e., putting 80 drugs and the underlying models in the training set, and 20 into test set)? Or were the data all mixed together, and 20% (including drugs in the test set) were used for validation? I’m slightly concerned by the potential of ”soft” data leaks between training/validation sets if the latter holds. Presumably, the real-world use case, especially for the inverse problem, will be to test drugs that were not seen in any form in the training process. I’m also not sure whether it’s okay to reuse cell models (sets of max conductances) between training and validation tests - wouldn’t it be better if these were also entirely distinct? Could you please comment on this?

      We completely agree with the main points of apprehension that training, validation and test sets all serve a distinct purpose and should not be arbitrarily mixed. However, this is only a result of the sub-optimal description of our datasets, which we heavily revised in Section 2.2.1 (Data, formerly 2.3.1). We now present the data using four distinct numbers: The initial training/validation data, now called data set #1 (formerly no number), is split 80%/20% into training and validation sets (for architectural choices) respectively. The presented evaluations in Section 2.3 (Evaluation) are purely performed on data set #2 (normal APs, formerly #1), #3 (EADs, formerly #2) and #4 (experimental).

      R I-(e) For the forward problem on EADs, I’m not sure if the 72% accuracy is that great (although I do agree that the traces in Fig 12-left also typically show substantial ICaL reactivation, but this definitely should be present, given the IKr and ICaL changes). I would suggest that you also consider the following design for the EAD investigation: include models with less severe upregulation of ICaL and downregulation of IKr, getting a population of models where a part manifests EADs and a part does not. Then you could run the emulator on the input data of this population and be able to quantify true, falsexpositive, negative detections. I think this is closer to a real-world use case where we have drug parameters and a cell population, and we want to quickly assess the arrhythmic risk, with some drugs being likely entirely nonrisky, some entirely risky, and some between (although I still am not convinced it’s that much of an issue to just simulate this in a couple of thousands of cells).

      Thank you for pointing out this alternative to address the EAD identification task. Even though the values chosen in Table 2 seem excessively large, we still only witnessed EADs in 171 of the 950 samples. Especially border cases, which are close to exhibiting EADs are hardest to estimate for the NN emulator. As suggested, we now include the study with the full 950 samples (non-EAD & EAD) and classify the emulator AP into one of the labels for each sample. The mentioned 72.5% now represent the sensitivity, whereas our accuracy in such a scenario becomes 90.8% (total ratio of correct classifications):

      "The data set #3 was used second and Appendix C shows all emulated APs, both containing the EAD and non-EAD cases. The emulation of all 950 APs took 0.76s on the GPU specified in Section 2.2.3 We show the emulation of all maximum conductances and the classification of the emulation. The comparison with the actual EAD classification (based on the criterion outlined in Appendix A) results in true-positive (EAD both in the simulation and emulation), false-negative (EAD in the simulation, but not in the emulation), false-positive (EAD in the emulation, but not in the simulation) and true-negative (no EAD both in the emulation and simulation). The emulations achieved 72.5% sensitivity (EAD cases correctly classified) and 94.9% specificity (non-EAD cases correctly classified), with an overall accuracy of 90.8% (total samples correctly classified). A substantial amount of wrongly classified APs showcase a notable proximity to the threshold of manifesting EADs. Figure 7 illustrates the distribution of RMSEs in the EAD APs between emulated and ground truth drugged APs. The average RMSE over all EAD APs was 14.5mV with 37.1mV being the maximum. Largest mismatches were located in phase 3 of the AP, in particular in emulated APs that did not fully repolarize." (Section 3.1.1)

      R I-(f) Figure 1 - I think a large number of readers will understand the mathematical notation describing inputs/outputs; that said, there may be a substantial number of readers who may find that hard to read (e.g. lab-based researchers, or simulation-based researchers not familiar with machine learning). At the same time, this is a very important part of the paper to explain what is done where, so I wonder whether using words to describe the inputs/outputs would not be more practical and easier to understand (e.g. ”drug-based conductance scaling factor” instead of ”s” ?). It’s just an idea - it needs to be tried to see if it wouldn’t make the figure too cluttered.

      We agree that the mathematical notation may be confusing to some readers. As a compromise between using verbose wording and mathematical notation, we introduced a legend in the lower right corner of the figure that shortly describes the notation in order to help with interpreting the figure.

      R I-(g) ”APs with a transmembrane potential difference of more than 10% of the amplitude between t = 0 and 1000 ms were excluded” - I’m not sure I understand what exactly you mean here - could you clarify?

      With this criterion, we try to discard data that is far away from fully repolarizing within the given time frame, which applies to 116 APs in data set #1 and 50 APs in data set #3. We added a small side note into the text:

      "APs with a transmembrane potential difference of more than 10% of the amplitude between t = 0 and 1000ms (indicative of an AP that is far away from full repolarization) were excluded." (Section 2.2.1)

      R I-(h) Speculation (for the future) - it looks like a tool like this could be equally well used to predict current traces, as well as action potentials. I wonder, would there be a likely benefit in feeding back the currents-traces predictions on the input of the AP predictor to provide additional information? Then again, this might be already encoded within the network - not sure.

      Although not possible with the chosen architecture (see also R I-(b)), it is worth thinking about an implementation in future works and to study differences to the current emulator.

      Entirely minor points:

      R I-(i) ”principle component analysis” → principal component analysis

      Fixed

      R I-(j) The paper will be probably typeset by elife anyway, but the figures are often quite far from their sections, with Results figures even overflowing into Discussion. This can be often fixed by using the !htb parameters (\begin{figure}[!htb]), or potentially by using ”\usepackage[section]{placeins}” and then ”\FloatBarrier” at the start and end of each section (or subsection) - this prevents floating objects from passing such barriers.

      Thank you for these helpful suggestions. We tried reducing the spacing between the figures and their references in the text, hopefully improving the reader’s experience.

      R I-(k) Alternans seems to be defined in Appendix A (as well as repo-/depolarization abnormalities), but is not really investigated. Or are you defining these just for the purpose of explaining what sorts of data were also included in the data?

      We defined alternans since this was an exclusion criterion for generating simulation data.

      Reviewer 2 - Recommendations

      R II-(a) Justification for methods selection: Explain the rationale behind important choices, such as the selection of specific parameters and algorithms.

      Thank you for this recommendation, we tried to increase transparency of our choices by introducing a separate data section that summarizes all data sets and their use cases in Section 2.2.1 and also collect many of the explanations there. Additionally we added an overview table (Table 1) of the utilized data.

      Author response table 1.

      Table 1: Summary of the data used in this study, along with their usage and the number of valid samples. Note that each AP is counted individually, also in cases of control/drug pairs.

      R II-(b) Interpretation of the evaluation results: After presenting the evaluation results, consider interpretations or insights into what the results mean for the performance of the emulator. Explain whether the emulator achieved the desired accuracy or compare it with other existing methods. In the revised version, we tried to further expand the discussion on possible applications of our emulator (Section 4.2). See also our response to C I-(a). To the best of our knowledge, there are currently no out-of-the-box methods available for directly comparing all experiments we considered in our work.

      Reviewer 3 - Recommendations

      R III-(a) In the introduction (Page 3) and then also in the 2.1 paragraph authors speak about the ”limit cycle”: Do you mean steady state conditions? In that case, it is more common to use steady state.

      When speaking about the limit cycle, we refer to what is also sometimes called the steady state, depending on the field of research and/or personal preference. We now mention both terms at the first occurence, but stick with the limit cycle terminology which can also be found in other works, see e.g. [Endresen and Skarland, 2000].

      R III-(b) On page 3, while comparing NN with GP emulators, I still don’t understand the key reason why NN can solve the discontinuous functions with more precision than GP.

      The potential problems in modeling sharp continuities using GPs is further explained in the referenced work [Ghosh et al., 2018] and further references therein:

      "Statistical emulators such as Gaussian processes are frequently used to reduce the computational cost of uncertainty quantification, but discontinuities render a standard Gaussian process emulation approach unsuitable as these emulators assume a smooth and continuous response to changes in parameter values [...] Applying GPs to model discontinuous functions is largely an open problem. Although many advances (see the discussion about non-stationarity in [Shahriari et al., 2016] and the references in there) have been made towards solving this problem, a common solution has not yet emerged. In the recent GP literature there are two specific streams of work that have been proposed for modelling non-stationary response surfaces including those with discontinuities. The first approach is based on designing nonstationary processes [Snoek et al., 2014] whereas the other approach attempts to divide the input space into separate regions and build separate GP models for each of the segmented regions. [...]"([Ghosh et al., 2018])

      We integrated a short segment of this explanation into Section 1.

      R III-(c) Why do authors prefer to use CARPentry and not directly openCARP? The use of CARPentry is purely a practical choice since the simulation pipeline was already set up. As we now point out however in Sec. 2.1 (Simulator), simulations can also be performed using any openly available ionic simulation tool, such as Myokit [Clerx et al., 2016], OpenCOR [Garny and Hunter, 2015] and openCARP [Plank et al., 2021]. We emphasized this in the text.

      "Note, that the simulations can also be performed using open-source software such as Myokit [Clerx et al., 2016], OpenCOR [Garny and Hunter, 2015] and openCARP [Plank et al., 2021]." (Section 2.1)

      R III-(d) In paragraph 2.1:

      (a) In this sentence: ”Various solver and sampling time steps were applied to generate APs and the biomarkers used in this study (see Appendix A)” this reviewer suggests putting the Appendix reference near “biomarkers”. In addition, a figure that shows the test of various solver vs. sampling time steps could be interesting and can be added to the Appendix as well.

      (b) Why did the authors set the relative difference below 5% for all biomarkers? Please give a reference to that choice. Instead, why choose 2% for the time step?

      1) We adjusted the reference to be closer to “biomarkers”. While we agree that further details on the influence of the sampling step would be of interest to some of the readers, we feel that it is far beyond the scope of this paper.

      2) There is no specific reference we can provide for the choice. Our goal was to reach 5% relative difference, which we surpassed by the chosen time steps of 0.01 ms (solver) and 0.05 ms (sampling), leading to only 2% difference. We rephrased the sentence in question to make this clear.

      "We considered the time steps with only 2% relative difference for all AP biomarkers (solver: 0.01ms; sampling: 0.05ms) to offer a sufficiently good approximation." (Section 2.1)

      R III-(e) In the caption of Figure 1 authors should include the reference for AP experimental data (are they from Orvos et al. 2019 as reported in the Experimental Data section?)

      We added the missing reference as requested. As correctly assumed, they are from [Orvos et al., 2019].

      R III-(f) Why do authors not use experimental data in the emulator development/training?

      For the supervised training of our NN emulator, we need to provide the maximum conductances of our chosen channels for each AP. While it would be beneficial to also include experimental data in the training to diversify the training data, the exact maximum conductances in our the considered retrospective experiments are not known. In the case such data would be available with low measurement uncertainty, it would be possible to include.

      R III-(g) What is TP used in the Appendix B? I could not find the acronymous explanation.

      We are sorry for the oversight, TP refers to the time-to-peak and is now described in Appendix A.

      R III-(h) Are there any reasons for only using ST and no S1? Maybe are the same?

      The global sensitivity analysis is further outlined in Appendix B, also showing S1 (first-order effects) and ST (variance of all interactions) together (Figure 11) [Herman and Usher, 2017] and their differences (e.g. in TP) Since S1 only captures first-order effects, it may fail to capture higher-order interactions between the maximum conductances, thus we favored ST.

      R III-(i) In Training Section Page 8. It is not clear why it is necessary to resample data. Can you motivate?

      The resampling part is motivated by exactly capturing the swift depolarization dynamics, whereas the output from CARPentry is uniformly sampled. This is now further highlighted in the text.

      "Then, the data were non-uniformly resampled from the original uniformly simulated APs, to emphasize the depolarization slope with a high accuracy while lowering the number of repolarization samples. For this purpose, we resamled the APs [...]" (Section 2.2.1)

      R III-(j) For the training of the neuronal network, the authors used the ADAM algorithm: have you tested any other algorithm?

      For training neural networks, ADAM has become the current de-facto standard and is certainly a robust choice for training our emulator. While there may exist slightly faster, or better-suited training algorithms, we witnessed (qualitative) convergence in the training (Equation (2)). We thus strongly believe that the training algorithm is not a limiting factor in our study.

      R III-(k) What is the amount of the drugs tested? Is the same dose reported in the description of the second data set or the values are only referring to experimental data? Moreover, it is not clear if in the description of experimental data, the authors are referring to newly acquired data (since they described in detail the protocol) or if they are obtained from Orvos et al. 2019 work.

      In all scenarios, we tested 5 different drugs (cisapride, dofetilide, sotalol, terfenadine, verapamil). We revised our previous presentation of the data available, and now try to give a concise overview over the utilized data (Section 2.2.1 and table 1) and drug comparison with the CiPA distributions (Table 5, former 4). Note that in the latter case, the available expected channel scaling factors by the CiPA distributions vary, but are now clearly shown in Table 5.

      R III-(l) In Figure 4, I will avoid the use of “control” in the legend since it is commonly associated with basal conditions and not with the drug administration.

      The terminology “control” in this context is in line with works from the CiPA initiative, e.g. [Li et al., 2017] and refers to the state of cell conditions before the drug wash-in. We added a minor note the first time we use the term control in the introduction to emphasize that we refer to the state of the cell before administering any drugs

      "To compute the drugged AP for given pharmacological parameters is a forward problem, while the corresponding inverse problem is to find pharmacological parameters for given control (before drug administration) and drugged AP." (Section 1)

      R III-(m) In Table 1 when you referred to Britton et al. 2017 work, I suggest adding also 10.1371/journal.pcbi.1002061.

      We added the suggested article as a reference.

      R III-(n) For the minimization problem, only data set #1 has been used. Have you tested data set #2?

      In the current scenario, we only tested the inverse problem for data set #2 (former #1). The main purpose for data set #3 (former #2), was to test the possibility to emulate EAD APs. Given the overall lower performance in comparison to data set #2 (former #1), we also expect deteriorated results in comparison to the existing inverse synthetic problem.

      R III-(o) In Figure 6 you should have the same x-axis (we could not see any points in the large time scale for many biomarkers). Why dVmMax is not uniformed distributed compared to the others? Can you comment on that?

      As suggested, we re-adjusted the x-range to show the center of distributions. Additionally, we denoted in each subplot the number of outliers which lie outside of the shown range. The error distribution on dVmMax exhibits a slightly off-center, left-tailed normal distribution, which we now describe a bit more in the revised text:

      "While the mismatches in phase 3 were simply a result of imperfect emulation, the mismatches in phase 0 were a result of the difficulty in matching the depolarization time exactly. [...] Likewise, the difficulty in exactly matching the depolarization time leads to elevated errors and more outliers in the biomarkers influenced by the depolarization phase (TP and dVmMax)," (Section 3.1.1)

      R III-(p) Page 14. Can the authors better clarify ”the average RMSE over all APs 13.6mV”: is it the mean for all histograms in Figure 7? (In Figure 5 is more evident the average RMSE).

      The average RMSE uses the same definition for Figures 5 and 7: It is the average over all the RMSEs for each pair of traces (simulated/emulated), though the amount of samples is much lower for the EAD data set and not normal distributed.

      R III-(q) In Table 4, the information on which drugs are considered should be added. For each channel, we added the names of the drugs for which respective data from the CiPA initiative were available.

      R III-(r) Pag. 18, second paragraph, there is a repetition of ”and”.

      Fixed

      R III-(s) The pair’s combination of scaling factors for simulating synthetic drugs reported in Table 2, can be associated with some effects of real drugs? In this case, I suggest including the information or justifying the choice.

      The scaling factors in Table 2 are used to create data set #3 (former #2), and is meant to provide several APs which expose EADs. This is described in more detail in the new data section, Section 2.2.1:

      "Data set #3: The motivation for creating data set #3 was to test the emulator on data of abnormal APs showing the repolarization abnormality EAD. This is considered a particularly relevant AP abnormality in pharmacological studies because of their role in the genesis of drug-induced ventricular arrhythmia’s [Weiss et al., 2010]. Drug data were created using ten synthetic drugs with the hERG channel and the Cav1.2 channel as targets. To this end, ten samples with pharmacological parameters for GKr and PCa (Table 2) were generated and the synthetic drugs were applied to the entire synthetic cardiomyocyte population by scaling GKr and PCa with the corresponding pharmacological parameter. Of the 1000 APs simulated, we discarded APs with a transmembrane potential difference of more than 10% of the amplitude between t = 0 and 1000ms (checked for the last AP), indicative of an AP that does not repolarize within 1000ms. This left us with 950 APs, 171 of which exhibit EAD (see Appendix C)." (Section 2.2.1)

      R III-(t) A general comment on the work is that the authors claim that their study highlights the potential of NN emulators as a powerful tool for increased efficiency in future quantitative systems pharmacology studies, but they wrote ”Larger inaccuracies were found in the inverse problem solutions on experimental data highlight inaccuracies in estimating the pharmacological parameters”: so, I was wondering how they can claim the robustness of NN use as a tool for more efficient computation in pharmacological studies.

      The discussed robustness directly refers to efficiently emulating steady-state/limit cycle APs from a set of maximum conductances (forward problem, Section 3.1.1). We extensively evaluated the algorithm and feel that given the low emulation RMSE of APs (< 1 mV), the statement is warranted. The inverse estimation, enabled through this rapid evaluation, performs well on synthetic data, but shows difficulties for experimental data. Note however that at this point there are multiple potential sources for these problems as highlighted in the Evaluation section (Section 4.1) and Table 5 (former 4) highlights the difference in accuracy of estimating per-channel maximum conductances, revealing a potentially large discrepancy. The emulator also offers future possibilities to incorporate additional informations in the forms of either priors, or more detailed measurements (e.g. calcium transients) and can be potentially improved to a point where also the inverse problem can be satisfactorily solved in experimental preparations, though further analysis will be required.

      References [Beck, 2017] Beck, A. (2017). First-order methods in optimization. SIAM.

      [Britton et al., 2013] Britton, O. J., Bueno-Orovio, A., Ammel, K. V., Lu, H. R., Towart, R., Gallacher, D. J., and Rodriguez, B. (2013). Experimentally calibrated population of models predicts and explains intersubject variability in cardiac cellular electrophysiology. Proceedings of the National Academy of Sciences, 110(23).

      [Chang et al., 2017] Chang, K. C., Dutta, S., Mirams, G. R., Beattie, K. A., Sheng, J., Tran, P. N., Wu, M., Wu, W. W., Colatsky, T., Strauss, D. G., and Li, Z. (2017). Uncertainty quantification reveals the importance of data variability and experimental design considerations for in silico proarrhythmia risk assessment. Frontiers in Physiology, 8.

      [Clerx et al., 2016] Clerx, M., Collins, P., de Lange, E., and Volders, P. G. A. (2016). Myokit: A simple interface to cardiac cellular electrophysiology. Progress in Biophysics and Molecular Biology, 120(1):100–114.

      [Endresen and Skarland, 2000] Endresen, L. and Skarland, N. (2000). Limit cycle oscillations in pacemaker cells. IEEE Transactions on Biomedical Engineering, 47(8):1134–1137.

      [Garny and Hunter, 2015] Garny, A. and Hunter, P. J. (2015). OpenCOR: a modular and interoperable approach to computational biology. Frontiers in Physiology, 6.

      [Gemmell et al., 2016] Gemmell, P., Burrage, K., Rodr´ıguez, B., and Quinn, T. A. (2016). Rabbit-specific computational modelling of ventricular cell electrophysiology: Using populations of models to explore variability in the response to ischemia. Progress in Biophysics and Molecular Biology, 121(2):169–184.

      [Ghosh et al., 2018] Ghosh, S., Gavaghan, D. J., and Mirams, G. R. (2018). Gaussian process emulation for discontinuous response surfaces with applications for cardiac electrophysiology models.

      [Herman and Usher, 2017] Herman, J. and Usher, W. (2017). SALib: An open-source python library for sensitivity analysis. J. Open Source Softw., 2(9):97.

      [Johnstone et al., 2016] Johnstone, R. H., Chang, E. T., Bardenet, R., de Boer, T. P., Gavaghan, D. J., Pathmanathan, P., Clayton, R. H., and Mirams, G. R. (2016). Uncertainty and variability in models of the cardiac action potential: Can we build trustworthy models? Journal of Molecular and Cellular Cardiology, 96:49–62.

      [Li et al., 2017] Li, Z., Dutta, S., Sheng, J., Tran, P. N., Wu, W., Chang, K., Mdluli, T., Strauss, D. G., and Colatsky, T. (2017). Improving the in silico assessment of proarrhythmia risk by combining hERG (human ether`a-go-go-related gene) channel–drug binding kinetics and multichannel pharmacology. Circulation: Arrhythmia and Electrophysiology, 10(2).

      [Muszkiewicz et al., 2016] Muszkiewicz, A., Britton, O. J., Gemmell, P., Passini, E., S´anchez, C., Zhou, X., Carusi, A., Quinn, T. A., Burrage, K., Bueno-Orovio, A., and Rodriguez, B. (2016). Variability in cardiac electrophysiology: Using experimentally-calibrated populations of models to move beyond the single virtual physiological human paradigm. Progress in Biophysics and Molecular Biology, 120(1):115–127.

      [Orvos et al., 2019] Orvos, P., Kohajda, Z., Szlov´ak, J., Gazdag, P., Arp´adffy-Lovas, T., T´oth, D., Geramipour, A.,´ T´alosi, L., Jost, N., Varr´o, A., and Vir´ag, L. (2019). Evaluation of possible proarrhythmic potency: Comparison of the effect of dofetilide, cisapride, sotalol, terfenadine, and verapamil on hERG and native iKr currents and on cardiac action potential. Toxicological Sciences, 168(2):365–380.

      [Paszke et al., 2019] Paszke, A., Gross, S., Massa, F., Lerer, A., Bradbury, J., Chanan, G., Killeen, T., Lin, Z., Gimelshein, N., Antiga, L., Desmaison, A., Kopf, A., Yang, E., DeVito, Z., Raison, M., Tejani, A., Chilamkurthy, S., Steiner, B., Fang, L., Bai, J., and Chintala, S. (2019). PyTorch: An Imperative Style, High-Performance Deep Learning Library. In Advances in Neural Information Processing Systems, volume 32. Curran Associates, Inc.

      [Plank et al., 2021] Plank, G., Loewe, A., Neic, A., Augustin, C., Huang, Y.-L., Gsell, M. A., Karabelas, E., Nothstein, M., Prassl, A. J., S´anchez, J., Seemann, G., and Vigmond, E. J. (2021). The openCARP simulation environment for cardiac electrophysiology. Computer Methods and Programs in Biomedicine, 208:106223.

      [Shahriari et al., 2016] Shahriari, B., Swersky, K., Wang, Z., Adams, R. P., and de Freitas, N. (2016). Taking the Human Out of the Loop: A Review of Bayesian Optimization. Proceedings of the IEEE, 104(1):148–175. Conference Name: Proceedings of the IEEE.

      [Snoek et al., 2014] Snoek, J., Swersky, K., Zemel, R., and Adams, R. (2014). Input Warping for Bayesian Optimization of Non-Stationary Functions. In Proceedings of the 31st International Conference on Machine Learning, pages 1674–1682. PMLR. ISSN: 1938-7228.

      [Weiss et al., 2010] Weiss, J. N., Garfinkel, A., Karagueuzian, H. S., Chen, P.-S., and Qu, Z. (2010). Early afterdepolarizations and cardiac arrhythmias. Heart Rhythm, 7(12):1891–1899.

    1. Author Response

      The following is the authors’ response to the original reviews.

      Public Reviews:

      Reviewer #1 (Public Review):

      Summary

      This article by Zhai et al, investigates sterol transport in bacteria. Synthesis of sterols is rare in bacteria but occurs in some, such as M capsulatus where the sterols are found primarily in the outer membrane. In a previous paper the authors discovered an operon consisting of five genes, with two of these genes encoding demethylases involved in sterol demethylation. In this manuscript, the authors set out to investigate the functions of the other three genes in the operon. Interestingly, through a bioinformatic analysis, they show that they are an inner membrane transporter of the RND family, a periplasmic binding protein, and an outer membrane-associated protein, all potentially involved with lipid transport, so providing a means of transporting the lipids to the outer membrane. These proteins are then extensively investigated through lipid pulldowns, binding analysis on all three, and X-ray crystallography and docking of the latter two.

      Strengths

      The lipid pulldowns and associated MST binding analysis are convincing, clearly showing that sterols are able to bind to these proteins. The structures of BstB and BstC are high resolution with excellent maps that allow docking studies to be carried out. These structures are distinct from sterol-binding proteins in eukaryotes.

      We thank the reviewer for their favorable impression of this work.

      Weaknesses

      While the docking and molecular dynamics studies are consistent with the binding of sterols to BstB and BstC, this is not backed up particularly well. The MST results of mutants in the binding pocket of BstB have relatively little effect, and while I agree with the authors this may be because of the extensive hydrophobic interactions that the ligand makes with the protein, it is difficult to make any firm conclusions about binding.

      We agree with the reviewer that at this point, there is no experimental evidence to define the sterol binding site in BstB. While in the manuscript we allude to the extensive hydrophobic interactions as being especially stabilizing and difficult to eliminate with one or two mutations, we are now also aware that hydrogen-bonding interactions with the polar head of the sterols are quite important (see data on BstC, where disruption of that interaction significantly reduces the equilibrium affinity for sterols). Our MD simulations show that at least 3 protein amino acids can participate in H-bonding with the sterols. Moreover, recent work from our lab show that even ligand site waters can extend an H-bonding network around the polar head of the lipid (Zhai et al., ChemBioChem 2023, 24, e202300156), thereby enabling H-bonding with amino acids that are further away from the ligand site. It is therefore difficult to predict which mutations will sufficiently destabilize the binding. While this question is one we will tackle in future studies focused on obtaining high-resolution substrate-bound structures of BstB or homologs, the findings reported here are still relevant and timely, and we posit will spur the discovery of functional homologs, including some in organisms that are more tractable.

      The authors also discuss the possibility of a secondary binding site in BstB based on a slight cavity in domain B next to a flexible loop. This is not backed up in any way and seems unlikely.

      The reviewer is correct in that the evidence for this second binding site weak. While the crystallographic structure shows a highly hydrophobic region and the binding studies suggests cooperativity exists in the binding of the 4methylsterol substrate, the docking studies do not strongly support binding at that site. As such, we have clarified in the manuscript that a second hydrophobic cavity is observed, but that its role in ligand interaction remains unexplored.

      Reviewer #2 (Public Review):

      Summary:

      In eukaryotes, sterols are crucial for signaling and regulating membrane fluidity, however, the mechanism governing cholesterol production and transport across the cell membrane in bacteria remains enigmatic. The manuscript by Zhai et al. sheds light on this topic by uncovering three potential cholesterol transport proteins. Through comprehensive bioinformatics analysis, the authors identified three genes bstA, bstB, and bstC encoding proteins which share homology with transporters, periplasmic binding proteins, and periplasmic components superfamily, respectively. Furthermore, the authors confirmed the specific interaction between these three proteins and C-4 methylated sterols and determined the structures of BstB and BstC. Combining these structural insights with molecular dynamics simulation, they postulated several plausible substrate binding sites within each protein.

      Strengths:

      The authors have identified 3 proteins that seem likely to be involved in sterol transport between the inner and outer membrane. The structures are of high quality, and the sterol binding experiments support a role for these proteins in sterol transport.

      We thank the reviewer for this positive view of our work.

      Weaknesses:

      While the author's model is very plausible, direct evidence for a role of BstABC in transport, or that the 3 proteins function together in a single pathway, is limited.

      The reviewer is correct that we were unable to demonstrate that the three proteins work together to transport 4methylsterols. This is not for lack of trying. We first attempted gene deletion studies, and as mentioned in the manuscript (with more details now provided in the experimental section), this appeared to be lethal. We then attempted in vitro exchange experiments, in which the proteins would be used to transfer sterols from sterol-loaded “heavy” liposomes to a sterol-free “light” liposomes – such exchange assays are frequently performed with eukaryotic sterol transporters (see Chung et al., Science 2015, https://doi.org/10.1126/science.aab1370). These assays were not successful because 1) sterols incorporated poorly into liposomes made with E. coli polar lipids and yielded leaky liposomes; 2) use of liposomes prepared with the TLE of M. capsulatus proved more stable, but no appreciable exchange was observed; we reasoned that this might be due to the absence of an energy source for BstA, the RND component for which we have expressed and purified only the soluble periplasmic domain. Given the technical difficulty of these in vitro transport experiments, we will continue to pursue in vivo demonstration of function as new homologs are identified.

      Reviewer #3 (Public Review):

      Summary:

      The work in this manuscript builds on prior efforts by this team to understand how sterols are biosynthesized and utilized in bacteria. The study reports a new function for three genes encoded near sterol biosynthesis enzymes, suggesting the resulting proteins function as a sterol transport system. Biochemical and structural characterization of the two soluble components of the pathway establishes that both proteins can bind sterols, with a preference for 4methylated derivatives. High-resolution x-ray structures of the apoproteins reveal hydrophobic cavities of the appropriate size to accommodate these substrates. Docking and molecular dynamics simulations confirm this observation and provide specific insights into residues involved in substrate binding.

      Strengths:

      The manuscript is comprehensive and well-written. The annotation of a new function in a set of proteins related to bacterial sterol usage is exciting and likely to enable further study of this phenomenon - which is currently not well understood. The work also has implications for improving our understanding of lipid usage in general among bacterial organisms.

      We thank the reviewer for this synopsis of our work.

      Weaknesses:

      The authors might consider moving some of the bioinformatics figures to the main text, given how much space is devoted to this topic in the results section.

      We have taken this advice and moved Figure S1 to the main manuscript.

      Reviewer #1 (Recommendations For The Authors):

      1. In the analysis of the MST data, the authors quote Hill coefficients. How reliable are these numbers? For BstB, for instance, it seems unlikely that more than one molecule would bind. Can the analysis be done without needing to include Hill coefficients?

      We used fits that did and did not invoke cooperativity – see below. We are certain that both BstA and BstB are better fit with cooperativity invoked.

      Author response image 1.

      1. In looking at the maps associated with the structures, which were included in the review package, I see that two citric acid molecules fit beautifully into the density where currently PEG has been modelled. This needs to be fixed and some comments may be appropriate in the manuscript.

      We thank the reviewer for calling our attention to this. Citric acid has now been added to the model, and we reason that these are present in the structure because citric acid was used in the crystallization condition. The revised model is now present in the PDB.

      1. It is not necessary to show the two molecules in the asymmetric unit in Figure 4 given that it is not a dimer. This doesn't add anything to the manuscript.

      We now show a single molecule of BstC in Figure 4 (now Figure 5).

      1. I wouldn't consider the loops shown in Figure S4 as disordered. They have slightly higher B-values but are not completely mobile.

      We did not refer to these loops as disordered. In the text, we say they “exhibit poor electron densities, suggesting conformational sampling of more than one state (Fig. S4A).”

      Reviewer #2 (Recommendations For The Authors):

      pg 7, "hinting at an astounding distinction": I might suggest a word other than astounding that conveys how statistically unlikely, unusual, etc. this result is.

      Thank you – we have removed “astounding”.

      pg 7, paragraph 2: Here the authors show that in the SSN analysis, BstB proteins cluster separately and suggest this implies a distinction in function. However, they also show that PhnD homologs do not cluster separately (distributed across multiple clusters), yet presumably have similar functions. I am not familiar with SSN, but it seems to me that the second statement about PhnD implies that the first statement about BstB might not be valid, i.e., if PhnD doesn't cluster based on function, on what basis can we conclude that BstB does? On what basis does clustering occur in the SSN analysis? Might it be driven by things other than function? This comment also concerns the final paragraph of this section.

      The reviewer is correct in that PhnD homologs occupy separate clusters of the SSN. Many of these homologs were crystallized with phosphate-like compounds, but it is possible that they have non-overlapping substrate scopes and are therefore functionally distinct. As for the basis of clustering, the SSN is fully sequence-based. What has been observed is that proteins with highly similar sequences can have similar functions – but this is not always true.

      pg 8, paragraph 1: The authors suggest that BstABC may be essential. This is probably not a critical claim and it might be simplest to just remove it, but if it is mentioned, the authors should probably explain what was attempted that failed, so a reader can assess the strength of the evidence supporting essentiality. For example, I don't see anything in the methods about genetic manipulations of M. capsulatus, so currently, this falls within the realm of "Data not shown".

      We have provided additional information about the experimental techniques used to do this. This statement was included so that it is understood that the reason for the experimental failure is unlikely to be technical in nature, as we have successfully deleted some sterol related genes while others remain intractable.

      Fig. 2A: It is unclear to me what is being plotted here, perhaps more experimental detail is required in the form of labels and/or legend. Is this a quantification of each sterol in each fraction separated by GC? There are essentially no methods provided for the GC-MS experiments. A reference is provided, but I think providing detailed methods for these specific experiments will provide a higher degree of scientific rigor. I am not sure what is standard for GCMS, but perhaps showing spectra in the supplement that establish the identity of the bound molecules as species I and II would be appropriate?

      Additional experimental details have been provided and the figure legend changed to be more clear. Moreover, we now clearly state that the chromatograms shown were used to identify lipids due to retention times for spectra that were previously published in Wei et al., 2016.

      pg 10-11, comparison with PhnD structure: Perhaps it is worth mentioning a 3rd possible explanation for the relative opening/closing of the cleft is simply crystal packing? I don't think it necessarily has to imply anything about a difference in function. Also, the focus seems to be on this pairwise comparison, but perhaps more insights could be gleaned from an analysis that included a wider range of homologs, especially if any are thought to bind hydrophobic substrates.

      This could be true, and we have included a statement to that effect. We are unaware of homologs shown to bind to large, hydrophobic molecules.

      I think that BstB is shown upside-down in sup movies relative to other figures. If it isn't changed, perhaps adding some labels would help orient the reader.

      We have rotated the movies to be more consistent with the figures.

      Fig. S7: No units are indicated for Kds (uM?).

      Thank you – this has been fixed.

      pg 11, paragraph 2. "adjacent to three residues: Glu118, Tyr120 and Asn192": The residue number used in the text doesn't seem to match the numbering in the PDB file. I think these residues correspond to Glu98, Tyr100, and Asn172 in the PDB file.

      We regret this error. The correct numbering for both structures is now present in the deposited PDB files (7T1M for BstB and 7T1S for BstC).

      pg 12, final paragraph: The authors present binding data for BstB variants with mutations in the putative sterol binding pocket identified in the structural and MD analyses. However, these mutants had no effect on binding. The authors rationalize this in terms of the size of the interface and hydrophobic nature (which indeed, may be correct and is very plausible), and it is worth noting that many of their mutations are to Ala and would largely preserve the hydrophobic nature of the cleft. However, these mutants raise questions about where sterols actually bind. No experimental evidence is presented that substrates bind in the cleft, it is only hypothesized based on structural homology, MD simulations, etc. These mutations formally provide evidence against the hypothesis being tested; I think that has to be discussed a bit more directly, alongside the caveats the authors already discuss about hydrophobicity, etc.

      This is a valid point by the reviewer, and it is one we have attempted to address with our statement in the manuscript and in our response to reviewer 1. We have modified the relevant text to more clearly state that there is as of yet no experimental evidence for the binding of sterols to the cavity identified via molecular docking.

      pg 13: Presumably this is not the full-length lipoprotein, but has been truncated/mutated in some way? Some statement of roughly what was purified/crystallized should be stated.

      The SI methods on protein purification states that the genes of BstB and BstC without their respective signal peptides were obtained.

      pg 13, last paragraph "TN1 exhibits hybrid hydrophobicity, with the sides horizontal to cavities being hydrophobic while the vertical sides are more hydrophilic". I don't really follow the horizontal vs vertical sides. Perhaps this could be described in a different way.

      Noted and changed to “TN1 is closer to the N-terminal face of the structure, while CA1 and CA2 are proximal to the C-terminal face and form two open hydrophobic pockets; TN1 exhibits a mixture of hydrophobic and hydrophilic amino acids (Fig. 4B and Fig. S9B, Table S4).”

      pg 15-16, "Comparison to eukaryotic sterol transporters": Perhaps this would be better suited for the discussion section? Could also be streamlined; it is mostly discussing and comparing eukaryotic sterol binding domains to each other, not to BstABC.

      Given that BstB and BstC are the first identified proteins (and putative transporters) for bacterial sterol engagement, we thought a careful description of the existing sterol transporters (which are all eukaryotic) was warranted.

      Reviewer #3 (Recommendations For The Authors):

      I have just two minor suggestions for the authors if they wish to comment on or address them.

      1. Do the three proteins (BstA/B/C) form any sort of complex? Perhaps this property was not assessed - but it seemed possible that the B and C components might constitute a shuttle for the membrane-bound transporter?

      This is an important observation – the unliganded version of these proteins show no appreciable affinity for each other. However, BstB (which would be expected to engage both with BstA and BstC) belongs to a family of proteins known to undergo significant conformational change upon substrate binding. It is possible that with substrate present, complexes are formed – we have yet to investigate this.

      1. In Figure S1, panel C - it appears that the label for the BstC cluster may have migrated away from the intended location. In this figure, it might also be useful to indicate in the caption the meaning of the red coloring of the nodes?

      The label is now fixed – thank you for drawing our attention to this.

    1. Author Response

      The following is the authors’ response to the original reviews.

      Reviewer #1 (Public Review):

      In this manuscript, the authors investigated the role of Elg1 in the regulation of telomere length. The main role of the Elg1/RLC complex is to unload the processivity factor PCNA, mainly after completion of synthesis of the Okazaki fragment in the lagging strand. They found that Elg1 physically interacts with the CST (Cdc13-Stn1- Ten1) and propose that Elg1 negatively regulates telomere length by mediating the interaction between Cdc13 and Stn1 in a pathway involving SUMOylation of both PCNA and Cdc13. Accumulation of SUMOylated PCNA upon deletion of ELG1 or overexpression of RAD30 leads to elongated telomeres. On the other hand, the interaction of Elg1 with Sten1 is SIM-dependent and occurs concurrently with telomere replication in late S phase. In contrast Elg1-Cdc13 interaction is mediated by PCNA-SUMO, is independent on the SIM of Elg1 but still dependent on Cdc13 SUMOylation. The authors present a model containing two main messages 1) PCNA- SUMO acts as a positive signal for telomerase activation 2) Elg1 promotes Cdc13/Stn1 interaction at the expense of Cdc13/Est1 interaction thus terminating telomerase action.

      The manuscript contains a large amount of data that make a major inroad on a new type of link between telomere replication and regulation of the telomerase. Nevertheless, the detailed choreography of the events as well as the role of PCNA- SUMO remain elusive and the data do not fully explain the role of the Stn1/Elg1 interaction. The data presented do not sufficiently support the claim that SUMO- PCNA is a positive signal for telomerase activation.

      We thank the reviewer for her/his review efforts and opinion. We have re-submitted a new version of the manuscript in which we clarify some of the criticisms presented. In a point-by-point letter we respond to all the specific queries.

      Reviewer #2 (Public Review):

      This paper purports to unveil a mechanism controlling telomere length through SUMO modifications controlling interactions between PCNA unloader Elg1 and the CST complex that functions at telomeres. This is an extremely interesting mechanism to understand, and this paper indeed reveals some interesting genetic results, leading to a compelling model, with potential impact on the field. The conclusions are largely supported by experiments examining protein-protein interactions at low resolution and ambiguous regarding directness of interactions like co-IP and yeast two-hybrid (Y2H) combined with genetics. However, some results appear contradictory and there's a lack of rigor in the experimental data needed to support claims. There is significant room for improvement and this work could certainly attain the quality needed to support the claims. The current version needs substantial revision and lacks the necessary experimental detail. Stronger support for the claims would add detail to help distinguish competing models.

      We thank the reviewer for her/his positive opinion. We have re-submitted a new version of the manuscript in which we clarify some of the criticisms presented by thereferees, and added all the missing experimental details. In a point-by-point letter we respond to all the specific queries.

      Reviewer #3 (Public Review):

      This paper reveals interesting physical connections between Elg1 and CST proteins that suggest a model where Elg1-mediated PCNA unloading is linked to regulation of telomere length extension via Stn1, Cdc13, and presumably Ten1 proteins. Some of these interactions appear to be modulated by sumolyation and connected with Elg1's PCNA unloading activity. The strength of the paper is in the observations of new interactions between CST, Elg1, and PCNA. These interactions should be of interest to a broad audience interested in telomeres and DNA replication.

      We thank the reviewer for her/his positive opinion. We have re-submitted a new version of the manuscript in which we clarify some of the criticisms presented. In a point-by-point letter we respond to all the specific queries.

      What is not well demonstrated from the paper is the functional significance of the interactions described. The model presented by the authors is one interpretation of the data shown, and proposes that the role of sumolyation is temporally regulate the Elg1, PCNA and CST interactions at telomeres. This model makes some assumptions that are not demonstrated by this work (such as Stn1 sumolyation, as noted) and are left for future testing. Alternative models that envision sumolyation as a key in promoting spatial localization could also be proposed based on the data here (as mentioned in the discussion), in addition to or instead of a role for sumolyation in enforcing a series of switches governing a tightly sequenced series of interactions and events at telomeres. Critically, the telomere length data from the paper indicates that the proposed model depicts interactions that are not necessary for telomerase activation or inhibition, as telomeres in pol30-RR strains are normal length and telomeres in elg1∆ strains are not nearly as elongated as in stn1 strains. One possibility mentioned in the paper is the PCNAS and Elg1 interactions are contributing to the negative regulation of telomerase under certain conditions that are not defined in this work. Could it also be possible that the role of these interactions is not primarily directed toward modulating telomerase activity? It will be of interest to learn more about how these interactions and regulation by Sumo function intersect with regulation of telomere extension.

      We present compelling evidence for a role of SUMOylated PCNA in telomere length regulation. Figure 1 shows that this modification is both necessary and sufficient to elongate the telomeres, indicating that PCNA SUMOylation plays a positive role in telomere elongation. The model we present is consistent with all our results. There are, of course, possible alternative models, but they usually fail to explain some of the results. We agree that the fact that pol30-RR presents normal-sized telomeres implies that SUMO-PCNA is not required for telomerase to solve the "end replication problem", but rather is needed for "sustained" activity of telomerase. Since elongated telomeres (by absence of Elg1 or by over-expression of SUMO-PCNA) was the phenotype monitored, this may require sustained telomerase activity. Similar results were seen in the past for Rnr1 (Maicher et al., 2017), and this mode depends on Mec1, rather than Tel1 (Harari and Kupiec, 2018). Telomere length regulation is complex, and we may not yet understand the whole picture. It appears that for normal “end replication problem” solution, very little telomerase activity may be needed, and spontaneous interactions at a low level may suffice. Future work may find the conditions at which telomerase switches from "end replication problem" to "sustained" activity. We have added further explanations on this subject to the Discussion section.

      We suspect, but could not prove, a role for Stn1 SUMOylation in the interactions. SUMOylation is usually transient, and notoriously hard to detect, and despite the fact that many telomeric proteins are SUMOylated, Stn1 SUMOylation could not be shown directly by us and others (Hang et al, 2011).

      Reviewer #1 (Recommendations For The Authors):

      Suggestions for improved or additional experiments, data or analyses.

      • My main concern is the claim that SUMOylated PCNA acts as a positive signal for telomerase activation. Yet the pol30-RR mutant has no impact on telomere length. The explanation of the authors is not entirely convincing.

      We are aware that the regulation of telomere length is complex, and we may not fully understand it yet. Just consider the fact that ~500 genes participate in determining the final telomere length of a yeast (Askree et al., 2004). Since mutation in EACH of these genes has a phenotype, the implication is that the joint action of 500 players determines the outcome (a dialogue of 500 participants). Having said this, we clearly show in figure 1 that mutations that prevent PCNA SUMOylation prevent telomere length elongation in cells lacking Elg1, and overexpressing SUMOylated PCNA is enough to elongate the telomeres. Thus, SUMOylation of PCNA does act as a positive signal for elongation.

      However, it appears that to fulfill the minimal requirement of dealing with the "end- replication problem", PCNA SUMOylation is not required, and only a "sustained activity" mode requires the S-PCNA signal (as we have also shown, surprisingly, for RNR1, Maicher et al. 2017). This sustained activity mode depends on Mec1, rather than Tel1 (Harari and Kupiec, 2018). Since elongated telomeres (by absence of Elg1 or by over-expression of SUMO-PCNA) was the phenotype monitored, this may require sustained telomerase activity. Telomere length regulation is complex, and we may not yet understand the whole picture. It appears that for normal “end replication problem” solution, very little telomerase activity may be needed, and spontaneous interactions at a low level may suffice (for example, unmodified PCNA may promote telomerase activity at a lower level than that of SUMO-PCNA. Future work may find the conditions at which telomerase switches from "end replication problem" to "sustained" activity.

      We have added further explanations on this subject to the Discussion section.

      • The model is entitled « Elg1 negatively regulates the telomere length by forming an interaction with the CST complex ». Nevertheless, expression of PCNA-RR completely reversed the long telomere phenotype of elg1∆ cells. Thus it appears that although the interaction between Stn1 and Cdc13 is reduced in the absence of Elg1, Elg1/Stn1 interaction is not instrumental in the formation of the CST complex and thus in the termination of telomerase activity. Does the elg1∆SIM mutant that does not interact with Stn1 impact telomere length?

      • In the model part (lane 318), it is argued that the complex Elg1-Stn1 unloads SUMOylated PCNA. Elg1-Stn1 interaction depends on the SIM of Elg1. This SIM is however not required for Elg1's function in genome-wide SUMO-PCNA unloading, is it required specifically at telomeres?

      The interactions between Elg1 and SUMOylated PCNA are carried out through both the SIM and the Threonines 386 and 387 (Shemesh et al, 2017). Consistently, the single elg1-SIM mutant has telomeres of normal length, and its effects on telomere length can only be seen when combined with mutations in the Threonines (elg1- TT386/7AA or elg1-TT386/7DD). Although the unloading of SUMOylated PCNA by Elg1 is important, the gene is not essential, and PCNA is either eventually unloaded by RFC, or spontaneously dis-assembles. This explains why the telomere length does not reach the same length in the absence of Elg1 as in the absence of, say, Stn1.

      • The model suggests that Elg1 promotes the interaction between Cdc13 and Stn1. This is based on the data presented in Figure 5 E and F. This is an important result. Because the experiment has been done on cells synchronized in S phase and the Elg1/Stn1 interaction occurs specifically at the end of S-phase, the FACS profile should be shown or a control provided to show that the two conditions are comparable.

      The FACS profile for this experiment is shown in Figure 5C.

      • Does the interaction between Cdc13 and Pol30 depend on the SUMOyaltion of POL30 ?

      Yes. We have added this as new Figure S2, and presented the results together with Figure 3 (Figure 3 is already too crowded).

      Others points :

      • Fig 1 : it should be mentioned in the Materials and Methods or in the figure legend how the average telomere lengths (horizontal bar) were calculated from the teloblot, as the position of the bar is not always intuitive

      We estimate telomere length by using TelQuant (Rubinstein et al., 2014). We have added this to the Methods section.

      -Fig 2 : Owing to the large span of telomere length in the stn1 mutants, the epistatic relationship between elg1∆ and stn1 mutants is poorly illustrated by the teloblot.

      We repeated this experiment several times, and stn1 mutants consistently gave a very spread telomere length. In ALL the blots, however, the double mutants elg1 stn1 showed a telomere length similar to that of the single stn1 mutant, and never longer.

      • It is mentioned that other mutants in the collection showed epistasis. Are any of these mutants related to telomere replication or the proposed model?

      Since we used the collection of non-essential mutants (so far), it was quite devoid of genes involved in DNA replication, which are mostly essential. An exception was siz1, which showed epistasis with elg1Δ.

      • The section entitled « Elg1's functional activity is essential for its interaction with Cdc13 » (lane 205) is difficult to follow. The hierarchy between the different mutants of Elg1 on their capacity to unload PCNA is not totally in agreement with the data published in Itzkovich et al 2023 and Shemesh et al. 2017. In particular it appears to me from these papers that elg1-WalkerA 238 (KK343/4AA) mutant did not show a defect in contrast to elg1-WalkerA 238(KK343/4DD).

      We are sorry for the typo in the results. We used the elg1-WalkerA (KK343/4DD) allele, which has a normal SIM but no activity. In a nutshell, we used mutants that either did or did not show unloading activity and/or SIM. The results clearly show that you need to unload PCNA in order for the N-ter of Elg1 to interact with Cdc13.

      • Are the synchronization done at 30{degree sign}C ?

      Yes. We have added the information to the Methods section.

      • ChIP experiments are not described in the Materials and Methods

      We apologize for this. They are now described.

      • In the figure 6, the PCNA rings are curiously placed at the beginning of the Okasaki fragments.

      We thank the referee for noticing, we have corrected the figure.

      Reviewer #2 (Recommendations For The Authors):

      This paper purports to unveil a mechanism controlling telomere length through SUMO modifications controlling interactions between PCNA unloader Elg1 and the CST complex that functions at telomeres. This is an extremely interesting mechanism to understand, and this paper indeed reveals some interesting genetic results, leading to a compelling model, with potential impact on the field. The conclusions are largely supported by experiments examining protein-protein interactions at low resolution and ambiguous regarding directness of interactions like co-IP and yeast two-hybrid (Y2H) combined with genetics. However, some results appear contradictory and there's a lack of rigor in the experimental data needed to support claims. There is significant room for improvement and this work could certainly attain the quality needed to support the claims. The current version needs substantial revision and lacks necessary experimental detail. Stronger support for the claims would add detail to help distinguish competing models.

      Specific comments:

      Insufficient technical detail: I could find no explanation of how overexpression was achieved. No description of how teloChIP is performed, either for the PCNA IP or how the sequence analysis is performed. Too limited details on growth like exact temperatures for the cell cycle time course.

      We have significantly expanded the Methods section to include all the technical information.

      Please do not bold and underline text for emphasis-EVER

      We have removed those from the text.

      Lines 130-132: they have not shown "accumulation of SUMOylated PCNA" anywhere; this is an inference.

      We have modified the text, it says: ”show that SUMOylated PCNA, and not unmodified or ubiquitinated PCNA, is both necessary and sufficient for telomere elongation in the presence or in the absence of Elg1.”

      Fig 2A Can authors show any other very long-telomere mutant like stn1 that does show enhancement in combination with elg1∆ to show feasibility of such phenotype?

      We don't think it is appropriate for the paper, but we have systematically created double mutants with elg1Δ and found many additive and even synergistic interactions. Here is an example. in Author response image 1, taken from the PhD thesis of Taly Ben-Shitrit, a PhD student in the lab.

      Author response image 1.

      What about cdc13 or ten1? Epistatic?

      We did not test telomere length in combination with Ten1. Combining elg1 with cdc13-50 resulted in synergistic elongation. Given the complex genetic relationship between Stn1/Ten1 and Cdc13, it is hard to interpret this result.

      Seems tenuous to use Y2H to decipher protein-protein interactions occurring out of context (i.e., not at telomere but at reporter gene promoter)

      Y2H is a great method to detect interactions, even if they are transient. Whenever possible, we confirm our findings using co-IP or telo-ChIP.

      Lines 268-270: It would be more accurate to state "can be" instead of "becomes" or "is" as they have not shown that SUMOylation or PCNA unloading have occurred.

      We agree, and have changed the text.

      Cdc13snm protein level?

      Unfortunately our Western blot is not presentable, but the level of Cdc13snm was similar to that of the wt Cdc13, and this result has been already published by Hang et al., 2011.

      Fig S3A: If SUMOylated Cdc13 mediates the Stn1-Elg1 interaction, why is Stn1-Elg1 interaction maintained in cdc13snm strain? This result seems to directly contradict the premise and overall conclusion of this section that Cdc13-SUMO mediates the (Y2H) interaction of Elg1 and Stn1.

      According to our model, the interaction between Stn1 and Elg1 takes place upstream, and only then this complex interacts with SUMOylated Cdc13. Hence, if Cdc13 cannot be SUMOylated, the interaction Elg1-Stn1 is not lost, although Stn1 fails to interact with Cdc13, leading to a telomeric phenotype.

      Line 279: which data establishes Stn1-Elg1 interaction as direct? Fig 2B co-Ip indicates physical but not necessarily direct interaction, but later the authors suggest that the interaction requires a SUMOylated intermediary, and Y2H in Fig. S3B doesn't demonstrate direct interaction.

      We have changed the text, taking out the word "direct".

      Co-Ip shows that interaction of Elg1 with Stn1 occurs mainly during later Sphase and with an overall delay compared to initial Elg1-Pol3 interaction.Co-IP Interaction between Cdc13 and Stn1 is reduced in the absence of Elg1

      The subsection title: "The interaction of Elg1 with Stn1 takes place at telomeres only at late S-phase" is not well supported by the data. I agree the data are consistent with the idea of the interactions occurring at telomeres but there's no direct evidence of this.

      We have changed the subsection title. It now reads: " The interaction of Elg1 with Stn1 takes place only at late S-phase"

      Model: Is unloading happening at the fork? Doesn't PCNA unloading have to follow its loading which occurred behind the fork particularly on the lagging strand? Model now suggest that Stn1 itself is SUMOylated.

      Yes, according to the model Elg1 moves with the fork, unloading PCNA from the lagging strand. Once Elg1 reaches the telomeres, it interacts with Stn1 (Figure 5). This interaction requires SUMOylation of Stn1 or of some other protein, which is not PCNA (Figure 3D) nor Cdc13 (Figure S3A) and could be Stn1 itself or another telomeric protein (Hang et al., 2011)

      Title is rather vague.

      We think it summarizes what we present in the paper.

      Abstract:

      "We report that SUMOylated PCNA acts as a signal that positively regulates telomerase activity."

      I don't think this is supported or a good description of what they find

      Figure 1B clearly shows that SUMO-PCNA is both necessary and sufficient for telomere elongation.

      "and dissected the mechanism by which Elg1 and Stn1 negatively regulates telomere elongation, coordinated by SUMO."

      Again, I don't think this is sufficiently supported and the model invokes SUMOylation events not demonstrated like Stn1, which might be a significant step forward.

      On the positive side, their model makes several predictions that they could test much more directly and rigorously: for example, examining the impact of the relevant mutations in the recruitment of proteins to the telomere.

      We have dissected the mechanism, and future work will be devoted to examining the impact of the relevant mutations in the recruitment of proteins to the telomere.

      Reviewer #3 (Recommendations For The Authors):

      Comments:

      1) The telomere length analysis data presented here is consistent with an interpretation that Stn1 and Elg1 play roles in a similar telomere maintenance pathway because the telomere restriction fragment pattern in the double mutants are not longer than the stn1 single mutants. No comment is made with respect to the yellow bars in Figure 2 that presumably measure telomere length appearing to be slightly shorter than in the stn1 single mutants. It may be interesting and informative if the double mutants do in fact have some phenotype distinct from the single stn1 mutants. Is there an impact on viability in the double mutant?

      Given the variable telomeric phenotype of the single stn1 mutants, slight variations in the measurement of the median telomere size are expected. The difference observed is not likely to be significant. What is important is that the double mutants with elg1 do not show longer telomeres. In terms of fitness, the stn1 mutants grow slightly slowly, but the elg1 mutation does not slow them down further.

      2) It is somewhat surprising that no additional telomere length analysis is included that actually tests the proposed model, including whether this path could be operational only under certain conditions. Maybe this is a topic of the next paper?

      Indeed, future work will explore the conditions under which PCNA SUMOylation is essential, and those under which is only needed.

      3) Were the error bars in Figure 5F determined only from the experiment in E? Does this represent error in measuring the data from one biological replicate? The type of error should be made clear to avoid readers assuming the data represents measurements from more than one sample in more than one experiment. The data would be stronger if it represented measurements from multiple experiments.

      The graph was made with data from three biological replicates. We show the best blot in Figure 5E. We have now stressed this in the Figure Legend.

      4) Why was only one two hybrid reporter shown? Having the multiple reporters can give confidence in interactions. (Not a big deal here given the nice co-IP data.)

      We thought that it is enough to show one reporter, as the results with a different reporter (B-gal assay) led to the same conclusions. since this did not add information and made the paper too lengthy (and boring), we took them out. In any case all data was verified by co-IP.

      5) Line 414 - what are the 32P-radio labeled PCR fragments? Are these solely comprised of TG1-3 repeats of some length? A bit more detail in this aspect of the method could be helpful.

      We have added an explanation on the probe in the Methods section.

      6) Line 432-433 - which anti-HA or anti-My antibodies are these? (very minor detail)

      We have added the details.

    1. Author response:

      The following is the authors’ response to the original reviews.

      Public Reviews: 

      Reviewer #1 (Public review): 

      Summary: 

      Liu and colleagues applied the hidden Markov model on fMRI to show three brain states underlying speech comprehension. Many interesting findings were presented: brain state dynamics were related to various speech and semantic properties, timely expression of brain states (rather than their occurrence probabilities) was correlated with better comprehension, and the estimated brain states were specific to speech comprehension but not at rest or when listening to non-comprehensible speech. 

      Strengths: 

      Recently, the HMM has been applied to many fMRI studies, including movie watching and rest. The authors cleverly used the HMM to test the external/linguistic/internal processing theory that was suggested in comprehension literature. I appreciated the way the authors theoretically grounded their hypotheses and reviewed relevant papers that used the HMM on other naturalistic datasets. The manuscript was well written, the analyses were sound, and the results had clear implications. 

      Weaknesses: 

      Further details are needed for the experimental procedure, adjustments needed for statistics/analyses, and the interpretation/rationale is needed for the results. 

      For the Experimental Procedure, we will provide a more detailed description about stimuli, and the comprehension test, and upload the audio files and corresponding transcriptions as the supplementary dataset. 

      For statistics/analyses, we have reproduced the states' spatial maps using unnormalized activity pattern. For the resting state, we observed a state resembling the baseline state described in Song, Shim, & Rosenberg (2023). However, for the speech comprehension task, all three states were characterized by network activities varying largely from zero. In addition, we have re-generated the null distribution for behaviorbrain state correlations using circular shift. The results are largely consistent with the previous findings. We have also made some other adjustment to the analyses or add some new analyses as recommended by the reviewer. We will revise the manuscript to incorporate these changes.

      For the interpretation/rationale: We will add a more detailed interpretation for the association between state occurrence and semantic coherence. Briefly speaking, higher semantic coherence may allow for the brain to better accumulate information over time.

      State #2 seems to be involved in the integration of information at shorter timescales (hundreds of milliseconds) while State #3 seems to be involved in the longer timescales (seconds). 

      We greatly appreciate the reviewer for the insightful comments and constructive suggestions.  

      Reviewer #2 (Public review): 

      Liu et al. applied hidden Markov models (HMM) to fMRI data from 64 participants listening to audio stories. The authors identified three brain states, characterized by specific patterns of activity and connectivity, that the brain transitions between during story listening. Drawing on a theoretical framework proposed by Berwick et al. (TICS 2023), the authors interpret these states as corresponding to external sensory-motor processing (State 1), lexical processing (State 2), and internal mental representations (State 3). States 1 and 3 were more likely to transition to State 2 than between one another, suggesting that State 2 acts as a transition hub between states. Participants whose brain state trajectories closely matched those of an individual with high comprehension scores tended to have higher comprehension scores themselves, suggesting that optimal transitions between brain states facilitated narrative comprehension. 

      Overall, the conclusions of the paper are well-supported by the data. Several recent studies (e.g., Song, Shim, and Rosenberg, eLife, 2023) have found that the brain transitions between a small number of states; however, the functional role of these states remains under-explored. An important contribution of this paper is that it relates the expression of brain states to specific features of the stimulus in a manner that is consistent with theoretical predictions. 

      (1) It is worth noting, however, that the correlation between narrative features and brain state expression (as shown in Figure 3) is relatively low (~0.03). Additionally, it was unclear if the temporal correlation of the brain state expression was considered when generating the null distribution. It would be helpful to clarify whether the brain state expression time courses were circularly shifted when generating the null. 

      In the revision, we generated the null distribution by circularly shifting the state time courses. The results remain consistent with our previous findings: p = 0.002 for the speech envelope, p = 0.007 for word-level coherence, and p = 0.001 for clause-level coherence.

      We note that in other studies which examined the relationship between brain activity and word embedding features, the group-mean correlation values are similarly low but statistically significant and theoretically meaningful (e.g., Fernandino et al., 2022; Oota et al., 2022). We think these relatively low correlations are primarily due to the high level of noise inherent in neural data. Brain activity fluctuations are shaped by a variety of factors, including task-related cognitive processing, internal thoughts, physiological states, as well as arousal and vigilance. Additionally, the narrative features we measured may account for only a small portion of the cognitive processes occurring during the task. As a result, the variance in narrative features can only explain a limited portion of the overall variance in brain activity fluctuations.

      We will replace Figure 3 and the related supplementary figures with new ones, in which the null distribution is generated via circular shift. Furthermore, we will expand our discussion to address why the observed brain-stimuli correlations are relatively small, despite their statistical significance.

      (2) A strength of the paper is that the authors repeated the HMM analyses across different tasks (Figure 5) and an independent dataset (Figure S3) and found that the data was consistently best fit by 3 brain states. However, it was not entirely clear to me how well the 3 states identified in these other analyses matched the brain states reported in the main analyses. In particular, the confusion matrices shown in Figure 5 and Figure S3 suggests that that states were confusable across studies (State 2 vs. State 3 in Fig. 5A and S3A, State 1 vs. State 2 in Figure 5B). I don't think this takes away from the main results, but it does call into question the generalizability of the brain states across tasks and populations. 

      We identified matching states across analyses based on similarity in the activity patterns of the nine networks. For each candidate state identified in other analyses, we calculate the correlation between its network activity pattern and the three predefined states from the main analysis, and set the one it most closely resembled to be its matching state. For instance, if a candidate state showed the highest correlation with State #1, it was labelled State #1 accordingly. 

      Each column in the confusion matrix depicts the similarity of each candidate state with the three predefined states. In Figure S3 (analysis for the replication dataset), the highest similarity occurred along the diagonal of the confusion matrix. This means that each of the three candidate states was best matched to State #1, State #2, and State #3, respectively, maintaining a one-to-one correspondence between the states from two analyses.

      For the comparison of speech comprehension task with the resting and the incomprehensible speech condition, there was some degree of overlap or "confusion."

      In Figure 5A, there were two candidate states showing the highest similarity to State #2. In this case, we labelled the candidate state with the strongest similarity as State #2, while the other candidate state is assigned as State #3 based on the ranking of similarity. This strategy was also applied to naming of states for the incomprehensible condition. The observed confusion supports the idea that the tripartite-state space is not an intrinsic, task-free property. To make the labeling clearer in the presentation of results, we will use a prime symbol (e.g., State #3') to indicate cases where such confusion occurred, helping to distinguish these ambiguous matches.

      (3) The three states identified in the manuscript correspond rather well to areas with short, medium, and long temporal timescales (see Hasson, Chen & Honey, TiCs, 2015).

      Given the relationship with behavior, where State 1 responds to acoustic properties, State 2 responds to word-level properties, and State 3 responds to clause-level properties, the authors may want to consider a "single-process" account where the states differ in terms of the temporal window for which one needs to integrate information over, rather than a multi-process account where the states correspond to distinct processes. 

      The temporal window hypothesis provides a more fitting explanation for our results. Based on the spatial maps and their modulation by speech features, States #1, #2, and #3 seem to correspond to short, medium, and long processing timescales, respectively. We will update the discussion to reflect this interpretation.

      We sincerely appreciate the constructive suggestions from the two anonymous reviewers, which have been highly valuable in improving the quality of the manuscript.  

      Recommendations for the authors:

      Reviewer #1 (Recommendations for the authors): 

      (1) The "Participants and experimental procedure" section deserves more details. I've checked Liu et al. (2020), and the dataset contained 43 participants aged 20-75 years, whereas this study contained data from 64 young adults and 30 old adult samples. The previous dataset seems to have two stories, whereas this study seems to have three. Please be specific, given that the dataset does not seem the same. Could the authors also include more descriptions of what the auditory stories were? For example, what were the contents, and how were they recorded? 

      The citation is partially incorrect. The dataset of young adults is shared with our work published in (2022). The 64 participants listened to one of three stories told by a female college student in Mandarin, recounting her real-life experience of hiking, a graduate admission interview, and her first time taking a flight, respectively. The sample of older adults is from our work published in (2020), which includes 30 older adults and additionally 13 young adults. The stimuli in this case were two stories told by an older woman in a Chinese dialect, describing her experience in Thailand and riding a warship, respectively. Since we aim to explore whether the main results can be replicated on a different age group, we excluded the 13 young adults from the analysis. 

      All the stories were recorded during fMRI scanning using a noise-canceling microphone (FOMRI-III; Optoacoustics Ltd, Or-Yehuda, Israel) positioned above the speaker’s mouth. The audio recordings were subsequently processed offline with Adobe Audition 3.0 (Adobe Systems Inc., USA) to further eliminate MRI scanner noise.

      In the revised manuscript, we have updated the citation, and provided a more detailed description of the stimuli in the supplementary material. We have also uploaded the audio files along with their corresponding transcriptions to GitHub.

      (2) I am curious about individual differences in comprehension scores. Did participants have less comprehension of the audio-narrated story because the story was a hard-tocomprehend narrative or because the audio quality was low? Could the authors share examples of comprehension tests? 

      We believe two factors contribute to the individual differences in comprehension scores. First, the audio quality is indeed moderately lower than in dailylife story-listening conditions. This is because those stories were recorded and played during fMRI scanning. Although a noise-canceling equipment was used, there were still some noises accompanying the speech, which may have made speech perception and comprehension more difficult than usual.

      Second, the comprehension test measured how much information about the story (including both main themes and details) participants could recall. Specifically, participants were asked to retell the stories in detail immediately after the scanning session. Following this free recall, the experimenters posed a few additional questions drawn from a pre-prepared list, targeting information not mentioned in their recall. If participants experienced lapses of attention or did not store the incoming information into memory promptly, they might fail to recall the relevant content. In several studies, such a task has been called a narrative recall test. However, memory plays a crucial role in real-time speech comprehension, while comprehension affects the depth of processing during memory encoding, thereby influencing subsequent recall performance. To align with prior work (e.g., Stephens et al., 2010) and our previous publications, we chose to referred to this task as narrative comprehension. 

      In the revised manuscript, we have provided a detailed description about the comprehension test (Line 907-933) and share the examples on GitHub. 

      (3) Regarding Figure 3, what does it mean for a state occurrence to follow semantic coherence? Is there a theoretical reason why semantic coherence was measured and related to brain state dynamics? A related empirical question is: is it more likely for the brain states to transition from one state to another when nearby time points share low semantic similarity compared to chance? 

      We analyzed semantic coherence and sound envelope as they capture different layers of linguistic and acoustic structure that unfold over varying temporal scales. Changes in the sound envelope typically occur on the order of milliseconds to a few hundred milliseconds, changes in word-level semantic coherence span approximately 0.24 ± 0.15 seconds, and changes in clause-level semantic coherence extend to 3.2 ± 1.7 seconds. Previous theory and empirical studies suggest that the timescales of information accumulation vary hierarchically, progressing from early sensory areas to higher-order areas (Hasson et al., 2015; Lerner et al., 2011). Based on this work, we anticipate that the three brain states, which are respectively associated with the auditory and sensory motor network, the language network and the DMN, would be selectively modulated by these speech properties corresponding to distinct timescales. 

      Accordingly, when a state occurrence aligns with (clause-level) semantic coherence, it suggests that this state is engaged in processing information accumulated at the clause level (i.e., its semantic relationship). Higher coherence facilitates better accumulation, making it more likely for the associated brain state to be activated. 

      We analyzed the relationship between state transition probability and semantic coherence, but did not find significant results. Here, the transition probability was calculated as Gamma(t) – Gamma(t-1), where Gamma refers to the state occurrence probability. The lack of significant findings may be because brain state transitions are driven primarily by more slowly changing factors. Indeed, we found the average dwell time of the three states ranges from 9.66 to 15.29s, which is a much slower temporal dynamics compared to the relatively rapid shifts in acoustic/semantic properties. 

      In the revised version, we have updated the Introduction to clarify the rational for selecting the three speech properties and to explore their relationship with brain dynamics (Line 111-118)

      (4) When running the HMM, the authors iterated K of 2 to 10 and K = 4, 10, and 12. However, the input features of the model consist of only 9 functional networks. Given that the HMM is designed to find low-dimensional latent state sequences, the choice of the number of latent states being higher than the number of input features sounds odd to me - to my speculation, it is bound to generate almost the exact same states as 9 networks and/or duplicates of the same state. I suggest limiting the K iterations from 2 to 8. For replication with Yeo et al.'s 7 networks, K iteration should also be limited to K of less than 7, or optionally, Yeo's 7 network scheme could be replaced with a 17network scheme. 

      We understand your concern. However, the determination of the number (K) of hidden states is not directly related to the number of features (in this case, the number of networks), but rather depends on the complexity of the time series and the number of underlying patterns. Given that each state corresponds to a distinct combination of the features, even a small number of features can be used to model a system with complex temporal behaviors and multiple states. For instance, for a system with n features, assuming each is a binary variable (0 or 1), there are maximally 2<sup>n</sup> possible underlying states. 

      In our study, we recorded brain activity over 300 time points and used the 9 networks as features. At different time points, the brain can exhibit distinct spatial configurations, reflected in the relative activity levels of the nine networks and their interactions. To accurately capture the temporal dynamics of brain activity, it is essential to explore models that allow for more states than the number of features. We note that in other HMM studies, researchers have also explored states more than the number of networks to find the best number of hidden states (e.g., Ahrends et al., 2022; Stevner et al., 2019). 

      Furthermore, Ahrends et al. (2022) suggested that “Based on the HCP-dataset, we estimate as a rule of thumb that the ratio of observations to free parameters per state should not be inferior to 200”, where free parameters per state is [𝐾 ∗(𝐾 −1)+ (𝐾 −1)+𝐾 ∗𝑁 ∗(𝑁 +1)/2]/𝐾. According to this, there should be above 10, 980 observations when the number of states (K) is 10 (the maximal number in our study) and the number of networks (N) is 9. In our group-level HMM model, there were 64 (valid runs) * 300 (TR) = 19200 observations for young adults, and 50 (valid runs) * 210 (TR) = 10500 observations for older adults. Aside from the older adults' data being slightly insufficient (4.37% less than the suggestion), all other hyperparameter combinations in this study meet the recommended number of observations. 

      (5) In Figure 2, the authors write that the states' spatial maps were normalized for visualization purposes. Could the authors also show visualization of brain states that are not normalized? The reason why I ask is, for example, in Song, Shim, & Rosenberg (2023), the base state was observed which had activity levels all close to the mean (which is 0 because the BOLD activity was normalized). If the activity patterns of this brain state were to be normalized after state estimation, the base state would have looked drastically different than what is reported. 

      We derived the spatial maps of the states using unnormalized activity patterns, with the BOLD signals Z-score normalized to a mean of zero. Under the speech comprehension task, the three states exhibited relatively large fluctuations in network activity levels. The activity ranges were as follows: [-0.71 to 0.51] for State #1, [-0.26 to 0.30] for State #2, and [-0.82 to 0.40] for State #3. For the resting state, we observed a state resembling the baseline state as described in Song, Shim, & Rosenberg (2023), with activity values ranging from -0.133 to 0.09. 

      In the revision, we have replaced the states' spatial maps with versions showing unnormalized activity patterns. 

      (6) In line 297, the authors speculate that "This may be because there is too much heterogeneity among the older adults". To support this speculation, the authors can calculate the overall ISC of brain state dynamics among older adults and compare it to the ISC estimated from younger adults.  

      We analyzed the overall ISC of brain state dynamics, and found the ISC was indeed significantly lower among the older adults than that among the younger adults. We have revised this statement as follows:

      These factors can diminish the inter-subject correlation of brain state dynamics— indeed, ISCs among older adults were significantly lower than those among younger adults (Figure S5)—and reduce ISC's sensitivity to individual differences in task performance (Line 321-326).

      Other comments: 

      (7) In Figure 4, the authors showed a significant positive correlation between head movement ISC with the best performer and comprehension scores. Does the average head movement of all individuals negatively correlate with comprehension scores, given that the authors argue that "greater task engagement is accompanied by decreased movement"? 

      We examined the relationship between participants' average head movement across the comprehension task and their comprehension scores. There was no significant correlation (r = 0.041, p = 0.74). In the literature (e.g. ,Ballenghein et al., 2019) , the relationship between task engagement and head movement was also assessed at the moment-by-moment level, rather than by using time-averaged data.

      Real-time head movements reflect fluctuations in task engagement and cognitive state. In contrast, mean head movement, as a static measure, fails to capture these changes, and thus is not effective in predicting task performance.

      (8) The authors write the older adults sample, the "independent dataset". Technically, however, this dataset cannot be independent because they were collected at the same time by the same research group. I would advise replacing the word independent to something like second dataset or replication dataset. 

      We have replaced the phrase “independent dataset” with “replication dataset”. 

      (9) Pertaining to a paragraph starting in line 586: For non-parametric permutation tests, the authors note that the time courses of brain state expression were "randomly shuffled". How was this random shuffling done: was this circular-shifted randomly, or were the values within the time course literally shuffled? The latter approach, literal shuffling of the values, does not make a fair null distribution because it does not retain temporal regularities (autocorrelation) that are intrinsic to the fMRI signals. Thus, I suggest replacing all non-parametric permutation tests with random circular shifting of the time series (np. roll in python).  

      In the original manuscript, the time course was literally shuffled. In the revised version, we circular-shifted the time course randomly (circshift.m in Matlab) to generate the null distribution. The results remain consistent with our previous findings: p = 0.002 for the speech envelope, p = 0.007 for word-level coherence, and p = 0.001 for clause-level coherence (Line 230-235). 

      (10) The p value calculation should be p = (1+#(chance>=observed))/(1+#iterations) for one-tailed test and p = (1+#(abs(chance)>=abs(observed)))/(1+#iterations) for twotailed test. Thus, if 5,000 iterations were run and none of the chances were higher than the actual observation, the p-value is p = 1/5001, which is the minimal value it can achieve. 

      Have corrected. 

      (11) State 3 in Figure S2 does not resemble State 3 of the main result. Could the authors explain why they corresponded State 3 of the Yeo-7 scheme to State 3 of the nineparcellation scheme, perhaps using evidence of spatial overlap? 

      The correspondence of states between the two schemes was established using evidence of state expression time course. 

      To assess temporal overlap, we calculated Pearson’s correlation between each candidate state obtained by the Yeo-7 scheme and the three predefined states obtained by the nine-network parcellation scheme in terms of state expression probabilities. The time courses of the 64 participants were concatenated, resulting in 19200 (300*64) time points for each state. The one that the candidate state most closely resembled was set to be its corresponding state. For instance, if a candidate state showed the highest correlation with State #1, it was labelled State #1 accordingly. As demonstrated in the confusion matrix, each of the three candidate states was best matched to State #1, State #2, and State #3, respectively, maintaining a one-to-one correspondence between the states from the two schemes.

      We also assessed the spatial overlap between the two schemes. First, a state activity value was assigned to each voxel across the whole brain (including a total of 34,892 voxels covered by both parcellation schemes). This is done for each brain state. Next, we calculated Spearman’s correlation between each candidate state obtained by the Yeo-7 scheme and the three predefined states obtained by the nine-network scheme in terms of whole-brain activities. The pattern of spatial overlap is consistent with the pattern of temporal overlap, such that each of the three candidate states was best matched to State #1, State #2, and State #3, respectively.

      Author response image 1.

      We noted that the networks between the two schemes are not well aligned in their spatial location, especially for the DMN (as shown below). This may lead to the low spatial overlap of State #3, which is dominated by DMN activity. Consequently, establishing state correspondence based on temporal information is more appropriate in this context. We therefore only reported the results of temporal overlap in the manuscript. 

      We have added a paragraph in the main text for “Establishing state correspondence between analyses” (Line 672-699). We have also updated the associated figures (Fig.S2, Fig.S3 and Fig.5)

      Author response image 2.

      (12) Line 839: gamma parameter, on a step size of? 

      (16) Figure 3. Please add a legend in the "Sound envelope" graph what green and blue lines indicate. The authors write Coh(t) and Coh(t, t+1) at the top and Coh(t) and Coh(t+1) at the bottom. Please be consistent with the labeling. Shouldn't they be Coh(t-1, t) and Coh(t, t+1) to be exact for both? 

      Have corrected. 

      (17) In line 226, is this one-sample t-test compared to zero? If so, please write it inside the parentheses. In line 227, the authors write "slightly weaker"; however, since this is not statistically warranted, I suggest removing the word "slightly weaker" and just noting significance in both States 1 and 2.  

      Have corrected.

      (18) In line 288, please fix "we also whether". 

      Have corrected. 

      (19) In Figure 2C, what do pink lines in the transition matrix indicate? Are they colored just to show authors' interests, or do they indicate statistical significance? Please write it in the figure legend.   

      Yes, the pink lines indicate a meaningful trend, showing that the between-state transition probabilities are significantly higher than those in permutation.

      We have added this information to the figure legend. 

      Reviewer #2 (Recommendations for the authors):

      (1) It is unclear how the correspondence between states across different conditions and datasets was computed. Given the spatial autocorrelation of brain maps, I recommend reporting the Dice coefficient along with a spin-test permutation to test for statistical significance.  

      The state correspondence between different conditions and between the two datasets are established using evidence of spatial overlap. The spatial overlap between states was quantified by Pearson’s correlation using the activity values (derived from HMM) of the nine networks. For each candidate state identified in other analyses (for the Rest, MG and older-adult datasets), we calculate the correlation between its network activity pattern and the three predefined states from the main analysis (for the young-adults dataset), and set the one it most closely resembled to be its matching state. For instance, if a candidate state showed the highest correlation with State #1, it was labelled State #1 accordingly. 

      For the comparison between the young and older adults’ datasets (as shown below), the largest spatial overlap occurred along the diagonal of the confusion matrix, with high correlation values. This means that each of the three candidate states was best matched to State #1, State #2, and State #3, respectively, maintaining a one-to-one correspondence between the states from the two datasets. As the HMM is modelled at the level of networks which lack accurate coordinates, we did not apply the spin-test to assess the statistical significance of overlap. Instead, we extracted the state activity patterns from the 1000 permutations (wherein the original BOLD time courses were circularly shifted and an HMM was conducted) for the older-adults dataset. Applying the similar state-correspondence strategy, we generated a null distribution of spatial overlap. The real overlap of the three states was greater than and 97.97%, 95.34% and 92.39% instances from the permutation (as shown below). 

      Author response image 3.

      For the comparison of main task with the resting and the incomprehensible speech condition, there was some degree of confusion: there were two candidate states showing the highest similarity to State #2. In this case, we labeled the most similar candidate as State #2. The other candidate was then assigned to the predefined state with which it had the second-highest correlation. We used a prime symbol (e.g., State #3') to denote cases where such confusion occurred. These findings support our conclusion that the tripartite-organization of brain states is not a task-free, intrinsic property.

      When establishing the correspondence between the Yeo-7 network and the ninenetwork parcellation schemes, we primarily relied on evidence from temporal overlap measures, as a clear network-level alignment between the two parcellation schemes is lacking. Temporal overlap was quantified by calculating the correlation of state occurrence probabilities between the two schemes. To achieve this, we concatenated the time courses of 64 participants, resulting in a time series consisting of 19,200 time points (300 time points per participant) for each state. Each of the three candidate states from the Yeo-7 network scheme was best matched to State #1, State #2, and State #3 from the main analyses, respectively. To determine the statistical significance of the temporal overlap, we circular shifted each participant’s time course of state expression obtained from the Yeo-7network scheme for 1000 times. Applying the same strategy to find the matching states, we generated a null distribution of overlap. The real overlap was much higher than the instances from permutation. 

      Author response image 4.

      In the revision, we have provided detailed description for how the state correspondence is established and reported the statistical significance of those correspondence (Line 671-699). The associated figures have also been updated (Fig.5, Fig. S2 and Fig.S3).  

      (2) Please clarify if circle-shifting was applied to the state expression time course when generating the null distribution for behavior-brain state correlations reported in Figure (3). This seems important to control for the temporal autocorrelation in the time courses.  

      We have updated the results by using circle-shifting to generated the null distribution. The results are largely consistent with the previous on without circular shifting (Line 230-242). 

      (3) Figure 3: What does the green shaded area around the sound envelope represent? In the caption, specify whether the red line in the null distributions indicates the mean or median R between brain state expression and narrative features. It would also be beneficial to report this value in the main text. 

      The green shaded area indicated the original amplitude of speech signal, while blue line indicates the smoothed, low-frequency contour of amplitude changes over time (i.e., speech envelope). We have updated the figure and explained this in the figure caption. 

      The red line in the null distributions indicates the R between brain state expression and narrative features for the real data. and reported the mean R of the permutation in the main text. 

      (4) The manuscript is missing a data availability statement (https://elifesciences.org/inside-elife/51839f0a/for-authors-updates-to-elife-s-datasharing-policies). 

      We have added a statement of data availability in the revision, as follows: 

      “The raw and processed fMRI data are available on OpenNeuro: https://openneuro.org/datasets/ds005623. The experimental stimuli, behavioral data and main scripts used in the analyses are provided on Github. ”

      (5) There is a typo in line 102 ("perceptual alalyses"). 

      Have corrected. 

      We sincerely thank the two reviewers for their constructive feedback, thorough review, and the time they dedicated to improving our work.

      Reference: 

      Ahrends, C., Stevner, A., Pervaiz, U., Kringelbach, M. L., Vuust, P., Woolrich, M. W., & Vidaurre, D. (2022). Data and model considerations for estimating timevarying functional connectivity in fMRI. Neuroimage, 252, 119026. 

      Ballenghein, U., Megalakaki, O., & Baccino, T. (2019). Cognitive engagement in emotional text reading: concurrent recordings of eye movements and head motion. Cognition and Emotion. 

      Fernandino, L., Tong, J.-Q., Conant, L. L., Humphries, C. J., & Binder, J. R. (2022). Decoding the information structure underlying the neural representation of concepts. Proceedings of the national academy of sciences, 119(6), e2108091119. https://doi.org/10.1073/pnas.2108091119  

      Hasson, U., Chen, J., & Honey, C. J. (2015). Hierarchical process memory: memory as an integral component of information processing. Trends in Cognitive Sciences, 19(6), 304-313. 

      Lerner, Y., Honey, C. J., Silbert, L. J., & Hasson, U. (2011). Topographic mapping of a hierarchy of temporal receptive windows using a narrated story [Article]. Journal of Neuroscience, 31(8), 2906-2915. https://doi.org/10.1523/JNEUROSCI.3684-10.2011  

      Liu, L., Li, H., Ren, Z., Zhou, Q., Zhang, Y., Lu, C., Qiu, J., Chen, H., & Ding, G. (2022). The “two-brain” approach reveals the active role of task-deactivated default mode network in speech comprehension. Cerebral Cortex, 32(21), 4869-4884. 

      Liu, L., Zhang, Y., Zhou, Q., Garrett, D. D., Lu, C., Chen, A., Qiu, J., & Ding, G. (2020). Auditory–Articulatory Neural Alignment between Listener and Speaker during Verbal Communication. Cerebral Cortex, 30(3), 942-951. https://doi.org/10.1093/cercor/bhz138

    1. Author Response

      The following is the authors’ response to the original reviews.

      eLife assessment

      This valuable study addresses the long-term effect of warming and altered precipitation on microbial growth, as a proxy for understanding the impact of global warming. While the methods are compelling and the evidence supporting the claims is solid, additional analysis of the data would strengthen the study, which should be of broad interest to microbial ecologists and microbiologists.

      We sincerely appreciate your assessment and thoughtful comments, which are valuable and very helpful for improving our manuscript. We have carefully considered all comments, and made extensive, thorough corrections and additional analysis of the data, which we hope to meet with approval.

      Reviewer #1 (Public Review):

      Warming and precipitation regime change significantly influences both above-ground and below-ground processes across Earth's ecosystems. Soil microbial communities, which underpin the biogeochemical processes that often shape ecosystem function, are no exception to this, and although research shows they can adapt to this warming, population dynamics and ecophysiological responses to these disturbances are not currently known. The Qinghai-Tibet Plateau, the Third Pole of the Earth, is considered among the most sensitive ecosystems to climate change. The manuscript described an integrated, trait-based understanding of these dynamics with the qSIP data. The experimental design and methods appear to be of sufficient quality. The data and analyses are of great value to the larger microbial ecological community and may help advance our understanding of how microbial systems will respond to global change. There are very few studies in which the growth rates of bacterial populations from multifactorial manipulation experiments on the Qinghai-Tibet Plateau have been investigated via qSIP, and the large quantity of data that comprises the study described in this manuscript, will substantially advance our knowledge of bacterial responses to warming and precipitation manipulations.

      We appreciate the encouragement and positive comments.

      Specific comments:

      (1) Please add some names of microbial groups with most common for the growth rates.

      We have added the sentence “The members in Solirubrobacter and Pseudonocardia genera had high growth rates under changed climate regimes” In the Abstract (Line 57-58).

      (2) L47-48, consider changing "microbial growth and death" to "microbial eco-physiological processes (e.g., growth and death)", and changing "such eco-physiological traits" to "such processes".

      Done (Line 47 and 48).

      (3) L50-51, the author estimated bacterial growth in alpine meadow soils of the Tibetan Plateau after warming and altered precipitation manipulation in situ. Actually, the soil samples were collected and incubated in the laboratory rather than in the field like the previous experiment conducted by Purcell et al. (2021, Global Change Biology). "In situ" would lead me to believe that the qSIP incubation was conducted in the field, so I think the use of the word in situ is inappropriate here. [https://onlinelibrary.wiley.com/doi/full/10.1111/gcb.15911]

      Agreed. We have deleted “in situ”.

      (4) L52, what does "interactive global change factors" mean?

      We have revised this sentence to “the growth of major taxa was suppressed by the single and combined effects of temperature and precipitation” (Line 52-53).

      (5) L61, in my opinion, "Microbial diversity" belongs to the category of species composition, rather than ecosystem functional services. Please revise it.

      Agree. We have deleted it.

      (6) L69, consider changing "further" to "thus".

      Done (Line 70).

      (7) L82, delete "The evidence is overwhelming that".

      Done.

      (8) L85-90, these two sentences have similar meanings, please express them concisely.

      We have deleted the sentence “Altered precipitation, particularly drought or heavy precipitation events, also tends to negatively influence soil processes and biodiversity”.

      (9) L91, the effect of drought on soil microorganisms is lacking here.

      We have added the sentence “Reduced precipitation affects soil processes notably by directly stressing soil organisms, and also altering the supply of substrates to microbes via dissolution, diffusion, and transport” in the Introduction (Line87-89).

      (10) L102, "Growth" should be highlighted here, as changes in relative abundance can also be classified as population dynamics. The use of the term "population dynamics" will eliminate the highlight of this study in calculating the growth rate of microbial species in in-situ soil based on qSIP. Consider changing "population dynamics" to "population-growth responses" or something like that.

      Done (Line 98).

      (11) L105, please note that this citation focuses on plant physiological characteristics.

      We have revised the reference (Line 102).

      (12) L115, "soil temperature, water availability" should be considered as a direct impact of climate change, rather than an indirect impact on microorganisms.

      We have deleted them.

      (13) L134-135, please clarify the interaction types between which climate factors.

      We have deleted this sentence.

      (14) L135-138, suggest modifying or deleting this sentence. The results in this study are already eco-physiological data and do not need to be further "understood and predicted".

      We have deleted this sentence.

      (15) L150, "The experimental design has been described in previously". I think this refers to another study and not the actual incubations in this study. Also in L198, suggest a change to "Incubation conditions were similar to those previously described". So, it's clear it's not the same experiment.

      We have revised these sentences to “has been described previously in (Ma et al., 2017)” (Line 136) and “according to a previous publication” (Line 194).

      Reference:

      Ma, Z., Liu, H., Mi, Z., Zhang, Z., Wang, Y., Xu, W. et al. (2017). Climate warming reduces the temporal stability of plant community biomass production. Nature Communications, 8, 15378.

      (16) L188, change "pre-wet soil samples" to "pre-wet samples" and change "soil samples for 48h incubation" to "incubation samples". What does "pre-wet" mean? Does it represent soil pre-cultivation?

      Done. The pre-wet samples, i.e., the soil samples before incubation (T = 0 d), were used to estimate the initial microbial composition. "pre-wet" does not mean soil pre-cultivation. We have added the description “A portion of the air-dried soil samples was taken as the pre-wet treatment (i.e., before incubation without H2O addition)” in MATERIALS AND METHODS (Line 174-175).

      (17) Unify the time unit of incubation (hour or day). Consider changing "48 h" to "2 d" in Materials and Methods.

      Done.

      (18) L247, what version of RDP Classifier was used?

      We used RDP v16 database for taxonomic annotation. We have added this information in the revision (Line 246).

      (19) L270, "average molecular weights".

      Done (Line 268).

      (20) L272-275, based on the preceding description, it appears that the culture period was limited to 48 hours. Please confirm it.

      Apologize for this mistake. We have revised it (Line 273).

      (21) L297, switch the order of the first two sentences of this paragraph.

      Done (Line 297).

      (22) L331, change "smaller-than-additive" to "smaller than their expected additive effect".

      Done (Line 331).

      (23) L374 and 381, I struggle with why "larger combined effects" than single factor effects represent higher degree of antoninism, and I think it should be "smaller combined effects".

      Agree. We have revised it according to this suggestion (Line 369 and 374).

      (24) L375, remove "than that of drought and warming".

      Done.

      (25) L405, simplify the expression, change "between different warming and rainfall regimes" to "between climate regimes"

      We have deleted this sentence.

      (26) L406-408, species are already on the phylogenetic tree and they can not "clustered at the phylogenetic branches", but the functional traits of microbes can. Please revise it.

      We have revised this sentence to “Overall, the most incorporators whose growth was influenced by the antagonistic interaction of T × P showed significant phylogenetic clustering (i.e., species clustered at the phylogenetic branches; NTI > 0, P < 0.05)” (Line 402-404).

      (27) L409, the same as above, and consider removing "The incorporators subjected to". We have revised this sentence to “The incorporators whose growth subjected to the additive interaction of warming × drought also showed significant phylogenetic clustering (P < 0.05)” (Line 404-406).

      (28) L412, consider changing "incorporators subjected to the synergistic interaction" to "the synergistic growth responses under multifactorial changes".

      We have revised the sentence to “incorporators whose growth is influenced by the synergistic interaction showed phylogenetically random distribution under both climate scenarios (P > 0.05)” (Line 407-409).

      (29) L505-506, please add a reference for this sentence.

      Done (Line 488).

      (30) L511-514, It should be noted that the production of MBC does not necessarily imply a net change in the C pool size. The accelerated growth rates may result in expedited turnover of MBC, rather than an increase in carbon sequestration.

      Thanks. We have deleted this sentence.

      (31) Language precision. In the discussion section there must be some additional caveats introduced to some of the claims the authors are making. For instance, L518, the author should clarify that "in this study, the bacterial growth in alpine grassland may be influenced by antagonistic interactions between multiple climatic factors after a decadal-long experiment". Because other studies may exhibit different results due to the focus on different ecosystem functions as well as environmental conditions. As such, softening of the language is recommended- lines are noted below- and these will not adjust the outcomes of this study, but support more precise interpretation.

      We have revised the sentence to “In this study, a decade-long experiment revealed that bacterial growth in alpine meadows is primarily influenced by the antagonistic interaction between T × P” (Line 497-499).

      (32) Picrust analysis is a good way to connect species and their functions, especially Picrust2, which updated the reference database and optimized the algorithm to improve its prediction accuracy (Douglas et al., 2020, Nature Biotechnology). However, the link between microbial taxonomy and microbial metabolism is still not straightforward, especially in diverse microbial communities like soils. The authors should introduce caveats within discussion that they know the limitations of their methods. For context, as a reader who does metabolisms in soils, I found myself somewhat disappointed when piecrust data was introduced and not properly caveated. Particularly, it might be helpful to introduce briefly in the last paragraph of the results. These caveats are necessary to not potentially overstate the author's findings, and to make sure the reader knows the authors understand the very clear limitations of these methods. [https://www.nature.com/articles/s41587-020-0548-6]

      Thanks. We have introduced caveats in DISCUSSION, that is “This is, however, still to be verified, as the functional output from PICRUSt2 is less likely to resolve rare environment-specific functions (Douglas et al., 2020)” (Line 540-542).

      Reference:

      Douglas, G., Maffei, V., Zaneveld, J., Yurgel, S., Brown, J., Taylor, C. et al. (2020). PICRUSt2 for prediction of metagenome functions. Nature Biotechnology, 38, 1-5.

      (33) Although the author has explained the potential causes for the negative effects of different climate change factors (i.e., warming, drought, and wet) on microbial growth, there seems to be a lack of a summary assertion and an extension on how climate change affects microbial growth and related ecosystem functions. It is recommended to make a general summary of the results in the last part of Discussion.

      We have added a general summary in the last paragraph of DISCUSSION, that is “Our results demonstrated that both warming and altered precipitation negatively affect the growth of grassland bacteria; However, the combined effects of warming and altered precipitation on the growth of ~70% soil bacterial taxa were smaller than the single-factor effects, suggesting antagonistic interaction. This suggests the development of multifactor manipulation experiments in precise prediction of future ecosystem services and feedbacks under climate change scenarios” (Line 552-558).

      (34) L546, please add the taxonomic information for "OTU 14".

      Done (Line 533).

      (35) L800, change "The phylogenetic tree" to "A phylogenetic tree".

      Done (Line 762).

      Reviewer #2 (Public Review):

      Summary:

      The authors aimed to describe the effect of different temperature and precipitation regimes on microbial growth responses in an alpine grassland ecosystem using quantitative 18O stable isotope probing. It was found that all climate manipulations had negative effects on microbial growth, and that single-factor manipulations exerted larger negative effects as compared to combined-factor manipulations. The degree of antagonism between factors was analyzed in detail, as well as the differential effect of these divergent antagonistic responses on microbial taxa that incorporated the isotope. Finally, a hypothetical functional profiling was performed based on taxonomic affiliations. This work gives additional evidence that altered warming and precipitation regimes negatively impact microbial growth.

      Strengths:

      A long term experiment with a thorough experimental design in apparently field conditions is a plus for this work, making the results potentially generalisable to the alpine grassland ecosystem. Also, the implementation of a qSIP approach to determine microbial growth ensures that only active members of the community are assessed. Finally, particular attention was given to the interaction between factors and a robust approach was implemented to quantify the weight of the combined-factor manipulations on microbial growth.

      We appreciate the reviewer’s positive comments.

      Weaknesses:

      The methodology does not mention whether the samples taken for the incubations were rhizosphere soil, bulk soil or a mix between both type of soils. If the samples were taken from rhizosphere soil, I wonder how the plants were affected by the infrared heaters and if the resulting shadow (also in the controls with dummy heaters) had an effect on the plants and the root exudates of the parcels as compared to plants outside the blocks? If the samples were bulk soil, are the results generalisable for a grassland ecosystem? In my opinion, it is needed to add more info on the origin of the soil samples and how these were taken.

      The samples taken for the incubations can be considered as a mixture of rhizosphere and bulk soils. During soil sampling, we did not use conventional rhizosphere soil collection methods. However, there is a certain proportion of fragmented roots in the soil samples we collected, indicating that soil properties are influenced by plants. We have added this description in MATERIALS AND METHODS (Line 158).

      To minimize the impact of physical shading on the plants, each sampling point was as far away from infrared heaters as possible. We have added this information of soil collection in MATERIALS AND METHODS, that is “In each plot, three soil cores of the topsoil (0-5 cm in depth) were randomly collected and combined as a composite sample, which can be considered as a mixture of rhizosphere and bulk soils. Each sampling point was as far away from infrared heaters as possible to minimize the impact of physical shading on the plants. The fresh soil samples were shipped to the laboratory and sieved (2-mm) to remove root fragments and stones.” (Line 157-162).

      Previous studies based on our field experiment assessed the effects of warming and altered precipitation on soil microbial communities (Zhang et al., 2016), the temporal stability of plant community biomass (Ma et al., 2017), shifting plant species composition and grassland primary production (Liu et al., 2018). These studies provide guidance for the experiment design and execution.

      Reference:

      Zhang, KP., Shi, Y., Jing, X. et al. (2016). Effects of Short-Term Warming and Altered Precipitation on Soil Microbial Communities in Alpine Grassland of the Tibetan Plateau. Frontiers in Microbiology, 7, 1-11.

      Ma ZY., Liu, HY., Mi, ZR. et al. (2017). Climate warming reduces the temporal stabilityof plant community biomass production. Nature Communications, 8, 15378.

      Liu, HY., Mi, ZR., Lin, L. et al. (2018). Shifting plant species composition in response to climate change stabilizes grassland primary production. Proceedings of the National Academy of Sciences, 115, 4051-4056.

      The qSIP calculations reported in the methodology for this work are rather superficial and the reader must be experienced in this technique to understand how the incorporators were identified and their growth quantified. For instance, the GC content of taxa was calculated for reads clustered in OTUs, and it is not discussed in the text the validity of such approach working at genus level.

      We have added the description of qSIP calculations in Supplementary Materials.

      The approach of GC content calculation can be used at genus level (Koch et al., 2018). The GC content of each bacterial taxon (Gi) was calculated using the mean density for the unlabeled (WLIGHTi) treatments (Hungate et al. 2015), rather than OTU sequence information. We have revised the sentence in MATERIALS AND METHODS, that is “the number of 16S rRNA gene copies per OTU taxon (e.g., genus or OTU) in each density fraction was calculated by multiplying the relative abundance (acquisition by sequencing) by the total number of 16S rRNA gene copies (acquisition by qPCR)” (Line 255-258).

      Reference:

      Hungate, B., Mau, R., Schwartz, E., Caporaso, J., Dijkstra, P., Van Gestel, N. et al. (2015). Quantitative microbial ecology through stable isotope probing. Applied and Environmental Microbiology, 81, 7570-7581.

      Koch, B., McHugh, T., Hayer, M., Schwartz, E., Blazewicz, S., Dijkstra, P. et al. (2018). Estimating taxon-specific population dynamics in diverse microbial communities. Ecosphere, 9, e02090.

      The selection of V4-V5 region over V3-V4 region to quantify the number of copies of the 16S rRNA gene should be substantiated in the text. Classic works determined one decade ago that primer pairs that amplify V3-V4 are most suitable to assess soil bacterial communities. Hungate et al. (2015), worked with the V3-V4 region when establishing the qSIP method. Maybe the number of unassigned OTUs is related with the selection of this region.

      Both primer sets (V3-V4 and V4-V5 regions), are widely used across various sample sets, with highly similar in representing the total microbial community composition (Fadeev et al., 2021; Zhang et al., 2018).

      A previous study based on our Field Research Station of Alpine Grassland Ecosystem used V4-V5 primer pairs to investigated the effect of warming and altered precipitation on the overall bacterial community composition (Zhang et al., 2016).

      Another reason for choosing the V4-V5 primer set in this study was to integrate and compare the data with that of two previous qSIP studies (Ruan et al., 2023; Guo et al., submitted), both of them focused on the growth responses of active species to global change and used V4-V5 primer pairs.

      We have added an explanation about primer selection as “The V4-V5 primer pairs were chosen to facilitate integration and comparison with data from previous studies (Ruan et al., 2023; Zhang et al., 2016)” (Line 213-215).

      Reference:

      Fadeev, E., Cardozo-Mino, M.G., Rapp, J.Z. et al. (2021). Comparison of Two 16S rRNA Primers (V3–V4 and V4–V5) for Studies of Arctic Microbial Communities. Frontiers in Microbiology, 12

      Zhang, J.Y., Ding, X., Guan, R. et al. (2018). Evaluation of different 16S rRNA gene V regions for exploring bacterial diversity in a eutrophic freshwater lake. Science of The Total Environment, 618, 1254-1267.

      Zhang, K.P., Shi, Y., Jing, X. et al. (2016). Effects of Short-Term Warming and Altered Precipitation on Soil Microbial Communities in Alpine Grassland of the Tibetan Plateau. Frontiers in Microbiology, 7, 1-11.

      Ruan, Y., Kuzyakov, Y., Liu, X. et al. (2023). Elevated temperature and CO2 strongly affect the growth strategies of soil bacteria. Nature Communications, 14, 1-12.

      Guo, J.J., Kuzyakov, Y., Li, L. et al. (2023). Bacterial growth acclimation to long-term nitrogen input in soil. The ISME Journal, Submitted.

      Report of preprocessing and processing of the sequences does not comply state of the art standards. More info on how the sequences were handled is needed, taking into account that a significant part of the manuscript relies on taxonomic classification of such sequences. Also, an OTU approach for an almost species-dependent analysis (GC contents) should be replaced or complemented with an ASV or subOTUs approach, using denoisers such as DADA2 or deblur. Usage of functional prediction tools underestimates gene frequencies, including those related with biogeochemical significance for soil-carbon and nitrogen cycling.

      (1) We have complemented the information about sequence processing as “The raw sequences were quality-filtered using the USEARCH v.11.0 (Edgar, 2010). In brief, the paired-end sequences were merged and quality filtered with “fastq_mergepairs” and “fastq_filter” commands, respectively. Sequences < 370 bp and total expected errors > 0.5 were removed. Next, “fastx_uniques” command was implemented to remove redundant sequences. Subsequently, high-quality sequences were clustered into operational taxonomic units (OTUs) with “cluster_otus” commandat a 97% identity threshold, and the most abundant sequence from each OTU was selected as a representative sequence.” (Line 238-245).

      (2) We have complemented the zero-radius OTU (ZOTU) analysis by the unoise3 command in USEARCH (https://drive5.com/usearch/manual/pipe_otus.html), as shown in Fig. S1-S2. The results showed that overall growth responses of soil bacteria to warming and precipitation changes were similar based on OTU and ZOTU analyses, i.e., warming and altered precipitation tend to negatively affect the growth of grassland bacteria and the prevalence of antagonistic interactions of T × P. The similarity of results between the different methods is reflected at the overall community level, the phylum level, the genus level and the species (i.e., OTU or ZOTU) level (Fig. S1 and S2).

      Author response image 1.

      The growth responses of grassland bacteria to warming and altered precipitation based on ZOTU analysis. The results of growth rates at the community level (A), the phylum level (B), and the ZOTU level (C and D) were similar to those based on OTU analysis. C the single and combined factor effects of climate factors on species growth, by comparing with the growth rates in T0nP. D the proportions of species growth influenced by different interaction types of T × P. T0-P represents the ambient temperature and decreased precipitation; T+-P represents warming and decreased precipitation; T0cP represents ambient temperature and precipitation; T+cP represents warming and ambient precipitation; T0+P represents ambient temperature and enhanced precipitation; T++P represents warming and enhanced precipitation. Values represent mean and the error bars represent standard deviation. Different letters indicate significant differences between climate treatments.

      Author response image 2.

      The growth responses of grassland bacteria at the genus level to warming and altered precipitation based on OTU analysis (A and C) and ZOTU analysis (B and D). A and B the single and combined factor effects of climate factors on growth in genera, by comparing with those in T0nP. C and D the proportions of genera whose growth influenced by different interaction types of T × P.

      (3) Agreed. We have introduced the caveat about the limitation of usage of functional prediction tools to the end of DISCUSSION, that is “This is, however, still to be verified, as the functional output from PICRUSt2 is less likely to resolve rare environment-specific functions (Douglas et al., 2020)” (Line 540-542). The caveat ensures that the reader knows the limitations of these methods, and are not potentially overstate our findings.

      Reference:

      Douglas, G.M., Maffei, V.J., Zaneveld, J.R. et al. (2020) PICRUSt2 for prediction of metagenome functions. Nat Biotechnol. 38, 685–688.

      Reviewer #2 (Recommendations For The Authors):

      General suggestions:

      Regarding the qSIP method, would be of help to see the differences in density vs number of 16S rRNA gene abundance for the most responsive bacterial groups in the different treatments, taking into account that with only 7 fractions the entire change in bacterial growth was resolved.

      We have selected three representative bacterial taxa (OTU1 belonging to Bradyrhizobium, OTU14 belonging to Solirubrobacter, OTU15 belonging to Pseudoxanthomonas), which have high growth rates in climate change treatments. The result showed that the peaks in the 18O treatment are shifted "backwards" (greater average weighted buoyancy density) compared to the 16O treatment, indicating that these species assimilates the 18O isotope into the DNA molecules during growth.

      Author response image 3.

      The distribution of 16S rRNA gene abundance of three representative bacterial taxa (OTU1- Bradyrhizobium, OTU14-Solirubrobacter, and OTU15-Pseudoxanthomonas) in different buoyant density fractions. Values represent mean and the error bars represent standard deviation.

      Seven fractionated DNA samples were selected for sequencing because they contained more than 99% gene copy numbers of each samples (please see the Figure below). The DNA concentrations of other fractions were too low to construct sequencing libraries.

      Author response image 4.

      Relative abundance of 16S rRNA gene copies in each fraction. The fractions with density between 1.703 and 1.727 g ml-1 were selected because they contained more than 99% gene copy numbers of each sample. T0-P represents the ambient temperature and decreased precipitation; T+-P represents warming and decreased precipitation; T0cP represents ambient temperature and precipitation; T+cP represents warming and ambient precipitation; T0+P represents ambient temperature and enhanced precipitation; T++P represents warming and enhanced precipitation. Values represent mean and the error bars represent standard deviation.

      With such dataset additional multivariate analysis would be of help to better interpret the ecological framework.

      Thanks for the suggestion. Interpreting the ecological framework is meaningful for understanding microbial responses to environmental changes.

      The main objective of this study is to investigate the growth response of soil microbes in alpine grasslands to the temperature and precipitation changes, and the interaction between climate factors. Our results, as well as the results of complementary analyses (based on subOTU analyses, SHOWN BELOW), indicate that warming and altered precipitation tend to negatively affect the growth of grassland bacteria, and the prevalence of antagonistic interactions of T × P.

      We have emphasized our research objectives and main conclusions in the revised manuscript: “The goal of current study is to comprehensively estimate taxon-specific growth responses of soil bacteria following a decade of warming and altered precipitation manipulation on the alpine grassland of the Tibetan Plateau” (Line 112-114);

      “Our results demonstrated that both warming and altered precipitation negatively affect the growth of grassland bacteria; However, the combined effects of warming and altered precipitation on the growth of ~70% soil bacterial taxa were smaller than the single-factor effects, suggesting antagonistic interaction” (Line 552-556).

      Extension of interaction analysis and its conclusions should be shortened, summarizing the most relevant findings. In my opinion, it becomes a bit redundant.

      We have shortened the discussion of Extension of interaction analysis by deleting the little relevant contents.

      Below are some, but not all, examples that have been deleted or revised in the Discussion,

      (1) Deleted “This result supports our second hypothesis that the interactive effects between warming and altered precipitation on soil microbial growth are not simply additive”;

      (2) Deleted “A previous study suggested that multiple global change factors had negative effects on soil microbial diversity (Yang et al., 2021)”;

      (3) Revised “A meta‐analysis of experimental manipulation revealed that the combined effects of different climate factors were usually less than expected additive effects, revealing antagonistic interactions on soil C storage and nutrient cycling processes (Dieleman et al., 2012; Wu et al., 2011). Moreover, two experimental studies on N cycling and net primary productivity demonstrated that the majority of interactions among multiple factors are antagonistic rather than additive or synergistic, thereby dampening the net effects (Larsen et al., 2011; Shaw et al., 2002)” to “A range of ecosystem processes have been revealed to be potentially subject to antagonistic interactions between climate factors, for instance, net primary productivity (Shaw et al., 2002), soil C storage and nutrient cycling processes (Dieleman et al., 2012; Wu et al., 2011; Larsen et al., 2011)” (Line 499-503);

      (4) Revised “Previous evidences suggest that warming has a negative impact on soil carbon pools (Jansson & Hofmockel, 2020; Purcell et al., 2022). During the first phase of soil warming (~ 10 years), microbial activity increased, resulting in rapid soil carbon mineralization and respiration (Melillo et al., 2017)” to “Previous evidences suggest that warming has a negative impact on soil carbon pools (Jansson & Hofmockel, 2020; Purcell et al., 2022), mainly because of the rapid soil carbon mineralization and respiration (Melillo et al., 2017)” (Line 464-466).

      I strongly suggest a functional analysis based on shotgun sequencing or RNAseq approaches. With this approach this work would be able to answer who is growing under altered T and Precipitation regimes and what are those that are growing doing.

      Thanks for the suggestion. Metagenomic sequencing is a popular approach to evaluate potential functions of microbial communities in environment. However, there are two main reasons that limit the application of metagenomic or metatranscriptomic sequencing in this study: 1) Most of the fractionated samples in SIP experiment have low DNA concentration and do not meet the requirement of library construction for sequencing; 2) Metagenome and metatranscriptomics usually have relatively low sensitivity to rare species, reducing the diversity of detected active species.

      This study focused on active microbial taxa and their growth in response to multifactorial climate change. We have added the prospect in DISCUSSION, that is “This suggests the development of methods combining qSIP with metagenomes and metatranscriptomes to assess the functional shifts of active microorganisms under global change scenarios” (Line 542-544).

      Minor suggestions:

      L121. _As

      We have deleted this sentence and relocated the hypotheses in the last paragraph of INTRODUCTION (according to the suggestion of the reviewer #3).

      Line150. Described previously in.

      Done (Line 136).

      Line500. Check whether it is better to use the word acclimatization (Coordinated response to several simultaneous stressors) in exchange of acclimation

      We have revised it according to this suggestion (Line 481).

      Fig.4C Drought

      Done (Line 761).

      Reviewer #3 (Public Review):

      Summary:

      In this paper, Ruan et al. studied the long-term impact of warming and altered precipitations on the composition and growth of the soil microbial community. The researchers adopted an experimental approach to assess the impact of climate change on microbial diversity and functionality. This study was carried out within a controlled environment, wherein two primary factors were assessed: temperature (in two distinct levels) and humidity (across three different levels). These factors were manipulated in a full factorial design, resulting in a total of six treatments. This experimental setup was maintained for ten years. To analyze the active microbial community, the researchers employed a technique involving the incorporation of radiolabeled water into biomolecules (particularly DNA) through quantitative stable isotope probing. This allowed for the tracking of the active fraction of microbes, accomplished via isopycnic centrifugation, followed by Illumina sequencing of the denser fraction. This study was followed by a series of statistical analysis to identify the impact of these two variables on the whole community and specific taxonomic groups. The full factorial design arrangement enabled the researchers to discern both individual contributions as well as potential interactions among the variables

      Strengths:

      This work presents a timely study that assesses in a controlled fashion the potential impact of global warming and altered precipitations on microbial populations. The experimental setup, experimental approach and data analysis seem to be overall solid. I consider the paper of high interest for the whole community as it provides a baseline to the assessment of global warming on microbial diversity.

      Thanks for the encouragement and positive comments.

      Weaknesses:

      While taxonomic information is interesting, it would have been highly valuable to include transcriptomics data as well. This would allow us to understand what active pathways become enriched under warming and altered precipitations. Non-metabolic OTUs hold significance as well. The authors could have potentially described these non-incorporators and derived hypotheses from the gathered information. The work would have benefited from using more biological replicates of each treatment.

      Thanks for the valuable suggestions.

      (1) Metatranscriptomics can assess the functional profiles of the community, but it has relatively low sensitivity to rare species, which is difficult to correlate the function pathways with the assignment to the numerous active taxa identified by qSIP. Additionally, due to the low DNA concentration, most fractionated samples are difficult to construct sequencing libraries, while amplicon based sequencing analyses were allowed. This study therefore focused on active microbial taxa and their growth in response to multifactorial climate change. We have added the prospect in DISCUSSION, that is “This suggests the development of methods combining qSIP with metagenomes and metatranscriptomes to assess the functional shifts of active microorganisms under global change scenarios” (Line 542-544).

      (2) 18O-qSIP can identify the growing microbial species (i.e., 18O incorporators) in the environment rather than metabolically active taxa. These non-incorporators in our study were likely to be metabolically active, i.e., maintaining life activities without reproduction, or recently deceased (Blazewicz et al., 2013). Therefore, it is hard to distinguish whether these non-incorporators possess metabolic activity.

      (3) Agreed. The qSIP experiments involve the use of isotopes and the sequencing of a large number of DNA samples (90 samples per biological replicate in this study). Considering its high cost, we selected three replicates for analysis. We have explained this issue in MATERIALS AND METHODS, that is “Considering the cost of qSIP experiment (i.e., the use of isotopes and the sequencing of a large number of DNA samples), we randomly selected three out of the six plots, serving as three replicates for each treatment” (Line 154-157).

      Reference:

      Nuccio, E.E., Starr, E., Karaoz, U. et al. (2020) Niche differentiation is spatially and temporally regulated in the rhizosphere. ISME J 14, 999–1014.

      Blazewicz, S.J., Barnard, R.L., Daly, R.A., Firestone, M.K (2013). Evaluating rRNA as an indicator of microbial activity in environmental communities: limitations and uses. The ISME Journal, 7, 2061–2068.

      Reviewer #3 (Recommendations For The Authors):

      Major comments:

      The manuscript should be written in a clearer way. The language should be more direct, so the message is conveyed faster and clearer. Some sentences, for instance, could be shortened or re-organized. Below, you will find some examples.

      We have rewritten the sentences to make the manuscript clearer. Below are some, but not all, examples that have been revised:

      (1) Deleted “(reduced precipitation, hereafter ‘drought’, or enhanced precipitation, hereafter ‘wet’)” in INTRODUCTION;

      (2) Deleted “Controlled experiments simulating climate change have investigated changes in microbial community composition as measured by shifts in the relative abundances (Evans & Wallenstein, 2014; Barnard et al., 2015). However, changes in relative abundances may be poor indicators of population responses to environmental change in some cases (Blazewicz et al., 2020). Another challenge is the presence of a large number of inactive microbial cells in the soil, which hinders the direct, quantitative measure of the ecological drivers in population dynamics (Fierer, 2017; Lennon & Jones, 2011).” in DISCUSSION;

      (3) Deleted “This result supports our second hypothesis that the interactive effects between warming and altered precipitation on soil microbial growth are not simply additive” in DISCUSSION;

      (4) Deleted “A previous study suggested that multiple global change factors had negative effects on soil microbial diversity (Yang et al., 2021)” in DISCUSSION;

      (5) Revised “A meta‐analysis of experimental manipulation revealed that the combined effects of different climate factors were usually less than expected additive effects, revealing antagonistic interactions on soil C storage and nutrient cycling processes (Dieleman et al., 2012; Wu et al., 2011). Moreover, two experimental studies on N cycling and net primary productivity demonstrated that the majority of interactions among multiple factors are antagonistic rather than additive or synergistic, thereby dampening the net effects (Larsen et al., 2011; Shaw et al., 2002)” to “A range of ecosystem processes have been revealed to be potentially subject to antagonistic interactions between climate factors, for instance, net primary productivity (Shaw et al., 2002), soil C storage and nutrient cycling processes (Dieleman et al., 2012; Wu et al., 2011; Larsen et al., 2011)” in DISCUSSION (Line 499-503);

      (6) Revised “Previous evidences suggest that warming has a negative impact on soil carbon pools (Jansson & Hofmockel, 2020; Purcell et al., 2022). During the first phase of soil warming (~ 10 years), microbial activity increased, resulting in rapid soil carbon mineralization and respiration (Melillo et al., 2017)” to “Previous evidences suggest that warming has a negative impact on soil carbon pools (Jansson & Hofmockel, 2020; Purcell et al., 2022), mainly because of the rapid soil carbon mineralization and respiration (Melillo et al., 2017)” in DISCUSSION (Line 464-466).

      I'm curious about why, even though there were six replicates of the experiment, only three samples were collected for analysis. Metagenomic analyses tend to display high variability.

      The qSIP experiments involve the use of isotopes and the sequencing of a large number of DNA samples (90 samples per biological replicate in this study). Considering its high cost, we selected three replicates for analysis..

      In Fig. 3A, the absolute growth rates (16S copies/d*g) are shown. How do you know that the efficiency of DNA extraction was similar across all treatments and therefore the absolute numbers are comparable?

      To avoid differences in extraction efficiency caused by experimental procedures, all DNA samples were extracted by the same person (the first author) within 2-3 hours, and a unifying procedure of cell lysis and DNA extraction was used, i.e., the mechanical cell destruction was attained by multi-size beads beating at 6 m s-1 for 40 s, and then FastDNA™ SPIN Kit for Soil (MP Biomedicals, Cleveland, OH, USA) was used for DNA extraction.

      We have measured the concentration of extracted DNA and found no significant difference between treatments (Table for the response letter).

      Author response table 1.

      Soil DNA concentration in climate change treatments after qSIP incubation (measured by Qubit® DNA HS Assay Kits).

      Values represent mean and standard deviation. T0-P represents the ambient temperature and decreased precipitation; T+-P represents warming and decreased precipitation; T0cP represents ambient temperature and precipitation; T+cP represents warming and ambient precipitation; T0+P represents ambient temperature and enhanced precipitation; T++P represents warming and enhanced precipitation. The results of ANOVA indicated no significant difference of extracted DNA concentration between treatments (p > 0.05).

      We have introduced the caveat in the DISCUSSION, that is “Note that the experimental parameters such as DNA extraction and PCR amplification efficiencies also have significant effects on the accuracy of growth assessment. This alerts the need to standardize experimental practices to ensure more realistic and reliable results” (Line 544-547).

      Line 96-99 and 121-124: "Hypotheses are typically placed at the end of the final paragraph in the Introduction section. It is advisable to relocate them there and provide a clearer description of the paper's main goal."

      We have relocated the hypotheses at the end of INTRODUCTION, and the main goal of this study, that is “The goal of current study is to comprehensively estimate taxon-specific growth responses of soil bacteria following a decade of warming and altered precipitation manipulation on the alpine grassland of the Tibetan Plateau, by using the 18O-quantitative stable isotope probing (18O-qSIP)” (Line 112-115).

      Line 399: Although you describe the classification among antagonistic interactions in the Methods section, I think you should describe this in further detail here. Can you clarify how you carried out this categorization and how these results were interpreted considering the phylogenetic classification.

      We have added the description of antagonistic interactions, that is “The interaction type of T × P on the growth of ~70% incorporators was antagonistic (i.e., the combined effect size is smaller than the additive expectation) (Fig. 4C)” (Line 388-390).

      The interaction types between factors can be classified into three categories: additive, synergistic and antagonistic. Additive interactions are those in which the combined effect size of factors is equal to the sum of the single effects of the factors (i.e., additive expectation, Fig. 1B). Synergistic interactions refer to the effect size was larger than the additive expectation by the combined manipulation of factors. On the contrary, antagonistic interactions refer to the combined effect size of factors is smaller than the additive expectation. In this study, the antagonistic interactions were further divided into three sub-categories: weak antagonistic interaction, strong antagonistic interaction, and neutralizing effect (Fig. 1B). The weak antagonistic interaction refers to the combined effect size smaller than the additive expectation and larger than any of the single factor effects. The strong antagonistic interaction refers to that the combined effect size is smaller than any of the single factor effects but larger than 0. The neutralizing effect refers to that the combined effect size is equal to 0, implying that the effects of different factors cancel each other out.

      Methodologically, the single and combined effects of two climate factors and their interaction effects were calculated by the natural logarithm of response ratio (lnRR) and Hedges’ d, respectively (Yue et al., 2017).

      We have added the result interpretation about the phylogenetic distribution patterns of incorporators, that is “The degree of phylogenetic relatedness can indicate the processes that influenced community assembly, like the extent a community is shaped by environmental filtering (clustered by phylogeny) or competitive interactions (life strategy is phylogenetically random distribution) (Evans & Wallenstein, 2014; Webb et al., 2002).The results showed that the incorporators whose growth was influenced by the antagonistic interaction of T × P showed significant phylogenetic relatedness, indicating the occurrence of taxa more likely shaped by environment filtering (i.e., selection pressure caused by changes in temperature and moisture conditions). In contrast, the growing taxa affected by synergistic interactions of T × P showed random phylogenetic distributions (Table S1), which may be explained by competition between taxa with similar eco-physiological traits or changes in genotypes (possibly through horizontal gene transfer) (Evans & Wallenstein, 2014). We also found that the extent of phylogenetic relatedness to which taxa groups of T × P interaction types varied by climate scenarios, suggesting that different climate history processes influenced the ways bacteria survive temperature and moisture stress” (Line 515-529).

      Reference:

      Evans, S.E. & Wallenstein, M.D. (2014). Climate change alters ecological strategies of soil bacteria. Ecology Letters, 17, 155-164.

      Webb, C.O., Ackerly, D.D., McPeek, M.A. & Donoghue, M.J. (2002). Phylogenies and Community Ecology. Annual Review of Ecology and Systematics, 33, 475-505.

      Yue, K., Fornara, D.A., Yang, W., Peng, Y., Peng, C., Liu, Z. et al. (2017). Influence of multiple global change drivers on terrestrial carbon storage: additive effects are common. Ecology Letters, 20, 663-672.

      Line 407-8: What do you mean with "...clustered at the phylogenetic branches" and Line 410: "cluster near the tips of the phylogenetic tree". Can you please clarify?

      Sorry for the unclear statement. We have added the explanation of NTI, that is “Nearest taxon index (NTI) was used to determine whether the species in a particular growth response are more phylogenetically related to one another than to other species (i.e., close or clustering on phylogenetic tree). NTI is an indicator of the extent of terminal clustering, or clustering near the tips of the tree (Evans & Wallenstein, 2014; Webb et al., 2002)” (Line 397-401).

      Reference:

      Evans, S.E. & Wallenstein, M.D. (2014). Climate change alters ecological strategies of soil bacteria. Ecology Letters, 17, 155-164.

      Webb, C.O., Ackerly, D.D., McPeek, M.A. & Donoghue, M.J. (2002). Phylogenies and Community Ecology. Annual Review of Ecology and Systematics, 33, 475-505.

      Could you provide some info about the biochemistry of the incorporation of heavy water into DNA molecules? What specific enzymes are typically involved?

      Due to the low DNA concentration in most fractionated samples (less than 10 ng/μL, measured by Qubit DNA HS Assay Kits), only amplicon based sequencing analyses were allowed. This study therefore focused only on active microbial taxa and their growth in response to multifactorial climate change.

      What might be the impact of soil desiccation on bacterial survival and subsequent water uptake?

      Slow dehydration and air drying of soil is a very common phenomenon in nature (Koch et al., 2018). In this process, microorganisms will reduce metabolism, and shift towards a potentially active state (Blagodatskaya and Kuzyakov, 2013). A previous study suggested that the potentially active microbial population permanently existing in soil between the active and dormant physiological states. Even under long-term starvation the potentially active microorganisms maintain ‘physiological alertness’ to be ready to occasional substrate input (Blagodatskaya and Kuzyakov, 2013). These microorganisms are important participants in the biogeochemical cycle is the focus of this study.

      Replacing the environmental water in the soil with 18O-labelled water is a typical practice for qSIP studies (Hungate et al. 2015; Koch et al., 2018). This process may cause disturbance to the microbial community. In this study, the soil samples were placed in a thermostatic incubator (14℃ and 16℃), rather than air-drying at 25℃ (as used in most studies). The incubation temperature is relatively low (compared to 25℃) and there is no violent air convection in the incubator, resulting slower evaporation and no significant discoloration caused by severe soil dehydration after 48 h. The process of soil drying in this study simulated the natural phenomenon, i.e., slow water loss in soil.

      We have added the description in MATERIALS AND METHODS, that is “There is no violent air convection in the incubator and the incubation temperature is relatively low (compared to 25℃ used in previous studies), resulting slower evaporation and no significant discoloration caused by severe soil dehydration after 48 h” (Line 171-174).

      Reference:

      Blagodatskaya, E. & Kuzyakov, Y. (2013) Active microorganisms in soil: Critical review of estimation criteria and approaches. Soil Biology and Biochemistry, 67, 192-211.

      Hungate, B., Mau, R., Schwartz, E., Caporaso, J., Dijkstra, P., Van Gestel, N. et al. (2015). Quantitative microbial ecology through stable isotope probing. Applied and Environmental Microbiology, 81, 7570-7581.

      Koch, B., McHugh, T., Hayer, M., Schwartz, E., Blazewicz, S., Dijkstra, P. et al. (2018). Estimating taxon-specific population dynamics in diverse microbial communities. Ecosphere, 9, e02090.

      The analysis of the 180 incorporators is interesting as it defines what microbes are metabolically active and hence growing under the different conditions tested. Should not be worth to analyze the non-incorporators? Is it possible to identify a pattern to generate a hypothesis of why they are metabolically inactive based on this information? In the Methods section, the authors state that they identified a total of 6,938 OTUs, of which only 1,373 were found to be incorporators.

      Microbes exist in a range of metabolic states: growing, active (non-growth), dormant and recently deceased (Blazewicz et al., 2013), and there is still a lack of clear threshold for their identification. 18O-DNA qSIP can identified the growing microbial species (i.e., 18O incorporators) rather than all metabolic active taxa, because some cells are measurably metabolizing (catabolic and/or anabolic processes) without reproduction. Therefore, the non-incorporators in our study may be metabolically active, or not (recently deceased microorganisms). This study focuses on the growing microorganisms identified by 18O-qSIP.

      In this study, ~20% microbial taxa (1,373/6,938) were identified as 18O incorporators. Microorganisms in soils suffer from resource and energy constraints frequently (Blagodatskaya and Kuzyakov, 2013). The energy requirements of species in the growing state are much higher (~30 fold) than those in the non-growing state, so the percentage of growing bacterial taxa in soil tends to be low.

      Reference:

      Blazewicz, S.J., Barnard, R.L., Daly, R.A., Firestone, M.K (2013). Evaluating rRNA as an indicator of microbial activity in environmental communities: limitations and uses. The ISME Journal, 7, 2061–2068.

      Blagodatskaya, E. & Kuzyakov, Y. (2013) Active microorganisms in soil: Critical review of estimation criteria and approaches. Soil Biology and Biochemistry, 67, 192-211.

      Minor comments:

      Fig. 3A and 3B. Please show the results of the multiple comparisons.

      Done.

      Author response image 5.

      Bacterial growth responses to climate change and the interaction types between warming and altered precipitation. The growth rates (A), and responses (LnRR) of soil bacteria to warming and altered precipitation (B) at the whole community level. The growth rates (C), and responses of the dominant bacterial phyla (D) had similar trends with that of the whole community. Values represent mean and the error bars represent standard deviation. Different letters indicate significant differences between climate treatments.

      Fig. 4. This figure should be self-explanatory. This diagram is challenging to understand.

      We have revised Fig. 4 to improve clarity.

      Author response image 6.

      The growth responses and phylogenetic relationship of incorporators subjected to different interaction types under two climate scenarios. A phylogenetic tree of all incorporators observed in the grassland soils (A). The inner heatmap represents the single and combined factor effects of climate factors on species growth, by comparing with the growth rates in T0nP. The outer heatmap represents the interaction types between warming and altered precipitation under two climate change scenarios. The proportions of positive or negative responses in species growth to single and combined manipulation of climate factors by summarizing the data from the inner heatmap (B). The proportions of species growth influenced by different interaction types of T × P by summarizing the data from the outer heatmap (C).

      Fig. 4. It says "Dorought" instead of "drought"

      Done (Line 760).

      Line 109: "relieves" instead of "relieved"

      Done (Line 102).

      Line 129: Should be: "We classified the interaction types as additive, synergistic, antagonistic, null and neutralizing."

      Done (Line 117).

      Line 233: How were the 16S rRNA sequences from each density fraction analyzed?

      (1) Raw sequencing data processing:

      The raw 16S rRNA gene sequences of each density fraction were quality-filtered using the USEARCH v.11.0 (Edgar, 2010). The paired-end sequences were merged and quality filtered with “fastq_mergepairs” and “fastq_filter” commands, respectively. Sequences < 370 bp and total expected errors > 0.5 were removed. Next, “fastx_uniques” command was implemented to identify the unique sequences. Subsequently, high-quality sequences were clustered into operational taxonomic units (OTUs) with “cluster_otus” commandat a 97% identity threshold, and the most abundant sequence from each OTU was selected as a representative sequence. The taxonomic affiliation of the representative sequence was determined using the RDP classifier (Wang et al., 2007).

      (2) qSIP calculation:

      Sequencing data reflects the relative abundance of taxa in community. We multiply the OTU’s relative abundance (acquisition by sequencing) and the number of 16S rRNA gene copies (acquisition by qPCR) to obtain the number of gene copies per OTU in each fraction. Then, the proportion of gene copies of a specific OTU of each fraction relative to the total amount of gene copies in one sample was calculated and used as a weight value for further calculation of the average weighted buoyant density (the critical parameter for assessing microbial growth).

      Line 366: "Three single-factor ... between warming and altered precipitation" -> "The individual impact of warming, drought, and wet conditions resulted in the most substantial negative effects on bacterial growth compared with the effects of warming x drought and warming x wet. A result that illustrates the negative interactions between warming and modified precipitations patterns."

      Done (Line 365-368).

      Line 376: "Similar with the result of whole growth of bacteria community, the growth responses of the major bacterial phyla were also negatively influenced by single climate factors". This sentence is hard to read. Maybe something like this: "Growth of the major bacterial phyla was also negatively influenced by the individual climate factors".

      Done (Line 371-372).

      Line 383: "In particular, the effects of wet and warming neutralized each other, resulting the net effects became zero on the growth rates of the phyla Actinobacteria and Bacteroidetes". "In Actinobacteria and Bacteroidetes, the effect of wet and warming neutralized each other, as the combined effect of these two factors had no effect on growth".

      Done (Line 377-379).

      Line 390: "The individual warming treatment (T+nP) reduced the growth rates of 75% incorporators..." "Warming (T+nP) reduced the growth of 75% of the taxonomic groups, which was followed by drought and wet.

      Done (Line 384-385).

      Line 392: "The combined manipulations of warming and altered precipitation lowered the percentages of incorporators with negative responses compared with single factor manipulation, especially warming and enhanced precipitation manipulation" -> "Warming x drought and warming x wet had a smaller impact on the growth of incorporators, compared with single effects."

      Done (Line 385-387).

      Line 468. This sentence "To the best ..." is not necessary.

      We have deleted this sentence.

      Line 476. Is it really "synthesis" the word you want to use?

      We have deleted this sentence.

      Line 477. Maybe should written like this: "Consistent with our findings, a recent experimental study demonstrated that 15 years of warming reduced the growth rate of soil bacteria in a montane meadow in northern Arizona."

      Done (Line 459-461).

      Line 490 and 502. Consider using "however" only once in a paragraph.

      We have deleted the second “however” (Line 483).

      Line 555-559. Based on genomic data you cannot predict the functional role of microbes in the environment. These sentences are speculative. Please, consider using less strong affirmations and focus more on the pathways that are enriched in the incorporators.

      Agreed. We have deleted this part of content.

    1. Author Response

      The following is the authors’ response to the original reviews.

      This important study shows that two methods of sleep induction in the fly, optogenetically activation of the dorsal fan-shaped body (which is rapidly reversible and maintains a neuronal activity signature similar to wakefulness), and Gaboxadol-induced sleep (which shuts down neuronal activity), produce distinct forms of sleep and have different effects on brain-wide neural activity. The majority of the conclusions of the paper are supported by compelling data, but the evidence supporting the claim that the two interventions trigger distinct transcriptional responses is incomplete.

      Thank you for the helpful and detailed reviews. We feel that these have improved the manuscript considerably, and hopefully the additional figures in this Reply letter will help further convince our readers.

      Public Review

      In this study, Anthoney and coworkers continue an important, unique, and technologically innovative line of inquiry from the van Swinderen lab aimed at furthering our understanding of the different sleep stages that may exist in Drosophila. Here, they compare the physiological and transcriptional hallmarks of sleep that have been induced by two distinct means, a pharmacological block of GABA signaling and optogenetic activation of dorsal fan-shaped-body neurons. They first employ an incredibly impressive fly-on-the-ball 2-photon functional imaging setup to monitor neural activity during these interventions, and then perform bulk RNA sequencing of fly brains at different stages. These transcriptomic analyses leads them to (a) knocking out nicotinic acetyl-choline receptor subunits and (b) knocking down AkhR throughout the fly brain testing the impact of these genetic interventions on sleep behaviors in flies. Based on this work, the authors present evidence that optogenetically and pharmacologically induced sleep produces highly distinct brain-wide effects on physiology and transcription. The study is of significant interest, is easy to read, and the figures are mostly informative. However there are features of the experimental design and the interpretation of results that diminish enthusiasm.

      a- Conditions under which sleep is induced for behavioral vs neural and transcriptional studies

      1- There is a major conceptual concern regarding the relationships between the physiological and transcriptomic effects of optogenetic and pharmacological sleep promotion, and the effects that these manipulations have on sleep behavior. The authors show that these two means of sleep-induction produce remarkably distinct physiological and transcriptional responses, however, they also show that they produce highly similar effects on sleep behavior, causing an increase in sleep through increases in the duration of sleep bouts. If dFB neurons were promoting active sleep, the sleep it produces should be more fragmented than the sleep induced by the drug, because the latter is supposed to produce quiet sleep. Yet both manipulations seem to be biasing behavior toward quiet sleep.

      This is a correct observation, which is already evident in our sleep architecture data (Figure 2E-H): chronic optogenetic sleep induction promotes longer sleep bouts that are similar in structure (bout number vs bout duration) to those produced by THIP feeding. Since our plots in Figure 2E-H follow the 5min sleep criterion cutoff, upon the Reviewer’s advice we re-analyzed our optogenetic experiments for short (1-5min) sleep. These are graphed below in Author response image 1. As can be seen, and as suspected by the Reviewer, the optogenetic manipulation does not increase the total amount of short sleep; indeed, it decreases it compared to baseline (these are for the exact same data as in Figure 2). Optogenetic sleep induction does not create a bunch of short sleep bouts.

      Author response image 1.

      Short sleep in optogenetic experiments. A. Average baseline (±SEM) 1-5min sleep across a day and night. B. Average (±SEM) 1-5min sleep in optogenenetically-activated flies, across a day and night.

      We agree with the reviewer that this observation might seem inconsistent with the idea that optogenetic activation promotes active sleep, and that short sleep is active sleep. However, it does not necessarily follow that optogenetic activation has to produce short sleep. Indeed, we know from our brain imaging data (and the associated behavioral analysis) that active sleep will persist for as long as we induce it with red light. While we have not induced it for longer than 15 minutes (Tainton-Heap et al, Current Biology, 2021; Troup et al, J. of Neuroscience, 2023), this is already clearly longer than a <5min sleep bout. So our interpretation is that the longer sleep bouts induced by optogenetic activation are prolonged active sleep, rather than quiet sleep. In other words, this artificial sleep manipulation induces prolonged active sleep, rather than many short sleep bouts. This is of course different than what happens during spontaneous sleep. We have tried to be clearer about sleep bout durations in the revised manuscript (e.g., the new Figure 3), and we now admit early in the results (lines 376-380) that that we don’t know what optogenetic activation looks like in the fly brain beyond 15 minutes.

      2- The authors show that the pharmacological block of GABA signaling and the optogenetic activation of dorsal fan-shaped-body neurons cause different responses on brain activity. Based on these recordings and the behavioral and brain transcriptomic data they then claim that these responses correspond to different sleep states and are associated with the expression and repression of a different constellation of genes. Nevertheless, neural activity in animals was recorded following short stimulations whereas behavioral and transcriptomic data were obtained following chronic stimulation. In this regard, it would be interesting to determine how the 12-hour pharmacological intervention they employed for their transcriptomic analysis changes neural activity throughout the brain - 12 hours will likely be too long for the open-cuticle preps, but an in-between time-point (e.g. 1h) would probably be equally informative.

      The longest we’ve imaged brain activity for optogenetic sleep induction is 15 minutes, as discussed above. We see no changes in activity across this time, which would normally have led to a quiet sleep stage in spontaneous sleep recordings. Whole-brain imaging after 10 hours of optogenetic sleep induction (our RNA collection timepoint) is not realistic, and even 1 hour is difficult. We have however conducted overnight electrophysiological recordings (with multichannel silicon probes), where we activated the same R23E10 neurons for successive 20-minute bouts (alternating with 20min of no red light). We are preparing this work for publication (Van De Poll, et al). We see no evidence of optogenetic activation of this circuit ever producing anything resembling quiet sleep. Since we are not in a position to provide this new electrophysiological data in the current study, we are careful to clarify that we have not investigated what brain imaging looks like after chronic optogenetic activation (lines 376-380). We are showing through diverse lines of evidence that what is called sleep can look different in flies.

      b- Efficiency of THIP treatment under different conditions

      1- There are no data to quantify how THIP alters food consumption. It is evident that flies consume it otherwise they would not show increased sleep. However, they may consume different amounts of food overall than the minus THIP controls. This might have an influence on the animal's metabolism, which could at least explain the fact that metabolism-related genes are regulated (Figure 5). Therefore, in the current state, it is not possible to be certain that gene regulation events measured in this experiment are solely due to THIP effects on sleep.

      We have two arguments against this reasonable criticism. First, as discussed above, the optogenetic flies are sleeping at least as much as the THIP-fed flies, so in principle they also might be feeding less. But we see no metabolic gene downregulation in the optogenetic dataset. We include this counterargument in the discussion (lines 752-756). Then, together with our co-author Paul Shaw we have shown that THIP-fed flies are not eating less compared to controls (Dissel et al, Current Biology, 2015), by tracking dye consumption. We show those results again below in Author response image 2 to support our reasoning that feeding is not an issue.

      Author response image 2.

      Flies were fed blue dye in their food while being sleep deprived (SD), or while being induced to sleep with 0.1mg/ml THIP in their food, or both. Dye consumption was measured in triplicate for pooled groups of 16 flies. Average absorbance at 625nm (±stan dev) is shown. Experiments were not significantly different (ANOVA of means).

      2- A similar problem exists in the sleep deprivation experiments. If flies are snapped every 20 seconds, they may not have the freedom to consume appropriate amounts of food, and therefore their consumption of THIP or ATR may be smaller than in non-sleep deprived controls. Thus, it would be crucial to know whether the flies that are sleep-deprived (i.e. shaken every 20 seconds for 12 hours) actually consume comparable amounts of food (and therefore THIP) as those that are undisturbed. If not, then perhaps the transcriptional differences between the two groups are not sleep-specific, but instead reflect varying degrees of exposure to THIP.

      Please see our response to the similar critique above, and how Figure R2 addresses this concern.

      3- The authors should further discuss the slow action of THIP perfusion vs dFB activation, especially as flies only seem to fall asleep several minutes after THIP is being washed away. Is it a technical artifact? If not, it may not be unreasonable to hypothesize that THIP, at the concentration used, could prevent flies from falling asleep, and that its removal may lower the concentration to a point that allows its sleep-promoting action. The authors could easily test this by extending THIP treatment for another 4-5 minutes.

      The reviewer is partially correct in suggesting a technical artifact: THIP does not get washed away immediately after 5min of perfusion. The drip system we employ means that THIP concentration will slowly increase to the maximum concentration of 0.2mg/ml, and then slowly get diluted away at a rate of 1.25ml/minute (this is all in the Methods). In a previous study (Yap et al, Nature Communications, 2017) we used this exact same perfusion procedure to test a range of THIP concentrations, and settled on 0.2mg/ml as the lowest that reliably induced quiet sleep within 5 minutes. Higher concentrations induced quiet sleep faster, so the alternate explanation proposed by the Reviewer is not supported. We feel that our previous electrophysiological study provided the necessary groundwork for using the same approach and dosage here for our whole-brain imaging readout.

      c- Comments regarding the behavioral assays

      1- L319-322: the authors conclude that dFB stimulation and THIP consumption have similar behavioral effects on sleep. However, this is inaccurate as in Figure S1 they explain that one increases bout number in both day and night and the other one only during the day.

      We have now added a caveat about night bout architecture being different (lines 353-356). Figure S1 is now Figure 3.

      2- The behavioral definitions used for active and quiet sleep do not fit well with strong evidence that deep sleep (defined by lowered metabolic rates) is probably most closely associated with bouts of inactivity that are much longer than the >5min duration used here, i.e., probably 30min and longer (Stahl et al. 2017 Sleep 40: zsx084). Given that the authors are providing evidence that quiet sleep is correlated with changes in the expression of metabolism related genes, they should at least discuss the fact that reductions in metabolism have been shown to occur after relatively long bouts of inactivity and might reconsider their behavioral sleep analysis (i.e., their criteria for sleep state) with this in mind.

      Interestingly, induced sleep bout durations are on average longer for the optogenetic manipulation (40min vs 25min); this was evident in Figure S1C vs S1F (now Figure 3). So as discussed above, this provides a counterargument for sleep bout duration alone being indicative of metabolic processes associated with quiet sleep: the optogenetic dataset did not uncover metabolic-related pathways as relevant to that sleep manipulation. We refer to Stahl et al, Sleep, 2017, in our discussion (lines 748-750), making exactly this point about metabolic rates being decreased in longer sleep bouts, and flowing up with our observation that optogenetic flies sleep just as much, and their bouts are actually longer. So clearly different processes must be involved.

      d- Comments regarding the recordings of neuronal activity

      1- There is an additional concern regarding the proposed active and quiet sleep states that rest at the heart of this study. Here these two states in the fly are compared to the REM and NREM sleep states observed in mammals and the parallels between active fly sleep and REM and quiet fly sleep and NREM provide the framework for the study. The establishment of such parallel sleep states in the fly is highly significant and identifying the physiological and molecular correlates of distinct sleep stages in the fly is of critical importance to the field. However, the proposal that the dorsal fan shaped body (dFB) neurons promote active sleep runs counter to the prevailing model that these neurons act as a major site of sleep homeostasis. If quiet sleep were akin to NREM, wouldn't we expect the major site of sleep homeostasis in the brain to promote it? Furthermore, the authors state that the effects of dFB neuron excitation on transcription have "almost no overlap" (line 500) with the transcriptomic effects of sleep deprivation (Supplementary Table 3), which is not what would be expected if dFB neurons are tracking sleep pressure and promoting sleep, as suggested by a growing body of convergent work summarized on page four of the manuscript. Wouldn't the 10h excitation of the dFB neurons be predicted to mimic the effects of sleep deprivation if these neurons "...serve as the discharge circuit for the insect's sleep homeostat..." (line 60)? Shouldn't their prolonged excitation produce an artificial increase in sleep drive (even during sleep) that would favor deep, restorative sleep? How do the authors interpret their results with regard to the current prevailing model that dFB neurons act as a major site of sleep homeostasis? This study could be seen as evidence against it, but the authors do not discuss this in their Discussion.

      These are all excellent and thoughtful points, which have made us re-think parts of our discussion. First off, the potential comparison with REM and NREM is entirely speculative, and we have tried to make that more obvious in introduction) and the discussion (e.g, see lines 43, 708, 818). The evidence that the FB neurons (and maybe others) are involved in the homeostatic regulation of sleep is well-supported in the literature, so that part of the discussion holds. However, we concede that the timing of our sleep manipulations could benefit from more explanation. We conducted these during the flies’ subjective day, after the animals had presumably had a good night’s sleep. This means that we induced either kind of sleep for 10 daytime hours, which presumably replaced whatever behavioural states would ‘naturally’ be happening during the day. Female flies sleep less during the day than at night, and we have shown in previous work that daytime sleep quality is different than night-time sleep (van Alphen et al, Journal of Neuroscience, 2013), leading us to suggest that most ‘deep’ or quiet sleep happens at night, for flies. Following this reasoning, daytime optogenetic activation might not be depriving flies of much quiet sleep, or accumulating a deep sleep drive as the Reviewer proposes. Rather, both induced sleep manipulations could be providing 10 hours of either kind of sleep that the flies don’t really ‘need’. Why did we design it this way? Firstly, we were interested in simply asking what these chronic sleep manipulations do to gene expression in rested flies, and how they might be similar or different. We focussed on daytime manipulations to avoid precisely the confound of sleep pressure, and also because we observed red-light artifacts at night for our optogenetic experiments (which we reported). Our sleep deprivation strategy was designed specifically as a control for the THIP (Gaboxadol) experiments, to control for non-sleep related effects of the drug (see below our rationale for why this was less crucial for the optogenetic experiments). In conclusion, we had a logical rationale for how the experiments were done, centred on the straightforward question of whether these two different approaches to sleep induction were having similar effects in well-rested flies. In retrospect, we were not anticipating the Reviewer’s thoughtful logic regarding the dFB’s potential role in also regulating deep sleep homeostasis. We now provide some discussion along these lines to make readers aware of this line of reasoning, as well as our rationale for why prolonged optogenetic sleep induction was not sleep-depriving (lines 768-777).

      2- Regarding the physiological effects of Gaboxadol, to what extent is the quieting induced by this drug reminiscent of physiology of the brains of flies spontaneously meeting the behavioral criterion for quiet sleep? Given the relatively high dose of the drug being delivered to the de-sheathed brain in the imaging experiments (at least when compared to the dose used in the fly food), one worries that the authors may be inducing a highly abnormal brain state that might bear very little resemblance to the deeply sleeping brain under normal conditions. As the authors acknowledge, it is difficult to compare these two situations. Comparing the physiological state of brains put to sleep by Gaboxadol and brains that have spontaneously entered a deep sleep state therefore seems critical.

      As discussed above, our Gaboxadol (THIP) perfusion concentration (0.2mg/ml) was the minimal dosage that effectively induced sleep within 5 minutes, based upon previously published work (Yap et al, Nature Communications, 2017). Lower concentrations were unreliable, with some never inducing sleep at all. Comparisons with feeding THIP are tenuous, and we make that clear in our discussion (lines 731-735). Nevertheless, the Reviewer makes an excellent point about comparisons with spontaneous ‘quiet’ sleep. Here, we feel well supported (please see Author response image 3 below, comparing THIP-induced sleep (this work, B) and spontaneous sleep (A) from previous study). In our previous study (Tainton-Heap et al, 2021) we showed that neural activity and connectivity decreases during spontaneous quiet sleep. This is what we also see with THIP perfusion. In contrast, in Troup et al, J. of Neuroscience (2023) we confirm that neither neural activity nor connectivity changes during optogenetic R23E10 activation, and general anesthesia – unlike THIP – does NOT produce a quiet brain state. Our finding that THIP effects are nothing like general anesthesia (at the level of brain activity levels) suggests a physiological sleep state closer to spontaneous quiet sleep. We elaborate on this important observation in our results, also pointing to crucial differences with general anesthesia (lines 411-415).

      Author response image 3.

      THIP-induced sleep resembles quiet spontaneous sleep. A. Calcium imaging data from spontaneously sleeping flies, taken from Tainton-Heap et al, 2021. Left, percent neurons active; right, mean degree, a measure connectivity among active neurons. Both measures decrease during later stages of sleep. B. Calcium imaging data from flies induced to sleep with 5min of 0.2mg/ml THIP perfusion (this study). Left, percent neurons active; right, mean degree. Both measures are significantly decreased, resembling the later stages of spontaneous sleep, which we have termed ‘quiet sleep. Hence THIP-induced sleep resembles quiet sleep. Note that the genetic background is different in A and B, hence the different baseline activity levels.

      3- There are some issues with Figure 3, in particular 3C-D. It is not clear whether these panels show representative traces or an average, however both the baseline activity and fluorescence are different between C and D, in particular in their amplitude. Therefore, it is difficult to attribute the differences between C and D to the stimulation itself or to the previously different baseline. In addition, the fact that flies with dFB activation seem to keep a basal level of locomotor activity whereas THIP-treated ones don't is quite striking, however it is not being discussed. Finally, the authors claim that the flies eventually wake up from THIP-induced sleep (L360-361), however there are no data to support this statement.

      These are representative traces, which is a way of showing the raw calcium data (Cell ID) so readers can see for themselves that one manipulation silences whereas the other does not – even though flies become inactive for both. The Y-axis scale is standard deviation of the experiment mean. Since THIP decreases neural activity, then the baseline is comparatively higher. Since optogenetic activation does not change average neural activity levels, the baseline is centered on zero. This is an outcome of our analysis method and does not reflect any ‘true’ baseline. We have now clarified this in our figure legend. We now also confess that flies rendered asleep optogenetically can be ‘twitchy’ (line 374). Finally, we show data for 3 flies that were recorded until they woke up. The rest were verified behaviorally, after the experiment. This is now explained in the Methods.

      4- In Figure 4C, it is strange that the SEM is always exactly the same across the whole experiment. Readers should be aware that there might have been an issue when plotting the figure.

      This is not a mistake, the standard errors are just all quite close (between 0.17 and 0.22). This is because of the way we did the analysis, asking how many flies responded to each stimulus event, with incremental levels of responsiveness. This is explained in the Methods. The figure makes the important point of sleep and recovery.

      e- Comments regarding the transcript analyses

      1- General comment: the title of this manuscript is inaccurate - the "transcriptome" commonly refers to the entirety of all transcripts in a cell/tissue/organ/animal (including genes that are not differentially expressed following their interventions), and it is therefore impossible to "engage two non-overlapping transcriptomes" in the same tissue. Perhaps the word "transcriptional programs" or transcriptional profiles" would be more accurate here?

      We thank the Reviewer for this advice and have changed the title as proposed.

      2- Given the sensitivity of transcriptomic methods, there is a significant concern that the optogenetic experiments are not as well controlled as they could be. Given the need for supplemental all-trans retinal (ATR) for functional light gating of channelrhodopsins in the fly, it is convenient to use flies with Gal4-driven opsin that have not been given supplemental ATR as a negative control, particularly as a control for the effects of light. However, there is another critical control to do here. Flies bearing the UAS-opsin responder element but lacking the GAL4 driver and that have been fed ATR are critical for confirming that the observed effects of optogenetic stimulation are indeed caused by the specific excitation of the targeted neurons and not due to leaky opsin expression, or the effect of ATR feeding under light stimulation or some combination of these factors. Given the sensitivity of transcriptomic methods, it would be good to see that the candidate transcripts identified by comparing ATR+ and ATR- R23E10GAL4/UAS-Chrimson flies are also apparent when comparing R23E10GAL4/UAS-Chrimson (ATR+) with UAS-Chrimson (ATR+) alone.

      We have not done these experiments on UAS-Chrimson/+ controls. Like many others in our field, we viewed non-ATR flies as the best controls, because this involves identical genotypes. Since we were however aware that ATR feeding itself could be affect gene expression, we specifically checked for this with our early (1hour) collection timepoint. We only found 26 gene expression differences between ATR and -ATR flies at this early timepoint, compared with 277 for the 10-hour timepoint. We detail this rationale in our results, explaining why this is a convincing control for ATR feeding. If there was leaky opsin expression / activity, this would have been evident in our design. Regarding the cumulative effect of light, this would also have been accounted in our design, as only 1 hour would have elapsed in our first timepoint compared to 10 hours in our second. While the Reviewer is correct in saying that parental controls are called for in many Drosophila experiments, this becomes quickly unmanageable in transcriptomic studies, which is exactly why well-designed +ATR vs -ATR comparisons in the exact same strain are most appropriate. We feel that our 1-hr timepoint mostly addresses this concern.

      3- Figures about qPCR experiments (5G and 6G) are problematic. First, whereas the authors seem satisfied with the 'good correspondence' between their RNA-seq and qPCR results, this is true for only ~9/19 genes in 5G and 2/6 genes in 6G. Whereas discrepancies are not rare between RNA-seq and qPCR, the text in L460-461 and 540-541 is misleading. In addition, it is unclear whether the n=19 in L458 refers to the number of genes tested or the number of replicates. If the qPCR includes replicates, this should be more clearly mentioned, and error bars should be added to the corresponding figures.

      We consider that our qPCR validations were convincing, as they were all mostly changed in the ‘right’ direction. We agree that are some discrepancies, so have modified our language to reflect this. We have also clarified that 19 refers to the number of genes validated by qPCR in that THIP dataset. All qPCRs involved three technical replicates. We prefer to keep these histograms the way they are to convey these simple trends. For complete transparency, we now provide a supplemental Excel worksheet with all of the qPCR data, alongside corresponding RNAseq data and stats for the selected genes (Supplementary Table 9).

      4- There is a lack of error bars for all their RNAseq and qPCR comparisons, which is particularly surprising because the authors went to great lengths and analyzed an applaudably large amount of independent biological replicates, yet the variability observed in the corresponding molecular data is not reported.

      The genes reported in each of our datasets and associated supplemental figures and tables were all significant, as determined by criteria outlined in the Methods. However, we appreciate that readers might want to get a sense of the values and variances involved, as well as access to the entire gene datasets. We now provide all of these as additional ‘sheets’ in our existing supplemental tables (S2-S7), so this should be very easy to navigate and evaluate. In addition to the previously provided lists for significant genes, in the second Excel sheet (‘All genes’) readers will be able to see the data for all 5 replicates, for the significant genes as well as all other ~15,000 genes (listed in alphabetical order). We feel that this will be a helpful resource, because admittedly significance thresholds can still be a little arbitrary and some readers might want to look up ‘their’ genes of interest.

      Comments to authors

      Other comments

      1- Text in L441 & 606 is misleading. According to ref 52, AkhR is involved specifically in starvation-induced sleep loss, and not in general sleep regulation.

      Corrected.

      2- The language used in L568-570 and 573-574 is confusing. The authors should specify that the knock down of cholinergic subunits, rather than the subunits themselves is what causes sleep to increase or decrease.

      Corrected.

      3- The authors' investigation of cholinergic receptor subunits function is very preliminary, and it is difficult to draw any conclusion from what is presented here. In particular, their behavioral data is difficult to reconcile with the RNA-seq data showing overexpression of both short sleep increasing and short sleep decreasing subunits. Without knowing where in the brain these subunits are required for controlling sleep, the data in Figure 7 is difficult to appreciate.

      We have now conducted additional experiments where we specifically knocked down these alpha receptor subunits (all 7 of them) in the R23E10 neurons. This seemed an obvious knockdown location, to determine if any of these subunits regulated activity in the same sleep promoting neurons that were the focus of this study. We found that alpha1 knockdown in these neurons had similar sleep phenotypes, which we believe is an important result. Since this functional localisation is a logical ending for the paper, we have now made it the final figure.

      Suggestions & comments

      1- It would be interesting if the authors could discuss their findings that metabolism genes are downregulated in THIP flies in the context of recent work that showed upregulation of mitochondrial ROS after sleep deprivation (Kempf et al, 2019).

      We now add the Kempf 2019 reference and allude to how those findings could be consistent with ours.

      2- The fact that THIP-induced sleep persists long after THIP removal (Fig 3D) is very intriguing and interesting. This suggests that the drug might trigger a sleep-inducing pathway that can continue on its own without the drug, once activated.

      This is correct, and in stark contrast to the optogenetic manipulation we employ, which does not appear to show such sleep inertia. We have now added a sentence highlighting this interesting difference (lines 394-396).

      3- The authors identify many new genes regulated in response to specific methods for sleep induction. These are all potentially interesting candidates for further studies investigating the molecular basis of sleep. It would be interesting to know which of these genes are already known to display circadian expression patterns.

      By providing all of the gene lists, these are now available to ask questions such as these. We hesitate however to delve into this domain for this work, as our main goal was to compare these two kinds of sleep in flies.

      4- The brain-wide monitoring of neural activity invites a number of very exciting follow-up experiments - most importantly, it would be fascinating to establish, which neurons are active in the different phases the authors describe! Are these neurons that are involved in transmitting external visual stimuli to the central brain? Do they also project into the central complex? They could make use of the large collection of existing driver lines in the fly and they could also exploit the extraordinary knowledge of the connectome and transcriptome of the fly brain.

      Thank you for sharing our enthusiasm for these likely future directions.

      5- The Dalpha2,3,4,6 and 7 Knock-out strains they generate will be a useful reagent for the Drosophila neuroscience community once the efficiency/success of the knock-out has been confirmed by qPCR.

      These knockout strains have all been confirmed by our co-authors Hang Luong, Trent Perry, and Philip Batterham. These knockout confirmations are outlined in publications that we reference (Perry et al, 2021).

      Materials and methods:

      1- This study has employed custom-built apparatus and custom-written code/scripts, but these do not appear to be available to the reader. For the sake of replicability, the authors should make these available.

      The code/scripts are available via the University of Queensland research data management system as described in the Methods, and can be sent by the Lead Contact. The imaging hardware and analysis code are identical to what was described in a previous publication, and available as directed therein (Tainton-Heap et al, 2021).

      2- Also, the authors should give details on the food used to rear their flies. Fly media comes in several common forms and sleep is sensitive to diet.

      This has now been elaborated in the beginning of the Methods.

      3- The light regime used for optogenetic excitation of dFB neurons consists of 12h of uninterrupted bright red LED light. Most optogenetic stimulations consist of pulsed high frequency flashes interlaced with pauses in illumination. Can dFB neurons be driven constitutively with 12 hours of bright light?

      We showed in Tainton-Heap (2021) that 7Hz pulsed red light had exactly the same effect on R23E10/Chrimson readouts as continuous red light, which is why we opted here to provide continuous red light. That optogenetic sleep induction can be driven continuously for 12 hours is evident by our 24-hour sleep profiles. However, we agree that one could question whether sleep quality is similar after 12 hours. To address this, we did an additional experiment where we stimulated the flies hourly, to determine if their behavioural responsiveness to mechanical stimuli changed over the course of continued sleep induction, for both optogenetic and THIP-induced sleep. We present the data below in Author response image 4. As can be seen in these new analyses, while optogenetic sleep induction persists across 12 daytime hours (speed is close to zero throughout), flies do indeed become more responsive later in the day. This could have two different interpretations: either some sleep functions are being satisfied over time, or the activation regime is becoming less effective over time. Either way, these data show that at our 10-hour daytime timepoint, unstimulated flies are still largely inactive, even though their arousal thresholds might have gradually changed; so the uninterrupted red-light regime is still effective. The comparison with THIP is interesting: here there does not seem to be a change in responsiveness over time; the drug just decreases behavioral responsiveness throughout. Together, these experiments support our view that both approaches are sleep-promoting throughout the 12-hour day, although we appreciate that sleep quality is not identical.

      Author response image 4.

      A) The average speed of baseline (grey) and optogenetically-activated flies (green) across 24 hours. Red dots indicate vibration stimulus times. B) The average speed of control (grey) and THIP-fed flies (blue) across 24 hours. Flies are all R23E10/Chrimson. N= 87 for optogenetic, n=88 for -THIP, n=85 for +THIP.

      4- The authors use the SNAP apparatus to prevent THIP-treated flies from sleeping to tease out possible sleep-independent effects. This is an excellent control. Why have the authors not done the same with the optogenetic treatment? It's surprising not to see this control given the concern the authors express (lines 501 - 502) that the dFB manipulation might be paralyzing awake flies, which certainly seems possible given the light regimes used. Why not test this directly with SNAP?

      We appreciate that this may have been a valuable additional control. However, we designed this control for the THIP experiments specifically because of concerns about THIP’s (yet unknown) mechanism of action in flies. THIP is a gabaergic drug with most likely many off-target effects that have little to do with sleep, hence the need for a control where we compare to flies that ingested THIP but have been prevented from sleeping. In contrast, R23E10-driven sleep induction is exactly that, a circuit when activated that induces sleep. Whatever specific neurons might really be involved, the Gal4 circuit is sleep-inducing. This is well supported by multiple publications. The most appropriate control for assessing transcriptomic effects during optogenetic sleep here is not preventing sleep, but rather no increased sleep in flies that have not ingested ATR, and comparing that to effects of ATR alone, which is what we have done. Adding a sleep-deprivation layer onto both of these analyses may have been interesting, but a lot more analyses and not strictly required to identify relevant sleep-related genes. We have rephrased the misleading sentence about paralyzing flies, to instead clarify that lack of overlap with the SD dataset suggests that optogenetic activation is not preventing sleep functions from being engaged.

      5- A pairwise comparison of ZT01 and ZT10 does not address circadian expression cycles in a meaningful way. There will be strong effects of the LD cycle here. I suggest toning this down. (Though it is gratifying to see the expected changes in the core clock genes.)

      We have changed the language from ‘circadian’ to ‘light-dark’ to address this, although have kept the word ‘circadian’ when referring specifically to genes such as per, clock, timeless, etc.

      6- Line 109: There is a reference missing.

      We now provide the relevant reference.

      Results

      1- General comment regarding the figures: a general effort could be made to improve the design and quality of the figures and make them more readable. There are a lot of issues such as stretched or misaligned text, badly drawn frames, etc.

      We think we know which figures this might relate to (e.g., Figures 3,4B), so we have adjusted where appropriate.

      2- Instead of 'dFB-induced' (e.g., L77) it would be more accurate to use 'optogenetically-induced'

      Thank you for this helpful advice. We have changed our language throughout to say ‘optognetically-induced’

      3- Figure S1 should be integrated in the main figure to make the quantification more easily 4accessible.

      We have integrated Figure S1 into the main figures. It is now Figure 3.

      5- It would be good to include red light controls in Figure 2C, E, G.

      Making Figure S1 a main figure has better highlighted the fact that we have done red light controls (‘baseline’).

      6- line 313: Fig2E-H - these graphs would benefit if the authors made it more obvious where the maximum sleep amount would fall - i.e. the combination of bouts and minutes that add up to 12 hours (and therefore the entire day/night)

      If a fly were to sleep uninterrupted for all 12 hours of a day or night, that would amount to a sleep bout 720 minutes long. We do not feel that identifying this maximum on these graphs would be helpful. It should be clear from the data that a floor is reached with very few sleep bouts exceeding 60 minutes in our paradigm. To help orient the reader though, we now clarify in the figure legend that the maximum is 720 minutes or 12 hours.

      7- Fig. 2B, D: It was not clear why the authors took the 3-day average here. Doesn't that lead to a whole range of very different behaviors? I could, perhaps naively, imagine that a fly's behavior changes after 2 days of almost-permanent sleep?

      We took the 3-day average because the effect of THIP on each successive day was not significantly different (see Author response image 5, below). Flies wake up enough to have a good feed (see Author response image 2) and then go back to sleep. Since this is however an important point raised by the reviewer, we now mention in the Methods that sleep duration was not different among the 3 averaged days and nights (lines 193-195).

      Author response image 5.

      Data from THIP feeding experiment (Figure 2B) in manuscript, separated into 3 successive days and nights, with THIP-fed flies (blue) compared to controls (white). Averages  SD are shown, samples sizes are the same as in Figure 2D. No THIP data was significantly different across days and nights (ANOVA of means).

      8- In Figure 2C the authors compare optogenetically induced to "spontaneous sleep," which I think refers to baseline sleep before stimulation, according to the figure. I think the proper comparison would be to the red light control (ATR-); though see the comment above regarding optogenetic controls).

      This information was provided in Figure S1. We now provide it as a main Figure 3, as requested above.

      We also made a point about red light having an effect at night, which is why we focussed on daytime effects for our transcriptomic comparisons. We feel that the ATR-fed flies (minus red light) are an appropriate control here for optogenetically-induced sleep: same exact genotype and ATR feeding, just no optogenetic activation. We therefor would prefer to keep these graphs as they are, especially since we show -ATR data subsequently.

      9- Figures 3A and 4A are redundant; Figure 3B has some active ROIs that are outside of the brain. I am not sure how this is possible?

      We have removed the redundant 4A and replaced it with the THIP molecule to clearly signal what this figure is focussed on. In Figure 3B (now 4B), the brain mask is a visual estimate made from the middle of the image stack. Some neurons in other layers are outside this single-layer estimate. All neurons were all accounted for.

      10- Figure 4B is confusing. It took me a while to understand and so it can do with re-drawing in a more accessible way.

      We agree that this was confusing, e.g. there were too many arrows. We have redrawn and simplified (Now 5A).

      11- The authors state that flies wake up from THIP-induced sleep on the ball, but in Figure 4D there appears to be fewer samples for flies who have woken up from THIP (3) compared to those observed before THIP administration. Are flies dying?

      None of the flies died. Most flies were removed from imaging to confirm recovery, while 3 were left in our imaging setup to measure brain activity upon recovery. These results are in Figure 5C and now clarified in the Methods.

      12- Fig5C,D: I'm surprised that by far the most significant changes (in terms of log2-FC and p-val) occur in the sleep-deprived flies? It is not clear to me what the authors mean by effects that "relate waking process"? Perhaps they could elaborate on this?

      We have removed the phrase ‘relates to waking processes’. We now also remark on the high level of fold-change in many of these genes but refrain from discussing this further in the results. It is interesting though.

      13- The sentence in L425-428 is unclear - it would be good to rephrase this.

      We have rephrased this sentence, hopefully it’s clearer now.

      14- Text in L544-545 is confusing. What do you mean by 'less clear'?

      We have replaced ‘less clear’ with ‘not dominated by a single category’.

      15- It is unclear what is the control in Fig 7A. It would be good to mention what strain was used.

      Different knockout strains had different controls. These are identified in the figure legend and Methods.

      16- L579-581: it would be helpful to include this data in a supplementary figure.

      We now provide this as a supplementary figure as requested (Supplementary Figure 6).

      17- There is no information about R57C10 in the methods - it would be good to explain which neurons this line labels, and why you chose it.

      We now clarify in the methods that R57C10-Gal4 is a pan-neural driver, and provide a reference.

      18- Table S5 - If I'm not mistaken then the first line should say 1h, not 10h.

      Corrected

    1. Author response:

      The following is the authors’ response to the original reviews.

      We would like to thank the reviewers for helping us improve our article and software. The feedback that we received was very helpful and constructive, and we hope that the changes that we have made are indeed effective at making the software more accessible, the manuscript clearer, and the online documentation more insightful as well. A number of comments related to shared concerns, such as:

      • the need to describe various processing steps more clearly (e.g. particle picking, or the nature of ‘dust’ in segmentations)

      • describing the features of Ais more clearly, and explaining how it can interface with existing tools that are commonly used in cryoET

      • a degree of subjectivity in the discussion of results (e.g. about Pix2pix performing better than other networks in some cases.)

      We have now addressed these important points, with a focus on streamlining not only the workflow within Ais but also making interfacing between Ais and other tools easier. For instance, we explain more clearly which file types Ais uses and we have added the option to export .star files for use in, e.g., Relion, or meshes instead of coordinate lists. We also include information in the manuscript about how the particle picking process is implemented, and how false positives (‘dust’) can be avoided. Finally, all reviewers commented on our notion that Pix2pix can work ‘better’ despite reaching a higher loss after training. As suggested, we included a brief discussion about this idea in the supplementary information (Fig. S6) and used it to illustrate how Ais enables iteratively improving segmentation results. 

      Since receiving the reviews we have also made a number of other changes to the software that are not discussed below but that we nonetheless hope have made the software more reliable and easier to use. These include expanding the available settings, slight changes to the image processing that can help speed it up or avoid artefacts in some cases, improving the GUI-free usability of Ais, and incorporating various tools that should help make it easier to use Ais with remote data (e.g. doing annotation on an office PC, but model training on a more powerful remote PC). We have also been in contact with a number of users of the software, who reported issues or suggested various other miscellaneous improvements, and many of whom had found the software via the reviewed preprint.

      Reviewer 1 (Public Review):

      This paper describes "Ais", a new software tool for machine-learning-based segmentation and particle picking of electron tomograms. The software can visualise tomograms as slices and allows manual annotation for the training of a provided set of various types of neural networks. New networks can be added, provided they adhere to a Python file with an (undescribed) format. Once networks have been trained on manually annotated tomograms, they can be used to segment new tomograms within the same software. The authors also set up an online repository to which users can upload their models, so they might be re-used by others with similar needs. By logically combining the results from different types of segmentations, they further improve the detection of distinct features. The authors demonstrate the usefulness of their software on various data sets. Thus, the software appears to be a valuable tool for the cryo-ET community that will lower the boundaries of using a variety of machine-learning methods to help interpret tomograms. 

      We thank the reviewer for their kind feedback and for taking the time to review our article. On the basis of their  comments, we have made a number of changes to the software, article, and documentation, that we think have helped improve the project and render it more accessible (especially for interfacing with different tools, e.g. the suggestions to describe the file formats in more detail). We respond to all individual comments one-by-one below.

      Recommendations:

      I would consider raising the level of evidence that this program is useful to *convincing* if the authors would adequately address the suggestions for improvement below.

      (1) It would be helpful to describe the format of the Python files that are used to import networks, possibly in a supplement to the paper. 

      We have now included this information in both the online documentation and as a supplementary note (Supplementary Note 1). 

      (2) Likewise, it would be helpful to describe the format in which particle coordinates are produced. How can they be used in subsequent sub-tomogram averaging pipelines? Are segmentations saved as MRC volumes? Or could they be saved as triangulations as well? More implementation details like this would be good to have in the paper, so readers don't have to go into the code to investigate. 

      Coordinates: previously, we only exported arrays of coordinates as tab-separated .txt files, compatible with e.g. EMAN2. We now added a selection menu where users can specify whether to export either .star files or tsv .txt files, which together we think should cover most software suites for subtomogram averaging. 

      Triangulations: We have now improved the functionality for exporting triangulations. In the particle picking menu, there is now the option to output either coordinates or meshes (as .obj files). This was previously possible in the Rendering tab, but with the inclusion in the picking menu exporting triangulations can now be done for all tomograms at once rather than manually one by one.

      Edits in the text: the output formats were previously not clear in the text. We have now included this information in the introduction:

      “[…] To ensure compatibility with other popular cryoET data processing suites, Ais employs file formats that are common in the field, using .mrc files for volumes, tab-separated .txt or .star files for particle datasets, and the .obj file format for exporting 3D meshes.”

      (3) In Table 2, pix2pix has much higher losses than alternatives, yet the text states it achieves fewer false negatives and fewer false positives. An explanation is needed as to why that is. Also, it is mentioned that a higher number of epochs may have improved the results. Then why wasn't this attempted? 

      The architecture of Pix2pix is quite different from that of the other networks included in the test. Whereas all others are trained to minimize a binary cross entropy (BCE) loss, Pix2pix uses a composite loss function that is a weighted combination of the generator loss and a discriminator penalty, neither of which employ BCE. However, to be able to compare loss values, we do compute a BCE loss value for the Pix2pix generator after every training epoch. This is the value reported in the manuscript and in the software. Although Pix2pix’ BCE loss does indeed diminish during training, the model is not actually optimized to minimize this particular value and a comparison by BCE loss is therefore not entirely fair to Pix2pix. This is pointed out (in brief) in the legend to the able: 

      “Unlike the other architectures, Pix2pix is not trained to minimize the bce loss but uses a different loss function instead. The bce loss values shown here were computed after training and may not be entirely comparable.”

      Regarding the extra number of epochs for Pix2pix: here, we initially ran in to the problem that the number of samples in the training data was low for the number of parameters in Pix2pix, leading to divergence later during training. This problem did not occur for most other models, so we decided to keep the data for the discussion around Table 1 and Figure 2 limited to that initial training dataset. After that, we increased the sample size (from 58 to 170 positive samples) and trained the model for longer. The resulting model was used in the subsequent analyses. This was previously implicit in the text but is now mentioned explicitly and in a new supplementary figure. 

      “For the antibody platform, the model that would be expected to be one of the worst based on the loss values, Pix2pix, actually generates segmentations that are seem well-suited for the downstream processing tasks. It also output fewer false positive segmentations for sections of membranes than many other models, including the lowest-loss model UNet. Moreover, since Pix2pix is a relatively large network, it might also be improved further by increasing the number of training epochs. We thus decided to use Pix2pix for the segmentation of antibody platforms, and increased the size of the antibody platform training dataset (from 58 to 170 positive samples) to train a much improved second iteration of the network for use in the following analyses (Fig. S6).”

      (4) It is not so clear what absorb and emit mean in the text about model interactions. A few explanatory sentences would be useful here. 

      We have expanded this paragraph to include some more detail.

      “Besides these specific interactions between two models, the software also enables pitching multiple models against one another in what we call ‘model competition’. Models can be set to ‘emit’ and/or ‘absorb’ competition from other models. Here, to emit competition means that a model’s prediction value is included in a list of competing models. To absorb competition means that a model’s prediction value will be compared to all values in that list, and that this model’s prediction value for any pixel will be set to zero if any of the competing models’ prediction value is higher. On a pixel-by-pixel basis, all models that absorb competition are thus suppressed whenever their prediction value for a pixel is lower than that of any of the emitting models.”

      (5) Under Figure 4, the main text states "the model interactions described above", but because multiple interactions were described it is not clear which ones they were. Better to just specify again. 

      Changed as follows:

      “The antibody platform and antibody-C1 complex models were then applied to the respective datasets, in combination with the membrane and carbon models and the model interactions described above (Fig. 4b): the membrane avoiding carbon, and the antibody platforms colocalizing with the resulting membranes”.

      (6) The next paragraph mentions a "batch particle picking process to determine lists of particle coordinates", but the algorithm for how coordinates are obtained from segmented volumes is not described. 

      We have added a paragraph to the main text to describe the picking process:

      “This picking step comprises a number of processing steps (Fig. S7). First, the segmented (.mrc) volumes are thresholded at a user-specified level. Second, a distance transform of the resulting binary volume is computed, in which every nonzero pixel in the binary volume is assigned a new value, equal to the distance of that pixel to the nearest zero-valued pixel in the mask. Third, a watershed transform is applied to the resulting volume, so that the sets of pixels closest to any local maximum in the distance transformed volume are assigned to one group. Fourth, groups that are smaller than a user-specified minimum volume are discarded. Fifth, groups are assigned a weight value, equal to the sum of the prediction value (i.e. the corresponding pixel value in the input .mrc volume) of the pixels in the group. For every group found within close proximity to another group (using a user-specified value for the minimum particle spacing), the group with the lower weight value is discarded. Finally, the centroid coordinate of the grouped pixels is considered the final particle coordinate, and the list of all

      coordinates is saved in a tab-separated text file.

      “As an alternative output format, segmentations can also be converted to and saved as triangulated meshes, which can then be used for, e.g., membrane-guided particle picking. After picking particles, the resulting coordinates are immediately available for inspection in the Ais 3D renderer (Fig. S8).“

      The two supplementary figures are pasted below for convenience. Fig. S7 is new, while Fig. S8 was previously Fig. S10 -the reference to this figure was originally missing in the main text, but is now included.

      (7) In the Methods section, it is stated that no validation splits are used "in order to make full use of an input set". This sounds like an odd decision, given the importance of validation sets in the training of many neural networks. Then how is overfitting monitored or prevented? This sounds like a major limitation of the method. 

      In our experience, the best way of preparing a suitable model is to (iteratively) annotate a set of training images and visually inspect the result. Since the manual annotation step is the bottleneck in this process, we decided not to use validation split in order to make full use of an annotated training dataset (i.e. a validation split of 20% would mean that 20% of the manually annotated training data is not used for training)

      We do recognize the importance of using separate data for validation, or at least offering the possibility of doing so. We have now added a parameter to the settings (and made a Settings menu item available in the top menu bar) where users can specify what fraction (0, 10, 20, or 50%) of training datasets should be set aside for validation. If the chosen value is not 0%, the software reports the validation loss as well as the size of the split during training, rather than (as was done previously) the training loss. We have, however, set the default value for the validation split to 0%, for the same reason as before. We also added a section to the online documentation about using validation splits, and edited the corresponding paragraph in the methods section:

      “The reported loss is that calculated on the training dataset itself, i.e., no validation split was applied. During regular use of the software, users can specify whether to use a validation split or not. By default, a validation split is not applied, in order to make full use of an input set of ground truth annotations. Depending on the chosen split size, the software reports either the overall training loss or the validation loss during training.”

      (8) Related to this point: how is the training of the models in the software modelled? It might be helpful to add a paragraph to the paper in which this process is described, together with indicators of what to look out for when training a model, e.g. when should one stop training? 

      We have expanded the paragraph where we write about the utility of comparing different networks architectures to also include a note on how Ais facilitates monitoring the output of a model during training:

      “When taking the training and processing speeds in to account as well as the segmentation results, there is no overall best architecture. We therefore included multiple well-performing model architectures in the final library, in order to allow users to select from these models to find one that works well for their specific datasets. Although it is not necessary to screen different network architectures and users may simply opt to use the default (VGGNet), these results thus show that it can be useful to test different networks in order to identify one that is best. Moreover, these results also highlight the utility of preparing well-performing models by iteratively improving training datasets and re-training models in a streamlined interface. To aid in this process, the software displays the loss value of a network during training and allows for the application of models to datasets during training. Thus, users can inspect how a model’s output changes during training and decide whether to interrupt training and improve the training data or choose a different architecture.”

      (9) Figure 1 legend: define the colours of the different segmentations. 

      Done

      (10) It may be better to colour Figure 2B with the same colours as Figure 2A. 

      We tried this, but the effect is that the underlying density is much harder to see. We think the current grayscale image paired with the various segmentations underneath is better for visually identifying which density corresponds to membranes, carbon film, or antibody platforms.

      Reviewer 2 (Public Review):

      Summary: 

      Last et al. present Ais, a new deep learning-based software package for the segmentation of cryo-electron tomography data sets. The distinguishing factor of this package is its orientation to the joint use of different models, rather than the implementation of a given approach. Notably, the software is supported by an online repository of segmentation models, open to contributions from the community. 

      The usefulness of handling different models in one single environment is showcased with a comparative study on how different models perform on a given data set; then with an explanation of how the results of several models can be manually merged by the interactive tools inside Ais. 

      The manuscripts present two applications of Ais on real data sets; one is oriented to showcase its particlepicking capacities on a study previously completed by the authors; the second one refers to a complex segmentation problem on two different data sets (representing different geometries as bacterial cilia and mitochondria in a mouse neuron), both from public databases. 

      The software described in the paper is compactly documented on its website, additionally providing links to some YouTube videos (less than an hour in total) where the authors videocapture and comment on major workflows. 

      In short, the manuscript describes a valuable resource for the community of tomography practitioners. 

      Strengths: 

      A public repository of segmentation models; easiness of working with several models and comparing/merging the results. 

      Weaknesses: 

      A certain lack of concretion when describing the overall features of the software that differentiate it from others. 

      We thank the reviewer for their kind and constructive feedback. Following the suggestion to use the Pix2pix results to illustrate the utility of Ais for analyzing results, we have added a new supplementary figure (Fig. S6) and brief discussion, showing the use of Ais in iteratively improving segmentation results. We have also expanded the online documentation and included a note in the supplementary information about how models are saved/loaded (Supplemetary note 1) 

      Recommendations:

      I would like to ask the authors about some concerns about the Ais project as a whole: 

      (1) The website that accompanies the paper (aiscryoet.org), albeit functional, seems to be in its first steps. Is it planned to extend it? In particular, one of the major contributions of the paper (the maintenance of an open repository of models) could use better documentation describing the expected formats to submit models. This could even be discussed in the supplementary material of the manuscript, as this feature is possibly the most distinctive one of the paper. Engaging third-party users would require giving them an easier entry point, and the superficial mention of this aspect in the online documentation could be much more generous.

      We have added a new page to the online documentation, titled ‘Sharing models’ where we include an explanation of the structure of model files and demonstrate the upload page. We also added a note to the Supplementary Information that explains the file format for models, and how they are loaded/saved (i.e., that these standard keras model obects). 

      To make it easier to interface Ais with other tools, we have now also made some of the core functionality available (e.g. training models, batch segmentation) via the command line interface. Information on how to use this is included in the online documentation. All file formats are common formats used in cryoET, so that using Ais in a workflow with, e.g. AreTomo -> Ais -> Relion should now be more straightforward.

      (2) A different major line advanced by the authors to underpin the novelty of the software, is its claimed flexibility and modularity. In particular, the restrictions of other packages in terms of visualization and user interaction are mentioned. Although in the manuscript it is also mentioned that most of the functionalities in Ais are already available in major established packages, as a reader I am left confused about what exactly makes the offer of Ais different from others in terms of operation and interaction: is it just the two aspects developed in the manuscript (possibility of using different models and tools to operate model interaction)? If so, it should probably be stated; but if the authors want to pinpoint other aspects of the capacity of Ais to drive smoothly the interactions, they should be listed and described, instead of leaving it as an unspecific comment. As a potential user of Ais, I would suggest the authors add (maybe in the supplementary material) a listing of such features. Figure 1 does indeed carry the name "overview of (...) functionalities", but it is not clear to me which functionalities I can expect to be absent or differently solved on the other tools they mention.

      We have rewritten the part of the introduction where we previously listed the features as below. We think it should now be clearer for the reader to know what features to expect, as well as how Ais can interface with other software (i.e. what the inputs and outputs are). We have also edited the caption for Figure 1 to make it explicit that panels A to C represent the annotation, model preparation, and rendering steps of the Ais workflow and that the images are screenshots from the software.

      “In this report we present Ais, an open-source tool that is designed to enable any cryoET user – whether experienced with software and segmentation or a novice – to quickly and accurately segment their cryoET data in a streamlined and largely automated fashion. Ais comprises a comprehensive and accessible user interface within which all steps of segmentation can be performed, including: the annotation of tomograms and compiling datasets for the training of convolutional neural networks (CNNs), training and monitoring performance of CNNs for automated segmentation, 3D visualization of segmentations, and exporting particle coordinates or meshes for use in downstream processes. To help generate accurate segmentations, the software contains a library of various neural network architectures and implements a system of configurable interactions between different models. Overall, the software thus aims to enable a streamlined workflow where users can interactively test, improve, and employ CNNs for automated segmentation. To ensure compatibility with other popular cryoET data processing suites, Ais employs file formats that are common in the field, using .mrc files for volumes, tab-separated .txt or .star files for particle datasets, and the .obj file format for exporting 3D meshes.”

      “Figure 1 – an overview of the user interface and functionalities. The various panels represent sequential stages in the Ais processing workflow, including annotation (a), testing CNNs (b), visualizing segmentation (c). These images (a-c) are unedited screenshots of the software. a) […]”

      (3) Table 1 could have the names of the three last columns. The table has enough empty space in the other columns to accommodate this. 

      Done.

      (4) The comment about Pix2pix needing a larger number of training epochs (being a larger model than the other ones considered) is interesting. It also lends itself for the authors to illustrate the ability of their software to precisely do this: allow the users to flexibly analyze results and test hypothesis

      Please see the response to Reviewer 1 comment #3. We agree that this is a useful example of the ability to iterate between annotation and training, and have added an explicit mention of this in the text:

      “Moreover, since Pix2pix is a relatively large network, it might also be improved further by increasing the number of training epochs. In a second iteration of annotation and training, we thus increased the size of the antibody platform training dataset (from 58 to 170 positive samples) and generated an improved Pix2pix model for use in the following analyses.”

      Reviewer 3 (Public Review):

      We appreciate the reviewer’s extensive and very helpful feedback and are glad to read that they consider Ais potentially quite useful for the users. To address the reviewer’s comments, we have made various edits to the text, figures, and documentation, that we think have helped improve the clarity of our work. We list all edits below. 

      Summary

      In this manuscript, Last and colleagues describe Ais, an open-source software package for the semi-automated segmentation of cryo-electron tomography (cryo-ET) maps. Specifically, Ais provides a graphical user interface (GUI) for the manual segmentation and annotation of specific features of interest. These manual annotations are then used as input ground-truth data for training a convolutional neural network (CNN) model, which can then be used for automatic segmentation. Ais provides the option of several CNNs so that users can compare their performance on their structures of interest in order to determine the CNN that best suits their needs. Additionally, pre-trained models can be uploaded and shared to an online database. 

      Algorithms are also provided to characterize "model interactions" which allows users to define heuristic rules on how the different segmentations interact. For instance, a membrane-adjacent protein can have rules where it must colocalize a certain distance away from a membrane segmentation. Such rules can help reduce false positives; as in the case above, false negatives predicted away from membranes are eliminated. 

      The authors then show how Ais can be used for particle picking and subsequent subtomogram averaging and for the segmentation of cellular tomograms for visual analysis. For subtomogram averaging, they used a previously published dataset and compared the averages of their automated picking with the published manual picking. Analysis of cellular tomogram segmentation was primarily visual. 

      Strengths:

      CNN-based segmentation of cryo-ET data is a rapidly developing area of research, as it promises substantially faster results than manual segmentation as well as the possibility for higher accuracy. However, this field is still very much in the development and the overall performance of these approaches, even across different algorithms, still leaves much to be desired. In this context, I think Ais is an interesting package, as it aims to provide both new and experienced users with streamlined approaches for manual annotation, access to a number of CNNs, and methods to refine the outputs of CNN models against each other. I think this can be quite useful for users, particularly as these methods develop. 

      Weaknesses: 

      Whilst overall I am enthusiastic about this manuscript, I still have a number of comments: 

      (1) On page 5, paragraph 1, there is a discussion on human judgement of these results. I think a more detailed discussion is required here, as from looking at the figures, I don't know that I agree with the authors' statement that Pix2pix is better. I acknowledge that this is extremely subjective, which is the problem. I think that a manual segmentation should also be shown in a figure so that the reader has a better way to gauge the performance of the automated segmentation.

      Please see the answer to Reviewer 1’s comment #3.

      (2) On page 7, the authors mention terms such as "emit" and "absorb" but never properly define them, such that I feel like I'm guessing at their meaning. Precise definitions of these terms should be provided. 

      We have expanded this paragraph to include some more detail:

      “Besides these specific interactions between two models, the software also enables pitching multiple models against one another in what we call ‘model competition’. Models can be set to ‘emit’ and/or ‘absorb’ competition from other models. Here, to emit competition means that a model’s prediction value is included in a list of competing models. To absorb competition means that a model’s prediction value will be compared to all values in that list, and that this model’s prediction value for any pixel will be set to zero if any of the competing models’ prediction value is higher. On a pixel-by-pixel basis, all models that absorb competition are thus suppressed whenever their prediction value for a pixel is lower than that of any of the emitting models.” 

      (3) For Figure 3, it's unclear if the parent models shown (particularly the carbon model) are binary or not.

      The figure looks to be grey values, which would imply that it's the visualization of some prediction score. If so, how is this thresholded? This can also be made clearer in the text. 

      The figures show the grayscale output of the parent model, but this grayscale output is thresholded to produce a binary mask that is used in an interaction. We have edited the text to include a mention of thresholding at a user-specified threshold value:

      “These interactions are implemented as follows: first, a binary mask is generated by thresholding the parent model’s predictions using a user-specified threshold value. Next, the mask is then dilated using a circular kernel with a radius 𝑅, a parameter that we call the interaction radius. Finally, the child model’s prediction values are multiplied with this mask.”

      To avoid confusion, we have also edited the figure to show the binary masks rather than the grayscale segmentations. 

      (4) Figure 3D was produced in ChimeraX using the hide dust function. I think some discussion on the nature of this "dust" is in order, e.g. how much is there and how large does it need to be to be considered dust? Given that these segmentations can be used for particle picking, this seems like it may be a major contributor to false positives. 

      ‘Dust’ in segmentations is essentially unavoidable; it would require a perfect model that does not produce any false positives. However, when models are sufficiently accurate, the volume of false positives is typically smaller than that of the structures that were intended to be segmented. In these cases, discarding particles based on size is a practical way of filtering the segmentation results. Since it is difficult to generalize when to consider something ‘dust’ we decided to include this additional text in the Method’s section rather than in the main text:

      “… with the use of the ‘hide dust’ function (the same settings were used for each panel, different settings used for each feature).

      This ‘dust’ corresponds to small (in comparison to the segmented structures of interest) volumes of false positive segmentations, which are present in the data due to imperfections in the used models. The rate and volume of false positives can be reduced either by improving the models (typically by including more examples of the images of what would be false negatives or positives in the training data) or, if the dust particles are indeed smaller than the structures of interest, they can simply be discarded by filtering particles based on their volume, as applied here. In particle picking a ‘minimum particle volume’ is specified – particles with a smaller volume are considered ‘dust’.

      In combination with the newly included text about the method of converting volumes into lists of coordinates (see Reviewer 1’s comment #6).

      “Third, a watershed transform is applied to the resulting volume, so that the sets of pixels closest to any local maximum in the distance transformed volume are assigned to one group. Fourth, groups that are smaller than a user-specified minimum volume are discarded…”

      We think it should now be clearer that (some form of) discarding ‘dust’ is a step that is typically included in the particle picking process.

      (5) Page 9 contains the following sentence: "After selecting these values, we then launched a batch particle picking process to determine lists of particle coordinates based on the segmented volumes." Given how important this is, I feel like this requires significant description, e.g. how are densities thresholded, how are centers determined, and what if there are overlapping segmentations? 

      Please see the response to Reviewer 1’s comment #6.

      (6) The FSC shown in Figure S6 for the auto-picked maps is concerning. First, a horizontal line at FSC = 0 should be added. It seems that starting at a frequency of ~0.045, the FSC of the autopicked map increases above zero and stays there. Since this is not present in the FSC of the manually picked averages, this suggests the automatic approach is also finding some sort of consistent features. This needs to be discussed. 

      Thank you for pointing this out. Awkwardly, this was due to a mistake made while formatting the figure. In the two separate original plots, the Y axes had slightly different ranges, but this was missed when they were combined to prepare the joint supplementary figure. As a result, the FSC values for the autopicked half maps are displayed incorrectly. The original separate plots are shown below to illustrate the discrepancy:

      Author response image 1.

      The corrected figure is Figure S9 in the manuscript. The values of 44 Å and 46 Å were not determined from the graph and remain unchanged.

      (7) Page 11 contains the statement "the segmented volumes found no immediately apparent false positive predictions of these pores". This is quite subjective and I don't know that I agree with this assessment. Unless the authors decide to quantify this through subtomogram classification, I don't think this statement is appropriate. 

      We originally included this statement and the supplementary figure because we wanted to show another example of automated picking, this time in the more crowded environment of the cell. We do agree that it requires better substantiation, but also think that the demonstration of automated picking of the antibody platforms and IgG3-C1 complexes for subtomogram averaging suffices to demonstrate Ais’ picking capabilities. Since the supplementary information includes an example of picked coordinates rendered in the Ais 3D viewer (Figure S7) that also used the pore dataset, we still include the supplementary figure (S10) but have edited the statement to read:

      “Moreover, we could identify the molecular pores within the DMV, and pick sets of particles that might be suitable for use in subtomogram averaging (see Fig. S11).”

      We have also expanded the text that accompanies the supplementary figure to emphasize that results from automated picking are likely to require further curation, e.g. by classification in subtomogram averaging, and that the selection of particles is highly dependent on the thresholds used in the conversion from volumes to lists of coordinates.

      (8) In the methods, the authors note that particle picking is explained in detail in the online documentation. Given that this is a key feature of this software, such an explanation should be in the manuscript. 

      Please see the response to Reviewer 1’s comment #6. 

      Recommendations:

      (9) The word "model" seems to be used quite ambiguously. Sometimes it seems to refer to the manual segmentations, the CNN architectures, the trained models, or the output predictions. More precision in this language would greatly improve the readability of the manuscript.

      This was indeed quite ambiguous, especially in the introduction. We have edited the text to be clearer on these differences. The word ‘model’ is now only used to refer to trained CNNs that segment a particular feature (as in ‘membrane model’ or ‘model interactions’). Where we used terms such as ‘3D models’ to describe scenes rendered in 3D, we now use ‘3D visualizations’ or similar terms. Where we previously used the term ‘models’ to refer to CNN architectures, we now use terms such as ‘neural network architectures’ or ‘architecture’. Some examples:

      … with which one can automatically segment the same or any other dataset …

      Moreover, since Pix2pix is a relatively large network, …       

      … to generate a 3D visualization of ten distinct cellular …

      … with the use of the same training datasets for all network architectures …

      In Figure 1, the text in panels D and E is illegible. 

      We have edited the figure to show the text more clearly (the previous images were unedited screenshots of the website).

      (10) Prior to the section on model interactions, I was under the impression that all annotations were performed simultaneously. I think it could be clarified that models are generated per annotation type. 

      Multiple different features can be annotated (i.e. drawn by hand by the user) at the same time, but each trained CNN only segments one feature. CNNs that output segmentations for multiple features can be implemented straightforwardly, but this introduces the need to provide training data where for every grayscale image, every feature is annotated. This can make preparing the training data much more cumbersome. Reusability of the models is also hampered. We now mention the separateness of the networks explicitly in the introduction:

      “Multiple features, such as membranes, microtubules, ribosomes, and phosphate crystals, can be segmented and edited at the same time across multiple datasets (even hundreds). These annotations are then extracted and used as ground truth labels upon which to condition multiple separate neural networks, …”

      (11) On page 6, there is the text "some features are assigned a high segmentation value by multiple of the networks, leading to ambiguity in the results". Do they mean some false features? 

      To avoid ambiguity of the word ‘features’, we have edited the sentence to read:

      “… some parts of the image are assigned a high segmentation value by multiple of the networks, leading to false classifications and ambiguity in the results.”

      (12) Figures 2 and 3 would be easier to follow if they had consistent coloring. 

      We have changed the colouring in Figure 2 to match that of Figure 3 better:

      (13) For Figure 3D, I'm confused as to why the authors showed results from the tomogram in Figure 2B. It seems like the tomogram in Figure 3C would be a more obvious choice, as we would be able to see how the 2D slices look in 3D. This would also make it easier to see the effect of interactions on false negatives. Also, since the orientation of the tomogram in 2B is quite different than that shown in 3D, it's a bit difficult to relate the two.

      We chose to show this dataset because it exemplifies the effects of both model competition and model interactions better than the tomogram in Figure 3C. See Figure 3D and Author response image 2 for a comparison:

      Author response image 2.

      (14) I'm confused as to why the tomographic data shown in Figures 4D, E, and F are black on white while all other cryo-ET data is shown as white on black. 

      The images in Figure 4DEF are now inverted.

      (15) For Figure 5, there needs to be better visual cueing to emphasize which tomographic slices are related to the segmentations in Panels A and B. 

      We have edited the figure to show more clearly which grayscale image corresponds to which segmentation:

      (16) I don't understand what I should be taking away from Figures S1 and S2. There are a lot of boxes around membrane areas and I don't know what these boxes mean. 

      We have added a more descriptive text to these figures. The boxes are placed by the user to select areas of the image that will be sampled when saving training datasets.

    1. Author response:

      The following is the authors’ response to the original reviews.

      Public Reviews:

      Reviewer #1 (Public Review):

      Summary:

      Jin, Briggs, and colleagues use light sheet imaging to reconstruct the islet threedimensional Ca2+ network. The authors find that early/late responding (leader) cells are dynamic over time, and located at the islet periphery. By contrast, highly connected or hub cells are stable and located toward the islet center. Suggesting that the two subpopulations are differentially regulated by fuel input, glucokinase activation only influences leader cell phenotype, whereas hubs remain stable.

      Strengths:

      The studies are novel in providing the first three-dimensional snapshot of the beta cell functional network, as well as determining the localization of some of the different subpopulations identified to date. The studies also provide some consensus as to the origin, stability, and role of such subpopulations in islet function.

      We thank the reviewers for their positive assessment.

      Weaknesses:

      Experiments with metabolic enzyme activators do not take into account the influence of cell viability on the observed Ca2+ network data. Limitations of the imaging approach used need to be recognized and evaluated/discussed.

      We worked very hard to make sure the islets remained stable and healthy over the duration of imaging time course. We imaged the islet in 3D and observed that all betacells displayed glucose-dependent oscillations, which can only arise from functioning cells. From the raw calcium traces (displayed in the figures) we observed no detectable loss of signal over 60 min of continuous imaging regardless of drug treatment; this is because the laser excitation is below the bleach threshold for GCaMP6s, and it is bleaching that generates phototoxicity. To demonstrate this clearly, we performed a bleach test using 6x laser power; in this case calcium amplitude dropped 30% over a 60 min of imaging, however islet calcium oscillatory behavior was preserved. Light-sheet is well documented to be 1000x more gentle than other optical sectioning techniques, which is why it was chosen for this application.

      Regarding the limitations of imaging approach, we recognized studying islets ex vivo is necessarily performed in the absence of native surrounding tissue, as highlighted in the discussion.

      Reviewer #2 (Public Review):

      The manuscript by Erli Jin, Jennifer Briggs et al. utilizes light sheet microscopy to image islet beta cell calcium oscillations in 3D and determine where beta cell populations are located that begin and coordinate glucose-stimulated calcium oscillations. The light sheet technique allowed clear 3D mapping of beta cell calcium responses to glucose, glucokinase activation, and pyruvate kinase activation. The manuscript finds that synchronized beta-cells are found at the islet center, that leader beta cells showing the first calcium responses are located on the islet periphery, that glucokinase activation helped maintain beta cells that lead calcium responses, and that pyruvate kinase activation primarily increases islet calcium oscillation frequency. The study is well-designed, contains a significant amount of high-quality data, and the conclusions are largely supported by the results.

      It has recently been shown that beta cells within islets containing intact vasculature (such as those in a pancreatic slice) show different calcium responses compared to isolated islets (such as that shown in PMID: 35559734). It would be important to include some discussion about the potential in vitro artifacts in calcium that arise following islet isolation (this could be included in the discussion about the limitations of the study).

      Although isolated islets reproduce the slow oscillatory calcium behavior observed in vivo, we agree that missing elements such as blood flow, cholinergic innervation, and surrounding tissues may each impact islet calcium responses. Pancreatic regional blood flow also links the endocrine and exocrine signaling which can directly influence the behavior of beta cells. We have highlighted some of these issues in the discussion “In addition to α-cells, vasculature may also impact islet Ca2+ responses, and may induce additional heterogeneity in vivo.” (see line 375, Ref. 46).

      Reviewer #3 (Public Review):

      Summary:

      Jin, Briggs et al. made use of light-sheet 3D imaging and data analysis to assess the collective network activity in isolated mouse islets. The major advantage of using whole islet imaging, despite compromising on the speed of acquisition, is that it provides a complete description of the network, while 2D networks are only an approximation of the islet network. In static-incubation conditions, excluding the effects of perfusion, they assessed two subpopulations of beta cells and their spatial consistency and metabolic dependence.

      Strengths:

      The authors confirmed that coordinated Ca2+ oscillations are important for glycemic control. In addition, they definitively disproved the role of individual privileged cells, which were suggested to lead or coordinate Ca²⁺ oscillations. They provided evidence for differential regional stability, confirming the previously described stochastic nature of the beta cells that act as strongly connected hubs as well as beta cells in initiating regions (doi.org/10.1103/PhysRevLett.127.168101).

      The fact that islet cores contain beta cells that are more active and more coordinated has also been readily observed in high-frequency 2D recordings (e.g. DOI: 10.2337/db22-0952), suggesting that the high-speed capture of fast activity can partially compensate for incomplete topological information.

      They also found an increased metabolic sensitivity of mantle regions of an islet with a subpopulation of beta cells with a high probability of leading the islet activity which can be entrained by fuel input. They discuss a potential role of alpha/delta cell interaction, however relative lack of beta cells in the islet border region could also be a factor contributing to less connectivity and higher excitability.

      The Methods section contains a useful series of direct instructions on how to approach fast 3D imaging with currently available hardware and software.

      The Discussion is clear and includes most of the issues regarding the interpretation of the presented results.

      Some issues concerning inconsistencies between data presented and statements made as well as statistical analysis need to be addressed.

      Taken together it is a strong technical paper to demonstrate the stochasticity regarding the functions subpopulations of beta cells in the islets may have and how less well-resolved approaches (both missing spatial resolution as well as missing temporal resolution) led us to jump to unjustified conclusions regarding the fixed roles of individual beta cells within an islet.

      We thank the reviewers for the comments on the many strengths of the manuscript and address the specific critiques below.

      Recommendations for the authors:

      Reviewing Editor Comments:

      Essential revisions:

      (1) How useful is GK activation as a subpopulation-level perturbation, given that all beta cells would be affected? Previous studies by the authors have shown that GK gradients likely dictate subpopulation behaviour, so the concern here is that GK activation across all cells might mask the influence of such gradients i.e. a U-shaped effect. Also, does the GK activator differentially penetrate the islet such that first responders/leaders are more vulnerable than hubs?

      As we previously published, non-saturating concentrations of GK activator (as used here) have the same effect on calcium oscillations as raising glucose (PMID:33147484). In other words, the activator boosts the activity of the endogenous GK. To the second point, recent ex vivo islet studies (PMID: 28380380) document the islet penetration of a fluorescent glucose analogue within seconds even under static conditions, and in our study the islets calcium oscillations reached steady state, so we are not concerned about drug penetration. The real limitation with any drug study in the islet is that non-beta cells are also activated; this limitation is included in the discussion along with the recommendation that genetic tools are needed to assess the effect of GK activation in the various endocrine subpopulations. 

      An additional concern with the GK activation experiment is that GK activation might push beta cells into a more stressed state such that they are more susceptible to phototoxicity. Although the authors state that photobleaching is low, they provide no data to support such a statement. Given the long duration of imaging and acquisition rate, phototoxicity might be more of an issue, especially with GK activation. Some further analysis (e.g. apoptosis) would be useful here to exclude an effect of beta cell viability versus GK activation on the observed phenotype of the different subpopulations.

      Acute GK activation (for 30min) does not stress the islet; the drug has the same effect as raising glucose (PMID: 33147484). To determine whether photobleaching was impacted by GK activation, we examined the peak of consecutive oscillations in response to vehicle and GK activator. The average photobleaching was less than 2% of the calcium fluorescence over 30min of continuous imaging. Furthermore, GKa activation did not significantly increase photobleaching (see Author response image 1). 

      Author response image 1.

      To the reviewer’s second point, apoptosis cannot occur on the timescale of the drug treatment (30min), and raw calcium traces are included showing that all beta cells display oscillatory behavior throughout the course of the experiment.

      (2) The authors show that glucokinase activation increases the duration of islet calcium oscillations and in some islets (3 of 15 islets) causes "a Ca2+ plateau." The authors indicate that "Glucokinase, as the 'glucose sensor' for the β-cell, controls the input of glucose carbons into glycolysis, and opens KATP channels." It would be nice to have some experimental evidence that the change in oscillation rate caused by the glucokinase activator is due to KATP activation. This could be accomplished by treating islets with subthreshold KATP activators (e.g., diazoxide) or subthreshold KATP inhibitors (e.g., tolbutamide).

      The statement that glucokinase activation opens KATP channels was a typo; glucose metabolism closes KATP channels by raising the ATP/ADP ratio. We now include additional citations that document the relationship between GK and KATP and the oscillatory behavior. See Ref 22 (PMID: 33147484) and Ref 34 (PMID: 33147484).

      The manuscript finds that "Early phase cells were maintained to a greater degree upon GKa application." Yet GKa is proposed to activate KATP. Some discussion about how the early phase is maintained in cell populations by GKa activation in the context of KATP activity would be useful.

      As discussed above, we meant to say that GKa will close KATP and apologize for the confusion. As we mentioned in the discussion, early phase cells are most likely maintained to a great degree following GK activation as result of enhanced GK gradient and reduced effect of stochastic alpha cell input. 

      (3) Membrane potential depolarization precedes calcium channel activation and subsequent calcium entry. In many cases, electrical coupling across beta cells happens on millisecond timescale. It would be good to confirm that the calcium is showing the same time scale in terms of elevation following beta cell membrane potential depolarization. One concern is that the islet beta cells could be depolarizing at the same speed and lagging in terms of calcium channel activation and calcium entry.

      We thank the reviewer for making this point, which is almost certainly true, particularly since plasma membrane calcium influx is not the sole source of intracellular calcium. Previously published “simultaneous” recordings of Vm and calcium show their same phase relationship but do not have sufficient time resolution to capture depolarization of each cell. A quantification of phase lag would require the field to generate mice with voltage sensors expressed in beta cells; these tools are not yet available.  

      A related issue: in the text, the authors discuss changes in membrane potential (not been measured in this study), while in the figures they exclusively describe Ca2+ oscillations (which were measured). Examples are on lines 149, 150, 153, 154, 263. It is recommended that the silent and active phases in the Results section describe processes actually measured in this study as shown in 6A.

      To clarify, we did not use the term ‘membrane potential’ anywhere in the manuscript. We do sometimes refer to calcium influx as a proxy for membrane depolarization; we think this is valid given the abundant evidence that these processes are interdependent in beta cells.

      (4) It would be good to include the timing of the phases of calcium entry. When was the beta cell calcium entry monitored for the response time? Were the response times between the late and early phases consistent for each oscillation? It looks as if the start of the calcium upstroke was similar for many beta cells (such as for the Figure 2I traces). It would be nice to include a shorter time duration graph of calcium oscillation traces right when the upstroke starts. This would allow the community to observe the differences in the start time of calcium entry. 

      We agree this is an important point. We now include an inset showing the expanded time scale of the calcium upstroke in Fig.2I. The response time spread between early and late phase cells is now shown in Fig.7F (and in Author response image 2). We also quantified the coefficient of variation in the response time spread (0 = no variation and 1 = maximal variation) and found no significant differences between metabolic activators (Author response image 2). 

      Author response image 2.

      Also, for most of the GCaMP6s traces shown, the authors indicate that they are plotted as F/F0. However, this normalization (F/F0) is not done for the actual traces shown. For example, Figure 2D shows the traces starting from what looks to be 0 to 0.3 F/F0, but the traces for an F/F0 group should all start at 1. Please change this for all representative oscillations so the start of calcium entry for example traces all line up.

      This has been corrected in Fig. 2D, I and Fig. 3B. Also Fig.6 should be F not F/F0

      Reviewer #1 (Recommendations for the authors):

      (1) Line 53: "Silencing the electrical activity of these hub cells with optogenetics was found to abolish the coordination within that plane of the islet". The authors should acknowledge that studies also showed that beta cell transcription factor (Pdx1/Mafa) dosage was important for hub cell phenotype and islet function.

      Thank you, this reference to Nasteska et al. (PMID: 33514698, Ref. 16) has been added to the discussion.

      (2) Light sheet imaging is used to image the 3D islet volume. Whilst speed is undoubtedly an advantage of this technique, axial resolution is ~1.1 µm over 4 µm z-step size. How confident are the authors that single nuclei can be reliably identified given their ~6 µm size in a beta cell (e.g. do some elongated nuclear appear, which could be "doublets")?

      The axial resolution of 1.1 µm exceeds the resolution needed for the Nyquist criterion (i.e. sampling every 2-3 µm). As a practical matter, it is not possible to doublecount nuclei because the software will exclude nuclei that occupy the same volume. Only a very elongated nucleus (>10 µm) would be double counted and this does not occur.

      (3) The authors discuss the advantages of the light sheet imaging approach used, including speed and phototoxicity. Some more balance is needed here since other approaches such as two-photon excitation achieve similar speeds with much better axial resolution (see dozens of neural circuit studies).

      We are careful to point out that two-photon excitation has better axial resolution, better tissue penetration, and often higher speeds (kHz using linescans) – however these neuronal studies are limited to the cells in a few planes and the laser power is orders of magnitude higher than lightsheet. For this reason, two photon imaging has not been used to image islet calcium in three dimensions. The bottom line is lightsheet trades axial resolution for gentle volumetric imaging. 

      (4) Line 340: "Laser ablation or optogenetic inactivation of these early phase cells would be predicted to have little impact on islet function, as suggested previously by electrophysiological studies in which surface β-cells have been voltage-clamped with no impact on β-cell oscillations". This statement is slightly ambiguous since the authors showed in their previous studies that laser ablation of first responder cells/leaders was able to influence the Ca2+ network. Do the authors mean that laser ablation would only temporarily influence islet function before another cell picked up the role of a first responder/leader? As written, the sentence seems to imply that first responders/leaders are unimportant for the islet function.

      We intended to imply that the oscillatory system is sufficiently robust that a new cell take over when leader cells are ablated. We also cite Korosak et al. (PMID:34723613, Ref. 40) and Dwulet et al. (PMID: 33939712, Ref. 15) to make this point, although to clarify we are not examining first responders in this study.

      (5) Line 369: "In contrast with leader cells, we found that the highly synchronized cells are both spatially and temporally stable." The sentence needs qualifying- what would spatiotemporal stability be expected to confer on such a subpopulation?

      We believe that the spatiotemporal stability of highly synchronized cells is a consequence of beta cells in the center of the islet lacking the stochastic input of nearby alpha cells; we raise this point in the discussion: “The preponderance of α-cells on the periphery of mouse islets, which influence β-cell oscillation frequency, would be expected to disrupt β-cell synchronization on the periphery and stabilize it in the islet center – which is precisely the pattern of network activity we observed.” (see line 372). 

      (6) Line 370: "However, in conflict with the description of hub cells as intermingled with other cells throughout the islet, the location of such cells in 3D space is close to the center." The study by Johnston et al did not have the axial resolution to exclude that some cells might have been grouped together.

      We agree and have included the reviewer’s comment in the text (See line 384); that’s an important reason for conducting this 3D study.  

      (7) Line 380: "One explanation may be that paracrine communication within the islet determines which region of cells will show high or low degree. For example, more peripheral cells that are in contact with nearby δ-cells may show some suppression in their Ca2+ dynamics, and thus reduced synchronization." A potentially exciting future study. Should however probably cite DOI s41467-022-31373-6 here.

      We thank the reviewer for their input. This reference to Ren et al. (PMID:35764654) was previously included as Ref. 42 (now Ref. 45)

      Reviewer #3 (Recommendations for the authors):

      (1) There are in fact no radially oriented networks in the core of an islet (l. 130, Figure 4) apart from the fact that every hub has somewhat radially oriented edges. For radiality to have some general meaning, the normalized distance from the geometric center would need to be lower than 0.4. The networks are centrally located, which does not change the major conclusions of the study.

      Thank you for pointing out this imprecise language. We did not intend to imply that the functional network is orientated radially. We corrected the text (see line 131, 145) to indicate that the cells with high and low synchronization are distributed in a radial pattern. 

      (2) The study would benefit from acknowledging that Ca2+ influx is not a sole mechanism to drive insulin secretion and that KATP channels are not the sole target sensitive to changes in the cytosolic (global or local) ADP and ATP concentration or that there is an absolute concentration-dependence of these ligands on KATP channels. The relatively small conductance changes that have been found to be associated with active and silent phases (closing and opening of the KATP channels as interpreted by the authors, respectively, doi: 10.1152/ajpendo.00046.2013) and should be due to metabolic factors, could be also associated to desensitization of KATP channels to ATP due to the increase in cytosolic Ca2+ changes after intracellular Ca2+ flux (DOI: 10.1210/endo.143.2.8625) as they have been found to operate also at time scales, significantly faster (DOI: 10.2337/db22-0952) than reported before (refs. 21,22). Metabolic changes influence intracellular Ca2+ flux as well.

      The reviewer is absolutely correct that there are amplifying factors and other sources of calcium beyond plasma membrane influx and there are other mechanisms that regulate insulin secretion beyond calcium levels. These alternative mechanisms are introduced in Refs. 1-2, however they are not the focus of this study. 

      (3) There is no explanation for why KL divergence is so different between the pre-test regional consistency of the islets used to test the vehicle compared to those where GKa and PKa have been tested.

      We thank the reviewer for their careful observation. This arises because there are larger differences between preparations than within a preparation. This has been described previously (PMID: 16306370 and 20037650) and could be expected to account for the differences in KL divergence between animals. 

      (4) Statistical analysis would profit from testing the normality of the data distribution before choosing the statistical test and then learning the difference between parametric and nonparametric tests. For example, in Figures 3CD and 5EF, the data density is lower at the calculated mean than below and above this value and there are other examples in other figures too.

      We thank the reviewer for this very important comment, and we apologize for the oversight on our part. To address this comment, we conducted two normality tests: Anderson-Darling and Kolmogorov-Smirnov on all statistical analyses in the manuscript. If the data were not normally distributed, we changed the analysis to Wilcoxon matchedpairs signed rank test (non-parametric version of t-tests) or the Friedman test (nonparametric version of ANOVA). Three results were changed based on this statistical correction: Figure 4D, also 5F 3D (from P=0.01 to P=0.0526), Figure 5F  ¼ z-depth (P = 0.005 to P = 0.012). We have updated the manuscript methods, results, and figures accordingly. Importantly, these results did not change the main points of the paper.

    1. Author response:

      The following is the authors’ response to the original reviews.

      Public Reviews:

      Reviewer #1 (Public Review):

      Summary:

      The manuscript "Self-inhibiting percolation and viral spreading in epithelial tissue" describes a model based on 5-state cellular automata of development of an infection. The model is motivated and qualitatively justified by time-resolved measurements of expression levels of viral, interferon-producing, and antiviral genes. The model is set up in such a way that the crucial difference in outcomes (infection spreading vs. confinement) depends on the initial fraction of special virus-sensing cells. Those cells (denoted as 'type a') cannot be infected and do not support the propagation of infection, but rather inhibit it in a somewhat autocatalytic way. Presumably, such feedback makes the transition between two outcomes very sharp: a minor variation in concentration of ``a' cells results in qualitative change from one outcome to another. As in any percolation-like system, the transition between propagation and inhibition of infection goes through a critical state with all its attributes. A power-law distribution of the cluster size (corresponding to the fraction of infected cells) with a fairly universal exponent and a cutoff at the upper limit of this distribution.

      Strengths:

      The proposed model suggests an explanation for the apparent diversity of outcomes of viral infections such as COVID.

      Author response: We thank the referee for the concise and accurate summary of our work.

      Weaknesses:

      Those are not real points of weakness, though I think addressing them would substantially improve the manuscript.

      Author response: Below we will address these point by point.

      The key point in the manuscript is the reduction of actual biochemical processes to the NOVAa rules. I think more could be said about it, be it referring to a set of well-known connections between expression states of cells and their reaction to infection or justifying it as an educated guess.

      Author response: We have now improved this part in the model section. We have added a few sentences explaining how the cell state transitions are motivated by the UMAP results:

      “The cell state transitions triggered by IFN signaling or viral replication are known in viral infection, but how exactly the transitions are orchestrated for specific infections is poorly understood. The UMAP cell state distribution hints at possible preferred transitions between states. The closer two cell states are on the UMAP, the more likely transitions between them are, all else being equal. For instance, the antiviral state (𝐴) is easily established from a susceptible cell (𝑂), but not from the fully virus-hijacked cell (𝑉 ). The IFN-secreting cell state (𝑁) requires the co-presence of the viral and antiviral genes and thus the cell cluster is located between the antiviral state (𝐴) and virus-infected state (𝑉 ) but distant from the susceptible cells (𝑂).

      Inspired by the UMAP data visualization (Fig. 1a), we propose the following transitions between five main discrete cell states”

      Another aspect where the manuscript could be improved would be to look a little beyond the strange and 'not-so-relevant for a biomedical audience' focus on the percolation critical state. While the presented calculation of the precise percolation threshold and the critical exponent confirm the numerical skills of the authors, the probability that an actual infected tissue is right at the threshold is negligible. So in addition to the critical properties, it would be interesting to learn about the system not exactly at the threshold: For example, how the speed of propagation of infection depends on subcritical p_a and what is the cluster size distribution for supercritical p_a.

      Author response: We agree that further exploring the model away from the critical threshold is worthwhile. While our main focus has been on explaining the large degree of heterogeneity in outcomes – readily explained as a consequence of the sharp threshold-like behavior – we now include plots of the time-evolution of the infection (as well as the remaining states) over time for subcritical values of pa. The plots can be found in Figure S4 of the supplement.

      Reviewer #2 (Public Review):

      Xu et al. introduce a cellular automaton model to investigate the spatiotemporal spreading of viral infection. In this study, the author first analyzes the single-cell RNA sequencing data from experiments and identifies four clusters of cells at 48 hours post-viral infection, including susceptible cells (O), infected cells (V), IFN-secreting cells (N), and antiviral cells (A). Next, a cellular automaton model (NOVAa model) is introduced by assuming the existence of a transient pre-antiviral state (a). The model consists of an LxL lattice; each site represents one cell. The cells change their state following the rules depending on the interaction of neighboring cells. The model introduces a key parameter, p_a, representing the fraction of pre-antiviral state cells. Cell apoptosis is omitted in the model. Model simulations show a threshold-like behavior of the final attack rate of the virus when p_a changes continuously. There is a critical value p_c, so that when p_a < p_c, infections typically spread to the entire system, while at a higher p_a > p_c, the propagation of the infected state is inhibited. Moreover, the radius R that quantifies the diffusion range of N cells may affect the critical value p_c; a larger R yields a smaller value of the critical value p_c. The structure of clusters is different for different values of R; greater R leads to a different microscopic structure with fewer A and N cells in the final state. Compared with the single-cell RNA seq data, which implies a low fraction of IFN-positive cells - around 1.7% - the model simulation suggests R=5. The authors also explored a simplified version of the model, the OVA model, with only three states. The OVA model also has an outbreak size. The OVA model shows dynamics similar to the NOVAa model. However, the change in microstructure as a function of the IFN range R observed in the NOVAa model is not observed in the OVA model.

      Author response: We thank the referee for the comprehensive summary of our work.

      Data and model simulation mainly support the conclusions of this paper, but some weaknesses should be considered or clarified.

      Author response: Thank you - we will address these point by point below.

      (1) In the automaton model, the authors introduce a parameter p_a, representing the fraction of pre-antiviral state cells. The authors wrote: ``The parameter p_a can also be understood as the probability that an O cell will switch to the N or A state when exposed to the virus of IFNs, respectively.' Nevertheless, biologically, the fraction of pre-antiviral state cells does not mean the same value as the probability that an O cell switches to the N or A state. Moreover, in the numerical scheme, the cell state changes according to the deterministic role N(O)=a and N(a)=A. Hence, the probability p_a did not apply to the model simulation. It may need to clarify the exact meaning of the parameter p_a.

      Author response: We acknowledge that this was an imprecise formulation, and have now changed it.

      What we tried to convey with that comment was that, alternatively to having a certain fraction of cells be in the a state initially, one could instead have devised a model in which We should note that even the current model has a level of stochasticity, since we choose the cells to be updated with a constant probability rate - we choose N cells to update in each timestep, with replacement.

      However, based on your suggestion, we simulated a version of the dynamics which included stochastic conversion, i.e. each action of a cell on a nearby cell happens only with a probability p_conv (and the original model is recovered as the p_conv=1 scenario). Of course, this slows down the dynamics (or effectively rescales time by a factor p_conv), but crucially we find that it does not appreciably affect the location of the threshold p_c. Below we include a parameter scan across p_a values for R=1 and p_conv=0.5, which shows that the threshold continues to appear at around p_a=27%. each O-state cell simply had a probability to act as an a-state cell upon exposure to the virus or to interferons, i.e. to switch to an N state (if exposed to virus) or to the A state (if exposed to interferons). In this simplified model, there would be no functional difference, since it would simply amount to whether each cell had a probability to be designated an a-cell initially (as in our model), or upon exposure. So our remark mainly served to explain that the role of the p_a parameter is simply to encode that a certain fraction of virus-naive cells behave this way (whether predetermined or not).

      (2) The current model is deterministic. However, biologically, considering the probabilistic model may be more realistic. Are the results valid when the probability update strategy is considered? By the probability model, the cells change their state randomly to the state of the neighbor cells. The probability of cell state changes may be relevant for the threshold of p_a. It is interesting to know how the random response of cells may affect the main results and the critical value of p_a.

      Author response: This is a good point - we are firm believers in the importance of stochasticity. We should note that even the current model has a level of stochasticity, since we choose the cells to be updated with a constant probability rate - we choose N cells to update in each timestep, with replacement.

      However, based on your suggestion, we simulated a version of the dynamics which included stochastic conversion, i.e. each action of a cell on a nearby cell happens only with a probability p_conv (and the original model is recovered as the p_conv=1 scenario). Of course, this slows down the dynamics (or effectively rescales time by a factor p_conv), but crucially we find that it does not appreciably affect the location of the threshold p_c. Below we include a parameter scan across p_a values for R=1 and p_conv=0.5, which shows that the threshold continues to appear at around p_a=27%.

      We now discuss these findings in the supplement and include the figure below as Fig. S5.

      Author response image 1.

      (3) Figure 2 shows a critical value p_c = 27.8% following a simulation on a lattice with dimension L = 1000. However, it is unclear if dimension changes may affect the critical value.

      Author response: Re-running the simulations on a lattice 4x as large (i.e. L=2000) yields a similar critical value of 27-28% for R=1, so we are confident that finite size effects do not play a major role at L=1000 and beyond. For R=5, however, we find that a minimum lattice size greater than L=1000 is necessary to determine the critical threshold. Concretely, we find that the threshold value pc for R=5 changes somewhat when the lattice size is increased from 1000 to 2000, but is invariant under a change from 2000 to 3000, so we conclude that L=2000 is sufficient for R=5. The pc value for R=5 cited in the manuscript (~0.4%) was determined from simulations at L=2000.

      Reviewer #3 (Public Review):

      Summary:

      This study considers how to model distinct host cell states that correspond to different stages of a viral infection: from naïve and susceptible cells to infected cells and a minority of important interferon-secreting cells that are the first line of defense against viral spread. The study first considers the distinct host cell states by analyzing previously published single-cell RNAseq data. Then an agent-based model on a square lattice is used to probe the dependence of the system on various parameters. Finally, a simplified version of the model is explored, and shown to have some similarity with the more complex model, yet lacks the dependence on the interferon range. By exploring these models one gains an intuitive understanding of the system, and the model may be used to generate hypotheses that could be tested experimentally, telling us "when to be surprised" if the biological system deviates from the model predictions.

      Author response: Thank you for the summary! We agree with the role that you describe for a model such as this one.

      Strengths:

      -  Clear presentation of the experimental findings and a clear logical progression from these experimental findings to the modeling.

      -  The modeling results are easy to understand, revealing interesting behavior and percolation-like features.

      -  The scaling results presented span several decades and are therefore compelling. - The results presented suggest several interesting directions for theoretical follow-up work, as well as possible experiments to probe the system (e.g. by stimulating or blocking IFN secretion).

      Weaknesses:

      -  Since the "range" of IFN is an important parameter, it makes sense to consider lattice geometries other than the square lattice, which is somewhat pathological. Perhaps a hexagonal lattice would generalize better.

      -  Tissues are typically three-dimensional, not two-dimensional. (Epithelium is an exception). It would be interesting to see how the modeling translates to the three-dimensional case. Percolation transitions are known to be very sensitive to the dimensionality of the system.

      Author response: We agree that probing different lattice geometries (2- and 3-dimensional alike) would be interesting and worthwhile. However, for this manuscript, we prefer to confine the analysis to the current, simple case. We do agree, however, that an extensive exploration of the role of geometry is an interesting future possibility.

      -  The fixed time-step of the agent-based modeling may introduce biases. I would consider simulating the system with Gillespie dynamics where the reaction rates depend on the ambient system parameters.

      -  Single-cell RNAseq data typically involves data imputation due to the high sparsity of the measured gene expression. More information could be provided on this crucial data processing step since it may significantly alter the experimental findings.

      Justification of claims and conclusions:

      The claims and conclusions are well justified.

      Recommendations for the authors:

      Reviewer #1 (Recommendations For The Authors):

      It is necessary to explain what UMAP does. Is clustering done in the space of twenty-something original dimensions or 2D? How UMAP1 and UMAP2 are selected and are those the same in all plots?

      Author response: We have now added a few sentences to clarify the point raised above - the second snippet explains how clustering is performed:

      “As a dimension reduction algorithm, UMAP is a manifold learning technique that favors the preservation of local distances over global distances (McInnes et al., 2018; Becht et al., 2019). It constructs a weighted graph from the data points and optimizes the graph layout in the low-dimensional space.”

      “We cluster the cells with the principal components analysis (PCA) results from their gene expression. With the first 16 principal components, we calculate k-nearest neighbors and construct the shared nearest neighbor graph of the cells then optimize the modularity function to determine clusters. We present the cluster information on the UMAP plane and use the same UMAP coordinates for all the plots in this paper hereafter.”

      Figure 1, what do bars in the upper right corners of panels d,e,f, and g indicate? ``Averaged' refers to time average? Something is missing in ``Cell proportions are labeled with corresponding colors in a)' .

      Author response: Thank you - we have now modified the figure caption. The bars in the upper right corners of panels d, e, f are color keys for gene expression, the brighter the color is, the higher the gene expression is.

      “Averaged” gene expression refers to the mean expression of that particular gene across the cells within each indicated cluster.

      The lines in c) correspond to cell proportions in different states at different time points. The same state in 1) and c) is shown in the same color.

      Line 46, ``However' does not sound right in this context. Would ``Also' be better?

      Author response: We agree and have corrected it in the revised manuscript.

      Line 96``The viral genes are also partially expressed in these cells, but different from the 𝑁 cluster, the antiviral genes are fully expressed (Fig. S1 and S2).' The sentence needs to be rephrased.

      Author response: We have rephrased the sentence: “As in the N cluster, the viral gene E is barely detected in these cells, indicating incomplete viral replication. However, in contrast to the N cluster, the antiviral genes are expressed to their full extent (Fig. S1 and S2).”

      Line 126, missing "be", ``large' -> ``larger'.

      Author response: Thank you, we have now corrected these typos.

      Line 139-140 The logical link between ignoring apoptosis and the diffusion of IFN is unclear.

      Author response: We modified the sentence as “Here, we assume that the secretion of IFNs by the 𝑁 cells is a faster process than possible apoptosis (Wen et al., 1997; Tesfaigzi, 2006) of these cells and that the diffusion of IFNs to the neighborhood is not significantly affected by apoptosis.”

      Fig. 2a Do the yellow arrows show the effect of IFN and the purple arrows the propagation of viral infection?

      Author response: That is correct. We have added this information to the figure caption: “The straight black arrows indicate transitions between cell states. The curved yellow arrows indicate the effects of IFNs on activating antiviral states. The curved purple arrows indicate viral spread to cells with 𝑂 and 𝑎 states.”

      Fig. 3, n(s) as the axis label vs P(s) in the text? How do the curves in panel a) look when the p_a is well above or below p_c?

      Author response: Thank you. We have edited the labels in the figure to reflect the symbols used in the text.

      Boundary conditions? From Fig. 4, apparently periodic?

      Author response: Yes, we use periodic boundary conditions in the model. We clarify it in the model section now (last sentence).

      It will be good to see a plot with time dependences of all cell types for a couple of values of p_a, illustrating propagation and cessation of the infection.

      Author response: We agree, and have added a Figure S4 in the supplement which explores exactly that. Thank you for the suggestion.

      A verbal qualitative description of why p_a has such importance and how the infection is terminated for large p_a would help.

      Reviewer #2 (Recommendations For The Authors):

      Below are two minor comments:

      (1) In the single-cell RNA sequencing data analysis, the authors describe the cell clusters O, V, A, and N. However, showing how the clusters are identified from the data might be more straightforward.

      Author response: Technically, we cluster the cells using principal components analysis (PCA) results of their gene expression. With the first 16 principal components, we calculate k-nearest neighbors and construct the shared nearest neighbor graph of the cells and then optimize the modularity function to determine clusters. We manually annotate the clusters with O, V, A, and N based on the detected abundance of viral genes, antiviral genes, and IFNs.

      (2) In Figure 3, what does n(s) mean in Figure 3a? And what is the meaning of the distribution P(s) of infection clusters? It may be stated clearly.

      Author response: The use of n(s) was inconsistent, and we have now edited the figure to instead say P(s), to harmonize it with the text. P(s) is the distribution of cluster sizes, s, expressed as a fraction of the whole system. In other words, once a cluster has reached its final size, we record s=(N+V)/L^2 where N and V are the number of N and V state cells in the cluster (note that, by design, each simulation leads to a single cluster, since we seed the infection in one lattice point). We now indicate more clearly in the caption and the main text what exactly P(s) and s refer to.

      Reviewer #3 (Recommendations For The Authors):

      - Would the authors kindly share the simulation code with the community? Also, the data analysis code should be shared to follow current best practices. This needs to be standard practice in all publications. I would go as far as to say that in 2024 publishing a data analysis / simulation study without sharing the relevant code should be ostracized by the community.

      Author response: We absolutely agree and have created a GitHub repository in which we share the C++ source code for the simulations and a Python notebook for plotting. The public repository can be found at https://github.com/BjarkeFN/ViralPercolation. We add this information in supplement under section “Code availability”.

      ­

      - I would avoid the use of the wording "critical" threshold since this is almost guaranteed to infuriate a certain type of reader.

      ­

      - Line 265 has a curious use of " ... " which should be replaced with something more appropriate.

      Author response: Thank you for pointing it out! We have checked the typos.

    1. Author response:

      The following is the authors’ response to the original reviews

      Reviewer #1:

      The manuscript suggests the zebrafish homolog of ctla-4 and generates a new mutant in it. However, the locus that is mutated is confusingly annotated as both CD28 (current main annotation in ZFIN) and CTLA-4/CD152 (one publication from 2020), see: https://zfin.org/ZDB-GENE-070912-128. Both human CTLA-4 and CD28 align with relatively similar scores to this gene. There seem to be other orthologs of these receptors in the zebrafish genome, including CD28-like (https://zfin.org/ZDB-GENE-070912-309) which neighbors the gene annotated as CD28 (exhibiting similar synteny as human CD28 and CTLA-4). It would be helpful to provide more information to distinguish between this family of genes and to further strengthen the evidence that this mutant is in ctla-4, not cd28. Also, is one of these genes in the zebrafish genome (e.g. cd28l) potentially a second homolog of CTLA-4? Is this why this mutant is viable in zebrafish and not mammals? Some suggestions:

      (a) A more extensive sequence alignment that considers both CTLA-4 and CD28, potentially identifying the best homolog of each human gene, especially taking into account any regions that are known to produce the functional differences between these receptors in mammals and effectively assigns identities to the two genes annotated as "cd28" and "cd28l" as well as the gene "si:dkey-1H24.6" that your CD28 ORF primers seem to bind to in zebrafish.

      In response to the reviewer's insightful suggestions, we have conducted more extensive sequence alignment and phylogenetic analyses that consider both CTLA-4, CD28, and CD28-like molecules, taking into account key regions crucial for the functionalities and functional differences between these molecules across various species, including mammals and zebrafish.

      Identification of zebrafish Ctla-4: We identified zebrafish Ctla-4 as a homolog of mammalian CTLA-4 based on key conserved structural and functional characteristics. Structurally, the Ctla-4 gene shares similar exon organization compared to mammalian CTLA-4. Ctla-4 is a type I transmembrane protein with typical immunoglobulin superfamily features. Multiple amino acid sequence alignments revealed that Ctla-4 contains a <sup>113</sup>LFPPPY<sup>118</sup> motif and a <sup>123</sup>GNGT<sup>126</sup> motif in the ectodomain, and a tyrosine-based <sup>206</sup>YVKF<sup>209</sup> motif in the distal C-terminal region. These motifs closely resemble MYPPPY, GNGT, and YVKM motifs in mammalian CTLA-4s, which are essential for binding to CD80/CD86 ligands and molecular internalization and signaling inhibition. Despite only 23.7% sequence identity to human CTLA-4, zebrafish Ctla-4 exhibits a similar tertiary structure with a two-layer β-sandwich architecture in its extracellular IgV-like domain. Four cysteine residues responsible for the formation of two pairs of disulfide bonds (Cys<sup>20</sup>-Cys<sup>91</sup>/Cys<sup>46</sup>-Cys<sup>65</sup> in zebrafish and Cys<sup>21</sup>-Cys<sup>92</sup>/Cys<sup>48</sup>-Cys<sup>66</sup> in humans) that connect the two-layer β-sandwich are conserved. Additionally, a separate cysteine residue (Cys<sup>120</sup> in zebrafish and Cys<sup>120</sup> in humans) involved in dimerization is also present, and Western blot analysis under reducing and non-reducing conditions confirmed Ctla-4’s dimerization. Phylogenetically, Ctla-4 clusters with other known CTLA-4 homologs from different species with high bootstrap probability, while zebrafish Cd28 groups separately with other CD28s. Functionally, Ctla-4 is predominantly expressed on CD4<sup>+</sup> T and CD8<sup>+</sup> T cells in zebrafish. It plays a pivotal inhibitory role in T cell activation by competing with CD28 for binding to CD80/86, as validated through a series of both in vitro and in vivo assays, including microscale thermophoresis assays which demonstrated that Ctla-4 exhibits a significantly higher affinity for Cd80/86 than Cd28 (KD = 0.50 ± 0.25 μM vs. KD = 2.64 ± 0.45 μM). These findings confirm Ctla-4 as an immune checkpoint molecule, reinforcing its identification within the CTLA-4 family.

      Comparison between zebrafish Cd28 and "Cd28l": Zebrafish Cd28 contains an extracellular SYPPPF motif and an intracellular FYIQ motif. The extracellular SYPPPF motif is essential for binding to Cd80/CD86, while the intracellular FYIQ motif likely mediates kinase recruitment and co-stimulatory signaling. In contrast, the "Cd28l" molecule lacks the SYPPPF motif, which is critical for Cd80/CD86 binding, and exhibits strong similarity in its C-terminal 79 amino acids to Ctla-4 rather than Cd28. Consequently, "Cd28l" resembles an atypical Ctla-4-like molecule but fails to exhibit Cd80/CD86 binding activity.

      We have incorporated the relevant analysis results into the main text of the revised manuscript and updated Supplementary Figure 1. Additionally, we provide key supplementary analyses here for the reviewer's convenience.  

      Author response image 1.

      Illustrates the alignment of Ctla-4 (XP_005167576.1) and Ctla-4-like (XP_005167567.1, previously referred to as "Cd28l") in zebrafish, generated using ClustalX and Jalview. Conserved and partially conserved amino acid residues are highlighted in color gradients ranging from carnation to red, respectively. The B7-binding motif is encircled with a red square.

      (b) Clearer description in the main text of such an analysis to better establish that the mutated gene is a homolog of ctla-4, NOT cd28.

      We appreciate the reviewer's advice. Additional confirmation of zebrafish Ctla-4 is detailed in lines 119-126 of the revised manuscript.

      (c) Are there mammalian anti-ctla-4 and/or anti-cd28 antibodies that are expected to bind to these zebrafish proteins? If so, looking to see whether staining is lost (or western blotting is lost) in your mutants could be additionally informative. (Our understanding is that your mouse anti-Ctla-4 antibody is raised against recombinant protein generated from this same locus, and so is an elegant demonstration that your mutant eliminates the production of the protein, but unfortunately does not contribute additional information to help establish its homology to mammalian proteins).

      This suggestion holds significant value. However, a major challenge in fish immunology research is the limited availability of antibodies suitable for use in fish species; antibodies developed for mammals are generally not applicable. We attempted to use human and mouse anti-CTLA-4 and anti-CD28 antibodies to identify Ctla-4 and Cd28 in zebrafish, but the results were inconclusive, with no expected signals. This outcome likely arises from the low sequence identity between human/mouse CTLA-4 and CD28 and their zebrafish homologs (ranging from 21.3% to 23.7% for CTLA-4 and 21.2% to 24.0% for CD28). Therefore, developing specific antibodies against zebrafish Ctla-4 is essential for advancing this research.

      The methods section is generally insufficient and doesn't describe many of the experiments performed in this manuscript. Some examples:

      (a) No description of antibodies used for staining or Western blots (Figure1C, 1D, 1F).

      (b) No description of immunofluorescence protocol (Figure 1D, 1F).

      (c) No description of Western blot protocol (Figure 1C, 2C).

      (d) No description of electron microscopy approach (Figure 2K).

      (e) No description of the approach for determining microbial diversity (Entirety of Figure 6).

      (f) No description of PHA/CFSE/Flow experiments (Figure 7A-E).

      (g) No description of AlphaFold approach (Figures 7F-G).

      (h) No description of co-IP approach (Figure 7H).

      (i) No description of MST assay or experiment (Figure 7I).

      (j) No description of purification of recombinant proteins, generation of anti-Ctla-4 antibody, or molecular interaction assays (Figures S2 and S6).

      We apologize for this oversight. The methods section was inadvertently incomplete due to an error during the file upload process at submission. This issue has been addressed in the revised manuscript. We appreciate your understanding.

      Figure 5 suggests that there are more Th2 cells 1, Th2 cells 2, and NKT cells in ctla-4 mutants through scRNA-seq. However, as the cell numbers for these are low in both genotypes, there is only a single replicate for each genotype scRNA-seq experiment, and dissociation stress can skew cell-type proportions, this finding would be much more convincing if another method that does not depend on dissociation was used to verify these results. Furthermore, while Th2 cells 2 are almost absent in WT scRNA-seq, KEGG analysis suggests that a major contributor to their clustering may be ribosomal genes (Fig. 5I). Since no batch correction was described in the methods, it would be beneficial to verify the presence of this cluster in ctla-4 mutants and WT animals through other means, such as in situ hybridization or transgenic lines.   

      We are grateful for the insightful comments provided by the reviewer. Given that research on T cell subpopulations in fish is still in its nascent stages, the availability of specific marker antibodies and relevant transgenic strains remains limited. Our single-cell RNA sequencing (scRNA-seq) analysis revealed that a distinct Th2 subset 2 was predominantly observed in Ctla-4 mutants but was rare in wild-type zebrafish, it suggests that this subset may primarily arise under pathological conditions associated with Ctla-4 mutation. Due to the near absence of Th2 subset 2 in wild-type samples, KEGG enrichment analysis was performed exclusively on this subset from Ctla-4-deficient intestines. The ribosome pathway was significantly enriched, suggesting that these cells may be activated to fulfill their effector functions. However, confirming the presence of Th2 subset 2 using in situ hybridization or transgenic zebrafish lines is currently challenging due to the lack of lineage-specific markers for detailed classification of Th2 cell subsets and the preliminary nature of scRNA-seq predictions.

      To address the reviewers' suggestion to confirm compositional changes in Th2 and NKT cells using dissociation-independent methods, we quantified mRNA levels of Th2 (il4, il13, and gata3) and NKT (nkl.2, nkl.4, and prf1.1) cell marker genes via RT-qPCR in intestines from wild-type and mutant zebrafish. As shown in Figure S7B and S7C, these markers were significantly upregulated in Ctla-4-deficient intestines compared to wild-type controls. This indicates an overall increase in Th2 and NKT cell activity in mutant zebrafish, aligning with our scRNA-seq analysis and supports the validity of our initial findings.

      Before analyzing the scRNA-seq data, we performed batch correction using the Harmony algorithm via cloud-based Cumulus v1.0 on the aggregated gene-count matrices. This methodological detail has been included in the “Materials and Methods” section of the revised manuscript. Moreover, the RT-qPCR results are presented in Supplementary Figures S7B and S7C.

      Quality control (e.g., no. of UMIs, no. of genes, etc.) metrics of the scRNAseq experiments should be presented in the supplementary information for each sample to help support that observed differential expression is not merely an outcome of different sequencing depths of the two samples.

      As illustrated in Fig. S5, the quality control data have been supplemented to include the effective cell number of the sample, along with pre- and post-filtering metrics such as nFeature_RNA, nCount_RNA and mitochondrial percentage (percent.mito). Furthermore, scatter plots comparing the basic information of the sample cells before and after filtering are provided.

      Some references to prior research lack citations. Examples:

      (a)"Given that Ctla-4 is primarily expressed on T cells (Figure 1E-F), and its absence has been shown to result in intestinal immune dysregulation, indicating a crucial role of this molecule as a conserved immune checkpoint in T cell inhibition."

      The references were incorporated into line 71 of the revised manuscript.

      (b) Line 83: Cite evidence/review for the high degree of conservation in adaptive immunity.

      The references were incorporated into line 93 of the revised manuscript.

      (c) Lines 100-102: Cite the evidence that MYPPPY is a CD80/86 binding motif.

      The references were incorporated into line 117 of the revised manuscript.

      The text associated with Figure 8 (Lines 280-289) does not clearly state that rescue experiments are being done in mutant zebrafish.

      We have provided a clear explanation of the rescue experiments conducted in Ctla-4-deficient zebrafish. This revision has been incorporated into line 319.

      Line 102: Is there evidence from other animals that LFPPPY can function as a binding site for CD80/CD86? Does CD28 also have this same motif?

      The extracellular domains of CTLA-4 and CD28, which bind to CD80/CD86, are largely conserved across various species. This conservation is exemplified by a central PPP core motif, although the flanking amino acids exhibit slight variations. In mammals, both CTLA-4 and CD28 feature the conserved MYPPPY motif. By contrast, in teleost fish, such as rainbow trout, CTLA-4 contains an LYPPPY motif, while CD28 has an MYPPPI motif (Ref. 1). Grass carp CTLA-4 displays an LFPPPY motif, whereas its CD28 variant bears an IYPPPF motif. Yeast two-hybrid assays confirm that these motifs facilitate interactions between grass carp CTLA-4 and CD28 with CD80/CD86 (Ref. 2). Similarly, zebrafish Ctla-4 contains the LFPPPY motif observed in grass carp, while Cd28 exhibits a closely related SYPPPF motif.

      References:

      (1) Bernard, D et al. (2006) Costimulatory Receptors in a Teleost Fish: Typical CD28, Elusive CTLA-4. J Immunol. 176: 4191-4200.

      (2) Lu T Z et al. (2022) Molecular and Functional Analyses of the Primordial Costimulatory Molecule CD80/86 and Its Receptors CD28 and CD152 (CTLA-4) in a Teleost Fish. Frontiers in Immunology. 13:885005.

      Line 110-111: Suggest adding citation of these previously published scRNAseq data to the main text in addition to the current description in the Figure legend.

      The reference has been added in line 129 in the main text.

      Figure 3B: It would be helpful to label a few of the top differentially expressed genes in Panel B?

      The top differentially expressed genes have been labeled in Figure 3B.

      Figure 3G: It's unclear how this analysis was conducted, what this figure is supposed to demonstrate, and in its current form it is illegible.

      Figure 3G displays a protein-protein interaction network constructed from differentially expressed genes. The densely connected nodes, representing physical interactions among proteins, provide valuable insights for basic scientific inquiry and biological or biomedical applications. As proteins are crucial to diverse biological functions, their interactions illuminate the molecular and cellular mechanisms that govern both healthy and diseased states in organisms. Consequently, these networks facilitate the understanding of pathogenic and physiological processes involved in disease onset and progression.

      To construct this network, we first utilized the STRING database (https://string-db.org) to generate an initial network diagram using the differentially expressed genes. This diagram was subsequently imported into Cytoscape (version 3.9.1) for visualization and further analysis. Node size and color intensity reflect the density of interactions, indicating the relative importance of each protein. Figure 3G illustrates that IL1β was a central cytokine hub in the disease process of intestinal inflammation in Ctla-4-deficient zebrafish.

      Expression scale labeling:

      (a) Most gene expression scales are not clearly labeled: do they represent mean expression or scaled expression? Has the expression been log-transformed, and if so, which log (natural log? Log10? Log2?). See: Figure 3E, 3I, 4D, 4E, 5B, 5G, 5H, 6I.

      The gene expression scales are detailed in the figure legends. Specifically, Figures 3E, 3I, and 6I present heatmaps depicting row-scaled expression levels for the corresponding genes. In contrast, Figures 4D and 4E display heatmaps illustrating the mean expression of these genes. Additionally, the dot plots in Figures 5B, 5G, and 5H visualize the mean expression levels of the respective genes.

      (b) For some plots, diverging color schemes (i.e. with white/yellow in the middle) are used for non-diverging scales and would be better represented with a sequential color scale. See: 4D, 4E, and potentially others (not fully clear because of the previous point).

      The color schemes in Figures 4D and 4E have been updated to a sequential color scale. The gene expression data depicted in these figures represent mean expression values and have not undergone log transformation. This information has been incorporated into the figure legend for clarity.

      Lines 186-187: Though it is merely suggested, apoptotic gene expression can be upregulated as part of the dissociation process for single-cell RNAseq. This would be much stronger if supported by a staining, such as anti-Caspase 3.

      Following the reviewer's insightful recommendations, we conducted a TUNEL assay to evaluate apoptosis in the posterior intestinal epithelial cells of both wild-type and Ctla-4-deficient zebrafish. As expected, our results demonstrate a significant increase in epithelial cell apoptosis in Ctla-4-deficient zebrafish compared with wild-type fish. The corresponding data are presented in Figure S6D and have been incorporated into the manuscript. Detailed protocols for the TUNEL assay have also been included in the Materials and Methods section.

      Author response image 2.

      Illustrates the quantification of TUNEL-positive cells per 1 × 10<sup>4</sup> μm<sup>2/⁻</sup> in the posterior intestines of both wild-type (WT) and ctla-4<sup>⁻/⁻</sup> zebrafish (n = 5). The data demonstrate a comparative analysis of apoptotic cell density between the two genotypes.

      Lines 248-251: This manuscript demonstrates gut inflammation and also changes in microbial diversity, but I don't think it demonstrates an association between them, which would require an experiment that for instance rescues one of these changes and shows that it ameliorates the other change, despite still being a ctla-4 mutant.

      We appreciate the valuable comments from the reviewer. Recently, the relationship between inflammatory bowel disease (IBD) and gut microbial diversity has garnered considerable attention, with several key findings emerging from human IBD studies. For instance, patients with IBD (including ulcerative colitis and Crohn's disease) exhibit reduced microbial diversity, which is correlated with disease severity. This decrease in microbial richness is thought to stem from the loss of normal anaerobic bacteria, such as Bacteroides, Eubacterium, and Lactobacillus (Refs. 1-6). Research using mouse models has shown that inflammation increases oxygen and nitrate levels within the intestinal lumen, along with elevated host-derived electron acceptors, thereby promoting anaerobic respiration and overgrowth of Enterobacteriaceae (Ref 7). Consistent with these findings, our study observed a significant enrichment of Enterobacteriaceae in the inflamed intestines of Ctla-4-deficient zebrafish, which supporting the observations in mice. Despite this progress, the zebrafish model for intestinal inflammation remains under development, with limitations in available techniques for manipulating intestinal inflammation and reconstructing gut microbiota. These challenges hinder investigations into the association between intestinal inflammation and changes in microbial diversity. We plan to address these issues through ongoing technological advancements and further research. We thank the reviewer for their understanding.

      References:

      (1) Ott S J, Musfeldt M, Wenderoth D F, Hampe J, Brant O, Fölsch U R et al. (2004) Reduction in diversity of the colonic mucosa associated bacterial microflora in patients with active inflammatory bowel disease. Gut 53:685-693.

      (2) Manichanh C, Rigottier-Gois L, Bonnaud E, Gloux K, Pelletier E, Frangeul L et al. (2006) Reduced diversity of faecal microbiota in Crohn's disease revealed by a metagenomic approach. Gut 55:205-211.

      (3) Qin J J, Li R Q, Raes J, Arumugam M, Burgdorf K S, Manichanh C et al. (2010) A human gut microbial gene catalogue established by metagenomic sequencing. Nature 464:59-U70.

      (4) Sha S M, Xu B, Wang X, Zhang Y G, Wang H H, Kong X Y et al. (2013) The biodiversity and composition of the dominant fecal microbiota in patients with inflammatory bowel disease. Diagn Micr Infec Dis 75:245-251.

      (5) Ray K. (2015) IBD. Gut microbiota in IBD goes viral. Nat Rev Gastroenterol Hepatol 12:122.

      (6) Papa E, Docktor M, Smillie C, Weber S, Preheim S P, Gevers D et al. (2012) Non-Invasive Mapping of the Gastrointestinal Microbiota Identifies Children with Inflammatory Bowel Disease. Plos One 7: e39242-39254.

      (7) Hughes E R, Winter M G, Duerkop B A, Spiga L, de Carvalho T F, Zhu W H et al. (2017) Microbial Respiration and Formate Oxidation as Metabolic Signatures of Inflammation-Associated Dysbiosis. Cell Host Microbe 21:208-219.

      Lines 270-272 say that interaction between Cd28/ctla-4 and Cd80/86 was demonstrated through bioinformatics, flow-cytometry, and Co-IP. Does this need to reference Fig S6D for the flow data? Figures 7F-G are very hard to read or comprehend as they are very small. Figure 7H is the most compelling evidence of this interaction and might stand out better if emphasized with a sentence referencing it on its own in the manuscript. 

      In this study, we utilized an integrated approach combining bioinformatics prediction, flow cytometry, and co-immunoprecipitation (Co-IP) to comprehensively investigate and validate the interactions between Cd28/Ctla-4 and Cd80/86. Flow cytometry analysis, as depicted in Supplementary Figure 6D (revised as Supplementary Figure 8F), demonstrated the surface expression of Cd80/86 on HEK293T cells and quantified their interactions with Cd28 and Ctla-4. These experiments not only validated the interactions between Cd80/86 and Cd28/Ctla-4 but also revealed a dose-dependent relationship, providing robust supplementary evidence for the molecular interactions under investigation. Furthermore, in Figure 7F-G, the axis font sizes were enlarged to improve readability. Additionally, in response to reviewers' feedback, we have emphasized Figure 7H, which presents the most compelling evidence for molecular interactions, by including a standalone sentence in the text to enhance its prominence.

      For Figure 7A-E, for non-immunologists, it is unclear what experiment was performed here - it would be helpful to add a 1-sentence summary of the assay to the main text or figure legend.

      We apologize for this oversight. Figures 7A–E illustrate the functional assessment of the inhibitory role of Ctla-4 in Cd80/86 and Cd28-mediated T cell activation. A detailed description of the methodologies associated with Figures 7A–E is provided in the ‘Materials and Methods’ section of the revised manuscript.

      For Figure 7F-G, it is extremely hard to read the heat map legends and the X and Y-axis. Also, what the heatmaps show and how that fits the overall narrative can be elaborated significantly.

      We regret this oversight. To enhance clarity, we have increased the font size of the heatmap legends and the X and Y-axes, as shown in the following figure. Additionally, a detailed analysis of these figures is provided in lines 299–306 of the main text.

      In general, the main text that accompanies Figure 7 should be expanded to more clearly describe these experiments/analyses and their results.

      We have conducted a detailed analysis of the experiments and results presented in Figure 7. This analysis is described in lines 278-314.

      Reviewer #2:

      The scRNASeq assay is missing some basic characterization: how many WT and mutant fish were assayed in the experiment? how many WT and mutant cells were subject to sequencing? Before going to the immune cell types, are intestinal cell types comparable between the two conditions? Are there specific regions in the tSNE plot in Figure 4A abundant of WT or ctla-4 mutant cells?

      In the experiment, we analyzed 30 wild-type and 30 mutant zebrafish for scRNA-seq, with an initial dataset comprising 8,047 cells in the wild-type group and 8,321 cells in the mutant group. Sample preparation details are provided on lines 620-652. Due to the relatively high expression of mitochondrial genes in intestinal tissue, quality control filtering yielded 3,263 cells in the wild-type group and 4,276 cells in the mutant group. Given that the intestinal tissues were dissociated using identical protocols, the resulting cell types are comparable between the two conditions. Both the wild-type and Ctla-4-deficient groups contained enterocytes, enteroendocrine cells, smooth muscle cells, neutrophils, macrophages, B cells, and a cluster of T/NK/ILC-like cells. Notably, no distinct regions were enriched for either condition in the tSNE plot (Figure 4A).

      The cell proliferation experiment using PHA stimulation assay demonstrated the role of Ctla-4 in cell proliferation, while the transcriptomic evidence points towards activation rather than an overall expansion of T-cell numbers. This should be discussed towards a more comprehensive model of how subtypes of cells can be differentially proliferating in the disease model.

      In the PHA-stimulated T cell proliferation assay, we aimed to investigate the regulatory roles of Ctla-4, Cd28, and Cd80/86 in T cell activation, focusing on validating Ctla-4's inhibitory function as an immune checkpoint. While our study examined general regulatory mechanisms, it did not specifically address the distinct roles of Ctla-4 in different T cell subsets. We appreciate the reviewer's suggestion to develop a more comprehensive model that elucidates differential T cell activation across various subsets in disease models. However, due to the nascent stage of research on fish T cell subsets and limitations in lineage-specific antibodies and transgenic strains, such investigations are currently challenging. We plan to pursue these studies in the future. Despite these constraints, our single-cell RNA sequencing data revealed an increased proportion of Th2 subset cells in Ctla-4-deficient zebrafish, as evidenced by elevated expression levels of Th2 markers (Il4, Il13, and Gata3) via RT-qPCR (see Figures S7B). Notably, recent studies in mouse models have shown that naïve T cells from CTLA-4-deficient mice tend to differentiate into Th2 cells post-proliferation, with activated Th2 cells secreting higher levels of cytokines like IL-4, IL-5, and IL-13, thereby exerting their effector functions (Refs. 1-2). Consequently, our findings align with observations in mice, suggesting conserved CTLA-4 functions across species. We have expanded the "Discussion" section to clarify these points.

      References:

      (1) Bour-Jordan H, Grogan J L, Tang Q Z, Auger J A, Locksley R M, Bluestone J A et al. (2003) CTLA-4 regulates the requirement for cytokine-induced signals in T<sub>H</sub>2 lineage commitment. Nature Immunology 4: 182-188.

      (2) Khattri Roli, Auger, Julie A, Griffin Matthew D, Sharpe Arlene H, Bluestone Jeffrey A et al. (1999) Lymphoproliferative Disorder in CTLA-4 Knockout Mice Is Characterized by CD28-Regulated Activation of Th2 Responses. The Journal of Immunology 162:5784-5791.

      It would be nice if the authors could also demonstrate whether other tissues in the zebrafish have an inflammation response, to show whether the model is specific to IBD.

      In addition to intestinal tissues, we also performed histological analysis on the liver of Ctla-4-deficient zebrafish. The results showed that Ctla-4 deficiency led to mild edema in a few hepatocytes, and lymphocyte infiltration was not significant. Compared to the liver, we consider intestinal inflammation to be more pronounced.

      Some minor comments on terminology

      (a) "multiomics" usually refers to omics experiments with different modalities (e.g. transcriptomics, proteomics, metabolomics etc), while the current paper only has transcriptomics assays. I wouldn't call it "multiomics" analysis.

      We appreciate the reviewer's attention to this issue. The "multi-omics" has been revised to "transcriptomics".

      (b) In several parts of the figure legend the author mentioned "tSNE nonlinear clustering" (Figures 4A and 5A). tSNE is an embedding method rather than a clustering method.

      The "tSNE nonlinear clustering" has been revised to "tSNE embedding”.

      (c) Figure 1E is a UMAP rather than tSNE.

      The "tSNE" has been revised to "UMAP" in the figure legend in line 1043.

      Reviewer #3: 

      Line 28: The link is not directly reflected in this sentence describing CTLA-4 knockout mice.

      We appreciate the reviewer for bringing this issue to our attention. We have expanded our description of CTLA-4 knockout mice on lines 77-84.

      Line 80-83: There is a lack of details about the CTLA-4-deficient mice. The factor that Th2 response could be induced has been revealed in mouse model. See the reference entitled "CTLA-4 regulates the requirement for cytokine-induced signals in TH2 lineage commitment" published in Nature Immunology.

      We thank the reviewer for providing valuable references. We have added descriptions detailing the differentiation of T cells into Th2 cells in CTLA-4-deficient mice on lines 78–81, and the relevant references have been cited in the revised manuscript.

      To better introduce the CTLA-4 immunobiology, the paper entitled "Current Understanding of Cytotoxic T Lymphocyte Antigen-4 (CTLA-4) Signaling in T-Cell Biology and Disease Therapy" published in Molecules and Cells should be referred.

      We have provided additional details on CTLA-4 immunology (lines 75-84) and have included the relevant reference in the revised manuscript.

      In current results, there are many sentences that should be moved to the discussion, such as lines 123-124, lines 152-153, lines 199-200, and lines 206-207. So, the result sections just describe the results, and the discussions should be put together in the discussion.

      We have relocated these sentences to the 'Discussion' section and refined the writing.

      In the discussion, the zebrafish enteritis model, such as DSS/TNBS and SBMIE models, should also be compared with the current CTLA-4 knockout model. Also, the comparison between the current fish IBD model and the previous mouse model should also be included, to enlighten the usage of CTLA-4 knockout zebrafish IBD model.

      We compared the phenotypes of our current Ctla-4-knockout zebrafish IBD model with other models, including DSS-induced IBD models in zebrafish and mice, as well as TNBS- and SBM-induced IBD models in zebrafish. The details are included in the "Discussion" section (lines 353-365).

      As to the writing, the structure of the discussion is poor. The paragraphs are very long and hard to follow. Many findings from current results were not yet discussed. I just can't find any discussion about the alteration of intestinal microbiota.

      In response to the reviewers' constructive feedback, we have revised and enhanced the discussion section. Furthermore, we have integrated the most recent research findings relevant to this study into the discussion to improve its relevance and comprehensiveness.

      In the discussion, the aerobic-related bacteria in 16s rRNA sequencing results should be focused on echoing the histopathological findings, such as the emptier gut of CTLA-4 knockout zebrafish.

      As mentioned above, the discussion section has been revised and expanded to provide a better understanding of the potential interplay among intestinal inflammatory pathology, gut microbiota alterations, and immune cell dysregulation in Ctla-4-deficient zebrafish. Furthermore, promising avenues for future research that warrant further investigation were also discussed.

      In the current method, there are no descriptions for many used methods, which already generated results, such as WB, MLR, MST, Co-IP, AlphaFold2 prediction, and how to make currently used anti-zfCTLA4 antibody. Also, there is a lack of description of the method of the husbandry of knockout zebrafish line.

      We regret these flaws. The methods section was inadvertently incomplete due to an error during the file upload process at submission. This issue has been rectified in the revised manuscript. Additionally, Ctla-4-deficient zebrafish were reared under the same conditions as wild-type zebrafish, and the rearing methods are now described in the "Generation of Ctla-4-deficient zebrafish" section of the Materials and Methods.

      Line 360: the experimental zebrafish with different ages could be a risk for unstable intestinal health. See the reference entitled "The immunoregulatory role of fish-specific type II SOCS via inhibiting metaflammation in the gut-liver axis" published in Water Biology and Security. The age-related differences in zebrafish could be observed in the gut.

      We appreciate the reviewers' reminders. The Ctla-4 mutant zebrafish used in our experiments were 4 months old, while the wild-type zebrafish ranged from 4 to 6 months old. These experimental fish were relatively young and uniformly distributed in age. During our study, we examined the morphological structures of the intestines in zebrafish aged 4 to 6 months and observed no significant abnormalities. These findings align with previous research indicating no significant difference in intestinal health between 3-month-old and 6-month-old wild-type zebrafish (Ref. 1). Consequently, we conclude that there is no notable aging-related change in the intestines of zebrafish aged 4 to 6 months. This reduces the risk associated with age-related variables in our study. We have added an explanation stating that the Ctla-4 mutant zebrafish used in the experiments were 4 months old (Line 449) in the revised manuscript.

      Reference

      (1) Shan Junwei, Wang Guangxin, Li Heng, Zhao Xuyang et al. (2023) The immunoregulatory role of fish-specific type II SOCS via inhibiting metaflammation in the gut-liver axis. Water Biology and Security 2: 100131-100144.

      Section "Generation of Ctla-4-deficient zebrafish": There is a lack of description of PCR condition for the genotyping.

      The target DNA sequence was amplified at 94 °C for 4 min, followed by 35 cycles at 94°C for 30 s, 58°C for 30 s and 72°C for 30 s, culminating in a final extension at 72 °C for 10 min. The polymerase chain reaction (PCR) conditions are described in lines 458-460.

      How old of the used mutant fish? There should be a section "sampling" to provide the sampling details.

      The "Sampling" information has been incorporated into the "Materials and Methods" section of the revised manuscript. Wild-type and Ctla-4-deficient zebrafish of varying months were housed in separate tanks, each labeled with its corresponding birth date. Experiments utilized Ctla-4-deficient zebrafish aged 4 months and wild-type zebrafish aged between 4 to 6 months.

      Line 378-380: The index for the histopathological analysis should be detailed, rather than just provide a reference. I don't think these indexes are good enough to specifically describe the pathological changes of intestinal villi and mucosa. It is suggested to improve with detailed parameters. As described in the paper entitled "Pathology of Gastric Intestinal Metaplasia: Clinical Implications" published in Am J Gastroenterol., histochemical, normal gastric mucins are pH neutral, and they stain magenta with periodic acid-Schiff (PAS). In an inflamed gut, acid mucins replace the original gastric mucins and are stained blue with Alcian blue (AB). So, to reveal the pathological changes of goblet cells and involved mucin components, AB staining should be added. Also, for the number of goblet cells in the inflammatory intestine, combining PAS and AB staining is the best way to reveal all the goblet cells. In Figure 2, there were very few goblet cells. The infiltration of lymphocytes and the empty intestinal lumen could be observed. Thus, the ratio between the length of intestinal villi and the intestinal ring radius should calculated.

      In response to the reviewers’ valuable suggestions, we have augmented the manuscript by providing additional parameters related to the pathological changes observed in the Ctlta-4-deficient zebrafish intestines, including the mucin component changes identified through PAS and AB-PAS staining, the variations in the number of goblet cells evaluated by AB-PAS staining, and the ratio of intestinal villi length to the intestinal ring radius, as illustrated in the following figures. These new findings are detailed in the "Materials and Methods" (lines 563-566) and "Results" (lines 143-146) sections, along with Supplementary Figure S3 of the revised manuscript.

      Section "Quantitative real-time PCR": What's the machine used for qPCR? How about the qPCR validation of RNA seq data? I did not see any related description of data and methods for qPCR validation. In addition, beta-actin is not a stable internal reference gene, to analyze inflammation and immune-related gene expression. See the reference entitled "Actin, a reliable marker of internal control?" published in Clin Chim Acta. Other stable housekeeping genes, such as EF1alpha and 18s, could be better internal references.

      RT-qPCR experiments were conducted using a PCR thermocycler device (CFX Connect Real-Time PCR Detection System with Precision Melt Analysis<sup>TM</sup> Software, Bio-Rad, Cat. No. 1855200EM1). This information has been incorporated into lines 608-610 of the "Materials and Methods" section. In these experiments, key gene sequences of interest, including il13, mpx, and il1β, were extracted from RNA-seq data for RT-qPCR validation. To ensure accurate normalization, potential internal controls were evaluated, and β-actin was identified as a suitable candidate due to its consistent expression levels in the intestines of both wild-type and Ctla-4-deficient zebrafish. The use of β-actin as an internal control is further supported by its application in recent studies on intestinal inflammation (Refs 1–2).

      References:

      (1) Tang Duozhuang, Zeng Ting, Wang Yiting, Cui Hui et al. (2020) Dietary restriction increases protective gut bacteria to rescue lethal methotrexate-induced intestinal toxicity. Gut Microbes 12: 1714401-1714422.

      (2) Malik Ankit, Sharma Deepika et al. (2023) Epithelial IFNγ signaling and compartmentalized antigen presentation orchestrate gut immunity. Nature 623: 1044-1052.

      How to generate sCtla-4-Ig, Cd28-Ig and Cd80/86? No method could be found.

      We apologize for the omission of these methods. The detailed protocols have now been added to the "Materials and Methods" section of the revised manuscript (lines 464-481).

      Figure 5: As reviewed in the paper entitled "Teleost T and NK cell immunity" published in Fish and Shellfsh Immunology, two types of NK cell homologues have been described in fish: non-specific cytotoxic cells and NK-like cells. There is no NKT cell identified in the teleost yet. Therefore, "NKT-like" could be better to describe this cell type.

      We refer to "NKT" cells as "NKT-like" cells, as suggested.

      For the supplementary data of scRNA-seq, there lacks the details of expression level.

      The expression levels of the corresponding genes are provided in Supplemental Table 4.

      Supplemental Table 1: There are no accession numbers of amplified genes.

      The accession numbers of the amplified genes are included in Supplemental Table 1.

      The English needs further editing.

      We have made efforts to enhance the English to meet the reviewers' expectations.

      Line 32: The tense should be the past.

      This tense error has been corrected.

      Line 363-365: The letter of this approval should be provided as an attachment.

      The approval document is provided as an attachment.

      Line 376: How to distinguish the different intestinal parts? Were they judged as the first third, second third, and last third parts of the whole intestine?

      The differences among the three segments of zebrafish intestine are apparent. The intestinal tube narrows progressively from the anterior to the mid-intestine and then to the posterior intestine. Moreover, the boundaries between the intestinal segments are well-defined, facilitating the isolation of each segment.

      Line 404: Which version of Cytoscape was used?

      The version of Cytoscape used in this study is 3.9.1. Information about the Cytoscape version is provided on line 603.

      The product information of both percoll and cell strainer should be provided.

      The information regarding Percoll and cell strainers has been added on lines 626 and 628, respectively.

      Line 814: Here should be a full name to tell what is MST.

      The acronym MST stands for "Microscale Thermophoresis", a technique that has been referenced on lines 1157-1158.

    1. Author response:

      The following is the authors’ response to the original reviews

      Reviewer #1:

      (1) The mechanism by which fenofibrate rescues memory loss in Kallistatin-transgenic mice is unclear. As a PPARalpha agonist, does fenofibrate target the Kallistatin pathway directly or indirectly? Please provide a discussion based on literature supporting either possibility.

      Thank you for your important suggestion. Fenofibrate is indeed acting as a PPARα agonist. Fenofibrate has been shown to protect memory and cognitive function by downregulating α- and β-secretases[1]. Activation of PPARα can reduce Aβ plaques by upregulating ADAM10, thereby protecting memory and cognition[2]. Whereas, Fenofibrate can also act through a PPARα-independent pathway[3]. In our previous study, we proved that Fenofibrate can directly down-regulate the expression of Kallistatin in hepatocytes[4]. Here, our findings showed that Kallistatin induces cognitive memory deterioration by increasing amyloid-β plaques accumulation and tau protein hyperphosphorylation (Fig. 1-3), and Fenofibrate can directly down-regulate the serum level of Kallistatin (Fig. 8G). In addition, the expression of PPARα in the hippocampal tissue of Kallistatin (KAL-TG) mice showed no significant difference compared to the WT group (Author response image 1A-B). Therefore, we think Fenofibrate may improve memory and cognitive function at least in part through a PPARα-independent effect, which provides a new mechanism of Fenofibrate in AD with elevated Kallistatin levels.

      Author response image 1.

      (A-B) Protein levels of PPARα were tested by western blot analysis in hippocampal tissue, then statistically analyzed the above results.

      (2) The current study exclusively investigated the hippocampus. What about other cognitive memory-related regions, such as the prefrontal cortex? Including data from these regions or discussing the possibility of their involvement could provide a more comprehensive understanding of the role of Kallistatin in memory impairment.

      Thank you for your suggestion. In addition to hippocampal tissue analysis, we performed immunohistochemical detection of Aβ and phosphorylated Tau levels in the prefrontal cortex. Our findings revealed that KAL-TG mice exhibited significantly elevated Aβ and phosphorylated Tau levels in the prefrontal cortex compared to WT mice. These observations align with the pathological patterns observed in hippocampal tissues, demonstrating consistent neurodegenerative pathology across both the hippocampus and prefrontal cortex. The data for this part are seen as follows.

      Author response image 2.

      (A-B) Immunofluorescence staining of Aβ and phosphorylated tau (p-tau T231) was carried out in the prefrontal cortex tissue of KAL-TG and WT mice. Error bars represented the Standard Error of Mean (SEM); **p < 0.01. Scale bar, 100 μm.

      (3) Fenofibrate rescued phenotypes in Kallistatin-transgenic mice while rosiglitazone, a PPARgamma agonist, did not. This result contradicts the manuscript's emphasis on a PPARgamma-associated mechanism. Please address this inconsistency.

      Thank you for the reminder. In fact, our results showed a trend towards improved memory and cognitive function in KAL-TG mice treated with Rosiglitazone, although its effect is not as significant as that of Fenofibrate. Several studies have reported that Rosiglitazone has a beneficial effect on memory and cognitive function in mouse models of dementia, while these studies involve treatment periods of 3 to 4 months[5, 6], whereas our treatment period was only one month. Extending the treatment period with Rosiglitazone may result in a more pronounced improvement. In addition, Fenofibrate may have a PPAR-independent pathway by downregulating Kallistatin directly as discussed above and then show stronger effects.

      (4) Most of the immunohistochemistry images are unclear. Inserts have similar magnification to the original representative images, making judgments difficult. Please provide larger inserts with higher resolution.

      According to your suggestion, we provided larger inserts with higher resolution in Fig 3A and Fig 4B, as follows:

      (5) The immunohistochemistry images in different figures were taken from different hippocampal subregions with different magnifications. Please maintain consistency, or explain why CA1, CA3, or DG was analyzed in each experiment.

      Thank you for your advice. The trends of changes in different brain regions(including CA1, CA3, or DG) are consistent. Following your suggestion, we have now selected the DG region replaced the different hippocampal subregions with the DG area, and re-conducted the statistical analysis in Fig 5I & 6C, as follows. Due to the significant deposition of Aβ only in the CA1 region, Fig 2A was not replaced.

      (6) Figure 5B is missing a title. Please add a title to maintain consistency with other graphs.

      Thanks for your suggestion. We have added a title to Figure 5B, as follows:

      (7) Please list statistical methods used in the figure legends, such as t-test or One-way ANOVA with post-hoc tests.

      Thanks for your suggestion. We have listed the statistical methods used in the figure legends.

      Reviewer #2:

      (1) It was suggested that Kallistatin is primarily produced by the liver. The study demonstrates increased Kallistatin levels in the hippocampus tissue of AD mice. It would be valuable to clarify if Kallistatin is also increased in the liver of AD mice, providing a comprehensive understanding of its distribution in disease states.

      Thank you for your suggestion. We extracted liver tissue from APP/PS1 mice, and the Western blot results indicated that the expression of Kallistatin in the liver of APP/PS1 mice was elevated, as follows:

      Author response image 3.

      (A-B) Protein levels of Kallistatin were tested by western blot analysis in the liver tissue, then statistically analyzed the above results. Error bars represented the Standard Error of Mean (SEM); **p < 0.01.

      (2) Does Kallistatin interact directly with Notch1 ligands? Clarifying this interaction mechanism would enhance understanding of how Kallistatin influences Notch1 signaling in AD pathology.

      Thank you for your suggestion. This study reveals that Kallistatin directly binds to Notch1 and contributes to the activation of the Noch1-HES1 signaling pathway. As for whether Kallistatin can bind to the ligands of Notch1, it needs to conduct further investigations in future studies. Our preliminary data showed that Jagged1 was upregulated in the hippocampal tissues of KAL-TG mice by qPCR and Western blot analyses.

      Author response image 4.

      Kallistatin promoted Notch ligand Jagged1 expression to activate Notch1 signaling. (A) QPCR analysis of Notch ligands (Dll1, Dll3, Jagged1, Jagged2) expression in the 9 months hippocampus tissue. (B) Western blotting analysis of Notch ligand Jagged1 expression in the hippocampus tissue. (C) Western blotting analysis of Notch ligand Jagged1 expression in the hippocampus primary neuron. β-actin served as the loading control. Error bars represented the Standard Error of Mean (SEM); *p < 0.05.

      (3) Is there any observed difference in AD phenotype between male and female Kallistatin-transgenic (KAL-TG) mice? Including this information would address potential gender-specific effects on cognitive decline and pathology.

      Thank you for your suggestion. Actually, we have previously used female mice for Morris Water Maze experiments, and the results showed that both male and female KAL-TG mice exhibited a phenotype of decreased memory and cognitive function compared to the gender-matched WT group, while there was no significant difference between male and female KAL-TG mice as follows:

      Author response image 5.

      (A-D) Behavioral performance was assessed through the Morris water maze test. (A) The escape latency time was presented during 1-5 days. (B-D) Cognitive functions were evaluated by spatial probe test on day 6, then analyzing each group of mice crossing platform times(B), time percent in the targeted area (C), and the path traces heatmap (D). Error bars represented the Standard Error of Mean (SEM); F represents Female, M represents Male, and TG refers to KAL-TG; *p < 0.05.

      (4) It is recommended to include molecular size markers in Western blots for clarity and accuracy in protein size determination.

      Thank you for your reminder. We have shown the molecular weight of each bolt.

      (5) The language should be revised for enhanced readability and clarity, ensuring that complex scientific concepts are communicated effectively to a broader audience.

      According to your suggestion, we have polished the article for enhancing readability and clarity.

      Reviewer #3:

      (1) The authors did not illustrate whether the protective effect of fenofibrate against AD depends on Kallistatin.

      Thank you for your important suggestion. Fenofibrate is indeed acting as a PPARα agonist. Fenofibrate has been shown to protect memory and cognitive function by downregulating α- and β-secretases[1]. Activation of PPARα can reduce Aβ plaques by upregulating ADAM10, thereby protecting memory and cognition[2]. Whereas, Fenofibrate can also act through a PPARα-independent pathway[3]. In our previous study,we proved Fenofibrate can directly down-regulate the expression of KAL in hepatocytes[4]. Here, our findings showed that Kallistatin induces cognitive memory deterioration by increasing amyloid-β plaques accumulation and tau protein hyperphosphorylation (Fig. 1-3), and Fenofibrate can directly down-regulate the serum level of Kallistatin (Fig. 8G). In addition, the expression of PPARα in the hippocampal tissue of Kallistatin (KAL-TG) mice showed no significant difference compared to the WT group (Author response image 1-B). Therefore, we think Fenofibrate may improve memory and cognitive function at least in part through downregulatin Kallistatin. To conclusively determine whether fenofibrate’s therapeutic effects depend on Kallistatin, future studies should employ Kallistatin-knockout AD animal models to evaluate fenofibrate’s impact on cognitive and memory functions. These investigations will further clarify the mechanistic underpinnings of fenofibrate in AD therapy.

      (2) The conclusions are supported by the results, but the quality of some results should be improved.

      Thank you for your kind suggestion. We have updated the magnified images in the immunohistochemistry section of the article, ensuring that the fields of view for the immunohistochemistry are within the same brain region, and have shown the molecular weights in each bolt. Additionally, we have conducted a quantitative analysis of the protein levels in the Western blot results presented in Fig6&8.

      (3) Figures 2c, 3c, and 4a present the Western blot results of p-tau from mice of different ages on one membrane, showing age-dependent expression. The authors analyzed the results of mice of different ages in one statistical chart, which will create ambiguity with the results of the representative images. For example, the expression of p-tau 396 in the blot was lower in the WT-12 M group than in the WT-9 M group (Figure 3c), which is contradictory to the statistical analysis.

      Thank you for your reminder. The statistical presentation here does not match the figure. At that time, the WB experiments for the hippocampal tissue at each age group were conducted separately, and it was not appropriate to compare different age groups together. This graph cannot illustrate age dependency. We have replaced the statistical graph in Figure 3B&D, as follows:

      (4) Figure 4b shows that KAL-TG-9 M had greater BACE1 expression than KAL-TG-12 M. Furthermore, the nuclei are not uniformly colored. Please provide more representative figures.

      Thank you for your reminder. Due to the fact that these sets of data were not processed in a single batch, the ages in the graph are not comparable. Regarding the issue of inconsistent nuclear staining, we have provided another representative image from this group, as follows:

      (5) Unclear why the BACE1 and Aβ levels seems less with KAL+shHES1 treatment than GFP+shNC treatment (Fig 6H)? This finding contradicts the conclusion.

      Thank you for your reminder. This experiment was repeated three times, and here, we have represented the representative results along with the corresponding statistical data. There are no difference between KAL+shHES1 treatment and GFP+shNC treatment. We have updated the Fig. 6H.

      (6) The Western blot results in figure 6e-h, 8h-i, and S3-S5 were not quantified.

      Thank you for your reminder. We have added statistical graphs and original images of the pictures in figure 6e-h, 8h-i, and S3-S5.

      (7) The authors did not provide the detection range of the Aβ42 ELISA kit.

      Thank you for your suggestion. The Aβ42 ELISA kit is from the IBL, with the product number 27721. Its standard range is 1.56 - 100 pg/mL, and the sensitivity is 0.05 pg/mL.

      (8)The authors did not specify the sex of the mice. This is important since sex could have had a dramatic impact on the results.

      Thank you for your suggestion. The results we present in the text are all statistically obtained from male mice. Actually, we have previously used female mice for Morris Water Maze experiments, and the results showed that both male and female KAL-TG mice exhibited a phenotype of decreased memory and cognitive function compared to the gender-matched WT group, while there was no significant difference between male and female KAL-TG mice (Author response image 5).

      Minor:

      (1) In Figure 2b, there are no units for the vertical coordinates of the statistical graph.

      Thank you for your reminder. We have added units for the vertical coordinates in Figure 2b.

      (2) In Figure 2c, the left Y-axis title is lacking in the statistic chart.

      Thank you for your reminder. We have added the left Y-axis title in the statistic chart.

      Reference:

      (1) Assaf N, El-Shamarka ME, Salem NA, Khadrawy YA, El Sayed NS. Neuroprotective effect of PPAR alpha and gamma agonists in a mouse model of amyloidogenesis through modulation of the Wnt/beta catenin pathway via targeting alpha- and beta-secretases. Progress in Neuro-Psychopharmacology and Biological Psychiatry 2020, 97: 109793.

      (2) Rangasamy SB, Jana M, Dasarathi S, Kundu M, Pahan K. Treadmill workout activates PPARα in the hippocampus to upregulate ADAM10, decrease plaques and improve cognitive functions in 5XFAD mouse model of Alzheimer’s disease. Brain, Behavior, and Immunity 2023, 109: 204-218.

      (3) Yuan J, Tan JTM, Rajamani K, Solly EL, King EJ, Lecce L, et al. Fenofibrate Rescues Diabetes-Related Impairment of Ischemia-Mediated Angiogenesis by PPARα-Independent Modulation of Thioredoxin-Interacting Protein. Diabetes 2019, 68(5): 1040-1053.

      (4) Fang Z, Shen G, Wang Y, Hong F, Tang X, Zeng Y, et al. Elevated Kallistatin promotes the occurrence and progression of non-alcoholic fatty liver disease. Signal Transduct Target Ther 2024, 9(1): 66.

      (5) Nelson ML, Pfeifer JA, Hickey JP, Collins AE, Kalisch BE. Exploring Rosiglitazone's Potential to Treat Alzheimer's Disease through the Modulation of Brain-Derived Neurotrophic Factor. Biology (Basel) 2023, 12(7).

      (6) Pedersen WA, McMillan PJ, Kulstad JJ, Leverenz JB, Craft S, Haynatzki GR. Rosiglitazone attenuates learning and memory deficits in Tg2576 Alzheimer mice. Exp Neurol 2006, 199(2): 265-273.

    1. Author response:

      The following is the authors’ response to the original reviews.

      eLife Assessment

      This manuscript reveals important insights into the role of ipsilateral descending pathways in locomotion, especially following unilateral spinal cord injury. The study provides solid evidence that this method improves the injured side's ability to support weight, and as such the findings may lead to new treatments for stroke, spinal cord injuries, or unilateral cerebral injuries. However, the methods and results need to be better detailed, and some of the statistical analysis enhanced.

      Thank you for your assessment. We incorporated various text improvements in the final version of the manuscript to address the weaknesses you have pointed out. The specific improvements are outlined below.

      Public Reviews:

      Reviewer #1 (Public Review):

      Summary:

      This manuscript provides potentially important new information about ipsilateral cortical impact on locomotion. A number of issues need to be addressed.

      Strengths:

      The primary appeal and contribution of this manuscript are that it provides a range of different measures of ipsilateral cortical impact on locomotion in the setting of impaired contralateral control. While the pathways and mechanisms underlying these various measures are not fully defined and their functional impacts remain uncertain, they comprise a rich body of results that can inform and guide future efforts to understand cortical control of locomotion and to develop more effective rehabilitation protocols.

      Weaknesses:

      (1) The authors state that they used a cortical stimulation location that produced the largest ankle flexion response (lines 102-104). Did other stimulation locations always produce similar, but smaller responses (aside from the two rats that showed ipsilateral neuromodulation)? Was there any site-specific difference in response to stimulation location?

      We derived motor maps in each rat, akin to the representation depicted in Fig 6. In each rat, alternative cortical sites did, indeed, produce distal or proximal contralateral leg flexion responses. Distal responses were more likely to be evoked in the rostral portion of the array, similarly to proximal responses early after injury. This distribution in responses across different cortical sites is reported in this study (Fig. 6) and is consistent with our prior work. The Results section has been revised to provide additional clarification of the passage you indicated and context for the data presented in Figure 6:

      On page 4, we have clarified: “Stimulation through these channels produced a strong whole-leg flexion movement, with an evident distal component. From visual inspection, all responding electrodes in the array produced contralateral leg flexion, although with different strength of contraction for a fixed stimulation intensity (100μA). Moreover, some sites did not present a distal movement component, failing in eliciting ankle flexion and resulting in a generally weaker proximal flexion.”

      On page 12, we have further noted: “By visually inspecting the responses elicited by stimulation delivered through each of the array electrodes, we categorized movements as proximal or distal. This classification was based on whether the ankle participated in the evoked response or if the movement was restricted to the proximal hindlimb. Each leg was scored independently.”

      (2) Figure 2: There does not appear to be a strong relationship between the percentage of spared tissue and the ladder score. For example, the animal with the mild injury (based on its ladder score) in the lower left corner of Figure 2A has less than 50% spared tissue, which is less spared tissue than in any animal other than the two severe injuries with the most tissue loss. Is it possible that the ladder test does not capture the deficits produced by this spinal cord injury? Have the authors looked for a region of the spinal cord that correlates better with the deficits that the ladder test produces? The extent of damage to the region at the base of the dorsal column containing the corticospinal tract would be an appropriate target area to quantify and compare with functional measures.

      In Fig. S6 of our 2021 publication "Bonizzato and Martinez, Science Translational Medicine", we investigated the predictive value of tissue sparing in specific sub-regions of the spinal cord for ladder performance. Among others, we examined the correlation between the accuracy of left leg ladder performance in the acute state and the preservation of the corticospinal tract (CST). Our results indicated that dorsal CST sparing serves as a mild predictor for ladder deficits, confirming the results obtained in this study.

      (3) Lines 219-221: The authors state that "phase-coherent stimulation reinstated the function of this muscle, leading to increased burst duration (90{plus minus}18% of the deficit, p=0.004, t-test, Fig. 4B) and total activation (56{plus minus}13% of the deficit, p=0.014, t-test, Fig. 3B). This way of expressing the data is unclear. For example, the previous sentence states that after SCI, burst duration decreased by 72%. Does this mean that the burst duration after stimulation was 90% higher than the -72% level seen with SCI alone, i.e., 90% + -72% = +18%? Or does it mean that the stimulation recovered 90% of the portion of the burst duration that had been lost after SCI, i.e., -72% * (100%-90%)= -7%? The data in Figure 4 suggests the latter. It would be clearer to express both these SCI alone and SCI plus stimulation results in the text as a percent of the pre-SCI results, as done in Figure 4.

      Your assessment is correct; we intended to report that the stimulation recovered 90% of the portion of the burst duration that had been lost after SCI. This point has been clarified (see page 9):

      “…leading to increased burst duration (recovered 90±18% of the lost burst duration, p=0.004, t-test, Fig. 4B) and total activation (recovered 56±13% of the total activation, p=0.014, t-test, Fig. 3B)”

      (4) Lines 227-229: The authors claim that the phase-dependent stimulation effects in SCI rats are immediate, but they don't say how long it takes for these effects to be expressed. Are these effects evident in the response to the first stimulus train, or does it take seconds or minutes for the effects to be expressed? After the initial expression of these effects, are there any gradual changes in the responses over time, e.g., habituation or potentiation?

      The effects are immediately expressed at the very first occurrence of stimulation. We never tested a rat completely naïve to stimuli, as each treadmill session involves prior cortical mapping to identify a suitable active site for involvement in locomotor experiments. Yet, as demonstrated in Supplementary Video 1 accompanying our 2021 publication on contralateral effects of cortical stimulation, "Bonizzato and Martinez, Science Translational Medicine," the impact of phase-dependent cortical stimulation on movement modulation is instantaneous and ceases promptly upon discontinuation of the stimulation. We did not quantify potential gradual changes in responsiveness over time, but we cannot exclude that for long stimulation sessions (e.g., 30 min or more), stimulus amplitude may need to be slightly increased over time to compensate habituation.

      (5) Awake motor maps (lines 250-277): The analysis of the motor maps appears to be based on measurements of the percentage of channels in which a response can be detected. This analytic approach seems incomplete in that it only assesses the spatial aspect of the cortical drive to the musculature. One channel could have a just-above-threshold response, while another could have a large response; in either case, the two channels would be treated as the same positive result. An additional analysis that takes response intensity into account would add further insight into the data, and might even correlate with the measures of functional recovery. Also, a single stimulation intensity was used; the results may have been different at different stimulus intensities.

      We confirm that maps of cortical stimulation responsiveness may vary at different stimulus amplitudes. To establish an objective metric of excitability, we identified 100µA as a reliable stimulation amplitude across rats and used this value to build the ipsilateral motor representation results in Figure 6. This choice allows direct comparison with Figure 6 of our 2021 article, related to contralateral motor representation. The comparison reveals a lack of correlation with functional recovery metrics in the ipsilateral case, in contrast to the successful correlation achieved in the contralateral case.

      Regarding the incorporation of stimulation amplitudes into the analysis, as detailed in the Method section (lines 770-771), we systematically tested various stimulation amplitudes to determine the minimal threshold required for eliciting a muscle twitch, identified as the threshold value. This process was conducted for each electrode site.

      Upon reviewing these data, we considered the possibility of presenting an additional assessment of ipsilateral cortical motor representation based on stimulation thresholds. However, the representation depicted in the figure did not differ significantly from the data presented in Figure 6A. Furthermore, this representation introduced an additional weakness, as it was unclear how to represent the absence of a response in the threshold scale. We chose to arbitrarily designate it as zero on the inverse logarithmic scale, where, for reference, 100 µA is positioned at 0.2 and 50 µA at 0.5.

      In conclusion, we believe that the conclusions drawn from this analysis align substantially with those in the text. The addition of the threshold analysis, in our assessment, would not contribute significantly to improving the manuscript.

      Author response image 1.

      Threshold analysis

      Author response image 2.

      Occurrence probability analysis, for comparison.

      (6) Lines 858-860: The authors state that "All tests were one-sided because all hypotheses were strictly defined in the direction of motor improvement." By using the one-sided test, the authors are using a lower standard for assessing statistical significance that the overwhelming majority of studies in this field use. More importantly, ipsilateral stimulation of particular kinds or particular sites might conceivably impair function, and that is ignored if the analysis is confined to detecting improvement. Thus, a two-sided analysis or comparable method should be used. This appropriate change would not greatly modify the authors' current conclusions about improvements.

      Our original hypothesis, drawn from previous studies involving cortical stimulation in rats and cats, as well as other neurostimulation research for movement restoration, posited a favorable impact of neurostimulation on movement. Consistent with this hypothesis, we designed our experiments with a focus on enhancing movement, emphasizing a strict direction of improvement.

      It's important to note that a one-sided test is the appropriate match for a one-sided hypothesis, and it is not a lower standard in statistics. Each experiment we conducted was constructed around a strictly one-sided hypothesis: the inclusion of an extensor-inducing stimulus would enhance extension, and the inclusion of a flexion-inducing stimulus would enhance flexion. This rationale guided our choice of the appropriate statistical test.

      We acknowledge your concern regarding the potential for ipsilateral stimulation to have negative effects on locomotion, which might not be captured when designing experiments based on one-sided hypotheses. That is, when hypothesizing that an extensor stimulus would enhance extension (a one-sided hypothesis) in a functional task, and finding an opposite result (inhibition), statistical rigor would impose that we cannot present that result as significant. This concern is valid, and we explicitly mentioned our design choice it in the method section, Quantification and statistical analyses:

      “All tests were one-sided, as our hypotheses were strictly defined to predict motor improvement. Specifically, we hypothesized that delivering an extension-inducing stimulus would enhance leg extension, and delivering a flexion-inducing stimulus would enhance leg flexion. Consequently, any potentially statistically significant result in the opposite direction (e.g., inhibition) would not be considered. However, no such occurrences were observed.”

      As a final note, even if such opposite observations were made, they could serve as the basis for triggering an ad-hoc follow-up study.

      Reviewer #1 also provided several detailed suggestions in the section “Recommendations for the authors”. We estimated that each of them was beneficial for the correctness or for the readability of the text, and thus all were incorporated into the final version.

      Reviewer #2 (Public Review):

      Summary:

      The authors' long-term goals are to understand the utility of precisely phased cortex stimulation regimes on recovery of function after spinal cord injury (SCI). In prior work, the authors explored the effects of contralesion cortex stimulation. Here, they explore ipsilesion cortex stimulation in which the corticospinal fibers that cross at the pyramidal decussation are spared. The authors explore the effects of such stimulation in intact rats and rats with a hemisection lesion at the thoracic level ipsilateral to the stimulated cortex. The appropriately phased microstimulation enhances contralateral flexion and ipsilateral extension, presumably through lumbar spinal cord crossed-extension interneuron systems. This microstimulation improves weight bearing in the ipsilesion hindlimb soon after injury, before any normal recovery of function would be seen. The contralateral homologous cortex can be lesioned in intact rats without impacting the microstimulation effect on flexion and extension during gait. In two rats ipsilateral flexion responses are noted, but these are not clearly demonstrated to be independent of the contralateral homologous cortex remaining intact.

      Strengths:

      This paper adds to prior data on cortical microstimulation by the laboratory in interesting ways. First, the strong effects of the spared crossed fibers from the ipsi-lesional cortex in parts of the ipsi-lesion leg's step cycle and weight support function are solidly demonstrated. This raises the interesting possibility that stimulating the contra-lesion cortex as reported previously may execute some of its effects through callosal coordination with the ipsi-lesion cortex tested here. This is not fully discussed by the authors but may represent a significant aspect of these data. The authors demonstrate solidly that ablation of the contra-lesional cortex does not impede the effects reported here. I believe this has not been shown for the contra-lesional cortex microstimulation effects reported earlier, but I may be wrong. Effects and neuroprosthetic control of these effects are explored well in the ipsi-lesion cortex tests here.

      In the revised version of the manuscript, we incorporated various text improvements to address the points you have highlighted in your review. Additionally, we have integrated the suggested discussion topic on callosal coordination related to contralateral cortical stimulation. The discussion section now incorporates:

      “Since bi-cortical interactions in sculpting descending commands are known (Brus-Ramer et al., 2009), and in light of the changes we report in ipsilesional motor cortex excitability, the role of the ipsilateral cortex in mediating or supporting functional descending commands from the contralateral cortex, particularly the immediate increase in flexion of the affected hindlimb and long-term recovery of functional control (Bonizzato & Martinez, 2021), could be further explored.”

      The localization of the specific channels closest to the interhemispheric fissure (Fig. 7D) may suggest the involvement of transcallosal interactions in mediating the transmission of the cortical command generated in the ipsilateral motor cortex (Brus-Ramer, Carmel, & Martin, 2009). “While ablation experiments (Fig. 8) refute this hypothesis for ipsilateral extension control, they do not conclusively determine whether a different efferent pathway is involved in ipsilateral flexion control in this specific case."

      Weaknesses:

      Some data is based on very few rats. For example (N=2) for ipsilateral flexion effects of microstimulation. N=3 for homologous cortex ablation, and only ipsi extension is tested it seems. There is no explicit demonstration that the ipsilateral flexion effects in only 2 rats reported can survive the contra-lateral cortex ablation.

      We agree with this assessment. The ipsilateral flexion representation is here reported as a rare but consistent phenomenon, which we believe to have robustly described with Figure 7 experiments. We underlined in the text that the ablation experiment did not conclude on the unilateral-cortical nature of ipsilateral flexion effects, by replacing the sentence with the following:

      “While ablation experiments (Fig. 8) refute this hypothesis for ipsilateral extension control, they do not conclusively determine whether a different efferent pathway is involved in ipsilateral flexion control in this specific case."

      Some improvements in clarity and precision of descriptions are needed, as well as fuller definitions of terms and algorithms.

      Likely Impacts: This data adds in significant ways to prior work by the authors, and an understanding of how phased stimulation in cortical neuroprosthetics may aid in recovery of function after SCI, especially if a few ambiguities in writing and interpretation are fully resolved.

      The manuscript text has been revised in its final version, and we sought to eliminate all ambiguity in writing and data interpretation.

      In the section “Recommendations for the authors” Reviewer #2 also suggested to better define multiple terms throughout the manuscript. A clarification was added for each.

      The Reviewer pointed out that we might have overlooked a correlation between locomotor recovery and motor maps increase in Figure 6. We re-approached this evaluation and found that the reviewer is correct. We were led to think that there was no correlation by “horizontally” looking at whether motor map size across rats would predict locomotor scores (as it did in the case of contralateral cortex mapping, Bonizzato and Martinez, 2021). However we now found a strong correlation between changes that happen over time for each rat and locomotor recovery, a result that was only hinted with no appropriate quantification in the previous version of the manuscript. We have now reformulated the results of Figure 6 on page 12, to include this result, and we would like to thank the reviewer for having noticed this opportunity.

      Finally, we have expanded the discussion to include the following points:

      The possibility that hemi-cortex coordination of contralesional microstimulation inputs may explain the Sci Transl Med results for contralesional cortex ICMS, which warrants further investigation.

      The recognition that the ablation experiments do not provide conclusive evidence regarding ipsilateral flexion control and whether an alternative efferent pathway might be involved in this specific case.

      Reviewer #3 (Public Review):

      Summary:

      This article aims to investigate the impact of neuroprosthesis (intracortical microstimulation) implanted unilaterally on the lesion side in the context of locomotor recovery following unilateral thoracic spinal cord injury.

      Strength:

      The study reveals that stimulating the left motor cortex, on the same side as the lesion, not only activates the expected right (contralateral) muscle activity but also influences unexpected muscle activity on the left (ipsilateral) side. These muscle activities resulted in a substantial enhancement in lift during the swing phase of the contralateral limb and improved trunk-limb support for the ipsilateral limb. They used different experimental and stimulation conditions to show the ipsilateral limb control evoked by the stimulation. This outcome holds significance, shedding light on the engagement of the "contralateral projecting" corticospinal tract in activating not only the contralateral but also the ipsilateral spinal network.

      The experimental design and findings align with the investigation of the stimulation effect of contralateral projecting corticospinal tracts. They carefully examined the recovery of ipsilateral limb control with motor maps. They also tested the effective sites of cortical stimulation. The study successfully demonstrates the impact of electrical stimulation on the contralateral projecting neurons on ipsilateral limb control during locomotion, as well as identifying important stimulation spots for such an effect. These results contribute to our understanding of how these neurons influence bilateral spinal circuitry. The study's findings contribute valuable insights to the broader neuroscience and rehabilitation communities.

      Thank you for your assessment of this manuscript. The final version of the manuscript incoporates your suggestions for improving term clarity and we enhanced the discussion on the mechanisms of spinal network engagement, as outlined below.

      Weakness:

      The term "ipsilateral" lacks a clear definition in the title, abstract, introduction, and discussion, potentially causing confusion for the reader.

      [and later] However, in my opinion, readers can easily link the ipsilateral cortical network to the ipsilateral-projecting corticospinal tract, which is less likely to play a role in ipsilateral limb control in this study since this tract is disrupted by the thoracic spinal injury.

      In order to mitigate the risk of having readers linking the effects of ipsilateral cortical stimulation with ipsilateral-projecting corticospinal tract, we specified:

      In the abstract, we precise that our goal was: “to investigate the functional role of the ipsilateral motor cortex in rat movement through spared contralesional pathways.”

      In the introduction: “In most cases, this lesion also disrupts all spinal tracts descending on the same side as the cortex under investigation at the thoracic level, meaning that the transmission of cortical commands to the ipsilesional hindlimb must depend on crossed descending tracts (Fig. S1).”

      The unexpected ipsilateral (left) muscle activity is most likely due to the left corticospinal neurons recruiting not only the right spinal network but also the left spinal network. This is probably due to the joint efforts of the neuroprosthesis and activation of spinal motor networks which work bilaterally at the spinal level.

      We agree with your assessment and the discussion section now emphasizes the effects of supraspinal drive onto spinal circuits.

      In the section “Recommendations for the authors” Reviewer #3 suggested to provide an early reminder to the reader that the focus is on exploring the control of the ipsilateral limb through the corticospinal tract of the same side, projecting contralaterally. We did so in the abstract and introduction, as presented above.

      The reviewer also suggested that the discussion could be shorter. While we recognize it covers diverse subjects that may appeal to different readers, we believe omitting some sections could limit its overall scope. The manuscript underwent three revisions and a thorough dialogue with reviewers from diverse backgrounds, and we are hesitant to undo some of these improvements.

      Moreover, the section falls short of fully exploring the involvement of contralateral projecting corticospinal neurons in spinal networks for diverse motor behaviors. It could potentially delve into aspects like the potential impact of corticospinal inputs on gating the cross-extensor reflex loop and elucidating the mechanisms underlying the recruitment of the ipsilateral spinal network for generating ipsilateral limb movements. Is it a direct control on motor neurons or via existing spinal circuits?

      The discussion section now includes the potential spinal circuits through which corticospinal neurons may affect motor control and reflexes.

      Reviewer #3 also provided several detailed suggestions in the sub-section “Minor points”. We estimated that all of them were beneficial for the correctness or for the readability of the text, and thus were incorporated into the final version. Some of the questions raised were answered directly in the text (defining “% of chronic map” and rephrasing the original Line 479). We would like to answer here below two remaining questions:

      Fig. 3C I wonder what is the average latency between stimulation onset and onset of right ankle flexor activity. Is the latency fixed, or variable (which probably indicates that the Cortical activation signal is integrated with spinal CPG activity.)

      ICMS trains, unfortunately, do not allow for precise dissection of transmission timing. Single pulses at 100 µA are insufficient to generate motoneuron responses and require multiple pulses to build up cortical transmission. Alstermark et al. (Journal of Neurophysiology, 2004) used two to four stimuli with higher amplitudes to investigate forelimb transmission timing. In our 2021 Science Translational Medicine paper, we employed single pulses at 1 mA to establish transmission delays from the contralateral cortex to the ankle flexor. However, the circuits recruited at 1 mA are not directly comparable to those activated by shorter trains.

      In this study, we used cortical trains of approximately 14 pulses, typical of ICMS protocols. Each pulse could potentially be the first to generate a response volley in the ankle flexor, with delays measured at 30 to 60 ms from ICMS train onset. While we believe that cortical commands are necessarily integrated with spinal CPG activity—as indicated in Figures 1B and 3D, where timing is crucial and descending commands can be gated out if delivered off-phase—the variability in latency that we recorded could be attributed to any of the following factors: cortical activation build-up, integration within reticular relay networks, or CPG integration.

      Fig. 4A. Why is the activity of under contralateral ankle flexor intact condition is later than the stimulation condition?

      We timed the stimulation to coincide with the contralateral leg lift and did not adjust its onset relative to spontaneous walking in SCI rats. Although stimulation could induce leg lift, as shown in Fig. 4A, SCI rats exhibited a slightly earlier and stronger activation of the right (contralateral) ankle flexor muscle even during spontaneous walking. This phenomenon is attributed to the deficits observed on the left side. The stronger right leg bears the body weight, as illustrated in Fig. 3, and thus, during body advancement, the right leg is engaged sooner and more rapidly (with a shorter swing phase) to provide support (right foot forward).

    1. Author response:

      The following is the authors’ response to the original reviews.

      Public Reviews:

      Reviewer #1 (Public review):

      (1) Although there are many citations acknowledging relevant previous work, there often isn't a very granular attribution of individual previous findings to their sources. In the results section, it's sometimes ambiguous when the paper is recapping established background and when it is breaking new ground. For example, around equation 8 in the results (sv = r - rho*t), it would be good to refer to previous places where versions of this equation have been presented. Offhand, McNamara 1982 (Theoretical Population Biology) is one early instance and Fawcett et al. 2012 (Behavioural Processes) is a later one. Line 922 of the discussion seems to imply this formulation is novel here.

      We would like to clarify that original manuscript equation 8, , as we derive, is not new, as it is similarly expressed in prior foundational work by McNamara (1982), and we thank the reviewer for drawing our attention to the extension of this form by Fawcett, McNamara, Houston (2012).

      We now so properly acknowledge this foundational work and extension in the results section…

      “This global reward-rate equivalent immediate reward (see Figure 4) is the subjective value of a pursuit, svPursuit (or simply, sv, when the referenced pursuit can be inferred), as similarly expressed in prior foundational work (McNamara 1982), and subsequent extensions (see (Fawcett, McNamara, Houston (2012)).”

      …and in the Discussion section at the location referenced by the reviewer:

      “From it, we re-expressed the pursuit’s worth in terms of its global reward rate-equivalent immediate reward, i.e., its ‘subjective value’, reprising McNamara’s foundational formulation (McNamara 1982).”

      (2) The choice environments that are considered in detail in the paper are very simple. The simplicity facilitates concrete examples and visualizations, but it would be worth further consideration of whether and how the conclusions generalize to more complex environments. The paper considers "forgo" scenario in which the agent can choose between sequences of pursuits like A-B-A-B (engaging with option B at all opportunities, which are interleaved with a default pursuit A) and A-A-A-A (forgoing option B). It considers "choice" scenarios where the agent can choose between sequences like A-B-A-B and A-C-A-C (where B and C are larger-later and smaller-sooner rewards, either of which can be interleaved with the default pursuit). Several forms of additional complexity would be valuable to consider. [A] One would be a greater number of unique pursuits, not repeated identically in a predictable sequence, akin to a prey-selection paradigm. It seems to me this would cause t_out and r_out (the time and reward outside of the focal prospect) to be policy-dependent, making the 'apportionment cost' more challenging to ascertain. Another relevant form of complexity would be if there were [B] variance or uncertainty in reward magnitudes or temporal durations or if [C] the agent had the ability to discontinue a pursuit such as in patch-departure scenarios.

      A) We would like to note that the section “Deriving Optimal Policy from Forgo Decision-making worlds”, addresses the reviewer’s scenario of n-number of pursuits”, each occurring at their own frequency, as in prey selection, not repeating identically in a predictable sequence. Within our subsection “Parceling the world…”, we introduce the concept of dividing a world (such as that) into the considered pursuit type, and everything outside of it. ‘Outside’ would include any number of other pursuits currently part of any policy, as the reviewer intuits, thus making t<sup>out</sup> and r<sup>out</sup> policy dependent. Nonetheless, a process of excluding (forgoing) pursuits by comparing the ‘in’ to the ‘out’ reward rate (section “Reward-rate optimizing forgo policy…”) or its equivalent sv (section “The forgo decision can also be made from subjective value), would iteratively lead to the global reward rate maximizing policy. This manner of parceling into ‘in’ and ‘out’ thus simplifies visualization of what can be complex worlds. Simpler cases that resemble common experimental designs are given in the manuscript to enhance intuition.

      We thank the reviewer for this keen suggestion. We now include example figures (Supplemental 1 & 2) for multi-pursuit worlds which have the same (Supplemental 1) and different pursuit frequencies (Supplemental 2), which illustrate how this evaluation leads to reward-rate optimization. This addition demonstrates how an iterative policy would lead to reward rate maximization and emphasizes how parcellating a world into ‘in’ and ‘out’ of the pursuit type applies and is a useful device for understanding the worth of any given pursuit in more complex worlds. The policy achieving the greatest global reward rate can be realized through an iterative process where pursuits with lower reward rates than the reward rate obtained from everything other than the considered pursuit type are sequentially removed from the policy.

      B) We would also emphasize that the formulation here contends with variance or uncertainty in the reward magnitudes or temporal durations. The ‘in’ pursuit is the average reward and the average time of the considered pursuit type, as is the ‘out’ the average reward and average time outside of the considered pursuit type.

      C) In this work, we consider the worth of initiating one-or-another pursuit (from having completed a prior one), and not the issue of continuing within a pursuit (having already engaged it), as in patch/give-up. Handling worlds in which the agent may depart from within a pursuit, which is to say ‘give-up’ (as in patch foraging), is outside the scope of this work.

      (3) I had a hard time arriving at a solid conceptual understanding of the 'apportionment cost' around Figure 5. I understand the arithmetic, but it would help if it were possible to formulate a more succinct verbal description of what makes the apportionment cost a useful and meaningful quality to focus on.

      We thank the reviewer for pressing for a succinct and intuitive verbal description.

      We added the following succinct verbal description of apportionment cost… “Apportionment cost is the difference in reward that can be expected, on average, between a policy of taking versus a policy of not taking the considered pursuit, over a time equal to its duration.” This definition appears in new paragraphs (as below) describing apportionment cost in the results section “Time’s cost: opportunity & apportionment costs determine a pursuit’s subjective value”, and is accompanied by equations for apportionment cost, and a figure giving its geometric depiction (Figure 5). We also expanded original figure 5 and its legend (so as to illustrate the apportionment scaling factor and the apportionment cost), and its accompanying main text, to further illustrate and clarify apportionment cost, and its relationship to opportunity cost, and time’s cost.

      “What, then, is the amount of reward by which the opportunity cost-subtracted reward is scaled down to equal the sv of the pursuit? This amount is the apportionment cost of time. The apportionment cost of time (height of the brown vertical bar, Figure 5F) is the global reward rate after taking into account the opportunity cost (slope of the magenta-gold dashed line in Figure 5F) times the time of the considered pursuit. Equally, the difference between the inside and outside reward rates, times the time of the pursuit, is the apportionment cost when scaled by the pursuit’s weight, i.e., the fraction that the considered pursuit is to the total time to traverse the world (Equation 9, right hand side). From the perspective of decision-making policies, apportionment cost is the difference in reward that can be expected, on average, between a policy of taking versus a policy of not taking the considered pursuit, over a time equal to its duration (Equation 9 center, Figure 5F).

      Equation 9. Apportionment Cost.

      While this difference is the apportionment cost of time, the opportunity cost of time is the amount that would be expected from a policy of not taking the considered pursuit over a time equal to the considered pursuit’s duration. Together, they sum to Time’s Cost (Figure 5G). Expressing a pursuit’s worth in terms of the global reward rate obtained under a policy of accepting the pursuit type (Figure 5 left column), or from the perspective of the outside reward and time (Figure 5 right column), are equivalent. However, the latter expresses sv in terms that are independent of one another, conveys the constituents giving rise to global reward rate, and provides the added insight that time’s cost comprises an apportionment as well as an opportunity cost.”

      The above definition of apportionment cost adds to other stated relationships of apportionment cost found throughout the paper (original lines 434,435,447,450).

      I think Figure 6C relates to this, but I had difficulty relating the axis labels to the points, lines, and patterned regions in the plot.

      We thank the reviewer for pointing out that this figure can be made to be more easily understood.

      We have done so by breaking its key features over a greater number of plots so that no single panel is overloaded. We have also changed text in the legend to clarify how apportionment and opportunity costs add to constitute time’s cost, and also correspondingly in the main text.

      I also was a bit confused by how the mathematical formulation was presented. As I understood it, the apportionment cost essentially involves scaling the rest of the SV expression by t<sup>out</sup>/(t<sup>in</sup> + t<sup>out</sup>).

      The reviewer’s understanding is correct: the amount of reward of the pursuit that remains after subtracting the opportunity cost, when so scaled, is equivalent to the subjective value of that pursuit. The amount by which that scaling decreases the rest of the SV expression is equal to the apportionment cost of time.

      The way this scaling factor is written in Figure 5C, as 1/(1 + (1/t<sup>out</sup>) t<sup>in</sup>), seems less clear than it could be.

      To be sure, we present the formula in original Figure 5C in this manner to emphasize the opportunity cost subtraction as separable from the apportionment rescaling, expressing the opportunity cost subtraction and the apportionment scaling component of the equation as their own terms in parentheses.

      But we understand the reviewer to be referring to the manner by which we chose to express the scaling term. We presented it in this way in the original manuscript, (rather than its more elegant form recognized by the reviewer) to make direct connection to temporal discounting literature. In this literature, discounting commonly takes the same mathematical form as our apportionment cost scaling, but whereas the steepness of discounting in this literature is controlled by a free fit parameter, k, we show how for a reward rate maximizing agent, the equivalent k term isn’t a free fit parameter, but rather is the reciprocal of the time spent outside the considered pursuit type.

      We take the reviewer’s advice to heart, and now first express subjective value in the format that emphasizes opportunity cost subtraction followed by an apportionment downscaling, identifying the apportionment scaling term, t<sup>out</sup>/(t<sup>out</sup> + t<sup>in</sup>), ie the outside weight. Figure 5 now shows the geometric representation of apportionment scaling and apportionment cost. Only subsequently in the discounting function section then do we now in the revised manuscript rearrange this subjective value expression to resemble the standard discounting function form.

      Also, the apportionment cost is described in the text as being subtracted from sv rather than as a multiplicative scaling factor.

      What we describe in the original text is how apportionment cost is a component of time’s cost, and how sv is the reward less time’s cost. It would be correct to say that apportionment cost and opportunity cost are subtracted from the pursuit’s reward to yield the subjective value of the pursuit. This is what we show in the original Figure 5D graphically. Original Figure 5 and accompanying formulas at its bottom show the equivalence of expressing sv in terms of subtracting time’s cost as calculated from the global reward rate under a policy of accepting the considered pursuit, or, of subtracting opportunity cost and then scaling the opportunity cost subtracted reward by the apportionment scaling term, thereby accounting for the apportionment cost of time.

      The revision of original figure 5, its figure legend, and accompanying text now make clear the meaning of apportionment cost, how it can be considered a subtraction from the reward of a pursuit, or, equivalently, how it can be thought of as the result of scaling down of opportunity cost subtracted reward.

      It could be written as a subtraction, by subtracting a second copy of the rest of the SV expression scaled by t_in/(t_in + t_out). But that shows the apportionment cost to depend on the opportunity cost, which is odd because the original motivation on line 404 was to resolve the lack of independence between terms in the SV expression.

      On line 404 of the original manuscript, we point out that the simple equation―which is a reprisal of McNamara’s insight―is problematic in that its terms on the RHS are not independent: the global reward rate is dependent on the considered pursuit’s reward (see Fig5B). The alternative expression for subjective value that we derive expresses sv in terms that are all independent of one another. We may have unintentionally obscured that fact by having already defined rho<sup>in</sup> as r<sup>in</sup>/ t<sup>in</sup> and rho<sup>out</sup> as r<sup>out</sup>/t<sup>out</sup> on lines 306 and 307.

      Therefore, in the revision, Ap 8 is expressed so to keep clear that it uses terms that are all independent of one another, and only subsequently express this formula with the simplifying substitution, rho<sup>out</sup>.

      That all said, we understand the reviewer’s point to be that the parenthetical terms relating the opportunity cost and the apportionment rescaling both contain within them the parameter t<sup>out</sup>, and in this way these concepts we put forward to understand the alternative equation are non-independent. That is correct, but it isn’t at odds with our objective to express SV in terms that are independent with one another (which we do). Our motivation in introducing these concepts is to provide insight and intuition into the cost of time (especially now with a clear and simple definition of apportionment cost stated). We go to lengths to demonstrate their relationship to each other.

      (4) In the analysis of discounting functions (line 664 and beyond), the paper doesn't say much about the fact that many discounting studies take specific measures to distinguish true time preferences from opportunity costs and reward-rate maximization.

      We understand the reviewer’s comment to connote that temporal decision-making worlds in which delay time does not preclude reward from outside the current pursuit is a means to distinguish time preference from the impact of opportunity cost. One contribution of this work is to demonstrate that, from a reward-rate maximization framework, an accounting of opportunity cost is not sufficient to understand apparent time preferences as distinguishable from reward-rate maximization. The apportionment cost of time must also be considered to have a full appreciation of the cost of time. For instance, let us consider a temporal decision-making world in which there is no reward received outside the considered pursuit. In such a world, there is no opportunity cost of time, so apparent temporal discounting functions would appear as if purely hyperbolic as a consequence of the apportionment cost of time alone. Time preference, as revealed experimentally by the choices made between a SS and a LL reward, then, seem confounding, as preference can reverse from a SS to a LL option as the displacement of those options (maintaining their difference in time) increases (Green, Fristoe, and Myerson 1994; Kirby and Herrnstein 1995). While this shift, the so-called “Delay effect”, could potentially arise as a consequence of some inherent time preference bias of an agent, we demonstrate that a reward-rate maximal agent exhibits hyperbolic discounting, and therefore it would also exhibit the Delay effect, even though it has no time preference.

      In the revision we now make reference to the Delay Effect (in abstract, results new section “The Delay Effect” with new figure 14, and in the discussion), which is taken as evidence of time preference in human and animal literature, and note explicitly how a reward-rate maximizing agent would also exhibit this behavior as a consequence of apparent hyperbolic discounting.

      In many of the human studies, delay time doesn't preclude other activities.

      Our framework is generalizable to worlds in which being in pursuit does not preclude an agent from receiving reward during that time at the outside reward rate. Original Ap 13 solves for such a condition, and shows that in this context, the opportunity cost of time drops out of the SV equation, leaving only the consequences of the apportionment cost of time. We made reference to this case on lines 1032-1034 of the original manuscript: “In this way, such hyperbolic discounting models [models that do not make an accounting of opportunity cost] are only appropriate in worlds with no “outside” reward, or, where being in a pursuit does not exclude the agent from receiving rewards at the rate that occurs outside of it (Ap. 13).”

      The note and reference is fleeting in the original work. We take the reviewer’s suggestion and now add paragraphs in the discussion on the difference between humans and animals in apparent discounting, making specific note of human studies in which delay time doesn’t preclude receiving outside reward while engaged in a pursuit. Relatedly, hyperbolic discounting is oft considered to be less steep in humans than in animals. As the reviewer points out, these assessments are frequently made under conditions in which being in a pursuit does not preclude receiving reward from outside the pursuit. When humans are tested under conditions in which outside rewards are precluded, they exhibit far steeper discounting. We now include citation to that observation (Jimura et al. 2009). We handle such conditions in original AP 13, and show how, in such worlds, the opportunity cost of time drops out of the equation. The consequence of this is that the apparent discounting function would become less steep (the agent would appear as if more patient), consistent with reports.

      “Relating to the treatment of opportunity cost, we also note that many investigations into temporal discounting do not make an explicit distinction between situations in which 1) subjects continue to receive the usual rewards from the environment during the delay to a chosen pursuit, and 2) situations in which during a chosen pursuit’s delay no other rewards or opportunities will occur (Kable & Glimcher, 2007; Kirby & Maraković, 1996; McClure, Laibson, Loewenstein, & Cohen, 2004). Commonly, human subjects are asked to answer questions about their preferences between options for amounts they will not actually earn after delays they will not actually have to wait, during which it is unclear whether they are really investing time away from other options or not (Rosati et al., 2007). In contrast, in most animal experiments, subjects actually receive reward after different delays during which they do not receive new options or rewards. By our formulation, when a pursuit does not exclude the agent from receiving rewards at the rate that occurs outside, the opportunity cost of time drops out of the subjective value equation (Ap 12).

      Equation 10. The value of initiating a pursuit when pursuit does not exclude receiving rewards at the outside rate (Ap 12)

      Therefore, the reward-rate maximizing discounting function in these worlds is functionally equivalent to the situation in which the outside reward rate is zero, and will―lacking an opportunity cost―be less steep. This rationalizes why human discounting functions are often reported to be longer (gentler) than animal discounting functions: they are typically tested in conditions that negate opportunity cost, whereas animals are typically tested in conditions that enforce opportunity costs. Indeed, when humans are made to wait for actually received reward, their observed discounting functions are much steeper (Jimura et al. 2009). “

      In animal studies, rate maximization can serve as a baseline against which to measure additional effects of temporal discounting. This is an important caveat to claims about discounting anomalies being rational under rate maximization (e.g., line 1024).

      We agree that the purpose of this reward-rate maximizing framework is to serve as a point of comparison in which effects of temporal intervals and rewards that define the environment can be analyzed to better understand the manner in which animals and humans deviate from this ideal behavior. Our interest in this work is in part motivated by a desire to have a deeper understanding of what “true” time preference means. Using the reward-rate maximizing framework here provides a means to speak about time preferences (ie biases) in terms of deviation from optimality. From this perspective, a reward-rate maximal agent doesn’t exhibit time preference: its actions are guided solely by reward-rate optimizing valuation. Therefore, one contribution of this work is to show that purported signs of time preference (hyperbolic discounting, magnitude, sign, and (now) delay effect) can be explained without invoking time preference. What errors from optimality that remain following an proper accounting of reward-rate maximizing behavior should then, and only then, be considered from the lens of time preference (bias).

      (5) The paper doesn't feature any very concrete engagement with empirical data sets. This is ok for a theoretical paper, but some of the characterizations of empirical results that the model aims to match seem oversimplified. An example is the contention that real decision-makers are optimal in accept/reject decisions (line 816 and elsewhere). This isn't always true; sometimes there is evidence of overharvesting, for example.

      We would like to note that the scope of this paper is limited to examining the value of initiating a pursuit, rather than the value of continuing within a pursuit. The issue of continuing within a pursuit constitutes a third fundamental topology, which could be called give-up or patch-foraging, and is complex and warrants its own paper. In Give-up topologies, which are distinct from Forgo, and Choice topologies, the reviewer is correct in pointing out that the preponderance of evidence demonstrates that animals and humans are as if overpatient, adopting a policy of investing too much time within a pursuit, than is warranted_._ In Forgo instances, however, the evidence supports near optimality.

      (6) Related to the point above, it would be helpful to discuss more concretely how some of this paper's theoretical proposals could be empirically evaluated in the future. Regarding the magnitude and sign effects of discounting, there is not a very thorough overview of the several other explanations that have been proposed in the literature. It would be helpful to engage more deeply with previous proposals and consider how the present hypothesis might make unique predictions and could be evaluated against them.

      We appreciate the reviewer’s point that there are many existing explanations for these various ‘anomalous’ effects. We hold that the point of this work is to demonstrate that these effects are consistent with a reward-rate maximizing framework so do not require additional assumptions, like separate processes for small and large rewards, or the inclusion of a utility function.

      Nonetheless, there is a diversity of explanations for the sign and magnitude effect, and, (now with its explicit inclusion in the revision) the delay effect. Therefore, we now also include reference to additional work which proffers alternative explanations for the sign and magnitude effects, (as reviewed by (Kalenscher and Pennartz 2008; Frederick et al. 2002)), as well as a scalar timing account of non-stationary time preference (Gibbon, 1977).

      With respect to making predictions, this framework makes the following in regards to the magnitude, sign, and (now in the revision) delay effect: in Discussion, Magnitude effect subsection: “The Magnitude Effect should be observed, experimentally, to diminish when 1) increasing the outside time while holding the outside reward constant, (thus decreasing the outside reward rate), or when 2) decreasing the outside reward while holding the outside time constant (thus decreasing the outside reward rate). However, 3) the Magnitude Effect would exaggerate as the outside time increased while holding the outside reward rate constant.”, in Sign effect subsection: “…we then also predict that the size of the Sign effect would diminish as the outside reward rate decreases (and as the outside time increases), and in fact would invert should the outside reward rate turn negative (become net punishing), such that punishments would appear to discount more steeply than rewards.” Delay effect subsection: “...a sign of irrationality is that a preference reversal occurs at delays greater than what a reward-rate-maximizing agent would exhibit.”

      A similar point applies to the 'malapportionment hypothesis' although in this case there is a very helpful section on comparisons to prior models (line 1163). The idea being proposed here seems to have a lot in common conceptually with Blanchard et al. 2013, so it would be worth saying more about how data could be used to test or reconcile these proposals.

      We thank the reviewer for holding that the section of model comparisons to be very helpful. We believe the text previously dedicated to this issue to be sufficient in this regard. We have, however, adding substantively to the Malapportionment Hypothesis section (Discussion) and its accompanying figure, to make explicit a number of predictions from the Malapportionment hypothesis as it relates to Hyperbolic discounting, the Delay Effect, and the Sign and Magnitude Effects.

      Reviewer #1 Recommendations

      (1) As a general note about the figures, it would be helpful to specify, either graphically or in the caption, what fixed values of reward sizes and time intervals are being assumed for each illustration.

      Thank you for the suggestion. We attempted to keep graphs as uncluttered as possible, but agree that for original figures 4,5,16, and 17, which didn’t have numbered axes, that we should provide the amounts in the captions in the revised figures (4,5, and now 17,18). These figures did not have numerics as their shapes and display are to illustrate the form of the relationship between vectors, being general to the values they may take.

      We now include in the captions for these figures the parameter amounts used.

      (2) Should Equation 2 have t in the denominator instead of r?

      Indeed. We thank the reviewer for catching this typographical error.

      We have corrected it in the revision.

      (3) General recommendation:

      My view is that in order for the paper's eLife assessment to improve, it would be necessary to resolve points 1 through 4 listed under "weaknesses" in my public review, which pertain to clarity and acknowledgement of prior work. I think a lot hinges on whether the authors can respond to point #3 by making a more compelling case for the usefulness and generality of the 'apportionment cost' concept, since that idea is central to the paper's contribution.

      We believe these critical points (1-4) to improve the paper will now have been addressed to the reviewer’s satisfaction.

      Reviewer #2 (Public review):

      While the details of the paper are compelling, the authors' presentation of their results is often unclear or incomplete:

      (1) The mathematical details of the paper are correct but contain numerous notation errors and are presented as a solid block of subtle equation manipulations. This makes the details of the authors' approach (the main contribution of the paper to the field) highly difficult to understand.

      We thank the reviewers for having detected typographical errors regarding three equations. They have been corrected. The first typographical error in the original main text (Line 277) regards equation 2 and will be corrected so that equation 2 appears correctly as

      The second typo regards the definition of the considered pursuit’s reward rate which appear in the original main text (line 306), and has been corrected to appear as

      The third typographical error occurred in conversion from Google Sheets to Microsoft Word appearing in the original main text (line 703) and regards the subjective value expression when no reward is received in an intertrial interval (ITI). It has been corrected to appear as

      (2) One of the main contributions of the paper is the notion that time’s cost in decision-making contains an apportionment cost that reflects the allocation of decision time relative to the world. The authors use this cost to pose a hypothesis as to why subjects exhibit sub-optimal behavior in choice decisions. However, the equation for the apportionment cost is never clearly defined in the paper, which is a significant oversight that hampers the effectiveness of the authors' claims.

      We thank the reviewer for pressing on this critical point. Reviewers commonly identified a need to provide a concise and intuitive definition of apportionment cost, and to explicitly solve and provide for its mathematical expression.

      We added the following succinct verbal description of apportionment cost… “Apportionment cost is the difference in reward that can be expected, on average, between a policy of taking versus a policy of not taking the considered pursuit, over a time equal to its duration.” This definition appears in new paragraphs (as below) describing apportionment cost in the results section “Time’s cost: opportunity & apportionment costs determine a pursuit’s subjective value”, and is accompanied by equations for apportionment cost, and a figure giving its geometric depiction (Figure 5). We also expanded original figure 5 and its legend (so as to illustrate the apportionment scaling factor and the apportionment cost), and its accompanying main text, to further illustrate and clarify apportionment cost, and its relationship to opportunity cost, and time’s cost.

      “What, then, is the amount of reward by which the opportunity cost-subtracted reward is scaled down to equal the sv of the pursuit? This amount is the apportionment cost of time. The apportionment cost of time (height of the brown vertical bar, Figure 5F) is the global reward rate after taking into account the opportunity cost (slope of the magenta-gold dashed line in Figure 5F) times the time of the considered pursuit. Equally, the difference between the inside and outside reward rates, times the time of the pursuit, is the apportionment cost when scaled by the pursuit’s weight, i.e., the fraction that the considered pursuit is to the total time to traverse the world (Equation 9, right hand side). From the perspective of decision-making policies, apportionment cost is the difference in reward that can be expected, on average, between a policy of taking versus a policy of not taking the considered pursuit, over a time equal to its duration (Equation 9 center, Figure 5F).

      Equation 9. Apportionment Cost.

      While this difference is the apportionment cost of time, the opportunity cost of time is the amount that would be expected from a policy of not taking the considered pursuit over a time equal to the considered pursuit’s duration. Together, they sum to Time’s Cost (Figure 5G). Expressing a pursuit’s worth in terms of the global reward rate obtained under a policy of accepting the pursuit type (Figure 5 left column), or from the perspective of the outside reward and time (Figure 5 right column), are equivalent. However, the latter expresses sv in terms that are independent of one another, conveys the constituents giving rise to global reward rate, and provides the added insight that time’s cost comprises an apportionment as well as an opportunity cost.”

      (3) Many of the paper's figures are visually busy and not clearly detailed in the captions (for example, Figures 6-8). Because of the geometric nature of the authors' approach, the figures should be as clean and intuitive as possible, as in their current state, they undercut the utility of a geometric argument.

      We endeavored to make our figures as simple as possible. We have made in the revision changes to figures that we believe improve their clarity. These include: 1) breaking some figures into more panels when more than one concept was being introduced (such as in revised Figure 5 , 6, 7, and 8), 2) using the left hand y axis for the outside reward, and the right hand axis for the inside reward when plotting the “in” and “outside” reward, and indicating their respective numerics (which run in opposite directions), 3) adding a legend to the figures themselves where needed (revised figures 10, 11, 12, 14) 4) adding the values used to the figure captions, where needed, and 5) ensuring all symbols are indicated in legends.

      (4) The authors motivate their work by focusing on previously-observed behavior in decision experiments and tell the reader that their model is able to qualitatively replicate this data. This claim would be significantly strengthened by the inclusion of experimental data to directly compare to their model's behavior. Given the computational focus of the paper, I do not believe the authors need to conduct their own experiments to obtain this data; reproducing previously accepted data from the papers the authors' reference would be sufficient.

      Our objective was not to fit experimentally observed data, as is commonly the goal of implementation/computational models. Rather, as a theory, our objective is to rationalize the broad, curious, and well-established pattern of temporal decision-making behaviors under a deeper understanding of reward-rate maximization, and from that understanding, identify the nature of the error being committed by whatever learning algorithm and representational architecture is actually being used by humans and animals. In doing so, we make a number of important contributions. By identifying and analyzing reward-rate-maximizing equations, we 1) provide insight into what composes time’s cost and how the temporal structure of the world in which it is embedded (its ‘context’) impacts the value of a pursuit, 2) rationalize a diverse assortment of temporal decision-making behaviors (e.g., Hyperbolic discounting, the Magnitude Effect, the Sign Effect, and the Delay effect), explaining them with no assumed free-fit parameter, and then, by analyzing error in parameters enabling reward-rate maximization, 3) identify the likely source of error and propose the Malapportionment Hypothesis. The Malapportionment Hypothesis identifies the underweighting of a considered pursuit’s “outside”, and not error in pursuit’s reward rates, as the source of error committed by humans and animals. It explains why animals and humans can present as suboptimally ‘impatient’ in Choice, but as optimal in Forgo. At the same time, it concords with numerous and diverse observations in decision making regarding whether to initiate a pursuit. The nature of this error also, then, makes numerous predictions. These insights inform future computational and experimental work by providing strong constraints on the nature of the algorithm and representational architecture used to learn and represent the values of pursuits. Rigorous test of the Malapportionment Hypothesis will require wholly new experiments.

      In the revision, we also now emphasize and add predictions of the Malapportionment Hypothesis, updated its figure (Figure 21), its legend, and its paragraphs in the discussion.

      “We term this reckoning of the source of error committed by animals and humans the Malapportionment Hypothesis, which identifies the underweighting of the time spent outside versus inside a considered pursuit but not the misestimation of pursuit rates, as the source of error committed by animals and humans (Figure 21). This hypothesis therefore captures previously published behavioral observations (Figure 21A) showing that animals can make decisions to take or forgo reward options that optimize reward accumulation (Krebs et al., 1977; Stephens and Krebs, 1986; Blanchard and Hayden, 2014), but make suboptimal decisions when presented with simultaneous and mutually exclusive choices between rewards of different delays (Logue et al., 1985; Blanchard and Hayden, 2015; Carter and Redish, 2016; Kane et al., 2019). The Malapportionment Hypothesis further predicts that apparent discounting functions will present with greater curvature than what a reward-rate-maximizing agent would exhibit (Figure 21B). While experimentally observed temporal discounting would have greater curvature, the Malapportionment Hypothesis also predicts that the Magnitude (Figure 21C) and Sign effect (Figure 21D) would be less pronounced than what a reward-rate-maximizing agent would exhibit, with these effects becoming less pronounced the greater the underweighting. Finally, with regards to the Delay Effect (Figure 21E), the Malapportionment Hypothesis predicts that preference reversal would occur at delays greater than that exhibited by a reward-rate-maximizing agent, with the delay becoming more pronounced the greater the underweighting outside versus inside the considered pursuit by the agent.”

      (5) While the authors reference a good portion of the decision-making literature in their paper, they largely ignore the evidence-accumulation portion of the literature, which has been discussing time-based discounting functions for some years. Several papers that are both experimentally-(Cisek et al. 2009, Thurs et al. 2012, Holmes et al. 2016) and theoretically-(Drugowitsch et al. 2012, Tajima et al. 2019, Barendregt et al. 22) driven exist, and I would encourage the authors to discuss how their results relate to those in different areas of the field.

      In this manuscript, we consider the worth of initiating one or another pursuit having completed a prior one, and not the issue of continuing within a pursuit having already engaged in it. The worth of continuing a pursuit, as in patch-foraging/give-up tasks, constitutes a third fundamental time decision-making topology which is outside the scope of the current work. It engages a large and important literature, encompassing evidence accumulation, and requires a paper on the value of continuing a pursuit in temporal decision making, in its own right, that can use the concepts and framework developed here. The excellent works suggested by the reviewer will be most relevant to that future work concerning patch-foraging/give-up topologies.

      Reviewer #2 Recommendations:

      (1) In Equation 1, the term rho_d is referred to as the reward rate of the default pursuit, when it should be the reward of the default pursuit.

      Regarding Equation 1, it is formulated to calculate the average reward received and average time spent per unit time spent in the default pursuit. So, f<sub>i</sub> is the encounter rate of pursuit i for one unit of time spent in the default pursuit (lines 259-262). Added to the summation in the numerator, we have the average reward obtained in the default pursuit per unit time () and in the denominator we have the time spent in the default pursuit per unit time (1).

      We have added clarifying text to assist in meaning of the equation in Ap 1, and thank the reviewer for pointing out this need.

      (2) The notation for "in" and "out" of a considered pursuit type begins as being used to describe the contribution from a single pursuit (without inter-trial interval) towards global reward rate and the contribution of all other factors (other possible pursuits and inter-trial interval) towards global reward rate, respectively, but is then used to describe the pursuit's contribution and the inter-trial interval's contribution, respectively, to the global reward rate. This should be cleaned up to be consistent throughout, or at the very least, it should be addressed when this special case is considered the default.

      As understood by the reviewer, “in” and “out” of the considered pursuit type describes the general form by which a world can be cleaved into these two parts: the average time and reward received outside of the considered pursuit type for the average time and reward received within that pursuit type. A specific, simple, and common experimental instance would be a world composed of one or another pursuit and an intertrial interval.

      We now make clear how such a world composed of a considered pursuit and an inter trial interval would be but one special case. In example cases where t<sup>out</sup> represents the special case of an inter-trial interval, this is now stated clearly. For instance, we do so when discussing how a purely hyperbolic discounting function would apply in worlds in which no reward is received in t<sup>out</sup>, stating that this is often the case common to experimental designs where t<sup>out</sup> represents an intertrial interval with no reward. Importantly, by the new inclusion of illustrated worlds in the revision that have n-number pursuits that could occur from a default pursuit and 1) equal frequency (Supplemental 1), and 2) at differing frequencies (Supplemental 2), we make more clear the generalizability and utility of this t<sup>out</sup>/tin concept.

      (3) Figure 5 should make clear the decomposition of time's cost both graphically and functionally. As it stands, the figure does not define the apportionment cost.

      In the revision of original fig 5, we now further decompose the figure to effectively convey 1) what opportunity cost, and (especially) 2) the apportionment cost is, both graphically and mathematically, 3) how time’s cost is comprised by them, 4) how the apportionment scaling term scales the opportunity-cost-subtracted reward by time’s allocation to equal the subjective value, and 4) the equivalence between the expression of time’s cost using terms that are not independent of one another with the expression of time’s cost using terms that are independent of one another.

      (4) Figures 6-8 do not clearly define the dots and annuli used in panels B and C.

      We have further decomposed figures 6-8 so that the functional form of opportunity, apportionment, and time’s cost can be more clearly appreciated, and what their interrelationship is with respect to changing outside reward and outside time, and clearly identify symbols used in the corresponding legends.

      (5) The meaning of a negative subjective value should be specifically stated. Is it the amount a subject would pay to avoid taking the considered pursuit?

      As the reviewer intuits, negative subjective value can be considered the amount an agent ought be willing to pay to avoid taking the considered pursuit.

      We now include the following lines in “The forgo decision can also be made from subjective value” section in reference to negative subjective value…

      “A negative subjective value thus indicates that a policy of taking the considered pursuit would result in a global reward rate that is less than a policy of forgoing the considered pursuit. Equivalently, a negative subjective value can be considered the amount an agent ought be willing to pay to avoid having to take the considered pursuit.”

      (6) Why do you define the discounting function as the normalized subjective value? This choice should be justified, via literature citations or a well-described logical argument.

      The reward magnitude normalized subjective value-time function is commonly referred to as the temporal discounting function as it permits comparison of the discount rate isolated from a difference in reward magnitude and/or sign and is deeply rooted in historical precedent. As the reviewer points out, the term is overloaded, however, as investigations in which comparisons between the form of subjective value-time functions is not needed tend to refer to these functions as temporal discounting functions as well.

      We make clear in the revised text in the introduction our meaning and use of the term, the justification in doing so, and its historical roots.

      “Historically, temporal decision-making has been examined using a temporal discounting function to describe how delays in rewards influence their valuation. Temporal discounting functions describe the subjective value of an offered reward as a function of when the offered reward is realized. To isolate the form of discount rate from any difference in reward magnitude and sign, subjective value is commonly normalized by the reward magnitude when comparing subjective value-time functions (Strotz, 1956, Jimura, 2009). Therefore, we use the convention that temporal discounting functions are the magnitude-normalized subjective value-time function (Strotz, 1956).”

      Special addition. In investigating the historical roots of the discounting function prompted by the reviewer, we learned (Grüne-Yanoff 2015) that it was Mazur that simply added the “1+k” in the denominator of the hyperbolic discounting function. Our derivation for the reward-rate optimal agent makes clear why apparent temporal discounting functions ought have this general form.

      Therefore, we add the following to the “Hyperbolic Temporal Discounting Function section in the discussion…

      “It was Ainslie (Ainslie, 1975) who first understood that the empirically observed “preference reversals” between SS and LL pursuits could be explained if temporal discounting took on a hyperbolic form, which he initially conjectured to arise simply from the ratio of reward to delay (Grüne-Yanoff 2015). This was problematic, however, on two fronts: 1) as the time nears zero, the value curve goes to infinity, and 2) there is no accommodation of differences observed within and between subjects regarding the steepness of discounting. Mazur (Mazur, 1987) addressed these issues by introducing 1 + k into the denominator, providing for the now standard hyperbolic discounting function, . Introduction of “1” solved the first issue, though “it never became fully clear how to interpret this 1” (Grüne-Yanoff 2015; interviewing Ainslie). Introduction of the free-fit parameter, k, accommodated the variability observed across and within subjects by controlling the curvature of temporal discounting, and has become widely interpreted as a psychological trait, such as patience, or willingness to delay gratification (Frederick et al., 2002).”

      …continuing later in that section to explain why the reward-rate optimal agent would exhibit this general form…

      “Regarding form, our analysis reveals that the apparent discounting function of a reward-rate-maximizing agent is a hyperbolic function…

      …which resembles the standard hyperbolic discounting function, , in the denominator, where . Whereas Mazur introduced 1 + k to t in the denominator to 1) force the function to behave as t approaches zero, and 2) provide a means to accommodate differences observed within and between subjects, our derivation gives cause to the terms 1 and k, their relationship to one another, and to t in the denominator. First, from our derivation, “1” actually signifies taking t<sub>out</sub> amount of time expressed in units of t<sub>out</sub> (t<sub>out</sub>/t<sub>out</sub>=1) and adding it to t<sub>in</sub>  amount of time expressed in units of t<sub>out</sub> (ie, the total time to make a full pass through the world expressed in terms of how the agent apportions its time under a policy of accepting the considered pursuit).”

      Additional Correction. In revising the section, “Hyperbolic Temporal Discounting Functions” in the discussion, we also detected an error in our description of the meaning of suboptimal bias for SS. In the revision, the sentence now reads…

      More precisely, what is meant by this suboptimal bias for SS is that the switch in preference from LL to SS occurs at an outside reward rate that is lower—and/or an outside time that is greater —than what an optimal agent would exhibit.”

      (7) Figure 15B should have negative axes defined for the pursuit's now negative reward.

      Yes- excellent point.

      To remove ambiguity regarding the valence of inside and outside reward magnitudes, we have changed all such figures so that the left hand y-axis is used to signify the outside reward magnitude and sign, and so that the right hand y-axis is used to signify the inside reward magnitude and sign.

      With respect to the revision of original 15B, this change now makes clear that the inside reward label and numerics on the right hand side of the graph run from positive (top) to negative (bottom) values so that it can now be understood that the magnitude of the inside reward is negative in this figure (ie, a punishment). The left hand y-axis labeling the outside reward magnitude has numerics that run in the opposite direction, from negative (top) to positive (bottom). In this figure, the outside reward rate is positive whereas the inside reward rate is negative.

      (8) When comparing your discounting function to the TIMERR and Heuristic models, it would be useful to include a schematic plot illustrating the different obtainable behaviors from all models rather than just telling the reader the differences.

      We hold that the descriptions and references are sufficient to address these comparisons.

      (9) I would strongly suggest cleaning up all appendices for notation…

      The typographical errors that have been noted in these reviews have all been corrected. We believe the reviewer to be referring here to the manner that we had cross-referenced Equations in the appendices and main text which can lead to confusion between whether an equation number being referenced is in regard to its occurrence in the main text or its occurrence in the appendices.

      In the revision, we eliminate numbering of equations in the appendices except where an equation occurs in an appendix that is referenced within the main text. In the main text, important equations are numbered sequentially and note the appendix from which they derive. If an equation in an appendix is referenced in the main text, it is noted within the appendix it derives.

      …and replacing some of the small equation manipulations with written text describing the goal of each derivation.

      To increase clarity, we have taken the reviewer’s helpful suggestion, adding helper text in the appendices were needed, and have bolded the equations of importance within the Appendices (rather than removing equation manipulations making clear steps of derivation).

      (10) I would suggest moving the table in Appendix 11 to the main text where misestimation is referenced.

      So moved. This appendix now appears in the main text as table 1 “Definitions of misestimating global reward rate-enabling parameters”.

      Reviewer #3 (Public review):

      One broad issue with the paper is readability. Admittedly, this is a complicated analysis involving many equations that are important to grasp to follow the analyses that subsequently build on top of previous analyses.

      But, what's missing is intuitive interpretations behind some of the terms introduced, especially the apportionment cost without referencing the equations in the definition so the reader gets a sense of how the decision-maker thinks of this time cost in contrast with the opportunity cost of time.

      We thank the reviewer for encouraging us to formulate a succinct and intuitive statement as to the nature of apportionment cost. We thank the reviewer for pressing for a succinct and intuitive verbal description.

      We added the following succinct verbal description of apportionment cost… “Apportionment cost is the difference in reward that can be expected, on average, between a policy of taking versus a policy of not taking the considered pursuit, over a time equal to its duration.” This definition appears in a new paragraph (as below) describing apportionment cost in the results section “Time’s cost: opportunity & apportionment costs determine a pursuit’s subjective value”, and is accompanied by equations for apportionment cost, and a figure giving its geometric depiction (Figure 5). We also expanded original figure 5 and its legend (so as to illustrate the apportionment scaling factor and the apportionment cost), and its accompanying main text, to further illustrate and clarify apportionment cost, and its relationship to opportunity cost, and time’s cost.

      “What, then, is the amount of reward by which the opportunity cost-subtracted reward is scaled down to equal the sv of the pursuit? This amount is the apportionment cost of time. The apportionment cost of time (height of the brown vertical bar, Figure 5F) is the global reward rate after taking into account the opportunity cost (slope of the magenta-gold dashed line in Figure 5F) times the time of the considered pursuit. Equally, the difference between the inside and outside reward rates, times the time of the pursuit, is the apportionment cost when scaled by the pursuit’s weight, i.e., the fraction that the considered pursuit is to the total time to traverse the world (Equation 9, right hand side). From the perspective of decision-making policies, apportionment cost is the difference in reward that can be expected, on average, between a policy of taking versus a policy of not taking the considered pursuit, over a time equal to its duration (Equation 9 center, Figure 5F).

      Equation 9. Apportionment Cost.

      While this difference is the apportionment cost of time, the opportunity cost of time is the amount that would be expected from a policy of not taking the considered pursuit over a time equal to the considered pursuit’s duration. Together, they sum to Time’s Cost (Figure 5G). Expressing a pursuit’s worth in terms of the global reward rate obtained under a policy of accepting the pursuit type (Figure 5 left column), or from the perspective of the outside reward and time (Figure 5 right column), are equivalent. However, the latter expresses sv in terms that are independent of one another, conveys the constituents giving rise to global reward rate, and provides the added insight that time’s cost comprises an apportionment as well as an opportunity cost.”

      The above definition of apportionment cost adds to other stated relationships of apportionment cost found throughout the paper (original lines 434,435,447,450).

      Re-analysis of some existing empirical data through the lens of their presented objective functions, especially later when they describe sources of error in behavior.

      Our objective was not to fit experimentally observed data, as is commonly the goal of implementation/computational models. Rather, as a theory, our objective is to rationalize the broad, curious, and well-established pattern of temporal decision-making behaviors under a deeper understanding of reward-rate maximization, and from that understanding, identify the nature of the error being committed by whatever learning algorithm and representational architecture is actually being used by humans and animals. In doing so, we make a number of important contributions. By identifying and analyzing reward-rate-maximizing equations, we 1) provide insight into what composes time’s cost and how the temporal structure of the world in which it is embedded (its ‘context’) impacts the value of a pursuit, 2) rationalize a diverse assortment of temporal decision-making behaviors (e.g., Hyperbolic discounting, the Magnitude Effect, the Sign Effect, and the Delay effect), explaining them with no assumed free-fit parameter, and then, by analyzing error in parameters enabling reward-rate maximization, 3) identify the likely source of error and propose the Malapportionment Hypothesis. The Malapportionment Hypothesis identifies the underweighting of a considered pursuit’s “outside”, and not error in pursuit’s reward rates, as the source of error committed by humans and animals. It explains why animals and humans can present as suboptimally ‘impatient’ in Choice, but as optimal in Forgo. At the same time, it concords with numerous and diverse observations in decision making regarding whether to initiate a pursuit. The nature of this error also, then, makes numerous predictions. These insights inform future computational and experimental work by providing strong constraints on the nature of the algorithm and representational architecture used to learn and represent the values of pursuits. Rigorous test of the Malapportionment Hypothesis will require wholly new experiments.

      In the revision, we also now emphasize and add predictions of the Malapportionment Hypothesis, augmenting its figure (Figure 21), its legend, and its paragraphs in the discussion.

      “We term this reckoning of the source of error committed by animals and humans the Malapportionment Hypothesis, which identifies the underweighting of the time spent outside versus inside a considered pursuit but not the misestimation of pursuit rates, as the source of error committed by animals and humans (Figure 21). This hypothesis therefore captures previously published behavioral observations (Figure 21A) showing that animals can make decisions to take or forgo reward options that optimize reward accumulation (Krebs et al., 1977; Stephens and Krebs, 1986; Blanchard and Hayden, 2014), but make suboptimal decisions when presented with simultaneous and mutually exclusive choices between rewards of different delays (Logue et al., 1985; Blanchard and Hayden, 2015; Carter and Redish, 2016; Kane et al., 2019). The Malapportionment Hypothesis further predicts that apparent discounting functions will present with greater curvature than what a reward-rate-maximizing agent would exhibit (Figure 21B). While experimentally observed temporal discounting would have greater curvature, the Malapportionment Hypothesis also predicts that the Magnitude (Figure 21C) and Sign effect (Figure 21D) would be less pronounced than what a reward-rate-maximizing agent would exhibit, with these effects becoming less pronounced the greater the underweighting. Finally, with regards to the Delay Effect (Figure 21E), the Malapportionment Hypothesis predicts that preference reversal would occur at delays greater than that exhibited by a reward-rate-maximizing agent, with the delay becoming more pronounced the greater the underweighting outside versus inside the considered pursuit by the agent.”

      Reviewer #3 Recommendations:

      As mentioned above, the readability of this paper should be improved so that the readers can follow the derivations and your analyses better. To this end, careful numbering of equations, following consistent equation numbering formats, and differentiating between appendix referencing and equation numbering would have gone a long way in improving the readability of this paper. Some specific questions are noted below.

      To increase clarity, in the revision we eliminated numbering of equations in the appendices except where an equation occurs in an appendix that is referenced within the main text. In the main text, important equations are thus numbered sequentially as they appear and note the appendix from which they derive. If an equation in an appendix is referenced in the main text, it is noted within the appendix it derives.

      (1) In general, it is unclear what the default pursuit is. From the schematic on the left (forgo decision), it appears to be the time spent in between reward-giving pursuits. However, this schematic also allows for smaller rewards to be attained during the default pursuit as do subsequent equations that reference a default reward rate. Here is where an example would have really benefited the authors in getting their point across as to what the default pursuit is in practice in the forgo decisions and how the default reward rate could be modulated.

      (1) The description of the default pursuit has been modified in section “Forgo and Choice decision topologies” to now read… “After either the conclusion of the pursuit, if accepted, or immediately after rejection, the agent returns to a pursuit by default (the “default” pursuit). This default pursuit effectively can be a waiting period over which reward could be received, and reoccurs until the next pursuit opportunity becomes available.” (2) Additionally, helper text has been added to Ap1 regarding the meaning of time and reward spent in the default pursuit. Finally, (3) new figures concerning n-pursuits occurring at the same (Supplement 1) or different (Supplement 2) frequencies from a default pursuit is now added, providing examples as suggested by the reviewer.

      (2) I want to clarify my understanding of the topologies in Figure 1. In the forgo, do they roam in the "gold" pursuit indefinitely before they are faced with the purple pursuit? In general, comparing the 2 topologies, it seems like in the forgo decision, they can roam indefinitely in the gold topology or choose the purple but must return to the gold.

      The reviewer’s understanding of the topology is correct. The agent loops across one unit time in the default gold pursuit indefinitely, though the purple pursuit (or any pursuit that might exist in that world) occurs on exit from gold at its frequency per unit time. The default gold pursuit will then itself have an average duration in units of time spent in gold. As the reviewer states, the agent can re-enter into gold from having exited gold, and can enter gold from having exited purple, but cannot re-enter purple from having exited purple; rather, it must enter into the default pursuit.

      …Another point here is that this topology is highly simplified (only one considered pursuit). So it may be helpful to either add a schematic for the full topology with multiple pursuits or alternatively, provide the corresponding equations (at least in appendix 1 and 2) for the simplified topology so you can drive home the intuition behind derived expressions in these equations.

      We understand the reviewer to be noting that, while, the illustrated example is of the simple topology, the mathematical formulation handles the case of n-number pursuits, and that illustrating a world in which there are a greater number of pursuits, corresponding to original appendices 1&2, would assist readers in understanding the generality of these equations.

      An excellent suggestion. We have now n-pursuit world illustrations where each pursuit occurs at the same (Supplemental Figure 1) and at different frequencies (Supplemental Figure 2) to the manuscript, and have added text to assist in understanding the form of the equation and its relationship to unit time in the default pursuit in the main and in the appendices.

      (3) In Equation and Appendix 1, there are a few things that are unclear. Particularly, why is the expected time of the default option E(t_default )= 1/(∑_(i=1)^n f_i )? Similarly, why is the E(r_default )= ρ_d/(∑_(i=1)^n f_i )? Looking at the expression for E(r_default ), it implies that across all pursuits 1 through n, the default option is encountered only once. Ultimately, in Equation 1.4, (and Equation 1), the units of the two terms in the numerator don't seem to match. One is a reward rate (ρ_d) and the other is a reward value. This is the most important equation of the paper since the next several equations build upon this. Therefore, the lack of clarity here makes the reader less likely to follow along with the analysis in rigorous detail. Better explanations of the terms and better formatting will help alleviate some of these issues.

      The equation is formulated to calculate the average reward received and average time spent per unit time spent in the default pursuit. So, f<sub>i</sub> is the encounter rate of pursuit i for one unit of time spent in the default pursuit. Added to the summation in the numerator we have the average reward obtained in the default pursuit per unit time () and in the denominator we have the time spent in the default pursuit per unit time (1).

      Text explaining the above equation has been added to Ap 1.

      (4) In equation and appendix 2, I'm trying to relate the expressions for t_out and r_out to the definitions "average time spent outside the considered pursuit". If I understand the expression in Equation 2.4 on the right-hand side, the numerator is the total time spent in all of the pursuits in the environment and the denominator refers to the number of times the considered pursuit is encountered. It is unclear as to why this is the average time spent outside the considered pursuit. In my mind, the expression for average time spent outside the considered pursuit would look something like t_out=1+ ∑_(i≠in)〖p_i t_i 〗= 1+ ∑_(i≠in)〖f_i/(∑_(j=1)^n f_j ) * t_i 〗. It is unclear how these expressions are then equivalent.

      Regarding the following equation,

      f<sub>i</sub> is the probability that pursuit i will be encountered during a single unit of time spent in the default pursuit. The numerator of the expression is the average amount of time spent across all pursuits, excepting the considered pursuit, per unit time spent in the default pursuit. Note that the + 1 in the numerator is accounting for the unit of time spent in the default pursuit and is added outside of the sum. Since f<sub>in</sub> is the probability that the considered pursuit will be encountered per unit of time spent in the default pursuit, is the average amount of time spent in the default pursuit between encounters of the considered pursuit. By multiplying the average time spent across all outside pursuits per unit of time in the default pursuit by the average amount of time spent in the default pursuit between encounters of the considered pursuit, we get the average amount of time spent outside the considered pursuit per encounter of the considered pursuit. This is calculated as if the pursuit encounters are mutually exclusive within a single unit of time spent within the default pursuit, as this is the case as the length of our unit time (delta t) approaches zero.

      The above text explaining the equation has been added to Ap 2.

      (5) In Figure 3, one huge advantage of this separation into in-pursuit and out-of-pursuit patches is that the optimal reward rate maximizing rule becomes one that compares ρ_in and ρ_out. This contrasts with an optimal foraging rule which requires comparing to the global reward rate and therefore a circularity in solution. In practice, however, it is unclear how ρ_out will be estimated by the agent.

      How, in practice, a human or animal estimates the reward rates―be they the outside and/or global reward rate under a policy of accepting a pursuit―is the crux of the matter. This work identifies equations that would enable a reward-rate maximizing agent to calculate and execute optimal policies and emphasizes that the effective reward rates and weights of pursuits must be accurately appreciated for global reward rate optimization. In so doing, it makes a reckoning of behaviors commonly but erroneously treated as suboptimal. Then, by examining the consequences of misestimation of these enabling parameters, it identifies mis-weighting pursuits as the nature of the error committed by whatever algorithm and representational architecture is being used by humans and animals (the Malapportionment Hypothesis). This curious pattern identified and analyzed in this work thus provides a clue into the nature of the learning algorithm and means of representing the temporal structure of the environment that is used by humans and animals―the subject of future work.

      We note, however, that we do discuss existing models that grapple with how, in practice, how a human or animal may estimate the outside reward rate. Of particular importance is the TIMERR model, which estimates the outside reward rate from its past experience, and can make an accounting of many qualitative features widely observed. However, while appealing, it would mix prior ‘in’ and ‘outside’ experiences within that estimate, and so would fail to perform forgo tasks optimally. Something is still amiss, as this work demonstrates.

      (6) The apportionment time cost needs to be explained a little bit more intuitively. For instance, it is clear that the opportunity cost of time is the cost of not spending time in the rest of the environment relative to the current pursuit. But given the definition of apportionment cost here in lines 447- 448 "The apportionment cost relates to time's allocation in the world: the time spent within a pursuit type relative to the time spent outside that pursuit type, appearing in the denominator." The reference to the equation (setting aside the confusion regarding which equation) within the definition makes it a bit harder to form an intuitive interpretation of this cost. Please reference the equation being referred to in lines 447-448, and again, an example may help the authors communicate their point much better

      We thank the reviewer for pressing on this critical point.

      Action: We added the following succinct verbal description of apportionment cost… “Apportionment cost is the difference in reward that can be expected, on average, between a policy of taking versus a policy of not taking the considered pursuit, over a time equal to its duration.” This definition appears in a new paragraph (as below) describing apportionment cost in the results section “Time’s cost: opportunity & apportionment costs determine a pursuit’s subjective value”, and is accompanied by equations for apportionment cost, and a figure giving its geometric depiction (Figure 5).

      “What, then, is the amount of reward by which the opportunity cost-subtracted reward is scaled down to equal the sv of the pursuit? This amount is the apportionment cost of time. The apportionment cost of time (height of the brown vertical bar, Figure 5F) is the global reward rate after taking into account the opportunity cost (slope of the magenta-gold dashed line in Figure 5F) times the time of the considered pursuit. Equally, the difference between the inside and outside reward rates, times the time of the pursuit, is the apportionment cost when scaled by the pursuit’s weight, i.e., the fraction that the considered pursuit is to the total time to traverse the world (Equation 9, right hand side). From the perspective of decision-making policies, apportionment cost is the difference in reward that can be expected, on average, between a policy of taking versus a policy of not taking the considered pursuit, over a time equal to its duration (Equation 9 center, Figure 5F).

      Equation 9. Apportionment Cost.

      While this difference is the apportionment cost of time, the opportunity cost of time is the amount that would be expected from a policy of not taking the considered pursuit over a time equal to the considered pursuit’s duration. Together, they sum to Time’s Cost (Figure 5G). Expressing a pursuit’s worth in terms of the global reward rate obtained under a policy of accepting the pursuit type (Figure 5 left column), or from the perspective of the outside reward and time (Figure 5 right column), are equivalent. However, the latter expresses sv in terms that are independent of one another, conveys the constituents giving rise to global reward rate, and provides the added insight that time’s cost comprises an apportionment as well as an opportunity cost.”

      (7) The analyses in Figures 6 and 7 give a nice visual representation of how the time costs are distributed as a function of outside reward and time spent. However, without an expression for apportionment cost it is hard to intuitively understand these visualizations. This also relates to the previous point of requiring a more intuitive explanation of apportionment costs in relation to the opportunity cost of time. Based on my quick math, it seems that an expression for apportionment cost would be as follows: (r_in- ρ_out*t_in)*(t_in⁄t_out )/(t_in⁄t_out +1 ). The condition described in Figure 7 seems like the perfect place to compute the value of just apportionment cost when the opportunity cost is zero. It would be helpful to introduce the equation here.

      We designed original figure 7, as the reviewer appreciates, to emphasize that time has a cost even when there is no opportunity cost, being due entirely to the apportionment cost of time.

      We now provide the mathematical expression of apportionment cost and apportionment scaling in Figure 5, the point in the main text of its first occurrence.

      …and have expanded original figure 5, its legend (so as to illustrate the apportionment scaling factor and the apportionment cost), and its accompanying main text, to further illustrate and clarify apportionment cost, and its relationship to opportunity cost, and time’s cost.

      (8) The analysis regarding choice decisions is relatively straightforward, pending the concerns for the main equations listed above for the forgo decisions. Legends certainly would have helped me grasp Figures 10-12 better.

      We believe the reviewer is referring to missing labels for the Sooner Smaller pursuit, and the Larger Later Pursuit in these figures? We used the same conventions as in Figure 9, but we see now that adding these labels to these figures would be helpful, and add them in the revision.

      We have now added to the figures themselves figure legends indicating the Sooner Small Pursuit and the Larger Later Pursuit. We have also added to the main text to emphasize the points made in these figures regarding the impact of opportunity cost and apportionment cost.

      (9) The derivation of the temporal discounting function from subjective reward rate is much appreciated as it provides further evidence for potential equivalence between reward rate optimization and hyperbolic discounting, which is known to explain a slew of decision-making behaviors in the economics literature.

      We thank and greatly appreciate the reviewer for this recognition.

      In response to the reviewer’s comment, we have added text that further relates reward rate optimization to hyperbolic discounting…

      (1) We add discussion of how our normative derivation gives explanation to Mazur’s ad hoc addition of 1 + k to Ainslie’s reward/time hyperbolic discounting conception. See new first paragraph under “Hyperbolic Temporal Discounting Functions” for the historical origins of the standard hyperbolic equation (which are decidedly not normatively derived). And then see our discussion (new second paragraph in sections “The apparent discounting function of global….”) of how our normative derivation gives explanation to “1”, “k”, and their relationship to each other.

      (2) We add explicit treatment of the Delay Effect in a new “The Delay Effect” section of the results along with a figure, and in its corresponding Discussion section.

      Minor comments:

      (1) Typo in equation 2, should be t_i in the denominator within the summation, not r_i .

      We thank the reviewer for catching this typo, and have corrected it in the revision.

      (2) Before equation 6, typo when defining ρ_in= r_in/(t_in.). Should be t_in in the denominator, not r_out.

      We thank the reviewer for catching this typo, and have corrected it in the revision.

      (3) Please be consistent with equation numbers, placement of equation references, and the reason for placing appendix numbers. This will improve readability immensely.

      To increase clarity, in the revision we eliminated numbering of equations in the appendices except where an equation occurs in an appendix that is referenced within the main text. In the main text, important equations are thus numbered sequentially and note the appendix from which they derive. If an equation in an appendix is referenced in the main text, it is noted within the appendix it derives.

      (4) Line 505 - "dominants" should be dominates.

      Typo fixed as indicated

      (5) Figures 10-12: add legends to the figures.

      Now so included.

      (6) Lines 701-703: please rewrite the equation separately. It is highly unclear what rt is here.

      We thank the reviewer for bringing attention to this error. The error arose in converting from Google Sheets to Microsoft Word.

      The equation has now been corrected.

      Additional citations noted in reply and appearing in Main text

      Ainslie, George. 1975. “Specious Reward: A Behavioral Theory of Impulsiveness and Impulse Control.” Psychological Bulletin 59: 257–72.

      Frederick, Shane, George Loewenstein, Ted O. Donoghue, and T. E. D. O. Donoghue. 2002. “Time Discounting and Time Preference : A Critical Review.” Journal of Economic Literature 40: 351–401.

      Gibbon, John. 1977. “Scalar Expectancy Theory and Weber’s Law in Animal Timing.” Psychological Review 84: 279–325.

      Green, Leonard, Nathanael Fristoe, and Joel Myerson. 1994. “Temporal Discounting and Preference Reversals in Choice between Delayed Outcomes.” Psychonomic Bulletin & Review 1: 383–89.

      Grüne-Yanoff, Till. 2015. “Models of Temporal Discounting 1937-2000: An Interdisciplinary Exchange between Economics and Psychology.” Science in Context 28 (4): 675–713.

      Jimura, Koji, Joel Myerson, Joseph Hilgard, Todd S. Braver, and Leonard Green. 2009. “Are People Really More Patient than Other Animals? Evidence from Human Discounting of Real Liquid Rewards.” Psychonomic Bulletin & Review 16: 1071–75.

      Kalenscher, Tobias, and Cyriel M. A. Pennartz. 2008. “Is a Bird in the Hand Worth Two in the Future? The Neuroeconomics of Intertemporal Decision-Making.” Progress in Neurobiology 84 (3): 284–315.

      Kirby, Kris N., and R. J. Herrnstein. 1995. “Preference Reversals Due to Myopic Discounting of Delayed Reward.” Psychological Science 6 (2): 83–89.

      Mazur, James E. 1987. “An Adjusting Procedure for Studying Delayed Reinforcement.” In The Effect of Delay and of Intervening Events on Reinforcement Value., 55–73. Quantitative Analyses of Behavior, Vol. 5. Hillsdale, NJ, US: Lawrence Erlbaum Associates, Inc.

      McNamara, John. 1982. “Optimal Patch Use in a Stochastic Environment.” Theoretical Population Biology 21 (2): 269–88.

      Rosati, Alexandra G., Jeffrey R. Stevens, Brian Hare, and Marc D. Hauser. 2007. “The Evolutionary Origins of Human Patience: Temporal Preferences in Chimpanzees, Bonobos, and Human Adults.” Current Biology: CB 17: 1663–68.

      Strotz, R. H. 1956. “Myopia and Inconsistency in Dynamic Utility Maximization.” The Review of Economic Studies 23: 165–80.

    1. Author Response

      The following is the authors’ response to the original reviews.

      eLife assessment

      This important study convincingly shows that the less common D-serine stereoisomer is transported in the kidney by the neutral amino acid transporter ASCT2 and that it is a noncanonical substrate for sodium-coupled monocarboxylate transporter SMCTs. With a multihierarchical approach, this important study further shows that Ischemia-Reperfusion Injury in the kidney causes a specific increment in renal reabsorption carried out, in part, by ASCT2.

      Public Reviews:

      Reviewer #1 (Public Review):

      Most amino acids are stereoisomers in the L-enantiomer, but natural D-serine has also been detected in mammals and its levels shown to be connected to a number of different pathologies. Here, the authors convincingly show that D-serine is transported in the kidney by the neutral amino acid transporter ASCT2 and as a non-canonical substrate for the sodium-coupled monocarboxylate transporter SMCTs. Although both transport D-serine, this important study further shows in a mouse model for acute kidney injury that ASCT2 has the dominant role.

      Strengths:

      The paper combines proteomics, animal models, ex vivo transport analyses, and in vitro transport assays using purified components. The exhaustive methods employed provide compelling evidence that both transporters can translocate D-serine in the kidney.

      Weakness:

      In the model for acute kidney injury, the SMCTs proteins were not showing a significant change in expression levels and were rather analysed based on other, circumstantial evidence. Although its clear SMCTs can transport D-serine its physiological role is less obvious compared to ASCT2.

      We greatly value the reviewer's efforts and feedback in reviewing our manuscript. We acknowledge the reviewer's observation that the changes indicated by our proteomic results are not markedly pronounced. To reinforce our findings, we have incorporated an analysis of gene alterations at the single-cell level (snRNA-seq) from the publicly accessible IRI mouse model data (Figure supplement 7). The snRNA-seq data align with our proteomic data in terms of the general trend of gene/protein alterations, but reveal more substantial changes in both ASCT2 and SMCTs. These discrepancies might stem from the different quantification methods used, suggesting a possible underestimation in our label-free proteomic quantification. The differences we see between the functional changes in transporters and their quantification in proteomics can be explained by the unique challenges posed by membrane proteins. Post-translational modifications and the complex nature of multiple transmembrane domains often impact the accurate measurement of these proteins in proteomic studies. This complexity can lead to a mismatch between the actual functional changes occurring in the transporters and their perceived abundance or alterations as detected by proteomic methods (Figure 4A) (Schey KL et al. Biochemistry 2015, doi: 10.1021/bi301604j). However, this label-free quantitative proteomics approach is well-suited for our study, given its screening efficiency, compatibility with animal models, and the absence of a labeling requirement. We may consider incorporating alternative quantitative proteomic methods in future for a more thorough comparison. We have included these considerations in lines 351-356 of the revised manuscript.

      Manuscript lines 351-356

      “When evaluating the extent of gene/protein alterations between the control and IRI conditions, we observed that the gene alterations of both Asct2 and Smcts, as revealed by snRNAsequencing, are more pronounced than the protein alteration ratios obtained from proteomics. This discrepancy may stem from difficulty in the quantification method, especially for membrane transport proteins in label-free quantitative proteomics.”

      Regarding the roles of ASCT2 and SMCTs in renal D-serine transport, snRNA-seq showed that ASCT2 expression in the controls is less than 10% of the cell population. We suggest that ASCT2 contributes to D-serine reabsorption because of its high affinity and SMCTs (SMCT1 and SMCT2) would play a role in D-serine reabsorption in the cells without ASCT2 expression. In addition, we included other factors (the turnover rate and the presence of local canonical substrates) that may determine the capability of D-serine reabsorption. We have included this suggestion in the Discussion lines 386-404.

      Manuscript lines 386-404

      “Kinetics analysis of D-serine transport revealed the high affinity by ASCT2 (Km 167 µM) (Foster et al., 2016) and low affinity by SMCT1 (Km 3.39 mM; Figure 5E). In addition to transport affinity, the expression levels and co-localization of multiple transporters within the same cells are critical for elucidating the physiological roles of transporters or transport systems (Sakaguchi et al., 2024). In our proteome data, the chromatogram intensities of Smct1 (2.9 x 109 AU) and Smct2 (1.6 x 108 AU) were significantly higher than that of Asct2 (1.5 x 107 AU) in control mice (Table 1: abundance in Sham). While direct intensity comparisons between different proteins in mass spectrometry analyses are not precise, they can provide a general indication of relative protein amounts. This finding aligns with the snRNA-seq data, where Asct2 expression was found to be minimal, present in less than 10% of cell populations under both control and IRI conditions, suggesting that many cells do not express Asct2. Conversely, Smct1 and Smct2 show high and ubiquitous expression in control conditions, but their levels are markedly reduced in IRI conditions (Figure supplement 7). Our ex vivo assays demonstrate that both ASCT2 and SMCTs mediate D-serine transport (Figure 7B). Consequently, Asct2 may contribute to D-serine reabsorption due to its high affinity, whereas Smcts, owing to their abundance, particularly in cells lacking Asct2, likely play a significant role in D-serine reabsorption. Moreover, factors such as transport turnover rate (Kcat) and the presence of local canonical substrates are also vital in defining the overall contribution of Dserine transport systems.”

      Reviewer #2 (Public Review):

      Summary:

      The manuscript "A multi-hierarchical approach reveals D-1 serine as a hidden substrate of sodium-coupled monocarboxylate transporters" by Wiriyasermkul et al. is a resubmission of a manuscript, which focused first on the proteomic analysis of apical membrane isolated from mouse kidney with early Ischemia-Reperfusion Injury (IRI), a well-known acute kidney injury (AKI) model. In the second part, the transport of D-serine by Asct2, Smct1, and Smct2 has been characterized in detail in different model systems, such as transfected cells and proteoliposomes.

      Strengths:

      A major problem with the first submission was the explanation of the link between the two parts of the manuscript: it was not very clear why the focus on Asct2, Smct1, and Smct2 was a consequence of the proteomic analysis. In the present version of the manuscript, the authors have focused on the expression of membrane transporters in the proteome analysis, thus making the reason for studying Asct2, Smct1, and Smct2 transporters more clear. In addition, the authors used 2D-HPLC to measure plasma and urinary enantiomers of 20 amino acids in plasma and urine samples from sham and Ischemia-Reperfusion Injury (IRI) mice. The results of this analysis demonstrated the value of D-serine as a potential marker of renal injury. These changes have greatly improved the manuscript and made it more convincing.

      We deeply appreciate the reviewer’s comments on the manuscript. We have responded to the recommendations one by one in the later section.

      Reviewer #3 (Public Review):

      Summary:

      The main objective of this work has been to delve into the mechanisms underlying the increment of D-serine in serum, as a marker of renal injury.

      Strengths:

      With a multi-hierarchical approach, the work shows that Ischemia-Reperfusion Injury in the kidney causes a specific increment in renal reabsorption of D-serine that, at least in part, is due to the increased expression of the apical transporter ASCT2. In this way, the authors revealed that SMCT1 also transports D-serine.

      The experimental approach and the identification of D-serine as a new substrate for SMCT1 merit publication in Elife.

      The manuscript also supports that increased expression of ASCT2, even together with the parallel decreased expression of SMCT1, in renal proximal tubules underlies the increased reabsorption of D-serine responsible for the increment of this enantiomer in serum in a murine model of Ischemia-Reperfusion Injury.

      Weaknesses:

      Remains to be clarified whether ASCT2 has substantial stereospecificity in favor of D- versus L-serine to sustain a ~10-fold decrease in the ratio D-serine/L-serine in the urine of mice under Ischemia-Reperfusion Injury (IRI).

      It is not clear how the increment in the expression of ASCT2, in parallel with the decreased expression of SMCT1, results in increased renal reabsorption of D-serine in IRI.

      We thoughtfully appreciate the reviewer’s comment on the manuscript. Considering the alteration of D-/L-serine ratios, there are several factors including protein expression levels at both apical and basolateral sides, properties of the transporters (e.g. transport affinities, substrate stereoselectivities), and the expression of DAAO (D-amino acid oxidase) which selectively degrades D-amino acids. Moreover, the mechanism becomes more complicated when the transport systems of L- and D-enantiomers are different and have distinct stereoselectivities as in the case of serine. Future studies are required to complete the mechanism. However, we would like to explore the mechanism based on the current knowledge.

      From this study, we identified ASCT2 and SMCTs (SMCT1 and SMCT2) as D-serine transport systems. We showed that SMCT1 prefers D-serine. Although we did not analyze ASCT2 stereoselectivity, based on the previous studies, ASCT2 recognizes both D- and Lserine with high affinities and slightly prefers L-enantiomer (Km of 18.4 µM for L-serine in oocyte expression system (Utsunomiya-Tate et al. J Biol Chem 1996) and 167 µM for Dserine in oocyte expression system (Foster et al. Plos ONE 2016), and the IC50 of 0.7 mM for L-serine and 4.9 mM for D-serine (in HEK293 expression systems, Foster et al. PLOS ONE 2016). The proteomics showed an increase of ASCT2 (1.6-fold increase) and a decrease of SMCTs (1.7-fold decrease in SMCT1, and 1.3-fold decrease in SMCT2) in IRI conditions. The table below summarizes D-serine transport by ASCT2 and SMCTs.

      In the case of L-serine, ASCT2 and B0ATs (in particular B0AT3) have been revealed as L-serine transport systems in the kidneys (Bröer et al. Physiol Rev 2008; Singer et al. J Biol Chem 2009). Proteomics showed that B0ATs have higher expression levels than ASCT2 supporting the idea that B0ATs are the main L-serine transport system (Table S1: Abundance of B0AT1 = 1.34E+09, B0AT3 = 2.13E+08, ASCT2 = 1.46E+07). In IRI conditions, B0AT3 decreased 1.8 fold and B0AT1 decreased 1.1 fold. From these results, we included the contribution of B0ATs in L-serine transport in Author response table 1.

      Author response table 1.

      Taken together, we suggest that high ratios of D-/L-serine in IRI conditions are a combinational result of 1) increase of D-serine reabsorption by ASCT2 enhancement and SMCTs reduction and 2) decrease of L-serine reabsorption by B0ATs. We have included this suggestion in the Discussion lines 438-451.

      Manuscript lines 438-451

      “The enantiomeric profiles of serine revealed distinct plasma D/L-serine ratio, with low rations in the normal control but elevated ratios in IRI, despite the weak stereoselectivity of ASCT2 (Figure 1B). This observation suggested differential renal handling of D-serine compared to L-serine. While we identified SMCTs as a D-serine transport system, it has been reported that L-serine reabsorption is mediated by B0AT3 (Singer et al., 2009). We propose that the alterations in plasma and urinary D/L-serine ratios are the combined outcomes of: 1) transport systems for L-serine, and 2) transport systems for D-serine. In normal kidneys, the low plasma D/L-serine ratios could result from the efficient reabsorption of L-serine by B0AT3, coupled with the DAAO activity that degrades intracellular D-serine reabsorbed by SMCTs. In IRI conditions, our enantiomeric amino acid profiling revealed low plasma L-serine and high urinary L-serine (Figure supplements 1B, 2B). Additionally, the proteomic analysis indicated a reduction in B0AT3 levels (4h IRI/sham = 0.56 fold; 8h IRI/sham = 0.65 fold; Table S1). These observations suggest that the low L-serine reabsorption in IRI is a result of B0AT3 reduction.”

      Recommendations for the authors:

      Reviewer #1 (Recommendations For The Authors):

      This is a thorough study that was reviewed previously under the old system. I think the authors have strengthened their findings and have no further suggestions.

      We appreciate reviewer 1 for his/her effort and comments, which greatly contributed to improving this manuscript.

      Reviewer #2 (Recommendations For The Authors):

      The experiments seem to me to have been well performed and the data are readily available.

      Weaknesses:

      More than weakness I would speak of discussion points: I have a few suggestions that may help to make the paper more accessible to a general audience.

      (1) In the Introduction, when the authors introduce the term "micromolecules", it would be beneficial to provide a precise definition or clarification of what they mean by this term. Adding a brief explanation may help the reader to better understand the context.

      Following the reviewer’s comment, we have included the explanation of the micromolecule and membrane transport proteins in lines 41-43.

      Manuscript lines 41-43

      “Membrane transport proteins function to transport micromolecules such as nutrients, ions, and metabolites across membranes, thereby playing a pivotal role in the regulation of micromolecular homeostasis.”

      (2) In line 91, I suggest specifying that this is a renal IRI model.

      Following the reviewer’s comment, we have added the information that it is a renal IRI model of AKI (lines 90-92).

      Manuscript lines 90-92

      “We applied 2D-HPLC to quantify the plasma and urinary enantiomers of 20 amino acids of renal ischemia-reperfusion injury (IRI) mice, a model of AKI and AKI-to-CKD transition (Sasabe et al., 2014; Fu et al., 2018).”

      (3) Lines 167-168 state that Asct2 is localised to the apical side of the renal proximal tubules. Is there any expression of Asct2 in other nephron segments?

      To our knowledge, there is no report of ASCT2 expression in other nephron segments. Our immunofluorescent data of the ASCT2 staining in the whole kidney at the low magnification and another region of Figure 3 (below) as well as immunohistochemistry from Human Protein Atlas (update: Jun 9th, 2023) did not show a strong signal of ASCT2 expression in other regions besides the proximal tubules. Thus, we conclude that ASCT2 is mainly expressed in proximal tubules, but not in other nephron regions.

      Author response image 1.

      (4) Lines 225-226: Have the authors expressed the candidate genes in HEK293 cells with ASCT2 knockdown?

      This experiment was done by expressing the candidate genes in the presence of endogenous ASCT2. We have added the information in lines 225-227 to emphasize this process.

      Manuscript lines 225-227

      “Based on this finding, we utilized cell growth determination assay as the screening method even in the presence of endogenous ASCT2 expression. HEK293 cells were transfected with human candidate genes without ASCT2 knockdown.”

      (5) Lines 254-255: why was D-serine transport enhanced by ASCT2 knockdown in FlpInTRSMCT1 or 2 cells?

      We appreciate the reviewer to point out this data. We apologize for causing the confusion in the text. The total amount of D-serine uptake in the cells did not enhance but the net uptake (uptake subtracted from the background) was increased. This enhancement is a result of the lower background by ASCT2 knockdown. We have revised the texts and explained this result in more detail (lines 256-258).

      Manuscript lines 256-258

      “In the cells with ASCT2 knockdown, the background level was lower, thereby enhancing the D-[3H]serine transport contributed by both SMCT1 and SMCT2 (the net uptake after subtracted with background) (Figure 5C).”

      (6) Line 265: The low affinity of SMCT1 for D-serine alone makes it an unlikely transporter for urinary D-serine.

      We admitted the reviewer’s concern about the low affinity of SMCT1. However, Km at mM range is widely accepted for several low-affinity amino acid transporters such as proton-coupled amino acid transporter PAT1 (Km = 2 – 5 mM; Miyauchi et al. Biochem J 2010), cationic amino acid transporter CAT2A (Km = 3 – 4 mM; Closs et al. Biochem 1997), and large-neutral amino acid transporter LAT4 (Km = 17 mM; Bodoy et al. J Biol Chem 2005). In the kidneys, many compounds are well-known to be reabsorbed by the low-affinity but high-capacity (high-expression) transporters. Similarly, D-serine was reported to be reabsorbed by the low-affinity transporter (Kragh-Hansen and Sheikh, J Physiol 1984; Shimomura et al. BBA 1988; Silbernagl et al. Am J Physiol Renal Physiol 1999). Moreover, amino acid profile showed urinary D-serine in the range of 100 – 200 µM (Figure supplement 2). This concentration range could drive SMCT1 function (Figure 5). Combined with the high and ubiquitous expression of SMCT1, we propose that SMCT1 is a low-affinity but highcapacity D-serine transporter in the kidneys.

      snRNA-seq is a method that can directly compare the expression levels between different genes within the same cells. From Figure supplement 7, expression of SMCT1 is much more abundant than ASCT2. ASCT2 was presented in less than 10% of cell population. It is possible that 90% of the cells that do not express ASCT2 use SMCT1 to reabsorb Dserine.

      We have revised the Discussion regarding this comment (lines 386-404).

      Manuscript lines 386-404

      “Kinetics analysis of D-serine transport revealed the high affinity by ASCT2 (Km 167 µM) (Foster et al., 2016) and low affinity by SMCT1 (Km 3.39 mM; Figure 5E). In addition to transport affinity, the expression levels and co-localization of multiple transporters within the same cells are critical for elucidating the physiological roles of transporters or transport systems (Sakaguchi et al., 2024). In our proteome data, the chromatogram intensities of Smct1 (2.9 x 109 AU) and Smct2 (1.6 x 108 AU) were significantly higher than that of Asct2 (1.5 x 107 AU) in the control mice (Table 1: abundance in Sham). While direct intensity comparisons between different proteins in mass spectrometry analyses are not precise, they can provide a general indication of relative protein amounts. This finding aligns with the snRNA-seq data, where Asct2 expression was found to be minimal, present in less than 10% of cell populations under both control and IRI conditions, suggesting that many cells do not express Asct2. Conversely, Smct1 and Smct2 show high and ubiquitous expression in control conditions, but their levels are markedly reduced in IRI conditions (Figure supplement 7). Our ex vivo assays demonstrate that both ASCT2 and SMCTs mediate D-serine transport (Figure 7B). Consequently, Asct2 may contribute to D-serine reabsorption due to its high affinity, whereas Smcts, owing to their abundance, particularly in cells lacking Asct2, likely play a significant role in D-serine reabsorption. Moreover, factors such as transport turnover rate (Kcat) and the presence of local canonical substrates are also vital in defining the overall contribution of Dserine transport systems.”

      (7) Line 316: The authors state that there is a high tubular D-serine reabsorption in IRI and in line 424 there is an inactivation of DAAO during the pathology. This suggests that there is a reabsorption of D-serine mediated by a transport system in the basolateral membrane domain of proximal tubular cells. Do the authors have any information about this transporter?

      We agree with the reviewer that transporters at the basolateral membrane are important to complete the D-serine reabsorption in the kidney, and have included this issue in the original manuscript. We stated that transport systems at the basolateral side are necessary to be analyzed in order to complete the picture of D-serine transport systems in the kidney (lines 481-483 of the revised manuscript). However, we did not have any strong candidates for basolateral D-serine transport systems. Because we analyzed the proteome of BBMV, which concentrates on the apical membrane proteins, the analysis did not detect several transporters at the basolateral side.

      (8) In lines 462-463, the authors state: "It is suggested that PAT1 is less active at the apical membrane where the luminal pH is neutral". However, the pH of urine in the proximal tubules is normally acidic due to the high activity of NH3. I suggest rewording this sentence.

      Thank you for your comment. Proximal tubule (PT) is the first and the main region to maintain acid-base homeostasis in the kidney. In PT cells, NH3 secretes H+ to titrate luminal HCO3- and creates CO2, which is absorbed into PT cells and produces "new intracellular HCO3-", which is subsequently reabsorbed into the blood. Although ion fluxes in PT is to maintain the pH homeostasis, the pH regulation in both luminal and intracellular PT cells is highly dynamic. We totally agree with the reviewer and to follow that, we have revised the text by emphasizing the pH around PT segments, rather than the final urine pH, and leaving the discussion open for the possibility of PAT1 function in PT of normal kidneys (lines 474481).

      Manuscript lines 474-481

      “PAT1, a low-affinity proton-coupled amino acid transporter (Km in mM range), has been found at both sub-apical membranes of the S1 segment and inside of the epithelia (The Human Protein Atlas: https://www.proteinatlas.org; updated on Dec 7th, 2022) (Sagné et al., 2001; Vanslambrouck et al., 2010). PAT1 exhibits optimum function at pH 5 - 6 but very low activity at pH 7 (Miyauchi et al., 2005; Bröer, 2008b). Future research is required to address the significance of PAT1 on D-serine transport in the proximal tubule segments where pH regulation is known to be highly dynamic (Boron, 2006; Nakanishi et al., 2012; Bouchard and Mehta, 2022; Imenez Silva and Mohebbi, 2022).”

      Reviewer #3 (Recommendations For The Authors):

      The authors proposed that the increased expression of ASCT2, even together with the decreased expression of SMCT1/2, causes the increased renal reabsorption of D-serine that occurs in IRI. In the discussion, the main argument to sustain this hypothesis is the higher apparent affinity for D-serine of ASCT2 (<200 uM Km) versus SMCT1 (3.4 mM Km). In the Discussion section (page 18- 1st complete paragraph), the authors indicate that the Mass Spec intensities of SMCT1 and 2 are two and one order of magnitude higher respectively than that of ASCT2. This suggests that SMCT1 is clearly more expressed than ASCT2 in control conditions. IRI increments ASCT2 protein expression in brush-border membrane vesicle from kidney 1.6 folds and decreases that of SMCT1 0.6 folds. How this fold changes, even taking into account the lower Km of ASCT2 versus SMCT1 would explain the dramatic changes in the D-/L-serine ratios in plasma and urine in IRI? The authors might discuss whether other transport characteristics, even unknown (e.g., a higher turnover rate of ASCT2 vs SMCT1), would also contribute to the higher D-serine reabsorption in IRI.

      SMCT1 shows some enantiomer selectivity for D- vs L-serine. At 50 uM concentration the transport is almost double for D. vs L-serine, but is ASCT2 stereoselective between the two enantiomers of serine? Some of the authors of this manuscript showed in a previous paper that the basolateral transporter Asc1 also participates in the accumulation of D-serine in serum caused by renal tubular damage. (Serum D-serine accumulation after proximal renal tubular damage involves neutral amino acid transporter Asc-1. Suzuki M et al. Sci Rep. 2019 Nov 13;9(1):16705 (PMID: 31723194)). Asc1 shows no stereoselectivity between L- and D-serine. Can the authors discuss possible mechanisms resulting in increased renal reabsorption of Dserine than L-serine in IRI with the participation of transporters with modest stereoselectivity for D- vs L-serine?

      We appreciate the reviewer’s comments on the degree of protein alteration in proteomics, the functional contributions of ASCT2 and SMCTs, and the alteration of D/L ratios. We have included the possibilities of the technical concerns and the discussion on the roles of ASCT2 and SMCTs as follows.

      • Regarding the expression levels, proteomics and snRNA-seq showed the same tendency that ASCT2 increase and SMCTs decrease in IRI conditions. However, the degrees of alterations are more contrast in snRNA-seq. This may be due to the difference in quantification methods and probably points out the underestimated quantification of membrane transport proteins in label-free proteomics. The accuracy of protein quantifications in the label-free proteomics are often impacted by the presence of post-translational modifications and multiple trans-membrane domains like in the case of the membrane transport proteins (Schey KL et al. Biochemistry 2015, doi: 10.1021/bi301604j). Alternative methods of quantitative proteomics may be added in the future for a more thorough comparison. We have added this issue in lines 351-356 of the revised version.

      Manuscript lines 351-356

      “When evaluating the extent of gene/protein alterations between the control and IRI conditions, we observed that the gene alterations of both Asct2 and Smcts, as revealed by snRNA-sequencing, are more pronounced than the protein alteration ratios obtained from proteomics. This discrepancy may stem from difficulty in the quantification method, especially for membrane transport proteins in label-free quantitative proteomics.”

      • For the functional contributions of ASCT2 and SMCTs in the kidney, we admitted the reviewer’s concern about the low affinity of SMCT1. Following the reviewer’s comment, we have included other factors besides transport affinities, e.g. expression levels and turnover rates of the transporters. From the results of both proteomics and snRNA-seq, ASCT2 expression is significantly lower than SMCTs in the normal conditions. snRNA-seq showed that ASCT2 was presented in less than 10% of the cell population (Figure supplement 7). We propose that most of the cells that do not express ASCT2 may use SMCT1 to reabsorb D-serine. This topic was included in the revised manuscript lines 386-404.

      Manuscript lines 386-404

      “Kinetics analysis of D-serine transport revealed the high affinity by ASCT2 (Km 167 µM) (Foster et al., 2016) and low affinity by SMCT1 (Km 3.39 mM; Figure 5E). In addition to transport affinity, the expression levels and co-localization of multiple transporters within the same cells are critical for elucidating the physiological roles of transporters or transport systems (Sakaguchi et al., 2024). In our proteome data, the chromatogram intensities of Smct1 (2.9 x 109 AU) and Smct2 (1.6 x 108 AU) were significantly higher than that of Asct2 (1.5 x 107 AU) in the control mice (Table 1: abundance in Sham). While direct intensity comparisons between different proteins in mass spectrometry analyses are not precise, they can provide a general indication of relative protein amounts. This finding aligns with the snRNA-seq data, where Asct2 expression was found to be minimal, present in less than 10% of cell populations under both control and IRI conditions, suggesting that many cells do not express Asct2. Conversely, Smct1 and Smct2 show high and ubiquitous expression in control conditions, but their levels are markedly reduced in IRI conditions (Figure supplement 7). Our ex vivo assays demonstrate that both ASCT2 and SMCTs mediate D-serine transport (Figure 7B). Consequently, Asct2 may contribute to D-serine reabsorption due to its high affinity, whereas Smcts, owing to their abundance, particularly in cells lacking Asct2, likely play a significant role in D-serine reabsorption. Moreover, factors such as transport turnover rate (Kcat) and the presence of local canonical substrates are also vital in defining the overall contribution of D-serine transport systems.”

      • As for the dramatic alterations of D/L-serine ratios juxtaposed with minimal changes in ASCT2 and SMCTs expression level, we cautiously refrain from drawing a definitive conclusion regarding the entire mechanism. This caution is grounded in the scientific understanding of a comprehensive elucidation of both L-serine transport systems and D-serine transport systems at both apical and basolateral membranes. Nevertheless, we would like to suggest a mechanism at the apical membrane based on the current knowledge.

      For D-serine transport systems, we found ASCT2 and SMCTs contributions in this study. Meanwhile, L-serine was previously reported to be mediated mainly by the neutral amino acid transporters B0AT3 (in particular B0AT3; Bröer et al. Physiol Rev 2008; Singer et al. J Biol Chem 2009). Hence, the mechanism behind the alterations of D/L-serine ratios should include B0AT3 functions as well. In IRI conditions, B0AT3 decreased 1.8 fold. We suggest that high ratios of D-/L-serine in IRI conditions are a combined outcome of 1) increase of D-serine reabsorption by ASCT2 enhancement and SMCTs reduction, and 2) decrease of L-serine reabsorption by B0AT3. We have included this suggestion in the Discussion lines 438-451.

      Manuscript lines 438-451

      “The enantiomeric profiles of serine revealed distinct plasma D/L-serine ratios, with low ratios in the normal control but elevated ratios in IRI, despite the weak stereoselectivity of ASCT2 (Figure 1B). This observation suggested the differential renal handling of D-serine compared to L-serine. While we identified SMCTs as a Dserine transport system, it has been reported that L-serine reabsorption is mediated by B0AT3 (Singer et al., 2009). We propose that the alterations in plasma and urinary D/Lserine ratios are the combined outcomes of: 1) transport systems for L-serine, and 2) transport systems for D-serine. In normal kidneys, the low plasma D/L-serine ratios could result from the efficient reabsorption of L-serine by B0AT3, coupled with the DAAO activity that degrades intracellular D-serine reabsorbed by SMCTs. In IRI conditions, our enantiomeric amino acid profiling revealed low plasma L-serine and high urinary L-serine (Figure supplements 1B, 2B). Additionally, the proteomics analysis indicated a reduction in B0AT3 levels (4h IRI/sham = 0.56 fold; 8h IRI/sham = 0.65 fold; Table S1). These observations suggest that the low L-serine reabsorption in IRI is a result of B0AT3 reduction.”

      • In the case of Asc-1, it was reported to be a D-serine transporter in the brain (Rosenberg et al. J Neurosci 2013). Suzuki et al. 2019 showed the increase of Asc-1 in cisplatin-induced tubular injury. Notably, the mRNA of Asc-1 is predominantly found in Henle’s loop, distal tubules, and collecting ducts but not in proximal tubules, and its protein expression level is dramatically low in the kidney (Human Protein Atlas: update on Jun 19, 2023). Furthermore, in this study, Asc-1 expression was not detected in the brush border membrane proteome. Consequently, we have decided not to include Asc-1 in the Discussion of this study, which primarily focuses on the proximal tubules.
    1. Author Response

      The following is the authors’ response to the original reviews.

      necessary clarifications on some of the reviewers' suggestions.

      Reviewer #1 (Public Review):

      Weaknesses:

      • This is a pilot study with only 24 cases and 24 controls. Because the human microbiota entails individual variability, this work should be confirmed with a higher sample size to achieve enough statistical power.

      Thank you for your suggestion. Unlike the high sparsity of 16s rRNA, the data density of metagenomic data is higher. Based on the experience of previous research, the sample size used this time can basically meet the requirements. However, your suggestion is very valuable, increasing the sample size allows better in-depth analysis. Due to limitations of objective factors, it is difficult for us to continue to increase the sample size in this study.

      • The authors do not report here the use of blank controls. The use of this type of control is important to "subtract" the potential background from plasticware, buffer or reagents from the real signal. Lack of controls may lead to microbiome artefacts in the results. This can be seen in the results presented where the authors report some bacterial contaminants (Agrobacterium tumefaciensis, Aequorivita lutea, Chitinophagaceae, Marinobacter vinifirmus, etc) as part of the most common bacteria found in cervical samples.

      Thank you for your suggestion. Applying blank controls in low biomass areas can effectively avoid contamination caused by the environment or kits. This opinion is consistent with that published by Raphael Eisenhofer et al. in Trends in Microbiology. When designing this study, we considered that this study described a biomass-rich site, and the abundance of dominant species was much higher than that of the possible 'kitome', so we did not set a blank control. On the other hand, our main discussion object in this study is high-abundance species, and the species filtering threshold for some analyzes was raised to 50%. Therefore, we believe that the absence of the blank control has little effect on the conclusions of this study. However, your opinion is spot on. Failure to set up a negative control will affect our future research on rare species. We will add a description in the Limitations section of the Discussion section.

      • Samples used for this study were collected from the cervix. Why not collect samples from the uterine cavity and isthmocele fluid (for cases)? In their previous paper using samples from the same research protocol ((IRB no. 2019ZSLYEC-005S) they used endometrial tissue from the patients, so access to the uterine cavity was guaranteed.

      Thank you for your suggestion. In Author response image 1 we show the approximate location of our cervical swab sampling. There are two main reasons for choosing cervical swabs:

      1) The adsorption of swabs allows us to obtain sufficient nucleic acid for high-depth sequencing, while the isthmocele fluid varies greatly among patients, which will introduce unnecessary batch effects.

      2) Since the female reproductive tract is a continuous whole, our sampling location is close to the lesion in the cervix, which can be effectively studied. On the other hand, the microbial biomass of the endometrium is probably two orders of magnitude lower than that of the cervix, and it is difficult to avoid contamination of the lower genital tract when sampling.

      Based on the above reasons, we selected cervical swabs for our microbial data.

      Author response image 1.

      • Through the use of shotgun genomics, results from all the genomes of the organisms present in the sample are obtained. However, the authors have only used the metagenomic data to infer the taxonomical annotation of fungi and bacteria.

      Thank you for your suggestion. The advantage of metagenomics is that it can obtain all the nucleic acid information of the entire environment. However, in the study of the female reproductive tract, the database of viruses and archaea is still immature, in order to ensure the accuracy of the results, we did not conduct the study. Looking forward to the emergence of a mature database in the future.

      Reviewer #1 (Recommendations For The Authors):

      • It would be interesting to use another series of functional data coming from the metagenomic analyses (not only taxonomic) to expand and reinforce the results presented.

      Thank you for your suggestion. We have dissected the functional data of microbiota in the article.

      • The authors have previously published the 16S rRNA sequencing and transcriptomic analysis of the same set of patients. It would be nice to see the integration of all the datasets produced.

      Thank you for your suggestion. There is no doubt that integrating all the data will have more dimensional results. In our previous study we focused on microbe-host interactions. However, there is an unanswered question: What are the characteristics of the regulatory network within microbiota? Therefore, we answered this question in this study, exploring the complex interaction processes within microbial communities. In addition to direct effects, interactions between microbiota may also occur through special metabolite experiments. Therefore, we introduced the analysis of the untargeted metabolome. However, 16s rRNA can only provide bacterial information, so we did not integrate the data. In addition, the transcriptome provides host information and is not the focus of this study. However, your suggestion is very valuable, and we will integrate all the data in the next study on the exploration of treatment methods.

      Reviewer #2 (Public Review):

      Weaknesses: Methodological descriptions are minimal.

      Some example:

      *The CON group (line 147) has not been defined. I supposed it is the control group.

      • There are no statistics related to shotgun sequencing. How many reads have been sequenced? How many have been removed from the host? How many are left to study bacteria and fungi? Are these reads proportional among the 48 samples? If not, what method has been used to normalise the data?

      • ggClusterNet has numerous algorithms to better display the modules of the microbiome network. Which one has been used?

      Thank you for your suggestion. We have added details to the method.

      Reviewer #2 (Recommendations For The Authors):

      I think the author should take into account the points described in the "Weaknesses" section. The lack of detail extends to almost all the analyses that have been included in the manuscript. Although the results are sound, I think it is important to understand what has been analysed and how it has been analysed. It is important that all work is reproducible and this requires vital information.

      For example, what parameters have been used for bowtie2? has a local analysis been used? or end-to-end ? Some parameters like --very-sensitive are important for this kind of analysis. You can also use specific programs like kneaddata.

      The Raw data preprocessing section should be more detailed.

      The same with the "Taxa and functional annotation" section, how have the data been normalised? has any Zero-Inflated Gamma probabilistic model algorithm been taken into account? How were the 0 (no species detected) in the shallow samples treated?

      Which algorithms have been used for LEfSe ? Kluskal-Wallis->(Wilcoxon)->LDA ?

      Which p-value has been used as cut-off ? this p-value has been corrected for multiple testing?

      • Information on ggClusterNet should be included and explained.

      The first section of the results and Table 1 should be in the Materials and Methods.

      Thank you for your suggestion. We have added details to the method.

      In the fungi section, it is mentioned that 431 species have been found. They should be included in a supplementary table.

      How many bacteria were found? Please include them also in a supplementary table.

      Thank you for your suggestion. We have added the corresponding table.

      Reviewer #3 (Public Review):

      Major

      1. Smoke or drink conditions, as well as diseases like hypertension and diabetes are important factors that could influence the metabolism of the host, thus the authors should add them in the exclusion criteria in the Methods.

      Thanks to reviewer #3 for professional comments. We have made corresponding additions in the method section. We also followed this standard when recruiting subjects.

      1. The sample size of this study is not large enough to draw a convincing conclusion.

      Thank you for your suggestion. Unlike the high sparsity of 16s rRNA, the data density of metagenomic data is higher. Based on the experience of previous research, the sample size used this time can basically meet the requirements. However, your suggestion is very valuable, increasing the sample size allows better in-depth analysis. Due to limitations of objective factors, it is difficult for us to continue to increase the sample size in this study.

      Reviewer #3 (Recommendations For The Authors):

      Please recruit more samples.

      In addition, there are many formatting and grammatical mistakes in the manuscript.

      Minor

      1. In Line 24-25 of the "Composition and characteristics of fungal communities", the format of "Goyaglycoside A and Janthitrem E." shouldn't be italic.

      2. In Line 126 of the "Metabolite detection using liquid chromatography (LC) and mass spectrometry (MS)", the "10 ul" should be changed to "Ten ul". Beginning with arabic numerals in a sentence should be avoided.

      3. In Line 170 of the "Composition and characteristics of bacterial communities", the "162 differential species" should be "One hundred and sixty-two differential species".

      4. In Line 187 of the "Composition and characteristics of fungal communities", the "42 differential" should be "Forty-two differential".

      Thanks to reviewer #3 for professional comments. We have completely revised the language of the article.

    1. Author response:

      The following is the authors’ response to the original reviews.

      We would like to thank the reviewer and the editor for carefully reading our manuscript, and acknowledging the strength of combining quantitative analysis with semi-naturalistic experiments on mice social behavior. Please find below our response to both the public review and the recommendation to the authors. As a summary, we have included additional figures and texts such as 

      - a new Results subsection “Choosing timescales for analysis ” (page 6)

      - a new Materials and Methods subsection “Maximum entropy model with triplet interactions” (page 17)

      - new supplementary figures, which have current labels of:

      - Figure 2 - figure supplement 5

      - Figure 2 - figure supplement 6

      - Figure 2 - figure supplement 7

      - Figure 4 - figure supplement 1

      - Figure 4 - figure supplement 2    

      Public Reviews: 

      Reviewer #1 (Public Review): 

      Summary: 

      In this manuscript, Chen et al. investigate the statistical structure of social interactions among mice living together in the ECO-Hab. They use maximum entropy models (MEM) from statistical physics that include individual preferences and pair-wise interactions among mice to describe their collective behavior. They also use this model to track the evolution of these preferences and interactions across time and in one group of mice injected with TIMP-1, an enzyme regulating synaptic plasticity. The main result is that they can explain group behavior (the probability of being together in one compartment) by a MEM that only includes pair-wise interactions. Moreover, the impact of TIMP-1 is to increase the variance of the couplings J_ij, the preference for the compartment containing food, as well as the dissatisfaction triplet index (DTI). 

      Strengths: 

      The ECO-Hab is a really nice system to ask questions about the sociability of mice and to tease apart sociability from individual preference. Moreover, combining the ECO-Hab with the use of MEM is a powerful and elegant approach that can help statistically characterize complex interactions between groups of mice -- an important question that requires fine quantitative analysis. 

      Weaknesses: 

      However, there is a risk in interpreting these models. In my view, several of the comparisons established in the current study would require finer and more in-depth analysis to be able to establish firmer conclusions (see below). Also, the current study, which closely resembles previous work by Shemesh et al., finds a different result but does not provide the same quantitative model comparison included there, nor a conclusive explanation of why their results are different. In total, I felt that some of the results required more solid statistical testing and that some of the conclusions of the paper were not entirely justified. In particular, the results from TIMP-1 require proper interaction tests (group x drug) which I couldn't find. This is particularly important when the control group has a smaller N than the drug groups.  

      We would like to thank the reviewer and the editor for carefully reading our manuscript, and acknowledging the strength of combining quantitative analysis with semi-naturalistic experiments on mice social behavior. Thanks to the reviewer’s suggestion, we have improved our manuscript by 

      (1) A proper comparison with Shemesh et al., especially to include maximum entropy models with triplet interactions. We show that triplet models overfit even given the entire 10 day dataset, which limits our study to look at pairwise interactions.

      (2) Results on cross-validation for both triplet interaction models and pairwise interaction models, completed on aggregates of various length of days. This analysis showed that pairwise models overfit for single-day data, and led us to learn pairwise models only on 5day aggregation of data. We have updated the manuscript (both the text and the figures) to present these results.

      (3) New results that subsample the drug groups to the same size as the control group. The conclusions about TIMP-1 treated mice hordes hold when we compare groups of the same size. 

      Recommendations for the authors:  

      Reviewer #1 (Recommendations For The Authors): 

      (1) COMPARISON WITH PREVIOUS WORK. The comparison with the cited previous work of Shemesh et al. 2013 rests novelty to the use of ME models in characterizing social interactions between groups of mice as well as sheds doubts on the main claim of the manuscript, namely that second-order correlations are sufficient to describe the joint distribution of occupancies of all mice (in particular triplets; there is no quantification of the variance explained by model in panel Fig. 2D). In my view, to make the claim "These results show that pairwise interaction among mice are sufficient to assess the observed collective behavior", the authors should compare models with 2nd and 3rd order interactions and quantify how much of the total correlation can be explained by pair-wise interactions, triplet interactions, and so on. Without a proper model comparison, it is unclear how the authors can make such a claim. One thing observed by Shemesh et al. is that, on average, J_ij are negative. This does not seem to be the case in the current study and the authors should discuss why. 

      Finally, the explanation provided in the Discussion about this discrepancy (spatial resolution and different group size) are not completely satisfactory. With more animals, one would imagine that the impact of higher order correlations would increase (and not decrease) as the number of terms of 3rd, 4th, ... order will be very big. I would also think that the same could be true for the spatial scale: assessing interactions with a coarser spatial grid (whole cages in the case of the ECO-Hab) would allow for simultaneous interactions among more mice to happen compared with a situation in which the spatial grid is so small that only a few animals can fit in each subdivision. 

      We thank the reviewer for the recommendation. In the updated version of the manuscript, we explicitly learn the triplet interaction model. We show that because the number of mice in our experiment is much larger than Shemesh et al., a triplet model runs into the problem of overfitting.

      In particular, we found that the test set likelihood increases monotonically when the L2 regularization strength increases, which corresponds to a suppression of the triplet interaction strength (see additional supplementary figure, now Figure 2 - figure supplement 5). More specifically, for the range of regularization strength (β<sub>G</sub>) we tested (10<sup>-1</sup> < β<sub>G</sub> < 10<sup>1</sup>), the maximum test set likelihood is achieved at β<sub>G</sub> = 10<sup>1</sup>, which corresponds to . Notice that those learned triplet interactions are very close to zero. This means we should select a model with pairwise interactions over a model with triplet interactions.

      We have added the above reasoning in page 5, line 166-169 of the Results section with the sentence “Moreover, models with triplet interactions show signs of overfitting under crossvalidation, which is mitigated when the triplet interactions are suppressed close to zero using L2 regularization”,  a new subsection “Maximum entropy model with triplet interactions” in Materials and Methods (page 16-17, line 548 - 563) to describe the protocols of learning and crossvalidation for these triplet interaction models. 

      Furthermore, we extended the discussion about the difference between Shemesh et al. and our results in the Discussion section. In addition to the difference of spatial scales (chamber vs. location in the chamber), and the difference of group size and its impact on data analysis (N = 15 in our largest cohort and N = 4 in theirs), we added a discussion about the difference of experimental arena, which in Eco-HAB contains connected chambers that mimic the naturalistic environment, and in Shemesh et al. contains a single chamber. The change in the text is on page 12, between line 390 and line 394.

      We thank the reviewers for pointing out that the mean 2nd order interaction in Shemesh et al. is negative. One possibility is that the labeled areas in Shemesh et al. are much smaller than in our Eco-HAB setup, which could suggest that mice do have the space to stay in the same area, which will lead to a negative mean 2nd order interaction.

      (2) ASSESSMENT OF THE TEMPORAL EVOLUTION OF THE INTERACTIONS. The analysis of the stability of the social structure is not conclusive. First, I don't think the authors can conclude that "These results suggest that the structure of social interactions in a cohort as a whole is consistent across all days." If anything is preserved, they would be the statistics of that structure but not the structure itself (i.e., there is no evidence for that). The comparison of the stability of the mean <h\_i> and the mean <J\_ik> would also require a statistical test to be able to state that "Delta h_i changed more strongly from day to day (Fig. 3D, top panel) relative to the interaction measured as the Jij's." The same is true for the assessment of the TIMP: the differences found in the variability in J_ij and in the mean and variance of the h_i's, look noisy and would require a proper statistical test. The traces look quite variable across days in the control condition, so assessing differences may be difficult. Finally, it would be good to know if the variability in individual J_ij is because they truly vary from day to day or because estimating them within one day is difficult (statistical error). If the reason is the latter, one could decrease the temporal resolution to 2-3 days and see whether the estimated J_ijs are more stable. Perhaps, also for that reason, the summed interaction strength J_i is also more stable, simply because it aggregates more data and has a smaller statistical error. 

      We thank the reviewer for pointing out the necessity of assessing the temporal evolution of the interactions. The problem of shorter data duration leads to more noise in the estimation, together with the reviewer’s Comment 4 about the risk of overfitting, led us to add a new Results subsection “Choosing timescales for analysis” (page 6, line 171 to line 189). Specifically, we assess whether the pairwise maximum entropy model overfits using data from _K-_day aggregates, by computing the log-likelihood of both the training sets and the test sets,which is chosen to be 1 hour from the 6 hour data window of each day. We found that for single day data, the pairwise maximum entropy model overfits. In contrast, for data with aggregates of more or equal to 4 days of data, the pairwise model does not overfit. This new result is supported by an additional supplementary figure, now Figure 2 - figure supplement 6.

      To be consistent with later approaches in the manuscript where we consider the effects of TIMP1, we choose the analysis windows to be data aggregates from 5 days. This means for the experiment that collects a total of 10 days of data, there are only two time points, thus a study of the temporal evolution is limited to comparison between the first 5 days and the last 5 days of the experiment. We describe these results in the Results subsection “Stability of sociability over time” (page 6, line 190 - 220). An additional supplementary figure, now Figure 2 - figure supplement 7, shows in details the comparison of the inferred interaction strength J and the chamber preference between the first 5 days and the last 5 days for the 4 cohorts of male C57BL6/J mice, which shows the inferred interactions have a consistent variability across first and last 5 days, and across all cohorts. The small value of Pearsons’ correlation coefficient shows that the exact structure (pairspecific J<sub>ij</sub>) is not stable. At the end of the Results subsection “Stability of sociability over time”, we explicitly say that “This implies that the maximum entropy model does not infer a social structure that is stable over time.”

      (3) EFFECT OF TIMP-1. The reported effects of TIMP-1 on the variance of the J_ij seem very small and possibly caused by a few outlier J_ijs (perhaps from one or two animals) which

      are not present in the control group which seems to have fewer animals (N = 9 minus two mice that died after the surgery vs. N = 14 in the drug group), so the lack of a significant difference in the sigma[J_ij] could simply be due to a smaller N (a test for the interaction group x drug was not done). 

      The clearest effect of TIMP-1 seems to be a change in place preference (h_i) and not the interaction terms (J_ij) (Fig. 3F bottom). But this could be explained by a number of factors that have nothing to do with sociability such as that recovery from surgery makes them eat more/less. The fact that it seems to be present, as recognized by the authors, in the control group with no TIMP-1 and that this effect was not observed in the female group F1, puts into question the specificity and reproducibility of the result. 

      Finally, the effect of TIMP-1 in the DTI would require more statistics (testing the interaction group x drug). The fact that the control group has fewer animals (N = 9 vs. 15 and 13 in the drug groups), and that there is a weaker trend in the DTI of the control group to start high and then decrease, makes this test necessary.  

      Now, after we select a proper timescale to learn the pairwise maximum entropy model, we update the manuscript to present results only on 5-day aggregation of data (see updated Figure 3, updated supplementary figures, Figure 3 - figure supplement 1 and 2). For the variance of the J<sub>ij</sub>, the F-test between different 5-day aggregates before and after TIMP for the male drug group now shows a nonsignificant p-value after applying the Bonferroni correction. For the female drug group, the difference of the J<sub>ij</sub> variance is still significant. 

      To test the effect of different group size on DTI, we subsampled the drug groups by 1) subsampling the inferred interactions learned from the original N = 15 or N = 13 data, or 2) subsampling the mice colocalization data and then inferring the pairwise interactions.  In both cases, the resulting DTI for the subsampled drug group still exhibits the same global pattern as before, i.e. after TIMP-1 injection, DTI significantly increases, which after 5 days falls back to the baseline level. The results are supported by two additional supplementary figures, Figure 4 - figure supplement 1 and 2. This result is referred to in the text in the Results subsection “Impaired neuronal plasticity in the PL affects the structure of social interactions” (page 10, line 333 - 336): “Notably, the difference of the DTI is not due to the control group M4 has less mice, as subsampling both on the level of the inferred interactions (Figure 4 - figure supplement 1) and on the level of the mice locations (Figure 4 - figure supplement 2) give the same DTI for cohorts M1 and F1.”

      (4) MODEL COMPARISON. Any quantitative measure of "goodness" of the model , (i.e., comparison of the predictions of the model with triplet frequency as well as the distribution of p(K)) should be cross-validated. In particular, Fig. S2 needs to be cross-validated for the goodness of fit to be properly quantified. Is the analysis shown in Fig. 3F crossvalidated? Because otherwise, there is an expected increase in the likelihood simply explained by an increase in the number of parameters of the model (i.e., adding the J_ij's). 

      As discussed in our responses to Comment 1 and 2, we have added results about cross-validation in the new supplementary figures, Figure 2 – figure supplement 5 and 6 , for which we computed the test-set and training-set likelihood for maximum entropy models with pairwise interactions and also for models with triplet interactions. Figure 2 - figure supplement 6 shows the pairwise model does not overfit when we consider the aggregated data from more or equal to 4 days. 

      (5) EFFECT OF SLEEP. The comparison of p(K) between the data and the model requires a bit more investigation: the model underestimates instances in which almost all mice were in the same compartment (i.e., for K >= 13. p(K)_data >> p(K)_MEM; btw where is the pairwise point p(15) in Fig. 2E and Fig. S4?). Could this be because there were still short periods during the dark cycle in which all mice were asleep in one of the cages? As explained by the authors, sleep introduces very strong higher order correlations between animals as they like sleeping altogether. Knowing whether removing light periods was enough to remove this "sleep contamination" or not, would be important in order to interpret discrepancies between the pairwise model and the data. 

      Figure 2E shows that the pairwise maximum entropy model (in black) overestimates the data (in blue circles) for P(K) at large K (and not underestimates). In the data, we never observe all 15 mice being in the same box; hence P<sub>data</sub>(15) = 0, and does not show up in the log-scaled figure (same for Figure 2 - figure supplement 3). A possible explanation for the pairwise model overestimating P(K) at large K is that the finite-sized box limits the total number of mice that are comfortably staying in the same box. It can also be due to the fact that the number of time points at which K >= 13 is small and hence causes an underestimation due to finite data. We have added this interpretation of the discrepancy of P(K) to Section “Pairwise interaction model explains the statistics of social behavior” in page 6, line 160. 

      We thank the Reviewer for raising the point of “sleep contamination”. Indeed, Eco-HAB data, as do data from other 24h-testing behavioral systems, demonstrate distinct differences in activity levels during the light and dark phases of the light-dark cycle (Rydzanicz et al., EMBO Mol. Med., 2024). During the light phases, mice primarily sleep and, as noted, they huddle, so many individuals within the cohort tend to remain in close proximity for extended periods. We acknowledge that including such periods in the analysis could potentially introduce confounding effects to the model due to limited movement and interactions, and this is why we decided not to use this data. However, during the dark phases, mice are highly active, with individuals rarely staying in the same compartment for long periods. Specifically, in the dark phases, while there are occasional instances where a few mice may remain in the same compartment for over 1 hour, the majority exhibit considerable mobility, actively exploring and transitioning between compartments. We see no compelling reason to exclude these periods from our analysis, as such activity aligns with the natural behavioral repertoire of the mice and provides robust data for our model. Furthermore, it is well-established that mammals, including nocturnal species such as mice, are most active shortly after waking, typically at the onset of their active phase (i.e., the beginning of the dark phase). To ensure a conservative approach, we specifically analyzed the first 6 hours of the dark phase when the cumulative number of box visits is at its peak, indicating heightened activity levels. In our view, this period offers an optimal window for studying natural behaviors, including social interactions.

      Additionally, prior studies using the Eco-HAB system have consistently demonstrated that mice engage in social interactions both within the compartments and in the connecting tubes during the dark phase (Puścian et al., eLife, 2016, Winiarski et al. in press). Given this evidence and the observed behavioral dynamics in our data, the likelihood of mice being asleep during the analyzed periods of the dark phase is very low.

      We hope this clarification addresses the reviewer’s concerns and highlights the rationale underpinning our analysis choices. Thank you for raising this important point, which allowed us to provide additional context for our approach.

      (6) COMPARTMENT PREFERENCES. The differences between p(K) across compartments also would require a bit more attention: of a MEM with non-spatially dependent pair-wise interactions shows differences across compartments, it must be because of the terms h_{i,r} terms which contain a compartment index, right? Wouldn't this imply that the independence model, which always underrepresents data events with large K, already contains the difference in goodness of fit between compartments (1, 3) and (2, 4)? In the plots, it does not look like the goodness of the independent model depends on the compartment (the authors could compare directly the models' predictions between compartments). Moreover, when looking at Fig. 2C, it does not look like the value of h_{i,r} in compartments (1,3) is higher than in (2,4) (if anything, it would be the other way around). How can this be explained? It would be good to know if the difference across compartments comes from differences in the empirical p(K) or in the models' prediction? If the difference is in the data p(K), could it be that the compartments 2-4 showing higher p(K=15) (i.e., larger difference with the pairwise MEM prediction) are those chosen by mice to sleep during the light cycle? If not, what could explain these differences across compartments? Could the presence of food and water explain this difference? 

      The reviewer is correct, in the pairwise MEM, the difference across compartments enter in the box preference h<sub>ir</sub>. Greater h<sub>ir</sub> means compartment r is more attractive to mouse i. Because box 2 and 4 contain food and water, we expect that mice are more attracted to box 2 and 4, and this is what we see in Figure 2C, bottom subpanels. To reduce the number of parameters to look at, we introduce an index Δh<sub>i</sub> = h<sub>i2</sub> + h<sub>i4</sub> - h<sub>i1</sub> - h<sub>i3</sub>. This index Δh<sub>i</sub> is found to be mostly positive (see updated Figure 3C), which makes sense because mice are attracted to food and water. 

      Next we analyze the difference of P(K) across compartments (Figure 2 - figure supplement 3). There is already a difference in the P(K) calculated from empirical data. For example, P(K) in compartment 2 has a maximum at K = 5 while P(K) in compartment 1 has a maximum at K = 3

      One interesting observation is that it seems from Figure 2 - figure supplement 3 that the pairwise model explains P(K) in compartment 1 and compartment 3 better than in compartment 2 and in compartment 4. In compartment 2 and 4, the pairwise MEM overestimates P(K) for large K. An alternative MEM could include compartment-specific interaction strength, but it will also introduce 315 new parameters for a mice cohort with size N = 15.

      MINOR

      (1) A more quantitative comparison between in-cohort sociability and couplings J_ij as œwell as mean rates and parameters h_i is required. The matrices in Fig. 2C do look similar. So it is not clear how the comparison between these values is contributing to characterizing the correlation structure of the data. 

      The comparison between in-cohort sociability and coupling J<sub>ij</sub> is given by supplementary Figure 2 - figure supplement 2.  The key point for the model with the learned J<sub>ij</sub> reproducing the in-cohort sociability is given by Figure 2 - figure supplement 1.

      (2) Analysis of "in-state" probability is not explained. To me, it wasn't obvious what Fig. S5 is showing. I was assuming that this analysis was comparing the prediction of the MEM about the position of each animal at each time point, given its preference (h), pairwise interactions (J_ij), and the position of all other animals and the true position of the animal. But it seems like it is comparing the shape of the distribution of this prob across time between the data and the model (I guess the data had to be temporally binned in coarser temporal periods to yield prob values other than 0s and 1s). Also, not clear whether this analysis was done for each compartment separately and then averaged. This needs explanation. 

      The in-state probability is comparing the prediction of the MEM about the position of each animal at each time point, given its preference (h), pairwise interactions (J<sub>ij</sub>), and the position of all other animals and the true position of the animal. To achieve values between 0s and 1s, we bin the data temporally according to the model-predicted in-state probability. 

      We have added the explanation of in-state probability on page 6, line 163-166. We have also improved the description of in-state probability in Materials and Methods (subsection “Comparing in-state probability between model prediction and data”, line 493 - 503, page 15), and added a pointer from the main text to it. 

      (3) Looks like Fig. S3 is not cited in the text. 

      We added a pointer to Fig. S3 (now Figure 2 - figure supplement 2) in line 154. 

      (4) The authors say that "TIMP-1 release from the TIMP-1-loaded nanoparticles diminishes after 5 days." Does that mean from the day of the injection (4-5 days before the "After Day 1") or five days after reintroduced in the ECO-Hab? 

      It means five days after the mice were re-introduced in the ECO-Hab. We have updated the text in Results/Effects of impairing neuronal plasticity in the PL on subterritory preferences and sociability (the end of the first paragraph of this subsection) to 

      “The choice of five-day aggregated data for analysis is in line both with the proper timescales needed for the pairwise maximum entropy model to not overfit, and with the literature that TIMP-1 release from the TIMP-1-loaded nanoparticles is stable for 7-10 days after injection (Chaturvedi et al., 2014)  (i.e. 2-5 days after the mice are reintroduced to Eco-HAB).” (line 272 - 276, page 9)

      (5) In Methods, the authors should report the final N of each of the three groups. 

      The number of final N is reported in Table 1 (page 13). In the updated version, we have added a pointer to Table 1 in Materials and Methods/Animals, and in Materials and Methods/Exclude inactive and dead mice from analysis. We have also expanded the caption of Table 1 to clarify the difference between final N and initial N, and added a pointer to Materials and Methods/Exclude inactive and dead mice from analysis.

    1. Author response:

      The following is the authors’ response to the original reviews.

      Public Reviews:

      Reviewer #1 (Public Review):

      Summary:

      The authors attempt to fully characterize the immunoglobulin (Ig) heavy (H) chain repertoire of tumor-infiltrating B cells from three different cancer types by identifying the IgH repertoire overlap between these, their corresponding draining lymph nodes (DLNs), and peripheral B cells. The authors claim that B cells from tumors and DLNs have a closer IgH profile than those in peripheral blood and that DLNs are differentially involved with tumor B cells. The claim that tumor-resident B cells are more immature and less specific is made based on the characteristics of the CDR-H3 they express.

      Strengths:

      The authors show great expertise in developing in-house bioinformatics pipelines, as well as using tools developed by others, to explore the IgH repertoire expressed by B cells as a means of better characterizing tumor-associated B cells for the future generation of tumor-reactive antibodies as a therapy.

      Weaknesses:

      This paper needs major editing, both of the text and the figures, because as it stands it is convoluted and extremely difficult to follow. The conclusions reached are often not obvious from the figures themselves. Sufficient a priori details describing the framework for their analyses are not provided, making the outcome of their results questionable and leaving the reader wondering whether the findings are on solid ground.

      The authors are encouraged to explain in more detail the premises used in their algorithms, as well as the criteria they follow to define clonotypes, clonal groups, and clonal lineages, which are currently poorly defined and are crucial elements that may influence their results and conclusions.

      In response to this comment, we significantly expanded the paragraph dedicated to the tumor and non-tumor repertoire overlap and isotype composition. The following sections were added:

      First, we characterized the relative similarity of IGH repertoires derived from tumors, DLN, and PBMC on the individual CDR-H3 clonotype level. We define clonotype as an instance with an identical CDR-H3 nucleotide sequence  and identical V- and J- segment attribution (isotype attribution may be different). Unlike other authors, here we do not pool together similar CDR-H3 sequences to account for hypermutation. (Hypermutation analysis is done separately and defined as clonal group analysis. )

      As overlap metrics are dependent on overall repertoire richness, we normalized the comparison using the same number of top most frequent clonotypes of each isotype from each sample (N = 109). Repertoire data for each sample were split according to the immunoglobulin isotype, and the F2 metric was calculated for each isotype separately and plotted as an individual point.

      We also analyzed D metric, which represents the relative overlap diversity uninfluenced by clonotype frequency (Dij\=dij/(di*dj), where dij is the number of clonotypes present in both samples, while di and dj are the diversities of samples i and j respectively). The results for D metric are not shown, as they indicate a similar trend to that of F2 metric. This observation allows us to conclude that tumor IGH repertoires are more similar to the repertoires of lymph nodes than to those of peripheral blood, both if clonotype frequency is taken into account, and when it is not.

      Having excluded the IGHD gene segment from some of their analyses (at least those related to clonal lineage inference and phylogenetic trees), it is not well explained which region of CDR-H3 is responsible for the charge, interaction strength, and Kidera factors, since in some cases the authors mention that the central part of CDR-H3 consists of five amino acids and in others of seven amino acids.

      We considered different ways of calculating amino acid properties of CDR3 and used different parameters for sample-average and individual-sequence CDR3s. Now plots for Fig S6 C are updated  for consistency and the parameters depicted there are now calculated using 5 central amino acids, as in other sections.

      How can the authors justify that the threshold for CDR-H3 identity varies according to individual patient data? 

      Ideal similarity threshold may depend on several factors, such as sampling, sequencing depth etc. For example, imagine a sample picking up 100% of the clonal lineage sequences which differ only 1 amino acid from each other, and a worse quality sample/sequencing picking up only every other sequence. Obviously, the minimal threshold required to accumulate these into a cluster/clonal group  would be different for these two cases (1aa for the former, and ~2 aa for the latter for single-linkage clustering). Or, in other words, the more the sequencing depth, the more dense the clusters will be. The method of individual threshold tailoring relies on the following: https://changeo.readthedocs.io/en/latest/examples/cloning.html

      Although individual kidera factors that are significant in the context of our analysis are described in the text one by one on their first appearance, we now also added a sentence to describe Kidera factor analysis in general (page 8):

      Kidera factors are a set of scores which quantify physicochemical properties of protein sequences (Nakai et al. 1988). 188 physical properties of the 20 amino acids are encoded using dimension reduction techniques.

      Throughout the analyses, the reasons for choosing one type of cancer over another sometimes seem subjective and are not well justified in the text.

      Whenever possible, we pooled all patients with all cancer types together, because the number of available samples did not allow us to draw any significant conclusions comparing between individual cancer types. When analyzing and showing individual patient data, we also did not attempt to depict any cancer-type-specific findings, but it is inevitable that we name a specific cancer type when labelling a sample coming from a specific tumor.

      Overall, the narrative is fragmented. There is a lack of well-defined conclusions at the end of the results subheadings.

      In addition to the described above, a conclusion was added to the paragraph describing hypermutation analysis:

      IGHG clonotypes from lung cancer samples show higher number of hypermutations, possibly reflecting high mutational load found in lung cancer tissue. For melanoma, another cancer known for high mutational load, no statistically significant difference was found. This may be due to higher variance between melanoma samples, which hinders the analysis, or due to the small sample size.

      The exact same paragraph is repeated twice in the results section.

      Corrected.

      The authors have also failed to synchronise the actual number of main figures with the text, and some panels are included in the main figures that are neither described nor mentioned in the text  (Venn diagram Fig. 2A and phylogenetic tree Fig. 5D). Overall, the manuscript appears to have been rushed and not thoroughly read before submission.

      Corrected.

      Reviewers are forced to wade through, unravel, and validate poorly explained algorithms in order to understand the authors' often bold conclusions.

      We hope that the aforementioned additions to the text and also addition to the Figure 1 make the narrative more easily understandable.

      Reviewer #2 (Public Review):

      Summary:

      The authors sampled the B cell receptor repertoires of Cancers, their draining lymph nodes, and blood. They characterized the clonal makeup of all B cells sampled and then analyzed these clones to identify clonal overlap between tissues and clonal activation as expressed by their mutation level and CDR3 amino acid characteristics and length. They conclude that B cell clones from the Tumor interact more with their draining lymph node than with the blood and that there is less mutation/expansion/activation of B cell clones in Tumors. These conclusions are interesting but hard to verify due to the under-sampling and short sequencing reads as well as confusion as to when analysis is across all individuals or of select individuals.

      Strengths:

      The main strength of their analysis is that they take into account multiple characteristics of clonal expansion and activation and their different modes of visualization, especially of clonal expansion and overlap. The triangle plots once one gets used to them are very nice.

      Weaknesses:

      The data used appears inadequate for the conclusions reached. The authors' sample size of B cells is small and they do not address how it could be sufficient. At such low sampling rates, compounded by the plasmablast bias they mention, it is unclear if the overlap trends they observe show real trends. Analyzing only top clones by size does not solve this issue. As it could be that the top 100 clones of one tissue are much bigger than those of another and that all overlap trends are simply because the clones are bigger in one tissue or the other. i.e there is equal overlap of clones with blood but blood is not sufficiently sampled given its greater diversity and smaller clones.

      Regarding the number of clonotypes to be taken into account,  we were limited by the B cell infiltration of tumor samples and our ability to capture their repertoire. However, we use technical replicates on the level of cell suspension to ensure that at least top clonotypes are consistently sampled. So, this is how the data should be interpreted - as describing the most abundant clones in the repertoire (which also may be considered the most functionally relevant in case of tumor infiltrating lymphocytes).

      To analyze the repertoire overlap, we generally use the F2 metric that takes clone size into account - because we think that clone size is an important functional factor. However, we have now added the description of using D metric (does not include clone frequency as a parameter) - which shows exactly the same trend as F2 metric. So, both F2 and D overlap metrics support our conclusion of higher overlap between tumor and LN.

      The following text was added:

      We also analyzed D metric, which represents the relative overlap diversity uninfluenced by clonotype frequency (Dij\=dij/(di*dj), where dij is the number of clonotypes present in both samples, while di and dj are the diversities of samples i and j respectively). The results for D metric are not shown, as they indicate a similar trend to that of F2 metric. This observation allows us to conclude that tumor IGH repertoires are more similar to the repertoires of lymph nodes than to those of peripheral blood, both if clonotype frequency is taken into account, and when it is not.

      All in all, of course, the deeper the better, but given the data we were able to generate from the samples, this was the best approach to normalization that could be used.

      Similarly, the read length (150bp X2) is too short, missing FWR1 and CDR1 and often parts of FWR2 if CDR3 is long. As the authors themselves note (and as was shown in (Zhang 2015 - PMC4811607) this makes mutation analysis difficult.

      Indeed, we are aware of this problem, and therefore only a small part of the manuscript is dedicated to the hypermutation analysis. However, as the CDR-H3 region is the most mutated part, we still can capture significant diversity of mutations. To address the question of applicability of our data for the hypermutation phylogeny analysis, we compare the distribution of physico-chemical properties along the trees of hypermutation using the 150+150 and 300+300 data from the same donor and the same set of samples. The main conclusion is that neither for long, nor for short datasets could any correlation of physicochemical properties of the CDR-H3 region with the rank of the clonotype on the tree be found.  

      It also makes the identification of V genes and thus clonal identification ambiguous. This issue becomes especially egregious when clones are mutated.

      Again, this would be important for clonotype phylogeny analysis. However, for the simple questions that we address with our clonal group analysis, such as clonal group overlap between tissues etc, we consider this data acceptable, because if any mislabelling of V segment occurs, it is a) rare and b) is equally frequent in all types of samples. Therefore, any conclusions made are still valid despite this technical drawback.

      To directly address the question of mislabelling of V-genes in our data, we looked at the average number of different  V-genes attributed to the same nucleotide sequence of CDR-H3 region in the short (150+150) and long (300+300) datasets from the same donor. Indeed, some ambiguity of V-gene labelling is observed (see below), but we think that it is unlikely to influence any of our cautious conclusions.

      Author response image 1.

      Finally, it is not completely clear when the analysis is of single individuals or across all individuals. If it is the former the authors did not explain how they chose the individuals analyzed and if the latter then it is not clear from the figures which measurements belong to which individual (i.e they are mixing measurements from different people).

      We addressed this issue by adding a comment to each figure caption, describing whether a particular figure or panel describes individual or pooled data, and also whether the analysis is done on individual clonotype or clonal group level.

      Also, in case pooled data were used, we added the number of patients that was pooled for a particular type of analysis. This number differs from one type of analysis to the other, because not all the patients had a complete set of tissues, and also not all samples passed a quality check for a particular analysis.

      Here are the numbers listed:

      Fig 2A: N=6 (we were only considering those who had all three tissues)

      Fig 2C, N=14 (all)

      2D: N=14 (all)

      2E N=7 (have both tum and PBMC).

      2F N=9 (have both tum and PBMC).

      2G N=9 (have both tum and PBMC)

      2H N=7 (have both tum and LN)

      3A N=14 (all)

      3B N=11 (only those with tumor)

      3E - N=14

      7F N=11 (all that have tumor)

      Reviewer #3 (Public Review):

      In multiple cancers, the key roles of B cells are emerging in the tumor microenvironment (TME). The authors of this study appropriately introduce that B cells are relatively under-characterised in the TME and argue correctly that it is not known how the B cell receptor (BCR) repertoires across tumors, lymph nodes, and peripheral blood relate. The authors therefore supply a potentially useful study evaluating the tumor, lymph node, and peripheral blood BCR repertoires and site-to-site as well as intra-site relationships. The authors employ sophisticated analysis techniques, although the description of the methods is incomplete. Among other interesting observations, the authors argue that the tumor BCR repertoire is more closely related to that of draining lymph node (dLN) than the peripheral blood in terms of clonal and isotype composition. Furthermore, the author's findings suggest that tumor-infiltrating B cells (TIL-B) exhibit a less mature and less specific BCR repertoire compared with circulating B cells. Overall, this is a potentially useful work that would be of interest to both medical and computational biologists working across cancer. However, there are aspects of the work that would have benefitted from further analysis and areas of the manuscript that could be written more clearly and proofread in further detail.

      Major Strengths:

      (1) The authors provide a unique analysis of BCR repertoires across tumor, dLN, and peripheral blood. The work provides useful insights into inter- and intra-site BCR repertoire heterogeneity. While patient-to-patient variation is expected, the findings with regard to intra-tumor and intra-dLN heterogeneity with the use of fragments from the same tissue are of importance, contribute to the understanding of the TME, and will inform future study design.

      (2) A particular strength of the study is the detailed CDR3 physicochemical properties analysis which leads the authors to observations that suggest a less-specific BCR repertoire of TIL-B compared to circulating B cells.

      Major Weaknesses:

      The study would have benefitted from a deeper biological interpretation of the data. While given the low number of patients one can plausibly understand a reluctance to speculate about clinical details, there is limited discussion about what may contribute to observed heterogeneity.

      We indeed do not want to overinterpret our data, especially where it comes to the difference between types of cancer. On the other hand, extracting similar patterns between different cancer types allows to pinpoint mechanisms that are more general and do not depend on cancer type. As for the potential source of intratumoral heterogeneity that we observe, we think that it may be coming from the selective sampling of tertiary lymphoid structures. We include IHC data for TLS detection in the supplementary Fig.5.  Also, tumor mutation clonality may correlate with differential antibody response (i.e. different IGH clonotypes developing to recognize different antigens) – as has been previously described for TCRs by the lab of B.Chain in https://www.ncbi.nlm.nih.gov/pmc/articles/PMC6890490/.

      For example, for the analysis of three lymph nodes taken per patient which were examined for inter-LN heterogeneity, there is a lack of information regarding these lymph nodes.

      Unfortunately no clinical information about the lymph nodes was available.

      'LN3' is deemed as exhibiting the most repertoire overlap with the tumor but there is no discussion as to why this may be the case.

      The following phrases describes this in the “LN-to-LN heterogeneity in colorectal cancer” paragraph:

      Similarly, an unequal interaction of tumors with DLNs was observed at the level of hypermutating clonal groups.

      Functionally, this may again indicate that within a group of DLNs, nodes are unequal in terms of access to tumor antigens, and this inequality shapes the BCR repertoires within these lymph nodes.

      (2) At times the manuscript is difficult to follow. In particular, the 'Intra-LN heterogeneity' section follows the 'LN-LN heterogeneity in colorectal cancer' section and compares the overlap of LN fragments (LN11, LN21, LN31) with the tumor in two separate patients (Fig 6A). In the previous section (LN-LN), LN11, LN21, LN31 are names given to separate lymph nodes from the same patient. The fragments are referred to as 'LN2' and the nodes in the previous section are referred to similarly. This conflation of naming for nodes and fragments is confusing.

      We corrected this.

      (3) There is a duplicated paragraph in 'Short vs long trees' and the following section 'Productive involvement in hypermutation lineages depends on CDR3 characteristics.

      Corrected.

      Reviewer #1 (Recommendations For The Authors):

      - Figures:

      Figure 1A lacks resolution

      Corrected

      Figure 2A, Venn diagram: What do the colors indicate?

      Corrected

      Figure 5D, why include this tree when there is no mention of it in the text?

      Described

      Figures 8, 9, and 10 are not to be found. One should not have to figure out that they became supplementary in the end.

      Corrected

      Regarding the physicochemical properties of CDR-H3, what do the authors mean by "the central part"? Do the authors refer to the CDR-H3 loop, and if so, how is that defined when the IGHD gene segment is excluded from the analyses? Is it 5 amino acids (Productive involvement in hypermutating lineages depends on CDR3 characteristics, Page 21/39 in merged document) and (CDR3 properties, Page 8/39 in merged document), or 7 amino acids (Short vs long trees phylogeny analysis, Page 19/39 in merged document)? Please clarify.  

      We considered different ways of calculating amino acid properties of CDR3 and used different parameters for sample-average and individual-sequence CDR3s. Now plots for Fig S6 C are updated for consistency. IGHD segment was not excluded from the analysis. The reviewer might be confused by our description of phylogenetic inference, when an artificial outgroup with D segment deleted is added to the clonal group to facilitate the inference process. All other sequences were analyzed in their original form with the D segment. This way, we could avoid biases in phylogeny introduced by misassignment of D gene germline to the outgroup.

      What was the threshold for CDR-H3 identity in their analyses? How can the authors justify that this value changes according to individual patient datasets? (Materials & methods, Clonal lineage inference Page 29/39 in merged document).

      As described earlier, ideal similarity threshold may depend on several factors, such as sampling, sequencing depth etc. For example, imagine a sample picking up 100% of the clonal lineage sequences which differ only 1 amino acid from each other, and a worse quality sample/sequencing picking up only every other sequence. Obviously, the minimal threshold required to accumulate these into a clonotype would be different for these two cases (1aa for the former, and ~2 aa for the latter for single-linkage clustering). The method of individual threshold tailoring relies on this: https://changeo.readthedocs.io/en/latest/examples/cloning.html

      What is the difference between tumor-induced and tumor-infiltrating B cells? How can the authors discriminate between the two? Page 6/39 in the merged document.

      corrected to tumor-infiltrating

      "Added nucleotides" meaning N additions? Page 3/39 in the merged document.

      yes

      How many cancer patients were enrolled? 17 or 14(Materials & methods page 27/39 in the merged document)? Please clarify.   

      In the current project 14 patients were enrolled. The appropriate changes have been introduced in the final text. Supplementary table 2 has been added with the patient data.

      Abbreviations are used without full descriptions.

      According to reviewer’s recommendation, a list of abbreviations was added in the manuscript, and also full descriptions were added in the text upon first mentioning of the term.

      Use either CDR3 or CDR-H3

      We corrected the text to use CDR-H3 abbreviation throughout the text.

      Reviewer #2 (Recommendations For The Authors):

      I would like to start by apologizing for the time it took me to review.

      As I mentioned above there are issues with the clonal sampling of the sequencing length and the statistics in this paper. From reading the paper I am not sure if they are fixable but there are some things that could be tried.

      (1) The authors mention the diversity of their individual analysis - 17 individuals across 3 cancer types, but do not then systematically show us how the different things they measure track across the different individuals and cancer types. it is possible that some trends would be more convincing if we saw them happening again and again across all individuals. But, as I said above, the authors do not identify individuals clearly across all their types of analysis nor do they explain why sometimes they show analysis of specific individuals.

      For overlap analysis (Fig. 2 except panel B), CDR3 properties analysis (Fig. 3, Fig. S7), clonal group analysis (Fig. 4) we used pooled data on all cancers, unless it is indicated otherwise on the panel. For overlap analysis, we used Cytoscape graph (Fig. 2B) for one patient, mp3, to illustrate the findings that were made on pooled data. For other types of analysis, such as overlap between individual lymph nodes, or tumor fragments (Fig. 5, 6, 7 except panel F) pooled analysis is not possible due to the individual nature of the processes in question.

      (2) The authors do not address how lacking their sampling is nor the distribution of clone sizes in different tissues/ individuals/ subsets. Without such a discussion it is not clear how tenuous or convincing their conclusions are.

      (3) The short sequencing lengths limit the ability to exactly identify V and thus the germline root of clones, whose positions are mutated and clonal association of sequences. The authors appear to be aware of this as they often use the most common ancestor as the start of their analysis... however, again there are inconsistencies that are not clearly described in the text. in creating trees with change they defined roots as the putative germline and at least in most cases also in clone association although in some analyses potentially similar clones were collapsed into clonotypes. Again it is not clear when one method was used or the other and how the choice was made what to choose.

      Here we can only state that we consistently used the approach described in the Methods section, which was the following:

      First, the repertoires were clustered into clonal lineages using the criteria described in “Methods: Clonal lineage inference” Assuming that each clonotype sequence in the clonal lineage originated from the same ancestor, we try to recover the phylogeny. Please note that we refer to the individual BCR sequences as “clonotypes”, and to a group of clonotypes that presumably share a common ancestor - as “clonal lineage” or “clonal group”.

      The phylogeny of B-cell hypermutations was inferred for each clonal lineage of size five or more using the maximum likelihood method and the GTR GAMMA nucleotide substitution model. To find the most recent common ancestor (MRCA) or “root” of the tree, we used an artificial outgroup constructed as a conjugate of germline segments V and J defined by MIXCR and added it to the clonal lineage. The D segment was excluded from the outgroup formation, as there was insufficient confidence in the germline annotations due to its short length and high level of mutations. The rest of the clonotypes were still analyzed in their original form with D segment in place. Deleting D segment from the outgroup simply eliminates the risk of biasing the phylogeny by missasigning D segment germline sequence to the outgroup. The MUSCLE tool was used for multiple sequence alignment and RAxML software was used to build and root phylogenetic trees.

      (4) Beyond the statistical issues mentioned above: the unclear selection of individual examples for comparison and significance testing, the mixing of individuals and cancer types without clear identification, etc. there is in general a lack of coherence in the statistical analysis performed. specifically:

      (a) the authors should choose one cutoff for significance (0.01 for instance) and then just mention when things are significant and when not. There is no need and it is confusing to add the p-value for every comparison. P-values are not good measures of effect size.

      We corrected the figures and left p-values only where they are below significance threshold.

      (b) the Bonferroni correction used is not well characterized. For an alpha of 0.01 in Figures 3 C and D how many tests were performed?

      The number of tests performed that was used for Bonferroni-Holm correction equals the number of comparisons on the heatmap which makes it 39 for each heatmap on Fig 3C and 13 for Fig 3D.

      Finally some minor issues -

      (1) Not all acronyms are described, for instance, TME and TIL. The first time any acronym is used it should be spelled out.  -> Katya B- список сокращений

      (2) The figure captions are not all there...

      (a) there is no caption for Figure 3E.

      corrected

      (b) there are Figure 7 F and G panels but no Figure 7E panel and Figure F is described after Figure G.

      corrected

      (3) A few problems with wording -

      (a) bottom paragraph of page 3 - instead of :

      "different lymph nodes from one draining lymph node pool may be more or less involved"

      Corrected to "different lymph nodes from one draining lymph node pool may be differentially involved"

      (b) figure caption for figure 3a: instead of:

      "CDR3 are on average significantly higher in tumor"

      Corrected to "CDR3 are on average significantly longer in tumor"

      Reviewer #3 (Recommendations For The Authors):

      - FIG1A - Suggest expanding the legend to include more information on the computational analyses.

      added

      - PAGE SIX: Suggest adding a table or some text on patient characteristics. Numbers of unique clonotypes per sample etc. Are there differences in age/sex that need to be considered? Some clonotype information is available in S1 but some summary and statistics would be appreciated.

      Added patient information as Supplementary table 2.

      - PAGE SIX: F2 Metric, suggestion to explain why this was used vs. other metrics.

      We expanded the following paragraph to include information about F2 metric and D metric, and the reason why we are using F2.

      Repertoire data for each sample were split according to the immunoglobulin isotype, and the F2 metric was calculated for each isotype separately and plotted as an individual point. We used the repertoire overlap metric F2 (Сlonotype-wise sum of geometric mean frequencies of overlapping clonotypes), which accounts for both the number and frequency of overlapping clonotypes (Fig. 2A). As expected, significantly lower overlaps were observed between the IGH repertoires of peripheral blood and tumors compared to LN/tumor overlaps. The LN/PBMC overlap also tended to be lower, but the difference was not statistically significant. We also analyzed D metric, which represents the relative overlap diversity uninfluenced by clonotype frequency (Dij\=dij/(di*dj), where dij is the number of clonotypes present in both samples, while di and dj are the diversities of samples i and j respectively). The results for D metric are not shown, as they indicate a similar trend to that of F2 metric. This observation allows us to conclude that tumor IGH repertoires are more similar to the repertoires of tumor-draining LNs than to those of peripheral blood, both if clonotype frequency is taken into account, and when it is not.

      - PAGE SIX: Make clear in the text that mp3 is a patient.

      Added “melanoma patient mp3”

      - PAGE EIGHT: Suggest explaining kidera factors at first use - not all readers will know what they are.

      We expanded the following paragraph to add more information about Kidera factors:

      To explore CDR-H3 physicochemical properties, we calculated the mean charge, hydropathy, predicted interaction strength, and Kidera factors 1 - 9 (kf1-kf9) for five central amino acids of the CDR-H3 region for the 100 most frequent clonotypes of each sample using VDJtools. Kidera factors are a set of scores which quantify physicochemical properties of protein sequences 61. 188 physical properties of the 20 amino acids are encoded using dimension reduction techniques, to yield 9 factors which are used to quantitatively characterize physicochemical properties of amino acid sequences.

      - Fig 5D is not referred to.

      Corrected

    1. Author response:

      The following is the authors’ response to the original reviews

      eLife assessment 

      This valuable study aims to present a mathematical theory for why the periodicity of the hexagonal pattern of grid cell firing would be helpful for encoding 2D spatial trajectories. The idea is supported by solid evidence, but some of the comparisons of theory to the experimental data seem incomplete, and the reasoning supporting some of the assumptions made should be strengthened. The work would be of interest to neuroscientists studying neural mechanisms of spatial navigation. 

      We thank the reviewers for this assessment. We have addressed the comments made by reviewers and believe that the revised manuscript has theoretical and practical implications beyond the subfield of neuroscience concerned with mechanisms underpinning spatial memory and spatial navigation. Specifically, the demonstration that four simple axioms beget the spatial firing pattern of grid cells is highly relevant for the field of artificial intelligence and neuromorphic computing. This relevance stems from the fact that the four axioms define a set of four simple computational algorithms that can be implemented in future work in grid cell-inspired computational algorithms. Such algorithms will be impactful because they can perform path integration, a function that is independent of an animal’s or agent’s location and therefore generalizable. Moreover, because of the functional organization of grid cells into modules, the algorithm is also scalable. Generalizability and scalability are two highly sought-after properties of brain-inspired computational frameworks. We also believe that the question why grid cells emerge in the brain is a fundamental one. This manuscript is, to our knowledge, the first one that provides an interpretable and intuitive answer to why grid cells are observed in the brain. 

      Before addressing each comment, we would like to point out that the first sentence of the assessment appears misphrased. The study does not aim to present a theory for why the periodicity in grid cell firing would be helpful for encoding 2D spatial trajectories. To present a theory “for why grid cell firing would be helpful for encoding 2D trajectories”, one assumes the existence of grid cells a priori. Instead of assuming the existence of grid cells and deriving a computational function from grid cells, our study derives grid cells from a computational function, as correctly summarized by reviewers #1 and #3 in their individual statements. In contrast to previous normative models, we prove mathematically that spatial periodicity in grid cell firing is implied by a sequence code of trajectories. If the brain uses cell sequences to code for trajectories, spatially periodic firing must emerge. As correctly pointed out by reviewer #1, the underlying assumptions of this study are that the brain codes for trajectories and that it does so using cell sequences. In response to comments by reviewer #1, we now discuss these two assumptions more rigorously.

      Public Reviews:

      Reviewer #1 (Public Review): 

      Rebecca R.G. et al. set to determine the function of grid cells. They present an interesting case claiming that the spatial periodicity seen in the grid pattern provides a parsimonious solution to the task of coding 2D trajectories using sequential cell activation. Thus, this work defines a probable function grid cells may serve (here, the function is coding 2D trajectories), and proves that the grid pattern is a solution to that function. This approach is somewhat reminiscent in concept to previous works that defined a probable function of grid cells (e.g., path integration) and constructed normative models for that function that yield a grid pattern. However, the model presented here gives clear geometric reasoning to its case. 

      Stemming from 4 axioms, the authors present a concise demonstration of the mathematical reasoning underlying their case. The argument is interesting and the reasoning is valid, and this work is a valuable addition to the ongoing body of work discussing the function of grid cells. 

      However, the case uses several assumptions that need to be clearly stated as assumptions, clarified, and elaborated on: Most importantly, the choice of grid function is grounded in two assumptions: 

      (1) that the grid function relies on the activation of cell sequences, and 

      (2) that the grid function is related to the coding of trajectories. While these are interesting and valid suggestions, since they are used as the basis of the argument, the current justification could be strengthened (references 28-30 deal with the hippocampus, reference 31 is interesting but cannot hold the whole case). 

      We thank this reviewer for the overall positive and constructive criticism. We agree with this reviewer that our study rests on two premises, namely that 1) a code for trajectories exist, and 2) this code is implemented by cell sequences. We now discuss and elaborate on the data in the literature supporting the two premises.

      In addition to the work by Zutshi et al. (reference 31 in the original manuscript), we have now cited additional work presenting experimental evidence for sequential activity of neurons in the medial entorhinal cortex, including sequential activity of grid cells.

      We have added the following paragraph to the Discussion section:

      “Recent studies provided compelling evidence for sequential activity of neurons representing spatial trajectories. In particular, Gardner et al. (2022) demonstrated that the sequential activity of hundreds of simultaneously recorded grid cells in freely foraging rats represented spatial trajectories. Complementary preliminary results indicate that grid cells exhibit left-rightalternating “theta sweeps,” characterized by temporally compressed sequences of spiking activity that encode outwardly oriented trajectories from the current location (Vollan et al., 2024).

      The concept of sequential grid cell activity extends beyond spatial coding. In various experimental contexts, grid cells have been shown to encode non-spatial variables. For instance, in a stationary auditory task, grid cells fired at specific sounds along a continuous frequency axis (Aronov et al., 2017). Further studies revealed that grid cell sequences also represent elapsed time and distance traversed, such as during a delay period in a spatial alternation task (Kraus et al., 2015). Similar findings were reported for elapsed time encoded by grid cell sequences in mice performing a virtual “Door Stop” task (Heys and Dombeck, 2018).

      Additionally, spatial trajectories represented by temporally compressed grid cell sequences have been observed during sleep as replay events (Ólafsdóttir et al., 2016; O’Neill et al., 2017). Collectively, these studies demonstrate that sequential activity of neurons within the MEC, particularly grid cells, consistently encodes ordered experiences, suggesting a fundamental role for temporal structure in neuronal representations.

      The theoretical underpinnings of grid cell activity coding for ordered experiences have been explored previously by Rueckemann et al. (2021) who argued that the temporal order in grid cell activation allows for the construction of topologically meaningful representations, or neural codes, grounded in the sequential experience of events or spatial locations. However, while Rueckemann et al. argue that the MEC supports temporally ordered representations through grid cell activity, our findings suggest an inverse relationship: namely, that grid cell activity emerges from temporally ordered spatial experiences. Additional studies demonstrate that hippocampal place cells may derive their spatial coding properties from higher-order sequence learning that integrates sensory and motor inputs (Raju et al., 2024) and that hexagonal grids, if assumed a priori, optimally encode transitions in spatiotemporal sequences (Waniek, 2018).

      Together, experimental and theoretical evidence demonstrate the significance of sequential neuronal activity within the hippocampus and entorhinal cortex as a core mechanism for representing both spatial and temporal information and experiences.”

      The work further leans on the assumption that sequences in the same direction should be similar regardless of their position in space, it is not clear why that should necessarily be the case, and how the position is extracted for similar sequences in different positions. 

      We thank this reviewer for giving us the opportunity to clarify this point. We define a trajectory as a path taken in space (Definition 6). By this definition, a code for trajectories is independent of the animal’s spatial location. This is consistent with the definition of path integration, which is also independent of an animal’s spatial location. If the number of neurons is finite (Axiom #4) and the space is large, sequences must eventually repeat in different locations. This results in neural sequences coding for the same directions being identical at different locations. We have clarified this point under new Remark 6.1. in the Results section of the revised:

      “Remark 6.1. Note that a code for trajectories is independent of the animal’s spatial location, consistent with the definition of path integration. This implies that, if the number of neurons is finite (Axiom #4) and the space is large, sequences must eventually repeat in different location, resulting in neural sequences coding for the same trajectories at different locations.”

      The formal proof was already included in the original manuscript: “Generally speaking, starting in a firing field of element i and going along any set of firing fields, some element must eventually become active again since the total number of elements is finite by axiom 4. Once there is a repeat of one element’s firing field, the whole sequence of firing fields of all elements must repeat by axiom 1. More specifically, if we had a sequence 1,2, … , k, 1, t of elements, then 1,2 and 1, t both would code for traveling in the same direction from element 1, contradicting axiom 1.”

      Further: “More explicitly, assuming axioms 1 and 4, the firing fields of trajectory-coding elements must be spatially periodic, in the sense that starting at any point and continuing in a single direction, the initial sequence of locally active elements must eventually repeat with a repeat length of at least 3”.

      Regarding the question how an animal’s position is extracted for similar sequences in different positions, we agree with this reviewer that this is an important question when investigating the contributions of grid cells to the coding of space. However, since a code for trajectories is independent of spatial location, the question of how to extract an animal’s position from a trajectory code is irrelevant for this study.

      While a trajectory code by neural sequences begets grid cells, a spatial code by neural sequences does not. Nevertheless, grid cells could contribute to the coding of space (in addition to providing a trajectory code). However, while experimental evidence from studies with rodents and human subjects and theoretical work demonstrated the importance of grid cells for path integration (Fuhs and Touretzky, 2006; McNaughton et al., 2006; Moser et al., 2017), experimental studies have shown that grid cells contribute little to the coding of space by place cells (Hales et al., 2014). Yet, theoretical work (Mathis et al., 2012) showed that coherent activity of grid cells across different modules can provide a code for spatial location that is more accurate than spatial coding by place cells in the hippocampus. Importantly, such a spatial code by coherent activity across grid cell modules does not require location-dependent differences in neural sequences.

      The authors also strengthen their model with the requirement that grid cells should code for infinite space. However, the grid pattern anchors to borders and might be used to code navigated areas locally. Finally, referencing ref. 14, the authors claim that no existing theory for the emergence of grid cell firing that unifies the experimental observations on periodic firing patterns and their distortions under a single framework. However, that same reference presents exactly that - a mathematical model of pairwise interactions that unifies experimental observations. The authors should clarify this point. 

      We thank this reviewer for this valuable feedback. We agree that grid cells anchor to borders and may be used to code navigated areas locally. In fact, the trajectory code performs a local function, namely path integration, and the global grid pattern can only emerge from performing this local computation if the activity of at least one grid unit or element (we changed the wording from unit to element based on feedback from reviewer #3) is anchored to either a spatial location or a border. Yet, the trajectory code itself does not require anchoring to a reference frame to perform local path integration. Because of the local nature of the trajectory code, path integration can be performed locally without the emergence of a global grid pattern. This has been shown experimentally in mice performing a path integration task where changes in the location of a task-relevant object resulted in translations of grid patterns in single trials. Although no global grid pattern was observed, grid cells performed path integration locally within the multiple reference frames defined by the task-relevant object, and grid patterns were visible when the changes in the references frames were accounted for in computing the rate maps (Peng et al., 2023). The data by Peng et al. (2023) confirm that the anchoring of the grid pattern to borders and the emergence of the global pattern are not required for local coding of trajectories. The global pattern emerges only when the reference frame does not change. However, this global pattern itself might not serve any function. According to the trajectory code model, the beguiling grid pattern is merely a byproduct of a local path integration function that is independent of the animal’s current location (which makes the code generalizable across space). The reviewer is correct that, if the reference frame used to anchor the grid pattern did not change in infinite space, the trajectory code model of grid cell firing would predict an infinite global pattern. But does the proof implicitly assume that space is infinite? The trajectory code model makes the quantitative prediction that the field size increases linearly with an increase in grid spacing (the distance between two fields). If the field size remains fixed, periodicity will emerge in finite spaces that are larger than the grid spacing. We have clarified these points in the revised manuscript:

      “Notably, the trajectory code itself does not require anchoring to a reference frame to perform local path integration. Because of the local nature of the trajectory code, path integration can be performed locally without the emergence of a global grid pattern. This has been shown experimentally in mice performing a path integration task where changes in the location of a task-relevant object resulted in translations of grid patterns in single trials (Peng et al., 2023). Although no global grid pattern was observed because the reference frame was not fixed in space, grid cells performed path integration locally within the reference frame defined by the moving task-relevant object, and grid patterns were visible when the changes in the references frames were accounted for in computing the rate maps”.

      Regarding how the emergence of grid cells from a trajectory code relates to the theory of a local code by grid cells brought forward by Ginosar et al. (ref. 14), we argue that the local computational function suggested by Ginosar et al. is to provide a code for trajectories. The perspective article by Ginosar et al. provides an excellent review of the experimental data on grid cells that point to grid cells performing a local function (see also Kate Jeffery’s excellent review article (Jeffery, 2024) on the mosaic structure of the mammalian cognitive map.) Assuming the existence of grid cells a priori, Ginosar et al. then propose three possible functions of grid cells, all of which are consistent with the trajectory code model of grid cell firing. Yet, the perspective article remains agnostic, in our opinion, on the exact nature of the local computation that is carried out by grid cells. But without knowing the local computation underlying grid cell function, a unifying theory explaining the emergence of grid cells cannot be considered complete. In contrast, our manuscript identifies the local computational function as a trajectory code by cell sequences. We have clarified these points in the revised manuscript:

      “The influential hypothesis that grid cells provide a universal map for space is challenged by experimental data suggesting a yet to be identified local computational function of grid cells (Ginosar et al., 2023; Jeffery, 2024). Here, we identify this local computational function as a trajectory code.”

      The mathematical model of pairwise interactions described by Ginosar et al. is fundamentally different from the mathematical framework developed in our manuscript. The mathematical model by Ginosar et al. describes how pairwise interactions between already existent grid fields can explain distortions in the grid pattern caused by the environment’s geometry, reward zones, and dimensionality. However, the model does not explain why there is a grid pattern in the first place. In contrast, our trajectory model provides an explanation for why grid cells may exist by demonstrating that a grid pattern emerges from a trajectory code by cell sequences. We stand by our assessment that a unifying theory of grid cells is not complete if it takes the existence of the grid pattern for granted.

      Reviewer #2 (Public Review): 

      Summary: 

      In this work, the authors consider why grid cells might exhibit hexagonal symmetry - i.e., for what behavioral function might this hexagonal pattern be uniquely suited? The authors propose that this function is the encoding of spatial trajectories in 2D space. To support their argument, the authors first introduce a set of definitions and axioms, which then lead to their conclusion that a hexagonal pattern is the most efficient or parsimonious pattern one could use to uniquely label different 2D trajectories using sequences of cells. The authors then go through a set of classic experimental results in the grid cell literature - e.g. that the grid modules exhibit a multiplicative scaling, that the grid pattern expands with novelty or is warped by reward, etc. - and describe how these results are either consistent with or predicted by their theory. Overall, this paper asks a very interesting question and provides an intriguing answer. However, the theory appears to be extremely flexible and very similar to ideas that have been previously proposed regarding grid cell function. 

      We thank this reviewer for carefully reading the manuscript and their valuable feedback which helps us clarify major points of the study. One major clarification is that the theoretical/axiomatic framework we put forward does not assume grid cells a priori. In contrast, we start by hypothesizing a computational function that a brain region shown to be important for path integration likely needs to solve, namely coding for spatial trajectories. We go on to show that this computational function begets spatially periodic firing (grid maps). By doing so, we provide mathematical proof that grid maps emerge from solving a local computational function, namely spatial coding of trajectories. Showing the emergence of grid maps from solving a local computational function is fundamentally different from many previous studies on grid cell function, which assign potential functions to the existing grid pattern. As we discuss in the manuscript, our work is similar to using normative models of grid cell function. However, in contrast to normative models, we provide a rigorous and interpretable mathematical framework which provides geometric reasoning to its case.

      Major strengths: 

      The general idea behind the paper is very interesting - why *does* the grid pattern take the form of a hexagonal grid? This is a question that has been raised many times; finding a truly satisfying answer is difficult but of great interest to many in the field. The authors' main assertion that the answer to this question has to do with the ability of a hexagonal arrangement of neurons to uniquely encode 2D trajectories is an intriguing suggestion. It is also impressive that the authors considered such a wide range of experimental results in relation to their theory.  

      We thank this reviewer for pointing out the significance of the question addressed by our manuscript.

      Major weaknesses: 

      One major weakness I perceive is that the paper overstates what it delivers, to an extent that I think it can be a bit confusing to determine what the contributions of the paper are. In the introduction, the authors claim to provide "mathematical proof that ... the nature of the problem being solved by grid cells is coding of trajectories in 2-D space using cell sequences. By doing so, we offer a specific answer to the question of why grid cell firing patterns are observed in the mammalian brain." This paper does not provide proof of what grid cells are doing to support behavior or provide the true answer as to why grid patterns are found in the brain. The authors offer some intriguing suggestions or proposals as to why this might be based on what hexagonal patterns could be good for, but I believe that the language should be clarified to be more in line with what the authors present and what the strength of their evidence is. 

      We thank this reviewer for this assessment. While there is ample experimental evidence demonstrating the importance of grid cells for path integration, we agree with this reviewer that there may be other computational functions that may require or largely benefit from the existence of grid cells. We now acknowledge the fact that we have provided a likely teleological cause for the emergence of grid cells and that there might be other causes for the emergence of grid cells. We have changed the wording in the abstract and discussion sections to acknowledge that our study does provide a likely teleological cause. We choose “likely” because the computational function – trajectory coding – from which grid maps emerge is very closely associated to path integration, which numerous experimental and theoretical studies associate with grid cell function.

      Relatedly, the authors claim that they find a teleological reason for the existence of grid cells - that is, discover the function that they are used for. However, in the paper, they seem to instead assume a function based on what is known and generally predicted for grid cells (encode position), and then show that for this specific function, grid cells have several attractive properties. 

      We agree with this reviewer that we leveraged what is known about grid cells, in particular their importance for path integration, in finding a likely teleological cause. However, the major significance of our work is that we demonstrate that coding for spatial trajectories requires spatially periodic firing (grid cells).This is very different from assuming the existence of grid cells a priori and then showing that grid cells have attractive, if not optimal, properties for this function. If we had shown that grid cells optimized a code for trajectories, this reviewer would be correct: we would have suggested just another potential function of grid cells. Instead, we provide both proof and intuition that trajectory coding by cell sequences begets grid cells (not the other way around), thereby providing a likely teleological cause for the emergence of grid cells. As stated above, we clarified in the revised manuscript that we provide a likely teleological cause which requires additional experimental verification.

      There is also some other work that seems very relevant, as it discusses specific computational advantages of a grid cell code but was not cited here: https://www.nature.com/articles/nn.2901

      We thank this reviewer for pointing us toward this article by (Sreenivasan and Fiete, 2011). The revised manuscript now cites this article in the Introduction and Discussion sections. We agree that the article by (Sreenivasan and Fiete, 2011) discusses a specific computational advantage of a population code by grid cells, namely unprecedented robustness to noise in estimating the location from the spiking information of noisy neurons. However, the work by (Sreenivasan and Fiete, 2011) differs from our work in that the authors assume the existence of grid cells a priori.

      In addition, we now discuss other relevant work, namely work on the conformal isometry hypothesis  by (Schøyen et al., 2024) and (Xu et al., 2024), published as pre-prints after publication of the first version of our manuscript, as well as work on transition scale- spaces by Nicolai Waniek. (Xu et al., 2024) and (Schøyen et al., 2024) investigate conformal isometry in the coding of space by grid cells. Conformal isometry means that trajectories in neural space map trajectories in physical space. (Xu et al., 2024) show that the conformal isometry hypothesis can explain the spatially periodic firing pattern of grid cells. (Schøyen et al., 2024) further show that a module of seven grid cells emerges if space is encoded as a conformal isometry, ensuring equal representation in all directions. While the work by (Xu et al., 2024) and (Schøyen et al., 2024) arrive at very similar conclusions as stated in the current manuscript, the conformal isometry hypothesis provides only a partial answer to why grid cells exist because it doesn’t explain why conformal isometry is important or required. In contrast, a sequence code of trajectories provides an intuitive answer to why such a code is important for animal behavior. Furthermore, we included the work by Nicolai Waniek, (2018, 2020) in the Discussion, who demonstrated that the hexagonal arrangement of grid fields is optimal for coding transitions in space. 

      The paragraph added to the Discussion reads as follows:

      “As part of the proof that a trajectory code by cell sequences begets spatially periodic firing fields, we proved that the centers of the firing fields must be arranged in a hexagonal lattice. This arrangement implies that the neural space is a conformally isometric embedding of physical space, so that local displacements in neural space are proportional to local displacements of an animal or agent in physical space, as illustrated in Figure 5. This property has recently been introduced in the grid cell literature as the conformal isometry hypothesis(Schøyen et al., 2024; Xu et al., 2024). Strikingly, Schøyen et al.(Schøyen et al., 2024) arrive at similar if not identical conclusions regarding the geometric principles in the neural representations of space by grid cells.”

      A second major weakness was that some of the claims in the section in which they compared their theory to data seemed either confusing or a bit weak. I am not a mathematician, so I was not able to follow all of the logic of the various axioms, remarks, or definitions to understand how the authors got to their final conclusion, so perhaps that is part of the problem. But below I list some specific examples where I could not follow why their theory predicted the experimental result, or how their theory ultimately operated any differently from the conventional understanding of grid cell coding. In some cases, it also seemed that the general idea was so flexible that it perhaps didn't hold much predictive power, as extra details seemed to be added as necessary to make the theory fit with the data. 

      I don't quite follow how, for at least some of their model predictions, the 'sequence code of trajectories' theory differs from the general attractor network theory. It seems from the introduction that these theories are meant to serve different purposes, but the section of the paper in which the authors claim that various experimental results are predicted by their theory makes this comparison difficult for me to understand. For example, in the section describing the effect of environmental manipulations in a familiar environment, the authors state that the experimental results make sense if one assumes that sequences are anchored to landmarks. But this sounds just like the classic attractornetwork interpretation of grid cell activity - that it's a spatial metric that becomes anchored to landmarks. 

      We thank this reviewer for giving us the opportunity to clarify in what aspects the ‘sequence code of trajectories’ theory of grid cell firing differs from the classic attractor network models, in particular the continuous attractor network (CAN) model. First of all, the CAN model is a mechanistic model of grid cell firing that is specifically designed to simulate spatially periodic firing of grid cells in response to velocity inputs. In contrast, the sequence code of trajectories theory of grid cell firing resembles a normative model showing that grid cells emerge from performing a specific function. However, in contrast to previous normative models, the sequence code of trajectories model grounds the emergence of grid cell firing in a mathematical proof and both geometric reasoning and intuition. The proof demonstrates that the emergence of grid cells is the only solution to coding for trajectories using cell sequences. The sequence code of trajectories model of grid cell firing is agnostic about the neural mechanisms that implements the sequence code in a population of neurons. One plausible implementation of the sequence code of trajectories is in fact a CAN. In fact, the sequence code of trajectories theory predicts conformal isometry in the CAN, i.e., a trajectory in neural space is proportional to a trajectory of an animal in physical space. However, other mechanistic implementations are possible. We have clarified how the sequence code of trajectories theory of grid cells relates to the mechanistic CAN models of grid cells. 

      We added the following text to the Discussion section:

      “While the sequence code of trajectories-model of grid cell firing is agnostic about the neural mechanisms that implements the sequence code, one plausible implementation is a continuous attractor network (McNaughton et al., 2006; Burak and Fiete, 2009). Interestingly, a sequence code of trajectories begets conformal isometry in the attractor network, i.e., a trajectory in neural space is proportional to a trajectory of an animal in physical space.”

      It was not clear to me why their theory predicted the field size/spacing ratio or the orientation of the grid pattern to the wall. 

      We thank this reviewer for bringing to our attention that we lacked a proper explanation for why the sequence code of trajectories theory predicts the field size/spacing ration in grid maps. We have modified/added the following text to the Results section of the manuscript to clarify this point:

      “Because the sequence code of trajectories model of grid cell firing implies a dense packing of firing fields, the spacing between two adjacent grid fields must change linearly with a change in field size. It follows that the ratio between grid spacing and field size is fixed. When using the distance between the centers of two adjacent grid fields to measure grid spacing and a diameter-like metric to measure grid field size, we can compute the ratio of grid spacing to grid field size as √7≈2.65 (see Methods).”

      We are also grateful for this reviewer’s correctly pointing out that the explanation as to why the sequence code of trajectories predicts a rotation of the grid pattern relative to a set of parallel walls in a rectangular environment. We have now made explicit the underlying premise that a sequence of firing fields from multiple grid cells are aligned in parallel to a nearby wall of the environment. We cite additional experimental evidence supporting this premise. Concretely, we quote Stensola and Moser summarizing results reported in (Stensola et al. 2015): “A surprising observation, however, was that modules typically assumed one of only four distinct orientation configurations relative to the environment” (Stensola and Moser, 2016). Importantly, all of the four distinct orientations show the characteristic angular rotation. Intriguingly, this is predicted by the sequence code of trajectories-model under the premise that a sequence of firing fields aligns with one of the geometric boundaries of the environment, as shown in Author response image 1 below.

      Author response image 1.

      Under the premise that a sequence of firing fields aligns with one of the geometric boundaries (walls) of a square arena, there are precisely four possible distinct configurations of orientations. This is precisely what has been observed in experiments (Stensola et al., 2015; Stensola and Moser, 2016).

      We added clarifying language to the Results section: “Under the premise that a sequence of firing fields aligns with one of the geometric boundaries of the environment, the sequence code model explains that the grid pattern typically assume one of only four distinct orientation configurations relative to the environment41,46. Concretely, the four orientation configurations arise when one row of grid fields aligns with one of the two sets of parallel walls in a rectangular environment, and each arrangement can result in two distinct orientations (Figure 3B).”

      I don't understand how repeated advancement of one unit to the next, as shown in Figure 4E, would cause the change in grid spacing near a reward. 

      In familiar environments, spatial firing fields of place cells in hippocampal CA1 and CA3 tend to shift backwards with experience (Mehta et al., 2000; Lee et al., 2004; Roth et al., 2012; Geiller et al., 2017; Dong et al., 2021). This implies that the center of place fields move closer to each other. A potential mechanism has been suggested, namely NMDA receptor-dependent longterm synaptic plasticity (Ekstrom et al., 2001). When we apply the same principle observed for place fields on a linear track to grid fields anchored to a reward zone, grid fields will “gravitate” towards the reward side. A similar idea has been presented by (Ginosar et al., 2023) who use the analogy of reward locations as “black holes”. In contrast to (Ginosar et al., 2023), who we cite multiple times, our idea unifies observations on place cells and grid cells in 1-D and 2-D environments and suggests a potential mechanism. We changed the wording in the revised manuscript and clarified the underlying premises.

      I don't follow how this theory predicts the finding that the grid pattern expands with novelty. The authors propose that this occurs because the animals are not paying attention to fine spatial details, and thus only need a low-resolution spatial map that eventually turns into a higher-resolution one. But it's not clear to me why one needs to invoke the sequence coding hypothesis to make this point. 

      We agree with this reviewer that this point needs clarification. The sequence code model adds explanatory power to the hypothesis that the grid pattern in a novel environment reflects a lowresolution mapping of space or spatial trajectories because it directly links spatial resolution to both field size and spacing of a grid map. Concretely, the spatial resolution of the trajectory code is equivalent to the spacing between two adjacent spatial fields, and the spatial resolution is directly proportional to the grid spacing and field size. If one did not evoke the sequence coding hypothesis, one would need to explain how and why both spacing and field size are related to the spatial resolution of the grid map. Lastly, as written in the manuscript text, we point out that, while the experimentally observed expansion of grid maps is consistent with the sequence code of trajectory, it is not predicted by the theory without making further assumption. 

      The last section, which describes that the grid spacing of different modules is scaled by the square root of 2, says that this is predicted if the resolution is doubled or halved. I am not sure if this is specifically a prediction of the sequence coding theory the authors put forth though since it's unclear why the resolution should be doubled or halved across modules (as opposed to changed by another factor). 

      We agree with reviewer #2 that the exact value of the scaling factor is not predicted by the sequence coding theory. E.g., the sequence code theory does not explain why the spatial resolution doesn’t change by a factor 3 or 1.5 (resulting in changes in grid spacing by square root of 3 or square root of 1.5, respectively). We have changed the wording to reflect this important point. We further clarified in the revised manuscript that future work on multiscale representations using modules of grid cells needs to show why changing the spatial resolution across modules by a factor of 2 is optimal. Interestingly, a scale ratio of 2 is commonly used in computer vision, specifically in the context of mipmapping and Gaussian pyramids, to render images across different scales. Literature in the computer vision field describes why a scaling factor of 2 and the use of Gaussian filter kernels (compare with Gaussian firing fields) is useful in allowing a smooth and balanced transition between successive levels of an image pyramid (Burt and Adelson, 1983; Lindeberg, 2008). Briefly, larger factors (like 3) could result in excessive loss of detail between levels, while smaller factors (like 1.5) would not reduce the image size enough to justify additional levels of computation (that would come with the structural cost of having more grid cell modules in the brain). We have clarified these points in the Discussion section.

      Reviewer #3 (Public Review): 

      The manuscript presents an intriguing explanation for why grid cell firing fields do not lie on a lattice whose axes aligned to the walls of a square arena. This observation, by itself, merits the manuscript's dissemination to the eLife's audience. 

      We thank this reviewer for their positive assessment.

      The presentation is quirky (but keep the quirkiness!). 

      We kept the quirkiness.

      But let me recast the problem presented by the authors as one of combinatorics. Given repeating, spatially separated firing fields across cells, one obtains temporal sequences of grid cells firing. Label these cells by integers from $[n]$. Any two cells firing in succession should uniquely identify one of six directions (from the hexagonal lattice) in which the agent is currently moving. 

      Now, take the symmetric group $\Sigma$ of cyclic permutations on $n$ elements.  We ask whether there are cyclic permutations of $[n]$ such that 

      \left(\pi_{i+1} - \pi_i \right) \mod n \neq \pm 1 \mod n, \; \forall i. 

      So, for instance, $(4,2,3,1)$ would not be counted as a valid permutation of $(1,2,3,4)$, as $(2,3)$ and $(1,4)$ are adjacent. 

      Furthermore, given $[n]$, are there two distinct cyclic permutations such that {\em no} adjacencies are preserved when considering any pair of permutations (among the triple of the original ordered sequence and the two permutations)? In other words, if we consider the permutation required to take the first permutation into the second, that permutation should not preserve any adjacencies. 

      {\bf Key question}: is there any difference between the solution to the combinatorics problem sketched above and the result in the manuscript? Specifically, the text argues that for $n=7$ there is only {\em one} solution. 

      Ideally, one would strive to obtain a closed-form solution for the number of such permutations as a function of $n$.  

      This is a great question! We currently have a student working on describing all possible arrangements of firing fields (essentially labelings of the hexagonal lattice) that satisfy the axioms in 2D, and we expect that results on the number of such arrangements will come out of his work. We plan to publish those results separately, possibly targeting a more mathematical audience.   

      The argument above appears to only apply in the case that every row (and every diagonal) contains all of the elements 1,...,n. However, when n is not prime, there are often arrangements where rows and/or diagonals do not contain every element from 1,...,n. For example, some admissible patterns with 9 neurons have a repeat length of 3 in all directions (horizontally and both diagonals). As a result the construction listed here will not give a full count of all possible arrangements. 

      Recommendations for the authors:  

      Reviewer #1 (Recommendations For The Authors): 

      I think the concise style of mathematical proof is both a curse and a blessing. While it delivers the message, I think the fluency and readability of the mathematical proof could be improved with longer paragraphs and some more editing. 

      We have added some clarifications in the text that we hope improve the readability.

      Reviewer #3 (Recommendations For The Authors): 

      A minor qualm I have with the nomenclature: 

      On page 7: 

      “To prove this statement, suppose that row A consists of units $1, \dots , k$ repeating in this order. Then any row that contains any unit from $1, \dots, k$ must contain the full repeat $1, \dots , k$ by axiom 1. So any row containing any unit from $1,\dots , k$ is a translation of row A, and any unit that does not contain them is disjoint from row A.”

      The last use of `unit' at the end of this paragraph instead of `row' is confusing. Technically, the authors have given themselves license to use this term by defining a unit to be “either to a single cell or a cell assembly”. Yet modern algebra tends to use `unit' as meaning a ring element that has an inverse.  

      We have renamed “unit” to “element” to avoid confusion with the terminology in modern algebra.

    1. Author response:

      The following is the authors’ response to the original reviews.

      Editor’s summary:

      This paper by Castello-Serrano et al. addresses the role of lipid rafts in trafficking in the secretory pathway. By performing carefully controlled experiments with synthetic membrane proteins derived from the transmembrane region of LAT, the authors describe, model and quantify the importance of transmembrane domains in the kinetics of trafficking of a protein through the cell. Their data suggest affinity for ordered domains influences the kinetics of exit from the Golgi. Additional microscopy data suggest that lipid-driven partitioning might segregate Golgi membranes into domains. However, the relationship between the partitioning of the synthetic membrane proteins into ordered domains visualised ex vivo in GPMVs, and the domains in the TGN, remain at best correlative. Additional experiments that relate to the existence and nature of domains at the TGN are necessary to provide a direct connection between the phase partitioning capability of the transmembrane regions of membrane proteins and the sorting potential of this phenomenon.

      The authors have used the RUSH system to study the traffic of model secretory proteins containing single-pass transmembrane domains that confer defined affinities for liquid ordered (lo) phases in Giant Plasma Membrane derived Vesicles (GPMVs), out of the ER and Golgi. A native protein termed LAT partitioned into these lo-domains, unlike a synthetic model protein termed LAT-allL, which had a substituted transmembrane domain. The authors experiments provide support for the idea that ER exit relies on motifs in the cytosolic tails, but that accelerated Golgi exit is correlated with lo domain partitioning.

      Additional experiments provided evidence for segregation of Golgi membranes into coexisting lipid-driven domains that potentially concentrate different proteins. Their inference is that lipid rafts play an important role in Golgi exit. While this is an attractive idea, the experiments described in this manuscript do not provide a convincing argument one way or the other. It does however revive the discussion about the relationship between the potential for phase partitioning and its influence on membrane traffic.

      We thank the editors and scientific reviewers for thorough evaluation of our manuscript and for positive feedback. While we agree that our experimental findings present a correlation between trafficking rates and raft affinity, in our view, the synthetic, minimal nature of the transmembrane protein constructs in question makes a strong argument for involvement of membrane domains in their trafficking. These constructs have no known sorting determinants and are unlikely to interact directly with trafficking proteins in cells, since they contain almost no extramembrane amino acids. Yet, the LATTMD traffics through Golgi similarly to the full-length LAT protein, but quite different from mutants with lower raft phase affinity. We suggest that these observations can be best rationalized by involvement of raft domains in the trafficking fates and rates of these constructs, providing strong evidence (beyond a simple correlation) for the existence and relevance of such domains.

      We have substantially revised the manuscript to address all reviewer comments, including several new experiments and analyses. These revisions have substantially improved the manuscript without changing any of the core conclusions and we are pleased to have this version considered as the “version of record” in eLife.

      Below is our point-by-point response to all reviewer comments.

      ER exit:

      The experiments conducted to identify an ER exit motif in the C-terminal domain of LAT are straightforward and convincing. This is also consistent with available literature. The authors should comment on whether the conservation of the putative COPII association motif (detailed in Fig. 2A) is significantly higher than that of other parts of the C-terminal domain.

      Thank you for this suggestion, this information has now been included as Supp Fig 2B. While there are other wellconserved residues of the LAT C-terminus, many regions have relatively low conservation. In contrast, the essential residues of the COPII association motif (P148 and A150) are completely conserved across in LAT across all species analyzed.

      One cause of concern is that addition of a short cytoplasmic domain from LAT is sufficient to drive ER exit, and in its absence the synthetic constructs are all very slow. However, the argument presented that specific lo phase partitioning behaviour of the TMDs do not have a significant effect on exit from the ER is a little confusing. This is related to the choice of the allL-TMD as the 'non-lo domain' partitioning comparator. Previous data has shown that longer TMDs (23+) promote ER export (eg. Munro 91, Munro 95, Sharpe 2005). The mechanism for this is not, to my knowledge, known. One could postulate that it has something to do with the very subject of this manuscript- lipid phase partitioning. If this is the case, then a TMD length of 22 might be a poor choice of comparison. A TMD 17 Ls' long would be a more appropriate 'non-raft' cargo. It would be interesting to see a couple of experiments with a cargo like this.

      The basis for the claim that raft affinity has relatively minor influence on ER exit kinetics, especially in comparison to the effect of the putative COPII interaction motif, is in Fig 1G. We do observe some differences between constructs and they may be related to raft affinity, however we considered these relatively minor compared to the nearly 4-fold increase in ER efflux induced by COPII motifs.

      We have modified the wording in the manuscript to avoid the impression that we have ruled out an effect of raft affinity of ER exit.

      We believe that our observations are broadly consistent with those of Munro and colleagues. In both their work and ours, long TMDs were able to exit the ER. In our experiments, this was true for several proteins with long TMDs, either as fulllength or as TMD-only versions (see Fig 1G). We intentionally did not measure shorter synthetic TMDs because these would not have been comparable with the raft-preferring variants, which all require relatively long TMDs, as demonstrated in our previous work1,2. Thus, because our manuscript does not make any claims about the influence of TMD length on trafficking, we did not feel that experiments with shorter non-raft constructs would substantively influence our conclusions.

      However, to address reviewer interest, we did complete one set of experiments to test the effect of shortening the TMD on ER exit. We truncated the native LAT TMD by removing 6 residues from the C-terminal end of the TMD (LAT-TMDd6aa). This construct exited the ER similarly to all others we measured, revealing that for this set of constructs, short TMDs did not accumulate in the ER. ER exit of the truncated variant was slightly slower than the full-length LAT-TMD, but somewhat faster than the allL-TMD. These effects are consistent with our previous measurements with showed that this shortened construct has slightly lower raft phase partitioning than the LAT-TMD but higher than allL2. While these are interesting observations, a more thorough exploration of the effect of TMD length would be required to make any strong conclusion, so we did not include these data in the final manuscript.

      Author response image 1.

      Golgi exit:

      For the LAT constructs, the kinetics of Golgi exit as shown in Fig. 3B are surprisingly slow. About half of the protein Remains in the Golgi at 1 h after biotin addition. Most secretory cargo proteins would have almost completely exited the Golgi by that time, as illustrated by VSVG in Fig. S3. There is a concern that LAT may have some tendency to linger in the Golgi, presumably due to a factor independent of the transmembrane domain, and therefore cannot be viewed as a good model protein. For kinetic modeling in particular, the existence of such an additional factor would be far from ideal. A valuable control would be to examine the Golgi exit kinetics of at least one additional secretory cargo.

      We disagree that LAT is an unusual protein with respect to Golgi efflux kinetics. In our experiments, Golgi efflux of VSVG was similar to full-length LAT (t1/2 ~ 45 min), and both of these were similar to previously reported values3. Especially for the truncated (i.e. TMD) constructs, it is very unlikely that some factor independent of their TMDs affects Golgi exit, as they contain almost no amino acids outside the membrane-embedded TMD.

      Practically, it has proven somewhat challenging to produce functional RUSH-Golgi constructs. We attempted the experiment suggested by the reviewer by constructing SBP-tagged versions of several model cargo proteins, but all failed to trap in the Golgi. We speculate that the Golgin84 hook is much more sensitive to the location of the SBP on the cargo, being an integral membrane protein rather than the lumenal KDEL-streptavidin hook. This limitation can likely be overcome by engineering the cargo, but we did not feel that another control cargo protein was essential for the conclusions we presented, thus we did not pursue this direction further.

      Comments about the trafficking model

      (1) In Figure 1E, the export of LAT-TMD from the ER is fitted to a single-exponential fit that the authors say is "well described". This is unclear and there is perhaps something more complex going on. It appears that there is an initial lag phase and then similar kinetics after that - perhaps the authors can comment on this?

      This is a good observation. This effect is explainable by the mechanics of the measurement: in Figs 1 and 2, we measure not ‘fraction of protein in ER’ but ‘fraction of cells positive for ER fluorescence’. This is because the very slow ER exit of the TMD-only constructs present a major challenge for live-cell imaging, so ER exit was quantified on a population level, by fixing cells at various time points after biotin addition and quantifying the fraction of cells with observable ER localization (rather than tracking a single cell over time).

      For fitting to the kinetic model (which attempts to describe ‘fraction in ER/Golgi’) we re-measured all constructs by livecell imaging (see Supp Fig 5) to directly quantify relative construct abundance in the ER or Golgi. These data did not have the plateau in Fig 1E, suggesting that this is an artifact of counting “ER positive cells” which would be expected to have a longer lag than “fraction of protein in ER”. Notably however, t1/2 measured by both methods was similar, suggesting that the population measurement agrees well with single-cell live imaging.

      We have included all these explanations and caveats in the manuscript. We have also changed the wording from “well described” to “reasonably approximated”.

      (2) The model for Golgi sorting is also complicated and controversial, and while the authors' intention to not overinterpreting their data in this regard must be respected, this data is in support of the two-phase Golgi export model (Patterson et al PMID:18555781).

      The reviewers are correct, our observations and model are consistent with Patterson et al and it was a major oversight that a reference to this foundational work was not included. We have now added a discussion regarding the “two phase model” of Patterson and Lippincott-Schwartz.

      Furthermore contrary to the statement in lines 200-202, the kinetics of VSVG exit from the Golgi (Fig. S3) are roughly linear and so are NOT consistent with the previous report by Hirschberg et al.

      Regarding kinetics of VSVG, our intention was to claim that the timescale of VSVG efflux from the Golgi was similar to previously reported in Hirschberg, i.e. t1/2 roughly between 30-60 minutes. We have clarified this in the text. Minor differences in the details between our observations and Hirschberg are likely attributable to temperature, as those measurements were done at 32°C for the tsVSVG mutant.

      Moreover, the kinetics of LAT export from the Golgi (Fig. 3B) appear quite different, more closely approximating exponential decay of the signal. These points should be described accurately and discussed.

      Regarding linear versus exponential fits, we agree that the reality of Golgi sorting and efflux is far more complicated than accounted for by either the phenomenological curve fitting in Figs 1-3 or the modeling in Fig 4. In addition to the possibility of lateral domains within Golgi stacks, there is transport between stacks, retrograde traffic, etc. The fits in Figs 1-3 are not intended to model specifics of transport, but rather to be phenomenological descriptors that allowed us to describe efflux kinetics with one parameter (i.e. t1/2). In contrast, the more refined kinetic modeling presented in Figure 4 is designed to test a mechanistic hypothesis (i.e. coexisting membrane domains in Golgi) and describes well the key features of the trafficking data.

      Relationship between membrane traffic and domain partitioning:

      (1) Phase segregation in the GPMV is dictated by thermodynamics given its composition and the measurement temperature (at low temperatures 4degC). However at physiological temperatures (32-37degC) at which membrane trafficking is taking place these GPMVs are not phase separated. Hence it is difficult to argue that a sorting mechanism based solely on the partitioning of the synthetic LAT-TMD constructs into lo domains detected at low temperatures in GPMVs provide a basis (or its lack) for the differential kinetics of traffic of out of the Golgi (or ER). The mechanism in a living cell to form any lipid based sorting platforms naturally requires further elaboration, and by definition cannot resemble the lo domains generated in GPMVs at low temperatures.

      We thank the reviewers for bringing up this important point. GPMVs are a useful tool because they allow direct, quantitative measurements of protein partitioning between coexisting ordered and disordered phases in complex, cell-derived membranes. However, we entirely agree, that GPMVs do not fully represent the native organization of the living cell plasma membrane and we have previously discussed some of the relevant differences4,5. Despite these caveats, many studies have supported the cellular relevance of phase separation in GPMVs and the partitioning of proteins to raft domains therein 6-9. Most notably, elegant experiments from several independent labs have shown that fluorescent lipid analogs that partition to Lo domains in GPMVs also show distinct diffusive behaviors in live cells 6,7, strongly suggesting the presence of nanoscopic Lo domains in live cells. Similarly, our recent collaborative work with the lab of Sarah Veatch showed excellent agreement between raft preference in GPMVs and protein organization in living immune cells imaged by super-resolution microscopy10. Further, several labs6,7, including ours11, have reported nice correlations between raft partitioning in GPMVs and detergent resistance, which is a classical (though controversial) assay for raft association.

      Based on these points, we feel that GPMVs are a useful tool for quantifying protein preference for ordered (raft) membrane domains and that this preference is a useful proxy for the raft-associated behavior of these probes in living cells. We propose that this approach allows us to overcome a major reason for the historical controversy surrounding the raft field: nonquantitative and unreliable methodologies that prevented consistent definition of which proteins are supposed to be present in lipid rafts and why. Our work directly addresses this limitation by relating quantitative raft affinity measurements in a biological membrane with a relevant and measurable cellular outcome, specifically inter-organelle trafficking rates.

      Addressing the point about phase transition temperatures in GPMVs: this is the temperature at which macroscopic domains are observed. Based on physical models of phase separation, it has been proposed that macroscopic phase separation at lower temperatures is consistent sub-microscopic, nanoscale domains at higher temperatures8,12. These smaller domains can potentially be stabilized / functionalized by protein-protein interactions in cells13 that may not be present in GPMVs (e.g. because of lack of ATP).

      (2) The lipid compositions of each of these membranes - PM, ER and Golgi are drastically different. Each is likely to phase separate at different phase transition temperatures (if at all). The transition temperature is probably even lower for Golgi and the ER membranes compared to the PM. Hence, if the reported compositions of these compartments are to be taken at face value, the propensity to form phase separated domains at a physiological temperature will be very low. Are ordered domains even formed at the Golgi at physiological temperatures?

      It is a good point that the membrane compositions and the resulting physical properties (including any potential phase behavior) will be very different in the PM, ER, and Golgi. Whether ordered domains are present in any of these membranes in living cells remains difficult to directly visualize, especially for non-PM membranes which are not easily accessible by probes, are nanoscopic, and have complex morphologies. However, the fact that raft-preferring probes / proteins share some trafficking characteristics, while very similar non-raft mutants behave differently argues that raft affinity plays a role in subcellular traffic.

      (3) The hypothesis of 'lipid rafts' is a very specific idea, related to functional segregation, and the underlying basis for domain formation has been also hotly debated. In this article the authors conflate thermodynamic phase separation mechanisms with the potential formation of functional sorting domains, further adding to the confusion in the literature. To conclude that this segregation is indeed based on lipid environments of varying degrees of lipid order, it would probably be best to look at the heterogeneity of the various membranes directly using probes designed to measure lipid packing, and then look for colocalization of domains of different cargo with these domains.

      This is a very good suggestion, and a direction we are currently following. Unfortunately, due to the dynamic nature and small size of putative lateral membrane domains, combined with the interior of a cell being filled with lipophilic environments that overlay each other, directly imaging domains in organellar membranes with lipid packing probes remains extremely difficult with current technology (or at least available to us). We argue that the TMD probes used in this manuscript are a reasonable alternative, as they are fluorescent probes with validated selectivity for membrane compartments with different physical properties.

      Ultimately, the features of membrane domains suggested by a variety of techniques – i.e. nanometric, dynamic, relatively similar in composition to the surrounding membrane, potentially diverse/heterogeneous – make them inherently difficult to microscopically visualize. This is one reason why we believe studies like ours, which use a natural model system to directly quantify raft-associated behaviors and relate them to cellular effects (in our case, protein sorting), are a useful direction for this field.

      We believe we have been careful in our manuscript to avoid confusing language surrounding lipid rafts, phase separation, etc. Our experiments clearly show that mammalian membranes have the capacity to phase separate, that some proteins preferentially interact with more ordered domains, and that this preference is related to the subcellular trafficking fates and rates of these proteins. We have edited the manuscript to emphasize these claims and avoid the historical controversies and confusions.

      (4) In the super-resolution experiments (by SIM- where the enhancement of resolution is around two fold or less compared to optical), the authors are able to discern a segregation of the two types of Golgi-resident cargo that have different preferences for the lo-domains in GPMVs. It should be noted that TMD-allL and the LATallL end up in the late endosome after exit of the Golgi. Previous work from the Bonafacino laboratory (PMID: 28978644) has shown that proteins (such as M6PR) destined to go to the late endosome bud from a different part of the Golgi in vesicular carriers, while those that are destined for the cell surface first (including TfR) bud with tubular vesicular carriers. Thus at the resolution depicted in Fig 5, the segregation seen by the authors could be due to an alternative explanation, that these molecules are present in different areas of the Golgi for reasons different from phase partitioning. The relatively high colocalization of TfR with the GPI probe in Fig 5E is consistent with this explanation. TfR and GPI prefer different domains in the GPMV assays yet they show a high degree of colocalization and also traffic to the cell surface.

      This is a good point. Even at microscopic resolutions beyond the optical diffraction limit, we cannot make any strong claims that the segregation we observe is due to lateral lipid domains and not several reasonable alternatives, including separation between cisternae (rather than within), cargo vesicles moving between cisternae, or lateral domains that are mediated by protein assemblies rather than lipids. We have explicitly included this point in the Discussion: “Our SIM imaging suggests segregation of raft from nonraft cargo in the Golgi shortly (5 min) after RUSH release (Fig 5B), but at this level of resolution, we can only report reduced colocalization, not intra-Golgi protein distributions. Moreover, segregation within a Golgi cisterna would be very difficult to distinguish from cargo moving between cisternae at different rates or exiting via Golgi-proximal vesicles.”

      We have also added a similar caveat in the Results section of the manuscript: “These observations support the hypothesis that proteins can segregate in Golgi based on their affinity for distinct membrane domains; however, it is important to emphasize that this segregation does not necessarily imply lateral lipid-driven domains within a Golgi cisterna. Reasonable alternative possibilities include separation between cisternae (rather than within), cargo vesicles moving between cisternae, or lateral domains that are mediated by protein assemblies rather than lipids.”

      Finally, while probes with allL TMD do eventually end up in late endosomes (consistent with the Bonifacino lab’s findings which we include), they do so while initially transiting the PM2,11.

      Minor concerns:

      (1) Generally, the quantitation is high quality from difficult experimental data. Although a lot appears to be manual, it appears appropriately performed and interpreted. There are some claims that are made based on this quantitation, however, where there are no statistics performed. For example, figure 1B. Any quantitation with an accompanying conclusion should be subject to a statistical test. I think the quality of the model fits- this is particularly important.

      We appreciate the thoughtful feedback, the quantifications and fits were not trivial, but we believe important. We have added statistical significance to Figure 1B and others where it was missing.

      (2) Modulation of lipid levels in Fig 4E shows a significant change for the trafficking rate for the LAT-TMD construct and a not so significant change for all-TMD construct. However, these data are not convincing and appear to depend on a singular data point that appears to lower the mean value. In general, the experiment with the MZA inhibitor (Fig. 4D-F) is hard to interpret because cells will likely be sick after inhibition of sphingolipid and cholesterol synthesis. Moreover, the difference in effects for LAT-TMD and allL-TMD is marginal.

      We disagree with this interpretation. Fig 4E shows the average of three experiments and demonstrates clearly that the inhibitors change the Golgi efflux rate of LAT-TMD but not allL-TMD. This is summarized in the t1/2 quantifications of Fig 4F, which show a statistically significant change for LAT-TMD but not allL-TMD. This is not an effect of a singular data point, but rather the trend across the dataset.

      Further, the inhibitor conditions were tuned carefully to avoid cells becoming “sick”: at higher concentrations, cells did adopt unusual morphologies and began to detach from the plates. We pursued only lower concentrations, which cells survived for at least 48 hrs and without major morphological changes.

      (3) Line 173: 146-AAPSA-152 should read either 146-AAPSA-150 or 146-AAPSAPA-152, depending on what the authors intended.

      Thanks for the careful reading, we intended the former and it has been fixed.

      (4) What is the actual statistical significance in Fig. 3C and Fig. 3E? There is a single asterisk in each panel of the figure but two asterisks in the legend.

      Apologies, a single asterisk representing p<0.05 was intended. It has been fixed.

      (5) The code used to calculate the model. is not accessible. It is standard practice to host well-annotated code on Github or similar, and it would be good to have this publicly available.

      We have deposited the code on a public repository (doi: 10.5281/zenodo. 10478607) and added a note to the Methods.

      (1) Lorent, J. H. et al. Structural determinants and func7onal consequences of protein affinity for membrane ra=s. Nature communica/ons 8, 1219 (2017).PMC5663905

      (2) Diaz-Rohrer, B. B., Levental, K. R., Simons, K. & Levental, I. Membrane ra= associa7on is a determinant of plasma membrane localiza7on. Proc Natl Acad Sci U S A 111, 8500-8505 (2014).PMC4060687

      (3) Hirschberg, K. et al. Kine7c analysis of secretory protein traffic and characteriza7on of golgi to plasma membrane transport intermediates in living cells. J Cell Biol 143, 1485-1503 (1998).PMC2132993

      (4) Levental, K. R. & Levental, I. Giant plasma membrane vesicles: models for understanding membrane organiza7on. Current topics in membranes 75, 25-57 (2015)

      (5) Sezgin, E. et al. Elucida7ng membrane structure and protein behavior using giant plasma membrane vesicles. Nat Protoc 7, 1042-1051 (2012)

      (6) Komura, N. et al. Ra=-based interac7ons of gangliosides with a GPI-anchored receptor. Nat Chem Biol 12, 402-410 (2016)

      (7) Kinoshita, M. et al. Ra=-based sphingomyelin interac7ons revealed by new fluorescent sphingomyelin analogs. J Cell Biol 216, 1183-1204 (2017).PMC5379944

      (8) Stone, M. B., Shelby, S. A., Nunez, M. F., Wisser, K. & Veatch, S. L. Protein sor7ng by lipid phase-like domains supports emergent signaling func7on in B lymphocyte plasma membranes. eLife 6 (2017).PMC5373823

      (9) Machta, B. B. et al. Condi7ons that Stabilize Membrane Domains Also Antagonize n-Alcohol Anesthesia. Biophys J 111, 537-545 (2016)

      (10) Shelby, S. A., Castello-Serrano, I., Wisser, I., Levental, I. & S., V. Membrane phase separa7on drives protein organiza7on at BCR clusters. Nat Chem Biol in press (2023)

      (11) Diaz-Rohrer, B. et al. Rab3 mediates a pathway for endocy7c sor7ng and plasma membrane recycling of ordered microdomains Proc Natl Acad Sci U S A 120, e2207461120 (2023)

      (12) Veatch, S. L. et al. Cri7cal fluctua7ons in plasma membrane vesicles. ACS Chem Biol 3, 287-293 (2008)

      (13) Wang, H. Y. et al. Coupling of protein condensates to ordered lipid domains determines func7onal membrane organiza7on. Science advances 9, eadf6205 (2023).PMC10132753

    1. Author response:

      The following is the authors’ response to the original reviews.

      Reviewer #1 (Public Review):

      (1) The main hypothesis/conclusion is summarized in the abstract: "Our study presents an intriguing model of cilia length regulation via controlling IFT speed through the modulation of the size of the IFT complex." The data clearly document the remarkable correlation between IFT velocity and ciliary length in the different cells/tissues/organs analyzed. The experimental test of this idea, i.e., the knock-down of GFP-IFT88, further supports the conclusion but needs to be interpreted more carefully. While IFT particle size and train velocity were reduced in the IFT88 morphants, the number of IFT particles is even more decreased. Thus, the contributions of the reduction in train size and velocity to ciliary length are, in my opinion, not unambiguous. Also, the concept that larger trains move faster, likely because they dock more motors and/or better coordinating kinesin-2 and that faster IFT causes cilia to be longer, is to my knowledge, not further supported by observations in other systems (see below).

      Thank you for your comments. We agree with the reviewer that the final section on IFT train size, velocity, and ciliary length regulation requires additional evidence. The purpose of the knockdown experiments was to investigate the potential relationship between IFT speed and IFT train size. We hypothesize that a deficiency in IFT88 proteins may disrupt the regular assembly of IFT particles, leading to the formation of shorter IFT trains. Indeed, we observed a shorter IFT particles and slight reduction in the transport speed of IFT particles in the morphants. Certainly, it would be more convincing to distinguish these IFT trains through ultrastructural analysis. However, with current techniques, performing such analysis on the zebrafish model will be very difficult due to the limited sample size. In the revised version, we have tempered the conclusions in these sections, as suggested by other reviewers as well.

      (2) I think the manuscript would be strengthened if the IFT frequency would also be analyzed in the five types of cilia. This could be done based on the existing kymographs from the spinning disk videos. As mentioned above, transport frequency in addition to train size and velocity is an important part of estimating the total number of IFT particles, which bind the actual cargoes, entering/moving in cilia.

      Thank you. We have analyzed the entry frequency of IFT in five types of cilia, both anterior and posterior. The analysis indicates that longer cilia also exhibit a higher frequency of fluorescent particles entering the cilia. These results are presented in Figure 3J.

      (3) Here, the variation in IFT velocity in cilia of different lengths within one species is documented - the results document a remarkable correlation between IFT velocity and ciliary length. These data need to be compared to observations from the literature. For example, the velocity of IFT in the quite long (~ 100 um) olfactory cilia of mice is similar to that observed in the rather short cilia of fibroblasts (~0.6 um/s). In Chlamydomonas, IFT velocity is not different in long flagella mutants compared to controls. Probably data are also available for C. elegans or other systems. Discussing these data would provide a broader perspective on the applicability of the model outside of zebrafish.

      Thank you for your suggestions. We believe the most significant novelty of our manuscript is the discovery that IFT velocities are closely related to cilia length in an in vivo model system. Our data suggest that longer cilia may require faster IFT transport to maintain their stable length, powered by larger IFT trains. We did observe substantial variability in IFT velocities across different studies. For example, anterograde IFT transport ranges from 0.2 µm/s in mouse olfactory neurons (Williams et al, 2014) to 0.8 µm/s in 293T cells (See et al, 2016) and 0.4 µm/s in IMCD-3 cells (Broekhuis et al, 2014). Even in NIH-3T3 cells, two studies report significant differences, despite using the same IFT reporters: 0.3 µm/s versus 0.9 µm/s (Kunova Bosakova et al, 2018; Luo et al, 2017). These findings suggest that cell types and culture conditions can influence IFT velocities in vitro, which may not accurately represent in vivo conditions. Interestingly, research on mouse olfactory neurons showed a strong correlation between anterograde and retrograde IFT velocities. Additionally, IFT velocity is closely related to the cell types within the olfactory neuron population, consistent with our results (Williams et al., 2014). 

      Reviewer #2 (Public Review):

      Summary:

      In this study, the authors study intraflagellar transport (IFT) in cilia of diverse organs in zebrafish. They elucidate that IFT88-GFP (an IFT-B core complex protein) can substitute for endogenous IFT88 in promoting ciliogenesis and use it as a reporter to visualize IFT dynamics in living zebrafish embryos. They observe striking differences in cilia lengths and velocity of IFT trains in different cilia types, with smaller cilia lengths correlating with lower IFT speed. They generate several mutants and show that disrupting the function of different kinesin-2 motors and BBSome or altering post-translational modifications of tubulin does not have a significant impact on IFT velocity. They however observe that when the amount of IFT88 is reduced it impacts the cilia length, IFT velocity as well as the number and size of IFT trains. They also show that the IFT train size is slightly smaller in one of the organs with shorter cilia (spinal cord). Based on their observations they propose that IFT velocity determines cilia length and go one step further to propose that IFT velocity is regulated by the size of IFT trains.

      Strengths:

      The main highlight of this study is the direct visualization of IFT dynamics in multiple organs of a living complex multi-cellular organism, zebrafish. The quality of the imaging is really good. Further, the authors have developed phenomenal resources to study IFT in zebrafish which would allow us to explore several mechanisms involved in IFT regulation in future studies. They make some interesting findings in mutants with disrupted function of kinesin-2, BBSome, and tubulin modifying enzymes which are interesting to compare with cilia studies in other model organisms. Also, their observation of a possible link between cilia length and IFT speed is potentially fascinating.

      Weaknesses:

      The manuscript as it stands, has several issues.

      (1) The study does not provide a qualitative description of cilia organization in different cell types, the cilia length variation within the same organ, and IFT dynamics. The methodology is also described minimally and must be detailed with more care such that similar studies can be done in other laboratories.

      Thank you for your comments. We found that cilia length is generally consistent within the same cell types we examined, including those in the pronephric duct, spinal cord, and epidermal cells. However, we observed variability in cilia length within ear crista cilia. Upon comparing IFT velocities, we found no differences among these cilia, further confirming our conclusion that IFT velocity is directly related to cell type rather than cilia length. These new results are presented in Figure S4 of the revised version.

      We apologize for the lack of methodological details in the original manuscript. Following the reviewer's suggestion, we have added a detailed description of the methods used to generate the transgenic line and to perform IFT velocity analysis. These details are included in Figure S2 and are thoroughly described in the methods section of the revised manuscript.

      (2) They provide remarkable new observations for all the mutants. However, discussion regarding what the findings imply and how these observations align (or contradict) with what has been observed in cilia studies in other organisms is incomprehensive.

      Thank you for this suggestion. We initially submitted this paper as a report, which have word limits. We believe the main finding of our work is that IFT velocity is directly associated with cell type, with longer cilia requiring higher velocities to maintain their length. This association of IFT velocity with cell type has also been observed in mouse olfactory neurons(Williams et al., 2014). We have included a discussion of our findings, along with related data published in other organisms, in the revised version.

      (3) The analysis of IFT velocities, the main parameter they compare between experiments, is not described at all. The IFT velocities appear variable in several kymographs (and movies) and are visually difficult to see in shorter cilia. It is unclear how they make sure that the velocity readout is robust. Perhaps, a more automated approach is necessary to obtain more precise velocity estimates.

      Thank you for these comments. To measure the IFT velocities, we first used ImageJ software to generate a kymograph, where moving particles appear as oblique lines. The velocity of these particles can be calculated based on the slope of the lines (Zhou et al, 2001). In the initial version, most of the lines were drawn manually. To eliminate potential artifacts, we also used KymographDirect software to automatically trace the particle paths. The velocities obtained with this method were similar to those calculated manually. These new data are now shown in Figure S2 B-D. For shorter cilia, we only used particles with clear moving paths for our calculations. In the revised version, we have included a detailed description of the velocity analysis methods.

      (4) They claim that IFT speeds are determined by the size of IFT trains, based on their observations in samples with a reduced amount of IFT88. If this was indeed the case, the velocity of a brighter IFT train (larger train) would be higher than the velocity of a dimmer IFT train (smaller train) within the same cilia. This is not apparent from the movies and such a correlation should be verified to make their claim stronger.

      Thank you for these excellent suggestions. We measured the particle size and fluorescence intensity of 3 dpf crista cilia using high-resolution images acquired with Abberior STEDYCON. The results showed a positive correlation between the two. These data have been added to the revised version in Figure 5I, which includes both control and ift88 morphant data.

      (5) They make an even larger claim that the cilia length (and IFT velocity) in different organs is different due to differences in the sizes of IFT trains. This is based on a marginal difference they observe between the cilia of crista and the spinal cord in immunofluorescence experiments (Figure 5C). Inferring that this minor difference is key to the striking difference in cilia length and IFT velocity is incorrect in my opinion.

      Impact:

      Overall, I think this work develops an exciting new multicellular model organism to study IFT mechanisms. Zebrafish is a vertebrate where we can perform genetic modifications with relative ease. This could be an ideal model to study not just the role of IFT in connection with ciliary function but also ciliopathies. Further, from an evolutionary perspective, it is fascinating to compare IFT mechanisms in zebrafish with unicellular protists like Chlamydomonas, simple multicellular organisms like C elegans, and primary mammalian cell cultures. Having said that, the underlying storyline of this study is flawed in my opinion and I would recommend the authors to report the striking findings and methodology in more detail while significantly toning down their proposed hypothesis on ciliary length regulation. Given the technological advancements made in this study, I think it is fine if it is a descriptive manuscript and doesn't necessarily need a breakthrough hypothesis based on preliminary evidence.

      Thanks for with these comments. We agree with this reviewer that more evidences are required to explain why IFT is transported faster in longer cilia. In the revised version, we have modified and softened this section, focusing primarily on the novel findings of IFT velocity differences between cilia of varying lengths.

      Reviewer #3 (Public Review):

      Summary:

      A known feature of cilia in vertebrates and many, if not all, invertebrates is the striking heterogeneity of their lengths among different cell types. The underlying mechanisms, however, remain largely elusive. In the manuscript, the authors addressed this question from the angle of intraflagellar transport (IFT), a cilia-specific bidirectional transportation machinery essential to biogenesis, homeostasis, and functions of cilia, by using zebrafish as a model organism. They conducted a series of experiments and proposed an interesting mechanism. Furthermore, they achieved in situ live imaging of IFT in zebrafish larvae, which is a technical advance in the field.

      Strengths:

      The authors initially demonstrated that ectopically expressed Ift88-GFP through a certain heatshock induction protocol fully sustained the normal development of mutant zebrafish that would otherwise be dead by 7 dpf due to the lack of this critical component of IFT-B complex.

      Accordingly, cilia formations were also fully restored in the tissues examined. By imaging the IFT using Ift88-GFP in the mutant fish as a marker, they unexpectedly found that both anterograde and retrograde velocities of IFT trains varied among cilia of different cell types and appeared to be positively correlated with the length of the cilia.

      For insights into the possible cause(s) of the heterogeneity in IFT velocities, the authors assessed the effects of IFT kinesin Kif3b and Kif17, BBSome, and glycylation or glutamylation of axonemal tubulin on IFT and excluded their contributions. They also used a cilia-localized ATP reporter to exclude the possibility of different ciliary ATP concentrations. When they compared the size of Ift88-GFP puncta in crista cilia, which are long, and spinal cord cilia, which are relatively short, by imaging with a cutting-edge super-resolution microscope, they noticed a positive correlation between the puncta size, which presumably reflected the size of IFT trains, and the length of the cilia.

      Finally, they investigated whether it is the size of IFT trains that dictates the ciliary length. They injected a low dose (0.5 ng/embryo) of ift88 MO and showed that, although such a dosage did not induce the body curvature of the zebrafish larvae, crista cilia were shorter and contained less Ift88-GFP puncta. The particle size was also reduced. These data collectively suggested mildly downregulated expression levels of Ift88-GFP. Surprisingly, they observed significant reductions in both retrograde and anterograde IFT velocities. Therefore, they proposed that longer IFT trains would facilitate faster IFT and result in longer cilia.

      Weaknesses:

      The current manuscript, however, contains serious flaws that markedly limit the credibility of major results and findings. Firstly, important experimental information is frequently missing, including (but not limited to) developmental stages of zebrafish larvae assayed (Figures 1, 3, and 5), how the embryos or larvae were treated to express Ift88-GFP (Figures 3-5), and descriptions on sample sizes and the number of independent experiments or larvae examined in statistical results (Figures 3-5, S3, S6). For instance, although Figure 1B appears to be the standard experimental scheme, the authors provided results from 30-hpf larvae (Figure 3) that, according to Figure 1B, are supposed to neither express Ift88-GFP nor be genotyped because both the first round of heat shock treatment and the genotyping were arranged at 48 hpf. Similarly, the results that ovl larvae containing Tg(hsp70l:ift88 GFP) (again, because the genotype is not disclosed in the manuscript, one can only deduce) display normal body curvature at 2 dpf after the injection of 0.5 ng of ift88 MO (Fig 5D) is quite confusing because the larvae should also have been negative for Ift88-GFP and thus displayed body curvature. Secondly, some inferences are more or less logically flawed. The authors tend to use negative results on specific assays to exclude all possibilities. For instance, the negative results in Figures 4A-B are not sufficient to "suggest that the variability in IFT speeds among different cilia cannot be attributed to the use of different motor proteins" because the authors have not checked dynein-2 and other IFT kinesins. In fact, in their previous publication (Zhao et al., 2012), the authors actually demonstrated that different IFT kinesins have different effects on ciliogenesis and ciliary length in different tissues. Furthermore, instead of also examining cilia affected by Kif3b or Kif17 mutation, they only examined crista cilia, which are not sensitive to the mutations. Similarly, their results in Figures 4C-G only excluded the importance of tubulin glycylation or glutamylation in IFT. Thirdly, the conclusive model is based on certain assumptions, e.g., constant IFT velocities in a given cell type. The authors, however, do not discuss other possibilities.

      Thank you for pointing out the flaws in our experiments. We apologize for any confusion caused by the lack of detail in our descriptions. Regarding Figure 2B, we want to clarify that it depicts the procedure for heat shock experiments conducted for the ovl mutants' rescue assay, not the experimental procedure for IFT imaging. In the revised version, we have included detailed methods on how to induce the expression of Ift88-GFP via heat shock and the subsequent image processing. The procedure for heat induction is also shown in Figure S2A. We have also added the sample sizes for each experiment and descriptions of the statistical tests used in the appropriate sections of the revised version.

      Regarding the comments on the relationship between IFT speed variability and motor proteins, we completely agree with the reviewer. We have revised our description of this part accordingly.

      Lastly, the results shown in Figure 5D are from a wild-type background, not ovl mutants. We aimed to demonstrate that a lower dose of ift88 morpholino (0.5 ng) can partially knock down Ift88, allowing embryos to maintain a generally normal body axis, while the cilia in the ear crista became significantly shorter.

      Recommendations for the authors:

      Reviewer #1 (Recommendations For The Authors):

      Minor

      (I recommend adding page numbers and probably line numbers. This makes commenting easier)

      We have added page numbers and line numbers in the revised manuscript.

      Intro: Furthermore, ultra-high-resolution microscopy showed a close association between cilia length in different organs and the size of IFT fluorescent particles, indicating the presence of larger IFT trains in longer cilia.

      This correlation is not that strong and data are only available for 2 types of cilia.

      Thanks. We have modified this part.

      P5) cilia (Fig. 1D) -> (Fig. S1)

      Thanks. We have corrected this.

      P5) "These movies provide a great opportunity to compare IFT across different cilia." Rewrite: "This approach allows one to determine the velocity and frequency based of IFT based on kymographs" or similar. 

      Thank you for your correction, we have changed it in the revised manuscript.

      This observation suggests that cargo and motor proteins are more effectively coordinated in transporting materials, resulting in increased IFT velocity-a novel regulatory mechanism governing IFT speed in vertebrate cilia.

      This is a somewhat cryptic phrase, rewrite?

      We have modified this sentence.

      P6 and elsewhere: "IFT in the absence of Kif17 or Bbs proteins" I wonder if it would be better to provide subheadings summarizing the main observation instead of descriptive titles. This includes the title of the manuscript.

      Thanks for this suggestion. We have changed the title of subheadings in the revised manuscript. We prefer to keep the current title of this manuscript, as we think this paper is mainly to describe IFT in different types of cilia. 

      Is it known whether IFT protein and motors are alternatively spliced in the various ciliated cells of zebrafish? In this context, is it known whether the cells express IFT proteins at different levels?

      We analyzed the transcript isoforms of several ciliary genes, including ift88, ift52, ift70, ift172, and kif3a. Most of these IFT genes possess only a single transcript isoform. The Kif3a motor proteins have two isoforms (long and short isoforms), however, the shorter isoform contains only the motor domain and is presumed to be nonfunctional for IFT. While we cannot completely rule out this possibility, we consider it unlikely that the variation in IFT speed is due to alternative splicing in ciliary tissues.

      P6) The relation between osm-3 and Kif17 needs to be introduced briefly.  

      Thank you for pointing this out. We have added it in the proper place of the revised manuscript.

      P6) "IFT was driven by kinesin or dynein motor proteins along the ciliary axoneme." "is driven"?

      Delete phrase and IFT to the next sentence?

      We have deleted this sentence.

      P7) "Moreover, the mutants were able to survive to adulthood and there is no difference in the fertility or sperm motility between mutants and control siblings, which is slightly different from those observed in mouse mutants(Gadadhar et al., 2021)." Could some of these data be shown? 

      Thanks for this suggestion. When crossed with wild-type females, all homozygous mutants showed no difference in fertility compared to controls. The percentage of fertilization rates in mutants was 90.5% (n = 7), which was similar to wild-type (87.2%, n = 7). We determined the trajectories of free-swimming sperm by high-speed video microscopy. The vast majority of sperm in ttll3 mutant, similar to wild-type sperm, swim almost entirely along a straight path, which is different from what was observed in the mouse mutant (where 86% of TTLL3-/-TTLL8-/- sperm rotate in situ). We assessed cilia motility in the pronephric ducts of 5dpf embryos using high-speed video microscopy. The ttll3 mutant exhibited a rhythmic sinusoidal wave pattern similar to the control, and there was no significant difference in ciliary beating frequency. These new data are now included in Figure S7C-H.

      P7) "which has been shown early to reduce" earlier

      We have changed it. Thanks.

      Maybe the authors could speculate how the cells ensure the assembly of larger/faster trains in certain cells. Are the relative expression levels known or worth exploring?

      Thank you for these suggestions. We believe that longer cilia may maintain larger IFT particle pools in the basal body region, facilitating the assembly of large IFT trains. The higher frequency of IFT injection in longer cilia further supports this hypothesis. It is likely that cells with longer cilia have higher expression levels of IFT proteins. However, due to the lack of proper antibodies for IFT proteins in zebrafish, it is currently unfeasible to compare this. This experiment is certainly worth investigating in the future. We have added this discussion in the revised manuscript.

      Reviewer #2 (Recommendations for The Authors):

      Here are detailed comments for the authors:

      (1) The authors need to describe their methodology of imaging and what they observe in much greater detail. How were the different cilia types organized? Approximately how many were observed in every organ? How were they oriented? Were there length variations between cilia in the same organ? While imaging, were individual cilium mostly lying in a single focal plane of imaging or the authors often performed z-scans over multiple planes. Velocity measurement is highly variable if individual cilia are spanning over a large volume, with only part of it in focus in single plane acquisition.

      Thank you for your comments. We apologize for the lack of details in the methodology. We have added a detailed description in the 'Materials and Methods' section and illustrated the experimental paradigm in Figure S2A of the revised manuscript. In most tissues we examined, the length of cilia was relatively uniform, except in the crista. The cilia in the crista were significantly longer, with lengths varying between 5 and 30 μm, compared to those in other tissues. We categorized the cilia lengths in the crista into three groups at intervals of 10 μm and measured the anterograde and retrograde velocities of IFT in each group. The results, shown in Figure S4, revealed no significant difference in IFT velocity among the different cilia lengths within the same tissue.  Regarding the imaging, all IFT movies were captured in a single focal plane. In most cases, we did not observe significant velocity variability within the same cilium.

      (2) It is very difficult to directly observe the large differences in IFT velocity from the kymographs, especially in the case of shorter cilia and retrograde motion in them. The quality of the example kymographs could be improved and more zoomed in several cases.

      Thank you for this suggestion. We have modified this.

      (3) The authors do not describe at all, how velocity analysis was done on the kymographs? Were lines drawn manually on the kymographs? From the movies and the kymographs it is visible that the IFT motion is often variable and sometimes gets stuck. How did the authors determine the velocities of such trains? A single slope through the entire train or part of the train? Were they consistent with this? Such variable motion is not so easy to discern in the case of really short cilia. The authors could use a more automatic way of extracting velocities from kymographs using tools such as kymodirect or kymobutler. Keeping in mind that IFT velocity is the main parameter studied in this work, it is important that the analysis is robust.

      We apologize for the previous lack of detailed description. We utilized ImageJ software to generate kymographs, where particles appear as lines. For a moving particle, this line appears oblique. We manually drew lines on the kymographs, and the velocity of particles was calculated based on the slope (Zhou et al., 2001). We only analyzed particles that tracked the full length of the cilia. Following the reviewer's suggestions, we also used the automatic software KymographDirect to calculate the velocity of IFT particles. The results were similar to those calculated using the previous method. These new data are now shown in Figure S2B-D. For shorter cilia, we only used particles with clear moving paths for our calculations. In the revised version, we have included a detailed description of the velocity analysis methods.

      (4) In line with the previous point, as visible from the kymographs the velocity is significantly slower near the transition zone. Did the authors make sure they are not including the region around the transition zone while measuring the IFT velocity, especially in the case of shorter cilia?

      Thank you for the comment. In the revised manuscript, we automatically extracted the path of particle using KymographDirect software. Quantification of each particle's velocity versus position in crista reveals that anterograde IFT proceeds from the base to the tip at a relatively constant speed, whereas retrograde IFT undergoes a slightly acceleration process when returning to the base (Fig. S2E). This finding differs from observations in C. elegans, which dynein-2 first accelerating and then decelerating back to 1.2 μm/s adjacent to the ciliary base (Yi et al, 2017). We believe it is very unlikely that the slow IFT velocity is due to the calculation of IFT only in the transition zone of shorter cilia.

      (5) There are several fascinating findings in this work that the authors do not discuss properly. Firstly, do the authors have a hypothesis as to why IFT speeds are so radically different in different cilia types, given that they are driven by the same motor proteins and have the same ATP levels? They make a big claim in this paper that IFT train sizes correlate with train velocities. IFT trains have a highly ordered structure with regular binding sites for motor proteins. So, a smaller train would have a proportional number of motors attached to them. Why (and how) are the motors moving trains so slowly in some cilia and not in others? If there is no clear answer, the authors must put forward the open question with greater clarity.

      Thank you for the comment. We hypothesize that if multiple motors drive the movement of cargoes synergistically, it could increase the speed of IFT transport. An example supporting this hypothesis is the principle of multiple-unit high-speed trains, which use multiple motors in each individual car to achieve high speeds. Of course, this is just one hypothesis, and we cannot exclude other possibilities, such as the use of different adaptors in different cell types. We have revised our conclusions accordingly in the updated manuscript.

      (6) They find that IFT speeds do not change in kif17 mutants. Are the cilia length also similar (does not appear to be the case in Figure 4 and Figure S3)? Cilia length needs to be quantified. Further, they mention that in C elegans, heterotrimeric kinesin-2 and homodimeric kinesin-2 coordinate IFT. However, from several previous studies, we know that in Chlamydomonas and in mammalian cilia IFT is driven primarily by heterotrimeric kinesin-2 with no evidence that homodimeric kinesin-2 is linked with driving IFT. It appears to be the same in zebrafish. This is an interesting finding and needs to be discussed far more comprehensively.

      Thank you for your comments. We have previously shown that the number and length of crista cilia were grossly normal in kif17 mutants (Zhao et al, 2012). The length of crista cilia displayed slight variability even in wild-type larvae. We quantified the length of cilia in both the crista and neuromast within different mutants, and our analysis revealed no significant difference (see Author response image 1). We agree with the reviewer that Kif17 may play a minor role in driving IFT in cilia. However, previous studies have shown that KIF17 exhibits robust, processive particle movement in both the anterograde and retrograde directions along the entire olfactory sensory neuron cilia in mice. This suggests that, although not essential, KIF17 may also be involved in IFT (Williams et al., 2014). We have added more discussion about Kif17 and heterotrimeric kinesin in the appropriate section of the revised manuscript.

      Author response image 1.

      Statistical significance is based on Kruskal-Wallis statistic, Dunn's multiple comparisons test. n.s., not significant, p>0.05.

      (7) Again, they find that IFT speeds do not change in BBS-4 mutants. I have the same comment about the cilia length as for kif17 mutants. Further, the discussion for this finding is lacking. The authors mention that IFT is disrupted in BBSome mutants of C elegans. Is this the case in other organisms as well? Structural studies on IFT trains reveal that BBSomes are not part of the core structure, while other studies reveal that BBSomes are not essential for IFT. So perhaps the results here are not too surprising.

      We agree with the reviewer that BBSome is possibly not essential for IFT in most cilia. However, in the cilia of olfactory sensory neurons, BBSome is involved in IFT in both mice and nematodes (Ou et al, 2005; Williams et al., 2014). We have added more discussion about BBSome in the appropriate section of the revised manuscript.

      (8) No change in IFT velocities in kif3b mutants is rather surprising. The authors suggest that Kif3C homodimerizes to carry out IFT in the absence of Kif3B. Even if that is the case, the individual homodimer constituents of heterotrimeric kinesin-2 have been shown in previous studies to have different motor properties when homodimerized artificially. Why is IFT not affected in these mutants? This should be discussed. Also, the cilia lengths should be quantified.

      We think the presence of the Kif3A/Kif3C/KAP3 trimeric kinesin may substitute for the Kif3A/Kif3B/KAP3 motors in kif3b mutants, which show normal length of cristae cilia. The Kif3A/Kif3C/KAP3 trimeric kinesin may have similar transport speeds as the Kif3A/Kif3B/KAP3 motors. We did not propose that the Kif3C homodimer can drive the cargoes alone. We apologize for this misunderstanding. Additionally, we have reevaluated the IFT velocities among different lengths of cristae cilia and found no difference between longer and shorter cilia within the same cell types.

      (9) The findings with tubulin modifications should also be discussed in comparison to what has been observed in other organisms.

      We have added further discussion about this result in the revised manuscript.

      (10) The authors find that IFT velocity is lower in ift88 morphants. They also find that the cilia length is shorter (in which cilia type?). Immunofluorescence experiments show that the IFT particle number and size are lower in the ift88 morphants. How many organisms did they look at for this data? What is the experimental variability in intensity measurements in immunofluorescence experiments? Wouldn't the authors expect much higher variability in ift88 morphants (between individual organisms) due to different amounts of IFT88 than for wildtype?

      Thank you for your comments. We apologize for the lack of information regarding the number of organisms observed in Figure 5. These numbers have been added to the figure legends in the revised manuscript. When a low dose of ift88 morpholino was injected, we observed significant shortening of cilia in the ear crista, along with reduced IFT speed. We measured the fluorescence intensity of different IFT particles and found a positive correlation between IFT particle size and fluorescence intensity (Fig 5I). Moreover, the variability of cilia length in cristae is slightly higher in ift88 morphants. These new data have been included in the revised version.

      (11) From their observations they make the claim that IFT velocity is directly proportional to IFT train size. Now within every cilium, IFT trains have large size variations, given the variable intensities for different IFT trains. The authors themselves show that they resolve far more trains when imaging with STED (possibly because they are able to visualize the smaller trains). Is the IFT velocity within the same cilium directly correlated with the intensity of the train, both for wildtype and ift88 morphants? That is the most direct way the authors can test that their hypothesis is true. Higher intensity (larger train size) results in faster velocity. From a qualitative look at their movies, I do not see any strong evidence for that.

      Thank you for your comments. We have measured the particle size and fluorescence intensity of 3dpf crista cilia using high-resolution images acquired with Abberior STEDYCON. The results, shown in Figure 5I, demonstrate a positive correlation between particle size and fluorescence intensity.

      (12) Are the sizes of both anterograde and retrograde trains lower in ift88 morphants? It's not clear from the data. It should be clearly stated that the authors speculate this and this is not directly evident from the data.

      Because the size of IFT fluorescence particles is based on immunostaining results, not live imaging, we cannot determine whether they are anterograde or retrograde IFT particles.

      Therefore, we can only speculate that possibly both anterograde and retrograde trains are reduced in ift88 morphants.

      (13) The biggest claim in this paper is that the cilia lengths in different organs are different due to differences in IFT train sizes. This is based on highly preliminary data shown in Figure 5C (how many organisms did they measure?). The difference is marginal and the dataset for spinal cord cilia is really small. The internal variability within the same cilia type is larger than the difference. How is this tiny difference resulting in such a large difference in IFT speeds? I believe their conclusions based on this data are incorrect.

      From our results, we believe that IFT velocity is related to cell types rather than the length of cilia (Fig. S4), which has also been mentioned in previous studies (Williams et al., 2014).  We agree with the reviewer that the evidence for faster IFT speed due to larger train size is not very solid. We have accordingly softened our conclusion and mentioned other possibilities in the revised version.

      Minor comments:

      (1) The authors only mention the number of IFT particles for their data. They should provide the number of cilia and the number of organisms as well.

      Thank you for your suggestion. We added the number of cilia and organisms next to the number of particles in Figure 3, Figure S2-S5 and Table S1 of the revised manuscript.

      (2) Cilia and flagella are similar structurally but not the same. The authors should change the following sentence: In contrast to the localization of most organelles within cells, cilia (also known as flagellar) are microtubule-based structures that extend from the cell surface, facilitating a more straightforward quantification of their size.  

      Thank you for the detailed review. We have changed it in our revised manuscript. 

      (3) The authors should provide references here. For example, Chlamydomonas has two flagella with lengths ranging from 10 to 14 μm, while sensory cilia in C. elegans vary from approximately 1.5 μm to 7.5 μm. In most mammalian cells, the primary cilium typically measures between 3 and 10 μm.  

      We have added it in our revised manuscript. 

      (4) They should mention ovl mutants are IFT88 mutants when they introduce it in the main text.

      We have added it in our revised manuscript. 

      (5) Correct the grammar here: The velocity of IFT within different cilia also seems unchanged (Figure 4F, Movie S9, Table S1).  

      We have changed it. 

      (6) Correct the grammar here: Similarly, the IFT speeds also exhibited only slight changes in ccp5 morphants, which decreased the deglutamylase activities of Ccp5 and resulted in a hyperglutamylated tubulin

      We have changed it. 

      Reviewer #3 (Recommendations For The Authors):

      Introduction:

      1st paragraph, "flagellar" should be "flagella"; 2nd paragraph, "result a wide range of" should be "result in a...".  

      We have changed it. 

      Results and discussion:

      "...certain specialized cell types, including olfactory epithelia and pronephric duct, ...": olfactory epithelia and pronephric duct are tissues, not cells.  

      "...the GFP fluorescence of the transgene was prominently enriched in the cilia (Fig 1D)" : Fig 2D?  

      "The velocity of IFT within different cilia was also seems unchanged (Fig. 4 F, Movie S9, Table S1)": "was" and "seems" cannot be used together.  

      "...driven by b-actin2 promotor":    -actin2? 

      "...each dynein motor protein might propel multiple IFT complexes": The "protein" should be deleted.  

      Thanks. We have corrected all of these mistakes.  

      Figures:

      Figure 1: Dyes and antibodies used other than the anti-acetylated tubulin antibody should mentioned. The developmental stages of zebrafish used for the imaging are mostly missing.  

      Thanks. In the revised version, we have updated the figure legends to include descriptions of the antibodies, developmental stages, as well as N numbers.

      Figure 2B: What "hphs" means should be explained somewhere.  

      Thanks. We have added full name for these abbreviations.  

      Figures 3A-E: For clarity, the cilia whose IFT kymographs are shown should be marked. "Representative particle traces are marked with white lines in panels D and E" (legend): they are actually black lines. The authors should also clearly disclose the developmental stages of zebrafish used for the imaging.  

      Thank you for your comments. In the revised manuscript, the cilia used to generate the kymograph are marked by yellow arrows. We have updated the legend to change "white" to "black." Additionally, we have included the developmental stages of zebrafish used for imaging in Figure 3A.

      Figures 3G-K: The authors used quantification results from 4-dpf larvae and 30-hpf embryos for comparisons. Nevertheless, according to their experimental scheme in Figure 2B, 30-hpf embryos were not subjected to heat-shock treatment and genotyping. How could they express Ift88-GFP for the imaging? How could the authors choose larvae of the right genotypes? In addition, even if the authors heat-shocked them in time but forgot to mention, there are issues that need to be clarified experimentally and/or through citations, at least through discussions. Firstly, at 30 hpf, those motile cilia are probably still elongating. If this is the case, their final lengths would be longer than those presented (H; the authors need to disclose whether the lengths were measured from ciliary Ift88-GFP or another marker). In other words, the correlation with IFT velocities (H and I) might no longer exist when mature cilia were measured. Similarly, cilia undergo gradual disassembly during the cell cycle. Epidermal cells at 30-hpf are likely proliferating actively, and the average length of their cilia (H) would be shorter than that measured from quiescent epidermal cells in later stages.

      Thank you for these comments. First, we want to clarify that Figure 2B depicts the procedure for heat shock experiments conducted for the ovl mutants' rescue assay, not the experimental procedure for IFT imaging. We visualized IFT in five types of cilia using Tg (hsp70l: ift88-GFP) embryos without the ovl mutant background. In the revised manuscript, we have provided a detailed description of embryo treatment in the 'Materials and Methods' section and illustrated the experimental paradigm in Figure S2A. 

      Regarding the ciliary length differences between different developmental stages, we quantified cilia length in epidermal cells at 30 hpf versus 4 dpf, and in pronephric duct cilia at 30 hpf versus 48 hpf. Our analysis found no significant difference in length between earlier and later stages. Additionally, IFT velocities were comparable between these stages. These findings suggest that slower IFT velocities may not be attributed to the selection of different embryonic stages. Furthermore, we demonstrated that longer and shorter cilia maintain similar IFT velocities in crista cilia, indicating that elongated cilia within the same cell type exhibit comparable IFT velocities. These new results are presented in Figures S4 and S5 in the revised version.

      Secondly, do IFT velocities differ between elongating and mature cilia or remain relatively constant for a given cell type? The authors apparently take the latter for granted without even discussing the possibility of the former. In addition, whether the quantification results were from cilia of one or multiple fish, an important parameter to reflect the reproducibility, and sample sizes for the length data are not disclosed. The lack of descriptions on sample sizes and the number of independent experiments or larvae examined are actually common for statistical results in this manuscript.

      Thank you for your comments. We apologize for omitting the basic description of sample sizes and the number of cilia analyzed. We have addressed these issues in the revised manuscript. The length of 4dpf Crista cilia is variable, with longer cilia reaching up to 30 µm and shorter cilia measuring only around 5 µm within the same crista. We categorized the cilia length of Crista into three groups at intervals of 10 µm and measured anterograde and retrograde velocities of IFT in each group. The results revealed no significant difference in IFT velocity among elongating and mature cilia within crista. These supplementary data are now included in Figure S4.

      Figures 4A-B: When mutating neither Kif17 nor Kif3b affected the IFT of crista cilia, the data unlikely "suggest that the variability in IFT speeds among different cilia cannot be attributed to the use of different motor proteins". In fact, in the cited publication (Zhao et al., 2012), the authors used the same and additional mutants (Kif3c and Kif3cl) to demonstrate that different IFT-related kinesin motors have different effects on ciliogenesis and ciliary length in different tissues, results actually implying tissue-specific contributions of different kinesin motors to IFT. Furthermore, although likely only cytoplasmic dynein-2 is involved in the retrograde IFT, the authors cannot exclude the possibility that different combinations or isoforms of its many subunits and regulators contribute to the velocity regulation. Therefore, the authors need to reconsider their wording. This reviewer would suggest that the authors examine the IFT status of cilia that were previously reported to be shortened in the Kif3b mutant to see whether the correlation between ciliary length and IFT velocities still stands. This would actually be a critical assay to assess whether the proposed correlation is only a coincidence or indeed has a certain causality.

      Thank you for your comments. The shortened cilia observed in Kif3b mutants may be attributed to the presence of maternal Kif3b proteins, making it challenging to exclude the involvement of Kif3b motor. Regarding the relationship between IFT speed variability and motor proteins, we agree with the reviewer that we cannot entirely dismiss the possibility of different motors or adaptors being involved. We have revised our description of this aspect accordingly.

      Figures 4C-G: Similarly, when the authors found that tubulin glycylation or glutamylation has little effect on IFT, they cannot use these observations to exclude possible influences of other types of tubulin modifications on IFT. They should only stick to their observations.

      Yes, we agree. We have changed the description in the revised manuscript.

      Figure 5:

      A-C: When the authors only compared immotile cilia of crista with motile cilia of the spinal cord, it is hard to say whether the difference in particle size is correlated with ciliary length or motility. Cilia from more tissues should be included to strengthen their point, especially when the authors want to make this point the central one.

      D: The authors showed that ovl larvae containing Tg(hsp70l:ift88 GFP) (as they do not indicate the genotype, this reviewer can only deduce) display normal body curvature at 2 dpf after the injection of 0.5 ng of ift88 MO. Such a result, however, is quite confusing. According to their experimental scheme in Figure 2B, these larvae were not subjected to heat shock induction for Ift88-GFP. Do ovl larvae containing Tg(hsp70l:ift88 GFP) naturally display normal body curvature at 2 dpf? 

      Thank you for your comments. Due to technical limitations, comparing IFT particle size across different cilia using STED is challenging. We agree with this reviewer that the evidence supporting this aspect is relatively weak. Accordingly, we have modified and softened our conclusion in the revised version.

      Regarding the injection of ift88 morpholino, we want to clarify that we are injecting it into wildtype embryos, not oval mutants. The lower dose of ift88 morpholino (0.5ng) partially knocked down Ift88, allowing embryos to maintain a grossly normal body axis while resulting in shorter cilia in the ear crista.

      E: The authors need to indicate the developmental stage of the larvae examined. One piece of missing data is global expression levels of both endogenous (maternal) Ift88 and exogenous

      Ift88-GFP in zebrafish larvae that are either uninjected, 8-ng-ift88 MO-injected, or 0.5-ng-ift88 MO-injected, preferably at multiple time points up to 3 dpf. The results will clarify (1) the total levels of Ift88 following time; (2) the extent of downregulation the MO injections achieved at different developmental stages; and importantly (3) whether the low MO dosage (0. 5 ng) indeed allowed a persistent downregulation to affect IFT trains at 3 dpf, a time the authors made the assays for Figures 5F-J to reach the model (K). It will be great to include wild-type larvae for comparison.

      Thank you for these valuable suggestions. The ift88 morpholino (MO) was designed to block the splicing of ift88 transcripts and has been used in multiple studies. This morpholino specifically blocks the expression of endogenous ift88, while the expression of the Ift88-GFP transgene remains unaffected. It would be beneficial to titrate the expression level of Ift88 in the morphants at different stages. Unfortunately, we do not have access to a zebrafish Ift88 antibody. We assessed the effects of a lower amount of MO based on our observation that the fish maintained a normal body axis while exhibiting shorter cilia. Ideally, the amount of Ift88 should be lower in the morphants, considering the presence of ciliogenesis defects. We have included additional comments regarding this limitation in the revised version.

      Movies:

      Movies 1-5: Elapsed time is not provided. Furthermore, cilia in the pronephric duct and spinal cord are known to beat rapidly. Their motilities, however, appear to be largely compromised in Movies 3 and 4. Although the quantification results in Fig 3G imply that the authors imaged 30hpf embryos for such cilia, there is no statement on real conditions.

      Thank you for your comments. We apologize for missing elapsed time in our movies. We have addressed this issue in the revised manuscript. Motile cilia are difficult to image due to their fast beating. To immobilize the moving cilia and enable the capture of IFT movement within the cilia, we gently press the embryo with a round cover glass to inhibit the beating of cilia. Data from each embryo were collected within 5 minutes to avoid the impact of embryo death on the results. We have added detail description in the 'Materials and Methods' section.

      Materials:

      The sequence of morpholino oligonucleotide against ift88 is missing.  

      We have added the sequence of ift88 morpholino in the revised manuscript.

      References:

      Important references are missing, including (1) the paper by Leventea et al., 2016 (PMID: 27263414), which shows cilia morphologies in various zebrafish tissues with more detailed descriptions of tissue anatomies and experimental techniques; (2) papers documenting that dynein motors "move faster than Kinesin motors" in IFT of C. reinhardtii and C. elegans cilia; and (3) the paper by Li et al., 2020 (PMID: 33112235), in which the authors constructed a hybrid IFT kinesin to markedly reduced anterograde IFT velocity (~ 2.8 fold) and IFT injection rate in C. reinhardtii cilia and found only a mild reduction (~15%) in ciliary length. This paper is important because it is a pioneer one that elegantly investigated the relationship between IFT velocity and ciliary length. The findings, however, do not necessarily contradict the current manuscript due to differences in, e.g., model organisms and methodology.

      Thank you for the detailed review, we have cited these literatures in the proper place of the revised manuscript.

      Reference

      Broekhuis JR, Verhey KJ, Jansen G (2014) Regulation of cilium length and intraflagellar transport by the RCK-kinases ICK and MOK in renal epithelial cells. PLoS One 9: e108470

      Kunova Bosakova M, Varecha M, Hampl M, Duran I, Nita A, Buchtova M, Dosedelova H, Machat R, Xie Y, Ni Z et al (2018) Regulation of ciliary function by fibroblast growth factor signaling identifies FGFR3-related disorders achondroplasia and thanatophoric dysplasia as ciliopathies. Hum Mol Genet 27: 1093-1105

      Luo W, Ruba A, Takao D, Zweifel LP, Lim RYH, Verhey KJ, Yang W (2017) Axonemal Lumen Dominates Cytosolic Protein Diffusion inside the Primary Cilium. Sci Rep 7: 15793 Ou G, Blacque OE, Snow JJ, Leroux MR, Scholey JM (2005) Functional coordination of intraflagellar transport motors. Nature 436: 583-587

      See SK, Hoogendoorn S, Chung AH, Ye F, Steinman JB, Sakata-Kato T, Miller RM, Cupido T, Zalyte R, Carter AP et al (2016) Cytoplasmic Dynein Antagonists with Improved Potency and Isoform Selectivity. ACS Chem Biol 11: 53-60

      Williams CL, McIntyre JC, Norris SR, Jenkins PM, Zhang L, Pei Q, Verhey K, Martens JR (2014) Direct evidence for BBSome-associated intraflagellar transport reveals distinct properties of native mammalian cilia. Nat Commun 5: 5813

      Yi P, Li WJ, Dong MQ, Ou G (2017) Dynein-Driven Retrograde Intraflagellar Transport Is Triphasic in C. elegans Sensory Cilia. Curr Biol 27: 1448-1461 e1447

      Zhao C, Omori Y, Brodowska K, Kovach P, Malicki J (2012) Kinesin-2 family in vertebrate ciliogenesis. Proceedings of the National Academy of Sciences 109: 2388 - 2393

      Zhou HM, Brust-Mascher I, Scholey JM (2001) Direct visualization of the movement of the monomeric axonal transport motor UNC-104 along neuronal processes in living Caenorhabditis elegans. J Neurosci 21: 3749-3755

    1. Author response:

      The following is the authors’ response to the original reviews

      Reviewer #1:

      Summary:

      The experiment is interesting and well executed and describes in high detail fish behaviour in thermally stratified waters. The evidence is strong but the experimental design cannot distinguish between temperature and vertical position of the treatments.

      Strengths:

      High statistical power, solid quantification of behaviour.

      Weaknesses:

      A major issue with the experimental design is the vertical component of the experiment. Many thermal preference and avoidance experiments are run using horizontal division in shuttlebox systems or in annular choice flumes. These remove the vertical stratification component so that hot and cold can be compared equally, without the vertical layering as a confounding factor. The method chosen, with its vertical stratification, is inherently unable to control for this effect because warm water is always above, and cold water is always below. This complicates the interpretations and makes firm conclusions about thermal behaviour difficult.

      We highly appreciate this evaluation and have addressed the reviewer’s specific comments below.

      The sentence "Further, the metabolic performance (and thus functions including growth, reproduction, and locomotion) of ectotherms takes the form of a bell-shaped curve as a function of temperature6, peaking within a range of optimal temperatures (the 'preferendum') and going to zero at lower and upper temperature limits7." contains several over-simplifications and misconceptions:

      (1) Thermal performance curves are never bell-shaped.

      (2) The optimum for various traits often shows different TPCs.

      (3) The preferendum rarely lines up with the thermal optimum for various trait TPCs.

      (4) Performance for various traits rarely reaches zero at upper or lower limits, instead they can reach zero at less extreme temperatures (e.g. growth) or maintain high function all the way up to and sometimes beyond thermal limits (e.g. aerobic scope, heart rate).

      We highly appreciate this input. We have replaced that sentence with: L69-71: “Because temperature influences the rates of most physiological processes, rapid warming or cooling can affect fish performance traits, including metabolic rates, swimming ability, and thermal tolerance (Jutfelt et al. 2024).”

      The use of adaptation instead of acclimation is confusing. Adaptation should be reserved for evolutionary change. This is an issue in several parts of the manuscript.

      Thanks for this input, we have replaced the word adapt with acclimate in two instances: L79 and L398.

      It is not true that "very few quantitative studies of thermotaxis have been conducted in fish". There exists an extensive literature on thermal preference and avoidance in fish that the manuscript downplays.

      Thanks a lot for this input. We understand that thermal preference is ultimately driven by mechanistic responses to thermal gradients, and that thermotaxis and thermokinesis are the two mechanisms used by fish to navigate heterothermal environments. Our study and analysis are focused on understanding these mechanisms in vertically stratified conditions, not to understand thermal preferences per se. We have modified our text to clarify this aspect. Our literature review was focused on the behavioral mechanisms and our understanding is that the establishment of thermal preferences has a different goal compared to understanding how fish respond to rapid changes in water temperature. We have deleted that sentence and replaced it by (L107-110): “While the thermal preference of fish is a well-established field of research, very few quantitative studies of the behavioral mechanisms allowing fish to seek their preferendum (i.e. thermotaxis) have been conducted in fish.”

      (Methods) It is unclear why the blue dye was used in all experiments. The fish can see the differently coloured water layer and that may have affected their choices. Five control trials without dye were run but finding no difference there could also be due to low statistical power.

      We appreciate this comment. The blue dye was used to visualize the precise location of the thermal interface and was therefore necessary in all experiments (see Methods section ‘Visualization and evolution of the thermal interface’). We acknowledge that fish can perceive the colored water layer, but since the dye concentration and resulting color intensity were consistent across all treatments, we do not see how it could have acted as a confounding variable. While we recognize the possibility of some behavioral influence from the dye, the clear behavioral differences across treatments indicate that it was not a determining factor. To emphasize this we have added the following to the manuscript (L701-703): “Furthermore, because the dye concentration and resulting color intensity were consistent across all treatments, the dye did not act as a confounding variable in our statistical comparisons.”

      Regarding statistical power, our control experiment without dye (N = 16 fish, 4 replicates; see Fig. S34 and S35) provides sufficient statistical power to assess whether the dye influenced behavior. The reviewer indicated that the high statistical power was a strength of the paper, which aligns with our view that our study design enables robust statistical comparisons. It seems contradictory that statistical power is a concern for the control trials, given that our main experiments were conducted with a similar sample size. Indeed, the number of replicates used is consistent with similar studies and balances statistical rigor with the ethical goal of reducing the number of animals used in experimentation. To emphasize this, we have added the following to the manuscript (L865-868): “The number of replicates used in this study reflects a balance between statistical rigor and the ethical imperative to minimize the use of animals in experimentation. Regarding statistical power, our design (five replicates with groups of four fish each) is consistent with similar studies and represents an adequate sample size.”

      A major issue with the experimental design is the vertical component of the experiment. Many thermal preference and avoidance experiments are run using horizontal division in shuttlebox systems or in annular choice flumes. These remove the vertical stratification component so that hot and cold can be compared equally, without the vertical layering as a confounding factor. The method chosen, with its vertical stratification, is inherently unable to control for this effect because warm water is always above, and cold water is always below. This complicates the interpretations and makes firm conclusions about thermal behaviour difficult. This issue should be thoroughly discussed.

      Thank you very much for this comment. We revised the manuscript accordingly, to clearly indicate that our goal was to assess the response of fish to vertically thermally stratified water, a scenario that occurs frequently in nature. We have added the following paragraph the discussion (L523-530): “However, a generalization of our observations to horizontally oriented thermal gradients remains elusive. Our results are inherently tied to the vertical stratification created in our experiments. As warm water was always positioned above and cold water below, we could not control for the effect of vertical position (i.e., we could not do cold over warm layer experiments). This limits our ability to directly compare our findings to those obtained from horizontally oriented thermal gradients. On the other hand, the case we addressed is of direct environmental relevance, as natural waters often experience vertical thermal stratification.”

      It is unclear why the authors assume an "optimal temperature" (undefined for which trait) of 12°C for brown trout parr, and why they assume the preference temperature would match that "optimal" temperature. The thermal biology for any fish species is more complex than a single perfect temperature, with various traits showing differing optima and often a mismatch with the preferred temperature. The literature suggests brown trout growth optima between 13 and 16°C, and preference temperature has even been suggested to be as high as 21°C. In light of this, the authors' conclusion that brown trout avoid cold and don't avoid warm water is possibly misguided. It is possible that the brown trout had a preference temperature higher than 12°C, which should be acknowledged and discussed.

      This is indeed a very important aspect, which was partly (but indeed not fully) already addressed in the discussion. To reflect these considerations, we have expanded the existing paragraph in the discussion (additions are in yellow). (L422 - L439): “We conclude from the behavior of fish when warmer water was available that their acute thermal preferendum exceeded 12 °C, departing from the acclimation temperature we had chosen based on the thermal preferendum for trout reported in literature[33]. Indeed, the thermal biology for any fish species is more complex than a single, static thermal preferendum: Many internal and external factors, such as hypoxia, satiation, time of day, and life stage[5], can influence the temperature preference of fish. For example, the level of satiation can have an impact because when fish are well fed, their growth rate increases with body temperature as metabolic performance increases[40]. This modifies the preferred temperature, as observed in Bear Lake sculpin (Cottus extensus) that ascend into warmer water after feeding to stimulate digestion and thereby achieve a three-fold higher growth rate[41]. In contrast, field studies with adult fish have observed movement from warm to cold water in summer[42,43], allowing fish to lower their metabolic rate, likely in effort to conserve energy[2,44]. We propose that the behavior of trout parr upon exposure to warmer water in our experiments served to achieve a higher body temperature to ultimately increase growth rate, which is critical for this life stage[45,46]. Indeed, growth experiments on brown trout populations have shown that optimal growth temperatures can range between 15 and 19 °C, depending on the stream of origin[46].”

      The figures are unnecessarily complex and introduce a long list of abbreviations and Greek characters for no apparent reason. There are many simpler ways for showing the results so unclear why they are so opaque.

      We appreciate the reviewer’s feedback and agree on the importance of clarity, however (in the absence of specific suggestions) we did not make changes to the figures or the use of Greek characters (which align with convention), as we believe they effectively convey the results. We highlight that the data themselves are very rich (multiple fish, multiple phases, multiple treatments, etc.) and we wanted to convey this richness in a compact and transparent manner.

      Reviewer #2:

      This paper investigates an interesting question: how do fish react to and avoid thermal disturbances from the optimum that occur on fast timescales? Previous work has identified potential strategies for warm avoidance in fish on short timescales while strategies for cold avoidance are far more elusive. The work combines a clever experimental paradigm with careful analysis to show that trout parr avoid cold water by limiting excursions across a warm-cold thermal interface. While I found the paper interesting and convincing overall, there are a few omissions and choices in the presentation that limit interpretability and clarity.

      A main question concerns the thermal interface itself. The authors track this interface using a blue dye that is mixed in with either colder or warmer water before a gate is opened that leads to gravitational flow overlaying the two water temperatures. The dye likely allows to identify convective currents which could lead to rapid mixing of water temperatures. However, it is less clear whether it accurately reflects thermal diffusion. This is problematic as the authors identify upward turning behavior around the interface which appears to be the behavioral strategy for avoiding cold water but not warm water. Without knowing the extent of the gradient across the interface, it is hard to know what the fish are sensing. The authors appear to treat the interface as essentially static, leading them to the conclusion that turning away before the interface is reached is likely related to associative learning. However, thermal diffusion could very likely create a gradient across centimeters which is used as a cue by the fish to initiate the turn. In an ideal world, the authors would use a thermal camera to track the relationship between temperature and the dye interface. Absent that, the simulation that is mentioned in passing in the methods section should be discussed in detail in the main text, and results should be displayed in Figure 1. Error metrics on the parameters used in the simulation could then be used to identify turns in subsequent figures that likely are or aren't affected by a gradient formed across the interface.

      The authors assume that the thermal interface triggers the upward-turning behavior. However, an alternative explanation, which should be discussed, is that cold water increases the tendency for upward turns. This could be an adaptive strategy since for temperatures > 4C turning swimming upwards is likely a good strategy to reach warmer water.

      The paper currently also suffers from a lack of clarity which is largely created by figure organization. Four main and 38 supplemental figures are very unusual. I give some specific recommendations below but the authors should decide which data is truly supplemental, versus supporting important points made in the paper itself. There also appear to be supplemental figures that are never referenced in the text which makes traversing the supplements unnecessarily tedious.

      The N that was used as the basis for statistical tests and plots should be identified in the figures to improve interpretability. To improve rigor, the experimental procedures should be expanded.

      Specifically, the paper uses two thermal models which are not detailed at all in the methods section.

      We appreciate these crucial comments to our paper. We have addressed these points in detail below.

      As stated above, a characterization of the thermal interface is critical. Ideally via measurement or at least by expanding on the simulation.

      We appreciate the idea of using thermal cameras and, indeed, we had initially tried to use them. However, thermal cameras generally cannot see through plexiglass or glass-like material due to the way infrared radiation interacts with these materials. While thin plastics can transmit some infrared, thicker plastics and reflective materials like glass tend to block or reflect infrared light.

      We have attempted to better characterize the thermal interface thickness, namely the spatial extent of the thermal gradient over the time period of our experiments (20 min). Indeed, our simulations in the original SI were conducted precisely to estimate the thermal interface thickness, though based on thermal diffusion in still water, while turbulence generated by the moving gravity current can smear out the interface, particularly in the initial phase. To account for this in our in the reviewed manuscript, we adopted a phenomenological approach to estimate the initial increase in thickness of the thermal interface due to turbulence and present this refined simulation in our manuscript.

      Our analysis suggests that, rather than assuming an initial interface thickness of zero (as in the original version of the manuscript), the thermal diffusion simulations should begin with an initial thickness of 2.8 mm in TR1. To incorporate this adjustment, we set the initial interface thickness to 2.8 mm and ran the simulation forward for t = 20 min, assuming diffusion. This approach resulted in a final interface thickness ranging between 4 and 6 cm (see Fig. 29 in the Supplementary Information).

      To reflect this refinement, we have added a new paragraph (L717-758: "Characterization of the thermal gradient", to the Methods section. Additionally, we have updated Fig. S29 in the Supplementary Information and included an average (over time and across treatments) gradient thickness of 5 cm in Figs. 2 and 3 of the manuscript. The revised Figs. 2 and 3 now explicitly indicate the estimated vertical extent of the thermal gradient, with an extended caption detailing these changes.

      The simulation should be detailed in the methods so that its validity can be evaluated and ideally, it should involve curved interfaces as encountered in the experiment.

      To account for the effect of turbulence during the initial, inertia-dominated phase after the gate removal, we have provided a correction for the initial thickness of the interface (see the addition to the Methods section). Thank you for your suggestion regarding the incorporation of curved interfaces in the simulations. We believe that including curved interfaces in the simulations would not significantly affect the results. As shown in the manuscript, the interface is curved primarily during the initial phase of the process (first 2 min where the flow is inertia-dominated), which is currently not included in our data analysis (phase 1 begins 2 min after the gate removal).

      In that vein, distances from the interface rather than height above the interface should be reported for the fish.

      We acknowledge the reviewer’s suggestion to report distances from the interface rather than height above or below it. However, beyond the initial phase, we do not see a strong justification for using the orthogonal distance over the vertical distance, as the choice is inherently arbitrary (e.g., one could also measure the distance along the fish’s orientation vector). We have therefore kept our assessment based on the vertical distance.

      Absent measurements, the paragraph on associative learning should be struck from the discussion as it is purely speculative.

      We agree that the original paragraph on associative learning may have sounded overly speculative. However, after updating our manuscript with additional simulations of the thermal gradient's vertical extent, we found that fish perform upward turns not only above the thermal interface, but also before entering the thermal gradient itself. This observation makes us hesitant to attribute the response solely to thermotaxis. We believe it is essential to provide a plausible explanation—albeit speculative—for how fish initiate these turns before directly encountering the cold-water gradient. To support this, we have extended the discussion in this paragraph and added Supplementary Fig. 39. The new text now reads (additions in yellow): (L487 – 499): “Our findings show that fish were able to perform upward turns while still located above the thermal interface and that is, before actually sampling the cold water below the interface. In fact, our simulation of the vertical extent of the thermal gradient revealed that a substantial fraction of upward turns occurred before fish encountered the gradient itself — that is, prior to any sensory detection of the temperature change (Supplementary Fig. 39). This finding may be evidence of associative learning, whereby fish used information regarding the presence of colder water at depth obtained at prior times. While the current data do not provide conclusive evidence in this regard, they prompt the possibility that, rather than responding solely to immediate thermal cues, fish use spatial memory or associative learning to anticipate the location of colder water based on prior experience. Indeed, fish are able to perform associative learning based on non-visual cues[53], create mental maps of their surroundings54 and retain memory for hours[55], days[56] and months[57,58].”  

      The body-temperature simulations need to be detailed in the methods.

      Thanks for this comment. We have removed the supplementary text section and have included the paragraph “Body cooling during cold-water excursions” into the methods section of our manuscript (L804 - L829).

      Constant temperature experiments could be helpful in addressing the importance of a gradient/interface for triggering upward turning

      We agree, however, we were limited (for ethical reasons) to a maximum number of fish we could use in the experiments. Hence, we focused on getting approval to run experiments focused on the responses to thermal gradients. However, occupancy during the acclimation phase in 12 °C showed that fish were much more stationary and primarily occupied the lower half of the tank.

      A lot of ease of reading could be gained by labeling the conditions according to either the second temperature or perhaps even better the delta temperature (i.e. TR[-2C] instead of TR1).

      We agree that labeling conditions by the second temperature or delta temperature could in principle improve readability. However, since T_bottom and T_top are explicitly mentioned in each main figure at least once, they can be directly associated with the respective treatment. Therefore, we have opted to retain the current labeling for consistency.

      The figure legends are often short and do not accurately label all figure elements. This is especially true for supplemental figure legends which often appear rushed (e.g., the legend for Figure S2 stops mid-sentence, the legend of Figure S3 does not indicate what Ttop or Tbottom are).

      We appreciate the reviewer’s comment and have carefully revised all figure legends to ensure clarity and completeness. Specifically, we have corrected figure labels, expanded the descriptions for supplemental figures, and ensured that all elements are accurately defined. For instance, we have completed the legend for Figure S2 and clarified the definitions of T_top and T_bottom in Figure S3. Additionally, we have systematically reviewed all figure legends to prevent inconsistencies and omissions.

      For Figure S3, to improve clarity, plotting the standard deviation at different points in the tank across the phases could be more informative than the hard-to-distinguish multi-line plots in different shades of red.

      We appreciate the reviewer’s suggestion regarding Figure S3. However, the primary goal of this figure is to illustrate how the thermal interface moves over time. While plotting the standard deviation at different points in the tank could provide additional statistical insights, it would detract from the intended visualization of the interface dynamics. For this reason, we have opted to retain the current multi-line representation. Nevertheless, we have ensured that the figure is as clear as possible by refining the color contrast and improving the legend for better readability.

      There is an inconsistency in in-text citation styles (mixture of superscript and numbers in brackets).

      Thank you for pointing this out. We have carefully reviewed the manuscript and corrected any inconsistencies in the in-text citation style to ensure uniform formatting throughout.

      While the statement in the introduction, that increases in movement frequency could be purely metabolic in nature is correct, at least for larval zebrafish it has been shown that sensory neural activity is predictive of motor neuron activity and swim rates (Haesemeyer, 2018, cited by the authors).

      This is an interesting finding. It is however unclear to us why this information is crucial in our context of brown trout parr.

      Examples of summary results from Supplementary Figures 8-10 should be bundled in a main text figure since this appears to be important information supporting the conclusions.

      We agree that Supplementary Figures 8–10 contain important information (i.e. Boxplots) on vertical occupancy and the time individuals spent in different water temperatures. However, this information is already integrated into Figure 2C, D, F, and G, which display the vertical distributions of fish across treatments and over time. Given the current length of the manuscript, adding another main-text figure could dilute rather than enhance clarity. For this reason, we have opted to keep these details in the Supplementary Materials while ensuring they are appropriately referenced in the main text.

      The distributions of excursion length for all treatments should be graphed in a main figure to support the point made in the third paragraph of the "Trout parr... do not avoid warm water" section of the results.

      We appreciate the reviewer’s suggestion. However, we do not believe that plotting excursion length is necessary to support this statement, as the key finding is already well represented in the manuscript. Specifically, the transition to bimodal depth occupancy, with fish spending comparable time above and below the interface in warm-water treatments (TR6–TR9), is clearly conveyed in Figure 2F and Supplementary Figure 8B. Additionally, this information is explicitly stated in the results section (L235): "Fish did not avoid warmer water in any of the warm-water treatments (TR6–TR9). Instead, fish transitioned to a bimodal depth occupancy, with comparable time spent above and below the interface (Fig. 2F; Supplementary Fig. 8B)." Given this, we believe that adding an additional figure would not enhance clarity but may instead introduce redundancy.

      There should be a main figure panel that statistically compares the turn biases around the interface for the different conditions and the +/- 5cm interface line mentioned in the text should be visualized in the appropriate figures - incidentally, this length scale is on par with the diffusion seen in simulations further suggesting that fish in fact sense a gradient here rather than remembering an interface.

      To address the reviewer’s comment, we have made the following updates:

      • Extended and incorporated simulations of the thermal interface thickness (see Methods and Supplementary Fig. 29).

      • Plotted the vertical locations of up-turning events relative to the phase-averaged position of the thermal interface (see Supplementary Fig. 39), which includes the estimated 5 cm vertical extent of the thermal gradient.

      • Added the thermal interface thickness to the main figures (Fig. 3F,G and Fig. 2E,H) where applicable.

      While we do not claim that memory alone explains cold-water avoidance, our data still suggests that it may contribute to the observed behavior, particularly since a substantial number of upturns occurred before the fish entered the thermal gradient (see also Author response image 1 below). Our aim is not to statistically disentangle the relative contribution of thermotaxis versus associative learning, but to propose a plausible interpretation of this observed anticipatory behavior with due caution to clarify that this is only a possibility.

      Given that the thermal gradient is now visualized and characterized in detail, we respectfully suggest that an additional statistical comparison of turn biases would not add further clarity. We believe that is is evidence that vertical turning, away from the cold, occurred within and above the thermal gradient. However, we welcome the reviewer’s perspective and to demonstrate that turning points occur outside and above the thermal interface we have plotted them against gradient growth over time (see Author response image 1 below).

      Author response image 1.

      The colored area indicates the temporal growth of thermal interface thickness.

      Reviewer #3:

      In this study, the authors measured the behavioural responses of brown trout to the sudden availability of a choice between thermal environments. The data clearly show that these fish avoid colder temperatures than the acclimation condition, but generally have no preference between the acclimation condition or warmer water (though I think the speculation that the fish are slowly warming up is interesting). Further, the evidence is compelling that avoidance of cold water is a combination of thermotaxis and thermokinesis. This is a clever experimental approach and the results are novel, interesting, and have clear biological implications as the authors discuss. I also commend the team for an extremely robust, transparent, and clear explanation of the experimental design and analytical decisions. The supplemental material is very helpful for understanding many of the methodological nuances, though I admit that I found it overwhelming at times and wonder if it could be pruned slightly to increase readability. Overall, I think the conclusions are generally well-supported by the data, and I have no major concerns.

      Minor comments

      P2 intro paragraphs 1/3 - it is not clear that thermal preference generally reflects the thermal optimum, partly because it is not clear what trait is being optimized (fitness?). Some nuance here would be helpful, and would also link nicely to the discussion on p10.

      Thank you for this comment. We have now refined this section as follows (L67–71): "As most fish species are ectotherms, their body temperature fluctuates with the surrounding water temperature. Because temperature influences the rates of most physiological processes, rapid warming or cooling can affect fish performance traits, including metabolic rates, swimming ability, and thermal tolerance[6]."

      To further clarify how thermal preference relates to thermal optimum and what trait is being optimized, we have incorporated additional nuance in this section. Specifically, we now acknowledge that thermal preference may not always align with the thermal optimum for performance or fitness.

      P2 intro paragraph 2 - "adapt physiologically" implies evolution, but here you are referring to plasticity. Suggest saving the word "adapt/adaptation" for evolutionary changes (see also p9).

      Thank you for this comment. We have revised the wording to "acclimate physiologically" (L79) to more accurately reflect plastic responses rather than evolutionary adaptation.

      P7 - "This difference in probabilities (ρup - ρdown) was particularly large in the region immediately above and below the interface (-5 cm < D < 5 cm; Fig. 3F) and is a hallmark of a thermotactic behavior." I agree that the result provides compelling evidence for thermotaxis, but would it be possible to bolster this case by statistically testing for a difference in probabilities among the treatment groups here?

      In addition to Fig. 3F, we are presenting statistical evidence that for colder water temperatures, fish penetrate less deeply into the cold lower water. The decreasing trend was statistically significant (Mann–Kendall test: , p < 0.001; Supplementary Table 6) and is presented in Fig. 4C. The depth reached during each cold-water excursion is determined by the location of the vertical turning point, which redirects the fish upward toward the surface. We think this is sufficient evidence for thermotaxis.

      P9 paragraph 3 = "recent studies suggest that fish may instead respond to temporal changes of their internal body temperature." It seems like a citation is missing here. Would be useful to briefly summarize the evidence for internal temperature sensing that is the basis of this modelling exercise.

      Thanks, we have added that citation (L385).

      P10 "Our findings provide the first experimental evidence for this mode of behavioral thermoregulation in which fish navigate their heterothermal environment to achieve gradual body warming."

      I think this statement overreaches given the presented data. While there may be a trend towards fish in the warm treatment spending increasing amounts of time in the upper half of the tank, I do not see this pattern supported statistically. There is also no evidence of gradual body warming, and even if there was I disagree that this would constitute experimental evidence that this was happening "intentionally". By this reasoning, any shuttlebox experiment in which fish actively shuttle between relatively warm and cool sides to end up with a preference that is above the starting condition would also constitute evidence for gradual warming. Overall, this is an interesting pattern, but I do not think there is sufficient evidence to conclude that fish are strategically warming.

      We appreciate the reviewer’s comment and acknowledge that our original wording may have overstated the evidence. We have revised the sentence to better reflect the evdience presented (L411-415): “Our observations resemble this mode of behavioral thermoregulation, in which fish progressively favor warmer regions within a heterothermal environment. However, additional experimental evidence is required to determine the mechanisms underlying this behavior.”

      P11 "Despite the avoidance response of cold water, fish engaged in repeated cold-water excursions..."

      This is an interesting speculation, but I think it would be helpful to also point out that these fish are biased towards the bottom of the tank (based on control measurements) and this pattern may therefore simply reflect a desire to be lower in the water column.

      Thank you for this helpful comment. We have now added this point to the revised text, which reads (L475-477): “Despite the avoidance response to cold water, fish engaged in repeated cold-water excursions, potentially reflecting a behavioral strategy to map the thermal environment. This pattern may also reflect an inherent tendency to occupy the lower part of the tank, as observed during homogeneous temperature of 12 °C during the acclimation phase.”

      P13 - why was the dye always added to the right side of the tank, instead of being assigned to a side randomly? I think the control experiment is good evidence that the dye did not substantially affect behaviour, but it seems like it would have been nice to separate dye and novel temperature exposure.

      We agree that randomizing the side of dye application would have been ideal. The dye was consistently added to the right side to maintain procedural consistency, ensuring that the “incoming” or “novel” temperature was always dyed. That said, our control experiment provides strong evidence that the dye itself did not influence behavior (as discussed above and in the manuscript).

    1. Author response:

      The following is the authors’ response to the original reviews.

      Public Reviews:

      Reviewer #1 (Public Review):

      Summary:

      The major result in the manuscript is the observation of the higher order structures in a cryoET reconstruction that could be used for understanding the assembly of toroid structures. The crosslinking ability of ZapD dimers result in bending of FtsZ filaments to a constant curvature. Many such short filaments are stitched together to form a toroid like structure. The geometry of assembly of filaments - whether they form straight bundles or toroid like structures - depends on the relative concentrations of FtsZ and ZapD.

      Strengths:

      In addition to a clear picture of the FtsZ assembly into ring-like structures, the authors have carried out basic biochemistry and biophysical techniques to assay the GTPase activity, the kinetics of assembly, and the ZapD to FtsZ ratio.

      Weaknesses:

      The discussion does not provide an overall perspective that correlates the cryoET structural organisation of filaments with the biophysical data.

      The crosslinking nature of ZapD is already established in the field. The work carried out is important to understand the ring assembly of FtsZ. However, the availability of the cryoET observations can be further analysed in detail to derive many measurements that will help validate the model, and obtain new insights.

      We thank the reviewer for these insightful comments on our work. We have edited the manuscript to resolve and clarify most of the issues raised during the review process.

      Reviewer #2 (Public Review):

      Summary:

      In this paper, the authors set out to better understand the mechanism by which the FtsZ-associated protein ZapD crosslinks FtsZ filaments to assemble a large-scale cytoskeletal assembly. For this aim, they use purified proteins in solution and a combination of biochemical, biophysical experiments and cryo-EM. The most significant finding of this study is the observation of FtsZ toroids that form at equimolar concentrations of the two proteins.

      Strengths:

      Many experiments in this paper confirm previous knowledge about ZapD. For example, it shows that ZapD promotes the assembly of FtsZ polymers, that ZapD bundles FtsZ filaments, that ZapD forms dimers and that it reduces FtsZ's GTPase activity. The most novel discovery is the observation of different assemblies as a function of ZapD:FtsZ ratio. In addition, using CryoEM to describe the structure of toroids and bundles, the paper provides some information about the orientation of ZapD in relation to FtsZ filaments. For example, they found that the organization of ZapD in relation to FtsZ filaments is "intrinsic heterogeneous" and that FtsZ filaments were crosslinked by ZapD dimers pointing in all directions. The authors conclude that it is this plasticity that allows for the formation of toroids and its stabilization. Unfortunately, a high-resolution structure of the protein organization was not possible. These are interesting findings that in principle deserve publication.

      We thank the reviewer for this valuable assessment. We have made several changes to the manuscript to improve its readability and comprehensibility. In addition, we have addressed the reviewer’s main concerns in the point-by-point response below.

      Weaknesses:

      While the data is convincing, their interpretation has some substantial weaknesses that the authors should address for the final version of this paper.

      We have addressed most of the aspects highlighted by the reviewer to improve the quality and comprehensibility of our results.

      For example, as the authors are the first to describe FtsZ-ZapD toroids, a discussion why this has not been observed in previous studies would be very interesting, i.e. is it due to buffer conditions, sample preparation?

      Several factors may explain the absence of observed toroidal structures in other studies. FtsZ is a highly dynamic protein, and its behavior varies significantly with different environmental conditions, as detailed in the literature. These environmental factors include pH, salt concentration, protein type, GTP levels, and the purification strategy used. Previous research has employed negative stain electron microscopy (EM) to visualize ZapD-FtsZ structures. It is important to note that FtsZ is sensitive to surface effects when it is bound to or adsorbed onto membranes (Mateos-Gil et al. 2019 FEMS Microbiol Rev - DOI: 10.1093/femsre/fuy039). Therefore, the adsorption of FtsZ and ZapD onto the EM grid may influence the formation of higher order structures. In this study, we used cryo-electron microscopy (cryo-EM) and cryo-electron tomography (cryo-ET) to visualize the 3D organization of ZapD-mediated structures. This approach allows us to avoid staining artifacts and the distortion of structures caused by adsorption or drying of the grid. In addition, we can resolve single filaments. Our buffer conditions also differ slightly from those in previous studies, which may significantly impact the behavior of FtsZ, as illustrated in Supplementary Fig. 3.

      At parts of the manuscript, the authors try a bit too hard to argue for the physiological significance of these toroids. This, however, is at least very questionable, because: The typical diameter is in the range of 0.25-1.0 μm, which requires some flexibility of the filaments to be able to accommodate this. It's difficult to see how a FtsZ-ZapD toroid, which appears to be quite rigid with a narrow size distribution of 502 nm {plus minus} 55 nm could support cell division rather than stalling it at that cell diameter. which the authors say is similar to the E. coli cell.

      The toroidal structures formed by FtsZ and ZapD, with their characteristics similar to those of the bacterial division system, are significant in physiological contexts and warrant further study. The connections mediated by Zaps are expected to play a crucial role in filament organization, which is vital for the machinery enabling cellular constriction. Therefore, characterizing these structures in vitro can provide insight into divisome stabilization, assembly and constriction mechanisms. While we acknowledge the limitations of in vitro systems and do not expect to see the same toroidal structures in vivo, the way ZapD decorates and connects FtsZ filaments in vitro may resemble the processes that occur in the division ring formed inside the cell. This study represents an initial effort to characterize these toroidal structures, which could inspire further research and potentially reveal their physiological relevance.

      Regarding flexibility, it has been previously reported that an arrangement of loosely connected filaments forms the FtsZ ring. Our model is consistent with this observation despite the heterogeneity and density observed in the toroidal structures. We anticipate differences in vivo due to the high complexity of the cytoplasm, interactions with other cellular components, and attachment to the cell membrane, all of which would influence structural outcomes. However, our novel in vitro approach, which allows us to study FtsZ filament organization and connectivity – features that are challenging to explore in vivo and have not been thoroughly investigated before – has the potential to significantly advance our understanding of these structures. Consequently, these structures can aid our understanding of complex macrostructures in vivo, even if we have merely begun to scratch the surface of their characterization.

      Regarding the size of the toroids, we hypothesize that it reflects an optimal condition based on our experimental setup in solution. In vivo, these conditions are altered by interactions with various division partners, attachment to the plasma membrane, and system contraction. 

      We have better reformulated and edited the manuscript to discuss the potential physiological relevance of our toroidal structures.

      For cell division, FtsZ filaments are recruited to the membrane surface via an interaction of FtsA or ZipA the C-terminal peptide of FtsZ. As ZapD also binds to this peptide, the question arises who wins this competition or where is ZapD when FtsZ is recruited to the membrane surface? Can such a toroidal structure of FtsZ filaments form on the membrane surface? Additional experiments would be helpful, but a more detailed discussion on how the authors think ZapD could act on membrane-bound filaments would be essential.

      We appreciate this comment, which was indeed one of our main questions. The complexity of the division system raises many questions about the interaction of FtsZ with the plasma membrane. The competition between division components to interact with FtsZ and thus modulate its behavior is still largely unknown. FtsA and ZipA appear to have a greater affinity for the C-terminal domain (CTD) of FtsZ than ZapD. However, considering all FtsZ monomers forming a filament, we expect FtsZ filaments to interact with many different division partners. The ability of FtsZ to interact with many components is necessary to explain the current model of the system. According to this model, FtsZ filaments would be decorated by many different proteins, anchoring them to the membrane while crosslinking or promoting their disassembly in a spatiotemporally controlled manner. 

      We tried experiments combining FtsA, ZipA, and ZapD on supported lipid membranes and liposomes. However, they proved difficult to perform. We expect similar results to those observed for ZapA (Caldas et al. 2019 Nat Commun - DOI: 10.1038/s41467-019-13702-4). However, competition between proteins for interaction with the CTD of FtsZ adds an extra layer of complexity, making exploring this issue attractive in the future. However, as remarkably pointed out by Reviewer 3, our cryo-ET data of straight bundles provide new insights into how ZapD-FtsZ structures can bind to the plasma membrane. In these straight bundles, the CTDs of two parallel FtsZ filaments are oriented upwards. They can bind the plasma membrane directly or the ZapDs, which decorate the FtsZ filaments from above instead of from the side, as suggested previously (Schumacher et al. 2017 J Biol Chem - DOI: 10.1074/jbc.M116.773192), allowing ZapDs to interact with the membrane.

      The authors conclude that the FtsZ filaments are dynamic, which is essential for cell division. But the evidence for dynamic FtsZ filaments within these toroids seems rather weak, as it is solely the partial reassembly after addition of GTP. As ZapD significantly slows down GTP hydrolysis, I am not sure it's obvious to make this conclusion.

      FtsZ filaments are dynamic, as they can reassemble into macrostructures relatively quickly. Decreased GTPase activity is a good indicator of the formation of lateral interactions between filaments. For instance, under crowding conditions, FtsZ also reduces its GTPase activity, although the bundles disassemble very slowly over time (González et al. 2003 J. Biol. Chem - DOI: 10.1074/jbc.M305230200). We measured the GTPase activity during the first 5 minutes after GTP addition, conditions under which toroidal structures and bundles remain fully assembled. However, we expect GTPase activity to recover as the macrostructures disassemble, considering the reassembly of macrostructures after GTP resupply, which suggests that FtsZ filaments remain active and dynamic.

      On a similar note, on page 5 the authors claim that ZapD would transiently interact with FtsZ filaments. What is the evidence for this? They also say that this transient interaction could have a "mechanistic role in the functionality of FtsZ macrostructures." Could they elaborate?

      We have rephrased the whole paragraph in the revised version to clarify matters (page 10, lines 2434):

      “These results are consistent with the observation that ZapD interacts with FtsZ through its central hub, which provides additional spatial freedom to connect other filaments in different conformations. This flexibility allows different filament organizations and contributes to structural heterogeneity. In addition, these results suggest that these crosslinkers can act as modulators of the dynamics of the ring structure, spacing filaments apart and allowing them to slide in an organized manner. The ability of FtsZ to treadmill directionally, together with the parallel or antiparallel arrangement of short, transiently crosslinked filaments, is considered essential for the functionality of the Z ring and its ability to exert constrictive force34,36–38,50. Thus, Zap proteins can play a critical role in ensuring correct filament placement and stabilization, which is consistent with the toroidal structure formed by ZapD.”

      The author should also improve in putting their findings into the context of existing knowledge. For example:

      The authors observe a straightening of filament bundles with increasing ZapD concentration. This seems consistent with what was found for ZapA, but this is not explicitly discussed (Caldas et al 2019)

      We have discussed this similarity in the revised version of this manuscript (page 12, line 40 - page 13, line 8):

      “Understanding how the associative states of ZapA (as tetramers) and ZapD (as dimers), together with membrane tethering, influence the predominant structures formed in both systems is essential. The complexity of the division system raises important questions about the interaction dynamics between FtsZ and the plasma membrane. The competitive nature of the division components to engage with FtsZ and modulate its functionality remains to be thoroughly elucidated. It is important to note that FtsA and ZipA have a greater affinity for the C-terminal domain of FtsZ than ZapD. Our cryo-ET data on straight bundles provide new perspectives on how ZapD-FtsZ structures can effectively bind to the plasma membrane; in particular, the C-terminal domains of parallel FtsZ filaments are oriented upward, allowing direct membrane binding or interaction with ZapDs that reinforce these filaments from above, rather than from the side, as previously suggested.”

      A paragraph summarizing what is known about the properties of ZapD in vivo would be essential: i.e., what has been found regarding its intracellular copy number, location and dynamics?

      We thank the reviewer for this valuable suggestion. We describe the role of Zap proteins in vivo and the previous studies of ZapD in the introduction (page 2, lines 34 - page 3, line 17). Additionally, we added the estimated number of ZapD copies in the cell in the discussion (page 11, lines 2-7).

      In the introduction, the authors write that "GTP binding and hydrolysis induce a conformational change in each monomer that modifies its binding potential, enabling them to follow a treadmilling behavior". This seems inaccurate, as shown by Wagstaff et al. 2022, the conformational change of FtsZ is not associated with the nucleotide state. In addition, they write that FtsZ polymerization depends on the GTPase activity. It would be more accurate to write that polymerization depends on GTP, and disassembly on GTPase activity.”

      Following the reviewer's suggestions, we have adapted and corrected these text elements as follows (page 2, lines 7-9): 

      “FtsZ undergoes treadmilling due to polymerization-dependent GTP hydrolysis, allowing the ring to exhibit its dynamic behavior.”

      On page 2 they also write that "the mechanism underlying bundling of FtsZ filaments is unknown". I would disagree, the underlying mechanism is very well known (see for example Schumacher, MA JBC 2017), but how this relates to the large-scale organization of FtsZ filaments was not clear.

      We thank the reviewer for this comment. We have corrected and clarified the related text accordingly (page 3, lines 11-12):

      “…the link between FtsZ bundling, promoted by ZapD, and the large-scale organization of FtsZ filaments remains unresolved.”

      The authors describe the toroid as a dense 3D mesh, how would this be compatible with the Z-ring and its role for cell division? I don't think this corresponds to the current model of the Z-ring (McQuillen & Xiao, 2020). Apart from the fact it's a ring, I don't think the organization of FtsZ obviously similar to the current of the Z-ring in the bacterial cell, in particular because it's not obvious how FtsZ filaments can bind ZapD and membrane anchors simultaneously.

      We consider that the intrinsic characteristics of toroidal structures and the bacterial division ring have points in common. As indicated in the answer above, despite the differences and limitations that might result from an in vitro approach, the structures shown after ZapD crosslinking of FtsZ filaments can demonstrate intrinsic features occurring in vivo. The current model of the division ring consists of an arrangement of filaments loosely connected by crosslinkers in the center of the cell, forming a ring. This model is compatible with our findings, although many questions remain about the structural organization of the Z-ring in the cell.

      Reviewer 3 has brought a compelling new perspective to interpreting our cryo-ET data: ZapD decorates FtsZ from above, allowing ZapD or FtsZ to bind to the plasma membrane. We have discussed this point in more detail below. In the case of straight bundles, this favors the stacking of straight FtsZ filaments, whereas in the case of toroids, ZapD can also bind FtsZ filaments laterally and diagonally, and it is this less compact arrangement that could enable FtsZ bending and toroid size adjustment. 

      We have revised the text accordingly to incorporate the interpretation proposed by Reviewer 3 (page 12, lines 24-31):

      “The current model of the division ring consists of an array of filaments loosely connected by crosslinkers at the center of the cell, forming a ring. This model is consistent with our findings, although many questions remain regarding the structural organization of the Z ring within the cell. ZapD binds to FtsZ from above, allowing either ZapD or FtsZ to interact with the plasma membrane. In straight bundles, this facilitates the stacking of straight FtsZ filaments, while for toroids, ZapD can also bind FtsZ filaments diagonally. This less compact arrangement could allow bending of the FtsZ filaments and adjustment of toroid size.”

      The authors write that "most of these modulators" interact with FtsZ's CTP, but then later that ZapD is the only Zap protein that binds CTP. This seems to be inconsistent. Why not write that membrane anchors usually bind the CTP, most Zaps do not, but ZapD is the exception?

      We thank the reviewer for this pertinent suggestion, which we have followed in the revised version of the manuscript (page 2, lines 19-22):

      “Most of these modulators interact with FtsZ through its carboxy-terminal end, which modulates division assembly as a central hub.  ZapD is the only Zap protein known to crosslink FtsZ by binding its C-terminal domain, suggesting a critical Z ring structure stabilizing function.”

      I also have some comments regarding the experiments and their analysis:

      Regarding cryoET: the filaments appear like flat bands, even in the absence of ZapD, which further elongates these bands. Is this due to an anisotropic resolution? This distortion makes the conclusion that ZapD forms bi-spherical dimers unconvincing.

      The missing wedge caused by the limited angular range of the tomography data generates an elongation of the structures by a factor of 2 along the Z axis. This feature is visible in the undecorated FtsZ filament data (Supplementary Fig. 10). The more pronounced elongation along the Z-axis observed in the presence of ZapD indicates the presence of ZapD to connect two parallel FtsZ filaments along the Z-axis (see Supplementary Figs. 8, 9 and 10). We do not have sufficient resolution to precisely resolve ZapD proteins from the FtsZ filaments in the Z-axis, but we also observed bispherical ZapDs in the XY plane (Fig. 4b-d). Unfortunately, our data do not allow for a more detailed characterization.

      The authors say that the cryoET visualization provides crucial information on the length of the filaments within this toroid. How long are they? Could the authors measure it?

      Measuring the length of single filaments is not trivial, given the dense, heterogeneous mesh promoted by ZapD crosslinking. We tried to identify and track them, but the density of filaments and connections made precise measurement very difficult. Nevertheless, we could identify the formation of these toroids by an arrangement of short filaments (Supplementary Fig. 11) instead of continuous circular filaments.

      We have removed the following sentence text in the revised manuscript: “Visualization of ZapDmediated FtsZ toroidal structures by cryo-ET provided crucial information on the 3D organization, connectivity and length of filaments within the toroid.”

      Regarding the dimerization mutant of ZapD: there is actually no direct confirmation that mZapD is monomeric. Did the authors try SEC MALS or AUC? Accordingly, the statement that dimerization is "essential" seems exaggerated (although likely true).

      Unlike the wild-type ZapD protein, the mZapD mutant exists as a mixture of monomers (~15%) and dimers, as AUC assays performed at similar protein concentrations revealed. These results demonstrate that the mutant protein has a lower tendency to form dimers than the native ZapD protein. We have included the AUC data for mZapD in the supplementary material (Supp. Fig. 15a).

      What do the authors mean that toroid formation is compatible with robust persistence length? I.e. What does robust mean? It was recently shown that FtsZ filaments are actually surprisingly flexible, which matches well the fact that the diameter of the Z-ring must continuously decrease during cell division (Dunajova et al Nature Physics 2023).

      We have corrected this sentence in the revised version of the manuscript to improve clarity (page 11, lines 9-10): 

      “The persistence length and curvature of FtsZ filaments are optimized for forming bacterial-sized ring structures.”

      The authors claim that their observations suggest „that crosslinkers ... allows filament sliding in an organized fashion". As far as I know there is no evidence of filament sliding, as FtsZ monomers in living cells and in vitro are static.

      Filament sliding may be one of the factors contributing to the force generation mechanisms involved in cell division (Nguyen et al. 2021 J Bacteriol - DOI: 10.1128/JB.00576-20). Our results indicate that ZapD can separate filaments, creating space between them and facilitating their organization.

      Although the molecular dynamics of cell constriction are not yet fully understood, it is possible that filament sliding plays a role. If this is the case, the crosslinking of short FtsZ filaments in multiple directions by ZapD could provide the necessary flexibility to adjust the diameter of the constriction ring during bacterial division.

      What is the „proto-ring FtsA protein"?

      The proto-ring denotes the first molecular assembly of the Z-ring, which in E. coli consists of FtsZ, FtsA and ZipA (see, for example, Ortiz et al. 2016 FEMS Microbiol Rev - DOI: 10.1093/femsre/fuv040). To simplify matters, we have deleted the term “proto-ring” in the revised version of the MS.

      The authors refer to „increasing evidence" for „alternative network remodeling mechanisms that do not rely on chemical energy consumption as those in which entropic forces act through diffusible crosslinkers, similar to ZapD and FtsZ polymers." A reference should be given, I assume the authors refer to the study by Lansky et al 2015 of PRC on microtubules. However, I am not sure how the authors made the conclusion that this applies to FtsZ and ZapD, on which evidence is this assumption based?

      We refer to cytoskeletal network remodeling mechanisms independent of chemical energy consumption (Braun et al. 2016 Bioessays - DOI: 10.1002/bies.201500183) driven by entropic forces induced by macromolecular crowding agents or diffusible crosslinkers. The latter mechanism leads to an increase in filament overlap length and the contraction of filament networks. These mechanisms complement and act in synergy with energy-consuming processes (such as those involving nucleotide hydrolysis) to modulate actin- and microtubule-based cytoskeleton remodeling. Similarly, crosslinking proteins such as ZapD may contribute to remodeling the FtsZ division ring in the cell. 

      We have revised the corresponding text of the manuscript accordingly (page 13, lines 16-24):  “In addition, our findings could greatly enhance the understanding of how polymeric cytoskeletal networks are remodeled during essential cellular processes such as cell motility and morphogenesis. Although conventional wisdom points to molecular motors as the primary drivers of filament remodeling through energy consumption, there is increasing evidence that there are alternative mechanisms that do not rely on such energy, instead harnessing entropic forces via diffusible crosslinkers. This approach may also be applicable to ZapD and FtsZ polymers, suggesting a promising avenue for optimizing conditions in the reverse engineering of the division ring to enhance force generation in minimally reconstituted systems aimed at achieving autonomous cell division.”

      Some inconsistencies in supplementary figure 3: The normalized absorbances in panel a do not seem to agree with the absolute absorbance shown in panel e, i.e. compare maximum intensity for ZapD = 20 µM and 5 µM in both panels.

      We have corrected these inconsistencies in the revised version.

      It's not obvious to me why the structure formed by ZapD and FtsZ disassembles after some time even before GTP is exhausted, can the authors explain? As the structures disassemble, how is the "steadystate turbidity" defined? Do the structures also disassemble when they use a non-hydrolyzable analog of GTP?

      In the presence of ZapD, FtsZ rapidly forms higher order polymers after the addition of GTP, as shown by turbidity assays at 320 nm (the formation of single- or double-stranded FtsZ filaments in the absence of ZapD does not produce a significant increase in turbidity). Macrostructures formed by FtsZ in the presence of ZapD, while more stable than FtsZ filaments (which rapidly disassemble following GTP consumption), are also dynamic. These assembly reactions are GTP-dependent and considerably modify polymer dynamics. In agreement with our results, previous studies have shown that high concentrations of macromolecular crowders (such as Ficoll or dextran) promote the formation of dynamic FtsZ polymer networks (González et al. 2003 J. Biol. Chem - DOI: 10.1074/jbc.M305230200). In this case, FtsZ GTPase activity was significantly retarded compared with FtsZ filaments, resulting in a decrease in GTPase turnover. Similar mechanisms may apply to assembly reactions in the presence of ZapD.

      Parallel assembly studies replacing GTP with a slowly hydrolyzable GTP analog remain pending. We expect ZapD-containing FtsZ macrostructures to last assembled for longer but still disassemble upon GTP consumption, as occurs with the crowding-induced FtsZ polymer networks formed in the presence of nucleotide analogs.

      Accordingly, we have revised the corresponding text to clarify matters (page 4, line 37 – page 5 line 7). 

      Conclusion: Despite some weaknesses in the interpretation of their findings, I think this paper will likely motivate other structural studies on large scale assemblies of FtsZ filaments and its associated proteins. A systematic comparison of the effects of ZapA, ZapC and ZapD and how their different modes of filament crosslinking can result in different filament networks will be very useful to understand their individual roles and possible synergistic behavior.

      We appreciate the reviewer's remarks and comments, which provided us with valuable information and helped us considerably improve the revised manuscript.

      Reviewer #3 (Public Review):

      Summary:

      The authors provide the first image analysis by cryoET of toroids assembled by FtsZ crosslinked by ZapD. Previously toroids of FtsZ alone have been imaged only in projection by negative stain EM. The authors attempt to distinguish ZapD crosslinks from the underlying FtsZ filaments. I did not find this distinction convincing, especially because it seems inconsistent with the 1:1 stoichiometry demonstrated by pelleting. I was intrigued by one image showing straight filament pairs, which may suggest a new model for how ZapD crosslinks FtsZ filaments.

      We thank the reviewer for these valuable comments, to which we have responded in detail below. 

      Strengths:

      (1) The first image analysis of FtsZ toroids by cryoET.

      (2) The images are accompanied by pelleting assays that convincingly establish a 1:1 stoichiometry of FtsZ:ZapD subunits.

      (3) Fig. 5 shows an image of a pair of FtsZ filaments crosslinked by ZapD. This seems to have higher resolution than the toroids. Importantly, it suggests a new model for the structure of FtsZ-ZapD that resolves previously unrecognized conflicts. (This is discussed below under weaknesses, because it is so far only supported by a single image.)

      We thank the reviewer for this assessment and, in particular, for raising point 3, which provided a new perspective on the interpretation of our data. We have also included a new example of a straight bundle in Supplementary Fig. 13.

      Weaknesses:

      This paper reports a study by cryoEM of polymers and bundles assembled from FtsZ plus ZapD. Although previous studies by other labs have focused on straight bundles of filaments, the present study found toroids mixed with these straight bundles, and they focused most of their study on the toroids. In the toroids they attempt to delineate FtsZ filaments and ZapD crosslinks. A major problem here is with the stoichiometry. Their pelleting assays convincingly established a stoichiometry of 1:1, while the mass densities identified as ZapD are sparse and apparently well below the number of FtsZ (FtsZ subunits are not resolved in the reconstructions, but the continuous sheets or belts seem to have a lot more mass than the identified crosslinks.)  

      Apart from the stoichiometry I don't find the identification of crosslinks to be convincing. It is missing an important control - cryoET of toroids assembled from pure FtsZ, without ZapD.

      However, if I ignore these and jump to Fig. 5, I think there is an important discovery that resolves controversies in the present study as well as previous ones, controversies that were not even recognized. The controversy is illustrated by the Schumacher 2017 model (their Fig. 7), which is repeated in a simplified version in Fig. 1a of the present mss. That model has a two FtsZ filaments in a plane facing ZapD dimers which bridge them. In this planar model the C-terminal linker, and the ctd of FtsZ that binds ZapD facing each other and the ZapD in the middle, with. The contradiction arises because the C-terminus needs to face the membrane in order to attach and generate a bending force. The two FtsZ filaments in the planar model are facing 90{degree sign} away from the membrane. A related contradiction is that Houseman et al 2016 showed that curved FtsZ filaments have the C terminus on the outside of the curve. In a toroid the C termini should all be facing the outside. If the paired filaments had the C termini facing each other, they could not form a toroid because the two FtsZ filaments would be bending in opposite directions.

      Fig. 5 of the present ms seems to resolve this by showing that the two FtsZ filaments and ZapD are not planar, but stacked. The two FtsZ filaments have their C termini facing the same direction, let's say up, toward the membrane, and ZapD binds on top, bridging the two. The spacing of the ctd binding sites on the Zap D dimer is 6.5 nm, which would fit the ~8 nm width of the paired filament complex observed in the present cryoEM (Fig S13). In the Schumacher model the width would be about 20 nm. Importantly, the stack model has the ctd of each filament facing the same direction, so the paired filaments could attach to the membrane and bend together (using ctd's not bound by ZapD). Finally, the new arrangement would also provide an easy way for the complex to extend from a pair of filaments to a sheet of three or four or more. A problem with this new model from Fig. 5 is that it is supported by only a single example of the paired FtsZ-ZapD complex. If this is to be the basis of the interpretation, more examples should be shown. Maybe examples could be found with three or four FtsZ filaments in a sheet.

      We thank the reviewer for asking interesting questions and suggesting a compelling model for how ZapD could bind FtsZ filaments. Cryo-ET of straight bundles revealed that high ZapD density promotes vertical stacking of FtsZ filaments and decoration of FtsZ filaments by ZapD from above. In toroids, FtsZ filaments are vertically decorated by ZapD, which explains the high elongation of the filament structures observed, consisting of FtsZ-ZapD(-FtsZ) units. In addition, we observed a high abundance of diagonal connections between FtsZ filaments of different heights, revealing a certain flexibility/malleability of ZapD to link filaments that are not perfectly aligned vertically. This configuration could give rise to curved filaments and the overall toroid structure.

      The manuscript proposes that ZapD can bind FtsZ filaments in different directions. However, it seems to have a certain tendency to bind to the upper part of FtsZ filaments, stacking them vertically or vertically with a lateral shift (Supplementary Fig. 9). We also observe lateral connections, although the features of the toroidal structures limit their visualization. This enables both the binding to the membrane by ZapD or FtsZ and the formation of higher order FtsZ polymer structures. 

      In summary, ZapD is capable of linking FtsZ filaments in multiple directions, including from the upper part of the filaments as well as laterally or diagonally. At high concentrations of ZapD, the filaments become more compactly arranged, primarily stacking vertically, which results in the loss of curvature. In contrast, at lower concentrations of ZapD, the FtsZ filaments are less tightly packed, leading to curved filaments and an overall toroidal structure that may resemble the in vivo ring structures.

      We have edited our manuscript to accommodate this hypothesis, including the abstract and the cryoET section (page 7, lines 5-16): 

      “The isosurface confirmed the presence of extended structures along the Z-axis, well beyond the elongation expected from the missing wedge effect for single FtsZ filaments (for comparison, see Supplementary Fig. 10). The vertically extended structures appeared to correspond to filaments that were connected or decorated by additional densities along the Z-axis (Supplementary Fig. 9b). Importantly, these densities were only observed in the presence of ZapD (Supplementary Fig. 10b), suggesting that they represent ZapD connections (Fig. 3e and Supplementary Figs. 8e and 9b). We note that the resolution of the data is not sufficient to precisely resolve ZapD proteins from the FtsZ filaments in the Z-axis.

      These results suggest that the toroids are constructed and stabilized by interactions between ZapD and FtsZ, which are mainly formed along the Z-axis but also laterally and diagonally.”

      Page 7, lines 40-42: 

      “Cryo-ET imaging of ZapD-mediated FtsZ toroidal structures revealed a preferential vertical stacking and crosslinking of short ZapD filaments, which are also crosslinked laterally and diagonally, allowing for filament curvature.”

      And in the discussion (page 12, lines 27-31): 

      “ZapD binds to FtsZ from above, allowing either ZapD or FtsZ to interact with the plasma membrane. In straight bundles, this facilitates the stacking of straight FtsZ filaments, while for toroids, ZapD can also bind FtsZ filaments diagonally. This less compact arrangement could allow bending of the FtsZ filaments and adjustment of the toroid size.”

      What then should be done with the toroids? I am not convinced by the identification of ZapD as "connectors." I think it is likely that the ZapD is part of the belts that I discuss below, although the relative location of ZapD in the belts is not resolved. It is likely that the resolution in the toroid reconstructions of Fig. 4, S8,9 is less than that of the isolated pf pair in Fig. 5c.

      We agree with the reviewer's interpretation that ZapD can attach to FtsZ filaments from both above and laterally. The data from the straight bundles, which are more clearly resolved due to their thinner structure, demonstrate that ZapD can decorate FtsZ filaments vertically. Additionally, the toroidal data supports the notion that ZapD can act as a crosslinker between filaments that are not perfectly vertical, allowing for lateral offsets (see, for example, Fig. 4d) or lateral connections (Fig. 4b). 

      We recognize that the resolution and high density of structures in our cryo-ET data make it challenging to accurately annotate proteins or connectors. Despite this difficulty, we have made efforts to label and identify the ZapD proteins and connectors. We employed an arbitrary labeling method to assist with visual interpretation. However, we acknowledge that some errors may exist and that ZapD proteins were not labeled, particularly along the Z-axis, where the missing wedge limits our ability to distinguish between ZapD and FtsZ proteins (page 7, lines 8-13):

      “The vertically extended structures appeared to correspond to filaments that were connected or decorated by additional densities along the Z-axis (Supplementary Fig. 9b). Importantly, these densities were only observed in the presence of ZapD (Supplementary Fig. 10b), suggesting that they represent ZapD connections (Fig. 3e and Supplementary Figs. 8e and 9b). We note that the resolution of the data is not sufficient to precisely resolve ZapD proteins from the FtsZ filaments in the Z-axis. We note that the resolution of the data is not sufficient to precisely resolve ZapD proteins from the FtsZ filaments in the Z-axis.”

      We draw attention to the limitation of our manual segmentation in the text as follows (page 7, lines 20-24):

      “We manually labeled the connecting densities in the toroid isosurfaces to analyze their arrangement and connectivity with the FtsZ filaments. The high density of the toroids and the wide variety of conformations of these densities prevented the use of subtomogram averaging to resolve their structure and spatial arrangement within the toroids.”

      Importantly, If the authors want to pursue the location of ZapD in toroids, I suggest they need to compare their ZapD-containing toroids with toroids lacking ZapD. Popp et al 2009 have determined a variety of solution conditions that favor the assembly of toroids by FtsZ with no added protein crosslinker. It would be very interesting to investigate the structure of these toroids by the present cryoEM methods, and compare them to the FtsZ-ZapD toroids. I suspect that the belts seen in the ZapD toroids will not be found in the pure FtsZ toroids, confirming that their structure is generated by ZapD.

      The only reported toroidal structure of E. coli FtsZ can be found in the literature by Popp et al. (2009 Biopolymers – DOI: 10.1002/bip.21136). It is important to note that methylcellulose (MC) must be added to the working solution to induce the formation of these structures, as FtsZ toroids do not form in the absence of MC. The mechanisms by which MC promotes this assembly process go beyond mere excluded volume effects due to crowding, as the concentration of MC used is very low (less than 1 mg/ml), which is below the typical crowding regime. This suggests that there are additional interactions between MC and FtsZ. Such complexities and secondary interactions prevent the use of this system as a reliable control for the FtsZ toroidal structures reported here. Alternatively, we also considered the toroidal structures of FtsZ from Bacillus subtilis (Huecas et al. 2017 Biophys J - DOI: 10.1016/j.bpj.2017.08.046) and Cyanobacterium synechocystis (Wang et al. 2019 J Biol Chem – DOI: 10.1074/jbc.RA118.005200). However, these structures do not serve as appropriate controls due to the structural and molecular differences between these FtsZ proteins.

      Recommendations for the authors:  

      Reviewing Editor:

      While the three referees recognize and appreciate the importance of this work several technical and interpretational questions have been raised. There was a prolonged discussion amongst the three expert referees, and it was felt that the current version suffers from a number of problems that the authors need to consider. These are to do with 1. Stoichiometry of ZapD-FtsZ 2. the evidence for crosslinks 3. how the cryo-ET data correlates with the biophysical data 4. Physiological relevance of the elucidated structures. Please take note of the public reviews (strengths and weaknesses) as well as "Recommendations to the authors" sections below, if you choose to prepare a revision.

      In reading the reviews very carefully (as well as while following the ensuing robust discussion between the referees) I noticed that all points raised are extremely important to be addressed / reconciled (with experiments and / or discussion) for this study to become an outstanding contribution to bacterial cell biology field. I would therefore urge you to consider these carefully and revise the manuscript accordingly.

      We thank the editorial board and reviewers for their excellent work evaluating and reviewing our manuscript. Their constructive suggestions and comments have been taken into account in preparing the revised version. We have paid particular attention to the four points mentioned above by the reviewing editor. We hope that the new version and this point-by-point rebuttal letter will answer most of the questions and weaknesses raised by the reviewers.

      Reviewer #1 (Recommendations for the authors):

      Suggestions for improvement of the manuscript:

      (1) ZapD to FtsZ ratio:

      i) Page 3: Results section, paragraph 1:

      FtsZ to ZapD shows a 1:2 ratio. How does this explain cross linking by a dimeric species, as this will be equivalent to a 1:1 ratio of FtsZ and ZapD? The crystal structure in the reference cited has FtsZ peptide bound only to one side of the dimer, however a crosslinking effect can happen only if FtsZ binds to both protomers of ZapD dimer. If the decoration is not uniform as given in the toroid model based on cryoET, this should lead to a model with excess of FtsZ in the toroid?

      On page 3 of the original manuscript, we stated that the binding stoichiometry of ZapD to FtsZ was 2:1, based on estimates derived from sedimentation velocity experiments involving the unassembled GDP form of FtsZ. However, upon reanalyzing these experiments, we found that the previous characterization of the association mode was overly simplistic. We determined that there are two predominant molecular species of ZapD:FtsZ complexes in solution, which correspond to ZapD dimers bound to either one or two FtsZ monomers, resulting in stoichiometries of 2:1 and 1:1, respectively. The revised binding stoichiometry data for ZapD and GDP-FtsZ suggests the presence of 1:1 ZapD-FtsZ complexes which aligns with the idea that FtsZ polymers can be crosslinked by dimeric ZapD species. In mixtures where ZapD is present in excess over FtsZ, the crosslinking corresponds to 1:1 binding stoichiometries, leading to the formation of straight macrostructures. Conversely, when the concentration of ZapD is reduced in the reaction mixture, the resulting macrostructures take the form of toroids. In this scenario, there is an excess of FtsZ because only some of the FtsZ molecules within the polymers are crosslinked by ZapD dimers, resulting in a binding stoichiometry of approximately 0.4 ZapD molecules per FtsZ, as quantified by differential sedimentation experiments.

      We have rewritten the corresponding texts in the revised version to explain these matters (page 4 lines 14-18):

      “Sedimentation velocity analysis of mixtures of the two proteins revealed the presence of two predominant molecular species of ZapD:FtsZ complexes in solution. These complexes are compatible with ZapD dimers bound to one or two FtsZ monomers, corresponding to ZapD:FtsZ stoichiometries of 2:1 and 1:1, respectively (Supplementary Fig. 1a (III-IV)). This observation is consistent with the proposed interaction model.”

      ii) How does 40 - 80 uM of ZapD correspond to a molar ratio of approximately 6?

      It was a typo from previous versions. We have corrected it in the revised version. 

      iii) The ratios of ZapD to FtsZ are different when described later in page 4 in the context of the toroid. Are these ratios relevant compared to the contradicting ratios mentioned later in page 4?

      To clarify issues related to the binding of ZapD to FtsZ, we have rewritten the sections on ZapD binding stoichiometries to both FtsZ-GDP and FtsZ polymers in the presence of GTP (see page 4 lines 14-18 and page 5 lines 15-26).

      iv) Supplementary Figure 5:

      In the representative gel shown, the amount of ZapD in the pellet does not appear to be double compared to 10 and 30 uM concentrations. However, the estimated amount in the plot shown in panel (c) appears to indicate that that ZapD has approximately doubled at 30 uM compared to 10 uM. Please re-check the quantification.

      Without prior staining calibration of the gels, there is no simple quantitative relationship between gel band intensities after Coomassie staining and the amount of protein in a band (Darawshe et al. 1993 Anal Biochem - DOI: 10.1006/abio.1993.1581). The latter point precludes a quantitative comparison of pelleting / SDS-PAGE data and analytical sedimentation measurements.

      v) How can a consistent ratio being maintained be explained in an irregular structure of the toroid? The number of ZapD should be much less compared to FtsZ according to the model.

      See answers to points i) and iii)

      (2) GTPase activity and assembly/disassembly of toroids:

      i) Page 3, Results section: last paragraph:

      What is the explanation or hypothesis for decrease in GTPase activity upon ZapD binding? Given that FtsZ core is not involved in the interaction of the higher order assemblies, what is the probable reason on decrease in GTPase activity upon ZapA binding?

      Excluded volume effects caused by macromolecular crowding, such as high concentrations of Ficoll or dextran, promote the formation of dynamic FtsZ polymer networks (González et al. 2003 J. Biol. Chem - DOI: 10.1074/jbc.M305230200). In these conditions, FtsZ GTPase activity is significantly slowed down compared to the activity observed in FtsZ filaments formed without crowding, leading to a decreased GTPase turnover rate. Similar mechanisms may also apply to assembly reactions in the presence of ZapD (see, for example, Durand-Heredia et al. 2012 J Bacteriol - DOI: 10.1128/JB.0017612).

      ii) How is the decrease in GTPase activity compatible with dynamics of disassembly? Please substantiate on why disassembly is linked to transient interaction with ZapD. Shouldn't disassembly and transient interaction be linked to recovery of GTPase activity rates? 

      iii) Does the decrease in GTPase activity imply a reduced turnover of disassembly of FtsZ to monomers? Hence, how is the reduction in turbidity related to the decrease in GTPase activity? How does the GTPase activity change with time? iv) How can the decrease in GTPase activity with increasing ZapD be explained?

      We conducted GTPase activity assays within the first two minutes following GTP addition, a timeframe that promotes bundle formation. Previous studies, such as those by Durand-Heredia et al. (2012 J Bacteriol - DOI: 10.1128/JB.00176-12), have also indicated a reduction in GTPase activity during the initial moments of bundling. The reviewer’s suggestion that GTPase activity should recover after the disassembly of toroids is valid and warrants further investigation. To test this hypothesis, measuring GTPase activity over extended periods would be necessary. When comparing FtsZ filaments observed in vitro, we found that ZapD-containing FtsZ bundles exhibit decreased GTPase activity. Although we did not measure it directly, we anticipate a reduction in the rate of GTP exchange within the polymer, similar to the behavior of FtsZ bundles formed in the presence of crowders (González et al. 2003 J. Biol. Chem - DOI: 10.1074/jbc.M305230200), which also display a delay in GTPase activity. High levels of ZapD enhance bundling, which may explain the decrease in GTPase activity as ZapD levels increase.

      (3) Treadmilling and FtsZ filament organisation:

      If the FtsZ filaments are cross linked antiparallel, how can tread milling behaviour be explained? Doesn't tread milling imply a directionality of filament orientations in the FtsZ bundles?

      Our model can only suggest filament alignment. The latter is compatible with parallel and antiparallel filament organization.

      The correlation between observed effects on GTPase activity, treadmilling and ZapD interaction will provide an interesting insight to the model.

      Establishing a detailed correlation among these three factors could yield valuable insights into the mechanisms and potential physiological implications of the structural organization of FtsZ polymers influenced by crosslinking proteins and ZapD. To precisely characterize these interactions, further time-resolved assays in solution and reconstituted systems would be necessary, which is beyond the scope of this study.

      (4) Toroid dimensions and intrinsic curvature:

      i) Page 4: What is the correlation between the toroid dimensions and the intrinsic curvature of the FtsZ filaments? Given the thickness of ~ 127 nm, please provide an explanation of how the intrinsic curvature of FtsZ is compatible with both the inner and outer diameters of 500 nm and 380 nm.

      We added a paragraph for clarification (page 6, lines 20-24):

      “Previous studies have shown different FtsZ structures at different concentrations and buffer conditions. FtsZ filaments are flexible and can generate different curvatures ranging from mini rings of ~24 nm to intermediate circular filaments of ~300 nm or toroids of ~500 nm in diameter (reviewed in Erickson and Osawa 2017 Subcell Biochem - DOI: 10.1007/978-3-319-53047-5_5, and Wang et al. 2019 J Biol Chem - DOI: 10.1074/jbc.RA119.009621). It is reasonable to assume that FtsZ filaments can accommodate the toroid shape promoted by ZapD crosslinking.”

      ii) For the curvature of FtsZ filaments to be similar, the length of the filaments in the inner circles of the toroid have to be smaller than those in the outer circles? Is this true? Or are the FtsZ filaments of uniform length throughout?

      Due to the limitations in the resolution of the toroidal structure, we could not accurately measure the length or curvature of the filaments. Considering the FtsZ flexibility, these filaments may exhibit various curvatures and lengths, as previously mentioned.

      iii) Is the ZapD density uniform thought the inner and outer regions of the toroid?

      The heterogeneity found in the structures suggests a difference in ZapD binding densities; however, we lack quantitative data to confirm this. The outer regions are likely more exposed to the attachment of free ZapDs in the surrounding environment, which leads to the recruitment of more ZapDs and the formation of straight bundles. Supplementary Fig. 7b (right) features a zoomed-in image of a toroid adorned with globular densities in the outer areas, which may correspond to ZapD oligomers. Similar characteristics appear in the straight filaments illustrated in the panels of this figure. However, these features are absent or present in significantly lower quantities in toroids with a 1:1 ratio and toroids formed under a 1:6 ratio, suggesting that the external decoration is due to ZapD saturation. Unfortunately, we cannot provide further details on the characteristics of these protein associations.

      (5) Regular arrangement and toroid structure:

      i) Page 4: last section, first sentence: What is meant by 'regular' arrangement here? The word regular will imply a periodicity, which is not a feature of the bundles.

      We have rephrased the sentence in the revised manuscript as follows (page 5, lines 35-36): “Previous studies have visualized bundles with similar features using negative-stain transmission electron microscopy.”

      ii) Similarly, page 6 first sentence mentions about a conserved toroid structure. Which aspects of the toroid structure are conserved and what are the other toroids that are compared with?

      We noted several features that are conserved in the ZapD-mediated toroidal structures, including their diameter, thickness, height, and roundness, as shown in Fig. 2d-e and Supplementary Fig. 6b-c. However, the internal organization of the toroid does not exhibit a periodic or regular structure. We have rephrased this to say: “…resulting in a toroidal structure observed for the first time following the interaction between FtsZ and one of its natural partners in vitro.” (page 7, lines 42-43):

      iii) Discussion, para 1, last sentence: How is the toroid structural correlated with the bacterial cell FtsZ ring? What do the authors mean by 'structural compatibility' with the ring?

      The toroidal structures described in this work are consistent with the intermediate curved conformation of FtsZ polymers observed more generally across bacterial species and are likely to be part of the FtsZ structure responsible for constriction-force generation (Erickson and Osawa 2017 Subcell Biochem - DOI: 10.1007/978-3-319-53047-5_5). In the case of E. coli, if we assume an average of around 5000 FtsZ monomers in the polymeric form (two-thirds of the total found in dividing cells), this number of FtsZ molecules would be enough to encircle the cell around 6-8 times (considering the axial spacing between FtsZ monomers and the cell perimeter), which would be compatible with the structure adopting the form of a discontinuous toroidal assembly. 

      The term “structural compatibility” could be confusing, so we have removed it from the revised text. 

      iv) Discussion, para 2:

      Resemblance with the division ring in bacterial cells is mentioned in paragraph 2, however the features that are compared to claim resemblance comes later in the discussion. It will be helpful to rearrange the sections so that these are presented together.

      We have reorganized the sections following the reviewer’s suggestion.

      (6) CryoET of toroid and interpretation of the tomogram:

      i) Supplementary figure 10: It is not convincing that the indicated densities correspond to ZapD. Is the resolution and the quality of the tomogram sufficient to comment on the localisation of ZapD? It is challenging to see any interpretable difference between FtsZ filament dimers in 10a vs FtsZ+ZapD in panel (b).

      We acknowledge that localizing ZapDs in the structure is a challenge due to the limited resolution of the cryo-ET data (page 7, lines 11-13, 21-24). We have manually labeled putative ZapDs in the data and have done our best to identify the structures reasonably while recognizing the limitations of the segmentation. We use different colors to guide the eye without clearly stating what is or is not a ZapD. However, filaments found in 1:1 and 1:6 ratio toroids have a clear difference in thickness to those observed in the absence of ZapD. The filaments in 1:0 ratio toroids provide a reasonable control for elongation due to the missing wedge and allow us to attribute the extra filament thickness to ZapD densities confidently (page 7, lines 5-12).

      ii) How is it quantified that the elongation in Z is beyond the missing wedge effect? Please include the explanation for this in the methods or the relevant data as Supplementary figure panels.

      The missing wedge effect causes an elongation by a factor of 2 along the Z-axis. This elongation is evident in the filaments of the 1:0 ratio toroids. Consequently, the elongation in the filaments of the 1:1 and 1:6 ratio toroids exceed that observed due to the missing wedge effect. We have also added this information to the methods section (page 17, lines 31-33).

      iii) Segmentation analysis of the tomogram and many method details of analysis and interpretation of the tomography data has not been described. This is essential to understand the reliability of the interpretation of the tomography data.

      We provided thresholds for volume extraction as isosurfaces and clarified how the putative ZapDs are colored in the revised methods section (page 17, line 24-30). However, we could not perform quantitative analysis of the segmented structures.

      (7) Quantification of structural features of the toroid:

      i) Page 5 last sentence mentions that it provides crucial information on the connectivity and length of the filaments. Is it possible to show a quantification of these features in the toroid models?

      Based on our data, we hypothesize that ZapD crosslinks filaments by creating a network of short filaments rather than long ones. These short filaments assemble to form a complete ring. However, the current resolution of the data precludes precise quantification of this process.

      In the revised version, we have changed this last sentence to put the emphasis on the crosslinking geometry instead (page 7, lines 40-43):

      “Cryo-ET imaging of ZapD-mediated FtsZ toroidal structures revealed a preferential vertical stacking and crosslinking of short ZapD filaments, which are also crosslinked laterally and diagonally, allowing for filament curvature and resulting in a toroidal structure observed for the first time following the interaction between FtsZ and one of its natural partners in vitro.”

      ii) In toroids with increasing concentrations, will it be possible to quantify the number of blobs which have been interpreted as ZapD? Is this consistent with the data of FtsZ to ZapD ratios?

      These quantifications would assist in interpreting the data. However, due to the limited resolution of the data, we are reluctant to provide estimates.

      iii) What is the average length of the filaments in the toroid? Can this be quantified from the tomography data? Similarly, can there be an estimation of curvature of the filaments from the data?

      Unfortunately, the complexity of the toroidal structure and the limited resolution we achieved prevent us from providing accurate quantification. We attempted to track and measure the length of the filaments, but this proved challenging due to the high concentration of connections. Regarding curvature, the arrangement of the filaments into toroids makes it difficult to measure the curvature of each filament. Additionally, the filaments are not perfectly aligned, which suggests that there may be various curvatures present.

      iv) What is the average distance between the FtsZ filaments in the toroid? Does this correlate with the ZapD dimensions, when a model has been interpreted as ZapD?

      We measured the spacing (not the center-to-center distance) between filaments in the toroids and showed this in Supplementary Fig. 14b (sky blue). We observed that the distances are very similar to those found for straight bundles (light blue), with a slightly greater variability. We should point out here that the distances were measured in the XY plane to simplify the measurements.

      v) What is the estimate of average inter-filament distances within the toroid? (Similar data as in Figure 13 for bundles?) When the distance between filaments is less, is the angle between ZapD and FtsZ filament axis different from 90 degrees? This might help in validation of interpretation of some of the blobs as ZapD.

      The distances between the filaments presented in Supplementary Figure 14b include those for toroids (1:1 ratio, represented in sky blue) and straight bundles (1:6 ratio, shown in light blue). We focused solely on the distance between filaments in the XY plane and did not differentiate based on the connection angle. Although the distance may vary with changes in the angles between filaments, our data does not permit us to make any quantitative measurements regarding these variations.

      vi) How does the inter filament distance in the toroids compare with the dimensions of ZapD dimers, in the toroids and bundles? Is there a role played by the FtsZ linker in deciding the spacing?

      The dimension of a ZapD dimer is ~7 nm along the longest axis. Huecas et al. (2017 Biophys J - DOI: 10.1016/j.bpj.2017.08.046) estimated an interfilament distance of ~6.5-6.7 nm for toroids of FtsZ from Bacillus subtilis. These authors also observed a difference in this spacing as a function of the linker, assuming that linker length would modulate FtsZ-FtsZ interactions. We observe a similar spacing for double filaments (5.9 ± 0.8 nm) and a longer spacing in the presence of ZapD (7.88 ± 2.1 nm). Previous studies with ZapD did not measure the distance between filaments but hypothesized that distances of 6-12 nm are allowed based on the structure of the protein (Schumacher M. 2017 J Biol Chem - DOI: 10.1074/jbc.M116.773192). Longer linkers may also provide additional freedom to spread the filaments further apart and facilitate a higher degree of variability in the connections by ZapD. This discussion has been included in the revised text (page 6, line 10-18).

      (8) Crosslinking by ZapD and toroid reorganisation by transient interactions:

      i) Page 5, paragraph 2: Presence of putative ZapD decorating a single FtsZ': When ZapD is interacting with 2 FtsZ monomers within the same protofilament, it does not have any more valency to crosslink filaments. How do the authors propose that this can connect nearby filaments?

      We thank the reviewer for raising this interesting question. We see examples of ZapD dimers binding a filament through only one of the monomers, occupying one valency of the interaction and leaving one of the monomers available for another binding. We expect to see higher densities of ZapD in the outer regions of toroids simply because there are no longer (or not as frequent) FtsZ filaments available to be attached and join the overall toroid structure. Assuming that a ZapD dimer could bind the same FtsZ filament, this region would not be able to connect to other nearby filaments via these interactions.

      ii) Page 5: How are the authors coming up with the proposal of a reorganisation of toroid structures to a bundle? Given the extensive cross linking, a transition from a toroid to a bundle has to be a cooperative process and may not be driven by transient interactions. I would imagine that the higher concentration of ZapD will directly result in straight bundles because of the increased binding events of a dimer to one filament.

      Theoretically, this is correct. A certain degree of cooperativity linked to multivalent interactions would also favor the establishment of other ZapD connections. Furthermore, the formation of these structures occurs relatively quickly, within the first two minutes following the addition of GTP. We observed various intermediate structures, ranging from sparse filament bundles to toroids and straight filaments. However, the limited data prevents us from proposing a model that eventually explains the formation of higher-order structures over time.

      iii) Given such a highly cross-linked mesh, how can you justify transient interactions and loss of ZapD leading to disassembly? The possibility that ZapD can diffuse out of such a network seems impossible. Hence, what is the significance of a transient interaction? What is the basis of calling the interactions transient?

      We have noted that the term “transient” used to define the interaction between ZapD and FtsZ seems to generate confusion. Therefore, we have decided to replace this term to improve the readability of our manuscript, which has been edited accordingly.

      iv) Does the spacing between ZapD connections decide the curvature of the toroid?

      The FtsZ linker connected to ZapD molecules could modulate filament spacing and curvature, as previously suggested (Huecas et al. 2017 Biophys J - DOI: 10.1016/j.bpj.2017.08.046; Sundararajan and Goley 2017 J Biol Chem - DOI: 10.1074/jbc.M117.809939, and Sundararajan et al. 2018 Mol Microbiol - DOI: 10.1111/mmi.14081). In our structures, we observe a mixture of curvatures in the internal organization of the toroid. Despite the flexibility of FtsZ, filaments have a preferred curvature that FtsZ would initially determine. However, the amount of ZapD connections will eventually force the filament structure to adapt and align with neighboring filaments, facilitating connections with more ZapDs. Thus, the binding density of ZapD molecules significantly impacts FtsZ curvature rather than the ZapD connections themselves. However, the molecular mechanism describing the link between ZapD binding and polymer curvature remains unsolved.

      v) What is the difference in conditions between supplementary figure 6 and 12? Why is it that toroids are not observed in 12, for the same ratios?

      Both figures show images of samples under the same conditions. At high ZapD concentrations in the sample, we observe a mixture of structures ranging from single filaments, bundles, toroids, and straight bundles. In Supplementary Fig. 6, we have selected images of toroids, while in Supplementary Fig. 12, we have focused on single and double filaments. We aim to compare similar structures at different ZapD concentrations.

      (9) Correlation with in vivo observations:

      What is the approximate ratio of ZapD to FtsZ concentrations in the cell? In this context, within a cell which one - a toroid or bundle - will be preferred?

      Previous studies have estimated that E. coli cells contain approximately 5,000 to 15,000 FtsZ protein molecules, resulting in a concentration of around 3 to 10 µM (Rueda et al. 2003 J Bacteriol - DOI: 10.1128/JB.185.11.3344-3351.2003). Furthermore, only about two-thirds of these FtsZ molecules participate in forming the division ring (Stricker et al. 2002 PNAS - DOI: 10.1073/pnas.052595099). In contrast, ZapD is a low-abundance protein, with only around 500 molecules per cell (DurandHeredia et al. 2012 J Bacteriol - DOI: 10.1128/JB.00176-12), making it a relatively small fraction compared to the FtsZ molecules. Under these circumstances, toroidal structures are more likely to form than straight bundles, as the latter would require significantly higher concentrations of ZapD for proper assembly. We have added these considerations in the revised text (page 11, lines 1-7).

      (10) Interpretation of mZapD results:

      i) What is the experimental proof for weakened stability of the dimer? Rather than weakened stability, does this form a population of only monomeric ZapD or a proportion of non-functional or unfolded dimer? This requires to be shown by AUC or SEC to substantiate the claim of a weakened interface.

      We have provided new AUC results indicating that mZapD is partially monomeric, which suggests a weakened dimerization interface (page 9, line 15-16 and Supp. Fig. 15a). The assays revealed no signs of protein aggregation.

      ii) How does a weaker dimer result in thinner bundles and not toroids? A weaker dimer would imply that the number of ZapD linked to FtsZ will be less than the wild type, leading to less cross linking, which should lead to toroid formation rather than thinner bundles.

      This observation provides the most plausible explanation. However, we did not detect any toroidal structures, even at high concentrations of mZapD. This finding indicates that a more potent dimerization interface is essential for promoting the formation of toroidal structures rather than merely the number of ZapD-FtsZ connections. mZapD presumably has a reduced affinity for FtsZ, which, along with a weaker binding interface, may explain mZapD's inability to facilitate toroid formation.

      iii) This observation would imply that the geometry of the dimeric interaction plays a role in the bending of the FtsZ filaments into toroids? Please comment.

      Our data suggest that the binding density of ZapD to FtsZ polymers is a crucial factor governing the transition from toroidal structures to straight bundles. Toroids form when the polymers have excess free FtsZ (that ZapD does not crosslink). Additional factors, such as the orientation of the interactions, the length of the flexible linker, and the strength of the ZapD dimerization interface, are likely to contribute to these structural reorganizations. However, our current data do not allow for further analysis, and future experiments will be necessary to address these questions.

      (11) Curvature and plasticity of toroid:

      i) What are the factors that stabilise curved protofilaments/toroid structures in the absence of a cross linker, based on earlier studies from B. subtilis. A comparison will be insightful. ii) What is the effect of the linker length between FtsZ globular domain and CTP in the toroid spacing?

      Huecas et al. 2017 (Biophys J - DOI: 10.1016/j.bpj.2017.08.046) concluded that the disordered CTL of FtsZ serves as a spacer that modulates the self-organization of FtsZ polymers. They proposed that this intrinsically disordered CTL, which spans the gap between protofilament cores, provides approximately 70 Å of lateral spacing between the curved Bacillus subtilis FtsZ (BsFtsZ), forming toroidal structures. In contrast, the parallel filaments of tailless BsFtsZ mutants, which have a reduced spacing of 50 Å, will likely stick together, resulting in the straight bundles observed. In the full-length BsFtsZ filament, the flexibility allowed by the lateral association favors the coalescence of these curved protofilaments, leading to the formation of toroidal structures. 

      The role of the C-terminal tail of FtsZ in E. coli is critical for its functionality (Buske and Levin 2012 J Biol Chem - DOI: 10.1074/jbc.M111.330324). However, its structural involvement in complex formations remains unclear. Research indicates that any disordered peptide between 43 and 95 amino acids in length can function as a viable linker, while peptides that are significantly shorter or longer impede cell division (Gardner et al. 2013 Mol Microbiol - DOI: 10.1111/mmi.12279). Studies in E. coli and B. subtilis suggest that intrinsically disordered CTLs play a role in determining FtsZ assembly and function in vivo, and this role is dependent on the length, flexibility, and disorder of the tails. These aspects still require further exploration.

      iii) How is it concluded that the concentration of ZapD is modulating the behaviour of the toroid structure? ZapD as a molecule does not have much room for conformational flexibility beyond a few angstroms, in the absence of long flexible regions. Rather, shouldn't the linker length of FtsZ to the CTP decide the plasticity of the toroid?

      The length and flexibility of the linker can significantly influence structural interactions. As previously mentioned, a longer linker will likely enhance the range of interaction distances and orientations. However, specific interaction of ZapD and FtsZ is stronger than non-specific electrostatic FtsZ-FtsZ interactions, and this is not solely due to the flexibility of the linker. Instead, it can modulate the formation of either a toroidal structure or straight bundles.

      iv) "a minor free energy perturbation to bring about significant changes in the geometry of the fibers due to modifications in environmental conditions" - this sentence is not clear to me. How did the data described in the paper relate to minor free energy perturbations and how do environmental conditions affect this?

      This sentence aimed to convey the notion of polymorphism in FtsZ polymers. We acknowledge that the original version may have been unclear, so we have removed it in the new version of the manuscript (page 12, lines 1-2).

      (12) Missing controls:

      i) Supplementary Figure 2a: Interaction between ZapD and FtsZ: what was the negative control used in this experiment? Use of FtsZ with the CTP deletion or ZapD specific mutations will help in confirming that the Kd estimation is indeed driven by a specific interaction.

      Negative controls correspond to FtsZ and ZapD alone.

      ii) In a turbidity measurement, how will you distinguish between ZapD mediated bundling, ZapD independent bundling and FtsZ filaments alone? Here again, having a data with non-interacting mutational partners will make the data more reliable.

      The turbidity signal of individual proteins in the absence and presence of GTP is indistinguishable from that of the buffer. We have indicated this in the figure legend.

      iii) Control experiments to show that mZapD is folded (see point below) and to indeed prove that it is monomeric is missing.

      We have included the missing AUC data in the supplementary information (Supp Fig 15a).

      Minor points:

      -  Page 2, para 4: beta-sheet domain (instead of beta-strand)

      Done.

      -  Fig 2a and b: Why is a ratio mentioned in Figure 2a legend? I understood these images as individual proteins at 10 uM concentrations.

      That was a typing error; it corresponds to two individual proteins at 10 µM concentrations. 

      -  Fig 2. Y-axis - spelling of frequency (change in all figures where applicable)

      Corrected.

      -  Supplementary Figure 5: FtsZ 5 uM - change u to micro symbol. FtsZ - t is missing

      Corrected. 

      -  Molecular weight marker is xx. What does xx stand for?

      Corrected. 

      -  Fig 1: Units for GTPase activity on the y-axis is missing.

      Done.

      -  Suppl Fig 3: How was the normalisation carried out for the turbidity data?

      We have explained it the revised methods section. 

      -  Page 4, line 5: p missing in ZapD

      Done. 

      -  Page 5: paragraph 1, last sentence: stabilised or established?

      Done.

      -  Page 6: 3rd sentence from last: correct the sentence (one ZapD two FtsZ)

      Corrected. 

      -  Page 14: Fluorescence microscopy and FRAP experiments have not been described in the manuscript. Hence, these are not required in the methods.

      Corrected. 

      -  Please include representative gels of purified protein samples used in the assay for sample quality control.

      Controls for each protein are shown in Supplementary Fig. 5a as “control samples” corresponding to 5 µM of each protein before centrifugation.

      Reviewer #3 (Recommendations for the authors):

      Fig. S2a confirms and quantitates the interaction of ZapD with FtsZ-GDP monomers by F.A. It shows a surprisingly high Kd of ~10 µM. This seems important but it is ignored in the overall interpretation. Fig. S2b (FCS) suggests an even weaker interaction, but this may reflect higher order aggregates.

      As the reviewer points out, the interaction between ZapD and FtsZ in the GDP form is weak, consistent with the need for high concentrations of ZapD to form FtsZ macrostructures in the presence of GTP.

      We did not observe the formation of ZapD aggregates, even at higher protein (Author response image 1A) and salt (Author response image 1B) concentrations.

      Author response image 1.

      A) Sedimentation velocity (SV) profiles of ZapD over a concentration range of 2 to 30 µM in 50 mM KCl, 5 mM MgCl2, Tris-HCl pH 7. B) SV profiles of ZapD at 10 µM in different ionic strength concentrations in buffer 50-500 mM KCl, 5 mM MgCl2, 50 mM Tris-HCl pH 7. Abs280 measurements were collected at 48,000 rpm and 20 ºC. 

      Describing their assembly of toroids the authors state "Upon adding equimolar amounts of ZapD, corresponding to the subsaturating ZapD binding densities described in the previous section". My reading of Fig. 1b and S5 is that FtsZ is almost fully saturated at 1:1 concentration; In S5a at 5:5 µM about 25% of each is in the pellet, which is near 1:1 saturation. It is certainly >50% saturated. Shouldn't this be clarified to read "slightly substoichiometric. Of course, that undermines the identification of ZapD as such a substoichiometric number.

      We have rephrased the sentence following the reviewer’s suggestions to clarify matters (page 5, lines 39-40).

      The cryoET images in Fig. 3 are an average of five slices with a total thickness of 32 nm. The circular "short filaments..almost parallel" are therefore not single 5 nm diameter FtsZ filaments but must be alignment of filaments axially into sheets (or belts, the axial structure shown in Fig. S8e, discussed next). Importantly, the authors indicate "connections between filaments" by red arrows. This seems wrong for two reasons. (1) The "connections" are very sparse, and therefore not consistent with the near saturation of FtsZ by ZapD. (2) To show up in the 32 nm averaged slice, connections from multiple filaments would have to be aligned. Fig. 3e is a "view of the segmented toroidal structure." I think it shows sheets of filaments as noted above, and the suggested "crosslinks" are again very sparse and no more convincing.

      We thank the reviewer for pointing this out. This was an error on our part, which we have corrected in the figure legend of the revised version of the manuscript. The tomographic slice shown in Fig. 3a is an average of 5 slices, each with a pixel size of 0.86 nm, corresponding to a pixel size of 4.31 nm. It therefore corresponds to the thickness of a single FtsZ filament. The few red arrows indicate lateral connections between filaments, and as discussed earlier, ZapDs also crosslinks FtsZ filaments vertically, giving rise to the elongated structures observed in the Z-direction.

      All 3-D reconstructions and segmented renditions should have a scale bar. The axial cylindrical sheets seem to be confirmed and qualified in Fig. S8e. The cylindrical sheets are not continuous, but seem to consist of belt-like filaments that are ~8-10 nm wide in the axial direction. Adjacent belts are separated axially by ~5 nm gaps, and radially by 4-20 nm. The densest filaments in the projection image Fig. 3b are probably an axial superposition of 2-3 belts, while the lighter filaments may be individual belts.

      Fig. 4 shows a higher number of crosslinks but nowhere near a 1:1 stoichiometry. Most importantly to me, the identification of crosslinks vs filaments seems completely arbitrary. For example, if one colored grey all of the densities I 4a right panel, I would have no way to duplicate the distinctions shown in red and blue. Even if we accept the authors' distinction, it does not provide much structural insight. Continuous bands or sheets are identified as FtsZ, without any resolution of substructure, and any density outside these bands is ZapD. The spots identified as ZapD seem randomly dispersed and much too sparse to include all the ~1:1 ZapD.

      We appreciate the reviewer's comments. Scale bars are present in the tomographic slices but not in the 3D views, as these are perspective views, and it would be inappropriate to include scale bars. To provide context for the images, we added the dimensions of the toroids and toroid sections to the figure legends. 

      As previously mentioned, the resolution of our data limits our ability to accurately segment ZapD densities, especially in the Z direction. In Fig. 4, we have done our best to segment the ZapD densities at the top and sides of the FtsZ filaments, but many densities have been missed. We have clarified this point in the text and in the figure legend. We have clarified this point in both the text and the figure legends. This preliminary annotated view is meant to help illustrate the formation of the toroids. In Fig. 3, we have labeled only a few arrows to highlight the lateral connections between the FtsZ filaments; however, there are many more connections than those indicated.

      Fig. S12 explores the effect of increasing ZapD to 1:6, and the authors conclude "the high concentration of ZapD molecules increased the number of links between filaments and ultimately promoted the formation of straight bundles." However, the binding sites on FtsZ are already nearly saturated at 10:10.

      We cannot assume that all FtsZ binding sites are present at a 1:1 ratio. Our pelleting assay confirms the presence of both proteins in the pellet, but we should be cautious about quantification due to the limitations of this technique. Based on our cryo-EM experiments, the amount of ZapD associated with these structures is much lower. We hypothesize that ZapD proteins sediment with the large FtsZ structures, acting as an external decoration for the toroids. A single ZapD monomer may be bound to multiple outer filaments of the structures, which could effectively increase the total µM concentration observed in the pelleting assay. This situation may explain the enrichment of ZapD in the pellet at high concentrations, when theoretically only a 1:1 ratio should be possible. We have observed external decorations of ZapD at high concentrations (see Supplementary Fig. 6). We believe that the pelleting assay simplifies the system and should be used to complement the cryo-EM images.

      Minor points.

      In the Intro "..to follow a treadmilling behavior, similar to that of actin filaments.9-13." These refs have little to do with treadmilling. I suggest: Wagstaff..Lowe mBio 2017; Du..Lutkenhaus PNAS 2018; Corbin Erickson BJ 2020; Ruis..Fernandez-Tornero Plos Biol 2022.

      Following the reviewer’s suggestions, we have modified the references in the revised version. 

      The authors responded to a query during review stating that the concentration of ZapD always refers to the monomer subunit. That seems certainly the case for Fig. S1, but the caption to Fig. 1a confuses the stoichiometry issue: "expecting (sic) at around 2:1 FtsZ:ZapD." Perhaps it could be clarified by stating that the Fig. shows only half the FtsZ's occupied. But in Fig. 1b the absorbance reaches its maximum at equimolar FtsZ and ZapD. That means that all FtsZ's are bound to a ZapD monomer. Why not draw the model in 1A show that? Fig. S5 is also consistent with this 1:1 stoichiometry. And this might be the place to contrast the planar model with the stacked model suggested by Fig. 5 where the two FtsZ filaments are ~8 nm apart, and the ZapD bridging them is on top.

      We have revised the legend for Fig. 1a to improve its readability. In Fig. 1b, the absorbance data indicate that most FtsZ proteins form macrostructures; however, this does not imply that all FtsZ proteins are bound to ZapDs. Our findings demonstrate that this binding only occurs in the case of straight bundles.

      It may help to note that some previous studies have expressed the concentration of ZapD as the dimer. E.g., Roach..Khursigara 2016 found maximal pelleting at FtsZ:ZapD(dimer) of 2:1 (their Fig. 3), completely consistent with the 1:1 FtsZ:ZapD(monomer) in the present study.

      We recognize this discrepancy in the literature. Therefore, throughout the manuscript, the molar concentrations of both proteins are expressed in terms of the FtsZ and ZapD monomer species.

    1. Author response:

      The following is the authors’ response to the original reviews.

      Public Reviews:

      Reviewer 1 (Public Review):

      Summary:

      The authors propose that the energy landscape of animals can be thought of in the same way as the fundamental versus realized niche concept in ecology. Namely, animals will use a subset of the fundamental energy landscape due to a variety of factors. The authors then show that the realized energy landscape of eagles increases with age as the animals are better able to use the energy landscape. Strengths:

      This is a very interesting idea and that adds significantly to the energy landscape framework. They provide convincing evidence that the available regions used by birds increase with size.

      Weaknesses:

      Some of the measures used in the manuscript are difficult to follow and there is no mention of the morphometrics of birds or how these change with age (other than that they don’t change which seems odd as surely they grow). Also, there may need to be more discussion of other ontogenetic changes such as foraging strategies, home range size etc.

      We thank reviewer 1 for their interest in our study and for their constructive recommendations. We have included further discussions of these points in the manuscript and outline these changes in our responses to the detailed recommendations below.

      Reviewer 2 (Public Review):

      Summary:

      With this work, the authors tried to expand and integrate the concept of realized niche in the context of movement ecology by using fine-scale GPS data of 55 juvenile Golden eagles in the Alps. Authors found that ontogenic changes influence the percentage of area flyable to the eagles as individuals exploit better geographic uplifts that allow them to reduce the cost of transport.

      Strengths:

      Authors made insightful work linking changes in ontogeny and energy landscapes in large soaring birds. It may not only advance the understanding of how changes in the life cycle affect the exploitability of aerial space but also offer valuable tools for the management and conservation of large soaring species in the changing world.

      Weaknesses:

      Future research may test the applicability of the present work by including more individuals and/or other species from other study areas.

      We are thankful to reviewer 2 for their encouragement and positive assessment of our work. We have addressed their specific recommendations below.

      Recommendations for the authors:

      Reviewer 1 (Recommendations For The Authors):

      I found this to be a very interesting paper which adds some great concepts and ideas to the energy landscape framework. The paper is also concise and well-written. While I am enthusiastic about the paper there are areas that need clarifying or need to be made clearer. Specific comments below:

      Line 64: I disagree that competition is the fundamental driver of the realized niche. In some cases, it may be but in others, predation may be far more important (as an example).

      We agree with this point and have now clarified that competition is an example of a driver of the realized niche. We have also included predation as another example:

      "However, just as animals do not occupy the entirety of their fundamental Hutchinsonian niche in reality [1], for example due to competition or predation risk, various factors can contribute to an animal not having access to the entirety of its fundamental movement niche."

      Intro: I think the authors should emphasize that morphological changes with ontogeny will change the energy landscape for many animals. It may not be the case specifically with eagles but that won’t be true for other animals. For example, in many sharks, buoyancy increases with age.

      We agree and have now clarified that the developmental processes that we are interested in happen in addition to morphological changes:

      "In addition to morphological changes, as young animals progress through their developmental stages, their movement proficiency [2] and cognitive capabilities [3] improve and memory manifests [4]."

      Line 91-93: The idea that birds fine-tune motor performance to take advantage of updrafts is a very important one to the manuscript and should be discussed in a bit more detail. How? At the moment there is a single sentence and it doesn’t even have a citation yet this is the main crux of the changes in realized energy landscape with age. This point should be emphasized because, by the end of the introduction, it is not clear to me why the landscape should be cheaper as the birds age?

      Thank you for pointing out this missing information. We have now added examples to clarify how soaring birds fine-tune their motor performance when soaring. These include for example adopting high bank angles in narrow and weak thermals [5] and reducing gliding airspeed when the next thermal has not been detected [6]:

      "Soaring flight is a learned and acquired behavior [7, 8], requiring advanced cognitive skills to locate uplifts as well as fine-tuned locomotor skills for optimal adjustment of the body and wings to extract the most energy from them, for example by adopting high bank angles in narrow and weak thermals [5] and reducing gliding airspeed when the next thermal has not been detected [6]."

      Results:

      Line 106: explain the basics of the life history of the birds in the introduction. I have no idea what emigration refers to or the life history of these animals.

      Thank you for pointing out the missing background information. We have now added this

      information to the introduction:

      "We analyzed 46,000 hours of flight data collected from bio-logging devices attached to 55 wild-ranging golden eagles in the Central European Alps. These data covered the transience phase of natal dispersal (hereafter post-emigration). In this population, juveniles typically achieve independence by emigrating from the parental territory within 4-10 months after fledging. However, due to the high density of eagles and consequently the scarcity of available territories, the transience phase between emigration and settling by eventually winning over a territory is exceptionally long at well over 4 years. Our hypothesis posited that the realized energy landscape during this transience phase gradually expands as the birds age."

      What I still am having a hard time understanding is the flyability index. Is this just a measure of the area animals actively select and then the assumption that it’s a good region to fly within?

      We have modified our description of the flyability index for more clarity. In short, we built a step-selection model and made predictions using this model. The predictions estimate the probability of use of an area based on the predictors of the model. For the purpose of our study and what our predictors were (proxies for uplift + movement capacity), we interpreted the predicted values as the "flyability index". We have now clarified this in the methods section:

      "We made the predictions on the scale of the link function and converted them to values between 0 and 1 using the inverse logit function [9]. These predicted values estimated the probability of use of an area for flying based on the model. We interpreted these predicted values as the flyability index, representing the potential energy available in the landscape to support flight, based on the uplift proxies (TRI and distance to ridge line) and the movement capacity (step length) of the birds included in the model."

      It might also be useful to simply show the changes in the area the animals use with age as well (i.e. a simple utilization distribution). This should increase in age for many animals but would also be a reflection of the resources animals need to acquire as they get older.

      We have now added the figure S2 to the supplementary material. This plot was created by calculating the cumulative area used by the birds in each week after emigration. This was done by extracting the commuting flights for each week, converting these to line objects, overlapping the lines with a raster of 100*100 m cell size, counting the number of overlapping cells and calculating the area that they covered. We did not calculate UDs or MCPs because the eagles seem to be responding to linear features of the landscape, e.g. preferring ridgelines and avoiding valleys. Using polygons to estimate used areas would have made it difficult to ensure that decision-making with regards to these linear features was captured.

      In a follow-up project, a PhD student in the golden eagle consortium is exploring the individuals’ space use after emigration considering different environmental and social factors. The outcome of that study will further complete our understanding of the post-emigration behavior of juvenile golden eagles in the Alps.

      How much do the birds change in size over the ontogeny measured? This is never discussed.

      Thank you for bringing up this question. The morphometrics of juvenile golden eagles are not significantly different from the adults, except in the size of culmen and claws [10]. Body mass changes after fledging, because of the development of the pectoral muscles as the birds start flying. Golden eagles typically achieve adult-like size and mass within their natal territory before emigration, at which time we started quantifying the changes in energy landscape. Given our focus on post-emigration flight behavior, we do not expect any significant changes in size and body mass during our study period. We now cover this in the discussion:

      "Juvenile golden eagles complete their morphological development before gaining independence from their parents, with their size and wing morphology remaining stable during the post-emigration phase [10, 11]. Consequently, variations in flyability of the landscape for these birds predominantly reflect their improved mastery of soaring flight, rather than changes in their morphology."

      Discussion:

      Line 154: Could the increase in step length also be due to changes in search strategies with age? e.g. from more Brownian motion when scavenging to Levy search patterns when actively hunting?

      This is a very good point and we tried to look for evidence of this transition in the tracking data. We explored the first passage time for two individuals with a radius of 50 km to see if there is a clear transition from a Brownian to a Levy motion. The patterns that emerge are inconclusive and seem to point to seasonality rather than a clear transition in foraging strategy (Author response image 1). We have modified our statement in the discussion about the change in preference of step lengths indicating improve flight ability, to clarify that it is speculative:

      Author response image 1.

      First passage times using a 50 km radius for two randomly selected individuals.

      "Our findings also reveal that as the eagles aged, they adopted longer step lengths, which could indicate an increasing ability to sustain longer uninterrupted flight bouts."

      Methods:

      Line 229: What is the cutoff for high altitude or high speed?

      We used the Expectation-maximization binary clustering (EMbC) method to identify commuting flights. The EmbC method does not use hard cutoffs to cluster the data. Each data point was assigned to the distribution to which it most likely belonged based on the final probabilities after multiple iterations of the algorithm. Author response image 2 shows the distribution of points that were either used or not used based on the EmbC classification.

      Author response image 2.

      Golden eagle tracking points were either retained (used) or discarded (not used) for further data analysis based on the EmbC algorithm. The point were clustered based on ground speed and height above ground.

      Figure 1: The figure captions should stand on their own but in this case there is no information as to what the tests are actually showing.

      We have now updated the caption to provide information about the model:

      "Coefficient estimates of the step selection function predicting probability of use as a function of uplift proxies, week since emigration, and step length. All variables were z-transformed prior to modeling.

      The error bars show 95% confidence intervals."

      Reviewer 2 (Recommendations For The Authors):

      First, I want to congratulate you on this fantastic work. I enjoyed reading it. The manuscript is clear and well-written, and the findings are sound and relevant to the field of movement ecology. Also, the figures are neatly presented and easy to follow.

      I particularly liked expanding the old concept of fundamental vs realized niche into a movement ecology context. I believe that adds a fresh view into these widely accepted ecological assumptions on species niche, which may help other researchers build upon them to better understand movement "realms" on highly mobile animals in a rapidly changing world.

      I made some minor comments to the manuscript since it was hard to find important weaknesses in it, given the quality of your work. However, there was a point in the discussion that I feel deserves your attention (or rather a reflection) on how major biological events such as moulting could also influence birds to master the flying and exploitation of the energy landscape. You may find my suggestion quite subjective, but I think it may help expand your idea for future works and, what is more, link concepts such as energy landscapes, ontogeny, and important life cycle events such as moulting in large soaring birds. I consider this relevant from a mechanistic perspective to understand better how individuals negotiate all three concepts to thrive and persist in changing environments and to maximise their

      fitness.

      Once again, congratulations on this excellent piece of research.

      We thank the reviewer for their enthusiasm about our work and for bringing up important points about the biology of the species. Our detailed response are below.

      MINOR COMMENTS:

      (Note: Line numbers refer to those in the PDF version provided by the journal).

      Line 110: Distinguished (?)

      corrected

      Line 131: Overall, I agree with the authors’ discussion and very much liked how they addressed crucial points. However, I have a point about some missing non-discussed aspects of bird ecology that had not been mentioned.

      The authors argue that morphological traits are less important in explaining birds’ mastery of flight (thus exploiting all available options in the landscape). However, I think the authors are missing some fundamental aspects of bird biology that are known to affect birds’ flying skills, such as moult.

      The moulting process affects species’ flying capacity. Although previous works have not assessed moults’ impact on movement capacity, I think it is worth including the influence of flyability on this ecologically relevant process.

      For instance, golden eagles change their juvenile plumage to intermediate, sub-adult plumage in two or three moult cycles. During this process, the moulting process is incomplete and affects the birds’ aerodynamics, flying capacity, and performance (see Tomotani et al. 2018; Hedenström 2023). Thus, one could expect this process to be somewhat indirectly linked to the extent to which birds can exploit available resources.

      Hedenström, A. (2023). Effects of wing damage and moult gaps on vertebrate flight performance.

      Journal of Experimental Biology, 226(9), jeb227355. Tomotani, B. M., Muijres, F. T., Koelman, J., Casagrande, S., & Visser, M. E. (2018). Simulated moult reduces flight performance, but overlap with breeding does not affect breeding success in a longdistance migrant. Functional Ecology, 32(2), 389-401.

      We thank the reviewer for bringing up this relevant topic. We explored the literature listed by the reviewer and also other sources. We came to the conclusion that moulting does not impact our findings. In our study, we included data for eagles that had emigrated from the natal territories, with their fully grown feathers in juvenile plumage. The moulting schedule in juvenile birds is similar to that of adults: the timing, intensity, and sequence of feathers being replaced is consistent every year (Author response image 3). For these reasons, we do not believe that moulting stage noticeably impacts flight performance at the scale of our study (hourly flights). Fine details of soaring flight performance (aerodynamics within and between thermals) could differs during moulting of different primary and secondary feathers, but this is something that would occur every time the eagle replaces these feather and we do not expect it to be any different for juveniles. Such fine scale investigations are outside the scope of this study.

      Author response image 3.

      Moulting schedule of golden eagles [12]

      Lines 181-182: I don’t think trophic transitions rely only on individual flying skill changes. Furthermore, despite its predominant role, scavenging does not mean it is the primary source of food acquisition in golden eagles. This also depends on prey availability, and scavenging is an auxiliary font of easy-to-catch food.

      Scavenging implies detecting carcasses. Should this carcass appearance occur in highly rugged areas, the likelihood of detection also reduces notably. This is not to say that there are not more specialized carrion consumers, such as vultures, that may outcompete eagles in searching for such resources more

      efficiently.

      In summary, I don‘t think such transition relies only on flying skills but on other non-discussed factors such as knowledge accumulation of the area or even the presence of conspecifics.

      Line 183: This is precisely what I meant with my earlier comment.

      Thank you for the discussion on the interaction between flight development and foraging strategy. We explored the transition from scavenging to hunting above as a response to Reviewer 1, but did not find a clear transition. This is in line with your comment that the birds probably use both scavenging and hunting methods opportunistically.

      Lines 193-195: I will locate this sentence somewhere in this paragraph. As it is now, it seems a bit out of context. It could be a better fit at the end of the first point in line 203.

      Thank you for pointing out the issue with the flow. We have now added a transitional sentence before this one to improve the paragraph. The beginning of the conclusion now reads as follows, with the new sentence shown in boldface.

      "Spatial maps serve as valuable tools in informing conservation and management strategies by showing the general distribution and movement patterns of animals. These tools are crucial for understanding how animals interact with their environment, including human-made structures. Within this context, energy landscapes play an important role in identifying potential areas of conflict between animals and anthropogenic infrastructures such as wind farms. The predictability of environmental factors that shape the energy landscape has facilitated the development of these conservation tools, which have been extrapolated to animals belonging to the same ecological guild traversing similar environments."

      References

      (1) Colwell, R. K. & Rangel, T. F. Hutchinson’s duality: The once and future niche. Proceedings of the National Academy of Sciences 106, 19651–19658. doi:10.1073/pnas.0901650106 (2009).

      (2) Corbeau, A., Prudor, A., Kato, A. & Weimerskirch, H. Development of flight and foraging behaviour in a juvenile seabird with extreme soaring capacities. Journal of Animal Ecology 89, 20–28. doi:10.1111/1365-2656.13121 (2020).

      (3) Fuster, J. M. Frontal lobe and cognitive development. Journal of neurocytology 31, 373–385.

      doi:10.1023/A:1024190429920 (2002).

      (4) Ramsaran, A. I., Schlichting, M. L. & Frankland, P. W. The ontogeny of memory persistence and specificity. Developmental Cognitive Neuroscience 36, 100591. doi:10.1016/j.dcn.2018.09.002 (2019).

      (5) Williams, H. J., Duriez, O., Holton, M. D., Dell’Omo, G., Wilson, R. P. & Shepard, E. L. C. Vultures respond to challenges of near-ground thermal soaring by varying bank angle. Journal of Experimental Biology 221, jeb174995. doi:10.1242/jeb.174995 (Dec. 2018).

      (6) Williams, H. J., King, A. J., Duriez, O., Börger, L. & Shepard, E. L. C. Social eavesdropping allows for a more risky gliding strategy by thermal-soaring birds. Journal of The Royal Society Interface 15, 20180578. doi:10.1098/rsif.2018.0578 (2018).

      (7) Harel, R., Horvitz, N. & Nathan, R. Adult vultures outperform juveniles in challenging thermal soaring conditions. Scientific reports 6, 27865. doi:10.1038/srep27865 (2016).

      (8) Ruaux, G., Lumineau, S. & de Margerie, E. The development of flight behaviours in birds. Proceedings of the Royal Society B: Biological Sciences 287, 20200668. doi:10.1098/rspb.2020.

      0668 (2020).

      (9) Bolker, B., Warnes, G. R. & Lumley, T. Package gtools. R Package "gtools" version 3.9.4 (2022).

      (10) Bortolotti, G. R. Age and sex size variation in Golden Eagles. Journal of Field Ornithology 55,

      54–66 (1984).

      (11) Katzner, T. E., Kochert, M. N., Steenhof, K., McIntyre, C. L., Craig, E. H. & Miller, T. A. Birds of the World (eds Rodewald, P. G. & Keeney, B. K.) chap. Golden Eagle (Aquila chrysaetos), version 2.0. doi:10.2173/bow.goleag.02 (Cornell Lab of Ornithology, Ithaca, NY, USA, 2020).

      (12) Bloom, P. H. & Clark, W. S. Molt and sequence of plumages of Golden Eagles and a technique for in-hand ageing. North American Bird Bander 26, 2 (2001).

    1. Author response:

      The following is the authors’ response to the original reviews

      Public Reviews:

      Reviewer #1 (Public Review):

      Summary:

      In the manuscript by Tie et.al., the authors couple the methodology which they have developed to measure LQ (localization quotient) of proteins within the Golgi apparatus along with RUSH based cargo release to quantify the speed of different cargos traveling through Golgi stacks in nocodazole induced Golgi ministacks to differentiate between cisternal progression vs stable compartment model of the Golgi apparatus. The debate between cisternal progression model and stable compartment model has been intense and going on for decades and important to understand the basic way of function/organization of the Golgi apparatus. As per the stable compartment model, cisterna are stable structures and cargo moves along the Golgi apparatus in vesicular carriers. While as per cisternal progression model, Golgi cisterna themselves mature acquiring new identity from the cis face to the trans face and act as transport carriers themselves. In this work, authors provide a missing part regarding intra-Golgi speed for transport of different cargoes as well as the speed of TGN exit and based on the differences in the transport velocities for different cargoes tested favor a stable compartment model. The argument which authors make is that if there is cisternal progression, all the cargoes should have a similar intra-Golgi transport speed which is essentially the rate at which the Golgi cisterna mature. Furthermore, using a combination of BFA and Nocodazole treatments authors show that the compartments remain stable in cells for at least 30-60 minutes after BFA treatment.

      Strengths:

      The method to accurately measure localization of a protein within the Golgi stack is rigorously tested in the previous publications from the same authors and in combination with pulse chase approaches has been used to quantify transport velocities of cargoes through the Golgi. This is a novel aspect in this paper and differences in intra-Golgi velocities for different cargoes tested makes a case for a stable compartment model.

      Weaknesses:

      Experiments are only tested in one cell line (HeLa cells) and predominantly derived from experimental paradigm using RUSH assays where a secretory cargo is released in a wave (not the most physiological condition) and therefore additional approaches would make a more compelling case for the model.

      We have added datasets from 293T cells in the revamped manuscript.

      Reviewer #2 (Public Review):

      Summary:

      This manuscript describes the use of quantitative imaging approaches, which have been a key element of the labs work over the past years, to address one of the major unresolved discussions in trafficking: intra-Golgi transport. The approach used has been clearly described in the labs previous papers, and is thus clearly described. The authors clearly address the weaknesses in this manuscript and do not overstate the conclusions drawn from the data. The only weakness not addressed is the concept of blocking COPI transport with BFA, which is a strong inhibitor and causes general disruption of the system. This is an interesting element of the paper, which I think could be improved upon by using more specific COPI inhibitors instead, although I understand that this is not necessarily straightforward.

      I commend the authors on their clear and precise presentation of this body of work, incorporating mathematical modelling with a fundamental question in cell biology. In all, I think that this is a very robust body of work, that provides a sound conclusion in support of the stable compartment model for the Golgi.

      General points:

      The manuscript contains a lot of background in its results sections, and the authors may wish to consider rebalancing the text: The section beginning at Line 175 is about 90% background and 10% data. Could some data currently in supplementary be included here to redress this balance, or this part combined with another?

      In the revamped manuscript, we have moved the background information on rapid partitioning and rim progression models to the Introduction.

      Reviewer #3 (Public Review):

      The manuscript by Tie et al. provides a quantitative assessment of intra-Golgi transport of diverse cargos. Quantitative approaches using fluorescence microscopy of RUSH synchronized cargos, namely GLIM and measurement of Golgi residence time, previously developed by the author's team (publications from 20216 to 2022), are being used here.

      Most of the results have been already published by the same team in 2016, 2017, 2020 and 2021. In this manuscript, very few new data have been added. The authors have put together measurements of intra-Golgi transport kinetics and Golgi residence time of many cargos. The quantitative results are supported by a large number of Golgi mini-stacks/cells analyzed. They are discussed with regard to the intra-Golgi transport models being debated in the field, namely the cisternal maturation/progression model and the stable compartments model. However, over the past decades, the cisternal progression model has been mostly accepted thanks to many experimental data.

      The authors show that different cargos have distinct intra-Golgi transport kinetics and that the Golgi residence time of glycosyltransferases is high. From this and the experiment using brefeldinA, the authors suggest that the rim progression model, adapted from the stable compartments model, fits with their experimental data.

      Strengths:

      The major strength of this manuscript is to put together many quantitative results that the authors previously obtained and to discuss them to give food for thought about the intraGolgi transport mechanism.

      The analysis by fluorescence microscopy of intra-Golgi transport is tough and is a tour de force of the authors even if their approach show limitations, which are clearly stated. Their work is remarkable in regards to the numbers of Golgi markers and secretory cargos which have been analyzed.

      Weaknesses:

      As previously mentioned, most of the data provided here were already published and thus accessible for the community. Is there is a need to publish them again?

      The authors' discussion about the intra-Golgi transport model is rather simplistic. In the introduction, there is no mention of the most recent models, namely the rapid partitioning and the rim progression models. To my opinion, the tubular connections between cisternae and the diffusion/biochemical properties of cargos are not enough taken into account to interpret the results. Indeed, tubular connections and biochemical properties of the cargos may affect their transit through the Golgi and the kinetics with which they reach the TGN for Golgi exit.

      Nocodazole is being used to form Golgi mini-stacks, which are necessary to allow intra-Golgi measurement. The use of nocodazole might affect cellular homeostasis but this is clearly stated by the authors and is acceptable as we need to perturb the system to conduct this analysis. However, the manual selection of the Golgi mini-stack being analyzed raises a major concern. As far as I understood, the authors select the mini-stacks where the cargo and the Golgi reference markers are clearly detectable and separated, which might introduce a bias in the analysis.

      The terms 'Golgi residence time ' is being used but it corresponds to the residence time in the trans-cisterna only as the cargo has been accumulated in the trans-Golgi thanks to a 20{degree sign}C block. The kinetics of disappearance of the protein of interest is then monitored after 20{degree sign}C to 37{degree sign}C switch.

      Another concern also lies in the differences that would be introduced by different expression levels of the cargo on the kinetics of their intra-Golgi transport and of their packaging into post-Golgi carriers.

      Please see below for our replies to intra-Golgi transport models, the Golgi residence time, and different expression levels of cargos.

      Recommendations for the authors:

      Reviewer #1 (Recommendations For The Authors):

      The data shown by the authors to measure differential intra Golgi velocities based on previously established methodology make a case for a stable compartment model, however more data is needed to make a complete story and the clarity of presentation can be improved.

      We sincerely appreciate the reviewer's insightful, detailed, and constructive feedback. Your thoughtful comments have helped us refine our analyses, clarify key points, and strengthen the overall quality of our manuscript. We are grateful for the time and effort you have dedicated to reviewing our work and providing valuable suggestions. Your input has been instrumental in improving both the scientific rigor and presentation of our findings. Thank you for your thorough and thoughtful review.

      Main points:

      (1) Along with the studies in yeast, which authors describe in this paper, the main evidence for cisternal maturation model in mammalian cells comes from Bonfanti et.al., (https://doi.org/10.1016/S0092-8674(00)81723-7), which used EM to visualize a wave of Collagen through Golgi stacks. It is therefore important this work needs to include collagen as one of the cargos tested. Can the authors use the RUSH-Col1AGFP (see: https://doi.org/10.1083/jcb.202005166) as a cargo to monitor intra-Golgi velocities?

      I understand that Hela cells are not professional collagen-secreting, but the authors can use U2OS cells to measure collagen export and two other extreme (slow and fast) cargos to validate the same trend in intra-Golgi transport velocities is seen in other cell lines. This will address three concerns: a. This is not a Hela-specific phenomenon; b. Transport of large cargoes like collagen agree with their proposal; c. To see if the same cargo has the same (similar) intra-Golgi speed and the trend between different cargoes is conserved across cell lines.

      Due to the difficulty of manipulating and imaging the procollagen-I RUSH reporter, we selected the collagenX-RUSH reporter (SBP-GFP-collagenX) instead. Our previous study (Tie et al., eLife, 2028) demonstrated that SBP-GFP-collagenX assembles as a large molecular weight particle, each having ~ 190 copies of SBP-GFP-collagenX. With an estimated mean size of ~ 40 nm, these aggregates are not as large as FM4 aggregates and procollagen-I (> 300 nm) and, therefore, are not excluded from conventional transport vesicles, which typically have a size of 50 – 100 nm. However, collagenX has distinct intra-Golgi transport behaviour from conventional secretory cargos -- while conventional secretory cargos localize to the cisternal interior, collagenX partitions to the cisternal rim (Tie et al., eLife, 2028).

      We studied the intra-Golgi transport of SBP-GFP-collagenX in HeLa cells via GLIM and side averaging. The new results are included in Figure 3 of the revamped manuscript. CollagenX has similar intra-Golgi transport kinetics as conventional secretory cargos, displaying the first-order exponential function in LQ vs. time and velocity vs. time plots.

      The side-averaging images are consistent with previous and current results. collagenX displays a double-punctum during the intra-Golgi transport, indicating a cisternal rim localization, as expected for large secretory cargos. Therefore, our new data demonstrated that cisternal rim partitioned large-size secretory cargos might follow intra-Golgi transport kinetics similar to those of cisternal interior partitioned conventional secretory cargos.

      We tried SBP-GFP-CD59 and SBP-GFP-Tac-TC, cargos with fast and slow intra-Golgi transport velocities, respectively, in 293T cells. Results are included in Figure 2, Supplementary Figure 2, and Table 1 of the revamped manuscript. We found that SBP-GFPTac-TC showed similar t<sub>intra</sub>s, 17 and 14 min, respectively, in HeLa and 293T cells. Considering our previous finding that glycosylation has an essential role in the Golgi exit (Sun et al., JBC, 2020), the distinct intra-Golgi transport kinetics of SBP-GFP-CD59 (t<sub>intra</sub>s, 13 and 5 min, respectively, in HeLa and 293T cells) might be due to its distinct luminal glycosylation between HeLa and 293T cells. Supporting this hypothesis, SBP-GFP-Tac-TC does not have any glycosylation sites due to the truncation of the Tac luminal domain.

      (2) RUSH assay has its own caveats which authors also refer to in the manuscript. Authors should test their model by using pulse chase approaches by SNAP tagged constructs which will allow them to do pulse chase assays without the requirement to release cargo as a wave (see: doi: 10.1242/jcs.231373). It is not necessary to test all the cargoes but the two on the ends of the spectrum (slow and fast). To avoid massive overexpression, authors could express the proteins using weaker promoters. Authors could also use this approach to simultaneously measure the two cargoes by tagging them with CLIP and SNAP tags and doing the pulse chase simultaneously (see: DOI: 10.1083/jcb.202206132). In this case it may be difficult to stain both GM130 and TGN, but authors could monitor the rate of segregation from the GM130 signal.

      During the RUSH assay, the sudden release of a large amount of secretory reporters does not occur under native secretory conditions and, consequently, might introduce artifacts. The reviewer suggests using pulse-chase labeling of SNAP (or CLIP)-tagged secretory cargos, which occurs in a steady state and hence more closely resembles native secretory transport. This is an excellent suggestion. However, we have not yet tested this method due to the following concerns.

      The standard protocol involves blocking existing reporters, pulse-labeling newly synthesized reporters, and chasing their movement along the secretory pathway. However, the typical 20minute pulse labeling period used in the two references would be too long, as a substantial portion of the reporters would already reach the trans-Golgi or exit the Golgi before the chase begins. Conversely, reducing the pulse labeling time would significantly weaken the GLIM signal.

      (3) While the intra-Golgi velocities are different for different cargoes tested, authors should show a control that the arrival of the cargoes from ER to the cis-Golgi follows similar kinetics or if there are differences there is no correlation with the intra-Golgi velocities. In other words, do cargoes which show slow intra-Golgi velocities also take more time to reach the cis-Golgi and vice versa.

      In nocodazole-induced Golgi ministacks, the ER exit site, ERGIC, and cis-Golgi are spatially closely associated. At the earliest measurable time point—5 minutes after biotin treatment— we observed that the secretory cargo had already reached the cis-Golgi (Figure 2 and Supplementary Figure 2). The rapid ER-to-cis-Golgi transport exceeds the temporal resolution of our current protocol, making it difficult to address the reviewer’s question (see our reply to Minor Points (2) of Reviewer #2 for more detailed discussion on this).

      (4) Were the different cargos traveling (at different speeds) through Golgi at the rims, or in the middle of ministack, or by vesicles?

      Please also refer to our reply to Question 1 of Reviewer #1. For the nocodazole-induced Golgi ministack, we previously investigated the lateral cisternal localization of RUSH secretory reporters using our en face average imaging (Tie et al., eLife, 2018). We found that small or conventional cargos (such as CD59 and E-cadherin) partition to the cisternal interior while large cargos (collagenX and FM4-CD8a) partition to the cisternal rim during their intra-Golgi transport. Using GLIM, we showed that the intra-Golgi transport kinetics of collagenX is similar to that of small cargos as both follow the first-order exponential function (Figure 3A-C). Therefore, cisternal rim partitioned large size secretory cargos might have intra-Golgi transport kinetics similar to those of cisternal interior partitioned conventional secretory cargos.

      (5) Figure 4, under both nocodazole and BFA treatment for 30mins, would the stacks have the same number (274 nm per LQ) as thickness? Or does it shrink a little? Considering extended BFA treatment reduced intact Golgi ministacks. This is important to understand the LQ numbers of those Golgi proteins. Besides, can they include one ERGIC marker in this assay, would it be approaching cis-Golgi? Images used for quantification in Figure 4 should be shown in the main figure.

      We define the axial size of the Golgi ministack as the axial distance from the GM130 to the GalT-mCherry, d<sub>(GM130-GalT-mCherry)</sub>, measured using the Gaussian centers of their line intensity profiles. As the reviewer suggested, we measured the axial size of the ministack during the nocodazole and BFA treatment. Indeed, we found a decrease in the ministack axial size from 300 ± 10 nm at 0 min to 190 ± 30 nm at 30 min of BFA treatment. This observation is further confirmed by our side average imaging. The new data is presented in Fig. 6G.

      Our study focuses on changes in the organization of the Golgi ministack. So, we didn’t include ERGIC53 in the current analysis. Instead, we quantified the axial distance between GalTmCherry and CD8a-furin, d<sub>(GalT-mCherry-CD8a-furin)</sub>, and found that it decreased from 200 ± 20 nm at 0 min to 100 ± 30 nm at 30 min of BFA treatment, suggesting the collapse of the TGN. The collapse of the TGN is further visualized by our side average imaging. The new data is presented in Fig. 6H.

      Therefore, our new data demonstrates that the Golgi ministack shrinks, and the TGN collapses under BFA treatment.

      Minor points:

      (1) The LQ data come from confocal/airy scan images, but no such images were shown in this paper. The authors can't assume every reader to have prior knowledge of their previous work. It will be beneficial to have one example image and how the LQ was measured.

      As advised by the reviewer, we have prepared Supplementary Figure 1 to provide a brief illustration of the principle behind GLIM and image processing steps involved.

      (2) The cargos used in this paper need to be introduced: what are they, how were they used in previous literature. Especially the furin constructs come out of the blue (also see point 7).

      As suggested by the reviewer, we have included a schematic diagram in Fig. 1 of the revised manuscript to illustrate all RUSH reporters and their corresponding ER hooks. In this diagram, we also highlight the key sequence differences in the cytosolic tails of different furin mutants.

      Additionally, we have added references for each RUSH reporter at the beginning of the Results and Discussion section.

      (3) There are two categories of exocytosis, constitutive and regulated. It important to state that the phenomenon observed is in cells predominantly showing only constitutive secretion.

      As the reviewer advised, we have added the following sentences in the section titled “Limitations of the study”.

      “Third, all RUSH reporters used in this study are constitutive secretory cargos. As a result, the intra-Golgi transport dynamics observed here might not reflect those of regulated secretion, which involves the synchronized release of a large quantity of cargo in response to a specific signal.”

      (4) All the cargoes show a progressive reduction in instantaneous velocities from cis to medial to trans. Authors should discuss how do they mechanistically explain this. Is the rate of vesicle production progressively decreasing from cis to trans and if so, why?

      As our imaging methods cannot differentiate vesicles from the cisternal rim, we could not tell if the vesicle production rate had changed during the intra-Golgi transport. We have provided an explanation of the progressive reduction of the intra-Golgi transport velocity in the Results and Discussion section. Please see the text below.

      “The progressive reduction in intra-Golgi transport of secretory cargo might result from the enzyme matrix's retention at the trans-Golgi. As the secretory cargos progress along the Golgi stack from the cis to the trans-side, more and more cargos become temporarily retained in the trans-Golgi region, gradually reducing their overall intra-Golgi transport velocity. If the release or Golgi exit of these cargos from the enzyme matrix follows a constant probability per unit time, i.e., a first-order kinetics process, the rate of cargo exiting from the Golgi should follow the first-order exponential function. Since the mechanism underlying intra-Golgi transport kinetics reflects fundamental molecular and cellular processes of the Golgi, further experimental data are essential to rigorously test this hypothesis.”

      (5) The supp file 1 nicely listed the raw data for plotting, and n for numbers of ministacks. Could the authors also show number of cells or experiment repeats?

      In the revamped version of the Supplementary File 1, we have added the cell number for each LQ measurement.

      (6) This recent work used novel multiplexing methods to show that nocodazole-treated cells had similar protein organization as in control may be cited. It also showed the effect of BFA. https://www.cell.com/cell/abstract/S0092-8674(24)00236-8.

      We have added this reference to the Introduction section to support that nocodazole-induced Golgi ministacks have a similar organization as the native Golgi. However, our BFA treatment was combined with the nocodazole treatment, while this paper’s BFA treatment does not contain nocodazole.

      (7) Figure 1G-J, authors should show a schematic to show the difference between different furin constructs. Also, LQ values in Fig 1I start from 1. Authors may need to include even earlier timepoints.

      As suggested by the reviewer, we have shown the domain organization of wild type and mutant furin RUSH reporters in Figure 1, highlighting key amino acids in the cytosolic tail. Please also see our reply to Minor Points (2) of Reviewer #1.

      In the revised manuscript, Fig. 1l (SBP-GFP-CD8a-furin-AC #1) has been updated to become Fig. 2J. In this dataset, the first time point was selected at a relatively late stage (20 min), resulting in an initial LQ value of 0.92. However, this should not pose an issue, as SBP-GFPCD8a-furin-AC reaches a plateau of ~ 1.6. The number of data points is sufficient to capture the rising phase and fit the first-order exponential function curve with an adjusted R<sup>2</sup> = 0.99. Furthermore, we have four independent datasets in total on the intra-Golgi transport of SBPGFP-CD8a-furin-AC (#1-4), demonstrating the consistency of our measurements.

      (8) Figure 2A need to show the data points, not just the lines.

      In the revamped manuscript, Fig. 2A has been updated to become Fig. 4A. The plot of Fig. 4A is calculated based on Equation 3.

      So, it does not have data points. However, t<sub>intra</sub> is calculated based on the experimental LQ vs. t kinetic data. 

      (9) Imaging and camera settings like exposure time, pixel size, etc should be reported in Methods.

      As suggested by the reviewer, we have supplied this information in the Materials and Methods section of the revised manuscript.

      (1) The exposure time and pixel size for the wide-field microscopy:

      “The image pixel size is 65 nm. The range of exposure time is 400 – 5000 ms for each channel.”

      (2) The exposure time and pixel size for the spinning disk confocal microscopy: “The image pixel size is 89 nm. The range of exposure time is 200 – 500 ms for each channel.”

      (3) The pixel dwelling time and pixel size for the Airyscan microscopy:

      “For side averaging, images were acquired under 63× objective (NA 1.40), zoomed in 3.5× to achieve 45 nm pixel size using the SR mode. The pixel dwelling time is 1.16 µs.”

      Reviewer #2 (Recommendations For The Authors):

      We sincerely appreciate the reviewer's insightful, detailed, and constructive feedback. Your thoughtful comments have helped us refine our analyses, clarify key points, and strengthen the overall quality of our manuscript. We are grateful for the time and effort you have dedicated to reviewing our work and providing valuable suggestions. Your input has been instrumental in improving both the scientific rigor and presentation of our findings. Thank you for your thorough and thoughtful review.

      Minor points:

      (1) Equation 2: A should be in front of the ln2. It's already resolved in equation 3, so likely only needs changing in the text

      As suggested by the reviewer, we have changed it accordingly.

      (2) Line 152: Why is there a lack of experimental data? High ER background and low golgi signal make it difficult to select ministacks: would be good to see examples of these images. Is 0 a relevant timepoint as cargo is still at the ER? Instead would a timepoint <5' be better demonstrate initial arrival in fast cargo, and 0' discarded?

      We observed that RUSH reporters typically do not exit the ER in < 5 min of biotin treatment, resulting in a high ER background and low Golgi signal. Example images of SBP-GFP-CD59 are shown below (scale bar: 10 µm). Possible reasons include: 1) the time required for biotin diffusion into the ER, 2) the time needed to displace the RUSH hook from the RUSH reporter, and 3) the time for recruitment of RUSH reporters to ER exit sites. As a result, we could not obtain LQs for time points earlier than 5 min during the biotin chase.

      Author response image 1.

      Despite the challenge in measuring LQs at early time points, 0 is still a relevant time point. At t = 0 min, RUSH reporters should be at the ER membrane near the ER exit site, a definitive pre-Golgi location along the Golgi axis, although we still don’t have a good method to determine its LQ.

      (3) Table 1 Line 474: 1-3 independent replicates: is there a better way of incorporating this into the table to make it more streamlined? It would be useful to see each cargo as a mean with error. Is there a more demonstrative way to present the table, for example (but does not have to be) fastest cargo first (Tintra) as in Table 2?

      As suggested by the reviewer, we revised Table 1. We calculated the mean and SD of t<sub>intra</sub> and arranged our RUSH reporters in ascending order based on their t<sub>intra</sub> values.

      (4) Line 264 / Fig 3B: It's unclear to me why the VHH-anti-GFP-mCherry internalisation approach was used, when the cells were expressing GFP, that could be used for imaging. Also, this introduces a question over trafficking of the VHH itself, to access the same compartments as the GFP-proteins are localised. It would be useful to describe the choice of this approach briefly in the text.

      Here, the surface-labeling approach is used to investigate if GFP-Tac-TC possesses a Golgi retrieval pathway after its exocytosis to the plasma membrane. When VHH-anti-GFP-mCherry is added to the tissue culture medium, it binds to the cell surface-exposed GFP-fused MGAT1, MGAT2, Tac, Tac-TC, CD8a, and CD8a-TC. Next, VHH-anti-GFP-mCherry traces the internalized GFP-fused transmembrane proteins. The surface-labeling approach has two advantages in this case. 1) It is much more sensitive in revealing the minor number of GFPtransmembrane proteins at the plasma membrane and endosomes, which are usually drowned in the strong Golgi and ER background fluorescence in the GFP channel. 2) While the GFP fluorescence distribution has reached a dynamic equilibrium, the surface labeling approach can reveal the endocytic trafficking route and dynamics.

      As the reviewer suggested, we added the following sentence to describe the choice of the cellsurface labeling – “By binding to the cell surface-exposed GFP, VHH-anti-GFP-mCherry serves as a sensitive probe to track the endocytic trafficking itinerary of the above GFP-fused transmembrane proteins”. 

      Regarding the trafficking of VHH-anti-GFP-mCherry itself, in HeLa cells that do not express GFP-fused transmembrane proteins, VHH-anti-GFP-mCherry can be internalized by fluidphase endocytosis. However, the fluid-phase endocytosis is negligible under our experimental condition, as we previously demonstrated (Sun et al., JCS, 2021; PMID: 34533190).

      (5) 446 Typo "internalization"

      It has been corrected.

      Reviewer #3 (Recommendations For The Authors):

      Below are my recommendations for the authors to improve their manuscript:

      We sincerely appreciate the reviewer's insightful, detailed, and constructive feedback. Your thoughtful comments have helped us refine our analyses, clarify key points, and strengthen the overall quality of our manuscript. We are grateful for the time and effort you have dedicated to reviewing our work and providing valuable suggestions. Your input has been instrumental in improving both the scientific rigor and presentation of our findings. Thank you for your thorough and thoughtful review.

      (1) Line 48: Tie at al. 2016 is cited. Please add references to original work showing that cargos transit from cis to trans Golgi cisternae.

      After reviewing the literature, we identified two references that provide some of the earliest morphological evidence of secretory cargo transit from the cis- to the trans-Golgi:

      (1) Castle et al, JCB, 1972; PMID: 5025103

      (2) Bergmann and Singer, JCB, 1983; PMID: 6315743

      The first study utilized pulse-chase autoradiographic EM imaging to track secretory protein movement, while the second employed immuno-EM imaging to observe the synchronized release of VSVGtsO45. Accordingly, we have removed Tie et al., 2016 and replaced it with these newly identified references.

      (2) I would suggest to cite earlier (in the Introduction) the rapid partitioning and rim progression models.

      As suggested, we have moved the rapid partitioning and rim progression models to the Introduction section.

      (3) Figure 1: LQ vs. time plot for SBP-GFP-CD8a-furinAC (panel I, 0.9 to 1.75 in 150 min) is different from Fig 7G of Tie et al. 2016 (LQ O-1.5 in 100 min). Please comment on why those 2 sets of data are different.

      We appreciate the reviewer for pointing out this error. In our previous publication (Tie et al., MBoC, 2016), we presented a total of four datasets on SBP-GFP-CD8a-furin-AC. However, in the earlier version of our manuscript, we mistakenly listed only three datasets, inadvertently omitting Fig. 7G from Tie et al., MBoC, 2016.

      In the revised version, we have now included Fig. S2T (SBP-GFP-CD8a-furin-AC #4), which corresponds to Fig. 7G from Tie et al., MBoC, 2016.

      (4) As mentioned in the public review, I think measurement of the expression level of the cargos is necessary to compare their transport kinetics.

      The reviewer raises a valid concern that is challenging to address. All our data were obtained by imaging overexpressed reporters, and we assume that their overexpression does not significantly impact the Golgi or the secretory pathway. Our previous studies have demonstrated that overexpression does not substantially affect LQs (Figure S2 of Tie et al., MBoC, 2016, and Figure S1 of Tie et al., JCB, 2022).

      We acknowledge this concern as one of the limitations in our study at the end of our manuscript:

      “First, our approach relied on the overexpression of fluorescence protein-tagged cargos. The synchronized release of a large amount of cargo could significantly saturate and skew the intra-Golgi transport.” 

      (5) To my opinion, cisternal continuities would also affect retrograde transport (accelerate) (by diffusion for instance) and not only retrograde transport. Please comment on how this would affect intra-Golgi transport kinetics.

      We believe the reviewer is suggesting “cisternal continuities would also affect retrograde transport (accelerate) (by diffusion for instance) and not only anterograde transport.”

      Transient cisternal continuities have been reported to facilitate the anterograde transport of large quantities of secretory cargos (Beznoussenko et al., 2014; PMID: 24867214) (Marsh et al., 2004; PMID: 15064406) (Trucco et al., 2004; PMID: 15502824). However, we are not aware of any reports demonstrating that such continuities facilitate the retrograde transport of secretory cargo, although Trucco et al. (2004) speculated that Golgi enzymes might use these connections to diffuse bidirectionally (anterograde and retrograde direction). For this reason, we did not discuss this scenario in our manuscript.

      (6) Lines 188-190: I don't understand why the rapid partitioning model is excluded. Please detail more the arguments used for this statement.

      Below is the section from the Introduction that addresses the reviewer's question.

      “This model (rapid partitioning model) suggests that cargos rapidly diffuse throughout the Golgi stack, segregating into multiple post-translational processing and export domains, where cargos are packed into carriers bound for the plasma membrane. Nonetheless, synchronized traffic waves have been observed through various techniques, including EM (Trucco et al., 2004) and advanced light microscopy methods we developed, such as GLIM and side-averaging(Tie et al., 2016; Tie et al., 2022). These findings suggest that the rapid partitioning model might not accurately represent the true nature of the intra-Golgi transport.”

      (7) I would suggest replacing the 'Golgi residence time' by another name as it reflects mainly the time of Golgi exit if I am not mistaken.

      We believe the term “Golgi residence time” more accurately reflects the underlying mechanism – retention. The same approach to measure the Golgi residence time can also be applied to Golgi enzymes such as ST6GAL1. Its slow Golgi exit kinetics (t<sub>1/2</sub> = 5.3 hours) (Sun et al., JCS, 2021) should be primarily due to a strong Golgi retention at its steady state Golgi localization.

      In contrast, the conventional secretory cargos’ Golgi exit times are usually much shorter (t<sub>1/2</sub> < 20 min) (Table 2) due to weaker Golgi retention. In a broader sense, the Golgi exit kinetics of a secretory cargo should be influenced by its Golgi retention. Furthermore, we have consistently used the term “Golgi residence time” in our previous publications. So, we propose maintaining this terminology in the current manuscript.

      (8) Lines 300-306: I would suggest that the authors remove this part as it is highly speculative and not supported by data.

      We have relocated this discussion to the section titled "Our data supports the rim progression model, a modified version of the stable compartment model."

      Our enzyme matrix hypothesis offers a potential explanation for key observations, including the differential cisternal localization of small and large cargos and the interior localization of Golgi enzymes. Cryo-FIB-ET has shown that the interior of Golgi cisternae is enriched with densely packed Golgi enzymes (Engel et al., PNAS, 2015; PMID: 26311849), supporting this hypothesis.

      Additionally, this hypothesis helps explain the gradual reduction in intra-Golgi transport velocities of secretory cargos, as requested by Reviewer #1 (Minor Points 4). For these reasons, we propose retaining this discussion in the manuscript.

      (9) In Figure 3B, percentage of MGAT2-GFP cells with anti-GFP signal at the Golgi is of 41% while Sun et al. 2021 reported 25%, please comment this difference. Reply:

      We included more cells for the quantification. The percentage of cells showing Golgi localization of VHH-anti-GFP-mCherry is now 32% (n = 266 cells). The observed difference, 32% vs. 25% (Sun et al., JCS, 2021), is likely due to uncontrollable variations in experimental conditions, which might have influenced the endocytic Golgi targeting efficiency.

      (10) The effects of brefeldinA are pleiotropic as it disassembles COPI and clathrin coats but also induces tubulation of endosomes. I would recommend using Golgicide A, which is more specific.

      We agree with the reviewer that Golgicide A might be more specific as an inhibitor of Arf1. We will certainly consider using this inhibitor next time.

    1. Author Response:

      The following is the authors' response to the original reviews.

      Reviewer #1 (Public Review):

      The authors investigated state-dependent changes in evoked brain activity, using electrical stimulation combined with multisite neural activity across wakefulness and anesthesia. The approach is novel, and the results are compelling. The study benefits from an in-depth sophisticated analysis of neural signals. The effects of behavioral state on brain responses to stimulation are generally convincing.

      It is possible that the authors' use of "an average reference montage that removed signals common to all EEG electrodes" could also remove useful components of the signal, which are common across EEG electrodes, especially during deep anesthesia. For example, it is possible (in fact from my experience I would be surprised if it is not the case) that under isoflurane anesthesia, electrical stimulation induces a generalized slow wave or a burst of activity across the brain. Subtracting the average signal will simply remove that from all channels. This does not only result in signals under anesthesia being affected more by the referencing procedure than during waking but also will have different effects on different channels, e.g. depending on how strong the response is in a specific channel.

      We thank the reviewer for the positive comments and for raising this point. We do not believe that the average reference montage is obscuring an evoked slow wave in the isoflurane-anesthetized mice. Electrical stimulation did elicit a brief activation in nearby neurons that was followed by roughly 200 ms of quiescence, but no significant changes in firing in the other regions we recorded from (Author response image 1).

      Author response image 1

      ERP and evoked population activity during isoflurane anesthesia do not show evidence of global responses. (Top). ERP (-0.2 to +0.8 s around stimulus onset) with all EEG electrode traces superimposed. Data represented is the same: red traces have been processed with the average reference montage, black traces have not. (Bottom) Population mean firing rates from the areas of interest from the same experiment as above.

      We are familiar with the work from Dasilva et al. (2021), a study similar to ours because they also performed cortical electrical stimulation in mice anesthetized with isoflurane. They show widespread evoked multi-unit activity (derived from LFP) in isoflurane-anesthetized mice in response to electrical stimulation, but critical experimental differences may underlie the conflicting results presented in our study. Both works use similar levels of isoflurane to maintain anesthesia (we use a level roughly equivalent to their “deep” level). However, our experiments use only isoflurane, whereas Dasilva et al. induced anesthesia with ketamine and medetomidine followed by isoflurane. It has been shown that isoflurane and ketamine have different effects on neural dynamics (Sorrenti et al., 2021). Typically, isoflurane causes reduced spontaneous firing rates and decreased evoked response amplitudes compared to wakefulness, whereas ketamine has been shown to increase firing rates and evoked response amplitudes (Aasebø et al., 2017; Michelson & Kozai, 2018). Perhaps a more relevant difference are the electrical stimulation parameters used to perturb the brain. Dasilva et al. used 1 ms pulses of 500 μA, which would have a much larger effect than the stimulation used in this work, 0.2 ms pulses of 10-100 μA.

      Additionally, we would like to clarify that the average reference montage is not impacting the main findings of this work. As the reviewer correctly pointed out, the average reference montage does change the appearance of the ERP in the butterfly plots (Top panel in Author response image 1). However, all the quantitative analyses of the EEG-ERPs are performed on the global field power, computed by taking the standard deviation across all EEG channels, which is not affected by the average reference montage.

      Reviewer #2 (Public Review):

      […] The conclusions regarding the thalamic contributions to the ERP components are strongly supported by the data.

      The spatiotemporal complexity is almost a side point compared to what seems to be the most important point of the paper: showing the contribution of thalamic activity to some components of the cortical ERP. Scalp ERPs have long been regarded as purely cortical phenomena, just like most EEGs, and this study shows convincing evidence to the contrary.

      The data presented seemingly contradicts the results presented by Histed et al. (2009), who assert that cortical microstimulation only affects passing fibers near the tip of the electrodes, and results in distant, sparse, and somewhat random neural activation. In this study, it is clear that the maximum effect happens near the electrodes, decays with distance, and is not sparse at all, suggesting that not only passing fibers are activated but that also neuronal elements might be activated by antidromic propagation from the axonal hillock. This appears to offer proof that microstimulation might be much more effective than it was thought after the publication of Histed 2009, as the uber-successful use of DBS to treat Parkinson's disease has also shown.

      We thank the reviewer for their positive comments and thoughtful suggestions. We appreciate and agree with the reviewer’s perspective that the thalamic contribution to the cortical ERP is one of the key points of this study. We also thank the reviewer for their comment on the apparently contradictory results reported by Histed et al. (2009). This gives us the opportunity to further highlight the important contribution of our study to the field.

      First, we would like to highlight some key experimental differences between the two studies. In our study we used single pulse stimulation with currents between 10 and 100 μA, whereas Histed et al. used trains of pulses (100 ms in duration at 250 Hz) with lower current intensities (between 2 and 50 μA). We varied the depth of stimulation, targeting superficial and deep cortical layers; Histed et al. exclusively stimulated superficial cortical layers. In addition, the two studies used recording methods that are orthogonal in nature. We used Neuropixels probes that record from neurons that span all cortical layers depth-wise while Histed et al. used two-photon calcium imaging to record from a horizontal plane of neurons (again, in the superficial cortical layers).

      Because of these important methodological differences, it is more appropriate to compare the Histed et al. results to our results from superficial stimulation at comparable current intensities. In this case, we believe the two studies show similar results: stimulation activated a small fraction of neurons even hundreds of microns away from the stimulating electrode (see Figure 4A from our manuscript). However, our study adds an important observation pointing to the critical role of the depth of the stimulating electrode. We observe significant excitation of local cortical neurons (Figure 4D) and trans-synaptic activation of the thalamus only when we delivered deep stimulation (Figure5A). This effect is likely mediated by activation of large, myelinated cortico-thalamic fibers, which are thought to be more excitable that non-myelinated horizontal fibers (Tehovnik & Slocum, 2013).

      To summarize, Histed et al. (2009) concluded that microstimulation causes a sparse activation of a distributed set of neurons with little evidence of synaptically driven activation. Instead, we showed that microstimulation can robustly activate local neurons and trans-synaptically activate distant neurons when stronger stimuli are directed to deep cortical layers. Based on this, we conclude that electrical stimulation is indeed highly effective, and is a valid tool that can be used to probe and characterize the cortico-thalamo-cortical network of any behavioral state.

      ----------

      Reviewer #1 (Recommendations for the authors):

      1. I am not clear how "putative pyramidal" or RS and "putative inhibitory" fast-spiking neurons were identified. Please provide some further details on that, including average spike wave shapes, and distribution of firing rates, and it would be interesting to know the proportion of "putative" RS and FS neurons in your recorded population. Obviously, caution is warranted here because, without further work, you cannot be sure that those are indeed pyramidal cells or interneurons! Is this subdivision necessary at all?

      We added details regarding the cell-type classification to the Results (lines 136-140) and the Methods section. This classification is common practice in cortical extracellular electrophysiology recordings given that cell-type specific analyses can reveal important differences between the two putative populations (Barthó et al., 2004; Bortone et al., 2014; Bruno & Simons, 2002; Jia et al., 2016; Niell & Stryker, 2008; Sirota et al., 2008). Based on our findings that the two populations respond to electrical stimulation in similar ways (excitation followed by a period of quiescence and rebound excitation), we agree the subdivision is not necessary to support our conclusions. However, we believe that some readers will appreciate seeing the two putative populations presented separately.

      2. I wonder how the authors know whether the animals were awake, specifically when they were not running. Did you observe animals falling asleep when head-fixed? Providing some analyses of spontaneous EEG/LFP signals in each state could add some reassurance that only wakefulness was included, as intended.

      While we cannot conclusively rule out that mice were asleep during the “quiet wakefulness” periods we analyzed, we believe they are likely to be awake for two main reasons: 1) all the experiments are performed during the dark phase of the light/dark cycle, when the mice are less likely to enter a sleep state (Franken et al., 1999); 2) the animals are not undergoing specific training to promote drowsiness or sleep. Indeed, many sleep-focused studies in head-fixed mice are performed during the light phase of the animal’s cycle to maximize the likelihood of capturing sleep states (Kobayashi et al., 2023; Turner et al., 2020; Yüzgeç et al., 2018; Zhang et al., 2022). We have added this note to the Discussion section (lines 402-406).

      Because we do not specifically record during sleep states and our recording does not include electromyography, which is commonly used in conjunction with EEG to classify sleep stages, we cannot accurately perform spectral comparison between “quiet wakefulness” and sleep states in our recordings.

      3. I was unsure about the meaning of some of the terminology, specifically "rebound", "rebound spiking", "rebound excitation" etc. Why do you call it "rebound"?

      “Rebound” is a term often used to describe a period of enhanced spiking following a period of prolonged silence or inhibition (Guido & Weyand, 1995; Roux et al., 2014). Grenier et al. list “postinhibitory rebound excitation” as an intrinsic property of cortical and thalamic neurons (1998). We added this description to the text (lines 79-80).

      Reviewer #2 (Recommendations For The Authors):

      Regarding analysis, I would make three main points:

      Regarding the CSD analysis, I think the authors have done a good job of circumventing several of the known issues of this technique, especially by using ERPs rather than ongoing activity. However, although I do not immediately have access to the literature to back up this claim, I've heard that many assumptions behind CSD require a laminar structure with electrodes positioned perpendicular to these layers. In Figure 1B it seems like the neuropixels probe is not really perpendicular to the cortical layers, and I wonder if this might be an issue. I am also wondering how to interpret the thalamic CSD, as this structure is not laminar, lacks the mass of neatly stacked neuronal dipoles present in the cortex, and does not have an orderly array of synaptic inputs and outputs. I understand that CSD analysis helps minimize the contributions of volume conduction, but in this case, I also wonder if the thalamic CSD is even necessary to back up the paper's claims.

      One-dimensional CSD is computed assuming that the electrode is inserted perpendicular to cortex. This is mainly important for the interpretation of sinks and sources, since CSD can be also computed on radial voltages (e.g., EEG [Tenke & Kayser, 2012]). In general, our Neuropixels probes do not significantly deviate from perpendicular (mean deviation from perpendicular 15.3 degrees, minimum 5.2 degrees, and maximum 36.6 degrees). The probe represented in Figure 1B deviates from perpendicular by 31.2 degrees, which is an outlier compared to the rest of the insertions. Any deviation from perpendicular would result in the “effective” cortical thickness being larger by a factor of 1/cos(angle deviation from perpendicular) and thus would not affect the relative location of sources and sinks. We have added a statement to clarify this in the text (lines 126 and 454-456).

      We agree with the statement regarding CSD analysis in the thalamus. We originally included the CSD for the thalamus in Figure 2F for completeness. As the reviewer pointed out, thalamic CSD was not used to perform any subsequent analysis and is, therefore, not necessary to back up any claims. As such, we have removed CSD plot from Figure 2F to avoid any confusion and made a comment to this effect in the legend (lines 1175-1177).

      On the merits of using the z-score normalization for spike rates vs. other strategies like standardizing to maximum firing, I am aware that both procedures have limitations, but the z-score changes the range of the firing rate from [0, +Inf] to [-Inf, +Inf]. This does not seem correct considering that negative spiking rates do not exist. The standardization to maximum rate keeps the range within [0, 1], not creating negative rates. Another point that it will be worth discussing is the reported values of the z-scored values. For example, what does it mean to be 54 standard deviations away from the mean? 6 standard deviations is already a big distance from the mean.

      For Figure 2, we chose to represent the neural firing rates as z-scores because we found it important to report the magnitude of both the increase and decrease of the evoked firing rates in the post-stimulus period relative to the pre-stimulus rate. The normalization we used helps to visualize the magnitude of the effects of electrical stimulation in neuronal activity for both directions, which is an important result of the study. Despite the differences between the two normalization methods, the normalization based on the maximum firing does not significantly change the qualitative interpretation of Figure 2 in the manuscript (Author response image 2).

      Author response image 2

      Evoked firing rates for neurons in the areas of interest in response to deep stimulation in MO during the awake state. (Left) Firing rates of all neurons normalized by the average, pre-stimulus firing rate. (Right) Firing rates of all neurons normalized by the maximum post-stimulus firing rate.

      Regarding Figure 3 and the associated text, we would like to clarify that the magnitude metric is not simply a z-score value (with units of s.d.) but rather it is the integrated area under the z-scored response over the response window (with units of s.d.∙seconds). This can help explain why we see values of ~50 s.d.∙s. We chose to z-score firing rates, LFP, and CSD to normalize across the different signals and magnitudes of the evoked responses. We often observed the largest responses in the LFP (see Figure 3A), which may be partly due to the signal naturally having a larger dynamic range than the measured neural firing rates. Then we integrated the z-score response time series to capture the dynamic of the signal over the response window, rather than a static value such as the mean or maximum z-score. After performing a thorough literature search, we found no other ways to capture and compare the magnitudes of the different signals. We have added language to clarify the magnitude metric (lines 155-156) and added the appropriate units.

      In reporting the p-values, I recommend increasing the number of significant digits to four because the p-value seems to be the same for different tests in several places (e.g.: lines 207 to 218), which seems odd. I also wonder whether this could be an artifact of the z-scoring procedure. In the figures, I would like to advise the use of 1 asterisk to denote "weak evidence to reject the null hypothesis (0.05 > p > 0.01)" and two asterisks to denote "strong evidence to reject the null hypothesis (0.01 > p)", and make a note of it accordingly in the manuscript and/or figure legends.

      According to the reviewer’s suggestion, we have changed the statistics language to “* weak evidence to reject null hypothesis (0.05 > p > 0.01), ** strong evidence to reject null hypothesis (0.01 > p > 0.001), *** very strong evidence to reject null hypothesis (0.001 > p)” throughout the manuscript.

      We have also increased the number of significant digits to four throughout the manuscript. It is true that some of the p-values reported for Figure 3 (lines 169-180) are the same for different tests. This is not an artifact of the z-scoring, but rather a consequence of performing the Wilcoxon signed-rank test (an ordinal statistical test) with small sample numbers. Because the p-value depends only on the relative ordering, not the continuous distribution of values, the small sample size (N=6-14) increases the likelihood of obtaining the exact same p-value if the relative ordering of samples is the same.

      Line 202: If the magnitude corresponds to z-score data, please add "s.d." after the number, as z-scored values are expressed in standard deviation units. Please update this throughout the paper.

      As stated above the magnitude metric is the integrated area under the z-scored response over the response window (with units of s.d.∙seconds). We have added the correct units in all places.

      Line 214: Please report how the multiple comparisons correction was performed

      We have added the test used for multiple comparisons in line 169 (formerly line 214) and in the Methods section (line 770).

      Line 462: please replace "Neuropixels activity" with "LFP and single-unit activity".

      We changed the wording to specify “LFP, and single neuron responses…” (now line 337).

      Line 475: a short explanation of the bi-stability phenomena will be helpful for the reader.

      We added the following description: “a state characterized by spontaneous alternation between bouts of activity and periods of silence” (lines 350-351).

      Line 601: It is asserted that "Electrical stimulation directly activates local cells and axons that run near the stimulation site via activation of the axon initial segment" and the paper by Histed et al. 2009 is cited. This does not seem like an appropriate citation, as Histed et al. explicitly state that electrical microstimulation does not activate local neuronal bodies near the electrode tip. See my comment above.

      Upon further reading, we believe we are seeing evidence of direct axonal activation and subsequent antidromic activation of local cell bodies, as you suggested in your above comment and has been proposed by many including Histed et al. (2009) and Nowak and Bullier (1998). We edited our sentence accordingly, kept the Histed et al. citation, and added other relevant citations (lines 487-490).

      References

      • Aasebø, I. E. J., Lepperød, M. E., Stavrinou, M., Nøkkevangen, S., Einevoll, G., Hafting, T., & Fyhn, M. (2017). Temporal Processing in the Visual Cortex of the Awake and Anesthetized Rat. ENeuro, 4(4), 59–76. https://doi.org/10.1523/ENEURO.0059-17.2017

      • Barthó, P., Hirase, H., Monconduit, L., Zugaro, M., Harris, K. D., & Buzsáki, G. (2004). Characterization of Neocortical Principal Cells and Interneurons by Network Interactions and Extracellular Features. Journal of Neurophysiology, 92(1), 600–608. https://doi.org/10.1152/jn.01170.2003

      • Bortone, D. S., Olsen, S. R., & Scanziani, M. (2014). Translaminar Inhibitory Cells Recruited by Layer 6 Corticothalamic Neurons Suppress Visual Cortex. Neuron, 82, 474–485. https://doi.org/10.1016/j.neuron.2014.02.021

      • Bruno, R. M., & Simons, D. J. (2002). Feedforward Mechanisms of Excitatory and Inhibitory Cortical Receptive Fields. The Journal of Neuroscience, 22(24), 10966–10975. https://doi.org/10.1523/JNEUROSCI.22-24-10966.2002

      • Dasilva, M., Camassa, A., Navarro-Guzman, A., Pazienti, A., Perez-Mendez, L., Zamora-López, G., Mattia, M., & Sanchez-Vives, M. V. (2021). Modulation of cortical slow oscillations and complexity across anesthesia levels. NeuroImage, 224, 117415. https://doi.org/10.1016/j.neuroimage.2020.117415

      • Franken, P., Malafosse, A., & Tafti, M. (1999). Genetics of sleep regulation in mice-Franken et al Genetic Determinants of Sleep Regulation in Inbred Mice. SLEEP, 22(2). https://academic.oup.com/sleep/article/22/2/155/2731698

      • Grenier, F., Timofeev, I., & Steriade, M. (1998). Leading role of thalamic over cortical neurons during postinhibitory rebound excitation. Proceedings of the National Academy of Sciences of the United States of America, 95(23), 13929–13934. https://doi.org/10.1073/pnas.95.23.13929

      • Guido, W., & Weyand, T. (1995). Burst responses in thalamic relay cells of the awake behaving cat. Journal of Neurophysiology, 74(4), 1782–1786. https://doi.org/10.1152/JN.1995.74.4.1782

      • Histed, M. H., Bonin, V., & Reid, R. C. (2009). Direct Activation of Sparse, Distributed Populations of Cortical Neurons by Electrical Microstimulation. Neuron, 63(4), 508–522. https://doi.org/10.1016/j.neuron.2009.07.016

      • Jia, X., Siegle, J., Bennett, C., Gale, S., Denman, D. R., Koch, C., & Olsen, S. (2016). High-density extracellular probes reveal dendritic backpropagation and facilitate neuron classification 1 2. Journal of Neurophysiology, 121(5), 1831–1847. https://doi.org/10.1101/376863

      • Kobayashi, G., Tanaka, K. F., & Takata, N. (2023). Pupil Dynamics-derived Sleep Stage Classification of a Head-fixed Mouse Using a Recurrent Neural Network. The Keio Journal of Medicine, 2022-0020-OA. https://doi.org/10.2302/KJM.2022-0020-OA

      • Michelson, N. J., & Kozai, T. D. Y. (2018). Isoflurane and ketamine differentially influence spontaneous and evoked laminar electrophysiology in mouse V1. Journal of Neurophysiology, 120(5), 2232. https://doi.org/10.1152/JN.00299.2018

      • Niell, C. M., & Stryker, M. P. (2008). Highly selective receptive fields in mouse visual cortex. Journal of Neuroscience, 28(30), 7520–7536. https://doi.org/10.1523/JNEUROSCI.0623-08.2008

      • Nowak, L. G., & Bullier, J. (1998). Axons, but not cell bodies, are activated by electrical stimulation in cortical gray matter. II. Evidence from selective inactivation of cell bodies and axon initial segments. Experimental Brain Research, 118(4), 489–500. https://doi.org/10.1007/S002210050305/METRICS

      • Roux, L., Stark, E., Sjulson, L., & Buzsáki, G. (2014). In vivo optogenetic identification and manipulation of GABAergic interneuron subtypes. Current Opinion in Neurobiology, 26, 88–95. https://doi.org/10.1016/j.conb.2013.12.013

      • Sirota, A., Montgomery, S., Fujisawa, S., Isomura, Y., Zugaro, M., & Buzsáki, G. (2008). Entrainment of Neocortical Neurons and Gamma Oscillations by the Hippocampal Theta Rhythm. Neuron, 60(4), 683–697. https://doi.org/10.1016/j.neuron.2008.09.014

      • Sorrenti, V., Cecchetto, C., Maschietto, M., Fortinguerra, S., Buriani, A., & Vassanelli, S. (2021). Understanding the Effects of Anesthesia on Cortical Electrophysiological Recordings: A Scoping Review. International Journal of Molecular Sciences, 22(3), 1286. https://doi.org/10.3390/IJMS22031286

      • Tehovnik, E. J., & Slocum, W. M. (2013). Two-photon imaging and the activation of cortical neurons. Neuroscience, 245(March), 12–25. https://doi.org/10.1016/j.neuroscience.2013.04.022

      • Tenke, C. E., & Kayser, J. (2012). Generator localization by current source density (CSD): Implications of volume conduction and field closure at intracranial and scalp resolutions. Clinical Neurophysiology, 123(12), 2328–2345. https://doi.org/10.1016/J.CLINPH.2012.06.005

      • Turner, K. L., Gheres, K. W., Proctor, E. A., & Drew, P. J. (2020). Neurovascular coupling and bilateral connectivity during nrem and rem sleep. ELife, 9, 1. https://doi.org/10.7554/ELIFE.62071

      • Yüzgeç, Ö., Prsa, M., Zimmermann, R., & Huber, D. (2018). Pupil Size Coupling to Cortical States Protects the Stability of Deep Sleep via Parasympathetic Modulation. Current Biology, 28(3), 392. https://doi.org/10.1016/J.CUB.2017.12.049

      • Zhang, X., Landsness, E. C., Chen, W., Miao, H., Tang, M., Brier, L. M., Culver, J. P., Lee, J. M., & Anastasio, M. A. (2022). Automated sleep state classification of wide-field calcium imaging data via multiplex visibility graphs and deep learning. Journal of Neuroscience Methods, 366, 109421. https://doi.org/10.1016/J.JNEUMETH.2021.109421

    1. Author response:

      The following is the authors’ response to the current reviews.

      Reviewer #1 (Public Review):

      Overall the authors provide a very limited data set and in fact only a proof of concept that their sensor can be applied in vivo. This is not really a research paper, but a technical note. With respect to their observation of clustered activity, they now provide an overview image, next to zoomed details. However, from these images one cannot conclude 'by eye' any clustering event. This aligns with the very low r values. All neurons in the field show variable activity and a clustering is not really evident from these examples. Even within a cluster, there is variability. The authors now confirm that expression levels are indeed variable but are independent from the ratio measurements. Further, they controlled for specificity by including DAPT treatments, but opposite to their own in vitro data (in primary neurons) the ratios increased. The authors argue that both distance and orientation can either decrease or increase ratios and that the use of this biosensor should be explored model-by-model. This doesn't really confer high confidence and may hinder other groups in using this sensor reliably.

      Secondly, there is still no physiological relevance for this observation. The experiments are performed in wild-type mice, but it would be more relevant to compare this with a fadPSEN1 KI or a PSEN1cKO model to investigate the contribution of a gain of toxic function or LOF to the claimed cell non-autonomous activations. The authors acknowledge this shortcoming but argue that this is for a follow-up study.

      For instance, they only monitor activity in cell bodies, and miss all info on g-sec activity in neurites and synapses: what is the relevance of the cell body associated g-sec and can it be used as a proxy for neuronal g-sec activity? If cells 'communicate' g-sec activities, I would expect to see hot spots of activity at synapses between neurons.

      Without some more validation and physiologically relevant studies, it remains a single observation and rather a technical note paper, instead of a true research paper.

      The effect size was small, as stated in the original and revised manuscripts and the point-by-point responses to the 1st round review. Such subtle effects will likely be challenging to detect by eye. However, our unbiased quantification allowed us to detect a statistically significant linear correlation between the 720/670 ratio in each neuron and the average ratio in neighboring neurons, which we have verified using many different approaches (Figure 3, Figure 3—figure supplement 2, and Figure 4), and the correlation was canceled by the administration of g-secretase inhibitor (Figure 5). Such objective analysis made us more confident to conclude that g-secretase affects g-secretase in neighboring neurons.

      We would also like to make clear the design of the C99 720-670 biosensor. Both C99, the sensing domain that is cleaved by g-secretase, and the anchoring domain fused to miRFP670 are integrated into the membrane (Figure 1A). Therefore, how these two domains with four transmembrane regions are embedded in the membrane should affect the orientation between the donor, miRFP670, and the acceptor, miRFP720. As noted in our point-by-point responses to the initial review, we have previously validated that pharmacological inhibition of g-secretase significantly increases the FRET ratio in various cell lines, including CHO, MEF, BV2 cells, and mouse cortical primary neurons (Maesako et al., 2020; Houser et al., 2020, and unpublished observations). On the other hand, FRET reduction by g-secretase inhibition was found in mouse primary neurons derived from the cerebellum (unpublished observations) as well as the somatosensory cortex neurons in vivo (this study). While we could not use the exact same imaging set-up between cortical primary neurons in vitro and those in vivo due to different expression levels of the biosensor, we could do it for in vitro cortical primary neurons vs. in vitro cerebellum neurons. We found by the direct comparison that 720/670 ratios are significantly higher in the cerebellum than the cortex neurons even in the presence of 1 mM DAPT (Author response image 1), a concentration that nearly completely inhibits g-secretase activity. This suggests a different integration and stabilization pattern of the sensing and anchoring domains in the C99 720-670 biosensor between the cortex and cerebellum primary neurons, and thus, orientation between the donor and acceptor varies in the two neuronal types. We expect a similar scenario between cortical primary neurons in vitro and those in vivo. Of note, we have recently demonstrated that the cortex and cerebellum primary neurons exhibit distinct membrane properties (Lundin and Wieckiewicz et al., 2024 in revision), suggesting the different baseline FRET could be related to the different membrane properties between the cortex and cerebellum primary neurons. On the other hand, this raises a concern that 720/670 ratios can be affected not only by g-secretase activity but also by other cofounders, such as altered membrane properties. However, a small but significant correlation between the 720/670 ratio in a neuron and those ratios in its neighboring neurons is canceled by g-secretase inhibitor (Figure 5), suggesting that the correlation between the 720/670 ratio in a neuron and those in its neighboring neurons is most likely dependent on g-secretase activity. Taken together, we currently think orientation plays a significant role in our biosensor and would like to emphasize the importance of ensuring on a model-by-model basis whether the cleavage of the C99 720-670 biosensor by g-secretase increases or decreases 720/670 FRET ratios.

      Author response image 1.

      Furthermore, we co-expressed the C99 720-670 biosensor and visible range fluorescence reporters to record other biological events, such as changes in ion concentration, in cortex primary neurons. Interestingly, several biological events uniquely detected in the neurons with higher 720/670 ratios, which are expected to exhibit lower endogenous g-secretase activity, are recapitulated by pharmacological inhibition of g-secretase (unpublished observations), ensuring that higher 720/670 ratios are indicative of lower g-secretase activity in mouse cortex primary neurons. Such multiplexed imaging will help to further elucidate how the C99 720-670 biosensor behaves in response to the modulation of g-secretase activity.

      Lastly, the scope of this study was to develop and validate a novel imaging assay employing a NIR FRET biosensor to measure g-secretase activity on a cell-by-cell basis in live wild-type mouse brains. However, we do appreciate the reviewer’s suggestion and think employing this new platform in FAD PSEN1 knock-in (KI) or PSEN1 conditional knockout (cKO) mice would provide valuable information. Furthermore, we are keen to expand our capability to monitor g-secretase with subcellular resolution in live mouse brains in vivo, which we will explore in follow-up studies. Thank you for your thoughtful suggestions.

      Reference

      - Maesako M, Sekula NM, Aristarkhova A, Feschenko P, Anderson LC, Berezovska O. Visualization of PS/γ-Secretase Activity in Living Cells. iScience. 2020 Jun 26;23(6):101139.

      - Houser MC, Hou SS, Perrin F, Turchyna Y, Bacskai BJ, Berezovska O, Maesako M. A Novel NIR-FRET Biosensor for Reporting PS/γ-Secretase Activity in Live Cells. Sensors (Basel). 2020 Oct 22;20(21):5980.

      - Lundin B, Wieckiewicz N, Dickson JR, Sobolewski RGR, Sadek M, Armagan G, Perrin F, Hyman BT, Berezovska O, and Maesako M. APP is a regulator of endo-lysosomal membrane permeability. 2024 in revision

      Reviewer #2 (Public Review):

      Regarding the variability and spatial correlation- the dynamic range of the sensor previously reported in vitro is in the range of 20-30% change (Houser et al 2020) whereas the range of FR detected in vivo is between cells is significantly larger in this MS. This raises considerable doubts for specific detection of cellular activity.

      One direct way to test the dynamic range of the sensor in vivo, is to increase or decrease endogenous gamma-secretase activity and to ensure this experimental design allows to accurately monitor gamma-secretase activity. In the previous characterization of the reporter (Hauser et al 2020), DAPT application and inhibition of gamma-secretase activity results in increased FR (Figures 2 and 3 of Houser et al). This is in agreement with the design of the biosensor, since FR should be inversely correlated with enzymatic activity. Here, the authors repeated the experiment, and surprisingly found an opposite effect, in which DAPT significantly reduced FR.

      The authors maintain that this result could be due to differences in cell-types, However, this experiment was previously performed in cultures cortical neurons and many different cell types, as noted by the authors in their rebuttal.

      Instead, I would argue that these results further highlight the concerns of using FR in vivo, since based on their own data, there is no way to interpret this quantification. If DAPT reduces FR, does this mean we should now interpret the results of higher FR corresponds to higher g-sec activity? Given a number of papers from the authors claiming otherwise, I do not understand how one can interpret the results as indicating a cell-specific effect.

      In conclusion, without any ground truth, it is impossible to assess and interpret what FR measurements of this sensor in vivo mean. Therefore, the use of this approach as a way to study g-sec activity in vivo seems premature.

      Please find our response to reviewer 1’s similar critique above. Here, we again would like to re-clarify the design of our C99 720-670 biosensor. The orientation between the donor, miRFP670, and acceptor, miRFP720, is dependent on how C99, the sensing domain that is cleaved by g-secretase, and the anchoring domain are integrated into the membrane (Figure 1A). Although it was surprising to us, it is possible that g-secretase inhibition decreases 720/670 ratios if 1) the donor-acceptor orientation plays a significant role in FRET and 2) the baseline structure of the C99 720-670 biosensor is different between cell types. This appears to be the case between the cortex and cerebellum primary neurons (i.e., DAPT increases 720/670 ratios in the cortex neurons while decreasing in the cerebellum neurons), and we expect it in cortical neurons in vitro vs. in vivo as well. Hence, we recommend that users first validate whether the cleavage of the C99 720-670 biosensor by g-secretase increases or decreases 720/670 FRET ratios in their models. If DAPT increases 720/670 ratios (like in cortex primary neurons, CHO, MEF, and BV2 cells that we have validated), the results of higher ratios should be interpreted as lower g-secretase activity. If DAPT reduces 720/670 ratios (like in cerebellum primary neurons and the somatosensory cortex neurons in vivo), we should interpret the results of higher ratios corresponding to higher g-secretase activity. From a biosensing perspective, although we need to know which is the case on a model-by-model basis, we think whether g-secretase activity increases or decreases the 720/670 ratio is not critical; rather, if it can significantly change FRET efficiency is more important. Thank you for your critical comments.

      Reviewer #3 (Public Review):

      This paper builds on the authors' original development of a near infrared (NIR) FRET sensor by reporting in vivo real-time measurements for gamma-secretase activity in the mouse cortex. The in vivo application of the sensor using state-of-the-art techniques is supported by a clear description and straightforward data, and the project represents significant progress because so few biosensors work in vivo. Notably, the NIR biosensor is detectable to ~ 100 µm depth in the cortex. A minor limitation is that this sensor has a relatively modest ΔF as reported in Houser et al, which is an additional challenge for its use in vivo. Thus, the data is fully dependent on post-capture processing and computational analyses. This can unintentionally introduce biases but is not an insurmountable issue with the proper controls that the authors have performed here.

      The following opportunity for improving the system didn't initially present itself until the authors performed an important test of the FRET sensor in vivo following DAPT treatment. The authors get credit for diligently reporting the unexpected decrease in 720/670 FRET ratio. In turn this has led to a suggestion that this sensor would benefit from a control that is insensitive to gamma-secretase activity. FRET influences that are independent of gamma-secretase activity could be distinguished by this control.

      From previous results in cultured neurons, the authors expected an increase in FRET following DAPT treatment in vivo. These expectations fit with the sensor's mode-of-action because a block of gamma-secretase activity should retain the fluorophores in proximity. When the authors observed decreased FRET, the conclusion was that the sensor performs differently in different cellular contexts. However, a major concern is that mechanistically it is unclear how this could occur with this type of sensor. The relative orientation of fluorophores indeed can contribute to FRET efficiency in tension-based sensors. However, the proteolysis expected with gamma-secretase activity would release tension and orientation constraints. Thus, the major contributing FRET factor is expected to be distance, not orientation. Alternative possibilities that could inadvertently affect readouts include an additional DAPT target in vivo sequestering the inhibitor, secondary pH effects on FRET, photo-bleaching, or an unidentified fluorophore quencher in vivo stimulated by DAPT. Ultimately this new FRET sensor would benefit from a control that is insensitive to gamma-secretase activity. FRET influences that are independent of gamma-secretase activity could be distinguished by this control.

      Given that the anchoring domain is composed of three transmembrane regions and the linker connecting the donor, miRFP670, and the acceptor, miRFP720, is highly flexibility, we are still not sure if the orientation constraint of the C99 720-670 biosensor is canceled by g-secretase cleavage. This means that the orientation between the donor and acceptor in the cleaved form of the sensor can be different between model and model. As explained in response to the similar critique of reviewer 1, we found that the 720/670 ratio is significantly higher in the cerebellum than in the cortex neurons even in the presence of DAPT (Figure 1 for the review only). Therefore, we currently think the donor-acceptor orientation, both in the cleaved and non-cleaved forms of the sensor, plays a role in determining whether g-secretase activity increases or decreases the 720/670 ratio (but this view may change depends on the future discoveries).

      As the reviewer pointed out, the NIR g-secretase biosensor with no biological activity is important; however, a point mutation in the transmembrane region of the C99 sensing domain could also result in altered orientation between the donor, miRFP670, and the acceptor, miRFP720, since C99 is connected to the acceptor, which may bring additional complexity. Also, as noted in our point-by-point responses to the initial review, the mutation(s) that can fully block C99 processing by g-secretase has not been established. Therefore, we asked if a subtle but significant correlation we found between the 720/670 ratio in a neuron and those ratios in its neighboring neurons is canceled by g-secretase inhibitor administration. Since the correlation was abolished (Figure 5), it suggests that the correlation between the 720/670 ratio in a neuron and those ratios in the neighboring neurons depends on g-secretase activity.

      It is not fully established how g-secretase activity is spatiotemporally regulated; therefore, the development of more appropriate control biosensors and further validation of our findings with complementary approaches would be crucial in our follow-up studies. Thank you for your valuable comments.


      The following is the authors’ response to the original reviews.

      Reviewer #1 (Public Review):

      (1) Overall the authors provide a very limited data set and in fact only a proof of concept that their sensor can be applied in vivo. This is not really a research paper, but a technical note. With respect to their observation of clustered activity, the images do not convince me as they show only limited areas of interest: from these examples (for instance fig 5) one sees that merely all neurons in the field show variable activity and a clustering is not really evident from these examples. Even within a cluster, there is variability. With r values between 0.23 to .36, the correlation is not that striking. The authors herein do not control for expression levels of the sensor: for instance, can they show that in all neurons in the field, the sensor is equally expressed, but FRET activity is correlated in sets of neurons? Or are the FRET activities that are measured only in positively transduced neurons, while neighboring neurons are not expressing the sensor? Without such validation, it is difficult to make this conclusion.

      We appreciate the reviewer’s comment. We agree with the reviewer that this study is not testing a new hypothesis but rather developing and validating a novel tool. However, we do believe such a “technical note” is as important as a “research paper” since advancing technique(s) is the only way to break the barrier in our understanding of complex biological events. Therefore, this study aimed to develop and validate a novel imaging assay employing a recently engineered NIR FRET biosensor to measure γ-secretase activity (Houser et al., 2020) on a cell-by-cell basis in live mouse brains, enabling us for the first time to examine how γ-secretase activity is regulated in individual neurons in vivo, and uncover that γ-secretase activity may influence γ-secretase in neighboring neurons. Like the reviewer, we found that the cell-to-cell correlation is not that striking, as we clearly stated in the original manuscript: “Although the effect size is modest, we also found a statistically significant correlation between…” 

      We were also aware that there is variability in a cluster of neurons exhibiting similar γ-secretase activities. Per the reviewer’s request, the images have been expanded to the entire imaging field of view (new Figure 3A). Although the effect size is small, our unbiased quantification showed a statistically significant linear correlation between the 720/670 ratio in each neuron and the average ratio in five neighboring neurons (Figure 3, Figure 3—figure supplement 2, and Figure 4), and the correlation was canceled by the administration of γ-secretase inhibitor (Figure 5). These findings made it impossible to conclude that γ-secretase does not affect γ-secretase in neighboring neurons.

      Regarding the expression levels and pattern of the sensor, an AAV-based gene delivery approach employed in this study results in the expression of the sensor not in all but in selected neurons. We have newly performed immunohistochemistry, showing that approximately 40% of NeuN-positive neurons express the C99 720-670 biosensor (new Figure 1—figure supplement 2A and 2B).

      Reference

      - Houser MC, Hou SS, Perrin F, Turchyna Y, Bacskai BJ, Berezovska O, Maesako M. A Novel NIRFRET Biosensor for Reporting PS/γ-Secretase Activity in Live Cells. Sensors (Basel). 2020 Oct 22;20(21):5980. 

      (2) Secondly, I am lacking some more physiological relevance for this observation. The experiments are performed in wild-type mice, but it would be more relevant to compare this with a fadPSEN1 KI or a PSEN1cKO model to investigate the contribution of a gain of toxic function or LOF to the claimed cell non-autonomous activations. Or what would be the outcome if the sensor was targeted to glial cells?

      The AAV vector in this study encodes the human synapsin promoter and our new immunohistochemistry demonstrates that nearly 100% of the cells expressing the C99 720-670 sensor are NeuN positive, and we hardly detected the sensor expression in Iba-1 or GFAP-positive cells (new Figure 1— figure supplement 2A and 2C). 

      The mechanism underlying the cell non-autonomous regulation of γ-secretase remains unclear. As discussed in our manuscript, one of the potential hypotheses could be that secreted abeta42 plays a role (Zoltowska et al., 2023 eLife). Whereas this report focuses on the development and validation of a novel assay using wildtype mice, future follow-up studies employing FAD PSEN1 knock-in (KI) and PSEN1 conditional knockout (cKO) mice would allow us test the hypothesis above since abeta42 is known to increase in some FAD PSEN1 KI mice (Siman et al., 2000 J Neurosci, Vidal et al., 2012 FASEB J) while decreases in PSEN1 cKO mice (Yu et al., 2001 Neuron).  

      Reference

      - Siman R, Reaume AG, Savage MJ, Trusko S, Lin YG, Scott RW, Flood DG. Presenilin-1 P264L knockin mutation: differential effects on abeta production, amyloid deposition, and neuronal vulnerability. J Neurosci. 2000 Dec 1;20(23):8717-26. 

      - Vidal R, Sammeta N, Garringer HJ, Sambamurti K, Miravalle L, Lamb BT, Ghetti B. The Psen1-L166Pknock-in mutation leads to amyloid deposition in human wild-type amyloid precursor protein YAC transgenic mice. FASEB J. 2012 Jul;26(7):2899-910. 

      - Yu H, Saura CA, Choi SY, Sun LD, Yang X, Handler M, Kawarabayashi T, Younkin L, Fedeles B, Wilson MA, Younkin S, Kandel ER, Kirkwood A, Shen J. APP processing and synaptic plasticity in presenilin-1 conditional knockout mice. Neuron. 2001 Sep 13;31(5):713-26. 

      - Zoltowska KM, Das U, Lismont S, Enzlein T, Maesako M, Houser MC, Franco ML, Moreira DG, Karachentsev D, Becker A, Hopf C, Vilar M, Berezovska O, Mobley W, Chávez-Gutiérrez L. Alzheimer's disease linked Aβ42 exerts product feedback inhibition on γ-secretase impairing downstream cell signaling. eLife. 2023. 12:RP90690

      (3) For this reviewer it is not clear what resolution they are measuring activity, at cellular or subcellular level? In other words are the intensity spots neuronal cell bodies? Given g-sec activity are in all endosomal compartments and at the cell surface, including in the synapse, does NIR imaging have the resolution to distinguish subcellular or surface localized activities? If cells 'communicate' g-sec activities, I would expect to see hot spots of activity at synapses between neurons: is this possible to assess with the current setup? 

      Since this study aimed to determine how γ-secretase activity is regulated on a cell-by-cell basis in live mouse brains, the FRET signal was detected in neuronal cell bodies. While our current set-up for in vivo can only record γ-secretase activity with a cellular resolution, we previously detected predominant γ-secretase activity in the endo-lysosomal compartments (Maesako et al., 2022 J Neurosci) as well as in certain spots of neuronal processes (Maesako et al., 2020 iScience) in cultured primary neurons using the same microscope set-up. Therefore, future studies will expand our capability to monitor γ-secretase with subcellular resolution in live mouse brains in vivo.

      Reference

      - Maesako M, Sekula NM, Aristarkhova A, Feschenko P, Anderson LC, Berezovska O. Visualization of PS/γ-Secretase Activity in Living Cells. iScience. 2020 Jun 26;23(6):101139. 

      - Maesako M, Houser MCQ, Turchyna Y, Wolfe MS, Berezovska O. Presenilin/γ-Secretase Activity Is Located in Acidic Compartments of Live Neurons. J Neurosci. 2022 Jan 5;42(1):145-154. 

      (4) Without some more validation and physiological relevant studies, it remains a single observation and rather a technical note paper, instead of a true research paper.

      Please find our response above to the critique (1).  

      Reviewer #2 (Public Review):

      (1) Regarding the variability and spatial correlation- the dynamic range of the sensor previously reported in vitro is in the range of 20-30% change (Houser et al 2020) whereas the range of FR detected in vivo is between cells is significantly larger (Fig. 3). This raises considerable doubts for specific detection of cellular activity (see point 3).

      Please find our response below to the critique (2).

      (2) One direct way to test the dynamic range of the sensor in vivo, is to increase or decrease endogenous gamma-secretase activity and to ensure this experimental design allows to accurately monitor gamma-secretase activity. In the previous characterization of the reporter (Hauser et al 2020), DAPT application and inhibition of gammasecretase activity results in increased FR (Figures 2 and 3 of Houser et al). This is in agreement with the design of the biosensor, since FR should be inversely correlated with enzymatic activity. Here, while the authors repeat the same manipulation and apply DAPT to block gamma-secretase activity, it seems to induce the opposite effect and reduces FR (comparing figures 8 with figures 5,6,7). First, there is no quantification comparing FR with and without DAPT. Moreover, it is possible to conduct this experiment in the same animals, meaning comparing FR before and after DAPT in the same mouse and cell populations. This point is absolutely critical- if indeed FR is reduced following DAPT application, this needs to be explained since this contradicts the basic design and interpretation of the biosensor.

      We appreciate the reviewer’s comment. In our hand, overexpression of γ-secretase four components (PSEN, Nct, Aph1, and Pen2) is the only reliable and reproducible approach to increase the cellular activity of γ-secretase, which we successfully employed in vitro but not in vivo yet. Therefore, a γ-secretase inhibitor was used to determine the dynamic range of our FRET biosensor in vivo. FRET efficiency depends on the proximity and orientation of donor and acceptor fluorescent proteins. In our initial study, we engineered the original C99 EGFP-RFP biosensor (C99 R-G), and the replacement of EGFP and RFP with mTurquoise-GL and YPet, respectively, expanded the dynamic range of the sensor approximately 2 times. Moreover, extending the linker length from 20 a.a. to 80 a.a. increased the dynamic range 2.2 times (Maesako et al., 2020 iScience). Of note, the C99 720-670 NIR analog, which has the same 80 a.a. linker but miRFP670 and miRFP720 as the donor and acceptor, exhibited a slightly better dynamic range than the C99 Y-T sensor (Houser et al., 2020 Sensor). Our interpretation, at that time, was that the cleavage of the C99 720-670 biosensor by γ-secretase results in a longer distance between the donor and acceptor, and thus, the FRET ratio always increases by γ-secretase inhibition (i.e., proximity plays a more significant role than orientation in our biosensors). As expected, a significantly increased FRET ratio was detected in various cell lines by γ-secretase inhibitors, including CHO, MEF, BV2 cells, and mouse cortical primary neurons. Moreover, to further ensure the C99 720-670 biosensor records changes in γ-secretase activity, the multiplexing capability of the biosensor was utilized. In other words, we co-expressed the C99 720-670 biosensor and visible range fluorescence reporters to record other biological events, such as changes in ion concentration, etc., in cortex primary neurons. Strikingly, several biological events uniquely detected in the neurons with diminished endogenous γ-secretase activity, i.e., neurons with higher FRET ratios, are recapitulated by pharmacological inhibition of γ-secretase (unpublished observation). This approach has allowed us to ensure that increased FRET ratios are indicative of decreased endogenous γ-secretase activity in mouse cortical primary neurons. 

      However, as recommended by the reviewer, we have performed a new experiment to compare the FRET ratio before and after DAPT, a potent γ-secretase inhibitor, administration in the same mouse and cell populations. Surprisingly, we found that of DAPT significantly decreases 720/670 ratios, which is included in our revised manuscript (Figure 2—figure supplement 2C). This unexpected FRET reduction by γ-secretase inhibition was also found in mouse primary neurons derived from the cerebellum (unpublished observation). These findings suggest that orientation plays a significant role in our γ-secretase FRET biosensor and whether the FRET ratio is increased or decreased by the γ-secretase-mediated cleavage depends on cell types. Of note, the difference in FRET ratios with and without DAPT was comparable between primary cortex neurons (24.3%) and the somatosensory cortex neurons in vivo (22.1%). Our new findings suggest that how our biosensors report γ-secretase activity (i.e., increased vs. decreased FRET ratio) must be examined on a model-by-model basis, which is clearly noted in the revised manuscript: 

      Reference

      - Houser MC, Hou SS, Perrin F, Turchyna Y, Bacskai BJ, Berezovska O, Maesako M. A Novel NIRFRET Biosensor for Reporting PS/γ-Secretase Activity in Live Cells. Sensors (Basel). 2020 Oct 22;20(21):5980. 

      - Maesako M, Sekula NM, Aristarkhova A, Feschenko P, Anderson LC, Berezovska O. Visualization of PS/γ-Secretase Activity in Living Cells. iScience. 2020 Jun 26;23(6):101139. 

      (3) For further validation, I would suggest including in vivo measurements with a sensor version with no biological activity as a negative control, for example, a mutation that prevents enzymatic cleavage and FRET changes. This should be used to showcase instrumental variability and would help to validate the variability of FR is indeed biological in origin. This would significantly strengthen the claims regarding spatial correlation within population of cells.

      We fully agree with the reviewer that having a sensor version containing a mutation, which prevents enzymatic cleavage and thus FRET changes, as a negative control is preferable. In our previous study, we developed and validated the APP-based C99 Y-T and Notch1-based N100 Y-T biosensors (Maesako et al., 2020 iScience). It is well established that Notch1 cleavage is entirely blocked by Notch1 V1744G mutation (Schroeter et al., 1998 Nature; Huppert et al., 2000 Nature), and therefore, we introduced the mutation into N100 Y-T biosensor and used it as a negative control. On the other hand, such a striking mutation has never been identified in APP processing. To successfully monitor γ-secretase activity in deep tissue in vivo, we replaced Turquoise-GL and YPet in the C99 Y-T and N100 Y-T biosensors with miRFP670 and miRFP720, respectively. While the APP-based C99 720-670 biosensor allows recording γ-secretase activity (Houser et al., 2020 Sensors), we found the N100 720-670 sensor exhibits a very small dynamic range, not enabling to reliably measure γ-secretase activity. Taken together, there is not currently available NIR γ-secretase biosensor with no biological activity.

      Reference

      - Houser MC, Hou SS, Perrin F, Turchyna Y, Bacskai BJ, Berezovska O, Maesako M. A Novel NIRFRET Biosensor for Reporting PS/γ-Secretase Activity in Live Cells. Sensors (Basel). 2020 Oct 22;20(21):5980. 

      - Huppert SS, Le A, Schroeter EH, Mumm JS, Saxena MT, Milner LA, Kopan R. Embryonic lethality in mice homozygous for a processing-deficient allele of Notch1. Nature. 2000 Jun 22;405(6789):966-70. 

      - Maesako M, Sekula NM, Aristarkhova A, Feschenko P, Anderson LC, Berezovska O. Visualization of PS/γ-Secretase Activity in Living Cells. iScience. 2020 Jun 26;23(6):101139. 

      - Schroeter EH, Kisslinger JA, Kopan R. Notch-1 signalling requires ligand-induced proteolytic release of intracellular domain. Nature. 1998 May 28;393(6683):382-6. 

      (4) In general, confocal microcopy is not ideal for in vivo imaging. Although the authors demonstrate data collected using IR imaging increases penetration depth, out of focus fluorescence is still evident (Figure 4). Many previous papers have primarily used FLIM based analysis in combination with 2p microscopy for in vivo FRET imaging (Some examples: Ma et al, Neuron, 2018; Massengil et al, Nature methods, 2022; DIaz-Garcia et al, Cell Metabolism, 2017; Laviv et al, Neuron, 2020). This technique does not rely on absolute photon number and therefore has several advantage sin terms of quantification of FRET signals in vivo.

      It is therefore likely that use of previously developed sensors of gamma-secretase with conventional FRET pairs, might be better suited for in vivo imaging. This point should be at least discussed as an alternative.

      The reviewer notes that 2p-FLIM may provide certain advantages over our confocal spectral imaging approach for detecting in vivo FRET. In our response below, we will address both the FRET detection method (FLIM vs. spectral) and microscope modality (2p vs. confocal). 

      As noted by the reviewer, we do acknowledge that 2p-FLIM has been utilized to detect FRET in vivo. On the other hand, the ratiometric spectral FRET approach has also been utilized in many in vivo FRET studies (Kuchibhotla et al., 2008 Neuron; Kuchibhotla et al., 2014 PNAS; Hiratsuka et al., 2015 eLife; Maesako et al., 2017 eLife; Konagaya et al., 2017 Cell Rep; Calvo-Rodriguez et al., 2020 Nat Communi; Hino et al., 2022 Dev Cell). We think both approaches have advantages and disadvantages, as discussed in a previous review (Bajar et al., 2016 Sensors), but they complement each other. Indeed, we regularly employ FLIM in cell culture studies (Maesako et al., 2017 eLife; McKendell et al., 2022 Biosensors; Devkota 2024 Cell Rep), and our recent study also utilized 2p-FLIM for in vivo NIR imaging (although not for detecting FRET) (Hou et al., 2023, Nat Biomed Eng); therefore, we are confident that 2p-FLIM can be adapted in our follow-up studies for γ-secretase recording.

      Regarding microscope modality, we agree with the reviewer’s point that generally two-photon microscopy can achieve larger penetration depths than confocal microscopy and is therefore more ideal for in vivo FRET imaging. However, in this study, since our aim was to quantify γ-secretase activity in the superficial layers of the cortex (<200 microns in depth), both NIR confocal and multiphoton microscopies could be used to achieve this imaging objective. Additionally, we chose to use confocal microscopy with our NIR C99 720-670 probe due to the probe’s slightly but higher sensitivity compared to our C99 Y-T probe (Houser et al., 2020 Sensors). Imaging γ-secretase activity with our NIR C99-720-670 probe has the additional advantage that it will allow us in future studies to multiplex with visible FRET pairs using multiphoton microscopy in the same brain region. Furthermore, our demonstration of in vivo FRET imaging using NIR confocal microscopy avoids some of the issues associated with multiphoton microscopy, including potential phototoxicity due to high average and peak laser powers and the high complexity and costs of the instrumentation. For future studies aimed at interrogating γ-secretase activity in deeper cortical regions, multiphoton microscopy could be applied for FLIM or ratiometric spectral imaging of either our NIR or visible FRET probes. Per the reviewer’s request, we have added multiphoton FRET imaging as an alternative in the discussion section. 

      Reference

      - Bajar BT, Wang ES, Zhang S, Lin MZ, Chu J. A Guide to Fluorescent Protein FRET Pairs. Sensors (Basel). 2016 Sep 14;16(9):1488.  

      - Calvo-Rodriguez M, Hou SS, Snyder AC, Kharitonova EK, Russ AN, Das S, Fan Z, Muzikansky A,

      Garcia-Alloza M, Serrano-Pozo A, Hudry E, Bacskai BJ. Increased mitochondrial calcium levels

      associated with neuronal death in a mouse model of Alzheimer's disease. Nat Commun. 2020 May

      1;11(1):2146  

      - Devkota S, Zhou R, Nagarajan V, Maesako M, Do H, Noorani A, Overmeyer C, Bhattarai S, Douglas JT, Saraf A, Miao Y, Ackley BD, Shi Y, Wolfe MS. Familial Alzheimer mutations stabilize synaptotoxic γ-secretase-substrate complexes. Cell Rep. 2024 Feb 27;43(2):113761. 

      - Hino N, Matsuda K, Jikko Y, Maryu G, Sakai K, Imamura R, Tsukiji S, Aoki K, Terai K, Hirashima T, Trepat X, Matsuda M. A feedback loop between lamellipodial extension and HGF-ERK signaling specifies leader cells during collective cell migration. Dev Cell. 2022 Oct 10;57(19):2290-2304.e7.

      - Hiratsuka T, Fujita Y, Naoki H, Aoki K, Kamioka Y, Matsuda M. Intercellular propagation of extracellular signal-regulated kinase activation revealed by in vivo imaging of mouse skin. eLife. 2015 Feb 10;4:e05178.  

      - Hou SS, Yang J, Lee JH, Kwon Y, Calvo-Rodriguez M, Bao K, Ahn S, Kashiwagi S, Kumar ATN, Bacskai BJ, Choi HS. Near-infrared fluorescence lifetime imaging of amyloid-β aggregates and tau fibrils through the intact skull of mice. Nat Biomed Eng. 2023 Mar;7(3):270-280.  

      - Houser MC, Hou SS, Perrin F, Turchyna Y, Bacskai BJ, Berezovska O, Maesako M. A Novel NIRFRET Biosensor for Reporting PS/γ-Secretase Activity in Live Cells. Sensors (Basel). 2020 Oct 22;20(21):5980. 

      - Konagaya Y, Terai K, Hirao Y, Takakura K, Imajo M, Kamioka Y, Sasaoka N, Kakizuka A, Sumiyama K, Asano T, Matsuda M. A Highly Sensitive FRET Biosensor for AMPK Exhibits Heterogeneous AMPK Responses among Cells and Organs. Cell Rep. 2017 Nov 28;21(9):2628-2638.  

      - Kuchibhotla KV, Goldman ST, Lattarulo CR, Wu HY, Hyman BT, Bacskai BJ. Abeta plaques lead to aberrant regulation of calcium homeostasis in vivo resulting in structural and functional disruption of neuronal networks. Neuron. 2008 Jul 31;59(2):214-25  

      - Kuchibhotla KV, Wegmann S, Kopeikina KJ, Hawkes J, Rudinskiy N, Andermann ML, Spires-Jones TL, Bacskai BJ, Hyman BT. Neurofibrillary tangle-bearing neurons are functionally integrated in cortical circuits in vivo. Proc Natl Acad Sci U S A. 2014 Jan 7;111(1):510-4  

      - Maesako M, Horlacher J, Zoltowska KM, Kastanenka KV, Kara E, Svirsky S, Keller LJ, Li X, Hyman BT, Bacskai BJ, Berezovska O. Pathogenic PS1 phosphorylation at Ser367. Elife. 2017 Jan 30;6:e19720.  

      - McKendell AK, Houser MCQ, Mitchell SPC, Wolfe MS, Berezovska O, Maesako M. In-Depth

      Characterization of Endo-Lysosomal Aβ in Intact Neurons. Biosensors (Basel). 2022 Aug 20;12(8):663. 

      (Recommendations For The Authors):

      (5) Minor issues- Figure 4 describes the analysis procedure, which seems to be standard practice in the field. This can be described in the methods section rather than in the main figure.

      Per the reviewer’s suggestion, this figure has been moved to Figure 2—figure supplement 1. 

      Reviewer #3 (Public Review):

      (1) This paper builds on the authors' original development of a near infrared (NIR) FRET sensor by reporting in vivo real-time measurements for gamma-secretase activity in the mouse cortex. The in vivo application of the sensor using state of the art techniques is supported by a clear description and straightforward data, and the project represents significant progress because so few biosensors work in vivo. Notably, the NIR biosensor is detectable to ~ 100 µm depth in the cortex. A minor limitation is that this sensor has a relatively modest ΔF as reported in Houser et al, which is an additional challenge for its use in vivo. Thus, the data is fully dependent on post-capture processing and computational analyses. This can unintentionally introduce biases but is not an insurmountable issue with the proper controls that the authors have performed here.

      We appreciate the reviewer’s overall positive evaluation. As described in our response to the Reviewer 2’s critique (2), ΔF in vivo has been characterized (Figure 2—figure supplement 2C).

      (2) The observation of gamma-secretase signaling that spreads across cells is potentially quite interesting, but it can be better supported. An alternative interpretation is that there exist pre-formed and clustered hubs of high gamma-secretase activity, and that DAPT has stochastic or differential accessibility to cells within the cluster. This could be resolved by an experiment of induction, for example, if gamma-secretase activity is induced or activated at a specific locale and there was observed coordinated spreading to neighboring neurons with their sensor.

      We agree with the reviewer that the stochastic or differential accessibility of DAPT to cell clusters with different γ-secretase can be an alternative interpretation of our data, which is now included in the Discussion of the revised manuscript. Undoubtedly, the activation of γ-secretase would provide valuable information. However, as described in the response above to Reviewer 2’s critique #2, overexpressing the four components of γ-secretase (PSEN, Nct, Aph1, and Pen2) is the only reliable and reproducible approach to increasing the cellular activity of γ-secretase, which was achieved in our in vitro study but not yet in vivo. Our future study will develop and characterize the approach to induce γ-secretase activity to further perform detailed mechanistic studies.

      (3) Furthermore, to rule out the possibility that uneven viral transduction was not simply responsible for the observed clustering, it would be helpful to see an analysis of 670nm fluorescence alone.

      Our new analysis comparing 670 nm fluorescence intensity and that in five neighbor neurons shows a positive correlation (Figure 3—figure supplement 1A), suggesting that AAV was unevenly transduced. On the other hand, the 720/670 ratio (i.e., γ-secretase activity) is not correlated with 670 nm fluorescence intensity (i.e., C99 720-670 biosensor expression) (Figure 3—figure supplement 1B). This strongly suggests that, while C99 720-670 biosensor expression was not evenly distributed in the brain, the uneven probe expression did not impact the capability of γ-secretase recording.  

      Reviewer #3 (Recommendations For The Authors):

      (4) One minor suggestion might be to consider Figures 6-7 as orthogonal supporting analyses rather than "validation". It might then be helpful to present them together with Figure 5.

      We have moved the initial Figure 6 and 7 to Figure 3—figure supplement 2 and Figure 4, respectively.

    1. Author response:

      The following is the authors’ response to the original reviews

      Reviewer #1:

      (1) Line numbers are missing.

      Added

      (2) VR classroom. Was this a completely custom design based on Unity, or was this developed on top of some pre-existing code? Many aspects of the VR classroom scenario are only introduced (e.g., how was the lip-speech synchronisation done exactly?). Additional detail is required. Also, is or will the experiment code be shared publicly with appropriate documentation? It would also be useful to share brief example video-clips.

      We have added details about the VR classroom programming to the methods section (p. 6-7), and we have now included a video-example as supplementary material.

      “Development and programming of the VR classroom were done primarily in-house, using assets (avatars and environment) were sourced from pre-existing databases. The classroom environment was adapted from assets provided by Tirgames on TurboSquid (https://www.turbosquid.com/Search/Artists/Tirgames) and modified to meet the experimental needs. The avatars and their basic animations were sourced from the Mixamo library, which at the time of development supported legacy avatars with facial blendshapes (this functionality is no longer available in current versions of Mixamo). A brief video example of the VR classroom is available at: https://osf.io/rf6t8.

      “To achieve realistic lip-speech synchronization, the teacher’s lip movements were controlled by the temporal envelope of the speech, adjusting both timing and mouth size dynamically. His body motions were animated using natural talking gestures.”

      While we do intent to make the dataset publicly available for other researchers, at this point we are not making the code for the VR classroom public. However, we are happy to share it on an individual-basis with other researchers who might find it useful for their own research in the future.

      (3) "normalized to the same loudness level using the software Audacity". Please specify the Audacity function and parameters.

      We have added these details (p.7)

      “All sound-events were normalized to the same loudness level using the Normalize function in the audio-editing software Audacity (theaudacityteam.org, ver 3.4), with the peak amplitude parameter set to -5 dB, and trimmed to a duration of 300 milliseconds.“

      (4) Did the authors check if the participants were already familiar with some of the content in the mini-lectures?

      This is a good point. Since the mini-lectures spanned many different topics, we did not pre-screen participants for familiarity with the topics, and it is possible that some of the participants had some pre-existing knowledge.

      In hindsight, it would have been good to have added some reflective questions regarding participants prior knowledge as well as other questions such as level of interest in the topic and/or how well they understood the content. These are elements that we hope to include in future versions of the VR classroom.

      (5) "Independent Component Analysis (ICA) was then used to further remove components associated with horizontal or vertical eye movements and heartbeats". Please specify how this selection was carried out.

      Selection of ICA components was done manually based on visual inspection of their time-course patterns and topographical distributions, to identify components characteristic of blinks, horizontal eye-movements and heartbeats). Examples of these distinct components are provided in Author response image 1 below. These is now specified in the methods section.

      Author response image 1.

      (6) "EEG data was further bandpass filtered between 0.8 and 20 Hz". If I understand correctly, the data was filtered a second time. If that's the case, please do not do that, as that will introduce additional and unnecessary filtering artifacts. Instead, the authors should replace the original filter with this one (so, filtering the data only once). Please see de Cheveigne and Nelkn, Neuron, 2019 for an explanation. Also, please provide an explanation of the rationale for further restricting the cut-off bands in the methods section. Finally, further details on the filters should be included (filter type and order, for example).

      Yes, the data was indeed filtered twice. The first filter is done as part of the preprocessing procedure, in order to remove extremely high- and low- frequency noise but retain most activity within the range of “neural” activity. This broad range is mostly important for the ICA procedure, so as to adequately separate between ocular and neural contribution to the recorded signal.

      However, since both the speech tracking responses and ERPs are typically less broadband and are comprised mostly of lower frequencies (e.g., those that make up the speech-envelope), a second narrower filter was applied to improve TRF model-fit and make ERPs more interpretable.

      In both cases we used a fourth order zero-phase Butterworth IIR filter with 1-seconds of padding, as implemented in the Fieldtrip toolbox. We have added these details to the manuscript.

      (7) "(~ 5 minutes of data in total), which is insufficient for deriving reliable TRFs". That is a bit pessimistic and vague. What does "reliable" mean? I would tend to agree when talking about individual subject TRFs, which 5 min per participant can be enough at the group level. Also, this depends on the specific speech material. If the features are univariate or multivariate. Etc. Please narrow down and clarify this statement.

      We determined that the data in the Quiet condition (~5 min) was insufficient for performing reliable TRF analysis, by assessing whether its predictive-power was significantly better than chance. As shown in Author response image 2 below, the predictive power achieved using this data was not higher than values obtained in permuted data (p = 0.43). Therefore, we did not feel that it was appropriate to include TRF analysis of the Quiet condition in this manuscript. We have now clarified this in the manuscript (p. 10)

      Author response image 2.

      (8) "Based on previous research in by our group (Kaufman & Zion Golumbic 2023), we chose to use a constant regularization ridge parameter (λ= 100) for all participants and conditions". This is an insufficient explanation. I understand that there is a previous paper involved. However, such an unconventional choice that goes against the original definition and typical use of these methods should be clearly reported in this manuscript.

      We apologize for not clarifying this point sufficiently, and have added an explanation of this methodological choice (p.11):

      “The mTRF toolbox uses a ridge-regression approach for L2 regularization of the model to ensure better generalization to new data. We tested a range of ridge parameter values (λ's) and used a leave-one-out cross-validation procedure to assess the model’s predictive power, whereby in each iteration, all but one trials are used to train the model, and it is then applied to the left-out trial. The predictive power of the model (for each λ) is estimated as the Pearson’s correlation between the predicted neural responses and the actual neural responses, separately for each electrode, averages across all iterations. We report results of the model with the λ the yielded the highest predictive power at the group-level (rather than selecting a different λ for each participant which can lead to incomparable TRF models across participants; see discussion in Kaufman & Zion Golumbic 2023).”

      Assuming that the explanation will be sufficiently convincing, which is not a trivial case to make, the next issue that I will bring up is that the lambda value depends on the magnitude of input and output vectors. While the input features are normalised, I don't see that described for the EEG signals. So I assume they are not normalized. In that case, the lambda would have at least to be adapted between subjects to account for their different magnitude.

      We apologize for omitting this detail – yes, the EEG signals were normalized prior to conducting the TRF analysis. We have updated the methods section to explicitly state this pre-processing step (p.10).

      Another clarification, is that value (i.e., 100) would not be comparable either across subjects or across studies. But maybe the authors have a simple explanation for that choice? (note that this point is very important as this could lead others to use TRF methods in an inappropriate way - but I understand that the authors might have specific reasons to do so here). Note that, if the issue is finding a reliable lambda per subject, a more reasonable choice would be to use a fixed lambda selected on a generic (i.e., group-level) model. However selecting an arbitrary lambda could be problematic (e.g., would the results replicate with another lambda; and similarly, what if a different EEG system was used, with different overall magnitude, hence the different impact of the regularisation).

      We fully agree that selecting an arbitrary lambda is problematic (esp across studies). As clarified above, the group-level lambda chosen here for the encoding more was data-driven, optimized based on group-level predictive power.

      (9) "L2 regularization of the model, to reduce its complexity". Could the authors explain what "reduce its complexity" refers to?

      Our intension here was to state that the L2 regularization constrains the model’s weights so that it can better generalize between to left-out data. However, for clarity we have now removed this statement.

      (10) The same lambda value was used for the decoding model. From personal experience, that is very unlikely to be the optimal selection. Decoding models typically require a different (usually larger) lambda than forward models, which can be due to different reasons (different SNR of "input" of the model and, crucially, very different dimensionality).

      We agree with the reviewer that treatment of regularization parameters might not be identical for encoding and decoding models. Our initial search of lambda parameters was limited to λ= 0.01 - 100, with λ= 100 showing the best reconstruction correlations. However, following the reviewer’s suggestion we have now broadened the range and found that, in fact reconstruction correlations are further improved and the best lambda is λ= 1000 (see Author response image 3 below, left panel). Importantly, the difference in decoding reconstruction correlations between the groups is maintained regardless of the choice of lambda (although the effect-size varies; see Author response image 3, right panel). We have now updated the text to reflect results of the model with λ= 1000.

      Author response image 3.

      (11) Skin conductance analysis. Additional details are required. For example, how was the linear interpolation done exactly? The raw data was downsampled, sure. But was an anti-aliasing filter applied? What filter exactly? What implementation for the CDA was run exactly?

      We have added the following details to the methods section (p. 14):

      “The Skin Conductance (SC) signal was analyzed using the Ledalab MATLAB toolbox (version 3.4.9; Benedek and Kaernbach, 2010; http://www.ledalab.de/) and custom-written scripts. The raw data was downsampled to 16Hz using FieldTrip's ft_resampledata function, which applies a built-in anti-aliasing low-pass filter to prevent aliasing artifacts. Data were inspected manually for any noticeable artifacts (large ‘jumps’), and if present were corrected using linear interpolation in Ledalab. A continuous decomposition analysis (CDA) was employed to separate the tonic and phasic SC responses for each participant. The CDA was conducted using the 'sdeco' mode (signal decomposition), which iteratively optimizes the separation of tonic and phasic components using the default regularization settings.”

      (12) "N1- and P2 peaks of the speech tracking response". Have the authors considered using the N1-P2 complex rather than the two peaks separately? Just a thought.

      This is an interesting suggestion, and we know that this has been used sometimes in more traditional ERP literature. In this case, since neither peak was modulated across groups, we did not think this would yield different results. However, it is a good point to keep in mind for future work.

      (13) Figure 4B. The ticks are missing. From what I can see (but it's hard without the ticks), the N1 seems later than in other speech-EEG tracking experiments (where is closer to ~80ms). Could the authors comment on that? Or maybe this looks similar to some of the authors' previous work?

      We apologize for this and have added ticks to the figure.

      In terms of time-course, a N1 peak at around 100ms is compatible with many of our previous studies, as well as those from other groups.

      (14) Figure 4C. Strange thin vertical grey bar to remove.

      Fixed.

      (15) Figure 4B: What about the topographies for the TRF weights? Could the authors show that for the main components?

      Yes. The topographies of the main TRF components are similar to those of the predictive power and are compatible with auditory responses. We have added them to Figure 4B.

      (16) Figure 4B: I just noticed that this is a grand average TRF. That is ok (but not ideal) only because the referencing is to the mastoids. The more appropriate way of doing this is to look at the GFP, instead, which estimates the presence of dipoles. And then look at topographies of the components. Averaging across channels makes the plotted TRF weaker and noisier. I suggest adding the GFP to the plot. Also, the colour scale in Figure 4A is deceiving, as blue is usually used for +/- in plots of the weights. While that is a heatmap, where using a single colour or even yellow to red would be less deceiving at first look. Only cosmetics, indeed. The result is interesting nonetheless!

      We apologize for this, and agree with the reviewer that it is better not to average across EEG channels. In the revised Figure, we now show the TRFs based on the average of electrodes FC1, FC2, and FCz, which exhibited the strongest activity for the two main components.

      Following the previous comment, we have also included the topographical representation of the TRF main components, to give readers a whole-head perspective of the TRF.

      We have also fixed the color-scales.

      We are glad that the reviewer finds this result interesting!

      (17) Figure 4C. This looks like a missed opportunity. That metric shows a significant difference overall. But is that underpinned but a generally lower envelope reconstruction correlation, or by a larger deviation in those correlations (so, that metric is as for the control in some moments, but it drops more frequently due to distractibility)?

      We understand the reviewer’s point here, and ideally would like to be able to address this in a more fine-grained analysis, for example on a trial-by-trial basis. However, the design of the current experiment was not optimized for this, in terms of (for example) number of trials, the distribution of sound-events and behavioral outcomes. We hope to be able to address this issue in our future research.

      (18) I am not a fan of the term "accuracy" for indicating envelope reconstruction correlations. Accuracy is a term typically associated with classification. Regression models are typically measured through errors, loss, and sometimes correlations. 'Accuracy' is inaccurate (no joke intended).

      We accept this comment and now used the term “reconstruction correlation”.

      (19) Discussion. "The most robust finding in". I suggest using more precise terminology. For example, "largest effect-size".

      We agree and have changed the terminology (p. 31).

      (20) "individuals who exhibited higher alpha-power [...]". I probably missed this. But could the authors clarify this result? From what I can see, alpha did not show an effect on the group. Is this referring to Table 2? Could the authors elaborate on that? How does that reconcile with the non-significant effect of the group? In that same sentence, do you mean "and were more likely"? If that's the case, and they were more likely to report attentional difficulties, how is it that there is no group-effect when studying alpha?

      Yes, this sentence refers to the linear regression models described in Figure 10 and in Table 2. As the reviewer correctly points out, this is one place where there is a discrepancy between the results of the between-group analysis (ADHD diagnosis yes/no) and the regression analysis, which treats ADHD symptoms as a continuum, across both groups. The same is true for the gaze-shift data, which also did not show a significance between-group effect but was identified in the regression analysis as contributing to explaining the variance in ADHD symptoms.

      We discuss this point on pages 30-31, noting that “although the two groups are clearly separable from each other, they are far from uniform in the severity of symptoms experienced”, which motivated the inclusion of both analyses in this paper.

      At the bottom of p. 31 we specifically address the similarities and differences between the between-group and regression-based results. In our opinion, this pattern emphasizes that while neither approach is ‘conclusive’, looking at the data through both lenses contributes to an overall better understanding of the contributing factors, as well as highlighting that “no single neurophysiological measure alone is sufficient for explaining differences between the individuals – whether through the lens of clinical diagnosis or through report of symptoms”.

      (21) "why in the latter case the neural speech-decoding accuracy did not contribute to explaining ASRS scores [...]". My previous point 1 on separating overall envelope decoding from its deviation could help there. The envelope decoding correlation might go up and down due to SNR, while you might be more interested in the dynamics over time (i.e., looking at the reconstructions over time).

      Again, we appreciate this comment, but believe that this additional analysis is outside the scope of what would be reliably-feasible with the current dataset. However, since the data will be made publicly available, perhaps other researchers will have better ideas as to how to do this.

      (22) Data and code sharing should be discussed. Also, specific links/names and version numbers should be included for the various libraries used.

      We are currently working on organizing the data to make it publicly available on the Open Science Project.

      We have updated links and version numbers for the various toolboxes/software used, throughout the manuscript.

      Reviewer #2:

      (1) While it is highly appreciated to study selective attention in a naturalistic context, the readers would expect to see whether there are any potential similarities or differences in the cognitive and neural mechanisms between contexts. Whether the classic findings about selective attention would be challenged, rebutted, or confirmed? Whether we should expect any novel findings in such a novel context? Moreover, there are some studies on selective attention in the naturalistic context though not in the classroom, it would be better to formulate specific hypotheses based on previous findings both in the strictly controlled and naturalistic contexts.

      Yes, we fully agree that comparing results across different contexts would be extremely beneficial and important.

      The current paper serves as an important proof-first-concept demonstrating the plausibility and scientific potential of using combined EEG-VR-eyetracking to study neurophysiological aspects of attention and distractibility, but is also the basis for formulating specific hypothesis that will be tested in follow-up studies.

      If fact, a follow up study is already ongoing in our lab, where we are looking into this point, by testing users in different VR scenarios (e.g., classroom, café, office etc.), and assessing whether similar neurophysiological patterns are observed across contexts and to what degree they are replicable within and across individuals. We hope to share these data with the community in the near future.

      (2) Previous studies suggest handedness and hemispheric dominance might impact the processing of information in each hemisphere. Whether these issues have been taken into consideration and appropriately addressed?

      This is an interesting point. In this study we did not specifically control for handedness/hemispheric dominance, since most of the neurophysiological measured used here are sensory/auditory in their nature, and therefore potentially invariant to handedness. Moreover, the EEG signal is typically not very sensitive to hemispheric dominance, at least for the measures used here. However, this might be something to consider more explicitly in future studies. Nonetheless, we have added handedness information to the Methods section (p. 5): “46 right-handed, 3 left-handed”

      (3) It would be interesting to know how students felt about the Virtual Classroom context, whether it is indeed close to the real classroom or to some extent different.

      Yes, we agree. Obviously, the VR classroom differs in many ways from a real classroom, in terms of the perceptual experience, social aspects and interactive possibilities. We did ask participants about their VR experience after the experiment, and most reported feeling highly immersed in the VR environment and engaged in the task, with a strong sense of presence in the virtual-classroom.

      We note that, in parallel to the VR studies in our lab, we are also conducting experiments in real classrooms, and we hope that the cross-study comparison will be able to shed more light on these similarities/differences.

      (4) One intriguing issue is whether neural tracking of the teacher's speech can index students' attention, as the tracking of speech may be relevant to various factors such as sound processing without semantic access.

      Another excellent point. While separating the ‘acoustic’ and ‘semantic’ contributions to the speech tracking response is non-trivial, we are currently working on methodological approaches to do this (again, in future studies) following, for example, the hierarchical TRF approach used by Brodbeck et al. and others.

      (5) There are many results associated with various metrics, and many results did not show a significant difference between the ADHD group and the control group. It is difficult to find the crucial information that supports the conclusion. I suggest the authors reorganize the results section and report the significant results first, and to which comparison(s) the readers should pay attention.

      We apologize if the organization of the results section was difficult to follow. This is indeed a challenge when collecting so many different neurophysiological metrics.

      To facilitate this, we have now added a paragraph at the beginning of the result section, clarifying its structure (p.16):

      The current dataset is extremely rich, consisting of many different behavioral, neural and physiological responses. In reporting these results, we have separated between metrics that are associated with paying attention to the teacher (behavioral performance, neural tracking of the teacher’s speech, and looking at the teacher), those capturing responses to the irrelevant sound-events (ERPs and event-related changes in SC and gaze); as well as more global neurophysiological measures that may be associated with the listeners’ overall ‘state’ of attention or arousal (alpha- and beta-power and tonic SC).

      Moreover, within each section we have ordered the analysis such that the ones with significant effects are first. We hope that this contributes to the clarity of the results section.

      (6) The difference between artificial and non-verbal humans should be introduced earlier in the introduction and let the readers know what should be expected and why.

      We have added this to the Introduction (p. 4)

      (7) It would be better to discuss the results against a theoretical background rather than majorly focusing on technical aspects.

      We appreciate this comment. In our opinion, the discussion does contain a substantial theoretical component, both regarding theories of attention and attention-deficits, and also regarding their potential neural correlates. However, we agree that there is always room for more in depth discussion.

      Reviewer #3:

      Major:

      (1) While the study introduced a well-designed experiment with comprehensive physiological measures and thorough analyses, the key insights derived from the experiment are unclear. For example, does the high ecological validity provide a more sensitive biomarker or a new physiological measure of attention deficit compared to previous studies? Or does the study shed light on new mechanisms of attention deficit, such as the simultaneous presence of inattention and distraction (as mentioned in the Conclusion)? The authors should clearly articulate their contributions.

      Thanks for this comment.

      We would not say that this paper is able to provide a ‘more sensitive biomarker’ or a ‘new physiological measure of attention’ – in order to make those type of grand statements we would need to have much more converging evidence from multiple studies and using both replication and generalization approaches.

      Rather, from our perspective, the key contribution of this work is in broadening the scope of research regarding the neurophysiological mechanisms involved in attention and distraction.

      Specifically, this work:

      (1) Offers a significant methodological advancement of the field – demonstrating the plausibility and scientific potential of using combined EEG-VR-eyetracking to study neurophysiological aspects of attention and distractibility in contexts that ‘mimic’ real-life situations (rather than highly controlled computerized tasks).

      (2) Provides a solid basis formulating specific mechanistic hypothesis regarding the neurophysiological metrics associated with attention and distraction, the interplay between them, and their potential relation to ADHD-symptoms. Rather than being an end-point, we see these results as a start-point for future studies that emphasize ecological validity and generalizability across contexts, that will hopefully lead to improved mechanisms understanding and potential biomarkers of real-life attentional capabilities (see also response to Rev #2 comment #1 above).

      (3) Highlights differences and similarities between the current results and those obtained in traditional ‘highly controlled’ studies of attention (e.g., in the way ERPs to sound-events differ between ADHD and controls; variability in gaze and alpha-power; and more broadly about whether ADHD symptoms do or don’t map onto specific neurophysiological metrics). Again, we do not claim to give a definitive ’answer’ to these issues, but rather to provide a new type of data that can expands the conversation and address the ecological validity gap in attention research.

      (2) Based on the multivariate analyses, ASRS scores correlate better with the physiological measures rather than the binary deficit category. It may be worthwhile to report the correlation between physiological measures and ASRS scores for the univariate analyses. Additionally, the correlation between physiological measures and behavioral accuracy might also be interesting.

      Thanks for this. The beta-values reported for the regression analysis reflect the correlations between the different physiological measures and the ASRS scores (p. 30). From a statistical perspective, it is better to report these values rather than the univariate correlation-coefficients, since these represent the ‘unique’ relationship with each factor, after controlling for all the others.

      The univariate correlations between the physiological measures themselves, as well as with behavioral accuracy, are reported in Figure 10

      (3) For the TRF and decoding analysis, the authors used a constant regularization parameter per a previous study. However, the optimal regularization parameter is data-dependent and may differ between encoding and decoding analyses. Furthermore, the authors did not conduct TRF analysis for the quiet condition due to the limited ~5 minutes of data. However, such a data duration is generally sufficient to derive a stable TRF with significant predictive power (Mesik and Wojtczak, 2023).

      The reviewer raises two important points, also raised by Rev #1 (see above).

      Regarding the choice of regularization parameters, we have now clarified that although we used a common lambda value for all participants, it was selected in a data-driven manner, so as to achieve an optimal predictive power at the group-level.

      See revised methods section:

      “The mTRF toolbox uses a ridge-regression approach for L2 regularization of the model to ensure better generalization to new data. We tested a range of ridge parameter values (λ's) and used a leave-one-out cross-validation procedure to assess the model’s predictive power, whereby in each iteration, all but one trials are used to train the model, and it is then applied to the left-out trial. The predictive power of the model (for each λ) is estimated as the Pearson’s correlation between the predicted neural responses and the actual neural responses, separately for each electrode, averages across all iterations. We report results of the model with the λ the yielded the highest predictive power at the group-level (rather than selecting a different λ for each participant which can lead to incomparable TRF models across participants; see discussion in Kaufman & Zion Golumbic 2023).”

      Regarding whether data was sufficient in the Quiet condition for performing TRF analysis – we are aware of the important work by Mesik & Wojtczak, and had initially used this estimate when designing our study. However, when assessing the predictive-power of the TRF model trained on data from the Quiet condition, we found that it was not significantly better than chance (see Author response image 2, ‘real’ predictive power vs. permuted data). Therefore, we ultimately did not feel that it was appropriate to include TRF analysis of the Quiet condition in this manuscript. We have now clarified this in the manuscript (p. 10)

      (4) As shown in Figure 4, for ADHD participants, decoding accuracy appears to be lower than the predictive power of TRF. This result is surprising because more data (i.e., data from all electrodes) is used in the decoding analysis.

      This is an interesting point – however, in our experience it is not necessarily the case that decoding accuracy (i.e., reconstruction correlation with the stimulus) is higher than encoding predictive-power. While both metrics use Pearson’s’ correlations, they quantify the similarity between two different types of signals (the EEG and the speech-envelope). Although the decoding procedure does use data from all electrodes, many of them don’t actually contain meaningful information regarding the stimulus, and thus could just as well hinder the overall performance of the decoding.

      (5) Beyond the current analyses, the authors may consider analyzing inter-subject correlation, especially for the gaze signal analysis. Given that the area of interest during the lesson changes dynamically, the teacher might not always be the focal point. Therefore, the correlation of gaze locations between subjects might be better than the percentage of gaze duration on the teacher.

      Thanks for this suggestion. We have tried to look into this, however working with eye-gaze in a 3-D space is extremely complex and we are not able to calculate reliable correlations between participants.

      (6) Some preprocessing steps relied on visual and subjective inspection. For instance, " Visual inspection was performed to identify and remove gross artifacts (excluding eye movements) " (P9); " The raw data was downsampled to 16Hz and inspected for any noticeable artifacts " (P13). Please consider using objective processes or provide standards for subjective inspections.

      We are aware of the possible differences between objective methods of artifact rejection vs. use of manual visual inspection, however we still prefer the manual (subjective) approach. As noted, in this case only very large artifacts were removed, exceeding ~ 4 SD of the amplitude variability, so as to preserve as many full-length trials as possible.

      (7) Numerous significance testing methods were employed in the manuscript. While I appreciate the detailed information provided, describing these methods in a separate section within the Methods would be more general and clearer. Additionally, the authors may consider using a linear mixed-effects model, which is more widely adopted in current neuroscience studies and can account for random subject effects.

      Indeed, there are many statistical tests in the paper, given the diverse types of neurophysiological data collected here. We actually thought that describing the statistics per method rather than in a separate “general” section would be easier to follow, but we understand that readers might diverge in their preferences.

      Regarding the use of mixed-effect models – this is indeed a great approach. However, it requires deriving reliable metrics on a per-trial basis, and while this might be plausible for some of our metrics, the EEG and GSR metrics are less reliable at this level. This is why we ultimately chose to aggregate across trials and use a regular regression model rather than mixed-effects.

      (8) Some participant information is missing, such as their academic majors. Given that only two lesson topics were used, the participants' majors may be a relevant factor.

      To clarify – the mini-lectures presented here actually covered a large variety of topics, broadly falling within the domains of history, science and social-science and technology. Regarding participants’ academic majors, these were relatively diverse, as can be seen in Author response table 1 and Author response image 4.

      Author response table 1.

      Author response image 4.

      (9) Did the multiple regression model include cross-validation? Please provide details regarding this.

      Yes, we used a leave-one-out cross validation procedure. We have now clarified this in the methods section which now reads:

      “The mTRF toolbox uses a ridge-regression approach for L2 regularization of the model to ensure better generalization to new data. We tested a range of ridge parameter values (λ's) and used a leave-one-out cross-validation procedure to assess the model’s predictive power, whereby in each iteration, all but one trials are used to train the model, and it is then applied to the left-out trial. The predictive power of the model (for each λ) is estimated as the Pearson’s correlation between the predicted neural responses and the actual neural responses, separately for each electrode, averages across all iterations. We report results of the model with the λ the yielded the highest predictive power at the group-level (rather than selecting a different λ for each participant which can lead to incomparable TRF models across participants; see discussion in Kaufman & Zion Golumbic 2023).”

      Minor:

      (10) Typographical errors: P5, "forty-nine 49 participants"; P21, "$ref"; P26, "Table X"; P4, please provide the full name for "SC" when first mentioned.

      Thanks! corrected

    1. Author response:

      The following is the authors’ response to the original reviews.

      Public Reviews:

      Reviewer #1 (Public Review):

      Hippocampal place cells display a sequence of firing activities when the animal travels through a spatial trajectory at a behavioral time scale of seconds to tens of seconds. Interestingly, parts of the firing sequence also occur at a much shorter time scale: ~120 ms within individual cycles of theta oscillation. These so-called theta sequences are originally thought to naturally result from the phenomenon of theta phase precession. However, there is evidence that theta sequences do not always occur even when theta phase precession is present, for example, during the early experience of a novel maze. The question is then how they emerge with experience (theta sequence development). This study presents evidence that a special group of place cells, those tuned to fast-gamma oscillations, may play a key role in theta sequence development.

      The authors analyzed place cells, LFPs, and theta sequences as rats traveled a circular maze in repeated laps. They found that a group of place cells were significantly tuned to a particular phase of fast-gamma (FG-cells), in contrast to others that did not show such tunning (NFG-cells). The authors then omitted FG-cells or the same number of NFG-cells, in their algorithm of theta sequence detection and found that the quality of theta sequences, quantified by a weighted correlation, was worse with the FG-cell omission, compared to that with the NFG-cell omission, during later laps, but not during early laps. What made the FG-cells special for theta sequences? The authors found that FG-cells, but not NFG-cells, displayed phase recession to slow-gamma (25 - 45 Hz) oscillations (within theta cycles) during early laps (both FG- and NFG-cells showed slow-gamma phase precession during later laps). Overall, the authors conclude that FG-cells contribute to theta sequence development through slow-gamma phase precession during early laps.

      How theta sequences are formed and developed during experience is an important question, because these sequences have been implicated in several cognitive functions of place cells, including memory-guided spatial navigation. The identification of FG-cells in this study is straightforward. Evidence is also presented for the role of these cells in theta sequence development. However, given several concerns elaborated below, whether the evidence is sufficiently strong for the conclusion needs further clarification, perhaps, in future studies.

      We thank the reviewer for these positive comments.

      (1) The results in Figure 3 and Figure 8 seems contradictory. In Figure 8, all theta sequences displayed a seemingly significant weighted correlation (above 0) even in early laps, which was mostly due to FG-cell sequences but not NFG-cell sequences (correlation for NFG-sequences appeared below 0). However, in Figure 3H, omitting FG-cells and omitting NFG-cells did not produce significant differences in the correlation. Conversely, FG-cell and NFG-cell sequences were similar in later laps in Figure 8 (NFG-cell sequences appeared even better than FG-cell sequences), yet omitting NFG-cells produced a better correlation than omitting FG-cells. This confusion may be related to how "FG-cell-dominant sequences" were defined, which is unclear in the manuscript. Nevertheless, the different results are not easy to understand.

      We thank the reviewer for pointing out this important problem.  The potential contradictory can be interpreted by different sequence dataset included in Fig3 and Fig8, described as follows.

      (1) In Fig 3, all sequences decoded without either FG or NFG cells were included, defined as exFG-sequences and exNFG sequences, so that we couldn’t observe sequence development at early phase and thus the weighted correlation was low.  (2) In Fig8, however, the sequences with either FG or NFG cells firing across at least 3 slow gamma cycles were included, defined as FG-cell sequences and NFG-cell sequences.  This criterion ensures to investigate the relationship between sequence development and slow gamma phase precession, so that these sequences were contributed by cells likely to show slow gamma phase precession.  These definitions have been updated to the “Theta sequences detection” section of the Methods (Line 606-619).

      At early phase, there’s still no difference of weighted correlation between FG-cell sequences and NFG-cell sequences (Author response image 1A, Student’s t test, t(65)=0.2, p=0.8, Cohen's D=0.1), but the FG-cell sequences contained high proportion of slow gamma phase precession (Fig8F).  At late phase, both FG-cell sequences and NFG-cell sequences exhibited slow gamma phase precession, so that their weighted correlation were high with no difference (Author response image 1B, Student’s t test, t(62)=-1.1, p=0.3, Cohen's D=0.3).  This result further indicates that the theta sequence development requires slow gamma phase precession, especially for FG cells during early phase.

      Author response image 1.

      (2) The different contributions between FG-cells and NFG-cells to theta sequences are supposed not to be caused by their different firing properties (Figure 5). However, Figure 5D and E showed a large effect size (Cohen's D = 07, 0.8), although not significant (P = 0.09, 0.06). But the seemingly non-significant P values could be simply due to smaller N's (~20). In other parts of the manuscript, the effect sizes were comparable or even smaller (e.g. D = 0.5 in Figure 7B), but interpreted as positive results: P values were significant with large N's (~480 in Fig. 7B). Drawing a conclusion purely based on a P value while N is large often renders the conclusion only statistical, with unclear physical meaning. Although this is common in neuroscience publications, it makes more sense to at least make multiple inferences using similar sample sizes in the same study.

      We thank the reviewer for this kind suggestion.  We made multiple inferences using similar sample sizes as much as possible.  In Fig7B, we did the statistical analysis with sessions as samples, and we found the significant conclusion was maintained.  These results have been updated to the revised manuscript (Lines 269-270).and the Fig7B has been replaced correspondingly.

      (3) In supplementary Figure 2 - S2, FG-cells displayed stronger theta phase precession than NFG-cells, which could be a major reason why FG-cells impacted theta sequences more than NFG cells. Although factors other than theta phase precession may contribute to or interfere with theta sequences, stronger theta phase precession itself (without the interference of other factors), by definition, can lead to stronger theta sequences.

      This is a very good point.  The finding that FG-cells displayed stronger theta phase precession than NFG-cells was consistent with the finding of Guardamagna et al., 2023 Cell Rep, that the theta phase precession pattern emerged with strong fast gamma.  Since slow gamma phase precession occurred within theta cycles, it is hard to consider the contribution of these factors to theta sequences development, without taking theta phase precession into account.  But one should be noted that the theta sequences could not be developed even if theta phase precession existed from the very beginning of the exploration (Feng et al., 2025 J Neurosci).  These findings suggest that theta phase precession, together with other factors, impact theta sequence development.  However, the weight of each factor and their interaction still need to be further investigated.  We have discussed this possibility in the Discussion section (Lines 361- 373).

      (4) The slow-gamma phase precession of FG-cells during early laps is supposed to mediate or contribute to the emergence of theta sequences during late laps (Figure 1). The logic of this model is unclear. The slow-gamma phase precession was present in both early and late laps for FG-cells, but only present in late laps for NFG-cells. It seems more straightforward to hypothesize that the difference in theta sequences between early and later laps is due to the difference in slow-gamma phase precession of NFG cells between early and late laps. Although this is not necessarily the case, the argument presented in the manuscript is not easy to follow.

      We thank the reviewer for pointing this out.  The slow gamma phase precession was first found in my previous publication (Zheng et al., 2016 Neuron), which indicates a temporally compressed manner for coding spatial information related to memory retrieval.  In this case, we would expect that slow gamma phase precession occurred in all cells during late laps, because spatial information was retrieved when rats have been familiar with the environment.  However, during early laps when novel information was just encoded, there would be balance between fast gamma and slow gamma modulation of cells for upcoming encoding-retrieval transition.  A possibility is that FG-cells support this balance by receiving modulation of both fast gamma and slow gamma, but with distinct phase-coding modes (fast gamma phase locking and slow gamma phase precession) in a temporally coordinated manner.  We have discussed this possibility in the Discussion section (Lines 415- 428).

      (5) There are several questions on the description of methods, which could be addressed to clarify or strengthen the conclusions.

      (i) Were the identified fast- and slow-gamma episodes mutually exclusive?

      Yes, the fast- and slow-gamma episodes are mutually exclusive. We have added descriptions in the “Detection of gamma episodes” section in the Methods part (Lines 538-550).

      (ii) Was the task novel when the data were acquired? How many days (from the 1st day of the task) were included in the analysis? When the development of the theta sequence was mentioned, did it mean the development in a novel environment, in a novel task, or purely in a sense of early laps (Lap 1, 2) on each day?

      We thank the reviewer for pointing this out.  The task was not novel to rats in this dataset, because only days with good enough recording quality for sequence decoding were included in this paper, which were about day2-day10 for each rat.  However, we still observed the process of sequence formation because of the rat’s exploration interest during early laps.  Thus, when the development of the theta sequence was mentioned, it meant a sense of early laps on each day.

      (iii) How were the animals' behavioral parameters equalized between early and later laps? For example, speed or head direction could potentially produce the differences in theta sequences.

      This is a very good point.  In terms of the effect of running speed on theta sequences, we quantified the running speeds during theta sequences across trials 1-5.  We found that the rats were running at stable running speed, which has been reported in Fig.3F.  In terms of the effect of head direction on theta sequences, we measured the angle difference between head direction and running direction.  We found that the angle difference for each lap was distributed around 0, with no significant difference across laps (Fig.S3, Watson-Williams multi-sample test, F(4,55)=0.2, p=0.9, partial η<sup>2</sup>= 0.01).  These results indicate that the differences in theta sequences across trials cannot be interpreted by the variability of behavioral parameters.  We have updated these results and corresponding methods in the revised manuscript (Lines 172-175, Lines 507-511, with a new Fig.S3).

      Reviewer #2 (Public Review):

      This manuscript addresses an important question that has not yet been solved in the field, what is the contribution of different gamma oscillatory inputs to the development of "theta sequences" in the hippocampal CA1 region? Theta sequences have received much attention due to their proposed roles in encoding short-term behavioral predictions, mediating synaptic plasticity, and guiding flexible decision-making. Gamma oscillations in CA1 offer a readout of different inputs to this region and have been proposed to synchronize neuronal assemblies and modulate spike timing and temporal coding. However, the interactions between these two important phenomena have not been sufficiently investigated. The authors conducted place cell and local field potential (LFP) recordings in the CA1 region of rats running on a circular track. They then analyzed the phase locking of place cell spikes to slow and fast gamma rhythms, the evolution of theta sequences during behavior, and the interaction between these two phenomena. They found that place cells with the strongest modulation by fast gamma oscillations were the most important contributors to the early development of theta sequences and that they also displayed a faster form of phase precession within slow gamma cycles nested with theta. The results reported are interesting and support the main conclusions of the authors. However, the manuscript needs significant improvement in several aspects regarding data analysis, description of both experimental and analytical methods, and alternative interpretations, as I detail below.

      • The experimental paradigm and recordings should be explained at the beginning of the Results section. Right now, there is no description whatsoever which makes it harder to understand the design of the study.

      We thank the reviewer for this kind suggestion.  The description of experimental paradigm and recordings has been added to the beginning of the results section (Lines 114-119).

      • An important issue that needs to be addressed is the very small fraction of CA1 cells phased-locked to slow gamma rhythms (3.7%). This fraction is much lower than in many previous studies, that typically report it in the range of 20-50%. However, this discrepancy is not discussed by the authors. This needs to be explained and additional analysis considered. One analysis that I would suggest, although there are also other valid approaches, is to, instead of just analyzing the phase locking in two discrete frequency bands, compute the phase locking will all LFP frequencies from 25-100 Hz. This will offer a more comprehensive and unbiased view of the gamma modulation of place cell firing. Alternative metrics to mean vector length that is less sensitive to firing rates, such as pairwise phase consistency index (Vinck et a., Neuroimage, 2010), could be implemented. This may reveal whether the low fraction of phase-locked cells could be due to a low number of spikes entering the analysis.

      We thank the reviewer for this constructive suggestion.  A previous work also on Long-Evans rats showed that the proportion of slow gamma phase-locked cells during novelty exploration was ~20%, however it dropped to ~10% during familiar exploration (Fig.4E, Kitanishi et al., 2015 Neuron).  This suggests that the proportion of slow gamma phase-locked cells may decreased with familiarity of the environment, which supports our data.  In addition, we also calculated the pairwise phase consistency index in terms of the effect of spike counts on MVL.  We could observe that the tendency of PPC (Author response image 2A) and MVL (Author response image 2B) along frequency bands were consistent across different subsets of cells, suggesting that the determination of cell subsets by MVL metric was not biased by the low number of spikes.  These results further shed light to the contribution of slow gamma phase precession of place cells to theta sequence development.

      Author response image 2.

      • From the methods, it is not clear to me whether the reference LFP channel was consistently selected to be a different one that where the spikes analyzed were taken. This is the better practice to reduce the contribution of spike leakage that could substantially inflate the coupling with faster gamma frequencies. These analyses need to be described in more detail.

      We thank the reviewer for pointing this out.  In the main manuscript, we used local LFPs as the cells were recorded from the same tetrode.  In addition, we selected an individual tetrode which located at stratum pyramidale and at the center of the drive bundle for each rat.  We detected a similar proportion of FG-cells by using LFPs on this tetrode, compared with that using local LFPs (Author response image 3A-B, Chi-squared test, χ<sup>2</sup>= 0.9, p=0.4, Cramer V=0.03).  We further found that the PPC measurement of FG- and NFG-cells were different at fast gamma band by using central LFPs (Author response image 3D), consistent with that by using local LFPs (Author response image 3C).  Therefore, these results suggest that the findings related to fast gamma was not due to the contribution of spike leakage in the local LFPs.  We have updated the description in the manuscript (Lines 553-557, 566-568).

      Author response image 3.

      • The initial framework of the authors of classifying cells into fast gamma and not fast gamma modulated implies a bimodality that may be artificial. The authors should discuss the nuances and limitations of this framework. For example, several previous work has shown that the same place cell can couple to different gamma oscillations (e.g., Lastoczni et al., Neuron, 2016; Fernandez-Ruiz et al., Neuron, 2017; Sharif et al., Neuron,2021).

      We thank the reviewer for this kind suggestion.  We have cited these references and discussed the possibility of bimodal phase-locking in the manuscript (Lines 430-433).

      • It would be useful to provide a more thorough characterization of the physiological properties of FG and NFG cells, as this distinction is the basis of the paper. Only very little characterization of some place cell properties is provided in Figure 5. Important characteristics that should be very feasible to compare include average firing rate, burstiness, estimated location within the layer (i.e., deep vs superficial sublayers) and along the transverse axis (i.e., proximal vs distal), theta oscillation frequency, phase precession metrics (given their fundamental relationship with theta sequences), etc.

      We thank the reviewer for this constructive suggestion.  In addition to the characterizations shown in Fig5, we also analyzed firing rate, anatomical location and theta modulation to compare the physiological properties of FG- and NFG-cells.

      In terms of the firing properties of both types of cells, we found that the mean firing rate of FG-cell was higher than NFG-cell (Fig. 5A, Student's t-test, t(22) = 2.1, p = 0.04, Cohen's D = 0.9), which was consistent with the previous study that the firing rate was higher during fast gamma than during slow gamma (Zheng et al., 2015 Hippocampus).  However, the spike counts of excluded FG- and NFG-cells for decoding were similar (Fig. 5B, Student's t-test, t(22) = 1.2, p = 0.3, Cohen's D = 0.5), suggesting that the differences found in theta sequences cannot be accounted for by different decoding quality related to spike counts.  In addition, we measured the burstiness based on the distribution of inter-spike-intervals, and we found that the bursting probability of spikes was not significantly different between FG and NFG cells (Author response image 4A, Student's t-test, t(22) = 0.6, p=0.5, Cohen's d=0.3).

      In terms of theta modulation of cells, we first compared the theta frequency related to the firing of FG and NFG cells.  We detected the instantaneous theta frequency at each spike timing of FG and NFG cells, and found that it was not significantly different between cell types (Author response image 4B, Student's t-test, t(22) = -0.5, p=0.6, Cohen's d=0.2).  In addition, we found the proportion of cells with significant theta phase precession was greater in FG-cells than in NFG-cells (Fig. S2E).  However, the slope and starting phase of theta phase precession was not significantly different between FG and NFG cells (Author response image 4C, Student's t-test, t(21) = 0.3, p=0.8, Cohen's d=0.1; Author response image 4D, Watson-Williams test, F(1,21)=0.5, p=0.5, partial η<sup>2</sup>=0.02).

      In terms of the anatomical location of FG and NFG cells, we identified tetrode traces in slices for each cell.  We found that both FG and NFG cells were recorded from the deep layer of dorsal CA1, with no difference of proportions between cell types (Author response image 4E, Chi-squared test, χ<sup>2</sup>=0.5, p=0.5, Cramer V=0.05).  The distribution of FG-cells he NFG-cells along the transverse axis was also similar between cell types (Author response image 4F, χ<sup>2</sup>=0.08, p=0.8, Cramer V=0.02).

      Author response image 4.

      • It is not clear to me how the analysis in Figure 6 was performed. In Figure 6B I would think that the grey line should connect with the bottom white dot in the third panel, which would be the interpretation of the results.

      We thank the reviewer for raising this good point.  The grey line was just for intuitional observation, not a quantitative analysis.  We have removed the grey lines from all heat maps in Fig.6.

      Reviewer #3 (Public Review):

      [Editors' note: This review contains many criticisms that apply to the whole sub-field of slow/fast gamma oscillations in the hippocampus, as opposed to this particular paper. In the editors' view, these comments are beyond the scope of any single paper. However, they represent a view that, if true, should contextualise the interpretation of this paper and all papers in the sub-field. In doing so, they highlight an ongoing debate within the broader field.]

      Summary:

      The authors aimed to elucidate the role of dynamic gamma modulation in the development of hippocampal theta sequences, utilizing the traditional framework of "two gammas," a slow and a fast rhythm. This framework is currently being challenged, necessitating further analyses to establish and secure the assumed premises before substantiating the claims made in the present article.

      The results are too preliminary and need to integrate contemporary literature. New analyses are required to address these concerns. However, by addressing these issues, it may be possible to produce an impactful manuscript.

      We thank the reviewer for raising these important questions in the hippocampal gamma field.  We have done a lot of new analyses according to the comments to strengthen our manuscript.

      I. Introduction

      Within the introduction, multiple broad assertions are conveyed that serve as the premise for the research. However, equally important citations that are not mentioned potentially contradict the ideas that serve as the foundation. Instances of these are described below:

      (1) Are there multiple gammas? The authors launched the study on the premise that two different gamma bands are communicated from CA3 and the entorhinal cortex. However, recent literature suggests otherwise, offering that the slow gamma component may be related to theta harmonics:

      From a review by Etter, Carmichael and Williams (2023)

      "Gamma-based coherence has been a prominent model for communication across the hippocampal-entorhinal circuit and has classically focused on slow and fast gamma oscillations originating in CA3 and medial entorhinal cortex, respectively. These two distinct gammas are then hypothesized to be integrated into hippocampal CA1 with theta oscillations on a cycle-to-cycle basis (Colgin et al., 2009; Schomburg et al., 2014). This would suggest that theta oscillations in CA1 could serve to partition temporal windows that enable the integration of inputs from these upstream regions using alternating gamma waves (Vinck et al., 2023). However, these models have largely been based on correlations between shifting CA3 and medial entorhinal cortex to CA1 coherence in theta and gamma bands. In vivo, excitatory inputs from the entorhinal cortex to the dentate gyrus are most coherent in the theta band, while gamma oscillations would be generated locally from presumed local inhibitory inputs (Pernía-Andrade and Jonas, 2014). This predominance of theta over gamma coherence has also been reported between hippocampal CA1 and the medial entorhinal cortex (Zhou et al., 2022). Another potential pitfall in the communication-through-coherence hypothesis is that theta oscillations harmonics could overlap with higher frequency bands (Czurkó et al., 1999; Terrazas et al., 2005), including slow gamma (Petersen and Buzsáki, 2020). The asymmetry of theta oscillations (Belluscio et al., 2012) can lead to harmonics that extend into the slow gamma range (Scheffer-Teixeira and Tort, 2016), which may lead to a misattribution as to the origin of slow-gamma coherence and the degree of spike modulation in the gamma range during movement (Zhou et al., 2019)."

      And from Benjamin Griffiths and Ole Jensen (2023)

      "That said, in both rodent and human studies, measurements of 'slow' gamma oscillations may be susceptible to distortion by theta harmonics [53], meaning open questions remain about what can be attributed to 'slow' gamma oscillations and what is attributable to theta."

      This second statement should be heavily considered as it is from one of the original authors who reported the existence of slow gamma.

      Yet another instance from Schomburg, Fernández-Ruiz, Mizuseki, Berényi, Anastassiou, Christof Koch, and Buzsáki (2014):

      "Note that modulation from 20-30 Hz may not be related to gamma activity but, instead, reflect timing relationships with non-sinusoidal features of theta waves (Belluscio et al., 2012) and/or the 3rd theta harmonic."

      One of this manuscript's authors is Fernández-Ruiz, a contemporary proponent of the multiple gamma theory. Thus, the modulation to slow gamma offered in the present manuscript may actually be related to theta harmonics.

      With the above emphasis from proponents of the slow/fast gamma theory on disambiguating harmonics from slow gamma, our first suggestion to the authors is that they A) address these statements (citing the work of these authors in their manuscript) and B) demonstrably quantify theta harmonics in relation to slow gamma prior to making assertions of phase relationships (methodological suggestions below). As the frequency of theta harmonics can extend as high as 56 Hz (PMID: 32297752), overlapping with the slow gamma range defined here (25-45 Hz), it will be important to establish an approach that decouples the two phenomena using an approach other than an arbitrary frequency boundary.

      We agree with the reviewer that the theta oscillations harmonics could overlap with higher frequency bands including slow gamma, as the above reviews discussed.  In order to rule out the possibility of theta harmonics effects in this study, we added new analyses in this letter (see below).

      (2) Can gammas be segregated into different lamina of the hippocampus? This idea appears to be foundational in the premise of the research but is also undergoing revision.

      As discussed by Etter et al. above, the initial theory of gamma routing was launched on coherence values. However, the values reported by Colgin et al. (2009) lean more towards incoherence (a value of 0) rather than coherence (1), suggesting a weak to negligible interaction. Nevertheless, this theory is coupled with the idea that the different gamma frequencies are exclusive to the specific lamina of the hippocampus.

      Recently, Deschamps et al. (2024) suggested a broader, more nuanced understanding of gamma oscillations than previously thought, emphasizing their wide range and variability across hippocampal layers. This perspective challenges the traditional dichotomy of gamma sub-bands (e.g., slow vs. medium gamma) and their associated cognitive functions based on a more rigid classification according to frequency and phase relative to the theta rhythm. Moreover, they observed all frequencies across all layers.

      Similarly, the current source density plots from Belluscio et al. (2012) suggest that SG and FG can be observed in both the radiatum and lacunosum-moleculare.

      Therefore, if the initial coherence values are weak to negligible and both slow and fast gamma are observed in all layers of the hippocampus, can the different gammas be exclusively related to either anatomical inputs or psychological functions (as done in the present manuscript)? Do these observations challenge the authors' premise of their research? At the least, please discuss.

      We thank the reviewer for raising this point, which I believe still remains controversial in this field.  We also thank the reviewer for providing detailed proofs of existence forms of gamma rhythms.  The reviewer was considering 2 aspects of gamma: 1) the reasonability of dividing slow and fast gamma by specific frequency bands; 2) the existence of gamma across all hippocampal layers, which challenged the functional significance of different types of gamma rhythms.  Although the results in Douchamps et al., 2024 challenged the idea of rigid gamma sub-bands, we still could see separate slow and fast gamma components exclusively occurred along time course, with central frequency of slow gamma lower than ~60Hz and central frequency of fast gamma higher than ~60Hz (Fig.1b of Douchamps et al., 2024).  This was also seen in the rat dataset of this reference (Fig. S3).  Since their behavioral test required both memory encoding and retrieval processes, it was hard to distinguish the role of different gamma components as they may dynamically coordinate during complex memory process.  Thus, although the behavioral performance can be decoded from broad range of gamma, we still cannot deny the existence of difference gamma rhythms and their functional significance during difference memory phases.

      (3) Do place cells, phase precession, and theta sequences require input from afferent regions? It is offered in the introduction that "Fast gamma (~65-100Hz), associated with the input from the medial entorhinal cortex, is thought to rapidly encode ongoing novel information in the context (Fernandez-Ruiz et al., 2021; Kemere, Carr, Karlsson, & Frank, 2013; Zheng et al., 2016)".

      CA1 place fields remain fairly intact following MEC inactivation include Ipshita Zutshi, Manuel Valero, Antonio Fernández-Ruiz , and György Buzsáki (2022)- "CA1 place cells and assemblies persist despite combined mEC and CA3 silencing" and from Hadas E Sloin, Lidor Spivak, Amir Levi, Roni Gattegno, Shirly Someck, Eran Stark (2024) - "These findings are incompatible with precession models based on inheritance, dual-input, spreading activation, inhibition-excitation summation, or somato-dendritic competition. Thus, a precession generator resides locally within CA1."

      These publications, at the least, challenge the inheritance model by which the afferent input controls CA1 place field spike timing. The research premise offered by the authors is couched in the logic of inheritance, when the effect that the authors are observing could be governed by local intrinsic activity (e.g., phase precession and gamma are locally generated, and the attribution to routed input is perhaps erroneous). Certainly, it is worth discussing these manuscripts in the context of the present manuscript.

      We thank the review for this discussion.  The main purpose of our current study is to investigate the mechanism of theta sequence development along with learning, which may or may not dependent on theta phase precession of single place cells as it remains controversial in this field.  Also, there is a limitation in this study that all gamma components were recorded from stratum pyramidale, thus we cannot make any conclusion on the originate of gamma in modulating sequence development.

      II. Results

      (1) Figure 2-

      a. There is a bit of a puzzle here that should be discussed. If slow and fast frequencies modulate 25% of neurons, how can these rhythms serve as mechanisms of communication/support psychological functions? For instance, if fast gamma is engaged in rapid encoding (line 72) and slow gamma is related to the integration processing of learned information (line 84), and these are functions of the hippocampus, then why do these rhythms modulate so few cells? Is this to say 75% of CA1 neurons do not listen to CA3 or MEC input?

      The proportion ~25% was the part of place cells phase-locked to either slow or fast gamma.  However, one of the main findings in this study was that most cells were modulated by slow gamma as they fired at precessed slow gamma phase within a theta cycle (Figs 6-8), which would promote information compression for theta sequence development.  Therefore, we didn’t mean that only a small proportion of cells were modulated by gamma rhythms and contributed to this process.

      b. Figure 2. It is hard to know if the mean vector lengths presented are large or small. Moreover, one can expect to find significance due to chance. For instance, it is challenging to find a frequency in which modulation strength is zero (please see Figure 4 of PMID: 30428340 or Figure 7 of PMID: 31324673).

      i. Please construct the histograms of Mean Vector Length as in the above papers, using 1 Hz filter steps from 1-120Hz and include it as part of Figure 2 (i.e., calculate the mean vector length for the filtered LFP in steps of 1-2 Hz, 2-3 Hz, 3-4 Hz,... etc). This should help the authors portray the amount of modulation these neurons have relative to the theta rhythm and other frequencies. If the theta mean vector length is higher, should it be considered the primary modulatory influence of these neurons (with slow and fast gammas as a minor influence)?

      We thank the review for this suggestion.  We measured the mean vector length at 5Hz step (equivalent to 1Hz step), and we found that the FG-cells were phase-locked to fast gamma rhythms even stronger than that to theta (Author response image 2B, mean MVL of theta=0.126±0.007, mean MVL of theta=0.175±0.006, paired t-test, t(112)=-5.9, p=0.01, Cohen's d=0.7).  In addition, in some previous studies with significant fast gamma phase locking, the MVL values were around 0.15 by using broad gamma band (Kitanishi et al., 2015 Neuron, Lasztóczi et al., 2016 Neuron, Tomar et al., 2021 Front Behav Neurosci, and Asiminas et al., 2022 Molecular Autism), which was consistent with the value in this study.  Therefore, we don’t believe that fast gamma was only a minor influence of these neurons.

      ii. It is possible to infer a neuron's degree of oscillatory modulation without using the LFP. For instance, one can create an ISI histogram as done in Figure 1 here (https://www.biorxiv.org/content/10.1101/2021.09.20.461152v3.full.pdf+html; "Distinct ground state and activated state modes of firing in forebrain neurons"). The reciprocal of the ISI values would be "instantaneous spike frequency". In favor of the Douchamps et al. (2024) results, the figure of the BioRXiV paper implies that there is a single gamma frequency modulate as there is only a single bump in the ISIs in the 10^-1.5 to 10^-2 range. Therefore, to vet the slow gamma results and the premise of two gammas offered in the introduction, it would be worth including this analysis as part of Figure 2.

      By using suggested method, we calculated the ISI distribution on log scale for FG-cells and NFG-cells during behavior (Author response image 5).  We could observe that the ISI distribution of FG-cells had a bump in the 10<sup>-1.5</sup>= to 10<sup>-2</sup>= range (black bar), in particular in the fast gamma range (10<sup>-2</sup>= to 10<sup>-1.8</sup>=).

      Author response image 5.

      c. There are some things generally concerning about Figure 2.

      i. First, the raw trace does not seem to have clear theta epochs (it is challenging to ascertain the start and end of a theta cycle). Certainly, it would be worth highlighting the relationship between theta and the gammas and picking a nice theta epoch.

      We thank the review for this suggestion.  We've updated this figure with a nice theta epoch in the revised manuscript.

      ii. Also, in panel A, there looks to be a declining amplitude relationship between the raw, fast, and slow gamma traces, assuming that the scale bars represent 100uV in all three traces. The raw trace is significantly larger than the fast gamma. However, this relationship does not seem to be the case in panel B (in which both the raw and unfiltered examples of slow and fast gamma appear to be equal; the right panels of B suggest that fast gamma is larger than slow, appearing to contradict the A= 1/f organization of the power spectral density). Please explain as to why this occurs. Including the power spectral density (see below) should resolve some of this.

      We thank the review for pointing this out.  The scales of y-axis of LFPs tracs in Fig.2B was not consistent, which mislead the comparison of amplitude between slow and fast gamma.  We have unified y axis scales across different gamma types in the revised manuscript.  Moreover, we also have replaced these examples with more typical ones (also see the response below).

      iii. Within the example of spiking to phase in the left side of Panel B (fast gamma example)- the neuron appears to fire near the trough twice, near the peak twice, and somewhere in between once. A similar relationship is observed for the slow gamma epoch. One would conclude from these plots that the interaction of the neuron with the two rhythms is the same. However, the mean vector lengths and histograms below these plots suggest a different story in which the neuron is modulated by FG but not SG. Please reconcile this.

      We thank the review for pointing this out.  We found that the fast gamma phase locking was robust across FG-cells with fast gamma peak as the preferred phase.  Therefore, we have replaced these examples with more typical ones, so that the examples were consistent with the group effect.

      iv. For calculating the MVL, it seems that the number of spikes that the neuron fires would play a significant role. Working towards our next point, there may be a bias of finding a relationship if there are too few spikes (spurious clustering due to sparse data) and/or higher coupling values for higher firing rate cells (cells with higher firing rates will clearly show a relationship), forming a sort of inverse Yerkes-Dodson curve. Also, without understanding the magnitude of the MVL relative to other frequencies, it may be that these values are indeed larger than zero, but not biologically significant.

      - Please provide a scatter plot of Neuron MVL versus the Neuron's Firing Rate for 1) theta (7-9 Hz), 2) slow gamma, and 3) fast gamma, along with their line of best fit.

      - Please run a shuffle control where the LFP trace is shifted by random values between 125-1000ms and recalculate the MVL for theta, slow, and fast gamma. Often, these shuffle controls are done between 100-1000 times (see cross-correlation analyses of Fujisawa, Buzsaki et al.).

      - To establish that firing rate does not play a role in uncovering modulation, it would be worth conducting a spike number control, reducing the number of spikes per cell so that they are all equal before calculating the phase plots/MVL.

      We thank the review for raising this point.  Beside of the MVL value, we also calculated the pairwise phase consistency (PPC) as suggested by Reviewer2, which is not sensitive to the spike counts.  We found that the phase locking strength to either rhythm (theta or gamma) was comparable between MVL and PPC measurements (Author response image 2).  Moreover, we quantified the relationship between MVL and mean firing rate, as suggested.  We found that the MVL value for theta, slow gamma and fast gamma was negatively correlated with mean firing rate (Author response image 6, Pearson correlation, theta: R<sup>2</sup>= 0.06, Pearson’s r=-0.3, p=1.3×10<sup>-8</sup>=; slow gamma: R<sup>2</sup>= 0.1, Pearson’s r=-0.4, p=2.4×10<sup>-17</sup>=; fast gamma: R<sup>2</sup>= 0.03, Pearson’s r=-0.2, p=4.3×10<sup>-5</sup>=).  These results help us rule out the concerns of the effect of spikes counts on the phase modulation measurement.

      Author response image 6.

      (2) Something that I anticipated to see addressed in the manuscript was the study from Grosmark and Buzsaki (2016): "Cell assembly sequences during learning are "replayed" during hippocampal ripples and contribute to the consolidation of episodic memories. However, neuronal sequences may also reflect preexisting dynamics. We report that sequences of place-cell firing in a novel environment are formed from a combination of the contributions of a rigid, predominantly fast-firing subset of pyramidal neurons with low spatial specificity and limited change across sleep-experience-sleep and a slow-firing plastic subset. Slow-firing cells, rather than fast-firing cells, gained high place specificity during exploration, elevated their association with ripples, and showed increased bursting and temporal coactivation during postexperience sleep. Thus, slow- and fast-firing neurons, although forming a continuous distribution, have different coding and plastic properties."

      My concern is that much of the reported results in the present manuscript appear to recapitulate the observations of Grosmark and Buzsaki, but without accounting for differences in firing rate. A parsimonious alternative explanation for what is observed in the present manuscript is that high firing rate neurons, more integrated into the local network and orchestrating local gamma activity (PING), exhibit more coupling to theta and gamma. In this alternative perspective, it's not something special about how the neurons are entrained to the routed fast gamma, but that the higher firing rate neurons are better able to engage and entrain their local interneurons and, thus modulate local gamma. However, this interpretation challenges the discussion around the importance of fast gamma routed from the MEC.

      a. Please integrate the Grosmark & Buzsaki paper into the discussion.

      b. Also, please provide data that refutes or supports the alternative hypothesis in which the high firing rate cells are just more gamma modulated as they orchestrate local gamma activity through monosynaptic connections with local interneurons (e.g., Marshall et al., 2002, Hippocampal pyramidal cell-interneuron spike transmission is frequency dependent and responsible for place modulation of interneuron discharge). Otherwise, the attribution to a MEC routed fast gamma routing seems tenuous.

      c. It is mentioned that fast-spiking interneurons were removed from the analysis. It would be worth including these cells, calculating the MVL in 1 Hz increments as well as the reciprocal of their ISIs (described above).

      We thank the review for this suggestion.  Because we found the mean firing rate of FG-cells was higher than that of NFG-cells, it would be possible that the FG-cells are mainly overlapped with fast-firing cells (rigid cells) in Grosmark et al., 2016 Science.  Actually, in this study, we aimed to investigate how fast and slow gamma rhythms modulated neurons dynamically during learning, rather than defining new cell types.  Thus, we don’t think this work was just a replication of the previous publication.  We have added this description in the Discussion part (Lines 439-441).  In addition, we don’t have enough number of interneurons to support the analysis between interneurons and place cells.  Therefore, we couldn’t make any statement about where was the fast gamma originated (CA1 locally or routed from MEC) in this study.

      (3) Methods - Spectral decomposition and Theta Harmonics.

      a. It is challenging to interpret the exact parameters that the authors used for their multi-taper analysis in the methods (lines 516-526). Tallon-Baudry et al., (1997; Oscillatory γ-Band (30-70 Hz) Activity Induced by a Visual Search Task in Humans) discuss a time-frequency trade-off where frequency resolution changes with different temporal windows of analysis. This trade-off between time and frequency resolution is well known as the uncertainty principle of signal analysis, transcending all decomposition methods. It is not only a function of wavelet or FFT, and multi-tapers do not directly address this. (The multitaper method, by using multiple specially designed tapers -like the Slepian sequences- smooths the spectrum. This smoothing doesn't eliminate leakage but distributes its impact across multiple estimates). Given the brevity of methods and the issues of theta harmonics as offered above, it is worth including some benchmark trace testing for the multi-taper as part of the supplemental figures.

      i. Please spectrally decompose an asymmetric 8 Hz sawtooth wave showing the trace and the related power spectral density using the multiple taper method discussed in the methods.

      ii. Please also do the same for an elliptical oscillation (perfectly symmetrical waves, but also capable of casting harmonics). Matlab code on how to generate this time series is provided below:

      A = 1; % Amplitude

      T = 1/8; % Period corresponding to 8 Hz frequency

      omega = 2*pi/T; % Angular frequency

      C = 1; % Wave speed

      m = 0.9; % Modulus for the elliptic function (0<m<1 for cnoidal waves)

      x = linspace(0, 2*pi, 1000); % temporal domain

      t = 0; % Time instant

      % Calculate B based on frequency and speed

      B = sqrt(omega/C);

      % Cnoidal wave equation using the Jacobi elliptic function

      u = A .* ellipj(B.*(x - C*t), m).^2;

      % Plotting the cnoidal wave

      figure;

      plot(x./max(x), u);

      title('8 Hz Cnoidal Wave');

      xlabel('time (x)');

      ylabel('Wave amplitude (u)');

      grid on;

      The Symbolic Math Toolbox needs to be installed and accessible in your MATLAB environment to use ellipj. Otherwise, I trust that, rather than plotting a periodic orbit around a circle (sin wave) the authors can trace the movement around an ellipse with significant eccentricity (the distance between the two foci should be twice the distance between the co-vertices).

      We thank the review for this suggestion.  In the main text of manuscript, we only applied Morlet's wavelet method to calculate the time varying power of rhythms.  Multitaper method was used for the estimation of power spectra across running speeds, which was shown in the manuscript.  Therefore, we removed the description of Multitaper method and updated the Morlet's wavelet power spectral analysis in the Methods (Lines 541-544).

      As suggested, we estimated the power spectral densities of 8 Hz sawtooth and elliptical oscillation by using these methods, and compared them with the results from FFT.  We found that both the Multitaper's and Morlet's wavelet methods could well capture the 8Hz oscillatory components (Author response image 7).  However, we could observe harmonic components from FFT spectrum.

      Author response image 7.

      iii. Line 522: "The power spectra across running speeds and absolute power spectrum (both results were not shown).". Given the potential complications of multi-taper discussed above, and as each convolution further removes one from the raw data, it would be the most transparent, simple, and straightforward to provide power spectra using the simple fft.m code in Matlab (We imagine that the authors will agree that the results should be robust against different spectral decomposition methods. Otherwise, it is concerning that the results depend on the algorithm implemented and should be discussed. If gamma transience is a concern, the authors should trigger to 2-second epochs in which slow/fast gamma exceeds 3-7 std. dev. above the mean, comparing those resulting power spectra to 2-second epochs with ripples - also a transient event). The time series should be at least 2 seconds in length (to avoid spectral leakage issues and the issues discussed in Talon-Baudry et al., 1997 above).

      Please show the unmolested power spectra (Y-axis units in mV2/Hz, X-axis units as Hz) as a function of running speed (increments of 5 cm/s) for each animal. I imagine three of these PSDs for 3 of the animals will appear in supplemental methods while one will serve as a nice manuscript figure. With this plot, please highlight the regions that the authors are describing as theta, slow, and fast gamma. Also, any issues should be addressed should there be notable differences in power across animals or tetrodes (issues with locations along proximal-distal CA1 in terms of MEC/LEC input and using a local reference electrode are discussed below).

      As suggested, we firstly estimated the power spectra as a function of running speeds in each running lap, and showed them separately for each rat, by using the multitaper spectral analysis (Author response image 8).  In addition, to achieve unmolested power spectra, the short-time Fourier transform (STFT) was used for this analysis at the same frequency resolution (Author response image 9).  We could see that the power spectra were consistent between these two methods.  Notably, there seems no significant theta harmonic component in the slow gamma band range.

      The multitaper spectral analysis was performed as follows.  The power spectra were measured across different running speeds as described previously (Ahmed et al., 2012 J Neurosci; Zheng et al., 2015 Hippocampus; Zheng et al., 2016 eNeuro).  Briefly, the absolute power spectrum was calculated for 0.5s moving window and 0.2s step size of the LFPs recordings each lap, using the multitaper spectral analysis in the Chronux toolbox (Mitra and Bokil, 2008, http://chronux.org/) and STFT spectral analysis in Matlab script stft.m.  In the multitaper method, the time-bandwidth product parameter (TW) was set at 3, and the number of tapers (K) was set at 5.  In the STFT method, the FFT length was set at 2048, which was equivalent with the parameters used in multitaper method.  Running speed was calculated (see “Estimation of running speed and head direction” section in the manuscript) and averaged within each 0.5s time window corresponding to the LFP segments.  Then, the absolute power at each frequency was smoothed with a Gaussian kernel centered on given speed bin.  The power spectral as a function of running speed and frequency were plotted in log scale.  Also, the colormap was in log scale, allowing for comparisons across different frequencies that would otherwise be difficult due to the 1/f decay of power in physiological signals.

      Author response image 8.

      Author response image 9.

      iv. Schomberg and colleagues (2014) suggested that the modulation of neurons in the slow gamma range could be related to theta harmonics (see above). Harmonics can often extend in a near infinite as they regress into the 1/f background (contributing to power, but without a peak above the power spectral density slope), making arbitrary frequency limits inappropriate. Therefore, in order to support the analyses and assertions regarding slow gamma, it seems necessary to calculate a "theta harmonic/slow gamma ratio". Aru et al. (2015; Untangling cross-frequency coupling in neuroscience) offer that: " The presence of harmonics in the signal should be tested by a bicoherence analysis and its contribution to CFC should be discussed." Please test both the synthetic signals above and the raw LFP, using temporal windows of greater than 4 seconds (again, the large window optimizes for frequency resolution in the time-frequency trade-off) to calculate the bicoherence. As harmonics are integers of theta coupled to itself and slow gamma is also coupled to theta, a nice illustration and contribution to the field would be a method that uses the bispectrum to isolate and create a "slow gamma/harmonic" ratio.

      We thank the reviewer for providing the method regarding on the theta harmonics.  We firstly measured the theta harmonics on the synthesized signal by using the biphasic coherence method, and we could clearly observe the nonlinear coupling between theta rhythm and its harmonics (Author response image 10).

      Author response image 10.

      In addition, we also measured the bicoherence on raw traces during slow gamma episodes.  We did not see nonlinear coupling between slow gamma and theta bands in this real data (mean bicoherence=0.1±0.0002) compared with that in the synthesized signal (mean bicoherence=0.7 for elliptical waves and 0.5 for sawtooth waves), suggesting that the slow gamma detected in this study was not pure theta harmonic (Author response image 11C, F, I, in red boxes).  Therefore, we believe that the contribution of theta harmonic in slow gamma is not significant.

      Author response image 11.

      (4) I appreciate the inclusion of the histology for the 4 animals. Knerim and colleagues describe a difference in MEC projection along the proximal-distal axis of the CA1 region (https://www.ncbi.nlm.nih.gov/pmc/articles/PMC3866456/)- "There are also differences in their direct projections along the transverse axis of CA1, as the LEC innervates the region of CA1 closer to the subiculum (distal CA1), whereas the MEC innervates the region of CA1 closer to CA2 and CA3 (proximal CA1)" From the histology, it looks like some of the electrodes are in the part of CA1 that would be dominated by LEC input while a few are closer to where the MEC would project.

      a. How do the authors control for these differences in projections? Wouldn't this change whether or not fast gamma is observed in CA1?

      b. I am only aware of one manuscript that describes slow gamma in the LEC which appeared in contrast to fast gamma from the MEC (https://www.science.org/doi/10.1126/science.abf3119). One would surmise that the authors in the present manuscript would have varying levels of fast gamma in their CA1 recordings depending on the location of the electrodes in the Proximal-distal axis, to the extent that some of the more medial tetrodes may need to be excluded (as they should not have fast gamma, rather they should be exclusively dominated by slow gamma). Alternatively, the authors may find that there is equal fast gamma power across the entire proximal-distal axis. However, this would pose a significant challenge to the LEC/slow gamma and MEC/fast gamma routing story of Fernandez-Ruiz et al. and require reconciliation/discussion.

      c. Is there a difference in neuron modulation to these frequencies based on electrode location in CA1?

      We thank the reviewer for this concern, which was also raised by Reviewer2.  We aligned the physical location of LFP channels in the proximal-distal axis based on histology.  In our dataset, only 2 rats were recorded from both distal and proximal hippocampus, so we calculated the gamma power from both sites in these rats.  We found that slow power was higher from proximal tetrodes than that from distal tetrodes (Author response image 12, repeated measure ANOVA, F(1,7)=10.2, p=0.02, partial η <sup>2</sup>=0.8).  However, fast gamma power were similar between different recording sites (F(1,7)=0.008, p=0.9, partial η <sup>2</sup>=0.001).  These results are partially consistent with the LEC/slow gamma and MEC/fast gamma routing story of Fernandez-Ruiz’s work.  The main reason would be that all LFPs were recorded from tetrodes in stratum pyramidale, deep layer in particular (Author response image 4E), so that it was hard to precisely identify their distance to distal/proximal apical dendrites.

      Author response image 12.

      In terms of the anatomical location of FG and NFG cells, we identified tetrode traces in slices for each cell.  We found that both FG and NFG cells were recorded from the deep layer of dorsal CA1, with no difference of proportions between cell types (Author response image 4E, Chi-squared test, χ<sup>2</sup>=0.5, p=0.5, Cramer V=0.05).  The distribution of FG-cells he NFG-cells along the transverse axis was also similar between cell types (Author response image 4F, χ<sup>2</sup>=0.08, p=0.8, Cramer V=0.02).

      (5) Given a comment in the discussion (see below), it will be worth exploring changes in theta, theta harmonic, slow gamma, and fast gamma power with running speed as no changes were observed with theta sequences or lap number versus. Notably, Czurko et al., report an increase in theta and harmonic power with running speed (1999) while Ahmed and Mehta (2012) report a similar effect for gamma.

      a. Please determine if the oscillations change in power and frequency of the rhythms discussed above change with running speed using the same parameters applied in the present manuscript. The specific concern is that how the authors calculate running speed is not sensitive enough to evaluate changes.

      We thank the reviewer for this suggestion.  The description of running speed quantification has been updated in the Method (see “Estimation of running speed and head direction” section, Lines 501-511).  Overall, the sample frequency of running speed was25Hz which would be sensitive enough to evaluate the behavioral changes.

      By measuring the rhythmic power changing as a function of running speed (Author response image 8 and Author response image 9), we could observe that theta power was increased as running speed getting higher.  Consistent with the results in (Ahmed and Mehta, 2012) and our previous study (Zheng et al., 2015), the fast gamma power was increasing and slow gamma power was decreasing when running speed was getting high.

      In addition, we also estimated the rhythmic frequency as a function of running speed in the slow and fast episodes respectively.  We found that fast gamma frequency was increased with running speed (Author response image 13, linear regression, R<sup>2</sup>=0.4, corr=0.6, p=9.9×10<sup>-15</sup>), whereas slow gamma frequency was decreased with running speed (R<sup>2</sup>=0.2, corr=-0.4, p=8.8×10<sup>-6</sup>).  Although significant correlation was found between gamma frequency and running speed, consistent with the previous studies, the frequency change (~70-75Hz for fast gamma and ~30-28Hz for slow gamma) was not big enough to affect the sequence findings in this study.  In additiontheta frequency was maintained in either slow episodes (R<sup>2</sup>=0.02, corr=-0.1, p=0.1) or fast episodes (R<sup>2</sup>=0.004, corr=0.06, p=0.5), consistent with results in Fig.1G of Kropff et al., 2021 Neuron.

      Author response image 13.

      b. It is astounding that animals ran as fast as they did in what appears to be the first lap (Figure 3F), especially as rats' natural proclivity is thigmotaxis and inquisitive exploration in novel environments. Can the authors expand on why they believe their rats ran so quickly on the first lap in a novel environment and how to replicate this? Also, please include the individual values for each animal on the same plot.

      We thank the reviewer for pointing this out.  The task was not brand new to rats in this dataset, because only days with good enough recording quality for sequence decoding were included in this paper, which were about day2-day10 for each rat.  However, we still observed the process of sequence formation because of the rat’s exploration interest during early laps.  Thus, in terms exploration behaviors, the rats ran at relative high speeds across laps (Author response image 14, each gray line represents the running speed within an individual session).

      Author response image 14.

      c. Can the authors explain how the statistics on line 169 (F(4,44)) work? Specifically, it is challenging to determine how the degrees of freedom were calculated in this case and throughout if there were only 4 animals (reported in methods) over 5 laps (depicted in Figure 3F. Given line 439, it looks like trials and laps are used synonymously). Four animals over 5 laps should have a DOF of 16.

      This statistic result was performed with each session/day as a sample (n=12 sessions/days).  The statistics were generated by repeated measures ANOVA on 5 trials in 12 sessions, with a DOF of 44.

      (6) Throughout the manuscript, I am concerned about an inflation of statistical power. For example on line 162, F(2,4844). The large degrees of freedom indicate that the sample size was theta sequences or a number of cells. Since multiple observations were obtained from the same animal, the statistical assumption of independence is violated. Therefore, the stats need to be conducted using a nested model as described in Aarts et al. (2014; https://pubmed.ncbi.nlm.nih.gov/24671065/). A statistical consult may be warranted.

      We thank the reviewer for this suggestion.  We have replaced this statistic result by using generalized linear mixed model with ratID being a covariate.  These results have been updated in the revised manuscript (Lines 164-167).

      (7) It is stated that one tetrode served as a quiet recording reference. The "quiet" part is an assumption when often, theta and gamma can be volume conducted to the cortex (e.g., Sirota et al., 2008; This is often why laboratories that study hippocampal rhythms use the cerebellum for the differential recording electrode and not an electrode in the corpus callosum). Generally, high frequencies propagate as well as low frequencies in the extracellular milieu (https://www.eneuro.org/content/4/1/ENEURO.0291-16.2016). For transparency, the authors should include a limitation paragraph in their discussion that describes how their local tetrode reference may be inadvertently diminishing and/or distorting the signal that they are trying to isolate. Otherwise, it would be worth hearing an explanation as to how the author's approach avoids this issue.

      In terms of the locations of references, we had 2 screws above the cerebellum in the skull connected to the recording drive ground, and 1 tetrode in a quiet area of the cortex serving as the recording reference.  We agree that the theta and gamma can be volume conducted to the cortex which may affect the power of these rhythms in the stratum pyramidale.  However, we didn’t mean to measure or compare the absolute theta or gamma power in this study, as we only cared about the phase modulation of gamma to place cells.  Therefore, we believe the location of recording reference would not make significant effect on our conclusion.

      Apologetically, this review is already getting long. Moreover, I have substantial concerns that should be resolved prior to delving into the remainder of the analyses. e.g., the analyses related to Figure 3-5 assert that FG cells are important for sequences. However, the relationship to gamma may be secondary to either their relationship to theta or, based on the Grosmark and Buzsaki paper, it may just be a phenomenon coupled to the fast-firing cells (fast-firing cells showing higher gamma modulation due to a local PING dynamic). Moreover, the observation of slow gamma is being challenged as theta harmonics, even by the major proponents of the slow/fast gamma theory. Therefore, the report of slow gamma precession would come as an unsurprising extension should they be revealed to be theta harmonics (however, no control for harmonics was implemented; suggestions were made above). Following these amendments, I would be grateful for the opportunity to provide further feedback.

      III. Discussion.

      a. Line 330- it was offered that fast gamma encodes information while slow gamma integrates in the introduction. However, in a task such as circular track running (from the methods, it appears that there is no new information to be acquired within a trial), one would guess that after the first few laps, slow gamma would be the dominant rhythm. Therefore, one must wonder why there are so few neurons modulated by slow gamma (~3.7%).

      The proportion of ~3.7% was the part of place cells phase-locked to slow gamma.  However, we aimed to find that the slow gamma phase precession of place cells promoted the theta sequence development.  We would not expect the cells phase-locked to slow gamma if phase precession occurred.

      b. Line 375: The authors contend that: "...slow gamma, related to information compression, was also required to modulate fast gamma phase-locked cells during sequence development. We replicated the results of slow gamma phase precession at the ensemble level (Zheng et al., 2016), and furthermore observed it at late development, but not early development, of theta sequences." In relation to the idea that slow gamma may be coupled to - if not a distorted representation of - theta harmonics, it has been observed that there are changes in theta relative to novelty.

      i. A. Jeewajee, C. Lever, S. Burton, J. O'Keefe, and N. Burgess (2008) report a decrease in theta frequency in novel circumstances that disappears with increasing familiarity.

      ii. One could surmise that this change in frequency is associated with alterations in theta harmonics (observed here as slow gamma), challenging the author's interpretation.

      iii. Therefore, the authors have a compelling opportunity to replicate the results of Jeewajee et al., characterizing changes of theta along with the development of slow gamma precession, as the environment becomes familiar. It will become important to demonstrate, using bicoherence as offered by Aru et al., how slow gamma can be disambiguated from theta harmonics. Specifically, we anticipate that the authors will be able to quantify A) theta harmonics (the number, and their respective frequencies and amplitudes), B) the frequency and amplitude of slow gamma, and C) how they can be quantitatively decoupled. Through this, their discussion of oscillatory changes with novelty-familiarity will garner a significant impact.

      We think we have demonstrated that the slow gamma observed in this study was not purely theta harmonics.  We didn’t focus on the frequency change of slow gamma or theta rhythms in this study.  Further investigation will be carried out on this topic in the future.

      c. Broadly, it is interesting that the authors emphasize the gamma frequency throughout the discussion. Given that the power spectral density of the Local Field Potential (LFP) exhibits a log-log relationship between amplitude and frequency, as described by Buzsáki (2005) in "Rhythms of the Brain," and considering that the LFP is primarily generated through synaptic transmembrane currents (Buzsáki et al., 2012), it seems parsimonious to consider that the bulk of synaptic activity occurs at lower frequencies (e.g., theta). Since synaptic transmission represents the most direct form of inter-regional communication, one might wonder why gamma (characterized by lower amplitude rhythms) is esteemed so highly compared to the higher amplitude theta rhythm. Why isn't the theta rhythm, instead, regarded as the primary mode of communication across brain regions? A discussion exploring this question would be beneficial.

      We thank the reviewer for this deep thinking.  When stating the conclusion on gamma rhythms, we didn’t mean to weaken the role of theta rhythm.  Conversely, the fast or slow gamma episodes were detected riding on theta rhythms, and we believe that the information compression should occur at a finer scale within a theta cycle scale.  More investigation will be carried out on this topic in the future.

      Recommendations for the authors:

      Reviewer #1 (Recommendations For The Authors):

      (1) It is helpful to clearly define "FG-cell sequences" before the relevant results are described in the Results section. More importantly, the seemingly conflicting results between Figure 3 and Figure 8 may need to be clarified.

      The “exFG-sequences and exNFG sequences”, “FG-cell sequences and NFG-cell sequences” have been defined clearly in the revised manuscript.  Moreover, the seemingly conflicting results between Figure 3 and Figure 8 have been interpreted properly.

      (2) It is helpful to clearly state the N and what defines a sample whenever a result is described.

      In each statistical results, the N and what defines a sample have been clarified in the revised manuscript.

      (3) Addressing the questions regarding the methods (#5) would clarify some of the results.

      The questions regarding the Methods part has addressed in the revised manuscript.

      (4) Line #244: "successful" should be "successive"?

      Fixed.

      Reviewer #2 (Recommendations For The Authors):

      - The writing of the manuscript can be substantially improved.

      The manuscript can be substantially revised and updated.

      - I noticed that the last author of the manuscript is not the lead or corresponding and has only provided a limited contribution to this work (according to the detailed author contributions). The second to last author seems to be the main senior intellectual contributor and supervisor, together with the third to last author. This speaks of potential bad academic practices where a senior person whose intellectual contribution to the study is relatively minor takes the last author position, against the standard conventions on authorship worldwide. I strongly suggest that this is corrected.

      We thank the reviewer for raising this problem.  The last author Dr. Ming was also a senior author and supervised this project with large contribution.  We have fixed his role as a co-corresponding author in the revised manuscript.

    1. Author response:

      The following is the authors’ response to the original reviews.

      Summary of revisions

      Title

      We have changed the title of the manuscript to “Chromatin endogenous cleavage provides a global view of yeast RNA polymerase II transcription kinetics”.

      Text

      Additional discussion of the patterns for elongation factors added (detailed below).

      Small text changes throughout, as mentioned in the detailed response below.

      Figures

      Updated legend-image in Figure 2F to reflect correct colors

      Added Figure 2 – supplement 1F – RNAPII enrichment with shorter promoter dwell times

      Added Figure 2 - supplement 2 with ChIP-seq outcomes (and text legend)

      Removed gene numbers in Figure 5C and put them in the legend.

      Substituted Med1 and Med8 ChEC over Rap1 sites in Figure 5F.

      Moved kin28-is growth inhibition to Figure 5 – Supplement 1.

      Substituted a new panel overlaying the RNAPII enrichment over UASs or promoters for all three strains in Figure 7D.

      Improved the labeling and legend of Figure 7E

      Methods

      Added ChIP-seq performed to confirm that the MNase fusion proteins are able to produce the expected pattern for ChIP.

      Point-by-point response to reviewers’ comments

      Reviewer 1:

      (1) Extending this work to elongation factors Ctk1 and Spt5 unexpectedly give strong signals near the PIC location and little signals over the coding region. This, and mapping CTD S2 and S5 phosphorylation by ChEC suggests to me that, for some reason, ChEC isn't optimal for detecting components of the elongation complex over coding regions. 

      (3) mapping the elongation factors Spt5 and Ctk1 by ChEC gives unexpected results as the signals over the coding sequences appear weak but unexpectedly strong at promoters and terminators. It would be helpful if the authors could comment on reasons why ChEC may not work well with elongation factors. For example, could this be something to do with the speed of Pol elongation and/or the chromatin structure of coding sequences such that coding sequence DNA is less accessible to MNase cleavage? 

      (7) The mintbodys are an interesting attempt to measure Pol II CTD modifications during elongation but give unexpected results as the signals in the coding region are lower than at promoters and terminators. It seems like ChIP is still a much better option for elongation factors unless I'm missing something. 

      We agree with the reviewer that this is a point that could confuse the reader.  Therefore, we have devoted two additional paragraphs to possible interpretations of our data in the Discussion:

      ChEC with factors involved in elongation (Ctk1, Spt5, Ser2p-RNAPII), when normalized to total RNAPII, showed greater enrichment over the CDS (Figure 3G), as expected. However, it is surprising that we also observed clear enrichment of these factors at promoters (e.g. Figure 3A, E & F). The association of elongation factors with the promoter seems to be biologically relevant. Changes in transcription correlate with changes in ChEC enrichment for these factors and modifications (Figure 4C). Blocking initiation by inhibiting TFIIH kinase led to a reduction of Ser5p RNAPII and Ser2p RNAPII over both the promoter and the transcribed region (Figure 5G). This suggests either that the true signal of these factors over transcribed regions is less evident by ChEC than by ChIP or that ChEC can reveal interactions of elongation factors at early stages of transcription that are missed by ChIP. The expectations for enrichment of elongation factors and phosphorylated CTD are largely based on ChIP data. Because ChIP fails to capture RNAPII enrichment at UASs and promoters, it is possible that ChIP also fails to capture promoter interaction of factors involved in elongation as well.

      Factors important for elongation can also function at the promoter. For example, Ctk1 is required for the dissociation of basal transcription factors from RNAPII at the promoter (Ahn et al., 2009). Transcriptional induction leads to increases in Ctk1 ChEC enrichment both over the promoter and over the 3’ end of the transcribed region (Figure 4C). Dynamics of Spt4/5 association with RNAPII from in vitro imaging (Rosen et al., 2020) indicate that the majority of Spt4/5 binding to RNAPII does not lead to elongation; Spt4/5 frequently dissociates from DNA-bound RNAPII. Association of Spt4/5 with RNAPII may represent a slow, inefficient step in the transition to productive elongation. If so, then ChEC-seq2 may capture transient Spt4/5 interactions that occur prior to productive elongation, producing enrichment of Spt5 at the promoter.

      (2) Finally, the role of nuclear pore binding by Gcn4 is explored, although the results do not seem convincing (10) In Figure 7, it's not convincing to me that ChEC is revealing the reason for the transcriptional defect in the Gcn4 PD mutant. The plots in panel D look nearly the same and I don't follow the authors' description of the differences stated in the text. In panel A, replotting the data in some other way might make the transcriptional differences between WT and Gcn4 PD mutants more obvious. 

      The phenotype of the gcn4-pd mutant is a quantitative decrease in transcription and this leads to a quantitative decrease, rather than qualitative loss, of RNA polymerase II over the promoter, without impacting the association of RNA polymerase II over the UAS region. This effect is small but statistically significant (p = 4e5). We have changed the title of this section of the manuscript to “ChEC-seq2 suggests a role for the NPC in stabilizing promoter association of RNAPII”. Also, to make comparison clearer, we have plotted the data together in the revised figure (Figure 7D).

      The magnitude of the decrease is not large, but we would highlight that is almost as large as that produced by inhibiting the Kin28 kinase (Figure 5H). Because the promoter-bound RNAPII is poorly captured by ChIP, this effect might be difficult to observe by techniques other than ChEC. Obviously, more mechanistic studies will need to be performed to fully understand this phenotype, but this result supports a role for the interaction with the nuclear pore complex in either enhancing the transfer of RNA polymerase II from the enhancer to the promoter or in preventing its dissociation from the promoter.

      I think that the related methods cut&run/cut&tag have been used to map elongating pol II. The authors should summarize what is known from this approach in the introduction and/or discussion. 

      CUT&RUN has been used to map RNAPII in mammals, but we are not aware of reports in S. cerevisiae.  Work from the Henikoff Lab in yeast mapped transcription factors and histone modifications (PMIDs 28079019 and 31232687).  A report using CUT&RUN in a human cell line reported a promoter-5’ bias of RNAPII that appeared to be dependent on fragment length (PMID 33070289). Regardless, the report highlights a key distinction between yeast and other eukaryotes: paused RNAPII. Indeed, paused RNAPII dominates ChIP-seq tracks in metazoans, and so we are hesitant to speculate between CUT&RUN in other species vs. ChEC-seq2 in S. cerevisiae

      Are the Rpb1, Rpb3, TFIIA, and TFIIE cleavage patterns expected based on the known structure of the PIC (Figures 2C, E)? 

      Rpb1 and 3 show peaks at approximately -17 and +34 with respect to TATA. TFIIA (Toa2) shows peaks at -12 and + 12.  And TFIIE (Tfa1) shows a peak around +34 (Figure 2C & E):

      As shown in the supplementary movie (based on the cMed-PIC structure; PDB #5OQM; Schilbach et al., 2017), upon binding to TBP/TFIID, TFIIA would be expected to cleave slightly upstream and downstream of the protected TATA (-12 and +12), while TFIIE binds downstream after the +12 site is protected and would be closest to the +34 unprotected site (to the right in the image below). RNAPII, which binds the fully assembled PIC, should be able to access either the upstream site (-12) or the downstream site (+34). Rpb1’s unstructured carboxy terminal domain, to which MNase is fused, would give it maximum flexibility, which likely explains why Rpb1 cleaves both at -12 and +34, with a preference for -12. Rpb3 also cleaves both sites, but without an obvious preference. 

      Author response image 1.

      Author response image 2.

      cleavage at -12, +12 and +34

      Author response image 3.

      Highlighted sites corresponding to the peaks in TFIIA assembled with TBP:

      Author response image 4.

      The complete PIC, protecting the +12 site, but leaving the +34 site exposed: 

      (6) Figure 2 S1: Pol II ChIP in the coding region gives a better correlation with transcription vs ChEC in promoters. Also, Pol II ChIP at terminators is almost as good as ChEC at promoters for estimating transcription. This latter point seems at odds with the text. The authors should comment on this and modify the text as needed. 

      Thank you for this comment.  We have clarified the text.

      In Figures 4 and 5, it's hard to tell how well changes in transcription correlate with changes in Pol II ChEC signals. It might be helpful to have a scatterplot or some other type of plot so that this relationship can be better evaluated. 

      While we find corresponding increase/decrease in ChEC-seq2 signal in genes identified as up/downregulated by SLAM-seq, the magnitude in change is not well correlated between the two techniques.  This was not surprising, because neither ChIP nor ChEC correlate especially well with SLAM-seq (Figure 2 – supplement 1E).

      In Figure 5, it's unclear why Pol association with Rap1 is being measured. Buratowski/Gelles showed that Pol associates with strong acidic activators - presumably through Mediator. Rap1 supposedly does not bind Mediator - so how is Pol associating here? Perhaps it would be better to measure Pol binding at STM genes that show Mediator-UAS binding. 

      Thank you; this is a good point.  We chose Rap1 because we had generated high-confidence binding sites in our strains under these conditions by ChEC-seq2. The results suggest that RNAPII is recruited well to these sites and that this recruitment does not require TFIIB. However, in disagreement with the notion that Mediator does not interact with Rap1, ChEC with Mediator subunits Med1 and Med8 also show peaks at these sites (new Figure 5F; the old Figure 5F is now Figure 5 – Supplement 1).  Therefore, either these sites are co-occupied by other transcription factors that mind Mediator, or Mediator is recruited by Rap1.  In either case, this correlates with binding of RNAPII. 

      Reviewer 2:

      (1) The term "nascent transcription" is all too often used interchangeably for NET-seq, PRO-seq, 4sUseq, and other assays that often provide different types of information. The authors should make it clear their use of the term refers to SLAM-seq data. 

      We have clarified throughout the manuscript that nascent transcription measured by SLAM-seq.

      The authors should explicitly state that experiments were performed in S. cerevisiae in the Results section. 

      We have made it clear in the title and the text that these experiments were performed in S. cerevisiae.

      Lines 216-218 state that "None of the 24 predicted the strong signal over the transcribed region with promoter depletion characteristic of ChIP-seq". I understand the authors' point, but there are parameter combinations that produce a flat profile with slightly less signal over the promoter (e.g., 5 sec dwell times and 3000 bp/ min elongation rate). If flanking windows were included, this profile would look something like ChIP-seq. I'd encourage the authors to be more precise with their language. 

      Thank you for highlighting this over-statement.

      We have now clarified the text and added another supplementary panel as follows:

      “While some combinations predicted a relatively flat distribution across the gene with lower levels in the promoter, none of the 24 predicted the strong signal over the transcribed region with promoter depletion characteristic of ChIP-seq. Only very short promoter dwell times (i.e., < 1s), produced the low promoter occupancy seen in ChIP-seq (Figure 2 – supplement 1F).”

    1. Author response:

      The following is the authors’ response to the original reviews.

      Public Reviews:

      Reviewer #1 (Public Review):

      In the manuscript by Su et al., the authors present a massively parallel reporter assay (MPRA) measuring the stability of in vitro transcribed mRNAs carrying wild-type or mutant 5' or 3' UTRs transfected into two different human cell lines. The goal presented at the beginning of the manuscript was to screen for effects of disease-associated point mutations on the stability of the reporter RNAs carrying partial human 5' or 3' UTRs. However, the majority of the manuscript is dedicated to identifying sequence components underlying the differential stability of reporter constructs. This shows that TA dinucleotides are the most predictive feature of RNA stability in both cell lines and both UTRs.

      The effect of AU rich elements (AREs) on RNA stability is well established in multiple systems, and the present study confirms this general trend but points out variability in the consequence of seemingly similar motifs on RNA stability. For example, the authors report that a long stretch of Us has extreme opposite effects on RNA stability depending on whether it is preceded by an A (strongly destabilizing) or followed by an A (strongly stabilizing). While the authors interpretation of a context- dependence of the effect is certainly well-founded, it seems counterintuitive that the preceding or following A would be the (only) determining factor. This points to a generally reductionist approach taken by the authors in the analysis of the data and in their attempt to dissect the contribution of "AU rich sequences" to RNA stability, with a general tendency to reduce the size and complexity of the features (e.g. to dinucleotides). While this certainly increases the statistical power of the analysis due to the number of occurrences of these motifs, it limits the interpretability of the results. How do TA dinucleotides per se contribute to destabilizing the RNA, both in 5' and 3' UTRs, but (according to limited data presented) not in coding sequences? What is the mechanism? RBPs binding to TA dinucleotide containing sequences are suggested to "mask" the destabilizing effect, thereby leading to a more stable RNA. Gain of TA dinucleotides is reported to have a destabilizing effect, but again no hypothesis is provided as to the underlying molecular mechanism. In addition to reducing the motif length to dinucleotides, the notion of "context dependence" is used in a very narrow sense; especially when focusing on simple and short motifs, a more extensive analysis of the interdependence of these features (beyond the existing analysis of the relationship between TA- diNTs and GC content) could potentially reveal more of the context dependence underlying the seemingly opposite behavior of very similar motifs.

      (We have used UA instead of TA, as per the reviewer's suggestion)

      The contribution of coding region sequence to RNA stability has been extensively discussed (For example: doi.org/10.1016/j.molcel.2022.03.032; doi.org/10.1186/s13059-020-02251-5; doi.org/10.15252/embr.201948220; doi.org/10.1371/journal.pone.0228730; doi.org/10.7554/eLife.45396). While UA content at the third codon position (wobble position) has been implicated as a pro-degradation signal, codon optimality has emerged as the most prominent determinant for RNA stability. This indicates that the role of coding regions in RNA stability differs from that of UTRs due to the involvement of translation elongation. We did not intend to suggest that UA-dinucleotides in UTRs and coding regions have the same effect. 

      To ensure the representativeness of the features entered into the LASSO model, we pre-selected those with an occurrence greater than 10% among all UTRs. As a result, while motifs with very low occurrences were excluded from the analysis, there is no evidence to indicate a preference for dinucleotides by the LASSO model.

      We hypothesize that UA-dinucleotide may recruit endonucleases RNase A family, whose catalytic pockets exhibit a strong bias for UA dinucleotide (doi.org/10.1016/j.febslet.2010.04.018). Structures or protein bindings that block this recognition might stabilize RNAs. To gain further insight into the motif interactions, we investigated the interactions between UA and other 15 dinucleotides through more detailed analyses. We conducted a linear regression analysis investigating interactions between UA and the other 15 dinucleotides. The formula used below includes UA:

      , where all 𝛽 terms represent the regression coefficients, and , , and represent the number of UA dinucleotides, the number of other dinucleotides (other than UA), and the GC content of the i<sup>th</sup> UTR, respectively, and 𝜖<sub>i</sub> denotes the error term. For each dinucleotide, we tested the significance of 𝛽<sub>UAxGC%</sub> and 𝛽<sub>UAxDiNT</sub>, and compared their p-values using a quantile-quantile (QQ) plot. Author response image 1 shows that the interaction effect of UA dinucleotides with GC% is much more significant than interactions with the other 15 dinucleotides, as indicated by the inflated QQ plot of p-values. This suggests that GC content is a more critical contextual factor influencing UA dinucleotides' impact on RNA stability.

      Author response image 1.

      The present MPRAs measures the effect of UTR sequences in one specific reporter context and using one experimental approach (following the decay of in vitro transcribed and transfected RNAs). While this approach certainly has its merits compared to other approaches, it also comes with some caveats: RNA is delivered naked, without bound RBPs and no nuclear history, e.g. of splicing (no EJCs), editing and modifications. One way to assess the generalizability of the results as well as the context dependence of the effects is to perform the same analysis on existing datasets of RNA stability measurements obtained through other methods (e.g. transcription inhibition). Are TA dinucleotides universally the most predictive feature of RNA half-lives?

      Our system studies the stability control of RNA synthesized in vitro and delivered into human cells. While we did not intend to generalize our conclusions to endogenous RNAs, our approach contributes to the understanding of in vitro synthesized RNA used for cellular expression, such as in vaccines. It is known that endogenous RNAs undergo very different regulation. The most prominent factors controlling endogenous RNA stability are the density of splice junctions and the length of UTRs (doi.org/10.1186/s13059-022-02811-x; doi.org/10.1186/s12915-021-00949-x). To decipher the sequence regulation, these factors are controlled in our experiments. Therefore, we do not expect the dinucleotide features found by our approach to be generalized as the most predictive feature of RNA half-life in vivo. 

      The authors conclude their study with a meta-analysis of genes with increased TA dinucleotides in 5' and 3'UTRs, showing that specific functional groups are overrepresented among these genes. In addition, they provide evidence for an effect of disease-associated UTR mutations on endogenous RNA stability. While these elements link back to the original motivation of the study (screening for effects of point mutations in 5' and 3' UTRs), they provide only a limited amount of additional insights.

      We utilized the Taiwan Biobank to investigate whether mutations significantly affecting RNA stability also impact human biochemical measurements. Our findings indicate that these mutations indeed have a significant effect on various biochemical indices. This highlights the importance of our study, as it bridges basic science with potential applications in precision medicine. By linking specific UTR mutations with measurable changes in biochemical indices, our research underscores the potential for these findings to inform targeted medical interventions in the future.

      In summary, this manuscript presents an interesting addition to the long-standing attempts at dissecting the sequence basis of RNA stability in human cells. The analysis is in general very comprehensive and sound; however, at times the goal of the authors to find novelty and specificity in the data overshadows some analyses. One example is the case where the authors try to show that TA-dinucleotides and GC content are decoupled and not merely two sides of the same coin.

      They claim that the effect of TA dinucleotides is different between high- and low-GC content contexts but do not control for the fact that low GC-content regions naturally will contain more TA dinucleotides and therefore the effect sizes and the resulting correlation between TA-diNT rate and stability will be stronger (Fig. 5A). A more thorough analysis and greater caution in some of the claims could further improve the credibility of the conclusions.

      Low GC content implies a higher UA content but does not directly equate to a high UA-dinucleotide ratio. For instance, the sequence AUUGAACCUU has a lower GC content (0.3) compared to UAUAGGCCGC (0.6), yet it also has a lower UA-dinucleotide ratio (0 vs. 0.22). To address this concern more rigorously, we performed a stratified analysis based on UA-diNT rate. As shown in our Fig. S7C, even after stratifying by UA- dinucleotide ratio (upper panel high UA- dinucleotide ratio / lower panel low UA- dinucleotide ratio), we still observe that the destabilizing effect of UA is stronger in the low GC content group.

      Reviewer #2 (Public Review):

      Summary of goals:

      Untranslated regions are key cis-regulatory elements that control mRNA stability, translation, and translocation. Through interactions with small RNAs and RNA binding proteins, UTRs form complex transcriptional circuitry that allows cells to fine-tune gene expression. Functional annotation of UTR variants has been very limited, and improvements could offer insights into disease relevant regulatory mechanisms. The goals were to advance our understanding of the determinants of UTR regulatory elements and characterize the effects of a set of "disease-relevant" UTR variants.

      Strengths:

      The use of a massively parallel reporter assay allowed for analysis of a substantial set (6,555 pairs) of 5' and 3' UTR fragments compiled from known disease associated variants. Two cell types were used.

      The findings confirm previous work about the importance of AREs, which helps show validity and adds some detailed comparisons of specific AU-rich motif effects in these two cell types.

      Using a Lasso regression, TA-dinucleotide content is identified as a strong regulator of RNA stability in a context dependent manner based on GC content and presence of RNA binding protein binding motifs. The findings have potential importance, drawing attention to a UTR feature that is not well characterized.

      The use of complementary datasets, including from half-life analyses of RNAs and from random sequence library MRPA's, is a useful addition and supports several important findings. The finding the TA dinucleotides have explanatory power separate from (and in some cases interacting with) GC content is valuable.

      The functional enrichment analysis suggests some new ideas about how UTRs may contribute to regulation of certain classes of genes.

      Weaknesses:

      It is difficult to understand how the calculations for half-life were performed. The sequencing approach measures the relative frequency of each sequence at each time point (less stable sequences become relatively less frequent after time 0, whereas more stable sequences become relatively more frequent after time 0). Since there is no discussion of whether the abundance of the transfected RNA population is referenced to some external standard (e.g., housekeeping RNAs), it is not clear how absolute (rather than relative) half-lives were determined.

      We estimated decay constant λ and half-life (t<sub>1/2</sub>) by the following equations:

      where C<sub>i(t)</sub> and C<sub>i(t=0)</sub> are read count values of the ith replicate at time points 𝑡 and 0 (see also Methods). The absolute abundance was not required for the half-life calculation. 

      Fig. S1A and B are used to assess reproducibility. They show that read counts at a given time point correlate well across replicate experiments. However, this is not a good way to assess reproducibility or accuracy of the measurements of t1/2 are. (The major source of variability in read counts in these plots - especially at early time points - is likely the starting abundance of each RNA sequence, not stability.) This creates concerns about how well the method is measuring t1/2. Also creating concern is the observation that many RNAs are associated with half-lives that are much longer than the time points analyzed in the study. For example, based upon Figure S1 and Table S1 correctly, the median t1/2 for the 5' UTR library in HEK cells appears to be >700 minutes. Given that RNA was collected at 30, 75, and 120 minutes, accurate measurements of RNAs with such long half lives would seem to be very difficult.

      We estimated the half-life based on the following equations:

      where C<sub>i(t)</sub> and C<sub>i(t=0)</sub> are read count values of the ith replicate at time points 𝑡 and 0 (see also Methods). The calculation of the half-life involves first determining the decay constant 𝜆, which represents a constant rate of decay. Since 𝜆 is a constant, it is possible to accurately calculate it without needing data over the entire decay range. Our experimental design considers this by selecting appropriate time points to ensure a reliable estimation of 𝜆, and thus, the half-life. To determine the most suitable time points, we conducted preliminary experiments using RT-PCR.

      These experiments indicated that 30, 75, and 120 minutes provided an effective range for capturing the decay dynamics of the transcripts.

      There is no direct comparison of t1/2 between the two cell types studied for the full set of sequences studied. This would be helpful in understanding whether the regulatory effects of UTRs are generally similar across cell lines (as has been shown in some previous studies) or whether there are fundamental differences. The distribution of t1/2's is clearly quite different in the two cell lines, but it is important to know if this reflects generally slow RNA turnover in HEK cells or whether there are a large number of sequence-specific effects on stability between cell lines. A related issue is that it is not clear whether the relatively small number of significant variant effects detected in HEK cells versus SH-SY5Y cells is attributable to real biological differences between cell types or to technical issues (many fewer read counts and much longer half lives in HEK cells).

      For both cell lines, we selected oligonucleotides with R<sup>2</sup> > 0.5 and mean squared error (MSE) < 1 for analysis when estimating half-life (λ) by linear regression. This selection criterion was implemented to minimize the effect of experimental noise. After quality control, we selected common UTRs and compared the RNA half-lives of the two cell lines using a scatter plot. Author response image 2 shows that RNA half-lives are quite different between the cell lines, with a moderate similarity observed in the 5' UTRs (R = 0.21), while the correlation in the 3' UTRs is non-significant.

      Author response image 2.

      Despite the low correlation of mRNA half-life between the two cell lines, UA-dinucleotide and UA-rich sequences consistently emerge as the most significant destabilizing features, suggesting a shared regulatory mechanism across diverse cellular environments.

      The general assertion is made in many places that TA dinucleotides are the most prominent destabilizing element in UTRs (e.g., in the title, the abstract, Fig. 4 legend, and on p. 12). This appears to be true for only one of the two cell lines tested based on Fig. 3.

      UA-dinucleotides and other UA-rich sequences exhibit similar effects on RNA stability, as illustrated in Fig. S5A-C. In two cell lines, UA-dinucleotide and WWWWWW sequences were representatives of the same stability-affecting cluster. While the impact of UA-dinucleotides can be generalized, we have rephrased some statements for clarification to avoid any potential misunderstanding. For examples: 

      Abstract: “...We found that UA dinucleotides and UA-rich motifs are the most prominent destabilizing element.“

      p.10: “UA dinucleotides and UA-rich motifs are the most common and effective RNA destabilizing factor” 

      Figure 4: “The UTR UA dinucleotides and UA-rich motifs are the most common and influential RNA destabilizing factor.”

      Appraisal and impact:

      The work adds to existing studies that previously identified sequence features, including AREs and other RNA binding protein motifs, that regulate stability and puts a new emphasis on the role of "TA" (better "UA") dinucleotides. It is not clear how potential problems with the RNA stability measurements discussed above might influence the overall conclusions, which may limit the impact unless these can be addressed.

      It is difficult to understand whether the importance of TA dinucleotides is best explained by their occurrence in a related set of longer RBP binding motifs (see Fig 5J, these motifs may be encompassed by the "WWWWWW cluster") or whether some other explanation applies. Further discussion of this would be helpful. Does the LASSO method tend to collapse a more diverse set of longer motifs that are each relatively rare compared to the dinucleotide? It remains unclear whether TA dinucleotides are associated with less stability independent of the presence of the known larger WWWWWWW motif. As noted above, the importance of TA dinucleotides in the HEK experiments appears to be less than is implied in the text.

      To ensure the representativeness of the features entered into the LASSO model, we pre-selected those with an occurrence greater than 10% among all UTRs. There is no evidence to support a preference for dinucleotides by LASSO. To address whether the destabilizing effect of UA dinucleotides is part of the broader WWWWWW motif, we divided UA dinucleotides into two groups: those within the WWWWWW motif and those outside of it. Specifically, we divided UTRs into two categories: 'at least one UA within a WWWWWW motif' and 'no UA within a WWWWWW motif,' and visualized the results using a boxplot. As shown in Author response image 3, the destabilizing trend still remains for UA dinucleotides outside of the WWWWWW motif, although the effect appears to be more pronounced when UA is within the WWWWWW motif. This suggests that while UA dinucleotides have a destabilizing effect independently, their impact is amplified when they are part of the broader WWWWWW motif.

      Author response image 3.

      The inclusion of more than a single cell type is an acknowledgement of the importance of evaluating cell type-specific effects. The work suggests a number of cell type-specific differences, but due to technical issues (especially with the HEK data, as outlined above) and the use of only two cell lines, it is difficult to understand cell type effects from the work.

      The inclusion of both 3' and 5' UTR sequences distinguishes this work from most prior studies in the field. Contrasting the effects of these regions on stability is of interest, although the role of these UTRs (especially the 5' UTR) in translational regulation is not assessed here.

      We examined the role of UTR and UTR variants in translation regulation using polysome profiling. By both univariate analysis and an elastic regression model, we identified motifs of short repeated sequences, including SRSF2 binding sites, as mutation hotspots that lead to aberrant translation. Furthermore, these polysome-shifting mutations had a considerable impact on RNA secondary structures, particularly in upstream AUG-containing 5’ UTRs. Integrating these features, our model achieved high accuracy (AUROC > 0.8) in predicting polysome-shifting mutations in the test dataset. Additionally, metagene analysis indicated that pathogenic variants were enriched at the upstream open reading frame (uORF) translation start site, suggesting changes in uORF usage underlie the translation deficiencies caused by these mutations. Illustrating this, we demonstrated that a pathogenic mutation in the IRF6 5’ UTR suppresses translation of the primary open reading frame by creating a uORF. Remarkably, site-directed ADAR editing of the mutant mRNA rescued this translation deficiency. Because the regulation of translation and stability does not converge, we illustrate these two mechanisms in two separate manuscripts (this one and doi.org/10.1101/2024.04.11.589132).

      Reviewer #3 (Public Review):

      Summary:

      In their manuscript titled "Multiplexed Assays of Human Disease‐relevant Mutations Reveal UTR

      Dinucleotide Composition as a Major Determinant of RNA Stability" the authors aim to investigate the effect of sequence variations in 3'UTR and 5'UTRs on the stability of mRNAs in two different human cell lines.

      To do so, the authors use a massively parallel reporter assay (MPRA). They transfect cells with a set of mRNA reporters that contain sequence variants in their 3' or 5' UTRs, which were previously reported in human diseases. They follow their clearance from cells over time relative to the matching non-variant sequence. To analyze their results, they define a set of factors (RBP and miRNA binding sites, sequence features, secondary structure etc.) and test their association with differences in mRNA stability. For features with a significant association, they use clustering to select a subset of factors for LASSO regression and identify factors that affect mRNA stability.

      They conclude that the TA dinucleotide content of UTRs is the strongest destabilizing sequence feature. Within that context, elevated GC content and protein binding can protect susceptible mRNAs from degradation. They also show that TA dinucleotide content of UTRs affects native mRNA stability, and that it is associated with specific functional groups. Finally, they link disease associated sequence variants with differences in mRNA stability of reporters.

      Strengths:

      This work introduces a different MPRA approach to analyze the effect of genetic variants. While previous works in tissue culture use DNA transfections that require normalization for transcription efficiency, here the mRNA is directly introduced into cells at fixed amounts, allowing a more direct view of the mRNA regulation.

      The authors also introduce a unique analysis approach, which takes into account multiple factors that might affect mRNA stability. This approach allows them to identify general sequence features that affect mRNA stability beyond specific genetic variants, and reach important insights on mRNA stability regulation. Indeed, while the conclusions to genetic variants identified in this work are interesting, the main strength of the work involve general effect of sequence features rather than specific variants.

      The authors provide adequate supports for their claims, and validate their analysis using both their reporter data and native genes. For the main feature identified, TA di-nucleotides, they perform follow-up experiments with modified reporters that further strengthen their claims, and also validate the effect on native cellular transcripts (beyond reporters), demonstrating its validity also within native scenarios.

      The work provides a broad analysis of mRNA stability, across two mRNA regulatory segments (3'UTR and 5'UTR) and is performed in two separate cell-types. Comparison between two different cell-types is adequate, and the results demonstrate, as expected, the dependence of mRNA stability on the cellular context. Analysis of 3'UTR and 5'UTR regulatory effects also shows interesting differences and similarities between these two regulatory regions.

      Weaknesses:

      (1) The authors fail to acknowledge several possible confounding factors of their MPRA approach in the discussion.

      First, while transfection of mRNA directly into cells allows to avoid the need to normalize for differences in transcription, the introduction of naked mRNA molecules is different than native cellular mRNAs and could introduce biases due to differences in mRNA modifications, protein associations etc. that may occur co-transcriptionally.

      Second, along those lines, the authors also use in-vitro polyadenylation. The length of the polyA tail of the transfected transcripts could potentially be very different than that of native mRNAs and also affect stability.

      The transcripts used in our study were polyadenylated in vitro with approximately 100 nucleotides 

      (Fig. S1C), similar to the polyA tail lengths typically observed in vivo (dx.doi.org/10.1016/j.molcel.2014.02.007).  Additionally, these transcripts were capped to emulate essential mRNA characteristics and to minimize immune responses in recipient cells. This design allows us to study RNA decay for in vitro-synthesized RNA delivered into human cells, akin to RNA vaccines, but it does not necessarily extend to endogenous RNAs. As mentioned, endogenous RNAs undergo nuclear processing and are decorated by numerous trans factors, resulting in distinct regulatory mechanisms. We therefore provided a more discussion on these differences and their implications in the revised manuscript: “However, while our approach effectively assesses the stability of synthesized RNA in human cells, it may not fully capture the decay dynamics of nuclear-synthesized RNA, which can be influenced by endogenous modifications and trans-acting RNA binding factors. (p. 18)”

      (2) The analysis approach used in this work for identifying regulatory features in UTRs was not previously used. As such, lack of in-depth details of the methodology, and possibly also more general validation of the approach, is a drawback in convincing the reader in the validity of this approach and its results.

      In particular, a main point that is not addressed is how the authors decide on the set of "factors" used in their analysis? As choosing different sets of factors might affect the results of the analysis. 

      In our study, we employed the calculation of the Variance Inflation Factor (VIF) as a basis for selecting variables. This well-established method is widely used to detect variables with high collinearity, thus ensuring the robustness and reliability of our analysis. By identifying and excluding highly collinear variables, we aimed to minimize multicollinearity and improve the accuracy of our regression models. For more detailed information on the use of VIF in regression analysis, please refer to Akinwande, M., Dikko, H., and Samson, A. (2015). Variance Inflation Factor: As a Condition for the Inclusion of Suppressor Variable(s) in Regression Analysis. Open Journal of Statistics, 5, 754-767. doi: 10.4236/ojs.2015.57075. We have included the method details in the revised manuscript (p. 28) :”… to avoid multicollinearity caused by similar features that perturb feature selection, all features were clustered using single-linkage hierarchical clustering with the distance metric defined as one minus the absolute value of the Spearman correlation coefficient. We cut the tree at a specific height, and the feature that had the greatest influence on RNA stability, which was examined using a simple linear regression model, was selected to be the representative of each cluster. Then we calculated the variance inflation factor (VIF) value of the representative features. The VIFs were obtained by the following linear model and equations:

      where and are the estimated value of the jth feature and the value of the kth feature of the ith UTR (note that the kth feature is a feature other than the jth feature), and are the intercept and the regression coefficients of the linear model that regressed the jth feature on the other remaining features, and is the mean level of the jth feature of all UTRs.”

      For example, the choice to use 7-mer sequences within the factors set is not explained, particularly when almost all motifs that are eventually identified (Figure 3B-E) are shorter.

      The known RBP motifs are primarily 6-mer. To explore the possibility of discovering novel motifs that could significantly impact our model, we started with 7-mer sequences. However, our analysis revealed that including these additional variables did not improve the explanatory power of the model; instead, it reduced it. Consequently, our final model focuses on motifs shorter than 7-mer. We explained the motif selections in the revised manuscript (p. 9): “Given our discovery that the effect of AREs is heavily dependent on sequence content, we decided to further explore the effects of other sequence elements, i.e., beyond known regulatory motifs, in more detail. Since most reported RBP motifs are 6-mers, we initiated a search for novel motifs by analyzing the presence of all 7-mers in our massively parallel reporter assay (MPRA) library, correlating their occurrence with mRNA half-life.”

      In addition, the authors do not perform validations to demonstrate the validity of their approach on simulated data or well-established control datasets. Such analysis would be helpful to further convince the reader in the usefulness and robustness of the analysis.

      We acknowledge the importance of validating our approach on simulated data or well-established control datasets to demonstrate its robustness and reliability. However, to the best of our knowledge, there are currently no well-established control datasets available that perfectly correspond to our specific study context. Despite this, we will continue to search for any relevant datasets that could be utilized for this purpose in future work. This effort will help to further reinforce the confidence in our methodology and its findings.

      (3) The analysis and regression models built in this work are not thoroughly investigated relative to native genes within cells. The effect of sequence "factors" on native cellular transcripts' stability is not investigated beyond TA di-nucleotides, and it is unclear to what degree do other predicted factors also affect native transcripts.

      Our system studies the stability control of RNA synthesized in vitro and delivered into human cells. While we validated the UTR UA-dinucleotide effect in vivo, we did not intend to conclude that this is the most influential regulation for endogenous RNAs. It is known that endogenous RNAs undergo very different regulation. The most prominent factors controlling endogenous RNA stability are the density of splice junctions and the length of UTRs (doi.org/10.1186/s13059-022-02811-x; doi.org/10.1186/s12915-021-00949-x). To decipher the sequence regulation, we controlled for these factors in our experiments. Therefore, we acknowledge that several endogenous features, which were excluded by our approach, may serve as predictive features of RNA half-life in vivo. 

      Recommendations for the authors:

      Reviewer #1 (Recommendations For The Authors):

      Specific comments:  

      Some references are missing, e.g for the sentence:

      Please see the response below.

      "Similarly, point mutation of the GFPT1 3' UTR results in congenital myasthenic syndrome." (p5)

      The reference has been added to the text:

      Dusl, M., Senderek, J., Muller, J. S., Vogel, J. G., Pertl, A., Stucka, R., Lochmuller, H., David, R., & Abicht, A. (2015). A 3'-UTR mutation creates a microRNA target site in the GFPT1 gene of patients with congenital myasthenic syndrome. Human Molecular Genetics, 24(12), 34183426. https://doi.org/10.1093/hmg/ddv090 

      "...but there have been no systematic assessments of the explicit effects of variants of both UTRs on stability regulation." (not true in the current phrasing; e.g. PMIDs 32719458, 36156153, 34849835)

      These references have been added to the text. However, we have to point out that these studies do not focus on the effects of the disease-relevant variants. To clarify, we modified the sentence to "... systematic assessments of the explicit effects of disease-relevant variants in both UTRs on stability regulation are still absent."

      "Multiple approaches have revealed AREs as exerting a destabilizing effect on RNA stability (Barreau et al., 2005). (p8)

      The reference has been added to the text:

      Barreau, C., Paillard, L., & Osborne, H. B. (2005). AU-rich elements and associated factors: are there unifying principles? Nucleic Acids Research, 33(22), 7138-7150. https://doi.org/10.1093/nar/gki1012 

      "This effect is specific, as such ratios in the coding region are inconsequential." (p12)

      This refers to our findings of Fig. 4G and Supplemental Fig. S5F.

      What are the sequences at the 5' and 3'UTR without insertion of a library? 5'UTR library (especially in SH) has much longer half-life compared to 3'utr library (Fig S1D).

      There is no designed 5’UTR of the 3’UTR library, only the Kozak sequence derived from the pEGFPC1 vector. This may partially underlie the shorter half-life of the 3’ UTR library.

      Fig2A: What are the units? "half-life (log)" Do the numbers correspond to log10(min)?

      It represents ln (min). To clarify, we now use ‘ln t<sub>1/2</sub> (min)’ in all figures.

      Fig 2 and 3: This was done only on the wild-type sequences? Or all tested sequences together, wt and mut?

      It was done only on the wild-type sequences. To clarify, we modified the text to “we examined the effect of AREs on RNA stability of the ref alleles according to specific sequence content….(p.8)” and “We considered as many factors as possible to explain the half-life of our ref UTR libraries,…. (p.9)”. ‘ref’ stands for reference.

      "Furthermore, to avoid collinearity confounding our model, e.g., the effects of very similar factors (such as 'AA' and 'AAA' sequences), we clustered the factors according to their properties, and then only one representative factor from within a cluster (i.e., the one with the highest correlation to halflife within a cluster) was subjected to LASSO regression": Given the observed context dependence, e.g. in the case of poly-U stretches: Isn't this clustering leading to similar/identical motifs with different context being grouped together (such as polyU preceded by an A (strongly destabilizing, according to Fig 2B) or followed by one (strongly stabilizing, according to Fig 2B), resulting in ignoring the context or using one potential outcome while a motif from the same cluster can have the opposite effect?

      Thank you very much for pointing this out. To determine if considering different contextual effects within each feature cluster would enhance model performance, we modified our feature selection by choosing both the feature with the largest positive and the largest negative effect on RNA half-life in Step III of Figure 3A. We then split the data into a 2:1 training and testing set and repeated this process 100 times. Model performance was evaluated using mean average error (MAE), root mean squared error (RMSE), and adjusted R-squared. From Author response image 4, we observed no significant improvement in model performance using this new approach. Notably, in the SH-SY5Y 5' UTR model, our original method even outperformed the modified one, with statistically lower MAE and RMSE and a higher adjusted R-squared. Therefore, we believe our current approach remains appropriate.

      Author response image 4.

      "Overall, motifs that are at least two nucleotides long proved critical for RNA stability, supporting the sequence specificity of the decay process." Unclear why this supports the "sequence specificity"

      No monomers were selected as an explanatory factor. On the contrary, specific sequence combinations and order are important for the regulation. These findings suggest sequence-specific recognition for the decay process.

      Fig3: The same features were used in both cell lines? If yes: Since they were selected for their highest correlation with half-life, how was a common set chosen? If no: problematic to compare.

      Thank you for your question regarding feature selection across cell lines. Initially, the features were collected uniformly for both cell lines. However, subsequent feature selection steps were cell-type specific, focusing on identifying features with the greatest impact on RNA half-life in each context. This approach allows us to still compare model performance and discuss the similarities and differences in selected features across cell types. By maintaining a consistent starting point, we ensure that any observed differences reflect cell-specific regulatory dynamics.

      uORFs were not used as features?

      Thank you for pointing this out. At the beginning of our study, we investigated the impact of Kozak sequence strength (categorized as weak, moderate, strong, or optimal) on RNA half-life. However, we found that this feature performed poorly in predicting RNA stability, and as a result, we decided not to include upstream open reading frames (uORFs) or Kozak sequences in our subsequent analyses.

      Experimental reproducibility: Only correlations between replicates for the same time point is shown, but no comparison between time points or between decay rates. How reproducible were the paired differences between mut/wt?

      The decay rate was calculated by modeling the slope of a linear regression of all time points. Therefore, there is only one decay rate associated with a genotype. To rule out inconsistent data, we excluded any regression with a mean square error greater than 1, as this indicates a poor fit of the data points. 

      Fig 7C/p17: This does not establish a "causal relationship" as the authors claim.

      We agree with the reviewer’s suggestion. We have modified the text on p.17 to “to establish a correlation between UTR variants and health outcomes,…..”

      In the discussion, the authors claim that TA-diNTs are not only an opposite of the GC percentage and base this on Fig 5A.

      Fig 5A: The range of TA-diNTs is naturally much higher in the low GC group. To make the high and low GC content comparable (as the authors aim to do), the correlation should be assessed for the same range of TA dint in both cases.

      To address this concern more rigorously, we performed a stratified analysis based on UA-diNT rate. As shown in our Fig. S7C, even after stratifying by UA- dinucleotide ratio (upper panel high UA- dinucleotide ratio / lower panel low UA- dinucleotide ratio), we still observe that the destabilizing effect of UA is stronger in the low GC content group.

      Supplemental Figure S7. Interplay of GC content and TA dinucleotide on stability regulation, related to Figure 5. (C) Stratifications of both TA dinucleotide ratio and GC content showed that the destabilizing effect of TA dinucleotide is the most prominent under conditions of low TA dinucleotide ratio and low GC content. The same trend was observed for 5’ UTR (left) and 3’ UTR (right).

      The injection of in vitro transcribed and polyA/capped RNA certainly has advantages over other methods, but delivering naked mRNA without nuclear history might also lead to artifacts. The caveats of the approach should be discussed more extensively.

      We appreciate the suggestion and have hence added the following in the Discussion (p.18): “However, while our approach effectively assesses the stability of synthesized RNA in human cells, it may not fully capture the decay dynamics of nuclear-synthesized RNA, which can be influenced by endogenous modifications and trans-acting RNA binding factors.”

      "We unexpectedly identified many crucial regulatory features in 5' UTRs." Why was this unexpected?

      We initially thought the 3’ UTR would play a major role in stability regulation. To avoid confusion, we have removed the word ‘unexpected’ from the text (p. 20): "We identified many crucial regulatory features in 5' UTRs."

      "...a massively parallel reporter assay in which coding regions and human 5'/3' UTRs with diseaserelevant mutations were generated in vitro and then directly transfected into human cell lines to assess their decay patterns by next‐generation sequencing": also coding regions?

      Thanks for the question. Indeed, the coding region was not synthesized together with the UTR library. Therefore, we modified the text of p. 6 to “…we developed a massively parallel reporter assay in which human 5’/3’ UTRs with disease-relevant mutations were generated in vitro, ligated with the enhanced green fluorescence protein (EGFP) coding region, and then directly transfected into human cell lines to assess their decay patterns by next-generation sequencing.”

      Reviewer #2 (Recommendations For The Authors):

      Nomenclature: When discussing RNA sequences, "U" should be used in place of "T" (e.g., "UA dinucleotide").

      We have replaced the RNA sequence “T” with “U” of the text and figures.

      Abstract: "We examined the RNA degradation patterns mediated by the UTR library in multiple cell lines" - It would be clearer to state that two cell lines (rather than multiple) were used.

      We appreciate the suggestion. We have modified the abstract as suggested: “We examined the RNA degradation patterns mediated by the UTR library in two cell lines…"

      The manuscript refers to "wild-type (WT) and mutant (mt) alleles." (p. 7 and elsewhere). It would be better to use "reference" instead of "wild type" given that these are human populations.

      We appreciate the suggestion. All instances of ‘wild-type’ or ‘WT’ in the text and figures have been replaced with ‘reference’ or ‘ref’.

      In the introduction, it is stated that traditional MPRAs "cannot differentiate the effect of the UTRs on transcription, stability and, in some cases, even protein production, greatly limiting scientific interpretation." This is confusing, since these assays can and have been used in association with both RNA decay measurements and measurements of reporter protein levels that allow assessment of effects on stability and protein production (including in the cited references).

      We reason that the RNA steady-state level (e.g., sequencing the overall RNA normalized to DNA) or protein steady-state level (e.g., detecting the fluorescence signal) does not precisely reveal the decay kinetics of the RNA. Steady-state level is a result of production and decay, both of which UTRs contribute to. Similarly, the protein level is not a perfect estimate of the RNA decay.

      To clarify, we have modified the introduction (p. 5) to “Nevertheless, because the steady-state level is a result of production and decay, these approaches cannot differentiate the effect of the UTRs on transcription, stability and, in some cases, even protein production, greatly limiting scientific interpretation.” 

      Adding raw and normalized read count data from individual experiments (e.g., to Table S1) would make it more likely for others to use this dataset to address additional questions.

      All raw and processed sequencing data generated in this study have been submitted to the NCBI Gene Expression Omnibus (GEO; https://www.ncbi.nlm.nih.gov/geo/) under accession number GSE217518 (reviewer token snspaakujtsdpcv).

      The manuscript would benefit from further clarification about model selection. Additional details regarding how the features were clustered, and the actual clusters themselves should be included.

      It should be discussed why Lasso was chosen vs Ridge or Elastic Net, in the context of handling multicollinearity. Often, data is subsetted for training and validation, and model performance metrics are presented.

      Thank you for pointing out the need for further clarification on model selection. The features were clustered using single-linkage hierarchical clustering with the distance metric defined as one minus the absolute value of the Spearman correlation coefficient (this information has been added to the manuscript on p. 28: “…to avoid multicollinearity caused by similar features that perturb feature selection, all features were clustered using single-linkage hierarchical clustering with the distance metric defined as one minus the absolute value of the Spearman correlation coefficient.”). The resulting feature clusters are available in Supplemental Table S3. 

      Regarding model selection, we chose LASSO over ridge and elastic net primarily for feature selection, as ridge does not perform feature selection. Elastic net is essentially a hybrid of ridge regression and LASSO regularization, but we opted for LASSO for its simplicity and effectiveness in selecting a sparse set of important features.

      We also performed a 2:1 training and testing set analysis and have included these details in the manuscript. Model performance metrics, including correlation coefficient between observed and predicted values in the testing set, mean absolute error (MAE), root mean squared error (RMSE), mean absolute percentage error (MAPE), and R-squared, are provided in new Supplemental Table S4.

      Recommend reviewing and correcting verb tenses in the methods section.

      We appreciate the reviewer’s suggestion. We have corrected verb tenses in the methods section, which includes “The UTRs were defined by NCBI RefSeq and ENCODE V27. (p.21)”, “The variant was placed in the middle of the sequence….(p.22)”, and “eCLIP signals with value < 1 or p value > 0.05 were removed. (p.26)”

      Please add information about which cell type(s) are being used in each of the figure legends (e.g., in Figs. 2B and 5).

      We appreciate the reviewer’s suggestion. We have added the cell type information in the figure legends: “Figure 2…. (B) The ten most influential AREs in terms of RNA stability in SH-SY5Y cells.” And “Figure 5…..(A) MPRA data of SH-SY5Y cells stratified according to the GC content (GC%) of UTRs.”

      Recommend review of axis labels and consistency in formatting the log(half-lives) and including the base of the log and the time unit (minutes). Even better, converting axis labels from log minutes to minutes would make this easier to understand.

      Thank you for the suggestion regarding axis labels and consistency. We have unified the half-life label to ‘ln t<sub>1/2</sub> (min)’ in all figures. We chose not to convert the axis from logarithmic minutes to minutes because the original scale is highly skewed, which would hinder clear data visualization.

      The discussion refers to Figure 1D but Figure 1 only has A-C

      Thank you for pointing out this mistake. ‘Fig. 1D’ has been changed to ‘Fig. 1B’ in the text (p. 7 and p. 20).

      The analyses in Fig. 2 are interpreted as demonstrating that AREs destabilize RNAs. These analyses are examining associations, so it would be more appropriate to say that AREs are associated with destabilization (since it is formally possible that other sequences that are present in these UTR fragment cause destabilization). A similar issue arises on p. 10: "TA dinucleotides alone can negatively regulate RNA stability, with a Pearson's correlation coefficient of ‐0.287 for 5' UTRs and ‐0.377 for 3' UTRs (Fig. 4A,C)." This is an association and does not establish causation. Again on p. 17: "We identified several SNPs in UTRs that induce aberrant RNA expression and/or protein expression (Supplemental Table S7)." These may be causal but may simply be in LD with other variants that are causal.

      We agree that the association observed is not proven to be causal. Therefore, we modified the text as suggested: 

      “AUUUA/AUUA-containing AREs are associated with RNA destabilization.” (p. 8)

      “UA dinucleotides alone present a negative correlation with RNA stability, with a Pearson’s correlation coefficient of -0.287 for 5’ UTRs and -0.377 for 3’ UTRs.”  (p.10)

      “We identified several SNPs in UTRs that correlated with aberrant RNA expression and/or protein expression.”  (p. 17)

      Figure 4C is important in that it examines whether variant sequences that differ in a manner that changes the number of dinucleotide repeats affect stability. Please show the number (not just the percentage) of sequences in each category.

      Thank you for your insightful comment. We believe the figure you referred to is Figure 4E. We have updated the figure to include the number of sequences in each category.

      Figure 6A and B: The horizontal axes appear to be misaligned since the dotted vertical lines do not cross at 0. ?

      The dotted vertical lines represent the genomic background of the UA-diNT ratio. To clarify it, we have modified the legend to: “Figure 6……(A) The top ten biological processes for which the 5’ UTR UA-dinucleotide ratio most significantly deviated from the genomic background (dashed line).”

      It may be helpful to state what the dashed and solid lines represent on Figure 6 E/F. Please correct spelling of "Biological" in 6E.

      As per the reviewer’s suggestions, we have modified the legend of Figure 6 to: “………..(E) Biological processes for RNAs in which the UA-dinucleotide ratios of both 5’ and 3’ UTRs are significantly different from the genomic background (dashed lines). (F) Molecular functions for RNAs in which the UA-dinucleotide ratios of both 5’ and 3’ UTRs are significantly different from the genomic background (dashed lines). The thin solid lines represent the standard deviation of the UAdinucleotide ratio within the gene group.” 

      In addition, the spelling of “Biological” in Fig. 6E has been corrected.

      Reviewer #3 (Recommendations For The Authors):

      I have 3 points that I think could improve science and its presentation within the manuscript.

      (1) Most importantly, how well do LASSO regression models predict the stability of native transcripts? Such analysis can also be useful for comparison between two different cell-types. How well does the regression model learned (on reporters) within one cell-type predict mRNA stability (of reporters and native genes) in this cell-type and in the other cell-type? Similarly, models can also help to analyze the effects of 5'UTR and 3'UTR sequences on mRNA stability. In particular, how well does the regression model of each separate regulatory sequence (3'UTR or 5'UTR) is able to predict the stability of native genes in the cell? Can the predictions be improved by combining both 3'UTR and 5'UTR sequence features within the regression models?

      The decay model for native transcripts has been established in prior research (doi.org/10.1186/s13059-022-02811-x; doi.org/10.1186/s12915-021-00949-x), which indicates that exon junction density and transcript length are the primary determinants of RNA stability. Based on these findings, we designed the MPRA with fixed length and without splicing to focus on the contribution of primary sequences. We validated the destabilizing effect of UA dinucleotide on endogenous RNAs (Fig. 4G and Supplemental Fig. S5F) but do not recommend using our model to fully explain or predict the stability of native transcripts.

      To assess the model's cross-cell type predictive performance for RNA half-life, we employed the Regression Error Characteristic (REC) curve (Bi & Bennett, 2003). Similar to the receiver operating characteristic (ROC) curve, the REC curve illustrates the trade-off between error tolerance and accuracy, with better performance indicated by curves trending toward the upper left. We also computed the Area Over the Curve (AOC) as a performance metric, where lower values indicate better predictive ability. From Author response image 5, the REC curves reveal that cross-cell type prediction performance is suboptimal. The y-axis represents prediction accuracy, while the x-axis denotes error tolerance for the natural logarithm of RNA half-life (ln(𝑡<sub>1/2</sub>), in minutes).

      Author response image 5.

      In response to the suggestion of combining 5' and 3' UTR sequence features in the regression model, we believe this approach may not be ideal. As shown in Figure S1D, the distribution of RNA half-lives between 5' and 3' UTRs is significantly different, reflecting their distinct regulatory roles. Additionally, the base composition differs, with 5' UTRs having a higher GC content compared to 3' UTRs. Combining these datasets would likely make the origin of the sequence (5' or 3' UTR) the most predictive feature, thereby reducing the model's interpretability. Furthermore, our MPRA results, derived from separate 5’ or 3’ UTR library, do not support a combined model, further suggesting this approach may not be suitable with our data.

      The conclusions regarding genetic variants are interesting, yet the main strength of the work involves identifying general sequence features that affect mRNA stability rather than specific variants. I wonder if the authors have considered to shift the focus of the motivation part to reflect that?

      We appreciated the reviewer’s suggestion. We have revised the abstract and introductions to emphasize the general UTR regulation. Here is the revised abstract:

      UTRs contain crucial regulatory elements for RNA stability, translation and localization, so their integrity is indispensable for gene expression. Approximately 3.7% of genetic variants associated with diseases occur in UTRs, yet a comprehensive understanding of UTR variant functions remains limited due to inefficient experimental and computational assessment methods. To systematically evaluate the effects of UTR variants on RNA stability, we established a massively parallel reporter assay on 6,555 UTR variants reported in human disease databases. We examined the RNA degradation patterns mediated by the UTR library in two cell lines, and then applied LASSO regression to model the influential regulators of RNA stability. We found that UA dinucleotides and UA-rich motifs are the most prominent destabilizing element. Gain of UA dinucleotide outlined mutant UTRs with reduced stability. Studies on endogenous transcripts indicate that high UA-dinucleotide ratios in UTRs promote RNA degradation. Conversely, elevated GC content and protein binding on UA dinucleotides protect high-UA RNA from degradation. Further analysis reveals polarized roles of UA-dinucleotide-binding proteins in RNA protection and degradation. Furthermore, the UA-dinucleotide ratio of both UTRs is a common characteristic of genes in innate immune response pathways, implying a coordinated stability regulation through UTRs at the transcriptomic level. We also demonstrate that stability-altering UTRs are associated with changes in biobank-based health indices, underscoring the importance of precise UTR regulation for wellness. Our study highlights the importance of RNA stability regulation through UTR primary sequences, paving the way for further exploration of their implications in gene networks and precision medicine.

      Plots presenting correlations (e.g., Figure 4A, 4C) are more informative when plotted as density plots (i.e., using colorscale to show density of the dots at each part of the plot).

      We greatly appreciate the reviewer's insightful suggestion regarding the use of density plots for presenting correlations. We have modified Figures 4A and 4C in the revised manuscript to implement density plotting. The updated figures now utilize a colorscale that highlights areas of high and low data density.

    1. Author Response

      The following is the authors’ response to the original reviews.

      Public Reviews:

      Reviewer #1 (Public Review):

      Qin et al. set out to investigate the role of mechanosensory feedback during swallowing and identify neural circuits that generate ingestion rhythms. They use Drosophila melanogaster swallowing as a model system, focusing their study on the neural mechanisms that control cibarium filling and emptying in vivo. They find that pump frequency is decreased in mutants of three mechanotransduction genes (nompC, piezo, and Tmc), and conclude that mechanosensation mainly contributes to the emptying phase of swallowing. Furthermore, they find that double mutants of nompC and Tmc have more pronounced cibarium pumping defects than either single mutants or Tmc/piezo double mutants. They discover that the expression patterns of nompC and Tmc overlap in two classes of neurons, md-C and md-L neurons. The dendrites of md-C neurons warp the cibarium and project their axons to the subesophageal zone of the brain. Silencing neurons that express both nompC and Tmc leads to severe ingestion defects, with decreased cibarium emptying. Optogenetic activation of the same population of neurons inhibited filling of the cibarium and accelerated cibarium emptying. In the brain, the axons of nompC∩Tmc cell types respond during ingestion of sugar but do not respond when the entire fly head is passively exposed to sucrose. Finally, the authors show that nompC∩Tmc cell types arborize close to the dendrites of motor neurons that are required for swallowing, and that swallowing motor neurons respond to the activation of the entire Tmc-GAL4 pattern.

      Strengths:

      • The authors rigorously quantify ingestion behavior to convincingly demonstrate the importance of mechanosensory genes in the control of swallowing rhythms and cibarium filling and emptying

      • The authors demonstrate that a small population of neurons that express both nompC and Tmc oppositely regulate cibarium emptying and filling when inhibited or activated, respectively

      • They provide evidence that the action of multiple mechanotransduction genes may converge in common cell types

      Thank you for your insightful and detailed assessment of our work. Your constructive feedback will help to improve our manuscript.

      Weaknesses:

      • A major weakness of the paper is that the authors use reagents that are expressed in both md-C and md-L but describe the results as though only md-C is manipulated-Severing the labellum will not prevent optogenetic activation of md-L from triggering neural responses downstream of md-L. Optogenetic activation is strong enough to trigger action potentials in the remaining axons. Therefore, Qin et al. do not present convincing evidence that the defects they see in pumping can be specifically attributed to md-C.

      Thank you for your comments. This is important point that we did not adequately address in the original preprint. We have obtained imaging and behavioral results that strongly suggest md-C, rather than md-L, are essential for swallowing behavior.

      36 hours after the ablation of the labellum, the signals of md-L were hardly observable when GFP expression was driven by the intersection between Tmc-GAL4 & nompC-QF (see F Figure 3—figure supplement 1A). This observation indicates that the axons of md-L likely degenerated after 36 hours, and were unlikely to influence swallowing. Moreover, the projecting pattern of Tmc-GAL4 & nompC-QF>>GFP exhibited no significant changes in the brain post labellum ablation.

      Furthermore, even after labellum ablation for 36 hours, flies exhibited responses to light stimulation (see Figure 3—figure supplement 1B-C, Video 5) when ReaChR was expressed in md-C. We thus reasoned that md-C but not md-L, plays a crucial role in the swallowing process.

      • GRASP is known to be non-specific and prone to false positives when neurons are in close proximity but not synaptically connected. A positive GRASP signal supports but does not confirm direct synaptic connectivity between md-C/md-L axons and MN11/MN12.

      In this study, we employed the nSyb-GRASP, wherein the GRASP is expressed at the presynaptic terminals by fusion with the synaptic marker nSyb. This method demonstrates an enhanced specificity compared to the original GRASP approach.

      Additionally, we utilized +/ UAS-nSyb-spGFP1-10, lexAop-CD4-spGFP11 ; + / MN-LexA fruit flies as a negative control to mitigate potential false signals originating from the tool itself (Author response image 1, scale bar = 50μm). Beside the genotype Tmc-Gal4, Tub(FRT. Gal80) / UAS-nSyb-spGFP1-10, lexAop-CD4-spGFP11 ; nompC-QF, QUAS-FLP / MN-LexA fruit flies discussed in this manuscript, we also incorporated genotype Tmc-Gal4, Tub(FRT. Gal80) / lexAop-nSyb-spGFP1-10, UAS-CD4-spGFP11 ; nompC-QF, QUAS-FLP / MN-LexA fruit flies as a reverse control (Author response image 2). Unexpectedly, similar positive signals were observed, indicating that, positive signals may emerge due to close proximity between neurons even with nSyb-GRASP.

      Author response image 1.

      It should be noted that the existence of synaptic projections from motor neurons (MN) to md-C cannot be definitively confirmed at this juncture. At present, we can only posit the potential for synaptic connections between md-C and motor neurons. A more conclusive conclusion may be attainable with the utilization of comprehensive whole-brain connectome data in future studies.

      Author response image 2.

      • As seen in Figure 2—figure supplement 1, the expression pattern of Tmc-GAL4 is broader than md-C alone. Therefore, the functional connectivity the authors observe between Tmc expressing neurons and MN11 and 12 cannot be traced to md-C alone

      It is true that the expression pattern of Tmc-GAL4 is broader than that of md-C alone. Our experiments, including those flies expressing TNT in Tmc+ neurons, demonstrated difficulties in emptying (Figure 2A, 2D). Notably, we encountered challenges in finding fly stocks bearing UAS>FRT-STOP-P2X2. Consequently, we opted to utilize Tmc-GAL4 to drive UAS-P2X2 instead. We believe that the results further support our hypothesis on the role of md-C in the observed behavioral change in emptying.

      Overall, this work convincingly shows that swallowing and swallowing rhythms are dependent on several mechanosensory genes. Qin et al. also characterize a candidate neuron, md-C, that is likely to provide mechanosensory feedback to pumping motor neurons, but the results they present here are not sufficient to assign this function to md-C alone. This work will have a positive impact on the field by demonstrating the importance of mechanosensory feedback to swallowing rhythms and providing a potential entry point for future investigation of the identity and mechanisms of swallowing central pattern generators.

      Reviewer #2 (Public Review):

      In this manuscript, the authors describe the role of cibarial mechanosensory neurons in fly ingestion. They demonstrate that pumping of the cibarium is subtly disrupted in mutants for piezo, TMC, and nomp-C. Evidence is presented that these three genes are co-expressed in a set of cibarial mechanosensory neurons named md-C. Silencing of md-C neurons results in disrupted cibarial emptying, while activation promotes faster pumping and/or difficulty filling. GRASP and chemogenetic activation of the md-C neurons is used to argue that they may be directly connected to motor neurons that control cibarial emptying.

      The manuscript makes several convincing and useful contributions. First, identifying the md-C neurons and demonstrating their essential role for cibarium emptying provides reagents for further studying this circuit and also demonstrates the important of mechanosensation in driving pumping rhythms in the pharynx. Second, the suggestion that these mechanosensory neurons are directly connected to motor neurons controlling pumping stands in contrast to other sensory circuits identified in fly feeding and is an interesting idea that can be more rigorously tested in the future.

      At the same time, there are several shortcomings that limit the scope of the paper and the confidence in some claims. These include:

      a) the MN-LexA lines used for GRASP experiments are not characterized in any other way to demonstrate specificity. These were generated for this study using Phack methods, and their expression should be shown to be specific for MN11 and MN12 in order to interpret the GRASP experiments.

      Thanks for the suggestion. We have checked the expression pattern of MN-LexA, which is similar to MN-GAL4 used in previous work (Manzo et al., PNAS., 2012, PMID:22474379) . Here is the expression pattern:

      Author response image 3.

      b) There is also insufficient detail for the P2X2 experiment to evaluate its results. Is this an in vivo or ex vivo prep? Is ATP added to the brain, or ingested? If it is ingested, how is ATP coming into contact with md-C neuron if it is not a chemosensory neuron and therefore not exposed to the contents of the cibarium?

      The P2X2 experimental preparation was done ex vivo. We immersed the fly in the imaging buffer, as described in the Methods section under Functional Imaging. Following dissection and identification of the subesophageal zone (SEZ) area under fluorescent microscopy, we introduced ATP slowly into the buffer, positioned at a distance from the brain

      c) In Figure 3C, the authors claim that ablating the labellum will remove the optogenetic stimulation of the md-L neuron (mechanosensory neuron of the labellum), but this manipulation would presumably leave an intact md-L axon that would still be capable of being optogenetically activated by Chrimson.

      Please refer to the corresponding answers for reviewer 1 and Figure 3—figure supplement 1.

      d) Average GCaMP traces are not shown for md-C during ingestion, and therefore it is impossible to gauge the dynamics of md-C neuron activation during swallowing. Seeing activation with a similar frequency to pumping would support the suggested role for these neurons, although GCaMP6s may be too slow for these purposes.

      Profiling the dynamics of md-C neuron activation during swallowing is crucial for unraveling the operational model of md-C and validating our proposed hypothesis. Unfortunately, our assay faces challenges in detecting probable 6Hz fluorescent changes with GCaMP6s.

      In general, we observed an increase of fluorescent signals during swallowing, but movement of alive flies during swallowing influenced the imaging recording, so we could not depict a decent tracing for calcium imaging for md-C neurons. To enhance the robustness of our findings, patching the md-C neurons would be a more convincing approach. As illustrated in Figure 2, the somata of md-C neurons are situated in the cibarium rather than the brain. patching of the md-C neuron somata in flies during ingestion is difficult.

      e) The negative result in Figure 4K that is meant to rule out taste stimulation of md-C is not useful without a positive control for pharyngeal taste neuron activation in this same preparation.

      We followed methods used in the previous work (Chen et al., Cell Rep., 2019, PMID:31644916), which we believe could confirm that md-C do not respond to sugars.

      In addition to the experimental limitations described above, the manuscript could be organized in a way that is easier to read (for example, not jumping back and forth in figure order).

      Thanks for your suggestion and the manuscript has been reorganized.

      Reviewer #3 (Public Review):

      Swallowing is an essential daily activity for survival, and pharyngo-laryngeal sensory function is critical for safe swallowing. In Drosophila, it has been reported that the mechanical property of food (e.g. Viscosity) can modulate swallowing. However, how mechanical expansion of the pharynx or fluid content sense and control swallowing was elusive. Qin et al. showed that a group of pharyngeal mechanosensory neurons, as well as mechanosensory channels (nompC, Tmc, and Piezo), respond to these mechanical forces for regulation of swallowing in Drosophila melanogaster.

      Strengths:

      There are many reports on the effect of chemical properties of foods on feeding in fruit flies, but only limited studies reported how physical properties of food affect feeding especially pharyngeal mechanosensory neurons. First, they found that mechanosensory mutants, including nompC, Tmc, and Piezo, showed impaired swallowing, mainly the emptying process. Next, they identified cibarium multidendritic mechanosensory neurons (md-C) are responsible for controlling swallowing by regulating motor neuron (MN) 12 and 11, which control filling and emptying, respectively.

      Weaknesses:

      While the involvement of md-C and mechanosensory channels in controlling swallowing is convincing, it is not yet clear which stimuli activate md-C. Can it be an expansion of cibarium or food viscosity, or both? In addition, if rhythmic and coordinated contraction of muscles 11 and 12 is essential for swallowing, how can simultaneous activation of MN 11 and 12 by md-C achieve this? Finally, previous reports showed that food viscosity mainly affects the filling rather than the emptying process, which seems different from their finding.

      We have confirmed that swallowing sucrose water solution activated md-C neurons, while sucrose water solution alone could not (Figure 4J-K). We hypothesized that the viscosity of the food might influence this expansion process.

      While we were unable to delineate the activation dynamics of md-C neurons, our proposal posits that these neurons could be activated in a single pump cycle, sequentially stimulating MN12 and MN11. Another possibility is that the activation of md-C neurons acts as a switch, altering the oscillation pattern of the swallowing central pattern generator (CPG) from a resting state to a working state.

      In the experiments with w1118 flies fed with MC (methylcellulose) water, we observed that viscosity predominantly affects the filling process rather than the emptying process, consistent with previous findings. This raises an intriguing question. Our investigation into the mutation of mechanosensitive ion channels revealed a significant impact on the emptying process. We believe this is due to the loss of mechanosensation affecting the vibration of swallowing circuits, thereby influencing both the emptying and filling processes. In contrast, viscosity appears to make it more challenging for the fly to fill the cibarium with food, primarily attributable to the inherent properties of the food itself.

      Reviewer #4 (Public Review):

      A combination of optogenetic behavioral experiments and functional imaging are employed to identify the role of mechanosensory neurons in food swallowing in adult Drosophila. While some of the findings are intriguing and the overall goal of mapping a sensory to motor circuit for this rhythmic movement are admirable, the data presented could be improved.

      The circuit proposed (and supported by GRASP contact data) shows these multi-dendritic neurons connecting to pharyngeal motor neurons. This is pretty direct - there is no evidence that they affect the hypothetical central pattern generator - just the execution of its rhythm. The optogenetic activation and inhibition experiments are constitutive, not patterned light, and they seem to disrupt the timing of pumping, not impose a new one. A slight slowing of the rhythm is not consistent with the proposed function.

      Motor neurons implicated in patterned motions can be considered effectors of Central Pattern Generators (CPGs)(Marder et al., Curr Biol., 2001, PMID: 11728329; Hurkey et al., Nature., 2023, PMID:37225999). Given our observation of the connection between md-C neurons and motor neurons, it is reasonable to speculate that md-C neurons influence CPGs. Compared to the patterned light (0.1s light on and 0.1s light off) used in our optogenetic experiments, we noted no significant changes in their responses to continuous light stimulation. We think that optogenetic methods may lead to overstimulation of md-C neurons, failing to accurately mimic the expansion of the cibarium during feeding.

      Dysfunction in mechanosensitive ion channels or mechanosensory neurons not only disrupts the timing of pumping but also results in decreased intake efficiency (Figure 1E). The water-swallowing rhythm is generally stable in flies, and swallowing is a vital process that may involve redundant ion channels to ensure its stability.

      The mechanosensory channel mutants nompC, piezo, and TMC have a range of defects. The role of these channels in swallowing may not be sufficiently specific to support the interpretation presented. Their other defects are not described here and their overall locomotor function is not measured. If the flies have trouble consuming sufficient food throughout their development, how healthy are they at the time of assay? The level of starvation or water deprivation can affect different properties of feeding - meal size and frequency. There is no description of how starvation state was standardized or measured in these experiments.

      Defects in mechanosensory channel mutants nompC, piezo, and TMC, have been extensively investigated (Hehlert et al., Trends Neurosci., 2021, PMID:332570000). Mutations in these channels exhibit multifaceted effects, as illustrated in our RNAi experiments (see Figure 2E). Deprivation of water and food was performed in empty fly vials. It's important to note that the duration of starvation determines the fly's willingness to feed but not the pump frequency (Manzo et al., PNAS., 2012, PMID:22474379).

      In most cases, female flies were deprived water and food in empty vials for 24 hours because after that most flies would be willing to drink water. The deprivation time is 12 hours for flies with nompC and Tmc mutated or flies with Kir2.1 expressed in md-C neurons, as some of these flies cannot survive 24h deprivation.

      The brain is likely to move considerably during swallow, so the GCaMP signal change may be a motion artifact. Sometimes this can be calculated by comparing GCaMP signal to that of a co-expressed fluorescent protein, but there is no mention that this is done here. Therefore, the GCaMP data cannot be interpreted.

      We did not co-express a fluorescent protein with GCaMP for md-C. The head of the fly was mounted onto a glass slide, and we did not observe significant signal changes before feeding.

      Recommendations for the authors:

      Reviewer #1 (Recommendations For The Authors):

      .>Abstract: I disagree that swallow is the first step of ingestion. The first paragraph also mentions the final checkpoint before food ingestion. Perhaps sufficient to say that swallow is a critical step of ingestion.

      Indeed, it is not rigorous enough to say “first step”. This has been replaced by “early step”.

      Introduction:

      Line 59: "Silence" should be "Silencing"

      This has been replaced.

      Results:

      Lines 91-92: I am not clear about what this means. 20% of nompC and 20% of wild-type flies exhibit incomplete filling? So nompC is not different from wild-type?

      Sorry for the mistake. Viscous foods led to incomplete emptying (not incomplete filling), as displayed in Video 4. The swallowing behavior differs between nompC mutants and wild-type flies, as illustrated in Figure 1C, Figure 1—figure supplement 1A-C and video 1&5.

      When fed with 1% MC water solution (Figure 1—figure supplement 1E-H). We found that when fed with 1% MC watere solution, Tmc or piezo mutants displayed incomplete emptying, which could constitute a long time proportion of swallowing behavior; while only 20% of nompC flies and 20% of wild-type flies sporadically exhibit incomplete emptying, which is significantly different. Though the percent of flies displaying incomplete pump is similar between nompC mutant and wild-type files, you can find it quite different in video 1 and 5.

      Line 94: Should read: “while for foods with certain viscosity, the pump of Tmc or piezo mutants might"

      What evidence is there for weakened muscle motion? The phenotypes of all three mutants is quite similar, so concluding that they have roles in initiation versus swallowing strength is not well supported -this would be better moved to the discussion since it is speculative.

      Muscles are responsible for pumping the bolus from the mouth to the crop. In the case of Tmc or piezo mutants, as evidenced by incomplete filling for viscous foods (see Video 4), we speculate that the loss of sensory stimuli leads to inadequate muscle contraction. The phenotypes observed in Tmc and piezo mutants are similar yet distinct from those of the wild-type or nompC mutant, as shown in Video 1 and 4. The phrase "due to weakened muscle motion" has been removed for clarity.

      Line 146: If md-L neurons are also labeled by this intersection, then you are not able to know whether the axons seen in the brain are from md-L or md-C neurons. Line 148: cutting the labellum is not sufficient to ablate md-L neurons. The projections will still enter the brain and can be activated with optogenetics, even after severing the processes that reside in the labellum.

      Please refer to the responses for reviewer #1 (Public Review):” A major weakness of the paper…” and Figure 4.

      Line 162: If the fly head alone is in saline, do you know that the sucrose enters the esophagus? The more relevant question here is whether the md-C neurons respond to mechanical force. If you could artificially inflate the cibarium with air and see the md-C neurons respond that would be a more convincing result. So far you only know that these are activated during ingestion, but have not shown that they are activated specifically by filling or emptying. In addition, you are not only imaging md-C (md-L is also labeled). This caveat should be mentioned.

      We followed the methods outlined in the previous work (Chen et al., Cell Rep., 2019, PMID:31644916), which suggested that md-C neurons do not respond to sugars. While we aimed to mechanically stimulate md-C neurons, detecting signal changes during different steps of swallowing is challenging. This aspect could be further investigated in subsequent research with the application of adequate patch recording or two-photon microscopy (TPM).

      Figure 3: It is not clear what the pie charts in Figure 3 A refer to. What are the three different rows, and what does blue versus red indicate?

      Figure 3A illustrates three distinct states driven by CsChrimson light stimulation of md-C neurons, with the proportions of flies exhibiting each state. During light activation, flies may display difficulty in filling, incomplete filling, or a normal range of pumping. The blue and red bars represent the proportions of flies showing the corresponding state, as indicated by the black line.

      Figure 4: Where are the example traces for J? The comparison in K should be average dF/F before ingestion compared with average dF/F during ingestion. Comparing the in vitro response to sucrose to the in vivo response during ingestion is not a useful comparison.

      Please refer to the answers for reviewer #2 question d).

      Reviewer #2 (Recommendations For The Authors):

      Suggested experiments that would address some of my concerns listed in the public review include:

      a) high resolution SEZ images of MN-LexA lines crossed to LexAop-GFP to demonstrate their specificity

      b) more detail on the P2X2 experiment. It is hard to make suggestions beyond that without first seeing the details.

      c) presenting average GCaMP traces for all calcium imaging results

      d) to rule out taste stimulation of md-C (Figure 4K) I would suggest performing more extensive calcium imaging experiments with different stimuli. For example, sugar, water, and increasing concentrations of a neutral osmolyte (e.g. PEG) to suppress the water response. I think that this is more feasible than trying to get an in vitro taste prep to be convincing.

      Please refer to the responses for public review of reviewer #2.

      Reviewer #3 (Recommendations For The Authors):

      Below I list my suggestions as well as criticisms.

      (1) It would be excellent if the authors could demonstrate whether varying levels of food viscosity affect md-C activation.

      That is a good point, and could be studied in future work.

      (2) It is not clear whether an intersectional approach using TMC-GAL4 and nompC-QF abolishes labelling of the labellar multidendritic neurons. If this is the case, please show labellar multidendritic neurons in TMC-GAL4 only flies and flies using the intersectional approach. Along with this question, I am concerned that labellum-removed flies could be used for feeding assay.

      Intersectional labelling using TMC-GAL4 and nompC-QF could not abolish labelling of the labellar multidendritic neurons (Author response image 4). Labellum-removed flies could be used for feeding assay (Figure 3—figure supplement 1B-C, video 5), but once LSO or cibarium of fly was damaged, swallowing behavior would be affected. Removing labellum should be very careful.

      Author response image 4.

      (3) Please provide the detailed methods for GRASP and include proper control.

      Please refer to the responses for public review of reviewer #1.

      (4) The authors hypothesized that md-C sequentially activates MN11 and 12. Is the time gap between applying ATP on md-C and activation of MN11 or MN12 different? Please refer to the responses for public review of reviewer #3. The time gap between applying ATP on md-C and activation of MN11 or MN12 didn’t show significant differences, and we think the reason is that the ex vivo conditions could not completely mimic in vivo process.

      I found the manuscript includes many errors, which need to be corrected.

      (1) The reference formatting needs to be rechecked, for example, lines 37, 42, and 43.

      (2) Line 44-46: There is some misunderstanding. The role of pharyngeal mechanosensory neurons is not known compared with chemosensory neurons.

      (3) Line 49: Please specify which type of quality of food. Chemical or physical?

      (4) Line 80 and Figure 1B-D Authors need to put filling and emptying time data in the main figure rather than in the supplementary figure. Otherwise, please cite the relevant figures in the text(S1A-C).

      (5) Line 84-85; Is "the mutant animals" indicating only nompC? Please specify it.

      (6) Figure 1a: It is hard to determine the difference between the series of images. And also label filling and emptying under the time.

      (7) S1E-H: It is unclear what "Time proportion of incomplete pump" means. Please define it.

      (8) Please reorganize the figures to follow the order of the text, for example, figures 2 and 4

      (9) Figure 4A. There is mislabelling in Figure 4A. It is supposed to be phalloidin not nc82.

      (10) Figure 4K: It does not match the figure legend and main text.

      (11) Figure 4D and G: Please indicate ATP application time point.

      Thanks for your correction and all the points mentioned were revised.

      Reviewer #4 (Recommendations For The Authors):

      The figures need improvement. 1A has tiny circles showing pharynx and any differences are unclear.

      The expression pattern of some of these drivers (Supplement) seems quite broad. The tmc nompC intersection image in Figure 1F is nice but the cibarium images are hard to interpret: does this one show muscle expression? What are "brain" motor neurons? Where are the labellar multi-dendritic neurons?

      Tmc nompC intersection image show no expression in muscles. Somata of motor neurons 12 or 11 situated at SEZ area of brain, while somata of md-C neurons are in the cibarium. Image of md-L neurons was posted in response for reviewer #3 (Recommendations For The Authors):

      Why do the assays alternate between swallowing food and swallowing water?

      Thank for your suggestion, figure 1A has been zoomed-in. The Tmc nompC intersection image in Figure 2F displayed the position of md-C neurons in a ventral perspective, and muscles were not labelled. We stained muscles in cibarium by phalloidin and the image is illustrated in Figure 4A, while we didn’t find overlap between md-C neurons and muscles. Image of md-L neurons were posted as Author response image 4.

      In the majority of our experiments, we employed water to test swallowing behavior, while we used methylcellulose water solution to test swallowing behavior of mechanoreceptor mutants, and sucrose solution for flies with md-C neurons expressing GCaMP since they hardly drank water when their head capsules were open.

      How starved or water-deprived were the flies?

      One day prior to the behavioral assays, flies were transferred to empty vials (without water or food) for 24 hours for water deprivation. Flies who could not survive 24h deprivation would be deprived for 12h.

      How exactly was the pumping frequency (shown in Fig 1B) measured? There is no description in the methods at all. If the pump frequency is scored by changes in blue food intensity (arbitrary units?), this seems very subjective and maybe image angle dependent. What was camera frame rate? Can it capture this pumping speed adequately? Given the wealth of more quantitative methods for measuring food intake (eg. CAFE, flyPAD), it seems that better data could be obtained.

      How was the total volume of the cibarium measured? What do the pie charts in Figure 3A represent?

      The pump frequency was computed as the number of pumps divided by the time scale, following the methodology outlined in Manzo et al., 2012. Swallowing curves were plotted using the inverse of the blue food intensity in the cibarium. In this representation, ascending lines signify filling, while descending lines indicate emptying (see Figure 2D, 3B). We maintain objectivity in our approach since, during the recording of swallowing behavior, the fly was fixed, and we exclusively used data for analysis when the Region of Interest (ROI) was in the cibarium. This ensures that the intensity values accurately reflect the filling and emptying processes. Furthermore, we conducted manual frame-by-frame checks of pump frequency, and the results align with those generated by the time series analyzer V3 of ImageJ.

      For the assessment of total volume of ingestion, we referred the methods of CAFE, utilizing a measurable glass capillary. We then calculated the ingestion rate (nL/s) by dividing the total volume of ingestion by the feeding time.

      The changes seem small, in spite of the claim of statistical significance.

      The observed stability in pump frequency within a given genotype underscores the significance of even seemingly small changes, which is statistically significant. We speculate that the stability in swallowing frequency suggests the existence of a redundant mechanism to ensure the robustness of the process. Disruption of one channel might potentially be partially compensated for by others, highlighting the vital nature of the swallowing mechanism.

      How is this change in pump frequency consistent with defects in one aspect of the cycle - either ingestion (activation) or expulsion (inhibition)?

      Please refer to Figure 2, 3. Both filling and emptying process were affects, while inhibition mainly influences emptying time (Figure 1—figure supplement 1).

      for the authors:

      Line 48: extensively

      Line 62 - undiscovered.

      Line 107, 463: multi

      Line 124: What is "dysphagia?" This is an unusual word and should be defined.

      Line 446: severe

      Line 466: in the cibarium or not?

      Thanks for your correction and all the places mentioned were revised.

    1. Author Response

      The following is the authors’ response to the original reviews.

      Thank you for organizing the reviews for our manuscript: Behavioral entrainment to rhythmic auditory stimulation can be modulated by tACS depending on the electrical stimulation field properties,” and for the positive eLife assessment. We also thank the reviewers for their constructive comments. We have addressed every comment, which has helped to improve the transparency and readability of the manuscript. The main changes to the manuscript are summarized as follows:

      1. Surrogate distributions were created for each participant and session to estimate the effect of tACS-phase lag on behavioral entrainment to the sound that could have occurred by chance or because of our analysis method (R1). The actual tACS-amplitude effects were normalized relative to the surrogate distribution, and statistical analysis was performed on the normalized (z-score) values. This analysis did not change our main outcome: that tACS modulates behavioral entrainment to the sound depending on the phase lag between the auditory and the electrical signals. This analysis has now been incorporated into the Results section and in Fig. 3c-d.

      2. Two additional supplemental figures were created to include the single-participant data related to Fig. 3b and 3e (R2).

      3. Additional editing of the manuscript has been performed to improve the readability.

      Below, you will find a point-by-point response to the reviewers’ comments.

      Reviewer #1 (Public Review):

      We are grateful for the reviewer’s positive assessment of the potential impact of our study. The reviewer’s primary concerns were 1) the tACS lag effects reported in the manuscript might be noise because of the realignment procedure, and 2) no multiple comparisons correction was conducted in the model comparison procedure.

      In response to point 1), we have reanalyzed the data in exactly the manner prescribed by the reviewer. Our effects remain, and the new control analysis strengthens the manuscript. 2) In the context of model comparison, the model selection procedure was not based on evaluating the statistical significance of any model or predictor. Instead, the single model that best fit the data was selected as the model with the lowest Akaike’s information criterion (AIC), and its superiority relative to the second-best model was corroborated using the likelihood ratio test. Only the best model was evaluated for significance and analyzed in terms of its predictors and interactions. This model is an omnibus test and does not require multiple comparison correction unless there are posthoc decompositions. For similar approaches, see (Kasten et al., 2019).

      Below, we have responded to each comment specifically or referred to this general comment.

      Summary of what the authors were trying to achieve.

      This paper studies the possible effects of tACS on the detection of silence gaps in an FM-modulated noise stimulus. Both FM modulation of the sound and the tACS are at 2Hz, and the phase of the two is varied to determine possible interactions between the auditory and electric stimulation. Additionally, two different electrode montages are used to determine if variation in electric field distribution across the brain may be related to the effects of tACS on behavioral performance in individual subjects.

      Major strengths and weaknesses of the methods and results.

      The study appears to be well-powered to detect modulation of behavioral performance with N=42 subjects. There is a clear and reproducible modulation of behavioral effects with the phase of the FM sound modulation. The study was also well designed, combining fMRI, current flow modeling, montage optimization targeting, and behavioral analysis. A particular merit of this study is to have repeated the sessions for most subjects in order to test repeat-reliability, which is so often missing in human experiments. The results and methods are generally well-described and well-conceived. The portion of the analysis related to behavior alone is excellent. The analysis of the tACS results is also generally well described, candidly highlighting how variable results are across subjects and sessions. The figures are all of high quality and clear. One weakness of the experimental design is that no effort was made to control for sensation effects. tACS at 2Hz causes prominent skin sensations which could have interacted with auditory perception and thus, detection performance.

      The reviewer is right that we did not control for the sensation effects in our paradigm. We asked the participants to rate the strength of the perceived stimulation after each run. However, this information was used only to assess the safety and tolerability of the stimulation protocol. Nevertheless, we did not consider controlling for skin sensations necessary given the within-participant nature of our design (all participants experienced all six tACS–audio phase lag conditions, which were identical in their potential to cause physical sensations; the only difference between conditions was related to the timing of the auditory stimulus). That is, while the reviewer is right that 2-Hz tACS can indeed induce skin sensation under the electrodes, in this study, we report the effects that depend on the tACS-phase lag relative to the FM-stimulus. Note that the starting phase of the FM-stimulus was randomized across trials within each block (all six tACS audio lags were presented in each block of stimulation). We have no reason to expect the skin sensation to change with the tACS-audio lag from trial to trial, and therefore do not consider this to be a confound in our design. We have added some sentences with this information to the Discussion section:

      Pages 16-17, lines 497-504: “Note that we did not control for the skin sensation induced by 2-Hz tACS in this experiment. Participants rated the strength of the perceived stimulation after each run. However, this information was used only to assess the safety and tolerability of the stimulation protocol. It is in principle possible that skin sensation would depend on tACS phase itself. However, in this study, we report effects that depend on the relationship between tACS-phase and FM-stimulus phase, which changed from trial to trial as the starting phase of the FM-stimulus was randomized across trials. We have no reason to expect the skin sensation to change with the tACS-audio lag and therefore do not consider this to be a confound in our data.”

      Appraisal of whether the authors achieved their aims, and whether the results support their conclusions.

      Unfortunately, the main effects described for tACS are encumbered by a lack of clarity in the analysis. It does appear that the tACS effects reported here could be an artifact of the analysis approach. Without further clarification, the main findings on the tACS effects may not be supported by the data.

      Likely impact of the work on the field, and the utility of the methods and data to the community.

      The central claim is that tACS modulates behavioral detection performance across the 0.5s cycle of stimulation. However, neither the phase nor the strength of this effect reproduces across subjects or sessions. Some of these individual variations may be explainable by individual current distribution. If these results hold, they could be of interest to investigators in the tACS field.

      The additional context you think would help readers interpret or understand the significance of the work.

      The following are more detailed comments on specific sections of the paper, including details on the concerns with the statistical analysis of the tACS effects.

      The introduction is well-balanced, discussing the promise and limitations of previous results with tACS. The objectives are well-defined.

      The analysis surrounding behavioral performance and its dependence on the phase of the FM modulation (Figure 3) is masterfully executed and explained. It appears that it reproduces previous studies and points to a very robust behavioral task that may be of use in other studies.

      Again, we would like to thank the reviewer for the positive assessment of the potential impact of our work and for the thoughtful comments regarding the methodology. For readability in our responses, we have numbered the comments below.

      1. There is a definition of tACS(+) vs tACS(-) based on the relative phase of tACS that may be problematic for the subsequent analysis of Figures 4 and 5. It seems that phase 0 is adjusted to each subject/session. For argument's sake, let's assume the curves in Fig. 3E are random fluctuations. Then aligning them to best-fitting cosine will trivially generate a FM-amplitude fluctuation with cosine shape as shown in Fig. 4a. Selecting the positive and negative phase of that will trivially be larger and smaller than a sham, respectively, as shown in Fig 4b. If this is correct, and the authors would like to keep this way of showing results, then one would need to demonstrate that this difference is larger than expected by chance. Perhaps one could randomize the 6 phase bins in each subject/session and execute the same process (fit a cosine to curves 3e, realign as in 4a, and summarize as in 4b). That will give a distribution under the Null, which may be used to determine if the contrast currently shown in 4b is indeed statistically significant.

      We agree with the reviewer’s concerns regarding the possible bias induced by the realignment procedure used to estimate tACS effects. Certainly, when adjusting phase 0 to each participant/session’s best tACS phase (peak in the fitting cosine), selecting the positive phase of the realigned data will be trivially larger than sham (Fig. 4a). This is why the realigned zero-phase and opposite phase (trough) bins were excluded from the analysis in Fig. 4b. Therefore, tACS(+) vs. tACS(-) do not represent behavioral entrainment at the peak positive and negative tACS lags, as both bins were already removed from the analysis. tACS(+) and tACS(-) are the averages of two adjacent bins from the positive and negative tACS lags, respectively (Zoefel et al., 2019). Such an analysis relies on the idea that if the effect of tACS is sinusoidal, presenting the auditory stimulus at the positive half cycle should be different than when the auditory stimulus lags the electrical signal by the other half. If the effect of tACS was just random noise fluctuations, there is no reason to assume that such fluctuations would be sinusoidal; therefore, any bias in estimating the effect of tACS should be removed when excluding the peak to which the individual data were realigned. Similar analytical procedures have been used previously in the literature (Riecke et al., 2015; Riecke et al., 2018). We have modified the colors in Fig. 4a and 4c (former 4b) and added a new panel to the figure (new 4b) to make the realignment procedure, including the exclusion of the realigned peak and trough data, more visually obvious.

      Moreover, we very much like the reviewer’s suggestion to normalize the magnitude of the tACS effect using a permutation strategy. We performed additional analyses to normalize our tACS effect in Fig. 4c by the probability of obtaining the effect by chance. For each subject and session, tACS-phase lags were randomized across trials for a total of 1000 iterations. For each iteration, the gaps were binned by the FM-stimulus phase and tACS-lag. For each tACS-lag, the amplitude of behavioral entrainment to the FM-stimulus was estimated (FM-amplitude), as shown in Fig. 3. Similar to the original data, a second cosine fit was estimated for the FM-amplitude by tACS-lag. Optimal tACS-phase was estimated from the cosine fit and FM-amplitude values were realigned. Again, the realigned phase 0 and trough were removed from the analysis, and their adjacent bins were averaged to obtain the FM-amplitude at tACS(+) and tACS(−), as shown in Fig. 4c. We then computed the difference between 1) tACS(+) and sham, 2) tACS(-) and sham, and 3) tACS(+) and tACS (-), for the original data and the permuted datasets. This procedure was performed for each participant and session to estimate the size of the tACS effect for the original and surrogate data. The original tACS effects were transformed to z-scores using surrogate distributions, providing us with an estimate of the size of the real effect relative to chance. We then computed one-sample t-tests to compare whether the effects of tACS were statistically significant. In fact, this analysis showed that the tACS effects were still statistically significant. This analysis has been added to the Results and Methods sections and is included in Figure 4d.

      Page 10, lines 282-297: “In order to further investigate whether the observed tACS effect was significantly larger than chance and not an artifact of our analysis procedure (33), we created 1000 surrogate datasets per participant and session by permuting the tACS lag designation across trials. The same binning procedure, realignment, and cosine fits were applied to each surrogate dataset as for the original data. This yielded a surrogate distribution of tACS(+) and tACS(-) values for each participant and session. These values were averaged across sessions since the original analysis did not show a main effect of session. We then computed the difference between tACS(+) and sham, tACS(-) and sham, and tACS(+) and tACS(-), separately for the original and surrogate datasets. The obtained difference for the original data where then z-scored using the mean and standard deviation of the surrogate distribution. Note that in this case we used data of all 42 participants who had at least one valid session (37 participants with both sessions). Three one-sample t-tests were conducted to investigate whether the size of the tACS effect obtained in the original data was significantly larger than that obtained by chance (Fig. 4d). This analysis showed that all z-scores were significantly higher than zero (all t(41) > 2.36, p < 0.05, all p-values corrected for multiple comparisons using the Holm-Bonferroni method).”

      Page 31, lines 962-972: “To further control that the observed tACS effects were not an artifact of the analysis procedure, the difference between the tACS conditions (sham, tACS(+), and tACS(-)) were normalized using a permutation approach. For each participant and session, 1000 surrogate datasets were created by permuting the tACS lag designation across trials. The same binning procedure, realignment, and cosine fits were applied to each surrogate dataset as for the original data (see above). FM-amplitude at sham, tACS(+) and tACS(-) were averaged across sessions since the original analysis did not show a main effect of session. Difference between tACS conditions were estimated for the original and surrogate datasets and the resulting values from the original data were z-scored using the mean and standard deviation from the surrogate distributions. One-sample t-tests were conducted to test the statistical significance of the z-scores. P-values were corrected for multiple comparisons using the Holm-Bonferroni method.”

      1. Results of Fig 5a and 5b seem consistent with the concern raised above about the results of Fig. 4. It appears we are looking at an artifact of the realignment procedure, on otherwise random noise. In fact, the drop in "tACS-amplitude" in Fig. 5c is entirely consistent with a random noise effect.

      Please see our response to the comment above.

      1. To better understand what factors might be influencing inter-session variability in tACS effects, we estimated multiple linear models ..." this post hoc analysis does not seem to have been corrected for multiple comparisons of these "multiple linear models". It is not clear how many different things were tried. The fact that one of them has a p-value of 0.007 for some factors with amplitude-difference, but these factors did not play a role in the amplitude-phase, suggests again that we are not looking at a lawful behavior in these data.

      We suspect that the reviewer did not have access to the supplemental materials where all tables (relevant here is Table S3) are provided. This post hoc analysis was performed as an exploratory analysis to better understand the factors that could influence the inter-session variability of tACS effects. In Table S3, we provide the formula for each of the seven models tested, including their Akaike information criteria corrected for small samples (AICc), R2, F, and p-values. As described in the methods section, the winning model was selected as the model with the smallest AICc. A similar procedure has been previously used in the literature (Kasten et al., 2019). Moreover, to ensure that our winning model was better at explaining the data than the second-best unrestricted model, we used the likelihood ratio test. After choosing the winning model and before reporting the significance of the predictors, we examined the significance of the model in and of itself, taking into account its R2 as well as F- and p-values relative to a constant model. Thus, only one model is being evaluated in terms of statistical significance. Therefore, to our understanding, there are no multiple comparisons to correct for. We added the information regarding the selection procedure, hoping this will make the analysis clearer.

      See page 12, lines 354-360: “This model was selected because it had the smallest Akaike’s information criterion (corrected for small samples), AICc. Moreover, the likelihood ratio test showed no evidence for choosing the more complex unrestricted model (stat = 2.411, p = 0.121). Following the same selection criteria, the winning model predicting inter-session variability in tACS-phase, included only the factor gender (Table S4). However, this model was not significant in and of itself when compared to a constant model (F-statistic vs. constant model: 3.05, p = 0.09, R2 = 0.082).”

      1. "So far, our results demonstrate that FM-stimulus driven behavioral modulation of gap detection (FM-amplitude) was significantly affected by the phase lag between the FM-stimulus and the tACS signal (Audio-tACS lag) ..." There appears to be nothing in the preceding section (Figures 4 and 5) to show that the modulation seen in 3e is not just noise. Maybe something can be said about 3b on an individual subject/session basis that makes these results statistically significant on their own. Maybe these modulations are strong and statistically significant, but just not reproducible across subjects and sessions?

      Please see our response to the first comment regarding the validity of our analysis for proving the significant effect of tACS lag on modulating behavioral entrainment to the FM-stimulus (FM-amplitude), and the new control analysis. After performing the permutation tests, to make sure the reported effects are not noise, our statistical analysis still shows that tACS-lag does significantly modulate behavioral entrainment to the sound (FM-amplitude). Thus, the reviewer is right to say “these modulations are strong and statistically significant, just not reproducible across subjects and sessions”. In this regard, we consider our evaluation of session-to-session reliability of tACS effects is of high relevance for the field, as this is often overlooked in the literature.

      1. "Inter-individual variability in the simulated E-field predicts tACS effects" Authors here are attempting to predict a property of the subjects that was just shown to not be a reliable property of the subject. Authors are picking 9 possible features for this, testing 33 possible models with N=34 data points. With these circumstances, it is not hard to find something that correlates by chance. And some of the models tested had interaction terms, possibly further increasing the number of comparisons. The results reported in this section do not seem to be robust, unless all this was corrected for multiple comparisons, and it was not made clear?

      We thank the reviewer very much for this comment. While the reviewer is right that in these models, we are trying to predict an individual property (tACS-amplitude) that was not test–retest reliable across sessions, we still consider this to be a valid analysis. Here, we take the tACS-amplitude averaged across sessions, trying to predict the probability of a participant to be significantly modulated by tACS, in general, regardless of day-to-day variability. Regarding the number of multiple regression models, how we chose the winning model and the appropriateness/need of multiple-comparisons correction in this case, please see our explanation under “Reviewer 1 (Public review)” and our response to comment 3.

      1. "Can we reduce inter-individual variability in tACS effects ..." This section seems even more speculative and with mixed results.

      We agree with the reviewer that this section is a bit speculative. We are trying to plant some seeds for future research can help move the field forward in the quest for better stimulation protocols. We have added a sentence at the end of the section to explicitly say that more evidence is needed in this regard.

      Page 14, lines 428-429: “At this stage, more evidence is needed to prove the superiority of individually optimized tACS montages for reducing inter-individual variability in tACS effects.”

      Given the concerns with the statistical analysis above, there are concerns about the following statements in the summary of the Discussion:

      1. "2) does modulate the amplitude of the FM-stimulus induced behavioral modulation (FM-amplitude)"

      This seems to be based on Figure 4, which leaves one with significant concerns.

      Please see response to comment 1. We hope the reviewer is satisfied with our additional analysis to make sure the effect of tACS here reported is not noise.

      1. "4) individual variability in tACS effect size was partially explained by two interactions: between the normal component of the E-field and the field focality, and between the normal component of the E-field and the distance between the peak of the electric field and the functional target ROIs."

      The complexity of this statement alone may be a good indication that this could be the result of false discovery due to multiple comparisons.

      We respectfully disagree with the reviewer’s opinion that this is a complex statement. We think that these interaction effects are very intuitive as we explain in the results and discussion sections. These significant interactions show that for tACS to be effective, it matters that current gets to the right place and not to irrelevant brain regions. We believe this finding is of great importance for the field, since most studies on the topic still focus mostly on predicting tACS effects from the absolute field strength and neglect other properties of the electric field.

      For the same reasons as stated above, the following statements in the Abstract do not appear to have adequate support in the data:

      "We observed that tACS modulated the strength of behavioral entrainment to the FM sound in a phase-lag specific manner. ... Inter-individual variability of tACS effects was best explained by the strength of the inward electric field, depending on the field focality and proximity to the target brain region. Spatially optimizing the electrode montage reduced inter-individual variability compared to a standard montage group."

      Please see response to all previous comments

      In particular, the evidence in support of the last sentence is unclear. The only finding that seems related is that "the variance test was significant only for tACS(-) in session 2". This is a very narrow result to be able to make such a general statement in the Abstract. But perhaps this can be made clearer.

      We changed this sentence in the abstract to:

      Page 2, lines 41-43: “Although additional evidence is necessary, our results also provided suggestive insights that spatially optimizing the electrode montage could be a promising tool to reduce inter-individual variability of tACS effects.”

      Reviewer #3 (Public Review):

      In "Behavioral entrainment to rhythmic auditory stimulation can be modulated by tACS depending on the electrical stimulation field properties" Cabral-Calderin and collaborators aimed to document 1) the possible advantages of personalized tACS montage over standard montage on modulating behavior; 2) the inter-individual and inter-session reliability of tACS effects on behavioral entrainment and, 3) the importance of the induced electric field properties on the inter-individual variability of tACS.

      To do so, in two different sessions, they investigated how the detection of silent gaps occurring at random phases of a 2Hz- amplitude modulated sound could be enhanced with 2Hz tACS, delivered at different phase lags. In addition, they evaluated the advantage of using spatially optimized tACS montages (information-based procedure - using anatomy and functional MRI to define the target ROI and simulation to compare to a standard montage applied to all participants) on behavioral entrainment. They first show that the optimized and the standard montages have similar spatial overlap to the target ROI. While the optimized montage induced a more focal field compared to the standard montage, the latter induced the strongest electric field. Second, they show that tACS does not modify the optimal phase for gap detection (phase of the frequency-modulated sound) but modulates the strength of behavioral entrainment to the frequency-modulated sound in a phase-lag specific manner. However, and surprisingly, they report that the optimal tACS lag, and the magnitude of the phasic tACS effect were highly variable across sessions. Finally, they report that the inter-individual variability of tACS effects can be explained by the strength of the inward electric field as a function of the field focality and on how well it reached the target ROI.

      The article is interesting and well-written, and the methods and approaches are state-of-the-art.

      Strengths:

      • The information-based approach used by the authors is very strong, notably with the definition of subject-specific targets using a fMRI localizer and the simulation of electric field strength using 3 different tACS montages (only 2 montages used for the behavioral experiment).

      • The inter-session and inter-individual variability are well documented and discussed. This article will probably guide future studies in the field.

      Weaknesses:

      • The addition of simultaneous EEG recording would have been beneficial to understand the relationship between tACS entrainment and the entrainment to rhythmic auditory stimulation.

      We are grateful for the Reviewer’s positive assessment of our work and for the reviewer’s recommendations. We agree with the reviewer that adding simultaneous EEG or MEG to our design would have been beneficial to understand tACS effects. However, as the reviewer might be familiar with, such combination also possesses additional challenges due to the strong artifacts induced by tACS in the EEG signals, which is at the frequency of interest and several orders of magnitude higher than the signal of interest. Unfortunately, the adequate setup for simultaneous tACS-EEG was not available at the moment of the study. Nevertheless, since we are using a paradigm that we have repeatedly studied in the past and have shown it entrains neural activity and modulates behavior rhythmically, we are confident our results are of interest on their own. For readability of our answers, we numbered to comments below.

      1. It would have been interesting to develop the fact that tACS did not "overwrite" neural entrainment to the auditory stimulus. The authors try to explain this effect by mentioning that "tACS is most effective at modulating oscillatory activity at the intended frequency when its power is not too high" or "tACS imposes its own rhythm on spiking activity when tACS strength is stronger than the endogenous oscillations but it decreases rhythmic spiking when tACS strength is weaker than the endogenous oscillations". However, it is relevant to note that the oscillations in their study are by definition "not endogenous" and one can interpret their results as a clear superiority of sensory entrainment over tACS entrainment. This potential superiority should be discussed, documented, and developed.

      We thank the reviewer very much for this remark. We completely agree that our results could be interpreted as a clear superiority of sensory entrainment over tACS entrainment. We have now incorporated this possibility in the discussion.

      Page 16, line 472-478: “Alternatively, our results could simply be interpreted as a clear superiority of the auditory stimulus for entrainment. In other words, sensory entrainment might just be stronger than tACS entrainment in this case where the stimulus rhythm was strong and salient. It would be interesting to further test whether this superiority of sensory entrainment applies to all sensory modalities or if there is a particular advantage for auditory stimuli when they compete with electrical stimulation. However, answering this question was beyond the scope of our study and needs further investigations with more appropriate paradigms.”

      1. The authors propose that "by applying tACS at the right lag relative to auditory rhythms, we can aid how the brain synchronizes to the sounds and in turn modulate behavior." This should be developed as the authors showed that the tACS lags are highly variable across sessions. According to their results, the optimal lag will vary for each tACS session and subtle changes in the montage could affect the effects.

      We thank the reviewer for this remark. We believe that the right procedure in this case would be using close-loop protocols where the optimal tACS-lag is estimated online as we discuss in the summary and future directions sub-section. We tried to make this clearer in the same sentence that the reviewer mentioned.

      Page 17, line 506-508: “Since optimal tACS phase was variable across participants and sessions, this approach would require closed-loop protocols where the optimal tACS lag is estimated online (see next section).”

      1. In a related vein, it would be very useful to show the data presented in Figure 3 (panels b,d,e) for all participants to allow the reader to evaluate the quality of the data (this can be added as a supplementary figure).

      Thank you very much for the suggestion. We have added two new supplemental figures (Fig S1 and S2) to show individual data for Fig. 3b and 3e. Note that Fig. 3d already shows the individual data as each circle represents optimal FM-phase for a single participant.

      Reviewer #1 (Recommendations For The Authors):

      Minor comments:

      "was optimized in SimNIBS to focus the electric field as precisely as possible at the target ROI" It appears that some form of constrained optimization was used. It would be good to clarify which method was used, including a reference.

      Indeed, SimNIBS implements a constrained optimization approach based on pre-calculated lead fields. We have added the corresponding reference. All parameters used for the optimization are reported in the methods (see sub-section Electric field simulations and montage optimization). Regarding further specifics, the readers are invited to check the MATLAB code that was used for the optimization which is made available at: https://osf.io/3yutb

      "Thus, each montage has its pros and cons, and the choice of montage will depend on which of these dependent measures is prioritized." Well put. It would be interesting to know if authors considered optimizing for intensity on target. That would give the strongest predicted intensity on target, which seems like an important desideratum. Individualizing for something focal, as expected, did not give the strongest intensity. In fact, the method struggled to achieve the desired intensity of 0.1V/m in some subjects. It would be interesting to have a discussion about why this particular optimization method was selected.

      The specific optimization method used in this study was somewhat arbitrary, as there is no standard in the field. It was validated in prior studies, where it was also demonstrated that it performs favorably compared to alternative methods (Saturnino et al., 2019; Saturnino et al., 2021). The underlying physics of the head volume conductor generally limits the maximally achievable focality, and requires a tradeoff between focality and the desired intensity in the target. This tradeoff depends on the maximal amount of current that can be injected into the electrodes due to safety limits (4 mA in total in our case). Further constraints of the optimization in our application were the simultaneous targeting of two areas, and achieving field directions in the targets roughly parallel to those of auditory dipoles. Given the combination of these constraints, as the reviewer noticed, we could not even achieve the desired intensity of .1V/m in some subjects. As we wanted to stimulate both auditory cortices equally, our priority was to have the E-fields as similar as possible between hemispheres. Future studies optimizing for only one target would be easier to optimize for target intensity (assuming the same maximal total current injection). Alternatively, relaxing the constraint on direction and optimizing only for field intensity would help to increase the field intensities in the targets, but would lead to differing field directions in the two targets. As an example, see Rev. Fig.1 below. We extensively discuss some of these points in the discussion section: “Are individually optimized tACS montage better?” (Pages 21-22).

      Additionally, we added a few sentences in the Results and Methods giving more details about the optimization approach.

      Page 5, lines 115-116: “Using individual finite element method (FEM) head models (see Methods) and the lead field-based constrained optimization approach implemented in SimNIBS (31)”

      Page 27, lines 819-822: “The optimization pipeline employed the approach described in (31) and was performed in two steps. First, a lead field matrix was created per individual using the 10-10 EEG virtual cap provided in SimNIBS and performing electric field simulations based on the default tissue conductivities listed below.”

      Author response image 1.

      E-field distributions for one example participant. Brain maps show the results from the same optimization procedure described in the main manuscript but with no constraint for the current direction (top) or constraining the current direction (bottom). Note that the desired intensity of .1 V/m can be achieved when the current direction is not constrained.

      The terminology of "high-definition HD" used here is unconventional and may confuse some readers. The paper cited for ring electrodes (18) does not refer to it as HD. A quick search for high-definition HD yields mostly papers using many small electrodes, not ring electrodes. They look more like what was called "individualized". More conventional would be to call the first configuration a "ring-electrode", and the "individualized" configuration might be called "individualized HD".

      We thank the reviewer for this remark. We changed the label of the high-definition montage to ring-electrode. Regarding the individualized configuration, we prefer not to use individualized HD as it has the same number of electrodes as the standard montage.

      "So far, we have evaluated whether tACS at different phase lags interferes with stimulus-brain synchrony and modulates behavioral signatures of entrainment" The paper does not present any data on stimulus-brain synchrony. There is only an analysis of behavior and stimulus/tACS phase.

      We agree with the reviewer. To be more careful with such statement we now modified the sentence to say:

      Page 10, lines 303-304: “So far, we have evaluated whether tACS at different phase lags modulates behavioral signatures of entrainment: FM-amplitude and FM-phase.”

      "However, the strength of the tACS effect was variable across participants." and across sessions, and the phase also was variable across subjects and sessions.

      "tACS-amplitude estimates were averaged across sessions since the session did not significantly affect FM-amplitude (Fig. 5a)." More importantly, the authors show that "tACS-amplitude" was not reproducible across sessions.

      Unfortunately, we did not understand what the reviewer is suggesting here, and would have to ask the reviewer in this case to provide us with more information.

      References

      Kasten FH, Duecker K, Maack MC, Meiser A, Herrmann CS (2019) Integrating electric field modeling and neuroimaging to explain inter-individual variability of tACS effects. Nat Commun 10:5427. Riecke L, Sack AT, Schroeder CE (2015) Endogenous Delta/Theta Sound-Brain Phase Entrainment Accelerates the Buildup of Auditory Streaming. Curr Biol 25:3196-3201.

      Riecke L, Formisano E, Sorger B, Baskent D, Gaudrain E (2018) Neural Entrainment to Speech Modulates Speech Intelligibility. Curr Biol 28:161-169 e165.

      Saturnino GB, Madsen KH, Thielscher A (2021) Optimizing the electric field strength in multiple targets for multichannel transcranial electric stimulation. J Neural Eng 18.

      Saturnino GB, Siebner HR, Thielscher A, Madsen KH (2019) Accessibility of cortical regions to focal TES: Dependence on spatial position, safety, and practical constraints. Neuroimage 203:116183.

      Zoefel B, Davis MH, Valente G, Riecke L (2019) How to test for phasic modulation of neural and behavioural responses. Neuroimage 202:116175.

    1. Author response:

      The following is the authors’ response to the original reviews

      Public Reviews:  

      Reviewer #1 (Public review):  

      Summary:  

      This work examines the binding of several phosphonate compounds to a membrane-bound pyrophosphatase using several different approaches, including crystallography, electron paramagnetic resonance spectroscopy, and functional measurements of ion pumping and pyrophosphatase activity. The work attempts to synthesize these different approaches into a model of inhibition by phosphonates in which the two subunits of the functional dimer interact differently with the phosphonate.  

      Strengths:  

      This study integrates a variety of approaches, including structural biology, spectroscopic measurements of protein dynamics, and functional measurements. Overall, data analysis was thoughtful, with careful analysis of the substrate binding sites (for example calculation of POLDOR omit maps).  

      Weaknesses:  

      Unfortunately, the protein did not crystallize with the more potent phosphonate inhibitors. Instead, structures were solved with two compounds with weak inhibitory constants >200 micromolar, which limits the molecular insight into compounds that could possibly be developed into small molecule inhibitors. Likewise, the authors choose to focus the spectroscopy experiments on these weaker binders, missing an opportunity to provide insight into the interaction between more potent binders and the protein. 

      We acknowledge the reviewer concern regarding the choice of weaker inhibitors. We attempted cocrystallization with all available inhibitors, including those with higher potency. However, despite numerous efforts, these potent inhibitors yielded low-resolution crystals, making them unsuitable for detailed structural analysis. Therefore, we chose to focus on the weaker binders, as we were able to obtain high-quality crystal structures for these compounds. This allowed us to perform DEER spectroscopy and monitor conformational TmPPase state ensembles in solution with the added advantage of accurately analysing the data against structural models derived from X-ray crystallography. Using these weaker inhibitors enabled a more precise interpretation of the DEER data, thus providing reliable insights into the conformational dynamics and inhibition mechanism. As suggested by the reviewer, in the revised version, we add new DEER experiments, conditions and analysis on two of the more potent inhibitors (alendronate and pamidronate) to provide additional insight into their interactions. Furthermore, we also implemented additional DEER data on the cytoplasmic side of TmPPase; at a new site we identified (with the advantage of being an endogenous cysteine residue) and spin labelled (C599R1), given the DEER data for the previous T211R1cytoplasmic site were difficult to interpret owing to the highly dynamic nature of this region. The new pair C599R1 yielded high-quality DEER traces and indicated more clearly than T211R1, distance distributions consistent with asymmetry across the sampled conditions.  Again, as suggested by the reviewer, alendronate and pamidronate DEER measurements were also recorded for this site (cytoplasmic side; C599R1) as well as the periplasmic side (525R1).

      In general, the manuscript falls short of providing any major new insight into membrane-bound pyrophosphatases, which are a very well-studied system. Subtle changes in the structures and ensemble distance distributions suggest that the molecular conformations might change a little bit under different conditions, but this isn't a very surprising outcome. It's not clear whether these changes are functionally important, or just part of the normal experimental/protein ensemble variation. 

      We respectfully disagree with the reviewer. The scale of motions particularly seen in solution (and now on a new reliable spin pair (C599R1) located on the cytoplasmic side) correspond to those seen in the full panoply of crystal structures of mPPases. Some proteins undergo very large conformational changes during catalysis – such as the rotary ATPase. This one does not, meaning that the precise motions we describe here are relevant and observed in solution for the first time. Conformational changes in the ensemble, whether large or small, represent essential protein motions which underlie key mPPase catalytic function. These dynamic transitions are extremely challenging to monitor, especially in so many conditions and our DEER spectroscopy data demonstrate the sensitivity and resolution necessary to monitor these subtle changes in equilibria, even if these are only a few Angstroms. For several of the conditions we investigated by DEER in solution, corresponding X-ray structures have been solved, with the derived distances agreeing well with the DEER distributions. This further validates the biological relevance of the structures, and reveals the complete conformational ensemble, intractable using other current approaches. Indeed, some conformational states were previously seen using serial time-resolved X-ray static structures and were consistent with asymmetry.

      The ZLD-bound crystal structure doesn't predict the DEER distances, and the conformation of Na+ binding site sidechains in the ZLD structure doesn't predict whether sodium currents occur. This might suggest that the ZLD structure captures a conformation that does not recapitulate what is happening in solution/ a membrane. 

      We agree with the reviewer that the ZLD-bound crystal structure does not predict the DEER distances. However, we believe this discrepancy arises from the steric bulkiness of ZLD inhibitor, which prevents the closure of the hydrolytic centre. Additionally, the absence of Na+ at the ion gate in the ZLD-bound structure suggests that Na+ transport does not occur, a conclusion further supported by our electrometric measurements. We agree with the reviewer; distances observed in the DEER experiments might represent a potential new conformation in solution, not captured by the static X-ray structure, thereby offering new insights into the dynamic nature of the protein under physiological conditions. This serves to emphasize the complementarity of the DEER approach to Xray crystallography and redoubles the importance of using both techniques. Finally, the static X-ray structures have not captured the asymmetric conformations that must exist to explain half-of-thesites reactivity, where DEER yields distance distributions, across all 16 cases tested here (two mutants with eight conditions each), that are consistent with asymmetry.

      Reviewer #2 (Public review):  

      Summary:  

      Crystallographic analysis revealed the asymmetric conformation of the dimer in the inhibitor-bound state. Based on this result, which is consistent with previous time-resolved analysis, authors verified the dynamics and distance between spin introduced label by DEER spectroscopy in solution and predicted possible patterns of asymmetric dimer.  

      Strengths:  

      Crystal structures with inhibitor bound provide detailed coordination in the binding pocket thus useful information for the mPPase field and maybe for drug development.  

      Weaknesses:  

      The distance information measured by DEER is advantageous for verifying the dynamics and structure of membrane protein in solution. However, regarding T211 data, which, as the authors themselves stated, lacks measurement precision, it is unclear for readers how confident one can judge the conclusion leading from these data for the cytoplasmic side. 

      We thank the reviewer for acknowledging the advantageous use of the DEER methodology for identifying dynamic states of membrane proteins in solution. In our original manuscript, we used two sites in our analysis: S525 (periplasm) and T211 (cytoplasm), in which S525R1 yielded highquality DEER data, while T211R1 yielded weak (or no) visual oscillations, leading to broad distributions for the several conditions tested. In the revised manuscript, we now added a third site at the cytoplasmic side (C599R1 located at TMH14), which yielded high-quality DEER data and comparable to S525R1. Both C599R1 and C525R1 spin pairs generated distance distributions for all 16 conditions (two mutants of eight conditions each) that were described well by the solution-state ensemble adopting a predominantly asymmetric conformation.  

      Furthermore, we have tailored our interpretation of the T211R1 DEER data, and refrain from using the data to draw conclusions about the TmPPase conformational ensemble in the presence of different inhibitors. However, we still opted to include the T211R1 data in the SI because they confirm an important structural feature of mPPase in solution conditions; the intrinsically dynamic behaviour of the loop5-6 where T211 is located. This observation in solution is also consistent with our previous (Kellosalo et al., Science, 2012; Li et al., Nat. Commun, 2016; Vidilaseris et al., Sci. Adv., 2019; Strauss et al., EMBO Rep., 2024) and current X-ray crystallography data. To reiterate, we excluded T211R1 from any analysis relating to mPPase asymmetry and our conclusions were entirely based on the S525R1 and new C599R1 DEER data, which allowed us to monitor both sides on the membrane.  

      The distance information for the luminal site, which the authors claim is more accurate, does not indicate either the possibility or the basis for why it is the ensemble of two components and not simply a structure with a shorter distance than the crystal structure.  

      We thank the reviewer for pointing out this possibility and alternative interpretation of our DEER data. We now provide further analysis to show that our DEER data from both membrane sides reporters are highly consistent with (although they cannot completely exclude) asymmetry and rephrase to be inclusive of other possibilities. Importantly, this additional possibility does not affect the current interpretation of the data in our manuscript. Furthermore, we have removed Fig. 6 from the manuscript, and we now include a direct comparison of the in silico predicted distribution coming from the asymmetric hybrid structure with the 8 conditions tested, for both mutants (i.e. S525R1 and C599R1).

      Reviewer #3 (Public review):  

      Summary:  

      Membrane-bound pyrophosphatases (mPPases) are homodimeric proteins that hydrolyze pyrophosphate and pump H+/Na+ across membranes. They are attractive drug targets against protist pathogens. Non-hydrolysable PPi analogue bisphosphonates such as risedronate (RSD) and pamidronate (PMD) serve as primary drugs currently used. Bisphosphonates have a P-C-P bond, with its central carbon can accommodate up to two substituents, allowing a large compound variability. Here the authors solved two TmPPase structures in complex with the bisphosphonates etidronate (ETD) and zoledronate (ZLD) and monitored their conformational ensemble using DEER spectroscopy in solution. These results reveal the inhibition mechanism of these compounds, which is crucial for developing future small molecule inhibitors.  

      Strengths:  

      The authors show that seven different bisphosphonates can inhibit TmPPase with IC50 values in the micromolar range. Branched aliphatic and aromatic modifications showed weaker inhibition.  

      High-resolution structures for TmPPase with ETD (3.2 Å) and ZLD (3.3 Å) are determined. These structures reveal the binding mode and shed light on the inhibition mechanism. The nature of modification on the bisphosphonate alters the conformation of the binding pocket.  

      The conformational heterogeneity is further investigated using DEER spectroscopy under several conditions.  

      Weaknesses:  

      The authors observed asymmetry in the TmPPase-ELD structure above the hydrolytic center. The structural asymmetry arises due to differences in the orientation of ETD within each monomer at the active site. As a result, loop5-6 of the two monomers is oriented differently, resulting in the observed asymmetry. The authors attempt to further establish this asymmetry using DEER spectroscopy experiments. However, the (over)interpretation of these data leads to more confusion than any further understanding. DEER data suggest that the asymmetry observed in the TmPPase-ELD structure in this region might be funneled from the broad conformational space under the crystallization conditions. 

      We respectfully disagree with the reviewer. The asymmetry was previously established using serial time crystallography (Strauss et al., EMBO Rep, 2024) and biochemical assays (e.g. Malinen et al., Prot. Sci., 2022; Artukka et al., Biochem J, 2018; Luoto et al., PNAS, 2013) and partially seen in one static structure (Vidilaseris et al., Sci Adv 2019). DEER data here also show that the previously proposed asymmetry is also present (and this presence of asymmetry is consistent across all DEER data) within the TmPPase conformational ensemble in solution conditions. Although we cannot rule out the possibility that the TmPPase monomers adopt a metastable intermediate state, in such a case we would expect the distance changes reported by DEER to be symmetric across both membrane sides. However, we observe a symmetry breaking between the cytoplasmic and periplasmic TmPPase sites. Indeed, DEER data yield distance distributions similar to that of the hybrid asymmetric structure under all: apo, +Ca, +Ca/ETD, +ETD, +ZLD, +IDP, +PAM, +ALE conditions.

      DEER data for position T211R1 at the enzyme entrance reveal a highly flexible conformation of loop56 (and do not provide any direct evidence for asymmetry, Figure EV8).

      Please see relevant response above. We acknowledge that T211 is indeed situated on a highly dynamic loop, which is important for gating and our DEER data confirm the high flexibility of this protein region. Given we have not observed dipolar oscillations, leading to broad distributions, we have stated in the original manuscript that we will not establish the presence of any asymmetry in solution on the basis of T211, rather relying on the S525R1 and the new C599R1 sites, for which we have acquired high-quality DEER data, as was also pointed out and has been commented on by all reviewers. We have provided data at the C599R1 position (same cytoplasmic side as 211 for which we have now limited our analysis to a minimum) which further provides evidence for asymmetry, including two new conditions.

      Similarly, data for position S521R1 near the exit channel do not directly support the proposed asymmetry for ETD.  

      The reviewer appears to suggest that we hold the S525R1 DEER data as direct proof of asymmetry; this is combative on the grounds that to directly prove asymmetry would require time-resolved DEER measurements, far beyond the scope of this work. Rather, we have applied DEER measurements to explore whether asymmetry (observed previously via time-resolved X-ray crystallography) is also present (or indeed a possibility) in solution. All our S525R1 and C599R1 DEER data (recorded for eight conditions) are consistent with asymmetry (see also detailed response above).

      Despite the high quality of the data, they reveal a very similar distance distribution. The reported changes in distances are very small (+/- 0.3 nm), which can be accommodated by a change of spin label rotamer distribution alone. Further, these spin labels are located on a flexible loop, thereby making it difficult to directly relate any distance changes to the global conformation

      We thank the reviewer for recognising the high quality of our DEER data for the S525R1 site which we now complement with a new pair on the cytoplasmic facing membrane side (C599R1) with DEER data of comparable quality as for S525R1, where visual oscillations in the raw traces for both spin pairs, as in our case, reportedly lead to highly accurate and reliable distributions, able to separate (in fortuitous cases) helical movements of only a few Angstroms (Peter et al., Nature Comms 13:4396, 2022; Klose et al., Biophys J 120:4842-4858, 2021). The ability of DEER/PELDOR offering near Angstrom resolution was also previously demonstrated by the acquisition and solution of highresolution multi-subunit spin-labelled membrane protein structures (Pliotas at al., PNAS, 2012; Pliotas et al., Nat Struct Mol Biol, 2015; Pliotas, Methods Enzymol, 2017) as well as its ability in detecting small (and of similar to mPPase magnitude) conformational changes in different integral membrane protein systems (Kapsalis et al., Nature Comms, 2019; Kubatova et al., PNAS, 2023; Schmidt et al., JACS, 2024; Lane et al., Structure, 2024; Hett et al., JACS, 2021; Zhao et al., Nature, 2024), occurring under different conditions and/or stimuli in solution and/or lipid environment. The changes here are not below the detection sensitivity of DEER (e.g. ~ 7 Angstroms between the two modal distance extremes (+Ca vs +IDP for S525R1), and with all other conditions showing intermediate changes.  

      We agree with the reviewer that these changes are relatively small, but they are expected for membrane ion pumps. Indeed, none of the mPPase structures show helical movements of greater than half a turn, and that only in helices 6 and 12. There appear to be larger-scale loop closing motions of the 5-6 loop that includes T211, due to the presence of E217 which binds to one of the Mg<sup>2+</sup> ions that coordinate the leaving group phosphate. This is, inter alia, the reason that this loop is so flexible: it cannot order before substrate is bound.  

      The reviewer suggests that the subtle distance shifts detected arise only from changes of label rotamer distribution. However, the concerted nature of the modal distance shifts with respect to multiple different conditions at a single labelling site strongly suggests that preferential rotamer orientations are not the cause. Indeed, for so many spin labels to undergo an arbitrary shift that the modal distance of the entire distribution changes – and in the absence of any conformational change – appears improbable. Here we have the resolution to detect such subtle differences by DEER, given there are unambiguous shifts in our time domain data (i.e. the position of the minimum of the first dipolar oscillation) (Fig 4) and these are reflected in the modal distances in the distributions. We also refrain from performing any quantitative analysis and use qualitative trends in modal distance shifts only; all which support our proposed model of a symmetry breaking across the membrane face. To further belabour this point, we do not quantify the DEER data (for instance through parametric fitting) to extract populations of different conformational states and we appreciate that to do so would be highly prone to error; however we do (and can, we feel without over-interpretation) assert that the modal distances shift.  

      The interpretations listed below are not supported by the data presented:  

      (1) 'In the presence of Ca2+, the distance distribution shifts towards shorter distances, suggesting that the two monomers come closer at the periplasmic side, and consistent with the predicted distances derived from the TmPPase:Ca structure.'

      Problem: This is a far-stretched interpretation of a tiny change, which is not reliable for the reasons described in the paragraph above. 

      While the authors overall agree with the reviewer assessment that ±0.3 nm is a small (not a minor) change, there are literature examples quantifying (or using for quantification) distribution peaks separated by similar Δr. (Kubatova et al., PNAS, 2023; Schmidt et al., JACS, 2024; Hett et al., JACS, 2021; Zhao et al., Nature, 2024). However, the time-domain data clearly indicate the position of the first minimum of the dipolar oscillation shifts to shorter dipolar evolution time. The sensitivity of the time-domain data to subtle changes in dipolar coupling frequency is significantly improved compared to the distance distributions.

      Importantly, we have fitted Gaussians to the experimental distance distributions of 525R1 output by the Comparative Deer Analyzer 2.0 and observed a change in the distribution width in presence of Ca2+, implying the rotameric freedom of the spin label is restricted. However, the CW-EPR for 525R1 indicate that the rotational correlation time of the spin label is highly consistent between conditions (the spectra are almost identical); this cannot be explained simply by rotameric preference of the spin label (as asserted by the reviewer 3), as there is no (further) immobilisation observed from the CW-EPR of apo-state (Figure EV9) to that in presence of Ca2+. Furthermore, in the absence of conformational changes, it is reasonable to assume (and demonstrable from the CW-EPR data) that the rotamer cloud should not significantly change between conditions. However, Gaussian fits of the two extreme cases yielding the longest (i.e., in presence of IDP) and shortest (in presence of ZLD) modal distances for the 525R1 DEER data indicated significant (i.e., above the noise floor after Tikhonov validation) probability density for the IDP condition at 50 Å (P(r) = 0.18). This occurs at four standard deviations above the mean of the Guassian fit to the +ZLD condition, which by random chance should occur with <0.007% probability.  

      As in previous response, the method can detect changes of such magnitude which are not small, but physiologically relevant and expected for integral membrane proteins, such as mPPases. Indeed, even in equal (or more) complex systems such as heptameric mechanosensitive channel proteins DEER provided sub-Angstrom accuracy, when a spin labelled high resolution XRC structure was solved (Pliotas et al., PNAS, 2012; Pliotas et al., Nat Struct Mol Biol, 2015). Despite this being an ideal case where DEER accuracy was experimentally validated another high-resolution structural method on modified membrane protein and is not very common it demonstrates the power of the method, especially when strong oscillations are present in the raw DEER data (as here for mPPase S525R1, and C599R1), even when multiple distances are present, Angstrom resolution is achievable in such challenging protein classes.

      (2) 'Based on the DEER data on the IDP-bound TmPPase, we observed significant deviations between the experimental and the in silico distances derived from the TmPPase:IDP X-ray structure for both cytoplasmic- (T211R1) and periplasmic-end (S525R1) sites (Figure 4D and Figure EV8D). This deviation could be explained by the dimer adopting an asymmetric conformation under the physiological conditions used for DEER, with one monomer in a closed state and the other in an open state.'  

      Problem: The authors are trying to establish asymmetry using the DEER data. Unfortunately, no significant difference is observed (between simulation and experiment) for position 525 as the authors claim (Figure 4D bottom panel). The observed difference for position 112 must be accounted for by the flexibility and the data provide no direct evidence for any asymmetry.  

      Reviewer 3 is incorrect in suggesting that we are trying to prove asymmetry through the DEER data. That is a well-known fact in the literature (e.g. Vidilaseris et al, Sci Adv 2019) where we show (1) that the exit channel inhibitor ATC (i.e. close to S525R1) binds better in solution to the TmPPase:PPi complex than the TmPPase:PPi<sub>2</sub> complex, and (2) that ATC binds in an asymmetric fashion to the TmPPase:IDP<sub>2</sub> complex with just one ATC dimer on one of the exit channels. We merely use the DEER data to support this well-established fact.  

      However, because we agree that the DEER data in presence of IDP does not provide direct proof for asymmetry; particularly for the cytoplasmic facing mutant T211R1, we have refrained from interpreting T211R1 data beyond being a highly dynamic loop region (as evidenced by the broad distributions). As pointed out by the reviewer, the differences in distance distributions between conditions observed for T211R1 likely arise from conformational heterogeneity in solution. Furthermore, we now report DEER data on another new site (C599R1), which is also on the cytoplasmic side and yields high quality DEER data comparable to the S525R1 data (commended for their quality by both the reviewers). The C599R1 measurements show that in all conditions tested, highly similar distributions are observed, inconsistent with the in silico predicted distance distributions from the symmetric X-ray structures, but consistent with an asymmetric hybrid structure (i.e. open-closed) in solution. Importantly, the difference between the fully open (6.8 nm modal distance) and fully closed (4.8 nm modal distance) states of the C599R1 dimer is larger than for the S525R1 dimer pair. Thus, delineating the asymmetric hybrid conformation from the symmetric conformations is more robust.

      (3) 'Our new structures, together with DEER distance measurements that monitor the conformational ensemble equilibrium of TmPPase in solution, provide further solid experimental evidence of asymmetry in gating and transitional changes upon substrate/inhibitor binding.'  

      Problem: See above. The DEER data do not support any asymmetry. 

      We feel that the reviewer comments here are somewhat unfounded. All the DEER data (for 525R1 periplasmic and C599R1 cytoplasmic sites are described, most parsimoniously, using an asymmetric hybrid structure. In particular, the new C599R1 distance distributions are poorly described by the symmetric X-ray crystal structures, with a conserved modal distance of approx. 5.8 nm throughout the tested conditions that aligns nicely with the in silico predictions from the asymmetric hybrid structure. Additionally, all S525R1 and C599R1 data well exceed the relevant criteria of the recent white paper (Schiemann et al., 2021, JACS) from the EPR community to be considered reliably interpretable (strong visual oscillations in the raw traces; signal-to-noise ratio .r.t modulation depth of > 20 in all cases; replicates have been performed and added into the maintext or supplementary; near quantitative labelling efficiency (evidenced by lack of free spin label signal in the CW-EPR spectra); analysed using the CDA (now Figure EV10) to avoid confirmation bias).

      While the DEER data do not prove asymmetry, we do not claim proof of asymmetry in the above sentence. We concede to rephrase the offending sentence above as: “Our new structures, together with DEER distance measurements that monitor the conformational ensemble of TmPPase in solution, do not exclude asymmetry in gating and transitional changes upon substrate/inhibitor binding and are consistent with our proposed model.” We feel that this reframed conjecture of asymmetry is well founded; indeed, comparing all the 16 experimentally derived DEER distance distributions for the 525R1 and 599R1 sites with in-silico modelling performed on the hybridised asymmetric structure (i.e., comprised of one monomer bound to Ca2+ and another bound to IDP) yields overlap coefficients (Islam and Roux, JPC B, 2015) of >0.85. This implies the envelope of the modelled distance distribution is quantitatively inside the envelope of the experimental distance distributions. Thus, the DEER data support asymmetry (previously observed by time-resolved XRC) in solution, and while we appreciate that ideally one would measure time-resolved DEER to directly correlate kinetics of conformational changes within the ensemble to the catalytic cycle of mPPase, (and this is something we aim to do in the future), it is far beyond the scope of this study.

      Indeed, half-of-the-sites reactivity has been demonstrated in at least the following papers

      (Vidilaseris et al, Sci Acv. ,2019, Strauss et al, EMBO Rep. 2024, Malinen et al Prot Sci, 2022, Artukka et al Biochem J, 2018; Luoto et al, PNAS, 2013). Half-of-the sites activity requires asymmetry in the mechanism, and therefore asymmetric motions in the active site (viz 211) and exit channel (viz 525). As mentioned above, we have demonstrated this for other inhibitors (Vidilaseris et al 2019) and as part of a time-resolved experiment (Strauss et al 2024). In fact, given the wealth of evidence showing that the symmetrical crystal structures sample a non- or less-productive conformation of the protein, it would be quixotic to propose the DEER experiments - in solution - do not generate asymmetric conformations. It certainly doesn’t obey Occam’s razor of choosing the simplest possible explanation that covers the data.

      (4) Based on these observations, and the DEER data for +IDP, which is consistent with an asymmetric conformation of TmPPase being present in solution, we propose five distinct models of TmPPase (Figure 7).  

      Problem: Again, the DEER data do not support any asymmetry and the authors may revisit the proposed models. 

      We have redressed the proposed models and limited them to four asymmetric models to clearly illustrate the apo/+Ca/+Ca:ETD-state (model 1) and highlight the distinct binding patterns of various inhibitors (ETD, ZLD and IDP; model 2-4), which result in a variety of closed/open-open states. In this version, we clarify that the proposed models are not solely based on the DEER data but all DEER data recorded for multiple conditions, inhibitors and for two opposite membrane side facing reporters are highly consistent, and are grounded in both current and previously solved structures, with the DEER data providing additional consistency with these models.

      (5) 'In model 2 (Figure 7), one active site is semi-closed, while the other remains open. This is supported by the distance distributions for S525R1 and T211R1 for +Ca/ETD informed by DEER, which agrees with the in silico distance predictions generated by the asymmetric TmPPase:ETD X-ray structure'  

      Problem: Neither convincing nor supported by the data 

      We respectfully disagree with the reviewer. However, owing to the conformational heterogeneity of T211R1, we now exclude T211R1 data from quantitative interpretation of changes to the conformational ensemble. Instead, we include new DEER data from site C599R1, which provides high-quality and convincing data that is consistent with asymmetry at the cytoplasmic face, and inconsistent with in silico distance distributions derived from symmetric X-ray crystal structures. Furthermore, the S525R1 distance distributions for the +ETD (corresponding to +Ca/ETD) and +ZLD conditions were directly compared with both the apo-state distance distribution (corresponding to a fully open, symmetric conformation) and the in silico predicted distributions of the asymmetric hybrid structure (corresponding to an open-closed conformation). Overlap coefficients were calculated (given in the main text) that indicated the +ETD (corresponding to +Ca/ETD) and +ZLD S525R1 distributions were more consistent with the apo-state distance distribution. This suggests that while on the cytosolic face of the membrane, an open-closed conformation is favoured, on the periplasmic face, a symmetric open-open conformation is favoured.

      Recommendations for the authors:  

      Reviewer #1 (Recommendations for the authors):   

      (1) The DEER experiments were performed with the two crystallized inhibitors, ETD and ZLD, along with previously characterized IDP. It would increase the impact of a tighter-binding phosphonate was examined since the inhibitory mechanism of these molecules is of greater interest. 

      We acknowledge the reviewer concern regarding the choice of weaker inhibitors. We chose to focus on the weaker binders, as we were able to obtain high-quality crystal structures for these compounds. This allowed us to perform DEER spectroscopy with the added advantage of accurately analysing the data against structural models derived from X-ray crystallography. In the revised version, we also include results from alendronate and pamidronate, two of the tighter inhibitors, which show similar and consistent results to the others.

      (2) I'm not able to find the concentrations of ETD and ZLD used for the DEER experiments. This information should be added to the Methods section on sample prep for EPR. 

      The information is already mentioned in the Method section on sample preparation for EPR spectroscopy (page 24), where we indicated that the protein aliquots were incubated with a final concentration of 2 mM inhibitors or 10 mM CaCl2 (30 min, RT). However, we recognise that this may not have been sufficiently clear. To clarify, we now explicitly state that the concentration of ETD and ZLD (amongst other inhibitors) used for the DEER experiments is 2 mM.  

      (3) There should be additional detail about the electrometry replicates. Does "triplicate" mean three measurements on the same sensor, three different sensors, and different protein preparations? At a minimum, data should be collected from three different sensors to ensure that the negative results (lack of current) for ETD and ZLD are not due to a failed sensor prep. In addition, Data from the other replicates should be shown in a supplementary figure, either the traces, or in a summary figure. Are the traces shown collected on the same sensor? They could be, in principle, since the inhibitor is washed away after each perfusion. 

      Yes, by 'triplicate', we mean three measurements taken on the same sensor. All traces shown were collected from a single sensor. Thank you for your advice; we now show here additional data from other sensors that display the same pattern. As for the possibility of a failed sensor preparation, this is unlikely since we always ensure the sensor quality with the substrate (PPi) as a positive control after each measurement.

      Author response image 1.

      (4) I'm confused by the NEM modification assay, and I don't think there is enough information in this manuscript for a reader to figure out what is happening. Why is the protein active if an inhibitor is present? I understand that there is a conformational change in the presence of the inhibitor that buries a cysteine, but the inhibitor itself should diminish function, correct? Is the inhibitor removed before testing the function? In addition, it would be clearer if the cysteines that are modified are indicated in the main text. I don't understand what is being shown in Figure Ev2. Shouldn't the accessible cysteines in the apo form be shown? Finally, the sentence "IDP has been reported to prevent the NEM modification..." does not make sense to me. Should the word "by" be removed from this sentence? 

      We apologize for the confusion. Yes, the inhibitors were removed before testing the protein function. In Figure EV2, the accessible cysteines are shown for both the apo and IDP-bound states. As seen, the accessible cysteines in the IDP-bound states are fewer than those in the apo state, meaning fewer cysteines are available for modification. Consequently, more activity is retained when IDP binds due to the reduction in accessible cysteines. We have addressed this in the manuscript (see the method section on the NEM modification assay).

      (5) Why does the model in Figure 7 show the small molecules bound to only one subunit, when they are crystallized in both subunits? 

      We propose that the small molecules bound to the two subunits in the crystal structure is likely a result of substrate inhibition, given the excess inhibitor used during crystallisation (e.g. Artukka, et al., Biochemical Journal, 2018; Vidilaseris, et al., Science Advances, 2022). Our PELDOR data indicate that in solution, the small molecules bound to TmPPase are in an intermediate state between both subunits being closed and both being open, most likely with at least one subunit in an open state. This is also consistent with previous kinetic studies (Anashkin, V. A., et al., International Journal of Molecular Sciences, 22, 2021), which showed that the binding constant of IDP to the second subunit is around 120 times higher than that of the first subunit.

      (6) The authors argue that the two ETDs bound in the two protomers adopt distinct conformations. Can this be further supported, for example, by swapping the position of the two ETDs between the two protomers and calculating a difference map (there should be corresponding negative/positive density if the modelling of the two different conformations is robust)? 

      As per the reviewer suggestion, we swapped the positions of the two ETDs between the protomers and calculated the difference electron density map. This analysis, presented in Figure EV3, reveals corresponding negative and positive electron density peaks, indicating that the ETDs indeed adopt distinct conformations in each protomer, supporting the accuracy of our modeling.

      (7) Are the changes in loop conformation possibly due to crystal packing differences for the two protomers? 

      We examined the crystal packing of the two protomers and found no interactions at the loop regions (red coloured in Author response image 2 below) that could be attributed to crystal packing differences. Therefore, we rule out this possibility.

      Author response image 2.

      (8) Typos:  

      Legend for Figure EV2 cystine - cysteine  

      Page 14, last sentence of the first paragraph: further - further  

      Figure 6 legend: there is no reference to panel B.  

      Thanks for pointing out the typos, now they are fixed.

      Reviewer #2 (Recommendations for the authors):  

      (1) T211 is located on the same loop where ligand/inhibitor-coordinating side chains (E217, D218) are located. It has not been tested whether spin labeling here would affect inhibitor binding. 

      We test all the mutant(s) activity before spin labelling, but not the activity of the spin-labelled mutants. MTSSL spin labels are typically not structurally perturbing. In particular, the T211R1 site that the reviewer is referring to is now not included in our interpretation of conformational changes occurring during mPPase’s functional cycle.

      (2) Why should the spin label be introduced to T211, which is recognized as a flexible region in the crystal structure? Authors should search for suitable residues except for T211 and other residues in this loop to evaluate the cytoplasmic distance. 

      We acknowledge the reviewer’s concern regarding the flexibility of the T211 region for spin labelling. Given the challenges associated with TmPPase, including reduced protein expression, loss of function, or inaccessibility upon spin labelling at certain sites, we have explored alternative residues. After extensive testing, we identified C599 as a suitable site for spin labelling resulting in high-quality DEER data. The results from spin labelling at C599 have been incorporated into the revised manuscript.

      (3) On the other hand, DEER data for S525 is solid, as the authors stated. This residue is located on the luminal side of the enzyme. However, the description of the luminal side structure and the comparison of symmetric/asymmetric dimer in this par are missing in the paper. 

      We thank the viewer for their positive assessment of the S525R1 DEER data. The data for 525 and now also for 599 spin pairs are indeed solid given the strong visual oscillation we observed particularly in such a challenging system.   

      We presented the periplasmic sites in the crystal structure dimer (Figure 4A), highlighting both the symmetrical region and the asymmetric model in Figure 4. In the revised version, we include additional details about this region and our rationale for labeling at position S525.

      (4) The conclusion models (Figure 7) are misleading. In the crystal structure, the 5-6Loop distance between each monomer should be close given the location of the dimer interface, and the actual distance between T211 in the structure (for example, in 5lzq) is about 10A. Nevertheless, the model depicts this distance longer than S525 (40.7A in 5LZQ), which would give a false impression. 

      We would like to apologize for the misleading model. We have now corrected the models to ensure they are consistent with their respective regions in the crystal structures.

      (5) P8 last paragraph  

      It is hard to imagine that in a crystal lattice, the straight inhibitor always binds to monomer A, and the neighboring monomer is always attached to a slightly tilted inhibitor, which causes asymmetry. For example, wouldn't it mean that it would first bind to one of them, which would then affect the neighboring monomer via 5-6 Loop, which would then affect its binding pose? So in this case, the inhibitor did not ARAISE asymmetry, and this is where it is misleading for readers. 

      We apologize for the confusion. What we intended to convey is that the first inhibitor binds to one protomer, which then affects the conformation of the neighbouring monomer, ultimately influencing its binding pose. This is required for half-of-the-sites reactivity, which is well-established in this system. This is reflected in our crystal structure, where we observed asymmetry in the loop 5-6 region and the ETD orientation between the two protomers. We have addressed this in the manuscript accordingly.

      (6) P11 L4 EV10 instead of EV8? 

      Thanks for pointing out. We have corrected it accordingly.

      (7) P11 L5 It is difficult to determine whether the peak is broad or sharp. Should be evaluated quantitatively by showing the half-value width of the peak. This may also be helpful to judge whether the peak is a mixture of two components or a single one. 

      We have taken this analysis out and rephrased the offending sentence. We have also added the FWHM values as the Reviewer suggested, and corresponding standard deviations for the distance distributions (under approximation as Gaussian distribution).   

      (8) Throughout the paper, the topology of the enzyme may be difficult to follow for readers who are not experts in this field. Please indicate the membrane plane's location or a figure's viewpoint in the caption. 

      We acknowledge the importance of making our figures accessible to all readers. In the revised manuscript, we have enhanced the clarity of our figures by explicitly indicating the membrane plane’s location and specifying the viewpoint in each figure caption. For example, we have added annotations such as “Top view of the superposition of chain A (cyan) and chain B (wheat), showing the relative movements (black arrow) of helices. The membrane plane is indicated by dashed lines.”

      (9) Figure 2B Check the color of the helix.  

      IDP and ETD are almost the same color, so it is difficult to see the superposition. It would be easier to understand the reading by, for example, using a lighter or transparent color set only for IDPs.  

      We acknowledge the reviewer concern regarding the colour similarity between the IDP and ETD in Figure 2B, which hinders clear differentiation. To enhance visual distinction, we have adjusted the colour scheme by changing the TmPPase:IDP structure colour to light blue. This modification improves the clarity of the superposition, making the structural differences more discernible.

      (10) Figure 2C Check the coordination state (dotted line), there appears to be coordination between E217Cg and Mg. Also, water that is located near N492 appears to be a bit distant from Mg, why does this act as a ligand? Stereo view or view from different angles, and distance information would help the reader understand the bonding state in more detail.  

      Yes, we confirm that Mg<sup>2+</sup> is coordinated by the oxygen atoms from both the side chain and main chain of residue E217. The water molecule near N492 is not directly coordinated with Mg<sup>2+</sup> but interacts with the O5 atom of one of the phosphate groups in ETD. To enhance clarity, we have updated Figure 2C (and other related figures) to include stereo views.  

      (11) Figure 5A: in the Bottom view (lower left), the symmetric dimer does not look symmetric. Better to view from a 2-fold axis exactly.  

      We have taken this figure out entirely and instead add a direct comparison to the in silico predicted distribution from the asymmetric hybrid structure to all 16 experimental DEER distributions. We have added the symmetric and asymmetric structures to Fig. 4A and view the symmetric structure along the 2-fold axis, as suggested.   

      (12) Figure 5B: Indicate which data is plotted in the caption.  

      As mentioned above, we have taken this figure out, as we felt quantifying two overlapping populations from a single Gaussian was over-interpretation of the data, and at the suggestion of reviewer 3, we have tailored our interpretation here.  

      (13) Figure EV8:  

      Because the authors discuss a lot about their conclusive model based on this data, Figure EV8 should be treated as a main figure, not a supplement. However, this reviewer has serious concerns about the measurement in this figure. Because DEER for T211 is too noisy, I don't see the point in discussing this in detail. For example, in the Ca/ETD data, there is a peak near 50A, but it would be difficult for TM5 to move away from this distance unless the protein unfolds. I do not find it meaningful to discuss using measurement results in which such an impossible distance is detected as a peak.  

      A: Show top view as in Figure 5  

      D: 2nd row dotted line. Regarding the in silico model that is used as a reference to compare the distance information, the distance of 40-50 A for T211 in the Ca-bound form is hard to imagine. PDB 4av6 model shows that T211 is disordered and not visible, but given the position of the TM5 helix, it does not appear to be that different from the IDR binding structure (5LZQ, 10A between two T211). The structures of in silico models are not shown in the figure, as it is only mentioned as modeled in Rossetafold. Please indicate their structures, especially focused on the relative orientation of T211 and S525 in the dimer, which would allow readers to determine the distances.  

      We acknowledge the reviewer’s concerns regarding Figure EV8 and the DEER data for T211R1. Upon re-evaluation, we recognize that the non-oscillating nature of the DEER data for T211R1 leads to broad distributions, indicating increased conformational dynamics, which is expected for a highly dynamic loop. Consequently, we have limited the discussion and interpretation of T211R1 in the revised manuscript and focused more on C599R1.

      Reviewer #3 (Recommendations for the authors):  

      A careful interpretation of the data in view of these limitations and without directly linking to asymmetry could solve the problem of the over-interpretation of the DEER data.  

      We respectfully disagree with the reviewer. Please see our detailed response above.  

      Additional comments:  

      (1) Did the authors use a Cys-less construct for spin labeling and DEER experiments?  

      We utilized a nearly Cys-less construct in which all native cysteines were mutated to serine, except for Cys183, which was retained due to its buried location and functional importance. We then introduced single cysteine mutations for spin labelling. For C599, Ser599 was reverted to cysteine.

      (2) The time data for position T211R1 is too short for most cases (Figure EV8D) for a reliable distance determination. No confidence interval is given for the '+Ca' sample distance distributions.  

      We recorded longer time traces for two of the conditions to better assign the background. We did not use the 211R1 data to reach any conclusions regarding asymmetry, which were based on the 525R1 and the 599R1 data. We now simply include T211R1 data to indicate the high mobility observed at loop5-6. We have added the confidence interval for the +Ca condition.  

      (3) It is recommended to mention the 2+1 artefact obvious at the end of the DEER data. 

      In the methods section, we have mentioned that the “2+1” artefact present at the end of the S525R1, and T211R1 DEER data likely arises from using a 65 MHz offset, rather than an 80 MHz offset (as for the C599R1 data), which avoids significant overlap of the pump and detection pulses. We also mention in the methods section that owing to the intense “2+1” artefact, the decision was made to truncate the artefact away, to minimise the impact on data treatment. As for motivation to use the lower offset of 65 MHz, we did so to maximise the achievable signal-to-noise ratio (SNR), as particularly for the T211R1 data, the detected echo was quite weak. This was further exacerbated by the poor transverse relaxation time observed at that site.  

      (4) Please check the number of significant digits for all the reported values. 

      We have addressed the number of significant digits as requested.

      (5) Please report the mean distances from DEER experiments with the standard deviation or FWHM.

      We have addressed this in the revised manuscript, we report modal distances rather than the mean distances and provide the FWHM and standard deviation.

    1. Author response:

      The following is the authors’ response to the original reviews

      Reviewer #1:

      Weaknesses:

      (1) Only Experiment 1 of Rademaker et al (2019) is reanalyzed. The previous study included another experiment (Expt 2) using different types of distractors which did result in distractor-related costs to neural and behavioral measures of working memory. The Rademaker et al (2019) study uses these two results to conclude that neural WM representations are protected from distraction when distraction does not impact behavior, but conditions that do impact behavior also impact neural WM representations. Considering this previous result is critical for relating the present manuscript's results to the previous findings, it seems necessary to address Experiment 2's data in the present work

      We thank the reviewer for the proposal to analyze Experiment 2 where subjects completed the same type of visual working memory task, but instead had either a flashing orientation distractor or a naturalistic (gazebo or face) distractor present during two-thirds of the trials. As the reviewer points out, unlike Experiment 1, these two conditions in Experiment 2 had a behavioral impact on recall accuracy, when compared to the blank delay. We have now run the temporal cross-decoding analysis, temporally-stable neural subspace analysis, and condition cross-decoding analysis in Experiment 2. The results from the stable subspace analysis are present in Figure 3, while the results from the temporal cross-decoding analysis and condition cross-decoding analysis are present in the Supplementary Data.

      First, we are unable to draw strong conclusions from the temporal cross-decoding analysis, as the decoding accuracies across time in Experiment 2 are much lower compared to Experiment 1. In some ROIs of the naturalistic distractor condition we see that some diagonal elements are not part of the above-chance decoding cluster, making it difficult to draw any conclusions regarding dynamic clusters. We do see some dynamic coding in the naturalistic condition in V3 where the off-diagonals do not show above-chance decoding. Since the temporal cross-decoding provides low accuracies, we do not examine the dynamics of neural subspaces across time.

      We do, however, run the stable subspace analysis on the flashing orientation distractor condition. Just like in Experiment 1, we examine temporally stable target and distractor subspaces. When projecting the distractor onto the working memory target subspace, we see a higher overlap between the two as compared to Experiment 1. A similar pattern is seen also when projecting the target onto the distractor subspace. We still see an above-chance principal angle between the target and distractor; however, this angle is qualitatively smaller compared to Experiment 1. This shows that the degree of separation between the two neural subspaces is impacted by behavioral performance during recall.

      (2) Primary evidence for 'dynamic coding', especially in the early visual cortex, appears to be related to the transition between encoding/maintenance and maintenance/recall, but the delay period representations seem overall stable, consistent with previous findings

      We agree with the reviewer that we primarily see dynamic coding between the encoding/maintenance and at the end of the maintenance periods, implying the WM representations are stable in most ROIs. The only place where we argue that we might see more dynamic coding during the delay itself is in V1 during the noise distractor trials in Experiment 1.

      (3) Dynamicism index used in Figure 1f quantifies the proportion of off-diagonal cells with significant differences in decoding performance from the diagonal cell. It's unclear why the proportion of time points is the best metric, rather than something like a change in decoding accuracy. This is addressed in the subsequent analysis considering coding subspaces, but the utility of the Figure 1f analysis remains weakly justified.

      We agree that other metrics can also provide a summary of dynamics; here, the dynamicism index just acts as a summary visualizing the dynamic elements. It offers an intuitive way to visualize peaks and troughs of the dynamic code across the extent of the trial.

      (4) There is no report of how much total variance is explained by the two PCs defining the subspaces of interest in each condition, and timepoint. It could be the case that the first two principal components in one condition (e.g., sensory distractor) explain less variance than the first two principal components of another condition.

      We thank the reviewer for this comment. We have now included the percent variance explained for the two PCs in both the temporally-stable target and distractor subspace and the dynamic subspace analysis. The percent-explained is comparable across analyses; the first PC ranges from 43-50% and the second ranges from 28-37%. The PCs within each analysis (dynamic no-distractor, orientation and noise distractor; temporally-stable target and distractor) are even closer in range (Figure 2c and 3d).

      (5) Converting a continuous decoding metric (angular error) to "% decoding accuracy" serves to obfuscate the units of the actual results. Decoding precision (e.g., sd of decoding error histogram) would be more interpretable and better related to both the previous study and behavioral measures of WM performance.

      We thank the reviewer for the comments. FCA is a linear function of the angular error that uses the following equation:

      We think that the FCA does not obfuscate the results, but instead provides an intuitive scale where 0% accuracy corresponds to a 180° error, 50% to a 90° error and so on. This also makes it easy to reverse-calculate the absolute error if need be. Our lab has previously used this method in other neuroimaging papers with continuous variables (Barbieri et al. 2023, Weber et al. 2024).

      We do, however, agree that “% decoding accuracy” does not provide an accurate reflection of the metric used. We have thus now changed “% decoding accuracy” to “Accuracy (% FCA)”.

      (6) This report does not make use of behavioral performance data in the Rademaker et al (2019) dataset.

      We have now analyzed Experiment 2 which, as previously mentioned by the reviewer and unlike Experiment 1, showed a decrease in recall accuracy during the two distractor conditions. We address the results from Experiment 2 in a previous response (please see Weaknesses 1).

      We do not, however, relate single subject behavioral performance to neural measurements, as we do not think there is enough power to do so with a small number of subjects in both Experiment 1 and 2. 

      (7) Given there were observed differences between individual retinotopic ROIs in the temporal cross-decoding analyses shown in Figure 1, the lack of data presented for the subspace analyses for the corresponding individual ROIs is a weakness

      We have now included an additional supplementary figure that shows individual plots of each ROI for the temporally stable subspace analysis for both Experiment 1 and Experiment 2 (Supplementary Figure 5). 

      Reviewer #1 (Recommendations For The Authors):

      (1) Is there any relationship between stable/dynamic coding properties and aspects of behavioral performance? This seems like a major missed opportunity to better understand the behavioral relevance or importance of the proposed dynamic and orthogonal coding schemes. For example, is it the case that participants who have more orthogonal coding subspaces between orientation distractor and remembered orientation show less of a behavioral consequence to distracting orientations? Less induced bias? I know these differences weren't significant at the group level in the original study, but maybe individual variability in the metrics of this study can explain differences in performance between participants in the reported dataset

      As mentioned in the previous response, we do not run individual correlations between dynamic or orthogonal coding metrics and behavioral performance, because of the small number of subjects in both experiments. We believe that for a brain-behavior correlation between average behavioral error of subjects and an average brain measure, we would need a larger sample size.  

      (2) The voxel selection procedure differs from the original study. The authors should add additional detail about the number of voxels included in their analyses, and how this number of voxels compares to that used in the original study.

      We have now added a figure summarizing the number of voxels selected across participants. We do select fewer voxels compared to Rademaker et al. 2019 (see their Supplementary Tables 9 and 10 and our Supplementary Figure 8). For example we have ~500 voxels on average in V1 in Experiment 1, while the original study had ~1000. As mentioned in the methods, we aimed to select voxels that reliably responded to both the perception localizer conditions and the working memory trials.

      (3) Lines 428-436 specify details about how data is rescaled prior to decoding. The procedure seems to estimate rescaling factors according to some aspect of the training data, and then apply this rescaling to the training and testing data. Is there a possibility of leakage here? That is - do aspects of the training data impact aspects of the testing data, and could a decoder pick up on such leakage to change decoding? It seems this is performed for each training/testing timepoint pair, and so the temporal unfolding of results may depend on this analysis choice.

      Thank you for the suggestion. To prevent data leakage, the mean and standard deviation are computed exclusively from the training set. These scaling parameters are then applied to the test set, ensuring that no information from the test set influences the training process. This transformation simply adjusts the test set to the same scale as the training data, without exposing the model to unseen test data during training.

      (4) Figure 1d, V1: it looks like the 'dynamics' are a bit non-symmetric - perhaps the authors could comment on this detail of the results? Why would we expect there would be a dynamic cluster on one side of the diagonal, but not the other? Given that this region, condition is the primary evidence for a dynamic code that's not related to the beginning/end of delay (see other comments), figuring this out is of particular importance.

      We thank the reviewer for this question. We think that this is just due to small numerical differences in the upper and lower triangles of the matrix, rather than a neuroscientifically interesting effect. However, this is only a speculative observation.

      (5) I think it's important to address the issue I raised in "weaknesses" about variance explained by the top N principal components in each condition. What are we supposed to learn from data projected into subspaces fit to different conditions if the subspaces themselves are differently useful?

      Thank you, this has now been addressed in a previous comment (please see Weakness 4). 

      Reviewer #2:

      Weaknesses:

      (1) An alternative interpretation of the temporal dynamic pattern is that working memory representations become less reliable over time. As shown by the authors in Figure 1c and Figure 4a, the on-diagonal decoding accuracy generally decreased over time. This implies that the signal-to-noise ratio was decreasing over time. Classifiers trained with data of relatively higher SNR and lower SNR may rely on different features, leading to poor generalization performance. This issue should be addressed in the paper.

      We thank the reviewer for raising this issue and we have now run three simulations that aim to address whether a changing SNR across time might create dynamic clusters. 

      In the first simulation we created a dataset of 200 voxels that have a sine or cosine response function to orientations between 1° to 180°, the same orientations as the remembered target. A circular shift is applied to each voxel to vary preferred (or maximal) responses of each simulated voxel. We then assess the decoding performance under different SNR conditions during training and testing. For each of the seven iterations we selected 108 responses (out of 180) to train on and 108 to test on. To increase variability the selected trials differed in each iteration. Random white noise was applied to the data and thus the SNR was independently scaled according to the specified levels for train and test data. We then use the same pSVR decoder as in the temporal cross decoding analysis to train and test. 

      The second and third simulations more directly address whether increased noise levels  would induce the decoder to rely on different features of the no-distractor and noise distractor data. We use empirical data from the primary visual cortex (V1; where dynamic coding was seen in the noise distractor trials) under the no-distractor and noise distractor conditions for the second and third simulations, respectively. Data from time points 5.6–8.8 seconds after stimulus onset are averaged across five TRs. As in the first simulation, SNR is systematically manipulated by adding white noise. Additionally, to see whether the initial decrease in SNR and subsequent increase would result in dynamic coding clusters, we initially increased and subsequently decreased the amplitude of added noise. The same pSVR decoder was used to train and test on the data with different levels of added noise.

      We see an absence of dynamic elements in the SNR cross-decoding matrices, as the decoding accuracy primarily depends on the training data rather than test data. This results in some off-diagonal values in the decoding matrix that are higher, rather than smaller, than corresponding on-diagonal elements.

      We have now added a Methods section explaining the simulations in more detail and Supplementary Figure 9 showing the SNR cross-decoding matrices. 

      (2) The paper tests against a strong version of stable coding, where neural spaces representing WM contents must remain identical over time. In this version, any changes in the neural space will be evidence of dynamic coding. As the paper acknowledges, there is already ample evidence arguing against this possibility. However, the evidence provided here (dynamic coding cluster, angle between coding spaces) is not as strong as what prior studies have shown for meaningful transformations in neural coding. For instance, the principal angle between coding spaces over time was smaller than 8 degrees, and around 7 degrees between sensory distractors and WM contents. This suggests that the coding space for WM was largely overlapping across time and with that for sensory distractors. Therefore, the major conclusion that working memory contents are dynamically coded is not well-supported by the presented results.

      We thank the reviewer for this comment. The principal angles we calculate are above-baseline, meaning that we subtract the within-subspace principal angles from the between-subspace principal angles and take the average. Thus a 7 degree difference does not imply that there are only 7 degrees separating e.g. the sensory distractor from the target; it just indicates that the separation is 7 degrees above chance. 

      (3) Relatedly, the main conclusions, such as "VWM code in several visual regions did not generalize well between different time points" and "VWM and feature-matching sensory distractors are encoded in separable coding spaces" are somewhat subjective given that cross-condition generalization analyses consistently showed above chance-level performance. These results could be interpreted as evidence of stable coding. The authors should use more objective descriptions, such as 'temporal generalization decoding showed reduced decoding accuracy in off-diagonals compared to on-diagonals.

      Thank you, we agree that our previous claims might have been too strong. We have now toned down our statements in the Abstract and use “did not fully generalize” and “VWM and feature-matching sensory distractors are encoded in coding spaces that do not fully overlap.”

      Reviewer #2 (Recommendations For The Authors):

      Weakness 1 can potentially be addressed with data simulations that fix the signal pattern, vary the noise pattern, and perform the same temporal generalization analysis to test whether changes in SNR can lead to seemingly dynamic coding formats.

      Thank you for the great suggestion. We have now run the suggested simulations. Please see above (response to Weakness 1).

      There are mismatches in the statistical symbols shown in Figure 4 and Supplementary Table 2. It seems that there was a swap between the symbols for the noise between-condition and noise within-condition.

      Thank you, this has now been fixed.

    1. Author response:

      The following is the authors’ response to the original reviews.

      Public Reviews:

      Reviewer #1 (Public Review): 

      (1) In Figure 1, the authors show that TF3C binds to the amino terminus of MYCN (Myc box I region), as shown previously. The data in Figure 1 B-D support, but do not rigorously confirm a 'direct' interaction because it has not been ruled out that accessory proteins mediating the association may be present in the mixture.

      In Figure 1B-D we have purified MYCN and the TFIIIC/TauA complex separately and then mixed the purified preparations, demonstrating that the purified proteins interact. We have additionally performed mass spectrometry, which shows that the TauA/MYCN complex is formed without further accessory proteins, as the molecular weight would be higher. Based on the Coomassie stained SDS-PAGE gels, there is no plausible contaminating band in the purified complex that could be mediating the interaction between MYCN and TauA, either in the purified complex (Figure 1C), or in the purified protein used to reconstitute the complex (Figure S1A & S1B).

      (2) The authors indicate in Figure 2 that TF3C has essentially no effect on MYCNdependent gene expression and/or transcription elongation. Yet a previous study (PMID: 29262328) associated with several of the same authors concluded that TF3C positively affects transcription elongation. The authors make no attempt to reconcile these disparate results and need to clarify this point.

      We agree that the data in this manuscript do not support the role on transcription elongation. This point was also raised by Reviewer 3. Comparing our new results to the data published previously we can summarize that the data sets in the two studies show three key results: First, the traveling ratio of RNAPII changes upon induction of MYCN. Second, RNAPII decreases at the transcription start side and third, it increases towards the end side.

      We agree that in the previous study we linked the traveling ratio directly to elongation. However performing ChIP-seq with different RNAPII antibodies showed us that for example RNAPII (N20), which is unfortunately discontinued, gives different results compared to RNAPII (A10). Combining our new results using the RNAPII (8WG16) antibody shows that the traveling ratio is not only reflecting transcription elongation but also includes that the RNAPII is kicked-off chromatin at the start side.

      (3) Figures 2B and C show that unphosphorylated pol2 is TSS-centered, and Ser2-P pol2 occupation is centered beyond the TES. From this data, however, the reader can't tell how much of the phospho-Ser2- pol2 is centered on the TSS. The authors should include overall plots over TSS and TES, and also perhaps the gene-body to allow a better comparison for TSS and TES plotted for both antibodies over the collected gene sets.

      We focused on the TSS for unphosphorylated RNAPII and the TES for pSer2-RNAPII, as these are the regions with specific enrichment of the respective antibodies. As requested for comparison, we now include metagenes showing TSS, gene-body, and TES for both antibodies as new Figure S2A and B. Additionally, we included density plots for unphosphorylated RNAPII at the TES as well as for pSer2-RNAPII at the TSS as a Figure for the Reviewers (Figure 1).

      (4) The authors see more TF3C at promoters in cells with MYCN (Figure 2F). What are the levels of TF3C in the absence and presence of MYCN?

      As shown in the immunoblot in Figure S1E, TF3C5 levels do not change upon induction of MYCN. We therefore think that MYCN helps to recruit TFIIIC5 to RNAPII promoter sites. This is also in accordance to what we previously reported 1.

      (5) The finding that TF3C is increased at TSS (Figure 2F) doesn't necessarily indicate that 1) MYCN is recruiting TF3C there, and 2) that this is due to the phosphorylation status of pol2. It could mean many other things. The logic of conflating these 3 points based on the data shown is questionable.

      We showed previously that knock-down of MYCN affects TFIIIC5 binding, showing that MYCN is required for binding of TFIIIC5 at promoter sites 1.

      Additionally, we included data with DRB treated cells (Figure 2F), which prevents RNAPII loading by preventing downstream de novo elongation. Those data show that TFIIIC5 binding at the TSS is massively increased upon induction of MYCN and additionally upon treatment with DRB. Conversely, we observed that the major effect of TFIIIC knock-down was at the nonphosphorylated RNAPII at the TSS on MYCN induction (Figure 2B). Therefore, we would argue that our assumption fits well to the data presented in the manuscript.

      (6) Figure 3A doesn't add much to the paper, as it is overplotted and no relationship is clear, except that Pol2 and MYCN occupy many of the same sites. Perhaps a less complex or different type of plot would allow the interactions to be better visible.

      We agree with the comment and since in another comment we were asked to show the same window for all shown Hi-ChIP data plots, we changed Figure 3A.

      (7) That depletion of TF3C leads to increased promoter hubs may or may not have anything to do with its association with MYCN (Figure 4E). This could be a direct consequence of its known structural function in cohesin complexes, and the MYCN changes as a secondary consequence of this (also see point 4, above).

      As shown in Büchel et al. (2017) 1 MYCN is needed to recruit RAD21 and depletion of RAD21 has no impact on the recruitment of MYCN. Since RAD21 is part of the cohesin complex we would exclude that the MYCN changes are a secondary consequence.

      (8) Depletion of TF3C5 results in a loss of EXOSC5 (exosome) at TSS in the presence and absence of MYCN (Figure 5B). As TF3C5 is a cohesin, could this simply be a consequence of genomic structure changes?

      We agree that the discovered changes in EXOSC5 can be due to depletion of TFIIIC5. TFIIIC has been shown to recruit cohesin 1 and condensin complexes 2, as well as inducing chromatin architectural changes 3. However, MYCN is needed to recruit TFIIIC and depletion of TFIIIC had no impact on MYCN recruitment 1. Furthermore, MYCN has been shown to recruit exosome 4. Therefore, we would argue that either MYCN can directly play a role or thru chromatin architectural changes.

      (9) The authors suggest that RNA dynamics are affected by changes in exosome function (RNA degradation, etc). What effect, if any does TF3C depletion have on the overall gene expression profile?

      We show in the manuscript that TFIIIC depletion in unperturbed cells has no effect on the global gene expression profile in the time frame analyzed (Figure 2E and S2B).

      Reviewer #2 (Public Review):

      (1) Dynamic inferences are made without kinetic experiments.

      While we agree that we did not collect kinetic data to study the dynamics of RNA polymerase we would argue that the integration of our different data sets make it possible to draw conclusions about dynamic interferences. The transcription cycle and its sequential steps have been well described. In this sense, we use the non-phosphorylated RNAPII data that is situated between RNAPII recruitment and initiation and RNAPII-pSer2 that shows pause-release to elongation to draw conclusions on the dynamic. Likewise, we also made use of our previous published datasets.

      Reviewer #2 (Recommendations For The Authors):  

      (1) A number of changes are reported in hub size, expression, etc. upon treatment with tamoxifen to activate MCN-ER. But MYC is already present in the SHEP cells, so why doesn't MYC support these same phenomena? It would seem that either the ability to cooperate with TFIIIC to clear non-productive polymerase complexes from promoters is particular to MYCN, or else it reflects a quantitative increase in total MYC proteins due to the entry of MYCN-ER into the nucleus with tamoxifen. The authors should address or discuss this issue.

      It could be that protein levels are the limiting factor between MYC and MYCN observed effects in this system. This interpretation would be in accordance with the results of Lorenzin et al. 5, which reported that different levels of MYC had different targets based on the affinity to Eboxes and protein level. A similar profile of MYC levels compared to function was also reported regarding SPT5 6. Those high protein levels mimic what is found in certain tumors in contrast to physiological levels. In this sense, the observed differences can also be between physiological and oncological levels of MYC proteins.

      On the other hand, it has been described both a core MYC- and an isoform specific-signature of target genes. MYCN is described to be involved in gene expression during the S-phase of the cell cycle 7. This suggests that there are differences between MYC and MYCN other than gene sets. The interaction with TFIIIC appears to be one of these differences. We have found multiple TFIIIC subunits as part of the MYCN interactome, but the interaction of TFIIIC with MYC is weaker and we are uncertain how relevant it is 7,8. We show here that depletion of different subunits of the TFIIIC complex show a MYCN-dependent growth defect (Figure 1 E). Similarly, nuclear exosome is a MYCN-specific dependence 4, and we show here that MYCNdependent recruitment of the exosome requires TFIIIC5. We take this as an indication that there is an intrinsic difference between MYC and MYCN and that MYCN engages TFIIIC for this pathway.

      (2) Reciprocal to TFIIIC recruitment to MYCN- rRNA, and other RNAPIII genes. Does this happen targets would be MYCN association with tRNA genes, 5S, and if so, is this association TFIIIC dependent? What happens to the expression of these genes?

      We did observe MYCN in interactions involving tRNA and other RNAPIII sites, such as SINE elements and tRNAs (Figure 4B, 4D, S3F, and S4B). There was no relevant number of 5S rRNA involved in interactions – either because the difficulty to properly map these repetitive regions or due to biology. In any case, none of those regions appeared to be specifically dependent on TFIIIC as the overall number of interactions increased in TFIIIC depletion regardless of the genomic annotation (Figure S4B). Regarding the expression of RNAPIII genes, we are constrained by technical limitations of poly(A) enrichment RNA-seq to globally analyze it in an unbiased way. However, we addressed this point for tRNAs expression in an earlier work 1 and found that tRNA levels do not change upon TFIIIC depletion. We think this is because tRNAs are stable transcripts and RNAPIII recycling can occur in a TFIIICindependent manner 9. Conversely, we reported no significant expression changes in RNAPII genes upon TFIIIC depletion in this work.

      (3) The authors show that TFIIIC depletion does not alter the RNA-expression profile; how do they account for this? Can they comment on "background" transcription that it would seem should be suppressed by TFIIIC-dependent removal of various hypofunctional polymerases?

      Since TFIIIC is important for the removal of non-functional RNAPII we would not expect changes to the gene expression profile upon depletion of TFIIIC in the time frame analyzed. Monitoring the elongating form of RNAPII by measuring pSer2 indeed shows us that transcription elongation is not affected.

      (4) Global changes in expression are difficult to assess with DESEQ2. This hypernormalizing algorithm is not really suited to distinguish differential, but universal upregulation from some targets being truly upregulated while others are downregulated. The authors should comment.

      The authors acknowledge that DESEQ2 relies on the conjecture that genewise estimates of dispersion are generally unchanged among samples. We address this comment in two different ways. We include those in the Figure for the Reviewers (Figure 2). The first was to sequence samples deeper to avoid any bias created by random effect of lower coverage, the range of total reads increased from 6.8-9.3 to 16.5-20.7 million reads. The second was to compare the fold average bin dot plot for RNA-seq of SH-EP-MYCN-ER showing mRNA expression normalized by control per bin using the DESEQ2 (Figure 2A) normalization to TMM in edgeR (Figure 2B) and to quantile normalization (Figure 2C). No major differences were found from the original data or using the different methods, but we updated the Figure 2E in the manuscript to include the deeper sequencing dataset, we also adjusted it to show -/+ MYCN and transformed to log2 to make it more intuitive. Overall, it enhances our original understanding that gene expression remains largely unaffected by TFIIIC5 knockdown.

      (5) On page 7, the authors claim that MYCN-ER increased Ser-2 can reflect MYCN-stimulated transcription elongation. In fact, without kinetic studies, this is not fully supported. Accumulation of Ser-2 RNAPII along a gene can reflect increased initiation of full-speed RNAPs or a pile-up of RNAPs slowing down. This should be resolved or qualified.

      While we agree that we did not collect kinetic data to study the dynamics of RNA polymerase we would argue that the integration of our different data sets make it possible to draw conclusions about dynamic interferences. We showed on the one side that pSer-2 accumulates on the TES and on the other side the induction of MYCN-ER up-regulates gene expression which proves productive transcription elongation.

      (6) pLHiChIP needs to be better described, the Mumbach reference is not sufficient.

      We have reformulated the pLHiChIP in the method section and hope that this will provide now a better description of the method.

      (7) Can the authors recheck all the labels in Figure 2D-I believe there is an error involving + or - MYCN.

      We carefully rechecked all the labels in Figure 2 and it was correct as it was. We understand the confusion that may have created comparing Figure 2D and Figure 2E. To avoid confusion, we updated Figure 2E to show the same direction of Figure 2D. We also log2 transformed the y-axis of Figure 2E to foster a more intuitive reading.

      (8) Why are there different scales for the regions of chromosome 17 shown in Figures 3 and 4? It would be easier to compare if the examples were all shown at the same scale (about 2 MB is shown in another Figure).

      We now show the same region of chromosome 17 in Figure 3 and 4.

      Reviewer #3 (Public Review):

      (1) The connection between the three major findings presented in this study regarding the role of TFIIIC in the regulation of MYCN function remains unclear. Specifically, how the TFIIICdependent restriction of MYCN localization to promoter hubs enhances the association of factors involved in nascent RNA degradation to prevent the accumulation of inactive RNA polymerase II at promoters is not apparent. As they are currently presented, these findings appear as independent observations. Cross-comparison of the different datasets obtained may provide some insight into addressing this question.

      We previously observed that TFIIIC does not affect MYCN recruitment, while MYCN affects TFIIIC binding 1. Moreover, our group reported that MYCN recruits exosome 4 and BRCA1 to promoter-proximal regions 10 to clear out non-functional RNAPII. We are currently reporting that MYCN-TFIIIC complexes exclude non-functional RNAPII. However, MYCN-active promoter hubs have more RNAPII and more transcription than MYCN-active promoter outside hubs. Furthermore, TFIIIC binding occurs upstream of BRCA1 and exosome recruitments as depletion of TFIIIC leads to recruitment decrease of both factors. Therefore, we argue that TFIIIC is required for the proper function of those MYCN-active promoter hubs.

      (2) Another concern involves the disparities in RNA polymerase II ChIP-seq results between this study and earlier ones conducted by the same group. In Figure 2, the authors demonstrate that activation of MYCN results in a reduction of non-phosphorylated RNA polymerase II across all expressed genes. This discovery contradicts prior findings obtained using the same methodology, where it was concluded that the expression of MYCN had no significant effect on the chromatin association of hypo-phosphorylated RNA polymerase II (Buchel et al, 2017). In this regard, the choice of the 8WG16 antibody raises concern, as fluctuations in the signal may be attributed to changes in the phosphorylation levels of the Cterminal domain. It remains unclear why the authors decided against using antibodies targeting the N-terminal domain of RNA polymerase II, which are unaffected by phosphorylation and consistently demonstrated a significant signal reduction upon MYCN activation in their previous studies (Buchel et al, 2017) (Herold et al, 2019). Similarly, the authors previously proposed that depletion of TFIIIC5 abrogates the MYCN-dependent increase of Ser2phosphorylated RNA polymerase II (Buchel et al, 2017), whereas they now show that it has no obvious impact. These aspects need clarification.

      We politely disagree that our discoveries are contradicting each other. Comparing our new results to the data published previously we can summarize that the data sets in the two studies show three key results: First, the traveling ratio of RNAPII changes upon induction of MYCN. Second, RNAPII decreases at the transcription start side and third, it increases towards the end side.

      We agree that in the previous study we linked the traveling ratio directly to elongation. However performing ChIP-seq with different RNAPII antibodies showed us that for example RNAPII (N20), which is unfortunately discontinued, gives different results compared to RNAPII (A10). Combining our new results using the RNAPII (8WG16) antibody shows that the traveling ratio is not only reflecting transcription elongation but also includes that the RNAPII is kicked-off chromatin at the start side.

      In the previous study we only performed manual ChIP experiments for RNAPII (8WG16) and pSer2. Now we did a global analysis which is more meaningful and is also reflected in the RNA sequencing data.

      (3) Finally, the varied techniques employed to explore the role of TFIIIC in MYCNdependent recruitment of nascent RNA degradation factors make it challenging to draw definitive conclusions about which factor is affected and which one is not. While conducting ChIPseq experiments for all factors may be beyond the scope of this manuscript, incorporating proximity ligation assays (PLA) or ChIP-qPCR assays with each factor would have enabled a more direct and comprehensive comparison.

      We understand the criticism that we are comparing different assays. We have performed PLAs with different antibodies. Since the controls of the PLAs were not sufficient for us, we refrain from using them. ChIP-qPCR experiments are much more challenging to do side by side compared to PLAs, which is why we decided against looking at all factors with this method.

      Recommendations For The Authors:

      Reviewer #3 (Recommendations For The Authors):

      (1) Figure 2: Why did the authors choose the 8WG16 antibody? Does TFIIIC5 depletion suppress the MYCN-dependent reduction of total RNA polymerase II binding to promoters that they consistently showed in previous studies? Given that phosphorylation of the CTD impacts 8WG16 recognition, including Ser5-phosphorylated RNA polymerase II ChIPseq experiments might clarify this issue.

      We used the RNAPII (8WG16) antibody to exactly map non-phosphorylated RNAPII which shows us the binding of non-functional RNAPII.

      (2) Figures 3 and 4: As it stands, the manuscript does not convincingly establish a functional connection between the results in Figures 2, 3, and 4 or elucidate potential mechanisms. Are changes in RNA polymerase II levels upon MYCN activation more pronounced at promoters located at MYCN hubs? Do changes in MYCN-enriched chromatin contacts upon TFIIIC5 depletion somehow correlate with alterations in RNA polymerase II levels? Performing similar cross-comparisons as in Figure 3C may help address this issue. Furthermore, it not clear how the authors concluded that MYCN/TFIIIC5-bound genes are not part of these so-called promoter hubs.

      In Figure 3C we show that RNAPII levels are more pronounced upon MYCN activation at promoters located at MYCN hubs. Additionally, we show non-phosphorylated ChIP-seq on TSS and RNAPII-pSer2 ChIP-seq on TES density plots for promoters with MYCN interactions in the Figure for the Reviewers (Figure 3). We found no other difference than binding compared to the overall global analysis for all expressed genes showed in Figure 2B and Figure 2C. This goes on the same direction of the high expression observed of those genes in MYCN interactions observed in Figure 3C.

      The changes observed in Figures 2B and 2C are global and do include the promoters with MYCN interactions. At the same time, it is required a higher number of replicates to statistically distinguish the MYCN interaction differences between TFIIIC5 presence and depletion. We acknowledge this limitation, and we therefore restrain any attempt towards this end. We base our conclusions on the other parts of the manuscript and on our previous studies that show that MYCN recruits TFIIIC, BRCA1, and the exosome to promoter proximal regions 1,4,10.

      (3) Figure 5: According to the PLA results, activation of MYCN could enhance RNA polymerase II-NELFE interaction in a TFIIC5-dependent manner. Considering the raised issues regarding the use of the 8WG16 antibody, this result might be of relevance.

      Nevertheless, PLA does not seem to be the optimal technique to address these questions, and I would rather suggest performing ChIP-qPCR experiments for all the factors to be compared. Finally, do the authors conclude that the TFIIIC5 effect on MYCN-dependent changes in RNA polymerase II depends upon the recruitment of EXOSC5 and BRCA1? If so, it would be interesting to determine whether depletion of these factors phenocopies the effects observed with TFIIC5.

      We understand the criticism that we are comparing different assays. We have performed PLAs with different antibodies. Since the controls of the PLAs were not sufficient for us, we refrain from using them.

      (4) In Figure S2 the labels should be EtOH, 4-OHT, and Input.

      We changed this accordingly.

      (5) On page 7, the sentence "We have shown previously that TFIIIC5 depletion does not cause significant changes in expression of multiple tRNA genes that are transcribed by RNAPIII (Buchel et al., 2017)" appears to lack a connection.

      We agree with the reviewer and we deleted this sentence from the manuscript.

      Author response image 1.

      (A) Density plot of ChIP-Rx signal for non-phosphorylated RNAPII. Data show mean (line) ± standard error of the mean (SEM indicated by the shade) of different gene sets based on an RNA-seq of SH-EP-MYCN-ER cells ± 4-OHT. The y-axis shows the number of spike-in normalized reads and it is centered to the TES ± 2 kb. N = number of genes in the gene set defined in the methods. (B) Density plot of ChIP-Rx signal for RNAPII pSer2 as described for panel A. The signal is centered to the TSS ± 2 kb.

      Author response image 2.

      Bin dot plot for RNA-seq of SH-EP-MYCN-ER showing mRNA expression normalized by control per bin comparing the fold average using DESEQ2 (A), normalization to TMM in edgeR (B) and to quantile normalization (C).

      Author response image 3.

      Average density plot of ChIP-Rx signal for non-phosphorylated RNAPII (A) or RNAPII pSer2 (B) at promoters with MYCN interactions.

      References

      (1) Büchel, G., Carstensen, A., Mak, K.-Y., Roeschert, I., Leen, E., Sumara, O., Hofstetter, J., Herold, S., Kalb, J., and Baluapuri, A. (2017). Association with Aurora-A controls NMYC-dependent promoter escape and pause release of RNA polymerase II during the cell cycle. Cell reports 21, 3483-3497.

      (2) Yuen, K.C., Slaughter, B.D., and Gerton, J.L. (2017). Condensin II is anchored by TFIIIC and H3K4me3 in the mammalian genome and supports the expression of active dense gene clusters. Sci Adv 3, e1700191. 10.1126/sciadv.1700191.

      (3) Ferrari, R., de Llobet Cucalon, L.I., Di Vona, C., Le Dilly, F., Vidal, E., Lioutas, A., Oliete, J.Q., Jochem, L., Cutts, E., Dieci, G., et al. (2020). TFIIIC Binding to Alu Elements Controls Gene Expression via Chromatin Looping and Histone Acetylation. Mol Cell 77, 475-487 e411. 10.1016/j.molcel.2019.10.020.

      (4) Papadopoulos, D., Solvie, D., Baluapuri, A., Endres, T., Ha, S.A., Herold, S., Kalb, J., Giansanti, C., Schulein-Volk, C., Ade, C.P., et al. (2021). MYCN recruits the nuclear exosome complex to RNA polymerase II to prevent transcription-replication conflicts. Mol Cell. 10.1016/j.molcel.2021.11.002.

      (5) Lorenzin, F., Benary, U., Baluapuri, A., Walz, S., Jung, L.A., von Eyss, B., Kisker, C., Wolf, J., Eilers, M., and Wolf, E. (2016). Different promoter affinities account for specificity in MYC-dependent gene regulation. Elife 5. 10.7554/eLife.15161.

      (6) Baluapuri, A., Hofstetter, J., Dudvarski Stankovic, N., Endres, T., Bhandare, P., Vos, S.M., Adhikari, B., Schwarz, J.D., Narain, A., Vogt, M., et al. (2019). MYC Recruits SPT5 to RNA Polymerase II to Promote Processive Transcription Elongation. Mol Cell 74, 674-687 e611. 10.1016/j.molcel.2019.02.031.

      (7) Baluapuri, A., Wolf, E., and Eilers, M. (2020). Target gene-independent functions of MYC oncoproteins. Nat Rev Mol Cell Biol. 10.1038/s41580-020-0215-2.

      (8) Koch, H.B., Zhang, R., Verdoodt, B., Bailey, A., Zhang, C.D., Yates, J.R., 3rd, Menssen, A., and Hermeking, H. (2007). Large-scale identification of c-MYCassociated proteins using a combined TAP/MudPIT approach. Cell Cycle 6, 205-217. 10.4161/cc.6.2.3742.

      (9) Ferrari, R., Rivetti, C., Acker, J., and Dieci, G. (2004). Distinct roles of transcription factors TFIIIB and TFIIIC in RNA polymerase III transcription reinitiation. Proc Natl Acad Sci U S A 101, 13442-13447. 10.1073/pnas.0403851101.

      (10) Herold, S., Kalb, J., Büchel, G., Ade, C.P., Baluapuri, A., Xu, J., Koster, J., Solvie, D., Carstensen, A., and Klotz, C. (2019). Recruitment of BRCA1 limits MYCN-driven accumulation of stalled RNA polymerase. Nature 567, 545-549.

    1. Author response:

      The following is the authors’ response to the original reviews.

      Reviewer #1 (Public Review):

      Summary:

      The manuscript by Dubicka and co-workers on calcification in miliolid foraminifera presents an interesting piece of work. The study uses confocal and electron microscopy to show that the traditional picture of calcification in porcelaneous foraminifera is incorrect.

      Strengths:

      The authors present high-quality images and an original approach to a relatively solid (so I thought) model of calcification.

      Weaknesses:

      There are several major shortcomings. Despite the interesting subject and the wonderful images, the conclusions of this manuscript are simply not supported at all by the results. The fluorescent images may not have any relation to the process of calcification and should therefore not be part of this manuscript. The SEM images, however, do point to an outdated idea of miliolid calcification. I think the manuscript would be much stronger with the focus on the SEM images and with the speculation of the physiological processes greatly reduced.

      We agree that fluorescence studies presented in the paper are not an unequivocal proof by itself for calcification model utilised by studied Miliolida species. However, fluorescence data combined with SEM studies, especially overlap of the elements that show autofluorescence upon excitation at 405 nm (emission 420–480 nm) and acidic vesicles marked by p_H-_sensitive LysoGlow84, may be a hint indicating ACC-bearing vesicles.

      We will tone down the the physiological interpretation based on fluorescence studies in the revised version of the manuscript.

      Nevertheless, we think that our fluorescent life-imaging experiments provides important observations in miliolida, which is scarce in the existing literature, and therefore are worth being presented as they might be very helpful in better understanding of full calcification model in the future.

      Reviewer #2 (Public Review):

      Summary:

      Dubicka et al. in their paper entitled " Biocalcification in porcelaneous foraminifera" suggest that in contrast to the traditionally claimed two different modes of test calcification by rotallid and porcelaneous miliolid formaminifera, both groups produce calcareous tests via the intravesicular mineral precursors (Mg-rich amorphous calcium carbonate). These precursors are proposed to be supplied by endocytosed seawater and deposited in situ as mesocrystals formed at the site of new wall formation within the organic matrix. The authors did not observe the calcification of the needles within the transported vesicles, which challenges the previous model of miliolid mineralization. Although the authors argue that these two groups of foraminifera utilize the same calcification mechanism, they also suggest that these calcification pathways evolved independently in the Paleozoic.

      We do not argue that Miliolida and Rotallida utilize exactly the same calcification mechanism but the both groups use less divergent crystallization pathways, where mesocrystalline chamber walls are created by accumulating and assembling particles of pre-formed liquid amorphous mineral phase.

      Strengths:<br /> The authors document various unknown aspects of calcification of Pseudolachlanella eburnea and elucidate some poorly explained phenomena (e.g., translucent properties of the freshly formed test) however there are several problematic observations/interpretations which in my opinion should be carefully addressed.

      Weaknesses:

      (1) The authors (line 122) suggest that "characteristic autofluorescence indicates the carbonate content of the vesicles (Fig. S2), which are considered to be Mg-ACCs (amorphous MgCaCO3) (Fig. 2, Movies S4 and S5)". Figure S2 which the authors refer to shows only broken sections of organic sheath at different stages of mineralization. Movie S4 shows that only in a few regions some vesicles exhibit red autofluorescence interpreted as Mg-ACC (S5 is missing but probably the authors were referring to S3). In their previous paper (Dubicka et al 2023: Heliyon), the authors used exactly the same methodology to suggest that these are intracellularly formed Mg-rich amorphous calcium carbonate particles that transform into a stable mineral phase in rotaliid Aphistegina lessonii. However, in Figure 1D (Dubicka et al 2023) the apparently carbonate-loaded vesicles show the same red autofluorescence as the test, whereas in their current paper, no evidence of autofluorescence of Mg-ACC grains accumulated within the "gel-like" organic matrix is given. The S3 and S4 movies show circulation of various fluorescing components, but no initial phase of test formation is observable (numerous mineral grains embedded within the o rganic matrix - Figures 3A and B - should be clearly observed also as autofluorescence of the whole layer). Thus the crucial argument supporting the calcification model (Figure 5) is missing.

      This is correct that we did not observe the initial phase of test formation in vivo. Therefore, it is not our crucial argument supporting novel components of the new calcification model. We suspect that vesicles preparing and transporting Mg-ACC are produced way before their docking and deposition into the new wall, because such seawater vesicles were observed between the chamber formation stages (Goleń and Tyszka, 2024, personal communication based on independent experiments on a closely related miliolid taxon). It means that our in vivo experiments most likely represent a long, dynamic stage of vesicles formation via seawater endocytosis, their modification (incl. Mg-ACC formation) before the stage of exocytosis during the new chamber formation. Our crucial arguments supporting the calcification model come from the SEM imaging of the specimens fixed during chamber formation, as well as from the transparency of the new chamber wall during its progressive calcification.

      There is no support for the following interpretation (lines 199-203) "The existence of intracellular, vesicular intermediate amorphous phase (Mg-ACC pools), which supply successive doses of carbonate material to shell production, was supported by autofluorescence (excitation at 405 nm; Fig. 2; Movies S3 and S4; see Dubicka et al., 2023) and a high content of Ca and Mg quantified from the area of cytoplasm by SEM-EDS analysis (Fig. S6)."

      We used laser line 405nm and multiphoton excitaton to detect ACCs. These wavelengths (partly) permeate the shell to excite ACCs autofluorescence. The autofluorescence of the shells is present as well but not clearly visible in movieS4 as the fluorescence of ACCs is stronger. This may be related to the plane/section of the cell which is shown. The laser permeates the shell above the ACCs (short distance) but to excite the shell CaCO3 around foraminifera in the same three-dimensional section where ACCs are shown, the light must pass a thick CaCO3 area due to the three-dimensional structure of the foraminiferan shell. Therefore, the laser light intensity is reduced. In a revised version a movie/image with reduced threshold is shown.

      Author response image 1.

      Autofluorescence image of studied Miliolida species (exc. 405 nm) showing algal chlorophyll (blue) and CaCO3 (red), both ACC and calcite shell.

      It would be very convenient if it was possible to visualize ACC by illumination with a blacklight, but there are very many organic molecules that have an autofluorescence excited by ~405 nm. One of the examples is NADH (Lee et al., 2015. Kor J Physiol Pharmac 19(4): 373-382), an omnipresent molecule in any cell (couldn't copy the appropriate picture here, but the reference has a figure with the em/exc spectra).

      The paper of Lee et al. 2015 shows that the excitation spectrum of NADH is ending close to 400 nm. This means that NADH is not or only very weakly excitable at 405nm, what we used as the excitation laser line. 

      (2) The authors suggest that "no organic matter was detected between the needles of the porcelain structures (Figures 3E; 3E; S4C, and S5A)". Such a suggestion, which is highly unusual considering that biogenic minerals almost by definition contain various organic components, was made based only on FE-SEM observation. The authors should either provide clearcut evidence of the lack of organic matter (unlikely) or may suggest that intense calcium carbonate precipitation within organic matrix gel ultimately results in a decrease of the amount of the organic phase (but not its complete elimination), alike the pure calcium carbonate crystals are separated from the remaining liquid with impurities ("mother liquor"). On the other hand, if (249-250) "organic matrix involved in the biomineralization of foraminiferal shells may contain collagen-like networks", such "laminar" organization of the organic matrix may partly explain the arrangement of carbonate fibers parallel to the surface as observed in Fig. 3E1.

      We agree with the reviewer that biogenic minerals should by definition contain some organic components. We just wrote that "no organic matter was detected between the needles of the porcelain structures” that means that we did not detect any organic structures based only on our FE-SEM observations. We will rephrase this part of the text to avoid further confusion.

      (3) The author's observations indeed do not show the formation of individual skeletal crystallites within intracellular vesicles, however, do not explain either what is the structure of individual skeletal crystallites and how they are formed. Especially, what are the structures observed in polarized light (and interpreted as calcite crystallites) by De Nooijer et al. 2009? The author's explanation of the process (lines 213-216) is not particularly convincing "we suspect that the OM was removed from the test wall and recycled by the cell itself".

      Thank you for this comment. We will do our best to supplement our explanations. We are aware about the structures observed in polarized light by De Nooijer et al. (2009). However, Goleń et al. (2022, Prostist; + 2 other citations) showed that organic polymers may also exhibit light polarization. Additional experimental studies are needed to separate these types of polarization. We will try to investigate this issue in our future research.

      (4) The following passage (lines 296-304) which deals with the concept of mesocrystals is not supported by the authors' methodology or observations. The authors state that miliolid needles "assembled with calcite nanoparticles, are unique examples of biogenic mesocrystals (see Cölfen and Antonietti, 2005), forming distinct geometric shapes limited by planar crystalline faces" (later in the same passage the authors say that "mesocrystals are common biogenic components in the skeletons of marine organisms" (are they thus unique or are they common)? It is my suggestion to completely eliminate this concept here until various crystallographic details of the miliolid test formation are well documented.

      Our intension was to express that mesocrystals are common biogenic components in the skeletons of marine organisms however such a miliolid needles forming distinct geometric shapes limited by planar crystalline faces are unique.

      Reviewer #1 (Recommendations For The Authors):

      Below, I have summarized my main criticisms.

      (1) The movies S1-S4 do not indicate what is described. The videos show indeed seawater (S1), cell membranes (S2), and autofluorescence and acidic vesicles (S3 and S4). The presence of all these intracellular structures is not surprising: any eukaryotic cell will have those. The authors, however, claim that they participate in the process of calcification, which is simply not shown. One of the main arguments seems the presence of 'carbonate pools', in the caption these are even claimed to be 'Mg-ACC pools', but this is by no means revealed by an excitation of 405nm/ emission between 420 and 490 nm. It would be very convenient if it was possible to visualize ACC by illumination with a blacklight, but there are very many organic molecules that have an autofluorescence excited by ~405 nm. One of the examples is NADH (Lee et al., 2015. Kor J Physiol Pharmac 19(4): 373-382), an omnipresent molecule in any cell (couldn't copy the appropriate picture here, but the reference has a figure with the em/exc spectra).

      The paper of Lee et al. 2015 shows that the excitation spectrum of NADH is ending close to 400 nm. This means that NADH is not or only very weakly excitable at 405nm, what we used as the excitation laser line. 

      The fluorescence by this excitation/ emission couple unlikely indicates the vesicles in which these foraminifera calcify. Therefore, most of the interpretation of the authors on what happens with the calcitic needles is not based on results but remains pure speculation.

      The fluorescence autofluorescence upon excitation at 405 nm (emission 420–480 nm is typical for CaCO3 both for biocalcite and amorphous calcium carbonate, what was proven by laboratory synthesis of amorphous calcium carbonate (Dubicka et al., in preparation).

      (2) The results mention 'granules', which are the supposed Mg-ACC-containing vesicles, but the movies simply don't show any granules. Only fluorescence. Again, the results show a lot of vesicles with autofluorescence, but these are not necessarily related to calcification. Proof could be supplied by showing that the same fluorescent vesicles are 'used up' when the specimens under observation are making a new chamber, but until that is done, the fate of all these vesicles remains uncertain and once more, may not be involved in calcification at all.

      We suspect that vesicles preparing and transporting Mg-ACC are produced way before their docking and deposition into the new wall, because such seawater vesicles were observed between the chamber formation stages (Goleń and Tyszka, 2024, personal communication based on independent experiments on a closely related miliolid taxon). It means that our in vivo experiments most likely represent a long, dynamic stage of vesicles formation via seawater endocytosis, their modification (incl. Mg-ACC formation) before the stage of exocytosis during the new chamber formation. Our crucial arguments supporting the calcification model come from the SEM imaging of the specimens fixed during chamber formation, as well as from the transparency of the new chamber wall during its progressive calcification.

      (3) The Methods are unclear. How long were the foraminifers kept before being placed under the microscope? Were they fed with anything? This is important since the chlorophyll should not be from any food source. I didn't know that this foraminiferal species has photosynthetic symbionts: genera like Quinqueloculina don't. Is there any reference for this? Normally, I wouldn't care that much, but the authors find the presence of (facultative) symbionts important (lines 305-336). I am a bit suspicious about this since the only evidence for the presence of photosynthetic symbionts is because of the autofluorescence. As the authors said, commonly these miliolid species are regarded as symbiont-barren, so additional proof for these symbionts is necessary.

      We agree that additional proof is needed for the presence of photosynthetic symbionts. We rephrased the manuscript accordingly.

      (4) It is also unclear (Methods) at what stage the miliolids were photographed (Figure 3). How did chamber formation proceed, what was the timing of the photographs, etc. These pictures are to me the most interesting finding of this study, but need to be described much better.

      All individuals of living foraminifera were fixed at the overall stage of chamber formation. However, every individual presents a complete set of successive steps (substages) of chamber wall calcification fixed at once. Fig. 3A and B present nearly the most proximal (youngest) part of the new chamber with a thick wall of calcite nanograins within a gel-like organic matrix. Fig. 3C and D present a bit more distal (intermediate) part of the calcified chamber. Fig. 3E shows the most distal part of the new chamber. This part is anchored to the older, underlying solid calcified chamber (not shown in this figure). All these steps are synchronous, however, represent gradual successive stages of calcification. The main text and Figs 4 and 5 explain this phenomenon in details.

      There are many small issues with the text too. These include:

      Line 28/29: in many other groups, calcification is thought to be polyphyletic (e.g. sponges: Chombard et al., 1997. Biol Bull 193: 359-367).

      Corrected

      Line 29/30: there may be even more 'types of shells'. The first author has shown in earlier papers that nodosarids have a unique shell architecture. Spirillinids also seem to have their own way of calcification. It is unclear what is meant here by 'two contrasting models'.

      By now there are known only two models of foraminiferal calcification. Lagenida biocalcification has not been studied.

      Line 33: 'Both groups'? This paper only shows calcification in miliolids.

      However, we refer to previous study.

      Line 42: Perhaps, but there is no data on the pseudopodial network in this manuscript.

      We refer to Angell, 1980 studies

      Line 43: Likely, but that is not what this manuscript is showing.

      Line 42-44: The authors should make a choice and be clear. The point of this paper is that miliolids and rotalids calcify in ways that are actually not as different as they seemed previously. Still, they are said to have different 'chamber formation modes'. If they are calcifying in a similar way (which I think is not necessarily supported by the results), isn't calcification in these groups like variations on the same theme? How does this relate to the independent origins of calcification within these two groups?

      Our intension is to show that Miliolida and Rotaliida utilize less divergent calcification pathways, following the recently discovered biomineralization principles.

      Line 49-51: is this a well-established distinction? If so, please add a reference. If not: what is fundamentally different between B and C? Does only the size of the intracellular vesicle matter?

      Rephrased

      Line 60: please include a reference for the intracellular calcification by coccolithophores.

      Added

      Line 67: this is wrong. It is the alignment of the needles at the surface that makes them all reflect light in the same way and gives the shells a porcelaneous appearance. A close-up of the miliolid's shell surface shows this arrangement. Underneath this layer, the orientation of the needles is more random.

      We referred to Johan Hohenegger papers.

      Line 114: how else?

      Line 114-116: I don't see the relevance here. If seawater is taken up, the vesicle containing this seawater has to have a membrane around it. By definition. The text here ('These vesicles') suggests that Calcein and FM1-43 were combined (which they easily could have), but the methods describe that they are used successively.

      Yes, we used two dyes separately.

      Lines 122-130: I think the interpretation of this autofluorescence signal is wrong. Even if it was true, these lines belong to the Discussion.

      This paragraph has been placed within discussion

      Line 138: What are 'mobile clusters'? I don't see a relation between the location of the symbionts and the other vesicles (Figure 2).

      Line 147-148: How can an SEM image show the absence of organic matter?

      We meant the absence of the gel-like OM visible in the previous stages of the chamber formation

      Line 148: Should be 'Figs. 3E; 3E1; S4C'.

      Corrected

      Lines 143-150: this can be merged with the following paragraph.

      Done

      Lines 151-169: why is there no indication of the time? Figures 3 and 4 link the pictures in time to show the development of the growing chamber wall. However, neither here nor in the methods, is there any recording of the time after the beginning of chamber formation. Now, the images are linked (Figure 4) as if they were taken at regular intervals, but this is not documented.

      Lines 170-184: this should go to the Discussion.

      Done

      Line 193-195: this is likely, but not visible in Figure 1.

      It was visible by optical microscopy and described by Angell, 1980

      Line 199-201: I don't understand this: the fluorescent vesicles were not observed during chamber formation so any link between the SEM and CLSM scans remains pure speculation.

      Line 203-204: needed for what?

      For better documentation of Miliolid ACC-bearing granules

      Line 220: is this shown in any of the images? 

      Angell, 1980

      Line 230: It sounds nice, but I don't think a 'paradigm shift' is appropriate here. However interesting and important foraminiferal biomineralization is, the authors show that the crystals of miliolids are likely formed differently than previously thought. If this is a 'paradigm shift', then most scientific findings are.

      In our opinion this is definitely a shift of paradigm

      Line 231: I don't think anyone suggested miliolids and coccolithophores share 'the same' pathway. They are shown (cocco's) and thought (miliolids) to secrete their calcite intracellularly.

      Changed to similar, intracellular

      Line 258: References should only be to peer-reviewed studies.

      Line 430: Burgers'

      Corrected

      Reviewer #2 (Recommendations For The Authors):

      Please separate clearly the results (observations) from the discussion (interpretations): various interpretational/commentary phrases should be removed from the Results section to Discussion e.g., lines 124-130, 131-135.

      Interpretation have been separated from results as suggested by Reviewer.

      [line 49] " living cells have evolved three major skeleton crystallization pathways". I would rather say "organisms" not "cells" as the coordination of the calcification process in multicellular organisms clearly involves processes that are beyond the individual cell activity.

      Corrected

    1. Author response:

      The following is the authors’ response to the original reviews.

      Response to the Reviewer #1 (Public review):

      We greatly appreciate the reviewer’s high evaluation of our paper and helpful comments. As expected, we revealed that the CCL17/CCL22–CCR4 axes play an important role in guiding Tregs to the atherosclerotic aorta. Interestingly, we also demonstrated that these axes are critical for Treg-dependent regulation of proinflammatory T cell responses in lymphoid tissues and atherosclerotic aortas, which is a previously unrecognized role for CCR4 in regulating inflammatory immune responses. However, the role of the CCL17/CCL22–CCR4 axes in regulating inflammatory immune responses and atherosclerosis has not been fully elucidated and further investigation is needed.

      Response to the reviewer #2 (Public review):

      We greatly appreciate the reviewer’s high evaluation of our paper and helpful comments and suggestions. We isolated CD4<sup>+</sup>CD25<sup>+</sup> T cells and used them as Tregs in several experiments. As the reviewer pointed out, we realize that CD4<sup>+</sup>CD25<sup>+</sup> T cell population contains some activated effector T cells. However, in consideration of the high expression levels of the most reliable Treg marker Foxp3 in isolated CD4<sup>+</sup>CD25<sup>+</sup> T cells determined by flow cytometry, we believe that our method for separating Tregs would be acceptable.

      Regarding the role of Th17 cells in atherosclerosis, conflicting results have been reported. Therefore, it is unclear whether augmented Th17 cell immune responses contribute to accelerated atherosclerosis in Ccr4<sup>-/-</sup>Apoe<sup>-/-</sup> mice.

      As the reviewer pointed out, it is important to consider the clinical relevance of our findings. We analyzed public database to determine if Ccr4 single nucleotide polymorphisms correlate with a higher incidence of atherosclerotic cardiovascular disease. However, no evidence supporting the clinical relevance of our findings was found.

      Response to the Reviewer #3 (Public review):

      We greatly appreciate the reviewer’s high evaluation of our paper and helpful comments and suggestions. In accordance with the reviewer’s suggestion, we described the detailed methods and carefully performed data analysis regarding flow cytometry, which would strengthen the conclusion of this study.

      We understood the importance of reviewer’s claim that CCR4 deficiency does not shift the Th1 cell/Treg balance toward Th1 cell responses in all lymphoid tissues. CCR4 deficiency promoted the accumulation of Th1 cells but did not affect the accumulation of Tregs in the atherosclerotic aorta, which led to the shift of the Th1 cell/Treg balance toward Th1 cell responses. The frequencies of both Tregs and Th1 cells in peripheral lymphoid tissues were increased by CCR4 deficiency, while these CCR4-deficient Tregs exhibited impaired suppressive function. Given this, we speculate that CCR4 deficiency may shift the Th1 cell/Treg balance toward Th1 cell responses in peripheral lymphoid tissues. However, it is difficult to clearly show this. We revised the manuscript accordingly.

      Although the reviewer pointed out the possibility that modulation of the Th1 cell/Th17 cell balance might be responsible for the changes in aortic inflammatory cells in Ccr4<sup>-/-</sup>Apoe<sup>-/-</sup> mice, the role of Th17 cells in atherosclerosis remain controversial. However, we cannot completely exclude the possibility of the involvement of the Th17 response modulation in accelerated atherosclerosis in Ccr4<sup>-/-</sup>Apoe<sup>-/-</sup> mice.

      As the limitation of this study, the phenotypic heterogeneity and dynamics of aortic leukocytes could not be revealed by flow cytometric analysis. Single-cell proteomic and transcriptomic approaches would provide additional important information on various aortic cells including immune cells and vascular cells.

      Reviewer #1 (Recommendations for the authors):

      Issue (1) Ideally, CCR4 could be deleted on Foxp3+ cells and some staining on double positive Rorg+Foxp3+ done. On the other side, a whole gene expression of infiltrated Foxp3 and effector could be also helpful. More challenging, it would be important to see whether those CCR4-specific Trges could or not regulate effector infiltrating cells.

      As the reviewer suggested, single-cell proteomic and transcriptomic approaches would be helpful to reveal the phenotypic heterogeneity and dynamics of aortic leukocytes including Tregs. Also, the use of conditional knockout mice would reveal the precise role of CCR4-expressing Tregs in regulating aortic immune cell infiltration and atherosclerosis.

      Reviewer #2 (Recommendations for the authors):

      Minor Suggestions:

      Issue (1) In supplementary Figure 1, CCR4 expression would be better represented by dot plots rather than histograms.

      We revised Supplementary Figure 1A through 1C.

      Issue (2) The reduction in CD103 expression shown in Figure 2E at 8 weeks should be discussed.

      In Figure 2E, we found that the expression of CD103 in peripheral LN Tregs was slightly lower in 8-week-old Ccr4<sup>-/-</sup>Apoe<sup>-/-</sup> mice than in age-matched Apoe<sup>-/-</sup> mice, while there was no difference in its expression levels between 18-week-old Apoe<sup>-/-</sup> and Ccr4<sup>-/-</sup>Apoe<sup>-/-</sup> mice. In addition, there was no significant difference in the mRNA expression of this molecule in splenic Tregs between 8-week-old Apoe<sup>-/-</sup> and Ccr4<sup>-/-</sup>Apoe<sup>-/-</sup> mice. Based on the minor effect of CCR4 deficiency on CD103 expression in Tregs, reduced CD103 expression in Ccr4<sup>-/-</sup>Apoe<sup>-/-</sup> mice does not seem to be an important change.

      Issue (3) The increased expression of CD86 by DCs should be discussed.

      The upregulated CD86 expression on DCs in Ccr4<sup>-/-</sup>Apoe<sup>-/-</sup> mice might be explained by the data on a Treg-DC coculture experiment showing the impaired cell–cell contacts between CCR4-deficient Tregs and DCs. On the other hand, the expression of another important costimulatory molecule CD80 on DCs was not altered in these mice, which is not consistent with the data on the above coculture experiment. The reason why only CD86 expression on DCs was upregulated in Ccr4<sup>-/-</sup>Apoe<sup>-/-</sup> mice remains unclear.

      Issue (4) In Figures 5F-H, using larger dots would enhance visibility.

      We revised the graphs in Figure 5F-H.

      Issue (5) In Figure 5I, since the data is normalized, a one-sample t-test is more appropriate.

      In accordance with the reviewer’s suggestion, we reconsidered the data analysis. Because there was a dramatic difference in the absolute number of Kaede-expressing Tregs accumulated in the aorta among experiments, we were worried that the statistical analysis of the combined data from multiple experiments might draw a wrong conclusion. We have decided to show the representative data from 3 independent experiments in Figure 5I.

      Issue (6) On page 11, line 256, the text mentions IL4 and IL10 being detected by cytokine array; however, the figures do not show these cytokines.

      We are afraid that the reviewer might have misunderstood the data. The cytokine levels of IL-4 and IL-10 could not be detected by cytokine array analysis. Accordingly, we carefully revised the text in the manuscript.

      Issue (7). On page 14, lines 326-330, the text should be revised for clarity.

      We revised the text in the manuscript.

      Issue (8) Several data are marked as "not shown"; some of this information is relevant and should be included in the supplementary figures.

      We showed the data on CCL17 and CCL22 expression in peripheral LNs in Supplementary Figure 2.

      Major Suggestions:

      Issue (1) FoxP3 expression should be evaluated post-isolation of CD4<sup>+</sup>CD25<sup>+</sup> T cells, and FoxP3- CD4<sup>+</sup>CD25<sup>+</sup> T cells should be characterized. Tregs could be more effectively isolated using FoxP3eGFP mice.

      After isolation of CD4<sup>+</sup>CD25<sup>+</sup> T cells (the purity was >95%), we examined Foxp3 expression by flow cytometry and found that most of these cells express Foxp3 (Supplementary Figure 10). Therefore, CD4<sup>+</sup>CD25<sup>+</sup> T cells without Foxp3 expression, which are considered contaminated effector T cells, are minor cells and would not substantially affect the results. Nonetheless, the use of Foxp3-eGFP mice would enable us to isolate Tregs more accurately.

      Issue (2) In Figure 3, it would be interesting to evaluate whether there are RORgt+Tbet+ (IL17+IFNg+) cells. These cells would be pathogenic, whereas RORgt+CD73+ cells would be non-pathogenic.

      We analyzed CD4<sup>+</sup> T cells producing both IL-17 and IFN-γ in the peripheral lymphoid tissues of Apoe<sup>-/-</sup> and Ccr4<sup>-/-</sup>Apoe<sup>-/-</sup> mice. We found that this cell population was quite rare and that there was no significant difference its proportion between the 2 groups, suggesting the possible minor contribution of this cell population to the atherosclerosis phenotype.

      Author response image 1.

      Issue (3) Different time points after adoptive cell transfer should be evaluated to confirm reduced migration to the atherosclerotic aorta.

      It would be interesting to evaluate Treg migration to the atherosclerotic aorta at different time points after Treg transfer. However, it seems difficult to accurately evaluate the migration of Tregs at later time points because they would proliferate in the aorta.

      Issue (4) The authors could evaluate whether Ccr4 SNPs correlate with an increased risk of atherosclerosis.

      As the reviewer pointed out, it is important to consider the clinical relevance of our findings. However, there is no evidence supporting that Ccr4 single nucleotide polymorphisms correlate with a higher incidence of atherosclerotic cardiovascular disease.

      Issue (5) The authors could evaluate if the transfer of Apoe<sup>-/-</sup> Tregs rescues early atherosclerosis development in Ccr4<sup>-/-</sup>Apoe<sup>-/-</sup> mice.

      To confirm whether transfer of CCR4-intact Tregs rescues the development of early atherosclerotic lesions in Ccr4<sup>-/-</sup>Apoe<sup>-/-</sup> mice, we injected Ccr4<sup>-/-</sup>Apoe<sup>-/-</sup> mice with saline or Tregs from Apoe<sup>-/-</sup> or Ccr4<sup>-/-</sup>Apoe<sup>-/-</sup> mice and analyzed the aortic root atherosclerotic lesions of recipient Ccr4<sup>-/-</sup>Apoe<sup>-/-</sup> mice. However, we found no significant difference in the aortic sinus plaque area among the 3 groups. We described this result in the results section and included the data in Supplementary Figure 8.

      Reviewer #3 (Recommendations for the authors):

      Analysis of TCD4<sup>+</sup> cell populations in different tissues:

      Issue (1) The description of flow cytometry analysis is incomplete and requires clarification. Please detail the use of controls to ensure correct analysis, including the following: i) cell viability; ii) staining controls to define positive and negative cells; iii) the gating strategy used to identify cell populations in each lymphoid tissue and aorta (please provide them as supplementary figures).

      As we thought that most of the prepared cells would be viable, we did not check their viability. Based on our previous work where various immune cells including Tregs, effector memory T cells, and helper T cell subsets were clearly detected, in this study we performed flow cytometric analysis of these immune cells without preparing negative controls stained with isotype control antibodies. The gating strategy of flow cytometric analysis of various immune cells in peripheral lymphoid tissues was reported in our previous report (J Am Heart Assoc 2024; 13: e031639). We provided the gating strategy of flow cytometric analysis of helper T cells and Tregs in the aorta in Supplementary Figure 9.

      Issue (2) The phenotype/differentiation markers used for analysing T CD4<sup>+</sup> cell subsets differ between lymphoid tissues and aortic lesions; might this influence results? If so, please comment on that.

      As the number of aortic T cells was quite few compared with that in peripheral lymphoid tissues, it seemed difficult to precisely detect aortic T cells including various helper T cell subsets and Tregs by intracellular cytokine staining. Therefore, we decided to analyze these cells by evaluating transcription factors specific for helper T cell subsets. The difference in the markers used for analyzing T cell subsets would not considerably influence the results.

      Issue (3) Considering my observations about the effect of CCR4 deficiency on the T CD4<sup>+</sup> differentiation profile in different tissues, I suggest comparing Th1/Treg and Th17/Treg ratios in all examined tissues. The modulation of the Th17/Th1 balance could shape inflammation.

      The Th1 cell/Treg balance is shifted toward Th1 cell responses in the atherosclerotic aorta of Ccr4<sup>-/-</sup>Apoe<sup>-/-</sup> mice, while this balance would not be altered in the peripheral lymphoid tissues. It remains unclear whether CCR4 deficiency affects the Th17 cell/Treg ratio. We do not think that it is important to investigate the effect of CCR4 deficiency on the balance of Th17 cell/Treg or Th17 cell/Th1 cell because the role of Th17 cell responses in atherosclerosis remains controversial.

      Issue (4) Cell numbers of recovered Treg from para-aortic lymphoid nodes and aortic tissues might not allow Treg functional assays. Analysis by flow cytometry of biomarkers of Treg activation state would be more informative than by quantifying mRNA expression levels. In particular, TGFβ analysis at the mRNA level does not provide much more information about the suppressive activity of Treg, and even at the protein level, the recognition of the active form of this cytokine is required. Analysis of PD1 (for exhausted cell phenotype) and Treg apoptosis along the stages of atherosclerosis could also yield useful information.

      We performed flow cytometric analysis of activation markers CTLA-4 and CD103, cell exhaustion marker PD1, and apoptosis in Tregs in the para-aortic LNs of Apoe<sup>-/-</sup> or Ccr4<sup>-/-</sup>Apoe<sup>-/-</sup> mice, and found no major differences in the expression levels of these molecules or the proportion of apoptotic cells between the 2 groups. We showed these data below.

      Author response image 2.

      Unfortunately, we failed to evaluate the activity of TGF-β in Tregs because an appropriate experimental method for precisely detecting its active form was unavailable.

      Issue (5) Regarding the result´s interpretation, I recommend being precise when concluding to avoid misunderstanding. A shift in the T CD4<sup>+</sup> response in lymphoid tissues might be interpreted as a modulation of the T cell differentiation process, which strongly depends on signals derived from DCs, which were not the focus of this study.

      There are two possible mechanisms for the altered CD4<sup>+</sup> T cell responses in peripheral lymphoid tissues, which include the modulation of their differentiation and proliferation processes. These processes are substantially regulated by DCs whose function could be favorably modulated by CCR4-expressing Tregs as described in the manuscript. Therefore, we think that the interactions between Tregs and DCs are crucial for shifting the CD4<sup>+</sup> T cell responses in peripheral lymphoid tissues, though it remains unclear which process plays a major role in regulating CD4<sup>+</sup> T cell polarization.

      Suppression studies:

      Issue (1) In vitro assays. According to the methodology suppression studies were performed using Treg collected from peripheral lymphoid nodes and spleen, but it is unclear whether these cells were analysed separately or as a pool (this was not clarified in the legend of Figure 5 either). Besides, be precise about which cells were used as antigen-presenting cells in the Treg suppression assay.

      In in vitro Treg suppression assay, we used Tregs purified from peripheral lymph nodes and spleen as a pool. We used splenocytes as antigen-presenting cells in Treg suppression assay. We revised the manuscript accordingly.

      Issue (2) Obtaining CD4<sup>+</sup>CD25<sup>+</sup> and CD4<sup>+</sup>CD25-. The control of the purity and viability of cell preparations from CCR4 deficient and CCR4 sufficient Apoe<sup>-/-</sup> mice should be included as a supplementary material; these purified cells were used in in vitro suppressive assays and in vivo cell transfer experiments, being relevant information to guarantee results. Since this control was performed by flow cytometry, I wonder whether Foxp3 levels were also checked.

      We included the data on the purity and viability of CD4<sup>+</sup>CD25<sup>+</sup> Tregs and CD4<sup>+</sup>CD25<sup>-</sup> T cells from Apoe<sup>-/-</sup> or Ccr4<sup>-/-</sup>Apoe<sup>-/-</sup> mice in Supplementary Figure 10. After the isolation of CD4<sup>+</sup>CD25<sup>+</sup> T cells, we examined Foxp3 expression by flow cytometry and found that most of these cells express Foxp3.

      Issue (3) For in vitro assays, IL-2, IL-10, and TGFβ measurement in culture supernatants could confirm and provide more information about Treg function.

      As both CD4<sup>+</sup>CD25<sup>+</sup> Tregs and CD4<sup>+</sup>CD25<sup>-</sup> T cells would produce various cytokines in in vitro Treg suppression assay, it is difficult to determine which cells mainly produce the above cytokines. Therefore, measurement of these cytokines would not provide more information about Treg function.

      Issue (4) It would be interesting to assess whether CCR4-mediated DC-Treg interaction is equally important to regulate Th1 than Th17 and Th2 activation; this likely requires using different settings to favour each activation profile.

      Based on our findings, we speculate that CCR4 may play an important role in regulating not only Th1 cell responses but also Th2 and Th17 cell responses by maintaining the interactions between Tregs and DCs. However, it may not be meaningful to investigate the effect of CCR4 deficiency on these T cell responses because the roles of Th2 and Th17 cell responses in atherosclerosis remain controversial.

      Issue (5) The authors showed that the presence of Treg decreased CD80 and CD86 surface levels in DCs in vitro, remarking a lower capacity of Treg derived from CCR4-deficient mice (Figure 5B). However, the fact that CD86 on splenic CD11c+MHC-II+ DCs in 8-week-old Ccr4<sup>-/-</sup>Apoe<sup>-/-</sup> mice was significantly higher than in Apoe<sup>-/-</sup> was underestimated (Supplementary Figure 4). This data needs reconsideration as it might indicate an in vivo more permissive activation state of DCs in Ccr4<sup>-/-</sup>Apoe<sup>-/-</sup> mice than in Apoe<sup>-/-</sup> mice, explaining the augmented effector T cell response observed in these mice (Figure 2).

      Our finding of the upregulated CD86 expression on DCs in Ccr4<sup>-/-</sup>Apoe<sup>-/-</sup> mice could be explained by the data on a Treg-DC coculture experiment showing the impaired ability of CCR4-deficient Tregs to downregulate CD80 and CD86 expression on DCs. As the reviewer pointed out, our data may indicate more permissive activation state of DCs and subsequent augmentation of effector T cell responses in Ccr4<sup>-/-</sup>Apoe<sup>-/-</sup> mice, which may be derived from impaired Treg suppressive function.

      Assays for chemokine levels and influence on T cell activation and traffic:

      Issue (1) Considering the findings described by Döring et al. (reference 24 in the paper), monitoring CCL22, CCL17, and CCL3 levels in the aorta and lymph nodes along atherosclerosis development would help in understanding when and how CCL17/CCL20-CCR4 might influence T cell activation and traffic. I wonder whether these chemokines were assayed by qPCR in lymphoid nodes and aorta from CCR4-deficient and sufficient Apoe<sup>-/-</sup> mice. The authors report that CCR8 (capable also of binding CCL17) was unaltered by CCR4 deficiency in splenic and para-aortic lymph nodes Treg from 8 and 18 weeks-old mice, respectively (Supplementary Figure 5 and 6), although a trend towards a high-level was observed for splenic Treg. It would be informative to evaluate CCR8 Treg levels along with atherosclerosis progress.

      As it is considered that the mRNA expression levels of chemokines do not necessarily reflect their protein expression levels, we did not analyze the mRNA expression of Ccl17 or Ccl22 by quantitative reverse transcription PCR. Instead of this, we evaluated the protein expression of CCL17 and CCL22 not only in the aorta but also in the peripheral lymph nodes of 18-week-old wild-type, Apoe<sup>-/-</sup>, and Ccr4<sup>-/-</sup>Apoe<sup>-/-</sup> mice by immunohistochemistry. We found no marked differences in their expression levels in peripheral lymph nodes among these mice and included the data in Supplementary Figure 2.

      As we focused on the role of the CCL17/CCL22–CCR4 axes in atherosclerosis, we did not examine the expression of CCL3 that is not directly related to these axes. The evaluation of CCR8+ Treg proportion is beyond the scope of this study, though we are interested in the change of this population by CCR4 deficiency associated with atherosclerotic lesion development.

      Issue (2) According to IFNγ and IL-17 expressing TCD4<sup>+</sup> subclasses, Th1 and Th17 cell subset levels increase in the spleen (Figure 3B-D) and para-aortic lymphoid nodes (Figure 4E) in CCR4 absence. A comparison of the CCR4 dependence for the migration of Th17 and Th1 cell subsets to the aorta was not performed in this atherosclerosis model; this study could help to understand the mechanisms associated with the aortic inflammation development.

      To evaluate the migration of Th1 or Th17 cells in the aorta, we need to specifically isolate them from the peripheral lymphoid tissues of Apoe<sup>-/-</sup> or Ccr4<sup>-/-</sup>Apoe<sup>-/-</sup> mice and adoptively transfer them into recipient Apoe<sup>-/-</sup> mice. However, it is impossible to isolate alive Th1 or Th17 cells because specific cell surface markers that enable us to separate these cells are unavailable.

      Issue (3) The numbers of Kaede Treg cells detected in the aorta were extremely low in both Apoe<sup>-/-</sup> and Ccr4<sup>-/-</sup>Apoe<sup>-/-</sup> mice (Figure 5I), opening results to question. Besides, the flow cytometry assay used for determining Kaede Treg cells in tissues was not well described. How were cell viability and formation of doublets examined to avoid artefacts? The gating strategy used to ensure a confident analysis of Kaede Tregs, particularly in the aorta, should be included as supplementary material.

      The extremely low number of Kaede-expressing Tregs migrated in the aorta of Apoe<sup>-/-</sup> and Ccr4<sup>-/-</sup>Apoe<sup>-/-</sup> mice may be derived from the small number of the transferred Tregs. As another explanation for this finding, Tregs may rarely migrate in the aorta under hypercholesterolemic conditions. We did not check the viability or doublets of Kaede-expressing Tregs because we thought that such experimental procedures would not considerably affect the results. We provided the gating strategy of flow cytometric analysis of Kaede-expressing Tregs in peripheral lymphoid tissues and aortas in Supplementary Figure 11.

      Other comments:

      Issue (1) As an alternative for statistical data analysis from independent experiments, two-way ANOVA with Tukey's post hoc (for data normally distributed) or the Mack Skillings exact test with Conover´s post hoc multiple comparison test (for a two-way layout in non-parametric conditions) could improve analysis.

      We performed statistical analysis in Figure 5A according to the reviewer’s suggestion.

      Issue (2) For future work, employing recombinant pseudo-receptor proteins capable of neutralizing chemokines (doi: 10.1016/j.jhep.2021.08.029) might help as an alternative to complete knockout mice.

      We thank the reviewer for giving us the information on an interesting approach as an alternative to CCR4-deficient mice.

    1. Author Response

      The following is the authors’ response to the original reviews.

      REVIEWER 1

      The claim that olivooid-type feeding was most likely a prerequisite transitional form to jet-propelled swimming needs much more support or needs to be tailored to olivooids. This suggests that such behavior is absent (or must be convergent) before olivooids, which is at odds with the increasing quantities of pelagic life (whose modes of swimming are admittedly unconstrained) documented from Cambrian and Neoproterozoic deposits. Even among just medusozoans, ancestral state reconstruction suggests that they would have been swimming during the Neoproterozoic (Kayal et al., 2018; BMC Evolutionary Biology) with no knowledge of the mechanics due to absent preservation.

      Thanks for your suggestions. Yes, we agree with you that the ancestral swimming medusae may appear before the early Cambrian, even at the Neoproterozoic deposits. However, discussions on the affinities of Ediacaran cnidarians are severely limited because of the lack of information concerning their soft anatomy. So, it is hard to detect the mechanics due to absent preservation. Olivooids found from the basal Cambrian Kuanchuanpu Formation can be reasonably considered as cnidarians based on their radial symmetry, external features, and especially the internal anatomies (Bengtson and Yue 1997; Dong et al. 2013; 2016; Han et al. 2013; 2016; Liu et al. 2014; Wang et al. 2017; 2020; 2022). The valid simulation experiment here was based on the soft tissue preserved in olivooids.

      While the lack of ambient flow made these simulations computationally easier, these organisms likely did not live in stagnant waters even within the benthic boundary layer. The absence of ambient unidirectional laminar current or oscillating current (such as would be found naturally) biases the results.

      Many thanks for your suggestion concerning the lack of ambient flow in the simulations. We revised the section “Perspectives for future work and improvements” (lines 381-392 in our revised version of manuscript). Conducting the simulations without ambient flow can reduce the computational cost and, of course, making the simulation easier, while adding ambient flow can lead to poorer convergency and more technical issues. Meanwhile, we strongly agreed that these (benthic) organisms did not live in stagnant waters, as discussed in Liu et al. 2022. However, reducing computational complexity is not the main reason that the ambient flow was not incorporated in the simulations. As we discussed in section “Perspectives for future work and improvements”, our work focuses on the theoretical effect caused by the dynamics (based on fossil observation and hypothesis) of polyp on ambient environment (i.e., how fast the organism inhales water from ambient environment) rather than effect caused by ambient flow on organism (e.g., drag forces), which was what previous palaeontological CFD simulations mainly focused based on fossil morphology and hydrodynamics. To this end, we mainly concern the flow velocity above or near peridermal aperture (and vorticity computed in this paper) generated only by polyp’s dynamics itself without the interference of ambient flow (as many CFD simulations for modern jellyfish, i.e., McHenry & Jed 2003; Gemmell et al. 2013; Sahin et al. 2009. All those simulations were conducted under hydrostatic conditions). Adding ambient flow to our simulations “biases” the flow velocity profiles we expect to obtain in this case.

      Nevertheless, we do agree that the ambient unidirectional laminar current or oscillating current plays an important role in feeding and respiration behavior of Quadrapyrgites. Further investigations need to be realized by designing a set of new insightful simulations and is beyond the scope of this work. We conducted CFD simulations incorporated with a randomly generated surface that imitated uneven seabed, where unidirectional laminar current and oscillating current (or vortex) were formed and exerted on Quadrapyrgites located in different places on the surface (Zhang et al. 2022). We assumed that combining the method we used in Zhang et al. 2022 and the velocity profiles collected in this work to conduct new simulations may be a promising way to further investigate the effect of the ambient current on organisms’ active feeding behavior.

      There is no explanation for how this work could be a breakthrough in simulation gregarious feeding as is stated in the manuscript.

      Thanks for your suggestion. We revised the section “Perspectives for future work and improvements” (lines 396-404 in our revised version of manuscript).

      Conducting simulations of gregarious active feeding behavior generally need to model multi (or clustered) organisms, which is beyond the present computational capability. However, exploiting the simulation result and thus building a simplified model can be possible to realize that, as we may apply an inlet or outlet boundary condition to the peridermal aperture of Quadrapyrgites with corresponding exhale or inhale flow velocity profiles collected in this work. By doing this we can obtain a simplified version of an active feeding Quadrapyrgites model without using computational expensive moving mesh feature. Such a model can be used solely or in cluster to investigate gregarious feeding behavior incorporated with ambient current. Those above are explicit explanations for how this work could be a “breakthrough” in simulation gregarious feeding. However, we modified the corresponding description in section “Perspectives for future work and improvements” to make it more appropriate.

      Throughout the manuscript there are portions that are difficult to digest due to grammar, which I suspect is due to being written in a second language. This is particularly problematic when the reader is attempting to understand if the authors are stating an idea is well documented versus throwing out hypotheses/interpretations.

      Thanks. Our manuscript was checked and corrected by a native speaker of English again.

      Line-by-line:

      L023: "Although fossil evidence suggests..."

      L026: "demonstrated" instead of "proven"

      We corrected them accordingly.

      L030: "The hydrostatic simulations show that the..." Maybe I'm confused by the wording, but shouldn't this be the case since it's a set part of the model?

      As is demonstrated in our manuscript, all the simulations were conducted under “hydrostatic” environment. We originally intend to use the description “hydrostatic” here to emphasize the simulation condition we set in our work. However, it can literally lead to misunderstanding that some of the simulations we conducted are “hydrostatic” while the others are not. To this end, deleting the word “hydrostatic” here (line 30) may be appropriate to eliminate confusion.

      L058: "lacking soft tissue" Haootia preservation suggests it is soft tissue (Liu et al., 2014), unless the preceding sentence is not including Haootia, in which case this section is confusingly worded

      Thank you. We deleted the sentence “However, their affinities are not without controversy as the lacking soft tissue.”

      L085: change "proxy"

      Yes, we changed to “Considering their polypoid shape and cubomedusa-type anatomy, the hatched olivooids appear to a type of periderm-bearing polyp-shaped medusa (Wang et al. 2020) (lines 86-88).”

      L092: "assist in feeding" has this been stated before? Citation needed, else this interpretation should primarily be in the discussion

      Yes, you are right. We cited the reference at the end of the mentioned sentence (lines 91-94).

      L095: Remove "It is suggested that"

      Thanks for your suggestions. We corrected it.

      L100: "Probably the..." here to the end belongs in the discussion and not introduction.

      Thanks for your suggestions. We corrected the sentences.

      L108: "an abapical"

      Thanks for your suggestions. We revised it in line 107.

      L112: "for some distance" be specific or remove

      Yes, we deleted “for some distance” in line 111.

      L133: I can't find a corresponding article to Zhang et al., 2022. Is this the correct reference?

      The article Zhang et al. 2022 (entitled “Effect of boundary layer on simulation of microbenthic fossils in coastal and shallow seas”.) was in press at the time when we first submitted this manuscript. We complemented the corresponding term in References with the doi (10.13745/j.esf.sf.2023.5.32), which may help readers to locate this article easier.

      L138: You can't be positive that your simulations "provide a good reproduction of the movement." You have attempted to reconstruct said movement, but the language here is overly firm - as is "pave a new way"

      Thanks for your suggestions. We corrected the corresponding description (lines 138-140) to make it more rigorous.

      L149: "No significant change" implies statistics were computed that are not presented here.

      The statistics were computed by using built-in function of Excel and presented in Table supplement 2 (deposited in figshare, https://doi.org/10.6084/m9.figshare.23282627.v2) rather than in manuscript. To be specific, the error computations are followed by the formula of relative error, which is defined by:

      where u_z denotes the velocity profile collected on each cut point z with the current mesh parameters, u_z^* denotes the velocity profile collected on each cut point z with the next finer mesh parameters, i denotes each time step (from 0.01 to 4.0). In this case, the total average error was computed by averaging the sum of each 〖error〗_i on corresponding time step. The results are red marked in Table supplement 2. We revised the corresponding description in lines 140-146

      L152: "line graphs" >> "profiles"

      Thanks for your suggestions. We corrected it in line 144.

      L159: remove "significant" unless statistics are being reported, in which case those need to be explained in detail.

      Thanks for your suggestions. We removed "significant" and corrected the corresponding sentences in lines 150-153 to make them more rigorous.

      L159: I would recommend including a supplemental somewhere that shows how tall the modeled Quadrapyrgites is and where the cut lines exist above it.

      Many thanks for your suggestions. Corresponding complementation was made in the last paragraph of section “Computational fluid dynamics” (line 455 and line 535). We agree that it is appropriate to elucidate the height of modeled Quadrapyrgites and the position of each cut point. Hence, we add a supplementary figure (entitled Figure supplement 1) to illustrate those above.

      L183: "The maximum vorticity magnitude was set..." I do not follow what this threshold is based on the current phrasing.

      The vorticity magnitude mentioned here is the visualisation range of the color scalebar, which can be set manually set in the software. The positive number represent the vortex rotated counterclockwise, while the negative number represent that rotated clockwise on the cut plane. In this case, the visualisation range is [-0.001,0.001] (i.e., the absolute value of 0.001 is the threshold), as the color scalebar in Figure 7. Decreasing the threshold, for example, setting the visualisation range to [-0.0001,0.0001], can capture smaller vorticity on the cut plane, as the figure below on the left. Otherwise, setting the range to [-0.01,0.01] will focus on bigger vorticity, as the figure below on the right. We found [-0.001,0.001] could be an appropriate parameter to visualize the vortex near periderm based on our trial. To be more rigorous and to avoid confusion, we modified the description in the corresponding place of the manuscript (lines 172-174).

      Author response image 1.

      L201: "3.9-4 s"

      Thanks, we corrected it in line 191.

      L269: "Sahin et al.,..." add to the next paragraph

      Yes, we rearranged the corresponding two paragraphs (lines 258-289).

      L344: "Higher expansion-contraction..." this needs references and/or more justification.

      Thanks. We deleted the sentence.

      L446: two layers of hexahedral elements is a very low number for meshing boundary layer flow

      Many thanks for your question. We agree that an appropriate hexahedral elements mesh for boundary layer is essential to recover boundary flow, especially in cases where turbulence model incorporated with wall function is adopted such as the standard k-epsilon model. In this case, the boundary flow is not the main point since the velocity profile was collected above periderm aperture rather than near no-slip wall region. What else, we do not need drag (related to sheer stress and pressure difference) computations in this case, which requires a more accurate flow velocity reconstruction near no-slip walls as what previous palaeontological CFD simulations have done. Thus, we think two layers of hexahedral elements are enough. What else, hexahedral elements added to periderm aperture domain, as illustrated in figure below, can let the velocity near wall vary smoothly and thus can benefit the convergency of simulations.

      Author response image 2.

      L449: similar to comments regarding lines 146-148, key information is missing here. Figure 3C appears to be COMSOL's default meshing routine. While it is true that the domain is discretized in a non-uniform manner, no information is provided as to what mesh parameters were "tuned" to determine "optimal settings" or what those settings are (or how they are optimal).

      Many thanks for your question. Specific mesh parameters were listed in Table supplement 3 and corresponding descriptions and modifications were made both in lines 475-479 and lines 542-549. In most CFD cases, the mesh parameters need to be tuned to ensure a balance between computational cost and accuracy. If the difference of the result obtained from present mesh and that obtained from the next finer mesh ranges from 5% -10%, the present mesh is expected to be “optimal”. To achieve this, we prescribed several sets of different mesh (mainly concerning maximum and minimum element size) to each subdomain (domain of the inner cavity, domain of the peridermal aperture and domain outside of fossil model) of the whole computational domain in the test model. Subsequently, we refined the mesh step by step as much as possible and adjust the element size of subdomains to find suitable mesh parameters, that is how the mesh parameters were "tuned". We agree that we should explicit what mesh parameters were tuned and what those settings are.

      Figure 7 should have the timesteps included and the scaling of the arrows should be explicit in the caption

      Many thanks for your suggestions. We intended to use the white arrows to represent the velocity orientation rather than true velocity scale in Figure 7 (Instead, the white arrows in Animation supplement 1 represent a normalized velocity profile). To avoid confusion, we revised Figure 7 with timesteps and arrows represent a normalized velocity profile, making it consistent with Animation supplement 1. Corresponding modification is also made in the caption of Figure 7.

      The COMSOL simulation files (raw data) are missing from the supplemental data. These should be posted to Dryad or here.

      We uploaded the files to Dryad (https://datadryad.org/stash/share/QGDSqLh8HOll7ofl6JWVrqM57Rp62ZPjvZU0AQQHwTY), and added the corresponding link to section “Data Availability Statement”.

      REVIEWER 2

      Lines 319-334: The omission in this paragraph of Paraconularia ediacara Leme, Van Iten and Simoes (2022) from the terminal Ediacaran of Brazil is a serious matter, as (1) the medusozoan affinities of this fossil are every bit as well established as those of anabaritids, Sphenothallus, Cambrorhytium and Byronia, and (2) P. ediacara was a large (centimetric) polyp, the presence of which in Precambrian times is thus a problem for the simple evolutionary scenario (very small polyps followed later in evolutionary history by large polyps) outlined in the paragraph. Thus, Paraconularia ediacara must be mentioned in this paper, both in connection with the early evolution of size in cnidarian polyps and in other places where the early evolution of cnidarians is discussed.

      Thanks for your important suggestions. We added some sentences in lines 323-326 as following: “Significantly, the large-bodied, skeletonized conulariids-like Paraconularia found from the terminal Ediacaran Tamengo Formation of Brazil confirmed their ancient predators like the extant medusozoans and suggested the origin of cnidarians even farther into the deep evolutionary scenario (Leme et al. 2022).”

      Line 23. Delete the word, been.

      Line 25. Replace conjecture with conjectural.

      Line 26. Delete the word, the before calyx-like.

      Line 32. Replace consisting with consistent.

      Thanks for your suggestions. We all corrected them.

    1. Author Response

      The following is the authors’ response to the original reviews.

      Thanks for your comments and suggestions concerning our manuscript entitled “miR-252 targeting temperature receptor CcTRPM to mediate the transition from summer-form to winter-form of Cacopsylla chinensis”. These comments are all of great important and extremely helpful for revising and improving our manuscript. We have revised the manuscript carefully according to all your comments. Our point-by-point responses to the comments are listed below.

      Reviewer #1 (Recommendations For The Authors):

      1) If the authors wish to improve their phylogenetic analysis, I strongly suggest using their hemipteran sequences alongside the Drosophila homolog and at least all of the human paralogs. This should be generally sufficient to recapitulate the generally accepted TRPM phylogeny. If the authors contend that this is in fact a separate lineage from other insect TRPMs, a phylogeny that is as taxonomically inclusive as possible, and as methodologically rigorous as possible, would be ideal.

      Thanks for your great suggestion. We have redid the phylogenetic analysis in Figure S1B using CcTRPM sequence with homologs from other 16 species, including 8 human paralogs, 1 Mus musculus homolog, 1 Drosophila homolog, and 6 insect homologs. The relative description was added in Line 489-491 and Line 1044-1049 of our revised manuscript.

      2) If the authors wish to conclude that this is a cold-sensitive ion channel, I strongly suggest repeating at least the Ca2+ imaging with a cold stimulus. In the absence of this experiment, I think that the conclusions need to be significantly softened/hedged, making it clear that the only evidence of cold sensitivity is indirect (resulting from the knockdown experiments).

      Thanks for your excellent suggestion. We have performed Ca2+ imaging with a cold stimulus of 10°C. As expected, there was a clear increase of Ca2+ concentration was observed when treated with cold stimulus of 10°C, which was similar with menthol treatment. So, we could get the solid conclusion that CcTRPM is a direct cold-sensitive ion channel in C. chinensis. We also have added the Ca2+ imaging result with a cold stimulus of 10°C in Figure 2D and moved the results of Ca2+ imaging with menthol treatment to Figure S2I. The related results and methods were added in Line 193-200, Line 919-923, and Line 1065-1069 of our revised manuscript.

      3) Lines 173 and 181: The method used to identify the putative transmembrane domains was not described (although the 3D model does have the correct TRP structure, these methodological details would be appreciated).

      Thanks for your great suggestion. We used an online software of SMART (a Simple Modular Architecture Research Tool) to identify the putative transmembrane domains of CcTRPM, and have added these methodological details in Line 485-487 of Materials and Methods of our revised manuscript.

      4) Lines 176-178: The authors state that "phylogenetic analysis revealed that CcTRPM was most closely related to the DcTRPM homologue (Diaphorina citri, XP_017299512.2), which was consistent with the evolutionary relationships predicted from the multiple alignment of amino acid sequences." The meaning of this sentence is unclear to me. I'm not sure what it means to be "consistent with the evolutionary relationships predicted from the multiple alignment of amino acid sequences."

      Thanks for your excellent suggestion. We have revised this sentence in Line176 to 179 of our revised manuscript.

      5) Lines 474-475: The authors state that the NCBI database was used to identify homologous sequences, but there isn't sufficient methodological detail to repeat the search. For example, was this a BLASTP search? Was it taxonomically restricted? What statistical thresholds for homology inference were used? These details would be much appreciated.

      Thanks for your great suggestion. We used BLASTP of NCBI database to identify homologous sequences and preferred the representative species that TRPM sequences have been reported. We have added more description about the methodological detail of phylogenetic analysis in Line 489 to 491 of our revised manuscript.

      6) It would be very interesting, but not critical, to know if menthol and borneol alone have an effect on cuticle thickness.

      Thanks for your excellent suggestion. Actually, we performed the experiments of menthol and borneol alone on cuticle thickness at the beginning. Under 25°C condition, treatment of menthol and borneol alone induced 30-40% transition of 1st instar nymphs from summer-form to winter-form, but only had some slight effect on cuticle thickness, not strong as 10°C of low temperature, because of the opposite effect of 25°C. However, under 10°C condition, we could not know whether the effect on cuticle thickness is from 10°C of low temperature, or direct from menthol and borneol alone.

      7) It would be interesting, but not critical, to confirm the authors' ab initio protein folding by comparing their model to the AlphaFold2-derived model, either by folding it themselves or extracting it from the AlphaFold Protein Structure Database, if it has already been folded by DeepMind.

      Thanks for your great suggestion. We have predicted the tertiary protein structures of CcTRPM with AlphaFold2 software and the result was shown in Author response image 1. Compared with the result in Figure 2A, the conserved ankyrin repeats (ANK) and six transmembrane domains were almost similar.

      Author response image 1.

      The tertiary structures of CcTRPM predicted with AlphaFold2 software.

      8) Figures 1F-G, 3F, 4A-B, 5G-J, S6C, and S7C-D do not plot replicates (although these are plotted in other figures).

      Thanks for your excellent suggestion. Besides Figure 1F-G was stacked grouped graph type and could not add the plot replicates, we have added the plot replicates in Figures 3F, 4A-B, 5G-J, S6C, and S7C-D of our revised manuscript.

      9) Figure 5A-C, and associated text: The significance of these findings is somewhat lost on me, coming from a position of general naivety concerning chitin biosynthesis. My interpretation of Figure 5A was that each of these steps was a necessary component of chitin biosynthesis. It was thus surprising that not all of the steps were required. I think it would be exceptionally helpful if the authors spent more time describing this pathway, alternative pathways to generating the intermediate steps, and ultimately, their hypothesis of why only two steps seem critical.

      Thanks for your great suggestion. The signal pathway of chitin biosynthesis in Figure 5A was modified from the paper of Doucet and Retnakaran, 2012. De novo biosynthesis of chitin has eight enzymatic steps, including 1 Trehalose, 2 enzymes in Glycolysis, 4 enzymes in Hexosamine pathway, and 1 Chitin synthesis. Glycolysis and hexosamine pathway are two complex cellular metabolic processes within organisms. We supposed that there are two reasons for not all of these steps were required: (1) the function of some enzymes may be replaced or supplemented by other enzymes, for examples, function of hexokinase and glucokinase was similar. (2) The reason for no obviously phenotypic defects might be cause by insufficient interference efficiency of RNAi. So, it’s worth to further study the functions of these chitin biosynthesis enzymes by CRISPR-Cas9 in future. We have added more describing about this chitin biosynthesis pathway in Line 379-390 of our revised manuscript.

      Reviewer #2 (Recommendations For The Authors):

      1) Line 19, should be morphological transition.

      Thanks for your excellent suggestion. We have changed “behavioral transition” to “morphological transition” in Line 19 of our revised manuscript.

      2) Line 21, delete the novel.

      Thanks for your excellent suggestion. We have deleted the word of “novel” in Line 21 of our revised manuscript.

      3) Fig. 2B, did authors examine the CcTRPM expression level before 3 d? Given that CcTRPM acts as a cold sensor, it is supposed to respond to temperature change quickly.

      Thanks for your excellent suggestion. We have examined the CcTRPM expression level in 1 d and 2 d after 10°C treatment compared with 25°C treatment. As expected, CcTRPM expression levels were also obviously increased in 1 d and 2 d after 10°C treatment. We have added the relative results in Figure S2F and relative description in Line 184-185, Line 500, and Line 1059-1060 of our revised manuscript.

      4) Fig. 2I, from the figure legend and the text in the panel, it's hard for readers to understand what the authors intend to say. This data is important since knockdown of CcTRPM decreases the winter-form from 90% to 30% at 10℃. Provide more information in the figure legend.

      Thanks for your excellent suggestion. We have added more information in the figure legend of Figure 2I in Line 933-939 of our revised manuscript.

      5) Line 224, ...CcTRPM functions as a molecular switch to modulate the transition from .... The phrase 'molecular switch' is inappropriate because knockdown of CcTRPM partially decreases the form ratio as shown in Fig.2I instead of reversing the effect completely. So, use other words instead of 'molecular switch'.

      Thanks for your excellent suggestion. We have changed “a molecular switch” to “an essential molecular signal” in Line 225 of our revised manuscript.

      6) Fig. 4G, this data is important. It's nice to see that this data is provided.

      Thanks for your excellent suggestion. We have provided the data of Figure 4G in Table S2 of our revised manuscript.

      7) Authors showed that CcTRPM functions as a cold receptor to regulate the transition of C. chinensis from summer-form to winter-form. Does this mean that a heat receptor gene functions oppositely by transiting winter-form into summer-form? Did the authors test the function of a heat TRP in the form transition? At least, discuss this in the discussion part.

      Thanks for your excellent suggestion. TRPV ion channel has been reported to function as a heat receptor in mammals by David Julius (Caterina et al., 1997; Cao et al., 2013). So, we supposed TRPV maybe function as a heat receptor to induce the transition from winter-form to summer-form in C. chinensis. The relative tests are on going. We have added two references in Line 681-686 and some discussion about the heat receptor in Line 341-345 of our revised manuscript.

      8) Line 433, which tissue was used for transmission electron microscopy?

      Thanks for your excellent suggestion. The thorax was used for transmission electron microscopy, and we have added the information in Line 448 and Line 453 of our revised manuscript.

      9) How is the conservation of miR-252? Does the regulatory role of CcTRPM and miR-252 apply to the psylla family in addition to C. chinensis?

      Thanks for your excellent suggestion. Besides C. chinensis, the phenomenon of summer-form and winter-form also existed in other psylla species, like Cyamophila willieti. Because of no genomic information was reported in most psylla species, we could not evaluate the conservation of miR-252 between different psylla species. However, it is worth and interesting to clarify whether the function of TRPM and miR-252 were conserved in the future.

    1. Author response:

      The following is the authors’ response to the original reviews.

      eLife assessment

      This is a valuable study in which the authors provide an expression profile of the human blood fluke, Schistosoma mansoni. A strength of this solid study is in its inclusion of in situ hybridisation to validate the predictions of the transcript analysis.

      We thank the reviewers and the editor for their effort and expertise in reviewing our manuscript. We have made changes based on the reviews and believe this has greatly strengthened our manuscript. We appreciate their insightful comments and suggestions.

      Public Reviews:

      Reviewer #1 (Public Review):

      In this work, the authors provide a valuable transcriptomic resource for the intermediate free-living transmission stage (miracidium larva) of the blood fluke. The single-cell transcriptome inventory is beautifully supplemented with in situ hybridization, providing spatial information and absolute cell numbers for many of the recovered transcriptomic states. The identification of sex-specific transcriptomic states within the populations of stem cells was particularly unexpected. The work comprises a rich resource to complement the biology of this complex system, however falls short in some technical aspects of the bioinformatic analyses of the generated sequence data.

      (1) Four sequencing libraries were generated and then merged for analysis, however, the authors fail to document any parameters that would indicate that the clustering does not suffer from any batch effects.

      We thank the reviewer for this comment which has given us the opportunity to elaborate on this interesting point. Consequently, we have added evidence to show that the data do not suffer from batch effects between samples (e.g. between sorted samples 1 and 4, and unsorted samples 2 and 3). We now show that there are contributions to all clusters from sorted and unsorted samples and highlight the benefits to using both conditions in a cell atlas with unknown cell types.

      Accordingly, we have now added the following paragraph to line 153:

      There were contributions from sorted and unsorted samples in almost all clusters (except ciliary plates). We found that some cell/tissue types had similar recovery from both methods (e.g. Stem A, Muscle 2, and Tegument), others were preferentially recovered by sorting (e.g Neuron 1, Neuron 4, and Stem E), and some were depleted by sorting (e.g. Parenchyma 1, Protonephridia, and Ciliary plates) (Supplementary Figure 1) , Supplementary Table 4). This variation in recovery, therefore, enabled us to maximise the discovery and inclusion of different cell types in the atlas.

      We have now added a Supplementary Figure 1 showing the contribution of sorted and unsorted cells to the Seurat clusters. We have also included a Supplementary Table 4 detailing the cell number contribution for both conditions and the percentages in order to easily compare differential recovery between cell types.

      These are added to the manuscript.

      (2) Additionally, the authors switch between analysis platforms without a clear motivation or explanation of what the fundamental differences between these platforms are. While in theory, any biologically robust observation should be recoverable from any permutation of analysis parameters, it has been recently documented that the two popular analysis platforms (Seurat - R and scanPy python) indeed do things slightly differently and can give different results (https://www.biorxiv.org/content/10.1101/2024.04.04.588111v1). For this reason, I don't think that one can claim that Seurat fails to find clusters resolved by SAM without running a similar pipeline on the cluster alone as was done with SAM/scanPy here. The manuscript itself needs to be checked carefully for misleading statements in this regard.

      We thank the reviewer for this comment and agree that it’s important to increase the clarity on this matter. We have added additional detail to explain that results of subclustering Neuron 1 using Seurat and SAM/ScanPy were broadly similar, but that we presented the results from the SAM/ScanPy analysis due to the strengths of SAM in detecting small differences in gene expression (Tarashanky et al., 2019 PMID: 31524596). We have included here the UMAP showing subclustering of Neuron 1 in Seurat for comparison.

      Author response image 1.

      UMAP showing subclustering of Neuron 1 cluster in Seurat (SCT normalisation, PC = 19, resolution = 0.3).

      We’ve added this additional text to the ‘Neuron abundance and diversity’ section on line 220:

      We explored whether Neuron 1 could be further subdivided into transcriptionally distinct cells by subclustering (Supplementary Figure 2; Supplementary Table 6) using the self-assembling manifold (SAM) algorithm (Tarashansky et al., 2019) with ScanPy (Wolf et al., 2018), given its reported strength in discerning subtle variation in gene expression (Tarashansky et al., 2019), although a similar topology was subsequently found using Seurat.

      (3) Similarly, the manuscript contains many statements regarding clusters being 'connected to', or forming a 'bridge' on the UMAP projection. One must be very careful about these types of statements, as the relative position of cells on a reduced-dimension cell map can be misleading (see Chari and Pachter 2023). To support these types of interpretations, the authors should provide evidence of gene expression transitions that support connectivity as well as stability estimates of such connections under different parameter conditions. Otherwise, these descriptors hold little value and should be dropped and the transcriptomic states simply defined as clusters with no reference to their positions on the UMAP.

      We thank the reviewer for this thoughtful comment. We agree and have rephrased those statements accordingly e.g. line numbers 218, 439, 543, and 557.

      (4) The underlying support for the clusters as transcriptomically unique identities is not well supported by the dot plots provided. The authors used very permissive parameters to generate marker lists, which hampers the identification of highly specific marker genes. This permissive approach can allow for extensive lists of upregulated genes for input into STRING/GO analyses, this is less useful for evaluating the robustness of the cluster states. Running the Seurat::FindAllMarkers with more stringent parameters would give a more selective set of genes to display and thereby increase the confidence in the reader as to the validity of profiles selected as being transcriptomically unique.

      The Reviewer is correct in noting that we used a permissive approach to enable a better understanding of the biology of each cluster, based on analysing enriched functions. However, we disagree about the suitability of the approach for finding markers. First, the permissive approach produced longer candidate lists, but those with the best AUC scores for each cluster are at the top of the list for each cluster. Second, some of the markers with lower expression also revealed interesting biology (e.g. Notum in the muscles). Furthermore, we used filtering on the marker genes lists to increase the minimum marker gene scores for analyses such as the GO analyses (details in the GO section of the methods). It’s important to stress that our approach also utilised validation by FISH for top marker genes, as well as biologically informative genes that were lower down the marker gene list.

      (5) Figure 5B shows a UMAP representation of cell positions with a statement that the clustering disappears. As a visual representation of this phenomenon, the UMAP is a very good tool, however, to make this statement you need to re-cluster your data after the removal of this gene set and demonstrate that the data no longer clusters into A/B and C/D.

      We’ve added Supplementary Figure 13 to show that after removing WSR and ZSR genes and reclustering, the data no longer clusters in A/B and C/D, even at a higher resolution where clusters appear oversplit.

      Also, as a reader, these data beg the question: which genes are removed here? Is there an over-representation of any specific 'types' of genes that could lead to any hypotheses of the function? Perhaps the STRING/GO analyses of this gene set could be informative.

      We have performed GO-enrichment analyses on W-specific genes, Z-specific genes and both together compared to the rest of the genome, but we did not find very informative results (see Supplementary Table 13 that we have now added, line 464). This may be due to the large difference in size. There are approx 900 Z-specific genes (males two copy, females one copy), while approx 30 W-specific genes many of which have homologs in the Z-specific region of the genome. Instead we suggest that tissue-specific regulation of gene dosage compensation is the more likely explanation as reported for other species (Valsecchi et al. 2018).

      (6) How do the proportions of cell types characterized via in situ here compare to the relative proportions of clusters obtained? It does not correspond to the percentages of the clusters captured (although this should be quantified in a similar manner in order to make this comparison direct: 10,686/20,478 = ~50% vs. 7%), how do you interpret this discrepancy? While this is mentioned in the discussion, there is no sufficient postulation as to why you have an overabundance of the stem cells compared to their presence in the tissue. While it is true that you could have a negative selection of some cell types, for example as stated the size of the penetration glands exceeds both that of the 10x capabilities (40uM), and the 30uM filters used in the protocol, this does not really address why over half of the captured cells represent 'stem cells'. A more realistic interpretation would be biological rather than merely technical. For example, while the composition of the muscle cells and the number of muscle transcriptomes captured are quite congruent at ~20%, the organism is composed of more than 50% of neurons, but only 15% of the transcriptomic states are assigned to neuronal. Could it be that a large fraction of the stem cells are actually neural progenitors? Are there other large inconsistencies between the cluster sizes and the fraction of expected cells? Could you look specifically at early transcription factors that are found in the neurons (or other cell types) within the various stem cell populations to help further refine the precursor/cell type relationships?

      Yes, it is really interesting that more than 50% of cells in the animal are neurons whereas more than 50% of cells in scRNAseq data are stem cells. This dataset provides a unique opportunity to compare tissue composition in the whole animal to the corresponding single cell RNAseq dataset.

      The table (in Supplementary Table 17) shows the percentage of cells from each tissue type in the miracidium (identified via in situ hybridisation of tissue-type marker genes) and in the scRNAseq to understand this phenomenon.

      This table shows that the single cell protocol used in this study negatively selected for nerves and tegument, and positively selected for stem and parenchyma. The composition of the muscle and protonephridia cells and the number of muscle and protonephridia transcriptomes captured are quite congruent.

      This technical finding is also biologically consistent. For instance, the tegument cells span the body wall muscles, with the cell bodies below and a syncytial layer above. It is not known how the tegument fragments during the dissociation process, and which parts of the cells get packaged by the 10X GEMs. Because of tegumental structure, the cells are likely prone to damage, and therefore we speculate that is why the tegument cells are under-represented in our 10X data. Unusually shaped fragments may not have been captured in 10X GEMs and of those that were, damaged or distressed tegument cells/fragments may have been excluded post-sequencing, by QC filters including cell calling, mitochondrial percentage and low transcript count (e.g. if there there was a tegumental fragment with 100 transcripts it would have not passed QC). Stem cells are spherical with a large nucleus:cytoplasm ratio, likely making them more robust during dissociation and more likely to be captured in 10X GEMs.

      We don’t think that a large fraction of the stem cells are actually neural progenitors because:

      (1) we used previously reported marker genes of different tissue types to identify the single cell RNAseq clusters, e.g. Ago2-1 for stem cells, which has been used in multiple life stages.

      (2) The stem cell transcriptomes express many previously reported stem cell marker genes.

      (3) We found that the stem cells from the single cell data generally had higher numbers of transcripts than the other cell types which is consistent with the Wang et al. 2013 observation that RNA marker POPO-1 could distinguish germinal (stem) cells from other cell types as they are RNA rich.

      (4) We also found higher numbers of ribosomal related transcripts in our stem cell transcriptomes, which is consistent with Pan’s observation that part of the distinct morphology of stem cells is densely packed ribosomes in the cytoplasm.

      In order to elaborate on this discussion we have generated new visualisations:

      (1) A UMAP of the stem cell marker ago2-1 (Supplementary figure 10), to further illustrate our evidence in classifying the stem cell clusters

      (2) A co-expression plot of the stem cell marker ago2-1 with neural marker complexin to confirm that there is little coexpression (the most coexpression being in Neuron 1 and Stem F). We identified that 15.56% of cells in the Stem F cluster show some expression of complexin (neural marker), suggesting that a small fraction of Stem F may be early/precursor neurons, but the gene expression indicates that the majority of cells in Stem F are more likely to be stem cells than any other tissue type. There is little to no complexin expression in the other stem clusters.

      (3) Expression plots of the 5 neurogenins (TFs involved in neuronal differentiation) we could identify using WormBase ParaSite in these data. Four of the five showed very little expression, and not in specific clusters. The fifth (Smp_072470) showed slightly more expression, though still sparse, mostly across the stem and neural clusters not enough to indicate that any of the stem clusters are neural progenitors.

      Author response image 2.

      Coexpression UMAP showing the expression of stem cell marker Ago2-1 and neural marker complexin.

      Author response image 3.

      UMAPs showing the expression five putative neurogenins of S.mansoni.

      Reviewer #2 (Public Review):

      Summary:

      In this manuscript the authors have generated a single-cell atlas of the miracidium, the first free-living stage of an important human parasite, Schistosoma mansoni. Miracidia develop from eggs produced in the mammalian (human) host and are released into freshwater, where they can infect the parasite's intermediate snail host to continue the life cycle. This study adds to the growing single-cell resources that have already been generated for other life-cycle stages and, thus, provides a useful resource for the field.

      Strengths:

      Beyond generating lists of genes that are differentially expressed in different cell types, the authors validated many of the cluster-defining genes using in situ hybridization chain reaction. In addition to providing the field with markers for many of the cell types in the parasite at this stage, the authors use these markers to count the total number of various cell types in the organism. Because the authors realized that their cell isolation protocols were biasing the cell types they were sequencing, they applied a second method to help them recover additional cell types.

      Schistosomes have ZW sex chromosomes and the authors make the interesting observation that the stem cells at this stage are already expressing sex (i.e. W)-specific genes.

      Weaknesses:

      The sample sizes upon which the in situ hybridization results and cell counts are based are either not stated (in most cases) or are very small (n=3). This lack of clarity about biological replicates and sample sizes makes it difficult for the reader to assess the robustness of the results and the extremely small sample sizes (when provided) are a missed opportunity to explore the variability of the system, or lack thereof.

      We have now added more details about the methods we used for validating cell type marker genes by in situ hybridisation. We have added to the methods that ‘We carried out at least three in situ hybridisation experiments for each marker gene we validated (each experiment was a biological replicate). From each experiment we imaged (by confocal microscopy) at least 10 miracidia (technical replicates) per marker gene experiment.’ on line 1036.

      In the figure legends we have added the number of miracidia that were screened, and documented the percentage of the screened larvae that showed the in situ gene expression pattern that is seen in the images in the figures, and that we described in the text.

      We manually segmented the nuclei of pan tissue marker genes, and we did this for one miracidium in the case of all tissues, except stem cells where we segmented stem cells in five larvae. Manual segmentation of gene expression in a confocal z-stack is very time consuming. We consider that the variability of different cell and tissue types (stereotypy) between miracidia is beyond the scope of this paper and can be investigated in future work.

      Although assigning transcripts to a given cell type is usually straightforward via in situ experiments, the authors fail to consider the potential difficulty of assigning the appropriate nuclei to cells with long cytoplasmic extensions, like neurons. In the absence of multiple markers and a better understanding of the nervous system, it seems likely that the authors have overestimated the number of neurons and misassigned other cell types based on their proximity to neural projections.

      This is a valid point, and we acknowledge the difficulties of assigning a nucleus to a cell using mRNA expression only and in the absence of a cell membrane marker. We tried to address this issue by labelling the cell membranes using an antibody against beta catenin after the HCR in situ protocol. This method has been used successfully on sections on slides (Schulte et al., 2024), but we failed to get usable results in our miracidia whole-mounts. The beta catenin localisation marked the membranes of the gland cells but didn’t do the same for the neurons or other cell types (see image below).

      Author response image 4.

      Image showing a maximum intensity projection of a subvolume of a confocal z-stack of a miracidia wholemount in situ hybridisation (by HCR) for paramyosin counterstained with a beta catenin antibody (1:600 concentration of Sigma C2206). The cell membrane of a lateral gland is clearly labelled, but those of the neurons of the brain and the paramyosin+ muscle cells are not.

      Our observation that 57% of the cells in a miracidium are nerves is high compared to the C.elegans hermaphrodite adult in which 302 out of 959 cells are neurons (Hobert et al., 2016), few studies have equivalent data with which to make comparisons. Despite this, and the limitation described above, we believe that we have not overestimated the number of neural cells. During the process of validating the marker genes and closely examining gene expression in hundreds of miracidia, we noted that the nuclei of different tissue types are distinct and recognisable (see figure below). The nuclei of stem, tegument and parenchymal cells are comparatively large and spherical with obvious nucleoli (i). The four nuclei of the apical gland cell are angular, pentagonal in shape and sit adjoining each other (inside red dashed circle, i-iii), those of the two lateral glands are bilaterally symmetrical and surrounded by flask shaped cytoplasm (arrows, iv). The nuclei of the body wall muscle cells are peripheral and flattened on the outer edge (iii). The notum+ muscle cell nuclei are anterior of the apical gland (manuscript Figure 2E). The only other two tissue types are the nerves and protonephridia, and their nuclei are smaller and more compact/condensed. In situ expression of the protonephridia marker suggests that 6 cells make up the protonephridial system (manuscript Figure 4 B&E). Therefore, by process of elimination, the remaining nuclei should belong to neurons. The complexin expression pattern supports this and we counted 209 nuclei that were surrounded by cpx transcript expression. To help the reader interpret this for themselves we have added confocal z-stacks of miracidia where tissue level markers have been multiplexed (supplementary videos 18-20). We counted all tissue type cells individually and the tissue type cell numbers added up to the overall cell count.

      Author response image 5.

      Image showing the diversity of nucleus morphology between tissue types in the miracidium.

      Biologically, it is not surprising that this larva is dominated by neural cells. It must navigate a complex aquatic environment and identify a suitable mollusc host in less than 12 hours. It is a non-feeding vehicle that must deliver the stem cells to a suitable environment where they can develop into the subsequent life cycle stage. Accordingly, the cell type composition reflects this challenge.

      The conclusion that germline genes are expressed in the miracidia stem cells seems greatly overstated in the absence of any follow-up validation. The expression scales for genes like eled and boule are more than 3 orders of magnitude smaller than those used for any of the robustly expressed genes presented throughout the paper. These scales are undefined, so it isn't entirely clear what they represent, but neither of these genes is detected at levels remotely high (or statistically significant) enough to survive filters for cluster-defining genes.

      Given that germ cells often develop early in embryogenesis and arrest the cell cycle until later in development, and that these transcripts reveal no unspliced forms, it seems plausible that the authors are detecting some maternally supplied transcripts that have yet to be completely degraded.

      We agree that the expression of genes such as eled and boule are low. We made this clear in the figure legends and text, and have now added scale information to the figure legends. We did not explore these genes as cluster-defining genes, partly due to their comparatively low levels of expression, but as genes already reported to be important in germ line specification. We found the expression of these genes to be consistent with our hypothesis that the Kappa stem cells may include germ line segregated cells, but our hypothesis does not rest on these lower-expressed genes.

      It is certainly possible that we have detected some maternally supplied transcripts in the miracidia stem cells. However experiments to distinguish between zygotic and maternal transcripts using metabolic labelling of zygotic transcripts (e.g. Fishman et al. 2023) would be hard in this species due to the hard egg capsule and its ectolethical embryogenesis. Therefore this is out of scope for this work, but this would be a very interesting topic to follow up on and develop tools for.

      We have added these sentences to the Discussion ln 746 ‘Intriguingly, the presence of spliced-only copies of the germline defining genes eled and boule could suggest that they are maternal transcripts that have been restricted to the primordial germ cells during embryogenesis, as is the case in Zebrafish embryos (Fishman et al., 2023). An alternative explanation is that unspliced transcripts exist for these lowly expressed genes but their abundance was below our threshold for detection.’

      Reviewer #1 (Recommendations For The Authors):

      Ln 138: specify the version of Seurat used, and reference the primary papers for this software. Also, from the dot plot shown here, these do not all appear to be supported by unique gene sets. How was the final clustering determined? This information is in the methods section, but a summary here could make it more robust for the readership.

      In addition to the details in the methods section, we have added the version and referenced the version-specific primary paper for Seurat when it is first mentioned. We have also summarised the methods used to select the final clustering when we first present the results to aid in clarity.

      We added to line 140 ‘Using Seurat (version 4.3.0) (Hao et al., 2021), 19 distinct clusters of cells were identified, along with putative marker genes best able to discriminate between the populations (Figure 1C & D and Supplementary Table 2 and 3). We used Seurat’s JackStraw and ElbowPlot, along with molecular cross-validation to select the number of principal components, and Seurat’s clustree to select a resolution where clusters were stable (Hao et al., 2021).’

      Ln 147: isn't seven stem cell clusters a lot? See comment in public review.

      We did not have preconceived expectations of the number of stem cell clusters, and were guided by the data and gene expression. In doing so we also discovered that four of those clusters were likely only two ‘biologically or functionally distinct’ clusters, but these split into four clusters based on the expression of genes on the sex-specific regions of the chromosomes, which was both unexpected and interesting.

      Figure 1D: gene model names are un-informative for the general reader. Can you provide any putative gene identities here to render this plot interpretable? For example in the main text you state that Smp-085540 is paramyosin; please use this annotation in all your visual material (as is used in Figure 2A).

      We have added gene names to the dotplots in all figures with the locus identifier (minus the ‘Smp’ prefix) in brackets after the gene name.

      Ln 191:196 Identification of the two muscle clusters as circular and longitudinal muscles is very well supported. However, it would be interesting to look specifically at the genes that are different here. Did the authors attempt to specifically pull out genes differentially expressed between these two groups, or only examine the output of FindAllMarkers at this point?

      We did indeed look specifically for genes differentially expressed between the muscle clusters, the results of which can be found in Supplementary Table 5 (Line 206). This analysis revealed “Wnt-11-1 (circular) and MyoD (longitudinal) were among the most differentially expressed genes”, which were important findings in our understanding of the muscle cells in the miracidium.

      Ln 207: "connected to stem F" - does this refer specifically to their relative positions on the UMAP in Figure 1C? One must be very careful about these types of statements, as the relative position of cells on a reduced-dimension cell map can be misleading (public review).

      We agree, and have rephrased accordingly.

      Ln 209:211: Here the authors switch from Seurat (R) as an analysis package, to SAM (python) for subset analysis of one large neural cluster. The results indicate that there may be small populations of transcriptomically distinct neural subtypes also within the neural1 cluster, but that the vast majority of these cells do not express unique transcriptomic profiles. Also in the supplementary material for this (SF1) there is a question of whether or not there is any clustering according to batch effects.

      In general, I find the neuronal section a little difficult to follow and it is unclear how many unique profiles are present and which are documented with in situ. I would recommend re-running the analysis on the entire neural subset (n1:5: complexin positive) and generating an inventory of putatively unique neural states with the associated in situ validation altogether in a main figure.

      In response to comments above we have both clarified our reasoning for using SAM analysis, and presented more details on possible batch effects. We have gone through the neural system results in order to make it clearer for the reader to follow.

      Ln 236: here the authors introduce a STRING analysis for the first time. Also, this method requires some introduction for the general audience in terms of its goals and general functionality and output.

      We used STRING analysis on some well defined clusters to provide additional clues about function. At the first mention of STRING (neuron 3 results) we have added the following statement to give more introduction to the reader: “STRING analysis of the top 100 markers of Neuron 3 predicted two protein interaction networks with functional enrichment: ….”

      Ln. 280:281. It is unclear why Steger et al is referenced here. In what way does a description of neural and glandular cell transcriptomic similarity in a Cnidarian inform your data on a member of the playhelmenthes? (which should also be referenced in the introduction: to which phylogenetic lineage does Schistosoma belong).

      We have now added that the Schistosoma belong to the Platyhelminths on the first line of the introduction.

      Ln 295 we have added ‘We expected to find a discrete cluster(s) for the penetration glands, and that it would show similarities to the neural clusters (as glandular cells arise from neuroglandular precursor cells in other animals, such as the sea anemone, Nematostella vectensis, Steger et al., 2022).’

      Ln 339: explain the motivation for generating a further plate-based scRNA of the ciliary plates.

      We wished to include the ciliary plates alongside the gland cells for plate based RNAseq as they are unique to the miracidium stage and wanted to make sure we had captured them in this study.

      Ln 345: Define the tegumental cells for the general reader.

      We have added further description on tegument cells in the introduction and tegument results section, e.g. on line 61, 366).

      Ln 365: "this cluster" is imprecise. Which cluster are we looking at here?' Also: were flame cells already described morphologically at this stage, or is this the first description of the protonephridial system for this stage of the life cycle?

      We have now clarified which cluster we are talking about in the text. The flame cells have been described using TEM before (Pan, 1980).

      Stem Cells: also here you refer to cells as 'bridge' which refers to the configuration of the UMAP. While this is likely a biological representation of a different differentiation state, the nomination of this based solely on the UMAP representation should be avoided.

      We have rephrased this.

      Figure 5B: What is neuron 6? This was Neuron 3 in Figure 1.

      Thank you for spotting these mistakes in the labelling, we have corrected them now.

      Ln 421:438 - Here you represent a UMAP representation of the cell positions, but state that the clustering disappears. See comment in Public Review.

      Modified accordingly, see response in public review.

      Ln 472 "Cells in stem E, F, and G in silico clusters might be stressed/damaged/dying cells or cells in transcriptionally transitional states." Is there any evidence supporting either of these conclusions?

      We found that 15.56% of the cells in Stem F expressed the neural marker complexin, leading us to consider the possibility that a fraction of these cells may be neural precursors. Stem F also had some cells with a mitochondrial % near the maximum threshold we set, suggesting they could be experiencing some stress. Since we could not identify clear markers for these clusters, their function and a more specific identity, beyond ‘stem’, is not yet known.

      That the two stem cell populations contribute to different parts of the next life cycle stage is interesting. The combined analysis suffers from the same issues as the previous analysis in terms of sample distribution; are the 'grey' sporocyst cells also contributing to the stem A/B (kappa) C/D (delta/phi) clusters? This is not possible to tell from the plot as the miracidia may simply be plotted on the top. A different representation of sample contribution to clusters is warranted.

      We have made an alternative visualisation here to demonstrate that the miracidia cells are not plotted on top of the sporocyst stem cells. Unfortunately this visual is hampered as there is not a straightforward way to split the panels. In the figure below, the left pane shows the miracidia cells, and the right pane shows the sporocyst cells. Below that, we have included the original figure for comparison. It can be clearly seen that there are three miracidia tegument cells in the sporocyst tegument cluster, and one sporocyst cell in the miracidia stem cells (Stem E), but the miracidia A/B and C/D stem cells are not plotted on top of any sporocyst cells.

      Author response image 6.

      Methods: Why is the multiplet rate estimate at >50% for the unsorted sample?

      We have added more detail on this: “The estimated doublet rate was calculated based on 10X loading guidelines and adjusted for our sample concentrations”.

      Reviewer #2 (Recommendations For The Authors):

      (1) The manuscript would benefit from a more careful consideration of what was already known based on previous literature, which would help the authors to better put their results in context. For example, previous work suggested that one of the sporocyst stem cell populations (phi) gives rise to tegument and other temporary larval structures; this appears not to be mentioned here. The model in Figure 7 suggests that two of the stem cell populations are gone at day 15 post-infection; the literature shows that those cells can still be detected at this stage (there are just far fewer of them).

      We have added the definition of Kappa, Delta and Phi as per Wang et al (2018) in the stem cell results p13 ln 428.

      We have amended Figure 7 to include further elements from the Wang et al (2018) paper that show that mother sporocyst stem cells classified as delta and phi are still detectable on day 15 post-infection in mother sporocysts.

      We intentionally didn’t put too much emphasis on fitting our data to the model of Wang et al (2018), because a) it’s a different life cycle stage and b) the single cell data the model was based on was from 35 stem cells and gathered using a different method, c) more recent data (Diaz, Attenborough et al. 2024) with 119 stem cells from sporocysts did not recover the same populations of stem cells. We therefore linked our data to previous literature where it was relevant but focused on being led by the data we gathered (>10,000 stem cells).

      (2) To add some detail to the public comment about the lack of clarity about sample sizes and biological replicates, and how this leads to questions about the robustness of the results, Figures 4 B and F show the expression pattern for the same parenchyma marker (Smp_318890) in two different samples. The patterns appear quite distinctive. In B, the cell bodies are so clearly labeled that the signal appears oversaturated. In F the cell bodies are barely apparent. Based on the single-cell clustering, it should be possible to distinguish between Parenchyma clusters 1 and 2 based on the levels of this transcript. Careful quantification of signal intensity from multiple samples across multiple experiments might enable the authors to detect such differences.

      The reason the expression patterns look different between panels 4Bii and 4F is that in 4Bii we have manually segmented the nuclei of the parenchymal cells in order to count them, whereas in the images in 4F there is no segmentation. We have made this more clear in this legend now, and also in the legends of Figures 2,3, and 5. If there was any signal intensity difference between parenchyma 1 and 2 cells based on expression of the marker gene, Smp_318890, it was not obvious. We carried out 6 experiments for parenchyma markers, multiplexing the pan-parenchyma marker, Smp_318890, with markers for parenchyma 2 but we were unable to distinguish between the two populations.

      (3) The authors find that the "somatic" stem cells in miracidia seem to combine attributes of the previously defined delta and phi stem cells from sporocysts. Because the 3 classes of sporocyst stem cells were defined by expression of nanos-2 and fgfrA, using those probes in in-situ experiments could have helped them resolve whether or not the miracidial cells represent precursors that can adopt either fate or if the heterogeneity is already present in miracidia.

      In silico expression of the marker genes for the 3 classes of sporocyst stem cells didn’t support those three classes in the miracidia stem cells (See supplementary table 10). We further subclustered the delta/phi cells to see if we could recover separate delta and phi populations but we were unable to do so. We therefore did not pursue in situ experiments of these genes. We instead prioritised cluster-defining genes in the miracidia stem cell populations rather than cluster defining genes in the sporocyst (defined by Wang et al., 2018), but we still explored these in silico. For example, instead of using klf to define Kappa (Wang et al 2018), we used UPPA to validate the Kappa population as it showed similar expression to klf but higher expression levels and was specific to that population. However, like Wang et al 2018, we did use p53, which is a cluster marker of delta and phi in sporocysts, as it showed clear and high expression in our miracidia delta/phi population. We were guided by our data and our knowledge of the literature. More in depth single cell RNAseq is needed from the mother and daughter sporocyst stages to understand the heterogeneity and fates of these stem populations.

      (4) Scale bars should be included throughout the figures and the scale should be defined either on the figure or in the legend. Similarly, all the scales used for velocity and expression analysis should be defined.

      We have added scale bars to all figures and legends.

      The statements “Gene expression has been log-normalised and scaled using Seurat(v. 4.3.0)”, “Gene expression has been normalised (CPM) and log-transformed using scvelo(v. 0.2.4)”, or “Library size was normalised and gene expression values were log-normalised using SAM (v1.0.1) and Scanpy (v1.8.2)” has been added to all figures as appropriate.

      (5) The table entitled In situ hybridization probes (Supplementary Table 15) contains no probe sequences, so any interested reader wishing to use these probes would have to design their own. To ensure the reproducibility of the results presented here, the authors should provide the probe sequences they used.

      In Supplementary Table 15 we have added the Molecular Instruments Lot number of all the probes used. Anyone wanting to repeat the experiment can order the same probes from the company.

      (6) It is unclear how useful the supplemental figures showing the STRING enrichment analyses will be for readers. Unannotated Smp gene identifiers provide no way to help readers digest the information in these hairballs. It would probably be best to replace the Smp names with useful annotations based on their orthologs; if not, these figures could probably be dropped entirely. (Also, the bottom panel of Supplementary Figure 7 has the word "Lorem" embedded on one of the connecting nodes.)

      “Lorem” has been removed.

      Many of the genes in these analyses do not have short descriptions, therefore we have used Smp gene identifiers in the STRING analysis supplementary figures. These ‘Smp_’ numbers can be used to search WormBase Parasite, where a description can be found and the history of the gene ID traced. This latter function facilitates searching for these genes in the literature and consistency between versions as gene models are updated.

      Minor edits

      (1) Figures 4A-D aren't cited in the text until after 4E-F are. It seems like moving the section on protonephridial cells (line 364) before the section on tegumental cells (line 345) better reflects the order of the figures.

      Thank you for flagging this, we have updated the in-text citations of Figure 4.

      (2) In-text references to Sarfati et al, 2021 should be to Nanes Sarfati, as listed in the references. Poteaux et al 2023 is cited in the text, but not in the reference list.

      Both of these have been fixed.

    1. Author response:

      The following is the authors’ response to the original reviews

      Public Reviews:

      Reviewer #1 (Public review):

      The authors track the motion of multiple consortia of Multicellular Magnetotactic Bacteria moving through an artificial network of pores and report a discovery of a simple strategy for such consortia to move fast through the network: an optimum drift speed is attained for consortia that swim a distance comparable to the pore size in the time it takes to align the with an external magnetic field. The authors rationalize their observations using dimensional analysis and numerical simulations. Finally, they argue that the proposed strategy could generalize to other species by demonstrating the positive correlation between the swimming speed and alignment time based on parameters derived from literature.

      Strengths:

      The underlying dimensional analysis and model convincingly rationalize the experimental observation of an optimal drift velocity: the optimum balances the competition between the trapping in pores at large magnetic fields and random pore exploration for weak magnetic fields.

      Weaknesses:

      The convex pore geometry studied here creates convex traps for cells, which I expect enhances their trapping. The more natural concave geometries, resulting from random packing of spheres, would create no such traps. In this case, whether a non-monotonic dependence of the drift velocity on the Scattering number would persist is unclear.

      We agree that convex walls increase the time that consortia remain trapped in pores at high magnetic fields. Since the non-monotonic behavior of the drift velocity with the Scattering number arises largely due to these long trapping times, we agree that experiments using concave pores are likely to show a peak drift velocity that is diminished or erased.

      However, we disagree that a random packing of spheres or similar particles provides an appropriate model for natural sediment, which is not composed exclusively of hard particles in a pure fluid. Pore geometry is also influenced by clogging. Biofilms growing within a network of convex pillars in two-dimensional microfluidic devices have been observed to connect neighboring pillars, thereby forming convex pores. Similar pore structures appear in simulations of biofilm growth between spherical particles in three dimensions. Moreover, the salt marsh sediment in which MMB live is more complex than simple sand grains, as cohesive organic particles are abundant. Experiments in microfluidic channels show that cohesive particles clog narrow passageways and form pores similar to those analyzed here. Thus, we expect convex pores to be present and even common in natural sediment where clogging plays a role.

      The concentration of convex pores in the experiments presented here is almost certainly much higher than in nature. Nonetheless, since magnetotactic bacteria continuously swim through the pore space, they are likely to regularly encounter such convexities. Efficient navigation of the pore space thus requires that magnetotactic bacteria be able to escape these traps. In the original version of this manuscript, this reasoning was reduced to only one or two sentences. That was a mistake, and we thank the reviewer for prompting us to expand on this point. As the reviewer notes, this reasoning is central to the analysis and should have been featured more prominently. In the final version, we will devote considerable space to this hypothesis and provide references to support the claims made above.

      The reviewer suggests that the generality of this work depends on our finding a ”positive correlation between the swimming speed and alignment [rate] based on parameters derived from literature.” We wish to emphasize that, in addition to predicting this correlation, our theory also predicts the function that describes it. The black line in Figure 3 is not fitted to the parameters found in the literature review; it is a pure prediction.

      Reviewer #2 (Public review):

      The authors have made microfluidic arrays of pores and obstacles with a complex shape and studied the swimming of multicellular magnetotactic bacteria through this system. They provide a comprehensive discussion of the relevant parameters of this system and identify one dimensionless parameter, which they call the scattering number and which depends on the swimming speed and magnetic moment of the bacteria as well as the magnetic field and the size of the pores, as the most relevant. They measure the effective speed through the array of pores and obstacles as a function of that parameter, both in their microfluidic experiments and in simulations, and find an optimal scattering number, which they estimate to reflect the parameters of the studied multicellular bacteria in their natural environment. They finally use this knowledge to compare different species to test the generality of this idea.

      Strengths:

      This is a beautiful experimental approach and the observation of an optimal scattering number (likely reflecting an optimal magnetic moment) is very convincing. The results here improve on similar previous work in two respects: On the one hand, the tracking of bacteria does not have the limitations of previous work, and on the other hand, the effective motility is quantified. Both features are enabled by choices of the experimental system: the use the multicellular bacteria which are larger than the usual single-celled magnetotactic bacteria and the design of the obstacle array which allows the quantification of transition rates due to the regular organization as well as the controlled release of bacteria into this array through a clever mechanism.

      Weaknesses:

      Some of the reported results are not as new as the authors suggest, specifically trapping by obstacles and the detrimental effect of a strong magnetic field have been reported before as has the hypothesis that the magnetic moment may be optimized for swimming in a sediment environment where there is a competition of directed swimming and trapping. Other than that, some of the key experimental choices on which the strength of the approach is based also come at a price and impose some limitations, namely the use of a non-culturable organism and the regular, somewhat unrealistic artificial obstacle array.

      In the “Recommendations for the Authors,” this reviewer drew our attention to a manuscript that absolutely should have been prominently cited. As the reviewer notes, our manuscript meaningfully expands upon this work. We are pleased to learn that the phenomena discussed here are more general than we initially understood. It was an oversight not to have found this paper earlier. The final version will better contextualize our work and give due credit to the authors. We sincerely appreciate the reviewer for bringing this work to our attention.

      We disagree that the use of non-culturable organisms and our unrealistic array should be considered serious weaknesses. While any methodological choice comes with trade-offs, we believe these choices best advance our aims. First, the goal of our research, both within and beyond this manuscript, is to understand the phenotypes of magnetotactic bacteria in nature. While using pure cultures enables many useful techniques, phenotypic traits may drift as strains undergo domestication. We therefore prioritize studying environmental enrichments.

      Clearly, an array of obstacles does not fully represent natural heterogeneity. However, using regular pore shapes allows us to average over enough consortium-wall collisions to enable a parameter-free comparison between theory and experiment. Conducting an analysis like this with randomly arranged obstacles would require averaging over an ensemble of random environments, which is practically challenging given the experimental constraints. Since we find good agreement between theory and experiment in simple geometries, we are now in a position to justify extending our theory to more realistic geometries. Additionally, we note that a microfluidic device composed of a random arrangement of obstacles would also be a poor representation of environmental heterogeneity, as pore shape and network topology differ between two and three dimensions.

      Recommendations for the Authors: 

      Reviewer #1 (Recommendations for the authors):

      My main suggestion is for the authors to describe the limitations of their approach in the case of concave pores.

      As we noted in our public comments, this was a very useful comment to hear from you and one that has been repeated as we have spoken about these results to colleagues. Convexities here represent an experimentally simple way to force bacteria to back track through the maze, as they must through natural sediment. We have greatly expanded this discussion to clarify this reasoning (lines 84–105). We provide references to three types of physical processes that may lead to such traps. First, as in figure 1 of Kurz et al, biofilm (white) can fill the spaces between convex pillars to create covexities. Additionally, clogging by cohesive particles can make narrow passageways between convex particles impassible. An example of clogging is shown in figure 6 of Dressaire & Sauret 2017. Finally, air bubbles trapped in the sediment can create pore-scale dead ends that require bacteria to backtrack. The full references are provided in the main text.

      Small points:

      (1) How many trajectories were used to produce Figures 2 b and c?

      We have modified the caption to note that these data represent the measured transition rates of a total 938 consortia at various Scattering numbers. Each consortium may pass between pores many times.

      (2) Can the authors describe in more detail how Equation (3) is derived? Why doesn’t it depend on the gap size between the pores?

      We have provided a derivation of this equation in Appendix 2 of the new version. This derivation shows that the drift velocity U<sub>drift</sub> is proportional to the pore diameter and difference between the transition rates.

      The proportionality constant α depends on how the pores are connected together in space. In the original version, we wanted to highlight the role of the asymmetry of the transition rates, so we imagined a one dimensional network of pores without gaps. In this case, α \= 1. This reasoning was poorly explained in the previous version and we thank the reviewer for pointing this deficiency out. In the new version, we include the gap size and use the layout of pores in a square lattice with gaps, which is shown in figure 1. The proportionality constant for a square lattice in the absence of gaps√ would be 1/2. The limitations of photolithography require some gap that increase the proportionality constant to α \= 0.8344.

      We have updated the text, equation (3), and the figures to account for the finite gap sizes.

      (3) I found the second part of the abstract, related to the comparison between diverse bacteria, to be slightly misleading. Upon first reading, my expectation was that the authors carried out experiments with different species.

      We have modified the abstract to make clear that we rely on values taken from a literature review.

      (4) More information is needed on how many trajectories were used to produce the probability densities in Figures 1b-d. How were the densities computed?

      The probability distributions give the probability that a pixel in a pore is covered by a consortium. They reflect between 1.2 and 7 million measurements (depending on the panel) of the instantaneous positions of consortia. We have added a section (Lines 453–469) to Materials and Methods that describes exactly how these distributions were calculated.

      Reviewer #2 (Recommendations for the authors):

      (1) As mentioned under Weaknesses in the Public review, some results are less new than claimed here. The existence of an optimal magnetic moment has been shown by Codutti et al eLife eLife13:RP98001 in very similar experiments, where it was also proposed that this may be an evolutionary adaptation to the sediment habitat. The paper here provides additional evidence for this, and with better tracking and quantification, but previous work should be discussed. Likewise, the work by Dekharghani et al. that is mentioned rather suddenly in the Results section appears to be a crucial previous state of the art and could already be mentioned in the introduction.

      We thank the reviewer for bringing this paper, which came out as we were writing this manuscript, to our attention. The hypothesis that there is an optimal phenotype that balances magnetotaxis with obstacle avoidance—and that natural selection could guide organisms to this optimum—goes back to at least 2022. It seems that Codutti et al independently came up with this same hypothesis and provided the first test.

      We have substantively rewritten the introduction (Lines 46–58) to better contextualize our work and give due attention to Dekharghani et al.

      (2) The first paragraph of Results also contains background information and could be moved into the introduction.

      As part of the rewrite to better contextualize our work, we moved the first two paragraphs of results to the introduction.

      (3) I found Figure 1 a bit confusing and it took me some time to understand the geometry. I think the black obstacles are very dominant to the viewer’s eye and draw attention away from the essentially circular shape of the pores. Likewise, I am not sure that cutting the neighboring pores off in a circular fashion in Figures 1b-d was the best choice. The authors should think about whether the presentation can be improved. Likewise, when describing the direction of the field in the text, I would suggest adding that it is along the horizontal direction in Figure 1.

      We have modified the figure and the text as the reviewer suggests.

      (4) That collisions with a pore wall are an important mechanism of changing direction is clear and it is nice to see the paper demonstrate that this mechanism is dominant over rotational diffusion. However, this may not be universal, as (i) rotational diffusion is more important for smaller cells and (ii) interaction with walls can result in all kinds of different behaviors than complete randomization (e.g. swimming along the walls as shown in microfluidic chambers, Ostapenko et al. Phys Rev Lett 2018, Codutti et al. eLife 2022, or reversals, Kuhn et al PNAS 2017). Here, it appears that complete randomization of the direction is an assumption, but this could be tested/quantified by analyzing the trajectories.

      This is an excellent point. We have modified the text to describe qualitatively how these tendencies would shift the Critical Scattering number. We also note in the text that there is evidence of these differences in Fig 3. The Desulfobacterota are shifted upwards in Fig 3 relative to the α-proteobacteria. This shift indicates that Desulfobacterota tend to live at slightly greater scattering numbers of 0.9±0.3 than the α-proteobacteria, which live at scattering number 0.37 ± 0.03. It is likely that this difference reflects taxonomic differences in rotational diffusion and cell-wall interactions.

      It is true that total randomization of the direction is indeed an assumption, and it is stated as such in line 189. We performed all of the numerics to find the solid curves in Fig 2 before we got any experimental data and so, at the time, total randomization seemed like a fair choice. Looking at Fig 2b, it is clear that these numerics systematically overestimate k<sub>−</sub>. We believe that this error is do to the assumption of total randomization.

      As this effect is small and does not change any of the conclusions of the paper and Codutti et al were able to publish their paper in the time that we were writing ours, we feel some urgency to move forward.

      (5) From the manuscript it is not fully clear to what extent experiments and simulations are or can be quantitatively compared. For example: is the curve (“fit”) in Figure 2c based on the simulations? Is there an explicit expression or is this just a spline or something like that? Why does Figure 5 (simulation) show the velocity as a function of Sc<sup>−1</sup>and Figure 2 (experiment) as a function of Sc? It looks to me as if a quantitative comparison could be achieved.

      The original version of Figure 2 shows a quantitative comparison between theory and experiment with no fit parameters. The data points are the result of experiments in which consortia are tracked as they as they move between connected pores. The solid line is a found by interpolating a smooth curve through the data from simulations. As we make clear in the new version (Lines 537–551), this blue curve is the most probable smooth curve that explains the simulations.

      We have added the simulations to figure 2 so that a single panel includes the data, the simulations, and the smooth curve. To further make clear that this comparison is quantitative and parameter free, we have added a panel to Figure 2. This panel directly compares the prediction to observation and is independent of the blue curve.

      As was noted (deep within the methods section) in the original version, our numerics can exactly simulate Sc = ∞. Consequently, it was reasonable to simulate parameters that are uniformly spaced in Sc<sup>−1</sup>.

      (6) While I like the idea behind Figure 3, the data shown here is not as convincing as suggested. If one looks at the data without the black line, I think one gets a weaker dependence. The correlation between U<sub>0</sub> and γ<sub>geo</sub> is likely not as strong as it seems. Calculating a correlation coefficient might be helpful here. In any case, the assumptions going into this figure should be discussed more explicitly and the results should in my opinion be phrased more cautiously (I tend to believe what the authors claim, but I don’t think the evidence for this point is very strong).

      We appreciate the reviewer’s skepticism. However, we believe that the data are stronger than one might understand from the previous text. We have rewritten the text (Lines 219–291) and included new analysis, figures, and explanation to make three points clear.

      (a) It is surprising that speed, magnetic moment, and mobility all vary tremendously(between one and three orders of magnitude) across taxa and environment, however, their dimensionless combination Sc is narrowly distributed. We have added a panel to Fig. 3 to show the measured Scattering numbers.

      It is notable that there are no adjusted parameters in the calculation of the Scattering numbers: it is a simple dimensionless combination of phenotypic and environmental parameters. All but one of these parameters (the pore size) is measured either by us or by other authors. The pore radius is likely narrowly distributed. We measure it at our field site and, when it is not reported, we use a value typical of the geological and fluvial environment. Just as the size of sand grains does not vary greatly between the beaches of Australia, Africa, and California, it is a good assumption that the pore spaces that host these magnetotactic bacteria do not vary tremendously in size.

      (b) In the new version we compare the Scattering number statistics to a parameterfree null model of phenotypic diversity. We argue in the text that it is appropriate to bootstrap over the phenotypic diversity of species. This null model provides the correct method to calculate p-values as the variability in the Scattering numbers is neither identically distributed nor normally distributed.

      We use this null model to show that—given the measured phenotypic diversity across species—the probability that fifteen random species would fall within the measured range of Scattering numbers that is consistent with optimal navigation is ∼ 10<sup>−6</sup>. This result is strong evidence that the phenotypic variables exhibit the correlations that are predicted by our analysis.

      (c) The correlation between U<sub>0</sub>/r and γ<sub>geo</sub> is reasonably strong. I think that our choice of axes in Fig 3, which were chosen to fit the legend, make the data look flatter than then they actually are. Here are the same data plotted without the line with tighter axes:

      Author response image 1.

      With the exception of the very first point and the very last point, the data appear to our eyes to be pretty correlated. This impression is born out by a calculation of the correlation coefficient which gives 0.77. The p-value is 4 × 10<sup>−4</sup>. We have included these values in the main text to clarify that this correlation is both statistically significant and of primary importance.

      (7) There is a comment at the end of the discussion that the evolutionary hypothesis could be tested by transferring the magnetotaxis genes to nonmagnetotactic organisms. This would indeed be highly desirable, but this is very difficult as indicated by the successful efforts in that direction (which often are only moderately magnetic/magnetotactic), see Kolinko et al Nature Nanotech 2014, Dziuba et al Nature Nanotech 2024.

      Thank you for highlighting these references, which we have included. We agree that these experiments will be challenging. Our results make a prediction about the evolution of these strains, so it seems worth mentioning this fact. We feel that this manuscript is not the correct space for a detailed description of challenges that we will encounter should we pursue this direction of study.

      (8) A section on how the bacterial samples were obtained could be added in Methods.

      We have done so.

      Additional Changes

      (1) In the original version, we feared that the consortia in the microfluidic device arepoorly representative of the natural population. Consequently, we used the values from previous experiments, which we performed using consortia taken from the same pond. Since submitting this manuscript we have undertaken new experiments that allowed us to measure the Scattering number of individual consortia. It turns out the effect is smaller than we worried. We have included these measurements in the new version. We find that even as the most common phenotypes vary over the course of time, the Scattering number remains constant. This result is additional evidence that there is strong selective pressure to optimally navigate.

      As a result of these additions, we have added an author, Julia Hernandez, who contributed to these experiments and analysis.

      (2) We have expanded the table of phenotypic variable in Appendix 1 to make it easier forother researchers to reproduce our analysis.

    1. Author response:

      The following is the authors’ response to the original reviews

      Public Reviews:

      Reviewer #1 (Public review):

      Hearing and balance rely on specialized ribbon synapses that transmit sensory stimuli between hair cells and afferent neurons. Synaptic adhesion molecules that form and regulate transsynaptic interactions between inner hair cells (IHCs) and spiral ganglion neurons (SGNs) are crucial for maintaining auditory synaptic integrity and, consequently, for auditory signaling. Synaptic adhesion molecules such as neurexin-3 and neuroligin-1 and -3 have recently been shown to play vital roles in establishing and maintaining these synaptic connections ( doi: 10.1242/dev.202723 and DOI: 10.1016/j.isci.2022.104803). However, the full set of molecules required for synapse assembly remains unclear.

      Karagulan et al. highlight the critical role of the synaptic adhesion molecule RTN4RL2 in the development and function of auditory afferent synapses between IHCs and SGNs, particularly regarding how RTN4RL2 may influence synaptic integrity and receptor localization. Their study shows that deletion of RTN4RL2 in mice leads to enlarged presynaptic ribbons and smaller postsynaptic densities (PSDs) in SGNs, indicating that RTN4RL2 is vital for synaptic structure. Additionally, the presence of "orphan" PSDs-those not directly associated with IHCs-in RTN4RL2 knockout mice suggests a developmental defect in which some SGN neurites fail to form appropriate synaptic contacts, highlighting potential issues in synaptic pruning or guidance. The study also observed a depolarized shift in the activation of CaV1.3 calcium channels in IHCs, indicating altered presynaptic functionality that may lead to impaired neurotransmitter release. Furthermore, postsynaptic SGNs exhibited a deficiency in GluA2/3 AMPA receptor subunits, despite normal Gria2 mRNA levels, pointing to a disruption in receptor localization that could compromise synaptic transmission. Auditory brainstem responses showed increased sound thresholds in RTN4RL2 knockout mice, indicating impaired hearing related to these synaptic dysfunctions.

      The findings reported here significantly enhance our understanding of synaptic organization in the auditory system, particularly concerning the molecular mechanisms underlying IHC-SGN connectivity. The implications are far-reaching, as they not only inform auditory neuroscience but also provide insights into potential therapeutic targets for hearing loss related to synaptic dysfunction.

      We would like to thank the reviewer for appreciating the work and the advice that helped us to further improve the manuscript. We have carefully addressed all concerns, please see our point-per-point response below and the revised manuscript.

      Reviewer #2 (Public review):

      Summary:

      Kargulyan et al. investigate the function of the transsynaptic adhesion molecule RTN4RL2 in the formation and function of ribbon synapses between type I spiral ganglion neurons (SGNs) and inner hair cells. For this purpose, they study constitutive RTN4RL2 knock-out mice. Using immunohistochemistry, they reveal defects in the recruitment of protein to ribbon synapses in the knockouts. Serial block phase EM reveals defects in SGN projections in mutants. Electrophysiological recordings suggest a small but statistically significant depolarized shift in the activation of Cav1.3 Ca<sup>2+</sup> channels. Auditory thresholds are also elevated in the mutant mice. The authors conclude that RTN4RL2 contributes to the formation and function of auditory afferent synapses to regulate auditory function.

      We would like to thank the reviewer for appreciating the work and the advice that helped us to further improve the manuscript. We have carefully addressed all concerns, please see our point-per-point response below and the revised manuscript.

      Strengths:

      The authors have excellent tools to analyze ribbon synapses.

      Weaknesses:

      However, there are several concerns that substantially reduce my enthusiasm for the study.

      (1) The analysis of the expression pattern of RTN4RL2 in Figure 1 is incomplete. The authors should show a developmental time course of expression up into maturity to correlate gene expression with major developmental milestones such as axon outgrowth, innervation, and refinement. This would allow the development of models supporting roles in axon outgrowth versus innervation or both.

      We agree that it would be valuable to show the developmental time course of RTN4RL2 expression. In response to the reviewer’s comment, we are providing RNAscope data from developmental ages E11.5, E12.5 and E16 in Figure 1. RTN4RL2 shows expression at E11.5/E12.5 both in the spiral ganglion and hair cell region, with first onset in the hair cells. We conclude that RTN4RL2 is expressed highest during fiber growth at embryonic stages and is downregulated during postnatal development maintaining low levels of expression during adulthood.

      (2) It would be important to improve the RNAscope data. Controls should be provided for Figure 1B to show that no signal is observed in hair cells from knockouts. The authors apparently already have the sections because they analyzed gene expression in SGNs of the knock-outs (Figure 1C).

      In Figure 1C gene expression in SGNs was assessed at p40, while the expression in hair cells is provided for p1 animals. Unfortunately, we do not have KO controls for p1 animals. However, as indicated in our manuscript, previously published RNA expression datasets do find RTN4RL2 expression in hair cells. Therefore, we think it is unlikely that our results are unspecific.

      (3) It is unclear from the immunolocalization data in Figure 1D if all type I SGNs express RTN4RL2. Quantification would be important to properly document the presence of RTN4RL2 in all or a subset of type I SGNs. If only a subset of SGNs express RTN4RL2, it could significantly affect the interpretation of the data. For example, SGNs selectively projecting to the pillar or modiolar side of hair cells could be affected. These synapses significantly differ in their properties.

      According to already published single cell RNAseq dataset from Shrestha et al., 2018, RTN4RL2 expression does not seem to show a clear type I SGN subtype specificity (Author response image 1). In response to the reviewer’s comment, we have further performed anti-Parvalbumin (PV) and anti-calretinin (CR) immunostainings in mid-modiolar cryosections of RTN4RL2<sup>+/+</sup> and RTN4RL2<sup>-/-</sup> cochleae. Parvalbumin was chosen to label all SGNs and CALB2 was chosen primarily as a type Ia SGN marker (Sun et al., 2018). We present the data from all analyzed samples below (figure 2 of this rebuttal letter). Cell segmentation masks of PV positive cells were obtained using Cellpose 2.0 and the average CR intensity was calculated in those masks. While the distributions of CR intensity and the ratio of CR and PV intensities are slightly shifted in RTN4RL2<sup>-/-</sup> cochleae, we take the data to suggest that the composition of the spiral ganglion by molecular type I SGN subtypes is largely unchanged in RTN4RL2<sup>-/-</sup> mice.

      Author response image 1.

      Author response image 1 cites single cell RNAseq data of Brikha R Shrestha, Chester Chia, Lorna Wu, Sharon G Kujawa, M Charles Liberman, Lisa V Goodrich. Sensory neuron diversity in the inner ear is shaped by activity. Cell. 2018 Aug 23; 174(5):1229-1246.e17. doi: 10.1016/j.cell/2018.07.007

      Author response image 2.

      Calretinin intensity distribution in spiral ganglion of RTN4RL2<sup>+/+</sup> and RTN4RL2<sup>-/-</sup> mice. (A) Mid-modiolar cochlear cryosections from RTN4RL2<sup>+/+</sup> (top) and RTN4RL2<sup>-/-</sup> (bottom) mice immunolabeled against Parvalbumin (PV) and Calretinin (CR). Scale bar = 20 mm. (B) Distribution of CR intensity in PV positive cells (N = 3 for each genotype). (C) Distribution of the ratio of CR and PV intensities (N = 3 for each genotype).

      (4) It is important to show proper controls for the RTN4RL2 immunolocalization data to show that no staining is observed in knockouts.

      Unfortunately, our recent attempts to perform RTN4RL2 immunostainings on cryosections failed and therefore, we decided to remove the RTNr4RL2 immunostainings from Figure 1. We have adjusted the results section accordingly.

      (5) The authors state in the discussion that no staining for RTN4RL2 was observed at synaptic sites. This is surprising. Did the authors stain multiple ages? Was there perhaps transient expression during development? Or in axons indicative of a role in outgrowth, not synapse formation?

      We thank the reviewer for the comment. We have now tried RTN4RL2 immunostainings on cryosections at several developmental stages, but unfortunately this time did not succeed to obtain reproducible and reliable results. Therefore, we decided to also remove the previous immunostainings from Figure 1. We have adjusted the results section as well as removed our statement of not detecting RTN4RL2 near the synaptic regions from the discussion.

      (6) In Figure 2 it seems that images in mutants are brighter compared to wildtypes. Are exposure times equivalent? Is this a consistent result?

      Yes, the samples were prepared in parallel, imaged and analyzed in the same manner.

      No, we did not observe consistent differences in brightness and also did not find it in the exemplary images of figure 2.

      (7) The number of synaptic ribbons for wildtype in Figure 2 is at 10/IHCs, and in Figure 2 Supplementary Figure 2 at 20/IHCs (20 is more like what is normally reported in the literature). The value for mutant similarly drastically varies between the two figures. This is a significant concern, especially because most differences that are reported in synaptic parameters between wild-type and mutants are far below a 2-fold difference.

      The key message is that there is no difference in the numbers of ribbons and synapses between the genotypes for the cochlear apex (~10 ribbons/IHCs, Figure 2 and Figure 2-figure supplement 2) and the mid- and base of the cochlea (more ribbons/IHCs, Figure 2-figure supplement 2). Figure 2-figure supplement 3 (now Figure 3) shows that there is a massive reduction of postsynaptic GluA2, while both Figure 2 and Figure 2-figure supplement 2 indicate that the number synapses is normal. These are two different data sets and while we closely collaborated and also shared the Moser lab protocols and analysis routines, we agree that there is a difference in the absolute synapse count, which most likely was an observer difference and different choice of tonotopic positions of analysis. In Figure 2 only the apical hair cells have been analyzed. The Moser lab, since establishing the immunofluorescence-based quantification of synapse number (Khimich et al., 2005) reported tonotopic differences in synapse counts (focus of Meyer et al., 2009 and reported by others: e.g. Kujawa and Liberman, 2009): apical and basal IHCs lower synapse numbers than mid-cochlear IHCs.

      (8) The authors report differences in ribbon volume between wild-type and mutant. Was there a difference between the modiolar/pillar region of hair cells? It is known that synaptic size varies across the modiolar-pillar axis. Maybe smaller synapses are preferentially lost?

      We thank the reviewer for the comment. Unfortunately, our already acquired datasets from 3-week-old mice did not allow us to check whether the previously described modiolar-pillar gradient of the ribbon size was collapsed in RTN4RL2<sup>-/-</sup> mice due to the not so well-preserved morphology of the inner hair cells in our preparations. However, since the number of the ribbons is not changed in the RTN4RL2 KO mice, we do not think that the increase in the ribbon size is due to the loss of small ribbons. In response to the reviewers comment we have analyzed the modiolar-pillar gradient of the ribbon size in IHCs of middle turn of the cochlea form a newly acquired dataset of 14-week-old mice. We took the fluorescence intensity of Ctbp2 positive puncta as a proxy for the ribbon size. In these older mice we found a preserved modiolar-pillar gradient of the ribbon size (larger ribbons at the modiolar side). We summarized the results in the below Author response image 3.

      Author response image 3.

      The modiolar-pillar gradient of ribbon size is preserved in RTN4RL2<sup>-/-</sup> IHCs. (A) Maximum intensity projections of approximately 2 IHCs stained against Vglut3 and Ctbp2 from 14-week-old RTN4RL2<sup>+/+</sup> (left) and RTN4RL2<sup>-/-</sup> (right) mice. Scale bar = 5 mm. (B) Synaptic ribbons on the modiolar side show higher fluorescence intensity than the ones on the pillar side of mid-cochlear IHCs in both RTN4RL2<sup>+/+</sup> (left, N=2) RTN4RL2<sup>-/-</sup> (right, N=2) mice. (C) Average fluorescence intensity of modiolar ribbons per IHC is higher than the average fluorescence intensity of pillar ribbons (paired t-test, p < 0.001).

      (9) The authors show in Figure 2 - Supplement 3 that GluA2/3 staining is absent in the mutants. Are GluA4 receptors upregulated? Otherwise, synaptic transmission should be abolished, which would be a dramatic phenotype. Antibodies are available to analyze GluA4 expression, the experiment is thus feasible. Did the authors carry out recordings from SGNs?

      In response to the reviewer’s comment, we have performed GluA4 stainings in RTN4LR2<sup>-/-</sup> mice and did not detect any GluA4 positive signal in the mutants (new Figure 3-figure supplement 1). Unfortunately, our animal breeding license was expired at the time we received the reviews and that is why our results are from 14-week-old animals. To verify that the absence of GluA4 signal is not due to potential PSD loss in 14-week-old RTN4RL2<sup>-/-</sup>, we have additionally performed anti-Ctbp2, anti-Homer1 and anti-Vglut3 stainings in 14-week-old animals. Despite the reduced number, we still observed juxtaposing pre- and postsynaptic puncta. We assume that the reviewer asks for patch-clamp recordings from SGNs, which are, as we are confident the reviewer is aware of, technically very challenging and beyond the scope of the present study but an important objective for future studies.  In response to the reviewers comment we have added a statement to the discussion pointing to these patch-clamp recordings from SGNs as important objective for future studies.

      (10) The authors use SBEM to analyze SGN projections and synapses. The data suggest that a significant number of SGNs are not connected to IHCs. A reconstruction in Figure 3 shows hair cells and axons. It is not clear how the outline of hair cells was derived, but this should be indicated. Also, is this a defect in the formation of synapses and subsequent retraction of SGN projections? Or could RTN4RL2 mutants have a defect in axonal outgrowth and guidance that secondarily affects synapses? To address this question, it would be useful to sparsely label SGNs in mutants, for example with AAV vectors expression GFP, and to trace the axons during development. This would allow us to distinguish between models of RTN4RL2 function. As it stands, it is not clear that RTN4RL2 acts directly at synapses.

      We agree with the reviewer on the value of a developmental study of afferent connectivity but consider this beyond the scope of the present study. In response to the reviewer's comment, we have replaced the IHC outlines with volume-reconstructed IHCs in Figure 3B (now Figure 4B). Moreover, as shown in Figure 3F (now Figure 4F), most if not all type-I SGNs (both with and without ribbon) were unbranched in the mutants just like in wildtype (also shown for a larger sample in Hua et al., 2021), arguing against morphological abnormality during development.

      (11) The authors observe a tiny shift in the operation range of Ca<sup>2+</sup> channels that has no effect on synaptic vesicle exocytosis. It seems very unlikely that this difference can explain the auditory phenotype of the mutant mice.

      We assume that the statement refers to the normal exocytosis of mutant IHCs at the potential of maximal Ca<sup>2+</sup> influx (Figure 3G and H, now Figure 4G and H). We would like to note that this experiment was performed to probe for a deficit of synapse function beyond that of the Ca<sup>2+</sup> channel activation, but did not address the impact of the altered voltage—dependence of Ca<sup>2+</sup> channel activation. In response to the reviewer’s comment, we have now added further discussion to more clearly communicate that for the range of receptor potentials achieved near sound threshold we expect impaired IHC exocytosis as the Ca<sup>2+</sup> channels require slightly more depolarization for activation in the mutant IHCs.

      (12) ABR recordings were conducted in whole-body knockouts. Effects on auditory thresholds could be a secondary consequence of perturbation along the auditory pathway. Conditional knockouts or precisely designed rescue experiments would go a long way to support the authors' hypothesis. I realize that this is a big ask and floxed mice might not be available to conduct the study.

      Thanks for this helpful comment and, indeed, unfortunately, we do not have conditional KO mice at our disposal. We totally agree that this will be important also for clarifying the role of IHC vs. SGN expression of RTN4RL2. In response to the reviewer’s comment, we now discussed the shortcoming of using constitutive RTN4RL2<sup>-/-</sup> mice and added this important experiment on IHC and SGN specific deletion of RTN4RL2 as an objective of future studies.

      Reviewer #3 (Public review):

      In this study, the authors used RNAscope and immunostaining to confirm the expression of RTN4RL2 RNA and protein in hair cells and spiral ganglia. Through RTN4RL2 gene knockout mice, they demonstrated that the absence of RTN4RL2 leads to an increase in the size of presynaptic ribbons and a depolarized shift in the activation of calcium channels in inner hair cells. Additionally, they observed a reduction in GluA2/3 AMPA receptors in postsynaptic neurons and identified additional "orphan PSDs" not paired with presynaptic ribbons. These synaptic alterations ultimately resulted in an increased hearing threshold in mice, confirming that the RTN4RL2 gene is essential for normal hearing. These data are intriguing as they suggest that RTN4RL2 contributes to the proper formation and function of auditory afferent synapses and is critical for normal hearing. However, a thorough understanding of the known or postulated roles of RTN4Rl2 is lacking.

      We would like to thank the reviewer for appreciating the work and the advice that helped us to further improve the manuscript. We have carefully addressed all concerns, please see our point-per-point response below and the revised manuscript.

      While the conclusions of this paper are generally well supported by the data, several aspects of the data analysis warrant further clarification and expansion.

      (1) A quantitative assessment is necessary in Figure 1 when discussing RNA and protein expression. It would be beneficial to show that expression levels are quantitatively reduced in KO mice compared to wild-type mice. This suggestion also applies to Figure 2-supplement 3.D, which examines expression levels.

      The processing of our control and KO samples for RNAscope was not strictly done in parallel and therefore we would like to refrain from quantitative comparison.

      (2) In Figure 2, the authors present a morphological analysis of synapses and discuss the presence of "orphan PSDs." I agree that Homer1 not juxtaposed with Ctbp2 is increased in KO mice compared to the control group. However, in quantifying this, they opted to measure the number of Homer1 juxtaposed with Ctbp2 rather than directly quantifying the number of Homer1 not juxtaposed with Ctbp2. Quantifying the number of Homer1 not juxtaposed with Ctbp2 would more clearly represent "orphan PSDs" and provide stronger support for the discussion surrounding their presence.

      We appreciate the reviewer’s comment. We did not perform this analysis primarily because “orphan” Homer1 puncta, as seen in our immunostainings, are distributed away from hair cells in diverse morphologies and sizes. This makes distinguishing them from unspecific immunofluorescent spots—also present in wild-type samples—challenging. In response to the reviewer’s request, we analyzed the number of “orphan” Homer1 puncta in our previously acquired RTN4RL2<sup>+/+</sup> and RTN4RL2<sup>-/-</sup> samples. Using the surface algorithm in Imaris software, we applied identical parameters across all samples to create surfaces for Homer1-positive puncta (total Homer1 puncta). We quantified “orphan” Homer1 puncta as the difference between total and ribbon-juxtaposing Homer1 puncta and normalized this number to the IHC count. Our results showed 4.3 vs. 26.8 “orphan” Homer1 puncta per IHC in RTN4RL2<sup>+/+</sup> and RTN4RL2<sup>-/-</sup> samples, respectively. We note that variations in acquired volumes between samples may introduce confounding effects.

      (3) In Figure 2, Supplementary 3, the authors discuss GluA2/3 puncta reduction and note that Gria2 RNA expression remains unchanged. However, there is an issue with the lack of quantification for Gria2 RNA expression. Additionally, it is noted that RNA expression was measured at P4. While the timing for GluA2/3 puncta assessment is not specified, if it was assessed at 3 weeks old as in Figure 2's synaptic puncta analysis, it would be inappropriate to link Gria2 RNA expression with GluA2/3 protein expression at P4. If RNA and protein expression were assessed at P4, please indicate this timing for clarity.

      GluA2/3 immunostainings were performed in 1 to 1.5-month-old animals. We apologize for not indicating this before and have now included it in Figure 3 legend. The processing of our control and KO samples for RNAscope was not strictly done in parallel and therefore we would like to refrain from quantitative comparison.

      (4) In Figure 3, the authors indicate that RTN4RL2 deficiency reduces the number of type 1 SGNs connected to ribbons. Given that the number of ribbons remains unchanged (Figure 2), it is important to clearly explain the implications of this finding. It is already known that each type I SGN forms a single synaptic contact with a single IHC. The fact that the number of ribbons remains constant while additional "orphan PSDs" are present suggests that the overall number of SGNs might need to increase to account for these findings. An explanation addressing this would be helpful.

      In Figure 3 (now Figure 4), we found additional type-1 SGNs that are unconnected to IHC, in good agreement with “orphan PSDs” observed under the light microscope. Indeed, we also confirmed monosynaptic, unbranched fiber morphology (Figure 3F, now Figure 4F). Together, these results imply about a 20% increase in the overall number of SGNs, which however we did not observe in SGN soma counting.

      (5) In Figure 4F and 5Cii, could you clarify how voltage sensitivity (k) was calculated? Additionally, please provide an explanation for the values presented in millivolts (mV).

      Voltage sensitivity (k) was calculated as the slope of the Boltzmann fit to the fractional activation curves: , Where G is conductance, G<sub>max</sub> is the maximum conductance, V<sub>m</sub> is the membrane potential, V<sub>half</sub> is the voltage corresponding to the half maximal activation of Ca<sup>2+</sup> channels and k (slope of the curve) is the voltage sensitivity of Ca<sup>2+</sup> channel activation. We have now added this to our Materials and Methods section.

      (6) In Figure 6, the author measured the threshold of ABR at 2-4 months old. Since previous figures confirming synaptic morphology and function were all conducted on 3-week-old mice, it would be better to measure ABR at 3 weeks of age if possible.

      ABR measurements for comparisons in a cohort of age-matched mice require fully developed individuals. 3 weeks is the minimum age that is regarded for a mature ear. However, variation in developmental differences among one litter is very frequent that affects normal hearing thresholds. From our own experience we do not regard the ear fully functional before 6 weeks of age. Then hearing thresholds are lowest indicating full functionality. Since the C57BL/6 background strain has a genetic defect in the Cadherin 23-coding gene (Cdh23) at the ahl locus of mouse chromosome 10 these mice exhibit early onset and progression of age-related hearing loss starting at 5–8 months (Hunter & Willott, 1987). Therefore, we chose a “safe” time window for stable and unaffected ABR recordings of 2-4 months to provide most representative data.

      Recommendations for the authors:

      Reviewer #1 (Recommendations for the authors):

      (1) Please include information on the validation of all the antibodies used in this study, or reference the relevant work where the antibodies were previously validated.

      In response to the reviewer’s comment, we have now included a table listing all primary antibodies used in this study. Where possible, we provide references for knockout (KO) validation. Otherwise, we refer to the manufacturer’s information, as provided in the respective datasheets.

      (2) Figure 2 illustrates the pre- and postsynaptic changes observed in RTN4RL2 knockout (KO) mice. Please specify the age of the mice and the cochlear region depicted and analyzed in Figure 2.

      We thank the reviewer for the comment. The IHCs of apical cochlear region were analyzed in mice at 3 weeks of age. We have now added this to the figure legend.

      (3) The discovery of orphan SGN neurites in RTN4RL2 KO mice is particularly intriguing. I wonder whether the additional Homer1-positive puncta illustrated in Figure 2 are present in these orphan SGN neurites, which would suggest that they may be functional. Conducting immunohistochemistry (IHC) labeling for type I SGN neurites using an anti-Tuj1 antibody, along with Homer1, would help localize the additional Homer1 puncta shown in Figure 2. Additionally, the "extra" Homer1 puncta appears less striking in the data presented in Figure 2-Supplement 2. Quantifying the number of Homer1 puncta in wild-type versus KO mice across different cochlear regions will help visualize the Figure 2-Supplement 2 data and relate the presence of extra neurites to the increased auditory brainstem response (ABR) thresholds observed at all frequencies.

      We thank the reviewer for the comment and we agree that localizing orphan PSDs on the SGN neurites would be very useful. Unfortunately, the animal breeding license in the Göttingen lab had expired. At the time we received the reviews we only had access to 14-week-old animals and could not perform the stainings in animals which would have comparable age range to the rest of the study (3-4 weeks). The phenotype of extra Homer1 puncta was not as drastic in 14-week-old animals as it was in previously stained 3-week-old animals. Nevertheless, we still tried NF200, Homer1 and Vglut3 immunostainings in 14-week-old animals. We present representative single imaging planes of NF200, Homer1 and Vglut3 stainings in Author response image 4. Additionally, we provide exemplary images from 7-week-old RTN4RL2<sup>-/-</sup>, where it looks like that the orphan Homer1 puncta are found on calretinin positive neurites.

      Author response image 4.

      Attempts to localize “orphan” Homer1 patches on type I SGN neurites. (A) Single exemplary imaging planes of apical IHC region from RTN4RL2<sup>+/+</sup> (left) and RTN4RL2<sup>-/-</sup> (right) mice immunolabeled against NF200, Vglut3 and Homer1. White arrows show putative “orphan” Homer1 puncta on NF200 positive neurites. Scale bar = 5 mm. (B) Maximum intensity projections of representative confocal stacks of IHCs from RTN4RL2<sup>-/-</sup> mice immunolabeled against Calretinin and Homer1. Scale bars = 5 mm. White arrows show possible “orphan” Homer1 puncta on Calretinin positive boutons.

      (4) The authors noted a reduction in the number of GluA2/3-positive puncta in RTN4RL2 KOs, as shown in Figure 2-Supplement 3. However, in the Results section (page 5, line 124), it is unclear whether the authors refer to a reduction in fluorescence intensity or the number of puncta. Please clarify this.

      We thank the reviewer for the comment. We refer to the number and have now added this to the manuscript.

      (5) I find it particularly interesting that, despite the presence of smaller but synaptically engaged Homer1-positive SGN neurites, these appear to lack or present a reduction in the number of GluA2/3 puncta, and that GluA2/3 puncta are observed in non-ribbon juxtaposed neurites. Therefore, I suggest including GluA2/3 (Fig2 supplement 3) data in the main figure. It would be valuable to determine whether the orphan neurites express both Homer1 and GluA2/3, which could indicate that the defect is not solely due to reduced GluA2/3 expression at the formed synapses, but also to the presence of additional orphan synapses. I would also mention in the discussion how the phenotype of the RTN4L2 KO compares to the GluA2/3 KO and if the lack of GluA2/3 at the AZ could explain the increase in ABR threshold. Quantification of GluA2/3 puncta at the apical, middle, and basal region would also help understand the auditory phenotype of the KO mice.

      We have changed Figure2-figure supplement 3 to become a main figure (Figure 3) based on the recommendation of the reviewer. We agree, that it would be valuable to perform immunohistochemistry combining anti-GluA2/3 and anti-Homer1 and anti-Ctbp2 antibodies to see if the “orphan” Homer1 patches house GluA2/3 not juxtaposing synaptic ribbons. Unfortunately, as mentioned above, due to the expiration of our animal breeding and experimentation licenses we did not manage to do those experiments. We have however performed stainings with anti-GluA4 antibodies and could not detect GluA4 signal in RTN4RL2<sup>-/-</sup> mice (Figure 3-figure supplement 1). This potentially could explain the more drastic ABR threshold elevation in RTN4RL2<sup>-/-</sup> mice compared to e.g. GluA3 KO mice. We have now made this clearer in our discussion.

      (6) I suggest considering the use of color-blind friendly palettes for figures and graphs in this manuscript to enhance clarity and ensure that the findings are accessible to a wider audience and improve the overall effectiveness of the presentation. Please use color-blind-friendly schemes in Figure 1 and Figure 2 Supplement 3.

      Done.

      (7) Could you please explain what "XX {plus minus} Y, SD = W" means in the figure legends?

      Mean ± SEM (standard error of the mean), SD (standard deviation) are indicated in the legends. In response to the reviewer comment we have now added an explanation in the Materials and Methods –> Data analysis and statistics section.

      (8) Please include information about the ear tested (left or right or both).

      Both ears were tested. Since there was no significant difference between right and left ear we did not further consider this factor. We will add this fact more precisely in the Material and methods section.

      Reviewer #3 (Recommendations for the authors):

      (1) Line 90: Why not show this control, it is a nice control.

      Unfortunately, our recent attempts to perform RTN4RL2 immunostaining on cryosections were unsuccessful. Therefore, we decided to remove RTN4RL2 immunostaining from Figure 1 and have adjusted the results section accordingly.

      (2) Line 94: Please provide a reference for these interactions.

      Done.

    1. Author Response

      The following is the authors’ response to the original reviews.

      Reviewer #1 (Public Review):

      The Hedgehog (HH) protein family is important for embryonic development and adult tissue maintenance. Deregulation or even temporal imbalances in the activity of one of the main players in the HH field, sonic hedgehog (SHH), can lead to a variety of human diseases, ranging from congenital brain disorders to diverse forms of cancers. SHH activates the GLI family of transcription factors, yet the mechanisms underlying GLI activation remain poorly understood. Modification and activation of one of the main SHH signalling mediators, GLI2, depends on its localization to the tip of the primary cilium. In a previous study the lab had provided evidence that SHH activates GLI2 by stimulating its phosphorylation on conserved sites through Unc-51-like kinase 3 (ULK3) and another ULK family member, STK36 (Han et al., 2019). Recently, another ULK family member, ULK4, was identified as a modulator of the SHH pathway (Mecklenburg et al. 2021). However, the underlying mechanisms by which ULK4 enhances SHH signalling remained unknown. To address this question, the authors employed complex biochemistry-based approaches and localization studies in cell culture to examine the mode of ULK4 activity in the primary cilium in response to SHH. The study by Zhou et al. demonstrates that ULK4, in conjunction with STK36, promotes GLI2 phosphorylation and thereby SHH pathway activation. Further experiments were conducted to investigate how ULK4 interacts with SHH pathway components in the primary cilium. The authors show that ULK4 interacts with a complex formed between STK36 and GLI2 and hypothesize that ULK4 functions as a scaffold to facilitate STK36 and GLI2 interaction and thereby GLI2 phosphorylation by STK36. Furthermore, the authors provide evidence that ULK4 and STK36 co-localize with GLI2 at the ciliary tip of NIH 3T3 cells, and that ULK4 and STK36 depend on each other for their ciliary tip accumulation. Overall, the described ULK4-mediated mechanism of SHH pathway modulation is based on detailed and rigorous Co-IP experiments and kinase assays as well as confocal imaging localization studies. The authors used various mutated and wild-type constructs of STK36 and ULK4 to decipher the mechanisms underlying GLI2 phosphorylation at the tip of the primary cilium. These novel results on SHH pathway activation add valuable insight into the complexity of SHH pathway regulation. The data also provide possible new strategies for interfering with SHH signalling which has implications in drug development (e.g., cancer drugs).

      However, it will be necessary to explore additional model systems, besides NIH3T3, HEK293 and MEF cell cultures, to conclude on the universality of the mechanisms described in this study. Ultimately, it needs to be addressed whether ULK4 modulates SHH pathway activity in vivo. Is there evidence that genetic ablation of ULK4 in animal models leads to less efficient SHH pathway induction? It also remains to be resolved how ULK3 and ULK4 act in distinct or common manners to promote SHH signalling. Another remaining question is, whether cell type- and tissue-specific features exist, that play a role in ULK3- versus ULK4-dependent SHH pathway modulation. In particular for the studies on ciliary tip localization of factors, relevant for SHH pathway transduction, a higher temporal resolution will be needed in the future as well as a deeper insight into tissue/ cell type-specific mechanisms. These caveats, mentioned here, don't have to be addressed in new experiments for the revision of this manuscript but could be discussed.

      We agree with the reviewer that it would be important to investigate in the future the in vivo function Ulk4 in Shh signaling, the relationship between Ulk3 and Ulk4/Stk36, and possible cell type/tissue specificity of these two kinase systems. This will need the generation of single and double knockout mice and examine Hh related phenotypes in different tissues and developmental stages. The precise mechanism by which Ulk4 and Stk36 are translocated to the ciliary tip is also an important and unsolved issue. We include several paragraphs in the “discussion” section to address these outstanding questions for future study.

      Reviewer #2 (Public Review):

      The authors provide solid molecular and cellular evidence that ULK4 and STK36 not only interact, but that STK36 is targeted (transported?) to the cilium by ULK4. Their data helps generate a model for ULK4 acting as a scaffold for both STK36 and its substrate, Gli2, which appear to co-localise through mutual binding to ULK4. This makes sense, given the proposed role of most pseuodkinases as non-catalytic signaling hubs. There is also an important mechanistic analysis performed, in which ULK4 phosphorylation in an acidic consensus by STK36 is demonstrated using IP'd STK36 or an inactive 'AA' mutant, which suggests this phosphorylation is direct.

      The major strength of the study is the well-executed combination of logical approaches taken, including expression of various deletion and mutation constructs and the careful (but not always quantified in immunoblot) effects of depleting and adding back various components in the context of both STK36 and ULK3, which broadens the potential impact of the work. The biochemical analysis of ULK4 phosphorylation appears to be solid, and the mutational study at a particular pair of phosphorylation sites upstream of an acidic residue (notably T2023) is further strong evidence of a functional interaction between ULK4/STK36. The possibility that ULK4 requires ATP binding for these mechanisms is not approached, though would provide significant insight: for example it would be useful to ask if Lys39 in ULK4 is involved in any of these processes, because this residue is likely important for shaping the ULK4 substrate-binding site as a consequence of ATP binding; this was originally shown in PMID 24107129 and discussed more recently in PMID: 33147475 in the context of the large amount of ULK4 proteomics data released.

      The reviewer raised an interesting question of whether ATP binding to the pseudokinase domain of Ulk4 might be required for its function, i.e., by regulating the interaction with its binding partner. In a recent study (Preuss et al. 2020;PMID: 33147475), the critical Lys39 for ATP binding was converted to Arg (KR mutation); however, unlike in most kinases the KR mutation affect ATP binding, the K39R mutation in the Ulk4 pseudokinase did not affect ATP binding although it slightly increased ADP binding (PMID: 33147475). Another mutation made by Preuss et al(PMID: 33147475), N239L, affected protein stability, making it impossible to determine whether this mutation affect ATP binding. Therefore, in the absence of clear approach to perturb ATP binding without affecting the overall structure of Ulk4, it would be challenging to address whether ATP binding regulates the ability of Ulk4 to bind its substrates. Nevertheless, we discuss the possibility that ATP binding might regulate Ulk4/Stk36 interaction and Shh signaling.

      The discussion is excellent, and raises numerous important future work in terms of potential transportation mechanisms of this complex. It also explains why the ULK4 pseudokinase domain is linked to an extended C-terminal region. Does AF2 predict any structural motifs in this region that might support binding to Gli2?

      The extended C-terminal domain of Ulk4 contains Arm/HEAT repeats (protein-protein interacting domain), which are predicted by AF2 to form alpha helixes.

      A weakness in the study, which is most evident in Figure 1, where Ulk4 siRNA is performed in the NIH3T3 model (and effects on Shh targets and Gli2 phosphorylation assessed), is that we do not know if ULK4 protein is originally present in these cells in order to actually be depleted. Also, we are not informed if the ULK4 siRNA has an effect on the 'rescue' by HA-ULK4; perhaps the HA-ULK4 plasmid is RNAi resistant, or if not, this explains why phosphorylation of Gli2 never reaches zero? Given the important findings of this study, it would be useful for the authors to comment on this, and perhaps discuss if they have tried to evaluate endogenous levels of ULK4 (and Stk36) in these cells using antibody-based approaches, ideally in the presence and absence of Shh. The authors note early on the large number of binding partners identified for ULK4, and siRNA may unwittingly deplete some other proteins that could also be involved in ULK4 transport/stability in their cellular model.

      Due to the lack of reliable Ulk4 and Stk36 antibodies, we were unable to confirm knockdown efficiency by western blot analysis. Therefore, we relied on the measure Ulk4 and STk36 mRNA expression by RT-qPCR to estimate the knockdown efficiency (Fig 1- figure supplement 1). We used mouse Ulk4 shRNA to carry out the knockdown experiments in NIH3T3 and MEF cells while the human version of Ulk4 (hUlk4) was used for the rescue experiments (Fig 1- figure supplement 2; Fig. 8). We have confirmed that the mUlk4 shRNA targeting sequence is not conserved in hUlk4; therefore, the hULK4 construct is RNAi resistant. The rescue experiments strongly argue that the effect of Ulk4 RNAi on Shh signaling is due to loss of endogenous Ulk4. This argument is further strengthened by the observations that mutations that affected Ulk4 and Stk36 ciliary tip localization also affected Shh signaling such as Gli2 phosphorylation and Ptch1/Gli expression (Fig. 8).

      The sequence of ULK4 siRNAs is not included in the materials and methods as far as I can see.

      We have added the mouse Ulk4 RNAi target sequence in the revised version.

      Reviewer #3 (Public Review):

      In this manuscript, Zhou et al. demonstrate that the pseudokinase ULK4 has an important role in Hedgehog signaling by scaffolding the active kinase Stk36 and the transcription factor Gli2, enabling Gli2 to be phosphorylated and activated.

      Through nice biochemistry experiments, they show convincingly that the N-terminal pseudokinase domain of ULK4 binds Stk36 and the C-terminal Heat repeats bind Gli2.

      Lastly, they show that upon Sonic Hedgehog signaling, ULK4 localizes to the cilia and is needed to localize Stk36 and Gli2 for proper activation.

      This manuscript is very solid and methodically shows the role of ULK4 and STK36 throughout the whole paper, with well controlled experiments. The phosphomimetic and incapable mutations are very convincing as well. I think this manuscript is strong and stands as is, and there is no need for additional experiments.

      Overall, the strengths are the rigor of the methods, and the convincing case they bring for the formation of the ULK4-Gli2-Stk36 complex. There are no weaknesses noted. I think a little additional context for what is being observed in the immunofluorescence might benefit readers who are not familiar with these cell types and structures.

      We thank this reviewer for the positive comments.

      Recommendations For the Authors

      Reviewer #1 (Recommendations For The Authors):

      This elegant study has been thoroughly and thoughtfully designed and the dataset is solid. The biochemistry results are overall very convincing. Some data lack quantification and there needs to be more information on data analyses and statistics. The following suggestions and comments aim at strengthening the manuscript.

      1. Please provide quantification normalized to input for IP experiments (Figures 1 E - F; Figure 8 C). More information on data analyses and statistics should be provided and included as information in the figure legends.

      Thanks for the suggestions, we have done the quantification and statistics analyses for Figures 1E-G and Figure8 C as requested.

      1. Did the authors investigate whether overexpressing hULK4 in the control NIH3T3 cells leads to an increase in pS230/232 (related to Figure 1E)? This would nicely support the notion of a promoting effect of ULK4 on GLI2 phosphorylation.

      We did not. We speculated that overexpressing hULK4 may not significantly promote GLI2 phosphorylation because Ulk4 is a pseudokinase and endogenous Stk36 (the kinase partner of Ulk4) is limited.

      1. The CO-IP experiments to show GLI2 activation were performed in NIH3T3 cells, whereas HEK293 cells were used for the experiments shown in Figure 2. Is there a specific reason for switching between cell lines also for experiments shown in Figures 3 C- I? Did the authors repeat some of the key experiments in both cell lines?

      In mammalian cells, Shh-induced activation of GLI2 depends on primary cilia (Han et al., 2019). NIH3T3 cells form the primary cilia but HEK293T cells do not. Therefore, we used NIH3T3 cells to examine the processes that are regulated by the Shh treatment assay (e.g., the Shh-induced phosphorylation of GLI2 and STK36). The HEK293 cells were used to map binding domain between ULK4 and STK36/GLI2/SUFU due to the high transfection efficiency.

      1. In Figure 2 D-E the authors nicely showed that hUlk4N-HA interacted with CFP-Stk36 but not with Myc-Gli2/Fg-Sufu whereas hUlk4C-HA formed a complex with Myc-Gli2/Fg-Sufu but not with CFP-Stk36. In Figure 4E the authors showed in their Co-IP experiments that Fg-Stk36 and Myc-Gli2 form a complex independent of SHH treatment. Did the authors see some pull down of Stk36, still in complex with Gli2, using hUlk4C IP and pull down of Gli2, still in complex with Stk36, using hUlk4N IP?

      We did not test that. As we have shown in Figures 4A and 4E, knockdown of endogenous ULK4 nearly abolished the interaction between Myc-GLi2 and Fg-Stk36, suggesting that Ulk4 is the major scaffold to bring Skt36 and Gli2 together, and that there is little if any direct interaction between GLi2 and Stk36.

      1. Another method to verify hULK4-Stk36-Gli2 complex formation (Figure 4) would be helpful. For example, proximity ligation assays, tripartite split GFP assays, or colocalization based on expansion STED immunofluorescence microscopy could be performed to temporally and spatially resolve localization of Ulk4, Stk36 and Gli2 upon SHH stimulation in the primary cilium

      Thanks for the suggestions. We think that our current study using biochemical and cell biology approaches have provide sufficient evidence that Ulk4, Stk36 and Gli2 form complexes. We will keep in mind of those more sophisticated methods in our future endeavors.

      1. Please provide more representative images of Ulk4, Stk36 and Gli2 localization in NIH3T3 cells or lower magnification overview images showing more than one cell (Figure 5).

      We have provided more representative images in Figure 5- figure supplement 1A-F of the revised manuscript.

      1. Confirmation of the results shown in Figure 5 in a second cell line would strengthen the data.

      We have confirmed the results in MEFs (see Figure 5- figure supplement 1G-J)

      1. Did the authors add immunofluorescence for tubulin as a ciliary base marker to ensure correct assignment of ciliary tip versus ciliary base localization for quantification experiments (Figures 5 - 8)?

      It has been well documented that GLi2 is accumulated at the ciliary tip in respond to Shh treatment; therefore, we used Gli2 as a marker for ciliary tip where both Ulk4 and Stk36 were also accumulated. γ tubulin staining could be another marker to assign the ciliary tip vs base; however, the antibody combination we have did not allow us to simultaneously stain γ tubulin and acetylated tubulin (Ac-Tub).

      1. SMO localization as a further readout of SHH pathway activation might be considered to be added for some of the key results (e.g., Figure 6). Is SMO trafficking affected after depletion or overexpression of ULK4?

      Due to the lack of a workable antibody to detect endogenous Smo in our hands, we did not determine whether the trafficking of SMO is affected after depletion or overexpression of ULK4. However, we noticed that a recent study reported that the SHH-induced ciliary SMO accumulation was impaired in Ulk4 siRNA treated cells (Mecklenburg et al. 2021). We include this information and its implication in the discussion section

      1. Do the authors see ULK4 only at the ciliary tip after SHH stimulation or is there also a dynamic time-dependent localization along the ciliary shaft? The image in Figure 6E (dKO + Stk36 WT) seems to show ULK4 also in the shaft.

      Unlike Smo that is evenly distributed alone the axoneme of primary cilia, ULK4 is mainly accumulated at ciliary tips upon Shh stimulation. Ulk4 is also located at low levels outside the cilia and sometimes in the ciliary shaft during its transit to the ciliary tip (e.g., see Figure 5- figure supplement 1F1-2; J1-2).

      1. Is the immunofluorescence signal for Ulk4 significantly reduced after shRNA treatment to deplete Ulk4 (Figure 6A)?

      We constructed a cell line that stably expressed ULK4 shRNA. The knockdown efficiency was determined by measuring Ulk4 mRNA expression (Fig 1_figure supplement 1). Because we were unable to obtain a reliable ULK4 antibody for immunostaining, we did not examine by whether ULK4 signal was depleted by Ulk4 shRNA.

      1. The labelled ciliary tip resembles in some cases images seen for ciliary abscission. The authors could use membrane/ciliary membrane markers to ensure "intraciliary" localization of the investigated factors.

      Thanks for the suggestion. We will try that in our future experiments.

      1. How many replicates were used in the three independent quantitative RT-PCR experiments (Figure 1 A-D)?

      We used 3 replicates in each independent quantitative RT-PCR assay.

      1. Please provide p values or statement on no significance for the comparison between Ulk3 single and Ulk3/Ulk4 double knockdown (Figure 1C) and between Stk36 single and Stk36/Ulk4 double knockdown (Figure 1D; Fig1_Figure Supplement 2).

      Thanks for the suggestion, we have added the p value or “ns” as asked.

      1. Figure legends in general are a bit short could have some more detailed information.

      Thank you for the suggestion, we have revised the Figure legends as asked.

      1. What do the asterisks present in Figure 4 C-D?

      Thanks for the suggestion. The asterisks in Figure 4C-D indicated the full length STK36 and truncated form STK36N and STK36C fragments. We that included this information in the figure legend.

      1. The authors state that a previous study described ULK4 as a genetic modifier for holoprosencephaly and that this raised the possibility that ULK4 may participate in HH signal transduction. Primary ciliary localization of ULK4 in mouse neuronal tissue and SHH pathway modulation by ULK4 in cell culture have been shown by Mecklenburg et al. 2021 before. Maybe the authors could rephrase their introduction and discussion accordingly.

      Thanks for the suggestion, we have changed the introduction and discussion accordingly.

      1. Overexpression studies in heterologous systems using tagged proteins can potentially have an influence on their subcellular localization and function. Please discuss this caveat.

      We have mentioned this caveat in the “discussion” section of the revised manuscript. However, we have tried to express the transgene at low levels using the lentiviral vector containing a weak promoter to ensure that the exogenously expressed proteins are still regulated by Hh signaling. We have also confirmed that the tagged Ulk4 and Stk36 can rescue the loss of endogenous genes.

      1. More details in the Methods section should be provided on the SHH induction in NIH3T3 cells, HEK293 cells and MEFs.

      We have revised the methods section on Shh induction.

      1. ULK4 is known to have at least three isoforms that exhibit varying abundance across developmental stages in mice and humans (Lang et al., 2014) (DOI:10.1242/jcs.137604). Can the authors speculate on potential common and distinct functions of the different ULK4 isoforms on SHH pathway modulation based on their present results?

      It is interesting that Ulk4 has multiple isoforms in both mouse and human. Several short isoforms in both mouse and human lack the pseudokinase domain while one short isoform in mouse lacks the C-terminal region essential for Ulk4 ciliary tip localization. We speculate that the C-terminally deleted isoform may not have a function in the Shh pathway based on our results shown in Fig. 7 and 8 but might still have functions in other cellular processes.

      Reviewer #2 (Recommendations For The Authors):

      The paper is well written, and clear throughout, with excellent (up-to-date) citations to the field.

      We thank reviewer #2 for the positive comments.

      Reviewer #3 (Recommendations For The Authors):

      My only quibble is that the immunofluorescence images are a little confusing, especially to people outside of the field. Please include an image of the whole field and improve the captions. Is that a single cell for each cilia? Why are there so few cilia? The DAPI makes it seem like What are we looking at? Are those multiple nuclei in Figure 6? They seem a little small if that's the 5 uM scale bar

      We provide uncropped images of Figure 5E to show the entire cells (below). We have added some context to improve the captions. Most of the mammalian cells such as MEF and NIH3T3 cells contain a single primary cilium; however, mutilated cells do exist. The DAPI staining indicated the nuclei. The cells shown in Figure 6 have single nucleus (the scale should be 2 µM). Due to the unevenness of DAPI signals in the nuclei, only the strong signals (puncta) were shown for individual nuclei.

      Author response image 1.

      One small typo: GLL2 instead of GLI2 on line 363

      Thanks, we have corrected this spelling mistake.

    1. Author Response

      The following is the authors’ response to the original reviews.

      Reviewer #1 (Public Review):

      The present work establishes 14-3-3 proteins as binding partners of spastin and suggests that this binding is positively regulated by phosphorylation of spastin. The authors show evidence that 14-3-3 >- spastin binding prevents spastin ubiquitination and final proteasomal degradation, thus increasing the availability of spastin. The authors measured microtubule severing activity in cell lines and axon regeneration and outgrowth as a prompt to spastin activity. By using drugs and peptides that separately inhibit 14-3-3 binding or spastin activity, they show that both proteins are necessary for axon regeneration in cell culture and in vivo models in rats.

      The following is an account of the major strengths and weaknesses of the methods and results.

      Major strengths

      -The authors performed pulldown assays on spinal cord lysates using GST-spastin, then analyzed pulldowns via mass spectrometry and found 3 peptides common to various forms of 14-3-3 proteins. In co-expression experiments in cell lines, recombinant spastin co-precipitated with all 6 forms of 14-3-3 tested.

      -By protein truncation experiments they found that the Microtubule Binding Domain of spastin contained the binding capability to 14-3-3. This domain contained a putative phosphorylation site, and substitutions that cannot be phosphorylated cannot bind to spastin.

      -spastin overexpression increased neurite growth and branching, and so did the phospho null spastin. On the other hand, the phospho mimetic prevents all kinds of neurite development.

      -Overexpression of GFP-spastin shows a turn-over of about 12 hours when protein synthesis is inhibited by cycloheximide. When 14-3-3 is co-overexpressed, GFP-spastin does not show a decrease by 12 hours. When S233A is expressed, a turn-over of 9 hours is observed, indicating that the ability to be phosphorylated increases the stability of the protein.

      -In support of that notion, the phospho-mimetic S233D makes it more stable, lasting as much as the over-expression of 14-3-3.

      -Authors show that spastin can be ubiquitinated, and that in the presence of ubiquitin, spastin-MT severing activity is inhibited.

      -By combining FCA with Spastazoline, the authors claim that FCA increased regeneration is due to increased spastin Activity in various models of neurite outgrowth and regeneration in cell culture and in vivo, the authors show impressive results on the positive effect of FCA in regeneration, and that this is abolished when spastin is inhibited.

      Major weaknesses

      -However convincing the pull-downs of the expressed proteins, the evidence would be stronger if a co-immunoprecipitation of the endogenous proteins were included.

      We thank the reviewer for their succinct summary of the main results and strengths of our study. We acknowledge the reviewers' valuable suggestions and agree that performing endogenous co-immunoprecipitation (co-IP) experiments in neurons is crucial for supporting our conclusions. To address this question, cortical neurons were cultured in vitro for endogenous IP experiment. The cortical neurons were cultured using a neurobasal medium supplemented with 2% B27, and using cytarabine to inhibit the proliferation of glial cells. The proteins were then extracted and subjected to the immunoprecipitation experiments using antibodies against spastin. The results, as shown in Fig.1C in the revised manuscript, clearly demonstrate that 14-3-3 protein indeed interacts with spastin within neurons.

      -To better establish the impact of spastin phosphorylation in the interaction, there is no indication that the phosphomimetic (S233D) can better bind spastin, and this result is contradicting to the conclusion of the authors that spastin-14-3-3 interaction is necessary for (or increases) spastin function.

      Thank you for your valuable and constructive comments. We agree with your consideration. To reinforce the importance of phosphorylated spastin in this binding model, we conducted additional experiments by transfecting S233D into 293T cells and performed immunoprecipitation experiments (Fig.2H). The results clearly demonstrate that spastin (S233D) exhibits enhanced binding to spastin, indicating that phosphorylation at the S233 site is critical for this interaction. Additionally, we observed that spastin (S233D) maintains its binding to 14-3-3 even in the presence of staurosporine. This data further supports and strengthens our conclusions.

      -To fully support the authors' suggestion that 14-3-3 and spastin work in the same pathway to promote regeneration, I believe that some key observations are missing.

      1-There is no evidence showing that 14-3-3 overexpression increases the total levels of spastin, not only its turnover.

      Thank you for your consideration and valuable input. We have previously demonstrated that overexpression of 14-3-3 leads to an increase in the protein levels of spastin in the absence of CHX (Fig.3E&F). Furthermore, we also observed an upregulated protein levels of spastin S233D compared to the wild-type (Fig.3G). We have now included these results in the revised manuscript.

      2- There is no indication that increasing the ubiquitination of spastin decreases its levels. To suggest that proteasomal activity is affecting the levels of a protein, one would expect that proteasomal inhibition (with bortezomib or epoxomycin), would increase its levels.

      Thanks for your concern. We believe that this evidence is critical. Indeed, another study by our team is working to elucidate the ubiquitination degradation pathway of spastin. In addition, a previous study has shown that phosphorylation of the S233 site of spastin can affect its protein stability (Spastin recovery in hereditary spastic paraplegia by preventing neddylation-dependent degradation, doi:10.26508/lsa.202000799.). To better support our conclusions, we have supplemented the results in Fig.3L&M. The results showed that the proteasome inhibitor MG132 could significantly increase the protein level of spastin, whereas CHX could significantly decrease the protein level of spastin, and the degradation of spastin is significantly hindered in the presence of both CHX and MG132. This experiment also further showed that ubiquitination of spastin reduced its protein level.

      3- Authors show that S233D increases MT severing activity, and explain that it is related to increased binding to 14-3-3. An alternative explanation is that phosphorylation at S233 by itself could increase MT severing activity. The authors could test if purified spastin S233D alone could have more potent enzymatic activity.)

      We appreciate the reviewer’s consideration. After investigating the interaction between 14-3-3 and spastin, we first aimed to determine whether the S233 phosphorylation mutation of spastin influenced its microtubule-severing activity. We found that overexpression of both S233A and S233D mutants resulted in significant microtubule severing (as indicated by a significant decrease in microtubule fluorescence intensity) (Fig.S2). Furthermore, it is noteworthy that S233 is located outside the microtubule-binding domain (MTBD, 270-328 amino acids) and the AAA region (microtubule-severing region, 342-599 amino acids) of spastin. Based on our initial observations, we believe that the phosphorylation of the S233 residue in spastin does not impact its microtubule-severing function. Additionally, under the same experimental conditions, we observed that the green fluorescence intensity of GFP-spastin S233D was significantly higher than that of GFP-spastin S233A. Based on these phenomena, we speculated that phosphorylation of the S233 residue of spastin might affect its protein stability, leading us to conduct further experiments. Furthermore, we fully acknowledge the reviewer's concern; however, due to technical limitations, we were unable to perform an in vitro assay to test the microtubule-severing activity of spastin. We have provided an explanation for this consideration in the revised version.

      -Finally, I consider that there are simpler explanations for the combined effect of FC-A and spastazoline. FC-A mechanism of action can be very broad, since it will increase the binding of all 14-3-3 proteins with presumably all their substrates, hence the pathways affected can rise to the hundreds. The fact that spastazoline abolishes FC-A effect, may not be because of their direct interaction, but because spastin is a necessary component of the execution of the regeneration machinery further downstream, in line with the fact that spastizoline alone prevented outgrowth and regeneration, and in agreement with previous work showing that normal spastin activity is necessary for regeneration.

      We appreciate the considerations raised by the reviewer. It is evident that spastin is not the exclusive substrate protein for 14-3-3, and it is challenging to demonstrate that 14-3-3 promotes nerve regeneration and recovery of spinal cord injury directly through spastin in vivo. However, we have identified the importance of 14-3-3 and spastin in the process of nerve regeneration. Importantly, we have conducted supplementary experiments to support the stabalization of spastin by FC-A treatment within neurons (Fig.4M), as well as the repair process of spinal cord injury in vivo (Fig.5D). The results showed that FC-A treatment in cortical neurons could enhance the stability of spastin protein levels, and we also demonstrated a consistent trend of upregulated protein levels of spastin and 14-3-3 following spinal cord injury. Moreover, the protein levels were significantly elevated in the the FC-A group of mice. These results also support that 14-3-3 enhances spastin protein stability to promote spinal cord injury repair. The manuscript was revised accordingly.

      Reviewer #2 (Public Review):

      Summary:

      The idea of harnessing small molecules that may affect protein-protein interactions to promote axon regeneration is interesting and worthy of study. In this manuscript, Liu et al. explore a 14-3-3-spastin complex and its role in axon regeneration.

      Strengths:

      Some of the effects of FC-A on locomotor recovery after spinal cord contusion look interesting.

      Weaknesses:

      The manuscript falls short of establishing that a 14-3-3-spastin complex is important for any FC-A-dependent effects and there are several issues with data quality that make it difficult to interpret the results. Importantly, the effects of the spastin inhibitor have a major impact on neurite outgrowth suggesting that cells simply cannot grow in the presence of the inhibitor and raising serious questions about any selectivity for FC-A - dependent growth. Aspects of the histology following spinal cord injury were not convincing.

      We sincerely appreciate the reviewer for evaluating our manuscript. Given the multitude of substrates that interact with 14-3-3, and considering spastin's indispensable role in neuroregeneration, it is indeed challenging to experimentally establish that FC-A's neuroregenerative effect is directly mediated through spastin in vivo. Therefore, we have provided additional crucial evidence regarding the changes in spastin protein levels following spinal cord injury, as well as the application of FC-A after spinal cord injury. Furthermore, we have made relevant adjustments to the uploaded images to enhance the resolution of the presented figures, as detailed in the subsequent response.

      Reviewer #3 (Public Review):

      Summary: The current manuscript c laims that 14-3-3 interacts with spastin and that the 14-3-3/spastin interaction is important to regulate axon regeneration after spinal cord injury.

      Strengths:

      In its present form, this reviewer identified no clear strengths for this manuscript.

      Weaknesses:

      In general, most of the figures lack sufficient quality to allow analyses and support the author's claims (detailed below). The legends also fail to provide enough information on the figures which makes it hard to interpret some of them. Most of the quantifications were done based on pseudo-replication. The number of independent experiments (that should be defined as n) is not shown. The overall quality of the written text is also low and typos are too many to list. The original nature of the spinal cord injury-related experiments is unclear as the role of 14-3-3 (and spastin) in axon regeneration has been extensively explored in the past.

      We sincerely appreciate the careful consideration and rigorous evaluation provided by the reviewer. In the revised version, we have made effort to present high-resolution figures and provide more detailed figure legends. Furthermore, we have made relevant adjustments to the statistical methods in accordance with the reviewer's suggestions. The manuscript has also undergone a thorough review and correction process to eliminate any writing-related errors. Please refer to the following response.

      To the best of our knowledge, there has been no clear reports on the efficacy of 14-3-3 in the repair of spinal cord injury. Kaplan A et al. (doi: 10.1016/j.neuron.2017.02.018) reported a reduction in die-back of the corticospinal tract following spinal cord injury using FC-A as a filler in situ in the lesion site. However, the specific effects of FC-A on spinal cord injury, such as motor function and neural reactivity, as well as the expression characteristic of 14-3-3 after spinal cord injury, have not been extensively elucidated. Additionally, prior research on spastin's role in axon regeneration primarily focused on the effects in Drosophila, and its regenerative effects in the central nervous system of adult mammals after injury have not been reported. Therefore, our study provides crucial insights into the importance of 14-3-3 and spastin in the process of spinal cord injury repair in mammals.

      Reviewer #1 (Recommendations For The Authors):

      There are many spelling and grammar errors, please revise. Examples:

      -approach revealed14-3-3

      -We have detected different many 14-3-3 peptides

      -Line 1057 (D) 14-3-3 agnoist FC-A

      -There is a discrepancy between panel names and figure legend in Figure 4.

      -There is another discrepancy between the color coding of treatments in Figure 7. All panels show "injury" in red and FC-A in orange, but in panel E, these are swapped. This is confusing to readers.

      Thank you for the thorough and rigorous review. We have re-colored the relevant chart. The manuscript has also undergone a thorough review to eliminate any writing-related errors.

      Most images from confocal microscopy are blurred or low resolution. They should be sharper for the type of microscopy used.

      We have adjusted and re-uploaded the images with higher resolution. Additionally, we have enlarged the relevant images.

      The list of all peptides retrieved in the Mass-Spec analyses of the GST-spastin pulldown must be publicly available, according to eLife rules.

      Thank you for your suggestion. We have now uploaded the mass spectrometry data.

      To determine where the 14-3-3/spastin protein142 complex functions in neurons, we double stained hippocampal neurons with spastin143 and 14-3-3 antibody, and found that 14-3-3 was colocalized with spastin in the entire144 cell compartment (Figure 1C).

      Colocalization by confocal fluorescence microscopy is not evidence for protein complexes.

      While co-localization experiments may not directly demonstrate protein-protein interactions, they can still provide valuable insights into the cellular localization of the proteins and suggest potential interactions between them. Therefore, we adjusted the statement.

      Fig1F- Co-immunoprecipitation assay results confirmed that all 14-3-3 isoforms could form direct complexes with spastin.

      CoIP in cells overexpressing the proteins is not evidence that it is direct. That they can interact directly with each other can be extracted from the evidence in vitro with purified proteins.

      We agree with this and we have changed our statement accordingly.

      For a broad audience to have a better understanding, the authors have to explain their a.a. subtitucions of Serine233, one being mimicking phosphorylation (S233D) and the other rendering the protein not being able to be phosphorylated in that position (S233A).

      We appreciate the suggestion. We have provided a more detailed explanation in revised manuscript.

      The panel of neuronas in Fig2G is mislabeled, because it is twice spastin S233A, instead of S233D.

      We apologize for this mistake and we have corrected it in the panel.

      FCA may increase the interaction of 14-3-3 with any of its substrates, including spastin. One would appreciate evidence that FCA increases the MT-severing activity of spastin, as assumed by authors

      We appreciate the reviewer’s suggestion. In this study, we overexpressed spastin to investigate its microtubule severing activity. It is important to note that overexpressing spastin significantly exceeds the normal physiological concentration of the protein. Using excessive amounts of FC-A to enhance the interaction between 14-3-3 and spastin in cells can lead to cell toxicity. Therefore, we chose to overexpress 14-3-3 instead of employing excessive FC-A.

      In Fig2F, the interaction of 14-3-3 with Spas-S233D would have been very informative.

      Thank you for the constructive suggestions from the reviewer. We have supplemented the corresponding co-immunoprecipitation experiments (Fig.).

      The functional effect of S233A and S233D does not correlate with a function of 14-3-3 in neurite outgrowth. This is because S233A does not interact with 14-3-3, however, it is as good as WT spastin... meaning that binding of 14-3-3 with spastin is not necessary...

      We appreciate the reviewer's consideration. The observed phenomenon of spastin WT and S233A promoting axon growth do not align with the physiological state within neurons. This may mask the true effects of S233A or S233D on neuronal axon growth. It is documented that the proper dosage of spastin is essential for neuronal growth and regeneration, as excessive or insufficient amounts can hinder axon growth. Excessive spastin levels can disrupt the overall cellular MTs. Therefore, spastin were moderately expressed by adjusting the transfection dosage and duration. Nevertheless, we were unable to precisely control the expression levels of spastin for both WT and S233A, also resulting in an overexpression state compared to the physiological state. As a result, the crucial role of spastin S233 in neural growth under physiological conditions may be masked. We have addressed this issue in the revised version of our manuscript.

      In panels 3C and D it is not clear if it does contain 14-3-3.... it seems it does not... but clarify.

      We apologize for any confusion. Since there is endogenous 14-3-3 present in the cells, we utilized spastin S233A and S233D to mimic the binding pattern with 14-3-3 according to the established interaction model. This information has been clarified in the original manuscript.

      Line 217 should indicate Figure 3, not Figure 5

      We have made the corresponding corrections.

      In F3G, it is intriguing that the input blot shows a decrease in Ubiquitin proteins when there is expression of flag ubiquitin...

      We apologize for the error in our presentation. In the control group, we actually overexpressed Flag-ubiquitin and GFP instead of Flag and GFP-spastin. Additionally, to further elucidate the impact of different phosphorylation states on spastin ubiquitination and degradation, we have conducted additional ubiquitination experiments (Fig.3N), which are now included in the revised version of our manuscript.

      S233 mutations seem to affect the effective turnover of spastin, but does not seem to change the levels of the spastin protein...hence, the conclusion that 14-3-3 protects from degradation is overstated.

      We thank the reviewers for the careful review and we have revised the statement accordingly.

      The mode of action of R18 FCA should be introduced earlier in the text.

      Thank you for the reviewer's correction. We have provided a corresponding description of the effects of FC-A and R18 on the interaction between 14-3-3 and spastin in the ubiquitination experiments section of the manuscript.

      Line 296 reads: Our results revealed that levels of 14-3-3 protein remained high even at 30 DPI, indicating that 14-3-3 plays an important role in the recovery of spinal cord injury.

      This is overstated since it can well be that an upregulated protein is inhibitory. We thank the reviewers for their consideration and we have made adjustments accordingly.

      It is not clear if 14-3-3 prevents ubiquitination of spastin, then its levels should be higher... it is noteworthy that they did not measure its levels in nerve tissue after injury. For example, in experiments shown in Figure 5A, it would have been very useful the observation of the levels of spastin.

      We appreciate the reviewer's consideration. We have now included the assessment of spastin protein levels following spinal cord injury. Additionally, we have collected the injured spinal cord lysates in mice treated with FC-A for western blot analysis. The results revealed that the expression trend of 14-3-3 protein is largely consistent with spastin after spinal cord injury. Furthermore, the treatment with FC-A was found to enhance the expression of spastin after spinal cord injury (Fig. 5C&D)."

      Panel 5G reads "nerve regeneration across the lesion site", but it actually measured NF levels, according to the legend.

      Thanks to the reviewers for the critical review. We have revised the chart accordingly.

      361 "BMS" should be explained in the results section for a better understanding of the results by non-experts.

      Thank you to the reviewers for their suggestions. We have explained this in the results section accordingly.

      Reviewer #2 (Recommendations For The Authors):

      1. The results of the mass spec and co-IP in Figure 1 are unclear.

      a) Are all of the peptides in Fig. 1A from 14-3-3 and were there only 3 14-3-3 peptides that were identified?

      The mass spectrum results did identify only three 14-3-3 peptides, and these three peptides were highly conserved across all isoforms.

      b) The blot in panel B needs to show the input band for spastin and 14-3-3 from the same gel and not spliced so that the level of enrichment can be evaluated in the co-IP.

      Thanks to the reviewer's comments, we have presented the whole gel (Fig.1B)

      c) Further, does an IP for 14-3-3 co-precipitate spastin?

      Thank you for your concern. We appreciate your feedback. Our 14-3-3 antibody is capable of Western blot experiments and recognizes all subtypes (Pan 14-3-3, Cell Signaling Technology, Cat #8312). Unfortunately, it is not suitable for immunoprecipitation (IP) experiments. Therefore, we have employed additional approaches, namely immunoprecipitation and pull-down assays, to further investigate the interaction between 14-3-3 and spastin.

      1. It is difficult to say anything about 14-3-3 - spastin co-localization in hippocampal neurons (1c) since 14-3-3 labels the entire hippocampal neuron so any protein will co-localize.

      We appreciate the comments. The co-localization experiments have provided evidence of the relative expression of both 14-3-3 and spastin in neurons, suggesting their potential interaction within neuronal cells. We have made the necessary revisions to accurately describe the results of the co-localization experiments in the manuscript.

      To further investigate the interaction between 14-3-3 and spastin within neurons, we have conducted additional co-immunoprecipitation (Co-IP) experiments using cortical neuron lysates (Fig.1C).

      1. The molecular weight of 14-3-3 is 25-28 kDa but the band in panel 1B and in subsequent figures it is below 15 kDa. Fig. 1F - the spastin band also seems to be low compared to predicted molecular weight and other W. Blot reports in the literature so some indication of how the antibody was validated would be important.

      Apologies for the mistakes. We have carefully re-evaluated the western blot images (See Author response image 1). We have confirmed that the molecular weight of the 14-3-3 protein is approximately 33 kDa. In the case of spastin, its molecular weight is around 55-70 kDa. Additionally, the GFP-spastin fusion protein has an estimated molecular weight of approximately 90 kDa. We have conducted a thorough verification and made appropriate adjustments to the molecular weight labels in all western blot images.

      Author response image 1.

      1. Fig 1G is a co-immunoprecipitation and it is not clear what the authors mean by "direct complexes" as claimed in line 150 of the results since this does not show direct binding between 14-3-3 and spastin. None of the assays in Fig. 1 assess "direct" binding between the two proteins and the authors should be clear in their interpretation.

      We agree with the reviewer's comments and have removed the word "direct" from the text.

      1. Fig. 1D - there is no validation that staurosporine (protein kinase inhibitor, not protein kinase as per typo in Line 167) affects the phosphorylation levels of spastin.

      Thank you for your valuable comments. In our group, we have conducted another study that has confirmed the involvement of CAMKII in mediating spastin phosphorylation. Furthermore, we have found that the addition of staurosporine significantly reduces the phosphorylation levels of spastin (unpublished results). In response to the reviewer's comment, we are pleased to provide western blot experiments demonstrating the effect of staurosporine on reducing spastin phosphorylation. The phosphorylation levels of spastin were assessed using a Pan Phospho antibody (Fig.2D).

      1. Fig. 2F - it would be important to test if spastin S233D interacts more robustly with 14-3-3 and if this is insensitive to staurosporine.

      Thank you for your comments. The suggestion provided by the reviewer is highly significant for supporting our conclusion that "phosphorylation of spastin is a prerequisite for its interaction with 14-3-3." Therefore, we have conducted additional immunoprecipitation experiments to further supplement our findings (Fig.2H). The experimental results demonstrate that the binding affinity between spastin S233D and 14-3-3 is stronger compared to spastin WT.

      1. Line 179 "Next, we transfected Ser233 mutation of spastin (spastin S233A or spastin S233D) with flag tagged 14-3-3 and generated Pearson's correlation coefficients. Results revealed that spastin 181 S233D was markedly colocalized with 14-3-3, with minimal colocalization with spastin S233A (Figure 2A-B)." Assuming the authors are referring to supplemental Figure 2, the 14-3-3 covers the entire cell thus I think measures of co-localization are uninterpretable.

      We agree with the reviewer's comment. We realize that 14-3-3θ exhibits a ubiquitous cellular distribution, which renders the measurement of its co-localization coefficients inconclusive. Therefore, we have decided to remove Supplementary Figure 2 from the manuscript.

      1. Line 189 "Consistent with earlier results, spastin promoted neurite outgrowth, as evidenced by both the length and total branches of neurite." - It is unclear what earlier results the authors are referring to. The authors should clarify how they determined the "moderate" expression level.

      We thank the review’s suggestions. The "earlier results" mentioned here refers to previously published articles, we now have added relevant references. Existing literature indicates that an appropriate dosage of spastin is necessary for neuronal growth and regeneration. However, both excessive and insufficient amounts of spastin are detrimental to axonal growth. Excessive spastin disrupts the overall microtubule network within cells. We controlled plasmid transfection dosage and transfection durations to achieve moderate expression. We have provided an explanation of these details in the revised version.

      1. The effects of WT spastin and spastin S233A were similar in spite of the fact that S233A does not bind to 14-3-3, which is inconsistent with the author's model that spastin-14-3-3 binding promotes growth. Line 191 - the authors mention that spastin S233D was toxic but I do not see any cell death measurements. I assume the bottom right panel in Fig. 2G labelled as spastin S233A is mislabeled and should be S233D.

      In response to comment 8, the transfection of both wild-type (WT) spastin and S233A mutant failed to precisely control the expression levels around the physiological concentration. Consequently, we observed an overexpression of spastin in both cases, which obscured the critical role of S233 phosphorylation in neurite outgrowth. We have addressed this issue in the revised version of the manuscript.

      1. Fig. 3. Does spastin(S233D) bind constitutively to 14-3-3? Why is spastin S233A not less stable than WT spastin based on the author's model?

      We propose that 14-3-3 is more likely to interact with spastin S233D in a non-constitutive manner. The instability of the S233A protein is attributed to the disruption of its ubiquitination degradation process due to the absence of 14-3-3 binding.

      1. The ubiquitin blot in Fig. 3G is not convincing and not quantified.

      We acknowledge the mislabeling in our figures. In the control group, Flag-Ubiquitin was also overexpressed, and we transfected GFP as a control instead of GFP-spastin. To further enhance the reliability, we conducted additional ubiquitination experiments (Fig.3N), which revealed a significant increase in spastin (S233A) ubiquitination levels compared to the WT group, consistent with previous research findings (Spastin recovery in hereditary spastic paraplegia by preventing neddylation-dependent degradation, doi:10.26508/lsa.202000799). Additionally, we observed that the addition of R18 could partially enhance spastin ubiquitination levels, as quantitatively illustrated in the figure (Fig.3O). This result further underscores the inhibitory role of 14-3-3 in the ubiquitination degradation pathway of spastin.

      1. I do not understand how the glutamate injury fits with the narrative (Fig. 4C).

      Excessive glutamate exposure can induce severe intracellular oxidative stress reactions, leading to the disruption of physiological processes such as mitochondrial energy production. This, in turn, results in the swelling and lysis of neuronal processes, a phenomenon known as neuronal necrosis. During this state, neurite maintenance is obstructed, and neurites exhibit swelling and breakage (Glutamate-induced neuronal death: a succession of necrosis or apoptosis depending on mitochondrial function. Neuron. 1995 Oct;15(4):961-73). We have provided a more comprehensive explanation of this phenomenon in the revised version of our manuscript.

      1. Some commentary about the selectivity of spastazoline to inhibit spastin should be included - it would be helpful if the authors could explain that this is a spastin inhibitor in the manuscript. FC-A still seems to promote growth in the presence of spastazoline suggesting that the FC-A effects are not dependent on spastin (Fig. 4E). The statistical analysis section of the materials and methods indicates that multiple groups were analyzed by one-way ANOVA. This seems unusual since the controls for cellular transfection are different than for small molecules (FC-A) and for peptides such as R18. As such, there is no vehicle control for the FC-A condition and it is difficult to assess the FC-A vs Spastazoline vs FA-A + Spastoazoline. The authors should clarify (Fig. 4E-J)

      Thank you for the reviewer’s suggestions. In the revised version, we have provided a more detailed explanation of the specific inhibition of spastin's severing function by spastazoline.

      We observed that FC-A, in combination with spastazoline, still exhibited a certain degree of promotion in neurite growth compared to the injury group under the glutamate circumstances. Evidently, spastin is not the exclusive substrate for 14-3-3, and FC-A might delay cellular oxidative stress reactions by facilitating the interaction of 14-3-3 with other substrates, such as the FOXO transcription factors as mentioned in the introduction. Nevertheless, our results still demonstrate that the addition of spastazoline significantly diminishes the promoting effect of FC-A on neurite growth, indicating that FC-A affects neuronal growth by impacting spastin.

      Furthermore, in the drug-treated groups, we overexpressed GFP to trace the morphology of neurons. Culture media were exchanged following transfection, and during media exchange, drugs were added. And an equivalent amount of DMSO or ethanol were added as controls to rule out the influence of solvents on neurons.

      1. There is a good possibility that spastin is required for all axon regeneration and that there is no selectivity for the FC-A pathway and this is a major issue with the interpretation of the manuscript (Fig 4K-L).

      We acknowledge this point. Clearly, spastin is not the exclusive substrate for 14-3-3, and our experimental evidence does not establish that 14-3-3 solely promotes neuronal regeneration through spastin. Nevertheless, we have identified the significance of 14-3-3 and spastin in the process of neural regeneration. Furthermore, we conducted complementary experiments to support the stability of spastin by FC-A treatment both in vitro and in vivo. We found an enhanced protein expression in cortical neurons after FC-A treatment (Fig.4M). Also, the results indicate a consistent elevation trend in the protein levels of spastin and 14-3-3 following spinal cord injury (Fig.5C&H). Moreover, in the FC-A group of mice, there was a significant increase in spastin protein levels (Fig.5D&I). These results also support that 14-3-3 promotes spinal cord injury repair by enhancing spastin protein stability.

      1. Fig. 5C- it is unclear where the photomicrographs were taken relative to the lesion.

      We obtained tissue sections from the lesion core and the above segments for histological analysis. Given the scarcity of neural compartment at the injury center, we select tissue slices as close as possible to lesion core to illustrate the relationship between 14-3-3 and the injured neurons. We have provided an explanation of this in the revised version of the manuscript.

      1. The authors need to provide some evidence that the FC-A and spastazoline compounds are accessing the CNS following IP injection.

      We thank the review’s suggestion. Although direct visualization evidence of FC-A and spastazoline entering the CNS is challenging to obtain, several indicators suggest drug penetration into spinal cord tissue. Firstly, behavioral and electrophysiological experiments in vivo demonstrate that drug injections indeed affect the neural activity of mice. Secondly, following spinal cord injury, the blood-spinal cord barrier was disrupted at the injury site, combined with the fact that both FC-A (molecular weight: 680.82 Da) and spastazoline (molecular weight: 382.51 Da) are small molecule drugs, these increases the likelihood of these small molecules entering the injured spinal cord tissue. Furthermore, our microtubule staining results indicated that FC-A and spastazoline did influence the acetylation ratio of microtubules. These findings support the drug penetration into spinal cord tissue.

      1. Some quantification of Fig. 5D would be important to support the contention that the lesion site is impacted by FC-A treatment.

      Thank you for the suggestion. We have included quantitative analysis for Figure 5D (Figure) as recommended.

      1. The NF and 5-HT staining in Fig. 5D and in Fig. 7A and B does not clearly define fibers and is not convincing.

      We appreciate the concerns. While we did not present whole nerve fibers, we therefore employed NF and 5-HT immunoreactive fluorescence intensity as an indicator to assess the regeneration of nerve fibers as previously described, but not axons per square millimeter (Baltan S, et, al. J Neurosci. 2011 Mar 16;31(11):3990-9; Iwai M, et, al. Stroke. 2010 May;41(5):1032-7; Wang Y, et, al. Elife. 2018 Sep 12;7:e39016; Altmann C, et, al. Mol Neurodegeneration. 2016 Oct 22;11(1):69).

      Our results showed that in the spinal cord injury group, there was strongly decreased NF-positive stainning (with a slight increase in 5-HT). In contrast, the FC-A treatment group exhibited a significant higher abundance of NF-positive signals (or an increased 5-HT signal) in the lesion site, which also suggests the reparative effect of FC-A on nerves. We also intend to refine our immunohistochemical methods in future experiments.

      Minor Comments: 1. Line 80 -84. To my knowledge the only manuscripts examining the effects of spastin in axon regeneration models includes the analysis in drosophila (i.e. ref 15 and 16) and a study in sciatic nerve that reported an index of functional recovery but did not perform any histology to assess axon regeneration phenotypes. The literature should be more accurately reflected in the introduction.

      We appreciate the suggestions from the reviewer. In the revised version, we have provided further clarification on the novelty of spastin in the spinal cord injury repair process.

      1. Line 73: The meaning of the following statement needs to be clarified: "spastin has two major isoforms, namely M1 and M87, coded form different initial sites."

      We have provided additional elaboration for this statement in the revised version.

      1. Line 216: Results indicated that GFP-spastin could be ubiquitinated, while inhibiting the 217 binding of 14-3-3/spastin promoted spastin ubiquitination (Figure 5G)." - Should be Fig 3G

      Sorry about the mistake. We have made the corresponding changes in the revised version.

      1. Line 255: "Briefly, we established a neural injury model as previously described(31)" - the basics of the injury model need to be described in this manuscript.

      In the revised version, we have provided further elaboration on the glutamate-induced neuronal injury model.

      Reviewer #3 (Recommendations For The Authors):

      Figure 1: A- Both legend and text fail to provide detail on this specific panel.

      We have provided a more detailed and comprehensive description of the legend and results in this section.

      B- Is the contribution of non-neuronal cells for co-IPs relevant? Co-IP with isolated neuronal extracts (instead of spinal cord tissue) should be performed.

      We thank the review’s suggestion. To further elucidate their interaction within neurons, cortical neurons were cultured (Cultured in Neurobasal medium supplemented with 2%B27 and cytarabine was used to inhibit glial cell growth) and cells were lysed for co-IP experiments (Fig.1C), and the results demonstrated the interaction between 14-3-3 and spastin within neurons.

      C- Both spastin and 14-3-3 appear to label the entire neuron with similar intensities throughout the entire cell which is rather unusual. Conditions of immunofluorescence should be improved and z-projections should be provided to support co-localization.

      Thanks for the comment. Our dual-labeling experiments indicated that 14-3-3 exhibits a characteristic pattern of whole-cell distribution. Therefore, this result cannot confirm the interaction between 14-3-3 and spastin within neurons, but it does provide evidence regarding the intracellular distribution patterns of 14-3-3 and spastin. Consequently, we supplemented neuronal endogenous co-IP experiments to further demonstrate the direct interaction between 14-3-3 and spastin within neurons, and we have modified the wording in the revised version accordingly.

      D- xx and yy axis information is either lacking or incomplete.

      We have made the corrections to the figures.

      E- It would be useful to show the conservation between the different 14-3-3 isoforms.

      We appreciate the suggestions. We have included a conservation analysis of 14-3-3 to assist readers in better understanding these results (Fig.1F).

      Figure 2:

      D- The experiment using a general protein kinase inhibitor does not allow concluding that the specific phosphorylation of spastin is sufficient for binding to 14-3-3. An alternative phosphorylated protein might be involved in the process.

      We appreciate the reviewer's consideration. We believe this serves as a prerequisite condition to demonstrate that "14-3-3 binding to spastin requires spastin phosphorylation." In fact, another project in our group has confirmed that CAMK II can mediate spastin phosphorylation, and the addition of staurosporine significantly reduces spastin phosphorylation levels (unpublished results). Here, we provide the western blot experiment showing the decrease in spastin phosphorylation under staurosporine treatment, with phosphorylation levels detected using the Pan Phospho antibody (Fig.2D).

      H and I- Pseudo-replication. Only independent experiments should be plotted and not data on multiple cells obtained in the same experiment. Please indicate the number of independent experiments.

      We appreciate the reviewer's correction. We now have included the mean value of three independent experiments and we have made relevant revisions to the statistical charts.

      Figure 3:

      The rationale for the hypothesis that spastin S233D transfection might upregulate the expression of spastin relative to WT and spastin S233A is unclear.

      We appreciate the reviewer's consideration. We have supplemented the relevant results, as depicted in the Fig.3G, which demonstrates that 14-3-3 can enhance the protein levels of spastin, and phosphorylated spastin (S233D) exhibits a significantly increased protein level compared to wild-type spastin. These findings indicate that 14-3-3 not only inhibits the degradation of spastin but also increases its protein levels.

      I- pseudo-replication. Please plot and do statistical analysis of independent experiments.

      Thank you for the reviewer's corrections. We have made the necessary revisions.

      Figure 4: E-J: I- pseudo-replication. Please plot and do statistical analysis of independent experiments.

      Thank you for the reviewer's corrections. We have made the necessary revisions.

      Figure 5:

      B- Please show individual data points.

      Thank you for the reviewer's corrections. We have made the necessary revisions.

      D- Longitudinal images of spinal cords where spastazoline was used cannot correspond to contusion as there is a very sharp discontinuity between the rostral and caudal spinal cord tissue. A full transection seems to have occurred. Alternatively, technical problems with tissue collection/preservation might have occurred.

      Thank you for the reviewer's consideration. The sharp discontinuity observed in the spastazoline group is not due to modeling issues but rather a result of the drug's effects on the injury site. This is primarily because spastin plays a crucial role not only in neuronal development but also in mitosis. Since the highly active proliferation of stromal cells at the injury site, . spastazoline may inhibit the proliferation of injury site-related stormal cells, thereby impeding the wound healing process following spinal cord injury, resulting in the observed discontinuous injury gap. We have made the corresponding revision accordingly.

      E- Images do not have the quality to allow analysis. 5HT staining should not be considered as a clear axonal labeling is not seen. This is also the case for neurofilament staining.

      We appreciate the concerns. While we did not present whole nerve fibers, we therefore employed NF and 5-HT immunoreactive fluorescence intensity as an indicator to assess the regeneration of nerve fibers as previously described, but not axons per square millimeter (Baltan S, et, al. J Neurosci. 2011 Mar 16;31(11):3990-9; Iwai M, et, al. Stroke. 2010 May;41(5):1032-7; Wang Y, et, al. Elife. 2018 Sep 12;7:e39016; Altmann C, et, al. Mol Neurodegeneration. 2016 Oct 22;11(1):69).

      Our results showed that in the spinal cord injury group, there was strongly decreased NF-positive stainning (with a slight increase in 5-HT). In contrast, our FC-A treatment group exhibited a significant higher abundance of NF-positive signals (or an increased 5-HT signal) in the lesion site, which also suggests the reparative effect of FC-A on nerves. We also intend to refine our immunohistochemical methods in future experiments.

      F- Images do not allow analysis. Higher magnifications are needed.

      Thank you for the reviewer's consideration. We have now included higher-magnification images (Fig.5M) to address this concern.

      Figure 7:

      Same issues as in Figure 5.

      A- Images do not have the quality to allow analysis. 5HT staining should not be considered as a clear axonal labeling is not seen.

      B- Images do not have the quality to allow analysis. Neurofilament staining should not be considered as clear axonal labeling is not seen. MBP staining does not have a pattern consistent with myelin staining

      We appreciate the concerns. While we did not present whole nerve fibers, we therefore employed NF and 5-HT immunoreactive fluorescence intensity as an indicator to assess the regeneration of nerve fibers as previously described, but not axons per square millimeter (Baltan S, et, al. J Neurosci. 2011 Mar 16;31(11):3990-9; Iwai M, et, al. Stroke. 2010 May;41(5):1032-7; Wang Y, et, al. Elife. 2018 Sep 12;7:e39016; Altmann C, et, al. Mol Neurodegeneration. 2016 Oct 22;11(1):69). In this study, sagittal slices were used. MBP covers the axonal surface, indicating its co-localization with the axons. However, as we did not present intact nerve fibers, so we were unable to show the typical myelin staining of MBP.

    1. Author Response:

      The following is the authors' response to the original reviews.

      We were pleased with the overall enthusiastic comments of the reviewers:

      • Reviewer #1: “This manuscript by Mahlandt, et al. presents a significant advance in the manipulation of endothelial barriers with spatiotemporal precision”

      • Reviewer #2: “The immediate and repeatable responses of barrier integrity changes upon light-on and light-off switches are fascinating and impressive.”

      • Reviewer #3: “, these molecular tools will be of broad interest to cell biologists interested in this family of GTPases.”

      We thank the reviewers for their fair and constructive comments that helped us to improve the manuscript.

      Reviewer #1 (Recommendations For The Authors):

      1) This paper is likely to attract a diverse audience. However, the order of data presented in this manuscript can be confusing or challenging to follow for the naive reader. This is because the tool characterization is split into two parts: before the barrier strength assay (selection of optogenetic platform and tool expression) and after (characterization of cell morphology with global and local optogenetic stimulation). Reorganizing the results such that the barrier strength results follows from an understanding of individual cell responses to stimulation may improve the ability of this readership to understand the factors at play in the changes in barrier strength observed when opto-RhoGEFs are activated.

      We appreciate this idea, and we initially structured the paper in the proposed order and then decided, that we wanted to put more focus on the barrier strength results by already presenting them in the second figure. Therefore, we prefer to keep this order of figures.

      2) While the description of the selection of iLID as the study's optogenetic platform is clear, a better job could be done motivating the need for engineering new optogenetic tools for the control of GEF recruitment. Given that iLID-based tools for GEFs of RhoA, Rac1, and Cdc42 already exist, some of which are cited in the introduction, more information on why these tools were not used would be helpful-were these tools tested in endothelial cells and found lacking.

      The original system has the domain structure DHPH-tagRFP-SspB. But we wanted to work with a SspB-FP-GEF construct, which would allow easy exchange of the FP and the DHPH domain. This modular approach allowed us to generate and compare the mCherry, iRFP647 and HaloTag version. We don’t want to claim that we engineered an entirely new optogenetic tool but rather optimized an existing one with different tags. To make this more clear we added : ‘The membrane tag of the original iLID was changed to an optimized anchor. In addition, we modified the sequence of the domains to SspB, tag, GEF to simplify the exchange of GEF and genetically encoded tag. A set of plasmids with different fluorescent tags was created for more flexibility in co-imaging.’

      3) Comment on the reason behind using DHPH vs. DH domains for each GEF is needed.

      We have previously found (and this is supported by biochemical analysis of GEF activity) that the selected domains provide the best activity. We will add reference and the following to the text: ‘Their catalytic active DHPH domains were used for ITSN1 and TIAM1 (Reinhard et al., 2019).  In case of p63 the DH domain only was used, because the PH domain of p63 inhibits the GEF activity (Van Unen et al., 2015) (Fig. 1E).

      4) Since multiple Rho GTPases (e.g., RhoA, RhoB, RhoC) exist and Rho is used as the name of the GTPase family, please use RhoA where applicable for clarity.

      Since the RhoGEFp63 will activate RhoA/B/C we would rather not refer to RhoA only. We will clarify this in the text: ‘Three GEFs were selected, ITSN1, TIAM1 and RhoGEFp63, which are known to specifically activate respectively Cdc42, Rac and Rho and their isoforms.’

      5) A brief comment on the use of HeLa cells for protein engineering and characterization (versus the endothelial cells motivated in the introduction) may be helpful.

      We added the following to the text: ‘HeLa cells were used for the tool optimization because of easier handling and  higher transfection rate in comparison to endothelial cells.

      Minor suggestions:

      In figure 1C, line sections showing intensity profiles before and after protein dimerization might further emphasize the change in biosensor localization.

      We are not a fan of intensity profiles as the profile depends strongly on the position of the line and it basically turns a 2D image in 1D data, for a single image. So, we prefer to stick to the quantification as shown in panel 1B (which shows data from multiple cells).

      Reviewer #2 (Recommendations For The Authors):

      1)The study has analyzed the effects of light-induced activation of the three optogenetic constructs in endothelial cells on their barrier function (electrical resistance) at high cell density and correlated the findings with the cellular overlap-producing effects on endothelial cells cultured at sparse cell density. It should be tried to show these effects at a cell density where these light-induced effects increase electrical resistance. Lifeact with different chromophores in adjacent cells might be useful.

      We had attempted to measure the overlap in a monolayer by taking advantage of the Halotag and the variety of dyes available by staining one pool of cells red with JF 552 nm and the other far red with the JF 635 nm dye. However, the cells need at least 24 h to form a monolayer and by then they had exchanged the dye and red and far red pool could not be distinguished any longer.

      Therefore, we used the Lck-mTq2-iLID construct, which already marks the plasma membrane of the cells. We created a mosaic monolayer of cells expressing mScarlet-CaaX and cells expressing Lck-mTq2-iLID + SspB-HaloTag-TIAM(DHPH). We observed and increase in the overlap between cells under this condition. The results have been added to figure 4 - figure supplement 2I&J. To the text we added:

      'Additionally, cell-cell membrane overlap increased about 20 %, up on photo-activation of OptoTIAM, in a mosaic expression monolayer (figure 4 - figure supplement 2I,J, Animation 22)‘

      2) The authors correctly state that some reports have shown that S1P can increase endothelial barrier function in VE-cadherin independent ways and these are related to Rac and Cdc42. This was also shown for Tie-2 in vitro and even in vitro in the absence of VE-cadherin and should also be mentioned.

      We added the following to the text: ‘Not only S1P promotes endothelial barrier independent from VE-cadherin, also Tie2 can increase barrier resistance in the absence of VE-cadherin (Frye et al. 2015).

      Since a blocking antibody against VE-cadherin was used, a negative control antibody should be tested which also binds to endothelial cells.

      To visualize the cell-cell junctions in the experiment shown in Supplemental Fig 3.1, we added a non-blocking VE-cadherin antibody that is directly labeled with ALEXA 647 and shows normal junction morphology. These experiments already give an indication that the live labeling antibody of VE-cadherin does not disturb the junction morphology. However, when we added the blocking antibody against VE-cadherin, known to interfere with the trans-interactions of VE-cadherin, a rapid disruption of the junctions is observed.

      Additionally, previous work has shown, that VE-cadherin labeling antibody does not interfere with junction dynamics and function (see Figure 2.A, Kroon et al. 2014 ‘Real-time imaging of endothelial cell-cell junctions during neutrophil transmigration under physiological flow’, jove.). We have added the figures below, showing that addition of the control IgG and VE-cadherin 55-7H1 Abs at the timepoint where the dotted line is, did not interfere with the resistance whereas the blocking Ab drastically reduced resistance. We have added this reference to the results. ‘Previous work has shown the specific blocking effect of this antibody in comparison to the VE-cadherin (55-7H1) labeling antibody (Kroon et al., 2014).’

      Author response image 1.

      Reviewer #3 (Recommendations For The Authors):

      Additional comments for the authors:

      1) The introduction is very long and would benefit from a more concise emphasis on the information required to put the work and results in context and understand their importance.

      Comment: we appreciate the comment of the reviewer. However, we wish to introduce the topic and the tools thoroughly and therefore we chose to keep the introduction as it is.

      2) The N-terminal membrane-binding domain does not homogeneously translocate to the plasma membrane, since lck is a raft-associated kinase. Please comment on this.

      In our hands, the Lck is among the most selective and efficient tags for plasma membrane localization (https://doi.org/10.1101/160374). We do observe homogeneous translocation, but our resolution is limited to ~200 nm and so we cannot exclude that the Lck concentrates in structures smaller than 200 nm. Given the robust performance of the lck-based iLID anchor in the optogenetics experiments, we think that the Lck anchor is a good choice.

      3) Figure 1D is not very clear. What does 25 or 36% change mean? If iLID tg is conjugated to these sequences, its cytosolic localization should be reduced versus iLID alone. Is this what the graph wants to express? If so, please, label properly the ordinate axis in the graph (% of non-tagged iLID values?)

      The graph is representing the recruitment efficiency of SspB to the plasma membrane for the two different membrane tags, targeting iLID to the plasma membrane. The recruitment efficiency was measured by the depletion of SspB-mScarlet intensity in the cytosol, up on light activation, and represented as a change in percentage.

      We added the following to the title of the graph_: SspB recruitment efficiency for Plasma Membrane tagged iLID._

      4) Supplemental figures in the main text. Fig S1D in the text refers to data in Fig S1E and Fig S1E is supposed to be Fig S1F? (page 11).

      That is correct. The mistakes have been corrected (and this is now renamed to figure 1 - figure supplement 1E and 1F).

      5) Figure 3. Contribution of VE-cadherin. Other junctional complexes, such as tight junctions may also intervene. However, these results would also suggest that cell-substrate adhesion rather than cell-cell junctions may modulate the barrier properties, as it has been previously demonstrated for example by imatinib-mediated activation of Rac1 (Aman et al. Circulation 2012). The ECIS system used to measure TEER in the quantitative barrier function assays can modulate these measurements and discriminate between paracellular permeability (Rb) and cell-substrate adhesion (alpha). Please, provide whether the optogenetic modulation of these GTPases does indeed regulate Rb or alpha.

      The measured impedance is made up of two components: capacitance and resistance. At relatively high AC frequencies (> 32,000 Hz) more current capacitively couples directly through the plasma membranes. At relatively low frequencies (≤ 4000 Hz), the current flows in the solution channels under and between adjacent endothelial cells’ (https://www.biophysics.com/whatIsECIS.php).

      Therefore, the high frequency impedance is representing cell-substrate adhesion whereas the low frequency responds more strongly to changes in cell-cell junction connections.

      We only measured at 4000 Hz, representing the paracellular permeability. We chose a single frequency to maximize time resolution.

      We have added this extra comment to the legend of the figure: ‘(B) Resistance of a monolayer of BOECs stably expressing Lck-mTurquoise2-iLID, solely as a control (grey), and either SspB-HaloTag-TIAM1(DHPH)(purple)/ ITSN1(DHPH) (blue) or p63RhoGEF(DH) (green) measured with ECIS at 4000 Hz, representing paracellular permeability, every 10 s.

    1. Author response:

      The following is the authors’ response to the original reviews.

      We appreciate your comments and suggestions on our manuscript.

      In particular, we have measured the affinity between the middle tail domain of myosin-5a (Myo5a-MTD) and the actin-binding domain of melanophilin (Mlph-ABD) using microscale thermophoresis, and obtained the Kd of ~0.56 uM, which is similar to the Kd of the globular tail domain of myosin-5a (Myo5a-GTD) to the GTD-binding motif of melanophilin (Mlph-GTBM). Moreover, we have performed Western blot of the lysate of transfected cells, showing that the proteins of the dominant negative construct and the negative control were expressed at similar lever without noticeable degradation.

      We appreciate the editors’ and reviewers’ comment on how melanophilin might be regulated in binding to the exon-G of myosin-5 and to actin filaments. Phosphorylation of melanophilin by protein kinase A is one possible mechanism. We will investigate this issues in our future study.

      We also took this opportunity to correct several minor errors in the manuscript. Textual alterations can be viewed in the “tracked change” version of the manuscript. Below is the comments from the editors and the two reviewers together with our point-by-point responses.

      eLife assessment

      This study represents a useful description of a third interaction site between melanophilin and myosin-5a which is important in regulating the distribution of pigment granules in melanocytes. While much of the data forms a solid case for this interaction, the inclusion of important controls for the cellular studies and measurement of interaction affinities would have been helpful.

      Public Reviews:

      Reviewer #1 (Public Review):

      Interactions known to be important for melanosome transport include exon F and the globular tail domain (GTD) of MyoVa with Mlph. Motivated by a discrepancy between in vitro and cell culture results regarding necessary interactions for MyoVa to be recruited to the melanosome, the authors used a series of pull-down and pelleting assays experiments to identify an additional interaction that occurs between exon G of MyoVa and Mlph. This interaction is independent of and synergistic with the interaction of Mlph with exon F. However, the interaction of the actin-binding domain of Mlph can occur either with exon G or with the actin filament, but not both simultaneously. These data lead to a modified recruitment model where both exon F and exon G enhance the binding of Mlph to auto-inhibited MyoVa, and then via an unidentified switch (PKA?) the actin-binding domain of Mlph dissociates from MyoVa and interacts with the actin filament to enhance MyoVa processivity.

      The only weakness noted is that the authors could have had a more complete story if they pursued whether PKA phosphorylation/dephosphorylation of Mlph is indeed the switch for the actin-binding domain of Mlph to interact with exon G versus the actin filament.

      We thank Reviewer #1 for careful reading of the manuscript and appreciation of the study. We agree with the Reviewer that it is important to understand how the actin-binding domain of Mlph switch its interaction with the exon-G of Myo5a and actin filament. We would like to pursue this direction in our future research.

      Reviewer #2 (Public Review):

      The authors identify a third component in the interaction between myosin Va and melanophilin- an interaction between a 32-residue sequence encoded by exon-g in myosin Va and melanophilin's actin-binding domain. This interaction has implications for how melanosome motility may be regulated.

      While this work is largely well done and certainly publishable following needed revisions (e.g. some affinity measurements, necessary controls for the dominant negative experiments), I believe that additional work would be required to make a more compelling case. First, the study provides just one more piece to a well-developed story (the role of exon-F and the GTD in myosin Va: melanophilin (Mlph) interaction), much of which was published 20 years ago by several labs. Second, the study does not demonstrate a physiological significance for their findings other than that exon-G plays an auxiliary role in the binding of myosin Va to Mlph. For example, what dictates the choice between Mlph's actin binding domain (ABD) binding to actin or to exon-G. Is it a PTM or local actin concentration? It is unlikely to be alternative splicing as exon-G is present in all spliced isoforms of myosin Va. And what changes re melanosome dynamics in cells between these two alternatives? Similarly, the paper does not provide any in vitro evidence that binding to exon-G instead of actin effects the processivity of a Rab27a/Myosin Va/Mlph transport complex. For example, if the ABD sticks to exon-G instead of actin, does that block Mlph's ability to promote processivity through its interaction with the actin filament during transport? In summary, given that the authors did not directly test their model either in vitro or in cells, I do not think this story represent a significant conceptual advance.

      We thank Reviewer #2 for careful reading of the manuscript and the suggestions of improving the manuscript. As suggested by the reviewer, we have measured the affinity between the middle tail domain of Myo5a (Myo5a-MTD) and Mlph-ABD (Kd ~0.562 uM), which is similar to that between the globular tail domain of Myo5a (Myo5a-GTD) and the GTBM of Mlph. In addition, we have performed additional experiments showing the integrity and the expression level of the dominant negative constructs in the transfected cells.

      We believe more extensive experiments are required to address other questions raised by the reviewer. For example, what dictates the choice between Mlph's actin binding domain (ABD) binding to actin or to exon-G is an open question. As we proposed, phosphorylation by protein kinase A is only one possible mechanism. We would like to pursue them in our future research.

      Recommendations for the authors:

      The reviewing editor feels strongly that addressing some of the points raised by the reviewers would make this a more compelling manuscript. In particular, a measurement of the affinity of the relevant fragments from melanophilin and myosin-5a would indicate that the interaction might be physiologically relevant. Concerning the dominant negative experiments, the lack of effect of an expressed fragment could be that the expressed fragments were simply degraded or expressed at too low of a level to be competing. The reviewer gives guidelines on how to address this. Reviewer #2 made a point that it would be compelling if the effect of phosphorylation as suggested in the model was tested, but we all agree that this could well be the subject of a later study. In addition, the authors make a very interesting proposal for how protein kinase A could be involved in this regulation as has been suggested previously. Perhaps the use of phosphomimetic mutations could give some insight into this. Such experiments, if consistent with the proposed model would certainly raise the impact of this study. Finally, a very clear periodicity in hydrophobic amino acids is apparent in the interacting sequences of both Myo5 (yrisLykrMidLmeqLekqdktVrkLkkqLkvFakkIgeLevgqmen) and Mlph (tdeeLseMedrVamtAseVqqAeseIsdIesrIaaLra). This is strongly suggesting a leucine-zipper-like coiled coil, rather than an interaction mediated solely by charge. Recent softwares (and easily accessible too) like AlphaFold multimer might yield important structural insight into the binding configuration and might help rationalize the effect of the mutations herein.

      We thank the editors and the reviewers for their suggestions of improving the manuscript. We have performed the several essential experiments to address the concerns raised by the reviewers.

      (1) Regarding the affinity of the relevant fragments from melanophilin and myosin-5a. We have measured the affinity between Mlph-ABD and Myo5a-MTD using MST (Kd ~562 nM) (see revised Figure 3A).

      (2) Regarding the concerns on the dominant negative experiments. We have examined the molecular sizes and expression levels of  Mlph or Myo5a constructs by Western blots. First, we show that all constructs have correct molecular size in transfected cells (see revised Figure 6C and 7D), indicating that the inability of Myo5a or Mlph truncations to generate dilute-like phenotypes was not due to the intracellular degradation of the EGFP fusion protein. Second, by correcting for the percentage of transfected cells, we show that the overall expression levels of the wild-type construct and the mutants are roughly equal. Third, we categorized the expression levels into high and low, and calculated percentage of the DN phenotype in high and low expression levels. The results are consistent with the percentage of DN phenotype in total EGFP fusion protein cells.

      (3) Regarding the suggestion to investigate the effect of phosphorylation by protein kinase A on Mlph-ABD’s interaction with Myo5a and actin filament. We understand that it is important to elucidate the mechanism by which the actin-binding domain of Mlph switch its interaction with the exon-G of Myo5a and actin filament. However, as we proposed, phosphorylation by protein kinase A is one possible mechanism, and more extensive experiments are required to address this question. Therefore, we would like to pursue it in our future research.

      (4) Regarding the suggestion to predict the interaction between the exon-G of myosin-5a and Mlph-ABD using AlphaFold. We have used AlphaFold multimer to predict the Myo5a-MTD/Mlph-ABD interaction. Remarkably, the AlphaFold predicted that the binding of Myo5a-MTD with Mlph-ABD is mediated by an antiparallel coiled-coil formed by Myo5a (1430-1467) and Mlph (450-481), just as predicted by the editors. This prediction is also consistent with our finding that the exon-G of Myo5a interacts with Mlph-ABD. However, the predicted model cannot explain our mutagenesis results. We will pursue this point in the future research. Nevertheless, we are grateful to the editors for bringing this idea to our attention, because it will help us to design experiments to investigate the nature of Myo5a-exon-G/Mlph-ABD interaction.

      Reviewer #1 (Recommendations For The Authors):

      Specific minor comments

      Q1: In figs 6-7 an overlay between DAPI and EGFP would be helpful for the reader to see perinuclear distribution.

      As suggested, we have added the merged images of DAPI and EGFP in the revised Figure 6 and 7.

      Q2: The delta symbol in the pdf text was corrupted.

      The corrupted delta symbol has been fixed in the revised manuscript.

      Reviewer #2 (Recommendations For The Authors):

      Q1: Please explain in detail early in the text what exon-G is - length, position in the tail, and evidence that it is a coiled coil (CC). Of note, is it only long enough for about 4 heptad repeats? Has it been shown biochemically to form a CC? Is the CC irreversible? What would be the consequence of removing the exon-G CC on the ability of surrounding regions to bind Mlph (exon-F and the GTD)?

      We thank the reviewer for this suggestion. In the revision, we added a new paragraph (the first paragraph in the results section) and revised Figure 1A to introduce the middle tail domain and alternatively spliced exons of Myo5a.

      Exon-G is 32 amino acids in length, located at the C-terminal region of the middle tail domain, immediately before the globular tail domain. Exon-G region was predicted to form a short coiled-coil by using on-line tools (such as paircoil), and this prediction has not been tested biochemically. Moreover, we do not know whether the exon-G coiled-coil is reversible or not.

      We have not examined the effect of removing the whole exon-G on the interaction between the GTD and Mlph-GTBM. The exon-G (residues 1436-1467) and the GTD core (residues 1498-1877) are separated by a long loop of 31 residues. We therefore expect that the removing the exon-G will not affect the GTD/Mlph-GTBM interaction.

      Physically, exon-F is immediately followed by exon-G, and those two regions might interfere with each other. In our preliminary study, we found that removing the whole exon-G abolished the interaction between exon-F and Mlph-EFBD. On the other hand, removing the C-terminal half (residues 1454-1467) of exon-G had little effect the interaction between exon-F and Mlph-EFBD (see Figure 2C). In this work, we intentionally selected the later construct for functional analysis of the exon-G/Mlph-ABD interaction, because removing the C-terminal half of exon-G abolishes the interaction with Mlph-ABD, but does not affect the exon-F/Mlph-EFBD interaction.

      Q2: Figures 1-3. While the pulldown experiments demonstrating an interaction between Mlph-ABD residues 446-571 and Myo5a-MTD are a good start, one would like to see affinity measurements to gauge the likelihood that this interaction is physiologically relevant. The same goes for the pulldown experiments demonstrating an interaction between (i) the C-terminal half of exon-G (residues 1453-1467) and the Mlph-ABD, (ii) between residues 1411-1467 (a short peptide containing exon-F and exon-G) and the Mlph-ABD, and (iii) between residues 1436-1467 (a short peptide containing exon-G) and the Mlph-ABD. This would also apply to the pulldowns in 3C-3E where versions of the proteins with charge residue changes were tested.

      We agree the reviewer’s opinion that determination of the affinities between Mlph-ABD and Myo5a-MTD and their variants will be helpful in understanding the physiological relevance of Exon-G/Mlph-ABD interaction. However, the extensive experiments suggested by the reviewer require many high quality, purified proteins, which are not trivial.

      Nevertheless, we think it is important to know the affinity between Myo5a-MTD and Mlph-ABD (both wild-type), as this parameter can be used for the comparison of the three interactions between Myo5a and Mlph. Therefore, we have obtained the affinity between Myo5a-MTD and Mlph-ABD using microscale thermophoresis (MST). The dissociation constant (Kd) of Myo5a-MTD to Mlph-ABD is 0.562±0.169 uM, which is similar to that between Myo5a-GTD and Mlph-GTBM (~1 uM) (Geething & Spudich (2007) JBC 282:21518). Consistent with GST pulldown results, MST shows that deletion of C-terminal half of exon-G (1453-1467) greatly decreases the MST signals (see revised Figure 3A).

      Q3: While the domain negative (DN) approach to testing functional significance is OK, rescuing dilute/myosin Va null melanocytes with full-length myosin Va containing the various deletions would have been more convincing. Also, the authors must show (i) that the DN constructs are the correct size in transfected cells (i.e. are not degraded), and (ii) that they are expressed at roughly equal levels (either by doing Westerns and correcting for the percent of transfected cells, or by measuring total cellular fluorescence in transfected cells). Without this information, it remains possible that constructs not exhibiting a DN effect are simply degraded or poorly expressed. This applies to all the DN data in Figures 6 and 7.

      We agree with the reviewer that Myo5a null melanocytes is ideal for investigating exon G function. Unfortunately, we do not have Myo5a null melanocytes derived from dilute mice.

      To confirm the integrity of the overexpressed proteins in the transfected cells, we performed Western blot of those proteins, including  EGFP-Mlph-RBD (wild-type and two mutants) and Myo5a-Tail (wild-type and G mutant), in the lysate of the transfected cells. Western blots show that all those proteins have correct molecular masses, indicating no degradation of those overexpressed proteins (see revised Figure 6C and 7C). Moreover, by correcting for the percentage of transfected cells, we show that the overall expression levels in each transfected cell of the wild-type construct and the mutants are roughly equal. This information is included in the revised manuscript (Line 222-225; 237-241).

      Q4: The authors scored the DN phenotype as yes/no but it mostly likely varies depending on the degree of over-expression. Showing that the degree of melanosome centralization scales with the degree of overexpression, and that the correlation between expression level and phenotype varies depending on the construct would strengthen the results.

      We agree with the reviewer’s prediction that the degree of DN phenotype should depend on the of over-expression level. We analyzed the EGFP signals of transfected cells and found very few cells with medium expression level. Therefore, we simply categorized the expression levels into high and low, and calculated the DN phenotype in each categories as shown in the table below. These results are consistent with the expectation that the degree of DN phenotype depends on the over-expression level of the transfected constructs.

      Author response table 1.

      Percentage of the EGFP-expressing cells with perinuclear aggregation of melanosomes

      Q5: The conclusion from the data in Figure 8A- "the presence of both exon-F and exon-G is insufficient for binding to the Mlph occupied by Myo5a, but sufficient for binding to the unoccupied Mlph"- should be verified by also doing the experiment in myosin Va knockdown cells.

      We agree. Unfortunately, our RNAi knockdown of Myo5a in melanocytes by RNAi is not ideal and we do not have Myo5a knockout melanocytes. We will pursue this point in the future.

      Q6: Line 213 "three Mlph-binding regions, i.e., exon-F, exon-F, and GTD (Figure 7A)" has a typo.

      This typo has been corrected.

      Q7: The authors should provide high mag insets for the images in Figure 8.

      As suggested, we have revised Figure 8 by including high mag insets for the images.

    1. Author response:

      The following is the authors’ response to the original reviews.

      Reviewer #1:

      In this manuscript by Napoli et al, the authors study the intracellular function of Cytosolic S100A8/A9 a myeloid cell soluble protein that operates extracellularly as an alarmin, whose intracellular function is not well characterized. Here, the authors utilize state-of-the-art intravital microscopy to demonstrate that adhesion defects observed in cells lacking S100A8/A9 (Mrp14-/-) are not rescued by exogenous S100A8/A9, thus highlighting an intrinsic defect. Based on this result subsequent efforts were employed to characterize the nature of those adhesion defects.

      The authors thank reviewer #1 for his/her insightful comments and suggestions. Please find our point to point responses below.

      (1) Ex vivo characterization of the function of S100A8/A9 in adhesion, spreading, and calcium signaling requires at least one rescue experiment to support the direct role of these proteins in the biological processes under study.

      We thank the reviewer for this comment. We agree that rescue experiments would be helpful to confirm the direct role of intracellular S100A8/A9 in adhesion, spreading, and Ca2+ signaling. Although transfection of primary cells, especially neutrophils, poses challenges due to their short half-life, we now have undertaken additional in vitro rescue experiments. Specifically, we used extracellular S100A8/A9 and coated Ibidi flow chambers with E-selectin, ICAM-1 and CXCL1 alone or alongside S100A8/A9, and measured rolling and adhesion of blood neutrophils. Our data reveal that extracellular S100A8/A9 can induce increased adhesion in WT neutrophils but fails to rescue the adhesion defect in Mrp14-/- neutrophils (Author response image 1). This result corroborates our in vivo findings, emphasizing that the observed adhesion defect is due to the lack of intracellular S100A8/A9.

      Author response image 1.

      Extracellular S100A8/A9 does not rescue the adhesion defect in Mrp14/- neutrophils. Analysis of number of adherent leukocytes FOV-1 normalized to the WBC of WT and Mrp14-/- mice. Whole blood was harvested through a carotid artery catheter and perfused with a high precision pump at constant shear rate using flow cambers coated with either E-selectin, ICAM-1 and CXCL1 or E-selectin, ICMA-1, CXCL1 and S100A8/A9. [mean+SEM, n=5 mice per group, 12 (WT) and 14 (Mrp14-/-) flow chambers, 2way ANOVA, Sidak’s multiple comparison]. ns, not significant; *p≤0.05, **p≤0.01, ***p≤0.001.

      (2) There is room for improvement in the analysis of signaling pathways presented in Figures 3 H and I. Western blots and analyses are not convincing, in particular for p-Pax.

      We acknowledge the reviewer's concern regarding the clarity of the signaling pathway analysis, particularly the western blots for p-Paxillin. To address this, we have repeated the western blot experiments using murine neutrophils. Our new data confirm the defective paxillin phosphorylation upon CXCL1 stimulation and ICAM-1 binding in the absence of cytosolic S100A8/A9. We have now integrated these new findings with the original data and included the updated results in the manuscript (Figure 3I revised). These enhanced analyses provide a more robust and convincing demonstration of the signaling defects in Mrp14-/- neutrophils.

      (3) At least one western blot showing a knockdown of S100A8/A9 should be included towards the beginning of the result section.

      We appreciate the reviewer's suggestion to include a western blot demonstrating the knockout of S100A8/A9 early in the results section. In a recent publication by our group, we have already demonstrated the absence of S100A8/A9 at the protein level in Mrp14-/- neutrophils via western blotting ([1], please refer to Extended Data Fig. 1h). We agree that visual confirmation of the absence of S100A8/A9 protein is crucial for establishing the validity of our study.

      (4) The Ca2+ measurements at LFA-1 nanoclusters using the Mrp14-/- Lyz2xGCamP5 are interesting; It is understood that the authors are correcting calcium levels by normalizing by LFA-1 cluster areas and that seems fine to me. The issue is that the total calcium signal seems decreased in Mrp14-/- cells compared to WT cells (Fig. 4E)...why is totalCa2+ low? Please discuss.

      We thank the reviewer for this insightful comment. Indeed, our observations reveal reduced overall Ca2+ levels in Mrp14-/- neutrophils compared to WT neutrophils. Initially, we noticed a general decrease in Ca2+ intensity (Author response image 2A-B) and lifetime in Mrp14-/- neutrophils (Author response image 2C-D). Further analysis indicated that these differences in Ca2+ levels are localized specifically to the LFA-1 nanocluster sites. In contrast, the cytosolic Ca2+ levels outside of the LFA-1 nanocluster areas were comparable between Mrp14-/- and WT neutrophils (Figure 4H-J). This suggests that the reduced total Ca2+ levels observed in Mrp14-/- neutrophils are primarily due to the impaired Ca2+ supply at the LFA-1 nanocluster areas. Our data support the notion that cytosolic S100A8/A9 plays a crucial role in actively supplying Ca2+ to LFA-1 nanoclusters during neutrophil crawling. In the absence of S100A8/A9, the increase in overall Ca2+ levels (summing both inside and outside LFA-1 nanocluster areas) is minimal, further highlighting the specific role of S100A8/A9 in maintaining localized Ca2+ concentrations at these crucial sites.

      Author response image 2.

      Overall Ca2+ levels in WT and Mrp14-/- neutrophils (A) Representative confocal images of neutrophils from WT Lyz2xGCaMP5 and Mrp14-/- Lyz2xGCaMP5 mice, labeled with Lyz2 td Tomato marker. The images illustrate overall cytosolic Ca2+ levels during neutrophil crawling flow chambers coated with E-selectin, ICAM-1, and CXCL1 (scale bar=10μm). (B) Quantitative analysis of total cytosolic Ca2+ intensity in single cells from WT Lyz2xGCaMP5 and Mrp14-/- Lyz2xGCaMP5 neutrophils measured over three time intervals: min 0-1, 5-6 and 9-10 [mean+SEM, n=5 mice per group, 56 (WT) and 54 (Mrp14-/-) neutrophils, 2way ANOVA, Sidak’s multiple comparison]. (C) Representative traces and (D) single cell analysis of total Ca2+ lifetime over the first 5 minutes in WT Lyz2xGCaMP5 and Mrp14-/- Lyz2xGCaMP5 neutrophils crawling on Eselectin, ICAM-1, and CXCL1 coated flow chambers recorded with FLIM microscopy [mean+SEM, n=3 mice per group, 111 (WT) and 95 (Mrp14-/-) neutrophils, 2way ANOVA, Sidak’s multiple comparison]. ns, not significant; *p≤0.05, **p≤0.01, ***p≤0.001.

      (5) Even if the calcium level outside LFA-1 nanoclusters is not significant (Figure 4J), the data at min 9-10 in Figure 4J seems to be affected by a single event that may be an outlier. Additional data may be needed here.

      We appreciate the reviewer’s attention to this detail. To address the concern regarding a potential outlier in the Ca2+ level measurements at 9-10 minutes in Figure 4J, we rigorously tested the dataset using the GraphPad outlier calculator. The analysis revealed that no data point was statistically identified as an outlier. Given that the current dataset is robust and the statistical analysis confirms the integrity of the data, we believe that the results accurately reflect the biological variability observed in our experiments. Therefore, we have not added additional data points at this stage but remain open to discussing this further.

      (6) Finally, even though there is less calcium at LFA-1 clusters, that does not necessarily mean that "cytosolic S100A8/A9 plays an important role in Ca2+ "supply" at LFA-1 adhesion spots" as proposed. S100A8/A9 may play an indirect role in calcium availability. The analysis of the subcellular localization of S100A8/A9 at LFA-1 clusters together with calcium dynamics in stimulated WT cells would help support the authors' interpretation, which although possibly correct, seems speculative at this point.

      We thank the reviewer for this insightful comment and fully agree that additional evidence regarding the subcellular localization of S100A8/A9 would strengthen our conclusions. Although live cell imaging of intracellular S100A8/A9 was initially challenging due to technical limitations, we have now performed additional experiments to address this issue. We conducted end-point measurements where we allowed WT neutrophils to crawl on E-selectin, ICAM-1, and CXCL1 coated flow chambers for 10 minutes. Following this, we fixed and permeabilized the cells to stain intracellular S100A9, along with LFA-1 and a cell tracker for segmentation. Confocal microscopy and subsequent single-cell analysis revealed a significant enrichment of S100A8/A9 at LFA-1 positive nanocluster areas compared to the surrounding cytosol (Figure 4K and 4L, new). This finding supports our hypothesis that S100A8/A9 plays a direct role in the localized supply of Ca2+ at LFA-1 adhesion spots, thus facilitating efficient neutrophil crawling under shear stress. These new data have been included in the revised manuscript, providing stronger evidence for our proposed mechanism.

      Reviewer #2:

      Napoli et al. provide a compelling study showing the importance of cytosolic S100A8/9 in maintaining calcium levels at LFA-1 nanoclusters at the cell membrane, thus allowing the successful crawling and adherence of neutrophils under shear stress. The authors show that cytosolic S100A8/9 is responsible for retaining stable and high concentrations of calcium specifically at LFA-1 nanoclusters upon binding to ICAM-1, and imply that this process aids in facilitating actin polymerisation involved in cell shape and adherence. The authors show early on that S100A8/9 deficient neutrophils fail to extravasate successfully into the tissue, thus suggesting that targeting cytosolic S100A8/9 could be useful in settings of autoimmunity/acute inflammation where neutrophil-induced collateral damage is unwanted.

      The authors appreciate reviewer #2's insightful comments and suggestions. Below are our detailed responses:

      (1) Extravasation is shown to be a major defect of Mrp14-/- neutrophils, but the Giemsa staining in Figure 1H seems to be quite unspecific to me, as neutrophils were determined by nuclear shape and granularity. It would have perhaps been more clear to use immunofluorescence staining for neutrophils instead as seen in Supplementary Figure 1A (staining for Ly6G or other markers instead of S100A9).

      We acknowledge the reviewer's concern. However, Giemsa staining is a well-established method in hematology, histology, cytology, and bacteriology, widely recognized for its ability to distinguish leukocyte subsets based on nuclear shape and cytoplasmic characteristics. This method is extensively documented in the literature [2-5]. Its advantages are the easy morphological discrimination of leukocytes based on nuclear and cytoplasmic shape and conformation (Author response image 3).

      Author response image 3.

      Giemsa staining of extravasated leukocyte subsets. (A) Representative image of Giemsa-stained cremaster muscle tissue post-TNF stimulation. The image clearly differentiates leukocyte subsets (white arrow = neutrophils, yellow arrow = eosinophils, red arrow = monocytes). Scale bar = 50µm.

      (2) The representative image for Mrp14-/- neutrophils used in Figure 4K to demonstrate Ripley's K function seems to be very different from that shown above in Figures 4C and 4F.

      The reviewer correctly observed that the cell in Figure 4K is different from those in Figures 4C and 4F. This is intentional, as Figure 4K is meant to show a representative image that accurately reflects the overall results of the experiments. We assure the reviewer that all cells analyzed in Figures 4C and 4F were also included in the analysis for Figure 4K.

      (3) Although the authors have done well to draw a path linking cytosolic S100A8/9 to actin polymerisation and subsequently the arrest and adherence of neutrophils in vitro, the authors can be more explicit with the analysis - for example, is the F-actin co-localized with the LFA-1 nanoclusters? Does S100A8/9 localise to the membrane with LFA-1 upon stimulation? Lastly, I think it would have been very useful to close the loop on the extravasation observation with some in vitro evidence to show that neutrophils fail to extravasate under shear stress.

      We thank the reviewer for this comment and questions. 

      Concerning the co-localization of F-actin with LFA-1 nanoclusters and S100A8/9 localization: We appreciate the reviewer's interest in the co-localization between F-actin and LFA-1. Unfortunately, due to the limitations of our GCaMP5 mouse model (with neutrophils labeled with td-Tomato and eGFP for LyzM and Ca2+), we could only stain for either LFA-1 or F-actin at a time. However, in our F-actin movies, we observed that F-actin predominantly localizes at the rear of the cell, while LFA-1 is more uniformly distributed at the plasma membrane.

      Regarding S100A8/A9 localization, as mentioned in response to Reviewer 1's sixth point, we now conducted endpoint measurements. We stained neutrophils with cell tracker green CMFDA and LFA-1, allowed them to crawl on E-selectin, ICAM-1, and CXCL1-coated flow chambers, and then performed intracellular S100A9 staining after fixation and permeabilization. Our analysis shows higher S100A9 intensity at LFA-1 positive areas compared to LFA-1 negative areas (Figure 4K and 4L, new). This indicates that S100A8/A9 indeed concentrates Ca2+ at LFA-1 nanoclusters, supporting adhesion and post-arrest modification events under flow.

      Regarding the extravasation defect under shear stress: To address the reviewer's suggestion, we performed transwell migration assays under static conditions. Our results show no significant difference in transmigration between WT and Mrp14-/- neutrophils without flow, indicating that the extravasation defect in Mrp14-/- neutrophils is shear-dependent. This supports our hypothesis that S100A8/A9-mediated Ca2+ supply at LFA-1 nanoclusters is critical under flow conditions (Author response image 4).

      Author response image 4.

      Static Transmigration assay. (a) Transmigration of WT and Mrp14-/- neutrophils in static transwell assays (3um pore size, 45min migration time) showing spontaneously migration (PBS) or migration towards CXCL1. [mean+SEM, n=3 mice per group, 2way ANOVA, Sidak’s multiple comparison]. ns, not significant; *p≤0.05, **p≤0.01, ***p≤0.001.

      Additional References

      (1) Pruenster, M., et al., E-selectin-mediated rapid NLRP3 inflammasome activation regulates S100A8/S100A9 release from neutrophils via transient gasdermin D pore formation. Nature Immunology, 2023. 24(12): p. 2021-2031.

      (2) Kuwano, Y., et al., Rolling on E- or P-selectin induces the extended but not high-affinity conformation of LFA-1 in neutrophils. Blood, 2010. 116(4): p. 617-24.

      (3) Porse, B., Mouse Hematology – A Laboratory Manual. European Journal of Haematology, 2010. 84(6): p. 554-554.

      (4) Frommhold, D., et al., Protein C concentrate controls leukocyte recruitment during inflammation and improves survival during endotoxemia after efficient in vivo activation. Am J Pathol, 2011. 179(5): p. 2637-50.

      (5) Braach, N., et al., RAGE Controls Activation and Anti-Inflammatory Signalling of Protein C. PLOS ONE, 2014. 9(2): p. e89422.

    1. Author response:

      The following is the authors’ response to the original reviews.

      Reviewer #1 (Public review):

      In the presented study, the authors aim to explore the role of nociceptors in the fine particulate matter (FPM) mediated Asthma phenotype, using rodent models of allergic airway inflammation. This manuscript builds on previous studies and identify transcriptomic reprogramming and an increased sensitivity of the jugular nodose complex (JNC) neurons, one of the major sensory ganglia for the airways, on exposure to FPM along with Ova during the challenge phase. The authors then use OX-314 a selectively permeable form of lidocaine, and TRPV1 knockouts to demonstrate that nociceptor blocking can reduce airway inflammation in their experimental setup. The authors further identify the presence of Gfra3 on the JNC neurons, a receptor for the protein Artemin, and demonstrate their sensitivity to Artemin as a ligand. They further show that alveolar macrophages release Artemin on exposure to FPM.

      We thank the reviewer for their valuable comments, which have significantly enhanced the quality of our manuscript. A point-by-point rebuttal is provided below.

      Strength

      The study builds on results available from multiple previous work and presents important results which allow insights into the mixed phenotypes of Asthma seen clinically. In addition, by identifying the role of nociceptors, they identify potential therapeutic targets which bear high translational potential.

      Weakness

      While the results presented in the study are highly relevant, there is a need for further mechanistic dissection to allow better inferences. Currently certain results seem associative. Also, certain visualisations and experimental protocols presented in the manuscript need careful assessment and interpretation. While Asthma is a chronic disease, the presented results are particularly important to explore Asthma exacerbations in response to acute exposure to air pollutants. This is relevant in today's age of increasing air pollution and increasing global travel.

      Major

      The JNC is a major group of neurons responsible for receiving sensory inputs from the airways. However, the DRG also contains nociceptors and is known to receive afference from the upper airways. An explanation of why the study was restricted to the JNC would be important.

      We acknowledge that some afferents to the upper airways do arise from the DRG, specifically in the upper thoracic segments (T1–T5). We have added a statement in the text to note this subset of nociceptive and spinally mediated pathways. However, the preponderance of evidence indicates that the majority of airway and lung afferents (70–80%, sometimes up to 90%) originate from the jugular–nodose complex (JNC). Given this large imbalance—and because our study focuses on the mechanosensory, and chemosensory functions mediated primarily by the JNC—we restricted our analysis to this main vagal pathway. By contrast, DRG innervation, though functionally important for nociception and irritation-related reflexes, accounts for a smaller yet significant (~20–30%) fraction of the total afferent pool. The referenced tracing studies[1,2] support this distribution and are cited to clarify our rationale for emphasizing the JNC in our work.

      Similarly, the role of the Artemin in the study remains associative. The study results present that Artemin sensitize nociceptors to lead to an increased inflammatory response (Supplementary Figure 2), however, both upstream and downstream evidence for this inference needs to be dissected further. For instance, the evidence for the role of Artemin in the model comes from ex vivo experiments with alveolar macrophages, but not in the experimental model created. Blocking or activation experiments could be performed, along with investigating the change in the total number of nociceptors with Artemin exposure. Similarly, the downstream effects of the potential Artemin-mediated JNC stimulation should be explored in the context of this experimental setup. A detailed dissection of the mechanisms is important. Additionally, it is also important to discuss the hypothesis leading to the selection of Artemin as a target, which currently seems arbitrary.

      Our data show that exogenous i) OVA-FPM exposed AM secrete Artemin and that ii) recombinant Artemin can sensitize nociceptors, potentially heightening the inflammatory response. As suggested, we agree that more upstream and downstream evidence is needed for definitive mechanistic insight. In response, we have expanded our experiments to include intravital microscopy, which demonstrates impaired motility of alveolar macrophages and neutrophils in nociceptor-ablated mice, suggesting a bidirectional crosstalk between AMs and nociceptor neurons.  

      In future studies, we will perform blocking or activation studies to clarify Artemin’s in vivo effects and confirm its role in modulating airway nociceptors. We also recognize the importance of examining whether Artemin exposure alters the phenotype of these neurons and lung innervation density. As recommended, we plan targeted interventions (e.g., Artemin-neutralizing antibodies or overexpression strategies) to delineate the mechanisms by which Artemin-mediated nociceptor stimulation influences the local inflammatory environment.

      We have expanded our discussion to clarify that Artemin is a recognized growth factor known to sensitize certain sensory neurons, including those responsive to tissue injury and inflammation. This literature-based rationale guided our hypothesis that Artemin might increase nociceptor reactivity in the lung and thereby influence alveolar macrophage function. By combining ex vivo and intravital approaches, we have begun to map these interactions but agree that further in vivo studies are necessary to confirm causality, dissect signal transduction pathways, and fully validate Artemin’s contributions to AM–nociceptor crosstalk. We have revised our manuscript accordingly to highlight these limitations.

      A deeper exploration of the inflammatory parameters could be performed. The multiplex analysis of the cytokine analysis shows a reduction in certain cytokines like IL-6 and MCP (figure 3F), which needs to be discussed. Additionally, investigating the change in proportions of the different immune cell populations is important, which currently restricts the eosinophil and neutrophil counts in the BAL. This is also important as the study builds on work from Prof. Chang's group, which also identified the expansion of an invariant iNKT cell population by FPM, regulatory in nature. Adding data on airway hyperresponsiveness, if possible, would be a welcome addition, considering Asthma as the disease context.

      We thank the reviewer for highlighting the need for a more comprehensive exploration of inflammatory parameters. To address these concerns:

      (1) Cytokine Analysis: We re-ran all statistical analyses, including the CBA and ELISA assays, and confirmed that TNFα and Artemin are the only differentially expressed cytokines across experimental groups. We have expanded the Discussion to emphasize TNFα’s role in this context.

      (2) Immune Cell Profiling in BALF: Our data show that co-exposure with FPM exacerbates CD45+ cells, eosinophil, neutrophil, T-cells and monocyte infiltration. Notably, CD45+ cells and neutrophils were the only population reduced under nociceptor neuron loss-of-function conditions (QX314–treated or TRPV1-DTA mice, Author response image 1).

      Of note, we also confirmed these data using intravital imaging and in a second line of nociceptor ablated mice (NaV1.8DTA). We are aware of Prof. Chang’s work suggesting expansion of an invariant iNKT cell population this population in future

      (3) Airway Hyperresponsiveness (AHR): We recognize that adding AHR data would strengthen the asthma-related context. Unfortunately, we are not currently equipped to perform AHR measurements, but we intend to include this in future experiments to provide a more complete assessment of airway function.

      Author response image 1.

      The authors could revisit the data presented in terms of visualization. For instance, the pooled data presented in some of the figures is probably leading to a wide variation which makes interpretation more difficult. Presenting data separately for each experimental replicate might help the reader. This is also important considering the possible variation seen between experiments (for instance, in Figure 3A and 3C and 3B and 3D, the neutrophil and eosinophil panels for the same groups seem to have an almost 2-fold difference.). Similarly, in the cytokine analysis, the authors have used a common axis for depicting all cytokine values which leads to difficulties in interpretation (Figure 3F). Analysis of the RNA seq results and the DEGs could be revisited to include pathway analysis etc (Figure 2), and the supplementary information could include detailed lists of the major target genes.

      To address this query, we have completely reformatted all graphs and included both gene lists and lists of enriched pathways for all three comparisons in Supplementary Table 1. We also confirmed our flow cytometry analysis functionally by performing intravital imaging.

      The authors should also consider citing the previous experimental setup used for some particular protocols. For instance, the use of the specified protocol for OVA in a C57 background needs to be justified, as there are various protocols reported in the literature. Additionally, doses used in some experiments seem arbitrary (The FPM and Artemin exposure in Figure 4). Depicting the dose-response curve or citing previous literature for the same would be important. Similarly, different sample sizes seen in experiments should be explained, whether they are due to mortality, failure to exhibit phenotypes, or due to technical failures. The RNA seq experiment mentions only 2 biological replicates in one of the groups which should be addressed either by increasing the sample size or by replicating the experiment. Moreover, nested comparisons in experiments performed for Figure 1 need to be performed. Neurons isolated from each mouse should be maintained and analysed separately to retain biological replicates to better represent the heterogeneity.

      We appreciate the request for clarity regarding the experimental protocols and sample sizes:

      OVA Model in C57BL/6 Mice: We adapted a previously published OVA protocol in C57BL/6 mice[3-5] (PMID: 39661516), which uses two doses of sensitization to compensate for the lower Th2 response compared to BALB/c[6]. We increased the dose of OVA (100 µg) because our initial experiments produced low eosinophil infiltration. Although this dosage is on the higher side, some studies have noted local IFNγ induction in C57BL/6 mice; however, we did not detect IFNγ in our setup.

      FPM and Artemin Doses: We did not perform a full dose-response assay for FPM and Artemin but used 100 ng/mL as reported in prior literature, where TRPA1 and TRPV1 mRNA were upregulated after 18 hours of incubation[7]. This reference has been added for clarity.

      Sample Sizes and Exclusions: One control mouse was excluded from the RNA-seq experiment because a parallel PCA analysis indicated it was an outlier. This was the only exclusion in the study, and this have been indicated in the method section of the article.  

      Nested Comparisons and Biological Replicates: We reanalyzed the relevant data with a nested one-way ANOVA and updated the figures accordingly. Neurons isolated from each mouse were first averaged to preserve biological replicates and capture potential heterogeneity; and data was analysed on the per mouse averages.

      The manuscript should be more detailed regarding the statistics employed. Currently, there is a section mentioned in the methods section, but details of corrections employed and specific stats for specific experiments should be described. There are also some minor grammatical errors and incomplete sentences in the manuscript which should be corrected. The authors should also consider a more expansive literature review in the introduction/discussion sections.

      We have updated the figure legends and methods to include more detailed information on the specific statistical tests used for each experiment. In addition, we have fixed minor grammatical errors and incomplete sentences throughout the manuscript. Finally, we have expanded our Introduction and Discussion to include additional references and a broader literature context.

      Reviewer #2 (Public review):

      The authors sought to investigate the role of nociceptor neurons in the pathogenesis of pollutionmediated neutrophilic asthma.

      We thank the reviewer for their valuable comments, which have significantly enhanced the quality of our manuscript. A point-by-point rebuttal is provided below.

      Strength

      The authors utilize TRPV1 ablated mice to confirm effects of intranasally administered QX-314 utilized to block sodium currents. The authors demonstrate that via artemin, which is upregulated in alveolar macrophages in response to pollution, sensitizes JNC neurons thereby increasing their responsiveness to pollution. Ablation or inactivity of nociceptor neurons prevented the pollution induced increase in inflammation.

      Weakness

      While neutrophilic, the model used doesn't appear to truly recapitulate a Th2/Th17 phenotype.  No IL-17A is visible/evident in the BALF fluid within the model. (Figure 3F). Unclear of the relevance of the RNAseq dataset, none of the identified DEGs were evaluated in the context of mechanism. The authors overall achieved the aim of demonstrating that nociceptor neurons are important to the pathogenesis of pollutionexacerbated asthma. Their results support their conclusions overall, although there are ways the study findings can be strengthened. This work further evaluates how nociceptor neurons contribute to asthma pathogenesis important for consideration while proposing treatment strategies for undertreated asthma endotypes.

      Major

      Utilizing a different model, one using house dust mite or alternaria alternata or similar that is able to induce a true Th2/th17 type response that is also more translatable to humans for confirmation.

      We appreciate the suggestion to use additional allergen models. In a pilot study, we did observe increased Artemin in the BALF of house dust mite–treated mice, although the levels were low under our current dosing schedule (20 µg/dose daily from Day 0–4 and Day 7–9, with sacrifice on Day 10; Auhtor response image 2). Conversely, using an Alternaria alternata model at 100 µg/dose daily from Day 0–2 (sacrificed on Day 3) did not yield a detectable increase in Artemin. We suspect these findings may reflect the specific dose and timing used. We plan to refine our protocols (e.g., longer exposures or higher doses) for HDM and/or Alternaria to better model a Th2/Th17 response and further validate our observations in a setting more translatable to human asthma.

      Author response image 2.

      Additional analysis, maybe pathway analysis on the RNAseq dataset presented in Figure 2. Unclear how these genes are relevant/how they affect functionality. At present it is acceptable to say they are transcriptionally reprogramed, but no protein evaluation is provided which would get more at function, however, the authors do show some functional data in Figure 1, so maybe this could somehow be discussed/related to Figure 2.

      We have expanded our RNA-seq analysis to include gene lists and enriched pathways for all three comparisons in Supplementary Table 1. We have also revised our discussion to align these transcriptomic changes with the functional data shown in Figure 1. While we have not yet performed protein-level validation for all identified genes, the patterns observed in our RNA-seq dataset suggest pathways potentially tied to nociceptor activation and the downstream inflammatory response. We plan to conduct targeted protein analyses in future studies to further substantiate these findings.

      Histology and localization of neutrophils/nociceptor neurons/alveolar macrophages would enhance the study findings.

      We appreciate the reviewer’s suggestion to include histological data showing the distribution of neutrophils, nociceptor neurons, and alveolar macrophages. While we have not yet performed detailed histological staining of these cell types, we have added live in-vivo intravital microscopy data (Figure 4) that illustrate impaired AM and neutrophil motility in nociceptor-ablated mice. We plan to include additional histological analyses in future studies to further localize these cells in the lung tissue.

      Minor:

      The first 3 figures are small and hard to read.

      We have enlarged Figures 1 and 3 in the revised manuscript to improve readability. We have also added the corresponding gene lists and enriched pathways to Supplementary Table 1 for clarity.

      The figures are mislabeled in the text. Figure 2 is discussed twice in two different contexts; the second mention is supposed to be labeled as Figure 2.

      We corrected the mislabeled figures in the text, ensuring that each figure is referenced accurately.

      Figure 4 isn't cited in the text. I think it is supposed to be referenced in the paragraph before the discussion starts and is currently labeled as Figure 1.

      We have updated the text to properly cite Figure 4 in the relevant paragraph before the Discussion begins, rather than labeling it as Figure 1.

      Notating which statistical analysis was used with each figure/subfigure would be beneficial. Also, it's important to notate if the data was analyzed for multiple comparisons.

      We have revised each figure/subfigure legend to specify the statistical tests used, including information on whether corrections for multiple comparisons were applied. This provides a clearer understanding of how each dataset was analyzed.

      Reviewer #3 (Public review):

      Asthma is a complex disease that includes endogenous epithelial, immune, and neural components that respond awkwardly to environmental stimuli. Small airborne particles with diameters in the range of 2.5 micrometers or less, so-called PM2.5, are generally thought to contribute to some forms of asthma. These forms of asthma may have increased numbers of neutrophils and/or eosinophils present in bronchoalveolar lavage fluid and are difficult to treat effectively as they tend to be poorly responsive to steroids. Here, Wang and colleagues build on a recent model that incorporated PM2.5 which was found to have a neutrophilic component. Wang altered the model to provide an extra kick via the incorporation of ovalbumin. Building on their prior expertise linking nociceptors and inflammation, they find that silencing TRPV1-expressing neurons either pharmacologically or genetically, abrogated inflammation and the accumulation of neutrophils. By examining bronchoalveolar lavage fluid, they found not only that levels of the number of cytokines were increased, but also that artemin, a protein that supports neuronal development and function, was elevated, which did not occur in nociceptor-ablated mice. They also found that alveolar macrophages exposed to PM2.5 particles had increased artemin transcription, suggesting a further link between pollutants, and immune and neural interactions.

      We thank the reviewer for their valuable comments, which have significantly enhanced the quality of our manuscript. A point-by-point rebuttal is provided below.

      Weakness

      There are substantial caveats that must be attached to the suggestions by the authors that targeting nociceptors might provide an approach to the treatment of neutrophilic airway inflammation in pollutiondriven asthma in general and wildfire-associated respiratory problems in particular.  

      These caveats include the uncertainty of the relevance of the conventional source of PM2.5, to pollution and asthma. According to the National Institute of Standards and Technology (NIST), the standard reference material (SRM) 2786 is a mix obtained from an air intake system in the Czech Republic. It is not clear exactly what is in the mix, and a recent bioRxiv preprint, https://www.biorxiv.org/content/10.1101/2023.08.18.553903v3.full.pdf reveals the presence of endotoxin. Care should thus be taken in interpreting data using particulate matter. Regarding wildfires, there is data that indicates that such exposure is toxic to macrophages. What impact might that then have on the production of cytokines, and artemin, in humans?

      We recognize the potential limitations of using SRM2786 (obtained from a Czech air-intake system) as a model for realworld PM2.5 exposure. Our rationale for choosing SRM2786 is that it is commercially available and represents a broad spectrum of ambient air pollutants, in contrast to more specialized sources like diesel exhaust particles. However, we acknowledge in the discussion the presence of endotoxin in SRM2786, as suggested by recent reports, and agree that this may influence immune responses and should be considered when interpreting our data.

      Regarding wildfire-associated exposure, we are aware that certain components of wildfire smoke can be toxic to macrophages. We do not think this play a significant role in the current study design as number of AMs, as determined by flow cytometry and intravital microscopy, are similar when comparing OVA-exposed mice to OVA-FPM exposed animals. Thus, these results rule out significant AM toxicity by FPM.

      Ultimately, while our findings suggest that modulating nociceptor activity may reduce neutrophilic inflammation, we emphasize that additional research—including different PM2.5 sources, validation of endotoxin levels, and in vivo confirmation in human-relevant models—is necessary before drawing definitive conclusions about treating pollutiondriven asthma or wildfire-induced respiratory problems.

      The Introductory paragraph implies links between wildfire events, particular exposure, and neutrophilic asthma. I am not aware of such a link having been established, in which case the paragraph needs revision. In the paragraph that begins with 'Urban pollution', it is suggested that eosinophilic asthma is treatment responsive in comparison to the neutrophilic form. That may not be the case, and they may often these cellular components may occur together. In much of the manuscript, there is a mismatch between the text and the figure numbers. For example, in the Results, Figure 2 should be Figure 3 some of the time, and Figure 3 is actually Figure 4, while the reference to Figure 1F-H is Figure 4H. Please check carefully.

      (a) Introduction Paragraph and Wildfire–Neutrophilic Asthma Link

      We add references to the introduction to support the link between wildfire, respiratory symptoms and the link to neutrophilic asthma [8-12].

      (b) Distinction Between Eosinophilic and Neutrophilic Asthma

      We recognize that eosinophilic and neutrophilic airway infiltrates can co-occur in the same individual and that treatment responsiveness can vary considerably. Our intention was to note that conventional asthma therapies (e.g., inhaled corticosteroids) are generally more effective for eosinophilic-driven disease than for neutrophilic phenotypes, but we agree that these inflammatory endotypes often overlap in clinical practice. We have revised the text in the “Urban pollution” section to acknowledge this complexity and to clarify that inflammatory cell populations in asthma are not always discrete.

      Figure Numbering and Text–Figure Mismatch

      We sincerely apologize for the confusion caused by mismatched figure labels and references in the Results section. We have carefully reviewed and corrected all figure references throughout the manuscript to ensure accuracy.

      References

      (1) Kim, S. H. et al. Mapping of the Sensory Innervation of the Mouse Lung by Specific Vagal and Dorsal Root Ganglion Neuronal Subsets. eNeuro 9 (2022). https://doi.org/10.1523/ENEURO.0026-22.2022

      (2) McGovern, A. E. et al. Evidence for multiple sensory circuits in the brain arising from the respiratory system: an anterograde viral tract tracing study in rodents. Brain Struct Funct 220, 3683-3699 (2015). https://doi.org/10.1007/s00429-014-0883-9

      (3) Shen, C.-C., Wang, C.-C., Liao, M.-H. & Jan, T.-R. A single exposure to iron oxide nanoparticles attenuates antigen-specific antibody production and T-cell reactivity in ovalbumin-sensitized BALB/c mice. International journal of nanomedicine, 1229-1235 (2011).  

      (4) Delayre-Orthez, C., De Blay, F., Frossard, N. & Pons, F. Dose-dependent effects of endotoxins on allergen sensitization and challenge in the mouse. Clinical & Experimental Allergy 34, 1789-1795 (2004).  

      (5) Morokata, T., Ishikawa, J. & Yamada, T. Antigen dose defines T helper 1 and T helper 2 responses in the lungs of C57BL/6 and BALB/c mice independently of splenic responses. Immunology letters 72, 119-126 (2000).  

      (6) Li, L., Hua, L., He, Y. & Bao, Y. Differential effects of formaldehyde exposure on airway inflammation and bronchial hyperresponsiveness in BALB/c and C57BL/6 mice. PLoS One 12, e0179231 (2017).  

      (7) Ikeda-Miyagawa, Y. et al. Peripherally increased artemin is a key regulator of TRPA1/V1 expression in primary afferent neurons. Molecular pain 11, s12990-12015-10004-12997 (2015).  

      (8) Baan, E. J. et al. Characterization of Asthma by Age of Onset: A Multi-Database Cohort Study. J Allergy Clin Immunol Pract 10, 1825-1834 e1828 (2022). https://doi.org/10.1016/j.jaip.2022.03.019

      (9) de Nijs, S. B., Venekamp, L. N. & Bel, E. H. Adult-onset asthma: is it really different? Eur Respir Rev 22, 44-52 (2013). https://doi.org/10.1183/09059180.00007112

      (10) Gianniou, N. et al. Acute effects of smoke exposure on airway and systemic inflammation in forest firefighters. J Asthma Allergy 11, 81-88 (2018). https://doi.org/10.2147/JAA.S136417

      (11) Noah, T. L., Worden, C. P., Rebuli, M. E. & Jaspers, I. The Effects of Wildfire Smoke on Asthma and Allergy. Curr Allergy Asthma Rep 23, 375-387 (2023). https://doi.org/10.1007/s11882-023-01090-1

      (12) Wilgus, M. L. & Merchant, M. Clearing the Air: Understanding the Impact of Wildfire Smoke on Asthma and COPD. Healthcare (Basel) 12 (2024). https://doi.org/10.3390/healthcare12030307

    1. Author Response

      The following is the authors’ response to the current reviews.

      We thank the editors and reviewers for their helpful comments, which have allowed us to improve the manuscript.

      Response to reviewer 2

      We thank the reviewer for this positive feedback, which requires no further revision.

      Response to reviewer 3

      We thank the reviewer for highlighting these additional points and provide further explanations on these below.

      Firstly, we started the analysis from a baseline of year 2000 because the largest international donor (the Global Fund) uses baseline malaria levels in the period 2000-2004 as the basis of their current allocation calculations (The Global Fund, Description of the 2020-2022 Allocation Methodology, December 2019). In the paper we compare our optimal strategy to a simplified version of this method, represented by our “proportional allocation” strategy.

      Even if our simulations started in the year 2015, a direct comparison with the Global Technical Strategy for Malaria 2016-2030 would not be possible due to the different approaches taken. The GTS was developed to progress towards malaria elimination globally and set ambitious targets of at least 90% reduction in malaria case incidence and mortality rates and malaria elimination in at least 35 countries by 2030 compared to 2015. Mathematical modelling at the time suggested that 90% coverage of WHO-recommended interventions (vector control, treatment and seasonal malaria chemoprevention) would be needed to approach this target (Griffin et al. 2016, Lancet Infectious Diseases). The global annual investment requirements to meet GTS targets were estimated at US$6.4 billion by 2020 and US$8.7 billion by 2030 (Patouillard et al. 2017, BMJ Global Health). This strategy therefore considers what resources would be required to achieve a specific global target, but not the optimized allocation of resources.

      Investments into malaria control have consistently been below the estimated requirements for the GTS milestones (World Health Organization 2022, World Malaria Report 2022). In our study, we therefore take a different perspective on how limited budgets can be optimally allocated to a single intervention (insecticide-treated nets) across countries/settings to achieve the best possible outcome for two objectives that are different to the GTS milestones (either minimizing the global case burden, or minimizing both the global case burden and the number of settings not having yet reached a pre-elimination phase). As stated in the discussion, our estimate of allocating 76% of very low budgets to high-transmission settings was similar to the global investment targets estimated for the GTS, where the 20 countries with the highest burden in 2015 were estimated to require 88% of total investments (Patouillard et al. 2017, BMJ Global Health). Nevertheless, we also show that if higher budgets were available, allocating the majority to low-transmission settings co-endemic for P. falciparum and P. vivax would achieve the largest reduction in global case burden. We acknowledge the modelling of a single intervention as one of the key limitations of this analysis, but this simplification was necessary in order to perform the complex optimisation problem. Computationally it would not have been feasible to optimize across a multitude of intervention and coverage combinations.

      A further limitation raised by the reviewer is the lack of cross-species immunity between P. falciparum and P. vivax in our model. While cross-reactivity between antibodies against these two species has been observed in previous studies and the potential implications of this would be important to explore in future work, we did not include it here as little is known to date about the epidemiological interactions between different malaria parasite species (Muh et al. 2020, PLoS Neglected Tropical Diseases).

      Lastly, we did not assume that transmission was homogenous within the four transmission settings in our study (very low, low, moderate, high); transmission dynamics were simulated separately in each country, accounting for heterogeneous mosquito bite exposure. However, results were summarised for the broader transmission settings since many other country-specific factors were not accounted for (see discussion) and the findings should not be used to inform individual country allocation decisions.


      The following is the authors’ response to the original reviews.

      Author response to peer review

      We thank the reviewers for their insightful comments, which raise several important points regarding our study. As the reviewers have recognised, we introduced a number of simplifications in order to perform this complex optimisation problem, such as by restricting the analysis to a single intervention (insecticide-treated nets) and modelling countries at a national level. Despite their clear relevance to the study, computationally it would not have been feasible to run the multitude of scenarios suggested by reviewer 1, which we recognise as a limitation. As such we agree with the assessment that this study primarily represents a thought experiment, based on substantive modelling and aggregate scenario-based analysis, to assess whether current policies are aligned with an optimal allocation strategy or whether there might be a need to consider alternative strategies. The findings are relevant primarily to global funders and should not be used to inform individual country allocation decisions, and also point to avenues for further research. This perspective also underlies our decision to start the analysis from a baseline of year 2000 as opposed to modelling the current 2023 malaria situation: the largest international donor (the Global Fund) uses baseline malaria levels in the period 2000-2004 as the basis of their allocation calculations (The Global Fund, Description of the 2020-2022 Allocation Methodology, December 2019) (1). A simplified version of this method is represented by our “proportional allocation” strategy. We have made several revisions to the manuscript to address the points raised by the reviewers, as detailed below.

      Reviewer #1 (Public Review):

      1. The authors present a back-of-the-envelope exploration of various possible resource allocation strategies for ITNs. They identify two optimal strategies based on two slightly different objective functions and compare 3 simple strategies to the outcomes of the optimal strategies and to each other. The authors consider both P falciparum and P vivax and explore this question at the country level, using 2000 prevalence estimates to stratify countries into 4 burden categories. This is a relevant question from a global funder perspective, though somewhat less relevant for individual countries since countries are not making decisions at the global scale.

      Thank you for this summary of the paper. We agree that our analysis is of relevance to global funders, but is not meant to inform individual country allocation decisions. In the discussion, we now state:

      p. 12 L19: “Therefore, policy decisions should additionally be based on analysis of country-specific contexts, and our findings are not informative for individual country allocation decisions.”

      1. The authors have made various simplifications to enable the identification of optimal strategies, so much so that I question what exactly was learned. It is not surprising that strategies that prioritize high-burden settings would avert more cases.

      Thank you for raising this point. Indeed, several simplifying assumptions were necessary to ensure the computational feasibility of this complex optimization problem. As a result, our study primarily represents a thought experiment to assess whether current policies are aligned with an optimal allocation strategy or whether there might be a need to consider alternative strategies. As now further outlined in the introduction, approaches to this have differed over time and it remains a relevant debate for malaria policy.

      p. 2 L22: “However, there remains a lack of consensus on how best to achieve this longer-term aspiration. Historically, large progress was made in eliminating malaria mainly in lower-transmission countries in temperate regions during the Global Malaria Eradication Program in the 1950s, with the global population at risk of malaria reducing from around 70% of the world population in 1950 to 50% in 2000 (2). Renewed commitment to malaria control in the early 2000s with the Roll Back Malaria initiative subsequently extended the focus to the highly endemic areas in sub-Saharan Africa (3).”

      We believe our findings not only confirm an “expected” outcome – that prioritizing high-burden settings would avert more cases – but also clearly illustrate various consequences of different allocation strategies that are implemented or considered in reality, which may not be so obvious. For example, we found that initially allocating a larger share of the budget to high-transmission countries could be both almost optimal in terms of reducing clinical cases and maximising the number of countries reaching pre-elimination. We also observed a trade-off between reducing burden and reducing the global population at risk (“shrinking the map”) through a focus on near-elimination settings, and estimate the loss in burden reduction when following an elimination target.

      1. Generally, I found much of the text confusing and some concepts were barely explained, such that the logic was difficult to follow.

      Thank you for bringing this to our attention, and we regret to hear the manuscript was confusing to read. We believe that the revisions made as a result of the reviewer comments have now made the manuscript much easier to follow. We additionally passed the manuscript to a colleague to identify confusing passages, and have added a number of sentences to clarify key concepts and improve the structure.

      1. I am not sure why the authors chose to stratify countries by 2000 PfPR estimates and in essence explore a counterfactual set of resource allocation strategies rather than begin with the present and compare strategies moving forward. I would think that beginning in 2020 and modeling forward would be far more relevant, as we can't change the past. Furthermore, there was no comparison with allocations and funding decisions that were actually made between 2000 and 2020ish so the decision to begin at 2000 is rather confusing.

      Thank you for pointing this out. We have now made the rationale for this choice clearer in the manuscript. Our main reason for this was to allow comparison with the Global Fund funding allocation, which is largely based on malaria disease burden in 2000-2004. As stated in the paper, malaria prevalence estimates in the year 2000 are commonly considered to represent a “baseline” endemicity level, before large-scale implementation of interventions in the following decades. In the manuscript, the transmission-related element of the Global Fund allocation algorithm is represented in our “proportional allocation” strategy. Previously this was only mentioned in the methods, but we have now added the following in the results to address this comment of the reviewer:

      p. 6 L12: “Strategies prioritizing high- or low-transmission settings involved sequential allocation of funding to groups of countries based on their transmission intensity (from highest to lowest EIR or vice versa). The proportional allocation strategy mimics the current allocation algorithm employed by the Global Fund: budget shares are mainly distributed according to malaria disease burden in the 2000-2004 period. To allow comparison with this existing funding model, we also started allocation decisions from the year 2000.”

      The Global Fund framework additionally considers economic capacity and other specific factors, and we have now also included a direct comparison with the 2020-2022 Global Fund allocation in Supplementary Figure S12 (see Author response image 1).

      We agree that looking at allocation decisions from 2020 onward would also constitute a very interesting question. However, the high dimensionality in scenarios to consider for this would currently make it computationally infeasible to run on the global level. Not only would it have to include all interventions currently implemented and available for malaria at different levels of coverage, but also the option of scaling down existing interventions. Instead, our priority in this paper was to conduct a thought experiment including both P. falciparum and P. vivax on a large geographical scale.

      Author response image 1.

      Impact of the proportional allocation strategy and the 2020-2022 Global Fund allocation on global malaria cases (panel A) and the total population at risk of malaria (panel B) at varying budgets. Both strategies use the same algorithm for budget share allocation based on malaria disease burden in 2000-2004, but the Global Fund allocation additionally involves an economic capacity component and specific strategic priorities.

      1. I realize this is a back-of-the-envelope assessment (although it is presented to be less approximate than it is, and the title does not reveal that the only intervention strategy considered is ITNs) but the number and scope of modeling assumptions made are simply enormous. First, that modeling is done at the national scale, when transmission within countries is incredibly heterogeneous. The authors note a differential impact of ITNs at various transmission levels and I wonder how the assumption of an intermediate average PfPR vs modeling higher and lower PfPR areas separately might impact the effect of the ITNs.

      Thank you for this comment. We agree the title could be more specific and have changed this to “Resource allocation strategies for insecticide-treated bednets to achieve malaria eradication”.

      Regarding the scale of ITN allocation, it is true that allocation at a sub-national scale could affect the results. However, considering this at a national scale is most relevant for our analysis because this is the scale at which global funding allocation decisions are made in practice. A sentence explaining this has been added in the methods.

      p. 15 L8: “The analysis was conducted on the national level, since this scale also applies to funding decisions made by international donors (1).”

      Further considering different geographical scales would also require introducing other assumptions, for example about how different countries would distribute funding sub-nationally, whether specific countries would take cooperative or competitive approaches to tackle malaria within a region or in border areas, and about delays in the allocation of bednets in specific regions. These interesting questions were outside of the scope of this work, but certainly require further investigation.

      1. Second, the effect of ITNs will differ across countries due to variations in vector and human behavior and variation in insecticide resistance and susceptibility to the ITNs. The authors note this as a limitation but it is a little mind-boggling that they chose not to account for either factor since estimates are available for the historical period over which they are modeling.

      Thank you for pointing this out. We did consider this and mentioned it as a limitation. Nevertheless, the complexity of accounting for this should also be recognised; for example, there is substantial uncertainty about the precise relationship between insecticide resistance and the population-level effect of ITNs (Sherrard-Smith et al., 2022, Lancet Planetary Health) (4). Additionally, our simulations extend beyond the 2000-2023 period so further assumptions about future changes to these factors would also be required. Simplifying assumptions are inherent to all mathematical modelling studies and we consider these particular simplifications acceptable given the high-level nature of the analysis.

      1. Third, the assumption that elimination is permanent and nothing is needed to prevent resurgence is, as the authors know, a vast oversimplification. Since resources will be needed to prevent resurgence, it appears this assumption may have a substantial impact on the authors' results.

      Thank you for this comment. In the discussion, we have now expanded on this:

      p. 13 L3: “While our analysis presents allocation strategies to progress towards eradication, the results do not provide insight into allocation of funding to maintain elimination. In practice, the threat of malaria resurgence has important implications for when to scale back interventions.”

      We believe that from a global perspective, the questions of funding allocation to achieve elimination vs to maintain it can currently still be considered separately given the large time-scales involved. The cost of preventing resurgence is not known, and one major problem in accounting for this would also be to identify relevant timescales to quantify this over.

      1. The decision to group all settings with EIR > 7 together as "high transmission" may perhaps be driven by WHO definitions but at a practical level this groups together countries with EIR 10 and EIR 500. Why not further subdivide this group, which makes sense from a technical perspective when thinking about optimal allocation strategies?

      Thank you for pointing this out. The WHO categories used are better interpreted in terms of the corresponding prevalence, which places countries with a prevalence of over 35% in the high transmission categories (WHO Guidelines for malaria, 31 March 2022) (5). We felt this is appropriate given that we are looking at theoretical global allocation patterns and do not aim to make recommendations for specific groups of countries or individual countries within sub-Saharan Africa that would be distinguished through the use of higher cut-offs. In our analysis, all 25 countries in the high transmission category were located in sub-Saharan Africa.

      1. The relevance of this analysis for elimination is a little questionable since no one eliminates with ITNs alone, to the best of my understanding.

      Thank you for this comment. We indeed state in the paper that ITNs alone are not sufficient to eliminate malaria. However, we still think that our analysis is relevant for elimination by taking a more theoretical perspective on reducing transmission using interventions. Starting from the 2000 baseline (or current levels) globally, large-scale transmission reductions such as those achieved by mass ITN distribution still represent the first key step on the path to malaria eradication, as shown in previous modelling work (Griffin et al., 2016, Lancet Infectious Diseases) (6). In the final phase of elimination, the WHO also recommends the addition of more targeted and reactive interventions (WHO Guidelines for malaria, 31 March 2022) (5). Our changes to the title of the article (“Resource allocation strategies for insecticide-treated bednets to achieve malaria eradication”) should now better reflect that we consider ITNs as just one necessary component to achieve malaria eradication.

      Reviewer #2 (Public Review):

      1. Schmit et al. analyze and compare different strategies for the allocation of funding for insecticide-treated nets (ITNs) to reduce the global burden of malaria. They use previously published models of Plasmodium falciparum and Plasmodium vivax malaria transmission to quantify the effect of ITN distribution on clinical malaria numbers and the population at risk. The impact of different resource allocation strategies on the reduction of malaria cases or a combination of malaria cases and achieving pre-elimination is considered to determine the optimal strategy to allocate global resources to achieve malaria eradication.

      Strengths:

      Schmit et al. use previously published models and optimization for rigorous analysis and comparison of the global impact of different funding allocation strategies for ITN distribution. This provides evidence of the effect of three different approaches: the prioritization of high-transmission settings to reduce the disease burden, the prioritization of low-transmission settings to "shrink the malaria map", and a resource allocation proportional to the disease burden.

      Thank you for providing this summary and outline of the strengths of the paper.

      1. Weaknesses:

      The analysis and optimization which provide the evidence for the conclusions and are thus the central part of this manuscript necessitate some simplifying assumptions which may have important practical implications for the allocation of resources to reduce the malaria burden. For example, seasonality, mosquito species-specific properties, stochasticity in low transmission settings, and changing population sizes were not included. Other challenges to the reduction or elimination of malaria such as resistance of parasites and mosquitoes or the spread of different mosquito species as well as other beneficial interventions such as indoor residual spraying, seasonal malaria chemoprevention, vaccinations, combinations of different interventions, or setting-specific interventions were also not included. Schmit et al. clearly state these limitations throughout their manuscript.

      The focus of this work is on ITN distribution strategies, other interventions are not considered. It also provides a global perspective and analysis of the specific local setting (as also noted by Schmit et al.) and different interventions as well as combinations of interventions should also be taken into account for any decisions.

      Thank you for raising these points. As outlined at the beginning of our response, for computational reasons we indeed had to introduce several simplifying assumptions to perform this complex optimisation problem. As a result of these factors you highlighted, our study should primarily be interpreted as a thought experiment to assess whether current policies are aligned with an optimal allocation strategy or whether there might be a need to consider alternative strategies. The findings are relevant primarily to global funders and should not be used to inform individual country allocation decisions, which we have further clarified in the manuscript.

      1. Nonetheless, the rigorous analysis supports the authors' conclusions and provides evidence that supports the prioritization of funding of ITNs for settings with high Plasmodium falciparum transmission. Overall, this work may contribute to making evidence-based decisions regarding the optimal prioritization of funding and resources to achieve a reduction in the malaria burden.

      Thank you for this positive assessment of our work.

      Reviewer #1 (Recommendations For The Authors):

      1. L144: last paragraph, the focus on endemic equilibrium: I did not really understand this, when 39 years is mentioned later is that a different analysis? How are cases averted calculated in a time-agnostic endemic equilibrium analysis? Perhaps a little more detail here would be helpful.

      A further explanation of this has been added in the results and methods.

      p. 8 L 22: “To evaluate the robustness of the results, we conducted a sensitivity analysis on our assumption on ITN distribution efficiency. Results remained similar when assuming a linear relationship between ITN usage and distribution costs (Figure S10). While the main analysis involves a single allocation decision to minimise long-term case burden (leading to a constant ITN usage over time in each setting irrespective of subsequent changes in burden), we additionally explored an optimal strategy with dynamic re-allocation of funding every 3 years to minimise cases in the short term.”

      p. 17 L25: “To ensure computational feasibility, 39 years was used as it was the shortest time frame over which the effect of re-distribution of funding from countries having achieved elimination could be observed.”

      p. 18 L 9: “Global malaria case burden and the population at risk were compared between baseline levels in 2000 and after reaching an endemic equilibrium under each scenario for a given budget.”

      1. L148: what is proportional allocation by disease burden and how is that different from prioritizing high-transmission settings?

      Further details have been added in the text.

      p. 6 L12: “Strategies prioritizing high- or low-transmission settings involved sequential allocation of funding to groups of countries based on their transmission intensity (from highest to lowest EIR or vice versa). The proportional allocation strategy mimics the current allocation algorithm employed by the Global Fund: budget shares are mainly distributed according to malaria disease burden in the 2000-2004 period. To allow comparison with this existing funding model, we also started allocation decisions from the year 2000.”

      1. L198-9: did low transmission settings get the majority of funding at intermediate and maximum budgets because they have the most population (I think so, based on Fig 1)?

      Yes, this is correct. We state in the results: “the optimized distribution of funding to minimize clinical burden depended on the available global budget and was driven by the setting-specific transmission intensity and the population at risk”.

      1. L206: what is ITN distribution efficiency? This is not explained. What is the 39-year period? Why this duration?

      Further explanations have been added in the results section, which were previously only detailed in the methods:

      p. 8 L 22: “To evaluate the robustness of the results, we conducted a sensitivity analysis on our assumption on ITN distribution efficiency. Results remained similar when assuming a linear relationship between ITN usage and distribution costs (Figure S10)."

      p. 17 L25: “To ensure computational feasibility, 39 years was used as it was the shortest time frame over which the effect of re-distribution of funding from countries having achieved elimination could be observed.”

      1. L218: what is "no intervention with a high budget"? is this a phrasing confusion?

      Yes, this has been changed.

      p. 9 L14: “We estimated that optimizing ITN allocation to minimize global clinical incidence could, at a high budget, avert 83% of clinical cases compared to no intervention.”

      1. L235-7: on comparing these results to previous work on the 20 highest-burden countries: is the definition of "high" similar enough across these studies that this is a relevant comparison?

      We believe this is reasonably comparable, as looking at the 20 highest-burden countries encompasses almost the entire high-transmission group in our work (25 countries in total), on which the comparison is made.

      1. L267-70: I didn't understand this sentence at all.

      Thanks for flagging this. The sentence referred to is: “Allocation proportional to disease burden did not achieve as great an impact as other strategies because the funding share assigned to settings was constant irrespective of the invested budget and its impact, and we did not reassign excess funding in high-transmission settings to other malaria interventions.”

      The previously mentioned added details on the proportional allocation strategy in the manuscript should now make this clearer, together with this clarification:

      p. 11 L17: “In modelling this strategy, we did not reassign excess funding in high-transmission settings to other malaria interventions, as would likely occur in practice.”

      For proportional allocation, a fixed proportion of the budget is calculated for each country based on disease burden, as described in the Global Fund allocation documentation (see Methods). However, since ITNs are the only intervention considered, this leads to a higher budget being allocated than is needed in some countries (i.e. where more funding doesn’t translate into further health gains).

      1. L339 EIR range: 80 is high at the country level but areas within countries probably went as high as 500 back in 2000. How does this affect the modeled estimates of ITN impact?

      The question of sub-national differences in transmission has been addressed in the public review comments. Briefly, we consider the national scale to be most relevant for our analysis because this is the scale at which global funding allocation decisions are made in practice. Although, as you correctly point out, the EIR affects ITN impact, it is not possible to conclude what the average effect of this would be on the country level without considering the following factors and introducing further assumptions on these: how would different countries distribute funding sub-nationally? Which countries would take cooperative or competitive approaches to tackle malaria within a region or in border areas? Would there be delays in the allocation of bednets in specific regions? These interesting questions were outside of the scope of this work, but certainly require further investigation.

      1. L347 population size constant: births and deaths are still present, is that right? Unclear from this sentence

      Yes, this is correct. Full details on the model can be found in the Supplementary Materials.

      1. L370 estimating ITN distribution required to achieve simulated population usage: is this a single relationship for all of Africa? Is it based on ITNs distributed 2:1 -> % access -> % usage? So it accounts for allocation inefficiency?

      Yes, this is represented by a single relationship for all of Africa to account for allocation inefficiency and is based on observed patterns across the continent and methodology developed in a previous publication (Bertozzi-Villa et al., 2021, Nature Communications) (7). Full details can be found in the Supplementary Materials (“Relationship between distribution and usage of insecticide-treated nets (ITNs)”, p. 21).

      1. L375: the ITN unit cost is assumed constant across countries and time (I think, it doesn't say explicitly), is this a good assumption?

      Yes, this is correct. We consider this a reasonable assumption within the scope of the paper. While delivery costs likely vary across countries, international funders usually have pooled procurement mechanisms for ITNs (The Global Fund, 2023, Pooled Procurement Mechanism Reference Pricing: Insecticide-Treated Nets).

      1. L399: "single allocation of a constant ITN usage" it is not explained what exactly this means

      Further explanations have been added in the manuscript.

      p. 8 L24: “While the main analysis involves a single allocation decision to minimise long-term case burden (leading to a constant ITN usage over time in each setting irrespective of subsequent changes in burden), we additionally explored an optimal strategy with dynamic re-allocation of funding every 3 years to minimise cases in the short term.”

      Reviewer #2 (Recommendations For The Authors):

      1. Additionally to the public comments, the only major comment is that in this reviewer's opinion, the focus on ITNs as the only intervention should be made clearer at different places in the manuscript (e.g. in the discussion lines 303-304). Otherwise, there are only some minor comments (see below).

      We have now modified the following sentence and also included this suggestion in the title (“Resource allocation strategies for insecticide-treated bednets to achieve malaria eradication”).

      p. 13 L8: “Our analysis demonstrates the most impactful allocation of a global funding portfolio for ITNs to reduce global malaria cases.”

      1. Minor comments:
      2. It may be of interest to compare the maximum budget obtained from the optimization with other estimates of required funding and actual available funding.

      Thank you for this interesting suggestion. Our maximum budget estimates are similar to the required investments projected for the WHO Global Technical Strategy: US$3.7 billion for ITNs in our analysis compared to between US$6.8 and US$10.3 billion total annual resources between 2020 and 2030, of which an estimated 55% would be required for (all) vector control (US$3.7 - US$5.7 billion) (Patouillard et al., 2016, BMJ Global Health) (8). However, it is well known that current spending is far below these requirements: total investments in malaria were estimated to be about US$3.1 billion per year in the last 5 years (World Health Organization, 2022, World Malaria Report 2022) (9).

      1. Line 177: should "Figure S7" be bold?

      Yes, this has been corrected.

      1. Line 218: what does "no intervention with high budget" mean? Should this simply be "no intervention"?

      This has been changed.

      p. 9 L14: “We estimated that optimizing ITN allocation to minimize global clinical incidence could, at a high budget, avert 83% of clinical cases compared to no intervention.”

      1. In this reviewer's opinion it would be easier for the reader if the weighting term in the objective function would be added in the Materials and Methods section. The weighting could be added without extending the section substantially and the explanation in lines 390-393 may be easier to understand.

      Thank you for this suggestion. We agree and have added this in the main manuscript.

      References

      1. The Global Fund. Description of the 2020-2022 Allocation Methodology 2019 [Available from: https://www.theglobalfund.org/media/9224/fundingmodel_2020-2022allocations_methodology_en.pdf.

      2. Hay SI, Guerra CA, Tatem AJ, Noor AM, Snow RW. The global distribution and population at risk of malaria: past, present, and future. Lancet Infect Dis. 2004;4(6):327-36.

      3. Feachem RGA, Phillips AA, Hwang J, Cotter C, Wielgosz B, Greenwood BM, et al. Shrinking the malaria map: progress and prospects. The Lancet. 2010;376(9752):1566-78.

      4. Sherrard-Smith E, Winskill P, Hamlet A, Ngufor C, N'Guessan R, Guelbeogo MW, et al. Optimising the deployment of vector control tools against malaria: a data-informed modelling study. The Lancet Planetary Health. 2022;6(2):e100-e9.

      5. World Health Organization. WHO Guidelines for malaria, 31 March 2022. Geneva: World Health Organization; 2022. Contract No.: Geneva WHO/UCN/GMP/ 2022.01 Rev.1.

      6. Griffin JT, Bhatt S, Sinka ME, Gething PW, Lynch M, Patouillard E, et al. Potential for reduction of burden and local elimination of malaria by reducing Plasmodium falciparum malaria transmission: a mathematical modelling study. The Lancet Infectious Diseases. 2016;16(4):465-72.

      7. Bertozzi-Villa A, Bever CA, Koenker H, Weiss DJ, Vargas-Ruiz C, Nandi AK, et al. Maps and metrics of insecticide-treated net access, use, and nets-per-capita in Africa from 2000-2020. Nature Communications. 2021;12(1):3589.

      8. Patouillard E, Griffin J, Bhatt S, Ghani A, Cibulskis R. Global investment targets for malaria control and elimination between 2016 and 2030. BMJ global health. 2017;2(2):e000176.

      9. World Health Organization. World malaria report 2022. Geneva: World Health Organization; 2022. Report No.: 9240064893.

    1. Author Response

      The following is the authors’ response to the original reviews.

      eLife assessment

      This work presents H3-OPT, a deep learning method that effectively combines existing techniques for the prediction of antibody structure. This work is important because the method can aid the design of antibodies, which are key tools in many research and industrial applications. The experiments for validation are solid.

      Comments to Author:

      Several points remain partially unclear, such as:

      1). Which examples constitute proper validation;

      Thank you for your kind reminder. We have modified the text of the experiments for validation to identify which examples constitute proper validation. We have corrected the “Finally, H3-OPT also shows lower Cα-RMSDs compared to AF2 or tFold-Ab for the majority of targets in an expanded benchmark dataset, including all antibody structures from CAMEO 2022” into “Finally, H3-OPT also shows lower Cα-RMSDs compared to AF2 or tFold-Ab for the majority (six of seven) of targets in an expanded benchmark dataset, including all antibody structures from CAMEO 2022” and added the following sentence in the experimental validation section of our revised manuscript to clarify which examples constitute proper validation: “AlphaFold2 outperformed IgFold on these targets”.

      2) What the relevance of the molecular dynamics calculations as performed is;

      Thank you for your comment, and I apologize for any confusion. The goal of our molecular dynamics calculations is to compare the differences in binding affinities, an important issue of antibody engineering, between AlphaFold2-predicted complexes and H3-OPT-predicted complexes. Molecular dynamics simulations enable the investigation of the dynamic behaviors and interactions of these complexes over time. Unlike other tools for predicting binding free energy, MM/PBSA or MM/GBSA calculations provide dynamic properties of complexes by sampling conformational space, which helps in obtaining more accurate estimates of binding free energy. In summary, our molecular dynamics calculations demonstrated that the binding free energies of H3-OPT-predicted complexes are closer to those of native complexes. We have included the following sentence in our manuscript to provide an explanation of the molecular dynamics calculations: “Since affinity prediction plays a crucial role in antibody therapeutics engineering, we performed MD simulations to compare the differences in binding affinities between AF2-predicted complexes and H3-OPT-predicted complexes.”.

      3) The statistics for some of the comparisons;

      Thank you for the comment. We have incorporated statistics for some of the comparisons in the revised version of our manuscript and added the following sentence in the Methods section: “We conducted two-sided t-test analyses to assess the statistical significance of differences between the various groups. Statistical significance was considered when the p-values were less than 0.05. These statistical analyses were carried out using Python 3.10 with the Scipy library (version 1.10.1).”.

      4) The lack of comparison with other existing methods.

      We appreciate your valuable comments and suggestions. Conducting comparisons with a broader set of existing methods can further facilitate discussions on the strengths and weaknesses of each method, as well as the accuracy of our method. In our study, we conducted a comparison of H3-OPT with many existing methods, including AlphaFold2, HelixFold-Single, ESMFold, and IgFold. We demonstrated that several protein structure prediction methods, such as ESMFold and HelixFold-Single, do not match the accuracy of AlphaFold2 in CDR-H3 prediction. Additionally, we performed a detailed comparison between H3-OPT, AlphaFold2, and IgFold (the latest antibody structure prediction method) for each target.

      We sincerely thank the comment and have introduced a comparison with OmegaFold. The results have been incorporated into the relevant sections (Fig 4a-b) of the revised manuscript.

      Author response image 1.

      Public Reviews

      Comments to Author:

      Reviewer #1 (Public Review):

      Summary:

      The authors developed a deep learning method called H3-OPT, which combines the strength of AF2 and PLM to reach better prediction accuracy of antibody CDR-H3 loops than AF2 and IgFold. These improvements will have an impact on antibody structure prediction and design.

      Strengths:

      The training data are carefully selected and clustered, the network design is simple and effective.

      The improvements include smaller average Ca RMSD, backbone RMSD, side chain RMSD, more accurate surface residues and/or SASA, and more accurate H3 loop-antigen contacts.

      The performance is validated from multiple angles.

      Weaknesses:

      1) There are very limited prediction-then-validation cases, basically just one case.

      Thanks for pointing out this issue. The number of prediction-then-validation cases is helpful to show the generalization ability of our model. However, obtaining experimental structures is both costly and labor-intensive. Furthermore, experimental validation cases only capture a limited portion of the sequence space in comparison to the broader diversity of antibody sequences.

      To address this challenge, we have collected different datasets to serve as benchmarks for evaluating the performance of H3-OPT, including our non-redundant test set and the CAMEO dataset. The introduction of these datasets allows for effective assessments of H3-OPT’s performance without biases and tackles the obstacle of limited prediction-then-validation cases.

      Reviewer #2 (Public Review):

      This work provides a new tool (H3-Opt) for the prediction of antibody and nanobody structures, based on the combination of AlphaFold2 and a pre-trained protein language model, with a focus on predicting the challenging CDR-H3 loops with enhanced accuracy than previously developed approaches. This task is of high value for the development of new therapeutic antibodies. The paper provides an external validation consisting of 131 sequences, with further analysis of the results by segregating the test sets into three subsets of varying difficulty and comparison with other available methods. Furthermore, the approach was validated by comparing three experimentally solved 3D structures of anti-VEGF nanobodies with the H3-Opt predictions

      Strengths:

      The experimental design to train and validate the new approach has been clearly described, including the dataset compilation and its representative sampling into training, validation and test sets, and structure preparation. The results of the in-silico validation are quite convincing and support the authors' conclusions.

      The datasets used to train and validate the tool and the code are made available by the authors, which ensures transparency and reproducibility, and allows future benchmarking exercises with incoming new tools.

      Compared to AlphaFold2, the authors' optimization seems to produce better results for the most challenging subsets of the test set.

      Weaknesses:

      1) The scope of the binding affinity prediction using molecular dynamics is not that clearly justified in the paper.

      We sincerely appreciate your valuable comment. We have added the following sentence in our manuscript to justify the scope of the molecular dynamics calculations: “Since affinity prediction plays a crucial role in antibody therapeutics engineering, we performed MD simulations to compare the differences in binding affinities between AF2-predicted complexes and H3-OPT-predicted complexes.”.

      2) Some parts of the manuscript should be clarified, particularly the ones that relate to the experimental validation of the predictions made by the reported method. It is not absolutely clear whether the experimental validation is truly a prospective validation. Since the methodological aspects of the experimental determination are not provided here, it seems that this may not be the case. This is a key aspect of the manuscript that should be described more clearly.

      Thank you for the reminder about experimental validation of our predictions. The sequence identities of the wild-type nanobody VH domain and H3 loop, when compared with the best template, are 0.816 and 0.647, respectively. As a result, these mutants exhibited low sequence similarity to our dataset, indicating the absence of prediction bias for these targets. Thus, H3-OPT outperformed IgFold on these mutants, demonstrating our model's strong generalization ability. In summary, the experimental validation actually serves as a prospective validation.

      Thanks for your comments, we have added the following sentence to provide the methodological aspects of the experimental determination: “The protein expression, purification and crystallization experiments were described previously. The proteins used in the crystallization experiments were unlabeled. Upon thawing the frozen protein on ice, we performed a centrifugation step to eliminate any potential crystal nucleus and precipitants. Subsequently, we mixed the protein at a 1:1 ratio with commercial crystal condition kits using the sitting-drop vapor diffusion method facilitated by the Protein Crystallization Screening System (TTP LabTech, mosquito). After several days of optimization, single crystals were successfully cultivated at 21°C and promptly flash-frozen in liquid nitrogen. The diffraction data from various crystals were collected at the Shanghai Synchrotron Research Facility and subsequently processed using the aquarium pipeline.”

      3) Some Figures would benefit from a clearer presentation.

      We sincerely thanks for your careful reading. According to your comments, we have made extensive modifications to make our presentation more convincing and clearer (Fig 2c-f).

      Author response image 2.

      Reviewer #3 (Public Review):

      Summary:

      The manuscript introduces a new computational framework for choosing 'the best method' according to the case for getting the best possible structural prediction for the CDR-H3 loop. The authors show their strategy improves on average the accuracy of the predictions on datasets of increasing difficulty in comparison to several state-of-the-art methods. They also show the benefits of improving the structural predictions of the CDR-H3 in the evaluation of different properties that may be relevant for drug discovery and therapeutic design.

      Strengths:

      The authors introduce a novel framework, which can be easily adapted and improved. The authors use a well-defined dataset to test their new method. A modest average accuracy gain is obtained in comparison to other state-of-the art methods for the same task while avoiding testing different prediction approaches.

      Weaknesses:

      1) The accuracy gain is mainly ascribed to easy cases, while the accuracy and precision for moderate to challenging cases are comparable to other PLM methods (see Fig. 4b and Extended Data Fig. 2). That raises the question: how likely is it to be in a moderate or challenging scenario? For example, it is not clear whether the comparison to the solved X-ray structures of anti-VEGF nanobodies represents an easy or challenging case for H3-OPT. The mutant nanobodies seem not to provide any further validation as the single mutations are very far away from the CDR-H3 loop and they do not disrupt the structure in any way. Indeed, RMSD values follow the same trend in H3-OPT and IgFold predictions (Fig. 4c). A more challenging test and interesting application could be solving the structure of a designed or mutated CDR-H3 loop.

      Thank you for your rigorous consideration. When the experimental structure is unavailable, it is difficult to directly determinate whether the target is easy-to-predict or challenging. We have conducted our non-redundant test set in which the number of easy-to-predict targets is comparable to the other two groups. Due to the limited availability of experimental antibody structures, especially nanobody structures, accurately predicting CDR-H3 remains a challenge. In our manuscript, we discuss the strengths and weakness of AlphaFold2 and other PLM-based methods, and we introduce H3-OPT as a comprehensive solution for antibody CDR3 modeling.

      We also appreciate your comment on experimental structures. We fully agree with your opinion and made attempts to solve the experimental structures of seven mutants, including two mutants (Y95F and Q118N) which are close to CDR-H3 loop. Unfortunately, we tried seven different reagent kits with a total of 672 crystallization conditions, but were unable to obtain crystals for these mutants. Despite the mutants we successfully solved may not have significantly disrupted the structures of CDR-H3 loops, they have still provided valuable insights into the differences between MSA-based methods and MSA-free methods (such as IgFold) for antibody structure modeling.

      We have further conducted a benchmarking study using two examples, PDBID 5U15 and 5U0R, both consisting of 18 residues in CDR-H3, to evaluate H3-OPT's performance in predicting mutated H3 loops. In the first case (target 5U15), AlphaFold2 failed to provide an accurate prediction of the extended orientation of the H3 loop, resulting in a less accurate prediction (Cα-RMSD = 10.25 Å) compared to H3-OPT (Cα-RMSD = 5.56 Å). In the second case (target 5U0R, a mutant of 5U15 in CDR3 loop), AlphaFold2 and H3-OPT achieved Cα-RMSDs of 6.10 Å and 4.25 Å, respectively. Additionally, the Cα-RMSDs of OmegaFold predictions were 8.05 Å and 9.84 Å, respectively. These findings suggest that both AlphaFold2 and OmegaFold effectively captured the mutation effects on conformations but achieved lower accuracy in predicting long CDR3 loops when compared to H3-OPT.

      2) The proposed method lacks a confidence score or a warning to help guide the users in moderate to challenging cases.

      We appreciate your suggestions and we have trained a separate module to predict confidence scores. We used the MSE loss for confidence prediction, where the label error was calculated as the Cα deviation of each residue after alignment. The inputs of this module are the same as those used for H3-OPT, and it generates a confidence score ranging from 0 to 100.

      3) The fact that AF2 outperforms H3-OPT in some particular cases (e.g. Fig. 2c and Extended Data Fig. 3) raises the question: is there still room for improvements? It is not clear how sensible is H3-OPT to the defined parameters. In the same line, bench-marking against other available prediction algorithms, such as OmegaFold, could shed light on the actual accuracy limit. We totally understand your concern. Many papers have suggested that PLM-based models are computationally efficient but may have unsatisfactory accuracy when high-resolution templates and MSA are available (Fast, accurate antibody structure prediction from deep learning on massive set of natural antibodies, Ruffolo, J. A. et al, 2023). However, the accuracy of AF2 decreased substantially when the MSA information is limited. Therefore, we directly retained high-confidence structures of AF2 and introduced a PSPM to improve the accuracy of the targets with long CDR-H3 loops and few sequence homologs. The improvement in mean Cα-RMSD demonstrated the room for accurately predicting CDR-H3 loops.

      We also appreciate your kind comment on defined parameters. In fact, once a benchmark dataset is established, determining an optimal cutoff value through parameter searching can indeed further improve the performance of H3-OPT in CDR3 structure prediction. However, it is important to note that this optimal cutoff value heavily depends on the testing dataset being used. Therefore, we provide a recommended cutoff value and offer a program interface for users who wish to manually define the cutoff value based on their specific requirements. Here, we showed the average Cα-RMSDs of our test set under different confidence cutoffs and the results have been added in the text accordingly.

      Author response table 1.

      We also appreciate your reminder, and we have conducted a benchmark against OmegaFold. The results have been included in the manuscript (Fig 4a-b).

      Author response image 3.

      Reviewer #1 (Recommendations For The Authors):

      1) In Fig 3a, please also compare IgFold and H3-OPT (merge Fig. S2 into Fig 3a)

      In Fig 3b, please separate Sub2 and Sub3, and add IgFold's performance.

      Thank you very much for your professional advice. We have made revisions to the figures based on your suggestions.

      Author response image 4.

      2) For the three experimentally solved structures of anti-VEGF nanobodies, what are the sequence identities of the VH domain and H3 loop, compared to the best available template? What is the length of the H3 loop? Which category (Sub1/2/3) do the targets belong to? What is the performance of AF2 or AF2-Multimer on the three targets?

      We feel sorry for these confusions. The sequence identities of the VH domain and H3 loop are 0.816 and 0.647, respectively, comparing with the best template. The CDR-H3 lengths of these nanobodies are both 17. According to our classification strategy, these nanobodies belong to Sub1. The confidence scores of these AlphaFold2 predicted loops were all higher than 0.8, and these loops were accepted as the outputs of H3-OPT by CBM.

      3) Is AF2-Multimer better than AF2, when using the sequences of antibody VH and antigen as input?

      Thanks for your suggestions. Many papers have benchmarked AlphaFold2-Multimer for protein complex modeling and demonstrated the accuracy of AlphaFold2-Multimer on predicting the protein complex is far from satisfactory (Benchmarking AlphaFold for protein complex modeling reveals accuracy determinants, Rui Yin, et al., 2022). Additionally, there is no significantly difference between AlphaFold2 and AlphaFold2-Multimer on antibody modeling (Structural Modeling of Nanobodies: A Benchmark of State-of-the-Art Artificial Intelligence Programs, Mario S. Valdés-Tresanco, et al., 2023)

      From the data perspective, we employed a non-redundant dataset for training and validation. Since these structures are valuable, considering the antigen sequence would reduce the size of our dataset, potentially leading to underfitting.

      4) For H3 loop grafting, I noticed that only identical target and template H3 sequences can trigger grafting (lines 348-349). How many such cases are in the test set?

      We appreciate your comment from this perspective. There are thirty targets in our database with identical CDR-H3 templates.

      Reviewer #2 (Recommendations For The Authors):

      • It is not clear to me whether the three structures apparently used as experimental confirmation of the predictions have been determined previously in this study or not. This is a key aspect, as a retrospective validation does not have the same conceptual value as a prospective, a posteriori validation. Please note that different parts of the text suggest different things in this regard "The model was validated by experimentally solving three structures of anti-VEGF nanobodies predicted by H3-OPT" is not exactly the same as "we then sought to validate H3-OPT using three experimentally determined structures of anti-VEGF nanobodies, including a wild-type (WT) and two mutant (Mut1 and Mut2) structures, that were recently deposited in protein data bank". The authors are kindly advised to make this point clear. By the way, "protein data bank" should be in upper case letters.

      We gratefully thank you for your feedback and fully understand your concerns. To validate the performance of H3-OPT, we initially solved the structures of both the wild-type and mutants of anti-VEGF nanobodies and submitted these structures to Protein Data Bank. We have corrected “that were recently deposited in protein data bank” into “that were recently deposited in Protein Data Bank” in our revised manuscript.

      • It would be good to clarify the goal and importance of the binding affinity prediction, as it seems a bit disconnected from the rest of the paper. Also, it would be good to include the production MD runs as Sup, Mat.

      Thanks for your valuable comment. We have added the following sentence in our manuscript to clarify the goal and importance of the molecular dynamics calculations: “Since affinity prediction plays a crucial role in antibody therapeutics engineering, we performed MD simulations to compare the differences in binding affinities between AF2-predicted complexes and H3-OPT-predicted complexes.”. The details of production runs have been described in Method section.

      • Has any statistical test been performed to compare the mean Cα-RMSD values across the modeling approaches included in the benchmark exercise?

      Thanks for this kind recommendation. We conducted a statistical test to assess the performance of different modeling approaches and demonstrated significant improvements with H3-OPT compared to other methods (p<0.001). Additionally, we have trained H3-OPT with five random seeds and compared mean Cα-RMSD values with all five models of AF2. Here, we showed the average Cα-RMSDs of H3-OPT and AlphaFold2.

      Author response table 1.

      • In Fig. 2c-f, I think it would be adequate to make the ordering criterion of the data points explicit in the caption or the graph itself.

      We appreciate your comment and suggestion. We have revised the graph in the manuscript accordingly.

      Author response image 5.

      • Please revise Figure S2 caption and/or its content. It is not clear, in parts b and c, which is the performance of H3-OPT. Why weren´t some other antibody-specific tools such as IgFold included in this comparison?

      Thanks for your comments. The performance of H3-OPT is not included in Figure S2. Prior to training H3-OPT, we conducted several preliminary studies, and the detailed results are available in the supplementary sections. We showed that AlphaFold2 outperformed other methods (including AI-based methods and TBM methods) and produced sub-angstrom predictions in framework regions. The comparison of IgFold with other methods was discussed in a previous work (Fast, accurate antibody structure prediction from deep learning on massive set of natural antibodies, Ruffolo, J. A. et al, 2023). In that study, we found that IgFold largely yielded results comparable to AlphaFold2 but with lower prediction cost. Additionally, we have also conducted a detailed comparison of CDR-H3 loops with IgFold in our main text.

      • It is stated that "The relative binding affinities of the antigen-antibody complexes were evaluated using the Python script...". Which Python script?

      Thank you for your comments, and I apologize for the confusion. This python script is a module of AMBER software, we have corrected “The relative binding affinities of the antigen-antibody complexes were evaluated using the python script” into “The relative binding affinities of the antigen-antibody complexes were evaluated using the MMPBSA module of AMBER software”.

      Reviewer #3 (Recommendations For The Authors):

      Does H3-OPT improve the AF2 score on the CDR-H3? It would be interesting to see whether grafted and PSPM loops improve the pLDDT score by using for example AF2Rank [https://doi.org/10.1103/PhysRevLett.129.238101]. That could also be a way to include a confidence score into H3-OPT.

      We are so grateful for your kind question. H3-OPT could not provide a confidence score for output in current version, so we did not know whether H3-OPT improve the AF2 score or not.

      We appreciate your kind recommendations and have calculated the pLDDT scores of all models predicted by H3-OPT and AF2 using AF2Rank. We showed that the average of pLDDT scores of different predicted models did not match the results of Cα-RMSD values.

      Author response table 3.

      Therefore, we have trained a separate module to predict the confidence score of the optimized CDR-H3 loops. We hope that this module can provide users with reliable guidance on whether to use predicted CDR-H3 loops.

      The test case of Nb PDB id. 8CWU is an interesting example where AF2 outperforms H3-OPT and PLMs. The top AF2 model according to ColabFold (using default options and no template [https://doi.org/10.1038/s41592-022-01488-1]) shows a remarkably good model of the CDR-H3, explaining the low Ca-RMSD in the Extended Data Fig. 3. However, the pLDDT score of the 4 tip residues (out of 12), forming the hairpin of the CDR-H3 loop, pushes down the average value bellow the CBM cut-off of 80. I wonder if there is a lesson to learn from that test case. How sensible is H3-OPT to the CBM cut-off definition? Have the authors tried weighting the residue pLDDT score by some structural criteria before averaging? I guess AF2 may have less confidence in hydrophobic tip residues in exposed loops as the solvent context may not provide enough support for the pLDDT score.

      Thanks for your valuable feedback. We showed the average Cα-RMSDs of our test set under different confidence cutoffs and the results have been added in the text accordingly.

      Author response table 4.

      We greatly appreciate your comment on this perspective. Inspired on your kind suggestions, we will explore the relationship between cutoff values and structural information in related work. Your feedback is highly valuable as it will contribute to the development of our approach.

      A comparison against the new folding prediction method OmegaFold [https://doi.org/10.1101/2022.07.21.500999] is missed. OmegaFold seems to outperform AF2, ESM, and IgFold among others in predicting the CDR-H3 loop conformation (See [https://doi.org/10.3390/molecules28103991] and [https://doi.org/10.1101/2022.07.21.500999]). Indeed, prediction of anti-VEGF Nb structure (PDB WT_QF_0329, chain B in supplementary data) by OmegaFold as implemented in ColabFold [https://colab.research.google.com/github/sokrypton/ColabFold/blob/main/beta/omegafold.ipynb] and setting 10 cycles, renders Ca-RMSD 1.472 Å for CDR-H3 (residues 98-115).

      We appreciate your valuable suggestion. We have added the comparison against OmegaFold in our manuscript. The results have been included in the manuscript (Fig 4a-b).

      Author response image 6.

      In our test set, OmegaFold outperformed ESMFold in predicting the CDR-H3 loop conformation. However, it failed to match the accuracy of AF2, IgFold, and H3-OPT. We discussed the difference between MSA-based methods (such as AlphaFold2) and MSA-free methods (such as IgFold) in predicting CDR-H3 loops. Similarly, OmegaFold provided comparative results with HelixFold-Single and other MSA-free methods but still failed to match the accuracy of AlphaFold2 and H3-OPT on Sub1.

      The time-consuming step in H3-OPT is the AF2 prediction. However, most of the time is spent in modeling the mAb and Nb scaffolds, which are already very well predicted by PLMs (See Fig. 4 in [https://doi.org/10.3390/molecules28103991]). Hence, why not use e.g. OmegaFold as the first step, whose score also correlates to the RMSD values [https://doi.org/10.3390/molecules28103991]? If that fails, then use AF2 or grafting. Alternatively, use a PLM model to generate a template, remove/mask the CDR loops (at least CDR-H3), and pass it as a template to AF2 to optimize the structure with or without MSA (e.g. using AF2Rank).

      Thanks for your professional feedbacks. It is really true that the speed of MSA searching limited the application of high-throughput structure prediction. Previous studies have demonstrated that the deep learning methods performed well on framework residues. We once tried to directly predict the conformations of CDR-H3 loops using PLM-based methods, but this initial version of H3-OPT lacking the CBM could not replicate the accuracy of AF2 in Sub1. Similarly, we showed that IgFold and OmegaFold also provide lower accuracy in Sub1 (average Cα-RMSD is 1.71 Å and 1.83 Å, respectively, whereas AF2 predicted an average of 1.07 Å). Therefore, The predictions of AlphaFold2 not only produce scaffolds but also provide the highest quality of CDR-H3 loops when high-resolution templates and MSA are available.

      Thank you once again for your kind recommendation. In the current version of H3-OPT, we have highlighted the strengths of H3-OPT in combining the AF2 and PLM models in various scenarios. AF2 can provide accurate predictions for short loops with fewer than 10 amino acids, and PLM-based models show little or no improvement in such cases. In the next version of H3-OPT, as the first step, we plan to replace the AF2 models with other methods if any accurate MSA-free method becomes available in the future.

      Line 115: The statement "IgFold provided higher accuracy in Sub3" is not supported by Fig. 2a.

      We are sorry for our carelessness. We have corrected “IgFold provided higher accuracy in Sub3” into “IgFold provided higher accuracy in Sub3 (Fig. 3a)”.

      Lines 195-203: What is the statistical significance of results in Fig 5a and 5b?

      Thank you for your kind comments. The surface residues of AF2 models are significantly higher than those of H3-OPT models (p < 0.005). In Fig. 5b, H3-OPT models predicted lower values than AF2 models in terms of various surface properties, including polarity (p <0.05) and hydrophilicity (p < 0.001).

      Lines 212-213: It is not easy to compare and quantify the differences between electrostatic maps in Fig. 5d. Showing a Dmap (e.g. mapmodel - mapexperiment) would be a better option. Additionally, there is no methodological description of how the maps were generated nor the scale of the represented potential.

      Thank you for pointing this out. We have modified the figure (Fig. 5d) according to your kind recommendation and added following sentences to clarify the methodological description on the surface electrostatic potential:

      “Analysis of surface electrostatic potential

      We generated two-dimensional projections of CDR-H3 loop’s surface electrostatic potential using SURFMAP v2.0.0 (based on GitHub from February 2023: commit: e0d51a10debc96775468912ccd8de01e239d1900) with default parameters. The 2D surface maps were calculated by subtracting the surface projection of H3-OPT or AF2 predicted H3 loops to their native structures.”

      Author response image 7.

      Lines 237-240 and Table 2: What is the meaning of comparing the average free energy of the whole set? Why free energies should be comparable among test cases? I think the correct way is to compare the mean pair-to-pair difference to the experimental structure. Similarly, reporting a precision in the order of 0.01 kcal/mol seems too precise for the used methodology, what is the statistical significance of the results? Were sampling issues accounted for by performing replicates or longer MDs?

      Thanks for your rigorous advice and pointing out these issues. We have modified the comparisons of free energies of different predicted methods and corrected the precision of these results. The average binding free energies of H3-OPT complexes is lower than AF2 predicted complexes, but there is no significant difference between these energies (p >0.05).

      Author response table 4.

      Comparison of binding affinities obtained from MD simulations using AF2 and H3-OPT.

      Thanks for your comments on this perspective. Longer MD simulations often achieve better convergence for the average behavior of the system, while replicates provide insights into the variability and robustness of the results. In our manuscript, each MD simulation had a length of 100 nanoseconds, with the initial 90 nanoseconds dedicated to achieving system equilibrium, which was verified by monitoring RMSD (Root Mean Square Deviation). The remaining 10 nanoseconds of each simulation were used for the calculation of free energy. This approach allowed us to balance the need for extensive sampling with the verification of system stability.

      Regarding MD simulations for CDR-H3 refinement, its successful application highly depends on the starting conformation, the force field, and the sampling strategy [https://doi.org/10.1021/acs.jctc.1c00341]. In particular, the applied plan MD seems a very limited strategy (there is not much information about the simulated times in the supplementary material). Similarly, local structure optimizations with QM methods are not expected to improve a starting conformation that is far from the experimental conformation.

      Thank you very much for your valuable feedback. We fully agree with your insights regarding the limitations of MD simulations. Before training H3-OPT, we showed the challenge of accurately predicting CDR-H3 structures. We then tried to optimize the CDR-H3 loops by computational tools, such as MD simulations and QM methods (detailed information of MD simulations is provided in the main text). Unfortunately, these methods were not expected to improve the accuracy of AF2 predicted CDR-H3 loops. These results showed that MD simulations and QM methods not only are time-consuming, but also failed to optimize the CDR-H3 loops. Therefore, we developed H3-OPT to tackle these issues and improve the accuracy of CDR3-H3 for the development of antibody therapeutics.

      Text improvements

      Relevant statistical and methodological parameters are presented in a dispersed manner throughout the text. For example, the number of structures in test, training, and validation datasets is first presented in the caption of Fig. 4. Similarly, the sequence identity % to define redundancy is defined in the caption of Fig. 1a instead of lines 87-88, where authors define "we constructed a non-redundant dataset with 1286 high-resolution (<2.5 Å)". Is the sequence redundancy for the CDR-H3 or the whole mAb/Nb?

      Thank you for pointing out these issues. We have added the number of structures in each subgroup in the caption of Fig. 1a: “Clustering of the filtered, high-resolution structures yielded three datasets for training (n = 1021), validation (n = 134), and testing (n = 131).” and corrected “As data quality has large effects on prediction accuracy, we constructed a non-redundant dataset with 1286 high-resolution (<2.5 Å) antibody structures from SAbDab” into “As data quality has large effects on prediction accuracy, we constructed a non-redundant dataset (sequence identity < 0.8) with 1286 high-resolution (<2.5 Å) antibody structures from SAbDab” in the revised manuscript. The sequence redundancy applies to the whole mAb/Nb.

      The description of ablation studies is not easy to follow. For example, what does removing TGM mean in practical terms (e.g. only AF2 is used, or PSPM is applied if AF2 score < 80)? Similarly, what does removing CBM mean in practical terms (e.g. all AF2 models are optimized by PSPM, and no grafting is done)? Thanks for your comments and suggestions. We have corrected “d, Differences in H3-OPT accuracy without the template module. e, Differences in H3-OPT accuracy without the CBM. f, Differences in H3-OPT accuracy without the TGM.” into “d, Differences in H3-OPT accuracy without the template module. This ablation study means only PSPM is used. e, Differences in H3-OPT accuracy without the CBM. This ablation study means input loop is optimized by TGM and PSPM. f, Differences in H3-OPT accuracy without the TGM. This ablation study means input loop is optimized by CBM and PSPM.”.

      Authors should report the values in the text using the same statistical descriptor that is used in the figures to help the analysis by the reader. For example, in lines 223-224 a precision score of 0.75 for H3-OPT is reported in the text (I assume this is the average value), while the median of ~0.85 is shown in Fig. 6a.

      Thank you for your careful checks. We have corrected “After identifying the contact residues of antigens by H3-OPT, we found that H3-OPT could substantially outperform AF2 (Fig. 6a), with a precision of 0.75 and accuracy of 0.94 compared to 0.66 precision and 0.92 accuracy of AF2.” into “After identifying the contact residues of antigens by H3-OPT, we found that H3-OPT could substantially outperform AF2 (Fig. 6a), with a median precision of 0.83 and accuracy of 0.97 compared to 0.64 precision and 0.95 accuracy of AF2.” in proper place of manuscript.

      Minor corrections

      Lines 91-94: What do length values mean? e.g. is 0-2 Å the RMSD from the experimental structure?

      We appreciate your comment and apologize for any confusion. The RMSD value is actually from experimental structure. The RMSD value evaluates the deviation of predicted CDR-H3 loop from native structure and also represents the degree of prediction difficulty in AlphaFold2 predictions. We have added following sentence in the proper place of the revised manuscript: “(RMSD, a measure of the difference between the predicted structure and an experimental or reference structure)”.

      Line 120: is the "AF2 confidence score" for the full-length or CDR-H3?

      We gratefully appreciate for your valuable comment and have corrected “Interestingly, we observed that AF2 confidence score shared a strong negative correlation with Cα-RMSDs (Pearson correlation coefficient =-0.67 (Fig. 2b)” into “Interestingly, we observed that AF2 confidence score of CDR-H3 shared a strong negative correlation with Cα-RMSDs (Pearson correlation coefficient =-0.67 (Fig. 2b)” in the revised manuscript.

      Line 166: Do authors mean "Taken" instead of "Token"?

      We are really sorry for our careless mistakes. Thank you for your reminder.

      Line 258: Reference to Fig. 1 seems wrong, do authors mean Fig. 4?

      We sincerely thank the reviewer for careful reading. As suggested by the reviewer, we have corrected the “Fig. 1” into “Fig. 4”.

      Author response image 7.

      Point out which plot corresponds to AF2 and which one to H3-OPT

      Thanks for pointing out this issue. We have added the legends of this figure in the proper positions in our manuscript.

    1. Author Response

      The following is the authors’ response to the current reviews.

      We thank both reviewers for their detailed and positive assessment of our work.

      To Reviewer #2, we have now explicated the pattern -- (QXQXQX>3)4 where X>3 denotes any length of three or more residues of any composition -- in the first paragraph of the discussion.

      To Reviewer #3, we have made slight modifications to the text in the “Q zippers poison themselves” results section, to attempt to further clarify the mechanism of self-poisoning.

      Briefly, the reviewer questions if an alternative model -- where inhibition involves non-structured rather than Q-zipper containing oligomers -- better explains the data. We provided two lines of evidence that we believe exclude this alternative model. First, we point out in the first paragraph of the “Q zippers poison themselves” section that the cells that unexpectedly lack amyloid in the high concentration regime have negligible levels of AmFRET, indicating that the inhibitory oligomers themselves occur at low concentrations regardless of the total concentration, and are therefore limited by a kinetic barrier. Second, we point out in the third paragraph of the section that the severity of amyloid inhibition with respect to concentration has a sequence dependence that matches the expectation of converging phase boundaries for crystal polymorphs -- specifically, inhibition is most severe for sequences that have a local Q density just high enough to form a Q zipper on both sides of each strand. Inhibition relaxed for sequences having more or less Qs than that threshold. In contrast, disordered oligomerization is not expected to have such a dependence on the precise pattern of Qs and Ns.


      The following is the authors’ response to the original reviews.

      We are pleased that the editors find our study valuable. We find that the reviewers’ criticisms largely arise from misunderstandings inherent to the conceptually challenging nature of the topic, rather than fundamental flaws, as we will elaborate here. We are grateful for the opportunity afforded by eLife to engage reviewers in what we intend to be a constructive public dialogue.

      Response to Reviewer 1

      This review is highly critical but lacks specifics. The reviewer’s criticisms reflect a position that seems to dismiss a critical role for (or perhaps even the existence of) conformational ordering in polyQ amyloid, which is untenable.

      The reviewer states that our objective to characterize the amyloid nucleus “rests on the assertion that polyQ forms amyloid structures to the exclusion of all other forms of solids”. We do not fully agree with this assertion because our findings show that detectable aggregation is rate-limited by conformational ordering, as evident by 1) its discontinuous relationship to concentration, 2) its acceleration by a conformational template, and 3) its strict dependence on very specific sequence features that are consistent with amyloid structure but not disordered aggregation).

      We strongly disagree with the reviewer’s subjective statement that we have not critically assessed our findings and that they do not stand up to scrutiny. This statement seems to rest on the perceived contradiction of our findings with that of Crick et al. 2013. Contrary to the reviewer’s assessment, we argue here that the conclusions of Crick et al. do more to support than to refute our findings. Briefly, Crick et al. investigated the aggregation of synthetic Q30 and Q40 peptides in vitro, wherein fibrils assembled from high concentrations of peptide were demonstrated to have saturating concentrations in the low micromolar range. As explained below, this finding of a saturating concentration does not refute our results. More relevant to the present work are their findings that “oligomers” accumulated over an hours-long timespan in solutions that are subsaturated with respect to fibrils, and these oligomers themselves have (nanomolar) critical concentrations. The authors postulated that the oligomers result from liquid–liquid demixing of intrinsically disordered polyglutamine. However, phase separation by a peptide is expected to fix its concentration in both the solute and condensed phases, and, because disordered phase separation is faster than amyloid formation, the postulated explanation removes the driving force for any amyloid phase with a critical solubility greater than that of the oligomers. In place of this interpretation that truly does appear to -- in the reviewer’s words -- “contradict basic physical principles of how homopolymers self-assemble”, we interpret these oligomers as evidence of Q zipper-containing self-poisoned multimers, rounded as an inherent consequence of self-poisoning (Ungar et al., 2005), and plausibly akin to semicrystalline spherulites that have been observed in other polymer crystal and amyloid-forming systems (Crist and Schultz, 2016; Vetri and Foderà, 2015). Importantly, the physical parameters governing the transition between amyloid spherulites and fibrils have been characterized in the case of insulin (Smith et al. 2012), where it was found that spherulites form at lower protein concentrations than fibrils. This mirrors the observation by Crick et al. that fibrils have a higher solubility limit than the spherical oligomers. . Further rebuttal to the perceived incompatibility of monomeric nucleation with the existence of a critical concentration for amyloid

      We appreciate that the concept of a monomeric nucleus can superficially appear inconsistent with the fact that crystalline solids such as polyQ amyloid have a saturating concentration, but this is only true if one neglects that polyQ amyloids are polymer crystals with intramolecular ordering. The perceived discrepancy is perhaps most easily dispelled by the fact that folded proteins can form crystals, and the folded state of the protein. These crystals have critical concentrations, and the protein subunits within them each have intramolecular crystalline order (in the form of secondary structure). When placed in a subsaturated solution, the protein crystals dissolve into the constituent monomers, and yet those monomers still retain intramolecular order. Our present findings for polyQ are conceptually no different.

      To further extrapolate this simple example to polyQ, one can also draw on the now well-established phenomenon of secondary nucleation, whereby transient interactions of soluble species with ordered species leads to their own ordering (Törnquist et al., 2018). Transience is important here because it implies that intramolecular ordering can in principle propagate even in solutions that are subsaturated with respect to bulk crystallization. This is possible in the present case because the pairing of sufficiently short beta strands (equivalent to “stems” in the polymer crystal literature) will be more stable intramolecularly than intermolecularly, due to the reduced entropic penalty of the former. Our elucidation that Q zipper ordering can occur with shorter strands intramolecularly than intermolecularly (Fig. S4C-D) demonstrates this fact. It is also evident from published descriptions of single molecule “crystals” formed in sufficiently dilute solutions of sufficiently long polymers (Hong et al., 2015; Keller, 1957; Lauritzen and Hoffman, 1960).

      In suggesting that a saturating concentration for amyloid rules out monomeric nucleation, the reviewer assumes that the Q zipper-containing monomer must be stable relative to the disordered ensemble. This is not inherent to our claim. The monomeric nucleating structure need not be more stable than the disordered state, and monomers may very well be disordered at equilibrium at low concentrations. To be clear, our claim requires that the Q zipper-containing monomer is both on pathway to amyloid and less stable than all subsequent species that are on pathway to amyloid. The former requirement is supported by our extensive mutational analysis. The latter requirement is supported by our atomistic simulations showing the Q zipper-containing monomer is stabilized by dimerization (included in our 2021 preprint). Hence, requisite ordering in the nucleating monomer is stabilized by intermolecular interactions. We provide in Author response image 1 an illustration to clarify what we believe to be the discrepancy between our claim and the reviewer’s interpretation.

      Author response image 1.

      That the rate-limiting fluctuation for a crystalline phase can occur in a monomer can also be understood as a consequence of Ostwald’s rule of stages, which describes the general tendency of supersaturated solutes, including amyloid forming proteins (Chakraborty et al., 2023), to populate metastable phases en route to more stable phases (De Yoreo, 2022; Schmelzer and Abyzov, 2017). Our findings with polyQ are consistent with a general mechanism for Ostwald’s rule wherein the relative stabilities of competing polymorphs differ with the number of subunits (De Yoreo, 2022; Navrotsky, 2004). As illustrated in Fig. 6 of Navrotsky, a polymorph that is relatively stable at small particle sizes tends to give way to a polymorph that -- while initially unstable -- becomes more stable as the particles grow. The former is analogous to our early stage Q zipper composed of two short sheets with an intramolecular interface, while the latter is analogous to the later stage Q zipper composed of longer sheets with an intermolecular interface. Subunit addition stabilizes the latter more than the former, hence the initial Q zipper that is stabilized more by intra- than intermolecular interactions will mature with growth to one that is stabilized more by intermolecular interactions.

      We have added a new figure (Fig. 6) to the manuscript to illustrate qualitative features of the amyloid pathway we have deduced for polyQ.

      Rebuttal to the perceived necessity of in vitro experiments

      The overarching concern of this reviewer and reviewing editor is whether in-cell assays can inform on sequence-intrinsic properties. We understand this concern. We believe however that the relative merit of in-cell assays is largely a matter of perspective. The truly sequence-intrinsic behavior of polyQ, i.e. in a vacuum, is less informative than the “sequence-intrinsic” behaviors of interest that emerge in the presence of extraneous molecules from the appropriate biological context. In vitro experiments typically include a tiny number of these -- water, ions, and sometimes a crowding agent meant to approximate everything else. Obviously missing are the myriad quinary interactions with other proteins that collectively round out the physiological solvent. The question is what experimental context best approximates that of a living human neuron under which the pathological sequence-dependent properties of polyQ manifest. We submit that a living yeast cell comes closer to that ideal than does buffer in a test tube.

      The reviewer’s statements that our findings must be validated in vitro ignores the fact -- stressed in our introduction -- that decades of in vitro work have not yet generated definitive evidence for or against any specific nucleus model. In addition to the above, one major problem concerns the large sizes of in vitro systems that obscure the effects of primary nucleation. For example, a typical in vitro experimental volume of e.g. 1.5 ml is over one billion-fold larger than the femtoliter volume of a cell. This means that any nucleation-limited kinetics of relevant amyloid formation are lost, and any alternative amyloid polymorphs that have a kinetic growth advantage -- even if they nucleate at only a fraction the rate of relevant amyloid -- will tend to dominate the system (Buell, 2017). Novel approaches are clearly needed to address these problems. We present such an approach, stretch it to the limit (as the reviewer notes) across multiple complementary experiments, and arrive at a novel finding that is fully and uniquely consistent with all of our own data as well as the collective prior literature.

      That the preceding considerations are collectively essential to understand relevant amyloid behavior is evident from recent cryoEM studies showing that in vitro-generated amyloid structures generally differ from those in patients (Arseni et al., 2022; Bansal et al., 2021; Radamaker et al., 2021; Schmidt et al., 2019; Schweighauser et al., 2020; Yang et al., 2022). This is highly relevant to the present discourse because each amyloid structure is thought to emanate from a different nucleating structure. This means that in vitro experiments have broadly missed the mark in terms of the relevant thermodynamic parameters that govern disease onset and progression. Note that the rules laid out via our studies are not only consistent with structural features of polyQ amyloid in cells, but also (as described in the discussion) explain why the endogenous structure of a physiologically relevant Q zipper amyloid differs from that of polyQ.

      A recent collaboration between the Morimoto and Knowles groups (Sinnige et al.) investigated the kinetics of aggregation by Q40-YFP expressed in C. elegans body wall muscle cells, using quantitative approaches that have been well established for in vitro amyloid-forming systems of the type favored by the reviewer. They calculate a reaction order of just 1.6, slightly higher than what would be expected for a monomeric nucleus but nevertheless fully consistent with our own conclusions when one accounts for the following two aspects of their approach. First, the polyQ tract in their construct is flanked by short poly-Histidine tracts on both sides. These charges very likely disfavor monomeric nucleation because all possible configurations of a four-stranded bundle position the beginning and end of the Q tract in close proximity, and Q40 is only just long enough to achieve monomeric nucleation in the absence of such destabilization. Second, the protein is fused to YFP, a weak homodimer (Landgraf et al., 2012; Snapp et al., 2003). With these two considerations, our model -- which was generated from polyQ tracts lacking flanking charges or an oligomeric fusion -- predicts that amyloid nucleation by their construct will occur more frequently as a dimer than a monomer. Indeed, their observed reaction order of 1.6 supports a predominantly dimeric nucleus. Like us and others, Sinnige et al. did not observe phase separation prior to amyloid formation. This is important because it not only argues against nucleation occurring in a condensate, it also suggests that the reaction order they calculated has not been limited by the concentration-buffering effect of phase separation.

      While we agree that our conclusions rest heavily on DAmFRET data (for good reason), we do provide supporting evidence from molecular dynamics simulations, SDD-AGE, and microscopy.

      To summarize, given the extreme limitations of in vitro experiments in this field, the breadth of our current study, and supporting findings from another lab using rigorous quantitative approaches, we feel that our claims are justified without in vitro data.

      Rebuttals to other critiques

      We do not deny that flanking domains can modulate the kinetics and stability of polyQ amyloid. However, as stated and referenced in the introduction, they do not appear to change the core structure. We have also added a paragraph concerning flanking domains to the discussion, and acknowledged that “the extent to which our findings will translate in these different contexts remains to be determined.” Nevertheless, that the intrinsic behavior of the polyQ tract itself is central to pathology is evident from the fact that the nine pathologic polyQ proteins have similar length thresholds despite different functions, flanking domains, interaction partners, and expression levels.

      The reviewer states that we found nucleation potential to require 60 Qs in a row. Our data are collectively consistent with nucleation occurring at and above approximately 36 Qs, a point repeated in the paper. The reviewer may be referring to our statement, ”Sixty residues proved to be the optimum length to observe both the pre- and post-nucleated states of polyQ in single experiments”. The purpose of this statement is simply to describe the practical consideration that led us to use 60 Qs for the bulk of our assays. We do appreciate that the fraction of AmFRET-positive cells is very low for lengths just above the threshold, especially Q40. They are nevertheless highly significant (p = 0.004 in [PIN+] cells, one-tailed T-test), and we have modified the figure and text to clarify this.

      The reviewer characterizes self-poisoning as the hallmark of crystallization from polymer melts, which would be problematic for our conclusions if self-poisoning were limited to this non-physiological context. In fact the term was first used to describe crystallization from solution (Organ et al., 1989), wherein the phenomenon is more pronounced (Ungar et al., 2005).

      Response to Reviewer 2

      We thank the reviewer for their detailed and helpful critique.

      The reviewer correctly notes that the majority of our manipulations were conducted with 60-residue long tracts (which corresponds to disease onset in early adulthood), and this length facilitates intramolecular nucleation. However, we also analyzed a length series of polyQ spanning the pathological threshold, as well as a synthetic sequence designed explicitly to test the model nucleus structure with a tract shorter than the pathological threshold, and both experiments corroborate our findings.

      The reviewer mentions “several caveats” that come with our result, but their subsequent elaboration suggests they are to be interpreted more as considerations than caveats. We agree that increasing sequence complexity will tend to increase homogeneity, but this is exactly the motivation of our approach. We explicitly set out to determine the minimal complexity sequence sufficient to specify the nucleating conformation, which we ultimately identified in terms of secondary and tertiary structure. We do not specify which parts of a long polyQ tract correspond to which parts of the structure, because, as the reviewer points out, they can occur at many places. Hence, depending on the length of the polyQ tract, the nucleus we describe may have any length of sequence connecting the strand elements. We do not think that the effects of N-residue placement can be interpreted as a confounding influence on hairpin position because the striking even-odd pattern we observe implicates the sides of beta strands rather than the lengths. Moreover, we observe this pattern regardless of the residue used (Gly, Ser, Ala, and His in addition to Asn).

      We thank the reviewer for noting the novelty and plausibility of the self-poisoning connection. We would like to elaborate on our finding that self-poisoning inhibits nucleation (in addition to elongation), as this will be confusing to many readers. While self-poisoning is claimed to inhibit primary nucleation in the polymer crystal literature (Ungar et al., 2005; Zhang et al., 2018), the semantics of “nucleation” in this context warrants clarification. Technically, the same structure can be considered a nucleus in one context but not in another. The Q zipper monomer, even if it is rate-limiting for amyloid formation at low concentrations (and is therefore the “nucleus”), is not necessarily rate-limiting when self-poisoned at high concentrations. Whether it comprises the nucleus in this case depends on the rates of Q zipper formation relative to subunit addition to the poisoned state. If the latter happens slower than Q zipper formation de novo, it can be said that self-poisoning inhibits nucleation, regardless of whether the Q zipper formed. We suspect this to be the mechanism by which preemptive oligomerization blocks nucleation in the case of polyQ, though other mechanisms may be possible.

      We believe the revised text also now incorporates the remaining suggestions of this reviewer, with two exceptions. 1) We retain the phrase “hidden pattern”, because we believe our data argue for a nucleus whose formation requires that Qs occur in a pattern that we now elaborate as (QXQXQX>3)4 where X>3 denotes any length of three or more residues of any composition. In amyloids formed from long polyQ molecules, the nucleus will involve any subset of 12 Qs that match this pattern. 2) We decided not to re-order the mansucript to discuss self-poisoning after establishing the monomer nucleus (even though we agree that doing so would improve the logical flow) because the interpretation of the data with respect to self-poisoning helps to establish critical strand lengths, and self-poisoning creates an anomaly in the DAmFRET data that is difficult to ignore. We add text clarifying that high local concentrations “effectively shifts the rate-limiting step to the growth of a higher order relatively-disordered species”.

      Response to Reviewer 3

      We thank the reviewer for their helpful comments.

      We opted to retain Figures 1A and B because we think they are important for comprehending the subject and objectives of the study. We modified the former to attempt to make it more clear. We have also elaborated on DAmFRET as it is a relatively new approach that may be unfamiliar to many readers. Beyond this, we refer the reviewer and readers to our cited prior work describing the theory and interpretation of DAmFRET. Note that the y-axes of DAmFRET plots are not raw FRET but rather “AmFRET”, a ratio of FRET to total expression level. As explained thoroughly in our cited prior work, the discontinuity of AmFRET with expression level indicates that the high AmFRET-population formed via a disorder-to-order transition. When the query protein is predicted to be intrinsically disordered, the discontinuous transition to high AmFRET invariably (among hundreds of proteins tested in prior published and unpublished work) signifies amyloid formation as corroborated by SDD-AGE and tinctorial assays.

      When performed using standard flow cytometry as in the present study, every AmFRET measurement corresponds to a cell-wide average, and hence does not directly inform on the distribution of the protein between different stoichiometric species. As there is only one fluorophore per protein molecule, monomeric nuclei have no signal. DAmFRET can distinguish cells expressing monomers from stable dimers from higher order oligomers (see e.g. Venkatesan et al. 2019), and we are therefore quite confident that AmFRET values of zero correspond to cells in which a vast majority of the respective protein is not in homo-oligomeric species (i.e. is monomeric or in hetero-complexes with endogenous proteins). The exact value of AmFRET, even for species with the same stoichiometry, will depend both on the effect of their respective geometries on the proximity of mEos3.1 fluorophores, and on the fraction of protein molecules in the species. Hence, we only attempt to interpret the plateau values of AmFRET (where the fraction of protein in an assembled state approaches unity) as directly informing on structure, as we did in Fig. S3A.

      We believe that AmFRET decreases with longer polyQ because the mass fraction of fluorophore decreases in the aggregate, simply because the extra polypeptide takes up volume in the aggregate.

      Yes, the fraction of positive cells in a discontinuous DAmFRET plot does increase with time. However, given the more laborious data collection and derivation of nucleation kinetics in a system with ongoing translation, especially across hundreds of experiments with other variables, ours is a snapshot measurement to approximately derive the relative contributions of intra- and intermolecular fluctuations to the nucleation barrier, rather than the barrier’s magnitude.

      We have revised the tautological statement by removing “non-amyloid containing”.

      Concerning the correlation of our data with the pathological length threshold -- as we state in the first results section, “Our data recapitulated the pathologic threshold -- Q lengths 35 and shorter lacked AmFRET, indicating a failure to aggregate or even appreciably oligomerize, while Q lengths 40 and longer did acquire AmFRET in a length and concentration-dependent manner”. Hence, most of our experiments were conducted with 60Q not because it resembles the pathological threshold, but rather because it was most convenient for DAmFRET experiments.

      Self-poisoning is a widely observed and heavily studied phenomenon in polymer crystal physics, though it seems not yet to have entered the lexicon of amyloid biologists. We were new to this concept before it emerged as an extremely parsimonious explanation for our results. As described in the text, two pieces of evidence exclude the alternative mechanism suggested by the reviewer -- that non-structured oligomers form and subsequently engage and inhibit the template. Specifically, 1) inhibition occurs without any detectable FRET, even at high total protein concentration, indicating the species do not form in a concentration-dependent manner that would be expected of disordered oligomers; and 2) inhibition itself has strict sequence requirements that match those of Q zippers. Hence our data collectively suggest that inhibition is a consequence of the deposition of partially ordered molecules onto the templating surface.

      We have softened the subheading and text of the relevant section in the discussion to more clearly indicate the speculative nature of our statements concerning the possible role of self-poisoned oligomers in toxicity.

      We stand by our statement 'that kinetically arrested aggregates emerge from the same nucleating event responsible for amyloid formation', as this follows directly from self-poisoning.

      Regarding the arguments for lateral and axial growth, we agree that the data are indirect. However, that polyQ forms lamellar amyloids both in vitro and in vivo is now established, so we do not feel it necessary to rigorously show that here. Nevertheless, we need to include this section primarily because it introduces the fact that ordering in polyQ amyloid occurs in the lateral as well as axial dimensions, and the onset of lateral ordering (lamellar growth) explains the very different behaviors of QU and QB sequences apparent on the DAmFRET plots. Ultimately, the two dimensions of growth are important to understand self-poisoning and maturation of the short nucleating zipper to amyloid.

      References

      Arseni D, Hasegawa M, Murzin AG, Kametani F, Arai M, Yoshida M, Ryskeldi-Falcon B. 2022. Structure of pathological TDP-43 filaments from ALS with FTLD. Nature 601:139–143. doi:10.1038/s41586-021-04199-3

      Bansal A, Schmidt M, Rennegarbe M, Haupt C, Liberta F, Stecher S, Puscalau-Girtu I, Biedermann A, Fändrich M. 2021. AA amyloid fibrils from diseased tissue are structurally different from in vitro formed SAA fibrils. Nat Commun 12:1013. doi:10.1038/s41467-021-21129-z

      Buell AK. 2017. The Nucleation of Protein Aggregates - From Crystals to Amyloid Fibrils. Int Rev Cell Mol Biol 329:187–226. doi:10.1016/bs.ircmb.2016.08.014

      Chakraborty D, Straub JE, Thirumalai D. 2023. Energy landscapes of Aβ monomers are sculpted in accordance with Ostwald’s rule of stages. Sci Adv 9:eadd6921. doi:10.1126/sciadv.add6921 Crist B, Schultz JM. 2016. Polymer spherulites: A critical review. Prog Polym Sci 56:1–63. doi:10.1016/j.progpolymsci.2015.11.006

      De Yoreo JJ. 2022. Casting a bright light on Ostwald’s rule of stages. Proc Natl Acad Sci USA 119. doi:10.1073/pnas.2121661119

      Hong Y, Yuan S, Li Z, Ke Y, Nozaki K, Miyoshi T. 2015. Three-Dimensional Conformation of Folded Polymers in Single Crystals. Phys Rev Lett 115:168301. doi:10.1103/PhysRevLett.115.168301 Keller A. 1957. A note on single crystals in polymers: Evidence for a folded chain configuration. Philosophical Magazine 2:1171–1175. doi:10.1080/14786435708242746

      Landgraf D, Okumus B, Chien P, Baker TA, Paulsson J. 2012. Segregation of molecules at cell division reveals native protein localization. Nat Methods 9:480–482. doi:10.1038/nmeth.1955

      Lauritzen JI, Hoffman JD. 1960. Theory of Formation of Polymer Crystals with Folded Chains in Dilute Solution. J Res Natl Bur Stand A Phys Chem 64A:73–102. doi:10.6028/jres.064A.007

      Navrotsky A. 2004. Energetic clues to pathways to biomineralization: precursors, clusters, and nanoparticles. Proc Natl Acad Sci USA 101:12096–12101. doi:10.1073/pnas.0404778101

      Ohhashi Y, Ito K, Toyama BH, Weissman JS, Tanaka M. 2010. Differences in prion strain conformations result from non-native interactions in a nucleus. Nat Chem Biol 6:225–230. doi:10.1038/nchembio.306

      Organ SJ, Ungar G, Keller A. 1989. Rate minimum in solution crystallization of long paraffins. Macromolecules 22:1995–2000. doi:10.1021/ma00194a078

      Radamaker L, Baur J, Huhn S, Haupt C, Hegenbart U, Schönland S, Bansal A, Schmidt M, Fändrich M. 2021. Cryo-EM reveals structural breaks in a patient-derived amyloid fibril from systemic AL amyloidosis. Nat Commun 12:875. doi:10.1038/s41467-021-21126-2

      Sahoo B, Singer D, Kodali R, Zuchner T, Wetzel R. 2014. Aggregation behavior of chemically synthesized, full-length huntingtin exon1. Biochemistry 53:3897–3907. doi:10.1021/bi500300c

      Schmelzer JWP, Abyzov AS. 2017. How do crystals nucleate and grow: ostwald’s rule of stages and beyond In: Šesták J, Hubík P, Mareš JJ, editors. Thermal Physics and Thermal Analysis, Hot Topics in Thermal Analysis and Calorimetry. Cham: Springer International Publishing. pp. 195–211. doi:10.1007/978-3-319-45899-1_9

      Schmidt M, Wiese S, Adak V, Engler J, Agarwal S, Fritz G, Westermark P, Zacharias M, Fändrich M. 2019. Cryo-EM structure of a transthyretin-derived amyloid fibril from a patient with hereditary ATTR amyloidosis. Nat Commun 10:5008. doi:10.1038/s41467-019-13038-z

      Schweighauser M, Shi Y, Tarutani A, Kametani F, Murzin AG, Ghetti B, Matsubara T, Tomita T, Ando T, Hasegawa K, Murayama S, Yoshida M, Hasegawa M, Scheres SHW, Goedert M. 2020. Structures of α-synuclein filaments from multiple system atrophy. Nature 585:464–469. doi:10.1038/s41586-020-2317-6

      Snapp EL, Hegde RS, Francolini M, Lombardo F, Colombo S, Pedrazzini E, Borgese N, Lippincott-Schwartz J. 2003. Formation of stacked ER cisternae by low affinity protein interactions. J Cell Biol 163:257–269. doi:10.1083/jcb.200306020

      Törnquist M, Michaels TCT, Sanagavarapu K, Yang X, Meisl G, Cohen SIA, Knowles TPJ, Linse S. 2018. Secondary nucleation in amyloid formation. Chem Commun 54:8667–8684. doi:10.1039/c8cc02204f

      Ungar G, Putra EGR, de Silva DSM, Shcherbina MA, Waddon AJ. 2005. The Effect of Self-Poisoning on Crystal Morphology and Growth Rates In: Allegra G, editor. Interphases and Mesophases in Polymer Crystallization I, Advances in Polymer Science. Berlin, Heidelberg: Springer Berlin Heidelberg. pp. 45–87. doi:10.1007/b107232

      Vetri V, Foderà V. 2015. The route to protein aggregate superstructures: Particulates and amyloid-like spherulites. FEBS Lett 589:2448–2463. doi:10.1016/j.febslet.2015.07.006

      Wild EJ, Boggio R, Langbehn D, Robertson N, Haider S, Miller JRC, Zetterberg H, Leavitt BR, Kuhn R, Tabrizi SJ, Macdonald D, Weiss A. 2015. Quantification of mutant huntingtin protein in cerebrospinal fluid from Huntington’s disease patients. The Journal of Clinical Investigation.

      Yang Y, Arseni D, Zhang W, Huang M, Lövestam S, Schweighauser M, Kotecha A, Murzin AG, Peak-Chew SY, Macdonald J, Lavenir I, Garringer HJ, Gelpi E, Newell KL, Kovacs GG, Vidal R, Ghetti B, Ryskeldi-Falcon B, Scheres SHW, Goedert M. 2022. Cryo-EM structures of amyloid-β 42 filaments from human brains. Science 375:167–172. doi:10.1126/science.abm7285

      Zhang X, Zhang W, Wagener KB, Boz E, Alamo RG. 2018. Effect of Self-Poisoning on Crystallization Kinetics of Dimorphic Precision Polyethylenes with Bromine. Macromolecules 51:1386–1397. doi:10.1021/acs.macromol.7b02745

    2. Author Response

      eLife assessment

      In this valuable study, the authors investigate the mechanism of amyloid nucleation in a cellular system using their novel ratiometric measurements and uncover interesting insights regarding the role of polyglutamine length and the sequence features of glutamine-rich regions on amyloid formation. Overall, the problem is significant and being able to assess nucleation in cells is of considerable relevance. The data, as presented and analyzed, are currently still incomplete. The specific claims would be stronger if based on in vitro measurements that avoid the intricacies of specific cellular systems and that are more suitable for assessing sequence-intrinsic properties.

      We are pleased that the editors find our study valuable. We find that the reviewers’ criticisms largely arise from misunderstandings inherent to the conceptually challenging nature of the topic, rather than fundamental flaws, as we will elaborate here. We are grateful for the opportunity afforded by eLife to engage reviewers in a constructive public dialogue.

      Reviewer #1 (Public Review):

      The authors take on the challenge of defining the core nucleus for amyloid formation by polyglutamine tracts. This rests on the assertion that polyQ forms amyloid structures to the exclusion of all other forms of solids. Using their unique assay, deployed in yeast, the authors attempt to infer the size of the nucleus that templates amyloid formation by polyQ. Further, through a series of sequence titrations, all studied using a single type of assay, the authors converge on an assertion stating that a single polyQ molecule is the nucleus for amyloid formation, that 12-residues make up the core of the nucleus, that it takes ca. 60 Qs in a row to unmask this nucleation potential, and that polyQ amyloid formation belongs to the same universality class as self-poisoned crystallization, which is the hallmark of crystallization from polymer melts formed by large, high molecular weight synthetic polymers. Unfortunately, the authors have decided to lean in hard on their assertions without a critical assessment of whether their findings stand up to scrutiny. If their findings are truly an intrinsic property of polyQ molecules, then their findings should be reconstituted in vitro. Unfortunately, careful and rigorous experiments in vitro show that there is a threshold concentration for forming fibrillar solids. This threshold concentration depends on the flanking sequence context on temperature and on solution conditions. The existence of a threshold concentration defies the expectation of a monomer nucleus. The findings disagree with in vitro data presented by Crick et al., and ignored by the authors. Please see: https://doi.org/10.1073/pnas.1320626110. These reports present data from very different assays, the importance of which was underscored first by Regina Murphy and colleagues. The work of Crick et al., provides a detailed thermodynamic framework - see the SI Appendix. This framework dove tails with theory and simulations of Zhang and Muthukumar, which explains exactly how a system like polyQ might work (https://doi.org/10.1063/1.3050295). The picture one paints is radically different from what the authors converge upon. One is inclined to lean toward data that are gleaned using multiple methods in vitro because the test tube does not have all the confounding effects of a cellular milieu, especially when it comes to focusing on sequence-intrinsic conformational transitions of a protein. In addition to concerns about the limitations of the DAmFRET method, which based on the work of the authors in their collaborative paper by Posey et al., are being stretched to the limit, there is the real possibility that the cellular milieu, unique to the system being studied, is enabling transitions that are not necessarily intrinsic to the sequence alone. A nod in this direction is the work of Marc Diamond, which showed that having stabilized the amyloid form of Tau through coacervation, there is a large barrier that limits the loss of amyloid-like structure for Tau. There may well be something similar going on with the polyQ system. If the authors could show that their data are achievable in vitro without anything but physiological buffers one would have more confidence in a model that appears to contradict basic physical principles of how homopolymers self-assemble. Absent such additional evidence, numerous statements seem to be too strong. There are also several claims that are difficult to understand or appreciate.

      Rebuttal to the perceived necessity of in vitro experiments

      The overarching concern of this reviewer and reviewing editor is whether in-cell assays can inform on sequence-intrinsic properties. We understand this concern. We believe however that the relative merit of in-cell assays is largely a matter of perspective. The truly sequence-intrinsic behavior of polyQ, i.e. in a vacuum, is less informative than the “sequence-intrinsic” behaviors of interest that emerge in the presence of extraneous molecules from the appropriate biological context. In vitro experiments typically include a tiny number of these -- water, ions, and sometimes a crowding agent meant to approximate everything else. Obviously missing are the myriad quinary interactions with other proteins that collectively round out the physiological solvent. The question is what experimental context best approximates that of a living human neuron under which the pathological sequence-dependent properties of polyQ manifest. We submit that a living yeast cell comes closer to that ideal than does buffer in a test tube.

      The reviewer’s statements that our findings must be validated in vitro ignores the fact -- stressed in our introduction -- that decades of in vitro work have not yet generated definitive evidence for or against any specific nucleus model. In addition to the above, one major problem concerns the large sizes of in vitro systems that obscure the effects of primary nucleation. For example, a typical in vitro experimental volume of e.g. 1.5 ml is over one billion-fold larger than the femtoliter volume of a cell. This means that any nucleation-limited kinetics of relevant amyloid formation are lost, and any alternative amyloid polymorphs that have a kinetic growth advantage -- even if they nucleate at only a fraction the rate of relevant amyloid -- will tend to dominate the system (Buell, 2017). Novel approaches are clearly needed to address these problems. We present such an approach, stretch it to the limit (as the reviewer notes) across multiple complementary experiments, and arrive at a novel finding that is fully and uniquely consistent with all of our own data as well as the collective prior literature.

      That the preceding considerations are collectively essential to understand relevant amyloid behavior is evident from recent cryoEM studies showing that in vitro-generated amyloid structures generally differ from those in patients (Arseni et al., 2022; Bansal et al., 2021; Radamaker et al., 2021; Schmidt et al., 2019; Schweighauser et al., 2020; Yang et al., 2022). This is highly relevant to the present discourse because each amyloid structure is thought to emanate from a different nucleating structure. This means that in vitro experiments have broadly missed the mark in terms of the relevant thermodynamic parameters that govern disease onset and progression. Note that the rules laid out via our studies are not only consistent with structural features of polyQ amyloid in cells, but also (as described in the discussion) explain why the endogenous structure of a physiologically relevant Q zipper amyloid differs from that of polyQ.

      A recent collaboration between the Morimoto and Knowles groups (Sinnige et al.) investigated the kinetics of aggregation by Q40-YFP expressed in C. elegans body wall muscle cells, using quantitative approaches that have been well established for in vitro amyloid-forming systems of the type favored by the reviewer. They calculate a reaction order of just 1.6, slightly higher than what would be expected for a monomeric nucleus but nevertheless fully consistent with our own conclusions when one accounts for the following two aspects of their approach. First, the polyQ tract in their construct is flanked by short poly-Histidine tracts on both sides. These charges very likely disfavor monomeric nucleation because all possible configurations of a four-stranded bundle position the beginning and end of the Q tract in close proximity, and Q40 is only just long enough to achieve monomeric nucleation in the absence of such destabilization. Second, the protein is fused to YFP, a weak homodimer (Landgraf et al., 2012; Snapp et al., 2003). With these two considerations, our model -- which was generated from polyQ tracts lacking flanking charges or an oligomeric fusion -- predicts that amyloid nucleation by their construct will occur more frequently as a dimer than a monomer. Indeed, their observed reaction order of 1.6 supports a predominantly dimeric nucleus. Like us and others, Sinnige et al. did not observe phase separation prior to amyloid formation. This is important because it not only argues against nucleation occurring in a condensate, it also suggests that the reaction order they calculated has not been limited by the concentration-buffering effect of phase separation.

      While we agree that our conclusions rest heavily on DAmFRET data (for good reason), we do provide supporting evidence from molecular dynamics simulations, SDD-AGE, and microscopy.

      To summarize, given the extreme limitations of in vitro experiments in this field, the breadth of our current study, and supporting findings from another lab using rigorous quantitative approaches, we feel that our claims are justified without in vitro data.

      Rebuttal to the perceived incompatibility of monomeric nucleation with the existence of a critical concentration for amyloid

      We appreciate that the concept of a monomeric nucleus can superficially appear inconsistent with the fact that crystalline solids such as polyQ amyloid have a saturating concentration, but this is only true if one neglects that polyQ amyloids are polymer crystals with intramolecular ordering. The perceived discrepancy is perhaps most easily dispelled by protein crystallography. Folded proteins form crystals. These crystals have critical concentrations, and the protein subunits within them each have intramolecular crystalline order (in the form of secondary structure). To extrapolate these familiar examples to our present finding with polyQ, one need only appreciate the now well-established phenomenon of secondary nucleation, whereby transient interactions of soluble species with the ordered species leads to their own ordering (Törnquist et al., 2018). Transience is important here because it implies that intramolecular ordering can in principle propagate even in solutions that are subsaturated with respect to bulk crystallization. This is possible in the present case because the pairing of sufficiently short beta strands (equivalent to “stems” in the polymer crystal literature) will be more stable intramolecularly than intermolecularly, due to the reduced entropic penalty of the former. Our elucidation that Q zipper ordering can occur with shorter strands intramolecularly than intermolecularly (Fig. S4C-D) demonstrates this fact. It is also evident from published descriptions of single molecule “crystals” formed in sufficiently dilute solutions of sufficiently long polymers (Hong et al., 2015; Keller, 1957; Lauritzen and Hoffman, 1960).

      In suggesting that a saturating concentration for amyloid rules out monomeric nucleation, the reviewer assumes that the Q zipper-containing monomer must be stable relative to the disordered ensemble. This is not inherent to our claim and in fact opposes the definition of a nucleus. The monomeric nucleating structure need not be more stable than the disordered state, and monomers may very well be disordered at equilibrium at low concentrations. To be clear, our claim requires that the Q zipper-containing monomer is both on pathway to amyloid and less stable than all subsequent species that are on pathway to amyloid. The former requirement is supported by our extensive mutational analysis. The latter requirement is supported by our atomistic simulations showing the Q zipper-containing monomer is stabilized by dimerization (see our 2021 preprint). Hence, requisite ordering in the nucleating monomer is stabilized by intermolecular interactions. We provide in Author response image 1 an illustration to clarify what we believe to be the discrepancy between our claim and the reviewer’s interpretation.

      Author response image 1.

      That the rate-limiting fluctuation for a crystalline phase can occur in a monomer can also be understood as a consequence of Ostwald’s rule of stages, which describes the general tendency of supersaturated solutes, including amyloid forming proteins (Chakraborty et al., 2023), to populate metastable phases en route to more stable phases (De Yoreo, 2022; Schmelzer and Abyzov, 2017). Our findings with polyQ are consistent with a general mechanism for Ostwald’s rule wherein the relative stabilities of competing polymorphs differ with the number of subunits (De Yoreo, 2022; Navrotsky, 2004). As illustrated in Fig. 6 of Navrotsky, a polymorph that is relatively stable at small particle sizes tends to give way to a polymorph that -- while initially unstable -- becomes more stable as the particles grow. The former is analogous to our early stage Q zipper composed of two short sheets with an intramolecular interface, while the latter is analogous to the later stage Q zipper composed of longer sheets with an intermolecular interface. Subunit addition stabilizes the latter more than the former, hence the initial Q zipper that is stabilized more by intra- than intermolecular interactions will mature with growth to one that is stabilized more by intermolecular interactions.

      We apologize to the Pappu group for neglecting to cite Crick et al. 2013 in the current preprint. Contrary to the reviewer’s assessment, however, we find that the conclusions of this valuable study do more to support than to refute our findings. Briefly, Crick et al. investigated the aggregation of synthetic Q30 and Q40 peptides in vitro, wherein fibrils assembled from high concentrations of peptide were demonstrated to have saturating concentrations in the low micromolar range. As explained above, this finding of a saturating concentration does not refute our results. More relevant to the present work are their findings that “oligomers” accumulated over an hours-long timespan in solutions that are subsaturated with respect to fibrils, and these oligomers themselves have (nanomolar) critical concentrations. The authors postulated that the oligomers result from liquid–liquid demixing of intrinsically disordered polyglutamine. However, phase separation by a peptide is expected to fix its concentration in both the solute and condensed phases, and, because disordered phase separation is inherently faster than amyloid formation, the postulated explanation removes the driving force for any amyloid phase with a critical solubility greater than that of the oligomers. In place of this interpretation that truly does appear to -- in the reviewer’s words -- “contradict basic physical principles of how homopolymers self-assemble”, we interpret these oligomers as evidence of our Q zipper-containing self-poisoned multimers, rounded as an inherent consequence of self-poisoning (Ungar et al., 2005), and likely akin to semicrystalline spherulites that have been observed in other polymer crystal and amyloid-forming systems (Crist and Schultz, 2016; Vetri and Foderà, 2015). That Crick et al. also observed the formation of a relatively labile amyloid phase when the reactions were started with 50 uM peptide is unsurprising in light of the aforementioned kinetic advantage that large reaction volumes can confer to labile polymorphs, and that high concentrations (in this case, orders of magnitude higher than the likely physiological concentration of polyQ (Wild et al., 2015)) can favor the formation of labile amyloid polymorphs (Ohhashi et al., 2010). Indeed, a contemporaneous study by the Wetzel group using very similar peptide constructs and polyQ lengths -- but beginning with lower concentrations -- found that the relevant saturating concentrations for amyloid lie below their limit of detection of 100 nM (Sahoo et al., 2014).

      Rebuttals to other critiques

      The reviewer states that we found nucleation potential to require 60 Qs in a row. Our data are collectively consistent with nucleation occurring at and above approximately 36 Qs, a point repeated in the paper. The reviewer may be referring to our statement, ”Sixty residues proved to be the optimum length to observe both the pre- and post-nucleated states of polyQ in single experiments”. The purpose of this statement is simply to describe the practical consideration that led us to use 60 Qs for the bulk of our assays. We do appreciate that the fraction of AmFRET-positive cells is very low for lengths just above the threshold, especially Q40. They are nevertheless highly significant (p = 0.004 in [PIN+] cells, one-tailed T-test), and we will modify the figure and text to clarify this.

      The reviewer characterizes self-poisoning as the hallmark of crystallization from polymer melts, which would be problematic for our conclusions if self-poisoning were limited to this non-physiological context. In fact the term was first used to describe crystallization from solution (Organ et al., 1989), wherein the phenomenon is more pronounced (Ungar et al., 2005).

      Reviewer #2 (Public Review):

      Numerous neurodegenerative diseases are thought to be driven by the aggregation of proteins into insoluble filaments known as "amyloids". Despite decades of research, the mechanism by which proteins convert from the soluble to insoluble state is poorly understood. In particular, the initial nucleation step is has proven especially elusive to both experiments and simulation. This is because the critical nucleus is thermodynamically unstable, and therefore, occurs too infrequently to directly observe. Furthermore, after nucleation much faster processes like growth and secondary nucleation dominate the kinetics, which makes it difficult to isolate the effects of the initial nucleation event. In this work Kandola et al. attempt to surmount these obstacles using individual yeast cells as microscopic reaction vessels. The large number of cells, and their small size, provides the statistics to separate the cells into pre- and post-nucleation populations, allowing them to obtain nucleation rates under physiological conditions. By systematically introducing mutations into the amyloid-forming polyglutamine core of huntingtin protein, they deduce the probable structure of the amyloid nucleus. This work shows that, despite the complexity of the cellular environment, the seemingly random effects of mutations can be understood with a relatively simple physical model. Furthermore, their model shows how amyloid nucleation and growth differ in significant ways, which provides testable hypotheses for probing how different steps in the aggregation pathway may lead to neurotoxicity.

      In this study Kandola et al. probe the nucleation barrier by observing a bimodal distribution of cells that contain aggregates; the cells containing aggregates have had a stochastic fluctuation allowing the proteins to surmount the barrier, while those without aggregates have yet to have a fluctuation of suitable size. The authors confirm this interpretation with the selective manipulation of the PIN gene, which provides an amyloid template that allows the system to skip the nucleation event.

      In simple systems lacking internal degrees of freedom (i.e., colloids or rigid molecules) the nucleation barrier comes from a significant entropic cost that comes from bringing molecules together. In large aggregates this entropic cost is balanced by attractive interactions between the particles, but small clusters are unable to form the extensive network of stabilizing contacts present in the larger aggregates. Therefore, the initial steps in nucleation incur an entropic cost without compensating attractive interactions (this imbalance can be described as a surface tension). When internal degrees of freedom are present, such as the conformational states of a polypeptide chain, there is an additional contribution to the barrier coming from the loss of conformational entropy required to the adopt aggregation-prone state(s). In such systems the clustering and conformational processes do not necessarily coincide, and a major challenge studying nucleation is to separate out these two contributions to the free energy barrier. Surprisingly, Kandola et al. find that the critical nucleus occurs within a single molecule. This means that the largest contribution to the barrier comes from the conformational entropy cost of adopting the beta-sheet state. Once this state is attained, additional molecules can be recruited with a much lower free energy barrier.

      There are several caveats that come with this result. First, the height of the nucleation barrier(s) comes from the relative strength of the entropic costs compared to the binding affinities. This balance determines how large a nascent nucleus must grow before it can form interactions comparable to a mature aggregate. In amyloid nuclei the first three beta strands form immature contacts consisting of either side chain or backbone contacts, whereas the fourth strand is the first that is able to form both kinds of contacts (as in a mature fibril). This study used relatively long polypeptides of 60 amino acids. This is greater than the 20-40 amino acids found in amyloid-forming molecules like ABeta or IAPP. As a result, Kandola et al.'s molecules are able to fold enough times to create four beta strands and generate mature contacts intramolecularly. The authors make the plausible claim that these intramolecular folds explain the well-known length threshold (L~35) observed in polyQ diseases. The intramolecular folds reduce the importance of clustering multiple molecules together and increase the importance of the conformational states. Similarly, manipulating the sequence or molecular concentrations will be expected to manipulate the relative magnitude of the binding affinities and the clustering entropy, which will shift the relative heights of the entropic barriers.

      The reviewer correctly notes that the majority of our manipulations were conducted with 60-residue long tracts (which corresponds to disease onset in early adulthood), and this length facilitates intramolecular nucleation. However, we also analyzed a length series of polyQ spanning the pathological threshold, as well as a synthetic sequence designed explicitly to test the model nucleus structure with a tract shorter than the pathological threshold, and both experiments corroborate our findings.

      The authors make an important point that the structure of the nucleus does not necessarily resemble that of the mature fibril. They find that the critical nucleus has a serpentine structure that is required by the need to form four beta strands to get the first mature contacts. However, this structure comes at a cost because residues in the hairpins cannot form strong backbone or zipper interactions. Mature fibrils offer a beta sheet template that allows incoming molecules to form mature contacts immediately. Thus, it is expected that the role of the serpentine nucleus is to template a more extended beta sheet structure that is found in mature fibrils.

      A second caveat of this work is the striking homogeneity of the nucleus structure they describe. This homogeneity is likely to be somewhat illusory. Homopolymers, like polyglutamine, have a discrete translational symmetry, which implies that the hairpins needed to form multiple beta sheets can occur at many places along the sequence. The asparagine residues introduced by the authors place limitations on where the hairpins can occur, and should be expected to increase structural homogeneity. Furthermore, the authors demonstrate that polyglutamine chains close to the minimum length of ~35 will have strict limitations on where the folds must occur in order to attain the required four beta strands.

      We are unsure how to interpret the above statements as a caveat. We agree that increasing sequence complexity will tend to increase homogeneity, but this is exactly the motivation of our approach. We explicitly set out to determine the minimal complexity sequence sufficient to specify the nucleating conformation, which we ultimately identified in terms of secondary and tertiary structure. We do not specify which parts of a long polyQ tract correspond to which parts of the structure, because, as the reviewer points out, they can occur at many places. Hence, depending on the length of the polyQ tract, the nucleus we describe may have any length of sequence connecting the strand elements. We do not think that the effects of N-residue placement can be interpreted as a confounding influence on hairpin position because the striking even-odd pattern we observe implicates the sides of beta strands rather than the lengths. Moreover, we observe this pattern regardless of the residue used (Gly, Ser, Ala, and His in addition to Asn).

      A novel result of this work is the observation of multiple concentration regimes in the nucleation rate. Specifically, they report a plateau-like regime at intermediate regimes in which the nucleation rate is insensitive to protein concentration. The authors attribute this effect to the "self-poisoning" phenomenon observed in growth of some crystals. This is a valid comparison because the homogeneity observed in NMR and crystallography structures of mature fibrils resemble a one-dimensional crystal. Furthermore, the typical elongation rate of amyloid fibrils (on the order of one molecule per second) is many orders of magnitude slower than the molecular collision rate (by factors of 10^6 or more), implying that the search for the beta-sheet state is very slow. This slow conformational search implies the presence of deep kinetic traps that would be prone to poisoning phenomena. However, the observation of poisoning in nucleation during nucleation is striking, particularly in consideration of the expected disorder and concentration sensitivity of the nucleus. Kandola et al.'s structural model of an ordered, intramolecular nucleus explains why the internal states responsible for poisoning are relevant in nucleation.

      We thank the reviewer for noting the novelty and plausibility of the self-poisoning connection. We would like to elaborate on our finding that self-poisoning inhibits nucleation (in addition to elongation), as this could prove confusing to some readers. While self-poisoning is claimed to inhibit primary nucleation in the polymer crystal literature (Ungar et al., 2005; Zhang et al., 2018), the semantics of “nucleation” in this context warrants clarification. Technically, the same structure can be considered a nucleus in one context but not in another. The Q zipper monomer, even if it is rate-limiting for amyloid formation at low concentrations (and is therefore the “nucleus”), is not necessarily rate-limiting when self-poisoned at high concentrations. Whether it comprises the nucleus in this case depends on the rates of Q zipper formation relative to subunit addition to the poisoned state. If the latter happens slower than Q zipper formation de novo, it can be said that self-poisoning inhibits nucleation, regardless of whether the Q zipper formed. We suspect this to be the mechanism by which preemptive oligomerization blocks nucleation in the case of polyQ, though other mechanisms may be possible.

      To achieve these results the authors used a novel approach involving a systematic series of simple sequences. This is significant because, while individual experiments showed seemingly random behavior, the randomness resolved into clear trends with the systematic approach. These trends provided clues to build a model and guide further experiments.

      Reviewer #3 (Public Review):

      Kandola et al. explore the important and difficult question regarding the initiating event that triggers (nucleates) amyloid fibril growth in glutamine-rich domains. The researchers use a fluorescence technique that they developed, dAMFRET, in a yeast system where they can manipulate the expression level over several orders of magnitude, and they can control the length of the polyglutamine domain as well as the insertion of interfering non-glutamine residues. Using flow cytometry, they can interrogate each of these yeast 'reactors' to test for self-assembly, as detected by FRET.

      In the introduction, the authors provide a fairly thorough yet succinct review of the relevant literature into the mechanisms of polyglutamine-mediated aggregation over the last two decades. The presentation as well as the illustrations in Figure 1A and 1B are difficult to understand, and unfortunately, there is no clear description of the experimental technique that would allow the reader to connect the hypothetical illustrations to the measurement outcomes. The authors do not explain what the FRET signal specifically indicates or what its intensity is correlated to. FRET measures distance between donor and acceptor, but can it be reliably taken as an indicator of a specific beta-sheet conformation and of amyloid? Does the signal increase with both nucleation and with elongation, and is the signal intensity the same if, e.g., there were 5 aggregates of 10 monomers each versus 50 monomeric nuclei? Is there a reason why the AmFRET signal intensity decreases at longer Q even though the number of cells with positive signal increases? Does the number of positive cells increase with time? The authors state later that 'non-amyloid containing cells lacked AmFRET altogether', but this seems to be a tautology - isn't the lack of AmFRET taken as a proof of lack of amyloid? Overall, a clearer description of the experimental method and what is actually measured (and validation of the quantitative interpretation of the FRET signal) would greatly assist the reader in understanding and interpreting the data.

      We believe the difficulty in understanding the illustrations in Figure 1A and 1B is inherent to the subject. We agree that elaborating how DAmFRET works would help the reader, and will add a few sentences to this end. Beyond this, we refer the reviewer and readers to our cited prior work describing the theory and interpretation of DAmFRET. Note that the y-axes of DAmFRET plots are not raw FRET but rather “AmFRET”, a ratio of FRET to total expression level. As explained thoroughly in our cited prior work, the discontinuity of AmFRET with expression level indicates that the high AmFRET-population formed via a disorder-to-order transition. When the query protein is predicted to be intrinsically disordered, the discontinuous transition to high AmFRET invariably (among hundreds of proteins tested in prior published and unpublished work) signifies amyloid formation as corroborated by SDD-AGE and tinctorial assays.

      When performed using standard flow cytometry as in the present study, every AmFRET measurement corresponds to a cell-wide average, and hence does not directly inform on the distribution of the protein between different stoichiometric species. As there is only one fluorophore per protein molecule, monomeric nuclei have no signal. DAmFRET can distinguish cells expressing monomers from stable dimers from higher order oligomers (see e.g. Venkatesan et al. 2019), and we are therefore quite confident that AmFRET values of zero correspond to cells in which a vast majority of the respective protein is not in homo-oligomeric species (i.e. is monomeric or in hetero-complexes with endogenous proteins). The exact value of AmFRET, even for species with the same stoichiometry, will depend both on the effect of their respective geometries on the proximity of mEos3.1 fluorophores, and on the fraction of protein molecules in the species. Hence, we only attempt to interpret the plateau values of AmFRET (where the fraction of protein in an assembled state approaches unity) as directly informing on structure, as we did in Fig. S3A.

      We believe that AmFRET decreases with longer polyQ because the mass fraction of fluorophore decreases in the aggregate, simply because the extra polypeptide takes up volume in the aggregate.

      Yes, the fraction of positive cells in a discontinuous DAmFRET plot does increase with time. However, given the more laborious data collection and derivation of nucleation kinetics in a system with ongoing translation, especially across hundreds of experiments with other variables, ours is a snapshot measurement to approximately derive the relative contributions of intra- and intermolecular fluctuations to the nucleation barrier, rather than the barrier’s magnitude.

      We will revise the tautological statement by removing “non-amyloid containing”.

      The authors demonstrate that their assay shows that the fraction of cells with AmFRET signal increases strongly with an increase in polyQ length, with a 'threshold around 50-60 glutamines. This roughly correlates with the Q-length dependence of disease. The experiments in which asparagine or other amino acids are inserted at variable positions in the glutamine repeat are creative and thorough, and the data along with the simulations provide compelling support for the proposed Q zipper model. The experiments shown in Figure 5 are strongly supportive of a model where formation of the beta-sheet nucleus is within a monomer. This is a potentially important result, as there are conflicting data in the literature as to whether the nucleus in polyQ is monomer.

      We thank the reviewer for these comments. We wish to clarify one important point, however, concerning the correlation of our data with the pathological length threshold. As we state in the first results section, “Our data recapitulated the pathologic threshold -- Q lengths 35 and shorter lacked AmFRET, indicating a failure to aggregate or even appreciably oligomerize, while Q lengths 40 and longer did acquire AmFRET in a length and concentration-dependent manner”. Hence, most of our experiments were conducted with 60Q not because it resembles the pathological threshold, but rather because it was most convenient for DAmFRET experiments.

      I did not find the argument, that their data shows the Q zipper grows in two dimensions, compelling; there are more direct experimental methods to answer this question. I was also confused by the section that Q zippers poison themselves. It would be easier for the reader to follow if the authors first presented their results without interpretation. The data seem more consistent with an argument that, at high concentrations, non-structured polyQ oligomers form which interfere with elongation into structured amyloid assemblies - but such oligomers would not be zippers.

      Self-poisoning is a widely observed and heavily studied phenomenon in polymer crystal physics, though it seems not yet to have entered the lexicon of amyloid biologists. We were new to this concept before it emerged as an extremely parsimonious explanation for our results. As described in the text, two pieces of evidence exclude the alternative mechanism suggested by the reviewer -- that non-structured oligomers form and subsequently engage and inhibit the template. Specifically, 1) inhibition occurs without any detectable FRET, even at high total protein concentration, indicating the species do not form in a concentration-dependent manner that would be expected of disordered oligomers; and 2) inhibition itself has strict sequence requirements that match those of Q zippers. Hence our data collectively suggest that inhibition is a consequence of the deposition of partially ordered molecules onto the templating surface.

      Although some speculation or hypothesizing is perfectly appropriate in the discussion, overall the authors stretch this beyond what can be supported by the results. A couple of examples: The conclusion that toxicity arises from 'self-poisoned polymer crystals' is not warranted, as there is no relevant data presented in this manuscript. The authors refer to findings 'that kinetically arrested aggregates emerge from the same nucleating event responsible for amyloid formation', but I cannot recall any evidence for this statement in the results section.

      We restricted any mention of toxicity to the introduction and a section in the discussion that is not worded as conclusive. Nevertheless, we will soften the subheading and text of the relevant section in the discussion to more clearly indicate the speculative nature of the statements.

      We stand by our statement 'that kinetically arrested aggregates emerge from the same nucleating event responsible for amyloid formation', as this follows directly from self-poisoning.

      Bibliography

      Arseni D, Hasegawa M, Murzin AG, Kametani F, Arai M, Yoshida M, Ryskeldi-Falcon B. 2022. Structure of pathological TDP-43 filaments from ALS with FTLD. Nature 601:139–143. doi:10.1038/s41586-021-04199-3

      Bansal A, Schmidt M, Rennegarbe M, Haupt C, Liberta F, Stecher S, Puscalau-Girtu I, Biedermann A, Fändrich M. 2021. AA amyloid fibrils from diseased tissue are structurally different from in vitro formed SAA fibrils. Nat Commun 12:1013. doi:10.1038/s41467-021-21129-z

      Buell AK. 2017. The Nucleation of Protein Aggregates - From Crystals to Amyloid Fibrils. Int Rev Cell Mol Biol 329:187–226. doi:10.1016/bs.ircmb.2016.08.014

      Chakraborty D, Straub JE, Thirumalai D. 2023. Energy landscapes of Aβ monomers are sculpted in accordance with Ostwald’s rule of stages. Sci Adv 9:eadd6921. doi:10.1126/sciadv.add6921 Crist B, Schultz JM. 2016. Polymer spherulites: A critical review. Prog Polym Sci 56:1–63. doi:10.1016/j.progpolymsci.2015.11.006

      De Yoreo JJ. 2022. Casting a bright light on Ostwald’s rule of stages. Proc Natl Acad Sci USA 119. doi:10.1073/pnas.2121661119

      Hong Y, Yuan S, Li Z, Ke Y, Nozaki K, Miyoshi T. 2015. Three-Dimensional Conformation of Folded Polymers in Single Crystals. Phys Rev Lett 115:168301. doi:10.1103/PhysRevLett.115.168301

      Keller A. 1957. A note on single crystals in polymers: Evidence for a folded chain configuration. Philosophical Magazine 2:1171–1175. doi:10.1080/14786435708242746

      Landgraf D, Okumus B, Chien P, Baker TA, Paulsson J. 2012. Segregation of molecules at cell division reveals native protein localization. Nat Methods 9:480–482. doi:10.1038/nmeth.1955

      Lauritzen JI, Hoffman JD. 1960. Theory of Formation of Polymer Crystals with Folded Chains in Dilute Solution. J Res Natl Bur Stand A Phys Chem 64A:73–102. doi:10.6028/jres.064A.007

      Navrotsky A. 2004. Energetic clues to pathways to biomineralization: precursors, clusters, and nanoparticles. Proc Natl Acad Sci USA 101:12096–12101. doi:10.1073/pnas.0404778101

      Ohhashi Y, Ito K, Toyama BH, Weissman JS, Tanaka M. 2010. Differences in prion strain conformations result from non-native interactions in a nucleus. Nat Chem Biol 6:225–230. doi:10.1038/nchembio.306

      Organ SJ, Ungar G, Keller A. 1989. Rate minimum in solution crystallization of long paraffins. Macromolecules 22:1995–2000. doi:10.1021/ma00194a078

      Radamaker L, Baur J, Huhn S, Haupt C, Hegenbart U, Schönland S, Bansal A, Schmidt M, Fändrich M. 2021. Cryo-EM reveals structural breaks in a patient-derived amyloid fibril from systemic AL amyloidosis. Nat Commun 12:875. doi:10.1038/s41467-021-21126-2

      Sahoo B, Singer D, Kodali R, Zuchner T, Wetzel R. 2014. Aggregation behavior of chemically synthesized, full-length huntingtin exon1. Biochemistry 53:3897–3907. doi:10.1021/bi500300c

      Schmelzer JWP, Abyzov AS. 2017. How do crystals nucleate and grow: ostwald’s rule of stages and beyond In: Šesták J, Hubík P, Mareš JJ, editors. Thermal Physics and Thermal Analysis, Hot Topics in Thermal Analysis and Calorimetry. Cham: Springer International Publishing. pp. 195–211. doi:10.1007/978-3-319-45899-1_9

      Schmidt M, Wiese S, Adak V, Engler J, Agarwal S, Fritz G, Westermark P, Zacharias M, Fändrich M. 2019. Cryo-EM structure of a transthyretin-derived amyloid fibril from a patient with hereditary ATTR amyloidosis. Nat Commun 10:5008. doi:10.1038/s41467-019-13038-z

      Schweighauser M, Shi Y, Tarutani A, Kametani F, Murzin AG, Ghetti B, Matsubara T, Tomita T, Ando T, Hasegawa K, Murayama S, Yoshida M, Hasegawa M, Scheres SHW, Goedert M. 2020. Structures of α-synuclein filaments from multiple system atrophy. Nature 585:464–469. doi:10.1038/s41586-020-2317-6

      Snapp EL, Hegde RS, Francolini M, Lombardo F, Colombo S, Pedrazzini E, Borgese N, Lippincott-Schwartz J. 2003. Formation of stacked ER cisternae by low affinity protein interactions. J Cell Biol 163:257–269. doi:10.1083/jcb.200306020

      Törnquist M, Michaels TCT, Sanagavarapu K, Yang X, Meisl G, Cohen SIA, Knowles TPJ, Linse S. 2018. Secondary nucleation in amyloid formation. Chem Commun 54:8667–8684. doi:10.1039/c8cc02204f

      Ungar G, Putra EGR, de Silva DSM, Shcherbina MA, Waddon AJ. 2005. The Effect of Self-Poisoning on Crystal Morphology and Growth Rates In: Allegra G, editor. Interphases and Mesophases in Polymer Crystallization I, Advances in Polymer Science. Berlin, Heidelberg: Springer Berlin Heidelberg. pp. 45–87. doi:10.1007/b107232

      Vetri V, Foderà V. 2015. The route to protein aggregate superstructures: Particulates and amyloid-like spherulites. FEBS Lett 589:2448–2463. doi:10.1016/j.febslet.2015.07.006

      Wild EJ, Boggio R, Langbehn D, Robertson N, Haider S, Miller JRC, Zetterberg H, Leavitt BR, Kuhn R, Tabrizi SJ, Macdonald D, Weiss A. 2015. Quantification of mutant huntingtin protein in cerebrospinal fluid from Huntington’s disease patients. The Journal of Clinical Investigation.

      Yang Y, Arseni D, Zhang W, Huang M, Lövestam S, Schweighauser M, Kotecha A, Murzin AG, Peak-Chew SY, Macdonald J, Lavenir I, Garringer HJ, Gelpi E, Newell KL, Kovacs GG, Vidal R, Ghetti B, Ryskeldi-Falcon B, Scheres SHW, Goedert M. 2022. Cryo-EM structures of amyloid-β 42 filaments from human brains. Science 375:167–172. doi:10.1126/science.abm7285

      Zhang X, Zhang W, Wagener KB, Boz E, Alamo RG. 2018. Effect of Self-Poisoning on Crystallization Kinetics of Dimorphic Precision Polyethylenes with Bromine. Macromolecules 51:1386–1397. doi:10.1021/acs.macromol.7b02745

    1. Author response:

      The following is the authors’ response to the original reviews.

      Public Reviews:

      Reviewer 1:

      (1) In Figure 1, it is curious that the authors only chose E.coli and staphytlococcus sciuri to test the induction of Chi3l1. What about other bacteria? Why does only E.coli but not staphytlococcus sciuri induce chi3l1 production? It does not prove that the gut microbiome induces the expression of Chi3l1. If it is the effect of LPS, does it trigger a cell death response or inflammatory responses that are known to induce chi3l1 production? What is the role of peptidoglycan in this experiment? Also, it is recommended to change WT to SPF in the figure and text, as no genetic manipulation was involved in this figure.

      Thank you for your valuable feedback and insightful suggestions. In our study, we tried to identify bacteria from murine gut contents and feces using 16S sequencing. However, only E. coli and Staphylococcus sciuri were identified (Figure 1D). Consequently, our experiments were limited to these two bacterial strains. While we have not tested other bacteria, our data suggest that not all bacteria can induce the expression of Chi3l1. Given that E. coli is Gram-negative and Staphylococcus sciuri is Gram-positive, we hypothesized that the difference in their ability to induce Chi3l1 expression might be due to variations between Gram-negative and Gram-positive bacteria, such as the presence of lipopolysaccharides (LPS).

      To test this hypothesis, we used LPS to induce Chi3l1 expression. Consistent with our hypothesis, LPS successfully induced Chi3l1 expression (Figure 1F&G). Additionally, we observed that Chi3l1 expression is significantly upregulated in specific pathogen-free (SPF) mice compared to germ-free mice (Figure 1A), demonstrating that the gut microbiome induces the expression of Chi3l1.

      Although we have not examined cell death or inflammatory responses, the protective role of Chi3l1 shown in Figure 5 suggests that any such responses would be mild and negligible. Regarding the role of peptidoglycan in the induction of Chi3l1 expression in DLD-1 cells, we have not yet explored this aspect. However, we agree with your suggestion that it would be worthwhile to investigate this in future experiments.

      We have also made the suggested modifications to the labeling (Figure 1A) and the clarification in the revised manuscript accordingly (page 3, Line 95-96; Line 102-106).

      Thank you again for your constructive feedback.

      (2) In Figure 2, the binding between Chi3l1 and PGN needs better characterization, regarding the affinity and how it compares with the binding between Chi3l1 and chitin. More importantly, it is unclear how this interaction could facilitate the colonization of gram-positive bacteria.

      Thank you for your insightful suggestions and we have performed the suggested experiments and included the results in the revised manuscript (Figure 2E-G, page 3-4, Line 132-146).

      Our results indicate that Chi3l1 interact with PGN in a dose-increase manner (Figure 2E). In contrast, the binding between Chi3l1 and chitin did not exhibit dose dependency (Figure 2E). These findings suggest a specific and distinct binding mechanism for Chi3l1 with PGN compared to chitin.

      We conducted DLD-1 cell-bacteria adhesion experiments, using GlmM mutant (PGN synthesis mutant) and K12 (wild-type) bacteria to test their adhesion capabilities. The results showed that the adhesion ability of the GlmM mutant to cells significantly decreased (Figure 2F). Additionally, after knocking down Chi3l1 in DLD-1 cells, we observed a decreased bacterial adhesion (Figure 2G). These findings suggest that Chi3l1 and PGN interaction plays a crucial role in bacterial adhesion.

      (3) In Figure 3, the abundance of furmicutes and other gram-positive species is lower in the knockout mice. What is the rationale for choosing lactobacillus in the following transfer experiments?

      We appreciate your thorough review. Among the Gram-positive bacteria that we have sequenced and analyzed, Lactobacillus occupies the largest proportion. Given the significant presence and established benefits of Lactobacillus, we chose it for the subsequent transfer experiments to leverage its known properties and availability, thereby ensuring the robustness and reproducibility of our findings.This is supported by the study referenced below.

      Lamas B, Richard ML, Leducq V, Pham HP, Michel ML, Da Costa G, Bridonneau C, Jegou S, Hoffmann TW, Natividad JM, Brot L, Taleb S, Couturier-Maillard A, Nion-Larmurier I, Merabtene F, Seksik P, Bourrier A, Cosnes J, Ryffel B, Beaugerie L, Launay JM, Langella P, Xavier RJ, Sokol H. CARD9 impacts colitis by altering gut microbiota metabolism of tryptophan into aryl hydrocarbon receptor ligands. Nat Med. 2016 Jun;22(6):598-605. doi: 10.1038/nm.4102. Epub 2016 May 9. PMID: 27158904; PMCID: PMC5087285.

      (4) FDAA-labeled E. faecalis colonization is decreased in the knockouts. Is it specific for E. faecalis, or it is generally true for all gram-positive bacteria? What about the colonization of gram-negative bacteria?

      Thank you for your insightful suggestions and we have investigated the colonization of gram-negative bacteria, OP50-mcherry (a strain of E.coli that express mCherry) and included the results in the updated manuscript (Supplementary Figure 3B, page 5, Line 197-200). We performed rectal injection of both wildtype and Chi11-/- mice with mCherry-OP50, and found that Chi11-/- mice had much higher colonization of E. coli compared to wildtype mice.

      (5) In Figure 5, the fact that FMT did not completely rescue the phenotype may point to the role of host cells in the processes. The reason that lactobacillus transfer did completely rescue the phenotypes could be due to the overwhelming protective role of lactobacillus itself, as the experiments were missing villin-cre mice transferred with lactobacillus.

      Thank you for your valuable feedback and thorough review. In our study, pretreatment with antibiotics in mice to eliminate gut microbiota demonstrated that IEC∆Chil1 mice exhibited a milder colitis phenotype (Supplementary Figure 4). This suggests that Chi3l1-expressing host cells are likely to play a detrimental role in colitis. Consequently, the failure of FMT to completely rescue the phenotype is likely due to the incomplete preservation of bacteria in the feces during the transfer experiment.

      We agree with your assessment of the protective role of lactobacillus. This also explains the significant difference in colitis phenotype between Villin-cre and IEC∆Chil1 mice (Figure 5B-E), as lactobacillus levels are significantly lower in IEC∆Chil1 mice (Figure 4F). Given the severity of colitis in Villin-cre mice at 7 days post-DSS, even if lactobacillus were transferred back to these mice, it is unlikely to result in a significant improvement.

      (6) Conflicting literature demonstrating the detrimental roles of Chi3l1 in mouse IBD model needs to be acknowledged and discussed.

      Thank you for your insightful suggestions and we have included additional discussions in the revised manuscript (page 6-7, Line 258-274).

      Reviewer #2 (Public Review):

      (1) Images are of great quality but lack proper quantification and statistical analysis. Statements such as "substantial increase of Chi3l1 expression in SPF mice" (Fig.1A), "reduced levels of Firmicutes in the colon lumen of IEC ∆ Chil1" (Fig.3F), "Chil1-/- had much lower colonization of E.faecalis" (Fig.4G), or "deletion of Chi3l1 significantly reduced mucus layer thickness" (Supplemental Figure 3A-B) are subjective. Since many conclusions were based on imaging data, the authors must provide reliable measures for comparison between conditions, as long as possible, such as fluorescence intensity, area, density, etc, as well as plots and statistical analysis.

      Thank you for your insightful suggestions and we have performed the suggested statistical analysis on most of the figures and included the analysis in the revised manuscript (Figure 1A, Figure 3E&F, Supplementary Figure 3B&C).Given large quantity of dietary fiber intertwined with bacteria, it is challenging to make a reliable quantification of bacteria in Figure 4G. However, it is easy to distinguish bacteria from dietary fiber under the microscope. We have exclusively analyzed gut sections from six mice in each group, and the results are consistent between the two groups.

      (2) In the fecal/Lactobacillus transplantation experiments, oral gavage of Lactobacillus to IECChil1 mice ameliorated the colitis phenotype, by preventing colon length reduction, weight loss, and colon inflammation. These findings seem to go against the notion that Chi3l1 is necessary for the colonization of Lactobacillus in the intestinal mucosa. The authors could speculate on how Lactobacillus administration is still beneficial in the absence of Chi3l1. Perhaps, additional data showing the localization of the orally administered bacteria in the gut of Chi3l1 deficient mice would clarify whether Lactobacillus are more successfully colonizing other regions of the gut, but not the mucus layer. Alternatively, later time points of 2% DSS challenge, after Lactobacillus transplantation, would suggest whether the gut colonization by Lactobacillus and therefore the milder colitis phenotype, is sustained for longer periods in the absence of Chi3l1.

      Thank you for your thorough review and insightful suggestions. Since we pretreated mice with antibiotics, the intestinal mucus layer is likely damaged according to a previous study (PMID: 37097253). Therefore, gavaged Lactobacillus cannot colonize in the mucus layer. Moreover, existing studies have shown that the protective effect of Lactobacillus is mainly derived from its metabolites or thallus components, rather than the living bacteria itself (PMID: 36419205, PMID: 27516254).

      Zhan M, Liang X, Chen J, Yang X, Han Y, Zhao C, Xiao J, Cao Y, Xiao H, Song M. Dietary 5-demethylnobiletin prevents antibiotic-associated dysbiosis of gut microbiota and damage to the colonic barrier. Food Funct. 2023 May 11;14(9):4414-4429. doi: 10.1039/d3fo00516j. PMID: 37097253.

      Montgomery TL, Eckstrom K, Lile KH, Caldwell S, Heney ER, Lahue KG, D'Alessandro A, Wargo MJ, Krementsov DN. Lactobacillus reuteri tryptophan metabolism promotes host susceptibility to CNS autoimmunity. Microbiome. 2022 Nov 23;10(1):198. doi: 10.1186/s40168-022-01408-7. PMID: 36419205.

      Piermaría J, Bengoechea C, Abraham AG, Guerrero A. Shear and extensional properties of kefiran. Carbohydr Polym. 2016 Nov 5;152:97-104. doi: 10.1016/j.carbpol.2016.06.067. Epub 2016 Jun 23. PMID: 27516254.

      Reviewer #3 (Public Review):

      The claim that mucus-associated Ch3l1 controls colonization of beneficial Gram-positive species within the mucus is not conclusive. The study should take into account recent discoveries on the nature of mucus in the colon, namely its mobile fecal association and complex structure based on two distinct mucus barrier layers coming from proximal and distal parts of the colon (PMID: ). This impacts the interpretation of how and where Ch3l1 is expressed and gets into the mucus to promote colonization. It also impacts their conclusions because the authors compare fecal vs. tissue mucus, but most of the mucus would be attached to the feces. Of the mucus that was claimed to be isolated from the WT and IEC Ch3l1 KO, this was not biochemically verified. Such verification (e.g. through Western blot) would increase confidence in the data presented. Further, the study relies upon relative microbial profiling, which can mask absolute numbers, making the claim of reduced overall Gram-positive species in mice lacking Ch3l1 unproven. It would be beneficial to show more quantitative approaches (e.g. Quantitative Microbial Profiling, QMP) to provide more definitive conclusions on the impact of Ch3l1 loss on Gram+ microbes.

      You raise an excellent point about the data interpretation, and we appreciate your insightful suggestions. We have included the discussion regarding the recent discoveries in the revised manuscript (page 7-8, Line 304-312). According to the recent discovery, the mucus in the proximal colon forms a primary encapsulation barrier around fecal material, while the mucus in the distal colon forms a secondary barrier. Our findings indicate that Chi3l1 is expressed throughout the entire colon, including the proximal, middle, and distal sections (See Author response image 1 below, P.S. Chi3l1 detection in colon presented in the manuscript are from the middle section). This suggests that Chi3l1 likely promotes bacterial colonization across the entire colon. Despite most mucus being expelled with feces, the

      constant production of mucus and the minimal presence of Chi3l1 in feces (Figure 4C) indicate that Chi3l1 continuously plays a role in promoting the colonization of microbiota.

      Author response image 1.

      Chi3l1 express in the proximal and distal colon. Immunofluoresence staining on proximal and distal colon sections to detect Chi3l1 (Red) expression. Nuclei were detected with DAPI (blue). Scale bars, 50um.

      Given the isolation method of the mucus layer, we followed the paper titled "The Antibacterial Lectin RegIIIγ Promotes the Spatial Segregation of Microbiota and Host in the Intestine" (PMID: 21998396). Although we did not find a suitable marker representative of the mucus layer for western blotting, we performed protein mass spectrometry on the isolated mucus layers and analyzed the data by comparing it with established research ("Proteomic Analyses of the Two Mucus Layers of the Colon Barrier Reveal That Their Main Component, the Muc2 Mucin, Is Strongly Bound to the Fcgbp Protein," PMID: 19432394). Our data showed a high degree of overlap with the proteins identified in established studies (see Author response image 2 below).

      Author response image 2.

      Comparison of mucus layer proteins identified by mass spectrometry between Our team and the Hansson team Mucus layer proteins identified by mass spectrometry between our team and the Hansson team (PMID: 19432394) are compared.

      Due to a lack of expertise, it has been challenging for us to perform reliable QMP experiments. However, since QMP involves qPCR combined with bacterial sequencing, we conducted 16S rRNA sequencing and confirmed the quantity of certain bacteria by qPCR (revised manuscript, Figure 3B, H, Figure 4E, F, Supplementary Figure 3A). Therefore, our data is reliable to some extent.

      Other weaknesses lie in the execution of the aims, leaving many claims incompletely substantiated. For example, much of the imaging data is challenging for the reader to interpret due to it being unfocused, too low of magnification, not including the correct control, and not comparing the same regions of tissues among different in vivo study groups. Statistical rigor could be better demonstrated, particularly when making claims based on imaging data. These are often presented as single images without any statistics (i.e. analysis of multiple images and biological replicates). These images include the LTA signal differences, FISH images, Enterococcus colonization, and mucus thickness.

      Thank you for your thorough review and insightful suggestions. We have performed the recommended statistical analysis on most of the figures and included the analysis in the revised manuscript (Figure 1A, Figure 3E&F, Supplementary Figure 3B&C). We have also added arrows in Figure 2B to make the figure easier to understand. Additionally, we repeated some key experiments to show the same regions of tissues among different groups. We will upload higher resolution figures during the revision. Thank you again for your constructive feedback.

      Recommendations for the authors:

      Reviewer #1 (Recommendations For The Authors):

      It is recommended to change WT to SPF in the figure and text, as no genetic manipulation was involved in Figure 1.

      Thank you for your insightful suggestion. We have also made the suggested modifications to the labeling (revised manuscript, Figure 1A).

      Reviewer #2 (Recommendations For The Authors):

      The manuscript is well-written, but it would benefit from a critical reading to correct some typos and small grammar issues. Histological and IF images would be more informative if they contained arrows and labels guiding the reader's attention to what the authors want to show. More details about the structures shown in the figures should be included in the legends.

      Thank you for your thorough review and insightful suggestions. We have revised the manuscript to correct noticeable typos and grammar issues. Arrows have been added to Figure 2A&B to make the figures easier to understand. Additionally, we have included a detailed description of the structural similarities and differences between chitin and peptidoglycan in the figure legend ( revised manuscript, page 19, line 730-733).

      Minor points:

      • Page 1, line 36: Please correct "mice models" to "mouse models".

      Thank you for your insightful suggestion and we have made the suggested correction in the revised manuscript (page 1, line 41).

      • Page 3, line 110: "by comparing the structure of chitin with that of peptidoglycan (PGN), a component of bacterial cells walls, we observed that they have similar structures (Fig.2A)". Although both structures are shown side-by-side, no similarities are mentioned or highlighted in the text, figure, or legend.

      Thank you for your insightful suggestion and we have included a detailed description of the structural similarities and differences between chitin and peptidoglycan in the figure legend (revised manuscript, page 19, line 730-733).

      • Fig.5C and Fig.5G: y axis brings "weight (%)". I believe the authors mean "weight change (%)"?

      We agrees with your suggestion and has corrected the labeling according to your suggestion (revised manuscript, Figure 5C and G)

      • Page 8: Genotyping method is described as a protocol. Please modify it.

      Thank you for your constructive suggestion and we have modified the genotyping method in the revised manuscript (page 8, line 339-349)

      • Please expand on the term "scaffold model" used in the abstract and discussion.

      Thank you for your thorough review. In this model, Chi3l1 acts as a key component of the scaffold. By binding to bacterial cell wall components like peptidoglycan, Chi3l1 helps anchor and organize bacteria within the mucus layer. This interaction facilitates the colonization of beneficial bacteria such as Lactobacillus, which are important for gut health. We included more descriptions regarding scaffold model in the revised manuscript (page 6, line 248-250)

      • Discussion session often recapitulates results description, which makes the text repetitive.

      Thank you for your constructive suggestion and we have removed unnecessary results description in the discussion session in the revised manuscript.

      Reviewer #3 (Recommendations For The Authors):

      Major comments

      (1) Figure 1A. The staining is very faint, and hard to see. The reader cannot be certain those are Ch311-positive cells. Higher Mag is needed.

      Thank you for your insightful suggestion and we have included the higher resolution figures in the revised manuscript Figure 1A.

      (2) The mucus is produced largely by the proximal colon, is adherent to the feces, and mobile with the feces (PMID: 33093110). Therefore it is important to determine where the Ch311 is being expressed to be released into the lumen. Further Ch3l1 expression studies are needed to be done in both proximal and distal colon.

      Thank you for your thorough review and insightful suggestions. We have addressed this part in our public review. Additionally, we agree with your suggestions and will conduct further studies on Chi3l1 expression in both the proximal and distal colon.

      (3) Figure 1B. The image is out of focus for the Ileum, and the DAPI signal needs to be brought up for the colon. Which part of the colon is this? The UEA1+ cells do not really look like goblet cells. A better image with clearer goblet cells is needed.

      Thank you for your constructive suggestions. In the revised manuscript, we have included higher-resolution images (Figure 1B). The middle colon (approximately 3 to 4 cm distal from the cecum) was harvested for staining. In addition to UEA-1, we utilized anti-MUC2 antibody to label goblet cells in this colon segment (see Author response image 3 below). The patterns of goblet cells identified by UEA-1 or MUC2 antibodies are similar. The UEA-1-positive cells shown in Figure 1B are presumed to be goblet cells.

      Author response image 3.

      Goblet Cell Distribution in the Middle Colon. Goblet cells in the middle segment of the colon (approximately 3 to 4 cm distal from the cecum) were detected using immunofluorescence with antibodies against UEA-1 (green) and MUC2 (red). Scale bar=50μm. Representative images are shown from three mice individually stained for each antibody.

      (4) Figure 1G. There needs to be some counterstain or contrast imaging to show evidence that cells are present in the untreated sample.

      Thank you for your insightful suggestions. We have annotated the cells present in the untreated sample based on the overexposure in the revised manuscript (Figure 1G).

      (5) Figure 3B. Is this absolute quantification? How were the data normalized to allow comparison of microbial loads?

      Thank you for your thorough review. Figure 3B presents absolute quantification data based on the methodology described in the paper titled "The Antibacterial Lectin RegIIIγ Promotes the Spatial Segregation of Microbiota and Host in the Intestine" (PMID: 21998396). Briefly, we amplified a short segment (179 bp) of the 16S rRNA gene using conserved 16S rRNA-specific primers and OP50 (a strain of E. coli) as the template. After gel extraction and concentration measurement, the PCR products were diluted to gradient concentrations (0.16, 0.32, 0.64, 1.28, 2.56, 5.12, 10.24, 20.48 pg/µl). These gradient concentrations were used as templates for qPCR to generate a standard curve based on Ct values and bacterial concentration. The standard curve is used to calculate bacterial concentration in the samples. The data presented in Figure 3B represent the weight of bacteria/milligram sample, calculated as (bacterial concentration x bacterial volume) / (weight of feces or gut content).

      (6) Figure 3D. The major case is made for a dramatic reduction in Gram+ species, but Figure 1D does not show a dramatic change. Is this difference significant?

      Thank you for your thorough review. We don’t think we are clear about your question. However, there was no significant difference in Figure 3D. The dramatic reduction in Gram+ species are made based on the LTA, Firmicutes FISH, individual species comparison between WT and KO mice, bacterial QPCR results together (Figure 3E-H).

      (7) Figures 3E and 3F. These stainings are alone not convincing of reduced Gram+ in the KOs. Some stats are required for these images. An independent complementary method is also needed to quantify these with statistics since this data is so central to the study's conclusions.

      Thank you for your constructive suggestions. We have included statistical analysis in the revised manuscript (Figure 3E and F). Given large quantity of dietary fiber intertwined with bacteria, it is challenging to make a reliable quantification of bacteria in Figure 3E. However, it is easy to distinguish bacteria from dietary fiber under the microscope. We have exclusively analyzed gut sections from six mice in each group, and the results are consistent with the Firmicutes FISH results. Complementary method such as bacterial QPCR have been employed to quantify these (Figure 4E, F). Due to a lack of expertise, it has been challenging for us to perform reliable QMP experiments.

      (8) Figure 3G. To make quantitative conclusions, the authors need to do quantitative microbial profiling (QMP) of the microbiota. Relative abundance masks absolute numbers, which could be increased. There are qPCR-based QMP platforms the authors could use (PMID: PMIDs: 31940382, 33763385).

      Thank you for your constructive suggestions. Due to a lack of expertise, it has been challenging for us to perform reliable QMP experiments. However, since QMP involves qPCR combined with bacterial sequencing, we conducted 16S rRNA sequencing and confirmed the quantity of certain bacteria by qPCR (revised manuscript, Figure 3B, H, Figure 4E, F, Supplementary Figure 3A). In addition to the original bacterial qPCR data presented in the manuscript, we included another bacterial species, Turicibater. Consistent with the 16S rRNA sequencing analysis data, qPCR results showed that Turicibacter was more abundant in IECΔChil1 mice than Villin-cre mice (revised manuscript, supplementary Figure 3A, page 4, line 171-173) Therefore, our data is reliable to some extent.

      (9) Figure 4B. The data nicely shows Ch3l1 in mucus. However, no data supports the authors' main claim Ch3h1 binds Gram-positive bacteria in situ. Dual staining of Ch3l1 with Firmicutes probe would be supportive to show this interaction is happening in vivo.

      You raise an excellent point, and we agree with your suggestion that we should confirm Chi3l1 binding to Gram-positive bacteria in situ. During the study, we attempted dual staining of Chi3l1 with a universal bacterial 16S FISH probe several times, but we were unsuccessful. Despite various optimizations of the protocol, we were only able to detect bacteria, not Chi3l1. It appears that the antibody is not suitable for this method.

      (10) Figures 4D - F. Because mucus is associated with feces (PMID: ), the data with feces likely contains both Muc2/mucus and Feces. Therefore, it is unclear what the "mucus" is referring to in these figures. To support the authors' conclusions, there needs to be some validation that mucus was purified in the assays. This must be confirmed at a minimum by PAS staining on SDS PAGE gel (should be very high molecular weight) or Western blot with UEA lectin.

      Thank you for your insightful suggestions. As mentioned in the public review, the mucus layer was isolated following the protocol described in the paper titled "The Antibacterial Lectin RegIIIγ Promotes the Spatial Segregation of Microbiota and Host in the Intestine" (PMID: 21998396). Briefly, after harvesting the middle colon from the mice, we cut open the colon longitudinally. After removing the gut contents, the lumen was vigorously rinsed in PBS while holding one end with forceps. The pellet obtained after centrifuging the rinsate was used as our mucus sample. Fresh feces were collected immediately after the mice defecated in a new, empty cage. We performed Western blot analysis to detect UEA lectin but were unsuccessful.

      However, as noted in the public review, we conducted protein mass spectrometry on the isolated mucus layers and analyzed the data by comparing it with established research ("Proteomic Analyses of the Two Mucus Layers of the Colon Barrier Reveal That Their Main Component, the Muc2 Mucin, Is Strongly Bound to the Fcgbp Protein," PMID: 19432394). Our data showed a high degree of overlap with the proteins identified in these established studies.

      (11) Figure 4E/F: The units of measurement are in pg/cm2, implying picogram per area. Can the authors please explain what this unit is referring to?

      We are grateful for your thorough review. The unit pg/cm ² represents picograms per square centimeter. Figures 4E and 4F present absolute quantification data based on the methodology described in the paper titled "The Antibacterial Lectin RegIIIγ Promotes the Spatial Segregation of Microbiota and Host in the Intestine" (PMID: 21998396). Briefly, we harvested a 3x0.5 cm section of colon and a 9x0.4 cm section of ileum. And then we collected the mucus layer as previously described (responses to question 10). We measured bacterial concentration as described in response to question 5 using the equation (y = -1.53ln(x) + 13.581), where x represents the bacterial concentration and y represents the Ct value. After obtaining the bacterial concentration, we multiplied it by the volume of the rinsate and divided it by the area to obtain the values for pg/cm² used in the figures.

      (12) Figure 5E. Normal tissues appear to be from different colon regions from colitis tissues: the "Normal" looks like the proximal colon, while "Colitis" looks like the Distal colon. They cannot be directly compared.

      Thank you for your insightful suggestion. We have now included the updated image in the revised manuscript as Figure 5E to compare the same region of the colons.

      (13) Similarly, in Figure 5I it appears different colon regions are being compared between groups: Proximal colon in the bottom panels, and distal in the top panels. Since the proximal colon is less damaged by DSS, this data could be misleading.

      Thank you for your insightful suggestion. We have now included the updated image in the revised manuscript as Figure 5I to compare the same region of the colons.

      (14) In the DSS studies, are the VillinCre and IEC Chit3l1 mice co-housed littermates?

      Thank you for your insightful suggestion. In the DSS studies, the Villin-Cre and IECΔChil1 mice are not co-housed littermates. However, they are derived from the same lineage and are housed in the same rack within the same room of the animal facility.

      (15) Supplementary Figure 3: Mucus thickness images; are they representative? Stats are needed on multiple mice to support the claim that the mucus is thinner.

      Thank you for your insightful suggestion. The images are representative of 4 mice each group. We have now included the statistical analysis in the revised manuscript Supplementary Figure 3C&D.

      Minor

      (1) Introduction: Reference to "mucosal layer": "Mucosal" and "Mucus" are different things. "Mucosal" refers to the epithelium, lamina propria, and muscularis mucosa. "Mucus" refers to the secreted mucus gel, the focus of the authors' study. Therefore, the statement "mucosal layer" is not proper. "Mucosal layer" should be changed to "mucus layer."

      Thank you for your constructive suggestions and we have learned a lot from it. We have made the replacement of “mucosal layer” to “mucus layer in the revised manuscript.

      (2) Line 366 and related lines: Feces cannot be "dissolved". "Resuspended" is a better term.

      Thank you for your constructive suggestion and we have made the changes of “dissolved” to “resuspended” in the revised manuscript.

      (3) Lines 36-37 and 43-44 are redundant to each other.

      Thank you for your constructive suggestion and we have removed the lines 36-37 in the revised manuscript.

    1. Author response:

      The following is the authors’ response to the current reviews.

      Reviewer #1:

      Summary:

      The authors study age-related changes in the excitability and firing properties of sympathetic neurons, which they ascribe to age-related changes in the expression of KCNQ (Kv7, "M-type") K+ currents in rodent sympathetic neurons, whose regulation by GPCRs has been most thoroughly studied for over 40 years.

      Strengths:

      The strengths include the rigor of the current-clamp and voltage-clamp experiments and the lovely, crisp presentation of the data, The separation of neurons into tonic, phasic and adapting classes is also interesting, and informative. The ability to successfully isolate and dissociate peripheral ganglia from such older animals is also quite rare and commendable! There is much useful detail here.

      Thank you for recognizing the effort we put on presenting the data and analyzing the neuronal populations. I also believe the ability to isolate neurons from old animals is worth communicating to the scientific community.

      Weaknesses:

      Where the manuscript becomes less compelling is in the rapamycin section, which does not provide much in the way of mechanistic insights. As such, the effect is more of an epi-phenomenon of unclear insight, and the authors cannot ascribe a signaling mechanism to it that is supported by data. Thus, this latter part rather undermines the overall impact and central advance of the manuscript. The problem is exacerbated by the controversial and anecdotal nature of the entire mTor/aging field, some of whose findings have very unfortunately had to be recently retracted.

      I would strongly recommend to the authors that they end the manuscript with their analysis of the role of M current/KCNQ channels in the numerous age-related changes in sympathetic neuron function that they elegantly report, and save the rapamycin, and possible mTor action, for a separate line of inquiry that the authors could develop in a more thorough and scholarly way.

      Whereas the description of the data are very nice and useful, the manuscript does not provide much in the way of mechanistic insights. As such, the effect is more of an epi-phenomenon of unclear insight, and the authors cannot ascribe changes in signaling mechanisms, such as that of M1 mAChRs to the phenomena that is supported by data.

      I appreciate the new comment. We had agreed that our rapamycin experiments did not allow to ascribe the mechanism to the signaling pathway of mTOR. The new comment mentions M1 mAChRs signaling as another potential signaling mechanism. Our work centered on determining whether aging altered the function of sympathetic motor neurons and defining the mechanism. We presented evidence showing that the mechanism is a reduction of the M-current. We did not attempt to identify the signaling mechanism linking aging to a reduction in M-current. Therefore, we agree with the reviewer that we do not provide further details on the mechanism and that that remains an open question. However, I find it harsh to say that “the effect is more of an epiphenomenon of unclear insight”. How could we possibly test that the effect of aging on the excitability of these neurons only arises as a secondary effect or that is not causal? How could we test for sufficiency and necessity of aging? How could we modify the state of aging to test for causality? We would have to reverse aging and show that the effect on the excitability is gone. And that is exactly what we tried to do with the rapamycin experiment.

      Reviewer #1 (Recommendations For The Authors):

      (1) The significance values greater than p < 0.05 do not add anything and distract focus from the results that are meaningful. Fig. 5 is a good example. What does p = 0.7 mean? Or p = 0.6? Does this help the reader with useful information?

      I thank Reviewer 1 for raising this question. We have attempted different versions of how we report p values, as we want to make sure to address rigor and transparency in reporting data. As corresponding author, I favor reporting p values for all statistical comparisons. To help the reader identifying what we considered statistically significant, we color coded the p values, with red for p-value<0.05 and black for p-value>0.05. As a reader, seeing a p-value=0.7 allows me to know that the authors performed an analysis comparing these conditions and found the mean not to be different. Not presenting the p-value makes me wonder whether the authors even analyzed those groups. In other words, I value more the ability to analyze the data seeing all p-values than not being distracted by not-significant p-values. This is just my preference.

      (2) Fig. 1 is not informative and should be removed.

      I thank Reviewer 1 for the suggestion. In previous drafts of the manuscript, this figure was included only as a panel. However, we decided it was better to guide the reader into the scope of our work. This is part of our scientific style and, therefore, we prefer to keep the figure.

      (3) The emphasis on a particular muscarinic agonist favored by many ion channel physiologists, oxotremorine, is not meaningful (lines 192, 198). The important point is stimulation of muscarinic AChRs, which physiologically are stimulated by acetylcholine. The particular muscarinic agonist used is unimportant. Unless mandated by eLife, "cholinergic type 1 muscarinic receptors" are usually referred to as M1 mAChRs, or even better is "Gq-coupled M1 mAChRs." I don't think that Kruse and Whitten, 2021 were the first to demonstrate the increase in excitability of sympathetic neurons from stimulation of M1 mAChRs. Please try and cite in a more scholarly fashion.

      A) I have modified lines 192 and 198 removing mention to oxotremorine.

      B) I have modified the nomenclature used to refer to cholinergic type 1 muscarinic receptors.

      C) I cited references on the role of M current on sympathetic motor neuron excitability. I also removed the reference (Kruse and Whitten, 2021) referring only on the temporal correlation between the decrease of KCNQ current with excitability.

      (4) The authors may want to use the term "M current" (after defining it) as the current produced by KCNQ2&3-containing channels in sympathetic neurons, and reserve "KCNQ" or "Kv7" currents as those made by cloned KCNQ/Kv7 channels in heterologous systems. A reason for this is to exclude currents KCNQ1-containing channels, which most definitely do not contribute to the "KCNQ" current in these cells. I am not mandating this, but rather suggesting it to conform with the literature.

      Thank you for the suggestion. I have modified the text to use the term M current. I maintain the use of KCNQ only when referring to KCNQ channel, such as in the section describing the abundance of KCNQ2.

      (5) The section in the text on "Aging reduces KCNQ current" is confusing. Can the authors describe their results and their interpretation more directly?

      I am not sure to understand the request. I assumed point 5 and 6 are related and decided to answer point 6.

      (6) Please explain the meaning of the increase in KCNQ2 abundance with age in Fig. 6G. How is this increase in KCNQ2 expression consistent with an increase in excitability? The explanation of "The decrease in KCNQ current and the increase in the abundance of KCNQ2 protein suggest a potential compensatory mechanism that occurs during aging, which we are actively investigating in an independent study." is rather odd, considering that the entire thesis of this paper is that changes in excitability and firing properties are underlied by changes in KCNQ2/3 channel expression/density. Suddenly, is this not the case?? What about KCNQ3? It would be very enlightening if the authors would just quantify the ratio of KCNQ2:KCNQ3 subunits in M-type channels in young and old mice using simple TEA dose/response curves (see Shapiro et al., JNS, 2000; Selyanko et al., J. Physiol., Hadley et al., Br. J. Pharm., 2001 and a great many more). It is also surprising that the authors did not assess or probe for differences in mAChR-induced suppression of M current between SCG neurons of young and old mice. This would seem to be a fundamental experiment in this line of inquiry.

      A. Please explain the meaning of the increase in KCNQ2 abundance with age in Fig. 6G. How is this increase in KCNQ2 expression consistent with an increase in excitability? The explanation of "The decrease in KCNQ current and the increase in the abundance of KCNQ2 protein suggest a potential compensatory mechanism that occurs during aging, which we are actively investigating in an independent study." is rather odd, considering that the entire thesis of this paper is that changes in excitability and firing properties are underlied by changes in KCNQ2/3 channel expression/density. Suddenly, is this not the case?? Our interpretation is that the decrease in M current is not caused by a decrease in the abundance of KCNQ (2) channels. We do not claim that changes in excitability are underlied by a reduction in the expression or density of KCNQ2 channels. On the contrary, our working hypothesis is that the reduction in M current is caused by changes in traffic, degradation, posttranslational modifications, or cofactors for KCNQ2 or KCNQ3 channels. We have modified the description in the results section to clarify this concept.

      B. What about KCNQ3? Unfortunately, we did not find an antibody to detect KCNQ3 channels. I have added a sentence to state this.

      C. KCNQ2:KCNQ3 subunits in M-type channels in young and old mice using simple TEA dose/response curves. This is a great idea. Thank you for the suggestion. Is this a necessary experiment for the acceptance of this manuscript?

      D. It is also surprising that the authors did not assess or probe for differences in mAChR-induced suppression of M current between SCG neurons of young and old mice. This would seem to be a fundamental experiment in this line of inquiry. Reviewer 1 is correct. We did not assess for differences in the suppression of M current by mAChR activation. We do not see the connection of this experiment with the scope of the current investigation.

      (7) Why do the authors use linopirdine instead of XE-991? Both are dirty drugs hardly specific to KCNQ channels at 25 uM concentrations, but linopirdine less so. The Methods section lists the source of XE991 used in the study, not linopirdine. Is there an error?

      A. Why do the authors use linopirdine instead of XE-991? After validation of KCNQ2/3 inhibition by Linopirdine, we found the effect on membrane potential recordings to be reproducible. Linopirdine has also been reported to be reversible. We wanted to assess reversibility on the excitability of young neurons. We did not find the effect to be reversible. We performed experiments applying XE-991 while recording the membrane potential. XE-991 did not show a clear effect. I was not surprised by this. It is very likely that the pharmacological inhibition of one channel leads to the activation of other channel types. This is highlighted in the work by Kimm, Khaliq, and Bean, 2015. “Further experiments revealed that inhibiting either BK or Kv2 alone leads to recruitment of additional current through the other channel type during the action potential as a consequence of changes in spike shape.” In fact, it was quite remarkable that the aged and young phenotypes were mimicked by targeting KCNQ pharmacologically.

      B. Both are dirty drugs hardly specific to KCNQ channels at 25 uM concentrations, but linopirdine less so. I have added a sentence to point out that linopirdine is less potent than XE-991. It reads: “We want to point out that linopirdine is less potent than XE-991 and that it has been reported to activate TRPV1 channels (Neacsu and Babes, 2010). Despite this limitation, the application of linopirdine to young sympathetic motor neurons led to depolarization and firing of action potentials.”

      C. The Methods section lists the source of XE991 used in the study, not linopirdine. Is there an error? Thank you for pointing out this. I have added information for both retigabine and linopirdine in the Methods section, both were missing.

      (8) Can the authors use a more scientific explanation of RTG action than "activating KCNQ channels?" For instance, RTG induces both a negative-shift in the voltage-dependance of activation and a voltage-independent increase in the open probability, both of which differing in detail between KCNQ2 and KCNQ3 subunits. The authors are free to use these exact words. Thus, the degree of "activation" is very dependent upon voltage at any voltages negative to the saturating voltages for channel activation.

      I have modified the text to reflect your suggestion.

      (9) Methods: did the authors really use "poly-l-lysine-coated coverslips?" Almost all investigators use poly-D-lysine as a coating for mammalian tissue-culture cells and more substantial coatings such as poly-D-lysine + laminin or rat-tail collagen for peripheral neurons, to allow firm attachment to the coverslip.

      That is correct. We used poly-L-lysine-coated coverslips. Sympathetic motor neurons do not adhere to poly-D-Lysine.

      (10) As a suggestion, sampling M-type/KCNQ/Kv7 current at 2 kHz is not advised, as this is far faster than the gating kinetics of the channels. Were the signals filtered?

      It is correct. Currents were sampled at 2KHz. Data were low-pass filtered at 3 KHz. Our conditions are not far from what is reported by others. Some sample at 10KHz and even 50 KHz. Others do not report the sample frequency.

      Reviewer #2:

      Weaknesses:

      None, the revised version of the manuscript has addressed all my concerns.

      I am glad we were able to satisfy previous concerns.

      Reviewer #3:

      The main weakness is that this study is a descriptive tabulation of changes in the electrophysiology of neurons in culture, and the effects shown are correlative rather than establishing causality.

      Allow me to clarify our previous responses and determine how this aligns with your concerns. In the previous revision, Reviewer 3 wrote: “It is difficult to know from the data presented whether the changes in KCNQ channels are in fact directly responsible for the observed changes in membrane excitability.” And suggested to “use of blockers and activators to provide greater relevance.” I assumed these comments were the main concern and that doing such experiments was enough to satisfy the criticism. It is discouraging to see that our experiments did not satisfy the concerns of the reviewer of being correlative.

      If Reviewer 3 is referring to stablishing causality between aging and a reduction in M current, I would like to emphasize that such endeavor is complicated as there is not a clear experiment to solve that issue. Our best attempt was to reverse aging with rapamycin, but the recommendation was to remove those experiments.

      … but the specifics of the effects and relevance to intact preparations are unclear. Additional experiments in slice cultures would provide greater significance on the potential relevance of the findings for intact preparations.

      I apologize for missing this point in the previous revision. The proposed experiments will require an upward microscope coupled to an electrophysiology rig. Unfortunately, I do not have the equipment to do these experiments.

      Summary of recommendations from the three reviewers:

      Please make corrections as suggested by reviewer 1 to improve the manuscript. Specifically, reviewer 1 suggests making changes to p values in Figure 5,

      It is not clear what the suggested changes are. The comment from Reviewer 1 says: The significance values greater than p < 0.05 do not add anything and distract focus from the results that are meaningful. If the suggested change is to remove p values > 0.05, I have explained my rational for keeping those values. If the Journal has a specific format on how to report p-values, I will be happy to make appropriate changes.

      and the importance of citing original scholarly works related to effects of increase in excitability of sympathetic neurons by M1 receptors, and the terminology for M currents and KCNQ currents. These changes will improve the manuscript and are strongly recommended.

      I cited original papers on that area, and changed the terminology for M current. I kept KCNQ when referring to the channel protein or abundance.

      The section dealing with Aging Reduces KCNQ currents seems to contain a lot of extraneous information especially in the last part of the long paragraph and this section should be rewritten for improved clarity… and - the implications or lack thereof - of the correlation of KCNQ with AP firing rates.

      A. I removed extraneous information in that section. It now reads: Previous work by our group and others demonstrated that cholinergic stimulation leads to a decrease in M current and increases the excitability of sympathetic motor neurons at young ages \cite{RN67,RN68,RN69,RN71, RN72, RN73, RN74, RN75}. The molecular determinants of the M current are channels formed by KCNQ2 and KCNQ3 in these neurons \cite{RN76, RN77, RN70}. Thus, Figure 6A shows a voltage response (measured in current-clamp mode) and a consecutive M current recording (measured in voltage-clamp mode) in the same neuron upon stimulation of cholinergic type 1 muscarinic receptors. It illustrates the temporal correlation between the decrease of M current with the increase in excitability and firing of APs upon activation with oxotremorine. This strong dependence led us to hypothesize that aging decreases M current, leading to a depolarized RMP and hyperexcitability (Figure 6B). For these experiments, we measured the RMP and evoked activity using perforated patch, followed by the amplitude of M current using a whole-cell voltage clamp in the same cell. We also measured the membrane capacitance as a proxy for cell size. Interestingly, M current density was smaller by 29\% in middle age (7.5 ± 0.7 pA/pF) and by 55\% in old (4.8 ± 0.7 pA/pF) compared to young (10.6 ± 1.5 pA/pF) neurons (Figure 6C-D). The average capacitance was similar in young (30.8 ± 2.2 pF), middle-aged (27.4 ± 1.2 pF), and old (28.8 ± 2.3 pF) neurons (Figure 6E), suggesting that aging is not associated with changes in cell size of sympathetic motor neurons, and supporting the hypothesis that aging alters the levels of M current. Next, we tested the effect on the abundance of the channels mediating M current. Contrary to our expectation, we observed that KCNQ2 protein levels were 1.5 ± 0.1 -fold higher in old compared to young neurons (Figure 6F-G). Unfortunately, we did not find an antibody to detect consistently KCNQ3 channels. We concluded that the decrease in M current is not caused by a decrease in the abundance of KCNQ2 protein.

      B. and - the implications or lack thereof - of the correlation of KCNQ with AP firing rates. I am not sure to understand the request on the section of the correlation of KCNQ with AP firing rate. I divided the long paragraph.

      The apparent lack of correlation between KCNQ current and KCNQ2 protein needs to be better explained. This is a central part of the study and this result undercuts the premise of the paper.

      Indeed, total KCNQ2 protein abundance increases while M current decreases. We do not claim in our work that changes in excitability are caused by a reduction in the expression or density of KCNQ2 channels. On the contrary, our current working hypothesis is that the reduction in M current is caused by changes in traffic, degradation, posttranslational modifications, or cofactors for KCNQ2 or KCNQ3 channels. I have modified the description in the results section and discussion to clarify this concept.

      Additionally, the poor specificity of Linordipine for KCNQ should be pointed out in the limitations.

      I pointed this limitation. It reads: We want to point out that linopirdine is less potent than XE-991 and that it has been reported to activate TRPV1 channels (Neacsu and Babes, 2010). Despite this limitation, the application of linopirdine to young sympathetic motor neurons led to depolarization and firing of action potentials.

      Finally, the editor notes that the author response should not contain ambiguities in what was addressed in the revision. In the original summary of consolidated revisions that were requested, one clearly and separately stated point (point 4) was that experiments in slice cultures should be strongly considered to extend the significance of the work to an intact brain preparation. The author response letter seems to imply that this was done, but this is not the case. The author response seems to have combined this point with another separate point (point 3) about using KCNQ drugs, and imply that all concerns were addressed. Authors should be clear about what revisions were in fact addressed.

      As corresponding author, and direct responsible of the document provided for the reply to the reviewers, I apologize for my mistake. After reviewing this comment, I realized I did not respond to the Major points in the section of the Recommendations for the authors from Reviewer 3. I missed that entire section. My previous responses addressed the Public review of reviewer 3. When doing so, I did not separate the sentences, omitting the request on performing the experiment in slices.


      The following is the authors’ response to the original reviews.

      Reviewer #1

      Summary:

      The authors study age-related changes in the excitability and firing properties of sympathetic neurons, which they ascribe to age-related changes in the expression of KCNQ (Kv7, "M-type") K+ currents in rodent sympathetic neurons, whose regulation by GPCRs has been most thoroughly studied for over 40 years. The authors suggest the ingestion of rapamycin may partially reverse the age-related decrease in M-channel expression. With the rapamycin part included, it is unclear how this work will impact the field of age-related neuronal dysfunction, as the mechanistic information is not strong.

      Strengths:

      The strengths include the rigor of the current-clamp and voltage-clamp experiments, the lovely, crisp presentation of the data, and the expert statistics. The separation of neurons into tonic, phasic, and adapting classes is also interesting, and informative. The writing is also elegant, and crisp. The above is especially true of the manuscript up until the part dealing with the effects of rapamycin, which becomes less compelling.

      We appreciate the thoughtful comments and constructive feedback to improve the impact of the manuscript.

      Weaknesses:

      Where the manuscript becomes less compelling is in the rapamycin section, which does not provide much in the way of mechanistic insights. As such, the effect is more of an epi-phenomenon of unclear insight, and the authors cannot ascribe a signaling mechanism to it that is supported by data. Thus, this latter part rather undermines the overall impact and central advance of the manuscript. The problem is exacerbated by the controversial and anecdotal nature of the entire mTor/aging field, some of whose findings have very unfortunately had to be recently retracted.

      I would strongly recommend to the authors that they end the manuscript with their analysis of the role of M current/KCNQ channels in the numerous age-related changes in sympathetic neuron function that they elegantly report, and save the rapamycin, and possible mTor action, for a separate line of inquiry that the authors could develop in a more thorough and scholarly way.

      We agree with the reviewer in that we cannot ascribe a signaling mechanism to the reversibility observed with rapamycin. Therefore, we are following the recommendation of the reviewer and have removed the rapamycin section.

      We want to emphasize that, in the aging field, any advancement in the knowledge of how drugs such as rapamycin reverse age-associated phenotypes is of crucial importance. These drugs, commonly referred to as aging interventions, include rapamycin, calorie restriction, elamipretide, and metformin. We could have used any of these interventions. And yet, the cellular and molecular mechanisms for each one of these anti-aging drugs are unknown.

      We want to note that, although the nature of the mTOR field is controversial, the effect of rapamycin in extending lifespan and improving health is not. At least these authors have not been able to find retracted papers on that subject or notices from the NIA alerting on this issue. We kindly request the reviewer to provide the references related to rapamycin that were retracted so we can evaluate how that affects the rigor of the premise for our future work.

      As authors, we also find it important to note that we are confident of our observations regarding the effect of rapamycin, and that we are not removing this section because we are retracting our claims. We will use these data to continue our research of the mechanism behind the effect of aging on sympathetic motor neurons.

      Reviewer #2:

      Summary:

      This research shows compelling and detailed evidence showing that aging influences intrinsic membrane properties of peripheral sympathetic motor neurons such that they become more excitable. Furthermore, the authors present convincing evidence that the oral administration of the anti-aging drug Rapamycin partially reversed hyperexcitability in aged neurons. This study also investigates the molecular mechanisms underlying age-associated hyperexcitability in mouse sympathetic motor neurons. In that regard, the authors found an age-associated reduction of an outward current having properties similar to KCNQ2/Q3 potassium current. They suggested a reduction of KCNQ2/Q3 current density in aged neurons as a potential mechanism behind their overactivity.

      Strengths:

      Detailed and rigorous analysis of electrical responses of peripheral sympathetic motor neurons using electrophysiology (perforated patch and whole-cell recordings). Most of the conclusions of this paper are well supported by the data.

      We thank the reviewer for valuing our effort to present a detailed and rigorous analysis.

      Weaknesses:

      (1) The identity of the age-associated reduced current as KCNQ2/Q3 is not corroborated by pharmacology (blocking the current with the specific blocker XE-991).

      We have performed experiments using blockers of KCNQ channels. See responses below.

      (2) The manuscript does not include a direct test of the reduction of KCNQ current as the mechanism behind age-induced hyperexcitability.

      Thank you for raising this point. We have performed experiments blocking KCNQ channels with Linopiridine in young neurons and found that the pharmacological reduction of KCNQ current was enough to depolarize the cell and, in some cases, elicit the firing of action potentials. We present the results in a new figure. We also added the description in the Results section.

      Reviewer #3:

      This is a descriptive study of membrane excitability and Na+ and K+ current amplitudes of sympathetic motor neurons in culture. The main findings of the study are that neurons isolated from aged animals show increased membrane excitability manifested as increased firing rates in response to electrical stimulation and changes in related membrane properties including depolarized resting membrane potential, increased rheobase, and spontaneous firing. By contrast, neuron cultures from young mice show little to no spontaneous firing and relatively low firing rates in response to current injection. These changes in excitability correlate with significant reductions in the magnitude of KCNQ currents in aged neurons compared to young neurons. Treating cultures with the immunosuppressive drug, rapamycin, which has known antiaging effects in model animals appears to reverse the firing rates in aged neurons and enhance KCNQ current. The authors conclude that aging promotes hyperexcitability of sympathetic motor neurons.

      The electrophysiological cataloging of the neuronal properties is generally well done, and the experiments are performed using perforated patch recordings which preserve the internal constituents of neurons, providing confidence that the effects seen are not due to washout of regulators from the cells.

      The main weakness is that this study is a descriptive tabulation of changes in the electrophysiology of neurons in culture, and the effects shown are correlative rather than establishing causality. It is difficult to know from the data presented whether the changes in KCNQ channels are in fact directly responsible for the observed changes in membrane excitability.

      We appreciate the constructive criticism. In an attempt to assess whether changes in KCNQ are in fact directly responsible for the changes in membrane excitability, we have performed experiments blocking KCNQ channels with Linopirdine in young neurons and found that the pharmacological reduction of KCNQ current was enough to depolarize the cell and, in some cases, elicit the firing of action potentials. Conversely, we activated KCNQ channels in old neurons with retigabine and found that the pharmacological activation was enough to hyperpolarize the membrane potential and stop the firing of action potentials. This effect was reversible. These two experiments provide solid evidence to our statement that age-associated reduction of KCNQ activity is responsible for the hyperexcited state in sympathetic motor neurons. We present the results in a new figure (Figure 8). We also added the description in the Results section.

      Furthermore, a notable omission seems to be the analysis of Ca2+ currents which have been widely linked to alterations in membrane properties in aging.

      We thank the reviewer for the comment. We did omit to include data on our studies of calcium currents. We agree that the study of the effect of calcium currents is relevant as it can influence the afterhyperpolarization. Furthermore, we believe that potential effects on calcium currents need to be studied in relation to other physiological processes that depend on calcium, including excitation-transcription coupling, calcium handling, and neurotransmitter release. Adding this information to this manuscript would only contribute to the tabulation of effects that we observe in sympathetic motor neurons with aging. As our main goal was to determine the ion channels responsible for the hyperexcited state, voltage-gated calcium channels or other calcium sources could have reflected a more indirect mechanism as compared to changes in sodium or potassium currents. We will continue our investigation on calcium currents and report our observations in the future, but for now, we have decided to leave it out of this work.

      As well, additional experiments in slice cultures would provide greater significance on the potential relevance of the findings for intact preparations. Finally, experiments using KCNQ blockers and activators could provide greater relevance that the observed changes in KCNQ are indeed connected to changes in membrane excitability.

      We are happy to report that we have performed these experiments and that the results strengthen the conclusion that changes in KCNQ are connected to changes in membrane excitability.

      Recommendations for the authors:

      We recommend the following essential revisions summarized from the reviews:

      (1) Is the change in KCNQ current responsible for the altered membrane excitability? What happens to membrane excitability when KCNQ is partially blocked (see reviewer 2 comment below)? Conversely, what happens to the excitability of aged neurons if KCNQ is activated (e.g., with retigabine)? (see reviewer 3 comment below). Results of these important experiments are needed to support the argument that KCNQ underlies the alterations in firing and membrane excitability.

      We have responded to this point. Thank you for the suggested experiments. In summary, the new experiments show that blocking KCNQ channels in young neurons lead to depolarization, and in some cases, the firing of action potentials. Conversely, the activation of KCNQ channels in aged neurons leads to hyperpolarization and a cease of firing. We have added a new figure and reported the results in the Results section.

      (2) Rapamycin experiments are underdeveloped and weak. These should be further developed by examining the effects of KCNQ blockers to see if their effects on membrane excitability are reversed. Also, see comment 2 from reviewer 1.

      We have followed the recommendation by reviewer 1 and removed the section on rapamycin.

      (3) The study should examine voltage-gated calcium currents to determine potential changes in these currents with aging. See reviewer 3 comments.

      We thank the reviewer for the comment. We performed preliminary experiments and found that aging impacts calcium currents. However, we omitted to include the data. In our opinion, the changes in calcium currents are outside the scope of this work, as the changes could be related to physiological processes that go beyond the control of firing. Effects on calcium currents need to be studied in relation to other physiological processes that depend on calcium, including excitation-transcription coupling, calcium handling, and neurotransmitter release. The study of the relationship between changes in calcium currents and those physiological processes would require multiple experiments and detailed analysis. We will continue our investigation on calcium currents and report our observations in the future, but for now, we have decided to leave it out of this work.

      We have also edited suggestions in the Figures and Legends.

      (2) In Fig.4 panel H, Y-axis must be # AP at 100 pA.

      We corrected the axis in Figure 4H.

      (3) In Legend Fig. 5, the number of cells for each subpopulation (n) needs to be corrected. In plots F-I, n= 9, 7, and 3 seem to be the number of adapting cells for 12-, 64- and 115w-old, respectively, instead of the number of single, phasic, and old cells for 12-week-old mice. A similar correction seems to be needed for 64-week-old and 115-week-old.

      We corrected the n number in Figure 5.

      (4) In Figure 6 panel C, it would be helpful for a reader to align the voltage protocol depicted with the current shown.

      We have aligned the voltage protocol to the current traces.

      (5) In the legend of Figure 7, the description of panel A ends with "Magnitude of voltage step to elicit each trace is shown in black", however in panel A there is no voltage depiction. In the description of panel D, "N = X animals, n=x cells" must be corrected.

      We have modified the legend to clarify. It now reads: “Text at the right of each current trace corresponds to the voltage used to elicit that current.”

      New Figure 8

      Author response image 1.

      Pharmacological inhibition and activation of KCNQ channels mimic the age-dependent phenotype. A. Membrane potential recordings from two young neurons treated with 25 μM linopirdine during the time illustrated by the light gray box. No holding current was applied. B. Left: Summary of the resting membrane potential measured before (light orange) and after (dark orange) the application of linopirdine. Right: Summary of the depolarization produced by linopirdine calculated by subtracting the post-drug voltage from the pre-drug voltage (V). Data points are from N = 2 animals, n = 8 cells, 14-week-old mice. C. Membrane potential recordings from two aged neurons treated with 10 μM retigabine during the time illustrated by the light gray box. No holding current was applied. D. Left: Summary of the resting membrane potential measured before (light purple) and after (dark purple) the application of retigabine. Right: Summary of the hyperpolarization produced by retigabine calculated by subtracting the post-drug voltage from the pre-drug voltage (V). Data points are from N = 2 animals, n = 7 cells, 120-week-old mice. P-values are shown at the top of the graphs.

    1. Author Response

      The following is the authors’ response to the current reviews.

      Joint Public Review

      This study is concerned with the general question as to how pools of synaptic vesicles are organized in presynaptic terminals to support different types of transmitter release, such as fast synchronous and asynchronous release. To address this issue, the authors employed the classical method of load- ing synaptic vesicle membranes with FM-styryl dyes and assessing dye destaining during repetitive synapse stimulation by live imaging as a readout of the mobilization of vesicles for fusion. Among other 1ndings, the authors provide evidence indicating that there are multiple reserve vesicle pools, that quickly and slowly mobilized reserves do not mix, and that vesicle fusion does not follow a mono-exponential time course, leading to the notion that two separate reserve pools of vesicles - slowly vs. rapidly mobilizing - feed two distinct releasable pools - reluctantly vs. rapidly releasing. These 1ndings are valuable to the 1eld of synapse biology, where the organization of synaptic vesicle pools that support synaptic transmission in different temporal and stimulation regimes has been a focus of intense experimentation and discussion for more than two decades.

      On the other hand, the present study has limitations, so that the authors’ key conclusions remain incompletely supported by the data, and alternative interpretations of the data remain possible. The approach of using bulk FM-styryl dye destaining as a readout of precise vesicle arrangements and pools in a population of functionally very diverse synapses bears problems. In essence, the approach is ’blind’ to many additional processes and confounding factors that operate in the back- ground, from other forms of release to inter-synaptic vesicle exchange. Further, averaging signals over many - functionally very diverse - synapses makes it diicult to distinguish the dynamics of separate vesicle pools within single synapses from a scenario where different kinetics of release originate from different types of synapses with different release probabilities.

      We thank the editors and reviewers for their time and patience, and are happy that they found our results valuable.

      We do not have a clear understanding of what the alternative interpretations might be - beyond those already addressed - but would like to. At present, we believe that the evidence for parallel processing of slowly and quickly mobilized reserve vesicles is solid and hope that people who are open to the possibility will evaluate the reasoning described within our report. The hypothesis that reserves are kept separate because they feed distinct subdivisions of the readily releasable pool remains to be tested.

      Beyond that, we have used FM-dye de-staining as a bulk measurement of sub-synaptic events in the sense that we have made no attempt to measure mobilization of isolated individual vesicles. We do not see how this necessarily leaves viable alternative interpretations, but this is diZcult to evaluate without knowing what the alternatives might be. On the other hand, the FM-dye technique has had good resolution at the level of distinguishing between individual synapses since at least Murthy et al. (2001). For our part, we are con1dent that our analysis in Figure 3 combined with the results in Figures 4-11 shows that the multiple reserve pools co-occur in many individual presynaptic terminals. We did not use electron microscopy to con1rm that all of the punctae analyzed in Figure 3 were indeed single synapses, but the reviewers did not recommend this, and we believe there is already enough published about the spatial distribution of synapses in cell culture to be con1dent that many of the punctae that are smaller than 1.5 µm were individuals.

      Overall, we have attempted to address all of the individual concerns raised by reviewers, and our understanding is that these concerns and our responses will be available on the eLife website. The reviewers were not convinced on every point, but these are cases where the nature of the concern was not clear to us. We hope that people who share these concerns will check out our responses and contact us with any further questions or alternative interpretations.

      (1) The authors sincerely addressed many of the previous concerns, mainly by clari1cation. The data are consistent with the authors’ hypothesis. The pool concept is somewhat similar to that of Richards et al (2000) and Rey et al (2015). The authors further propose that two reserve pools feed vesicles to two readily-releasable pools independently.

      To clarify further: The possibility that distinct reserve pools feed distinct readily releasable pools is predicted by our working model, and is something that we would like to test in the future, but is not a conclusion of the present study. Instead, in the present study, we tested the prediction that quickly and slowly mobilized reserve vesicles are processed in parallel without making assumptions about the the underlying mechanism.

      Unfortunately, the heterogeneity among individual synapses remains a concern as shown in (some of) the raw data (Fig. 3 and supplements).

      We emphasize that we have not attempted to minimize the extensive heterogeneity among synapses, but actually highlight this. In fact, we chose the image in Figure 3 for an example in part because of the lower left region replicated in Figure 3 supplement 2 demonstrating extensive heterogeneity along what appears to be a single axon. We are not the 1rst to notice the heterogeneity (see Waters and Smith, 2002), but we do provide a new possible explanation which, if correct, might be impor- tant for understanding biological computation (see our Discussion). At the same time, we believe that our evidence for multiple reserve pools within individual synapses with heterogenous properties is compelling. We see no contradiction, and indeed, our conclusion that the ratio of slowly to quickly mobilized varies extensively between synapses can only be correct if individual synapses contain mul- tiple types. We hope that people who are interested in our conclusions will evaluate the evidence and reasoning presented in our report.

      Bulk imaging of FM de-staining does not really measure the fraction of non-stained vesicles, which changes dynamically during stimulation, so that the situation calls for an independent readout of stained and non-stained vesicles. Moreover, direct correspondence between two speci1c stimulation frequencies (with long stimulation) and vesicle pools is not straightforward. These issues make the experimentally measured pools not well-de1ned.

      We think that the reviewer is suggesting an alternative scenario where decreases in the fractional rate of FM-dye de-staining seen during 1 Hz stimulation might be caused by a large (4-fold) increase in the total size of the reserve pool that dilutes the stained vesicles by mixing. This scenario is consis- tent with the results in Figures 2 and 4-7, and initially seems plausible because previous studies have shown that many vesicles are not mobilized, and therefore are not stained, during our standard load- ing protocol of 100 s at 20 Hz (Harata et al., 2001). However, liberation of this "deep reserve" as an explanation for the decrease in fractional destaining is not compatible with the results in Figures 10-11 that rule out mixing. For example, liberation of the deep reserve would cause fractional destaining to appear equally depressed during subsequent 20 Hz stimulation, and Figure 10 shows that this is not the case. The scenario cannot be rescued by postulating that the subsequent 20 Hz stimulation caused the deep reserve to quickly recapture the liberated vesicles because Figure 11D-E shows that fractional de-staining continues to be depressed at the very beginning of a second 1 Hz train that follows the 20 Hz stimulation.

      (2) The authors’ latest round of responses did not alleviate most of my major previous concerns. The additional data now shown in Fig 3 rely on conceptually the same type of bulk measurements and thus suffer from the same limitations as outlined in the earlier review.

      We believe that the new evidence in Figure 3 for multiple reserve pools at individual synapses is strong when evaluated in combination with the results in Figures 4-11. We do not, at present, see how the fact that FM-dye destaining is used as a bulk measurement at the sub-synaptic level could undercut our logic.

      Moreover, the image of neuronal cultures shown in Fig. 3 might be problematic. It shows very bright staining with large round lumps, which may be indicative of unhealthy cultures.

      Unhealthy cultures are not a concern because we used strict quantitative criteria to assess health that are better than we have seen elsewhere (details below). We think the reviewer might be reacting to the way we rendered the image; i.e., as “overexposed”. We did this to highlight the dimmest punctae, which is a key element of the analysis. The same image rendered with less contrast is now displayed in Author response image 1 (3rd panel from left).

      Author response image 1.

      Image to left is a reproduction of the example image in Figure 3, which was the average of 120 time lapse raw data images; scale bar is 20 µm. The second image is a replicate except all 69 punctae that were included in the study are occluded by 1.5 µm × 1.5 µm yellow squares. The third image is another replicate except with a different brightness setting. The rightmost image is one of the raw data images with brightness matched to the third image.

      More details (relevance to in vivo is in point 4):

      (1) Identifying unhealthy cultures is straightforward with our technique because synapses in un- healthy cultures destain spontaneously. Our criteria for accepting experiments for further analy- sis was less than 1.5 % spontaneous rundown/minute. This is a better way to judge health than we have seen elsewhere because it eliminates subjective decisions, and would be equally appli- cable for microscopes and imaging software of any quality. For our part, we used a 25X objective with a low numerical aperture and low intensity illumination that allowed us to completely avoid photobleaching. The images will look worse to some compared to when acquired with a higher quality microscope, but the absence of photobleaching is an important bene1t because it allowed us to avoid complicated corrections.

      (2) Stained areas larger than 1.5 µm across - such as the ones noted by the reviewer - were expressly excluded from our study because they could have been clusters of multiple synapses. The size criteria are detailed in the Legend of Figure 3. Punctae and larger areas that were excluded are the ones that are not occluded by yellow squares in the 2nd image from the left, above; at least two of the largest were likely clusters of synapses that were out of focus. Nevertheless, despite being excluded, it is unlikely that the stained areas larger than 1.5 µm in the image in Figure 3 were characteristic of unhealthy cultures because these areas did not de-stain spontaneously, but instead de-stained in response to 1 and 20 Hz electrical stimulation much like the small punctae that were included in the analysis.

      (3) Electron microscopy results have shown that individual synapses vary >10-fold in size, so a large range of brightness is expected (Murthy et al., 2001). The large range would either make the brighter punctae and clusters appear to be overexposed in a printed image, or render the dimmer punctae invisible. We have opted to present an image with overall brightness adjusted so that the dimmest punctae are visible. This is appropriate because one of the concerns was that analyzing the dimmest punctae would reveal underlying populations where the rate of fractional destaining was constant. In the end, no evidence for underlying populations emerged, which supports the conclusion that the decreases in fractional destaining occur at individual synapses. Note that adjusting brightness for example images was unavoidable; we used the camera in a range that was far below saturation and, because of this, images presented without adjusting brightness would appear to be completely black.

      (4) Primary cell cultures are non-physiological by de1nition, so the concept of health is intrinsically arbitrary, and relevance to synapses in brains is questioned routinely. However, the new 1ndings in the present report are that: (1) individual hippocampal synapses contain multiple reserve pools; (2) the reserves remain separate but are not distinguishable by the timing of mobilization when the frequency of stimulation is high; and (3) the reserves are nevertheless processed in parallel even when the frequency of stimulation is high. Of these, 1nding (1) has been reported previously for other synapse types, but 1ndings (2) and (3) were both unexpected, and 1nding (3) was not compatible with current concepts. Nevertheless, all three 1ndings were predicted by a model that was developed to explain orthogonal results from studies of intact synapses in ex vivo slices that did not 1t with current concepts either, as referenced in the Introduction. Because of this, we think that the parallel processing of quickly and slowly mobilized reserve vesicles likely occurs in individual Schaffer collateral synapses in vivo, and is not a cell culture artifact; the alternative would be too much of an unlikely coincidence.

      References

      Harata N, Pyle JL, Aravanis AM, Mozhayeva M, Kavalali ET & Tsien RW (2001). Limited numbers of recycling vesicles in small CNS nerve terminals: implications for neural signaling and vesicular cycling. Trends in Neurosciences 24, 637–43.

      Murthy VN, Schikorski T, Stevens CF & Zhu Y (2001). Inactivity produces increases in neurotransmitter release and synapse size. Neuron 32, 673–82.

      Waters J & Smith SJ (2002). Vesicle pool partitioning in2uences presynaptic diversity and weighting in rat hippocampal synapses. Journal of Physiology 541, 811–23.


      The following is the authors’ response to the original reviews.

      Reviewer 1

      Mahfooz et al. investigated the time course of synaptic vesicle fusion of cultured mouse hippocampal synapses using FM-styryl dyes. The major finding is that the FM destaining time course deviates from a mono-exponential function during 1 Hz, but not 20 Hz stimulation. The deviation from a mono-exponential function was also seen during a second stimulus train applied after recovery periods of several minutes, or after depletion of the readily-releasable vesicle pool. Furthermore, this "decreased fractional destaining" was unlikely due to long-term synaptic depression, or incomplete dye clearance. Fractional destaining was enhanced when the dye was loaded with 1 Hz compared with 20 Hz stimulation, suggesting that vesicles recycled during 1 Hz stimulation are predominantly sorted into a rapidly mobilized pool. Finally, they show that 20 Hz stimulation does not affect the decrease in fractional destaining induced and recorded during 1 Hz stimulation. Based on these observations, they put forward a model in which slowly and quickly resupplied synaptic vesicles are mobilized in parallel.

      The demonstration that FM destaining time courses deviate from single exponentials during 1 Hz stimulation (Figs 2-3) is a starting point used to rule out simple models where vesicles intermix freely and to introduce a mathematical technique for quantifying the extent of the deviations that is essential for the analysis of later experiments, where curve fitting could not be used. We then:

      1) Show that the deviation from simple models is not caused by depletion of the readily releasable pool, as noted by the reviewer;

      2) rule out a number of explanations for the deviation that do not involve reserve pools at all, again as noted;

      3) provide affirmative evidence for the presence of multiple reserve pools by labeling them with distinct colors;

      4) show that the vesicles within the distinct reserve pools do not intermix even when activity is intense enough to drive destaining with single exponential kinetics.

      We believe that the 4th point - documented in Figs 10-11 - is a key element.

      Beyond that, we note that our working model arose from previous studies, as referenced in the Introduction, not from the present results. The model did predict the parallel processing of quickly and slowly mobilized reserves, and the present study was designed to test this prediction. In that sense, the evidence in the current study supports our working model, not the other way around.

      In any case, most readers in the near term will be more interested in the serial versus parallel question, and less in precisely what the present results mean for evaluating our working model. Because of this, we emphasize that evidence for parallel processing of separate reserve pools depends solely on experimental results within the study, and not on modeling. As a consequence, the evidence will continue to be equally strong even if problems with our working model arise later on (lines 382-386).

      We do have additional unpublished evidence for the working model that does not bear directly on the parallel versus serial question. Some of this was removed from an earlier version of the manuscript and some has been newly gathered since the original submission. We will publish the additional evidence at a later point. We decided not to include it in the present manuscript expressly to avoid confusion about the relationship between modeling and the evidence for parallel processing in general.

      The paper addresses an interesting question - the relationship between the resupply and release of synaptic vesicles. The study is based on a lot of data of high quality. Most data are solid. However, some of the major conclusions are not well supported by the data. Moreover, it remains unclear how speci1c the findings are to the experimental design.

      The following points should be addressed:

      1) Most traces display a decrease in fluorescence intensity before stimulation. Data with a decrease in baseline fluorescence intensity of up to 1.5 % were considered for the analysis (Fig 2-supplement 2). I may have missed it, but were the data corrected for the observed decrease in baseline fluorescence intensity? (In the model shown in Appendix 1 Figure 1, they correct for "rundown"). For instance, are the residuals shown in Fig 2D, E based on corrected data? In case the data would not be corrected for a decrease in baseline fluorescence, would the decay kinetics also deviate from a single exponential after correction?

      We did not correct for rundown - as now noted on lines 96-97 - except in the figure in the Appendix, noted by the reviewer, where the uncorrected and corrected time courses are plotted side by side for easy comparison. However, our study includes an analysis showing that correcting for rundown during 1 Hz stimulation would increase - not decrease - the deviation from a single exponential (2 bars in rightmost panel in Fig 2C, and lines 113-116 of Results), so the absence of a correction does not weaken our conclusions.

      2) The analysis of "fractional destaining" is not clear to me. How many intervals of which length were chosen and why? For instance, the intervals often differ in length, number and do not cover the complete decay (e.g., Fig 2B).

      We calculated fractional destaining from longer intervals at later times because the overall amount of stain was less, meaning signal/noise was less, and scatter was more. We did this because increased scatter at later times could be counteracted by estimating the slope of destaining from longer intervals. An additional bene1t is that elongating the later intervals allowed us to plot only 6 bars for 25 min of 1 Hz destaining, which works better visually than 17.

      Increasing the interval length for later times is mathematically sound because the key factor causing distortions related to deviations from linearity is not the length of the interval per se but, instead, the fractional destaining over the interval. The fractional destaining is greater at the start of 1Hz stimulation, thus requiring shorter intervals.

      It would be possible to choose inappropriately long intervals that would distort estimates of the change in fractional destaining. However, we now include Fig 2-supplement 6 – which includes all 17 1.5 min intervals - to con1rm that any distortions after the first interval were minimal. The Appendix predicts a biologically important distortion for the first interval which we are following up, but this would underestimate the true deviation from quickly mixing pools, so would not be problematic for the present conclusions.

      Sometimes, only the interval right after stimulation onset was considered (e.g., Fig 7, 8).

      Figs 7, 8 in the previous version are now Figs 8, 9.

      This is appropriate because the goal was to estimate the fractional destaining at the very start, before the quickly mobilized fraction has destained.

      How quickly fractional destaining is expected to revert to the lowest value seen after 15 min of 1Hz stimulation in Fig 2 (and elsewhere) depends very much on assumptions - such as the number of reserve pools, etc. We sought to avoid this kind of additional analysis because we are keen to avoid the impression that our main conclusions depend on the speci1cs of modeling.

      How sensitive are the changes in fractional destaining to the choice of the intervals?

      Minimally. This can be seen by eye because the magenta lines in Fig 2B 1t the data well, but see Fig 2-supplement 6 for a quantitative comparison.

      For instance, would fractional destaining be increased if later intervals would have been chosen for the second 20 Hz stimulus in the experiment shown in Fig 9B?

      Previous Fig 9B is now Fig 10B.

      We cannot be certain, but think it probably would not be different. Neither an increase nor a decrease would be problematic for our conclusions.

      More detail: There is not enough data to evaluate this specifically for Fig 10B because the total amount of stain remaining at later intervals is little, meaning signal/noise is low, which causes extensive experimental scatter. However, synapses were even more extensively destained prior to time course c of Figure2-supplement 2C, which nevertheless matches time courses a, b, and d.

      I propose fitting all baseline-corrected data with a single and a double-exponential function (as well as single exponential plus line?) and reporting the corresponding time constants (slopes) and amplitudes.

      As noted above, we purposefully do not baseline correct data in a way that would make this possible. However, we do include exponential fits when appropriate, in Fig 2D-E, Fig 2- supplement 1, Fig 2-supplement-7, Fig 2-supplement-8, and Fig 12B.

      Indeed, the absence of any change in the weighting parameter despite substantial changes for both time constants seen after raising the temperature to 35C (Fig 2-supplement-8 vs Fig12B) is notable because it suggests that the contents of the reserve pools are not altered by changing temperature, even though vesicle trafficking is accelerated. Fig 2-supplement-8 is a supplementary figure because the result is outside the scope of the main point, not because the quality is lower than for other figures.

      Beyond that, exponential fits would not be adequate for most of the study because many experiments - including the core experiments in Figs 10-11 - require discontinuous stimulation, such as when we stop stimulating at 1 Hz, rest for minutes, and then start up again at 1 or 20 Hz. And, although widely used, exponentials are non-linear equations after all. Even when they can be used to quantify time courses, the fractional destaining measurement is almost always more informative, in the technical sense, because it avoids complications when estimating the importance of deviations occurring at the two extremes versus deviations in the middle of the time course.

      3) Along the same lines, is the average slow time constant indeed around 40 min? (Are the data shown in Fig 2 S7 based on an average?) If this would be the case, I suggest conducting a control experiment with a recording time > 40 min. Would fitting an exponential or a line to baseline data (without stimulation) also give a similar slow component?

      Fig 2-supplement 7 in the previous version is now Fig 2-supplement 8.

      First, yes, the time course shown in Fig 2-supplement 8 is the mean across preparations. The time courses of the individual preparations were quanti1ed as the median value of the individual ROIs before averaging.

      Second, no, fitting baseline data would give an approximately 3-fold greater time constant (i.e., 120 min) because fractional destaining decreases by about 3-fold when we stop stimulating after 25 min of 1 Hz stimulation (i.e., Fig 2C, 3B, and many others).

      The key point is that fractional destaining decreases greatly over long trains of 1 Hz stimulation.

      For Fig 2, we saw a 2.7+/-0.1-fold decrease before accounting for baseline destaining (lines 106-110), which increased to a 4.4-fold decrease when we did account for baseline destaining (lines 113-116). Overall, the 2.7-fold value is simultaneously a safe minimum boundary, and much greater than the value of 1.0 expected from models where vesicles mix freely.

      Note that future studies will show that even the 4.4-fold value is probably an underestimate because 1 Hz stimulation misses a fast component at the very beginning of the time courses, as predicted in the Appendix.

      4) How speci1c are the findings to 1 Hz (and 20 Hz) stimulation? From which frequency onward can a decrease in fractional destaining be no longer observed?

      Our logic depends only on the premise that we are able to find some frequency where fractional destaining no longer decreases. We knew that 20 Hz was a good place to start because of previous electrophysiological experiments - frequency jumps (Fig 1 of Wesseling and Lo, 2002 and Fig 2C of Garcia-Perez and Wesseling, 2008), and trains of action potentials followed by osmotic shocks (Fig 2A of Garcia-Perez et al., 2008) - showing that 20 Hz stimulation is enough to nearly completely exhaust the readily releasable pool. This is noted in lines 202-203, and Box 2.

      would previous stimulation with frequencies <20 Hz interfere with fractional destaining? These control experiments would help assessing how general/speci1c the findings are.

      Yes (Figs 4 and 11A at 1 Hz). Also, we have done experiments at 0.1 Hz, which will be published later; some of these were actually removed from an earlier version of the manuscript because the results are primarily relevant to deciding between particular parallel models, and are not relevant to the conclusion of the present study that quickly and slowly mobilized reserves are processed in parallel.

      Similarly, a major conclusion of the paper - the parallel mobilization of two vesicle pools - is largely based on these two stimulation frequencies. Can they exclude that mixing between the two pools occurs at other frequencies?

      We cannot exclude the possibility of breakdown at a higher frequency, but this would not undercut our conclusions. We do not have plans to try this experiment because: (1) a positive result would be open to concerns about non-physiologically heavy stimulation; and (2) a negative result would be difficult to interpret because of the possibility that the axons cannot follow at higher frequencies.

      6) Some information in the methods section is lacking. For instance, which species is the cell culture based on?

      Mice from both sexes were used. This is now speci1ed in the Methods.

      Reviewer 2

      By using optical monitoring of synaptic vesicles with FM1-43 at hippocampal synapses, the authors try to show the evidence for two parallel reserve pools of synaptic vesicles, which feed the vesicles to the readily releasable pool. The major strength of the study is the use of a quantitative model, which can be readily testable by experiments: in the course of the study, the authors propose the best vesicle pool model, which fits the experimental data "averaged over synapses" nicely. On the other hand, the weak point of the study comes from the optical method and the data: bulk imaging of vesicle dynamics monitored at each synapse is noisy and the signals vary considerably among synapses. Therefore, the average signals over many synapses may not reflect the vesicle dynamics of two reserve pools within a synapse, but something else, such as the different kinetics of release from multiple synapses with different release probability. Nevertheless, a new framework of two reserve pools offers a testable hypothesis of vesicle dynamics, and the use of single vesicle tracking and EM may allow one to give a de1nitive answer in the future studies Therefore, the study may be of interest to the community of synaptic neurobiology.

      1) The current version includes a new figure (Fig 3) showing that the deviations from single pool models seen in populations are caused by deviations occurring at the level of single synapses. The heterogeneity between synapses actually causes population statistics to underestimate - not overestimate - the mean and median size of the deviations at individuals.

      We think the new evidence in Fig 3 and supplements is conclusive without follow-on EM of the same punctae given the substantial body of already published EM on similar cultures. Essentially, the only way to explain the results without invoking multiple reserve pools in individual synapses would be to say that individual synapses ALWAYS come in clumps containing multiple types and are NEVER separated from neighbors by more than 1.5 microns - even when the clumps are separated from each other by 5 microns. There is already clear evidence against this.

      2) No new model is proposed here, see the first response to the first reviewer.

      3) We are not aware of alternative hypotheses that could account for our results, so cannot evaluate if single vesicle tracking and EM could add meaningful additional support.

      1) The existence of non-stained vesicles complicates the interpretation of the data. Because the release by 20 Hz and 1 Hz stimulation do not entirely reflect the release from fast and slow vesicle pools. the estimation of non-stained vesicles using synaptopHluorin (+ba1lomycin) and EPSCs would be helpful to examine fraction of non-stained / stained vesicles over time (with stimulation, the ratio may change dynamically, which may bring complications).

      Non-stained vesicles are not a complication, but instead a key element of our logic which is included in the diagrams in Boxes 1 and 2 and Figure 9. That is, quickly and slowly mobilized reserves can be distinguished at 1 Hz precisely because 1 Hz is not intense enough to exhaust the readily releasable pool (Box 2). The corollary is that stained vesicles must be replaced by non-stained vesicles, because otherwise 1 Hz stimulation would exhaust the readily releasable pool. And this is why FM-dyes (plus a beta-cyclodextrin during washing) are ideal for the current questions whereas other techniques, such as electrophysiology or synaptopHluorin imaging are obviously indispensable for other questions, but could not replace the FM-dyes in the current study. This is now noted on lines 86-89.

      We are aware that synaptopHluorin + ba1lomycin could, in principle, accomplish some of the same goals. However, ba1lomycin ended up being toxic when applied for tens of minutes, as it would have to be in our experiments. And, we do not see what critical question is not already answered with strong evidence using FM dyes.

      2) Individual synapses show marked differences in the time course of de-staining, suggesting differences in release probability. The averaging of the whole data may reflect "average" behavior of synapses, but for example, bi-exponential time course may reflect high Pr and low Pr synapses, rather than vesicle recruitment.

      The authors may comment on this issue.

      See newly added Fig 3, and responses above.

      3) Some differences are very small (Fig 10, the same amplitude as bleaching time course), and I am not certain if the observed differences are meaningful, given low signal to noise ratio in each synapse.

      Fig 10 in the previous version is Fig 11 in the current version.

      Even if correct, this would not be problematic because 20 Hz stimulation clearly did not cause fractional destaining to return to the initial value when stimulation was resumed at 1 Hz (compare d and f in Fig 11E). In any case, Figs 2C, 3B, 5B, 7B, and Fig 10-supplement 2A all show that the minimum fractional destaining value during 1 Hz stimulation is about 3-fold greater than during subsequent rest intervals, which is not a small difference. Also, note that Fig 2-supplement 3 shows that photobleaching likely did not play a role.

      Reviewer 3

      Reviewer #3 (Recommendations For The Authors):

      This study attempts to conceptualize the long-standing question of vesicle pool organization in presynaptic terminals. Authors used classical FM dye release experiments to support a hypothesis that rapidly and slowly releasing vesicles are mobilized in parallel without intermixing. This modular model is also supported indirectly by the authors’ recent findings of molecular links that connect a subset of vesicles in linear chains (published elsewhere).

      Our study should be seen as a test of the hypothesis that quickly and slowly mobilized reserves are processed in parallel. The evidence is independent of any modeling, and would continue to be equally strong if our working model turns out to be incorrect (lines 382-386).

      The scope of the original model was limited by a number of caveats. The main concerns included a limited data set measured in bulk from a highly heterogeneous synapse population, and a complex interrelationship between vesicle mobilization and the bulk FM dye de-staining kinetics. The second major limitation was measurements being performed at room temperature, which inhibits or alters a number of critical synaptic processes that are being modeled. This includes the efficiency of exo/endocytosis coupling, vesicle mobility and release site refractory period, which are stimulus- and temperature-dependent, but were not accounted for in the original model.

      The present study contains experiments at body temperature (Fig 12 and Fig 12-supplement 1 in the current version) and analyses of individual synapses (especially Fig 3 in the current version). To our knowledge all results are consistent with everything that is known about the efficiency of exo/endocytosis coupling, vesicle mobility and release site refractory periods.

      The authors made strong efforts to address previous concerns. However, the main conceptual point, i.e. linking the bulk FM dye de-staining kinetics with precise arrangement of vesicle pools, is not well supported and is generally highly problematic because it ignores many additional processes and confounding factors.

      For example, vesicle exchange between neighboring synapses constitutes from 15% to over 50% of total recycling vesicle population, and therefore is a major contributing factor to FM dye loss/redistribution, but is not considered in this study. Additionally, this vesicle exchange process undergoes calcium/activity-dependent changes, contributing to difficulty in interpreting the current experiments comparing FM de-staining at different stimulation frequencies.

      We do not see how exchange of vesicles between synapses could be a problem for our logic, so cannot evaluate this without a more detailed description of the concern. Instead, our results rule out random inter-synaptic exchange between quickly and slowly mobilized reserve pools because this would show up in our assays as mixing, which does not occur. We think there are three remaining possibilities:

      1) vesicles are exchanged primarily between quickly mobilized reserve pools

      2) vesicles are exchanged primarily between slowly mobilized reserve pools

      3) vesicles in quickly mobilized reserve pools are targeted to quickly mobilized reserve pools in other synapses and vesicles in slowly mobilized reserve pools are targeted to slowly mobilized reserve pools in other synapses.

      It would be interesting to know which of these is correct, but this is outside the scope of the current study.

      Moreover, other forms of release, such as asynchronous release, contribute a large fraction of released vesicles, but are not factored in. Asynchronous release varies widely in synapse population from 0.1 to >0.4 of synchronous release, but is entirely ignored. Spontaneous release may also contribute to FM dye loss over extended 25min recordings used.

      Spontaneous release and asynchronous release are not caveats.

      First, spontaneous: We suspect that spontaneous release contributes to the background destaining rate, but this is 3-fold slower than the minimum during 1 Hz stimulation on average (Figs 2C, 3C, 5B etc), so we know that the slowly mobilized reserve is mobilized by low frequency trains of action potentials (lines 410-412). Note that a different outcome - where the rate of destaining decreased to a very low level during long trains of 1 Hz stimulation - would not have been consistent with the idea that slowly mobilized vesicles are only released spontaneously because the remaining fluorescence can always be destained rapidly by increasing the stimulation intensity to 20 Hz (e.g., see examples in Fig 3).

      Second, asynchronous: We know that slowly mobilized reserves must be released synchronously at 35C because the asynchronous component is eliminated at this temperature (Huson et al., 2019), without altering the quantity of slowly mobilized reserves that are mobilized by 1 Hz stimulation (lines 350-360 of Results, and 445-452 of Discussion; we can con1rm from our own unpublished experiments that the disappearance of asynchronous release at 35C is a robust phenomenon in these cell cultures). Asynchronous release of slowly mobilized vesicles might occur at room temperature, but this would not argue against the conclusion that slowly mobilized vesicles are processed in parallel with quickly mobilized.

      Speci1c comments:

      Points 1-4 are already addressed above.

      5) The notion of the chained vesicles is somewhat confusing: how does the "first" vesicle located at the plasma membrane/release site get released if it is attached to the chain? Wouldn’t this "first" vesicle be non-immediately releasable since it must first be liberated? Since all vesicles shown in the Figure 1 have chains attached to them, what vesicle population then give rise to sub-millisecond release?

      This is not a concern relevant to the present study because none of the conclusions rely on the model in any way (see Introduction, and lines 382-386 of the Discussion). Beyond that: We previously published clear evidence that docked vesicles are tethered to non-docked vesicles (Figure 8 of Wesseling et al., 2019). We see no reason to suspect that a tether to an internal vesicle would prevent the docked vesicle from priming for release.

      7) Model: For fitting de-staining during 20 Hz stimulation, authors state that it was necessary to allow >5-fold Facilitation. This seems to be non-physiologically relevant, since previous studies found only very mild facilitation at room temperature (typically below a factor of 1.5-2.0) and the authors themselves state that, at most, a 1.3 fold facilitation was found.

      If the 1.3-fold facilitation estimate comes from us, it must have been in a different context.

      Most estimates of facilitation that are published are heavily convolved with simultaneous depression, and there is additionally a saturation mechanism for readily releasable vesicles with high release probability that is not widely known (Garcia-Perez and Wesseling, 2008). The standard method for eliminating the depression is to lower the probability of release by lowering extracellular [Ca2+], which additionally relieves occlusion by the saturation mechanism. And, lowering [Ca2+] uncovers an enormous amount facilitation at synapses in hippocampal cell culture. For example, see Figure 2B of Stevens and Wesseling (1999), which shows a 7-fold enhancement during 9 Hz stimulation, and Figure 3 of the same study, which shows a linear relationship with frequency. Taken together these two results suggest 15-fold enhancement during 20 Hz stimulation, which far exceeds the 5-fold value needed at inefficient release sites to make our working model 1t the FM-dye destaining results.

      References

      Garcia-Perez E, Lo DC & Wesseling JF (2008). Kinetic isolation of a slowly recovering component of short-term depression during exhaustive use at excitatory hippocampal synapses. Journal of Neurophysiology 100, 781–95.

      Garcia-Perez E & Wesseling JF (2008). Augmentation controls the fast rebound from depression at excitatory hippocampal synapses. Journal of Neurophysiology 99, 1770–86.

      Huson V, van Boven MA, Stuefer A, Verhage M & Cornelisse LN (2019). Synaptotagmin-1 enables frequency coding by suppressing asynchronous release in a temperature dependent manner. Scienti1c reports 9, 11341.

      Stevens CF & Wesseling JF (1999). Augmentation is a potentiation of the exocytotic process. Neuron 22, 139–46.

      Wesseling JF & Lo DC (2002). Limit on the role of activity in controlling the release-ready supply of synaptic vesicles. Journal of Neuroscience 22, 9708–20.

      Wesseling JF, Phan S, Bushong EA, Siksou L, Marty S, Pérez-Otaño I & Ellisman M (2019). Sparse force-bearing bridges between neighboring synaptic vesicles. Brain Structure and Function 224, 3263–3276.

    1. Author Response

      The following is the authors’ response to the original reviews.

      Recommendations

      Recommendation #1: Address potential confounds in the experimental design:

      (1a) Confounding factors between baseline to early learning. While the visual display of the curved line remains constant, there are at least three changes between these two phases: 1) the presence of reward feedback (the focus of the paper); 2) a perturbation introduced to draw a hidden, mirror-symmetric curved line; 3) instructions provided to use reward feedback to trace the line on the screen (intentionally deceitful). As such, it remains unclear which of these factors are driving the changes in both behavior and bold signals between the two phases. The absence of a veridical feedback phase in which participants received reward feedback associated with the shown trajectory seems like a major limitation.

      (1b) Confounding Factors Between Early and Late Learning. While the authors have focused on interpreting changes from early to late due to the explore-exploit trade-off, there are three additional factors possibly at play: 1) increasing fatigue, 2) withdrawal of attention, specifically related to individuals who have either successfully learned the perturbation within the first few trials or those who have simply given up, or 3) increasing awareness of the perturbation (not clear if subjective reports about perturbation awareness were measured.). I understand that fMRI research is resource-intensive; however, it is not clear how to rule out these alternatives with their existing data without additional control groups. [Another reviewer added the following: Why did the authors not acquire data during a control condition? How can we be confident that the neural dynamics observed are not due to the simple passage of time? Or if these effects are due to the task, what drives them? The reward component, the movement execution, increased automaticity?]

      We have opted to address both of these points above within a single reply, as together they suggest potential confounding factors across the three phases of the task. We would agree that, if the results of our pairwise comparisons (e.g., Early > Baseline or Late > Early) were considered in isolation from one another, then these critiques of the study would be problematic. However, when considering the pattern of effects across the three task phases, we believe most of these critiques can be dismissed. Below, we first describe our results in this context, and then discuss how they address the reviewers’ various critiques.

      Recall that from Baseline to Early learning, we observe an expansion of several cortical areas (e.g., core regions in the DMN) along the manifold (red areas in Fig. 4A, see manifold shifts in Fig. 4C) that subsequently exhibit contraction during Early to Late learning (blue areas in Fig. 4B, see manifold shifts in Fig. 4D). We show this overlap in brain areas in Author response image 1 below, panel A. Notably, several of these brain areas appear to contract back to their original, Baseline locations along the manifold during Late learning (compare Fig. 4C and D). This is evidenced by the fact that many of these same regions (e.g., DMN regions, in Author response image 1 panel A below) fail to show a significant difference between the Baseline and Late learning epochs (see Author response image 1 panel B below, which is taken from supplementary Fig 6). That is, the regions that show significant expansion and subsequent contraction (in Author response image 1 panel A below) tend not to overlap with the regions that significantly changed over the time course of the task (in Author response image 1 panel B below).

      Author response image 1.

      Note that this basic observation above is not only true of our regional manifold eccentricity data, but also in the underlying functional connectivity data associated with individual brain regions. To make this second point clearer, we have modified and annotated our Fig. 5 and included it below. Note the reversal in seed-based functional connectivity from Baseline to Early learning (leftmost brain plots) compared to Early to Late learning (rightmost brain plots). That is, it is generally the case that for each seed-region (A-C) the areas that increase in seed-connectivity with the seed region (in red; leftmost plot) are also the areas that decrease in seed-connectivity with the seed region (in blue; rightmost plot), and vice versa. [Also note that these connectivity reversals are conveyed through the eccentricity data — the horizontal red line in the rightmost plots denote the mean eccentricity of these brain regions during the Baseline phase, helping to highlight the fact that the eccentricity of the Late learning phase reverses back towards this Baseline level].

      Author response image 2.

      Critically, these reversals in brain connectivity noted above directly counter several of the critiques noted by the reviewers. For instance, this reversal pattern of effects argues against the idea that our results during Early Learning can be simply explained due to the (i) presence of reward feedback, (ii) presence of the perturbation or (iii) instructions to use reward feedback to trace the path on the screen. Indeed, all of these factors are also present during Late learning, and yet many of the patterns of brain activity during this time period revert back to the Baseline patterns of connectivity, where these factors are absent. Similarly, this reversal pattern strongly refutes the idea that the effects are simply due to the passage of time, increasing fatigue, or general awareness of the perturbation. Indeed, if any of these factors alone could explain the data, then we would have expected a gradual increase (or decrease) in eccentricity and connectivity from Baseline to Early to Late learning, which we do not observe. We believe these are all important points when interpreting the data, but which we failed to mention in our original manuscript when discussing our findings.

      We have now rectified this in the revised paper, where we now write in our Discussion:

      “Finally, it is important to note that the reversal pattern of effects noted above suggests that our findings during learning cannot be simply attributed to the introduction of reward feedback and/or the perturbation during Early learning, as both of these task-related features are also present during Late learning. In addition, these results cannot be simply explained due to the passage of time or increasing subject fatigue, as this would predict a consistent directional change in eccentricity across the Baseline, Early and Late learning epochs.”

      However, having said the above, we acknowledge that one potential factor that our findings cannot exclude is that they are (at least partially) attributable to changes in subjects’ state of attention throughout the task. Indeed, one can certainly argue that Baseline trials in our study don’t require a great deal of attention (after all, subjects are simply tracing a curved path presented on the screen). Likewise, for subjects that have learned the hidden shape, the Late learning trials are also likely to require limited attentional resources (indeed, many subjects at this point are simply producing the same shape trial after trial). Consequently, the large shift in brain connectivity that we observe from Baseline to Early Learning, and the subsequent reversion back to Baseline-levels of connectivity during Late learning, could actually reflect a heightened allocation of attention as subjects are attempting to learn the (hidden) rewarded shape. However, we do not believe that this would reflect a ‘confound’ of our study per se — indeed, any subject who has participated in a motor learning study would agree that the early learning phase of a task is far more cognitively demanding than Baseline trials and Late learning trials. As such, it is difficult to disentangle this ‘attention’ factor from the learning process itself (and in fact, it is likely central to it).

      Of course, one could have designed a ‘control’ task in which subjects must direct their attention to something other than the learning task itself (e.g., divided attention paradigm, e.g., Taylor & Thoroughman, 2007, 2008, and/or perform a secondary task concurrently (Codol et al., 2018; Holland et al., 2018), but we know that this type of manipulation impairs the learning process itself. Thus, in such a case, it wouldn’t be obvious to the experimenter what they are actually measuring in brain activity during such a task. And, to extend this argument even further, it is true that any sort of brain-based modulation can be argued to reflect some ‘attentional’ process, rather than modulations related to the specific task-based process under consideration (in our case, motor learning). In this regard, we are sympathetic to the views of Richard Andersen and colleagues who have eloquently stated that “The study of how attention interacts with other neural processing systems is a most important endeavor. However, we think that over-generalizing attention to encompass a large variety of different neural processes weakens the concept and undercuts the ability to develop a robust understanding of other cognitive functions.” (Andersen & Cui, 2007, Neuron). In short, it appears that different fields/researchers have alternate views on the usefulness of attention as an explanatory construct (see also articles from Hommel et al., 2019, “No one knows what attention is”, and Wu, 2023, “We know what attention is!”), and we personally don’t have a dog in this fight. We only highlight these issues to draw attention (no pun intended) that it is not trivial to separate these different neural processes during a motor learning study.

      Nevertheless, we do believe these are important points worth flagging for the reader in our paper, as they might have similar questions. To this end, we have now included in our Discussion section the following text:

      “It is also possible that some of these task-related shifts in connectivity relate to shifts in task-general processes, such as changes in the allocation of attentional resources (Bédard and Song, 2013; Rosenberg et al., 2016) or overall cognitive engagement (Aben et al., 2020), which themselves play critical roles in shaping learning (Codol et al., 2018; Holland et al., 2018; Song, 2019; Taylor and Thoroughman, 2008, 2007; for a review of these topics, see Tsay et al., 2023). Such processes are particularly important during the earlier phases of learning when sensorimotor contingencies need to be established. While these remain questions for future work, our data nevertheless suggest that this shift in connectivity may be enabled through the PMC.”

      Finally, we should note that, at the end of testing, we did not assess participants' awareness of the manipulation (i.e., that they were, in fact, being rewarded based on a mirror image path). In hindsight, this would have been a good idea and provided some value to the current project. Nevertheless, it seems clear that, based on several of the learning profiles observed (e.g., subjects who exhibited very rapid learning during the Early Learning phase, more on this below), that many individuals became aware of a shape approximating the rewarded path. Note that we have included new figures (see our responses below) that give a better example of what fast versus slower learning looks like. In addition, we now note in our Methods that we did not probe participants about their subjective awareness re: the perturbation:

      “Note that, at the end of testing, we did not assess participants’ awareness of the manipulation (i.e., that they were, in fact, being rewarded based on a mirror image path of the visible path).”

      Recommendation #2: Provide more behavioral quantification.

      (2a) The authors chose to only plot the average learning score in Figure 1D, without an indication of movement variability. I think this is quite important, to give the reader an impression of how variable the movements were at baseline, during early learning, and over the course of learning. There is evidence that baseline variability influences the 'detectability' of imposed rotations (in the case of adaptation learning), which could be relevant here. Shading the plots by movement variability would also be important to see if there was some refinement of the moment after participants performed at the ceiling (which seems to be the case ~ after trial 150). This is especially worrying given that in Fig 6A there is a clear indication that there is a large difference between subjects' solutions on the task. One subject exhibits almost a one-shot learning curve (reaching a score of 75 after one or two trials), whereas others don't seem to really learn until the near end. What does this between-subject variability mean for the authors' hypothesized neural processes?

      In line with these recommendations, we have now provided much better behavioral quantification of subject-level performance in both the main manuscript and supplementary material. For instance, in a new supplemental Figure 1 (shown below), we now include mean subject (+/- SE) reaction times (RTs), movement times (MTs) and movement path variability (our computing of these measures are now defined in our Methods section).

      As can be seen in the figure, all three of these variables tended to decrease over the course of the study, though we note there was a noticeable uptick in both RTs and MTs from the Baseline to Early learning phase, once subjects started receiving trial-by-trial reward feedback based on their movements. With respect to path variability, it is not obvious that there was a significant refinement of the paths created during late learning (panel D below), though there was certainly a general trend for path variability to decrease over learning.

      Author response image 3.

      Behavioral measures of learning across the task. (A-D) shows average participant reward scores (A), reaction times (B), movement times (C) and path variability (D) over the course of the task. In each plot, the black line denotes the mean across participants and the gray banding denotes +/- 1 SEM. The three equal-length task epochs for subsequent neural analyses are indicated by the gray shaded boxes.

      In addition to these above results, we have also created a new Figure 6 in the main manuscript, which now solely focuses on individual differences in subject learning (see below). Hopefully, this figure clarifies key features of the task and its reward structure, and also depicts (in movement trajectory space) what fast versus slow learning looks like in the task. Specifically, we believe that this figure now clearly delineates for the reader the mapping between movement trajectory and the reward score feedback presented to participants, which appeared to be a source of confusion based on the reviewers’ comments below. As can be clearly observed in this figure, trajectories that approximated the ‘visible path’ (black line) resulted in fairly mediocre scores (see score color legend at right), whereas trajectories that approximated the ‘reward path’ (dashed black line, see trials 191-200 of the fast learner) resulted in fairly high scores. This figure also more clearly delineates how fPCA loadings derived from our functional data analysis were used to derive subject-level learning scores (panel C).

      Author response image 4.

      Individual differences in subject learning performance. (A) Examples of a good learner (bordered in green) and poor learner (bordered in red). (B) Individual subject learning curves for the task. Solid black line denotes the mean across all subjects whereas light gray lines denote individual participants. The green and red traces denote the learning curves for the example good and poor learners denoted in A. (C) Derivation of subject learning scores. We performed functional principal component analysis (fPCA) on subjects’ learning curves in order to identify the dominant patterns of variability during learning. The top component, which encodes overall learning, explained the majority of the observed variance (~75%). The green and red bands denote the effect of positive and negative component scores, respectively, relative to mean performance. Thus, subjects who learned more quickly than average have a higher loading (in green) on this ‘Learning score’ component than subjects who learned more slowly (in red) than average. The plot at right denotes the loading for each participant (open circles) onto this Learning score component.

      The reviewers note that there are large individual differences in learning performance across the task. This was clearly our hope when designing the reward structure of this task, as it would allow us to further investigate the neural correlates of these individual differences (indeed, during pilot testing, we sought out a reward structure to the task that would allow for these intersubject differences). The subjects who learn early during the task end up having higher fPCA scores than the subjects who learn more gradually (or learn the task late). From our perspective, these differences are a feature, and not a bug, and they do not negate any of our original interpretations. That is, subjects who learn earlier on average tend to contract their DAN-A network during the early learning phase whereas subjects who learn more slowly on average (or learn late) instead tend to contract their DAN-A network during late learning (Fig. 7).

      (2b) In the methods, the authors stated that they scaled the score such that even a perfectly traced visible path would always result in an imperfect score of 40 patients. What happens if a subject scores perfectly on the first try (which seemed to have happened for the green highlighted subject in Fig 6A), but is then permanently confronted with a score of 40 or below? Wouldn't this result in an error-clamp-like (error-based motor adaptation) design for this subject and all other high performers, which would vastly differ from the task demands for the other subjects? How did the authors factor in the wide between-subject variability?

      We think the reviewers may have misinterpreted the reward structure of the task, and we apologize for not being clearer in our descriptions. The reward score that subjects received after each trial was based on how well they traced the mirror-image of the visible path. However, all the participant can see on the screen is the visible path. We hope that our inclusion of the new Figure 6 (shown above) makes the reward structure of the task, and its relationship to movement trajectories, much clearer. We should also note that, even for the highest performing subject (denoted in Fig. 6), it still required approximately 20 trials for them to reach asymptote performance.

      (2c) The study would benefit from a more detailed description of participants' behavioral performance during the task. Specifically, it is crucial to understand how participants' motor skills evolve over time. Information on changes in movement speed, accuracy, and other relevant behavioral metrics would enhance the understanding of the relationship between behavior and brain activity during the learning process. Additionally, please clarify whether the display on the screen was presented continuously throughout the entire trial or only during active movement periods. Differences in display duration could potentially impact the observed differences in brain activity during learning.

      We hope that with our inclusion of the new Supplementary Figure 1 (shown above) this addresses the reviewers’ recommendation. Generally, we find that RTs, MTs and path variability all decrease over the course of the task. We think this relates to the early learning phase being more attentionally demanding and requiring more conscious effort, than the later learning phases.

      Also, yes, the visible path was displayed on the screen continuously throughout the trial, and only disappeared at the 4.5 second mark of each trial (when the screen was blanked and the data was saved off for 1.5 seconds prior to commencement of the next trial; 6 seconds total per trial). Thus, there were no differences in display duration across trials and phases of the task. We have now clarified this in the Methods section, where we now write the following:

      “When the cursor reached the target distance, the target changed color from red to green to indicate that the trial was completed. Importantly, other than this color change in the distance marker, the visible curved path remained constant and participants never received any feedback about the position of their cursor.”

      (2d) It is unclear from plots 6A, 6B, and 1D how the scale of the behavioral data matches with the scaling of the scores. Are these the 'real' scores, meaning 100 on the y-axis would be equivalent to 40 in the task? Why then do all subjects reach an asymptote at 75? Or is 75 equivalent to 40 and the axis labels are wrong?

      As indicated above, we clearly did a poor job of describing the reward structure of our task in our original paper, and we now hope that our inclusion of Figure 6 makes things clear. A ‘40’ score on the y-axis would indicate that a subject has perfectly traced the visible path whereas a perfect ‘100’ score would indicate that a subject has perfectly traced the (hidden) mirror image path.

      The fact that several of the subjects reach asymptote around 75 is likely a byproduct of two factors. Firstly, the subjects performed their movements in the absence of any visual error feedback (they could not see the position of a cursor that represented their hand position), which had the effect of increasing motor variability in their actions from trial to trial. Secondly, there appears to be an underestimation among subjects regarding the curvature of the concealed, mirror-image path (i.e., that the rewarded path actually had an equal but opposite curvature to that of the visible path). This is particularly evident in the case of the top-performing subject (illustrated in Figure 6A) who, even during late learning, failed to produce a completely arched movement.

      (2e) Labeling of Contrasts: There is a consistent issue with the labeling of contrasts in the presented figures, causing confusion. While the text refers to the difference as "baseline to early learning," the label used in figures, such as Figure 4, reads "baseline > early." It is essential to clarify whether the presented contrast is indeed "baseline > early" or "early > baseline" to avoid any misinterpretation.

      We thank the reviewers for catching this error. Indeed, the intended label was Early > Baseline, and this has now been corrected throughout.

      Recommendation #3. Clarify which motor learning mechanism(s) are at play.

      (3a) Participants were performing at a relatively low level, achieving around 50-60 points by the end of learning. This outcome may not be that surprising, given that reward-based learning might have a substantial explicit component and may also heavily depend on reasoning processes, beyond reinforcement learning or contextual recall (Holland et al., 2018; Tsay et al., 2023). Even within our own data, where explicit processes are isolated, average performance is low and many individuals fail to learn (Brudner et al., 2016; Tsay et al., 2022). Given this, many participants in the current study may have simply given up. A potential indicator of giving up could be a subset of participants moving straight ahead in a rote manner (a heuristic to gain moderate points). Consequently, alterations in brain networks may not reflect exploration and exploitation strategies but instead indicate levels of engagement and disengagement. Could the authors plot the average trajectory and the average curvature changes throughout learning? Are individuals indeed defaulting to moving straight ahead in learning, corresponding to an average of 50-60 points? If so, the interpretation of brain activity may need to be tempered.

      We can do one better, and actually give you a sense of the learning trajectories for every subject over time. In the figure below, which we now include as Supplementary Figure 2 in our revision, we have plotted, for each subject, a subset of their movement trajectories across learning trials (every 10 trials). As can be seen in the diversity of these trajectories, the average trajectory and average curvature would do a fairly poor job of describing the pattern of learning-related changes across subjects. Moreover, it is not obvious from looking at these plots the extent to which poor learning subjects (i.e., subjects who never converge on the reward path) actually ‘give up’ in the task — rather, many of these subjects still show some modulation (albeit minor) of their movement trajectories in the later trials (see the purple and pink traces). As an aside, we are also not entirely convinced that straight ahead movements, which we don’t find many of in our dataset, can be taken as direct evidence that the subject has given up.

      Author response image 5

      Variability in learning across subjects. Plots show representative trajectory data from each subject (n=36) over the course of the 200 learning trials. Coloured traces show individual trials over time (each trace is separated by ten trials, e.g., trial 1, 10, 20, 30, etc.) to give a sense of the trajectory changes throughout the task (20 trials in total are shown for each subject).

      We should also note that we are not entirely opposed to the idea of describing aspects of our findings in terms of subject engagement versus disengagement over time, as such processes are related at some level to exploration (i.e., cognitive engagement in finding the best solution) and exploitation (i.e., cognitively disengaging and automating one’s behavior). As noted in our reply to Recommendation #1 above, we now give some consideration of these explanations in our Discussion section, where we now write:

      “It is also possible that these task-related shifts in connectivity relates to shifts in task-general processes, such as changes in the allocation of attentional resources (Bédard and Song, 2013; Rosenberg et al., 2016) or overall cognitive engagement (Aben et al., 2020), which themselves play critical roles in shaping learning (Codol et al., 2018; Holland et al., 2018; Song, 2019; Taylor and Thoroughman, 2008, 2007; for a review of these topics, see Tsay et al., 2023). Such processes are particularly important during the earlier phases of learning when sensorimotor contingencies need to be established. While these remain questions for future work, our data nevertheless suggest that this shift in connectivity may be enabled through the PMC.”

      (3b) The authors are mixing two commonly used paradigms, reward-based learning, and motor adaptation, but provide no discussion of the different learning processes at play here. Which processes were they attempting to probe? Making this explicit would help the reader understand which brain regions should be implicated based on previous literature. As it stands, the task is hard to interpret. Relatedly, there is a wealth of literature on explicit vs implicit learning mechanisms in adaptation tasks now. Given that the authors are specifically looking at brain structures in the cerebral cortex that are commonly associated with explicit and strategic learning rather than implicit adaptation, how do the authors relate their findings to this literature? Are the learning processes probed in the task more explicit, more implicit, or is there a change in strategy usage over time? Did the authors acquire data on strategies used by the participants to solve the task? How does the baseline variability come into play here?

      As noted in our paper, our task was directly inspired by the reward-based motor learning tasks developed by Dam et al., 2013 (Plos One) and Wu et al., 2014 (Nature Neuroscience). What drew us to these tasks is that they allowed us to study the neural bases of reward-based learning mechanisms in the absence of subjects also being able to exploit error-based mechanisms to achieve learning. Indeed, when first describing the task in the Results section of our paper we wrote the following:

      “Importantly, because subjects received no visual feedback about their actual finger trajectory and could not see their own hand, they could only use the score feedback — and thus only reward-based learning mechanisms — to modify their movements from one trial to the next (Dam et al., 2013; Wu et al., 2014).”

      If the reviewers are referring to ‘motor adaptation’ in the context in which that terminology is commonly used — i.e., the use of sensory prediction errors to support error-based learning — then we would argue that motor adaptation is not a feature of the current study. It is true that in our study subjects learn to ‘adapt’ their movements across trials, but this shaping of the movement trajectories must be supported through reinforcement learning mechanisms (and, of course, supplemented by the use of cognitive strategies as discussed in the nice review by Tsay et al., 2023). We apologize for not being clearer in our paper about this key distinction and we have now included new text in the introduction to our Results to directly address this:

      “Importantly, because subjects received no visual feedback about their actual finger trajectory and could not see their own hand, they could only use the score feedback — and thus only reward-based learning mechanisms — to modify their movements from one trial to the next (Dam et al., 2013; Wu et al., 2014). That is, subjects could not use error-based learning mechanisms to achieve learning in our study, as this form of learning requires sensory errors that convey both the change in direction and magnitude needed to correct the movement.”

      With this issue aside, we are well aware of the established framework for thinking about sensorimotor adaptation as being composed of a combination of explicit and implicit components (indeed, this has been a central feature of several of our other recent neuroimaging studies that have explored visuomotor rotation learning, e.g., Gale et al., 2022 PNAS, Areshenkoff et al., 2022 elife, Standage et al., 2023 Cerebral Cortex). However, there has been comparably little work done on these parallel components within the domain of reinforcement learning tasks (though see Codol et al., 2018; Holland et al., 2018, van Mastrigt et al., 2023; see also the Tsay et al., 2023 review), and as far as we can tell, nothing has been done to date in the reward-based motor learning area using fMRI. By design, we avoided using descriptors of ‘explicit’ or ‘implicit’ in our study because our experimental paradigm did not allow a separate measurement of those two components to learning during the task. Nevertheless, it seems clear to us from examining the subjects’ learning curves (see supplementary figure 2 above), that individuals who learn very quickly are using strategic processes (such as action exploration to identify the best path) to enhance their learning. As we noted in an above response, we did not query subjects after the fact about their strategy use, which admittedly was a missed opportunity on our part.

      Author response image 6.

      With respect to the comment on baseline variability and its relationship to performance, this is an interesting idea and one that was explored in the Wu et al., 2014 Nature Neuroscience paper. Prompted by the reviewers, we have now explored this idea in the current data set by testing for a relationship between movement path variability during baseline trials (all 70 baseline trials, see Supplementary Figure 1D above for reference) and subjects’ fPCA score on our learning task. However, when we performed this analysis, we did not observe a significant positive relationship between baseline variability and subject performance. Rather, we actually found a trend towards a negative relationship (though this was non-significant; r=-0.2916, p=0.0844). Admittedly, we are not sure what conclusions can be drawn from this analysis, and in any case, we believe it to be tangential to our main results. We provide the results (at right) for the reviewers if they are interested. This may be an interesting avenue for exploration in future work.

      Recommendation #4: Provide stronger justification for brain imaging methods.

      (4a) Observing how brain activity varies across these different networks is remarkable, especially how sensorimotor regions separate and then contract with other, more cognitive areas. However, does the signal-to-noise ratio in each area/network influence manifold eccentricity and limit the possible changes in eccentricity during learning? Specifically, if a region has a low signal-to-noise ratio, it might exhibit minimal changes during learning (a phenomenon perhaps relevant to null manifold changes in the striatum due to low signal-to-noise); conversely, regions with higher signal-to-noise (e.g., motor cortex in this sensorimotor task) might exhibit changes more easily detected. As such, it is unclear how to interpret manifold changes without considering an area/network's signal-to-noise ratio.

      We appreciate where these concerns are coming from. First, we should note that the timeseries data used in our analysis were z-transformed (mean zero, 1 std) to allow normalization of the signal both over time and across regions (and thus mitigate the possibility that the changes observed could simply reflect mean overall signal changes across different regions). Nevertheless, differences in signal intensity across brain regions — particularly between cortex and striatum — are well-known, though it is not obvious how these differences may manifest in terms of a task-based modulation of MR signals.

      To examine this issue in the current data set, we extracted, for each subject and time epoch (Baseline, Early and Late learning) the raw scanner data (in MR arbitrary units, a.u.) for the cortical and striatal regions and computed the (1) mean signal intensity, (2) standard deviation of the signal (Std) and (3) temporal signal to noise ratio (tSNR; calculated by mean/Std). Note that in the fMRI connectivity literature tSNR is often the preferred SNR measure as it normalizes the mean signal based on the signal’s variability over time, thus providing a general measure of overall ‘signal quality’. The results of this analysis, averaged across subjects and regions, is shown below.

      Author response image 7.

      Note that, as expected, the overall signal intensity (left plot) of cortex is higher than in the striatum, reflecting the closer proximity of cortex to the receiver coils in the MR head coil. In fact, the signal intensity in cortex is approximately 38% higher than that in the striatum (~625 - 450)/450). However, the signal variation in cortex is also greater than striatum (middle plot), but in this case approximately 100% greater (i.e., (~5 - 2.5)/2.5)). The result of this is that the tSNR (mean/std) for our data set and the ROI parcellations we used is actually greater in the striatum than in cortex (right plot). Thus, all else being equal, there seems to have been sufficient tSNR in the striatum for us to have detected motor-learning related effects. As such, we suspect the null effects for the striatum in our study actually stem from two sources.

      The first likely source is the relatively lower number of striatal regions (12) as compared to cortical regions (998) used in our analysis, coupled with our use of PCA on these data (which, by design, identifies the largest sources of variation in connectivity). In future studies, this unbalance could be rectified by using finer parcellations of the striatum (even down to the voxel level) while keeping the same parcellation of cortex (i.e., equate the number of ‘regions’ in each of striatum and cortex). The second likely source is our use of a striatal atlas (the Harvard-Oxford atlas) that divides brain regions based on their neuroanatomy rather than their function. In future work, we plan on addressing this latter concern by using finer, more functionally relevant parcellations of striatum (such as in Tian et al., 2020, Nature Neuroscience). Note that we sought to capture these interrelated possible explanations in our Discussion section, where we wrote the following:

      “While we identified several changes in the cortical manifold that are associated with reward-based motor learning, it is noteworthy that we did not observe any significant changes in manifold eccentricity within the striatum. While clearly the evidence indicates that this region plays a key role in reward-guided behavior (Averbeck and O’Doherty, 2022; O’Doherty et al., 2017), there are several possible reasons why our manifold approach did not identify this collection of brain areas. First, the relatively small size of the striatum may mean that our analysis approach was too coarse to identify changes in the connectivity of this region. Though we used a 3T scanner and employed a widely-used parcellation scheme that divided the striatum into its constituent anatomical regions (e.g., hippocampus, caudate, etc.), both of these approaches may have obscured important differences in connectivity that exist within each of these regions. For example, areas such the hippocampus and caudate are not homogenous areas but themselves exhibit gradients of connectivity (e.g., head versus tail) that can only be revealed at the voxel level (Tian et al., 2020; Vos de Wael et al., 2021). Second, while our dimension reduction approach, by design, aims to identify gradients of functional connectivity that account for the largest amounts of variance, the limited number of striatal regions (as compared to cortex) necessitates that their contribution to the total whole-brain variance is relatively small. Consistent with this perspective, we found that the low-dimensional manifold architecture in cortex did not strongly depend on whether or not striatal regions were included in the analysis (see Supplementary Fig. 6). As such, selective changes in the patterns of functional connectivity at the level of the striatum may be obscured using our cortex x striatum dimension reduction approach. Future work can help address some of these limitations by using both finer parcellations of striatal cortex (perhaps even down to the voxel level)(Tian et al., 2020) and by focusing specifically on changes in the interactions between the striatum and cortex during learning. The latter can be accomplished by selectively performing dimension reduction on the slice of the functional connectivity matrix that corresponds to functional coupling between striatum and cortex.”

      (4b) Could the authors clarify how activity in the dorsal attention network (DAN) changes throughout learning, and how these changes also relate to individual differences in learning performance? Specifically, on average, the DAN seems to expand early and contract late, relative to the baseline. This is interpreted to signify that the DAN exhibits lesser connectivity followed by greater connectivity with other brain regions. However, in terms of how these changes relate to behavior, participants who go against the average trend (DAN exhibits more contraction early in learning, and expansion from early to late) seem to exhibit better learning performance. This finding is quite puzzling. Does this mean that the average trend of expansion and contraction is not facilitative, but rather detrimental, to learning? [Another reviewer added: The authors do not state any explicit hypotheses, but only establish that DMN coordinates activity among several regions. What predictions can we derive from this? What are the authors looking for in the data? The work seems more descriptive than hypothesis-driven. This is fine but should be clarified in the introduction.]

      These are good questions, and we are glad the reviewers appreciated the subtlety here. The reviewers are indeed correct that the relationship of the DAN-A network to behavioral performance appears to go against the grain of the group-level results that we found for the entire DAN network (which we note is composed of both the DAN-A and DAN-B networks). That is, subjects who exhibited greater contraction from Baseline to Early learning and likewise, greater expansion from Early to Late learning, tended to perform better in the task (according to our fPCA scores). However, on this point it is worth noting that it was mainly the DAN-B network which exhibited group-level expansion from Baseline to Early Learning whereas the DAN-A network exhibited negligible expansion. This can be seen in Author response image 8 below, which shows the pattern of expansion and contraction (as in Fig. 4), but instead broken down into the 17-network parcellation. The red asterisk denotes the expansion from Baseline to Early learning for the DAN-B network, which is much greater than that observed for the DAN-A network (which is basically around the zero difference line).

      Author response image 8.

      Thus, it appears that the DAN-A and DAN-B networks are modulated to a different extent during the task, which likely contributes to the perceived discrepancy between the group-level effects (reported using the 7-network parcellation) and the individual differences effects (reported using the finer 17-network parcellation). Based on the reviewers’ comments, this seems like an important distinction to clarify in the manuscript, and we have now described this nuance in our Results section where we now write:

      “...Using this permutation testing approach, we found that it was only the change in eccentricity of the DAN-A network that correlated with Learning score (see Fig. 7C), such that the more the DAN-A network decreased in eccentricity from Baseline to Early learning (i.e., contracted along the manifold), the better subjects performed at the task (see Fig. 7C, scatterplot at right). Consistent with the notion that changes in the eccentricity of the DAN-A network are linked to learning performance, we also found the inverse pattern of effects during Late learning, whereby the more that this same network increased in eccentricity from Early to Late learning (i.e., expanded along the manifold), the better subjects performed at the task (Fig. 7D). We should note that this pattern of performance effects for the DAN-A — i.e., greater contraction during Early learning and greater expansion during Late learning being associated with better learning — appears at odds with the group-level effects described in Fig. 4A and B, where we generally find the opposite pattern for the entire DAN network (composed of the DAN-A and DAN-B subnetworks). However, this potential discrepancy can be explained when examining the changes in eccentricity using the 17-network parcellation (see Supplementary Figure 8). At this higher resolution level we find that these group-level effects for the entire DAN network are being largely driven by eccentricity changes in the DAN-B network (areas in anterior superior parietal cortex and premotor cortex), and not by mean changes in the DAN-A network. By contrast, our present results suggest that it is the contraction and expansion of areas of the DAN-A network (and not DAN-B network) that are selectively associated with differences in subject learning performance.”

      Finally, re: the reviewers’ comments that we do not state any explicit hypotheses etc., we acknowledge that, beyond our general hypothesis stated at the outset about the DMN being involved in reward-based motor learning, our study is quite descriptive and exploratory in nature. Such little work has been done in this research area (i.e., using manifold learning approaches to study motor learning with fMRI) that it would be disingenuous to have any stronger hypotheses than those stated in our Introduction. Thus, to make the exploratory nature of our study clear to the reader, we have added the following text (in red) to our Introduction:

      “Here we applied this manifold approach to explore how brain activity across widely distributed cortical and striatal systems is coordinated during reward-based motor learning. We were particularly interested in characterizing how connectivity between regions within the DMN and the rest of the brain changes as participants shift from learning the relationship between motor commands and reward feedback, during early learning, to subsequently using this information, during late learning. We were also interested in exploring whether learning-dependent changes in manifold structure relate to variation in subject motor performance.”

      We hope these changes now make it obvious the intention of our study.

      (4c) The paper examines a type of motor adaptation task with a reward-based learning component. This, to me, strongly implicates the cerebellum, given that it has a long-established crucial role in adaptation and has recently been implicated in reward-based learning (see work by Wagner & Galea). Why is there no mention of the cerebellum and why it was left out of this study? Especially given that the authors state in the abstract they examine cortical and subcortical structures. It's evident from the methods that the authors did not acquire data from the cerebellum or had too small a FOV to fully cover it (34 slices at 4 mm thickness 136 mm which is likely a bit short to fully cover the cerebellum in many participants). What was the rationale behind this methodological choice? It would be good to clarify this for the reader. Related to this, the authors need to rephrase their statements on 'whole-brain' connectivity matrices or analyses - it is not whole-brain when it excludes the cerebellum.

      As we noted above, we do not believe this task to be a motor adaptation task, in the sense that subjects are not able to use sensory prediction errors (and thus error-based learning mechanisms) to improve their performance. Rather, by denying subjects this sensory error feedback they are only able to use reinforcement learning processes, along with cognitive strategies (nicely covered in Tsay et al., 2023), to improve performance. Nevertheless, we recognize that the cerebellum has been increasingly implicated in facets of reward-based learning, particularly within the rodent domain (e.g., Wagner et al., 2017; Heffley et al., 2018; Kostadinov et al., 2019, etc.). In our study, we did indeed collect data from the cerebellum but did not include it in our original analyses, as we wanted (1) the current paper to build on prior work in the human and macaque reward-learning domain (which focuses solely on striatum and cortex, and which rarely discusses cerebellum, see Averbeck & O’Doherty, 2022 & Klein-Flugge et al., 2022 for recent reviews), and, (2) allow this to be a more targeted focus of future work (specifically we plan on focusing on striatal-cerebellar interactions during learning, which are hypothesized based on the neuroanatomical tract tracing work of Bostan and Strick, etc.). We hope the reviewers respect our decisions in this regard.

      Nevertheless, we acknowledge that based on our statements about ‘whole-brain’ connectivity and vagueness about what we mean by ‘subcortex,’ that this may be confusing for the reader. We have now removed and/or corrected such references throughout the paper (however, note that in some cases it is difficult to avoid reference to “whole-brain” — e.g., “whole-brain correlation map” or “whole-brain false discovery rate correction”, which is standard terminology in the field).

      In addition, we are now explicit in our Methods section that the cerebellum was not included in our analyses.

      “Each volume comprised 34 contiguous (no gap) oblique slices acquired at a ~30° caudal tilt with respect to the plane of the anterior and posterior commissure (AC-PC), providing whole-brain coverage of the cerebrum and cerebellum. Note that for the current study, we did not examine changes in cerebellar activity during learning.”

      (4d) The authors centered the matrices before further analyses to remove variance associated with the subject. Why not run a PCA on the connectivity matrices and remove the PC that is associated with subject variance? What is the advantage of first centering the connectivity matrices? Is this standard practice in the field?

      Centering in some form has become reasonably common in the functional connectivity literature, as there is considerable evidence that task-related (or cognitive) changes in whole-brain connectivity are dwarfed by static, subject-level differences (e.g., Gratton, et al, 2018, Neuron). If covariance matrices were ordinary scalar values, then isolating task-related changes could be accomplished simply by subtracting a baseline scan or mean score; but because the space of covariance matrices is non-Euclidean, the actual computations involved in this subtraction are more complex (see our Methods). However, fundamentally (and conceptually) our procedure is simply ordinary mean-centering, but adapted to this non-Euclidean space. Despite the added complexity, there is considerable evidence that such computations — adapted directly to the geometry of the space of covariance matrices — outperform simpler methods, which treat covariance matrices as arrays of real numbers (e.g. naive substraction, see Dodero et al. & Ng et al., references below). Moreover, our previous work has found that this procedure works quite well to isolate changes associated with different task conditions (Areshenkoff et al., 2021, Neuroimage; Areshenkoff et al., 2022, elife).

      Although PCA can be adapted to work well with covariance matrix valued data, it would at best be a less direct solution than simply subtracting subjects' mean connectivity. This is because the top components from applying PCA would be dominated by both subject-specific effects (not of interest here), and by the large-scale connectivity structure typically observed in component based analyses of whole-brain connectivity (i.e. the principal gradient), whereas changes associated with task-condition (the thing of interest here) would be buried among the less reliable components. By contrast, our procedure directly isolates these task changes.

      References cited above:

      Dodero, L., Minh, H. Q., San Biagio, M., Murino, V., & Sona, D. (2015, April). Kernel-based classification for brain connectivity graphs on the Riemannian manifold of positive definite matrices. In 2015 IEEE 12th international symposium on biomedical imaging (ISBI) (pp. 42-45). IEEE.

      Ng, B., Dressler, M., Varoquaux, G., Poline, J. B., Greicius, M., & Thirion, B. (2014). Transport on Riemannian manifold for functional connectivity-based classification. In Medical Image Computing and Computer-Assisted Intervention–MICCAI 2014: 17th International Conference, Boston, MA, USA, September 14-18, 2014, Proceedings, Part II 17 (pp. 405-412). Springer International Publishing.

      (4e) Seems like a missed opportunity that the authors just use a single, PCA-derived measure to quantify learning, where multiple measures could have been of interest, especially given that the introduction established some interesting learning-related concepts related to exploration and exploitation, which could be conceptualized as movement variability and movement accuracy. It is unclear why the authors designed a task that was this novel and interesting, drawing on several psychological concepts, but then chose to ignore these concepts in the analysis.

      We were disappointed to hear that the reviewers did not appreciate our functional PCA-derived measure to quantify subject learning. This is a novel data-driven analysis approach that we have previously used with success in recent work (e.g., Areshenkoff et al., 2022, elife) and, from our perspective, we thought it was quite elegant that we were able to describe the entire trajectory of learning across all participants along a single axis that explained the majority (~75%) of the variance in the patterns of behavioral learning data. Moreover, the creation of a single behavioral measure per participant (what we call a ‘Learning score’, see Fig. 6C) helped simplify our brain-behavior correlation analyses considerably, as it provided a single measure that accounts for the natural auto-correlation in subjects’ learning curves (i.e., that subjects who learn quickly also tend to be better overall learners by the end of the learning phase). It also avoids the difficulty (and sometimes arbitrariness) of having to select specific trial bins for behavioral analysis (e.g., choosing the first 5, 10, 20 or 25 trials as a measure of ‘early learning’, and so on). Of course, one of the major alternatives to our approach would have involved fitting an exponential to each subject’s learning curves and taking measures like learning rate etc., but in our experience we have found that these types of models don’t always fit well, or derive robust/reliable parameters at the individual subject level. To strengthen the motivation for our approach, we have now included the following text in our Results:

      “To quantify this variation in subject performance in a manner that accounted the auto-correlation in learning performance over time (i.e., subjects who learned more quickly tend to exhibit better performance by the end of learning), we opted for a pure data-driven approach and performed functional principal component analysis (fPCA; (Shang, 2014)) on subjects’ learning curves. This approach allowed us to isolate the dominant patterns of variability in subject’s learning curves over time (see Methods for further details; see also Areshenkoff et al., 2022).”

      In any case, the reviewers may be pleased to hear that in current work in the lab we are using more model-based approaches to attempt to derive sets of parameters (per participant) that relate to some of the variables of interest described by the reviewers, but that we relate to much more dynamical (shorter-term) changes in brain activity.

      (4f) Overall Changes in Activity: The manuscript should delve into the potential influence of overall changes in brain activity on the results. The choice of using Euclidean distance as a metric for quantifying changes in connectivity is sensitive to scaling in overall activity. Therefore, it is crucial to discuss whether activity in task-relevant areas increases from baseline to early learning and decreases from early to late learning, or if other patterns emerge. A comprehensive analysis of overall activity changes will provide a more complete understanding of the findings.

      These are good questions and we are happy to explore this in the data. However, as mentioned in our response to query 4a above, it is important to note that the timeseries data for each brain region was z-scored prior to analysis, with the aim of removing any mean changes in activity levels (note that this is a standard preprocessing step when performing functional connectivity analysis, given that mean signal changes are not the focus of interest in functional connectivity analyses).

      To further emphasize these points, we have taken our z-scored timeseries data and calculated the mean signal for each region within each task epoch (Baseline, Early and Late learning, see panel A in figure below). The point of showing this data (where each z-score map looks near identical across the top, middle and bottom plots) is to demonstrate just how miniscule the mean signal changes are in the z-scored timeseries data. This point can also be observed when plotting the mean z-score signal across regions for each epoch (see panel B in figure below). Here we find that Baseline and Early learning have a near identical mean activation level across regions (albeit with slightly different variability across subjects), whereas there is a slight increase during late learning — though it should be noted that our y-axis, which measures in the thousandths, really magnifies this effect.

      To more directly address the reviewers’ comments, using the z-score signal per region we have also performed the same statistical pairwise comparisons (Early > Baseline and Late>Early) as we performed in the main manuscript Fig. 4 (see panel C in Author response image 9 below). In this plot, areas in red denote an increase in activity from Baseline to Early learning (top plot) and from Early to Late learning (bottom plot), whereas areas in blue denote a decrease for those same comparisons. The important thing to emphasize here is that the spatial maps resulting from this analysis are generally quite different from the maps of eccentricity that we report in Fig. 4 in our paper. For instance, in the figure below, we see significant changes in the activity of visual cortex between epochs but this is not found in our eccentricity results (compare with Fig. 4). Likewise, in our eccentricity results (Fig. 4), we find significant changes in the manifold positioning of areas in medial prefrontal cortex (MPFC), but this is not observed in the activation levels of these regions (panel C below). Again, we are hesitant to make too much of these results, as the activation differences denoted as significant in the figure below are likely to be an effect on the order of thousandths of a z-score (e.g., 0.002 > 0.001), but this hopefully assuages reviewers’ concerns that our manifold results are solely attributable to changes in overall activity levels.

      We are hesitant to include the results below in our paper as we feel that they don’t add much to the interpretation (as the purpose of z-scoring was to remove large activation differences). However, if the reviewers strongly believe otherwise, we would consider including them in the supplement.

      Author response image 9.

      Examination of overall changes in activity across regions. (A) Mean z-score maps across subjects for the Baseline (top), Early Learning (middle) and Late learning (bottom) epochs. (B) Mean z-score across brain regions for each epoch. Error bars represent +/- 1 SEM. (C) Pairwise contrasts of the z-score signal between task epochs. Positive (red) and negative (blue) values show significant increases and decreases in z-score signal, respectively, following FDR correction for region-wise paired t-tests (at q<0.05).

    1. Author response:

      The following is the authors’ response to the original reviews.

      Reviewer 1:

      In the article titled "Polyphosphate discriminates protein conformational ensembles more efficiently than DNA promoting diverse assembly and maturation behaviors," Goyal and colleagues investigate the role of negatively charged biopolymers, i.e., polyphosphate (polyP) and DNA, play in phase separation of cytidine repressor (CytR) and fructose repressor (FruR). The authors find that both negative polymers drive the formation of metastable protein/polymer condensates. However, polyPdriven condensates form more gel- or solid-like structures over time while DNA-driven condensates tend to dissipate over time. The authors link this disparate condensate behavior to polyP-induced structures within the enzymes. Specifically, they observe the formation of polyproline II-like structures within two tested enzyme variants in the presence of polyP. Together their results provide a unique insight into the physical and structural mechanism by which two unique negatively charged polymers can induce distinct phase transitions with the same protein. This study will be a welcomed addition to the condensate field and provide new molecular insights into how binding partner-induced structural changes within a given protein can affect the mesoscale behavior of condensates. The concerns outlined below are meant to strengthen the manuscript.

      Recommendation:

      We value the reviewer’s positive comments and appreciate time taken to provide detailed feedback that has certainly helped improve our manuscript.

      Major Concerns:

      (1) The biggest concern in this manuscript lies with experiments comparing polyP45, which has a net negative charge of -47, and double-stranded DNA of 45 base pairs (as stated in the methods), which will have a net negative charge of -90. Given the dependence of phase separation and phase transitions on not only net charge but charge density, this is an important factor to consider when comparing the effect of these molecules. It is unclear how or if the authors considered these factors in the design of their experiments. Because of the factor of 2 difference in net charge over the same number of polymer chain components, i.e. a chain of 45 pi vs. a chain of 45 double-stranded base pairs, it is unclear if the results from polyP vs. DNA are directly comparable. One solution would be to repeat all DNA experiments using single-stranded DNA so that the net charge is similar to polyP over the same chain length. Another possibility would be to repeat DNA experiments using a doublestranded DNA of 23 base pairs. This would allow for a nearly equal net charge (-46 vs. -47 for polyP), but the charge density would still be 2X polyP. As it stands now, the perceived differences in DNA vs. polyP behavior may be an artifact arising from the difference in net charge and charge density between DNA and polyP.

      To address the reviewer’s concerns regarding charge density differences between polyP and DNA, we conducted an experiment using a higher DNA concentration (11.24 µM) to obtain charge equivalence between the two experiments (i.e. the total concentration of charges). As shown in Figure S5, even at higher DNA concentration, the condensates undergo progressive dissolution over time. This observation indicates that the differential maturation of condensates, arising from distinct initial protein ensembles, are governed by the intrinsic properties of polyP. Charge density (i.e. the number of charges per unit volume of the polymer), on the other hand, is an intrinsic feature of the polymer which is naturally different between DNA and polyP. In fact, the primary result of our work is our observation that polyP can discern the starting ensembles more efficiently, likely through actively engaging and interacting with the ensemble while DNA appears to be a passive player. The differences are not an artifact as they arise from fundamental features of two natural anionic polymers found within cells. In other words, the outcomes could be very different if the concentration of one polymer dominates over the other (see the response below).

      (2) One outstanding question the authors do not address relates to how mixtures of CytR or FruR, DNA, and polyP behave. In the bacterial cytoplasm, these molecules are all in the same compartment (admittedly that compartment is not well mixed due to unique condensate-driven organization). Would the authors expect to see similar effects of polyP and DNA if they were in the same solution? Perhaps the authors could run a set of experiments where they vary the ratios of DNA and polyP to probe how increased levels of "stress", i.e. increased levels of polyP vs. DNA, alter the formation and behavior of enzymatic condensates.

      Following this comment, we investigated the phase separation behavior of CytR WT in the presence of different charge ratios of polyP-DNA mixtures. As seen in Author response image 1,panel A below, the outcomes are highly sensitive to the starting concentrations: at higher charge concentration of polyP (left panel), the OD and ThT fluorescence intensity is high at lower time points, both decrease and increase again. Fluorescence microscopy images (panel B) reveal similar trends, but the more fascinating outcome are the FRAP recovery profiles which recover extremely fast and fully at zero time point (panel C) despite aggregation-like tendencies observed in ThT fluorescence assays. However, at longer time points (20 and 40 mins) the FRAP recovery is significantly weaker but recovers to ~65% at 1 hour (panel C). At high relative polyP concentrations with respect to DNA, droplets are formed first which then transition into aggregates (liquid-to-solid transition; middle image in panel A). At relatively high DNA concentrations it appears that both droplets and aggregates co-exist as both OD and ThT fluorescence are moderately high. Given these complex behaviors, we have not included the same in the current manuscript as we still do not fully understand the origins of these differences. In fact, we are planning to extend this study by exploring the combinations in detail to understand the relative roles played by the two polymers in ternary mixtures.

      Author response image 1.

      (3) In Figure 1H, the recovery trace shows the fractional recovery of DM to near WT levels. It is clear from the images that recovery of the bleached region occurs, but the overall fluorescence intensity of DM is much lower than WT, even when accounting for the difference in starting condensate sizes in the Pre-Bleach images. Shouldn't this qualitative difference in total fluorescence be reflected in the quantitative trace?

      In Figure 2H, as the reviewer rightly points out, there is a clear difference in the absolute fluorescence intensity between WT and DM condensates. We would like to clarify that the recovery traces shown in Figure 2I were normalized to the pre-bleach intensity of each individual condensate to reflect fractional recovery. This normalization is intended to highlight the relative mobility of the protein within each condensate, but it does not capture the difference in total fluorescence intensity between WT and DM.

      (4) A description of the molten-globular variant Y19A FruR should be included in the main text where the variant is introduced. There is currently no additional description of the molten-globular variant in the Supplement as suggested by the manuscript.

      Figure 6A depicts the three-dimensional structure of FruR WT, with tyrosine residues Y19 and Y28, shown in red, forming stacking interactions. In the Y19A mutant, the loss of these interactions results in little changes in secondary structure (as shown in Figure 6E) but disrupts the protein’s tertiary structure, resulting in a molten globular state. The FruR work is now published in JPCB and can be found at https://doi.org/10.1021/acs.jpcb.4c03895, and is also appropriately cited in the revised version (reference 53).

      (5) Throughout the manuscript, the authors discuss polyP and DNA being able (or unable) to "distinguish" between different variants of CytR and FruR. This is confusing and suggests that DNA or polyP can choose to bind one form over another. The authors should re-work the language in this section to better reflect their direct observations for the behavior of protein in CD experiments and condensate behavior in imaging and turbidity experiments.

      We have now modified the text where necessary. The experiments were not done in the presence of both polyP and DNA, but in isolation (protein + polyP or protein + DNA). Hence, our aim is to convey that polyP is the polymer that leads to variable outcomes because of its ability to ‘interact’ differently with the different starting ensembles.

      Minor Concerns:

      (1) For all Figures, please include the number of measurements, i.e., N = ...

      We have updated all figure legends to include the number of measurements, indicated as N = ..., as suggested.

      (2) For all Figures, please place panel labels, i.e., A, B, C, etc., in the same respective location for each panel. As currently mapped out, it is difficult to easily determine which data are associated with each panel because the IDs are in various locations.

      Due to variations in data presentation and spacing within individual plots, it was challenging to place all labels in exactly the same position without obscuring important details. We have therefore maintained the labels as they were before.

      (3) In the introduction, it would be helpful for the authors to specify exactly what is meant by chaperone. Given the context, it seems that the authors refer to the chaperone activity as one that prevents aggregation. Is this correct?

      We refer to chaperone activity specifically as the ability to prevent aggregation of proteins. We have now clarified this definition in the Introduction section of the revised manuscript.

      (4) The results for experiments shown in Figure 3 need additional setup in the text. Were these measurements taken immediately after mixing WT, DM, or P33A with polyP? If so, why do condensates immediately appear and then dissipate before ThT-detected aggregates begin forming? Or were condensates allowed to form and then transferred to a different buffer, after which measurements were taken? Without a brief description of the experimental setup, interpreting the results is difficult.

      The condensates appear immediately after adding polyP to protein solutions, indicating that the condensate phase is kinetically accessible on mixing polyP with DM or the WT. As illustrated in Figure 3A and 3B, for WT protein, the condensates undergo liquid to solid transition over the time as this likely is the most thermodynamically stable phase. Effectively, this work is to convey that it is important to look at time-dependence of even droplets when formed as they may not be the most stable phase.

      (5) Please include images of P33A over the time course of the experiment in Figure 3B.

      We have included the representative images of P33A in presence of polyP over the time in Figure 3B in the revised manuscript.

      (6) In Figures 3D, E, G, and H, please plot each measurement separately with mean and standard deviation to enable the reader to see each data point.

      We have now revised Figures 3D, E, G, and H to show individual data points along with the mean and standard deviation.

      (7) In the top paragraph on page 12, "fast-moving molecules" can be replaced with "dynamic molecules", as this offers a better description of the FRAP data.

      We have incorporated the suggested changes.

      (8) In the "Structural changes within the condensates spans over three hours" results section on page 15, the conclusion reads "In summary, we find that both the WT and the DM 'unfold' on forming condensates with polyP..." The way this is written suggests that WT and DM behave in a similar manner. Given the CD data, however, it seems that by 4 hours, DM forms alpha helices while the WT does not. This suggests that while each unfolds, the conformation at 4 hours is different. The summary should reflect these differences.

      We fully agree with the reviewer on this. The summary is now modified to include the fact the DM forms alpha helices at 4 hours while the WT does not.

      (9) At the end of the first paragraph of the results section "DNA does not discriminate the conformational ensembles" the authors should refer to Figure 2G, where they show the altered morphology of polP-P33A condensates.

      We have now included the reference to Figure 2G.

      (10) The authors refer to droplets "solubilizing" throughout the manuscript. It seems that dissolve is a better term to use. Solubilize is better associated with individual biomolecules while dissolve is better associated with condensate behavior.

      We thank the reviewer for pointing this out. We have revised the manuscript to replace “solubilize” with “dissolve”.

      (11) In Figures 5L and 5N, please change the Y-axis scale so that each curve is visible on the plot.

      We have adjusted the Y-axis scale in Figures 5L, 5M, and 5N to ensure that each curve is clearly visible and for easier comparison among the variants.

      (12) The authors should show an image of FruR WT and Y19A with DNA for a direct comparison with experiments in which FruR and polyP were used. The addition of turbidity measurements of samples shown in Figure 6D will offer another direct comparison. As written, there is no way for the author to directly compare the effects of polyP and DNA on FruR phase transitions.

      As suggested, we have now included representative images of FruR WT and Y19A with DNA (Figure 6K and 6L) to enable a direct comparison with the FruR–polyP experiments. Also, we have already shown turbidity measurements in Figure 6B and 6C corresponding to the samples shown in Figure 6D.

      Reviewer 2:

      In this study, Goyal et al demonstrate that the assembly of proteins with polyphosphate into either condensates or aggregates can reveal information on the initial protein ensemble. They show that, unlike DNA, polyphosphate is able to effectively discriminate against initial protein ensembles with different conformational heterogeneity, structure, and compactness. The authors further show that the protein native ensemble is vital on whether polyphosphate induces phase separation or aggregation, whereas DNA induces a similar outcome regardless of the initial protein ensemble. This work provides a way to improve our mechanistic understanding of how conformational transitions of proteins may regulate or drive LLPS condensate and aggregate assemblies within biological systems.

      We thank the reviewer for the favorable comments on the manuscript.

      Major Concerns:

      (1) The authors are using bacterial proteins (CytR and FruR) and solely represent polyphosphates as polyP45 (a polyphosphate with 45 Pi units). However, in bacterial systems, polyphosphates can be significantly longer (in the order of 100s to 1000 Pi units). Additionally, the experiments were run at neutral pH (7.0), and though this is fairly appropriate for the cytoplasm, volutin granules (where polyphosphates often accumulate) are typically considered slightly acidic (pH 5.5-6.5). From a physiological perspective, understanding how pH and the length of polyphosphate influence the ability to induce condensates or aggregates could be of importance.

      We appreciate the reviewer’s insightful comments regarding the physiological relevance of polyphosphate length and pH. In our current study, we used polyP45 as it is easily available commercially and we conducted our experiments at pH 7 to mimic the general cytoplasm conditions. We agree that polyphosphates in bacterial cells can be significantly longer (hundreds to thousands of Pi units) and conducting experiments at slightly more acidic environment would be physiologically relevant. We plan to use longer polyP from Regene Tiss Inc. and acidic pH to explore how polyphosphate-induced phase separation of CytR vary with pH as a part of a future study. One could imagine doing all the experiments listed in the manuscript at different pH conditions for the different variants, but this could not be a part of the current work which has a specific focus on the differences in maturation properties depending on the nature of starting ensemble. However, the pKa values of the internal hydroxyl groups is ~2.2 (DOI:10.2147/IJN.S389819) indicating that the polyP carries near identical charges in the pH range between 4-7, and hence we expect little change in the charged status of polyP. On the other hand, the protonation states of charged amino acids within CytR could vary with pH, thus influencing its assembly properties.

      (2) In the study, the longest metastable condensate induced by polyphosphate lasted approximately 3 hours before resolubilizing. It would be nice if the authors were able to generate a longer-lived condensate phase that would enable further mechanistic studies (e.g., NMR).

      We agree that generating longer-lived condensates would be highly valuable for mechanistic studies. However, the formation and stability of condensates is an intrinsic property of protein, and optimizing different conditions for a longer-lived condensate phase is beyond the scope of the current study. It is possible that the condensates are long-lived with longer polyP, but it is not clear if this would indeed be the case. We would also like to state here that while it is common to report on the liquid-to-solid transition in condensates, the intrinsic metastability of droplets (when there is no aggregation) is rarely reported. One possibility is to mutationally introduce cysteine residues and induce the formation of disulphide bridges (as done in a recent work, doi: 10.1021/jacs.4c09557) that make the condensate highly stable kinetically; however, this would also complicate the interpretation as the mechanism of condensate formation might be very different. We have therefore reported our results as an observation arising from differences in the nature of the poly-anionic polymers.

      (3) The authors showed that CytR DM (fully folded), CytR WT (minor state folded), and CytR P33A (highly disordered) with polyphosphates lead to longer-lived condensates that resolubilize, shorterlived condensates that aggregate, and immediate aggregating, respectively. Whereas FruR (folded) and FruR Y19A (molten globular) with polyphosphate induce spontaneous aggregation and short-lived condensates, respectively. I would expect FruR to be more similar to CytR DM and FruR Y19A more similar to CytR WT in terms of structure and conformational dynamics and plasticity, yet they have opposing results. This raises a bit of concern. Meaning, that though polyphosphate discriminates between the different ensembles, is it actually possible to obtain information on the initial ensemble composition?

      In the current study, we show that CytR WT (less structured) and FruR Y19A (molten globule) form short-lived condensates that aggregate. We agree with the reviewer that while CytR DM (fully folded) forms condensates that dissolve over time, FruR WT (fully folded) variant forms aggregates immediately upon polyP addition. The observations show that polyP can discriminate between different protein conformations, in contrast to DNA, which does not show such selectivity. However, we acknowledge that while polyP-induced behavior reflects aspects of protein ensemble properties, it does not provide direct insight into the nature of the initial conformational ensemble.

      (4) In the case of FruR with polyphosphate, no CD for the secondary structure analysis was provided as it was for CytR. It would be useful to see if the polyphosphate-induced structural changes observed for CytR hold true for FruR as well.

      We thank the reviewer for the suggestion. In response, we have performed far-UV CD experiments on FruR variants in the presence of polyP. Similar to the CytR WT, FruR WT shows unfolding upon polyP addition. A similar outcome is noted for the Y19A variant though there is significant residual helix content in the condensate unlike the WT. The CD spectra of FruR variants have been added to Figure 6.

      Minor Concerns/Suggestions:

      Under conclusion, third paragraph, first sentence. This sentence reads, "Our observations thus establish that polyP efficiently discriminates the conformational features of proteins than DNA, contributing to the diverse outcomes."

      We thank the reviewer for pointing this out. The sentence has been revised for clarity. It now reads “Our observations establish that polyP is more sensitive to the conformational features of proteins than DNA, thereby contributing to the diverse outcomes.”

      One experimental suggestion. Seeing that protein dynamics and plasticity seem to play a role. For either CytR WT or DM, it would be interesting to see the influence of temperature. Altering the temperature is a good way to perturb the population distribution of conformation sub-states and to alter kinetics. It may be that at a lower temperature (maybe 5C) for the WT you reduce conformational dynamics and you obtain results more similar to that of the DM. Alternatively, heating the DM would be another option. Obviously, there are additional challenges that may arise with changing the temperature, but if it were to work I think it could add some value.

      We thank the reviewer for the thoughtful suggestion. Due to limitations in our current experimental setup (as the reviewer notes as ‘challenges’)- the confocal set up does not have a temperature controller - we will not be to perform temperature-controlled assays. However, the ‘structure’ of CytR variants do not vary much between 280 – 298 K, and this is one of the reasons for choosing three variants without altering any other thermodynamic property. If temperature were varied, the dynamics of polyP would also change and hence the true molecule origins of any differences we might observe will be confounded by the dynamic effects on polyP as well. In this work, we have eliminated any dynamic differences in polyP by performing the experiments at a fixed temperature.

    1. Author response:

      The following is the authors’ response to the current reviews.

      Reviewer #1 (Public Review):

      Summary:

      This manuscript explores the impact of serotonin on olfactory coding in the antennal lobe of locusts and odor-evoked behavior. The authors use serotonin injections paired with an odorevoked palp-opening response assay and bath application of serotonin with intracellular recordings of odor-evoked responses from projection neurons (PNs).

      Strengths:

      The authors make several interesting observations, including that serotonin enhances behavioral responses to appetitive odors in starved and fed animals, induces spontaneous bursting in PNs, directly impacts PN excitability, and uniformly enhances PN responses to odors.

      Weaknesses:

      The one remaining issue to be resolved is the theoretical discrepancy between the physiology and the behavior. The authors provide a computational model that could explain this discrepancy and provide the caveat that while the physiological data was collected from the antennal lobe, but there could be other olfactory processing stages involved. Indeed other processing stages could be the sites for the computational functions proposed by the model. There is an additional caveat which is that the physiological data were collected 5-10 minutes after serotonin application whereas the behavioral data were collected 3 hours after serotonin application. It is difficult to link physiological processes induced 5 minutes into serotonin application to behavioral consequences 3 hours subsequent to serotonin application. The discrepancy between physiology and behavior could easily reflect the timing of action of serotonin (i.e. differences between immediate and longer-term impact).

      For our behavioral experiments, we waited 3 hours after serotonin injection to allow serotonin to penetrate through the layers of air sacks and the sheath, and for the locusts to calm down and recover their baseline POR activity levels. For the physiology experiments, we noticed that the quality of the patch decreased over time after serotonin introduction. Hence, it was difficult to hold cells for that long. However, the point raised by the reviewer is well-taken. We have performed additional experiments to show that the changes in POR levels to different odorants are rapid and can be observed within 15 minutes of injecting serotonin (Author response image 2) and that the physiological changes in PNs (bursting spontaneous activity, maintenance of temporal firing patterns, and increase odor-evoked responses) persists when the cells are held for longer duration (i.e. 3 hours akin to our behavioral experiments). It is worth noting that 3-hour in-vivo intracellular recordings are not easily achievable and come with many experimental constraints. So far, we have managed to record from two PNs that were held for this long and add them to this rebuttal to support our conclusions. (Author response image 1).

      Author response image 1.

      Spontaneous and odor-evoked responses in individual PNs remain consistent for three hours after serotonin introduction into the recording chamber/bath. (A) Representative intracellular recording showing membrane potential fluctuations in a projection neuron (PN) in the antennal lobe. Spontaneous and odor-evoked responses to four odorants (pink color bars, 4 s duration) are shown before (control) and after serotonin application (5HT). Voltage traces 30 minutes (30min), 1 hour (1h), 2 hours (2h), and 3 hours (3h) after 5HT application are shown to illustrate the persisting effect of serotonin during spontaneous and odor-evoked activity periods. (B) Rasterized spiking activities in two recorded PNs are shown. Spontaneous and odor-evoked responses are shown in all 5 consecutive trials. Note that the odor-evoked response patterns are maintained, but the spontaneous activity patterns are altered after serotonin introduction.

      Author response image 2.

      Palp-opening response (POR) patterns to different odorants remain consistent following serotonin introduction. The probability of PORs is shown as a bar plot for four different odorants; hexanol (green), benzaldehyde (blue), linalool (red), and ammonium (purple). PORs before serotonin injection (solid bars) are compared against response levels after serotonin injection (striped bars). As can be noted, PORs to the four odorants remain consistent when tested 15 minutes and 3 hours after (5HT) serotonin injection.

      Overall, the study demonstrates the impact of serotonin on odor-evoked responses of PNs and odor-guided behavior in locusts. Serotonin appears to have non-linear effects including changing the firing patterns of PNs from monotonic to bursting and altering behavioral responses in an odor-specific manner, rather than uniformly across all stimuli presented.

      We thank the reviewer for again providing very useful feedback for improving our manuscript.

      Reviewer #2 (Public Review):

      Summary:

      The authors investigate the influence of serotonin on feeding behavior and electrophysiological responses in the antennal lobe of locusts. They find that serotonin injection changes behavior in an odor-specific way. In physiology experiments, they can show that projection neurons in the antennal lobe generally increase their baseline firing and odor responses upon serotonin injection. Using a modeling approach the authors propose a framework on how a general increase in antennal lobe output can lead to odor-specific changes in behavior.

      Strengths:

      This study shows that serotonin affects feeding behavior and odor processing in the antennal lobe of locusts, as serotonin injection increases activity levels of projection neurons. This study provides another piece of evidence that serotonin is a general neuromodulator within the early olfactory processing system across insects and even phyla.

      Weaknesses:

      I still have several concerns regarding the generalizability of the model and interpretation of results. The authors cannot provide evidence that serotonin modulation of projection neurons impacts behavior.

      This is true and likely to be true for any study linking neural responses to behavior. There are multiple circuits and pathways that would get impacted by a neuromodulator like serotonin. What we showed with our physiology is how spontaneous and odor-evoked responses in the very first neural network that receives olfactory sensory neuron input are altered by serotonin. Given the specificity of the changes in behavioral outcomes (i.e. odor-specific increase and decrease in an appetitive behavior) and non-specificity in the changes at the level of individual PNs (general increase in odor-evoked spiking activity), we presented a relatively simple computational model to address the apparent mismatch between neural and behavioral responses. (Author response image 4).

      The authors show that odor identity is maintained after 5-HT injection, however, the authors do not show if PN responses to different odors were differently affected after serotonin exposure.

      The PN responses to different odorants changed in a qualitatively similar fashion. (Author response image 3)

      Author response image 3.

      PN activity before and after 5HT application are compared for different cellodor combinations. As can be noted, the changes are qualitatively similar in all cases. After 5HT application, the baseline activity became more bursty, but the odor-evoked response patterns were robustly maintained for all odorants.

      Regarding the model, the authors show that the model works for odors with non-overlapping PN activation. However, only one appetitive, one neutral, and one aversive odor has been tested and modeled here. Can the fixed-weight model also hold for other appetitive and aversive odors that might share more overlap between active PNs? How could the model generate BZA attraction in 5-HT exposed animals (as seen in behavior data in Figure 1) if the same PNs just get activated more?

      Author response image 4.

      Testing the generality of the proposed computational model. To test the generality of the model proposed we used a published dataset [Chandak and Raman, 2023]: Neural dataset – 89 PN responses to a panel of twenty-two odorants; Behavioral dataset – probability of POR responses to the same twenty-two odorants. We built the model using just the three odorants overlapping between the two datasets: hexanol, benzaldehyde and linalool. The true probability of POR values of the twenty odorants and the POR probability predicted by the model are shown for all twenty-two odorants as a scatter plot. As can be noted, there is a high correlation (0.79) between the true and the predicted values.

      The authors should still not exclude the possibility that serotonin injections could affect behavior via modulation of other cell types than projection neurons. This should still be discussed, serotonin might rather shut down baseline activation of local inhibitory neurons - and thus lead to the interesting bursting phenotypes, which can also be seen in the baseline response, due to local PN-to-LN feedback.

      As we agreed, there could be other cells that are impacted by serotonin release. Our goal in this study was to characterize how spontaneous and odor-evoked responses in the very first neural network that receives olfactory sensory neuron input are altered by serotonin. Within this circuit, there are local inhibitory neurons (LNs), as correctly indicated by this reviewer. Surprisingly, our preliminary data indicates that LNs are not shut down but also have an enhanced odor-evoked neural response. (Author response image 5.) Further data would be needed to verify this observation and determine the mechanism that mediate the changes in PN excitability. Irrespective, since PN activity should incorporate the effects of changes in the local neuron responses and is the sole output from the antennal lobe that drives all downstream odor-evoked activity, we focused on them in this study.

      Author response image 5.

      Representative traces showing intracellular recording from a local neuron in the antennal lobe. Five consecutive trials are shown. Note that LNs in the locust antennal lobe are non-spiking. The LN activity before, during, and after the presentation of benzaldehyde and hexanol (colored bar; 4s) are shown. The Left and Right panels show LN activity before and after the application of 5HT. As can be noted, 5HT did not shut down odor-evoked activity in this local neuron.

      The authors did not fully tone down their claims regarding causality between serotonin and starved state behavioral responses. There is no proof that serotonin injection mimics starved behavioral responses.

      Specific minor issues:<br /> It is still unclear how naturalistic the chosen odor concentrations are. This is especially important as behavioral responses to different concentrations of odors are differently modulated after serotonin injection (Figure 2: Linalool and Ammonium). The new method part does not indicate the concentrations of odors used for electrophysiology.

      All odorants were diluted to 0.01-10% concentration by volume in either mineral oil or distilled water. This information is included in the Methods section. For most odorants used in the study, the lower concentrations only evoked a very weak neural response, and the higher concentrations evoked more robust responses. The POR responses for these odorants at various concentrations chosen are included in Figure 2. Note, that the responses to linalool and ammonium remained weak throughout the concentration changes, compared to hexanol and benzaldehyde.

      Did all tested PNs respond to all odorants?

      No, only a subset of them responses to each odorant. These responses have been well characterized in earlier publications [included refs].

      The authors do not show if PN responses to different odors were differently affected after serotonin exposure. They describe that ON responses were robust, but OFF responses were less consistent after 5-HT injection. Was this true across all odors tested? Example traces are shown, but the odor is not indicated in Figure 4A. Figure 4D shows that many odor-PN combinations did not change their peak spiking activity - was this true across odorants? In Figure 5 - are PNs ordered by odor-type exposure?

      Also, Figure 6A only shows example trajectories for odorants - how does the average look? Regarding the data used for the model - can the new dataset from the 82 odor-PN pairs reproduce the activation pattern of the previously collected dataset of 89 pairs?

      What is shown in Figure 6A is the trial-averaged response trajectory combining activities of all 82 odor-PN pairs. 82 odor-PN pair was collected intracellularly examining the responses to four odorants before and after 5HT application. The second dataset involving 89 PN responses to 22 odorants was collected extracellularly. They have qualitative similarities in each odorant activate a unique subset of those neurons.

      The authors toned down their claims that serotonin injection can mimic the starved state behavioral response. However, some sentences still indicate this finding and should also be toned down:

      last sentence of introduction - "In sum, our results provide a more systems-level view of how a specific neuromodulator (serotonin) alters neural circuits to produce flexible behavioral outcomes."

      We believe we showed this with our computational model, how uniform changes in the neural responses could lead to variable and odor-specific changes in behavioral PORs.

      discussion: "Finally, fed locusts injected with serotonin generated similar appetitive responses to food-related odorants as starved locusts indicating the role of serotonin in hunger statedependent modulation of odor-evoked responses." This claim is not supported.

      Figure 7 shows that the fed locusts had lower POR to hex and bza. The POR responses significantly increased after the 5HT application. However, we have rephrased this sentence to limit our claims to this result. "Finally, fed locusts injected with serotonin generated similar appetitive palp-opening responses to food-related odorants as observed in starved locusts”

      last results: "However, consistent with results from the hungry locusts, the introduction of serotonin increased the appetitive POR responses to HEX and BZA. Intriguingly, the appetitive responses of fed locusts treated with 5HT were comparable or slightly higher than the responses of hungry locusts to the same set of odorants."

      Again this sentence simply describes the result shown in Figure 7.

      In Figure 7 - BZA response seems unchanged in hungry and fed animals and only 5-HT injection enhances the response. There is only one example where 5-HT application and starvation induce the same change in behavior - N=1 is not enough to conclude that serotonin influences food-driven behaviors.

      The reviewer is ignoring the lack of changes to PORs to linalool and ammonium. Taken together, serotonin increased PORs to only two of the four odorants in starved locusts. The responses after 5HT modulation to these four odorants were similar in fed locusts treated with 5HT and starved locusts.

      Also, this seems to be wrongly interpreted in Figure 7: "It is worth noting that responses to LOOL and AMN, non-food related odorants with weaker PORs, remained unchanged in fed locusts treated with 5HT." The authors indicate a significant reduction in POR after 5-HT injection on LOOL response in Figure 7.

      Revised.<br /> It is worth noting that responses to LOOL and AMN, non-food related odorants with weaker PORs, and reduced in fed locusts treated with 5HT."

      Also, the newly added sentence at the end of the discussion does not make sense: "However, since 5HT increased behavioral responses in both fed and hungry locusts, the precise role of 5HT modulation and whether it underlies hunger-state dependent modulation of appetitive behavior still remains to be determined."<br /> The authors did not test 5-HT injection in starved animals

      The results shown in Figure 1 compare the POR responses of starved locusts before and after 5HT introduction.

      We again thank the reviewer for useful feedback to further improve our manuscript.


      The following is the authors’ response to the original reviews.

      Reviewer #1 (Public Review):

      Summary:

      This manuscript explores the impact of serotonin on olfactory coding in the antennal lobe of locusts and odor-evoked behavior. The authors use serotonin injections paired with an odor-evoked palp-opening response assay and bath application of serotonin with intracellular recordings of odor-evoked responses from projection neurons (PNs).

      Strengths:

      The authors make several interesting observations, including that serotonin enhances behavioral responses to appetitive odors in starved and fed animals, induces spontaneous bursting in PNs, and uniformly enhances PN responses to odors. Overall, I had no technical concerns. Weaknesses:

      While there are several interesting observations, the conclusions that serotonin enhanced sensitivity specifically and that serotonin had feeding-state-specific effects, were not supported by the evidence provided. Furthermore, there were other instances in which much more clarification was needed for me to follow the assumptions being made and inadequate statistical testing was reported.

      Major concerns.

      • To enhance olfactory sensitivity, the expected results would be that serotonin causes locusts to perceive each odor as being at a relatively higher concentration. The authors recapitulate a classic olfactory behavioral phenomenon where higher odor concentrations evoke weaker responses which is indicative of the odors becoming aversive. If serotonin enhanced the sensitivity to odors, then the dose-response curve should have shifted to the left, resulting in a more pronounced aversion to high odor concentrations. However, the authors show an increase in response magnitude across all odor concentrations. I don't think the authors can claim that serotonin enhances the behavioral sensitivity to odors because the locusts no longer show concentration-dependent aversion. Instead, I think the authors can claim that serotonin induces increased olfactory arousal.

      The reviewer makes a valid point. Bath application of serotonin increased POR behavioral responses across all odor concentrations, and concentration-dependent aversion was also not observed. Furthermore, the monotonic relationship between projection neuron responses and the intensity of current injection is altered when serotonin is exogenously introduced (see Author response image 1; see below for more explanation). Hence, our data suggests that serotonin alters the dose-response relationship between neural/behavioral responses and odor intensity. As recommended, we have followed what the reviewer has suggested and revised our claim to serotonin inducing increase in olfactory arousal. The new physiology data has been added as Supplementary Figure 3 to the revised manuscript.

      • The authors report that 5-HT causes PNs to change from tonic to bursting and conclude that this stems from a change in excitability. However, excitability tests (such as I/V plots) were not included, so it's difficult to disambiguate excitability changes from changes in synaptic input from other network components.

      To confirm that the PN excitability did indeed change after serotonin application, we performed a new set of current-clamp recordings. In these experiments, we monitored the spiking activities in individual PNs as we injected different levels of current injections (200 – 1000 pico Amperes). Note that locust LNs that provide recurrent inhibition arborize and integrate inputs from a large number of sensory neurons and projection neurons. Therefore, activating a single PN should not activate the local neurons and therefore the antennal lobe network.

      We found that the total spiking activity monotonically increased with the magnitude of the current injection in all four PNs recorded (Author response image 1). However, after serotonin injection, we found that the spiking activity remained relatively stable and did not systematically vary with the magnitude of the current injection. While the changes in odor-evoked responses may incorporate both excitability changes in individual PNs and recurrent feedback inhibition through GABAergic LNs, these results from our current injection experiments unambiguously indicate that there are changes in excitability at the level of individual PNs. We have added this result to the revised manuscript.

      Author response image 1.

      Current-injection induced spiking activity in individual PNs is altered after serotonin application. (A) Representative intracellular recordings showing membrane potential fluctuations as a function of time for one projection neuron (PNs) in the locust antennal lobe. A two-second window when a positive 200-1000pA current was applied is shown. Firing patterns before (left) and after (right) serotonin application are shown for comparison. Note, the spiking activity changes after the 5HT application. The black bar represents the 20mV scale. (B) Dose-response curves showing the average number of action potentials (across 5 trials) during the 2second current pulse before (green) and after (purple) serotonin for each recorded PN. Note that the current intensity was systematically increased from 200 pA to 1000 pA. The (C) The mean number of spikes across the four recorded cells during current injection is shown. The color progression represents the intensity of applied current ranging 200pA (leftmost bar) to 1000pA (rightmost bar). The dose-response trends before (green) and after (purple) 5HT application are shown for comparison. The error bars represent SEM across the four cells.

      • There is another explanation for the theoretical discrepancy between physiology and behavior, which is that odor coding is further processing in higher brain regions (ie. Other than the antennal lobe) not studied in the physiological component of this study. This should at least be discussed.

      This is a valid argument. For our model of neural mapping onto behavior to work, we only need the odorant that evokes or suppresses PORs to activate a distinct set of neurons. Having said that, our extracellular recording results (Fig. 6E) indicate that hexanol (high POR) and linalool (low POR) do activate highly non-overlapping sets of PNs in the antennal lobe. Hence, our results suggest that the segregation of neural activity based on behavioral relevance already begins in the antennal lobe. We have added this clarification to the discussion section.

      • The authors cannot claim that serotonin underlies a hunger state-dependent modulation, only that serotonin impacts responses to appetitive odors. Serotonin enhanced PORs for starved and fed locusts, so the conclusion would be that serotonin enhances responses regardless of the hunger state. If the authors had antagonized 5-HT receptors and shown that feeding no longer impacts POR, then they could make the claim that serotonin underlies this effect. As it stands, these appear to be two independent phenomena.

      This is also a valid point. We have clarified this in the revised manuscript.

      Reviewer #2 (Public Review):

      Summary:

      The authors investigate the influence of serotonin on feeding behavior and electrophysiological responses in the antennal lobe of locusts. They find that serotonin injection changes behavior in an odorspecific way. In physiology experiments, they can show that antennal lobe neurons generally increase their baseline firing and odor responses upon serotonin injection. Using a modeling approach the authors propose a framework on how a general increase in antennal lobe output can lead to odorspecific changes in behavior. The authors finally suggest that serotonin injection can mimic a change in a hunger state.

      Strengths:

      This study shows that serotonin affects feeding behavior and odor processing in the antennal lobe of locusts, as serotonin injection increases activity levels of antennal lobe neurons. This study provides another piece of evidence that serotonin is a general neuromodulator within the early olfactory processing system across insects and even phyla. Weaknesses:

      I have several concerns regarding missing control experiments, unclear data analysis, and interpretation of results.

      A detailed description of the behavioral experiments is lacking. Did the authors also provide a mineral oil control and did they analyze the baseline POR response? Is there an increase in baseline response after serotonin exposure already at the behavioral output level? It is generally unclear how naturalistic the chosen odor concentrations are. This is especially important as behavioral responses to different concentrations of odors are differently modulated after serotonin injection (Figure 2: Linalool and Ammonium).

      POR protocol: Sixth instar locusts (Schistocera americana) of either sex were starved for 24-48 hours before the experiment or taken straight from the colony and fed blades of grass for the satiated condition. Locusts were immobilized by placing them in the plastic tube and securing their body with black electric tape (see Author response image 2). Locusts were given 20 - 30 minutes to acclimatize after placement in the immobilization tube. As can be noted, the head of the locusts along with the antenna and maxillary palps protruded out of this immobilization tube so they can be freely moved by the locusts. Note that the maxillary palps are sensory organs close to the mouth parts that are used to grab food and help with the feeding process.

      It is worth noting that our earlier studies had shown that the presentation of ‘appetitive odorants’ triggers the locust to open their maxillary palps even when no food is presented (Saha et al., 2017; Nizampatnam et al., 2018; Nizampatnam et al., 2022; Chandak and Raman, 2023.) Furthermore, our earlies results indicate that the probability of palp opening varies across different odorants (Chandak and Raman, 2023). We chose four odorants that had a diverse range of palp-opening: supra-median (hexanol), median (benzaldehyde), and sub-median (linaool). Therefore, each locust in our experiments was presented with one concentration of four odorants (hexanol, benzaldehyde, linalool, and ammonium) in a pseudorandomized order. The odorants were chosen based on our physiology results such that they evoked different levels of spiking activities.

      The odor pulse was 4 s in duration and the inter-pulse interval was set to 60 s. The experiments were recorded using a web camera (Microsoft) placed right in front of the locusts. The camera was fully automated with the custom MATLAB script to start recording 2 seconds before the odor pulse and end recording at odor termination. An LED was used to track the stimulus onset/offset. The POR responses were manually scored offline. Responses to each odorant were scored a 0 or 1 depending on if the palps remained closed or opened. A positive POR was defined as a movement of the maxillary palps during the odor presentation time window as shown on the locust schematic (Main Paper Figure 1).

      Author response image 2.

      Pictures showing the behavior experiment setup and representative palp-opening responses in a locust.

      As the reviewer inquired, we performed a new series of POR experiments, where we explored POR responses to mineral oil and hexanol, before and after serotonin injection. For this study, we used 10 locusts that were starved 24-48 hours before the experiment. Note that hexanol was diluted at 1% (v/v) concentration in mineral oil. Our results reveal that locusts PORs to hexanol (~ 50% PORs) were significantly higher than those triggered by mineral oil (~10% PORs). Injection of serotonin increased the POR response rate to hexanol but did not alter the PORs evoked by mineral oil (Author response image 3).

      Author response image 3.

      Serotonin does not alter the palp-opening responses evoked by paraffin oil. The PORs before and after (5HT) serotonin injection are summarized and shown as a bar plot for hexanol and paraffin oil. Striped bars signify the data collected after 5HT injection. Significant differences are identified in the plot (one-tailed paired-sample t-test; (*p<0.05).

      Regarding recordings of potential PNs - the authors do not provide evidence that they did record from projection neurons and not other types of antennal lobe neurons. Thus, these claims should be phrased more carefully.

      In the locust antennal lobe, only the cholinergic projection neurons fire full-blown sodium spikes. The GABAergic local neurons only fire calcium ‘spikelets’ (Laurent, TINS, 1996; Stopfer et al., 2003; see Author response image 4 for an example). Hence, we are pretty confident that we are only recording from PNs. Furthermore, due to the physiological properties of the LNs, their signals being too small, they are also not detected in the extracellular recordings from the locust antennal lobe. Hence, we are confident with our claims and conclusion.

      Author response image 4.

      PN vs LN physiological differences: Left: A representative raw voltage traces recorded from a local neuron before, during, and after a 4-second odor pulse are shown. Note that the local neurons in the locust antennal lobe do not fire full-blown sodium spikes but only fire small calcium spikelets. On the right: A representative raw voltage trace recorded from a representative projection neuron is shown for comparison. Clear sodium spikes are clearly visible during spontaneous and odor-evoked periods. The gray bar represents 4 seconds of odor pulse. The vertical black bar represents the 40mV.

      The presented model suggests labeled lines in the antennal lobe output of locusts. Could the presented model also explain a shift in behavior from aversion to attraction - such as seen in locusts when they switch from a solitarious to a gregarious state? The authors might want to discuss other possible scenarios, such as that odor evaluation and decision-making take place in higher brain regions, or that other neuromodulators might affect behavioral output. Serotonin injections could affect behavior via modulation of other cell types than antennal lobe neurons. This should also be discussed - the same is true for potential PNs - serotonin might not directly affect this cell type, but might rather shut down local inhibitory neurons.

      There are multiple questions here. First, regarding solitary vs. gregarious states, we are currently repeating these experiments on solitary locusts. Our preliminary results (not included in the manuscript) indicate that the solitary animals have increased olfactory arousal and respond with a higher POR but are less selective and respond similarly to multiple odorants. We are examining the physiology to determine whether the model for mapping neural responses onto behavior could also explain observations in solitary animals.

      Second, this reviewer makes the point raised by Reviewer 1. We agree that odor evaluation and decisionmaking might take place in higher brain regions. All we could conclude based on our data is that a segregation of neural activity based on behavioral relevance might provide the simplest approach to map non-specific increase in stimulus-evoked neural responses onto odor-specific changes in behavioral outcome. Furthermore, our results indicate that hexanol and linalool, two odorants that had an increase and decrease in PORs after serotonin injection, had only minimal neural response overlap in the antennal lobe. These results suggest that the formatting of neural activity to support varying behavioral outcomes might already begin in the antennal lobe. We have added this to our discussion.

      Third, regarding serotonin impacting PNs, we performed a new set of current-clamp experiments to examine this issue (Author response image 1). Our results clearly show that projection neuron activity in response to current injections (that should not incorporate feedback inhibition through local neurons) was altered after serotonin injection. Therefore, the observed changes in the odor-evoked neural ensemble activity should incorporate modulation at both individual PN level and at the network level. We have added this to our discussion as well.

      Finally, the authors claim that serotonin injection can mimic the starved state behavioral response. However, this is only shown for one of the four odors that are tested for behavior (HEX), thus the data does not support this claim.

      We note that Hex is the only appetitive odorant in the panel. But, as reviewer 1 has also brought up a similar point, we have toned down our claims and will investigate this carefully in a future study.

      Recommendations for the authors:

      Reviewer #1 (Recommendations For The Authors):

      • Was the POR of the locusts towards linalool and ammonium higher than towards a blank odor cartridge? I ask because the locusts appear to be less likely to respond to these odors and so I am concerned that this assay is not relevant to the ecological context of these odors. In other words, perhaps serotonin did not enhance the responses to these odors in this assay, because this is not a context in which locusts would normally respond to these odors.

      The POR response to linalool and ammonium is lower and comparable to that of paraffin oil. Serotonin does not increase POR responses to paraffin oil but does increase response to hexanol (an appetitive odorant). We have clarified this using new data (Author response image 5).

      • It seems to me that Figure 5C is the crux for understanding the potential impact of 5-HT on odor coding, but it is somewhat confusing and underutilized. Is the implication that 5-HT decorrelates spontaneous activity such that when an odor stimulus arrives, the odor-evoked activity deviates to a greater degree? The authors make claims about this figure that require the reader to guess as to the aspect of the figure to which they are referring.

      The reviewer makes an astute observation. Yes, the spontaneous activity in the antennal lobe network before serotonin introduction is not correlated with the ensemble spontaneous activity after serotonin bath application. Remarkably, the odor-evoked responses were highly similar, both in the reduced PCA space and when assayed using high-dimensional ensemble neural activity vectors. Whether the changes in network spontaneous activity have a function in odor detection and recognition is not fully understood and cannot be convincingly answered using our data. But this is something that we had pondered.

      • The modeling component summarized in Figure 6 needs clarification and more detail. Perhaps example traces associated with positive weighting within neural ensemble 1 relative to neural ensemble 2? I struggled to understand conceptually how the model resolved the theoretical discrepancy between physiology and behavior.

      As recommended, here is a plot showing the responses of four PNs that had positive weights to hexanol and linalool. As can be expected, each PN in this group had higher responses to hexanol and no response to linalool. Further, the four PNs that received negative weights had response only to linalool.

      Author response image 5.

      Odor-evoked responses of four PNs that received positive weights in the model (top panel), and four PNs that were assigned negative weights in the model (bottom).

      • Was there a significant difference between the PORs of hungry vs. fed locusts? The authors state that they differ and provide statistics for the comparisons to locusts injected with 5-HT, but then don't provide any statistical analyses of hungry vs. fed animals.

      The POR responses to HEX (an appetitive odorant) were significantly different between the hungry and starved locusts.

      Author response image 6.

      A bar plot summarizing PORs to all four odors for satiated locust (highlighted with stripes), before (dark shade), and after 5HT injection (lighter shade). To allow comparison before 5HT injection for starved locust plotted as well (without stripes). The significance was determined using a one-tailed paired-sample ttest(*p<0.05).

      • Were any of the effects of 5-HT on odor-evoked PN responses significant? No statistics are provided.

      We examined the distribution of odor-evoked responses in PNs before and after 5HT introduction. We found that the overall distribution was not significantly different between the two (one-tailed pairedsample t-test; p = 0.93).

      Author response image 7.

      Comparison of the distribution of odor-evoked PN responses before (green) and after (purple) 5HT introduction. One-tailed paired sample t-test was used to compare the two distributions.

      • The authors interchangeably use "serotonin", "5HT" and "5-HT" throughout the manuscript, but this should be consistent.

      This has been fixed in the revised manuscript.

      • On page 2 the authors provide an ecological relevance for linalool as being an additive in pesticides, however, linalool is a common floral volatile chemical. Is the implication that locusts have learned to associate linalool with pesticides?

      Linalool is a terpenoid alcohol that has a floral odor but has also been used as a pesticide and insect repellent [Beier et al., 2014]. As shown in Author response image 2, it evoked the least POR responses amongst a diverse panel of 22 odorants that were tested. We have clarified how we chose odorants based on the prior dataset in the Methods section.

      • In Figure 1, there should be a legend in the figure itself indicating that the black box indicates the absence of POR and the white box indicates presence, rather than just having it in the legend text.

      Done.

      • In Figure 2, the raw data from each animal can be moved to the supplements. The way it is presented is overwhelming and the order of comparisons is difficult to follow.

      Done.

      • For the induction of bursting in PNs by the application of 5-HT, were there any other metrics observed such as period, duration of bursts, or peak burst frequency? The authors rely on ISI, but there are other bursting metrics that could also be included to understand the nature of this observation. In particular, whether the bursts are likely due to changes in intrinsic biophysical properties of the PNs or polysynaptic effects.

      We could use other metrics as the reviewer suggests. Our main point is that the spontaneous activity of individual PNs changed. We have added a new current-injection experiments to show that the PNs output to square pulses of current becomes different after serotonin application (Author response image 1)

      • Were 4-vinyl anisole, 1-nonanol, and octanoic acid selected as additional odors because they had particular ecological relevance, or was it for the diversity of chemical structure?

      These odorants were selected based on both, chemical structure and ecological relevance. The logic behind this was to have a very diverse odor panel that consisted of food odorant – Hexanol, aggregation pheromone – 4-vinyl anisole, sex pheromone – benzaldehyde, acid – octanoic acid, base – ammonium, and alcohol – 1-nonanol. Additionally, we selected these odors based on previous neural and behavioral data on these odorants (Chandak and Raman, 2023, Traner and Raman, 2023, Nizampatnam et al, 2022 & 2018; Saha et al., 2017 & 2013).

      Reviewer #2 (Recommendations For The Authors):

      The electrophysiology dataset combines all performed experiments across all tested different PN-odor pairs. How many odors have been tested in a single PN and how many PNs have been tested for a single odor? This information is not present in the current manuscript. Can the authors exclude that there are odor-specific modulations?

      In total, our dataset includes recordings from 19 PNs. Seven PNs were tested on a panel of seven odorants (4-vinyl anisole, 1-nonanol, octanoic acid, Hex, Bza, Lool, and Amn), and the remaining twelve were tested with the four main odorants used in the study (Hex, Bza, Lool, and Amn). This information has been added to the Methods section

      How did the authors choose the concentrations of serotonin injections and bath applications - is this a naturalistic amount?

      The serotonin concentration for ephys experiments was chosen based on trial-error experiments:

      0.01mM was the highest concentration that did not cause cell death. For the behavioral experiments, we increased the concentration (0.1 M) due to the presence of anatomical structures in the locust's head such as air sacks, sheath as well as hemolymph which causes some degree of dilution that we cannot control.

      Behavior experiments were performed 3 hours after injection - ephys experiments 5-10 minutes following bath application. Can the authors exclude that serotonin affects neural processing differently on these different timescales?

      We cannot exclude this possibility. We did ePhys experiments 5-10 minutes after bath application as it would be extremely hard to hold cells for that long.

      A longer delay was required for our behavioral experiments as the locusts tended to be a bit more agitated with larger spontaneous movements of palps as well as exhibited unprompted vomiting. A 3hour period allowed the locust to regain its baseline level movements after 5HT introduction. [This information has been added to the methods section of the revised manuscript]

      Concerning the analysis of electrophysiological data. The authors should correct for changes in the baseline before performing PCA analysis. And how much of the variance is explained by PC1 and PC2?

      We did not correct for baseline changes or subtract baseline as we wanted to show that the odor-evoked neural responses still robustly encoded information about the identity of the odorant.

      The authors should perform dye injections after recordings to visualize the cell type they recorded from. Serotonin might affect also other cell types in the antennal lobe.

      As mentioned above, in the locust antennal lobe only PNs fire full-blown sodium spikes, and LNs only fire calcium spikelets (Author response image 4). Since these signals are small, they will be buried under the noise floor when using extracellular recording electrodes for monitoring responses in the AL antennal lobe.

      Hence we are pretty certain what type of cells we are recording from.

      There were several typos in the manuscript, please check again.

      We have fixed many of the grammatical errors and typos in the revised version.

    1. Author response:

      The following is the authors’ response to the original reviews

      Public Reviews:

      Reviewer #1 (Public Review):

      Summary:

      Most studies in sensory neuroscience investigate how individual sensory stimuli are represented in the brain (e.g., the motion or color of a single object). This study starts tackling the more difficult question of how the brain represents multiple stimuli simultaneously and how these representations help to segregate objects from cluttered scenes with overlapping objects.

      Strengths

      The authors first document the ability of humans to segregate two motion patterns based on differences in speed. Then they show that a monkey's performance is largely similar; thus establishing the monkey as a good model to study the underlying neural representations.

      Careful quantification of the neural responses in the middle temporal area during the simultaneous presentation of fast and slow speeds leads to the surprising finding that, at low average speeds, many neurons respond as if the slowest speed is not present, while they show averaged responses at high speeds. This unexpected complexity of the integration of multiple stimuli is key to the model developed in this paper.

      One experiment in which attention is drawn away from the receptive field supports the claim that this is not due to the involuntary capture of attention by fast speeds.

      A classifier using the neuronal response and trained to distinguish single-speed from bi-speed stimuli shows a similar overall performance and dependence on the mean speed as the monkey. This supports the claim that these neurons may indeed underlie the animal's decision process.

      The authors expand the well-established divisive normalization model to capture the responses to bi-speed stimuli. The incremental modeling (eq 9 and 10) clarifies which aspects of the tuning curves are captured by the parameters.

      We thank the Reviewer for the thorough summary of the findings and supportive comments.

      Weaknesses

      While the comparison of the overall pattern of behavioral performance between monkeys and humans is important, some of the detailed comparisons are not well supported by the data. For instance, whether the monkey used the apparent coherence simply wasn't tested and a difference between 4 human subjects and a single monkey subject cannot be tested statistically in a meaningful manner. I recommend removing these observations from the manuscript and leaving it at "The difference between the monkey and human results may be due to species differences or individual variability" (and potentially add that there are differences in the task as well; the monkey received feedback on the correctness of their choice, while the humans did not.)

      Thanks for the suggestion. We agree and have modified the text accordingly. We now state on page 8, lines 189-191, "The difference between the monkey and human results may be due to species differences or individual variability. The differences in behavioral tasks may also play a role – the monkey received feedback on the correctness of the choice, whereas human subjects did not."

      A control experiment aims to show that the "fastest speed takes all" behavior is general by presenting two stimuli that move at fast/slow speeds in orthogonal directions. The claim that these responses also show the "fastest speed takes all" is not well supported by the data. In fact, for directions in which the slow speed leads to the largest response on its own, the population response to the bi-speed stimulus is the average of the response to the components (This is fine. One model can explain all direction tuning curve, which also explain averaging at the slower speed stronger directions). Only for the directions where the fast speed stimulus is the preferred direction is there a bias towards the faster speed (Figure 7A). The quantification of this effect in Figure 7B seems to suggest otherwise, but I suspect that this is driven by the larger amplitude of Rf in Figure 8, and the constraint that ws and wf are constant across directions. The interpretation of this experiment needs to be reconsidered.

      The Reviewer raised a good question. Our model with fixed weights for faster and slower components across stimulus directions provided a parsimonious explanation for the whole tuning curve, regardless of whether the faster component elicited a stronger response than the slower component. Because the model can be well constrained by the measured direction-tuning curves, we did not restrain 𝑤 and 𝑤 to sum to one, which is more general. The linear weighted summation (LWS) model fits the neuronal responses to the bi-speed stimuli very well, accounting for an average of 91.8% (std = 7.2%) of the response variance across neurons. As suggested by the Reviewer, we now use the normalization model to fit the data with fixed weights across all motion directions. The normalization model also provides a good fit, accounting for an average of 90.5% (std = 7.1%) of the response variance across neurons.

      Note that in the new Figure 8A, at the left side of the tuning curve (i.e., at negative vector average (VA) directions), where the slower component moving in a more preferred direction of the neurons than the faster component, the bi-speed response (red curve) is slightly lower than the average of the component response (gray curve), indicating a bias toward the weaker faster component. Therefore, the faster speed bias does not occur only when the faster component moves in the more preferred direction. This can also be seen in the direction-tuning curves of an example neuron that we added to the figure (new Fig. 8B). The peak responses to the slower and faster component were about the same, but the neuron still showed a faster-speed bias. At negative VA directions, the red curve is lower than the response average (gray curve) and is biased toward the weaker (faster) component.  

      The faster-speed bias also occurs when the peak response to the slower component is stronger than the faster component. As a demonstration, Author response image 1 1 shows an example MT neuron that has a slow preferred speed (PS = 1.9 deg/s) and was stimulated by two speeds of 1.2 and 4.8 deg/s. The peak response to the faster component (blue) was weaker than that to the slower component (green). However, this neuron showed a strong bias toward the faster component. A normalization model fit with fixed weights for the faster and slower components (black curve) described the neuronal response to both speeds (red) well. This neuron was not included in the neuron population shown in Figure 8 because it was not tested with stimulus speeds of 2.5 and 10 deg/s.

      Author response image 1.

      An example MT neuron was tested with stimulus speeds of 1.2 and 4.8 deg/s. The preferred speed of this neuron was 1.9 deg/s. Fixed weights of 0.59 for the faster component and 0.12 for the slower component described the responses to the bispeed stimuli well using a normalization model. The neuron showed a faster-speed bias although its peak response to the slower component was higher than that of the faster component.

      We modified the text to clarify these points:

      Page 19, lines 405 – 410, “The bi-speed response was biased toward the faster component regardless of whether the response to the faster component was stronger (in positive VA directions) or weaker (in negative VA directions) than that to slower component (Fig. 8A). The result from an example neuron further demonstrated that, even when the peak firing rates of the faster and slower component responses were similar, the response elicited by the bi-speed stimuli was still biased toward the faster component (Fig. 8B). ”

      Page 19, lines 421 – 427, “Because the model can be well constrained by the measured direction-tuning curves, it is not necessary to require 𝑤 and 𝑤 to sum to one, which is more general. An implicit assumption of the model is that, at a given pair of stimulus speeds, the response weights for the slower and faster components are fixed across motion directions. The model fitted MT responses very well, accounting for an average of 91.8% of the response variance (std = 7.2%, N = 21) (see Methods). The success of the model supports the assumption that the response weights are fixed across motion directions.”

      Reviewer #2 (Public Review):

      Summary:

      This is a paper about the segmentation of visual stimuli based on speed cues. The experimental stimuli are random dot fields in which each dot moves at one of two velocities. By varying the difference between the two speeds, as well as the mean of the two speeds, the authors estimate the capacity of observers (human and non-human primates) to segment overlapping motion stimuli. Consistent with previous work, perceptual segmentation ability depends on the mean of the two speeds. Recordings from area MT in monkeys show that the neuronal population to compound stimuli often shows a bias towards the faster-speed stimuli. This bias can be accounted for with a computational model that modulates single-neuron firing rates by the speed preferences of the population. The authors also test the capacity of a linear classifier to produce the psychophysical results from the MT data.

      Strengths:

      Overall, this is a thorough treatment of the question of visual segmentation with speed cues. Previous work has mostly focused on other kinds of cues (direction, disparity, color), so the neurophysiological results are novel. The connection between MT activity and perceptual segmentation is potentially interesting, particularly as it relates to existing hypotheses about population coding.

      We thank the Reviewer for the summary and comments.

      Weaknesses:

      Page 10: The relationship between (R-Rs) and (Rf-Rs) is described as "remarkably linear". I don't actually find this surprising, as the same term (Rs) appears on both the x- and y-axes. The R^2 values are a bit misleading for this reason.

      The Reviewer is correct that subtracting a common term Rs from R and Rf would introduce correlation between (R-Rs) and (Rf-Rs). To address this concern, we conducted an additional analysis. We showed that, at most speed pairs, the R^2 values between (R-Rs) and (Rf-Rs) based on the data are significantly higher than the R^2 values between (R’-Rs) and (RfRs), in which R’ was a random combination of Rs and Rf. Since the same Rs was commonly subtracted in calculating R^2 (data) and R^2 (simulation), the difference between R^2 (data) and R^2 (simulation) suggests that the response pattern of R contributes to the additional correlation.

      We now acknowledge this confounding factor and describe the new analysis results on page 14, lines 309 – 326. Please also see the response to Reviewer 3 about a similar concern.

      Figure 9: I'm confused about the linear classifier section of the paper. The idea makes sense - the goal is to relate the neuronal recordings to the psychophysical data. However the results generally provide a poor quantitative match to the psychophysical data. There is mention of a "different paper" (page 26) involving a separate decoding study, as well as a preprint by Huang et al. (2023) that has better decoding results. But the Huang et al. preprint appears to be identical to the current manuscript, in that neither has a Figure 12, 13, or 14. The text also says (page 26) that the current paper is not really a decoding study, but the linear classifier (Figure 9F) is a decoder, as noted on page 10. It sounds like something got mixed up in the production of two or more papers from the same dataset.

      We apologize for the confusion regarding the reference of Huang et al. (2023, bioRxiv). We referred to an earlier version of this bioRxiv manuscript (version 1), which included decoding analysis. In the bibliography, we provided two URLs for this pre-print. While the second link was correct, the first URL automatically links to the latest version (version 2), which did not have the abovementioned decoding analysis.

      The analysis in Figure 9 is to apply a classifier to discriminate two-speed from singlespeed stimuli, which is a decoding analysis as the Reviewer pointed out. We revised the result section about the classifier to make it clear what the classifier can and cannot explain (pages 2223, lines 516-534). We also included a sentence at the end of this section that leads to additional decoding analysis to extract motion speed(s) from MT population responses (page 23, lines 541543), “To directly evaluate whether the population neural responses elicited by the bi-speed stimulus carry information about two speeds, it is important to conduct a decoding analysis to extract speed(s) from MT population responses.”

      In any case, I think that some kind of decoding analysis would really strengthen the current paper by linking the physiology to the psychophysics, but given the limitations of the linear classifier, a more sophisticated approach might be necessary -- see for example Zemel, Dayan, and Pouget, 1998. The authors might also want to check out closely related work by Treue et al. (Nature Neuroscience 2000) and Watamaniuk and Duchon (1992).

      We thank the Reviewer for the suggestion and agree that it is useful to incorporate additional decoding analysis that can better link physiology results to psychophysics. The decoding analysis we conducted was motivated by the framework proposed by Zemel, Dayan, and Pouget (1998), and also similar to the idea briefly mentioned in the Discussion of Treue et al. (2000). We have added the decoding analysis to this paper on pages 25-32.  

      What do we learn from the normalization model? Its formulation is mostly a restatement of the results - that the faster and slower speeds differentially affect the combined response. This hypothesis is stated quantitatively in equation 8, which seems to provide a perfectly adequate account of the data. The normalization model in equation 10 is effectively the same hypothesis, with the mean population response interposed - it's not clear how much the actual tuning curve in Figure 10A even matters, since the main effect of the model is to flatten it out by averaging the functions in Figure 10B. Although the fit to the data is reasonable, the model uses 4 parameters to fit 5 data points and is likely underconstrained; the parameters other than alpha should at least be reported, as it would seem that sigma is actually the most important one. And I think it would help to examine how robust the statistical results are to different assumptions about the normalization pool.

      In the linear weighted summation model (LWS) model (Eq. 8), the weights Ws and Wf are free parameters. We think the value of the normalization model (Eq. 9) is that it provides an explanation of what determines the response weights. We agree with the Reviewer that using the normalization model (Eq. 9) with 4 parameters to fit 5 data points of the tuning curves to bispeed stimuli of individual neurons is under-constrained. We, therefore, removed the section using the normalization model to fit overlapping stimuli moving in the same direction at different speeds.

      A better way to constrain the normalization model is to use the full direction-tuning curves of MT neurons in response to two stimulus components moving in different directions at different speeds, as shown in Figure 8. We now use the normalization model (Eq. 9) to fit this data set (also suggested by Reviewer 1), in addition to the LWS model. We now report the median values of the model parameters of the normalization model, including the exponent n, sigma, alpha, and the constant c. We also compared the normalization model fit with the linear summation (LWS) model. We discuss the limitations of our data set and what needs to be done in future studies. The revisions are on page 20, lines 434-467 in the Results, and pages 34-35, lines 818-829 in Discussion.

      Reviewer #3 (Public Review):

      Summary:

      This study concerns how macaque visual cortical area MT represents stimuli composed of more than one speed of motion.

      Strengths:

      The study is valuable because little is known about how the visual pathway segments and preserves information about multiple stimuli. The study presents compelling evidence that (on average) MT neurons represent the average of the two speeds, with a bias that accentuates the faster of the two speeds. An additional strength of the study is the inclusion of perceptual reports from both humans and one monkey participant performing a task in which they judged whether the stimuli involved one vs two different speeds. Ultimately, this study raises intriguing questions about how exactly the response patterns in visual cortical area MT might preserve information about each speed, since such information could potentially be lost in an average response as described here, depending on assumptions about how MT activity is evaluated by other visual areas.

      Weaknesses:

      My main concern is that the authors are missing an opportunity to make clear that the divisive normalization, while commonly used to describe neural response patterns in visual areas (and which fits the data here), fails on the theoretical front as an explanation for how information about multiple stimuli can be preserved. Thus, there is a bit of a disconnect between the goal of the paper - how does MT represent multiple stimuli? - and the results: mostly averaging responses which, while consistent with divisive normalization, would seem to correspond to the perception of a single intermediate speed. This is in contrast to the psychophysical results which show that subjects can at least distinguish one from two speeds. The paper would be strengthened by grappling with this conundrum in a head-on manner.

      We thank the Reviewer for the constructive comments. We agree with the Reviewer that it is important to connect the encoding of multiple speeds with the perception. The Reviewer also raised an important question regarding whether multiple speeds can be extracted from population neural responses, given the encoding rules characterized in this study.

      It is a hard problem to extract multiple stimulus values from the population neural response. Inspired by the theoretical framework proposed by Zemel et al. (1998), we conducted a detailed decoding study to extract motion speed(s) from MT population responses. We used the decoded speed(s) to perform a discrimination task similar to our psychophysics task and compared the decoder's performance with perception. We found that, at X4 speed difference, we could decode two speeds based on MT response, and the decoder's performance was similar to that of perception. However, at X2 speed difference, except at the slowest speeds of 1.25 and 2.5 deg/s, the decoder cannot extract two speeds and cannot differentiate between a bi-speed stimulus and a single log-mean speed stimulus. We have added the decoding analysis to this paper on pages 25-32. We also discuss the implications and limitations of these results (pages 35-36, lines 852-884).

      Recommendations for the authors:

      Reviewer #1 (Recommendations For The Authors):

      Classifier:

      One question I have is how the classifier's performance scales with the number of neurons used in the analysis. Here that number is set to the number that was recorded, but it is a free parameter in this analysis. Why does the arbitrary choice of 100 neurons match the animals' performance?

      We apologize for the unclearness of this point. The decoding using the classifier was based on the neural responses of 100 recorded MT neurons in our data set. The number of 100 neurons was not a free parameter. We need to reconstruct the population neural response based on the responses of the recorded neurons and their preferred speeds (red and black dots in Figure 9A-E).  

      We spline-fitted the reconstructed population neural responses (red and black curves in Figure 9-E). One way to change the number of neurons used for the decoding is to resample N points along the spline-fitted population responses, using N as a free parameter. However, we think it is better to conduct decoding based on the responses from the recorded neurons rather than based on interpolated responses. We now clarify on page 22, lines 520-522, that we based on the responses of the 100 recorded neurons in our dataset to do the classification (decoding).

      Normalization Model:

      Although the model is phenomenological, a schematic circuit diagram could help the reader understand how this could work (I think this is worthwhile even though the data cannot distinguish among different implementations of divisive normalization).

      Thanks for this suggestion. We agree that a circuit diagram would help the readers understand how the model works. However, as the Reviewer pointed out, our data cannot distinguish between different implementations of the model. For example, divisive normalization can occur on the inputs to MT neurons or on MT neurons themselves. The circuit mechanism of weighting the component responses is not clear either. A schematic circuit diagram then mainly serves to recapitulate the normalization model in Equation 9. We, therefore, choose not to add a schematic circuit diagram at this time. We are interested in developing a circuit model to account for how visual neurons represent multiple stimuli in future studies.

      Another suggestion is that the time courses could be used to constrain the model; the fact that it takes a while after the onset of the slow-speed response for averaging to reveal itself suggests the presence of inertia/hysteresis in the circuit).

      We agree that the time course of MT responses could be used to constrain the model. This is also why we think it is important to document the time course in this paper. We now state in the Results, page 17, lines 354-357:

      “At slow speeds, the very early faster-speed bias suggests a likely role of feedforward inputs to MT on the faster-speed bias. The slightly delayed reduction (normalization) in the bispeed response relative to the stronger component response also helps constrain the circuit model for divisive normalization.”

      Two-Direction Experiment:

      Applying the normalization model to this dataset could help determine its generality.

      This is a good point. We now apply the normalization model (Eq. 9) to fit this data set with the full direction tuning curves in response to two stimuli moving in different directions at different speeds. Please also see the response to Reviewer 2 about the normalization model fit.

      The results of the normalization model fit are now described on page 20 and Figure 8A, B, D.

      Reviewer #2 (Recommendations For The Authors):

      In terms of impact, I would say that the presentation is geared largely toward people who go to VSS. To broaden the appeal, the authors might consider a more general formulation of the four hypotheses stated at the bottom of page 3. These are prominent ideas in systems neuroscience - population encoding, Bayesian inference, etc.

      We thank the Reviewer for the suggestion. We have revised the Introduction accordingly on pages 3-4, lines 43-69. Please also see the response to Reviewer 3 about the Introduction.

      Figure 5: It might be helpful to show the predictions for different hypotheses. If the response to the transparent stimulus is equal to that of the faster stimulus, you will have a line with slope 1. If it is equal to the response to the slow stimulus, all points will lie on the x-axis. In between you get lines with slopes less than 1.

      In Figures 5F1 and 5F2, we show dotted lines indicating faster-all (i.e., faster-componenttake-all), response averaging, and slower-all (i.e., slower-component-take-all) on the X-axis. We show those labels in between Figs. 5F1 and F2.

      Figure 6: The analysis is not motivated by any particular question, and the results are presented without any quantitation. This section could be better motivated or else removed.

      We now better motivate the section about the response time course on page 16, lines 336 – 339: “The temporal dynamics of the response bias toward the faster component may provide a useful constraint on the neural model that accounts for this phenomenon. We therefore examined the timecourse of MT response to the bi-speed stimuli. We asked whether the faster-speed bias occurred early in the neuronal response or developed gradually.”

      On page 17, lines 354-357, we also state that “At slow speeds, the very early faster-speed bias suggests a likely role of feedforward inputs to MT on the faster-speed bias. The slightly delayed reduction (normalization) in the bi-speed response relative to the stronger component response also helps constrain the circuit model for divisive normalization.”

      Equation (9): There appears to be an "S" missing in the denominator.

      We double-checked and did not see a missing "S" in Equation 9, on page 20.  

      Reviewer #3 (Recommendations For The Authors):

      This is an impressive study, with the chief strengths being the computational/theoretical motivation and analyses and the inclusion of psychophysics together with primate neurophysiology. The manuscript is well-written and the figures are clear and convincing (with a couple of suggestions detailed below).

      We thank the Reviewer for the comments.

      Specific suggestions:

      (1) Intro para 3

      "It is conceivable that the responses of MT neurons elicited by two motion speeds may follow one of the following rules: (1) averaging the responses elicited by the individual speed components; (2) bias toward the speed component that elicits a stronger response, i.e. "soft-max operation" (Riesenhuber and Poggio, 1999); (3) bias toward the slower speed component, which may better represent the more probable slower speeds in nature scenes (Weiss et al., 2002); (4) bias toward the faster speed component, which may benefit the segmentation of a faster-moving stimulus from a slower background."

      This would be a good place to point out which of these options is likely to preserve vs. lose information and how.

      It seems to me that only #2 is clearly information-preserving, assuming that there are neurons with a variety of different speed preferences such that different neurons will exhibit different "winners". #1 would predict subjects would perceive only an intermediate speed, whereas #3 would predict perceiving only/primarily the slower speed and #4 would predict only/primarily perceiving the faster speed.

      The difference between "only" and "primarily" would depend on whether the biases are complete or only partial. I acknowledge that the behavioral task in the study is not a "report all perceived speeds" task, but rather a 1 vs 2 speeds task, so the behavioral assay is not a direct assessment of the question I'm raising here, but I think it should still be possible to write about the perceptual implications of these different possibilities for encoding in an informative way.

      Thanks for the suggestions. We have revised this paragraph in the Introduction on pages 3 – 4, lines 43 – 69.

      (2) Analysis clarifications

      The section "Relationship between the responses to bi-speed stimuli and constituent stimulus components" could use some clarification/rearrangement/polish. I had to read it several times. Possibly, rearrangement, simplification/explanation of nomenclature, and building up from a simpler to a more complex case would help. If I understand correctly, the outcome of the analysis is to obtain a weight value for every combination of slow and fast speeds used. The R's in equation 5 are measured responses, observed on the single stimulus and combined stimulus trials. It was not clear to me if the R's reflect average responses or individual trial responses; this should be clarified. Ws = 1- wf so in essence only 1 weight is computed for each combination. Then, in the subsequent sections of the manuscript, the authors explore whether the weight computed for each stimulus combination is the same or does it vary across conditions. If I have this right, then walking through these steps will aid the reader.

      The Reviewer is correct. We now walk through these steps and better state the rationale for this approach. The R's in Equation 5 are trial-averaged responses, not trial-by-trial responses.

      We have clarified these points on page 13.

      To take a particular example, the sentence "Using this approach to estimate the response weights for individual neurons can be inaccurate because, at each speed pair, the weights are determined only by three data points" struck me as a rather backdoor way to get at the question. Is the estimate noisy? Or does the weighting vary systematically across speeds? I think the authors are arguing the latter; if so, it would be valuable to say so.

      We wanted to estimate the weighting for each speed pair and determine whether the weights change with the stimulus speeds. Indeed, we found that the weights change systematically across speed pairs. The issue was not because the estimate was noisy (see below in response to the second paragraph for point 3.  

      We have clarified this point in the text, on page 13, lines 273 – 280: “Our goal was to estimate the weights for each speed pair and determine whether the weights change with the stimulus speeds. In our main data set, the two speed components moved in the same direction. To determine the weights of 𝑤 and w<sub>f</sub> for each neuron at each speed pair, we have three data points R, R<sub>s</sub>, and R<sub>f</sub>, which are trial-averaged responses. Since it is not possible to solve for both variables, 𝑤 and w<sub>f</sub>, from a single equation (Eq. 5) with three data values, we introduced an additional constraint: 𝑤 + w<sub>f</sub> =1. While this constraint may not yield the exact weights that would be obtained with a fully determined system, it nevertheless allows us to characterize how the relative weights vary with stimulus speed.”

      (3) Figure 5

      Related to the previous point, Figures 5A-E are subject to a possible confound. When plotting x vs y values, it is critical that the x and y not depend trivially on the same value. Here, the plots are R-Rs and Rf-Rs. Rs, therefore, is contained in both the x and y values. Assume, for the sake of argument, that R and Rf are constants, whereas Rs is drawn from a distribution of random noise. When Rs, by chance, has an extreme negative value, R-Rs and Rf-Rs will be large positive values. The solution to this artificial confound is to split the trials that generate Rs into two halves and subtract one half from R and the other half from Rf. Then, the same noisy draw will not be contributing to both x and y. The above is what is needed if the authors feel strongly about including this analysis.

      The Reviewer is correct that subtracting a common term (Rs) would introduce a correlation between (R-Rs) and (Rf-Rs) (Reviewer 2 also raised this point). R's in Equations 5, 6, 7 (and Figure 5A-E) are trial-averaged responses. So, we cannot address the issue by dividing R’s into two halves. Our results showed that the regression slope (W<sub>f</sub>) changed from near 1 to about 0.5 as the stimulus speeds increased, and the correlation coefficient between (R – Rs) and (R<sub>f</sub> – Rs) was high at slow stimulus speeds. To determine whether these results can be explained by the confounding factor of subtracting a common term Rs, rather than by the pattern of R in representing two speeds, we did an additional analysis. We acknowledged the issue and described the new analysis on page 13, lines 303 – 326:

      “Our results showed that the bi-speed response showed a strong bias toward the faster component when the speeds were slow and changed progressively from a scheme of ‘fastercomponent-take-all’ to ‘response-averaging’ as the speeds of the two stimulus components increased (Fig. 5F1). We found similar results when the speed separation between the stimulus components was small (×2), although the bias toward the faster component at low stimulus speeds was not as strong as x4 speed separation (Fig. 5A2-F2 and Table 1).  

      In the regression between (𝑅 – 𝑅<sub>s</sub>) and (𝑅<sub>f</sub> – 𝑅<sub>s</sub>), 𝑅<sub>s</sub> was a common term and therefore could artificially introduce correlations. We wanted to determine whether our estimates of the regression slope (𝑤<sub>f</sub>) and the coefficient of determination (𝑅<sup>2</sup>) can be explained by this confounding factor. At each speed pair and for each neuron from the data sample of the 100 neurons shown in Figure 5, we simulated the response to the bi-speed stimuli (𝑅 <sub>e</sub>) as a randomly weighted sum of 𝑅<sub>f</sub> and 𝑅<sub>s</sub> of the same neuron.

      𝑅<sub>e</sub> = 𝑎𝑅<sub>f</sub> + (1 − 𝑎)𝑅<sub>s</sub>,

      in which 𝑎 was a randomly generated weight (between 0 and 1) for 𝑅<sub>f</sub>, and the weights for 𝑅<sub>f</sub> and 𝑅<sub>s</sub> summed to one. We then calculated the regression slope and the correlation coefficient between the simulated 𝑅<sub>e</sub> - 𝑅<sub>s</sub> and 𝑅<sub>f</sub> - 𝑅<sub>s</sub> across the 100 neurons. We repeated the process 1000 times and obtained the mean and 95% confidence interval (CI) of the regression slope and the 𝑅<sup>2</sup>. The mean slope based on the simulated responses was 0.5 across all speed pairs. The estimated slope (𝑤<sub>f</sub>) based on the data was significantly greater than the simulated slope at slow speeds of 1.25/5, 2.5/10 (Fig. 5F1), and 1.25/2.5, 2.5/5, and 5/10 degrees/s (Fig. 5F2) (bootstrap test, see p values in Table 1). The estimated 𝑅<sup>2</sup> based on the data was also significantly higher than the simulated 𝑅<sup>2</sup> for most of the speed pairs (Table 1). These results suggest that the faster-speed bias at the slow stimulus speeds and the consistent response weights across the neuron population at each speed pair are not analysis artifacts.”

      However, I don't see why the analysis is needed at all. Can't Figure 5F be computed on its own? Rather than computing weights from the slopes in 5A-E, just compute the weights from each combination of stimulus conditions for each neuron, subject to the constraint ws=1-wf. I think this would be simpler to follow, not subject to the noise confound described in the previous point, and likely would make writing about the analysis easier.

      We initially tried the suggested approach to determine the weights of the individual neurons. The weights from each speed combination for each neuron are calculated by:  𝑤<sub>s</sub> = , 𝑤<sub>f</sub> , and 𝑤<sub>s</sub> and 𝑤<sub>f</sub> sum to 1. 𝑅, 𝑅<sub>f</sub> and  𝑅<sub>s</sub> are the responses to the same motion direction. Using this approach to estimate response weights for individual neurons can be unreliable, particularly when 𝑅<sub>f</sub> and 𝑅<sub>s</sub> are similar. This situation often arises when the two speeds fall on opposite sides of the neuron's preferred speed, resulting in a small denominator (𝑅<sub>f</sub> - 𝑅<sub>s</sub>) and, consequently, an artificially inflated weight estimate. We therefore used an alternative approach. We estimated the response weights for the neuronal population at each speed pair (𝑅<sub>f</sub> - 𝑅<sub>s</sub>) using linear regression of (𝑅 - 𝑅<sub>s</sub>) against (𝑅<sub>f</sub> - 𝑅<sub>s</sub>). The slope is the weight for the faster component for the population. This approach overcame the difficulty of determining the response weights for single neurons.

      Nevertheless, if the data provide better constraints, it is possible to estimate the response weights for each speed pair for individual neurons. For example, we can calculate the weights for single neurons by using stimuli that move in different directions at two speeds. By characterizing the full direction tuning curves for R, R<sub>f</sub>, and Rs, we have sufficient data to constrain the response weights for single neurons, as we did for the speed pair of 2.5 and 10º/s in Figure 8. In future studies, we can use this approach to measure the response weights for single neurons at different speed pairs and average the weights across the neuron population.  

      We explain these considerations in the Results (pages 13–14, lines 265-326) and Discussion (pages 34-35, lines 818-829).

      (4) Figure 7

      Bidirectional analysis. It would be helpful to have a bit more explanation for why this analysis is not subject to the ws=1-wf constraint. In Figure 7B, a line could be added to show what ws + wf =1 would look like (i.e. a line with slope -1 going from (0,1) to (1,0); it looks like these weights are a little outside that line but there is still a negative trend suggesting competition.

      For the data set when visual stimuli move in the same direction at different speeds, we included a constraint that W<sub>s</sub> and W<sub>f</sub> sum to 1. This is because one cannot solve two independent variables (Ws and Wf) using one equation R = W<sub>s</sub> · R<sub>s</sub> + W<sub>f</sub> R<sub>f</sub>, with three data values (R, Rs, Rf).

      In the dataset using bi-directional stimuli (now Fig. 8), we can use the full direction tuning curves to constrain the linear weighted (LWS) summation model and the normalization model. So, we did not need to impose the additional constraint that Ws and Wf sum to one, which is more general. We now clarify this in the text, on page 19, lines 421-423.

      As suggested, we added a line showing Ws + Wf = 1 for the LWS model fit (Fig. 8C) and the normalization model fit (Fig. 8D) (also see page 21, lines 482-484). Although 𝑤 and 𝑤 are not constrained to sum to one in the model fits, the fitted weights are roughly aligned with the dashed lines of Ws + Wf = 1.

      (5) Attention task

      General wording suggestions - a caution against using "attention" as a causal/mechanistic explanation as opposed to a hypothesized cognitive state. For example, "We asked whether the faster-speed bias was due to bottom-attention being drawn toward the faster stimulus component". This could be worded more conservatively as whether the bias is "still present if attention is directed elsewhere" - i.e. a description of the experimental manipulation.

      We intended to test the hypothesis of whether the faster-speed bias can be explained by attention automatically drawn to the faster component and therefore enhance the contribution of the faster component to the bi-speed response. We now state it as a possible explanation to be tested. We changed the subtitle of this section to be more conservative: “Faster-speed bias still present when attention was directed away from the RFs”, on page 18, line 363.

      We also modified the text on page 18, lines 364-367: “One possible explanation for the faster-speed bias may be that bottom-up attention is drawn toward the faster stimulus component, enhancing the response to the faster component. To address this question, we asked whether the faster-speed bias was still present if attention was directed away from the RFs.”

      Relatedly, in the Discussion, the section on "Neural mechanisms", the sentence "The faster-speed bias was not due to an attentional modulation" should be rephrased as something like 'the bias survived or was still present despite an attentional modulation requiring the monkey to attend elsewhere'.

      Our motivation for doing the attention-away experiment was to determine whether a bottom-up attentional modulation can explain the faster-speed bias. We now describe the results as suggested by the Reviewer. But we’d also like to interpret the implications of the results. In Discussion, page 34, lines 789-790, we now state: “We found that the faster-speed bias was still present when attention was directed away from the RFs, suggesting that the faster-speed bias cannot be explained by an attentional modulation.”  

      (6) "A model that accounts for the neuronal responses to bi-speed stimuli". This section opens with: "We showed that the neuronal response in MT to a bi-speed stimulus can be described by a weighted sum of the neuron's responses to the individual speed components". "Weighted average" would be more appropriate here, given that ws = 1-wf.

      As mentioned above, the added constraint of Ws+Wf = 1 was only a practical solution for determining the weights for the data set using visual stimuli moving in the same direction. More generally, Ws and Wf do not need to sum to one. As such, we prefer the wording of weighted sum.

      (7) "As we have shown previously using visual stimuli moving transparently in different directions, a classifier's performance of discriminating a bi-directional stimulus from a singledirection stimulus is worse when the encoding rule is response-averaging than biased toward one of the stimulus components" - this is important! Can this be worked into the Introduction?

      Yes, we now also mention this point in the Introduction regarding response averaging on page 4, lines 54-57: “While decoding two stimuli from a unimodal response is theoretically possible (Zemel et al., 1998; Treue et al., 2000), response averaging may result in poorer segmentation compared to encoding schemes that emphasize individual components, as demonstrated in neural coding of overlapping motion directions (Xiao and Huang, 2015).” Also, please see the response to point 1 above.

      (8) Minor, but worth catching now - is the use of initials for human participants consistent with best practices approved at your institution?

      Thanks for checking. The letters are not the initials of the human subjects. They are coded characters. We have clarified it in the legend of Figure 1, on page 7, line 168.

    1. Author Response

      Reviewer #1 (Public Review):

      Summary:

      In this paper, the effects of two sensory stimuli (visual and somatosensory) on fMRI responsiveness during absence seizures were investigated in GEARS rats with concurrent EEG recordings. SPM analysis of fMRI showed a significant reduction in whole-brain responsiveness during the ictal period compared to the interictal period under both stimuli, and this phenomenon was replicated in a structurally constrained whole-brain computational model of rat brains.

      The conclusion of this paper is that whole-brain responsiveness to both sensory stimuli is inhibited and spatially impeded during seizures.

      I also suggest the manuscript should be written in a way that is more accessible to readers who are less familiar with animal experiments. In addition, the implementation and interpretation of brain simulations need to be more careful and clear.

      Several sections of the manuscript were clarified and simplified to be more accessible. Also, implementation and interpretations of brain simulations were modified to be more precise.

      Strengths:

      1) ZTE imaging sequence was selected over traditional EPI sequence as the optimal way to perform fMRI experiments during absence seizures.

      2) A detailed classification of stimulation periods is achieved based on the relative position in time of the stimulation period with respect to the brain state.

      3) A whole-brain model embedded with a realistic rat connectome is simulated on the TVB platform to replicate fMRI observations.

      We thank the reviewer for indicating the strengths of our manuscript.

      Weaknesses:

      1) The analysis in this paper does not directly answer the scientific question posed by the authors, which is to explore the mechanisms of the reduced brain responsiveness to external stimuli during absence seizures (in terms of altered information processing), but merely characterizes the spatial involvement of such reduced responsiveness. The same holds for the use of mean-field modeling, which merely reproduces experimental results without explaining them mechanistically as what the authors have claimed at the head of the paper.

      We agree with the reviewer that the manuscript does not answer specifically about the mechanisms of reduced brain responsiveness. The main scientific question addressed in the manuscript was to compare whole-brain responsiveness of stimulus between ictal and interictal states. The sentence that can lead to misinterpretations in the manuscript abstract: “The mechanism underlying the reduced responsiveness to external stimulus remains unknown.” was therefore modified to the following “The whole-brain spatial and temporal characteristics of reduced responsiveness to external stimulus remains unknown”.

      2) The implementations of brain simulations need to be more specific.

      Contribution:

      The contribution of this paper is performing fMRI experiments under a rare condition that could provide fresh knowledge in the imaging field regarding the brain's responsiveness to environmental stimuli during absence seizures.

      Reviewer #2 (Public Review):

      Summary:

      This study examined the possible effect of spike-wave discharges (SWDs) on the response to visual or somatosensory stimulation using fMRI and EEG. This is a significant topic because SWDs often are called seizures and because there is non-responsiveness at this time, it would be logical that responses to sensory stimulation are reduced. On the other hand, in rodents with SWDs, sensory stimulation (a noise, for example) often terminates the SWD/seizure.

      In humans, these periods of SWDs are due to thalamocortical oscillations. A certain percentage of the normal population can have SWDs in response to photic stimulation at specific frequencies. Other individuals develop SWDs without stimulation. They disrupt consciousness. Individuals have an absent look, or "absence", which is called absence epilepsy.

      The authors use a rat model to study the responses to stimulation of the visual or somatosensory systems during and in between SWDs. They report that the response to stimulation is reduced during the SWDs. While some data show this nicely, the authors also report on lines 396-8 "When comparing statistical responses between both states, significant changes (p<0.05, cluster-) were noticed in somatosensory auditory frontal..., with these regions being less activated in interictal state (see also Figure 4). That statement is at odds with their conclusion.

      We thank the reviewer for noting this discrepancy. The statement should have been written vice versa and it has been corrected as: “When comparing statistical responses between both states, significant changes (p<0.05, cluster-level corrected) were noticed in the somatosensory, auditory and frontal cortices: these regions were less activated in ictal than in interictal state (see also Figure 4).”

      They also conclude that stimulation slows the pathways activated by the stimulus. I do not see any data proving this. It would require repeated assessments of the pathways in time.

      We agree with the reviewer that there are no data showing slowing of the pathways in response to stimulus. However, we are a bit confused about this comment, as to what part in conclusion section it refers to. We did not intentionally claim that stimulation slows the activated pathways in the manuscript.

      The authors also study the hemodynamic response function (HRF) and it is not clear what conclusions can be made from the data.

      Hemodynamic response functions were studied for two reasons:

      • To account for possible change in HRF during the detection of activated regions. Indeed, a physiological change in HRF can mask the detection of an activation when the software uses a standard HRF to convolve the design matrix (David et al. 2008).

      • To characterize the shape and polarity of fMRI activations in brain regions that we noticed to be differently activated between ictal and interictal states and evaluate whether alteration in activation was associated to alteration in hemodynamic.

      The observed HRF decreases (rather than increases) in the cortex when stimulation was applied during SWD, was discussed in section 4.4., where we speculated that neuronal suppression caused by SWD can prevent responsiveness. In this case, the decreased HRF could either be a consequence or a cause of the observed neuronal suppression. The assumption that the HRF reduction is causal would be supported by a possible vascular steal effect from other activation regions. However, in the conclusion section we did not state this and therefore the following sentence was added to conclusions: “Moreover, the detected decreases in the cortical HRF when sensory stimulation was applied during spike-and-wave discharges, could play a role in decreased sensory perception. Further studies are required to evaluate whether this HRF change is a cause or a consequence of the reduced neuronal response”.

      Finally, the authors use a model to analyze the data. This model is novel and while that is a strength, its validation is unclear. The conclusion is that the modeling supports the conclusions of the study, which is useful.

      Details about the model were added.

      Strengths:

      Use of fMRI and EEG to study SWDs in rats.

      Weaknesses:

      Several aspects of the Methods and Results are unclear.

      Reviewer #3 (Public Review):

      Summary:

      This is an interesting paper investigating fMRI changes during sensory (visual, tactile) stimulation and absence seizures in the GAERS model. The results are potentially important for the field and do suggest that sensory stimulation may not activate brain regions normally during absence seizures. However the findings are limited by substantial methodological issues that do not enable fMRI signals related to absence seizures to be fully disentangled from fMRI signals related to the sensory stimuli.

      Strengths:

      Investigating fMRI brain responses to sensory stimuli during absence seizures in an animal model is a novel approach with the potential to yield important insights.

      The use of an awake, habituated model is a valid and potentially powerful approach.

      Weaknesses:

      The major difficulty with interpreting the results of this study is that the duration of the visual and auditory stimuli was 6 seconds, which is very close to the mean seizure duration per Table 1. Therefore the HRF model looking at fMRI responses to visual or auditory stimuli occurring during seizures was simultaneously weighting both seizure activity and the sensory (visual or auditory) stimuli over the same time intervals on average. The resulting maps and time courses claiming to show fMRI changes from visual or auditory stimulation during seizures will therefore in reality contain some mix of both sensory stimulation-related signals and seizure-related signals. The main claim that the sensory stimuli do not elicit the same activations during seizures as they do in the interictal period may still be true. However the attempts to localize these differences in space or time will be contaminated by the seizure-related signals.

      The claims that differences were observed for example between visual cortex and superior colliculus signals with visual stim during seizures vs. interictal are unconvincing due to the above.

      We understand this concern expressed by the reviewer and agree that seizure-related signals must be considered in the analysis when studying stimulation responses. Therefore, in modelling the responses in the SPM framework, we considered both stimulation and seizure-only states as regressors of interest and used seizure-only responses as nuisance regressors to account for error variance. Thereby, the effects caused by the stimulation should be, in theory, separated as much as possible from the effects caused by the seizure itself. Additionally, the cases where stimulations occurred fully inside a seizure (included in Figure 3, “...stimulation during ictal state) actually had a longer average seizure duration of 45 ± 60 s, therefore being much longer than 6s which an average duration taken from all seizures.

      However, we acknowledge that there is a potential that some leftover effects from a seizure are still present, and we have noted this caution in the “Physiologic and methodologic considerations” section: “We note a caution that presented maps and time courses showing fMRI changes from visual or whisker stimulation during seizures may contain mixture of both sensory stimulation-related signals and seizure-related signals. To minimize this contamination, we considered in SPM both stimulation and seizure-only states as regressors of interest and used seizure-only responses as nuisance regressors to account for error variance. Thereby, the effects caused by the seizure itself should be separated as much as possible from the effects caused by stimulation.”

      The maps shown in Figure 3 do not show clear changes in the areas claimed to be involved.

      We clarified the overall appearance of Figure 3, by enlarging the selected cross sections for better anatomical differentiation and added anterior and posterior directions on all images.

      Reviewer #1 (Recommendations For The Authors):

      1) The implementations of brain simulations need to be more specific: How is the stimulation applied in the mean-field model in terms of its mathematical expression? The state variable of the model is the rate of neuronal firing, but how is it subsequently converted into fMRI responses? How are the statistical plots calculated? How much does this result depend on the model parameter?

      Further details and explanations about the model have now been added to the manuscript. The stimulation of a specific region is simulated as an increase in the excitatory input to the specific node. In particular we use a square function for representing the stimulus (see for example panel A in Figure 6–figure supplement 1). As the referee mentions, the model describes the dynamics of the neuronal firing rates. This provides direct information about neuronal activity and responsiveness for which all the statistical analyses of the simulations shown in the paper were performed using the firing rates. For these analyses, no conversion to fMRI was needed. To build the statistical maps, an ANOVA (analysis of variance) test was used. The ANOVA test is originally designed to assess the significance of the change in the mean between two samples, and is calculated via an F-test as the ratio of the variance between and within samples. In our case it allowed us to assess the impact of the stimulation on the ongoing neuronal activity by performing a comparison of the timeseries of the firing rate with and without stimulation (this was performed independently for each state). For the results presented in this paper, the ANOVA analysis was performed using the “f_oneway” function of the scipy.stats. module in python. Regarding the dependence on the model parameter, the main results obtained in our paper are related with the responsiveness of the system under two quantitatively different types of ongoing dynamics: an asynchronous irregular activity (interictal period) and an oscillatory SWD type of dynamics (ictal period). In particular, we show how for the SWD dynamics the activity evoked by the stimulus is overshadowed by the ongoing activity which imposes a strong limitation in the response of the system and the propagation of the stimulus. In this sense, the main results of the simulations are very general, and no significant dependence on specific cellular or network parameters was observed within a physiologically relevant range or should be expected. Nevertheless, we point out that, as mentioned in the text, the key parameter that triggers the transition between the two types of dynamics is the strength of the adaptation current (in particular the strength of the spike-triggered adaptation parameter ‘b’ described in the Supplementary information), which in addition has the capacity of controlling the frequency of the oscillations. In the paper, this parameter was set such that the SWD frequency falls within the range observed in the GAERS (between 7-12Hz). We believe that further analysis around the region of transition between states, in particular from a dynamical point of view, could be of relevance for future work.

      2) In the abstract, what exactly does "typical information flow in functional pathways" mean and which part of the results does this refer to?

      We note that this sentence was overly complicated. By “typical information flow”, we were referring to sensory responsiveness during interictal state. Therefore, we made the following modifications to the abstract: “These results suggest that sensory processing observed during an interictal state can be hindered or even suppressed by the occurrence of an absence seizure, potentially contributing to decreased responsiveness.”

      3) Figure 4 - Figure Supplement 1 performed an analysis of comparing states between 'when stimulation ended a seizure' and 'stimulation during an ictal period'. The authors should explain more clearly in the manuscript what is the reason and significance of considering the state of 'when stimulation ended a seizure'. And how is a seizure considered to be terminated by stimulation rather than ending spontaneously?

      We have now added explanations to the manuscript section 2.5.3 as why this state was also of interest: “The case when stimulation ended a seizure is particularly interesting for studying the spatial and temporal aspects explaining shift from ictal, i.e. non-responsiveness state, to non-ictal, i.e. responsiveness state.” We agree that there is a possibility that seizures ended spontaneously at the same time as stimulus was applied but argue that seizures most probably end due to stimulation, based on results published previously (https://doi.org/10.1016/j.brs.2012.05.009).

      4) In Section 3.1, some detailed descriptions of methods should be moved to Section 2, e.g. how the spatial and temporal SNR is obtained and the description of bad quality data. Also, I suggest the significance of selecting the optimal MRI sequence be stated earlier in the paper, as Section 3.1 cannot be expected from reading the abstract and introduction.

      We moved some technical explanations of SNRs from section 3.1. to section 2.4.1. Significance of the selection of the MRI sequence is also now stated earlier in the introduction section: “For this purpose, the functionality of ZTE sequence was first piloted, and selected over traditional EPI sequence for its lower acoustic noise and reduced magnetic susceptibility artefacts. The selected MRI sequence thus appeared optimal for awake EEG-fMRI measurements.”

      Some minor issues:

      1) How is ROI defined in this paper? What type of atlas is used?

      Anatomical ROIs were drawn based on Paxinos and Watson rat brain atlas 7th edition. Region was selected if there were statistically significant activations detected inside that region, based on activation maps. We clarified the definition of ROI as the following: “Anatomical ROIs, based on Paxinos atlas (Paxinos and Watson rat brain atlas 7th edition), were drawn on the brain areas where statistical differences were seen in activation maps.”

      2) Section 4.3.2, "In addition, some responses were seen in the somatosensory cortex during the seizure state, which may be due to the fact that the linear model used did not completely remove the effect of the seizure itself" What is the reason for the authors to make such comments?

      This claim was made because we saw similar trend of responses (deactivation) in F-contrast maps in the somatosensory cortex, when comparing “stimulation during ictal state” maps to "seizure map", leading us to assume that the effect of seizure was still apparent in the maps (even though “seizure only” states were used as nuisance regressors). However, as this claim is highly speculative, we have decided to delete this sentence in the manuscript.

      3) Abbreviations such as SPM, HRF, CBF, etc. are not defined in the manuscript.

      Definitions for these abbreviations were added.

      4) Supplementary information-AdEx mean-field model, 've and vi', e and i should be subscripted.

      Subscripts were added.

      Reviewer #2 (Recommendations For The Authors):

      Below are more detailed questions and concerns. Many questions are about the Methods, which seem to be written by a specialist. However, there are also questions about the experimental approach and conclusions.

      One of the strengths of the study is the use of fMRI and EEG. However, to allow rats to be still in the magnet, isoflurane was used, and then as soon as rats recovered they were imaged. However isoflurane has effects on the brain long after the rats have appeared to wake up. Moreover, to train rats to be still, repetitive isoflurane sessions had to be used. Repetitive isoflurane should have a control of some kind, or be discussed as a limitation.

      The repetitive use of isoflurane is indeed an important limiting factor that was not yet discussed in the manuscript. We have added the following sentences to the “Physiologic and methodologic considerations” section:

      “As the used awake habituation and imaging protocol didn’t allow us to avoid the usage of isoflurane during the preparation steps, we cannot rule out the possible effect of using repetitive anesthesia on brain function. However, duration (~15 min) and concentration of anesthesia (~1.5%) during these steps were still moderate, whereas extended durations (1-3 h) of either single or repetitive isoflurane exposures have been used in previous studies where long-term effects on brain function have been observed (Long II et al., 2016; Stenroos et al., 2021). Moreover, there was a 5-15 min waiting period between the cessation of anesthesia and initiation of fMRI scan, to avoid the potential short-term effects of isoflurane that has been found to be most prominent during the 5 min after isoflurane cessation (Dvořáková et al., 2022).

      An assumption of the study is that interictal periods are normal. However, they may not be. A control is necessary. One also wants to know how often GAERS have spontaneous spike-wave discharges (SWDs), what the authors call seizures. The reason is that the more common the SWDs, the less likely interictal periods are normal. It seems from the Methods that rats were selected if they had frequent seizures so many could be captured in a recording session. Those without frequent seizures were discarded.

      A good control would be a normal rat that has spontaneous SWDs, since almost all rat strains have them, especially with age and in males (PMID: 7700522). However, whether they are frequent enough might be a problem. Alternatively, animals could be studied with rare seizures to assess the normal baseline, and compared to interictal states in GAERS.

      We appreciate this concern raised by the Reviewer. Even though it would be interesting to study different strains and SWD frequency dependence, the aim of this study was to compare interictal vs ictal states in this specific animal model. We also understand that interictal periods could not necessarily model “normal” state and therefore went through the manuscript again to remove any claims referring to this.

      About the mechanisms of SWDs, the authors should update their language which seems imprecise and lacks current citations (starting on line 71):

      "Although the origin of absence seizures is not fully understood, current studies on rat models of absence seizures suggest that they arise from atypical excitatory-inhibitory patterns in the barrel field of the somatosensory cortex (Meeren et al. 2002; Polack et al. 2007) and lead to synchronous cortico-thalamic activity (Holmes, Brown, and Tucker 2004)."

      Some of the best explanations for SWDs that I know of are from the papers of John Huguenard. His reviews are excellent. They discuss the mechanisms of thalamocortical oscillations.

      We have reformatted the sentences discussing the mechanism of SWDs and included the explanations provided by manuscripts from Huguenard and McCafferty et al.: “Although the origin of absence seizures is not fully understood, current studies on rat models of absence seizures suggest that they arise from excitatory drive in the barrel field of the somatosensory cortex (Meeren et al. 2002; Polack et al. 2007, 2009, David et al., 2008) and then propagate to other structures (David et al., 2008) including thalamus, knowing to play an essential role during the ictal state (Huguenard, 2019). Notably, the thalamic subnetwork is believed to play a role in coordinating and spacing SWDs via feedforward inhibition together with burst firing patterns. These lead to the rhythms of neuronal silence and activation periods that are detected in SWD waves and spikes (McCafferty et al., 2018; Huguenard, 2019).”

      The following also is not precise:

      "Although seizures are initially triggered by hyperactive somatosensory cortical neurons, the majority of neuronal populations are deactivated rather than activated during the seizure, resulting in an overall decrease in neuronal activity during SWD (McCafferty et al. 2023)." What neuronal populations? Cortex? Which neurons in the cortex? Those projecting to the thalamus? What about thalamocortical relay cells? Thalamic gabaergic neurons?

      Lines 85-8: "In addition, a previous fMRI study on GAERS, which measured changes in cerebral blood volume, found both deactivated and activated brain areas during seizures (David et al. 2008). Which areas and conditions led to reduced activity? Increased activity? How was it surmised?

      "concurrent stimuli and therefore could contribute to the alterations in behavioral responsiveness" - This idea has been raised before by others (Logthetis, Barth). Please discuss these as the background for this study.

      The particular section was modified to the following:

      “Previous results on GAERS have indicated that, during an absence seizure, hyperactive electrophysiological activity in the somatosensory cortex can contribute to bilateral and regular SWD firing patterns in most parts of the cortex. These patterns propagate to different cortical areas (retrosplenial, visual, motor and secondary sensory), basal ganglia, cerebellum, substantia nigra and thalamus (David et al. 2008; Polack et al. 2007). Although SWDs are initially triggered by hyperactive somatosensory cortical neurons, neuronal firing rates, especially in majority of frontoparietal cortical and thalamocortical relay neurons, are decreased rather than increased during SWD, resulting in an overall decrease in activity in these neuronal populations (McCafferty et al. 2023). Previous fMRI studies have demonstrated blood volume or BOLD signal decreases in several cortical regions including parietal and occipital cortex, but also, quite surprisingly, increases in subcortical regions such as thalamus, medulla and pons (David et al., 2008; McCafferty et al., 2023). In line with these findings, graph-based analyses have shown an increased segregation of cortical networks from the rest of the brain (Wachsmuth et al. 2021). Altogether, alterations in these focal networks in the animal models of epilepsy impairs cognitive capabilities needed to process specific concurrent stimuli during SWD and therefore could contribute to the lack of behavioral responsiveness (Chipaux et al. 2013; Luo et al. 2011; Meeren et al. 2002; Studer et al. 2019), although partial voluntary control in certain stimulation schemes can be still present (Taylor et al., 2017).”

      Please discuss the mean-field model more. What are its assumptions? What is its validation? Do other models also provide the same result?

      We have now extended the discussion and explanation of the mean-field model, both in the main text and in the Supplementary information. The mean-field model is a statistical tool to estimate the mean activity of large neuronal populations, and as such its main assumptions are centered around the size of the population analyzed and the characteristic times of the neuronal dynamics under study. It has been shown that the formalism is valid for characteristic times of neuronal dynamics with a lower bond in the order of few milliseconds and with population size of in the order thousands of neurons (see El Boustani and Destexhe, Neural computation 2009; and Di Volo et al, Neural computation 2019), with both conditions satisfied in the simulations made for this work. Regarding the validation, the model has been extensively validated and used for simulating different brain states (Di Volo et al. 2009; Goldman et al. 2023), signal propagation in cortical circuits (Zerlaut et al, 2018) and to perform whole-brain simulations (Goldman et al, 2023). The standard validation of the mean-field implies its comparison with the activity obtained from the corresponding spiking neural network. For completeness we show in Author response image 1 an example of the SWD type of dynamics obtained from a spiking neural network together with the one obtained from the mean-field. This figure has been added now to the Supplementary information of the paper. Regarding the extension of the results to other models, we think that the generality of our results is an interesting point from our work. The main results obtained from our simulation are related with the responsiveness of the system during two different type of ongoing activity: in the interictal state there is a significant variation on the ongoing activity evoked by the stimulation that is propagated to other regions, while in the SWD state the evoked activity is overshadowed by the ongoing activity which imposes a strong limit to the responsiveness of the system and the propagation of the signal. In this sense, the results of the simulations are very general and should be extensible to other models. Of course, the advantage of using a model like ours is the capability of reproducing the different states, its applicability to large scale simulations, and the fact that it is built from biologically relevant single-cell models (AdEx).

      Author response image 1.

      Comparison of the SWD dynamics in the mean-field model and the underlying spiking-neural network of AdEx neurons. A) Raster plot (top) and mean firing rate (bottom) from an SWD type of dynamics obtained from the spiking- network simulations. The network is made of 8000 excitatory neurons and 2000 inhibitory neurons. Neurons in the network are randomly connected with probability p=0.05 for inhibitory-inhibitory and excitatory-inhibitory connections, and p=0.06 for excitatory-excitatory connections. Cellular parameters correspond to the ones used in the mean-field, with spike-triggered adaptation for excitatory neurons set to b=200pA. We show the results for excitatory (green) and inhibitory (red) neurons. B) Mean-firing rate obtained from a single mean-field model. We see that, although the amplitude of oscillations is larger in the spiking-network, the mean-field can correctly capture the general dynamics and frequency of the oscillations.

      Line 11: "rats were equally divided by gender." Given n=11, does that mean 5 males and 6 females or the opposite?

      Out of 11 animals, 6 were males, and 5 females. This is now mentioned in the manuscript.

      What was the type of food?

      Type of food was added to the manuscript (Extrudat, vitamin-fortified, irradiated > 25 kGy)

      What were the electrodes?

      This was provided in the manuscript. Carbon fiber filament was produced by World Precision Instruments. The tips of this filament were spread to brush-like shape to increase the contact surface above the skull.

      "low noise zero echo time (ZTE) MRI sequence"- please explain for the non-specialist or provide references.

      Reference added.

      Lines 148-150: "The length of habituation period was selected based on pilot experiments and was sufficient for rats to be in low-stress state and produce absence seizures inside the magnet." How do the authors know the rats were in a low-stress state?

      This claim was based on two factors. At the end of the habituation protocol, the motion of animals was considerably decreased according to previous study using similar restraint/habituation protocol (DOI: 10.3389/fnins.2018.00548). In this study the decreased motion is also correlated with decreased blood corticosterone levels which reduced to baseline levels (indicating low-stress state) after 4 days of habituation. Another factor is when epileptic rodents are continuously recorded for 24h, most SWDs occur during a state of passive wakefulness or drowsiness (Lannes et al. 1988, Coenen et al. 1991) . Either way, as we don’t have a way to provide direct evidence of low-stress state, we modified the sentence to the following:

      “The length of habituation period was selected based on pilot experiments to provide low-motion data therefore giving rats a better chance to be in a low-stress state and thus produce absence seizures inside the magnet.”

      Lines 150-2: "Respiration rate and motion were monitored during habituation sessions using a pressure pillow and video camera to estimate stress level." What were the criteria for a high stress level?

      Criteria for high (or low) stress levels were based mostly on motion levels according to previous study (DOI: 10.1016/s0149-7634(05)80005-3). Still, as we didn’t measure direct measures of stress, we modified the sentence to the following:

      “Pressure pillow and video camera were used to estimate physiological state, via breathing rate, and motion level, respectively.”

      Lines 152-3: "During the last habituation session, EEG was measured to confirm that the rats produced a sufficient amount of absence seizures (10 or more per session)." If 10 min, the rats would basically be seizing the entire session, leading to doubt about what the interictal state was.

      The length of the last habituation session was 60min and the fMRI scan 45min. Given that rats produced ~40-50 seizures during fMRI scan, on average they produced ~1 seizures/min, and one seizure lasting on average of 5-6s, giving ~45s periods for interictal states. 10 or more seizures were used as a threshold to give statistically meaningful findings based on pilot experiments.

      Line 153: "Total of 2-5 fMRI experiments were conducted per rat within a 1-3-week period." What was the schedule for each animal? A table would be useful. If it varied, how do the authors know this was justified?

      Please see Figure 1–figure supplement 2 for examples of habituation timelines for individual rats:

      We found an error when stating 2-5 fMRI experiments, but it should be 3-5 fMRI experiments. This was corrected. We had an aim to acquire 12-14 sessions per stimulation condition and once a sufficient number of sessions were acquired, part of the animals was not used further. Two of the animals that were found to have good quality EEG and produced sufficient amounts of SWDs were kept, and briefly retrained for later second stimulation condition experiments. This was done to replace animals that needed to be excluded in the second stimulation condition due to bad quality EEG or lost implant. Extended use of some animals could theoretically bring slight variation to results but could actually be an advantage as animals were already well trained providing low-motion data.

      "Before and after each habituation session, rats were given a treat of sugar water and/or chocolate cereals as positive reinforcement. " How much and what was the concentration of sugar water; chocolate cereal?

      Rats were given 3 chocolate cereals and/or 1% sugar water. This was added to the manuscript now.

      Line 188: "We relied on pilot calibration of the heated water to maintain the body temperature" Please explain.

      Sentence was clarified:

      “We relied on pilot calibration of the temperature of heated water circulating inside animal bed to maintain the normal body temperature of ~37 °C"

      Line 190: "After manual tuning and matching of the transmit-receive coil, shimming and anatomical imaging" Please explain for the non-specialist.

      Sentence was simplified:

      “After routine preparation steps in the MRI console were done"

      Lines 199-201: "Anatomical imaging was conducted with a T1-FLASH sequence (TR: 530 ms, TE: 4 ms, flip angle 196 18{degree sign}, bandwidth 39,682 kHz, matrix size 128 x 128, 51 slices, field-of-view 32 x 32 mm², resolution 0.25 x 0.25 x 0.5 mm3). fMRI was performed with a 3D ZTE sequence (TR: 0.971 ms, TE: 0 ms, flip angle 4{degree sign}, pulse length 1 µs, bandwidth 150 kHz, oversampling 4, matrix size 60 x 60 x 60, field-of-view 30 x 30 x 60 mm3 , resolution of 0.5 x 0.5 x 1 mm3 , polar under sampling factor 5.64 nr. of projections 2060 resulting to a volume acquisition time of about 2 s). A total of 1350 volumes (45 min) were acquired." Please explain for the non-specialist.

      These technical parameters are provided for the sake of repeatability. Section was however clarified as the following and citation was added:

      Anatomical imaging was conducted with a T1-FLASH sequence (repetition time: 530 ms, echo time: 4 ms, flip angle 18°, bandwidth 39,682 kHz, matrix size 128 x 128, 51 slices, field-of-view 32 x 32 mm², spatial resolution 0.25 x 0.25 x 0.5 mm3). fMRI was performed with a 3D ZTE sequence (repetition time: 0.971 ms, TE: 0 ms, flip angle 4°, pulse length 1 µs, bandwidth 150 kHz, oversampling 4, matrix size 60 x 60 x 60, field-of-view 30 x 30 x 60 mm3, spatial resolution of 0.5 x 0.5 x 1 mm3, polar under sampling factor 5.64, number of projections 2060 resulting to a volume acquisition time of about 2 s (look Wiesinger & Ho, 2022 for parameter explanations)). A total of 1350 volumes (45 min) were acquired.

      "Visual (n=14 sessions, 5 rats) and somatosensory whisker (n=14 sessions, 4 rats)" - Please explain how multiple sessions were averaged for a single rat. Please justify the use of different numbers of sessions per rat.

      All the sessions belonging to the same stimulus scheme (multiple sessions per rat) were put at the once as sessions in SPM analysis together with all the stimulus conditions belonging to these sessions. Justifications for using a different number of sessions per rat, were given above.

      Lines 205-206: "For the visual stimulation, light pulses (3 Hz, 6 s total length, pulse length 166 ms) were produced by a blue led, and light was guided through two optical fibers to the front of the rat's eyes. What wavelength of blue? Why blue? Is the stimulation strong? Weak?

      Wavelength was 470 nm and brightness 7065 mcd with a current of 20mA. Blue was selected as it is in the frequency range that rat can differentiate and this color has been used in previous literature ( https://doi.org/10.1016/j.neuroimage.2020.117542, https://doi.org/10.1016/j.jneumeth.2021.109287)

      Line 212: "Stimulation parameters were based on previous rat stimulation fMRI studies to produce robust responses" What is a robust response? One where a lot of visual cortical voxels are activated?

      Sentence was corrected as the following:

      “Stimulation parameters were based on previous rat stimulation fMRI studies and chosen to activate voxels widely in visual and somatosensory pathways, correspondingly.”

      Line 245: "Seizures were confirmed as SWDs if they had a typical regular pattern, had at least double the amplitude compared to baseline signal..." What was the "typical" pattern? What baseline signal was it compared to? Was the baseline measured as an amplitude? Peak to trough?

      Sentence was corrected to the following:

      “Seizures were confirmed as SWDs if they had a typical regular spike and wave pattern with 7-12 Hz frequency range and had at least double the amplitude compared to baseline signal. All other signals were classified as baseline i.e. signal absent of a distinctive 7-12 Hz frequency power but spread within frequencies from 1 to 90 Hz.”

      "using rigid, affine, and SYN registrations" Please explain for the non-specialist.

      Corrected as the following:

      “using rigid, affine (linear) and SYN (non-linear) registrations”

      Line 274-5: "However, there were also intermediate cases where the seizure started or ended during the stimulation block (Figure 1 - Figure Supplement 1). These intermediate cases were modeled as confounds" Why confounds? They could be very interesting because the stimulation may not be affected if timed at the end of the seizure. What was the definition of start and end? Defining the onset and end of seizures is tricky.

      We agree that these cases are also highly interesting. Indeed, all the intermediate cases were also analyzed separately but not included in the manuscript (other than the case when stimulation immediately ended a seizure) as no statistical findings were found when comparing these cases to the baseline. E.g. for the case when stimulation was applied towards the end of seizure, it provided weakened responses but still stronger compared to case when stimulation was applied fully during a seizure (indicating some responsiveness after the cessation of seizure). As these intermediate cases led to results with higher variance, we considered them as confounds in the general linear model (i.e. reducing unwanted variance from the results of interests).

      Definition of onset and end of seizure can be difficult in some cases. When looking at the signal itself, especially towards the end of seizure the amplitude of SWDs can get weaker and thus the shift from seizure to baseline signal can be more problematic to differentiate. However, when looking at the power spectrum the boundaries were more easily detectable. Thus, in the definitions of onsets and ends of seizure we relied on both the signal and power spectrum (stated in the manuscript).

      "in the SPM analysis" Please explain for the non-specialist.

      Definition of SPM together with a link to software site was added.

      Line 276: "of fMRI data (see 2.5.3.) and thus explained variance that was not accounted for by the main effects of interest. " Please clarify.

      Clarified as:

      “Intermediate cases, where the seizure started or ended during the stimulation block (Figure 1–figure supplement 1), were considered as confounds of no-interest in the SPM analysis of fMRI data and the explained variance caused by the confounds were reduced from the main effects of interests”

      Line 277: "Additionally, a contrast..." What is meant?

      This chapter in 2.5.3. was modified as a whole to be more clear.

      Line 278-9: "...was given to two cases: i) when stimulation ended a seizure (0-2 s between stimulation start and seizure end)..." Again, how is the seizure onset and end defined?

      Look comment above.

      Lines 281-2: "Stimulations that did not fully coincide with a seizure were considered as nuisance regressors in the second level analysis." What is meant by nuisance regressor?

      Reference to SPM 12 manual was given for technical terms referring to analysis software.

      Lines 283-8: "Motion periods were also included as multiple regressors (not convolved with a basis function) to be used as nuisance regressors. Stimulations that coincided with a motion above 0.3% of the voxel size were not considered stimulation inputs. Stimulation and seizure inputs were convolved with "3 gamma distribution basis functions" (i.e. 3rd 285 order gamma) in SPM (option: basis functions, gamma functions, order: 3), to account for temporal and dispersion variations in the hemodynamic response. The choice of 3rd order gamma was based on the expectation that time-to peak and shape of HRFs of seizure could vary across voxels (David et al. 2008)." Please explain the technical terms.

      Reference for SPM 12 manual was given for technical terms referring to analysis software, and HRF was defined.

      "BAMS rat connectome" - Please explain the technical terms.

      Modified as:

      “…connection matrix of the rat nervous system (BAMS rat connectome, Bota, Dong, and Swanson 2012).”

      Results

      After removing problematic animals and sessions, was there sufficient power? There probably wasn't enough to determine sex differences.

      After removing problematic sessions, we found statistically significant results (multiple comparison corrected) results in both activation maps, and hemodynamic responses. To determine sex differences, there were not enough animals for statistical findings (p>0.05).

      Figure 2 - I don't understand "tSNR" here. What is the point here?

      B vs C. Are these different brain areas or the same but SNR was adjusted?

      D. Where is FD explained? I think explaining what the parts of the figure show would be helpful.

      tSNR, the temporal signal-to-noise ratio, demonstrates the behavior of noise through time. Readers who are planning to mimic the used awake fMRI protocol together with the single loop coil, might be interested on data quality aspect, and ability for the coil to capture signal from noise, as it is one of the most important factors in fMRI designs where small signal changes have to be distinguished from the background noise.

      B and C illustrate the same brain area, but B was acquired with high resolution anatomical scanning (T1 FLASH), and C was acquired with low resolution ZTE scanning. We clarified the figure legend to the following:

      “…spatial signal-to-noise ratios of an illustrative high resolution anatomical T1-FLASH (B), and low resolution ZTE image (C)

      FD was explained in section 2.5.1. Some parts of the explanation were clarified: “Framewise displacement (FD) (Figure 2E) was calculated as follows. First, the differential of successive motion parameters (x, y, z translation, roll, pitch, yaw rotation) was calculated. Then absolute value was taken from each parameter and rotational parameters were divided by 5 mm (as estimate of the rat brain radius) to convert degrees to millimeters (Power et al. 2012). Lastly, all the parameters were summed together.”

      Table 1 has no statistical comparisons.

      Table 1 is purely an illustration of stimulation and seizure occurrence. There is no specific interest to compare stimulation types (in what state of seizure it occurred) as it does not provide any meaningful inferences to the study.

      Statistical activation maps - it is not clear how this was done.

      Creation of statistical maps are explained in section 2.5.3.

      Line 384-5: "In addition, some responses were observed in the somatosensory cortex during a seizure state, probably due to incomplete nuisance removal of the effect of the seizure itself by the linear model used." I don't see why the authors would not suggest that the result is logical given that stimuli should activate the somatosensory cortex.

      Sentence was modified as the following:

      “In addition, responses were observed in the somatosensory cortex during a seizure state”

      Fig 3 "F-contrast maps." Please explain.

      Creation of statistical maps are explained in section 2.5.3.

      HRF- please define. The ROI selection is unclear - it "was based on statistical differences seen in activation maps." But how were ROIs drawn? Also, why were HRFs examined at the end of seizures?

      HRF was defined, and definitions of HRF and ROI were moved from results section 3.3. to method section 2.5.3.

      Definition of ROI was clarified:

      “Anatomical ROIs, based on Paxinos atlas (Paxinos and Watson rat brain atlas 7th edition), were drawn on the brain areas where statistical differences were seen in activation maps.”

      HRFs were estimated additionally at the end of seizure as it was specifically interesting to study brain state shifts from ictal to interictal. This shift was also providing us statistically significant findings in means that brain responses differed from ictal stimulation.

      Line 421: "Interestingly, the response amplitude was higher when the stimulation ended a seizure compared to when it did not" Why is this interesting?

      Word “interestingly” was changed to “additionally” to avoid any inferences in the results section.

      Line 427: "Notably, HRFs amplitudes were both negatively and positively signed during the ictal 427 state, depending on the brain region." Why is this notable?

      Word “notably” was removed to avoid any inferences in the results section.

      Please explain the legends of Figures 4 and 6 more clearly.

      Figure 4, and figure 4 – figure supplement 1, legends were clarified:

      “HRFs was calculated in selected ROI, belonging to visual or somatosensory area, by multiplying gamma basis functions (Figure 1–figure supplement 1, B) with their corresponding average beta values over a ROI and taking a sum of these values.”

      Using the comments above as a guide, please revise the Discussion to be more precise and more clear about what was shown and what can be concluded in light of limitations. Please ensure the literature is cited where appropriate.

      Some parts of the discussion and conclusion sections were modified.

      Reviewer #3 (Recommendations For The Authors):

      Minor comments:

      Formatting: fMRI maps in Figures 3 and 5 should be more clearly labeled, indicating anterior and posterior directions on all images, and the cross sections should be enlarged to enable anatomical areas to be more clearly differentiated.

      Anterior and posterior directions were added, and cross sections were enlarged.

      The Methods section 2.41 and other places in the text, and Figure 2 - Figure Supplement 1 say that there was less artifact on the EEG with ZTA than with GE-EPI. However the EEG shown in Figure 2 - Figure Supplement 1 Part C shows much more artifact in the left (ZTE) trace than the right (GE-EPI) trace. This apparent contradiction should be resolved.

      The figure was actually demonstrating the relative change to the signal when MRI sequences were on, and by this standard, the ZTE produced both less amplitude and frequency changes than EPI. In the example figure, the baseline fluctuations in the EEG trace in the left were higher in amplitude than in the right, and this could potentially lead to misconception of ZTE producing more noise. Figure legend was clarified to highlight relative change:

      “ZTE also caused relatively less artificial noise on EEG signal, keeping both amplitude of the signal and frequencies relatively more intact, which improved live detection of absence seizures.”

      Figure 2 - Supplement 1, part B horizontal axis should provide units.

      Units were added.

      Figure 2 - Supplement 1, legend last sentence says arrows mark the beginning of each "sequence." Is this a typo and should this instead say "each seizure"?

      Should state “each fMRI sequence” which was corrected.

      Line 307, Methods "to reveal brain areas where ictal stimulation provided higher amplitude response than interictal" - should this be reversed, ie weren't the authors analyzing a contrast to determine where interictal signals were higher than ictal signals?

      This should be reversed, and was corrected, thank you for noting this.

      Figure 6 - Figure Supplement 1, the scales are very different for many of the plots so they are hard to compare. Especially in the ictal periods (D, E, F) it is hard to see if any changes are happening during ictal stimulation similar to interictal stimulation due to very different scales. The activity related to SWD is so large that it overshadows the rest and perhaps should be subtracted out.

      We point out that Figure 6 - Figure Supplement 1 reproduces with a higher level of detail the results shown of Figure 6 from the main text, where all signals are plotted in the same scale. The difference between scales used in this figure is intended, and its purpose is to show and highlight the large differences observed on the ongoing activity and the evoked response between the two states (ictal and interictal). In interictal periods the ongoing activity is characterized by fluctuations around a baseline level whose variance is highly affected by the application of the stimulus. On the contrary, ictal periods are characterized by large oscillations, with periods of high and synchronized activity followed by periods of nearly no activity, where the effect of the stimulus on the dynamics is overshadowed by the ongoing dynamics (both from local and from afferent nodes) as the referee mentions, and which imposes a strong limit to the responsiveness of the system and the propagation of the signal.

    1. Author Response

      The following is the authors’ response to the original reviews.

      Reviewer #1 (Public Review):

      Assessment:

      The manuscript titled 'Rab7 dependent regulation of goblet cell protein CLCA1 modulates gastrointestinal 1 homeostasis' by Gaur et al discusses the role of Rab7 in the development of ulcerative colitis by regulating the lysosomal degradation of Clca1, a mucin protease. The manuscript presents interesting data and provides a potential molecular mechanism for the pathological alterations observed in ulcerative colitis. Gaur et al demonstrate that Rab7 levels are lowered in UC and CD. However, a similar analysis of Rab7 levels in ulcerative colitis (UC) and Crohn's disease (CD) patient samples was conducted recently (Du et al, Dev Cell, 2020) which showed that Rab7 levels are found to be elevated under these conditions. While Gaur et al have briefly mentioned Du et al's paper in passing in the discussion, they need to discuss these contradictory results in their paper and clarify these differences. Additionally, Du et al are not included in the list of references.

      Strengths:

      The manuscript used a multi-pronged approach and compares patient samples, mouse models of DSS, and protocols that allow differentiation of goblet cells. They also use a nanogel-based delivery system for siRNAs, which is ideal for the knockdown of specific genes in the gut.

      Weaknesses:

      (1) Du et al, Dev Cell 2020 (https://doi.org/10.1016/j.devcel.2020.03.002) have previously shown that Rab7 levels are elevated in a similar set of colonic samples (age group, number etc.) from UC and CD patients. Gaur et al have not discussed this paper or its findings in detail, which directly contradicts their results. Clarification regarding this should be provided.

      We thank and appreciate the reviewer for bringing this point.

      The results shown by Du et al, Dev Cell, 2020 depict elevated expression of Rab7 in UC and CD patients compared to controls. In first occurrence, these results appear contradictory, but there may be a few possible explanations for this.

      Firstly, Rab7 expression levels may fluctuate in the tissue depending on the degree of the gut inflammation. This can be concluded from our observations in DSS-mice dynamics model and the human patient samples with mild and moderate UC. Furthermore, Du et al provide no information of the severity of the condition among the patients employed in the study. Our motive, in the current work, was to emphasize this aspect. This point was mentioned in the discussion section of the manuscript. However, in view of the reviewer’s concern, we have now added a detailed comment on this in the main text of the revised version of the manuscript.

      Secondly, the control biopsies in our investigation were acquired from non-IBD patients, and not what was done by Du et al., wherein biopsies from the normal para-carcinoma region of the colorectal cancer patients were used. One cannot overlook the fact that physiological and molecular changes are apparent even in non-inflamed regions in the gut of an IBD or CRC patient. It is possible that the observed discrepancy arises due to the differences in the sample type used for comparing the Rab7 expression.

      Finally, the main sub-tissue region showing a decrease in Rab7 expression in UC samples, appeared to be the Goblet cells which was not covered by Du et al.

      Keeping these points in mind we do not think that there is a contradiction in our findings with that of Du et al., 2020. In the revised submission some of these explanations are incorporated (Lines 106-109).

      This was an oversight from our side. We have actually mentioned Du et al., 2020 in the discussion (line number 345) but somehow the reference was missing in the main list. We have ensured that the reference is included in the revised version and that their findings are included both in main text and in the discussion.

      Reviewer #2 (Public Review):

      Summary:

      In this work, the authors report a role for the well-studied GTPase Rab7 in gut homeostasis. The study combines cell culture experiments with mouse models and human ulcerative colitis patient tissues to propose a model where, Rab7 by delivering a key mucous component CLCA1 to lysosomes, regulates its secretion in the goblet cells. This is important for the maintenance of mucous permeability and gut microbiota composition. In the absence of Rab7, CLCA1 protein levels are higher in tissues as well as the mucus layer, corroborating with the anticorrelation of Rab7 (reduced) and CLCA1 (increased) from ulcerative colitis patients. The authors conclude that Rab7 maintains CLCA1 level by controlling its lysosomal degradation, thereby playing a vital role in mucous composition, colon integrity, and gut homeostasis.

      Strengths:

      The biggest strength of this manuscript is the combination of cell culture, mouse model, and human tissues. The experiments are largely well done and, in most cases, the results support their conclusions. The authors go to substantial lengths to find a link, such as alteration in microbiota, or mucus proteomics.

      Weaknesses:

      (1) There are also some weaknesses that need to be addressed. The association of Rab7 with UC in both mice and humans is clear, however, claims on the underlying mechanisms are less clear. Does Rab7 regulate specifically CLCA1 delivery to lysosomes, or is it an outcome of a generic trafficking defect?

      We thank the reviewer for the insightful comment. We would like to bring forth the following explanation for each these concerns:

      Our immunofluorescence imaging experiments revealed co-localization of Rab7 protein with CLCA1 and the lysosomes (Fig 7I). In addition, the absence of Rab7 affects the transport of CLCA1 to lysosomes (Fig 7J). This demonstrates that Rab7 may be involved in regulation of CLCA1 transport (presumably along with other cargo), to lysosomes selectively. However, we do recognize that the point raised by the reviewer about possible effect of a generic trafficking defect is valid.

      (2) CLCA1 is a secretory protein, how does it get routed to lysosomes, i.e., through Golgi-derived vesicles, or by endocytosis of mucous components? Mechanistic details on how CLCA1 is routed to lysosomes will add substantial value.

      As mentioned in the manuscript, the trafficking of CLCA1 protein or CLCA1-containing vesicles within the goblet cell is unknown, with no information on the proteins involved in its mobility. The switching of CLCA1 containing vesicles from the secretory route to lysosomes needs extensive investigation involving overall trafficking of the protein. Taken together, the complete answer to both these important questions will need a series of experiments and those may be interesting avenues for future research.

      (3) Why does the level of Rab7 fluctuate during DSS treatment (Fig 1B)?

      This is a very thoughtful point from the reviewer. We detected a distinct pattern of Rab7 expression fluctuation in intestinal epithelial cells after DSS-dynamics treatment in mice. Perhaps, these changes are the result of complex cellular signaling in response to the DSS treatment. Rab7, being a fundamental protein involved in protein sorting pathway, is expected to undergo alteration based on cells requirement. Presently there are no reports suggesting the regulatory mechanisms that govern Rab7 levels in the gut.

      (4) Does the reduction seen in Rab7 levels (by WB) also reflect in reduced Rab7 endosome numbers?

      We observed reduction in Rab7 expression both at RNA and protein levels. To confirm whether this alteration will lead to reduced Rab7 positive endosome numbers may require detailed investigations.

      (5) Are other late endosomal (and lysosomal) populations also reduced upon DSS treatment and UC? Is there a general defect in lysosomal function?

      There are no direct evidences showing reduction in the late endosomal and lysosomal population during gut inflammation, but few studies link lysosomal dysfunction with risk for colitis (doi: 10.1016/j.immuni.2016.05.007).

      (6) The evidence for lysosomal delivery of CLCA1 (Fig 7 I, J) is weak. Although used sometimes in combination with antibodies, lysotracker red is not well compatible with permeabilization and immunofluorescence staining. The authors can substantiate this result further using lysosomal antibodies such as Lamp1 and Lamp2. For Fig 7J, it will be good to see a reduction in Rab7 levels upon KD in the same cell.

      We used Lysotracker red in live cells followed by fixation. So, permeabilization issues were resolved. Lamp1, as suggested by the reviewer, is definitely a better marker for lysosomes in immunofluorescence studies, but is also shown to mark late endosomes (doi: 10.1083/jcb.132.4.565). As Rab7 protein also marks the late endosomes, using Lamp1 may leave the ambiguity of CLCA1 in Rab7 positive late endosomes versus lysosomes. Nevertheless, we have carried out this experiment, as suggested by the reviewer, by staining the cells with LAMP1 (author response image 1). As demonstrated in our previous data, the colocalization of CLCA1 with LAMP1 positive vesicles decreased upon Rab7 knockdown. Also, we observed a decrease in the intensity of LAMP1 staining in cells with Rab7 knockdown. Additionally, we noted a reduction in the LAMP1 staining intensity in cells where Rab7 was knocked down. This observation can be attributed to the decrease in the presence of Rab7-positive vesicles or late endosomes which also exhibit LAMP1 staining.

      Author response image 1.

      (A) Representative confocal images of HT29-MTX-E12 cells transfected with either scrambled siRNA (control) or Rab7 siRNA (Rab7Knockdown). Cells are stained with CLCA1 (green) using antiCLCA1 antibody and lysosomes with LAMP1. (B) Graph shows quantitation of colocalization between CLCA1 and LAMP1 from images (n=20) using Mander’s overlap coefficient. Inset shows zoomed areas of the image with colocalization puncta (yellow) marked with arrows.

      (7) In this connection, Fig S3D is somewhat confusing. While it is clear that the pattern of Muc2 in WT and Rab7-/- cells are different, how this corroborates with the in vivo data on alterations in mucus layer permeability -- as claimed -- is not clear.

      The data in Fig. S3D suggest the involvement of Rab7 in packaging of Muc2. The whole idea for doing this experiment was to support our observation in the Rab7KD-mice model where mucus layer was seen to be loose and more permeable in Rab7 deficient mice.

      (8) Overall, the work shows a role for a well-studied GTPase, Rab7, in gut homeostasis. This is an important finding and could provide scope and testable hypotheses for future studies aimed at understanding in detail the mechanisms involved.

      We thank the reviewer for this comment.

      Recommendations for the authors:

      Reviewer #1 (Recommendations For The Authors):

      Specific questions to the authors:

      (1) Why is the dotted line in Fig. 1c at -7.5? What does this signify?

      Response: The dotted line was intended to represent the baseline; in the revised manuscript it is corrected and placed at y=0.

      (2) Du et al should be cited. Fig 6 K-Q from Du et al should be discussed and reasons for contradictory findings should be given in greater detail, rather than a single sentence in the discussion.

      Response: The reference for Du et al is included in the list and the possible reasons the findings of the current work are discussed in the main text (Line 106-109).

      (3) Fig1. Why are Rab7 levels low even in remission patient samples? Can DSS be withdrawn to induce remission followed by analysis of colonic samples?

      Response: A possible explanation for this observation could be that the restoration of Rab7 levels may not immediately follow the resolution of clinical symptoms in remission patients. After the remission initiation, the normalization of cellular processes, including the regulation of Rab7 expression, might exhibit a time lag. A thorough investigation of Rab7 levels and the allied pathways at different time points during the remission phase could provide deeper insights into the gradual dynamics of recovery. As suggested by the reviewer, DSS withdrawal induced recovery model can be utilized for understanding the same and could be a good approach for future investigations.

      (4) Fig. 2: Single-channel fluorescence should be shown.

      Response: The single channel fluorescence images are incorporated in Fig. S2.

      (5) Line 456 should be modified. 'Blind pathologist' does not read well!

      Response: The line has been modified with ‘Blinded pathologist’.

      (6) Other inflammatory markers, cytokine levels should be looked at in addition to TNF alpha.

      Response: TNF-α is a crucial mediator in intestinal inflammation, actively contributing to the development of IBD. Elevated levels of TNF-α are observed in patients of IBD (Billmeier U. et al, World J Gastroenterol. 2016). In the current work, while probing for TNF-α our primary objective was to examine this significant indicator of colitis following Rab7 knockdown in mice, aiming to gain insights into heightened gut inflammation.

      (7) Quantitation of S3D should be provided.

      Response: The dispersed expression of Muc2 was observed in n=20 cells per sample and it was a qualitative observation. The aim was to identify any changes in Muc2 packaging under Rab7 knockout conditions.

      (8) Microbiota analysis should include Rab7KD+DSS mice.

      Response: We understand the importance of this point, however, in the current work our primary objective was to specifically investigate changes in microbial diversity and abundance in Rab7KD mice compared to both DSS+CScr and CScr mice. Rab7KD+DSS mice is expected to show higher dysbiosis in comparison to DSS+CScr.

      (9) Fig 6 H and I, G. How do Clca1 levels reduce in Rab7kd +DSS relative to Scr+DSS while they are higher in Rab7kd compared to Scr. Comment.

      Response: The decreased expression of CLCA1 in the mucus of DSS+Rab7KD mice can be attributed to a consequence of significant reduction in goblet cell numbers in these mice, as evidenced by the observed loss of these cells (Fig.S3 B and Fig. S3C). CLCA1 is exclusively secreted by goblet cells, so a decline in their numbers directly affects CLCA1 levels.

      (10) How are Rab7 levels downregulated? What is the predicted mechanism?

      Response: While our current study didn't explore this aspect, it's worth noting that Rab7 protein levels undergo regulation through various mechanisms, including post-translational modifications such as Ubiquitination and SUMOylation. These modifications are known to regulate Rab7 stability, transport and recycling. Specific experiments conducted during this study (work not included in the manuscript) indicated the participation of SENP7, a deSUMOylase, in controlling the stability of Rab7 protein, particularly in the context of colitis. Additionally, goblet cell specific mechanisms are also likely to be controlling the Rab7 in the gut.

      (11) What is the explanation for opposite changes in CLCa1 RNA (down) and protein (up).

      Response: The reduction in CLCA1 at the RNA level could be associated with the decrease in goblet cell numbers during colitis. Our investigation indicates that Rab7 predominantly influences CLCA1 at the protein level by impacting its degradation pathway. It is important to acknowledge that not all the alterations in CLCA1 observed during colitis can be solely attributed to Rab7, but our study has identified a connection between Rab7 and CLCA1.

      (12) In light of Du et al, it would be interesting to see how the number of peroxisomes changes upon alteration of Rab7 levels.

      Response: The suggestion by the reviewer is noteworthy. Since, being an altogether different domain, it deviates from the primary objectives of current work. Here, our goal was specifically on exploring the role of Rab7 in goblet cell functioning. Thus is an attractive theme for future investigations.

      (13) While Gaur et al suggest in their discussion that Du et al may have observed an upregulation in Rab7 levels in different cell types of the intestine, this is not apparent from the data provided. Tissue sections should be carefully analysed to provide data supporting this observation. Differences in reagents used (antibodies) should also be considered. As far as the human patient data is concerned, it does not appear that the sample stages are very different across the two manuscripts (based on age, inclusion criteria etc.).

      Response: This has been explained in detail in our public comments.

      Reviewer #2 (Recommendations For The Authors):

      (1) In general, image-based measurements could be done better (for example, object-based statistics than pixel-based overlaps) and represented differently. It is difficult to appreciate the reduction in Rab7 levels in goblet cells in Fig 2 A, C. It might be good to show the channels separately, and perhaps use an intensity gradient LUT for the Rab7 channel.

      Response: The single channel fluorescence images are incorporated in Fig. S2.

      (2) The EM images, and particularly Fig 2F are not convincing, with an oddly square-shaped vesicle. I'm not sure what value they are adding to the interpretation.

      Response: The observed square-shaped vesicle in Fig. 2F could be attributed to the dynamic nature of vesicles within a cell. This dynamicity allows them to adopt various shapes depending on their state and function within the cell. The presence of Rab7 near vacuoles of goblet cells signify its probable involvement in the regulation of secretory function of these cells which is the key aspect being covered in this work.

      (3) A general method question concerns the definition of the distal colon. How is this decided, particularly when colon lengths are reduced upon DSS treatment?

      Response: The murine colon is divided into proximal and distal colon of mouse and has a visual difference of inner folds which are quite prominent in proximal colon. Additionally, the portion towards the rectum (predominantly distal colon) was majorly utilized for the experiments. In each case the various experimental groups were matched for the respective areas.

      (4) The use of an in vivo intestine-specific Rab7 silencing model is good. Why does Rab7 KD itself not capitulate aspects of DSS treatment, rather it seems to exacerbate it.

      Response: Our objective was to determine whether the downregulation of Rab7 during colitis was the cause or consequence of gut inflammation. Interestingly, our investigation using the murine Rab7 knockdown model revealed that the reduction of Rab7 expression in the intestine exacerbates inflammation. Subsequent analysis demonstrated that the absence of Rab7 disrupts goblet cell secretory function, consequently contributing to heightened inflammation. Our findings overall suggest that Rab7 downregulation is not merely a consequence but plays a contributory role in aggravating inflammation in the context of colitis.

      (5) The axes labels in Fig 5 are not readable. It is unclear how Rab7 KD is more similar in gut microbiota phenotypes to DSS than to CScr.

      Response: The microbial analysis revealed an abnormal composition of gut microbiota in Rab7KD mice compared to CScr. Interestingly, this composition exhibited some similarity to the inflamed gut microbiota observed in DSSScr mice. The analysis further demonstrated a shift in microbial diversity in Rab7KD mice, showcasing characteristics akin to those observed in inflamed mice. This similarity in gut microbiota phenotypes between Rab7KD and DSSScr suggests a potential link or influence of Rab7 downregulation on the microbiota, contributing to the observed similarities with DSS-induced inflammation.

      (6) The use of mucous proteomics to identify mechanisms of Rab7-mediated phenotype is a good approach. The replicates in the proteomics dataset (Fig 6F) do not seem to match. Detailing of methodology used for analysis will help to overcome these doubts.

      Response: The identified proteins in different samples of mucus proteomics were subjected to label free quantification. Subsequently, the significantly altered proteins were subjected to analysis with the False Discovery Rate (FDR) to control for potential false positives and ascertain the validity of the findings.

      (7) It will be good to see the immunoblots showing the negative correlation between Rab7 and CLCL1 in Fig 7D.

      Response: Fig. 7C shows western blot for protein expression of CLCA1of the same control and UC samples which were used in Fig. 1F to show Rab7 expression. Fig. 7D is the quantitative correlation plot for Fig. 1F (Rab7 expression) and Fig. 7C (CLCA1 expression).

      (8) Why is UC different from the DSS model for Rab7 gene expression but not protein levels? Endosomal counts could help address this.

      Response: We encountered challenges in accurately counting the individual puncta of Rab7 expression in immunofluorescence images due to the nature of tissue samples. Locating endosomes within a single cell proved to be challenging, and the proximity of many puncta made it difficult to delineate them individually. Despite these technical difficulties, the intriguing prospect of correlating Rab7 expression with endosomal counts remains a compelling aspect that may well be area for future investigations.

    1. Author response:

      (1) General Statements

      As you will see in our attached rebuttal to the reviewers, we have added several new experiments and revised manuscript to fully address their concerns.

      (2) Point-by-point description of the revisions

      Reviewer #1:

      Evidence, reproducibility and clarity

      Summary:

      The manuscript by Yang et al. describes a new CME accessory protein. CCDC32 has been previously suggested to interact with AP2 and in the present work the authors confirm this interaction and show that it is a bona fide CME regulator. In agreement with its interaction with AP2, CCDC32 recruitment to CCPs mirrors the accumulation of clathrin. Knockdown of CCDC32 reduces the amount of productive CCPs, suggestive of a stabilisation role in early clathrin assemblies. Immunoprecipitation experiments mapped the interaction of CCDC42 to the α-appendage of the AP2 complex α-subunit. Finally, the authors show that the CCDC32 nonsense mutations found in patients with cardio-facial-neuro-developmental syndrome disrupt the interaction of this protein to the AP2 complex. The manuscript is well written and the conclusions regarding the role of CCDC32 in CME are supported by good quality data. As detailed below, a few improvements/clarifications are needed to reinforce some of the conclusions, especially the ones regarding CFNDS.

      We thank the referee for their positive comments. In light of a recently published paper describing CCDC32 as a co-chaperone required for AP2 assembly (Wan et al., PNAS, 2024, see reviewer 2), we have added several additional experiments to address all concerns and consequently gained further insight into CCDC32-AP2 interactions and the important dual role of CCDC32 in regulating CME. 

      Major comments:

      (1) Why did the protein could just be visualized at CCPs after knockdown of the endogenous protein? This is highly unusual, especially on stable cell lines. Could this be that the tag is interfering with the expressed protein function rendering it incapable of outcompeting the endogenous? Does this points to a regulated recruitment?

      The reviewer is correct, this would be unusual; however, it is not the case. We misspoke in the text (although the figure legend was correct) these experiments were performed without siRNA knockdown and we can indeed detect eGFP-CCDC32 being recruited to CCPs in the presence of endogenous protein. Nonetheless, we repeated the experiment to be certain (see Author response image 1).  

      Author response image 1.

      Cohort-averaged fluorescence intensity traces of CCPs (marked with mRuby-CLCa) and CCP-enriched eGFPCCDC32(FL).

      (2) The disease mutation used in the paper does not correspond to the truncation found in patients. The authors use an 1-54 truncation, but the patients described in Harel et al. have frame shifts at the positions 19 (Thr19Tyrfs*12) and 64 (Glu64Glyfs*12), while the patient described in Abdalla et al. have the deletion of two introns, leading to a frameshift around amino acid 90. Moreover, to be precisely test the function of these disease mutations, one would need to add the extra amino acids generated by the frame shift. For example, as denoted in the mutation description in Harel et al., the frameshift at position 19 changes the Threonine 19 to a Tyrosine and ads a run of 12 extra amino acids (Thr19Tyrfs*12).

      The label of the disease mutant p.(Thr19Tyrfs12) and p.(Glu64Glyfs12) is based on a 194aa polypeptide version of CCDC32 initiated at a nonconventional start site that contains a 9 aa peptide (VRGSCLRFQ) upstream of the N-terminus we show. Thus, we are indeed using the appropriate mutation site (see: https://www.uniprot.org/uniprotkb/Q9BV29/entry). The reviewer is correct that we have not included the extra 12 aa in our construct; however as these residues are not present in the other CFNDS mutants, we think it unlikely that they contribute to the disease phenotype.  Rather, as neither of the clinically observed mutations contain the 78-98 aa sequence required for AP2 binding and CME function, we are confident that this defect contributed to the disease. Thus, we are including the data on the CCDC32(1-54) mutant, as we believe these results provide a valuable physiological context to our studies. 

      (3) The frameshift caused by the CFNDS mutations (especially the one studied) will likely lead to nonsense mediated RNA decay (NMD). The frameshift is well within the rules where NMD generally kicks in. Therefore, I am unsure about the functional insights of expressing a diseaserelated protein which is likely not present in patients.

      We thank the reviewer for bringing up this concern. However, as shown in new Figure S1, the mutant protein is expressed at comparable levels as the WT, suggesting that NMD is not occurring.

      (4) Coiled coils generally form stable dimers. The typically hydrophobic core of these structures is not suitable for transient interactions. This complicates the interpretation of the results regarding the role of this region as the place where the interaction to AP2 occurs. If the coiled coil holds a stable CCDC32 dimer, disrupting this dimer could reduce the affinity to AP2 (by reduced avidity) to the actual binding site. A construct with an orthogonal dimeriser or a pulldown of the delta78-98 protein with of the GST AP2a-AD could be a good way to sort this issue.

      We were unable to model a stable dimer (or other oligomer) of this protein with high confidence using Alphafold 3.0. Moreover, we were unable to detect endogenous CCDC32 coimmunoprecipitating with eGFP-CCDC32 (Fig. S6C). Thus, we believe that the moniker, based solely on the alpha-helical content of the protein is a misnomer.  We have explained this in the main text.

      Minor comments:

      (1) The authors interchangeably use the term "flat CCPs" and "flat clathrin lattices". While these are indeed related, flat clathrin lattices have been also used to refer to "clathrin plaques". To avoid confusion, I suggest sticking to the term "flat CCPs" to refer to the CCPs which are in their early stages of maturation.

      Agreed. Thank you for the suggestion. We have renamed these structures flat clathrin assemblies, as they do not acquire the curvature needed to classify them as pits, and do not grow to the size that would classify then as plaques. 

      Significance

      General assessment:

      CME drives the internalisation of hundreds of receptors and surface proteins in practically all tissues, making it an essential process for various physiological processes. This versatility comes at the cost of a large number of molecular players and regulators. To understand this complexity, unravelling all the components of this process is vital. The manuscript by Yang et al. gives an important contribution to this effort as it describes a new CME regulator, CCDC32, which acts directly at the main CME adaptor AP2. The link to disease is interesting, but the authors need to refine their experiments. The requirement for endogenous knockdown for recruitment of the tagged CCDC32 is unusual and requires further exploration.

      Advance:

      The increased frequency of abortive events presented by CCDC32 knockdown cells is very interesting, as it hints to an active mechanism that regulates the stabilisation and growth of clathrin coated pits. The exact way clathrin coated pits are stabilised is still an open question in the field.

      Audience:

      This is a basic research manuscript. However, given the essential role of CME in physiology and the growing number of CME players involved in disease, this manuscript can reach broader audiences.

      We thank the referee for recognizing the ‘interesting’ advances our studies have made and for considering these studies as ‘an important contribution’ to ‘an essential process for various physiological processes’ and able ‘to reach broader audiences’. We have addressed and reconciled the reviewer’s concerns in our revised manuscript. 

      Field of expertise of the reviewer:

      Clathrin mediated endocytosis, cell biology, microscopy, biochemistry.

      Reviewer #2:

      Evidence, reproducibility and clarity

      In this manuscript, the authors demonstrate that CCDC32 regulates clathrin-mediated endocytosis (CME). Some of the findings are consistent with a recent report by Wan et al. (2024 PNAS), such as the observation that CCDC32 depletion reduces transferrin uptake and diminishes the formation of clathrin-coated pits. The primary function of CCDC32 is to regulate AP2 assembly, and its depletion leads to AP2 degradation. However, this study did not examine AP2 expression levels. CCDC32 may bind to the appendage domain of AP2 alpha, but it also binds to the core domain of AP2 alpha.

      We thank the reviewer for drawing our attention to the Wan et al. paper, that appeared while this work was under review.  However, our in vivo data are not fully consistent with the report from Wan et al. The discrepancies reveal a dual function of CCDC32 in CME that was masked by complete knockout vs siRNA knockdown of the protein, and also likely affected by the position of the GFP-tag (C- vs N-terminal) on this small protein. Thus:

      -  Contrary to Wan et al., we do not detect any loss of AP2 expression (see new Figure S3A-B) upon siRNA knockdown. Most likely the ~40% residual CCDC32 present after siRNA knockdown is sufficient to fulfill its catalytic chaperone function but not its structural role in regulating CME beyond the AP2 assembly step.  

      - Contrary to Wan et al., we have shown that CCDC32 indeed interacts with intact AP2 complex (Figure S3C and 6B,C) showing that all 4 subunits of the AP2 complex co-IP with full length eGFP-CCDC32. Interestingly, whereas the full length CCDC32 pulls down the intact AP2 complex, co-IP of the ∆78-98 mutant retains its ability to pull down the β2-µ2 hemicomplex, its interactions with α:σ2 are severely reduced.  While this result is consistent with the report of Wan et al that CCDC32 binds to the α:σ2 hemi-complex, it also suggests that the interactions between CCDC32 and AP2 are more complex and will require further studies.

      - Contrary to Wan et al., we provide strong evidence that CCDC32 is recruited to CCPs. Interestingly, modeling with AlphaFold 3.0 identifies a highly probably interaction between alpha helices encoded by residues 66-91 on CCDC32 and residues 418-438 on α. The latter are masked by µ2-C in the closed confirmation of the AP2 core, but exposed in the open confirmation triggered by cargo binding, suggesting that CCDC32 might only bind to membrane-bound AP2.

      Thus, our findings are indeed novel and indicate striking multifunctional roles for CCDC32 in CME, making the protein well worth further study. 

      (1) Besides its role in AP2 assembly, CCDC32 may potentially have another function on the membrane. However, there is no direct evidence showing that CCDC32 associates with the plasma membrane.

      We disagree, our data clearly shows that CCDC32 is recruited to CCPs (Fig. 1B) and that CCPs that fail to recruit CCDC32 are short-lived and likely abortive (Fig. 1C). Wan et al. did not observe any colocalization of C-terminally tagged CCDC32 to CCPs, whereas we detect recruitment of our N-terminally tagged construct, which we also show is functional (Fig. 6F).  Further, we have demonstrated the importance of the C-terminal region of CCDC32 in membrane association (see new Fig. S7).  Thus, we speculate that a C-terminally tagged CCDC32 might not be fully functional. Indeed, SIM images of the C-terminally-tagged CCDC32 in Wan et al., show large (~100 nm) structures in the cytosol, which may reflect aggregation. 

      (2) CCDC32 binds to multiple regions on AP2, including the core domain. It is important to distinguish the functional roles of these different binding sites.

      We have localized the AP2-ear binding region to residues 78-99 and shown these to be critical for the functions we have identified. As described above we now include data that are complementary to those of Wan et al. However, our data also clearly points to additional binding modalities. We agree that it will be important and map these additional interactions and identify their functional roles, but this is beyond the scope of this paper.  

      (3) AP2 expression levels should be examined in CCDC32 depleted cells. If AP2 is gone, it is not surprising that clathrin-coated pits are defective.

      Agreed and we have confirmed this by western blotting (Figure S3A-B) and detect no reduction in levels of any of the AP2 subunits in CCDC32 siRNA knockdown cells. As stated above this could be due to residual CCDC32 present in the siRNA KD vs the CRISPR-mediated gene KO.

      (4) If the authors aim to establish a secondary function for CCDC32, they need to thoroughly discuss the known chaperone function of CCDC32 and consider whether and how CCDC32 regulates a downstream step in CME.

      Agreed. We have described the Wan et al paper, which came out while our manuscript was in review, in our Introduction.  As described above, there are areas of agreement and of discrepancies, which are thoroughly documented and discussed throughout the revised manuscript.  

      (5) The quality of Figure 1A is very low, making it difficult to assess the localization and quantify the data.

      The low signal:noise in Fig. 1A the reviewer is concerned about is due to a diffuse distribution of CCDC32 on the inner surface of the plasma membrane. We now, more explicitly describe this binding, which we believe reflects a specific interaction mediated by the C-terminus of CCDC32; thus the degree of diffuse membrane binding we observe follows: eGFP-CCDC32(FL)> eGFPCCDC32(∆78-98)>eGFP-CCDC32(1-54)~eGFP/background (see new Fig. S7). Importantly, the colocalization of CCDC32 at CCPs is confirmed by the dynamic imaging of CCPs (Fig 1B).

      (6) In Figure 6, why aren't AP2 mu and sigma subunits shown?

      Agreed. Not being aware of CCDC32’s possible dual role as a chaperone, we had assumed that the AP2 complex was intact.  We have now added this data in Figure 6 B,C and Fig. S3C, as discussed above. 

      Page 5, top, this sentence is confusing: "their surface area (~17 x 10 nm<sup>2</sup>) remains significantly less than that required for the average 100 nm diameter CCV (~3.2 x 103 nm<sup>2</sup>)."

      Thank you for the criticism. We have clarified the sentence and corrected a typo, which would definitely be confusing.  The section now reads,  “While the flat CCSs we detected in CCDC32 knockdown cells were significantly larger than in control cells (Fig. 4D, mean diameter of 147 nm vs. 127 nm, respectively), they are much smaller than typical long-lived flat clathrin lattices (d≥300 nm)(Grove et al., 2014). Indeed, the surface area of the flat CCSs that accumulate in CCDC32 KD cells (mean ~1.69 x 10<sup>4</sup> nm<sup>2</sup>) remains significantly less than the surface area of an average 100 nm diameter CCV (~3.14 x 10<sup>4</sup> nm<sup>2</sup>). Thus, we refer to these structures as ‘flat clathrin assemblies’ because they are neither curved ‘pits’ nor large ‘lattices’. Rather, the flat clathrin assemblies represent early, likely defective, intermediates in CCP formation.” 

      Significance

      Overall, while this work presents some interesting ideas, it remains unclear whether CCDC32 regulates AP2 beyond the assembly step.

      Our responses above argue that we have indeed established that CCDC32 regulates AP2 beyond the assembly step. We have also identified several discrepancies between our findings and those reported by Wan et al., most notably binding between CCDC32 and mature AP2 complexes and the AP2-dependent recruitment of CCDC32 to CCPs.  It is possible that these discrepancies may be due to the position of the GFP tag (ours is N-terminal, theirs is C-terminal; we show that the N-terminal tagged CCDC32 rescues the knockdown phenotype, while Wan et al., do not provide evidence for functionality of the C-terminal construct). 

      Reviewer #3: 

      Evidence, reproducibility and clarity (Required): 

      In this manuscript, Yang et al. characterize the endocytic accessory protein CCDC32, which has implications in cardio-facio-neuro-developmental syndrome (CFNDS). The authors clearly demonstrate that the protein CCDC32 has a role in the early stages of endocytosis, mainly through the interaction with the major endocytic adaptor protein AP2, and they identify regions taking part in this recognition. Through live cell fluorescence imaging and electron microscopy of endocytic pits, the authors characterize the lifetimes of endocytic sites, the formation rate of endocytic sites and pits and the invagination depth, in addition to transferrin receptor (TfnR) uptake experiments. Binding between CCDC32 and CCDC32 mutants to the AP2 alpha appendage domain is assessed by pull down experiments. Together, these experiments allow deriving a phenotype of CCDC32 knock-down and CCDC32 mutants within endocytosis, which is a very robust system, in which defects are not so easily detected. A mutation of CCDC32, known to play a role in CFNDS, is also addressed in this study and shown to have endocytic defects.

      We thank the reviewer for their positive remarks regarding the quality of our data and the strength of our conclusions.  

      In summary, the authors present a strong combination of techniques, assessing the impact of CCDC32 in clathrin mediated endocytosis and its binding to AP2, whereby the following major and minor points remain to be addressed: 

      - The authors show that CCDC32 depletion leads to the formation of brighter and static clathrin coated structures (Figure 2), but that these were only prevalent to 7.8% and masked the 'normal' dynamic CCPs. At the same time, the authors show that the absence of CCDC32 induces pits with shorter life times (Figure 1 and Figure 2), the 'majority' of the pits.

      Clarification is needed as to how the authors arrive at these conclusions and these numbers. The authors should also provide (and visualize) the corresponding statistics. The same statement is made again later on in the manuscript, where the authors explain their electron microscopy data. Was the number derived from there? 

      These points are critical to understanding CCDC32's role in endocytosis and is key to understanding the model presented in Figure 8. The numbers of how many pits accumulate in flat lattices versus normal endocytosis progression and the actual time scales could be included in this model and would make the figure much stronger. 

      Thank you for these comments.  We understand the paradox between the visual impression and the reality of our dynamic measurements. We have been visually misled by this in previous work (Chen et al., 2020), which emphasizes the importance of unbiased image analysis afforded to us through the well-documented cmeAnalysis pipeline, developed by us (Aguet et al., 2013) and now used by many others (e.g. (He et al., 2020)). 

      The % of static structures was not derived from electron microscopy data, but quantified using cmeAnalysis, which automatedly provides the lifetime distribution of CCPs. We have now clarified this in the manuscript and added a histogram (Fig. S4) quantifying the fraction of CCPs in lifetime cohorts  <20s, 21-60s, 61-100s, 101-150s and >150s (static). 

      - In relation to the above point, the statistics of Figure 2E-G and the analysis leading there should also be explained in more detail: For example, what are the individual points in the plot (also in Figures 6G and 7G)? The authors should also use a few phrases to explain software they use, for example DASC, in the main text. 

      Each point in these bar graphs represents a movie, where n≥12. These details have been added to the respective figure legend. We have also added a brief description of DASC analysis in the text. 

      -  There are several questions related to the knock-down experiments that need to be addressed:

      Firstly, knock-down of CCDC32 does not seem to be very strong (Figure S2B). Can the level of knock-down be quantified? 

      We have now quantified the KD efficiency. It is ~60%. This turns out to be fortuitous (see responses to reviewer 2), as a recent publication, which came out after we completed our study, has shown by CRISPR-mediated knockout, that CCD32 also plays an essential chaperone function required for AP2 assembly.  We do not see any reduction in AP2 levels or its complex formation under our conditions (see new Supplemental Figure S3), which suggests that the effects of CCDC32 on CCP dynamics are more sensitive to CCDC32 concentration than its roles as a chaperone. Our phenotypes would have been masked by more efficient depletion of CCDC32.  

      In page 6 it is indicated that the eGFP-CCDC32(1-54) and eGFP-CCDC32(∆78-98) constructs are siRNA-resistant. However in Fig S2B, these proteins do not show any signal in the western blot, so it is not clear if they are expressed or simply not detected by the antibody. The presence of these proteins after silencing endogenous CCDC32 needs to be confirmed to support Figures 6 and Figures 7, which critically rely on the presence of the CCDC32 mutants. 

      Unfortunately, the C-terminally truncated CCDC32 proteins are not detected because they lack the antibody epitope, indeed even the ∆78-98 deletion is poorly detected (compare the GFP blot in new S1A with the anti-CCDC32 blot in S1B).  However, these constructs contain the same siRNA-resistance mutation as the full length protein. That they are expressed and siRNA resistant can be seen in Fig. S2A (now Fig. S1A) blotting for GFP.

      In Figures 6 and 7, siRNA knock-down of CCDC32 is only indicated for sub-figures F to G. Is this really the case? If not, the authors should clarify. The siRNA knock-down in Figure 1 is also only mentioned in the text, not in the figure legend. The authors should pay attention to make their figure legends easy to understand and unambiguous. 

      No, it is not the case.  Thank you for pointing out the uncertainty. We have added these details to the Figure legends and checked all Figure legends to ensure that they clearly describe the data shown.  

      - It is not exactly clear how the curves in Figure 3C (lower panel) on the invagination depth were obtained. Can the authors clarify this a bit more? For example, what are kT and kE in Figure 3A? What is I0? And how did the authors derive the logarithmic function used to quantify the invagination depth? In the main text, the authors say that the traces were 'logarithmically transformed'. This is not a technical term. The authors should refer to the actual equation used in the figure. 

      This analysis was developed by the Kirchhausen lab (Saffarian and Kirchhausen, 2008). We have added these details and reference them in the Figure legend and in the text. We also now use the more accurate descriptor ‘log-transformed’.

      - In the discussion, the claim 'The resulting dysregulation of AP2 inhibits CME, which further results in the development of CFNDS.' is maybe a bit too strong of a statement. Firstly, because the authors show themselves that CME is perturbed, but by no means inhibited. Secondly, the molecular link to CFNDS remains unclear. Even though CCDC32 mutants seem to be responsible for CFNDS and one of the mutant has been shown in this study to have a defect in endocytosis and AP2 binding, a direct link between CCDC32's function in endocytosis and CFNDS remains elusive. The authors should thus provide a more balanced discussion on this topic. 

      We have modified and softened our conclusions, which now read that the phenotypes we see likely “contribute to” rather than “cause” the disease.

      - In Figure S1, the authors annotate the presence of a coiled-coil domain, which they also use later on in the manuscript to generate mutations. Could the authors specify (and cite) where and how this coiled-coil domain has been identified? Is this predicted helix indeed a coiled-coil domain, or just a helix, as indicated by the authors in the discussion?

      See response to Reviewer 1, point 4.  We have changed this wording to alpha-helix. The ‘coiled-coil’ reference is historical and unlikely a true reflection of CCDC32 structure. AlphaFold 3.0 predictions were unable to identify with certainly any coiled-coil structures, even if we modelled potential dimers or trimers; and we find no evidence of dimerization of CCDC32 in vivo. We have clarified this in the text.

      Minor comments

      - In general, a more detailed explanation of the microscopy techniques used and the information they report would be beneficial to provide access to the article also to non-expert readers in the field. This concerns particularly the analysis methods used, for example: 

      How were the cohort-averaged fluorescence intensity and lifetime traces obtained? 

      How do the tools cmeAnalysis and DASC work? A brief explanation would be helpful. 

      We have expanded Methods to add these details, and also described them in the main text. 

      - The axis label of Figure 2B is not quite clear. What does 'TfnR uptake % of surface bound' mean? Maybe the authors could explain this in more detail in the figure legend? Is the drop in uptake efficiency also accessible by visual inspection of the images? It would be interesting to see that. 

      This is a standard measure of CME efficiency. 'TfnR uptake % of surface bound' = Internalized TfnR/Surface bound TfnR. Again, images may be misleading as defects in CME lead to increased levels of TfnR on the cell surface, which in turn would result in more Tfn uptake even if the rate of CME is decreased.

      - Figure 4: How is the occupancy of CCPs in the plasma membrane measured? What are the criteria used to divide CCSs into Flat, Dome or Sphere categories? 

      We have expanded Methods to add these details. Based on the degree of invagination, the shapes of CCSs were classified as either: flat CCSs with no obvious invagination; dome-shaped CCSs that had a hemispherical or less invaginated shape with visible edges of the clathrin lattice; and spherical CCSs that had a round shape with the invisible edges of clathrin lattice in 2D projection images. In most cases, the shapes were obvious in 2D PREM images. In uncertain cases, the degree of CCS invagination was determined using images tilted at ±10–20 degrees. The area of CCSs were measured using ImageJ and used for the calculation of the CCS occupancy on the plasma membrane.

      - Figure 5B: Can the authors explain, where exactly the GFP was engineered into AP2 alpha? This construct does not seem to be explained in the methods section. 

      We have added this information. The construct, which corresponds to an insertion of GFP into the flexible hinge region of AP2, at aa649, was first described by (Mino et al., 2020) and shown to be fully functional.  This information has been added to the Methods section.

      - Figure S1B: The authors should indicate the colour code used for the structural model.

      We have expanded our structural modeling using AlphaFold 3.0 in light of the recent publication suggesting the CCDC32 interacts with the µ2 subunit and does not bind full length AP2. These results are described in the text. The color coding now reflects certainty values given by AlphaFold 3.0 (Fig. S6B, D). 

      - The list of primers referred to in the materials and methods section does not exist. There is a Table S1, but this contains different data. The actual Table S1 is not referenced in the main text. This should be done. 

      We apologize for this error. We have now added this information in Table S2.

      Significance (Required):

      In this study, the authors analyse a so-far poorly understood endocytic accessory protein, CCDC32, and its implication for endocytosis. The experimental tool set used, allowing to quantify CCP dynamics and invagination is clearly a strength of the article that allows assessing the impact of an accessory protein towards the endocytic uptake mechanism, which is normally very robust towards mutations. Only through this detailed analysis of endocytosis progression could the authors detect clear differences in the presence and absence of CCDC32 and its mutants. If the above points are successfully addressed, the study will provide very interesting and highly relevant work allowing a better understanding of the early phases in CME with implication for disease. 

      The study is thus of potential interest to an audience interested in CME, in disease and its molecular reasons, as well as for readers interested in intrinsically disordered proteins to a certain extent, claiming thus a relatively broad audience. The presented results may initiate further studies of the so-far poorly understood and less well known accessory protein CCDC32.

      We thank the reviewer for their positive comments on the significance of our findings and the importance of our detailed phenotypic analysis made possible by quantitative live cell microscopy. We also believe that our new structural modeling of CCDC32 and our findings of complex and extensive interactions with AP2 make the reviewers point regarding intrinsically disordered proteins even more interesting and relevant to a broad audience.  We trust that our revisions indeed address the reviewer’s concerns. 

      The field of expertise of the reviewer is structural biology, biochemistry and clathrin mediated endocytosis. Expertise in cell biology is rather superficial.

      References:

      Aguet, F., Costin N. Antonescu, M. Mettlen, Sandra L. Schmid, and G. Danuser. 2013. Advances in Analysis of Low Signal-to-Noise Images Link Dynamin and AP2 to the Functions of an Endocytic Checkpoint. Developmental Cell. 26:279-291.

      Chen, Z., R.E. Mino, M. Mettlen, P. Michaely, M. Bhave, D.K. Reed, and S.L. Schmid. 2020. Wbox2: A clathrin terminal domain–derived peptide inhibitor of clathrin-mediated endocytosis. Journal of Cell Biology. 219.

      Grove, J., D.J. Metcalf, A.E. Knight, S.T. Wavre-Shapton, T. Sun, E.D. Protonotarios, L.D. Griffin, J. Lippincott-Schwartz, and M. Marsh. 2014. Flat clathrin lattices: stable features of the plasma membrane. Mol Biol Cell. 25:3581-3594.

      He, K., E. Song, S. Upadhyayula, S. Dang, R. Gaudin, W. Skillern, K. Bu, B.R. Capraro, I. Rapoport, I. Kusters, M. Ma, and T. Kirchhausen. 2020. Dynamics of Auxilin 1 and GAK in clathrinmediated traffic. J Cell Biol. 219.

      Mino, R.E., Z. Chen, M. Mettlen, and S.L. Schmid. 2020. An internally eGFP-tagged α-adaptin is a fully functional and improved fiduciary marker for clathrin-coated pit dynamics. Traffic. 21:603-616.

      Saffarian, S., and T. Kirchhausen. 2008. Differential evanescence nanometry: live-cell fluorescence measurements with 10-nm axial resolution on the plasma membrane. Biophys J. 94:23332342.

    1. Author response:

      Reviewer #1 (Evidence, reproducibility and clarity):

      Minor comments:

      In the results section (lines 498-499), the authors describe free kinetochores in many cells without associated spindle microtubules. However, some nuclei appear to have kinetochores, as presented in Figure 6. Could the authors clarify how this conclusion was derived using transmission electron microscopy (TEM) without serial sectioning, as this is not explicitly mentioned in the materials and methods?

      We observed free kinetochores in the ALLAN-KO parasites with no associated spindle microtubules (see Fig. 6Gh), while kinetochores are attached to spindle microtubules in WT-GFP cells (see Fig. 6Gc). To provide further evidence we analysed additional images and found that ALLAN-KO cells have free kinetochores in the centre of nucleus, unattached to spindle microtubules. We provide some more images clearly showing free kinetochores in these cells (new supplementary Fig. S11).

      However, in the ALLAN mutant, this difference is not absolute: in a search of over 50 cells, one example of a cell with a “normal” nuclear spindle and attached kinetochores was observed.

      The use of serial sectioning has limitations for examining small structures like kinetochores in whole cells. The limitations of the various techniques (for example, SBF-SEM vs tomography) are highlighted in our previous study (Hair et al 2022; PMID: 38092766), and we consider that examining a population of randomly sectioned cells provides a better understanding of the overall incidence of specific features.

      Discussion Section:

      Could the authors expand on why SUN1 and ALLAN are not required during asexual replication, even though they play essential roles during male gametogenesis?

      We observed no phenotype in asexual blood stage parasites associated with the sun1 and allan gene deletions. Several other Plasmodium berghei gene knockout parasites with a phenotype in sexual stages, for example CDPK4 (PMID: 15137943), SRPK (PMID: 20951971), PPKL (PMID: 23028336) and kinesin-5 (PMID: 33154955) have no phenotype in blood stages, so perhaps this is not surprising. One explanation may be the substantial differences in the mode of cell division between these two stages. Asexual blood stages produce new progeny (merozoites) over 24 hours with closed mitosis and asynchronous karyokinesis during schizogony, while male gametogenesis is a rapid process, completed within 15 min to produce eight flagellated gametes. During male gametogenesis the nuclear envelope must expand to accommodate the increased DNA content (from 1N to 8N) before cytokinesis. Furthermore, male gametogenesis is the only stage of the life cycle to make flagella, and axonemes must be assembled in the cytoplasm to produce the flagellated motile male gametes at the end of the process. Thus, these two stages of parasite development have some very different and specific features.

      Lines 611-613 states: "These loops serve as structural hubs for spindle assembly and kinetochore attachment at the nuclear MTOC, separating nuclear and cytoplasmic compartments." Could the authors elaborate on the evidence supporting this statement?

      We observed the loops/folds in the nuclear envelope (NE) as revealed by SUN1-GFP and 3D TEM images during male gametogenesis. These folds/loops occur mainly in the vicinity of the nuclear MTOC where the spindles are assembled (as visualised by EB1 fluorescence) and attached to kinetochores (as visualised by NDC80 fluorescence). These loops/folds may form due to the contraction of the spindle pole back to the nuclear periphery, inducing distortion of the NE. Since there is no physical segregation of chromosomes during the three rounds of mitosis (DNA increasing from 1N to 8N), we suggest that these folds provide additional space for spindle and kinetochore dynamics within an intact NE to maintain separation from the cytoplasm (as shown by location of kinesin-8B).

      In lines 621-622, the authors suggest that ALLAN may have a broader role in NE remodelling across the parasite's lifecycle. Could they reflect on or remind readers of the finding that ALLAN is not essential during the asexual stage?

      ALLAN-GFP is expressed throughout the parasite life cycle but as the reviewer points out, a functional role is more pronounced during male gametogenesis. This does not mean that it has no role at other stages of the life cycle even if there is no obvious phenotype following deletion of the gene during the asexual blood stage. The fact that ALLAN is not essential during the asexual blood stage is noted in lines 628-29.

      Reviewer #2 (Evidence, reproducibility and clarity):

      Introduction

      Line 63: The authors stat: "NE is integral to mitosis, supporting spindle formation, kinetochore attachment, and chromosome segregation..". Seemingly at odds, they also say (Line 69) that 'open' "mitosis is "characterized by complete NE disassembly".

      The authors could explain better the ideas presented in their quoted review from Dey and Baum, which points out that truly 'open' and 'closed' topologies may not exist and that even in 'open' mitosis, remnants of the NE may help support the mitotic spindle.

      We have modified the sentence in which we discuss current opinions about ‘open’ and ‘closed’ mitosis. It is believed that there is no complete disassembly of the NE during open mitosis and no completely intact NE during closed mitosis, respectively. In fact, the NE plays a critical role in the different modes of mitosis during MTOC organisation and spindle dynamics. Please see the modified lines 64-71.

      Results

      Fig 7 is the final figure; but would be more useful upfront.

      We have provided a new introductory figure (Fig 1) showing a schematic of conventional /canonical LINC complexes and evidence of SUN protein functions in model eukaryotes and compare them to what is known in apicomplexans.

      Fig 1D. The authors generated a C-terminal GFP-tagged SUN1 transfectants and used ultrastructure expansion microscopy (U-ExM) and structured illumination microscopy (SIM) to examine SUN1-GFP in male gametocytes post-activation. The immuno-labelling of SUN1-GFP in these fixed cells appears very different to the live cell images of SUN1-GFP. The labelling profile comprises distinct punctate structures (particularly in the U-ExM images), suggesting that paraformaldehyde fixation process, followed by the addition of the primary and secondary antibodies has caused coalescing of the SUN1-GFP signal into particular regions within the NE.

      We agree with the reviewer. Fixation with paraformaldehyde (PFA) results in a coalescence of the SUN1-GFP signal. We have also tried methanol fixation (see new Fig. S2), but a similar problem was encountered.

      Given these fixation issues, the suggestion that the SUN1-GFP signal is concentrated at the BB/ nuclear MTOC and "enriched near spindle poles" needs further support.

      These statements seem at odd with the data for live cell imaging where the SUN1-GFP seems evenly distributed around the nuclear periphery. Can the observation be quantitated by calculating the percentage of BB/ nuclear MTOC structures with associated SUN1-GFP puncta? If not, I am not convinced these data help understand the molecular events.

      We agree with the reviewer that whilst the live cell imaging showed an even distribution of SUN1-GFP signal, after fixation with either PFA or methanol, then SUN1-GFP puncta are observed in addition to the peripheral location around the stained DNA (Hoechst) (See Fig. S2; puncta are indicated by arrows). These SUN1-GFP labelled puncta were observed at the junction of the nuclear MTOC and the basal body (Fig. 2F). Quantification of the distribution showed that these SUN1-GFP puncta are associated with nuclear MTOC in more than 90 % of cells (18 cells examined). Live cell imaging of the dual labelled parasites; SUN1xkinesin-8B (Fig. 2H) and SUN1x EB1 (Fig. 2I) provides further support for the association of SUN1-GFP puncta with BB (kinesin-8B) /nuclear MTOC (EB1).

      The authors then generated dual transfectants and examined the relative locations of different markers in live cells. These data are more informative.

      The authors state; " ..SUN1-GFP marked the NE with strong signals located near the nuclear MTOCs situated between the BB tetrads". The nuclear MTOCs are not labelled in this experiment. The SUN1-GFP signal between the kinesin-8B puncta is evident as small puncta on regions of NE distortion. I would prefer to not describe this signal as "strong". The signal is stronger in other regions of the NE.

      We have modified the sentence on line 213 to accommodate this suggestion.

      Line 219. The authors state; "..SUN1-GFP is partially colocalized with spindle poles as indicated by EB1,.. it shows no overlap with kinetochores (NDC80)." The authors should provide an analysis of the level of overlap at a pixel by pixel level to support this statement.

      We now provide the overlap at a pixel-by-pixel level for representative images, and we have quantified more cells (n>30), as documented in the new Fig. S4A. We have also modified the sentence on line 219 to reflect these additions.

      The SUN1 construct is C-terminally GFP-tagged. By analogy with human SUN1, the C-terminal SUN domain is expected to be in the NE lumen. That is in a different compartment to EB1, which is located in the nuclear lumen (on the spindle). Thus, the overlap of signal is expected to be minimal.

      We agree with the reviewer that the overlap between EB1 and Sun1 signals is expected to be minimal. We have quantified the data and included it in Supplementary Fig. S4A.

      Similarly, given that EB1 and NDC80 are known to occupy overlapping locations on the spindle, it seems unlikely that SUN1 can overlap with one and not the other.

      We agree with the reviewer’s analysis that EB1 and NDC80 occupy overlapping locations on the spindle, although the length of NDC80 is less at the ends of spindles (see Author response image 1A) as shown in our previous study where we compared the locations of two spindle proteins, ARK2 and EB1, with that of NDC80 (Zeeshan et al, 2022; PMID: 37704606). In the present study we observed that Sun1-GFP partially overlaps with EB1 at the ends of the spindle, but not with NDC80. Please see Author response image 1B.

      Author response image 1.

      I note on Line 609, the authors state "Our study demonstrates that SUN1 is primarily localized to the nuclear side of the NE.." As per Fig 7D, and as discussed above, the bulk of the protein, including the SUN1 domain, is located in the space between the INM and the ONM.

      We appreciate the reviewer’s correction; we have now modified the sentence to indicate that the protein is largely localized in the space between the INM and the ONM on line 617.

      Interestingly, as the authors point out, nuclear membrane loops are evident around EB1 and NDC80 focal regions. The data suggests that the contraction of the spindle pole back to the nuclear periphery induces distortion of the NE.

      We agree with the reviewer’s suggestion that the data indicate that contraction of spindle poles back to the nuclear periphery may induce distortion of the NE.

      The author should discuss further the overlap of findings of this study with that from a recent manuscript (https://doi.org/10.1016/j.cels.2024.10.008). That Sayers et al. study identified a complex of SUN1 and ALLC1 as essential for male fertility in P. berghei. Sayers et al. also provide evidence that this complex particulate in the linkage of the MTOC to the NE and is needed for correct mitotic spindle formation during male gametogenesis.

      We thank the reviewer for this suggestion. The study by Sayers et al, (2024) was published while our manuscript was under preparation. It was interesting to see that these complementary studies have similar findings about the role of SUN1 and the novel complex of SUN1-ALLAN. Our study contains a more detailed, in-depth analysis both by Expansion and TEM of SUN1. We include additional studies on the role of ALLAN.  We discuss the overlap in the findings of the two studies in lines 590-605.

      While the work is interesting, the conclusions may need to be tempered. The authors suggestion that in the absence of KASH-domain proteins, the SUN1-ALLAN complex forms a non-canonical LINC complex (that is, a connection across the NE), that "achieves precise nuclear and cytoskeletal coordination".

      We have toned down the wording of this conclusion in lines 665-677.

      In other organisms, KASH interacts with the C-terminal domain on SUN1, which as mentioned above is located between the INM and ONM. By contrast, ALLAN interacts with the N-terminal domain of SUN1, which is located in the nuclear lumen. The SUN1-ALLAN interaction is clearly of interest, and ALLAN might replace some of the roles of lamins. However, the protein that functionally replaces KASH (i.e. links SUN1 to the ONM) remains unidentified.

      We agree with reviewer, and future studies will need to focus on identifying the KASH replacement that links SUN1 to the ONM.

      It may also be premature to suggest that the SUN1-ALLAN complex is promising target for blocking malaria transmission. How would it be targeted?

      We have deleted the sentence that raised this suggestion.

      While the above datasets are interesting and internally consistent, there are two other aspects of the manuscript that need further development before they can usefully contribute to the molecular story.

      The authors undertook a transcriptomic analysis of Δsun1 and WT gametocytes, at 8 and 30 min post-activation, revealing moderate changes (~2-fold change) in different genes. GO-based analysis suggested up-regulation of genes involved in lipid metabolism. Given the modest changes, it may not be correct to conclude that "lipid metabolism and microtubule function may be critical functions for gametogenesis that can be perturbed by sun1 deletion." These changes may simply be a consequence of the stalled male gametocyte development.

      Following the reviewer’s suggestion we have moved these data to the supplementary information (Fig. S5D-I) and toned down their discussion in the results and discussion sections.

      The authors have then undertaken a detailed lipid analysis of the Δsun1 and WT gametocytes, before and after activation. Substantial changes in lipid metabolites might not be expected in such a short period of time. And indeed, the changes appear minimal. Similarly, there are only minor changes in a few lipid sub-classes between Δsun1 and WT gametocytes. In my opinion, the data are not sufficient to support the authors conclusion that "SUN1 plays a crucial role, linking lipid metabolism to NE remodelling and gamete formation."

      In agreement with the reviewer’s comments we have moved  these data to supplementary information (Fig. S6) and substantially toned down the conclusions based on these findings.

      Reviewer #3 (Evidence, reproducibility and clarity):

      Major comments:

      My main concern with this manuscript is that the authors do conclude not only that SUN1 is important for spindle formation and basal body segregation, but also that it influences for lipid metabolism and NE dynamics. I don't think the data supports this conclusion, for several reasons listed below. I would suggest to remove this claim from the manuscript or at least tone it down unless more supporting data are provided, in particular showing any change in NE dynamics in the SUN1-KO. Instead I would recommend to focus on the more interesting role of SUN1-ALLAN in bipartite MTOC organisation, which likely explains all observed phenotypes (including those in later stages of the parasite life cycle). In addition, some aspects of the knockout phenotype should be quantified to a bit deeper level.

      In more detail:

      - The lipidomics analysis is clearly the weakest point of the manuscript: The authors state that there are significant changes in some lipid populations between WT and sun1-KO, and between activated and non-activated cells, yet no statistical analysis is shown and the error bars are quite high compared to only minor changes in the means. For some discussed lipids, the result text does not match the graphs, e.g. PA, where the increase upon activation is more pronounced in the SUN1-KO vs WT (contrary to the text), or MAG, which is reduced in the SUN1-KO vs WT (contrary to the text). I don't see the discussed changes in arachidonic acid levels and myristic acid levels in the data either. Even if the authors find after analysis some statistically significant differences between some groups, they should carefully discuss the biological significance of these differences. As it is, I do not think the presented data warrants the conclusion that deletion of SUN1 changes lipid homeostasis, but rather shows that overall lipid homeostasis is not majorly affected by gametogenesis or SUN1 deletion. As a minor comment, if you decide to keep the lipidomics analysis in the manuscript, please state how many replicates were done.

      As detailed above we have moved the lipidomics data to supplementary information (Fig. S6) and substantially toned down the discussion of these data in the results and discussion sections.

      - I can't quite follow the logic why the authors performed transcriptomic analysis of the SUN1 and how they chose their time points. Their data up to this point indicate that SUN1 has a structural or coordinating role in the bipartite MTOC during male gametogenesis. Based on that it is rather unlikely that SUN1 KO directly leads to transcriptional changes within the 8 min of exflagellation. Isn't it more likely that transcriptional differences are purely a downstream effect of incomplete/failed gametogenesis? This is particularly true for the comparison at 30 min, which compares a mixture of exflagellated/emerged gametes and zygotes in WT to a mixture of aberrant, arrested gametes in the knockout, which will likely not give any meaningful insight. The by far most significant GO-term is then also nuclear-transcribed mRNA catabolic process, which is likely not related at all to SUN1 function (and the authors do not even comment on this in the main text). I would therefore suggest removing the 30 min data set from this manuscript. As a minor point, I would suggest highlighting some of the top de-regulated gene IDs in the volcano plots and stating their function. Also, please state how you prepared the cells for the transcriptomes and in how many replicates this was done.

      As suggested by the reviewer we have removed the 30 min post activation data from the manuscript. We have also moved the rest of the transcriptomics data to supplementary information (Fig. S5) and toned down the presentation of this aspect of the work in the results and discussion sections.

      - Live-cell imaging of SUN1-GFP does nicely visualise the NE during gametogenesis, showing a highly dynamic NE forming loops and folds, which is very exciting to see. It would be beneficial to also show a video from the life-cell imaging.

      We have now added videos to the manuscript as suggested by the reviewer. Please see the supplementary Videos S1 and S2.

      In their discussion, the authors state multiple times that NE dynamics are changed upon SUN1 KO. Yet, they do not provide data supporting this claim, i.e. that the extended loops and folds found in the nuclear envelope during gametogenesis are affected in any way by the knockout of SUN1 or ALLAN. What happens to the NE in absence of SUN1? Are there less loops and folds? In absence of a reliable NE marker this may not be entirely easy to address, but at least some SBF-SEM images of the sun1-KO gametocytes could provide insight.

      It was difficult to provide SBF-SEM images as that work is beyond the scope of this manuscript. We will consider this approach in our future work. We re-examined many of our TEM images of SUN1-KO and ALLAN-KO parasites and did find some micrographs showing aberrant nuclear membrane folding (<5%) (Please see Author response image 2). However, we also observed similar structures in some of the WT-GFP samples (<5%), so we do not think this is a strong phenotype of the SUN1 or ALLAN mutants.

      Author response image 2.

       

      - I think the exciting part of the manuscript is the cell biological role of SUN1 on male gametogenesis, which could be carved out a bit more by a more detailed phenotyping. Specifically it would be good to quantify

      (1) If DNA replication to an octoploid state still occurs in SUN1-KO and ALLAN-KO,

      DNA replication is not affected in the SUN1-KO and ALLAN-KO mutants: DNA content increases to 8N (data added in Fig. 3J and Fig. S10F).

      (2) The proportion of anucleated gametes in WT and the KO lines

      We have added these data in Fig. 3K and Fig. S10G

      (3) A quantification of the BB clustering phenotype (in which proportion of cells do the authors see this phenotype). This could be addressed by simple fixed immunofluorescence images of the respective WT/KO lines at various time points after activation (or possibly by reanalysis of the already obtained images) and would really improve the manuscript.

      We have reanalysed the BB clustering phenotype and added the quantitative data in Fig. 4E and Fig. S7.

      Especially the claim that emerged SUN1-KO gametes lack a nucleus is currently only based on single slices of few TEM cells and would benefit from a more thorough quantification in both SUN1- and ALLAN-Kos

      We have examined many microgametes (100+ sections). In WT parasites a small proportion of gametes can appear to lack a nucleus if it does not extend all the way to the apical and basal ends (Hair et al. 2022). However, the proportion of microgametes that appear to lack a nucleus (no nucleus seen in any section) was much higher in the SUN1 mutant. In contrast, this difference was not as clear cut in the ALLAN mutant with a small proportion of intact (with axoneme and nucleus) microgametes being observed.

      We have done additional analysis of male gametes, looking for the presence of the nucleus by live cell imaging after DNA staining with Hoechst. These data are added in Fig. 3K (for Sun1-KO) and Fig. S10G (for Allan-KO).

      - The TEM suggests that in the SUN1-KO, kinetochores are free in the nucleus. Are all kinetochores free or do some still associate to a (minor/incorrectly formed) spindle? The authors could address this by tagging NDC80 in the KO lines.

      Our observation and quantification of the data indicated that 100% of kinetochores were attached to spindle microtubules and that 0% were unattached kinetochores in the WT parasites. However, the exact opposite was found for the SUN1 mutant with 100% unattached kinetochores and 0% attached. The result was not quite as clear cut in the ALLAN mutant, with 98% unattached and 2% attached. An important observation was the lack of separation of the nuclear poles and any spindle formation. Spindle formation was never or very rarely observed in the mutants.

      - Finally, I think it is curious that in contrast to SUN1, ALLAN seems to be less important, with some KO parasite completing the life cycle. Maybe a more detailed phenotyping as above gives some more hints to where the phenotypic difference between the two proteins lies. I would assume some ALLAN-KO cells can still segregate the basal body. Can the authors speculate/discuss in more detail why these two proteins seems to have slightly different phenotypes?

      We agree with the reviewer. Overall, the ALLAN-KO has a less prominent phenotype than that of the Sun1-KO. The main difference is that in the ALLAN-KO mutant some basal body segregation can occur, leading to the production of some fertile microgametocytes, and ookinetes, and oocyst formation (Fig. 8). Approximately 5% of oocysts sporulated to release infective sporozoites that could infect mice in bite back experiments and complete the life cycle. In contrast the Sun1-KO mutant made no healthy oocysts, or infective sporozoites, and could not complete the life cycle in bite back experiments. We have analysed the phenotype in detail and provide quantitative data for gametocyte stages by EM and ExM in Figs. 4 and S8 (SUN1) and Figs. 7 and S11 (ALLAN). We have also performed detailed analysis of oocyst and sporozoite stages and included the data in Fig. 3 (SUN1) and S10 (ALLAN).

      Based on the location, and functional and interactome data, we think that SUN1 plays a central role in coordinating nucleoplasm and cytoplasmic events as a key component of the nuclear membrane lumen, whereas ALLAN is located in the nucleoplasm. Deleting the SUN1 gene may disrupt the connection between INM and ONM whereas the deletion of ALLAN may affect only the INM.

      Some additional points where the data is not entirely sound yet or could be improved:

      - Localisation of SUN1: There seems to be a discrepancy between SUN1-GFP location as observed by live cell microscopy, and by Expansion Microscopy (ExM), similar for ALLAN-GFP. By live-cell microscopy, the SUN1 localisation is much more evenly distributed around the NE, while the localisation in ExM is much more punctuated, and e.g. in Figure 1E seems to be within the nucleus. Do the authors have an explanation for this? Also, in Fig. 1D there are two GFP foci at the cell periphery (bottom left of the image), which I would think are not SUN1-Foci, as they seem to be outside of the cell. Is the antibody specific? Was there a negative control done for the antibody (WT cells stained with GFP antibodies after ExM)?

      High resolution SIM and expansion microscopy showed that the SUN1-GFP molecules coalesce to form puncta, in contrast to the more uniform distribution observed by live cell imaging. This apparent difference may be due to a better resolution that could not be achieved by live cell imaging. We agree with the reviewer that the two green foci are outside of the cell. As a negative control we have used WT-ANKA cells (which contain no GFP) and the anti-GFP antibody, which gave no signal. This confirms the specificity of the antibody (please see the new Fig. S3). 

      - The authors argue that SIM gave unexpected results due to PFA fixation leading to collapse of the NE loops. However, they also fix their ExM cells and their EM cells with PFA and do not observe a collapse, at least from what I see in the two presented images and in the 3D reconstruction. Is there something else different in the sample preparation?

      There was no difference in the fixation process for samples examined by SIM and ExM, but we used an anti-GFP antibody in ExM to visualise the SUN1-GFP, while in SIM the images of GFP signal were collected directly after fixation.  We used both PFA and methanol as fixative, and both methods showed a coalescing of the SUN1-GFP signal (please see the new Fig. S2 and S3).

      Can the authors trace their NE in ExM according to the NHS-Ester signal?

      We could trace the NE in the ExM by the NHS-ester signal and observed that the SUN1-GFP signal was largely coincident with the NE (Please see the new Fig. S3B).

      - Fig 2D: It would be good to not just show images of oocysts but actually quantify their size from images. Also, have the authors determined the sporozoite numbers in SUN1-KO?

      We have measured oocyst size (data added in new Fig. 3) and added the sporozoite quantification data in Fig. 3D.

      - Line 481-483: the authors state that oocyst size is reduced in ALLAN-KO but do not show the data. Please quantify oocyst size or at least show representative images. Also the drastic decrease in sporozoite numbers (Fig. 6D, E) is not mentioned in the text. Please add reference to Fig S7D when talking about the bite back data.

      We have added the oocyst size data in Fig. S10. We mention the changes in sporozoite numbers (now  shown in Fig. 7D, E), and refer to  the bite back data shown in current Fig. 7E.

      - Fig S1C, 6C: Both WB images are stitched, but this is not clearly indicated e.g. by leaving a small gap between the lanes. Also please show a loading control along with the western blots. Also there seems to be a (unspecific?) band in the control, running at the same height as Allan-GFP WB. What exactly is the control?

      We have provided the original blot showing the bands of ALLAN-GFP and SUN1-GFP. As a positive control, we used an RNA associated protein (RAP-GFP) that is highly expressed in Plasmodium and regularly used in our lab for this purpose.

      - Regarding the crossing experiment: The authors conclude from this cross that SUN1 is only needed in males, yet for this conclusion they would need to also show that a cross with a female line does not rescue the phenotype. The authors should repeat the cross with a male-deficient line to really test if the phenotype is an exclusively male phenotype. In addition, line 270-272 states that no oocysts/sporozoites were detected in sun1-ko and nek4-ko parasites. However, the figure 2E shows only oocysts, not sporozoites, and shows also that sun1-ko does form oocysts, albeit dead ones.

      We have now performed the experiment of crossing the Sun1-KO parasite line with a male deficient line (Hap2-KO) and added the data in Fig. 3I. We have added images showing sporozoites in oocysts.

      - In Fig S1 the authors show that they also generated a SUN1-mCherry line, yet they do not use it in any of the presented experiments (unless I missed it). Would it be beneficial to cross the SUN1-mCherry line with the Allan1-GFP line to test colocalisation (possibly also by expansion microscopy)?

      We did generate a SUN1-mCherry line, with the intent to cross ALLAN-GFP and SUN1-mCherry lines and observe the co-location of the proteins. Despite multiple attempts this cross was unsuccessful. This may have been due to their close proximity such that the addition of both GFP and mCherry was difficult to facilitate a proper protein-protein interaction between either of the proteins.

      - Line 498: "In a significant proportion of cells" - What was the proportion of cells, and what does significant mean in this context?

      Approximately 67% of cells showed the clumping of BBs. We have now added the numbers in Figs. 6H and S11I.

      - The authors should discuss a bit more how their work relates to the work of Sayers et al. 2024, which also identified the SUN1-ALLAN complex. The paper is cited, but only very briefly commented on.

      We have extended this discussion now in lines 590-605.

      Suggestions how to improve the writing and data presentation.

      - General presentation of microscopy images: Considering that large parts of the manuscript are based on microscopy data, their presentation could be improved. Single-channel microscopy images would benefit from being depicted in gray scale instead of color, which would make it easier to see the structures and intensities (especially for blue channels).

      Whilst we agree with the reviewer, sometimes it is difficult to see the features in the merged images. Therefore, we would like to request to be allowed to retain the colours, which can be easily followed in both individual and merged images.

      Also, it would be good to harmonize in which panels arrows are shown (e.g. Fig 1G, where some white arrows are in the SUN1-GFP panel, while others are in the merge panel, but they presumably indicate the same thing.). At the same time, Fig 1H doesn't have any with arrows, even though the figure legend states so.

      We apologise for this lack of consistency, and we have now added arrows wherever they are missing to harmonise in the presentations.

      Fig 3A and S4 show the same experiment but are coloured in different colours (NHS-Eester in green vs grey scale).

      - Are the scale bars of all expansion microscopy images adjusted for the expansion factor?

      Yes, the scale bars are adjusted accordingly.

      - The figure legends would benefit from streamlining, as they have very different style between figures (eg Fig. 6 which has a concise figure legend vs microscopy figures where figure legends are very long and describe not only the figure but the results)

      The figure legends have been streamlined, with removal of the description of results.

      - Line 155-156: The text makes it sound like the expression only happens after activation. is that the case? Are these images activated or non-activated gametocytes?

      They are expressed before activation, but the signal intensifies after activation. Images from before and after activation of gametocytes have been added in Fig. S1F.

      - Line 267: Reference to the original nek4-KO paper missing

      This reference is now included.

      - Line 301: The reference to Figure 2J seems to be a bit arbitrarily placed. Also, this schematic of lipid metabolism is never discussed in relation to the transcriptomic or lipidomic data.

      We have moved these data to supplementary information and modified the text.

      - Line 347-349 states that gametes emerged, but the referenced figure shows activated gametocytes before exflagellation.

      We have corrected the text to the start of exflagellation.

      - Line 588: Spelling mistake in SUN1-domain

      Corrected.

      - Line 726/731: i missing in anti-GFP

      Corrected.

      - Line 787-789: statement of scale bar and number of cells imaged is not at the right position in the figure legend.

      Moved to right place

      - Line 779, 783: "shades of green" should be just "green". Same goes for line 986, 989 with "shades of grey"

      Changed.

      - Line 974, 976: please correct to WT-GFP and dsun1

      Corrected.

      - Line 1041, 1044: WT-GFP instead of WTGFP.

      Corrected to WT-GFP.

      - Fig 1B, D, E, Fig S1G, H: What are the time points of imaging?

      We have added the time points to the images in these figures.

      - Fig 1D/Line 727: the scale of the scale bar on the inset is missing.

      We have added the scale bar.

      - Fig 3 E-G and 6H-J: Please indicate total number of cells/images analysed per quantification, either in the graphs themselves or in the figure legend.

      We indicate now the number of cells analysed in individual figures and also in Fig. S5C and S8C, respectively.

      - Fig 5B: What is NP

      Nuclear Pole (NP), also known as the nuclear/acentriolar MTOC (Zeeshan et al 2022; PMID: 35550346).

      - Fig S1B/D: The legend states that there is an arrow indicating the band, but there is none.

      We have added the arrow.

      - Fig S2C: Is the scale bar really the same for the zygote and the ookinete?

      We have checked this and used the same for both zygote and ookinete.

      - Fig S3C, S7C: which stages was qRT-PCR done on?

      Gametocytes activated for 8 min.

      - Fig. S3D, S7D: According to the figure legend, three independent experiments were performed. How many mice were used per experiment? It would be good to depict the individual data points instead of the bar graph. For S7D, 3 data points are depicted (one in WT, two in allan-KO), what do they mean?

      The bite back experiment was performed using 15-20 mosquitoes infected with WT-GFP and gene knockout lines to feed on one naïve mouse each, in three different experiments. We have now included the data points in the bar diagrams.

      - Fig S3: Panel letters E and G are missing

      We have updated the lettering in current Fig. S5

      - Fig 3D: Please indicate what those boxes are. I presume that these are the insets show in b, e and j, but it is never mentioned. J is not even larger than i. Also, f is quite cropped, it would be good to see the large-scale image it comes from to see where in the nucleus these kinetochores are placed. Were there unbound kinetochores found in WT?

      We mention the boxes in the figure legends. It is rare to find unbound kinetochores in WT parasite. We provide large scale and zoomed-in images of free kinetochores in Fig. S8.

      - Fig S4: Insets are not mentioned in the figure legend. Please add scale bar to zoom-ins

      We now describe the insets in the figure legends and have added scale bars to the zoomed-in images.

      - Fig S5A, B: Please indicate which inset belongs to which sub-panel. Where does Ac stem from?

      We have now included the full image showing the inset (new Fig. S8).

      - Fig S5C and S8C: Change "DNA" to "Nucleus".

      We have changed “DNA” to “Nucleus”. Now they are Fig. S8K and S11I.

      Reviewer #3 (Significance):

      Yet, the statement that SUN1 is also important for lipid homoeostasis and NE dynamics is currently not backed up by sufficient data. I believe that the manuscript would benefit from removing the less convincing transcriptomic and lipidomic datasets and rather focus on more deeply characterising the cell biology of the knockouts. This way, the results would be interesting not only for parasitologists, but also for more general cell biologists.

      We have moved the lipidomics and transcriptomics data to supplementary information and toned down the emphasis on these data to make the manuscript more focused on the cell biology and analysis of the genetic KO data.

    1. Author response:

      eLife assessment

      This study is a detailed investigation of how chromatin structure influences replication origin function in yeast ribosomal DNA, with focus on the role of the histone deacetylase Sir2 and the chromatin remodeler Fun30. Convincing evidence shows that Sir2 does not affect origin licensing but rather affects local transcription and nucleosome positioning which correlates with increased origin firing. However, the evidence remains incomplete as the methods employed do not rigorously establish a key aspect of the mechanism, fully address some alternative models, or sufficiently relate to prior results. Overall, this is a valuable advance for the field that could be improved to establish a more robust paradigm.

      Public Reviews:

      Reviewer #1 (Public Review):

      Summary:

      This paper presents a mechanistic study of rDNA origin regulation in yeast by SIR2. Each of the ~180 tandemly repeated rDNA gene copies contains a potential replication origin. Early-efficient initiation of these origins is suppressed by Sir2, reducing competition with origins distributed throughout the genome for rate-limiting initiation factors. Previous studies by these authors showed that SIR2 deletion advances replication timing of rDNA origins by a complex mechanism of transcriptional de-repression of a local PolII promoter causing licensed origin proteins (MCMcomplexes) to re-localize (slide along the DNA) to a different (and altered) chromatin environment. In this study, they identify a chromatin remodeler, FUN30, that suppresses the sir2∆ effect, and remarkably, results in a contraction of the rDNA to about one-quarter it's normal length/number of repeats, implicating replication defects of the rDNA. Through examination of replication timing, MCM occupancy and nucleosome occupancy on the chromatin in sir2, fun30, and double mutants, they propose a model where nucleosome position relative to the licensed origin (MCM complexes) intrinsically determines origin timing/efficiency. While their interpretations of the data are largely reasonable and can be interpreted to support their model, a key weakness is the connection between Mcm ChEC signal disappearance and origin firing. While the cyclical chromatin association-dissociation of MCM proteins with potential origin sequences may be generally interpreted as licensing followed by firing, dissociation may also result from passive replication and as shown here, displacement by transcription and/or chromatin remodeling.

      While it is true that both transcription and passive replication can cause the signal of MCM-ChEC to disappear, neither can cause selective disappearance of the displaced complex without affecting the non-displaced complex.  Indeed, in the case of transcription, RNA polymerase transcribing C-pro would have to first dislodge the normally positioned MCM complex before even reaching the displaced complex.  Furthermore, deletion of FUN30 leads to both more C-pro transcription and less disappearance of the displaced MCM complex.  It is important to keep in mind that this cannot somehow reflect continuous replenishment of displaced MCMs with newly loaded MCMs, since the cells are in S phase and licensing is restricted to G1. 

      Moreover, linking its disappearance from chromatin in the ChEC method with such precise resolution needs to be validated against an independent method to determine the initiation site(s). Differences in rDNA copy number and relative transcription levels also are not directly accounted for, obscuring a clearer interpretation of the results.

      Copy number reduction of the magnitude caused by deletion of SIR2 and FUN30 does not suppress the sir2D effect (i.e. early replication of the rDNA), but rather exacerbates it.  In particular, deletion of SIR2 and FUN30 causes the rDNA to shrink to approximately 35 copies.  Kwan et al., 2023 (PMID: 36842087) have shown that reduction of rDNA copy number to 35 causes a dramatic acceleration of rDNA replication in a SIR2 strain.  Thus, the effect of rDNA size on replication timing reinforces our conclusion that deletion of FUN30 suppresses rDNA replication.

      However, to address this concern directly, in the revision we will include 2 D gels in fob1 strains with equal number of repeats that allows to conclude that the effect of FUN30 deletion in suppressing rDNA origin firing is independent of either rDNA size or FOB1. The figure of the critical 2 D gels is shown below in the reply to reviewer 2.

      Nevertheless, this paper makes a valuable advance with the finding of Fun30 involvement, which substantially reduces rDNA repeat number in sir2∆ background. The model they develop is compelling and I am inclined to agree, but I think the evidence on this specific point is purely correlative and a better method is needed to address the initiation site question. The authors deserve credit for their efforts to elucidate our obscure understanding of the intricacies of chromatin regulation. At a minimum, I suggest their conclusions on these points of concern should be softened and caveats discussed. Statistical analysis is lacking for some claims.

      Strengths are the identification of FUN30 as suppressor, examination of specific mutants of FUN30 to distinguish likely functional involvement. Use of multiple methods to analyze replication and protein occupancies on chromatin. Development of a coherent model.

      Weaknesses are failure to address copy number as a variable; insufficient validation of ChEC method relationship to exact initiation locus; lack of statistical analysis in some cases. 

      The two potential initiation sites that one would monitor (non-displaced and displaced) are separated by less than 150 base pairs, and other techniques simply do not have the resolution necessary to distinguish such differences.  Furthermore, as we suggest in the manuscript, our results are consistent with a model in which it is only the displaced MCM complex that is activated, whether in sir2 or WT.  If no genotype-dependent difference in initiation sites is even expected, it would be hard to interpret even the most precise replication-based assays.  However, the reviewer is correct that this is a novel technique and that confirmation with a well-established technique is comforting, therefore we are performing ChIP experiments to corroborate, to the extent possible, the conclusions that we reached with ChEC. 

      We appreciate the reviewer pointing out that some statistical analyses were lacking, and we will correct this in a revised manuscript.

      Additional background and discussion for public review:

      This paper broadly addresses the mechanism(s) that regulate replication origin firing in different chromatin contexts. The rDNA origin is present in each of ~180 tandem repeats of the rDNA sequence, representing a high potential origin density per length of DNA (9.1kb repeat unit). However, the average origin efficiency of rDNA origins is relatively low (~20% in wild-type cells), which reduces the replication load on the overall genome by reducing competition with origins throughout the genome for limiting replication initiation factors. Deletion of histone deacetylase SIR2, which silences PolII transcription within the rDNA, results in increased early activation or the rDNA origins (and reduced rate of overall genome replication). Previous work by the authors showed that MCM complexes loaded onto the rDNA origins (origin licensing) were laterally displaced (sliding) along the rDNA, away from a well-positioned nucleosome on one side. The authors' major hypothesis throughout this work is that the new MCM location(s) are intrinsically more efficient configurations for origin firing. The authors identify a chromatin remodeling enzyme, FUN30, whose deletion appears to suppress the earlier activation of rDNA origins in sir2∆ cells. Indeed, it appears that the reduction of rDNA origin activity in sir2∆ fun30∆ cells is severe enough to results in a substantial reduction in the rDNA array repeat length (number of repeats); the reduced rDNA length presumably facilitates it's more stable replication and maintenance.

      Analysis of replication by 2D gels is marginally convincing, using 2D gels for this purpose is very challenging and tricky to quantify. The more quantitative analysis by EdU incorporation is more convincing of the suppression of the earlier replication caused by SIR2 deletion.

      To address the mechanism of suppression, they analyze MCM positioning using ChEC, which in G1 cells shows partial displacement of MCM from normal position A to positions B and C in sir2∆ cells and similar but more complete displacement away from A to positions B and C in sir2fun30 cells. During S-phase in the presence of hydroxyurea, which slows replication progression considerably (and blocks later origin firing) MCM signals redistribute, which is interpreted to represent origin firing and bidirectional movement of MCMs (only one direction is shown), some of which accumulate near the replication fork barrier, consistent with their interpretation. They observe that MCMs displaced (in G1) to sites B or C in sir2∆ cells, disappear more rapidly during S-phase, whereas the similar dynamic is not observed in sir2∆fun30∆. This is the main basis for their conclusion that the B and C sites are more permissive than A. While this may be the simplest interpretation, there are limitations with this assay that undermine a rigorous conclusion (additional points below). The main problem is that we know the MCM complexes are mobile so disappearance may reflect displacement by other means including transcription which is high is the sir2∆ background. Indeed, the double mutant has greater level of transcription per repeat unit which might explain more displaced from A in G1. Thus, displacement might not always represent origin firing. Because the sir2 background profoundly changes transcription, and the double mutant has a much smaller array length associated with higher transcription, how can we rule out greater accessibility at site A, for example in sir2∆, leading to more firing, which is suppressed in sir2 fun30 due to greater MCM displacement away from A?

      I think the critical missing data to solidly support their conclusions is a definitive determination of the site(s) of initiation using a more direct method, such as strand specific sequencing of EdU or nascent strand analysis. More direct comparisons of the strains with lower copy number to rule out this facet. As discussed in detail below, copy number reduction is known to suppress at least part of the sir2∆ effect so this looms over the interpretations. I think they are probably correct in their overall model based on the simplest interpretation of the data but I think it remains to be rigorously established. I think they should soften their conclusions in this respect.

      Reviewer #2 (Public Review):

      Summary:

      In this manuscript, the authors follow up on their previous work showing that in the absence of the Sir2 deacetylase the MCM replicative helicase at the rDNA spacer region is repositioned to a region of low nucleosome occupancy. Here they show that the repositioned displaced MCMs have increased firing propensity relative to non-displaced MCMs. In addition, they show that activation of the repositioned MCMs and low nucleosome occupancy in the adjacent region depend on the chromatin remodeling activity of Fun30.

      Strengths:

      The paper provides new information on the role of a conserved chromatin remodeling protein in the regulation of origin firing and in addition provides evidence that not all loaded MCMs fire and that origin firing is regulated at a step downstream of MCM loading.

      Weaknesses:

      The relationship between the author's results and prior work on the role of Sir2 (and Fob1) in regulation of rDNA recombination and copy number maintenance is not explored, making it difficult to place the results in a broader context. Sir2 has previously been shown to be recruited by Fob1, which is also required for DSB formation and recombination-mediated changes in rDNA copy number. Are the changes that the authors observe specifically in fun30 sir2 cells related to this pathway? Is Fob1 required for the reduced rDNA copy number in fun30 sir2 double mutant cells? 

      Strains lacking SIR2 have unstable rDNA size, and FOB1 deletion stabilizes rDNA size in sir2 background. Likewise, FOB1 deletion influences the kinetics  rDNA size reduction in sir2 fun30 cells. However, the main effect of Fun30 in sir2 cells we were interested in, suppression of rDNA replication, is preserved in fob1 background, arguing that the observed effect is independent of Fob1 (see figure below). Given that the main focus of the paper is regulation of rDNA origins activity and that these changes were independent of Fob1, we had elected not to include these results in the original manuscript but will gladly include them in the revision.

      Besides refuting the possible role of Fob1 in the FUN30-mediated activation of rDNA origin firing in sir2 cells, the use of fob1 background enabled us compare the activation of rDNA origins in the sir2 and sir2 fun30 strains with equally short rDNA size. The 2-D gels demonstrate a dramatic suppression of rDNA origin activity upon deletion of FUN30 in the sir2 fob1 strains with 35 rDNA copies.

      Author response image 1.

      The deletion of FUN30 diminishes the replication bubble signal in a fob1 sir2 strain with 35 rDNA copies by more than tenfold. The single rARS signal, marked with the arrow, originates from the rightmost rDNA repeat. This specific rightmost rDNA NheI fragment is approximately 25 kb in size, distinctly larger than the 4.7 kb NheI 1N rARS-containing fragments that originate from the internal rDNA repeats.

      Reviewer #3 (Public Review):

      Summary:

      Heterochromatin is characterized by low transcription activity and late replication timing, both dependent on the NAD-dependent protein deacetylase Sir2, the founding member of the sirtuins. This manuscript addresses the mechanism by which Sir2 delays replication timing at the rDNA in budding yeast. Previous work from the same laboratory (Foss et al. PLoS Genetics 15, e1008138) showed that Sir2 represses transcription-dependent displacement of the Mcm helicase in the rDNA. In this manuscript, the authors show convincingly that the repositioned Mcms fire earlier and that this early firing partly depends on the ATPase activity of the nucleosome remodeler Fun30. Using read-depth analysis of sorted G1/S cells, fun30 was the only chromatin remodeler mutant that somewhat delayed replication timing in sir2 mutants, while nhp10, chd1, isw1, htl1, swr1, isw2, and irc5 had not effect. The conclusion was corroborated with orthogonal assays including two-dimensional gel electrophoresis and analysis of EdU incorporation at early origins. Using an insightful analysis with an Mcm-MNase fusion (Mcm-ChEC), the authors show that the repositioned Mcms in sir2 mutants fire earlier than the Mcm at the normal position in wild type. This early firing at the repositioned Mcms is partially suppressed by Fun30. In addition, the authors show Fun30 affects nucleosome occupancy at the sites of the repositioned Mcm, providing a plausible mechanism for the effect of Fun30 on Mcm firing at that position. However, the results from the MNAse-seq and ChEC-seq assays are not fully congruent for the fun30 single mutant. Overall, the results support the conclusions providing a much better mechanistic understanding how Sir2 affects replication timing at rDNA.

      The reason that the results for the fun30 single mutant appear incongruent, with a larger signal of the +2 nucleosome in the MNase-seq plot but a negligible signal in the ChEC-seq plot is the paucity of displaced Mcm in the fun30 single mutant. Given the relative absence of displaced MCMs, the MCM-MNase fusion protein can't "light up" the +2 nucleosome.  We will comment on this in the revision to clarify this. 

      Strengths

      (1) The data clearly show that the repositioned Mcm helicase fires earlier than the Mcm in the wild type position.

      (2) The study identifies a specific role for Fun30 in replication timing and an effect on nucleosome occupancy around the newly positioned Mcm helicase in sir2 cells.

      Weaknesses

      (1) It is unclear which strains were used in each experiment.

      (2) The relevance of the fun30 phospho-site mutant (S20AS28A) is unclear.

      (3) For some experiments (Figs. 3, 4, 6) it is unclear whether the data are reproducible and the differences significant. Information about the number of independent experiments and quantitation is lacking. This affects the interpretation, as fun30 seems to affect the +3 nucleosome much more than let on in the description.

      We appreciate the reviewer pointing out places in which our manuscript omitted key pieces of information (items 1 and 3), and we will fix these oversights in our revision. 

      With regard to point 2, we had written: 

      “Fun30 is also known to play a role in the DNA damage response; specifically, phosphorylation of Fun30 on S20 and S28 by CDK1 targets Fun30 to sites of DNA damage, where it promotes DNA resection (Chen et al. 2016; Bantele et al. 2017). To determine whether the replication phenotype that we observed might be a consequence of Fun30's role in the DNA damage response, we tested non-phosphorylatable mutants for the ability to suppress early replication of the rDNA in sir2; these mutations had no effect on the replication phenotype (Figure 2B), arguing against a primary role for Fun30

      in DNA damage repair that somehow manifests itself in replication.”

      We will expand on this to clarify our point in the revision.

    1. Author rsponse:

      Public Reviews:

      Reviewer #1 (Public review):

      Summary:

      In this paper, the authors have performed an antigenic assay for human seasonal N1 neuraminidase using antigens and mouse sera from 2009-2020 (with one avian N1 antigen). This shows two distinct antigen groups. There is poorer reactivity with sera from 2009-2012 against antigens from 2015-2019, and poorer reactivity with sera from 2015-2020 against antigens from 2009-2013. There is a long branch separating these two groups. However, 321 and 423 are the only two positions that are consistently different between the two groups. Therefore these are the most likely cause of these antigenic differences.

      Strengths:

      (1) A sensible rationale was given for the choice of sera, in terms of the genetic diversity.

      (2) There were two independent batches of one of the antigens used for generating sera, which demonstrated the level of heterogeneity in the experimental process.

      (3) Replicate of the Wisconsin/588/2019 antigen (as H1 and H6) is another useful measure of heterogeneity.

      (4) The presentation of the data, e.g. Figure 2, clearly shows two main antigenic groups.

      (5) The most modern sera are more recent than other related papers, which demonstrates that has been no major antigenic change.

      Weaknesses:

      (1) Issues with experimental methods

      As I am not an experimentalist, I cannot comment fully on the experimental methods. However, I note that BALB/c mice sera were used, whereas outbred ferret sera are typically used in influenza antigenic characterisation, so the antigenic difference observed may not be relevant in humans. Similarly, the mice were immunised with an artificial NA immunogen where the typical approach would be to infect the ferret with live virus intra-nasally.

      Indeed, ferrets are the gold standard model for the study of influenza. The main reason for this is the susceptibility of ferrets to infection with primary human influenza virus isolates and their ability to transmit human influenza A and B viruses. Although mouse models often require the use of mouse-adapted influenza virus strains, it is still the most used model to study new developments on influenza vaccine.

      In our previous publication we performed a parallel analysis of sera of ferrets that were primed by infection and boosted by recombinant protein, as well as mice that, like in this study that focuses on N1 NA, were prime-boosted with purified recombinant NA proteins in the presence of an adjuvant. Our data indicate that the NAI responses in immune sera from infected ferrets after infection and after boost enables similar antigenic classification and correlated strongly with those induced in mice that had been prime-boosted with adjuvanted recombinant NA (Catani et al., eLife 2024). To a large extend, the immunogenicity of an antigen relies on epitope accessibility, which may dictate a universal rule of immunogenicity and antigenicity (Altman et al., 2015).

      (2) Five mice sera were generated per immunogen and then pooled, but data was not presented that demonstrated these sera were sufficiently homogenous that this approach is valid.

      Although individual sera was not tested here. Based on previous studies from our group we are confident that a prime-boost schedule with 1 µg of adjuvanted soluble tetrameric NA, induces a highly homogeneous response in mice (Catani et al., 2022).

      (3) There were no homologous antigens for most of the sera. This makes the responses difficult to interpret as the homologous titre is often used to assess the overall reactivity of a serum. The sequence of the antigens used is not described, which again makes it difficult to interpret the results.

      The absence of homologous antigens may indeed make interpretation more difficult. However, we have observed that homologous sera do not always coincide with the highest reactivity, although highest reactivity is always found within an antigenic cluster. A sequence comparison would be appropriate to improve interpretability of the data. Therefore, a sequence alignment and a pairwise comparison will be provided in the revised manuscript as supplement. 

      (4) To be able to untangle the effects of the individual substitutions at 321, 386, and 432, it would have been useful to have included the naturally occurring variants at these positions, or to have generated mutants at these positions. Gao et al clearly show an antigenic difference with ferret sera correlated separately with N386K and I321V/K432E.

      The prevalence of single amino acid substitutions in N1 NA of clinical H1N1 virus strains isolated between 2009 and 2024 is minimal, which may indicate reduced fitness (see Author response image 1) in strains with these substitutions in NA. Nevertheless, we agree that the rescue of single mutants would provide important evidence to untangle those individual impacts on antigenicity. We plan to generate mutants with substitution at these positions in NA of A/Wisconsin/588/2019 H1N1 and determine the NAI against our panel of sera.

      Author response image 1.

      Prevalence of the indicated N1 NA substitutions in all clinical human H1N1 isolates with unique sequences deposited in the GISAID data bank since 2009.

      (5) The challenge experiments in Gao et al showed that NI titre was not a good correlate of protection, so that limits the interpretation of these results.

      On the contrary, challenges experiments confirmed that drift occurred in NA from H1N1 viruses isolated between 2009 (CA/09) and 2015 (MI/15). The dilution of transferred sera to equal inhibitory titers indicate that the homologous ferret sera (shown in figure 5e-f)(Gao et al., 2019) is still effective in protecting against infection while heterologous sera are not. This result emphasises that the nature of the homologous NAI response is well-suited for protection against a homologous challenge, although mechanistic data was not provided.

      Issues with the computational methods

      (6) The NAI titres were normalised using the ELISA results, and the motivation for this is not explained. It would be nice to see the raw values.

      Mice were immunized with different batches of recombinant protein. Each of those batches may have distinct intrinsic immunogenicity, as observed in Figure 1d. For that reason, NAI values were normalized using homologous ELISA titers induced by each respective NA antigen. A table with the raw values will be included in the revised manuscript.

      (7) It is not clear what value the random forest analysis adds here, given that positions 321 and 432 are the only two that consistently differ between the two groups.

      The substitutions at position 321 and 432 are indeed the only 2 consistently differing amino acids among the tested N1s. Although their correlation with antigenic clustering may be obvious after analysis, a random forest analysis would enable to reveal less obvious substitutions that contribute to the antigenic diversity. In the future, we intend to expand this methodology to strains that are not currently included in the panel. A random forest model is a relatively simple and performant method to deal with a new dataset.

      (8) As with the previous N2 paper, the metric for antigenic distance (the root mean square of the difference between the titres for two sera) is not one that would be consistent when different sera are included. More usual metrics of distance are Archetti-Horsfall, fold down from homologous, or fold down from maximum.

      The antigenic distances calculated prior to our random forest does use fold-difference as metrics as log2(max(EC50) / EC50). After having obtained the fold-difference values, a pairwise dissimilarity matrix was calculated to obtain the average antigenic distance between pairs of sera. A more detailed description of the methodology will be included in the methods session, including the R-code.

      (9) Antigenic cartography of these data is fraught. I wonder whether 2 dimensions are required for what seems like a 1-dimensional antigenic difference - certainly, the antigens, excluding the H5N1, are in a line. The map may be skewed by the high reactivity Brisbane/18 antigen. It is not clear if the column bases (normalisation factors for calculating antigenic distance) have been adjusted to account for the lack of homologous antigens. It is typical to present antigenic maps with a 1:1 x:y ratio.

      Antigenic cartography will be repeated excluding H5N1 and/or Brisbane/18 antigen. Data will be provided in the final rebuttal letter.

      Issues with interpretation

      (10) Figure 2 shows the NAI titres split into two groups for the antigens, however, A/Brisbane is an outlier in the second antigenic group with high reactivity.

      Indeed, A/Brisbane/02/2018 has overall higher IC50 values. However, it still falls into the same cluster that we called AG2. Highlighting A/Brisbane/02/2018 may lead to the misinterpretation of a non-existent antigenic group. 

      (11) Following Gao et al, I think you can claim that it is more likely that the antigenic change is due to K432E than I321V, based on a comparison of the amino acid change.

      Indeed, we would expect that substitution of the basic arginine to an acidic glutamate is more likely to impact antigenicity than the isoleucine-to-valine apolar substitution. Testing of mutant reassortants with single mutations may provide the definitive answer for that question.

      Appraisal:

      Taking into account the limitations of the experimental techniques (which I appreciate are due to resource constraints), this paper meets its aim of measuring the antigenic relationships between 2009-2020 seasonal N1s, showing that there were two main groups. The authors discovered that the difference between the two antigenic groups was likely attributable to positions 321 and 432, as these were the only two positions that were consistently different between the two groups. They came to this finding by using a random forest model, but other simpler methods could have been used.

      Impact:

      This paper contributes to the growing literature on the potential benefit of NA in the influenza vaccine.

      Reviewer #2 (Public review):

      Summary:

      In this study, Catani et al. have immunized mice with 17 recombinant N1 neuraminidases (NAs) from human isolates circulating between 2009-2020 to investigate antigenic diversity. NA inhibition (NAI) titers revealed two groups that were antigenically and phylogenetically distinct. Machine learning was used to estimate the antigenic distances between the N1 NAs and mutations at residues K432E and I321V were identified as key determinants of N1 NA antigenicity.

      Strengths:

      Observation of mutations associated with N1 antigenic drift.

      Weaknesses:

      Validation that K432E and I321V are responsible for antigenic drift was not determined in a background strain with native K432 and I321 or the restitution of antibody binding by reversion to K432 and I321 in strains that evaded sera.

      Reassortant A/Wisconsin/588/2019 with E432K, V321I and also K386N single mutations will be rescued and tested against the panel of sera.

    1. Author response:

      eLife Assessment

      This valuable study presents a theoretical model of how punctuated mutations influence multistep adaptation, supported by empirical evidence from some TCGA cancer cohorts. This solid model is noteworthy for cancer researchers as it points to the case for possible punctuated evolution rather than gradual genomic change. However, the parametrization and systematic evaluation of the theoretical framework in the context of tumor evolution remain incomplete, and alternative explanations for the empirical observations are still plausible.

      We thank the editor and the reviewers for their thorough engagement with our work. The reviewers’ comments have drawn our attention to several important points that we have addressed in the updated version. We believe that these modifications have substantially improved our paper.

      There were two major themes in the reviewers’ suggestions for improvement. The first was that we should demonstrate more concretely how the results in the theoretical/stylized modelling parts of our paper quantitatively relate to dynamics in cancer.

      To this end, we have now included a comprehensive quantification of the effect sizes of our results across large and biologically-relevant parameter ranges. Specifically, following reviewer 1’s suggestion to give more prominence to the branching process, we have added two figures (Fig S3-S4) quantifying the likelihood of multi-step adaptation in a branching process for a large range of mutation rates and birth-death ratios. Formulating our results in terms of birth-death ratios also allowed us to provide better intuition regarding how our results manifest in models with constant population size vs models of growing populations. In particular, the added figure (Fig S3) highlights that the effect size of temporal clustering on the probability of successful 2-step adaptation is very sensitive to the probability that the lineage of the first mutant would go extinct if it did not acquire a second mutation. As a result, the phenomenon we describe is biologically likely to be most effective in those phases during tumor evolution in which tumor growth is constrained. This important pattern had not been described sufficiently clearly in the initial version of our manuscript, and we thank both reviewers for their suggestions to make these improvements.

      The second major theme in the reviewers’ suggestions was focused on how we relate our theoretical findings to readouts in genomic data, with both reviewers pointing to potential alternative explanations for the empirical patterns we describe.

      We have now extended our empirical analyses following some of the reviewers’ suggestions. Specifically, we have included analyses investigating how the contribution of reactive oxygen species (ROS)-related mutation signatures correlates with our proxies for multi-step adaptation; and we have included robustness checks in which we use Spearman instead of Pearson correlations. Moreover, we have included more discussion on potential confounds and the assumptions going into our empirical analyses as well as the challenges in empirically identifying the phenomena we describe.

      Below, we respond in detail to the individual comments made by each reviewer.

      Public Reviews:

      Reviewer #1 (Public review):

      Summary:

      Grasper et al. present a combined analysis of the role of temporal mutagenesis in cancer, which includes both theoretical investigation and empirical analysis of point mutations in TCGA cancer patient cohorts. They find that temporally elevated mutation rates contribute to cancer fitness by allowing fast adaptation when the fitness drops (due to previous deleterious mutations). This may be relevant in the case of tumor suppressor genes (TSG), which follow the 2-hit hypothesis (i.e., biallelic 2 mutations are necessary to deactivate TS), and in cases where temporal mutagenesis occurs (e.g., high APOBEC, ROS). They provide evidence that this scenario is likely to occur in patients with some cancer types. This is an interesting and potentially important result that merits the attention of the target audience. Nonetheless, I have some questions (detailed below) regarding the design of the study, the tools and parametrization of the theoretical analysis, and the empirical analysis, which I think, if addressed, would make the paper more solid and the conclusion more substantiated.

      Strengths:

      Combined theoretical investigation with empirical analysis of cancer patients.

      Weaknesses:

      Parametrization and systematic investigation of theoretical tools and their relevance to tumor evolution.

      We sincerely thank Reviewer 1 for their comments. As communicated in more detail in the point-by-point replies to the “Recommendations for the authors”, we have revised the paper to address these comments in various ways. To summarize, Reviewer 1 asked for (1) more comprehensive analyses of the parameter space, especially in ranges of small fitness effects and low mutation rates; (2) additional clarifications on details of mechanisms described in the manuscript; and (3) suggested further robustness checks to our empirical analyses. We have addressed these points as follows: we have added detailed analyses of dynamics and effect sizes for branching processes (see Sections SI2 and SI3 in the Supplementary Information, as well as Figures S3 and S4). As suggested, these additions provide characterizations of effect sizes in biologically relevant parameter ranges (low mutation rates and smaller fitness effect sizes), and extend our descriptions to processes with dynamically changing population sizes. Moreover, we have added further clarifications at suggested points in the manuscript, e.g. to elaborate on the non-monotonicities in Fig 3. Lastly, we have undertaken robustness checks using Spearman rather than Pearson correlation coefficients to quantify relations between TSG deactivation and APOBEC signature contribution, and have performed analyses investigating dynamics of reactive oxygen species-associated mutagenesis instead of APOBEC.

      Reviewer #2 (Public review):

      This work presents theoretical results concerning the effect of punctuated mutation on multistep adaptation and empirical evidence for that effect in cancer. The empirical results seem to agree with the theoretical predictions. However, it is not clear how strong the effect should be on theoretical grounds, and there are other plausible explanations for the empirical observations.

      Thank you very much for these comments. We have now substantially expanded our investigations of the parameter space as outlined in the response to the “eLife Assessment” above and in the detailed comments below (A(1)-A(3)) to convey more quantitative intuition for the magnitude of the effects we describe for different phases of tumor evolution. We agree that there could be potential additional confounders to our empirical investigations besides the challenges regarding quantification that we already described in our initial version of the manuscript. We have thus included further discussion of these in our manuscript (see replies to B(1)-B(3)), and we have expanded our empirical analyses as outlined in the response to the “eLife Assessment”.

      For various reasons, the effect of punctuated mutation may be weaker than suggested by the theoretical and empirical analyses:

      (A1) The effect of punctuated mutation is much stronger when the first mutation of a two-step adaptation is deleterious (Figure 2). For double inactivation of a TSG, the first mutation--inactivation of one copy--would be expected to be neutral or slightly advantageous. The simulations depicted in Figure 4, which are supposed to demonstrate the expected effect for TSGs, assume that the first mutation is quite deleterious. This assumption seems inappropriate for TSGs, and perhaps the other synergistic pairs considered, and exaggerates the expected effects.

      Thank you for highlighting this discrepancy between Figure 2 and Figure 4. For computational efficiency and for illustration purposes, we had opted for high mutation rates and large fitness effects in Figure 2; however, our results are valid even in the setting of lower mutation rates and fitness effects. To improve the connection to Figure 4, and to address other related comments regarding parameter dependencies, we have now added more detailed quantification of the effects we describe (Figures SF3 and SF4) to the revised manuscript. These additions show that the effects illustrated in Figure 2 retain large effect sizes when going to much lower mutation rates and much smaller fitness effects. Indeed, while under high mutation rates we only see the large relative effects if the first mutation is highly deleterious, these large effects become more universal when going to low mutation rates.

      In general, it is correct that the selective disadvantage (or advantage) conveyed by the first mutation affects the likelihood of successful 2-step adaptations. It is also correct that the magnitude of the ‘relative effect’ of temporal clustering on valley-crossing is highest if the lineage with only the first of the two mutations is vanishingly unlikely to produce a second mutant before going extinct. If the first mutation is strongly deleterious, the lineage of such a first mutant is likely to quickly go extinct – and therefore also more likely to do so before producing a second mutant.

      However, this likelihood of producing the second mutant is also low if the mutation rate is low. As our added figure (Figure SF3) illustrates, at low mutation rates appropriate for cancer cells, is insensitive to the magnitude of the fitness disadvantage for large parts of the parameter space. Especially in populations of constant size (approximated by a birth/death ratio of 1), the relative effects for first mutations that reduce the birth rate by 0.5 or by 0.05 are indistinguishable (Figure SF3f).

      Moreover, the absolute effect (f<sub>k</sub> - f<sub>1</sub>), as we discuss in the paper (Figures SF2 and SF3) is largest in regions of the parameter space in which the first mutant is not infinitesimally unlikely to produce a second mutant (and f<sub>k</sub>  and f<sub>1</sub> would be infinitesimally small), but rather in parameter regions in which this first mutant has a non-negligible chance to produce a second mutant. The absolute effect (f<sub>k</sub> - f<sub>1</sub>) therefore peaks around fitness-neutral first mutations. While the next comment (below) says that our empirical investigations more closely resemble comparisons of relative effects and not absolute effects, we would expect that the observations in our data come preferentially from multi-step adaptations with large absolute effect since the absolute effect is maximal when both f<sub>k</sub> and f<sub>1</sub> are relatively high.

      In summary, we believe Figure 2, while having exaggerated parameters for very defendable reasons, is not a misleading illustration of the general phenomenon or of its applicability in biological settings, as effect sizes remain large when moving to biologically realistic parameter ranges. To clarify this issue, we have largely rewritten the relevant paragraphs in the results section and have added two additional figures (Figures SF3 and SF4) as well as a section in the SI with detailed discussion (SI2).

      (A2) More generally, parameter values affect the magnitude of the effect. The authors note, for example, that the relative effect decreases with mutation rate. They suggest that the absolute effect, which increases, is more important, but the relative effect seems more relevant and is what is assessed empirically.

      Thank you for this comment. As noted in the replies to the above comments, we have now included extensive investigations of how sensitive effect sizes are to different parameter choices. We also apologize for insufficiently clearly communicating how the quantities in Figure 4 relate to the findings of our theoretical models.

      The challenge in relating our results to single-timepoint sequencing data is that we only observe the mutations that a tumor has acquired, but we do not directly observe the mutation rate histories that brought about these mutations. As an alternative readout, we therefore consider (through rough proxies: TSGs and APOBEC signatures) the amount of 2-step adaptations per acquired/retained mutation. While we unfortunately cannot control for the average mutation rate in a sample, we motivate using this “TSG-deactivation score” by the hypothesis that for any given mutation rate, we expect a positive relationship between the amount of temporal clustering and the amount of 2-step adaptations per acquired/retained mutation. This hypothesis follows directly from our theoretical model where it formally translates to the statement that for a fixed μ, f<sub>k</sub> is increasing in k.

      However, while both quantities f<sub>k</sub>/f<sub>1</sub> or f<sub>k</sub> - f<sub>1</sub> from our theoretical model relate to this hypothesis – both are increasing in k –, neither of them maps directly onto the formulation of our empirical hypothesis.

      We have now rewritten the relevant passages of the manuscript to more clearly convey our motivation for constructing our TSG deactivation score in this form (P. 4-6).

      (A3) Routes to inactivation of both copies of a TSG that are not accelerated by punctuation will dilute any effects of punctuation. An example is a single somatic mutation followed by loss of heterozygosity. Such mechanisms are not included in the theoretical analysis nor assessed empirically. If, for example, 90% of double inactivations were the result of such mechanisms with a constant mutation rate, a factor of two effect of punctuated mutagenesis would increase the overall rate by only 10%. Consideration of the rate of apparent inactivation of just one TSG copy and of deletion of both copies would shed some light on the importance of this consideration.

      This is a very good point, thank you. In our empirical analyses, the main motivation was to investigate whether we would observe patterns that are qualitatively consistent with our theoretical predictions, i.e. whether we would find positive associations between valley-crossing and temporal clustering. Our aim in the empirical analyses was not to provide a quantitative estimate of how strongly temporally clustered mutation processes affect mutation accumulation in human cancers. We hence restricted attention to only one mutation process which is well characterized to be temporally clustered (APOBEC mutagenesis) and to only one category of (epi)genomic changes (SNPs, in which APOBEC signatures are well characterized). Of course, such an analysis ignores that other mutation processes (e.g. LOH, copy number changes, methylation in promoter regions, etc.) may interact with the mechanisms that we consider in deactivating Tumor suppressor genes.

      We have now updated the text to include further discussion of this limitation and further elaboration to convey that our empirical analyses are not intended as a complete quantification of the effect of temporal clustering on mutagenesis in-vivo (P. 10,11).

      Several factors besides the effects of punctuated mutation might explain or contribute to the empirical observations:

      (B1) High APOBEC3 activity can select for inactivation of TSGs (references in Butler and Banday 2023, PMID 36978147). This selective force is another plausible explanation for the empirical observations.

      Thank you for making this point. We agree that increased APOBEC3 activity, or any other similar perturbation, can change the fitness effect that any further changes/perturbations to the cell would bring about. Our empirical analyses therefore rely on the assumption that there are no major confounding structural differences in selection pressures between tumors with different levels of APOBEC signature contributions. We have expanded our discussion section to elaborate on this potential limitation (P. 10-11).

      While the hypothesis that APOBEC3 activity selects for inactivation of TSGSs has been suggested, there remain other explanations. Either way, the ways in which selective pressures have been suggested to change would not interfere relevantly with the effects we describe. The paper cited in the comment argues that “high APOBEC3 activity may generate a selective pressure favoring” TSG mutations as “APOBEC creates a high [mutation] burden, so cells with impaired DNA damage response (DDR) due to tumor suppressor mutations are more likely to avert apoptosis and continue proliferating”. To motivate this reasoning, in the same passage, the authors cite a high prevalence of TP53 mutations across several cancer types with “high burden of APOBEC3-induced mutations”, but also note that “this trend could arise from higher APOBEC3 expression in p53-mutated tumors since p53 may suppress APOBEC3B transcription via p21 and DREAM proteins”.

      Translated to our theoretical framework, this reasoning builds on the idea that APOBEC3 activity increases the selective advantage of mutants with inactivation of both copies of a TSG. In contrast, the mechanism we describe acts by altering the chances of mutants with only one TSG allele inactivated to inactivate the second allele before going extinct. If homozygous inactivation of TSGs generally conveys relatively strong fitness advantages, lineages with homozygous inactivation would already be unlikely to go extinct. Further increasing the fitness advantage of such lineages would thus manifest mostly in a quicker spread of these lineages, rather than in changes in the chance that these lineages survive. In turn, such a change would have limited effect on the “rate” at which such 2-step adaptations occur, but would mostly affect the speed at which they fixate. It would be interesting to investigate these effects empirically by quantifying the speed of proliferation and chance of going extinct for lineages that newly acquired inactivating mutations in TSGs.

      Beyond this explicit mention of selection pressures, the cited paper also discusses high occurrences of mutations in TSGs in relation to APOBEC. These enrichments, however, are not uniquely explained by an APOBEC-driven change in selection pressures. Indeed, our analyses would also predict such enrichments.

      (B2) Without punctuation, the rate of multistep adaptation is expected to rise more than linearly with mutation rate. Thus, if APOBEC signatures are correlated with a high mutation rate due to the action of APOBEC, this alone could explain the correlation with TSG inactivation.

      Thank you for making this point. Indeed, an identifying assumption that we make is that average mutation rates are balanced between samples with a higher vs lower APOBEC signature contribution. We cannot cleanly test this assumption, as we only observe aggregate mutation counts but not mutation rates. However, the fact that we observe an enrichment for APOBEC-associated mutations among the set of TSG-inactivating mutations (see Figure 4F) would be consistent with APOBEC-mutations driving the correlations in Fig 4D, rather than just average mutation rates. We have now added a paragraph to our manuscript to discuss these points (P. 10-11).

      (B3) The nature of mutations caused by APOBEC might explain the results. Notably, one of the two APOBEC mutation signatures, SBS13, is particularly likely to produce nonsense mutations. The authors count both nonsense and missense mutations, but nonsense mutations are more likely to inactivate the gene, and hence to be selected.

      Thank you for making this point.  We have included it in our discussion of potential confounders/limitations in the revised manuscript (P. 10-11).

    1. Author response:

      Reviewer 1:

      Summary:

      This paper describes molecular dynamics simulations (MDS) of the dynamics of two T-cell receptors (TCRs) bound to the same major histocompatibility complex molecule loaded with the same peptide (pMHC). The two TCRs (A6 and B7) bind to the pMHC with similar affinity and kinetics, but employ different residue contacts. The main purpose of the study is to quantify via MDS the differences in the inter- and intra-molecular motions of these complexes, with a specific focus on what the authors describe as catch-bond behavior between the TCRs and pMHC, which could explain how T-cells can discriminate between different peptides in the presence of weak separating force.

      Strengths:

      The authors present extensive simulation data that indicates that, in both complexes, the number of high-occupancy interdomain contacts initially increases with applied load, which is generally consistent with the authors’ conclusion that both complexes exhibit catch-bond behavior, although to different extents. In this way, the paper somewhat expands our understanding of peptide discrimination by T-cells.

      The reviewer makes thoughtful assessments of our manuscript. While our manuscript is meant to be a “short” contribution, our significant new finding is that even for TCRs targeting the same pMHC, having similar structures, and leading to similar functional outcomes in conventional assays, their response to applied load can be different. This supports out recent experimental work where TCRs targeting the same pMHC differed in their catch bond characteristics, and importantly, in their response to limiting copy numbers of pMHCs on the antigen-presenting cell (Akitsu et al., Sci. Adv., 2024; cited in our manuscript). Our present manuscript provides the physical basis where two similar TCRs respond to applied load differently. In the revised manuscript, we will make this point clearer.

      Weaknesses:

      While generally well supported by data, the conclusions would nevertheless benefit from a more concise presentation of information in the figures, as well as from suggesting experimentally testable predictions.

      Following the reviewers’ suggestions, we will update figures and use Figure Supplements to make the main figures more concise and to simplify the overall presentation.

      Regarding testable predictions, one prediction would be that B7 TCR will exhibit weaker catch bond behavior than A6. This is an important prediction because the two TCRs targeting the same pMHC have similar structures and are functionally similar in conventional assays. This prediction can be tested by single-molecule optical tweezers experiments. We also predict the A6 TCR may perform better when the number of pMHC molecules presented are limited, analogous to our recent experiments on different TCRs, Akitsu et al., Sci. Adv. (2024).

      Another testable prediction for the conservation of the basic allostery mechanism is to test the Cβ FG-loop deletion mutant located at the hinge region of the β chain, yet its deletion severely impairs the catch bond formation. These predictions will be mentioned and discussed in the updated manuscript.

      Reviewer 2:

      In this work, Chang-Gonzalez and coworkers follow up on an earlier study on the force-dependence of peptide recognition by a T-cell receptor using all-atom molecular dynamics simulations. In this study, they compare the results of pulling on a TCR-pMHC complex between two different TCRs with the same peptide. A goal of the paper is to determine whether the newly studied B7 TCR has the same load-dependent behavior mechanism shown in the earlier study for A6 TCR. The primary result is that while the unloaded interaction strength is similar, A6 exhibits more force stabilization.

      This is a detailed study, and establishing the difference between these two systems with and without applied force may establish them as a good reference setup for others who want to study mechanobiological processes if the data were made available, and could give additional molecular details for T-Cell-specialists. As written, the paper contains an overwhelming amount of details and it is difficult (for me) to ascertain which parts to focus on and which results point to the overall take-away messages they wish to convey.

      As mentioned above and as the reviewer correctly pointed out, the condensed appearance of this manuscript arose largely because we intended it to be a Research Advances article as a short follow up study of our previous paper on A6 TCR published in eLife. Most of the analysis scripts for the A6 TCR study are already available on Github. We will additionally deposit sample structures and simulation scripts for the B7 TCR. Trajectory will be provided upon request given their large size.

      Regarding the focus issue, it is in part due to the complex nature of the problem, which required simulations under different conditions and multi-faceted analyses. Concisely presenting the complex analyses also has been a challenge in our previous papers on TCR simulations (Hwang et al., PNAS 2020; Chang-Gonzalez et al., eLife, 2024 – both are cited in our manuscript). With updated figures and texts, we expect that the presentation will be a lot clearer. But even in the present form, the reviewer points out the main take-away message well: “The primary result is that while the unloaded interaction strength is similar, A6 exhibits more force stabilization.

      Detailed comments:

      (1) In Table 1 - are the values of the extension column the deviation from the average length at zero force (that is what I would term extension) or is it the distance between anchor points (which is what I would assume based on the large values. If the latter, I suggest changing the heading, and then also reporting the average extension with an asterisk indicating no extensional restraints were applied for B7-0, or just listing 0 load in the load column. Standard deviation in this value can also be reported. If it is an extension as I would define it, then I think B7-0 should indicate extension = 0+/- something.

      The distance between anchor points could also be labeled in Figure 1A.

      “Extension” is the distance between anchor points (blue spheres at the ends of the added strands in Fig. 1A). While its meaning should be clear in the section “Laddered extensions” in MD simulation protocol, at first glance it may lead to confusion. In a strict sense, use of “extension” for the distance is a misnomer, but we have used it in our previous two papers (Hwang et al., PNAS 2020; Chang-Gonzalez et al., eLife, 2024), so we prefer to keep it for consistency. Instead, in the caption of Table 1, we will explain its meaning, and also explicitly label it in Fig. 1A, as the reviewer suggested.

      Please also note that the no-load case B7<sup>0</sup> does not have a particular extension that yields zero load on average. It would in fact be very difficult to find such an extension (distance between two anchor points). To simulate the system without load, we separately built a TCR-pMHC complex without added linkers, and held the distal part of pMHC with weak harmonic restraints (explained in sections “Structure preparation” and “Systems without load”). In this way, no external force is applied to TCR as it moves relative to pMHC. We will clarify this when introducing B7<sup>0</sup> in the Results section.

      (2) As in the previous paper, the authors apply ”constant force” by scanning to find a particular bond distance at which a desired force is selected, rather than simply applying a constant force. I find this approach less desirable unless there is experimental evidence suggesting the pMHC and TCR were forced to be a particular distance apart when forces are applied. It is relatively trivial to apply constant forces, so in general, I would suggest this would have been a reasonable comparison. Line 243-245 speculates that there is a difference in catch bonding behavior that could be inferred because lower force occurs at larger extensions, but I do not believe this hypothesis can be fully justified and could be due to other differences in the complex.

      There is indeed experimental evidence that the TCR-pMHC complex operates under constant separation. The spacing between a T-cell and an antigen-presenting cell is maintained by adhesion molecules such as the CD2CD58 pair, as explained in our paper on the A6 TCR, (Chang-Gonzalez et al., eLife, 2024; please see the bottom paragraph on page 4 of the paper). In in vitro single-molecule experiments, pulling to a fixed separation and holding is also commonly done. Detailed comparison between constant extension vs. constant force simulations is definitely a subject of our future study. We will clarify these points when explaining about the constant extension (or separation).

      Regarding line 243–245, we agree with the reviewer that without further tests, lower forces at larger extensions per se cannot be an indicator that B7 forms a weaker catch bond. But with additional insight, it does have an indirect relevance. In addition to fewer TCR-pMHC contacts (Fig. 1C of our manuscript), the intra-TCR contacts are also reduced compared to those of A6 (Fig. 1D vs. Chang-Gonzalez et al., eLife, 2024, Fig. 8A,B, first column; reproduced in the figure in our response to reviewer 3 below). This shows that the B7 TCR forms a looser complex with pMHC compared to A6. With its higher compliance, the B7 TCR-pMHC complex needs to be under a greater extension than A6 to apply comparable levels of force, and it would be more difficult to achieve load-induced stabilization of the TCR-pMHC interface, hence a weaker catch bond. We will add this point when explaining the weaker catch bond behavior of B7.

      (3) On a related note, the authors do not refer to or consider other works using MD to study force-stabilized interactions (e.g. for catch bonding systems), e.g. these cases where constant force is applied and enhanced sampling techniques are used to assess the impact of that applied force: https://www.cell.com/biophysj/fulltext/S0006-3495(23)00341-7, https://www.biorxiv.org/content/10.1101/2024.10.10.617580v1. I was also surprised not to see this paper on catch bonding in pMHC-TCR referred to, which also includes some MD simulations: https://www.nature.com/articles/s41467-023-38267-1

      We thank the reviewer for bringing the three papers to our attention, which are:

      (1) Languin-Cattoën, Sterpone, and Stirnemann, Biophys. J. 122:2744 (2023): About bacterial adhesion protein FimH.

      (2) Peña Ccoa, et al., bioRxiv (2024): About actin binding protein vinculin.

      (3) Choi et al., Nat. Comm. 14:2616 (2023): About a mathematical model of the TCR catch bond.

      Catch bond mechanisms of FimH and vinculin are different from that of TCR in that FimH and vinculin have relatively well-defined weak- and strong-binding states where there are corresponding crystal structures. Availability of the end-state structures enable using simulation approaches such as enhanced sampling of individual states and studying the transition between the two states. In contrast, TCR does not have any structurally well-defined weakor strong-binding states, which requires a different approach. As demonstrated in our current manuscript as well as in our previous two papers (Hwang et al., PNAS 2020; Chang-Gonzalez et al., eLife, 2024), our microsecond-long simulations of the complex under realistic pN-level loads and a combination of analysis methods are effective for elucidating the catch bond mechanism of TCR. In the revised manuscript, we will cite the two papers, to compare the TCR catch bond mechanism with those of FimH and vinculin, which will offer a broader perspective.

      The third paper (Choi, 2023) proposes a mathematical model to analyze extensive sets of data, and also perform new experiments and additional simulations. Of note, their model assumptions are based mainly on the steered MD (SMD) simulation in their previous paper (Wu, et al., Mol. Cell. 73:1015, 2019). In their model, formation of a catch bond (called catch-slip bond in Choi’s paper) requires partial unfolding of MHC and tilting of the TCR-pMHC interface. While further studies are needed to find whether those changes are indeed required, even so, the question remains regarding how the complex in the fully folded state can bear load and enter such a state in the first place. Our current and previous simulation studies suggest a mechanism by which ligand- and load-dependent responses occur as the first obligatory step of catch bond formation, after which partial unfolding and/or extensive conformational transitions may occur, as described in our recent paper (Akitsu et al., Sci. Adv., 2024). In the revised manuscript, we will cite Wu’s paper and briefly explain the above.

      (4) The authors should make at least the input files for their system available in a public place (github, zenodo) so that the systems are a more useful reference system as mentioned above. The authors do not have a data availability statement, which I believe is required.

      As mentioned above, we will make sample input files and coordinates available on Github. Data availability statement will be added.

      Reviewer 3:

      Summary:

      The paper by Chang-Gonzalez et al. is a molecular dynamics (MD) simulation study of the dynamic recognition (load-induced catch bond) by the T cell receptor (TCR) of the complex of peptide antigen (p) and the major histocompatibility complex (pMHC) protein. The methods and simulation protocols are essentially identical to those employed in a previous study by the same group (Chang-Gonzalez et al., eLife 2024). In the current manuscript, the authors compare the binding of the same pMHC to two different TCRs, B7 and A6 which was investigated in the previous paper. While the binding is more stable for both TCRs under load (of about 10-15 pN) than in the absence of load, the main difference is that, with the current MD sampling, B7 shows a smaller amount of stable contacts with the pMHC than A6.

      Strengths:

      The topic is interesting because of the (potential) relevance of mechanosensing in biological processes including cellular immunology.

      Weaknesses:

      The study is incomplete because the claims are based on a single 1000-ns simulation at each value of the load and thus some of the results might be marred by insufficient sampling, i.e., statistical error. After the first 600 ns, the higher load of B7high than B7low is due mainly to the simulation segment from about 900 ns to 1000 ns (Figure 1D). Thus, the difference in the average value of the load is within their standard deviation (9 +/- 4 pN for B7low and 14.5 +/- 7.2 for B7high, Table 1). Even more strikingly, Figure 3E shows a lack of convergence in the time series of the distance between the V-module and pMHC, particularly for B70 (left panel, yellow) and B7low (right panel, orange). More and longer simulations are required to obtain a statistically relevant sampling of the relative position and orientation of the V-module and pMHC.

      The reviewer uses data points during the last 100 ns to raise an issue with sampling. But since we are using realistic pN range forces, force fluctuates more slowly. In fact, in our simulation of B7<sup>high</sup>, while the force peaks near 35 pN at 500 ns (Fig. 1D of our manuscript; reproduced as panels C and D below), the contact heat map shows no noticeable changes around 500 ns (Fig. 2C of our manuscript). Thus, a wider time window must be considered rather than focusing on instantaneous force.

      We believe the reviewer’s concern about sampling arose also due to a lack of clear explanation. Author response image 1 below contains panels from our earlier eLife paper on the A6 TCR. Panels A and B are from Fig. 8 of the A6 paper, and panels C and D are from Fig. 1D of our present manuscript. The high-load simulations in both cases (outlined circles) fluctuate widely in force so that one might argue that sampling was insufficient. However, unless one is interested in finding the precise value of force for a given extension, sampling in our simulations was reasonable enough to distinguish between high- and low-force behaviors. To support this, we show panel E below, which is from Appendix 3–Fig. 1 of our A6 paper. Added to this panel are the average forces and standard deviations of B7<sup>low</sup> and B7<sup>high</sup> from Table 1 of our manuscript (red squares). Please note that all of the data were measured after 500 ns. Except for Y8A<sup>low</sup> and dFG<sup>low</sup> of A6 (explained below), all of the data points lie on nearly a straight line.

      Author response image 1.

      Thermodynamically, the force and position of the restraint (blue spheres in Fig. 1A of our manuscript) form a pair of generalized force and the corresponding spatial variable in equilibrium at temperature 300 K, which is akin to the pressure P and volume V of an ideal gas. If V is fixed, P fluctuates. Denoting the average and std of pressure as ⟨P⟩ and ∆P, respectively, Burgess showed that ∆P/P⟩ is a constant (Eq. 5 of Burgess, Phys. Lett. A, 44:37; 1973). In the case of the TCRαβ-pMHC system, although individual atoms are not ideal gases, since their motion leads to the fluctuation in force on the restraints, the situation is analogous to the case where pressure arises from individual ideal gas molecules hitting the confining wall as the restraint. Thus, the near-linear behavior in panel E above is a consequence of the system being many-bodied and at constant temperature. The linearity is also an indirect indicator that sampling of force was reasonable. The fact that A6 and B7 data show a common linear profile further demonstrates the consistency in our force measurement. That said, the B7 data points (red in panel E) are elevated slightly above nearby A6 data points. This is consistent with B7 forming an overall weaker complex, both at the TCR-pMHC interface (panels A vs. C) and within intra-TCR interfaces (panels B vs. D), which can be seen by the wider ranges of color bars in panels A and B for A6 compared to panels C and D for B7.

      About the two outliers of A6, Y8A<sup>low</sup> is for an antagonist peptide and dFG<sup>low</sup> is the Cβ FG-loop deletion mutant. Interestingly, both cases had reduced numbers of contacts with pMHC, which likely caused a wider conformational motion, hence greater fluctuation in force.

      A similar argument applies to Fig. 3E of our manuscript. If precise values of the V-module to pMHC distance were needed, longer or duplicate simulations would be necessary, however, Fig. 3E as it currently stands clearly shows that B7<sup>high</sup> maintains more stable interface compared to B7<sup>low</sup>, which is consistent with all other measures we used, such as Fig. 3B (Hamming distance), Fig. 3C (buried surface area), and Fig. 4A–E (Vα-Vβ motion and CDR3 distance). They are also consistent with our simulations of A6.

      Thus, rather than relying on peculiarities of individual trajectories, we analyze data in multiple ways and draw conclusions based on features that are consistent across different simulations. Please also note that reviewer 1 mentioned that our conclusions are “generally well supported by data.”

      We will update our manuscript to concisely explain the above and also will add Panel E above as a supplement of Fig. 1.

      It is not clear why ”a 10 A distance restraint between alphaT218 and betaA259 was applied” (section MD simulation protocol, page 9).

      αT218 and βA259 are the residues attached to a leucine-zipper handle in in vitro optical trap experiments (Das, et al., PNAS 2015). In T cells, those residues also connect to transmembrane helices. Author response image 2 is a model of N15 TCR used in experiments in Das’ paper, constructed based on PDB 1NFD. Blue spheres represent Cα atoms corresponding to αT218 and βA259 of B7 TCR. Their distance is 6.7 ˚A. The 10-˚A distance restraint in simulation was applied to mimic the presence of the leucine zipper that prevents excessive separation of the added strands. The distance restraint is a flat-bottom harmonic potential which is activated only when the distance between the two atoms exceeds 10 ˚A, which we did not clarify in our original manuscript. The same restraint was used in our previous studies on JM22 and A6 TCRs.

      We will add the figure as a supplement of Fig. 1, cite Das’ paper, and also update description of the distance restraint in the MD simulation protocol section.

      Author response image 2.

    1. Author response:

      Public Reviews:

      We thank the reviewers for their overall positive assessments and constructive feedback

      Reviewer #1 (Public Review):

      Summary:

      The study explored the biomechanics of kangaroo hopping across both speed and animal size to try and explain the unique and remarkable energetics of kangaroo locomotion.

      Strengths:

      The study brings kangaroo locomotion biomechanics into the 21st century. It is a remarkably difficult project to accomplish. There is excellent attention to detail, supported by clear writing and figures.

      Weaknesses:

      The authors oversell their findings, but the mystery still persists.

      The manuscript lacks a big-picture summary with pointers to how one might resolve the big question.

      General Comments

      This is a very impressive tour de force by an all-star collaborative team of researchers. The study represents a tremendous leap forward (pun intended) in terms of our understanding of kangaroo locomotion. Some might wonder why such an unusual species is of much interest. But, in my opinion, the classic study by Dawson and Taylor in 1973 of kangaroos launched the modern era of running biomechanics/energetics and applies to varying degrees to all animals that use bouncing gaits (running, trotting, galloping and of course hopping). The puzzling metabolic energetics findings of Dawson & Taylor (little if any increase in metabolic power despite increasing forward speed) remain a giant unsolved problem in comparative locomotor biomechanics and energetics. It is our "dark matter problem".

      Thank you for the kind words

      This study is certainly a hop towards solving the problem. But, the title of the paper overpromises and the authors present little attempt to provide an overview of the remaining big issues.

      We will modify the title to reflect this comment.  

      The study clearly shows that the ankle and to a lesser extent the mtp joint are where the action is. They clearly show in great detail by how much and by what means the ankle joint tendons experience increased stress at faster forward speeds.

      Since these were zoo animals, direct measures were not feasible, but the conclusion that the tendons are storing and returning more elastic energy per hop at faster speeds is solid.

      The conclusion that net muscle work per hop changes little from slow to fast forward speeds is also solid.

      Doing less muscle work can only be good if one is trying to minimize metabolic energy consumption. However, to achieve greater tendon stresses, there must be greater muscle forces. Unless one is willing to reject the premise of the cost of generating force hypothesis, that is an important issue to confront.

      Further, the present data support the Kram & Dawson finding of decreased contact times at faster forward speeds. Kram & Taylor and subsequent applications of (and challenges to) their approach supports the idea that shorter contact times (tc) require recruiting more expensive muscle fibers and hence greater metabolic costs. Therefore, I think that it is incumbent on the present authors to clarify that this study has still not tied up the metabolic energetics across speed problems and placed a bow atop the package.

      Fortunately, I am confident that the impressive collective brain power that comprises this author list can craft a paragraph or two that summarizes these ideas and points out how the group is now uniquely and enviably poised to explore the problem more using a dynamic SIMM model that incorporates muscle energetics (perhaps ala' Umberger et al.). Or perhaps they have other ideas about how they can really solve the problem.

      You have raised important points, thank you for this feedback. We will add a paragraph discussing the limitations of our study and ensure the revised manuscript makes it clear which mysteries remain. We intend to address muscle forces, contact time, and energetics in future work when we have implemented all hindlimb muscles within the musculoskeletal model.  

      I have a few issues with the other half of this study (i.e. animal size effects). I would enjoy reading a new paragraph by these authors in the Discussion that considers the evolutionary origins and implications of such small safety factors. Surely, it would need to be speculative, but that's OK.

      We will integrate this into the discussion.

      Reviewer #2 (Public Review):

      Summary

      This is a fascinating topic that has intrigued scientists for decades. I applaud the authors for trying to tackle this enigma. In this manuscript, the authors primarily measured hopping biomechanics data from kangaroos and performed inverse dynamics.

      While these biomechanical analyses were thorough and impressively incorporated collected anatomical data and an Opensim model, I'm afraid that they did not satisfactorily address how kangaroos can hop faster and not consume more metabolic energy, unique from other animals.

      Noticeably, the authors did not collect metabolic data nor did they model metabolic rates using their modelling framework. Instead, they performed a somewhat traditional inverse dynamics analysis from multiple animals hopping at a self-selected speed.

      We aimed to provide a joint-level explanation, but we will address the limitations of not modelling the energy consumers themselves (the skeletal muscles) in the revised manuscript. We plan to expand upon muscle level energetics in the future with a more detailed MSK model.

      Within these analyses, the authors largely focused on ankle EMA, discussing its potential importance (because it affects tendon stress, which affects tendon strain energy, which affects muscle mechanics) on the metabolic cost of hopping. However, EMA was roughly estimated (CoP was fixed to the foot, not measured)…

      As noted in our methods, EMA was not calculated from a fixed centre of pressure (CoP). We did fix the medial-lateral position, owing to the fact that both feet contacted the force plate together, but the anteroposterior movement of the CoP was recorded by the force plate and thus allowed to move. We report the movement (or lack of movement) in our results. The anterior-posterior axis is the most relevant to lengthening or shortening the distance of the ‘out-lever’ R, and thereby EMA.

      It is necessary to assume fixed medial-lateral position because a single force trace and CoP is recorded when two feet land on the force plate. The medial-lateral forces on each foot cancel out so there is no overall medial-lateral movement if the forces are symmetrical (e.g. if the kangaroo is hopping in a straight path and one foot is not in front of the other). We only used symmetrical trials so that the anterior-posterior movement of the CoP would be reliable.

      and did not detectibly associate with hopping speed (see results).

      Yet, the authors interpret their EMA findings as though it systematically related with speed to explain their theory on how metabolic cost is unique in kangaroos vs. other animals.

      Indeed, the relationship between R and speed (and therefore EMA and speed) was not significant. However, the significant change in ankle height with speed, combined with no systematic change in COP at midstance, demonstrates that R would get longer at faster speeds. If we consider the nonsignificant relationship between R and speed to indicate that there is no change in R, then these two results conflict. We could not find a flaw in our methods, so instead concluded that the nonsignificant relationship between R and speed may be due to a small change in R being undetectable in our data. Taking both results into account, we think it is more likely that there is a non-detectable change in R, rather than no change in R with speed, but we presented both results for transparency.

      These speed vs. biomechanics relationships were limited by comparisons across different animals hopping at different speeds and could have been strengthened using repeated measures design.

      There is significant variation in speed within individuals, not just between individuals. The preferred speed of kangaroos is 2-4.5 m/s, but most individuals show a wide range within this. Eight of our 16 kangaroos had a maximum speed that was between 1-2m/s faster than their slowest trial. Repeated measures of these eight individuals comprises 78 out of the 100 trials.

      It would be ideal to collect data across the full range of speeds for all individuals, but it is not feasible in this type of experimental setting. Interference such as chasing is dangerous to kangaroos as they are prone to strong adverse reactions to stress.

      There are also multiple inconsistencies between the authors' theory on how mechanics affect energetics and the cited literature, which leaves me somewhat confused and wanting more clarification and information on how mechanics and energetics relate.

      We will ensure that this is clearer in the revised manuscript.

      My apologies for the less-than-favorable review, I think that this is a neat biomechanics study - but am unsure if it adds much to the literature on the topic of kangaroo hopping energetics in its current form.

      Reviewer #3 (Public Review):

      Summary:

      The goal of this study is to understand how, unlike other mammals, kangaroos are able to increase hopping speed without a concomitant increase in metabolic cost. They use a biomechancial analysis of kangaroo hopping data across a range of speeds to investigate how posture, effective mechanical advantage, and tendon stress vary with speed and mass. The main finding is that a change in posture leads to increasing effective mechanical advantage with speed, which ultimately increases tendon elastic energy storage and returns via greater tendon strain. Thus kangaroos may be able to conserve energy with increasing speed by flexing more, which increases tendon strain.

      Strengths:

      The approach and effort invested into collecting this valuable dataset of kangaroo locomotion is impressive. The dataset alone is a valuable contribution.

      Thank you!

      Weaknesses:

      Despite these strengths, I have concerns regarding the strength of the results and the overall clarity of the paper and methods used (which likely influences how convincingly the main results come across).

      (1) The paper seems to hinge on the finding that EMA decreases with increasing speed and that this contributes significantly to greater tendon strain estimated with increasing speed. It is very difficult to be convinced by this result for a number of reasons:

      • It appears that kangaroos hopped at their preferred speed. Thus the variability observed is across individuals not within. Is this large enough of a range (either within or across subjects) to make conclusions about the effect of speed, without results being susceptible to differences between subjects?

      Apologies, this was not clear in the manuscript. Kangaroos hopping at their preferred speed means we did not chase or startle them into high speeds to comply with ethics and enclosure limitations. Thus we did not record a wide range of speed within the bounds of what kangaroos are capable of (up to 12 m/s), but for the range we did measure (~2-4.5 m/s), there is variation hopping speed within each individual kangaroo. Out of 16 individuals, eight individuals had a difference of 1-2m/s between their slowest and fastest trials, and these kangaroos accounted for 78 out of 100 trials. Of the remainder, six individuals had three for fewer trials each, and two individual had highly repeatable speeds (3 out of 4, and 6 out of 7 trials were within 0.5 m/s). We will ensure this is clear in the revised manuscript.

      In the literature cited, what was the range of speeds measured, and was it within or between subjects?

      For other literature, to our knowledge the highest speed measured is ~9.5m/s (see supplementary Fig1b) and there were multiple measures for several individuals (see methods Kram & Dawson 1998).

      • Assuming that there is a compelling relationship between EMA and velocity, how reasonable is it to extrapolate to the conclusion that this increases tendon strain and ultimately saves metabolic cost?

      They correlate EMA with tendon strain, but this would still not suggest a causal relationship (incidentally the p-value for the correlation is not reported).

      We will add supporting literature on the relationship between metabolic cost and tendon stress (or strain), to elaborate on why the correlation between EMA and stress is important.

      Tendon strain could be increasing with ground reaction force, independent of EMA.

      Even if there is a correlation between strain and EMA, is it not a mathematical necessity in their model that all else being equal, tendon stress will increase as ema decreases? I may be missing something, but nonetheless, it would be helpful for the authors to clarify the strength of the evidence supporting their conclusions.

      Yes, GRF also contributes to the increase in tendon stress in the mechanism we propose. We have illustrated this in Fig 6, however we will make this clearer in the revised discussion.

      • The statistical approach is not well-described. It is not clear what the form of the statistical model used was and whether the analysis treated each trial individually or grouped trials by the kangaroo. There is also no mention of how many trials per kangaroo, or the range of speeds (or masses) tested.

      The methods include the statistical model with the variables that we used, as well as the kangaroo masses (13.7 to 26.6 kg, mean: 20.9 ± 3.4 kg). We will move the range of speeds from the supplementary material to the results or figure captions. We will add information on the number of trials per kangaroo to the methods.

      We did not group the data e.g. by using an average speed per individual for all their trials, or by comparing fast to slow groups (this was for display purposes in our figures, which we will make clearer in the methods).

      Related to this, there is no mention of how different speeds were obtained. It seems that kangaroos hopped at a self-selected pace, thus it appears that not much variation was observed. I appreciate the difficulty of conducting these experiments in a controlled manner, but this doesn't exempt the authors from providing the details of their approach.

      • Some figures (Figure 2 for example) present means for one of three speeds, yet the speeds are not reported (except in the legend) nor how these bins were determined, nor how many trials or kangaroos fit in each bin. A similar comment applies to the mass categories. It would be more convincing if the authors plotted the main metrics vs. speed to illustrate the significant trends they are reporting.

      Thank you for this comment. The bins are used only for display purposes and not within the analysis. In the revised manuscript, we will ensure this is clear.

      (2) The significance of the effects of mass is not clear. The introduction and abstract suggest that the paper is focused on the effect of speed, yet the effects of mass are reported throughout as well, without a clear understanding of the significance. This weakness is further exaggerated by the fact that the details of the subject masses are not reported.

      Indeed, the primary aim of our study was to explore the influence of speed, given the uncoupling of energy from hopping speed in kangaroos. We included mass to ensure that the effects of speed were not driven by body mass (i.e.: that larger kangaroos hopped faster).  

      (3) The paper needs to be significantly re-written to better incorporate the methods into the results section. Since the results come before the methods, some of the methods must necessarily be described such that the study can be understood at some level without turning to the dedicated methods section. As written, it is very difficult to understand the basis of the approach, analysis, and metrics without turning to the methods.

      We agree, and in the revised manuscript will incorporate some of the methodological details within the results.

      Author response image 1.

    1. Author response:

      Public Reviews:

      Reviewer #1 (Public review):

      Summary:

      Pradhan et al investigated the potential gustatory mechanisms that allow flies to detect cholesterol. They found that flies are indifferent to low cholesterol and avoid high cholesterol. They further showed that the ionotropic receptors Ir7g, Ir51b, and Ir56d are important for the cholesterol sensitivity in bitter neurons. The figures are clear and the behavior result is interesting. However, I have several major comments, especially on the discrepancy of the expression of these Irs with other lab published results, and the confusing finding that the same receptors (Ir7g, Ir51b) have been implicated in the detection of various seemingly unrelated compounds.

      Strengths:

      The results are very well presented, the figures are clear and well-made, text is easy to follow.

      Weaknesses:

      (1) Regarding the expression of Ir56d. The reported Ir56d expression pattern contradicts multiple previous studies (Brown et al., 2021 eLife, Figure 6a-c; Sanchez-Alcaniz et al., 2017 Nature Communications, Figure 4e-h; Koh et al., 2014 Neuron, Figure 3b). These studies, using three different driver lines, consistently showed Ir56d expression in sweet-sensing neurons and taste peg neurons. Importantly, Sanchez-Alcaniz et al. demonstrated that Ir56d is not expressed in Gr66a-expressing (bitter) neurons. This discrepancy is critical since Ir56d is identified as the key subunit for cholesterol detection in bitter neurons, and misexpression of Ir7g and Ir51b together is insufficient to confer cholesterol sensitivity (Fig.4b,d). Which Ir56d-GAL4 (and Gr66a-I-GFP) line was used in this study? Is there additional evidence (scRNA sequencing, in-situ hybridization, or immunostaining) supporting Ir56d expression in bitter neurons?

      We agree that the expression pattern of Ir56d diverges from two prior reports . The studies by Brown et al. and Koh et al. employed the same Ir56d-GAL4 driver line, which exhibited expression in sweet-sensing gustatory receptor neurons (GRNs) and taste peg neurons, but not bitter GRNs (the Sanchez-Alcaniz et al. paper did not use an Ir56d-Gal4).

      In our study, we used a Ir56d-GAL4 driver line (KDRC:2307) and the Gr66a-I-GFP reporter line (Weiss et al., 2011 Neuron). This is a crucial distinction, as differences in the regulatory regions used to generate different driver lines are well known to underlie differences in expression patterns. Our double-labeling experiments revealed co-expression of Ir56d with Gr66a-positive bitter GRNs specifically within the S6 and S7 sensilla—types previously shown to exhibit strong electrophysiological responses to cholesterol (Figure 2—figure supplement 1F).

      We believe this observation is biologically significant and consistent with our functional data. Specifically, targeted expression of Ir56d in bitter neurons using the Gr33a-GAL4 was sufficient to rescue cholesterol avoidance behavior in Ir56d<sup>1</sup> mutants (Figure 3G). These results demonstrate that Ir56d plays a functional role in bitter GRNs for cholesterol detection. The convergence of genetic, behavioral, and electrophysiological data presented in our study provides compelling support for this previously unappreciated expression pattern and function of Ir56d.

      (2) Ir51b has previously been implicated in detecting nitrogenous waste (Dhakal 2021), lactic acid (Pradhan 2024), and amino acids (Aryal 2022), all by the same lab. Additionally, both Ir7g and Ir51b have been implicated in detecting cantharidin, an insect-secreted compound that flies may or may not encounter in the wild, by the same lab. Is Ir51b proposed to be a specific receptor for these chemically distinct compounds or a general multimodal receptor for aversive stimuli? Unlike other multimodal bitter receptors, the expression level of Ir51b is rather low and it's unclear which subset of GRNs express this receptor. The chemical diversity among nitrogenous waste, amino acids, lactic acid, cantharidin, and cholesterol raises questions about the specificity of these receptors and warrants further investigation and at a minimum discussion in this paper. Given the wide and seemingly unrelated sensitivity of Ir51b and Ir7g to these compounds I'm leaning towards the hypothesis that at least some of these is non-specific and ecologically irrelevant without further supporting evidence from the authors.

      While it is true that IR51b and IR7g are responsive to a range of compounds, they share chemical features such as nitrogen-containing groups, hydrophobicity, or amphipathic structures suggesting that recognition of these chemicals may be mediated by the same or overlapping domains within the receptor complexes. These features could facilitate binding to a structurally diverse yet chemically related groups of aversive ligands.

      In the case of cholesterol, while its sterol ring system is distinct from the other compounds, it shares hydrophobic and amphipathic properties that may enable interaction with these receptors via similar structural motifs. Importantly, our data demonstrates that Ir51b and Ir7g are necessary but not sufficient on their own to confer cholesterol sensitivity, indicating that additional co-factors or receptor subunits are required for full functionality (Figure 4B, D). Furthermore, our dose-response analysis (Figure 3F) shows that Ir7g is particularly important at higher cholesterol concentrations, supporting the idea of graded sensitivity rather than indiscriminate activation. This suggests that these receptors may have evolved to recognize cholesterol and its analogs (e.g., phytosterols such as stigmasterol, yet to be tested), which are naturally found in the fly’s diet (e.g., yeast and plant-derived matter), as ecologically relevant cues signaling microbial contamination, lipid imbalance, or dietary overconsumption.

      We acknowledge the reviewer’s concern regarding the relatively low expression levels of Ir51b and Ir7g. However, we note that low transcript abundance does not necessarily equate to diminished physiological relevance. Finally, we agree that the chemical diversity of ligands associated with Ir51b and Ir7g warrants deeper investigation, particularly through structure-function studies aimed at identifying ligand-binding domains and receptor-ligand interactions at atomic resolution.

      (3) The Benton lab Ir7g-GAL4 reporter shows no expression in adults. Additionally, two independent labellar RNA sequencing studies (Dweck, 2021 eLife; Bontonou et al., 2024 Nature Communications) failed to detect Ir7g expression in the labellum. This contradicts the authors' previous RT-PCR results (Pradhan 2024 Fig. S4, Journal of Hazardous Materials) showing Ir7g expression in the labellum. Additionally the Benton and Carlson lab Ir51b-GAL4 reporters show no expression in adults as well. Please address these inconsistencies.

      With respect to Ir7g, we acknowledge that the Ir7g-GAL4 reporter line from the Benton lab does not exhibit detectable expression in adult labella. Furthermore, two independent transcriptomic studies—Dweck et al., 2021 (eLife) and Bontonou et al., 2024 (Nature Communications) also did not detect Ir7g transcripts in bulk RNA-seq datasets derived from adult labella. However, our previously published RT-PCR data (Pradhan et al., 2024, Journal of Hazardous Materials, Fig. S4) revealed Ir7g expression in labellar tissue, albeit at low levels. Our RT-PCR includes an internal control (tubulin) with the same reaction tube with control and the Ir7g mutant as a negative control. Therefore, we stand behind the findings that Ir7g is expressed in the labellum.

      We would like to point out that RT-PCR is more sensitive and better-suited to detect low-abundance transcripts than bulk RNA-seq, which may fail to capture transcripts due to limitations in depth of coverage. Moreover, immunohistochemistry can have limitations in detecting very low expression levels. Costa et al. 2013 (Translational lung cancer research) states that “RNA-Seq technique will not likely replace current RT-PCR methods, but will be complementary depending on the needs and the resources as the results of the RNA-Seq will identify those genes that need to then be examined using RT-PCR methods”.

      Similarly, regarding Ir51b, while the GAL4 reporter lines from the Benton and Carlson labs do not show robust adult expression, our RT-PCR and functional data strongly support a role for Ir51b in labellar bitter GRNs. Specifically, Ir51b<sup>1</sup> mutants display electrophysiological deficits in response to cholesterol (Figure 2A–B), and these defects are rescued by expressing Ir51b in Gr33a-positive bitter neurons (Figure 3G), providing functional validation of the RT-PCR expression.

      (4) The premise that high cholesterol intake is harmful to flies, which makes sensory mechanisms for cholesterol avoidance necessary, is interesting but underdeveloped. Animal sensory systems typically evolve to detect ecologically relevant stimuli with dynamic ranges matching environmental conditions. Given that Drosophila primarily consume fruits and plant matter (which contain minimal cholesterol) rather than animal-derived foods (which contain higher cholesterol), the ecological relevance of cholesterol detection requires more thorough discussion. Furthermore, at high concentrations, chemicals often activate multiple receptors beyond those specifically evolved for their detection. If the cholesterol concentrations used in this study substantially exceed those encountered in the fly's natural diet, the observed responses may represent an epiphenomenon rather than an ecologically and ethologically relevant sensory mechanism. What is the cholesterol content in flies' diet and how does that compare to the concentrations used in this paper?

      Drosophila melanogaster cannot synthesize sterols de novo, and must acquire them from its diet. In natural environments, flies acquire sterols from fermenting fruit, decaying plant matter, and yeast, which contain trace amounts of phytosterols (e.g., stigmasterol, β-sitosterol) and ergosterol. While the exact sterol concentrations in these sources remain uncharacterized, our behavioral assays used concentrations (0.001–0.01% by weight) that align with the low levels expected in such nutrient-limited ecological niches.

      In our study, the cholesterol concentrations tested ranged from 0.001% to 0.1%, thereby spanning both the physiologically relevant and slightly elevated range. Importantly, avoidance behaviors and receptor activation were most prominent at 0.1% cholesterol. While it is true that high chemical concentrations may elicit off-target effects via broad receptor activation, our genetic and electrophysiological data indicate that the observed responses are mediated by specific ionotropic receptors (Ir51b, Ir7g, Ir56d) and not merely generalized chemical stress.

      Ecologically, elevated sterol levels may also signal conditions unsuitable for egg-laying or larval development. For example, high levels of cholesterol or other sterols may occur in substrates colonized by pathogenic microbes, decaying animal tissue, or in cases of abnormal microbial fermentation, which could represent a nutritional or microbial hazard. The avoidance of cholesterol may help signal the flies to avoid consuming decaying animal tissue. In this context, sensory detection of excessive cholesterol might serve as a protective function.

      Reviewer #2 (Public review):

      Summary:

      In Cholesterol Taste Avoidance in Drosophila melanogaster, Pradhan et al. used behavioral and electrophysiological assays to demonstrate that flies can: (1) detect cholesterol through a subset of bitter-sensing gustatory receptor neurons (GRNs) and (2) avoid consuming food with high cholesterol levels. Mechanistically, they identified five members of the IR family as necessary for cholesterol detection in GRNs and for the corresponding avoidance behavior. Ectopic expression experiments further suggested that Ir7g + Ir56d or Ir51b + Ir56d may function as tuning receptors for cholesterol detection, together with the Ir25a and Ir76b co-receptors.

      Strengths:

      The experimental design of this study was logical and straightforward. Leveraging their expertise in the Drosophila taste system, the research team identified the molecular and cellular basis of a previously unrecognized taste category, expanding our understanding of gustation. A key strength of the study was its combination of electrophysiological recordings with behavioral genetic experiments.

      Weaknesses:

      My primary concern with this study is the lack of a systematic survey of the IRs of interest in the labellum GRNs. Consequently, there is no direct evidence linking the expression of putative cholesterol IRs to the B GRNs in the S6 and S7 sensilla.

      Specifically, the authors need to demonstrate that the IR expression pattern explains cholesterol sensitivity in the B GRNs of S6 and S7 sensilla, but not in other sensilla. Instead of providing direct IR expression data for all candidate IRs (as shown for Ir56d in Figure 2-figure supplement 1F), the authors rely on citations from several studies (Lee, Poudel et al. 2018; Dhakal, Sang et al. 2021; Pradhan, Shrestha et al. 2024) to support their claim that Ir7g, Ir25a, Ir51b, and Ir76b are expressed in B GRNs (Lines 192-194). However, none of these studies provide GAL4 expression or in situ hybridization data to substantiate this claim.

      Without a comprehensive IR expression profile for GRNs across all taste sensilla, it is difficult to interpret the ectopic expression results observed in the B GRN of the I9 sensillum or the A GRN of the L-sensillum (Figure 4). It remains equally plausible that other tuning IRs-beyond the co-receptor Ir25a and Ir76b-could interact with the ectopically expressed IRs to confer cholesterol sensitivity, rather than the proposed Ir7g + Ir56d or Ir51b + Ir56d combinations.

      We provide electrophysiological data demonstrating that the S6 and S7 sensilla respond to cholesterol (Figure 1D). This finding is consistent with the hypothesis that these sensilla harbor the complete receptor complexes necessary for cholesterol detection. In our electrophysiological recordings, only those bitter GRNs that co-express Ir56d along with either Ir7g or Ir51b generate action potentials in response to cholesterol. Other S-type sensilla lacking one or more of these subunits remain unresponsive, reinforcing the idea that these components are necessary for receptor function and sensory coding of cholesterol. Moreover, in the cholesterol-insensitive I9 sensillum (based on our mapping results using electrophysiology), co-expression of either Ir7g + Ir56d or Ir51b + Ir56d conferred de novo cholesterol sensitivity (Figure 4B). Importantly, no cholesterol response was observed when any of these IRs was expressed alone or when Ir7g + Ir51b were co-expressed without Ir56d. These findings strongly argue against the possibility that endogenous tuning IRs in I9 sensilla (e.g., Ir25a, Ir76b) are sufficient to generate cholesterol responsiveness.

      Furthermore, based on the literature, Ir25a and Ir76b are endogenously expressed in I- and L-type sensilla. Thus, their presence alone is insufficient for cholesterol responsiveness. These data support the model that cholesterol sensitivity depends on a specific, multi-subunit receptor complex (e.g., Ir7g + Ir25a + Ir56d + Ir76b or Ir51b + Ir25a + Ir56d + Ir76b).

      In conclusion, while we acknowledge that our data do not provide a full anatomical map of IR expression across all sensilla, our results strongly support the idea that cholesterol sensitivity in S6 and S7 sensilla arises from specific combinations of IRs expressed in the B GRNs.

      Reviewer #3 (Public review):

      Summary:

      Whether and how animals can taste cholesterol is not well understood. The study provides evidence that 1) cholesterol activates a subset of bitter-sensing gustatory receptor neurons (GRNs) in the fly labellum, but not other types of GRNs, 2) flies show aversion to high concentrations of cholesterol, and this is mediated by bitter GRNs, and 3) cholesterol avoidance depends on a specific set of ionotropic receptor (IR) subunits acting in bitter GRNs. The claims of the study are supported by electrophysiological recordings, genetic manipulations, and behavioral readouts.

      Strengths:

      Cholesterol taste has not been well studied, and the paper provides new insight into this question. The authors took a comprehensive and rigorous approach in several different parts of the paper, including screening the responses of all 31 labellar sensilla, screening a large panel of receptor mutants, and performing misexpression experiments with nearly every combination of the 5 IRs identified. The effects of the genetic manipulations are very clear and the results of electrophysiological and behavioral studies match nicely, for the most part. The appropriate controls are performed for all genetic manipulations.

      Weaknesses:

      The weaknesses of the study, described below, are relatively minor and do not detract from the main conclusions of the paper.

      (1) The paper does not state what concentrations of cholesterol are present in Drosophila's natural food sources. Are the authors testing concentrations that are ethologically Drosophila melanogaster primarily feeds on fermenting fruits and associated microbial communities, especially yeast, which serve as major sources of dietary sterols. These natural food sources are known to contain phytosterols such as stigmasterol and β-sitosterol. One study quantified phytosterols (e.g., stigmasterol, sitosterol) in fruits, reporting concentrations between 1.6–32.6 mg/100 g edible portion (~0.0016–0.0326% wet weight) (Han et al 2008). The range we tested falls within this range. Additionally, ergosterol, the principal sterol in yeast and a structural analog of cholesterol, is present at levels of about 0.005% to 0.02% in yeast-rich environments.

      To ensure physiological relevance, we designed our behavioral assays to include a broad concentration range of cholesterol, from 10<sup>-5</sup>% to 10<sup>-1</sup>%. This spans both physiological levels (0.001–0.01%), which are comparable to those found in the natural diet, and supra-physiological levels (e.g., 0.1%), which exceed natural exposure but help define the threshold for aversive behavior.

      Our results demonstrate that flies begin to avoid cholesterol at concentrations ≥10<sup>-3</sup>% more (Figure 3A), which falls within the upper physiological range and may reflect the threshold beyond which cholesterol or related sterols become deleterious. At these higher concentrations, excess sterols may disrupt membrane fluidity, interfere with hormone signaling, or promote microbial overgrowth—all of which could compromise fly health.

      (2) The paper does not state or show whether the expression of IR7g, IR51b, and IR56d is confined to bitter GRNs. Bitter-specific expression of at least some of these receptors would be necessary to explain why bitter GRNs but not sugar GRNs (or other GRN types) normally show cholesterol responses.

      We show the Ir56d-Gal4 is co-expressed with Gr66a-GFP in S6/S7 sensilla, indicating that it is expressed in bitter GRNs (Figure 2—figure supplement 1F). In the case of Ir7g and Ir51b, there are no reporters or antibodies to address expression. However, previously they have been shown to be expressed in bitter GRNs using RT-PCR (Dhakal et al. 2021, Communications Biology; Pradhan et al. 2024, Journal of Hazardous Materials). In addition, we provide functional evidence that bitter GRNs are required for the cholesterol response since silencing bitter GRNs abolishes cholesterol-induced action potentials (Figure 1E–F). Moreover, we showed that we could rescue the Ir7g<sup>1</sup>, Ir51b<sup>1</sup> and Ir56d<sup>1</sup> mutant phenotypes only when we expressed the cognate transgenes in bitter GRNs using the Gr33a-GAL4 (Figure 3G). Thus, while Ir7g/Ir51b are not exclusive to bitter GRNs, their functional role in cholesterol detection is bitter-GRN-specific.

      (3) The authors only investigated the responses of GRNs in the labellum, but GRN responses in the leg may also contribute to the avoidance of cholesterol feeding. Alternatively, leg GRNs might contribute to cholesterol attraction that is unmasked when bitter GRNs are silenced. In support of this possibility, Ahn et al. (2017) showed that Ir56d functions in sugar GRNs of the leg to promote appetitive responses to fatty acids.

      This is an interesting idea. Indeed, when bitter GRNs are hyperpolarized, the flies exhibit a strong attraction to cholesterol. Nevertheless, the cellular basis for cholesterol attraction and whether it is mediated by GRNs in the legs will require a future investigation.

      (4) The authors might consider using proboscis extension as an additional readout of taste attraction or aversion, which would help them more directly link the labellar GRN responses to a behavioral readout. Using food ingestion as a readout can conflate the contribution of taste with post-ingestive effects, and the regulation of food ingestion also may involve contributions from GRNs on multiple organs, whereas organ-specific contributions can be dissociated using proboscis extension. For example, does presenting cholesterol on the proboscis lead to aversive responses in the proboscis extension assay (e.g., suppression of responses to sugar)? Does this aversion switch to attraction when bitter GRNs are silenced, as with the feeding assay?

      We thank the reviewer for the suggestion regarding the use of the proboscis extension reflex (PER) assay to strengthen the link between labellar GRN activity and behavioral responses to cholesterol.

      Author response image 1.

      Our PER assay results shown above indicate that cholesterol presentation on the labellum or forelegs leads to an aversive response, as evidenced by a significant reduction in proboscis extension when compared to control stimuli (Author response image 1A. 2% sucrose or 2% sucrose with 10<sup>-1</sup>% cholesterol was applied to labellum or forelegs and the percent PER was recorded. n=6. Data were compared using single-factor ANOVA coupled with Scheffe’s post-hoc test. Statistical significance was compared with the control. Means ± SEMs. **p<0.01). This finding supports the idea that cholesterol is detected by labellar and leg GRNs and elicits behavioral avoidance. In contrast, sucrose stimulation robustly induces proboscis extension, as expected for an appetitive stimulus. We confirmed the defects of due to each Ir mutant by presenting the stimuli to the labellum (Author response image 1B). Together, these PER results provide a more direct behavioral correlate of labellar and leg GRN activation and reinforce our conclusion that cholesterol is sensed as an aversive tastant through the labellar bitter GRNs.

      (5) The authors claim that the cholesterol receptor is composed of IR25a, IR76b, IR56d, and either IR7g or IR51b. While the authors have shown that IR25a and IR76b are each required for cholesterol sensing, they did not show that both are required components of the same receptor complex. If the authors are relying on previous studies to make this assumption, they should state this more clearly. Otherwise, I think further misexpression experiments may be needed where only IR25a or IR76b, but not both, are expressed in GRNs.

      In our study, we relied on prior work demonstrating that Ir25a and Ir76b function as broadly required co-receptors in most IR-dependent chemosensory pathways (Ganguly et al., 2017; Lee et al., 2018). These studies showed that Ir25a and Ir76b are co-expressed in many GRNs across multiple taste modalities. Functional IR complexes often fail to form or signal properly in the absence of these co-receptors. Thus, it is widely accepted in the field that Ir25a and Ir76b function together as a core heteromeric scaffold for diverse IR complexes, akin to co-receptors in other ionotropic glutamate receptor families. We state that while Ir25a and Ir76b are presumed co-receptors in the cholesterol receptor complex based on their conserved roles, their direct physical interaction with Ir7g, Ir51b, and Ir56d remains to be demonstrated.

      In support of this model, we note that in our ectopic expression experiments using I9 sensilla, which endogenously express Ir25a and Ir76b, introduction of either Ir7g + Ir56d or Ir51b + Ir56d was sufficient to confer cholesterol sensitivity (Figure 4B). We obtained a similar result in L6 sensilla (Figure 4D), which also endogenously express Ir25a and Ir76b. These findings imply that both co-receptors are already present in these sensilla and are likely part of the functional complex. However, we agree that we have not directly tested the requirement for both co-receptors in a minimal reconstitution context, such as expressing only Ir25a or Ir76b alongside tuning IRs in an otherwise null background. Such an experiment would indeed provide more direct evidence of their joint requirement in the receptor complex. Future studies, including heterologous expression experiments, will be necessary to define the cholesterol-receptor complexes.

    1. Author Response

      Reviewer #1 (Public Review):

      The authors introduce a computational model that simulates the dendrites of developing neurons in a 2D plane, subject to constraints inspired by known biological mechanisms such as diffusing trophic factors, trafficked resources, and an activity-dependent pruning rule. The resulting arbors are analyzed in terms of their structure, dynamics, and responses to certain manipulations. The authors conclude that 1) their model recapitulates a stereotyped timecourse of neuronal development: outgrowth, overshoot, and pruning 2) Neurons achieve near-optimal wiring lengths, and Such models can be useful to test proposed biological mechanisms- for example, to ask whether a given set of growth rules can explain a given observed phenomenon - as developmental neuroscientists are working to understand the factors that give rise to the intricate structures and functions of the many cell types of our nervous system.

      Overall, my reaction to this work is that this is just one instantiation of many models that the author could have built, given their stated goals. Would other models behave similarly? This question is not well explored, and as a result, claims about interpreting these models and using them to make experimental predictions should be taken warily. I give more detailed and specific comments below.

      We thank the reviewer for the summary of the work. We find the criticism “that this is one instantiation of many models [we] could have built” can apply to any model. To quote George Box, “all models are wrong, but some models are useful” was the moto that drove our modeling approach. In principle, there are infinitely many possible models. We chose one of the most minimalistic models which implements known biological mechanisms including activity-independent and -dependent phases of dendritic growth, and constrained parameters based on experimental data. We compare the proposed model to other alternatives in the Discussion section, especially to the models of Hermann Cuntz which propose very different strategies for growth.

      However, the reviewer is right that within the type of model we chose, we could have more extensively explored the sensitivity to parameters. In the revised manuscript we will investigate the sensitivity of model output to variations of specific parameters, as explained below.

      Point 1.1. Line 109. After reading the rest of the manuscript, I worry about the conclusion voiced here, which implies that the model will extrapolate well to manipulations of all the model components. How were the values of model parameters selected? The text implies that these were selected to be biologically plausible, but many seem far off. The density of potential synapses, for example, seems very low in the simulations compared to the density of axons/boutons in the cortex; what constitutes a potential synapse? The perfect correlations between synapses in the activity groups is flawed, even for synapses belonging to the same presynaptic cell. The density of postsynaptic cells is also orders of magnitude of, etc. Ideally, every claim made about the model's output should be supported by a parameter sensitivity study. The authors performed few explorations of parameter sensitivity and many of the choices made seem ad hoc.

      It is indeed important to clarify how the model parameters were selected. Here we provide a short justification for some of these parameters, which will be included in the revised manuscript.

      1) Potential synapse density: We modelled 1,500 potential synapses in a cortical sheet of size 185x185 microns squared. We used 1 pixel per μm to capture approximately 1 μm thick dendrites. Therefore, we started with initial density of 0.044 potential synapses per μm^2. From Author Response Image 1 we can see that at the end of our simulation time ~1,000 potential synapses remain. So in fact, the density of potential synapses is totally sufficient, since not many potential synapses end up connected. The rapid slowing down of growth in our model is not due to a depletion of potential synaptic partners as the number of potential synapses remains high. Nonetheless, we will explore this in the revised manuscript. (this figure will be included in the revised submission):

      2) Stabilized synapse density: Since ~1,000 of the potential synapses in the modeled cortical sheet remain available, ~500 become connected to the dendrites of the 9 somas in the modeled cortical sheet. This means that the density of stable connected synapses is approximately 0.015 synapses per μm^2. This is also the number that is shown in Figure 3b, which is about 60 synapses stabilized per cell. This density is much easier to compare to experimental data, and below we provide some numbers from literature we already cited in the manuscript as well as a recent preprint.

      In the developing cortex:

      • Leighton, Cheyne and Lohmann 2023 https://doi.org/10.1101/2023.03.02.530772 find up to 0.4 synapses per μm in pyramidal neurons in vivo in the developing mouse visual cortex at P8 to P13. This is almost identical to our value of 0.4 synapses per μm.

      • Ultanir et al., 2007 https://doi.org/10.1073/pnas.0704031104 find 0.7 to 1.7 spines per μm in pyramidal neurons in vivo in L2/3 of the developing mouse cortex, at P10 to P20.

      • Glynn et al., 2011 https://doi.org/10.1038/nn.2764 find 0.1 to 0.7 spines per μm^2 in pyramidal neurons in vivo and in vitro in L2/3 of the developing mouse cortex, at P8 to P60.

      In the developing hippocampus:

      Although these values vary somewhat across experiments, in most cases they are in agreement with our chosen values, especially when taking into account that we are modeling development (rather than adulthood).

      3) Soma/neuron density: Indeed, we did not exactly mention this number anywhere in the paper. But from the figures we can infer 9 somas growing dendrites on an area of ~34,000 μm^2. Thus, neuron density would be 300 neurons per mm^2. This number seems a bit low after a short search through the literature. For e.g. Keller et al., 2018 https://www.frontiersin.org/articles/10.3389/fnana.2018.00083/full reports about 90,000 neurons per mm^3, albeit in adulthood.

      We are also performing a sensitivity analysis where some of these parameters are varied and will include this in the revised manuscript. In particular:

      (1) We will vary the nature of the input correlations. In the current model, the synapses in each correlated group receive spike trains with a perfect correlation and there are no correlations across the groups. We will reduce the correlations within group and add non-zero correlations across the groups.

      (2) We will vary the density of the neuronal somas. We expect that higher densities of somas will either yield smaller dendritic areas because the different neurons compete more or result in a state where nearby neurons have to complement each other regarding their activity preferences.

      (3) We will introduce dynamics in the potential synapses to model the dynamics of axons. We plan to explore several scenarios. We could introduce a gradual increase in the density of potential synapses and implement a cap on the number of synapses that can be alive at the same time, and vary that cap. We could also introduce a lifetime of each synapse (following for example a lognormal distribution). A potential synapse can disappear if it does not form a stable synapse in its lifetime, in which case it could move to a different location.

      Point 1.2. Many potentially important phenomena seem to be excluded. I realize that no model can be complete, but the choice of which phenomena to include or exclude from this model could bias studies that make use of it and is worth serious discussion. The development of axons is concurrent with dendrite outgrowth, is highly dynamic, and perhaps better understood mechanistically. In this model, the inputs are essentially static. Growing dendrites acquire and lose growth cones that are associated with rapid extension, but these do not seem to be modeled. Postsynaptic firing does not appear to be modeled, which may be critical to activity-dependent plasticity. For example, changes in firing are a potential explanation for the global changes in dendritic pruning that occur following the outgrowth phase.

      As the reviewer concludes, no model can be complete. In agreement with this, here we would like to quote a paragraph from a very nice paper by Larry Abbott (“Theoretical Neuroscience Rising, Neuron 2008 https://www.sciencedirect.com/science/article/pii/S0896627308008921) which although published more than 10 years ago, still applies today:

      “Identifying the minimum set of features needed to account for a particular phenomenon and describing these accurately enough to do the job is a key component of model building. Anything more than this minimum set makes the model harder to understand and more difficult to evaluate. The term ‘‘realistic’’ model is a sociological rather than a scientific term. The truly realistic model is as impossible and useless a concept as Borges’ ‘‘map of the empire that was of the same scale as the empire and that coincided with it point for point’’ (Borges, 1975). […] The art of modeling lies in deciding what this subset should be and how it should be described.”

      We have clearly stated in the Introduction (e.g. lines 37-75) which phenomena we include in the model and why. The Discussion also compares our model to others (lines 315-373), pointing out that most models either focus on activity-independent or activity-dependent phases. We include both, combining literature on molecular gradients and growth factors, with activity-dependent connectivity refinements instructed by spontaneous activity. We could not think of a more tractable, more minimalist model that would include both activity-independent or activity-dependent aspects. Therefore, we feel that the current manuscript provides sufficient motivation but also a discussion of limitations of the current model.

      Regarding including the concurrent development of axons, we agree this is very interesting and currently not addressed in the model. As noted at the bottom of our reply to point 1.1, bullet (3) we are now revising the manuscript to include a simplified form of axonal dynamics by allowing changes in the lifetime and location of potential synapses, which come from axons of presynaptic partners.

      Regarding postsynaptic firing, this is indeed super relevant and an important point to consider. In one of our recent publications (Kirchner and Gjorgjieva, 2021 https://www.nature.com/articles/s41467-021-23557-3), we studied only an activity-dependent model for the organization of synaptic inputs on non-growing dendrites which have a fixed length. There, we considered the effect of postsynaptic firing and demonstrated that it plays an important role in establishing a global organization of synapses on the entire dendritic tree of the neuron, and not just local dendritic branches. For example, we showed that could that it could lead to the emergence of retinotopic maps which have been found experimentally (Iacaruso et al., 2017 https://www.nature.com/articles/nature23019). Since we use the same activity-dependent plasticity model in this paper, we expect that the somatic firing will have the same effect on establishing synaptic distributions on the entire dendritic tree. We will make a note of this in the Discussion in the revised paper.

      Point 1.3. Line 167. There are many ways to include activity -independent and -dependent components into a model and not every such model shows stability. A key feature seems to be that larger arbors result in reduced growth and/or increased retraction, but this could be achieved in many ways (whether activity dependent or not). It's not clear that this result is due to the combination of activity-dependent and independent components in the model, or conceptually why that should be the case.

      We never argued for model uniqueness. There are always going to be many different models (at different spatial and temporal scales, at different levels of abstraction). We can never study all of them and like any modeling study in systems neuroscience we have chosen one model approach and investigated this approach. We do compare the current model to others in the Discussion. If the reviewers have a specific implementation that we should compare our model to as an alternative, we could try, but not if this means doing a completely separate project.

      Point 1.4. Line 183. The explanation of overshoot in terms of the different timescales of synaptic additions versus activity-dependent retractions was not something I had previously encountered and is an interesting proposal. Have these timescales been measured experimentally? To what extent is this a result of fine-tuning of simulation parameters?

      We found that varying the amount of BDNF controls the timescale of the activity-dependent plasticity (see our Figure 5c). Hence, changing the balance between synaptic additions vs. retractions is already explored in Figure 5e and f. Here we show that the overshoot and retraction does not have to be fine-tuned but may be abolished if there is too much activity-dependent plasticity.

      Regarding the relative timescales of synaptic additions vs. retractions: since the first is mainly due to activity-independent factors, and the second due to activity-dependent plasticity, the questions is really about the timescales of the latter two. As we write in the Introduction (lines 60-62), manipulating activity-dependent synaptic transmission has been found to not affect morphology but rather the density and specificity of synaptic connections (Ultanir et al. 2007 https://doi.org/10.1073/pnas.0704031104), supporting the sequential model we have (although we do not impose the sequence, as both activity-independent and activity-dependent mechanisms are always “on”; but note that activity-dependent plasticity can only operate on synapses that have already formed).

      Point 1.5. Line 203. This result seems at odds with results that show only a very weak bias in the tuning distribution of inputs to strongly tuned cortical neurons (e.g. work by Arthur Konnerth's group). This discrepancy should be discussed.

      First, we note that the correlated activity experienced by our modeled synapses (and resulting synaptic organization) does not necessarily correspond to visual orientation, or any stimulus feature, for that matter.

      Nonetheless, this is a very interesting question and there is some variability in what the experimental data show. Many studies have shown that synapses on dendrites are organized into functional synaptic clusters: across brain regions, developmental ages and diverse species from rodent to primate (Kleindienst et al. 2011; Takahashi et al. 2012; Winnubst et al. 2015; Gökçe et al., 2016; Wilson et al. 2016; Iacaruso et al., 2017; Scholl et al., 2017; Niculescu et al. 2018; Kerlin et al. 2019; Ju et al. 2020). Interestingly, some in vivo studies have reported lack of fine-scale synaptic organization (Varga et al. 2011; X. Chen et al. 2011; T.-W. Chen et al. 2013; Jia et al. 2010; Jia et al. 2014), while others reported clustering for different stimulus features in different species. For example, dendritic branches in the ferret visual cortex exhibit local clustering of orientation selectivity but do not exhibit global organization of inputs according to spatial location and receptive field properties (Wilson et al. 2016; Scholl et al., 2017). In contrast, synaptic inputs in mouse visual cortex do not cluster locally by orientation, but only by receptive field overlap, and exhibit a global retinotopic organization along the proximal-distal axis (Iacaruso et al., 2017). We proposed a theoretical framework to reconcile these data: combining activity-dependent plasticity similar to the BDNF-proBDNF model that we used in the current work, and a receptive field model for the different species (Kirchner and Gjorgjieva, 2021 https://www.nature.com/articles/s41467-021-23557-3). We can mention this aspect in the revised manuscript.

      Point 1.6. Line 268. How does the large variability in the size of the simulated arbors relate to the relatively consistent size of arbors of cortical cells of a given cell type? This variability suggests to me that these simulations could be sensitive to small changes in parameters (e.g. to the density or layout of presynapses).

      As noted at the bottom of our reply to point 1.1, bullet (3) we are now revising the manuscript to include changes in the lifetime and location of potential synapses.

      Point 1.7. The modeling of dendrites as two-dimensional will likely limit the usefulness of this model. Many phenomena- such as diffusion, random walks, topological properties, etc - fundamentally differ between two and three dimensions.

      The reviewer is right about there being differences between two and three dimensions. But a simpler model does not mean a useless model even if not completely realistic. We have ongoing work that extends the current model to 3D but is beyond the scope of the current paper. In systems neuroscience, people have found very interesting results making such simplified geometric assumptions about networks, for instance the one-dimensional ring model has been used to uncover fundamental insights about computations even though highly simplified and abstracted.

      Point 1.8. The description of wiring lengths as 'approximately optimal' in this text is problematic. The plotted data show that the wiring lengths are several deviations away from optimal, and the random model is not a valid instantiation of the 2D non-overlapping constraints the authors imposed. A more appropriate null should be considered.

      We did not use the term “optimal” in line with previous literature. We wrongly referred to the minimal wiring length as the optimal wiring length, but neurons can optimize their wiring not only by minimizing their dendritic length (e.g. work of Hermann Cuntz). In the revised manuscript, we will replace the term “optimal wiring” with “minimal wiring”. Then we will compare the wiring length in the model with the theoretically minimal wiring length, the random wiring length and the actual data.

      Point 1.9. It's not clear to me what the authors are trying to convey by repeatedly labeling this model as 'mechanistic'. The mechanisms implemented in the model are inspired by biological phenomena, but the implementations have little resemblance to the underlying biophysical mechanisms. Overall my impression is that this is a phenomenological model intended to show under what conditions particular patterns are possible. Line 363, describing another model as computational but not mechanistic, was especially unclear to me in this context.

      What we mean by mechanistic is that we implement equations that model specific mechanisms i.e. we have a set of equations that implement the activity-independent attraction to potential synapses (with parameters such as the density of synapses, their spatial influence, etc) and the activity-dependent refinement of synapses (with parameters such as the ratio of BDNF and proBDNF to induce potentiation vs depression, the activity-dependent conversion of one factor to the other, etc). This is a bottom-up approach where we combine multiple elements together to get to neuronal growth and synaptic organization. This approach is in stark contrast to the so-called top-down or normative approaches where the method would involve defining an objective function (e.g. minimal dendritic length) which depends on a set of parameters and then applying a gradient descent or other mathematical optimization technique to get at the parameters that optimize the objective function. This latter approach we would not call mechanistic because it involves an abstract objective function (who could say what a neuron or a circuit should be trying to optimize) and a mathematical technique for how to optimize the function (we don’t know of neurons can compute gradients of abstract objective functions).

      Hence our model is mechanistic, but it does operate at a particular level of abstraction/simplification. We don’t model individual ion channels, or biophysics of synaptic plasticity (opening and closing of NMDA channels, accumulation of proteins at synapses, protein synthesis). We do, however, provide a biophysical implementation of the plasticity mechanism though the BDNF/proBDNF model which is more than most models of plasticity achieve, because they typically model a phenomenological STDP or Hebbian rule that just uses activity patterns to potential or depress synaptic weights, disregarding how it could be implemented.

      Reviewer #2 (Public Review):

      This work combines a model of two-dimensional dendritic growth with attraction and stabilisation by synaptic activity. The authors find that constraining growth models with competition for synaptic inputs produces artificial dendrites that match some key features of real neurons both over development and in terms of final structure. In particular, incorporating distance-dependent competition between synapses of the same dendrite naturally produces distinct phases of dendritic growth (overshoot, pruning, and stabilisation) that are observed biologically and leads to local synaptic organisation with functional relevance. The approach is elegant and well-explained, but makes some significant modelling assumptions that might impact the biological relevance of the results.

      Strengths:

      The main strength of the work is the general concept of combining morphological models of growth with synaptic plasticity and stabilisation. This is an interesting way to bridge two distinct areas of neuroscience in a manner that leads to findings that could be significant for both. The modelling of both dendritic growth and distance-dependent synaptic competition is carefully done, constrained by reasonable biological mechanisms, and well-described in the text. The paper also links its findings, for example in terms of phases of dendritic growth or final morphological structure, to known data well.

      Weaknesses:

      The major weaknesses of the paper are the simplifying modelling assumptions that are likely to have an impact on the results. These assumptions are not discussed in enough detail in the current version of the paper.

      1) Axonal dynamics.

      A major, and lightly acknowledged, assumption of this paper is that potential synapses, which must come from axons, are fixed in space. This is not realistic for many neural systems, as multiple undifferentiated neurites typically grow from the soma before an axon is specified (Polleux & Snider, 2010). Further, axons are also dynamic structures in early development and, at least in some systems, undergo activity-dependent morphological changes too (O'Leary, 1987; Hall 2000). This paper does not consider the implications of joint pre- and post-synaptic growth and stabilisation.

      We thank the reviewer for the summary of the strengths and weaknesses of the work. While we feel that including a full model of axonal dynamics is beyond the scope of the current manuscript, some aspects of axonal dynamics can be included. In a revised model, we will introduce a gradual increase in the density of potential synapses and implement a cap on the number of synapses that can be alive at the same time, and vary that cap. We plan to also introduce a lifetime of each synapse (following for example a lognormal distribution). A potential synapse can disappear if it does not form a stable synapse in its lifetime, in which case it could move to a different location. See also our reply to reviewer comment 1.1, bullet (3).

      2) Activity correlations

      On a related note, the synapses in the manuscript display correlated activity, but there is no relationship between the distance between synapses and their correlation. In reality, nearby synapses are far more likely to share the same axon and so display correlated activity. If the input activity is spatially correlated and synaptic plasticity displays distance-dependent competition in the dendrites, there is likely to be a non-trivial interaction between these two features with a major impact on the organisation of synaptic contacts onto each neuron.

      We are exploring the amount of correlation (between and within correlated groups) to include in the revised manuscript (see also our reply to reviewer comment 1.1, bullet (1)).

      However, previous experimental work, (Kleindienst et al., 2011 https://doi.org/10.1016/j.neuron.2011.10.015) has provided anatomical and functional analyses that it is unlikely that the functional synaptic clustering on dendritic branches is the result of individual axons making more than one synapse (see pg. 1019).

      3) BDNF dynamics

      The models are quite sensitive to the ratio of BDNF to proBDNF (eg Figure 5c). This ratio is also activity-dependent as synaptic activation converts proBDNF into BDNF. The models assume a fixed ratio that is not affected by synaptic activity. There should at least be more justification for this assumption, as there is likely to be a positive feedback relationship between levels of BDNF and synaptic activation.

      The reviewer is correct. We used the BDNF-proBDNF model for synaptic plasticity based on our previous work: Kirchner and Gjorgjieva, 2021 https://www.nature.com/articles/s41467-021-23557-3.

      There, we explored only the emergence of functionally clustered synapses on static dendrites which do not grow. In the Methods section (Parameters and data fitting) we justify the choice of the ratio of BDNF to proBDNF from published experimental work. We also performed sensitivity analysis (Supplementary Fig. 1) and perturbation simulations (Supplementary Fig. 3), which showed that the ratio is crucial in regulating the overall amount of potentiation and depression of synaptic efficacy, and therefore has a strong impact on the emergence and maintenance of synaptic organization. Since we already performed all this analysis, we do not expect there will be any differences in the current model which includes dendritic growth, as the activity-dependent mechanism has such a different timescale.

      A further weakness is in the discussion of how the final morphologies conform to principles of optimal wiring, which is quite imprecise. 'Optimal wiring' in the sense of dendrites and axons (Cajal, 1895; Chklovskii, 2004; Cuntz et al, 2007, Budd et al, 2010) is not usually synonymous with 'shortest wiring' as implied here. Instead, there is assumed to be a balance between minimising total dendritic length and minimising the tree distance (ie Figure 4c here) between synapses and the site of input integration, typically the soma. The level of this balance gives the deviation from the theoretical minimum length as direct paths to synapses typically require longer dendrites. In the model this is generated by the guidance of dendritic growth directly towards the synaptic targets. The interpretation of the deviation in this results section discussing optimal wiring, with hampered diffusion of signalling molecules, does not seem to be correct.

      We agree with this comment. We had wrongly used the term “optimal wiring” as neurons can optimize their wiring not only by minimizing their dendritic length but other factors as noted by the reviewer. In the revised manuscript will replace the term “optimal wiring” with “minimal wiring” and discuss these differences to previous work.

      Reviewer #3 (Public Review):

      The authors propose a mechanistic model of how the interplay between activity-independent growth and an activity-dependent synaptic strengthening/weaken model influences the dendrite shape, complexity and distribution of synapses. The authors focus on a model for stellate cells, which have multiple dendrites emerging from a soma. The activity independent component is provided by a random pool of presynaptic sites that represent potential synapses and that release a diffusible signal that promotes dendritic growth. Then a spontaneous activity pattern with some correlation structure is imposed at those presynaptic sites. The strength of these synapses follow a learning rule previously proposed by the lab: synapses strengthen when there is correlated firing across multiple sites, and synapses weaken if there is uncorrelated firing with the relative strength of these processes controlled by available levels of BDNF/proBDNF. Once a synapse is weakened below a threshold, the dendrite branch at that site retracts and loses its sensitivity to the growth signal

      The authors run the simulation and map out how dendrites and synapses evolve and stabilize. They show that dendritic trees growing rapidly and then stabilize by balancing growth and retraction (Figure 2). They also that there is an initial bout of synaptogenesis followed by loss of synapses, reflecting the longer amount of time it takes to weaken a synapse (Figure 3). They analyze how this evolution of dendrites and synapses depends on the correlated firing of synapses (i.e. defined as being in the same "activity group"). They show that in the stabilized phase, synapses that remain connected to a given dendritic branch are likely to be from same activity group (Figure 4). The authors systemically alter the learning rule by changing the available concentration of BDNF, which alters the relative amount of synaptic strengthening, which in turn affects stabilization, density of synapses and interestingly how selective for an activity group one dendrite is (Figure 5). In addition the authors look at how altering the activity-independent factors influences outgrowth (Figure 6). Finally, one of the interesting outcomes is that the resulting dendritic trees represent "optimal wiring" solutions in the sense that dendrites use the shortest distance given the distribution of synapses. They compare this distribute to one published data to see how the model compared to what has been observed experimentally.

      There are many strengths to this study. The consequence of adding the activity-dependent contribution to models of synapto- and dendritogenesis is novel. There is some exploration of parameters space with the motivation of keeping the parameters as well as the generated outcomes close to anatomical data of real dendrites. The paper is also scholarly in its comparison of this approach to previous generative models. This work represented an important advance to our understanding of how learning rules can contribute to dendrite morphogenesis

      We thank the reviewer for the positive evaluation of the work and the suggestions below.

    1. Author response:

      Reviewer #1 (Evidence, reproducibility and clarity):

      Authors has provided a mechanism by which how presence of truncated P53 can inactivate function of full length P53 protein. Authors proposed this happens by sequestration of full length P53 by truncated P53.

      In the study, performed experiments are well described.

      My area of expertise is molecular biology/gene expression, and I have tried to provide suggestions on my area of expertise. The study has been done mainly with overexpression system and I have included few comments which I can think can be helpful to understand effect of truncated P53 on endogenous wild type full length protein. Performing experiments on these lines will add value to the observation according to this reviewer.

      Major comments:

      (1) What happens to endogenous wild type full length P53 in the context of mutant/truncated isoforms, that is not clear. Using a P53 antibody which can detect endogenous wild type P53, can authors check if endogenous full length P53 protein is also aggregated as well? It is hard to differentiate if aggregation of full length P53 happens only in overexpression scenario, where lot more both of such proteins are expressed. In normal physiological condition P53 expression is usually low, tightly controlled and its expression get induced in altered cellular condition such as during DNA damage. So, it is important to understand the physiological relevance of such aggregation, which could be possible if authors could investigate effect on endogenous full length P53 following overexpression of mutant isoforms.

      Thank you very much for your insightful comments.

      (1) To address “what happens to endogenous wild-type full-length P53 in the context of mutant/truncated isoforms," we employed a human A549 cell line expressing endogenous wild-type p53 under DNA damage conditions such as an etoposide treatment(1). We choose the A549 cell line since similar to H1299, it is a lung cancer cell line (www.atcc.org). For comparison, we also transfected the cells with 2 μg of V5-tagged plasmids encoding FLp53 and its isoforms Δ133p53 and Δ160p53. As shown in Author response image 1A, lanes 1 and 2, endogenous p53 expression, remained undetectable in A549 cells despite etoposide treatment, which limits our ability to assess the effects of the isoforms on the endogenous wild-type FLp53. We could, however, detect the V5-tagged FLp53 expressed from the plasmid using anti-V5 (rabbit) as well as with antiDO-1 (mouse) antibody (Author response image 1). The latter detects both endogenous wildtype p53 and the V5-tagged FLp53 since the antibody epitope is within the Nterminus (aa 20-25). This result supports the reviewer’s comment regarding the low level of expression of endogenous p53 that is insufficient for detection in our experiments.   

      In summary, in line with the reviewer’s comment that ‘under normal physiological conditions p53 expression is usually low,’ we could not detect p53 with an anti-DO-1 antibody. Thus, we proceeded with V5/FLAG-tagged p53 for detection of the effects of the isoforms on p53 stability and function. We also found that protein expression in H1299 cells was more easily detectable than in A549 cells (Compare Author response image 1A and B). Thus, we decided to continue with the H1299 cells (p53-null), which would serve as a more suitable model system for this study.  

      (2) We agree with the reviewer that ‘It is hard to differentiate if aggregation of full-length p53 happens only in overexpression scenario’. However, it is not impossible to imagine that such aggregation of FLp53 happens under conditions when p53 and its isoforms are over-expressed in the cell. Although the exact physiological context is not known and beyond the scope of the current work, our results indicate that at higher expression, p53 isoforms drive aggregation of FLp53. Given the challenges of detecting endogenous FLp53, we had to rely on the results obtained with plasmid mediated expression of p53 and its isoforms in p53-null cells.

      Author response image 1.

      Comparative analysis of protein expression in A549 and H1299 cells. (A) A549 cells (p53 wild-type) were treated with etoposide to induce endogenous wild-type p53 expression. To assess the effects of FLp53 and its isoforms Δ133p53 and Δ160p53 on endogenous wild-type p53 aggregation, A549 cells were transfected with 2 μg of V5-tagged p53 expression plasmids, with or without etoposide (20μM for 8h) treatment. Western blot analysis was done with the anti-V5 (rabbit) to detect V5-tagged proteins and anti-DO-1 (mouse), the latter detects both endogenous wild-type p53 and V5-tagged FLp53. The merged image corresponds to the overlay between the V5 and DO1 antibody signals. (B) H1299 cells (p53-null) were transfected with 2 μg V5tagged p53 expression plasmids or the empty vector control pcDNA3.1. Western blot analysis was done with the anti-V5 (mouse) antibody. 

      (2) Can presence of mutant P53 isoforms can cause functional impairment of wild type full length endogenous P53? That could be tested as well using similar ChIP assay authors has performed, but instead of antibody against the Tagged protein if the authors could check endogenous P53 enrichment in the gene promoter such as P21 following overexpression of mutant isoforms. May be introducing a condition such as DNA damage in such experiment might help where endogenous P53 is induced and more prone to bind to P53 target such as P21.

      Thank you very much for your valuable comments and suggestions. To investigate the potential functional impairment of endogenous wild-type p53 by p53 isoforms, we initially utilized A549 cells (p53 wild-type), aiming to monitor endogenous wild-type p53 expression following DNA damage. However, as mentioned and demonstrated in Author response image 1, endogenous p53 expression was too low to be detected under these conditions, making the ChIP assay for analyzing endogenous p53 activity unfeasible. Thus, we decided to utilize plasmid-based expression of FLp53 and focus on the potential functional impairment induced by the isoforms.

      (3) On similar lines, authors described:

      "To test this hypothesis, we escalated the ratio of FLp53 to isoforms to 1:10. As expected, the activity of all four promoters decreased significantly at this ratio (Figure 4A-D). Notably, Δ160p53 showed a more potent inhibitory effect than Δ133p53 at the 1:5 ratio on all promoters except for the p21 promoter, where their impacts were similar (Figure 4E-H). However, at the 1:10 ratio, Δ133p53 and Δ160p53 had similar effects on all transactivation except for the MDM2 promoter (Figure 4E-H)."

      Again, in such assay authors used ratio 1:5 to 1:10 full length vs mutant. How authors justify this result in context (which is more relevant context) where one allele is Wild type (functional P53) and another allele is mutated (truncated, can induce aggregation). In this case one would except 1:1 ratio of full-length vs mutant protein, unless other regulation is going which induces expression of mutant isoforms more than wild type full length protein. Probably discussing on these lines might provide more physiological relevance to the observed data.

      Thank you for raising this point regarding the physiological relevance of the ratios used in our study.

      (1) In the revised manuscript (lines 193-195), we added in this direction that “The elevated Δ133p53 protein modulates p53 target genes such as miR‑34a and p21, facilitating cancer development(2, 3). To mimic conditions where isoforms are upregulated relative to FLp53, we increased the ratios to 1:5 and 1:10.” This approach aims to simulate scenarios where isoforms accumulate at higher levels than FLp53, which may be relevant in specific contexts, as also elaborated above.

      (2) Regarding the issue of protein expression, where one allele is wild-type and the other is isoform, this assumption is not valid in most contexts. First, human cells have two copies of TPp53 gene (one from each parent). Second, the TP53 gene has two distinct promoters: the proximal promoter (P1) primarily regulates FLp53 and ∆40p53, whereas the second promoter (P2) regulates ∆133p53 and ∆160p53(4, 5). Additionally, ∆133TP53 is a p53 target gene(6, 7) and the expression of Δ133p53 and FLp53 is dynamic in response to various stimuli. Third, the expression of p53 isoforms is regulated at multiple levels, including transcriptional, post-transcriptional, translational, and post-translational processing(8). Moreover, different degradation mechanisms modify the protein level of p53 isoforms and FLp53(8). These differential regulation mechanisms are regulated by various stimuli, and therefore, the 1:1 ratio of FLp53 to ∆133p53 or ∆160p53 may be valid only under certain physiological conditions. In line with this, varied expression levels of FLp53 and its isoforms, including ∆133p53 and ∆160p53, have been reported in several studies(3, 4, 9, 10). 

      (3) In our study, using the pcDNA 3.1 vector under the human cytomegalovirus (CMV) promoter, we observed moderately higher expression levels of ∆133p53 and ∆160p53 relative to FLp53 (Author response image 1B). This overexpression scenario provides a model for studying conditions where isoform accumulation might surpass physiological levels, impacting FLp53 function. By employing elevated ratios of these isoforms to FLp53, we aim to investigate the potential effects of isoform accumulation on FLp53.

      (4) Finally does this altered function of full length P53 (preferably endogenous one) in presence of truncated P53 has any phenotypic consequence on the cells (if authors choose a cell type which is having wild type functional P53). Doing assay such as apoptosis/cell cycle could help us to get this visualization.

      Thank you for your insightful comments. In the experiment with A549 cells (p53 wild-type), endogenous p53 levels were too low to be detected, even after DNA damage induction. The evaluation of the function of endogenous p53 in the presence of isoforms is hindered, as mentioned above. In the revised manuscript, we utilized H1299 cells with overexpressed proteins for apoptosis studies using the Caspase-Glo® 3/7 assay (Figure 7). This has been shown in the Results section (lines 254-269). “The Δ133p53 and Δ160p53 proteins block pro-apoptotic function of FLp53.

      One of the physiological read-outs of FLp53 is its ability to induce apoptotic cell death(11). To investigate the effects of p53 isoforms Δ133p53 and Δ160p53 on FLp53-induced apoptosis, we measured caspase-3 and -7 activities in H1299 cells expressing different p53 isoforms (Figure 7). Caspase activation is a key biochemical event in apoptosis, with the activation of effector caspases (caspase-3 and -7) ultimately leading to apoptosis(12). The caspase-3 and -7 activities induced by FLp53 expression was approximately 2.5 times higher than that of the control vector (Figure 7). Co-expression of FLp53 and the isoforms Δ133p53 or Δ160p53 at a ratio of 1: 5 significantly diminished the apoptotic activity of FLp53 (Figure 7). This result aligns well with our reporter gene assay, which demonstrated that elevated expression of Δ133p53 and Δ160p53 impaired the expression of apoptosis-inducing genes BAX and PUMA (Figure 4G and H). Moreover, a reduction in the apoptotic activity of FLp53 was observed irrespective of whether Δ133p53 or Δ160p53 protein was expressed with or without a FLAG tag (Figure 7). This result, therefore, also suggests that the FLAG tag does not affect the apoptotic activity or other physiological functions of FLp53 and its isoforms. Overall, the overexpression of p53 isoforms Δ133p53 and Δ160p53 significantly attenuates FLp53-induced apoptosis, independent of the protein tagging with the FLAG antibody epitope.”

      Referees cross-commenting

      I think the comments from the other reviewers are very much reasonable and logical.

      Especially all 3 reviewers have indicated, a better way to visualize the aggregation of full-length wild type P53 by truncated P53 (such as looking at endogenous P53# by reviewer 1, having fluorescent tag #by reviewer 2 and reviewer 3 raised concern on the FLAG tag) would add more value to the observation.

      Thank you for these comments. The endogenous p53 protein was undetectable in A549 cells induced by etoposide (Figure R1A). Therefore, we conducted experiments using FLAG/V5-tagged FLp53.  To avoid any potential side effects of the FLAG tag on p53 aggregation, we introduced untagged p53 isoforms in the H1299 cells and performed subcellular fractionation. Our revised results, consistent with previous FLAG-tagged p53 isoforms findings, demonstrate that co-expression of untagged isoforms with FLAG-tagged FLp53 significantly induced the aggregation of FLAG-FLp53, while no aggregation was observed when FLAG-tagged FLp53 was expressed alone (Supplementary Figure 6). These results clearly indicate that the FLAG tag itself does not contribute to protein aggregation. 

      Additionally, we utilized the A11 antibody to detect protein aggregation, providing additional validation (Figure 8 from Jean-Christophe Bourdon et al. Genes Dev. 2005;19:2122-2137). Given that the fluorescent proteins (~30 kDa) are substantially bigger than the tags used here (~1 kDa) and may influence oligomerization (especially GFP), stability, localization, and function of p53 and its isoforms, we avoided conducting these vital experiments with such artificial large fusions. 

      Reviewer #1 (Significance):

      The work in significant, since it points out more mechanistic insight how wild type full length P53 could be inactivated in the presence of truncated isoforms, this might offer new opportunity to recover P53 function as treatment strategies against cancer.

      Thank you for your insightful comments. We appreciate your recognition of the significance of our work in providing mechanistic insights into how wild-type FLp53 can be inactivated by truncated isoforms. We agree that these findings have potential for exploring new strategies to restore p53 function as a therapeutic approach against cancer. 

      Reviewer #2 (Evidence, reproducibility and clarity):

      The manuscript by Zhao and colleagues presents a novel and compelling study on the p53 isoforms, Δ133p53 and Δ160p53, which are associated with aggressive cancer types. The main objective of the study was to understand how these isoforms exert a dominant negative effect on full-length p53 (FLp53). The authors discovered that the Δ133p53 and Δ160p53 proteins exhibit impaired binding to p53-regulated promoters. The data suggest that the predominant mechanism driving the dominant-negative effect is the coaggregation of FLp53 with Δ133p53 and Δ160p53.

      This study is innovative, well-executed, and supported by thorough data analysis. However, the authors should address the following points:

      (1) Introduction on Aggregation and Co-aggregation: Given that the focus of the study is on the aggregation and co-aggregation of the isoforms, the introduction should include a dedicated paragraph discussing this issue. There are several original research articles and reviews that could be cited to provide context.

      Thank you very much for the valuable comments. We have added the following paragraph in the revised manuscript (lines 74-82): “Protein aggregation has become a central focus of modern biology research and has documented implications in various diseases, including cancer(13, 14, 15). Protein aggregates can be of different types ranging from amorphous aggregates to highly structured amyloid or fibrillar aggregates, each with different physiological implications. In the case of p53, whether protein aggregation, and in particular, co-aggregation with large N-terminal deletion isoforms, plays a mechanistic role in its inactivation is yet underexplored. Interestingly, the Δ133p53β isoform has been shown to aggregate in several human cancer cell lines(16). Additionally, the Δ40p53α isoform exhibits a high aggregation tendency in endometrial cancer cells(17). Although no direct evidence exists for Δ160p53 yet, these findings imply that p53 isoform aggregation may play a major role in their mechanisms of actions.”

      (2) Antibody Use for Aggregation: To strengthen the evidence for aggregation, the authors should consider using antibodies that specifically bind to aggregates.

      Thank you for your insightful suggestion. We addressed protein aggregation using the A11 antibody which specifically recognizes amyloid-like protein aggregates. We analyzed insoluble nuclear pellet samples prepared under identical conditions as described in Figure 6B. To confirm the presence of p53 proteins, we employed the anti-p53 M19 antibody (Santa Cruz, Cat No. sc-1312) to detect bands corresponding to FLp53 and its isoforms Δ133p53 and Δ160p53. The monomer FLp53 was not detected (Figure 8, lower panel, Jean-Christophe Bourdon et al. Genes Dev. 2005;19:2122-2137), which may be attributed to the lower binding affinity of the anti-p53 M19 antibody to it. These samples were also immunoprecipitated using the A11 antibody (Thermo Fischer Scientific, Cat No. AHB0052) to detect aggregated proteins. Interestingly, FLp53 and its isoforms, Δ133p53 and Δ160p53, were clearly visible with Anti-A11 antibody when co-expressed at a 1:5 ratio suggesting that they underwent co-aggregation. However, no FLp53 aggregates were observed when it was expressed alone (Author response image 2). These results support the conclusion in our manuscript that Δ133p53 and Δ160p53 drive FLp53 aggregation. 

      Author response image 2.

      Induction of FLp53 Aggregation by p53 Isoforms Δ133p53 and Δ160p53. H1299 cells transfected with the FLAG-tagged FLp53 and V5-tagged Δ133p53 or Δ160p53 at a 1:5 ratio. The cells were subjected to subcellular fractionation, and the resulting insoluble nuclear pellet was resuspended in RIPA buffer. The samples were heated at 95°C until the pellet was completely dissolved, and then analyzed by Western blotting. Immunoprecipitation was performed using the A11 antibody, which specifically recognizes amyloid protein aggregates, and the anti-p53 M19 antibody, which detects FLp53 as well as its isoforms Δ133p53 and Δ160p53. 

      (3) Fluorescence Microscopy: Live-cell fluorescence microscopy could be employed to enhance visualization by labeling FLp53 and the isoforms with different fluorescent markers (e.g., EGFP and mCherry tags).

      We appreciate the suggestion to use live-cell fluorescence microscopy with EGFP and mCherry tags for the visualization FLp53 and its isoforms. While we understand the advantages of live-cell imaging with EGFP / mCherry tags, we restrained us from doing such fusions as the GFP or corresponding protein tags are very big (~30 kDa) with respect to the p53 isoform variants (~30 kDa).  Other studies have shown that EGFP and mCherry fusions can alter protein oligomerization, solubility and aggregation(18, 19) Moreover, most fluorescence proteins are prone to dimerization (i.e. EGFP) or form obligate tetramers (DsRed)(20, 21, 22), potentially interfering with the oligomerization and aggregation properties of p53 isoforms, particularly Δ133p53 and Δ160p53.

      Instead, we utilized FLAG- or V5-tag-based immunofluorescence microscopy, a well-established and widely accepted method for visualizing p53 proteins. This method provided precise localization and reliable quantitative data, which we believe meet the needs of the current study. We believe our chosen method is both appropriate and sufficient for addressing the research question.

      Reviewer #2 (Significance):

      The manuscript by Zhao and colleagues presents a novel and compelling study on the p53 isoforms, Δ133p53 and Δ160p53, which are associated with aggressive cancer types. The main objective of the study was to understand how these isoforms exert a dominant negative effect on full-length p53 (FLp53). The authors discovered that the Δ133p53 and Δ160p53 proteins exhibit impaired binding to p53-regulated promoters. The data suggest that the predominant mechanism driving the dominant-negative effect is the coaggregation of FLp53 with Δ133p53 and Δ160p53.

      We sincerely thank the reviewer for the thoughtful and positive comments on our manuscript and for highlighting the significance of our findings on the p53 isoforms, Δ133p53 and Δ160p53. 

      Reviewer #3 (Evidence, reproducibility and clarity):

      In this manuscript entitled "Δ133p53 and Δ160p53 isoforms of the tumor suppressor protein p53 exert dominant-negative effect primarily by coaggregation", the authors suggest that the Δ133p53 and Δ160p53 isoforms have high aggregation propensity and that by co-aggregating with canonical p53 (FLp53), they sequestrate it away from DNA thus exerting a dominantnegative effect over it.

      First, the authors should make it clear throughout the manuscript, including the title, that they are investigating Δ133p53α and Δ160p53α since there are 3 Δ133p53 isoforms (α, β, γ), and 3 Δ160p53 isoforms (α, β, γ).

      Thank you for your suggestion. We understand the importance of clearly specifying the isoforms under study. Following your suggestion, we have added α in the title, abstract, and introduction and added the following statement in the Introduction (lines 57-59): “For convenience and simplicity, we have written Δ133p53 and Δ160p53 to represent the α isoforms (Δ133p53α and Δ160p53α) throughout this manuscript.” 

      One concern is that the authors only consider and explore Δ133p53α and Δ160p53α isoforms as exclusively oncogenic and FLp53 dominant-negative while not discussing evidences of different activities. Indeed, other manuscripts have also shown that Δ133p53α is non-oncogenic and non-mutagenic, do not antagonize every single FLp53 functions and are sometimes associated with good prognosis. To cite a few examples:

      (1) Hofstetter G. et al. D133p53 is an independent prognostic marker in p53 mutant advanced serous ovarian cancer. Br. J. Cancer 2011, 105, 15931599.

      (2) Bischof, K. et al. Influence of p53 Isoform Expression on Survival in HighGrade Serous Ovarian Cancers. Sci. Rep. 2019, 9,5244.

      (3) Knezovi´c F. et al. The role of p53 isoforms' expression and p53 mutation status in renal cell cancer prognosis. Urol. Oncol. 2019, 37, 578.e1578.e10.

      (4) Gong, L. et al. p53 isoform D113p53/D133p53 promotes DNA doublestrand break repair to protect cell from death and senescence in response to DNA damage. Cell Res. 2015, 25, 351-369.

      (5) Gong, L. et al. p53 isoform D133p53 promotes efficiency of induced pluripotent stem cells and ensures genomic integrity during reprogramming. Sci. Rep. 2016, 6, 37281.

      (6) Horikawa, I. et al. D133p53 represses p53-inducible senescence genes and enhances the generation of human induced pluripotent stem cells. Cell Death Differ. 2017, 24, 1017-1028.

      (7) Gong, L. p53 coordinates with D133p53 isoform to promote cell survival under low-level oxidative stress. J. Mol. Cell Biol. 2016, 8, 88-90.

      Thank you very much for your comment and for highlighting these important studies. 

      We agree that Δ133p53 isoforms exhibit complex biological functions, with both oncogenic and non-oncogenic potentials. However, our mission here was primarily to reveal the molecular mechanism for the dominant-negative effects exerted by the Δ133p53α and Δ160p53α isoforms on FLp53 for which the Δ133p53α and Δ160p53α isoforms are suitable model systems. Exploring the oncogenic potential of the isoforms is beyond the scope of the current study and we have not claimed anywhere that we are reporting that. We have carefully revised the manuscript and replaced the respective terms e.g. ‘prooncogenic activity’ with ‘dominant-negative effect’ in relevant places (e.g. line 90). We have now also added a paragraph with suitable references that introduces the oncogenic and non-oncogenic roles of the p53 isoforms.

      After reviewing the papers you cited, we are not sure that they reflect on oncogenic /non-oncogenic role of the Δ133p53α isoform in different cancer cases.  Although our study is not about the oncogenic potential of the isoforms, we have summarized the key findings below:

      (1) Hofstetter et al., 2011: Demonstrated that Δ133p53α expression improved recurrence-free and overall survival (in a p53 mutant induced advanced serous ovarian cancer, suggesting a potential protective role in this context.

      (2) Bischof et al., 2019: Found that Δ133p53 mRNA can improve overall survival in high-grade serous ovarian cancers. However, out of 31 patients, only 5 belong to the TP53 wild-type group, while the others carry TP53 mutations.

      (3) Knezović et al., 2019: Reported downregulation of Δ133p53 in renal cell carcinoma tissues with wild-type p53 compared to normal adjacent tissue, indicating a potential non-oncogenic role, but not conclusively demonstrating it.

      (4) Gong et al., 2015: Showed that Δ133p53 antagonizes p53-mediated apoptosis and promotes DNA double-strand break repair by upregulating RAD51, LIG4, and RAD52 independently of FLp53.

      (5) Gong et al., 2016: Demonstrated that overexpression of Δ133p53 promotes efficiency of cell reprogramming by its anti-apoptotic function and promoting DNA DSB repair. The authors hypotheses that this mechanism is involved in increasing RAD51 foci formation and decrease γH2AX foci formation and chromosome aberrations in induced pluripotent stem (iPS) cells, independent of FL p53.

      (6) Horikawa et al., 2017: Indicated that induced pluripotent stem cells derived from fibroblasts that overexpress Δ133p53 formed noncancerous tumors in mice compared to induced pluripotent stem cells derived from fibroblasts with complete p53 inhibition. Thus, Δ133p53 overexpression is "non- or less oncogenic and mutagenic" compared to complete p53 inhibition, but it still compromises certain p53-mediated tumor-suppressing pathways. “Overexpressed Δ133p53 prevented FL-p53 from binding to the regulatory regions of p21WAF1 and miR-34a promoters, providing a mechanistic basis for its dominant-negative

      inhibition of a subset of p53 target genes.”

      (7) Gong, 2016: Suggested that Δ133p53 promotes cell survival under lowlevel oxidative stress, but its role under different stress conditions remains uncertain.

      We have revised the Introduction to provide a more balanced discussion of Δ133p53’s dule role (lines 62-73):

      “The Δ133p53 isoform exhibit complex biological functions, with both oncogenic and non-oncogenic potentials. Recent studies demonstrate the non-oncogenic yet context-dependent role of the Δ133p53 isoform in cancer development. Δ133p53 expression has been reported to correlate with improved survival in patients with TP53 mutations(23, 24), where it promotes cell survival in a nononcogenic manner(25, 26), especially under low oxidative stress(27). Alternatively, other recent evidences emphasize the notable oncogenic functions of Δ133p53 as it can inhibit p53-dependent apoptosis by directly interacting with the FLp53 (4, 6). The oncogenic function of the newly identified Δ160p53 isoform is less known, although it is associated with p53 mutation-driven tumorigenesis(28) and in melanoma cells’ aggressiveness(10). Whether or not the Δ160p53 isoform also impedes FLp53 function in a similar way as Δ133p53 is an open question. However, these p53 isoforms can certainly compromise p53-mediated tumor suppression by interfering with FLp53 binding to target genes such as p21 and miR-34a(2, 29) by dominant-negative effect, the exact mechanism is not known.” On the figures presented in this manuscript, I have three major concerns:

      (1) Most results in the manuscript rely on the overexpression of the FLAGtagged or V5-tagged isoforms. The validation of these construct entirely depends on Supplementary figure 3 which the authors claim "rules out the possibility that the FLAG epitope might contribute to this aggregation. However, I am not entirely convinced by that conclusion. Indeed, the ratio between the "regular" isoform and the aggregates is much higher in the FLAG-tagged constructs than in the V5-tagged constructs. We can visualize the aggregates easily in the FLAG-tagged experiment, but the imaging clearly had to be overexposed (given the white coloring demonstrating saturation of the main bands) to visualize them in the V5-tagged experiments. Therefore, I am not convinced that an effect of the FLAG-tag can be ruled out and more convincing data should be added. 

      Thank you for raising this important concern. We have carefully considered your comments and have made several revisions to clarify and strengthen our conclusions.

      First, to address the potential influence of the FLAG and V5 tags on p53 isoform aggregation, we have revised Figure 2 and removed the previous Supplementary Figure 3, where non-specific antibody bindings and higher molecular weight aggregates were not clearly interpretable. In the revised Figure 2, we have removed these potential aggregates, improving the clarity and accuracy of the data.

      To further rule out any tag-related artifacts, we conducted a coimmunoprecipitation assay with FLAG-tagged FLp53 and untagged Δ133p53 and Δ160p53 isoforms. The results (now shown in the new Supplementary Figure 3) completely agree with our previous result with FLAG-tagged and V5tagged Δ133p53 and Δ160p53 isoforms and show interaction between the partners. This indicates that the FLAG / V5-tags do not influence / interfere with the interaction between FLp53 and the isoforms. We have still used FLAGtagged FLp53 as the endogenous p53 was undetectable and the FLAG-tagged FLp53 did not aggregate alone. 

      In the revised paper, we added the following sentences (Lines 146-152): “To rule out the possibility that the observed interactions between FLp53 and its isoforms Δ133p53 and Δ160p53 were artifacts caused by the FLAG and V5 antibody epitope tags, we co-expressed FLAG-tagged FLp53 with untagged Δ133p53 and Δ160p53. Immunoprecipitation assays demonstrated that FLAGtagged FLp53 could indeed interact with the untagged Δ133p53 and Δ160p53 isoforms (Supplementary Figure 3, lanes 3 and 4), confirming formation of hetero-oligomers between FLp53 and its isoforms. These findings demonstrate that Δ133p53 and Δ160p53 can oligomerize with FLp53 and with each other.”

      Additionally, we performed subcellular fractionation experiments to compare the aggregation and localization of FLAG-tagged FLp53 when co-expressed either with V5-tagged or untagged Δ133p53/Δ160p53. In these experiments, the untagged isoforms also induced FLp53 aggregation, mirroring our previous results with the tagged isoforms (Supplementary Figure 5). We’ve added this result in the revised manuscript (lines 236-245): “To exclude the possibility that FLAG or V5 tags contribute to protein aggregation, we also conducted subcellular fractionation of H1299 cells expressing FLAG-tagged FLp53 along with untagged Δ133p53 or Δ160p53 at a 1:5 ratio. The results showed (Supplementary Figure 6) a similar distribution of FLp53 across cytoplasmic, nuclear, and insoluble nuclear fractions as in the case of tagged Δ133p53 or Δ160p53 (Figure 6A to D). Notably, the aggregation of untagged Δ133p53 or Δ160p53 markedly promoted the aggregation of FLAG-tagged FLp53 (Supplementary Figure 6B and D), demonstrating that the antibody epitope tags themselves do not contribute to protein aggregation.” 

      We’ve also discussed this in the Discussion section (lines 349-356): “In our study, we primarily utilized an overexpression strategy involving FLAG/V5tagged proteins to investigate the effects of p53 isoforms Δ133p53 and Δ160p53 on the function of FLp53. To address concerns regarding potential overexpression artifacts, we performed the co-immunoprecipitation (Supplementary Figure 6) and caspase-3 and -7 activity (Figure 7) experiments with untagged Δ133p53 and Δ160p53. In both experimental systems, the untagged proteins behaved very similarly to the FLAG/V5 antibody epitopecontaining proteins (Figures 6 and 7 and Supplementary Figure 6). Hence, the C-terminal tagging of FLp53 or its isoforms does not alter the biochemical and physiological functions of these proteins.”

      In summary, the revised data set and newly added experiments provide strong evidence that neither the FLAG nor the V5 tag contributes to the observed p53 isoform aggregation.

      (2) The authors demonstrate that to visualize the dominant-negative effect, Δ133p53α and Δ160p53α must be "present in a higher proportion than FLp53 in the tetramer" and the need at least a transfection ratio 1:5 since the 1:1 ration shows no effect. However, in almost every single cell type, FLp53 is far more expressed than the isoforms which make it very unlikely to reach such stoichiometry in physiological conditions and make me wonder if this mechanism naturally occurs at endogenous level. This limitation should be at least discussed.

      Thank you for your insightful comment. However, evidence suggests that the expression levels of these isoforms such as Δ133p53, can be significantly elevated relative to FLp53 in certain physiological conditions(3, 4, 9). For example, in some breast tumors, with Δ133p53 mRNA is expressed at a much levels than FLp53, suggesting a distinct expression profile of p53 isoforms compared to normal breast tissue(4). Similarly, in non-small cell lung cancer and the A549 lung cancer cell line, the expression level of Δ133p53 transcript is significantly elevated compared to non-cancerous cells(3). Moreover, in specific cholangiocarcinoma cell lines, the Δ133p53 /TAp53 expression ratio has been reported to increase to as high as 3:1(9). These observations indicate that the dominant-negative effect of isoform Δ133p53 on FLp53 can occur under certain pathological conditions where the relative amounts of the FLp53 and the isoforms would largely vary. Since data on the Δ160p53 isoform are scarce, we infer that the long N-terminal truncated isoforms may share a similar mechanism.

      (3) Figure 5C: I am concerned by the subcellular location of the Δ133p53α and Δ160p53α as they are commonly considered nuclear and not cytoplasmic as shown here, particularly since they retain the 3 nuclear localization sequences like the FLp53 (Bourdon JC et al. 2005; Mondal A et al. 2018; Horikawa I et al, 2017; Joruiz S. et al, 2024). However, Δ133p53α can form cytoplasmic speckles (Horikawa I et al, 2017) when it colocalizes with autophagy markers for its degradation.

      The authors should discuss this issue. Could this discrepancy be due to the high overexpression level of these isoforms? A co-staining with autophagy markers (p62, LC3B) would rule out (or confirm) activation of autophagy due to the overwhelming expression of the isoform.

      Thank you for your thoughtful comments. We have thoroughly reviewed all the papers you recommended (Bourdon JC et al., 2005; Mondal A et al., 2018; Horikawa I et al., 2017; Joruiz S. et al., 2024)(4, 29, 30, 31). Among these, only the study by Bourdon JC et al. (2005) provided data regarding the localization of Δ133p53(4). Interestingly, their findings align with our observations, indicating that the protein does not exhibit predominantly nuclear localization in the Figure 8 from Jean-Christophe Bourdon et al. Genes Dev. 2005;19:2122-2137. The discrepancy may be caused by a potentially confusing statement in that paper(4).

      The localization of p53 is governed by multiple factors, including its nuclear import and export(32). The isoforms Δ133p53 and Δ160p53 contain three nuclear localization sequences (NLS)(4). However, the isoforms Δ133p53 and Δ160p53 were potentially trapped in the cytoplasm by aggregation and masking the NLS. This mechanism would prevent nuclear import. 

      Further, we acknowledge that Δ133p53 co-aggregates with autophagy substrate p62/SQSTM1 and autophagosome component LC3B in cytoplasm by autophagic degradation during replicative senescence(33). We agree that high overexpression of these aggregation-prone proteins may induce endoplasmic reticulum (ER) stress and activates autophagy(34). This could explain the cytoplasmic localization in our experiments. However, it is also critical to consider that we observed aggregates in both the cytoplasm and the nucleus (Figures 6B and E and Supplementary Figure 6B). While cytoplasmic localization may involve autophagy-related mechanisms, the nuclear aggregates likely arise from intrinsic isoform properties, such as altered protein folding, independent of autophagy. These dual localizations reflect the complex behavior of Δ133p53 and Δ160p53 isoforms under our experimental conditions.

      In the revised manuscript, we discussed this in Discussion (lines 328-335): “Moreover, the observed cytoplasmic isoform aggregates may reflect autophagy-related degradation, as suggested by the co-localization of Δ133p53 with autophagy substrate p62/SQSTM1 and autophagosome component LC3B(33). High overexpression of these aggregation-prone proteins could induce endoplasmic reticulum stress and activate autophagy(34). Interestingly, we also observed nuclear aggregation of these isoforms (Figure 6B and E and Supplementary Figure 6B), suggesting that distinct mechanisms, such as intrinsic properties of the isoforms, may govern their localization and behavior within the nucleus. This dual localization underscores the complexity of Δ133p53 and Δ160p53 behavior in cellular systems.”

      Minor concerns:

      -  Figure 1A: the initiation of the "Δ140p53" is shown instead of "Δ40p53"

      Thank you! The revised Figure 1A has been created in the revised paper.

      -  Figure 2A: I would like to see the images cropped a bit higher, so the cut does not happen just above the aggregate bands

      Thank you for this suggestion. We’ve changed the image and the new Figure 2 has been shown in the revised paper.

      -  Figure 3C: what ratio of FLp53/Delta isoform was used?

      We have added the ratio in the figure legend of Figure 3C (lines 845-846) “Relative DNA-binding of the FLp53-FLAG protein to the p53-target gene promoters in the presence of the V5-tagged protein Δ133p53 or Δ160p53 at a 1: 1 ratio.”

      -  Figure 3C suggests that the "dominant-negative" effect is mostly senescencespecific as it does not affect apoptosis target genes, which is consistent with Horikawa et al, 2017 and Gong et al, 2016 cited above. Furthermore, since these two references and the others from Gong et al. show that Δ133p53α increases DNA repair genes, it would be interesting to look at RAD51, RAD52 or Lig4, and maybe also induce stress.

      Thank you for your thoughtful comments and suggestions. In Figure 3C, the presence of Δ133p53 or Δ160p53 only significantly reduced the binding of FLp53 to the p21 promoter. However, isoforms Δ133p53 and Δ160p53 demonstrated a significant loss of DNA-binding activity at all four promoters: p21, MDM2, and apoptosis target genes BAX and PUMA (Figure 3B). This result suggests that Δ133p53 and Δ160p53 have the potential to influence FLp53 function due to their ability to form hetero-oligomers with FLp53 or their intrinsic tendency to aggregate. To further investigate this, we increased the isoform to FLp53 ratio in Figure 4, which demonstrate that the isoforms Δ133p53 and Δ160p53 exert dominant-negative effects on the function of FLp53. 

      These results demonstrate that the isoforms can compromise p53-mediated pathways, consistent with Horikawa et al. (2017), which showed that Δ133p53α overexpression is "non- or less oncogenic and mutagenic" compared to complete p53 inhibition, but still affects specific tumor-suppressing pathways. Furthermore, as noted by Gong et al. (2016), Δ133p53’s anti-apoptotic function under certain conditions is independent of FLp53 and unrelated to its dominantnegative effects.

      We appreciate your suggestion to investigate DNA repair genes such as RAD51, RAD52, or Lig4, especially under stress conditions. While these targets are intriguing and relevant, we believe that our current investigation of p53 targets in this manuscript sufficiently supports our conclusions regarding the dominant-negative effect. Further exploration of additional p53 target genes, including those involved in DNA repair, will be an important focus of our future studies.

      - Figure 5A and B: directly comparing the level of FLp53 expressed in cytoplasm or nucleus to the level of Δ133p53α and Δ160p53α expressed in cytoplasm or nucleus does not mean much since these are overexpressed proteins and therefore depend on the level of expression. The authors should rather compare the ratio of cytoplasmic/nuclear FLp53 to the ratio of cytoplasmic/nuclear Δ133p53α and Δ160p53α.

      Thank you very much for this valuable suggestion. In the revised paper, Figure 5B has been recreated.  Changes have been made in lines 214215: “The cytoplasm-to-nucleus ratio of Δ133p53 and Δ160p53 was approximately 1.5-fold higher than that of FLp53 (Figure 5B).” 

      Referees cross-commenting

      I agree that the system needs to be improved to be more physiological.

      Just to precise, the D133 and D160 isoforms are not truncated mutants, they are naturally occurring isoforms expressed in almost every normal human cell type from an internal promoter within the TP53 gene.

      Using overexpression always raises concerns, but in this case, I am even more careful because the isoforms are almost always less expressed than the FLp53, and here they have to push it 5 to 10 times more expressed than the FLp53 to see the effect which make me fear an artifact effect due to the overwhelming overexpression (which even seems to change the normal localization of the protein).

      To visualize the endogenous proteins, they will have to change cell line as the H1299 they used are p53 null.

      Thank you for these comments. We’ve addressed the motivation of overexpression in the above responses. We needed to use the plasmid constructs in the p53-null cells to detect the proteins but the expression level was certainly not ‘overwhelmingly high’. 

      First, we tried the A549 cells (p53 wild-type) under DNA damage conditions, but the endogenous p53 protein was undetectable. Second, several studies reported increased Δ133p53 level compared to wild-type p53 and that it has implications in tumor development(2, 3, 4, 9). Third, the apoptosis activity of H1299 cells overexpressing p53 proteins was analyzed in the revised manuscript (Figure 7). The apoptotic activity induced by FLp53 expression was approximately 2.5 times higher than that of the control vector under identical plasmid DNA transfection conditions (Figure 7). These results rule out the possibility that the plasmid-based expression of p53 and its isoforms introduced artifacts in the results. We’ve discussed this in the Results section (lines 254269).

      Reviewer #3 (Significance):

      Overall, the paper is interesting particularly considering the range of techniques used which is the main strength.

      The main limitation to me is the lack of contradictory discussion as all argumentation presents Δ133p53α and Δ160p53α exclusively as oncogenic and strictly FLp53 dominant-negative when, particularly for Δ133p53α, a quite extensive literature suggests a not so clear-cut activity.

      The aggregation mechanism is reported for the first time for Δ133p53α and Δ160p53α, although it was already published for Δ40p53α, Δ133p53β or in mutant p53.

      This manuscript would be a good basic research addition to the p53 field to provide insight in the mechanism for some activities of some p53 isoforms.

      My field of expertise is the p53 isoforms which I have been working on for 11 years in cancer and neuro-degenerative diseases

      Thank you very much for your positive and critical comments. We’ve included a fair discussion on the oncogenic and non-oncogenic function of Δ133p53 in the Introduction following your suggestion (lines 62-73). 

      References

      (1) Pitolli C, Wang Y, Candi E, Shi Y, Melino G, Amelio I. p53-Mediated Tumor Suppression: DNA-Damage Response and Alternative Mechanisms. Cancers 11,  (2019).

      (2) Fujita K, et al. p53 isoforms Delta133p53 and p53beta are endogenous regulators of replicative cellular senescence. Nature cell biology 11, 1135-1142 (2009).

      (3) Fragou A, et al. Increased Δ133p53 mRNA in lung carcinoma corresponds with reduction of p21 expression. Molecular medicine reports 15, 1455-1460 (2017).

      (4) Bourdon JC, et al. p53 isoforms can regulate p53 transcriptional activity. Genes & development 19, 2122-2137 (2005).

      (5) Ghosh A, Stewart D, Matlashewski G. Regulation of human p53 activity and cell localization by alternative splicing. Molecular and cellular biology 24, 7987-7997 (2004).

      (6) Aoubala M, et al. p53 directly transactivates Δ133p53α, regulating cell fate outcome in response to DNA damage. Cell death and differentiation 18, 248-258 (2011).

      (7) Marcel V, et al. p53 regulates the transcription of its Delta133p53 isoform through specific response elements contained within the TP53 P2 internal promoter. Oncogene 29, 2691-2700 (2010).

      (8) Zhao L, Sanyal S. p53 Isoforms as Cancer Biomarkers and Therapeutic Targets. Cancers 14,  (2022).

      (9) Nutthasirikul N, Limpaiboon T, Leelayuwat C, Patrakitkomjorn S, Jearanaikoon P. Ratio disruption of the ∆133p53 and TAp53 isoform equilibrium correlates with poor clinical outcome in intrahepatic cholangiocarcinoma. International journal of oncology 42, 1181-1188 (2013).

      (10) Tadijan A, et al. Altered Expression of Shorter p53 Family Isoforms Can Impact Melanoma Aggressiveness. Cancers 13,  (2021).

      (11) Aubrey BJ, Kelly GL, Janic A, Herold MJ, Strasser A. How does p53 induce apoptosis and how does this relate to p53-mediated tumour suppression? Cell death and differentiation 25, 104-113 (2018).

      (12) Ghorbani N, Yaghubi R, Davoodi J, Pahlavan S. How does caspases regulation play role in cell decisions? apoptosis and beyond. Molecular and cellular biochemistry 479, 1599-1613 (2024).

      (13) Petronilho EC, et al. Oncogenic p53 triggers amyloid aggregation of p63 and p73 liquid droplets. Communications chemistry 7, 207 (2024).

      (14) Forget KJ, Tremblay G, Roucou X. p53 Aggregates penetrate cells and induce the coaggregation of intracellular p53. PloS one 8, e69242 (2013).

      (15) Farmer KM, Ghag G, Puangmalai N, Montalbano M, Bhatt N, Kayed R. P53 aggregation, interactions with tau, and impaired DNA damage response in Alzheimer's disease. Acta neuropathologica communications 8, 132 (2020).

      (16) Arsic N, et al. Δ133p53β isoform pro-invasive activity is regulated through an aggregation-dependent mechanism in cancer cells. Nature communications 12, 5463 (2021).

      (17) Melo Dos Santos N, et al. Loss of the p53 transactivation domain results in high amyloid aggregation of the Δ40p53 isoform in endometrial carcinoma cells. The Journal of biological chemistry 294, 9430-9439 (2019).

      (18) Mestrom L, et al. Artificial Fusion of mCherry Enhances Trehalose Transferase Solubility and Stability. Applied and environmental microbiology 85,  (2019).

      (19) Kaba SA, Nene V, Musoke AJ, Vlak JM, van Oers MM. Fusion to green fluorescent protein improves expression levels of Theileria parva sporozoite surface antigen p67 in insect cells. Parasitology 125, 497-505 (2002).

      (20) Snapp EL, et al. Formation of stacked ER cisternae by low affinity protein interactions. The Journal of cell biology 163, 257-269 (2003).

      (21) Jain RK, Joyce PB, Molinete M, Halban PA, Gorr SU. Oligomerization of green fluorescent protein in the secretory pathway of endocrine cells. The Biochemical journal 360, 645-649 (2001).

      (22) Campbell RE, et al. A monomeric red fluorescent protein. Proceedings of the National Academy of Sciences of the United States of America 99, 7877-7882 (2002).

      (23) Hofstetter G, et al. Δ133p53 is an independent prognostic marker in p53 mutant advanced serous ovarian cancer. British journal of cancer 105, 1593-1599 (2011).

      (24) Bischof K, et al. Influence of p53 Isoform Expression on Survival in High-Grade Serous Ovarian Cancers. Scientific reports 9, 5244 (2019).

      (25) Gong L, et al. p53 isoform Δ113p53/Δ133p53 promotes DNA double-strand break repair to protect cell from death and senescence in response to DNA damage. Cell research 25, 351-369 (2015).

      (26) Gong L, et al. p53 isoform Δ133p53 promotes efficiency of induced pluripotent stem cells and ensures genomic integrity during reprogramming. Scientific reports 6, 37281 (2016).

      (27) Gong L, Pan X, Yuan ZM, Peng J, Chen J. p53 coordinates with Δ133p53 isoform to promote cell survival under low-level oxidative stress. Journal of molecular cell biology 8, 88-90 (2016).

      (28) Candeias MM, Hagiwara M, Matsuda M. Cancer-specific mutations in p53 induce the translation of Δ160p53 promoting tumorigenesis. EMBO reports 17, 1542-1551 (2016).

      (29) Horikawa I, et al. Δ133p53 represses p53-inducible senescence genes and enhances the generation of human induced pluripotent stem cells. Cell death and differentiation 24, 1017-1028 (2017).

      (30) Mondal AM, et al. Δ133p53α, a natural p53 isoform, contributes to conditional reprogramming and long-term proliferation of primary epithelial cells. Cell death & disease 9, 750 (2018).

      (31) Joruiz SM, Von Muhlinen N, Horikawa I, Gilbert MR, Harris CC. Distinct functions of wild-type and R273H mutant Δ133p53α differentially regulate glioblastoma aggressiveness and therapy-induced senescence. Cell death & disease 15, 454 (2024).

      (32) O'Brate A, Giannakakou P. The importance of p53 location: nuclear or cytoplasmic zip code? Drug resistance updates : reviews and commentaries in antimicrobial and anticancer chemotherapy 6, 313-322 (2003).

      (33) Horikawa I, et al. Autophagic degradation of the inhibitory p53 isoform Δ133p53α as a regulatory mechanism for p53-mediated senescence. Nature communications 5, 4706 (2014).

      (34) Lee H, et al. IRE1 plays an essential role in ER stress-mediated aggregation of mutant huntingtin via the inhibition of autophagy flux. Human molecular genetics 21, 101-114 (2012).

    1. Author Response

      Reviewer #1 (Public Review):

      Weaknesses:

      1) The authors should better review what we know of fungal Drosophila microbiota species as well as the ecology of rotting fruit. Are the microbiota species described in this article specific to their location/setting? It would have been interesting to know if similar species can be retrieved in other locations using other decaying fruits. The term 'core' in the title suggests that these species are generally found associated with Drosophila but this is not demonstrated. The paper is written in a way that implies the microbiota members they have found are universal. What is the evidence for this? Have the fungal species described in this paper been found in other studies? Even if this is not the case, the paper is interesting, but there should be a discussion of how generalizable the findings are.

      The reviewer inquires as to whether the microbial species described in this article are ubiquitously associated with Drosophila or not. Indeed, most of the microbes described in this manuscript are generally recognized as species associated with Drosophila spp. For example, species such as Hanseniaspora uvarum, Pichia kluyveri, and Starmerella bacillaris have been detected in or isolated from Drosophila spp. collected in European countries as well as the United States and Oceania (Chandler et al., 2012; Solomon et al., 2019). As for the bacteria, species belonging to the genera Pantoea, Lactobacillus, Leuconostoc, and Acetobacter have also previously been detected in wild Drosophila spp. (Chandler et al., 2011). These elucidations will be incorporated into our revised manuscript.

      Nevertheless, the term “core” in the manuscript title may lead to misunderstanding, as the generality does not ensure the ubiquitous presence of these microbial species in every individual fly. Considering this point, we will replace the term with an expression more appropriate to our context.

      2) Can the authors clearly demonstrate that the microbiota species that develop in the banana trap are derived from flies? Are these species found in flies in the wild? Did the authors check that the flies belong to the D. melanogaster species and not to the sister group D. simulans?

      Can the authors clearly demonstrate that the microbiota species that develop in the banana trap are derived from flies? Are these species found in flies in the wild?

      The reviewer asked whether the microbial species identified in the fermented banana samples were derived from flies. To address this question, additional experiments under more controlled conditions, such as the inoculation of specific species of wild flies onto fresh bananas, would be needed. Nevertheless, the microbes may potentially originate from wild flies, as supported by the literature cited in our response to the Weakness 1).

      Alternative sources for microbial provenance also merit consideration. For example, microbial entities may be inherently present in unfermented bananas through the infiltration of peel injuries (lines 1141-1142 of the original manuscript). In addition, they could be introduced by insects other than flies, given that both rove beetles (Staphylinidae) and sap beetles (Nitidulidae) were observed in some of the traps. These possibilities will be incorporated into the 'MATERIALS AND METHODS' and 'DISCUSSION' sections of our revised manuscript.

      Did the authors check that the flies belong to the D. melanogaster species and not to the sister group D. simulans?

      Our sampling strategy was designed to target not only D. melanogaster but also other domestic Drosophila species, such as D. simulans, that inhabit human residential areas. After adult flies were caught in each trap, we identified the species as shown in Table S1, thereby showing the presence of either or both D. melanogaster and D. simulans. We will provide these descriptions in MATERIALS AND METHODS and DISCUSSION.

      3) Did the microarrays highlight a change in immune genes (ex. antibacterial peptide genes)? Whatever the answer, this would be worth mentioning. The authors described their microarray data in terms of fed/starved in relation to the Finke article. They should clarify if they observed significant differences between species (differences between species within bacteria or fungi, and more generally differences between bacteria versus fungi).

      Did the microarrays highlight a change in immune genes (ex. antibacterial peptide genes)? Whatever the answer, this would be worth mentioning.

      Regarding the antimicrobial peptide genes, statistical comparisons of our RNA-seq data across different conditions were impracticable because most of them showed low expression levels (refer to Author response table 1, which exhibits the RNA-seq data of the yeast-fed larvae; similar expression profiles were observed in the bacteria-fed larvae). While a subset of genes exhibited significantly elevated expression in the non-supportive conditions relative to the supportive ones, this can be due to intra-sample variability rather than due to distinct nutritional environments. Therefore, it would be difficult to discuss a change in immune genes in the paper. Additionally, the previous study that conducted larval microarray analysis (Zinke et al., 2002) did not explicitly focus on immune genes.

      Author response table 1.

      Antimicrobial peptide genes are not up-regulated by any of the microbes. Antimicrobial peptides gene expression profiles of whole bodies of first-instar larvae fed on yeasts. TPM values of all samples and comparison results of gene expression levels in the larvae fed on supportive and non-supportive yeasts are shown. Antibacterial peptide genes mentioned in Hanson and Lemaitre, 2020 are listed. NA or na, not available.

      They should clarify if they observed significant differences between species (differences between species within bacteria or fungi, and more generally differences between bacteria versus fungi).

      We did not observe significant differences between species within bacteria or fungi, or between bacteria and fungi. For example, the gene expression profiles of larvae fed on the various supporting microbes showed striking similarities to each other, as evidenced by the heat map showing the expression of all genes detected in larvae fed either yeast or bacteria (Author response image 1). Similarities were also observed among larvae fed on distinct non-supporting microbes.

      Author response image 1.

      Gene expression profiles of larvae fed on the various supporting microbes show striking similarities to each other. Heat map showing the gene expression of the first-instar larvae that fed on yeasts or bacteria. Freshly hatched germ-free larvae were placed on banana agar inoculated with each microbe and collected after 15 h feeding to examine gene expression of the whole body. Note that data presented in Figures 3A and 4C in the original manuscript, which are obtained independently, are combined to generate this heat map. The labels under the heat map indicate the microbial species fed to the larvae, with three samples analyzed for each condition. The lactic acid bacteria (“LAB”) include Lactiplantibacillus plantarum and Leuconostoc mesenteroides, while the lactic acid bacterium (“AAB”) represents Acetobacter orientalis. “LAB + AAB” signifies mixtures of the AAB and either one of the LAB species. The asterisk in the label highlights a sample in a “LAB” condition (Leuconostoc mesenteroides), which clustered separately from the other “LAB” samples. Brown abbreviations of scientific names are for the yeast-fed conditions. H. uva, Hanseniaspora uvarum; K. hum, Kazachstania humilis; M. asi, Martiniozyma asiatica; S. cra, Saccharomycopsis crataegensis; P. klu, Pichia kluyveri; S. bac, Starmerella bacillaris; S. cer, S. cerevisiae BY4741 strain.

      Only a handful of genes showed different expression patterns between larvae fed on yeast and those fed on bacteria, without any enrichment for specialized gene functions. Thus, it is challenging to discuss the potential differential impacts, if any, of yeast and bacteria on larval growth.

      4) The whole paper - and this is one of its merits - points to a role of the Drosophila larval microbiota in processing the fly food. Are these bacterial and fungal species found in the gut of larvae/adults? Are these species capable of establishing a niche in the cardia of adults as shown recently in the Ludington lab (Dodge et al.,)? Previous studies have suggested that microbiota members stimulate the Imd pathway leading to an increase in digestive proteases (Erkosar/Leulier). Are the microbiota species studied here affecting gut signaling pathways beyond providing branched amino acids?

      The whole paper - and this is one of its merits - points to a role of the Drosophila larval microbiota in processing the fly food. Are these bacterial and fungal species found in the gut of larvae/adults? Are these species capable of establishing a niche in the cardia of adults as shown recently in the Ludington lab (Dodge et al.,)?

      Although we did not investigate the microbiota in the gut of either larvae or adults, we did compare the microbiota within surface-sterilized larvae or adults with those in food samples. We found that adult flies and early-stage food sources, as well as larvae and late-stage food sources, harbor similar microbial species (Figure 1F). Additionally, previous examinations of the gut microbiota in wild adult flies have identified microbial species or taxa congruent with those we isolated from our foods (Chandler et al., 2011; Chandler et al., 2012). We have elaborated on this in our response to Weakness 1).

      While we did not investigate whether these species are capable of establishing a niche in the cardia of adults, we will cite the study by Dodge et al., 2023 in our revised manuscript and discuss the possibility that predominant microbes in adult flies may show a propensity for colonization.

      Previous studies have suggested that microbiota members stimulate the Imd pathway leading to an increase in digestive proteases (Erkosar/Leulier). Are the microbiota species studied here affecting gut signaling pathways beyond providing branched amino acids?

      The reviewer inquires whether the supportive microbes in our study stimulate gut Imd signaling pathways and induce the expression of digestive protease genes, as demonstrated in a previous study (Erkosar et al., 2015). According to our RNA-seq data, it seems unlikely that the supportive microbes stimulate the signaling pathway. Figures contained in Author response image 2 provide the statistical comparisons of expression levels for seven protease genes between the supportive and the non-supportive conditions. These genes did not exhibit a consistent upregulation in the presence of the supportive microbes (H. uva or K. hum in Author response image 2A; Le mes + A. ori in Author response image 2B). Rather, they exhibited a tendency to be upregulated under the non-supportive microbes (St. bac or Pi. klu in Author response image 2A; La. pla in Author response image 2B).

      Author response image 2.

      Most of the peptidase genes reported by Erkosar et al., 2015 are more highly expressed under the non-supportive conditions than the supportive conditions. Comparison of the expression levels of seven peptidase genes derived from the RNA-seq analysis of yeast-fed (A) or bacteria-fed (B) first-instar larvae. A previous report demonstrated that the expression of these genes is upregulated upon association with a strain of Lactiplantibacillus plantarum, and that the PGRP-LE/Imd/Relish signaling pathway, at least partially, mediates the induction (Erkosar et al., 2015). H. uva, Hanseniaspora uvarum; K. hum, Kazachstania humilis; P. klu, Pichia kluyveri; S. bac, Starmerella bacillaris; La. pla, Lactiplantibacillus plantarum; Le. mes, Leuconostoc mesenteroides; A. ori, Acetobacter orientalis; ns, not significant.

      Reviewer #2 (Public Review):

      Weaknesses:

      The experimental setting that, the authors think, reflects host-microbe interactions in nature is one of the key points. However, it is not explicitly mentioned whether isolated microbes are indeed colonized in wild larvae of Drosophila melanogaster who eat bananas. Another matter is that this work is rather descriptive and a few mechanical insights are presented. The evidence that the nutritional role of BCAAs is incomplete, and molecular level explanation is missing in "interspecies interactions" between lactic acid bacteria (or yeast) and acetic acid bacteria that assure their inhabitation. Apart from these matters, the future directions or significance of this work could be discussed more in the manuscript.

      The experimental setting that, the authors think, reflects host-microbe interactions in nature is one of the key points. However, it is not explicitly mentioned whether isolated microbes are indeed colonized in wild larvae of Drosophila melanogaster who eat bananas.

      The reviewer asks whether the isolated microbes were colonized in the larval gut. Previous studies on microbial colonization associated with Drosophila have predominantly focused on adults (Pais et al. PLOS Biology, 2018), rather than larval stages. Developing larvae continually consume substrates which are already subjected to microbial fermentation and abundant in live microbes until the end of the feeding larval stage. Therefore, we consider it difficult to discuss microbial colonization in the larval gut. We will add this point in the DISCUSSION of the revised manuscript.

      Another matter is that this work is rather descriptive and a few mechanical insights are presented. The evidence that the nutritional role of BCAAs is incomplete, and molecular level explanation is missing in "interspecies interactions" between lactic acid bacteria (or yeast) and acetic acid bacteria that assure their inhabitation.

      While recognizing the importance of comprehensive mechanistic analysis, this study includes all experimentally feasible data. Elucidation of more detailed molecular mechanisms lies beyond the scope of this study and will be the subject of future research.

      Regarding the nutritional role of BCAAs, the incorporation of BCAAs enabled larvae fed with the non-supportive yeast to grow to the second instar. This observation suggests that consumption of BCAAs upregulates diverse genes involved in cellular growth processes in larvae. We have discussed the hypothetical interaction between lactic acid bacteria (LAB) and acetic acid bacteria (AAB) in the manuscript (lines 402-405): LAB may facilitate lactate provision to AAB, consequently enhancing the biosynthesis of essential nutrients such as amino acids. To test this hypothesis, future experiments will include the supplementation of lactic acid to AAB culture plates and the co-inoculating LAB mutant strains defective in lactate production with AABs, to assess both larval growth and continuous larval association with AABs. With respect to AAB-yeast interactions, metabolites released from yeast cells might benefit AAB growth, and this possibility will be investigated through the supplementation of AAB culture plates with candidate metabolites identified in the cell suspension supernatants of the late-stage yeasts.

      Apart from these matters, the future directions or significance of this work could be discussed more in the manuscript.

      We appreciate the reviewer's recommendations and will include additional descriptions regarding these aspects in the DISCUSSION section.

      Reviewer #3 (Public Review):

      Weaknesses:

      Despite describing important findings, I believe that a more thorough explanation of the experimental setup and the steps expected to occur in the exposed diet over time, starting with natural "inoculation" could help the reader, in particular the non-specialist, grasp the rationale and main findings of the manuscript. When exactly was the decision to collect early-stage samples made? Was it when embryos were detected in some of the samples? What are the implications of bacterial presence in the no-fly traps? These samples also harbored complex microbial communities, as revealed by sequencing. Were these samples colonized by microbes deposited with air currents? Were they the result of flies that touched the material but did not lay eggs? Could the traps have been visited by other insects? Another interesting observation that could be better discussed is the fact that adult flies showed a microbiome that more closely resembles that of the early-stage diet, whereas larvae have a more late-stage-like microbiome. It is easy to understand why the microbiome of the larvae would resemble that of the late-stage foods, but what about the adult microbiome? Authors should discuss or at least acknowledge the fact that there must be a microbiome shift once adults leave their food source. Lastly, the authors should provide more details about the metabolomics experiments. For instance, how were peaks assigned to leucine/isoleucine (as well as other compounds)? Were both retention times and MS2 spectra always used? Were standard curves produced? Were internal, deuterated controls used?

      When exactly was the decision to collect early-stage samples made? Was it when embryos were detected in some of the samples?

      We collected traps and early-stage samples 2.5 days after setting up the traps. This time frame was determined by pilot experiments. A shorter collection time resulted in a greater likelihood of obtaining no-fly traps, whereas a longer collection time caused larval overcrowding, as well as adults’ deaths from drowning in the liquid seeping out of fruits. These procedural details will be delineated in the MATERIALS AND METHODS section of the revised manuscript.

      What are the implications of bacterial presence in the no-fly traps? These samples also harbored complex microbial communities, as revealed by sequencing. Were these samples colonized by microbes deposited with air currents? Were they the result of flies that touched the material but did not lay eggs? Could the traps have been visited by other insects?

      We assume that the origins of the microbes detected in the no-fly trap foods vary depending on the species. For instance, Colletotrichum musae, the fungus that causes banana anthracnose, may have been present in fresh bananas before trap placement. The filamentous fungi could have originated from airborne spores, but they could also have been introduced by insects that feed on these fungi. We will include these possibilities in the DISCUSSION section of the revised manuscript.

      Another interesting observation that could be better discussed is the fact that adult flies showed a microbiome that more closely resembles that of the early-stage diet, whereas larvae have a more late-stage-like microbiome. It is easy to understand why the microbiome of the larvae would resemble that of the late-stage foods, but what about the adult microbiome? Authors should discuss or at least acknowledge the fact that there must be a microbiome shift once adults leave their food source.

      We are grateful for the reviewer's insightful suggestions regarding shifts in the adult microbiome. We plan to include in the DISCUSSION section of the revised manuscript the possibility that the microbial composition may change substantially during pupal stages and that microbes obtained after eclosion could potentially form the adult gut microbiota.

      Lastly, the authors should provide more details about the metabolomics experiments. For instance, how were peaks assigned to leucine/isoleucine (as well as other compounds)? Were both retention times and MS2 spectra always used? Were standard curves produced? Were internal, deuterated controls used?

      We appreciate the reviewer's advice. Detailed methods of the metabolomic experiments will be included in our revised manuscript.

    1. Author response:

      Reviewer #1 (Public review):

      (1) Some details are not described for experimental procedures. For example, what were the pharmacological drugs dissolved in, and what vehicle control was used in experiments? How long were pharmacological drugs added to cells?

      We apologise for the oversight. These details have now been added to the methods section of the manuscript as well as to the relevant figure legends.

      Briefly, latrunculin was used at a final concentration of 250 nM and Y27632 at a final concentration of 50 μM. Both drugs were dissolved in DMSO. The vehicle controls were effected with the highest final concentration of DMSO of the two drugs.

      The details of the drug treatments and their duration was added to the methods and to figures 6, S10, and S12.

      (2) Details are missing from the Methods section and Figure captions about the number of biological and technical replicates performed for experiments. Figure 1C states the data are from 12 beads on 7 cells. Are those same 12 beads used in Figure 2C? If so, that information is missing from the Figure 2C caption. Similarly, this information should be provided in every figure caption so the reader can assess the rigor of the experiments. Furthermore, how heterogenous would the bead displacements be across different cells? The low number of beads and cells assessed makes this information difficult to determine.

      We apologise for the oversight. We have now added this data to the relevant figure panels.

      To gain a further understanding of the heterogeneity of bead displacements across cells, we have replotted the relevant graphs using different colours to indicate different cells. This reveals that different cells appear to behave similarly and that the behaviour appears controlled by distance to the indentation or the pipette tip rather than cell identity.

      We agree with the reviewer that the number of cells examined is low. This is due to the challenging nature of the experiments that signifies that many attempts are necessary to obtain a successful measurement.

      The experiments in Fig 1C are a verification of a behaviour documented in a previous publication [1]. Here, we just confirm the same behaviour and therefore we decided that only a small number of cells was needed.

      The experiments in Fig 2C (that allow for a direct estimation of the cytoplasm’s hydraulic permeability) require formation of a tight seal between the glass micropipette and the cell, something known as a gigaseal in electrophysiology. The success rate of this first step is 10-30% of attempts for an experienced experimenter. The second step is forming a whole cell configuration, in which a hydraulic link is formed between the cell and the micropipette. This step has a success rate of ~ 50%. Whole cell links are very sensitive to any disturbance. After reaching the whole cell configuration, we applied relatively high pressures that occasionally resulted in loss of link between the cell and the micropipette. In summary, for the 12 successful measurements, hundreds of unsuccessful attempts were carried out.

      (3) The full equation for displacement vs. time for a poroelastic material is not provided. Scaling laws are shown, but the full equation derived from the stress response of an elastic solid and viscous fluid is not shown or described.

      We thank the reviewer for this comment. Based on our experiments, we found that the cytoplasm behaves as a poroelastic material. However, to understand the displacements of the cell surface in response to localised indentation, we show that we also need to take the tension of the sub membranous cortex into account. In summary, the interplay between cell surface tension generated by the cortex and the poroelastic cytoplasm controls the cell behaviour. To our knowledge, no simple analytical solutions to this type of problem exist.

      In Fig 1, we show that the response of the cell to local indentation is biphasic with a short time-scale displacement followed by a longer time-scale one. In Figs 2 and 3, we directly characterise the kinetics of cell surface displacement in response to microinjection of fluid. These kinetics are consistent with the long time-scale displacement but not the short time-scale one. Scaling considerations led us to propose that tension in the cortex may play a role in mediating the short time-scale displacement. To verify this hypothesis, we have now added new data showing that the length-scale of an indentation created by an AFM probe depends on tension in the cortex (Fig S5).

      In a previous publication [2], we derived the temporal dynamics of cell surface displacement for a homogenous poroelastic material in response to a change in osmolarity. In the current manuscript, the composite nature of the cell (membrane, cortex, cytoplasm) needs to be taken into account as well as a realistic cell shape. Therefore, we did not attempt to provide an analytical solution for the displacement of the cell surface versus time in the current work. Instead, we turned to finite element modelling to show that our observations are qualitatively consistent with a cell that comprises a tensed sub membranous actin cortex and a poroelastic cytoplasm (Fig 4). We have now added text to make this clearer for the reader.

      Reviewer #2 (Public review):

      Comments & Questions:

      The authors state, "Next, we sought to quantitatively understand how the global cellular response to local indentation might arise from cellular poroelasticity." However, the evidence presented in the following paragraph appears more qualitative than strictly quantitative. For instance, the length scale estimate of ~7 μm is only qualitatively consistent with the observed ~10 μm, and the timescale 𝜏𝑧 ≈ 500 ms is similarly described as "qualitatively consistent" with experimental observations. Strengthening this point would benefit from more direct evidence linking the short timescale to cell surface tension. Have you tried perturbing surface tension and examining its impact on this short-timescale relaxation by modulating acto-myosin contractility with Y-27632, depolymerizing actin with Latrunculin, or applying hypo/hyperosmotic shocks?

      Upon rereading our manuscript, we agree with the reviewer that some of our statements are too strong. We have now moderated these and clarified the goal of that section of the text.

      The reviewer asks if we have examined the effect of various perturbations on the short time-scale displacements. In our experimental conditions, we cannot precisely measure the time-scale of the fast relaxation because its duration is comparable to the frame rate of our image acquisition. However, we examined the amplitude of the displacement of the first phase in response to sucrose treatment and we have carried out new experiments in which we treat cells with 250nM Latrunculin to partially depolymerise cellular F-actin. Neither of these treatments had an impact on the amplitude of vertical displacements (Author response image 1).

      The absence of change in response to Latrunculin may be because the treatment decreases both the elasticity of the cytoplasm E and the cortical tension γ. As the length-scale l of the deformation of the surface scales as , the two effects of latrunculin treatment may therefore compensate one another and result in only small changes in l. We have now added this data to supplementary information and comment on this in the text.

      Author response image 1:

      Amplitude of the short time-scale displacements of beads in response to AFM indentation at δx=0µm for control cells, sucrose treated cells, and cells treated with Latrunculin B. n indicates the number of cells examined and N the number of beads.

      The reviewer’s comment also made us want to determine how cortical tension affects the length-scale of the cell surface deformation created by localised micro indentation. To isolate the role of the cortex from that of cell shape, we decided to examine rounded mitotic cells. In our experiments, we indented a mitotic cell expressing a membrane targeted GFP with a sharp AFM tip (Author response image 2).

      In our experiments, we adjusted force to generate a 2μm depth indentation and we imaged the cell profile with confocal microscopy before and during indentation. Segmentation of this data allowed us to determine the cell surface displacement resulting from indentation and measure a length scale of deformation. In control conditions, the length scale created by deformation is on the order of 1.2μm. When we inhibited myosin contractility with blebbistatin, the length-scale of deformation decreased significantly to 0.8 μm, as expected if we decrease the surface tension γ without affecting the cytoplasmic elasticity. We have now added this data to our manuscript.

      Author response image 2.

      (a) Overlay of the zx profiles of a mitotic cell before (green) and during indentation (red). The cell membrane is labelled with CellMask DeepRed. The arrowhead indicates the position of the AFM tip. Scale bar 10µm. (b) Position of the membrane along the top half of the cell before (green) and during (red) indentation. The membrane position is derived from segmentation of the data in (a). Deformation is highly localised and membrane profiles overlap at the edges. The tip position is marked by an *. (c) The difference in membrane height between pre-indentation and indentation profiles plotted in (b) with the tip located at x=0. (d) Schematic of the cell surface profile during indentation and the corresponding length scale of the deformation induced by indentation. (e) Measured length scale for an indentation ~2µm for DMSO control l=1.2±0.2µm (n=8 cells) and with blebbistatin treatment (100µM) l=0.8±0.4µm (n=9 cells) (p= 0.016

      The authors demonstrate that the second relaxation timescale increases (Figure 1, Panel D) following a hyperosmotic shock, consistent with cytoplasmic matrix shrinkage, increased friction, and consequently a longer relaxation timescale. While this result aligns with expectations, is a seven-fold increase in the relaxation timescale realistic based on quantitative estimates given the extent of volume loss?

      We thank the reviewer for this interesting question. Upon re-examining our data, we realised that the numerical values in the text related to the average rather than the median of our measurements. The median of the poroelastic time constant increases from ~0.4s in control conditions to 1.4s in sucrose, representing approximately a 3.5-fold increase.

      Previous work showed that HeLa cell volume decreases by ~40% in response to hyperosmotic shock [3]. The fluid volume fraction in cells is ~65-75%. If we assume that the water is contained in N pores of volume , we can express the cell volume as with V<sub>s</sub> the volume of the solid fraction. We can rewrite with ϕ = 0.42 -0.6. As V<sub>s</sub> does not change in response to osmotic shock, we can rewrite the volume change to obtain the change in pore size .

      The poroelastic diffusion constant scales as and the poroelastic timescale scales as . Therefore, the measured change in volume leads to a predicted increase in poroelastic diffusion time of 1.7-1.9-fold, smaller than observed in our experiments. This suggests that some intuition can be gained in a straightforward manner assuming that the cytoplasm is a homogenous porous material.

      However, the reality is more complex and the hydraulic pore size is distinct from the entanglement length of the cytoskeleton mesh, as we discussed in a previous publication [4]. When the fluid fraction becomes sufficiently small, macromolecular crowding will impact diffusion further and non-linearities will arise. We have now added some of these considerations to the discussion.

      If the authors' hypothesis is correct, an essential physiological parameter for the cytoplasm could be the permeability k and how it is modulated by perturbations, such as volume loss or gain. Have you explored whether the data supports the expected square dependency of permeability on hydraulic pore size, as predicted by simple homogeneity assumptions?

      We thank the reviewer for this comment. As discussed above, we have explored such considerations in a previous publication (see discussion in [4]). Briefly, we find that the entanglement length of the F-actin cytoskeleton does play a role in controlling the hydraulic pore size but is distinct from it. Membrane bounded organelles could also contribute to setting the pore size. In our previous publication, we derived a scaling relationship that indicates that four different length-scales contribute to setting cellular rheology: the average filament bundle length, the size distribution of particles in the cytosol, the entanglement length of the cytoskeleton, and the hydraulic pore size. Many of these length-scales can be dynamically controlled by the cell, which gives rise to complex rheology. We have now added these considerations to our discussion.

      Additionally, do you think that the observed decrease in k in mitotic cells compared to interphase cells is significant? I would have expected the opposite naively as mitotic cells tend to swell by 10-20 percent due to the mitotic overshoot at mitotic entry (see Son Journal of Cell Biology 2015 or Zlotek Journal of Cell Biology 2015).

      We thank the reviewer for this interesting question. Based on the same scaling arguments as above, we would expect that a 10-20% increase in cell volume would give rise to 10-20% increase in diffusion constant. However, we also note that metaphase leads to a dramatic reorganisation of the cell interior and in particular membrane-bounded organelles. In summary, we do not know why such a decrease could take place. We now highlight this as an interesting question for further research.

      Based on your results, can you estimate the pore size of the poroelastic cytoplasmic matrix? Is this estimate realistic? I wonder whether this pore size might define a threshold above which the diffusion of freely diffusing species is significantly reduced. Is your estimate consistent with nanobead diffusion experiments reported in the literature? Do you have any insights into the polymer structures that define this pore size? For example, have you investigated whether depolymerizing actin or other cytoskeletal components significantly alters the relaxation timescale?

      We thank the reviewer for this comment. We cannot directly estimate the hydraulic pore size from the measurements performed in the manuscript. Indeed, while we understand the general scaling laws, the pre-factors of such relationships are unknown.

      We carried out experiments aiming at estimating the hydraulic pore size in previous publications [3,4] and others have shown spatial heterogeneity of the cytoplasmic pore size [5]. In our previous experiments, we examined the diffusion of PEGylated quantum dots (14nm in hydrodynamic radius). In isosmotic conditions, these diffused freely through the cell but when the cell volume was decreased by a hyperosmotic shock, they no longer moved [3,4]. This gave an estimate of the pore radius of ~15nm.

      Previous work has suggested that F-actin plays a role in dictating this pore size but microtubules and intermediate filaments do not [4].

      There are no quantifications in Figure 6, nor is there a direct comparison with the model. Based on your model, would you expect the velocity of bleb growth to vary depending on the distance of the bleb from the pipette due to the local depressurization? Specifically, do blebs closer to the pipette grow more slowly?

      We apologise for the oversight. The quantifications are presented in Fig S10 and Fig S12. We have now modified the figure legends accordingly.

      Blebs are very heterogenous in size and growth velocity within a cell and across cells in the population in normal conditions [6]. Other work has shown that bleb size is controlled by a competition between pressure driving growth and actin polymerisation arresting it[7]. Therefore, we did not attempt to determine the impact of depressurisation on bleb growth velocity or size.

      In experiments in which we suddenly increased pressure in blebbing cells, we did notice a change in the rate of growth of blebs that occurred after we increased pressure (Author response image 3). However, the experiments are technically challenging and we decided not to perform more.

      Author response image 3:

      A. A hydraulic link is established between a blebbing cell and a pipette. At time t>0, a step increase in pressure is applied. B. Kymograph of bleb growth in a control cell (top) an in a cell subjected to a pressure increase at t=0s (bottom). Top: In control blebs, the rate of growth is slow and approximately constant over time. The black arrow shows the start of blebbing. Bottom: The black arrow shows the start of blebbing. The dashed line shows the timing of pressure application and the red arrow shows the increase in growth rate of the bleb when the pressure increase reaches the bleb. This occurs with a delay δt.

      I find it interesting that during depressurization of the interphase cells, there is no observed volume change, whereas in pressurization of metaphase cells, there is a volume increase. I assume this might be a matter of timescale, as the microinjection experiments occur on short timescales, not allowing sufficient time for water to escape the cell. Do you observe the radius of the metaphase cells decreasing later on? This relaxation could potentially be used to characterize the permeability of the cell surface.

      We thank the reviewer for this comment.

      First, we would like to clarify that both metaphase and interphase cells increase their volume in response to microinjection. The effect is easier to quantify in metaphase cells because we assume spherical symmetry and just monitor the evolution of the radius (Fig 3). However, the displacement of the beads in interphase cells (Fig 2) clearly shows that the cell volume increases in response to microinjection. For both interphase and metaphase cells, when the injection is prolonged, the membrane eventually detaches from the cortex and large blebs form until cell lysis. In contrast to the reviewer’s intuition, we never observe a relaxation in cell volume, probably because we inject fluid faster than the cell can compensate volume change through regulatory mechanisms involving ion channels.

      When we depressurise metaphase cells, we do not observe any change in volume (Fig S10). This contrasts with the increase that we observe upon pressurisation. The main difference between these two experiments is the pressure differential. During depressurisation experiments, this is the hydraulic pressure within the cell ~500Pa (Fig 6A); whereas during pressurisation experiments, this is the pressure in the micropipette, ranging from 1.4-10 kPa (Fig 3). We note in particular that, when we used the lowest pressures in our experiments, the increase in volume was very slow (see Fig 3C). Therefore, we agree with the reviewer that it is likely the magnitude of the pressure differential that explains these differences.

      I am curious about the saturation of the time lag at 30 microns from the pipette in Figure 4, Panel E for the model's prediction. A saturation which is not clearly observed in the experimental data. Could you comment on the origin of this saturation and the observed discrepancy with the experiments (Figure E panel 2)? Naively, I would have expected the time lag to scale quadratically with the distance from the pipette, as predicted by a poroelastic model and the diffusion of displacement. It seems weird to me that the beads start to move together at some distance from the pipette or else I would expect that they just stop moving. What model parameters influence this saturation? Does membrane permeability contribute to this saturation?

      We thank the reviewer for pointing this out. In our opinion, the saturation occurring at 30 microns arises from the geometry of the model. At the largest distance away from the micropipette, the cortex becomes dominant in the mechanical response of the cell because it represents an increasing proportion of the cellular material.

      To test this hypothesis, we will rerun our finite element models with a range of cell sizes. This will be added to the manuscript at a later date.

      Reviewer #3 (Public review):

      Weaknesses: I have two broad critical comments:

      (1) I sense that the authors are correct that the best explanation of their results is the passive poroelastic model. Yet, to be thorough, they have to try to explain the experiments with other models and show why their explanation is parsimonious. For example, one potential explanation could be some mechanosensitive mechanism that does not involve cytoplasmic flow; another could be viscoelastic cytoskeletal mesh, again not involving poroelasticity. I can imagine more possibilities. Basically, be more thorough in the critical evaluation of your results. Besides, discuss the potential effect of significant heterogeneity of the cell.

      We thank the reviewer for these comments and we agree with their general premise.

      Some observations could qualitatively be explained in other ways. For example, if we considered the cell as a viscoelastic material, we could define a time constant with η the viscosity and E the elasticity of the material. The increase in relaxation time with sucrose treatment could then be explained by an increase in viscosity. However, work by others has previously shown that, in the exact same conditions as our experiment, viscoelasticity cannot account for the observations[1]. In its discussion, this study proposed poroelasticity as an alternative mechanism but did not investigate that possibility. This was consistent with our work that showed that the cytoplasm behaves as a poroelastic material and not as a viscoelastic material [4]. Therefore, we decided not to consider viscoelasticity as possibility. We now explain this reasoning better and have added a sentence about a potential role for mechanotransductory processes in the discussion.

      (2) The study is rich in biophysics but a bit light on chemical/genetic perturbations. It could be good to use low levels of chemical inhibitors for, for example, Arp2/3, PI3K, myosin etc, and see the effect and try to interpret it. Another interesting question - how adhesive strength affects the results. A different interesting avenue - one can perturb aquaporins. Etc. At least one perturbation experiment would be good.

      We agree with the reviewer. In our previous studies, we already examined what biological structures affect the poroelastic properties of cells [2,4]. Therefore, the most interesting aspect to examine in our current work would be perturbations to the phenomenon described in Fig 6G and, in particular, to investigate what volume regulation mechanisms enable sustained intracellular pressure gradients. However, these experiments are particularly challenging and with very low throughput. Therefore, we feel that these are out of the scope of the present report and we mention these as promising future directions.

      References:

      (1) Rosenbluth, M. J., Crow, A., Shaevitz, J. W. & Fletcher, D. A. Slow stress propagation in adherent cells. Biophys J 95, 6052-6059 (2008). https://doi.org/10.1529/biophysj.108.139139

      (2) Esteki, M. H. et al. Poroelastic osmoregulation of living cell volume. iScience 24, 103482 (2021). https://doi.org/10.1016/j.isci.2021.103482

      (3) Charras, G. T., Mitchison, T. J. & Mahadevan, L. Animal cell hydraulics. J Cell Sci 122, 3233-3241 (2009). https://doi.org/10.1242/jcs.049262

      (4) Moeendarbary, E. et al. The cytoplasm of living cells behaves as a poroelastic material. Nat Mater 12, 253-261 (2013). https://doi.org/10.1038/nmat3517

      (5) Luby-Phelps, K., Castle, P. E., Taylor, D. L. & Lanni, F. Hindered diffusion of inert tracer particles in the cytoplasm of mouse 3T3 cells. Proc Natl Acad Sci U S A 84, 4910-4913 (1987). https://doi.org/10.1073/pnas.84.14.4910

      (6) Charras, G. T., Coughlin, M., Mitchison, T. J. & Mahadevan, L. Life and times of a cellular bleb. Biophys J 94, 1836-1853 (2008). https://doi.org/10.1529/biophysj.107.113605

      (7) Tinevez, J. Y. et al. Role of cortical tension in bleb growth. Proc Natl Acad Sci U S A 106, 18581-18586 (2009). https://doi.org/10.1073/pnas.0903353106

    1. Author response:

      Reviewer #1 (Evidence, reproducibility and clarity (Required)): 

      Summary: 

      Laura Morano and colleagues have performed a screen to identify compounds that interfere with the formation of TopBP1 condensates. TopBP1 plays a crucial role in the DNA damage response, and specifically the activation of ATR. They found that the GSK-3b inhibitor AZD2858 reduced the formation of TopBP1 condensates and activation of ATR and its downstream target CHK1 in colorectal cancer cell lines treated with the clinically relevant irinotecan active metabolite SN-38. This inhibition of TopBP1 condensates by AZD2858 was independent from its effect on GSK-3b enzymatic activity. Mechanistically, they show that AZD2858 thus can interfere with intra-S-phase checkpoint signaling, resulting in enhanced cytostatic and cytotoxic effects of SN-38 (or SN-38+Fluoracil aka FOLFIRI) in vitro in colorectal carcinoma cell lines. 

      Major comments: 

      Overall the work is rigorous and the main conclusions are convincing. However, they only show the effects of their combination treatments on colorectal cancer cell lines. I'm worried that blocking the formation of TopB1 condensates will also be detrimental in non-transformed cells. Furthermore it is somewhat disappointing that it remains unclear how AZD2858 blocks selfassembly of TopBP1 condensates, although I understand that unraveling this would be complex and somewhat out-of-reach for now. 

      We appreciate your feedback and fully recognize the importance of understanding how AZD2858 blocks the assembly of TopBP1 condensates. While we understand your disappointment, addressing this question remains a key focus for us. Keeping in mind that unravelling such a mechanism in vitro or in vivo is rather challenging, we have consulted an expert who has made efforts to predict the potential docking sites of AZD2858 on TopBP1, which may provide valuable insights for future experimental investigations. Using an AlphaFold model (no crystal or cryo-EM structure available) and looking for suitable pockets or cavities in which AZD2858 could bind, the analyses, though requiring cautious interpretation, suggested that AZD2858 may target the BRCT1 and BRCT8 domains (as shown below, two pockets n°1 and 7 with sufficient volume and surrounded by b-sheets structures like other GSK3 inhibitor) of TopBP1.

      However, these are preliminary results that require further exploration and experimental validation to confirm their significance and mechanistic implications.

      Author response image 1.

      Here are some specific points for improvement: 

      (1) The authors conclude that "These data supports [sic] the feasibility of targeting condensates formed in response to DNA damage to improve chemotherapy-based cancer treatments". To support this conclusion the authors need to show that proliferating non-transformed cells (e.g. primary cell cultures or organoids) can tolerate the combination of AZD2858 + SN-38 (or FOLFIRI) better than colorectal cancer cells. 

      We would like to thank the reviewer for this vital suggestion to prove that this combination is effective on tumor cells and not very toxic on healthy cells. We therefore used a healthy colon cell line (CCD841) and tested the efficacy of each treatment alone (FOLFIRI and AZD2858) as well as the combination FOLFIRI+AZD2858. We compared the results obtained in the CCD841 cell line with those obtained in the HCT116 colorectal cancer cell line. The results presented below show not only that each treatment alone is much less effective on CCD841 lines, but also that the combination is not synergistic.

      Author response image 2.

      Page 19 "This suggests that the combination... arrests the cell cycle before mitosis in a DNAPKsc-dependent manner." I find the remark that this arrest would be DNA-PKcs-dependent too speculative. I suppose that the authors base this claim on reference 55 but if they want to support this claim they need to prove this by adding DNA-PKcs inhibitors to their treated cells. 

      Thank you for your thoughtful comment. We agree with the reviewer that claiming the G2/M arrest is DNA-PKcs-dependent without direct experimental evidence is speculative. While we initially based this hypothesis on reference 55, we acknowledge that further experiments, such as the use of DNA-PKcs inhibitors, would be necessary to robustly support this claim.

      Given that this observation was intended as a potential explanation for the G2/M arrest observed at 6 and 12 hours of treatment with AZD2858 + SN-38 (compared to SN-38 alone), and considering that exploring this pathway is not the primary focus of our study, we have decided to remove this hypothesis from both the figure and the text to avoid any ambiguity.

      We appreciate the reviewer’s input and will consider investigating this pathway in future studies.

      (2) When discussing Figure S5B the authors claim that SN-38 + AZD2858 progressively increases the fractions of BrdU positive cells, but this is not supported by statistical analysis.

      The fractions are still very small, so I would like to see statistics on these data. Alternatively, the authors could take out this conclusion. 

      Thank you for your valuable comment. In response, we have conducted a statistical analysis (Mann-Whitney test) on the data, and the results have been added to Figure S5C for the 6-hour time point and Figure S5D for the 12-hour time point, based on three independent biological replicates. We hope this provides the necessary clarification.

      Minor comments: 

      - Page 5 Materials and methods - Cell culture. Last sentence "Add in what medium you cultured them" looks like an internal review remark and should probably be removed? 

      We apologize for this oversight. The medium has now been specified, and the sentence has been removed.

      - The numbers in all the synergy matrices (in white font) are extremely small and virtually unreadable, and visually distracting. I recommend taking these out altogether. 

      We believe that the reduction in figure quality may be due to the PDF compression, which affected the resolution of the figures. We are happy to provide high-resolution versions of the figures separately for clarity. If the issue persists even with the higher resolution, we will consider removing the numbers, as suggested.

      - The legends of the synergy matrices (for example Fig 1D, 4E, 5, 6) are often extremely small, making it difficult to understand them intuitively. Please enlarge them and label them more clearly, and use larger fonts. In the legend of Figure 5D,E a green matrix indicating % live cells is mentioned but I don't see it. Do they mean the grey matrix? 

      We have enlarged the figure legends and will provide high-resolution versions of the figures to ensure all details are clearly readable. Regarding Figure 5D,E: we acknowledge that the color may appear differently (more green or gray) depending on the display or printer settings. To avoid any confusion, we have corrected the legend to specify that the color in question is khaki, rather than green. Moreover, following suggestions of the reviewer #2, these figures have been respectively moved to Figure S6B and S6C.

      - Figure S2. Perhaps I misunderstand the PML body experiment but the authors seem to use PML body formation to support their idea that AZD2858 blocks TopBP1 condensate formation and not just any condensate formation. However, if this is the case they would need a proper positive control, i.e. an additional experimental condition in which they do see PLM bodies. 

      Arsenic is a well-known positive control for experiments involving PML bodies due to its ability to induce specific responses in PML proteins and modify PML nuclear bodies (NBs) structure and function (Jaffray et al., 2023, JCB ; Zhu et al., 1997, PNAS). Thus, we used Arsenic as a positive control and observed a significant increase in PML NBs vs the other conditions (Kruskal-Wallis test) as indicated below. We thus implemented the results in the corresponding figure S2B and text.

      Author response image 3.

      PML condensates were tested after 2 h of incubation. AZD2858 : 100nM ; SN-38 : 300nM ; Arsenic : 6µM. ****: p<0.0001 (Kruskal-Wallis test).

      - The quantification of the flow cytometry data needs to be clarified. I find it strange that in the figures (for example Figure 3A and 3C) representative examples are shown of apparently 3 replicates, and that the percentages shown in these examples are then the given in the text as the overall numbers; for example on page 18 "...BrdU incorporation increased from 16.11% (SN38 alone) to 41.83% (combination)...". This type of description is done in multiple places in the Results section and is confusing. It would be clearer if the authors show proper quantifications (mean +/- sem) of the percentages of (the relevant) gated populations. Besides, I don't think it make a lot of sense to mention in the text the percentages with 2 decimals behind the comma. This suggests a level of precision that does not seem justified in flow cytometry data. Finally, all flow cytometry plots look visually very busy and all the text is crammed in with really small fonts. Cleaning them up and enlarging the fonts of the remaining text/numbers would really improve the readability of the figures. 

      Thank you for your helpful comments. We understand your concern regarding the flow cytometry quantification. Indeed, the percentages presented in the figures are derived from representative replicates, and we acknowledge that this presentation could be confusing. To address this, we have included a table summarizing the data from all replicates to improve readability [Table S2 and S3 in the new version]. Second, we specified in the text that the data are representative biological replicates when needed. Third, we have performed statistical analyses on the three replicates when necessary, as shown in Supplementary Figure S5C-F in the new version. The text has been revised to reflect the correct statistical interpretation.

      Regarding the use of two decimal, we are unable to remove them due to limitations in the software (Kaluza) used for flow cytometry analysis. However, we agree that this level of precision may not be warranted, and we have revised the text where appropriate to reduce confusion.

      - In Figure 5G the authors show that FOLFIRI + AZD2858 are synergistic in two SN-38-resistant cell lines. They conclude that this combination may overcome drug resistance. But tried to figure out the used FOLFIRI concentrations used in these cell lines and they still seem far higher than the SN-38-sensitive HCT116 cell lines, so I would like to see a bit more nuance in their interpretation. I think overcoming drug resistance is an overstatement, and perhaps alleviating would be a better term 

      Thank you for highlighting this important point; we have adjusted the text accordingly.

      - The legend in Table S2 refers to Figure 5A-B; this should be Figure 4A-B. 

      Thank you, this has been corrected and Table S2 is now moved to Table S4 .

      Reviewer #1 (Significance (Required)): 

      The finding that AZD2858 block TOPbp1 condensate formation via a pleiotropic effect of this compound is interesting and convincing. To my best knowledge it's a novel finding which is interesting to the potential target audience mentioned below. Their findings that inhibition of TOPbp1 condensation and ATR signaling via AZD2858 may synergize with FOLFIRI therapy in colorectal cancer cells are still very preliminary, because the effects on non-cancerous cells are not tested. 

      Researchers involved in early cancer drug discovery and cell biologists studying DNA damage responses in cancer cells seem to me typical audience interested and influenced by this paper. 

      I'm a cell biologist studying cell cycle fate decisions, and adaptation of cancer cells & stem cells to (drug-induced) stress. My expertise aligns well with the work presented throughout this paper. 

      Reviewer #2 (Evidence, reproducibility and clarity (Required)): 

      The authors have extended their previous research to develop TOPBP1 as a potential drug target for colorectal cancer by inhibiting its condensation. Utilizing an optogenetic approach, they identified the small molecule AZD2858, which inhibits TOPBP1 condensation and works synergistically with first-line chemotherapy to suppress colorectal cancer cell growth. The authors investigated the mechanism and discovered that disrupting TOPBP1 assembly inhibits the ATR/Chk1 signaling pathway, leading to increased DNA damage and apoptosis, even in drug-resistant colorectal cancer cell lines. Addressing the following concerns would enhance clarity and further in vivo work may improve significance: 

      (1) How does the optogenetic method for inducing condensates compare to the DNA damage induction mechanism? 

      Optogenetics provides a versatile and precise approach for controlling the condensation of scaffold proteins in both space and time. This method enables us to study the role of biomolecular condensates with minute-scale resolution, separating their formation from potentially confounding upstream events, such as DNA damage, and providing valuable insights into their specific function. Importantly, based on our previous publications on TopBP1 or SLX4 optogenetic condensates, we have substantial evidence indicating that light-induced condensates closely mimic those formed in response to DNA damage:

      - Functional similarity: Optogenetic condensates recapitulate endogenous condensates formed upon exposure of the cells of DNA damaging agents, and include most known partner proteins involved in the DNA damage response. It was shown for light induced-TopBP1 and SLX4 condensates (1-3).

      - Dynamic reversibility: Optogenetic condensates and DNA damage induced condensates are both dynamic and reversible. They dissolve within 15 minutes of light deactivation or after removal of the damaging agent (1,3).

      - Chromatin association: Both optogenetic and DNA damage-induced condensates are bound to chromatin or localized at sites of DNA damage (3).

      - Regulation: Both types of condensates are regulated similarly, with their formation triggered by the same signaling pathways. ATR basal activity drives the nucleation of opto-TopBP1 condensates and endogenous TopBP1 structures upon light exposure (1). Likewise, sumoylation modifications regulate the formation of opto-SLX4 condensates and endogenous SLX4 condensates (3).

      - Structurally: Using super-resolution imaging by stimulation-emission-depletion (STED) microscopy, we observed that endogenous SLX4 nanocondensates formed globular clusters that were indistinguishable from recombinant light induced SLX4 condensates (1,3).  

      (1) Frattini C, Promonet A, Alghoul E, Vidal-Eychenie S, Lamarque M, Blanchard MP, et al. TopBP1 assembles nuclear condensates to switch on ATR signaling. Molecular Cell. 18 mars 2021;81(6):1231-1245.e8. 

      (2) Alghoul E, Basbous J, Constantinou A. An optogenetic proximity labeling approach to probe the composition of inducible biomolecular condensates in cultured cells. STAR Protocols. 2021;2(3):100677. 

      (3) Alghoul E, Basbous J, Constantinou A. Compartmentalization of the DNA damage response: Mechanisms and functions. DNA Repair. août 2023;128:103524.

      (2) Why wasn't the initial screen conducted on the HCT116-SN50 resistant cell line? 

      Thank you for raising this important question, which we also considered at the outset of the project. After careful consideration, we decided to use the HCT116 WT cells in order to obtain initial data from an unmodified cell line. It is worth mentioning that HCT116-SN50 cells exhibit slower proliferation compared to WT cells, and they also express an efflux pump capable of pumping out SN38. We were concerned that these factors might interfere with the optogenetic assay, which is why we chose to perform the screen using the WT HCT116 cells.

      (3) The labels in Fig. 1D are difficult to recognize. 

      This issue was also raised by Reviewer #1. We suspect that the PDF conversion may have reduced the resolution of the figures, so we will provide them separately in high resolution. In addition, we have increased the size of some labels to improve their clarity.

      The selected cell image in Fig. 2A for SN-38 seems over-representative; unselected cells appear similar to other groups. Why does AZD2858 itself induce TopBP1 condensates in the plot, yet this is not evident in the images? 

      Thank you for your comment; we have updated the figure with a more representative image. We indeed observe that AZD2858 alone induces a slight increase in TopBP1 condensates. However, this increase did not lead to the activation of the ATR/Chk1 signaling pathway, as shown by the Western blot data presented in Fig. 2B. In addition, AZD2858 specifically prevents the formation of TopBP1 condensates induced by SN38 treatment, and the level of TopBP1 condensates does not return to the basal levels observed in untreated cells, but rather to those observed with AZD2858 treatment. During the 2-hour AZD2858 treatment, the progression of replication forks was unaffected (Fig. 3A and 3B). However, when AZD2858 was added alone to the Xenopus egg extracts, there was increased recruitment of TopBP1 to the chromatin (Fig. 2E). This result suggests that AZD2858 alone can induce the assembly of TopBP1 on chromatin to initiate DNA replication (a well-established role of TopBP1), but the number and concentration of TopBP1 molecules did not reach levels sufficient to activate the ATR/Chk1 pathway.

      (4) In Fig. 3A, despite the drastic change in the FACS plot shape, the quantifications appear quite similar. 

      Thank you for this insightful observation. The gates for the S phase were intentionally set wider to avoid biasing the results and inadvertently excluding the population that incorporates BrdU weakly (but still incorporates it) in the SN-38 only condition. As a result, the percentage of cells within this gate remains similar, even though the overall shape of the FACS plot changes, reflecting a shift in the distribution of BrdU incorporation. This point has now been clarified in the legend of the Figure 3A.

      This effect can also be attributed to the relatively short treatment time (2 hours), which captures early changes in DNA synthesis. The effect becomes more pronounced at later time points, as shown in Figure 3C. For example, after 6 hours of treatment, the percentage of BrdU-positive cells increases from 15% with SN-38 alone to 41% with the AZD2858 combination, demonstrating a clearer impact on DNA synthesis. A graph summarizing the statistical analysis has been added to Figure S5C for the 6-hour time point and Figure S5D for the 12-hour time point, based on data from three independent biological replicates.

      (5) The results section is imbalanced; Figs. 5 and 6 could be combined into one figure. 

      We have combined Figures 5 and 6 into a single figure to optimize the presentation of results. To avoid overloading the new figure, some of the data have been moved to supplementary figures, ensuring the main figure remains clear and focused.

      (6) An in vivo study is anticipated to assess the drug's efficacy. 

      Although AZD2858 was developed a few years ago, there is a limited amount of in vivo data available, which led us to consider potential issues related to the drug's biodistribution or its pharmacokinetics (PK). Despite these concerns, we proceeded with preliminary in vivo studies, testing various diluents and injection routes for AZD2858. However, we observed that the compound was not effective in vivo. Given the strong synergistic effects observed in vitro, we concluded that AZD2858 was likely not being distributed properly in the mice. As a result, we have decided to conduct a more detailed investigation into the pharmacokinetics (PK), pharmacodynamics (PD), and absorption, distribution, metabolism, and excretion (ADME) of AZD2858 to better understand its in vivo behavior and efficacy. Therefore, the in vivo evaluation of AZD2858 will be addressed in a separate study specifically focused on this aspect.

      Reviewer #2 (Significance (Required)): 

      Addressing the stated concerns would enhance clarity and further in vivo work may improve significance. 

      Reviewer #3 (Evidence, reproducibility and clarity (Required)): 

      Summary 

      In 2021 (PMID: 33503405) and 2024 (PMID: 38578830) Constantinou and colleagues published two elegant papers in which they demonstrated that the Topbp1 checkpoint adaptor protein could assemble into mesoscale phase-separated condensates that were essential to amplify activation of the PIKK, ATR, and its downstream effector kinase, Chk1, during DNA damage signalling. A key tool that made these studies possible was the use of a chimeric Topbp1 protein bearing a cryptochrome domain, Cry2, which triggered condensation of the chimeric Topbp1 protein, and thus activation of ATR and Chk1, in response to irradiation with blue light without the myriad complications associated with actually exposing cells to DNA damage. 

      In this current report Morano and co-workers utilise the same optogenetic Topbp1 system to investigate a different question, namely whether Topbp1 phase-condensation can be inhibited pharmacologically to manipulate downstream ATR-Chk1 signalling. This is of interest, as the therapeutic potential of the ATR-Chk1 pathway is an area of active investigation, albeit generally using more conventional kinase inhibitor approaches. 

      The starting point is a high throughput screen of 4730 existing or candidate small molecule anticancer drugs for compounds capable of inhibiting the condensation of the Topbp1-Cry2mCherry reporter molecule in vivo. A surprisingly large number of putative hits (>300) were recorded, from which 131 of the most potent were selected for secondary screening using activation of Chk1 in response to DNA damage induced by SN-38, a topoisomerase inhibitor, as a surrogate marker for Topbp1 condensation. From this the 10 most potent compounds were tested for interactions with a clinically used combination of SN-38 and 5-FU (FOLFIRI) in terms of cytotoxicity in HCT116 cells. The compound that synergised most potently with FOLFIRI, the GSK3-beta inhibitor drug AZD2858, was selected for all subsequent experiments. 

      AZD2858 is shown to suppress the formation of Topbp1 (endogenous) condensates in cells exposed to SN-38, and to inhibit activation of Chk1 without interfering with activation of ATM or other endpoints of damage signalling such as formation of gamma-H2AX or activation of Chk2 (generally considered to be downstream of ATM). AZD2858 therefore seems to selectively inhibit the Topbp1-ATR-Chk1 pathway without interfering with parallel branches of the DNA damage signalling system, consistent with Topbp1 condensation being the primary target. Importantly, neither siRNA depletion of GSK3-beta, or other GSK3-beta inhibitors were able to recapitulate this effect, suggesting it was a specific non-canonical effect of AZD2858 and not a consequence of GSK3-beta inhibition per se. 

      To understand the basis for synergism between AZD2858 and SN-38 in terms of cell killing, the effect of AZD2858 on the replication checkpoint was assessed. This is a response, mediated via ATR-Chk1, that modulates replication origin firing and fork progression in S-phase cell under conditions of DNA damage or when replication is impeded. SN-38 treatment of HCT116 cells markedly suppresses DNA replication, however this was partially reversed by co-treatment with AZD2858, consistent with the failure to activate ATR-Chk1 conferring a defect in replication checkpoint function. 

      Figures 4 and 5 demonstrate that AZD2858 can markedly enhance the cytotoxic and cytostatic effects of SN-38 and FOLFIRI through a combination of increased apoptosis and growth arrest according to dosage and treatment conditions. Figure 6 extends this analysis to cells cultured as spheroids, sometimes considered to better represent tumor responses compared to single cell cultures. 

      Major comments 

      Most of the data presented is of good technical quality and supports the conclusions drawn. There are however a small number of instances where this is not true; ie where the data are of insufficient technical quality, or where the description or interpretation of the results is at variance with the data which is presented. Some examples: 

      (1) Fig.2E - the claim that "we observed an increase in RPA, Topb1 and Pol-epsilon levels when CPT and AZD2858 were added together" do not seem to be justified by the data provided. It is also unclear what the purpose/ significance of this experiment is. 

      Thank you for pointing out the contradiction in Figure 2E. Upon review, we identified an error in the labeling of conditions (CPT and AZD2858 were inadvertently swapped). The corrected figure now clearly shows that, at the 60-minute timepoint after starting replication, the combination of

      CPT and AZD2858 results in a greater accumulation of TopBP1, Pol ε, and RPA on chromatin compared to CPT alone. We have revised the sentence to: "Our data demonstrate that combining CPT and AZD2858 earlier enhances the accumulation of replication-related factors (RPA, TopBP1, and Pol ε) on chromatin compared to CPT treatment alone, particularly visible at the 60minute after starting replication."

      The significance of this experiment lies in its connection to the earlier observation that AZD2858 restores BrdU incorporation when combined with SN-38, as shown in flow cytometry data (Figure 3A). At a molecular level, this was further supported by DNA fiber assays, which revealed that replication tracks (CldU tracts) were longer in the combination treatment compared to SN-38 alone (Figure 3B).

      To strengthen and validate these findings, we chose to employ the Xenopus egg extract system for several reasons. This model provides a highly controlled environment where DNA replication occurs without confounding effects from transcription or translation. Moreover, replication is limited to a single round, offering a unique opportunity to specifically interrogate replication mechanisms. These attributes make the Xenopus model an ideal system to confirm that AZD2858 facilitates replication recovery in the presence of replication stress induced by agents like CPT. This will lead, in longer treatment, to accumulation of DNA damage and apoptosis (Figure 3D-E and Figure 4A-D)

      (2) Figs. 3 A and C certainly show that the SN-38-mediated suppression of DNA synthesis is modified and partially alleviated by co-treatment with AZD2858. The statement however that "prolonged co-incubation with AZD2858 for 6 and 12 hours effectively abolished the SN-38 induced S-phase checkpoint" is clearly misleading. If this were true, then the BrdU incorporation profiles of the respective samples would be similar or identical to control, which clearly they are not. Clearly AZD2858 is affecting the imposition of the S-phase checkpoint in some way, but not "abolishing" it. 

      We appreciate the reviewer’s detailed observations regarding Figures 3A and 3C and the phrasing in our manuscript. We agree that the term "abolished" is not precise in describing the effects of AZD2858 on the SN-38-induced S-phase checkpoint.

      To clarify: our data indicate that co-treatment with AZD2858 modifies and partially alleviates the SN-38-induced suppression of DNA synthesis, as demonstrated by increased BrdU incorporation relative to SN-38 treatment alone. However, as the reviewer correctly points out, the BrdU incorporation profiles of the co-treated samples do not fully return to control non treated cells levels. This suggests that while AZD2858 significantly mitigates the S-phase checkpoint, it does not completely abolish it.

      We have revised the statement in the manuscript to better reflect these findings, as follows: "Prolonged co-incubation with AZD2858 for 6 and 12 hours significantly alleviated the SN-38induced S-phase checkpoint, as evidenced by the partially increased BrdU incorporation. However, the population of co-treated cells is heterogeneous: some cells exhibit BrdU incorporation levels similar to those of untreated control cells, while others incorporate BrdU at levels comparable to cells treated with SN-38 alone. This indicates that AZD2858 does not fully restore DNA synthesis to control levels across the entire cell population."

      This revised phrasing aligns with the data presented and acknowledges the partial recovery of DNA synthesis observed. Thank you for bringing this to our attention and helping us improve the accuracy of our conclusions.

      (3) Fig. 3 E. The western blots of pDNA-PKcs (S2056) and total DNA-PKcs are really not interpretable. It is possible to sympathise that these reagents are probably extremely difficult to work with and obtain clear results, however uninterpretable results are not acceptable. 

      We agree that the data presented in the Fig3E are difficult to interpret. As noted by Reviewer 1, we recognize the challenge of obtaining clear and reliable results with these specific reagents. Based on this feedback, and to ensure the robustness of our conclusions, we have decided to exclude these specifics blots from the revised manuscript.

      We believe that this adjustment will enhance the clarity and reliability of the manuscript while focusing on the other, more interpretable data presented. Thank you for pointing this out, and we appreciate your understanding.

      (4) Fig. 3D. This is a puzzling image. Described as a PFGE assay, it presumably depicts an agarose gel, with intact genomic DNA at the top and a discrete band below representing fragmented genomic DNA. This is a little surprising, as fragmented genomic DNA does not usually appear as a specific band but as a heterogenous population or "smear". Nevertheless, even if one accepts this premise, it is unclear what is meant by "DSBs remained elevated after the combined treatment" when the intensity of this band is equivalent for both SN-38 and SN-38 + AZD2858 treatments. 

      We thank the reviewer for his insightful comments regarding the PFGE results in Figure 3D. We agree that the appearance of a discrete band, rather than a heterogeneous smear, is atypical for fragmented genomic DNA in this assay. However, by enhancing the signal intensity (as shown below), the expected smear becomes more appreciable.

      Author response image 4.

      Regarding the interpretation of the band intensities, we agree that the signals for SN-38 and SN38 + AZD2858 appear similar under these specific conditions. At the relatively high concentration of SN-38 used in this experiment (300 nM), it is indeed challenging to observe a more pronounced effect on DNA breaks. This is why we proposed the "DSBs remained elevated after the combined treatment" because the band intensity of SN-38 single agent treated cells or combined with AZD2858 is comparable. However, we note a slightly more intense γH2AX signal over time when AZD2858 is combined with SN-38 compared to SN-38 alone (Figure 3E). Furthermore, under lower, sub-optimal doses of SN-38 and over extended incubation treatment (48h), we observe a clearer increase in fragmented DNA bands, as demonstrated in Figure 4D.

      Minor comments 

      (1) Fig. 1. A surprisingly large number of compounds scored positive in the primary screen for inhibition of Topbp1 condensation (>300). Of the 131 of these selected for secondary screening using Chk1 activation (S345 phosphorylation) as a readout approximately 2/3 were negative, implying that a majority of the tested compounds inhibited Topbp1 condensation but not Chk1 activation. What could explain that?

      Thank you for this thoughtful comment. The discrepancy between the large number of compounds scoring positive for TopBP1 condensation inhibition and the smaller number inhibiting Chk1 activation (S345 phosphorylation) could be attributed to several factors:

      • Different cell lines and induction methods: The initial screen was conducted in HEK293 TrexFlpin cells overexpressing optoTopBP1, while the secondary screen used HCT116 cells. In addition, the methods used to induce the respective pathways were distinct: in the primary screen, we employed a blue light induction of opto-TopBP1 condensates, whereas in the secondary screen, we used an SN-38 treatment to induce DNA replication stress and activate the Chk1 pathway. These differences could account for the varying responses observed in the two screens.

      • The compounds that inhibited TopBP1 condensation might not fully block Chk1 activation. While they disrupt TopBP1 condensation, they may still allow for partial activation of Chk1 or Chk1 activation through alternative mechanisms. For instance, Chk1 activation could be mediated by other signaling pathways or molecules, such as ETAA1, a known Chk1 activator (1). Thus, TopBP1 condensation inhibition does not necessarily translate to complete inhibition of Chk1 activation, especially if ETAA1 is employed by cells as a rescue activator.

      • Some compounds may affect chromosome dynamics, potentially generating mechanical forces or torsional stress that could activate the ATR/Chk1 pathway independently of TopBP1

      (2).

      These factors suggest that while the compounds effectively disrupt TopBP1 condensation, they may not always fully inhibit the downstream Chk1 activation, pointing to the complexity of the DNA damage response pathways. 

      (1) Bass, T. E. et al. ETAA1 acts at stalled replication forks to maintain genome integrity. Nat Cell Biol 18, 1185–1195 (2016).

      (2) Kumar, A. et al. ATR Mediates a Checkpoint at the Nuclear Envelope in Response to Mechanical Stress. Cell 158, 633–646 (2014).

      (2) Fig. 2D. The protein-protein interaction assay shown demonstrates that AZD2858 ablates the light-induced auto-interaction between exogenous opto-Topbp1 molecules and ATR plus or minus SN-38, but clearly endogenous Topbp1 molecules do not participate. Why is this? 

      The biotin proximity labeling assay was conducted without exposing cells to light, using a TurboID module fused to TopBP1-mCherry-CRY2. Stable cell lines were then generated in HEK293 TrexFlpIn cells, where endogenous TopBP1 is still expressed. Upon adding doxycycline, the recombinant TurboID-TopBP1-mCherry-Cry2 (opto-TopBP1) is induced at levels comparable to endogenous TopBP1 (Fig 2D).

      Since the opto-TopBP1 construct exhibits behavior similar to that of endogenous TopBP1 (1), we used it to investigate whether TopBP1 self-assembly and its interaction with ATR are influenced by AZD2858 alone or in combination with SN38. Our results show that treatment with SN38 increases the proximity between opto-TopBP1 and the endogenous TopBP1 (not fused to TurboID). However, AZD2858, either alone or in combination with SN38, disrupts the selfassembly of recombinant TopBP1 with itself as well as its interaction with endogenous TopBP1.

      (1) Frattini C, Promonet A, Alghoul E, Vidal-Eychenie S, Lamarque M, Blanchard MP, et al. TopBP1 assembles nuclear condensates to switch on ATR signaling. Molecular Cell. 18 mars 2021;81(6):1231-1245.e8.

      Reviewer #3 (Significance (Required)): 

      Significance 

      Liquid phase separation of protein complexes is increasingly recognised as a fundamental mechanism in signal transduction and other cellular processes. One recent and important example was that of Topbp1, whose condensation in response to DNA damage is required for efficient activation of the ATR-Chk1 pathway. The current study asks a related but distinct question; can protein condensation be targeted by drugs to manipulate signalling pathways which in the main rely on protein kinase cascades? 

      Here, the authors identify an inhibitor of GSK3-beta as a novel inhibitor of DNA damage-induced Topbp1 condensation and thus of ATR-Chk1 signalling. 

      This work will be of interest to researchers in the fields of DNA damage signalling, biophysics of protein condensation, and cancer chemotherapy.

    1. Author response:

      Public Reviews:

      Reviewer #1 (Public review):

      In this paper by Brickwedde et al., the authors observe an increase in posterior alpha when anticipating auditory as opposed to visual targets. The authors also observe an enhancement in both visual and auditory steady-state sensory evoked potentials in anticipation of auditory targets, in correlation with enhanced occipital alpha. The authors conclude that alpha does not reflect inhibition of early sensory processing, but rather orchestrates signal transmission to later stages of the sensory processing stream. However, there are several major concerns that need to be addressed in order to draw this conclusion.

      First, I am not convinced that the frequency tagging method and the associated analyses are adequate for dissociating visual vs auditory steady-state sensory evoked potentials.

      Second, if the authors want to propose a general revision for the function of alpha, it would be important to show that alpha effects in the visual cortex for visual perception are analogous to alpha effects in the auditory cortex for auditory perception.

      Third, the authors propose an alternative function for alpha - that alpha orchestrates signal transmission to later stages of the sensory processing stream. However, the supporting evidence for this alternative function is lacking. I will elaborate on these major concerns below.

      (1) Potential bleed-over across frequencies in the spectral domain is a major concern for all of the results in this paper. The fact that alpha power, 36Hz and 40Hz frequency-tagged amplitude and 4Hz intermodulation frequency power is generally correlated with one another amplifies this concern. The authors are attaching specific meaning to each of these frequencies, but perhaps there is simply a broadband increase in neural activity when anticipating an auditory target compared to a visual target?

      We appreciate the reviewer’s insightful comment regarding the potential bleed-over across frequencies in the spectral domain. We fully acknowledge that the trade-off between temporal and frequency resolution is a challenge, particularly given the proximity of the frequencies we are examining.

      To address this concern, we performed additional analyses to investigate whether there is indeed a broadband increase in neural activity when anticipating an auditory target as compared to a visual target, as opposed to distinct frequency-specific effects. Our results show that the bleed-over between frequencies is minimal and does not significantly affect our findings. Specifically, we repeated the analyses using the same filter and processing steps for the 44 Hz frequency. At this frequency, we did not observe any significant differences between conditions.

      These findings suggest that the effects we report are indeed specific to the 40 Hz frequency band and not due to a general broadband increase in neural activity. We hope this addresses the reviewer’s concern and strengthens the validity of our frequency-specific results.

      Author response image 1.

      Illustration of bleeding over effects over a span of 4 Hz. A, 40 Hz frequency-tagging data over the significant cluster differing between when expecting an auditory versus a visual target (identical to Fig. 9 in the manuscript). B, 44 Hz signal over the same cluster chosen for A. The analysis was identical with the analysis performed in  A, apart from the frequency for the band-pass filter.

      We do, however, not specifically argue against the possibility of a broadband increase when anticipating an auditory compared to a visual target. But even a broadband-increase would directly contradict the alpha inhibition hypothesis, which poses that an increase in alpha completely disengages the whole cortex. We will clarify this point in the revised manuscript.

      (2) Moreover, 36Hz visual and 40Hz auditory signals are expected to be filtered in the neocortex. Applying standard filters and Hilbert transform to estimate sensory evoked potentials appears to rely on huge assumptions that are not fully substantiated in this paper. In Figure 4, 36Hz "visual" and 40Hz "auditory" signals seem largely indistinguishable from one another, suggesting that the analysis failed to fully demix these signals.

      We appreciate the reviewer’s insightful concern regarding the filtering and demixing of the 36 Hz visual and 40 Hz auditory signals, and we share the same reservations about the reliance on standard filters and the Hilbert transform method.

      To address this, we would like to draw attention to Author response image 1, which demonstrates that a 4 Hz difference is sufficient to effectively demix the signals using our chosen filtering and Hilbert transform approach. We believe that the reason the 36 Hz visual and 40 Hz auditory signals show similar topographies lies not in incomplete demixing but rather in the possibility that this condition difference reflects sensory integration, rather than signal contamination.

      This interpretation is further supported by our findings with the intermodulation frequency at 4 Hz, which also suggests cross-modal integration. Furthermore, source localization analysis revealed that the strongest condition differences were observed in the precuneus, an area frequently associated with sensory integration processes. We will expand on this in the discussion section to better clarify this point.

      (3) The asymmetric results in the visual and auditory modalities preclude a modality-general conclusion about the function of alpha. However, much of the language seems to generalize across sensory modalities (e.g., use of the term 'sensory' rather than 'visual').

      We thank the reviewer for pointing this out and agree that in some cases we have not made a good enough distinction between visual and sensory. We will make sure, that when using ‘sensory’, we either describe overall theories, which are not visual-exclusive or refer to the possibility of a broad sensory increase. However, when directly discussing our results and the interpretation thereof, we will now use ‘visual’ in the revised manuscript.

      (4) In this vein, some of the conclusions would be far more convincing if there was at least a trend towards symmetry in source-localized analyses of MEG signals. For example, how does alpha power in the primary auditory cortex (A1) compare when anticipating auditory vs visual target? What do the frequency-tagged visual and auditory responses look like when just looking at the primary visual cortex (V1) or A1?

      We thank the reviewer for this important suggestion and have added a virtual channel analysis. We were however, not interested in alpha power in primary auditory cortex, as we were specifically interested in the posterior alpha, which is usually increased when expecting an auditory compared to a visual target (and used to be interpreted as a blanket inhibition of the visual cortex). We will improve upon the clarity concerning this point in the manuscript.

      We have however, followed the reviewer’s suggestion of a virtual channel analysis, showing that the condition differences are not observable in primary visual cortex for the 36 Hz visual signal and in primary auditory cortex for the 40 Hz auditory signal. Our data clearly shows that there is an alpha condition difference in V1, while there no condition difference for 36 Hz in V1 and for 40 Hz in Heschl’s Gyrus (see Author response image 2).

      Author response image 2.

      Virtual channels for V1 and Helschl’s gyrus. A, alpha power for the virtual channel created in V1 (Calcerine_L and Calcerine_R from AAL atlas; Tzourio-Mazoyer et al., 2002, NeuroImage). A cluster permutation analysis over time (between -2 and 0) revealed a significant condition difference between ~ -2 and -1.7 s (p = 0.0449). B, 36 Hz frequency-tagging signal for the virtual channel created in V1 (equivalent to the procedure in A). The same cluster permutation as performed in A revealed no significant condition differences. C, 40 Hz frequency-tagging signal for the virtual channel created in Heschl’s gryrus (Heschl_L and Heschl_R from AAL atlas; Tzourio-Mazoyer et al., 2002, NeuroImage). The same cluster permutation as performed in A revealed no significant condition differences.

      (5) Blinking would have a huge impact on the subject's ability to ignore the visual distractor. The best thing to do would be to exclude from analysis all trials where the subjects blinked during the cue-to-target interval. The authors mention that in the MEG experiment, "To remove blinks, trials with very large eye-movements (> 10 degrees of visual angle) were removed from the data (See supplement Fig. 5)." This sentence needs to be clarified since eye-movements cannot be measured during blinking. In addition, it seems possible to remove putative blink trials from EEG experiments as well, since blinks can be detected in the EEG signals.

      We thank the reviewer for mentioning that we were making this point confusing. From the MEG-data, we removed eyeblinks using ICA. Alone for the supplementary Fig. 5 analysis, we used the eye-tracking data to confirm that participants were in fact fixating the centre of the screen. For this analysis, we removed trials with blinks (which can be seen in the eye-tracker as huge amplitude movements or as large eye-movements in degrees of visual angle; see Author response image 3 below to show a blink in the MEG data and the according eye-tracker data in degrees of visual angle). We will clarify this in the methods section.

      As for the concern closed eyes to ignore visual distractors, in both experiments we can observe highly significant distractor cost in accuracy for visual distractors, which we hope will convince the reviewer that our visual distractors were working as intended.

      Author response image 3.

      Illustration of eye-tracker data for a trial without and a trial with a blink. All data points recorded during this trial are plottet. A, ICA component 1, which reflects blinks and its according data trace in a trial. No blink is visible. B, eye-tracker data transformed into degrees of visual angle for the trial depicted in A. C, ICA component 1, which reflects blinks and its according data trace in a trial. A clear blink is visible. D, eye-tracker data transformed into degrees of visual angle for the trial depicted in C.

      (6) It would be interesting to examine the neutral cue trials in this task. For example, comparing auditory vs visual vs neutral cue conditions would be indicative of whether alpha was actively recruited or actively suppressed. In addition, comparing spectral activity during cue-to-target period on neutral-cue auditory correct vs incorrect trials should mimic the comparison of auditory-cue vs visual-cue trials. Likewise, neutral-cue visual correct vs incorrect trials should mimic the attention-related differences in visual-cue vs auditory-cue trials.

      We thank the reviewer for this suggestion. We have analysed the neutral cue trials in the EEG dataset (see suppl. Fig. 1) and will expand this figure to show all conditions. There were no significant differences to auditory or visual cues, but descriptively alpha power was higher for neutral cues compared to visual cues and lower for neutral cues compared to auditory cues. While this may suggest that for visual trials alpha is actively suppressed and for auditory trials actively recruited, we do not feel comfortable to make this claim, as the neutral condition may not reflect a completely neutral state. The neutral task can still be difficult, especially because of the uncertainty of the target modality.

      As for the analysis of incorrect versus correct trials, we love the idea, but unfortunately the accuracy rate was quite high so that the number of incorrect trials would not be sufficient to perform a reliable analysis.

      (7) In the abstract, the authors state that "This implies that alpha modulation does not solely regulate 'gain control' in early sensory areas but rather orchestrates signal transmission to later stages of the processing stream." However, I don't see any supporting evidence for the latter claim, that alpha orchestrates signal transmission to later stages of the processing stream. If the authors are claiming an alternative function to alpha, this claim should be strongly substantiated.

      We thank the reviewer for pointing out, that we have not sufficiently explained our case. The first point refers to gain control akin to the alpha inhibition hypothesis, which claims that increases in alpha disengage a whole cortical area. Since we have confirmed the alpha increase in our data to originate from primary visual cortex through source analysis, this should lead to decreased visual processing. The increase in 36 Hz visual processing therefore directly contradicts the alpha inhibition hypothesis. We propose an alternative explanation for the functionality of alpha activity in this task. Through pulsed inhibition, information packages of relevant visual information could be transmitted down the processing stream, thereby enhancing relevant visual signal transmission. We believe the fact that the enhanced visual 36 Hz signal we found correlated with visual alpha power on a trial-by-trial basis, and did not originate from primary visual cortex, but from areas known for sensory integration supports our claim.

      We will make this point clearer in our revised manuscript.

      Reviewer #2 (Public review):

      Brickwedde et al. investigate the role of alpha oscillations in allocating intermodal attention. A first EEG study is followed up with a MEG study that largely replicates the pattern of results (with small to be expected differences). They conclude that a brief increase in the amplitude of auditory and visual stimulus-driven continuous (steady-state) brain responses prior to the presentation of an auditory - but not visual - target speaks to the modulating role of alpha that leads them to revise a prevalent model of gating-by-inhibition.

      Overall, this is an interesting study on a timely question, conducted with methods and analysis that are state-of-the-art. I am particularly impressed by the author's decision to replicate the earlier EEG experiment in MEG following the reviewer's comments on the original submission. Evidently, great care was taken to accommodate the reviewer's suggestions.

      We thank the reviewer for the positive feedback and expression of interest in the topic of our manuscript.

      Nevertheless, I am struggling with the report for two main reasons: It is difficult to follow the rationale of the study, due to structural issues with the narrative and missing information or justifications for design and analysis decisions, and I am not convinced that the evidence is strong, or even relevant enough for revising the mentioned alpha inhibition theory. Both points are detailed further below.

      We thank the reviewer for raising this important point. We will revise our introduction and results in line with the reviewer’s suggestions, hoping that our rationale will then be easier to follow and that our evidence will be more convincing.

      Strength/relevance of evidence for model revision: The main argument rests on 1) a rather sustained alpha effect following the modality cue, 2) a rather transient effect on steady-state responses just before the expected presentation of a stimulus, and 3) a correlation between those two. Wouldn't the authors expect a sustained effect on sensory processing, as measured by steady-state amplitude irrespective of which of the scenarios described in Figure 1A (original vs revised alpha inhibition theory) applies? Also, doesn't this speak to the role of expectation effects due to consistent stimulus timing? An alternative explanation for the results may look like this: Modality-general increased steady-state responses prior to the expected audio stimulus onset are due to increased attention/vigilance. This effect may be exclusive (or more pronounced) in the attend-audio condition due to higher precision in temporal processing in the auditory sense or, vice versa, too smeared in time due to the inferior temporal resolution of visual processing for the attend-vision condition to be picked up consistently. As expectation effects will build up over the course of the experiment, i.e., while the participant is learning about the consistent stimulus timing, the correlation with alpha power may then be explained by a similar but potentially unrelated increase in alpha power over time.

      We thank the reviewer for raising these insightful questions and suggestions.

      It is true that our argument rests on a rather sustained alpha effect and a rather transient effect on steady-state responses and a correlation between the two. However, this connection would not be expected under the alpha inhibition hypothesis, which states that alpha activity would inhibit a whole cortical area (when irrelevant to the task), exerting “gain control”. This notion directly contradicts our results of the “irrelevant” visual information a) being transmitted at all and b) increasing.

      However, it has been shown on many occasions that alpha activity exerts pulsed inhibition, so we proposed an alternative theory of an involvement in signal transmission. In this case, the cyclic inhibition would serve as an ordering system, which only allows for high-priority information to pass, resulting in higher signa-to-noise. We do not make a claim about how fast or when these signals are transmitted in relation to alpha power. For instance, it could be that alpha power increases as a preparatory state even before signal is actually transmitted.  Zhigalov (2020 Hum. Brain M.) has shown that in V1, frequency-tagging responses were up-and down regulated with attention – independent of alpha activity.

      But we do believe that the fact that visual alpha power correlates on a trial-by-trial level with visual 36 Hz frequency-tagging increases and (a relationship which has not been found in V1, see Zhigalov 2020, Hum. Brain Mapp.) suggest a strong connection. Furthermore, the fact that the alpha modulation originates from early visual areas and occurs prior to any frequency-tagging changes, while the increase in frequency-tagging can be observed in areas which are later in the processing stream (such as the precuneus) is strongly indicative for an involvement of alpha power in the transmission of this signal. We cannot fully exclude alternative accounts and mechanisms which effect both alpha power and frequency-tagging responses. 

      We do believe that the alternative account described by the reviewer does not contradict our theory, as we do believe that the alpha power modulation may reflect an expectation effect (and the idea that it could be related to the resolution of auditory versus visual processing is very interesting!). It is also possible that this expectation is, as the reviewer suggests, related to attention/vigilance and might result in a modality-general signal increase. And indeed, we can observe an increase in the frequency-tagging response in sensory integration areas. Accordingly, we believe that the alternative explanation provided by the reviewer contradicts the alpha inhibition hypothesis, but not necessarily our alternative theory.

      We will revise the discussion, which we hope will make our case stronger and easier to follow. Additionally, we will mention the possibility for alternative explanations as well as the possibility, that alpha networks fulfil different roles in different locations/task environments.

      Structural issues with the narrative and missing information: Here, I am mostly concerned with how this makes the research difficult to access for the reader. I list the major points below:

      In the introduction the authors pit the original idea about alpha's role in gating against some recent contradictory results. If it's the aim of the study to provide evidence for either/or, predictions for the results from each perspective are missing. Also, it remains unclear how this relates to the distinction between original vs revised alpha inhibition theory (Fig. 1A). Relatedly if this revision is an outcome rather than a postulation for this study, it shouldn't be featured in the first figure.

      We agree with the reviewer that we have not sufficiently clarified our goal as well as how different functionalities of alpha oscillations would lead to different outcomes. We will revise the introduction and restructure the results and hope that it will be easier to follow.

      The analysis of the intermodulation frequency makes a surprise entrance at the end of the Results section without an introduction as to its relevance for the study. This is provided only in the discussion, but with reference to multisensory integration, whereas the main focus of the study is focussed attention on one sense. (Relatedly, the reference to "theta oscillations" in this sections seems unclear without a reference to the overlapping frequency range, and potentially more explanation.) Overall, if there's no immediate relevance to this analysis, I would suggest removing it.

      We thank the reviewer for pointing this out and will add information about this frequency to the introduction part. We believe that the intermodulation frequency analysis is important, as it potentially supports the notion that condition differences in the visual-frequency tagging response are related to downstream processing rather than overall visual information processing in V1. We would therefore prefer to leave this analysis in the manuscript.

      Reviewer #3 (Public review):

      Brickwedde et al. attempt to clarify the role of alpha in sensory gain modulation by exploring the relationship between attention-related changes in alpha and attention-related changes in sensory-evoked responses, which surprisingly few studies have examined given the prevalence of the alpha inhibition hypothesis. The authors use robust methods and provide novel evidence that alpha likely exhibits inhibitory control over later processing, as opposed to early sensory processing, by providing source-localization data in a cross-modal attention task.

      This paper seems very strong, particularly given that the follow-up MEG study both (a) clarifies the task design and separates the effect of distractor stimuli into other experimental blocks, and (b) provides source-localization data to more concretely address whether alpha inhibition is occurring at or after the level of sensory processing, and (c) replicates most of the EEG study's key findings.

      We are very grateful to the reviewer for their positive feedback and evaluation of our work.

      There are some points that would be helpful to address to bolster the paper. First, the introduction would benefit from a somewhat deeper review of the literature, not just reviewing when the effects of alpha seem to occur, but also addressing how the effect can change depending on task and stimulus design (see review by Morrow, Elias & Samaha (2023).

      We thank the reviewer for this suggestion and agree. We will add a paragraph to the introduction which refers to missing correlation studies and the impact of task design.

      Additionally, the discussion could benefit from more cautionary language around the revision of the alpha inhibition account. For example, it would be helpful to address some of the possible discrepancies between alpha and SSEP measures in terms of temporal specificity, SNR, etc. (see Peylo, Hilla, & Sauseng, 2021). The authors do a good job speculating as to why they found differing results from previous cross-modal attention studies, but I'm also curious whether the authors think that alpha inhibition/modulation of sensory signals would have been different had the distractors been within the same modality or whether the cues indicated target location, rather than just modality, as has been the case in so much prior work?

      We thank the reviewer for suggesting these interesting discussion points and will include a paragraph in our discussion which goes deeper into these topics.

      Overall, the analyses and discussion are quite comprehensive, and I believe this paper to be an excellent contribution to the alpha-inhibition literature.

    1. Author response:

      To Reviewer #1:

      Thank you for your thorough review and comments on our work, which you described as “the role of neuritin in T cell biology studied here is new and interesting.”.  We have summarized your comments into two categories: biology and investigation approach, experimental rigor, and data presentation.

      Biology and Investigation approach comments:

      (1) Questions regarding the T cell anergy model:

      Major point “(4) Figure 1E-H. The authors assume that this immunization protocol induces anergic cells, but they provide no experimental evidence for this. It would be useful to show that T cells are indeed anergic in this model, especially those that are OVA-specific. The lack of IL-2 production by Cltr cells could be explained by the presence of fewer OVA-specific cells, rather than by an anergic status.”

      T cell anergy is a well-established concept first described by Schwartz’s group. It refers to the hyporesponsive T cell functional state in antigen-experienced CD4 T cells (Chappert and Schwartz, 2010; Fathman and Lineberry, 2007; Jenkins and Schwartz, 1987; Quill and Schwartz, 1987).  Anergic T cells are characterized by their inability to expand and to produce IL2 upon subsequent antigen re-challenge. In this paper, we have borrowed the existing in vivo T cell anergy induction model used by Mueller’s group for T cell anergy induction (Vanasek et al., 2006).  Specifically, Thy1.1+ Ctrl or Nrn1-/- TCR transgenic OTII cells were co-transferred with the congenically marked Thy1.2+ WT polyclonal Treg cells into TCR-/- mice.  After anergy induction, the congenically marked TCR transgenic T cells were recovered by sorting based on Thy1.1+ congenic marker, and subsequently re-stimulation ex vivo with OVA323-339 peptide. We evaluated the T cell anergic state based on OTII cell expansion in vivo and IL2 production upon OVA323-339 restimulation ex vivo.  

      “The authors assume that this immunization protocol induces anergic cells, but they provide no experimental evidence for this.”

      Because the anergy model by Mueller's group is well established (Vanasek et al., 2006), we did not feel that additional effort was required to validate this model as the reviewer suggested. Moreover, the limited IL2 production among the control cells upon restimulation confirms the validity of this model.

      “The lack of IL-2 production by Cltr cells could be explained by the presence of fewer OVAspecific cells, rather than by an anergic status”.

      Cells from Ctrl and Nrn1-/- mice on a homogeneous TCR transgenic (OTII) background were used in these experiments. The possibility that substantial variability of TCR expression or different expression levels of the transgenic TCR could have impacted IL2 production rather than anergy induction is unlikely.

      Overall, we used this in vivo anergy model to evaluate the Nrn1-/- T cell functional state in comparison to Ctrl cells under the anergy induction condition following the evaluation of Nrn1 expression, particularly in anergic T cells.  Through studies using this anergy model, we observed a significant change in Treg induction among OTII cells. We decided to pursue the role of Nrn1 in Treg cell development and function rather than the biology of T cell anergy as evidenced by subsequent experiments.

      Minor points “(6) On which markers are anergic cells sorted for RNAseq analysis?”

      Cells were sorted out based on their congenic marker marking Ctrl or Nrn1-/- OTII cells transferred into the host mice.  We did not specifically isolate anergic cells for sequencing.

      (2) Question regarding the validity of iTreg differentiation model.

      Major point: “(5) Figure 2A-C and Figure 3. The use of iTregs to try to understand what is happening in vivo is problematic. iTregs are cells that have probably no equivalent in vivo, and so may have no physiological relevance. In any case, they are different from pTreg cells generated in vivo. Working with pTreg may be challenging, that is why I would suggest generating data with purified nTreg. Moreover, it was shown in the article of Gonzalez-Figueroa 2021 that Nrn1-/- nTreg retained a normal suppressive function, which would not be what is concluded by the authors of this manuscript. Moreover, we do not even know what the % of Foxp3 cells is in the iTreg used (after differentiation and 20h of re-stimulation) and whether this % is the same between Ctlr and Nrn1 KO cells.”.

      We thank Reviewer #1 for their feedback. While it is true that iTregs made in vitro and in vivo generated pTregs display several distinctions (e. g., differences in Foxp3 expression stability, for example), we strongly disagree with this statement by Revieweer#1 “The use of iTregs to try to understand what is happening in vivo is problematic. iTregs are cells that have probably no equivalent in vivo, and so may have no physiological relevance.” The induced Treg cell (iTreg) model was established over 20 years ago (Chen et al., 2003; Zheng et al., 2002), and the model is widely adopted with over 2000 citations. Further, it has been instrumental in understanding different aspects of regulatory T cell biology (Hurrell et al., 2022; John et al., 2022; Schmitt and Williams, 2013; Sugiura et al., 2022).   

      Because we have observed reduced pTreg generation in vivo, we choose to use the in vitro iTreg model system to understand the mechanistic changes involved in Treg cell differentiation and function, specifically, neuritin’s role in this process. We have made no claim that iTreg cell biology is identical to pTreg generated in vivo or nTreg cells. However, the iTreg culture system has proved to be a good in vitro system for deciphering molecular events involved in complex processes. As such, it remains a commonly used approach by many research groups in the Treg cell field (Hurrell et al., 2022; John et al., 2022; Sugiura et al., 2022). Moreover, applying the iTreg in vitro culture system has been instrumental in helping us identify the cell electrical state change in Nrn1-/- CD4 cells and revealed the biological link between Nrn1 and the ionotropic AMPA receptor (AMPAR), which we will discuss in the subsequent discussion. It is technically challenging to use nTreg cells for T cell electrical state studies due to their heterogeneous nature from development in an in vivo environment and the effect of manipulation during the nTreg cell isolation process, which can both affect the T cell electrical state.   

      “Moreover, it was shown in the article of Gonzalez-Figueroa 2021 that Nrn1-/- nTreg retained a normal suppressive function, which would not be what is concluded by the authors of this manuscript.” 

      We have also carried out nTreg studies in vitro in addition to iTreg cells. Similar to Gonzalez-Figueroa et al.'s findings, we did not observe differences in suppression function between Nrn1-/- and WT nTreg using the in vitro suppression assay. However, Nrn1-/- nTreg cells revealed reduced suppression function in vivo (Fig. 2D-L). In fact, Gonzalez-Figueroa et al. observed reduced plasma cell formation after OVA immunization in Treg-specific Nrn1-/- mice, implicating reduced suppression from Nrn1-/- follicular regulatory T (Tfr) cells. Thus, our observation of the reduced suppression function of Nrn1-/- nTreg toward effector T cell expansion, as presented in Fig. 2D-L, does not contradict the results from Gonzalez-Figueroa et al. Rather, the conclusions of these two studies agree that Nrn1 can play important roles in immune suppression observable in vivo that are not captured readily by the in vitro suppression assay.

      “Moreover, we do not even know what the % of Foxp3 cells is in the iTreg used (after differentiation and 20h of re-stimulation) and whether this % is the same between Ctlr and Nrn1 KO cells.”

      We have stated in the manuscript on page 7 line 208 that “Similar proportions of Foxp3+ cells were observed in Nrn1-/- and Ctrl cells under the iTreg culture condition, suggesting that Nrn1 deficiency does not significantly impact Foxp3+ cell differentiation”. In the revised manuscript, we will include the data on the proportion of Foxp3+ cells before iTreg restimulation.

      (3) Confirmation of transcriptomic data regarding amino acids or electrolytes transport change

      Minor point“(3) Would not it be possible to perform experiments showing the ability of cells to transport amino acids or electrolytes across the plasma membrane? This would be a more interesting demonstration than transcriptomic data.”

      We appreciate Review# 1’s suggestion regarding “perform experiments showing the ability of cells to transport amino acids or electrolytes across the plasma membrane”.  We have indeed already performed such experiments corroborating the transcriptomics data on differential amino acid and nutrient transporter expression. Specifically, we loaded either iTreg or Th0 cells with membrane potential (MP) dye and measured MP level change after adding the complete set of amino acids (complete AA).  Upon entry, the charge carried by AAs may transiently affect cell membrane potential. Different AA transporter expression patterns may show different MP change patterns upon AA entry, as we showed in Author response image 1. We observed reduced MP change in Nrn1-/- iTreg compared to the Ctrl, whereas in the context of Th0 cells, Nrn1-/- showed enhanced MP change than the Ctrl. We can certainly include these data in the revised manuscript.

      Author response image 1.

      Membrane potential change induced by amino acids entry. a. Nrn1-/- or WT iTreg cells loaded with MP dye and MP change was measured upon the addition of a complete set of AAs. b. Nrn1-/- or WT Th0 cells loaded with MP dye and MP change was measured upon the addition of a complete set of AAs.

      (4) EAE experiment data assessment

      Minor point ”(5) Figure 5F. How are cells re-stimulated? If polyclonal stimulation is used, the experiment is not interesting because the analysis is done with lymph node cells. This analysis should either be performed with cells from the CNS or with MOG restimulation with lymph node cells.”

      In the EAE study, the Nrn1-/- mice exhibit similar disease onset but a protracted non-resolving disease phenotype compared to the WT control mice.  Several reasons may contribute to this phenotype: 1. Enhanced T effector cell infiltration/persistence in the central nervous system (CNS); 2. Reduced Treg cell-mediated suppression to the T effector cells in the CNS; 3. Protracted non-resolving inflammation at the immunization site has the potential to continue sending T effector cells into CNS, contributing to persistent inflammation. Based on this reasoning, we examined the infiltrating T effector cell number and Treg cell proportion in the CNS.  We also restimulated cells from draining lymph nodes close to the inflammation site, looking for evidence of persistent inflammation.  When mice were harvested around day 16 after immunization, the inflammation at the local draining lymph node should be at the contraction stage.  We stimulated cells with PMA and ionomycin intended to observe all potential T effector cells involved in the draining lymph node rather than only MOG antigen-specific cells.  We disagree with Reviewer #1’s assumption that “This analysis should either be performed with cells from the CNS or with MOG restimulation with lymph node cells.”. We think the experimental approach we have taken has been appropriately tailored to the biological questions we intended to answer.

      Experimental rigor and data presentation.

      (1) Data labeling and additional supporting data

      Major points (2) The authors use Nrn1+/+ and Nrn1+/- cells indiscriminately as control cells on the basis of similar biology between Nrn1+/+ and Nrn1+/- cells at homeostasis. However, it is quite possible that the Nrn1+/- cells have a phenotype in situations of in vitro activation or in vivo inflammation (cancer, EAE). It would be important to discriminate Nrn1+/- and Nrn1+/+ cells in the data or to show that both cell types have the same phenotype in these conditions too.

      (3) Figure 1A-D. Since the authors are using the Nrp1 KO mice, it would be important to confirm the specificity of the anti-Nrn1 mAb by FACS. Once verified, it would be important to add FACS results with this mAb in Figures 1A-C to have single-cell and quantitative data as well.

      Minor points  

      (1) Line 119, 120 of the text. It is said that one of the most up-regulated genes in anergic cells is Nrn1 but the data is not shown.

      (2) For all figures showing %, the titles of the Y axes are written in an odd way. For example, it is written "Foxp3% CD4". It would be more conventional and clearer to write "% Foxp3+ / CD4+" or "% Foxp3+ among CD4+".

      (4) For certain staining (Figure 3E, H) it would be important to show the raw data, in addition to MFI or % values.

      We can adapt the labeling and provide additional data, including Nrn1 staining on Treg cells and flow graphs for pmTOR and pS6 staining (Fig. 3H), as requested by Reviewer #1.

      (2) Experimental rigor:

      General comments:

      “However, it is disappointing that reading this manuscript leaves an impression of incomplete work done too quickly.”

      We were discouraged to receive the comment, “this manuscript leaves an impression of incomplete work done too quickly.” Our study of this novel molecule began without any existing biological tools such as antibodies, knockout mice, etc.  Over the past several years, we have established our own antibodies for Nrn1 detection, obtained and characterized Nrn1 knockout mice, and utilized multiple approaches to identify the molecular mechanism of Nrn1 function. Through the use of the in vitro iTreg system described in this manuscript, we identified the association of Nrn1 deficiency with cell electrical state change, potentially connected to AMPAR function. We have further corroborated our findings by generating Nrn1 and AMPAR T cell specific double knockout mice and confirmed that T cell specific AMPAR deletion could abrogate the phenotype caused by the Nrn1 deficiency (see Author response image 2).  We did not include the double knockout data in the current manuscript because AMPAR function has not yet been studied thoroughly in T cell biology, and we feel this topic warrants examination in its own right.  However, the unpublished data support the finding that Nrn1 modulates the T cell electrical state and, consequently, metabolism, ultimately influencing tolerance and immunity.  In its current form, the manuscript represents the first characterization of the novel molecule Nrn1 in anergic cells, Tregs, and effector T cells. While this work has led to several exciting additional questions, we disagree that the novel characterization we have presented Is incomplete. We feel that our present data set, which squarely highlights Nrn1’s role as an important immune regulator while shedding unprecedented light on the molecular events involved, will be of considerable interest to a broad field of researchers.

      “Multiple models have been used, but none has been studied thoroughly enough to provide really conclusive and unambiguous data. For example, 5 different models were used to study T cells in vivo. It would have been preferable to use fewer, but to go further in the study of mechanisms.”

      We have indeed used multiple in vivo models to reveal Nrn1's function in Treg differentiation, Treg suppression function, T effector cell differentiation and function, and the overall impact on autoimmune disease. Because the impact of ion channel function is often context-dependent, we examined the biological outcome of Nrn1 deficiency in several in vivo contexts.  We would appreciate it if Reviewer#1 would provide a specific example, given the Nrn1 phenotype, of how to proceed deeper to investigate the electrical change in the in vivo models.

      “Major points (1) A real weakness of this work is the fact that in most of the results shown, there are few biological replicates with differences that are often small between Ctrl and Nrn1 -/-. The systematic use of student's t-test may lead to thinking that the differences are significant, which is often misleading given the small number of samples, which makes it impossible to know whether the distributions are Gaussian and whether a parametric test can be used. RNAseq bulk data are based on biological duplicates, which is open to criticism.”

      We respectfully disagree with Reviewer #1 on the question of statistical power and significance to our work. We have used 5-8 mice/group for each in vivo model and 3-4 technical replicates for the in vitro studies, with a minimum of 2-3 replicate experiments. These group sizes and replication numbers are in line with those seen in high-impact publications. While some differences between Ctrl and Nrn1-/- appear small, they have significant biological consequences, as evidenced by the various Nrn1-/- in vivo phenotypes. Furthermore, we believe we have subjected our data to the appropriate statistical tests to ensure rigorous analysis and representation of our findings.

      To Reviewer #2.

      We thank Reviewer #2 for the careful review of the manuscript. We especially appreciate the comments that “The characterizations of T cell Nrn1 expression both in vitro and in vivo are comprehensive and convincing. The in vivo functional studies of anergy development, Treg suppression, and EAE development are also well done to strengthen the notion that Nrn1 is an important regulator of CD4 responsiveness.”

      “The major weakness of this study stems from a lack of a clear molecular mechanism involving Nrn1. “  

      We fully understand this comment from Reviewer #2. The main mechanism we identified contributing to the functional defect of Nrn1-/- T cells involves novel effects on the electric and metabolic state of the cells. Although we referenced neuronal studies that indicate Nrn1 is the auxiliary protein for the ionotropic AMPA-type glutamate receptor (AMPAR) and may affect AMPAR function, we did not provide any evidence in this manuscript as the topic requires further in-depth study.   

      For the benefit of this discussion, we include our preliminary Nrn1 and AMPAR double knockout data (Author response image 2), which indicates that abrogating AMPAR expression can compensate for the defect caused by Nrn1 deficiency in vitro and in vivo. This preliminary data supports the notion that Nrn1 modulates AMPAR function, which causes changes in T cell electric and metabolic state, influencing T cell differentiation and function.  

      Author response image 2.

      Deletion of AMPAR expression in T cells compensates for the defect caused by Nrn1 deficiency. Nrn1-/- mice were crossed with T cell-specific AMPAR knockout mice (AMPARfl/flCD4Cre+) mice. The following mice were generated and used in the experiment: T cell specific AMPAR-knockout and Nrn1 knockout mice (AKONKO), Nrn1 knockout mice (AWTNKO), Ctrl mice (AWTNWT). a. Deletion of AMPAR compensates for the iTreg cell defect observed in Nrn1-/- CD4 cells. iTreg live cell proportion, cell number, and Ki67 expression among Foxp3+ cells 3 days after aCD3 restimulation. b. Deletion of AMPAR in T cells abrogates the enhanced autoimmune response in Nrn1-/- Mouse in the EAE disease model. Mouse relative weight change and disease score progression after EAE disease induction.  

      Ion channels can influence cell metabolism through multiple means (Vaeth and Feske, 2018; Wang et al., 2020). First, ion channels are involved in maintaining cell resting membrane potential. This electrical potential difference across the cell membrane is essential for various cellular processes, including metabolism (Abdul Kadir et al., 2018; Blackiston et al., 2009; Nagy et al., 2018; Yu et al., 2022). Second, ion channels facilitate the movement of ions across cell membranes. These ions are essential for various metabolic processes. For example, ions like calcium (Ca2+), potassium (K+), and sodium (Na+) play crucial roles in signaling pathways that regulate metabolism (Kahlfuss et al., 2020). Third, ion channel activity can influence cellular energy balance due to ATP consumption associated with ion transport to maintain ion balances (Erecińska and Dagani, 1990; Gerkau et al., 2019). This, in turn, can impact processes like ATP production, which is central to cellular metabolism. Thus, ion channel expression and function determine the cell’s bioelectric state and contribute to cell metabolism (Levin, 2021).

      Because the AMPAR function has not been thoroughly studied using a genetic approach in T cells, we do not intend to include the double knockout data in this manuscript before fully characterizing the T cell-specific AMPAR knockout mice.  

      “Although the biochemical and informatics studies are well-performed, it is my opinion that these results are inconclusive in part due to the absence of key "naive" control groups. This limits my ability to understand the significance of these data.

      Specifically, studies of the electrical and metabolic state of Nrn1-/- inducible Treg cells (iTregs) would benefit from similar data collected from wild-type and Nrn1-/- naive CD4 T cells.”

      We appreciate the reviewer’s comments. This comment reflects two concerns in data interpretation:

      (1) Are Nrn1-/- naïve T cells fundamentally different from WT cells? Does this fundamental difference contribute to the observed electrical and metabolic phenotype in iTreg or Th0 cells? This is a very good question we will perform the experiments as the reviewer suggested. While Nrn1 is expressed at a basal (low) level in naïve T cells, deletion of Nrn1 may cause changes in naïve T cell phenotype.   

      (2) Is the Nrn1-/- phenotype caused by Nrn1 functional deficiency or due to the secondary effect of Nrn1 deletion, such as non-physiological cell membrane structure changes?

      We have done the following experiment to address this concern.  We have cultured WT T cells in the presence of Nrn1 antibody and compared the outcome with Nrn1-/- iTreg cells (Author response image 3). WT iTreg cells under antibody blockade exhibited similar changes as Nrn1-/- iTreg cells, confirming the physiological relevance of the Nrn1-/- phenotype.

      Author response image 3.

      Nrn1 antibody blockade in WT iTreg cell culture caused similar phenotypic change as in Nrn1-/- iTreg cells. Nrn1-/- and WT CD4 cells were differentiated under iTreg condition in the presence of anti-Nrn1 (aNrn1) antibody or isotype control for 3 days. Cells were restimulated with anti-CD3 and in the presence of aNrn1 or isotype. a. MP measured 18hr after anti-CD3 restimulation. b. live CD4 cell number and proportion of Ki67 expression among live cells three days after restimulation. c. The proportion of Foxp3+ cells among live cells three days after restimulation.  

      Reference:

      Abdul Kadir, L., M. Stacey, and R. Barrett-Jolley. 2018. Emerging Roles of the Membrane Potential: Action Beyond the Action Potential. Front Physiol 9:1661.

      Blackiston, D.J., K.A. McLaughlin, and M. Levin. 2009. Bioelectric controls of cell proliferation: ion channels, membrane voltage and the cell cycle. Cell Cycle 8:3527-3536.

      Chappert, P., and R.H. Schwartz. 2010. Induction of T cell anergy: integration of environmental cues and infectious tolerance. Current opinion in immunology 22:552-559.

      Chen, W., W. Jin, N. Hardegen, K.J. Lei, L. Li, N. Marinos, G. McGrady, and S.M. Wahl. 2003. Conversion of peripheral CD4+CD25- naive T cells to CD4+CD25+ regulatory T cells by TGF-beta induction of transcription factor Foxp3. The Journal of experimental medicine 198:1875-1886.

      Erecińska, M., and F. Dagani. 1990. Relationships between the neuronal sodium/potassium pump and energy metabolism. Effects of K+, Na+, and adenosine triphosphate in isolated brain synaptosomes. J Gen Physiol 95:591-616.

      Fathman, C.G., and N.B. Lineberry. 2007. Molecular mechanisms of CD4+ T-cell anergy. Nat Rev Immunol 7:599-609.

      Gerkau, N.J., R. Lerchundi, J.S.E. Nelson, M. Lantermann, J. Meyer, J. Hirrlinger, and C.R. Rose. 2019. Relation between activity-induced intracellular sodium transients and ATP dynamics in mouse hippocampal neurons. The Journal of physiology 597:5687-5705.

      Hurrell, B.P., D.G. Helou, E. Howard, J.D. Painter, P. Shafiei-Jahani, A.H. Sharpe, and O. Akbari. 2022. PD-L2 controls peripherally induced regulatory T cells by maintaining metabolic activity and Foxp3 stability. Nature communications 13:5118.

      Jenkins, M.K., and R.H. Schwartz. 1987. Antigen presentation by chemically modified splenocytes induces antigen-specific T cell unresponsiveness in vitro and in vivo. The Journal of experimental medicine 165:302-319.

      John, P., M.C. Pulanco, P.M. Galbo, Jr., Y. Wei, K.C. Ohaegbulam, D. Zheng, and X. Zang. 2022. The immune checkpoint B7x expands tumor-infiltrating Tregs and promotes resistance to anti-CTLA-4 therapy. Nature communications 13:2506.

      Kahlfuss, S., U. Kaufmann, A.R. Concepcion, L. Noyer, D. Raphael, M. Vaeth, J. Yang, P. Pancholi, M. Maus, J. Muller, L. Kozhaya, A. Khodadadi-Jamayran, Z. Sun, P. Shaw, D. Unutmaz, P.B. Stathopulos, C. Feist, S.B. Cameron, S.E. Turvey, and S. Feske. 2020. STIM1-mediated calcium influx controls antifungal immunity and the metabolic function of nonpathogenic Th17 cells. EMBO molecular medicine 12:e11592.

      Levin, M. 2021. Bioelectric signaling: Reprogrammable circuits underlying embryogenesis, regeneration, and cancer. Cell 184:1971-1989.

      Nagy, E., G. Mocsar, V. Sebestyen, J. Volko, F. Papp, K. Toth, S. Damjanovich, G. Panyi, T.A. Waldmann, A. Bodnar, and G. Vamosi. 2018. Membrane Potential Distinctly Modulates Mobility and Signaling of IL-2 and IL-15 Receptors in T Cells. Biophys J 114:2473-2482.

      Quill, H., and R.H. Schwartz. 1987. Stimulation of normal inducer T cell clones with antigen presented by purified Ia molecules in planar lipid membranes: specific induction of a long-lived state of proliferative nonresponsiveness. Journal of immunology (Baltimore, Md. : 1950) 138:3704-3712.

      Schmitt, E.G., and C.B. Williams. 2013. Generation and function of induced regulatory T cells. Frontiers in immunology 4:152.

      Sugiura, A., G. Andrejeva, K. Voss, D.R. Heintzman, X. Xu, M.Z. Madden, X. Ye, K.L. Beier, N.U. Chowdhury, M.M. Wolf, A.C. Young, D.L. Greenwood, A.E. Sewell, S.K. Shahi, S.N. Freedman, A.M. Cameron, P. Foerch, T. Bourne, J.C. Garcia-Canaveras, J. Karijolich, D.C. Newcomb, A.K. Mangalam, J.D. Rabinowitz, and J.C. Rathmell. 2022. MTHFD2 is a metabolic checkpoint controlling effector and regulatory T cell fate and function. Immunity 55:65-81.e69.

      Vaeth, M., and S. Feske. 2018. Ion channelopathies of the immune system. Current opinion in immunology 52:39-50.

      Vanasek, T.L., S.L. Nandiwada, M.K. Jenkins, and D.L. Mueller. 2006. CD25+Foxp3+ regulatory T cells facilitate CD4+ T cell clonal anergy induction during the recovery from lymphopenia. Journal of immunology (Baltimore, Md. :1950) 176:5880-5889.

      Wang, Y., A. Tao, M. Vaeth, and S. Feske. 2020. Calcium regulation of T cell metabolism. Current opinion in physiology 17:207-223.

      Yu, W., Z. Wang, X. Yu, Y. Zhao, Z. Xie, K. Zhang, Z. Chi, S. Chen, T. Xu, D. Jiang, X. Guo, M. Li, J. Zhang, H. Fang, D. Yang, Y. Guo, X. Yang, X. Zhang, Y. Wu, W. Yang, and D. Wang. 2022. Kir2.1-mediated membrane potential promotes nutrient acquisition and inflammation through regulation of nutrient transporters. Nature communications 13:3544.

      Zheng, S.G., J.D. Gray, K. Ohtsuka, S. Yamagiwa, and D.A. Horwitz. 2002. Generation ex vivo of TGF-beta-producing regulatory T cells from CD4+CD25- precursors. Journal of immunology (Baltimore, Md. : 1950) 169:4183-4189.

    1. Author response:

      Reviewer #1 (Public review):

      Summary:

      Loh and colleagues investigate valence encoding in the mesolimbic dopamine system. Using an elegant approach, they show that sucrose, which normally evokes strong dopamine neuron activity and release in the nucleus accumbens, is made aversive via conditioned taste aversion, the same sucrose stimulus later evokes much less dopamine neuron activity and release. Thus, dopamine activity can dynamically track the changing valence of an unconditioned stimulus. These results are important for helping clarify valence and value related questions that are the matter of ongoing debate regarding dopamine functions in the field.

      Strengths:

      This is an elegant way to ask this question, the within subject's design and the continuity of the stimulus is a strong way to remove a lot of the common confounds that make it difficult to interpret valence-related questions. I think these are valuable studies that help tie up questions in the field while also setting up a number of interesting future directions. There are number of control experiments and tweaks to the design that help eliminate a number of competing hypotheses regarding the results. The data are clearly presented and contextualized.

      Weaknesses for consideration:

      The focus on one relatively understudied region of the rat striatum for dopamine recordings could potentially limit generalization of the findings. While this can be determined in future studies, the implications should be further discussed in the current manuscript.

      We agree that the manuscript would benefit from providing a stronger rationale for our recording sites and acknowledging the potential for regional differences in dopamine signaling. We have made the following additions to the manuscript:

      Added to the Discussion: “Recordings were targeted to the lateral VTA and the corresponding approximate terminal site in the NAc lateral shell (Lammel et al., 2008). Subregional differences in dopamine activity likely contribute to mixed findings on dopamine and affect. For example, dopamine in the NAc lateral shell differentially encodes cues predictive of rewarding sucrose and aversive footshock, which is distinct from NAc medial shell dopamine responses (de Jong et al., 2019). Our findings are similar to prior work from our group targeting recordings to the NAc dorsomedial shell (Hsu et al., 2020; McCutcheon et al., 2012; Roitman et al., 2008): there, intraoral sucrose increased NAc dopamine release while the response in the same rats to quinine was significantly lower.”

      Reviewer #2 (Public review):

      Summary:

      Koh et al. report an interesting manuscript studying dopamine binding in the lateral accumbens shell of rats across the course of conditioned taste aversion. The question being asked here is how does the dopamine system respond to aversion? The authors take advantage of unique properties of taste aversion learning (notably, within-subjects remapping of valence to the same physical stimulus) to address this.

      They combine a well controlled behavioural design (including key, unpaired controls) with fibre photometry of dopamine binding via GrabDA and of dopamine neuron activity by gCaMP, careful analyses of behaviour (e.g., head movements; home cage ingestion), the authors show that, 1) conditioned taste aversion of sucrose suppresses the activity of VTA dopamine neurons and lateral shell dopamine binding to subsequent presentations of the sucrose tastant; 2) this pattern of activity was similar to the innately aversive tastant quinine; 3) dopamine responses were negatively correlated with behavioural (inferred taste reactivity) reactivity; and 4) dopamine responses tracked the contingency of between sucrose and illness because these responses recovered across extinction of the conditioned taste aversion.

      Strengths:

      There are important strengths here. The use of a well-controlled design, the measurement of both dopamine binding and VTA dopamine neuron activity, the inclusion of an extinction manipulation; and the thorough reporting of the data. I was not especially surprised by these results, but these data are a potentially important piece of the dopamine puzzle (e.g., as the authors note, salience-based argument struggles to explain these data).

      Weaknesses for consideration:

      (1) The focus here is on the lateral shell. This is a poorly investigated region in the context of the questions being asked here. Indeed, I suspect many readers might expect a focus on the medial shell. So, I think this focus is important. But, I think it does warrant greater attention in both the introduction and discussion. We do know from past work that there can be extensive compartmentalisation of dopamine responses to appetitive and aversive events and many of the inconsistent findings in the literature can be reconciled by careful examination of where dopamine is assessed. I do think readers would benefit from acknowledgement this - for example it is entirely reasonable to suppose that the findings here may be specific to the lateral shell.

      As with our response to Reviewer 1, we agree that we should provide further rationale for focusing our recordings on the lateral shell and acknowledge potential differences in dopamine dynamics across NAc subregions. In addition to the changes in the Discussion detailed in our response to Reviewer 1, we have made the following additions to the Introduction:

      Added to the Introduction: “NAc lateral shell dopamine differentially encodes cues predictive of rewarding (i.e., sipper spout with sucrose) and aversive stimuli (i.e., footshock), which is distinct from other subregions (de Jong et al., 2019). It is important to note that other regions of the NAc may serve as hedonic hotspots (e.g. dorsomedial shell; or may more closely align with the signaling of salience (e.g. ventromedial shell; (Yuan et al., 2021)).”

      (2) Relatedly, I think readers would benefit from an explicit rationale for studying the lateral shell as well as consideration of this in the discussion. We know that there are anatomical (PMID: 17574681), functional (PMID: 10357457), and cellular (PMID: 7906426) differences between the lateral shell and the rest of the ventral striatum. Critically, we know that profiles of dopamine binding during ingestive behaviours there can be highly dissimilar to the rest of ventral striatum (PMID: 32669355). I do think these points are worth considering.

      There are several reasons why dopamine dynamics were recorded in the NAc lateral shell:

      (1) Dopamine neurons in more medial aspects of the VTA preferentially target the NAc medial shell and core whereas dopamine neurons in the lateral VTA – our target for VTA DA recordings – project to the lateral shell of the NAc (Lammel et al., 2008). Thus, our goal was to sample NAc release dynamics in areas that receive projections from our cell body recording sites.

      (2) Cues predictive of reward availability (i.e., sipper spout with sucrose) and aversive stimuli (i.e., footshock) are differentially encoded by NAc lateral shell dopamine, which is distinct from NAc ventromedial shell dopamine responses (de Jong et al., 2019). These findings suggest a role for NAc lateral shell dopamine in the encoding of a stimulus’s valence, which made the subregion an area of interest for further examination.

      (3) With respect to the medial NAc shell specifically, extensive literature had already shown it to be a ‘hedonic hotspot’ (Morales and Berridge, 2020; Yuan et al., 2021) whereas the ventral portion is more mixed with respect to valence (Yuan et al., 2021). We had previously shown that intraoral infusions of primary taste stimuli of opposing valence (i.e., sucrose and quinine) evoke differential responses in dopamine release within the NAc dorsomedial shell (Roitman et al., 2008). We more recently replicated differential dopamine responses from dopamine cell bodies in the lateral VTA (Hsu et al., 2020) and thus endeavored to the possibility of changing dopamine responses in the lateral VTA to the same stimulus as its valence changes. As a result of these choices, measuring dopamine release in the lateral shell was a logical choice. The field would greatly benefit from continued future work surveying the entirety of the VTA DA projection terminus. 

      We have included these points of justification in the Introduction and Discussion sections.

      (3) I found the data to be very thoughtfully analysed. But in places I was somewhat unsure:

      (a) Please indicate clearly in the text when photometry data show averages across trials versus when they show averages across animals.

      We have now explicitly indicated in the figure legends of Figures 1, 3, 5, 7, and 8:

      (1) In heat maps, each row represents the averaged (across rats) response on that trial.

      (2) Traces below heat maps represent the response to infusion averaged first across trials for each rat and then across all rats.

      (3) Insets represent the average z-score across the infusion period averaged first across all trials for each rat and then across all rats.

      (b) I did struggle with the correlation analyses, for two reasons.

      (i) First, the key finding here is that the dopamine response to intraoral sucrose is suppressed by taste aversion. So, this will significantly restrict the range of dopamine transients, making interpretation of the correlations difficult.

      The overall hypothesis is that the dopamine response would correlate with the valence of a taste stimulus – even and especially when the stimulus remained constant but its valence changed. We inferred valence from the behavioral reactivity to the stimulus – reasoning that an appetitive taste will evoke minimal movement of the nose and paws (presumably because the animals are primarily engaging in small mouth movements associated with ingestion as shown by the seminal work of Grill and Norgren (1978) and the many studies published by the K.C. Berridge group) whereas an aversive taste will evoke significantly more movement as the rats engage in rejection responses (e.g. forelimb flails, chin rubs, etc.). When we conducted our regression analyses we endeavored to be as transparent as possible and labeled each symbol based on group (Unpaired vs Paired) and day (Conditioning vs Test). Both behavioral reactivity and dopamine responses change – but only for the Paired rats across days. In this sense, we believe the interpretation is clear. However, the Reviewer raises an important criticism that there would essentially be a floor effect with dopamine responses. We believe this is mitigated by data acquired across extinction and especially in Figure 9B. Here, the observations that dopamine responses fall to near zero but return to pre-conditioning levels in the Paired group with strong correlation between dopamine and behavioral reactivity throughout would hopefully partially allay the Reviewer’s concerns. See Part ii below for further support.

      (ii) Second, the authors report correlations by combining data across groups/conditions. I understand why the authors have done this, but it does risk obscuring differences between the groups. So, my question is: what happens to this trend when the correlations are computed separately for each group? I suspect other readers will share the same question. I think reporting these separate correlations would be very helpful for the field -

      regardless of the outcome.

      To address this concern, we performed separate regression analyses for Paired and Unpaired rats and provide the table below to detail results where data were combined across groups or separated. Expectedly, all analyses in Paired rats indicated a significant inverse relationship between dopamine and behavioral reactivity. Afterall, it is only in this group where behavioral reactivity to the taste stimulus changes as function of conditioning. Perhaps even more striking is that in almost all comparisons, even when restricting the regression analysis to Unpaired rats, we still observed a significant inverse relationship between dopamine and behavioral reactivity in most experiments. We have outlined the separated correlations below (asterisks denote slopes significantly different from 0; * p<0.05; ** p<0.01; *** p<0.005; **** p<0.001):

      Author response table 1.

      (4) Figure 1A is not as helpful as it might be. I do think readers would expect a more precise reporting of GCaMP expression in TH+ and TH- neurons. I also note that many of the nuances in terms of compartmentalisation of dopamine signalling discussed above apply to ventral tegmental area dopamine neurons (e.g. medial v lateral) and this is worth acknowledging when interpreting t

      Others have reported (Choi et al., 2020) and quantified (Hsu et al., 2020) GCaMP6f expression in TH+ neurons. While we didn’t report these quantifications, our observations were very much in line with previous quantifications from our laboratory (Hsu et al. 2020).

      We agree that we should elaborate on VTA subregional differences and have answered this response above (See responses to Reviewer 1 Weakness #1 and Reviewer 2 Weakness #2).

      Reviewer #3 (Public review):

      Summary:

      This study helps to clarify the mixed literature on dopamine responses to aversive stimuli. While it is well accepted that dopamine in the ventral striatum increases in response to various rewarding and appetitive stimuli, aversive stimuli have been shown to evoke phasic increases or decreasing depending on the exact aversive stimuli, behavioral paradigm, and/or dopamine recording method and location examined. Here the authors use a well-designed set of experiments to show differential responses to an appetitive primary reward (sucrose) that later becomes a conditioned aversive stimulus (sucrose previously paired with lithium chloride in a conditioned taste aversion paradigm). The results are interesting and add valuable data to the question of how the mesolimbic dopamine system encodes aversive stimuli, however, the conclusions are strongly stated given that the current data do not necessarily align with prior conflicting data in terms of recording location, and it is not clear exactly how to interpret the generally biphasic dopamine response to the CTA-sucrose which also evolves over exposures within a single session.

      Strengths:

      • The authors nicely demonstrate that their two aversive stimuli examined, quinine and sucrose following CTA, evoked aversive facial expressions and paw movements that differed from those following rewarding sucrose to support that the stimuli experienced by the rats differ in valence.

      • Examined dopamine responses to the exact same sensory stimuli conditioned to have opposing valences, avoiding standard confounds of appetitive and aversive stimuli being sensed by different sensory modalities (i.e., sweet taste vs. electric shock)

      • The authors examined multiple measurements of dopamine activity - cell body calcium (GCaMP6f) in midbrain and release in NAc (Grab-DA2h), which is useful as the prior mixed literature on aversive dopamine responses comes from a variety of recording methods.

      • Correlations between sucrose preference and dopamine signals demonstrate behavioral relevance of the differential dopamine signals.

      • The delayed testing experiment in Figure 7 nicely controls for the effect of time to demonstrate that the "rewarding" dopamine response to sucrose only recovers after multiple extinction sucrose exposures to extinguish the CTA.

      Weaknesses for consideration:

      (1) Regional differences in dopamine signaling to aversive stimuli are mentioned in the introduction and discussion. For instance, the idea that dopamine encodes salience is strongly argued against in the discussion, but the paper cited as arguing for that (Kutlu et al. 2021) is recording from the medial core in mice. Given other papers cited in the text about the regional differences in dopamine signaling in the NAc and from different populations of dopamine neurons in midbrain, it's important to mention this distinction wrt to salience signaling. Relatedly, the text says that the lateral NAc shell was targeted for accumbens recordings, but the histology figure looks like the majority of fibers were in the anterior lateral core of NAc. For the current paper to be a convincing last word on the issue, it would be extremely helpful to have similar recordings done in other parts of the NAc to do a more thorough comparison against other studies.

      As the Reviewer notes, NAc dopamine recordings were aimed at the lateral NAc shell. It is possible that some dopamine neurons lying within the anterior lateral core were recorded. Fiber photometry and the size of the fiber optics cannot definitively identify the precise location and number of dopamine neurons from which we recorded. Still, recording sites did not systematically differ between groups. Further, the within-subjects design helps to mitigate any potential biases for one subregion over another. The results presented in the manuscript strongly support a valence code. It is difficult to be the ‘last word’ on this topic and we suspect debate will continue. We used taste stimuli for appetitive and aversive stimuli – whereas many in the field will continue to use other noxious stimuli (e.g. foot shock) that likely recruit different circuits en route to the VTA. And there may very well be a different regional profile for dopamine signaling with different noxious stimuli. Moreover, we used intraoral infusion to avoid confounds of stimulus avoidance and competing motivations (e.g. food or fluid deprivation). We believe that this is one of the most important and unique features of our report. Recent work supports a role for phasic increases in dopamine in avoidance of noxious stimuli (Jung et al., 2024) and it will be critical for the field to reflect on the differences between avoidance and aversion. Moreover, in ongoing studies we aspire to fully survey dopamine signaling in conditioned taste aversion across the medial-lateral and dorsal-ventral axes of the VTA and NAc.

      (2) Dopamine release in the NAc never dips below baseline for the conditioned sucrose. Is it possible to really consider this as a signal for valence per se, as opposed to it being a weaker response relative to the original sucrose response?

      Indeed, NAc dopamine release to intraoral quinine nor aversive sucrose doesn’t dip below baseline but rather dopamine binding doesn’t change from pre-infusion baseline levels. It should be noted that VTA dopamine cell body activity does indeed dip below baseline in response to aversive sucrose. Moreover, using fast-scan cyclic voltammetry, we showed that dopamine release dips below baseline in the NAc dorsomedial shell in response to intraoral quinine (Roitman et al., 2008). The differences across recording sites may reflect regional differences but they may also reflect differences in recording approaches. GrabDA2h, used here, has relatively slow kinetics that may obscure dips below baseline (see response Weakness# 8 below).

      (3) Related to this, the main measure of the dopamine signal here, "mean z-score," obscures the temporal dynamics of the aversive dopamine response across a trial. This measure is used to claim that sucrose after CTA is "suppressing" dopamine neuron activity and release, which is true relative to the positive valence sucrose response. However, both GRAB-DA and cell-body GCaMP measurements show clear increases after onset of sucrose infusion before dipping back to baseline or slightly below in the average of all example experiments displayed. One could point to these data to argue either that aversive stimuli cause phasic increases in dopamine (due to the initial increase) or decreases (due to the delayed dip below baseline) depending on the measurement window. Some discussion of the dynamics of the response and how it relates to the prior literature would be useful.

      We have used mean z-score to do much of our quantitative analyses but the Reviewer raises the intriguing possibility that we are masking an initial increase in dopamine release and VTA DA activity evoked by aversive taste by doing so. We included the heat maps in the manuscript to be as transparent as possible about the time course of dopamine responses – both within a trial and across trials. The Reviewer’s point prompted us to reflect further on the heat maps and recognize that trials early in the session often showed a brief increase in dopamine for aversive sucrose but this response dissipated (NAc dopamine release) or flipped (VTA DA cell body activity) over trials. We now quantitatively characterize this feature by looking at the timecourse of dopamine responses in each third of the trials (1-10, 11-20, 21-30; see Author response images 1,2 and 3). As we infer the valence of the stimulus from nose and paw movements (behavioral reactivity), it is especially striking that we a similar timecourse for changes in behavior. Collectively, the data may reflect an updating process that is relatively slow and requires experience of the stimulus in a new (aversive) state – that is, a model-free process. While our experiments were not designed to test the updating of dopamine responses and discern their participation in model-based versus model-free learning processes – another debate in the dopamine field (Cone et al., 2016; Deserno et al., 2021)– the data reflect a model-free process. This is further supported in the experiment involving multiple conditioning sessions, where dopamine ‘dips’ are observed in trials 1-10 on Conditioning Day 3 and Extinction Day 1 when the new value of sucrose has been established. Finally, the relatively slow updating of the value of sucrose is reflected in older literature using a continuous intraoral infusion. Using this approach, rats began rejecting the saccharin infusion only after ~2min rather than immediately (Schafe et al., 1998; Schafe and Bernstein, 1996; Wilkins and Bernstein, 2006).   

      Author response image 1.

      Author response image 2.

      Author response image 3.

      (4) Would this delayed below-baseline dip be visible with a shorter infusion time?

      While our experiments did not explore this parameter, it would be interesting to parametrically vary infusion duration times and examine differences in dopamine responses. However, we believe the most parsimonious explanation is that the ‘dip’ in VTA cell body activity develops as a function of the slow updating of the value of sucrose reflective of a model-free process. We recognize that this is mere speculation.

      (5) Does the max of the increase or the dip of the decrease better correlate with the behavioral measures of aversion (orofacial, paw movements) or sucrose preference than "mean z-score" measure used here?

      It seems plausible that finding the most extreme value from baseline could better correlate to behavioral measures. Time courses to max increase and max decrease are different. Moreover, with appetitive sucrose, there are often multiple transients that occur throughout a single intraoral infusion. Coupled with a noisy time course for individual components of behavioral reactivity, we determined that averaging data across the whole infusion period (i.e. mean z-score) was the most objective way we could analyze the dopamine and behavioral responses to taste stimuli.

      (6) The authors argue strongly in the discussion against the idea that dopamine is encoding "salience." Could this initial peak (also seen in the first few trials of quinine delivery, fig 1c color plot) be a "salience" response?

      Our response above to the potential for ‘mixed’ dopamine responses to aversive sucrose led to additional analyses that support a slow updating of both behavior and dopamine to the new, aversive value of sucrose. Quinine is innately aversive and thus the Reviewer rightly points out that even here we observe an increase in dopamine release evoked by quinine on the first few trials (as observed in the heat map). We’d like to note, though, that the order of stimulus exposure was counterbalanced across rats. In those rats first receiving a sucrose session, quinine initially caused a modest increase in dopamine release during the first 10 trials (which is more pronounced in the first 2 trials). In the subsequent 2 blocks of 10 trials, no such increase was observed. Interestingly, in rats for which quinine was their first stimulus, we did not see an increase in dopamine release on the first few trials (see Author response image 4). We speculate that the initial sucrose session required the value of intraoral infusions to be updated when quinine was delivered to these rats and that, once more, the updating process may be slow and akin to a model-free process. This analysis, at present, is underpowered but will direct future attention in follow-up work.

      Author response image 4.

      (7) Related to this, the color plots showing individual trials show a reduction in the increases to positive valence sucrose across conditioning day trials and a flip from infusion-onset increase to delayed increases across test day trials. This evolution across days makes it appear that the last few conditioning day trials would be impossible to discriminate from the first few test day trials in the CTA-paired. Presumably, from strength of CTA as a paradigm, the sucrose is already aversive to the animals at the first trial of test day. Why do the authors think the response evolves across this session?

      As the Reviewer noted, Points 3-7 are related. We have speculated that the evolving dopamine response in Paired rats across test day trials reflects a model-free process. Importantly, as in the manuscript, our additional analyses once again show a tight relationship between behavioral reactivity and the dopamine response across the test session trials. It is important to note, though, that these experiments were not designed to test if responses reflect model-free or model-based processes.

      (8) Given that most of the work is using a conditioned aversive stimulus, the comparison to a primary aversive tastant quinine is useful. However, the authors saw basically no dopamine response to a primary aversive tastant quinine (measured only with GRAB-DA) and saw less noticeable decreases following CTA for NAc recordings with GRAB-DA2h than with cell body GCaMP. Given that they are using the high-affinity version of the GRAB sensor, this calls into question whether this is a true difference in release vs. soma activity or issue of high affinity release sensor making decreases in dopamine levels more difficult to observe.

      We share the same speculation as the Reviewer. Using fast-scan cyclic voltammetry, albeit measuring dopamine concentration in the dorsomedial shell, we observed a clear decrease from baseline with intraoral infusions of quinine (Roitman et al., 2008). Using fiber photometry here, the Reviewer and we note that GRAB_DA2h is a high-affinity (i.e., EC50: 7nM) dopamine sensor with relatively long off-kinetics (i.e., t1/2 decay time: 7300ms) (Labouesse et al., 2020). It may therefore be much more difficult to observe decreases (below baseline) using this sensor. The publication of new dopamine sensors - with lower affinity, faster kinetics, and greater dynamic range (Zhuo et al., 2024) – introduces opportunities for comparison and the greater potential for capturing decreases below baseline. Due to the poorer kinetics associated with GRAB_DA2h, we would not assert that direct comparisons between the GCaMP- and GRAB-based signals observed here represent true differences between somatic and terminal activity.

      References

      Choi JY, Jang HJ, Ornelas S, Fleming WT, Fürth D, Au J, Bandi A, Engel EA, Witten IB. 2020. A Comparison of Dopaminergic and Cholinergic Populations Reveals Unique Contributions of VTA Dopamine Neurons to Short-Term Memory. Cell Rep 33. doi:10.1016/j.celrep.2020.108492

      Cone JJ, Fortin SM, McHenry JA, Stuber GD, McCutcheon JE, Roitman MF. 2016. Physiological state gates acquisition and expression of mesolimbic reward prediction signals. Proc Natl Acad Sci U S A 113. doi:10.1073/pnas.1519643113

      de Jong JW, Afjei SA, Pollak Dorocic I, Peck JR, Liu C, Kim CK, Tian L, Deisseroth K, Lammel S. 2019. A Neural Circuit Mechanism for Encoding Aversive Stimuli in the Mesolimbic Dopamine System. Neuron 101. doi:10.1016/j.neuron.2018.11.005

      Deserno L, Moran R, Michely J, Lee Y, Dayan P, Dolan RJ. 2021. Dopamine enhances model-free credit assignment through boosting of retrospective model-based inference. Elife 10. doi:10.7554/eLife.67778

      Hsu TM, Bazzino P, Hurh SJ, Konanur VR, Roitman JD, Roitman MF. 2020. Thirst recruits phasic dopamine signaling through subfornical organ neurons. Proc Natl Acad Sci U S A 117:30744–30754. doi:10.1073/PNAS.2009233117/-/DCSUPPLEMENTAL

      Jung K, Krüssel S, Yoo S, An M, Burke B, Schappaugh N, Choi Y, Gu Z, Blackshaw S, Costa RM, Kwon HB. 2024. Dopamine-mediated formation of a memory module in the nucleus accumbens for goal-directed navigation. Nat Neurosci. doi:10.1038/s41593-024-01770-9

      Labouesse MA, Cola RB, Patriarchi T. 2020. GPCR-based dopamine sensors—A detailed guide to inform sensor choice for in vivo imaging. Int J Mol Sci. doi:10.3390/ijms21218048

      Lammel S, Hetzel A, Häckel O, Jones I, Liss B, Roeper J. 2008. Unique Properties of Mesoprefrontal Neurons within a Dual Mesocorticolimbic Dopamine System. Neuron 57. doi:10.1016/j.neuron.2008.01.022

      McCutcheon JE, Ebner SR, Loriaux AL, Roitman MF, Tobler PN. 2012. Encoding of aversion by dopamine and the nucleus accumbens. Front Neurosci 6. doi:10.3389/fnins.2012.00137

      Morales I, Berridge KC. 2020. ‘Liking’ and ‘wanting’ in eating and food reward: Brain mechanisms and clinical implications. Physiol Behav. doi:10.1016/j.physbeh.2020.113152

      Roitman MF, Wheeler RA, Wightman RM, Carelli RM. 2008. Real-time chemical responses in the nucleus accumbens differentiate rewarding and aversive stimuli. Nature Neuroscience 2008 11:12 11:1376–1377. doi:10.1038/nn.2219

      Schafe GE, Bernstein IL. 1996. Forebrain contribution to the induction of a brainstem correlate of conditioned taste aversion: I. The amygdala. Brain Res 741. doi:10.1016/S0006-8993(96)00906-7

      Schafe GE, Thiele TE, Bernstein IL. 1998. Conditioning method dramatically alters the role of amygdala in taste aversion learning. Learning and Memory 5. doi:10.1101/lm.5.6.481

      Wilkins EE, Bernstein IL. 2006. Conditioning method determines patterns of c-fos expression following novel taste-illness pairing. Behavioural Brain Research 169. doi:10.1016/j.bbr.2005.12.006

      Yuan L, Dou YN, Sun YG. 2021. Topography of reward and aversion encoding in the mesolimbic dopaminergic system. Journal of Neuroscience 39. doi:10.1523/JNEUROSCI.0271-19.2019

      Zhuo Y, Luo B, Yi X, Dong H, Miao X, Wan J, Williams JT, Campbell MG, Cai R, Qian T, Li F, Weber SJ, Wang L, Li B, Wei Y, Li G, Wang H, Zheng Y, Zhao Y, Wolf ME, Zhu Y, Watabe-Uchida M, Li Y. 2024. Improved green and red GRAB sensors for monitoring dopaminergic activity in vivo. Nat Methods 21. doi:10.1038/s41592-023-02100-w

    1. Author response:

      Reviewer #1 (Public review):

      Summary:

      This is an interesting study on the role of FGF signaling in the induction of primitive streak-like cells (PS-LC) in human 2D-gastruloids. The authors use a previously characterized standard culture that generates a ring of PS-LCs (TBXT+) and correlate this with pERK staining. A requirement for FGF signaling in TBXT induction is demonstrated via pharmacological inhibition of MEK and FGFR activity. A second set of culture conditions (with no exogenous FGFs) suggests that endogenous FGFs are required for pERK and TBXT induction. The authors then characterize, via scRNA-seq, various components of the FGF pathway (genes for ligands, receptors, ERK regulators, and HSPG regulation). They go on to characterize the pFGFR1, receptor isoforms, and polarized localization of this receptor. Finally, they perform FGF4 inhibition and use a cell line with a limited FGF17 inactivation (heterozygous null) and show that loss of these FGFs reduces PS-LC and derivative cell types.

      Strengths:

      (1) As the authors point out, the role of FGF signaling in gastrulation is less well understood than other signaling pathways. Hence this is a valuable contribution to that field.

      (2) The FGF4 and FGF17 loss-of-function experiments in Figure 5 are very intriguing. This is especially so given the intriguing observation that these FGFs appear to be dominating in this model of human gastrulation, in contrast to what FGFs dominate in mice, chicks, and frogs.

      (3) In general this paper is valuable as a further development of the Human gastruloid system and the role of FGF signaling in the induction of PS-CLs. The wide net that the authors cast in characterizing the FGF ligand gene, receptor isoforms, and downstream components provides a foundation for future work. As the authors write near the beginning of the Discussion "Many questions remain."

      We thank the reviewer for these positive comments.

      Weaknesses:

      (1) FGFs are cell survival factors in various aspects of development. The authors fail to address cell death due to loss of FGF signaling in their experiments. For example, in Figure 1E (which requires statistical analysis) and 1G (the bottom FGFRi row), there appears to be a significant amount of cell loss. Is this due to cell death? The authors should address the question of whether the role of FGF/ERK signaling is to keep the cells alive.

      Indeed, FGF also strongly affects cell number and it is an interesting question to what extent this depends on ERK. Our manuscript focuses instead on the role of FGF/ERK signaling in cell fate patterning. However, as mentioned in our discussion, figure 1de show that doxycycline induced pERK leads to more TBXT+ cells than the control without restoring cell number, suggesting the role of FGF in controlling cell number is independent of the requirement for FGF/ERK in PS-LC differrentiation. Unpublished data below showing a MEK inhibitor dose response further supports this: low doses of MEKi are sufficient to inhibit differentiation without affecting cell number. To address the reviewer’s question we will include this data in the revised manuscript and perform several additional experiments to determine in more detail how cell death and proliferation depend on FGF.

      Author response image 1.

      MEK affects differentiation and cell number at different doses. a-c) control and MEKi (0.3uM) treated colonies with similar cell number but different TBXT expression. d-f) quantification of cell number per colonies (d), percentage of TBXT-positive cell per colony (e), and the distribution of pERK intensities for different doses of MEK inhibitor (f). N>6 colonies per condition. MEKi = PD0325901. Scalebar = 50 micron.

      (2) Regarding the sparse cells in 1G, is there a reduction in cell number only with FGFRi and not MEKi? Is this reproducible? Gattiglio et al (Development, 2023, PMID: 37530863) present data supporting a "community effect" in the FGF-induced mesoderm differentiation of mouse embryonic stem cells. Could a community effect be at play in this human system (especially given the images in the bottom row of 1G)? If the authors don't address this experimentally they should at least address the ideas in Gattoglio et al.

      Indeed, FGFRi reproducibly affects cell number more than MEKi, in line with the fact that pathways downstream of FGF other than MAPK/ERK (e.g. PI3K) play important roles in cell survival and growth. We think the lack of differentiation in MEKi and FGFRi in Fig.1g cannot be attributed to a loss of cells combined with a community effect. This is because without FGFRi or MEKi cells also differentiate to primitive streak at much lower densities than those shown, consistent with the data we show above in response to (1), which argue against a primarily indirect effect of FGF on PS-LC differentiation through cell density. In the context of directed differentiation (rather than 2D gastruloids), we will show this in a controlled manner by repeating the experiment in Fig.1g while adjusting cell seeding densities to obtain similar final cell densities in all three conditions. We will also include Gattoglio et al. in our revised discussion.

      (3) Do the FGF4 and FGF17 LOF experiments in Figure 5 affect cell numbers like FGFRi in Figure 1?

      It seems the effect on cell number is small but we will analyze this carefully and include it in the revised manuscript. A small effect would be consistent with our unpublished data below showing a near uniform proliferation rate. This in turn suggests that low levels of pERK in the center are sufficient to maintain proliferation there while the much higher pERK levels in the PS-LC ring (that we think depend on FGF4 and FGF17) do not signifcantly increase the proliferation rate (see Fig.1 in the manuscript for the pERK pattern). Thus, loss of high pERK in PS-LC ring while maintaining low pERK throughout would not be expected to have a major impact on cell number but would impact differentiation. In contrast, loss of all FGF signaling through FGFRi does dramatically affect cell number. This is again consistent with the data provided in response to (1) showing that ERK levels can be reduced to a point where PS-LC differentiation is lost without significantly affecting cell number. We will include the data below in the revised manuscript.

      Author response image 2.

      Why examine PS-LC induction only in FGF17 heterozygous cells and not homozygous FGF17 nulls?

      We were unable to obtain homozygous FGF17 nulls, it is not clear if there is a reason for this. We will try again and otherwise attempt to corroborate our findings with further knockdown data.

      (4) The idea that FGF8 plays a dominant role during gastrulation of other species but not humans is so intriguing it warrants deeper testing. The authors dismiss FGF8 because its mRNA "...levels always remained low." (line 363) as well as the data published in Zhai et al (PMID: 36517595) and Tyser et al (PMID: 34789876). But there are cases in mouse development where a gene was expressed at levels so low, that it might be dismissed, and yet LOF experiments revealed it played a role or even was required in a developmental process. The authors should consider FGF8 inhibition or inactivation to explore its potential role, despite its low levels of expression.

      We agree with the reviewer that FGF8 is worth investigating further and we will now pursue this.

      (5) Redundancy is a common feature in FGF genetics. What is the effect of inhibiting FGF4 in FGF17 LOF cells?

      We will attempt to do the experiment the reviewer suggests.

      (6) I suggest stating that the authors take more caution in describing FGF gradients. For example, in one Results heading they write "Endogenous FGF4 and FGF17 gradients underly the ERK activity pattern.", implying an FGF protein gradient. However, they only present data for FGF mRNA , not protein. This issue would be clarified if they used proper nomenclature for gene, mRNA (italics), and protein (no italics) throughout the paper.

      We will edit the paper to more clearly distinguish protein and mRNA.

      Reviewer #2 (Public review):

      Summary:

      The role of FGFs in embryonic development and stem cell differentiation has remained unclear due to its complexity. In this study, the authors utilized a 2D human stem cell-based gastrulation model to investigate the functions of FGFs. They discovered that FGF-dependent ERK activity is closely linked to the emergence of primitive streak cells. Importantly, this 2D model effectively illustrates the spatial distribution of key signaling effectors and receptors by correlating these markers with cell fate markers, such as T and ISL1. Through inhibition and loss-of-function studies, they further corroborated the needs of FGF ligands. Their data shows that FGFR1 is the primary receptor, and FGF2/4/17 are the key ligands for primitive streak development, which aligns with observations in primate embryos. Additional experiments revealed that the reduction of FGF4 and FGF17 decreases ERK activity.

      Strengths:

      This study provides comprehensive data and improves our understanding of the role of FGF signaling in primate primitive streak formation. The authors provide new insights related to the spatial localization of the key components of FGF signaling and attempt to reveal the temporal dynamics of the signal propagation and cell fate decision, which has been challenging.

      Weaknesses:

      Given the solid data, the work only partially clarifies the complex picture of FGF signaling, so details remain somewhat elusive. The findings lack a strong punchline, which may limit their broader impact.

      We thank this reviewer for their valuable feedback and the compliment on the solidity of our data. The punchline of our work is that FGF4- and FGF17-dependent ERK signaling plays a key role in human PS-LC differentiation, and that these are different FGFs than those thought to drive mouse gastrulation. A second key point is that like BMP and TGFβ signaling, FGF signaling is restricted to the basolateral sides of pluripotent stem cell colonies due to polarized receptor expression, which is crucial for understanding the response to exogenous ligands added to the cell medium. Indeed, many facets of FGF signaling remain to investigated in the future, such as how FGF regulates and is regulated by other signals, which we will dedicate a different manuscript to.

      Reviewer #3 (Public review):

      Jo and colleagues set out to investigate the origins and functions of localized FGF/ERK signaling for the differentiation and spatial patterning of primitive streak fates of human embryonic stem cells in a well-established micropattern system. They demonstrate that endogenous FGF signaling is required for ERK activation in a ring-domain in the micropatterns, and that this localized signaling is directly required for differentiation and spatial patterning of specific cell types. Through high-resolution microscopy and transwell assays, they show that cells receive FGF signals through basally localized receptors. Finally, the authors find that there is a requirement for exogenous FGF2 to initiate primitive streak-like differentiation, but endogenous FGFs, especially FGF4 and FGF17, fully take over at later stages.

      Even though some of the authors' findings - such as the localized expression of FGF ligands during gastrulation and the importance of FGF/ERK signaling for cell differentiation in the primitive streak - have been reported in model organisms before, this is one of the first studies to investigate the role of FGF signaling during primitive streak-like differentiation of human cells. In doing so, the paper reports a number of interesting and valuable observations, namely the basal localization of FGF receptors which mirrors that of BMP and Nodal receptors, as well as the existence of a positive feedback loop centered on FGF signaling that drives primitive-streak differentiation. The authors also perform a comparison of the role of different FGFs across species and try to assign specific functions to individual FGFs. In the absence of clean genetic loss-of-function cell lines, this part of the work remains less strong.

      We thank the reviewer for emphasizing the value of our findings in a human model for gastrulation. We agree more loss-of-function experiments would provide further insight into the role of different FGFs, and we plan to provide additional data along these lines in the revised manuscript.

    1. Author response:

      We thank the reviewers for their thoughtful comments and constructive suggestions. We describe how we will address each point below and are grateful for the guidance on areas where our work could be clarified or expanded. In particular, we note the following:

      Selection scan summary statistics: In our revised manuscript, we will include summary statistics from the selection scans. We believe this addition will enhance transparency and provide additional context for readers.

      Reporting of outliers: As highlighted by the editor, the reviewers expressed differing views on the most appropriate way to report outliers. To provide a comprehensive and balanced presentation, we will report both the empirical selection statistics and the corresponding converted p-values. This dual approach will allow readers to fully interpret the results under both perspectives.

      Methodological considerations: We have carefully considered the reviewers' methodological suggestions and will incorporate them into our revisions where possible. These changes strengthen the rigor and clarity of the analyses.

      Public Reviews:

      Reviewer #1 (Public review):

      Summary:

      The paper reports an analysis of whole-genome sequence data from 40 Faroese. The authors investigate aspects of demographic history and natural selection in this population. The key findings are that the Faroese (as expected) have a small population size and are broadly of Northwest European ancestry. Accordingly, selection signatures are largely shared with other Northwest European populations, although the authors identify signals that may be specific to the Faroes. Finally, they identify a few predicted deleterious coding variants that may be enriched in the Faroes.

      Strengths:

      The data are appropriately quality-controlled and appear to be of high quality. Some aspects of the Faroese population history are characterized, in particular, by the relatively (compared to other European populations) high proportion of long runs of homozygosity, which may be relevant for disease mapping of recessive variants. The selection analysis is presented reasonably, although as the authors point out, many aspects, for example differences in iHS, can reflect differences in demographic history or population-specific drift and thus can't reliably be interpreted in terms of differences in the strength of selection.

      Weaknesses:

      The main limitations of the paper are as follows:

      (1) The data are not available. I appreciate that (even de-identified) genotype data cannot be shared; however, that does substantially reduce the value of the paper. Minimally, I think the authors should share summary statistics for the selection scans, in line with the standard of the field.

      We agree with the reviewer that sharing the selection scan results is important, so in the next revision of this manuscript we will make the selection scan summary statistics publicly available, and clearly lay out the guidelines and research questions for which the data can be accessed.

      (2) The insight into the population history of the Faroes is limited, relative to what is already known (i.e., they were settled around 1200 years ago, by people with a mixture of Scandinavian and British ancestry, have a small effective population size, and any admixture since then comes from substantially similar populations). It's obvious, for example, that the Faroese population has a smaller bottleneck than, say, GBR.

      More sophisticated analyses (for example, ARG-based methods, or IBD or rare variant sharing) would be able to reveal more detailed and fine-scale information about the history of the populations that is not already known. PCA, ADMIXTURE, and HaplotNet analysis are broad summaries, but the interesting questions here would be more specific to the Faroes, for example, what are the proportions of Scandinavian vs Celtic ancestry? What is the date and extent of sex bias (as suggested by the uniparental data) in this admixture? I think that it is a bit of a missed opportunity not to address these questions.

      We clarify that we did quantify the proportions of various ancestry components as estimated by HaploNet in main text Figure 5 and supplemental figures S5 and S6. In our revisions, we will include the average global ancestry of the various components in the Main Text so that this result is more clear.

      We agree that more fine-scale demographic analyses would be informative. We have begun working on an estimation of the admixture date, for example, but have encountered problems with using different standard date estimation software, which give very inconsistent and unstable results. We suspect this might be due to the strong bottleneck experienced in the history of the Faroe Islands breaking one or more of the assumptions of these methods. We will continue working on this problem in coming months, possibly using simulations to assess where the problem might be. We recognize that our relatively small sample size places limits on the fine-scale demographic analyses that can be performed. We are addressing this in ongoing work by generating a larger cohort, which we hope will enable more detailed inference in the future.

      (3) I don't really understand the rationale for looking at HLA-B allele frequencies. The authors write that "ankylosing spondylitis (AS) may be at a higher prevalence in the Faroe Islands (unpublished data), however, this has not been confirmed by follow-up epidemiological studies". So there's no evidence (certainly no published evidence) that AS is more prevalent, and hence nothing to explain with the HLA allele frequencies?

      We agree that no published studies have confirmed a higher prevalence of ankylosing spondylitis (AS) in the Faroe Islands. Our recruitment data suggest that AS might be more common than in other European populations, but we understand that this is only based on limited, unpublished observations and what we are hearing from the community. We emphasized in our original manuscript that this is based on observational evidence from the FarGen project. However, as this reviewer pointed out, we can be more clear that this prevalence has not been formally studied.

      In our next revision we will clarify in the text that our recruitment data suggest a higher prevalence of AS may be possible, but more formal epidemiological studies are needed to confirm this observation. The reason we study HLA-B allele frequencies is to see if the genetic background of the Faroese population could help explain this possible difference, since HLA-B27 is already known to play a strong role in AS.

      Reviewer #2 (Public review):

      In this paper, Hamid et al present 40 genomes from the Faroe Islands. They use these data (a pilot study for an anticipated larger-scale sequencing effort) to discuss the population genetic diversity and history of the sample, and the Faroes population. I think this is an overall solid paper; it is overall well-polished and well-written. It is somewhat descriptive (as might be expected for an explorative pilot study), but does make good use of the data.

      The data processing and annotation follows a state-of-the-art protocol, and at least I could not find any evidence in the results that would pinpoint towards bioinformatic issues having substantially biased some of the results, and at least preliminary results lead to the identification of some candidate disease alleles, showing that small, isolated cohorts can be an efficient way to find populations with locally common, but globally rare disease alleles.

      I also enjoyed the population structure analysis in the context of ancient samples, which gives some context to the genetic ancestry of Faroese, although it would have been nice if that could have been quantified, and it is unfortunate that the sampling scheme effectively precludes within-Faroes analyses.

      We note that although the ancestry proportions are not specified in the main text, we did quantify ancestry proportions in the modern Faroese individuals and other ancient samples, and we visualized these proportions in Figure 5 and Supplementary Figures S5 and S6. As stated in our response to Reviewer #1, in our revisions, we will more clearly state the average global ancestry of the various components in the Main Text.

      I am unfortunately quite critical of the selection analysis, both on a statistical level and, more importantly, I do not believe it measures what the authors think it does.

      Major comments:

      (1) Admixture timing/genomic scaling/localization:

      As the authors lay out, the Faroes were likely colonized in the last 1,000-1,500 years, i.e., 40-60 generations ago. That means most genomic processes that have happened on the Faroese should have signatures that are on the order of ~1-2cM, whereas more local patterns likely indicate genetic history predating the colonization of the islands. Yet, the paper seems to be oblivious to this (to me) fascinating and somewhat unique premise. Maybe this thought is wrong, but I think the authors miss a chance here to explain why the reader should care beyond the fact that the small populations might have high-frequency risk alleles and the Faroes are intrinsically interesting, but more importantly, it also makes me think it leads to some misinterpretations in the selection analysis

      See response to point #3

      (2) ROH:

      Would the sampling scheme impact ROH? How would it deal with individuals with known parental coancestry? As an example of what I mean by my previous comment, 1MB is short enough in that I would expect most/many 1MB ROH-tracts to come from pedigree loops predating the colonization of the Faroes. (i.e, I am actually quite surprised that there isn't much more long ROH, which makes me wonder if that would be impacted by the sampling scheme).

      The sampling scheme was designed to choose 40 Faroese individuals that were representative of the different regions and were minimally related. There were no pairs of third-degree relatives or closer (pi-hat > 0.125) in either the Faroese cohort or the reference populations. It is possible that this sampling scheme would reduce the amount of longer ROHs in the population, but we should still be able to see overall patterns of ROH reflective of bottlenecks in the past tens of generations. Additionally, based on this reviewer's earlier comment, 1 Mb ROHs would still be relevant to demographic events in the last 40-60 generations given that on average 1 cM corresponds to 1 Mb in humans, though we recognize that is not an exact conversion.

      That said, the “sum total amount of the genome contained in long ROH” as we described in the manuscript includes all ROHs greater than 1Mb. Although we group all ROHs longer than 1Mb into one category in the current manuscript, we can look more specifically at the distribution of the longer ROH in future revisions and add discussion into what this might tell us about the timing of bottlenecks. 

      For now, we share a plot of the distribution in ROH lengths across all individuals for each cohort. As this plot shows, there certainly are ROHs longer than 1Mb in the Faroese cohort, and on average there is a higher proportion of long ROH particularly in the 5-15 Mb range in the Faroese cohort relative to the other cohorts.

      Author response image 1.

      (3) Selection scan:

      We are talking about a bottlenecked population that is recently admixed (Faroese), compared to a population (GBR) putatively more closely related to one of its sources. My guess would be that selection in such a scenario would be possibly very hard to detect, and even then, selection signals might not differentiate selection in Faroese vs. GBR, but rather selection/allele frequency differences between different source populations. I think it would be good to spell out why XP-EHH/iHS measures selection at the correct time scale, and how/if these statistics are expected to behave differently in an admixed population.

      The reviewer brings up good points about the utility of classical selection statistics in populations that are admixed or bottlenecked, and whether the timescale at which these statistics detect selection is relevant for understanding the selective history of the Faroese population. We break down these concerns separately.

      (1) Bottlenecks: Recent bottlenecks result in higher LD within a population. However, demographic events such as bottlenecks affect global genomic patterns while positive selection is expected to affect local genomic patterns. For this reason, iHS and XP-EHH statistics are standardized against the genome-wide background, to account for population-specific demographic history.

      (2) Admixture: The term “admixture” has different interpretations depending on the line of inquiry and the populations being studied. Across various time and geographic scales, all human populations are admixed to some degree, as gene flow between groups is a common fixture throughout our history. For example,

      even the modern British population has “admixed” ancestry from North / West European sources as well, dating to at least as recently as the Medieval & Viking periods (Gretzinger et al. 2022, Leslie et al. 2015), yet we do not commonly consider it an “admixed” population, and we are not typically concerned about applying haplotype-based statistics in this population. This is due to the low divergence between the source populations. In the case of the Faroe Islands, we believe admixture likely occurred on a similar timescale. We see low variance in ancestry proportions estimated by HaploNet, both from the historical Faroese individuals (250BP) and the modern samples. This indicates admixture predating the settlement of the Faroe Islands, where recombination has had time to break up long ancestry tracts and the global ancestry proportions have reached an equilibrium. That is, these ancestry patterns suggest that the modern Faroese are most likely descended from already admixed founders. We mention this as a likely possibility in the main text: “This could have occurred either via a mixture of the original “West Europe” ancestry with individuals of predominantly “North Europe” ancestry, or a by replacement with individuals that were already of mixed ancestry at the time of arrival in the islands (the latter are not uncommon in Viking Age mainland Europe).” And, as with the case of the British population, the closely-related ancestral sources for the Faroese founders were likely not so diverged as to have differences in allele frequencies and long-range haplotypes that would disrupt signals of selection from iHS or XP-EHH.

      (3) Time scale: It is certainly possible, and in fact likely, that iHS measures selection older than the settlement of the Faroe Islands. In our manuscript, we calculated iHS in both the Faroese and the closely related British cohort, and we highlight in the main Main Text that the top signals, with the exception of LCT, are shared between the two cohorts, indicative of selection that began prior to the population split. iHS is a commonly calculated statistic, and it is often calculated in a single population without comparing to others, so we feel it is important to show our result demonstrating these shared selection signals. In future revisions, we will emphasize in the main text that we are not claiming to have identified selection that occurred in the Faroese population post-settlement with the iHS statistic. As far as XP-EHH, it is a statistic designed to identify differentiated variants that are fixed or approaching fixation in one population but not others. The time-scale of selection that XP-EHH can detect would therefore be dependent on the populations used for comparison. As XP-EHH has the best power to identify alleles that are fixed or approaching fixation in one population but not others, it is less likely to detect older selection events / incomplete sweeps from the source populations.

      In our next revision, we will more clearly state limitations of these statistics under various population histories, and clarify the time-scale at which we are detecting selection for iHS vs XP-EHH.

      (4) Similarly, for the discussion of LCT, I am not convinced that the haplotypes depicted here are on the right scale to reflect processes happening on the Faroes. Given the admixture/population history, it at the very least should be discussed in the context of whether the 13910 allele frequency on the Faroes is at odds with what would be expected based on the admixture sources.

      We agree that more investigation into the LCT allele frequency in the other ancient samples may provide some insight into the selection history, particularly in light of ancient admixture. Please note, we did look at the allele frequency of the LCT allele rs4988235 and stated in the main text that it was present at high frequencies in the historical (250BP) Faroese samples. The frequency of this allele in the imputed historical Faroese samples is 82% while the allele is present at ~74% frequency in modern samples. We did not report the exact percentage in the main text because the sample size of the historical samples (11 individuals) is small and coverage of ancient samples is low, leading to potential errors in imputation. However, we can try to calculate the LCT allele frequency in other ancient samples, and assuming that we have good proxies for the sources at the time of admixture, we may calculate the expected allele frequency in the admixed ancestors of the Faroese founders in the next revision.

      (5) I am lacking information to evaluate the procedure for turning the outliers into p-values. Both iHS and XP-EHH are ratio statistics, meaning they might be heavy-tailed if one is not careful, and the central limit theorem may not apply. It would be much easier (and probably sufficient for the points being made here) to reframe this analysis in terms of empirical outliers.

      Given that there are disagreements on the best approach to reporting selection scan results from the reviewers, in our revision, we can additionally supply both the standardized iHS / XP-EHH values in the supplementary information as well as these values transformed to p-values. As the p-values are derived from the empirical distribution, the “significant” p-values are also empirical outliers from the empirical distribution, so the conclusions of the manuscript do not change. We found that the p-value approach and controlling for FDR is more conservative, with fewer signals reaching “significance” than are considered empirical outliers based on common approaches such as IQR or arbitrary percentile cutoffs.

      (6) Oldest individual predating gene flow: It seems impossible to make any statements based on a single individual. Why is it implausible that this person (or their parents), e.g., moved to the Faroes within their lifetime and died there?

      We agree with the reviewer that this is a plausible explanation, and in future revisions we will update the main text to acknowledge this possibility.

    1. Author response:

      Reviewer #1 (Public review):

      Wang et al., recorded concurrent EEG-fMRI in 107 participants during nocturnal NREM sleep to investigate brain activity and connectivity related to slow oscillations (SO), sleep spindles, and in particular their co-occurrence. The authors found SO-spindle coupling to be correlated with increased thalamic and hippocampal activity, and with increased functional connectivity from the hippocampus to the thalamus and from the thalamus to the neocortex, especially the medial prefrontal cortex (mPFC). They concluded the brain-wide activation pattern to resemble episodic memory processing, but to be dissociated from task-related processing and suggest that the thalamus plays a crucial role in coordinating the hippocampal-cortical dialogue during sleep.

      The paper offers an impressively large and highly valuable dataset that provides the opportunity for gaining important new insights into the network substrate involved in SOs, spindles, and their coupling. However, the paper does unfortunately not exploit the full potential of this dataset with the analyses currently provided, and the interpretation of the results is often not backed up by the results presented. I have the following specific comments.

      Thank you for your thoughtful and constructive feedback. We greatly appreciate your recognition of the strengths of our dataset and findings Below, we address your specific comments and provide responses to each point you raised to ensure our methods and results are as transparent and comprehensible as possible. We hope these revisions address your comments and further strengthen our manuscript. Thank you again for the constructive feedback.

      (1) The introduction is lacking sufficient review of the already existing literature on EEG-fMRI during sleep and the BOLD-correlates of slow oscillations and spindles in particular (Laufs et al., 2007; Schabus et al., 2007; Horovitz et al., 2008; Laufs, 2008; Czisch et al., 2009; Picchioni et al., 2010; Spoormaker et al., 2010; Caporro et al., 2011; Bergmann et al., 2012; Hale et al., 2016; Fogel et al., 2017; Moehlman et al., 2018; Ilhan-Bayrakci et al., 2022). The few studies mentioned are not discussed in terms of the methods used or insights gained.

      We acknowledge the need for a more comprehensive review of prior EEG-fMRI studies investigating BOLD correlates of slow oscillations and spindles. However, these articles are not all related to sleep SO or spindle. Articles (Hale et al., 2016; Horovitz et al., 2008; Laufs, 2008; Laufs, Walker, & Lund, 2007; Spoormaker et al., 2010) mainly focus on methodology for EEG-fMRI, sleep stages, or brain networks, which are not the focus of our study. Thank you again for your attention to the comprehensiveness of our literature review, and we will expand the introduction to include a more detailed discussion of the existing literature, ensuring that the contributions of previous EEG-fMRI sleep studies are adequately acknowledged.

      Introduction, Page 4 Lines 62-76

      “Investigating these sleep-related neural processes in humans is challenging because it requires tracking transient sleep rhythms while simultaneously assessing their widespread brain activation. Recent advances in simultaneous EEG-fMRI techniques provide a unique opportunity to explore these processes. EEG allows for precise event-based detection of neural signal, while fMRI provides insight into the broader spatial patterns of brain activation and functional connectivity (Horovitz et al., 2008; Huang et al., 2024; Laufs, 2008; Laufs, Walker, & Lund, 2007; Schabus et al., 2007; Spoormaker et al., 2010). Previous EEG-fMRI studies on sleep have focused on classifying sleep stages or examining the neural correlates of specific waves (Bergmann et al., 2012; Caporro et al., 2012; Czisch et al., 2009; Fogel et al., 2017; Hale et al., 2016; Ilhan-Bayrakcı et al., 2022; Moehlman et al., 2019; Picchioni et al., 2011). These studies have generally reported that slow oscillations are associated with widespread cortical and subcortical BOLD changes, whereas spindles elicit activation in the thalamus, as well as in several cortical and paralimbic regions. Although these findings provide valuable insights into the BOLD correlates of sleep rhythms, they often do not employ sophisticated temporal modeling (Huang et al., 2024), to capture the dynamic interactions between different oscillatory events, e.g., the coupling between SOs and spindles.”

      (2) The paper falls short in discussing the specific insights gained into the neurobiological substrate of the investigated slow oscillations, spindles, and their interactions. The validity of the inverse inference approach ("Open ended cognitive state decoding"), assuming certain cognitive functions to be related to these oscillations because of the brain regions/networks activated in temporal association with these events, is debatable at best. It is also unclear why eventually only episodic memory processing-like brain-wide activation is discussed further, despite the activity of 16 of 50 feature terms from the NeuroSynth v3 dataset were significant (episodic memory, declarative memory, working memory, task representation, language, learning, faces, visuospatial processing, category recognition, cognitive control, reading, cued attention, inhibition, and action).

      Thank you for pointing this out, particularly regarding the use of inverse inference approaches such as “open-ended cognitive state decoding.” Given the concerns about the indirectness of this approach, we decided to remove its related content and results from Figure 3 in the main text and include it in Supplementary Figure 7. We will refocus the main text on direct neurobiological insights gained from our EEG-fMRI analyses, particularly emphasizing the hippocampal-thalamocortical network dynamics underlying SO-spindle coupling, and we will acknowledge the exploratory nature of these findings and highlight their limitations.

      Discussion, Page 17-18 Lines 323-332

      “To explore functional relevance, we employed an open-ended cognitive state decoding approach using meta-analytic data (NeuroSynth: Yarkoni et al. (2011)). Although this method usefully generates hypotheses about potential cognitive processes, particularly in the absence of a pre- and post-sleep memory task, it is inherently indirect. Many cognitive terms showed significant associations (16 of 50), such as “episodic memory,” “declarative memory,” and “working memory.” We focused on episodic/declarative memory given the known link with hippocampal reactivation (Diekelmann & Born, 2010; Staresina et al., 2015; Staresina et al., 2023). Nonetheless, these inferences regarding memory reactivation should be interpreted cautiously without direct behavioral measures. Future research incorporating explicit tasks before and after sleep would more rigorously validate these potential functional claims.”

      (3) Hippocampal activation during SO-spindles is stated as a main hypothesis of the paper - for good reasons - however, other regions (e.g., several cortical as well as thalamic) would be equally expected given the known origin of both oscillations and the existing sleep-EEG-fMRI literature. However, this focus on the hippocampus contrasts with the focus on investigating the key role of the thalamus instead in the Results section.

      We appreciate your insight regarding the relative emphasis on hippocampal and thalamic activation in our study. We recognize that the manuscript may currently present an inconsistency between our initial hypothesis and the main focus of the results. To address this concern, we will ensure that our Introduction and Discussion section explicitly discusses both regions, highlighting the complementary roles of the hippocampus (memory processing and reactivation) and the thalamus (spindle generation and cortico-hippocampal coordination) in SO-spindle dynamics.

      Introduction, Page 5 Lines 87-103

      “To address this gap, our study investigates brain-wide activation and functional connectivity patterns associated with SO-spindle coupling, and employs a cognitive state decoding approach (Margulies et al., 2016; Yarkoni et al., 2011)—albeit indirectly—to infer potential cognitive functions. In the current study, we used simultaneous EEG-fMRI recordings during nocturnal naps (detailed sleep staging results are provided in the Methods and Table S1) in 107 participants. Although directly detecting hippocampal ripples using scalp EEG or fMRI is challenging, we expected that hippocampal activation in fMRI would coincide with SO-spindle coupling detected by EEG, given that SOs, spindles, and ripples frequently co-occur during NREM sleep. We also anticipated a critical role of the thalamus, particularly thalamic spindles, in coordinating hippocampal-cortical communication.

      We found significant coupling between SOs and spindles during NREM sleep (N2/3), with spindle peaks occurring slightly before the SO peak. This coupling was associated with increased activation in both the thalamus and hippocampus, with functional connectivity patterns suggesting thalamic coordination of hippocampal-cortical communication. These findings highlight the key role of the thalamus in coordinating hippocampal-cortical interactions during human sleep and provide new insights into the neural mechanisms underlying sleep-dependent brain communication. A deeper understanding of these mechanisms may contribute to future neuromodulation approaches aimed at enhancing sleep-dependent cognitive function and treating sleep-related disorders.”

      Discussion, Page 16-17 Lines 292-307

      “When modeling the timing of these sleep rhythms in the fMRI, we observed hippocampal activation selectively during SO-spindle events. This suggests the possibility of triple coupling (SOs–spindles–ripples), even though our scalp EEG was not sufficiently sensitive to detect hippocampal ripples—key markers of memory replay (Buzsáki, 2015). Recent iEEG evidence indicates that ripples often co-occur with both spindles (Ngo, Fell, & Staresina, 2020) and SOs (Staresina et al., 2015; Staresina et al., 2023). Therefore, the hippocampal involvement during SO-spindle events in our study may reflect memory replay from the hippocampus, propagated via thalamic spindles to distributed cortical regions.

      The thalamus, known to generate spindles (Halassa et al., 2011), plays a key role in producing and coordinating sleep rhythms (Coulon, Budde, & Pape, 2012; Crunelli et al., 2018), while the hippocampus is found essential for memory consolidation (Buzsáki, 2015; Diba & Buzsá ki, 2007; Singh, Norman, & Schapiro, 2022). The increased hippocampal and thalamic activity, along with strengthened connectivity between these regions and the mPFC during SO-spindle events, underscores a hippocampal-thalamic-neocortical information flow. This aligns with recent findings suggesting the thalamus orchestrates neocortical oscillations during sleep (Schreiner et al., 2022). The thalamus and hippocampus thus appear central to memory consolidation during sleep, guiding information transfer to the neocortex, e.g., mPFC.”

      (4) The study included an impressive number of 107 subjects. It is surprising though that only 31 subjects had to be excluded under these difficult recording conditions, especially since no adaptation night was performed. Since only subjects were excluded who slept less than 10 min (or had excessive head movements) there are likely several datasets included with comparably short durations and only a small number of SOs and spindles and even less combined SO-spindle events. A comprehensive table should be provided (supplement) including for each subject (included and excluded) the duration of included NREM sleep, number of SOs, spindles, and SO+spindle events. Also, some descriptive statistics (mean/SD/range) would be helpful.

      We appreciate your recognition of our sample size and the challenges associated with simultaneous EEG-fMRI sleep recordings. We acknowledge the importance of transparently reporting individual subject data, particularly regarding sleep duration and the number of detected SOs, spindles, and SO-spindle events. To address this, we will provide comprehensive tables in the supplementary materials, contains descriptive information about sleep-related characteristics (Table S1), as well as detailed information about sleep waves at each sleep stage for all 107 subjects(Table S2-S4), listing for each subject:(1)Different sleep stage duration; (2)Number of detected SOs; (3)Number of detected spindles; (4)Number of detected SO-spindle coupling events; (5)Density of detected SOs; (6)Density of detected spindles; (7)Density of detected SO-spindle coupling events.

      However, most of the excluded participants were unable to fall asleep or had too short a sleep duration, so they basically had no NREM sleep period, so it was impossible to count the NREM sleep duration, SO, spindle, and coupling numbers.

      Supplementary Materials, Page 42-54, Table S1-S4

      (Consider of the length, we do not list all the tables here. Please refer to the revised manuscript.)

      (5) Was the 20-channel head coil dedicated for EEG-fMRI measurements? How were the electrode cables guided through/out of the head coil? Usually, the 64-channel head coil is used for EEG-fMRI measurements in a Siemens PRISMA 3T scanner, which has a cable duct at the back that allows to guide the cables straight out of the head coil (to minimize MR-related artifacts). The choice for the 20-channel head coil should be motivated. Photos of the recording setup would also be helpful.

      Thank you for your comment regarding our choice of the 20-channel head coil for EEG-fMRI measurements. We acknowledge that the 64-channel head coil is commonly used in Siemens PRISMA 3T scanners; however, the 20-channel coil was selected due to specific practical and technical considerations in our study. In particular, the 20-channel head coil was compatible with our EEG system and ensured sufficient signal-to-noise ratio (SNR) for both EEG and fMRI acquisition. The EEG electrode cables were guided through the lateral and posterior openings of the head coil, secured with foam padding to reduce motion and minimize MR-related artifacts. Moreover, given the extended nature of nocturnal sleep recordings, the 20-channel coil allowed us to maintain participant comfort while still achieving high-quality simultaneous EEG-fMRI data.

      We have made this clearer in the revised manuscript.

      Methods, Page 20 Lines 385-392

      “All MRI data were acquired using a 20-channel head coil on a research-dedicated 3-Tesla Siemens Magnetom Prisma MRI scanner. Earplugs and cushions were provided for noise protection and head motion restriction. We chose the 20-channel head coil because it was compatible with our EEG system and ensured sufficient signal-to-noise ratio (SNR) for both EEG and fMRI acquisition. The EEG electrode cables were guided through the lateral and posterior openings of the head coil, secured with foam padding to reduce motion and minimize MR-related artifacts. Moreover, given the extended nature of nocturnal sleep recordings, the 20-channel coil helped maintain participant comfort while still achieving high-quality simultaneous EEG-fMRI data.”

      (6) Was the EEG sampling synchronized to the MR scanner (gradient system) clock (the 10 MHz signal; not referring to the volume TTL triggers here)? This is a requirement for stable gradient artifact shape over time and thus accurate gradient noise removal.

      Thank you for raising this important point. We confirm that the EEG sampling was synchronized to the MR scanner’s 10 MHz gradient system clock, ensuring a stable gradient artifact shape over time and enabling accurate artifact removal. This synchronization was achieved using the standard clock synchronization interface of the EEG amplifier, minimizing timing jitter and drift. As a result, the gradient artifact waveform remained stable across volumes, allowing for more effective artifact correction during preprocessing. We appreciate your attention to this critical aspect of EEG-fMRI data acquisition.

      We have made this clearer in the revised manuscript.

      Methods, Page 19-20 Lines 371-383

      “EEG was recorded simultaneously with fMRI data using an MR-compatible EEG amplifier system (BrainAmps MR-Plus, Brain Products, Germany), along with a specialized electrode cap. The recording was done using 64 channels in the international 10/20 system, with the reference channel positioned at FCz. In order to adhere to polysomnography (PSG) recording standards, six electrodes were removed from the EEG cap: one for electrocardiogram (ECG) recording, two for electrooculogram (EOG) recording, and three for electromyogram (EMG) recording. EEG data was recorded at a sample rate of 5000 Hz, the resistance of the reference and ground channels was kept below 10 kΩ, and the resistance of the other channels was kept below 20 kΩ. To synchronize the EEG and fMRI recordings, the BrainVision recording software (BrainProducts, Germany) was utilized to capture triggers from the MRI scanner. The EEG sampling was synchronized to the MR scanner’s 10 MHz gradient system clock, ensuring a stable gradient artifact shape over time and enabling accurate artifact removal. This was achieved via the standard clock synchronization interface of the EEG amplifier, minimizing timing jitter and drift.”

      (7) The TR is quite long and the voxel size is quite large in comparison to state-of-the-art EPI sequences. What was the rationale behind choosing a sequence with relatively low temporal and spatial resolution?

      We acknowledge that our chosen TR and voxel size are relatively long and large compared to state-of-the-art EPI sequences. This decision was made to optimize the signal-to-noise ratio (SNR) and reduce susceptibility-related distortions, which are particularly critical in EEG-fMRI sleep studies where head motion and physiological noise can be substantial. A longer TR allowed us to sample whole-brain activity with sufficient coverage, while a larger voxel size helped enhance BOLD sensitivity and minimize partial volume effects in deep brain structures such as the thalamus and hippocampus, which are key regions of interest in our study. We appreciate your concern and hope this clarification provides sufficient rationale for our sequence parameters.

      We have made this clearer in the revised manuscript.

      Methods, Page 20-21 Lines 398-408

      “Then, the “sleep” session began after the participants were instructed to try and fall asleep. For the functional scans, whole-brain images were acquired using k-space and steady-state T2*-weighted gradient echo-planar imaging (EPI) sequence that is sensitive to the BOLD contrast. This measures local magnetic changes caused by changes in blood oxygenation that accompany neural activity (sequence specification: 33 slices in interleaved ascending order, TR = 2000 ms, TE = 30 ms, voxel size = 3.5 × 3.5 × 4.2 mm<sup>3</sup>, FA = 90°, matrix = 64 × 64, gap = 0.7 mm). A relatively long TR and larger voxel size were chosen to optimize SNR and reduce susceptibility-related distortions, which are critical in EEG-fMRI sleep studies where head motion and physiological noise can be substantial. The longer TR allowed whole-brain coverage with sufficient temporal resolution, while the larger voxel size helped enhance BOLD sensitivity and minimize partial volume effects in deep brain structures (e.g., the thalamus and hippocampus), which are key regions of interest in this study.”

      (8) The anatomically defined ROIs are quite large. It should be elaborated on how this might reduce sensitivity to sleep rhythm-specific activity within sub-regions, especially for the thalamus, which has distinct nuclei involved in sleep functions.

      We appreciate your insight regarding the use of anatomically defined ROIs and their potential limitations in detecting sleep rhythm-specific activity within sub-regions, particularly in the thalamus. Given the distinct functional roles of thalamic nuclei in sleep processes, we acknowledge that using a single, large thalamic ROI may reduce sensitivity to localized activity patterns. To address this, we will discuss this limitation in the revised manuscript, acknowledging that our approach prioritizes whole-structure effects but may not fully capture nucleus-specific contributions.

      Discussion, Page 18 Lines 333-341

      “Despite providing new insights, our study has several limitations. First, our scalp EEG did not directly capture hippocampal ripples, preventing us from conclusively demonstrating triple coupling. Second, the combination of EEG-fMRI and the lack of a memory task limit our ability to parse fine-grained BOLD responses at the DOWN- vs. UP-states of SOs and link observed activations to behavioral outcomes. Third, the use of large anatomical ROIs may mask subregional contributions of specific thalamic nuclei or hippocampal subfields. Finally, without a memory task, we cannot establish a direct behavioral link between sleep-rhythm-locked activation and memory consolidation. Future studies combining techniques such as ultra-high-field fMRI or iEEG with cognitive tasks may refine our understanding of subregional network dynamics and functional significance during sleep.”

      (9) The study reports SO & spindle amplitudes & densities, as well as SO+spindle coupling, to be larger during N2/3 sleep compared to N1 and REM sleep, which is trivial but can be seen as a sanity check of the data. However, the amount of SOs and spindles reported for N1 and REM sleep is concerning, as per definition there should be hardly any (if SOs or spindles occur in N1 it becomes by definition N2, and the interval between spindles has to be considerably large in REM to still be scored as such). Thus, on the one hand, the report of these comparisons takes too much space in the main manuscript as it is trivial, but on the other hand, it raises concerns about the validity of the scoring.

      We appreciate your concern regarding the reported presence of SOs and spindles in N1 and REM sleep and the potential implications. Our detection method for detecting SO, spindle, and coupling were originally designed only for N2&N3 sleep data based on the characteristics of the data itself, and this method is widely recognized and used in the sleep research (Hahn et al., 2020; Helfrich et al., 2019; Helfrich et al., 2018; Ngo, Fell, & Staresina, 2020; Schreiner et al., 2022; Schreiner et al., 2021; Staresina et al., 2015; Staresina et al., 2023). While, because the detection methods for SO and spindle are based on percentiles, this method will always detect a certain number of events when used for other stages (N1 and REM) sleep data, but the differences between these events and those detected in stage N23 remain unclear. We will acknowledge the reasons for these results in the Methods section and emphasize that they are used only for sanity checks.

      Methods, Page 25 Lines 515-524

      “We note that the above methods for detecting SOs, spindles, and their couplings were originally developed for N2 and N3 sleep data, based on the specific characteristics of these stages. These methods are widely recognized in sleep research (Hahn et al., 2020; Helfrich et al., 2019; Helfrich et al., 2018; Ngo, Fell, & Staresina, 2020; Schreiner et al., 2022; Schreiner et al., 2021; Staresina et al., 2015; Staresina et al., 2023). However, because this percentile-based detection approach will inherently identify a certain number of events if applied to other stages (e.g., N1 and REM), the nature of these events in those stages remains unclear compared to N2/N3. We nevertheless identified and reported the detailed descriptive statistics of these sleep rhythms in all sleep stages, under the same operational definitions, both for completeness and as a sanity check. Within the same subject, there should be more SOs, spindles, and their couplings in N2/N3 than in N1 or REM (see also Figure S2-S4, Table S1-S4).”

      (10) Why was electrode F3 used to quantify the occurrence of SOs and spindles? Why not a midline frontal electrode like Fz (or a number of frontal electrodes for SOs) and Cz (or a number of centroparietal electrodes) for spindles to be closer to their maximum topography?

      We appreciate your suggestion regarding electrode selection for SO and spindle quantification. Our choice of F3 was primarily based on previous studies (Massimini et al., 2004; Molle et al., 2011), where bilateral frontal electrodes are commonly used for detecting SOs and spindles. Additionally, we considered the impact of MRI-related noise and, after a comprehensive evaluation, determined that F3 provided an optimal balance between signal quality and artifact minimization. We also acknowledge that alternative electrode choices, such as Fz for SOs and Cz for spindles, could provide additional insights into their topographical distributions.

      (11) Functional connectivity (hippocampus -> thalamus -> cortex (mPFC)) is reported to be increased during SO-spindle coupling and interpreted as evidence for coordination of hippocampo-neocortical communication likely by thalamic spindles. However, functional connectivity was only analysed during coupled SO+spindle events, not during isolated SOs or isolated spindles. Without the direct comparison of the connectivity patterns between these three events, it remains unclear whether this is specific for coupled SO+spindle events or rather associated with one or both of the other isolated events. The PPIs need to be conducted for those isolated events as well and compared statistically to the coupled events.

      We appreciate your critical perspective on our functional connectivity analysis and the interpretation of hippocampus-thalamus-cortex (mPFC) interactions during SO-spindle coupling. We acknowledge that, in the current analysis, functional connectivity was only examined during coupled SO-spindle events, without direct comparison to isolated SOs or isolated spindles. To address this concern, we have conducted PPI analyses for all three ROIs(Hippocampus, Thalamus, mPFC) and all three event types (SO-spindle couplings, isolated SOs, and isolated spindles). Our results indicate that neither isolated SOs nor isolated Spindles yielded significant connectivity changes in all three ROIs, as all failed to survive multiple comparison corrections. This suggests that the observed connectivity increase is specific to SO-spindle coupling, rather than being independently driven by either SOs or spindles alone.

      Results, Page 14 Lines 248-255

      “Crucially, the interaction between FC and SO-spindle coupling revealed that only the functional connectivity of hippocampus -> thalamus (ROI analysis, t<sub>(106)</sub> = 1.86, p = 0.0328) and thalamus -> mPFC (ROI analysis, t<sub>(106)</sub> = 1.98, p = 0.0251) significantly increased during SO-spindle coupling, with no significant changes in all other pathways (Fig. 4e). We also conducted PPI analyses for the other two events (SOs and spindles), and neither yielded significant connectivity changes in the three ROIs, as all failed to survive whole-brain FWE correction at the cluster level (p < 0.05). Together, these findings suggest that the thalamus, likely via spindles, coordinates hippocampal-cortical communication selectively during SO-spindle coupling, but not isolated SOs or spindle events alone.”

      (12) The limited temporal resolution of fMRI does indeed not allow for easily distinguishing between fMRI activation patterns related to SO-up- vs. SO-down-states. For this, one could try to extract the amplitudes of SO-up- and SO-down-states separately for each SO event and model them as two separate parametric modulators (with the risk of collinearity as they are likely correlated).

      We appreciate your insightful comment regarding the challenge of distinguishing fMRI activation patterns related to SO-up vs. SO-down states due to the limited temporal resolution of fMRI. While our current analysis does not differentiate between these two phases, we acknowledge that separately modeling SO-up and SO-down states using parametric modulators could provide a more refined understanding of their distinct neural correlates. However, as you notes, this approach carries the risk of collinearity, and there is indeed a high correlation between the two amplitudes across all subjects in our results (r=0.98). Future studies could explore more on leveraging high-temporal-resolution techniques. While implementing this in the current study is beyond our scope, we will acknowledge this limitation in the Discussion section.

      Discussion, Page 17 Lines 308-322

      “An intriguing aspect of our findings is the reduced DMN activity during SOs when modeled at the SO trough (DOWN-state). This reduced DMN activity may reflect large-scale neural inhibition characteristic of the SO trough. The DMN is typically active during internally oriented cognition (e.g., self-referential processing or mind-wandering) and is suppressed during external stimuli processing (Yeshurun, Nguyen, & Hasson, 2021). It is unlikely, however, that this suppression of DMN during SO events is related to a shift from internal cognition to external responses given it is during deep sleep time. Instead, it could be driven by the inherent rhythmic pattern of SOs, which makes it difficult to separate UP- from DOWN-states (the two temporal regressors were highly correlated, and similar brain activation during SOs events was obtained if modelled at the SO peak instead, Fig. S5). Since the amplitude at the SO trough is consistently larger than that at the SO peak, the neural activation we detected may primarily capture the large-scale inhibition from DOWN-state. Interestingly, no such DMN reduction was found during SO-spindle coupling, implying that coupling may involve distinct neural dynamics that partially re-engage DMN-related processes, possibly reflecting memory-related reactivation. Future research using high-temporal-resolution techniques like iEEG could clarify these possibilities.

      Discussion, Page 18 Lines 333-341

      “Despite providing new insights, our study has several limitations. First, our scalp EEG did not directly capture hippocampal ripples, preventing us from conclusively demonstrating triple coupling. Second, the combination of EEG-fMRI and the lack of a memory task limit our ability to parse fine-grained BOLD responses at the DOWN- vs. UP-states of SOs and link observed activations to behavioral outcomes. Third, the use of large anatomical ROIs may mask subregional contributions of specific thalamic nuclei or hippocampal subfields. Finally, without a memory task, we cannot establish a direct behavioral link between sleep-rhythm-locked activation and memory consolidation. Future studies combining techniques such as ultra-high-field fMRI or iEEG with cognitive tasks may refine our understanding of subregional network dynamics and functional significance during sleep.

      (13) L327: "It is likely that our findings of diminished DMN activity reflect brain activity during the SO DOWN-state, as this state consistently shows higher amplitude compared to the UP-state within subjects, which is why we modelled the SO trough as its onset in the fMRI analysis." This conclusion is not justified as the fact that SO down-states are larger in amplitude does not mean their impact on the BOLD response is larger.

      We appreciate your concern regarding our interpretation of diminished DMN activity reflecting the SO down-state. We acknowledge that the current expression is somewhat misleading, and our interpretation of it is: it could be driven by the inherent rhythmic pattern of SOs, which makes it difficult to separate UP- from DOWN-states (the two temporal regressors were highly correlated, and similar brain activation during SOs events was obtained if modelled at the SO peak instead). Since the amplitude at the SO trough is consistently larger than that at the SO peak, the neural activation we detected may primarily capture the large-scale inhibition from DOWN-state. And we will make this clear in the Discussion section.

      Discussion, Page 17 Lines 308-322

      “An intriguing aspect of our findings is the reduced DMN activity during SOs when modeled at the SO trough (DOWN-state). This reduced DMN activity may reflect large-scale neural inhibition characteristic of the SO trough. The DMN is typically active during internally oriented cognition (e.g., self-referential processing or mind-wandering) and is suppressed during external stimuli processing (Yeshurun, Nguyen, & Hasson, 2021). It is unlikely, however, that this suppression of DMN during SO events is related to a shift from internal cognition to external responses given it is during deep sleep time. Instead, it could be driven by the inherent rhythmic pattern of SOs, which makes it difficult to separate UP- from DOWN-states (the two temporal regressors were highly correlated, and similar brain activation during SOs events was obtained if modelled at the SO peak instead, Fig. S5). Since the amplitude at the SO trough is consistently larger than that at the SO peak, the neural activation we detected may primarily capture the large-scale inhibition from DOWN-state. Interestingly, no such DMN reduction was found during SO-spindle coupling, implying that coupling may involve distinct neural dynamics that partially re-engage DMN-related processes, possibly reflecting memory-related reactivation. Future research using high-temporal-resolution techniques like iEEG could clarify these possibilities.

      (14) Line 77: "In the current study, while directly capturing hippocampal ripples with scalp EEG or fMRI is difficult, we expect to observe hippocampal activation in fMRI whenever SOs-spindles coupling is detected by EEG, if SOs- spindles-ripples triple coupling occurs during human NREM sleep". Not all SO-spindle events are associated with ripples (Staresina et al., 2015), but hippocampal activation may also be expected based on the occurrence of spindles alone (Bergmann et al., 2012).

      We appreciate your clarification regarding the relationship between SO-spindle coupling and hippocampal ripples. We acknowledge that not all SO-spindle events are necessarily accompanied by ripples (Staresina et al., 2015). However, based on previous research, we found that hippocampal ripples are significantly more likely to occur during SO-spindle coupling events. This suggests that while ripple occurrence is not guaranteed, SO-spindle coupling creates a favorable network state for ripple generation and potential hippocampal activation. To ensure accuracy, we will revise the manuscript to delete this misleading sentence in the Introduction section and acknowledge in the Discussion that our results cannot conclusively directly observe the triple coupling of SO, spindle, and hippocampal ripples.

      Discussion, Page 18 Lines 333-341

      “Despite providing new insights, our study has several limitations. First, our scalp EEG did not directly capture hippocampal ripples, preventing us from conclusively demonstrating triple coupling. Second, the combination of EEG-fMRI and the lack of a memory task limit our ability to parse fine-grained BOLD responses at the DOWN- vs. UP-states of SOs and link observed activations to behavioral outcomes. Third, the use of large anatomical ROIs may mask subregional contributions of specific thalamic nuclei or hippocampal subfields. Finally, without a memory task, we cannot establish a direct behavioral link between sleep-rhythm-locked activation and memory consolidation. Future studies combining techniques such as ultra-high-field fMRI or iEEG with cognitive tasks may refine our understanding of subregional network dynamics and functional significance during sleep.”

      Reviewer #2 (Public review):

      In this study, Wang and colleagues aimed to explore brain-wide activation patterns associated with NREM sleep oscillations, including slow oscillations (SOs), spindles, and SO-spindle coupling events. Their findings reveal that SO-spindle events corresponded with increased activation in both the thalamus and hippocampus. Additionally, they observed that SO-spindle coupling was linked to heightened functional connectivity from the hippocampus to the thalamus, and from the thalamus to the medial prefrontal cortex-three key regions involved in memory consolidation and episodic memory processes.

      This study's findings are timely and highly relevant to the field. The authors' extensive data collection, involving 107 participants sleeping in an fMRI while undergoing simultaneous EEG recording, deserves special recognition. If shared, this unique dataset could lead to further valuable insights. While the conclusions of the data seem overall well supported by the data, some aspects with regard to the detection of sleep oscillations need clarification.

      The authors report that coupled SO-spindle events were most frequent during NREM sleep (2.46 [plus minus] 0.06 events/min), but they also observed a surprisingly high occurrence of these events during N1 and REM sleep (2.23 [plus minus] 0.09 and 2.32 [plus minus] 0.09 events/min, respectively), where SO-spindle coupling would not typically be expected. Combined with the relatively modest SO amplitudes reported (~25 µV, whereas >75 µV would be expected when using mastoids as reference electrodes), this raises the possibility that the parameters used for event detection may not have been conservative enough - or that sleep staging was inaccurately performed. This issue could present a significant challenge, as the fMRI findings are largely dependent on the reliability of these detected events.

      Thank you very much for your thorough and encouraging review. We appreciate your recognition of the significance and relevance of our study and dataset, particularly in highlighting how simultaneous EEG-fMRI recordings can provide complementary insights into the temporal dynamics of neural oscillations and their associated spatial activation patterns during sleep. In the sections that follow, we address each of your comments in detail. We have revised the text and conducted additional analyses wherever possible to strengthen our argument, clarify our methodological choices. We believe these revisions improve the clarity and rigor of our work, and we thank you for helping us refine it.

      We appreciate your insightful comments regarding the detection of sleep oscillations. Our methods for detecting SOs, spindles, and their couplings were originally developed for N2 and N3 sleep data, based on the specific characteristics of these stages. These methods are widely recognized in sleep research (Hahn et al., 2020; Helfrich et al., 2019; Helfrich et al., 2018; Ngo, Fell, & Staresina, 2020; Schreiner et al., 2022; Schreiner et al., 2021; Staresina et al., 2015; Staresina et al., 2023). However, because this percentile-based detection approach will inherently identify a certain number of events if applied to other stages (e.g., N1 and REM), the nature of these events in those stages remains unclear compared to N2/N3. We nevertheless identified and reported the detailed descriptive statistics of these sleep rhythms in all sleep stages, under the same operational definitions, both for completeness and as a sanity check. Within the same subject, there should be more SOs, spindles, and their couplings in N2/N3 than in N1 or REM. We will acknowledge the reasons for these results in the Methods section and emphasize that they are used only for sanity checks.

      Regarding the reported SO amplitudes (~25 µV), during preprocessing, we applied the Signal Space Projection (SSP) method to more effectively remove MRI gradient artifacts and cardiac pulse noise. While this approach enhances data quality, it also reduces overall signal power, leading to systematically lower reported amplitudes. Despite this, our SO detection in NREM sleep (especially N2/N3) remain physiologically meaningful and are consistent with previous fMRI studies using similar artifact removal techniques. We appreciate your careful evaluation and valuable suggestions.

      In addition, we will provide comprehensive tables in the supplementary materials, contains descriptive information about sleep-related characteristics (Table S1), as well as detailed information about sleep waves at each sleep stage for all 107 subjects(Table S2-S4), listing for each subject:(1)Different sleep stage duration; (2)Number of detected SOs; (3)Number of detected spindles; (4)Number of detected SO-spindle coupling events; (2)Density of detected SOs; (3)Density of detected spindles; (4)Density of detected SO-spindle coupling events.

      Methods, Page 25 Lines 515-524

      “We note that the above methods for detecting SOs, spindles, and their couplings were originally developed for N2 and N3 sleep data, based on the specific characteristics of these stages. These methods are widely recognized in sleep research (Hahn et al., 2020; Helfrich et al., 2019; Helfrich et al., 2018; Ngo, Fell, & Staresina, 2020; Schreiner et al., 2022; Schreiner et al., 2021; Staresina et al., 2015; Staresina et al., 2023). However, because this percentile-based detection approach will inherently identify a certain number of events if applied to other stages (e.g., N1 and REM), the nature of these events in those stages remains unclear compared to N2/N3. We nevertheless identified and reported the detailed descriptive statistics of these sleep rhythms in all sleep stages, under the same operational definitions, both for completeness and as a sanity check. Within the same subject, there should be more SOs, spindles, and their couplings in N2/N3 than in N1 or REM (see also Figure S2-S4, Table S1-S4).”

      Supplementary Materials, Page 42-54, Table S1-S4

      (Consider of the length, we do not list all the tables here. Please refer to the revised manuscript.)

      Reviewer #3 (Public review):

      Summary:

      Wang et al., examined the brain activity patterns during sleep, especially when locked to those canonical sleep rhythms such as SO, spindle, and their coupling. Analyzing data from a large sample, the authors found significant coupling between spindles and SOs, particularly during the upstate of the SO. Moreover, the authors examined the patterns of whole-brain activity locked to these sleep rhythms. To understand the functional significance of these brain activities, the authors further conducted open-ended cognitive state decoding and found a variety of cognitive processing may be involved during SO-spindle coupling and during other sleep events. The authors next investigated the functional connectivity analyses and found enhanced connectivity between the hippocampus, the thalamus, and the medial PFC. These results reinforced the theoretical model of sleep-dependent memory consolidation, such that SO-spindle coupling is conducive to systems-level memory reactivation and consolidation.

      Strengths:

      There are obvious strengths in this work, including the large sample size, state-of-the-art neuroimaging and neural oscillation analyses, and the richness of results.

      Weaknesses:

      Despite these strengths and the insights gained, there are weaknesses in the design, the analyses, and inferences.

      Thank you for your detailed and thoughtful review of our manuscript. We are delighted that you recognize our advanced analysis methods and rich results of neuroimaging and neural oscillations as well as the large sample size data. In the following sections, we provide detailed responses to each of your comments. And we have revised the text and conducted additional analyses to strengthen our arguments and clarify our methodological choices. We believe these revisions enhance the clarity and rigor of our work, and we sincerely appreciate your thoughtful feedback in helping us refine the manuscript.

      (1) A repeating statement in the manuscript is that brain activity could indicate memory reactivation and thus consolidation. This is indeed a highly relevant question that could be informed by the current data/results. However, an inherent weakness of the design is that there is no memory task before and after sleep. Thus, it is difficult (if not impossible) to make a strong argument linking SO/spindle/coupling-locked brain activity with memory reactivation or consolidation.

      We appreciate your suggestion regarding the lack of a pre- and post-sleep memory task in our study design. We acknowledge that, in the absence of behavioral measures, it is hard to directly link SO-spindle coupling to memory consolidation in an outcome-driven manner. Our interpretation is instead based on the well-established role of these oscillations in memory processes, as demonstrated in previous studies. We sincerely appreciate this feedback and will adjust our Discussion accordingly to reflect a more precise interpretation of our findings.

      Discussion, Page 18 Lines 333-341

      “Despite providing new insights, our study has several limitations. First, our scalp EEG did not directly capture hippocampal ripples, preventing us from conclusively demonstrating triple coupling. Second, the combination of EEG-fMRI and the lack of a memory task limit our ability to parse fine-grained BOLD responses at the DOWN- vs. UP-states of SOs and link observed activations to behavioral outcomes. Third, the use of large anatomical ROIs may mask subregional contributions of specific thalamic nuclei or hippocampal subfields. Finally, without a memory task, we cannot establish a direct behavioral link between sleep-rhythm-locked activation and memory consolidation. Future studies combining techniques such as ultra-high-field fMRI or iEEG with cognitive tasks may refine our understanding of subregional network dynamics and functional significance during sleep.”

      (2) Relatedly, to understand the functional implications of the sleep rhythm-locked brain activity, the authors employed the "open-ended cognitive state decoding" method. While this method is interesting, it is rather indirect given that there were no behavioral indices in the manuscript. Thus, discussions based on these analyses are speculative at best. Please either tone down the language or find additional evidence to support these claims.

      Moreover, the results from this method are difficult to understand. Figure 3e showed that for all three types of sleep events (SO, spindle, SO-spindle), the same mental states (e.g., working memory, episodic memory, declarative memory) showed opposite directions of activation (left and right panels showed negative and positive activation, respectively). How to interpret these conflicting results? This ambiguity is also reflected by the term used: declarative memory and episodic memories are both indexed in the results. Yet these two processes can be largely overlapped. So which specific memory processes do these brain activity patterns reflect? The Discussion shall discuss these results and the limitations of this method.

      We appreciate your critical assessment of the open-ended cognitive state decoding method and its interpretational challenges. Given the concerns about the indirectness of this approach, we decided to remove its related content and results from Figure 3 in the main text and include it in Supplementary Figure 7.

      Due to the complexity of memory-related processes, we acknowledge that distinguishing between episodic and declarative memory based solely on this approach is not straightforward. We will revise the Supplementary Materials to explicitly discuss these limitations and clarify that our findings do not isolate specific cognitive processes but rather suggest general associations with memory-related networks.

      Discussion, Page 17-18 Lines 323-332

      “To explore functional relevance, we employed an open-ended cognitive state decoding approach using meta-analytic data (NeuroSynth: Yarkoni et al. (2011)). Although this method usefully generates hypotheses about potential cognitive processes, particularly in the absence of a pre- and post-sleep memory task, it is inherently indirect. Many cognitive terms showed significant associations (16 of 50), such as “episodic memory,” “declarative memory,” and “working memory.” We focused on episodic/declarative memory given the known link with hippocampal reactivation (Diekelmann & Born, 2010; Staresina et al., 2015; Staresina et al., 2023). Nonetheless, these inferences regarding memory reactivation should be interpreted cautiously without direct behavioral measures. Future research incorporating explicit tasks before and after sleep would more rigorously validate these potenial functional claims.”

      (3) The coupling strength is somehow inconsistent with prior results (Hahn et al., 2020, eLife, Helfrich et al., 2018, Neuron). Specifically, Helfrich et al. showed that among young adults, the spindle is coupled to the peak of the SO. Here, the authors reported that the spindles were coupled to down-to-up transitions of SO and before the SO peak. It is possible that participants' age may influence the coupling (see Helfrich et al., 2018). Please discuss the findings in the context of previous research on SO-spindle coupling.

      We appreciate your concern regarding the temporal characteristics of SO-spindle coupling. We acknowledge that the SO-spindle coupling phase results in our study are not identical to those reported by Hahn et al. (2020); Helfrich et al. (2018). However, these differences may arise due to slight variations in event detection parameters, which can influence the precise phase estimation of coupling. Notably, Hahn et al. (2020) also reported slight discrepancies in their group-level coupling phase results, highlighting that methodological differences can contribute to variability across studies. Furthermore, our findings are consistent with those of Schreiner et al. (2021), further supporting the robustness of our observations.

      That said, we acknowledge that our original description of SO-spindle coupling as occurring at the "transition from the lower state to the upper state" was not entirely precise. The -π/2 phase represents the true transition point, while our observed coupling phase is actually closer to the SO peak rather than strictly at the transition. We will revise this statement in the manuscript to ensure clarity and accuracy in describing the coupling phase.

      Discussion, Page 16 Lines 283-291

      “Our data provide insights into the neurobiological underpinnings of these sleep rhythms. SOs, originating mainly in neocortical areas such as the mPFC, alternate between DOWN- and UP-states. The thalamus generates sleep spindles, which in turn couple with SOs. Our finding that spindle peaks consistently occurred slightly before the UP-state peak of SOs (in 83 out of 107 participants), concurs with prior studies, including Schreiner et al. (2021). Yet it differs from some results suggesting spindles might peak right at the SO UP-state (Hahn et al., 2020; Helfrich et al., 2018). Such discrepancies could arise from differences in detection algorithms, participant age (Helfrich et al., 2018), or subtle variations in cortical-thalamic timing. Nonetheless, these results underscore the importance of coordinated SO-spindle interplay in supporting sleep-dependent processes.”

      (4) The discussion is rather superficial with only two pages, without delving into many important arguments regarding the possible functional significance of these results. For example, the author wrote, "This internal processing contrasts with the brain patterns associated with external tasks, such as working memory." Without any references to working memory, and without delineating why WM is considered as an external task even working memory operations can be internal. Similarly, for the interesting results on SO and reduced DMN activity, the authors wrote "The DMN is typically active during wakeful rest and is associated with self-referential processes like mind-wandering, daydreaming, and task representation (Yeshurun, Nguyen, & Hasson, 2021). Its reduced activity during SOs may signal a shift towards endogenous processes such as memory consolidation." This argument is flawed. DMN is active during self-referential processing and mind-wandering, i.e., when the brain shifts from external stimuli processing to internal mental processing. During sleep, endogenous memory reactivation and consolidation are also part of the internal mental processing given the lack of external environmental stimulation. So why during SO or during memory consolidation, the DMN activity would be reduced? Were there differences in DMN activity between SO and SO-spindle coupling events?

      We appreciate your concerns regarding the brevity of the discussion and the need for clearer theoretical arguments. We will expand this section to provide more in-depth interpretations of our findings in the context of prior literature. Regarding working memory (WM), we acknowledge that our phrasing was ambiguous. We will modify this statement in the Discussion section.

      For the SO-related reduction in DMN activity, we recognize the need for a more precise explanation. This reduced DMN activity may reflect large-scale neural inhibition characteristic of the SO trough. The DMN is typically active during internally oriented cognition (e.g., self-referential processing or mind-wandering) and is suppressed during external stimuli processing (Yeshurun, Nguyen, & Hasson, 2021). It is unlikely, however, that this suppression of DMN during SO events is related to a shift from internal cognition to external responses given it is during deep sleep time. Instead, it could be driven by the inherent rhythmic pattern of SOs, which makes it difficult to separate UP- from DOWN-states (the two temporal regressors were highly correlated, and similar brain activation during SOs events was obtained if modelled at the SO peak instead). Since the amplitude at the SO trough is consistently larger than that at the SO peak, the neural activation we detected may primarily capture the large-scale inhibition from DOWN-state.

      To address your final question, we have conducted the additional post hoc comparison of DMN activity between isolated SOs and SO-spindle coupling events. Our results indicate that

      DMN activation during SOs was significantly lower than during SO-spindle coupling (t<sub>(106)</sub> = -4.17, p < 1e-4). This suggests that SO-spindle coupling may involve distinct neural dynamics that partially re-engage DMN-related processes, possibly reflecting memory-related reactivation. We appreciate your constructive feedback and will integrate these expanded analyses and discussions into our revised manuscript.

      Results, Page 11 Lines 199-208

      “Spindles were correlated with positive activation in the thalamus (ROI analysis, t<sub>(106)</sub> = 15.39, p < 1e-4), the anterior cingulate cortex (ACC), and the putamen, alongside deactivation in the DMN (Fig. 3c). Notably, SO-spindle coupling was linked to significant activation in both the thalamus (ROI analysis, t<sub>(106)</sub> \= 3.38, p = 0.0005) and the hippocampus (ROI analysis, t<sub>(106)</sub> \= 2.50, p = 0.0070, Fig. 3d). However, no decrease in DMN activity was found during SO-spindle coupling, and DMN activity during SO was significantly lower than during coupling (ROI analysis, t<sub>(106)</sub> \= -4.17, p < 1e-4). For more detailed activation patterns, see Table S5-S7. We also varied the threshold used to detect SO events to assess its effect on hippocampal activation during SO-spindle coupling and observed that hippocampal activation remained significant when the percentile thresholds for SO detection ranged between 71% and 80% (see Fig. S6).”

      Discussion, Page 17-18 Lines 308-332

      “An intriguing aspect of our findings is the reduced DMN activity during SOs when modeled at the SO trough (DOWN-state). This reduced DMN activity may reflect large-scale neural inhibition characteristic of the SO trough. The DMN is typically active during internally oriented cognition (e.g., self-referential processing or mind-wandering) and is suppressed during external stimuli processing (Yeshurun, Nguyen, & Hasson, 2021). It is unlikely, however, that this suppression of DMN during SO events is related to a shift from internal cognition to external responses given it is during deep sleep time. Instead, it could be driven by the inherent rhythmic pattern of SOs, which makes it difficult to separate UP- from DOWN-states (the two temporal regressors were highly correlated, and similar brain activation during SOs events was obtained if modelled at the SO peak instead, Fig. S5). Since the amplitude at the SO trough is consistently larger than that at the SO peak, the neural activation we detected may primarily capture the large-scale inhibition from DOWN-state. Interestingly, no such DMN reduction was found during SO-spindle coupling, implying that coupling may involve distinct neural dynamics that partially re-engage DMN-related processes, possibly reflecting memory-related reactivation. Future research using high-temporal-resolution techniques like iEEG could clarify these possibilities.

      To explore functional relevance, we employed an open-ended cognitive state decoding approach using meta-analytic data (NeuroSynth: Yarkoni et al. (2011)). Although this method usefully generates hypotheses about potential cognitive processes, particularly in the absence of a pre- and post-sleep memory task, it is inherently indirect. Many cognitive terms showed significant associations (16 of 50), such as “episodic memory,” “declarative memory,” and “working memory.” We focused on episodic/declarative memory given the known link with hippocampal reactivation (Diekelmann & Born, 2010; Staresina et al., 2015; Staresina et al., 2023). Nonetheless, these inferences regarding memory reactivation should be interpreted cautiously without direct behavioral measures. Future research incorporating explicit tasks before and after sleep would more rigorously validate these potential functional claims.”

      Reviewing Editor Comment:

      The reviewers think that you are working on a relevant and important topic. They are praising the large sample size used in the study. The reviewers are not all in line regarding the overall significance of the findings, but they all agree the paper would strongly benefit from some extra work, as all reviewers raise various critical points that need serious consideration.

      We appreciate your recognition of the relevance and importance of our study, as well as your acknowledgment of the large sample size as a strength of our work. We understand that there are differing perspectives regarding the overall significance of our findings, and we value the constructive critiques provided. We are committed to addressing the key concerns raised by all reviewers, including refining our analyses, clarifying our interpretations, and incorporating additional discussions to strengthen the manuscript. Below, we address your specific recommendations and provide responses to each point you raised to ensure our methods and results are as transparent and comprehensible as possible. We believe that these revisions will significantly enhance the rigor and impact of our study, and we sincerely appreciate your thoughtful feedback in helping us improve our work.

      Reviewer #1 (Recommendations for the authors):

      (1) The phrase "overnight sleep" suggests an entire night, while these were rather "nocturnal naps". Please rephrase.

      Thank you for pointing this out. We have revised the phrasing in our manuscript to "nocturnal naps" instead of "overnight sleep" to more accurately reflect the duration of the sleep recordings.

      (2) Sleep staging results (macroscopic sleep architecture) should be provided in more detail (at least min and % of the different sleep stages, sleep onset latency, total sleep duration, total recording duration), at least mean/SD/range.

      Thank you for this suggestion. We will provide comprehensive tables in the supplementary materials, contains descriptive information about sleep-related characteristics. This information will help provide a clearer overview of the macroscopic sleep architecture in our dataset.

      Supplementary Materials, Page 42, Table S1

      Author response table 1.

      Descriptive results of demographic information and sleep characteristics. Note: The total recorded time is equal to the awake time plus the total sleep time. The sleep onset latency is the time taken to reach the first sleep epoch. The Sleep Efficiency is the ratio of actual sleep time to total recording time.

      Reviewer #2 (Recommendations for the authors):

      In order to allow for a better estimation of the reliability of the detected sleep events, please:

      (1) Provide densities and absolute numbers of all detected SOs and spindles (N1, NREM, and REM sleep).

      Thank you for pointing this out. We will provide comprehensive tables in the supplementary materials, contains detailed information about sleep waves at each sleep stage for all 107 subjects (Table S2-S4), listing for each subject:1) Different sleep stage duration; 2) Number of detected SOs; 3) Number of detected spindles; 4) Number of detected SO-spindle coupling events; 5) Density of detected SOs; 6) Density of detected spindles; 7) Density of detected SO-spindle coupling events.

      Supplementary Materials, Page 43-54, Table S2-S4

      (Consider of the length, we do not list all the tables here. Please refer to the revised manuscript.)

      (2) Show ERPs for all detected SOs and spindles (per sleep stage).

      Thank you for the suggestion. We will provide ERPs for all detected SOs and spindles, separated by sleep stage (N1, N2&N3, and REM) in supplementary Fig. S2-S4. These ERP waveforms will help illustrate the characteristic temporal profiles of SOs and spindles across different sleep stages.

      Methods, Page 25, Line 525-532

      “Event-related potentials (ERP) analysis. After completing the detection of each sleep rhythm event, we performed ERP analyses for SOs, spindles, and coupling events in different sleep stages. Specifically, for SO events, we took the trough of the DOWN-state of each SO as the zero-time point, then extracted data in a [-2 s to 2 s] window from the broadband (0.1–30 Hz) EEG and used [-2 s to -0.5 s] for baseline correction; the results were then averaged across 107 subjects (see Fig. S2a). For spindle events, we used the peak of each spindle as the zero-time point and applied the same data extraction window and baseline correction before averaging across 107 subjects (see Fig. S2b). Finally, for SO-spindle coupling events, we followed the same procedure used for SO events (see Fig. 2a, Figs. S3–S4).”

      Supplementary Materials, Page 36-38, Fig. S2-S4

      Author response image 1.

      ERPs of SOs and spindles coupling during different sleep stages across all 107 subjects. a. ERP of SOs in different sleep stages using the broadband (0.1–30 Hz) EEG data. We align the trough of the DOWN-state of each SO at time zero (see Methods for details). The orange line represents the SO ERP in the N1 stage, the black line represents the SO ERP in the N2&N3 stage, and the green line represents the SO ERP in the REM stage. b. ERP of spindles in different sleep stages using the broadband (0.1–30 Hz) EEG data. We align the peak of each spindle at time zero (see Methods for details). The color scheme is the same as in panel a.

      Author response image 2.

      ERP and time-frequency patterns of SO-spindle coupling in the N1 stage. The averaged temporal frequency pattern and ERP across all instances of SO-spindle coupling, computed over all subjects, following the same procedure as in Fig. 2a, but for N1 stage.

      Author response image 3.

      ERP and time-frequency patterns of SO-spindle coupling in the REM stage. The averaged temporal frequency pattern and ERP across all instances of SO-spindle coupling, computed over all subjects, again following the same procedure as in Fig. 2a, but for REM stage.

      (3) Provide detailed info concerning sleep characteristics (time spent in each sleep stage etc.).

      Thank you for this suggestion. Same as the response above, we will provide comprehensive tables in the supplementary materials, contains descriptive information about sleep-related characteristics.

      Supplementary Materials, Page 42, Table S1 (same as above)

      (4) What would happen if more stringent parameters were used for event detection? Would the authors still observe a significant number of SO spindles during N1 and REM? Would this affect the fMRI-related results?

      Thank you for this suggestion. Our methods for detecting SOs, spindles, and their couplings were originally developed for N2 and N3 sleep data, based on the specific characteristics of these stages. These methods are widely recognized in sleep research (Hahn et al., 2020; Helfrich et al., 2019; Helfrich et al., 2018; Ngo, Fell, & Staresina, 2020; Schreiner et al., 2022; Schreiner et al., 2021; Staresina et al., 2015; Staresina et al., 2023). However, because this percentile-based detection approach will inherently identify a certain number of events if applied to other stages (e.g., N1 and REM), the nature of these events in those stages remains unclear compared to N2/N3. We nevertheless identified and reported the detailed descriptive statistics of these sleep rhythms in all sleep stages, under the same operational definitions, both for completeness and as a sanity check. Within the same subject, there should be more SOs, spindles, and their couplings in N2/N3 than in N1 or REM (see also Figure S2-S4, Table S1-S4).

      Furthermore, in order to explore the impact of this on our fMRI results, we conducted an additional sensitivity analysis by applying different detection parameters for SOs. Specifically, we adjusted amplitude percentile thresholds for SO detection (the parameter that has the greatest impact on the results). We used the hippocampal activation value during N2&N3 stage SO-spindle coupling as an anchor value and found that when the parameters gradually became stricter, the results were similar to or even better than the current results. However, when we continued to increase the threshold, the results began to gradually decrease until the threshold was increased to 80%, and the results were no longer significant. This indicates that our results are robust within a specific range of parameters, but as the threshold increases, the number of trials decreases, ultimately weakening the statistical power of the fMRI analysis.

      Thank you again for your suggestions on sleep rhythm event detection. We will add the results in Supplementary and revise our manuscript accordingly.

      Results, Page 11, Line 199-208

      “Spindles were correlated with positive activation in the thalamus (ROI analysis, t<sub>(106)</sub> = 15.39, p < 1e-4), the anterior cingulate cortex (ACC), and the putamen, alongside deactivation in the DMN (Fig. 3c). Notably, SO-spindle coupling was linked to significant activation in both the thalamus (ROI analysis, t<sub>(106)</sub> \= 3.38, p = 0.0005) and the hippocampus (ROI analysis, t<sub>(106)</sub> \= 2.50, p = 0.0070, Fig. 3d). However, no decrease in DMN activity was found during SO-spindle coupling, and DMN activity during SO was significantly lower than during coupling (ROI analysis, t<sub>(106)</sub> \= -4.17, p < 1e-4). For more detailed activation patterns, see Table S5-S7. We also varied the threshold used to detect SO events to assess its effect on hippocampal activation during SO-spindle coupling and observed that hippocampal activation remained significant when the percentile thresholds for SO detection ranged between 71% and 80% (see Fig. S6).”

      Supplementary Materials, Page 40, Fig. S6

      Author response image 4.

      Influence of the percentile threshold for SO detection on hippocampal activation (ROI) during SO-spindle coupling. We changed the percentile threshold for SO event detection in the EEG data analysis and then reconstructed the GLM design matrix based on the SO events detected at each threshold. The brain-wide activation pattern of SO-spindle couplings in the N2/3 stage was extracted using the same method as shown in Fig. 3. The gray horizontal line represents the significant range (71%–80%). * p < 0.05.

      Finally, we sincerely thank all again for your thoughtful and constructive feedback. Your insights have been invaluable in refining our analyses, strengthening our interpretations, and improving the clarity and rigor of our manuscript. We appreciate the time and effort you have dedicated to reviewing our work, and we are grateful for the opportunity to enhance our study based on your recommendations.

      References:

      Bergmann, T. O., Mölle, M., Diedrichs, J., Born, J., & Siebner, H. R. (2012). Sleep spindle-related reactivation of category-specific cortical regions after learning face-scene associations. NeuroImage, 59(3), 2733-2742.

      Buzsáki, G. (2015). Hippocampal sharp wave‐ripple: A cognitive biomarker for episodic memory and planning. Hippocampus, 25(10), 1073-1188.

      Caporro, M., Haneef, Z., Yeh, H. J., Lenartowicz, A., Buttinelli, C., Parvizi, J., & Stern, J. M. (2012). Functional MRI of sleep spindles and K-complexes. Clinical neurophysiology, 123(2), 303-309.

      Coulon, P., Budde, T., & Pape, H.-C. (2012). The sleep relay—the role of the thalamus in central and decentral sleep regulation. Pflügers Archiv-European Journal of Physiology, 463, 53-71.

      Crunelli, V., Lőrincz, M. L., Connelly, W. M., David, F., Hughes, S. W., Lambert, R. C., Leresche, N., & Errington, A. C. (2018). Dual function of thalamic low-vigilance state oscillations: rhythm-regulation and plasticity. Nature Reviews Neuroscience, 19(2), 107-118.

      Czisch, M., Wehrle, R., Stiegler, A., Peters, H., Andrade, K., Holsboer, F., & Sämann, P. G. (2009). Acoustic oddball during NREM sleep: a combined EEG/fMRI study. PloS one, 4(8), e6749.

      Diba, K., & Buzsáki, G. (2007). Forward and reverse hippocampal place-cell sequences during ripples. Nature Neuroscience, 10(10), 1241.

      Diekelmann, S., & Born, J. (2010). The memory function of sleep. Nature Reviews Neuroscience, 11(2), 114-126.

      Fogel, S., Albouy, G., King, B. R., Lungu, O., Vien, C., Bore, A., Pinsard, B., Benali, H., Carrier, J., & Doyon, J. (2017). Reactivation or transformation? Motor memory consolidation associated with cerebral activation time-locked to sleep spindles. PloS one, 12(4), e0174755.

      Hahn, M. A., Heib, D., Schabus, M., Hoedlmoser, K., & Helfrich, R. F. (2020). Slow oscillation-spindle coupling predicts enhanced memory formation from childhood to adolescence. Elife, 9, e53730.

      Halassa, M. M., Siegle, J. H., Ritt, J. T., Ting, J. T., Feng, G., & Moore, C. I. (2011). Selective optical drive of thalamic reticular nucleus generates thalamic bursts and cortical spindles. Nature Neuroscience, 14(9), 1118-1120.

      Hale, J. R., White, T. P., Mayhew, S. D., Wilson, R. S., Rollings, D. T., Khalsa, S., Arvanitis, T. N., & Bagshaw, A. P. (2016). Altered thalamocortical and intra-thalamic functional connectivity during light sleep compared with wake. NeuroImage, 125, 657-667.

      Helfrich, R. F., Lendner, J. D., Mander, B. A., Guillen, H., Paff, M., Mnatsakanyan, L., Vadera, S., Walker, M. P., Lin, J. J., & Knight, R. T. (2019). Bidirectional prefrontal-hippocampal dynamics organize information transfer during sleep in humans. Nature Communications, 10(1), 3572.

      Helfrich, R. F., Mander, B. A., Jagust, W. J., Knight, R. T., & Walker, M. P. (2018). Old brains come uncoupled in sleep: slow wave-spindle synchrony, brain atrophy, and forgetting. Neuron, 97(1), 221-230. e224.

      Horovitz, S. G., Fukunaga, M., de Zwart, J. A., van Gelderen, P., Fulton, S. C., Balkin, T. J., & Duyn, J. H. (2008). Low frequency BOLD fluctuations during resting wakefulness and light sleep: A simultaneous EEG‐fMRI study. Human brain mapping, 29(6), 671-682.

      Huang, Q., Xiao, Z., Yu, Q., Luo, Y., Xu, J., Qu, Y., Dolan, R., Behrens, T., & Liu, Y. (2024). Replay-triggered brain-wide activation in humans. Nature Communications, 15(1), 7185.

      Ilhan-Bayrakcı, M., Cabral-Calderin, Y., Bergmann, T. O., Tüscher, O., & Stroh, A. (2022). Individual slow wave events give rise to macroscopic fMRI signatures and drive the strength of the BOLD signal in human resting-state EEG-fMRI recordings. Cerebral Cortex, 32(21), 4782-4796.

      Laufs, H. (2008). Endogenous brain oscillations and related networks detected by surface EEG‐combined fMRI. Human brain mapping, 29(7), 762-769.

      Laufs, H., Walker, M. C., & Lund, T. E. (2007). ‘Brain activation and hypothalamic functional connectivity during human non-rapid eye movement sleep: an EEG/fMRI study’—its limitations and an alternative approach. Brain, 130(7), e75.

      Margulies, D. S., Ghosh, S. S., Goulas, A., Falkiewicz, M., Huntenburg, J. M., Langs, G., Bezgin, G., Eickhoff, S. B., Castellanos, F. X., & Petrides, M. (2016). Situating the default-mode network along a principal gradient of macroscale cortical organization. Proceedings of the National Academy of Sciences, 113(44), 12574-12579.

      Massimini, M., Huber, R., Ferrarelli, F., Hill, S., & Tononi, G. (2004). The sleep slow oscillation as a traveling wave. Journal of Neuroscience, 24(31), 6862-6870.

      Moehlman, T. M., de Zwart, J. A., Chappel-Farley, M. G., Liu, X., McClain, I. B., Chang, C., Mandelkow, H., Özbay, P. S., Johnson, N. L., & Bieber, R. E. (2019). All-night functional magnetic resonance imaging sleep studies. Journal of neuroscience methods, 316, 83-98.

      Molle, M., Bergmann, T. O., Marshall, L., & Born, J. (2011). Fast and slow spindles during the sleep slow oscillation: disparate coalescence and engagement in memory processing. Sleep, 34(10), 1411-1421.

      Ngo, H.-V., Fell, J., & Staresina, B. (2020). Sleep spindles mediate hippocampal-neocortical coupling during long-duration ripples. Elife, 9, e57011.

      Picchioni, D., Horovitz, S. G., Fukunaga, M., Carr, W. S., Meltzer, J. A., Balkin, T. J., Duyn, J. H., & Braun, A. R. (2011). Infraslow EEG oscillations organize large-scale cortical– subcortical interactions during sleep: a combined EEG/fMRI study. Brain research, 1374, 63-72.

      Schabus, M., Dang-Vu, T. T., Albouy, G., Balteau, E., Boly, M., Carrier, J., Darsaud, A., Degueldre, C., Desseilles, M., & Gais, S. (2007). Hemodynamic cerebral correlates of sleep spindles during human non-rapid eye movement sleep. Proceedings of the National Academy of Sciences, 104(32), 13164-13169.

      Schreiner, T., Kaufmann, E., Noachtar, S., Mehrkens, J.-H., & Staudigl, T. (2022). The human thalamus orchestrates neocortical oscillations during NREM sleep. Nature communications, 13(1), 5231.

      Schreiner, T., Petzka, M., Staudigl, T., & Staresina, B. P. (2021). Endogenous memory reactivation during sleep in humans is clocked by slow oscillation-spindle complexes. Nature Communications, 12(1), 3112.

      Singh, D., Norman, K. A., & Schapiro, A. C. (2022). A model of autonomous interactions between hippocampus and neocortex driving sleep-dependent memory consolidation. Proceedings of the National Academy of Sciences, 119(44), e2123432119.

      Spoormaker, V. I., Schröter, M. S., Gleiser, P. M., Andrade, K. C., Dresler, M., Wehrle, R., Sämann, P. G., & Czisch, M. (2010). Development of a large-scale functional brain network during human non-rapid eye movement sleep. Journal of Neuroscience, 30(34), 11379-11387.

      Staresina, B. P., Bergmann, T. O., Bonnefond, M., van der Meij, R., Jensen, O., Deuker, L., Elger, C. E., Axmacher, N., & Fell, J. (2015). Hierarchical nesting of slow oscillations, spindles and ripples in the human hippocampus during sleep. Nature Neuroscience, 18(11), 1679-1686.

      Staresina, B. P., Niediek, J., Borger, V., Surges, R., & Mormann, F. (2023). How coupled slow oscillations, spindles and ripples coordinate neuronal processing and communication during human sleep. Nature Neuroscience, 1-9.

      Yarkoni, T., Poldrack, R. A., Nichols, T. E., Van Essen, D. C., & Wager, T. D. (2011). Large-scale automated synthesis of human functional neuroimaging data. Nature methods, 8(8), 665-670.

      Yeshurun, Y., Nguyen, M., & Hasson, U. (2021). The default mode network: where the idiosyncratic self meets the shared social world. Nature Reviews Neuroscience, 1-12.

    1. Author response:

      Public Reviews:

      Reviewer #1 (Public review):

      Summary

      The authors describe a method for gastruloid formation using mouse embryonic stem cells (mESCs) to study YS and AGM-like hematopoietic differentiation. They characterise the gastruloids during nine days of differentiation using a number of techniques including flow cytometry and single-cell RNA sequencing. They compare their findings to a published data set derived from E10-11.5 mouse AGM. At d9, gastruloids were transplanted under the adrenal gland capsule of immunocompromised mice to look for the development of cells capable of engrafting the mouse bone marrow. The authors then applied the gastruloid protocol to study overexpression of Mnx1 which causes infant AML in humans.

      In the introduction, the authors define their interpretation of the different waves of hematopoiesis that occur during development. 'The subsequent wave, known as definitive, produces: first, oligopotent erythro-myeloid progenitors (EMPs) in the YS (E8-E8.5); and later myelo-lymphoid progenitors (MLPs - E9.5-E10), multipotent progenitors (MPPs - E10-E11.5), and hematopoietic stem cells (HSCs - E10.5-E11.5), in the aorta-gonadmesonephros (AGM) region of the embryo proper.' Herein they designate the yolk sac-derived wave of EMP hematopoiesis as definitive, according to convention, although paradoxically it does not develop from intraembryonic mesoderm or give rise to HSCs.

      The apparent perplexity of the Reviewer with our definition of primitive and definitive waves is somewhat surprising, as it is widely used in the field (e.g. PMID: 18204427; PMID: 28299650; PMID: 33681211). Definitive haematopoiesis, encompassing EMP, MLP, MPP and HSC, highlights their origin from haemogenic hendothelium, generation of mature cells with adult characteristics from progenitors with multilineage potential and direct and indirect developmental contributions to the intra-embryonic and time-restricted generation of HSCs.

      General comments

      The authors make the following claims in the paper:

      (1) The development of a protocol for hemogenic gastruloids (hGx) that recapitulates YS and AGM-like waves of blood from HE.

      (2) The protocol recapitulates both YS and EMP-MPP embryonic blood development 'with spatial and temporal accuracy'.

      (3) The protocol generates HSC precursors capable of short-term engraftment in an adrenal niche.

      (4) Overexpression of MNX1 in hGx transforms YS EMP to 'recapitulate patient transcriptional signatures'.

      (5) hGx is a model to study normal and leukaemic embryonic hematopoiesis.

      There are major concerns with the manuscript. The statements and claims made by the authors are not supported by the data presented, data is overinterpreted, and the conclusions cannot be justified. Furthermore, the data is presented in a way that makes it difficult for the reader to follow the narrative, causing confusion. The authors have not discussed how their hGx compares to the previously published mouse embryoid body protocols used to model early development and hematopoiesis. the data is presented in a way that makes it difficult for the reader to follow the narrative, causing confusion. The authors have not discussed how their hGx compares to the previously published mouse embryoid body protocols used to model early development and hematopoiesis.

      Specific points

      (1) It is claimed that HGxs capture cellularity and topography of developmental blood formation. The hGx protocol described in the manuscript is a modification of a previously published gastruloid protocol (Rossi et al 2022). The rationale for the protocol modifications is not fully explained or justified. There is a lack of novelty in the presented protocol as the only modifications appear to be the inclusion of Activin A and an extension of the differentiation period from 7 to 9 days of culture. No direct comparison has been made between the two versions of gastruloid differentiation to justify the changes.

      The Reviewer paradoxically claims that the protocol is not novel and that it differs from a previous publication in at least 2 ways – the patterning pulse and the length of the protocol. Of these, the patterning pulse is key. As documented in Fig. S1, we cannot obtain Flk1-GFP expression in the absence of Activin A. Expression of Flk1 is a fundamental step in haemato-endothelial specification and, accordingly, we do not see CD41 or CD45+ cells in the absence of Activin A. Also, in our hands, there is a clear time-dependent progression of marker expression, with sequential acquisition of CD41 and CD45, with the latter not detectable until 192h (Fig. 1C-D), another key difference relative to the Rossi et al (2022) protocol. The 192h-timepoint, we argue in the manuscript, and present further evidence for in this rebuttal, corresponds to the onset of AGM-like haematopoiesis. We have empirically extended the protocol to maximise the CD45+ cell output (Fig. S1B-D).

      The inclusion of Activin A at high concentration at the beginning of differentiation would be expected to pattern endoderm rather than mesoderm. BMP signaling is required to induce Flk1+ mesoderm, even in the presence of Wnt.

      Again, we call the Reviewer’s attention to Fig. S1 which clearly shows that Activin A (with no BMP added) is required for induction of Flk1 expression, in the presence of Wnt. Activin A in combination with Wnt, is used in other protocols of haemato-endothelial differentiation from pluripotent cells, with no BMP added in the same step of patterning and differentiation (PMID: 39227582; PMID: 39223325). In the latter protocol, we also call the Reviewer’s attention to the fact that a higher concentration of Activin A precludes the need for BMP4 addition. Finally, one of us has recently reported that Activin A, on its own, will induce FLK1, as well as other anterior mesodermal progenitors (https://www.biorxiv.org/content/10.1101/2025.01.11.632562v1)..) In addressing the Reviewer’s concerns with the dose of Activin A used, we titrated its concentration against activation of Flk1, confirming optimal Flk1-GFP expression at the 100ng/ml dose used in the manuscript.

      Author response image 1.

      Dose-dependent requirement of Activin A for induction of Flk1 expression in haemogenic gastruloids. Composite GFP and brightfield live imaging of Flk1-GFP haemogenic gastruloids at 96h. Images were acquired using a Cytation5 instrument (Thermo). Images are representative of 12 gastruloids per condition.

      FACS analysis of the hGx during differentiation is needed to demonstrate the co-expression of Flk1-GFP and lineage markers such as CD34 to indicate patterning of endothelium from Flk1+ mesoderm. The FACS plots in

      Fig. 1 show c-Kit expression but very little VE-cadherin which suggests that CD34 is not induced. Early endoderm expresses c-Kit, CXCR4, and Epcam, but not CD34 which could account for the lack of vascular structures within the hGx as shown in Fig. 1E.

      We were surprised by the Reviewer’s comment that there are no endothelial structures in our gastruloids. The presence of a Flk1-GFP+ network is visible in the GFP images in Fig.1B, from 144h onwards, also shown in Author response image 2A. In addition, our single-cell RNA-seq data, included in the manuscript, confirms the presence of endothelial cells with a developing endothelial, including arterial, programme. This can be seen in Fig. 2B, F of the manuscript and is represented in Author response image 2B. In contrast with the Reviewer’s claims that no endothelial cells are formed, the data show that Kdr (Flk1)+ cells co-express Cdh5/VE-Cadherin and indeed Cd34, attesting to the presence of an endothelial programme. Arterial markers Efnb2, Flt1, and Dll4 are present. A full-blown programme, which also includes haemogenic markers including Sox17, Esam, Cd44 and Mecom is clear at early (144h) and, particularly at late (192h) timepoints in cells sorted on detection of surface c-Kit (Author response image 2B). Further to the data shown in B, already present in the manuscript, we also document co-expression of Flk1-GFP and CD34 by flow cytometry (Author response image 2C).

      Author response image 2.

      Haemogenic gastruloids have a branched vascular network. A. Whole-mount confocal imaging of 144h-haemogenic gastruloids. B. Differentiation of an arterial endothelial programme in haemogenic gastruloids; singlecell RNA-seq data of differentiating haemogenic gastruloids, sorted on cell surface expression of c-Kit at 144 and 192h; gene expression colour scale from yellow (low) to orange (high); grey = no detectable expression. C. Flow cytometry plots of 216h-haemogenic gastruloids showing detection of haemato-endothelial marker CD34.

      (2) The protocol has been incompletely characterised, and the authors have not shown how they can distinguish between either wave of Yolk Sac (YS) hematopoiesis (primitive erythroid/macrophage and erythro-myeloid EMP) or between YS and intraembryonic Aorta-Gonad-Mesonephros (AGM) hematopoiesis. No evidence of germ layer specification has been presented to confirm gastruloid formation, organisation, and functional ability to mimic early development. Furthermore, differentiation of YS primitive and YS EMP stages of development in vitro should result in the efficient generation of CD34+ endothelial and hematopoietic cells. There is no flow cytometry analysis showing the kinetics of CD34 cell generation during differentiation. Benchmarking the hGx against developing mouse YS and embryo data sets would be an important verification.

      The Reviewer is correct that we have not provided detailed characterisation of the different germ layers, as this was not the focus of the study. In that context, we were surprised by the earlier comment assuming co-expression of c-kit, Cxcr4 and Epcam, which we did not show, while overlooking the endothelial programme reiterated above, which we have presented.

      Given our focus on haemato-endothelial specification, we have started the single-cell RNA-seq characterisation of the haemogenic gastruloid at 120h and have not looked specifically at earlier timepoints of embryo patterning.

      This said, we show the presence of neuroectodermal cells in cluster 9; on the other hand, cluster 7 includes hepatoblast-like cells, denoting endodermal specification. We are happy to include this characterisation, to the extent that it is present, in a revised version of the manuscript. However, in the absence of earlier timepoints and given the bias towards mesodermal specification, we expect that specification of ectodermal and endodermal programmes may be incomplete.

      In respect of the contention regarding the capture of YS-like and AGM-like haematopoiesis, we have presented evidence in the manuscript that haemogenic cells generated during gastruloid differentiation, particularly at late 192h and 216h timepoints project onto highly purified c-Kit+ CD31+ Gfi1-expressing cells from mouse AGM (PMID: 38383534), providing support for the recapitulation of the corresponding developmental stage. In distinguishing between YS-like and AGM-like haematopoiesis, we call the Reviewer’s attention to the replotting of the single-cell RNA-seq data already in the manuscript, which we provided in response to point 1 (Author response image 2B), which highlights an increase in Sox17, but not Sox18, expression in the 192h haemogenic endothelium, which suggests an association with AGM haematopoiesis (PMID: 20228271). A significant association of Cd44 and Procr expression with the same time-point (Fig. 2F in the manuscript), further supports an AGM-like endothelial-to-haematopoietic transition at the 192h timepoint.

      Following on the Reviewer’s comments about CD34, we also inspected co-expression of CD34 with CD41 and CD45, the latter co-expression present in, although not necessarily exclusive to, AGM haematopoiesis.

      Reassuringly, we observed clear co-expression with both markers (Author response image 3), in addition to a CD41+CD34-population, which likely reflects YS EMP-independent erythropoiesis. Interestingly, marker expression is responsive to the levels of Activin A used in the patterning pulse, with the 100ng/ml Activin A used in our protocol superior to 75ng/ml.

      Author response image 3.

      Association of CD34 with CD41 and CD45 expression is Activin A-responsive and supports the presence of definitive haematopoiesis. A. Flow cytometry analysis of CD34 and CD41 expression in 216h-haemogenic gastruloids; two doses of Activin A were used in the patterning pulse with CHI99021 between 48-72h. FMO controls shown. B. Flow cytometry analysis of CD34 and CD45 at 216h in the same experimental conditions.

      We agree that it remains challenging to identify markers exclusive to AGM haematopoiesis, which is operationally equated with generation of transplantable haematopoietic stem cells. While HSC generation is a key event characteristic of the AGM, not all AGM haematopoiesis corresponds to HSCs, an important point in evaluating the data presented in the manuscript, and indeed acknowledged by us.

      Author response image 4.

      Clustering of haemogenic gastruloid cells sorted on the basis of haemato-endothelial surface markers CD41, C-Kit and CD45. A. Leiden clustering to single-cell RNA-seq data. B. Time stamps of sorted haemogenic gastruloid cells in A. C. Surface marker stamps of cells in A.

      Given the centrality of this point in comments by all the Reviewers, we have conducted projections of our single-cell RNA-seq data against two studies which (1) capture arterial and haemogenic specification in the para-splanchnopleura (pSP) and AGM region between E8.0 and E11 (Hou et al, PMID: 32203131), and (2) uniquely capture YS, AGM and FL progenitors and the AGM endothelial-to-haematopoietic transition (EHT) in the same scRNA-seq dataset (Zhu et al, PMID: 32392346).

      Focusing the analysis on the subsets of haemogenic gastruloid cells sorted as CD41+ (144h) CKit+ (144h and 192h) and CD45+ (192h and 216h) (Author response image 4AC), we show:

      (1) That a subset of haemato-endothelial cells from haemogenic gastruloids at 144h to 216h project onto intra-embryonic cells spanning E8.25 to E10 (Author response image 5A-B). This is in agreement with our interpretation that 216h are no later than the MPP/pre-HSC state of embryonic development, requiring further maturation to generate long-term engrafting HSC.

      (2) That haemogenic gastruloids contain YS-like (including EMP-like) and AGM-like haematopoietic cells (Author response image 6A-B). Significantly, some of the cells, particularly c-Kit-sorted cells with a candidate endothelial and HE-like signature project onto AGM pre-HE and HE, as well as IAHC, and later, predominantly 216h cells, have characteristics of MPP/LMPP-like cells from the FL.

      Altogether, the data support the notion that haemogenic gastruloids capture YS and AGM haematopoiesis until E10, as suggested by us in the manuscript. We thought it was important to share this preliminary data with the Editors at an early stage, and we will incorporate a deeper analysis in a revised version of the manuscript.

      Single-cell RNA sequencing was used to compare hGx with mouse AGM. The authors incorrectly conclude that ' ..specification of endothelial and HE cells in hGx follows with time-dependent developmental progression into putative AGM-like HE..' And, '...HE-projected hGx cells.......expressed Gata2 but not Runx1, Myb, or Gfi1b..' Hemogenic endothelium is defined by the expression of Runx1 and Gfli1b is downstream of Runx1.

      As a hierarchy of regulation, Gata2 precedes and drives Runx1 expression at the specification of HE (PMID: 17823307; PMID: 24297996), while Runx1 drives the EHT, upstream of Gfi1b in haematopoietic clusters (PMID: 34517413).

      Author response image 5.

      Projection of sorted haemogenic gastruloid cells onto Hou et al dataset (PMID: 32203131) analysing development of mouse intra-embryonic haematopoiesis. A. Time signatures of Hou et al data. B. Projection of Leiden clusters in Author response image 4A. Methodology as described in our manuscript; 68% gastruloid cells projected.

      Author response image 6.

      Projection of sorted haemogenic gastruloid cells onto Zhu et al dataset (PMID: 32392346), capturing arterial endothelial and haemogenic endothelial development, in reference to YS, AGM and FL haematopoietic progenitors. A. Functional cluster classification as per Zhu et al. B. Projection of Leiden clusters in Author response image 4A. Methodology as detailed in our manuscript; 58% gastruloid cells projected. Haematopoietic clusters annotated as in A.

      (3) The hGx protocol 'generates hematopoietic SC precursors capable of short-term engraftment' is not supported by the data presented. Short-term engraftment would be confirmed by flow cytometric detection of hematopoietic cells within the recipient bone marrow, spleen, thymus, and peripheral blood that expressed the BFP transgene. This analysis was not provided. PCR detection of transcripts, following an unspecified number of amplification cycles, as shown in Figure 3G (incorrectly referred to as Figure 3F in the legend) is not acceptable evidence for engraftment.

      We provide the full flow cytometry analysis of spleen engraftment in the 5 mice which received implantation of 216h-haemogenic gastruloids in the adrenal gland; an additional (control) animal received adrenal injection of PBS (Author response image 7). The animals were analysed at 4 weeks. In this experiment, the bone marrow collection was limiting, and material was prioritised for PCR.

      We had previously provided only representative plots of flow cytometry analysis of bone marrow and spleen in Fig. S4E, which we described as low-level engraftment. The analysis was complemented with genomic DNA PCR, where detection was present in only some of the replicates tested per animal. We confirm that PCR analysis used conventional 40 cycles; the sensitivity was shown in Fig. S4F. As shown in Fig. 3 A-C, no more than 7 CD45+CD144+ multipotent cells are present per haemogenic gastruloid, with 3 haemogenic gastruloids implanted in the adrenal gland of each transplanted animal. We argue that the low level of cytometric and molecular engraftment at 4 weeks, from haemogenic gastruloid-derived progenitors that have not progressed beyond a stage equivalent to E10 Author response image 5A-B) and that we have described as requiring additional maturation in vivo, are not surprising.

      Author response image 7.

      BFP engraftment of Nude recipient mice 4 weeks after unilateral adrenal implantation of 216h-haemogenic gastruloids. Flow cytometry analysis of spleen engraftment. Genomic PCR analysis is shown in Fig. 3G of the manuscript.

      Transplanted hGx formed teratoma-like structures, with hematopoietic cells present at the site of transplant only analysed histologically. Indeed, the quality of the images provided does not provide convincing validation that donor-derived hematopoietic cells were present in the grafts.

      As stated in the text, the images mean to illustrate that the haemogenic gastruloids developed in situ. The observation of donor-derived blood cells in the implanted haemogenic gastruloids would not correspond to engraftment, as we have amply demonstrated that they have generated blood cells in vitro. There is no evidence that there are remaining pluripotent cells in the haemogenic gastruloid after 9 days of differentiation, and it is therefore not clear that these are teratomas

      There is no justification for the authors' conclusion that '... the data suggest that 216h hGx generate AGM-like pre-HSC capable of at least short-term multilineage engraftment upon maturation...'. Indeed, this statement is in conflict with previous studies demonstrating that pre-HSCs in the dorsal aorta of the mouse embryo are immature and actually incapable of engraftment.

      We have clearly stated that we do not see haematopoietic engraftment through transplantation of dissociated haemogenic gastruloids, which reach the E10 state containing pre-HSC (Author response image 5). Instead, we observed rare myelo-erythroid (in the manuscript) and myelo-lymphoid (Author response image 9 below, in response to Reviewer 2) engraftment upon in vivo maturation of haemogenic gastruloids with preserved 3D organisation. These statements are not contradictory.

      The statement '...low-level production of engrafting cells recapitulates their rarity in vivo, in agreement with the embryo-like qualities of the gastruloid system....' is incorrect. Firstly, no evidence has been provided to show the hGx has formed a dorsal aorta facsimile capable of generating cells with engrafting capacity. Secondly, although engrafting cells are rare in the AGM, approximately one per embryo, they are capable of robust and extensive engraftment upon transplantation.

      We are happy to rephrase the statement to simply say that “…the data suggest that 216h haemogenic gastruloids contain candidate AGM-like progenitors with some short-term engraftment potential but incomplete functional maturation.” To be clear, with our existing statement we meant to highlight that the production of definitive AGM-like haematopoietic progenitors (not all of which are engrafting) in haemogenic gastruloids does not correspond to non-physiological single-lineage programming. We did not claim that we achieved production of HSC, which would be long-term engrafting.

      (4) Expression MNX1 transcript and protein in hematopoietic cells in MNX1 rearranged acute myeloid leukaemia (AML) is one cause of AML in infants. In the hGX model of this disease, Mnx1 is overexpressed in the mESCs that are used to form gastruloids. Mnx1 overexpression seems to confer an overall growth advantage on the hGx and increase the serial replating capacity of the small number of hematopoietic cells that are generated. The inefficiency with which the hGx model generates hematopoietic cells makes it difficult to model this disease. The poor quality of the cytospin images prevents accurate identification of cells. The statement that the kit-expressing cells represent leukemic blast cells is not sufficiently validated to support this conclusion. What other stem cell genes are expressed? Surface kit expression also marks mast cells, frequently seen in clonogenic assays of blood cells. Flow cytometric and gene expression analyses using known markers would be required.

      The haemogenic gastruloid model generates haematopoietic and haemato-endothelial cells. MNX1 expands Kit+ cells at 144h, which we show to have a haemato-endothelial signature (manuscript Fig. 2B, which we replotted in Author response image 2B).

      Serial replating of CFC assays is a conventional in vitro assay of leukaemia transformation. Critically, colony replating is not maintained in EV control cells, attesting to the transformation potential of MNX1.

      Although we have not fully-traced the cellular hierarchy of MNX1-driven transformation in the haemogenic gastruloid system, the in vitro replating expands a Kit+ cell (Fig. 5E), which reflects the surface phenotype of the leukaemia, also recapitulated in the mouse model initiated by MNX1-overexpressing FL cells. Importantly, it recapitulates the transcriptional profile of MNX1-leukaemia patients (Fig. 6C), which is uniquely expressed by MNX1144h and replated colony cells, but not to MNX1 216h gastruloid cells, arguing against a generic signature of MNX1 overexpression (Fig. 6B). Importantly, the MNX1-transformation of haemogenic gastruloid cells is superior to the FL leukaemia model at capturing the unique transcriptional features of MNX1-driven leukaemia, distinct from other forms of AML in the same age group (Fig S7). It is possible that this corresponds to a preleukaemia event, and we will explore this in future studies, which are beyond the proof-of-principle nature of this paper.

      (5) In human infant MNX1 AML, the mutation is thought to arise at the fetal liver stage of development. There is no evidence that this developmental stage is mimicked in the hGx model.

      We never claim that the haemogenic gastruloid model mimics the foetal liver. We propose that susceptibility to MNX1 is at the HE-to-EMP transition. Moreover, and importantly, contrary to the Reviewer’s statement, there is no evidence in the literature that the mutation arises in the foetal liver stage, just that the mutation arises before birth (PMID: 38806630), which is different. In a mouse model of MNX1 overexpression, the authors achieve leukaemia engraftment upon MNX1 overexpression in foetal liver, but not in bone marrow cells (PMID: 37317878). This is in agreement with a vulnerability of embryonic / foetal, but not adult cells to the MNX1 expression caused by the translocation. However, haematopoietic cells in the foetal liver originate from YS and AGM precursors, so the origin of the MNX1-susceptible cells can be in those locations, rather than the foetal liver itself.

      Reviewer #2 (Public review):<br /> Summary:<br /> In this manuscript, the authors develop an exciting new hemogenic gastruloid (hGX) system, which they claim reproduces the sequential generation of various blood cell types. The key advantage of this cellular system would be its potential to more accurately recapitulate the spatiotemporal emergence of hematopoietic progenitors within their physiological niche compared to other available in vitro systems. The authors present a large set of data and also validate their new system in the context of investigating infant leukemia.<br /> Strengths:<br /> The development of this new in vitro system for generating hematopoietic cells is innovative and addresses a significant drawback of current in vitro models. The authors present a substantial dataset to characterize this system, and they also validate its application in the context of investigating infant leukemia.<br /> Weaknesses:<br /> The thorough characterization and full demonstration that the cells produced truly represent distinct waves of hematopoietic progenitors are incomplete. The data presented to support the generation of late yolk sac (YS) progenitors, such as lymphoid cells, and aortic-gonad-mesonephros (AGM)-like progenitors, including pre-hematopoietic stem cells (pre-HSCs), by this system are not entirely convincing. Given that this is likely the manuscript's most crucial claim, it warrants further scrutiny and direct experimental validation. Ideally, the identity of these progenitors should be further demonstrated by directly assessing their ability to differentiate into lymphoid cells or fully functional HSCs. Instead, the authors primarily rely on scRNA-seq data and a very limited set of markers (e.g., Ikzf1 and Mllt3) to infer the identity and functionality of these cells. Many of these markers are shared among various types of blood progenitors, and only a well-defined combination of markers could offer some assurance of the lymphoid and pre-HSC nature of these cells, although this would still be limited in the absence of functional assays.<br /> The identification of a pre-HSC-like CD45⁺CD41⁻/lo c-Kit⁺VE-Cadherin⁺ cell population is presented as evidence supporting the generation of pre-HSCs by this system, but this claim is questionable. This FACS profile may also be present in progenitors generated in the yolk sac such as early erythro-myeloid progenitors (EMPs). It is only within the AGM context, and in conjunction with further functional assays demonstrating the ability of these cells to differentiate into HSCs and contribute to long-term repopulation, that this profile could be strongly associated with pre-HSCs. In the absence of such data, the cells exhibiting this profile in the current system cannot be conclusively identified as true pre-HSCs.

      At this preliminary response stage, we present 2 additional pieces of evidence to support our claims that we capture YS and AGM stages of haematopoietic development. In future experiments, we can complement these with functional assays, including co-culture with OP9 and OP9-DL stroma.

      Author response image 8.

      EZH2 inhibition affects CD41+ cellular output in haemogenic gastruloids at 144, but not 216h. A. Flow cytometry analysis of CD41 expression in 144h-haemogenic gastruloid treated with 0.5μM EZH2 inhibitor GSK126 from 120h. DMSO (0.05%), vehicle. 1 of 2 independent experiments (average CD41+: DMSO, 21.20%; GSK126, 12.10%; CD45 not detected). B. Flow cytometry analysis of CD41 and CD45 expression in 216h gastruloids, treated with DMSO or GSK216. (DMSO: average CD41+, 15.28%; average CD45+ 0.46%. GSK126: average CD41+, 23.78%; average CD45+, 2.08%).

      In Author response images 5 and 6, we project our single-cell RNA-seq data onto (1) developing intra-embryonic pSP and AGM between E8 and E11 (Author response image 5) and (2) a single-cell RNA-seq study of HE development which combines haemogenic and haematopoietic cells from the YS, the developing HE and IAHC in the AGM, and FL (Author response image 6). Our data maps E8.25-E10 (Author response image 5) and captures YS EMP and erythroid and myeloid progenitors, as well as AGM pre-HE, HE and IAHC, with some cells matching HSPC and LMPP (Author response image 6), as suggested by the projection onto the Thambyrajah et al data set (Fig. S3 in the manuscript).

      Given the difficulty in finding markers that specifically associate with AGM haematopoiesis, we inspected the possibility of capturing different regulatory requirements at different stages of gastruloid development mirroring differential effects in the embryo. Polycomb EZH2 is specifically required for EMP differentiation in the YS, but does not affect AGM-derived haematopoiesis; it is also not required for primitive erythroid cells (PMID: 29555646; PMID: 34857757). We treated haemogenic gastruloids from 120h onwards with either DMSO (0.05%) or GSK126 (0.5μM), and inspected the cellularity of gastruloids at 144h, which we equate with YS-EMP, and 216h – putatively AGM haematopoiesis (Author response image 8). We show that EZH2 inhibition / GSK126 treatment specifically reduces %CD41+ cells at 144h (Author response image 8A), but does not reduce %CD41+ or %CD45+ cells at 216h (Author response image 8B).

      Although preliminary, these data, together with the scRNA-seq projections described, provide evidence to our claim that 144h haemogenic gastruloids capture YS EMPs, while CD41+ and CD45+ cells isolated at 216h reflect AGM progenitors. We cannot conclude as to the functional nature of the AGM cells from this experiment.

      The engraftment data presented are also not fully convincing, as the observed repopulation is very limited and evaluated only at 4 weeks post-transplantation. The cells detected after 4 weeks could represent the progeny of EMPs that have been shown to provide transient repopulation rather than true HSCs.

      We clearly state that there is low level engraftment and do not claim to have generated HSC. We describe cells with short-term engraftment potential. Although the cells we show in the manuscript at 4 weeks could be EMPs (Author response image 7 and Fig. 3 and S3), we now have 8-week analysis of implant recipients, in which we observed, again low-level, engraftment of the recipient bone marrow in 1:3 animals (Author response image 9). This engraftment is myeloid-lymphoid and therefore likely to have originated in a later progenitor. To be clear, we do not claim that this corresponds to the presence of HSC. It nevertheless supports the maturation of progenitors with engraftment potential.

      Author response image 9.

      Flow cytometry BFP engraftment of recipient bone marrow 8-weeks post implantation of 216hhaemogenic gastruloids in the adrenal gland of Nude mice. 1:3 animals show BFP CD45+ engraftment in the myeloid (Mac1+) and B-lymphoid (B220+) lineages. 3 haemogenic gastruloids were implanted unilaterally in the adrenal gland of each animal. A. Engrafted animal, showing CD45+ BFP cells of myeloid (CD11b) and B-lymphoid affiliation (B220). B. Non-engrafted mouse recipient of haemogenic gastruloid implants.

      Reviewer #3 (Public review):<br /> In this study, the authors employ a mouse ES-derived "hemogenic gastruloid" model which they generated and which they claim to be able to deconvolute YS and AGM stages of blood production in vitro. This work could represent a valuable resource for the field. However, in general, I find the conclusions in this manuscript poorly supported by the data presented. Importantly, it isn't clear what exactly are the "YS" and the "AGM"-like stages identified in the culture and where is the data that backs up this claim. In my opinion, the data in this manuscript lack convincing evidence that can enable us to identify what kind of hematopoietic progenitor cells are generated in this system. Therefore, the statement that "our study has positioned the MNX1-OE target cell within the YS-EMP stage (line 540)" is not supported by the evidence presented in this study. Overall, the system seems to be very preliminary and requires further optimization before those claims can be made.<br /> Specific comments below:<br /> (1) The flow cytometric analysis of gastruloids presented in Figure 1 C-D is puzzling. There is a large % of c-Kit+ cells generated, but few VE-Cad+ Kit+ double positive cells. Similarly, there are many CD41+ cells, but very few CD45+ cells, which one would expect to appear toward the end of the differentiation process if blood cells are actually generated. It would be useful to present this analysis as consecutive gating (i.e. evaluating CD41 and CD45 within VE-Cad+ Kit+ cells, especially if the authors think that the presence of VE-Cad+ Kit+ cells is suggestive of EHT). The quantification presented in D is misleading as the scale of each graph is different.

      Fig. 1C-D provide an overview of haemogenic markers during the timecourse of haemogenic gastruloid differentiation, and does indeed show a late up-regulation of CD45, as the Reviewer points out would be expected. The %CD45+ cells is indeed low. However, we should point out that the haemogenic gastruloid protocol, although biased towards mesodermal outputs, does not aim to achieve pure haematopoietic specification, but rather place it in its embryo-like context. Consecutive gating at the 216h-timepoint is shown and quantified in Fig. 3A-B. We refute that the scale is misleading. It is a necessity to represent the data in a way that is interpretable by the reader: the gates (in C) are truly representative and annotated, as are the plot axes (in D).

      (2) The imaging presented in Figure 1E is very unconvincing. C-Kit and CD45 signals appear as speckles and not as membrane/cell surfaces as they should. This experiment should be repeated and nuclear stain (i.e. DAPI) should be included.

      We include the requested images below (Author response image 10).

      Author response image 10.

      Confocal images of haematopoietic production in haemogenic gastruloids. Wholemount, cleared haemogenic gastruloids were stained for CD45 (pseudo-coloured red) and c-Kit antigens (pseudo-coloured yellow) with indirect staining, as described in the manuscript. Flk1-GFP signal is shown in green. Nuclei are contrasted with DAPI. (A) 192h. (B) 216h.

      (3) Overall, I am not convinced that hematopoietic cells are consistently generated in these organoids. The authors should sort hematopoietic cells and perform May-Grunwald Giemsa stainings as they did in Figure 6 to confirm the nature of the blood cells generated.

      It is factual that the data are reproducible and complemented by functional assays shown in Fig. 3, which clearly demonstrate haematopoietic output. The single-cell RNA-seq data also show expression of a haematopoietic programme. Nevertheless, we include Giemsa-Wright’s stained cytospins obtained at 216h to illustrate haematopoietic output (Reviewer Fig. 11). Inevitably, the cytospins will be inconclusive as to the presence of endothelial-to-haematopoietic transition or the generation of haematopoietic stem/progenitor cells, as these cells do not have a distinctive morphology.

      Author response image 11.

      Cytospin of dissociated haemogenic gastruloids at 216h. Cytospins were stained with Giemsa-Wright’s stain and are visualised with a 40x objective. Annotated are cells in the monocytic (dashed open arrow), granulocytic (solid open arrow), megakaryocytic (solid arrow) and erythroid (asterisk) lineages; arrowheads indicate cells with a non-specific blast-like morphology. Representative image.

      (4) The scRNAseq in Figure 2 is very difficult to interpret. Specific points related to this:<br /> - Cluster annotation in Figure 2a is missing and should be included.<br /> - Why do the heatmaps show the expression of genes within sorted cells? Couldn't the authors show expression within clusters of hematopoietic cells as identified transcriptionally (which ones are they? See previous point)? Gene names are illegible.<br /> - I see no expression of Hlf or Myb in CD45+ cells (Figure 2G). Hlf is not expressed by any of the populations examined (panels E, F, G). This suggests no MPP or pre-HSC are generated in the culture, contrary to what is stated in lines 242-245. (PMID 31076455 and 34589491).<br /> Later on, it is again stated that "hGx cells... lacked detection of HSC genes like Hlf, Gfi1, or Hoxa9" (lines 281-283). To me, this is proof of the absence of AGM-like hematopoiesis generated in those gastruloids.

      Author response image 12.

      Expression of endothelial, haemogenic and haematopoietic genes in haemogenic gastruloid cells sorted at 144h, 192h and 216h. UMAP as in Author response image 4. Pecam (CD31) and CD34 represent endothelial genes also detected in haemogenic endothelium. CD44 is specifically enriched at the endothelial-to-haemogenic transition. Mecom is detected in haemogenic endothelium and haematopoietic progenitors. Mllt3 and Runx1 are haematopoietic markers. Hoxa9 and Hlf are associated with haematopoietic stem and progenitor cells and their detection is rare in haemogenic gastruloids at 216h.

      For a combination of logistic and technical reasons, we performed single-cell RNA-seq using the Smart-Seq2 platform, which is inherently low throughput. We overcame the issue of cell coverage by complementing whole-gastruloid transcriptional profiling at successive time-points with sorting of subpopulations of cells based on individual markers documented in Fig. 1. We clearly stated which platform was used as well as the number and type of cells profiled (Fig. S2A and lines 172-179 of the manuscript), and our approach is standard. We will review our representation of the data in a revised manuscript. Nevertheless, at this stage, we provide plots of the expression of key haematopoietic markers over UMAPs of haemogenic gastruloid timecourse (Author response image 12). We also show preliminary qRT-PCR data with increased Hlf expression upon extension of the protocol to 264h (Author response image 13), further confirming haematopoietic specification, including of candidate definitive progenitor cells, in the haemogenic gastruloid model.

      Author response image 13.

      Hlf expression is up-regulated in late stage haemogenic gastruloids. Quantitative RT-PCR analysis of Hlf expression in unfractionated haemogenic gastruloids cultured for 264h. From 168h onwards, haemogenic gastruloids were cultured in N2B27 in the presence of VEGF, SCF, FLT3L and TPO, all recombinant mouse cytokines, as described in the manuscript. Shown are mean±standard deviation of n=5 replicates from 2 mouse ES cell lines, respectively Flk1-GFP and Rosa26-BFP::Flk1-GFP, reported in the manuscript; 2-tailed unpaired t-test with Welch correction.

      (5) Mapping of scRNA-Seq data onto the dataset by Thambyrajah et al. is not proof of the generation of AGM HE. The dataset they are mapping to only contains AGM cells, therefore cells do not have the option to map onto something that is not AGM. The authors should try mapping to other publicly available datasets also including YS cells.

      We have done this and the data are presented in Author response image 5 and 6. As detailed in response to Reviewer 1, we have conducted projections of our single-cell RNA-seq data against two studies which (1) capture arterial and haemogenic specification in the para-splanchnopleura (pSP) and AGM region between E8.0 and E11 (Hou et al, PMID: 32203131) (Author response image 5), and (2) uniquely capture YS, AGM and FL progenitors and the AGM endothelial-to-haematopoietic transition (EHT) in the same scRNA-seq dataset (Zhu et al, PMID: 32392346) (Author response image 6). Specifically in answering the Reviewers’ point, we show that different subsets of haemogenic gastruloid cells sorted on haemogenic surface markers c-Kit, CD41 and CD45 cluster onto pre-HE and HE, intra-aortic clusters and FL progenitor compartments, and to YS EMP and erythroid and myeloid progenitors. This lends support to our claim that the haemogenic gastruloid system specifies both YS-like and AGM-like cells.

      (6) Conclusions in Figure 3, named "hGx specify cells with preHSC characteristics" are not supported by the data presented here. Again, I am not convinced that hematopoietic cells can be efficiently generated in this system, and certainly not HSCs or pre-HSCs.

      We have provided evidence, both in the manuscript and in this response to Reviewers, that there is haematopoietic specification, including of progenitor cells, in the haemogenic gastruloid system (Fig. 3 and Author response image 7,9). We have added data in this response that supports the specification of YS-like and AGM-like cells (Author response image 5-6, 8). Importantly, we have never claimed that haemogenic gastruloids generate HSC. We accept the Reviewer’s comment that we have not provided sufficient evidence for the specification of pre-HSC-like cells. We will re-phrase Fig. 3 conclusion as “Haemogenic gastruloids specify cells with characteristics of definitive haematopoietic progenitors”.

      - FACS analysis in 3A is again very unconvincing. I do not think the population identified as c-Kit+ CD144+ is real. Also, why not try gating the other way around, as commonly done (e.g. VE-Cad+ Kit+ and then CD41/CD45)?

      There is nothing unconventional about our gating strategy, which was done from a more populated gate onto the less abundant one to ensure that the results are numerically more robust. In the case of haemogenic gastruloids, unlike the AGM preparations the Reviewer may be referring to, CD41 and CD45+ cells are more abundant as there is no circulation of more differentiated haematopoietic cells away from the endothelial structures. This said, we did perform the gating as suggested (Author response image 14), indeed confirming that most VE-cad+ Kit+ cells are CD45+. Interestingly VE-cad+Kit- are predominantly CD41+, reinforcing the true haematopoietic nature of these cells.

      Author response image 14.

      Flow cytometry analysis of VE-cadherin+ cells in haemogenic gastruloids at 216h of the differentiation protocol, probing co-expression of CD45, CD41 and c-Kit.

      - The authors must have tried really hard, but the lack of short- or long-engraftment in a number of immunodeficient mouse models (lines 305-313) really suggests that no blood progenitors are generated in their system. I am not familiar with the adrenal gland transplant system, but it seems like a very non-physiological system for trying to assess the maturation of putative pre-HSCs. The data supporting the engraftment of these mice, essentially seen only by PCR and in some cases with a very low threshold for detection, are very weak, and again unconvincing. It is stated that "BFP engraftment of the Spl and BM by flow cytometry was very low level albeit consistently above control (Fig. S4E)" (lines 337-338). I do not think that two dots in a dot plot can be presented as evidence of engraftment.

      We have presented the data with full disclosure and do not deny that the engraftment achieved is low-level and short-term, indicating incomplete maturation of definitive haematopoietic progenitors in the current haemogenic gastruloid system. However, we call the Reviewer’s attention to the fact that detection of BFP+ cells by PCR and flow cytometry in the recipient animals at 4 weeks is consistent between the 2 methods (Author response image 7).

      Furthermore, we have now also been able to detect low-level myelo-lymphoid engraftment in the bone marrow 8 weeks after adrenal implantation, again suggesting the presence of a small number of definitive haematopoietic progenitors that potentially mature from the 3 haemogenic gastruloids implanted (Author response image 9).

      (7) Given the above, I find that the foundations needed for extracting meaningful data from the system when perturbed are very shaky at best. Nevertheless, the authors proceed to overexpress MNX1 by LV transduction, a system previously shown to transform fetal liver cells, mimicking the effect of the t(7;12) AML-associated translocation. Comments on this section:<br /> - The increase in the size of the organoid when MNX1 is expressed is a very unspecific finding and not necessarily an indication of any hematopoietic effect of MNX1 OE.

      We agree with the Reviewer on this point; it is nevertheless a reproducible observation which we thought relevant to describe for completeness and data reproducibility.

      - The mild increase of cKit+ cells (Figure 4E) at the 144hr timepoint and the lack of any changes in CD41+ or CD45+ cells suggests that the increase in Kit+ cells % is not due to any hematopoietic effect of MNX1 OE. No hematopoietic GO categories are seen in RNA seq analysis, which supports this interpretation. Could it be that just endothelial cells are being generated?

      The Reviewer is correct that the MNX1-overexpressing cells have a strong endothelial signature, which is present in the patients (Fig. 4A). We investigated a potential link with c-Kit by staining cells from the replating colonies during the process of in vitro transformation with CD31. We observed that 40-50% of c-Kit+ cells (20-30% total colony cells) co-expressed CD31(Author response image 15), at least at early plating. These cells co-exist with haematopoietic cells, namely Ter119+ cells, as expected from the YS-like erythroid and EMP-like affiliation of haematopoietic output from 144h-haemogenic gastruloids (Fig. 5F).

      Author response image 15.

      Endothelial affiliation of MNX1-oe replating cells from haemogenic gastruloid. A. Representative flow cytometry plot of plate 1 CFC from MNX1-overexpressing haemogenic gastruloids at 144h. B. Quantification of the proportion of CD31+c-Kit+ cells in plates 1 and 2 of MNX1-oe-driven in vitro transformation.

      (8) There seems to be a relatively convincing increase in replating potential upon MNX1-OE, but this experiment has been poorly characterized. What type of colonies are generated? What exactly is the "proportion of colony forming cells" in Figures 5B-D? The colony increase is accompanied by an increase in Kit+ cells; however, the flow cytometry analysis has not been quantified.

      Given the inability to replate control EV cells, there is not a population to compare with in terms of quantification. The level of c-Kit+ represented in Fig. 5E is achieved at plate 2 or 3 (depending on the experiment), both of which are significantly enriched for colony-forming cells relative to control (Fig. 5B, D).

      (9) Do hGx cells engraft upon MNX1-OE? This experiment, which appears not to have been performed, is essential to conclude that leukemic transformation has occurred.

      For the purpose of this study, we are satisfied with confirmation of in vitro transformation potential of MNX1 haemogenic gastruloids, which can be used for screening purposes. Although interesting, in vivo leukaemia engraftment from haemogenic gastruloids is beyond the scope of this study.

    1. Author response:

      We would like to thank the three reviewers for the careful review and thoughtful comments on our manuscript. In addition to providing useful suggestions, they uncovered some embarrassing oversights on our part, related to experimental details including number of embryos, and quantification of variance in the observed changes for some of the experiments, which were inadvertently omitted in the submission. We provide below an initial response to the reviewer’s public reviews and expect to submit a revised manuscript comprehensively addressing all their concerns.

      I would like to start by addressing some of their most critical comments related to validation of the tools used to reduce soxB1 gene family function in the embryo.  In the absence of the critical supplementary data that we inadvertently failed to include, the reviewers were left with an understandable, but we feel erroneous impression, that there was insufficient validation of mutant and knockdown tools. 

      Reviewer #2 says “The sox2y589 mutant line is not properly verified in this manuscript, which could be done by examining ant-Sox2 antibody labeling, Western blot analysis or…”

      This validation, which had been performed previously both with antibody staining and with western blot analysis, was inadvertently omitted from the supplementary data submitted with the paper. The western blot data is shown here.

      Author response image 1.

      Validation of sox2 mutant phenotype with Western blot.

      Lysates were prepared from 25 embryos selected as wild type or potentially mutant based on the “loss of L1” phenotype at 6 dpf. This polyclonal antibody recognizes within the last 16 amino acids of the C-terminal.

      Author response image 2.

      Validation of sox2 mutant phenotype with antibody staining.

      Though in this experiment there was considerable background in the red channel, and it shows the lateral line nerve, loss of nuclear Sox2 expression is evident in the deposited neuromast of an embryo identified as a mutant based on its delayed deposition of the L1 neuromast.

      This data and a repeat of the antibody staining showing the primordium with loss of Sox2 will be included in a revised manuscript.

      Furthermore, Reviewer #2 comments “the authors show that the anti-Sox2 and antiSox3 antibody labeling is reduced but not absent in sox2 MO1 and sox3 MO-injected embryos, but do not show antibody labeling of the sox2 MO and sox3 MO-double injected embryos to determine if there is an additional knockdown”

      This will be included in a revised manuscript.

      Reviewer #2:

      The authors acknowledge that the sox2 MO1 used in this manuscript also alters sox3 function, but do not redo the experiments with a specific sox2 MO

      This is not exactly true. Having discovered sox2 MO1 simultaneously reduces sox2 and sox3 function, three new morpholinos were obtained based on another paper (Kamachi et al 2008), which had quantitatively assessed efficacy of three sox2 specific morpholinos (sox2 MO2, sox2 MO3, and sox2 MO4). The effects of these morpholinos on the pattern of L1 deposition was compared to that of sox2 MO1. This comparison was shown in supplementary Figure 2 and is included below. It shows that the sox2 specific morpholinos resulted in a poorly penetrant delay in deposition of L1, comparable to that of a sox2 mutant, which was quantified in supplementary Figure 3B. The observations with these three sox2 specific morpholinos independently supported the observations made with the sox2 mutant that reduction of sox2 on its own results in a delay in deposition of the first neuromast with low penetrance and that to effectively examine the role of these SoxB1 genes in the primordium their function needs to be compromised in a combinatorial manner. A conclusion that was independently supported by observations made by crossing sox1a, sox2 and sox3 mutants (Figure 3 and Supplementary Figure 3). Therefore, even though the initial use of a sox2 morpholino, which simultaneously knocks down sox3, was unintentional, its use turned out to be useful. It allowed us to examine effects of knocking down sox2 and sox3 with a single morpholino. Furthermore, though this project was initiated more than 15 years ago to specifically understand sox2 function, our focus had shifted to understanding the role of soxB1 family members sox1a, sox2 and sox3 functioning together as an interacting system that regulates Wnt activity in the primordium. Considering this broader focus, reflected in the title of the paper, it was not a priority to repeat every experiment previously done with the sox2MO1 with the new sox2 specific morpholinos. Instead, having acknowledged the “limitations” of sox2MO1, we used it to better understand effects of combinatorial reduction of SoxB1 function.

      Reviewer #1:

      It is not exactly clear what underlies the apparent redundancy. It would be helpful if the soxb gene family member expression was reported after loss of each.

      As suggested by reviewer #1, we had previously looked changes in expression of each of the soxB1 factors following loss of individual soxB1 factors but not included it in the supplementary data with the original submission. Independent of a reproducible and consistent expansion sox1a expression into the trailing zone, following loss of sox2 function, which is reported in the paper and quantified here where 10/10 mutant embryos showed the expansion (compare region within bracket in WT and sox2<sup>-/-</sup>), no consistent changes in the expression of other soxB1 family members was observed as part of a mechanism that might account for compensation when function of a particular soxB1 factor is soxB1 factor is lost. The data shown above together with more extensive quantification of changes will be included in a revised version of the manuscript. At this time the only consistent change was the expansion of sox1a to the trailing zone when lost. The data trailing zone when sox2 function is lost. This change reflects dependence of sox1a on Wnt activity and the fact that Wnt activity expands into the trailing zone when sox2 function is lost.  

      Author response image 3.

      Reviewer #3:

      Given that the expression patterns of Sox1a and Sox3 are not merely different but are largely reciprocal, the mechanistic basis of their very similar double mutant phenotypes with Sox2 remains opaque.

      The simplest way to think about compensation for gene function in a network is to think of it being determined by expression of a homolog or another gene with a similar function being expressed in a similar or overlapping domain.  However, it is more useful to think of Sox2 function in the primordium as part of a interacting network of SoxB1 factors whose differential regulatory mechanisms create a robust system that simultaneously regulates two key aspects of Wnt activity in the primordium; how high Wnt activity is allowed to get in the leading zone and how effectively it is shut off to facilitate protoneuromast maturation in the trailing zone. These features of Wnt activity influence both when and where nascent protoneuromasts will form in the wake of a progressively shrinking Wnt system and where they undergo effective maturation and stabilization prior to deposition. Changes in individual SoxB1 expression patterns provide some hints about how some SoxB1 factors may compensate when function of one or more of these factors is compromised. However, a deeper understanding of robustness and “compensation” will require a systems level understanding of this gene regulatory network with computational models, which we are currently working on in our group. It remains possible, for example, that how far into the trailing zone the Wnt activity has an influence is regulated at least in part by how high it is allowed to get in the leading zone by sox1a. Conversely, how high Wnt activity gets in the leading zone may be influenced by how effectively it is shut off in the trailing zone by sox2 and sox3, as this influences the size of the Wnt system, which in turn can influence the overall level of Wnt activity. In this manner Sox1a may cooperate with Sox2 and Sox3 to limit both how high Wnt activity is allowed to get in the primordium and to effectively shut it off in the trailing zone.

      Reviewer #3:

      Related to this, the authors discuss that Sox1a/Sox2 double knockdown produces a more severe phenotype than Sox2/Sox3 double knockdown, yet this difference is not obviously reflected in the data.

      The severity of the sox1a/sox2 double mutant phenotype compared to that of the sox2/sox3 double mutant is shown in Figure 3 K and N, and quantified in Supplementary Figure 3A. Simultaneous loss of sox2 and sox3 results in a small but relatively penetrant delay in where the first stable neuromast is deposited (Figure 2 N). By contrast, loss of sox2 and sox1a together consistently results in a longer delay in deposition of the first stable (Figure 2 K). A new graph, shown below, which will be incorporated in the revised paper, shows that there is a significant difference in the pattern of L1 deposition in sox1a<sup>-/-</sup>, sox2<sup>-/-</sup> and sox2<sup>-/-</sup>, sox3<sup>-/-</sup> double mutants. 

      Author response image 4.

      All 3 datasets found to be normally distributed by Shapiro-Wilk test. 1-way ANOVA showed significance (<0.0001), with Tukey’s multiple comparisons test showing significant difference between all 3 conditions. (***p=0.0008, ****p<0.0001)

      Reviewer #1:

      It would be good to more clearly state why sox3 is not regulated by Wnt given its expression is inhibited by the delta TCF construct (Figure 2M).

      The explanation for why we believe sox3 expression is determined by Fgf signaling, and not Wnt activity requires integrating what is observed both with induction of the delta TCF construct and the dominant negative Fgf receptor (DN FgfR). Loss of sox3 expression with induced expression of the delta TCF construct could result from loss of Wnt activity or the downstream loss of Fgf activity, which is ultimately dependent on Fgfs secreted by Wnt active cells in the leading domain. Distinguishing between these possibilities is based on inhibition of FGF signaling with the DN FgfR, described in the next paragraph. Heat Shock induced expression of DN FgfR expression results in loss of FGF signaling and the simultaneous expansion of Wnt activity into the trailing zone. As explained in the original text, loss of sox3 expression in this context, rather than its expansion, suggests its expression is determined by Fgf signaling not Wnt activity. We will emphasize that its loss, rather than its expansion, following induction of DN FgfR, indicates its expression is determined by Fgf signaling not Wnt activity.

      Reviewer #2:

      The manuscript lacks quantification of many of the experiments, making it difficult to conclude their significance.

      One of the biggest inadvertent omissions of the paper was the inadequate quantification of some of the results. Quantification of results with considerable variation in the outcome, like the pattern of L1 deposition,  was provided following manipulations where various combinations of sox1a, sox2, and sox3 function was lost (Figures 3, supplementary Figures 2 and 3) or where sox2MO1/sox3MO was used with or without IWR (Figure 5 and Figure 6). However, numbers for the experiments in Figures 2 were omitted in the Figure legend, where typically about 10 embryos for each manipulation were photographed, scored, and a representative image was used to make the figure. In these experiments  there was a very consistent result with 100% of the embryos showing changes represented by each panel in Figure 2. The only exception was Figure 2Y where 9/10 embryos showed the described change. Similarly in Figure 4 there was a consistent result and 100% of embryos showed the change shown. Numbers and statistics for these results will be included in a revised manuscript.

      Reviewer #2:

      The statistical analysis in Figure 5 and Supplementary Figures 2 and 3 should be one-way ANOVA or Kruskal-Wallis with a Dunn's multiple comparisons test rather than pair-wise comparisons.

      The analysis has been re-done following the reviewer’s suggestions. The analysis confirms the primary conclusions of the original submission, and this analysis will be incorporated in a revised manuscript. However, to improve the power of the analysis, experiments with low numbers of embryos will be repeated.

      See redone graphs in Figure 5 and supplementary Figure 2 and 3.

    1. Author response:

      Reviewer #1 (Public Review):

      Summary:

      For many years, there has been extensive electrophysiological research investigating the relationship between local field potential patterns and individual cell spike patterns in the hippocampus. In this study, using state-of-the-art imaging techniques, they examined spike synchrony of hippocampal cells during locomotion and immobility states. In contrast to conventional understanding of the hippocampus, the authors demonstrated that hippocampal place cells exhibit prominent synchronous spikes locked to theta oscillations.

      Strengths:

      The voltage imaging used in this study is a highly novel method that allows recording not only suprathreshold-level spikes but also subthreshold-level activity. With its high frame rate, it offers time resolution comparable to electrophysiological recordings. Moreover, it enables the visualization of actual cell locations, allowing for the examination of spatial properties (e.g., Figure 4G).

      We thank the reviewer for pointing out the technical novelty of this work.

      Weaknesses:

      There is a notable deviation from several observations obtained through conventional electrophysiological recordings. Particularly, as mentioned below in detail, the considerable differences in baseline firing rates and no observations of ripple-triggered firing patterns raise some concerns about potential artifacts from imaging and analysis, such as cell toxicity, abnormal excitability, and false detection of spikes. While these findings are intriguing if the validity of these methods is properly proven, accepting the current results as new insights is challenging.

      We appreciate the reviewer’s insightful comments regarding the intriguing aspect of our findings. Indeed, the emergence of a novel form of CA1 population synchrony presents exciting implications for hippocampal memory research and beyond.

      While we acknowledge the deviations from conventional electrophysiological recordings, we respectfully contend that these differences do not necessarily imply methodological flaws. All experiments and analyses were conducted with meticulous adherence to established standards in the field.

      Regarding the observed variations in averaging firing rates, it is important to note the well-documented heterogeneity in CA1 pyramidal neuron firing rates, spanning from 0.01 to 10 Hz, with a skewed distribution toward lower frequencies (Mizuseki et al., 2013). Our exclusion criteria for neurons with low estimated firing rates may have inadvertently biased the selection towards more active neurons. Moreover, prior research has indicated that averaging firing rates tend to increase during exposure to novel environments (Karlsson et al., 2008), and among deep-layer CA1 pyramidal neurons (Mizuseki et al., 2011). Given our recording setup in a highly novel environment and the predominance of deep CA1 pyramidal neurons in our sample, the observed higher averaging firing rates could be influenced by these factors. Considering these points, our mean firing rates (3.2 Hz) are reasonable estimations compared to previously reported values obtained from electrophysiological recordings (2.1 Hz in McHugh et al., 1996 and 2.4-2.6 Hz in Buzsaki et al., 2003).

      Regarding concerns about potential cell toxicity, previous studies have shown that Voltron expression and illumination do not significantly alter membrane resistance, membrane capacitance, resting membrane potentials, spike amplitudes, and spike width (see Abdelfattah 2019, Science, Supplementary Figure 11 and 12). In our recordings, imaged neurons exhibit preserved membrane and dendritic morphology during and after experiments (Author response image 1), supporting the absence of significant toxicity.

      Author response image 1.

      Voltron-expressing neurons exhibit preserved membrane and dendritic morphology. (A) Images of two-photon z-stack maximum intensity projection showing Voltron-expressing neurons taken after voltage image experiments in vivo. (B) Post-hoc histological images of neurons being voltage-imaged.

      Regarding spike detection, we use validated algorithms (Abdelfattah et al., 2019 and 2023) to ensure robust and reliable detection of spikes. Spiking activity was first separated from slower subthreshold potentials using high-pass filtering. This way, a slow fluorescence increase will not be detected as a spike, even if its amplitude is large. We benchmarked the detection algorithm in computer simulation. The sensitivity and specificity of the algorithm exceed 98% at the level of signal-to-noise ratio of our recordings. While we acknowledge that a small number of spikes, particularly those occurring later in a burst, might be missed due to their smaller amplitudes (as illustrated in Figure 1 and 2 of the manuscript), we anticipate that any missed spikes would lead to a decrease rather than an increase in synchrony between neurons. Overall, we are confident that spike detection is performed in a rigorous and robust manner.

      To further strengthen these points, we will include the following in the revision:

      (1) Histological images of recorded neurons during and after experiments.

      (2) Further details regarding the validation of spike detection algorithms.

      (3) Analysis of publicly available electrophysiological datasets.

      (4) Discussion regarding the reasons behind the novelty of some of our findings compared to previous observations.

      In conclusion, we assert that our experimental and analysis approach upholds rigorous standards. We remain committed to reconciling our findings with previous observations and welcome further scrutiny and engagement from the scientific community to explore the intriguing implications of our findings.

      Reviewer #2 (Public Review):

      Summary:

      This study employed voltage imaging in the CA1 region of the mouse hippocampus during the exploration of a novel environment. The authors report synchronous activity, involving almost half of the imaged neurons, occurred during periods of immobility. These events did not correlate with SWRs, but instead, occurred during theta oscillations and were phased-locked to the trough of theta. Moreover, pairs of neurons with high synchronization tended to display non-overlapping place fields, leading the authors to suggest these events may play a role in binding a distributed representation of the context.

      We thank the reviewer for a thorough and thoughtful review of our paper.

      Strengths:

      Technically this is an impressive study, using an emerging approach that allows single-cell resolution voltage imaging in animals, that while head-fixed, can move through a real environment. The paper is written clearly and suggests novel observations about population-level activity in CA1.

      We thank the reviewer for pointing out the technical strength and the novelty of our observations.

      Weaknesses:

      The evidence provided is weak, with the authors making surprising population-level claims based on a very sparse data set (5 data sets, each with less than 20 neurons simultaneously recorded) acquired with exciting, but less tested technology. Further, while the authors link these observations to the novelty of the context, both in the title and text, they do not include data from subsequent visits to support this. Detailed comments are below:

      We understand the reviewer’s concerns regarding the size of the dataset. Despite this limitation, it is important to note that synchronous ensembles beyond what could be expected from chance (jittering) were detected in all examined data. In the revision, we plan to add more data, including data from subsequent visits, to further strengthen our findings.

      (1) My first question for the authors, which is not addressed in the discussion, is why these events have not been observed in the countless extracellular recording experiments conducted in rodent CA1 during the exploration of novel environments. Those data sets often have 10x the neurons simultaneously recording compared to these present data, thus the highly synchronous firing should be very hard to miss. Ideally, the authors could confirm their claims via the analysis of publicly available electrophysiology data sets. Further, the claim of high extra-SWR synchrony is complicated by the observation that their recorded neurons fail to spike during the limited number of SWRs recorded during behavior- again, not agreeing with much of the previous electrophysiological recordings.

      We understand the reviewer’s concern. We will examine publicly available electrophysiology datasets to gain further insights into any similarities and differences to our findings. Based on these results, we will discuss why these events have not been previously observed/reported.

      (2) The authors posit that these events are linked to the novelty of the context, both in the text, as well as in the title and abstract. However, they do not include any imaging data from subsequent days to demonstrate the failure to see this synchrony in a familiar environment. If these data are available it would strengthen the proposed link to novelty if they were included.

      We thank the reviewer’s constructive suggestion. We will acquire more datasets from subsequent visits to gain further insights into these synchronous events.

      3) In the discussion the authors begin by speculating the theta present during these synchronous events may be slower type II or attentional theta. This can be supported by demonstrating a frequency shift in the theta recording during these events/immobility versus the theta recording during movement.

      We thank the reviewer’s constructive suggestion. We did demonstrate a frequency shift to a lower frequency in the synchrony-associated theta during immobility than during locomotion (see Fig. 4B, the red vs. blue curves). We will enlarge this panel and specifically refer to it in the corresponding discussion paragraph.

      (4) The authors mention in the discussion that they image deep-layer PCs in CA1, however, this is not mentioned in the text or methods. They should include data, such as imaging of a slice of a brain post-recording with immunohistochemistry for a layer-specific gene to support this.

      We thank the reviewer’s constructive suggestion. We do have images of brain slices post-recordings (Author response image 2). Imaged neurons are clearly located in the deep CA1 pyramidal layer. We will add these images and quantification in the revised manuscript.

      Author response image 2.

      Imaged neurons are located in the deep pyramidal layer of the dorsal hippocampal CA1 region.

      Reviewer #3 (Public Review):

      Summary:

      In the present manuscript, the authors use a few minutes of voltage imaging of CA1 pyramidal cells in head-fixed mice running on a track while local field potentials (LFPs) are recorded. The authors suggest that synchronous ensembles of neurons are differentially associated with different types of LFP patterns, theta and ripples. The experiments are flawed in that the LFP is not "local" but rather collected in the other side of the brain, and the investigation is flawed due to multiple problems with the point process analyses. The synchrony terminology refers to dozens of milliseconds as opposed to the millisecond timescale referred to in prior work, and the interpretations do not take into account theta phase locking as a simple alternative explanation.

      We genuinely appreciate the reviewer’s feedback and acknowledge the concerns raised. However, we believe these concerns can be effectively addressed without undermining the validity of our conclusions. With this in mind, we respectfully disagree with the assessment that our experiments and investigation are flawed. Please allow us to address these concerns and offer additional context to support the validity of our study.

      Weaknesses:

      The two main messages of the manuscript indicated in the title are not supported by the data. The title gives two messages that relate to CA1 pyramidal neurons in behaving head-fixed mice: (1) synchronous ensembles are associated with theta (2) synchronous ensembles are not associated with ripples.

      There are two main methodological problems with the work:

      (1) Experimentally, the theta and ripple signals were recorded using electrophysiology from the opposite hemisphere to the one in which the spiking was monitored. However, both signals exhibit profound differences as a function of location: theta phase changes with the precise location along the proximo-distal and dorso-ventral axes, and importantly, even reverses with depth. And ripples are often a local phenomenon - independent ripples occur within a fraction of a millimeter within the same hemisphere, let alone different hemispheres. Ripples are very sensitive to the precise depth - 100 micrometers up or down, and only a positive deflection/sharp wave is evident.

      We appreciate the reviewer’s consideration regarding the collection of LFP from the contralateral hemisphere. While we acknowledge the limitation of this design, we believe that our findings still offer valuable insights into the dynamics of synchronous ensembles. Despite potential variations in theta phases with recording locations and depth, we find that the occurrence and amplitudes of theta oscillations are generally coordinated across hemispheres (Buzsaki et al., Neurosci., 2003). Therefore, the presence of prominent contralateral LFP theta around the times of synchronous ensembles in our study (see Figure 4A of the manuscript) strongly supports our conclusion regarding their association with theta oscillations, despite the collection of LFP from the opposite hemisphere.

      In addition, in our manuscript, we specifically mentioned that the “preferred phases” varied from session to session, likely due to the variability of recording locations (see Line 254-256). Therefore, we think that the reviewer’s concern regarding theta phase variability has already been addressed in the present manuscript.

      Regarding ripple oscillations, while we recognize that they can sometimes occur locally, the majority of ripples occur synchronously in both hemispheres (up to 70%, see Szabo et al., Neuron, 2022; Buzsaki et al., Neurosci., 2003). Therefore, using contralateral LFP to infer ripple occurrence on the ipsilateral side has been a common practice in the field, employed by many studies published in respectable journals (Szabo et al., Neuron, 2022; Terada et al., Nature, 2021; Dudok et al., Neuron, 2021; Geiller et al., Neuron, 2020). Furthermore, our observation that 446 synchronous ensembles during immobility do not co-occur with contralateral ripples, and the remaining 313 ensembles during locomotion are not associated with ripples, as ripples rarely occur during locomotion. Therefore, our conclusion that synchronous ensembles are not associated with ripple oscillations is supported by data.

      (2) The analysis of the point process data (spike trains) is entirely flawed. There are many technical issues: complex spikes ("bursts") are not accounted for; differences in spike counts between the various conditions ("locomotion" and "immobility") are not accounted for; the pooling of multiple CCGs assumes independence, whereas even conditional independence cannot be assumed; etc.

      We acknowledge the reviewer’s concern regarding spike train analysis. Indeed, complex bursts or different behavioral conditions can lead to differences in spike counts that could potentially affect the detection of synchronous ensembles. However, our jittering procedure (see Line 121-132) is designed to control for the variation of spike counts. Importantly, while the jittered spike trains also contain the same spike count variations, we found 7.8-fold more synchronous events in our data compared to jitter controls (see Figure 1G of the manuscript), indicating that these factors cannot account for the observed synchrony.

      To explicitly demonstrate that complex bursts cannot account for the observed synchrony, we have performed additional analysis to remove all latter spikes in bursts and only count the single and the first spikes of bursts. Importantly, we found that this procedure did not change the rate and size of synchronous ensembles, nor did it significantly alter the grand-average CCG (see Author response image 3). The results of this analysis explicitly rule out a significant effect of complex spikes on the analysis of synchronous ensembles.

      Author response image 3.

      Population synchrony remains after the removal of spikes in bursts. (A) The grand-average cross correlogram (CCG) was calculated using spike trains without latter spikes in bursts. The gray line represents the mean grand average CCG between reference cells and randomly selected cells from different sessions. (B) Pairwise comparison of the event rates of population synchrony between spike trains containing all spikes and spike trains without latter spikes in bursts. Bar heights indicate group means (n=10 segments, p=0.036, Wilcoxon signed-rank test). (C) Histogram of the ensemble sizes as percentages of cells participating in the synchronous ensembles.

      Beyond those methodological issues, there are two main interpretational problems: (1) the "synchronous ensembles" may be completely consistent with phase locking to the intracellular theta (as even shown by the authors themselves in some of the supplementary figures).

      We agree with the reviewer that the synchronous ensembles are indeed consistent with theta phase locking. However, it is important to note that theta phase locking alone does not necessarily imply population synchrony. In fact, theta phase locking has been shown to “reduce” population synchrony in a previous study (Mizuseki et al., 2014, Phil. Trans. R. Soc. B.). Thus, the presence of theta phase locking cannot be taken as a simple alternative explanation of the synchronous ensembles.

      To directly assess the contribution of theta phase locking to synchronous ensembles, we have performed a new analysis to randomize the specific theta cycles in which neurons spike, while keeping the spike phases constant. This manipulation disrupts spike co-occurrence while preserving theta phase locking, allowing us to test whether theta phase locking alone can explain the population synchrony, or whether spike co-occurrence in specific cycles is required. The grand-average CCG shows a much smaller peak compared to the original peak (Author response image 4A). Moreover, synchronous event rates show a 4.5-fold decrease in the randomized data compared to the original event rates (Author response image 4B). Thus, the new analysis reveals theta phase locking alone cannot account for the population synchrony.

      Author response image 4.

      Drastic reduction of population synchrony by randomizing spikes to other theta cycles while preserving the phases. (A) The grand-average cross correlogram (CCG) was calculated using original spike trains (black) and randomized spike trains where theta phases of the spikes are kept the same but spike timings were randomly moved to other theta cycles (red). (B) Pairwise comparison of the event rates of population synchrony between the original spike trains and randomized spike trains (n=10 segments, p=0.002, Wilcoxon signed-rank test). Bar heights indicate group means. ** p<0.01

      (2) The definition of "synchrony" in the present work is very loose and refers to timescales of 20-30 ms. In previous literature that relates to synchrony of point processes, the timescales discussed are 1-2 ms, and longer timescales are referred to as the "baseline" which is actually removed (using smoothing, jittering, etc.).

      Regarding the timescale of synchronous ensembles, we acknowledge that it varies considerably across studies and cell types. However, it is important to note that a timescale of dozens, or even hundreds of milliseconds is common for synchrony terminology in CA1 pyramidal neurons (see Csicsvari et al., Neuron, 2000; Harris et al., Science, 2003; Malvache et al., Science, 2016; Yagi et al., Cell Reports, 2023). In fact, a timescale of 20-30 ms is considered particularly important for information transmission and storage in CA1, as it matches the membrane time constant of pyramidal neurons, the period of hippocampal gamma oscillations, and the time window for synaptic plasticity. Therefore, we believe that this timescale is relevant and in line with established practices in the field.

    1. Author response:

      eLife Assessment

      This useful study integrates experimental methods from materials science with psychophysical methods to investigate how frictional stabilities influence tactile surface discrimination. The authors argue that force fluctuations arising from transitions between frictional sliding conditions facilitate the discrimination of surfaces with similar friction coefficients. However, the reliance on friction data obtained from an artificial finger, together with the ambiguous correlative analyses relating these measurements to human psychophysics, renders the findings incomplete.

      Our main goal with this paper was to show that the most common metric, i.e. average friction coefficient—widely used in tactile perception and device design—is fundamentally unsound, and to offer a secondary parameter that is compatible with the fact that human motion is unconstrained, leading to dynamic interfacial mechanics. In contrast with the summary assessment, we also note that the average friction coefficients in our study were not particularly similar, ranging from differences of 0.4 – 1, a typical range seen in most studies. We believe some of the comments originate from a misinterpretation of our statistically significant, but negative correlation between human results and friction coefficients – which leads to the spurious conclusion that nearly identical objects should be very easy to tell apart, thus supporting our central argument for the need of an alternative. We understand the Reviewers wanting to see that we can demonstrate that humans using instabilities in situ. This is seemingly reasonable, but we explain the significant challenges and fundamental unknowns to those experiments. However, we modified our title to reflect our focus on offering an alternative to the average coefficient of friction.

      We do not think it was feasible, at this stage, to demonstrate that humans use friction instabilities through direct manipulation and observation in human participants. In short, there are still several fundamental unknowns: (1) a decision-making model would need to be created, but it is unknown if tactile decision making follows other models, (2) it is further unknown what constitutes “tactile evidence”, though at our manuscript’s conclusion, we propose that friction instabilities are better suited for to be tactile evidence than the averaging of friction coefficients from a narrow range of human exploration (3) in the design of samples, from a friction mechanics and materials perspective, it is not at this point, possible to pre-program surfaces a priori to deliver friction instabilities and instead must be experimentally determined – especially when attempting to achieve this in controlled surfaces that do not create other overriding tactile cues, like macroscopic bumps or large differences in surface roughness. (4) Given that the basis for tactile percepts, like which object feels “rougher” or “smoother” is not sufficiently established and we have seen leads to confusion, it is necessary to use a 3-alternative forced choice task which avoids asking objects along a preset perceptual dimension – a challenge recognized by Reviewer 3. However, this would bring in issues of memory in the decision-making model. (5) The prior points are compounded by the fact that, we believe, tactile exploration must be performed in an unconstrained manner, i.e., without an apparatus generating motion onto a stationary finger. Work by Liu et al. (IEEE ToH, 2024) showed that recreating friction obtained during free exploration onto a stationary finger was uninterpretable by the participants, hinting at the importance of efference copies(1). We believe that each of the above-mentioned issues constitutes a significant advance in knowledge and would require discussion and dissemination with the community. Finally, one of our overarching goals is to create a consistent method to characterize surfaces, and given individual variability in human fingers and motion, a machine-based method that can rapidly, consistently, and sufficiently replicate tactile exploration is needed.

      Finally, we also justify our use of a mock finger to provide a method to characterize surfaces in tactile studies that other researchers could reasonably recreate, without creating a standard around individual humans, considering the variability in finger shape and motion during exploration. We do not believe this is an “either-or” argument, but rather that standardized methods to characterize surfaces and devices are greatly needed in the field. From these standardized methods, like surface roughness, some tabulated values of friction coefficient, or surface energy, etc., the current metrics to parameterize results are largely incapable of capturing the dynamic changes in forces expected during human tactile exploration.

      Our changes to the manuscript (Page 1 & SI Page 1, Title)

      “Alternatives to Friction Coefficient: Role of Frictional Instabilities for Fine Touch Perception”

      Reviewer 1 (Public review):

      Summary:

      In this paper, Derkaloustian et. al look at the important topic of what affects fine touch perception. The observations that there may be some level of correlation with instabilities are intriguing. They attempted to characterize different materials by counting the frequency (occurrence #, not of vibration) of instabilities at various speeds and forces of a PDMS slab pulled lengthwise over the material. They then had humans make the same vertical motion to discriminate between these samples. They correlated the % correct in discrimination with differences in frequency of steady sliding over the design space as well as other traditional parameters such as friction coefficient and roughness. The authors pose an interesting hypothesis and make an interesting observation about the occurrences of instability regimes in different materials while in contact with PDMS, which is interesting for the community to see in the publication. It should be noted that the finger is complex, however, and there are many factors that may be quite oversimplified with the use of the PDMS finger, and the consideration and discounting of other parameters are not fully discussed in the main text or SI. Most importantly, however, the conclusions as stated do not align with the primary summary of the data in Figure 2.

      Strengths:

      The strength of this paper is in its intriguing hypothesis and important observation that instabilities may contribute to what humans are detecting as differences in these apparently similar samples.

      We thank Reviewer 1 for their time on the manuscript, recognizing the approach we took, and offering constructive feedback. We believe that our conclusions, in fact, are supported by the primary summary of the data in Figure 2 but we believe that our use of R<sup>2</sup> could have led to misinterpretation. The trend with friction coefficient and percent correct was indeed statistically significant but was spurious because the slope was negative. In the revision, we add clarifying comments throughout, change from R<sup>2</sup> to r as to highlight the negative trend, and adjust the figures to better focus on friction coefficient.

      Finally, we added a new section to discuss the tradeoffs between using a real human finger versus a mock finger, and which situations may warrant the use of one or the other. In short, for our goal of characterizing surfaces to be used in tactile experiments, we believe a mock finger is more sustainable and practical than using real humans because human fingers are unique per participant, humans move their fingers at constantly changing pressures and velocities, and friction generated during free exploring human cannot be satisfactorily replicated by moving a sample onto a stationary finger. But, we do not disagree that for other types of experiments, characterizing a human participant directly may be more advantageous.

      Weaknesses:

      Comment 1 - The most important weakness is that the findings do not support the statements of findings made in the abstract. Of specific note in this regard is the primary correlation in Figure 2B between SS (steady sliding) and percent correct discrimination. Of specific note in this regard is the primary correlation in Figure 2B between SS (steady sliding) and percent correct discrimination. While the statistical test shows significance (and is interesting!), the R-squared value is 0.38, while the R-squared value for the "Friction Coefficient vs. Percent Correct" plot has an R-squared of 0.6 and a p-value of < 0.01 (including Figure 2B). This suggests that the results do not support the claim in the abstract: "We found that participant accuracy in tactile discrimination was most strongly correlated with formations of steady sliding, and response times were negatively correlated with stiction spikes. Conversely, traditional metrics like surface roughness or average friction coefficient did not predict tactile discriminability."

      We disagree that the trend with friction coefficient suggests the results do not support the claim because the correlation was found to be negative. However, we could have made the comparison more apparent and expanded on this point, given its novelty.

      While the R<sup>2</sup> value corresponding to the “Friction Coefficient vs. Percent Correct” plot is notably higher, our results show that the slope is negative, which would be statistically spurious. This is because a negative correlation between percent correct (accuracy in discriminating surfaces) and difference in friction coefficient means that the more similar two surfaces are (by friction coefficient), the easier it would be for people to tell them apart. That is, it incorrectly concludes that two identical surfaces would be much easier to tell apart than two surfaces with greatly different friction coefficients.

      This is counterintuitive to nearly all existing results, but we believe our samples were well-positioned to uncover this trend by minimizing variability, by controlling multiple physical parameters in the samples, and that the friction coefficient — typically calculated in the field as an average friction coefficient — ignores all the dynamic changes in forces present in elastic systems undergoing mesoscale friction, i.e., human touch, as seen in Fig. 1 in a mock finger and Fig. 3 in a real finger. By demonstrating this statistically spurious trend, we believe this strongly supports our premise that an alternative to friction coefficient is needed in the design of tactile psychophysics and haptic interfaces.

      We believe that this could have been misinterpreted, so we took several steps to improve clarity, given the importance of this finding: we separated the panel on friction coefficient to its own panel, we changed from R<sup>2</sup> to r throughout, and we added clarifying text. We also added a small section focusing on this spurious trend.

      Our changes to the manuscript (Page 10)

      “To compare the value of looking at frictional instabilities, we also performed GLMM fits on common approaches in the field, like a friction coefficient or material property typically used in tactile discrimination, shown in Fig. 2D-E. Interestingly, in Fig. 2D, we observed a spurious, negative correlation between friction coefficient (typically and often problematically simplified as across all tested conditions) and accuracy (r = -0.64, p < 0.01); that is, the more different the surfaces are by friction coefficient, the less people can tell them apart. This spurious correlation would be the opposite of intuition, and further calls into question the common practice of using friction coefficients in touch-related studies. The alternative, two-term model which includes adhesive contact area for friction coefficient(29) was even less predictive (see Fig. S6A of SI). We believe such a correlation could not have been uncovered previously as our samples are minimal in their physical variations. Yet, the dynamic changes in force even within a single sample are not considered, despite being a key feature of mesoscale friction during human touch.

      We investigate different material properties in Fig. 2E. Differences in average roughness R<sub>a</sub> (or other parameters, like root mean square roughness R<sub>rms</sub> (Fig. S6A of SI) did not show a statistically significant correlation to accuracy. Though roughness is a popular parameter, correlating any roughness parameter to human performance here could be moot: the limit of detecting roughness differences has previously been defined as 13 nm on structured surfaces33 and much higher for randomly rough surfaces,(46) all of which are magnitudes larger than the roughness differences between our surfaces. The differences in contact angle hysteresis – as an approximation of the adhesion contributions(47) – do not present any statistically significant effects on performance.”

      Comment 2, Part 1

      Along the same lines, other parameters that were considered such as the "Percent Correct vs. Difference in Sp" and "Percent Correct vs. Difference in SFW" were not plotted for consideration in the SI. It would be helpful to compare these results with the other three metrics in order to fully understand the relationships.

      We have added these plots to the SI. We note that we had checked these relationships and discussed them briefly, but did not include the plot. The plots show that the type of instability was not as helpful as its presence or absence.

      Our changes to the manuscript (Page 9)

      “Furthermore, a model accounting for slow frictional waves alone specifically shows a significant, negative effect on performance (p < 0.01, Fig. S5 of SI), suggesting that in these samples and task, the type of instability was not as important.”

      Added (SI Page 4)

      “and no correlation between accuracy and stiction spikes (Fig. S5).”

      Comment 2, Part 2

      Other parameters such as stiction magnitude and differences in friction coefficient over the test space could also be important and interesting.

      We agree these are interesting and have thought about them. We are aware that others, like Gueorguiev et al., have studied stiction magnitudes, and though there was a correlation, the physical differences in surface roughness (glass versus PMMA) investigated made it unclear if these could be generalized further(2). We are unsure how to proceed here with a satisfactory analysis of stiction magnitude, given that stiction spikes are not always generated. In fact, Fig. 1 shows that for many velocities and pressures, they do not form. However, we offer some speculation on why stiction spikes may be overrepresented in the literature because:

      (1) They are prone to being created if the finger was loaded for a long time onto a surface prior to movement, thus creating adhesion by contact aging which is unlike active human exploration. We avoid this by discarding the first pull in our measurements, and is a standard practice in mechanical characterization if contact aging needs to be avoided.

      (2) The ranges of velocities and pressures explored were small.

      (3) In an effort to generate strong tactile stimuli, highly adhesive or rough surfaces are used.

      (4) They are visually distinctive on a plot, but we are unaware of any mechanistic reason that mechanoreceptors would be extremely sensitive to this low frequency event over other signals.

      In ongoing work, however, we are always cognizant that if stiction spikes are a dominant factor, then a secondary analysis on their magnitude would be important.

      We interpret “difference in friction coefficient over the test space” to be, for a single surface, like C4, to find the highest average friction for a condition of single velocity and mass and subtract that from the lowest average friction for a condition of single velocity and mass. We calculated the difference in friction coefficient in the typical manner of the field, by averaging all data collected at all velocities and masses and assigning a single value for all of a surface, like C4. We had performed this, and have the data, but we are wary of overinterpreting secondary and tertiary metrics because they do not have any fundamental basis in traditional tribology, and this value, if used by humans, would suggest that they rapidly explore a large parameter space to find a “maximum” and “minimum” friction. Furthermore, the range in friction across the test space, after averaging, may in fact, be smaller than the range of friction in a single measurement. For example, in Fig. 1B, the friction coefficient can be calculated by dividing the data by the normal force ([applied mass + 6 g finger] × gravity). The friction coefficient in a single run varies widely, as expected.

      Fig. 2D shows a GLMM fit between percent correct responses across our pairs and the differences in friction coefficient for each pair, where we see a spurious negative correlation. As we had the data of all average friction coefficients for each condition for a given material, we also looked at the difference in maximum and minimum friction coefficients. For our tested pairs, these differences also lined up on a statistically significant, negative GLMM fit (r = -0.86, p < 0.005). However, the values for a given surface can vary drastically, with an interquartile range of 1.20 to 2.09 on a single surface. We fit participant accuracy to the differences in these IQRs across pairs. This also led to a negative GLMM fit (r = -0.65, p < 0.05). However, we are hesitant to add this to the manuscript for the reasons stated previously.

      Comment 3, Part 1

      Beyond this fundamental concern, there is a weakness in the representativeness of the PDMS finger, the vertical motion, and the speed of sliding to real human exploration.

      Overall, this is a continuous debate that we think offers two solutions. There is always a tradeoff between using a synthetic model of a finger versus a real human finger, and there is a place for both models. That is, while our mock finger will be more successful the closer it is to a human finger, it is not our goal to fully replace a human finger, rather our goal is to provide a method of characterizing surfaces that is indeed relevant on the length scale of human touch.

      The usefulness of the mock finger is in isolating the features of each surface that is independent of human variability, i.e., instabilities that form without changing loading conditions between sliding motions or even within one sliding motion. Of course, with this method, we still require confirmation of these features still forming during human exploration, which we show in Fig. 3.

      We believe that this method of characterizing surfaces at the mesoscale will ultimately lead to more successful human studies on tactile perception. Currently, and as shown in the paper, characterizing surfaces through traditional techniques, such as a commercial tribometer (friction coefficient, using a steel or hard metal ball), roughness (via atomic force microscopy or some other metrology), surface energy are less predictive. Thus, we believe this mock finger is stronger than the current state-of-the-art characterizing surfaces (we are also aware of a commercial mock finger company, but we were unable to purchase or obtain an evaluation model).

      One of the main – and severe – limitations of using a human finger is that all fingers are different, meaning any study focusing on a particular user may not apply to others or be recreated easily by other researchers. We cannot set a standard for replication around a real human finger as that participant may no longer be available, or willing to travel the world as a “standard”. Furthermore, the method in which changes their pressures and velocities is different. We note that this is a challenge unique to touch perception – how an object is touched changes the friction generated, and thus the tactile stimulus generated, whereas a standardized stimulus is more straightforward for light or sound.

      However, we do emphasize that we have strongly considered the balance between feasibility and ecological validity in the design of a mock finger. We have a mock finger, with the three components of stiffness of a human finger (more below). Furthermore, we have also successfully used this mock finger in correlations with human psychophysics in previous work, where findings from our mechanical experiments were predictive of human performance(3-6).

      Our changes to the manuscript Added (Page 2-3)

      “Mock finger as a characterization tool

      In this work, we use a mechanical setup with a PDMS mock finger to derive tactile predictors from controlled friction traces alternative to average friction coefficients. While there is a tradeoff in selecting a synthetic finger over a more accurate, real human finger in modeling touch, our aim to design a method of mesoscale surface characterization for more successful studies on tactile perception cannot be fulfilled using one human participant as a standard. We believe that with sufficient replication of surface and bulk properties as well as contact geometry, and controlled friction measurements collected at loading conditions observed during a tactile discrimination task, we can isolate unique frictional features of a set of surfaces that do not arise from human-to-human variability.

      The major component of a human finger, by volume, is soft tissue (~56%)(22), resulting in an effective modulus close to 100 kPa(23,24). In order to achieve this same softness, we crosslink PDMS in a 1×1×5 cm mold at a 30:1 elastomer:crosslinker ratio. However, two more features impart increased stiffness in a human finger. Most of this added rigidity is derived from the bone at the fingertip, the distal phalanx(23–25), which we mimic with an acrylic bone within our PDMS network. The stratum corneum, the stiffer, glassier outer layer of skin(26), is replicated with the surface of the mock finger glassified, or further crosslinked, after 8 hours of UV-Ozone treatment(27). This treatment also modifies the surface properties of the native PDMS to align with those of a human finger more closely. It minimizes the viscoelastic tack at the surface, resulting in a comparable non-sticky surface. At least one day after treatment, the finger surface returns to moderate hydrophilicity (~60º), as is typically observed for a real finger(28).

      The initial contact area formed before a friction trace is collected is a rectangle of 1×1 cm. While this shape is not entirely representative of a human finger with curves and ridges, human fingers flatten out enough to reduce the effects of curvature with even very light pressures(28–30). This implies that regardless of finger pressure, the contact area is largely load-independent, which is more accurately replicated with a rectangular mock finger. It is still a challenge to control pressure distribution with this planar interface, but non-uniform pressures are also expected during human exploration.

      Lastly, we consider fingerprints vs. flat fingers. A key finding of our previous work is that while fingerprints enhanced frictional dynamics at certain conditions, key features were still maintained with a flat finger.7 Furthermore, for some loading conditions, the more amplified signals could also result in more similar friction traces for different surfaces. We have continued to use flat fingers in our mechanical experiments, and have observed good agreement between these friction traces and human experiments(7,8,21,31).”

      (Page 3-4, Materials and Methods)

      “Mock Finger Preparation

      Friction forces across all six surfaces were measured using a custom apparatus with a polydimethylsiloxane (PDMS, Dow Sylgard 184) mock finger that mimics a human finger’s

      mechanical properties and contact mechanics while exploring a surface relatively closely(7,8). PDMS and crosslinker were combined in a 30:1 ratio to achieve a stiffness of 100 kPa comparable to a real finger, then degassed in a vacuum desiccator for 30 minutes. We are aware that the manufacturer recommended crosslinking ratio for Sylgard 184 is 10:1 due to potential uncrosslinked liquid residues(32), but further crosslinking concentrated at the surface prevents this. The prepared PDMS was then poured into a 1×1×5 cm mold also containing an acrylic 3D-printed “bone” to attach applied masses on top of the “fingertip” area contacting a surface during friction testing. After crosslinking in the mold at 60ºC for 1 hour, the finger was treated with UV-Ozone for 8 hours out of the mold to minimize viscoelastic tack.

      Mechanical Testing

      A custom device using our PDMS mock finger was used to collect macroscopic friction force traces replicating human exploration(7,8). After placing a sample surface on a stage, the finger was lowered at a slight angle such that an initial 1×1 cm rectangle of “fingertip” contact area could be established. We considered a broad range of applied masses (M \= 0, 25, 75, and 100 g) added onto the deadweight of the finger (6 g) observed during a tactile discrimination task. The other side of the sensor was connected to a motorized stage (V-508 PIMag Precision Linear Stage, Physikinstrumente) to control both displacement (4 mm across all conditions) and sliding velocity (v \= 5, 10, 25, and 45 mm s<sup>-1</sup>). Forces were measured at all 16 combinations of mass and velocity via a 250 g Futek force sensor (k \= 13.9 kN m<sup>-1</sup>) threaded to the bone, and recorded at an average sampling rate of 550 Hz with a Keithley 7510 DMM digitized multimeter. Force traces were collected in sets of 4 slides, discarding the first due to contact aging. Because some mass-velocity combinations were near the boundaries of instability phase transitions, not all force traces at these given conditions exhibited similar profiles.

      Thus, three sets were collected on fresh spots for each condition to observe enough occurrences of multiple instabilities, at a total of nine traces per combination for each surface.”

      Added References (Page 13)

      M. Murai, H.-K. Lau, B. P. Pereira and R. W. H. Pho, J. Hand Surg., 1997, 22, 935–941.

      A. Abdouni, M. Djaghloul, C. Thieulin, R. Vargiolu, C. Pailler-Mattei and H. Zahouani, R. Soc. Open Sci., DOI:10.1098/rsos.170321.

      P.-H. Cornuault, L. Carpentier, M.-A. Bueno, J.-M. Cote and G. Monteil, J. R. Soc. Interface, DOI:10.1098/rsif.2015.0495.

      K. Qian, K. Traylor, S. W. Lee, B. Ellis, J. Weiss and D. Kamper, J. Biomech., 2014, 47, 3094– 3099.

      Y. Yuan and R. Verma, Colloids Surf. B Biointerfaces, 2006, 48, 6–12.

      Y.-J. Fu, H. Qui, K.-S. Liao, S. J. Lue, C.-C. Hu, K.-R. Lee and J.-Y. Lai, Langmuir, 2010, 26, 4392–4399.

      Comment 3, Part 2

      “The real finger has multiple layers with different moduli. In fact, the stratum corneum cells, which are the outer layer at the interface and determine the friction, have a much higher modulus than PDMS. The real finger has multiple layers with different moduli. In fact, the stratum corneum cells, which are the outer layer at the interface and determine the friction, have a much higher modulus than PDMS.

      We have approximated the softness of the finger with 100 kPa crosslinked PDMS, which is close to what has been reported for the bulk of a human fingertip(8,9). However, as mentioned in the Materials and Methods, there are two additional features of the mock finger that impart greater strength. The PDMS surrounds a rigid, acrylic bone comparable to the distal phalanx, which provides an additional layer of higher modulus(10). Additionally, the 8-hour UV-Ozone treatment decreases the viscoelastic tack of the pristine PDMS by glassifying, or further crosslinking the surface of the finger(11), therefore imparting greater stiffness at the surface similar to the contributions of the stratum corneum, along with a similar surface energy(12). This technique is widely used in wearables(13), soft robotics(14), and microfluidics(15) to induce both these material changes. Additionally, the finger is used at least a day after UV-Ozone treatment is completed in order for the surface to return to moderate hydrophilicity, similar to the outermost layer of human skin(16).

      Comment 3, Part 3

      In addition, the slanted position of the finger can cause non-uniform pressures across the finger. Both can contribute to making the PDMS finger have much more stick-slip than a real finger.

      To ensure that there is minimal contribution from the slanted position of the finger, an initial contact area of 1×1 cm is established before sliding and recording friction measurements. As the PDMS finger is a soft object, the portion in contact with a surface flattens and the contact area remains largely unchanged during sliding. Any additional stick-slip after this alignment step is caused by contact aging at the interface, but the first trace we collect is always discarded to only consider stick-slip events caused by surface chemistry. We recognize that it is difficult to completely control the pressure distribution due to the planar interface, but this is also expected when humans freely explore a surface.

      Comment 3, Part 4

      In fact, if you look at the regime maps, there is very little space that has steady sliding. This does not represent well human exploration of surfaces. We do not tend to use a force and velocity that will cause extensive stick-slip (frequent regions of 100% stick-slip) and, in fact, the speeds used in the study are on the slow side, which also contributes to more stick-slip. At higher speeds and lower forces, all of the materials had steady sliding regions.

      We are not aware of published studies that extensively show that humans avoid stickslip regimes. In fact, we are aware familiar with literature where stiction spike formation is suppressed – a recent paper by AliAbbasi, Basdogan et. al. investigates electroadhesion and friction with NaCl solution-infused interfaces, resulting in significantly steadier forces(17). We also directly showed evidence of instability formation that we observed during human exploration in Fig. 3B-C. These dynamic events are common, despite the lack of control of normal forces and sliding velocities. We also note that Reviewer 1, Comment 2, was suggesting that we further explore possible trends from parameterizing the stiction spike.

      We note that many studies have often not gone at the velocities and masses required for stiction spikes – even though these masses and velocities would be routinely seen in free exploration – this is usually due to constraints of equipment(18). Sliding events during human free exploration of surfaces can exceed 100 mm/s for rapid touches. However, for the surfaces investigated here, we observe that large regions of stick-slip can emerge at velocities as low as 5 mm/s depending on the applied load. The incidence of steady sliding appears more dependent on the applied mass, with almost no steady sliding observed at or above 75 g. Indeed, the force categorization along our transition zones is the main point of the paper.

      Comment 3, Part 5

      Further, on these very smooth surfaces, the friction and stiction are more complex and cannot dismiss considerations such as finger material property change with sweat pore occlusion and sweat capillary forces. Also, the vertical motion of both the PDMS finger and the instructed human subjects is not the motion that humans typically use to discriminate between surfaces.

      We did not describe the task sufficiently. Humans were only given the instruction to slide their finger along a single axis from top to bottom of a sample, not vertical as in azimuthal to gravity. We have updated our wording in the manuscript to reflect this.

      Our changes to the manuscript (Page 4)

      “Participants could touch for as long as they wanted, but were asked to only use their dominant index fingers along a single axis to better mimic the conditions for instability formation during mechanical testing with the mock finger.”

      (Page 11)

      “The participant was then asked to explore each sample simultaneously, and ran over each surface in strokes along a single axis until the participant could decide which of the two had “more friction”.”

      Comment 3, Part 6

      Finally, fingerprints may not affect the shape and size of the contact area, but they certainly do affect the dynamic response and detection of vibrations.

      We are aware of the nuance. Our previous work on the role of fingerprints on friction experienced by a PDMS mock finger showed enhanced signals with the incorporation of ridges on the finger and used a rate-and-state model of a heterogenous, elastic body to find corresponding trends (though there is no existing model of friction that can accurately model experiments on mesoscale friction)(7). The key conclusion was that a flat finger still preserved key dynamic features, and the presence of stronger or more vibrations could result in more similar forces for different surfaces depending on the sliding conditions.

      This is also in the context that we are seeking to provide a reasonable and experimentally accessible method to characterize surfaces, which will always be better as we get closer in replicating a true human finger. But our goal here was to replicate the finger sufficiently for use in human studies. We believe the more appropriate metric of success is if the mock finger is more successful than replacing traditional characterization experiments, like friction coefficient, roughness, surface energy, etc.

      Comment 4

      This all leads to the critical question, why are friction, normal force, and velocity not measured during the measured human exploration and in a systematic study using the real human finger? The authors posed an extremely interesting hypothesis that humans may alter their speed to feel the instability transition regions. This is something that could be measured with a real finger but is not likely to be correlated accurately enough to match regime boundaries with such a simplified artificial finger.

      We are excited that our manuscript offers a tractable manner to test the hypothesis that tactile decision-making models use friction instabilities as evidence. However, we lay out the challenges and barriers, and how the scope of this paper will lead us in that direction. We also clarify that our goals are to provide a method to characterize samples to better design tactile interfaces in haptics or in psychophysical experiments and raise awareness that the common methods of sample characterization in touch by an average friction coefficient or roughness is fundamentally unsound.

      In short, in our view, to further support our findings on instabilities would require answering:

      (1) Which one, or combination of, of the multiple swipes that people make responsible for a tactile decision? (The need for a decision-making model)

      (2) Establish what is, or may be, tactile evidence.

      (3) Establish tactile decision-making models are similar or different than existing decision-making models.

      (4) Test the hypothesis, in these models, that friction instabilities are evidence, and not some other unknown metric. This requires design samples that vary in the amount of evidence generated, but this evidence cannot be controlled directly. Rather, the samples indirectly vary evidence by how likely it is for a human to generate different types of friction instabilities during standard exploration.

      (5) Design a task that does not require the use of subjective tactile descriptors, like “which one feels rougher”, which we see cause confusion in participants, which will likely require accounting for memory effects.

      We elaborate these points below:

      To successfully perform this experiment, we note that freely exploring humans make multiple strokes on a surface. Therefore, we would need to construct a decision-making model. It has not yet been demonstrated whether tactile decision making follows visual decision making, but perhaps to start, we can assume it does. Then, in the design of our decision-making paradigm, we immediately run into the problem: What is tactile evidence?

      From Fig. 3C, we already can see that identifying evidence is challenging. Prior to this manuscript, people may have chosen the average force, or the highest force. Or we may choose the average friction force. Then, after deciding on the evidence, we need to find a method to manipulate the evidence, i.e., create samples or a machine that causes high friction, etc. We show that during the course of human touch, due to the dynamic nature of friction, the average can change a large amount and sample design becomes a central barrier to experiments. Others may suggest immobilizing the finger and applying a known force, but given how much friction changes with human exploration, there is no known method to make a machine recreate temporally and spatially varying friction forces during sliding onto a stationary finger. Finally, perhaps most importantly, in addition to mechanical challenges, a study by Liu, Colgate et al. showed that even if they recorded the friction (2D) of a finger exploring a surface and then replicated the same friction forces onto a finger, the participant could not determine which surface the replayed friction force was supposed to represent.1 This supports that the efference copy is important, that the forces in response to expected motion are important to determine friction. Finally, there is no known method to design instabilities a priori. They must be found through experiments. Especially since if we were to introduce, say a bump or a trough, then we bring in confounding variables to how participants tell surfaces apart.

      Furthermore, even if we had some consistent method to create tactile “evidence”, the paradigm also deserves some consideration. In our experience, the 3-AFC task we perform is important because the vocabulary for touch has not been established. That is, in 3-AFC, by asking to determine which one sample is unlike the others, we do not have to ask the participant questions like “which one is rougher” or “which one has less friction”. In contrast, 2-AFC, which is better for decision-making models because it does not include memory, requires the asking of a perceptual question like: “which one is rougher?”. In our ongoing work, taking two silane coatings, we found that participants could easily identify which surface is unlike the others above chance in a 3-AFC, but participants, even within their own trials, could not consistently identify one silane as perceptually “rougher” by 2-AFC. To us, this calls into question the validity of tactile descriptors, but is beyond the scope of this manuscript.

      This is not our only goal, but in the context of human exploration, in this manuscript here, we believed it was important to identify a mechanical parameter that was consistent with how humans explore surfaces, but was also a parameter that could characterize to some consistent property of a surface – irrespective of whether a human was touching it. We thought that designing human decision-making models and paradigms around the friction coefficient would not be successful.

      Given the scope of these challenges, we do not think it would be possible to establish these conceptual sequences in a single manuscript.

      Reviewer 2 (Public review):

      Summary:

      In this paper, the authors want to test the hypothesis that frictional instabilities rather than friction are the main drivers for discriminating flat surfaces of different sub-nanometric roughness profiles.

      They first produced flat surfaces with 6 different coatings giving them unique and various properties in terms of roughness (picometer scale), contact angles (from hydrophilic to hydrophobic), friction coefficient (as measured against a mock finger), and Hurst exponent.

      Then, they used those surfaces in two different experiments. In the first experiment, they used a mock finger (PDMS of 100kPA molded into a fingertip shape) and slid it over the surfaces at different normal forces and speeds. They categorized the sliding behavior as steady sliding, sticking spikes, and slow frictional waves by visual inspection, and show that the surfaces have different behaviors depending on normal force and speed. In a second experiment, participants (10) were asked to discriminate pairs of those surfaces. It is found that each of those pairs could be reliably discriminated by most participants.

      Finally, the participant's discrimination performance is correlated with differences in the physical attributes observed against the mock finger. The authors found a positive correlation between participants' performances and differences in the count of steady sliding against the mock finger and a negative correlation between participants' reaction time and differences in the count of stiction spikes against the mock finger. They interpret those correlations as evidence that participants use those differences to discriminate the surfaces.

      Strengths:

      The created surfaces are very interesting as they are flat at the nanometer scale, yet have different physical attributes and can be reliably discriminated.”

      We thank Reviewer 2 for their notes on our manuscript. The responses below address the reviewer’s comments and recommendations for revised work.

      Weaknesses:

      Comment 1

      In my opinion, the data presented in the paper do not support the conclusions. The conclusions are based on a correlation between results obtained on the mock finger and results obtained with human participants but there is no evidence that the human participants' fingertips will behave similarly to the mock finger during the experiment. Figure 3 gives a hint that the 3 sliding behaviors can be observed in a real finger, but does not prove that the human finger will behave as the mock finger, i.e., there is no evidence that the phase maps in Figure 1C are similar for human fingers and across different people that can have very different stiffness and moisture levels.

      The mechanical characterization conducted with the mock finger seeks to extract significant features of friction traces of a set of surfaces to use as predictors of tactile discriminability. The goal is to find a consistent method to characterize surfaces for use in tactile experiments that can be replicated by others and used prior to any human experiments. However, in the overall response and in a response to a similar comment by Reviewer 1, we also explain why we believe experiments on humans to establish this fact is not yet reasonable.

      Comment 2

      I believe that the authors collected the contact forces during the psychophysics experiments, so this shortcoming could be solved if the authors use the actual data, and show that the participant responses can be better predicted by the occurrence of frictional instabilities than by the usual metrics on a trial by trial basis, or at least on a subject by subject basis. I.e. Poor performers should show fewer signs of differences in the sliding behaviors than good performers.

      To fully implement this, a decision-making model is necessary because, as a counter example, a participant could have generated 10 swipes of SFW and 1 swipe of a Sp, but the Sp may have been the most important event for making a tactile decision. This type of scenario is not compatible with the analysis suggested — and similar counterpoints can be made for other types of seemingly straightforward analysis.

      While we are interested and actively working on this, the study here is critical to establish types of evidence for a future decision-making model. We know humans change their friction constantly during real exploration, so it is unclear which of these constantly changing values we should input into the decision making model, and the future challenges we anticipate are explained in Comment 1.

      Comment 3

      The sample size (10) is very small.

      We recognize that, with all factors being equal, this sample size is on the smaller end. However, we emphasize the degree of control of samples is far above typical, with minimal variations in sample properties such as surface roughness, and every sample for every trial was pristine. Furthermore, the sample preparation (> 300 individual wafers were used) and cost became a factor. Although not typically appropriate, and thus not included in the manuscript, a post-hoc power analysis for our 100 trials of our pair that was closest to chance, P4, (53%, closest to chance at 33%) showed a power of 98.2%, suggesting that the study was appropriately powered.

      Reviewer 2 (Recommendations for the authors):

      Comment 1

      Differences in SS and Sp (Table 2) are NOT physical or mechanical differences but are obtained by counting differences in the number of occurrences of each sliding behavior. It is rather a weird choice.

      We disagree that differences in SS and Sp are not physical or mechanical, as these are well-established phenomena in the soft matter and tribology literature(19-21). These are known as “mechanical instabilities” and generated due to the effects of two physical phenomena: the elasticity of the finger (which is constant in our mechanical testing) and the friction forces present (which change per sample type). The motivation behind using these different shapes is that the instabilities, in some conditions, can be invariant to external factors like velocity. This would be quite advantageous for human exploration because, unlike friction coefficient, which changes with nearly any factor, including velocity and mass, the instabilities being invariant to velocity would mean that we are accurately characterizing a unique identifier of the surface even though velocity may be variable.

      This “weird choice” is the central innovation of this paper. This choice was necessary because we demonstrated that the common usage of friction coefficient is fundamentally flawed: we see that friction coefficient suggests that surface which are more different would feel more similar – indeed the most distinctive surfaces would be two surfaces that are identical, which is clearly spurious. One potential explanation for why we were able to see this is effect is because our surfaces have similar (< 0.6 nm variability) roughness, removing potential confounding factors, and this type of low roughness control has not been used in tactile studies to the best of our knowledge.

      Comment 2

      Figures 2B-C: why are the x-data different than Table 2?

      The x-data in Fig. 2B-C are the absolute differences in the number of occurrences measured for a given instability type or material property out of 144 pulls. Modeling the human participant results in our GLMMs required the independent variables to be in this form rather than percentages. We initially chose to list percent differences in Table 2 to highlight the ranges of differences instead of an absolute value, but have added both for clarity.

      Our changes to the manuscript (Page 7)

      “To determine if humans can detect these three different instabilities, we selected six pairs of surfaces to create a broad range of potential instabilities present across all three types. These are summarized in Table 2, where the first column for each instability is the difference in occurrence of that instability formed between each pair, and the second is the percent difference.”

      Comment 3

      "We constructed a set of coated surfaces with physical differences which were imperceptible by touch but created different types of instabilities based on how quickly a finger is slid and how hard a human finger is pressed during sliding." Yet, in your experiment, participants could discriminate them, so this is incoherent.

      To clarify the point, macroscopic objects can differ in physical shape and in chemical composition. What we meant was that the physical differences, i.e., roughness, were below a limit (Skedung et al.) that participants, without a coating, would not be able to tell these apart(22). Therefore, the reason people could tell our surfaces apart was due to the chemical composition of the surface, and not any differences in roughness or physical effects like film stiffness (due to the molecular-scale thinness of the surface coatings, they are mechanically negligible). However, we concede that at the molecular scale, the traditional macroscopic distinction between physical and chemical is blurred.

      We have made minor revisions to the wording in the abstract. We clarify that the surface coatings had physical differences in roughness that were smaller than 0.6 nm, which based purely on roughness, would not be expected to be distinguishable to participants. Therefore, the reason participants can tell these surfaces apart is due to differences in friction generated by chemical composition, and we were able to minimize contributions from physical differences in the sample our study.

      Our changes to the manuscript (Page 1, Abstract)

      “We constructed a set of coated surfaces with minimal physical differences that by themselves, are not perceptible to people, but instead, due to modification in surface chemistry, the surfaces created different types of instabilities based on how quickly a finger is slid and how hard a human finger is pressed during sliding.”

      Reviewer 3 (Public review):

      Strengths:  

      The paper describes a new perspective on friction perception, with the hypothesis that humans are sensitive to the instabilities of the surface rather than the coefficient of friction. The paper is very well written and with a comprehensive literature survey.

      One of the central tools used by the author to characterize the frictional behavior is the frictional instabilities maps. With these maps, it becomes clear that two different surfaces can have both similar and different behavior depending on the normal force and the speed of exploration. It puts forward that friction is a complicated phenomenon, especially for soft materials.

      The psychophysics study is centered around an odd-one-out protocol, which has the advantage of avoiding any external reference to what would mean friction or texture for example. The comparisons are made only based on the texture being similar or not.

      The results show a significant relationship between the distance between frictional maps and the success rate in discriminating two kinds of surface.”

      We thank Reviewer 3 for their notes and interesting discussion points on our manuscript. Below, we address the reviewer’s feedback and comments on related works.

      Weaknesses:

      Comment 1

      The main weakness of the paper comes from the fact that the frictional maps and the extensive psychophysics study are not made at the same time, nor with the same finger. The frictional maps are produced with an artificial finger made out of PDMS which is a poor substitute for the complex tribological properties of skin.

      A similar comment was made by Reviewers 1 and 2 and parts are replicated below. We are not claiming that our PDMS fingers are superior to real fingers, but rather, we cannot establish standards in the field by using real human fingers that vary between subjects and researchers. We believe the mock finger we designed is a reasonable mimic of the human finger by matching surface energy, heterogeneous mechanical structure, and the ability to test multiple physiologically relevant pressures and sliding velocities.

      We achieve a heterogeneous mechanical structure with the 3 primary components of stiffness of a human finger. The effective modulus of ~100 kPa, from soft tissue,8,9 is obtained with a 30:1 ratio of PDMS to crosslinker. The PDMS also surrounds a rigid, acrylic bone comparable to the distal phalanx, which provides an additional layer of higher modulus.10 Additionally, the 8-hour UV-Ozone treatment decreases the viscoelastic tack of the pristine PDMS by glassifying, or further crosslinking the surface of the finger,11 therefore imparting greater stiffness at the surface similar to the contributions of the stratum corneum, along with a similar surface energy.12 The finger is used at least a day after UV-Ozone treatment is completed in order for the surface to return to moderate hydrophilicity, similar to the outermost layer of human skin.16 We also discuss the shape of the contact formed. To ensure that there is minimal contribution from the slanted position of the finger, an initial contact area of 1×1 cm is established before sliding and recording friction measurements. As the PDMS finger is a soft object, the portion in contact with a surface flattens and the contact area remains largely unchanged during sliding. We recognize that it is difficult to completely control the pressure distribution due to the planar interface, but this variation is also expected when humans freely explore a surface. Finally, we consider flat vs. fingerprinted fingers. Our previous work on the role of fingerprints on friction experienced by a PDMS mock finger showed enhanced signals with the incorporation of ridges on the finger and used a rate-andstate model of a heterogenous, elastic body to find corresponding trends.7 The key conclusion was that a flat finger still preserved key dynamic features, and the presence of stronger or more vibrations could result in more similar forces for different surfaces depending on the sliding conditions. We note that we have subsequently used the controlled mechanical data collected with this flat mock finger in correlations with human psychophysics in previous work, where findings from our mechanical experiments were predictive of human performance.3–6 Ultimately, we see from our prior work and here that, despite the drawbacks of our mock finger, it outperforms other standard characterization technique in providing information about the mesoscale that correlates to tactile perception. We have added these details to the manuscript.

      We also note that an intermediate option, replicating real fingers, even in a mold, may also inadvertently limit trends from characterization to a specific finger. One of the main – and severe – limitations of using a human finger is that all fingers are different, meaning any study focusing on a particular user may not apply to others or be recreated easily by other researchers. We cannot set a standard for replication around a real human finger as that participant may no longer be available, or willing to travel the world as a “standard”. Furthermore, the method in which a single person changes their pressures and velocities as they touch a surface is highly variable. We also note that in the Summary Response, we noted that a study by Colgate et al. (IEEE ToH 2024) demonstrated that efference copies may be important, and thus constraining a human finger and replaying the forces recorded during free exploration will not lead to the participant identifying a surface with any consistency. Thus, it is important to allow humans to freely explore surfaces, but creates nearly limitless variability in friction forces.

      This is also against the backdrop that we are seeking to provide a method to characterize surfaces, which will be aided as we get closer in replicate a true human finger. Indeed, the more features we replicate, the more successful the mechanical data will be in correlating to tactile distinguishability. But reasonably, our success would be in replacing traditional characterization experiments, not in recreating the forces of an arbitrary human finger.

      Our changes to the manuscript Added (Page 2-3)

      “Mock finger as a characterization tool

      In this work, we use a mechanical setup with a PDMS mock finger to derive tactile predictors from controlled friction traces alternative to average friction coefficients. While there is a tradeoff in selecting a synthetic finger over a more accurate, real human finger in modeling touch, our aim to design a method of mesoscale surface characterization for more successful studies on tactile perception cannot be fulfilled using one human participant as a standard. We believe that with sufficient replication of surface and bulk properties as well as contact geometry, and controlled friction measurements collected at loading conditions observed during a tactile discrimination task, we can isolate unique frictional features of a set of surfaces that do not arise from human-to-human variability.

      The major component of a human finger, by volume, is soft tissue (~56%)(22), resulting in an effective modulus close to 100 kPa(23,24). In order to achieve this same softness, we crosslink PDMS in a 1×1×5 cm mold at a 30:1 elastomer:crosslinker ratio. However, two more features impart increased stiffness in a human finger. Most of this added rigidity is derived from the bone at the fingertip, the distal phalanx(23-25), which we mimic with an acrylic bone within our PDMS network. The stratum corneum, the stiffer, glassier outer layer of skin(26), is replicated with the surface of the mock finger glassified, or further crosslinked, after 8 hours of UV-Ozone treatment(27). This treatment also modifies the surface properties of the native PDMS to align with those of a human finger more closely. It minimizes the viscoelastic tack at the surface, resulting in a comparable non-sticky surface. At least one day after treatment, the finger surface returns to moderate hydrophilicity (~60º), as is typically observed for a real finger(28).

      The initial contact area formed before a friction trace is collected is a rectangle of 1×1 cm. While this shape is not entirely representative of a human finger with curves and ridges, human fingers flatten out enough to reduce the effects of curvature with even very light pressures(28-30). This implies that regardless of finger pressure, the contact area is largely load-independent, which is more accurately replicated with a rectangular mock finger. It is still a challenge to control pressure distribution with this planar interface, but non-uniform pressures are also expected during human exploration.

      Lastly, we consider fingerprints vs. flat fingers. A key finding of our previous work is that while fingerprints enhanced frictional dynamics at certain conditions, key features were still maintained with a flat finger(7). Furthermore, for some loading conditions, the more amplified signals could also result in more similar friction traces for different surfaces. We have continued to use flat fingers in our mechanical experiments, and have observed good agreement between these friction traces and human experiments(7,8,21,31).”

      (Page 3-4, Materials and Methods)

      “Mock Finger Preparation

      Friction forces across all six surfaces were measured using a custom apparatus with a polydimethylsiloxane (PDMS, Dow Sylgard 184) mock finger that mimics a human finger’s

      mechanical properties and contact mechanics while exploring a surface relatively closely(7,8). PDMS and crosslinker were combined in a 30:1 ratio to achieve a stiffness of 100 kPa comparable to a real finger, then degassed in a vacuum desiccator for 30 minutes. We are aware that the manufacturer recommended crosslinking ratio for Sylgard 184 is 10:1 due to potential uncrosslinked liquid residues(32), but further crosslinking concentrated at the surface prevents this. The prepared PDMS was then poured into a 1×1×5 cm mold also containing an acrylic 3D-printed “bone” to attach applied masses on top of the “fingertip” area contacting a surface during friction testing. After crosslinking in the mold at 60ºC for 1 hour, the finger was treated with UV-Ozone for 8 hours out of the mold to minimize viscoelastic tack.  

      Mechanical Testing

      A custom device using our PDMS mock finger was used to collect macroscopic friction force traces replicating human exploration(7,8). After placing a sample surface on a stage, the finger was lowered at a slight angle such that an initial 1×1 cm rectangle of “fingertip” contact area could be established. We considered a broad range of applied masses (M \= 0, 25, 75, and 100 g) added onto the deadweight of the finger (6 g) observed during a tactile discrimination task. The other side of the sensor was connected to a motorized stage (V-508 PIMag Precision Linear Stage, Physikinstrumente) to control both displacement (4 mm across all conditions) and sliding velocity (v \= 5, 10, 25, and 45 mm s<sup>-1</sup>). Forces were measured at all 16 combinations of mass and velocity via a 250 g Futek force sensor (k \= 13.9 kN m<sup>-1</sup>) threaded to the bone, and recorded at an average sampling rate of 550 Hz with a Keithley 7510 DMM digitized multimeter. Force traces were collected in sets of 4 slides, discarding the first due to contact aging. Because some mass-velocity combinations were near the boundaries of instability phase transitions, not all force traces at these given conditions exhibited similar profiles. Thus, three sets were collected on fresh spots for each condition to observe enough occurrences of multiple instabilities, at a total of nine traces per combination for each surface.”

      Added References (Page 13)

      M. Murai, H.-K. Lau, B. P. Pereira and R. W. H. Pho, J. Hand Surg., 1997, 22, 935–941.

      A. Abdouni, M. Djaghloul, C. Thieulin, R. Vargiolu, C. Pailler-Mattei and H. Zahouani, R. Soc. Open Sci., DOI:10.1098/rsos.170321.

      P.-H. Cornuault, L. Carpentier, M.-A. Bueno, J.-M. Cote and G. Monteil, J. R. Soc. Interface, DOI:10.1098/rsif.2015.0495.

      K. Qian, K. Traylor, S. W. Lee, B. Ellis, J. Weiss and D. Kamper, J. Biomech., 2014, 47, 3094– 3099.

      Y. Yuan and R. Verma, Colloids Surf. B Biointerfaces, 2006, 48, 6–12.

      Y.-J. Fu, H. Qui, K.-S. Liao, S. J. Lue, C.-C. Hu, K.-R. Lee and J.-Y. Lai, Langmuir, 2010, 26, 4392–4399.

      Comment 2

      The evidence would have been much stronger if the measurement of the interaction was done during the psychophysical experiment. In addition, because of the protocol, the correlation is based on aggregates rather than on individual interactions.

      Our Response: We agree that this would have helped further establish our argument, but in the overall statement and in other reviewer responses, we describe the significant challenges to establishing this.

      To fully implement this, a decision-making model is necessary because, as a counter example, a participant could have generated 10 swipes of SFW and 1 swipe of a Sp, but the Sp may have been the most important event for making a tactile decision. We also clarify that our goals are to provide a method to characterize samples to better design tactile interfaces in haptics or in psychophysical experiments.

      In short, in our view, to develop a decision-making model, the challenges are as follows:

      (1) Which one, or combination of, of the multiple swipes that people make responsible for a tactile decision?

      (2) Establish what is, or may be, tactile evidence.

      (3) Establish tactile decision-making models are similar or different than existing decision-making models.

      (4) Test the hypothesis, in these models, that friction instabilities are evidence, and not some other unknown metric.

      (5) Design a task that does not require the use of subjective tactile descriptors, like “which one feels rougher”, which we see cause confusion in participants, which will likely require accounting for memory effects.

      (6) Design samples that vary in the amount of evidence generated, but this evidence cannot be controlled directly. Rather, the samples indirectly vary evidence by how likely it is for a human to generate different types of friction instabilities during standard exploration.

      We elaborate these points below:

      To successfully perform this experiment, we note that freely exploring humans make multiple strokes on a surface. Therefore, we would need to construct a decision-making model. It has not yet been demonstrated whether tactile decision making follows visual decision making, but perhaps to start, we can assume it does. Then, in the design of our decision-making paradigm, we immediately run into the problem: What is tactile evidence?

      From Fig. 3C, we already can see that identifying evidence is challenging. Prior to this manuscript, people may have chosen the average force, or the highest force. Or we may choose the average friction force. Then, after deciding on the evidence, we need to find a method to manipulate the evidence, i.e., create samples or a machine that causes high friction, etc. We show that during the course of human touch, due to the dynamic nature of friction, the average can change a large amount and sample design becomes a central barrier to experiments. Others may suggest to immobilize the finger and applying a known force, but given how much friction changes with human exploration, there is no known method to make a machine recreate temporally and spatially varying friction forces during sliding onto a stationary finger. Finally, perhaps most importantly, in addition to mechanical challenges, a study by Liu, Colgate et al. showed that even if they recorded the friction (2D) of a finger exploring a surface and then replicated the same friction forces onto a finger, the participant could not determine which surface the replayed friction force was supposed to represent.1 This supports that the efference copy is important, that the forces in response to expected motion are important to determine friction. Finally, there is no known method to design instabilities a priori. They must be found through experiments, especially since if we were to introduce, say a bump or a trough, then we bring in confounding variables to how participants tell surfaces apart.

      Furthermore, even if we had some consistent method to create tactile “evidence”, the paradigm also deserves some consideration. In our experience, the 3-AFC task we perform is important because the vocabulary for touch has not been established. That is, in 3-AFC, by asking to determine which one sample is unlike the others, we do not have to ask the participant questions like “which one is rougher” or “which one has less friction”. In contrast, 2-AFC, which is better for decision-making models because it does not include memory, requires the asking of a perceptual question like: “which one is rougher?”. In our ongoing work, taking two silane coatings, we found that participants could easily identify which surface is unlike the others above chance in a 3-AFC, but participants, even within their own trials, could not consistently identify one silane as perceptually “rougher” by 2-AFC. To us, this calls into question the validity of tactile descriptors, but is beyond the scope of the current manuscript.

      This is not our only goal, but in the context of human exploration, in this manuscript here, we believed it was important to identify a mechanical parameter that was consistent with how humans explore surfaces, but was also a parameter that could characterize to some consistent property of a surface – irrespective of whether a human was touching it. We thought that designing human decision-making models and paradigms around the friction coefficient would not be successful.

      Given the scope of these challenges, we do not think it would be possible to establish this conceptual sequence in a single manuscript.

      Comment 3

      The authors compensate with a third experiment where they used a 2AFC protocol and an online force measurement. But the results of this third study, fail to convince the relation.

      With this experiment, our central goal was to demonstrate that the instabilities we have identified with the PDMS finger also occur with a human finger. Several instances of SS, Sp, and SFW were recorded with this setup as a participant touched surfaces in real time.

      Comment 4

      No map of the real finger interaction is shown, bringing doubt to the validity of the frictional map for something as variable as human fingers.

      Real fingers change constantly during exploration, and friction is state-dependent, meaning that the friction will depend on how the person was moving the moment prior. Therefore, a map is only valid for a single human movement – even if participants all were instructed to take a single swipe and start from zero motion, humans are unable to maintain constant velocities and pressures. Clearly, this is not sustainable for any analysis, and these drawbacks apply to any measured parameter, whether instabilities suggested here, or friction coefficients used throughout. We believe the difficulty of this approach emphasizes why a standard map of characterization of a surface by a mock finger, even with its drawbacks, is a viable path forward.

      Reviewer 3 (Recommendations for the authors):

      Comment 1

      It would be interesting to comment on a potential connection between the frictional instability maps and Schalamack waves

      Schallamach waves are a subset of slow frictional waves (SFW). Schallmach waves are very specifically defined. They are a are pockets of air that form between a soft sliding object and rigid surface, and propagate rear-to-front (retrograde waves) as a soft object is slid and buckles due to adhesive pinning. Wrinkles form at the detached portion of the soft material, until the interface reattaches and the process repeats.23 There is typically a high burden of proof to establish a Schallamach wave over a more general slow frictional wave. We note that it would be exceeding difficult to design samples that can reliably create subsets of SFW, but we are aware that this may be an interesting question at a future point in our work.

      Comment 2

      The force sensors look very compliant, and given the dynamic nature of the signal, it is important to characterize the frequency response of the system to make sure that the fluctuations are not amplified.

      Our Response: Thank you for noticing. We mistyped the sensor spring constant as 13.9 N m<sup>-1</sup> instead of kN m<sup>-1</sup>. However, below we show how the instabilities are derived from the mechanics at the interface due to the compliance of the finger. The “springs” of the force sensor and PDMS finger are connected in parallel. Since k<sub>sensor</sub> = 13.9 kN m<sup>-1</sup>, the spring constant of the system overall reflects the compliance of the finger, and highlights the oscillations arising solely from stick-slip. A sample calculation is shown below.

      Author response image 1.

      Fitting a line to the initial slope of the force trace for C6 gives the equation y = 25.679_x_ – 0.2149. The slope here represents force data over time data, and is divided by the velocity (25 mm/s) to determine 𝐹𝐹 the spring constant of the system . This value is lower than ksensor = 13.9 kN/m, indicating that the “springs” representing the force sensor and PDMS finger are connected in parallel: . The finger is the compliant component of the system, with k<sub>finger</sub> = 0.902 N/m, and of course, real human fingers are also compliant so this matches our goals with the design of the mock finger.

      Our changes to the manuscript (Page 4)

      (k \= 13.9 kN m<sup>-1</sup>)

      Comment 3

      The authors should discuss about the stochastic nature of friction:

      Wiertlewski, Hudin, Hayward, IEEE WHC 2011

      Greenspon, McLellan, Lieber, Bensmaia, JRSI 2020”

      We believe that, given the references, this comment on “stochastic” refers to the macroscopically-observable fluctuations (i.e., the mechanical “noise” which is not due to instrument noise) in friction arising from the discordant network of stick-slip phenomena occurring throughout the contact zone, and not the stochastic nature of nanoscale friction that occurs thermal fluctuations nor due to statistical distributions in bond breaking associated with soft contact.

      We first note that our small-scale fluctuations do not arise from a periodic surface texture that dominates in the frequency regime. However, even on our comparatively smooth surfaces, we do expect fluctuations due to nanoscale variation in contact, generation of stick-slip across at microscale length scales that occur either concurrently or discordantly across the contact zone, and the nonlinear dependence of friction to nearly any variation in state and composition(7).

      Perhaps the most relevant to the manuscript is that a major advantage of analysis by friction is that it sidesteps these ever-present microscale fluctuations, leading to more clearly defined classifiers or categories during analysis. Wiertlewski et. al. showed repeated measurements in their systems ultimately gave rise to consistent frequencies(24) (we think their system was in a steady sliding regime and the patterning gave rise to underlying macroscopic waves). These consistent frequencies, at least in soft systems and absent obvious macroscopic patterned features, would be expected to arise from the instability categories and we see them throughout.

      Comment 4

      It is stated that "we observed a spurious, negative correlation between friction coefficient and accuracy”.

      What makes you qualify that correlation as spurious?

      We mean this as in the statistical definition of “spurious”.

      This correlation would indicate that by the metric of friction coefficient, more different surfaces are perceived more similarly. Thus, two very different surfaces, like Teflon and sandpaper, by friction coefficient would be expected to feel very similar. Two nearly identical surfaces would be expected to feel very different – but of course, humans cannot consistently distinguish two identical surfaces. This finding is counterintuitive and refutes that friction coefficient is a reliable classifier of surfaces by touch. We do not think it is productive to determine a mechanism for a spurious correlation, but perhaps one reason we were able to observe this is because our study, to the best of our knowledge, is unique for having samples that are controlled in their physical differences in roughness and surface features.

      Our changes to the manuscript (Page 10)

      “To compare the value of looking at frictional instabilities, we also performed GLMM fits on common approaches in the field, like a friction coefficient or material property typically used in tactile discrimination, shown in Fig. 2D-E. Interestingly, in Fig. 2D, we observed a spurious, negative correlation between friction coefficient (typically and often problematically simplified as across all tested conditions) and accuracy (r = -0.64, p < 0.01); that is, the more different the surfaces are by friction coefficient, the less people can tell them apart. This spurious correlation would be the opposite of intuition, and further calls into question the common practice of using friction coefficients in touch-related studies. The alternative, two-term model which includes adhesive contact area for friction coefficient(29) was even less predictive (see Fig. S6A of SI). We believe such a correlation could not have been uncovered previously as our samples are minimal in their physical variations. Yet, the dynamic changes in force even within a single sample are not considered, despite being a key feature of mesoscale friction during human touch.

      We investigate different material properties in Fig. 2E. Differences in average roughness R<sub>a</sub> (or other parameters, like root mean square roughness R<sub>rms</sub> (Fig. S6A of SI) did not show a statistically significant correlation to accuracy. Though roughness is a popular parameter, correlating any roughness parameter to human performance here could be moot: the limit of detecting roughness differences has previously been defined as 13 nm on structured surfaces(33) and much higher for randomly rough surfaces(46), all of which are magnitudes larger than the roughness differences between our surfaces. The differences in contact angle hysteresis – as an approximation of the adhesion contributions(47) – do not present any statistically significant effects on performance.”

      Comment 5

      The authors should comment on the influence of friction on perceptual invariance. Despite inducing radially different frictional behavior for various conditions, these surfaces are stably perceived. Maybe this is a sign that humans extract a different metric?

      We agree – we are excited that frictional instabilities may offer a more stable perceptual cue because they are not prone to fluctuations (Recommendations for the authors, Comment 3) and instability formation, in many conditions, is invariant to applied pressures and velocities – thus forming large zones where a human may reasonable encounter a given instability.

      Raw friction is highly prone to variation during human exploration (in alignment with Recommendations for the authors, Comment 3), but ongoing work seeks to explain tactile constancy, or the ability to identify objects despite these large changes in force. Very recently published work by Fehlberg et. al. identified the role of modulating finger speed and normal force in amplifying the differences in friction coefficient between materials in order to identify them(25), and we postulate that their work may be streamlined and consistent with the idea of friction instabilities, though we have not had a chance to discuss this in-depth with the authors yet.

      We think that the instability maps show a viable path forward to how surfaces are stably perceived, and instabilities themselves show a potential mechanism: mathematically, instabilities for given conditions can be invariant to velocity or mass, creating zones where a certain instability is encountered. This reduces the immense variability of friction to a smaller, more stable classification of surfaces (e.g., a 30% SS surface or a 60% SS surface). A given surface will typically produce the same instability at a specific condition (we found some boundaries are extremely condition sensitive, but many conditions are not), whereas a single friction trace which is highly prone to variation is not a stable metric.

      Added References (Page 14)

      53 M. Fehlberg, E. Monfort, S. Saikumar, K. Drewing and R. Bennewitz, IEEE Trans. Haptics, 2024, 17, 957–963.

      References

      Z. Liu, J.-T. Kim, J. A. Rogers, R. L. Klatzky and J. E. Colgate, IEEE Trans. Haptics, 2024, 17, 441– 450.

      D. Gueorguiev, S. Bochereau, A. Mouraux, V. Hayward and J.-L. Thonnard, Sci Rep, 2016, 6, 25553.

      C. W. Carpenter, C. Dhong, N. B. Root, D. Rodriquez, E. E. Abdo, K. Skelil, M. A. Alkhadra, J. Ramírez, V. S. Ramachandran and D. J. Lipomi, Mater. Horiz., 2018, 5, 70–77.

      A. Nolin, A. Licht, K. Pierson, C.-Y. Lo, L. V. Kayser and C. Dhong, Soft Matter, 2021, 17, 5050– 5060.

      A. Nolin, K. Pierson, R. Hlibok, C.-Y. Lo, L. V. Kayser and C. Dhong, Soft Matter, 2022, 18, 3928– 3940.

      Z. Swain, M. Derkaloustian, K. A. Hepler, A. Nolin, V. S. Damani, P. Bhattacharyya, T. Shrestha, J. Medina, L. Kayser and C. Dhong, J. Mater. Chem. B, DOI:10.1039/D4TB01646G.

      C. Dhong, L. V. Kayser, R. Arroyo, A. Shin, M. Finn, A. T. Kleinschmidt and D. J. Lipomi, Soft Matter, 2018, 14, 7483–7491.

      A. Abdouni, M. Djaghloul, C. Thieulin, R. Vargiolu, C. Pailler-Mattei and H. Zahouani, Royal Society Open Science, DOI:10.1098/rsos.170321.

      P.-H. Cornuault, L. Carpentier, M.-A. Bueno, J.-M. Cote and G. Monteil, Journal of The Royal Society Interface, DOI:10.1098/rsif.2015.0495.

      K. Qian, K. Traylor, S. W. Lee, B. Ellis, J. Weiss and D. Kamper, J Biomech, 2014, 47, 3094–3099.

      Y.-J. Fu, H. Qui, K.-S. Liao, S. J. Lue, C.-C. Hu, K.-R. Lee and J.-Y. Lai, Langmuir, 2010, 26, 4392– 4399.

      Y. Yuan and R. Verma, Colloids Surf B Biointerfaces, 2006, 48, 6–12.

      G. Yu, J. Hu, J. Tan, Y. Gao, Y. Lu and F. Xuan, Nanotechnology, 2018, 29, 115502.

      L. Zheng, S. Dong, J. Nie, S. Li, Z. Ren, X. Ma, X. Chen, H. Li and Z. L. Wang, ACS Appl. Mater. Interfaces, 2019, 11, 42504–42511.

      K. Ma, J. Rivera, G. J. Hirasaki and S. L. Biswal, Journal of Colloid and Interface Science, 2011, 363, 371–378.

      A. Mavon, H. Zahouani, D. Redoules, P. Agache, Y. Gall and Ph. Humbert, Colloids and Surfaces B: Biointerfaces, 1997, 8, 147–155.

      E. AliAbbasi, M. Muzammil, O. Sirin, P. Lefèvre, Ø. G. Martinsen and C. Basdogan, IEEE Trans. Haptics, 2024, 17, 841–849.

      G. Corniani, Z. S. Lee, M. J. Carré, R. Lewis, B. P. Delhaye and H. P. Saal, eLife, DOI:10.7554/eLife.93554.1.

      J. N. Israelachvili, Intermolecular and Surface Forces, Academic Press, 2011.

      S. Das, N. Cadirov, S. Chary, Y. Kaufman, J. Hogan, K. L. Turner and J. N. Israelachvili, J R Soc Interface, 2015, 12, 20141346.

      B. N. J. Persson, O. Albohr, C. Creton and V. Peveri, The Journal of Chemical Physics, 2004, 120, 8779–8793.

      L. Skedung, M. Arvidsson, J. Y. Chung, C. M. Stafford, B. Berglund and M. W. Rutland, Sci Rep, 2013, 3, 2617.

      K. Viswanathan, N. K. Sundaram and S. Chandrasekar, Soft Matter, 2016, 12, 5265–5275.

      M. Wiertlewski, C. Hudin and V. Hayward, in 2011 IEEE World Haptics Conference, 2011, pp. 25– 30.

      M. Fehlberg, E. Monfort, S. Saikumar, K. Drewing and R. Bennewitz, IEEE Transactions on Haptics, 2024, 17, 957–963.

    1. Author response:

      Public Reviews:<br /> Reviewer #1 (Public review):

      Summary:

      The manuscript discusses the role of phosphorylated ubiquitin (pUb) by PINK1 kinase in neurodegenerative diseases. It reveals that elevated levels of pUb are observed in aged human brains and those affected by Parkinson's disease (PD), as well as in Alzheimer's disease (AD), aging, and ischemic injury. The study shows that increased pUb impairs proteasomal degradation, leading to protein aggregation and neurodegeneration. The authors also demonstrate that PINK1 knockout can mitigate protein aggregation in aging and ischemic mouse brains, as well as in cells treated with a proteasome inhibitor. While this study provided some interesting data, several important points should be addressed before being further considered.

      Strengths:

      (1) Reveals a novel pathological mechanism of neurodegeneration mediated by pUb, providing a new perspective on understanding neurodegenerative diseases.

      (2) The study covers not only a single disease model but also various neurodegenerative diseases such as Alzheimer's disease, aging, and ischemic injury, enhancing the breadth and applicability of the research findings.

      Weaknesses:

      (1) PINK1 has been reported as a kinase capable of phosphorylating Ubiquitin, hence the expected outcome of increased p-Ub levels upon PINK1 overexpression. Figures 5E-F do not demonstrate a significant increase in Ub levels upon overexpression of PINK1 alone, whereas the evident increase in Ub expression upon overexpression of S65A is apparent. Therefore, the notion that increased Ub phosphorylation leads to protein aggregation in mouse hippocampal neurons is not yet convincingly supported.

      Indeed, overexpression of sPINK1* alone caused little change in Ub levels in the soluble fraction (Figure 5E), which is expected. Ub in the soluble fraction is in a relatively stable, buffered state. However, overexpression of sPINK1* resulted in an increase in Ub levels in the insoluble fraction, indicating protein aggregation. The molecular weight of Ub in the insoluble fraction was predominantly below 70 kDa, implying that phosphorylation inhibits Ub chain elongation.

      To further examine this, we used the Ub/S65A mutant to antagonize Ub phosphorylation, and found that the aggregation at low molecular weight was significantly reduced, indicating a partial restoration of proteasomal activity. The increase in Ub levels in both the soluble and insoluble fractions likely results from the high rate of ubiquitination driven by the elevated levels of Ub. Notably, the overexpressed Ub/S65A was detected in the Western blot using the wild-type Ub antibody, which accounts for the apparently increased Ub level.

      When overexpressing Ub/S65E, we again saw an increase in Ub levels in the insoluble fraction (but no increase in the soluble fraction), with low molecular weight bands even more prominent than those observed with sPINK1* transfection. These findings collectively support the conclusion that sPINK1* promotes protein aggregation through Ub phosphorylation.

      (2) The specificity of PINK1 and p-Ub antibodies requires further validation, as a series of literature indicate that the expression of the PINK1 protein is relatively low and difficult to detect under physiological conditions.

      We acknowledge the challenges in achieving optimal specificity for commercially available and custom-generated antibodies targeting PINK1 and pUb, particularly given the low endogenous levels of these proteins under physiological conditions. Despite these limitations, we observed robust immunofluorescent staining for PINK1 (Figures 1A, 1C, and 1G) and pUb (Figures 1B, 1D, and 1G) in human brain samples from Alzheimer's disease (AD) patients, as well as in mouse brains from models of AD and cerebral ischemia. The significant elevation of PINK1 and pUb under these pathological conditions likely accounts for the clear visualization. To validate antibody specificity, we have included images from pink1-/- mice as negative controls in the revised manuscript (Figure 1C and 1D, third panel).

      In addition, we detected a significant increase in pUb levels in aged mouse brains compared to young ones (Figures 1E and 1F). Notably, in pink1-/- mice, pUb levels remained unchanged between young and aged groups, despite some background signal, further supporting the conclusion that pUb accumulation during aging is PINK1-dependent.

      In HEK293 cells, pink1-/- cells served as a negative control for PINK1 (Figure 2B and 2C) and for pUb (Figure 2D and 2E). While the Western blot using the pUb antibody displayed some nonspecific background, pUb levels in pink1-/- cells remained unchanged across all MG132 treatment conditions (Figures 2D and 2E), further attesting the reliability of our findings.

      (3) In Figure 6, relying solely on Western blot staining and Golgi staining under high magnification is insufficient to prove the impact of PINK1 overexpression on neuronal integrity and cognitive function. The authors should supplement their findings with immunostaining results for MAP2 or NeuN to demonstrate whether neuronal cells are affected.

      Thank you for raising this important point. We included NeuN immunofluorescent staining in Figure 5—figure supplement 2 of the original manuscript. The results demonstrate a significant loss of NeuN-positive cells in the hippocampus following Ub/S65E overexpression, while no apparent change in NeuN-positive cells was observed with sPINK1* transfection alone. These findings provide evidence of neuronal loss in response to Ub/S65E, further supporting the impact of pUb elevation on neuronal integrity.

      While we did not perform MAP2 immunostaining, we included complementary analyses to assess neuronal integrity. Specifically, we performed Western blotting to determine MAP2 protein levels and used Golgi staining to study neuronal morphology and synaptic structure in greater detail. These analyses revealed that overexpression of sPINK1* or Ub/S65E decreased MAP2 levels and caused damage to synaptic structures (Figures 6F and 6H). Importantly, the deleterious effects of sPINK1* overexpression could be rescued by co-expression of Ub/S65A, further underscoring the role of pUb in mediating these changes.

      Together, our NeuN immunostaining, MAP2 analysis, and Golgi staining provide strong evidence for the impact of PINK1 overexpression and pUb elevation on neuronal integrity and synaptic health. We believe these complementary approaches sufficiently address the reviewer’s concern and highlight the pathological consequences of elevated pUb levels.

      (4) The authors should provide more detailed figure captions to facilitate the understanding of the results depicted in the figures.

      Figure captions will be updated with more details in the revised manuscript.

      (5) While the study proposes that pUb promotes neurodegeneration by affecting proteasomal function, the specific molecular mechanisms and signaling pathways remain to be elucidated.

      The specific molecular mechanisms and signaling pathways through which pUb promotes neurodegeneration are likely multifaceted and interconnected. Mitochondrial dysfunction appears to be a central contributor to neurodegeneration following sPINK1* overexpression. This is supported by (1) an observed increase in full-length PINK1, indicative of impaired mitochondrial quality control, and (2) proteomic data revealing enhanced mitophagy at 30 days post-transfection and substantial mitochondrial injury by 70 days post-transfection. The progressive damage to mitochondria caused by protein aggregates can cause further neuronal injury and degeneration.

      In addition, reduced proteasomal activity may result in the accumulation of inhibitory proteins that are normally degraded by the ubiquitin-proteasome system. Our proteomics analysis identified a >54-fold increase in CamK2n1 (UniProt ID: Q6QWF9), an endogenous inhibitor of CaMKII activation, following sPINK1* overexpression. This is particularly significant because the accumulation of CamK2n1 could suppress CaMKII activation and, subsequently, inhibit the CREB signaling pathway (illustrated below). As CREB is essential for synaptic plasticity and neuronal survival, its inhibition may further amplify neurodegenerative processes.

      While our study identifies proteasomal dysfunction and mitochondrial damage as key initial triggers, downstream effects—such as disruptions in signaling pathways like CaMKII-CREB—likely contribute to a broader cascade of pathological events. These findings highlight the complexity of pUb-mediated neurodegeneration and suggest that further exploration of downstream mechanisms is necessary to fully elucidate the pathways involved.

      We plan to include the proteomics data, in the revised manuscript, of mouse brain tissues at 30 days and 70 days post-transfection, to further highlight this downstream effect upon proteasomal dysfunction.

      Author response image 1.

      Reviewer #2 (Public review):

      Summary:

      The manuscript makes the claim that pUb is elevated in a number of degenerative conditions including Alzheimer's Disease and cerebral ischemia. Some of this is based on antibody staining which is poorly controlled and difficult to accept at this point. They confirm previous results that a cytosolic form of PINK1 accumulates following proteasome inhibition and that this can be active. Accumulation of pUb is proposed to interfere with proteostasis through inhibition of the proteasome. Much of the data relies on over-expression and there is little support for this reflecting physiological mechanisms.

      Weaknesses:

      The manuscript is poorly written. I appreciate this may be difficult in a non-native tongue, but felt that many of the problems are organisational. Less data of higher quality, better controls and incision would be preferable. Overall the referencing of past work is lamentable.

      Methods are also very poor and difficult to follow.<br /> Until technical issues are addressed I think this would represent an unreliable contribution to the field.

      (1) Antibody specificity and detection under pathological conditions

      We acknowledge the limitations of commercially available antibodies for detecting PINK1 and pUb. Despite these challenges, our findings demonstrate a significant increase in PINK1 and pUb levels under pathological conditions, such as Alzheimer's disease (AD) and ischemia. Additionally, we observed an increase in pUb level during brain aging, further highlighting its relevance in this particular physiological process. To ensure reliable quantification of PINK1 and pUb levels, we used pink1-/- mice and HEK293 cells as negative controls. For example, PINK1 levels were extremely low in control cells but increased dramatically after 2 hours of oxygen-glucose deprivation (OGD) and 6 hours of reperfusion (Figure 1H). Together, these controls validate that the observed elevations in PINK1 and pUb are specific and linked to pathological or certain physiological conditions.

      (2)  Overexpression as a model for pathological conditions

      To investigate whether the inhibitory effects of sPINK1* on the ubiquitin-proteasome system (UPS) are dependent on its kinase activity, we utilized a kinase-dead version of sPINK1* as a negative control. Since PINK1 has multiple substrates, we further explored whether its effects on UPS inhibition were mediated specifically by ubiquitin phosphorylation. For this, we used Ub/S65A (a phospho-null mutant) to antagonize Ub phosphorylation by sPINK1*, and Ub/S65E (a phospho-mimetic mutant) to mimic phosphorylated Ub. These well-defined controls ensured the robustness of our conclusions.

      While overexpression does not perfectly replicate physiological conditions, it serves as a valuable model for studying pathological scenarios such as neurodegeneration and brain aging, where pUb levels are known to increase. For example, we observed a 30.4% increase in pUb levels in aged mouse brains compared to young brains (Figure 1F). Similarly, in our sPINK1* overexpression model, pUb levels increased by 43.8% and 59.9% at 30- and 70-days post-transfection, respectively, compared to controls (Figures 5A and 5C). Notably, co-expression of sPINK1* with Ub/S65A almost entirely prevented sPINK1* accumulation (Figure 5B), indicating that an active UPS can efficiently degrade sPINK1*. Collectively, these findings show that sPINK1* accumulation inhibits UPS activity, a defect that can be rescued by the phospho-null Ub mutant. Thus, this overexpression model closely mimics pathological conditions and offers valuable insights into pUb-mediated proteasomal dysfunction.

      (3) Organization of the manuscript

      We believe the structure of the manuscript is justified and systematically addresses the key aspects of the study in a logic flow:

      (a) Evidence for the increase of PINK1 and pUb in multiple pathological and physiological conditions.

      (b) Identification of the sources and consequences of sPINK1 and pUb elevation.

      (c) Mechanistic insights into how pUb inhibits UPS-mediated degradation.

      (d) Validation of these findings using pink1-/- mice and cells.

      (e) Evidence of the reciprocal relationship between proteasomal inhibition and pUb elevation, culminating in neurodegeneration.

      (f) Demonstration of elevated pUb levels and protein aggregation in the hippocampus following sPINK1* overexpression, supported by proteomic analyses, behavioral tests, Western blotting, and Golgi staining.

      Thus, this organization provides a clear and cohesive narrative, culminating in the demonstration that sPINK1* overexpression induces hippocampal neuron degeneration.

      (4) Revisions to writing, referencing, and methodology

      We will improve the clarity and flow of the manuscript, add more references to properly acknowledge prior work, and incorporate additional details into the Methods section to enhance readability and reproducibility. These improvements should address the organizational and technical concerns raised, while strengthen the overall quality of the manuscript.

      Reviewer #3 (Public review):

      Summary:

      This study aims to explore the role of phosphorylated ubiquitin (pUb) in proteostasis and its impact on neurodegeneration. By employing a combination of molecular, cellular, and in vivo approaches, the authors demonstrate that elevated pUb levels contribute to both protective and neurotoxic effects, depending on the context. The research integrates proteasomal inhibition, mitochondrial dysfunction, and protein aggregation, providing new insights into the pathology of neurodegenerative diseases.

      Strengths:

      - The integration of proteomics, molecular biology, and animal models provides comprehensive insights.

      - The use of phospho-null and phospho-mimetic ubiquitin mutants elegantly demonstrates the dual effects of pUb.

      - Data on behavioral changes and cognitive impairments establish a clear link between cellular mechanisms and functional outcomes.

      Weaknesses:

      - While the study discusses the reciprocal relationship between proteasomal inhibition and pUb elevation, causality remains partially inferred.

      The reciprocal cycle between proteasomal inhibition and pUb elevation can be initiated by various factors that impair proteasomal activity. These factors include Aβ accumulation, ATP depletion, reduced expression of proteasome components, and covalent modifications of proteasomal subunits—all well-established contributors to the progressive decline in proteasome function. Once initiated, this cycle would become self-perpetuating, with the accumulation of sPINK1 and pUb driving a feedback loop of deteriorating proteasomal activity.

      In the current study, this reciprocal relationship between sPINK1/pUb elevation and proteasomal dysfunction is depicted in Figure 4A. Our results demonstrate that increased sPINK1 or PINK1 levels, such as through overexpression, can initiate this cycle. Crucially, co-expression of Ub/S65A effectively rescues the cells from this cycle, highlighting the pivotal role of pUb in driving proteasomal inhibition and establishing causality in this relationship. At the animal level, pink1 knockout could prevent protein aggregation upon aging and cerebral ischemia (Figures 1E and 1G).

      Mitochondrial injury is a likely source of elevated PINK1 and pUb levels. A recent study showed that efficient mitophagy is necessary to prevent pUb accumulation (bioRxiv 2023.02.14.528378), suggesting that mitochondrial damage can trigger this cycle. In another study (bioRxiv 2024.07.03.601901), the authors found that mitochondrial damage could enhance PINK1 transcription, further increasing cytoplasmic PINK1 levels and exacerbating the cycle.

      - The role of alternative pathways, such as autophagy, in compensating for proteasomal dysfunction is underexplored.

      Elevated sPINK1 has been reported to enhance autophagy (Autophagy 2016, 12: 632-647), potentially compensating for the impaired UPS. One mechanism involves the phosphorylation of p62 by sPINK1, which enhances autophagy activity. In our study, we did observe increased autophagic activity upon sPINK1* overexpression, as shown in Figure 2I (middle panel, without BALA). This increased autophagy may help degrade ubiquitinated proteins induced by puromycin, partially compensating for the proteasomal dysfunction.

      This compensation might explain why protein aggregation only increased slightly, though statistically significant, at 70 days post sPINK1* transfection (Figure 5F). Additionally, we observed a slight, though statistically insignificant, increase in LC3II levels in the hippocampus of mouse brains at 70 days post sPINK1* transfection (Figure 5—figure supplement 6), further supporting the notion of autophagy activation.

      However, while autophagy may provide some compensation, its effect is likely limited. Autophagy and UPS differ significantly in their roles and mechanisms of degradation. Autophagy is a bulk degradation pathway that is generally non-selective, targeting long-lived proteins, damaged organelles, and intracellular pathogens. In contrast, the UPS is highly selective, primarily degrading short-lived regulatory proteins, misfolded proteins, and proteins tagged for degradation.

      Together, we found that sPINK1* overexpression enhanced autophagy-mediated protein degradation while simultaneously impairing UPS-mediated degradation. This suggests that while autophagy may provide partial compensation for proteasomal dysfunction, it is not sufficient to fully counterbalance the selective degradation functions of the UPS.

      - The immunofluorescence images in Figure 1A-D lack clarity and transparency. It is not clear whether the images represent human brain tissue, mouse brain tissue, or cultured cells. Additionally, the DAPI staining is not well-defined, making it difficult to discern cell nuclei or staging. To address these issues, lower-magnification images that clearly show the brain region should be provided, along with improved DAPI staining for better visualization. Furthermore, the Results section and Figure legends should explicitly indicate which brain region is being presented. These concerns raise questions about the reliability of the reported pUb levels in AD, which is a critical aspect of the study's findings.

      We will include low-magnification images in the supplementary figures of the revised manuscript to provide a broader context for the immunofluorescence data presented in Figure 1. DAPI staining at higher magnifications will also be provided to improve visualization of cell nuclei and overall tissue structure. Additionally, we will indicate the brain regions examined in the corresponding figure legends, and incorporate more details in the Results section to provide clearer descriptions of the samples and brain regions analyzed.

      The human brain samples presented in Figure 1 are from the cingulate gyrus region of Alzheimer's disease (AD) patients. Our analysis revealed that PINK1 is primarily localized within cell bodies, while pUb is more abundant around Aβ plaques, likely in nerve terminals. These additional clarifications and supplementary figures should provide greater transparency and improve the reliability of our findings.

      - Figure 4B should also indicate which brain region is being presented.

      The images were taken for layer III-IV in the neocortex of mouse brains, which information will be incorporated in the figure legend of the revised manuscript.

    1. Author Response:

      This work presents valuable information about the specificity and promiscuity of toxic effector and immunity protein pairs. The evidence supporting the claims of the authors is currently incomplete, as there is concern about the methodology used to analyze protein interactions, which did not take potential differences in expression levels, protein folding, and/or transient interaction into account. Other methods to measure the strength of interactions and structural predictions would improve the study. The work will be of interest to microbiologists and biochemists working with toxin-antitoxin and effector-immunity proteins.

      We thank the reviewers for considering this manuscript. We agree that this manuscript provides a valuable and cross-discipline introduction to new EI pair protein families where we focus on the EI pair’s flexibility and impacts on community structure. As such, we believe we have provided a solid foundation for future studies to examine non-cognate interactions and their possible effects on microbial communities. This, by definition, leaves some areas “incomplete” and, therefore, open for further investigations. While the methods we show do take into account potential differences in binding assays, we will more explicitly address how “expression, protein folding, and/or transient binding” may play into this expanded EI pair model upon revision and temper the discussion of the proposed model. We have responded to the reviewers’ public comments (italicized below).

      Public Reviews:

      Note: Reviewer 1, who appeared to focus on a subset of the manuscript rather than the whole, based their comments on several inaccuracies, which we discuss below. We found the tone in this reviewer's comments to be, at times, inappropriate, e.g., using "harsh" and "simply too drastic" to imply that common structure-function analyses were outside of the field-standard methods. We also note that the reviewer took a somewhat atypical step in reviewing this manuscript by running and analyzing the potential protein-complex data in AlphaFold2 but did not discuss areas of low confidence within that model that may contradict their conclusions. We are concerned their approach muddled valid scientific criticisms with problematic conclusions.

      Reviewer #1 (Public Review):

      In this manuscript, Knecht, Sirias et al describe toxin-immunity pair from Proteus mirabilis. Their observations suggest that the immunity protein could protect against non-cognate effectors from the same family. They analyze these proteins by dissecting them into domains and constructing chimeras which leads them to the conclusion that the immunity can be promiscuous and that the binding of immunity is insufficient for protective activity.

      Strengths:

      The manuscript is well written and the data are very well presented and could be potentially interesting. The phylogenetic analysis is well done, and provides some general insights.

      Weaknesses:

      1) Conclusions are mostly supported by harsh deletions and double hybrid assays. The later assays might show binding, but this method is not resolutive enough to report the binding strength. Proteins could still bind, but the binding might be weaker, transient, and out-competed by the target binding.

      The phrasing of structure-function analyses as “harsh” is a bit unusual, as other research groups regularly use deletions and hybrid studies. Given the known caveats to deletion and domain substitutions, we included point-mutation analyses for both the effector and immunity proteins, as found on lines 105 - 113 and 255 - 261 in the current manuscript. These caveats are also why we coupled the in vitro binding analyses with in vivo protection experiments in two distinct experimental systems (E. coli and P. mirabilis). Based on this manuscript’s introductory analysis (where we define and characterize the genes, proteins, interactions, phylogenetics, and incidences in human microbiomes), the next apparent questions are beyond the scope of this study. Future approaches would include analyzing purified proteins from these effector (E) and immunity (I) protein families using biochemical assays, such as X-ray crystallography, circular dichroism spectroscopy, among others.

      (Interestingly, most papers in the EI field do not measure EI protein affinity (Jana et al., 2019, Yadav et al., 2021). Notable exceptions are earlier colicin research (Wallis et al., 1995) and a new T6SS EI paper (Bosch et al., 2023) published as we submitted this manuscript.)

      2) While the authors have modeled the structure of toxin and immunity, the toxin-immunity complex model is missing. Such a model allows alternative, more realistic interpretation of the presented data. Firstly, the immunity protein is predicted to bind contributing to the surface all over the sequence, except the last two alpha helices (very high confidence model, iPTM>0.8). The N terminus described by the authors contributes one of the toxin-binding surfaces, but this is not the sole binding site. Most importantly, other parts of the immunity protein are predicted to interact closer to the active site (D-E-K residues). Thus, based on the AlphaFold model, the predicted mechanism of immunization remains physically blocking the active site. However, removing the N terminal part, which contributes large interaction surface will directly impact the binding strength. Hence, the toxin-immunity co-folding model suggests that proper binding of immunity, contributed by different parts of the protein, is required to stabilize the toxin-immunity complex and to achieve complete neutralization. Alternative mechanisms of neutralization might not be necessary in this case and are difficult to imagine for a DNAse.

      In response to the reviewer’s comment, we again reviewed the RdnE-RdnI AlphaFold2 complex predictions with the most updated version of ColabFold (1.5.2-patch with PDB100 and MMseq2) and have included them at the end of the responses [1].

      However, the literature reports that computational predictions of E-I complexes often do not match experimental structural results (Hespanhol et al., 2022, Bosch et al., 2023). As such, we chose not to include the predicted cognate and non-cognate RdnE-I complexes from ColabFold (which uses AlphaFold2) and will not include this data in revised manuscripts. (It is notable that reviewer 1 found the proposed expanded model and research so interesting as to directly input and examine the AI-predicted RdnE-RdnI protein interactions in AlphaFold2.)

      Discussion of the prevailing toxin-immunity complex model is in the introduction (lines 45-48) and Figure 5E. Further, there are various known mechanisms for neutralizing nucleases and other T6SS effectors, which we briefly state in the discussion (lines 359 - 361). More in-depth, these molecular mechanisms include active-site blocking (Benz et al., 2012), allosteric-site binding (Kleanthous et al., 1999 and Lu et al., 2014), enzymatic neutralization of the target (Ting et al., 2021), and structural disruption of both the active and binding sites (Bosch et al., 2023). Given this diversity of mechanisms, we did not presume to speculate on the as-of-yet unknown mechanism of RdnI protection.

      3) Dissection of a toxin into two domains is also not justified from a structural point of view, it is probably based on initial sequence analyses. The N terminus (actually previously reported as Pone domain in ref 21) is actually not a separate domain, but an integral part of the protein that is encased from both sides by the C terminal part. These parts might indeed evolve faster since they are located further from the active site and the central core of the protein. I am happy to see that the chimeric toxins are active, but regarding the conservation and neutralization, I am not surprised, that the central core of the protein fold is highly conserved. However, "deletion 2" is quite irrelevant - it deletes the central core of the protein, which is simply too drastic to draw any conclusions from such a construct - it will not fold into anything similar to an original protein, if it will fold properly at all.

      The reviewer’s comment highlights why we turned to the chimera proteins to dissect the regions of RdnE (formerly IdrD-CT), as the deletions could result in misfolded proteins. (We initially examined RdnE in the years before the launch of AlphaFold2.) However, the reviewer is incorrect regarding the N-terminus of RdnE. The PoNe domain, while also a subfamily of the PD-(D/E)XK superfamily, forms a distinct clade of effectors from the PD-(D/E)XK domain in RdnE (formally IdrD-CT) as seen in Hespanhol et al., 2022; this is true for other DNAse effectors as well. Many studies analyzing effectors within the PD-(D/E)XK superfamily only focus on the PD-(D/E)XK domain, removing just this domain from the context of the whole protein (Hespanhol et al., 2022; Jana et al., 2019). Of note, in RdnE, this region alone (containing the DNA-binding domain) is insufficient for DNAse activity (unlike in PoNe).

      4) Regarding the "promiscuity" there is always a limit to how similar proteins are, hence when cross-neutralization is claimed authors should always provide sequence similarities. This similarity could also be further compared in terms of the predicted interaction surface between toxin and immunity.

      Reviewer 1 points out a fundamental property of protein-protein interactions that has been isolated away from the impacts of such interactions on bacterial community structure. We have provided the whole protein alignments in supplemental figure 3, the summary images in Figure 3D, and the protein phylogenetic trees in Figure 3C. We encourage others to consider the protein alignments as percent amino acid sequence similarity is not necessarily a good gauge for protein function and interactions. RuBisCo is one example of how protein sequence similarity can be small while functions remain highly conserved. These data are publicly available on the OSF website associated with this manuscript https://osf.io/scb7z/, and we hope the community explores the data there.

      In consideration of the enthusiasm to deeply dive into the primary research data, we have included the pairwise sequence identities across the entire proteins here: Proteus RdnI vs. Rothia RdnI: 23.6%; Proteus RdnI vs. Prevotella RdnI: 16.3%, Proteus RdnI vs. Pseudomonas RdnI: 14.6%; Rothia RdnI vs. Prevotella RdnI: 22.4%, Rothia RdnI vs. Pseudomonas RdnI: 17.6%; Prevotella RdnI vs. Pseudomonas RdnI: 19.5%. (As stated in response to reviewer 1 comment 2, we do not find it appropriate to make inferences based on AlphaFold2-predicted protein complexes.)

      Overall, it looks more like a regular toxin-immunity couple, where some cross-reactions with homologues are possible, depending on how far the sequences have deviated. Nevertheless, taking all of the above into account, these results do not challenge toxin-immunity specificity dogma.

      In this manuscript, we did not intend to dismiss the E-I specificity model but rather point out its limitations and propose an important expansion of that model that accounts for cross-protection and survival against attacks from other genera. We agree that it is commonly considered that deviations in amino acid sequence over time could result in cross-binding and protection (see lines 364-368). However, the impacts of such cross-binding on community structure, bacterial survival, and strain evolution have rarely been considered or addressed in prior literature, with exceptions such as in Zhang et al., 2013 and Bosch et al., 2023. One key insight we propose and show in this manuscript is that cross-binding can be a fitness benefit in mixed communities; therefore, it could be selected for evolutionarily (lines 378-380), even potentially in host microbiomes.

      Reviewer #2 (Public Review):

      Summary:

      The manuscript by Knecht et al entitled "Non-cognate immunity proteins provide broader defenses against interbacterial effectors in microbial communities" aims at characterizing a new type VI secretion system (T6SS) effector immunity pair using genetic and biochemical studies primarily focused on Proteus mirabilis and metagenomic analysis of human-derived data focused on Rothia and Prevotella sequences. The authors provide evidence that RdnE and RdnI of Proteus constitute an E-I pair and that the effector likely degrades nucleic acids. Further, they provide evidence that expression of non-cognate immunity derived from diverse species can provide protection against RdnE intoxication. Overall, this general line of investigation is underdeveloped in the T6SS field and conceptually appropriate for a broad audience journal. The paper is well-written and, aside from a few cases, well-cited. As detailed below however, there are several aspects of this paper where the evidence provided is somewhat insufficient to support the claims. Further, there are now at least two examples in the literature of non-cognate immunity providing protection against intoxication, one of which is not cited here (Bosch et al PMID 37345922 - the other being Ting et al 2018). In general therefore I think that the motivating concept here in this paper of overturning the predominant model of interbacterial effector-immunity cognate interactions is oversold and should be dialed back.

      We agree that analyses focusing on flexible non-cognate interactions and protection are underdeveloped within the T6SS field and are not fully explored within a community structure. These ideas are rapidly growing in the field, as evidenced by the references provided by the reviewer. As stated earlier, we did not intend to overturn the prevailing model but rather propose an expanded model that accounts for protection against attacks from foreign genera.

      Strengths:

      One of the major strengths of this paper is the combination of diverse techniques including competition assays, biochemistry, and metagenomics surveys. The metagenomic analysis in particular has great potential for understanding T6SS biology in natural communities. Finally, it is clear that much new biology remains to be discovered in the realm of T6SS effectors and immunity.

      Weaknesses:

      The authors have not formally shown that RdnE is delivered by the T6SS. Is it the case that there are not available genetics tools for gene deletion for the BB2000 strain? If there are genetic tools available, standard assays to demonstrate T6SS-dependency would be to interrogate function via inactivation of the T6SS (e.g. by deleting tssC).

      Our research group showed that the T6SS secretes RdnE (previously IdrD) in Wenren et al., 2013 (cited in lines 71-73). We later confirmed T6SS-dependent secretion by LC-MS/MS (Saak et al., 2017).

      For swarm cross-phyla competition assays (Figure 4), at what level compared to cognate immunity are the non-cognate immunity proteins being expressed? This is unclear from the methods and Figure 4 legend and should be elaborated upon. Presumably these non-cognate immunity proteins are being overexpressed. Expression level and effector-to-immunity protein stoichiometry likely matters for interpretation of function, both in vitro as well as in relevant settings in nature. It is important to assess if native expression levels of non-cognate cross-phyla immunity (e.g. Rothia and Prevotella) protect similarly as the endogenously produced cognate immunity. This experiment could be performed in several ways, for example by deleting the RdnE-I pair and complementing back the Rothia or Prevotella RdnI at the same chromosomal locus, then performing the swarm assay. Alternatively, if there are inducible expression systems available for Proteus, examination of protection under varying levels of immunity induction could be an alternate way to address this question. Western blot analysis comparing cognate to non-cognate immunity protein levels expressed in Proteus could also be important. If the authors were interested in deriving physical binding constants between E and various cognate and non-cognate I (e.g. through isothermal titration calorimetry) that would be a strong set of data to support the claims made. The co-IP data presented in supplemental Figure 6 are nice but are from E. coli cells overexpressing each protein and do not fully address the question of in vivo (in Proteus) native expression.

      P. mirabilis strain ATCC29906 does not encode the rdnE and rdnI genes on the chromosome (NCBI BioSample: SAMN00001486) (line 151). Production of the RdnI proteins, including the cognate Proteus RdnI, comes from equivalent transgenic expression vectors. Specifically, the rdnI genes were expressed under the flaA promoter in P. mirabilis strain ATCC29906 (Table 1) for the swarm competition assays found in Figure 2C and Figure 4. This promoter results in constitutive expression in swarming cells (Belas et al., 1991; Jansen et al., 2003).

      Lines 321-324, the authors infer differences between E and I in terms of read recruitment (greater abundance of I) to indicate the presence of orphan immunity genes in metagenomic samples (Figure 5A-D). It seems equally or perhaps more likely that there is substantial sequence divergence in E compared to the reference sequence. In fact, metagenomes analyzed were required only to have "half of the bases on reference E-I sequence receiving coverage". Variation in coverage again could reflect divergent sequence dipping below 90% identity cutoff. I recommend performing metagenomic assemblies on these samples to assess and curate the E-I sequences present in each sample and then recalculating coverage based on the exact inferred sequences from each sample.

      This comment raises the challenges with metagenomic analyses. It was difficult to balance specificity to a particular species’ DNA sequence with the prevalence of any homologous sequence in the sample. Given the distinction in binding interactions among the examined four species, we opted to prioritize specificity, accepting that we were losing access to some rdnE and rdnI sequences in that decision. We chose a 90% identity cutoff, which, through several in silica controls, ensured that each sequence we identified was the rdnE or rdnI gene from that specific species. For the Version of Record, we will revisit this decision and consider trying to account for sequence divergence by lowering the identity cutoffs as suggested.

      A description of gene-level read recruitment in the methods section relating to metagenomic analysis is lacking and should be provided.

      Noted. We will also include the raw code and sequences on the OSF website associated with this manuscript https://osf.io/scb7z/.

      Reviewer #3 (Public Review):

      [...] Strengths:

      The authors presented a strong rationale in the manuscript and characterized the molecular mechanism of the RdnE effector both in vitro and in the heterologous expression model. The utilization of the bacterial two-hybrid system, along with the competition assays, to study the protective action of RdnI immunity is informative. Furthermore, the authors conducted bioinformatic analyses throughout the manuscript, examining the primary sequence, predicted structural, and metagenomic levels, which significantly underscore the significance and importance of the EI pair.

      Weaknesses:

      1. The interaction between RdnI and RdnE appears to be complex and requires further investigation. The manuscript's data does not conclusively explain how RdnI provides a "promiscuous" immunity function, particularly concerning the RdnI mutant/chimera derivatives. The lack of protection observed in these cases might be attributed to other factors, such as a decrease in protein expression levels or misfolding of the proteins. Additionally, the transient nature of the binding interaction could be insufficient to offer effective defenses.

      Yes, we agree with the reviewer and hope that grant reviewers’ share this colleague’s enthusiasm for understanding the detailed molecular mechanisms of RdnE-RdnI binding across genera. We will continue to emphasize such caveats as the next frontier is clearly understanding the molecular mechanisms for RdnI cognate or non-cognate protection. We address the concerns regarding expression levels in the response to reviewer 2, comment 2.

      1. The results from the mixed population competition lack quantitative analysis. The swarm competition assays only yield binary outcomes (Yes or No), limiting the ability to obtain more detailed insights from the data.

      The mixed swam assay is needed when studying T6SS effectors that are primarily secreted during Proteus’ swarming activity (Saak et al. 2017, Zepeda-Rivera et al. 2018). This limitation is one reason we utilize in vitro, in vivo, and bioinformatic analyses. Though the swarm competition assay yields a binary outcome, we are confident that the observed RdnI protection is due to interaction with a trans-cell RdnE via an active T6SS. By contrast, many manuscripts report co-expression of the EI pair (Yadev et al., 2021, Hespanhol et al., 2022) rather than secreted effectors, as we have achieved in this manuscript.

      1. The discovery of cross-species protection is solely evident in the heterologous expression-competition model. It remains uncertain whether this is an isolated occurrence or a common characteristic of RdnI immunity proteins across various scenarios. Further investigations are necessary to determine the generality of this behavior.

      We agree, which is why we submitted this paper as a launching point for further investigations into the generality of non-cognate interactions and their potential impact on community structure.

      Comments from Reviewing Editor:

      • In addition to the references provided by reviewer#2, the first manuscript to show non-cognate binding of immunity proteins was Russell et al 2012 (PMID: 22607806).
      • IdrD was shown to form a subfamily of effectors in this manuscript by Hespanhol et al 2022 PMID: 36226828 that analyzed several T6SS effectors belonging to PDDExK, and it should be cited.

      We appreciate that the reviewer and eLife staff pointed out missed citations. A revised manuscript will incorporate those studies and cite them appropriately.

      [1] The Proteus RdnE in complex with either the Prevotella or Pseudomonas RdnI showed low confidence at the interface (pIDDT ~50-70%); this AI-predicted complex might support the lack of binding seen in the bacterial two-hybrid assay. On the other hand, the Proteus and Rothia RdnI N-terminal regions show higher confidence at the interface with RdnE. Despite this, the C-terminus of the Proteus RdnI shows especially low confidence (pIDDT ~50%) where it might interact near RdnE’s active site (as suggested by reviewer 1). Given this low confidence and the already stated inaccuracies of AI-generated complexes, we would rather wait for crystallization data to inform potential protection mechanisms of RdnI.

      Author response image 1.

    1. Author response:

      Description of the planned revisions

      Reviewer #1 (Evidence, reproducibility and clarity):

      Summary

      The authors focused on medaka retinal organoids to investigate the mechanism underlying the eye cup morphogenesis. The authors succeeded to induce lens formation in fish retinal organoids using 3D suspension culture with minimal growth factor-containing media containing the Hepes. At day 1, Rx3:H2B-GFP+ cells appear in the surface region of organoids. At day 1.5, Prox1+cells appear in the interface area between the organoid surface and the core of central cell mass, which develops a spherical-shaped lens later. So, Prox1+ cells covers the surface of the internal lens cell core. At day 2, foxe3:GFP+ cells appear in the Prox1+ area, where early lens fiber marker, LFC, starts to be expressed. In addition, foxe3:GFP+ cells show EdU+ incorporation, indicating that foxe3:GFP+ cells have lens epithelial cell-characters. At day 4, cry:EGFP+ cells differentiate inside the spherical lens core, whose the surface area consists of LFC+ and Prox1+ cells. Furthermore, at day 4, the lens core moves towards the surface of retinal organoids to form an eye-cup like structure, although this morphogenesis "inside out" mechanism is different from in vivo cellular "outside -in" mechanism of eye cup formation. From these data, the authors conclude that optic cup formation, especially the positioning of the lens, is established in retinal organoids though the different mechanism of in vivo morphogenesis.

      Overall, manuscript presentation is nice. However, there are still obscure points to understand background mechanism. My comments are shown below.

      Major comments

      (1) At the initial stage of retinal organoid morphogenesis, a spherical lens is centrally positioned inside the retinal organoids, by covering a central lens core by the outer cell sheet of retinal precursor cells. I wonder if the formation of this structure may be understood by differential cell adhesive activity or mechanical tension between lens core cells and retinal cell sheet, just like the previous study done by Heisenberg lab on the spatial patterning of endoderm, mesoderm and ectoderm (Nat. Cell Biol. 10, 429 - 436 (2008)). Lens core cells may be integrated inside retinal cell mass by cell sorting through the direct interaction between retinal cells and lens cells, or between lens cells and the culture media. After day 1, it is also possible to understand that lens core moves towards the surface of retinal organoids, if adhesive/tensile force states of lens core cells may be change by secretion of extracellular matrix. I wonder if the authors measure physical property, adhesive activity and solidness, of retinal precursor cells and lens core cells. If retinal organoids at day 1 are dissociated and cultured again, do they show the same patterning of internal lens core covering by the outer retinal cell sheet?

      The question, whether different adhesive activity is involved in cell sorting and lens formation is indeed very intriguing. To address this point, we will include additional experiment (see Revision Plan, experiment 1). This experiment will be based on the dissociation and re-aggregation of lens-forming organoids as suggested by the reviewer. To monitor cell type specific sorting, we will employ a lens progenitor reporter line Foxe3::GFP and the retina-specific Rx2::H2B-RFP. If different adhesive activities of lens and retinal progenitor cells are involved and drive the process of cell sorting, dissociation and re-aggregation will result in cell sorting based on their identity. 

      (2) Optic cup is evaginated from the lateral wall of neuroepithelium of the diencephalon. In zebrafish, cell movement occurs from the pigment epithelium to the neural retina during eye morphogenesis in an FGF-dependent manner. How the medaka optic cup morphogenesis is coordinated? I also wonder if the authors conduct the tracking of cell migration during optic cup morphogenesis to reveal how cell migration and cell division are regulated in lens of the Medaka retinal organoids. It is also interesting to examine how retinal cell movement is coordinated during Medaka retinal organoids.

      Looking into the detail of how optic cup-looking tissue arrangement of ocular organoids is achieved on cellular level is of course interesting. Our previous study showed that optic vesicles of medaka retinal organoids do not form optic cups (for details please see Zilova et al., 2021, eLIFE). We assume that the formation of cup-looking structure of the ocular organoids is mediated by the following processes: establishment of retina and lens domains at the specific region of the organoid – retina on the surface and lens in the center (see Figure S2 d and Figure 3e, and Figure 4). Further dislocation of the centrally formed lens towards the organoid periphery through the retina layer, places the lens to the periphery while retinal cells stay static. We assume that the “cup-like” shape is acquired by extrusion of the lens from the center of the organoid. To clarify this process with respect to tissue rearrangements and cell movements, we will include additional experiments (see Revision Plan, experiment 2) and follow lens- and retina-fated cells (by employing lens-specific Foxe3::GFP and retina-specific Rx2::H2B-RFP reporter lines) through the process of lens extrusion to dissect individual contribution of retinal/lens cells to this process (cross-reference with Reviewer #2).

      (3) The authors showed that blockade of FGF signaling affects lens fiber differentiation in day 1-2, whereas lens formation seems to be intact in the presence of FGF receptor inhibitor in day 0-1. I suggest the authors to examine which tissue is a target of FGF signaling in retinal organoids, using markers such as pea3, which is a downstream target of ERK branch of FGF signaling. Since FGF signaling promotes cell proliferation, is the lens core size normal in SU5402-treated organoids from day 0 to day 1?

      Assessing the activity of FGF signaling (cross-reference to Reviewer #3) in the organoids is indeed an important point. To address which tissue is the target of FGF signaling we will include additional experiments and assess the phosphorylation status of ERK (pERK) and expression of the ERK downstream target pea3, as suggested by the reviewer (see Revision Plan, experiment 3). That will allow to identify the tissue within the organoid responding to the Fgf signaling.

      Lens core size of organoids treated with SU5402 from day 0 to day 1 is fully comparable to the control (please see Figure 6b).

      (4) Fig. 3f and 3g indicate that there is some cell population located between foxe3:GFP+ cells and rx2:H2B-RFP+ cells. What kind of cell-type is occupied in the interface area between foxe3:GFP+ cells and rx2:H2B-RFP+ cells?

      That is for sure an interesting question. We are aware of this population of cells. We currently do not have data that would with certainty clarify the fate of those cells. We are currently following up on that question with the use of scRNA sequencing, however we will not be able to address this question in the current manuscript.

      (5) Fig. 5e indicates the depth of Rx3 expression at day 1. Is the depth the thickness of Rx3 expressing cell sheet, which covers the central lens core in the organoids? If so, I wonder if total cell number of Rx3 expressing cell sheet may be different in each seeded-cell number, because thickness is the same across each seeded-cell number, but the surface area size may be different depending on underneath the lens core size. Please clarify this point.

      Yes. Figure 5e indicates the thickness of the cell sheet expressing Rx3 that lies on the surface of the organoid. Indeed, the number of Rx3-expressing cells (and lens cells) scales with the size of the organoid as stated in the submitted manuscript.

      (6) Noggin application inhibits lens formation at day 0-1. BMP signaling regulates formation of lens placode and olfactory placode at the early stage of development. It is interesting to examine whether Noggin-treated organoid expands olfactory placode area. Please check forebrain territory markers.

      What tissue differentiates at the expense of the lens in BMP inhibitor-treated organoids is of course an intriguing question. To address the identity of cells differentiated under this condition we will include an additional experiment (see Revision Plan, experiment 4 as suggested by the reviewer). We will check for the expression of Lhx2, Otx2 and Huc/D to address this point.

      I have no minor comments

      Referees cross-commenting

      I agree that all reviewers have similar suggestions, which are reasonable and provided the same estimated time for revision.

      Reviewer #1 (Significance):

      Strength:

      This study is unique. The authors examined eye cup morphogenesis using fish retinal organoids. Eye cup normally consists of the lens, the neural retina, pigment epithelium and optic stalk. However, retinal organoids seem to be simple and consists of two cell types, lens and retina. Interestingly, a similar optic cup-like structure is achieved in both cases; however, underlying mechanism is different. It is interesting to investigate how eye morphogenesis is regulated in retinal organoids,under the unconstrained embryo-free environment.

      Limitation:

      Description is OK, but analysis is not much profound. It is necessary to apply a bit more molecular and cellular level analysis, such as tracking of cell movement and visualization of FGF signnaling in organoid tissues.

      Advancement:

      The current study is descriptive. Need some conceptual advance, which impact cell biology field or medical science.

      Audience:

      The target audience of current study are still within ophthalmology and neuroscience community people, maybe translational/clinical rather than basic biology. To beyond specific fields, need to formulate a general principle for cell and developmental biology.

      Reviewer #2 (Evidence, reproducibility and clarity):

      In this study from Stahl et al., the authors demonstrate that medaka pluripotent embryonic cells can self-organise into eye organoids containing both retina and lens tissues. While these organoids can self-organize into an eye structure that resembles the vertebrate eye, they are built from a fundamentally different morphogenetic process – an “inside-out” mechanism where the lens forms centrally and moves outward, rather than the normal “outside-in” embryonic process. This is a very interesting discovery, both for our understanding of developmental biology and the potential for tissue engineering applications. The study would benefit from some additional experiments and a few clarifications.

      The authors suggest that the lens cells are the ones that move from the central to a more superficial position. Is this an active movement of lens cells or just the passive consequence of the retina cells acquiring a cup shape? Are the retina cells migrating behind the lens or the lens cells pushing outwards? High-resolution imaging of organoid cup formation, tracking retina cells in combination with membrane labeling of all cells would help elucidate the morphogenetic processes occurring in the organoids. Membrane labeling would also be useful as Prox1 positive lens cells appear elongated in embryos while in the organoids, cell shapes seem less organised, less compact and not elongated (for example as shown in Fig 3f,g).

      Looking into the detail of how optic cup-looking tissue arrangement of ocular organoids is achieved on cellular level is of course interesting. We assume that the formation of cup-looking structures of the ocular organoids is mediated by following processes: establishment of retina and lens domains at a specific region of the organoid – retina on the surface and lens in the center (see Figure S2 d and Figure 3e, and Figure 4). Further dislocation of centrally formed lenses towards the organoid periphery through the retina layer, place the lens to the periphery while retinal cells stay static. We assume that the “cup-like” shape is acquired by extrusion of the lens. To clarify this process with respect to tissue rearrangements and cell movements, we will include additional experiments (see Revision Plan, experiment 2). We will follow lens- and retina-fated cells (by employing lens-specific Foxe3::GFP and retina-specific Rx2::H2B-RFP reporter lines) through the process of lens extrusion to dissect the individual contribution of retinal/lens cells to this process (cross-reference with Reviewer #1).

      The organoids could be a useful tool to address how cell fate is linked to cell shape acquisition. In the forming organoids, retinal tissue initially forms on the outside, while non-retinal tissue is located in the centre; this central tissue later expresses lens markers. Do the authors have any insights into why fate acquisition occurs in this pattern? Is there a difference in proliferation rates between the centrally located cells and the external ones? Could it be that highly proliferative cells give rise to neural retina (NR), while lower proliferating cells become lens?

      The question how is the retinal and lens domain established in this specific manner is indeed intriguing and very interesting. We dedicated a part of the discussion to this topic. We discuss the role of the diffusion limit and the potential contribution of BMB and FGF signaling to this arrangement. Additional experiments (see Revision Plan, experiment 3) addressing the source and target tissues of FGF and BMP signaling in the organoid will ultimately bring more clarity to our understanding of the tissue arrangements in the organoid. 

      Although analysis of the proliferation rate of the cells at the surface and in the central region of the organoid might possibly show some differences in the proliferation rates between lens and retinal cells, we do not have any indications, that the proliferation rate itself would be instructive or superior to the cell fate decisions.

      What happens in organoids that do not form lenses? Do these organoids still generate foxe3 positive cells that fail to develop into a proper lens structure? And in the absence of lens formation, does the retina still acquire a cup shape?

      Lens formation is primarily dependent on acquisition/specification of Foxe3-expressing lens placode progenitors. If those are not present, a lens does not develop. Once Foxe3-expressing progenitors are established, a lens is formed in unperturbed conditions (measured by the presence of expression of crystallin proteins). In such conditions, organoids that do not have a lens, do not carry Foxe3-expressing cells.

      In the absence of the lens, the organoid is composed of retinal neuroepithelium, that does not form an optic cup (for details of such phenotypes please see Zilova et al., 2021, eLIFE).

      The author suggest that lens formation occurs even in the absence of Matrigel. Is the process slower in these conditions? Are the resulting organoids smaller? While there are indeed some LFC expressing cells by day2, these cells are not very well organised and the pattern of expression seems dotty. Moreover, LFC staining seems to localise posterior to the LFC negative, lens-like structure (e.g. Fig.S1 3o’clock).

      How do these organoids develop beyond day 4? Do they maintain their structural integrity at later stages?

      The role of HEPES in promoting organoid formation is intriguing. Do the authors have any insights into why it is important in this context? Have the authors tried other culture conditions and does culture condition influence the morphogenetic pathways occurring within the organoids?

      We thank the reviewer for pointing this out. We were not clear in the wording and describing of our observation. Indeed, Matrigel is not required for acquisition of lens fate, which can be demonstrated with the expression of lens-specific markers. However, the presence of Matrigel has a profound impact on the structural aspects of organoid formation. Matrigel is essential for organization of retinal-committed cells into the retinal epithelium (Zilova et al., 2021, eLIFE). The absence of the structure of the retinal epithelium can indeed negatively impact on the cellular organization and the overall lens structure. To clarify the contribution of the Matrigel to the speed of organoid lens development and to the overall structure of the organoid lens we will perform additional experiments (see Revision Plan, experiment 5). With the use of Foxe3::GFP reporter line we will measure the onset of the lens-specific gene expression. In addition, we will use the immunohistochemistry to assess the gross morphology and size of the organoids grown without the Matrigel (cross-reference with Reviewer #3).

      The role of the HEPES in lens formation is indeed very intriguing and currently under investigation. As HEPES is mainly used to regulate pH of the culture media and pH might have an impact on multiple cellular processes, it will require significant time investment to dissect molecular mechanism underlying the effect of HEPES on the process of lens formation (cross reference with Reviewer #3) and therefore cannot be addressed in the current manuscript.

      Referees cross-commenting

      Pleased to see that all the other reviewers are positive about the study and raise similar concerns and comments

      Reviewer #2 (Significance):

      This is a very interesting paper, and it will be important to determine whether this alternative morphogenetic process is specific to medaka or if similar developmental routes can be recapitulated in organoid cultures from other vertebrate species.

      Reviewer #3 (Evidence, reproducibility and clarity):

      Summary:

      The manuscript by Stahl and colleagues reports an approach to generate ocular organoids composed of retinal and lens structures, derived from Medaka blastula cells. The authors present a comprehensive characterisation of the timeline followed by lens and retinal progenitors, showing these have distinct origins, and that they recapitulate the expression of differentiation markers found in vivo. Despite this molecular recapitulation, morphogenesis is strikingly different, with lens progenitors arising at the centre of the organoid, and subsequently translocating to the outside.

      Comments:

      - The manuscript presents a beautiful set of high quality images showing expression of lens differentiation markers over time in the organoids. The set of experiments is very robust, with high numbers of organoids analysed and reproducible data. The mechanism by which lens specification is promoted in these organoids is, however, poorly analysed, and the reader does not get a clear understanding of what is different in these experiments, as compared to previous attempts, to support lens differentiation. There is a mention to HEPES supplementation, but no further analysis is provided, and the fact that the process is independent of ECM contradicts, as the authors point out, previous reports. The manuscript would benefit from a more detailed analysis of the mechanisms that lead to lens differentiation in this setting.

      The role of the HEPES in lens formation is indeed very intriguing and under current investigation. As HEPES is mainly used to regulate pH of the culture media and pH might have an impact on multiple cellular processes it will require a significant time investment to dissect molecular mechanism underlying the effect of HEPES on the process of lens formation (cross reference with Reviewer #2) and therefore unfortunately cannot be addressed in the current manuscript.

      To clarify the contribution of the Matrigel to the organoid lens development we will perform additional experiments (see Revision Plan, experiment 5). With the use of Foxe3::GFP reporter line we will measure the onset of the lens-specific gene expression. In addition, we will use the immunohistochemistry to assess the gross morphology and size of the organoids grown without the Matrigel (cross-reference with Reviewer #2).

      - The markers analysed to show onset of lens differentiation in the organoids seem to start being expressed, in vivo, when the lens placode starts invaginating. An analysis of earlier stages is not presented. This would be very informative, allowing to determine whether progenitors differentiate as placode and neuroepithelium first, to subsequently continue differentiating into lens and retina, respectively. Could early placodal and anterior neural plate markers be analysed in the organoids? This would provide a more complete sequence of lens vs retina differentiation in this model.

      Yes. The figures show the expression of lens and retinal markers in the embryo in later developmental stages and the timing of their expression can be documented with higher temporal resolution. In the revised version of the manuscript, we will provide the information about the onset of expression of Rx3::H2B-GFP (retina) and Foxe3::GFP (lens) (see Author response image 1). Rx3 represents one of the earlies markers labeling the presumptive eye field within the region of the anterior neural plate (S16, late gastrula). FoxE3::GFP expression can be detected within the head surface ectoderm before the lens placode is formed showing that Foxe3 is a suitable marker of placodal progenitors in medaka.

      We are convinced that the onset of Rx3 and Foxe3-driven reporters is early enough to make the claim about the separate origin of the lens (placodal) and retinal (anterior neuroectoderm) tissues within the ocular organoids.

      Author response image 1.

      - The analysis of BMP and Fgf requirement for lens formation and differentiation is suggestive, but the source of these signals is not resolved or mentioned in the manuscript. Are BMP4 and Fgf8 expressed by the organoids? Where are they coming from?

      Indeed, addressing the source of BMP and FGF activation would bring more clarity in understanding the mechanism of retina/lens specification within the ocular organoids (cross reference with Reviewer #1). To address this point, we will include additional experiments (see Revision Plan, experiment 3). We will analyze the expression of respective ligands (Bmp4 and Fgf8) and activation of downstream effectors of BMP and FGF signaling pathways within the ocular organoids as suggested by Reviewer #1 and Reviewer #3.

      - The fact that the lens becomes specified in the centre of the organoid is striking, but it is for me difficult to visualise how it ends up being extruded from the organoid. Did the authors try to follow this process in movies? I understand that this may be technically challenging, but it would certainly help to understand the process that leads to the final organisation of retinal and lens tissues in the organoid. There is no discussion of why the morphogenetic mechanism is so different from the in vivo situation. The manuscript would benefit from explicitly discussing this.

      Following the extruding lens in vivo is indeed very relevant suggestion. To clarify the process of ocular organoid formation in the respect of tissue rearrangements and cell movements, we will include additional experiment (see Revision Plan, experiment 2). We will follow lens- and retina-fated cells (by employing lens-specific Foxe3::GFP and retina-specific Rx2::H2B-RFP reporter lines) through the process of lens extrusion (cross-reference with Reviewer #1 and Reviewer #2).

      Referees cross-commenting

      We all seem to have similar comments and concerns. I think overall the suggestions are feasible and realistic for the timeframe provided.

      Reviewer #3 (Significance):

      This study describes a reproducible approach to differentiate ocular organoids composed of lens and retinal tissues. The characterisation of lens differentiation in this model is very detailed, and despite the morphogenetic differences, the molecular mechanisms show many similarities to the in vivo situation. The manuscript however does not highlight, in my opinion, why this model may be relevant. Clearly articulating this relevance, particularly in the discussion, will enhance the study and provide more clarity to the readers regarding the significance of the study for the field of organoid research, ocular research and regenerative studies.

      Revision Plan:

      (1) To address whether differential adhesion properties of retinal and lens progenitors mediate cell sorting to establish retina and lens domains in the organoids (Reviewer #1, comment 1), we will perform dissociation of the organoids on day 1 and subsequential re-aggregation. This experiment will allow to follow cell type specific adhesion properties of lens and retinal progenitor cells. We will employ lens progenitor reporter line Foxe3::GFP and retina-specific Rx2::H2B-RFP to monitor cell type specific sorting with fluorescent microscopy.

      (2)   Multiple reviewers (Reviewer #1, Reviewer #2, Reviewer #3) asked for the presentation of detailed in vivo imaging experiment showing individual contributions of retina- and lens- fated cells to the resulting tissue organization withing the ocular organoid. We will perform in vivo live imaging experiment to follow the movements of individual lens (Foxe3::GFP) and retinal (Rx2::H2B-GFP) cells from day 1 to day 2 of organoid development to address this point.

      (3) Reviewer #1 and Reviewer #3 raised questions concerning the role of FGF and BMP signaling and sources of these signaling pathway activities in ocular organoid tissue arrangement. To address this point and bring more light into the molecular mechanisms regulating lens and retina tissue arrangement in the organoid, we will perform additional experiment. We will assess the expression of candidate FGF and BMP ligands (Fgf8, Bmp7 and Bmp4) and activation of downstream effectors (p-ERK, p-SMAD) and the direct transcriptional target of Fgf signaling (Pea3) in the developing organoids. This will allow the identification of the tissue producing the ligand on one site and tissue responding to the signaling on the other site and help out to narrow down the molecular mechanism controlling tissue arrangements in the organoid.

      (4) We will analyze the expression of forebrain territory markers in organoids treated with the BMP inhibitor to identify the identity of the tissue differentiating at the expense of lens under the BMP inhibition (suggested by Reviewer #1). We will label Noggin-treated organoids with the antibodies against Lhx2, Otx2 and HuC/D to address this point.

      (5) We will provide more comprehensive analysis of the organoids grown without the Matrigel and compare them to the organoids grown in the presence of the Matrigel (mentioned by Reviewer #2 and Reviewer #3). With the use of lens progenitor-specific Foxe3::GFP reporter line, we will measure the onset of the lens-specific gene expression. In addition, we will use the immunohistochemistry to assess the gross morphology and size of the organoids grown without the Matrigel.

      Description of analyses that authors prefer not to carry out

      Reviewer #1:

      (4) Fig. 3f and 3g indicate that there is some cell population located between foxe3:GFP+ cells and rx2:H2B-RFP+ cells. What kind of cell-type is occupied in the interface area between foxe3:GFP+ cells and rx2:H2B-RFP+ cells?

      That is for sure interesting question. We are aware of this population of cells. We currently do not have a data that would with certainty clarify the fate of those cells. We are currently following up on that question with the use of scRNA sequencing, however we will not be able to address this question in the current manuscript.

      Reviewer #2:

      The role of HEPES in promoting organoid formation is intriguing. Do the authors have any insights into why it is important in this context? Have the authors tried other culture conditions and does culture condition influence the morphogenetic pathways occurring within the organoids?

      The role of the HEPES in lens formation is indeed very intriguing and under current investigation. As HEPES is mainly used to regulate pH of the culture media and pH might have impact on multiple cellular processes it will require significant time investment to dissect molecular mechanism underlying the effect of the HEPES on the process of lens formation (cross reference with Reviewer #3) and cannot be addressed in the current manuscript.

      Is there a difference in proliferation rates between the centrally located cells and the external ones? Could it be that highly proliferative cells give rise to neural retina (NR), while lower proliferating cells become lens?

      Although analysis of the proliferation rate of the cells at the surface and in the central region of the organoid might possibly show some differences in the proliferation rates between lens and retinal cells, we do not have any indications, that the proliferation rate itself would be instructive or superior to the cell fate decisions.

    1. Author response:

      Public Reviews:

      Reviewer #1 (Public review):

      Summary:

      This paper is an elegant, mostly observational work, detailing observations that polysome accumulation appears to drive nucleoid splitting and segregation. Overall I think this is an insightful work with solid observations.

      Thank you for your appreciation and positive comments. In our view, an appealing aspect of this proposed biophysical mechanism for nucleoid segregation is its self-organizing nature and its ability to intrinsically couple nucleoid segregation to biomass growth, regardless of nutrient conditions.

      Strengths:

      The strengths of this paper are the careful and rigorous observational work that leads to their hypothesis. They find the accumulation of polysomes correlates with nucleoid splitting, and that the nucleoid segregation occurring right after splitting correlates with polysome segregation. These correlations are also backed up by other observations:

      (1) Faster polysome accumulation and DNA segregation at faster growth rates.

      (2) Polysome distribution negatively correlating with DNA positioning near asymmetric nucleoids.

      (3) Polysomes form in regions inaccessible to similarly sized particles.

      These above points are observational, I have no comments on these observations leading to their hypothesis.

      Thank you!

      Weaknesses:

      It is hard to state weaknesses in any of the observational findings, and furthermore, their two tests of causality, while not being completely definitive, are likely the best one could do to examine this interesting phenomenon.

      It is indeed difficult to prove causality in a definitive manner when the proposed coupling mechanism between nucleoid segregation and gene expression is self-organizing, i.e., does not involve a dedicated regulatory molecule (e.g., a protein, RNA, metabolite) that we could have depleted through genetic engineering to establish causality. We are grateful to the reviewer for recognizing that our two causality tests are the best that can be done in this context.

      Points to consider / address:

      Notably, demonstrating causality here is very difficult (given the coupling between transcription, growth, and many other processes) but an important part of the paper. They do two experiments toward demonstrating causality that help bolster - but not prove - their hypothesis. These experiments have minor caveats, my first two points.

      (1) First, "Blocking transcription (with rifampicin) should instantly reduce the rate of polysome production to zero, causing an immediate arrest of nucleoid segregation". Here they show that adding rifampicin does indeed lead to polysome loss and an immediate halting of segregation - data that does fit their model. This is not definitive proof of causation, as rifampicin also (a) stops cell growth, and (b) stops the translation of secreted proteins. Neither of these two possibilities is ruled out fully.

      That’s correct; cell growth also stops when gene expression is inhibited, which is consistent with our model in which gene expression within the nucleoid promotes nucleoid segregation and biomass growth (i.e., cell growth), inherently coupling these two processes. This said, we understand the reviewer’s point: the rifampicin experiment doesn’t exclude the possibility that protein secretion and cell growth drive nucleoid segregation. We are assuming that the reviewer is envisioning an alternative model in which sister nucleoids would move apart because they would be attached to the membrane through coupled transcription-translation-protein secretion (transertion) and the membrane would expand between the separating nucleoids, similar to the model proposed by Jacob et al in 1963 (doi:10.1101/SQB.1963.028.01.048). There are several observations arguing against this cell elongation/transertion model.

      (1) For this alternative mechanism to work, membrane growth must be localized at the middle of the splitting nucleoids (i.e., midcell position for slow growth and ¼ and ¾ cell positions for fast growth) to create a directional motion. To our knowledge, there is no evidence of such localized membrane incorporation. Furthermore, even if membrane growth was localized at the right places, the fluidity of the cytoplasmic membrane (PMID: 6996724, 20159151, 24735432, 27705775) would be problematic. To circumvent the membrane fluidity issue, one could potentially evoke an additional connection to the rigid peptidoglycan, but then again, peptidoglycan growth would have to be localized at the middle of the splitting nucleoid. However, peptidoglycan growth is dispersed early in the cell division cycle when the nucleoid splitting happens in fast growing cells and only appears to be zonal after the onset of cell constriction (PMID: 35705811, 36097171, 2656655).

      (2) Even if we ignore the aforementioned caveats, Paul Wiggins’s group ruled out the cell elongation/transertion model by showing that the rate of cell elongation is slower than the rate of chromosome segregation (PMID: 23775792). In the revised manuscript, we wil clarify this point and provide confirmatory data showing that the cell elongation rate is indeed slower than the nucleoid segregation rate, indicating that it cannot be the main driver.

      (3) Furthermore, our correlation analysis comparing the rate of nucleoid segregation to the rate of either cell elongation or polysome accumulation argues that polysome accumulation plays a larger role than cell elongation in nucleoid segregation. These data were already shown in Figure 1H and Figure 1 – figure supplement 3 of the original manuscript but were not highlighted in this context. We will revise the text to clarify this point.

      (4) The asymmetries in nucleoid compaction that we described in our paper are predicted by our model. We do not see how they could be explained by cell growth or protein secretion.

      (5) We also show that polysome accumulation at ectopic sites (outside the nucleoid) results in correlated nucleoid dynamics, consistent with our proposed mechanism. These nucleoid dynamics cannot be explained by cell growth or protein secretion (transertion).

      (1a) As rifampicin also stops all translation, it also stops translational insertion of membrane proteins, which in many old models has been put forward as a possible driver of nucleoid segregation, and perhaps independent of growth. This should at last be mentioned in the discussion, or if there are past experiments that rule this out it would be great to note them.

      It is not clear to us how the attachment of the DNA to the cytoplasmic membrane could alone create a directional force to move the sister nucleoids. We agree that old models have proposed a role for cell elongation (providing the force) and transertion (providing the membrane tether).  Please see our response above for the evidence (from the literature and our work) against it. This was mentioned in the introduction and Results section, but we agree that this was not well explained. We will add experimental data and revise the text to clarify these points.

      (1b) They address at great length in the discussion the possibility that growth may play a role in nucleoid segregation. However, this is testable - by stopping surface growth with antibiotics. Cells should still accumulate polysomes for some time, it would be easy to see if nucleoids are still segregated, and to what extent, thereby possibly decoupling growth and polysome production. If successful, this or similar experiments would further validate their model.

      We reviewed the literature and could not find a drug that stops cell growth without stopping gene expression. Any drug that affects the membrane integrity or potential stops gene expression, which requires ATP.  However, our experiment in which we drive polysome accumulation at ectopic sites decouples polysome accumulation from cell growth. In this experiment, by redirecting most of chromosome gene expression to a single plasmid-encoded gene, we reduce the rate of cell growth but still create a large accumulation of polysomes at an ectopic location. This ectopic polysome accumulation is sufficient to affect nucleoid dynamics in a correlated fashion. In the revised manuscript, we will clarify this point and add model simulations to show that our experimental observations are predicted by our model.

      (2) In the second experiment, they express excess TagBFP2 to delocalize polysomes from midcell. Here they again see the anticorrelation of the nucleoid and the polysomes, and in some cells, it appears similar to normal (polysomes separating the nucleoid) whereas in others the nucleoid has not separated. The one concern about this data - and the differences between the "separated" and "non-separated" nuclei - is that the over-expression of TagBFP2 has a huge impact on growth, which may also have an indirect effect on DNA replication and termination in some of these cells. Could the authors demonstrate these cells contain 2 fully replicated DNA molecules that are able to segregate?

      We will perform the requested experiment.

      (3) What is not clearly stated and is needed in this paper is to explain how polysomes do (or could) "exert force" in this system to segregate the nucleoid: what a "compaction force" is by definition, and what mechanisms causes this to arise (what causes the "force") as the "compaction force" arises from new polysomes being added into the gaps between them caused by thermal motions.

      They state, "polysomes exert an effective force", and they note their model requires "steric effects (repulsion) between DNA and polysomes" for the polysomes to segregate, which makes sense. But this makes it unclear to the reader what is giving the force. As written, it is unclear if (a) these repulsions alone are making the force, or (b) is it the accumulation of new polysomes in the center by adding more "repulsive" material, the force causes the nucleoids to move. If polysomes are concentrated more between nucleoids, and the polysome concentration does not increase, the DNA will not be driven apart (as in the first case) However, in the second case (which seems to be their model), the addition of new material (new polysomes) into a sterically crowded space is not exerting force - it is filling in the gaps between the molecules in that region, space that needs to arise somehow (like via Brownian motion). In other words, if the polysome region is crowded with polysomes, space must be made between these polysomes for new polysomes to be inserted, and this space must be made by thermal (or ATP-driven) fluctuations of the molecules. Thus, if polysome accumulation drives the DNA segregation, it is not "exerting force", but rather the addition of new polysomes is iteratively rectifying gaps being made by Brownian motion.

      We apologize for the understandable confusion. In our picture, the polysomes and DNA (conceptually considered as small plectonemic segments) basically behave as dissolved particles. If these particles were noninteracting, they would simply mix. However, both polysomes and DNA segments are large enough to interact sterically. So as density increases, steric avoidance implies a reduced conformational entropy and thus a higher free energy per particle. We argue (based on Miangolarra et al. PNAS 2021 PMID: 34675077 and Xiang et al. Cell 2021 PMID: 34186018) that the demixing of polysomes and DNA segments occurs because DNA segments pack better with each other than they do with polysomes. This raises the free energy cost associated with DNA-polysome interactions compared to DNA-DNA interactions.  We model this effect by introducing a term in the free energy χ_np, which refer to as a repulsion between DNA and polysomes, though as explained above it arises from entropic effects. At realistic cellular densities of DNA and polysomes this repulsive interaction is strong enough to cause the DNA and polysomes to phase separate.

      This same density-dependent free energy that causes phase separation can also give rise to forces, just in the way that a higher pressure on one side of a wall can give rise to a net force on the wall. Indeed, the “compaction force” we refer to is fundamentally an osmotic pressure difference. At some stages during nucleoid segregation, the region of the cell between nucleoids has a higher polysome concentration, and therefore a higher osmotic pressure, than the regions near the poles. This results in a net poleward force on the sister nucleoids that drives their migration toward the poles. This migration continues until the osmotic pressure equilibrates. Therefore, both phase separation (due to the steric repulsion described above) and nonequilibrium polysome production and degradation (which creates the initial accumulation of polysomes around midcell) are essential ingredients for nucleoid segregation.

      This will be clarified in the revised text, with the support of additional simulation results.

      The authors use polysome accumulation and phase separation to describe what is driving nucleoid segregation. Both terms are accurate, but it might help the less physically inclined reader to have one term, or have what each of these means explicitly defined at the start. I say this most especially in terms of "phase separation", as the currently huge momentum toward liquid-liquid interactions in biology causes the phrase "phase separation" to often evoke a number of wider (and less defined) phenomena and ideas that may not apply here. Thus, a simple clear definition at the start might help some readers.

      Phase separation means that the DNA-polysome steric repulsion is strong enough to drive their demixing, which creates a compact nucleoid. As mentioned in a previous point, this effect is captured in the free energy by the χ_np term, which is an effective repulsion between DNA and polysomes, though as explained above it arises from entropic effects.

      In the revised manuscript, we will illustrate this with our theoretical model by initializing a cell with a diffuse nucleoid and low polysome concentration. For the sake of simplicity, we assume that the cell does not elongate. We observe that the DNA-polysome steric repulsion is sufficient to compact the nucleoid and place it at mid-cell.

      (4) Line 478. "Altogether, these results support the notion that ectopic polysome accumulation drives nucleoid dynamics". Is this right? Should it not read "results support the notion that ectopic polysome accumulation inhibits/redirects nucleoid dynamics"?

      We think that this is correct; the ectopic polysome accumulation drives nucleoid dynamics. In our theoretical model, we can introduce polysome production at fixed sources to mimic the experiments where ectopic polysome production is achieved by high plasmid expression (Fig. 6). The model is able to recapitulate the two main phenotypes observed in experiments. These new simulation results will be added to the revised manuscript.

      (5) It would be helpful to clarify what happens as the RplA-GFP signal decreases at midcell in Figure 1- is the signal then increasing in the less "dense" parts of the cell? That is, (a) are the polysomes at midcell redistributing throughout the cell? (b) is the total concentration of polysomes in the entire cell increasing over time?

      It is a redistribution—the RplA-GFP signal remains constant in concentration from cell birth to division (Figure 1 – Figure Supplement 1E). This will be clarified in the revised text.

      (6) Line 154. "Cell constriction contributed to the apparent depletion of ribosomal signal from the mid-cell region at the end of the cell division cycle (Figure 1B-C and Movie S1)" - It would be helpful if when cell constriction began and ended was indicated in Figures 1B and C.

      Good idea. We will add markers to indicate the start of cell constriction. We will also indicate that cell birth and division correspond to the first and last images/timepoint in Fig. 1B and C, respectively.

      (7) In Figure 7 they demonstrate that radial confinement is needed for longitudinal nucleoid segregation. It should be noted (and cited) that past experiments of Bacillus l-forms in microfluidic channels showed a clear requirement role for rod shape (and a given width) in the positing and the spacing of the nucleoids.

      Wu et al, Nature Communications, 2020 . "Geometric principles underlying the proliferation of a model cell system" https://dx.doi.org/10.1038/s41467-020-17988-7

      Good point. We will add this reference. Thank you.

      (8) "The correlated variability in polysome and nucleoid patterning across cells suggests that the size of the polysome-depleted spaces helps determine where the chromosomal DNA is most concentrated along the cell length. This patterning is likely reinforced through the displacement of the polysomes away from the DNA dense region"

      It should be noted this likely functions not just in one direction (polysomes dictating DNA location), but also in the reverse - as the footprint of compacted DNA should also exclude (and thus affect) the location of polysomes

      We agree that the effects could go both ways at this early stage of the story. We will revise the text accordingly.  

      (9) Line 159. Rifampicin is a transcription inhibitor that causes polysome depletion over time. This indicates that all ribosomal enrichments consist of polysomes and therefore will be referred to as polysome accumulations hereafter". Here and throughout this paper they use the term polysome, but cells also have monosomes (and 2 somes, etc). Rifampicin stops the assembly of all of these, and thus the loss of localization could occur from both. Thus, is it accurate to state that all transcription events occur in polysomes? Or are they grouping all of the n-somes into one group?

      In the discussion, we noted that our term “polysomes” also includes monosomes for simplicity, but we agree that the term should have been defined much earlier. This will be done in the revised manuscript.

      Thank you for the valuable comments and suggestions!

      Reviewer #2 (Public review):

      Summary:

      The authors perform a remarkably comprehensive, rigorous, and extensive investigation into the spatiotemporal dynamics between ribosomal accumulation, nucleoid segregation, and cell division. Using detailed experimental characterization and rigorous physical models, they offer a compelling argument that nucleoid segregation rates are determined at least in part by the accumulation of ribosomes in the center of the cell, exerting a steric force to drive nucleoid segregation prior to cell division. This evolutionarily ingenious mechanism means cells can rely on ribosomal biogenesis as the sole determinant for the growth rate and cell division rate, avoiding the need for two separate 'sensors,' which would require careful coupling.

      Terrific summary! Thank you for your positive assessment.

      Strengths:

      In terms of strengths; the paper is very well written, the data are of extremely high quality, and the work is of fundamental importance to the field of cell growth and division. This is an important and innovative discovery enabled through a combination of rigorous experimental work and innovative conceptual, statistical, and physical modeling.

      Thank you!

      Weaknesses:

      In terms of weaknesses, I have three specific thoughts.

      Firstly, my biggest question (and this may or may not be a bona fide weakness) is how unambiguously the authors can be sure their ribosomal labeling is reporting on polysomes, specifically. My reading of the work is that the loss of spatial density upon rifampicin treatment is used to infer that spatial density corresponds to polysomes, yet this feels like a relatively indirect way to get at this question, given rifampicin targets RNA polymerase and not translation. It would be good if a more direct way to confirm polysome dependence were possible.

      The heterogeneity of ribosome distribution inside E. coli cells has been attributed to polysomes by many labs (PMID: 25056965, 38678067, 22624875, 31150626, 34186018, 10675340).  The attribution is also consistent with single-molecule tracking experiments showing that slow-moving ribosomes (polysomes) are excluded by the nucleoid whereas fast-diffusing ribosomes (free ribosomal subunits) are distributed throughout the cytoplasm (PMID: 25056965, 22624875).

      Furthermore, inhibition of translation initiation with kasugamycin treatment, which decreases the pool of polysomes, results in a homogenization of ribosomes and expansion of the nucleoid (see Author response image 1). This further supports the rifampicin experiments. Given that the attribution of ribosome heterogeneity to polysomes is well accepted in the field, we would prefer to not include these kasugamycin data in the revised manuscript because long-term exposure to this drug leads to nucleoid re-compaction (PMID: 25250841 and PMID: 34186018). This secondary effect may possibly be due to a dysregulated increase in synthesis of naked rRNAs (PMID: 14460744, PMID: 2114400, and PMID: 2448483) or ribosome aggregation, which we are currently investigating.

      Author response image 1.

      Effects of kasugamycin treatment on the intracellular distribution of ribosomes and nucleoids. Representative single cell (CJW7323) growing in M9gluCAAT.  Kasugamycin (3 mg/mL) was added at time = 0 min. Show is the early response (0-30 min) to the drug characterized by the homogenization of the ribosomal RplA-GFP fluorescence and the expansion of the HupA-mCherry-labeled nucleoids. For each segmented cell, the RplA-GFP and HupA-mCherry signals were normalized by the average fluorescence.

      Second, the authors invoke a phase separation model to explain the data, yet it is unclear whether there is any particular evidence supporting such a model, whether they can exclude simpler models of entanglement/local diffusion (and/or perhaps this is what is meant by phase separation?) and it's not clear if claiming phase separation offers any additional insight/predictive power/utility. I am OK with this being proposed as a hypothesis/idea/working model, and I agree the model is consistent with the data, BUT I also feel other models are consistent with the data. I also very much do not think that this specific aspect of the paper has any bearing on the paper's impact and importance.

      We appreciate the reviewer’s comment, but the output of our reaction-diffusion model is a bona fide phase separation (spinodal decomposition). So, we feel that we need to use the term when reporting the modeling results. Inside the cell, the situation is more complicated. As the reviewer points out, there likely are entanglements (not considered in our model) and other important factors (please see our discussion on the model limitations). This said, we will revise our text to clarify our terms and proposed mechanism.

      Finally, the writing and the figures are of extremely high quality, but the sheer volume of data here is potentially overwhelming. I wonder if there is any way for the authors to consider stripping down the text/figures to streamline things a bit? I also think it would be useful to include visually consistent schematics of the question/hypothesis/idea each of the figures is addressing to help keep readers on the same page as to what is going on in each figure. Again, there was no figure or section I felt was particularly unclear, but the sheer volume of text/data made reading this quite the mental endurance sport! I am completely guilty of this myself, so I don't think I have any super strong suggestions for how to fix this, but just something to consider.

      We agree that there is a lot to digest. We will add schematics and a didactic simulation. We will also try to streamline the text.

      Reviewer #3 (Public review):

      Summary:

      Papagiannakis et al. present a detailed study exploring the relationship between DNA/polysome phase separation and nucleoid segregation in Escherichia coli. Using a combination of experiments and modelling, the authors aim to link physical principles with biological processes to better understand nucleoid organisation and segregation during cell growth.

      Strengths:

      The authors have conducted a large number of experiments under different growth conditions and physiological perturbations (using antibiotics) to analyse the biophysical factors underlying the spatial organisation of nucleoids within growing E. coli cells. A simple model of ribosome-nucleoid segregation has been developed to explain the observations.

      Weaknesses:

      While the study addresses an important topic, several aspects of the modelling, assumptions, and claims warrant further consideration.

      Thank you for your feedback. Please see below for a response to each concern. 

      Major Concerns:

      Oversimplification of Modelling Assumptions:

      The model simplifies nucleoid organisation by focusing on the axial (long-axis) dimension of the cell while neglecting the radial dimension (cell width). While this approach simplifies the model, it fails to explain key experimental observations, such as:

      (1) Inconsistencies with Experimental Evidence:

      The simplified model presented in this study predicts that translation-inhibiting drugs like chloramphenicol would maintain separated nucleoids due to increased polysome fractions. However, experimental evidence shows the opposite-separated nucleoids condense into a single lobe post-treatment (Bakshi et al 2014), indicating limitations in the model's assumptions/predictions. For the nucleoids to coalesce into a single lobe, polysomes must cross the nucleoid zones via the radial shells around the nucleoid lobes.

      We do not think that the results from chloramphenicol-treated cells are inconsistent with our model. Our proposed mechanism predicts that nucleoids will condense in the presence of chloramphenicol, consistent with experiments. It also predicts that nucleoids that were still relatively close at the time of chloramphenicol treatment could fuse if they eventually touched through diffusion (thermal fluctuation) to reduce their interaction with the polysomes and minimize their conformational energy. Fusion is, however, not expected for well-separated nucleoids since their diffusion is slow in the crowded cytoplasm. This is consistent with our experimental observations: In the presence of a growth-inhibitory concentration of chloramphenicol (70 μg/mL), nucleoids in relatively close proximity can fuse, but well-separated nucleoids condense and do not fuse. Since the growth rate inhibition is not immediate upon chloramphenicol treatment, many cells with well-separated condensed nucleoids divide during the first hour. As a result, the non-fusion phenotype is more obvious in non-dividing cells, achieved by pre-treating cells with the cell division inhibitor cephalexin (50μg/mL). In these polyploid elongated cells, well-separated nucleoids condensed but did not fuse, not even after an hour in the presence of chloramphenicol (as illustrated in Author response image 2).

      In Bakshi et al, 2014, nucleoid fusion was shown for a single cell in which the sister nucleoids were relatively close to each other at the time of chloramphenicol treatment. Population statistics were provided for the relative length and width of the nucleoids, but not for the fusion events. So, it is unclear whether the illustrated fusion was universal or not. Also, we note that Bakshi et al (2014) used a chloramphenicol concentration of 300 μg/mL, which is 20-fold higher than the minimal inhibitory concentration for growth, compared to 70 μg/mL in our experiments.

      Author response image 2.

      Effects of chloramphenicol treatment on the intracellular distribution of ribosomes and nucleoids in non-dividing cells. Exponentially growing cells (M9glyCAAT at 30°C) were pre-treated with cephalexin for one hour before being spotted on an 1% agarose pad for time-lapse imaging. The agarose pad contained M9glyCAAT, cephalexin, and chloramphenicol.  (A) Phase contrast, RplA-GFP fluorescence and HupA-mCherry fluorescence images of a representative single cell. Three timepoints are shown, including the first image after spotting on the agarose pad (at 0 min), 30 minutes and one hour of chloramphenicol treatment. (B) One-dimensional profiles of the ribosomal (RplA-GFP) and nucleoid (HupA-mCherry) fluorescence from the cells shown in panel A. These intensity profiles correspond to the average fluorescence along the medial axis of the cell considering a 6-pixel region (0.4 μm) centered on the central line of the cell. The fluorescence intensity is plotted along the relative cell length, scaled from 0 to 100% between the two poles, illustrating the relative nucleoid length (L<sub>DNA</sub>/L<sub>cell</sub>) that was plotted by Bakshi et al in 2014 (PMID: 25250841).

      (2) The peripheral localisation of nucleoids observed after A22 treatment in this study and others (e.g., Japaridze et al., 2020; Wu et al., 2019), which conflicts with the model's assumptions and predictions. The assumption of radial confinement would predict nucleoids to fill up the volume or ribosomes to go near the cell wall, not the nucleoid, as seen in the data.

      The reviewer makes a good point that DNA attachment to the membrane through transertion likely contributes to the nucleoid being peripherally localized in A22 cells. We will revise the text to add this point. However, we do not think that this contradicts the proposed nucleoid segregation mechanism based on phase separation and out-of-equilibrium dynamics described in our model. On the contrary, by attaching the nucleoid to the cytoplasmic membrane along the cell width, transertion might help reduce the diffusion and thus exchange of polysomes across nucleoids. We will revise the text to discuss transertion over radial confinement.

      (3) The radial compaction of the nucleoid upon rifampicin or chloramphenicol treatment, as reported by Bakshi et al. (2014) and Spahn et al. (2023), also contradicts the model's predictions. This is not expected if the nucleoid is already radially confined.

      We originally evoked radial confinement to explain the observation that polysome accumulations do not equilibrate between DNA-free regions. We agree that transertion is an alternative explanation. Thank you for bringing it to our attention. However, please note that this does not contradict the model. In our view, it actually supports the 1D model by providing a reasonable explanation for the slow exchange of polysomes across DNA-free regions. The attachment of the nucleoid to the membrane along the cell width may act as diffusion barrier. We will revise the text and the title of the manuscript accordingly.

      (4) Radial Distribution of Nucleoid and Ribosomal Shell:

      The study does not account for well-documented features such as the membrane attachment of chromosomes and the ribosomal shell surrounding the nucleoid, observed in super-resolution studies (Bakshi et al., 2012; Sanamrad et al., 2014). These features are critical for understanding nucleoid dynamics, particularly under conditions of transcription-translation coupling or drug-induced detachment. Work by Yongren et al. (2014) has also shown that the radial organisation of the nucleoid is highly sensitive to growth and the multifork nature of DNA replication in bacteria.

      We will discuss the membrane attachment. Please see the previous response.

      The omission of organisation in the radial dimension and the entropic effects it entails, such as ribosome localisation near the membrane and nucleoid centralisation in expanded cells, undermines the model's explanatory power and predictive ability. Some observations have been previously explained by the membrane attachment of nucleoids (a hypothesis proposed by Rabinovitch et al., 2003, and supported by experiments from Bakshi et al., 2014, and recent super-resolution measurements by Spahn et al.).

      We agree—we will add a discussion about membrane attachment in the radial dimension. See previous responses.

      Ignoring the radial dimension and membrane attachment of nucleoid (which might coordinate cell growth with nucleoid expansion and segregation) presents a simplistic but potentially misleading picture of the underlying factors.

      As mentioned above, we will discuss membrane attachment in the revised manuscript.

      This reviewer suggests that the authors consider an alternative mechanism, supported by strong experimental evidence, as a potential explanation for the observed phenomena:

      Nucleoids may transiently attach to the cell membrane, possibly through transertion, allowing for coordinated increases in nucleoid volume and length alongside cell growth and DNA replication. Polysomes likely occupy cellular spaces devoid of the nucleoid, contributing to nucleoid compaction due to mutual exclusion effects. After the nucleoids separate following ter separation, axial expansion of the cell membrane could lead to their spatial separation.

      This “membrane attachment/cell elongation” model is reminiscent to the hypothesis proposed by Jacob et al in 1963 (doi:10.1101/SQB.1963.028.01.048). There are several lines of evidence arguing against it as the major driver of nucleoid segregation:

      (Below is a slightly modified version of our response to a comment from Reviewer 1—see page 3)

      (1) For this alternative model to work, axial membrane expansion (i.e., cell elongation) would have to be localized at the middle of the splitting nucleoids (i.e., midcell position for slow growth and ¼ and ¾ cell positions for fast growth) to create a directional motion. To our knowledge, there is no evidence of such localized membrane incorporation.  Furthermore, even if membrane growth was localized at the right places, the fluidity of the cytoplasmic membrane (PMID: 6996724, 20159151, 24735432, 27705775) would be problematic. To go around this fluidity issue, one could potentially evoke a potential connection to the rigid peptidoglycan, but then again, peptidoglycan growth would have to be localized at the middle of the splitting nucleoid to “push” the sister nucleoid apart from each other. However, peptidoglycan growth is dispersed prior to cell constriction (PMID: 35705811, 36097171, 2656655).

      (2) Even if we ignore the aforementioned caveats, Paul Wiggins’s group ruled out the cell elongation/transertion model by showing that the rate of cell elongation is slower than the rate of chromosome segregation (PMID: 23775792). In the revised manuscript, we will provide additional data showing that the cell elongation rate is indeed slower than the nucleoid segregation rate.

      (3) Furthermore, our correlation analysis comparing the rate of nucleoid segregation to the rate of either cell elongation or polysome accumulation argues that polysome accumulation plays a larger role than cell elongation in nucleoid segregation. These data were already shown in the original manuscript (Figure 1I and Figure 1 – figure supplement 3) but were not highlighted in this context. We will revise the text to clarify this point.

      (4) The membrane attachment/cell elongation model does not explain the nucleoid asymmetries described in our paper (Figure 3), whereas they can be recapitulated by our model.

      (5) The cell elongation/transertion model cannot predict the aberrant nucleoid dynamics observed when chromosomal expression is largely redirected to plasmid expression. In the revised manuscript, we will add simulation results showing that these nucleoid dynamics are predicted by our model.

      In line of these arguments, we do not believe that a mechanism based on membrane attachment and cell elongation is the major driver of nucleoid segregations. However, we do believe that it may play a complementary role (see “Nucleoid segregation likely involves multiple factors” in the Discussion). We will revise this section to clarify our thoughts and mention the potential role of transertion.

      Incorporating this perspective into the discussion or future iterations of the model may provide a more comprehensive framework that aligns with the experimental observations in this study and previous work.

      As noted above, we will revise the text to mention about transertion.

      Simplification of Ribosome States:

      Combining monomeric and translating ribosomes into a single 'polysome' category may overlook spatial variations in these states, particularly during ribosome accumulation at the mid-cell. Without validating uniform mRNA distribution or conducting experimental controls such as FRAP or single-molecule measurements to estimate the proportions of ribosome states based on diffusion, this assumption remains speculative.

      Indeed, for simplicity, we adopt an average description of all polysomes with an average diffusion coefficient and interaction parameters, which is sufficient for capturing the fundamental mechanism underlying nucleoid segregation. To illustrate that considering multiple polysome species does not change the physical picture, we consider an extension of our model, which contains three polysome species, each with a different diffusion coefficient (D<SUB>P</SUB> = 0.018, 0.023, or 0.028 μm<sup>2</sup>/s), reflecting that polysomes with more ribosomes will have a lower diffusion coefficient. Simulation of this model reveals that the different polysome species have essentially the same concentration distribution, suggesting that the average description in our minimal model is sufficient for our purposes. We will present these new simulation results in the revised manuscript.

    1. Author response:

      eLife assessment

      This study provides valuable information on the mechanism of PepT2 through enhanced-sampling molecular dynamics, backed by cell-based assays, highlighting the importance of protonation of selected residues for the function of a proton-coupled oligopeptide transporter (hsPepT2). The molecular dynamics approaches are convincing, but with limitations that could be addressed in the manuscript, including lack of incorporation of a protonation coordinate in the free energy landscape, possibility of protonation of the substrate, errors with the chosen constant pH MD method for membrane proteins, dismissal of hysteresis emerging from the MEMENTO method, and the likelihood of other residues being affected by peptide binding. Some changes to the presentation could be considered, including a better description of pKa calculations and the inclusion of error bars in all PMFs. Overall, the findings will appeal to structural biologists, biochemists, and biophysicists studying membrane transporters.

      We would like to express our gratitude to the reviewers for providing their feedback on our manuscript, and also for recognising the variety of computational methods employed, the amount of sampling collected and the experimental validation undertaken. Following the individual reviewer comments, as addressed point-by-point below, we will shortly prepare a revised version of this paper. Intended changes to the revised manuscript are marked up in bold font in the detailed responses below, but before that we address some of the comments made above in the general assessment:

      • “lack of incorporation of a protonation coordinate in the free energy landscape”. We acknowledge that of course it would be highly desirable to treat protonation state changes explicitly and fully coupled to conformational changes. However, at this point in time, evaluating such a free energy landscape is not computationally feasible (especially considering that the non-reactive approach taken here already amounts to almost 1ms of total sampling time). Previous reports in the literature tend to focus on either simpler systems or a reduced subset of a larger problem. As we were trying to obtain information on the whole transport cycle, we decided to focus here on non-reactive methods.

      • “possibility of protonation of the substrate”. The reviewers are correct in pointing out this possibility, which we had not discussed explicitly in our manuscript. Briefly, while we describe a mechanism in which protonation of only protein residues (with an unprotonated ligand) can account for driving all the necessary conformational changes of the transport cycle, there is some evidence for a further intermediate protonation site in our data (as we commented on in the first version of the manuscript as well), which may or may not be the substrate itself. A future explicit treatment of the proton movements through the transporter, when it will become computationally tractable to do so, will have to include the substrate as a possible protonation site; for the present moment, we will amend our discussion to alert the reader to the possibility that the substrate could be an intermediate to proton transport. This has repercussions for our study of the E56 pKa value, where – if protons reside with a significant population at the substrate C-terminus – our calculated shift in pKa upon substrate binding could be an overestimate, although we would qualitatively expect the direction of shift to be unaffected. However, we also anticipate that treating this potential coupling explicitly would make convergence of any CpHMD calculation impractical to achieve and thus it may be the case that for now only a semi-quantitative conclusion is all that can be obtained.

      • “errors with the chosen constant pH MD method for membrane proteins”. We acknowledge that – as reviewer #1 has reminded us – the AMBER implementation of hybrid-solvent CpHMD is not rigorous for membrane proteins, and as such we will add a cautionary note to our paper. We will also explain how the use of the ABFE thermodynamic cycle calculations helps to validate the CpHMD results in a completely orthogonal manner (we will promote this validation which was in the supplementary figures into the main text in the revised version). We therefore remain reasonably confident in the results presented with regards to the reported pKa shift of E56 upon substrate binding, and suggest that if the impact of neglecting the membrane in the implicit-solvent stage of CpHMD is significant, then there is likely an error cancellation when considering shifts induced by the incoming substrate.

      • “dismissal of hysteresis emerging from the MEMENTO method”. We have shown in our method design paper how the use of the MEMENTO method drastically reduces hysteresis compared to steered MD and metadynamics for path generation, and find this improvement again for PepT2 in this study. We will address reviewer #3’s concern about our presentation on this point by revising our introduction of the MEMENTO method, as detailed in the response below.

      • “the likelihood of other residues being affected by peptide binding”. In this study, we have investigated in detail the involvement of several residues in proton-coupled di-peptide transport by PepT2. Short of the potential intermediate protonation site mentioned above, the set of residues we investigate form a minimal set of sorts within which the important driving forces of alternating access can be rationalised. We have not investigated in substantial detail here the residues involved in holding the peptide in the binding site, as they are well studied in the literature and ligand promiscuity is not the problem of interest here. It remains entirely possible that further processes contribute to the mechanism of driving conformational changes by involving other residues not considered in this paper. We will make our speculation that an ensemble of different processes may be contributing simultaneously more explicit in our revision, but do not believe any of our conclusions would be affected by this.

      As for the additional suggested changes in presentation, we will provide the requested details on the CpHMD analysis. Furthermore, we will use the convergence data presented separately in figures S12 and S16 to include error bars on our 1D-reprojections of the 2D-PMFs in figures 3, 4 and 5. (Note that we will opt to not do so in figures S10 and S15 which collate all 1D PMF reprojections for the OCC ↔ OF and OCC ↔ IF transitions in single reference plots, respectively, to avoid overcrowding those necessarily busy figures). We are also changing the colours schemes of these plots in our revision to improve accessibility.

      Reviewer #1 (Public Review):

      The authors have performed all-atom MD simulations to study the working mechanism of hsPepT2. It is widely accepted that conformational transitions of proton-coupled oligopeptide transporters (POTs) are linked with gating hydrogen bonds and salt bridges involving protonatable residues, whose protonation triggers gate openings. Through unbiased MD simulations, the authors identified extra-cellular (H87 and D342) and intra-cellular (E53 and E622) triggers. The authors then validated these triggers using free energy calculations (FECs) and assessed the engagement of the substrate (Ala-Phe dipeptide). The linkage of substrate release with the protonation of the ExxER motif (E53 and E56) was confirmed using constant-pH molecular dynamics (CpHMD) simulations and cellbased transport assays. An alternating-access mechanism was proposed. The study was largely conducted properly, and the paper was well-organized. However, I have a couple of concerns for the authors to consider addressing.

      We would like to note here that it may be slightly misleading to the reader to state that “The linkage of substrate release with the protonation of the ExxER motif (E53 and E56) was confirmed using constant-pH molecular dynamics (CpHMD) simulations and cell-based transport assays.” The cellbased transport assays confirmed the importance of the extracellular gating trigger residues H87, S321 and D342 (as mentioned in the preceding sentence), not of the substrate-protonation link as this line might be understood to suggest.

      (1) As a proton-coupled membrane protein, the conformational dynamics of hsPepT2 are closely coupled to protonation events of gating residues. Instead of using semi-reactive methods like CpHMD or reactive methods such as reactive MD, where the coupling is accounted for, the authors opted for extensive non-reactive regular MD simulations to explore this coupling. Note that I am not criticizing the choice of methods, and I think those regular MD simulations were well-designed and conducted. But I do have two concerns.

      a) Ideally, proton-coupled conformational transitions should be modelled using a free energy landscape with two or more reaction coordinates (or CVs), with one describing the protonation event and the other describing the conformational transitions. The minimum free energy path then illustrates the reaction progress, such as OCC/H87D342- → OCC/H87HD342H → OF/H87HD342H as displayed in Figure 3.

      We concur with the reviewer that the ideal way of describing the processes studied in our paper would be as a higher-dimensional free energy landscapes obtained from a simulation method that can explicitly model proton-transfer processes. Indeed, it would have been particularly interesting and potentially informative with regards to the movement of protons down into the transporter in the OF → OCC → IF sequence of transitions. As we note in our discussion on the H87→E56 proton transfer:

      “This could be investigated using reactive MD or QM/MM simulations (both approaches have been employed for other protonation steps of prokaryotic peptide transporters, see Parker et al. (2017) and Li et al. (2022)). However, the putative path is very long (≈ 1.7 nm between H87 and E56) and may or may not involve a large number of intermediate protonatable residues, in addition to binding site water. While such an investigation is possible in principle, it is beyond the scope of the present study.”

      Where even sampling the proton transfer step itself in an essentially static protein conformation would be pushing the boundaries of what has been achieved in the field, we believe that considering the current state-of-the-art, a fully coupled investigation of large-scale conformational changes and proton-transfer reaction is not yet feasible in a realistic/practical time frame. We also note this limitation already when we say that:

      “The question of whether proton binding happens in OCC or OF warrants further investigation, and indeed the co-existence of several mechanisms may be plausible here”.

      Nonetheless, we are actively exploring approaches to treat uptake and movement of protons explicitly for future work.

      In our revision, we will expand on our discussion of the reasoning behind employing a nonreactive approach and the limitations that imposes on what questions can be answered in this study.

      Without including the protonation as a CV, the authors tried to model the free energy changes from multiple FECs using different charge states of H87 and D342. This is a practical workaround, and the conclusion drawn (the OCC→ OF transition is downhill with protonated H87 and D342) seems valid. However, I don't think the OF states with different charge states (OF/H87D342-, OF/H87HD342-, OF/H87D342H, and OF/H87HD342H) are equally stable, as plotted in Figure 3b. The concern extends to other cases like Figures 4b, S7, S10, S12, S15, and S16. While it may be appropriate to match all four OF states in the free energy plot for comparison purposes, the authors should clarify this to ensure readers are not misled.

      The reviewer is correct in their assessment that the aligning of PMFs in these figures is arbitrary; no relative free energies of the PMFs to each other can be estimated without explicit free energy calculations at least of protonation events at the end state basins. The PMFs in our figures are merely superimposed for illustrating the differences in shape between the obtained profiles in each condition, as discussed in the text, and we will make this clear in the appropriate figure captions in our revision.

      b) Regarding the substrate impact, it appears that the authors assumed fixed protonation states. I am afraid this is not necessarily the case. Variations in PepT2 stoichiometry suggest that substrates likely participate in proton transport, like the Phe-Ala (2:1) and Phe-Gln (1:1) dipeptides mentioned in the introduction. And it is not rigorous to assume that the N- and C-termini of a peptide do not protonate/deprotonate when transported. I think the authors should explicitly state that the current work and the proposed mechanism (Figure 8) are based on the assumption that the substrates do not uptake/release proton(s).

      This is indeed an assumption inherent in the current work. While we do “speculate that the proton movement processes may happen as an ensemble of different mechanisms, and potentially occur contemporaneously with the conformational change” we do not in the current version indicate explicitly that this may involve the substrate. We will make clear the assumption and this possibility in the revised version of our paper. Indeed, as we discuss, there is some evidence in our PMFs of an additional protonation site not considered thus far, which may or may not be the substrate. We will make note of this point in the revised manuscript.

      As for what information can be drawn from the given experimental stoichiometries, we note in our paper that “a 2:1 stoichiometry was reported for the neutral di-peptide D-Phe-L-Ala and 3:1 for anionic D-Phe-L-Glu. (Chen et al., 1999) Alternatively, Fei et al. (1999) have found 1:1 stoichiometries for either of D-Phe-L-Gln (neutral), D-Phe-L-Glu (anionic), and D-Phe-L-Lys (cationic).”

      We do not assume that it is our place to arbit among the apparent discrepancies in the experimental data here, although we believe that our assumed 2:1 stoichiometry is additionally “motivated also by our computational results that indicate distinct and additive roles played by two protons in the conformational cycle mechanism”.

      (2) I have more serious concerns about the CpHMD employed in the study.

      a) The CpHMD in AMBER is not rigorous for membrane simulations. The underlying generalized Born model fails to consider the membrane environment when updating charge states. In other words, the CpHMD places a membrane protein in a water environment to judge if changes in charge states are energetically favorable. While this might not be a big issue for peripheral residues of membrane proteins, it is likely unphysical for internal residues like the ExxER motif. As I recall, the developers have never used the method to study membrane proteins themselves. The only CpHMD variant suitable for membrane proteins is the membrane-enabled hybrid-solvent CpHMD in CHARMM. While I do not expect the authors to redo their CpHMD simulations, I do hope the authors recognize the limitations of their method.

      We will discuss the limitations of the AMBER CpHMD implementation in the revised version. However, despite that, we believe we have in fact provided sufficient grounds for our conclusion that substrate binding affects ExxER motif protonation in the following way:

      In addition to CpHMD simulations, we establish the same effect via ABFE calculations, where the substrate affinity is different at the E56 deprotonated vs protonated protein. This is currently figure S20, though in the revised version we will move this piece of validation into a new panel of figure 6 in the main text, since it becomes more important with the CpHMD membrane problem in mind. Since the ABFE calculations are conducted with an all-atom representation of the lipids and the thermodynamic cycle closes well, it would appear that if the chosen CpHMD method has a systematic error of significant magnitude for this particular membrane protein system, there may be the benefit of error cancellation. While the calculated absolute pKa values may not be reliable, the difference made by substrate binding appears to be so, as judged by the orthogonal ABFE technique.

      Although the reviewer does “not expect the authors to redo their CpHMD simulations”, we consider that it may be helpful to the reader to share in this response some results from trials using the continuous, all-atom constant pH implementation that has recently become available in GROMACS (Aho et al 2022, https://pubs.acs.org/doi/10.1021/acs.jctc.2c00516) and can be used rigorously with membrane proteins, given its all-atom lipid representation.

      Unfortunately, when trying to titrate E56 in this CpHMD implementation, we found few protonationstate transitions taking place, and the system often got stuck in protonation state–local conformation coupled minima (which need to interconvert through rearrangements of the salt bridge network involving slow side-chain dihedral rotations in E53, E56 and R57). Author response image 1 shows this for the apo OF state, Author response image 2 shows how noisy attempts at pKa estimation from this data turn out to be, necessitating the use of a hybrid-solvent method.

      Author response image 1.

      All-atom CpHMD simulations of apo-OF PepT2. Red indicates protonated E56, blue is deprotonated.

      Author response image 2.

      Difficulty in calculating the E56 pKa value from the noisy all-atom CpHMD data shown in Author response image 1

      b) It appears that the authors did not make the substrate (Ala-Phe dipeptide) protonatable in holosimulations. This oversight prevents a complete representation of ligand-induced protonation events, particularly given that the substrate ion pairs with hsPepT2 through its N- & C-termini. I believe it would be valuable for the authors to acknowledge this potential limitation.

      In this study, we implicitly assumed from the outset that the substrate does not get protonated, which – as by way of response to the comment above – we will acknowledge explicitly in revision. This potential limitation for the available mechanisms for proton transfer also applies to our investigation of the ExxER protonation states. In particular, a semi-grand canonical ensemble that takes into account the possibility of substrate C-terminus protonation may also sample states in which the substrate is protonated and oriented away from R57, thus leaving the ExxER salt bridge network in an apo-like state. The consequence would be that while the direction of shift in E56 pKa value will be the same, our CpHMD may overestimate its magnitude. It would thus be interesting to make the C-terminus protonatable for obtaining better quantitative estimates of the E56 pKa shift (as is indeed true in general for any other protein protonatable residue, though the effects are usually assumed to be negligible). We do note, however, that convergence of the CpHMD simulations would be much harder if the slow degree of freedom of substrate reorientation (which in our experience takes 10s to 100s of ns in this binding pocket) needs to be implicitly equilibrated upon protonation state transitions. We will discuss such considerations in the revision.

      Reviewer #2 (Public Review):

      This is an interesting manuscript that describes a series of molecular dynamics studies on the peptide transporter PepT2 (SLC15A2). They examine, in particular, the effect on the transport cycle of protonation of various charged amino acids within the protein. They then validate their conclusions by mutating two of the residues that they predict to be critical for transport in cell-based transport assays. The study suggests a series of protonation steps that are necessary for transport to occur in Petp2. Comparison with bacterial proteins from the same family shows that while the overall architecture of the proteins and likely mechanism are similar, the residues involved in the mechanism may differ.

      Strengths:

      This is an interesting and rigorous study that uses various state-of-the-art molecular dynamics techniques to dissect the transport cycle of PepT2 with nearly 1ms of sampling. It gives insight into the transport mechanism, investigating how the protonation of selected residues can alter the energetic barriers between various states of the transport cycle. The authors have, in general, been very careful in their interpretation of the data.

      Weaknesses:

      Interestingly, they suggest that there is an additional protonation event that may take place as the protein goes from occluded to inward-facing but they have not identified this residue.

      We have indeed suggested that there may be an additional protonation site involved in the conformational cycle that we have not been able to capture, which – as we discuss in our paper – might be indicated by the shapes of the OCC ↔ IF PMFs given in Figure S15. One possibility is for this to be the substrate itself (see the response to reviewer #1 above) though within the scope of this study the precise pathway by which protons move down the transporter and the exact ordering of conformational change and proton transfer reactions remains a (partially) open question. We acknowledge this and denote it with question marks in the mechanistic overview we give in Figure 8, and also “speculate that the proton movement processes may happen as an ensemble of different mechanisms, and potentially occur contemporaneously with the conformational change”.

      Some things are a little unclear. For instance, where does the state that they have defined as occluded sit on the diagram in Figure 1a? - is it truly the occluded state as shown on the diagram or does it tend to inward- or outward-facing?

      Figure 1a is a simple schematic overview intended to show which structures of PepT2 homologues are available to use in simulations. This was not meant to be a quantitative classification of states. Nonetheless, we can note that the OCC state we derived has extra- and intracellular gate opening distances (as measured by the simple CVs defined in the methods and illustrated in Figure 2a) that indicate full gate closure at both sides. In particular, although it was derived from the IF state via biased sampling, the intracellular gate opening distance in the OCC state used for our conformational change enhanced sampling was comparable to that of the OF state (ie, full closure of the gate), see Figure S2b and the grey bars therein. Therefore, we would schematically classify the OCC state to lie at the center of the diagram in Figure 1a. Furthermore, it is largely stable over triplicates of 1 μslong unbiased MD, where in 2/3 replicates the gates remain stable, and the remaining replicate there is partial opening of the intracellular gate (as shown in Figure 2 b/c under the “apo standard” condition). We comment on this in the main text by saying that “The intracellular gate, by contrast, is more flexible than the extracellular gate even in the apo, standard protonation state”, and link it to the lower barrier for transition to IF than to OF. We did this by saying that “As for the OCC↔OF transitions, these results explain the behaviour we had previously observed in the unbiased MD of Figure 2c.” We acknowledge this was not sufficiently clear and will add details to the latter sentence in revision to help clarify better the nature of the occluded state.

      The pKa calculations and their interpretation are a bit unclear. Firstly, it is unclear whether they are using all the data in the calculations of the histograms, or just selected data and if so on what basis was this selection done. Secondly, they dismiss the pKa calculations of E53 in the outward-facing form as not being affected by peptide binding but say that E56 is when there seems to be a similar change in profile in the histograms.

      In our manuscript, we have provided two distinct analyses of the raw CpHMD data. Firstly, we analysed the data by the replicates in which our simulations were conducted (Figure 6, shown as bar plots with mean from triplicates +/- standard deviation), where we found that only the effect on E56 protonation was distinct as lying beyond the combined error bars. This analysis uses the full amount of sampling conducted for each replicate. However, since we found that the range of pKa values estimated from 10ns/window chunks was larger than the error bars obtained from the replicate analysis (Figures S17 and S18), we sought to verify our conclusion by pooling all chunk estimates and plotting histograms (Figure S19). We recover from those the effect of substrate binding on the E56 protonation state on both the OF and OCC states. However, as the reviewer has pointed out (something we did not discuss in our original manuscript), there is a shift in the pKa of E53 of the OF state only. In fact, the trend is also apparent in the replicate-based analysis of Figure 6, though here the larger error bars overlap. In our revision, we will add more details of these analyses for clarity (including more detailed figure captions regarding the data used in Figure 6) as well as a discussion of the partial effect on the E53 pKa value.

      We do not believe, however, that our key conclusions are negatively affected. If anything, a further effect on the E53 pKa which we had not previously commented on (since we saw the evidence as weaker, pertaining to only one conformational state) would strengthen the case for an involvement of the ExxER motif in ligand coupling.

      Reviewer #3 (Public Review):

      Summary:

      Lichtinger et al. have used an extensive set of molecular dynamics (MD) simulations to study the conformational dynamics and transport cycle of an important member of the proton-coupled oligopeptide transporters (POTs), namely SLC15A2 or PepT2. This protein is one of the most wellstudied mammalian POT transporters that provides a good model with enough insight and structural information to be studied computationally using advanced enhanced sampling methods employed in this work. The authors have used microsecond-level MD simulations, constant-PH MD, and alchemical binding free energy calculations along with cell-based transport assay measurements; however, the most important part of this work is the use of enhanced sampling techniques to study the conformational dynamics of PepT2 under different conditions.

      The study attempts to identify links between conformational dynamics and chemical events such as proton binding, ligand-protein interactions, and intramolecular interactions. The ultimate goal is of course to understand the proton-coupled peptide and drug transport by PepT2 and homologous transporters in the solute carrier family.

      Some of the key results include:

      (1) Protonation of H87 and D342 initiate the occluded (Occ) to the outward-facing (OF) state transition.

      (2) In the OF state, through engaging R57, substrate entry increases the pKa value of E56 and thermodynamically facilitates the movement of protons further down.

      (3) E622 is not only essential for peptide recognition but also its protonation facilitates substrate release and contributes to the intracellular gate opening. In addition, cell-based transport assays show that mutation of residues such as H87 and D342 significantly decreases transport activity as expected from simulations.

      Strengths:

      (1) This is an extensive MD-based study of PepT2, which is beyond the typical MD studies both in terms of the sheer volume of simulations as well as the advanced methodology used. The authors have not limited themselves to one approach and have appropriately combined equilibrium MD with alchemical free energy calculations, constant-pH MD, and geometry-based free energy calculations. Each of these 4 methods provides a unique insight regarding the transport mechanism of PepT2.

      (2) The authors have not limited themselves to computational work and have performed experiments as well. The cell-based transport assays clearly establish the importance of the residues that have been identified as significant contributors to the transport mechanism using simulations.

      (3) The conclusions made based on the simulations are mostly convincing and provide useful information regarding the proton pathway and the role of important residues in proton binding, protein-ligand interaction, and conformational changes.

      Weaknesses:

      (1) Some of the statements made in the manuscript are not convincing and do not abide by the standards that are mostly followed in the manuscript. For instance, on page 4, it is stated that "the K64-D317 interaction is formed in only ≈ 70% of MD frames and therefore is unlikely to contribute much to extracellular gate stability." I do not agree that 70% is negligible. Particularly, Figure S3 does not include the time series so it is not clear whether the 30% of the time where the salt bridge is broken is in the beginning or the end of simulations. For instance, it is likely that the salt bridge is not initially present and then it forms very strongly. Of course, this is just one possible scenario but the point is that Figure S3 does not rule out the possibility of a significant role for the K64-D317 salt bridge.

      The reviewer is right to point out that the statement and Figure S3 as they stand do not adequately support our decision to exclude the K64-D317 salt-bridge in our further investigations. The violin plot shown in Figure S3, visualised as pooled data from unbiased 1 μs triplicates, does indeed not rule out a scenario where the salt bridge only formed late in our simulations (or only in some replicates), but then is stable. Therefore, in our revision, we will include the appropriate time-series of the salt bridge distances, showing how K64-D317 is initially stable but then falls apart in replicate 1, and is transiently formed and disengaged across the trajectories in replicates 2 and 3. We will also remake the data for this plot as we discovered a bug in the relevant analysis script that meant the D170-K642 distance was not calculated accurately. The results are however almost identical, and our conclusions remain.

      (2) Similarly, on page 4, it is stated that "whether by protonation or mutation - the extracellular gate only opens spontaneously when both the H87 interaction network and D342-R206 are perturbed (Figure S5)." I do not agree with this assessment. The authors need to be aware of the limitations of this approach. Consider "WT H87-prot" and "D342A H87-prot": when D342 residue is mutated, in one out of 3 simulations, we see the opening of the gate within 1 us. When D342 residue is not mutated we do not see the opening in any of the 3 simulations within 1 us. It is quite likely that if rather than 3 we have 10 simulations or rather than 1 us we have 10 us simulations, the 0/3 to 1/3 changes significantly. I do not find this argument and conclusion compelling at all.

      If the conclusions were based on that alone, then we would agree. However, this section of work covers merely the observations of the initial unbiased simulations which we go on to test/explore with enhanced sampling in the rest of the paper, and which then lead us to the eventual conclusions.

      Figure S5 shows the results from triplicate 1 μs-long trajectories as violin-plot histograms of the extracellular gate opening distance, also indicating the first and final frames of the trajectories as connected by an arrow for orientation – a format we chose for intuitively comparing 48 trajectories in one plot. The reviewer reads the plot correctly when they analyse the “WT H87-prot” vs “D342A H87-prot” conditions. In the former case, no spontaneous opening in unbiased MD is taking place, whereas when D342 is mutated to alanine in addition to H87 protonation, we see spontaneous transition in 1 out of 3 replicates. However, the reviewer does not seem to interpret the statement in question in our paper (“the extracellular gate only opens spontaneously when both the H87 interaction network and D342-R206 are perturbed”) in the way we intended it to be understood. We merely want to note here a correlation in the unbiased dataset we collected at this stage, and indeed the one spontaneous opening in the case comparison picked out by the reviewer is in the condition where both the H87 interaction network and D342-R206 are perturbed. In noting this we do not intend to make statistically significant statements from the limited dataset. Instead, we write that “these simulations show a large amount of stochasticity and drawing clean conclusions from the data is difficult”. We do however stand by our assessment that from this limited data we can “already appreciate a possible mechanism where protons move down the transporter pore” – a hypothesis we investigate more rigorously with enhanced sampling in the rest of the paper. We will revise the section in question to make clearer that the unbiased MD is only meant to give an initial hypothesis here to be investigated in more detail in the following sections. In doing so, we will also incorporate, as we had not done before, the case (not picked out by the reviewer here but concerning the same figure) of S321A & H87 prot. In the third replicate, this shows partial gate opening towards the end of the unbiased trajectory (despite D342 not being affected), highlighting further the stochastic nature that makes even clear correlative conclusions difficult to draw.

      (3) While the MEMENTO methodology is novel and interesting, the method is presented as flawless in the manuscript, which is not true at all. It is stated on Page 5 with regards to the path generated by MEMENTO that "These paths are then by definition non-hysteretic." I think this is too big of a claim to say the paths generated by MEMENTO are non-hysteretic by definition. This claim is not even mentioned in the original MEMENTO paper. What is mentioned is that linear interpolation generates a hysteresis-free path by definition. There are two important problems here: (a) MEMENTO uses the linear interpolation as an initial step but modifies the intermediates significantly later so they are no longer linearly interpolated structures and thus the path is no longer hysteresisfree; (b) a more serious problem is the attribution of by-definition hysteresis-free features to the linearly interpolated states. This is based on conflating the hysteresis-free and unique concepts. The hysteresis in MD-based enhanced sampling is related to the presence of barriers in orthogonal space. For instance, one may use a non-linear interpolation of any type and get a unique pathway, which could be substantially different from the one coming from the linear interpolation. None of these paths will be hysteresis-free necessarily once subjected to MD-based enhanced sampling techniques.

      We certainly do not intend to claim that the MEMENTO method is flawless. The concern the reviewer raises around the statement "These paths are then by definition non-hysteretic" is perhaps best addressed by a clarification of the language used and considering how MEMENTO is applied in this work.

      Hysteresis in the most general sense denotes the dependence of a system on its history, or – more specifically – the lagging behind of the system state with regards to some physical driver (for example the external field in magnetism, whence the term originates). In the context of biased MD and enhanced sampling, hysteresis commonly denotes the phenomenon where a path created by a biased dynamics method along a certain collective variable lags behind in phase space in slow orthogonal degrees of freedom (see Figure 1 in Lichtinger and Biggin 2023, https://doi.org/10.1021/acs.jctc.3c00140). When used to generate free energy profiles, this can manifest as starting state bias, where the conformational state that was used to seed the biased dynamics appears lower in free energy than alternative states. Figure S6 shows this effect on the PepT2 system for both steered MD (heavy atom RMSD CV) + umbrella sampling (tip CV) and metadynamics (tip CV). There is, in essence, a coupled problem: without an appropriate CV (which we did not have to start with here), path generation that is required for enhanced sampling displays hysteresis, but the refinement of CVs is only feasible when paths connecting the true phase space basins of the two conformations are available. MEMENTO helps solve this issue by reconstructing protein conformations along morphing paths which perform much better than steered MD paths with respect to giving consistent free energy profiles (see Figure S7 and the validation cases in the MEMENTO paper), even if the same CV is used in umbrella sampling.

      There are still differences between replicates in those PMFs, indicating slow conformational flexibility propagated from end-state sampling through MEMENTO. We use this to refine the CVs further with dimensionality reduction (see the Method section and Figure S8), before moving to 2D-umbrella sampling (figure 3). Here, we think, the reviewer’s point seems to bear. The MEMENTO paths are ‘non-hysteretic by definition’ with respect to given end states in the sense that they connect (by definition) the correct conformations at both end-states (unlike steered MD), which in enhanced sampling manifests as the absence of the strong starting-state bias we had previously observed (Figure S7 vs S6). They are not, however, hysteresis-free with regards to how representative of the end-state conformational flexibility the structures given to MEMENTO really were, which is where the iterative CV design and combination of several MEMENTO paths in 2D-PMFs comes in.

      We also cannot make a direct claim about whether in the transition region the MEMENTO paths might be separated from the true (lower free energy) transition paths by slow orthogonal degrees of freedom, which may conceivably result in overestimated barrier heights separating two free energy basins. We cannot guarantee that this is not the case, but neither in our MEMENTO validation examples nor in this work have we encountered any indications of a problem here.

      We hope that the reviewer will be satisfied by our revision, where we will replace the wording in question by a statement that the MEMENTO paths do not suffer from hysteresis that is otherwise incurred as a consequence of not reaching the correct target state in the biased run (in some orthogonal degrees of freedom).

    1. Author Response

      Response to the Reviews

      We are grateful for these balanced, nuanced evaluations of our work concerning the observed epistatic trends and our interpretations of their mechanistic origins. Overall, we think the reviewers have done an excellent job at recognizing the novel aspects of our findings while also discussing the caveats associated with our interpretations of the biophysical effects of these mutations. We believe it is important to consider both of these aspects of our work in order to appreciate these advances and what sorts of pertinent questions remain.

      Notably, both reviewers suggest that a lack of experimental approaches to compare the conformational properties of GnRHR variants weakens our claims. We would first humbly suggest that this constitutes a more general caveat that applies to nearly all investigations of the cellular misfolding of α-helical membrane proteins. Whether or not any current in vitro folding measurements report on conformational transitions that are relevant to cellular protein misfolding reactions remains an active area of debate (discussed further below). Nevertheless, while we concede that our structural and/ or computational evaluations of various mutagenic effects remain speculative, prevailing knowledge on the mechanisms of membrane protein folding suggest our mutations of interest (V276T and W107A) are highly unlikely to promote misfolding in precisely the same way. Thus, regardless of whether or not we were able experimentally compare the relevant folding energetics of GnRHR variants, we are confident that the distinct epistatic interactions formed by these mutations reflect variations in the misfolding mechanism and that they are distinct from the interactions that are observed in the context of stable proteins. In the following, we provide detailed considerations concerning these caveats in relation to the reviewers’ specific comments.

      Reviewer #1 (Public Review):

      The paper carries out an impressive and exhaustive non-sense mutagenesis using deep mutational scanning (DMS) of the gonadotropin-releasing hormone receptor for the WT protein and two single point mutations that I) influence TM insertion (V267T) and ii) influence protein stability (W107A), and then measures the effect of these mutants on correct plasma membrane expression (PME).

      Overall, most mutations decreased mGnRHR PME levels in all three backgrounds, indicating poor mutational tolerance under these conditions. The W107A variant wasn't really recoverable with low levels of plasma membrane localisation. For the V267T variant, most additional mutations were more deleterious than WT based on correct trafficking, indicating a synergistic effect. As one might expect, there was a higher degree of positive correlation between V267T/W107A mutants and other mutants located in TM regions, confirming that improper trafficking was a likely consequence of membrane protein co-translational folding. Nevertheless, context is important, as positive synergistic mutants in the V27T could be negative in the W107A background and vice versa. Taken together, this important study highlights the complexity of membrane protein folding in dissecting the mechanism-dependent impact of disease-causing mutations related to improper trafficking.

      Strengths

      This is a novel and exhaustive approach to dissecting how receptor mutations under different mutational backgrounds related to co-translational folding, could influence membrane protein trafficking.

      Weaknesses

      The premise for the study requires an in-depth understanding of how the single-point mutations analysed affect membrane protein folding, but the single-point mutants used seem to lack proper validation.

      Given our limited understanding of the structural properties of misfolded membrane proteins, it is unclear whether the relevant conformational effects of these mutations can be unambiguously validated using current biochemical and/ or biophysical folding assays. X-ray crystallography, cryo-EM, and NMR spectroscopy measurements have demonstrated that many purified GPCRs retain native-like structural ensembles within certain detergent micelles, bicelles, and/ or nanodiscs. However, helical membrane protein folding measurements typically require titration with denaturing detergents to promote the formation of a denatured state ensemble (DSE), which will invariably retain considerable secondary structure. Given that the solvation provided by mixed micelles is clearly distinct from that of native membranes, it remains unclear whether these DSEs represent a reasonable proxy for the misfolded conformations recognized by cellular quality control (QC, see https://doi.org/10.1021/acs.chemrev.8b00532). Thus, the use and interpretation of these systems for such purposes remains contentious in the membrane protein folding community. In addition to this theoretical issue, we are unaware of any instances in which GPCRs have been found to undergo reversible denaturation in vitro- a practical requirement for equilibrium folding measurements (https://doi.org/10.1146/annurev-biophys-051013-022926). We note that, while the resistance of GPCRs to aggregation, proteolysis, and/ or mechanical unfolding have also been probed in micelles, it is again unclear whether the associated thermal, kinetic, and/ or mechanical stability should necessarily correspond to their resistance to cotranslational and/ or posttranslational misfolding. Thus, even if we had attempted to validate the computational folding predictions employed herein, we suspect that any resulting correlations with cellular expression may have justifiably been viewed by many as circumstantial. Simply put, we know very little about the non-native conformations are generally involved in the cellular misfolding of α-helical membrane proteins, much less how to measure their relative abundance. From a philosophical standpoint, we prefer to let cells tell us what sorts of broken protein variants are degraded by their QC systems, then do our best to surmise what this tells us about the relevant properties of cellular DSEs.

      Despite this fundamental caveat, we believe that the chosen mutations and our interpretation of their relevant conformational effects are reasonably well-informed by current modeling tools and by prevailing knowledge on the physicochemical drivers of membrane protein folding and misfolding. Specifically, the mechanistic constraints of translocon-mediated membrane integration provide an understanding of the types of mutations that are likely to disrupt cotranslational folding. Though we are still learning about the protein complexes that mediate membrane translocation (https://doi.org/10.1038/s41586-022-05336-2), it is known that this underlying process is fundamentally driven by the membrane depth-dependent amino acid transfer free energies (https://doi.org/10.1146/annurev.biophys.37.032807.125904). This energetic consideration suggests introducing polar side chains near the center of a nascent TMDs should almost invariably reduce the efficiency of topogenesis. To confirm this in the context of TMD6 specifically, we utilized a well-established biochemical reporter system to confirm that V276T attenuates its translocon-mediated membrane integration (Fig. S1)- at least in the context of a chimeric protein. We also constructed a glycosylation-based topology reporter for full-length GnRHR, but ultimately found its’ in vitro expression to be insufficient to detect changes in the nascent topological ensemble. In contrast to V276T, the W107A mutation is predicted to preserve the native topological energetics of GnRHR due to its position within a soluble loop region. W107A is also unlike V276T in that it clearly disrupts tertiary interactions that stabilize the native structure. This mutation should preclude the formation of a structurally conserved hydrogen bonding network that has been observed in the context of at least 25 native GPCR structures (https://doi.org/10.7554/eLife.5489). However, without a relevant folding assay, the extent to which this network stabilizes the native GnRHR fold in cellular membranes remains unclear. Overall, we admit that these limitations have prevented us from measuring how much V276T alters the efficiency of GnRHR topogenesis, how much the W107A destabilizes the native fold, or vice versa. Nevertheless, given these design principles and the fact that both reduce the plasma membrane expression of GnRHR, as expected, we are highly confident that the structural defects generated by these mutations do, in fact, promote misfolding in their own ways. We also concede that the degree to which these mutagenic perturbations are indeed selective for specific folding processes is somewhat uncertain. However, it seems exceedingly unlikely that these mutations should disrupt topogenesis and/ or the folding of the native topomer to the exact same extent. From our perspective, this is the most important consideration with respect to the validity of the conclusions we have made in this manuscript.

      Furthermore, plasma membrane expression has been used as a proxy for incorrect membrane protein folding, but this not necessarily be the case, as even correctly folded membrane proteins may not be trafficked correctly, at least, under heterologous expression conditions. In addition, mutations can affect trafficking and potential post-translational modifications, like glycosylation.

      While the reviewer is correct that the sorting of folded proteins within the secretory pathway is generally inefficient, it is also true that the maturation of nascent proteins within the ER generally bottlenecks the plasma membrane expression of most α-helical membrane proteins. Our group and several others have demonstrated that the efficiency of ER export generally appears to scale with the propensity of membrane proteins to achieve their correct topology and/ or to achieve their native fold (see https://doi.org/10.1021/jacs.5b03743 and https://doi.org/10.1021/jacs.8b08243). Notably, these investigations all involved proteins that contain native glycosylation and various other post-translational modification sites. While we cannot rule out that certain specific combinations of mutations may alter expression through their perturbation of post-translational GnRHR modifications, we feel confident that the general trends we have observed across hundreds of variants predominantly reflect changes in folding and cellular QC. This interpretation is supported by the relationship between observed trends in variant expression and Rosetta-based stability calculations, which we identified using unbiased unsupervised machine learning approaches (compare Figs. 6B & 6D).

      Reviewer #2 (Public Review):

      Summary:

      In this paper, Chamness and colleagues make a pioneering effort to map epistatic interactions among mutations in a membrane protein. They introduce thousands of mutations to the mouse GnRH Receptor (GnRHR), either under wild-type background or two mutant backgrounds, representing mutations that destabilize GnRHR by distinct mechanisms. The first mutant background is W107A, destabilizing the tertiary fold, and the second, V276T, perturbing the efficiency of cotranslational insertion of TM6 to the membrane, which is essential for proper folding. They then measure the surface expression of these three mutant libraries, using it as a proxy for protein stability, since misfolded proteins do not typically make it to the plasma membrane. The resulting dataset is then used to shed light on how diverse mutations interact epistatically with the two genetic background mutations. Their main conclusion is that epistatic interactions vary depending on the degree of destabilization and the mechanism through which they perturb the protein. The mutation V276T forms primarily negative (aggravating) epistatic interactions with many mutations, as is common to destabilizing mutations in soluble proteins. Surprisingly, W107A forms many positive (alleviating) epistatic interactions with other mutations. They further show that the locations of secondary mutations correlate with the types of epistatic interactions they form with the above two mutants.

      Strengths:

      Such a high throughput study for epistasis in membrane proteins is pioneering, and the results are indeed illuminating. Examples of interesting findings are that: (1) No single mutation can dramatically rescue the destabilization introduced by W107A. (2) Epistasis with a secondary mutation is strongly influenced by the degree of destabilization introduced by the primary mutation. (3) Misfolding caused by mis-insertion tends to be aggravated by further mutations. The discussion of how protein folding energetics affects epistasis (Fig. 7) makes a lot of sense and lays out an interesting biophysical framework for the findings.

      Weaknesses:

      The major weakness comes from the potential limitations in the measurements of surface expression of severely misfolded mutants. This point is discussed quite fairly in the paper, in statements like "the W107A variant already exhibits marginal surface immunostaining" and many others. It seems that only about 5% of the W107A makes it to the plasma membrane compared to wild-type (Figures 2 and 3). This might be a low starting point from which to accurately measure the effects of secondary mutations.

      The reviewer raises an excellent point that we considered at length during the analysis of these data and the preparation of the manuscript. Though we remain confident in the integrity of these measurements and the corresponding analyses, we now realize this aspect of the data merits further discussion and documentation in our forthcoming revision, in which we will outline the following specific lines of reasoning.

      Still, the authors claim that measurements of W107A double mutants "still contain cellular subpopulations with surface immunostaining intensities that are well above or below that of the W107A single mutant, which suggests that this fluorescence signal is sensitive enough to detect subtle differences in the PME of these variants". I was not entirely convinced that this was true.

      We made this statement based on the simple observation that the surface immunostaining intensities across the population of recombinant cells expressing the library of W107A double mutants was consistently broader than that of recombinant cells expressing W107A GnRHR alone (see Author response image 1 for reference). Given that the recombinant cellular library represents a mix of cells expressing ~1600 individual variants that are each present at low abundance, the pronounced tails within this distribution presumably represent the composite staining of many small cellular subpopulations that express collections of variants that deviate from the expression of W107A to an extent that is significant enough to be visible on a log intensity plot.

      Author response image 1.

      Firstly, I think it would be important to test how much noise these measurements have and how much surface immunostaining the W107A mutant displays above the background of cells that do not express the protein at all.

      For reference, the average surface immunostaining intensity of HEK293T cells transiently expressing W107A GnRHR was 2.2-fold higher than that of the IRES-eGFP negative, untransfected cells within the same sample- the WT immunostaining intensity was 9.5-fold over background by comparison. Similarly, recombinant HEK293T cells expressing the W107A double mutant library had an average surface immunostaining intensity that was 2.6-fold over background across the two DMS trials. Thus, while the surface immunostaining of this variant is certainly diminished, we were still able to reliably detect W107A at the plasma membrane even under distinct expression regimes. We will include these and other signal-to-noise metrics for each experiment in a new table in the revised version of this manuscript.

      Beyond considerations related to intensity, we also previously noticed the relative intensity values for W107A double mutants exhibited considerable precision across our two biological replicates. If signal were too poor to detect changes in variant expression, we would have expected a plot of the intensity values across these two replicates to form a scatter. Instead, we found DMS intensity values for individual variants to be highly correlated from one replicate to the next (Pearson’s R= 0.97, see Author response image 2 for reference). This observation empirically demonstrates that this assay consistently differentiated between variants that exhibit slightly enhanced immunostaining from those that have even lower immunostaining than W107A GnRHR.

      Author response image 2.

      But more importantly, it is not clear if under this regimen surface expression still reports on stability/protein fitness. It is unknown if the W107A retains any function or folding at all. For example, it is possible that the low amount of surface protein represents misfolded receptors that escaped the ER quality control.

      While we believe that such questions are outside the scope of this work, we certainly agree that it is entirely possible that some of these variants bypass QC without achieving their native fold. This topic is quite interesting to us but is quite challenging to assess in the context of GPCRs, which have complex fitness landscapes that involve their propensity to distinguish between different ligands, engage specific components associated with divergent downstream signaling pathways, and navigate between endocytic recycling/ degradation pathways following activation. In light of the inherent complexity of GPCR function, we humbly suggest our choice of a relatively simple property of an otherwise complex protein may be viewed as a virtue rather than a shortcoming. Protein fitness is typically cast as the product of abundance and activity. Rather than measuring an oversimplified, composite fitness metric, we focused on one variable (plasma membrane expression) and its dominant effector (folding). We believe restraining the scope in this manner was key for the elucidation of clear mechanistic insights.

      The differential clustering of epistatic mutations (Fig. 6) provides some interesting insights as to the rules that dictate epistasis, but these too are dominated by the magnitude of destabilization caused by one of the mutations. In this case, the secondary mutations that had the most interesting epistasis were exceedingly destabilizing. With this in mind, it is hard to interpret the results that emerge regarding the epistatic interactions of W107A. Furthermore, the most significant positive epistasis is observed when W107A is combined with additional mutations that almost completely abolish surface expression. It is likely that either mutation destabilizes the protein beyond repair. Therefore, what we can learn from the fact that such mutations have positive epistasis is not clear to me. Based on this, I am not sure that another mutation that disrupts the tertiary folding more mildly would not yield different results. With that said, I believe that the results regarding the epistasis of V276T with other mutations are strong and very interesting on their own.

      We agree with the reviewer. In light of our results we believe it is virtually certain that the secondary mutations characterized herein would be likely to form distinct epistatic interactions with mutations that are only mildly destabilizing. Indeed, this insight reflects one of the key takeaway messages from this work- stability-mediated epistasis is difficult to generalize because it should depend on the extent to which each mutation changes the stability (ΔΔG) as well as initial stability of the WT/ reference sequence (ΔG, see Figure 7). Frankly, we are not so sure we would have pieced this together as clearly had we not had the fortune (or misfortune?) of including such a destructive mutation like W107A as a point of reference.

      Additionally, the study draws general conclusions from the characterization of only two mutations, W107A and V276T. At this point, it is hard to know if other mutations that perturb insertion or tertiary folding would behave similarly. This should be emphasized in the text.

      We agree and will be sure to emphasize this point in the revised manuscript.

      Some statistical aspects of the study could be improved:

      1. It would be nice to see the level of reproducibility of the biological replicates in a plot, such as scatter or similar, with correlation values that give a sense of the noise level of the measurements. This should be done before filtering out the inconsistent data.

      We thank the reviewer for this suggestion and will include scatters for each genetic background like the one shown above in the supplement of the revised version of the manuscript.

      1. The statements "Variants bearing mutations within the C- terminal region (ICL3-TMD6-ECL3-TMD7) fare consistently worse in the V276T background relative to WT (Fig. 4 B & E)." and "In contrast, mutations that are 210 better tolerated in the context of W107A mGnRHR are located 211 throughout the structure but are particularly abundant among residues 212 in the middle of the primary structure that form TMD4, ICL2, and ECL2 213 (Fig. 4 C & F)." are both hard to judge. Inspecting Figures 4B and C does not immediately show these trends, and importantly, a solid statistical test is missing here. In Figures 4E and F the locations of the different loops and TMs are not indicated on the structure, making these statements hard to judge.

      We apologize for this oversight and thank the reviewer for pointing this out. We will include additional statistical tests to reinforce these conclusions in the revised version of the manuscript.

      1. The following statement lacks a statistical test: "Notably, these 98 variants are enriched with TMD variants (65% TMD) relative to the overall set of 251 variants (45% TMD)." Is this enrichment significant? Further in the same paragraph, the claim that "In contrast to the sparse epistasis that is generally observed between mutations within soluble proteins, these findings suggest a relatively large proportion of random mutations form epistatic interactions in the context of unstable mGnRHR variants". Needs to be backed by relevant data and statistics, or at least a reference.

      We will include additional statistical tests for this in the revised manuscript and will ensure the language we use is consistent with the strength of the indicated statistical enrichment.

    1. Author response:

      Reviewer #1 (Public review):

      This work regards the role of Aurora Kinase A (AurA) in trained immunity. The authors claim that AurA is essential to the induction of trained immunity. The paper starts with a series of experiments showing the effects of suppressing AurA on beta-glucan-trained immunity. This is followed by an account of how AurA inhibition changes the epigenetic and metabolic reprogramming that are characteristic of trained immunity. The authors then zoom in on specific metabolic and epigenetic processes (regulation of S-adenosylmethionine metabolism & histone methylation). Finally, an inhibitor of AurA is used to reduce beta-glucan's anti-tumour effects in a subcutaneous MC-38 model.

      Strengths:

      With the exception of my confusion around the methods used for relative gene expression measurements, the experimental methods are generally well-described. I appreciate the authors' broad approach to studying different key aspects of trained immunity (from comprehensive transcriptome/chromatin accessibility measurements to detailed mechanistic experiments). Approaching the hypothesis from many different angles inspires confidence in the results (although not completely - see weaknesses section). Furthermore, the large drug-screening panel is a valuable tool as these drugs are readily available for translational drug-repurposing research.

      We thank the reviewer for the positive and encouraging comments.

      Weaknesses:

      (1) The manuscript contains factual inaccuracies such as: (a) Intro: the claim that trained cells display a shift from OXPHOS to glycolysis based on the paper by Cheng et al. in 2014; this was later shown to be dependent on the dose of stimulation and actually both glycolysis and OXPHOS are generally upregulated in trained cells (pmid 32320649).

      We appreciate the reviewer for pointing out this inaccuracy, and we will revise our statement to ensure accurate and updated description. We are aware that trained immunity involves different metabolic pathways, including both glycolysis and oxidative phosphorylation[1, 2]. We also detected Oxygen Consumption Rate (OCR, as detailed in comment#8) but observed no increase of oxygen consumption in trained BMDMs while previous study reported decreased oxidative phosphorylation[3]. We will discuss the potential reasons underlying such different results.

      (b) Discussion: Trained immunity was first described as such in 2011, not decades ago.

      We are sorry for the inaccurate description, and we will correct the statement in our revised manuscript as “Despite the fact that the concept of “trained immunity” has been proposed since 2011, the mechanisms that regulate trained immunity are still not completely understood.”

      (2) The authors approach their hypothesis from different angles, which inspires a degree of confidence in the results. However, the statistical methods and reporting are underwhelming.

      (a) Graphs depict mean +/- SEM, whereas mean +/- SD is almost always more informative. (b) The use of 1-tailed tests is dubious in this scenario. Furthermore, in many experiments/figures the case could be made that the comparisons should be considered paired (the responses of cells from the same animal are inherently not independent due to their shared genetic background and, up until cell isolation, the same host factors like serum composition/microbiome/systemic inflammation etc). (c) It could be explained a little more clearly how multiple testing correction was done and why specific tests were chosen in each instance.

      Thank you for the suggestions and we will revise all data presented as mean ± SEM in the manuscript to mean ± SD, and provide a detailed description of how multiple comparisons were performed and explain the rationale behind the different comparison methods used. Previous studies have shown that knockdown of GNMT increases intracellular SAM level and knockdown of GNMT is commonly used as a method to upregulate SAM[4-6]. Thus we used 1-tailed test in Figure 3J.

      (d) Most experiments are done with n = 3, some experiments are done with n = 5. This is not a lot. While I don't think power analyses should be required for simple in vitro experiments, I would be wary of drawing conclusions based on n = 3. It is also not indicated if the data points were acquired in independent experiments. ATAC-seq/RNA-seq was, judging by the figures, done on only 2 mice per group. No power calculations were done for the in vivo tumor model.

      We are sorry for the confusion in our description in figure legends. As for in vitro studies, we performed at least three independent experiments (BMs isolated from different mice) but we only display technical replicates data from one experiment in our manuscript. As for seq data, we acknowledge the reviewer's concern regarding the small sample size (n=2) in our RNA-seq/ATAC-seq experiment. We consider the sequencing experiment mainly as an exploratory approach, and performed rigorous quality control and normalization of the sequencing data to ensure the reliability of our findings. While we understand that a larger sample size would be ideal for drawing more definitive conclusions, we believe that the current data offer valuable preliminary insights that can inform future studies with larger cohorts. As a complementary method, we conducted ChIP PCR for detecting active histone modification enrichment in Il6 and Tnf region to further verify the increased accessibility of trained immunity induced inflammatory genes and reliability of our conclusions despite the small sample size. We hope this clarifies our approach, and we would be happy to further acknowledge and discuss the limitations of the current study.

      For the in vivo experiment, we determined the sample size by referring to the animal numbers used for similar experiments in literatures. And according to a reported resource equation approach for calculating sample size in animal studies[7], n=5-7 is suitable for most of our mouse experiments. We will describe the details in the revised methods part.

      (e) Furthermore, the data spread in many experiments (particularly BMDM experiments) is extremely small. I wonder if these are true biological replicates, meaning each point represents BMDMs from a different animal? (disclaimer: I work with human materials where the spread is of course always much larger than in animal experiments, so I might be misjudging this.).

      We are sorry for the confusion in our description in figure legends. In vivo experiments represent individual mice as biological replicates, the exact values of n are reported in figure legends and each point represents data from a different animal (Figure 1I, and Figure 6). The in vitro cell assay was performed in triplicates, each experiment was independently replicated at least three times and points represents technical replicates.

      (3) Maybe the authors are reserving this for a separate paper, but it would be fantastic if the authors would report the outcomes of the entire drug screening instead of only a selected few. The field would benefit from this as it would save needless repeat experiments. The list of drugs contains several known inhibitors of training (e.g. mTOR inhibitors) so there must have been more 'hits' than the reported 8 Aurora inhibitors.

      Thank you for your suggestion and we will report the outcomes of the entire drug screening in the revised manuscript.

      (4) Relating to the drug screen and subsequent experiments: it is unclear to me in supplementary figure 1B which concentrations belong to secondary screens #1/#2 - the methods mention 5 µM for the primary screen and "0.2 and 1 µM" for secondary screens, is it in this order or in order of descending concentration?

      Thank you for your comments and we are sorry for unclear labelled results in supplementary 1B. We performed secondary drug screen at two concentrations, and drug concentrations corresponding to secondary screen#1 and #2 are 0.2, 1 μM respectively. That is to say, it is just in this order, not in an order of descending concentration.

      (a) It is unclear if the drug screen was performed with technical replicates or not - the supplementary figure 1B suggests no replicates and quite a large spread (in some cases lower concentration works better?)

      Thank you for your question. The drug screen was performed without technical replicates. Actually, we observed s a lower concentration works better in some cases. This might be due to the fact that the drug's effect correlates positively with its concentration only within a specific range (as seen in comment#4).

      (5) The methods for (presumably) qPCR for measuring gene expression in Figure 1C are missing. Which reference gene was used and is this a suitably stable gene?

      We are sorry for the omission for the qPCR method. The mRNA expression of Il6 and Tnf in trained BMDMs was normalized to untrained BMDMs and β-actin served as a reference gene. And we will describe in detail in our revised manuscript.

      (6) From the complete unedited blot image of Figure 1D it appears that the p-Aurora and total Aurora are not from the same gel (discordant number of lanes and positioning). This could be alright if there are no/only slight technical errors, but I find it misleading as it is presented as if the actin (loading control to account for aforementioned technical errors!) counts for the entire figure.

      Thanks for this comment. In the original data, p-Aurora and total Aurora were from different gels. In this experiment the membrane stripping/reprobing after p-Aurora antibody did now work well, so we couldn’t get all results from one gel, and we had to run another gel using the same samples to blot with anti-aurora antibody. Yes we should have provided separated actin blots as loading controls for this experiment. We will repeat the experiment and provide original data of three biological replicates to confirm the experiment result.

      Figure 2: This figure highlights results that are by far not the strongest ones - I think the 'top hits' deserve some more glory. A small explanation on why the highlighted results were selected would have been fitting.

      We appreciate the valuable suggestion. We will make a discussion in our revised manuscript.

      (7) Figure 3 incl supplement: the carbon tracing experiments show more glucose-carbon going into TCA cycle (suggesting upregulated oxidative metabolism), but no mito stress test was performed on the seahorse.

      We appreciate this question raised by the reviewer. We previously performed seahorse XF analyze to measure mito stress in β-glucan trained BMDMs in combination with alisertib (data not shown in our submitted manuscript). The results showed no increase in oxidative phosphorylation under β-glucan stimulation.

      Author response image 1.

      (8) Inconsistent use of an 'alisertib-alone' control in addition to 'medium', 'b-glucan', 'b-glucan + alisertib'. This control would be of great added value in many cases, in my opinion.

      Thank you for your comment. We appreciate that including “alisertib-alone” group throughout all the experiments may add more value to the findings. We set the aim of the current study to investigate the role of Aurora kinase A in trained immunity. Therefore, in most settings, we did not focus on the role of aurora kinase A without β-glucan stimulation. Initially, we showed in Figure 1B and 1C that alisertib alone in a concentration lower than 1μM (included) does not affect the response to secondary stimulus. In a previous report, the authors showed that Aurora A inhibitor alone did not affect trained immunity[8]. Thus, we did not include this control group in all of the experiments.

      (9) Figure 4A: looking at the unedited blot images, the blot for H3K36me3 appears in its original orientation, whereas other images appear horizontally mirrored. Please note, I don't think there is any malicious intent but this is quite sloppy and the authors should explain why/how this happened (are they different gels and the loading sequence was reversed?)

      Thank you for pointing out this error. After checking the original data, we found that we indeed misassembled the orientation of several blots. We went through the assembling process and figured out that some orientations were assembled according to the loading sequences but not saved, so that the orientations in Figure 4A were not consistent with the unedited blot image. We are sorry for the careless mistake, and we will double check to make sure all the blots are correctly assembled in the revised manuscript.

      (10) For many figures, for example prominently figure 5, the text describes 'beta-glucan training' whereas the figures actually depict acute stimulation with beta-glucan. While this is partially a semantic issue (technically, the stimulation is 'the training-phase' of the experiment), this could confuse the reader.

      Thanks for the reviewer’s suggestion and we will reorganize our language to ensure clarity and avoid any inconsistencies that might lead to misunderstanding.

      (11) Figure 6: Cytokines, especially IL-6 and IL-1β, can be excreted by tumour cells and have pro-tumoral functions. This is not likely in the context of the other results in this case, but since there is flow cytometry data from the tumour material it would have been nice to see also intracellular cytokine staining to pinpoint the source of these cytokines.

      Thanks for the reviewer’s suggestion. To address potential concerns raised by the reviewers, we will perform intracellular cytokines staining in tumor experiments with mice trained with β-glucan or in combination with alisertib followed MC38 inoculation.

      Reviewer #2 (Public review):

      Summary:

      This manuscript investigates the inhibition of Aurora A and its impact on β-glucan-induced trained immunity via the FOXO3/GNMT pathway. The study demonstrates that inhibition of Aurora A leads to overconsumption of SAM, which subsequently impairs the epigenetic reprogramming of H3K4me3 and H3K36me3, effectively abolishing the training effect.

      Strengths:

      The authors identify the role of Aurora A through small molecule screening and validation using a variety of molecular and biochemical approaches. Overall, the findings are interesting and shed light on the previously underexplored role of Aurora A in the induction of β-glucan-driven epigenetic change.

      We thank the reviewer for the positive and encouraging comments.

      Weaknesses:

      Given the established role of histone methylations, such as H3K4me3, in trained immunity, it is not surprising that depletion of the methyl donor SAM impairs the training response. Nonetheless, this study provides solid evidence supporting the role of Aurora A in β-glucan-induced trained immunity in murine macrophages. The part of in vivo trained immunity antitumor effect is insufficient to support the final claim as using Alisertib could inhibits Aurora A other cell types other than myeloid cells.

      We appreciate the question raised by the reviewer. Though SAM generally acts as methyl donor, whether the epigenetic reprogram in trained immunity is directly linked to SAM metabolism is not known. In our study, we provided evidence suggesting the necessity of SAM maintenance in supporting trained immunity. As for in vivo tumor model, tumor cells were subcutaneously inoculated 24 h after oral administration of alisertib. Previous studies showed alisertib administered orally had a half-life of 10 h and 90% concentration reduction in serum after 24 h [9, 10]. Therefore, we suppose that tumor cells are more susceptible to long-term effects of drugs on the immune system rather than directly affected by alisertib. To further address the reviewer’s concern, we will perform bone marrow transplantation (trained mice as donor and naïve mice as recipient) to clarify the mechanistic contribution of trained immunity versus off-target effects.

      Cited references

      (1) Ferreira, A.V., et al., Metabolic Regulation in the Induction of Trained Immunity. Semin Immunopathol, 2024. 46(3-4): p. 7.

      (2) Keating, S.T., et al., Rewiring of glucose metabolism defines trained immunity induced by oxidized low-density lipoprotein. J Mol Med (Berl), 2020. 98(6): p. 819-831.

      (3) Li, X., et al., Maladaptive innate immune training of myelopoiesis links inflammatory comorbidities. Cell, 2022. 185(10): p. 1709-1727.e18.

      (4) Luka, Z., S.H. Mudd, and C. Wagner, Glycine N-methyltransferase and regulation of S-adenosylmethionine levels. J Biol Chem, 2009. 284(34): p. 22507-11.

      (5) Hughey, C.C., et al., Glycine N-methyltransferase deletion in mice diverts carbon flux from gluconeogenesis to pathways that utilize excess methionine cycle intermediates. J Biol Chem, 2018. 293(30): p. 11944-11954.

      (6) Simile, M.M., et al., Nuclear localization dictates hepatocarcinogenesis suppression by glycine N-methyltransferase. Transl Oncol, 2022. 15(1): p. 101239.

      (7) Arifin, W.N. and W.M. Zahiruddin, Sample Size Calculation in Animal Studies Using Resource Equation Approach. Malays J Med Sci, 2017. 24(5): p. 101-105.

      (8) Benjaskulluecha, S., et al., Screening of compounds to identify novel epigenetic regulatory factors that affect innate immune memory in macrophages. Sci Rep, 2022. 12(1): p. 1912.

      (9) Yang, J.J., et al., Preclinical drug metabolism and pharmacokinetics, and prediction of human pharmacokinetics and efficacious dose of the investigational Aurora A kinase inhibitor alisertib (MLN8237). Drug Metab Lett, 2014. 7(2): p. 96-104.

      (10) Palani, S., et al., Preclinical pharmacokinetic/pharmacodynamic/efficacy relationships for alisertib, an investigational small-molecule inhibitor of Aurora A kinase. Cancer Chemother Pharmacol, 2013. 72(6): p. 1255-64.

    1. Author response:

      Point-by-point description of the revisions

      Reviewer #1 (Evidence, reproducibility, and clarity):

      The work by Pinon et al describes the generation of a microvascular model to study Neisseria meningitidis interactions with blood vessels. The model uses a novel and relatively high throughput fabrication method that allows full control over the geometry of the vessels. The model is well characterized. The authors then study different aspects of Neisseriaendothelial interactions and benchmark the bacterial infection model against the best disease model available, a human skin xenograft mouse model, which is one of the great strengths of the paper. The authors show that Neisseria binds to the 3D model in a similar geometry that in the animal xenograft model, induces an increase in permeability short after bacterial perfusion, and induces endothelial cytoskeleton rearrangements. Finally, the authors show neutrophil recruitment to bacterial microcolonies and phagocytosis of Neisseria. The article is overall well written, and it is a great advancement in the bioengineering and sepsis infection field, and I only have a few major comments and some minor.

      Major comments:

      Infection-on-chip. I would recommend the authors to change the terminology of "infection on chip" to better reflect their work. The term is vague and it decreases novelty, as there are multiple infection on chips models that recapitulate other infections (recently reviewed in https://doi.org/10.1038/s41564-024-01645-6) including Ebola, SARS-CoV-2, Plasmodium and Candida. Maybe the term "sepsis on chip" would be more specific and exemplify better the work and novelty. Also, I would suggest that the authors carefully take a look at the text and consider when they use VoC or to current term IoC, as of now sometimes they are used interchangeably, with VoC being used occasionally in bacteria perfused experiments.

      We thank Reviewer #1 for this suggestion. Indeed, we have chosen to replace the term "Infection-on-Chip" by "infected Vessel-on-chip" to avoid any confusion in the title and the text. Also, we have removed all the terms "IoC" which referred to "Infection-on-Chip" and replaced with "VoC" for "Vessel-on-Chip". We think these terms will improve the clarity of the main text.

      Author response image 1.

      F-actin (red) and ezrin (yellow) staining after 3h of infection with N. meningitidis (green) in 2D (top) and 3D (bottom) vessel-on-chip models.

      Fig 3 and Supplementary 3: Permeability. The authors suggest that early 3h infection with Neisseria do not show increase in vascular permeability in the animal model, contrary to their findings in the 3D in vitro model. However, they show a non-significant increase in permeability of 70 KDa Dextran in the animal xenograft early infection. This seems to point that if the experiment would have been done with a lower molecular weight tracer, significant increases in permeability could have been detected. I would suggest to do this experiment that could capture early events in vascular disruption.

      Comparing permeability under healthy and infected conditions using Dextran smaller than 70 kDa is challenging. Previous research (1) has shown that molecules below 70 kDa already diffuse freely in healthy tissue. Given this high baseline diffusion, we believe that no significant difference would be observed before and after N. meningitidis infection and these experiments were not carried out. As discussed in the manuscript, bacteria induced permeability in mouse occurs at later time points, 16h post infection as shown previoulsy (2). As discussed in the manuscript, this difference between the xenograft model and the chip likely reflect the absence in the chip of various cell types present in the tissue parenchyma.

      The authors show the formation of actin of a honeycomb structure beneath the bacterial microcolonies. This only occurred in 65% of the microcolonies. Is this result similar to in vitro 2D endothelial cultures in static and under flow? Also, the group has shown in the past positive staining of other cytoskeletal proteins, such as ezrin in the ERM complex. Does this also occur in the 3D system?

      We thank the Reviewer #1 for this suggestion.

      • According to this recommendation, we imaged monolayers of endothelial cells in the flat regions of the chip (the two lateral channels) using the same microscopy conditions (i.e., Obj. 40X N.A. 1.05) that have been used to detect honeycomb structures in the 3D vessels in vitro. We showed that more than 56% of infected cells present these honeycomb structures in 2D, which is 13% less than in 3D, and is not significant due to the distributions of both populations. Thus, we conclude that under both in vitro conditions, 2D and 3D, the amount of infected cells exhibiting cortical plaques is similar. We have added the graph and the confocal images in Figure S4B and lines 418-419 of the revised manuscript.

      • We recently performed staining of ezrin in the chip and imaged both the 3D and 2D regions. Although ezrin staining was visible in 3D (Fig. 1 of this response), it was not as obvious as other markers under these infected conditions and we did not include it in the main text. Interpretation of this result is not straight forward as for instance the substrate of the cells is different and it would require further studies on the behaviour of ERM proteins in these different contexts.

      One of the most novel things of the manuscript is the use of a relatively quick photoablation system. I would suggest that the authors add a more extensive description of the protocol in methods. Could this technique be applied in other laboratories? If this is a major limitation, it should be listed in the discussion.

      Following the Reviewer’s comment, we introduced more detailed explanations regarding the photoablation:

      • L157-163 (Results): "Briefly, the chosen design is digitalized into a list of positions to ablate. A pulsed UV-LASER beam is injected into the microscope and shaped to cover the back aperture of the objective. The laser is then focused on each position that needs ablation. After introducing endothelial cells (HUVEC) in the carved regions,…"

      • L512-516 (Discussion): "The speed capabilities drastically improve with the pulsing repetition rate. Given that our laser source emits pulses at 10kHz, as compared to other photoablation lasers with repetitions around 100 Hz, our solution could potentially gain a factor of 100."

      • L1082-1087 (Materials and Methods): "…, and imported in a python code. The control of the various elements is embedded and checked for this specific set of hardware. The code is available upon request." Adding these three paragraphs gives more details on how photoablation works thus improving the manuscript.

      Minor comments:

      Supplementary Fig 2. The reference to subpanels H and I is swapped.

      The references to subpanels H and I have been correctly swapped back in the reviewed version.

      Line 203: I would suggest to delete this sentence. Although a strength of the submitted paper is the direct comparison of the VoC model with the animal model to better replicate Neisseria infection, a direct comparison with animal permeability is not needed in all vascular engineering papers, as vascular permeability measurements in animals have been well established in the past.

      The sentence "While previously developed VoC platforms aimed at replicating physiological permeability properties, they often lack direct comparisons with in vivo values." has been removed from the revised text.

      Fig 3: Bacteria binding experiments. I would suggest the addition of more methodological information in the main results text to guarantee a good interpretation of the experiment. First, it would be better that wall shear stress rather than flow rate is described in the main text, as flow rate is dependent on the geometry of the vessel being used. Second, how long was the perfusion of Neisseria in the binding experiment performed to quantify colony doubling or elongation? As per figure 1C, I would guess than 100 min, but it would be better if this information is directly given to the readers.

      We thank Reviewer #1 for these two suggestions that will improve the text clarity (e.g., L316). (i) Indeed, we have changed the flow rate in terms of shear stress. (ii) Also, we have normalized the quantification of the colony doubling time according to the first time-point where a single bacteria is attached to the vessel wall. Thus, early adhesion bacteria will be defined by a longer curve while late adhesion bacteria by a shorter curve. In total, the experiment lasted for 3 hours (modifications appear in L318 and L321-326).

      Fig 4: The honeycomb structure is not visible in the 3D rendering of panel D. I would recommend to show the actin staining in the absence of Neisseria staining as well.

      According to this suggestion, a zoom of the 3D rendering of the cortical plaque without colony had been added to the figure 4 of the revised manuscript.

      Line 421: E-selectin is referred as CD62E in this sentence. I would suggest to use the same terminology everywhere.

      We have replaced the "CD62E" term with "E-selectin" to improve clarity.

      Line 508: "This difference is most likely associated with the presence of other cell types in the in vivo tissues and the onset of intravascular coagulation". Do the authors refer to the presence of perivascular cells, pericytes or fibroblasts? If so, it could be good to mention them, as well as those future iterations of the model could include the presence of these cell types.

      By "other cell types", we refer to pericytes (3), fibroblasts (4), and perivascular macrophages (5), which surround endothelial cells and contribute to vessel stability. The main text was modified to include this information (Lines 548 and 555-570) and their potential roles during infection disussed.

      Discussion: The discussion covers very well the advantages of the model over in vitro 2D endothelial models and the animal xenograft but fails to include limitations. This would include the choice of HUVEC cells, an umbilical vein cell line to study microcirculation, the lack of perivascular cells or limitations on the fabrication technique regarding application in other labs (if any).

      We thank Reviewer #1 for this suggestion. Indeed, our manuscript may lack explaining limitations, and adding them to the text will help improve it:

      • The perspectives of our model include introducing perivascular cells surrounding the vessel and fibroblasts into the collagen gel as discussed previously and added in the discussion part (L555-570).

      • Our choice for HUVEC cells focused on recapitulating the characteristics of venules that respect key features such as the overexpression of CD62E and adhesion of neutrophils during inflammation. Using microvascular endothelial cells originating from different tissues would be very interesting. This possibility is now mentioned in the discussion lines 567-568.

      • Photoablation is a homemade fabrication technique that can be implemented in any lab harboring an epifluorescence microscope. This method has been more detailed in the revised manuscript (L1085-1087).

      Line 576: The authors state that the model could be applied to other systemic infections but failed to mention that some infections have already been modelled in 3D bioengineered vascular models (examples found in https://doi.org/10.1038/s41564-024-01645-6). This includes a capillary photoablated vascular model to study malaria (DOI: 10.1126/sciadv.aay724).

      Thes two important references have been introduced in the main text (L84, 647, 648).

      Line 1213: Are the 6M neutrophil solution in 10ul under flow. Also, I would suggest to rewrite this sentence in the following line "After, the flow has been then added to the system at 0.7-1 µl/min."

      We now specified that neutrophils are circulated in the chip under flow conditions, lines 1321-1322.

      Significance

      The manuscript is comprehensive, complete and represents the first bioengineered model of sepsis. One of the major strengths is the carful characterization and benchmarking against the animal xenograft model. Its main limitations is the brief description of the photoablation methodology and more clarity is needed in the description of bacteria perfusion experiments, given their complexity. The manuscript will be of interest for the general infection community and to the tissue engineering community if more details on fabrication methods are included. My expertise is on infection bioengineered models.

      Reviewer #2 (Evidence, reproducibility, and clarity):

      Summary:

      The authors develop a Vessel-on-Chip model, which has geometrical and physical properties similar to the murine vessels used in the study of systemic infections. The vessel was created via highly controllable laser photoablation in a collagen matrix, subsequent seeding of human endothelial cells and flow perfusion to induce mechanical cues. This vessel could be infected with Neisseria meningitidis, as a model of systemic infection. In this model, microcolony formation and dynamics, and effects on the host were very similar to those described for the human skin xenograft mouse, which is the current gold standard for these studies, and were consistent with observations made in patients. The model could also recapitulate the neutrophil response upon N. meningitidis systemic infection.

      Major comments:

      I have no major comments. The claims and the conclusions are supported by the data, the methods are properly presented and the data is analyzed adequately. Furthermore, I would like to propose an optional experiment could improve the manuscript. In the discussion it is stated that the vascular geometry might contribute to bacterial colonization in areas of lower velocity. It would be interesting to recapitulate this experimentally. It is of course optional but it would be of great interest, since this is something that can only be proven in the organ-on-chip (where flow speed can be tuned) and not as much in animal models. Besides, it would increase impact, demonstrating the superiority of the chip in this area rather than proving to be equal to current models.

      We have conducted additional experiments on infection in different vascular geometries now added these results figure 3/S3 and lines 288-305. We compared sheared stress levels as determined by Comsol simulation and experimentally determined bacterial adhesion sites. In the conditions used, the range of shear generated by the tested geometries do not appear to change the efficiency of bacterial adhesion. These results are consistent with a previous study from our group which show that in this range of shear stresses the effect on adhesion is limited (6) . Furthermore, qualitative observations in the animal model indicate that bacteria do not have an obvious preference in terms of binding site.

      Minor comments:

      I have a series of suggestions which, in my opinion, would improve the discussion. They are further elaborated in the following section, in the context of the limitations.

      • How to recapitulate the vessels in the context of a specific organ or tissue? If the pathogen is often found in the luminal space of other organs after disseminating from the blood, how can this process be recapitulated with this mode, if at all?

      For reasons that are not fully understood, postmortem histological studies reveal bacteria only inside blood vessels but rarely if ever in the organ parenchyma. The presence of intravascular bacteria could nevertheless alter cells in the tissue parenchyma. The notable exception is the brain where bacteria exit the bacterial lumen to access the cerebrospinal fluid. The chip we describe is fully adapted to develop a blood brain barrier model and more specific organ environments. This implies the addition of more cell types in the hydrogel. A paragraph on this topic has been added (Lines 548 and 552-570).

      • Similarly, could other immune responses related to systemic infection be recapitulated? The authors could discuss the potential of including other immune cells that might be found in the interstitial space, for example.

      This important discussion point has been added to the manuscript (L623-636). As suggested by Reviewer #2, other immune cells respond to N. meningitis and can be explored using our model. For instance, macrophages and dendritic cells are activated upon N. meningitis infection, eliminate the bacteria through phagocytosis, produce pro-inflammatory cytokines and chemokines potentially activating lymphocytes (7). Such an immune response, yet complex, would be interesting to study in our model as skin-xenograft mice are deprived of B and T lymphocytes to ensure acceptance of human skin grafts.

      • A minor correction: in line 467 it should probably be "aspects" instead of "aspect", and the authors could consider rephrasing that sentence slightly for increased clarity.

      We have corrected the sentence with "we demonstrated that our VoC strongly replicates key aspects of the in vivo human skin xenograft mouse model, the gold standard for studying meningococcal disease under physiological conditions." in lines 499-503.

      Strengths and limitations

      The most important strength of this manuscript is the technology they developed to build this model, which is impressive and very innovative. The Vessel-on-Chip can be tuned to acquire complex shapes and, according to the authors, the process has been optimized to produce models very quickly. This is a great advancement compared with the technologies used to produce other equivalent models. This model proves to be equivalent to the most advanced model used to date, but allows to perform microscopy with higher resolution and ease, which can in turn allow more complex and precise image-based analysis. However, the authors do not seem to present any new mechanistic insights obtained using this model. All the findings obtained in the infection-on-chip demonstrate that the model is equivalent to the human skin xenograft mouse model, and can offer superior resolution for microscopy. However, the advantages of the model do not seem to be exploited to obtain more insights on the pathogenicity mechanisms of N. meningitidis, host-pathogen interactions or potential applications in the discovery of potential treatments. For example, experiments to elucidate the role of certain N. meningiditis genes on infection could enrich the manuscript and prove the superiority of the model. However, I understand these experiments are time-consuming and out of the scope of the current manuscript. In addition, the model lacks the multicellularity that characterizes other similar models. The authors mention that the pathogen can be found in the luminal space of several organs, however, this luminal space has not been recapitulated in the model. Even though this would be a new project, it would be interesting that the authors hypothesize about the possibilities of combining this model with other organ models. The inclusion of circulating neutrophils is a great asset; however it would also be interesting to hypothesize about how to recapitulate other immune responses related to systemic infection.

      We thank Reviewer #2 for his/her comment on the strengths and limitations of our work. The difficulty is that our study opens many futur research directions and applications and we hope that the work serves as the basis for many future studies but one can only address a limited set of experiments in a single manuscript.

      • Experiments investigating the role of N. meningitidis genes require significant optimization of the system. Multiplexing is a potential avenue for future development, which would allow the testing of many mutants. The fast photoablation approach is particularly amenable to such adaptation.

      • Cells and bacteria inside the chambers could be isolated and analyzed at the transcriptomic level or by flow cytometry. This would imply optimizing a protocol for collecting cells from the device via collagenase digestion, for instance. This type of approach would also benefit from multiplexing to enhance the number of cells.

      • As mentioned above, the revised manuscript discusses the multicellular capabilities of our model, including the integration of additional immune cells and potential connections to other organ systems. We believe that these approaches are feasible and valuable for studying various aspects of N. meningitidis infection.

      Advance

      The most important advance of this manuscript is technical: the development of a model that proves to be equivalent to the most complex model used to date to study meningococcal systemic infections. The human skin xenograft mouse model requires complex surgical techniques and has the practical and ethical limitations associated with the use of animals. However, the Infection-on-chip model is completely in vitro, can be produced quickly, and allows to precisely tune the vessel’s geometry and to perform higher resolution microscopy. Both models were comparable in terms of the hallmarks defining the disease, suggesting that the presented model can be an effective replacement of the animal use in this area.

      Other vessel-on-chip models can recapitulate an endothelial barrier in a tube-like morphology, but do not recapitulate other complex geometries, that are more physiologically relevant and could impact infection (in addition to other non-infectious diseases). However, in the manuscript it is not clear whether the different morphologies are necessary to study or recapitulate N. meningitidis infection, or if the tubular morphologies achieved in other similar models would suffice.

      Audience

      This manuscript might be of interest for a specialized audience focusing on the development of microphysiological models. The technology presented here can be of great interest to researchers whose main area of interest is the endothelium and the blood vessels, for example, researchers on the study of systemic infections, atherosclerosis, angiogenesis, etc. Thus, the tool presented (vessel-on-chip) can have great applications for a broad audience. However, even when the method might be faster and easier to use than other equivalent methods, it could still be difficult to implement in another laboratory, especially if it lacks expertise in bioengineering. Therefore, the method could be more of interest for laboratories with expertise in bioengineering looking to expand or optimize their toolbox. Alternatively, this paper present itself as an opportunity to begin collaborations, since the model could be used to test other pathogen or conditions.

      Field of expertise:

      Infection biology, organ-on-chip, fungal pathogens.

      I lack the expertise to evaluate the image-based analysis.

      References

      (1) Gyohei Egawa, Satoshi Nakamizo, Yohei Natsuaki, Hiromi Doi, Yoshiki Miyachi, and Kenji Kabashima. Intravital analysis of vascular permeability in mice using two-photon microscopy. Scientific Reports, 3(1):1932, Jun 2013. ISSN 2045-2322. doi: 10.1038/srep01932.

      (2) Valeria Manriquez, Pierre Nivoit, Tomas Urbina, Hebert Echenique-Rivera, Keira Melican, Marie-Paule Fernandez-Gerlinger, Patricia Flamant, Taliah Schmitt, Patrick Bruneval, Dorian Obino, and Guillaume Duménil. Colonization of dermal arterioles by neisseria meningitidis provides a safe haven from neutrophils. Nature Communications, 12(1):4547, Jul 2021. ISSN 2041-1723. doi: 10.1038/s41467-021-24797-z.

      (3) Mats Hellström, Holger Gerhardt, Mattias Kalén, Xuri Li, Ulf Eriksson, Hartwig Wolburg, and Christer Betsholtz. Lack of pericytes leads to endothelial hyperplasia and abnormal vascular morphogenesis. Journal of Cell Biology, 153(3):543–554, Apr 2001. ISSN 0021-9525. doi: 10.1083/jcb.153.3.543.

      (4) Arsheen M. Rajan, Roger C. Ma, Katrinka M. Kocha, Dan J. Zhang, and Peng Huang. Dual function of perivascular fibroblasts in vascular stabilization in zebrafish. PLOS Genetics, 16(10):1–31, 10 2020. doi: 10.1371/journal.pgen.1008800.

      (5) Huanhuan He, Julia J. Mack, Esra Güç, Carmen M. Warren, Mario Leonardo Squadrito, Witold W. Kilarski, Caroline Baer, Ryan D. Freshman, Austin I. McDonald, Safiyyah Ziyad, Melody A. Swartz, Michele De Palma, and M. Luisa Iruela-Arispe. Perivascular macrophages limit permeability. Arteriosclerosis, Thrombosis, and Vascular Biology, 36(11):2203–2212, 2016. doi: 10.1161/ATVBAHA. 116.307592.

      (6) Emilie Mairey, Auguste Genovesio, Emmanuel Donnadieu, Christine Bernard, Francis Jaubert, Elisabeth Pinard, Jacques Seylaz, Jean-Christophe Olivo-Marin, Xavier Nassif, and Guillaume Dumenil. Cerebral microcirculation shear stress levels determine Neisseria meningitidis attachment sites along the blood–brain barrier . Journal of Experimental Medicine, 203(8):1939–1950, 07 2006. ISSN 0022-1007. doi: 10.1084/jem.20060482.

      (7) Riya Joshi and Sunil D. Saroj. Survival and evasion of neisseria meningitidis from macrophages. Medicine in Microecology, 17:100087, 2023. ISSN 2590-0978. doi: https://doi.org/10.1016/j.medmic. 2023.100087.

    1. Author Response:

      Assessment note: “Whereas the results and interpretations are generally solid, the mechanistic aspect of the work and conclusions put forth rely heavily on in vitro studies performed in cultured L6 myocytes, which are highly glycolytic and generally not viewed as a good model for studying muscle metabolism and insulin action.”

      While we acknowledge that in vitro models may not fully recapitulate the complexity of in vivo systems, we believe that our use of L6 myotubes is appropriate for studying the mechanisms underlying muscle metabolism and insulin action. As mentioned below (reviewer 2, point 1), L6 myotubes possess many important characteristics relevant to our research, including high insulin sensitivity and a similar mitochondrial respiration sensitivity to primary muscle fibres. Furthermore, several studies have demonstrated the utility of L6 myotubes as a model for studying insulin sensitivity and metabolism, including our own previous work (PMID: 19805130, 31693893, 19915010).

      In addition, we have provided evidence of the similarities between L6 cells overexpressing SMPD5 and human muscle biopsies at protein levels and the reproducibility of the negative correlation between ceramide and Coenzyme Q observed in L6 cells in vivo, specifically in the skeletal muscle of mice in chow diet. These findings support the relevance of our in vitro results to in vivo muscle metabolism.

      Finally, we will supplement our findings by demonstrating a comparable relationship between ceramide and Coenzyme Q in mice exposed to a high-fat diet, to be shown in Supplementary Figure 4 H-I. Further animal experiments will be performed to validate our cell-line based conclusions. We hope that these additional results address the concerns raised by the reviewer and further support the relevance of our in vitro findings to in vivo muscle metabolism and insulin action.

      Points from reviewer 1:

      1. Although the authors' results suggest that higher mitochondrial ceramide levels suppress cellular insulin sensitivity, they rely solely on a partial inhibition (i.e., 30%) of insulin-stimulated GLUT4-HA translocation in L6 myocytes. It would be critical to examine how much the increased mitochondrial ceramide would inhibit insulin-induced glucose uptake in myocytes using radiolabel deoxy-glucose.

      Response: The primary impact of insulin is to facilitate the translocation of glucose transporter type 4 (GLUT4) to the cell surface, which effectively enhances the maximum rate of glucose uptake into cells. Therefore, assessing the quantity of GLUT4 present at the cell surface in non-permeabilized cells is widely regarded as the most reliable measure of insulin sensitivity (PMID: 36283703, 35594055, 34285405). Additionally, plasma membrane GLUT4 and glucose uptake are highly correlated. Whilst we have routinely measured glucose uptake with radiolabelled glucose in the past, we do not believe that evaluating glucose uptake provides a better assessment of insulin sensitivity than GLUT4.

      We will clarify the use of GLUT4 translocation in the Results section:

      “...For this reason, several in vitro models have been employed involving incubation of insulin sensitive cell types with lipids such as palmitate to mimic lipotoxicity in vivo. In this study we will use cell surface GLUT4-HA abundance as the main readout of insulin response...”

      1. Another important question to be addressed is whether glycogen synthesis is affected in myocytes under these experimental conditions. Results demonstrating reductions in insulin-stimulated glucose transport and glycogen synthesis in myocytes with dysfunctional mitochondria due to ceramide accumulation would further support the authors' claim.

      Response: We have carried out supplementary experiments to investigate glycogen synthesis in our insulin-resistant models. Our approach involved L6-myotubes overexpressing the mitochondrial-targeted construct ASAH1 (as described in Fig. 3). We then challenged them with palmitate and measured glycogen synthesis using 14C radiolabeled glucose. Our observations indicated that palmitate suppressed insulin-induced glycogen synthesis, which was effectively prevented by the overexpression of ASAH1 (N = 5, * p<0.05). These results provide additional evidence highlighting the role of dysfunctional mitochondria in muscle cell glucose metabolism.

      These data will be added to Supplementary Figure 4K and the results modified as follows:

      “Notably, mtASAH1 overexpression protected cells from palmitate-induced insulin resistance without affecting basal insulin sensitivity (Fig. 3E). Similar results were observed using insulin-induced glycogen synthesis as an ortholog technique for Glut4 translocation. These results provide additional evidence highlighting the role of dysfunctional mitochondria in muscle cell glucose metabolism (Sup. Fig. 5K). Importantly, mtASAH1 overexpression did not rescue insulin sensitivity in cells depleted…”

      We will add to the method section:

      “L6 myotubes overexpressing ASAH were grown and differentiated in 12-well plates, as described in the Cell lines section, and stimulated for 16 h with palmitate-BSA or EtOH-BSA, as detailed in the Induction of insulin resistance section.

      On day seven of differentiation, myotubes were serum starved in plain DMEM for 3 and a half hours. After incubation for 1 hour at 37C with 2 µCi/ml D-[U-14C]-glucose in the presence or absence of 100 nM insulin, glycogen synthesis assay was performed, as previously described (Zarini S. et al., J Lipid Res, 63(10): 100270, 2022).”

      1. In addition, it would be critical to assess whether the increased mitochondrial ceramide and consequent lowering of energy levels affect all exocytic pathways in L6 myoblasts or just the GLUT4 trafficking. Is the secretory pathway also disrupted under these conditions?

      Response: As the secretory pathway primarily involves the synthesis and transportation of soluble proteins that are secreted into the extracellular space, and given that the majority of cellular transmembrane proteins (excluding those of the mitochondria) use this pathway to arrive at their ultimate destination, we believe that the question posed by the reviewer is highly challenging and beyond the scope of our research. We will add this to the discussion:

      “...the abundance of mPTP associated proteins suggesting a role of this pore in ceramide induced insulin resistance (Sup. Fig. 6E). In addition, it is yet to be determined whether the trafficking defect is specific to Glut4 or if it affects the exocytic-secretory pathway more broadly…”

      Points from reviewer 2:

      1. The mechanistic aspect of the work and conclusions put forth rely heavily on studies performed in cultured myocytes, which are highly glycolytic and generally viewed as a poor model for studying muscle metabolism and insulin action. Nonetheless, the findings provide a strong rationale for moving this line of investigation into mouse gain/loss of function models.

      Response: The relative contribution of the anaerobic (glycolysis) and aerobic (mitochondria) contribution to the muscle metabolism can change in L6 depending on differentiation stage. For instance, Serrage et al (PMID30701682) demonstrated that L6-myotubes have a higher mitochondrial abundance and aerobic metabolism than L6-myoblasts. Others have used elegant transcriptomic analysis and metabolic characterisation comparing different skeletal muscle models for studying insulin sensitivity. For instance, Abdelmoez et al in 2020 (PMID31825657) reported that L6 myotubes exhibit greater insulin-stimulated glucose uptake and oxidative capacity compared with C2C12 and Human Mesenchymal Stem Cells (HMSC). Overall, L6 cells exhibit higher metabolic rates and primarily rely on aerobic metabolism, while C2C12 and HSMC cells rely on anaerobic glycolysis. It is worth noting that L6 myotubes are the cell line most closely related to adult human muscle when compared with other muscle cell lines (PMID31825657). Our presented results in Figure 6 H and I provide evidence for the similarities between L6 cells overexpressing SMPD5 and human muscle biopsies. Additionally, in Figure 3J-K, we demonstrate the reproducibility of the negative correlation between ceramide and Coenzyme Q observed in L6 cells in vivo, specifically in the skeletal muscle of mice in chow diet. Furthermore, we have supplemented these findings by demonstrating a comparable relationship in mice exposed to a high-fat diet, as shown in Supplementary Figure 4 H-I (refer to point 4). We will clarify these points in the Discussion:

      “In this study, we mainly utilised L6-myotubes, which share many important characteristics with primary muscle fibres relevant to our research. Both types of cells exhibit high sensitivity to insulin and respond similarly to maximal doses of insulin, with Glut4 translocation stimulated between 2 to 4 times over basal levels in response to 100 nM insulin (as shown in Fig. 1-4 and (46,47)). Additionally, mitochondrial respiration in L6-myotubes have a similar sensitivity to mitochondrial poisons, as observed in primary muscle fibres (as shown in Fig. 5 (48)). Finally, inhibiting ceramide production increases CoQ levels in both L6-myotubes and adult muscle tissue (as shown in Fig. 2-3). Therefore, L6-myotubes possess the necessary metabolic features to investigate the role of mitochondria in insulin resistance, and this relationship is likely applicable to primary muscle fibres”.

      We will also add additional data - in point 2 - from differentiated human myocytes that are consistent with our observations from the L6 models. Additional experiments are in progress to further extend these findings.

      1. One caveat of the approach taken is that exposure of cells to palmitate alone is not reflective of in vivo physiology. It would be interesting to know if similar effects on CoQ are observed when cells are exposed to a more physiological mixture of fatty acids that includes a high ratio of palmitate, but better mimics in vivo nutrition.

      Response: Palmitate is widely recognized as a trigger for insulin resistance and ceramide accumulation, which mimics the insulin resistance induced by a diet in rodents and humans. Previous studies have compared the effects of a lipid mixture versus palmitate on inducing insulin resistance in skeletal muscle, and have found that the strong disruption in insulin sensitivity caused by palmitate exposure was lessened with physiologic mixtures of fatty acids, even with a high proportion of saturated fatty acids. This was associated, in part, to the selective partitioning of fatty acids into neutral lipids (such as TAG) when muscle cells are exposed to physiologic lipid mixtures (Newsom et al PMID25793412). Hence, we think that using palmitate is a better strategy to study lipid-induced insulin resistance in vitro. We will add to results:

      “In vitro, palmitate conjugated with BSA is the preferred strategy for inducing insulin resistance, as lipid mixtures tend to partition into triacylglycerides (33)”.

      We are also performing additional in vivo experiments to add to the physiological relevance of the findings.

      1. While the utility of targeting SMPD5 to the mitochondria is appreciated, the results in Figure 5 suggest that this manoeuvre caused a rather severe form of mitochondrial dysfunction. This could be more representative of toxicity rather than pathophysiology. It would be helpful to know if these same effects are observed with other manipulations that lower CoQ to a similar degree. If not, the discrepancies should be discussed.

      Response: We conducted a staining procedure using the mitochondrial marker mitoDsRED to observe the effect of SMPD5 overexpression on cell toxicity. The resulting images, displayed in the figure below (Author response image 1), demonstrate that the overexpression of SMPD5 did not result in any significant changes in cell morphology or impact the differentiation potential of our myoblasts into myotubes.

      Author response image 1.

      In addition, we evaluated cell viability in HeLa cells following exposure to SACLAC (2 uM) to induce CoQ depletion (left panel). Specifically, we measured cell death by monitoring the uptake of Propidium iodide (PI) as shown in the right panel. Our results demonstrated that Saclac-induced CoQ depletion did not lead to cell death at the doses used for CoQ depletion (Author response image 2).

      Author response image 2.

      Therefore, we deemed it improbable that the observed effect is caused by cellular toxicity, but rather represents a pathological condition induced by elevated levels of ceramides. We will add to discussion:

      “...downregulation of the respirasome induced by ceramides may lead to CoQ depletion. Despite the significant impact of ceramide on mitochondrial respiration, we did not observe any indications of cell damage in any of the treatments, suggesting that our models are not explained by toxic/cell death events.”

      1. The conclusions could be strengthened by more extensive studies in mice to assess the interplay between mitochondrial ceramides, CoQ depletion and ETC/mitochondrial dysfunction in the context of a standard diet versus HF diet-induced insulin resistance. Does P053 affect mitochondrial ceramide, ETC protein abundance, mitochondrial function, and muscle insulin sensitivity in the predicted directions?

      Response: We would like to note that the metabolic characterization and assessment of ETC/mitochondrial function in these mice (both fed a high-fat (HF) and chow diet, with or without P053) were previously published (Turner N, PMID30131496). In addition to this, we have conducted targeted metabolomic and lipidomic analyses to investigate the impact of P053 on ceramide and CoQ levels in HF-fed mice. As illustrated in the figures below (Author response image 3), the administration of P053 led to a reduction in ceramide levels (left panel) and an increase in CoQ levels (right panel) in HF-fed mice, which is consistent with our in vitro findings.

      Author response image 3.

      We will add to results:

      “…similar effect was observed in mice exposed to a high fat diet for 5 wks (Supp. Fig. 4H-I further phenotypic and metabolic characterization of these animals can be found in (41))”

      We will further perform more in-vivo studies to corroborate these findings.

    1. Author response:

      eLife assessment

      This useful study reports how neuronal activity in the prefrontal cortex maps time intervals during which animals have to wait until reaching a reward and how this mapping is preserved across days. However, the evidence supporting the claims is incomplete as these sequential neuronal patterns do not necessarily represent time but instead may be correlated with stereotypical behavior and restraint from impulsive decision, which would require further controls (e.g. behavioral analysis) to clarify the main message. The study will be of interest to neuroscientists interested in decision making and motor control. 

      We thank the editors and reviewers for the constructive comments. In light of the questions mentioned by the reviewers, we plan to perform additional analyses in our revision, particularly aiming to address issues related to single-cell scalability, and effects of motivation and movement. We believe these additional data will greatly improve the rigor and clarity of our study. We are grateful for the review process of eLife.

      Public Reviews:

      Reviewer #1 (Public Review):

      Summary:

      This paper investigates the neural population activity patterns of the medial frontal cortex in rats performing a nose poking timing task using in vivo calcium imaging. The results showed neurons that were active at the beginning and end of the nose poking and neurons that formed sequential patterns of activation that covaried with the timed interval during nose poking on a trial-by-trial basis. The former were not stable across sessions, while the latter tended to remain stable over weeks. The analysis on incorrect trials suggests the shorter non-rewarded intervals were due to errors in the scaling of the sequential pattern of activity. 

      Strengths:

      This study measured stable signals using in vivo calcium imaging during experimental sessions that were separated by many days in animals performing a nose poking timing task. The correlation analysis on the activation profile to separate the cells in the three groups was effective and the functional dissociation between beginning and end, and duration cells was revealing. The analysis on the stability of decoding of both the nose poking state and poking time was very informative. Hence, this study dissected a neural population that formed sequential patterns of activation that encoded timed intervals. 

      We thank the reviewer for the positive comments.

      Weaknesses: 

      It is not clear whether animals had enough simultaneously recorded cells to perform the analyzes of Figures 2-4. In fact, rat 3 had 18 responsive neurons which probably is not enough to get robust neural sequences for the trial-by-trial analysis and the correct and incorrect trial analysis. 

      We thank the reviewer for the comment. We would like to mention that the 18 cells plotted in Supplementary figure 1 were only from the duration cell category. To improve the clarity of our results, we are going to provide information regarding the number of cells from each rat in our revision. In general, we imaged more than 50 cells from each rat. We would also like to point to the data from individual trials in Supplementary figure 1B showing robust sequentiality.

      In addition, the analysis of behavioral errors could be improved. The analysis in Figure 4A could be replaced by a detailed analysis on the speed, and the geometry of neural population trajectories for correct and incorrect trials.

      We thank the reviewer for the suggestions. We are going to conduct the analysis as the reviewer recommended. We agree with the reviewer that better presentation of the neural activity will be helpful for the readers.

      In the case of Figure 4G is not clear why the density of errors formed two clusters instead of having a linear relation with the produce duration. I would be recommendable to compute the scaling factor on neuronal population trajectories and single cell activity or the computation of the center of mass to test the type III errors. 

      We would like to mention that the prediction errors plotted in this graph were calculated from two types of trials. The correct trials tended to show positive time estimation errors while the incorrect trials showed negative time estimation errors. We believe that the polarity switch between these two types suggested a possible use of this neural mechanism to time the action of the rats.

      In addition, we are going to perform the analysis suggested by the reviewer in our revision. We agree that different ways of analyzing the data would provide better characterization of the scaling effect.

      Due to the slow time resolution of calcium imaging, it is difficult to perform robust analysis on ramping activity. Therefore, I recommend downplaying the conclusion that: "Together, our data suggest that sequential activity might be a more relevant coding regime than the ramping activity in representing time under physiological conditions." 

      We agree with the reviewer and we have mentioned this caveat in our original manuscript. We are going to rephrase the sentence as the reviewer suggested during our revision.

      Reviewer #2 (Public Review):

      In this manuscript, Li and collaborators set out to investigate the neuronal mechanisms underlying "subjective time estimation" in rats. For this purpose, they conducted calcium imaging in the prefrontal cortex of water-restricted rats that were required to perform an action (nosepoking) for a short duration to obtain drops of water. The authors provided evidence that animals progressively improved in performing their task. They subsequently analyzed the calcium imaging activity of neurons and identify start, duration, and stop cells associated with the nose poke. Specifically, they focused on duration cells and demonstrated that these cells served as a good proxy for timing on a trial-by-trial basis, scaling their pattern of actvity in accordance with changes in behavioral performance. In summary, as stated in the title, the authors claim to provide mechanistic insights into subjective time estimation in rats, a function they deem important for various cognitive conditions. 

      This study aligns with a wide range of studies in system neuroscience that presume that rodents solve timing tasks through an explicit internal estimation of duration, underpinned by neuronal representations of time. Within this framework, the authors performed complex and challenging experiments, along with advanced data analysis, which undoubtedly merits acknowledgement. However, the question of time perception is a challenging one, and caution should be exercised when applying abstract ideas derived from human cognition to animals. Studying so-called time perception in rats has significant shortcomings because, whether acknowledged or not, rats do not passively estimate time in their heads. They are constantly in motion. Moreover, rats do not perform the task for the sake of estimating time but to obtain their rewards are they water restricted. Their behavior will therefore reflects their motivation and urgency to obtain rewards. Unfortunately, it appears that the authors are not aware of these shortcomings. These alternative processes (motivation, sensorimotor dynamics) that occur during task performance are likely to influence neuronal activity. Consequently, my review will be rather critical. It is not however intended to be dismissive. I acknowledge that the authors may have been influenced by numerous published studies that already draw similar conclusions. Unfortunately, all the data presented in this study can be explained without invoking the concept of time estimation. Therefore, I hope the authors will find my comments constructive and understand that as scientists, we cannot ignore alternative interpretations, even if they conflict with our a priori philosophical stance (e.g., duration can be explicitly estimated by reading neuronal representation of time) and anthropomorphic assumptions (e.g., rats estimate time as humans do). While space is limited in a review, if the authors are interested, they can refer to a lengthy review I recently published on this topic, which demonstrates that my criticism is supported by a wide range of timing experiments across species (Robbe, 2023). In addition to this major conceptual issue that cast doubt on most of the conclusions of the study, there are also several major statistical issues. 

      Main Concerns 

      (1) The authors used a task in which rats must poke for a minimal amount of time (300 ms and then 1500 ms) to be able to obtain a drop of water delivered a few centimeters right below the nosepoke. They claim that their task is a time estimation task. However, they forget that they work with thirsty rats that are eager to get water sooner than later (there is a reason why they start by a short duration!). This task is mainly probing the animals ability to wait (that is impulse control) rather than time estimation per se. Second, the task does not require to estimate precisely time because there appear to be no penalties when the nosepokes are too short or when they exceed. So it will be unclear if the variation in nosepoke reflects motivational changes rather than time estimation changes. The fact that this behavioral task is a poor assay for time estimation and rather reflects impulse control is shown by the tendency of animals to perform nose-pokes that are too short, the very slow improvement in their performance (Figure 1, with most of the mice making short responses), and the huge variability. Not only do the behavioral data not support the claim of the authors in terms of what the animals are actually doing (estimating time), but this also completely annhilates the interpretation of the Ca++ imaging data, which can be explained by motivational factors (changes in neuronal activity occurring while the animals nose poke may reflect a growing sens of urgency to check if water is available). 

      We would like to respond to the reviewer’s comments 1, 2 and 4 together since they all focus on the same issue. We thank the reviewer for the very thoughtful comments and for sharing his detailed reasoning from a recently published review (Robbe, 2023). A lot of the discussion goes beyond the scope of this study and we agree that whether there is an explicit representation of time (an internal clock) in the brain is a difficult question to answer, particularly by using animal behaviors. In fact, even with fully conscious humans and elaborated task design, we think it is still questionable to clearly dissociate the neural substrate of “timing” from “motor”. In the end, it may as well be that as the reviewer cited from Bergson’s article, the experience of time cannot be measured.

      Studying the neural representation of any internal state may suffer from the same ambiguity. With all due respect, however, we would like to limit our response in the scope of our results. According to the reviewer, two alternative interpretations of the task-related sequential activity exist: 1, duration cells may represent fidgeting or orofacial movements and 2, duration cells may represent motivation or motion plan of the rats. To test the first alternative interpretation, we will perform a more comprehensive analysis of the behavior data at all the limbs and visible body parts of the rat during nose poke and analyze its periodicity among different trials, although the orofacial movements may not be visible to us.

      Regarding the second alternative interpretation, we think our data in the original Figure 4G argues against it. In this graph, we plotted the decoding error of time using the duration cells’ activity against the actual duration of the trials. If the sequential activity of durations cells only represents motivation, then the errors should distribute evenly across different trial times, or linearly modulated by trial durations. The unimodal distribution we observed (Figure 4G and see Author response image 1 below for a re-plot without signs) suggests that the scaling factor of the sequential activity represents information related to time. And the fact that this unimodal distribution centered at the time threshold of the task provides strong evidence for the active use of scaling factor for time estimation. In order to further test the relationship to motivation, we will measure the time interval between exiting nose poke to the start of licking water reward as an independent measurement of motivation for each trial. We will analyze and report whether this measurement correlates with the nose poking durations in our data in the revision.

      Author response image 1.

      Furthermore, whether the scaling sequential activity we report represents behavioral timing or true time estimation, the reviewer would agree that these activities correlate with the animal’s nose poking durations, and a previous study has showed that PFC silencing led to disruption of the mouse’s timing behavior (PMID: 24367075). The main surprising finding of the paper is that these duration cells are different from the start and end cells in terms of their coding stability. Thus, future studies dissecting the anatomical microcircuit of these duration cells may provide further clue regarding whether they receive inputs from thirst or reward-related brain regions. This may help partially resolve the “time” vs. “motor” debate the reviewer mentioned.

      (2) A second issue is that the authors seem to assume that rats are perfectly immobile and perform like some kind of robots that would initiate nose pokes, maintain them, and remove them in a very discretized manner. However, in this kind of task, rats are constantly moving from the reward magazine to the nose poke. They also move while nose-poking (either their body or their mouth), and when they come out of the nose poke, they immediately move toward the reward spout. Thus, there is a continuous stream of movements, including fidgeting, that will covary with timing. Numerous studies have shown that sensorimotor dynamics influence neural activity, even in the prefrontal cortex. Therefore, the authors cannot rule out that what the records reflect are movements (and the scaling of movement) rather than underlying processes of time estimation (some kind of timer). Concretely, start cells could represent the ending of the movement going from the water spout to the nosepoke, and end cells could be neurons that initiate (if one can really isolate any initiation, which I doubt) the movement from the nosepoke to the water spout. Duration cells could reflect fidgeting or orofacial movements combined with an increasing urgency to leave the nose pokes.

      (3)The statistics should be rethought for both the behavioral and neuronal data. They should be conducted separately for all the rats, as there is likely interindividual variability in the impulsivity of the animals.

      We thank the reviewer for the comment, yet we are not quite sure what specifically was asked by the reviewer. There is undoubtedly variance among individual animals. One of the core reasons for statistical comparison is to compare the group difference with the variance due to sampling. It appears that the reviewer would like to require we conduct our analysis using each rat individually. We will conduct and report analysis with individual rat in Figure 1C, Figure 2C, G, K, Figure 4F in our revised manuscript.

      (4) The fact that neuronal activity reflects an integration of movement and motivational factors rather than some abstract timing appears to be well compatible with the analysis conducted on the error trials (Figure 4), considering that the sensorimotor and motivational dynamics will rescale with the durations of the nose poke. 

      (5) The authors should mention upfront in the main text (result section) the temporal resolution allowed by their Ca+ probe and discuss whether it is fast enough in regard of behavioral dynamics occurring in the task. 

      We thank the reviewer for the suggestion. We have originally mentioned the caveat of calcium imaging in the interpretation of our results. We will incorporate more texts for this purpose during our revision. In terms of behavioral dynamics (start and end of nose poke in this case), we think calcium imaging could provide sufficient kinetics. However, the more refined dynamics related to the reproducibility of the sequential activity or the precise representation of individual cells on the scaled duration may be benefited from improved time resolution.

      Recommendations for the authors:

      Reviewer #1 (Recommendations For The Authors):

      (1) Please refer explicitly to the three types of cells in the abstract. 

      We will modify the abstract as suggested during revision.

      (2) Please refer to the work of Betancourt et al., 2023 Cell Reports, where a trial-by-trail analysis on the correlation between neural trajectory dynamics in MPC and timing behavior is reported. In that same paper the stability of neural sequences across task parameters is reported. 

      We will cite and discuss this study in our revised paper.

      (3) Please state the number of studied animals at the beginning of the results section. 

      We will provide this information as requested. The number of animals were also plotted in Figure 1D for each analysis.

      (4) Why do the middle and right panels of Figure 2E show duration cells. 

      Figure 2E was intended to show examples of duration cells’ activity. We included different examples of cells that peak at different points in the scaled duration. We believe these multiple examples would give the readers a straight forward impression of these cells’ activity patterns.

      (5) Which behavioral sessions of Figure 1B were analyzed further. 

      We will label the analyzed sessions in Figure 1B during our revision.

      (6) In Figure 3A-C please increase the time before the beginning of the trial in order to visualize properly the activation patterns of the start cells. 

      We thank the reviewer for the suggestion and will modify the figure accordingly during revision.

      (7) Please state what could be the behavioral and functional effect of the ablation of the cortical tissue on top of mPFC. 

      We thank the reviewer for the question. In our experience, mice with lens implanted in mPFC did not show observable different to mice without surgery regarding the acquisition of the task and the distribution of the nose-poke durations. Although we could not rule out the effect on other cognitive process, the mice appeared to be intact in the scope of our task. We will provide these behavior data during our revision.

    1. Author response:

      Public Reviews:

      Reviewer #1 (Public review):

      The authors present exciting new experimental data on the antigenic recognition of 78 H3N2 strains (from the beginning of the 2023 Northern Hemisphere season) against a set of 150 serum samples. The authors compare protection profiles of individual sera and find that the antigenic effect of amino acid substitutions at specific sites depends on the immune class of the sera, differentiating between children and adults. Person-to-person heterogeneity in the measured titers is strong, specifically in the group of children's sera. The authors find that the fraction of sera with low titers correlates with the inferred growth rate using maximum likelihood regression (MLR), a correlation that does not hold for pooled sera. The authors then measure the protection profile of the sera against historical vaccine strains and find that it can be explained by birth cohort for children. Finally, the authors present data comparing pre- and post- vaccination protection profiles for 39 (USA) and 8 (Australia) adults. The data shows a cohort-specific vaccination effect as measured by the average titer increase, and also a virus-specific vaccination effect for the historical vaccine strains. The generated data is shared by the authors and they also note that these methods can be applied to inform the bi-annual vaccine composition meetings, which could be highly valuable.

      Thanks for this nice summary of our paper.

      The following points could be addressed in a revision:

      (1) The authors conclude that much of the person-to-person and strain-to-strain variation seems idiosyncratic to individual sera rather than age groups. This point is not yet fully convincing. While the mean titer of an individual may be idiosyncratic to the individual sera, the strain-to-strain variation still reveals some patterns that are consistent across individuals (the authors note the effects of substitutions at sites 145 and 275/276). A more detailed analysis, removing the individual-specific mean titer, could still show shared patterns in groups of individuals that are not necessarily defined by the birth cohort.

      As the reviewer suggests, we normalized the titers for all sera to the geometric mean titer for each individual in the US-based pre-vaccination adults and children. This is only for the 2023-circulating viral strains. We then faceted these normalized titers by the same age groups we used in Figure 6, and the resulting plot is shown below. Although there are differences among virus strains (some are better neutralized than others), there are not obvious age group-specific patterns (eg, the trends in the two facets are similar). To us this suggests that at least for these relatively closely related recent H3N2 strains, the strain-to-strain variation does not obviously segregate by age group. Obviously, it is possible (we think likely) that there would be more obvious age-group specific trends if we looked at a larger swath of viral strains covering a longer time range (eg, over decades of influenza evolution). We plan to add the new plots shown below to a supplemental figure in the revised manuscript.

      Author response image 1.

      Author response image 2.

      (2) The authors show that the fraction of sera with a titer below 138 correlates strongly with the inferred growth rate using MLR. However, the authors also note that there exists a strong correlation between the MLR growth rate and the number of HA1 mutations. This analysis does not yet show that the titers provide substantially more information about the evolutionary success. The actual relation between the measured titers and fitness is certainly more subtle than suggested by the correlation plot in Figure 5. For example, the clades A/Massachusetts and A/Sydney both have a positive fitness at the beginning of 2023, but A/Massachusetts has substantially higher relative fitness than A/Sydney. The growth inference in Figure 5b does not appear to map that difference, and the antigenic data would give the opposite ranking. Similarly, the clades A/Massachusetts and A/Ontario have both positive relative fitness, as correctly identified by the antigenic ranking, but at quite different times (i.e., in different contexts of competing clades). Other clades, like A/St. Petersburg are assigned high growth and high escape but remain at low frequency throughout. Some mention of these effects not mapped by the analysis may be appropriate.

      Thanks for the nice summary of our findings in Figure 5. However, the reviewer is misreading the growth charts when they say that A/Massachusetts/18/2022 has a substantially higher fitness than A/Sydney/332/2023. Figure 5a shows the frequency trajectory of different variants over time. While A/Massachusetts/18/2022 reaches a higher frequency than A/Sydney/332/2023, the trajectory is similar and the reason that A/Massachusetts/18/2022 reached a higher max frequency is that it started at a higher frequency at the beginning of 2023. The MLR growth rate estimates differ from the maximum absolute frequency reached: instead, they reflect how rapidly each strain grows relative to others. In fact, A/Massachusetts/18/2022 and A/Sydney/332/2023 have similar growth rates, as shown in Supplementary Figure 6b. Similarly, A/Saint-Petersburg/RII-166/2023 starts at a low initial frequency but then grows even as A/Massachusetts/18/2022 and A/Sydney/332/2023 are declining, and so has a higher growth rate than both of those. In the revised manuscript, we will clarify how viral growth rates are estimated from frequency trajectories, and how growth rate differs from max frequency.

      (3) For the protection profile against the vaccine strains, the authors find for the adult cohort that the highest titer is always against the oldest vaccine strain tested, which is A/Texas/50/2012. However, the adult sera do not show an increase in titer towards older strains, but only a peak at A/Texas. Therefore, it could be that this is a virus-specific effect, rather than a property of the protection profile. Could the authors test with one older vaccine virus (A/Perth/16/2009?) whether this really can be a general property?

      We are interested in studying immune imprinting more thoroughly using sequencing-based neutralization assays, but we note that the adults in the cohorts we studied would have been imprinted with much older strains than included in this library. As this paper focuses on the relative fitness of contemporary strains with minor secondary points regarding imprinting, these experiments are beyond the scope of this study. We’re excited for future work (from our group or others) to explore these points by making a new virus library with strains from multiple decades of influenza evolution.

      Reviewer #2 (Public review):

      This is an excellent paper. The ability to measure the immune response to multiple viruses in parallel is a major advancement for the field, which will be relevant across pathogens (assuming the assay can be appropriately adapted). I only have a few comments, focused on maximising the information provided by the sera.

      Thanks very much!

      Firstly, one of the major findings is that there is wide heterogeneity in responses across individuals. However, we could expect that individuals' responses should be at least correlated across the viruses considered, especially when individuals are of a similar age. It would be interesting to quantify the correlation in responses as a function of the difference in ages between pairs of individuals. I am also left wondering what the potential drivers of the differences in responses are, with age being presumably key. It would be interesting to explore individual factors associated with responses to specific viruses (beyond simply comparing adults versus children).

      We’re excited by this idea! We plan to include these analyses in our revised pre-print.

      Relatedly, is the phylogenetic distance between pairs of viruses associated with similarity in responses?

      As above, we like this idea and our revised pre-print will include this analysis.

      Figure 5C is also a really interesting result. To be able to predict growth rates based on titers in the sera is fascinating. As touched upon in the discussion, I suspect it is really dependent on the representativeness of the sera of the population (so, e.g., if only elderly individuals provided sera, it would be a different result than if only children provided samples). It may be interesting to compare different hypotheses - so e.g., see if a population-weighted titer is even better correlated with fitness - so the contribution from each individual's titer is linked to a number of individuals of that age in the population. Alternatively, maybe only the titers in younger individuals are most relevant to fitness, etc.

      We’re very interested in these analyses, but suggest they may be better explored in subsequent works that could sample more children, teenagers and adults across age groups. Our sera set, as the reviewer suggests, may be under-powered to perform the proposed analysis on subsetted age groups of our larger age cohorts.

      In Figure 6, the authors lump together individuals within 10-year age categories - however, this is potentially throwing away the nuances of what is happening at individual ages, especially for the children, where the measured viruses cross different groups. I realise the numbers are small and the viruses only come from a small numbers of years, however, it may be preferable to order all the individuals by age (y-axis) and the viral responses in ascending order (x-axis) and plot the response as a heatmap. As currently plotted, it is difficult to compare across panels

      This is a good suggestion, and a revised pre-print will include heatmaps of the different cohorts, ordered by ages of individuals.

      Reviewer #3 (Public review):

      The authors use high-throughput neutralisation data to explore how different summary statistics for population immune responses relate to strain success, as measured by growth rate during the 2023 season. The question of how serological measurements relate to epidemic growth is an important one, and I thought the authors present a thoughtful analysis tackling this question, with some clear figures. In particular, they found that stratifying the population based on the magnitude of their antibody titres correlates more with strain growth than using measurements derived from pooled serum data. However, there are some areas where I thought the work could be more strongly motivated and linked together. In particular, how the vaccine responses in US and Australia in Figures 6-7 relate to the earlier analysis around growth rates, and what we would expect the relationship between growth rate and population immunity to be based on epidemic theory.

      Thank you for this nice summary. This reviewer also notes that the text related to figures 6 and 7 are more secondary to the main story presented in figures 3-5. The main motivation for including figures 6 and 7 were to demonstrate the wide-ranging applications of sequencing-based neutralization data, and this can certainly be clarified in minor text revisions.

    1. Author Response

      Public Reviews

      We thank both reviewers for taking the time and effort to think critically about our paper and point out areas where it can be improved. In this document, we do our best to clarify any misunderstandings with the hope that further consideration about the strengths and weaknesses of our approach will be possible. Our responses are in bold.

      Reviewer #1 (Public Review):

      Summary:

      In their manuscript, Schmidlin, Apodaca, et al try to answer fundamental questions about the evolution of new phenotypes and the trade-offs associated with this process. As a model, they use yeast resistance to two drugs, fluconazole and radicicol. They use barcoded libraries of isogenic yeasts to evolve thousands of strains in 12 different environments. They then measure the fitness of evolved strains in all environments and use these measurements to examine patterns in fitness trade-offs. They identify only six major clusters corresponding to different trade-off profiles, suggesting the vast genotypic landscape of evolved mutants translates to a highly constrained phenotypic space. They sequence over a hundred evolved strains and find that mutations in the same gene can result in different phenotypic profiles.

      Overall, the authors deploy innovative methods to scale up experimental evolution experiments, and in many aspects of their approach tried to minimize experimental variation.

      We thank the reviewer for this positive assessment of our work. We are happy that the reviewer noted what we feel is a unique strength of our approach: we scaled up experimental evolution by using DNA barcodes and by exploring 12 related selection pressures. Despite this scaling up, we still see phenotypic convergence among the 744 adaptive mutants we study.

      The environments we study represent 12 different concentrations or combinations of two drugs, radicicol and fluconazole. Our hope is that this large dataset (774 mutants x 12 environments) will be useful, both to scientists who are generally interested in the genetic and phenotypic underpinnings of adaptation, and to scientists specifically interested in the evolution of drug resistance.

      Weaknesses:

      (1) One of the objectives of the authors is to characterize the extent of phenotypic diversity in terms of resistance trade-offs between fluconazole and radicicol. To minimize noise in the measurement of relative fitness, the authors only included strains with at least 500 barcode counts across all time points in all 12 experimental conditions, resulting in a set of 774 lineages passing this threshold. This corresponds to a very small fraction of the starting set of ~21 000 lineages that were combined after experimental evolution for fitness measurements.

      This is a misunderstanding that we will work to clarify in the revision. Our starting set did not include 21,000 adaptive lineages. The total number of unique adaptive lineages in this starting set is much lower than 21,000 for two reasons.

      First, ~21,000 represents the number of single colonies we isolated in total from our evolution experiments. Many of these isolates possess the same barcode, meaning they are duplicates. Second, and more importantly, most evolved lineages do not acquire adaptive mutations, meaning that many of the 21,000 isolates are genetically identical to their ancestor. In our revised manuscript, we will explicitly state that these 21,000 isolated lineages do not all represent unique, adaptive lineages. In figure 2 and all associated text, we will change the word “lineages” to “isolates,” where relevant.

      More broadly speaking, several previous studies have demonstrated that diverse genetic mutations converge at the level of phenotype, and have suggested that this convergence makes adaptation more predictable (PMID33263280, PMID37437111, PMID22282810, PMID25806684). Our study captures mutants that are overlooked in previous studies, such as those that emerge across subtly different selection pressures (e.g., 4 𝜇g/ml vs. 8 𝜇g/ml flu) and those that are undetectable in evolutions lacking DNA barcodes. Thus, while our experimental design misses some mutants (see next comment), it captures many others. Note that 774 adaptive lineages is more than most previous studies. Thus, we feel that “our work – showing that 774 mutants fall into a much smaller number of groups” is important because it “contributes to growing literature suggesting that the phenotypic basis of adaptation is not as diverse as the genetic basis (lines 161 - 162).”

      As the authors briefly remark, this will bias their datasets for lineages with high fitness in all 12 environments, as all these strains must be fit enough to maintain a high abundance.

      The word “briefly” feels a bit unfair because we discuss this bias on 3 separate occasions (on lines 146 - 147, 260 - 264, and in more detail on 706 - 714). We even walk through an example of a class of mutants that our study misses. We say, “our study is underpowered to detect adaptive lineages that have low fitness in any of the 12 environments. This is bound to exclude large numbers of adaptive mutants. For example, previous work has shown some FLU resistant mutants have strong tradeoffs in RAD (Cowen and Lindquist 2005). Perhaps we are unable to detect these mutants because their barcodes are at too low a frequency in RAD environments, thus they are excluded from our collection of 774.”

      In our revised version, we will add more text to the first mention of these missing mutants (lines 146 - 147) so that the implications are more immediately made apparent.

      While we “miss” some classes of mutants, we “catch” other classes that may have been missed in previous studies of convergence. For example, we observe a unique class of FLU-resistant mutants that primarily emerged in evolution experiments that lack FLU (Figure 3). Thus, we think that the unique design of our study, surveying 12 environments, allows us to make a novel contribution to the study of phenotypic convergence.

      One of the main observations of the authors is phenotypic space is constrained to a few clusters of roughly similar relative fitness patterns, giving hope that such clusters could be enumerated and considered to design antimicrobial treatment strategies. However, by excluding all lineages that fit in only one or a few environments, they conceal much of the diversity that might exist in terms of trade-offs and set up an inclusion threshold that might present only a small fraction of phenotypic space with characteristics consistent with generalist resistance mechanisms or broadly increased fitness. This has important implications regarding the general conclusions of the authors regarding the evolution of trade-offs.

      We discussed these implications in some detail in the 16 lines mentioned above (146 - 147, 260 - 264, 706 - 714). To add to this discussion, we will also add the following sentence to the end of the paragraph on lines 697 - 714: “This could complicate (or even make impossible) endeavors to design antimicrobial treatment strategies that thwart resistance”.

      We will also add a new paragraph that discusses these implications earlier in our manuscript. This paragraph will highlight the strengths of our method (e.g., that we “catch” classes of mutants that are often overlooked) while being transparent about the weaknesses of our approach (e.g., that we “miss” mutants with strong tradeoffs).

      (2) Most large-scale pooled competition assays using barcodes are usually stopped after ~25 to avoid noise due to the emergence of secondary mutations.

      The rate at which new mutations enter a population is driven by various factors such as the mutation rate and population size, so choosing an arbitrary threshold like 25 generations is difficult.

      We conducted our fitness competition following previous work using the Levy/Blundell yeast barcode system, in which the number of generations reported varies from 32 to 40 (PMID33263280, PMID27594428, PMID37861305, see PMID27594428 for detailed calculation of the fraction of lineages biased by secondary mutations in this system).

      The authors measure fitness across ~40 generations, which is almost the same number of generations as in the evolution experiment. This raises the possibility of secondary mutations biasing abundance values, which would not have been detected by the whole genome sequencing as it was performed before the competition assay.

      We understand how the reviewer came to this misunderstanding and will adjust our revised manuscript accordingly. Previous work has demonstrated that, in this particular evolution platform, most of the mutations actually occur during the transformation that introduces the DNA barcodes (PMID25731169). In other words, these mutations do not accumulate during the 40 generations of evolution, they are already there. So the observation that we collect a genetically diverse pool of adaptive mutants after 40 generations of evolution is not evidence that 40 generations is enough time for secondary mutations to bias abundance values.

      (3) The approach used by the authors to identify and visualize clusters of phenotypes among lineages does not seem to consider the uncertainty in the measurement of their relative fitness. As can be seen from Figure S4, the inter-replicate difference in measured fitness can often be quite large. From these graphs, it is also possible to see that some of the fitness measurements do not correlate linearly (ex.: Med Flu, Hi Rad Low Flu), meaning that taking the average of both replicates might not be the best approach.

      This concern, and all subsequent concerns, seem to be driven by either (a) general concerns about the noisiness of fitness measurements obtained from large-scale barcode fitness assays or (b) general concerns about whether the clusters obtained from our dimensional reduction approach capture this noise as opposed to biologically meaningful differences.

      We will respond to each concern point-by-point, but want to start by generally stating that (a) our particular large-scale barcode fitness assay has several features that diminish noise, and (b) we devote 4 figures and 200 lines of text to demonstrating that these clusters capture biologically meaningful differences between mutants (and not noise).

      In terms of this specific concern, we performed an analysis of noise in the submitted manuscript: Our noisiest fitness measurements correspond to barcodes that are the least abundant and thus suffer the most from stochastic sampling noise. These are also the barcodes that introduce the nonlinearity the reviewer mentions. We removed these from our dataset by increasing our coverage threshold from 500 reads to 5,000 reads. The clusters did not collapse, which suggests that they were not capturing noise (Figure S7 panel B). But we agree with the reviewer that this analysis alone is not sufficient to conclude that the clusters distinguish groups of mutants with unique fitness tradeoffs.

      Because the clustering approach used does not seem to take this variability into account, it becomes difficult to evaluate the strength of the clustering, especially because the UMAP projection does not include any representation of uncertainty around the position of lineages.

      To evaluate the strength of the clustering, we performed numerous analyses including whole genome sequencing, growth experiments, reclustering, and tracing the evolutionary origins of each cluster (Figures 5 - 8). All of these analyses suggested that our clusters capture groups of mutants that have different fitness tradeoffs. We will adjust our revised manuscript to make clear that we do not rely on the results of a clustering algorithm alone to draw conclusions about phenotypic convergence.

      We are also grateful to the reviewer for helping us realize that, as written, our manuscript is not clear with regard to how we perform clustering. We are not using UMAP to decide which mutant belongs to which cluster. Recent work highlights the importance of using an independent clustering method (PMID37590228). Although this recent work addresses the challenge of clustering much higher dimensional data than we survey here, we did indeed use an independent clustering method (gaussian mixture model). In other words, we use UMAP for visualization but not clustering. We also confirm our clustering results using a second independent method (hierarchical clustering; Figure S8). And in our revised manuscript, will confirm with a third method (PCA, see below). We will adjust the main text and the methods section to make these choices clearer.

      This might paint a misleading picture where clusters appear well separate and well defined but are in fact much fuzzier, which would impact the conclusion that the phenotypic space is constricted.

      The salient question is whether the clusters are so “fuzzy” that they are not meaningful. That interpretation seems unreasonable. Our clusters group mutants with similar genotypes, evolutionary histories, and fitness tradeoffs (Figures 5 - 8). Clustering mutants with similar behaviors is important and useful. It improves phenotypic prediction by revealing which mutants are likely to have at least some phenotypic effects in common. And it also suggests that the phenotypic space is constrained, at least to some degree, which previous work suggests is helpful in predicting evolution (PMID33263280, PMID37437111, PMID22282810, PMID25806684).

      (4) The authors make the decision to use UMAP and a gaussian mixed model to cluster and represent the different fitness landscapes of their lineages of interest. Their approach has many caveats. First, compared to PCA, the axis does not provide any information about the actual dissimilarities between clusters. Using PCA would have allowed a better understanding of the amount of variance explained by components that separate clusters, as well as more interpretable components.

      The components derived from PCA are often not interpretable. It’s not obvious that each one, or even the first one, will represent some intuitive phenotype, like resistance to fluconazole.

      Moreover, we see many non-linearities in our data. For example, fitness in a double drug environment is not predicted by adding up fitness in the relevant single drug environments. Also, there are mutants that have high fitness when fluconazole is absent or abundant, but low fitness when mild concentrations are present. These types of nonlinearities can make the axes in PCA very difficult to interpret, plus these nonlinearities can be missed by PCA, thus we prefer other clustering methods.

      We will adjust our revised manuscript to explain these reasons why we chose UMAP and GMM over PCA.

      Also, we will include PCA in the supplement of our revised manuscript. Please find below PC1 vs PC2, with points colored according to the cluster assignment in figure 4 (i.e. using a gaussian mixture model). It appears the clusters are largely preserved.

      Author response image 1.

      Second, the advantages of dimensional reduction are not clear. In the competition experiment, 11/12 conditions (all but the no drug, no DMSO conditions) can be mapped to only three dimensions: concentration of fluconazole, concentration of radicicol, and relative fitness. Each lineage would have its own fitness landscape as defined by the plane formed by relative fitness values in this space, which can then be examined and compared between lineages.

      We worry that the idea stems from apriori notions of what the important dimensions should be. It also seems like this would miss important nonlinearities such as our observation that low fluconazole behaves more like a novel selection pressure than a dialed down version of high fluconazole.

      Also, we believe the reviewer meant “fitness profile” and not “fitness landscape”. A fitness landscape imagines a walk where every “step” is a mutation. Most lineages in barcoded evolution experiments possess only a single adaptive mutation. A single-step walk is not enough to build a landscape, though others are expanding barcoded evolution experiments beyond the first step (PMID34465770, PMID31723263), so maybe one day this will be possible.

      Third, the choice of 7 clusters as the cutoff for the multiple Gaussian model is not well explained. Based on Figure S6A, BIC starts leveling off at 6 clusters, not 7, and going to 8 clusters would provide the same reduction as going from 6 to 7. This choice also appears arbitrary in Figure S6B, where BIC levels off at 9 clusters when only highly abundant lineages are considered.

      We agree. We did not rely on the results of BIC alone to make final decisions about how many clusters to include. We thank the reviewer for pointing out this gap in our writing. We will adjust our revised manuscript to explain that we ultimately chose to describe 6 clusters that we were able to validate with follow-up experiments. In figures 5, 6, 7, and 8, we use external information to validate the clusters that we report in figure 4. And in lines 697 – 714, we explain that there are may be additional clusters beyond those we tease apart in this study.

      This directly contradicts the statement in the main text that clusters are robust to noise, as more a stringent inclusion threshold appears to increase and not decrease the optimal number of clusters. Additional criteria to BIC could have been used to help choose the optimal number of clusters or even if mixed Gaussian modeling is appropriate for this dataset.

      We are under the following impression: If our clustering method was overfitting, i.e. capturing noise, the optimal number of clusters should decrease when we eliminate noise. It increased. In other words, the observation that our clusters did not collapse (i.e. merge) when we removed noise suggests these clusters were not capturing noise.

      More generally, our validation experiments, described below, provide additional evidence that our clusters capture meaningful differences between mutants (and not noise).

      (5) Large-scale barcode sequencing assays can often be noisy and are generally validated using growth curves or competition assays.

      Some types of bar-seq methods, in particular those that look at fold change across two time points, are noisier than others that look at how frequency changes across multiple timepoints (PMID30391162). Here, we use the less noisy method. We also reduce noise by using a stricter coverage threshold than previous work (e.g., PMID33263280), and by excluding batch effects by performing all experiments simultaneously (PMID37237236).

      The main assay we use to measure fitness has been previously validated (PMID27594428). No subsequent study using this assay validates using the methods suggested by the reviewer (see PMID37861305, PMID33263280, PMID31611676, PMID29429618, PMID37192196, PMID34465770, PMID33493203).

      More to the point, bar-seq has been used, without the reviewer’s suggested validation, to demonstrate that the way some mutant’s fitness changes across environments is different from other mutants (PMID33263280, PMID37861305, PMID31611676, PMID33493203, PMID34596043). This is the same thing that we use bar-seq to demonstrate.

      For all of these reasons, we are hesitant to confirm bar-seq itself as a valid way to infer fitness. It seems this is already accepted as a standard in our field.

      Having these types of results would help support the accuracy of the main assay in the manuscript and thus better support the claims of the authors.

      We don’t agree that fitness measurements obtained from this bar-seq assay generally require validation. But we do agree that it is important to validate whether the mutants in each of our 6 clusters indeed are different from one another in meaningful ways, in particular, in that they have different fitness tradeoffs. We have four figures (5 - 8) and 200 lines of text dedicated to validating whether our clusters capture reproducible and biologically meaningful differences between mutants. Happily, one of these figures (Fig 7) includes growth curves, which are exactly the type of validation experiment asked for by the reviewer.

      Below, we walk through the different types of validation experiments that are present in our original manuscript, and additional validation experiments that we plan to include in the revised version. We are hopeful that these validation experiments are sufficient, or at the very least, that this list empowers reviewers to point out where more work is needed.

      (1) Mutants from different clusters have different growth curves: In our original manuscript, we measured growth curves corresponding to a fitness tradeoff that we thought was surprising. Mutants in clusters 4 and 5 both have fitness advantages in single drug conditions. While mutants from cluster 4 also are advantageous in the double drug conditions, mutants from cluster 5 are not! We validated these different behaviors by studying growth curves for a mutant from each cluster (Figures 7 and S10).

      (2) Mutants from different clusters have different evolutionary origins: In our original manuscript, we came up with a novel way to ask whether the clusters capture different types of adaptive mutants. We asked whether the mutants in each cluster originate from different evolution experiments. Indeed they often do (see pie charts in Figures 6, 7, 8). This method also provides evidence supporting each cluster’s differing fitness tradeoffs.

      For example, mutants in cluster 5 appear to have a tradeoff in a double drug condition (described above). They rarely originate from that evolution condition, unlike mutants in nearby cluster 4 (see Figure 7).

      (3) Mutants from each cluster often fall into different genes: In our original manuscript, we sequenced many of these mutants and show that mutants in the same gene are often found in the same cluster. For example, all 3 IRA1 mutants are in cluster 6 (Fig 8), both GPB2 mutants are in cluster 4 (Figs 7 & 8), and 35/36 PDR mutants are in either cluster 2 or 3 (Figs 5 & 6).

      (4) Mutants from each cluster have behaviors previously observed in the literature: In our original manuscript, we compared our sequencing results to the literature and found congruence. For example, PDR mutants are known to provide a fitness benefit in fluconazole and are found in clusters that have high fitness in fluconazole (lines 457 - 462). Previous work suggests that some mutations to PDR have different tradeoffs than others, which is what we see (lines 540 - 542). IRA1 mutants were previously observed to have high fitness in our “no drug” condition, and are found in the cluster that has the highest fitness in the “no drug” condition (lines 642 - 646). Previous work even confirms the unusual fitness tradeoff we observe where IRA1 and other cluster 6 mutants have low fitness only in low concentrations of fluconazole (lines 652 - 657).

      (5) Mutants largely remain in their clusters when we use alternate clustering methods: In our original manuscript, we performed various different reclustering and/or normalization approaches on our data (Fig 6, S5, S7, S8, S9). The clusters of mutants that we observe in figure 4 do not change substantially when we recluster the data. We will add PCA (see above) to these analyses in our revised manuscript.

      (6) We will include additional data showing that mutants in different clusters have different evolutionary origins: Cluster 1 is defined by high fitness in low fluconazole that declines with increasing fluconazole (see Fig 4E and Fig 5C). In our revised manuscript, we will show that cluster 1 lineages were overwhelmingly sampled from evolutions conducted in our lowest concentration of fluconazole (see figure panel A below). No other cluster’s evolutionary history shows this pattern (figures 6, 7, and 8).

      (7) We will include additional data showing that mutants in different clusters have different growth curves: Cluster 1 lineages are unique in that their fitness advantage is specific to low flu and trades off in higher concentrations of fluconazole. We obtained growth curves for three cluster 1 mutants (2 SUR1 mutants and 1 UPC2 mutant). We compared them to growth curves for three PDR mutants (from clusters 2 and 3). Cluster 1 mutants appear to have the highest growth rates and reach the higher carrying capacity in low fluconazole (see red and green lines in Author response image 2 panel B below). But the cluster 1 mutants are negatively affected by higher concentrations of fluconazole, much more so than the mutants from clusters 2 and 3 (see Author response image 2 panel C below). This is consistent with the different fitness tradeoffs we observe for each cluster (figures 4 and 5). We will include a more detailed version of this analysis and the figures below in our revised manuscript.

      Author response image 2.

      Validation experiments demonstrate that cluster 1 mutants have uniquely high fitness in only the lowest concentration of fluconazole. (A) The mutant lineages in cluster 1 were largely sampled from evolution experiments performed in low flu. This is not true of other clusters (see pie charts in main manuscript). (B) In low flu (4 𝜇g/ml), Cluster 1 lineages (red/UPC2 and green/SUR1) grow faster and achieve higher density than lineages from clusters 2 and 3 (blue/PDR). This is consistent with barseq measurements demonstrating that cluster 1 mutants have the highest fitness in low flu. (C) Cluster 1 lineages are sensitive to increasing flu concentrations (SUR1 and UPC2 mutants, middle and rightmost graphs). This is apparent in that the gray (8 𝜇g/ml flu) and light blue (32 𝜇g/ml flu) growth curves rise more slowly and reach lower density than the dark blue curves (4 𝜇g/ml flu). But this is not the case for the PDR mutants from clusters 2 and 3 (leftmost graph). These observations are consistent with the bar-seq fitness data presented in the main manuscript (Fig 4E).

      With all of these validation efforts combined, we are hopeful that the reviewer is now more convinced that our clusters capture groups of mutants with different fitness tradeoffs (as opposed to noise). We want to conclude by saying that we are grateful to the reviewer for making us think deeply about areas where we can include additional validation efforts as well as areas where we can make our manuscript clearer.

      Reviewer #2 (Public Review):

      Summary:

      Schmidlin & Apodaca et al. aim to distinguish mutants that resist drugs via different mechanisms by examining fitness tradeoffs across hundreds of fluconazole-resistant yeast strains. They barcoded a collection of fluconazole-resistant isolates and evolved them in different environments with a view to having relevance for evolutionary theory, medicine, and genotypephenotype mapping.

      Strengths:

      There are multiple strengths to this paper, the first of which is pointing out how much work has gone into it; the quality of the experiments (the thought process, the data, the figures) is excellent. Here, the authors seek to induce mutations in multiple environments, which is a really large-scale task. I particularly like the attention paid to isolates with are resistant to low concentrations of FLU. So often these are overlooked in favour of those conferring MIC values >64/128 etc. What was seen is different genotype and fitness profiles. I think there's a wealth of information here that will actually be of interest to more than just the fields mentioned (evolutionary medicine/theory).

      We are very grateful for this positive review. This was indeed a lot of work! We are happy that the reviewer noted what we feel is a unique strength of our manuscript: that we survey adaptive isolates across multiple environments, including low drug concentrations.

      Weaknesses:

      Not picking up low fitness lineages - which the authors discuss and provide a rationale as to why. I can completely see how this has occurred during this research, and whilst it is a shame I do not think this takes away from the findings of this paper. Maybe in the next one!

      We thank the reviewer for these words of encouragement and will work towards catching more low fitness lineages in our next project.

      In the abstract the authors focus on 'tradeoffs' yet in the discussion they say the purpose of the study is to see how many different mechanisms of FLU resistance may exist (lines 679-680), followed up by "We distinguish mutants that likely act via different mechanisms by identifying those with different fitness tradeoffs across 12 environments". Whilst I do see their point, and this is entirely feasible, I would like a bit more explanation around this (perhaps in the intro) to help lay-readers make this jump. The remainder of my comments on 'weaknesses' are relatively fixable, I think:

      We think that phrasing the “jump” as a question might help lay readers get from point A to point B. So, in the introduction of our revised manuscript, we will add a paragraph roughly similar to this one: “If two groups of drug-resistant mutants have different fitness tradeoffs, does it mean that they provide resistance through different underlying mechanisms? Alternatively, it could mean that both provide drug resistance via the same mechanism, but some mutations come with a cost that others don’t pay. However, another way to phrase this alternative is to say that both groups of mutants affect fitness through different suites of mechanisms that are only partially overlapping. And so, by identifying groups of mutants with different fitness tradeoffs, we argue that we will be uncovering sets of mutations that impact fitness through different underlying mechanisms. The ability to do so would be useful for genotype-phenotype mapping endeavors.”

      In the introduction I struggle to see how this body of research fits in with the current literature, as the literature cited is a hodge-podge of bacterial and fungal evolution studies, which are very different! So example, the authors state "previous work suggests that mutants with different fitness tradeoffs may affect fitness through different molecular mechanisms" (lines 129-131) and then cite three papers, only one of which is a fungal research output. However, the next sentence focuses solely on literature from fungal research. Citing bacterial work as a foundation is fine, but as you're using yeast for this I think tailoring the introduction more to what is and isn't known in fungi would be more appropriate. It would also be great to then circle back around and mention monotherapy vs combination drug therapy for fungal infections as a rationale for this study. The study seems to be focused on FLU-resistant mutants, which is the first-line drug of choice, but many (yeast) infections have acquired resistance to this and combination therapy is the norm.

      In our revised manuscript, we will carefully review all citations. The issue may stem from our attempt to reach two different groups of scientists. We ourselves are broadly interested in the structure of the genotype-phenotype-fitness map (PMID33263280, PMID32804946). Though the 3 papers the reviewer mentions on lines 132 - 133 all pertain to yeast, we cite them because they are studies about the complexity of this map. Their conclusions, in theory, should apply broadly, beyond yeast. Similarly, the reason we cite papers from yeast, as well as bacteria and cancer, is that we believe general conclusions about the genotype-phenotype-fitness map should apply broadly. For example, the sentence the reviewer highlights, “previous work suggests that mutants with different fitness tradeoffs may affect fitness through different molecular mechanisms” is a general observation about the way genotype maps to fitness. So we cited papers from across the tree of life to support this sentence.

      On the other hand, because we study drug resistant mutations, we also hope that our work is of use to scientists studying the evolution of resistance. We agree with the reviewer that in this regard, some of our findings may be especially pertinent to the evolution of resistance to antifungal drugs. We will consider this when reviewing the citations in our revised manuscript and add some text to clarify these points.

      Methods: Line 769 - which yeast? I haven't even seen mention of which species is being used in this study; different yeast employ different mechanisms of adaptation for resistance, so could greatly impact the results seen. This could help with some background context if the species is mentioned (although I assume S. cerevisiae).

      In the revised manuscript, we will make clear that we study S. cerevisiae.

      In which case, should aneuploidy be considered as a mechanism? This is mentioned briefly on line 556, but with all the sequencing data acquired this could be checked quickly?

      We like this idea and we are working on it, but it is not straightforward. The reviewer is correct in that we can use the sequencing data that we already have. But calling aneuploidy with certainty is tough because its signal can be masked by noise. In other words, some regions of the genome may be sequenced more than others by chance. Given this is not straightforward, at least not for us, this analysis will likely have to wait for a subsequent paper.

      I think the authors could be bolder and try and link this to other (pathogenic) yeasts. What are the implications of this work on say, Candida infections?

      Perhaps because our background lies in general study of the genotype-phenotype map, we did not want to make bold assertions about how our work might apply to pathogenic yeasts. But we see how this could be helpful and will add some discussion points about this. Specifically, we will discuss which of the genes and mutants we observe are also found in Candida. We will also investigate whether our observation that low fluconazole represents a seemingly unique challenge, not just a milder version of high fluconazole, has any corollary in the Candida literature.

    1. Author response:

      We thank the reviewers for their thorough reading and thoughtful feedback. Below, we provisionally address each of the concerns raised in the public reviews, and outline our planned revision that aims to further clarify and strengthen the manuscript.

      In our response, we clarify our conceptualization of elasticity as a dimension of controllability, formalizing it within an information-theoretic framework, and demonstrating that controllability and its elasticity are partially dissociable. Furthermore, we provide clarifications and additional modeling results showing that our experimental design and modeling approach are well-suited to dissociating elasticity inference from more general learning processes, and are not inherently biased to find overestimates of elasticity. Finally, we clarify the advantages and disadvantages of our canonical correlation analysis (CCA) approach for identifying latent relationships between multidimensional data sets, and provide additional analyses that strengthen the link between elasticity estimation biases and a specific psychopathology profile.

      Reviewer 1:

      This research takes a novel theoretical and methodological approach to understanding how people estimate the level of control they have over their environment, and how they adjust their actions accordingly. The task is innovative and both it and the findings are well-described (with excellent visuals). They also offer thorough validation for the particular model they develop. The research has the potential to theoretically inform the understanding of control across domains, which is a topic of great importance.

      We thank the reviewer for their favorable appraisal and valuable suggestions, which have helped clarify and strengthen the study’s conclusion. 

      An overarching concern is that this paper is framed as addressing resource investments across domains that include time, money, and effort, and the introductory examples focus heavily on effort-based resources (e.g., exercising, studying, practicing). The experiments, though, focus entirely on the equivalent of monetary resources - participants make discrete actions based on the number of points they want to use on a given turn. While the same ideas might generalize to decisions about other kinds of resources (e.g., if participants were having to invest the effort to reach a goal), this seems like the kind of speculation that would be better reserved for the Discussion section rather than using effort investment as a means of introducing a new concept (elasticity of control) that the paper will go on to test.

      We thank the reviewer for pointing out a lack of clarity regarding the kinds of resources tested in the present experiment. Investing additional resources in the form of extra tickets did not only require participants to pay more money. It also required them to invest additional time – since each additional ticket meant making another attempt to board the vehicle, extending the duration of the trial, and attentional effort – since every attempt required precisely timing a spacebar press as the vehicle crossed the screen. Given this involvement of money, time, and effort resources, we believe it would be imprecise to present the study as concerning monetary resources in particular. That said, we agree with the Reviewer that results might differ depending on the resource type that the experiment or the participant considers most. Thus, in our revision of the manuscript, we will make sure to clarify the kinds of resources the experiment involved, and highlight the open question of whether inferences concerning the elasticity of control generalize across different resource domains.

      Setting aside the framing of the core concepts, my understanding of the task is that it effectively captures people's estimates of the likelihood of achieving their goal (Pr(success)) conditional on a given investment of resources. The ground truth across the different environments varies such that this function is sometimes flat (low controllability), sometimes increases linearly (elastic controllability), and sometimes increases as a step function (inelastic controllability). If this is accurate, then it raises two questions.

      First, on the modeling front, I wonder if a suitable alternative to the current model would be to assume that the participants are simply considering different continuous functions like these and, within a Bayesian framework, evaluating the probabilistic evidence for each function based on each trial's outcome. This would give participants an estimate of the marginal increase in Pr(success) for each ticket, and they could then weigh the expected value of that ticket choice (Pr(success)*150 points) against the marginal increase in point cost for each ticket. This should yield similar predictions for optimal performance (e.g., opt-out for lower controllability environments, i.e., flatter functions), and the continuous nature of this form of function approximation also has the benefit of enabling tests of generalization to predict changes in behavior if there was, for instance, changes in available tickets for purchase (e.g., up to 4 or 5) or changes in ticket prices. Such a model would of course also maintain a critical role for priors based on one's experience within the task as well as over longer timescales, and could be meaningfully interpreted as such (e.g., priors related to the likelihood of success/failure and whether one's actions influence these). It could also potentially reduce the complexity of the model by replacing controllability-specific parameters with multiple candidate functions (presumably learned through past experience, and/or tuned by experience in this task environment), each of which is being updated simultaneously.

      Second, if the reframing above is apt (regardless of the best model for implementing it), it seems like the taxonomy being offered by the authors risks a form of "jangle fallacy," in particular by positing distinct constructs (controllability and elasticity) for processes that ultimately comprise aspects of the same process (estimation of the relationship between investment and outcome likelihood). Which of these two frames is used doesn't bear on the rigor of the approach or the strength of the findings, but it does bear on how readers will digest and draw inferences from this work. It is ultimately up to the authors which of these they choose to favor, but I think the paper would benefit from some discussion of a common-process alternative, at least to prevent too strong of inferences about separate processes/modes that may not exist. I personally think the approach and findings in this paper would also be easier to digest under a common-construct approach rather than forcing new terminology but, again, I defer to the authors on this.

      We thank the reviewer for suggesting this interesting alternative modeling approach. We agree that a Bayesian framework evaluating different continuous functions could offer advantages, particularly in its ability to generalize to other ticket quantities and prices. We will attempt to implement this as an alternative model and compare it with the current model.  

      We also acknowledge the importance of avoiding a potential "jangle fallacy". We entirely agree with the Reviewer that elasticity and controllability inferences are not distinct processes. Specifically, we view resource elasticity as a dimension of controllability, hence the name of our ‘elastic controllability’ model. In response to this and other Reviewers’ comments, we now offer a formal definition of elasticity as the reduction in uncertainty about controllability due to knowing the amount of resources the agent is able and willing to invest (see further details in response to Reviewer 3 below).  

      With respect to how this conceptualization is expressed in the modelling, we note that the representation in our model of maximum controllability and its elasticity via different variables is analogous to how a distribution may be represented by separate mean and variance parameters. Ultimately, even in the model suggested by the Reviewer, there would need to be a dedicated variable representing elasticity, such as the probability of sloped controllability functions. A single-process account thus allows that different aspects of this process would be differently biased (e.g., one can have an accurate estimate of the mean of a distribution but overestimate its variance). Therefore, our characterization of distinct elasticity and controllability biases (or to put it more accurately, ‘elasticity of controllability bias’ and ‘maximum controllability bias’) is consistent with a common construct account. 

      That said, given the Reviewer’s comments, we believe that some of the terminology we used may have been misleading. In our planned revision, we will modify the text to clarify that we view elasticity as a dimension of controllability that can only be estimated in conjunction with controllability. 

      Reviewer 2:

      This research investigates how people might value different factors that contribute to controllability in a creative and thorough way. The authors use computational modeling to try to dissociate "elasticity" from "overall controllability," and find some differential associations with psychopathology. This was a convincing justification for using modeling above and beyond behavioral output and yielded interesting results. Interestingly, the authors conclude that these findings suggest that biased elasticity could distort agency beliefs via maladaptive resource allocation. Overall, this paper reveals some important findings about how people consider components of controllability.

      We appreciate the Reviewer's positive assessment of our findings and computational approach to dissociating elasticity and overall controllability.

      The primary weakness of this research is that it is not entirely clear what is meant by "elastic" and "inelastic" and how these constructs differ from existing considerations of various factors/calculations that contribute to perceptions of and decisions about controllability. I think this weakness is primarily an issue of framing, where it's not clear whether elasticity is, in fact, theoretically dissociable from controllability. Instead, it seems that the elements that make up "elasticity" are simply some of the many calculations that contribute to controllability. In other words, an "elastic" environment is inherently more controllable than an "inelastic" one, since both environments might have the same level of predictability, but in an "elastic" environment, one can also partake in additional actions to have additional control overachieving the goal (i.e., expend effort, money, time).

      We thank the reviewer for highlighting the lack of clarity in our concept of elasticity. We first clarify that elasticity cannot be entirely dissociated from controllability because it is a dimension of controllability. If no controllability is afforded, then there cannot be elasticity or inelasticity. This is why in describing the experimental environments, we only label high-controllability, but not low-controllability, environments as ‘elastic’ or ‘inelastic’. For further details on this conceptualization of elasticity, and a planned revision of the text, see our response above to Reviewer 1. 

      Second, we now clarify that controllability can also be computed without knowing the amount of resources the agent is able and willing to invest, for instance by assuming infinite resources available or a particular distribution of resource availabilities. However, knowing the agent’s available resources often reduces uncertainty concerning controllability. This reduction in uncertainty is what we define as elasticity. Since any action requires some resources, this means that no controllable environment is entirely inelastic if we also consider agents that do not have enough resources to commit any action. However, even in this case environments can differ in the degree to which they are elastic. For further details on this formal definition, see our response to Reviewer 3 below. We will make these necessary clarifications in the revised manuscript. 

      Importantly, whether an environment is more or less elastic does not determine whether it is more or less controllable. In particular, environments can be more controllable yet less elastic. This is true even if we allow that investing different levels of resources (i.e., purchasing 0, 1, 2, or 3 tickets) constitute different actions, in conjunction with participants’ vehicle choices. Below, we show this using two existing definitions of controllability. 

      Definition 1, reward-based controllability<sup>1</sup>: If control is defined as the fraction of available reward that is controllably achievable, and we assume all participants are in principle willing and able to invest 3 tickets, controllability can be computed in the present task as:

      where P(S' \= goal ∣ 𝑆, 𝐴, 𝐶 ) is the probability of reaching the treasure from present state 𝑆 when taking action A and investing C resources in executing the action. In any of the task environments, the probability of reaching the goal is maximized by purchasing 3 tickets (𝐶 = 3) and choosing the vehicle that leads to the goal (𝐴 = correct vehicle). Conversely, the probability of reaching the goal is minimized by purchasing 3 tickets (𝐶 = 3) and choosing the vehicle that does not lead to the goal (𝐴 = wrong vehicle). This calculation is thus entirely independent of elasticity, since it only considers what would be achieved by maximal resource investment, whereas elasticity consists of the reduction in controllability that would arise if the maximal available 𝐶 is reduced. Consequently, any environment where the maximum available control is higher yet varies less with resource investment would be more controllable and less elastic. 

      Note that if we also account for ticket costs in calculating reward, this will only reduce the fraction of achievable reward and thus the calculated control in elastic environments.   

      Definition 2, information-theoretic controllability<sup>2</sup>: Here controllability is defined as the reduction in outcome entropy due to knowing which action is taken:

      I(S'; A, C | S) = H(S'|S) - H(S'|S, A, C)

      where H(S'|S) is the conditional entropy of the distribution of outcomes S' given the present state 𝑆, and H(S'|S, A, C) is the conditional entropy of the outcome given the present state, action, and resource investment. 

      To compare controllability, we consider two environments with the same maximum control:

      • Inelastic environment: If the correct vehicle is chosen, there is a 100% chance of reaching the goal state with 1, 2, or 3 tickets. Thus, out of 7 possible action-resource investment combinations, three deterministically lead to the goal state (≥1 tickets and correct vehicle choice), three never lead to it (≥1 tickets and wrong vehicle choice), and one (0 tickets) leads to it 20% of the time (since walking leads to the treasure on 20% of trials).

      • Elastic Environment: If the correct vehicle is chosen, the probability of boarding it is 0% with 1 ticket, 50% with 2 tickets, and 100% with 3 tickets. Thus, out of 7 possible actionresource investment combinations, one deterministically leads to the goal state (3 tickets and correct vehicle choice), one never leads to it (3 tickets and wrong vehicle choice), one leads to it 60% of the time (2 tickets and correct vehicle choice: 50% boarding + 50% × 20% when failing to board), one leads to it 10% of time (2 ticket and wrong vehicle choice), and three lead to it 20% of time (0-1 tickets).

      Here we assume a uniform prior over actions, which renders the information-theoretic definition of controllability equal to another definition termed ‘instrumental divergence’3,4. We note that changing the uniform prior assumption would change the results for the two environments, but that would not change the general conclusion that there can be environments that are more controllable yet less elastic. 

      Step 1: Calculating H(S'|S)

      For the inelastic environment:

      P(goal) = (3 × 100% + 3 × 0% + 1 × 20%)/7 = .46, P(non-goal) = .54  H(S'|S) = – [.46 × log<sub>2</sub>(.46) + .54 × log<sub>2</sub>(.54)] \= 1 bit

      For the elastic environment:

      P(goal) \= (1 × 100% + 1 × 0% + 1 × 60% + 1 × 10% + 3 × 20%)/7 \= .33, P(non-goal) \= .67  H(S'|S) = – [.33 × log<sub>2</sub>(.33) + .67 × log<sub>2</sub>(.67)] \= .91 bits

      Step 2: Calculating H(S'|S, A, C)

      Inelastic environment: Six action-resource investment combinations have deterministic outcomes entailing zero entropy, whereas investing 0 tickets has a probabilistic outcome (20%). The entropy for 0 tickets is: H(S'|C \= 0) \= -[.2 × log<sub>2</sub>(.2) + 0.8 × log<sub>2</sub> (.8)] = .72 bits. Since this actionresource investment combination is chosen with probability 1/7, the total conditional entropy is approximately .10 bits

      Elastic environment: 2 actions have deterministic outcomes (3 tickets with correct/wrong vehicle), whereas the other 5 actions have probabilistic outcomes:

      2 tickets and correct vehicle (60% success): 

      H(S'|A = correct, C = 2) = – [.6 × log<sub>2</sub>(.6) + .4 × log<sub>2</sub>(.4)] \= .97 bits 2 tickets and wrong vehicle (10% success): 

      H(S'|A = wrong, C = 2) = – [.1 × <sub>2</sub>(.1) + .9 × <sub>2</sub>(.9)] \= .47 bits 0-1 tickets (20% success):

      H(S'|C = 0-1) = – [.2 × <sub>2</sub>(.2) + .8 × <sub>2</sub> .8)] \= .72 bits

      Thus the total conditional entropy of the elastic environment is: H(S'|S, A, C) = (1/7) × .97 + (1/7) × .47 + (3/7) × .72 \= .52 bits

      Step 3: Calculating I(S' | A, S)  

      Inelastic environment: I(S'; A, C | S) = H(S'|S) – H(S'|S, A, C) = 1 – 0.1 = .9 bits 

      Elastic environment: I(S'; A, C | S) = H(S'|S) – H(S'|S, A, C) = .91 – .52 = .39 bits

      Thus, the inelastic environment offers higher information-theoretic controllability (.9 bits) compared to the elastic environment (.39 bits). 

      Of note, even if each combination of cost and goal reaching is defined as a distinct outcome, then information-theoretic controllability is higher for the inelastic (2.81 bits) than for the elastic (2.30 bits) environment. 

      In sum, for both definitions of controllability, we see that environments can be more elastic yet less controllable. We will amend the manuscript to clarify this distinction between controllability and its elasticity.

      Reviewer 3:

      A bias in how people infer the amount of control they have over their environment is widely believed to be a key component of several mental illnesses including depression, anxiety, and addiction. Accordingly, this bias has been a major focus in computational models of those disorders. However, all of these models treat control as a unidimensional property, roughly, how strongly outcomes depend on action. This paper proposes---correctly, I think---that the intuitive notion of "control" captures multiple dimensions in the relationship between action and outcome is multi-dimensional. In particular, the authors propose that the degree to which outcome depends on how much *effort* we exert, calling this dimension the "elasticity of control". They additionally propose that this dimension (rather than the more holistic notion of controllability) may be specifically impaired in certain types of psychopathology. This idea thus has the potential to change how we think about mental disorders in a substantial way, and could even help us better understand how healthy people navigate challenging decision-making problems.

      Unfortunately, my view is that neither the theoretical nor empirical aspects of the paper really deliver on that promise. In particular, most (perhaps all) of the interesting claims in the paper have weak empirical support.

      We appreciate the Reviewer's thoughtful engagement with our research and recognition of the potential significance of distinguishing between different dimensions of control in understanding psychopathology. We believe that all the Reviewer’s comments can be addressed with clarifications or additional analyses, as detailed below.  

      Starting with theory, the elasticity idea does not truly "extend" the standard control model in the way the authors suggest. The reason is that effort is simply one dimension of action. Thus, the proposed model ultimately grounds out in how strongly our outcomes depend on our actions (as in the standard model). Contrary to the authors' claims, the elasticity of control is still a fixed property of the environment. Consistent with this, the computational model proposed here is a learning model of this fixed environmental property. The idea is still valuable, however, because it identifies a key dimension of action (namely, effort) that is particularly relevant to the notion of perceived control. Expressing the elasticity idea in this way might support a more general theoretical formulation of the idea that could be applied in other contexts. See Huys & Dayan (2009), Zorowitz, Momennejad, & Daw (2018), and Gagne & Dayan (2022) for examples of generalizable formulations of perceived control.

      We thank the Reviewer for the suggestion that we formalize our concept of elasticity to resource investment, which we agree is a dimension of action. We first note that we have not argued against the claim that elasticity is a fixed property of the environment. We surmise the Reviewer might have misread our statement that “controllability is not a fixed property of the environment”. The latter statement is motivated by the observation that controllability is often higher for agents that can invest more resources (e.g., a richer person can buy more things). We will clarify this in our revision of the manuscript.

      To formalize elasticity, we build on Huys & Dayan’s definition of controllability(1) as the fraction of reward that is controllably achievable, 𝜒 (though using information-theoretic definitions(2,3) would work as well). To the extent that this fraction depends on the amount of resources the agent is able and willing to invest (max 𝐶), this formulation can be probabilistically computed without information about the particular agent involved, specifically, by assuming a certain distribution of agents with different amounts of available resources. This would result in a probability distribution over 𝜒. Elasticity can thus be defined as the amount of information obtained about controllability due to knowing the amount of resources available to the agent: I(𝜒; max 𝐶). We will add this formal definition to the manuscript.  

      Turning to experiment, the authors make two key claims: (1) people infer the elasticity of control, and (2) individual differences in how people make this inference are importantly related to psychopathology. Starting with claim 1, there are three sub-claims here; implicitly, the authors make all three. (1A) People's behavior is sensitive to differences in elasticity, (1B) people actually represent/track something like elasticity, and (1C) people do so naturally as they go about their daily lives. The results clearly support 1A. However, 1B and 1C are not supported. Starting with 1B, the experiment cannot support the claim that people represent or track elasticity because the effort is the only dimension over which participants can engage in any meaningful decision-making (the other dimension, selecting which destination to visit, simply amounts to selecting the location where you were just told the treasure lies). Thus, any adaptive behavior will necessarily come out in a sensitivity to how outcomes depend on effort. More concretely, any model that captures the fact that you are more likely to succeed in two attempts than one will produce the observed behavior. The null models do not make this basic assumption and thus do not provide a useful comparison.

      We appreciate the reviewer's critical analysis of our claims regarding elasticity inference, which as detailed below, has led to an important new analysis that strengthens the study’s conclusions. However, we respectfully disagree with two of the Reviewer’s arguments. First, resource investment was not the only meaningful decision dimension in our task, since participant also needed to choose the correct vehicle to get to the right destination. That this was not trivial is evidenced by our exclusion of over 8% of participants who made incorrect vehicle choices more than 10% of the time. Included participants also occasionally erred in this choice (mean error rate = 3%, range [0-10%]). 

      Second, the experimental task cannot be solved well by a model that simply tracks how outcomes depend on effort because 20% of the time participants reached the treasure despite failing to board their vehicle of choice. In such cases, reward outcomes and control were decoupled. Participants could identify when this was the case by observing the starting location, which was revealed together with the outcome (since depending on the starting location, the treasure location was automatically reached by walking). To determine whether participants distinguished between control-related and non-control-related reward, we have now fitted a variant of our model to the data that allows learning from each of these kinds of outcomes by means of a different free parameter. The results show that participants learned considerably more from control-related outcomes. They were thus not merely tracking outcomes, but specifically inferred when outcomes can be attributed to control. We will include this new analysis in the revised manuscript.

      Controllability inference by itself, however, still does not suffice to explain the observed behavior. This is shown by our ‘controllability’ model, which learns to invest more resources to improve control, yet still fails to capture key features of participants’ behavior, as detailed in the manuscript. This means that explaining participants’ behavior requires a model that not only infers controllability—beyond merely outcome probability—but also assumes a priori that increased effort could enhance control. Building these a priori assumption into the model amounts to embedding within it an understanding of elasticity – the idea that control over the environment may be increased by greater resource investment. 

      That being said, we acknowledge the value in considering alternative computational formulations of adaptation to elasticity. Thus, in our revision of the manuscript, we will add a discussion concerning possible alternative models.  

      For 1C, the claim that people infer elasticity outside of the experimental task cannot be supported because the authors explicitly tell people about the two notions of control as part of the training phase: "To reinforce participants' understanding of how elasticity and controllability were manifested in each planet, [participants] were informed of the planet type they had visited after every 15 trips." (line 384).

      We thank the reviewer for highlighting this point. We agree that our experimental design does not test whether people infer elasticity spontaneously. Our research question was whether people can distinguish between elastic and inelastic controllability. The results strongly support that they can, and this does have potential implications for behavior outside of the experimental task. Specifically, to the extent that people are aware that in some contexts additional resource investment improve control, whereas in other contexts it does not, then our results indicate that they would be able to distinguish between these two kinds of contexts through trial-and-error learning. That said, we agree that investigating whether and how people spontaneously infer elasticity is an interesting direction for future work. We will clarify the scope of the present conclusions in the revised manuscript.

      Finally, I turn to claim 2, that individual differences in how people infer elasticity are importantly related to psychopathology. There is much to say about the decision to treat psychopathology as a unidimensional construct. However, I will keep it concrete and simply note that CCA (by design) obscures the relationship between any two variables. Thus, as suggestive as Figure 6B is, we cannot conclude that there is a strong relationship between Sense of Agency and the elasticity bias---this result is consistent with any possible relationship (even a negative one). The fact that the direct relationship between these two variables is not shown or reported leads me to infer that they do not have a significant or strong relationship in the data.

      We agree that CCA is not designed to reveal the relationship between any two variables. However, the advantage of this analysis is that it pulls together information from multiple variables. Doing so does not treat psychopathology as unidimensional. Rather, it seeks a particular dimension that most strongly correlates with different aspects of task performance. This is especially useful for multidimensional psychopathology data because such data are often dominated by strong correlations between dimensions, whereas the research seeks to explain the distinctions between the dimensions. Similar considerations hold for the multidimensional task parameters, which although less correlated, may still jointly predict the relevant psychopathological profile better than each parameter does in isolation. Thus, the CCA enabled us to identify a general relationship between task performance and psychopathology that accounts for different symptom measures and aspects of controllability inference. 

      Using CCA can thus reveal relationships that do not readily show up in two-variable analyses. Indeed, the direct correlation between Sense of Agency (SOA) and elasticity bias was not significant – a result that, for completeness, we will now report in the supplementary materials along with all other direct correlations. We note, however, that the CCA analysis was preregistered and its results were replicated. Furthermore, an auxiliary analysis specifically confirmed the contributions of both elasticity bias (Figure 6D, bottom plot) and, although not reported in the original paper, of the Sense of Agency score (SOA; p\=.03 permutation test) to the observed canonical correlation. Participants scoring higher on the psychopathology profile also overinvested resources in inelastic environments but did not futilely invest in uncontrollable environments (Figure 6A), providing external validation to the conclusion that the CCA captured meaningful variance specific to elasticity inference. The results thus enable us to safely conclude that differences in elasticity inferences are significantly associated with a profile of controlrelated psychopathology to which SOA contributed significantly.  

      Finally, whereas interpretation of individual CCA loadings that were not specifically tested remains speculative, we note that the pattern of loadings largely replicated across the initial and replication studies (see Figure 6B), and aligns with prior findings. For instance, the positive loadings of SOA and OCD match prior suggestions that a lower sense of control leads to greater compensatory effort(7), whereas the negative loading for depression scores matches prior work showing reduced resource investment in depression(5-6).

      We will revise the text to better clarify the advantageous and disadvantageous of our analytical approach, and the conclusions that can and cannot be drawn from it.

      There is also a feature of the task that limits our ability to draw strong conclusions about individual differences in elasticity inference. As the authors clearly acknowledge, the task was designed "to be especially sensitive to overestimation of elasticity" (line 287). A straightforward consequence of this is that the resulting *empirical* estimate of estimation bias (i.e., the gamma_elasticity parameter) is itself biased. This immediately undermines any claim that references the directionality of the elasticity bias (e.g. in the abstract). Concretely, an undirected deficit such as slower learning of elasticity would appear as a directed overestimation bias. When we further consider that elasticity inference is the only meaningful learning/decisionmaking problem in the task (argued above), the situation becomes much worse. Many general deficits in learning or decision-making would be captured by the elasticity bias parameter. Thus, a conservative interpretation of the results is simply that psychopathology is associated with impaired learning and decision-making.

      We apologize for our imprecise statement that the task was ‘especially sensitive to overestimation of elasticity’, which justifiably led to Reviewer’s concern that slower elasticity learning can be mistaken for elasticity bias. To make sure this was not the case, we made use of the fact that our computational model explicitly separates bias direction (λ) from the rate of learning through two distinct parameters, which initialize the prior concentration and mean of the model’s initial beliefs concerning elasticity (see Methods pg. 22). The higher the concentration of the initial beliefs (𝜖), the slower the learning. Parameter recovery tests confirmed that our task enables acceptable recovery of both the bias λ<sub>elasticity</sub> (r=.81) and the concentration 𝝐<sub>elasticity</sub> (r=.59) parameters. And importantly, the level of confusion between the parameters was low (confusion of 0.15 for 𝝐<sub>elasticity</sub>→ λ<sub>elasticity</sub> and 0.04 for λ<sub>elasticity</sub>→ 𝝐<sub>elasticity</sub>). This result confirms that our task enables dissociating elasticity biases from the rate of elasticity learning. 

      Moreover, to validate that the minimal level of confusion existing between bias and the rate of learning did not drive our psychopathology results, we re-ran the CCA while separating concentration from bias parameters. The results (Author response image 1) demonstrate that differences in learning rate (𝜖) had virtually no contribution to our CCA results, whereas the contribution of the pure bias (𝜆) was preserved. 

      We will incorporate these clarifications and additional analysis in our revised manuscript.

      Author response image 1.

      Showing that a model parameter correlates with the data it was fit to does not provide any new information, and cannot support claims like "a prior assumption that control is likely available was reflected in a futile investment of resources in uncontrollable environments." To make that claim, one must collect independent measures of the assumption and the investment.

      We apologize if this and related statements seemed to be describing independent findings. They were merely meant to describe the relationship between model parameters and modelindependent measures of task performance. It is inaccurate, though, to say that they provide no new information, since results could have been otherwise. For instance, instead of a higher controllability bias primarily associating with futile investment of resources in uncontrollable environments, it could have been primarily associated with more proper investment of resources in high-controllability environments. Additionally, we believe these analyses are of value to readers who seek to understand the role of different parameters in the model. In our planned revision, we will clarify that the relevant analyses are merely descriptive. 

      Did participants always make two attempts when purchasing tickets? This seems to violate the intuitive model, in which you would sometimes succeed on the first jump. If so, why was this choice made? Relatedly, it is not clear to me after a close reading how the outcome of each trial was actually determined.

      We thank the reviewer for highlighting the need to clarify these aspects of the task in the revised manuscript. 

      When participants purchased two extra tickets, they attempted both jumps, and were never informed about whether either of them succeeded. Instead, after choosing a vehicle and attempting both jumps, participants were notified where they arrived at. This outcome was determined based on the cumulative probability of either of the two jumps succeeding. Success meant that participants arrived at where their chosen vehicle goes, whereas failure meant they walked to the nearest location (as determined by where they started from). 

      Though it is unintuitive to attempt a second jump before seeing whether the first succeed, this design choice ensured two key objectives. First, that participants would consistently need to invest not only more money but also more effort and time in planets with high elastic controllability. Second, that the task could potentially generalize to the many real-world situations where the amount of invested effort has to be determined prior to seeing any outcome, for instance, preparing for an exam or a job interview. 

      It should be noted that the model is heuristically defined and does not reflect Bayesian updating. In particular, it overestimates control by not using losses with less than 3 tickets (intuitively, the inference here depends on your beliefs about elasticity). I wonder if the forced three-ticket trials in the task might be historically related to this modeling choice.

      We apologize for not making this clear, but in fact losing with less than 3 tickets does reduce the model’s estimate of available control. It does so by increasing the elasticity estimates

      (a<sub>elastic≥1</sub>, a<sub>elastic2</sub> parameters), signifying that more tickets are needed to obtain the maximum available level of control, thereby reducing the average controllability estimate across ticket investment options. 

      It would be interesting to further develop the model such that losing with less than 3 tickets would also impact inferences concerning the maximum available control, depending on present beliefs concerning elasticity, but the forced three-ticket purchases already expose participants to the maximum available control, and thus, the present data may not be best suited to test such a model. These trials were implemented to minimize individual differences concerning inferences of maximum available control, thereby focusing differences on elasticity inferences. We will discuss the Reviewer’s suggestion for a potentially more accurate model in the revised manuscript. 

      References

      (1) Huys, Q. J. M., & Dayan, P. (2009). A Bayesian formulation of behavioral control. Cognition, 113(3), 314– 328.

      (2) Ligneul, R. (2021). Prediction or causation? Towards a redefinition of task controllability. Trends in Cognitive Sciences, 25(6), 431–433.

      (3) Mistry, P., & Liljeholm, M. (2016). Instrumental divergence and the value of control. Scientific Reports, 6, 36295.

      (4) Lin, J. (1991). Divergence measures based on the Shannon entropy. IEEE Transactions on Information Theory, 37(1), 145–151

      (5) Cohen RM, Weingartner H, Smallberg SA, Pickar D, Murphy DL. Effort and cognition in depression. Arch Gen Psychiatry. 1982 May;39(5):593-7. doi: 10.1001/archpsyc.1982.04290050061012. PMID: 7092490.

      (6) Bi R, Dong W, Zheng Z, Li S, Zhang D. Altered motivation of effortful decision-making for self and others in subthreshold depression. Depress Anxiety. 2022 Aug;39(8-9):633-645. doi: 10.1002/da.23267. Epub 2022 Jun 3. PMID: 35657301; PMCID: PMC9543190.

      (7) Tapal, A., Oren, E., Dar, R., & Eitam, B. (2017). The Sense of Agency Scale: A measure of consciously perceived control over one's mind, body, and the immediate environment. Frontiers in Psychology, 8, 1552

    1. Author response: 

      We thank the reviewers for their feedback on our paper. We have taken all their comments into account in revising the manuscript. We provide a point-by-point response to their comments, below.

      Reviewer #1:

      Major comments:

      The manuscript is clearly written with a level of detail that allows others to reproduce the imaging and cell-tracking pipeline. Of the 22 movies recorded one was used for cell tracking. One movie seems sufficient for the second part of the manuscript, as this manuscript presents a proof-of-principle pipeline for an imaging experiment followed by cell tracking and molecular characterisation of the cells by HCR. In addition, cell tracking in a 5-10 day time-lapse movie is an enormous time commitment.

      My only major comment is regarding "Suppl_data_5_spineless_tracking". The image file does not load.

      It looks like the wrong file is linked to the mastodon dataset. The "Current BDV dataset path" is set to "Beryl_data_files/BLB mosaic cut movie-02.xml", but this file does not exist in the folder. Please link it to the correct file.

      We have corrected the file path in the updated version of Suppl. Data 5.

      Minor comments:

      The authors state that their imaging settings aim to reduce photo damage. Do they see cell death in the regenerating legs? Is the cell death induced by the light exposure or can they tell if the same cells die between the movies? That is, do they observe cell death in the same phases of regeneration and/or in the same regions of the regenerating legs?

      Yes, we observe cell death during Parhyale leg regeneration. We have added the following sentence to explain this in the revised manuscript: "During the course of regeneration some cells undergo apoptosis (reported in Alwes et al., 2016). Using the H2B-mRFPruby marker, apoptotic cells appear as bright pyknotic nuclei that break up and become engulfed by circulating phagocytes (see bright specks in Figure 2F)."

      We now also document apoptosis in regenerated legs that have not been subjected to live imaging in a new supplementary figure (Suppl. Figure 3),  and we refer to these observations as follows: "While some cell death might be caused by photodamage, apoptosis can also be observed in similar numbers in regenerating legs that have not been subjected to live imaging (Suppl. Figure 3)."

      Based on 22 movies, the authors divide the regeneration process into three phases and they describe that the timing of leg regeneration varies between individuals. Are the phases proportionally the same length between regenerating legs or do the authors find differences between fast/slow regenerating legs? If there is a difference in the proportions, why might this be?

      Both early and late phases contribute to variation in the speed of regeneration, but there is no clear relationship between the relative duration of each phase and the speed of regeneration. We now present graphs supporting these points in a new supplementary figure (Suppl. Figure 2).  

      To clarify this point, we have added the following sentence in the manuscript: "We find that the overall speed of leg regeneration is determined largely by variation in the speed of the early (wound closure) phase of regeneration, and to a lesser extent by variation in later phases when leg morphogenesis takes place (Suppl. Figure 2 A,B). There is no clear relationship between the relative duration of each phase and the speed of regeneration (Suppl. Figure 2 A',B')."

      Based on their initial cell tracing experiment, could the authors elaborate more on what kind of biological information can be extracted from the cell lineages, apart from determining which is the progenitor of a cell? What does it tell us about the cell population in the tissue? Is there indication of multi- or pluripotent stem cells? What does it say about the type of regeneration that is taking place in terms of epimorphosis and morphallaxis, the old concepts of regeneration?

      In the first paragraph of Future Directions we describe briefly the kind of biological information that could be gained by applying our live imaging approach with appropriate cell-type markers (see below). We do not comment further, as we do not currently have this information at hand. Regarding the concepts of epimorphosis and morphallaxis, as we explain in Alwes et al. 2016, these terms describe two extreme conditions that do not capture what we observe during Parhyale leg regeneration. Our current work does not bring new insights on this topic.

      Page 5. The authors mention the possibility of identifying the cell ID based on transcriptomic profiling data. Can they suggest how many and which cell types they expect to find in the last stage based on their transcriptomic data?

      We have added this sentence: "Using single-nucleus transcriptional profiling, we have identified approximately 15 transcriptionally-distinct cell types in adult Parhyale legs (Almazán et al., 2022), including epidermis, muscle, neurons, hemocytes, and a number of still unidentified cell types."

      Page 6. Correction: "..molecular and other makers.." should be "..molecular and other markers.."

      Corrected

      Page 8. The HCR in situ protocol probably has another important advantage over the conventional in situ protocol, which is not mentioned in this study. The hybridisation step in HCR is performed at a lower temperature (37˚C) than in conventional in situ hybridisation (65˚C, Rehm et al., 2009). In other organisms, a high hybridisation temperature affects the overall tissue morphology and cell location (tissue shrinkage). A lower hybridisation temperature has less impact on the tissue and makes manual cell alignment between the live imaging movie and the fixed HCR in situ stained specimen easier and more reliable. If this is also the case in Parhyale, the authors must mention it.

      This may be correct, but all our specimens were treated at 37˚C, so we cannot assess whether hybridisation temperature affects morphological preservation in our specimens.

      Page 9. The authors should include more information on the spineless study. What been is spineless? What do the cell lineages tell about the spineless progenitors, apart from them being spread in the tissue at the time of amputation? Do spineless progenitors proliferate during regeneration? Do any spineless expressing cells share a common progenitor cell?

      We now point out that spineless encodes a transcription factor. We provide a summary of the lineages generating spineless-expressing cells in Suppl. Figure 6, and we explain that "These epidermal progenitors undergo 0, 1 or 2 cell divisions, and generate mostly spineless-expressing cells (Suppl. Figure 5)."

      Page 10. Regarding the imaging temperature, the Materials and Methods state "... a temperature control chamber set to 26 or 27˚C..."; however, in Suppl. Data 1, 26˚C and 29˚C are indicated as imaging temperatures. Which is correct?

      We corrected the Methods by adding "with the exception of dataset li51, imaged at 29°C"

      Page 10. Regarding the imaging step size, the Materials and Methods state "...step size of 1-2.46 µm..."; however, Suppl. Data 1 indicate a step size between 1.24 - 2.48 µm. Which is correct?

      We corrected the Methods.

      Page 11. Correct "...as the highest resolution data..." to "...at the highest resolution data..."

      The original text is correct ("standardised to the same dimensions as the highest resolution data").

      Page 11. Indicate which supplementary data set is referred to: "Using Mastodon, we generated ground truth annotations on the original image dataset, consisting of 278 cell tracks, including 13,888 spots and 13,610 links across 55 time points (see Supplementary Data)."

      Corrected

      p. 15. Indicate which supplementary data set is referred to: "In this study we used HCR probes for the Parhyale orthologues of futsch (MSTRG.441), nompA (MSTRG.6903) and spineless (MSTRG.197), ordered from Molecular Instruments (20 oligonucleotides per probe set). The transcript sequences targeted by each probe set are given in the Supplementary Data."

      Corrected

      Figure 3. Suggestion to the overview schematics: The authors might consider adding "molting" as the end point of the red bar (representing differentiation).

      The time of molting is not known in the majority of these datasets, because the specimens were fixed and stained prior to molting. We added the relevant information in the figure legend: "Datasets li-13 and li-16 were recorded until the molt; the other recordings were stopped before molting."

      Figure 4B': Please indicate that the nuclei signal is DAPI.

      Corrected

      Supplementary figure 1A. Word is missing in the figure legend: ...the image also shows weak…

      Corrected

      Supplementary Figure 2: Please indicate the autofluorescence in the granular cells. Does it correspond to the yellow cells?

      Corrected

      Video legend for video 1 and 2. Please correct "H2B-mREFruby" to "H2B-mRFPruby".

      Corrected

      Reviewer #2:

      Major comments:

      MC 1. Given that most of the technical advances necessary to achieve the work described in this manuscript have been published previously, it would be helpful for the authors to more clearly identify the primary novelty of this manuscript. The abstract and introduction to the manuscript focus heavily on the technical details of imaging and analysis optimization and some additional summary of the implications of these advances should be included here to aid the reader.

      This paper describes a technical advance. While previous work (Alwes et al. 2016) established some key elements of our live imaging approach, we were not at that time able to record the entire time course of leg regeneration (the longest recordings were 3.5 days long). Here we present a method for imaging the entire course of leg regeneration (up to 10 days of imaging), optimised to reduce photodamage and to improve cell tracking. We also develop a method of in situ staining in cuticularised adult legs (an important technical breakthrough in this experimental system), which we combine with live imaging to determine the fate of tracked cells. We have revised the abstract and introduction of the paper to point out these novelties, in relation to our previous publications.

      In the abstract we explain: "Building on previous work that allowed us to image different parts of the process of leg regeneration in the crustacean Parhyale hawaiensis, we present here a method for live imaging that captures the entire process of leg regeneration, spanning up to 10 days, at cellular resolution. Our method includes (1) mounting and long-term live imaging of regenerating legs under conditions that yield high spatial and temporal resolution but minimise photodamage, (2) fixing and in situ staining of the regenerated legs that were imaged, to identify cell fates, and (3) computer-assisted cell tracking to determine the cell lineages and progenitors of identified cells. The method is optimised to limit light exposure while maximising tracking efficiency."

      The introduction includes the following text: "Our first systematic study using this approach presented continuous live imaging over periods of 2-3 days, capturing key events of leg regeneration such as wound closure, cell proliferation and morphogenesis of regenerating legs with single-cell resolution (Alwes et al., 2016). Here, we extend this work by developing a method for imaging the entire course of leg regeneration, optimised to reduce photodamage and to improve cell tracking. We also develop a method of in situ staining of gene expression in cuticularised adult legs, which we combine with live imaging to determine the fate of tracked cells."

      MC 2. The description of the regeneration time course is nicely detailed but also very qualitative. A major advantage of continuous recording and automated cell tracking in the manner presented in this manuscript would be to enable deeper quantitative characterization of cellular and tissue dynamics during regeneration. Rather than providing movies and manually annotated timelines, some characterization of the dynamics of the regeneration process (the heterogeneity in this is very very interesting, but not analyzed at all) and correlating them against cellular behaviors would dramatically increase the impact of the work and leverage the advances presented here. For example, do migration rates differ between replicates? Division rates? Division synchrony? Migration orientation? This seems to be an incredibly rich dataset that would be fascinating to explore in greater detail, which seems to me to be the primary advance presented in this manuscript. I can appreciate that the authors may want to segregate some biological findings from the method, but I believe some nominal effort highlighting the quantitative nature of what this method enables would strengthen the impact of the paper and be useful for the reader. Selecting a small number of simple metrics (eg. Division frequency, average cell migration speed) and plotting them alongside the qualitative phases of the regeneration timeline that have already been generated would be a fairly modest investment of effort using tools that already exist in the Mastodon interface, I would roughly estimate on the order of an hour or two per dataset. I believe that this effort would be well worth it and better highlight a major strength of the approach.

      The primary goal of this work was to establish a robust method for continuous long-term live imaging of regeneration, but we do appreciate that a more quantitative analysis would add value to the data we are presenting. We tried to address this request in three steps:

      First, we examined whether clear temporal patterns in cell division, cell movements or other cellular features can be observed in an accurately tracked dataset (li13-t4, tracked in Sugawara et al. 2022). To test this we used the feature extraction functions now available on the Mastodon platform (see link). We could discern a meaningful temporal pattern for cell divisions (see below); the other features showed no interpretable pattern of variation.

      Second, we asked whether we could use automated cell tracking to analyse the patterns of cell division in all our datasets. Using an Elephant deep learning model trained on the tracks of the li13-t4 dataset, we performed automated cell tracking in the same dataset, and compared the pattern of cell divisions from the automated cell track predictions with those coming from manually validated cell tracks. We observed that the automated tracks gave very imprecise results, with a high background of false positives obscuring the real temporal pattern (see images below, with validated data on the left, automated tracking on the right). These results show that the automated cell tracking is not accurate enough to provide a meaningful picture on the pattern of cell divisions.

      Third, we tried to improve the accuracy of detection of dividing cells by additional training of Elephant models on each dataset (to lower the rate of false positives), followed by manual proofreading. Given how labour intensive this is, we could only apply this approach to 4 additional datasets. The results of this analysis are presented in Figure 4.

      Author response image 1.

      MC 3. The authors describe the challenges faced by their described approach:

      Using this mode of semi-automated and manual cell tracking, we find that most cells in the upper slices of our image stacks (top 30 microns) can be tracked with a high degree of confidence. A smaller proportion of cell lineages are trackable in the deeper layers.

      Given that the authors quantify this in Table 1, it would aid the reader to provide metrics in the manuscript text at this point. Furthermore, the metrics provided in Table 1 appear to be for overall performance, but the text describes that performance appears to be heavily depth dependent. Segregating the performance metrics further, for example providing DET, TRA, precision and recall for superficial layers only and for the overall dataset, would help support these arguments and better highlight performance a potential adopter of the method might expect.

      In the revised manuscript we have added data on the tracking performance of Elephant in relation to imaging depth in Suppl. Figure 3. These data confirm our original statement (which was based on manual tracking) that nuclei are more challenging to track in deeper layers.

      We point to these new results in two parts of the paper, as follows: "A smaller proportion of cells are trackable in the deeper layers (see Suppl. Figure 3)", and "Our results, summarised in Table 1A, show that the detection of nuclei can be enhanced by doubling the z resolution at the expense of xy resolution and image quality. This improvement is particularly evident in the deeper layers of the imaging stacks, which are usually the most challenging to track (Suppl. Figure 3)."

      MC 4. Performance characterization in Table 1 appears to derive from a single dataset that is then subsampled and processed in different ways to assess the impact of these changes on cell tracking and detection performance. While this is a suitable strategy for this type of optimization it leaves open the question of performance consistency across datasets. I fully recognize that this type of quantification can be onerous and time consuming, but some attempt to assess performance variability across datasets would be valuable. Manual curation over a short time window over a random sampling of the acquired data would be sufficient to assess this.

      We think that similar trade-offs will apply to all our datasets because tracking performance is constrained by the same features, which are intrinsic to our system; e.g. by the crowding of nuclei in relation to axial resolution, or the speed of mitosis in relation to the temporal resolution of imaging. We therefore do not see a clear rationale for repeating this analysis. On a practical level, our existing image datasets could not be subsampled to generate the various conditions tested in Table 1, so proving this point experimentally would require generating new recordings, and tracking these to generate ground truth data. This would require months of additional work.

      A second, related question is whether Elephant would perform equally well in detecting and tracking nuclei across different datasets. This point has been addressed in the Sugawara et al. 2022 paper, where the performance of Elephant was tested on diverse fluorescence datasets.

      Reviewer #3:

      Major comments:

      • The authors should clearly specify what are the key technical improvements compared to their previous studies (Alwes et al. 2016, Elife; Konstantinides & Averof 2014, Science). There, the approaches for mounting, imaging, and cell tracking are already introduced, and the imaging is reported to run for up to 7 days in some cases.

      In Konstantinides and Averof (2014) we did not present any live imaging at cellular resolution. In Alwes et al. (2016) we described key elements of our live imaging approach, but we were never able to record the entire time course of leg regeneration. The longest recordings in that work were 3.5 days long.

      We have revised the abstract and introduction to clarify the novelty of this work, in relation to our previous publications. Please see our response to comment MC1 of reviewer 2.

      • While the authors mention testing the effect of imaging parameters (such as scanning speed and line averaging) on the imaging/tracking outcome, very little or no information is provided on how this was done beyond the parameters that they finally arrived to.

      Scan speed and averaging parameters were determined by measuring contrast and signal-to-noise ratios in images captured over a range of settings. We have now added these data in Supplementary Figure 1.

      • The authors claim that, using the acquired live imaging data across entire regeneration time course, they are now able to confirm and extend their description of leg regeneration. However, many claims about the order and timing of various cellular events during regeneration are supported only by references to individual snapshots in figures or supplementary movies. Presenting a more quantitative description of cellular processes during regeneration from the acquired data would significantly enhance the manuscript and showcase the usefulness of the improved workflow.

      The events we describe can be easily observed in the maximum projections, available in Suppl. Data 2. Regarding the quantitative analysis, please see our response to comment MC2 of reviewer 2.  

      • Table 1 summarizes the performance of cell tracking using simulated datasets of different quality. However only averages and/or maxima are given for the different metrics, which makes it difficult to evaluate the associated conclusions. In some cases, only 1 or 2 test runs were performed.

      The metrics extracted from each of the three replicates, per dataset, are now included in Suppl. Data 4.

      We consistently used 3 replicates to measure tracking performance with each of the datasets. The "replicates" column label in Table 1 referred to the number of scans that were averaged to generate the image, not to the replicates used for estimating the tracking performance. To avoid confusion, we changed that label to "averaging".

      • OPTIONAL: An imaging approach that allows using the current mounting strategy but could help with some of the tradeoffs is using a spinning-disk confocal microscope instead of a laser scanning one. If the authors have such a system available, it could be interesting to compare it with their current scanning confocal setup.

      Preliminary experiments that we carried out several years ago on a spinning disk confocal (with a 20x objective and the CSU-W1 spinning disk) were not very encouraging, and we therefore did not pursue this approach further. The main problem was bad image quality in deeper tissue layers.

      Minor comments:

      • The presented imaging protocol was optimized for one laser wavelength only (561 nm) - this should be mentioned when discussing the technical limitations since animals tend to react differently to different wavelengths. Same settings might thus not be applicable for imaging a different fluorescent protein.

      In the second paragraph of the Results section, we explain that we perform the imaging at long wavelengths in order to minimise photodamage. It should be clear to the readers that changing the excitation wavelength will have an impact for long-term live imaging.

      • For transferability, it would be useful if the intensity of laser illumination was measured and given in the Methods, instead of just a relative intensity setting from the imaging software. Similarly,more details of the imaging system should be provided where appropriate (e.g., detector specifications).

      We have now measured the intensity of the laser illumination and added this information in the

      Methods: "Laser power was typically set to 0.3% to 0.8%, which yields 0.51 to 1.37 µW at 561 nm (measured with a ThorLabs Microscope Slide Power Sensor, #S170C)."

      Regarding the imaging system and the detector, we provide all the information that is available to us on the microscope's technical sheets.

      • The versions of analysis scripts associated with the manuscript should be uploaded to an online repository that permanently preserves the respective version.

      The scripts are now available on gitbub and online repositories. The relevant links are included in the revised manuscript.

    1. Reviewer #2 (Public Review):

      Summary:

      The goal of the authors in this study is to develop a more reliable approach for quantifying codon usage such that it is more comparable across species. Specifically, the authors wish to estimate the degree of adaptive codon usage, which is potentially a general proxy for the strength of selection at the molecular level. To this end, the authors created the Codon Adaptation Index for Species (CAIS) that controls for differences in amino acid usage and GC% across species. Using their new metric, the authors find a previously unobserved negative correlation between the overall adaptiveness of codon usage and body size across 118 vertebrates. As body size is negatively correlated with effective population size and thus the general strength of natural selection, the negative correlation between CAIS and body size is expected. The authors argue this was previously unobserved due to failures of other popular metrics such as Codon Adaptation Index (CAI) and the Effective Number of Codons (ENC) to adequately control for differences in amino acid usage and GC content across species. Most surprisingly, the authors also find a positive relationship between CAIS and the overall "disorderedness" of a species protein domains. As some of these results are unexpected, which is acknowledged by the authors, I think it would be particularly beneficial to work with some simulated datasets. I think CAIS has the potential to be a valuable tool for those interested in comparing codon adaptation across species in certain situations. However, I have certain theoretical concerns about CAIS as a direct proxy for the efficiency of selection when the mutation bias changes across species.

      Strengths:

      (1) I appreciate that the authors recognize the potential issues of comparing CAI when amino acid usage varies and correct for this in CAIS. I think this is sometimes an under-appreciated point in the codon usage literature, as CAI is a relative measure of codon usage bias (i.e. only considers synonyms). However, the strength of natural selection on codon usage can potentially vary across amino acids, such that comparing mean CAI between protein regions with different amino acid biases may result in spurious signals of statistical significance (see Cope et al. Biochemica et Biophysica Acta - Biomembranes 2018 for a clear example of this).

      (2) The authors present numerous analysis using both ENC and mean CAI as a comparison to CAIS, helping given a sense of how CAIS corrects for some of the issues with these other metrics. I also enjoyed that they examined the previously unobserved relationship between codon usage bias and body size, which has bugged me ever since I saw Kessler and Dean 2014. The result comparing protein disorder to CAIS was particularly interesting and unexpected.

      (3) The CAIS metric presented here is generally applicable to any species that has an annotated genome with protein-coding sequences.

      Weaknesses:

      (1) The main weakness of this work is that it lacks simulated data to confirm that it works as expected. This would be particularly useful for assessing the relationship between CAIS and the overall effect of protein structure disorder, which the authors acknowledge is an unexpected result. I think simulations could also allow the authors to assess how their metric performs in situations where mutation bias and natural selection act in the same direction vs. opposite directions. Additionally, although I appreciate their comparisons to ENC and mean CAI, the lack of comparison to other popular codon metrics for calculating the overall adaptiveness of a genome (e.g. dos Reis et al.'s statistic, which is a function of tRNA Adaptation Index (tAI) and ENC) may be more appropriate. Even if results are similar to , CAIS has a noted advantage that it doesn't require identifying tRNA gene copy numbers or abundances, which I think are generally less readily available than genomic GC% and protein-coding sequences.

      The authors mention the selection-mutation-drift equilibrium model, which underlies the basic ideas of this work (e.g. higher results in stronger selection on codon usage), but a more in-depth framing of CAIS in terms of this model is not given. I think this could be valuable, particularly in addressing the question "are we really estimating what we think we're estimating?"

      Let's take a closer look at the formulation for RSCUS. From here on out, subscripts will only be used to denote the codon and it will be assumed that we are only considering the case of for some species

      I think what the authors are attempting to do is "divide out" the effects of mutation bias (as given by , such that only the effects of natural selection remain, i.e. deviations from the expected frequency based on mutation bias alone represent adaptive codon usage. Consider Gilchrist et al. MBE 2015, which says that the expected frequency of codon at selection-mutation-drift equilibrium in gene for an amino acid with synonymous codons is

      where is the mutation bias, is the strength of selection scaled by the strength of drift, and is the gene expression level of gene \(g\). In this case, \ and reflect the strength and direction of mutation bias and natural selection relative to a reference codon, for which . Assuming the selection-mutation-drift equilibrium model is generally adequate to model the true codon usage patterns in a genome (as I do and I think the authors do, too), the could be considered the expected observed frequency codon in gene .

      Let's re-write the in the form of Gilchrist et al., such that it is a function of mutation bias . For simplicity, we will consider just the two-codon case and assume the amino acid sequence is fixed. Assuming GC% is at equilibrium, the term and can be written as

      where is the mutation rate from nucleotides to. As described in Gilchrist et al. MBE 2015 and Shah and Gilchrist PNAS 2011, the mutation bias . This can be expressed in terms of the equilibrium GC content by recognizing that

      As we are assuming the amino acid sequence is fixed, the probability of observing a synonymous codon at an amino acid becomes just a Bernoulli process.

      If we do this, then

      Recall that in the Gilchrist et al. framework, the reference codon has . Thus, we have recovered the Gilchrist et al. model from the formulation of under the assumption that natural selection has no impact on codon usage and codon NNG is the pre-defined reference codon. To see this, plug in 0 for in equation (1).

      We can then calculate the expected RSCUS using equation (1) (using notation and equation (6) for the two codon case. For simplicity assume, we are only considering a gene of average expression (defined as . Assume in this case that NNG is the reference codon .

      This shows that the expected value of RSCUS for a two-codon amino acid is expected to increase as the strength of selection increases, which is desired. Note that in Gilchrist et al. is formulated in terms of selection against a codon relative to the reference, such that a negative value represents that a codon is favored relative to the reference. If (i.e. selection does not favor either codon), then . Also note that the expected RSCUS does not remain independent of the mutation bias. This means that even if (i.e. the strength of natural selection) does not change between species, changes to the strength and direction of mutation bias across species could impact RSCUS. Assuming my math is right, I think one needs to be cautious when interpreting CAIS as representative of the differences in the efficiency of selection across species except under very particular circumstances. One such case could be when it is known that mutation bias varies little across the species of interest. Looking at the species used in this manuscript, most of them have a GC content ranging around 0.41, so I suspect their results are okay.

      Although I have not done so, I am sure this could be extended to the 4 and 6 codon amino acids.

      Another minor weakness of this work is that although the method is generally applicable to any species with an annotated genome and the code is publicly available, the code itself contains hard-coded values for GC% and amino acid frequencies across the 118 vertebrates. The lack of a more flexible tool may make it difficult for less computationally-experienced researchers to take advantage of this method.

    1. Author response:

      Public Reviews:

      Reviewer #1 (Public review):

      Summary:

      The image analysis pipeline is tested in analysing microscopy imaging data of gastruloids of varying sizes, for which an optimised protocol for in toto image acquisition is established based on whole mount sample preparation using an optimal refractive index matched mounting media, opposing dual side imaging with two-photon microscopy for enhanced laser penetration, dual view registration, and weighted fusion for improved in toto sample data representation. For enhanced imaging speed in a two-photon microscope, parallel imaging was used, and the authors performed spectral unmixing analysis to avoid issues of signal cross-talk.

      In the image analysis pipeline, different pre-treatments are done depending on the analysis to be performed (for nuclear segmentation - contrast enhancement and normalisation; for quantitative analysis of gene expression - corrections for optical artifacts inducing signal intensity variations). Stardist3D was used for the nuclear segmentation. The study analyses into properties of gastruloid nuclear density, patterns of cell division, morphology, deformation, and gene expression.

      Strengths:

      The methods developed are sound, well described, and well-validated, using a sample challenging for microscopy, gastruloids. Many of the established methods are very useful (e.g. registration, corrections, signal normalisation, lazy loading bioimage visualisation, spectral decomposition analysis), facilitate the development of quantitative research, and would be of interest to the wider scientific community.

      We thank the reviewer for this positive feedback.

      Weaknesses:

      A recommendation should be added on when or under which conditions to use this pipeline.

      We thank the reviewer for this valuable feedback, which will be addressed in the revision. In general, the pipeline is applicable to any tissue, but it is particularly useful for large and dense 3D samples—such as organoids, embryos, explants, spheroids, or tumors—that are typically composed of multiple cell layers and have a thickness greater than 50 µm.

      The processing and analysis pipeline are compatible with any type of 3D imaging data (e.g. confocal, 2 photon, light-sheet, live or fixed).

      - Spectral unmixing to remove signal cross-talk of multiple fluorescent targets is typically more relevant in two-photon imaging due to the broader excitation spectra of fluorophores compared to single-photon imaging. In confocal or light-sheet microscopy, alternating excitation wavelengths often circumvents the need for unmixing. Spectral decomposition performs even better with true spectral detectors; however, these are usually not non-descanned detectors, which are more appropriate for deep tissue imaging. Our approach demonstrates that simultaneous cross-talk-free four-color two-photon imaging can be achieved in dense 3D specimen with four non-descanned detectors and co-excitation by just two laser lines. Depending on the dispersion in optically dense samples, depth-dependent apparent emission spectra need to be considered.

      - Nuclei segmentation using our trained StarDist3D model is applicable to any system under two conditions: (1) the nuclei exhibit a star-convex shape, as required by the StarDist architecture, and (2) the image resolution is sufficient in XYZ to allow resampling. The exact sampling required is object- and system-dependent, but the goal is to achieve nearly isotropic objects with diameters of approximately 15 pixels while maintaining image quality. In practice, images containing objects that are natively close to or larger than 15 pixels in diameter should segment well after resampling. Conversely, images with objects that are significantly smaller along one or more dimensions will require careful inspection of the segmentation results.

      - Normalization is broadly applicable to multicolor data when at least one channel is expected to be ubiquitously expressed within its domain. Wavelength-dependent correction requires experimental calibration using either an ubiquitous signal at each wavelength. Importantly, this calibration only needs to be performed once for a given set of experimental conditions (e.g., fluorophores, tissue type, mounting medium).

      - Multi-scale analysis of gene expression and morphometrics is applicable to any 3D multicolor image. This includes both the 3D visualization tools (Napari plugins) and the various analytical plots (e.g., correlation plots, radial analysis). Multi-scale analysis can be performed even with imperfect segmentation, as long as segmentation errors tend to cancel out when averaged locally at the relevant spatial scale. However, systematic errors—such as segmentation uncertainty along the Z-axis due to strong anisotropy—may accumulate and introduce bias in downstream analyses. Caution is advised when analyzing hollow structures (e.g., curved epithelial monolayers with large cavities), as the pipeline was developed primarily for 3D bulk tissues, and appropriate masking of cavities would be needed.

      Reviewer #2 (Public review):

      Summary:

      This study presents an integrated experimental and computational pipeline for high-resolution, quantitative imaging and analysis of gastruloids. The experimental module employs dual-view two-photon spectral imaging combined with optimized clearing and mounting techniques to image whole-mount immunostained gastruloids. This approach enables the acquisition of comprehensive 3D images that capture both tissue-scale and single-cell level information.

      The computational module encompasses both pre-processing of acquired images and downstream analysis, providing quantitative insights into the structural and molecular characteristics of gastruloids. The pre-processing pipeline, tailored for dual-view two-photon microscopy, includes spectral unmixing of fluorescence signals using depth-dependent spectral profiles, as well as image fusion via rigid 3D transformation based on content-based block-matching algorithms. Nuclei segmentation was performed using a custom-trained StarDist3D model, validated against 2D manual annotations, and achieving an F1 score of 85+/-3% at a 50% intersection-over-union (IoU) threshold. Another custom-trained StarDist3D model enabled accurate detection of proliferating cells and the generation of 3D spatial maps of nuclear density and proliferation probability. Moreover, the pipeline facilitates detailed morphometric analysis of cell density and nuclear deformation, revealing pronounced spatial heterogeneities during early gastruloid morphogenesis.

      All computational tools developed in this study are released as open-source, Python-based software.

      Strengths:

      The authors applied two-photon microscopy to whole-mount deep imaging of gastruloids, achieving in toto visualization at single-cell resolution. By combining spectral imaging with an unmixing algorithm, they successfully separated four fluorescent signals, enabling spatial analysis of gene expression patterns.

      The entire computational workflow, from image pre-processing to segmentation with a custom-trained StarDist3D model and subsequent quantitative analysis, is made available as open-source software. In addition, user-friendly interfaces are provided through the open-source, community-driven Napari platform, facilitating interactive exploration and analysis.

      We thank the reviewer for this positive feedback.

      Weaknesses:

      The computational module appears promising. However, the analysis pipeline has not been validated on datasets beyond those generated by the authors, making it difficult to assess its general applicability.

      We agree that applying our analysis pipeline to published datasets—particularly those acquired with different imaging systems—would be valuable. However, only a few high-resolution datasets of large organoid samples are publicly available, and most of these either lack multiple fluorescence channels or represent 3D hollow structures. Our computational pipeline consists of several independent modules: spectral filtering, dual-view registration, local contrast enhancement, 3D nuclei segmentation, image normalization based on a ubiquitous marker, and multiscale analysis of gene expression and morphometrics.

      Spectral filtering has already been applied in other systems (e.g. [7] and [8]), but is here extended to account for imaging depth-dependent apparent emission spectra of the different fluorophores. In our pipeline, we provide code to run spectral filtering on multichannel images, integrated in Python. In order to apply the spectral filtering algorithm utilized here, spectral patterns of each fluorophore need to be calibrated as a function of imaging depth, which depend on the specific emission windows and detector settings of the microscope.

      Image normalization using a wavelength-dependent correction also requires calibration on a given imaging setup to measure the difference in signal decay among the different fluorophores species. To our knowledge, the calibration procedures for spectral-filtering and our image-normalization approach have not been performed previously in 3D samples, which is why validation on published datasets is not readily possible. Nevertheless, they are described in detail in the Methods section, and the code used—from the calibration measurements to the corrected images—is available open-source at the Zenodo link in the manuscript.

      Dual-view registration, local contrast enhancement, and multiscale analysis of gene expression and morphometrics are not limited to organoid data or our specific imaging modalities. If we identify suitable datasets to validate these modules, we will include them in the revised manuscript.

      To evaluate our 3D nuclei segmentation model, we plan to test it on diverse systems, including gastruloids stained with the nuclear marker Draq5 from Moos et al. [1]; breast cancer spheroids; primary ductal adenocarcinoma organoids; human colon organoids and HCT116 monolayers from Ong et al. [2]; and zebrafish tissues imaged by confocal microscopy from Li et al [3]. These datasets were acquired using either light-sheet or confocal microscopy, with varying imaging parameters (e.g., objective lens, pixel size, staining method).

      Preliminary results are promising (see Author response image 1). We will provide quantitative comparisons of our model’s performance on these datasets, using annotations or reference predictions provided by the original authors where available.

      Author response image 1.

      Qualitative comparison of our custom Stardist3D segmentation strategy on diverse published 3D nuclei datasets. We show one slice from the XY plane for simplicity. (a) Gastruloid stained with the nuclear marker DRAQ5 imaged with an open-top dual-view and dual-illumination LSM [1]. (b) Breast cancer spheroid [2]. (c) Primary pancreatic ductal adenocarcinoma organoids imaged with confocal microscopy[2]. (d) Human colon organoid imaged with LSM laser scanning confocal microscope [2]. (e) Monolayer HCT116 cells imaged with LSM laser scanning confocal microscope [2]. (f) Fixed zebrafish embryo stained for nuclei and imaged with a Zeiss LSM 880 confocal microscopy [3].

      Besides, the nuclei segmentation component lacks benchmarking against existing methods.

      We agree with the reviewer that a benchmark against existing segmentation methods would be very useful. We tried different pre-trained models:

      - CellPose, which we tested in a previous paper ([4]) and which showed poor performances compared to our trained StarDist3D model.

      - DeepStar3D ([2]) is only available in the software 3DCellScope. We could not benchmark the model on our data, because the free and accessible version of the software is limited to small datasets. An image of a single whole-mount gastruloid with one channel, having dimensions (347,467,477) was too large to be processed, see screenshot below. The segmentation model could not be extracted from the source code and tested externally because the trained DeepStar3D weights are encrypted.

      Author response image 2.

      Screenshot of the 3DCellScore software. We could not perform 3D nuclei segmentation of a whole-mount gastruloids because the image size was too large to be processed.

      - AnyStar ([5]), which is a model trained from the StarDist3D architecture, was not performing well on our data because of the heterogeneous stainings. Basic pre-processing such as median and gaussian filtering did not improve the results and led to wrong segmentation of touching nuclei. AnyStar was demonstrated to segment well colon organoids in Ong et al, 2025 ([2]), but the nuclei were more homogeneously stained. Our Hoechst staining displays bright chromatin spots that are incorrectly labeled as individual nuclei.

      - Cellos ([6]), another model trained from StarDist3D, was also not performing well. The objects used for training and to validate the results are sparse and not touching, so the predicted segmentation has a lot of false negatives even when lowering the probability threshold to detect more objects. Additionally, the network was trained with an anisotropy of (9,1,1), based on images with low z resolution, so it performed poorly on almost isotropic images. Adapting our images to the network’s anisotropy results in an imprecise segmentation that can not be used to measure 3D nuclei deformations.

      We tried both Cellos and AnyStar predictions on a gastruloid image from Fig. S2 of our main manuscript. Author response image 3 displays the results qualitatively compared to our trained model Stardist-tapenade. For the revision of the paper, we will perform a comprehensive benchmark of these state-of-the-art routines, including quantitative assessment of the performance.

      Author response image 3.

      Qualitative comparison of two published segmentation models versus our model. We show one slice from the XY plane for simplicity. Segmentations are displayed with their contours only. (Top left) Gastruloid stained with Hoechst, image extracted from Fig S2 of our manuscript. (Top right) Same image overlayed with the prediction from the Cellos model, showing many false negatives. (Bottom left) Same image overlayed with the prediction from our Stardist-tapenade model. (Bottom right) Same image overlayed with the prediction from the AnyStar model, false positives are indicated with a red arrow.

      Appraisal:

      The authors set out to establish a quantitative imaging and analysis pipeline for gastruloids using dual-view two-photon microscopy, spectral unmixing, and a custom computational framework for 3D segmentation and gene expression analysis. This aim is largely achieved. The integration of experimental and computational modules enables high-resolution in toto imaging and robust quantitative analysis at the single-cell level. The data presented support the authors' conclusions regarding the ability to capture spatial patterns of gene expression and cellular morphology across developmental stages.

      Impact and utility:

      This work presents a compelling and broadly applicable methodological advance. The approach is particularly impactful for the developmental biology community, as it allows researchers to extract quantitative information from high-resolution images to better understand morphogenetic processes. The data are publicly available on Zenodo, and the software is released on GitHub, making them highly valuable resources for the community.

      We thank the reviewer for these positive feedbacks.

      Reviewer #3 (Public review):

      Summary

      The paper presents an imaging and analysis pipeline for whole-mount gastruloid imaging with two-photon microscopy. The presented pipeline includes spectral unmixing, registration, segmentation, and a wavelength-dependent intensity normalization step, followed by quantitative analysis of spatial gene expression patterns and nuclear morphometry on a tissue level. The utility of the approach is demonstrated by several experimental findings, such as establishing spatial correlations between local nuclear deformation and tissue density changes, as well as the radial distribution pattern of mesoderm markers. The pipeline is distributed as a Python package, notebooks, and multiple napari plugins.

      Strengths

      The paper is well-written with detailed methodological descriptions, which I think would make it a valuable reference for researchers performing similar volumetric tissue imaging experiments (gastruloids/organoids). The pipeline itself addresses many practical challenges, including resolution loss within tissue, registration of large volumes, nuclear segmentation, and intensity normalization. Especially the intensity decay measurements and wavelength-dependent intensity normalization approach using nuclear (Hoechst) signal as reference are very interesting and should be applicable to other imaging contexts. The morphometric analysis is equally well done, with the correlation between nuclear shape deformation and tissue density changes being an interesting finding. The paper is quite thorough in its technical description of the methods (which are a lot), and their experimental validation is appropriate. Finally, the provided code and napari plugins seem to be well done (I installed a selected list of the plugins and they ran without issues) and should be very helpful for the community.

      We thank the reviewer for his positive feedback and appreciation of our work.

      Weaknesses

      I don't see any major weaknesses, and I would only have two issues that I think should be addressed in a revision:

      (1) The demonstration notebooks lack accompanying sample datasets, preventing users from running them immediately and limiting the pipeline's accessibility. I would suggest to include (selective) demo data set that can be used to run the notebooks (e.g. for spectral unmixing) and or provide easily accessible demo input sample data for the napari plugins (I saw that there is some sample data for the processing plugin, so this maybe could already be used for the notebooks?).

      We thank the reviewer for this relevant suggestion. The 7 notebooks were updated to automatically download sample tests. The different parts of the pipeline can now be run immediately: https://github.com/GuignardLab/tapenade/tree/chekcs_on_notebooks/src/tapenade/notebooks

      (2) The results for the morphometric analysis (Figure 4) seem to be only shown in lateral (xy) views without the corresponding axial (z) views. I would suggest adding this to the figure and showing the density/strain/angle distributions for those axial views as well.

      We agree with the reviewer that a morphometric analysis based on the axial views would be informative and plan to perform this analysis for the revision.

      (1) Moos, F., Suppinger, S., de Medeiros, G., Oost, K.C., Boni, A., Rémy, C., Weevers, S.L., Tsiairis, C., Strnad, P. and Liberali, P., 2024. Open-top multisample dual-view light-sheet microscope for live imaging of large multicellular systems. Nature Methods, 21(5), pp.798-803.

      (2) Ong, H.T., Karatas, E., Poquillon, T., Grenci, G., Furlan, A., Dilasser, F., Mohamad Raffi, S.B., Blanc, D., Drimaracci, E., Mikec, D. and Galisot, G., 2025. Digitalized organoids: integrated pipeline for high-speed 3D analysis of organoid structures using multilevel segmentation and cellular topology. Nature Methods, 22(6), pp.1343-1354.

      (3) Li, L., Wu, L., Chen, A., Delp, E.J. and Umulis, D.M., 2023. 3D nuclei segmentation for multi-cellular quantification of zebrafish embryos using NISNet3D. Electronic Imaging, 35, pp.1-9.

      (4) Vanaret, J., Dupuis, V., Lenne, P. F., Richard, F., Tlili, S., & Roudot, P. (2023). A detector-independent quality score for cell segmentation without ground truth in 3D live fluorescence microscopy. IEEE Journal of Selected Topics in Quantum Electronics, 29(4: Biophotonics), 1-12.

      (5) Dey, N., Abulnaga, M., Billot, B., Turk, E. A., Grant, E., Dalca, A. V., & Golland, P. (2024). AnyStar: Domain randomized universal star-convex 3D instance segmentation. In Proceedings of the IEEE/CVF Winter Conference on Applications of Computer Vision (pp. 7593-7603).

      (6) Mukashyaka, P., Kumar, P., Mellert, D. J., Nicholas, S., Noorbakhsh, J., Brugiolo, M., ... & Chuang, J. H. (2023). High-throughput deconvolution of 3D organoid dynamics at cellular resolution for cancer pharmacology with Cellos. Nature Communications, 14(1), 8406.

      (7) Rakhymzhan, A., Leben, R., Zimmermann, H., Günther, R., Mex, P., Reismann, D., ... & Niesner, R. A. (2017). Synergistic strategy for multicolor two-photon microscopy: application to the analysis of germinal center reactions in vivo. Scientific reports, 7(1), 7101.

      (8) Dunsing, V., Petrich, A., & Chiantia, S. (2021). Multicolor fluorescence fluctuation spectroscopy in living cells via spectral detection. Elife, 10, e69687.

    1. Author response:

      Reviewer 1:

      There are no significant weaknesses to signal in the manuscript. However, in order to fully conclude that there is no obvious advantage for the linguistic dimension in neonates, it would have been most useful to test a third condition in which the two dimensions were pitted against each other, that is, in which they provide conflicting information as to the boundaries of the words comprised in the artificial language. This last condition would have allowed us to determine whether statistical learning weighs linguistic and non-linguistic features equally, or whether phonetic content is preferentially processed.

      We appreciate the reviewers' suggestion that a stream with conflicting information would provide valuable insights. In the present study, we started with a simpler case involving two orthogonal features (i.e., phonemes and voices), with one feature being informative and the other uninformative, and we found similar learning capacities for both. Future work should explore whether infants—and humans more broadly—can simultaneously track regularities in multiple speech features. However, creating a stream with two conflicting statistical structures is challenging. To use neural entrainment, the two features must lead to segmentation at different chunk sizes so that their effects lead to changes in power/PLV at different frequencies—for instance, using duplets for the voice dimension and triplets for the linguistic dimension  (or vice versa). Consequently, the two dimensions would not be directly comparable within the same participant in terms of the number of distinguishable syllables/voices, memory demand, or SNR given the 1/F decrease in amplitude of background EEG activity. This would involve comparisons between two distinct groups counter-balancing chunk size and linguistic non-linguistic dimension. Considering the test phase, words for one dimension would have been part-words for the other dimension. As we are measuring differences and not preferences, interpreting the results would also have been difficult. Additionally, it may be difficult to find a sufficient number of clearly discriminable voices for such a design (triplets imply 12 voices). Therefore, an entirely different experimental paradigm would need to be developed.

      If such a design were tested, one possibility is that the regularities for the two dimensions are calculated in parallel, in line with the idea that the calculation of statistical regularities is a ubiquitous implicit mechanism (see Benjamin et al., 2024, for a proposed neural mechanism). Yet, similar to our present study, possibly only phonetic features would be used as word candidates. Another possibility is that only one informative feature would be explicitly processed at a time due to the serial nature of perceptual awareness, which may prioritise one feature over the other.

      Note: The reviewer’s summary contains a typo: syllabic rate (4 Hz) –not 2 Hz, and word rate (2 Hz) –not 4 Hz.

      Reviewer 2:

      N400: I am skeptical regarding the interpretation of the phoneme-specific ERP effect as a precursor of the N400 and would suggest toning it down. While the authors are correct in that infant ERP components are typically slower and more posterior compared to adult components, and the observed pattern is hence consistent with an adult N400, at the same time, it could also be a lot of other things. On a functional level, I can't follow the author's argument as to why a violation in phoneme regularity should elicit an N400, since there is no evidence for any semantic processing involved. In sum, I think there is just not enough evidence from the present paradigm to confidently call it an N400.

      The reviewer is correct that we cannot definitively determine the type of processing reflected by the ERP component that appears when neonates hear a triplet after exposure to a stream with phonetic regularities. We interpreted this component as a precursor to the N400, based on prior findings in speech segmentation tasks without semantic content, where a ~400 ms component emerged when adult participants recognised pseudowords (Sander et al., 2002) or during structured streams of syllables (Cunillera et al., 2006, 2009). Additionally, the component we observed had a similar topography and timing to those labelled as N400 in infant studies, where semantic processing was involved (Parise et al., 2010; Friedrich & Friederici, 2011).

      Given our experimental design, the difference we observed must be related to the type of regularity during familiarisation (either phonemes or voices). Thus, we interpreted this component as reflecting lexical search— a process which could be triggered by a linguistic structure but which would not be relevant to a non-linguistic regularity such as voices. However, we are open to alternative interpretations. In any case, this difference between the two streams reveals that computing regularities based on phonemes versus voices does not lead to the same processes. We will revise and tone down the corresponding part of the discussion to clarify that it is just a possible interpretation of the results.  

      Female and male voices: Why did the authors choose to include male and female voices? While using both female and male stimuli of course leads to a higher generalizability, it also introduces a second dimension for one feature that is not present for this other (i.e., phoneme for Experiment 1 and voice identity plus gender for Experiment 2). Hence, couldn't it also be that the infants extracted the regularity with which one gender voice followed the other? For instance, in List B, in the words, one gender is always followed by the other (M-F or F-M), while in 2/3 of the part-words, the gender is repeated (F-F and M-M). Wouldn't you expect the same pattern of results if infants learned regularities based on gender rather than identity?

      We used three female and three male voices to maximise acoustic variability. The streams were synthesised using MBROLA, which provides a limited set of artificial voices. Indeed, there were not enough French voices of acceptable quality, so we also used two Italian voices (the phonemes used existed in both Italian and French).

      Voices differ in timbre, and female voices tend to be higher pitched. However, it is sometimes difficult to categorise low-pitched female voices and high-pitched male voices. Given that gender may be an important factor in infants' speech perception (newborns, for instance, prefer female voices at birth), we conducted tests to assess whether this dimension could have influenced our results.  

      We first quantified the transitional probabilities matrices during the structured stream of Experiment 2, considering that there are only two types of voices: Female and Male.  

      For List A, all transition probabilities are equal to 0.5 (P(M|F), P(F|M), P(M|M), P(F|F)), resulting in flat TPs throughout the stream (see Author response image 1, top). Therefore, we would not expect neural entrainment at the word rate (2 Hz), nor would we anticipate ERP differences between the presented duplets in the test phase.

      For List B, P(M|F)=P(F|M)=0.66 while P(M|M)=P(F|F)=0.33. However, this does not produce a regular pattern of TP drops throughout the stream (see Author response image 1, bottom). As a result, strong neural entrainment at 2 Hz was unlikely, although some degree of entrainment might have occasionally occurred due to some drops occurring at a 2 Hz frequency. Regarding the test phase, all three Words and only one Part-word presented alternating patterns (TP=0.6). Therefore, the difference in the ERPs between Words and Partwords in List B might be attributed to gender alternation.  

      However, it seems unlikely that gender alternation alone explains the entire pattern of results, as the effect is inconsistent and appears in only one of the lists. To rule out this possibility, we analysed the effects in each list separately.

      Author response image 1.

      Transition probabilities (TPs) across the structured stream in Experiment 2, considering voices processed by gender (Female or Male). Top: List A. Bottom: List B.

      We computed the mean activation within the time windows and electrodes of interest and compared the effects of word type and list using a two-way ANOVA. For the difference between Words and Part-words over the positive cluster, we observed a main effect of word type (F(1,31) = 5.902, p = 0.021), with no effects of list or interactions (p > 0.1). Over the negative cluster, we again observed a main effect of word type (F(1,31) = 10.916, p = 0.0016), with no effects of list or interactions (p > 0.1). See Author response image 2.  

      Author response image 2.

      Difference in ERP voltage (Words – Part-words) for the two lists (A and B); W=Words; P=Part-Words, 

      We conducted a similar analysis for neural entrainment during the structured stream on voices. A comparison of entrainment at 2 Hz between participants who completed List A and List B showed no significant differences (t(30) = -0.27, p = 0.79). A test against zero for each list indicated significant entrainment in both cases (List A: t(17) = 4.44, p = 0.00036; List B: t(13) = 3.16, p = 0.0075). See Author response image 3.

      Author response image 3.

      Neural entrainment at 2Hz during the structured stream of Experiment 2 for Lists A and B.

      Words entrainment over occipital electrodes: Do you have any idea why the duplet entrainment effect occurs over the electrodes it does, in particular over the occipital electrodes (which seems a bit unintuitive given that this is a purely auditory experiment with sleeping neonates).

      Neural entrainment might be considered as a succession of evoked response induced by the stream. After applying an average reference in high-density EEG recordings, the auditory ERP in neonates typically consists of a central positivity and a posterior negativity with a source located at the electrical zero in a single-dipole model (i.e. approximately in the superior temporal region (Dehaene-Lambertz & Dehaene, 1994). In adults, because of the average reference (i.e. the sum of voltages is equal to zero at each time point) and because the electrodes cannot capture the negative pole of the auditory response, the negativity is distributed around the head. In infants, however, the brain is higher within the skull, allowing for a more accurate recording of the negative pole of the auditory ERP (see Author response image 4 for the location of electrodes in an infant head model).  

      Besides the posterior electrodes, we can see some entrainment on more anterior electrodes that probably corresponds to the positive pole of the auditory ERP.

      Author response image 4.

      International 10–20 sensors' location on the skull of an infant template, with the underlying 3-D reconstruction of the grey-white matter interface and projection of each electrode to the cortex. Computed across 16 infants (from Kabdebon et al, Neuroimage, 2014). The O1, O2, T5, and T6 electrodes project lower than in adults.

      Reviewer 3:

      (1) While it's true that voice is not essential for language (i.e., sign languages are implemented over gestures; the use of voices to produce non-linguistic sounds, like laughter), it is a feature of spoken languages. Thus I'm not sure if we can really consider this study as a comparison between linguistic and non-linguistic dimensions. In turn, I'm not sure that these results show that statistical learning at birth operates on non-linguistic features, being voices a linguistic dimension at least in spoken languages. I'd like to hear the authors' opinions on this.

      On one hand, it has been shown that statistical learning (SL) operates across multiple modalities and domains in human adults and animals. On the other hand, SL is considered essential for infants to begin parsing speech. Therefore, we aimed to investigate whether SL capacities at birth are more effective on linguistic dimensions of speech, potentially as a way to promote language learning.

      We agree with the reviewer that voices play an important role in communication (e.g., for identifying who is speaking); however, they do not contribute to language structure or meaning, and listeners are expected to normalize across voices to accurately perceive phonemes and words. Thus, voices are speech features but not linguistic features. Additionally, in natural speech, there are no abrupt voice changes within a word as in our experiment; instead, voice changes typically occur on a longer timescale and involve only a limited number of voices, such as in a dialogue. Therefore, computing regularities based on voice changes would not be useful in real-life language learning. We considered that contrasting syllables and voices was an elegant way to test SL beyond its linguistic dimension, as the experimental paradigm is identical in both experiments.  

      Along the same line, in the Discussion section, the present results are interpreted within a theoretical framework showing statistical learning in auditory non-linguistic (string of tones, music) and visual domains as well as visual and other animal species. I'm not sure if that theoretical framework is the right fit for the present results.

      (2) I'm not sure whether the fact that we see parallel and independent tracking of statistics in the two dimensions of speech at birth indicates that newborns would be able to do so in all the other dimensions of the speech. If so, what other dimensions are the authors referring to?

      The reviewer is correct that demonstrating the universality of SL requires testing additional modalities and acoustic dimensions. However, we postulate that SL is grounded in a basic mechanism of long-term associative learning, as proposed in Benjamin et al. (2024), which relies on a slow decay in the representation of a given event. This simple mechanism, capable of operating on any representational output, accounts for many types of sequence learning reported in the literature (Benjamin et al., in preparation). We will revise the discussion section to clarify this theoretical framework.

      (3) Lines 341-345: Statistical learning is an evolutionary ancient learning mechanism but I do not think that the present results are showing it. This is a study on human neonates and adults, there are no other animal species involved therefore I do not see a connection with the evolutionary history of statistical learning. It would be much more interesting to make claims on the ontogeny (rather than philogeny) of statistical learning, and what regularities newborns are able to detect right after birth. I believe that this is one of the strengths of this work.

      We did not intend to make claims about the phylogeny of SL. Since SL appears to be a learning mechanism shared across species, we use it as a framework to suggest that SL may arise from general operational principles applicable to diverse neural networks. Thus, while it is highly useful for language acquisition, it is not specific to it. We will revise this section to tone down our claims.  

      (4) The description of the stimuli in Lines 110-113 is a bit confusing. In Experiment 1, e.g., "pe" and "tu" are both uttered by the same voice, correct? ("random voice each time" is confusing). Whereas in Experiment 2, e.g., "pe" and "tu" are uttered by different voices, for example, "pe" by yellow voice and "tu" by red voice. If this is correct, then I recommend the authors to rephrase this section to make it more clear.

      To clarify, in Experiment 1, the voices were randomly assigned to each syllable, with the constraint that no voice was repeated consecutively. This means that syllables within the same word were spoken by different voices, and each syllable was heard with various voices throughout the stream. As a result, neonates had to retrieve the words based solely on syllabic patterns, without relying on consistent voice associations or specific voice relationships.

      In Experiment 2, the design was orthogonal: while the syllables were presented in a random order, the voices followed a structured pattern. Similar to Experiment 1, each syllable (e.g., “pe” and “tu”) was spoken by different voices. The key difference is that in Experiment 2, the structured regularities were applied to the voices rather than the syllables. In other words, the “green” voice was always followed by the “red” voice for example but uttered different syllables.

      We will revise the methods section to clarify these important points.

      (5) Line 114: the sentence "they should compute a 36 x 36 TPs matrix relating each acoustic signal, with TPs alternating between 1/6 within words and 1/12 between words" is confusing as it seems like there are different acoustic signals. Can the authors clarify this point?

      Thank you for highlighting this point. To clarify, our suggestion is that neonates might not track regularities between phonemes and voices as separate features. Instead, they may treat each syllable-voice combination as a distinct item—for example, "pe" spoken by the "yellow" voice is one item, while "pe" spoken by the "red" voice is another. Under this scenario, there would be a total of 36 unique items (6 syllables × 6 voices), and infants would need to track regularities between these 36 combinations.

      We will rephrase this sentence in the manuscript to make it clearer.

    1. Author Response

      eLife assessment

      This study presents potentially valuable results on glutamine-rich motifs in relation to protein expression and alternative genetic codes. The author's interpretation of the results is so far only supported by incomplete evidence, due to a lack of acknowledgment of alternative explanations, missing controls and statistical analysis and writing unclear to non experts in the field. These shortcomings could be at least partially overcome by additional experiments, thorough rewriting, or both.

      We thank both the Reviewing Editor and Senior Editor for handling this manuscript and will submit our revised manuscript after the reviewed preprint is published by eLife.  

      Reviewer #1 (Public Review):

      Summary

      This work contains 3 sections. The first section describes how protein domains with SQ motifs can increase the abundance of a lacZ reporter in yeast. The authors call this phenomenon autonomous protein expression-enhancing activity, and this finding is well supported. The authors show evidence that this increase in protein abundance and enzymatic activity is not due to changes in plasmid copy number or mRNA abundance, and that this phenomenon is not affected by mutants in translational quality control. It was not completely clear whether the increased protein abundance is due to increased translation or to increased protein stability.

      In section 2, the authors performed mutagenesis of three N-terminal domains to study how protein sequence changes protein stability and enzymatic activity of the fusions. These data are very interesting, but this section needs more interpretation. It is not clear if the effect is due to the number of S/T/Q/N amino acids or due to the number of phosphorylation sites.

      In section 3, the authors undertake an extensive computational analysis of amino acid runs in 27 species. Many aspects of this section are fascinating to an expert reader. They identify regions with poly-X tracks. These data were not normalized correctly: I think that a null expectation for how often poly-X track occur should be built for each species based on the underlying prevalence of amino acids in that species. As a result, I believe that the claim is not well supported by the data.

      Strengths

      This work is about an interesting topic and contains stimulating bioinformatics analysis. The first two sections, where the authors investigate how S/T/Q/N abundance modulates protein expression level, is well supported by the data. The bioinformatics analysis of Q abundance in ciliate proteomes is fascinating. There are some ciliates that have repurposed stop codons to code for Q. The authors find that in these proteomes, Q-runs are greatly expanded. They offer interesting speculations on how this expansion might impact protein function.

      Weakness

      At this time, the manuscript is disorganized and difficult to read. An expert in the field, who will not be distracted by the disorganization, will find some very interesting results included. In particular, the order of the introduction does not match the rest of the paper.

      In the first and second sections, where the authors investigate how S/T/Q/N abundance modulates protein expression levels, it is unclear if the effect is due to the number of phosphorylation sites or the number of S/T/Q/N residues.

      There are three reasons why the number of phosphorylation sites in the Q-rich motifs is not relevant to their autonomous protein expression-enhancing (PEE) activities:

      First, we have reported previously that phosphorylation-defective Rad51-NTD (Rad51-3SA) and wild-type Rad51-NTD exhibit similar autonomous PEE activity. Mec1/Tel1-dependent phosphorylation of Rad51-NTD antagonizes the proteasomal degradation pathway, increasing the half-life of Rad51 from ∼30 min to ≥180 min (Ref 27; Woo, T. T. et al. 2020).

      1. T. T. Woo, C. N. Chuang, M. Higashide, A. Shinohara, T. F. Wang, Dual roles of yeast Rad51 N-terminal domain in repairing DNA double-strand breaks. Nucleic Acids Res 48, 8474-8489 (2020).

      Second, in our preprint manuscript, we have also shown that phosphorylation-defective Rad53-SCD1 (Rad51-SCD1-5STA) also exhibits autonomous PEE activity similar to that of wild-type Rad53-SCD (Figure 2D, Figure 4A and Figure 4C).

      Third, as revealed by the results of our preprint manuscript (Figure 4), it is the percentages, and not the numbers, of S/T/Q/N residues that are correlated with the PEE activities of Q-rich motifs.

      The authors also do not discuss if the N-end rule for protein stability applies to the lacZ reporter or the fusion proteins.

      The autonomous PEE function of S/T/Q-rich NTDs is unlikely to be relevant to the N-end rule. The N-end rule links the in vivo half-life of a protein to the identity of its N-terminal residues. In S. cerevisiae, the N-end rule operates as part of the ubiquitin system and comprises two pathways. First, the Arg/N-end rule pathway, involving a single N-terminal amidohydrolase Nta1, mediates deamidation of N-terminal asparagine (N) and glutamine (Q) into aspartate (D) and glutamate (E), which in turn are arginylated by a single Ate1 R-transferase, generating the Arg/N degron. N-terminal R and other primary degrons are recognized by a single N-recognin Ubr1 in concert with ubiquitin-conjugating Ubc2/Rad6. Ubr1 can also recognize several other N-terminal residues, including lysine (K), histidine (H), phenylalanine (F), tryptophan (W), leucine (L) and isoleucine (I) (Bachmair, A. et al. 1986; Tasaki, T. et al. 2012; Varshavshy, A. et al. 2019). Second, the Ac/N-end rule pathway targets proteins containing N-terminally acetylated (Ac) residues. Prior to acetylation, the first amino acid methionine (M) is catalytically removed by Met-aminopeptides, unless a residue at position 2 is non-permissive (too large) for MetAPs. If a retained N-terminal M or otherwise a valine (V), cysteine (C), alanine (A), serine (S) or threonine (T) residue is followed by residues that allow N-terminal acetylation, the proteins containing these AcN degrons are targeted for ubiquitylation and proteasome-mediated degradation by the Doa10 E3 ligase (Hwang, C. S., 2019).

      A. Bachmair, D. Finley, A. Varshavsky, In vivo half-life of a protein is a function of its amino-terminal residue. Science 234, 179-186 (1986).

      T. Tasaki, S. M. Sriram, K. S. Park, Y. T. Kwon, The N-end rule pathway. Annu Rev Biochem 81, 261-289 (2012).

      A. Varshavsky, N-degron and C-degron pathways of protein degradation. Proc Natl Acad Sci 116, 358-366 (2019).

      C. S. Hwang, A. Shemorry, D. Auerbach, A. Varshavsky, The N-end rule pathway is mediated by a complex of the RING-type Ubr1 and HECT-type Ufd4 ubiquitin ligases. Nat Cell Biol 12, 1177-1185 (2010).

      The PEE activities of these S/T/Q-rich domains are unlikely to arise from counteracting the N-end rule for two reasons. First, the first two amino acid residues of Rad51-NTD, Hop1-SCD, Rad53-SCD1, Sup35-PND, Rad51-ΔN, and LacZ-NVH are MS, ME, ME, MS, ME, and MI, respectively, where M is methionine, S is serine, E is glutamic acid and I is isoleucine. Second, Sml1-NTD behaves similarly to these N-terminal fusion tags, despite its methionine and glutamine (MQ) amino acid signature at the N-terminus.

      The most interesting part of the paper is an exploration of S/T/Q/N-rich regions and other repetitive AA runs in 27 proteomes, particularly ciliates. However, this analysis is missing a critical control that makes it nearly impossible to evaluate the importance of the findings. The authors find the abundance of different amino acid runs in various proteomes. They also report the background abundance of each amino acid. They do not use this background abundance to normalize the runs of amino acids to create a null expectation from each proteome. For example, it has been clear for some time (Ruff, 2017; Ruff et al., 2016) that Drosophila contains a very high background of Q's in the proteome and it is necessary to control for this background abundance when finding runs of Q's.

      We apologize for not explaining sufficiently well the topic eliciting this reviewer’s concern in our preprint manuscript. In the second paragraph of page 14, we cite six references to highlight that SCDs are overrepresented in yeast and human proteins involved in several biological processes (32, 74), and that polyX prevalence differs among species (43, 75-77).

      1. Cheung HC, San Lucas FA, Hicks S, Chang K, Bertuch AA, Ribes-Zamora A. An S/T-Q cluster domain census unveils new putative targets under Tel1/Mec1 control. BMC Genomics. 2012;13:664.

      2. Mier P, Elena-Real C, Urbanek A, Bernado P, Andrade-Navarro MA. The importance of definitions in the study of polyQ regions: A tale of thresholds, impurities and sequence context. Comput Struct Biotechnol J. 2020;18:306-13.

      3. Cara L, Baitemirova M, Follis J, Larios-Sanz M, Ribes-Zamora A. The ATM- and ATR-related SCD domain is over-represented in proteins involved in nervous system development. Sci Rep. 2016;6:19050.

      4. Kuspa A, Loomis WF. The genome of Dictyostelium discoideum. Methods Mol Biol. 2006;346:15-30.

      5. Davies HM, Nofal SD, McLaughlin EJ, Osborne AR. Repetitive sequences in malaria parasite proteins. FEMS Microbiol Rev. 2017;41(6):923-40.

      6. Mier P, Alanis-Lobato G, Andrade-Navarro MA. Context characterization of amino acid homorepeats using evolution, position, and order. Proteins. 2017;85(4):709-19.

      We will cite the two references by Kiersten M. Ruff in our revised manuscript.

      K. M. Ruff and R. V. Pappu, (2015) Multiscale simulation provides mechanistic insights into the effects of sequence contexts of early-stage polyglutamine-mediated aggregation. Biophysical Journal 108, 495a.

      K. M. Ruff, J. B. Warner, A. Posey and P. S. Tan (2017) Polyglutamine length dependent structural properties and phase behavior of huntingtin exon1. Biophysical Journal 112, 511a.

      The authors could easily address this problem with the data and analysis they have already collected. However, at this time, without this normalization, I am hesitant to trust the lists of proteins with long runs of amino acid and the ensuing GO enrichment analysis.

      Ruff KM. 2017. Washington University in St.

      Ruff KM, Holehouse AS, Richardson MGO, Pappu RV. 2016. Proteomic and Biophysical Analysis of Polar Tracts. Biophys J 110:556a.

      We thank Reviewer #1 for this helpful suggestion and now address this issue by means of a different approach described below.

      Based on a previous study (43; Palo Mier et al. 2020), we applied seven different thresholds to seek both short and long, as well as pure and impure, polyX strings in 20 different representative near-complete proteomes, including 4X (4/4), 5X (4/5-5/5), 6X (4/6-6/6), 7X (4/7-7/7), 8-10X (≥50%X), 11-10X (≥50%X) and ≥21X (≥50%X).

      To normalize the runs of amino acids and create a null expectation from each proteome, we determined the ratios of the overall number of X residues for each of the seven polyX motifs relative to those in the entire proteome of each species, respectively. The results of four different polyX motifs are shown below, i.e., polyQ (Author response image 1), polyN (Author response image 2), polyS (Author response image 3) and polyT (Author response image 4).

      Author response image 1.

      Q contents in 7 different types of polyQ motifs in 20 near-complete proteomes. The five ciliates with reassigned stops codon (TAAQ and TAGQ) are indicated in red. Stentor coeruleus, a ciliate with standard stop codons, is indicated in green.  

      Author response image 2.

      N contents in 7 different types of polyN motifs in 20 near-complete proteomes. The five ciliates with reassigned stops codon (TAAQ and TAGQ) are indicated in red. Stentor coeruleus, a ciliate with standard stop codons, is indicated in green.

      Author response image 3.

      S contents in 7 different types of polyS motifs in 20 near-complete proteomes. The five ciliates with reassigned stops codon (TAAQ and TAGQ) are indicated in red. Stentor coeruleus, a ciliate with standard stop codons, is indicated in green.  

      Author response image 4.

      T contents in 7 different types of polyT motifs in 20 near-complete proteomes. The five ciliates with reassigned stops codon (TAAQ and TAGQ) are indicated in red. Stentor coeruleus, a ciliate with standard stop codons, is indicated in green.

      The results summarized in these four new figures support that polyX prevalence differs among species and that the overall X contents of polyX motifs often but not always correlate with the X usage frequency in entire proteomes (43; Palo Mier et al. 2020).

      Most importantly, our results reveal that, compared to Stentor coeruleus or several non-ciliate eukaryotic organisms (e.g., Plasmodium falciparum, Caenorhabditis elegans, Danio rerio, Mus musculus and Homo sapiens), the five ciliates with reassigned TAAQ and TAGQ codons not only have higher Q usage frequencies, but also more polyQ motifs in their proteomes (Figure 1). In contrast, polyQ motifs prevail in Candida albicans, Candida tropicalis, Dictyostelium discoideum, Chlamydomonas reinhardtii, Drosophila melanogaster and Aedes aegypti, though the Q usage frequencies in their entire proteomes are not significantly higher than those of other eukaryotes (Figure 1). Due to their higher N usage frequencies, Dictyostelium discoideum, Plasmodium falciparum and Pseudocohnilembus persalinus have more polyN motifs than the other 23 eukaryotes we examined here (Figure 2). Generally speaking, all 26 eukaryotes we assessed have similar S usage frequencies and percentages of S contents in polyS motifs (Figure 3). Among these 26 eukaryotes, Dictyostelium discoideum possesses many more polyT motifs, though its T usage frequency is similar to that of the other 25 eukaryotes (Figure 4).

      In conclusion, these new normalized results confirm that the reassignment of stop codons to Q indeed results in both higher Q usage frequencies and more polyQ motifs in ciliates.  

      Reviewer #2 (Public Review):

      Summary:

      This study seeks to understand the connection between protein sequence and function in disordered regions enriched in polar amino acids (specifically Q, N, S and T). While the authors suggest that specific motifs facilitate protein-enhancing activities, their findings are correlative, and the evidence is incomplete. Similarly, the authors propose that the re-assignment of stop codons to glutamine-encoding codons underlies the greater user of glutamine in a subset of ciliates, but again, the conclusions here are, at best, correlative. The authors perform extensive bioinformatic analysis, with detailed (albeit somewhat ad hoc) discussion on a number of proteins. Overall, the results presented here are interesting, but are unable to exclude competing hypotheses.

      Strengths:

      Following up on previous work, the authors wish to uncover a mechanism associated with poly-Q and SCD motifs explaining proposed protein expression-enhancing activities. They note that these motifs often occur IDRs and hypothesize that structural plasticity could be capitalized upon as a mechanism of diversification in evolution. To investigate this further, they employ bioinformatics to investigate the sequence features of proteomes of 27 eukaryotes. They deepen their sequence space exploration uncovering sub-phylum-specific features associated with species in which a stop-codon substitution has occurred. The authors propose this stop-codon substitution underlies an expansion of ploy-Q repeats and increased glutamine distribution.

      Weaknesses:

      The preprint provides extensive, detailed, and entirely unnecessary background information throughout, hampering reading and making it difficult to understand the ideas being proposed. The introduction provides a large amount of detailed background that appears entirely irrelevant for the paper. Many places detailed discussions on specific proteins that are likely of interest to the authors occur, yet without context, this does not enhance the paper for the reader.

      The paper uses many unnecessary, new, or redefined acronyms which makes reading difficult. As examples:

      (1) Prion forming domains (PFDs). Do the authors mean prion-like domains (PLDs), an established term with an empirical definition from the PLAAC algorithm? If yes, they should say this. If not, they must define what a prion-forming domain is formally.

      The N-terminal domain (1-123 amino acids) of S. cerevisiae Sup35 was already referred to as a “prion forming domain (PFD)” in 2006 (Tuite, M. F. 2006). Since then, PFD has also been employed as an acronym in other yeast prion papers (Cox, B.S. et al. 2007; Toombs, T. et al. 2011).

      M. F., Tuite, Yeast prions and their prion forming domain. Cell 27, 397-407 (2005).

      B. S. Cox, L. Byrne, M. F., Tuite, Protein Stability. Prion 1, 170-178 (2007).

      J. A. Toombs, N. M. Liss, K. R. Cobble, Z. Ben-Musa, E. D. Ross, [PSI+] maintenance is dependent on the composition, not primary sequence, of the oligopeptide repeat domain. PLoS One 6, e21953 (2011).

      (2) SCD is already an acronym in the IDP field (meaning sequence charge decoration) - the authors should avoid this as their chosen acronym for Serine(S) / threonine (T)-glutamine (Q) cluster domains. Moreover, do we really need another acronym here (we do not).

      SCD was first used in 2005 as an acronym for the Serine (S)/threonine (T)-glutamine (Q) cluster domain in the DNA damage checkpoint field (Traven, A. and Heierhorst, J. 2005). Almost a decade later, SCD became an acronym for “sequence charge decoration” (Sawle, L. et al. 2015; Firman, T. et al. 2018).

      A. Traven and J, Heierhorst, SQ/TQ cluster domains: concentrated ATM/ATR kinase phosphorylation site regions in DNA-damage-response proteins. Bioessays. 27, 397-407 (2005).

      L. Sawle and K, Ghosh, A theoretical method to compute sequence dependent configurational properties in charged polymers and proteins. J. Chem Phys. 143, 085101(2015).

      T. Firman and Ghosh, K. Sequence charge decoration dictates coil-globule transition in intrinsically disordered proteins. J. Chem Phys. 148, 123305 (2018).

      (3) Protein expression-enhancing (PEE) - just say expression-enhancing, there is no need for an acronym here.

      Thank you. Since we have shown that addition of Q-rich motifs to LacZ affects protein expression rather than transcription, we think it is better to use the “PEE” acronym.

      The results suggest autonomous protein expression-enhancing activities of regions of multiple proteins containing Q-rich and SCD motifs. Their definition of expression-enhancing activities is vague and the evidence they provide to support the claim is weak. While their previous work may support their claim with more evidence, it should be explained in more detail. The assay they choose is a fusion reporter measuring beta-galactosidase activity and tracking expression levels. Given the presented data they have shown that they can drive the expression of their reporters and that beta gal remains active, in addition to the increase in expression of fusion reporter during the stress response. They have not detailed what their control and mock treatment is, which makes complete understanding of their experimental approach difficult. Furthermore, their nuclear localization signal on the tag could be influencing the degradation kinetics or sequestering the reporter, leading to its accumulation and the appearance of enhanced expression. Their evidence refuting ubiquitin-mediated degradation does not have a convincing control.

      Based on the experimental results, the authors then go on to perform bioinformatic analysis of SCD proteins and polyX proteins. Unfortunately, there is no clear hypothesis for what is being tested; there is a vague sense of investigating polyX/SCD regions, but I did not find the connection between the first and section compelling (especially given polar-rich regions have been shown to engage in many different functions). As such, this bioinformatic analysis largely presents as many lists of percentages without any meaningful interpretation. The bioinformatics analysis lacks any kind of rigorous statistical tests, making it difficult to evaluate the conclusions drawn. The methods section is severely lacking. Specifically, many of the methods require the reader to read many other papers. While referencing prior work is of course, important, the authors should ensure the methods in this paper provide the details needed to allow a reader to evaluate the work being presented. As it stands, this is not the case.

      Thank you. As described in detail below, we have now performed rigorous statistical testing using the GofuncR package.

      Overall, my major concern with this work is that the authors make two central claims in this paper (as per the Discussion). The authors claim that Q-rich motifs enhance protein expression. The implication here is that Q-rich motif IDRs are special, but this is not tested. As such, they cannot exclude the competing hypothesis ("N-terminal disordered regions enhance expression").

      In fact, “N-terminal disordered regions enhance expression” exactly summarizes our hypothesis.

      On pages 12-13 and Figure 4 of our preprint manuscript, we explained our hypothesis in the paragraph entitled “The relationship between PEE function, amino acid contents, and structural flexibility”.

      The authors also do not explore the possibility that this effect is in part/entirely driven by mRNA-level effects (see Verma Na Comms 2019).

      As pointed out by the first reviewer, we show evidence that the increase in protein abundance and enzymatic activity is not due to changes in plasmid copy number or mRNA abundance (Figure 2), and that this phenomenon is not affected by translational quality control mutants (Figure 3).

      As such, while these observations are interesting, they feel preliminary and, in my opinion, cannot be used to draw hard conclusions on how N-terminal IDR sequence features influence protein expression. This does not mean the authors are necessarily wrong, but from the data presented here, I do not believe strong conclusions can be drawn. That re-assignment of stop codons to Q increases proteome-wide Q usage. I was unable to understand what result led the authors to this conclusion.

      My reading of the results is that a subset of ciliates has re-assigned UAA and UAG from the stop codon to Q. Those ciliates have more polyQ-containing proteins. However, they also have more polyN-containing proteins and proteins enriched in S/T-Q clusters. Surely if this were a stop-codon-dependent effect, we'd ONLY see an enhancement in Q-richness, not a corresponding enhancement in all polar-rich IDR frequencies? It seems the better working hypothesis is that free-floating climate proteomes are enriched in polar amino acids compared to sessile ciliates.

      Thank you. These comments are not supported by the results in Figure 1.

      Regardless, the absence of any kind of statistical analysis makes it hard to draw strong conclusions here.

      We apologize for not explaining more clearly the results of Tables 5-7 in our preprint manuscript.

      To address the concerns about our GO enrichment analysis by both reviewers, we have now performed rigorous statistical testing for SCD and polyQ protein overrepresentation using the GOfuncR package (https://bioconductor.org/packages/release/bioc/html/GOfuncR.html). GOfuncR is an R package program that conducts standard candidate vs. background enrichment analysis by means of the hypergeometric test. We then adjusted the raw p-values according to the Family-wise error rate (FWER). The same method had been applied to GO enrichment analysis of human genomes (Huttenhower, C., et al. 2009).

      Curtis Huttenhower, C., Haley, E. M., Hibbs, M., A., Dumeaux, V., Barrett, D. R., Hilary A. Coller, H. A., and Olga G. Troyanskaya, O., G. Exploring the human genome with functional maps, Genome Research 19, 1093-1106 (2009).

      The results presented in Author response image 5 and Author response image 6 support our hypothesis that Q-rich motifs prevail in proteins involved in specialized biological processes, including Saccharomyces cerevisiae RNA-mediated transposition, Candida albicans filamentous growth, peptidyl-glutamic acid modification in ciliates with reassigned stop codons (TAAQ and TAGQ), Tetrahymena thermophila xylan catabolism, Dictyostelium discoideum sexual reproduction, Plasmodium falciparum infection, as well as the nervous systems of Drosophila melanogaster, Mus musculus, and Homo sapiens (74). In contrast, peptidyl-glutamic acid modification and microtubule-based movement are not overrepresented with Q-rich proteins in Stentor coeruleus, a ciliate with standard stop codons.

      1. Cara L, Baitemirova M, Follis J, Larios-Sanz M, Ribes-Zamora A. The ATM- and ATR-related SCD domain is over-represented in proteins involved in nervous system development. Sci Rep. 2016;6:19050.

      Author response image 5.

      Selection of biological processes with overrepresented SCD-containing proteins in different eukaryotes. The percentages and number of SCD-containing proteins in our search that belong to each indicated Gene Ontology (GO) group are shown. GOfuncR (Huttenhower, C., et al. 2009) was applied for GO enrichment and statistical analysis. The p values adjusted according to the Family-wise error rate (FWER) are shown. The five ciliates with reassigned stop codons (TAAQ and TAGQ) are indicated in red. Stentor coeruleus, a ciliate with standard stop codons, is indicated in green.

      Author response image 6.

      Selection of biological processes with overrepresented polyQ-containing proteins in different eukaryotes. The percentages and numbers of polyQ-containing proteins in our search that belong to each indicated Gene Ontology (GO) group are shown. GOfuncR (Huttenhower, C., et al. 2009) was applied for GO enrichment and statistical analysis. The p values adjusted according to the Family-wise error rate (FWER) are shown. The five ciliates with reassigned stops codons (TAAQ and TAGQ) are indicated in red. Stentor coeruleus, a ciliate with standard stop codons, is indicated in green.

    1. Author response:

      Reviewer #1 (Public review):

      Summary:

      Jocher, Janssen, et al examine the robustness of comparative functional genomics studies in primates that make use of induced pluripotent stem cell-derived cells. Comparative studies in primates, especially amongst the great apes, are generally hindered by the very limited availability of samples, and iPSCs, which can be maintained in the laboratory indefinitely and defined into other cell types, have emerged as promising model systems because they allow the generation of data from tissues and cells that would otherwise be unobservable.

      Undirected differentiation of iPSCs into many cell types at once, using a method known as embryoid body differentiation, requires researchers to manually assign all cell types in the dataset so they can be correctly analysed. Typically, this is done using marker genes associated with a specific cell type. These are defined a priori, and have historically tended to be characterised in mice and humans and then employed to annotate other species. Jocher, Janssen, et al ask if the marker genes and features used to define a given cell type in one species are suitable for use in a second species, and then quantify the degree of usefulness of these markers. They find that genes that are informative and cell type specific in a given species are less valuable for cell type identification in other species, and that this value, or transferability, drops off as the evolutionary distance between species increases.

      This paper will help guide future comparative studies of gene expression in primates (and more broadly) as well as add to the growing literature on the broader challenges of selecting powerful and reliable marker genes for use in single-cell transcriptomics.

      Strengths:

      Marker gene selection and cell type annotation is a challenging problem in scRNA studies, and successful classification of cells often requires manual expert input. This can be hard to reproduce across studies, as, despite general agreement on the identity of many cell types, different methods for identifying marker genes will return different sets of genes. The rise of comparative functional genomics complicates this even further, as a robust marker gene in one species need not always be as useful in a different taxon. The finding that so many marker genes have poor transferability is striking, and by interrogating the assumption of transferability in a thorough and systematic fashion, this paper reminds us of the importance of systematically validating analytical choices. The focus on identifying how transferability varies across different types of marker genes (especially when comparing TFs to lncRNAs), and on exploring different methods to identify marker genes, also suggests additional criteria by which future researchers could select robust marker genes in their own data.

      The paper is built on a substantial amount of clearly reported and thoroughly considered data, including EBs and cells from four different primate species - humans, orangutans, and two macaque species. The authors go to great lengths to ensure the EBs are as comparable as possible across species, and take similar care with their computational analyses, always erring on the side of drawing conservative conclusions that are robustly supported by their data over more tenuously supported ones that could be impacted by data processing artefacts such as differences in mappability, etc. For example, I like the approach of using liftoff to robustly identify genes in non-human species that can be mapped to and compared across species confidently, rather than relying on the likely incomplete annotation of the non-human primate genomes. The authors also provide an interactive data visualisation website that allows users to explore the dataset in depth, examine expression patterns of their own favourite marker genes and perform the same kinds of analyses on their own data if desired, facilitating consistency between comparative primate studies.

      We thank the Reviewer for their kind assessment of our work.

      Weaknesses and recommendations:

      (1) Embryoid body generation is known to be highly variable from one replicate to the next for both technical and biological reasons, and the authors do their best to account for this, both by their testing of different ways of generating EBs, and by including multiple technical replicates/clones per species. However, there is still some variability that could be worth exploring in more depth. For example, the orangutan seems to have differentiated preferentially towards cardiac mesoderm whereas the other species seemed to prefer ectoderm fates, as shown in Figure 2C. Likewise, Supplementary Figure 2C suggests a significant unbalance in the contributions across replicates within a species, which is not surprising given the nature of EBs, while Supplementary Figure 6 suggests that despite including three different clones from a single rhesus macaque, most of the data came from a single clone. The manuscript would be strengthened by a more thorough exploration of the intra-species patterns of variability, especially for the taxa with multiple biological replicates, and how they impact the number of cell types detected across taxa, etc.

      You are absolutely correct in pointing out that the large clonal variability in cell type composition is a challenge for our analysis. We also noted the odd behavior of the orangutan EBs, and their underrepresentation of ectoderm. There are many possible sources for these variable differentiation propensities: clone, sample origin (in this case urine) and individual. However, unfortunately for the orangutan, we have only one individual and one sample origin and thus cannot say whether this germ layer preference says something about the species or is due to our specific sample.

      Because of this high variability from multiple sources, getting enough cell types with an appreciable overlap between species was limiting to analyses. In order to be able to derive meaningful conclusions from intra-species analyses and the impact of different sources of variation on cell type propensity, we would need to sequence many more EBs with an experimental design that balances possible sources of variation. This would go beyond the scope of this study.

      Instead, here we control for intra-species variation in our analyses as much as possible: For the analysis of cell type specificity and conservation the comparison is relative for the different specificity degrees (Figure 3C).  For the analysis of marker gene conservation, we explicitly take intra-species variation into account (Figure 4D).

      The same holds for the temporal aspect of the data, which is not really discussed in depth despite being a strength of the design. Instead, days 8 and 16 are analysed jointly, without much attention being paid to the possible differences between them.

      Concerning the temporal aspect, indeed we knowingly omitted to include an explicit comparison of day 8 and day 16 EBs, because we felt that it was not directly relevant to our main message. Our pseudotime analysis showed that the differences of the two time points were indeed a matter of degree and not so much of quality. All major lineages were already present at day 8 and even though day 8 cells had on average earlier pseudotimes, there was a large overlap in the pseudotime distributions between the two sampling time points (Author response image 1). That is why we decided to analyse the data together.

      Are EBs at day 16 more variable between species than at day 8? Is day 8 too soon to do these kinds of analyses?

      When we started the experiment, we simply did not know what to expect. We were worried that cell types at day 8 might be too transient, but longer culture can also introduce biases. That is why we wanted to look at two time points, however as mentioned above the differences are in degree.

      Concerning the cell type composition: yes, day 16 EBs are more heterogeneous than day 8 EBs. Firstly, older EBs have more distinguishable cell types and hence even if all EBs had identical composition, the sampling variance would be higher given that we sampled a similar number of cells from both time points. Secondly, in order to grow EBs for a longer time, we moved them from floating to attached culture on day 8 and it is unclear how much variance is added by this extra handling step.

      Are markers for earlier developmental progenitors better/more transferable than those for more derived cell types?

      We did not see any differences in the marker conservation between early and late cell types, but we have too little data to say whether this carries biological meaning.

      Author response image 1.

      Pseudotime analysis for a differentiation trajectory towards neurons. Single cells were first aggregated into metacells per species using SEACells (Persad et al. 2023). Pluripotent and ectoderm metacells were then integrated across all four species using Harmony and a combined pseudotime was inferred with Slingshot (Street et al. 2018), specifying iPSCs as the starting cluster. Here, lineage 3 is shown, illustrating a differentiation towards neurons. (A) PHATE embedding colored by pseudotime (Moon et al. 2019). (B) PHATE embedding colored by celltype. (C) Pseudotime distribution across the sampling timepoints (day 8 and day 16) in different species.

      (2) Closely tied to the point above, by necessity the authors collapse their data into seven fairly coarse cell types and then examine the performance of canonical marker genes (as well as those discovered de novo) across the species. However some of the clusters they use are somewhat broad, and so it is worth asking whether the lack of specificity exhibited by some marker genes and driving their conclusions is driven by inter-species heterogeneity within a given cluster.

      Author response image 2.

      UMAP visualization for the Harmony-integrated dataset across all four species for the seven shared cell types, colored by cell type identity (A) and species (B).

      Good point, if we understand correctly, the concern is that in our relatively broadly defined cell types, species are not well mixed and that this in turn is partly responsible for marker gene divergence. This problem is indeed difficult to address, because most approaches to evaluate this require integration across species which might lead to questionable results (see our Discussion).

      Nevertheless, we attempted an integration across all four species. To this end, we subset the cells for the 7 cell types that we found in all four species and visualized cell types and species in the UMAPs above (Author response image 2).

      We see that cardiac fibroblasts appear poorly integrated in the UMAP, but they still have very transferable marker genes across species. We quantified integration quality using the cell-specific mixing score (cms) (Lütge et al. 2021) and indeed found that the proportion of well integrated cells is lowest for cardiac fibroblasts (Author response image 3A). On the other end of the cms spectrum, neural crest cells appear to have the best integration across species, but their marker transferability between species is rather worse than for cardiac fibroblasts (Supplementary Figure 9). Cell-type wise calculated rank-biased overlap scores that we use for marker gene conservation show the same trends (Author response image 3B) as the F1 scores for marker gene transferability.  Hence, given our current dataset we do not see any indication that the low marker gene conservation is a result of too broadly defined cell types.

      Author response image 3.

      (A) Evaluation of species mixing per cell type in the Harmony-integrated dataset, quantified by the fraction of cells with an adjusted cell-specific mixing score (cms) above 0.05. (B) Summary of rank-biased overlap (RBO) scores per cell type to assess concordance of marker gene rankings for all species pairs.

      Reviewer #2 (Public review):

      Summary:

      The authors present an important study on identifying and comparing orthologous cell types across multiple species. This manuscript focuses on characterizing cell types in embryoid bodies (EBs) derived from induced pluripotent stem cells (iPSCs) of four primate species, humans, orangutans, cynomolgus macaques, and rhesus macaques, providing valuable insights into cross-species comparisons.

      Strengths:

      To achieve this, the authors developed a semi-automated computational pipeline that integrates classification and marker-based cluster annotation to identify orthologous cell types across primates. This study makes a significant contribution to the field by advancing cross-species cell type identification.

      We thank the reviewer for their positive and thoughtful feedback.

      Weaknesses:

      However, several critical points need to be addressed.

      (1) Use of Liftoff for GTF Annotation

      The authors used Liftoff to generate GTF files for Pongo abelii, Macaca fascicularis, and Macaca mulatta by transferring the hg38 annotation to the corresponding primate genomes. However, it is unclear why they did not use species-specific GTF files, as all these genomes have existing annotations. Why did the authors choose not to follow this approach?

      As Reviewer 1 also points out, also we have observed that the annotation of non-human primates often has truncated 3’UTRs. This is especially problematic for 3’ UMI transcriptome data as the ones in the 10x dataset that we present here. To illustrate this we compared the Liftoff annotation derived from Gencode v32,  that we also used throughout our manuscript to the Ensembl gene annotation Macaca_fascicularis_6.0.111. We used transcriptomes from human and cynomolgus iPSC bulk RNAseq  (Kliesmete et al. 2024) using the Prime-seq protocol (Janjic et al. 2022) which is very similar to 10x in that it also uses 3’ UMIs. On average using Liftoff produces higher counts than the Ensembl annotation (Author response image 4A). Moreover, when comparing across species, using Ensembl for the macaque leads to an asymmetry in differentially expressed genes, with apparently many more up-regulated genes in humans. In contrast, when we use the Liftoff annotation, we detect fewer DE-genes and a similar number of genes is up-regulated in macaques as in humans (Author response image 4B). We think that the many more DE-genes are artifacts due to mismatched annotation in human and cynomolgus macaques. We illustrate this for the case of the transcription factor SALL4 in Author response image 4 C,D.  The Ensembl annotation reports 2 transcripts, while Liftoff from Gencode v32 suggests 5 transcripts, one of which has a longer 3’UTR. This longer transcript is also supported by Nanopore data from macaque iPSCs. The truncation of the 3’UTR in this case leads to underestimation of the expression of SALL4 in macaques and hence SALL4 is detected as up-regulated in humans (DESeq2: LFC= 1.34, p-adj<2e-9). In contrast, when using the Liftoff annotation SALL4 does not appear to be DE between humans and macaques (LFC=0.33, p.adj=0.20).

      Author response image 4. 

      (A) UMI-counts/ gene for the same cynomolgus macaque iPSC samples. On the x-axis the gtf file from Ensembl Macaca_fascicularis_6.0.111 was used to count and on the y-axis we used our filtered Liftoff annotation that transferred the human gene models from Gencode v32. (B) The # of DE-genes between human  and cynomolgus iPSCs detected with DESeq2. In Liftoff, we counted human samples using Gencode v32 and compared it to the Liftoff annotation of the same human gene models to macFas6. In Ensembl, we use Gencode v32 for the human and  Ensembl Macaca_fascicularis_6.0.111 for the Macaque. For both comparisons we subset the genes to only contain one to one orthologues as annotated in biomart. Up and down regulation is relative to human expression. C) Read counts for one example gene SALL4. Here we used in addition to the Liftoff and Ensembl annotation also transcripts derived from Nanopore cDNA sequencing of cynomolgus iPSCs. D) Gene models for SALL4 in the space of MacFas6 and a coverage for iPSC-Prime-seq bulk RNA-sequencing.

      (2) Transcript Filtering and Potential Biases

      The authors excluded transcripts with partial mapping (<50%), low sequence identity (<50%), or excessive length differences (>100 bp and >2× length ratio). Such filtering may introduce biases in read alignment. Did the authors evaluate the impact of these filtering choices on alignment rates?

      We excluded those transcripts from analysis in both species, because they present a convolution of sequence-annotation differences and expression. The focus in our study is on regulatory evolution and we knowingly omit marker differences that are due to a marker being mutated away, we will make this clearer in the text of a revised version.

      (3) Data Integration with Harmony

      The methods section does not specify the parameters used for data integration with Harmony. Including these details would clarify how cross-species integration was performed.

      We want to stress  that none of our conservation and marker gene analyses relies on cross-species integration. We only used the Harmony integrated data for visualisation in Figure 1 and the rough germ-layer check up in Supplementary Figure S3.  We will add a better description in the revised version.

      References

      Janjic, Aleksandar, Lucas E. Wange, Johannes W. Bagnoli, Johanna Geuder, Phong Nguyen, Daniel Richter, Beate Vieth, et al. 2022. “Prime-Seq, Efficient and Powerful Bulk RNA Sequencing.” Genome Biology 23 (1): 88.

      Kliesmete, Zane, Peter Orchard, Victor Yan Kin Lee, Johanna Geuder, Simon M. Krauß, Mari Ohnuki, Jessica Jocher, Beate Vieth, Wolfgang Enard, and Ines Hellmann. 2024. “Evidence for Compensatory Evolution within Pleiotropic Regulatory Elements.” Genome Research 34 (10): 1528–39.

      Lütge, Almut, Joanna Zyprych-Walczak, Urszula Brykczynska Kunzmann, Helena L. Crowell, Daniela Calini, Dheeraj Malhotra, Charlotte Soneson, and Mark D. Robinson. 2021. “CellMixS: Quantifying and Visualizing Batch Effects in Single-Cell RNA-Seq Data.” Life Science Alliance 4 (6): e202001004.

      Moon, Kevin R., David van Dijk, Zheng Wang, Scott Gigante, Daniel B. Burkhardt, William S. Chen, Kristina Yim, et al. 2019. “Visualizing Structure and Transitions in High-Dimensional Biological Data.” Nature Biotechnology 37 (12): 1482–92.

      Persad, Sitara, Zi-Ning Choo, Christine Dien, Noor Sohail, Ignas Masilionis, Ronan Chaligné, Tal Nawy, et al. 2023. “SEACells Infers Transcriptional and Epigenomic Cellular States from Single-Cell Genomics Data.” Nature Biotechnology 41 (12): 1746–57.

      Street, Kelly, Davide Risso, Russell B. Fletcher, Diya Das, John Ngai, Nir Yosef, Elizabeth Purdom, and Sandrine Dudoit. 2018. “Slingshot: Cell Lineage and Pseudotime Inference for Single-Cell Transcriptomics.” BMC Genomics 19 (1): 477.

    1. Author response:

      Public Reviews:

      Reviewer #1 (Public review):

      We thank the reviewer for his valuable input and careful assessment, which have significantly improved the clarity and rigor of our manuscript.

      Summary:

      Mazer & Yovel 2025 dissect the inverse problem of how echolocators in groups manage to navigate their surroundings despite intense jamming using computational simulations.

      The authors show that despite the 'noisy' sensory environments that echolocating groups present, agents can still access some amount of echo-related information and use it to navigate their local environment. It is known that echolocating bats have strong small and large-scale spatial memory that plays an important role for individuals. The results from this paper also point to the potential importance of an even lower-level, short-term role of memory in the form of echo 'integration' across multiple calls, despite the unpredictability of echo detection in groups. The paper generates a useful basis to think about the mechanisms in echolocating groups for experimental investigations too.

      Strengths:

      (1) The paper builds on biologically well-motivated and parametrised 2D acoustics and sensory simulation setup to investigate the various key parameters of interest

      (2) The 'null-model' of echolocators not being able to tell apart objects & conspecifics while echolocating still shows agents successfully emerge from groups - even though the probability of emergence drops severely in comparison to cognitively more 'capable' agents. This is nonetheless an important result showing the direction-of-arrival of a sound itself is the 'minimum' set of ingredients needed for echolocators navigating their environment.

      (3) The results generate an important basis in unraveling how agents may navigate in sensorially noisy environments with a lot of irrelevant and very few relevant cues.

      (4) The 2D simulation framework is simple and computationally tractable enough to perform multiple runs to investigate many variables - while also remaining true to the aim of the investigation.

      Weaknesses:

      There are a few places in the paper that can be misunderstood or don't provide complete details. Here is a selection:

      (1) Line 61: '... studies have focused on movement algorithms while overlooking the sensory challenges involved' : This statement does not match the recent state of the literature. While the previous models may have had the assumption that all neighbours can be detected, there are models that specifically study the role of limited interaction arising from a potential inability to track all neighbours due to occlusion, and the effect of responding to only one/few neighbours at a time e.g. Bode et al. 2011 R. Soc. Interface, Rosenthal et al. 2015 PNAS, Jhawar et al. 2020 Nature Physics.

      We appreciate the reviewer's comment and the relevant references. We have revised the manuscript accordingly to clarify the distinction between studies that incorporate limited interactions and those that explicitly analyze sensory constraints and interference. We have refined our statement to acknowledge these contributions while maintaining our focus on sensory challenges beyond limited neighbor detection, such as signal degradation, occlusion effects, and multimodal sensory integration (see lines 61-64):

      While collective movement has been extensively studied in various species, including insect swarming, fish schooling, and bird murmuration (Pitcher, Partridge and Wardle, 1976; Partridge, 1982; Strandburg-Peshkin et al., 2013; Pearce et al., 2014; Rosenthal, Twomey, Hartnett, Wu, Couzin, et al., 2015; Bastien and Romanczuk, 2020; Davidson et al., 2021; Aidan, Bleichman and Ayali, 2024), as well as in swarm robotics agents performing tasks such as coordinated navigation and maze-solving (Faria Dias et al., 2021; Youssefi and Rouhani, 2021; Cheraghi, Shahzad and Graffi, 2022), most studies have focused on movement algorithms , often assuming full detection of neighbors (Parrish and Edelstein-Keshet, 1999; Couzin et al., 2002, 2005; Sumpter et al., 2008; Nagy et al., 2010; Bialek et al., 2012; Gautrais et al., 2012; Attanasi et al., 2014). Some models have incorporated limited interaction rules where individuals respond to one or a few neighbors due to sensory constraints (Bode, Franks and Wood, 2011; Jhawar et al., 2020). However, fewer studies explicitly examine how sensory interference, occlusion, and noise shape decision-making in collective systems (Rosenthal et al., 2015).

      (2) The word 'interference' is used loosely places (Line 89: '...took all interference signals...', Line 319: 'spatial interference') - this is confusing as it is not clear whether the authors refer to interference in the physics/acoustics sense, or broadly speaking as a synonym for reflections and/or jamming.

      To improve clarity, we have revised the manuscript to distinguish between different types of interference:

      · Acoustic interference (jamming): Overlapping calls that completely obscure echo detection, preventing bats from perceiving necessary environmental cues.

      · Acoustic interference (masking): Partial reduction in signal clarity due to competing calls.

      · Spatial interference: Physical obstruction by conspecifics affecting movement and navigation.

      We have updated the manuscript to use these terms consistently and explicitly define them in relevant sections (see lines 87-94 and 329-330). This distinction ensures that the reader can differentiate between interference as an acoustic phenomenon and its broader implications in navigation.

      (3) The paper discusses original results without reference to how they were obtained or what was done. The lack of detail here must be considered while interpreting the Discussion e.g. Line 302 ('our model suggests...increasing the call-rate..' - no clear mention of how/where call-rate was varied) & Line 323 '..no benefit beyond a certain level..' - also no clear mention of how/where call-level was manipulated in the simulations.

      All tested parameters, including call rate dynamics and call intensity variations, are detailed in the Methods section and Tables 1 and 2. Specifically:

      · Call Rate Variation: The Inter-Pulse Interval (IPI) was modeled based on documented echolocation behavior, decreasing from 100 msec during the search phase to 35 msec (~28 calls per second) at the end of the approach phase, and to 5 msec (200 calls per second) during the final buzz (see Table 2). This natural variation in call rate was not manually manipulated in the model but emerged from the simulated bat behavior.

      · Call Intensity Variation: The tested call intensity levels (100, 110, 120, 130 dB SPL) are presented in Table 1 under the “Call Level” parameter. The effect of increasing call intensity was analyzed in relation to exit probability, jamming probability, and collision rate. This is now explicitly referenced in the Discussion.

      We have revised the manuscript to explicitly reference these aspects in the Results and Discussion sections.

      Reviewer #2 (Public review):

      We are grateful for the reviewer’s insightful feedback, which has helped us clarify key aspects of our research and strengthen our conclusions.

      This manuscript describes a detailed model of bats flying together through a fixed geometry. The model considers elements that are faithful to both bat biosonar production and reception and the acoustics governing how sound moves in the air and interacts with obstacles. The model also incorporates behavioral patterns observed in bats, like one-dimensional feature following and temporal integration of cognitive maps. From a simulation study of the model and comparison of the results with the literature, the authors gain insight into how often bats may experience destructive interference of their acoustic signals and those of their peers, and how much such interference may actually negatively affect the groups' ability to navigate effectively. The authors use generalized linear models to test the significance of the effects they observe.

      In terms of its strengths, the work relies on a thoughtful and detailed model that faithfully incorporates salient features, such as acoustic elements like the filter for a biological receiver and temporal aggregation as a kind of memory in the system. At the same time, the authors' abstract features are complicating without being expected to give additional insights, as can be seen in the choice of a two-dimensional rather than three-dimensional system. I thought that the level of abstraction in the model was perfect, enough to demonstrate their results without needless details. The results are compelling and interesting, and the authors do a great job discussing them in the context of the biological literature.

      The most notable weakness I found in this work was that some aspects of the model were not entirely clear to me.

      For example, the directionality of the bat's sonar call in relation to its velocity. Are these the same?

      For simplicity, in our model, the head is aligned with the body, therefore the direction of the echolocation beam is the same as the direction of the flight.

      Moreover, call directionality (directivity) is not directly influenced by velocity. Instead, directionality is estimated using the piston model, as described in the Methods section. The directionality is based on the emission frequency and is thus primarily linked to the behavioral phases of the bat, with frequency shifts occurring as the bat transitions from search to approach to buzz phases. During the approach phase, the bat emits calls with higher frequencies, resulting in increased directionality. This is supported by the literature (Jakobsen and Surlykke, 2010; Jakobsen, Brinkløv and Surlykke, 2013). This phase is also associated with a natural reduction in flight speed, which is a well-documented behavioral adaptation in echolocating bats (Jakobsen et al., 2024).

      To clarify this in the manuscript, we have updated the text to explicitly state that directionality follows phase-dependent frequency changes rather than being a direct function of velocity, see lines 460-465.

      If so, what is the difference between phi_target and phi_tx in the model equations?

      represents the angle between the bat and the reflected object (target).

      the angle [rad], between the masking bat and target (from the transmitter’s perspective)

      refers to the angle between the transmitting conspecific and the receiving focal bat, from the transmitter’s point of view.

      represents the angle between the receiving bat and the transmitting bat, from the receiver’s point of view.

      These definitions have been explicitly stated in the revised manuscript to prevent any ambiguity (lines 467-468). Additionally, a Supplementary figure demonstrating the geometrical relations has been added to the manuscript.

      Author response image 1.

      What is a bat's response to colliding with a conspecific (rather than a wall)?

      In nature, minor collisions between bats are common and typically do not result in significant disruptions to flight (Boerma et al., 2019; Roy et al., 2019; Goldstein et al., 2024).Given this, our model does not explicitly simulate the physical impact of a collision event. Instead, during the collision event the bat keeps decreasing its velocity and changing its flight direction until the distance between bats is above the threshold (0.4 m). We assume that the primary cost of such interactions arises from the effort required to avoid collisions, rather than from the collision itself. This assumption aligns with observations of bat behavior in dense flight environments, where individuals prioritize collision avoidance rather than modeling post-collision dynamics.

      From the statistical side, it was not clear if replicate simulations were performed. If they were, which I believe is the right way due to stochasticity in the model, how many replicates were used, and are the standard errors referred to throughout the paper between individuals in the same simulation or between independent simulations, or both?

      The number of repetitions for each scenario is detailed in Table 1, but we included it in a more prominent location in the text for clarity. Specifically, we now state (Lines 274-275):

      "The number of repetitions for each scenario was as follows: 1 bat: 240; 2 bats: 120; 5 bats: 48; 10 bats: 24; 20 bats: 12; 40 bats: 12; 100 bats: 6."

      Regarding the reported standard errors, they are calculated across all individuals within each scenario, without distinguishing between different simulation trials.

      We clarified in the revised text (Lines 534-535 in Statistical Analysis)

      Overall, I found these weaknesses to be superficial and easily remedied by the authors. The authors presented well-reasoned arguments that were supported by their results, and which were used to demonstrate how call interference impacts the collective's roost exit as measured by several variables. As the authors highlight, I think this work is valuable to individuals interested in bat biology and behavior, as well as to applications in engineered multi-agent systems like robotic swarms.

      Reviewer #3 (Public review):

      We sincerely appreciate the reviewer’s thoughtful comments and the time invested in evaluating our work, which have greatly contributed to refining our study.

      We would like to note that in general, our model often simplifies some of the bats’ abilities, under the assumption that if the simulated bats manage to perform this difficult task with simpler mechanisms, real better adapted bats will probably perform even better. This thought strategy will be repeated in several of the answers below.

      Summary:

      The authors describe a model to mimic bat echolocation behavior and flight under high-density conditions and conclude that the problem of acoustic jamming is less severe than previously thought, conflating the success of their simulations (as described in the manuscript) with hard evidence for what real bats are actually doing. The authors base their model on two species of bats that fly at "high densities" (defined by the authors as colony sizes from tens to tens of thousands of individuals and densities of up to 33.3 bats/m2), Pipistrellus kuhli and Rhinopoma microphyllum. This work fits into the broader discussion of bat sensorimotor strategies during collective flight, and simulations are important to try to understand bat behavior, especially given a lack of empirical data. However, I have major concerns about the assumptions of the parameters used for the simulation, which significantly impact both the results of the simulation and the conclusions that can be made from the data. These details are elaborated upon below, along with key recommendations the authors should consider to guide the refinement of the model.

      Strengths:

      This paper carries out a simulation of bat behavior in dense swarms as a way to explain how jamming does not pose a problem in dense groups. Simulations are important when we lack empirical data. The simulation aims to model two different species with different echolocation signals, which is very important when trying to model echolocation behavior. The analyses are fairly systematic in testing all ranges of parameters used and discussing the differential results.

      Weaknesses:

      The justification for how the different foraging phase call types were chosen for different object detection distances in the simulation is unclear. Do these distances match those recorded from empirical studies, and if so, are they identical for both species used in the simulation?

      The distances at which bats transition between echolocation phases are identical for both species in our model (see Table 2). These distances are based on well-documented empirical studies of bat hunting and obstacle avoidance behavior (Griffin, Webster and Michael, 1958; Simmons and Kick, 1983; Schnitzler et al., 1987; Kalko, 1995; Hiryu et al., 2008; Vanderelst and Peremans, 2018). These references provide extensive evidence that insectivorous bats systematically adjust their echolocation calls in response to object proximity, following the characteristic phases of search, approach, and buzz.

      To improve clarity, we have updated the text to explicitly state that the phase transition distances are empirically grounded and apply equally to both modeled species (lines 430-447).

      What reasoning do the authors have for a bat using the same call characteristics to detect a cave wall as they would for detecting a small insect?

      In echolocating bats, call parameters are primarily shaped by the target distance and echo strength. Accordingly, there is little difference in call structure between prey capture and obstacles-related maneuvers, aside from intensity adjustments based on target strength (Hagino et al., 2007; Hiryu et al., 2008; Surlykke, Ghose and Moss, 2009; Kothari et al., 2014). In our study, due to the dense cave environment, the bats are found to operate in the approach phase nearly all the time, which is consistent with natural cave emergence, where they are navigating through a cluttered environment rather than engaging in open-space search. For one of the species (Rhinopoma M.), we also have empirical recordings of individuals flying under similar conditions (Goldstein et al., 2024). Our model was designed to remain as simple as possible while relying on conservative assumptions that may underestimate bat performance. If, in reality, bats fine-tune their echolocation calls even earlier or more precisely during navigation than assumed, our model would still conservatively reflect their actual capabilities.

      We actually used logarithmically frequency modulated (FM) chirps, generated using the MATLAB built-in function chirp(t, f0, t1, f1, 'logarithmic'). This method aligns with the nonlinear FM characteristics of Pipistrellus kuhlii (PK) and Rhinopoma microphyllum (RM) and provides a realistic approximation of their echolocation signals. We acknowledge that this was not sufficiently emphasized in the original text, and we have now explicitly highlighted this in the revised version to ensure clarity (sell Lines 447-449 in Methods).

      The two species modeled have different calls. In particular, the bandwidth varies by a factor of 10, meaning the species' sonars will have different spatial resolutions. Range resolution is about 10x better for PK compared to RM, but the authors appear to use the same thresholds for "correct detection" for both, which doesn't seem appropriate.

      The detection process in our model is based on Saillant’s method using a filter bank, as detailed in the paper (Saillant et al., 1993; Neretti et al., 2003; Sanderson et al., 2003). This approach inherently incorporates the advantages of a wider bandwidth, meaning that the differences in range resolution between the species are already accounted for within the signal-processing framework. Thus, there is no need to explicitly adjust the model parameters for bandwidth variations, as these effects emerge from the applied method.

      Also, the authors did not mention incorporating/correcting for/exploiting Doppler, which leads me to assume they did not model it.

      The reviewer is correct. To maintain model simplicity, we did not incorporate the Doppler effect or its impact on echolocation. The exclusion of Doppler effects was based on the assumption that while Doppler shifts can influence frequency perception, their impact on jamming and overall navigation performance is minor within the modelled context.

      The maximal Doppler shifts expected for the bats in this scenario are of ~ 1kHz. These shifts would be applied variably across signals due to the semi-random relative velocities between bats, leading to a mixed effect on frequency changes. This variability would likely result in an overall reduction in jamming rather than exacerbating it, aligning with our previous statement that our model may overestimate the severity of acoustic interference. Such Doppler shifts would result in errors of 2-4 cm in localization (i.e., 200-400 micro-seconds) (Boonman, Parsons and Jones, 2003). 

      We have now explicitly highlighted this in the revised version (see Lines 468-470).

      The success of the simulation may very well be due to variation in the calls of the bats, which ironically enough demonstrates the importance of a jamming avoidance response in dense flight. This explains why the performance of the simulation falls when bats are not able to distinguish their own echoes from other signals. For example, in Figure C2, there are calls that are labeled as conspecific calls and have markedly shorter durations and wider bandwidths than others. These three phases for call types used by the authors may be responsible for some (or most) of the performance of the model since the correlation between different call types is unlikely to exceed the detection threshold. But it turns out this variation in and of itself is what a jamming avoidance response may consist of. So, in essence, the authors are incorporating a jamming avoidance response into their simulation.

      We fully agree that the natural variations in call design between the phases contribute significantly to interference reduction (see our discussion in a previous paper in Mazar & Yovel, 2020). However, we emphasize that this cannot be classified as a Jamming Avoidance Response (JAR). In our model, bats respond only to the physical presence of objects and not to the acoustic environment or interference itself. There is no active or adaptive adjustment of call design to minimize jamming beyond the natural phase-dependent variations in call structure. Therefore, while variation in call types does inherently reduce interference, this effect emerges passively from the modeled behavior rather than as an intentional strategy to avoid jamming.

      The authors claim that integration over multiple pings (though I was not able to determine the specifics of this integration algorithm) reduces the masking problem. Indeed, it should: if you have two chances at detection, you've effectively increased your SNR by 3dB.

      The reviewer is correct. Indeed, integration over multiple calls improves signal-to-noise ratio (SNR), effectively increasing it by approximately 3 dB per doubling of observations. The specifics of the integration algorithm are detailed in the Methods section, where we describe how sensory information is aggregated across multiple time steps to enhance detection reliability.

      They also claim - although it is almost an afterthought - that integration dramatically reduces the degradation caused by false echoes. This also makes sense: from one ping to the next, the bat's own echo delays will correlate extremely well with the bat's flight path. Echo delays due to conspecifics will jump around kind of randomly. However, the main concern is regarding the time interval and number of pings of the integration, especially in the context of the bat's flight speed. The authors say that a 1s integration interval (5-10 pings) dramatically reduces jamming probability and echo confusion. This number of pings isn't very high, and it occurs over a time interval during which the bat has moved 5-10m. This distance is large compared to the 0.4m distance-to-obstacle that triggers an evasive maneuver from the bat, so integration should produce a latency in navigation that significantly hinders the ability to avoid obstacles. Can the authors provide statistics that describe this latency, and discussion about why it doesn't seem to be a problem?

      As described in the Methods section, the bat’s collision avoidance response does not solely rely on the integration process. Instead, the model incorporates real-time echoes from the last calls, which are used independently of the integration process for immediate obstacle avoidance maneuvers. This ensures that bats can react to nearby obstacles without being hindered by the integration latency. The slower integration on the other hand is used for clustering, outlier removal and estimation wall directions to support the pathfinding process, as illustrated in Supplementary Figure 1.

      Additionally, our model assumes that bats store the physical positions of echoes in an allocentric coordinate system (x-y). The integration occurs after transforming these detections from a local relative reference frame to a global spatial representation. This allows for stable environmental mapping while maintaining responsiveness to immediate changes in the bat’s surroundings.

      See lines 518-523 in the revied version.

      The authors are using a 2D simulation, but this very much simplifies the challenge of a 3D navigation task, and there is an explanation as to why this is appropriate. Bat densities and bat behavior are discussed per unit area when realistically it should be per unit volume. In fact, the authors reference studies to justify the densities used in the simulation, but these studies were done in a 3D world. If the authors have justification for why it is realistic to model a 3D world in a 2D simulation, I encourage them to provide references justifying this approach.

      We acknowledge that this is a simplification; however, from an echolocation perspective, a 2D framework represents a worst-case scenario in terms of bat densities and maneuverability:

      · Higher Effective Density: A 2D model forces all bats into a single plane rather than distributing them through a 3D volume, increasing the likelihood of overlap in calls and echoes and making jamming more severe. As described in the text: the average distance to the nearest bat in our simulation is 0.27m (with 100 bats), whereas reported distances in very dense colonies are 0.5m, as observed in Myotis grisescens and Tadarida brasiliensis (Fujioka et al., 2021; Sabol and Hudson, 1995; Betke et al., 2008; Gillam et al, 2010)

      · Reduced Maneuverability: In 3D space, bats can use vertical movement to avoid obstacles and conspecifics. A 2D constraint eliminates this degree of freedom, increasing collision risk and limiting escape options.

      Thus, our 2D model provides a conservative difficult test case, ensuring that our findings are valid under conditions where jamming and collision risks are maximized. Additionally, the 2D framework is computationally efficient, allowing us to perform multiple simulation runs to explore a broad parameter space and systematically test the impact of different variables.

      To address the reviewer’s concern, we have clarified this justification in the revised text and will provide supporting references where applicable: (see Methods lines 407-412)

      The focus on "masking" (which appears to be just in-band noise), especially relative to the problem of misassigned echoes, is concerning. If the bat calls are all the same waveform (downsweep linear FM of some duration, I assume - it's not clear from the text), false echoes would be a major problem. Masking, as the authors define it, just reduces SNR. This reduction is something like sqrt(N), where N is the number of conspecifics whose echoes are audible to the bat, so this allows the detection threshold to be set lower, increasing the probability that a bat's echo will exceed a detection threshold. False echoes present a very different problem. They do not reduce SNR per se, but rather they cause spurious threshold excursions (N of them!) that the bat cannot help but interpret as obstacle detection. I would argue that in dense groups the mis-assignment problem is much more important than the SNR problem.

      There is substantial literature supporting the assumption that bats can recognize their own echoes and distinguish them from conspecific signals (Schnitzler and Bioscience, 2001‏; Kazial, Burnett and Masters, 2001; Burnett and Masters, 2002; Kazial, Kenny and Burnett, 2008; Chili, Xian and Moss, 2009; Yovel et al., 2009; Beetz and Hechavarría, 2022). However, we acknowledge that false echoes may present a major challenge in dense groups. To address this, we explicitly tested the impact of the self-echo identification assumption in our study see Results Figure 4: The impact of confusion on performance, and lines 345-355 in the Discussion.

      Furthermore, we examined a full confusion scenario, where all reflected echoes from conspecifics were misinterpreted as obstacle reflections (i.e., 100% confusion). Our results show that this significantly degrades navigation performance, supporting the argument that echo misassignment is a critical issue. However, we also explored a simple mitigation strategy based on temporal integration with outlier rejection, which provided some improvement in performance. This suggests that real bats may possess additional mechanisms to enhance self-echo identification and reduce false detections. See lines XX in the manuscript for further discussion.

      The criteria set for flight behavior (lines 393-406) are not justified with any empirical evidence of the flight behavior of wild bats in collective flight. How did the authors determine the avoidance distances? Also, what is the justification for the time limit of 15 seconds to emerge from the opening? Instead of an exit probability, why not instead use a time criterion, similar to "How long does it take X% of bats to exit?"

      While we acknowledge that wild bats may employ more complex behaviors for collision avoidance, we chose to implement a simplified decision-making rule in our model to maintain computational tractability.

      The avoidance distances (1.5 m from walls and 0.4 m from other bats) were selected as internal parameters to ensure coherent flight trajectories while maintaining a reasonable collision rate. These distances provide a balance between maneuverability and stability, preventing erratic flight patterns while still enabling effective obstacle avoidance. In the revised paper, we have added supplementary figures illustrating the effect of model parameters on performance, specifically focusing on the avoidance distance.

      The 15-second exit limit was determined as described in the text (Lines 403-404): “A 15-second window was chosen because it is approximately twice the average exit time for 40 bats and allows for a second corrective maneuver if needed.” In other words, it allowed each bat to circle the ‘cave’ twice to exit even in the most crowded environment. This threshold was set to keep simulation time reasonable while allowing sufficient time for most bats to exit successfully.

      We acknowledge that the alternative approach suggested by the reviewer—measuring the time taken for a certain percentage of bats to exit—is also valid. However, in our model, some outlier bats fail to exit and continue flying for many minutes, Such simulations would lead to excessive simulation times making it difficult to generate repetitions and not teaching us much – they usually resulted from the bat slightly missing the opening (see video S1. Our chosen approach ensures practical runtime constraints while still capturing relevant performance metrics.

      What is the empirical justification for the 1-10 calls used for integration?

      The "average exit time for 40 bats" is also confusing and not well explained. Was this determined empirically? From the simulation? If the latter, what are the conditions? Does it include masking, no masking, or which species?

      Previous studies have demonstrated that bats integrate acoustic information received sequentially over several echolocation calls (2-15), effectively constructing an auditory scene in complex environments (Ulanovsky and Moss, 2008; Chili, Xian and Moss, 2009; Moss and Surlykke, 2010; Yovel and Ulanovsky, 2017; Salles, Diebold and Moss, 2020). Additionally, bats are known to produce echolocation sound groups when spatiotemporal localization demands are high (Kothari et al., 2014). Studies have documented call sequences ranging from 2 to 15 grouped calls (Moss et al., 2010), and it has been hypothesized that grouping facilitates echo segregation.

      We did not use a single integration window - we tested integration sizes between 1 and 10 calls and presented the results in Figure 3A. This range was chosen based on prior empirical findings and to explore how different levels of temporal aggregation impact navigation performance. Indeed, the results showed that the performance levels between 5-10 calls integration window (Figure 3A)

      Regarding the average exit time for 40 bats, this value was determined from our simulations, where it represents the mean time for successful exits under standard conditions with masking.

      We have revised the text to clarify these details see, lines 466.

      References:

      Aidan, Y., Bleichman, I. and Ayali, A. (2024) ‘Pausing to swarm: locust intermittent motion is instrumental for swarming-related visual processing’, Biology letters, 20(2), p. 20230468. Available at: https://doi.org/10.1098/rsbl.2023.0468.

      Attanasi, A. et al. (2014) ‘Collective Behaviour without Collective Order in Wild Swarms of Midges’. Edited by T. Vicsek, 10(7). Available at: https://doi.org/10.1371/journal.pcbi.1003697.

      Bastien, R. and Romanczuk, P. (2020) ‘A model of collective behavior based purely on vision’, Science Advances, 6(6). Available at: https://doi.org/10.1126/sciadv.aay0792.

      Beetz, M.J. and Hechavarría, J.C. (2022) ‘Neural Processing of Naturalistic Echolocation Signals in Bats’, Frontiers in Neural Circuits, 16, p. 899370. Available at: https://doi.org/10.3389/FNCIR.2022.899370/BIBTEX.

      Betke, M. et al. (2008) ‘Thermal Imaging Reveals Significantly Smaller Brazilian Free-Tailed Bat Colonies Than Previously Estimated’, Journal of Mammalogy, 89(1), pp. 18–24. Available at: https://doi.org/10.1644/07-MAMM-A-011.1.

      Bialek, W. et al. (2012) ‘Statistical mechanics for natural flocks of birds’, Proceedings of the National Academy of Sciences, 109(13), pp. 4786–4791. Available at: https://doi.org/10.1073/PNAS.1118633109.

      Bode, N.W.F., Franks, D.W. and Wood, A.J. (2011) ‘Limited interactions in flocks: Relating model simulations to empirical data’, Journal of the Royal Society Interface, 8(55), pp. 301–304. Available at: https://doi.org/10.1098/RSIF.2010.0397.

      Boerma, D.B. et al. (2019) ‘Wings as inertial appendages: How bats recover from aerial stumbles’, Journal of Experimental Biology, 222(20). Available at: https://doi.org/10.1242/JEB.204255/VIDEO-3.

      Boonman, A.M., Parsons, S. and Jones, G. (2003) ‘The influence of flight speed on the ranging performance of bats using frequency modulated echolocation pulses’, The Journal of the Acoustical Society of America, 113(1), p. 617. Available at: https://doi.org/10.1121/1.1528175.

      Burnett, S.C. and Masters, W.M. (2002) ‘Identifying Bats Using Computerized Analysis and Artificial Neural Networks’, North American Symposium on Bat Research, 9.

      Cheraghi, A.R., Shahzad, S. and Graffi, K. (2022) ‘Past, Present, and Future of Swarm Robotics’, in Lecture Notes in Networks and Systems. Available at: https://doi.org/10.1007/978-3-030-82199-9_13.

      Chili, C., Xian, W. and Moss, C.F. (2009) ‘Adaptive echolocation behavior in bats for the analysis of auditory scenes’, Journal of Experimental Biology, 212(9), pp. 1392–1404. Available at: https://doi.org/10.1242/jeb.027045.

      Couzin, I.D. et al. (2002) ‘Collective Memory and Spatial Sorting in Animal Groups’, Journal of Theoretical Biology, 218(1), pp. 1–11. Available at: https://doi.org/10.1006/jtbi.2002.3065.

      Couzin, I.D. et al. (2005) ‘Effective leadership and decision-making in animal groups on the move’, Nature, 433(7025), pp. 513–516. Available at: https://doi.org/10.1038/nature03236.

      Davidson, J.D. et al. (2021) ‘Collective detection based on visual information in animal groups’, Journal of the Royal Society, 18(180), p. 2021.02.18.431380. Available at: https://doi.org/10.1098/rsif.2021.0142.

      Faria Dias, P.G. et al. (2021) ‘Swarm robotics: A perspective on the latest reviewed concepts and applications’, Sensors. Available at: https://doi.org/10.3390/s21062062.

      Fujioka, E. et al. (2021) ‘Three-Dimensional Trajectory Construction and Observation of Group Behavior of Wild Bats During Cave Emergence’, Journal of Robotics and Mechatronics, 33(3), pp. 556–563. Available at: https://doi.org/10.20965/jrm.2021.p0556.

      Gautrais, J. et al. (2012) ‘Deciphering Interactions in Moving Animal Groups’, PLOS Computational Biology, 8(9), p. e1002678. Available at: https://doi.org/10.1371/JOURNAL.PCBI.1002678.

      Gillam, E.H. et al. (2010) ‘Echolocation behavior of Brazilian free-tailed bats during dense emergence flights’, Journal of Mammalogy, 91(4), pp. 967–975. Available at: https://doi.org/10.1644/09-MAMM-A-302.1.

      Goldstein, A. et al. (2024) ‘Collective Sensing – On-Board Recordings Reveal How Bats Maneuver Under Severe 4 Acoustic Interference’, Under Review, pp. 1–25.

      Griffin, D.R., Webster, F.A. and Michael, C.R. (1958) ‘THE ECHOLOCATION OF FLYING INSECTS BY BATS ANIMAL BEHAVIOUR , Viii , 3-4’.

      Hagino, T. et al. (2007) ‘Adaptive SONAR sounds by echolocating bats’, International Symposium on Underwater Technology, UT 2007 - International Workshop on Scientific Use of Submarine Cables and Related Technologies 2007, pp. 647–651. Available at: https://doi.org/10.1109/UT.2007.370829.

      Hiryu, S. et al. (2008) ‘Adaptive echolocation sounds of insectivorous bats, Pipistrellus abramus, during foraging flights in the field’, The Journal of the Acoustical Society of America, 124(2), pp. EL51–EL56. Available at: https://doi.org/10.1121/1.2947629.

      Jakobsen, L. et al. (2024) ‘Velocity as an overlooked driver in the echolocation behavior of aerial hawking vespertilionid bats’. Available at: https://doi.org/10.1016/j.cub.2024.12.042.

      Jakobsen, L., Brinkløv, S. and Surlykke, A. (2013) ‘Intensity and directionality of bat echolocation signals’, Frontiers in Physiology, 4 APR(April), pp. 1–9. Available at: https://doi.org/10.3389/fphys.2013.00089.

      Jakobsen, L. and Surlykke, A. (2010) ‘Vespertilionid bats control the width of their biosonar sound beam dynamically during prey pursuit’, 107(31). Available at: https://doi.org/10.1073/pnas.1006630107.

      Jhawar, J. et al. (2020) ‘Noise-induced schooling of fish’, Nature Physics 2020 16:4, 16(4), pp. 488–493. Available at: https://doi.org/10.1038/s41567-020-0787-y.

      Kalko, E.K. V. (1995) ‘Insect pursuit, prey capture and echolocation in pipistrelle bats (Microchirptera)’, Animal Behaviour, 50(4), pp. 861–880.

      Kazial, K.A., Burnett, S.C. and Masters, W.M. (2001) ‘ Individual and Group Variation in Echolocation Calls of Big Brown Bats, Eptesicus Fuscus (Chiroptera: Vespertilionidae) ’, Journal of Mammalogy, 82(2), pp. 339–351. Available at: https://doi.org/10.1644/1545-1542(2001)082<0339:iagvie>2.0.co;2.

      Kazial, K.A., Kenny, T.L. and Burnett, S.C. (2008) ‘Little brown bats (Myotis lucifugus) recognize individual identity of conspecifics using sonar calls’, Ethology, 114(5), pp. 469–478. Available at: https://doi.org/10.1111/j.1439-0310.2008.01483.x.

      Kothari, N.B. et al. (2014) ‘Timing matters: Sonar call groups facilitate target localization in bats’, Frontiers in Physiology, 5 MAY. Available at: https://doi.org/10.3389/fphys.2014.00168.

      Moss, C.F. and Surlykke, A. (2010) ‘Probing the natural scene by echolocation in bats’, Frontiers in Behavioral Neuroscience. Available at: https://doi.org/10.3389/fnbeh.2010.00033.

      Nagy, M. et al. (2010) ‘Hierarchical group dynamics in pigeon flocks’, Nature 2010 464:7290, 464(7290), pp. 890–893. Available at: https://doi.org/10.1038/nature08891.

      Neretti, N. et al. (2003) ‘Time-frequency model for echo-delay resolution in wideband biosonar’, The Journal of the Acoustical Society of America, 113(4), pp. 2137–2145. Available at: https://doi.org/10.1121/1.1554693.

      Parrish, J.K. and Edelstein-Keshet, L. (1999) ‘Complexity, Pattern, and Evolutionary Trade-Offs in Animal Aggregation’, Science, 284(5411), pp. 99–101. Available at: https://doi.org/10.1126/SCIENCE.284.5411.99.

      Partridge, B.L. (1982) ‘The Structure and Function of Fish Schools’, 246(6), pp. 114–123. Available at: https://doi.org/10.2307/24966618.

      Pearce, D.J.G. et al. (2014) ‘Role of projection in the control of bird flocks’, Proceedings of the National Academy of Sciences of the United States of America, 111(29), pp. 10422–10426. Available at: https://doi.org/10.1073/pnas.1402202111.

      Pitcher, T.J., Partridge, B.L. and Wardle, C.S. (1976) ‘A blind fish can school’, Science, 194(4268), pp. 963–965. Available at: https://doi.org/10.1126/science.982056.

      Rosenthal, S.B., Twomey, C.R., Hartnett, A.T., Wu, H.S., Couzin, I.D., et al. (2015) ‘Revealing the hidden networks of interaction in mobile animal groups allows prediction of complex behavioral contagion’, Proceedings of the National Academy of Sciences of the United States of America, 112(15), pp. 4690–4695. Available at: https://doi.org/10.1073/pnas.1420068112.

      Rosenthal, S.B., Twomey, C.R., Hartnett, A.T., Wu, H.S. and Couzin, I.D. (2015) ‘Revealing the hidden networks of interaction in mobile animal groups allows prediction of complex behavioral contagion’, Proceedings of the National Academy of Sciences of the United States of America, 112(15), pp. 4690–4695. Available at: https://doi.org/10.1073/PNAS.1420068112/-/DCSUPPLEMENTAL/PNAS.1420068112.SAPP.PDF.

      Roy, S. et al. (2019) ‘Extracting interactions between flying bat pairs using model-free methods’, Entropy, 21(1). Available at: https://doi.org/10.3390/e21010042.

      Sabol, B.M. and Hudson, M.K. (1995) ‘Technique using thermal infrared-imaging for estimating populations of gray bats’, Journal of Mammalogy, 76(4). Available at: https://doi.org/10.2307/1382618.

      Saillant, P.A. et al. (1993) ‘A computational model of echo processing and acoustic imaging in frequency- modulated echolocating bats: The spectrogram correlation and transformation receiver’, The Journal of the Acoustical Society of America, 94(5). Available at: https://doi.org/10.1121/1.407353.

      Salles, A., Diebold, C.A. and Moss, C.F. (2020) ‘Echolocating bats accumulate information from acoustic snapshots to predict auditory object motion’, Proceedings of the National Academy of Sciences of the United States of America, 117(46), pp. 29229–29238. Available at: https://doi.org/10.1073/PNAS.2011719117/SUPPL_FILE/PNAS.2011719117.SAPP.PDF.

      Sanderson, M.I. et al. (2003) ‘Evaluation of an auditory model for echo delay accuracy in wideband biosonar’, The Journal of the Acoustical Society of America, 114(3), pp. 1648–1659. Available at: https://doi.org/10.1121/1.1598195.

      Schnitzler, H., Bioscience, E.K.- and 2001‏, undefined (no date) ‘Echolocation by insect-eating bats: we define four distinct functional groups of bats and find differences in signal structure that correlate with the typical echolocation ‏’, academic.oup.com‏HU Schnitzler, EKV Kalko‏Bioscience, 2001‏•academic.oup.com‏ [Preprint]. Available at: https://academic.oup.com/bioscience/article-abstract/51/7/557/268230 (Accessed: 17 March 2025).

      Schnitzler, H.-U. et al. (1987) ‘The echolocation and hunting behavior of the bat,Pipistrellus kuhli’, Journal of Comparative Physiology A, 161(2), pp. 267–274. Available at: https://doi.org/10.1007/BF00615246.

      Simmons, J.A. and Kick, S.A. (1983) ‘Interception of Flying Insects by Bats’, Neuroethology and Behavioral Physiology, pp. 267–279. Available at: https://doi.org/10.1007/978-3-642-69271-0_20.

      Strandburg-Peshkin, A. et al. (2013) ‘Visual sensory networks and effective information transfer in animal groups’, Current Biology. Cell Press. Available at: https://doi.org/10.1016/j.cub.2013.07.059.

      Sumpter, D.J.T. et al. (2008) ‘Consensus Decision Making by Fish’, Current Biology, 18(22), pp. 1773–1777. Available at: https://doi.org/10.1016/J.CUB.2008.09.064.

      Surlykke, A., Ghose, K. and Moss, C.F. (2009) ‘Acoustic scanning of natural scenes by echolocation in the big brown bat, Eptesicus fuscus’, Journal of Experimental Biology, 212(7), pp. 1011–1020. Available at: https://doi.org/10.1242/JEB.024620.

      Theriault, D.H. et al. (no date) ‘Reconstruction and analysis of 3D trajectories of Brazilian free-tailed bats in flight‏’, cs-web.bu.edu‏ [Preprint]. Available at: https://cs-web.bu.edu/faculty/betke/papers/2010-027-3d-bat-trajectories.pdf (Accessed: 4 May 2023).

      Ulanovsky, N. and Moss, C.F. (2008) ‘What the bat’s voice tells the bat’s brain’, Proceedings of the National Academy of Sciences of the United States of America, 105(25), pp. 8491–8498. Available at: https://doi.org/10.1073/pnas.0703550105.

      Vanderelst, D. and Peremans, H. (2018) ‘Modeling bat prey capture in echolocating bats : The feasibility of reactive pursuit’, Journal of theoretical biology, 456, pp. 305–314.

      Youssefi, K.A.R. and Rouhani, M. (2021) ‘Swarm intelligence based robotic search in unknown maze-like environments’, Expert Systems with Applications, 178. Available at: https://doi.org/10.1016/j.eswa.2021.114907.

      Yovel, Y. et al. (2009) ‘The voice of bats: How greater mouse-eared bats recognize individuals based on their echolocation calls’, PLoS Computational Biology, 5(6). Available at: https://doi.org/10.1371/journal.pcbi.1000400.

      Yovel, Y. and Ulanovsky, N. (2017) ‘Bat Navigation’, The Curated Reference Collection in Neuroscience and Biobehavioral Psychology, pp. 333–345. Available at: https://doi.org/10.1016/B978-0-12-809324-5.21031-6.

    1. Author response:

      We thank the reviewers for their thorough evaluation and constructive feedback on our manuscript.

      We think that their valuable suggestions will strengthen the manuscript and help us clarify several important points.

      All reviewers acknowledged the importance of our theoretical results and network classification in making pattern formation analysis a more tractable problem. At the same time, they have also raised a number of important concerns that we shall carefully consider.

      A. A major clarification that the reviewers found important concerns the definition of non-trivial pattern transformations and its generalization to higher dimensions. In this regard, the reviewers’ comments are:

      Reviewer #1:

      (on non-trivial pattern transformations):

      (3) All modelling is confined to one spatial dimension, and the very definition of a "non-trivial" transformation is framed in terms of peak positions along a line, which clearly must be reformulated for higher dimensions. It's well-known that diffusions in 1, 2, and 3 dimensions are also dramatically different, so the relevance of the three-class taxonomy to real multicellular tissues remains unclear, or at least should be explained in more detail. Reviewer #2 (on non-trivial pattern transformations):

      (5) The definition of non-trivial pattern formation is provided only in the Supplementary Information, despite its central importance for interpreting the main results. It would significantly improve clarity if this definition were included and explained in the main text. Additionally, it remains unclear how the definition is consistently applied across the different initial conditions. In particular, the authors should clarify how slope-based measures are determined for both the random noise and sharp peak/step function initial states. Furthermore, the authors do not specify how the sign function is evaluated at zero. If the standard mathematical definition sgn(0)=0 is used, then even a simple widening of a peak could fulfill the criterion for nontrivial pattern transformation.

      We agree with Reviewer #2 that including a more detailed definition of non-trivial pattern transformation in the main text would enhance the clarity of the paper. The one-dimensional (1D) definition currently provided in the Supplementary Information was chosen because all computations presented therein involve exclusively one-dimensional patterns. However, we acknowledge that this definition, as it was, did not have a totally unambiguous generalization  to higher dimensions. Therefore, in a revised version of the manuscript, we will incorporate an expanded definition applicable to higher-dimensional cases.

      This general definition of a non-trivial pattern transformation should make no reference to the sign of spatial derivatives of either the initial or resulting patterns. Specifically, a pattern transformation is considered non-trivial if it satisfies the following criteria:

      - It is heterogeneous: The resulting pattern is heterogeneous in space.

      - It is rearranging: The arrangement of critical points (i.e. peaks, valleys and saddle points in a gene product concentration) along the domain in the resulting pattern of a gene product is different to the arrangement of critical points in its initial pattern. This includes the emergence of new critical points, the disappearance of existing ones, or the spatial displacement of critical points from one location to another.

      - It is non-replicating: The spatial arrangement of critical points in the pattern of one gene product must differ from that of any other upstream gene product.

      Nonetheless, our two initial patterns are spatially discontinuous functions: in homogeneous initial patterns, the white noise is discontinuous by definition; and for the spike and spike+homogeneous initial patterns, we use sharp spikes defined by the rectangular function, which is discontinuous at the spike boundaries. Therefore, the aforementioned definition should be supplemented with the following two ad hoc assumptions:

      - Homogeneous initial patterns do not comprise any critical point. White noise in this type of initial patterns represents small thermodynamic fluctuations around the steady state and, for the purpose of pattern transformation, this is equivalent to a constant concentration along the domain.

      - Spike and spike+homogeneous initial patterns each contain a single critical point located at the center of the spike. The sharp spikes, modeled using the rectangular function, serve as a theoretical idealization to facilitate mathematical analysis. Once diffusion begins to act, these sharp boundaries are smoothed into differentiable gradients, maintaining a unique critical point at the center of the initial spike, which is the most relevant information for pattern transformation.

      Finally, it is worth recalling that our gene network classification is fundamentally based on an analysis of the dispersion relation associated with the gene network, and the construction of this dispersion relation is independent of the spatial dimensionality of the domain (i.e. it does not require assuming any specific number of dimensions). The fact that the description of this dispersion relation was in the SI may have been non-ideal for the understandability of the article and will, consequently, be moved to the main text in an upcoming version of the article. Thus, the gene networks that can lead to pattern transformation are the same in 1D, 2D or 3D. As for the resulting patterns, the broad description we provide also applies to any number of dimensions; these would be periodic, non periodic as in the amplified noise patterns or non periodic as in the hierarchic networks. For the latter notice that, except for boundary effects that we later discuss, the spike initial condition is radially symmetric and thus, the patterns resulting from it will also be radially symmetric. We will make this point more explicit in a revised version of the article, especially since, as suggested, this important portion of the Supplementary Information will be incorporated into the main text.

      Reviewer 2 suggests that with our definition of non-trivial pattern transformation, the simple widening of a concentration peak would constitute a non-trivial pattern transformation. This is not the case, as already shown in the figures as a example, since in a widening there is no change in the position of the critical point. A different situation applies if a wide and completely flat concentration peak (i.e. a plateau) forms. As we will explain in the coming version this is not possible because of requirement R5.

      We think that this clarification of the definition of non-trivial pattern transformation will also help clarify the next point (B below) since it would make it clearer that this article does not intend to explain which specific resulting pattern would arise from any given gene network.

      B. The main concern among these relates to the validity of our linearization of the model equations and the extension of the results obtained for the linear system to the fully nonlinear system. In this regard, the reviewers’ comments are:

      Reviewer #1:

      (on linearization):

      (2) A central step in the model formulation is the linearisation of the reaction term around a homogeneous steady state; higher-order kinetics, including ubiquitous bimolecular sinks such as A + B → AB, are simply collapsed into the Jacobian without any stated amplitude bound on the perturbations. Because the manuscript never analyses how far this assumption can be relaxed, the robustness of the three-class taxonomy under realistic nonlinear reactions or large spike amplitudes remains uncertain.

      Reviewer #2:

      (on linearization):

      (2) Most of the proofs presented in the Supplementary Information rely on linearized versions of the governing equations, and it remains unclear how these results extend to the fully nonlinear system. We are concerned that the generality of the conclusions drawn from the linear analysis may be overstated in the main text. For example, in Section S3, the authors introduce the concept of dynamic equivalence of transitive chains (Proposition S3.1) and intracellular transitive M-branching (Proposition S3.2), which pertains to the system's steady-state behavior. However, the proof is based solely on the linearized equations, without additional justification for why the result should hold in the presence of nonlinearities. Moreover, the linearized system is used to analyze the response to a "spike initial pattern of arbitrary height C" (SI Chapter S5.1), yet it is not clear how conclusions derived from the linear regime can be valid for large perturbations, where nonlinear effects are expected to play a significant role. We encourage the authors to clarify the assumptions under which the linearized analysis remains valid and to discuss the potential limitations of applying these results to the nonlinear regime.

      In this article, we address two main questions: first, which gene network topologies can give rise to non-trivial pattern transformations; and second, which broad types of resulting patterns can these gene network topologies give rise to resulting pattern. Thus, we are not intending to explain which exact resulting patterns would arise from any given gene network (i.e. a gene network topology with specific functions and interaction strengths or weights), a question for which non-linearities do indeed matter.

      For most known gene regulatory networks, available empirical information is typically limited to the nature of gene product regulations -indicating whether they act as activators or inhibitors- while details about the specific functional form of these regulations are rare. For instance, given two gene products, i and j, the network may indicate that i acts as an activator of j, implying that the concentration of j increases with that of i. However, this increase could follow a variety of functional forms: it may be quadratic (e.g., ), cubic (e.g., ), or any other function f j(gi). As we explain in the description of our model, we restrict our study to functions with a monotonicity constraint: higher concentrations of i lead to increased production of j (i.e., ).  In other words, a given gene interaction is always inhibitory or activatory, it does not change of sign. This monotonicity constraint corresponds to requirement (R5) in our main text. This requirement it is based on the biologically plausible idea that the complexity of gene regulation in development stems more from the topology of gene networks than from the complexity of the regulation by which a gene product may regulate another (i.e. we use simple monotonic functions).

      Question 1: A critical part to understand question 1 is in the dispersion relation that was explained in SI. From the reviewers’ comments it is clear that having this crucial part in the main text of an upcoming version of the article would improve understandability, specially for question 1.

      In brief, any pattern transformation requires the initial pattern to change. The trigger of such change is a change in the concentration of some gene product, either conceptualized as a noise fluctuation (in the homogeneous initial pattern) or a regulated change in a specific point (in the spike initial pattern). Mathematically, both can be conceptualized as perturbations and, for pattern transformation to be possible, such perturbation should grow so that the initial pattern becomes unstable and can change to another resulting pattern.

      If the perturbation is small, one can use the standard linear perturbation analysis in S6.2 of our Supplementary Information. In other words, the linear analysis is enough to ascertain if a small perturbation would grow or not. A gene network in which this will not happen would be unable to lead to pattern transformation, whichever the nonlinear part of f(g). In that sense, the linear approximation provides a necessary condition that any gene network needs to fulfill to lead to pattern transformation.

      However, the linear analysis would not ascertain whether a specific gene network will actually lead to pattern transformation (i.e., the condition is not sufficient). This, as well as the shape of the specific resulting pattern, may actually depend on the non-linear parts too. As we discuss, based on the dispersion relation, and other complementing arguments along the article, we can also get some insights on the possible patterns from the linear approximation alone (question 2). This arguments hold thanks to the imposition of requirements (R1-R5) on function f(g), which prevent strange behaviors stemming from the nonlinear part of the equation.

      The amplitude bound of perturbations mentioned by Reviewer #1 is addressed by requirements (R2) and (R4). Although the solution to the linear system predicts unbounded growth of unstable eigenmodes, the assume functions f(g) on which the nonlinear terms  eventually halt this growth, thereby ensuring the boundedness of solutions as imposed by (R4). This assumption on the nonlinear part is literally requirement R2 on f(g) in the main text.

      The transitive chains and branchings in section S3 of the Supplementary Information mentioned by the Reviewer #2 are topological properties of gene networks and therefore they influence only the linear part of the reaction-diffusion equations. This is why the proofs in that section are based on the linearized equations. We agree that clarifying this point in the text, as suggested by the reviewer, would improve the reader’s understanding of the section.

      Regarding Reviewer #2’s concerns about large perturbations, we acknowledge that the phrasing using “arbitrary height” may be confusing. For the homogeneous initial conditions these perturbations are assumed to be small because they are actually molecular noise (otherwise the initial condition could not be considered homogenous in the classical sense of developmental biology models). In the spike initial conditions in hierarchic networks the perturbation is not necessarily small. For the analysis provided in the SI we indeed assume that the perturbations are small enough for the linear approximation to be possible. Notice, however, that since these networks require an intracellular self-activating loop upstream of the first extracellular signal, the effective perturbation would rapidly grow to a value determined by such loop.

      In general the height of the initial spike does not affect the fact that hierarchic networks can lead to non-trivial pattern transformation. By definition these networks require the secretion of an extracellular signal from the cells in the spike (otherwise no change in gene product concentrations can occur over space). By definition this signal is not produced by any other cells and, thus, its concentration is governed by diffusion from the spike and its production in the cells in the spike. Thus, whichever the initial height of the spike and whichever the non-linearities in f(g), the signal’s concentration would decrease with the distance from the spike. As explained in the main text, this would lead to non-trivial pattern transformations if other general conditions are met. In general, the height of the initial perturbation can affect which specific pattern transformation would arise from a specific gene network but not which gene network topologies can lead to pattern transformation. This will be more clearly stated in an upcoming version of the article. C. In the following, we respond to the remaining concerns raised by the reviewers:

      Reviewer #1:

      (1) The Results section is difficult to follow. Key logical steps and network configurations are described shortly in prose, which constantly require the reader to address either SI or other parts of the text (see numerous links on the requirements R1-R5 listed at the beginning of the paper) to gain minimal understanding. As a result, a scientifically literate but non-specialist reader may struggle to grasp the argument with a reasonable time invested.

      We acknowledge that the current version of the main text may not be as clear as we intended. Initially, we believed that placing the more technical mathematical passages in the Supplementary Information would make the main text more accessible to readers. However, we agree with the reviewer that including some of these computations in the main text could improve clarity. We also believe that adding a summary table outlining all the model’s requirements would further contribute to that goal.

      Reviewer #2:

      (1) We have serious concerns regarding the validity of the simulation results presented in the manuscript. Rather than simulating the full nonlinear system described by Equation (1), the authors base their results on a truncated expansion (Equation S.8.2) that captures only the time evolution of small deviations around a spatially homogeneous steady state. However, it remains unclear how this reduced system is derived from the full equations specifically, which terms are retained or neglected and why- and how the expansion of the nonlinear function can be steady-state independent, as claimed. Additionally, in simulations involving the spike plus homogeneous initial condition, it is not evident -or, where equations are provided, it is not correct- that the assumed global homogeneous background actually corresponds to a steady state of the full dynamics. We elaborate on these concerns in the following:

      We believe there has been a misunderstanding regarding the presentation of the model equations (S8.2) used throughout our simulations. Accordingly, we agree that this relevant section of the Supplementary Information should be rewritten in a revised version of the manuscript to clarify this issue. Below, we address all the concerns raised by the reviewer.

      Equation (S8.2) represents the full nonlinear system described in Equation (1). While we recognize that the model may oversimplify real biological processes, its purpose is to illustrate our general statements about pattern formation rather than to capture any specific or detailed mechanism. In this context, model (S8.2) offers three key advantages for our goals: it allows rapid manipulation of gene network topology simply by modifying the matrix J, making it ideal for illustrating pattern formation across different network classes; it accommodates gene networks of arbitrary size -unlike other models, such as the classical Gierer-Meinhardt model, which are limited to two-element Turing or noise-amplifying networks-; and, due to the simplicity of its nonlinear terms, this model involves relatively few free parameters, facilitating the fine-tuning needed to identify parameter regions where non-trivial pattern transformations occur.

      Indeed, we find that the ability of model (S8.2) to illustrate our results despite having such simple nonlinear terms -bearing in mind that at least some nonlinearity is always necessary for selforganization- strongly supports the claim that the capacity of a gene network to produce pattern transformations is fully determined by the linear part of Equation (1). In this sense, nonlinear terms primarily influence the precise parameter values at which these transformations occur and contribute to shaping specific features of the resulting patterns.

      Model (S8.2) has been successfully employed in pattern formation studies elsewhere in the literature; accordingly, we provide relevant bibliographic references to support its widespread use.

      We believe the misunderstanding arises from our explanation of the biological interpretation of the model. As noted in the accompanying bibliography, the model is based on a general reactiondiffusion mechanism assuming the existence of a steady state. However, this conceptual reactiondiffusion framework is not the same as our Equation (1); rather, it was introduced by the original proponents of the model in the seminal paper cited in our text. In this context, Equation (S8.2) describes small concentration perturbations around that steady state, where the variables represent deviations in concentration relative to the general steady state.

      The aforementioned general steady state corresponds to the trivial equilibrium point g≡0 in equations (S8.2). Consequently, all our simulations based on model (S8.2) start from this steady state, to which we add white noise to generate homogeneous initial patterns or a sharp spike for the two types of spike initial patterns.

      It is also worth noting that Equations (S8.2) represent a non-dimensional model.

      It is assumed that the homogeneous steady states are given by g_i=0 and g_i=c_i, where 1/c_i = \mu_i or \hat{\mu}_i, independently of the specific network structure. However, the basis for this assumption is unclear, especially since some of the functions do not satisfy this condition -for example, f5 as defined below Eq. S8.10.5. Moreover, if g_i=c_i does not correspond to a true steady state, then the time evolution of deviations from this state is not correctly described by Eq. S8.2, as the zeroth-order terms do not vanish in that case.

      From the explanations above, it is important to distinguish two scales in the process: the scale of small perturbations, where equations (S8.2) apply; and the global scale, where the conceptual general reaction-diffusion system operates. Since the specific form of this general system does not affect equations (S8.2), we assume that it follows any of the models cited in the text, which yield a non-zero steady state at .

      In this sense, Equation (S8.2) represent a small concentration deviation of such global system and g(t ,x) is a relative concentration where g≡0 represents the steady-state at are concentrations above , and g<0 are concentrations below .

      As previously mentioned, simulations are performed using Equations (S8.2) on the basis of the equilibrium point g≡0. The result of these simulations is then superimposed on the non-zero steady state and presented in the figures along the article.

      Using the full model instead of the simplified Equations (S8.2) may result in slightly different resulting patterns, but it does not affect the gene network’s ability to produce pattern transformations, nor does it alter the main structural properties of the patterns—for example, the periodic nature of patterns generated by Turing networks.

      Additionally, the equations used contain only linear terms and a cubic degradation term for each species g_i, while neglecting all quadratic terms and cubic terms involving cross-species interactions (i≠j). An explanation for this selective truncation is not provided, and without knowledge of the full equation (f), it is impossible to assess whether this expansion is mathematically justified. If, as suggested in the Supplementary Information, the linear and cubic terms are derived from f, then at the very least, the Jacobian matrix should depend on the background steady-state concentration. However, the equations for the small deviation around a steady state (including the Jacobian matrix) used in the simulations appear to be independent of the particular steady state concentration.

      The Jacobian of Equation (S8.2) is independent of g because g represents a small perturbation around a steady state of a general reaction-diffusion system. Consequently, the matrix J corresponds to the Jacobian of the general system evaluated at that steady state. Evaluating the Jacobian of equations (S8.2) at the equilibrium point g≡0 -which represents the general steady state- recovers the matrix J.

      This is why we believe that the differences observed between the spike-only initial condition and the spike superimposed on a homogeneous background are not due to the initial conditions themselves, but rather result from a modified reaction scheme introduced through a questionable cutoff.

      "In simulations with spike initial patterns, the reference value g≡0 represents an actual concentration of 0 and therefore, we must add to (S8.2) a Heaviside function Φ acting of f (i.e., Φ(f(g))=f(g) if f(g)>0 , Φ(f(g))=0 if f(g){less than or equal to}0 ) to prevent the existence of negative concentrations for any gene product (i.e., g_i<0 for some i )." (SI chapter S8).

      This cutoff alters the dynamics (no inhibition) and introduces a different reaction scheme between the two simulations. The need for this correction may itself reflect either a problem in the original equations (which should fulfill the necessary conditions and prevent negative concentrations (R4 in main text)) or the inappropriateness of using an expanded approximation which assumes independence on the steady state concentration. It is already questionable if the linearized equations with a cubic degradation term are valid for the spike initial conditions (with different background concentration values), as the amplitude of this perturbation seems rather large.

      For homogeneous and spike+homogeneous initial conditions, we interpret equations (S8.2) as small perturbations around a non-zero steady state of a general reaction-diffusion system. For spike-only initial conditions, that steady state is zero. As we mention before, g≡0 will then represent such steady-state of zero concentration, g>0 are positive concentrations of the general system, and g<0 would represent unfeasible negative concentrations of the general system. Therefore, the use of a cutoff function to handle such initial conditions is justified. Moreover, this cutoff function is the same as the one employed in the reference general system cited in our paper.

      We acknowledge that the cutoff influences the simulations and accounts for the differences observed between spike and spike+homogeneous initial conditions. However, this distinction reflects what occurs in real biological systems, which is precisely why we differentiate these two types of initial states. For instance, the emergence of a periodic pattern in a noise-amplifying network depends critically on the formation of regions with concentrations below the steady state near the initial spike. Such regions can form in spike-plus-homogeneous initial patterns but not in spike-only initial patterns, where concentrations below the steady state would correspond to biologically unfeasible negative values.

      Lastly, we note that under the current simulation scheme, it is not possible to meaningfully assess criteria RH2a and RH2b, as they rely on nonlinear interactions that are absent from the implemented dynamics.

      It is explicitly stated in the relevant subsections of Section S7 in the Supplementary Information that, for the simulations involving RH2a and RH2b, the function f(g) in equation (S8.2) is modified by adding an ad hoc quadratic term to enable the assessment of these criteria.

      (3) Several statements in the main text are presented without accompanying proof or sufficient explanation, which makes it difficult to assess their validity. In some cases, the lack of justification raises serious doubts about whether the claims are generally true. Examples are:

      "For the purpose of clarity we will explain our results as if these cells have a simple arrangement in space (e.g., a 1D line or a 2D square lattice) but, as we will discuss, our results shall apply with the same logic to any distribution of cells in space." (Main text l.145-l.148).

      We believe that the confusion in this statement arises from the ambiguous use of the phrase “our results”. We will revise the text to provide a more precise description. Specifically, by “our results,” we refer to the conclusion that it is possible to determine whether a gene network leads to nontrivial pattern transformations based solely on its topology. This conclusion is independent of the dimensionality of space, as none of our arguments rely on assumptions specific to spatial dimensions. While one-dimensional examples are used for clarity and illustration, the underlying reasoning applies generally. In an improved version of the article, we will clarify this point explicitly and move relevant arguments from the Supplementary Information into the main text.

      Critically, our classification of gene networks is ultimately based on an argument concerning the dispersion relation associated with the network, and the construction of this dispersion relation is independent of the spatial dimensionality of the domain. In this sense, the networks identified in the text as capable of producing pattern transformations will be able to generate non-trivial pattern transformations in any spatial domain and in any number of dimensions. While the specific parameter values that permit such transformations may vary depending on the geometry and dimensionality of the domain, the existence of at least one such parameter set remains unaffected.

      The geometry of the domain can influence the specific form of the resulting patterns, but it does not alter the broader class of patterns (e.g., periodic patterns, peaks emerging around a spike, etc.) that a given gene network topology can produce. One such geometric influence, commonly observed in simulations, involves boundary effects. For example, structures such as peaks or rings forming near the boundaries may appear higher, broader, or spatially shifted compared to those arising in the central regions of the domain. However, we think a pattern consisting of a periodic train of peaks where only those near the boundary are slightly different can still be classified as a periodic pattern.

      "For any non-trivial pattern transformation (as long as it is symmetric around the initial spike), there exists an H gene network capable of producing it from a spike initial pattern." (Main text l.366f).

      A justification for this statement is provided shortly after the claim, although we acknowledge that the current explanation is somewhat cumbersome and would benefit from a clearer presentation in a revised version of the main text.

      A more detailed justification is provided in the Supplementary Information, based on three key ideas. First, any pattern (provided it is symmetric with respect to the initial spike) can be described as an arrangement of peaks with varying heights and spatial positions along a one-dimensional domain. Second, there exists a simple gene network—the diamond network—that, through parameter tuning, can produce two peaks of arbitrary height and symmetric position relative to the initial spike. Third, by placing multiple diamond networks positively upstream of a common gene product, that gene product can express peaks at each location where the upstream diamond networks induce them. Under mild additional conditions, this mechanism allows the formation of essentially any symmetric pattern. These mild conditions, along with a detailed analysis of the diamond network’s ability to generate peaks with controllable height and position, are discussed in the Supplementary Information.

      "In 2D there are no peaks but concentric rings of high gene product concentration centered around the spike, while in 3D there are concentric spherical shells." (Main text l. 447ff).

      This result pertains specifically to pattern transformations arising from spike initial patterns. As defined in the text, spike initial patterns are radially symmetric. Since diffusion preserves radial symmetry, pattern transformations from spike initial patterns in two or three dimensions reduce to effectively one-dimensional transformations along each radial direction. In this framework, each pair of concentration peaks symmetric with respect to the spike in one dimension corresponds to a ring surrounding the spike in two dimensions, and each ring in two dimensions becomes a hollow spherical shell around the spike in three dimensions.

      We agree that including a brief section in the Supplementary Information to clarify these subtleties would be helpful for readers to better understand the generalization of certain patterns to higher dimensions.

      (4) The study identifies one-signal networks and examines how combinations of these structures can give rise to minimal pattern-forming subnetworks. However, the analysis of the combinations of these minimal pattern-forming subnetworks remains relatively brief, and the manuscript does not explore how the results might change if the subnetworks were combined in upstream and downstream configurations. In our view, it is not evident that all possible gene regulatory networks can be fully characterized by these categories, nor that the resulting patterns can be reliably predicted. Rather, the approach appears more suited to identifying which known subnetworks are present within a larger network, without necessarily capturing the full dynamics of more complex configurations.

      We acknowledge that our explanation regarding the combination of sub-networks was relatively brief, and we intend to address this in a revised version. Our argument that combining sub-networks does not produce qualitatively new types of pattern transformations -beyond those already described- is based on the dispersion relation. Although this relation was only detailed in the Supplementary Information, it is central to our argument and will therefore be moved to the main text. Below, we provide an outline of this argument:

      Our study identifies two distinct behaviors of the principal branch of the dispersion relation at large wavenumbers. Based on this, gene networks capable of pattern formation can be classified into two categories: networks of the first kind, where the real part of the principal branch diverges to infinity as the wavenumber increases; and networks of the second kind, where the real part of the principal branch converges to a positive finite value for large wavenumbers. Naturally this argument applies to any gene network irrespectively of which, or how many, sub-networks are used to built it.

      Any gene regulatory network capable of pattern formation falls into one of these two categories. We identified that networks of the first kind contain at least one Turing sub-network, whereas networks of the second kind include either an H sub-network or a noise-amplifying sub-network. In this way, the primary objective of our study -namely, achieving a topological classification of gene regulatory networks capable of pattern formation- is fulfilled. It is important to note that while the dispersion relation provides broad information about the possible resulting patterns a gene network topology can produce (e.g., periodic versus noisy), it does not specify the exact patterns that emerge for each particular set of parameter values.

      Finally, regarding the shape of the resulting patterns, Figure S10 in the Supplementary Information exemplifies the notion that the behavior of combined networks can be understood as a combination of the individual behaviors of each constituent sub-network (note that the contribution of each type of sub-network in the resulting pattern is readily distinguishable). Consequently, we focus our detailed analysis on the patterning properties of the fundamental classes.

      (6) The manuscript lacks a clear and detailed explanation of the underlying model and its assumptions. In particular, it is not well-defined what constitutes a "cell" in the context of the model, nor is it justified why spatial features of cells -such as their size or boundaries- can be neglected. Furthermore, the concept of the extracellular space in the one-dimensional model remains ambiguous, making it unclear which gene products are assumed to diffuse.

      The size of cells is ignored in our model because we assume that they are small enough with respect to the total size of the domain that the space continuous reaction-diffusion equation (equation (1) in the main text) holds. Conceptually, one could understand cells in our model each of the pieces in an even partition of the domain into small subdomains surrounding each position x. This is anyway the standard procedure in most models of pattern formation by reaction-diffusion in embryonic development.

      For extracellular signals, we assume that g(t ,x) corresponds to the concentration of the signal in the extracellular space surrounding the cell located at position x. The extracellular space is any fluid medium for which Fick Laws apply and, therfore, the Fickian diffusion term in equation (1) is valid.

      For intracellular gene products, we assume that g(t ,x) corresponds to the concentration of such gene product within the cell at position x (if the gene product in hand is a transcription factor, for example), or on its surface (if it is a membrane-bound receptor). When collapsed in the continuous equations there is not such difference between being strictly within the cell or on its boundary. The only important fact is that these gene products cannot diffuse.

      Regarding cell boundaries, let us consider an extracellular signal s that regulates a transcriptor factor i within cells (in our model, i is an intracellular gene product). Such regulation shall be mediated by a membrane-bound receptor, which corresponds to intracellular gene product j. In terms of the gene regulatory network this is sji. Cell boundary effects mentioned by the reviewer should be encapsulated in the specific functional form of the regulation function f(g), but they have no effect in the actual topology of the network. Consequently, they are out of the scope of this study: as we mentioned before, considering different non-linear terms for f(g) will affect the parameter range for which a gene network is capable of producing non-trivial pattern transformations, but not their overall ability to produce non-trivial pattern transformations (i.e., the existence of at least one choice of model parameters for which such transformations take place).

      Finally, we would like to once again express our sincere gratitude to all reviewers for their insightful and constructive feedback. We are confident that the thorough peer review process will significantly enhance both the clarity and depth of our work. We greatly value the detailed comments provided and will carefully incorporate them in the preparation of a revised manuscript, which we intend to submit in the coming months.

    1. Author Response

      Public Reviews:

      Reviewer #1 (Public Review):

      Summary:

      Given knowledge of the amino acid sequence and of some version of the 3D structure of two monomers that are expected to form a complex, the authors investigate whether it is possible to accurately predict which residues will be in contact in the 3D structure of the expected complex. To this effect, they train a deep learning model that takes as inputs the geometric structures of the individual monomers, per-residue features (PSSMs) extracted from MSAs for each monomer, and rich representations of the amino acid sequences computed with the pre-trained protein language models ESM-1b, MSA Transformer, and ESM-IF. Predicting inter-protein contacts in complexes is an important problem. Multimer variants of AlphaFold, such as AlphaFold-Multimer, are the current state of the art for full protein complex structure prediction, and if the three-dimensional structure of a complex can be accurately predicted then the inter-protein contacts can also be accurately determined. By contrast, the method presented here seeks state-of-the-art performance among models that have been trained end-to-end for inter-protein contact prediction.

      Strengths:

      The paper is carefully written and the method is very well detailed. The model works both for homodimers and heterodimers. The ablation studies convincingly demonstrate that the chosen model architecture is appropriate for the task. Various comparisons suggest that PLMGraph-Inter performs substantially better, given the same input than DeepHomo, GLINTER, CDPred, DeepHomo2, and DRN-1D2D_Inter. As a byproduct of the analysis, a potentially useful heuristic criterion for acceptable contact prediction quality is found by the authors: namely, to have at least 50% precision in the prediction of the top 50 contacts.

      We thank the reviewer for recognizing the strengths of our work!

      Weaknesses:

      My biggest issue with this work is the evaluations made using bound monomer structures as inputs, coming from the very complexes to be predicted. Conformational changes in protein-protein association are the key element of the binding mechanism and are challenging to predict. While the GLINTER paper (Xie & Xu, 2022) is guilty of the same sin, the authors of CDPred (Guo et al., 2022) correctly only report test results obtained using predicted unbound tertiary structures as inputs to their model. Test results using experimental monomer structures in bound states can hide important limitations in the model, and thus say very little about the realistic use cases in which only the unbound structures (experimental or predicted) are available. I therefore strongly suggest reducing the importance given to the results obtained using bound structures and emphasizing instead those obtained using predicted monomer structures as inputs.

      We thank the reviewer for the suggestion! We evaluated PLMGraph-Inter with the predicted monomers and analyzed the result in details (see the “Impact of the monomeric structure quality on contact prediction” section and Figure 3). To mimic the real cases, we even deliberately reduced the performance of AF2 by using reduced MSAs (see the 2nd paragraph in the ““Impact of the monomeric structure quality on contact prediction” section). We leave some of the results in the supplementary of the current manuscript (Table S2). We will move these results to the main text to emphasize the performance of PLMGraph-Inter with the predicted monomers in the revision.

      In particular, the most relevant comparison with AlphaFold-Multimer (AFM) is given in Figure S2, not Figure 6. Unfortunately, it substantially shrinks the proportion of structures for which AFM fails while PLMGraph-Inter performs decently. Still, it would be interesting to investigate why this occurs. One possibility would be that the predicted monomer structures are of bad quality there, and PLMGraph-Inter may be able to rely on a signal from its language model features instead. Finally, AFM multimer confidence values ("iptm + ptm") should be provided, especially in the cases in which AFM struggles.

      We thank the reviewer for the suggestion! Yes! The performance of PLMGraph-Inter drops when the predicted monomers are used in the prediction. However, it is difficult to say which is a fairer comparison, Figure 6 or Figure S2, since AFM also searched monomer templates (see the third paragraph in 7. Supplementary Information : 7.1 Data in the AlphaFold-Multimer preprint: https://www.biorxiv.org/content/10.1101/2021.10.04.463034v2.full) in the prediction. When we checked our AFM runs, we found that 99% of the targets in our study (including all the targets in the four datasets: HomoPDB, HeteroPDB, DHTest and DB5.5) employed at least 20 templates in their predictions, and 87.8% of the targets employed the native templates. We will provide the AFM confidence values of the AFM predictions in the revision.

      Besides, in cases where any experimental structures - bound or unbound - are available and given to PLMGraph-Inter as inputs, they should also be provided to AlphaFold-Multimer (AFM) as templates. Withholding these from AFM only makes the comparison artificially unfair. Hence, a new test should be run using AFM templates, and a new version of Figure 6 should be produced. Additionally, AFM's mean precision, at least for top-50 contact prediction, should be reported so it can be compared with PLMGraph-Inter's.

      We thank the reviewers for the suggestion! We would like to notify that AFM also searched monomer templates (see the third paragraph in 7. Supplementary Information : 7.1 Data in the AlphaFold-Multimer preprint: https://www.biorxiv.org/content/10.1101/2021.10.04.463034v2.full) in the prediction. When we checked our AFM runs, we found that 99% of the targets in our study (including all the targets in the four datasets: HomoPDB, HeteroPDB, DHTest and DB5.5) employed at least 20 templates in their predictions, and 87.8% of the targets employed the native template.

      It's a shame that many of the structures used in the comparison with AFM are actually in the AFM v2 training set. If there are any outside the AFM v2 training set and, ideally, not sequence- or structure-homologous to anything in the AFM v2 training set, they should be discussed and reported on separately. In addition, why not test on structures from the "Benchmark 2" or "Recent-PDB-Multimers" datasets used in the AFM paper?

      We thank the reviewer for the suggestion! The biggest challenge to objectively evaluate AFM is that as far as we known, AFM does not release the PDB ids of its training set and the “Recent-PDB-Multimers” dataset. “Benchmark 2” only includes 17 heterodimer proteins, and the number can be further decreased after removing targets redundant to our training set. We think it is difficult to draw conclusions from such a small number of targets. In the revision, we will analyze the performance of AFM on targets released after the date cutoff of the AFM training set, but with which we cannot totally remove the redundancy between the training and the test sets of AFM.

      It is also worth noting that the AFM v2 weights have now been outdated for a while, and better v3 weights now exist, with a training cutoff of 2021-09-30.

      We thank the reviewer for reminding the new version of AFM. The only difference between AFM V3 and V2 is the cutoff date of the training set. Our test set would have more overlaps with the training set of AFM V3, which is one reason that we think AFM V2 is more appropriate to be used in the comparison.

      Another weakness in the evaluation framework: because PLMGraph-Inter uses structural inputs, it is not sufficient to make its test set non-redundant in sequence to its training set. It must also be non-redundant in structure. The Benchmark 2 dataset mentioned above is an example of a test set constructed by removing structures with homologous templates in the AF2 training set. Something similar should be done here.

      We agree with the reviewer that testing whether the model can keep its performance on targets with no templates (i.e. non-redundant in structure) is important. We will perform the analysis in the revision.

      Finally, the performance of DRN-1D2D for top-50 precision reported in Table 1 suggests to me that, in an ablation study, language model features alone would yield better performance than geometric features alone. So, I am puzzled why model "a" in the ablation is a "geometry-only" model and not a "LM-only" one.

      Using the protein geometric graph to integrate multiple protein language models is the main idea of PLMGraph-Inter. Comparing with our previous work (DRN-1D2D_Inter), we consider the building of the geometric graph as one major contribution of this work. To emphasize the efficacy of this geometric graph, we chose to use the “geometry-only” model as the base model. We will further clarity this in the revision.

      Reviewer #2 (Public Review):

      This work introduces PLMGraph-Inter, a new deep-learning approach for predicting inter-protein contacts, which is crucial for understanding protein-protein interactions. Despite advancements in this field, especially driven by AlphaFold, prediction accuracy and efficiency in terms of computational cost) still remains an area for improvement. PLMGraph-Inter utilizes invariant geometric graphs to integrate the features from multiple protein language models into the structural information of each subunit. When compared against other inter-protein contact prediction methods, PLMGraph-Inter shows better performance which indicates that utilizing both sequence embeddings and structural embeddings is important to achieve high-accuracy predictions with relatively smaller computational costs for the model training.

      The conclusions of this paper are mostly well supported by data, but test examples should be revisited with a more strict sequence identity cutoff to avoid any potential information leakage from the training data. The main figures should be improved to make them easier to understand.

      We thank the reviewer for recognizing the significance of our work! We will revise the manuscript carefully to address the reviewer’s concerns.

      1. The sequence identity cutoff to remove redundancies between training and test set was set to 40%, which is a bit high to remove test examples having homology to training examples. For example, CDPred uses a sequence identity cutoff of 30% to strictly remove redundancies between training and test set examples. To make their results more solid, the authors should have curated test examples with lower sequence identity cutoffs, or have provided the performance changes against sequence identities to the closest training examples.

      We thank the reviewer for the valuable suggestion! Using different thresholds to reduce the redundancy between the test set and the training set is a very good suggestion, and we will perform the analysis in the revision. In the current version of the manuscript, the 40% sequence identity is used as the cutoff for many previous studies used this cutoff (e.g. the Recent-PDB-Multimers used in AlphaFold-Multimer (see: 7.8 Datasets in the AlphaFold-Multimer paper); the work of DSCRIPT: https://www.cell.com/action/showPdf?pii=S2405-4712%2821%2900333-1 (see: the PPI dataset paragraph in the METHODS DETAILS section of the STAR METHODS)). One reason for using the relatively higher threshold for PPI studies is that PPIs are generally not as conserved as protein monomers.

      We performed a preliminary analysis using different thresholds to remove redundancy when preparing this provisional response letter:

      Author response table 1.

      Table1. The performance of PLMGraph-Inter on the HomoPDB and HeteroPDB test sets using native structures(AlphaFold2 predicted structures).

      Method:

      To remove redundancy, we clustered 11096 sequences from the training set and test sets (HomoPDB, HeteroPDB) using MMSeq2 with different sequence identity threshold (40%, 30%, 20%, 10%) (the lowest cutoff for CD-HIT is 40%, so we switched to MMSeq2). Each sequence is then uniquely labeled by the cluster (e.g. cluster 0, cluster 1, …) to which it belongs, from which each PPI can be marked with a pair of clusters (e.g. cluster 0-cluster 1). The PPIs belonging to the same cluster pair (note: cluster n - cluster m and cluster n-cluster m were considered as the same pair) were considered as redundant. For each PPI in the test set, if the pair cluster it belongs to contains the PPI belonging to the training set, we remove that PPI from the test set.

      We will perform more detailed analyses in the revised manuscript.

      1. Figures with head-to-head comparison scatter plots are hard to understand as scatter plots because too many different methods are abstracted into a single plot with multiple colors. It would be better to provide individual head-to-head scatter plots as supplementary figures, not in the main figure.

      We thank the reviewer for the suggestion! We will include the individual head-to-head scatter plots as supplementary figures in the revision.

      3) The authors claim that PLMGraph-Inter is complementary to AlphaFold-multimer as it shows better precision for the cases where AlphaFold-multimer fails. To strengthen the point, the qualities of predicted complex structures via protein-protein docking with predicted contacts as restraints should have been compared to those of AlphaFold-multimer structures.

      We thank the reviewer for the suggestion! We will add this comparison in the revision.

      4) It would be interesting to further analyze whether there is a difference in prediction performance depending on the depth of multiple sequence alignment or the type of complex (antigen-antibody, enzyme-substrates, single species PPI, multiple species PPI, etc).

      We thank the reviewer for the suggestion! We will perform such analysis in the revision.

    1. Author response:

      eLife Assessment 

      This valuable study investigates how the neural representation of individual finger movements changes during the early period of sequence learning. By combining a new method for extracting features from human magnetoencephalography data and decoding analyses, the authors provide incomplete evidence of an early, swift change in the brain regions correlated with sequence learning, including a set of previously unreported frontal cortical regions. The addition of more control analyses to rule out that head movement artefacts influence the findings, and to further explain the proposal of offline contextualization during short rest periods as the basis for improvement performance would strengthen the manuscript. 

      We appreciate the Editorial assessment on our paper’s strengths and novelty.  We have implemented additional control analyses to show that neither task-related eye movements nor increasing overlap of finger movements during learning account for our findings, which are that contextualized neural representations in a network of bilateral frontoparietal brain regions actively contribute to skill learning.  Importantly, we carried out additional analyses showing that contextualization develops predominantly during rest intervals.

      Public Reviews:

      We thank the Reviewers for their comments and suggestions, prompting new analyses and additions that strengthened our report.

      Reviewer #1 (Public review): 

      Summary: 

      This study addresses the issue of rapid skill learning and whether individual sequence elements (here: finger presses) are differentially represented in human MEG data. The authors use a decoding approach to classify individual finger elements and accomplish an accuracy of around 94%. A relevant finding is that the neural representations of individual finger elements dynamically change over the course of learning. This would be highly relevant for any attempts to develop better brain machine interfaces - one now can decode individual elements within a sequence with high precision, but these representations are not static but develop over the course of learning. 

      Strengths: The work follows a large body of work from the same group on the behavioural and neural foundations of sequence learning. The behavioural task is well established and neatly designed to allow for tracking learning and how individual sequence elements contribute. The inclusion of short offline rest periods between learning epochs has been influential because it has revealed that a lot, if not most of the gains in behaviour (ie speed of finger movements) occur in these so-called micro-offline rest periods. The authors use a range of new decoding techniques, and exhaustively interrogate their data in different ways, using different decoding approaches. Regardless of the approach, impressively high decoding accuracies are observed, but when using a hybrid approach that combines the MEG data in different ways, the authors observe decoding accuracies of individual sequence elements from the MEG data of up to 94%. 

      We have previously showed that neural replay of MEG activity representing the practiced skill correlated with micro-offline gains during rest intervals of early learning, 1 consistent with the recent report that hippocampal ripples during these offline periods predict human motor sequence learning2.  However, decoding accuracy in our earlier work1 needed improvement.  Here, we reported a strategy to improve decoding accuracy that could benefit future studies of neural replay or BCI using MEG.

      Weaknesses: 

      There are a few concerns which the authors may well be able to resolve. These are not weaknesses as such, but factors that would be helpful to address as these concern potential contributions to the results that one would like to rule out. Regarding the decoding results shown in Figure 2 etc, a concern is that within individual frequency bands, the highest accuracy seems to be within frequencies that match the rate of keypresses. This is a general concern when relating movement to brain activity, so is not specific to decoding as done here. As far as reported, there was no specific restraint to the arm or shoulder, and even then it is conceivable that small head movements would correlate highly with the vigor of individual finger movements. This concern is supported by the highest contribution in decoding accuracy being in middle frontal regions - midline structures that would be specifically sensitive to movement artefacts and don't seem to come to mind as key structures for very simple sequential keypress tasks such as this - and the overall pattern is remarkably symmetrical (despite being a unimanual finger task) and spatially broad. This issue may well be matching the time course of learning, as the vigor and speed of finger presses will also influence the degree to which the arm/shoulder and head move. This is not to say that useful information is contained within either of the frequencies or broadband data. But it raises the question of whether a lot is dominated by movement "artefacts" and one may get a more specific answer if removing any such contributions. 

      Reviewer #1 expresses concern that the combination of the low-frequency narrow-band decoder results, and the bilateral middle frontal regions displaying the highest average intra-parcel decoding performance across subjects is suggestive that the decoding results could be driven by head movement or other artefacts.

      Head movement artefacts are highly unlikely to contribute meaningfully to our results for the following reasons. First, in addition to ICA denoising, all “recordings were visually inspected and marked to denoise segments containing other large amplitude artifacts due to movements” (see Methods). Second, the response pad was positioned in a manner that minimized wrist, arm or more proximal body movements during the task. Third, while head position was not monitored online for this study, the head was restrained using an inflatable air bladder, and head position was assessed at the beginning and at the end of each recording. Head movement did not exceed 5mm between the beginning and end of each scan for all participants included in the study. Fourth, we agree that despite the steps taken above, it is possible that minor head movements could still contribute to some remaining variance in the MEG data in our study. The Reviewer states a concern that “it is conceivable that small head movements would correlate highly with the vigor of individual finger movements”. However, in order for any such correlations to meaningfully impact decoding performance, such head movements would need to: (A) be consistent and pervasive throughout the recording (which might not be the case if the head movements were related to movement vigor and vigor changed over time); and (B) systematically vary between different finger movements, and also between the same finger movement performed at different sequence locations (see 5-class decoding performance in Figure 4B). The possibility of any head movement artefacts meeting all these conditions is extremely unlikely.

      Given the task design, a much more likely confound in our estimation would be the contribution of eye movement artefacts to the decoder performance (an issue appropriately raised by Reviewer #3 in the comments below). Remember from Figure 1A in the manuscript that an asterisk marks the current position in the sequence and is updated at each keypress. Since participants make very few performance errors, the position of the asterisk on the display is highly correlated with the keypress being made in the sequence. Thus, it is possible that if participants are attending to the visual feedback provided on the display, they may move their eyes in a way that is systematically related to the task.  Since we did record eye movements simultaneously with the MEG recordings (EyeLink 1000 Plus; Fs = 600 Hz), we were able to perform a control analysis to address this question. For each keypress event during trials in which no errors occurred (which is the same time-point that the asterisk position is updated), we extracted three features related to eye movements: 1) the gaze position at the time of asterisk position update (or keyDown event), 2) the gaze position 150ms later, and 3) the peak velocity of the eye movement between the two positions. We then constructed a classifier from these features with the aim of predicting the location of the asterisk (ordinal positions 1-5) on the display. As shown in the confusion matrix below (Author response image 1), the classifier failed to perform above chance levels (Overall cross-validated accuracy = 0.21817):

      Author response image 1.

      Confusion matrix showing that three eye movement features fail to predict asterisk position on the task display above chance levels (Fold 1 test accuracy = 0.21718; Fold 2 test accuracy = 0.22023; Fold 3 test accuracy = 0.21859; Fold 4 test accuracy = 0.22113; Fold 5 test accuracy = 0.21373; Overall cross-validated accuracy = 0.2181). Since the ordinal position of the asterisk on the display is highly correlated with the ordinal position of individual keypresses in the sequence, this analysis provides strong evidence that keypress decoding performance from MEG features is not explained by systematic relationships between finger movement behavior and eye movements (i.e. – behavioral artefacts).

      In fact, inspection of the eye position data revealed that a majority of participants on most trials displayed random walk gaze patterns around a center fixation point, indicating that participants did not attend to the asterisk position on the display. This is consistent with intrinsic generation of the action sequence, and congruent with the fact that the display does not provide explicit feedback related to performance. A similar real-world example would be manually inputting a long password into a secure online application. In this case, one intrinsically generates the sequence from memory and receives similar feedback about the password sequence position (also provided as asterisks), which is typically ignored by the user. The minimal participant engagement with the visual task display observed in this study highlights another important point – that the behavior in explicit sequence learning motor tasks is highly generative in nature rather than reactive to stimulus cues as in the serial reaction time task (SRTT).  This is a crucial difference that must be carefully considered when designing investigations and comparing findings across studies.

      We observed that initial keypress decoding accuracy was predominantly driven by contralateral primary sensorimotor cortex in the initial practice trials before transitioning to bilateral frontoparietal regions by trials 11 or 12 as performance gains plateaued.  The contribution of contralateral primary sensorimotor areas to early skill learning has been extensively reported in humans and non-human animals. 1,3-5  Similarly, the increased involvement of bilateral frontal and parietal regions to decoding during early skill learning in the non-dominant hand is well known.  Enhanced bilateral activation in both frontal and parietal cortex during skill learning has been extensively reported6-11, and appears to be even more prominent during early fine motor skill learning in the non-dominant hand12,13.  The frontal regions identified in these studies are known to play crucial roles in executive control14, motor planning15, and working memory6,8,16-18 processes, while the same parietal regions are known to integrate multimodal sensory feedback and support visuomotor transformations6,8,16-18, in addition to working memory19. Thus, it is not surprising that these regions increasingly contribute to decoding as subjects internalize the sequential task.  We now include a statement reflecting these considerations in the revised Discussion.

      A somewhat related point is this: when combining voxel and parcel space, a concern is whether a degree of circularity may have contributed to the improved accuracy of the combined data, because it seems to use the same MEG signals twice - the voxels most contributing are also those contributing most to a parcel being identified as relevant, as parcels reflect the average of voxels within a boundary. In this context, I struggled to understand the explanation given, ie that the improved accuracy of the hybrid model may be due to "lower spatially resolved whole-brain and higher spatially resolved regional activity patterns".

      We strongly disagree with the Reviewer’s assertion that the construction of the hybrid-space decoder is circular. To clarify, the base feature set for the hybrid-space decoder constructed for all participants includes whole-brain spatial patterns of MEG source activity averaged within parcels. As stated in the manuscript, these 148 inter-parcel features reflect “lower spatially resolved whole-brain activity patterns” or global brain dynamics. We then independently test how well spatial patterns of MEG source activity for all voxels distributed within individual parcels can decode keypress actions. Again, the testing of these intra-parcel spatial patterns, intended to capture “higher spatially resolved regional brain activity patterns”, is completely independent from one another and independent from the weighting of individual inter-parcel features. These intra-parcel features could, for example, provide additional information about muscle activation patterns or the task environment. These approximately 1150 intra-parcel voxels (on average, within the total number varying between subjects) are then combined with the 148 inter-parcel features to construct the final hybrid-space decoder. In fact, this varied spatial filter approach shares some similarities to the construction of convolutional neural networks (CNNs) used to perform object recognition in image classification applications. One could also view this hybrid-space decoding approach as a spatial analogue to common time-frequency based analyses such as theta-gamma phase amplitude coupling (PAC), which combine information from two or more narrow-band spectral features derived from the same time-series data.

      We directly tested this hypothesis – that spatially overlapping intra- and inter-parcel features portray different information – by constructing an alternative hybrid-space decoder (HybridAlt) that excluded average inter-parcel features which spatially overlapped with intra-parcel voxel features, and comparing the performance to the decoder used in the manuscript (HybridOrig). The prediction was that if the overlapping parcel contained similar information to the more spatially resolved voxel patterns, then removing the parcel features (n=8) from the decoding analysis should not impact performance. In fact, despite making up less than 1% of the overall input feature space, removing those parcels resulted in a significant drop in overall performance greater than 2% (78.15% ± SD 7.03% for HybridOrig vs. 75.49% ± SD 7.17% for HybridAlt; Wilcoxon signed rank test, z = 3.7410, p = 1.8326e-04) (Author response image 2).

      Author response image 2.

      Comparison of decoding performances with two different hybrid approaches. HybridAlt: Intra-parcel voxel-space features of top ranked parcels and inter-parcel features of remaining parcels. HybridOrig:  Voxel-space features of top ranked parcels and whole-brain parcel-space features (i.e. – the version used in the manuscript). Dots represent decoding accuracy for individual subjects. Dashed lines indicate the trend in performance change across participants. Note, that HybridOrig (the approach used in our manuscript) significantly outperforms the HybridAlt approach, indicating that the excluded parcel features provide unique information compared to the spatially overlapping intra-parcel voxel patterns.

      Firstly, there will be a relatively high degree of spatial contiguity among voxels because of the nature of the signal measured, i.e. nearby individual voxels are unlikely to be independent. Secondly, the voxel data gives a somewhat misleading sense of precision; the inversion can be set up to give an estimate for each voxel, but there will not just be dependence among adjacent voxels, but also substantial variation in the sensitivity and confidence with which activity can be projected to different parts of the brain. Midline and deeper structures come to mind, where the inversion will be more problematic than for regions along the dorsal convexity of the brain, and a concern is that in those midline structures, the highest decoding accuracy is seen. 

      We definitely agree with the Reviewer that some inter-parcel features representing neighboring (or spatially contiguous) voxels are likely to be correlated. This has been well documented in the MEG literature20,21 and is a particularly important confound to address in functional or effective connectivity analyses (not performed in the present study). In the present analysis, any correlation between adjacent voxels presents a multi-collinearity problem, which effectively reduces the dimensionality of the input feature space. However, as long as there are multiple groups of correlated voxels within each parcel (i.e. - the effective dimensionality is still greater than 1), the intra-parcel spatial patterns could still meaningfully contribute to the decoder performance. Two specific results support this assertion.

      First, we obtained higher decoding accuracy with voxel-space features [74.51% (± SD 7.34%)] compared to parcel space features [68.77% (± SD 7.6%)] (Figure 3B), indicating individual voxels carry more information in decoding the keypresses than the averaged voxel-space features or parcel-space features.  Second, Individual voxels within a parcel showed varying feature importance scores in decoding keypresses (Author response image 3). This finding supports the Reviewer’s assertion that neighboring voxels express similar information, but also shows that the correlated voxels form mini subclusters that are much smaller spatially than the parcel they reside in.

      Author response image 3.

      Feature importance score of individual voxels in decoding keypresses: MRMR was used to rank the individual voxel space features in decoding keypresses and the min-max normalized MRMR score was mapped to a structural brain surface. Note that individual voxels within a parcel showed different contribution to decoding.

       

      Some of these concerns could be addressed by recording head movement (with enough precision) to regress out these contributions. The authors state that head movement was monitored with 3 fiducials, and their time courses ought to provide a way to deal with this issue. The ICA procedure may not have sufficiently dealt with removing movement-related problems, but one could eg relate individual components that were identified to the keypresses as another means for checking. An alternative could be to focus on frequency ranges above the movement frequencies. The accuracy for those still seems impressive and may provide a slightly more biologically plausible assessment. 

      We have already addressed the issue of movement related artefacts in the first response above. With respect to a focus on frequency ranges above movement frequencies, the Reviewer states the “accuracy for those still seems impressive and may provide a slightly more biologically plausible assessment”. First, it is important to note that cortical delta-band oscillations measured with local field potentials (LFPs) in macaques is known to contain important information related to end-effector kinematics22,23 muscle activation patterns24 and temporal sequencing25 during skilled reaching and grasping actions. Thus, there is a substantial body of evidence that low-frequency neural oscillatory activity in this range contains important information about the skill learning behavior investigated in the present study. Second, our own data shows (which the Reviewer also points out) that significant information related to the skill learning behavior is also present in higher frequency bands (see Figure 2A and Figure 3—figure supplement 1). As we pointed out in our earlier response to questions about the hybrid space decoder architecture (see above), it is likely that different, yet complimentary, information is encoded across different temporal frequencies (just as it is encoded across different spatial frequencies). Again, this interpretation is supported by our data as the highest performing classifiers in all cases (when holding all parameters constant) were always constructed from broadband input MEG data (Figure 2A and Figure 3—figure supplement 1).  

      One question concerns the interpretation of the results shown in Figure 4. They imply that during the course of learning, entirely different brain networks underpin the behaviour. Not only that, but they also include regions that would seem rather unexpected to be key nodes for learning and expressing relatively simple finger sequences, such as here. What then is the biological plausibility of these results? The authors seem to circumnavigate this issue by moving into a distance metric that captures the (neural network) changes over the course of learning, but the discussion seems detached from which regions are actually involved; or they offer a rather broad discussion of the anatomical regions identified here, eg in the context of LFOs, where they merely refer to "frontoparietal regions". 

      The Reviewer notes the shift in brain networks driving keypress decoding performance between trials 1, 11 and 36 as shown in Figure 4A. The Reviewer questions whether these substantial shifts in brain network states underpinning the skill are biologically plausible, as well as the likelihood that bilateral superior and middle frontal and parietal cortex are important nodes within these networks.

      First, previous fMRI work in humans performing a similar sequence learning task showed that flexibility in brain network composition (i.e. – changes in brain region members displaying coordinated activity) is up-regulated in novel learning environments and explains differences in learning rates across individuals26.  This work supports our interpretation of the present study data, that brain networks engaged in sequential motor skills rapidly reconfigure during early learning.

      Second, frontoparietal network activity is known to support motor memory encoding during early learning27,28. For example, reactivation events in the posterior parietal29 and medial prefrontal30,31 cortex (MPFC) have been temporally linked to hippocampal replay, and are posited to support memory consolidation across several memory domains32, including motor sequence learning1,33,34.  Further, synchronized interactions between MPFC and hippocampus are more prominent during early learning as opposed to later stages27,35,36, perhaps reflecting “redistribution of hippocampal memories to MPFC” 27.  MPFC contributes to very early memory formation by learning association between contexts, locations, events and adaptive responses during rapid learning37. Consistently, coupling between hippocampus and MPFC has been shown during, and importantly immediately following (rest) initial memory encoding38,39.  Importantly, MPFC activity during initial memory encoding predicts subsequent recall40. Thus, the spatial map required to encode a motor sequence memory may be “built under the supervision of the prefrontal cortex” 28, also engaged in the development of an abstract representation of the sequence41.  In more abstract terms, the prefrontal, premotor and parietal cortices support novice performance “by deploying attentional and control processes” 42-44 required during early learning42-44. The dorsolateral prefrontal cortex DLPFC specifically is thought to engage in goal selection and sequence monitoring during early skill practice45, all consistent with the schema model of declarative memory in which prefrontal cortices play an important role in encoding46,47.  Thus, several prefrontal and frontoparietal regions contributing to long term learning 48 are also engaged in early stages of encoding. Altogether, there is strong biological support for the involvement of bilateral prefrontal and frontoparietal regions to decoding during early skill learning.  We now address this issue in the revised manuscript.

      If I understand correctly, the offline neural representation analysis is in essence the comparison of the last keypress vs the first keypress of the next sequence. In that sense, the activity during offline rest periods is actually not considered. This makes the nomenclature somewhat confusing. While it matches the behavioural analysis, having only key presses one can't do it in any other way, but here the authors actually do have recordings of brain activity during offline rest. So at the very least calling it offline neural representation is misleading to this reviewer because what is compared is activity during the last and during the next keypress, not activity during offline periods. But it also seems a missed opportunity - the authors argue that most of the relevant learning occurs during offline rest periods, yet there is no attempt to actually test whether activity during this period can be useful for the questions at hand here. 

      We agree with the Reviewer that our previous “offline neural representation” nomenclature could be misinterpreted. In the revised manuscript we refer to this difference as the “offline neural representational change”. Please, note that our previous work did link offline neural activity (i.e. – 16-22 Hz beta power and neural replay density during inter-practice rest periods) to observed micro-offline gains49.

      Reviewer #2 (Public review): 

      Summary 

      Dash et al. asked whether and how the neural representation of individual finger movements is "contextualized" within a trained sequence during the very early period of sequential skill learning by using decoding of MEG signal. Specifically, they assessed whether/how the same finger presses (pressing index finger) embedded in the different ordinal positions of a practiced sequence (4-1-3-2-4; here, the numbers 1 through 4 correspond to the little through the index fingers of the non-dominant left hand) change their representation (MEG feature). They did this by computing either the decoding accuracy of the index finger at the ordinal positions 1 vs. 5 (index_OP1 vs index_OP5) or pattern distance between index_OP1 vs. index_OP5 at each training trial and found that both the decoding accuracy and the pattern distance progressively increase over the course of learning trials. More interestingly, they also computed the pattern distance for index_OP5 for the last execution of a practice trial vs. index_OP1 for the first execution in the next practice trial (i.e., across the rest period). This "off-line" distance was significantly larger than the "on-line" distance, which was computed within practice trials and predicted micro-offline skill gain. Based on these results, the authors conclude that the differentiation of representation for the identical movement embedded in different positions of a sequential skill ("contextualization") primarily occurs during early skill learning, especially during rest, consistent with the recent theory of the "micro-offline learning" proposed by the authors' group. I think this is an important and timely topic for the field of motor learning and beyond. <br /> Strengths 

      The specific strengths of the current work are as follows. First, the use of temporally rich neural information (MEG signal) has a large advantage over previous studies testing sequential representations using fMRI. This allowed the authors to examine the earliest period (= the first few minutes of training) of skill learning with finer temporal resolution. Second, through the optimization of MEG feature extraction, the current study achieved extremely high decoding accuracy (approx. 94%) compared to previous works. As claimed by the authors, this is one of the strengths of the paper (but see my comments). Third, although some potential refinement might be needed, comparing "online" and "offline" pattern distance is a neat idea. 

      Weaknesses 

      Along with the strengths I raised above, the paper has some weaknesses. First, the pursuit of high decoding accuracy, especially the choice of time points and window length (i.e., 200 msec window starting from 0 msec from key press onset), casts a shadow on the interpretation of the main result. Currently, it is unclear whether the decoding results simply reflect behavioral change or true underlying neural change. As shown in the behavioral data, the key press speed reached 3~4 presses per second already at around the end of the early learning period (11th trial), which means inter-press intervals become as short as 250-330 msec. Thus, in almost more than 60% of training period data, the time window for MEG feature extraction (200 msec) spans around 60% of the inter-press intervals. Considering that the preparation/cueing of subsequent presses starts ahead of the actual press (e.g., Kornysheva et al., 2019) and/or potential online planning (e.g., Ariani and Diedrichsen, 2019), the decoder likely has captured these future press information as well as the signal related to the current key press, independent of the formation of genuine sequential representation (e.g., "contextualization" of individual press). This may also explain the gradual increase in decoding accuracy or pattern distance between index_OP1 vs. index_OP5 (Figure 4C and 5A), which co-occurred with performance improvement, as shorter inter-press intervals are more favorable for the dissociating the two index finger presses followed by different finger presses. The compromised decoding accuracies for the control sequences can be explained in similar logic. Therefore, more careful consideration and elaborated discussion seem necessary when trying to both achieve high-performance decoding and assess early skill learning, as it can impact all the subsequent analyses.

      The Reviewer raises the possibility that (given the windowing parameters used in the present study) an increase in “contextualization” with learning could simply reflect faster typing speeds as opposed to an actual change in the underlying neural representation. The issue can essentially be framed as a mixing problem. As correct sequences are generated at higher and higher speeds over training, MEG activity patterns related to the planning, execution, evaluation and memory of individual keypresses overlap more in time. Thus, increased overlap between the “4” and “1” keypresses (at the start of the sequence) and “2” and “4” keypresses (at the end of the sequence) could artefactually increase contextualization distances even if the underlying neural representations for the individual keypresses remain unchanged (assuming this mixing of representations is used by the classifier to differentially tag each index finger press). If this were the case, it follows that such mixing effects reflecting the ordinal sequence structure would also be observable in the distribution of decoder misclassifications. For example, “4” keypresses would be more likely to be misclassified as “1” or “2” keypresses (or vice versa) than as “3” keypresses. The confusion matrices presented in Figures 3C and 4B and Figure 3—figure supplement 3A in the previously submitted manuscript do not show this trend in the distribution of misclassifications across the four fingers.

      Moreover, if the representation distance is largely driven by this mixing effect, it’s also possible that the increased overlap between consecutive index finger keypresses during the 4-4 transition marking the end of one sequence and the beginning of the next one could actually mask contextualization-related changes to the underlying neural representations and make them harder to detect. In this case, a decoder tasked with separating individual index finger keypresses into two distinct classes based upon sequence position might show decreased performance with learning as adjacent keypresses overlapped in time with each other to an increasing extent. However, Figure 4C in our previously submitted manuscript does not support this possibility, as the 2-class hybrid classifier displays improved classification performance over early practice trials despite greater temporal overlap.

      We also conducted a new multivariate regression analysis to directly assess whether the neural representation distance score could be predicted by the 4-1, 2-4 and 4-4 keypress transition times observed for each complete correct sequence (both predictor and response variables were z-score normalized within-subject). The results of this analysis affirmed that the possible alternative explanation put forward by the Reviewer is not supported by our data (Adjusted R2 = 0.00431; F = 5.62). We now include this new negative control analysis result in the revised manuscript.

      Overall, we do strongly agree with the Reviewer that the naturalistic, self-paced, generative task employed in the present study results in overlapping brain processes related to planning, execution, evaluation and memory of the action sequence. We also agree that there are several tradeoffs to consider in the construction of the classifiers depending on the study aim. Given our aim of optimizing keypress decoder accuracy in the present study, the set of trade-offs resulted in representations reflecting more the latter three processes, and less so the planning component. Whether separate decoders can be constructed to tease apart the representations or networks supporting these overlapping processes is an important future direction of research in this area. For example, work presently underway in our lab constrains the selection of windowing parameters in a manner that allows individual classifiers to be temporally linked to specific planning, execution, evaluation or memory-related processes to discern which brain networks are involved and how they adaptively reorganize with learning. Results from the present study (Figure 4—figure supplement 2) showing hybrid-space decoder prediction accuracies exceeding 74% for temporal windows spanning as little as 25ms and located up to 100ms prior to the keyDown event strongly support the feasibility of such an approach.

      Related to the above point, testing only one particular sequence (4-1-3-2-4), aside from the control ones, limits the generalizability of the finding. This also may have contributed to the extremely high decoding accuracy reported in the current study. 

      The Reviewer raises a question about the generalizability of the decoder accuracy reported in our study. Fortunately, a comparison between decoder performances on Day 1 and Day 2 datasets does provide some insight into this issue. As the Reviewer points out, the classifiers in this study were trained and tested on keypresses performed while practicing a specific sequence (4-1-3-2-4). The study was designed this way as to avoid the impact of interference effects on learning dynamics. The cross-validated performance of classifiers on MEG data collected within the same session was 90.47% overall accuracy (4-class; Figure 3C). We then tested classifier performance on data collected during a separate MEG session conducted approximately 24 hours later (Day 2; see Figure 3—supplement 3). We observed a reduction in overall accuracy rate to 87.11% when tested on MEG data recorded while participants performed the same learned sequence, and 79.44% when they performed several previously unpracticed sequences. Both changes in accuracy are important with regards to the generalizability of our findings. First, 87.11% performance accuracy for the trained sequence data on Day 2 (a reduction of only 3.36%) indicates that the hybrid-space decoder performance is robust over multiple MEG sessions, and thus, robust to variations in SNR across the MEG sensor array caused by small differences in head position between scans.  This indicates a substantial advantage over sensor-space decoding approaches. Furthermore, when tested on data from unpracticed sequences, overall performance dropped an additional 7.67%. This difference reflects the performance bias of the classifier for the trained sequence, possibly caused by high-order sequence structure being incorporated into the feature weights. In the future, it will be important to understand in more detail how random or repeated keypress sequence training data impacts overall decoder performance and generalization. We strongly agree with the Reviewer that the issue of generalizability is extremely important and have added a new paragraph to the Discussion in the revised manuscript highlighting the strengths and weaknesses of our study with respect to this issue.

      In terms of clinical BCI, one of the potential relevance of the study, as claimed by the authors, it is not clear that the specific time window chosen in the current study (up to 200 msec since key press onset) is really useful. In most cases, clinical BCI would target neural signals with no overt movement execution due to patients' inability to move (e.g., Hochberg et al., 2012). Given the time window, the surprisingly high performance of the current decoder may result from sensory feedback and/or planning of subsequent movement, which may not always be available in the clinical BCI context. Of course, the decoding accuracy is still much higher than chance even when using signal before the key press (as shown in Figure 4 Supplement 2), but it is not immediately clear to me that the authors relate their high decoding accuracy based on post-movement signal to clinical BCI settings.

      The Reviewer questions the relevance of the specific window parameters used in the present study for clinical BCI applications, particularly for paretic patients who are unable to produce finger movements or for whom afferent sensory feedback is no longer intact. We strongly agree with the Reviewer that any intended clinical application must carefully consider these specific input feature constraints dictated by the clinical cohort, and in turn impose appropriate and complimentary constraints on classifier parameters that may differ from the ones used in the present study.  We now highlight this issue in the Discussion of the revised manuscript and relate our present findings to published clinical BCI work within this context.

      One of the important and fascinating claims of the current study is that the "contextualization" of individual finger movements in a trained sequence specifically occurs during short rest periods in very early skill learning, echoing the recent theory of micro-offline learning proposed by the authors' group. Here, I think two points need to be clarified. First, the concept of "contextualization" is kept somewhat blurry throughout the text. It is only at the later part of the Discussion (around line #330 on page 13) that some potential mechanism for the "contextualization" is provided as "what-and-where" binding. Still, it is unclear what "contextualization" actually is in the current data, as the MEG signal analyzed is extracted from 0-200 msec after the keypress. If one thinks something is contextualizing an action, that contextualization should come earlier than the action itself. 

      The Reviewer requests that we: 1) more clearly define our use of the term “contextualization” and 2) provide the rationale for assessing it over a 200ms window aligned to the keyDown event. This choice of window parameters means that the MEG activity used in our analysis was coincident with, rather than preceding, the actual keypresses.  We define contextualization as the differentiation of representation for the identical movement embedded in different positions of a sequential skill. That is, representations of individual action elements progressively incorporate information about their relationship to the overall sequence structure as the skill is learned. We agree with the Reviewer that this can be appropriately interpreted as “what-and-where” binding. We now incorporate this definition in the Introduction of the revised manuscript as requested.

      The window parameters for optimizing accurate decoding individual finger movements were determined using a grid search of the parameter space (a sliding window of variable width between 25-350 ms with 25 ms increments variably aligned from 0 to +100ms with 10ms increments relative to the keyDown event). This approach generated 140 different temporal windows for each keypress for each participant, with the final parameter selection determined through comparison of the resulting performance between each decoder.  Importantly, the decision to optimize for decoding accuracy placed an emphasis on keypress representations characterized by the most consistent and robust features shared across subjects, which in turn maximize statistical power in detecting common learning-related changes. In this case, the optimal window encompassed a 200ms epoch aligned to the keyDown event (t0 = 0 ms).  We then asked if the representations (i.e. – spatial patterns of combined parcel- and voxel-space activity) of the same digit at two different sequence positions changed with practice within this optimal decoding window.  Of course, our findings do not rule out the possibility that contextualization can also be found before or even after this time window, as we did not directly address this issue in the present study.  Ongoing work in our lab, as pointed out above, is investigating contextualization within different time windows tailored specifically for assessing sequence skill action planning, execution, evaluation and memory processes.

      The second point is that the result provided by the authors is not yet convincing enough to support the claim that "contextualization" occurs during rest. In the original analysis, the authors presented the statistical significance regarding the correlation between the "offline" pattern differentiation and micro-offline skill gain (Figure 5. Supplement 1), as well as the larger "offline" distance than "online" distance (Figure 5B). However, this analysis looks like regressing two variables (monotonically) increasing as a function of the trial. Although some information in this analysis, such as what the independent/dependent variables were or how individual subjects were treated, was missing in the Methods, getting a statistically significant slope seems unsurprising in such a situation. Also, curiously, the same quantitative evidence was not provided for its "online" counterpart, and the authors only briefly mentioned in the text that there was no significant correlation between them. It may be true looking at the data in Figure 5A as the online representation distance looks less monotonically changing, but the classification accuracy presented in Figure 4C, which should reflect similar representational distance, shows a more monotonic increase up to the 11th trial. Further, the ways the "online" and "offline" representation distance was estimated seem to make them not directly comparable. While the "online" distance was computed using all the correct press data within each 10 sec of execution, the "offline" distance is basically computed by only two presses (i.e., the last index_OP5 vs. the first index_OP1 separated by 10 sec of rest). Theoretically, the distance between the neural activity patterns for temporally closer events tends to be closer than that between the patterns for temporally far-apart events. It would be fairer to use the distance between the first index_OP1 vs. the last index_OP5 within an execution period for "online" distance, as well. 

      The Reviewer suggests that the current data is not convincing enough to show that contextualization occurs during rest and raises two important concerns: 1) the relationship between online contextualization and micro-online gains is not shown, and 2) the online distance was calculated differently from its offline counterpart (i.e. - instead of calculating the distance between last IndexOP5 and first IndexOP1 from a single trial, the distance was calculated for each sequence within a trial and then averaged).

      We addressed the first concern by performing individual subject correlations between 1) contextualization changes during rest intervals and micro-offline gains; 2) contextualization changes during practice trials and micro-online gains, and 3) contextualization changes during practice trials and micro-offline gains (Author response image 4). We then statistically compared the resulting correlation coefficient distributions and found that within-subject correlations for contextualization changes during rest intervals and micro-offline gains were significantly higher than online contextualization and micro-online gains (t = 3.2827, p = 0.0015) and online contextualization and micro-offline gains (t = 3.7021, p = 5.3013e-04). These results are consistent with our interpretation that micro-offline gains are supported by contextualization changes during the inter-practice rest period.

      Author response image 4.

      Distribution of individual subject correlation coefficients between contextualization changes occurring during practice or rest with  micro-online and micro-offline performance gains. Note that, the correlation distributions were significantly higher for the relationship between contextualization changes during rest and micro-offline gains than for contextualization changes during practice and either micro-online or offline gain.

      With respect to the second concern highlighted above, we agree with the Reviewer that one limitation of the analysis comparing online versus offline changes in contextualization as presented in the reviewed manuscript, is that it does not eliminate the possibility that any differences could simply be explained by the passage of time (which is smaller for the online analysis compared to the offline analysis). The Reviewer suggests an approach that addresses this issue, which we have now carried out.   When quantifying online changes in contextualization from the first IndexOP1 the last IndexOP5 keypress in the same trial we observed no learning-related trend (Author response image 5, right panel). Importantly, offline distances were significantly larger than online distances regardless of the measurement approach and neither predicted online learning (Author response image 6).

      Author response image 5.

      Trial by trial trend of offline (left panel) and online (middle and right panels) changes in contextualization. Offline changes in contextualization were assessed by calculating the distance between neural representations for the last IndexOP5 keypress in the previous trial and the first IndexOP1 keypress in the present trial. Two different approaches were used to characterize online contextualization changes. The analysis included in the reviewed manuscript (middle panel) calculated the distance between IndexOP1 and IndexOP5 for each correct sequence, which was then averaged across the trial. This approach is limited by the lack of control for the passage of time when making online versus offline comparisons. Thus, the second approach controlled for the passage of time by calculating distance between the representations associated with the first IndexOP1 keypress and the last IndexOP5 keypress within the same trial. Note that while the first approach showed an increase online contextualization trend with practice, the second approach did not.

      Author response image 6.

      Relationship between online contextualization and online learning is shown for both within-sequence (left; note that this is the online contextualization measure used in the reviewd manuscript) and across-sequence (right) distance calculation. There was no significant relationship between online learning and online contextualization regardless of the measurement approach.

      A related concern regarding the control analysis, where individual values for max speed and the degree of online contextualization were compared (Figure 5 Supplement 3), is whether the individual difference is meaningful. If I understood correctly, the optimization of the decoding process (temporal window, feature inclusion/reduction, decoder, etc.) was performed for individual participants, and the same feature extraction was also employed for the analysis of representation distance (i.e., contextualization). If this is the case, the distances are individually differently calculated and they may need to be normalized relative to some stable reference (e.g., 1 vs. 4 or average distance within the control sequence presses) before comparison across the individuals. 

      The Reviewer makes a good point here. We have now implemented the suggested normalization procedure in the analysis provided in the revised manuscript.

      Reviewer #3 (Public review): 

      Summary: 

      One goal of this paper is to introduce a new approach for highly accurate decoding of finger movements from human magnetoencephalography data via dimension reduction of a "multi-scale, hybrid" feature space. Following this decoding approach, the authors aim to show that early skill learning involves "contextualization" of the neural coding of individual movements, relative to their position in a sequence of consecutive movements. Furthermore, they aim to show that this "contextualization" develops primarily during short rest periods interspersed with skill training and correlates with a performance metric which the authors interpret as an indicator of offline learning. <br /> Strengths: 

      A clear strength of the paper is the innovative decoding approach, which achieves impressive decoding accuracies via dimension reduction of a "multi-scale, hybrid space". This hybrid-space approach follows the neurobiologically plausible idea of the concurrent distribution of neural coding across local circuits as well as large-scale networks. A further strength of the study is the large number of tested dimension reduction techniques and classifiers (though the manuscript reveals little about the comparison of the latter). 

      We appreciate the Reviewer’s comments regarding the paper’s strengths.

      A simple control analysis based on shuffled class labels could lend further support to this complex decoding approach. As a control analysis that completely rules out any source of overfitting, the authors could test the decoder after shuffling class labels. Following such shuffling, decoding accuracies should drop to chance level for all decoding approaches, including the optimized decoder. This would also provide an estimate of actual chance-level performance (which is informative over and beyond the theoretical chance level). Furthermore, currently, the manuscript does not explain the huge drop in decoding accuracies for the voxel-space decoding (Figure 3B). Finally, the authors' approach to cortical parcellation raises questions regarding the information carried by varying dipole orientations within a parcel (which currently seems to be ignored?) and the implementation of the mean-flipping method (given that there are two dimensions - space and time - what do the authors refer to when they talk about the sign of the "average source", line 477?). 

      The Reviewer recommends that we: 1) conduct an additional control analysis on classifier performance using shuffled class labels, 2) provide a more detailed explanation regarding the drop in decoding accuracies for the voxel-space decoding following LDA dimensionality reduction (see Fig 3B), and 3) provide additional details on how problems related to dipole solution orientations were addressed in the present study.  

      In relation to the first point, we have now implemented a random shuffling approach as a control for the classification analyses. The results of this analysis indicated that the chance level accuracy was 22.12% (± SD 9.1%) for individual keypress decoding (4-class classification), and 18.41% (± SD 7.4%) for individual sequence item decoding (5-class classification), irrespective of the input feature set or the type of decoder used. Thus, the decoding accuracy observed with the final model was substantially higher than these chance levels.  

      Second, please note that the dimensionality of the voxel-space feature set is very high (i.e. – 15684). LDA attempts to map the input features onto a much smaller dimensional space (number of classes-1; e.g. –  3 dimensions, for 4-class keypress decoding). Given the very high dimension of the voxel-space input features in this case, the resulting mapping exhibits reduced accuracy. Despite this general consideration, please refer to Figure 3—figure supplement 3, where we observe improvement in voxel-space decoder performance when utilizing alternative dimensionality reduction techniques.

      The decoders constructed in the present study assess the average spatial patterns across time (as defined by the windowing procedure) in the input feature space.  We now provide additional details in the Methods of the revised manuscript pertaining to the parcellation procedure and how the sign ambiguity problem was addressed in our analysis.

      Weaknesses: 

      A clear weakness of the paper lies in the authors' conclusions regarding "contextualization". Several potential confounds, described below, question the neurobiological implications proposed by the authors and provide a simpler explanation of the results. Furthermore, the paper follows the assumption that short breaks result in offline skill learning, while recent evidence, described below, casts doubt on this assumption. 

      We thank the Reviewer for giving us the opportunity to address these issues in detail (see below).

      The authors interpret the ordinal position information captured by their decoding approach as a reflection of neural coding dedicated to the local context of a movement (Figure 4). One way to dissociate ordinal position information from information about the moving effectors is to train a classifier on one sequence and test the classifier on other sequences that require the same movements, but in different positions50. In the present study, however, participants trained to repeat a single sequence (4-1-3-2-4). As a result, ordinal position information is potentially confounded by the fixed finger transitions around each of the two critical positions (first and fifth press). Across consecutive correct sequences, the first keypress in a given sequence was always preceded by a movement of the index finger (=last movement of the preceding sequence), and followed by a little finger movement. The last keypress, on the other hand, was always preceded by a ring finger movement, and followed by an index finger movement (=first movement of the next sequence). Figure 4 - Supplement 2 shows that finger identity can be decoded with high accuracy (>70%) across a large time window around the time of the key press, up to at least +/-100 ms (and likely beyond, given that decoding accuracy is still high at the boundaries of the window depicted in that figure). This time window approaches the keypress transition times in this study. Given that distinct finger transitions characterized the first and fifth keypress, the classifier could thus rely on persistent (or "lingering") information from the preceding finger movement, and/or "preparatory" information about the subsequent finger movement, in order to dissociate the first and fifth keypress. Currently, the manuscript provides no evidence that the context information captured by the decoding approach is more than a by-product of temporally extended, and therefore overlapping, but independent neural representations of consecutive keypresses that are executed in close temporal proximity - rather than a neural representation dedicated to context. 

      Such temporal overlap of consecutive, independent finger representations may also account for the dynamics of "ordinal coding"/"contextualization", i.e., the increase in 2-class decoding accuracy, across Day 1 (Figure 4C). As learning progresses, both tapping speed and the consistency of keypress transition times increase (Figure 1), i.e., consecutive keypresses are closer in time, and more consistently so. As a result, information related to a given keypress is increasingly overlapping in time with information related to the preceding and subsequent keypresses. The authors seem to argue that their regression analysis in Figure 5 - Figure Supplement 3 speaks against any influence of tapping speed on "ordinal coding" (even though that argument is not made explicitly in the manuscript). However, Figure 5 - Figure Supplement 3 shows inter-individual differences in a between-subject analysis (across trials, as in panel A, or separately for each trial, as in panel B), and, therefore, says little about the within-subject dynamics of "ordinal coding" across the experiment. A regression of trial-by-trial "ordinal coding" on trial-by-trial tapping speed (either within-subject or at a group-level, after averaging across subjects) could address this issue. Given the highly similar dynamics of "ordinal coding" on the one hand (Figure 4C), and tapping speed on the other hand (Figure 1B), I would expect a strong relationship between the two in the suggested within-subject (or group-level) regression. Furthermore, learning should increase the number of (consecutively) correct sequences, and, thus, the consistency of finger transitions. Therefore, the increase in 2-class decoding accuracy may simply reflect an increasing overlap in time of increasingly consistent information from consecutive keypresses, which allows the classifier to dissociate the first and fifth keypress more reliably as learning progresses, simply based on the characteristic finger transitions associated with each. In other words, given that the physical context of a given keypress changes as learning progresses - keypresses move closer together in time and are more consistently correct - it seems problematic to conclude that the mental representation of that context changes. To draw that conclusion, the physical context should remain stable (or any changes to the physical context should be controlled for). 

      The issues raised by Reviewer #3 here are similar to two issues raised by Reviewer #2 above and agree they must both be carefully considered in any evaluation of our findings.

      As both Reviewers pointed out, the classifiers in this study were trained and tested on keypresses performed while practicing a specific sequence (4-1-3-2-4). The study was designed this way as to avoid the impact of interference effects on learning dynamics. The cross-validated performance of classifiers on MEG data collected within the same session was 90.47% overall accuracy (4-class; Figure 3C). We then tested classifier performance on data collected during a separate MEG session conducted approximately 24 hours later (Day 2; see Figure 3—supplement 3). We observed a reduction in overall accuracy rate to 87.11% when tested on MEG data recorded while participants performed the same learned sequence, and 79.44% when they performed several previously unpracticed sequences. This classification performance difference of 7.67% when tested on the Day 2 data could reflect the performance bias of the classifier for the trained sequence, possibly caused by mixed information from temporally close keypresses being incorporated into the feature weights.

      Along these same lines, both Reviewers also raise the possibility that an increase in “ordinal coding/contextualization” with learning could simply reflect an increase in this mixing effect caused by faster typing speeds as opposed to an actual change in the underlying neural representation. The basic idea is that as correct sequences are generated at higher and higher speeds over training, MEG activity patterns related to the planning, execution, evaluation and memory of individual keypresses overlap more in time. Thus, increased overlap between the “4” and “1” keypresses (at the start of the sequence) and “2” and “4” keypresses (at the end of the sequence) could artefactually increase contextualization distances even if the underlying neural representations for the individual keypresses remain unchanged (assuming this mixing of representations is used by the classifier to differentially tag each index finger press). If this were the case, it follows that such mixing effects reflecting the ordinal sequence structure would also be observable in the distribution of decoder misclassifications. For example, “4” keypresses would be more likely to be misclassified as “1” or “2” keypresses (or vice versa) than as “3” keypresses. The confusion matrices presented in Figures 3C and 4B and Figure 3—figure supplement 3A in the previously submitted manuscript do not show this trend in the distribution of misclassifications across the four fingers.

      Following this logic, it’s also possible that if the ordinal coding is largely driven by this mixing effect, the increased overlap between consecutive index finger keypresses during the 4-4 transition marking the end of one sequence and the beginning of the next one could actually mask contextualization-related changes to the underlying neural representations and make them harder to detect. In this case, a decoder tasked with separating individual index finger keypresses into two distinct classes based upon sequence position might show decreased performance with learning as adjacent keypresses overlapped in time with each other to an increasing extent. However, Figure 4C in our previously submitted manuscript does not support this possibility, as the 2-class hybrid classifier displays improved classification performance over early practice trials despite greater temporal overlap.

      As noted in the above replay to Reviewer #2, we also conducted a new multivariate regression analysis to directly assess whether the neural representation distance score could be predicted by the 4-1, 2-4 and 4-4 keypress transition times observed for each complete correct sequence (both predictor and response variables were z-score normalized within-subject). The results of this analysis affirmed that the possible alternative explanation put forward by the Reviewer is not supported by our data (Adjusted R2 = 0.00431; F = 5.62). We now include this new negative control analysis result in the revised manuscript.

      Finally, the Reviewer hints that one way to address this issue would be to compare MEG responses before and after learning for sequences typed at a fixed speed. However, given that the speed-accuracy trade-off should improve with learning, a comparison between unlearned and learned skill states would dictate that the skill be evaluated at a very low fixed speed. Essentially, such a design presents the problem that the post-training test is evaluating the representation in the unlearned behavioral state that is not representative of the acquired skill. Thus, this approach would not address our experimental question: “do neural representations of the same action performed at different locations within a skill sequence contextually differentiate or remain stable as learning evolves”.

      A similar difference in physical context may explain why neural representation distances ("differentiation") differ between rest and practice (Figure 5). The authors define "offline differentiation" by comparing the hybrid space features of the last index finger movement of a trial (ordinal position 5) and the first index finger movement of the next trial (ordinal position 1). However, the latter is not only the first movement in the sequence but also the very first movement in that trial (at least in trials that started with a correct sequence), i.e., not preceded by any recent movement. In contrast, the last index finger of the last correct sequence in the preceding trial includes the characteristic finger transition from the fourth to the fifth movement. Thus, there is more overlapping information arising from the consistent, neighbouring keypresses for the last index finger movement, compared to the first index finger movement of the next trial. A strong difference (larger neural representation distance) between these two movements is, therefore, not surprising, given the task design, and this difference is also expected to increase with learning, given the increase in tapping speed, and the consequent stronger overlap in representations for consecutive keypresses. Furthermore, initiating a new sequence involves pre-planning, while ongoing practice relies on online planning (Ariani et al., eNeuro 2021), i.e., two mental operations that are dissociable at the level of neural representation (Ariani et al., bioRxiv 2023). 

      The Reviewer argues that the comparison of last finger movement of a trial and the first in the next trial are performed in different circumstances and contexts. This is an important point and one we tend to agree with. For this task, the first sequence in a practice trial (which is pre-planned offline) is performed in a somewhat different context from the sequence iterations that follow, which involve temporally overlapping planning, execution and evaluation processes.  The Reviewer is particularly concerned about a difference in the temporal mixing effect issue raised above between the first and last keypresses performed in a trial. However, in contrast to the Reviewers stated argument above, findings from Korneysheva et. al (2019) showed that neural representations of individual actions are competitively queued during the pre-planning period in a manner that reflects the ordinal structure of the learned sequence.  Thus, mixing effects are likely still present for the first keypress in a trial. Also note that we now present new control analyses in multiple responses above confirming that hypothetical mixing effects between adjacent keypresses do not explain our reported contextualization finding. A statement addressing these possibilities raised by the Reviewer has been added to the Discussion in the revised manuscript.

      In relation to pre-planning, ongoing MEG work in our lab is investigating contextualization within different time windows tailored specifically for assessing how sequence skill action planning evolves with learning.

      Given these differences in the physical context and associated mental processes, it is not surprising that "offline differentiation", as defined here, is more pronounced than "online differentiation". For the latter, the authors compared movements that were better matched regarding the presence of consistent preceding and subsequent keypresses (online differentiation was defined as the mean difference between all first vs. last index finger movements during practice).  It is unclear why the authors did not follow a similar definition for "online differentiation" as for "micro-online gains" (and, indeed, a definition that is more consistent with their definition of "offline differentiation"), i.e., the difference between the first index finger movement of the first correct sequence during practice, and the last index finger of the last correct sequence. While these two movements are, again, not matched for the presence of neighbouring keypresses (see the argument above), this mismatch would at least be the same across "offline differentiation" and "online differentiation", so they would be more comparable. 

      This is the same point made earlier by Reviewer #2, and we agree with this assessment. As stated in the response to Reviewer #2 above, we have now carried out quantification of online contextualization using this approach and included it in the revised manuscript. We thank the Reviewer for this suggestion.

      A further complication in interpreting the results regarding "contextualization" stems from the visual feedback that participants received during the task. Each keypress generated an asterisk shown above the string on the screen, irrespective of whether the keypress was correct or incorrect. As a result, incorrect (e.g., additional, or missing) keypresses could shift the phase of the visual feedback string (of asterisks) relative to the ordinal position of the current movement in the sequence (e.g., the fifth movement in the sequence could coincide with the presentation of any asterisk in the string, from the first to the fifth). Given that more incorrect keypresses are expected at the start of the experiment, compared to later stages, the consistency in visual feedback position, relative to the ordinal position of the movement in the sequence, increased across the experiment. A better differentiation between the first and the fifth movement with learning could, therefore, simply reflect better decoding of the more consistent visual feedback, based either on the feedback-induced brain response, or feedback-induced eye movements (the study did not include eye tracking). It is not clear why the authors introduced this complicated visual feedback in their task, besides consistency with their previous studies.

      We strongly agree with the Reviewer that eye movements related to task engagement are important to rule out as a potential driver of the decoding accuracy or contextualization effect. We address this issue above in response to a question raised by Reviewer #1 about the impact of movement related artefacts in general on our findings.

      First, the assumption the Reviewer makes here about the distribution of errors in this task is incorrect. On average across subjects, 2.32% ± 1.48% (mean ± SD) of all keypresses performed were errors, which were evenly distributed across the four possible keypress responses. While errors increased progressively over practice trials, they did so in proportion to the increase in correct keypresses, so that the overall ratio of correct-to-incorrect keypresses remained stable over the training session. Thus, the Reviewer’s assumptions that there is a higher relative frequency of errors in early trials, and a resulting systematic trend phase shift differences between the visual display updates (i.e. – a change in asterisk position above the displayed sequence) and the keypress performed is not substantiated by the data. To the contrary, the asterisk position on the display and the keypress being executed remained highly correlated over the entire training session. We now include a statement about the frequency and distribution of errors in the revised manuscript.

      Given this high correlation, we firmly agree with the Reviewer that the issue of eye movement-related artefacts is still an important one to address. Fortunately, we did collect eye movement data during the MEG recordings so were able to investigate this. As detailed in the response to Reviewer #1 above, we found that gaze positions and eye-movement velocity time-locked to visual display updates (i.e. – a change in asterisk position above the displayed sequence) did not reflect the asterisk location above chance levels (Overall cross-validated accuracy = 0.21817; see Author response image 1). Furthermore, an inspection of the eye position data revealed that a majority of participants on most trials displayed random walk gaze patterns around a center fixation point, indicating that participants did not attend to the asterisk position on the display. This is consistent with intrinsic generation of the action sequence, and congruent with the fact that the display does not provide explicit feedback related to performance. As pointed out above, a similar real-world example would be manually inputting a long password into a secure online application. In this case, one intrinsically generates the sequence from memory and receives similar feedback about the password sequence position (also provided as asterisks), which is typically ignored by the user. Notably, the minimal participant engagement with the visual task display observed in this study highlights an important difference between behavior observed during explicit sequence learning motor tasks (which is highly generative in nature) with reactive responses to stimulus cues in a serial reaction time task (SRTT).  This is a crucial difference that must be carefully considered when comparing findings across studies. All elements pertaining to this new control analysis are now included in the revised manuscript.

      The authors report a significant correlation between "offline differentiation" and cumulative micro-offline gains. However, it would be more informative to correlate trial-by-trial changes in each of the two variables. This would address the question of whether there is a trial-by-trial relation between the degree of "contextualization" and the amount of micro-offline gains - are performance changes (micro-offline gains) less pronounced across rest periods for which the change in "contextualization" is relatively low? Furthermore, is the relationship between micro-offline gains and "offline differentiation" significantly stronger than the relationship between micro-offline gains and "online differentiation"? 

      In response to a similar issue raised above by Reviewer #2, we now include new analyses comparing correlation magnitudes between (1) “online differention” vs micro-online gains, (2) “online differention” vs micro-offline gains and (3) “offline differentiation” and micro-offline gains (see Author response images 4, 5 and 6 above). These new analyses and results have been added to the revised manuscript. Once again, we thank both Reviewers for this suggestion.

      The authors follow the assumption that micro-offline gains reflect offline learning.

      This statement is incorrect. The original Bonstrup et al (2019) 49 paper clearly states that micro-offline gains must be carefully interpreted based upon the behavioral context within which they are observed, and lays out the conditions under which one can have confidence that micro-offline gains reflect offline learning.  In fact, the excellent meta-analysis of Pan & Rickard (2015) 51, which re-interprets the benefits of sleep in overnight skill consolidation from a “reactive inhibition” perspective, was a crucial resource in the experimental design of our initial study49, as well as in all our subsequent work. Pan & Rickard stated:

      “Empirically, reactive inhibition refers to performance worsening that can accumulate during a period of continuous training (Hull, 1943). It tends to dissipate, at least in part, when brief breaks are inserted between blocks of training. If there are multiple performance-break cycles over a training session, as in the motor sequence literature, performance can exhibit a scalloped effect, worsening during each uninterrupted performance block but improving across blocks52,53. Rickard, Cai, Rieth, Jones, and Ard (2008) and Brawn, Fenn, Nusbaum, and Margoliash (2010) 52,53 demonstrated highly robust scalloped reactive inhibition effects using the commonly employed 30 s–30 s performance break cycle, as shown for Rickard et al.’s (2008) massed practice sleep group in Figure 2. The scalloped effect is evident for that group after the first few 30 s blocks of each session. The absence of the scalloped effect during the first few blocks of training in the massed group suggests that rapid learning during that period masks any reactive inhibition effect.”

      Crucially, Pan & Rickard51 made several concrete recommendations for reducing the impact of the reactive inhibition confound on offline learning studies. One of these recommendations was to reduce practice times to 10s (most prior sequence learning studies up until that point had employed 30s long practice trials). They stated:

      “The traditional design involving 30 s-30 s performance break cycles should be abandoned given the evidence that it results in a reactive inhibition confound, and alternative designs with reduced performance duration per block used instead 51. One promising possibility is to switch to 10 s performance durations for each performance-break cycle Instead 51. That design appears sufficient to eliminate at least the majority of the reactive inhibition effect 52,53.”

      We mindfully incorporated recommendations from Pan and Rickard51  into our own study designs including 1) utilizing 10s practice trials and 2) constraining our analysis of micro-offline gains to early learning trials (where performance monotonically increases and 95% of overall performance gains occur), which are prior to the emergence of the “scalloped” performance dynamics that are strongly linked to reactive inhibition effects. 

      However, there is no direct evidence in the literature that micro-offline gains really result from offline learning, i.e., an improvement in skill level.

      We strongly disagree with the Reviewer’s assertion that “there is no direct evidence in the literature that micro-offline gains really result from offline learning, i.e., an improvement in skill level.”  The initial Bönstrup et al. (2019) 49 report was followed up by a large online crowd-sourcing study (Bönstrup et al., 2020) 54. This second (and much larger) study provided several additional important findings supporting our interpretation of micro-offline gains in cases where the important behavioral conditions clarified above were met (see Author response image 7 below for further details on these conditions).

      Author response image 7.

      Micro-offline gains observed in learning and non-learning contexts are attributed to different underlying causes. (A) Micro-offline and online changes relative to overall trial-by-trial learning. This figure is based on data from Bönstrup et al. (2019) 49. During early learning, micro-offline gains (red bars) closely track trial-by-trial performance gains (green line with open circle markers), with minimal contribution from micro-online gains (blue bars). The stated conclusion in Bönstrup et al. (2019) is that micro-offline gains only during this Early Learning stage reflect rapid memory consolidation (see also 54). After early learning, about practice trial 11, skill plateaus. This plateau skill period is characterized by a striking emergence of coupled (and relatively stable) micro-online drops and micro-offline increases. Bönstrup et al. (2019) as well as others in the literature 55-57, argue that micro-offline gains during the plateau period likely reflect recovery from inhibitory performance factors such as reactive inhibition or fatigue, and thus must be excluded from analyses relating micro-offline gains to skill learning.  The Non-repeating groups in Experiments 3 and 4 from Das et al. (2024) suffer from a lack of consideration of these known confounds.

      Evidence documented in that paper54 showed that micro-offline gains during early skill learning were: 1) replicable and generalized to subjects learning the task in their daily living environment (n=389); 2) equivalent when significantly shortening practice period duration, thus confirming that they are not a result of recovery from performance fatigue (n=118);  3) reduced (along with learning rates) by retroactive interference applied immediately after each practice period relative to interference applied after passage of time (n=373), indicating stabilization of the motor memory at a microscale of several seconds consistent with rapid consolidation; and 4) not modified by random termination of the practice periods, ruling out a contribution of predictive motor slowing (N = 71) 54.  Altogether, our findings were strongly consistent with the interpretation that micro-offline gains reflect memory consolidation supporting early skill learning. This is precisely the portion of the learning curve Pan and Rickard51 refer to when they state “…rapid learning during that period masks any reactive inhibition effect”.

      This interpretation is further supported by brain imaging evidence linking known memory-related networks and consolidation mechanisms to micro-offline gains. First, we reported that the density of fast hippocampo-neocortical skill memory replay events increases approximately three-fold during early learning inter-practice rest periods with the density explaining differences in the magnitude of micro-offline gains across subjects1. Second, Jacobacci et al. (2020) independently reproduced our original behavioral findings and reported BOLD fMRI changes in the hippocampus and precuneus (regions also identified in our MEG study1) linked to micro-offline gains during early skill learning. 33 These functional changes were coupled with rapid alterations in brain microstructure in the order of minutes, suggesting that the same network that operates during rest periods of early learning undergoes structural plasticity over several minutes following practice58. Third, even more recently, Chen et al. (2024) provided direct evidence from intracranial EEG in humans linking sharp-wave ripple events (which are known markers for neural replay59) in the hippocampus (80-120 Hz in humans) with micro-offline gains during early skill learning. The authors report that the strong increase in ripple rates tracked learning behavior, both across blocks and across participants. The authors conclude that hippocampal ripples during resting offline periods contribute to motor sequence learning. 2

      Thus, there is actually now substantial evidence in the literature directly supporting the assertion “that micro-offline gains really result from offline learning”.  On the contrary, according to Gupta & Rickard (2024) “…the mechanism underlying RI [reactive inhibition] is not well established” after over 80 years of investigation60, possibly due to the fact that “reactive inhibition” is a categorical description of behavioral effects that likely result from several heterogenous processes with very different underlying mechanisms.

      On the contrary, recent evidence questions this interpretation (Gupta & Rickard, npj Sci Learn 2022; Gupta & Rickard, Sci Rep 2024; Das et al., bioRxiv 2024). Instead, there is evidence that micro-offline gains are transient performance benefits that emerge when participants train with breaks, compared to participants who train without breaks, however, these benefits vanish within seconds after training if both groups of participants perform under comparable conditions (Das et al., bioRxiv 2024). 

      It is important to point out that the recent work of Gupta & Rickard (2022,2024) 55 does not present any data that directly opposes our finding that early skill learning49 is expressed as micro-offline gains during rest breaks. These studies are essentially an extension of the Rickard et al (2008) paper that employed a massed (30s practice followed by 30s breaks) vs spaced (10s practice followed by 10s breaks) to assess if recovery from reactive inhibition effects could account for performance gains measured after several minutes or hours. Gupta & Rickard (2022) added two additional groups (30s practice/10s break and 10s practice/10s break as used in the work from our group). The primary aim of the study was to assess whether it was more likely that changes in performance when retested 5 minutes after skill training (consisting of 12 practice trials for the massed groups and 36 practice trials for the spaced groups) had ended reflected memory consolidation effects or recovery from reactive inhibition effects. The Gupta & Rickard (2024) follow-up paper employed a similar design with the primary difference being that participants performed a fixed number of sequences on each trial as opposed to trials lasting a fixed duration. This was done to facilitate the fitting of a quantitative statistical model to the data.  To reiterate, neither study included any analysis of micro-online or micro-offline gains and did not include any comparison focused on skill gains during early learning. Instead, Gupta & Rickard (2022), reported evidence for reactive inhibition effects for all groups over much longer training periods. Again, we reported the same finding for trials following the early learning period in our original Bönstrup et al. (2019) paper49 (Author response image 7). Also, please note that we reported in this paper that cumulative micro-offline gains over early learning did not correlate with overnight offline consolidation measured 24 hours later49 (see the Results section and further elaboration in the Discussion). Thus, while the composition of our data is supportive of a short-term memory consolidation process operating over several seconds during early learning, it likely differs from those involved over longer training times and offline periods, as assessed by Gupta & Rickard (2022).

      In the recent preprint from Das et al (2024) 61,  the authors make the strong claim that “micro-offline gains during early learning do not reflect offline learning” which is not supported by their own data.   The authors hypothesize that if “micro-offline gains represent offline learning, participants should reach higher skill levels when training with breaks, compared to training without breaks”.  The study utilizes a spaced vs. massed practice group between-subjects design inspired by the reactive inhibition work from Rickard and others to test this hypothesis. Crucially, the design incorporates only a small fraction of the training used in other investigations to evaluate early skill learning1,33,49,54,57,58,62.  A direct comparison between the practice schedule designs for the spaced and massed groups in Das et al., and the training schedule all participants experienced in the original Bönstrup et al. (2019) paper highlights this issue as well as several others (Author response image 8):

      Author response image 8.

      (A) Comparison of Das et al. Spaced & Massed group training session designs, and the training session design from the original Bönstrup et al. (2019) 49 paper. Similar to the approach taken by Das et al., all practice is visualized as 10-second practice trials with a variable number (either 0, 1 or 30) of 10-second-long inter-practice rest intervals to allow for direct comparisons between designs. The two key takeaways from this comparison are that (1) the intervention differences (i.e. – practice schedules) between the Massed and Spaced groups from the Das et al. report are extremely small (less than 12% of the overall session schedule) and (2) the overall amount of practice is much less than compared to the design from the original Bönstrup report 49  (which has been utilized in several subsequent studies). (B) Group-level learning curve data from Bönstrup et al. (2019) 49 is used to estimate the performance range accounted for by the equivalent periods covering Test 1, Training 1 and Test 2 from Das et al (2024). Note that the intervention in the Das et al. study is limited to a period covering less than 50% of the overall learning range.

      First, participants in the original Bönstrup et al. study 49 experienced 157.14% more practice time and 46.97% less inter-practice rest time than the Spaced group in the Das et al. study (Author response image 8).  Thus, the overall amount of practice and rest differ substantially between studies, with much more limited training occurring for participants in Das et al.  

      Second, and perhaps most importantly, the actual intervention (i.e. – the difference in practice schedule between the Spaced and Massed groups) employed by Das et al. covers a very small fraction of the overall training session. Identical practice schedule segments for both the Spaced & Massed groups are indicated by the red shaded area in Author response image 8. Please note that these identical segments cover 94.84% of the Massed group training schedule and 88.01% of the Spaced group training schedule (since it has 60 seconds of additional rest). This means that the actual interventions cover less than 5% (for Massed) and 12% (for Spaced) of the total training session, which minimizes any chance of observing a difference between groups.

      Also note that the very beginning of the practice schedule (during which Figure R9 shows substantial learning is known to occur) is labeled in the Das et al. study as Test 1.  Test 1 encompasses the first 20 seconds of practice (alternatively viewed as the first two 10-second-long practice trials with no inter-practice rest). This is immediately followed by the Training 1 intervention, which is composed of only three 10-second-long practice trials (with 10-second inter-practice rest for the Spaced group and no inter-practice rest for the Massed group). Author response image 8 also shows that since there is no inter-practice rest after the third Training practice trial for the Spaced group, this third trial (for both Training 1 and 2) is actually a part of an identical practice schedule segment shared by both groups (Massed and Spaced), reducing the magnitude of the intervention even further.

      Moreover, we know from the original Bönstrup et al. (2019) paper49 that 46.57% of all overall group-level performance gains occurred between trials 2 and 5 for that study. Thus, Das et al. are limiting their designed intervention to a period covering less than half of the early learning range discussed in the literature, which again, minimizes any chance of observing an effect.

      This issue is amplified even further at Training 2 since skill learning prior to the long 5-minute break is retained, further constraining the performance range over these three trials. A related issue pertains to the trials labeled as Test 1 (trials 1-2) and Test 2 (trials 6-7) by Das et al. Again, we know from the original Bönstrup et al. paper 49 that 18.06% and 14.43% (32.49% total) of all overall group-level performance gains occurred during trials corresponding to Das et al Test 1 and Test 2, respectively. In other words, Das et al averaged skill performance over 20 seconds of practice at two time-points where dramatic skill improvements occur. Pan & Rickard (1995) previously showed that such averaging is known to inject artefacts into analyses of performance gains.

      Furthermore, the structure of the Test in Das et. al study appears to have an interference effect on the Spaced group performance after the training intervention.  This makes sense if you consider that the Spaced group is required to now perform the task in a Massed practice environment (i.e., two 10-second-long practice trials merged into one long trial), further blurring the true intervention effects. This effect is observable in Figure 1C,E of their pre-print. Specifically, while the Massed group continues to show an increase in performance during test relative to the last 10 seconds of practice during training, the Spaced group displays a marked decrease. This decrease is in stark contrast to the monotonic increases observed for both groups at all other time-points.

      Interestingly, when statistical comparisons between the groups are made at the time-points when the intervention is present (as opposed to after it has been removed) then the stated hypothesis, “If micro-offline gains represent offline learning, participants should reach higher skill levels when training with breaks, compared to training without breaks”, is confirmed.

      The data presented by Gupta and Rickard (2022, 2024) and Das et al. (2024) is in many ways more confirmatory of the constraints employed by our group and others with respect to experimental design, analysis and interpretation of study findings, rather than contradictory. Still, it does highlight a limitation of the current micro-online/offline framework, which was originally only intended to be applied to early skill learning over spaced practice schedules when reactive inhibition effects are minimized49. Extrapolation of this current framework to post-plateau performance periods, longer timespans, or non-learning situations (e.g. – the Non-repeating groups from Experiments 3 & 4 in Das et al. (2024)), when reactive inhibition plays a more substantive role, is not warranted. Ultimately, it will be important to develop new paradigms allowing one to independently estimate the different coincident or antagonistic features (e.g. - memory consolidation, planning, working memory and reactive inhibition) contributing to micro-online and micro-offline gains during and after early skill learning within a unifying framework.

      References

      (1) Buch, E. R., Claudino, L., Quentin, R., Bonstrup, M. & Cohen, L. G. Consolidation of human skill linked to waking hippocampo-neocortical replay. Cell Rep 35, 109193 (2021). https://doi.org:10.1016/j.celrep.2021.109193

      (2) Chen, P.-C., Stritzelberger, J., Walther, K., Hamer, H. & Staresina, B. P. Hippocampal ripples during offline periods predict human motor sequence learning. bioRxiv, 2024.2010.2006.614680 (2024). https://doi.org:10.1101/2024.10.06.614680

      (3) Classen, J., Liepert, J., Wise, S. P., Hallett, M. & Cohen, L. G. Rapid plasticity of human cortical movement representation induced by practice. J Neurophysiol 79, 1117-1123 (1998).

      (4) Karni, A. et al. Functional MRI evidence for adult motor cortex plasticity during motor skill learning. Nature 377, 155-158 (1995). https://doi.org:10.1038/377155a0

      (5) Kleim, J. A., Barbay, S. & Nudo, R. J. Functional reorganization of the rat motor cortex following motor skill learning. J Neurophysiol 80, 3321-3325 (1998).

      (6) Shadmehr, R. & Holcomb, H. H. Neural correlates of motor memory consolidation. Science 277, 821-824 (1997).

      (7) Doyon, J. et al. Experience-dependent changes in cerebellar contributions to motor sequence learning. Proc Natl Acad Sci U S A 99, 1017-1022 (2002).

      (8) Toni, I., Ramnani, N., Josephs, O., Ashburner, J. & Passingham, R. E. Learning arbitrary visuomotor associations: temporal dynamic of brain activity. Neuroimage 14, 1048-1057 (2001).

      (9) Grafton, S. T. et al. Functional anatomy of human procedural learning determined with regional cerebral blood flow and PET. J Neurosci 12, 2542-2548 (1992).

      (10) Kennerley, S. W., Sakai, K. & Rushworth, M. F. Organization of action sequences and the role of the pre-SMA. J Neurophysiol 91, 978-993 (2004). https://doi.org:10.1152/jn.00651.2003 00651.2003 [pii]

      (11) Hardwick, R. M., Rottschy, C., Miall, R. C. & Eickhoff, S. B. A quantitative meta-analysis and review of motor learning in the human brain. Neuroimage 67, 283-297 (2013). https://doi.org:10.1016/j.neuroimage.2012.11.020

      (12) Sawamura, D. et al. Acquisition of chopstick-operation skills with the non-dominant hand and concomitant changes in brain activity. Sci Rep 9, 20397 (2019). https://doi.org:10.1038/s41598-019-56956-0

      (13) Lee, S. H., Jin, S. H. & An, J. The difference in cortical activation pattern for complex motor skills: A functional near- infrared spectroscopy study. Sci Rep 9, 14066 (2019). https://doi.org:10.1038/s41598-019-50644-9

      (14) Battaglia-Mayer, A. & Caminiti, R. Corticocortical Systems Underlying High-Order Motor Control. J Neurosci 39, 4404-4421 (2019). https://doi.org:10.1523/JNEUROSCI.2094-18.2019

      (15) Toni, I., Thoenissen, D. & Zilles, K. Movement preparation and motor intention. Neuroimage 14, S110-117 (2001). https://doi.org:10.1006/nimg.2001.0841

      (16) Wolpert, D. M., Goodbody, S. J. & Husain, M. Maintaining internal representations: the role of the human superior parietal lobe. Nat Neurosci 1, 529-533 (1998). https://doi.org:10.1038/2245

      (17) Andersen, R. A. & Buneo, C. A. Intentional maps in posterior parietal cortex. Annu Rev Neurosci 25, 189-220 (2002). https://doi.org:10.1146/annurev.neuro.25.112701.142922 112701.142922 [pii]

      (18) Buneo, C. A. & Andersen, R. A. The posterior parietal cortex: sensorimotor interface for the planning and online control of visually guided movements. Neuropsychologia 44, 2594-2606 (2006). https://doi.org:S0028-3932(05)00333-7 [pii] 10.1016/j.neuropsychologia.2005.10.011

      (19) Grover, S., Wen, W., Viswanathan, V., Gill, C. T. & Reinhart, R. M. G. Long-lasting, dissociable improvements in working memory and long-term memory in older adults with repetitive neuromodulation. Nat Neurosci 25, 1237-1246 (2022). https://doi.org:10.1038/s41593-022-01132-3

      (20) Colclough, G. L. et al. How reliable are MEG resting-state connectivity metrics? Neuroimage 138, 284-293 (2016). https://doi.org:10.1016/j.neuroimage.2016.05.070

      (21) Colclough, G. L., Brookes, M. J., Smith, S. M. & Woolrich, M. W. A symmetric multivariate leakage correction for MEG connectomes. NeuroImage 117, 439-448 (2015). https://doi.org:10.1016/j.neuroimage.2015.03.071

      (22) Mollazadeh, M. et al. Spatiotemporal variation of multiple neurophysiological signals in the primary motor cortex during dexterous reach-to-grasp movements. J Neurosci 31, 15531-15543 (2011). https://doi.org:10.1523/JNEUROSCI.2999-11.2011

      (23) Bansal, A. K., Vargas-Irwin, C. E., Truccolo, W. & Donoghue, J. P. Relationships among low-frequency local field potentials, spiking activity, and three-dimensional reach and grasp kinematics in primary motor and ventral premotor cortices. J Neurophysiol 105, 1603-1619 (2011). https://doi.org:10.1152/jn.00532.2010

      (24) Flint, R. D., Ethier, C., Oby, E. R., Miller, L. E. & Slutzky, M. W. Local field potentials allow accurate decoding of muscle activity. J Neurophysiol 108, 18-24 (2012). https://doi.org:10.1152/jn.00832.2011

      (25) Churchland, M. M. et al. Neural population dynamics during reaching. Nature 487, 51-56 (2012). https://doi.org:10.1038/nature11129

      (26) Bassett, D. S. et al. Dynamic reconfiguration of human brain networks during learning. Proc Natl Acad Sci U S A 108, 7641-7646 (2011). https://doi.org:10.1073/pnas.1018985108

      (27) Albouy, G., King, B. R., Maquet, P. & Doyon, J. Hippocampus and striatum: dynamics and interaction during acquisition and sleep-related motor sequence memory consolidation. Hippocampus 23, 985-1004 (2013). https://doi.org:10.1002/hipo.22183

      (28) Albouy, G. et al. Neural correlates of performance variability during motor sequence acquisition. Neuroimage 60, 324-331 (2012). https://doi.org:10.1016/j.neuroimage.2011.12.049

      (29) Qin, Y. L., McNaughton, B. L., Skaggs, W. E. & Barnes, C. A. Memory reprocessing in corticocortical and hippocampocortical neuronal ensembles. Philos Trans R Soc Lond B Biol Sci 352, 1525-1533 (1997). https://doi.org:10.1098/rstb.1997.0139

      (30) Euston, D. R., Tatsuno, M. & McNaughton, B. L. Fast-forward playback of recent memory sequences in prefrontal cortex during sleep. Science 318, 1147-1150 (2007). https://doi.org:10.1126/science.1148979

      (31) Molle, M. & Born, J. Hippocampus whispering in deep sleep to prefrontal cortex--for good memories? Neuron 61, 496-498 (2009). https://doi.org:S0896-6273(09)00122-6 [pii] 10.1016/j.neuron.2009.02.002

      (32) Frankland, P. W. & Bontempi, B. The organization of recent and remote memories. Nat Rev Neurosci 6, 119-130 (2005). https://doi.org:10.1038/nrn1607

      (33) Jacobacci, F. et al. Rapid hippocampal plasticity supports motor sequence learning. Proc Natl Acad Sci U S A 117, 23898-23903 (2020). https://doi.org:10.1073/pnas.2009576117

      (34) Albouy, G. et al. Maintaining vs. enhancing motor sequence memories: respective roles of striatal and hippocampal systems. Neuroimage 108, 423-434 (2015). https://doi.org:10.1016/j.neuroimage.2014.12.049

      (35) Gais, S. et al. Sleep transforms the cerebral trace of declarative memories. Proc Natl Acad Sci U S A 104, 18778-18783 (2007). https://doi.org:0705454104 [pii] 10.1073/pnas.0705454104

      (36) Sterpenich, V. et al. Sleep promotes the neural reorganization of remote emotional memory. J Neurosci 29, 5143-5152 (2009). https://doi.org:10.1523/JNEUROSCI.0561-09.2009

      (37) Euston, D. R., Gruber, A. J. & McNaughton, B. L. The role of medial prefrontal cortex in memory and decision making. Neuron 76, 1057-1070 (2012). https://doi.org:10.1016/j.neuron.2012.12.002

      (38) van Kesteren, M. T., Fernandez, G., Norris, D. G. & Hermans, E. J. Persistent schema-dependent hippocampal-neocortical connectivity during memory encoding and postencoding rest in humans. Proc Natl Acad Sci U S A 107, 7550-7555 (2010). https://doi.org:10.1073/pnas.0914892107

      (39) van Kesteren, M. T., Ruiter, D. J., Fernandez, G. & Henson, R. N. How schema and novelty augment memory formation. Trends Neurosci 35, 211-219 (2012). https://doi.org:10.1016/j.tins.2012.02.001

      (40) Wagner, A. D. et al. Building memories: remembering and forgetting of verbal experiences as predicted by brain activity. Science (New York, N.Y.) 281, 1188-1191 (1998).

      (41) Ashe, J., Lungu, O. V., Basford, A. T. & Lu, X. Cortical control of motor sequences. Curr Opin Neurobiol 16, 213-221 (2006).

      (42) Hikosaka, O., Nakamura, K., Sakai, K. & Nakahara, H. Central mechanisms of motor skill learning. Curr Opin Neurobiol 12, 217-222 (2002).

      (43) Penhune, V. B. & Steele, C. J. Parallel contributions of cerebellar, striatal and M1 mechanisms to motor sequence learning. Behav. Brain Res. 226, 579-591 (2012). https://doi.org:10.1016/j.bbr.2011.09.044

      (44) Doyon, J. et al. Contributions of the basal ganglia and functionally related brain structures to motor learning. Behavioural brain research 199, 61-75 (2009). https://doi.org:10.1016/j.bbr.2008.11.012

      (45) Schendan, H. E., Searl, M. M., Melrose, R. J. & Stern, C. E. An FMRI study of the role of the medial temporal lobe in implicit and explicit sequence learning. Neuron 37, 1013-1025 (2003). https://doi.org:10.1016/s0896-6273(03)00123-5

      (46) Morris, R. G. M. Elements of a neurobiological theory of hippocampal function: the role of synaptic plasticity, synaptic tagging and schemas. The European journal of neuroscience 23, 2829-2846 (2006). https://doi.org:10.1111/j.1460-9568.2006.04888.x

      (47) Tse, D. et al. Schemas and memory consolidation. Science 316, 76-82 (2007). https://doi.org:10.1126/science.1135935

      (48) Berlot, E., Popp, N. J. & Diedrichsen, J. A critical re-evaluation of fMRI signatures of motor sequence learning. Elife 9 (2020). https://doi.org:10.7554/eLife.55241

      (49) Bonstrup, M. et al. A Rapid Form of Offline Consolidation in Skill Learning. Curr Biol 29, 1346-1351 e1344 (2019). https://doi.org:10.1016/j.cub.2019.02.049

      (50) Kornysheva, K. et al. Neural Competitive Queuing of Ordinal Structure Underlies Skilled Sequential Action. Neuron 101, 1166-1180 e1163 (2019). https://doi.org:10.1016/j.neuron.2019.01.018

      (51) Pan, S. C. & Rickard, T. C. Sleep and motor learning: Is there room for consolidation? Psychol Bull 141, 812-834 (2015). https://doi.org:10.1037/bul0000009

      (52) Rickard, T. C., Cai, D. J., Rieth, C. A., Jones, J. & Ard, M. C. Sleep does not enhance motor sequence learning. J Exp Psychol Learn Mem Cogn 34, 834-842 (2008). https://doi.org:10.1037/0278-7393.34.4.834

      53) Brawn, T. P., Fenn, K. M., Nusbaum, H. C. & Margoliash, D. Consolidating the effects of waking and sleep on motor-sequence learning. J Neurosci 30, 13977-13982 (2010). https://doi.org:10.1523/JNEUROSCI.3295-10.2010

      (54) Bonstrup, M., Iturrate, I., Hebart, M. N., Censor, N. & Cohen, L. G. Mechanisms of offline motor learning at a microscale of seconds in large-scale crowdsourced data. NPJ Sci Learn 5, 7 (2020). https://doi.org:10.1038/s41539-020-0066-9

      (55) Gupta, M. W. & Rickard, T. C. Dissipation of reactive inhibition is sufficient to explain post-rest improvements in motor sequence learning. NPJ Sci Learn 7, 25 (2022). https://doi.org:10.1038/s41539-022-00140-z

      (56) Jacobacci, F. et al. Rapid hippocampal plasticity supports motor sequence learning. Proceedings of the National Academy of Sciences 117, 23898-23903 (2020).

      (57) Brooks, E., Wallis, S., Hendrikse, J. & Coxon, J. Micro-consolidation occurs when learning an implicit motor sequence, but is not influenced by HIIT exercise. NPJ Sci Learn 9, 23 (2024). https://doi.org:10.1038/s41539-024-00238-6

      (58) Deleglise, A. et al. Human motor sequence learning drives transient changes in network topology and hippocampal connectivity early during memory consolidation. Cereb Cortex 33, 6120-6131 (2023). https://doi.org:10.1093/cercor/bhac489

      (59) Buzsaki, G. Hippocampal sharp wave-ripple: A cognitive biomarker for episodic memory and planning. Hippocampus 25, 1073-1188 (2015). https://doi.org:10.1002/hipo.22488

      (60) Gupta, M. W. & Rickard, T. C. Comparison of online, offline, and hybrid hypotheses of motor sequence learning using a quantitative model that incorporate reactive inhibition. Sci Rep 14, 4661 (2024). https://doi.org:10.1038/s41598-024-52726-9

      (61) Das, A., Karagiorgis, A., Diedrichsen, J., Stenner, M.-P. & Azanon, E. “Micro-offline gains” convey no benefit for motor skill learning. bioRxiv, 2024.2007.2011.602795 (2024). https://doi.org:10.1101/2024.07.11.602795

      (62) Mylonas, D. et al. Maintenance of Procedural Motor Memory across Brief Rest Periods Requires the Hippocampus. J Neurosci 44 (2024). https://doi.org:10.1523/JNEUROSCI.1839-23.2024

    1. Author response:

      eLife assessment

      This potentially useful study involves neuro-imaging and electrophysiology in a small cohort of congenital cataract patients after sight recovery and age-matched control participants with normal sight. It aims to characterize the effects of early visual deprivation on excitatory and inhibitory balance in the visual cortex. While the findings are taken to suggest the existence of persistent alterations in Glx/GABA ratio and aperiodic EEG signals, the evidence supporting these claims is incomplete. Specifically, small sample sizes, lack of a specific control cohort, and other methodological limitations will likely restrict the usefulness of the work, with relevance limited to scientists working in this particular subfield.

      As pointed out in the public reviews, there are only very few human models which allow for assessing the role of early experience on neural circuit development. While the prevalent research in permanent congenital blindness reveals the response and adaptation of the developing brain to an atypical situation (blindness), research in sight restoration addresses the question of whether and how atypical development can be remediated if typical experience (vision) is restored. The literature on the role of visual experience in the development of E/I balance in humans, assessed via Magnetic Resonance Spectroscopy (MRS), has been limited to a few studies on congenital permanent blindness. Thus, we assessed sight recovery individuals with a history of congenital blindness, as limited evidence from other researchers indicated that the visual cortex E/I ratio might differ compared to normally sighted controls.

      Individuals with total bilateral congenital cataracts who remained untreated until later in life are extremely rare, particularly if only carefully diagnosed patients are included in a study sample. A sample size of 10 patients is, at the very least, typical of past studies in this population, even for exclusively behavioral assessments. In the present study, in addition to behavioral assessment as an indirect measure of sensitive periods, we investigated participants with two neuroimaging methods (Magnetic Resonance Spectroscopy and electroencephalography) to directly assess the neural correlates of sensitive periods in humans. The electroencephalography data allowed us to link the results of our small sample to findings documented in large cohorts of both, sight recovery individuals and permanently congenitally blind individuals. As pointed out in a recent editorial recommending an “exploration-then-estimation procedure,” (“Consideration of Sample Size in Neuroscience Studies,” 2020), exploratory studies like ours provide crucial direction and specific hypotheses for future work.

      We included an age-matched sighted control group recruited from the same community, measured in the same scanner and laboratory, to assess whether early experience is necessary for a typical excitatory/inhibitory (E/I) ratio to emerge in adulthood. The present findings indicate that this is indeed the case. Based on these results, a possible question to answer in future work, with individuals who had developmental cataracts, is whether later visual deprivation causes similar effects. Note that even if visual deprivation at a later stage in life caused similar effects, the current results would not be invalidated; by contrast, they are essential to understand future work on late (permanent or transient) blindness.

      Thus, we think that the present manuscript has far reaching implications for our understanding of the conditions under which E/I balance, a crucial characteristic of brain functioning, emerges in humans.

      Finally, our manuscript is one of the first few studies which relates MRS neurotransmitter concentrations to parameters of EEG aperiodic activity. Since present research has been using aperiodic activity as a correlate of the E/I ratio, and partially of higher cognitive functions, we think that our manuscript additionally contributes to a better understanding of what might be measured with aperiodic neurophysiological activity.

      Public Reviews:

      Reviewer #1 (Public Review):

      Summary:

      In this human neuroimaging and electrophysiology study, the authors aimed to characterize the effects of a period of visual deprivation in the sensitive period on excitatory and inhibitory balance in the visual cortex. They attempted to do so by comparing neurochemistry conditions ('eyes open', 'eyes closed') and resting state, and visually evoked EEG activity between ten congenital cataract patients with recovered sight (CC), and ten age-matched control participants (SC) with normal sight.

      First, they used magnetic resonance spectroscopy to measure in vivo neurochemistry from two locations, the primary location of interest in the visual cortex, and a control location in the frontal cortex. Such voxels are used to provide a control for the spatial specificity of any effects because the single-voxel MRS method provides a single sampling location. Using MR-visible proxies of excitatory and inhibitory neurotransmission, Glx and GABA+ respectively, the authors report no group effects in GABA+ or Glx, no difference in the functional conditions 'eyes closed' and 'eyes open'. They found an effect of the group in the ratio of Glx/GABA+ and no similar effect in the control voxel location. They then performed multiple exploratory correlations between MRS measures and visual acuity, and reported a weak positive correlation between the 'eyes open' condition and visual acuity in CC participants.

      The same participants then took part in an EEG experiment. The authors selected only two electrodes placed in the visual cortex for analysis and reported a group difference in an EEG index of neural activity, the aperiodic intercept, as well as the aperiodic slope, considered a proxy for cortical inhibition. They report an exploratory correlation between the aperiodic intercept and Glx in one out of three EEG conditions.

      The authors report the difference in E/I ratio, and interpret the lower E/I ratio as representing an adaptation to visual deprivation, which would have initially caused a higher E/I ratio. Although intriguing, the strength of evidence in support of this view is not strong. Amongst the limitations are the low sample size, a critical control cohort that could provide evidence for a higher E/I ratio in CC patients without recovered sight for example, and lower data quality in the control voxel.

      Strengths of study:

      How sensitive period experience shapes the developing brain is an enduring and important question in neuroscience. This question has been particularly difficult to investigate in humans. The authors recruited a small number of sight-recovered participants with bilateral congenital cataracts to investigate the effect of sensitive period deprivation on the balance of excitation and inhibition in the visual brain using measures of brain chemistry and brain electrophysiology. The research is novel, and the paper was interesting and well-written.

      Limitations:

      (1.1) Low sample size. Ten for CC and ten for SC, and a further two SC participants were rejected due to a lack of frontal control voxel data. The sample size limits the statistical power of the dataset and increases the likelihood of effect inflation.

      Applying strict criteria, we only included individuals who were born with no patterned vision in the CC group. The population of individuals who have remained untreated past infancy is small in India, despite a higher prevalence of childhood cataract than Germany. Indeed, from the original 11 CC and 11 SC participants tested, one participant each from the CC and SC group had to be rejected, as their data had been corrupted, resulting in 10 participants in each group.

      It was a challenge to recruit participants from this rare group with no history of neurological diagnosis/intake of neuromodulatory medications, who were able and willing to undergo both MRS and EEG. For this study, data collection took more than 1.5 years.

      We took care of the validity of our results with two measures; first, assessed not just MRS, but additionally, EEG measures of E/I ratio. The latter allowed us to link results to a larger population of CC individuals, that is, we replicated the results of a larger group of 38 individuals (Ossandón et al., 2023) in our sub-group.

      Second, we included a control voxel. As predicted, all group effects were restricted to the occipital voxel.

      (1.2) Lack of specific control cohort. The control cohort has normal vision. The control cohort is not specific enough to distinguish between people with sight loss due to different causes and patients with congenital cataracts with co-morbidities. Further data from more specific populations, such as patients whose cataracts have not been removed, with developmental cataracts, or congenitally blind participants, would greatly improve the interpretability of the main finding. The lack of a more specific control cohort is a major caveat that limits a conclusive interpretation of the results.

      The existing work on visual deprivation and neurochemical changes, as assessed with MRS, has been limited to permanent congenital blindness. In fact, most of the studies on permanent blindness included only congenitally blind or early blind humans (Coullon et al., 2015; Weaver et al., 2013), or, in separate studies, only late-blind individuals (Bernabeu et al., 2009). Thus, accordingly, we started with the most “extreme” visual deprivation model, sight recovery after congenital blindness. If we had not observed any group difference compared to normally sighted controls, investigating other groups might have been trivial. Based on our results, subsequent studies in late blind individuals, and then individuals with developmental cataracts, can be planned with clear hypotheses.

      (1.3) MRS data quality differences. Data quality in the control voxel appears worse than in the visual cortex voxel. The frontal cortex MRS spectrum shows far broader linewidth than the visual cortex (Supplementary Figures). Compared to the visual voxel, the frontal cortex voxel has less defined Glx and GABA+ peaks; lower GABA+ and Glx concentrations, lower NAA SNR values; lower NAA concentrations. If the data quality is a lot worse in the FC, then small effects may not be detectable.

      Worse data quality in the frontal than the visual cortex has been repeatedly observed in the MRS literature, attributable to magnetic field distortions (Juchem & Graaf, 2017) resulting from the proximity of the region to the sinuses (recent example: (Rideaux et al., 2022)). Nevertheless, we chose the frontal control region rather than a parietal voxel, given the potential  neurochemical changes in multisensory regions of the parietal cortex due to blindness. Such reorganization would be less likely in frontal areas associated with higher cognitive functions. Further, prior MRS studies of the visual cortex have used the frontal cortex as a control region as well (Pitchaimuthu et al., 2017; Rideaux et al., 2022).

      In the present study, we checked that the frontal cortex datasets for Glx and GABA+ concentrations were of sufficient quality: the fit error was below 8.31% in both groups (Supplementary Material S3). For reference, Mikkelsen et al. reported a mean GABA+ fit error of 6.24 +/- 1.95% from a posterior cingulate cortex voxel across 8 GE scanners, using the Gannet pipeline. No absolute cutoffs have been proposed for fit errors. However, MRS studies in special populations (I/E ratio assessed in narcolepsy (Gao et al., 2024), GABA concentration assessed in Autism Spectrum Disorder (Maier et al., 2022)) have used frontal cortex data with a fit error of <10% to identify differences between cohorts (Gao et al., 2024; Pitchaimuthu et al., 2017). Based on the literature, MRS data from the frontal voxel of the present study would have been of sufficient quality to uncover group differences.

      In the revised manuscript, we will add the recently published MRS quality assessment form to the supplementary materials. Additionally, we would like to allude to our apriori prediction of group differences for the visual cortex, but not for the frontal cortex voxel.

      (1.4) Because of the direction of the difference in E/I, the authors interpret their findings as representing signatures of sight improvement after surgery without further evidence, either within the study or from the literature. However, the literature suggests that plasticity and visual deprivation drive the E/I index up rather than down. Decreasing GABA+ is thought to facilitate experience-dependent remodelling. What evidence is there that cortical inhibition increases in response to a visual cortex that is over-sensitised due to congenital cataracts? Without further experimental or literature support this interpretation remains very speculative.

      Indeed, higher inhibition was not predicted, which we attempt to reconcile in our discussion section. We base our discussion mainly on the non-human animal literature, which has shown evidence of homeostatic changes after prolonged visual deprivation in the adult brain (Barnes et al., 2015). It is also interesting to note that after monocular deprivation in adult humans, resting GABA+ levels decreased in the visual cortex (Lunghi et al., 2015). Assuming that after delayed sight restoration, adult neuroplasticity mechanisms must be employed, these studies would predict a “balancing” of the increased excitatory drive following sight restoration by a commensurate increase in inhibition (Keck et al., 2017). Additionally, the EEG results of the present study allowed for speculation regarding the underlying neural mechanisms of an altered E/I ratio. The aperiodic EEG activity suggested higher spontaneous spiking (increased intercept) and increased inhibition (steeper aperiodic slope between 1-20 Hz) in CC vs SC individuals (Ossandón et al., 2023).

      In the revised manuscript, we will more clearly indicate that these speculations are based primarily on non-human animal work, due to the lack of human studies on the subject.

      (1.5) Heterogeneity in the patient group. Congenital cataract (CC) patients experienced a variety of duration of visual impairment and were of different ages. They presented with co-morbidities (absorbed lens, strabismus, nystagmus). Strabismus has been associated with abnormalities in GABAergic inhibition in the visual cortex. The possible interactions with residual vision and confounds of co-morbidities are not experimentally controlled for in the correlations, and not discussed.

      The goal of the present study was to assess whether we would observe changes in E/I ratio after restoring vision at all. We would not have included patients without nystagmus in the CC group of the present study, since it would have been unlikely that they experienced congenital patterned visual deprivation. Amongst diagnosticians, nystagmus or strabismus might not be considered genuine “comorbidities” that emerge in people with congenital cataracts. Rather, these are consequences of congenital visual deprivation, which we employed as diagnostic criteria. Similarly, absorbed lenses are clear signs that cataracts were congenital. As in other models of experience dependent brain development (e.g. the extant literature on congenital permanent blindness, including anophthalmic individuals (Coullon et al., 2015; Weaver et al., 2013), some uncertainty remains regarding whether the (remaining, in our case) abnormalities of the eye, or the blindness they caused, are the factors driving neural changes. In case of people with reversed congenital cataracts, at least the retina is considered to be intact, as they would otherwise not receive cataract removal surgery.

      However, we consider it unlikely that strabismus caused the group differences, because the present study shows group differences in the Glx/GABA+ ratio at rest, regardless of eye opening or eye closure, for which strabismus would have caused distinct effects. By contrast, the link between GABA concentration and, for example, interocular suppression in strabismus, have so far been documented during visual stimulation (Mukerji et al., 2022; Sengpiel et al., 2006), and differed in direction depending on the amblyopic vs. non-amblyopic eye. Further, one MRS study did not find group differences in GABA concentration between the visual cortices of 16 amblyopic individuals and sighted controls (Mukerji et al., 2022), supporting that the differences in Glx/GABA+ concentration which we observed were driven by congenital deprivation, and not amblyopia-associated visual acuity or eye movement differences.  

      In the revised manuscript, we will discuss the inclusion criteria in more detail, and the aforementioned reasons why our data remains interpretable.

      (1.6) Multiple exploratory correlations were performed to relate MRS measures to visual acuity (shown in Supplementary Materials), and only specific ones were shown in the main document. The authors describe the analysis as exploratory in the 'Methods' section. Furthermore, the correlation between visual acuity and E/I metric is weak, and not corrected for multiple comparisons. The results should be presented as preliminary, as no strong conclusions can be made from them. They can provide a hypothesis to test in a future study.

      In the revised manuscript, we will clearly indicate that the exploratory correlation analyses are reported to put forth hypotheses for future studies.

      (1.7) P.16 Given the correlation of the aperiodic intercept with age ("Age negatively correlated with the aperiodic intercept across CC and SC individuals, that is, a flattening of the intercept was observed with age"), age needs to be controlled for in the correlation between neurochemistry and the aperiodic intercept. Glx has also been shown to negatively correlate with age.

      The correlation between chronological age and aperiodic intercept was observed across groups, but the correlation between Glx and the intercept of the aperiodic EEG activity was seen only in the CC group, even though the SC group was matched for age. Thus, such a correlation was very unlikely to  be predominantly driven by an effect of chronological age.

      In the revised manuscript, we will add the linear regressions with age as a covariate included below, for the relationship between aperiodic intercept and Glx concentration in the CC group. 

      a. A linear regression was conducted within the CC group to predict the intercept during visual stimulation, based on age and visual cortex Glx concentration. The results of the regression analysis indicated that the model explained a significant proportion of the variance in the aperiodic intercept, 𝑅2\=0.82_, t_(2,7)=16.1_, 𝑝=0.0024._ Note that the coefficient for age was not significant, 𝛽=0.007, t(7)=0.82, 𝑝=0.439. The regression coefficients and their respective statistics are presented in Author response table 1.

      Author response table 1.

      Regression Analysis Summary for Predicting Aperiodic Intercept (Visual Stimulation) in the CC group

      b. A linear regression was conducted to predict the intercept during eye opening at rest, based on age and visual cortex Glx concentration. The results of the regression analysis indicated that the model explained a significant proportion of the variance in the aperiodic intercept, 𝑅2\=0.842_, t_(2,7)=18.6,  𝑝=0.00159_._ Note that the coefficient for age was not significant, 𝛽=−0.005, t(7)=−0.90, 𝑝=0.400. The regression coefficients and their respective statistics are presented in Author response table 2.

      Author response table 2.

      Regression Analysis Summary for Predicting Aperiodic Intercept (Eyes Open) in the CC group

      c. Given that the Glx coefficient is significant in both models and age does not significantly predict either outcome, it can be concluded that Glx independently predicts the intercept of the aperiodic intercept.

      (1.8) Multiple exploratory correlations were performed to relate MRS to EEG measures (shown in Supplementary Materials), and only specific ones were shown in the main document. Given the multiple measures from the MRS, the correlations with the EEG measures were exploratory, as stated in the text, p.16, and in Figure 4. Yet the introduction said that there was a prior hypothesis "We further hypothesized that neurotransmitter changes would relate to changes in the slope and intercept of the EEG aperiodic activity in the same subjects." It would be great if the text could be revised for consistency and the analysis described as exploratory.

      In the revised manuscript, we will improve the phrasing. We consider the correlation analyses as exploratory due to our sample size and the absence of prior work. However, we did hypothesize that both MRS and EEG markers would concurrently be altered in CC vs SC individuals.

      (1.9) The analysis for the EEG needs to take more advantage of the available data. As far as I understand, only two electrodes were used, yet far more were available as seen in their previous study (Ossandon et al., 2023). The spatial specificity is not established. The authors could use the frontal cortex electrode (FP1, FP2) signals as a control for spatial specificity in the group effects, or even better, all available electrodes and correct for multiple comparisons. Furthermore, they could use the aperiodic intercept vs Glx in SC to evaluate the specificity of the correlation to CC.

      The aperiodic intercept and slope did not differ between CC and SC individuals for Fp1 and Fp2, suggesting the spatial specificity of the results. In the revised manuscript, we will add this analysis to the supplementary material.

      Author response image 1.

      Aperiodic intercept (top) and slope (bottom) for congenital cataract-reversal (CC, red) and age-matched normally sighted control (SC, blue) individuals. Distributions of these parameters are displayed as violin plots for three conditions; at rest with eyes closed (EC), at rest with eyes open (EO) and during visual stimulation (LU). Aperiodic parameters were calculated across electrodes Fp1 and Fp2. Solid black lines indicate mean values, dotted black lines indicate median values. Coloured lines connect values of individual participants across conditions.

      Further, Glx concentration in the visual cortex did not correlate with the aperiodic intercept in the SC group (Figure 4), suggesting that this relationship was indeed specific to the CC group.

      The data from all electrodes has been analyzed and published in other studies as well (Pant et al., 2023; Ossandón et al., 2023).

      Reviewer #2 (Public Review):

      Summary:

      The manuscript reports non-invasive measures of activity and neurochemical profiles of the visual cortex in congenitally blind patients who recovered vision through the surgical removal of bilateral dense cataracts. The declared aim of the study is to find out how restoring visual function after several months or years of complete blindness impacts the balance between excitation and inhibition in the visual cortex.

      Strengths:

      The findings are undoubtedly useful for the community, as they contribute towards characterising the many ways this special population differs from normally sighted individuals. The combination of MRS and EEG measures is a promising strategy to estimate a fundamental physiological parameter - the balance between excitation and inhibition in the visual cortex, which animal studies show to be heavily dependent upon early visual experience. Thus, the reported results pave the way for further studies, which may use a similar approach to evaluate more patients and control groups.

      Weaknesses:

      (2.1) The main issue is the lack of an appropriate comparison group or condition to delineate the effect of sight recovery (as opposed to the effect of congenital blindness). Few previous studies suggested an increased excitation/Inhibition ratio in the visual cortex of congenitally blind patients; the present study reports a decreased E/I ratio instead. The authors claim that this implies a change of E/I ratio following sight recovery. However, supporting this claim would require showing a shift of E/I after vs. before the sight-recovery surgery, or at least it would require comparing patients who did and did not undergo the sight-recovery surgery (as common in the field).

      Longitudinal studies would indeed be the best way to test the hypothesis that the lower E/I ratio in the CC group observed by the present study is a consequence of sight restoration. However, longitudinal studies involving neuroimaging are an effortful challenge, particularly in research conducted outside of major developed countries and dedicated neuroimaging research facilities. Crucially, however, had CC and SC individuals, as well as permanently congenitally blind vs SC individuals (Coullon et al., 2015; Weaver et al., 2013), not differed on any neurochemical markers, such a longitudinal study might have been trivial. Thus, in order to justify and better tailor longitudinal studies, cross-sectional studies are an initial step.

      (2.2) MR Spectroscopy shows a reduced GLX/GABA ratio in patients vs. sighted controls; however, this finding remains rather isolated, not corroborated by other observations. The difference between patients and controls only emerges for the GLX/GABA ratio, but there is no accompanying difference in either the GLX or the GABA concentrations. There is an attempt to relate the MRS data with acuity measurements and electrophysiological indices, but the explorative correlational analyses do not help to build a coherent picture. A bland correlation between GLX/GABA and visual impairment is reported, but this is specific to the patients' group (N=10) and would not hold across groups (the correlation is positive, predicting the lowest GLX/GABA ratio values for the sighted controls - the opposite of what is found). There is also a strong correlation between GLX concentrations and the EEG power at the lowest temporal frequencies. Although this relation is intriguing, it only holds for a very specific combination of parameters (of the many tested): only with eyes open, only in the patient group.

      We interpret these findings differently, that is, in the context of experiments from non-human animals and the larger MRS literature.

      Homeostatic control of E/I balance assumes that the ratio of excitation (reflected here by Glx) and inhibition (reflected here by GABA+) is regulated. Like prior work (Gao et al., 2024, 2024; Narayan et al., 2022; Perica et al., 2022; Steel et al., 2020; Takado et al., 2022; Takei et al., 2016), we assumed that the ratio of Glx/GABA+ is indicative of E/I balance rather than solely the individual neurotransmitter levels. One of the motivations for assessing the ratio vs the absolute concentration is that as per the underlying E/I balance hypothesis, a change in excitation would cause a concomitant change in inhibition, and vice versa, which has been shown in non-human animal work (Fang et al., 2021; Haider et al., 2006; Tao & Poo, 2005) and modeling research (Vreeswijk & Sompolinsky, 1996; Wu et al., 2022). Importantly, our interpretation of the lower E/I ratio is not just from the Glx/GABA+ ratio, but additionally, based on the steeper EEG aperiodic slope (1-20 Hz).  

      As in the discussion section and response 1.4, we did not expect to see a lower Glx/GABA+ ratio in CC individuals. We discuss the possible reasons for the direction of the correlation with visual acuity and aperiodic offset during passive visual stimulation, and offer interpretations and (testable) hypotheses.

      We interpret the direction of the  Glx/GABA+ correlation with visual acuity to imply that patients with highest (compensatory) balancing of the consequences of congenital blindness (hyperexcitation), in light of visual stimulation, are those who recover best. Note, the sighted control group was selected based on their “normal” vision. Thus, clinical visual acuity measures are not expected to sufficiently vary, nor have the resolution to show strong correlations with neurophysiological measures. By contrast, the CC group comprised patients highly varying in visual outcomes, and thus were ideal to investigate such correlations.

      This holds for the correlation between Glx and the aperiodic intercept, as well. Previous work has suggested that the intercept of the aperiodic activity is associated with broadband spiking activity in neural circuits (Manning et al., 2009). Thus, an atypical increase of spiking activity during visual stimulation, as indirectly suggested by “old” non-human primate work on visual deprivation (Hyvärinen et al., 1981) might drive a correlation not observed in healthy populations.

      In the revised manuscript, we will more clearly indicate in the discussion that these are possible post-hoc interpretations. We argue that given the lack of such studies in humans, it is all the more important that extant data be presented completely, even if the direction of the effects are not as expected.

      (2.3) For these reasons, the reported findings do not allow us to draw firm conclusions on the relation between EEG parameters and E/I ratio or on the impact of early (vs. late) visual experience on the excitation/inhibition ratio of the human visual cortex.

      Indeed, the correlations we have tested between the E/I ratio and EEG parameters were exploratory, and have been reported as such. The goal of our study was not to compare the effects of early vs. late visual experience. The goal was to study whether early visual experience is necessary for a typical E/I ratio in visual neural circuits. We provided clear evidence in favor of this hypothesis. Thus, the present results suggest the necessity of investigating the effects of late visual deprivation. In fact, such research is missing in permanent blindness as well.

      Reviewer #3 (Public Review):

      This manuscript examines the impact of congenital visual deprivation on the excitatory/inhibitory (E/I) ratio in the visual cortex using Magnetic Resonance Spectroscopy (MRS) and electroencephalography (EEG) in individuals whose sight was restored. Ten individuals with reversed congenital cataracts were compared to age-matched, normally sighted controls, assessing the cortical E/I balance and its interrelationship to visual acuity. The study reveals that the Glx/GABA ratio in the visual cortex and the intercept and aperiodic signal are significantly altered in those with a history of early visual deprivation, suggesting persistent neurophysiological changes despite visual restoration.

      My expertise is in EEG (particularly in the decomposition of periodic and aperiodic activity) and statistical methods. I have several major concerns in terms of methodological and statistical approaches along with the (over)interpretation of the results. These major concerns are detailed below.

      (3.1) Variability in visual deprivation:

      - The document states a large variability in the duration of visual deprivation (probably also the age at restoration), with significant implications for the sensitivity period's impact on visual circuit development. The variability and its potential effects on the outcomes need thorough exploration and discussion.

      We work with a rare, unique patient population, which makes it difficult to systematically assess the effects of different visual histories while maintaining stringent inclusion criteria such as complete patterned visual deprivation at birth. Regardless, we considered the large variance in age at surgery and time since surgery as supportive of our interpretation: group differences were found despite the large variance in duration of visual deprivation. Moreover, the existing variance was used to explore possible associations between behavior and neural measures, as well as neurochemical and EEG measures.

      In the revised manuscript, we will detail the advantages and disadvantages of our CC sample, with respect to duration of congenital visual deprivation.

      (3.2) Sample size:

      - The small sample size is a major concern as it may not provide sufficient power to detect subtle effects and/or overestimate significant effects, which then tend not to generalize to new data. One of the biggest drivers of the replication crisis in neuroscience.

      We address the small sample size in our discussion, and make clear that small sample sizes were due to the nature of investigations in special populations. It is worth noting that our EEG results fully align  with those of a larger sample of CC individuals (Ossandón et al., 2023), providing us confidence about their validity and reproducibility. Moreover, our MRS results and correlations of those with EEG parameters were spatially specific to occipital cortex measures, as predicted.

      The main problem with the correlation analyses between MRS and EEG measures is that the sample size is simply too small to conduct such an analysis. Moreover, it is unclear from the methods section that this analysis was only conducted in the patient group (which the reviewer assumed from the plots), and not explained why this was done only in the patient group. I would highly recommend removing these correlation analyses.

      We marked the correlation analyses as exploratory; note that we do not base most of our discussion on the results of these analyses. As indicated by Reviewer 1, reporting them allows for deriving more precise hypothesis for future studies. It has to be noted that we investigate an extremely rare population, tested outside of major developed economies and dedicated neuroimaging research facilities. In addition to being a rare patient group, these individuals come from poor communities. Therefore, we consider it justified to report these correlations as exploratory, providing direction for future research.

      (3.3) Statistical concerns:

      - The statistical analyses, particularly the correlations drawn from a small sample, may not provide reliable estimates (see https://www.sciencedirect.com/science/article/pii/S0092656613000858, which clearly describes this problem).

      It would undoubtedly be better to have a larger sample size. We nonetheless think it is of value to the research community to publish this dataset, since 10 multimodal data sets from a carefully diagnosed, rare population, representing a human model for the effects of early experience on brain development, are quite a lot.  Sample sizes in prior neuroimaging studies in transient blindness have most often ranged from n = 1 to n = 10. They nevertheless provided valuable direction for future research, and integration of results across multiple studies provides scientific insights.  

      Identifying possible group differences was the goal of our study, with the correlations being an exploratory analysis, which we have clearly indicated in the methods, results and discussion.

      - Statistical analyses for the MRS: The authors should consider some additional permutation statistics, which are more suitable for small sample sizes. The current statistical model (2x2) design ANOVA is not ideal for such small sample sizes. Moreover, it is unclear why the condition (EO & EC) was chosen as a predictor and not the brain region (visual & frontal) or neurochemicals. Finally, the authors did not provide any information on the alpha level nor any information on correction for multiple comparisons (in the methods section). Finally, even if the groups are matched w.r.t. age, the time between surgery and measurement, the duration of visual deprivation, (and sex?), these should be included as covariates as it has been shown that these are highly related to the measurements of interest (especially for the EEG measurements) and the age range of the current study is large.

      In our ANOVA models, the neurochemicals were the outcome variables, and the conditions were chosen as predictors based on prior work suggesting that Glx/GABA+ might vary with eye closure (Kurcyus et al., 2018). The study was designed based on a hypothesis of group differences localized to the occipital cortex, due to visual deprivation. The frontal cortex voxel was chosen to indicate whether these differences were spatially specific. Therefore, we conducted separate ANOVAs based on this study design.

      In the revised manuscript, we will add permutation analyses for our outcomes, as well as multiple regression models investigating whether the variance in visual history might have driven these results. Note that in the supplementary materials (S6, S7), we have reported the correlations between visual history metrics and MRS/EEG outcomes.

      The alpha level used for the ANOVA models specified in the methods section was 0.05. The alpha level for the exploratory analyses reported in the main manuscript was 0.008, after correcting for (6) multiple comparisons using the Bonferroni correction, also specified in the methods. Note that the p-values following correction are expressed as multiplied by 6, due to most readers assuming an alpha level of 0.05 (see response regarding large p-values).

      We used a control group matched for age and sex. Moreover, the controls were recruited and tested in the same institutes, using the same setup. We feel that we followed the gold standards for recruiting a healthy control group for a patient group.

      - EEG statistical analyses: The same critique as for the MRS statistical analyses applies to the EEG analysis. In addition: was the 2x3 ANOVA conducted for EO and EC independently? This seems to be inconsistent with the approach in the MRS analyses, in which the authors chose EO & EC as predictors in their 2x2 ANOVA.

      The 2x3 ANOVA was not conducted independently for the eyes open/eyes closed condition, the ANOVA conducted on the EEG metrics was 2x3 because it had group (CC, SC) and condition (eyes open (EO), eyes closed (EC) and visual stimulation (LU)) as predictors.

      - Figure 4: The authors report a p-value of >0.999 with a correlation coefficient of -0.42 with a sample size of 10 subjects. This can't be correct (it should be around: p = 0.22). All statistical analyses should be checked.

      As specified in the methods and figure legend, the reported p values in Figure 4 have been corrected using the Bonferroni correction, and therefore multiplied by the number of comparisons, leading to the seemingly large values.

      Additionally, to check all statistical analyses, we put the manuscript through an independent Statistics Check (Nuijten & Polanin, 2020) (https://michelenuijten.shinyapps.io/statcheck-web/) and will upload the consistency report with the revised supplementary material.

      - Figure 2c. Eyes closed condition: The highest score of the *Glx/GABA ratio seems to be ~3.6. In subplot 2a, there seem to be 3 subjects that show a Glx/GABA ratio score > 3.6. How can this be explained? There is also a discrepancy for the eyes-closed condition.

      The three subjects that show the Glx/GABA+ ratio > 3.6 in subplot 2a are in the SC group, whereas the correlations plotted in figure 2c are only for the CC group, where the highest score is indeed ~3.6.

      (3.4) Interpretation of aperiodic signal:

      - Several recent papers demonstrated that the aperiodic signal measured in EEG or ECoG is related to various important aspects such as age, skull thickness, electrode impedance, as well as cognition. Thus, currently, very little is known about the underlying effects which influence the aperiodic intercept and slope. The entire interpretation of the aperiodic slope as a proxy for E/I is based on a computational model and simulation (as described in the Gao et al. paper).

      Apart from the modeling work from Gao et al., multiple papers which have also been cited which used ECoG, EEG and MEG and showed concomitant changes in aperiodic activity with pharmacological manipulation of the E/I ratio (Colombo et al., 2019; Molina et al., 2020; Muthukumaraswamy & Liley, 2018). Further, several prior studies have interpreted changes in the aperiodic slope as reflective of changes in the E/I ratio, including studies of developmental groups (Favaro et al., 2023; Hill et al., 2022; McSweeney et al., 2023; Schaworonkow & Voytek, 2021) as well as patient groups (Molina et al., 2020; Ostlund et al., 2021).

      In the revised manuscript, we will cite those studies not already included in the introduction.

      - Especially the aperiodic intercept is a very sensitive measure to many influences (e.g. skull thickness, electrode impedance...). As crucial results (correlation aperiodic intercept and MRS measures) are facing this problem, this needs to be reevaluated. It is safer to make statements on the aperiodic slope than intercept. In theory, some of the potentially confounding measures are available to the authors (e.g. skull thickness can be computed from T1w images; electrode impedances are usually acquired alongside the EEG data) and could be therefore controlled.

      All electrophysiological measures indeed depend on parameters such as skull thickness and electrode impedance. As in the extant literature using neurophysiological measures to compare brain function between patient and control groups, we used a control group matched in age/ sex, recruited in the same region, tested with the same devices, and analyzed with the same analysis pipeline. For example, impedance was kept below 10 kOhm for all subjects. There is no evidence available suggesting that congenital cataracts are associated with changes in skull thickness that would cause the observed pattern of group results. Moreover, we cannot think of how any of the exploratory correlations between neurophysiological measures and MRS measures could be accounted for by a difference e.g. in skull thickness.

      - The authors wrote: "Higher frequencies (such as 20-40 Hz) have been predominantly associated with local circuit activity and feedforward signaling (Bastos et al., 2018; Van Kerkoerle et al., 2014); the increased 20-40 Hz slope may therefore signal increased spontaneous spiking activity in local networks. We speculate that the steeper slope of the aperiodic activity for the lower frequency range (1-20 Hz) in CC individuals reflects the concomitant increase in inhibition." The authors confuse the interpretation of periodic and aperiodic signals. This section refers to the interpretation of the periodic signal (higher frequencies). This interpretation cannot simply be translated to the aperiodic signal (slope).

      Prior work has not always separated the aperiodic and periodic components, making it unclear what might have driven these effects in our data. The interpretation of the higher frequency range was intended to contrast with the interpretations of lower frequency range, in order to speculate as to why the two aperiodic fits might go in differing directions. We will clarify our interpretation in the revised manuscript. Note that Ossandon et al. reported highly similar results (group differences for CC individuals and for permanently congenitally blind humans) for the aperiodic activity between 20-40 Hz and oscillatory activity in the gamma range. We will allude to these findings in the revised manuscript.

      - The authors further wrote: We used the slope of the aperiodic (1/f) component of the EEG spectrum as an estimate of E/I ratio (Gao et al., 2017; Medel et al., 2020; Muthukumaraswamy & Liley, 2018). This is a highly speculative interpretation with very little empirical evidence. These papers were conducted with ECoG data (mostly in animals) and mostly under anesthesia. Thus, these studies only allow an indirect interpretation by what the 1/f slope in EEG measurements is actually influenced.

      Note that Muthukumaraswamy et al. (2018) used different types of pharmacological manipulations and analyzed periodic and aperiodic MEG activity in addition to monkey ECoG (Medel et al., 2020) (now published as (Medel et al., 2023)) compared EEG activity in addition to ECoG data after propofol administration. The interpretation of our results are in line with a number of recent studies in developing (Hill et al., 2022; Schaworonkow & Voytek, 2021) and special populations using EEG. As mentioned above, several prior studies have used the slope of the 1/f component/aperiodic activity as an indirect measure of the E/I ratio (Favaro et al., 2023; Hill et al., 2022; McSweeney et al., 2023; Molina et al., 2020; Ostlund et al., 2021; Schaworonkow & Voytek, 2021), including studies using scalp-recorded EEG. We will make more clear in the introduction of the revised manuscript that this metric is indirect.

      While a full understanding of aperiodic activity needs to be provided, some convergent ideas have emerged . We think that our results contribute to this enterprise, since our study is, to the best of our knowledge, the first which assessed MRS measured neurotransmitter levels and EEG aperiodic activity.

      (3.5) Problems with EEG preprocessing and analysis:

      - It seems that the authors did not identify bad channels nor address the line noise issue (even a problem if a low pass filter of below-the-line noise was applied).

      As pointed out in the methods and Figure 1, we only analyzed data from two channels, O1 and O2, neither of which were rejected for any participant. Channel rejection was performed for the larger dataset, published elsewhere (Ossandón et al., 2023; Pant et al., 2023).

      In both published works, we did not consider frequency ranges above 40 Hz to avoid any possible contamination with line noise. Here, we focused on activity between 0 and 20 Hz, definitely excluding line noise contaminations. The low pass filter (FIR, 1-45 Hz) guaranteed that any spill-over effects of line noise would be restricted to frequencies just below the upper cutoff frequency.

      Additionally, a prior version of the analysis used the cleanline.m function to remove line noise before filtering, and the group differences remained stable. We will report this analysis in the supplementary version of the revised manuscript. Further, both groups were measured in the same lab, making line noise as an account for the observed group effects highly unlikely. Finally, any of the exploratory MRS-EEG correlations would be hard to explain if the EEG parameters would be contaminated with line noise.

      - What was the percentage of segments that needed to be rejected due to the 120μV criteria? This should be reported specifically for EO & EC and controls and patients.

      The mean percentage of 1 second segments rejected for each resting state condition is below. Mean percentage of 6.25 long segments rejected in each group for the visual stimulation condition are also included, and will be added to the revised manuscript:

      Author response table 3.

      - The authors downsampled the data to 60Hz to "to match the stimulation rate". What is the intention of this? Because the subsequent spectral analyses are conflated by this choice (see Nyquist theorem).

      This data were collected as part of a study designed to evoke alpha activity with visual white-noise, which ranged in luminance with equal power at all frequencies from 1-60 Hz, restricted by the refresh rate of the monitor on which stimuli were presented (Pant et al., 2023). This paradigm and method was developed by VanRullen and colleagues (Schwenk et al., 2020; Vanrullen & MacDonald, 2012), wherein the analysis requires the same sampling rate between the presented frequencies and the EEG data. The downsampling function used here automatically applies an anti-aliasing filter (EEGLAB 2019) .

      - "Subsequently, baseline removal was conducted by subtracting the mean activity across the length of an epoch from every data point." The actual baseline time segment should be specified.

      The time segment was the length of the epoch, that is, 1 second for the resting state conditions and 6.25 seconds for the visual stimulation conditions. This will be explicitly stated in the revised manuscript.

      - "We excluded the alpha range (8-14 Hz) for this fit to avoid biasing the results due to documented differences in alpha activity between CC and SC individuals (Bottari et al., 2016; Ossandón et al., 2023; Pant et al., 2023)." This does not really make sense, as the FOOOF algorithm first fits the 1/f slope, for which the alpha activity is not relevant.

      We did not use the FOOOF algorithm/toolbox in this manuscript. As stated in the methods, we used a 1/f fit to the 1-20 Hz spectrum in the log-log space, and subtracted this fit from the original spectrum to obtain the corrected spectrum. Given the pronounced difference in alpha power between groups (Bottari et al., 2016; Ossandón et al., 2023; Pant et al., 2023), we were concerned it might drive differences in the exponent values.  Our analysis pipeline had been adapted from previous publications of our group and other labs (Ossandón et al., 2023; Voytek et al., 2015; Waschke et al., 2017).

      We have conducted the analysis with and without the exclusion of the alpha range, as well as using the FOOOF toolbox both in the 1-20 Hz and 20-40 Hz ranges (Ossandón et al., 2023); The findings of a steeper slope in the 1-20 Hz range as well as lower alpha power in CC vs SC individuals remained stable. In Ossandón et al., the comparison between the piecewise fits and FOOOF fits led the authors to use the former as it outperformed the FOOOF algorithm for their data.

      - The model fits of the 1/f fitting for EO, EC, and both participant groups should be reported.

      In Figure 3 of the manuscript, we depicted the mean spectra and 1/f fits for each group. We will add the fit quality metrics and show individual subjects’ fits in the revised manuscript.

      (3.6) Validity of GABA measurements and results:

      - According the a newer study by the authors of the Gannet toolbox (https://analyticalsciencejournals.onlinelibrary.wiley.com/doi/abs/10.1002/nbm.5076), the reliability and reproducibility of the gamma-aminobutyric acid (GABA) measurement can vary significantly depending on acquisition and modeling parameter. Thus, did the author address these challenges?

      We took care of data quality while acquiring MRS data by ensuring appropriate voxel placement and linewidth prior to scanning. Acquisition as well as modeling parameters were constant for both groups, so they cannot have driven group differences.

      The linked article compares the reproducibility of GABA measurement using Osprey, which was released in 2020 and uses linear combination modeling to fit the peak as opposed to Gannet’s simple peak fitting (Hupfeld et al., 2024). The study finds better test-retest reliability for Osprey compared to Gannet’s method.

      As the present work was conceptualized in 2018, we used Gannet 3.0, which was the state-of-the-art edited spectral analysis toolbox at the time, and still is widely used. In the revised manuscript, we will include a supplementary section reanalyzing the main findings with Osprey.

      - Furthermore, the authors wrote: "We confirmed the within-subject stability of metabolite quantification by testing a subset of the sighted controls (n=6) 2-4 weeks apart. Looking at the supplementary Figure 5 (which would be rather plotted as ICC or Blant-Altman plots), the within-subject stability compared to between-subject variability seems not to be great. Furthermore, I don't think such a small sample size qualifies for a rigorous assessment of stability.

      Indeed, we did not intend to provide a rigorous assessment of within-subject stability. Rather, we aimed to confirm that data quality/concentration ratios did not systematically differ between the same subjects tested longitudinally; driven, for example, by scanner heating or time of day. As with the phantom testing, we attempted to give readers an idea of the quality of the data, as they were collected from a primarily clinical rather than a research site.

      In the revised manuscript we will remove the statement regarding stability, and add the Blant-Altman plot.

      - "Why might an enhanced inhibitory drive, as indicated by the lower Glx/GABA ratio" Is this interpretation really warranted, as the results of the group differences in the Glx/GABA ratio seem to be rather driven by a decreased Glx concentration in CC rather than an increased GABA (see Figure 2).

      We used the Glx/GABA+ ratio as a measure, rather than individual Glx or GABA+ concentration, which did not significantly differ between groups. As detailed in Response 2.2, we think this metric aligns better with an underlying E/I balance hypothesis and has been used in many previous studies (Gao et al., 2024; Liu et al., 2015; Narayan et al., 2022; Perica et al., 2022).

      Our interpretation of an enhanced inhibitory drive additionally comes from the combination of aperiodic EEG (1-20 Hz) and MRS measures, which, when considered together, are consistent with a decreased E/I ratio.

      In the revised manuscript, we will rephrase this sentence accordingly. 

      - Glx concentration predicted the aperiodic intercept in CC individuals' visual cortices during ambient and flickering visual stimulation. Why specifically investigate the Glx concentration, when the paper is about E/I ratio?

      As stated in the methods, we exploratorily assessed the relationship between all MRS parameters (Glx, GABA+ and Glx/GABA+ ratio) with the aperiodic parameters (slope, offset), and corrected for multiple comparisons accordingly. We think this is a worthwhile analysis considering the rarity of the dataset/population (see 1.2, 1.6, 2.1 and reviewer 1’s comments about future hypotheses). We only report the Glx – aperiodic intercept correlation in the main manuscript as it survived correction for multiple comparisons.

      (3.7) Interpretation of the correlation between MRS measurements and EEG aperiodic signal:

      - The authors wrote: "The intercept of the aperiodic activity was highly correlated with the Glx concentration during rest with eyes open and during flickering stimulation (also see Supplementary Material S11). Based on the assumption that the aperiodic intercept reflects broadband firing (Manning et al., 2009; Winawer et al., 2013), this suggests that the Glx concentration might be related to broadband firing in CC individuals during active and passive visual stimulation." These results should not be interpreted (or with very caution) for several reasons (see also problem with influences on aperiodic intercept and small sample size). This is a result of the exploratory analyses of correlating every EEG parameter with every MRS parameter. This requires well-powered replication before any interpretation can be provided. Furthermore and importantly: why should this be specifically only in CC patients, but not in the SC control group?

      We indicate clearly in all parts of the manuscript that these correlations are presented as exploratory. Further, we interpret the Glx-aperiodic offset correlation, and none of the others, as it survived the Bonferroni correction for multiple comparisons. We offer a hypothesis in the discussion section as to why such a correlation might exist in the CC but not the SC group (see response 2.2), and do not speculate further.

      (3.8) Language and presentation:

      - The manuscript requires language improvements and correction of numerous typos. Over-simplifications and unclear statements are present, which could mislead or confuse readers (see also interpretation of aperiodic signal).

      In the revision, we will check that speculations are clearly marked and typos are removed.

      - The authors state that "Together, the present results provide strong evidence for experience-dependent development of the E/I ratio in the human visual cortex, with consequences for behavior." The results of the study do not provide any strong evidence, because of the small sample size and exploratory analyses approach and not accounting for possible confounding factors.

      We disagree with this statement and allude to convergent evidence of both MRS and neurophysiological measures. The latter link to corresponding results observed in a larger sample of CC individuals (Ossandón et al., 2023).

      - "Our results imply a change in neurotransmitter concentrations as a consequence of *restoring* vision following congenital blindness." This is a speculative statement to infer a causal relationship on cross-sectional data.

      As mentioned under 2.1, we conducted a cross-sectional study which might justify future longitudinal work. In order to advance science, new testable hypotheses were put forward at the end of a manuscript.

      In the revised manuscript we will add “might imply” to better indicate the hypothetical character of this idea.

      - In the limitation section, the authors wrote: "The sample size of the present study is relatively high for the rare population , but undoubtedly, overall, rather small." This sentence should be rewritten, as the study is plein underpowered. The further justification "We nevertheless think that our results are valid. Our findings neurochemically (Glx and GABA+ concentration), and anatomically (visual cortex) specific. The MRS parameters varied with parameters of the aperiodic EEG activity and visual acuity. The group differences for the EEG assessments corresponded to those of a larger sample of CC individuals (n=38) (Ossandón et al., 2023), and effects of chronological age were as expected from the literature." These statements do not provide any validation or justification of small samples. Furthermore, the current data set is a subset of an earlier published paper by the same authors "The EEG data sets reported here were part of data published earlier (Ossandón et al., 2023; Pant et al., 2023)." Thus, the statement "The group differences for the EEG assessments corresponded to those of a larger sample of CC individuals (n=38) " is a circular argument and should be avoided.

      Our intention was not to justify having a small sample, but to justify why we think the results might be valid as they align with/replicate existing literature.

      In the revised manuscript, we will add a figure showing that the EEG results of the 10 subjects considered here correspond to those of the 28 other subjects of Ossandon et al. We will adapt the text accordingly, clearly stating that the pattern of EEG results of the ten subjects reported here replicate those of the 28 additional subjects of Ossandon et al. (2023).

      References

      Barnes, S. J., Sammons, R. P., Jacobsen, R. I., Mackie, J., Keller, G. B., & Keck, T. (2015). Subnetwork-specific homeostatic plasticity in mouse visual cortex in vivo. Neuron, 86(5), 1290–1303. https://doi.org/10.1016/J.NEURON.2015.05.010

      Bernabeu, A., Alfaro, A., García, M., & Fernández, E. (2009). Proton magnetic resonance spectroscopy (1H-MRS) reveals the presence of elevated myo-inositol in the occipital cortex of blind subjects. NeuroImage, 47(4), 1172–1176. https://doi.org/10.1016/j.neuroimage.2009.04.080

      Bottari, D., Troje, N. F., Ley, P., Hense, M., Kekunnaya, R., & Röder, B. (2016). Sight restoration after congenital blindness does not reinstate alpha oscillatory activity in humans. Scientific Reports. https://doi.org/10.1038/srep24683

      Colombo, M. A., Napolitani, M., Boly, M., Gosseries, O., Casarotto, S., Rosanova, M., Brichant, J. F., Boveroux, P., Rex, S., Laureys, S., Massimini, M., Chieregato, A., & Sarasso, S. (2019). The spectral exponent of the resting EEG indexes the presence of consciousness during unresponsiveness induced by propofol, xenon, and ketamine. NeuroImage, 189(September 2018), 631–644. https://doi.org/10.1016/j.neuroimage.2019.01.024

      Consideration of Sample Size in Neuroscience Studies. (2020). Journal of Neuroscience, 40(21), 4076–4077. https://doi.org/10.1523/JNEUROSCI.0866-20.2020

      Coullon, G. S. L., Emir, U. E., Fine, I., Watkins, K. E., & Bridge, H. (2015). Neurochemical changes in the pericalcarine cortex in congenital blindness attributable to bilateral anophthalmia. Journal of Neurophysiology. https://doi.org/10.1152/jn.00567.2015

      Fang, Q., Li, Y. T., Peng, B., Li, Z., Zhang, L. I., & Tao, H. W. (2021). Balanced enhancements of synaptic excitation and inhibition underlie developmental maturation of receptive fields in the mouse visual cortex. Journal of Neuroscience, 41(49), 10065–10079. https://doi.org/10.1523/JNEUROSCI.0442-21.2021

      Favaro, J., Colombo, M. A., Mikulan, E., Sartori, S., Nosadini, M., Pelizza, M. F., Rosanova, M., Sarasso, S., Massimini, M., & Toldo, I. (2023). The maturation of aperiodic EEG activity across development reveals a progressive differentiation of wakefulness from sleep. NeuroImage, 277. https://doi.org/10.1016/J.NEUROIMAGE.2023.120264

      Gao, Y., Liu, Y., Zhao, S., Liu, Y., Zhang, C., Hui, S., Mikkelsen, M., Edden, R. A. E., Meng, X., Yu, B., & Xiao, L. (2024). MRS study on the correlation between frontal GABA+/Glx ratio and abnormal cognitive function in medication-naive patients with narcolepsy. Sleep Medicine, 119, 1–8. https://doi.org/10.1016/j.sleep.2024.04.004

      Haider, B., Duque, A., Hasenstaub, A. R., & McCormick, D. A. (2006). Neocortical network activity in vivo is generated through a dynamic balance of excitation and inhibition. Journal of Neuroscience. https://doi.org/10.1523/JNEUROSCI.5297-05.2006

      Hill, A. T., Clark, G. M., Bigelow, F. J., Lum, J. A. G., & Enticott, P. G. (2022). Periodic and aperiodic neural activity displays age-dependent changes across early-to-middle childhood. Developmental Cognitive Neuroscience, 54, 101076. https://doi.org/10.1016/J.DCN.2022.101076

      Hupfeld, K. E., Zöllner, H. J., Hui, S. C. N., Song, Y., Murali-Manohar, S., Yedavalli, V., Oeltzschner, G., Prisciandaro, J. J., & Edden, R. A. E. (2024). Impact of acquisition and modeling parameters on the test–retest reproducibility of edited GABA+. NMR in Biomedicine, 37(4), e5076. https://doi.org/10.1002/nbm.5076

      Hyvärinen, J., Carlson, S., & Hyvärinen, L. (1981). Early visual deprivation alters modality of neuronal responses in area 19 of monkey cortex. Neuroscience Letters, 26(3), 239–243. https://doi.org/10.1016/0304-3940(81)90139-7

      Juchem, C., & Graaf, R. A. de. (2017). B0 magnetic field homogeneity and shimming for in vivo magnetic resonance spectroscopy. Analytical Biochemistry, 529, 17–29. https://doi.org/10.1016/j.ab.2016.06.003

      Keck, T., Hübener, M., & Bonhoeffer, T. (2017). Interactions between synaptic homeostatic mechanisms: An attempt to reconcile BCM theory, synaptic scaling, and changing excitation/inhibition balance. Current Opinion in Neurobiology, 43, 87–93. https://doi.org/10.1016/J.CONB.2017.02.003

      Kurcyus, K., Annac, E., Hanning, N. M., Harris, A. D., Oeltzschner, G., Edden, R., & Riedl, V. (2018). Opposite Dynamics of GABA and Glutamate Levels in the Occipital Cortex during Visual Processing. Journal of Neuroscience, 38(46), 9967–9976. https://doi.org/10.1523/JNEUROSCI.1214-18.2018

      Liu, B., Wang, G., Gao, D., Gao, F., Zhao, B., Qiao, M., Yang, H., Yu, Y., Ren, F., Yang, P., Chen, W., & Rae, C. D. (2015). Alterations of GABA and glutamate-glutamine levels in premenstrual dysphoric disorder: A 3T proton magnetic resonance spectroscopy study. Psychiatry Research - Neuroimaging, 231(1), 64–70. https://doi.org/10.1016/J.PSCYCHRESNS.2014.10.020

      Lunghi, C., Berchicci, M., Morrone, M. C., & Russo, F. D. (2015). Short‐term monocular deprivation alters early components of visual evoked potentials. The Journal of Physiology, 593(19), 4361. https://doi.org/10.1113/JP270950

      Maier, S., Düppers, A. L., Runge, K., Dacko, M., Lange, T., Fangmeier, T., Riedel, A., Ebert, D., Endres, D., Domschke, K., Perlov, E., Nickel, K., & Tebartz van Elst, L. (2022). Increased prefrontal GABA concentrations in adults with autism spectrum disorders. Autism Research, 15(7), 1222–1236. https://doi.org/10.1002/aur.2740

      Manning, J. R., Jacobs, J., Fried, I., & Kahana, M. J. (2009). Broadband shifts in local field potential power spectra are correlated with single-neuron spiking in humans. The Journal of Neuroscience : The Official Journal of the Society for Neuroscience, 29(43), 13613–13620. https://doi.org/10.1523/JNEUROSCI.2041-09.2009

      McSweeney, M., Morales, S., Valadez, E. A., Buzzell, G. A., Yoder, L., Fifer, W. P., Pini, N., Shuffrey, L. C., Elliott, A. J., Isler, J. R., & Fox, N. A. (2023). Age-related trends in aperiodic EEG activity and alpha oscillations during early- to middle-childhood. NeuroImage, 269, 119925. https://doi.org/10.1016/j.neuroimage.2023.119925

      Medel, V., Irani, M., Crossley, N., Ossandón, T., & Boncompte, G. (2023). Complexity and 1/f slope jointly reflect brain states. Scientific Reports, 13(1), 21700. https://doi.org/10.1038/s41598-023-47316-0

      Medel, V., Irani, M., Ossandón, T., & Boncompte, G. (2020). Complexity and 1/f slope jointly reflect cortical states across different E/I balances. bioRxiv, 2020.09.15.298497. https://doi.org/10.1101/2020.09.15.298497

      Molina, J. L., Voytek, B., Thomas, M. L., Joshi, Y. B., Bhakta, S. G., Talledo, J. A., Swerdlow, N. R., & Light, G. A. (2020). Memantine Effects on Electroencephalographic Measures of Putative Excitatory/Inhibitory Balance in Schizophrenia. Biological Psychiatry: Cognitive Neuroscience and Neuroimaging, 5(6), 562–568. https://doi.org/10.1016/j.bpsc.2020.02.004

      Mukerji, A., Byrne, K. N., Yang, E., Levi, D. M., & Silver, M. A. (2022). Visual cortical γ−aminobutyric acid and perceptual suppression in amblyopia. Frontiers in Human Neuroscience, 16. https://doi.org/10.3389/fnhum.2022.949395

      Muthukumaraswamy, S. D., & Liley, D. T. (2018). 1/F electrophysiological spectra in resting and drug-induced states can be explained by the dynamics of multiple oscillatory relaxation processes. NeuroImage, 179(November 2017), 582–595. https://doi.org/10.1016/j.neuroimage.2018.06.068

      Narayan, G. A., Hill, K. R., Wengler, K., He, X., Wang, J., Yang, J., Parsey, R. V., & DeLorenzo, C. (2022). Does the change in glutamate to GABA ratio correlate with change in depression severity? A randomized, double-blind clinical trial. Molecular Psychiatry, 27(9), 3833—3841. https://doi.org/10.1038/s41380-022-01730-4

      Nuijten, M. B., & Polanin, J. R. (2020). “statcheck”: Automatically detect statistical reporting inconsistencies to increase reproducibility of meta-analyses. Research Synthesis Methods, 11(5), 574–579. https://doi.org/10.1002/jrsm.1408

      Ossandón, J. P., Stange, L., Gudi-Mindermann, H., Rimmele, J. M., Sourav, S., Bottari, D., Kekunnaya, R., & Röder, B. (2023). The development of oscillatory and aperiodic resting state activity is linked to a sensitive period in humans. NeuroImage, 275, 120171. https://doi.org/10.1016/J.NEUROIMAGE.2023.120171

      Ostlund, B. D., Alperin, B. R., Drew, T., & Karalunas, S. L. (2021). Behavioral and cognitive correlates of the aperiodic (1/f-like) exponent of the EEG power spectrum in adolescents with and without ADHD. Developmental Cognitive Neuroscience, 48, 100931. https://doi.org/10.1016/j.dcn.2021.100931

      Pant, R., Ossandón, J., Stange, L., Shareef, I., Kekunnaya, R., & Röder, B. (2023). Stimulus-evoked and resting-state alpha oscillations show a linked dependence on patterned visual experience for development. NeuroImage: Clinical, 103375. https://doi.org/10.1016/J.NICL.2023.103375

      Perica, M. I., Calabro, F. J., Larsen, B., Foran, W., Yushmanov, V. E., Hetherington, H., Tervo-Clemmens, B., Moon, C.-H., & Luna, B. (2022). Development of frontal GABA and glutamate supports excitation/inhibition balance from adolescence into adulthood. Progress in Neurobiology, 219, 102370. https://doi.org/10.1016/j.pneurobio.2022.102370

      Pitchaimuthu, K., Wu, Q. Z., Carter, O., Nguyen, B. N., Ahn, S., Egan, G. F., & McKendrick, A. M. (2017). Occipital GABA levels in older adults and their relationship to visual perceptual suppression. Scientific Reports, 7(1). https://doi.org/10.1038/S41598-017-14577-5

      Rideaux, R., Ehrhardt, S. E., Wards, Y., Filmer, H. L., Jin, J., Deelchand, D. K., Marjańska, M., Mattingley, J. B., & Dux, P. E. (2022). On the relationship between GABA+ and glutamate across the brain. NeuroImage, 257, 119273. https://doi.org/10.1016/J.NEUROIMAGE.2022.119273

      Schaworonkow, N., & Voytek, B. (2021). Longitudinal changes in aperiodic and periodic activity in electrophysiological recordings in the first seven months of life. Developmental Cognitive Neuroscience, 47. https://doi.org/10.1016/j.dcn.2020.100895

      Schwenk, J. C. B., VanRullen, R., & Bremmer, F. (2020). Dynamics of Visual Perceptual Echoes Following Short-Term Visual Deprivation. Cerebral Cortex Communications, 1(1). https://doi.org/10.1093/TEXCOM/TGAA012

      Sengpiel, F., Jirmann, K.-U., Vorobyov, V., & Eysel, U. T. (2006). Strabismic Suppression Is Mediated by Inhibitory Interactions in the Primary Visual Cortex. Cerebral Cortex, 16(12), 1750–1758. https://doi.org/10.1093/cercor/bhj110

      Steel, A., Mikkelsen, M., Edden, R. A. E., & Robertson, C. E. (2020). Regional balance between glutamate+glutamine and GABA+ in the resting human brain. NeuroImage, 220. https://doi.org/10.1016/J.NEUROIMAGE.2020.117112

      Takado, Y., Takuwa, H., Sampei, K., Urushihata, T., Takahashi, M., Shimojo, M., Uchida, S., Nitta, N., Shibata, S., Nagashima, K., Ochi, Y., Ono, M., Maeda, J., Tomita, Y., Sahara, N., Near, J., Aoki, I., Shibata, K., & Higuchi, M. (2022). MRS-measured glutamate versus GABA reflects excitatory versus inhibitory neural activities in awake mice. Journal of Cerebral Blood Flow & Metabolism, 42(1), 197. https://doi.org/10.1177/0271678X211045449

      Takei, Y., Fujihara, K., Tagawa, M., Hironaga, N., Near, J., Kasagi, M., Takahashi, Y., Motegi, T., Suzuki, Y., Aoyama, Y., Sakurai, N., Yamaguchi, M., Tobimatsu, S., Ujita, K., Tsushima, Y., Narita, K., & Fukuda, M. (2016). The inhibition/excitation ratio related to task-induced oscillatory modulations during a working memory task: A multtimodal-imaging study using MEG and MRS. NeuroImage, 128, 302–315. https://doi.org/10.1016/J.NEUROIMAGE.2015.12.057

      Tao, H. W., & Poo, M. M. (2005). Activity-dependent matching of excitatory and inhibitory inputs during refinement of visual receptive fields. Neuron, 45(6), 829–836. https://doi.org/10.1016/J.NEURON.2005.01.046

      Vanrullen, R., & MacDonald, J. S. P. (2012). Perceptual echoes at 10 Hz in the human brain. Current Biology. https://doi.org/10.1016/j.cub.2012.03.050

      Voytek, B., Kramer, M. A., Case, J., Lepage, K. Q., Tempesta, Z. R., Knight, R. T., & Gazzaley, A. (2015). Age-related changes in 1/f neural electrophysiological noise. Journal of Neuroscience, 35(38). https://doi.org/10.1523/JNEUROSCI.2332-14.2015

      Vreeswijk, C. V., & Sompolinsky, H. (1996). Chaos in neuronal networks with balanced excitatory and inhibitory activity. Science, 274(5293), 1724–1726. https://doi.org/10.1126/SCIENCE.274.5293.1724

      Waschke, L., Wöstmann, M., & Obleser, J. (2017). States and traits of neural irregularity in the age-varying human brain. Scientific Reports 2017 7:1, 7(1), 1–12. https://doi.org/10.1038/s41598-017-17766-4

      Weaver, K. E., Richards, T. L., Saenz, M., Petropoulos, H., & Fine, I. (2013). Neurochemical changes within human early blind occipital cortex. Neuroscience. https://doi.org/10.1016/j.neuroscience.2013.08.004

      Wu, Y. K., Miehl, C., & Gjorgjieva, J. (2022). Regulation of circuit organization and function through inhibitory synaptic plasticity. Trends in Neurosciences, 45(12), 884–898. https://doi.org/10.1016/J.TINS.2022.10.006

    1. Author response:

      Reviewer #1 (Public review):

      (1) Legionella effectors are often activated by binding to eukaryote-specific host factors, including actin. The authors should test the following: a) whether Lfat1 can fatty acylate small G-proteins in vitro; b) whether this activity is dependent on actin binding; and c) whether expression of the Y240A mutant in mammalian cells affects the fatty acylation of Rac3 (Figure 6B), or other small G-proteins.

      We were not able to express and purify the full-length recombinant Lfat1 to perform fatty acylation of small GTPases in vitro. However, in cellulo overexpression of the Y240A mutant still retained ability to fatty acylate Rac3 and another small GTPase RheB (see Author response image 1 below). We postulate that under infection conditions, actin-binding might be required to fatty acylate certain GTPases due to the small amount of effector proteins that secreted into the host cell.

      Author response image 1.

      (2) It should be demonstrated that lysine residues on small G-proteins are indeed targeted by Lfat1. Ideally, the functional consequences of these modifications should also be investigated. For example, does fatty acylation of G-proteins affect GTPase activity or binding to downstream effectors?

      We have mutated K178 on RheB and showed that this mutation abolished its fatty acylation by Lfat1 (see Author response image 2 below). We were not able to test if fatty acylation by Lfat1 affect downstream effector binding.

      Author response image 2.

      (3) Line 138: Can the authors clarify whether the Lfat1 ABD induces bundling of F-actin filaments or promotes actin oligomerization? Does the Lfat1 ABD form multimers that bring multiple filaments together? If Lfat1 induces actin oligomerization, this effect should be experimentally tested and reported. Additionally, the impact of Lfat1 binding on actin filament stability should be assessed. This is particularly important given the proposed use of the ABD as an actin probe.

      The ABD domain does not form oligomer as evidenced by gel filtration profile of the ABD domain. However, we do see F-actin bundling in our in vitro -F-actin polymerization experiment when both actin and ABD are in high concentration (data not shown). Under low concentration of ABD, there is not aggregation/bundling effect of F-actin.

      (4) Line 180: I think it's too premature to refer to the interaction as having "high specificity and affinity." We really don't know what else it's binding to.

      We have revised the text and reworded the sentence by removing "high specificity and affinity."

      (5) The authors should reconsider the color scheme used in the structural figures, particularly in Figures 2D and S4.

      Not sure the comments on the color scheme of the structure figures.

      (6) In Figure 3E, the WT curve fits the data poorly, possibly because the actin concentration exceeds the Kd of the interaction. It might fit better to a quadratic.

      We have performed quadratic fitting and replaced Figure 3E.

      (7) The authors propose that the individual helices of the Lfat1 ABD could be expressed on separate proteins and used to target multi-component biological complexes to F-actin by genetically fusing each component to a split alpha-helix. This is an intriguing idea, but it should be tested as a proof of concept to support its feasibility and potential utility.

      It is a good suggestion. We plan to thoroughly test the feasibility of this idea as one of our future directions.

      (7) The plot in Figure S2D appears cropped on the X-axis or was generated from a ~2× binned map rather than the deposited one (pixel size ~0.83 Å, plot suggests ~1.6 Å). The reported pixel size is inconsistent between the Methods and Table 1-please clarify whether 0.83 Å refers to super-resolution.

      Yes, 0.83 Å is super-resolution. We have updated in the cryoEM table

      Reviewer #2 (Public review):

      Weaknesses:

      (1) The authors should use biochemical reactions to analyze the KFAT of Llfat1 on one or two small GTPases shown to be modified by this effector in cellulo. Such reactions may allow them to determine the role of actin binding in its biochemical activity. This notion is particularly relevant in light of recent studies that actin is a co-factor for the activity of LnaB and Ceg14 (PMID: 39009586; PMID: 38776962; PMID: 40394005). In addition, the study should be discussed in the context of these recent findings on the role of actin in the activity of L. pneumophila effectors.

      We have new data showed that Actin binding does not affect Lfat1 enzymatic activity. (see figure; response to Reviewer #1). We have added this new data as Figure S7 to the paper. Accordingly, we also revised the discussion by adding the following paragraph.

      “The discovery of Lfat1 as an F-actin–binding lysine fatty acyl transferase raised the intriguing question of whether its enzymatic activity depends on F-actin binding. Recent studies have shown that other Legionella effectors, such as LnaB and Ceg14, use actin as a co-factor to regulate their activities. For instance, LnaB binds monomeric G-actin to enhance its phosphoryl-AMPylase activity toward phosphorylated residues, resulting in unique ADPylation modifications in host proteins (Fu et al, 2024; Wang et al, 2024). Similarly, Ceg14 is activated by host actin to convert ATP and dATP into adenosine and deoxyadenosine monophosphate, thereby modulating ATP levels in L. pneumophila–infected cells (He et al, 2025). However, this does not appear to be the case for Lfat1. We found that Lfat1 mutants defective in F-actin binding retained the ability to modify host small GTPases when expressed in cells (Figure S7). These findings suggest that, rather than serving as a co-factor, F-actin may serve to localize Lfat1 via its actin-binding domain (ABD), thereby confining its activity to regions enriched in F-actin and enabling spatial specificity in the modification of host targets.”

      (2) The development of the ABD domain of Llfat1 as an F-actin domain is a nice extension of the biochemical and structural experiments. The authors need to compare the new probe to those currently commonly used ones, such as Lifeact, in labeling of the actin cytoskeleton structure.

      We fully agree with the reviewer’s insightful suggestion. However, a direct comparison of the Lfat1 ABD domain with commonly used actin probes such as Lifeact, as well as evaluation of the split α-helix probe (as suggested by Reviewer #1), would require extensive and technically demanding experiments. These are important directions that we plan to pursue in future studies.

    1. R0:

      Reviewer #1: Manuscript as reviewed meets PLOS Global Public Health publication requirements, the author(s) clearly presented the study background, methods, results, discussions and conclusion. My comments and revision request are minor formatting and suggested input. No ethics concerns at this time. Reviewer #2: This is a well-written paper with clear methodology. From the perspective of data science applied to public health, this manuscript does a great job of clearly discussing and defining its methodology, which are all the current best practices. Correcting for class imbalance was a good choice, given the low prevalence of EC in the survey population. The use of SMOTE on the training set only ensured minimal data leakage, and is the current best practice. Using such a large variety of machine learning models creates a challenge in describing each model well enough within one manuscript, and the author did a good job of balancing that challenge.

      I only have a few minor suggestions toc clarify the methodology of the manuscript:

      Please specify upfront how many observations were used in training and testing, and specify how many positive EC outcomes were included in the testing set. With such a low prevalence of a positive outcome in a relatively small set of observations, it is worth mentioning that there are perhaps only 10-20 positive outcomes being predicted in the test set. In the absence of weighting, it may be that characteristics of those few positive outcomes in test set are biasing the predictors, and this is worth mentioning.

      Please discuss how the initial 38 variables were selected from the survey. If there was an initial expert judgment on inclusion into the variable set for feature selection, that should be mentioned.

      Cluster design was mentioned in the PMA survey. This indicates that the survey includes survey weights of some kind. Please discuss whether those weights were addressed in the machine learning methods, or defend why they were not included in the model design. Survey weights can be included in machine learning models to make the predictors more representative of the population of interest.

      In the discussion, please discuss the impact of low precision, where there were many false positives compared to true positives. While it is mentioned, there are consequences (e.g., loss of trust) for low precision prediction models in public health, and this characteristic of the findings could be discussed more.

      Consider including a SHAP dependance plot, because potential interactions are discussed (e.g., knowledge and ad exposure) without showing evidence. A SHAP Dependence plot could take care of this.

      Consider explicitly discussing the limitation of cross-sectional survey data used for prediction, where proxies were used in place of quantitative evidence (e.g., exposure to ads to proxy perceptions).

      Overall, great work, timely, and well constructed. Reviewer #3: SEE word document attached with clear table

      Manuscript Number: PGPH-D-25-01837 Review report

      This manuscript demonstrates a significant strength in its application of advanced machine learning and Explainable AI (XAI) to address the critical public health challenge of low emergency contraceptive (EC) use in Ethiopia. By rigorously testing multiple models and using SMOTE to handle severe class imbalance, it identifies key modifiable predictors like primarily EC awareness and media exposure rather than static socioeconomic factors. The use of SHAP values transforms complex model outputs into actionable insights, revealing that knowledge gaps are the primary barrier. This approach provides a powerful, data-driven blueprint for designing targeted interventions, such as tailored media campaigns and improved health counselling, to effectively increase EC uptake and reduce unintended pregnancies. However, the following points may need to be considered, so as to improve the quality of the paper.

      Topic/ subtopic Issue Suggestions Title: Predicting Utilization of Emergency Contraceptive Usage in Ethiopia and Identifying Its Predictors Using Machine Learning Redundancy. "Utilization" and "Usage" mean the same thing. Predicting the Utilization of Emergency Contraception in Ethiopia and Identifying Its Predictors Using Machine Learning. Affiliation Inconsistent institution name. on page 1 says "College of Medicine Health Science" while first page of manuscript is "College of Health Science". Use consistent affiliation name Abstract "Traditional analyses have struggled to identify complex predictors." For flow, consider: Traditional statistical analyses have struggled to… Abstract "with SMOTE used to address class imbalance" – Grammar: This is a dependent clause. It should be connected to the previous sentence. ..., and the SMOTE was used to address class imbalance. Abstract "Findings highlight that knowledge gaps, not poverty or access, are key barriers to EC use." – Clarity: "access" is vague. Be more specific. ...not poverty or physical access barriers, are key. Introduction Page 3: "moderate’s" Change to moderates ("the way the education level moderate’s religion-based stigma"). Introduction "drives excessive maternal mortality rates of over 500 deaths per 100,000 live births, drives poverty cycles, constrains girls' and women's educational and economic opportunities, and overwhelms poor healthcare infrastructures." – The word "drives" is used twice in close succession. ...contributes to high maternal mortality rates of over 500 deaths per 100,000 live births, perpetuates cycles of poverty, constrains... Introduction "is a central preventive intervention" is a crucial preventive intervention Introduction "the use of EC remains embarrassingly low" "Embarrassingly" is subjective and informal. ...remains critically low. Introduction "tempts women to shun services" Word choice not good. ...pressures women to shun services. Introduction "woefully underserved" Informal. ...significantly underserved. Introduction "yield the predictive resolution necessary" "Resolution" unusual in this context. ...yield the predictive accuracy necessary Introduction "vastness tests for fairness" – Phrase is unclear and likely an error. Correct the phrase to clarity Methods Data Source & Inclusion Criteria: The criteria for selecting the 2,334 women from the larger PMA sample of 8,943 are not explicitly stated. Was it a complete case analysis? This needs clarification as it affects the generalizability of the findings. Clarify if sampling was done or it was a complete case study Methods "The dataset demonstrates low overall missing data prevalence" –"Prevalence" is for diseases outbreaks. The missing data were minimal overall; Methods "offering robust classifier building while preserving real performance measurement." ...facilitating the development of robust classifiers while preserving a realistic assessment of performance. Results "nailing 17 true positives" Informal word choice. ...correctly identifying 17 true positives... Results "It manages this recall strength at the expense of precision, though, which sits at approximately 11%." – "Sits at" is informal. It achieves this high recall at the expense of precision, which was approximately 11%. Results "The most influential positive feature was “heard_emergency”, indicating awareness of emergency services has the greatest influence..." add which . The most influential positive feature was “heard_emergency”, which indicates that awareness of emergency contraception has the greatest influence... Results "This resonates with core assumptions of health behavior theories like the Health Belief Model, which posit perceived knowledge as a harbinger of action." "Harbinger" misused. ...which posit knowledge as a prerequisite for action. Results Page 18: "radio-implemented" Change to radio-delivered or radio-based. Results "Even positive, this reflects continued systemic disincentives documented elsewhere" – Unclear Even not a correct word. Although positively associated, this factor reflects... Results "all the sources of blunting the effect of being in contact with the health system." Grammatically incorrect and unclear. ...all of which blunt the effect of health system contact. Results "One of the thoughtful discoveries of SHAP values was the sizeable negative impact" "Thoughtful" incorrect. A notable discovery from the SHAP analysis was. Results "Isolated use of SMOTE in the training set" – "Isolated" wrong word. Applying SMOTE exclusively to the training set Results "It shifted the ML model from being a prediction device to an analysis tool, not just deciding which features were significant, but the size and sign of their effects, and significantly, potential interactions" Not clear because of parallel verbs. It transformed the ML model from a prediction device into an analytical tool, revealing not only which features were significant but also the magnitude and direction of their effects, as well as potential interactions. Results "Simulation by counterfactual SHAP analysis suggests a hypothetical 30% increase in EC knowledge might boost utilization by approximately 12.7%, a valuable public health gain." The sentence needs clearer explanation. Counterfactual simulation using SHAP values (e.g., calculating the mean impact of increasing the "heard_emergency" feature value) suggested that a 30% increase in EC knowledge could potentially increase utilization by approximately 12.7%, representing a valuable public health gain. Results "Geographic ML modeling over the geographic data would also potentially be able to further optimize resource deployment" Repetition: "Geographic" used twice. Rewrite the sentence for clarity Results "the implied vulnerability evidenced by the 'forced pregnancy' variable (despite missing data concerns) underscore" Not clear as the subject-verb disagreement. .use the word..underscores. Methods Model Selection Justification: The list of eight algorithms is comprehensive, but justification for simpler models like Naive Bayes is weak. Justify the inclusion of Naïve Bayes. Is it possible because they were included as benchmarks. Methods Evaluation Metrics: AUC-ROC emphasized, but for imbalanced problems F1-Score or Precision-Recall AUC may be better. Also consider using F1-Score or Precision as the data is not balanced or Justify the use of AUC-ROC Methods Model Performance Presentation: Logistic Regression focus unclear since Gradient Boosting achieved higher AUC-ROC (0.85). Consider Gradient Boosting as it achieved AUC-ROC 0.85 OR Explain rationale (e.g., performance vs. interpretability). Results Confusion Matrix Analysis (Figure 3): Issue: The analysis states precision is "approximately 11%." Based on the described confusion matrix (TP=17, FP=138), precision is 17 / (17+138) = 11.0%. This is a critical weakness of the model that deserves more emphasis. It means ~89% of the people predicted to be EC users were actually non-users. This has huge implications for the cost and efficiency of any intervention based on this model Discuss this trade-off explicitly: "The model's high recall (85%) comes at the cost of low precision (11%), resulting in a high false positive rate. This suggests the model is well-suited as a screening tool where identifying most true cases is prioritized over resource efficiency, but would require secondary screening or low-cost interventions to target the large number of false positives." Discussion Addressing Limitations More Forcefully: Underreporting of EC likely major issue. Add: "A key limitation is the potential for significant underreporting of EC use due to social desirability bias and stigma..." Conclusion "myth-busting" Word choice is Informal. myth-dispelling Conclusion "stock guarantees of EC" Not clear Consider write as guaranteed EC stock availability Conclusion "This research provides an ethical and evidence-based blueprint to accelerate gains in reducing maternal mortality and advancing reproductive autonomy in Ethiopia and similar settings." – Awkward phrasing. .Conside rephrasing as ..blueprint to reduce maternal mortality and advance... Reviewer #4: This manuscript applies machine learning (ML) and explainable AI (XAI) methods to predict emergency contraceptive (EC) use among women in Ethiopia, using data from the 2023 PMA survey. The authors compare eight algorithms, address severe class imbalance with SMOTE, and use SHAP values to interpret predictors. They find that awareness of EC is the strongest predictor, followed by media exposure and health facility discussions, while demographic variables show limited predictive value.

      However, the results as currently presented are unreliable. Major inconsistencies in reported performance metrics (e.g., contradictory precision values, implausible Naive Bayes results, inflated accuracy) call into question the validity of the analyses. In addition, the small number of EC users makes the modeling unstable, and subgroup analyses are not feasible with this dataset. These issues, combined with over-interpretation of SHAP as causal, limit both the methodological credibility and substantive contribution of the paper.

      Contradictory precision results The performance metrics are inconsistent. Table 4 shows Logistic Regression with SMOTE achieving precision = 0.72 and recall = 0.85, yet the confusion matrix description reports precision at only ~11%. These cannot both be correct. This discrepancy raises questions about the accuracy of the reported results and must be clarified.

      Inflated accuracy The reported accuracy of 0.95 for Logistic Regression with SMOTE appears implausibly high given the extreme class imbalance (4.4% EC use). Accuracy is not an informative measure in this context, and such values raise concerns about potential data leakage or overly optimistic validation. The authors should confirm that the outcome variable or proxy features were not inadvertently included in the predictors.

      Over-interpretation of SHAP The SHAP analysis is framed in causal terms (e.g., a 30% increase in knowledge leading to a 12.7% increase in use). SHAP values describe associations within the model, not causal effects. The manuscript should temper these statements and present SHAP findings as indicators of relative predictive importance, not intervention outcomes.

      Implausible Naive Bayes results Naive Bayes is reported as having accuracy of only 0.06 pre-SMOTE. Given that 95% of the sample did not use EC, even a trivial majority-class classifier would achieve ~95% accuracy. Such a result suggests an error in coding or reporting that must be checked.

      Small minority class vs. model complexity Only 103 EC users were present in the dataset. Training and tuning eight algorithms with hyperparameter searches on such a small minority class risks overfitting and unstable results, even with SMOTE. This limitation should be acknowledged explicitly, with emphasis on the need for validation on independent samples.

      Subgroup analysis claims The manuscript claims fairness testing across subgroups (rural/urban, religion, age), but no results are presented. With so few EC users, subgroup analyses would be underpowered and unreliable. It would be more appropriate to note this limitation rather than imply subgroup robustness.

      Causality Issue The manuscript repeatedly interprets predictive associations as though they were causal effects. For example, SHAP values are used to suggest that increasing knowledge by 30% would increase EC use by 12.7%. Since the data are cross-sectional and observational, such statements are not justified. Machine learning models in this setting can identify predictive patterns, but they cannot establish causal relationships between predictors and outcomes. This overreach is particularly concerning because it could mislead policymakers or practitioners into believing the study provides evidence of causal effects. Reviewer #5: Summary This study investigates the underuse of emergency contraception in Ethiopia using a machine learning framework. Strengths include the application of multiple algorithms, careful handling of class imbalance, and the use of Explainable AI to interpret model outputs. The paper is generally well-structured, and the methodological workflow is presented clearly. At the same time, the results are presented in a way that overstates the model’s practical utility while giving insufficient attention to the precision–recall trade-off. The manuscript should be revised to consistently acknowledge the low precision across the abstract, results, and discussion, and to provide a clear justification for the relevance of a high-recall, low-precision model in this public health context. The limitation posed by the small number of positive cases in the validation set should also be explicitly discussed. Addressing these points is necessary to strengthen the scientific validity of the work. Specific comments 1. Title; It should be shortened to remove redundancy since Utilization and Usage mean the same thing 2. Abstract. I think something key was missed. The aurthors state a recall of 0.85 without mentioning the precision. I see that (Figure 3, page 20) show that the precision is approximately 11%. My understanding of this that for every 100 women the model flags as likely EC non-users who need intervention, 89 of them are false alarms. An abstract must present a balanced view of performance. 3. Methods (About the data): A sample size of 2,334 with a 4.4% prevalence means you only have ~103 positive cases (EC users). After an 80/20 train-test split, your test set contains only ~21 positive cases. This number is critically small and raises serious questions about the stability and generalizability of your reported performance metrics. A different random split could yield vastly different results. I suggest that such a major limitation is addressed upfront in the limitations section and acknowledged in the methods section. 4. Data balancing; I like the write up of this section 5. Evaluation Metrics; The text states the test set has 18.7% EC users, but the abstract and data balancing section state the overall prevalence is 4.4%. Please clarify this discrepancy. Is 18.7% a typo? Or did the stratified split result in a test set with a much higher prevalence than the overall dataset? This needs to be consistent. Could you also add the precision-recall plots, since you state that they were tracked. 6. Results: - In Table 4, the columns are F1 and Score. This seems like a typo. It should likely be a single column: F1 Score. Please correct. - Lastly, i think it would be good to acknowledge the weaknesses of SMOTE Reviewer #6: The title of the article is: Predicting Utilization of Emergency Contraceptive Usage in Ethiopia and Identifying Its Predictors Using Machine Learning. The author explains that traditional analyses have struggled to identify complex predictors and therefore they used machine learning (ML) and Explainable AI (XAI) to improve the prediction and interpretability of Emergency Contraceptive (EC) use. The paper can be published with the following corrections and some are extremely important. In particular methodological perspectives. Category Authors Contribution Comments Objectives The primary objectives are twofold:

      one, to predict the likelihood of EC use with far greater accuracy than conventional regression techniques;

      two, to identify the key modifiable socio-behavioural predictors e.g., self-efficacy, mass media exposure, provider perception, and women's autonomy through XAI methods like SHAP values to yield interpretability and actionable insights. First objective can be modified. Far greater is a vague statement. Measuring accuracy is an indicator of choosing between models but conventional regression techniques why has a problem in this study should focus on that.

      Second objective seems motivation of the study. This objective should be written in clear sentence. Identify predictors to yield interpretability and actionable insights are subjective things. These objective seems ambiguous.

      Methodological view Page 5: Methodologically, it represents a new contribution by rigorously testing the performance of eight alternative ML classifiers and developing an optimized analytical pipeline specifically designed to handle skewed healthcare datasets prevalent in rare outcomes like EC use

      Theoretically, it applies the Socio-Ecological Model (SEM) framework to hierarchically analyze predictors at levels of individual (knowledge, attitudes), interpersonal (partner communication, family influence), community (stigma norms, access), and policy (health system factors) providing an integrated explanation for the interrelating influences on EC behavior. It is not methodological contribution.

      Moreover, author mentioned theoretical contribution. However, it is just exploratory of the data.

      Methodology In page 4: In contrast to conventional statistical approaches, ML algorithms, such as random forests, gradient boosting machines (e.g., XGBoost), and neural networks, can particularly identify complex, high-dimensional patterns within diverse data sets, properly manage missing data, and produce personalized risk predictions with improved accuracy Author mentioned several times about conventional statistical technique. However, in the report author directly reported the model performance of ML. My suggestion is to first run the analysis using traditional or conventional methods and then compare with ML techniques. This is very important. Outcome Variable Page 8: The outcome of interest is EC Usage, a binary measure of whether emergency contraception was used in the last 12 months. This is the dependent variable for analysis. Redundant as at the beginning you mentioned outcome of interest is….. Missing data For handling missingness in our data, a stratified approach based on missingness mechanisms and rates was followed and so on……….. The author used many approaches and it is difficult to keep track. So it is better to explain step by step and pros and cons of each process. Moreover, explain why this approach is best in this study Variables Page 12

      Lots of category under one variable. Some category has very few observations. Justify the necessity. May be we can also show some cross-tabulation analysis result and report the p-value. Research Gap Page 19: The research goes beyond the correlational limitations of previous studies by utilizing predictive analytics to identify the modifiable factors and approximate their hypothetical effects What do you mean by correlational limitations? Moreover, over the report the previous studies were not mentioned in comparison to the authors current approaches. Sa add some recent references and explain the research gap. The Machine learning techniques are not new. So it is required to mention how those machine learning helps in your study as a novelty. All over the report there is a missing of synchronization and coherence of sentences. Moreover, the references, table titles etc are not space maintained. Abstract 1. SMOTE and SHAP 2. Conversely, recent reproductive events such as unintended pregnancy were linked to non-use. Static demographic factors showed poor predictive value. Findings highlight that knowledge gaps, not poverty or access, are key barriers to EC use. Tailored media campaigns and routine health counseling could enhance EC uptake. ML and XAI offer powerful tools for guiding targeted reproductive health interventions. 1. Did not mention what it is?

      1. The message of these sentences are not coherent. I think author can check the whole paper from an English native reviewer.

      R1:

      Reviewer #4: I appreciate the authors' thoughtful revisions and detailed responses. Several of my earlier comments were addressed—specifically, the correction of Naive Bayes reporting errors, improved acknowledgment of sample size limitations, and removal of unsupported subgroup analyses. These are welcome improvements. However, key concerns about the internal consistency of results, causal interpretation of SHAP analyses, and overextension of policy recommendations remain unresolved.

      First, while the outdated "11% precision" text has been removed, the confusion matrix values (TP=102, FP=180, FN=18) still do not correspond to the reported performance metrics. With these numbers, precision would equal roughly 0.36, not the 0.72 cited in Table 4. This suggests an ongoing internal inconsistency between the descriptive counts and the summary metrics. The lack of alignment raises continuing doubts about the reliability of the reported model performance.

      Second, the manuscript still places heavy emphasis on accuracy values approaching 0.92–0.95 despite a highly imbalanced outcome (4.4% EC use). Although the authors state that AUC-ROC and recall were prioritized, the presentation continues to foreground accuracy, which is misleading in this context. No calibration or uncertainty measures (e.g., Brier score, calibration curve) have been added, leaving the reader without a sense of how well the predicted probabilities reflect actual risk.

      Third, although the authors softened their language, the interpretation of SHAP values remains quasi-causal. The new statement—"counterfactual simulation using SHAP values … suggested that a 30% increase in EC knowledge could potentially increase utilization by approximately 12.7%", still presents SHAP outputs as if they represent real-world intervention effects. SHAP analysis identifies predictive associations within a model; it does not estimate the causal impact of changing a feature in the population. Likewise, subsequent phrases such as “integrating a predictive risk-scoring tool can help identify women at high risk” and “geographic machine learning modeling can optimize resource deployment” continue to frame the model as a validated operational tool. These remain prescriptive policy claims that move beyond what a cross-sectional, unvalidated predictive study can substantiate.

      Finally, while the tone of the manuscript has improved, the discussion still reads as policy advocacy rather than analytical interpretation. Phrases like "representing a valuable public health gain”" and "can help optimize resource deployment" give the impression of proven effectiveness rather than exploratory modeling. A clearer distinction between predictive insights and causal or operational evidence is necessary for the study to maintain methodological integrity.

    1. Thank you for submitting this paper. I think the paper requires substantial, major revisions to be published. Throughout the paper I noted many instances where references or examples would help make the intent clear. I also think the message of the paper would benefit from several figures to demonstrate workflows or ideas. The figures presented are essentially tables, and I think the message could be made clearer for the reader if they were presented as flow charts or at least with clear numbering to hook the ideas to the reader - e.g., Figures 1 & 2 would benefit from having numbers on the key ideas.

      The paper is lacking many instances of citation, and at times reads as though it is an essay delivering an opinion. I'm not sure if this is the type of article that the journal would like, but two examples of sentences missing citations are:

      1. "Over the last two decades, an unexpectedly large number of peer-reviewed findings across many scientific disciplines have been found to be irreproducible upon closer inspection." (Introduction, page 2)

      2. "A large number of examples cited in this context involves faulty software or inappropriate use of software" (Introduction, page 3)

      Two examples of sentences missing examples are:

      1. Experimental software evolves at a much faster pace than mature software, and documentation is rarely up to date or complete (in Mature vs. experimental software, page 7). Could the author provide more examples of what "experimental software" is? There is also consistent use of universal terms like "...is rarely up to date or complete", which would be better phrased as "is often not up to date or complete"

      2. There are various techniques for ensuring or verifying that a piece of software conforms to a formal specification.

      Overall the paper introduces many new concepts, and I think it would greatly benefit from being made shorter and more concise, with adding some key figures for the reader to refer back to to understand these new ideas. The paper is well written, and it is clear the author is a great writer, and has put a lot of thought into the ideas. However it is my opinion that because these ideas are so big and require so much unpacking, they are also harder to understand. The reader would benefit from having more guidance to come back to understand these ideas.

      I hope this review is helpful to the author.

      Review comments

      Introduction

      Highlight [page 2]: Ever since the beginnings of organized science in the 17th century, researchers are expected to put all facts supporting their conclusions on the table, and allow their peers to inspect them for accuracy, pertinence, completeness, and bias. Since the 1950s, critical inspection has become an integral part of the publication process in the form of peer review, which is still widely regarded as a key criterion for trustworthy results.

      • and Note [page 2]: Both of these statements feel like they should have some peer review, or reference on them, I believe. What was the beginnings of organised science in the 1600s? Why since the 1950s? Why not sooner? What happened then?

      Highlight [page 2]: Over the last two decades, an unexpectedly large number of peer-reviewed findings across many scientific disciplines have been found to be irreproducible upon closer inspection.

      Highlight [page 2]: In the quantitative sciences, almost all of today’s research critically relies on computational techniques, even when they are not the primary tool for investigation - and Note [page 2]: Again, it does feel like it would be great to acknowledge research in this space.

      Highlight [page 2]: But then, scientists mostly abandoned doubting.

      • and Note [page 2]: This feels like an essay, where show me the evidence for where you can say something like this?

      Highlight [page 2]: Automation bias

      • and Note [page 2]: What is automation bias?

      Highlight [page 3]: A large number of examples cited in this context involves faulty software or inappropriate use of software

      • and Note [page 3]: Can you provide some examples of the examples cited that you are referring to here?

      Highlight [page 3]: A particularly frequent issue is the inappropriate use of statistical inference techniques.

      • and Note [page 3]: Please provide citations to these frequent issues.

      Highlight [page 3]: The Open Science movement has made a first step towards dealing with automated reasoning in insisting on the necessity to publish scientific software, and ideally making the full development process transparent by the adoption of Open Source practices - and Note [page 3]: Could you provide an example of one of these Open Science movements?

      Highlight [page 3]: Almost no scientific software is subjected to independent review today.

      • and Note [page 3]: How can you justify this claim?

      Highlight [page 3]: In fact, we do not even have established processes for performing such reviews

      Highlight [page 3]: as I will show

      • and Note [page 3]: How will you show this?

      Highlight [page 3]: is as much a source of mistakes as defects in the software itself

      • and Note [page 3]: Again, this feels like a statement of fact without evidence or citation.

      Highlight [page 3]: This means that reviewing the use of scientific software requires particular attention to potential mismatches between the software’s behavior and its users’ expectations, in particular concerning edge cases and tacit assumptions made by the software developers. They are necessarily expressed somewhere in the software’s source code, but users are often not aware of them.

      • and Note [page 3]: The same can be said of assumptions for equations and mathematics - the problem here is dealing with abstraction of complexity and the potential unintended consequences.

      Highlight [page 4]: the preservation of epistemic diversity

      • and Note [page 4]: Please define epistemic diversity
      Reviewability of automated reasoning systems

      Highlight [page 5]: The five dimensions of scientific software that influence its reviewability.

      • and Note [page 5]: It might be clearer to number these in the figure, and also I might suggest changing the “convivial” - it’s a pretty unusual word?
      Wide-spectrum vs. situated software

      Highlight [page 6]: In between these extremes, we have in particular domain libraries and tools, which play a very important role in computational science, i.e. in studies where computational techniques are the principal means of investigation

      • and Note [page 6]: I’m not very clear on this example - can you provide an example of a “domain library” or “domain tool” ?

      Highlight [page 6]: Situated software is smaller and simpler, which makes it easier to understand and thus to review.

      • and Note [page 6]: I’m not sure I agree it is always smaller and simpler - the custom code for a new method could be incredibly complicated.

      Highlight [page 6]: Domain tools and libraries

      • and Note [page 6]: Can you give an example of this?
      Mature vs. experimental software

      Highlight [page 7]: Experimental software evolves at a much faster pace than mature software, and documentation is rarely up to date or complete

      • and Note [page 7]: Could the author provide more examples of what “experimental software” is? There is also consistent use of universal terms like “…is rarely up to date or complete”, which would be better phrased as “is often not up to date or complete”

      Highlight [page 7]: An extreme case of experimental software is machine learning models that are constantly updated with new training data.

      • and Note [page 7]: Such as…

      Highlight [page 7]: interlocutor

      • and Note [page 7]: suggest “middle man” or “mediator”, ‘interlocutor’ isn’t a very common word

      Highlight [page 7]: A grey zone

      • and Note [page 7]: I think it would be helpful to discuss black and white zones before this.

      Highlight [page 7]: The libraries of the scientific Python ecosystem

      • and Note [page 7]: Do you mean SciPy? https://scipy.org/. Can you provide an example of the frequent changes that break backward compatibility?

      Highlight [page 7]: too late that some of their critical dependencies are not as mature as they seemed to be

      • and Note [page 7]: Again, can you provide some evidence for this?

      Highlight [page 7]: The main difference in practice is the widespread use of experimental software by unsuspecting scientists who believe it to be mature, whereas users of instrument prototypes are usually well aware of the experimental status of their equipment.

      • and Note [page 7]: Again this feels like an assertion without evidence. Is this an essay, or a research paper?
      Convivial vs. proprietary software

      Highlight [page 8]: Convivial software [Kell 2020], named in reference to Ivan Illich’s book “Tools for conviviality” [Illich 1973], is software that aims at augmenting its users’ agency over their computation

      • and Note [page 8]: It would be really helpful if the author would define the word, “convivial” here. It would also be very useful if they went on to give an example of what they meant by: “…software that aims at augmenting its users’ agency over their computation.” How does it augment the users agency?

      Highlight [page 8]: Shaw recently proposed the less pejorative term vernacular developers [Shaw 2022]

      • and Note [page 8]: Could you provide an example of what makes “vernacular developers” different, or just what they mean by this term?

      Highlight [page 8]: which Illich has described in detail

      • and Note [page 8]: Should this have a citation to Illich then in this sentence?

      Highlight [page 8]: what has happened with computing technology for the general public

      • and Note [page 8]: Can you give an example of this. Do you mean the rise of Apple and Windows? MS Word? Facebook? A couple of examples would be really useful to make this point clear.

      Highlight [page 8]: tech corporations

      • and Note [page 8]: Suggest “tech corporations” be “technology corporations”.

      Highlight [page 8]: Some research communities have fallen into this trap as well, by adopting proprietary tools such as MATLAB as a foundation for their computational tools and models.

      • and Note [page 8]: Can you provide an example of the alternative here, what would be the way to avoid this trap - use software such as Octave, or?

      Highlight [page 8]: Historically, the Free Software movement was born in a universe of convivial technology.

      • and Note [page 8]: If it is historic, can you please provide a reference to this?

      Highlight [page 8]: most of the software they produced and used was placed in the public domain

      • and Note [page 8]: Can you provide an example of this? I’m also curious how the software was placed in the public domain if there was no way to distribute it via the internet.

      Highlight [page 8]: as they saw legal constraints as the main obstacle to preserving conviviality

      • and Note [page 8]: Again, these are conjectures that are lacking a reference or example, can you provide some examples of references of this?

      Highlight [page 9]: Software complexity has led to a creeping loss of user agency, to the point that even building and installing Open Source software from its source code is often no longer accessible to non-experts, making them dependent not only on the development communities, but also on packaging experts. An experience report on building the popular machine learning library PyTorch from source code nicely illustrates this point [Courtès 2021].

      • and Note [page 9]: Can you summarise what makes it difficult to install Open Source Software? Again, this statement feels like it is making a strong generalisation without clear evidence to support this. The article by Courtès (https://hpc.guix.info/blog/2021/09/whats-in-a-package/), actually notes that it’s straightforward to install PyTorch via pip, but using an alternative package manager causes difficulty. The point you are making here seems to be that building and installing most open source software is almost prohibitive, but I think you’ve given strong evidence for this claim, and I don’t understand how this builds into your overall argument.

      Highlight [page 9]: It survives mainly in communities whose technology has its roots in the 1980s, such as programming systems inheriting from Smalltalk (e.g. Squeak, Pharo, and Cuis), or the programmable text editor GNU Emacs.

      • and Note [page 9]: Can you give an example of how it survives in these communities?

      Highlight [page 9]: FLOSS has been rapidly gaining in popularity, and receives strong support from the Open Science movement

      • and Note [page 9]: Can you provide some evidence to back this statement up?

      Highlight [page 9]: the traditional values of scientific research.

      • and Note [page 9]: Can you state what you mean by “traditional values of scientific research”

      Highlight [page 9]: always been convivial

      • and Note [page 9]: Can you provide a further explanation of what makes them convivial?
      Transparent vs. opaque software

      Highlight [page 9]: Transparent software

      • and Note [page 9]: It might be useful to explain a distinction between transparent and open software - or to perhaps open with a statement for why we are talking about transparent and opaque software.

      Highlight [page 9]: Large language models are an extreme example.

      • and Note [page 9]: Based on your definition of transparent software - every action produces a visible result. If I type something into an LLM and get an immediate and visible result, how is this different? It is possible you are stating that the behaviour is able to be easily interpreted, or perhaps the behaviour is easy to understand?

      Highlight [page 10]: Even highly interactive software, for example in data analysis, performs nonobvious computations, yielding output that an experienced user can perhaps judge for plausibility, but not for correctness.

      • and Note [page 10]: Could you give a small example of this?

      Highlight [page 10]: It is much easier to develop trust in transparent than in opaque software.

      • and Note [page 10]: Can you state why it is easier to develop this trust?

      Highlight [page 10]: but also less important

      • and Note [page 10]: Can you state why it is less important?

      Highlight [page 10]: even a very weak trustworthiness indicator such as popularity becomes sufficient

      • and Note [page 10]: becomes sufficient for what? Reviewing? Why does it become sufficient?

      Highlight [page 10]: This is currently a much discussed issue with machine learning models,

      • and Note [page 10]: Given it is currently much discussed, could you link to at least 2 research articles discussing this point?

      Highlight [page 10]: treated extensively in the philosophy of science.

      • and Note [page 10]: Given that is has been treated extensively, can you please provide some key references after this statement? You do go on to cite one paper, but it would be helpful to mention at least a few key articles.
      Size of the minimal execution environment

      Highlight [page 11]: The importance of this execution environment is not sufficiently appreciated by most researchers today, who tend to consider it a technical detail

      • and Note [page 11]: This statement is a bit of a sweeping generalisation - why is it not sufficiently appreciated? What evidence do you have of this?

      Highlight [page 11]: Software environments have only recently been recognized as highly relevant for automated reasoning in science and beyond

      • and Note [page 11]: Where have they been only recently recognised?

      Highlight [page 11]: However, they have not yet found their way into mainstream computational science.

      • and Note [page 11]: Could you provide an example of what it might look like if they were in mainstream computational science? For example, https://github.com/ropensci/rix implements using reproducible environments for R with NIX. What makes this not mainstream? Are you talking about mainstream in the sense of MS Excel? SPSS/SAS/STATA?
      Analogies in experimental and theoretical science

      Highlight [page 12]: Non-industrial components are occasionally made for special needs, but this is discouraged by their high manufacturing cost

      • and Note [page 12]: Can you provide an example of this?

      Highlight [page 12]: cables

      • and Note [page 12]: What do you mean by a cable? As in a computer cable? An electricity cable?

      Highlight [page 13]: which an experienced microscopist will recognize. Software with a small defect, on the other hand, can introduce unpredictable errors in both kind and magnitude, which neither a domain expert nor a professional programmer or computer scientist can diagnose easily.

      • and Note [page 13]: I don’t think this is a fair comparison. Surely there must be instances of experiences microscopists not identifying defects? Similarly, why can’t there be examples of domain expert or professional programmer/computer scientist identifying errors. Don’t unit tests help protect us against some of our errors? Granted, they aren’t bullet proof, and perhaps act more like guard rails.

      Highlight [page 13]: where “traditional” means not relying on any form of automated reasoning.

      • and Note [page 13]: Can you give an example of what a “traditional” scientific model or theory
      Improving the reviewability of automated reasoning systems

      Highlight [page 14]: Figure 2: Four measures that can be taken to make scientific software more trustworthy.

      • and Note [page 14]: Could the author perhaps instead call these “four measures” or perhaps give them a better name, and number them?
      Review the reviewable

      Highlight [page 14]: mature wide-spectrum software

      • and Note [page 14]: Can you give an example of what “mature wide-spectrum software” is?

      Highlight [page 15]: The main difficulty in achieving such audits is that none of today’s scientific institutions consider them part of their mission.

      Science vs. the software industry

      Highlight [page 15]: Many computers, operating systems, and compilers were designed specifically for the needs of scientists.

      • and Note [page 15]: Could you give an example of this? E.g., FORTRAN? COBAL?

      Highlight [page 15]: Today, scientists use mostly commodity hardware

      • and Note [page 15]: Can you explain what you mean by “commodity hardware”, and give an example.

      Highlight [page 15]: even considered advantageous if it also creates a barrier to reverse- engineering of the software by competitors

      • and Note [page 15]: Can you give an example of this?

      Highlight [page 15]: few customers (e.g. banks, or medical equipment manufacturers) are willing to pay for

      • and Note [page 15]: What about software like SPSS/STATA/SAS - surely many many industries, and also researchers will pay for software like this that is considered mature?
      Emphasize situated and convivial software

      Highlight [page 16]: a convivial collection of more situated modules, possibly supported by a shared wide-spectrum layer.

      • and Note [page 16]: Could you give an example of what this might look like practically? Are you saying things like SciPy would be restructured into many separate modules, or?

      Highlight [page 16]: In terms of FLOSS jargon, users make a partial fork of the project. Version control systems ensure provenance tracking and support the discovery of other forks. Keeping up to date with relevant forks of one’s software, and with the motivations for them, is part of everyday research work at the same level as keeping up to date with publications in one’s wider community. In fact, another way to describe this approach is full integration of scientific software development into established research practices, rather than keeping it a distinct activity governed by different rules.

      • and Note [page 16]: Could the author provide a diagram or schematic to more clearly show how such a system would work with forks etc?

      Highlight [page 17]: a universe is very

      • and Note [page 17]: Perhaps this could be “would be very different” - since this doesn’t yet exist, right?

      Highlight [page 17]: Improvement thus happens by small-step evolution rather than by large-scale design. While this may look strange to anyone used to today’s software development practices, it is very similar to how scientific models and theories have evolved in the pre-digital era.

      • and Note [page 17]: I think some kind of schematic or workflow to compare existing practices to this new practice would be really useful to articulate these points. I also think this new method of development you are proposing should have a concrete name.

      Highlight [page 17]: Existing code refactoring tools can probably be adapted to support application-specific forks, for example via code specialization. But tools for working with the forks, i.e. discovering, exploring, and comparing code from multiple forks, are so far lacking. The ideal toolbox should support both forking and merging, where merging refers to creating consensual code versions from multiple forks. Such maintenance by consensus would probably be much slower than maintenance performed by a coordinated team.

      • and Note [page 17]: Perhaps an example of screenshot of a diff could be used to demonstrate that we can make these changes between two branches/commits, but comparing multiple is challenging?
      Make scientific software explainable

      Highlight [page 18]: An interesting line of research in software engineering is exploring possibilities to make complete software systems explainable [Nierstrasz and Girba 2022]. Although motivated by situated business applications, the basic ideas should be transferable to scientific computing

      • and Note [page 18]: Is this similar to concepts such as “X-AI” or “X-ML” - that is, “Explainable” Artificial Intelligence or Machine Learning?

      Highlight [page 18]: Unlike traditional notebooks, Glamorous Toolkit [feenk.com 2023],

      • and Note [page 18]: It appears that you have introduced “Glamorous Toolkit” as an example of these three principles? It feels like it should be introduced earlier in this paragraph?

      Highlight [page 18]: In Glamorous Toolkit, whenever you look at some code, you can access corresponding examples (and also other references to the code) with a few mouse clicks

      • and Note [page 18]: I think it would be very beneficial to show screenshots of what the author means - while I can follow the link to Glamorous Toolkit, bitrot is a thing, and that might go away, so it would good to see exactly what the author means when they discuss these examples.
      Use Digital Scientific Notations

      Highlight [page 18]: There are various techniques for ensuring or verifying that a piece of software conforms to a formal specification

      • and Note [page 18]: Can you give an example of these techniques?

      Highlight [page 18]: The use of these tools is, for now, reserved to software that is critical for safety or security,

      • and Note [page 18]: Again, could you give an example of this point? Which tools, and which software is critical for safety or security?

      Highlight [page 19]: formal specifications

      • and Note [page 19]: It would be really helpful if you could demonstrate an example of a formal specification so we can understand how they could be considered constraints.

      Highlight [page 19]: All of them are much more elaborate than the specification of the result they produce. They are also rather opaque.

      • and Note [page 19]: It isn’t clear to me how these are opaque - if the algorithm is defined, it can be understood, how is it opaque?

      Highlight [page 19]: Moreover, specifications are usually more modular than algorithms, which also helps human readers to better understand what the software does [Hinsen 2023]

      • and Note [page 19]: A tight example of this would be really useful to make this point clear. Perhaps with a figure of a specification alongside an algorithm.

      Highlight [page 19]: In software engineering, specifications are written to formalize the expected behavior of the software before it is written. The software is considered correct if it conforms to the specification.

      • and Note [page 19]: Is an example of this test drive development?

      Highlight [page 19]: A formal specification has to evolve in the same way, and is best seen as the formalization of the scientific knowledge. Change can flow from specification to software, but also in the opposite direction.

      • and Note [page 19]: Again, I think a good figure here would be very helpful in articulating this clearly.

      Highlight [page 19]: My own experimental Digital Scientific Notation, Leibniz [Hinsen 2024], is intended to resemble traditional mathematical notation as used e.g. in physics. Its statements are embeddable into a narrative, such as a journal article, and it intentionally lacks typical programming language features such as scopes that do not exist in natural language, nor in mathematical notation.

      • and Note [page 19]: Could we see an example of what this might look like?
      Conclusion

      Highlight [page 20]: Situated software is easy to recognize.

      • and Note [page 20]: Could you provide some examples?

      Highlight [page 20]: Examples from the reproducibility crisis support this view

      • and Note [page 20]: Can you provide some example papers that you mention here?

      Highlight [page 21]: The ideal structure for a reliable scientific software stack would thus consist of a foundation of mature software, on top of which a transparent layer of situated software, such as a script, a notebook, or a workflow, orchestrates the computations that together answer a specific scientific question. Both layers of such a stack are reviewable, as I have explained in section 3.1, but adequate reviewing processes remain to be enacted.

      • and Note [page 21]: Again, I think it would be very insightful for the reader to have a clear figure to rest these ideas upon.

      Highlight [page 21]: has been neglected by research institutions all around the world

      • and Note [page 21]: I do not think this is true - could you instead say “neglected my most/many” perhaps?
    2. Dear editors and reviewers, Thank you for your careful reading of my manuscript and the detailed and insightful feedback. It has contributed significantly to the improvements in the revised version. Please find my detailed responses below.

      1 Reviewer 1

      Thank you for this helpful review, and in particular for pointing out the need for more references, illustrations, and examples in various places of my manuscript. In the case of the section on experimental software, the search for examples made clear to me that the label was in fact badly chosen. I have relabeled the dimension as “stable vs. evolving software”, and rewritten the section almost entirely. Another major change motivated by your feedback is the addition of a figure showing the structure of a typical scientific software stack (Fig. 2), and of three case studies (section 2.7) in which I evaluate scientific software packages according to my five dimensions of reviewability. The discussion of conviviality (section 2.4), a concept that is indeed not widely known yet, has been much expanded. I have followed the advice to add references in many places. I have been more hesitant to follow the requests for additional examples and illustrations, because of the inevitable conflict with the equally understandable request to make the paper more compact. In many cases, I have preferred to refer to examples discussed in the literature. A few comments deserve a more detailed reply:

      Introduction

      Highlight [page 3]: In fact, we do not even have established processes for performing such reviews

      and Note [page 3]: I disagree, there is the Journal of Open Source Software: https://joss.theoj.org/, rOpenSci has a guide for development of peer review of statistical software: https://github.com/ropensci/statistical software-review-book, and also maintain a very clear process of software review: https://ropensci.org/software-review/

      As I say in the section “Review the reviewable”, these reviews are not independent critical examination of the software as I define it. Reviewers are not asked to evaluate the software’s correctness or appropriateness for any specific purpose. They are expected to comment only on formal characteristics of the software publication process (e.g. “is there a license?”), and on a few software engineering quality indicators (“is there a test suite?”).

      Highlight [page 3]: This means that reviewing the use of scientific software requires particular attention to potential mismatches between the software’s behavior and its users’ expectations, in particular concerning edge cases and tacit assumptions made by the software developers. They are necessarily expressed somewhere in the software’s source code, but users are often not aware of them.

      and Note [page 3]: The same can be said of assumptions for equations and mathematics- the problem here is dealing with abstraction of complexity and the potential unintended consequences.

      Indeed. That’s why we need someone other than the authors to go through mathematical reasoning and verify it. Which we do.

      Reviewability of automated reasoning systems

      Wide-spectrum vs. situated software

      Highlight [page 6]: Situated software is smaller and simpler, which makes it easier to understand and thus to review.

      and Note [page 6]: I’m not sure I agree it is always smaller and simpler- the custom code for a new method could be incredibly complicated.

      The comparison is between situated software and more generic software performing the same operation. For example, a script reading one specific CSV file compared to a subroutine reading arbitrary CSV files. I have yet to see a case in which abstraction from a concrete to a generic function makes code smaller or simpler.

      Convivial vs. proprietary software

      Highlight [page 8]: most of the software they produced and used was placed in the public domain

      and Note [page 8]: Can you provide an example of this? I’m also curious how the software was placed in the public domain if there was no way to distribute it via the internet.

      Software distribution in science was well organized long before the Internet, it was just slower and more expensive. Both decks of punched cards and magnetic tapes were routinely sent by mail. The earliest organized software distribution for science I am aware of was the DECUS Software Library in the early 1960s.

      Size of the minimal execution environment

      Note [page 11]: Could you provide an example of what it might look like if they were in mainstream computational science? For example, https://github.com/ropensci/rix implements using reproducible environments for R with NIX. What makes this not mainstream? Are you talking about mainstream in the sense of MS Excel? SPSS/SAS/STATA?

      I have looked for quantitative studies on software use in science that would allow to give a precise meaning to “mainstream”, but I have not been able to find any. Based on my personal experience, mostly with teaching MOOCs on computational science in which students are asked about the software they use, the most widely used platform is Microsoft Windows. Linux is already a minority platform (though overrepresented in computer science), and Nix users are again a small minority among Linux users.

      Analogies in experimental and theoretical science

      Highlight [page 13]: which an experienced microscopist will recognize. Soft ware with a small defect, on the other hand, can introduce unpredictable errors in both kind and magnitude, which neither a domain expert nor a professional programmer or computer scientist can diag- nose easily.

      and Note [page 13]: I don’t think this is a fair comparison. Surely there must be instances of experiences microscopists not identifying defects? Similarly, why can’t there be examples of domain expert or professional program mer/computer scientist identifying errors. Don’t unit tests help protect us against some of our errors? Granted, they aren’t bullet proof, and perhaps act more like guard rails.

      There are probably cases of microscopists not noticing defects, but my point is that if you ask them to look for defects, they know what to do (and I have made this clearer in my text). For contrast, take GROMACS (one of my case studies in the revised manuscript) and ask either an expert programmer or an experienced computational biophysicist if it correctly implements, say, the AMBER force field. They wouldn’t know what to do to answer that question, both because it is ill-defined (there is no precise definition of the AMBER force field) and because the number of possible mistakes and symptoms of mistakes is enormous. I have seen a protein simulation program fail for proteins whose number of atoms was in a narrow interval, defined by the size that a compiler attributed to a specific data structure. I was able to catch and track down this failure only because a result was obviously wrong for my use case. I have never heard of similar issues with microscopes.

      Improving the reviewability of automated reasoning systems

      Review the reviewable

      Highlight [page 15]: The main difficulty in achieving such audits is that none of today’s scientific institutions consider them part of their mission.

      and Note [page 15]: I disagree. Monash provides an example here where they view software as a first class research output: https://robjhyndman.com/files/EBS_research_software.pdf

      This example is about superficial reviews in the context of career evaluation. Other institutions have similar processes. As far as I know, none of them ask reviewers to look at the actual code and comment on its correctness or its suitability for some specific purpose.

      Science vs. the software industry

      Highlight [page 15]: few customers (e.g. banks, or medical equipment manufacturers) are willing to pay for

      and Note [page 15]: What about software like SPSS/STATA/SAS- surely many many industries, and also researchers will pay for software like this that is considered mature?

      I could indeed extend the list of examples to include various industries. Compared to the huge number of individuals using PCs and smartphones, that’s still few customers.

      Emphasize situated and convivial software

      Note [page 16]: Could the author provide a diagram or schematic to more clearly show how such a system would work with forks etc?

      I have decided the contrary: I have significantly shortened this section, removing all speculation about how the ideas could be turned into concrete technology. The reason is that I have been working on this topic since I wrote the reviewed version of this manuscript, and I have a lot more to say about it than would be reasonable to include in this work. This will become a separate article.

      Make scientific software explainable

      Note [page 18]: I think it would be very beneficial to show screenshots of what the author means- while I can follow the link to Glamorous Toolkit, bitrot is a thing, and that might go away, so it would good to see exactly what the author means when they discuss these examples.

      Unfortunately, static screenshots can only convey a limited impression of Glamorous Toolkit, but I agree that they have are a more stable support than the software itself. Rather than adding my own screenshots, I refer to a recent paper by the authors of Glamorous Toolkit that includes many screenshots for illustration.

      Use Digital Scientific Notations

      Highlight [page 19]: formal specifications and Note [page 19]: It would be really helpful if you could demonstrate an example of a formal specification so we can understand how they could be considered constraints.

      Highlight [page 19]: Moreover, specifications are usually more modular than algorithms, which also helps human readers to better understand what the software does [Hinsen 2023]

      and Note [page 19]: A tight example of this would be really useful to make this point clear. Perhaps with a figure of a specification alongside an algorithm.

      I do give an example: sorting a list. To write down an actual formalized version, I’d have to introduce a formal specification language and explain it, which I think goes well beyond the scope of this article. Illustrating modularity requires an even larger example. This is, however, an interesting challenge which I’d be happy to take up in a future article.

      Highlight [page 19]: In software engineering, specifications are written to formalize the expected behavior of the software before it is written. The software is considered correct if it conforms to the specification.

      and Note [page 19]: Is an example of this test drive development?

      Not exactly, though the underlying idea is similar: provide a condition that a result must satisfy as evidence for being correct. With testing, the condition is spelt out for one specific input. In a formal specification, the condition is written down for all possible inputs.

      2 Reviewer 2

      First of all, I would like to thank the reviewer for this thoughtful review. It addresses many points that require clarifications in the my article, which I hope to have done adequately in the revised version.

      One such point is the role and form of reviewing processes for software. I have made it clearer that I take “review” to mean “critical independent inspection”. It could be performed by the user of a piece of software, but the standard case should be a review performed by experts at the request of some institution that then publishes the reviewer’s findings. There is no notion of gatekeeping attached to such reviews. Users are free to ignore them. Given that today, we publish and use scientific software without any review at all, the risk of shifting to the opposite extreme of having reviewers become gatekeepers seems unlikely to me.

      Your comment on users being software developers addresses another important point that I had failed to make clear: conviviality is all about diminishing the distinction between developers and users. Users gain agency over their computations at the price of taking on more of a developer role. This is now stated explicitly in the revised article. Your hypothesis that I want scientific software to be convivial is only partially true. I want convivially structured software to be an option for scientists, with adequate infrastructure and tooling support, but I do not consider it to be the best approach for all scientific software.

      The paragraph on the relevance and importance of reviewing in your comment is a valid point of view but, unsurprisingly, not mine. In the grand scheme of science, no specific quality assurance measure is strictly necessary. There is always another layer above that will catch mistakes that weren’t detected in the layer below. It is thus unlikely that unreliable software will cause all of science to crumble. But from many perspectives, including overall efficiency, personal satisfaction of practitioners, and insight derived from the process, it is preferable to catch mistakes as closely as possible to their source. Pre-digital theoreticians have always double-checked their manual calculations before submitting their papers, rather than sending off unchecked results and count on confrontation with experiment for finding mistakes. I believe that we should follow this same approach with software. The cost of mistakes can be quite high. Consider the story of the five retracted protein structures that I cite in my article (Miller, 2006, 10.1126/science.314.5807.1856). The five publications that were retracted involved years of work by researchers, reviewers, and editors. In between their publication and their retraction, other protein crystallographers saw their work rejected because it was in contradiction with the high-profile articles that later turned out to be wrong. The whole story has probably involved a few ruined careers in addition to its monetary cost. In contrast, independent critical examination of the software and the research processes in which it was used would likely have spotted the problem rather quickly (Matthews, 2007).

      You point out that reviewability is also a criterion in choosing software to build on, and I agree. Building on other people’s software requires trusting it. Incorporating it into one’s own work (the core principle of convivial software) requires understanding it. This is in fact what motivated my reflections on this topic. I am not much interested in neatly separating epistemic and practical issues. I am a practitioner, my interest in epistemology comes from a desire for improving practices.

      Review holism is something I have not thought about before. I consider it both impossible to apply in practice and of little practical value. What I am suggesting, and I hope to have made this clearer in my revision, is that reviewing must take into account the dependency graph. Reviewing software X requires a prior review of its dependencies (possibly already done by someone else), and a consideration of how each dependency influences the software under consideration. However, I do not consider Donoho’s “frictionless reproducibility” a sufficient basis for trust. It has the same problem as the widespread practice of tacitly assuming a piece of software to be correct because it is widely used. This reasoning is valid only if mistakes have a high chance of being noticed, and that’s in my experience not true for many kinds of research software. “It works”, when pronounced by a computational scientist, really means “There is no evidence that it doesn’t work”.

      This is also why I point out the chaotic nature of computation. It is not about Humphreys’ “strange errors”, for which I have no solution to offer. It is about the fact that looking for mistakes requires some prior idea of what the symptoms of a mistake might be. Experienced researchers do have such prior ideas for scientific instruments, and also e.g. for numerical algorithms. They come from an understanding of the instruments and their use, including in particular a knowledge of how they can go wrong. But once your substrate is a Turing-complete language, no such understanding is possible any more. Every programmer has made the experience of chasing down some bug that at first sight seems impossible. My long-term hope is that scientific computing will move towards domain-specific languages that are explicitly not Turing-complete, and offer useful guarantees in exchange. Unfortunately, I am not aware of any research in this space.

      I fully agree with you that internalist justifications are preferable to reliabilistic ones. But being fundamentally a pragmatist, I don’t care much about that distinction. Indisputable justification doesn’t really exist anywhere in science. I am fine with trust that has a solid basis, even if there remains a chance of failure. I’d already be happy if every researcher could answer the question “why do you trust your computational results?” in a way that shows signs of critical reflection.

      What I care about ultimately is improving practices in computational science. Over the last 30 years, I have seen numerous mistakes being discovered by chance, often leading to abandoned research projects. Some of these mistakes were due to software bugs, but the most common cause was an incorrect mental model of what the software does. I believe that the best technique we have found so far to spot mistakes in science is critical independent inspection. That’s why I am hoping to see it applied more widely to computation.

      2.1 References

      Miller, G. (2006) A Scientist’s Nightmare: Software Problem Leads to Five Retractions. Science 314, 1856. https://doi.org/10.1126/science.314.5807.1856

      Matthews, B.W. (2007) Five retracted structure reports: Inverted or incorrect? Protein Science 16, 1013. https://doi.org/10.1110/ps.072888607

      3 Editor

      Bayesian methods often use MCMC, which is often slow and creates long chains of estimates; however, the chains will show if the likelihood does not have a clear maximum, which is usually from a badly specified model...

      That is an interesting observation I haven’t seen mentioned bedore. I agree that Bayesian inference is particularly amenable to inspection. One more reason to normalize inspection and inspectability in computational science.

      Some reflection on the growing use of AI to write software may be worthwhile.

      The use of AI in writing and reviewing software is a topic I have considered for this review, since the technology has evolved enormously since I wrote the current version of the manuscript. However, in view of reviewer 1’s constant admonition to back up statements with citations, I refrained from delving into this topic. We all know it’s happening, but it’s too early to observe a clear impact on research software. I have therefore limited myself to a short comment in the Conclusion section.

      I wondered if highly-used software should get more scrutiny.

      This is an interesting suggestion. If and when we get serious about reviewing code, resource allocation will become an important topic. For getting started, it’s probably more productive to review newly published code than heavily used code, because there is a better chance that authors actually act on the feedback and improve their code before it has many users. That in turn will help improve the reviewing process, which is what matters most right now, in my opinion.

      “supercomputers are rare”, should this be “relatively rare” or am I speaking from a privileged university where I’ve always had access to supercomputers.

      If you have easy access to supercomputer, you should indeed consider yourself privileged. But did you ever use supercomputer time for reviewing someone else’s work? I have relatively easy access to supercomputers as well, but I do have to make a re quest and promise to do innovative research with the allocated resources.

      I did think about “testthat” at multiple points whilst reading the paper (https://testthat.r-lib.org/)

      I hadn’t seen “testthat” before, not being much of a user of R. It looks interesting, and reminds me of similar test support features in Smalltalk which I found very helpful. Improving testing culture is definitely a valuable contribution to improving computational practices.

      Can badges on github about downloads and maturity help (page 7)?

      Badges can help, on GitHub or elsewhere, e.g. in scientific software catalogs. I see them as a coarse-grained output of reviewing. The right balance to find is between the visibility of a badge and the precision of a carefully written review report. One risk with badges is the temptation to automate the evaluation that leads to it. This is fine for quantitative measures such as test coverage, but what we mostly lack today is human expert judgement on software.

    1. Reviewer #1 (Public review):

      This paper describes a number of patterns of epistasis in a large fitness landscape dataset recently published by Papkou et al. The paper is motivated by an important goal in the field of evolutionary biology to understand the statistical structure of epistasis in protein fitness landscapes, and it capitalizes on the unique opportunities presented by this new dataset to address this problem.

      The paper reports some interesting previously unobserved patterns that may have implications for our understanding of fitness landscapes and protein evolution. In particular, Figure 5 is very intriguing. However, I have two major concerns detailed below. First, I found the paper rather descriptive (it makes little attempt to gain deeper insights into the origins of the observed patterns) and unfocused (it reports what appears to be a disjointed collection of various statistics without a clear narrative. Second, I have concerns with the statistical rigor of the work.

      (1) I think Figures 5 and 7 are the main, most interesting, and novel results of the paper. However, I don't think that the statement "Only a small fraction of mutations exhibit global epistasis" accurately describes what we see in Figure 5. To me, the most striking feature of this figure is that the effects of most mutations at all sites appear to be a mixture of three patterns. The most interesting pattern noted by the authors is of course the "strong" global epistasis, i.e., when the effect of a mutation is highly negatively correlated with the fitness of the background genotype. The second pattern is a "weak" global epistasis, where the correlation with background fitness is much weaker or non-existent. The third pattern is the vertically spread-out cluster at low-fitness backgrounds, i.e., a mutation has a wide range of mostly positive effects that are clearly not correlated with fitness. What is very interesting to me is that all background genotypes fall into these three groups with respect to almost every mutation, but the proportions of the three groups are different for different mutations. In contrast to the authors' statement, it seems to me that almost all mutations display strong global epistasis in at least a subset of backgrounds. A clear example is C>A mutation at site 3.

      1a. I think the authors ought to try to dissect these patterns and investigate them separately rather than lumping them all together and declaring that global epistasis is rare. For example, I would like to know whether those backgrounds in which mutations exhibit strong global epistasis are the same for all mutations or whether they are mutation- or perhaps position-specific. Both answers could be potentially very interesting, either pointing to some specific site-site interactions or, alternatively, suggesting that the statistical patterns are conserved despite variation in the underlying interactions.

      1b. Another rather remarkable feature of this plot is that the slopes of the strong global epistasis patterns seem to be very similar across mutations. Is this the case? Is there anything special about this slope? For example, does this slope simply reflect the fact that a given mutation becomes essentially lethal (i.e., produces the same minimal fitness) in a certain set of background genotypes?

      1c. Finally, how consistent are these patterns with some null expectations? Specifically, would one expect the same distribution of global epistasis slopes on an uncorrelated landscape? Are the pivot points unusually clustered relative to an expectation on an uncorrelated landscape?

      1d. The shapes of the DFE shown in Figure 7 are also quite interesting, particularly the bimodal nature of the DFE in high-fitness (HF) backgrounds. I think this bimodality must be a reflection of the clustering of mutation-background combinations mentioned above. I think the authors ought to draw this connection explicitly. Do all HF backgrounds have a bimodal DFE? What mutations occupy the "moving" peak?

      1e. In several figures, the authors compare the patterns for HF and low-fitness (LF) genotypes. In some cases, there are some stark differences between these two groups, most notably in the shape of the DFE (Figure 7B, C). But there is no discussion about what could underlie these differences. Why are the statistics of epistasis different for HF and LF genotypes? Can the authors at least speculate about possible reasons? Why do HF and LF genotypes have qualitatively different DFEs? I actually don't quite understand why the transition between bimodal DFE in Figure 7B and unimodal DFE in Figure 7C is so abrupt. Is there something biologically special about the threshold that separates LF and HF genotypes? My understanding was that this was just a statistical cutoff. Perhaps the authors can plot the DFEs for all backgrounds on the same plot and just draw a line that separates HF and LF backgrounds so that the reader can better see whether the DFE shape changes gradually or abruptly.

      1f. The analysis of the synonymous mutations is also interesting. However I think a few additional analyses are necessary to clarify what is happening here. I would like to know the extent to which synonymous mutations are more often neutral compared to non-synonymous ones. Then, synonymous pairs interact in the same way as non-synonymous pair (i.e., plot Figure 1 for synonymous pairs)? Do synonymous or non-synonymous mutations that are neutral exhibit less epistasis than non-neutral ones? Finally, do non-synonymous mutations alter epistasis among other mutations more often than synonymous mutations do? What about synonymous-neutral versus synonymous-non-neutral. Basically, I'd like to understand the extent to which a mutation that is neutral in a given background is more or less likely to alter epistasis between other mutations than a non-neutral mutation in the same background.

      (2) I have two related methodological concerns. First, in several analyses, the authors employ thresholds that appear to be arbitrary. And second, I did not see any account of measurement errors. For example, the authors chose the 0.05 threshold to distinguish between epistasis and no epistasis, but why this particular threshold was chosen is not justified. Another example: is whether the product s12 × (s1 + s2) is greater or smaller than zero for any given mutation is uncertain due to measurement errors. Presumably, how to classify each pair of mutations should depend on the precision with which the fitness of mutants is measured. These thresholds could well be different across mutants. We know, for example, that low-fitness mutants typically have noisier fitness estimates than high-fitness mutants. I think the authors should use a statistically rigorous procedure to categorize mutations and their epistatic interactions. I think it is very important to address this issue. I got very concerned about it when I saw on LL 383-388 that synonymous stop codon mutations appear to modulate epistasis among other mutations. This seems very strange to me and makes me quite worried that this is a result of noise in LF genotypes.

    1. Author response:

      The following is the authors’ response to the original reviews.

      Reviewer #1 (Public Review):

      Summary: 

      The idea is appealing, but the authors have not sufficiently demonstrated the utility of this approach.

      Strengths: 

      Novelty of the approach, potential impli=cations for discovering novel interactions

      Weaknesses:

      The Duong had introduced their highly elegant peptidisc approach several years ago. In this present work, they combine it with thermal proteome profiling (TPP) and attempt to demonstrate the utility of this combination for identifying novel membrane protein-ligand interactions.

      While I find this idea intriguing, and the approach potentially useful, I do not feel that the authors had sufficiently demonstrated the utility of this approach. My main concern is that no novel interactions are identified and validated. For the presentation of any new methodology, I think this is quite necessary. In addition, except for MsbA, no orthogonal methods are used to support the conclusions, and the authors rely entirely on quantifying rather small differences in abundances using either iBAQ or LFQ.

      We thank the reviewer for their thoughtful comments. In this revision, we have experimentally addressed the reviewer’s concerns in three ways:

      (1) To demonstrate the utility of our MM-TPP method over the detergent-based TPP workflow (termed DB-TPP), we performed a side-by-side comparison using ATP–VO₄ at 51 °C (Figure 3B and Figure 4A). From the DB-TPP dataset, 7.4% of all identified proteins were annotated as ATP-binding, while 6.4% of proteins differentially stabilized were annotated as ATP-binding. In contrast, in the MM-TPP dataset, 9.3% of all identified proteins were annotated as ATP-binding proteins, while 17% of proteins differentially stabilized were annotated as ATP-binding. The lack of enrichment in the detergent-based approach indicates that the observed differences are likely stochastic, rather than a result of specific ATP–VO₄-mediated stabilization as found with MM-TPP. For instance, several key proteins—BCS1, P2RY6, SLC27A2, ABCB1, ABCC2, and ABCC9— found differentially stabilized using the MM-TPP method showed no such pattern in the DB-TPP dataset. This divergence strongly supports the specificity and utility of our Peptidisc approach. 

      (2) To demonstrate that MM-TPP can resolve not only the broader effects of ATP–VO₄ but also specific ligand–protein interactions, we employed 2-methylthio-ADP (2-MeS-ADP), a selective agonist of the P2RY12 receptor [PMID: 24784220]. In that case, we observed clear thermal stabilization of P2RY12, with more than 6-fold increase in stability at both 51 °C and 57 °C (–log₁₀ p > 5.97; Figure 4B and Figure S4). Notably, no other proteins—including the structurally related but non-responsive P2RY6 receptor- showed comparable stabilization fold change at these temperatures.

      (3) To further probe the reproducibility of the method, we performed an independent MMTPP evaluation with ATP–VO₄ at 51 °C using data-independent acquisition (DIA), in contrast to the data-dependent acquisition (DDA) approach used in the initial study (Figure S5). Overall, 7.8% of all identified proteins were annotated as ATP-binding, and as before, this proportion increased to 17% among proteins with log₂ fold changes greater than 0.5. Specifically, BCS1 and SLC27A2 exhibited strong stabilization (log₂ fold change > 1), while P2RY6, ABCB11, ABCC2, and ABCG2 showed moderate stabilization (log₂ fold changes between 0.5 and 1), and consistent with previous results, P2RX4 was destabilized, with a log₂ fold change below –1. These findings support the consistency and reproducibility of the method across distinct data acquisition methods.

      My main concern is that no novel interactions are identified and validated. For the presentation of any new methodology, I think this is quite necessary.  

      The primary objective of our study is to establish and benchmark the MM-TPP workflow using known targets, rather than to discover novel ligand–protein interactions. Identifying new binders requires extensive screening and downstream validations, which we believe is beyond the scope of this methodological report. Instead, our study highlights the sensitivity and reliability of the MM-TPP approach by demonstrating consistent and reproducible results with well-characterized interactions.

      We respectfully disagree with the notion that introducing a new methodology must necessarily include the discovery of novel interactions. For instance, Martinez Molina et al. [PMID: 23828940] introduced the cellular thermal shift assay (CETSA) by validating established targets such as MetAP2 with TNP-470 and CDK2 with AZD-5438, without identifying novel protein–ligand pairs. Similarly, Kalxdorf et al. [PMID: 33398190] published their cell-surface thermal proteome profiling (CS-TPP) using Ouabain to stabilize the Na⁺/K⁺-ATPase pump in K562 cells, and SB431542 to stabilize its canonical target JAG1. In fact, when these methods revealed additional stabilizations, these were not validated but instead interpreted through reasoning grounded in the literature. For instance, they attributed the SB431542-induced stabilization of MCT1 to its reported role in cell migration and tumor invasiveness, and explained that SLC1A2 stabilization is related to the disruption of Na⁺/K⁺-ATPase activity by Ouabain. In the same way, our interpretation of ATP-VO₄–mediated stabilization of Mao-B is justified by predictive AlphaFold-3 rather than direct orthogonal assays, which are beyond the scope of our methodological presentation. 

      Collectively, the influential studies cited above have set methodological precedents by prioritizing validation and proof-of-concept over merely finding uncharacterized binders. In the same spirit, our work is centred on establishing MM-TPP as a robust platform for probing membrane protein–ligand interactions in a water-soluble format. The discovery of novel binders remains an exciting future direction—one that will build upon the methodological foundation laid by the present study.

      In addition, except for MsbA, no orthogonal methods are used to support the conclusions, and the authors rely entirely on quantifying rather small differences in abundances using either iBAQ or LFQ.

      We deliberately began this study with our model protein, MsbA, examined under both native and overexpressed conditions, to establish an adequation between MMTPP (Figure 2D) and biochemical stability assays (Figure 2A). This validation has provided us with the foundation to confidently extend MM-TPP to the mouse organ proteome. To demonstrate the validity of our workflow, we have used ATP-VO₄ because it has expected targets. 

      We note that orthogonal validation often requires overproduction and purification of the candidate proteins, including suitable antibodies, which is a true challenge for membrane proteins. Here, we demonstrate that MM-TPP can detect ligand-induced thermal shifts directly in native membrane preparations, without requiring protein overproduction or purification. We also emphasize several influential studies in TPP, including Martinez Molina et al. (PMID: 23828940) and Fang et al. (PMID: 34188175), which focused primarily on establishing and benchmarking the methodology, rather than on extensive orthogonal validation. In the same spirit, our study prioritizes methodological development, and accordingly, several orthogonal validations are now included in this revision.

      [...] and the authors rely entirely on quantifying rather small differences in abundances using either iBAQ or LFQ.

      To clarify, all analyses on ligand-induced stabilization or destabilization were carried out using LFQ values. The sole exception is on Figure 2B, where we used iBAQ values to depict the relative abundance of proteins within a single sample; this to show MsbA's relative level within the E. coli peptidisc library.

      Respectfully, we disagree with the assertion that we are “quantifying rather small differences in abundances using either iBAQ or LFQ.” We were able to clearly distinguish between stabilizations driven by specific ligands binding to their targets versus those caused by non-specific ligands with broader activity. This is further confirmed by comparing 2-MeS-ADP, a selective ligand for P2RY12, with ATP-VO₄, a highly promiscuous ligand, and AMP-PNP, which exhibits intermediate breadth. When tested in triplicate at 51 °C, 2-MeS-ADP significantly altered the thermal stability of 27 proteins,  AMP-PNP 44 proteins, and ATP-VO₄ 230 proteins, consistent with the expectation that broader ligands stabilize more proteins nonspecifically. Importantly, 2-MeS-ADP produced markedly stronger stabilization of its intended target, P2RY12 (–log<sub>10</sub>p = 9.32), than the top stabilized proteins for ATP–VO₄ (DNAJB3, –log₁₀p = 5.87) or AMP-PNP (FTH1, p = 5.34). Moreover, 2-MeS-ADP did not significantly stabilize proteins that were consistently stabilized by the broad ligands, such as SLC27A2, which was strongly stabilized by both ATP-VO<sub>4</sub> and AMP-PNP (–log<sub>10</sub> p>2.5). Together, these findings demonstrate that MMTPP can robustly distinguish between broad-spectrum and target-specific ligands, with selective ligands inducing stronger and more physiologically meaningful stabilization at their intended targets compared to promiscuous ligands.

      Finally, we emphasize that our findings are not marginal, but meet quantitative and statistical rigor consistent with best practices in proteomics. We apply dual thresholds combining effect size (|log₂FC| ≥ 1, i.e., at least a two-fold change) with statistical significance (FDR-adjusted p ≤ 0.05)—criteria commonly used in proteomics methodology studies (e.g., PMID: 24942700, 38724498). Moreover, the stabilization and destabilization events we report are reproducible across biological replicates (n = 3), consistent across adjacent temperatures for most targets, and technically robust across acquisition modes (DDA vs. DIA). Taken together, these results reflect statistically valid and biologically meaningful effects, fully aligned with standards set by prior published proteomics studies.

      Furthermore, the reported changes in abundances are solely based on iBAQ or LFQ analysis. This must be supported by a more quantitative approach such as SILAC or labeled peptides. In summary, I think this story requires a stronger and broader demonstration of the ability of peptidisc-TPP to identify novel physiologically/pharmacologically relevant interactions.

      With respect to labeling strategies, we deliberately avoided using TMT due to concerns about both cost and potential data quality issues. Some recent studies have documented the drawbacks of TMT in contexts directly relevant to our work. For example, a benchmarking study of LiP-MS workflows showed that although TMT increased proteome depth and reduced technical variance, it was less accurate in identifying true drug–protein interactions and produced weaker dose–response correlations compared with label-free DIA approaches [PMID: 40089063]. More broadly, technical reviews have highlighted that isobaric tagging is intrinsically prone to ratio compression and reporterion interference due to co-isolation and co-fragmentation of peptides, which flatten measured fold-changes and obscure biologically meaningful differences [PMID: 22580419, 22036744]. In terms of SILAC, the technique requires metabolic incorporation of heavy amino acids, which is feasible in cultured cells but not in physiologically relevant tissues such as the liver organ used here. SILAC mouse models exist, but they are expensive and time-consuming [PMID: 18662549, 21909926]. We are not a mouse lab, and introducing liver organ SILAC labeling in our workflow is beyond the scope of these revisions. We also note that several hallmark TPP studies have been successfully carried out using label-free quantification [PMID: 25278616, 26379230, 33398190, 23828940], establishing this as an accepted and widely applied approach in the field. 

      To further support our conclusions, we added controls showing that detergent solubilization of mouse liver membranes followed by SP4 cleanup fails to detect ATP-VO₄– mediated stabilization of ATP-binding proteins, underscoring the necessity of Peptidisc reconstitution for capturing ligand-induced thermal stabilization. We also present new data demonstrating selective stabilization of the P2Y12 receptor by its agonist 2-MeS-ADP, providing orthogonal, receptor-specific validation within the MM-TPP framework. Finally, an orthogonal DIA acquisition on separate replicates confirmed robust ATP-vanadate stabilization of ATP-binding proteins, including BCS1l and SLC27A2. Together, these additions reinforce that the observed stabilizations are genuine, physiologically relevant ligand–protein interactions and highlight the unique advantage of the Peptidisc-based workflow in capturing such events.

      Cited Reference:

      24784220: Zhang J, Zhang K, Gao ZG, et al. Agonist-bound structure of the human P2Y₁₂ receptor. Nature.  2014;509(7498):119-122. doi:10.1038/nature13288. 

      23828940: Martinez Molina D, Jafari R, Ignatushchenko M, et al. Monitoring drug target engagement in cells and tissues using the cellular thermal shift assay. Science. 2013;341(6141):84-87. doi:10.1126/science.1233606.

      33398190: Kalxdorf M, Günthner I, Becher I, et al. Cell surface thermal proteome profiling tracks perturbations and drug targets on the plasma membrane. Nat Methods. 2021;18(1):84-91. doi:10.1038/s41592-020-01022-1.

      34188175: Fang S, Kirk PDW, Bantscheff M, Lilley KS, Crook OM. A Bayesian semi-parametric model for thermal proteome profiling. Commun Biol. 2021;4(1):810. doi:10.1038/s42003-021-02306-8.

      24942700: Cox J, Hein MY, Luber CA, Paron I, Nagaraj N, Mann M. Accurate proteome-wide label-free quantification by delayed normalization and maximal peptide ratio extraction, termed MaxLFQ. Mol Cell Proteomics. 2014;13(9):2513-2526. doi:10.1074/mcp.M113.031591.

      38724498: Peng H, Wang H, Kong W, Li J, Goh WWB. Optimizing differential expression analysis for proteomics data via high-performing rules and ensemble inference. Nat Commun. 2024;15(1):3922. doi:10.1038/s41467-02447899-w. 

      40089063: Koudelka T, Bassot C, Piazza I. Benchmarking of quantitative proteomics workflows for limited proteolysis mass spectrometry. Mol Cell Proteomics. 2025;24(4):100945. doi:10.1016/j.mcpro.2025.100945.

      22580419: Christoforou AL, Lilley KS. Isobaric tagging approaches in quantitative proteomics: the ups and downs. Anal Bioanal Chem. 2012;404(4):1029-1037. doi:10.1007/s00216-012-6012-9. 

      22036744: Christoforou AL, Lilley KS. Isobaric tagging approaches in quantitative proteomics: the ups and downs. Anal Bioanal Chem. 2012;404(4):1029-1037. doi:10.1007/s00216-012-6012-9. 

      18662549: Krüger M, Moser M, Ussar S, et al. SILAC mouse for quantitative proteomics uncovers kindlin-3 as an essential factor for red blood cell function. Cell. 2008;134(2):353-364. doi:10.1016/j.cell.2008.05.033.

      21909926: Zanivan S, Krueger M, Mann M. In vivo quantitative proteomics: the SILAC mouse. Methods Mol Biol. 2012;757:435-450. doi:10.1007/978-1-61779-166-6_25. 

      25278616: Kalxdorf M, Becher I, Savitski MM, et al. Temperature-dependent cellular protein stability enables highprecision proteomics profiling. Nat Methods. 2015;12(12):1147-1150. doi:10.1038/nmeth.3651.

      26379230: Savitski MM, Reinhard FBM, Franken H, et al. Tracking cancer drugs in living cells by thermal profiling of the proteome. Science. 2015;346(6205):1255784. doi:10.1126/science.1255784. 

      33452728: Leuenberger P, Ganscha S, Kahraman A, et al. Cell-wide analysis of protein thermal unfolding reveals determinants of thermostability. Science. 2020;355(6327):eaai7825. doi:10.1126/science.aai7825. 

      23066101: Savitski MM, Zinn N, Faelth-Savitski M, et al. Quantitative thermal proteome profiling reveals ligand interactions and thermal stability changes in cells. Nat Methods. 2013;10(12):1094-1096. doi:10.1038/nmeth.2766.  

      30858367: Piazza I, Kochanowski K, Cappelletti V, et al. A machine learning-based chemoproteomic approach to identify drug targets and binding sites in complex proteomes. Nat Commun. 2019;10(1):1216. doi:10.1038/s41467019-09199-0. 

      Reviewer #2 (Public Review):

      Summary:

      The membrane mimetic thermal proteome profiling (MM-TPP) presented by Jandu et al. seems to be a useful way to minimize the interference of detergents in efficient mass spectrometry analysis of membrane proteins. Thermal proteome profiling is a mass spectrometric method that measures binding of a drug to different proteins in a cell lysate by monitoring thermal stabilization of the proteins because of the interaction with the ligands that are being studied. This method has been underexplored for membrane proteome because of the inefficient mass spectrometric detection of membrane proteins and because of the interference from detergents that are used often for membrane protein solubilization.

      Strengths:

      In this report the binding of ligands to membrane protein targets has been monitored in crude membrane lysates or tissue homogenates exalting the efficacy of the method to detect both intended and off-target binding events in a complex physiologically relevant sample setting.

      The manuscript is lucidly written and the data presented seems clear. The only insignificant grammatical error I found was that the 'P' in the word peptidisc is not capitalized in the beginning of the methods section "MM-TPP profiling on membrane proteomes". The clear writing made it easy to understand and evaluate what has been presented. Kudos to the authors.

      Weaknesses:

      While this is a solid report and a promising tool for analyzing membrane protein drug interactions, addressing some of the minor caveats listed below could make it much more impactful.

      The authors claim that MM-TPP is done by "completely circumventing structural perturbations invoked by detergents[1] ". This may not be entirely accurate, because before reconstitution of the membrane proteins in peptidisc, the membrane fractions are solubilized by 1% DDM. The solubilization and following centrifugation step lasts at least for 45 min. It is less likely that all the structural perturbations caused by DDM to various membrane proteins and their transient interactions become completely reversed or rescued by peptidisc reconstitution.

      We thank the reviewer for this insightful comment. In response, we have revised the sentence and expanded the discussion to clarify that the Peptidisc provides a complementary approach to detergent-based preparations for studying membrane proteins, preserving native lipid–protein interactions and stabilization effects that may be diminished in detergent.

      To further address the structural perturbations invoked by detergents, and as already detailed to our response to Reviewer 1, we have compared the thermal profile of the Peptidisc library to the mouse liver membranes solubilized with 1% DDM, after incubation with ATP–VO₄ at 51 °C (Figure 4A). The results with the detergent extract revealed random patterns of stabilization and destabilization, with only 6.4% of differentially stabilized proteins being ATP-binding—comparable to the 7.4% observed in the background. In contrast, in the Peptidisc library, 17% of differentially stabilized proteins were ATP-binding, compared to 9.3% in the background. Thus, while Peptidisc reconstitution does not fully avoid initial detergent exposure, these findings underscore the importance of implementing Peptidisc in the TPP workflow when dealing with membrane proteins.

      In the introduction, the authors make statements such as "..it is widely acknowledged that even mild detergents can disrupt protein structures and activities, leading to challenges in accurately identifying drug targets.." and "[peptidisc] libraries are instrumental in capturing and stabilizing IMPs in their functional states while preserving their interactomes and lipid allosteric modulators...'. These need to be rephrased, as it has been shown by countless studies that even with membrane protein suspended in micelles robust ligand binding assays and binding kinetics have been performed leading to physiologically relevant conclusions and identification of protein-protein and protein-ligand interactions.

      We thank the reviewer for this valuable feedback and fully agree with the point raised. In response, we have revised the Introduction and conclusion to moderate the language concerning the limitations of detergent use. We now explicitly acknowledge that numerous studies have successfully used detergent micelles for ligand-binding assays and kinetic analyses, yielding physiologically relevant insights into both protein–protein and protein–ligand interactions [e.g., PMID: 22004748, 26440106, 31776188].

      At the same time, we clarify that the Peptidisc method offers a complementary advantage, particularly in the context of thermal proteome profiling (TPP), which involves mass spectrometry workflows that are incompatible with detergents. In this setting, Peptidiscs facilitate the detection of ligand-binding events that may be more difficult to observe in detergent micelles.

      We have reframed our discussion accordingly to present Peptidiscs not as a replacement for detergent-based methods, but rather as a complementary tool that broadens the available methodological landscape for studying membrane protein interactions.

      If the method involves detergent solubilization, for example using 1% DDM, it is a bit disingenuous to argue that 'interactomes and lipid allosteric modulators' characterized by lowaffinity interactions will remain intact or can be rescued upon detergent removal. Authors should discuss this or at least highlight the primary caveat of the peptidisc method of membrane protein reconstitution - which is that it begins with detergent solubilization of the proteome and does not completely circumvent structural perturbations invoked by detergents.

      We would like to clarify that, in our current workflow, ligand incubation occurs after reconstitution into Peptidiscs. As such, the method is designed to circumvent the negative effects of detergent during the critical steps involving low-affinity interactions.

      That said, we fully acknowledge that Peptidisc reconstitution begins with detergent solubilization (e.g., 1% DDM), and we have revised the conclusion to explicitly state this important caveat. As the reviewer correctly points out, this initial step may introduce some structural perturbations or result in the loss of weakly associated lipid modulators.

      However, reconstitution into Peptidiscs rapidly restores a detergent-free environment for membrane proteins, which has been shown in our previous studies [PMID: 38577106, 38232390, 31736482, 31364989] to mitigate these effects. Specifically, we have demonstrated that time-limited DDM exposure, followed by Peptidisc reconstitution, minimizes membrane protein delipidation, enhances thermal stability, retains functionality, and preserves multi-protein assemblies.

      It would also be important to test detergents that are even milder than 1% DDM and ones which are harsher than 1% DDM to show that this method of reconstitution can indeed rescue the perturbations to the structure and interactions of the membrane protein done by detergents during solubilization step. 

      We selected 1% DDM based on our previous work [PMID: 37295717, 39313981,38232390], where it consistently enabled robust and reproducible solubilization for Peptidisc reconstitution. We agree that comparing milder detergents (e.g., LMNG) and harsher ones (e.g., SDC) would provide valuable insights into how detergent strength influences structural perturbations, and how effectively these can be mitigated by Peptidisc reconstitution. Preliminary data (not shown) from mouse liver membranes indicate broadly similar proteomic profiles following solubilization with DDM, LMNG, and SDC, although potential differences in functional activity or ligand binding remain to be investigated.

      Based on the methods provided, it appears that the final amount of detergent in peptidisc membrane protein library was 0.008%, which is ~150 uM. The CMC of DDM depending on the amount of NaCl could be between 120-170 uM.

      While we cannot entirely rule out the presence of residual DDM (0.008%) in the raw library, its free concentration may be lower than initially estimated. This is related to the formation of mixed micelles with the amphipathic peptide scaffold, which is supplied in excess during reconstitution. These mixed micelles are subsequently removed during the ultrafiltration step. Furthermore, in related work using His-tagged Peptidiscs [PMID: 32364744], we purified the library by nickel-affinity chromatography following a 5× dilution into a detergent-free buffer. Although this purification step reduced the number of soluble proteins, the same membrane proteins were retained, suggesting that any residual detergent does not significantly interfere with Peptidisc reconstitution. Supporting this, our MM-TPP assays on purified libraries (data not shown) consistently demonstrated stabilization of ATP-binding proteins (e.g., SLC27A2, DNAJB3), indicating that the observed ligand–protein interactions result from successful incorporation into Peptidiscs.

      Perhaps, to completely circumvent the perturbations from detergents other methods of detergentfree solubilization such as using SMA polymers and SMALP reconstitution could be explored for a comparison. Moreover, a comparison of the peptidisc reconstitution with detergent-free extraction strategies, such as SMA copolymers, could lend more strength to the presented method.

      We agree that detergent-free methods such as SMA polymers hold promise for membrane protein solubilization. However, in preliminary single-replicate experiments using SMA2000 at 51 °C in the presence of ATP–VO₄ (data not shown), we observed broad, non-specific stabilization effects. Of the 2,287 quantified proteins, 9.3% were annotated as ATP-binding, yet 9.9% of the 101 proteins showing a log₂ fold change >1 or <–1 were ATPbinding, indicating no meaningful enrichment. Given this lack of specificity and the limited dataset, we chose not to pursue further SMA experiments and have not included them here. However, in a recent study (https://doi.org/10.1101/2025.08.25.672181), we directly compared Peptidisc, SMA, and nanodiscs for liver membrane proteome profiling. In that work, Peptidisc outperformed both SMA and nanodiscs in detecting membrane protein dysregulation between healthy and diseased liver. By extension, we expect Peptidisc to offer superior sensitivity and specificity for detecting ligand-induced stabilization events, such as those observed here with ATP–vanadate.

      Cross-verification of the identified interactions, and subsequent stabilization or destabilizations, should be demonstrated by other in vitro methods of thermal stability and ligand binding analysis using purified protein to support the efficacy of the MM-TPP method. An example cross-verification using SDS-PAGE, of the well-studied MsbA, is shown in Figure 2. In a similar fashion, other discussed targets such as, BCS1L, P2RX4, DgkA, Mao-B, and some un-annotated IMPs shown in supplementary figure 3 that display substantial stabilization or destabilization should be cross-verified.

      We appreciate this suggestion and note that a similar point was raised in R1’s comment “In addition, except for MsbA, no orthogonal methods are used to support the conclusions, and the authors rely entirely on quantifying rather small differences in abundances using either iBAQ or LFQ.” We have developed a detailed response to R1 on this matter, which equally applies here. 

      Cited Reference:

      35616533: Young JW, Wason IS, Zhao Z, et al. Development of a Method Combining Peptidiscs and Proteomics to Identify, Stabilize, and Purify a Detergent-Sensitive Membrane Protein Assembly. J Proteome Res. 2022;21(7):1748-1758. doi:10.1021/acs.jproteome.2c00129. PMID: 35616533.

      31364989: Carlson ML, Stacey RG, Young JW, et al. Profiling the Escherichia coli membrane protein interactome captured in Peptidisc libraries. Elife. 2019;8:e46615. doi:10.7554/eLife.46615. 

      22004748: O'Malley MA, Helgeson ME, Wagner NJ, Robinson AS. Toward rational design of protein detergent complexes: determinants of mixed micelles that are critical for the in vitro stabilization of a G-protein coupled receptor. Biophys J. 2011;101(8):1938-1948. doi:10.1016/j.bpj.2011.09.018.

      26440106: Allison TM, Reading E, Liko I, Baldwin AJ, Laganowsky A, Robinson CV. Quantifying the stabilizing effects of protein-ligand interactions in the gas phase. Nat Commun. 2015;6:8551. doi:10.1038/ncomms9551.

      31776188: Beckner RL, Zoubak L, Hines KG, Gawrisch K, Yeliseev AA. Probing thermostability of detergentsolubilized CB2 receptor by parallel G protein-activation and ligand-binding assays. J Biol Chem. 2020;295(1):181190. doi:10.1074/jbc.RA119.010696.

      38577106: Jandu RS, Yu H, Zhao Z, Le HT, Kim S, Huan T, Duong van Hoa F. Capture of endogenous lipids in peptidiscs and effect on protein stability and activity. iScience. 2024;27(4):109382. doi:10.1016/j.isci.2024.109382.

      38232390: Antony F, Brough Z, Zhao Z, Duong van Hoa F. Capture of the Mouse Organ Membrane Proteome Specificity in Peptidisc Libraries. J Proteome Res. 2024;23(2):857-867. doi:10.1021/acs.jproteome.3c00825.

      31736482: Saville JW, Troman LA, Duong Van Hoa F. PeptiQuick, a one-step incorporation of membrane proteins into biotinylated peptidiscs for streamlined protein binding assays. J Vis Exp. 2019;(153). doi:10.3791/60661. 

      37295717: Zhao Z, Khurana A, Antony F, et al. A Peptidisc-Based Survey of the Plasma Membrane Proteome of a Mammalian Cell. Mol Cell Proteomics. 2023;22(8):100588. doi:10.1016/j.mcpro.2023.100588. 

      39313981: Antony F, Brough Z, Orangi M, Al-Seragi M, Aoki H, Babu M, Duong van Hoa F. Sensitive Profiling of Mouse Liver Membrane Proteome Dysregulation Following a High-Fat and Alcohol Diet Treatment. Proteomics. 2024;24(23-24):e202300599. doi:10.1002/pmic.202300599. 

      32364744: Young JW, Wason IS, Zhao Z, Rattray DG, Foster LJ, Duong Van Hoa F. His-Tagged Peptidiscs Enable Affinity Purification of the Membrane Proteome for Downstream Mass Spectrometry Analysis. J Proteome Res. 2020;19(7):2553-2562. doi:10.1021/acs.jproteome.0c00022.

      32591519: The M, Käll L. Focus on the spectra that matter by clustering of quantification data in shotgun proteomics. Nat Commun. 2020;11(1):3234. doi:10.1038/s41467-020-17037-3. 

      33188197: Kurzawa N, Becher I, Sridharan S, et al. A computational method for detection of ligand-binding proteins from dose range thermal proteome profiles. Nat Commun. 2020;11(1):5783. doi:10.1038/s41467-02019529-8. 

      26524241: Reinhard FBM, Eberhard D, Werner T, et al. Thermal proteome profiling monitors ligand interactions with cellular membrane proteins. Nat Methods. 2015;12(12):1129-1131. doi:10.1038/nmeth.3652. 

      23828940: Martinez Molina D, Jafari R, Ignatushchenko M, et al. Monitoring drug target engagement in cells and tissues using the cellular thermal shift assay. Science. 2013;341(6141):84-87. doi:10.1126/science.1233606. 

      32133759: Mateus A, Kurzawa N, Becher I, et al. Thermal proteome profiling for interrogating protein interactions. Mol Syst Biol. 2020;16(3):e9232. doi:10.15252/msb.20199232. 

      14755328: Dorsam RT, Kunapuli SP. Central role of the P2Y12 receptor in platelet activation. J Clin Invest. 2004;113(3):340-345. doi:10.1172/JCI20986. 

      Reviewer #1 (Recommendations for the authors):

      “The authors use iBAC or LFQ to compare across samples. This inconsistency is puzzling. As far as I know, LFQ should always be used when comparing across samples”

      As mentioned above, we use iBAQ only in Fig. 2B to illustrate within-sample relative abundance; all comparative analyses elsewhere use LFQ. We have updated the Fig. 2B legend to state this explicitly.

      We used iBAQ Fig. 2B as it provides a notion of protein abundance within a sample, normalizing the summed peptide intensities by the number of theoretically observable peptides. This normalization facilitates comparisons between proteins within the same sample, offering a clearer understanding of their relative molar proportions [PMID: 33452728]. LFQ, by contrast, is optimized for comparing the same protein across different samples. It achieves this by performing delayed normalization to reduce run-to-run variability and by applying maximal peptide ratio extraction, which integrates pairwise peptide intensity ratios across all samples to build a consistent protein-level quantification matrix [PMID: 24942700]. These features make LFQ more robust to missing values and technical variation, thereby enabling accurate detection of relative abundance changes in the same protein under different experimental conditions. This distinction is well supported by the proteomics literature: Smits et al. [PMID: 23066101] used iBAQ specifically to determine the relative abundance of proteins within one sample, whereas LFQ was applied for comparative analyses between conditions.

      “[Regarding Figure 2A] Why does the control also contain ATP-vanadate? Also, I am not aware of a commercially available chemical "ATP-VO4". I assume this is a mistake”

      The control condition in Figure 2A was mislabeled, and the figure has been corrected to remove this discrepancy. In our experiments, ATP and orthovanadate (VO<sub>4</sub>) were added together, and for simplicity this was annotated as “ATP-VO<sub>4</sub>.” 

      “[Regarding Figure 2B] What is the fold change in MsbA iBAQ values? It seems that the differences are quite small, and as such require a more quantitative approach than iBAQ (e.g SILAC or some other internal standard). In addition, what information does this panel add relative to 2C”

      The figure has been updated to clarify that the values shown are log₂transformed iBAQ intensities. Figures 2B and 2C are complementary: Figure 2B shows that in the control sample, MsbA’s peptide abundance decreases with temperatures (51, 56, and 61 °C) relative to the remaining bulk proteins. Figure 2C shows the specific thermal profiles of MsbA in control and ATP–vanadate conditions. To make this clearer, we have added a sentence to the Results section explaining the specific role of Figure 2B.

      Together, these panels indicate that the method can identify ligand-induced stabilization even for proteins whose abundance decreases faster than the bulk during the TPP assay. We have provided the rationale for not using SILAC or TMT labeling in our public response.

      “[Regarding Figure 2C] Although not mentioned in the legend, I assume this is iBAQ quantification, which as mentioned above isn't accurate enough for such small differences. In addition, I find this data confusing: why is MsbA more stable at the lower temperatures in the absence of ATP-vanadate? The smoothed-line representation is misleading, certainly given the low number of data points”

      The data presented represent LFQ values for MsbA, and we have updated the figure legend to clearly indicate this. Additionally, as suggested, we have removed the smoothing line to more accurately reflect the data. Regarding the reviewer’s concern about stability at lower temperatures, we note that MsbA exhibits comparable abundance at 38 °C and 46 °C under both conditions, with overlapping error bars. We therefore interpret these data as indicating no significant difference in stability at the lower temperatures, with ligand-dependent stabilization becoming apparent only at elevated temperatures. We do not exclude the possibility that MsbA stability at these temperatures is affected by the conformational dynamics of this ABC transporter upon ATP binding and hydrolysis.

      “[Regarding Figure 3A] is this raw LFQ data? Why did the authors suddenly change from iBAQ to LFQ? I find this inconsistency puzzling”

      To clarify, all analyses of protein stabilization or destabilization presented in the manuscript are based on LFQ values. The only instance where iBAQ was used is Figure 2B, where it served to illustrate the relative peptide abundance of MsbA within the same sample. We have revised the figure legends and text to make this distinction explicit and ensure consistency in presentation.

      “[Regarding Figure 3B] The non-specific ATP-dependent stabilization increases the likelihood of false positive hits. This limitation is not mentioned by the authors. I think it is important to show other small molecules, in addition to ATP. The authors suggest that their approach is highly relevant for drug screening. Therefore, a good choice is to test an effect of a known stabilizing drug (eg VX-809 and CFTR)”

      We thank the reviewer for this suggestion. As noted in the manuscript (results and discussion sections), ATP is a natural hydrotrope and is therefore expected to induce broad, non-specific stabilization effects, a phenomenon also observed in previous proteome-wide studies, which demonstrated ATP’s widespread influence on cytosolic protein solubility and thermal stability (PMID: 30858367). To demonstrate that MM-TPP can resolve specific ligand–protein interactions beyond these global ATP effects, we tested 2-methylthio-ADP (2-MeS-ADP), a selective agonist of P2RY12 (PMID: 14755328). In these experiments, we observed robust and reproducible stabilization of P2RY12 at both 51°C and 57°C, with no consistent stabilization of unrelated proteins across temperatures. This provides direct evidence that our workflow can distinguish specific from non-specific ligand-induced effects. We selected 2-MeS-ADP due to its structural stability and receptor higher-affinity over ADP, allowing us to extend our existing workflow while testing a receptor-specific interaction. We agree that extending this approach to clinically relevant small-molecule drugs, such as VX-809 with CFTR, would further underscore the pharmacological potential of MM-TPP, and we have now noted this as an important avenue for future studies.

      “X axis of Figure 3B: Log 2 fold difference of what? iBAQ? LFQ? Similar ambiguity regarding the Y axis of 3E. What peptide? And why the constant changes in estimating abundances?”

      We thank the reviewer for pointing out these inaccuracies in the figure annotations. As mentioned above, all analyses (except Figure 2B) are based on LFQ values. We have revised the figure legends and text to make this clear.

      In Figure 3E, “peptide intensity” refers to log2 LFQ peptide intensities derived from the BCS1L protein, as indicated in the figure caption. 

      “The authors suggest that P2RY6 and P2RY12 are stabilized by ADP, the hydrolysis product of ATP. Currently, the support for this suggestion is highly indirect. To support this claim, the authors need to directly show the effect of ADP. In reference to the alpha fold results shown in Figure 4D, the authors state that "Collectively, these data highlight the ability of MM-TPP to detect the side effects of parent compounds, an important consideration for drug development". To support this claim, it is necessary to show that Mao-B is indeed best stabilized with ADP or AMP, rather than ATP.”

      In this revision, we chose not to test ADP directly, as it is a broadly binding, relatively weak ligand that would likely stabilize many proteins without revealing clear target-specific effects. Since we had already evaluated ATP-VO₄, a similarly broad, non-specific ligand, additional testing with ADP would provide limited additional insight. Instead, we prioritized 2-methylthio-ADP, a selective agonist of P2RY12, to more effectively demonstrate the specificity of MM-TPP. With this ligand, we observed clear and reproducible stabilization of P2RY12, underscoring the ability of MM-TPP to resolve receptor–ligand interactions beyond ATP’s broad hydrotropic effects. Importantly, and as expected, we did not observe stabilization of the related purinergic receptor P2RY6, further supporting the specificity of the observed effect.

      We have also revised the AlphaFold-related statement in Figure 4D to adopt a more cautious tone: “Collectively, these data suggest that MM-TPP may detect potential side effects of parent compounds, an important consideration for drug development.” In this context, we use AlphaFold not as a validation tool, but rather as a structural aid to help rationalize why certain off-target proteins (e.g., ATP with Mao-B) exhibit stabilization.

      Reviewer #2 (Recommendations for the authors):

      “In the main text, it will be useful to include the unique peptides table of at least the targets discussed in the manuscript. For example, in presence of AMP-PNP at 51oC P2RY6 shows 4-6 peptides in all n=3 positive & negative ionization modes. But, for P2RY12 only 1-3 peptides were observed. Depending on the sequence length and the relative abundance in the cell of a protein of interest, the number of peptides observed could vary a lot per protein. Given the unique peptide abundance reported in the supplementary file, for various proteins in different conditions, it appears the threshold of observation of two unique peptides for a protein to be analyzed seems less stringent.”

      By applying a filter requiring at least two unique peptides in at least one replicate, we exclude, on average, 15–20% of the total identified proteins. We consider this a reasonable level of stringency that balances confidence in protein identification with the retention of relevant data. This threshold was selected because it aligns with established LC-MS/MS data analysis practices (PMID: 32591519, 33188197, 26524241), and we have included these references in the Methods section to justify our approach. We have included in this revision a Supplemental Table 2 showing the unique peptide counts for proteins highlighted in this study.  

      “It appears that the time of heat treatment for peptidisc library subjected to MM-TPP profiling was chosen as 3 min based on the results presented in Supplementary Figure 1A, especially the loss of MsbA observed in 1% DDM after 3 min heat perturbation. However, when reconstituted in peptidisc there seems to be no loss in MsbA even after 12 mins at 45oC. So, perhaps a longer heat treatment would be a more efficient perturbation.”

      Previous studies indicate that heat exposure of 3–5 minutes is optimal for visualizing protein denaturation (PMID: 23828940, 32133759). We have added a statement to the Results section to justify our choice of heat exposure. Although MsbA remains stable at 45 °C for extended periods, higher temperatures allow for more effective perturbation to reveal destabilization. Supplementary Figure 1A specifically illustrates MsbA instability in detergent environments.

      “Some of the stabilized temperatures listed in Table 1 are a bit confusing. For example, ABCC3 and ABCG2. In the case of ABCC3 stabilization was observed at 51oC and 60oC, but 56oC is not mentioned. In the same way, 51oC is not mentioned for ABCG2. You would expect protein to be stabilized at 56oC if it is stabilized at both 51oC and 60oC. So, it is unclear if the stabilizations were not monitored for these proteins at the missing temperatures in the table or if no peptides could be recorded at these temperatures as in the case of P2RX4 at 60oC in Figure 4C.”

      Both scenarios are represented in our data. For some proteins, like ABCG2, sufficient peptide coverage was achieved, but no stabilization was observed at intermediate temperatures (e.g., 56 °C), likely because the perturbation was not strong enough to reveal an effect. In other cases, such as ABCC3 at 56 °C or P2RX4 at 60 °C, the proteins were not detected due to insufficient peptide identifications at those temperatures, which explains their omission from the table. 

      “In Figure 4C, it is perplexing to note that despite n = 3 there were no peptide fragments detected for P2RX4 at 60oC in presence of ATP-VO4, but they were detected in presence of AMP-PNP. It will be useful to learn authors explanation for this, especially because both of these ligands destabilize P2RX4. In Figure 4B, it would have been great to see the effect of ADP too, to corroborate the theory that ATP metabolites could impact the thermal stability.”

      In Figure 4C, the absence of P2RX4 peptide detection at 60 °C with ATP–VO₄ mirrors variability observed in the corresponding control (n = 6). Specifically, neither the control nor ATP–VO₄ produced unique peptides for P2RX4 at 60 °C in that replicate, whereas peptides were detected at 60 °C in other replicates for both the control and AMPPNP, and at 64 °C for ATP–VO<sub>4</sub>, the controls, and AMP-PNP. Such missing values are a natural feature of MS-based proteomics and can arise from multiple technical factors, including inconsistent heating, incomplete digestion, stochastic MS injection, or interference from Peptidisc peptides. We therefore interpret the absence of peptides in this replicate as a technical artifact rather than evidence against protein destabilization. Importantly, the overall dataset consistently shows that both ATP–VO₄ and AMP-PNP destabilize P2RX4, supporting their characterization as broad, non-specific ligands with off-target effects.

      Because ATP and ADP belong to the same class of broadly binding, non-specific ligands, additional testing with ADP would not provide meaningful mechanistic insight. Instead, we chose to test 2-methylthio-ADP, a selective P2RY12 agonist. This experiment revealed robust, reproducible stabilization of P2RY12, without consistent effects on unrelated proteins at 51 °C and 57 °C, thereby demonstrating the ability of MM-TPP to detect specific receptor–ligand interactions.

      Finally, we note that P2RX4 is not a primary target of ATP–VO<sub>4</sub> or AMP-PNP. Consequently, the observed destabilization of P2RX4 is expected to be less pronounced than the strong, physiologically consistent stabilization of ABC transporters by ATP–VO<sub>4</sub>, as shown in Figure 3D, where the majority of ABC transporters are thermally stabilized across all tested temperatures.

      “As per Figure 4, P2Y receptors P2RY6 and P2RY12 both showed great thermal stability in presence of ATP-VO4 despite their preference for ADP. The authors argue this could be because of ATP metabolism, and binding of the resultant ADP to the P2RY6. If P2RX4 prefers ATP and not the metabolized product ADP that apparently is available, ideally you should not see a change in stability. A stark destabilization would indicate interaction of some sorts. P2X receptors are activated by ATP and are not naturally activated by AMP-PNP. So, destabilization of P2RX4 upon binding to ATP that can activate P2X receptors is conceivable. However, destabilization both in presence of ATP-VO4 and AMP-PNP is unclear. It is perhaps useful to test effect of ADP using this method, and maybe even compare some antagonists such as TNPATP.”

      In this study, we did not directly test ADP, as we had already demonstrated that MM-TPP detects stabilization by broad-binding ligands such as ATP–VO₄. Instead, we focused on a more selective ligand, 2-MeS-ADP, a specific agonist of P2RY12 [PMID: 14755328]. Here, we observed robust and reproducible stabilization of P2RY12 at 51 °C and 57 °C, while P2RY6 showed no significant changes, and no other proteins were consistently stabilized (Figure 4B, S4). This confirms that MM-TPP can distinguish specific ligand–receptor interactions from broader ATP-induced effects. To further explore the assay’s nuance and sensitivity, testing additional nucleotide ligands—including antagonists like TNP-ATP or ATPγS—would provide valuable insights, and we have identified this as an important future direction.